Download as pdf or txt
Download as pdf or txt
You are on page 1of 4170

Which of the following drugs increases the rate of gastric emptying in the vagotomised stomach?

Ondansetron

Metoclopramide

Cyclizine

Erythromycin

Chloramphenicol

Vagotomy seriously compromises gastric emptying which is why either a pyloroplasty or gastro-
enterostomy is routinely performed at the same time.

Chloramphenicol has no effect on gastric emptying. Ondansetron slows gastric emptying slightly.
Metoclopramide increases the rate of gastric emptying but its effects are mediated via the vagus
nerve. Erythromycin enhances gastric emptying by acting via the motilin receptor in the gut.
Please rate this question:

Discuss and give feedback


Next question

Gastric emptying

 The stomach serves both a mechanical and immunological function. Solid and liquid are
retained in the stomach during which time repeated peristaltic activity against a closed
pyloric sphincter will cause fragmentation of food bolus material. Contact with gastric acid will
help to neutralise any pathogens present.
 The amount of time material spends in the stomach is related to its composition and volume.
For example a glass of water will empty more quickly than a large meal. The presence of
amino acids and fat will all serve to delay gastric emptying.

Controlling factors
Neuronal stimulation of the stomach is mediated via the vagus and the parasympathetic nervous
system will tend to favor an increase in gastric motility. It is for this reason that individuals who have
undergone truncal vagotomy will tend to routinely require either a pyloroplasty or gastro-enterostomy
as they would otherwise have delayed gastric emptying.

The following hormonal factors are all involved:


Delay emptying Increase emptying

Gastric inhibitory peptide Gastrin

Cholecystokinin

Enteroglucagon

Diseases affecting gastric emptying


All diseases that affect gastric emptying may result in bacterial overgrowth, retained food and
eventually the formation of bezoars that may occlude the pylorus and make gastric emptying even
worse. Fermentation of food may cause dyspepsia, reflux and foul smelling belches of gas.

Iatrogenic
Gastric surgery can have profound effects on gastric emptying. As stated above any procedure that
disrupts the vagus can cause delayed emptying. Whilst this is particularly true of vagotomy, this
operation is now rarely performed. Surgeons are divided on the importance of vagal disruption that
occurs during an oesophagectomy, some will routinely perform a pyloroplasty and others will not.

When a distal gastrectomy is performed, the type of anastomosis performed will impact on emptying.
When a gastro-enterostomy is constructed, a posterior, retrocolic gastroenterostomy will empty
better than an anterior one.

Diabetic gastroparesis
This is predominantly due to neuropathy affecting the vagus nerve. The stomach empties poorly and
patients may have episodes of repeated and protracted vomiting. Diagnosis is made by upper GI
endoscopy and contrast studies, in some cases a radio nucleotide scan is needed to demonstrate
the abnormality more clearly. In treating these conditions, drugs such as metoclopramide will be less
effective as they exert their effect via the vagus nerve. One of the few prokinetic drugs that do not
work in this way is the antibiotic erythromycin.

Malignancies
Obviously a distal gastric cancer may obstruct the pylorus and delay emptying. In addition,
malignancies of the pancreas may cause extrinsic compression of the duodenum and delay
emptying. Treatment in these cases is by gastric decompression using a wide bore nasogastric tube
and insertion of a stent or, if that is not possible, by a surgical gastroenterostomy. As a general rule
gastroenterostomies constructed for bypass of malignancy are usually placed on the anterior wall of
the stomach (in spite of the fact that they empty less well). A Roux en Y bypass may also be
undertaken, but the increased number of anastomoses for this, in malignant disease that is being
palliated, is probably not justified.

Congenital Hypertrophic Pyloric Stenosis


This is typically a disease of infancy. Most babies will present around 6 weeks of age with projectile
non bile stained vomiting. It has an incidence of 2.4 per 1000 live births and is more common in
males. Diagnosis is usually made by careful history and examination and a mass may be palpable in
the epigastrium (often cited seldom felt!). The most important diagnostic test is an ultrasound that
usually demonstrates the hypertrophied pylorus. Blood tests may reveal a hypochloraemic metabolic
alkalosis if the vomiting is long standing. Once the diagnosis is made the infant is resuscitated and a
pyloromyotomy is performed (either open or laparoscopically). Once treated there are no long term
sequelae.
Next question
Which of the following is not secreted by the parietal cells?

Hydrochloric acid

Mucus

Magnesium

Intrinsic factor

Calcium

Chief of Pepsi cola = Chief cells secrete PEPSInogen

Parietal cells: secrete HCl, Ca, Na, Mg and intrinsic factor


Chief cells: secrete pepsinogen
Surface mucosal cells: secrete mucus and bicarbonate
Please rate this question:

Discuss and give feedback


Next question

Gastric secretions

A working knowledge of gastric secretions is important for surgery because peptic ulcers are
common, surgeons frequently prescribe anti secretory drugs and because there are still patients
around who will have undergone acid lowering procedures (Vagotomy) in the past.

Gastric acid

 Is produced by the parietal cells in the stomach


 pH of gastric acid is around 2 with acidity being maintained by the H+/K+ ATP ase pump. As
part of the process bicarbonate ions will be secreted into the surrounding vessels.
 Sodium and chloride ions are actively secreted from the parietal cell into the canaliculus.
This sets up a negative potential across the membrane and as a result sodium and
potassium ions diffuse across into the canaliculus.
 Carbonic anhydrase forms carbonic acid which dissociates and the hydrogen ions formed by
dissociation leave the cell via the H+/K+ antiporter pump. At the same time sodium ions are
actively absorbed. This leaves hydrogen and chloride ions in the canaliculus these mix and
are secreted into the lumen of the oxyntic gland.
This is illustrated diagrammatically below:

Image sourced from Wikipedia

Phases of gastric acid secretion


There are 3 phases of gastric secretion:

1. Cephalic phase (smell / taste of food)

 30% acid produced


 Vagal cholinergic stimulation causing secretion of HCL and gastrin release from G cells

2. Gastric phase (distension of stomach )

 60% acid produced


 Stomach distension/low H+/peptides causes Gastrin release
3. Intestinal phase (food in duodenum)

 10% acid produced


 High acidity/distension/hypertonic solutions in the duodenum inhibits gastric acid secretion
via enterogastrones (CCK, secretin) and neural reflexes.

Regulation of gastric acid production


Factors increasing production include:

 Vagal nerve stimulation


 Gastrin release
 Histamine release (indirectly following gastrin release) from enterchromaffin like cells

Factors decreasing production include:

 Somatostatin (inhibits histamine release)


 Cholecystokinin
 Secretin

The diagram below illustrates some of the factors involved in regulating gastric acid secretion and
the relevant associated pharmacology

Image sourced from Wikipedia

Below is a brief summary of the major hormones involved in food digestion:


Source Stimulus Actions

Gastrin G cells in Distension of Increase HCL, pepsinogen and IF secretion,


antrum of the stomach, extrinsic increases gastric motility, trophic effect on
stomach nerves gastric mucosa
Inhibited by: low
antral pH,
somatostatin

CCK I cells in Partially digested Increases secretion of enzyme-rich fluid


upper small proteins and from pancreas, contraction of gallbladder
intestine triglycerides and relaxation of sphincter of Oddi,
decreases gastric emptying, trophic effect on
pancreatic acinar cells, induces satiety

Secretin S cells in Acidic chyme, Increases secretion of bicarbonate-rich fluid


upper small fatty acids from pancreas and hepatic duct cells,
intestine decreases gastric acid secretion, trophic
effect on pancreatic acinar cells

VIP Small Neural Stimulates secretion by pancreas and


intestine, intestines, inhibits acid and pepsinogen
pancreas secretion

Somatostatin D cells in the Fat, bile salts and Decreases acid and pepsin secretion,
pancreas and glucose in the decreases gastrin secretion, decreases
stomach intestinal lumen pancreatic enzyme secretion, decreases
insulin and glucagon secretion
inhibits trophic effects of gastrin, stimulates
gastric mucous production

Next question
A 65 year old man is admitted for a below knee amputation. He is taking digoxin. Clinically the
patient has an irregularly irregular pulse. What would you expect to see when you examine the
jugular venous pressure?

Absent y waves

Slow y descent

Cannon waves

Steep y descent

Absent a waves

Jugular venous pressure

Absent a waves = Atrial fibrillation


Large a waves = Any cause of right ventricular hypertrophy, tricuspid stenosis
Cannon waves (extra large a waves) = Complete heart block
Prominent v waves = Tricuspid regurgitation
Slow y descent = Tricuspid stenosis, right atrial myxoma
Steep y descent = Right ventricular failure, constrictive pericarditis, tricuspid regurgitation

Theme from January 2013 exam

This patient has atrial fibrillation and is most likely to have absent a waves.
Please rate this question:

Discuss and give feedback


Next question

Jugular venous pressure

As well as providing information on right atrial pressure, the jugular vein waveform may provide
clues to underlying valvular disease. A non-pulsatile JVP is seen in superior vena caval obstruction.
Kussmaul's sign describes a paradoxical rise in JVP during inspiration seen in constrictive
pericarditis

'a' wave = atrial contraction

 large if atrial pressure e.g. tricuspid stenosis, pulmonary stenosis, pulmonary hypertension
 absent if in atrial fibrillation

Cannon 'a' waves

 caused by atrial contractions against a closed tricuspid valve


 are seen in complete heart block, ventricular tachycardia/ectopics, nodal rhythm, single
chamber ventricular pacing

'c' wave

 closure of tricuspid valve


 not normally visible

'v' wave

 due to passive filling of blood into the atrium against a closed tricuspid valve
 giant v waves in tricuspid regurgitation

'x' descent = fall in atrial pressure during ventricular systole

'y' descent = opening of tricuspid valve

Image sourced from Wikipedia

Next question
Which part of the ECG represents atrial depolarization?

P wave

Q wave

T wave

QRS complex

P-R interval

Theme from April 2013 exam


Theme from April 2014 exam
The P wave represents atrial depolarization. Note that atrial repolarization is obscured within the
QRS complex.
Please rate this question:

Discuss and give feedback


Next question

The normal ECG


Image sourced from Wikipedia

P wave

 Represents the wave of depolarization that spreads from the SA node throughout the atria
 Lasts 0.08 to 0.1 seconds (80-100 ms)
 The isoelectric period after the P wave represents the time in which the impulse is traveling
within the AV node

P-R interval

 Time from the onset of the P wave to the beginning of the QRS complex
 Ranges from 0.12 to 0.20 seconds in duration
 Represents the time between the onset of atrial depolarization and the onset of ventricular
depolarization

QRS complex

 Represents ventricular depolarization


 Duration of the QRS complex is normally 0.06 to 0.1 seconds
ST segment

 Isoelectric period following the QRS


 Represents period which the entire ventricle is depolarized and roughly corresponds to the
plateau phase of the ventricular action potential

T wave

 Represents ventricular repolarization and is longer in duration than depolarization


 A small positive U wave may follow the T wave which represents the last remnants of
ventricular repolarization.

Q-T interval

 Represents the time for both ventricular depolarization and repolarization to occur, and
therefore roughly estimates the duration of an average ventricular action potential.
 Interval ranges from 0.2 to 0.4 seconds depending upon heart rate.
 At high heart rates, ventricular action potentials shorten in duration, which decreases the Q-T
interval. Therefore the Q-T interval is expressed as a "corrected Q-T (QTc)" by taking the Q-
T interval and dividing it by the square root of the R-R interval (interval between ventricular
depolarizations). This allows an assessment of the Q-T interval that is independent of heart
rate.
 Normal corrected Q-Tc interval is less than 0.44 seconds.

Next question
A 45 year old male has alcoholic cirrhosis and decompensated liver failure, which of the following
clotting factors is least likely to be affected?

Factor V

Factor VII

Factor IX

Factor VIII

Factor XI

Factor VIII is synthesised in the endothelial cells of the liver rather than the liver itself and therefore
is less prone to the effects of hepatic dysfunction.

Please rate this question:

Discuss and give feedback

Next question

Abnormal coagulation

Cause Factors affected

Heparin Prevents activation factors 2,9,10,11

Warfarin Affects synthesis of factors 2,7,9,10


Cause Factors affected

DIC Factors 1,2,5,8,11

Liver disease Factors 1,2,5,7,9,10,11

Interpretation blood clotting test results

Disorder APTT PT Bleeding time

Haemophilia Increased Normal Normal

von Willebrand's disease Increased Normal Increased

Vitamin K deficiency Increased Increased Normal

Next question
A 23 year old is stabbed in the groin and develops hypovolaemic shock. What is the most likely
finding on analysis of his urine?

Decreased specific gravity

Increased specific gravity

Increased urinary glucose

Increased urinary protein

Increased red blood cells in the urine

Theme from April 2013 Exam


Hypovolaemic shock is likely to compromise renal blood flow especially if blood pressure falls below
the range at which the kidney is able to autoregulate its blood flow. The result of this will be an
increase of the specific gravity as water retention occurs in an attempt to maintain circulating
volume.
Please rate this question:

Discuss and give feedback


Next question

Renal Physiology

Overview

 Each nephron is supplied with blood from an afferent arteriole that opens onto the glomerular
capillary bed.
 Blood then flows to an efferent arteriole, supplying the peritubular capillaries and medullary
vasa recta.
 The kidney receives up to 25% of resting cardiac output.

Control of blood flow

 The kidney is able to autoregulate its blood flow between systolic pressures of 80- 180mmHg
so there is little variation in renal blood flow.
 This is achieved by myogenic control of arteriolar tone, both sympathetic input and hormonal
signals (e.g. renin) are responsible.
Glomerular structure and function

 Blood inside the glomerulus has considerable hydrostatic pressure.


 The basement membrane has pores that will allow free diffusion of smaller solutes, larger
negatively charged molecules such as albumin are unable to cross.
 The glomerular filtration rate (GFR) is equal to the concentration of a solute in the urine,
times the volume of urine produced per minute, divided by the plasma concentration
(assuming that the solute is freely diffused e.g. inulin).
 In clinical practice creatinine is used because it is subjected to very little proximal tubular
secretion.
 Although subject to variability, the typical GFR is 125ml per minute.
 Glomerular filtration rate = Total volume of plasma per unit time leaving the capillaries and
entering the bowman's capsule
 Renal clearance = volume plasma from which a substance is removed per minute by the
kidneys

Substances used to measure GFR have the following features:


1. Inert
2. Free filtration from the plasma at the glomerulus (not protein bound)
3. Not absorbed or secreted at the tubules
4. Plasma concentration constant during urine collection

Examples: inulin, creatinine

GFR = urine concentration (mmol/l) x urine volume (ml/min)


--------------------------------------------------------------------------
plasma concentration (mmol/l)

 The clearance of a substance is dependent not only on its diffusivity across the basement
membrane but also subsequent tubular secretion and / or reabsorption.
 So glucose which is freely filtered across the basement membrane is usually reabsorbed
from tubules giving a clearance of zero.

Tubular function

 Reabsorption and secretion of substances occurs in the tubules.


 In the proximal tubule substrates such as glucose, amino acids and phosphate are co-
transported with sodium across the semi permeable membrane.
 Up to two thirds of filtered water is reabsorbed in the proximal tubules.
 This will lead to increase in urea concentration in the distal tubule allowing for its increased
diffusion.
 Substances to be secreted into the tubules are taken up from the peritubular blood by tubular
cells.
 Solutes such as paraaminohippuric acid are cleared with a single passage through the
kidneys and this is why it is used to measure renal plasma flow. Ions such as calcium and
phosphate will have a tubular reabsorption that is influenced by plasma PTH levels.
 Potassium may be both secreted and re-absorbed and is co-exchanged with sodium.

Loop of Henle

 Approximately 60 litres of water containing 9000mmol sodium enters the descending limb of
the loop of Henle in 24 hours.
 Loops from the juxtamedullary nephrons run deep into the medulla.
 The osmolarity of fluid changes and is greatest at the tip of the papilla.
 The thin ascending limb is impermeable to water, but highly permeable to sodium and
chloride ions.
 This loss means that at the beginning of the thick ascending limb the fluid is hypo osmotic
compared with adjacent interstitial fluid.
 In the thick ascending limb the reabsorption of sodium and chloride ions occurs by both
facilitated and passive diffusion pathways.
 The loops of Henle are co-located with vasa recta, these will have similar solute
compositions to the surrounding extracellular fluid so preventing the diffusion and
subsequent removal of this hypertonic fluid.
 The energy dependent reabsorption of sodium and chloride in the thick ascending limb helps
to maintain this osmotic gradient.

Next question
A healthy man has a blood pressure of 120/80 mmHg and an intra cranial pressure of 17 mmHg.
What is the approximate cerebral perfusion pressure?

103 mmHg

63 mmHg

83 mmHg

91 mmHg

76 mmHg

Cerebral perfusion pressure= Mean arterial pressure - intra cranial pressure

Theme from September 2015 Exam


The mean arterial pressure can be calculated as:
MAP= Diastolic pressure+ 0.333(Systolic pressure- Diastolic pressure)
In this situation the MAP = 93.
The ICP is subtracted from this value; 93 - 17 = 76

Please rate this question:

Discuss and give feedback

Next question

Cerebral perfusion pressure

The cerebral perfusion pressure (CPP) is defined as being the net pressure gradient causing blood
flow to the brain. The CPP is tightly autoregulated to maximise cerebral perfusion. A sharp rise in
CPP may result in a rising ICP, a fall in CPP may result in cerebral ischaemia. It may be calculated
by the following equation:

CPP= Mean arterial pressure - Intra cranial pressure


Following trauma, the CPP has to be carefully controlled and the may require invasive monitoring of
the ICP and MAP.

Next question
There is decreased secretion of which one of the following hormones in response to major surgery:

Insulin

Cortisol

Renin

Anti diuretic hormone

Prolactin

Endocrine parameters reduced in stress response:

 Insulin
 Testosterone
 Oestrogen

Insulin is often released in decreased quantities following surgery.


Please rate this question:

Discuss and give feedback


Next question

Stress response: Endocrine and metabolic changes

 Surgery precipitates hormonal and metabolic changes causing the stress response.
 Stress response is associated with: substrate mobilization, muscle protein loss, sodium and
water retention, suppression of anabolic hormone secretion, activation of the sympathetic
nervous system, immunological and haematological changes.
 The hypothalamic-pituitary axis and the sympathetic nervous systems are activated and
there is a failure of the normal feedback mechanisms of control of hormone secretion.

A summary of the hormonal changes associated with the stress response:

Increased Decreased No Change


Increased Decreased No Change

Growth hormone Insulin Thyroid stimulating hormone

Cortisol Testosterone Luteinizing hormone

Renin Oestrogen Follicle stimulating hormone

Adrenocorticotrophic hormone (ACTH)

Aldosterone

Prolactin

Antidiuretic hormone

Glucagon

Sympathetic nervous system

 Stimulates catecholamine release


 Causes tachycardia and hypertension

Pituitary gland

 ACTH and growth hormone (GH) is stimulated by hypothalamic releasing factors,


corticotrophin releasing factor (CRF) and somatotrophin (or growth hormone releasing factor)
 Perioperative increased prolactin secretion occurs by release of inhibitory control
 Secretion of thyroid stimulating hormone (TSH), luteinizing hormone (LH) and follicle
stimulating hormone (FSH) does not change significantly
 ACTH stimulates cortisol production within a few minutes of the start of surgery. More ACTH
is produced than needed to produce a maximum adrenocortical response.

Cortisol

 Significant increases within 4-6 hours of surgery (>1000 nmol litre-1).


 The usual negative feedback mechanism fails and concentrations of ACTH and cortisol
remain persistently increased.
 The magnitude and duration of the increase correlate with the severity of stress and the
response is not abolished by the administration of corticosteroids.
 The metabolic effects of cortisol are enhanced:

Skeletal muscle protein breakdown to provide gluconeogenic precursors and amino acids for protein
synthesis in the liver
Stimulation of lipolysis
'Anti-insulin effect'
Mineralocorticoid effects
Anti-inflammatory effects

Growth hormone

 Increased secretion after surgery has a minor role


 Most important for preventing muscle protein breakdown and promote tissue repair by insulin
growth factors

Alpha Endorphin

 Increased

Antidiuretic hormone

 An important vasopressor and enhances haemostasis


 Renin is released causing the conversion of angiotensinogen to angiotensin I
 Angiotensin II formed by ACE on angiotensin 1, which causes the secretion of aldosterone
from the adrenal cortex. This increases sodium reabsorption at the distal convoluted tubule

Insulin

 Release inhibited by stress


 Occurs via the inhibition of the beta cells in the pancreas by the α2-adrenergic inhibitory
effects of catecholamines
 Insulin resistance by target cells occurs later
 The perioperative period is characterized by a state of functional insulin deficiency

Thyroxine (T4) and tri-iodothyronine (T3)

 Circulating concentrations are inversely correlated with sympathetic activity and after surgery
there is a reduction in thyroid hormone production, which normalises over a few days.

Metabolic effect of endocrine response

Carbohydrate metabolism

 Hyperglycaemia is a main feature of the metabolic response to surgery


 Due to increase in glucose production and a reduction in glucose utilization
 Catecholamines and cortisol promote glycogenolysis and gluconeogenesis
 Initial failure of insulin secretion followed by insulin resistance affects the normal responses
 The proportion of the hyperglycaemic response reflects the severity of surgery
 Hyperglycaemia impairs wound healing and increase infection rates

Protein metabolism

 Initially there is inhibition of protein anabolism, followed later, if the stress response is
severe, by enhanced catabolism
 The amount of protein degradation is influenced by the type of surgery and also by the
nutritional status of the patient
 Mainly skeletal muscle protein is affected
 The amino acids released form acute phase proteins (fibrinogen, C reactive protein,
complement proteins, a2-macroglobulin, amyloid A and ceruloplasmin) and are used for
gluconeogenesis
 Nutritional support has little effect on preventing catabolism

Lipid metabolism
Increased catecholamine, cortisol and glucagon secretion, and insulin deficiency, promotes lipolysis
and ketone body production.

Salt and water metabolism

 ADH causes water retention, concentrated urine, and potassium loss and may continue for 3
to 5 days after surgery
 Renin causes sodium and water retention
Cytokines

 Glycoproteins
 Interleukins (IL) 1 to 17, interferons, and tumour necrosis factor
 Synthesized by activated macrophages, fibroblasts, endothelial and glial cells in response to
tissue injury from surgery or trauma
 IL-6 main cytokine associated with surgery. Peak 12 to 24 h after surgery and increase by
the degree of tissue damage Other effects of cytokines include fever, granulocytosis,
haemostasis, tissue damage limitation and promotion of healing.

Modifying the response

 Opioids suppress hypothalamic and pituitary hormone secretion


 At high doses the hormonal response to pelvic and abdominal surgery is abolished.
However, such doses prolong recovery and increase the need for postoperative ventilatory
support
 Spinal anaesthesia can reduce the glucose, ACTH, cortisol, GH and epinephrine changes,
although cytokine responses are unaltered
 Cytokine release is reduced in less invasive surgery
 Nutrition prevents the adverse effects of the stress response. Enteral feeding improves
recovery
 Growth hormone and anabolic steroids may improve outcome
 Normothermia decreases the metabolic response

References
Deborah Burton, Grainne Nicholson, and George Hall
Endocrine and metabolic response to surgery .

Contin Educ Anaesth Crit Care Pain (2004) 4(5): 144-147 doi:10.1093/bjaceaccp/mkh040
Next question
A 28 year old man undergoes an ileocaecal resection to treat terminal ileal Crohns disease. Post
operatively he attends the clinic and complains of diarrhoea. His CRP is within normal limits and
small bowel enteroclysis shows no focal changes. Which of the following interventions is most likely
to be beneficial?

5 ASA drugs

Azathioprine

Pulsed methylprednisolone

Infliximab

Oral cholestyramine

Malabsorption of bile salts is a common cause of diarrhoea following ileal resection. A normal small
bowel study and CRP effectively excludes active Crohns disease and therefore immunomodulator
drugs are not appropriate.

Please rate this question:

Discuss and give feedback

Next question

Ileum

Anatomical overview
The terminal ileum comprises two fifths of the small intestine and has a diameter that is smaller than
that of the jejunum (typically 2cm at the ileocaecal valve). The ileum is attached to the abdominal
wall by a mesentery that contains more fat than that of the jejunum. The blood supply of the ileum is
derived from branches of the superior mesenteric artery, the vascular arcades of the ileum are more
densely packed than those of the jejunum. The wall of the ileum contains Peyers patches which are
aggregations of lymphoid tissue.
Function
The main function of the terminal ileum is absorption of vitamin B12 and bile salts. The
neuroendocrine cells in the wall of the ileum may secrete hormones. In surgical patients, resection of
the terminal ileum is a common procedure for conditions such as terminal ileal Crohns disease.
Where a significant proportion of the ileum is removed, patients are at increased risk of bile salt
malabsorption with the development of bile salt diarrhoea and increased risk of gallstones. The lack
of vitamin B12 may pre-dispose to macrocytic anaemia.

Next question
Which of the following statements related to the coagulation cascade is true?

The intrinsic pathway is the main pathway in coagulation

Heparin inhibits the activation of Factor 8

The activation of factor 8 is the point when the intrinsic and the extrinsic pathways meet

Tissue factor released by damaged tissue initiates the extrinsic pathway

Thrombin converts plasminogen to plasmin

Theme from April 2015 Exam


The extrinsic pathway is the main path of coagulation. Heparin inhibits the activation of factors
2,9,10,11. The activation of factor 10 is when both pathways meet. Thrombin converts fibrinogen to
fibrin. During fibrinolysis plasminogen is converted to plasmin to break down fibrin.
Please rate this question:

Discuss and give feedback


Next question

Coagulation cascade

Two pathways lead to fibrin formation

Intrinsic pathway (components already present in the blood)

 Minor role in clotting


 Subendothelial damage e.g. collagen
 Formation of the primary complex on collagen by high-molecular-weight kininogen (HMWK),
prekallikrein, and Factor 12
 Prekallikrein is converted to kallikrein and Factor 12 becomes activated
 Factor 12 activates Factor 11
 Factor 11 activates Factor 9, which with its co-factor Factor 8a form the tenase complex
which activates Factor 10

Extrinsic pathway (needs tissue factor released by damaged tissue)

 Tissue damage
 Factor 7 binds to Tissue factor
 This complex activates Factor 9
 Activated Factor 9 works with Factor 8 to activate Factor 10

Common pathway

 Activated Factor 10 causes the conversion of prothrombin to thrombin


 Thrombin hydrolyses fibrinogen peptide bonds to form fibrin and also activates factor 8 to
form links between fibrin molecules

Fibrinolysis
Plasminogen is converted to plasmin to facilitate clot resorption

Image sourced from Wikipedia

Intrinsic pathway Increased APTT Factors 8,9,11,12

Extrinsic pathway Increased PT Factor 7

Common pathway Increased APTT & PT Factors 2,5,10

Vitamin K dependent Factors 2,7,9,10


A 34 year old man receives morphine following an appendicectomy. He develops constipation as a
result. Which of the following best accounts for this process?

Stimulation of DOPA receptors

Inhibition of DOPA receptors

Stimulation of µ receptors

Stimulation of serotonin release

Inhibition of serotonin release

4 Types of opioid receptor:

 δ (located in CNS)- Accounts for analgesic and antidepressant effects


 k (mainly CNS)- analgesic and dissociative effects
 µ (central and peripheral) - causes analgesia, miosis, decreased gut motility
 Nociceptin receptor (CNS)- Affect of appetite and tolerance to µ agonists.

Theme from April 2013 Exam


Constipation is a common side effect of morphine treatment and stimulation of µ receptors accounts
for this process.
Please rate this question:

Discuss and give feedback


Next question

Morphine

Strong opiate analgesic. It is a pro- type narcotic drug and its effects mediated via the 4 types of
opioid receptor. Its clinical effects stem from binding to these receptor sites within the CNS and
gastrointestinal tract. Unwanted side effects include nausea, constipation, respiratory depression
and, if used long term, addiction .
It may be administered orally or intravenously. It can be reversed with naloxone.
Next question
Which of the following is not well absorbed following a gastrectomy?

Vitamin c

Zinc

Vitamin B12

Copper

Molybdenum

Vitamin B12. The others are unaffected

Post gastrectomy syndrome

 Rapid emptying of food from stomach into the duodenum: diarrhoea, abdominal pain,
hypoglycaemia
 Complications: Vitamin B12 and iron malabsorption, osteoporosis
 Treatment: High protein, low carbohydrate diet. Replace B12/Fe/Ca

Please rate this question:

Discuss and give feedback


Next question

Gastric emptying

 The stomach serves both a mechanical and immunological function. Solid and liquid are
retained in the stomach during which time repeated peristaltic activity against a closed
pyloric sphincter will cause fragmentation of food bolus material. Contact with gastric acid will
help to neutralise any pathogens present.
 The amount of time material spends in the stomach is related to its composition and volume.
For example a glass of water will empty more quickly than a large meal. The presence of
amino acids and fat will all serve to delay gastric emptying.

Controlling factors
Neuronal stimulation of the stomach is mediated via the vagus and the parasympathetic nervous
system will tend to favor an increase in gastric motility. It is for this reason that individuals who have
undergone truncal vagotomy will tend to routinely require either a pyloroplasty or gastro-enterostomy
as they would otherwise have delayed gastric emptying.

The following hormonal factors are all involved:

Delay emptying Increase emptying

Gastric inhibitory peptide Gastrin

Cholecystokinin

Enteroglucagon

Diseases affecting gastric emptying


All diseases that affect gastric emptying may result in bacterial overgrowth, retained food and
eventually the formation of bezoars that may occlude the pylorus and make gastric emptying even
worse. Fermentation of food may cause dyspepsia, reflux and foul smelling belches of gas.

Iatrogenic
Gastric surgery can have profound effects on gastric emptying. As stated above any procedure that
disrupts the vagus can cause delayed emptying. Whilst this is particularly true of vagotomy, this
operation is now rarely performed. Surgeons are divided on the importance of vagal disruption that
occurs during an oesophagectomy, some will routinely perform a pyloroplasty and others will not.

When a distal gastrectomy is performed, the type of anastomosis performed will impact on emptying.
When a gastro-enterostomy is constructed, a posterior, retrocolic gastroenterostomy will empty
better than an anterior one.

Diabetic gastroparesis
This is predominantly due to neuropathy affecting the vagus nerve. The stomach empties poorly and
patients may have episodes of repeated and protracted vomiting. Diagnosis is made by upper GI
endoscopy and contrast studies, in some cases a radio nucleotide scan is needed to demonstrate
the abnormality more clearly. In treating these conditions, drugs such as metoclopramide will be less
effective as they exert their effect via the vagus nerve. One of the few prokinetic drugs that do not
work in this way is the antibiotic erythromycin.

Malignancies
Obviously a distal gastric cancer may obstruct the pylorus and delay emptying. In addition,
malignancies of the pancreas may cause extrinsic compression of the duodenum and delay
emptying. Treatment in these cases is by gastric decompression using a wide bore nasogastric tube
and insertion of a stent or, if that is not possible, by a surgical gastroenterostomy. As a general rule
gastroenterostomies constructed for bypass of malignancy are usually placed on the anterior wall of
the stomach (in spite of the fact that they empty less well). A Roux en Y bypass may also be
undertaken, but the increased number of anastomoses for this, in malignant disease that is being
palliated, is probably not justified.
Congenital Hypertrophic Pyloric Stenosis
This is typically a disease of infancy. Most babies will present around 6 weeks of age with projectile
non bile stained vomiting. It has an incidence of 2.4 per 1000 live births and is more common in
males. Diagnosis is usually made by careful history and examination and a mass may be palpable in
the epigastrium (often cited seldom felt!). The most important diagnostic test is an ultrasound that
usually demonstrates the hypertrophied pylorus. Blood tests may reveal a hypochloraemic metabolic
alkalosis if the vomiting is long standing. Once the diagnosis is made the infant is resuscitated and a
pyloromyotomy is performed (either open or laparoscopically). Once treated there are no long term
sequelae.
Next question
Which vitamin is involved in the formation of collagen?

Vitamin A

Vitamin B

Vitamin C

Vitamin D

Vitamin E

Theme from September 2015 Exam


Vitamin C is needed for the hydroxylation of proline during collagen synthesis.
Please rate this question:

Discuss and give feedback


Next question

Collagen

Collagen is one of the most important structural proteins within the extracellular matrix, collagen
together with components such as elastin and glycosaminoglycans determine the properties of all
tissues.

 Composed of 3 polypeptide strands that are woven into a helix, usually a combination of
glycine with either proline or hydroxyproline plus another amino acid
 Numerous hydrogen bonds exist within molecule to provide additional strength
 Many sub types but commonest sub type is I (90% of bodily collagen), tissues with increased
levels of flexibility have increased levels of type III collagen
 Vitamin c is important in establishing cross links
 Synthesised by fibroblasts

Collagen Diseases
Disorders of collagen range from relatively common, acquired defects (typically aging), through to
rarer congenital disorders. The latter are exemplified by conditions such as osteogenesis imperfecta
and Ehlers Danlos syndromes.

Osteogenesis imperfecta:
-8 Subtypes
-Defect of type I collagen
-In type I the collagen is normal quality but insufficient quantity
-Type II- poor quantity and quality
-Type III- Collagen poorly formed, normal quantity
-Type IV- Sufficient quantity but poor quality
Patients have bones which fracture easily, loose joint and multiple other defects depending upon
which sub type they suffer from.

Ehlers Danlos:
-Multiple sub types
-Abnormality of types 1 and 3 collagen
-Patients have features of hypermobility.
-Individuals are prone to joint dislocations and pelvic organ prolapse. In addition to many other
diseases related to connective tissue defects.
Next question
A 56 year old man has long standing chronic pancreatitis and develops pancreatic insufficiency.
Which of the following will be absorbed normally?

Fat

Protein

Folic acid

Vitamin B12

None of the above

Pancreatic lipase is required for digestion of fat, Proteases facilitate protein and B12 absorption.
Folate digestion is independent of the pancreas.

Please rate this question:

Discuss and give feedback

Next question

Pancreas exocrine physiology

Composition of pancreatic secretions


Pancreatic secretions are usually 1000-1500ml per 24 hours and have a pH of 8.

Secretion Source Substances secreted

Trypsinogen
Procarboxylase
Enzymic Acinar cells
Amylase
Elastase
Sodium
Bicarbonate
Water
Ductal and
Aqueous Potassium
Centroacinar cells
Chloride
NB: Sodium and potassium reflect their plasma levels; chloride
and bicarbonate vary with flow rate

Regulation
The cephalic and gastric phases (neuronal and physical) are less important in regulating the
pancreatic secretions. The effect of digested material in the small bowel stimulates CCK release and
ACh which stimulate acinar and ductal cells. Of these CCK is the most potent stimulus. In the case
of the ductal cells these are potently stimulated by secretin which is released by the S cells of the
duodenum. This results in an increase in bicarbonate.

Enzyme activation
Trypsinogen is converted via enterokinase to active trypsin in the duodenum. Trypsin then activates
the other inactive enzymes

Next question
A 56 year old male presents to the acute surgical take with severe abdominal pain. He is normally fit
and well. He has no malignancy. The biochemistry laboratory contacts the ward urgently, his
corrected calcium result is 3.6 mmol/l. What is the medication of choice to treat this abnormality?

IV Pamidronate

Oral Alendronate

Dexamethasone

Vitamin D

Resonium salts

Theme from January 2012 exam

IV Pamidronate is the drug of choice as it most effective and has long lasting effects. Calcitonin
would need to be given with another agent, to ensure that the hypercalcaemia is treated once its
short term effects wear off. IV zoledronate is preferred in scenarios associated with malignancy.
Please rate this question:

Discuss and give feedback


Next question

Management of hypercalcaemia

 Free Ca is affected by pH (increased in acidosis) and plasma albumin concentration


 ECG changes include: Shortening of QTc interval
 Urgent management is indicated if:

Calcium > 3.5 mmol/l

Reduced consciousness

Severe abdominal pain

Pre renal failure


Management:

 Airway Breathing Circulation


 Intravenous fluid resuscitation with 3-6L of 0.9% Normal saline in 24 hours
 Concurrent administration of calcitonin will also help lower calcium levels
 Medical therapy (usually if Corrected calcium >3.0mmol/l)

Bisphosphonates

 Analogues of pryrophosphate
 Prevent osteoclast attachment to bone matrix and interfere with osteoclast activity
 Inhibit bone resorption.

Agents

Drug Side effects Notes

IV Pamidronate pyrexia, leucopaenia Most potent agent

IV Zoledronate response lasts 30 days Used for malignancy associated hypercalcaemia

Calcitonin

 Quickest onset of action however short duration (tachyphylaxis) therefore only given with a
second agent.

Prednisolone

 May be given in hypercalcaemia related to sarcoidosis, myeloma or vitamin D intoxication.

Next question
An over enthusiastic medical student decides to ask you questions about ECGs. Rather than
admitting your dwindling knowledge on this topic, you bravely attempt to answer her questions! One
question is what segment of the ECG represents ventricular repolarization?

QRS complex

Q-T interval

P wave

T wave

S-T segment

Theme from January 2012 exam

The T wave represents ventricular repolarization. The common sense approach to remembering
this, is to acknowledge that ventricular repolarization is the last phase of cardiac contraction and
should therefore correspond the the last part of the QRS complex.
Please rate this question:

Discuss and give feedback


Next question

The normal ECG


Image sourced from Wikipedia

P wave

 Represents the wave of depolarization that spreads from the SA node throughout the atria
 Lasts 0.08 to 0.1 seconds (80-100 ms)
 The isoelectric period after the P wave represents the time in which the impulse is traveling
within the AV node

P-R interval

 Time from the onset of the P wave to the beginning of the QRS complex
 Ranges from 0.12 to 0.20 seconds in duration
 Represents the time between the onset of atrial depolarization and the onset of ventricular
depolarization

QRS complex

 Represents ventricular depolarization


 Duration of the QRS complex is normally 0.06 to 0.1 seconds
ST segment

 Isoelectric period following the QRS


 Represents period which the entire ventricle is depolarized and roughly corresponds to the
plateau phase of the ventricular action potential

T wave

 Represents ventricular repolarization and is longer in duration than depolarization


 A small positive U wave may follow the T wave which represents the last remnants of
ventricular repolarization.

Q-T interval

 Represents the time for both ventricular depolarization and repolarization to occur, and
therefore roughly estimates the duration of an average ventricular action potential.
 Interval ranges from 0.2 to 0.4 seconds depending upon heart rate.
 At high heart rates, ventricular action potentials shorten in duration, which decreases the Q-T
interval. Therefore the Q-T interval is expressed as a "corrected Q-T (QTc)" by taking the Q-
T interval and dividing it by the square root of the R-R interval (interval between ventricular
depolarizations). This allows an assessment of the Q-T interval that is independent of heart
rate.
 Normal corrected Q-Tc interval is less than 0.44 seconds.

Next question
The oxygen-haemoglobin dissociation curve is shifted to the right in which of the following
scenarios?

Hypothermia

Respiratory alkalosis

Low altitude

Decreased 2,3-DPG in transfused red cells

Chronic iron deficiency anaemia

Mnemonic to remember causes of right shift of the oxygen dissociation curve:

CADET face RIGHT

C O2
A cidosis
2,3-DPG
E xercise
T emperature

The curve is shifted to the right when there is an increased oxygen requirement by the tissue. This
includes:

 Increased temperature
 Acidosis
 Increased DPG:

DPG is found in erythrocytes and is increased during glycolysis. It binds to the Hb molecule, thereby
releasing oxygen to tissues. DPG is increased in conditions associated with poor oxygen delivery to
tissues, such as anaemia and high altitude.
Please rate this question:

Discuss and give feedback


Next question

Oxygen Transport
Oxygen transport
Almost all oxygen is transported within erythrocytes. It has limited solubility and only 1% is carried as
solution. Therefore the amount of oxygen transported will depend upon haemoglobin concentration
and its degree of saturation.

Haemoglobin
Globular protein composed of 4 subunits. Haem consists of a protoporphyrin ring surrounding an
iron atom in its ferrous state. The iron can form two additional bonds; one with oxygen and the other
with a polypeptide chain. There are two alpha and two beta subunits to this polypeptide chain in an
adult and together these form globin. Globin cannot bind oxygen but is able to bind to carbon dioxide
and hydrogen ions, the beta chains are able to bind to 2,3 diphosphoglycerate. The oxygenation of
haemoglobin is a reversible reaction. The molecular shape of haemoglobin is such that binding of
one oxygen molecule facilitates the binding of subsequent molecules.

Oxygen dissociation curve

 The oxygen dissociation curve describes the relationship between the percentage of
saturated haemoglobin and partial pressure of oxygen in the blood. It is not affected by
haemoglobin concentration.
 Chronic anaemia causes 2, 3 DPG levels to increase, hence shifting the curve to the right

Haldane effect

 Shifts to left = for given oxygen tension there is increased saturation of Hb with oxygen i.e.
Decreased oxygen delivery to tissues

Bohr effect

 Shifts to right = for given oxygen tension there is reduced saturation of Hb with oxygen i.e.
Enhanced oxygen delivery to tissues
Image sourced from Wikipedia

Shifts to Left = Lower oxygen delivery Shifts to Right = Raised oxygen


delivery

 HbF, methaemoglobin, carboxyhaemoglobin


 low [H+] (alkali)  raised [H+] (acidic)
 low pCO2  raised pCO2
 low 2,3-DPG  raised 2,3-DPG*
 low temperature  raised temperature

*2,3-diphosphoglycerate
Next question
A 45 year old man undergoes a sub total colectomy and formation of end ileostomy. What is the
most likely sodium content per litre of ileostomy fluid?

120 mmol

60 mmol

20 mmol

210 mmol

180 mmol

Investigators in the 1960's dehydrated and measured the sodium content of ileostomy effluent and
determined this concentration. Not an experiment many would care to repeat!

Please rate this question:

Discuss and give feedback

Next question

Gastrointestinal secretions

Up to 7 litres of gastrointestinal secretions enter the lumen of the GI tract in a 24 hour period. The
absorptive function of the small bowel is such that by the time a formed stool is created, it will
contain, on average 200ml water.
The common secretions together with their approximate volumes are demonstrated below:

Origin of secretion Volume in ml / 24 hour period Na +mmol/L K+mmol/L Cl-mmol/L HCO3


Origin of secretion Volume in ml / 24 hour period Na +mmol/L K+mmol/L Cl-mmol/L HCO3

Salivary glands 1500 10 26 10 30

Stomach 1500 60 10 130

Duodenum 100-2000 140 80 80

Pancreas 1000 140 5 70 115

Bile 50-800 145 50 100 35

Jejunum/ileum 3000 140 50 104 30

Colon 100 60 30 40

The regulation of these secretions is dependent upon location. In the salivary glands a complex
interaction of flow rate governed by the autonomic nervous system. The exact composition of
sodium and potassium is regulated by aldosterone. In the stomach hormones such as gastrin play a
role and feedback is both endocrine and neurologically mediated (vagus). In the duodenum CCK is
released in response to duodenal distension and this causes contraction of the gallbladder and
release of bile.

Pancreatic secretions are affected by somatostatin. The secretions in the small bowel are affected
by the osmolality of the lumenal contents. This is in part due to the tightness of cellular junctions and
in this regard the jejunum is more permeable than the ileum. The practical implication of this is that if
an individual has an extensive intestinal resection and a high output, proximally sited stoma then
administration of hypotonic rather than isotonic solutions will result in worsening of electrolyte
disturbances as electrolyte rich secretions will enter the jejunum.

In some individuals a colectomy or similar procedure results in formation of an end or loop ileostomy.
Ileostomies typically lose between 500 and 1000ml over a 24 hour period and patients with high
output ileostomies can rapidly become dehydrated. Ileostomy effluent typically contains 126mmol/L
of sodium and 22mmol/L of potassium. Knowledge of this fluid composition should guide fluid
prescribing in replacing losses.

Next question
A 73 year old lady is admitted for a laparoscopic cholecystectomy. During her pre-operative
assessment it is noted that she is receiving furosemide for the treatment of hypertension. Where is
the site of action of this diuretic?

Proximal convoluted tubule

Descending limb of the loop of Henle

Ascending limb of the loop of Henle

Distal convoluted tubule

Collecting ducts

Action of furosemide = ascending limb of the loop of Henle

Furosemide and bumetanide are loop diuretics that act by inhibiting the Na-K-Cl cotransporter in the
thick ascending limb of the loop of Henle, reducing the absorption of NaCl.

Please rate this question:

Discuss and give feedback

Next question

Diuretic agents

The diuretic drugs are divided into three major classes, which are distinguished according to the site
at which they impair sodium reabsorption: loop diuretics in the thick ascending loop of Henle,
thiazide type diuretics in the distal tubule and connecting segment; and potassium sparing diuretics
in the aldosterone - sensitive principal cells in the cortical collecting tubule.
In the kidney, sodium is reabsorbed through Na+/ K+ ATPase pumps located on the basolateral
membrane. These pumps return reabsorbed sodium to the circulation and maintain low intracellular
sodium levels. This latter effect ensures a constant concentration gradient.
Physiological effects of commonly used diuretics

Carrier or channel Percentage of filtered sodium


Site of action Diuretic inhibited excreted

Ascending limb of loop of Frusemide Na+/K+ 2Cl - carrier Up to 25%


Henle

Distal tubule and connecting Thiazides Na+Cl- carrier Between 3 and 5%


segment

Cortical collecting tubule Spironolactone Na+/K+ ATP ase pump Between 1 and 2%

Next question
Which of the following physiological changes do not occur following tracheostomy?

Alveolar ventilation is increased.

Anatomical dead space is reduced by 50%.

Work of breathing is increased.

Proportion of ciliated epithelial cells in the trachea may decrease.

Splinting of the larynx may lead to swallowing difficulties.

Work of breathing is decreased which is one reasons it is popular option for weaning ventilated
patients. Humidified air in this setting helps to reduce the viscosity of mucous that forms.
Please rate this question:

Discuss and give feedback


Next question

Trachea

Trachea
Location C6 vertebra to the upper border of T5 vertebra (bifurcation)

Arterial and venous supply Inferior thyroid arteries and the thyroid venous plexus.

Nerve Branches of vagus, sympathetic and the recurrent nerves

Relations in the neck


Anterior(Superior to  Isthmus of the thyroid gland
inferior)  Inferior thyroid veins
 Arteria thyroidea ima (when that vessel exists)
 Sternothyroid
 Sternohyoid
 Cervical fascia
 Anastomosing branches between the anterior jugular
veins

Posterior Oesophagus.

Laterally  Common carotid arteries


 Right and left lobes of the thyroid gland
 Inferior thyroid arteries
 Recurrent laryngeal nerves

Relations in the thorax

Anterior

 Manubrium, the remains of the thymus, the aortic arch, left common carotid arteries, and the
deep cardiac plexus

Lateral

 In the superior mediastinum, on the right side is the pleura and right vagus; on its left side
are the left recurrent nerve, the aortic arch, and the left common carotid and subclavian
arteries.

Next question
What is the substrate of renin?

Aldosterone

Angiotensinogen

Angiotensin converting enzyme

Angiotensin I

Angiotensin II

Theme from January 2015 Exam


Renin hydrolyses angiotensinogen to form angiotensin I.
Image sourced from Wikipedia

Please rate this question:

Discuss and give feedback


Next question

Renin

Renin is secreted by juxtaglomerular cells and hydrolyses angiotensinogen to produce angiotensin I


Factors stimulating renin secretion

 Hypotension causing reduced renal perfusion


 Hyponatraemia
 Sympathetic nerve stimulation
 Catecholamines
 Erect posture

Factors reducing renin secretion

 Drugs: beta-blockers, NSAIDs

Next question
A 28 year old man is shot in the abdomen and haemorrhages. Which of the following substances will
produce vasoconstriction in response to this process?

Renin

Angiotensin I

Angiotensin II

Aldosterone

None of the above

Theme from April 2014 exam


Renin does not cause vasoconstriction. Angiotensin I is biologically inactive. Aldosterone will
increase blood pressure but does not have direct vasospastic effects.
Please rate this question:

Discuss and give feedback


Next question

Shock

 Shock occurs when there is insufficient tissue perfusion.


 The pathophysiology of shock is an important surgical topic and may be divided into the
following aetiological groups:
 Septic
 Haemorrhagic
 Neurogenic
 Cardiogenic
 Anaphylactic

Septic shock
Septic shock is a major problem and those patients with severe sepsis have a mortality rate in
excess of 40%. In those who are admitted to intensive care mortality ranges from 6% with no organ
failure to 65% in those with 4 organ failure.

Sepsis is defined as an infection that triggers a particular Systemic Inflammatory Response


Syndrome (SIRS). This is characterised by body temperature outside 36 oC - 38 o C, HR >90
beats/min, respiratory rate >20/min, WBC count >12,000/mm 3 or < 4,000/mm 3, altered mental state
or hyperglycaemia (in absence of diabetes).
Patients with infections and two or more elements of SIRS meet the diagnostic criteria for sepsis.
Those with organ failure have severe sepsis and those with refractory hypotension -septic shock.

During the septic process there is marked activation of the immune system with extensive cytokine
release. This may be coupled with or triggered by systemic circulation of bacterial toxins. These all
cause endothelial cell damage and neutrophil adhesion. The overall hallmarks are thus those
ofexcessive inflammation, coagulation and fibrinolytic suppression.

The surviving sepsis campaign (2012) highlights the following key areas for attention:

 Prompt administration of antibiotics to cover all likely pathogens coupled with a rigorous
search for the source of infection.
 Haemodynamic stabilisation. Many patients are hypovolaemic and require aggressive fluid
administration. Aim for CVP 8-12 cm H2O, MAP >65mmHg.
 Modulation of the septic response. This includes manoeuvres to counteract the changes and
includes measures such as tight glycaemic control. The routine use of steroids is not
advised.

In surgical patients, the main groups with septic shock include those with anastomotic leaks,
abscesses and extensive superficial infections such as necrotising fasciitis. When performing
surgery the aim should be to undertake the minimum necessary to restore physiology. These
patients do not fare well with prolonged surgery. Definitive surgery can be more safely undertaken
when physiology is restored and clotting in particular has been normalised.

Haemorrhagic shock
The average adult blood volume comprises 7% of body weight. Thus in the 70 Kg adult this will
equate to 5 litres. This changes in children (8-9% body weight) and is slightly lower in the elderly.

The table below outlines the 4 major classes of haemorrhagic shock and their associated
physiological sequelae:

Parameter Class I Class II Class III Class IV

Blood loss ml <750ml 750-1500ml 1500-2000ml >2000ml

Blood loss % <15% 15-30% 30-40% >40%

Pulse rate <100 >100 >120 >140

Blood pressure Normal Normal Decreased Decreased


Parameter Class I Class II Class III Class IV

Respiratory rate 14-20 20-30 30-40 >35

Urine output >30ml 20-30ml 5-15ml <5ml

Symptoms Normal Anxious Confused Lethargic

Decreasing blood pressure during haemorrhagic shock causes organ hypoperfusion and relative
myocardial ischaemia. The cardiac index gives a numerical value for tissue oxygen delivery and is
given by the equation: Cardiac index= Cardiac output/ body surface area. Where Hb is
haemoglobin concentration in blood and SaO2 the saturation and PaO2 the partial pressure of
oxygen. Detailed knowledge of this equation is required for the MRCS Viva but not for part A,
although you should understand the principle.

In patients suffering from trauma the most likely cause of shock is haemorrhage. However, the
following may also be the cause or occur concomitantly:

 Tension pneumothorax
 Spinal cord injury
 Myocardial contusion
 Cardiac tamponade

When assessing trauma patients it is worth remembering that in order to generate a palpable
femoral pulse an arterial pressure of >65mmHg is required.

Once bleeding is controlled and circulating volume normalised the levels of transfusion should be to
maintain a Hb of 7-8 in those with no risk factors for tissue hypoxia and Hb 10 for those who have
such risk factors.

Neurogenic shock
This occurs most often following a spinal cord transection, usually at a high level. There is
resultant interruption of the autonomic nervous system. The result is either decreased sympathetic
tone or increased parasympathetic tone, the effect of which is a decrease in peripheral vascular
resistance mediated by marked vasodilation.

This results in decreased preload and thus decreased cardiac output (Starlings law). There is
decreased peripheral tissue perfusion and shock is thus produced. In contrast with many other types
of shock peripheral vasoconstrictors are used to return vascular tone to normal.

Cardiogenic shock
In medical patients the main cause is ischaemic heart disease. In the traumatic setting direct
myocardial trauma or contusion is more likely. Evidence of ECG changes and overlying sternal
fractures or contusions should raise the suspicion of injury. Treatment is largely supportive and
transthoracic echocardiography should be used to determine evidence of pericardial fluid or direct
myocardial injury. The measurement of troponin levels in trauma patients may be undertaken but
they are less useful in delineating the extent of myocardial trauma than following MI.

When cardiac injury is of a blunt nature and is associated with cardiogenic shock the right side of the
heart is the most likely site of injury with chamber and or valve rupture. These patients require
surgery to repair these defects and will require cardiopulmonary bypass to achieve this. Some may
require intra aortic balloon pump as a bridge to surgery.

Anaphylactic shock
Anaphylaxis may be defined as a severe, life-threatening, generalised or systemic
hypersensitivity reaction.

Anaphylaxis is one of the few times when you would not have time to look up the dose of a
medication. The Resuscitation Council guidelines on anaphylaxis have recently been updated.
Adrenaline is by far the most important drug in anaphylaxis and should be given as soon as
possible. The recommended doses for adrenaline, hydrocortisone and chlorpheniramine are as
follows:

Adrenaline Hydrocortisone Chlorpheniramine

< 6 months 150 mcg (0.15ml 1 in 1,000) 25 mg 250 mcg/kg

6 months - 6 years 150 mcg (0.15ml 1 in 1,000) 50 mg 2.5 mg

6-12 years 300 mcg (0.3ml 1 in 1,000) 100 mg 5 mg

Adult and child 12 years 500 mcg (0.5ml 1 in 1,000) 200 mg 10 mg

Adrenaline can be repeated every 5 minutes if necessary. The best site for IM injection is the
anterolateral aspect of the middle third of the thigh.

Common identified causes of anaphylaxis

 food (e.g. Nuts) - the most common cause in children


 drugs
 venom (e.g. Wasp sting)

Next question
A 43 year old lady is recovering on the intensive care unit following a Whipples procedure. She has
a central venous line in situ. Which of the following will lead to the "y" descent on the waveform
trace?

Ventricular contraction

Emptying of the right atrium

Emptying of the right ventricle

Opening of the pulmonary valve

Cardiac tamponade

JVP
3 Upward deflections and 2 downward deflections

Upward deflections

 a wave = atrial contraction


 c wave = ventricular contraction
 v wave = atrial venous filling

Downward deflections

 x wave = atrium relaxes and tricuspid valve moves down


 y wave = ventricular filling

Theme from January 2012

The 'y' descent represents the emptying of the atrium and the filling of the right ventricle.
Please rate this question:

Discuss and give feedback


Next question

Cardiac physiology
 The heart has four chambers ejecting blood into both low pressure and high pressure
systems.
 The pumps generate pressures of between 0-25mmHg on the right side and 0-120 mmHg on
the left.
 At rest diastole comprises 2/3 of the cardiac cycle.
 The product of the frequency of heart rate and stroke volume combine to give the cardiac
output which is typically 5-6L per minute.

Detailed descriptions of the various waveforms are often not a feature of MRCS A (although they are
on the syllabus). However, they are a very popular topic for surgical physiology in the MRCS B
exam.

Electrical properties

 Intrinsic myogenic rhythm within cardiac myocytes means that even the denervated heart is
capable of contraction.
 In the normal situation the cardiac impulse is generated in the sino atrial node in the right
atrium and conveyed to the ventricles via the atrioventricular node.
 The sino atrial node is also capable of spontaneous discharge and in the absence of
background vagal tone will typically discharge around 100x per minute. Hence the higher
resting heart rate found in cardiac transplant cases. In the SA and AV nodes the resting
membrane potential is lower than in surrounding cardiac cells and will slowly depolarise from
-70mV to around -50mV at which point an action potential is generated.
 Differences in the depolarisation slopes between SA and AV nodes help to explain why the
SA node will depolarise first. The cells have a refractory period during which they cannot be
re-stimulated and this period allows for adequate ventricular filling. In pathological
tachycardic states this time period is overridden and inadequate ventricular filling may then
occur, cardiac output falls and syncope may ensue.

Parasympathetic fibres project to the heart via the vagus and will release acetylcholine. Sympathetic
fibres release nor adrenaline and circulating adrenaline comes from the adrenal medulla.
Noradrenaline binds to β 1 receptors in the SA node and increases the rate of pacemaker potential
depolarisation.

Cardiac cycle
Image sourced from Wikipedia

 Mid diastole: AV valves open. Ventricles hold 80% of final volume. Outflow valves shut.
Aortic pressure is high.

 Late diastole: Atria contract. Ventricles receive 20% to complete filling. Typical end diastolic
volume 130-160ml.

 Early systole: AV valves shut. Ventricular pressure rises. Isovolumetric ventricular


contraction. AV Valves bulge into atria (c-wave). Aortic and pulmonary pressure exceeded-
blood is ejected. Shortening of ventricles pulls atria downwards and drops intra atrial
pressure (x-descent).

 Late systole: Ventricular muscles relax and ventricular pressures drop. Although ventricular
pressure drops the aortic pressure remains constant owing to peripheral vascular resistance
and elastic property of the aorta. Brief period of retrograde flow that occurs in aortic recoil
shuts the aortic valve. Ventricles will contain 60ml end systolic volume. The average stroke
volume is 70ml (i.e. Volume ejected).
 Early diastole: All valves are closed. Isovolumetric ventricular relaxation occurs. Pressure
wave associated with closure of the aortic valve increases aortic pressure. The pressure dip
before this rise can be seen on arterial waveforms and is called the incisura. During systole
the atrial pressure increases such that it is now above zero (v- wave). Eventually atrial
pressure exceed ventricular pressure and AV valves open - atria empty passively into
ventricles and atrial pressure falls (y -descent )

The negative atrial pressures are of clinical importance as they can allow air embolization to occur if
the neck veins are exposed to air. This patient positioning is important in head and neck surgery to
avoid this occurrence if veins are inadvertently cut, or during CVP line insertion.

Mechanical properties

 Preload = end diastolic volume


 Afterload = aortic pressure

It is important to understand the principles of Laplace's law in surgery.

 It states that for hollow organs with a circular cross section, the total circumferential wall
tension depends upon the circumference of the wall, multiplied by the thickness of the wall
and on the wall tension.
 The total luminal pressure depends upon the cross sectional area of the lumen and the
transmural pressure. Transmural pressure is the internal pressure minus external pressure
and at equilibrium the total pressure must counterbalance each other.
 In terms of cardiac physiology the law explains that the rise in ventricular pressure that
occurs during the ejection phase is due to physical change in heart size. It also explains why
a dilated diseased heart will have impaired systolic function.

Starlings law

 Increase in end diastolic volume will produce larger stroke volume.


 This occurs up to a point beyond which cardiac fibres are excessively stretched and stroke
volume will fall once more. It is important for the regulation of cardiac output in cardiac
transplant patients who need to increase their cardiac output.

Baroreceptor reflexes

 Baroreceptors located in aortic arch and carotid sinus.


 Aortic baroreceptor impulses travel via the vagus and from the carotid via the
glossopharyngeal nerve.
 They are stimulated by arterial stretch.
 Even at normal blood pressures they are tonically active.
 Increase in baroreceptor discharge causes:
*Increased parasympathetic discharge to the SA node.
*Decreased sympathetic discharge to ventricular muscle causing decreased contractility and fall in
stroke volume.
*Decreased sympathetic discharge to venous system causing increased compliance.
*Decreased peripheral arterial vascular resistance

Atrial stretch receptors

 Located in atria at junction between pulmonary veins and vena cava.


 Stimulated by atrial stretch and are thus low pressure sensors.
 Increased blood volume will cause increased parasympathetic activity.
 Very rapid infusion of blood will result in increase in heart rate mediated via atrial receptors:
theBainbridge reflex.
 Decreases in receptor stimulation results in increased sympathetic activity this will decrease
renal blood flow-decreases GFR-decreases urinary sodium excretion-renin secretion by
juxtaglomerular apparatus-Increase in angiotensin II.
 Increased atrial stretch will also result in increased release of atrial natriuretic peptide.

Next question
Which of the following are not characteristic features of central chemoreceptors in the control of
ventilation?

They are located in the medulla oblongata

They are stimulated primarily by venous hypercapnia

They are relatively insensitive to hypoxia

They may be affected by changes in the pH of CSF

During acute hypercapnia the carotid receptors will be stimulated first

Theme from April 2014 exam


They are stimulated by arterial carbon dioxide. It takes longer to equilibrate than the peripheral
chemoreceptors located in the carotid. They are less sensitive to acidity due to the blood brain
barrier.
Please rate this question:

Discuss and give feedback


Next question

Control of ventilation

 Control of ventilation is coordinated by the respiratory centres, chemoreceptors, lung


receptors and muscles.
 Automatic, involuntary control of respiration occurs from the medulla.
 The respiratory centres control the respiratory rate and the depth of respiration.

Respiratory centres
Medullary Inspiratory and expiratory neurones. Has ventral group which controls
respiratory centre forced voluntary expiration and the dorsal group controls inspiration.
Depressed by opiates.

Apneustic centre Lower pons


Stimulates inspiration - activates and prolongs inhalation
Overridden by pneumotaxic control to end inspiration

Pneumotaxic centre Upper pons, inhibits inspiration at a certain point. Fine tunes the
respiratory rate.

Ventillatory variables

 Levels of pCO2 most important in ventilation control


 Levels of O2 are less important.
 Peripheral chemoreceptors: located in the bifurcation of carotid arteries and arch of the
aorta. They respond to changes in reduced pO2, increased H+ and increased pCO2 in
ARTERIAL BLOOD.
 Central chemoreceptors: located in the medulla. Respond to increased H+ in BRAIN
INTERSTITIAL FLUID to increase ventilation. NB the central receptors are NOT influenced
by O2levels.

Lung receptors include:

 Stretch receptors: respond to lung stretching causing a reduced respiratory rate


 Irritant receptors: respond to smoke etc causing bronchospasm
 J (juxtacapillary) receptors

Next question
A 32 year old man has a glomerular filtration rate of 110ml / minute at a systolic blood pressure of
120/80. If his blood pressure were to fall to 100/70 what would glomerular filtration rate be?

110ml / minute

100ml/ minute

55ml/ minute

25ml/ minute

75ml/ minute

Theme from April 2014 exam


The proposed drop in blood pressure falls within the range within which the kidney autoregulates its
blood supply. GFR will therefore remain unchanged.
Please rate this question:

Discuss and give feedback


Next question

Renal Physiology

Overview

 Each nephron is supplied with blood from an afferent arteriole that opens onto the glomerular
capillary bed.
 Blood then flows to an efferent arteriole, supplying the peritubular capillaries and medullary
vasa recta.
 The kidney receives up to 25% of resting cardiac output.

Control of blood flow

 The kidney is able to autoregulate its blood flow between systolic pressures of 80- 180mmHg
so there is little variation in renal blood flow.
 This is achieved by myogenic control of arteriolar tone, both sympathetic input and hormonal
signals (e.g. renin) are responsible.
Glomerular structure and function

 Blood inside the glomerulus has considerable hydrostatic pressure.


 The basement membrane has pores that will allow free diffusion of smaller solutes, larger
negatively charged molecules such as albumin are unable to cross.
 The glomerular filtration rate (GFR) is equal to the concentration of a solute in the urine,
times the volume of urine produced per minute, divided by the plasma concentration
(assuming that the solute is freely diffused e.g. inulin).
 In clinical practice creatinine is used because it is subjected to very little proximal tubular
secretion.
 Although subject to variability, the typical GFR is 125ml per minute.
 Glomerular filtration rate = Total volume of plasma per unit time leaving the capillaries and
entering the bowman's capsule
 Renal clearance = volume plasma from which a substance is removed per minute by the
kidneys

Substances used to measure GFR have the following features:


1. Inert
2. Free filtration from the plasma at the glomerulus (not protein bound)
3. Not absorbed or secreted at the tubules
4. Plasma concentration constant during urine collection

Examples: inulin, creatinine

GFR = urine concentration (mmol/l) x urine volume (ml/min)


--------------------------------------------------------------------------
plasma concentration (mmol/l)

 The clearance of a substance is dependent not only on its diffusivity across the basement
membrane but also subsequent tubular secretion and / or reabsorption.
 So glucose which is freely filtered across the basement membrane is usually reabsorbed
from tubules giving a clearance of zero.

Tubular function

 Reabsorption and secretion of substances occurs in the tubules.


 In the proximal tubule substrates such as glucose, amino acids and phosphate are co-
transported with sodium across the semi permeable membrane.
 Up to two thirds of filtered water is reabsorbed in the proximal tubules.
 This will lead to increase in urea concentration in the distal tubule allowing for its increased
diffusion.
 Substances to be secreted into the tubules are taken up from the peritubular blood by tubular
cells.
 Solutes such as paraaminohippuric acid are cleared with a single passage through the
kidneys and this is why it is used to measure renal plasma flow. Ions such as calcium and
phosphate will have a tubular reabsorption that is influenced by plasma PTH levels.
 Potassium may be both secreted and re-absorbed and is co-exchanged with sodium.

Loop of Henle

 Approximately 60 litres of water containing 9000mmol sodium enters the descending limb of
the loop of Henle in 24 hours.
 Loops from the juxtamedullary nephrons run deep into the medulla.
 The osmolarity of fluid changes and is greatest at the tip of the papilla.
 The thin ascending limb is impermeable to water, but highly permeable to sodium and
chloride ions.
 This loss means that at the beginning of the thick ascending limb the fluid is hypo osmotic
compared with adjacent interstitial fluid.
 In the thick ascending limb the reabsorption of sodium and chloride ions occurs by both
facilitated and passive diffusion pathways.
 The loops of Henle are co-located with vasa recta, these will have similar solute
compositions to the surrounding extracellular fluid so preventing the diffusion and
subsequent removal of this hypertonic fluid.
 The energy dependent reabsorption of sodium and chloride in the thick ascending limb helps
to maintain this osmotic gradient.

Next question
Which of the following does not stimulate insulin release?

Gastrin

Atenolol

Protein

Secretin

Vagal cholinergic action

Beta blockers inhibit the release of insulin.

Stimulation of insulin release:

 Glucose
 Amino acid
 Vagal cholinergic
 Secretin/Gastrin/CCK
 Fatty acids
 Beta adrenergic drugs

Please rate this question:

Discuss and give feedback


Next question

Insulin

Insulin is a peptide hormone, produced by beta cells of the pancreas, and is central to regulating
carbohydrate and fat metabolism in the body. Insulin causes cells in the liver, skeletal muscles, and
fat tissue to absorb glucose from the blood. In the liver and skeletal muscles, glucose is stored as
glycogen, and in fat cells (adipocytes) it is stored as triglycerides.

Structure
The human insulin protein is composed of 51 amino acids, and has a molecular weight of 5808 Da. It
is a dimer of an A-chain and a B-chain, which are linked together by disulfide bonds.

Synthesis
Pro-insulin is formed by the rough endoplasmic reticulum in pancreatic beta cells. Then pro-insulin is
cleaved to form insulin and C-peptide. Insulin is stored in secretory granules and released in
response to Ca2+.

Function

 Secreted in response to hyperglycaemia


 Glucose utilisation and glycogen synthesis
 Inhibits lipolysis
 Reduces muscle protein loss

Next question
A 63 year old female is referred to the surgical clinic with an iron deficiency anaemia. Her past
medical history includes a left hemi colectomy but no other co-morbidities. At what site is most
dietary iron absorbed?

Stomach

Duodenum

Proximal ileum

Distal ileum

Colon

Theme from April 2014 exam


Iron is best absorbed from the proximal small bowel (duodenum and jejunum) in the Fe 2+ state. Iron
is transported across the small bowel mucosa by a divalent membrane transporter protein (hence
the improved absorption of Fe 2+). The intestinal cells typically store the bound iron as ferritin. Cells
requiring iron will typically then absorb the complex as needed.
Please rate this question:

Discuss and give feedback


Next question

Iron metabolism

Absorption  Duodenum and upper jejunum


 About 10% of dietary iron absorbed
 Fe2+ (ferrous iron) much better absorbed than Fe3+ (ferric iron)
 Ferrous iron is oxidized to form ferric iron, which is combined with
apoferritin to form ferritin
 Absorption is regulated according to body's need
 Increased by vitamin C, gastric acid
 Decreased by proton pump inhibitors, tetracycline, gastric achlorhydria,
tannin (found in tea)

Transport In plasma as Fe3+ bound to transferrin


Storage Ferritin (or haemosiderin) in bone marrow

Excretion Lost via intestinal tract following desquamation

Distribution in body
Total body iron 4g

Haemoglobin 70%

Ferritin and haemosiderin 25%

4%
Myoglobin

Plasma iron 0.1%

Next question
Which of the following haemodynamic changes is not seen in hypovolaemic shock?

Decreased cardiac output

Increased heart rate

Reduced left ventricle filling pressures

Reduced blood pressure

Reduced systemic vascular resistance

Cardiogenic Shock:
e.g. MI, valve abnormality

increased SVR (vasoconstriction in response to low BP)


increased HR (sympathetic response)
decreased cardiac output
decreased blood pressure

Hypovolaemic shock:
blood volume depletion
e.g. haemorrhage, vomiting, diarrhoea, dehydration, third-space losses during major operations

increased SVR
increased HR
decreased cardiac output
decreased blood pressure

Septic shock:
occurs when the peripheral vascular dilatation causes a fall in SVR
similar response may occur in anaphylactic shock, neurogenic shock

reduced SVR
increased HR
normal/increased cardiac output
decreased blood pressure

SVR will typically increase


Please rate this question:

Discuss and give feedback


Next question
Shock

 Shock occurs when there is insufficient tissue perfusion.


 The pathophysiology of shock is an important surgical topic and may be divided into the
following aetiological groups:
 Septic
 Haemorrhagic
 Neurogenic
 Cardiogenic
 Anaphylactic

Septic shock
Septic shock is a major problem and those patients with severe sepsis have a mortality rate in
excess of 40%. In those who are admitted to intensive care mortality ranges from 6% with no organ
failure to 65% in those with 4 organ failure.

Sepsis is defined as an infection that triggers a particular Systemic Inflammatory Response


Syndrome (SIRS). This is characterised by body temperature outside 36 oC - 38 o C, HR >90
beats/min, respiratory rate >20/min, WBC count >12,000/mm 3 or < 4,000/mm 3, altered mental state
or hyperglycaemia (in absence of diabetes).

Patients with infections and two or more elements of SIRS meet the diagnostic criteria for sepsis.
Those with organ failure have severe sepsis and those with refractory hypotension -septic shock.

During the septic process there is marked activation of the immune system with extensive cytokine
release. This may be coupled with or triggered by systemic circulation of bacterial toxins. These all
cause endothelial cell damage and neutrophil adhesion. The overall hallmarks are thus those
ofexcessive inflammation, coagulation and fibrinolytic suppression.

The surviving sepsis campaign (2012) highlights the following key areas for attention:

 Prompt administration of antibiotics to cover all likely pathogens coupled with a rigorous
search for the source of infection.
 Haemodynamic stabilisation. Many patients are hypovolaemic and require aggressive fluid
administration. Aim for CVP 8-12 cm H2O, MAP >65mmHg.
 Modulation of the septic response. This includes manoeuvres to counteract the changes and
includes measures such as tight glycaemic control. The routine use of steroids is not
advised.

In surgical patients, the main groups with septic shock include those with anastomotic leaks,
abscesses and extensive superficial infections such as necrotising fasciitis. When performing
surgery the aim should be to undertake the minimum necessary to restore physiology. These
patients do not fare well with prolonged surgery. Definitive surgery can be more safely undertaken
when physiology is restored and clotting in particular has been normalised.

Haemorrhagic shock
The average adult blood volume comprises 7% of body weight. Thus in the 70 Kg adult this will
equate to 5 litres. This changes in children (8-9% body weight) and is slightly lower in the elderly.
The table below outlines the 4 major classes of haemorrhagic shock and their associated
physiological sequelae:

Parameter Class I Class II Class III Class IV

Blood loss ml <750ml 750-1500ml 1500-2000ml >2000ml

Blood loss % <15% 15-30% 30-40% >40%

Pulse rate <100 >100 >120 >140

Blood pressure Normal Normal Decreased Decreased

Respiratory rate 14-20 20-30 30-40 >35

Urine output >30ml 20-30ml 5-15ml <5ml

Symptoms Normal Anxious Confused Lethargic

Decreasing blood pressure during haemorrhagic shock causes organ hypoperfusion and relative
myocardial ischaemia. The cardiac index gives a numerical value for tissue oxygen delivery and is
given by the equation: Cardiac index= Cardiac output/ body surface area. Where Hb is
haemoglobin concentration in blood and SaO2 the saturation and PaO2 the partial pressure of
oxygen. Detailed knowledge of this equation is required for the MRCS Viva but not for part A,
although you should understand the principle.

In patients suffering from trauma the most likely cause of shock is haemorrhage. However, the
following may also be the cause or occur concomitantly:

 Tension pneumothorax
 Spinal cord injury
 Myocardial contusion
 Cardiac tamponade

When assessing trauma patients it is worth remembering that in order to generate a palpable
femoral pulse an arterial pressure of >65mmHg is required.
Once bleeding is controlled and circulating volume normalised the levels of transfusion should be to
maintain a Hb of 7-8 in those with no risk factors for tissue hypoxia and Hb 10 for those who have
such risk factors.

Neurogenic shock
This occurs most often following a spinal cord transection, usually at a high level. There is
resultant interruption of the autonomic nervous system. The result is either decreased sympathetic
tone or increased parasympathetic tone, the effect of which is a decrease in peripheral vascular
resistance mediated by marked vasodilation.

This results in decreased preload and thus decreased cardiac output (Starlings law). There is
decreased peripheral tissue perfusion and shock is thus produced. In contrast with many other types
of shock peripheral vasoconstrictors are used to return vascular tone to normal.

Cardiogenic shock
In medical patients the main cause is ischaemic heart disease. In the traumatic setting direct
myocardial trauma or contusion is more likely. Evidence of ECG changes and overlying sternal
fractures or contusions should raise the suspicion of injury. Treatment is largely supportive and
transthoracic echocardiography should be used to determine evidence of pericardial fluid or direct
myocardial injury. The measurement of troponin levels in trauma patients may be undertaken but
they are less useful in delineating the extent of myocardial trauma than following MI.

When cardiac injury is of a blunt nature and is associated with cardiogenic shock the right side of the
heart is the most likely site of injury with chamber and or valve rupture. These patients require
surgery to repair these defects and will require cardiopulmonary bypass to achieve this. Some may
require intra aortic balloon pump as a bridge to surgery.

Anaphylactic shock
Anaphylaxis may be defined as a severe, life-threatening, generalised or systemic
hypersensitivity reaction.

Anaphylaxis is one of the few times when you would not have time to look up the dose of a
medication. The Resuscitation Council guidelines on anaphylaxis have recently been updated.
Adrenaline is by far the most important drug in anaphylaxis and should be given as soon as
possible. The recommended doses for adrenaline, hydrocortisone and chlorpheniramine are as
follows:

Adrenaline Hydrocortisone Chlorpheniramine

< 6 months 150 mcg (0.15ml 1 in 1,000) 25 mg 250 mcg/kg

6 months - 6 years 150 mcg (0.15ml 1 in 1,000) 50 mg 2.5 mg

6-12 years 300 mcg (0.3ml 1 in 1,000) 100 mg 5 mg


Adult and child 12 years 500 mcg (0.5ml 1 in 1,000) 200 mg 10 mg

Adrenaline can be repeated every 5 minutes if necessary. The best site for IM injection is the
anterolateral aspect of the middle third of the thigh.

Common identified causes of anaphylaxis

 food (e.g. Nuts) - the most common cause in children


 drugs
 venom (e.g. Wasp sting)

Next question
An otherwise fit 30 year old male donates 500ml of blood. Which of the processes outlined below is
most likely to occur?

Reduction of urine output

Activation of the renin angiotensin system

Sweating

Fall in mean arterial pressure

Tachypnoea

Theme from 2011 Exam


The loss of 500ml (assuming a 70 Kg male) will usually be sufficient to activate the renin angiotensin
system. It is unlikely that it would cause any other physiological disturbance.

Please rate this question:

Discuss and give feedback

Next question

Bleeding

The initial response to bleeding, even if of relatively small volume is generalised splanchnic
vasoconstriction mediated by activation of the sympathetic nervous system. This process of
vasoconstriction is usually sufficient to maintain renal perfusion and cardiac output if the volume of
blood lost is small. Over the following hours the circulating fluid volume is restored and normal
haemodynamics resume. Loss of greater volumes of blood will typically result in activation in the
renin angiotensin system (see diagram below).
Image sourced from Wikipedia

Where the source of bleeding ceases these physiological measures will restore circulating volume.
Ongoing bleeding will result in haemorrhagic shock.
Blood loss is typically quantified by the degree of shock produced as outlined below:

Parameter Class I Class II Class III Class IV

Blood loss ml <750ml 750-1500ml 1500-2000ml >2000ml

Blood loss % <15% 15-30% 30-40% >40%

Pulse rate <100 >100 >120 >140

Blood pressure Normal Normal Decreased Decreased

Respiratory rate 14-20 20-30 30-40 >35

Urine output >30ml 20-30ml 5-15ml <5ml

Symptoms Normal Anxious Confused Lethargic

Next question
A 25 year old man is undergoing respiratory spirometry. He takes a maximal inspiration and
maximally exhales. Which of the following measurements will best illustrate this process?

Functional residual capacity

Vital capacity

Inspiratory capacity

Maximum voluntary ventilation

Tidal volume

Theme from April 2012 exam


Theme from April 2014 exam
The maximum voluntary ventilation is the maximal ventilation over the course of 1 minute.
Please rate this question:

Discuss and give feedback


Next question

Lung volumes

The diagram demonstrates lung volumes and capacities


Image sourced from Wikipedia

Definitions

Tidal volume (TV)  Is the volume of air inspired and expired during each
ventilatory cycle at rest.
 It is normally 500mls in males and 340mls in females.

Inspiratory reserve  Is the maximum volume of air that can be forcibly inhaled
volume (IRV) following a normal inspiration. 3000mls.

Expiratory reserve  Is the maximum volume of air that can be forcibly exhaled
volume (ERV) following a normal expiration. 1000mls.

Residual volume (RV)  Is that volume of air remaining in the lungs after a maximal
expiration.
 RV = FRC - ERV. 1500mls.

Functional residual  Is the volume of air remaining in the lungs at the end of a
capacity (FRC) normal expiration.
 FRC = RV + ERV. 2500mls.

Vital capacity (VC)  Is the maximal volume of air that can be forcibly exhaled after
a maximal inspiration.
 VC = TV + IRV + ERV. 4500mls in males, 3500mls in
females.

Total lung capacity  Is the volume of air in the lungs at the end of a maximal
(TLC) inspiration.
 TLC = FRC + TV + IRV = VC + RV. 5500-6000mls.

Forced vital capacity  The volume of air that can be maximally forcefully exhaled.
(FVC)
Next question
Which of the following does not decrease the functional residual capacity?

Obesity

Pulmonary fibrosis

Muscle relaxants

Laparoscopic surgery

Upright position

Increased FRC:

 Erect position
 Emphysema
 Asthma

Decreased FRC:

 Pulmonary fibrosis
 Laparoscopic surgery
 Obesity
 Abdominal swelling
 Muscle relaxants

When the patient is upright the diaphragm and abdominal organs put less pressure on the lung
bases, allowing for an increase in the functional residual capacity (FRC). Other causes of increased
FRC include:

 Emphysema
 Asthma

In addition to those listed above, causes of reduced FRC include:

 Abdominal swelling
 Pulmonary oedema
 Reduced muscle tone of the diaphragm
 Age

Please rate this question:

Discuss and give feedback


Next question

Lung volumes

The diagram demonstrates lung volumes and capacities

Image sourced from Wikipedia

Definitions

Tidal volume (TV)  Is the volume of air inspired and expired during each
ventilatory cycle at rest.
 It is normally 500mls in males and 340mls in females.

Inspiratory reserve  Is the maximum volume of air that can be forcibly inhaled
volume (IRV) following a normal inspiration. 3000mls.

Expiratory reserve  Is the maximum volume of air that can be forcibly exhaled
volume (ERV) following a normal expiration. 1000mls.

Residual volume (RV)  Is that volume of air remaining in the lungs after a maximal
expiration.
 RV = FRC - ERV. 1500mls.

Functional residual  Is the volume of air remaining in the lungs at the end of a
capacity (FRC) normal expiration.
 FRC = RV + ERV. 2500mls.

Vital capacity (VC)  Is the maximal volume of air that can be forcibly exhaled after
a maximal inspiration.
 VC = TV + IRV + ERV. 4500mls in males, 3500mls in
females.

Total lung capacity  Is the volume of air in the lungs at the end of a maximal
(TLC) inspiration.
 TLC = FRC + TV + IRV = VC + RV. 5500-6000mls.

Forced vital capacity  The volume of air that can be maximally forcefully exhaled.
(FVC)
Next question
Which of the following is the main site of dehydroepiandrosterone release?

Posterior pituitary

Zona reticularis of the adrenal gland

Zona glomerulosa of the adrenal gland

Juxtaglomerular apparatus of the kidney

Zona fasciculata of the adrenal gland

Adrenal cortex mnemonic: GFR - ACD

DHEA possesses some androgenic activity and is almost exclusively released from the adrenal
gland.
Please rate this question:

Discuss and give feedback


Next question

Renin-angiotensin-aldosterone system

Adrenal cortex (mnemonic GFR - ACD)

 Zona glomerulosa (on outside): mineralocorticoids, mainly aldosterone


 Zona fasciculata (middle): glucocorticoids, mainly cortisol
 Zona reticularis (on inside): androgens, mainly dehydroepiandrosterone (DHEA)

Renin

 Released by JGA cells in kidney in response to reduced renal perfusion, low sodium
 Hydrolyses angiotensinogen to form angiotensin I

Factors stimulating renin secretion


 Low BP
 Hyponatraemia
 Sympathetic nerve stimulation
 Catecholamines
 Erect posture

Angiotensin

 ACE in lung converts angiotensin I → angiotensin II


 Vasoconstriction leads to raised BP
 Stimulates thirst
 Stimulates aldosterone and ADH release

Aldosterone

 Released by the zona glomerulosa in response to raised angiotensin II, potassium, and
ACTH levels
 Causes retention of Na+ in exchange for K+/H+ in distal tubule

Image sourced from Wikipedia

Next question
Secretions from which of the following will contain the highest levels of potassium?

Rectum

Small bowel

Gallbladder

Pancreas

Stomach

Theme from April 2014 Exam


The rectum has the potential to generate secretions rich in potassium. This is the rationale behind
administration of resins for hyperkalaemia and the development of hypokalaemia in patients with
villous adenoma of the rectum.

Please rate this question:

Discuss and give feedback

Next question

Potassium secretion -GI tract

Potassium secretions

Salivary glands Variable may be up to 60mmol/L

Stomach 10 mmol/L
Bile 5 mmol/L

Pancreas 4-5 mmol/L

Small bowel 10 mmol/L

Rectum 30 mmol/L

The above table provides average figures only and the exact composition varies depending upon the
existence of disease, serum aldosterone levels and serum pH.

A key point to remember for the exam is that gastric potassium secretions are low. Hypokalaemia
may occur in vomiting, usually as a result of renal wasting of potassium, not because of potassium
loss in vomit.

Next question
What is the typical stroke volume in a resting 70 Kg man?

10ml

150ml

125ml

45ml

70ml

Theme from April 2014 Exam


Theme from September 2013 Exam
Stroke volumes range from 55-100ml.
Please rate this question:

Discuss and give feedback


Next question

Stroke volume-Cardiac physiology

The stroke volume equates to the volume of blood ejected from the ventricle during each cycle of
cardiac contraction. The volumes for both ventricles are typically equal and equate roughly to 70ml
for a 70Kg man. It is calculated by subtracting the end systolic volume from the end diastolic
volume.

Factors affecting stroke volume

 Cardiac size
 Contractility
 Preload
 Afterload

Next question
A patient loses 1.6L of fresh blood from their abdominal drain. Which of the following will not
decrease?

Cardiac output

Renin secretion

Firing of carotid baroreceptors

Firing of aortic baroreceptors

Blood pressure

Renin secretion will increase as systemic hypotension will cause impairment of renal blood flow.
Although the kidney can autoregulate its own blood flow over a range of systemic blood pressures a
loss of 1.6 L will usually produce an increase in renin secretion.
Please rate this question:

Discuss and give feedback


Next question

Shock

 Shock occurs when there is insufficient tissue perfusion.


 The pathophysiology of shock is an important surgical topic and may be divided into the
following aetiological groups:
 Septic
 Haemorrhagic
 Neurogenic
 Cardiogenic
 Anaphylactic

Septic shock
Septic shock is a major problem and those patients with severe sepsis have a mortality rate in
excess of 40%. In those who are admitted to intensive care mortality ranges from 6% with no organ
failure to 65% in those with 4 organ failure.

Sepsis is defined as an infection that triggers a particular Systemic Inflammatory Response


Syndrome (SIRS). This is characterised by body temperature outside 36 oC - 38 o C, HR >90
beats/min, respiratory rate >20/min, WBC count >12,000/mm 3 or < 4,000/mm 3, altered mental state
or hyperglycaemia (in absence of diabetes).
Patients with infections and two or more elements of SIRS meet the diagnostic criteria for sepsis.
Those with organ failure have severe sepsis and those with refractory hypotension -septic shock.

During the septic process there is marked activation of the immune system with extensive cytokine
release. This may be coupled with or triggered by systemic circulation of bacterial toxins. These all
cause endothelial cell damage and neutrophil adhesion. The overall hallmarks are thus those
ofexcessive inflammation, coagulation and fibrinolytic suppression.

The surviving sepsis campaign (2012) highlights the following key areas for attention:

 Prompt administration of antibiotics to cover all likely pathogens coupled with a rigorous
search for the source of infection.
 Haemodynamic stabilisation. Many patients are hypovolaemic and require aggressive fluid
administration. Aim for CVP 8-12 cm H2O, MAP >65mmHg.
 Modulation of the septic response. This includes manoeuvres to counteract the changes and
includes measures such as tight glycaemic control. The routine use of steroids is not
advised.

In surgical patients, the main groups with septic shock include those with anastomotic leaks,
abscesses and extensive superficial infections such as necrotising fasciitis. When performing
surgery the aim should be to undertake the minimum necessary to restore physiology. These
patients do not fare well with prolonged surgery. Definitive surgery can be more safely undertaken
when physiology is restored and clotting in particular has been normalised.

Haemorrhagic shock
The average adult blood volume comprises 7% of body weight. Thus in the 70 Kg adult this will
equate to 5 litres. This changes in children (8-9% body weight) and is slightly lower in the elderly.

The table below outlines the 4 major classes of haemorrhagic shock and their associated
physiological sequelae:

Parameter Class I Class II Class III Class IV

Blood loss ml <750ml 750-1500ml 1500-2000ml >2000ml

Blood loss % <15% 15-30% 30-40% >40%

Pulse rate <100 >100 >120 >140

Blood pressure Normal Normal Decreased Decreased


Parameter Class I Class II Class III Class IV

Respiratory rate 14-20 20-30 30-40 >35

Urine output >30ml 20-30ml 5-15ml <5ml

Symptoms Normal Anxious Confused Lethargic

Decreasing blood pressure during haemorrhagic shock causes organ hypoperfusion and relative
myocardial ischaemia. The cardiac index gives a numerical value for tissue oxygen delivery and is
given by the equation: Cardiac index= Cardiac output/ body surface area. Where Hb is
haemoglobin concentration in blood and SaO2 the saturation and PaO2 the partial pressure of
oxygen. Detailed knowledge of this equation is required for the MRCS Viva but not for part A,
although you should understand the principle.

In patients suffering from trauma the most likely cause of shock is haemorrhage. However, the
following may also be the cause or occur concomitantly:

 Tension pneumothorax
 Spinal cord injury
 Myocardial contusion
 Cardiac tamponade

When assessing trauma patients it is worth remembering that in order to generate a palpable
femoral pulse an arterial pressure of >65mmHg is required.

Once bleeding is controlled and circulating volume normalised the levels of transfusion should be to
maintain a Hb of 7-8 in those with no risk factors for tissue hypoxia and Hb 10 for those who have
such risk factors.

Neurogenic shock
This occurs most often following a spinal cord transection, usually at a high level. There is
resultant interruption of the autonomic nervous system. The result is either decreased sympathetic
tone or increased parasympathetic tone, the effect of which is a decrease in peripheral vascular
resistance mediated by marked vasodilation.

This results in decreased preload and thus decreased cardiac output (Starlings law). There is
decreased peripheral tissue perfusion and shock is thus produced. In contrast with many other types
of shock peripheral vasoconstrictors are used to return vascular tone to normal.

Cardiogenic shock
In medical patients the main cause is ischaemic heart disease. In the traumatic setting direct
myocardial trauma or contusion is more likely. Evidence of ECG changes and overlying sternal
fractures or contusions should raise the suspicion of injury. Treatment is largely supportive and
transthoracic echocardiography should be used to determine evidence of pericardial fluid or direct
myocardial injury. The measurement of troponin levels in trauma patients may be undertaken but
they are less useful in delineating the extent of myocardial trauma than following MI.

When cardiac injury is of a blunt nature and is associated with cardiogenic shock the right side of the
heart is the most likely site of injury with chamber and or valve rupture. These patients require
surgery to repair these defects and will require cardiopulmonary bypass to achieve this. Some may
require intra aortic balloon pump as a bridge to surgery.

Anaphylactic shock
Anaphylaxis may be defined as a severe, life-threatening, generalised or systemic
hypersensitivity reaction.

Anaphylaxis is one of the few times when you would not have time to look up the dose of a
medication. The Resuscitation Council guidelines on anaphylaxis have recently been updated.
Adrenaline is by far the most important drug in anaphylaxis and should be given as soon as
possible. The recommended doses for adrenaline, hydrocortisone and chlorpheniramine are as
follows:

Adrenaline Hydrocortisone Chlorpheniramine

< 6 months 150 mcg (0.15ml 1 in 1,000) 25 mg 250 mcg/kg

6 months - 6 years 150 mcg (0.15ml 1 in 1,000) 50 mg 2.5 mg

6-12 years 300 mcg (0.3ml 1 in 1,000) 100 mg 5 mg

Adult and child 12 years 500 mcg (0.5ml 1 in 1,000) 200 mg 10 mg

Adrenaline can be repeated every 5 minutes if necessary. The best site for IM injection is the
anterolateral aspect of the middle third of the thigh.

Common identified causes of anaphylaxis

 food (e.g. Nuts) - the most common cause in children


 drugs
 venom (e.g. Wasp sting)

Next question
What are the most likely effects of the release of vasopressin from the pituitary?

Vasoconstriction of the afferent glomerular arteriole

Increased permeability of the mesangial cells to glucose

Reduced permeability of the inner medullary portion of the collecting duct to urea

Increased secretion of aldosterone from the macula densa

Increased water permeability of the distal tubule cells of the kidney

ADH (vasopressin) results in the insertion of aquaporin channels in apical membrane of the distal
tubule and collecting ducts.
Please rate this question:

Discuss and give feedback


Next question

Renal Physiology

Overview

 Each nephron is supplied with blood from an afferent arteriole that opens onto the glomerular
capillary bed.
 Blood then flows to an efferent arteriole, supplying the peritubular capillaries and medullary
vasa recta.
 The kidney receives up to 25% of resting cardiac output.

Control of blood flow

 The kidney is able to autoregulate its blood flow between systolic pressures of 80- 180mmHg
so there is little variation in renal blood flow.
 This is achieved by myogenic control of arteriolar tone, both sympathetic input and hormonal
signals (e.g. renin) are responsible.

Glomerular structure and function


 Blood inside the glomerulus has considerable hydrostatic pressure.
 The basement membrane has pores that will allow free diffusion of smaller solutes, larger
negatively charged molecules such as albumin are unable to cross.
 The glomerular filtration rate (GFR) is equal to the concentration of a solute in the urine,
times the volume of urine produced per minute, divided by the plasma concentration
(assuming that the solute is freely diffused e.g. inulin).
 In clinical practice creatinine is used because it is subjected to very little proximal tubular
secretion.
 Although subject to variability, the typical GFR is 125ml per minute.
 Glomerular filtration rate = Total volume of plasma per unit time leaving the capillaries and
entering the bowman's capsule
 Renal clearance = volume plasma from which a substance is removed per minute by the
kidneys

Substances used to measure GFR have the following features:


1. Inert
2. Free filtration from the plasma at the glomerulus (not protein bound)
3. Not absorbed or secreted at the tubules
4. Plasma concentration constant during urine collection

Examples: inulin, creatinine

GFR = urine concentration (mmol/l) x urine volume (ml/min)


--------------------------------------------------------------------------
plasma concentration (mmol/l)

 The clearance of a substance is dependent not only on its diffusivity across the basement
membrane but also subsequent tubular secretion and / or reabsorption.
 So glucose which is freely filtered across the basement membrane is usually reabsorbed
from tubules giving a clearance of zero.

Tubular function

 Reabsorption and secretion of substances occurs in the tubules.


 In the proximal tubule substrates such as glucose, amino acids and phosphate are co-
transported with sodium across the semi permeable membrane.
 Up to two thirds of filtered water is reabsorbed in the proximal tubules.
 This will lead to increase in urea concentration in the distal tubule allowing for its increased
diffusion.
 Substances to be secreted into the tubules are taken up from the peritubular blood by tubular
cells.
 Solutes such as paraaminohippuric acid are cleared with a single passage through the
kidneys and this is why it is used to measure renal plasma flow. Ions such as calcium and
phosphate will have a tubular reabsorption that is influenced by plasma PTH levels.
 Potassium may be both secreted and re-absorbed and is co-exchanged with sodium.
Loop of Henle

 Approximately 60 litres of water containing 9000mmol sodium enters the descending limb of
the loop of Henle in 24 hours.
 Loops from the juxtamedullary nephrons run deep into the medulla.
 The osmolarity of fluid changes and is greatest at the tip of the papilla.
 The thin ascending limb is impermeable to water, but highly permeable to sodium and
chloride ions.
 This loss means that at the beginning of the thick ascending limb the fluid is hypo osmotic
compared with adjacent interstitial fluid.
 In the thick ascending limb the reabsorption of sodium and chloride ions occurs by both
facilitated and passive diffusion pathways.
 The loops of Henle are co-located with vasa recta, these will have similar solute
compositions to the surrounding extracellular fluid so preventing the diffusion and
subsequent removal of this hypertonic fluid.
 The energy dependent reabsorption of sodium and chloride in the thick ascending limb helps
to maintain this osmotic gradient.

Next question
Which of the following hormones is mainly responsible for sodium - potassium exchange in the
salivary ducts?

Vasopressin

Angiotensin I

Aldosterone

Somatostatin

Cholecystokinin

Aldosterone is responsible for regulating ion exchange in salivary glands. It acts on a sodium /
potassium ion exchange pump.It is a mineralocorticoid hormone derived from the zona glomerulosa
of the adrenal gland.
Please rate this question:

Discuss and give feedback


Next question

Parotid gland

Anatomy of the parotid gland


Location Overlying the mandibular ramus; anterior and inferior to the ear.

Salivary duct Crosses the masseter, pierces the buccinator and drains adjacent to the
2nd upper molar tooth (Stensen's duct).

Structures passing  Facial nerve (Mnemonic: The Zebra Buggered My Cat; Temporal
through the gland Zygomatic, Buccal, Mandibular, Cervical)
 External carotid artery
 Retromandibular vein
 Auriculotemporal nerve

Relations  Anterior: masseter, medial pterygoid, superficial temporal and


maxillary artery, facial nerve, stylomandibular ligament
 Posterior: posterior belly digastric muscle, sternocleidomastoid,
stylohyoid, internal carotid artery, mastoid process, styloid
process

Arterial supply Branches of external carotid artery

Venous drainage Retromandibular vein

Lymphatic drainage Deep cervical nodes

Nerve innervation  Parasympathetic-Secretomotor


 Sympathetic-Superior cervical ganglion
 Sensory- Greater auricular nerve

Parasympathetic stimulation produces a water rich, serous saliva. Sympathetic stimulation leads to
the production of a low volume, enzyme-rich saliva.
Next question
In a 70 Kg male, what proportion of total body fluid will be contributed by plasma?

50%

5%

35%

65%

25%

70 Kg male = 42 L water (60% of total body weight)

Please rate this question:

Discuss and give feedback

Next question

Fluid compartment physiology

Body fluid compartments comprise intracellular and extracellular compartments. The latter includes
interstitial fluid, plasma and transcellular fluid.
Typical figures are based on the 70 Kg male.

Body fluid volumes

Compartment Volume in litres Percentage of total volume

Intracellular 28 L 60-65%
Compartment Volume in litres Percentage of total volume

Extracellular 14 L 35-40%

Plasma 3L 5%

Interstitial 10 L 24%

Transcellular 1L 3%

Figures are approximate

Next question
A 23 year old man is undergoing an inguinal hernia repair under local anaesthesia. The surgeon
encounters a bleeding site which he manages with diathermy. About a minute or so later the patient
complains that he is able to feel the burning pain of the heat at the operative site. Which of the
following nerve fibres is responsible for the transmission of this signal?

A α fibres

A β fibres

B fibres

C fibres

None of the above

Slow transmission of mechanothermal stimuli is transmitted via C fibres.


A α fibres transmit information relating to motor proprioception, A β fibres transmit touch and
pressure and B fibres are autonomic fibres.
Please rate this question:

Discuss and give feedback


Next question

Pain - neuronal transmission

Somatic pain

 Peripheral nociceptors are innervated by either small myelinated fibres (A-gamma) fibres or
by unmyelinated C fibres.
 The A gamma fibres register high intensity mechanical stimuli. The C fibres usually register
high intensity mechanothermal stimuli.

Next question
What is the approximate volume of pancreatic secretions in a 24 hour period?

100ml

200ml

500ml

1500ml

3000ml

Typically the pancreas secretes between 1000 and 1500ml per day.

Please rate this question:

Discuss and give feedback

Next question

Pancreas exocrine physiology

Composition of pancreatic secretions


Pancreatic secretions are usually 1000-1500ml per 24 hours and have a pH of 8.

Secretion Source Substances secreted

Trypsinogen
Procarboxylase
Enzymic Acinar cells
Amylase
Elastase

Aqueous Sodium
Ductal and
Bicarbonate
Centroacinar cells Water
Potassium
Chloride
NB: Sodium and potassium reflect their plasma levels; chloride
and bicarbonate vary with flow rate

Regulation
The cephalic and gastric phases (neuronal and physical) are less important in regulating the
pancreatic secretions. The effect of digested material in the small bowel stimulates CCK release and
ACh which stimulate acinar and ductal cells. Of these CCK is the most potent stimulus. In the case
of the ductal cells these are potently stimulated by secretin which is released by the S cells of the
duodenum. This results in an increase in bicarbonate.

Enzyme activation
Trypsinogen is converted via enterokinase to active trypsin in the duodenum. Trypsin then activates
the other inactive enzymes

Next question
A 34 year old lady has just undergone a parathyroidectomy for primary hyperparathyroidism. The
operation is difficult and all 4 glands were explored. The wound was clean and dry at the conclusion
of the procedure and a suction drain inserted. On the ward she becomes irritable and develops
stridor. On examination, her neck is soft and the drain empty. Which of the following treatments
should be tried initially?

Administration of intravenous calcium gluconate

Administration of intravenous lorazepam

Removal of the skin closure on the ward

Direct laryngoscopy

Administration of calcichew D3 orally

Exploration of the parathyroid glands may result in impairment of the blood supply. Serum PTH
levels can fall quickly and features of hypocalcaemia may ensue, these include neuromuscular
irritability and laryngospasm. Prompt administration of intravenous calcium gluconate can be
lifesaving. The absence of any neck swelling and no blood in the drain would go against a contained
haematoma in the neck (which should be managed by removal of skin closure).
Please rate this question:

Discuss and give feedback


Next question

Calcium homeostasis

Calcium ions are linked to a wide range of physiological processes. The largest store of bodily
calcium is contained within the skeleton. Calcium levels are primarily controlled by parathyroid
hormone, vitamin D and calcitonin.

Hormonal regulation of calcium


Hormone Actions

Parathyroid hormone (PTH)  Increase calcium levels and decrease phosphate


levels
 Increases bone resorption
 Immediate action on osteoblasts to increase ca2+ in
Hormone Actions

extracellular fluid
 Osteoblasts produce a protein signaling molecule
that activate osteoclasts which cause bone
resorption
 Increases renal tubular reabsorption of calcium
 Increases synthesis of 1,25(OH)2D (active form
of vitamin D) in the kidney which increases
bowel absorption of Ca2+
 Decreases renal phosphate reabsorption

1,25-dihydroxycholecalciferol (the  Increases plasma calcium and plasma phosphate


active form of vitamin D)  Increases renal tubular reabsorption and gut
absorption of calcium
 Increases osteoclastic activity
 Increases renal phosphate reabsorption

Calcitonin  Secreted by C cells of thyroid


 Inhibits intestinal calcium absorption
 Inhibits osteoclast activity
 Inhibits renal tubular absorption of calcium

Both growth hormone and thyroxine also play a small role in calcium metabolism.
Next question
Theme: Interpretation of aterial blood gas results

A. pH 7.19, pCO2 10.2, pO2 16 (FiO2 85%), Bicarbonate 23.8, Base excess -2.2
mmol

B. pH 7.57, PaCO2 3.5, Pa O2 24.5 (FiO2 85%), Bicarbonate 23.5, Base excess
+1.8 mmol

C. pH 7.14, PaCO2 7.4, PaO2 8.9 (FiO2 40%), Bicarbonate 14 mmol, Base excess
-10.6

D. pH 7.36, PaCO2 7.3, PO2 8.9 (FiO2 40%), Bicarbonate 30.2, Base excess 5.3

E. pH 7.32, PCO2 3.8, PaO2 22.2 (FiO2 40%), Bicarbonate 19.1, Base excess -7.9

Which of the following arterial blood gases fit with the description below?

42. Acute respiratory acidosis

pH 7.19, pCO2 10.2, pO2 16 (FiO2 85%), Bicarbonate 23.8, Base excess -2.2 mmol

43. Metabolic acidosis with a compensatory respiratory alkalosis

You answered pH 7.19, pCO2 10.2, pO2 16 (FiO2 85%), Bicarbonate 23.8, Base excess -2.2 mmol

The correct answer is pH 7.32, PCO2 3.8, PaO2 22.2 (FiO2 40%), Bicarbonate 19.1, Base excess -7.9

44. Chronic respiratory acidosis with a compensatory metabolic alkalosis

You answered pH 7.19, pCO2 10.2, pO2 16 (FiO2 85%), Bicarbonate 23.8, Base excess -2.2 mmol

The correct answer is pH 7.36, PaCO2 7.3, PO2 8.9 (FiO2 40%), Bicarbonate 30.2, Base excess 5.3
Please rate this question:

Discuss and give feedback

Next question

Arterial blood gas interpretation

In advanced life support training, a 5 step approach to arterial blood gas interpretation is advocated.

1. How is the patient?

2. Is the patient hypoxaemic?


The Pa02 on air should be 10.0-13.0 kPa

3. Is the patient acidaemic (pH <7.35) or alkalaemic (pH >7.45)

4. What has happened to the PaCO2?


If there is acidaemia, an elevated PaCO2 will account for this

5. What is the bicarbonate level or base excess?


A metabolic acidosis will have a low bicarbonate level and a low base excess (< -2 mmol)
A metabolic alkalosis will have a high bicarbonate and a high base excess (> +2 mmol)

Next question
A 23 year old man presents with blunt abdominal trauma and a splenic bleed is suspected. He is
commenced on an infusion of tranexamic acid. Which of the following best describes its mechanism
of action?

Inhibition of plasmin

Inhibition of thrombin

Inhibition of factor II

Inhibition of factor Xa

Activation of factor VIII

Tranexamic acid inhibits plasmin and this prevents fibrin degradation.

Please rate this question:

Discuss and give feedback

Next question

Tranexamic acid

Tranexamic acid is a synthetic derivative of lysine. Its primary mode of action is as an anti fibrinolytic
that competitively inhibits the conversion of plasminogen to plasmin. Plasmin degrades fibrin and
therefore rendering plasmin inactive slows this process.
The role of tranexamic acid in trauma was investigated in the CRASH 2 trial and has been shown to
be of benefit in bleeding trauma when administered in the first 3 hours.

Next question
A 34 year old male donates a unit of blood. It is stored at 4 oC. After 72 hours which of the following
clotting factors will be most affected?

Factor V

Factor II

Factor VII

Factor IX

Factor XI

Factors V and VIII are sensitive to temperature which is the reason why FFP is frozen soon after
collection.
Please rate this question:

Discuss and give feedback


Next question

Blood products

Whole blood fractions

Fraction Key points

Packed red cells Used for transfusion in chronic anaemia and cases where infusion of large
volumes of fluid may result in cardiovascular compromise. Product obtained
by centrifugation of whole blood.

Platelet rich Usually administered to patients who are thrombocytopaenic and are bleeding
plasma or require surgery. It is obtained by low speed centrifugation.

Platelet Prepared by high speed centrifugation and administered to patients with


concentrate thrombocytopaenia.
Fresh frozen  Prepared from single units of blood.
plasma  Contains clotting factors, albumin and immunoglobulin.
 Unit is usually 200 to 250ml.
 Usually used in correcting clotting deficiencies in patients with hepatic
synthetic failure who are due to undergo surgery.
 Usual dose is 12-15ml/Kg-1.
 It should not be used as first line therapy for hypovolaemia.

Cryoprecipitate  Formed from supernatant of FFP.


 Rich source of Factor VIII and fibrinogen.
 Allows large concentration of factor VIII to be administered in small
volume.

SAG-Mannitol Removal of all plasma from a blood unit and substitution with:
Blood

 Sodium chloride
 Adenine
 Anhydrous glucose
 Mannitol

Up to 4 units of SAG M Blood may be administered. Thereafter whole blood


is preferred. After 8 units, clotting factors and platelets should be considered.

Cell saver devices


These collect patients own blood lost during surgery and then re-infuse it. There are two main types:

 Those which wash the blood cells prior to re-infusion. These are more expensive to purchase
and more complicated to operate. However, they reduce the risk of re-infusing contaminated
blood back into the patient.
 Those which do not wash the blood prior to re-infusion.

Their main advantage is that they avoid the use of infusion of blood from donors into patients and
this may reduce risk of blood borne infection. It may be acceptable to Jehovah's witnesses. It is
contraindicated in malignant disease for risk of facilitating disease dissemination.

Blood products used in warfarin reversal


In some surgical patients the use of warfarin can pose specific problems and may require the use of
specialised blood products

Immediate or urgent surgery in patients taking warfarin(1) (2):

1. Stop warfarin

2. Vitamin K (reversal within 4-24 hours)


-IV takes 4-6h to work (at least 5mg)
-Oral can take 24 hours to be clinically effective

3. Fresh frozen plasma


Used less commonly now as 1st line warfarin reversal
-30ml/kg-1
-Need to give at least 1L fluid in 70kg person (therefore not appropriate in fluid overload)
-Need blood group
-Only use if human prothrombin complex is not available

4. Human Prothrombin Complex (reversal within 1 hour)


-Bereplex 50 u/kg
-Rapid action but factor 6 short half life, therefore give with vitamin K

References
1. Dentali, F., C. Marchesi, et al. (2011). "Safety of prothrombin complex concentrates for rapid
anticoagulation reversal of vitamin K antagonists. A meta-analysis." Thromb Haemost 106(3): 429-
438.

2. http://www.transfusionguidelines.org/docs/pdfs/bbt-03warfarin-reversal-flowchart-2006.pdf
Next question
Cortisol is predominantly produced by which of the following?

Zona fasciculata of the adrenal

Zona glomerulosa of the adrenal

Zona reticularis of the adrenal

Adrenal medulla

Posterior lobe of the pituitary

Relative Glucocorticoid activity:

Hydrocortisone = 1
Prednisolone = 4
Dexamethasone = 25

Cortisol is produced by the zona fasciculata of the adrenal gland.


Please rate this question:

Discuss and give feedback


Next question

Cortisol

 Glucocorticoid
 Released by zona fasiculata of the adrenal gland
 90% protein bound; 10% active
 Circadian rhythm: High in the mornings
 Negative feedback via ACTH

Actions

 Glycogenolysis
 Gluconeogenesis
 Protein catabolism
 Lipolysis
 Stress response
 Anti-inflammatory
 Decrease protein in bones
 Increase gastric acid
 Increases neutrophils/platelets/red blood cells
 Inhibits fibroblastic activity

Next question
Which of the following is not an intravenous colloid?

Gelofusine

Dextran 40

Human albumin solution

Hydroxyethyl starch

Bicarbonate 8.4%

Bicarbonate is a crystalloid.
Please rate this question:

Discuss and give feedback


Next question

Pre operative fluid management

Fluid management has been described in the British Consensus guidelines on IV fluid
therapy for Adult Surgical patients (GIFTASUP) and by NICE (CG174 December 2013)

The Recommendations include:

 Use Ringer's lactate or Hartmann's when a crystalloid is needed for resuscitation or


replacement of fluids. Avoid 0.9% N. Saline (due to risk of hyperchloraemic acidosis) unless
patient vomiting or has gastric drainage.
 Use 4%/0.18% dextrose saline or 5% dextrose in maintenance fluids. It should not be used
in resuscitation or as replacement fluids.
 Adult maintenance fluid requirements are: Na 50-100 mmol/day and K 40-80 mmol/day in
1.5-2.5L fluid per day.
 Patients for elective surgery should NOT be nil by mouth for >2 hours (unless has disorder of
gastric emptying).
 Patients for elective surgery should be given carbohydrate rich drinks 2-3h before. Ideally
this should form part of a normal pre op plan to facilitate recovery.
 Avoid mechanical bowel preparation.
 If bowel prep is used, simultaneous administration of Hartmann's or Ringer's lactate should
be considered.
 Excessive fluid losses from vomiting should be treated with a crystalloid with potassium
replacement. 0.9% N. Saline should be given if there is hypochloraemia. Otherwise
Hartmann's or Ringer lactate should be given for diarrhoea/ileostomy/ileus/obstruction.
Hartmann's should also be given in sodium losses secondary to diuretics.
 High risk patients should receive fluids and inotropes.
 An attempt should be made to detect pre or operative hypovolaemia using flow based
measurements. If this is not available, then clinical evaluation is needed i.e. JVP, pulse
volume etc.
 In Blood loss or infection causing hypovolaemia should be treated with a balanced crystalloid
or colloid (or until blood available in blood loss). A critically ill patient is unable to excrete Na
or H20 leading to a 5% risk of interstitial oedema. Therefore 5% dextrose as well as colloid
should be given.
 If patients need IV fluid resuscitation, use crystalloids that contain sodium in the range 130-
154 mmol/l, with a bolus of 500 ml over less than 15 minutes (NICE Guidance CG 174).

Next question
Theme: Electrolyte disorders

A. Hypotonic hypovolaemic hyponatraemia


B. Hypotonic hypervolaemic hyponatraemia
C. Pseudohyponatraemia
D. Syndrome of inappropriate ADH secretion (SIADH)
E. Hypertonic hyponatraemia
F. Over administration 5% dextrose

Please select the most likely reason for hyponatraemia for each scenario given. Each option may be
used once, more than once or not at all.

49. A 73 year old man presents to pre operative clinic for an elective total hip replacement. He
is on frusemide for hypertension. He is found to have the following blood results:
Na 120
Urine Na 10 (low)
Serum osmolality 280 (normal)

Hypotonic hypovolaemic hyponatraemia

The blood results reflect extra-renal sodium loss. The body is trying to preserve the
sodium by not allowing any sodium into the urine (hence the low Na in the urine). Note
with renal sodium loss the Urinary sodium is high.

50. A 67 year old man presents to pre operative clinic for an elective hernia repair. He is on
frusemide for heart failure. He is found to have the following blood results:
Na 120
Urine Na 35 (high)
Urine osmolality 520 (high)
Serum osmolality 265 (low)

You answered Hypotonic hypovolaemic hyponatraemia

The correct answer is Syndrome of inappropriate ADH secretion (SIADH)

This blood picture fits with SIADH. SIADH causes retention of fluid from the urine
(concentrated urine) into the blood vessels, therefore diluting the fluid in the blood vessels
(low osmolality). Management involves removing the cause and fluid restriction.

51. A 77 year old man presents to pre operative clinic for a total knee replacement. He is on
frusemide for hypertension. He is known to have multiple myeloma. He is found to have
the following blood results:
Na 120
Serum osmolality 280 (normal)
Urine osmolality normal
Urine Na normal

You answered Hypotonic hypovolaemic hyponatraemia

The correct answer is Pseudohyponatraemia

Hyperlipidaemia and multiple myeloma are known to cause a pseudohyponatraemia.


SIADH:

 Low serum osmolality


 High/Normal urine osmolality

Please rate this question:

Discuss and give feedback


Next question

Hyponatraemia

This is commonly tested in the MRCS (despite most surgeons automatically seeking medical advice
if this occurs!). The most common cause in surgery is the over administration of 5% dextrose.

Hyponatraemia may be caused by water excess or sodium depletion. Causes of


pseudohyponatraemia include hyperlipidaemia (increase in serum volume) or a taking blood from a
drip arm. Urinary sodium and osmolarity levels aid making a diagnosis.

Classification
Urinary sodium > 20 Sodium depletion, renal loss Mnemonic: Syndrome of
mmol/l INAPPropriate Anti-
Diuretic Hormone:
 Patient often hypovolaemic In creased
 Diuretics (thiazides) Na (sodium)
 Addison's PP (urine)
 Diuretic stage of renal failure
 SIADH (serum osmolality
low, urine osmolality high,
urine Na high)
 Patient often euvolaemic
Urinary sodium < 20 Sodium depletion, extra-renal loss
mmol/l

 Diarrhoea, vomiting,
sweating
 Burns, adenoma of rectum (if
villous lesion and large)

Water excess (patient  Secondary


often hypervolaemic and hyperaldosteronism: CCF,
oedematous) cirrhosis
 Reduced GFR: renal failure
 IV dextrose, psychogenic
polydipsia

Management

Symptomatic Hyponatremia :

Acute hyponatraemia with Na <120: immediate therapy. Central Pontine Myelinolisis, may occur
from overly rapid correction of serum sodium. Aim to correct until the Na is > 125 at a rate of 1
mEq/h. Normal saline with frusemide is an alternative method.

The sodium requirement can be calculated as follows :

(125 - serum sodium) x 0.6 x body weight = required mEq of sodium


Next question
A 25 year old man undergoes an appendicetomy for appendicitis. The appendix is submitted for
histopathological evaluation. Which of the following is most likely to be identified microscopically?

Macrophages

Neutrophils

Fibroblasts

Lymphocytes

Stem cells

Theme from April 2015 Exam


Theme from January 2015 Exam
Neutrophil polymorphs are the cell type most commonly encountered in acute inflammation.
Please rate this question:

Discuss and give feedback


Next question

Acute inflammation

Inflammation is the reaction of the tissue elements to injury. Vascular changes occur, resulting in the
generation of a protein rich exudate. So long as the injury does not totally destroy the existing tissue
architecture, the episode may resolve with restoration of original tissue architecture.

Vascular changes

 Vasodilation occurs and persists throughout the inflammatory phase.


 Inflammatory cells exit the circulation at the site of injury.
 The equilibrium that balances Starlings forces within capillary beds is disrupted and a protein
rich exudate will form as the vessel walls also become more permeable to proteins.
 The high fibrinogen content of the fluid may form a fibrin clot. This has several important
immunomodulatory functions.

Sequelae
Resolution  Typically occurs with minimal initial injury
 Stimulus removed and normal tissue architecture results

Organisation  Delayed removal of exudate


 Tissues undergo organisation and usually fibrosis

Suppuration  Typically formation of an abscess or an empyema


 Sequestration of large quantities of dead neutrophils

Progression to chronic  Coupled inflammatory and reparative activities


inflammation  Usually occurs when initial infection or suppuration has
been inadequately managed

Causes

 Infections e.g. Viruses, exotoxins or endotoxins released by bacteria


 Chemical agents
 Physical agents e.g. Trauma
 Hypersensitivity reactions
 Tissue necrosis

Presence of neutrophil polymorphs is a histological diagnostic feature of acute inflammation


Next question
A 53 year old man is on the intensive care unit following an emergency abdominal aortic aneurysm
repair. He develops abdominal pain and diarrhoea and is profoundly unwell. His abdomen has no
features of peritonism. Which of the following arterial blood gas pictures is most likely to be present?

pH 7.45, pO2 10.1, pCO2 3.2, Base excess 0, Lactate 0

pH 7.35, pO2 8.0, pCO2 5.2, Base excess 2, Lactate 1

pH 7.20, pO2 9.0, pCO2 3.5, Base excess -10, Lactate 8

pH 7.29, pO2 8.9, pCO2 5.9, Base excess -4, Lactate 3

pH 7.30, pO2 9.2 pCO2 4.8, Base excess -2, lactate 1

This man is likely to have a metabolic acidosis secondary to a mesenteric infarct.


Please rate this question:

Discuss and give feedback


Next question

Disorders of acid - base balance

Disorders of acid- base balance are often covered in the MRCS part A, both in the SBA and EMQ
sections.

The acid-base normogram below shows how the various disorders may be categorised
Image sourced from Wikipedia

Metabolic acidosis

 This is the most common surgical acid - base disorder.


 Reduction in plasma bicarbonate levels.
 Two mechanisms:

1. Gain of strong acid (e.g. diabetic ketoacidosis)


2. Loss of base (e.g. from bowel in diarrhoea)
- Classified according to the anion gap, this can be calculated by:
(Na+ + K+) - (Cl- + HCO3-).
- If a question supplies the chloride level then this is often a clue that the anion gap should be
calculated. The normal range = 10-18 mmol/L

Normal anion gap ( = hyperchloraemic metabolic acidosis)

 Gastrointestinal bicarbonate loss: diarrhoea, ureterosigmoidostomy, fistula


 Renal tubular acidosis
 Drugs: e.g. acetazolamide
 Ammonium chloride injection
 Addison's disease

Raised anion gap


 Lactate: shock, hypoxia
 Ketones: diabetic ketoacidosis, alcohol
 Urate: renal failure
 Acid poisoning: salicylates, methanol

Metabolic acidosis secondary to high lactate levels may be subdivided into two types:

 Lactic acidosis type A: (Perfusion disorders e.g.shock, hypoxia, burns)


 Lactic acidosis type B: (Metabolic e.g. metformin toxicity)

Metabolic alkalosis

 Usually caused by a rise in plasma bicarbonate levels.


 Rise of bicarbonate above 24 mmol/L will typically result in renal excretion of excess
bicarbonate.
 Caused by a loss of hydrogen ions or a gain of bicarbonate. It is due mainly to problems of
the kidney or gastrointestinal tract

Causes

 Vomiting / aspiration (e.g. Peptic ulcer leading to pyloric stenosis, nasogastric suction)
 Diuretics
 Liquorice, carbenoxolone
 Hypokalaemia
 Primary hyperaldosteronism
 Cushing's syndrome
 Bartter's syndrome
 Congenital adrenal hyperplasia

Mechanism of metabolic alkalosis

 Activation of renin-angiotensin II-aldosterone (RAA) system is a key factor


 Aldosterone causes reabsorption of Na+ in exchange for H+ in the distal convoluted tubule
 ECF depletion (vomiting, diuretics) → Na+ and Cl- loss → activation of RAA system → raised
aldosterone levels
 In hypokalaemia, K+ shift from cells → ECF, alkalosis is caused by shift of H + into cells to
maintain neutrality
Respiratory acidosis

 Rise in carbon dioxide levels usually as a result of alveolar hypoventilation


 Renal compensation may occur leading to Compensated respiratory acidosis

Causes

 COPD
 Decompensation in other respiratory conditions e.g. Life-threatening asthma / pulmonary
oedema
 Sedative drugs: benzodiazepines, opiate overdose

Respiratory alkalosis

 Hyperventilation resulting in excess loss of carbon dioxide


 This will result in increasing pH

Causes

 Psychogenic: anxiety leading to hyperventilation


 Hypoxia causing a subsequent hyperventilation: pulmonary embolism, high altitude
 Early salicylate poisoning*
 CNS stimulation: stroke, subarachnoid haemorrhage, encephalitis
 Pregnancy

*Salicylate overdose leads to a mixed respiratory alkalosis and metabolic acidosis. Early stimulation
of the respiratory centre leads to a respiratory alkalosis whilst later the direct acid effects of
salicylates (combined with acute renal failure) may lead to an acidosis
Next question
A 48 year old women suffers blunt trauma to the head and develops respiratory compromise. As a
result she develops hypercapnia. Which of the following effects is most likely to ensue?

Cerebral vasoconstriction

Cerebral vasodilation

Cerebral blood flow will remain unchanged

Shunting of blood to peripheral tissues will occur in preference to CNS perfusion

None of the above

Hypercapnia will tend to produce cerebral vasodilation. This is of considerable importance in patients
with cranial trauma as it may increase intracranial pressure.
Please rate this question:

Discuss and give feedback


Next question

Applied neurophysiology

 Pressure within the cranium is governed by the Monroe-Kelly doctrine. This considers the
skull as a closed box. Increases in mass can be accommodated by loss of CSF. Once a
critical point is reached (usually 100- 120ml of CSF lost) there can be no further
compensation and ICP rises sharply. The next step is that pressure will begin to equate with
MAP and neuronal death will occur. Herniation will also accompany this process.
 The CNS can autoregulate its own blood supply. Vaso constriction and dilatation of the
cerebral blood vessels is the primary method by which this occurs. Extremes of blood
pressure can exceed this capacity resulting in risk of stroke. Other metabolic factors such as
hypercapnia will also cause vasodilation, which is of importance in ventilating head injured
patients.
 The brain can only metabolise glucose, when glucose levels fall, consciousness will be
impaired.

Next question
A patient is seen in clinic complaining of abdominal pain. Routine bloods show:

Na+ 142 mmol/l

K+ 4.0 mmol/l

Chloride 104 mmol/l

Bicarbonate 19 mmol/l

Urea 7.0 mmol/l

Creatinine 112 µmol/l

What is the anion gap?

4 mmol/L

14 mmol/L

20 mmol/L

21 mmol/L

23 mmol/L

The anion gap may be calculated by using (sodium + potassium) - (bicarbonate + chloride)

= (142 + 4.0) - (104 + 19) = 23 mmol/L


Please rate this question:

Discuss and give feedback


Next question

Anion gap

The anion gap is calculated by:

(sodium + potassium) - (bicarbonate + chloride)

A normal anion gap is 8-14 mmol/L


It is useful to consider in patients with a metabolic acidosis:

Causes of a normal anion gap or hyperchloraemic metabolic acidosis

 gastrointestinal bicarbonate loss: diarrhoea, ureterosigmoidostomy, fistula


 renal tubular acidosis
 drugs: e.g. acetazolamide
 ammonium chloride injection
 Addison's disease

Causes of a raised anion gap metabolic acidosis

 lactate: shock, hypoxia


 ketones: diabetic ketoacidosis, alcohol
 urate: renal failure
 acid poisoning: salicylates, methanol

Next question
A 73 year old man has an arterial line in situ. On studying the trace the incisura can be seen. What is
the physiological event which accounts for this process?

Atrial repolarisation

Mitral valve closure

Ventricular repolarisation

Elastic recoil of the aorta

Tricuspid valve closure

Theme from 2010 Exam

It is the temporary rise in aortic pressure occurring as a result of elastic recoil.


Please rate this question:

Discuss and give feedback


Next question

Cardiac physiology

 The heart has four chambers ejecting blood into both low pressure and high pressure
systems.
 The pumps generate pressures of between 0-25mmHg on the right side and 0-120 mmHg on
the left.
 At rest diastole comprises 2/3 of the cardiac cycle.
 The product of the frequency of heart rate and stroke volume combine to give the cardiac
output which is typically 5-6L per minute.

Detailed descriptions of the various waveforms are often not a feature of MRCS A (although they are
on the syllabus). However, they are a very popular topic for surgical physiology in the MRCS B
exam.

Electrical properties

 Intrinsic myogenic rhythm within cardiac myocytes means that even the denervated heart is
capable of contraction.
 In the normal situation the cardiac impulse is generated in the sino atrial node in the right
atrium and conveyed to the ventricles via the atrioventricular node.
 The sino atrial node is also capable of spontaneous discharge and in the absence of
background vagal tone will typically discharge around 100x per minute. Hence the higher
resting heart rate found in cardiac transplant cases. In the SA and AV nodes the resting
membrane potential is lower than in surrounding cardiac cells and will slowly depolarise from
-70mV to around -50mV at which point an action potential is generated.
 Differences in the depolarisation slopes between SA and AV nodes help to explain why the
SA node will depolarise first. The cells have a refractory period during which they cannot be
re-stimulated and this period allows for adequate ventricular filling. In pathological
tachycardic states this time period is overridden and inadequate ventricular filling may then
occur, cardiac output falls and syncope may ensue.

Parasympathetic fibres project to the heart via the vagus and will release acetylcholine. Sympathetic
fibres release nor adrenaline and circulating adrenaline comes from the adrenal medulla.
Noradrenaline binds to β 1 receptors in the SA node and increases the rate of pacemaker potential
depolarisation.

Cardiac cycle

Image sourced from Wikipedia

 Mid diastole: AV valves open. Ventricles hold 80% of final volume. Outflow valves shut.
Aortic pressure is high.

 Late diastole: Atria contract. Ventricles receive 20% to complete filling. Typical end diastolic
volume 130-160ml.
 Early systole: AV valves shut. Ventricular pressure rises. Isovolumetric ventricular
contraction. AV Valves bulge into atria (c-wave). Aortic and pulmonary pressure exceeded-
blood is ejected. Shortening of ventricles pulls atria downwards and drops intra atrial
pressure (x-descent).

 Late systole: Ventricular muscles relax and ventricular pressures drop. Although ventricular
pressure drops the aortic pressure remains constant owing to peripheral vascular resistance
and elastic property of the aorta. Brief period of retrograde flow that occurs in aortic recoil
shuts the aortic valve. Ventricles will contain 60ml end systolic volume. The average stroke
volume is 70ml (i.e. Volume ejected).

 Early diastole: All valves are closed. Isovolumetric ventricular relaxation occurs. Pressure
wave associated with closure of the aortic valve increases aortic pressure. The pressure dip
before this rise can be seen on arterial waveforms and is called the incisura. During systole
the atrial pressure increases such that it is now above zero (v- wave). Eventually atrial
pressure exceed ventricular pressure and AV valves open - atria empty passively into
ventricles and atrial pressure falls (y -descent )

The negative atrial pressures are of clinical importance as they can allow air embolization to occur if
the neck veins are exposed to air. This patient positioning is important in head and neck surgery to
avoid this occurrence if veins are inadvertently cut, or during CVP line insertion.

Mechanical properties

 Preload = end diastolic volume


 Afterload = aortic pressure

It is important to understand the principles of Laplace's law in surgery.

 It states that for hollow organs with a circular cross section, the total circumferential wall
tension depends upon the circumference of the wall, multiplied by the thickness of the wall
and on the wall tension.
 The total luminal pressure depends upon the cross sectional area of the lumen and the
transmural pressure. Transmural pressure is the internal pressure minus external pressure
and at equilibrium the total pressure must counterbalance each other.
 In terms of cardiac physiology the law explains that the rise in ventricular pressure that
occurs during the ejection phase is due to physical change in heart size. It also explains why
a dilated diseased heart will have impaired systolic function.
Starlings law

 Increase in end diastolic volume will produce larger stroke volume.


 This occurs up to a point beyond which cardiac fibres are excessively stretched and stroke
volume will fall once more. It is important for the regulation of cardiac output in cardiac
transplant patients who need to increase their cardiac output.

Baroreceptor reflexes

 Baroreceptors located in aortic arch and carotid sinus.


 Aortic baroreceptor impulses travel via the vagus and from the carotid via the
glossopharyngeal nerve.
 They are stimulated by arterial stretch.
 Even at normal blood pressures they are tonically active.
 Increase in baroreceptor discharge causes:

*Increased parasympathetic discharge to the SA node.


*Decreased sympathetic discharge to ventricular muscle causing decreased contractility and fall in
stroke volume.
*Decreased sympathetic discharge to venous system causing increased compliance.
*Decreased peripheral arterial vascular resistance

Atrial stretch receptors

 Located in atria at junction between pulmonary veins and vena cava.


 Stimulated by atrial stretch and are thus low pressure sensors.
 Increased blood volume will cause increased parasympathetic activity.
 Very rapid infusion of blood will result in increase in heart rate mediated via atrial receptors:
theBainbridge reflex.
 Decreases in receptor stimulation results in increased sympathetic activity this will decrease
renal blood flow-decreases GFR-decreases urinary sodium excretion-renin secretion by
juxtaglomerular apparatus-Increase in angiotensin II.
 Increased atrial stretch will also result in increased release of atrial natriuretic peptide.

Next question
A surgeon is considering using lignocaine as local anasthesia for a minor procedure. Which of the
following best accounts for its actions?

Blockade of neuronal acetylcholine receptors

Blockade of neuronal nicotinic receptors

Blockade of neuronal sodium channels

Blockade of neuronal potassium channels

Blockade of neuronal calcium channels

Lignocaine blocks sodium channels. They will typically be activated first, hence the pain some
patients experience on administration.
Please rate this question:

Discuss and give feedback


Next question

Local anaesthetic agents

Lidocaine

 An amide
 Local anaesthetic and a less commonly used antiarrhythmic (affects Na channels in the
axon)
 Hepatic metabolism, protein bound, renally excreted
 Toxicity: due to IV or excess administration. Increased risk if liver dysfunction or low protein
states. Note acidosis causes lidocaine to detach from protein binding.
 Drug interactions: Beta blockers, ciprofloxacin, phenytoin
 Features of toxicity: Initial CNS over activity then depression as lidocaine initially blocks
inhibitory pathways then blocks both inhibitory and activating pathways. Cardiac arrhythmias.
 Increased doses may be used when combined with adrenaline to limit systemic absorption.

Cocaine

 Pure cocaine is a salt, usually cocaine hydrochloride. It is supplied for local anaesthetic
purposes as a paste.
 It is supplied for clinical use in concentrations of 4 and 10%. It may be applied topically to the
nasal mucosa. It has a rapid onset of action and has the additional advantage of causing
marked vasoconstriction.
 It is lipophillic and will readily cross the blood brain barrier. Its systemic effects also include
cardiac arrhythmias and tachycardia.
 Apart from its limited use in ENT surgery it is otherwise used rarely in mainstream surgical
practice.

Bupivacaine

 Bupivacaine binds to the intracellular portion of sodium channels and blocks sodium influx
into nerve cells, which prevents depolarization.
 It has a much longer duration of action than lignocaine and this is of use in that it may be
used for topical wound infiltration at the conclusion of surgical procedures with long duration
analgesic effect.
 It is cardiotoxic and is therefore contra indicated in regional blockage in case the tourniquet
fails.
 Levobupivicaine (Chirocaine) is less cardiotoxic and causes less vasodilation.

Prilocaine

 Similar mechanism of action to other local anaesthetic agents. However, it is far less
cardiotoxic and is therefore the agent of choice for intravenous regional anaesthesia e.g.
Biers Block.

All local anaesthetic agents dissociate in tissues and this contributes to their therapeutic effect. The
dissociation constant shifts in tissues that are acidic e.g. where an abscess is present, and this
reduces the efficacy.

Doses of local anaesthetics


Agent Dose plain Dose with adrenaline

Lignocaine 3mg/Kg 7mg/Kg

Bupivacaine 2mg/Kg 2mg/Kg

Prilocaine 6mg/Kg 9mg/Kg

These are a guide only as actual doses depend on site of administration, tissue vascularity and co-
morbidities.
Maximum total local anaesthetic doses

 Lignocaine 1% plain - 3mg/ Kg - 200mg (20ml)


 Lignocaine 1% with 1 in 200,000 adrenaline - 7mg/Kg - 500mg (50ml)
 Bupivicaine 0.5% - 2mg/kg- 150mg (30ml)

Maximum doses are based on ideal body weight

Effects of adrenaline
Adrenaline may be added to local anaesthetic drugs. It prolongs the duration of action at the site of
injection and permits usage of higher doses (see above). It is contra indicated in patients taking
MAOI's or tricyclic antidepressants. The toxicity of bupivacaine is related to protein binding and
addition of adrenaline to this drug does not permit increases in the total dose of bupivacaine, in
contrast to the situation with lignocaine.

References
An excellent review is provided by:
French J and Sharp L. Local Anaesthetics. Ann R Coll Surg Engl 2012; 94: 76-80.
Next question
A 22 year old man suffers a blunt head injury. He is drowsy and has a GCS of 7 on admission.
Which of the following is the major determinant of cerebral blood flow in this situation?

Systemic blood pressure

Mean arterial pressure

Intra cranial pressure

Hypoxaemia

Acidosis

Theme from 2009 Exam

Hypoxaemia and acidosis may both affect cerebral blood flow. However, in the traumatic situation
increases in intracranial pressure are far more likely to occur especially when GCS is low. This will
adversely affect cerebral blood flow.
Please rate this question:

Discuss and give feedback


Next question

Cerebral blood flow

 CNS autoregulates its own blood supply


 Factors affecting the cerebral pressure include; systemic carbon dioxide levels, CNS
metabolism, CNS trauma, CNS pressure
 The PaCO2 is the most potent mediator
 Acidosis and hypoxaemia will increase cerebral blood flow but to a lesser degree
 Intra cranial pressure may increase in patients with head injuries and this can result in
impaired blood flow
 Intra cerebral pressure is governed by Monroe-Kelly Doctrine which considers brain as
closed box, changes in pressure are offset by loss of CSF. When this is no longer possible
ICP rises

Next question
A 43 year old man has recurrent episodes of dyspepsia and treatment is commenced with oral
antacids. Which of the hormones listed below is released in response to increased serum gastrin
levels and decreases intra gastric pH?

Cholecystokinin

Histamine

Somatostatin

Insulin

Vasoactive intestinal peptide

Theme from January 2013 Exam


Histamine is released from enterochromaffin cells in the stomach mucosa which stimulates acid
secretion. It is usually released in response to increased serum gastrin levels. Histamine blockers
(e.g. cimetidine) were extremely popular treatments until the advent of proton pump inhibitors.
Please rate this question:

Discuss and give feedback


Next question

Gastric secretions

A working knowledge of gastric secretions is important for surgery because peptic ulcers are
common, surgeons frequently prescribe anti secretory drugs and because there are still patients
around who will have undergone acid lowering procedures (Vagotomy) in the past.

Gastric acid

 Is produced by the parietal cells in the stomach


 pH of gastric acid is around 2 with acidity being maintained by the H +/K+ ATP ase pump. As
part of the process bicarbonate ions will be secreted into the surrounding vessels.
 Sodium and chloride ions are actively secreted from the parietal cell into the canaliculus.
This sets up a negative potential across the membrane and as a result sodium and
potassium ions diffuse across into the canaliculus.
 Carbonic anhydrase forms carbonic acid which dissociates and the hydrogen ions formed by
dissociation leave the cell via the H+/K+ antiporter pump. At the same time sodium ions are
actively absorbed. This leaves hydrogen and chloride ions in the canaliculus these mix and
are secreted into the lumen of the oxyntic gland.
This is illustrated diagrammatically below:

Image sourced from Wikipedia

Phases of gastric acid secretion


There are 3 phases of gastric secretion:

1. Cephalic phase (smell / taste of food)

 30% acid produced


 Vagal cholinergic stimulation causing secretion of HCL and gastrin release from G cells

2. Gastric phase (distension of stomach )

 60% acid produced


 Stomach distension/low H+/peptides causes Gastrin release
3. Intestinal phase (food in duodenum)

 10% acid produced


 High acidity/distension/hypertonic solutions in the duodenum inhibits gastric acid secretion
via enterogastrones (CCK, secretin) and neural reflexes.

Regulation of gastric acid production


Factors increasing production include:

 Vagal nerve stimulation


 Gastrin release
 Histamine release (indirectly following gastrin release) from enterchromaffin like cells

Factors decreasing production include:

 Somatostatin (inhibits histamine release)


 Cholecystokinin
 Secretin

The diagram below illustrates some of the factors involved in regulating gastric acid secretion and
the relevant associated pharmacology

Image sourced from Wikipedia

Below is a brief summary of the major hormones involved in food digestion:


Source Stimulus Actions

Gastrin G cells in Distension of Increase HCL, pepsinogen and IF secretion,


antrum of the stomach, extrinsic increases gastric motility, trophic effect on
stomach nerves gastric mucosa
Inhibited by: low
antral pH,
somatostatin

CCK I cells in Partially digested Increases secretion of enzyme-rich fluid


upper small proteins and from pancreas, contraction of gallbladder
intestine triglycerides and relaxation of sphincter of Oddi,
decreases gastric emptying, trophic effect on
pancreatic acinar cells, induces satiety

Secretin S cells in Acidic chyme, Increases secretion of bicarbonate-rich fluid


upper small fatty acids from pancreas and hepatic duct cells,
intestine decreases gastric acid secretion, trophic
effect on pancreatic acinar cells

VIP Small Neural Stimulates secretion by pancreas and


intestine, intestines, inhibits acid and pepsinogen
pancreas secretion

Somatostatin D cells in the Fat, bile salts and Decreases acid and pepsin secretion,
pancreas and glucose in the decreases gastrin secretion, decreases
stomach intestinal lumen pancreatic enzyme secretion, decreases
insulin and glucagon secretion
inhibits trophic effects of gastrin, stimulates
gastric mucous production

Next question
Which main group of receptors does dobutamine bind to?

α-1

α-2

ß-1

ß-2

D-1

Dobutamine is a sympathomimetic with both alpha- and beta-agonist properties; it displays a


considerable selectivity for beta1-cardiac receptors.

Please rate this question:

Discuss and give feedback

Next question

Inotropes and cardiovascular receptors

Inotropes are a class of drugs which work primarily by increasing cardiac output. They should be
distinguished from vasoconstrictor drugs which are used specifically when the primary problem is
peripheral vasodilatation.

Catecholamine type agents are commonly used and work by increasing cAMP levels by adenylate
cyclase stimulation. This in turn intracellular calcium ion mobilisation and thus the force of
contraction. Adrenaline works as a beta adrenergic receptor agonist at lower doses and an alpha
receptor agonist at higher doses. Dopamine causes dopamine receptor mediated renal and
mesenteric vascular dilatation and beta 1 receptor agonism at higher doses. This results in
increased cardiac output. Since both heart rate and blood pressure are raised, there is less overall
myocardial ischaemia. Dobutamine is a predominantly beta 1 receptor agonist with weak beta 2 and
alpha receptor agonist properties. Noradrenaline is a catecholamine type agent and predominantly
acts as an alpha receptor agonist and serves as a peripheral vasoconstrictor.

Phosphodiesterase inhibitors such as milrinone act specifically on the cardiac phosphodiesterase


and increase cardiac output.

Inotrope Cardiovascular receptor action

Adrenaline α-1, α-2, β-1, β-2

Noradrenaline α-1,( α-2), (β-1), (β-2)

Dobutamine β-1, (β 2)

Dopamine (α-1), (α-2), (β-1), D-1,D-2

Minor receptor effects in brackets

Effects of receptor binding

α-1, α-2 vasoconstriction

β-1 increased cardiac contractility and HR

β-2 vasodilatation

D-1 renal and spleen vasodilatation

D-2 inhibits release of noradrenaline

Next question
Which of the following is responsible for the release and synthesis of calcitonin?

Parathyroid glands

Anterior pituitary

Thyroid gland

Posterior pituitary

Adrenal glands

Calcitonin has the opposite effect of PTH and is released from the thyroid gland.
Please rate this question:

Discuss and give feedback


Next question

Calcium homeostasis

Calcium ions are linked to a wide range of physiological processes. The largest store of bodily
calcium is contained within the skeleton. Calcium levels are primarily controlled by parathyroid
hormone, vitamin D and calcitonin.

Hormonal regulation of calcium


Hormone Actions

Parathyroid hormone (PTH)  Increase calcium levels and decrease phosphate


levels
 Increases bone resorption
 Immediate action on osteoblasts to increase ca2+ in
extracellular fluid
 Osteoblasts produce a protein signaling molecule
that activate osteoclasts which cause bone
resorption
 Increases renal tubular reabsorption of calcium
 Increases synthesis of 1,25(OH)2D (active form
of vitamin D) in the kidney which increases
Hormone Actions

bowel absorption of Ca2+


 Decreases renal phosphate reabsorption

1,25-dihydroxycholecalciferol (the  Increases plasma calcium and plasma phosphate


active form of vitamin D)  Increases renal tubular reabsorption and gut
absorption of calcium
 Increases osteoclastic activity
 Increases renal phosphate reabsorption

Calcitonin  Secreted by C cells of thyroid


 Inhibits intestinal calcium absorption
 Inhibits osteoclast activity
 Inhibits renal tubular absorption of calcium

Both growth hormone and thyroxine also play a small role in calcium metabolism.
Next question
What is the half life of insulin in the circulation of a normal healthy adult?

Less than 30 minutes

Between 1 and 2 hours

Between 2 and 3 hours

Between 4 and 5 hours

Over 6 hours

Insulin is degraded by enzymes in the circulation. It typically has a half life of less than 30 minutes.
Abnormalities of the clearance of insulin may occur in type 2 diabetes.
Please rate this question:

Discuss and give feedback


Next question

Insulin

Insulin is a peptide hormone, produced by beta cells of the pancreas, and is central to regulating
carbohydrate and fat metabolism in the body. Insulin causes cells in the liver, skeletal muscles, and
fat tissue to absorb glucose from the blood. In the liver and skeletal muscles, glucose is stored as
glycogen, and in fat cells (adipocytes) it is stored as triglycerides.

Structure
The human insulin protein is composed of 51 amino acids, and has a molecular weight of 5808 Da. It
is a dimer of an A-chain and a B-chain, which are linked together by disulfide bonds.

Synthesis
Pro-insulin is formed by the rough endoplasmic reticulum in pancreatic beta cells. Then pro-insulin is
cleaved to form insulin and C-peptide. Insulin is stored in secretory granules and released in
response to Ca2+.

Function

 Secreted in response to hyperglycaemia


 Glucose utilisation and glycogen synthesis
 Inhibits lipolysis
 Reduces muscle protein loss
Which of the following statements about blood clotting is untrue?

Platelet adhesion to disrupted endothelium is dependent upon von Willebrand factor

Protein C is a vitamin K dependent substance

The bleeding time provides an assessment of platelet function

The prothrombin time tests the extrinsic system

Administration of aprotinin during liver transplantation surgery prolongs survival

Although aprotinin reduces fibrinolysis and thus bleeding, it is associated with increased risk of death
and was withdrawn in 2007. Protein C is dependent upon vitamin K and this may paradoxically
increase the risk of thrombosis during the early phases of warfarin treatment.
Please rate this question:

Discuss and give feedback


Next question

Coagulation cascade

Two pathways lead to fibrin formation

Intrinsic pathway (components already present in the blood)

 Minor role in clotting


 Subendothelial damage e.g. collagen
 Formation of the primary complex on collagen by high-molecular-weight kininogen (HMWK),
prekallikrein, and Factor 12
 Prekallikrein is converted to kallikrein and Factor 12 becomes activated
 Factor 12 activates Factor 11
 Factor 11 activates Factor 9, which with its co-factor Factor 8a form the tenase complex
which activates Factor 10

Extrinsic pathway (needs tissue factor released by damaged tissue)

 Tissue damage
 Factor 7 binds to Tissue factor
 This complex activates Factor 9
 Activated Factor 9 works with Factor 8 to activate Factor 10

Common pathway

 Activated Factor 10 causes the conversion of prothrombin to thrombin


 Thrombin hydrolyses fibrinogen peptide bonds to form fibrin and also activates factor 8 to
form links between fibrin molecules

Fibrinolysis
Plasminogen is converted to plasmin to facilitate clot resorption

Image sourced from Wikipedia

Intrinsic pathway Increased APTT Factors 8,9,11,12

Extrinsic pathway Increased PT Factor 7

Common pathway Increased APTT & PT Factors 2,5,10

Vitamin K dependent Factors 2,7,9,10

Next question
Theme: Management of vomiting

A. Ondansetron
B. Metoclopramide
C. Cyclizine
D. Erythromycin
E. Cisapride
F. Haloperidol

Please select the most appropriate drug for the given scenario. Each option may be used once,
more than once or not at all.

13. A 78 year old man with diabetes develops autonomic gastropathy with persistent and
troublesome vomiting.

You answered Ondansetron

The correct answer is Erythromycin

Unlike metoclopramide. the effects of erythromycin on gastric emptying are not mediated
via the vagus nerve.

14. A drug which blocks the chemoreceptor trigger zone in the area postrema.

Ondansetron

5 HT3 blockers are most effective for many types of nausea for this reason.

15. A 48 year old man with oesphageal varices has a profuse haemorrhage on the ward.

You answered Ondansetron

The correct answer is Metoclopramide

Intravenous metoclopramide causes increased oesophageal pressure and this may


temporarily slow the rate of haemorrhage whilst more definitive measures are instigated.

Please rate this question:

Discuss and give feedback


Next question
Vomiting

Reflex oral expulsion of gastric (and sometimes intestinal) contents - reverse peristalsis and
abdominal contraction

The vomiting centre is in part of the medulla oblongata and is triggered by receptors in several
locations:

 Labyrinthine receptors of ear (motion sickness)


 Over distention receptors of duodenum and stomach
 Trigger zone of CNS - many drugs (e.g., opiates) act here
 Touch receptors in throat

Next question
Which of the following cell types is least likely to be found in a wound 1 week following injury?

Macrophages

Fibroblasts

Myofibroblasts

Endothelial cells

Neutrophils

Theme from April 2012 Exam


Myofibroblasts are differentiated fibroblasts, in which the cytoskeleton contains actin filaments.
These cell types facilitate wound contracture and are the hallmark of a mature wound. They are
almost never found in wounds less than 1 month old.
Please rate this question:

Discuss and give feedback


Next question

Phases of wound healing

Phase Key features Cells Timeframe

Haemostasis  Vasospasm in adjacent vessels Erythrocytes and Seconds/


 Platelet plug formation and platelets Minutes
generation of fibrin rich clot

Inflammation  Neutrophils migrate into wound Neutrophils, Days


(function impaired in diabetes). fibroblasts and
 Growth factors released, including macrophages
basic fibroblast growth factor and
vascular endothelial growth factor.
 Fibroblasts replicate within the
adjacent matrix and migrate into
wound.
 Macrophages and fibroblasts
couple matrix regeneration and clot
substitution.

Regeneration  Platelet derived growth factor and Fibroblasts, Weeks


transformation growth factors endothelial cells,
stimulate fibroblasts and epithelial macrophages
cells.
 Fibroblasts produce a collagen
network.
 Angiogenesis occurs and wound
resembles granulation tissue.

Remodelling  Longest phase of the healing Myofibroblasts 6 weeks to 1


process and may last up to one year year
(or longer).
 During this phase fibroblasts
become differentiated
(myofibroblasts) and these
facilitate wound contraction.
 Collagen fibres are remodelled.
 Microvessels regress leaving a pale
scar.

Next question
The blood - brain barrier is not highly permeable to which of the following?

Carbon dioxide

Barbituates

Glucose

Oxygen

Hydrogen ions

The blood brain barrier is relatively impermeable to highly dissociated compounds.


Please rate this question:

Discuss and give feedback


Next question

Cerebrospinal fluid

The CSF fills the space between the arachnoid mater and pia mater (covering surface of the brain).
The total volume of CSF in the brain is approximately 150ml. Approximately 500 ml is produced by
the ependymal cells in the choroid plexus (70%), or blood vessels (30%). It is reabsorbed via the
arachnoid granulations which project into the venous sinuses.

Circulation
1. Lateral ventricles (via foramen of Munro)
2. 3rd ventricle
3. Cerebral aqueduct (aqueduct of Sylvius)
4. 4th ventricle (via foramina of Magendie and Luschka)
5. Subarachnoid space
6. Reabsorbed into the venous system via arachnoid granulations into superior sagittal sinus

Composition

 Glucose: 50-80mg/dl
 Protein: 15-40 mg/dl
 Red blood cells: Nil
 White blood cells: 0-3 cells/ mm3

Next question
A 43 year old presents to the urology clinic complaining of impotence. Which of the following will
occur in response to increased penile parasympathetic stimulation?

Detumescence

Ejaculation

Erection

Vasospasm of the penile branches of the pudendal artery

Contraction of the smooth muscle in the epididymis and vas deferens

Memory aid for erection


p=parasympathetic=points
s=sympathetic=shoots

Parasympathetic stimulation causes erection. Sympathetic stimulation will produce ejaculation,


detumescence and vasospasm of the pudendal artery. It will also cause contraction of the smooth
muscle in the epididymis and vas to convey the ejaculate.
Please rate this question:

Discuss and give feedback


Next question

Penile erection

Physiology of erection
Autonomic  Sympathetic nerves originate from T11-L2 and parasympathetic nerves
from S2-4 join to form pelvic plexus.
 Parasympathetic discharge causes erection, sympathetic discharge causes
ejaculation and detumescence.

Somatic Supplied by dorsal penile and pudendal nerves. Efferent signals are relayed from
nerves Onufs nucleus (S2-4) to innervate ischiocavernosus and bulbocavernosus muscles.

Autonomic discharge to the penis will trigger the veno-occlusive mechanism which triggers the flow
of arterial blood into the penile sinusoidal spaces. As the inflow increases the increased volume in
this space will secondarily lead to compression of the subtunical venous plexus with reduced venous
return. During the detumesence phase the arteriolar constriction will reduce arterial inflow and
thereby allow venous return to normalise.

Priapism
Prolonged unwanted erection, in the absence of sexual desire, lasting more than 4 hours.

Classification of priaprism
Low flow priaprism Due to veno-occlusion (high intracavernosal pressures).

 Most common type


 Often painful
 Often low cavernosal flow
 If present for >4 hours requires emergency treatment

High flow priaprism Due to unregulated arterial blood flow.

 Usually presents as semi rigid painless erection

Recurrent priaprism Typically seen in sickle cell disease, most commonly of high flow type.

Causes

 Intracavernosal drug therapies (e.g. for erectile dysfunction>


 Blood disorders such as leukaemia and sickle cell disease
 Neurogenic disorders such as spinal cord transection
 Trauma to penis resulting in arterio-venous malformations

Tests

 Exclude sickle cell/ leukaemia


 Consider blood sampling from cavernosa to determine whether high or low flow (low flow is
often hypoxic)

Management

 Ice packs/ cold showers


 If due to low flow then blood may be aspirated from copora or try intracavernosal alpha
adrenergic agonists.
 Delayed therapy of low flow priaprism may result in erectile dysfunction.
 Which of the following best accounts for the action of PTH in increasing serum calcium
levels?

Activation of vitamin D to increase absorption of calcium from the small intestine.

Direct stimulation of osteoclasts to absorb bone with release of calcium.

Stimulation of phosphate absorption at the distal convoluted tubule of the kidney.

Decreased porosity of the vessels at Bowmans capsule to calcium.

Vasospasm of the afferent renal arteriole thereby reducing GFR and calcium urinary loss.


Theme from April 2012 Exam
PTH increases the activity of 1-α-hydroxylase enzyme, which converts 25-
hydroxycholecalciferol to 1,25-dihydroxycholecalciferol, the active form of vitamin D.
Osteoclasts do not have a PTH receptor and effects are mediated via osteoblasts.
 Please rate this question:


 Discuss and give feedback

 Next question

 Parathyroid hormone

 Parathyroid hormone is secreted by the chief cells of the parathyroid glands. It acts to
increase serum calcium concentration by stimulation of the PTH receptors in the kidney and
bone. PTH has a plasma half life of 4 minutes.

Effects of PTH

Bone Binds to osteoblasts which signal to osteoclasts to cause resorption of bone and
release calcium.
Kidney Active reabsorption of calcium and magnesium from the distal convoluted tubule.
Decreases reabsorption of phosphate.

Intestine via Increases intestinal calcium absorption by increasing activated vitamin D. Activated
kidney vitamin D increases calcium absorption.

 Next question

Which of the following drugs does not cause syndrome of inappropriate anti diuretic hormone
release?

Haloperidol

Carbamazepine

Amitriptylline

Cyclophosphamide

Methotrexate

Drugs causing SIADH: ABCD

A nalgesics: opioids, NSAIDs


B arbiturates
C yclophosphamide/ Chlorpromazine/ Carbamazepine
D iuretic (thiazides)

Please rate this question:

Discuss and give feedback


Next question

Hyponatraemia

This is commonly tested in the MRCS (despite most surgeons automatically seeking medical advice
if this occurs!). The most common cause in surgery is the over administration of 5% dextrose.

Hyponatraemia may be caused by water excess or sodium depletion. Causes of


pseudohyponatraemia include hyperlipidaemia (increase in serum volume) or a taking blood from a
drip arm. Urinary sodium and osmolarity levels aid making a diagnosis.

Classification
Urinary sodium > 20 Sodium depletion, renal loss Mnemonic: Syndrome of
mmol/l INAPPropriate Anti-
Diuretic Hormone:
 Patient often hypovolaemic In creased
Na (sodium)
 Diuretics (thiazides) PP (urine)
 Addison's
 Diuretic stage of renal failure
 SIADH (serum osmolality
low, urine osmolality high,
urine Na high)
 Patient often euvolaemic

Urinary sodium < 20 Sodium depletion, extra-renal loss


mmol/l

 Diarrhoea, vomiting,
sweating
 Burns, adenoma of rectum (if
villous lesion and large)

Water excess (patient  Secondary


often hypervolaemic and hyperaldosteronism: CCF,
oedematous) cirrhosis
 Reduced GFR: renal failure
 IV dextrose, psychogenic
polydipsia

Management

Symptomatic Hyponatremia :

Acute hyponatraemia with Na <120: immediate therapy. Central Pontine Myelinolisis, may occur
from overly rapid correction of serum sodium. Aim to correct until the Na is > 125 at a rate of 1
mEq/h. Normal saline with frusemide is an alternative method.

The sodium requirement can be calculated as follows :

(125 - serum sodium) x 0.6 x body weight = required mEq of sodium


Next question
Which of the following changes are not typically seen in established dehydration?

Rising haematocrit

Urinary sodium <20mmol/ litre

Metabolic acidosis

Decreased serum urea to creatinine ratio

Hypernatraemia

Diagnosing dehydration can be complicated, laboratory features include:

 Hypernatraemia
 Rising haematocrit
 Metabolic acidosis
 Rising lactate
 Increased serum urea to creatinine ratio
 Urinary sodium <20 mmol/litre
 Urine osmolality approaching 1200mosmol/kg

Please rate this question:

Discuss and give feedback


Next question

Pre operative fluid management

Fluid management has been described in the British Consensus guidelines on IV fluid
therapy for Adult Surgical patients (GIFTASUP) and by NICE (CG174 December 2013)

The Recommendations include:

 Use Ringer's lactate or Hartmann's when a crystalloid is needed for resuscitation or


replacement of fluids. Avoid 0.9% N. Saline (due to risk of hyperchloraemic acidosis) unless
patient vomiting or has gastric drainage.
 Use 4%/0.18% dextrose saline or 5% dextrose in maintenance fluids. It should not be used
in resuscitation or as replacement fluids.
 Adult maintenance fluid requirements are: Na 50-100 mmol/day and K 40-80 mmol/day in
1.5-2.5L fluid per day.
 Patients for elective surgery should NOT be nil by mouth for >2 hours (unless has disorder of
gastric emptying).
 Patients for elective surgery should be given carbohydrate rich drinks 2-3h before. Ideally
this should form part of a normal pre op plan to facilitate recovery.
 Avoid mechanical bowel preparation.
 If bowel prep is used, simultaneous administration of Hartmann's or Ringer's lactate should
be considered.
 Excessive fluid losses from vomiting should be treated with a crystalloid with potassium
replacement. 0.9% N. Saline should be given if there is hypochloraemia. Otherwise
Hartmann's or Ringer lactate should be given for diarrhoea/ileostomy/ileus/obstruction.
Hartmann's should also be given in sodium losses secondary to diuretics.
 High risk patients should receive fluids and inotropes.
 An attempt should be made to detect pre or operative hypovolaemia using flow based
measurements. If this is not available, then clinical evaluation is needed i.e. JVP, pulse
volume etc.
 In Blood loss or infection causing hypovolaemia should be treated with a balanced crystalloid
or colloid (or until blood available in blood loss). A critically ill patient is unable to excrete Na
or H20 leading to a 5% risk of interstitial oedema. Therefore 5% dextrose as well as colloid
should be given.
 If patients need IV fluid resuscitation, use crystalloids that contain sodium in the range 130-
154 mmol/l, with a bolus of 500 ml over less than 15 minutes (NICE Guidance CG 174).

Next question
A 67 year old male is admitted to the surgical unit with acute abdominal pain. He is found to have a
right sided pneumonia. The nursing staff put him onto 15L O 2 via a non rebreathe mask. After 30
minutes the patient is found moribund, sweaty and agitated by the nursing staff. An arterial blood
gas reveals:

pH 7.15

pCO2 10.2

pO2 8

Bicarbonate 32

Base excess - 5.2

What is the most likely cause for this patients deterioration?

Acute respiratory alkalosis secondary to hyperventilation

Over administration of oxygen in a COPD patient

Metabolic acidosis secondary to severe pancreatitis

Metabolic alkalosis secondary to hypokalaemia

Acute respiratory acidosis secondary to pneumonia

Theme from April 2012 exam


This patient has an acute respiratory acidosis, however this is on a background of chronic
respiratory acidosis (due to COPD) with a compensatory metabolic alkalosis (the elevated
bicarbonate is the main clue to the chronic nature of the respiratory acidosis). This blood gas picture
is typical in a COPD patient who has received too much oxygen; these patients lose their hypoxic
drive for respiration, therefore retain CO2 and subsequently hypoventilate leading to respiratory
arrest. If the bicarbonate was normal, then the answer would be acute respiratory acidosis
secondary to pneumonia.

Please rate this question:

Discuss and give feedback

Next question

Arterial blood gas interpretation

In advanced life support training, a 5 step approach to arterial blood gas interpretation is advocated.

1. How is the patient?

2. Is the patient hypoxaemic?


The Pa02 on air should be 10.0-13.0 kPa

3. Is the patient acidaemic (pH <7.35) or alkalaemic (pH >7.45)

4. What has happened to the PaCO2?


If there is acidaemia, an elevated PaCO2 will account for this

5. What is the bicarbonate level or base excess?


A metabolic acidosis will have a low bicarbonate level and a low base excess (< -2 mmol)
A metabolic alkalosis will have a high bicarbonate and a high base excess (> +2 mmol)

Next question
Which of the following statements relating to the pharmacology of warfarin is untrue?

Interferes with clotting factors 2,7,9 and 10

It may not be clinically effective for up to 72 hours

The half life of warfarin is 40 hours

Warfarin has a large volume of distribution

It is metabolized in the liver

Factors 2,7,9,10 affected

Warfarin interferes with fibrin formation by affecting carboxylation of glutamic acid residues in factors
2,7,9 and 10. Factor 2 has the longest half life of approximately 60 hours, therefore it can take up to
3 days for warfarin to be fully effective. Warfarin has a small volume of distribution as it is protein
bound.
Please rate this question:

Discuss and give feedback


Next question

Warfarin

Warfarin is an oral anticoagulant which inhibits the reduction of vitamin K to its active hydroquinone
form, which in turn acts as a cofactor in the formation of clotting factor II, VII, IX and X (mnemonic =
1972) and protein C

Factors that may potentiate warfarin

 Liver disease
 P450 enzyme inhibitors, e.g.: amiodarone, ciprofloxacin
 Cranberry juice
 Drugs which displace warfarin from plasma albumin, e.g. NSAIDs
 Inhibit platelet function: NSAIDs

Side-effects
 Haemorrhage
 Teratogenic
 Skin necrosis: when warfarin is first started biosynthesis of protein C is reduced. This results
in a temporary procoagulant state after initially starting warfarin, normally avoided by
concurrent heparin administration. Thrombosis may occur in venules leading to skin
necrosis.

Next question
Which of the following does not cause an increased anion gap acidosis?

Uraemia

Paraldehyde

Diabetic ketoacidosis

Ethylene glycol

Acetazolamide

Causes of increased anion acidosis: MUDPILES

M - Methanol
U - Uraemia
D - DKA/AKA
P - Paraldehyde/phenformin
I - Iron/INH
L - Lactic acidosis
E - Ethylene glycol
S - Salicylates

Please rate this question:

Discuss and give feedback


Next question

Disorders of acid - base balance

Disorders of acid- base balance are often covered in the MRCS part A, both in the SBA and EMQ
sections.

The acid-base normogram below shows how the various disorders may be categorised
Image sourced from Wikipedia

Metabolic acidosis

 This is the most common surgical acid - base disorder.


 Reduction in plasma bicarbonate levels.
 Two mechanisms:

1. Gain of strong acid (e.g. diabetic ketoacidosis)


2. Loss of base (e.g. from bowel in diarrhoea)
- Classified according to the anion gap, this can be calculated by:
(Na+ + K+) - (Cl- + HCO3-).
- If a question supplies the chloride level then this is often a clue that the anion gap should be
calculated. The normal range = 10-18 mmol/L

Normal anion gap ( = hyperchloraemic metabolic acidosis)

 Gastrointestinal bicarbonate loss: diarrhoea, ureterosigmoidostomy, fistula


 Renal tubular acidosis
 Drugs: e.g. acetazolamide
 Ammonium chloride injection
 Addison's disease

Raised anion gap


 Lactate: shock, hypoxia
 Ketones: diabetic ketoacidosis, alcohol
 Urate: renal failure
 Acid poisoning: salicylates, methanol

Metabolic acidosis secondary to high lactate levels may be subdivided into two types:

 Lactic acidosis type A: (Perfusion disorders e.g.shock, hypoxia, burns)


 Lactic acidosis type B: (Metabolic e.g. metformin toxicity)

Metabolic alkalosis

 Usually caused by a rise in plasma bicarbonate levels.


 Rise of bicarbonate above 24 mmol/L will typically result in renal excretion of excess
bicarbonate.
 Caused by a loss of hydrogen ions or a gain of bicarbonate. It is due mainly to problems of
the kidney or gastrointestinal tract

Causes

 Vomiting / aspiration (e.g. Peptic ulcer leading to pyloric stenosis, nasogastric suction)
 Diuretics
 Liquorice, carbenoxolone
 Hypokalaemia
 Primary hyperaldosteronism
 Cushing's syndrome
 Bartter's syndrome
 Congenital adrenal hyperplasia

Mechanism of metabolic alkalosis

 Activation of renin-angiotensin II-aldosterone (RAA) system is a key factor


 Aldosterone causes reabsorption of Na+ in exchange for H+ in the distal convoluted tubule
 ECF depletion (vomiting, diuretics) → Na+ and Cl- loss → activation of RAA system → raised
aldosterone levels
 In hypokalaemia, K+ shift from cells → ECF, alkalosis is caused by shift of H + into cells to
maintain neutrality
Respiratory acidosis

 Rise in carbon dioxide levels usually as a result of alveolar hypoventilation


 Renal compensation may occur leading to Compensated respiratory acidosis

Causes

 COPD
 Decompensation in other respiratory conditions e.g. Life-threatening asthma / pulmonary
oedema
 Sedative drugs: benzodiazepines, opiate overdose

Respiratory alkalosis

 Hyperventilation resulting in excess loss of carbon dioxide


 This will result in increasing pH

Causes

 Psychogenic: anxiety leading to hyperventilation


 Hypoxia causing a subsequent hyperventilation: pulmonary embolism, high altitude
 Early salicylate poisoning*
 CNS stimulation: stroke, subarachnoid haemorrhage, encephalitis
 Pregnancy

*Salicylate overdose leads to a mixed respiratory alkalosis and metabolic acidosis. Early stimulation
of the respiratory centre leads to a respiratory alkalosis whilst later the direct acid effects of
salicylates (combined with acute renal failure) may lead to an acidosis
Next question
Which one of the following is least associated with thrombocytopenia?

Heparin therapy

Rheumatoid arthritis

Infectious mononucleosis

Liver disease

Pregnancy

Rheumatoid arthritis, unlike systemic lupus erythematous, is generally associated with a


thrombocytosis. In some cases of Felty's syndrome thrombocytopaenia may be seen secondary to
hypersplenism. This however represents a small percentage of patients with rheumatoid arthritis.
Please rate this question:

Discuss and give feedback


Next question

Thrombocytopenia

Causes of severe thrombocytopenia

 ITP
 DIC
 TTP
 haematological malignancy

Causes of moderate thrombocytopenia

 heparin induced thrombocytopenia (HIT)


 drug-induced (e.g. quinine, diuretics, sulphonamides, aspirin, thiazides)
 alcohol
 liver disease
 hypersplenism
 viral infection (EBV, HIV, hepatitis)
 pregnancy
 SLE/antiphospholipid syndrome
 vitamin B12 deficiency

Next question
Which of the following will increase the volume of pancreatic exocrine secretions?

Octreotide

Cholecystokinin

Aldosterone

Adrenaline

None of the above

Cholecystokinin will often increase the volume of pancreatic secretions.

Please rate this question:

Discuss and give feedback

Next question

Pancreas exocrine physiology

Composition of pancreatic secretions


Pancreatic secretions are usually 1000-1500ml per 24 hours and have a pH of 8.

Secretion Source Substances secreted

Trypsinogen
Procarboxylase
Enzymic Acinar cells
Amylase
Elastase

Aqueous Sodium
Ductal and
Bicarbonate
Centroacinar cells Water
Potassium
Chloride
NB: Sodium and potassium reflect their plasma levels; chloride
and bicarbonate vary with flow rate

Regulation
The cephalic and gastric phases (neuronal and physical) are less important in regulating the
pancreatic secretions. The effect of digested material in the small bowel stimulates CCK release and
ACh which stimulate acinar and ductal cells. Of these CCK is the most potent stimulus. In the case
of the ductal cells these are potently stimulated by secretin which is released by the S cells of the
duodenum. This results in an increase in bicarbonate.

Enzyme activation
Trypsinogen is converted via enterokinase to active trypsin in the duodenum. Trypsin then activates
the other inactive enzymes

Next question
Where is the majority of iron found in the body?

Bone

Haemoglobin

Ferritin and haemosiderin

Myoglobin

Plasma iron

Approximately 70% of body iron is found bound to haemoglobin.


Please rate this question:

Discuss and give feedback


Next question

Iron metabolism

Absorption  Duodenum and upper jejunum


 About 10% of dietary iron absorbed
 Fe2+ (ferrous iron) much better absorbed than Fe3+ (ferric iron)
 Ferrous iron is oxidized to form ferric iron, which is combined with
apoferritin to form ferritin
 Absorption is regulated according to body's need
 Increased by vitamin C, gastric acid
 Decreased by proton pump inhibitors, tetracycline, gastric achlorhydria,
tannin (found in tea)

Transport In plasma as Fe3+ bound to transferrin

Storage Ferritin (or haemosiderin) in bone marrow

Excretion Lost via intestinal tract following desquamation

Distribution in body
Total body iron 4g

Haemoglobin 70%

Ferritin and haemosiderin 25%

4%
Myoglobin

Plasma iron 0.1%

Next question
A 44 year old man recieves a large volume transfusion of whole blood. The whole blood is two
weeks old. Which of the following best describes its handling of oxygen?

It will have a low affinity for oxygen

Its affinity for oxygen is unchanged

It will more readily release oxygen in metabolically active tissues than fresh blood

The release of oxygen in metabolically active tissues will be the same as fresh blood

It will have an increased affinity for oxygen

Stored blood has less 2,3 DPG and therefore has a higher affinity for oxygen, this reduces its ability
to release it at metabolising tissues.
Please rate this question:

Discuss and give feedback


Next question

Oxygen Transport

Oxygen transport
Almost all oxygen is transported within erythrocytes. It has limited solubility and only 1% is carried as
solution. Therefore the amount of oxygen transported will depend upon haemoglobin concentration
and its degree of saturation.

Haemoglobin
Globular protein composed of 4 subunits. Haem consists of a protoporphyrin ring surrounding an
iron atom in its ferrous state. The iron can form two additional bonds; one with oxygen and the other
with a polypeptide chain. There are two alpha and two beta subunits to this polypeptide chain in an
adult and together these form globin. Globin cannot bind oxygen but is able to bind to carbon dioxide
and hydrogen ions, the beta chains are able to bind to 2,3 diphosphoglycerate. The oxygenation of
haemoglobin is a reversible reaction. The molecular shape of haemoglobin is such that binding of
one oxygen molecule facilitates the binding of subsequent molecules.

Oxygen dissociation curve


 The oxygen dissociation curve describes the relationship between the percentage of
saturated haemoglobin and partial pressure of oxygen in the blood. It is not affected by
haemoglobin concentration.
 Chronic anaemia causes 2, 3 DPG levels to increase, hence shifting the curve to the right

Haldane effect

 Shifts to left = for given oxygen tension there is increased saturation of Hb with oxygen i.e.
Decreased oxygen delivery to tissues

Bohr effect

 Shifts to right = for given oxygen tension there is reduced saturation of Hb with oxygen i.e.
Enhanced oxygen delivery to tissues

Image sourced from Wikipedia

Shifts to Left = Lower oxygen delivery Shifts to Right = Raised oxygen


delivery

 HbF, methaemoglobin, carboxyhaemoglobin


 low [H+] (alkali)
 low pCO2  raised [H+] (acidic)
 low 2,3-DPG  raised pCO2
 low temperature  raised 2,3-DPG*
 raised temperature

*2,3-diphosphoglycerate
Next question
A 43 year old lady undergoes a day case laparoscopic cholecystectomy. The operation is more
difficult than anticipated and a drain is placed to the operative site. Whilst in recovery, the patient
loses 1800ml of frank blood into the drain. Which of the following will not occur?

Release of aldosterone via the Bainbridge reflex

Reduced urinary sodium excretion

Increase in sympathetic discharge to ventricular muscle

Fall in parasympathetic discharge to the sino atrial node

Decreased stimulation from atrial pressure receptors

Theme from 2013 Exam


The Bainbridge reflex is the increase in heart rate mediated via atrial stretch receptors that occurs
following a rapid infusion of blood.
Please rate this question:

Discuss and give feedback


Next question

Cardiac physiology

 The heart has four chambers ejecting blood into both low pressure and high pressure
systems.
 The pumps generate pressures of between 0-25mmHg on the right side and 0-120 mmHg on
the left.
 At rest diastole comprises 2/3 of the cardiac cycle.
 The product of the frequency of heart rate and stroke volume combine to give the cardiac
output which is typically 5-6L per minute.

Detailed descriptions of the various waveforms are often not a feature of MRCS A (although they are
on the syllabus). However, they are a very popular topic for surgical physiology in the MRCS B
exam.

Electrical properties

 Intrinsic myogenic rhythm within cardiac myocytes means that even the denervated heart is
capable of contraction.
 In the normal situation the cardiac impulse is generated in the sino atrial node in the right
atrium and conveyed to the ventricles via the atrioventricular node.
 The sino atrial node is also capable of spontaneous discharge and in the absence of
background vagal tone will typically discharge around 100x per minute. Hence the higher
resting heart rate found in cardiac transplant cases. In the SA and AV nodes the resting
membrane potential is lower than in surrounding cardiac cells and will slowly depolarise from
-70mV to around -50mV at which point an action potential is generated.
 Differences in the depolarisation slopes between SA and AV nodes help to explain why the
SA node will depolarise first. The cells have a refractory period during which they cannot be
re-stimulated and this period allows for adequate ventricular filling. In pathological
tachycardic states this time period is overridden and inadequate ventricular filling may then
occur, cardiac output falls and syncope may ensue.

Parasympathetic fibres project to the heart via the vagus and will release acetylcholine. Sympathetic
fibres release nor adrenaline and circulating adrenaline comes from the adrenal medulla.
Noradrenaline binds to β 1 receptors in the SA node and increases the rate of pacemaker potential
depolarisation.

Cardiac cycle

Image sourced from Wikipedia

 Mid diastole: AV valves open. Ventricles hold 80% of final volume. Outflow valves shut.
Aortic pressure is high.

 Late diastole: Atria contract. Ventricles receive 20% to complete filling. Typical end diastolic
volume 130-160ml.
 Early systole: AV valves shut. Ventricular pressure rises. Isovolumetric ventricular
contraction. AV Valves bulge into atria (c-wave). Aortic and pulmonary pressure exceeded-
blood is ejected. Shortening of ventricles pulls atria downwards and drops intra atrial
pressure (x-descent).

 Late systole: Ventricular muscles relax and ventricular pressures drop. Although ventricular
pressure drops the aortic pressure remains constant owing to peripheral vascular resistance
and elastic property of the aorta. Brief period of retrograde flow that occurs in aortic recoil
shuts the aortic valve. Ventricles will contain 60ml end systolic volume. The average stroke
volume is 70ml (i.e. Volume ejected).

 Early diastole: All valves are closed. Isovolumetric ventricular relaxation occurs. Pressure
wave associated with closure of the aortic valve increases aortic pressure. The pressure dip
before this rise can be seen on arterial waveforms and is called the incisura. During systole
the atrial pressure increases such that it is now above zero (v- wave). Eventually atrial
pressure exceed ventricular pressure and AV valves open - atria empty passively into
ventricles and atrial pressure falls (y -descent )

The negative atrial pressures are of clinical importance as they can allow air embolization to occur if
the neck veins are exposed to air. This patient positioning is important in head and neck surgery to
avoid this occurrence if veins are inadvertently cut, or during CVP line insertion.

Mechanical properties

 Preload = end diastolic volume


 Afterload = aortic pressure

It is important to understand the principles of Laplace's law in surgery.

 It states that for hollow organs with a circular cross section, the total circumferential wall
tension depends upon the circumference of the wall, multiplied by the thickness of the wall
and on the wall tension.
 The total luminal pressure depends upon the cross sectional area of the lumen and the
transmural pressure. Transmural pressure is the internal pressure minus external pressure
and at equilibrium the total pressure must counterbalance each other.
 In terms of cardiac physiology the law explains that the rise in ventricular pressure that
occurs during the ejection phase is due to physical change in heart size. It also explains why
a dilated diseased heart will have impaired systolic function.
Starlings law

 Increase in end diastolic volume will produce larger stroke volume.


 This occurs up to a point beyond which cardiac fibres are excessively stretched and stroke
volume will fall once more. It is important for the regulation of cardiac output in cardiac
transplant patients who need to increase their cardiac output.

Baroreceptor reflexes

 Baroreceptors located in aortic arch and carotid sinus.


 Aortic baroreceptor impulses travel via the vagus and from the carotid via the
glossopharyngeal nerve.
 They are stimulated by arterial stretch.
 Even at normal blood pressures they are tonically active.
 Increase in baroreceptor discharge causes:

*Increased parasympathetic discharge to the SA node.


*Decreased sympathetic discharge to ventricular muscle causing decreased contractility and fall in
stroke volume.
*Decreased sympathetic discharge to venous system causing increased compliance.
*Decreased peripheral arterial vascular resistance

Atrial stretch receptors

 Located in atria at junction between pulmonary veins and vena cava.


 Stimulated by atrial stretch and are thus low pressure sensors.
 Increased blood volume will cause increased parasympathetic activity.
 Very rapid infusion of blood will result in increase in heart rate mediated via atrial receptors:
theBainbridge reflex.
 Decreases in receptor stimulation results in increased sympathetic activity this will decrease
renal blood flow-decreases GFR-decreases urinary sodium excretion-renin secretion by
juxtaglomerular apparatus-Increase in angiotensin II.
 Increased atrial stretch will also result in increased release of atrial natriuretic peptide.

Next question
Approximately what proportion of salivary secretions is provided by the submandibular glands?

10%

70%

40%

90%

20%

Although they are small, the submandibular glands provide the bulk of salivary secretions and
contribute 70%, the sublingual glands provide 5% and the remainder from the parotid.
Please rate this question:

Discuss and give feedback


Next question

Submandibular gland

Relations of the submandibular gland


Superficial Platysma, deep fascia and mandible
Submandibular lymph nodes
Facial vein (facial artery near mandible)
Marginal mandibular nerve
Cervical branch of the facial nerve

Deep Facial artery (inferior to the mandible)


Mylohoid muscle
Sub mandibular duct
Hyoglossus muscle
Lingual nerve
Submandibular ganglion
Hypoglossal nerve
Submandibular duct (Wharton's duct)

 Opens lateral to the lingual frenulum on the anterior floor of mouth.


 5 cm length
 Lingual nerve wraps around Wharton's duct. As the duct passes forwards it crosses medial
to the nerve to lie above it and then crosses back, lateral to it, to reach a position below the
nerve.

Innervation

 Sympathetic innervation- Derived from superior cervical ganglion


 Parasympathetic innervation- Submandibular ganglion via lingual nerve

Arterial supply
Branch of the facial artery. The facial artery passes through the gland to groove its deep surface. It
then emerges onto the face by passing between the gland and the mandible.

Venous drainage
Anterior facial vein (lies deep to the Marginal Mandibular nerve)

Lymphatic drainage
Deep cervical and jugular chains of nodes
Next question
Which is the least likely to cause hyperuricaemia?

Severe psoriasis

Lesch-Nyhan syndrome

Amiodarone

Diabetic ketoacidosis

Alcohol

Mnemonic of the drugs causing hyperuricaemia as a result of reduced excretion of urate

'Can't leap'

C iclosporin
A lcohol
N icotinic acid
T hiazides

L oop diuretics
E thambutol
A spirin
P yrazinamide

Decreased tubular secretion of urate occurs in patients with acidosis (eg, diabetic ketoacidosis,
ethanol or salicylate intoxication, starvation ketosis). The organic acids that accumulate in these
conditions compete with urate for tubular secretion.
Please rate this question:

Discuss and give feedback


Next question

Hyperuricaemia

 Increased levels of uric acid may be seen secondary to either increased cell turnover or
reduced renal excretion of uric acid. Hyperuricaemia may be found in asymptomatic patients
who have not experienced attacks of gout
 Hyperuricaemia may be associated with hyperlipidaemia and hypertension. It may also be
seen in conjunction with the metabolic syndrome

Increased synthesis

 Lesch-Nyhan disease
 Myeloproliferative disorders
 Diet rich in purines
 Exercise
 Psoriasis
 Cytotoxics

Decreased excretion

 Drugs: low-dose aspirin, diuretics, pyrazinamide


 Pre-eclampsia
 Alcohol
 Renal failure
 Lead

Next question
Which of the following statements relating to low molecular weight heparins is false?

They act via inhibition of Factor Xa

Large doses may be used prior to commencing cardiopulmonary bypass

They have a highly predictable pharmacokinetic profile

They are derivatives of unfractionated heparin

They have a molecular mass in the range of 3000-10000Da

As they are not easily reversed they are unsuitable for this purpose.
Please rate this question:

Discuss and give feedback


Next question

Heparin

 Causes the formation of complexes between antithrombin and activated thrombin/factors


7,9,10,11 & 12

Advantages of low molecular weight heparin

 Better bioavailability
 Lower risk of bleeding
 Longer half life
 Little effect on APTT at prophylactic dosages
 Less risk of HIT

Complications

 Bleeding
 Osteoporosis
 Heparin induced thrombocytopenia (HIT): occurs 5-14 days after 1st exposure
 Anaphylaxis
In surgical patients that may need a rapid return to theatre, administration of unfractionated heparin
is preferred; as low molecular weight heparins have a longer duration of action and are harder to
reverse.
Next question
A 43 year old lady presents with urinary incontinence. At which of the following locations is Onufs
nucleus likely to be found?

Medulla oblongata

Anterior horn of L5 nerve roots

Micturition centre in the Pons

Anterior horn of S2 nerve roots

None of the above

Onufs nucleus is located in the anterior horn of S2 and is the origin of neurones to the external
urethral sphincter.
Please rate this question:

Discuss and give feedback


Next question

Urinary incontinence

Involuntary passage of urine. Most cases are female (80%). It has a prevalence of 11% in those
aged greater than 65 years. The commonest variants include:

 Stress urinary incontinence (50%)


 Urge incontinence (15%)
 Mixed (35%)

Males
Males may also suffer from incontinence although it is a much rarer condition in men. A number of
anatomical factors contribute to this. Males have 2 powerful sphincters; one at the bladder neck and
the other in the urethra. Damage to the bladder neck mechanism is a factor in causing retrograde
ejaculation following prostatectomy. The short segment of urethra passing through the urogenital
diaphragm consists of striated muscle fibres (the external urethral sphincter) and smooth muscle
capable of more sustained contraction. It is the latter mechanism that maintains continence following
prostatectomy.

Females
The sphincter complex at the level of bladder neck is poorly developed in females. As a result the
external sphincter complex is functionally more important, its composition being similar to that of
males. Innervation is via the pudendal nerve and the neuropathy that may accompany obstetric
events may compromise this and lead to stress urinary incontinence.

Innervation
Somatic innervation to the bladder is via the pudendal, hypogastric and pelvic nerves. Autonomic
nerves travel in these nerve fibres too. Bladder filling leads to detrusor relaxation (sympathetic)
coupled with sphincter contraction. The parasympathetic system causes detrusor contraction and
sphincter relaxation. Overall control of micturition is centrally mediated via centres in the Pons.

Stress urinary incontinence

 50% of cases, especially in females.


 Damage (often obstetric) to the supporting structures surrounding the bladder may lead to
urethral hypermobility.
 Other cases due to sphincter dysfunction, usually from neurological disorders (e.g. Pudendal
neuropathy, multiple sclerosis).

Urethral mobility:
Pressure not transmitted appropriately to the urethra resulting in involuntary passage of urine during
episodes of raised intra-abdominal pressure.

Sphincter dysfunction:
Sphincter fails to adapt to compress urethra resulting in involuntary passage of urine. When the
sphincter completely fails there is often to continuous passage of urine.

Urge incontinence
In these patients there is sense of urgency followed by incontinence. The detrusor muscle in these
patients is unstable and urodynamic investigation will demonstrate overactivity of the detrusor
muscle at inappropriate times (e.g. Bladder filling). Urgency may be seen in patients with overt
neurological disorders and those without. The pathophysiology is not well understood but poor
central and peripheral co-ordination of the events surrounding bladder filling are the main
processes.

Assessment
Careful history and examination including vaginal examination for cystocele.
Bladder diary for at least 3 days
Consider flow cystometry if unclear symptomatology or surgery considered and diagnosis is unclear.
Exclusion of other organic disease (e.g. Stones, UTI, Cancer)

Management
Conservative measures should be tried first; Stress urinary incontinence or mixed symptoms should
undergo 3 months of pelvic floor exercise. Over active bladder should have 6 weeks of bladder
retraining.
Drug therapy for women with overactive bladder should be offered oxybutynin (or solifenacin if
elderly) if conservative measures fail.
In women with detrusor instability who fail non operative therapy a trial of sacral neuromodulation
may be considered, with conversion to permanent implant if good response. Augmentation
cystoplasty is an alternative but will involve long term intermittent self catheterisation.
In women with stress urinary incontinence a urethral sling type procedure may be undertaken.
Where cystocele is present in association with incontinence it should be repaired particularly if it lies
at the introitus.

NICE guidelines

 Initial assessment urinary incontinence should be classified as stress/urge/mixed.


 At least 3/7 bladder diary if unable to classify easily.
 Start conservative treatment before urodynamic studies if a diagnosis is obvious from the
history
 Urodynamic studies if plans for surgery.
 Stress incontinence: Pelvic floor exercises 3/12, if fails consider surgery.
 Urge incontinence: Bladder training >6/52, if fails for oxybutynin (antimuscarinic drugs) then
sacral nerve stimulation.
 Pelvic floor exercises offered to all women in their 1st pregnancy.

Next question
Which of the following is associated with reduced lung compliance?

Older age

Emphysematous type COPD

Decline in pulmonary blood flow

Adopting a vertical posture

Adjusting a ventilator to maintain high lung volumes

Increased lung compliance = Older age, COPD

Lung compliance is a measure of the ease of expansion of the lungs and thorax, determined by
pulmonary volume and elasticity. A high degree of compliance indicates a loss of elastic recoil of the
lungs, as in old age or emphysema. This increased lung compliance is due to loss of supportive
tissue around the airways. While a normal lung has a high passive elastic recoil, the sick lung has a
decreased elasticity (i.e. decreased transpulmonary pressure) which leads to increased lung
compliance.

Decreased compliance means that a greater change in pressure is needed for a given change in
volume, as in atelectasis, pulmonary fibrosis, pneumonia, or lack of surfactant.
Please rate this question:

Discuss and give feedback


Next question

Lung volumes

The diagram demonstrates lung volumes and capacities


Image sourced from Wikipedia

Definitions

Tidal volume (TV)  Is the volume of air inspired and expired during each
ventilatory cycle at rest.
 It is normally 500mls in males and 340mls in females.

Inspiratory reserve  Is the maximum volume of air that can be forcibly inhaled
volume (IRV) following a normal inspiration. 3000mls.

Expiratory reserve  Is the maximum volume of air that can be forcibly exhaled
volume (ERV) following a normal expiration. 1000mls.

Residual volume (RV)  Is that volume of air remaining in the lungs after a maximal
expiration.
 RV = FRC - ERV. 1500mls.

Functional residual  Is the volume of air remaining in the lungs at the end of a
capacity (FRC) normal expiration.
 FRC = RV + ERV. 2500mls.

Vital capacity (VC)  Is the maximal volume of air that can be forcibly exhaled after
a maximal inspiration.
 VC = TV + IRV + ERV. 4500mls in males, 3500mls in
females.
Total lung capacity  Is the volume of air in the lungs at the end of a maximal
(TLC) inspiration.
 TLC = FRC + TV + IRV = VC + RV. 5500-6000mls.

Forced vital capacity  The volume of air that can be maximally forcefully exhaled.
(FVC)
Next question
Which of the following statements relating to alveolar ventilation is untrue?

Anatomical dead space is measured by helium dilution

Physiological dead space is increased in PE

Alveolar ventilation is defined as the volume of fresh air entering the alveoli per minute

Anatomical dead space is increased by adrenaline

Type 2 pneumocytes in the alveoli secrete surfactant

Anatomical dead space is measured by Fowlers method.

A patient inhales 100% oxygen to empty the conducting zone gases of nitrogen and then exhales
through a mouthpiece which analyses the nitrogen concentration at the mouth. Initially the exhaled
gases contain no nitrogen as this is dead space gas; the nitrogen concentration will increase as the
alveolar gases are exhaled. Nitrogen which is measured following the breath of 100% oxygen must
then have come only from gas exchanging areas of the lung and not dead space.
Please rate this question:

Discuss and give feedback


Next question

Alveolar ventilation

 Minute ventilation is the total volume of gas ventilated per minute.

MV (ml/min)= tidal volume x Respiratory rate (resps/min).

 Dead space ventilation describes the volume of gas not involved in exchange in the blood.

There are 2 types:

1. Anatomical dead space: 150mls


 Volume of gas in the respiratory tree not involved in gaseous exchange: mouth, pharynx,
trachea, bronchi up to terminal bronchioles
 Measured by Fowlers method
 Increased by:
 Standing, increased size of person, increased lung volume and drugs causing
bronchodilatation e.g. Adrenaline

2. Physiological dead space: normal 150 mls, increases in ventilation/perfusion mismatch e.g. PE,
COPD, hypotension

 Volume of gas in the alveoli and anatomical dead space not involved in gaseous exchange.

 Alveolar ventilation is the volume of fresh air entering the alveoli per minute.

Alveolar ventilation = minute ventilation - Dead space volume


Next question
Which opioid receptor does morphine attach to?

mu

alpha

sigma

beta

kappa

Pethidine and other conventional opioids attach to this receptor.

Please rate this question:

Discuss and give feedback

Next question

Opioids

- Combine to specific opiate receptors in the CNS (periaqueductal grey matter, limbic system,
substantia gelatinosa)

- Morphine attaches to mu1 receptors

Next question
Which of the following inhibits the secretion of insulin?

Adrenaline

Lipids

Gastrin

Arginine

Vagal cholinergic activity

Inhibition of insulin release:

 Alpha adrenergic drugs


 Beta blockers
 Sympathetic nerves

Please rate this question:

Discuss and give feedback


Next question

Insulin

Insulin is a peptide hormone, produced by beta cells of the pancreas, and is central to regulating
carbohydrate and fat metabolism in the body. Insulin causes cells in the liver, skeletal muscles, and
fat tissue to absorb glucose from the blood. In the liver and skeletal muscles, glucose is stored as
glycogen, and in fat cells (adipocytes) it is stored as triglycerides.

Structure
The human insulin protein is composed of 51 amino acids, and has a molecular weight of 5808 Da. It
is a dimer of an A-chain and a B-chain, which are linked together by disulfide bonds.

Synthesis
Pro-insulin is formed by the rough endoplasmic reticulum in pancreatic beta cells. Then pro-insulin is
cleaved to form insulin and C-peptide. Insulin is stored in secretory granules and released in
response to Ca2+.
Function

 Secreted in response to hyperglycaemia


 Glucose utilisation and glycogen synthesis
 Inhibits lipolysis
 Reduces muscle protein loss

Next question
What is measured to obtain renal plasma flow?

Creatinine

Para-amino hippuric acid (PAH)

Inulin

Glucose

Protein

Theme from April 2016 Exam


Renal plasma flow = (amount of PAH in urine per unit time) / (difference in PAH concentration in the
renal artery or vein)

Normal value = 660ml/min

Please rate this question:

Discuss and give feedback

Next question

Renal Physiology

Overview

 Each nephron is supplied with blood from an afferent arteriole that opens onto the glomerular
capillary bed.
 Blood then flows to an efferent arteriole, supplying the peritubular capillaries and medullary
vasa recta.
 The kidney receives up to 25% of resting cardiac output.
Control of blood flow

 The kidney is able to autoregulate its blood flow between systolic pressures of 80- 180mmHg
so there is little variation in renal blood flow.
 This is achieved by myogenic control of arteriolar tone, both sympathetic input and hormonal
signals (e.g. renin) are responsible.

Glomerular structure and function

 Blood inside the glomerulus has considerable hydrostatic pressure.


 The basement membrane has pores that will allow free diffusion of smaller solutes, larger
negatively charged molecules such as albumin are unable to cross.
 The glomerular filtration rate (GFR) is equal to the concentration of a solute in the urine,
times the volume of urine produced per minute, divided by the plasma concentration
(assuming that the solute is freely diffused e.g. inulin).
 In clinical practice creatinine is used because it is subjected to very little proximal tubular
secretion.
 Although subject to variability, the typical GFR is 125ml per minute.
 Glomerular filtration rate = Total volume of plasma per unit time leaving the capillaries and
entering the bowman's capsule
 Renal clearance = volume plasma from which a substance is removed per minute by the
kidneys

Substances used to measure GFR have the following features:


1. Inert
2. Free filtration from the plasma at the glomerulus (not protein bound)
3. Not absorbed or secreted at the tubules
4. Plasma concentration constant during urine collection

Examples: inulin, creatinine

GFR = urine concentration (mmol/l) x urine volume (ml/min)


--------------------------------------------------------------------------
plasma concentration (mmol/l)

 The clearance of a substance is dependent not only on its diffusivity across the basement
membrane but also subsequent tubular secretion and / or reabsorption.
 So glucose which is freely filtered across the basement membrane is usually reabsorbed
from tubules giving a clearance of zero.
Tubular function

 Reabsorption and secretion of substances occurs in the tubules.


 In the proximal tubule substrates such as glucose, amino acids and phosphate are co-
transported with sodium across the semi permeable membrane.
 Up to two thirds of filtered water is reabsorbed in the proximal tubules.
 This will lead to increase in urea concentration in the distal tubule allowing for its increased
diffusion.
 Substances to be secreted into the tubules are taken up from the peritubular blood by tubular
cells.
 Solutes such as paraaminohippuric acid are cleared with a single passage through the
kidneys and this is why it is used to measure renal plasma flow. Ions such as calcium and
phosphate will have a tubular reabsorption that is influenced by plasma PTH levels.
 Potassium may be both secreted and re-absorbed and is co-exchanged with sodium.

Loop of Henle

 Approximately 60 litres of water containing 9000mmol sodium enters the descending limb of
the loop of Henle in 24 hours.
 Loops from the juxtamedullary nephrons run deep into the medulla.
 The osmolarity of fluid changes and is greatest at the tip of the papilla.
 The thin ascending limb is impermeable to water, but highly permeable to sodium and
chloride ions.
 This loss means that at the beginning of the thick ascending limb the fluid is hypo osmotic
compared with adjacent interstitial fluid.
 In the thick ascending limb the reabsorption of sodium and chloride ions occurs by both
facilitated and passive diffusion pathways.
 The loops of Henle are co-located with vasa recta, these will have similar solute
compositions to the surrounding extracellular fluid so preventing the diffusion and
subsequent removal of this hypertonic fluid.
 The energy dependent reabsorption of sodium and chloride in the thick ascending limb helps
to maintain this osmotic gradient.

Next question
Which of the following surgical procedures will have the greatest long term impact on a patients
calcium metabolism?

Distal gastrectomy

Cholecystectomy

Extensive small bowel resection

Sub total colectomy

Gastric banding for obesity

Calcium is mainly absorbed from the small bowel and this will have a direct long term impact on
calcium metabolism and increase the risk of osteoporosis. Gastric banding and distal gastrectomy
may affect a patients dietary choices but any potential deleterious nutritional intake may be
counteracted by administration of calcium supplements orally. Only 10% of calcium is absorbed from
the colon so that a sub total colectomy will only have a negligible effect.

Please rate this question:

Discuss and give feedback

Next question

Calcium homeostasis

Calcium ions are linked to a wide range of physiological processes. The largest store of bodily
calcium is contained within the skeleton. Calcium levels are primarily controlled by parathyroid
hormone, vitamin D and calcitonin.

Hormonal regulation of calcium


Hormone Actions

Parathyroid hormone (PTH)  Increase calcium levels and decrease phosphate levels
 Increases bone resorption
 Immediate action on osteoblasts to increase ca2+ in
extracellular fluid
 Osteoblasts produce a protein signaling molecule that
activate osteoclasts which cause bone resorption
 Increases renal tubular reabsorption of calcium
 Increases synthesis of 1,25(OH)2D (active form of
vitamin D) in the kidney which increases bowel
absorption of Ca2+
 Decreases renal phosphate reabsorption

1,25-dihydroxycholecalciferol (the  Increases plasma calcium and plasma phosphate


active form of vitamin D)  Increases renal tubular reabsorption and gut absorption
of calcium
 Increases osteoclastic activity
 Increases renal phosphate reabsorption

Calcitonin  Secreted by C cells of thyroid


 Inhibits intestinal calcium absorption
 Inhibits osteoclast activity
 Inhibits renal tubular absorption of calcium

Both growth hormone and thyroxine also play a small role in calcium metabolism.

Next question
A 52-year-old woman with a history of gastrectomy reports lethargy and a sore tongue. Blood tests
are reported as follows:

Hb 10.7 g/dl

MCV 121 fl

Plt 177 * 10^9/l

WBC 5.4 * 10^9/l

What is the most likely cause?

Vitamin B12 deficiency

Vitamin C deficiency

Iron deficiency anaemia

Anaemia of chronic disease

Vitamin E deficiency

A history of gastrectomy and a macrocytic anaemia should indicate a diagnosis of B12 deficiency.

Please rate this question:

Discuss and give feedback

Next question
Vitamin B12 deficiency

Vitamin B12 is mainly used in the body for red blood cell development and also maintenance of the
nervous system. It is absorbed after binding to intrinsic factor (secreted from parietal cells in the
stomach) and is actively absorbed in the terminal ileum. A small amount of vitamin B12 is passively
absorbed without being bound to intrinsic factor.

Causes of vitamin B12 deficiency

 pernicious anaemia
 post gastrectomy
 poor diet
 disorders of terminal ileum (site of absorption): Crohn's, blind-loop etc

Features of vitamin B12 deficiency

 macrocytic anaemia
 sore tongue and mouth
 neurological symptoms: e.g. Ataxia
 neuropsychiatric symptoms: e.g. Mood disturbances

Management

 if no neurological involvement 1 mg of IM hydroxocobalamin 3 times each week for 2 weeks,


then once every 3 months
 if a patient is also deficient in folic acid then it is important to treat the B12 deficiency first to
avoid precipitating subacute combined degeneration of the cord

Next question
A 43 year old lady is diagnosed with primary hyperparathyroidism. Her serum PTH levels are
elevated. An endocrine surgeon performs a parathyroidectomy. How long will it take for the serum
PTH levels to fall if the functioning adenoma has been successfully removed?

6 hours

24 hours

2 hours

1 hour

10 minutes

PTH has a very short half life usually less than 10 minutes. Therefore a demonstrable drop in serum
PTH should be identified within 10 minutes of removing the adenoma. This is useful clinically since it
is possible to check the serum PTH intraoperatively prior to skin closure and explore the other
glands if levels fail to fall.

Please rate this question:

Discuss and give feedback

Next question

Calcium homeostasis

Calcium ions are linked to a wide range of physiological processes. The largest store of bodily
calcium is contained within the skeleton. Calcium levels are primarily controlled by parathyroid
hormone, vitamin D and calcitonin.

Hormonal regulation of calcium


Hormone Actions

Parathyroid hormone (PTH)  Increase calcium levels and decrease phosphate levels
 Increases bone resorption
 Immediate action on osteoblasts to increase ca2+ in
extracellular fluid
 Osteoblasts produce a protein signaling molecule that
activate osteoclasts which cause bone resorption
 Increases renal tubular reabsorption of calcium
 Increases synthesis of 1,25(OH)2D (active form of
vitamin D) in the kidney which increases bowel
absorption of Ca2+
 Decreases renal phosphate reabsorption

1,25-dihydroxycholecalciferol (the  Increases plasma calcium and plasma phosphate


active form of vitamin D)  Increases renal tubular reabsorption and gut absorption
of calcium
 Increases osteoclastic activity
 Increases renal phosphate reabsorption

Calcitonin  Secreted by C cells of thyroid


 Inhibits intestinal calcium absorption
 Inhibits osteoclast activity
 Inhibits renal tubular absorption of calcium

Both growth hormone and thyroxine also play a small role in calcium metabolism.

Next question
Which of the following statements relating to abnormal coagulation is false?

Warfarin affects the synthesis of factor 2,7,9,10

The prothrombin time is prolonged in Haemophilia A

Cholestatic jaundice can cause vitamin K deficiency

Disseminated intravascular coagulation is associated with thrombocytopenia

Massive transfusion is associated with reduced levels of factor 5 and 8

In haemophilia A the APTT is prolonged and there is reduced levels of factor 8:C. The bleeding time
and PT are normal. Cholestatic jaundice prevents the absorption of the fat soluble vitamin K.
Massive transfusion (>10u blood or equivalent to the blood volume of a person) puts the patient at
risk of thrombocytopaenia, factor 5 and 8 deficiency.
Please rate this question:

Discuss and give feedback


Next question

Abnormal coagulation

Cause Factors affected

Heparin Prevents activation factors 2,9,10,11

Warfarin Affects synthesis of factors 2,7,9,10

DIC Factors 1,2,5,8,11

Liver disease Factors 1,2,5,7,9,10,11

Interpretation blood clotting test results


Disorder APTT PT Bleeding time

Haemophilia Increased Normal Normal

von Willebrand's disease Increased Normal Increased

Vitamin K deficiency Increased Increased Normal

Next question
A 34 year old man presents with a peptic ulcer. Which of the following is responsible for the release
of gastric acid?

Chief cells

Parietal cells

Brunners Glands

G Cells

None of the above

Parietal cells are responsible for the release of gastric acid. Brunners glands are found in the
duodenum.

Please rate this question:

Discuss and give feedback

Next question

Gastric secretions

A working knowledge of gastric secretions is important for surgery because peptic ulcers are
common, surgeons frequently prescribe anti secretory drugs and because there are still patients
around who will have undergone acid lowering procedures (Vagotomy) in the past.

Gastric acid

 Is produced by the parietal cells in the stomach


 pH of gastric acid is around 2 with acidity being maintained by the H +/K+ ATP ase pump. As
part of the process bicarbonate ions will be secreted into the surrounding vessels.
 Sodium and chloride ions are actively secreted from the parietal cell into the canaliculus.
This sets up a negative potential across the membrane and as a result sodium and
potassium ions diffuse across into the canaliculus.
 Carbonic anhydrase forms carbonic acid which dissociates and the hydrogen ions formed by
dissociation leave the cell via the H+/K+ antiporter pump. At the same time sodium ions are
actively absorbed. This leaves hydrogen and chloride ions in the canaliculus these mix and
are secreted into the lumen of the oxyntic gland.

This is illustrated diagrammatically below:

Image sourced from Wikipedia

Phases of gastric acid secretion


There are 3 phases of gastric secretion:

1. Cephalic phase (smell / taste of food)


 30% acid produced
 Vagal cholinergic stimulation causing secretion of HCL and gastrin release from G cells

2. Gastric phase (distension of stomach )

 60% acid produced


 Stomach distension/low H+/peptides causes Gastrin release

3. Intestinal phase (food in duodenum)

 10% acid produced


 High acidity/distension/hypertonic solutions in the duodenum inhibits gastric acid secretion
via enterogastrones (CCK, secretin) and neural reflexes.

Regulation of gastric acid production


Factors increasing production include:

 Vagal nerve stimulation


 Gastrin release
 Histamine release (indirectly following gastrin release) from enterchromaffin like cells

Factors decreasing production include:

 Somatostatin (inhibits histamine release)


 Cholecystokinin
 Secretin

The diagram below illustrates some of the factors involved in regulating gastric acid secretion and
the relevant associated pharmacology
Image sourced from Wikipedia

Below is a brief summary of the major hormones involved in food digestion:

Source Stimulus Actions

Gastrin G cells in Distension of Increase HCL, pepsinogen and IF secretion,


antrum of the stomach, extrinsic increases gastric motility, trophic effect on gastric
stomach nerves mucosa
Inhibited by: low
antral pH,
somatostatin

CCK I cells in upper Partially digested Increases secretion of enzyme-rich fluid from
small intestine proteins and pancreas, contraction of gallbladder and
triglycerides relaxation of sphincter of Oddi, decreases gastric
emptying, trophic effect on pancreatic acinar
cells, induces satiety

Secretin S cells in Acidic chyme, fatty Increases secretion of bicarbonate-rich fluid from
upper small acids pancreas and hepatic duct cells, decreases gastric
acid secretion, trophic effect on pancreatic acinar
intestine cells

VIP Small Neural Stimulates secretion by pancreas and intestines,


intestine, inhibits acid and pepsinogen secretion
pancreas

Somatostatin D cells in the Fat, bile salts and Decreases acid and pepsin secretion, decreases
pancreas and glucose in the gastrin secretion, decreases pancreatic enzyme
stomach intestinal lumen secretion, decreases insulin and glucagon
secretion
inhibits trophic effects of gastrin, stimulates
gastric mucous production

Next question
Which of the following does not lead to relaxation of the lower oesophageal sphincter?

Metoclopramide

Botulinum toxin type A

Nicotine

Alcohol

Theophylline

Metoclopramide acts directly on the smooth muscle of the LOS to cause it to contract.
Theophylline is a phosphodiesterase inhibitor (mimics action of prostaglandin E1) which causes
relaxation of the LOS.

Please rate this question:

Discuss and give feedback

Next question

Peristalsis

 Circular smooth muscle contracts behind the food bolus and longitudinal smooth muscle
propels the food through the oesophagus
 Primary peristalsis spontaneously moves the food from the oesophagus into the stomach (9
seconds)
 Secondary peristalsis occurs when food, which doesn't enter the stomach, stimulates stretch
receptors to cause peristalsis
 In the small intestine each peristalsis waves slows to a few seconds and causes mixture of
chyme
 In the colon three main types of peristaltic activity are recognised (see below)

Colonic peristalsis
Segmentation contractions Localised contractions in which the bolus is subjected to local forces to
maximise mucosal absorption

Antiperistaltic contractions Localised reverse peristaltic waves to slow entry into colon and
towards ileum maximise absorption

Mass movements Waves migratory peristaltic waves along the entire colon to empty the
organ prior to the next ingestion of food bolus

Next question
Which of the following is not released from the islets of Langerhans?

Pancreatic polypeptide

Glucagon

Secretin

Somatostatin

Insulin

Secretin is released from mucosal cells in the duodenum and jejunum.


Please rate this question:

Discuss and give feedback


Next question

Pancreas endocrine physiology

Hormones released from the islets of Langerhans


Beta cells Insulin (70% of total secretions)

Alpha cells Glucagon

Delta cells Somatostatin

F cells Pancreatic polypeptide

Next question
Which of the following is not classically seen in coning resulting from raised intra cranial pressure?

Coma

Hypotension

Unreactive mid sized pupils

Cheyne Stokes style respiratory efforts

Bradycardia

Cushings triad

 Widening of the pulse pressure


 Respiratory changes
 Bradycardia

Due to raised ICP systemic hypertension is usually seen. Compression of the respiratory centre will
typically result in Cheyne Stokes style respiration.
Please rate this question:

Discuss and give feedback

Next question

Coning

 The cranial vault is a confined cavity apart from infants with a non fused fontanelle.
 Rises in ICP may be accommodated by shifts of CSF.
 Once the CSF shifting has reached its capacity ICP will start to rise briskly.
 The brain autoregulates its blood supply, as ICP rises the systemic circulation will display
changes to try and meet the perfusion needs of the brain. Usually this will involve
hypertension.
 As ICP rises further, the brain will be compressed, cranial nerve palsies may be seen and
compression of essential centres in the brain stem will occur. When the cardiac centre is
involved bradycardia will often develop.

Next question
Control of ventilation. Which statement is false?

Peripheral chemoreceptors are located in the bifurcation of the carotid arteries and arch of the
aorta

Central chemoreceptors respond to changes in O2

The respiratory centres control the rate and depth of respiration

Involuntary control of respiration is from the medulla and pons

Irritant receptors cause bronchospasm

- Central chemoreceptors: Respond to increased H+ in BRAIN INTERSTITIAL FLUID to increase


ventilation.

Please rate this question:

Discuss and give feedback

Next question

Control of ventilation

 Control of ventilation is coordinated by the respiratory centres, chemoreceptors, lung


receptors and muscles.
 Automatic, involuntary control of respiration occurs from the medulla.
 The respiratory centres control the respiratory rate and the depth of respiration.

Respiratory centres

Medullary Inspiratory and expiratory neurones. Has ventral group which controls forced
respiratory centre voluntary expiration and the dorsal group controls inspiration. Depressed by
opiates.

Apneustic centre Lower pons


Stimulates inspiration - activates and prolongs inhalation
Overridden by pneumotaxic control to end inspiration

Pneumotaxic centre Upper pons, inhibits inspiration at a certain point. Fine tunes the respiratory rate.

Ventillatory variables

 Levels of pCO2 most important in ventilation control


 Levels of O2 are less important.
 Peripheral chemoreceptors: located in the bifurcation of carotid arteries and arch of the
aorta. They respond to changes in reduced pO2, increased H+ and increased pCO2 in
ARTERIAL BLOOD.
 Central chemoreceptors: located in the medulla. Respond to increased H + in BRAIN
INTERSTITIAL FLUID to increase ventilation. NB the central receptors are NOT influenced
by O2levels.

Lung receptors include:

 Stretch receptors: respond to lung stretching causing a reduced respiratory rate


 Irritant receptors: respond to smoke etc causing bronchospasm
 J (juxtacapillary) receptors

Next question
A 54 year old lady has her serum calcium measured. Assuming her renal function is normal, what
proportion of calcium filtered at the glomerulus will be reabsorbed by the renal tubules?

5%

15%

25%

50%

95%

Most filtered calcium is reabsorbed (95%) a rare disorder of familial hypercalcemic calciurea may
affect this proportion.

Please rate this question:

Discuss and give feedback

Next question

Calcium homeostasis

Calcium ions are linked to a wide range of physiological processes. The largest store of bodily
calcium is contained within the skeleton. Calcium levels are primarily controlled by parathyroid
hormone, vitamin D and calcitonin.

Hormonal regulation of calcium

Hormone Actions
Hormone Actions

Parathyroid hormone (PTH)  Increase calcium levels and decrease phosphate levels
 Increases bone resorption
 Immediate action on osteoblasts to increase ca2+ in
extracellular fluid
 Osteoblasts produce a protein signaling molecule that
activate osteoclasts which cause bone resorption
 Increases renal tubular reabsorption of calcium
 Increases synthesis of 1,25(OH)2D (active form of
vitamin D) in the kidney which increases bowel
absorption of Ca2+
 Decreases renal phosphate reabsorption

1,25-dihydroxycholecalciferol (the  Increases plasma calcium and plasma phosphate


active form of vitamin D)  Increases renal tubular reabsorption and gut absorption
of calcium
 Increases osteoclastic activity
 Increases renal phosphate reabsorption

Calcitonin  Secreted by C cells of thyroid


 Inhibits intestinal calcium absorption
 Inhibits osteoclast activity
 Inhibits renal tubular absorption of calcium

Both growth hormone and thyroxine also play a small role in calcium metabolism.

Next question
Which of the following does not cause hyperkalaemia?

Haemolysis

Burns

Familial periodic paralysis

Type 4 renal tubular acidosis

Severe malnutrition

'Machine' - Causes of Increased Serum K+

M - Medications - ACE inhibitors, NSAIDS


A - Acidosis - Metabolic and respiratory
C - Cellular destruction - Burns, traumatic injury
H - Hypoaldosteronism, haemolysis
I - Intake - Excessive
N - Nephrons, renal failure
E - Excretion - Impaired

Familial periodic paralysis has subtypes associated with hyper and hypokalaemia.

Please rate this question:

Discuss and give feedback

Next question

Hyperkalaemia

 Plasma potassium levels are regulated by a number of factors including aldosterone, acid-
base balance and insulin levels.
 Metabolic acidosis is associated with hyperkalaemia as hydrogen and potassium ions
compete with each other for exchange with sodium ions across cell membranes and in the
distal tubule.
 ECG changes seen in hyperkalaemia include tall-tented T waves, small P waves, widened
QRS leading to a sinusoidal pattern and asystole

Causes of hyperkalaemia

 Acute renal failure


 Drugs*: potassium sparing diuretics, ACE inhibitors, angiotensin 2 receptor blockers,
spironolactone, ciclosporin, heparin**
 Metabolic acidosis
 Addison's
 Tissue necrosis/rhabdomylosis: burns, trauma
 Massive blood transfusion

Foods that are high in potassium

 Salt substitutes (i.e. Contain potassium rather than sodium)


 Bananas, oranges, kiwi fruit, avocado, spinach, tomatoes

*beta-blockers interfere with potassium transport into cells and can potentially cause hyperkalaemia
in renal failure patients - remember beta-agonists, e.g. Salbutamol, are sometimes used as
emergency treatment

**both unfractionated and low-molecular weight heparin can cause hyperkalaemia. This is thought to
be caused by inhibition of aldosterone secretion
Next question
Which of the following statements is true of glucagon?

Produced in response to hyperglycaemia

Released by beta cells

Inhibits gluconeogenesis

Produced in response to an increase of amino acids

Composed of 2 alpha polypeptide chains linked by hydrogen bonds

Glucagon is a protein comprised of a single polypeptide chain.


Produced by alpha cells of pancreatic islets of Langerhans in response to hypoglycaemia and amino
acids.
It increases plasma glucose and ketones.
Please rate this question:

Discuss and give feedback


Next question

Glucagon

Glucagon, the hormonal antagonist to insulin, is released from the alpha cells of the Islets of
Langerhans in the pancreas. It will result in an increased plasma glucose level.

Stimulation Inhibition

Decreased plasma glucose Somatostatin

Increased catecholamines Insulin

Increased free fatty acids and keto acids


Increased plasma amino acids
Sympathetic nervous system Increased urea

Acetylcholine

Cholecystokinin

Next question
A 28 year old man undergoes a completion right hemicolectomy for treatment of a 5cm appendiceal
carcinoid. As part of his follow up he is due to undergo 24 hour urine collection for 5-HIAA. Which of
the following causes an elevated 5-HIAA in a 24-hour urine collection?

Naproxen

Oranges

Flucloxacillin

Amiodarone

Beef

It is important to be aware of what can falsely elevate 5-HIAA to avoid diagnosing carcinoid
syndrome incorrectly. These include:

Food: spinach, cheese, wine, caffeine, tomatoes


Drugs: Naproxen, Monoamine oxidase inhibitors
Recent surgery

Please rate this question:

Discuss and give feedback

Next question

Carcinoid syndrome

 Carcinoid tumours secrete serotonin


 Originate in neuroendocrine cells mainly in the intestine (midgut-distal ileum/appendix)
 Can occur in the rectum, bronchi
 Hormonal symptoms mainly occur when disease spreads outside the bowel
Clinical features

 Onset: insidious over many years


 Flushing face
 Palpitations
 Pulmonary valve stenosis and tricuspid regurgitation causing dyspnoea
 Asthma
 Severe diarrhoea (secretory, persists despite fasting)

Investigation

 5-HIAA in a 24-hour urine collection


 Somatostatin receptor scintigraphy
 CT scan
 Blood testing for chromogranin A

Treatment

 Octreotide
 Surgical removal

Next question
A 52 year old man develops septic shock following a Hartmans procedure for perforated diverticular
disease. He is started on an adrenaline infusion. Which of the following is least likely to occur?

Peripheral vasoconstriction

Coronary artery vasospasm

Gluconeogenesis

Lipolysis

Tachycardia

It's cardiac effects are mediated via β 1 receptors. The coronary arteries which have β 2 receptors
are unaffected.
Please rate this question:

Discuss and give feedback


Next question

Adrenaline

 Fight or Flight response

- Catecholamine (phenylalanine and tyrosine)


- Neurotransmitter and hormone
- Released by the adrenal glands
- Effects on α 1 and 2, β 1 and 2 receptors
- Effect on β 2 receptors in skeletal muscle vessels-causing vasodilation
- Increase cardiac output and total peripheral resistance
- Vasoconstriction in the skin and kidneys causing a narrow pulse pressure

Actions
α adrenergic receptors:

 Inhibits insulin secretion by the pancreas


 Stimulates glycogenolysis in the liver and muscle
 Stimulates glycolysis in muscle
β adrenergic receptors:

 Stimulates glucagon secretion in the pancreas


 Stimulates ACTH
 Stimulates lipolysis by adipose tissue

Next question
Intra cranial pressure is governed by the principles of the Monroe-Kellie doctrine. To which of the
following does this concept not apply?

A 2 month old child

A 2 year old child

A 5 year old child

A 10 year old child

An adult

The Monroe-Kelly Doctrine assumes that the cranial cavity is a rigid box. In children with non fused
fontanells this is not the case.
Please rate this question:

Discuss and give feedback


Next question

Applied neurophysiology

 Pressure within the cranium is governed by the Monroe-Kelly doctrine. This considers the
skull as a closed box. Increases in mass can be accommodated by loss of CSF. Once a
critical point is reached (usually 100- 120ml of CSF lost) there can be no further
compensation and ICP rises sharply. The next step is that pressure will begin to equate with
MAP and neuronal death will occur. Herniation will also accompany this process.
 The CNS can autoregulate its own blood supply. Vaso constriction and dilatation of the
cerebral blood vessels is the primary method by which this occurs. Extremes of blood
pressure can exceed this capacity resulting in risk of stroke. Other metabolic factors such as
hypercapnia will also cause vasodilation, which is of importance in ventilating head injured
patients.
 The brain can only metabolise glucose, when glucose levels fall, consciousness will be
impaired.

Next question
Which of the following is not caused by cortisol in the stress response?

Anti-inflammatory effects

Hypoglycaemia

Skeletal muscle protein breakdown

Stimulation of lipolysis

Mineralocorticoid effects

An 'anti insulin' effect occurs leading to hyperglycaemia.


Please rate this question:

Discuss and give feedback


Next question

Stress response: Endocrine and metabolic changes

 Surgery precipitates hormonal and metabolic changes causing the stress response.
 Stress response is associated with: substrate mobilization, muscle protein loss, sodium and
water retention, suppression of anabolic hormone secretion, activation of the sympathetic
nervous system, immunological and haematological changes.
 The hypothalamic-pituitary axis and the sympathetic nervous systems are activated and
there is a failure of the normal feedback mechanisms of control of hormone secretion.

A summary of the hormonal changes associated with the stress response:

Increased Decreased No Change

Growth hormone Insulin Thyroid stimulating hormone

Cortisol Testosterone Luteinizing hormone


Increased Decreased No Change

Renin Oestrogen Follicle stimulating hormone

Adrenocorticotrophic hormone (ACTH)

Aldosterone

Prolactin

Antidiuretic hormone

Glucagon

Sympathetic nervous system

 Stimulates catecholamine release


 Causes tachycardia and hypertension

Pituitary gland

 ACTH and growth hormone (GH) is stimulated by hypothalamic releasing factors,


corticotrophin releasing factor (CRF) and somatotrophin (or growth hormone releasing factor)
 Perioperative increased prolactin secretion occurs by release of inhibitory control
 Secretion of thyroid stimulating hormone (TSH), luteinizing hormone (LH) and follicle
stimulating hormone (FSH) does not change significantly
 ACTH stimulates cortisol production within a few minutes of the start of surgery. More ACTH
is produced than needed to produce a maximum adrenocortical response.

Cortisol

 Significant increases within 4-6 hours of surgery (>1000 nmol litre-1).


 The usual negative feedback mechanism fails and concentrations of ACTH and cortisol
remain persistently increased.
 The magnitude and duration of the increase correlate with the severity of stress and the
response is not abolished by the administration of corticosteroids.
 The metabolic effects of cortisol are enhanced:

Skeletal muscle protein breakdown to provide gluconeogenic precursors and amino acids for protein
synthesis in the liver
Stimulation of lipolysis
'Anti-insulin effect'
Mineralocorticoid effects
Anti-inflammatory effects

Growth hormone

 Increased secretion after surgery has a minor role


 Most important for preventing muscle protein breakdown and promote tissue repair by insulin
growth factors

Alpha Endorphin

 Increased

Antidiuretic hormone

 An important vasopressor and enhances haemostasis


 Renin is released causing the conversion of angiotensinogen to angiotensin I
 Angiotensin II formed by ACE on angiotensin 1, which causes the secretion of aldosterone
from the adrenal cortex. This increases sodium reabsorption at the distal convoluted tubule

Insulin

 Release inhibited by stress


 Occurs via the inhibition of the beta cells in the pancreas by the α2-adrenergic inhibitory
effects of catecholamines
 Insulin resistance by target cells occurs later
 The perioperative period is characterized by a state of functional insulin deficiency

Thyroxine (T4) and tri-iodothyronine (T3)


 Circulating concentrations are inversely correlated with sympathetic activity and after surgery
there is a reduction in thyroid hormone production, which normalises over a few days.

Metabolic effect of endocrine response

Carbohydrate metabolism

 Hyperglycaemia is a main feature of the metabolic response to surgery


 Due to increase in glucose production and a reduction in glucose utilization
 Catecholamines and cortisol promote glycogenolysis and gluconeogenesis
 Initial failure of insulin secretion followed by insulin resistance affects the normal responses
 The proportion of the hyperglycaemic response reflects the severity of surgery
 Hyperglycaemia impairs wound healing and increase infection rates

Protein metabolism

 Initially there is inhibition of protein anabolism, followed later, if the stress response is
severe, by enhanced catabolism
 The amount of protein degradation is influenced by the type of surgery and also by the
nutritional status of the patient
 Mainly skeletal muscle protein is affected
 The amino acids released form acute phase proteins (fibrinogen, C reactive protein,
complement proteins, a2-macroglobulin, amyloid A and ceruloplasmin) and are used for
gluconeogenesis
 Nutritional support has little effect on preventing catabolism

Lipid metabolism
Increased catecholamine, cortisol and glucagon secretion, and insulin deficiency, promotes lipolysis
and ketone body production.

Salt and water metabolism

 ADH causes water retention, concentrated urine, and potassium loss and may continue for 3
to 5 days after surgery
 Renin causes sodium and water retention

Cytokines

 Glycoproteins
 Interleukins (IL) 1 to 17, interferons, and tumour necrosis factor
 Synthesized by activated macrophages, fibroblasts, endothelial and glial cells in response to
tissue injury from surgery or trauma
 IL-6 main cytokine associated with surgery. Peak 12 to 24 h after surgery and increase by
the degree of tissue damage Other effects of cytokines include fever, granulocytosis,
haemostasis, tissue damage limitation and promotion of healing.

Modifying the response

 Opioids suppress hypothalamic and pituitary hormone secretion


 At high doses the hormonal response to pelvic and abdominal surgery is abolished.
However, such doses prolong recovery and increase the need for postoperative ventilatory
support
 Spinal anaesthesia can reduce the glucose, ACTH, cortisol, GH and epinephrine changes,
although cytokine responses are unaltered
 Cytokine release is reduced in less invasive surgery
 Nutrition prevents the adverse effects of the stress response. Enteral feeding improves
recovery
 Growth hormone and anabolic steroids may improve outcome
 Normothermia decreases the metabolic response

References
Deborah Burton, Grainne Nicholson, and George Hall
Endocrine and metabolic response to surgery .

Contin Educ Anaesth Crit Care Pain (2004) 4(5): 144-147 doi:10.1093/bjaceaccp/mkh040
Next question
Which of the following features does not put a patient at risk of refeeding syndrome?

BMI < 16 kg/m2

Alcohol abuse

Thyrotoxicosis

Chemotherapy

Diuretics

Diuretics increase the risk of re-feeding syndrome through a process of increasing the risk of
depletion of key electrolytes.
Please rate this question:

Discuss and give feedback


Next question

Nutrition - Refeeding syndrome

Refeeding syndrome describes the metabolic abnormalities which occur on feeding a person
following a period of starvation. The metabolic consequences include:

 Hypophosphataemia
 Hypokalaemia
 Hypomagnesaemia
 Abnormal fluid balance

These abnormalities can lead to organ failure.

Re-feeding problems
If patient not eaten for > 5 days, aim to re-feed at < 50% energy and protein levels

High risk for re-feeding problems


If one or more of the following:

 BMI < 16 kg/m 2


 Unintentional weight loss >15% over 3-6 months
 Little nutritional intake > 10 days
 Hypokalaemia, Hypophosphataemia or hypomagnesaemia prior to feeding (unless high)

If two or more of the following:

 BMI < 18.5 kg/m2


 Unintentional weight loss > 10% over 3-6 months
 Little nutritional intake > 5 days
 History of: alcohol abuse, drug therapy including insulin, chemotherapy, diuretics and
antacids

Prescription

 Start at up to 10 kcal/kg/day increasing to full needs over 4-7 days


 Start immediately before and during feeding: oral thiamine 200-300mg/day, vitamin B co
strong 1 tds and supplements
 Give K+ (2-4 mmol/kg/day), phosphate (0.3-0.6 mmol/kg/day), magnesium (0.2-0.4
mmol/kg/day)

Next question
Which of the following statements relating to the regulation of renal blood flow is untrue?

In a healthy 70Kg male, the glomerular filtration rate will be the same at a systolic blood
pressure of 120mmHg as a systolic blood pressure of 95 mmHg

Over 90% of the blood supply to the kidney is distributed to the cortex

The kidney receives approximately 25% of the total cardiac output at rest

A decrease in renal perfusion pressure will cause the juxtaglomerular cells to secrete renin

Systolic blood pressures of less than 65mmHg will cause the mesangial cells to secrete
aldosterone

The kidney autoregulates its blood supply over a range of systolic blood pressures. Drop in arterial
pressure is sensed by the juxtaglomerular cells and renin is released leading to the activation of the
renin-angiontensin system. Mesangial cells are contractile cells that are located in the tubule and
have no direct endocrine function.
Please rate this question:

Discuss and give feedback


Next question

Renal Physiology

Overview

 Each nephron is supplied with blood from an afferent arteriole that opens onto the glomerular
capillary bed.
 Blood then flows to an efferent arteriole, supplying the peritubular capillaries and medullary
vasa recta.
 The kidney receives up to 25% of resting cardiac output.

Control of blood flow

 The kidney is able to autoregulate its blood flow between systolic pressures of 80- 180mmHg
so there is little variation in renal blood flow.
 This is achieved by myogenic control of arteriolar tone, both sympathetic input and hormonal
signals (e.g. renin) are responsible.
Glomerular structure and function

 Blood inside the glomerulus has considerable hydrostatic pressure.


 The basement membrane has pores that will allow free diffusion of smaller solutes, larger
negatively charged molecules such as albumin are unable to cross.
 The glomerular filtration rate (GFR) is equal to the concentration of a solute in the urine,
times the volume of urine produced per minute, divided by the plasma concentration
(assuming that the solute is freely diffused e.g. inulin).
 In clinical practice creatinine is used because it is subjected to very little proximal tubular
secretion.
 Although subject to variability, the typical GFR is 125ml per minute.
 Glomerular filtration rate = Total volume of plasma per unit time leaving the capillaries and
entering the bowman's capsule
 Renal clearance = volume plasma from which a substance is removed per minute by the
kidneys

Substances used to measure GFR have the following features:


1. Inert
2. Free filtration from the plasma at the glomerulus (not protein bound)
3. Not absorbed or secreted at the tubules
4. Plasma concentration constant during urine collection

Examples: inulin, creatinine

GFR = urine concentration (mmol/l) x urine volume (ml/min)


--------------------------------------------------------------------------
plasma concentration (mmol/l)

 The clearance of a substance is dependent not only on its diffusivity across the basement
membrane but also subsequent tubular secretion and / or reabsorption.
 So glucose which is freely filtered across the basement membrane is usually reabsorbed
from tubules giving a clearance of zero.

Tubular function

 Reabsorption and secretion of substances occurs in the tubules.


 In the proximal tubule substrates such as glucose, amino acids and phosphate are co-
transported with sodium across the semi permeable membrane.
 Up to two thirds of filtered water is reabsorbed in the proximal tubules.
 This will lead to increase in urea concentration in the distal tubule allowing for its increased
diffusion.
 Substances to be secreted into the tubules are taken up from the peritubular blood by tubular
cells.
 Solutes such as paraaminohippuric acid are cleared with a single passage through the
kidneys and this is why it is used to measure renal plasma flow. Ions such as calcium and
phosphate will have a tubular reabsorption that is influenced by plasma PTH levels.
 Potassium may be both secreted and re-absorbed and is co-exchanged with sodium.

Loop of Henle

 Approximately 60 litres of water containing 9000mmol sodium enters the descending limb of
the loop of Henle in 24 hours.
 Loops from the juxtamedullary nephrons run deep into the medulla.
 The osmolarity of fluid changes and is greatest at the tip of the papilla.
 The thin ascending limb is impermeable to water, but highly permeable to sodium and
chloride ions.
 This loss means that at the beginning of the thick ascending limb the fluid is hypo osmotic
compared with adjacent interstitial fluid.
 In the thick ascending limb the reabsorption of sodium and chloride ions occurs by both
facilitated and passive diffusion pathways.
 The loops of Henle are co-located with vasa recta, these will have similar solute
compositions to the surrounding extracellular fluid so preventing the diffusion and
subsequent removal of this hypertonic fluid.
 The energy dependent reabsorption of sodium and chloride in the thick ascending limb helps
to maintain this osmotic gradient.

Next question
A 39 year old lady undergoes a laparoscopic cholecystectomy as a daycase. The operation is more
difficult than anticipated and the surgeon places a drain to the liver bed. In recovery 1.5 litres of
blood is seen to enter the drain. Which of the following substances is the first to be released in this
situation?

Angiotensinogen

Renin

Angiotensin I

Angiotensin II

Aldosterone

The decrease in blood pressure will be sensed by the juxtaglomerular cells in the kidney. This will
cause renin secretion.
Please rate this question:

Discuss and give feedback


Next question

Shock

 Shock occurs when there is insufficient tissue perfusion.


 The pathophysiology of shock is an important surgical topic and may be divided into the
following aetiological groups:
 Septic
 Haemorrhagic
 Neurogenic
 Cardiogenic
 Anaphylactic

Septic shock
Septic shock is a major problem and those patients with severe sepsis have a mortality rate in
excess of 40%. In those who are admitted to intensive care mortality ranges from 6% with no organ
failure to 65% in those with 4 organ failure.

Sepsis is defined as an infection that triggers a particular Systemic Inflammatory Response


Syndrome (SIRS). This is characterised by body temperature outside 36 oC - 38 o C, HR >90
beats/min, respiratory rate >20/min, WBC count >12,000/mm 3 or < 4,000/mm 3, altered mental state
or hyperglycaemia (in absence of diabetes).

Patients with infections and two or more elements of SIRS meet the diagnostic criteria for sepsis.
Those with organ failure have severe sepsis and those with refractory hypotension -septic shock.

During the septic process there is marked activation of the immune system with extensive cytokine
release. This may be coupled with or triggered by systemic circulation of bacterial toxins. These all
cause endothelial cell damage and neutrophil adhesion. The overall hallmarks are thus those
ofexcessive inflammation, coagulation and fibrinolytic suppression.

The surviving sepsis campaign (2012) highlights the following key areas for attention:

 Prompt administration of antibiotics to cover all likely pathogens coupled with a rigorous
search for the source of infection.
 Haemodynamic stabilisation. Many patients are hypovolaemic and require aggressive fluid
administration. Aim for CVP 8-12 cm H2O, MAP >65mmHg.
 Modulation of the septic response. This includes manoeuvres to counteract the changes and
includes measures such as tight glycaemic control. The routine use of steroids is not
advised.

In surgical patients, the main groups with septic shock include those with anastomotic leaks,
abscesses and extensive superficial infections such as necrotising fasciitis. When performing
surgery the aim should be to undertake the minimum necessary to restore physiology. These
patients do not fare well with prolonged surgery. Definitive surgery can be more safely undertaken
when physiology is restored and clotting in particular has been normalised.

Haemorrhagic shock
The average adult blood volume comprises 7% of body weight. Thus in the 70 Kg adult this will
equate to 5 litres. This changes in children (8-9% body weight) and is slightly lower in the elderly.

The table below outlines the 4 major classes of haemorrhagic shock and their associated
physiological sequelae:

Parameter Class I Class II Class III Class IV

Blood loss ml <750ml 750-1500ml 1500-2000ml >2000ml

Blood loss % <15% 15-30% 30-40% >40%

Pulse rate <100 >100 >120 >140

Blood pressure Normal Normal Decreased Decreased


Parameter Class I Class II Class III Class IV

Respiratory rate 14-20 20-30 30-40 >35

Urine output >30ml 20-30ml 5-15ml <5ml

Symptoms Normal Anxious Confused Lethargic

Decreasing blood pressure during haemorrhagic shock causes organ hypoperfusion and relative
myocardial ischaemia. The cardiac index gives a numerical value for tissue oxygen delivery and is
given by the equation: Cardiac index= Cardiac output/ body surface area. Where Hb is
haemoglobin concentration in blood and SaO2 the saturation and PaO2 the partial pressure of
oxygen. Detailed knowledge of this equation is required for the MRCS Viva but not for part A,
although you should understand the principle.

In patients suffering from trauma the most likely cause of shock is haemorrhage. However, the
following may also be the cause or occur concomitantly:

 Tension pneumothorax
 Spinal cord injury
 Myocardial contusion
 Cardiac tamponade

When assessing trauma patients it is worth remembering that in order to generate a palpable
femoral pulse an arterial pressure of >65mmHg is required.

Once bleeding is controlled and circulating volume normalised the levels of transfusion should be to
maintain a Hb of 7-8 in those with no risk factors for tissue hypoxia and Hb 10 for those who have
such risk factors.

Neurogenic shock
This occurs most often following a spinal cord transection, usually at a high level. There is
resultant interruption of the autonomic nervous system. The result is either decreased sympathetic
tone or increased parasympathetic tone, the effect of which is a decrease in peripheral vascular
resistance mediated by marked vasodilation.

This results in decreased preload and thus decreased cardiac output (Starlings law). There is
decreased peripheral tissue perfusion and shock is thus produced. In contrast with many other types
of shock peripheral vasoconstrictors are used to return vascular tone to normal.

Cardiogenic shock
In medical patients the main cause is ischaemic heart disease. In the traumatic setting direct
myocardial trauma or contusion is more likely. Evidence of ECG changes and overlying sternal
fractures or contusions should raise the suspicion of injury. Treatment is largely supportive and
transthoracic echocardiography should be used to determine evidence of pericardial fluid or direct
myocardial injury. The measurement of troponin levels in trauma patients may be undertaken but
they are less useful in delineating the extent of myocardial trauma than following MI.

When cardiac injury is of a blunt nature and is associated with cardiogenic shock the right side of the
heart is the most likely site of injury with chamber and or valve rupture. These patients require
surgery to repair these defects and will require cardiopulmonary bypass to achieve this. Some may
require intra aortic balloon pump as a bridge to surgery.

Anaphylactic shock
Anaphylaxis may be defined as a severe, life-threatening, generalised or systemic
hypersensitivity reaction.

Anaphylaxis is one of the few times when you would not have time to look up the dose of a
medication. The Resuscitation Council guidelines on anaphylaxis have recently been updated.
Adrenaline is by far the most important drug in anaphylaxis and should be given as soon as
possible. The recommended doses for adrenaline, hydrocortisone and chlorpheniramine are as
follows:

Adrenaline Hydrocortisone Chlorpheniramine

< 6 months 150 mcg (0.15ml 1 in 1,000) 25 mg 250 mcg/kg

6 months - 6 years 150 mcg (0.15ml 1 in 1,000) 50 mg 2.5 mg

6-12 years 300 mcg (0.3ml 1 in 1,000) 100 mg 5 mg

Adult and child 12 years 500 mcg (0.5ml 1 in 1,000) 200 mg 10 mg

Adrenaline can be repeated every 5 minutes if necessary. The best site for IM injection is the
anterolateral aspect of the middle third of the thigh.

Common identified causes of anaphylaxis

 food (e.g. Nuts) - the most common cause in children


 drugs
 venom (e.g. Wasp sting)

Next question
Which of the following drugs causes hyperkalaemia?

Heparin

Ciprofloxacin

Salbutamol

Levothyroxine

Codeine phosphate

Both unfractionated and low-molecular weight heparin can cause hyperkalaemia. This is thought to
be caused by inhibition of aldosterone secretion. Salbutamol is a recognised treatment for
hyperkalaemia.
Please rate this question:

Discuss and give feedback


Next question

Hyperkalaemia

 Plasma potassium levels are regulated by a number of factors including aldosterone, acid-
base balance and insulin levels.
 Metabolic acidosis is associated with hyperkalaemia as hydrogen and potassium ions
compete with each other for exchange with sodium ions across cell membranes and in the
distal tubule.
 ECG changes seen in hyperkalaemia include tall-tented T waves, small P waves, widened
QRS leading to a sinusoidal pattern and asystole

Causes of hyperkalaemia

 Acute renal failure


 Drugs*: potassium sparing diuretics, ACE inhibitors, angiotensin 2 receptor blockers,
spironolactone, ciclosporin, heparin**
 Metabolic acidosis
 Addison's
 Tissue necrosis/rhabdomylosis: burns, trauma
 Massive blood transfusion
Foods that are high in potassium

 Salt substitutes (i.e. Contain potassium rather than sodium)


 Bananas, oranges, kiwi fruit, avocado, spinach, tomatoes

*beta-blockers interfere with potassium transport into cells and can potentially cause hyperkalaemia
in renal failure patients - remember beta-agonists, e.g. Salbutamol, are sometimes used as
emergency treatment

**both unfractionated and low-molecular weight heparin can cause hyperkalaemia. This is thought to
be caused by inhibition of aldosterone secretion
Next question
A 25-year-old man who has been morbidly obese for the past five years is reviewed in the surgical
bariatric clinic. In this patient, release of which of the following hormones would increase appetite?

Leptin

Thyroxine

Adiponectin

Ghrelin

Serotonin

Obesity hormones

 leptin decreases appetite


 ghrelin increases appetite

Whilst thyroxine can increase appetite it does not fit with the clinical picture being described.
Please rate this question:

Discuss and give feedback


Next question

Obesity: physiology

Leptin
Leptin is thought to play a key role in the regulation of body weight. It is produced by adipose tissue
and acts on satiety centres in the hypothalamus and decreases appetite. More adipose tissue (e.g.
in obesity) results in high leptin levels.

Leptin stimulates the release of melanocyte-stimulating hormone (MSH) and corticotrophin-releasing


hormone (CRH). Low levels of leptin stimulates the release of neuropeptide Y (NPY)

Ghrelin
Where as leptin induces satiety, ghrelin stimulates hunger. It is produced mainly by the fundus of the
stomach and the pancreas. Ghrelin levels increase before meals and decrease after meals
Next question
Which of the following is not linked to excess glucocorticoids?

Osteonecrosis

Osteoporosis

Hypokalaemia

Hyponatraemia

Growth retardation in children

There are many adverse effects associated with excess glucocorticoids. Thinning of the skin,
osteonecrosis and osteoporosis are all common. Steroids are associated with retention of sodium
and water. Potassium loss may occur and hypokalaemic alkalosis has been reported.
Please rate this question:

Discuss and give feedback


Next question

Cortisol

 Glucocorticoid
 Released by zona fasiculata of the adrenal gland
 90% protein bound; 10% active
 Circadian rhythm: High in the mornings
 Negative feedback via ACTH

Actions

 Glycogenolysis
 Gluconeogenesis
 Protein catabolism
 Lipolysis
 Stress response
 Anti-inflammatory
 Decrease protein in bones
 Increase gastric acid
 Increases neutrophils/platelets/red blood cells
 Inhibits fibroblastic activity
A 54-year-old woman is admitted to the Surgical Admissions Unit with abdominal pain. Blood tests
taken on admission show the following:

Magnesium 0.40 mmol/l (normal value 0.7-1.0 mmol/l)

Which one of the following factors is most likely to be responsible for this result?

Excessive resuscitation with intravenous saline

Digoxin therapy

Diarrhoea

Hypothermia

Rhabdomyolysis

Please rate this question:

Discuss and give feedback


Next question

Hypomagnasaemia

Cause of low magnesium

 Diuretics
 Total parenteral nutrition
 Diarrhoea
 Alcohol
 Hypokalaemia, hypocalcaemia

Features

 Paraesthesia
 Tetany
 Seizures
 Arrhythmias
 Decreased PTH secretion → hypocalcaemia
 ECG features similar to those of hypokalaemia
 Exacerbates digoxin toxicity

Next question
A 43 year old man has a nasogastric tube inserted. The nurse takes a small aspirate of the fluid from
the stomach and tests the pH of the aspirate. What is the normal intragastric pH?

0.5

Theme from January 2012 Exam


The intragastric pH is usually 2. Administration of proton pump inhibitors can result in almost
complete abolition of acidity
Please rate this question:

Discuss and give feedback


Next question

Gastric secretions

A working knowledge of gastric secretions is important for surgery because peptic ulcers are
common, surgeons frequently prescribe anti secretory drugs and because there are still patients
around who will have undergone acid lowering procedures (Vagotomy) in the past.

Gastric acid

 Is produced by the parietal cells in the stomach


 pH of gastric acid is around 2 with acidity being maintained by the H +/K+ ATP ase pump. As
part of the process bicarbonate ions will be secreted into the surrounding vessels.
 Sodium and chloride ions are actively secreted from the parietal cell into the canaliculus.
This sets up a negative potential across the membrane and as a result sodium and
potassium ions diffuse across into the canaliculus.
 Carbonic anhydrase forms carbonic acid which dissociates and the hydrogen ions formed by
dissociation leave the cell via the H+/K+ antiporter pump. At the same time sodium ions are
actively absorbed. This leaves hydrogen and chloride ions in the canaliculus these mix and
are secreted into the lumen of the oxyntic gland.
This is illustrated diagrammatically below:

Image sourced from Wikipedia

Phases of gastric acid secretion


There are 3 phases of gastric secretion:

1. Cephalic phase (smell / taste of food)

 30% acid produced


 Vagal cholinergic stimulation causing secretion of HCL and gastrin release from G cells

2. Gastric phase (distension of stomach )

 60% acid produced


 Stomach distension/low H+/peptides causes Gastrin release
3. Intestinal phase (food in duodenum)

 10% acid produced


 High acidity/distension/hypertonic solutions in the duodenum inhibits gastric acid secretion
via enterogastrones (CCK, secretin) and neural reflexes.

Regulation of gastric acid production


Factors increasing production include:

 Vagal nerve stimulation


 Gastrin release
 Histamine release (indirectly following gastrin release) from enterchromaffin like cells

Factors decreasing production include:

 Somatostatin (inhibits histamine release)


 Cholecystokinin
 Secretin

The diagram below illustrates some of the factors involved in regulating gastric acid secretion and
the relevant associated pharmacology

Image sourced from Wikipedia

Below is a brief summary of the major hormones involved in food digestion:


Source Stimulus Actions

Gastrin G cells in Distension of Increase HCL, pepsinogen and IF secretion,


antrum of the stomach, extrinsic increases gastric motility, trophic effect on
stomach nerves gastric mucosa
Inhibited by: low
antral pH,
somatostatin

CCK I cells in Partially digested Increases secretion of enzyme-rich fluid


upper small proteins and from pancreas, contraction of gallbladder
intestine triglycerides and relaxation of sphincter of Oddi,
decreases gastric emptying, trophic effect on
pancreatic acinar cells, induces satiety

Secretin S cells in Acidic chyme, Increases secretion of bicarbonate-rich fluid


upper small fatty acids from pancreas and hepatic duct cells,
intestine decreases gastric acid secretion, trophic
effect on pancreatic acinar cells

VIP Small Neural Stimulates secretion by pancreas and


intestine, intestines, inhibits acid and pepsinogen
pancreas secretion

Somatostatin D cells in the Fat, bile salts and Decreases acid and pepsin secretion,
pancreas and glucose in the decreases gastrin secretion, decreases
stomach intestinal lumen pancreatic enzyme secretion, decreases
insulin and glucagon secretion
inhibits trophic effects of gastrin, stimulates
gastric mucous production

Next question
Which of the following is the equivalent of cardiac preload?

End diastolic volume

Stroke volume

Systemic vascular resistance

Mean arterial pressure

Peak systolic arterial pressure

Preload is the same as end diastolic volume. When it is increased slightly there is an associated
increase in cardiac output (Frank Starling principle). When it is markedly increased e.g. over 250ml
then cardiac output falls.
Please rate this question:

Discuss and give feedback


Next question

Cardiac physiology

 The heart has four chambers ejecting blood into both low pressure and high pressure
systems.
 The pumps generate pressures of between 0-25mmHg on the right side and 0-120 mmHg on
the left.
 At rest diastole comprises 2/3 of the cardiac cycle.
 The product of the frequency of heart rate and stroke volume combine to give the cardiac
output which is typically 5-6L per minute.

Detailed descriptions of the various waveforms are often not a feature of MRCS A (although they are
on the syllabus). However, they are a very popular topic for surgical physiology in the MRCS B
exam.

Electrical properties

 Intrinsic myogenic rhythm within cardiac myocytes means that even the denervated heart is
capable of contraction.
 In the normal situation the cardiac impulse is generated in the sino atrial node in the right
atrium and conveyed to the ventricles via the atrioventricular node.
 The sino atrial node is also capable of spontaneous discharge and in the absence of
background vagal tone will typically discharge around 100x per minute. Hence the higher
resting heart rate found in cardiac transplant cases. In the SA and AV nodes the resting
membrane potential is lower than in surrounding cardiac cells and will slowly depolarise from
-70mV to around -50mV at which point an action potential is generated.
 Differences in the depolarisation slopes between SA and AV nodes help to explain why the
SA node will depolarise first. The cells have a refractory period during which they cannot be
re-stimulated and this period allows for adequate ventricular filling. In pathological
tachycardic states this time period is overridden and inadequate ventricular filling may then
occur, cardiac output falls and syncope may ensue.

Parasympathetic fibres project to the heart via the vagus and will release acetylcholine. Sympathetic
fibres release nor adrenaline and circulating adrenaline comes from the adrenal medulla.
Noradrenaline binds to β 1 receptors in the SA node and increases the rate of pacemaker potential
depolarisation.

Cardiac cycle

Image sourced from Wikipedia

 Mid diastole: AV valves open. Ventricles hold 80% of final volume. Outflow valves shut.
Aortic pressure is high.

 Late diastole: Atria contract. Ventricles receive 20% to complete filling. Typical end diastolic
volume 130-160ml.
 Early systole: AV valves shut. Ventricular pressure rises. Isovolumetric ventricular
contraction. AV Valves bulge into atria (c-wave). Aortic and pulmonary pressure exceeded-
blood is ejected. Shortening of ventricles pulls atria downwards and drops intra atrial
pressure (x-descent).

 Late systole: Ventricular muscles relax and ventricular pressures drop. Although ventricular
pressure drops the aortic pressure remains constant owing to peripheral vascular resistance
and elastic property of the aorta. Brief period of retrograde flow that occurs in aortic recoil
shuts the aortic valve. Ventricles will contain 60ml end systolic volume. The average stroke
volume is 70ml (i.e. Volume ejected).

 Early diastole: All valves are closed. Isovolumetric ventricular relaxation occurs. Pressure
wave associated with closure of the aortic valve increases aortic pressure. The pressure dip
before this rise can be seen on arterial waveforms and is called the incisura. During systole
the atrial pressure increases such that it is now above zero (v- wave). Eventually atrial
pressure exceed ventricular pressure and AV valves open - atria empty passively into
ventricles and atrial pressure falls (y -descent )

The negative atrial pressures are of clinical importance as they can allow air embolization to occur if
the neck veins are exposed to air. This patient positioning is important in head and neck surgery to
avoid this occurrence if veins are inadvertently cut, or during CVP line insertion.

Mechanical properties

 Preload = end diastolic volume


 Afterload = aortic pressure

It is important to understand the principles of Laplace's law in surgery.

 It states that for hollow organs with a circular cross section, the total circumferential wall
tension depends upon the circumference of the wall, multiplied by the thickness of the wall
and on the wall tension.
 The total luminal pressure depends upon the cross sectional area of the lumen and the
transmural pressure. Transmural pressure is the internal pressure minus external pressure
and at equilibrium the total pressure must counterbalance each other.
 In terms of cardiac physiology the law explains that the rise in ventricular pressure that
occurs during the ejection phase is due to physical change in heart size. It also explains why
a dilated diseased heart will have impaired systolic function.
Starlings law

 Increase in end diastolic volume will produce larger stroke volume.


 This occurs up to a point beyond which cardiac fibres are excessively stretched and stroke
volume will fall once more. It is important for the regulation of cardiac output in cardiac
transplant patients who need to increase their cardiac output.

Baroreceptor reflexes

 Baroreceptors located in aortic arch and carotid sinus.


 Aortic baroreceptor impulses travel via the vagus and from the carotid via the
glossopharyngeal nerve.
 They are stimulated by arterial stretch.
 Even at normal blood pressures they are tonically active.
 Increase in baroreceptor discharge causes:

*Increased parasympathetic discharge to the SA node.


*Decreased sympathetic discharge to ventricular muscle causing decreased contractility and fall in
stroke volume.
*Decreased sympathetic discharge to venous system causing increased compliance.
*Decreased peripheral arterial vascular resistance

Atrial stretch receptors

 Located in atria at junction between pulmonary veins and vena cava.


 Stimulated by atrial stretch and are thus low pressure sensors.
 Increased blood volume will cause increased parasympathetic activity.
 Very rapid infusion of blood will result in increase in heart rate mediated via atrial receptors:
theBainbridge reflex.
 Decreases in receptor stimulation results in increased sympathetic activity this will decrease
renal blood flow-decreases GFR-decreases urinary sodium excretion-renin secretion by
juxtaglomerular apparatus-Increase in angiotensin II.
 Increased atrial stretch will also result in increased release of atrial natriuretic peptide.

Next question
A 48 year old man undergoes a right hemicolectomy for a large caecal polyp. In the immediate post
operative period which of the physiological processes described below is least likely to occur?

Glycogenolysis

Increased production of acute phase proteins

Increased cortisol production

Bronchoconstriction

Release of nitric oxide by vessels

Please rate this question:

Discuss and give feedback


Next question

Response to surgery

Sympathetic nervous system

 Noradrenaline from sympathetic nerves and adrenaline from adrenal medulla


 Blood diverted from skin and visceral organs; bronchodilatation, reduced intestinal motility,
increased glucagon and glycogenolysis, insulin reduced
 Heart rate and myocardial contractility are increased

Acute phase response

 TNF-α, IL-1, IL-2, IL-6, interferon and prostaglandins are released


 Excess cytokines may cause SIRS
 Cytokines increase the release of acute phase proteins

Endocrine response

 Hypothalamus, pituitary, adrenal axis


 Increases ACTH and cortisol production:

increases protein breakdown


increases blood glucose levels

 Aldosterone increases sodium re-absorption


 Vasopressin increases water re-absorption and causes vasoconstriction

Vascular endothelium

 Nitric oxide produces vasodilatation


 Platelet activating factor enhances the cytokine response
 Prostaglandins produce vasodilatation and induce platelet aggregation

Next question
 A 73 year old female is referred to the surgical clinic with an iron deficiency anaemia. As part
of the diagnostic work up the doctor requests a serum ferritin level. Which of the conditions
listed is most likely to lead to a falsely elevated result?

Locally perforated sigmoid colonic adenocarcinoma

Colonic angiodysplasia

Dieulafoy lesion of the stomach

Transitional cell carcinoma of the bladder

Endometrial adenocarcinoma


A locally perforated colonic tumour will typically cause an intense inflammatory response and
if peritonitis is not present clinically then at the very least a localised abscess. This
inflammatory process is the most likely (from the list) to falsely raise the serum ferritin level.
Angiodysplasia and dieulafoy lesions are mucosal arteriovenous malformations and unlikely
to result in considerable inflammatory activity.
 Please rate this question:


 Discuss and give feedback

 Next question

 Ferritin

 Ferritin is an intracellular protein that binds iron and stores it to be released in a controlled
fashion at sites where iron is required. Because iron and ferritin are bound the total body
ferritin levels may be decreased in cases of iron deficiency anaemia. Measurement of serum
ferritin levels can be useful in determining whether an apparently low haemoglobin and
microcytosis is truly caused by an iron deficiency state.
Ferritin is an acute phase protein and may be synthesised in increased quantities in
situations where inflammatory activity is ongoing. Falsely elevated results may therefore be
encountered clinically and need to be taken in context of the clinical picture and full blood
count results.
 Which of the following is least likely to cause a prolonged prothrombin time?

Cholestatic jaundice

Disseminated intravascular coagulation

Prolonged antibiotic treatment

Liver disease

Acquired factor 12 deficiency


Vitamin K deficiency results from cholestatic jaundice and prolonged antibiotic therapy.
Acquired factor 12 deficiency causes prolonged APTT.
 Please rate this question:


 Discuss and give feedback

 Next question

 Abnormal coagulation

Cause Factors affected

Heparin Prevents activation factors 2,9,10,11

Warfarin Affects synthesis of factors 2,7,9,10

DIC Factors 1,2,5,8,11


Cause Factors affected

Liver disease Factors 1,2,5,7,9,10,11


Interpretation blood clotting test results

Disorder APTT PT Bleeding time

Haemophilia Increased Normal Normal

von Willebrand's disease Increased Normal Increased

Vitamin K deficiency Increased Increased Normal

 Next question

Which statement about peristalsis is true?

Longitudinal smooth muscle propels the food bolus through the oesophagus

Secondary peristalsis occurs when there is no food bolus in the oesophagus

Food transfer from the oesophagus to the stomach is 4 seconds

Circular smooth muscle is not involved in peristalsis

Peristalsis only occurs in the oesophagus

Please rate this question:

Discuss and give feedback


Next question

Peristalsis

 Circular smooth muscle contracts behind the food bolus and longitudinal smooth muscle
propels the food through the oesophagus
 Primary peristalsis spontaneously moves the food from the oesophagus into the stomach (9
seconds)
 Secondary peristalsis occurs when food, which doesn't enter the stomach, stimulates stretch
receptors to cause peristalsis
 In the small intestine each peristalsis waves slows to a few seconds and causes mixture of
chyme
 In the colon three main types of peristaltic activity are recognised (see below)

Colonic peristalsis
Segmentation contractions Localised contractions in which the bolus is subjected to local
forces to maximise mucosal absorption

Antiperistaltic contractions Localised reverse peristaltic waves to slow entry into colon and
towards ileum maximise absorption
Mass movements Waves migratory peristaltic waves along the entire colon to
empty the organ prior to the next ingestion of food bolus

Next question
A 24 year old man is injured in a road traffic accident. He becomes oliguric and his renal function
deteriorates. Which of the options below would favor acute tubular necrosis over pre renal uraemia?

No response to intravenous fluids

Urinary sodium < 20mmol/L

Bland coloured urinary sediment

Increased urine specific gravity

None of the above

In acute tubular necrosis there is no response to intravenous fluids because the damage occurs from
within the renal system rather than as a result of volume depletion.

Please rate this question:

Discuss and give feedback

Next question

Acute renal failure: Pre renal failure vs. acute tubular necrosis

Prerenal uraemia - kidneys retain sodium to preserve volume

Pre-renal uraemia Acute tubular necrosis

Urine sodium < 20 mmol/L > 30 mmol/L


Pre-renal uraemia Acute tubular necrosis

Fractional sodium excretion* < 1% > 1%

Fractional urea excretion** < 35% >35%

Urine:plasma osmolality > 1.5 < 1.1

Urine:plasma urea > 10:1 < 8:1

Specific gravity > 1020 < 1010

Urine 'bland' sediment brown granular casts

Response to fluid challenge Yes No

*fractional sodium excretion = (urine sodium/plasma sodium) / (urine creatinine/plasma creatinine) x


100

**fractional urea excretion = (urine urea /blood urea ) / (urine creatinine/plasma creatinine) x 100

Next question
Which of the following is not an effect of cholecystokinin?

It causes gallbladder contraction

It increases the rate of gastric emptying

It relaxes the sphincter of oddi

It stimulates pancreatic acinar cells

It has a trophic effect on pancreatic acinar cells

It decreases the rate of gastric emptying.


Please rate this question:

Discuss and give feedback


Next question

Gastric secretions

A working knowledge of gastric secretions is important for surgery because peptic ulcers are
common, surgeons frequently prescribe anti secretory drugs and because there are still patients
around who will have undergone acid lowering procedures (Vagotomy) in the past.

Gastric acid

 Is produced by the parietal cells in the stomach


 pH of gastric acid is around 2 with acidity being maintained by the H +/K+ ATP ase pump. As
part of the process bicarbonate ions will be secreted into the surrounding vessels.
 Sodium and chloride ions are actively secreted from the parietal cell into the canaliculus.
This sets up a negative potential across the membrane and as a result sodium and
potassium ions diffuse across into the canaliculus.
 Carbonic anhydrase forms carbonic acid which dissociates and the hydrogen ions formed by
dissociation leave the cell via the H+/K+ antiporter pump. At the same time sodium ions are
actively absorbed. This leaves hydrogen and chloride ions in the canaliculus these mix and
are secreted into the lumen of the oxyntic gland.

This is illustrated diagrammatically below:


Image sourced from Wikipedia

Phases of gastric acid secretion


There are 3 phases of gastric secretion:

1. Cephalic phase (smell / taste of food)

 30% acid produced


 Vagal cholinergic stimulation causing secretion of HCL and gastrin release from G cells

2. Gastric phase (distension of stomach )

 60% acid produced


 Stomach distension/low H+/peptides causes Gastrin release

3. Intestinal phase (food in duodenum)


 10% acid produced
 High acidity/distension/hypertonic solutions in the duodenum inhibits gastric acid secretion
via enterogastrones (CCK, secretin) and neural reflexes.

Regulation of gastric acid production


Factors increasing production include:

 Vagal nerve stimulation


 Gastrin release
 Histamine release (indirectly following gastrin release) from enterchromaffin like cells

Factors decreasing production include:

 Somatostatin (inhibits histamine release)


 Cholecystokinin
 Secretin

The diagram below illustrates some of the factors involved in regulating gastric acid secretion and
the relevant associated pharmacology

Image sourced from Wikipedia

Below is a brief summary of the major hormones involved in food digestion:

Source Stimulus Actions


Gastrin G cells in Distension of Increase HCL, pepsinogen and IF secretion,
antrum of the stomach, extrinsic increases gastric motility, trophic effect on
stomach nerves gastric mucosa
Inhibited by: low
antral pH,
somatostatin

CCK I cells in Partially digested Increases secretion of enzyme-rich fluid


upper small proteins and from pancreas, contraction of gallbladder
intestine triglycerides and relaxation of sphincter of Oddi,
decreases gastric emptying, trophic effect on
pancreatic acinar cells, induces satiety

Secretin S cells in Acidic chyme, Increases secretion of bicarbonate-rich fluid


upper small fatty acids from pancreas and hepatic duct cells,
intestine decreases gastric acid secretion, trophic
effect on pancreatic acinar cells

VIP Small Neural Stimulates secretion by pancreas and


intestine, intestines, inhibits acid and pepsinogen
pancreas secretion

Somatostatin D cells in the Fat, bile salts and Decreases acid and pepsin secretion,
pancreas and glucose in the decreases gastrin secretion, decreases
stomach intestinal lumen pancreatic enzyme secretion, decreases
insulin and glucagon secretion
inhibits trophic effects of gastrin, stimulates
gastric mucous production

Next question
Which part of the jugular venous waveform is associated with the closure of the tricuspid valve?

a wave

c wave

x descent

y descent

v wave

JVP: C wave - closure of the tricuspid valve

The c wave of the jugular venous waveform is associated with the closure of the tricuspid valve.
Please rate this question:

Discuss and give feedback


Next question

Jugular venous pressure

As well as providing information on right atrial pressure, the jugular vein waveform may provide
clues to underlying valvular disease. A non-pulsatile JVP is seen in superior vena caval obstruction.
Kussmaul's sign describes a paradoxical rise in JVP during inspiration seen in constrictive
pericarditis

'a' wave = atrial contraction

 large if atrial pressure e.g. tricuspid stenosis, pulmonary stenosis, pulmonary hypertension
 absent if in atrial fibrillation

Cannon 'a' waves

 caused by atrial contractions against a closed tricuspid valve


 are seen in complete heart block, ventricular tachycardia/ectopics, nodal rhythm, single
chamber ventricular pacing
'c' wave

 closure of tricuspid valve


 not normally visible

'v' wave

 due to passive filling of blood into the atrium against a closed tricuspid valve
 giant v waves in tricuspid regurgitation

'x' descent = fall in atrial pressure during ventricular systole

'y' descent = opening of tricuspid valve

Image sourced from Wikipedia

Next question
Which one of the following serum proteins is most likely to increase in a patient with severe sepsis?

Transferrin

Transthyretin

Ferritin

Albumin

Cortisol binding protein

Ferritin can be markedly increased during acute illness. The other parameters tend to decrease
during an acute phase response.
Please rate this question:

Discuss and give feedback


Next question

Acute phase proteins

Acute phase proteins

 CRP
 procalcitonin
 ferritin
 fibrinogen
 alpha-1 antitrypsin
 caeruloplasmin
 serum amyloid A
 haptoglobin
 complement

During the acute phase response the liver decreases the production of other proteins (sometimes
referred to as negative acute phase proteins). Examples include:

 albumin
 transthyretin (formerly known as prealbumin)
 transferrin
 retinol binding protein
 cortisol binding protein

Levels of CRP are commonly measured in acutely unwell patients. CRP is a protein synthesised in
the liver and binds to phosphocholine in bacterial cells and on those cells undergoing apoptosis. In
binding to these cells it is then able to activate the complement system. CRP levels are known to
rise in patients following surgery. However, levels of greater than 150 at 48 hours post operatively
are suggestive of evolving complications.
Next question
Theme: Critical care

A. Hypovolaemia

B. Normal

C. Cardiogenic shock

D. Septic shock

For each of the scenarios outlined in the tables below, please select the most likely diagnosis from
the list. Each option may be used once, more than once or not at all.

72. A 45 year old man is admitted to the intensive care unit following a laparotomy. He has a central
line, pulmonary artery catheter and arterial lines inserted. The following results are obtained:

Pulmonary artery occlusion Systemic vascular


Cardiac output
pressure resistance

Low Low High

Hypovolaemia

Theme from April 2012 Exam


Cardiac output is lowered in hypovolaemia due to decreased preload.

73. A 75 year old man is admitted to the intensive care unit following a laparotomy. He has a central
line, pulmonary artery catheter and arterial lines inserted. The following results are obtained:

Pulmonary artery occlusion Systemic vascular


Cardiac output
pressure resistance

High Low High


You answered Hypovolaemia

The correct answer is Cardiogenic shock

In cardiogenic shock pulmonary pressures are often high. This is the basis for the use of
venodilators in the treatment of pulmonary oedema.

74. A 22 year old lady is admitted to the intensive care unit following a laparotomy. She has a central
line, pulmonary artery catheter and arterial lines inserted. The following results are obtained:

Pulmonary artery occlusion Systemic vascular


Cardiac output
pressure resistance

Low High Low

You answered Hypovolaemia

The correct answer is Septic shock

Decreased SVR is a major feature of sepsis. A hyperdynamic circulation is often present. This is the
reason for the use of vasoconstrictors.

Please rate this question:

Discuss and give feedback

Next question

Pulmonary artery occlusion pressure monitoring

The pulmonary artery occlusion pressure is an indirect measure of left atrial pressure, and thus filling
pressure of the left heart. The low resistance within the pulmonary venous system allows this useful
measurement to be made. The most accurate trace is made by inflating the balloon at the catheter
tip and "floating" it so that it occludes the vessel. If it is not possible to occlude the vessel in this way
then the measurement gained will be the pulmonary artery end diastolic pressure.

Interpretation of PAOP

PAOP mmHg Scenario

Normal 8-12

Low <5 Hypovolaemia

Low with pulmonary oedema <5 ARDS

High >18 Overload

When combined with measurements of systemic vascular resistance and cardiac output it is possible
to accurately classify patients.

Systemic vascular resistance


Derived from aortic pressure, right atrial pressure and cardiac output.

SVR=80(mean aortic pressure-mean right atrial pressure)/cardiac output

Next question
A 23 year old man has a routine ECG performed. Which part of the tracing obtained represents atrial
repolarisation?

P wave

T wave

Q-T Interval

P-R interval

None of the above

The process of atrial repolarisation is generally not visible on the ECG strip. It occurs during the
QRS complex.
Please rate this question:

Discuss and give feedback


Next question

The normal ECG


Image sourced from Wikipedia

P wave

 Represents the wave of depolarization that spreads from the SA node throughout the atria
 Lasts 0.08 to 0.1 seconds (80-100 ms)
 The isoelectric period after the P wave represents the time in which the impulse is traveling
within the AV node

P-R interval

 Time from the onset of the P wave to the beginning of the QRS complex
 Ranges from 0.12 to 0.20 seconds in duration
 Represents the time between the onset of atrial depolarization and the onset of ventricular
depolarization

QRS complex

 Represents ventricular depolarization


 Duration of the QRS complex is normally 0.06 to 0.1 seconds
ST segment

 Isoelectric period following the QRS


 Represents period which the entire ventricle is depolarized and roughly corresponds to the
plateau phase of the ventricular action potential

T wave

 Represents ventricular repolarization and is longer in duration than depolarization


 A small positive U wave may follow the T wave which represents the last remnants of
ventricular repolarization.

Q-T interval

 Represents the time for both ventricular depolarization and repolarization to occur, and
therefore roughly estimates the duration of an average ventricular action potential.
 Interval ranges from 0.2 to 0.4 seconds depending upon heart rate.
 At high heart rates, ventricular action potentials shorten in duration, which decreases the Q-T
interval. Therefore the Q-T interval is expressed as a "corrected Q-T (QTc)" by taking the Q-
T interval and dividing it by the square root of the R-R interval (interval between ventricular
depolarizations). This allows an assessment of the Q-T interval that is independent of heart
rate.
 Normal corrected Q-Tc interval is less than 0.44 seconds.

Next question
A 72-year-old woman is admitted to the acute surgical unit with profuse vomiting. Admission bloods
show the following:

Na+ 131 mmol/l

K+ 2.2 mmol/l

Urea 3.1 mmol/l

Creatinine 56 µmol/l

Glucose 4.3 mmol/l

Which one of the following ECG features is most likely to be seen?

Short PR interval

Short QT interval

Flattened P waves

J waves

U waves

Hypokalaemia - U waves on ECG

J waves are seen in hypothermia whilst delta waves are associated with Wolff Parkinson White
syndrome.
Please rate this question:

Discuss and give feedback


Next question

ECG features in hypokalemia

 U waves
 Small or absent T waves (occasionally inversion)
 Prolonged PR interval
 ST depression
 Long QT interval
One registered user suggests the following rhyme!

 In Hypokalaemia, U have no Pot and no T, but a long PR and a long QT!

Next question
The oxygen-haemoglobin dissociation curve is shifted to the left in:

With decreased 2,3-DPG in transfused red cells

Respiratory acidosis

High altitude

Pyrexia

Haemolytic anaemia

S shaped curve

The curve is shifted to the left when there is a decreased oxygen requirement by the tissue. This
includes:
1. Hypothermia
2. Alkalosis
3. Reduced levels of DPG:

 DPG is found in erythrocytes and is reduced in non exercising muscles, i.e. when there is
reduced glycolysis.

4. Polycythaemia
Please rate this question:

Discuss and give feedback


Next question

Oxygen Transport

Oxygen transport
Almost all oxygen is transported within erythrocytes. It has limited solubility and only 1% is carried as
solution. Therefore the amount of oxygen transported will depend upon haemoglobin concentration
and its degree of saturation.

Haemoglobin
Globular protein composed of 4 subunits. Haem consists of a protoporphyrin ring surrounding an
iron atom in its ferrous state. The iron can form two additional bonds; one with oxygen and the other
with a polypeptide chain. There are two alpha and two beta subunits to this polypeptide chain in an
adult and together these form globin. Globin cannot bind oxygen but is able to bind to carbon dioxide
and hydrogen ions, the beta chains are able to bind to 2,3 diphosphoglycerate. The oxygenation of
haemoglobin is a reversible reaction. The molecular shape of haemoglobin is such that binding of
one oxygen molecule facilitates the binding of subsequent molecules.

Oxygen dissociation curve

 The oxygen dissociation curve describes the relationship between the percentage of
saturated haemoglobin and partial pressure of oxygen in the blood. It is not affected by
haemoglobin concentration.
 Chronic anaemia causes 2, 3 DPG levels to increase, hence shifting the curve to the right

Haldane effect

 Shifts to left = for given oxygen tension there is increased saturation of Hb with oxygen i.e.
Decreased oxygen delivery to tissues

Bohr effect

 Shifts to right = for given oxygen tension there is reduced saturation of Hb with oxygen i.e.
Enhanced oxygen delivery to tissues
Image sourced from Wikipedia

Shifts to Left = Lower oxygen delivery Shifts to Right = Raised oxygen


delivery

 HbF, methaemoglobin, carboxyhaemoglobin


 low [H+] (alkali)  raised [H+] (acidic)
 low pCO2  raised pCO2
 low 2,3-DPG  raised 2,3-DPG*
 low temperature  raised temperature

*2,3-diphosphoglycerate
Next question
A homeless 42 year old male had an emergency inguinal hernia repair 24 hours previously. He has
a BMI of 15. He has been put on a feeding regime of 35 kcal/kg/day with no additional medications.
The nursing staff contact you as he has become confused and unsteady. On examination the patient
is disorientated to place, has diplopia and nystagmus. What is the most likely diagnosis?

Cerebellar stroke

Acute dystonic reaction

Cerebrovascular accident

Parkinsonism

Wernickes encephalopathy

Triad of Wernicke encephalopathy:

 Acute confusion
 Ataxia
 Opthalmoplegia

This patient has received a carbohydrate rich diet without any thiamine or vitamin B co strong
replacement. This has led to Wernickes encephalopathy, which classically presents with confusion,
ataxia and opthalmoplegia. Characteristically it is associated with chronic alcoholism, however it is
also known to occur post bariatric surgery.
Please rate this question:

Discuss and give feedback


Next question

Nutrition - Refeeding syndrome

Refeeding syndrome describes the metabolic abnormalities which occur on feeding a person
following a period of starvation. The metabolic consequences include:

 Hypophosphataemia
 Hypokalaemia
 Hypomagnesaemia
 Abnormal fluid balance
These abnormalities can lead to organ failure.

Re-feeding problems
If patient not eaten for > 5 days, aim to re-feed at < 50% energy and protein levels

High risk for re-feeding problems


If one or more of the following:

 BMI < 16 kg/m 2


 Unintentional weight loss >15% over 3-6 months
 Little nutritional intake > 10 days
 Hypokalaemia, Hypophosphataemia or hypomagnesaemia prior to feeding (unless high)

If two or more of the following:

 BMI < 18.5 kg/m2


 Unintentional weight loss > 10% over 3-6 months
 Little nutritional intake > 5 days
 History of: alcohol abuse, drug therapy including insulin, chemotherapy, diuretics and
antacids

Prescription

 Start at up to 10 kcal/kg/day increasing to full needs over 4-7 days


 Start immediately before and during feeding: oral thiamine 200-300mg/day, vitamin B co
strong 1 tds and supplements
 Give K+ (2-4 mmol/kg/day), phosphate (0.3-0.6 mmol/kg/day), magnesium (0.2-0.4
mmol/kg/day)

Next question
A 22 year old lady receives intravenous morphine for acute abdominal pain. Which of the following
best accounts for its analgesic properties?

Binding to δ opioid receptors in the brainstem

Binding to δ opioid receptors at peripheral nerve sites

Binding to β opioid receptors within the CNS

Binding to α opioid receptors within the CNS

Binding to µ opioid receptors within the CNS

4 Types of opioid receptor:

 δ (located in CNS)- Accounts for analgesic and antidepressant effects


 k (mainly CNS)- analgesic and dissociative effects
 µ (central and peripheral) - causes analgesia, miosis, decreased gut motility
 Nociceptin receptor (CNS)- Affect of appetite and tolerance to µ agonists.

Theme from April 2012 Exam


Please rate this question:

Discuss and give feedback


Next question

Morphine

Strong opiate analgesic. It is a pro- type narcotic drug and its effects mediated via the 4 types of
opioid receptor. Its clinical effects stem from binding to these receptor sites within the CNS and
gastrointestinal tract. Unwanted side effects include nausea, constipation, respiratory depression
and, if used long term, addiction .
It may be administered orally or intravenously. It can be reversed with naloxone.
Next question
Which one of the following reduces the secretion of renin?

Erect posture

Adrenaline

Hyponatraemia

Hypotension

Beta-blockers

Please rate this question:

Discuss and give feedback


Next question

Renin

Renin is secreted by juxtaglomerular cells and hydrolyses angiotensinogen to produce angiotensin I

Factors stimulating renin secretion

 Hypotension causing reduced renal perfusion


 Hyponatraemia
 Sympathetic nerve stimulation
 Catecholamines
 Erect posture

Factors reducing renin secretion

 Drugs: beta-blockers, NSAIDs

Next question
Which of the following stimulates prolactin release?

Leutinising hormone

Dopamine

Thyrotropin releasing hormone

Oestrogen

Follicle stimulating hormone

TRH stimulates prolactin release. Dopamine suppresses the release of prolactin.

Please rate this question:

Discuss and give feedback

Next question

Prolactin

Prolactin is a peptide hormone released from the anterior pituitary. It is under tonic dopamine
inhibition, thyrotropin releasing hormone has a stimulatory effect on release. Prolactin release
stimulates milk production but also reduces gonadal activity. It decreases GnRH pulsatility at the
hypothalamic level and to a lesser extent, blocks the action of LH on the ovary or testis.

Next question
Theme: Vitamin deficiency

A. Vitamin A

B. Vitamin B1

C. Vitamin B12

D. Vitamin B3

E. Vitamin C

F. Vitamin K

G. Vitamin D

Please select the vitamin deficiency most closely associated with the situation described. Each
option may be used once, more than once or not at all.

82. A 3 year old child presents with Rickets

You answered Vitamin A

The correct answer is Vitamin D

Vitamin D is needed to help mineralise bone. When this is deficient, mineralisation is inadequate
and deformities mayt result.

83. A 44 year old lady presents with jaundice. Following a minor ward based surgical procedure she
develops troublesome and persistent bleeding.

You answered Vitamin A

The correct answer is Vitamin K

Patients who are jaundiced usually have impaired absorption of vitamin K. This can result in loss
of the vitamin K dependent clotting factors and troublesome bleeding.
84. A 69 year old man who has been living in sheltered accommodation for many months, with
inadequate nutrition notices that his night vision is becoming impaired.

Vitamin A

Loss of vitamin A will result in impair rhodopsin synthesis and poor night vision.

Please rate this question:

Discuss and give feedback

Next question

Vitamin deficiency

Vitamin Effect of deficiency

A Night blindness
Epithelial atrophy
Infections

B1 Beriberi

B2 Dematitis and photosensitivity

B3 Pellagra

B12 Pernicious anaemia


C Poor wound healing
Impaired collagen synthesis

D Rickets (Children)
Osteomalacia (Adults)

K Clotting disorders

Next question
A 55 year old man undergoes a laparotomy and repair of incisional hernia. Which of the following
hormones is least likely to be released in increased quantities following the procedure?

Insulin

ACTH

Glucocorticoids

Aldosterone

Growth hormone

Insulin and thyroxine are often have reduced levels of secretion in the post operative period. This,
coupled with increased glucocorticoid release may cause difficulty in management of diabetes in
individuals with insulin resistance.
Please rate this question:

Discuss and give feedback


Next question

Stress response: Endocrine and metabolic changes

 Surgery precipitates hormonal and metabolic changes causing the stress response.
 Stress response is associated with: substrate mobilization, muscle protein loss, sodium and
water retention, suppression of anabolic hormone secretion, activation of the sympathetic
nervous system, immunological and haematological changes.
 The hypothalamic-pituitary axis and the sympathetic nervous systems are activated and
there is a failure of the normal feedback mechanisms of control of hormone secretion.

A summary of the hormonal changes associated with the stress response:

Increased Decreased No Change

Growth hormone Insulin Thyroid stimulating hormone


Increased Decreased No Change

Cortisol Testosterone Luteinizing hormone

Renin Oestrogen Follicle stimulating hormone

Adrenocorticotrophic hormone (ACTH)

Aldosterone

Prolactin

Antidiuretic hormone

Glucagon

Sympathetic nervous system

 Stimulates catecholamine release


 Causes tachycardia and hypertension

Pituitary gland

 ACTH and growth hormone (GH) is stimulated by hypothalamic releasing factors,


corticotrophin releasing factor (CRF) and somatotrophin (or growth hormone releasing factor)
 Perioperative increased prolactin secretion occurs by release of inhibitory control
 Secretion of thyroid stimulating hormone (TSH), luteinizing hormone (LH) and follicle
stimulating hormone (FSH) does not change significantly
 ACTH stimulates cortisol production within a few minutes of the start of surgery. More ACTH
is produced than needed to produce a maximum adrenocortical response.
Cortisol

 Significant increases within 4-6 hours of surgery (>1000 nmol litre-1).


 The usual negative feedback mechanism fails and concentrations of ACTH and cortisol
remain persistently increased.
 The magnitude and duration of the increase correlate with the severity of stress and the
response is not abolished by the administration of corticosteroids.
 The metabolic effects of cortisol are enhanced:

Skeletal muscle protein breakdown to provide gluconeogenic precursors and amino acids for protein
synthesis in the liver
Stimulation of lipolysis
'Anti-insulin effect'
Mineralocorticoid effects
Anti-inflammatory effects

Growth hormone

 Increased secretion after surgery has a minor role


 Most important for preventing muscle protein breakdown and promote tissue repair by insulin
growth factors

Alpha Endorphin

 Increased

Antidiuretic hormone

 An important vasopressor and enhances haemostasis


 Renin is released causing the conversion of angiotensinogen to angiotensin I
 Angiotensin II formed by ACE on angiotensin 1, which causes the secretion of aldosterone
from the adrenal cortex. This increases sodium reabsorption at the distal convoluted tubule

Insulin

 Release inhibited by stress


 Occurs via the inhibition of the beta cells in the pancreas by the α2-adrenergic inhibitory
effects of catecholamines
 Insulin resistance by target cells occurs later
 The perioperative period is characterized by a state of functional insulin deficiency
Thyroxine (T4) and tri-iodothyronine (T3)

 Circulating concentrations are inversely correlated with sympathetic activity and after surgery
there is a reduction in thyroid hormone production, which normalises over a few days.

Metabolic effect of endocrine response

Carbohydrate metabolism

 Hyperglycaemia is a main feature of the metabolic response to surgery


 Due to increase in glucose production and a reduction in glucose utilization
 Catecholamines and cortisol promote glycogenolysis and gluconeogenesis
 Initial failure of insulin secretion followed by insulin resistance affects the normal responses
 The proportion of the hyperglycaemic response reflects the severity of surgery
 Hyperglycaemia impairs wound healing and increase infection rates

Protein metabolism

 Initially there is inhibition of protein anabolism, followed later, if the stress response is
severe, by enhanced catabolism
 The amount of protein degradation is influenced by the type of surgery and also by the
nutritional status of the patient
 Mainly skeletal muscle protein is affected
 The amino acids released form acute phase proteins (fibrinogen, C reactive protein,
complement proteins, a2-macroglobulin, amyloid A and ceruloplasmin) and are used for
gluconeogenesis
 Nutritional support has little effect on preventing catabolism

Lipid metabolism
Increased catecholamine, cortisol and glucagon secretion, and insulin deficiency, promotes lipolysis
and ketone body production.

Salt and water metabolism

 ADH causes water retention, concentrated urine, and potassium loss and may continue for 3
to 5 days after surgery
 Renin causes sodium and water retention

Cytokines

 Glycoproteins
 Interleukins (IL) 1 to 17, interferons, and tumour necrosis factor
 Synthesized by activated macrophages, fibroblasts, endothelial and glial cells in response to
tissue injury from surgery or trauma
 IL-6 main cytokine associated with surgery. Peak 12 to 24 h after surgery and increase by
the degree of tissue damage Other effects of cytokines include fever, granulocytosis,
haemostasis, tissue damage limitation and promotion of healing.

Modifying the response

 Opioids suppress hypothalamic and pituitary hormone secretion


 At high doses the hormonal response to pelvic and abdominal surgery is abolished.
However, such doses prolong recovery and increase the need for postoperative ventilatory
support
 Spinal anaesthesia can reduce the glucose, ACTH, cortisol, GH and epinephrine changes,
although cytokine responses are unaltered
 Cytokine release is reduced in less invasive surgery
 Nutrition prevents the adverse effects of the stress response. Enteral feeding improves
recovery
 Growth hormone and anabolic steroids may improve outcome
 Normothermia decreases the metabolic response

References
Deborah Burton, Grainne Nicholson, and George Hall
Endocrine and metabolic response to surgery .

Contin Educ Anaesth Crit Care Pain (2004) 4(5): 144-147 doi:10.1093/bjaceaccp/mkh040
Next question
Which of the following is not a major function of the spleen in adults?

Iron reutilisation

Storage of platelets

Storage of monocytes

Haematopoeisis in haematological disorders

Storage red blood cells

The reservoir function of the spleen is less marked in humans than other animals (e.g. pigs) and in
normal individuals it can sequester between 5 and 10% of the red cell mass. The other stated
processes are major splenic functions and this accounts for the answer provided.
Please rate this question:

Discuss and give feedback


Next question

Spleen

The spleen is located in the left upper quadrant of the abdomen and its size can vary depending
upon the amount of blood it contains. The typical adult spleen is 12.5cm long and 7.5cm wide. The
usual weight of the adult spleen is 150g.
The exact position of the spleen can vary with respiratory activity, posture and the state of
surrounding viscera. It usually lies obliquely with its long axis aligned to the 9th, 10th and 11th ribs. It
is separated from these ribs by both diaphragm and pleural cavity. The normal spleen is not
palpable.

The shape of the spleen is influenced by the state of the colon and stomach. Gastric distension will
cause the spleen to resemble the shape of an orange segment. Colonic distension will cause it to
become more tetrahedral.

The spleen is almost entirely covered by peritoneum, which adheres firmly to its capsule. Recesses
of the greater sac separate it from the stomach and kidney. It develops from the upper dorsal
mesogastrium, remaining connected to the posterior abdominal wall and stomach by two folds of
peritoneum; the lienorenal ligament and gastrosplenic ligament. The lienorenal ligament is derived
from peritoneum where the wall of the general peritoneum meets the omental bursa between the left
kidney and spleen; the splenic vessels lie in its layers. The gastrosplenic ligament also has two
layers, formed by the meeting of the walls of the greater sac and omental bursa between spleen and
stomach, the short gastric and left gastroepiploic branches of the splenic artery pass in its layers.
Laterally, the spleen is in contact with the phrenicocolic ligament.

Relations
Superiorly Diaphragm

Anteriorly Gastric impression

Posteriorly Kidney

Inferiorly Colon

Tail of pancreas and splenic vessels (splenic artery divides here, branches pass to the
Hilum
white pulp transporting plasma)

Contents
White Immune function. Contains central trabecular artery. The germinal centres are supplied
pulp by arterioles called penicilliary radicles.

Red pulp Filters abnormal red blood cells.

Function

 Filtration of abnormal blood cells and foreign bodies such as bacteria.


 Immunity: IgM. Production of properdin, and tuftsin which help target fungi and bacteria for
phagocytosis.
 Haematopoiesis: up to 5th month gestation or in haematological disorders.
 Pooling: storage of 40% platelets.
 Iron reutilisation
 Storage monocytes

Disorders of the spleen


Massive splenomegaly

 Myelofibrosis
 Chronic myeloid leukaemia
 Visceral leishmaniasis (kala-azar)
 Malaria
 Gaucher's syndrome

Other causes (as above plus)

 Portal hypertension e.g. secondary to cirrhosis


 Lymphoproliferative disease e.g. CLL, Hodgkin's
 Haemolytic anaemia
 Infection: hepatitis, glandular fever
 Infective endocarditis
 Sickle-cell*, thalassaemia
 Rheumatoid arthritis (Felty's syndrome)

*the majority of adult patients with sickle-cell will have an atrophied spleen due to repeated infarction
Next question
Which one of the following is associated with increased lung compliance?

Kyphosis

Pulmonary oedema

Emphysema

Pulmonary fibrosis

Pneumonectomy

Please rate this question:

Discuss and give feedback


Next question

Respiratory physiology: lung compliance

Lung compliance is defined as change in lung volume per unit change in airway pressure

Causes of increased compliance

 age
 emphysema - this is due to loss alveolar walls and associated elastic tissue

Causes of decreased compliance

 pulmonary oedema
 pulmonary fibrosis
 pneumonectomy
 kyphosis

Next question
Which of the following areas is predominantly concerned with thermoregulation?

Hypothalamus

Anterior pituitary

Cerebellum

Brain stem

Temporal lobe

Theme from 2012 Exam


The hypothalamus is primarily concerned with thermoregulation. It may relay to the cerebral cortex
to induce behavioural adaptation to facilitate the thermoregulatory process.
Please rate this question:

Discuss and give feedback


Next question

Thermoregulation

 The hypothalamus is the main centre for thermoregulation. Peripheral and central
thermoreceptors relay to this region.
 Central thermoreceptors play the main role in maintenance of core temperature.
 Hypothalamus may initiate involuntary motor responses to raise body temperature
(e.g.shivering). It will also stimulate the sympathetic nervous system to produce peripheral
vasoconstriction and release of adrenaline from the adrenal medulla.
 Heat loss is governed by behavioural responses and by autonomic responses including
peripheral vasodilation.
 Heat loss can be maintained within the thermoneutral zone (25 to 30 degrees) although the
absolute value depends upon atmospheric humidity.
 Sepsis results in the release of cytokines that reset the thermoregulatory centre resulting in
fever.

Next question
Theme: Acid - base disorders

A. pH 7.64 pO2 10.0 kPa pCO2 2.8 kPa HCO3 20


B. pH 7.25 pO2 8.9 pCO2 3.2 HCO3 10
C. pH 7.20 pO2 6.2 pCO2 8.2 HCO3 27
D. pH 7.60 pO2 8.2 pCO2 5.8 HCO3 40
E. pH 7.45 pO2 7.2 pCO2 2.5 HCO3 24

Please match the diagnosis with the arterial blood gas result. Each option may be used once, more
than once or not at all.

89. Pulmonary embolus

You answered pH 7.64 pO2 10.0 kPa pCO2 2.8 kPa HCO3 20

The correct answer is pH 7.45 pO2 7.2 pCO2 2.5 HCO3 24

A combination of hypoxia and respiratory alkalosis should suggest a pulmonary embolus.


The respiratory alkalosis is due to hyperventilation associated with the pulmonary
embolism.

90. High output ureterosigmoidostomy

You answered pH 7.64 pO2 10.0 kPa pCO2 2.8 kPa HCO3 20

The correct answer is pH 7.25 pO2 8.9 pCO2 3.2 HCO3 10

There is acidosis. To compensate the patient will attempt to raise the pH level in the blood
by hyperventilating, hence the low CO2 level .

91. Widespread mesenteric infarction

You answered pH 7.64 pO2 10.0 kPa pCO2 2.8 kPa HCO3 20

The correct answer is pH 7.25 pO2 8.9 pCO2 3.2 HCO3 10

This is usually associated with acidosis, hyperventillation and reduction in bicarbonate.

Please rate this question:

Discuss and give feedback


Next question
Disorders of acid - base balance

Disorders of acid- base balance are often covered in the MRCS part A, both in the SBA and EMQ
sections.

The acid-base normogram below shows how the various disorders may be categorised

Image sourced from Wikipedia

Metabolic acidosis

 This is the most common surgical acid - base disorder.


 Reduction in plasma bicarbonate levels.
 Two mechanisms:

1. Gain of strong acid (e.g. diabetic ketoacidosis)


2. Loss of base (e.g. from bowel in diarrhoea)
- Classified according to the anion gap, this can be calculated by:
(Na+ + K+) - (Cl- + HCO3-).
- If a question supplies the chloride level then this is often a clue that the anion gap should be
calculated. The normal range = 10-18 mmol/L

Normal anion gap ( = hyperchloraemic metabolic acidosis)

 Gastrointestinal bicarbonate loss: diarrhoea, ureterosigmoidostomy, fistula


 Renal tubular acidosis
 Drugs: e.g. acetazolamide
 Ammonium chloride injection
 Addison's disease

Raised anion gap

 Lactate: shock, hypoxia


 Ketones: diabetic ketoacidosis, alcohol
 Urate: renal failure
 Acid poisoning: salicylates, methanol

Metabolic acidosis secondary to high lactate levels may be subdivided into two types:

 Lactic acidosis type A: (Perfusion disorders e.g.shock, hypoxia, burns)


 Lactic acidosis type B: (Metabolic e.g. metformin toxicity)

Metabolic alkalosis

 Usually caused by a rise in plasma bicarbonate levels.


 Rise of bicarbonate above 24 mmol/L will typically result in renal excretion of excess
bicarbonate.
 Caused by a loss of hydrogen ions or a gain of bicarbonate. It is due mainly to problems of
the kidney or gastrointestinal tract

Causes

 Vomiting / aspiration (e.g. Peptic ulcer leading to pyloric stenosis, nasogastric suction)
 Diuretics
 Liquorice, carbenoxolone
 Hypokalaemia
 Primary hyperaldosteronism
 Cushing's syndrome
 Bartter's syndrome
 Congenital adrenal hyperplasia

Mechanism of metabolic alkalosis


 Activation of renin-angiotensin II-aldosterone (RAA) system is a key factor
 Aldosterone causes reabsorption of Na+ in exchange for H+ in the distal convoluted tubule
 ECF depletion (vomiting, diuretics) → Na+ and Cl- loss → activation of RAA system → raised
aldosterone levels
 In hypokalaemia, K+ shift from cells → ECF, alkalosis is caused by shift of H + into cells to
maintain neutrality

Respiratory acidosis

 Rise in carbon dioxide levels usually as a result of alveolar hypoventilation


 Renal compensation may occur leading to Compensated respiratory acidosis

Causes

 COPD
 Decompensation in other respiratory conditions e.g. Life-threatening asthma / pulmonary
oedema
 Sedative drugs: benzodiazepines, opiate overdose

Respiratory alkalosis

 Hyperventilation resulting in excess loss of carbon dioxide


 This will result in increasing pH

Causes

 Psychogenic: anxiety leading to hyperventilation


 Hypoxia causing a subsequent hyperventilation: pulmonary embolism, high altitude
 Early salicylate poisoning*
 CNS stimulation: stroke, subarachnoid haemorrhage, encephalitis
 Pregnancy

*Salicylate overdose leads to a mixed respiratory alkalosis and metabolic acidosis. Early stimulation
of the respiratory centre leads to a respiratory alkalosis whilst later the direct acid effects of
salicylates (combined with acute renal failure) may lead to an acidosis
Next question
A 45 year old male is diagnosed with carcinoma of the head of the pancreas. He reports that his
stool sticks to the commode and will not flush away. Loss of which of the following enzymes is most
likely to be responsible for this problem?

Lipase

Amylase

Trypsin

Elastase

None of the above

Theme from April 2014 Exam


Loss of lipase is one of the key features in the development of steatorrhoea which typically consists
of pale and offensive stools that are difficult to flush away.
Please rate this question:

Discuss and give feedback


Next question

Pancreatic cancer

 Adenocarcinoma
 Risk factors: Smoking, diabetes, adenoma, familial adenomatous polyposis
 Mainly occur in the head of the pancreas (70%)
 Spread locally and metastasizes to the liver
 Carcinoma of the pancreas should be differentiated from other periampullary tumours with
better prognosis

Clinical features

 Weight loss
 Painless jaundice
 Epigastric discomfort (pain usually due to invasion of the coeliac plexus is a late feature)
 Pancreatitis
 Trousseau's sign: migratory superficial thrombophlebitis
Investigations

 USS: May miss small lesions


 CT Scanning (pancreatic protocol). If unresectable on CT then no further staging needed
 PET/CT for those with operable disease on CT alone
 ERCP/ MRI for bile duct assessment
 Staging laparoscopy to exclude peritoneal disease

Management

 Head of pancreas: Whipple's resection (SE dumping and ulcers). Newer techniques include
pylorus preservation and SMA/ SMV resection
 Carcinoma body and tail: poor prognosis, distal pancreatectomy, if operable
 Usually adjuvent chemotherapy for resectable disease
 ERCP and stent for jaundice and palliation
 Surgical bypass may be needed for duodenal obstruction

Next question
Which of the following drugs does not interfere with the laboratory analysis of serum cortisol levels?

Dexamethasone

Prednisolone

Hydrocortisone IV

Hydrocortisone PO

Hydrocortisone IM

Prednisolone and it's metabolites can chemically mimic cortisol in radio-immunoassay techniques of
laboratory analysis.

Dexamethasone can be given as glucorticoid replacement during testing for addisons or adrenal
insufficiency as it does not interfere with cortisol levels. For example, if you have a patient with
polymyalgia rheumatica and they are on long term prednisolone, you can replace the prednisolone
with dexamethasone to undertake a short synacthen test.
Please rate this question:

Discuss and give feedback


Next question

Cortisol

 Glucocorticoid
 Released by zona fasiculata of the adrenal gland
 90% protein bound; 10% active
 Circadian rhythm: High in the mornings
 Negative feedback via ACTH

Actions

 Glycogenolysis
 Gluconeogenesis
 Protein catabolism
 Lipolysis
 Stress response
 Anti-inflammatory
 Decrease protein in bones
 Increase gastric acid
 Increases neutrophils/platelets/red blood cells
 Inhibits fibroblastic activity

Next question
An elderly lady who presented with weight loss and malabsorption was found to have amyloid of the
small bowel. On presentation she was found to have osteomalacia and was hypocalcaemic. Over
the past seven days she has received total parenteral nutrition with adequate calcium replacement.
Despite this she remained hypocalcaemic. Deficiency of which of the following electrolytes is most
likely to account for this process?

Magnesium

Potassium

Sodium

Phosphate

None of the above

Theme from September 2015 Exam


Patients with malabsorption may develop magnesium deficiency, although her TPN feeds may have
contained magnesium it may not have been sufficient to correct her losses. Sodium, phosphate and
potassium would not have this effect on serum calcium.

Please rate this question:

Discuss and give feedback

Next question

Combined deficiency of magnesium and calcium

Magnesium is required for both PTH secretion and its action on target tissues. Hypomagnesaemia
may both cause hypocalcaemia and render patients unresponsive to treatment with calcium and
vitamin D supplementation.

Magnesium is the fourth most abundant cation in the body. The body contains 1000mmol, with half
contained in bone and the remainder in muscle, soft tissues and extracellular fluid. There is no one
specific hormonal control of magnesium and various hormones including PTH and aldosterone affect
the renal handling of magnesium.

Magnesium and calcium interact at a cellular level also and as a result decreased magnesium will
tend to affect the permeability of cellular membranes to calcium, resulting in hyperexcitability.

Next question
A 19 year old man is attacked outside a club and beaten with a baseball bat. He sustains a blow to
the right side of his head. He is brought to the emergency department and a policy of observation is
adopted. His glasgow coma score deteriorates and he becomes comatose. Which of the following
haemodynamic parameters is most likely to be present?

Hypertension and bradycardia

Hypotension and tachycardia

Hypotension and bradycardia

Hypertension and tachycardia

Normotension and bradycardia

Theme from April 2013 Exam


Hypertension and bradycardia are seen prior to coning. The brain autoregulates its blood supply by
controlling systemic blood pressure.
Please rate this question:

Discuss and give feedback


Next question

Head injury

Patients who suffer head injuries should be managed according to ATLS principles and extra cranial
injuries should be managed alongside cranial trauma. Inadequate cardiac output will compromise
CNS perfusion irrespective of the nature of the cranial injury.

Types of traumatic brain injury


Bleeding into the space between the dura mater and the skull. Often results from
acceleration-deceleration trauma or a blow to the side of the head. The majority
of extradural haematomas occur in the temporal region where skull fractures
cause a rupture of the middle meningeal artery.

Extradural
Features
haematoma

 Raised intracranial pressure


 Some patients may exhibit a lucid interval
Bleeding into the outermost meningeal layer. Most commonly occur around the
frontal and parietal lobes. May be either acute or chronic.
Subdural
haematoma Risk factors include old age and alcoholism.

Slower onset of symptoms than a extradural haematoma.

Usually occurs spontaneously in the context of a ruptured cerebral aneurysm, but


Subarachnoid
may be seen in association with other injuries when a patient has sustained a
haemorrhage
traumatic brain injury.

Pathophysiology

 Primary brain injury may be focal (contusion/ haematoma) or diffuse (diffuse axonal injury)
 Diffuse axonal injury occurs as a result of mechanical shearing following deceleration,
causing disruption and tearing of axons
 Intra-cranial haematomas can be extradural, subdural or intracerebral, while contusions may
occur adjacent to (coup) or contralateral (contre-coup) to the side of impact
 Secondary brain injury occurs when cerebral oedema, ischaemia, infection, tonsillar or
tentorial herniation exacerbates the original injury. The normal cerebral auto regulatory
processes are disrupted following trauma rendering the brain more susceptible to blood flow
changes and hypoxia
 The Cushings reflex (hypertension and bradycardia) often occurs late and is usually a pre
terminal event

Management

 Where there is life threatening rising ICP such as in extra dural haematoma and whilst
theatre is prepared or transfer arranged use of IV mannitol/ frusemide may be required.
 Diffuse cerebral oedema may require decompressive craniotomy
 Exploratory Burr Holes have little management in modern practice except where scanning
may be unavailable and to thus facilitate creation of formal craniotomy flap
 Depressed skull fractures that are open require formal surgical reduction and debridement,
closed injuries may be managed non operatively if there is minimal displacement.
 ICP monitoring is appropriate in those who have GCS 3-8 and normal CT scan.
 ICP monitoring is mandatory in those who have GCS 3-8 and abnormal CT scan.
 Hyponatraemia is most likely to be due to syndrome of inappropriate ADH secretion.
 Minimum of cerebral perfusion pressure of 70mmHg in adults.
 Minimum cerebral perfusion pressure of between 40 and 70 mmHg in children.

Interpretation of pupillary findings in head injuries


Pupil size Light response Interpretation
Pupil size Light response Interpretation

Unilaterally dilated Sluggish or fixed 3rd nerve compression secondary to tentorial


herniation

Bilaterally dilated Sluggish or fixed  Poor CNS perfusion


 Bilateral 3rd nerve palsy

Unilaterally dilated or Cross reactive (Marcus - Optic nerve injury


equal Gunn)

Bilaterally constricted May be difficult to  Opiates


assess  Pontine lesions
 Metabolic encephalopathy

Unilaterally Preserved Sympathetic pathway disruption


constricted
Next question
Which of the following drugs does not cause pseudohaematuria?

Rifampicin

Quinine

Noradrenaline

Levodopa

Phenytoin

Rifampicin, phenytoin, levodopa, methyldopa, and quinine all cause pseudohaematuria.


Please rate this question:

Discuss and give feedback


Next question

Haematuria

Causes of haematuria

Trauma  Injury to renal tract


 Renal trauma commonly due to blunt injury (others penetrating
injuries)
 Ureter trauma rare: iatrogenic
 Bladder trauma: due to RTA or pelvic fractures

Infection  Remember TB

Malignancy  Renal cell carcinoma (remember paraneoplastic syndromes):


painful or painless
 Urothelial malignancies: 90% are transitional cell carcinoma, can
occur anywhere along the urinary tract. Painless haematuria.
 Squamous cell carcinoma and adenocarcinoma: rare bladder
tumours
 Prostate cancer
 Penile cancers: SCC

Renal disease  Glomerulonephritis

Stones  Microscopic haematuria common

Structural  Benign prostatic hyperplasia (BPH) causes haematuria due to


abnormalities hypervascularity of the prostate gland
 Cystic renal lesions e.g. polycystic kidney disease
 Vascular malformations
 Renal vein thrombosis due to renal cell carcinoma

Coagulopathy  Causes bleeding of underlying lesions

Drugs  Cause tubular necrosis or interstitial nephritis: aminoglycosides,


chemotherapy
 Interstitial nephritis: penicillin, sulphonamides, and NSAIDs
 Anticoagulants

Benign  Exercise

Gynaecological  Endometriosis: flank pain, dysuria, and haematuria that is cyclical

Iatrogenic  Catheterisation
 Radiotherapy; cystitis, severe haemorrhage, bladder necrosis

Pseudohaematuria For example following consumption of beetroot

References
Http://bestpractice.bmj.com/best-practice/monograph/316/overview/aetiology.html
Next question
A 74-year-old woman with thyroid cancer is admitted due to shortness of breath. What is the best
investigation to assess for possible compression of the upper airways?

Arterial blood gases

Forced vital capacity

Transfer factor

Peak expiratory flow rate

Flow volume loop

Flow volume loop is the investigation of choice for upper airway compression.

Please rate this question:

Discuss and give feedback

Next question

Flow volume loop

A normal flow volume loop is often described as a 'triangle on top of a semi circle'

Flow volume loops are the most suitable way of assessing compression of the upper airway

Next question
Which of the following statements relating to cerebrospinal fluid is untrue?

The choroid plexus is only present in the lateral ventricles

Total CSF volume is 100-150ml

CSF pressure is usually 10-15mmHg

The cerebral aqueduct connects the third and fourth ventricles

The foramen of Luschka are paired and lie laterally in the fourth ventricle

The choroid plexus lies in all ventricles.


Please rate this question:

Discuss and give feedback


Next question

Cerebrospinal fluid

The CSF fills the space between the arachnoid mater and pia mater (covering surface of the brain).
The total volume of CSF in the brain is approximately 150ml. Approximately 500 ml is produced by
the ependymal cells in the choroid plexus (70%), or blood vessels (30%). It is reabsorbed via the
arachnoid granulations which project into the venous sinuses.

Circulation
1. Lateral ventricles (via foramen of Munro)
2. 3rd ventricle
3. Cerebral aqueduct (aqueduct of Sylvius)
4. 4th ventricle (via foramina of Magendie and Luschka)
5. Subarachnoid space
6. Reabsorbed into the venous system via arachnoid granulations into superior sagittal sinus

Composition

 Glucose: 50-80mg/dl
 Protein: 15-40 mg/dl
 Red blood cells: Nil
 White blood cells: 0-3 cells/ mm3

Next question
Which substance can be used to achieve the most accurate measurement of the glomerular filtration
rate?

Glucose

Protein

Inulin

Creatinine

Para-amino hippuric acid

Theme from April 2016 exam


Creatinine declines with age due to decline in renal function and muscle mass. Glucose, protein
(amino acids) and PAH are reabsorbed by the kidney.
Please rate this question:

Discuss and give feedback


Next question

Renal Physiology

Overview

 Each nephron is supplied with blood from an afferent arteriole that opens onto the glomerular
capillary bed.
 Blood then flows to an efferent arteriole, supplying the peritubular capillaries and medullary
vasa recta.
 The kidney receives up to 25% of resting cardiac output.

Control of blood flow

 The kidney is able to autoregulate its blood flow between systolic pressures of 80- 180mmHg
so there is little variation in renal blood flow.
 This is achieved by myogenic control of arteriolar tone, both sympathetic input and hormonal
signals (e.g. renin) are responsible.
Glomerular structure and function

 Blood inside the glomerulus has considerable hydrostatic pressure.


 The basement membrane has pores that will allow free diffusion of smaller solutes, larger
negatively charged molecules such as albumin are unable to cross.
 The glomerular filtration rate (GFR) is equal to the concentration of a solute in the urine,
times the volume of urine produced per minute, divided by the plasma concentration
(assuming that the solute is freely diffused e.g. inulin).
 In clinical practice creatinine is used because it is subjected to very little proximal tubular
secretion.
 Although subject to variability, the typical GFR is 125ml per minute.
 Glomerular filtration rate = Total volume of plasma per unit time leaving the capillaries and
entering the bowman's capsule
 Renal clearance = volume plasma from which a substance is removed per minute by the
kidneys

Substances used to measure GFR have the following features:


1. Inert
2. Free filtration from the plasma at the glomerulus (not protein bound)
3. Not absorbed or secreted at the tubules
4. Plasma concentration constant during urine collection

Examples: inulin, creatinine

GFR = urine concentration (mmol/l) x urine volume (ml/min)


--------------------------------------------------------------------------
plasma concentration (mmol/l)

 The clearance of a substance is dependent not only on its diffusivity across the basement
membrane but also subsequent tubular secretion and / or reabsorption.
 So glucose which is freely filtered across the basement membrane is usually reabsorbed
from tubules giving a clearance of zero.

Tubular function

 Reabsorption and secretion of substances occurs in the tubules.


 In the proximal tubule substrates such as glucose, amino acids and phosphate are co-
transported with sodium across the semi permeable membrane.
 Up to two thirds of filtered water is reabsorbed in the proximal tubules.
 This will lead to increase in urea concentration in the distal tubule allowing for its increased
diffusion.
 Substances to be secreted into the tubules are taken up from the peritubular blood by tubular
cells.
 Solutes such as paraaminohippuric acid are cleared with a single passage through the
kidneys and this is why it is used to measure renal plasma flow. Ions such as calcium and
phosphate will have a tubular reabsorption that is influenced by plasma PTH levels.
 Potassium may be both secreted and re-absorbed and is co-exchanged with sodium.

Loop of Henle

 Approximately 60 litres of water containing 9000mmol sodium enters the descending limb of
the loop of Henle in 24 hours.
 Loops from the juxtamedullary nephrons run deep into the medulla.
 The osmolarity of fluid changes and is greatest at the tip of the papilla.
 The thin ascending limb is impermeable to water, but highly permeable to sodium and
chloride ions.
 This loss means that at the beginning of the thick ascending limb the fluid is hypo osmotic
compared with adjacent interstitial fluid.
 In the thick ascending limb the reabsorption of sodium and chloride ions occurs by both
facilitated and passive diffusion pathways.
 The loops of Henle are co-located with vasa recta, these will have similar solute
compositions to the surrounding extracellular fluid so preventing the diffusion and
subsequent removal of this hypertonic fluid.
 The energy dependent reabsorption of sodium and chloride in the thick ascending limb helps
to maintain this osmotic gradient.

Next question
A 45 year old man sustains a closed head injury. He is initially alert, however, his level of
consciousness deteriorates on arrival at hospital. An intra cranial pressure monitor is inserted. What
is the normal intracranial pressure?

35 - 45mm Hg

45 - 55mm Hg

<15mm Hg

25 - 35mm Hg

25 - 30 mm Hg

The normal intracranial pressure is between 7 and 15 mm Hg. The brain can accommodate
increases up to 24 mm Hg, thereafter clinical features will become evident.
Please rate this question:

Discuss and give feedback


Next question

Applied neurophysiology

 Pressure within the cranium is governed by the Monroe-Kelly doctrine. This considers the
skull as a closed box. Increases in mass can be accommodated by loss of CSF. Once a
critical point is reached (usually 100- 120ml of CSF lost) there can be no further
compensation and ICP rises sharply. The next step is that pressure will begin to equate with
MAP and neuronal death will occur. Herniation will also accompany this process.
 The CNS can autoregulate its own blood supply. Vaso constriction and dilatation of the
cerebral blood vessels is the primary method by which this occurs. Extremes of blood
pressure can exceed this capacity resulting in risk of stroke. Other metabolic factors such as
hypercapnia will also cause vasodilation, which is of importance in ventilating head injured
patients.
 The brain can only metabolise glucose, when glucose levels fall, consciousness will be
impaired.

Next question
A 55-year-old man with a history of type 2 diabetes mellitus, bipolar disorder and chronic obstructive
pulmonary disease has bloods taken during a pre operative assessment of an inguinal hernia repair:

Na+ 125 mmol/l

K+ 3.8 mmol/l

Bicarbonate 24 mmol/l

Urea 3.7 mmol/l

Creatinine 92 µmol/l

Due to his smoking history a chest x-ray is ordered which is reported as normal. The Consultant
asks you what is the most likely cause for the hyponatraemia?

Metformin

Lithium

Carbamazepine

Sodium valproate

Pioglitazone

SIADH - drug causes: carbamazepine, sulfonylureas, SSRIs, tricyclics

Lithium can cause diabetes insipidus but this is generally associated with a high sodium. Lithium
only tends to cause raised antidiuretic hormone levels following a severe overdosage.
Please rate this question:

Discuss and give feedback


Next question

syndrome of inappropriate antidiuretic hormone (SIADH): causes

Malignancy

 especially small cell lung cancer


 also: pancreas, prostate

Neurological

 stroke
 subarachnoid haemorrhage
 subdural haemorrhage
 meningitis/encephalitis/abscess

Infections

 tuberculosis
 pneumonia

Drugs

 sulfonylureas
 SSRIs, tricyclics
 carbamazepine
 vincristine
 cyclophosphamide

Other causes

 positive end-expiratory pressure (PEEP)


 porphyrias

Next question
A 24 year old man is involved in a road traffic accident. His right leg is trapped for 6 hours whilst he
is moved. On examination his foot is insensate and a dorsalis pedis pulse is only weakly felt. Which
of the biochemical abnormalities listed below is most likely to be present?

Alkalosis

Hypercalcaemia

Hypocalcaemia

Hyperkalaemia

Hyponatraemia

In this scenario the patient will have a compartment syndrome, delayed diagnosis and muscle death.
The effect of muscle death will result in the release of potassium. It is also highly likely that there will
be a degree of renal impairment, the result of which is that the serum potassium is likely to be high.
Please rate this question:

Discuss and give feedback


Next question

Hyperkalaemia

 Plasma potassium levels are regulated by a number of factors including aldosterone, acid-
base balance and insulin levels.
 Metabolic acidosis is associated with hyperkalaemia as hydrogen and potassium ions
compete with each other for exchange with sodium ions across cell membranes and in the
distal tubule.
 ECG changes seen in hyperkalaemia include tall-tented T waves, small P waves, widened
QRS leading to a sinusoidal pattern and asystole

Causes of hyperkalaemia

 Acute renal failure


 Drugs*: potassium sparing diuretics, ACE inhibitors, angiotensin 2 receptor blockers,
spironolactone, ciclosporin, heparin**
 Metabolic acidosis
 Addison's
 Tissue necrosis/rhabdomylosis: burns, trauma
 Massive blood transfusion

Foods that are high in potassium

 Salt substitutes (i.e. Contain potassium rather than sodium)


 Bananas, oranges, kiwi fruit, avocado, spinach, tomatoes

*beta-blockers interfere with potassium transport into cells and can potentially cause hyperkalaemia
in renal failure patients - remember beta-agonists, e.g. Salbutamol, are sometimes used as
emergency treatment

**both unfractionated and low-molecular weight heparin can cause hyperkalaemia. This is thought to
be caused by inhibition of aldosterone secretion
Next question
A 39 year old lady has recurrent attacks of biliary colic. What is the approximate volume of bile to
enter the duodenum per 24 hours?

500 mL

50 mL

100 mL

2000 mL

150 mL

Between 500 mL and 1.5 L of bile enters the small bowel daily. Most bile salts are recycled by the
enterohepatic circulation. When the gallbladder contracts the lumenal pressure is approximately
25cm water, which is why biliary colic may be so painful.

Please rate this question:

Discuss and give feedback

Next question

Bile

Bile is produced at a rate of between 500ml and 1500mL per day. Bile is composed of bile salts,
bicarbonate, cholesterol, steroids and water. There are three main factors regulating bile flow;
hepatic secretion, gall bladder contraction and sphincter of oddi resistance. Bile salts are absorbed
in the terminal ileum (and recycled to the liver). Over 90% of all bile salts are recycled in this way,
such that the total pool of bile salts is recycled up to six times a day.

Primary bile salts


Cholate and chenodeoxycholate.

Secondary bile salts


Formed by bacterial action on primary bile salts. These are deoxycholate and lithocholate. Of these
deoxycholate is reabsorbed, whilst lithocholate is insoluble and excreted.

Pathophysiology of gallstones
Bile salts have a detergent action. They aggregate to form micelles and these have a lipid centre in
which fats may be transported. Excessive quantities of cholesterol cannot be transported in this way
and will tend to precipitate, resulting in the formation of cholesterol rich gallstones.

Next question
At which of the following sites is the most water absorbed?

Right colon

Left colon

Stomach

Jejunum

Duodenum

Water absorption in the gastrointestinal tract predominantly occurs in the small bowel (jejunum and
ileum). The colon is an important site of water absorption, however, its overall contribution is
relatively small. The importance of the colonic component to water absorption may increase
following extensive small bowel resections.

Please rate this question:

Discuss and give feedback

Next question

Water absorption

During a 24 hours period the average person will ingest up to 2000ml of liquid orally. In addition a
further 8000ml of fluid will enter the small bowel as gastrointestinal secretions. Intestinal water
absorption is a passive process and is related to solute load. In the jejunum the active absorption of
glucose and amino acids will create a concentration gradient that water will flow across. In the ileum
most water is absorbed by a process of facilitated diffusion (with sodium).
Approximately 150ml of water enters the colon daily, most is absorbed, the colon can adapt to, and
increase this amount following resection.

Next question
Which of the following is not a characteristic of the proximal convoluted tubule in the kidney?

Up to 95% of filtered amino acids will be reabsorbed at this site

It is a risk of damage in a patient with compartment syndrome due to a tibial fracture

It is responsible for absorbing more than 50% of filtered water

Its secretory function is most effective at low systolic blood pressures (typically less than
100mmHg)

Glucose is reabsorbed by a process of facilitated diffusion

The proximal convoluted tubule may undergo necrosis in situations such as compartment syndrome.
It is responsible for reabsorbing up to two thirds of filtered water. Low systolic blood pressures
(below the renal autoregulatory range) are a risk factor for acute tubular necrosis. Within the
autoregulatory range the absolute value of systolic BP has little effect.
Please rate this question:

Discuss and give feedback


Next question

Renal Physiology

Overview

 Each nephron is supplied with blood from an afferent arteriole that opens onto the glomerular
capillary bed.
 Blood then flows to an efferent arteriole, supplying the peritubular capillaries and medullary
vasa recta.
 The kidney receives up to 25% of resting cardiac output.

Control of blood flow

 The kidney is able to autoregulate its blood flow between systolic pressures of 80- 180mmHg
so there is little variation in renal blood flow.
 This is achieved by myogenic control of arteriolar tone, both sympathetic input and hormonal
signals (e.g. renin) are responsible.
Glomerular structure and function

 Blood inside the glomerulus has considerable hydrostatic pressure.


 The basement membrane has pores that will allow free diffusion of smaller solutes, larger
negatively charged molecules such as albumin are unable to cross.
 The glomerular filtration rate (GFR) is equal to the concentration of a solute in the urine,
times the volume of urine produced per minute, divided by the plasma concentration
(assuming that the solute is freely diffused e.g. inulin).
 In clinical practice creatinine is used because it is subjected to very little proximal tubular
secretion.
 Although subject to variability, the typical GFR is 125ml per minute.
 Glomerular filtration rate = Total volume of plasma per unit time leaving the capillaries and
entering the bowman's capsule
 Renal clearance = volume plasma from which a substance is removed per minute by the
kidneys

Substances used to measure GFR have the following features:


1. Inert
2. Free filtration from the plasma at the glomerulus (not protein bound)
3. Not absorbed or secreted at the tubules
4. Plasma concentration constant during urine collection

Examples: inulin, creatinine

GFR = urine concentration (mmol/l) x urine volume (ml/min)


--------------------------------------------------------------------------
plasma concentration (mmol/l)

 The clearance of a substance is dependent not only on its diffusivity across the basement
membrane but also subsequent tubular secretion and / or reabsorption.
 So glucose which is freely filtered across the basement membrane is usually reabsorbed
from tubules giving a clearance of zero.

Tubular function

 Reabsorption and secretion of substances occurs in the tubules.


 In the proximal tubule substrates such as glucose, amino acids and phosphate are co-
transported with sodium across the semi permeable membrane.
 Up to two thirds of filtered water is reabsorbed in the proximal tubules.
 This will lead to increase in urea concentration in the distal tubule allowing for its increased
diffusion.
 Substances to be secreted into the tubules are taken up from the peritubular blood by tubular
cells.
 Solutes such as paraaminohippuric acid are cleared with a single passage through the
kidneys and this is why it is used to measure renal plasma flow. Ions such as calcium and
phosphate will have a tubular reabsorption that is influenced by plasma PTH levels.
 Potassium may be both secreted and re-absorbed and is co-exchanged with sodium.

Loop of Henle

 Approximately 60 litres of water containing 9000mmol sodium enters the descending limb of
the loop of Henle in 24 hours.
 Loops from the juxtamedullary nephrons run deep into the medulla.
 The osmolarity of fluid changes and is greatest at the tip of the papilla.
 The thin ascending limb is impermeable to water, but highly permeable to sodium and
chloride ions.
 This loss means that at the beginning of the thick ascending limb the fluid is hypo osmotic
compared with adjacent interstitial fluid.
 In the thick ascending limb the reabsorption of sodium and chloride ions occurs by both
facilitated and passive diffusion pathways.
 The loops of Henle are co-located with vasa recta, these will have similar solute
compositions to the surrounding extracellular fluid so preventing the diffusion and
subsequent removal of this hypertonic fluid.
 The energy dependent reabsorption of sodium and chloride in the thick ascending limb helps
to maintain this osmotic gradient.

Next question
An arterial blood gas sample is taken and the following results obtained;

PaO2 8kPa

PaCO2 4kPa

pH 7.4

With which of the following are these values most consistent?

Compensated metabolic alkalosis

Pulmonary atelectasis

Alveolar hypoventilation

Residing at 4500M for 48 hours

LAD occlusion

The patient has low oxygen tension and low carbon dioxide. The pH is normal so there is
compensation for a long standing condition in which oxygenation is reduced. There is neither
alkalosis, nor hypoventilation as the carbon dioxide is low. At very high altitude, the low oxygen
tension can exceed the anaerobic threshold and carbon dioxide levels increase.

Please rate this question:

Discuss and give feedback

Next question

Arterial blood gas interpretation


In advanced life support training, a 5 step approach to arterial blood gas interpretation is advocated.

1. How is the patient?

2. Is the patient hypoxaemic?


The Pa02 on air should be 10.0-13.0 kPa

3. Is the patient acidaemic (pH <7.35) or alkalaemic (pH >7.45)

4. What has happened to the PaCO2?


If there is acidaemia, an elevated PaCO2 will account for this

5. What is the bicarbonate level or base excess?


A metabolic acidosis will have a low bicarbonate level and a low base excess (< -2 mmol)
A metabolic alkalosis will have a high bicarbonate and a high base excess (> +2 mmol)

Next question
Which of the following does not cause a normal anion gap acidosis?

Pancreatic fistula

Acetazolamide

Uraemia

Ureteric diversion

Renal tubular acidosis

Normal Gap Acidosis: HARDUP

H - Hyperalimentation/hyperventilation
A - Acetazolamide
R - Renal tubular acidosis
D - Diarrhoea
U - Ureteral diversion
P - Pancreatic fistula/parenteral saline

Uraemia will typically cause a high anion gap acidosis. It is one of the unmeasured anions.
Please rate this question:

Discuss and give feedback


Next question

Disorders of acid - base balance

Disorders of acid- base balance are often covered in the MRCS part A, both in the SBA and EMQ
sections.

The acid-base normogram below shows how the various disorders may be categorised
Image sourced from Wikipedia

Metabolic acidosis

 This is the most common surgical acid - base disorder.


 Reduction in plasma bicarbonate levels.
 Two mechanisms:

1. Gain of strong acid (e.g. diabetic ketoacidosis)


2. Loss of base (e.g. from bowel in diarrhoea)
- Classified according to the anion gap, this can be calculated by:
(Na+ + K+) - (Cl- + HCO3-).
- If a question supplies the chloride level then this is often a clue that the anion gap should be
calculated. The normal range = 10-18 mmol/L

Normal anion gap ( = hyperchloraemic metabolic acidosis)

 Gastrointestinal bicarbonate loss: diarrhoea, ureterosigmoidostomy, fistula


 Renal tubular acidosis
 Drugs: e.g. acetazolamide
 Ammonium chloride injection
 Addison's disease

Raised anion gap


 Lactate: shock, hypoxia
 Ketones: diabetic ketoacidosis, alcohol
 Urate: renal failure
 Acid poisoning: salicylates, methanol

Metabolic acidosis secondary to high lactate levels may be subdivided into two types:

 Lactic acidosis type A: (Perfusion disorders e.g.shock, hypoxia, burns)


 Lactic acidosis type B: (Metabolic e.g. metformin toxicity)

Metabolic alkalosis

 Usually caused by a rise in plasma bicarbonate levels.


 Rise of bicarbonate above 24 mmol/L will typically result in renal excretion of excess
bicarbonate.
 Caused by a loss of hydrogen ions or a gain of bicarbonate. It is due mainly to problems of
the kidney or gastrointestinal tract

Causes

 Vomiting / aspiration (e.g. Peptic ulcer leading to pyloric stenosis, nasogastric suction)
 Diuretics
 Liquorice, carbenoxolone
 Hypokalaemia
 Primary hyperaldosteronism
 Cushing's syndrome
 Bartter's syndrome
 Congenital adrenal hyperplasia

Mechanism of metabolic alkalosis

 Activation of renin-angiotensin II-aldosterone (RAA) system is a key factor


 Aldosterone causes reabsorption of Na+ in exchange for H+ in the distal convoluted tubule
 ECF depletion (vomiting, diuretics) → Na+ and Cl- loss → activation of RAA system → raised
aldosterone levels
 In hypokalaemia, K+ shift from cells → ECF, alkalosis is caused by shift of H + into cells to
maintain neutrality
Respiratory acidosis

 Rise in carbon dioxide levels usually as a result of alveolar hypoventilation


 Renal compensation may occur leading to Compensated respiratory acidosis

Causes

 COPD
 Decompensation in other respiratory conditions e.g. Life-threatening asthma / pulmonary
oedema
 Sedative drugs: benzodiazepines, opiate overdose

Respiratory alkalosis

 Hyperventilation resulting in excess loss of carbon dioxide


 This will result in increasing pH

Causes

 Psychogenic: anxiety leading to hyperventilation


 Hypoxia causing a subsequent hyperventilation: pulmonary embolism, high altitude
 Early salicylate poisoning*
 CNS stimulation: stroke, subarachnoid haemorrhage, encephalitis
 Pregnancy

*Salicylate overdose leads to a mixed respiratory alkalosis and metabolic acidosis. Early stimulation
of the respiratory centre leads to a respiratory alkalosis whilst later the direct acid effects of
salicylates (combined with acute renal failure) may lead to an acidosis
Next question
Which one of the following would cause a rise in the carbon monoxide transfer factor (TLCO)?

Emphysema

Pulmonary embolism

Pulmonary haemorrhage

Pneumonia

Pulmonary fibrosis

Transfer factor

 raised: asthma, haemorrhage, left-to-right shunts, polycythaemia


 low: everything else

Where alveolar haemorrhage occurs the TLCO tends to increase due to the enhanced uptake of
carbon monoxide by intra-alveolar haemoglobin.
Please rate this question:

Discuss and give feedback


Next question

Transfer factor

The transfer factor describes the rate at which a gas will diffuse from alveoli into blood. Carbon
monoxide is used to test the rate of diffusion. Results may be given as the total gas transfer (TLCO)
or that corrected for lung volume (transfer coefficient, KCO)

Causes of a raised TLCO Causes of a lower TLCO

 asthma  pulmonary fibrosis


 pulmonary haemorrhage (Wegener's, Goodpasture's)  pneumonia
 left-to-right cardiac shunts  pulmonary emboli
 polycythaemia  pulmonary oedema
 hyperkinetic states  emphysema
 male gender, exercise  anaemia
 low cardiac output

KCO also tends to increase with age. Some conditions may cause an increased KCO with a normal
or reduced TLCO

 pneumonectomy/lobectomy
 scoliosis/kyphosis
 neuromuscular weakness
 ankylosis of costovertebral joints e.g. ankylosing spondylitis

Next question
Which of the following is least likely to be associated with hypercalcaemia?

Thiazides

Antacids

Coeliac disease

Sarcoidosis

Zolinger-Ellison syndrome

Mnemonic for the causes of hypercalcaemia:

CHIMPANZEES

C alcium supplementation
H yperparathyroidism
I atrogentic (Drugs: Thiazides)
M ilk Alkali syndrome
P aget disease of the bone
A cromegaly and Addison's Disease
N eoplasia
Z olinger-Ellison Syndrome (MEN Type I)
E xcessive Vitamin D
E xcessive Vitamin A
S arcoidosis

Patients with coeliac disease tend to develop hypocalcaemia due to malabsorption of calcium by the
bowel.
Please rate this question:

Discuss and give feedback


Next question

Hypercalcaemia

Main causes

 Malignancy (most common cause in hospital in-patients)


 Primary hyperparathyroidism (commonest cause in non hospitalised patients)
Less common

 Sarcoidosis (extrarenal synthesis of calcitriol )


 Thiazides, lithium
 Immobilisation
 Pagets disease
 Vitamin A/D toxicity
 Thyrotoxicosis
 MEN
 Milk alkali syndrome

Clinical features
Stones, bones, abdominal groans, and psychic moans
High serum calcium levels result in decreased neuronal excitability. Therefore sluggish reflexes,
muscle weakness and constipation may occur.
Next question
Release of somatostatin from the pancreas will result in which of the following?

Decrease in pancreatic exocrine secretions

Contraction of the gallbladder

Increase in the rate of gastric emptying

Increased synthesis of growth hormone

Increased insulin release

Octreotide reduces exocrine pancreatic secretions so is used to treat high output pancreatic fistulae
(though parenteral feeding is most effective). Other uses include variceal bleeding and treatment of
acromegaly.
Inhibits growth hormone and insulin release (when released from pancreas).
Somatostatin is also released by the hypothalamus causing a negative feedback response on
growth hormone.

Please rate this question:

Discuss and give feedback

Next question

Somatostatin

Somatostatin is produced in the D cells of the pancreatic islets. It is also produced in the gut
(enterochromaffin cells) and is found in brain tissue. Those substances that stimulate insulin release
will also induce somatostatin secretion. It is an inhibitor of growth hormone, it also delays gastric
emptying and reduces gastrin secretion.
It reduces pancreatic exocrine secretions and may be used therapeutically to treat pancreatic
fistulae.
Somatostatinomas are rare pancreatic endocrine tumours and will result in the clinical
manifestations of diabetes mellitus, gallstones and steatorrhoea.

Next question
A 34 year old lady develops septic shock and features of the systemic inflammatory response
syndrome as a complication of cholangitis. Which of the following is not a typical feature of this
condition?

Body temperature less than 36oC or greater than 38oC

Respiratory rate >20

Lactate <4 mmol/L

High levels of tumour necrosis factor α

WCC >12,000 mm3

Septic shock will typically result in end organ hypoperfusion and as a result lactate levels will often
be high. In the surviving sepsis campaign it is suggested that elevated lactate levels are an
independent indicator for vasopressor support in patient with sepsis. The WCC may be paradoxically
low in severe sepsis, although it is most often elevated.
Please rate this question:

Discuss and give feedback


Next question

Shock

 Shock occurs when there is insufficient tissue perfusion.


 The pathophysiology of shock is an important surgical topic and may be divided into the
following aetiological groups:
 Septic
 Haemorrhagic
 Neurogenic
 Cardiogenic
 Anaphylactic

Septic shock
Septic shock is a major problem and those patients with severe sepsis have a mortality rate in
excess of 40%. In those who are admitted to intensive care mortality ranges from 6% with no organ
failure to 65% in those with 4 organ failure.

Sepsis is defined as an infection that triggers a particular Systemic Inflammatory Response


Syndrome (SIRS). This is characterised by body temperature outside 36 oC - 38 o C, HR >90
beats/min, respiratory rate >20/min, WBC count >12,000/mm 3 or < 4,000/mm 3, altered mental state
or hyperglycaemia (in absence of diabetes).

Patients with infections and two or more elements of SIRS meet the diagnostic criteria for sepsis.
Those with organ failure have severe sepsis and those with refractory hypotension -septic shock.

During the septic process there is marked activation of the immune system with extensive cytokine
release. This may be coupled with or triggered by systemic circulation of bacterial toxins. These all
cause endothelial cell damage and neutrophil adhesion. The overall hallmarks are thus those
ofexcessive inflammation, coagulation and fibrinolytic suppression.

The surviving sepsis campaign (2012) highlights the following key areas for attention:

 Prompt administration of antibiotics to cover all likely pathogens coupled with a rigorous
search for the source of infection.
 Haemodynamic stabilisation. Many patients are hypovolaemic and require aggressive fluid
administration. Aim for CVP 8-12 cm H2O, MAP >65mmHg.
 Modulation of the septic response. This includes manoeuvres to counteract the changes and
includes measures such as tight glycaemic control. The routine use of steroids is not
advised.

In surgical patients, the main groups with septic shock include those with anastomotic leaks,
abscesses and extensive superficial infections such as necrotising fasciitis. When performing
surgery the aim should be to undertake the minimum necessary to restore physiology. These
patients do not fare well with prolonged surgery. Definitive surgery can be more safely undertaken
when physiology is restored and clotting in particular has been normalised.

Haemorrhagic shock
The average adult blood volume comprises 7% of body weight. Thus in the 70 Kg adult this will
equate to 5 litres. This changes in children (8-9% body weight) and is slightly lower in the elderly.

The table below outlines the 4 major classes of haemorrhagic shock and their associated
physiological sequelae:

Parameter Class I Class II Class III Class IV

Blood loss ml <750ml 750-1500ml 1500-2000ml >2000ml

Blood loss % <15% 15-30% 30-40% >40%

Pulse rate <100 >100 >120 >140


Parameter Class I Class II Class III Class IV

Blood pressure Normal Normal Decreased Decreased

Respiratory rate 14-20 20-30 30-40 >35

Urine output >30ml 20-30ml 5-15ml <5ml

Symptoms Normal Anxious Confused Lethargic

Decreasing blood pressure during haemorrhagic shock causes organ hypoperfusion and relative
myocardial ischaemia. The cardiac index gives a numerical value for tissue oxygen delivery and is
given by the equation: Cardiac index= Cardiac output/ body surface area. Where Hb is
haemoglobin concentration in blood and SaO2 the saturation and PaO2 the partial pressure of
oxygen. Detailed knowledge of this equation is required for the MRCS Viva but not for part A,
although you should understand the principle.

In patients suffering from trauma the most likely cause of shock is haemorrhage. However, the
following may also be the cause or occur concomitantly:

 Tension pneumothorax
 Spinal cord injury
 Myocardial contusion
 Cardiac tamponade

When assessing trauma patients it is worth remembering that in order to generate a palpable
femoral pulse an arterial pressure of >65mmHg is required.

Once bleeding is controlled and circulating volume normalised the levels of transfusion should be to
maintain a Hb of 7-8 in those with no risk factors for tissue hypoxia and Hb 10 for those who have
such risk factors.

Neurogenic shock
This occurs most often following a spinal cord transection, usually at a high level. There is
resultant interruption of the autonomic nervous system. The result is either decreased sympathetic
tone or increased parasympathetic tone, the effect of which is a decrease in peripheral vascular
resistance mediated by marked vasodilation.

This results in decreased preload and thus decreased cardiac output (Starlings law). There is
decreased peripheral tissue perfusion and shock is thus produced. In contrast with many other types
of shock peripheral vasoconstrictors are used to return vascular tone to normal.
Cardiogenic shock
In medical patients the main cause is ischaemic heart disease. In the traumatic setting direct
myocardial trauma or contusion is more likely. Evidence of ECG changes and overlying sternal
fractures or contusions should raise the suspicion of injury. Treatment is largely supportive and
transthoracic echocardiography should be used to determine evidence of pericardial fluid or direct
myocardial injury. The measurement of troponin levels in trauma patients may be undertaken but
they are less useful in delineating the extent of myocardial trauma than following MI.

When cardiac injury is of a blunt nature and is associated with cardiogenic shock the right side of the
heart is the most likely site of injury with chamber and or valve rupture. These patients require
surgery to repair these defects and will require cardiopulmonary bypass to achieve this. Some may
require intra aortic balloon pump as a bridge to surgery.

Anaphylactic shock
Anaphylaxis may be defined as a severe, life-threatening, generalised or systemic
hypersensitivity reaction.

Anaphylaxis is one of the few times when you would not have time to look up the dose of a
medication. The Resuscitation Council guidelines on anaphylaxis have recently been updated.
Adrenaline is by far the most important drug in anaphylaxis and should be given as soon as
possible. The recommended doses for adrenaline, hydrocortisone and chlorpheniramine are as
follows:

Adrenaline Hydrocortisone Chlorpheniramine

< 6 months 150 mcg (0.15ml 1 in 1,000) 25 mg 250 mcg/kg

6 months - 6 years 150 mcg (0.15ml 1 in 1,000) 50 mg 2.5 mg

6-12 years 300 mcg (0.3ml 1 in 1,000) 100 mg 5 mg

Adult and child 12 years 500 mcg (0.5ml 1 in 1,000) 200 mg 10 mg

Adrenaline can be repeated every 5 minutes if necessary. The best site for IM injection is the
anterolateral aspect of the middle third of the thigh.

Common identified causes of anaphylaxis

 food (e.g. Nuts) - the most common cause in children


 drugs
 venom (e.g. Wasp sting)

Next question
Which of the following stimulates gastric acid secretion?

Cholecystokinin

Gastric inhibitory peptide

Secretin

Histamine

Somatostatin

Gastrin: From G cells: stimulates gastric acid production

Pepsin: Digestion of protein, secretion occurs simultaneously with gastrin

Secretin: From mucosal cells in the duodenum and jejunum: inhibits gastric acid, stimulates bile and
pancreatic juice production

Gastric inhibitory peptide: (produced in response to fatty acids) inhibits gastrin release and acid
secretion from parietal cells

Cholecystokinin: From mucosal cells in the duodenum and jejunum (produced in response to fatty
acids) inhibits acid secretion from parietal cells, causes gallbladder contraction and relaxation of
sphincter of Oddi

Somatostatin: From D cells

Please rate this question:

Discuss and give feedback

Next question

Hormonal control of gastric acid secretion


Source Stimulus Actions

Gastrin G cells in Distension of Increase HCL, pepsinogen and IF secretion,


antrum of the stomach, extrinsic increases gastric motility, trophic effect on gastric
stomach nerves mucosa
Inhibited by: low
antral pH,
somatostatin

CCK I cells in upper Partially digested Increases secretion of enzyme-rich fluid from
small intestine proteins and pancreas, contraction of gallbladder and
triglycerides relaxation of sphincter of Oddi, decreases gastric
emptying, trophic effect on pancreatic acinar
cells, induces satiety

Secretin S cells in Acidic chyme, fatty Increases secretion of bicarbonate-rich fluid from
upper small acids pancreas and hepatic duct cells, decreases gastric
intestine acid secretion, trophic effect on pancreatic acinar
cells

VIP Small Neural Stimulates secretion by pancreas and intestines,


intestine, inhibits acid and pepsinogen secretion
pancreas

Somatostatin D cells in the Fat, bile salts and Decreases acid and pepsin secretion, decreases
pancreas and glucose in the gastrin secretion, decreases pancreatic enzyme
stomach intestinal lumen secretion, decreases insulin and glucagon
secretion
inhibits trophic effects of gastrin, stimulates
gastric mucous production

Next question
Which of the following statements relating to gastric acid secretions are untrue?

In parietal cells carbonic anhydrase generates hydrogen ions which are then actively
secreted

The cephalic phase is abolished following truncal vagotomy

The intestinal phase accounts for 60% of gastric acid produced

Histamine acts in a paracrine manner on H2 receptors

H2 receptor antagonists will not completely abolish gastric acid production

The intestinal phase of gastric acid secretion accounts for only 10% of gastric acid produced.
Please rate this question:

Discuss and give feedback


Next question

Gastric secretions

A working knowledge of gastric secretions is important for surgery because peptic ulcers are
common, surgeons frequently prescribe anti secretory drugs and because there are still patients
around who will have undergone acid lowering procedures (Vagotomy) in the past.

Gastric acid

 Is produced by the parietal cells in the stomach


 pH of gastric acid is around 2 with acidity being maintained by the H +/K+ ATP ase pump. As
part of the process bicarbonate ions will be secreted into the surrounding vessels.
 Sodium and chloride ions are actively secreted from the parietal cell into the canaliculus.
This sets up a negative potential across the membrane and as a result sodium and
potassium ions diffuse across into the canaliculus.
 Carbonic anhydrase forms carbonic acid which dissociates and the hydrogen ions formed by
dissociation leave the cell via the H+/K+ antiporter pump. At the same time sodium ions are
actively absorbed. This leaves hydrogen and chloride ions in the canaliculus these mix and
are secreted into the lumen of the oxyntic gland.

This is illustrated diagrammatically below:


Image sourced from Wikipedia

Phases of gastric acid secretion


There are 3 phases of gastric secretion:

1. Cephalic phase (smell / taste of food)

 30% acid produced


 Vagal cholinergic stimulation causing secretion of HCL and gastrin release from G cells

2. Gastric phase (distension of stomach )

 60% acid produced


 Stomach distension/low H+/peptides causes Gastrin release
3. Intestinal phase (food in duodenum)

 10% acid produced


 High acidity/distension/hypertonic solutions in the duodenum inhibits gastric acid secretion
via enterogastrones (CCK, secretin) and neural reflexes.

Regulation of gastric acid production


Factors increasing production include:

 Vagal nerve stimulation


 Gastrin release
 Histamine release (indirectly following gastrin release) from enterchromaffin like cells

Factors decreasing production include:

 Somatostatin (inhibits histamine release)


 Cholecystokinin
 Secretin

The diagram below illustrates some of the factors involved in regulating gastric acid secretion and
the relevant associated pharmacology

Image sourced from Wikipedia

Below is a brief summary of the major hormones involved in food digestion:


Source Stimulus Actions

Gastrin G cells in Distension of Increase HCL, pepsinogen and IF secretion,


antrum of the stomach, extrinsic increases gastric motility, trophic effect on
stomach nerves gastric mucosa
Inhibited by: low
antral pH,
somatostatin

CCK I cells in Partially digested Increases secretion of enzyme-rich fluid


upper small proteins and from pancreas, contraction of gallbladder
intestine triglycerides and relaxation of sphincter of Oddi,
decreases gastric emptying, trophic effect on
pancreatic acinar cells, induces satiety

Secretin S cells in Acidic chyme, Increases secretion of bicarbonate-rich fluid


upper small fatty acids from pancreas and hepatic duct cells,
intestine decreases gastric acid secretion, trophic
effect on pancreatic acinar cells

VIP Small Neural Stimulates secretion by pancreas and


intestine, intestines, inhibits acid and pepsinogen
pancreas secretion

Somatostatin D cells in the Fat, bile salts and Decreases acid and pepsin secretion,
pancreas and glucose in the decreases gastrin secretion, decreases
stomach intestinal lumen pancreatic enzyme secretion, decreases
insulin and glucagon secretion
inhibits trophic effects of gastrin, stimulates
gastric mucous production

Next question
A 22 year old man is undergoing a daycase excision of a sebaceous cyst. He is needle phobic and
as the surgeon approaches with the needle the patient begins to hyperventilate. He soon develops
circumoral parasthesia and muscular twitching. Which of the following is the most likely explanation
for this event?

Temporal lobe epilepsy

Reduction in ionised calcium levels

Increase in ionised calcium levels

Fall in serum PTH levels

Rise in serum PTH levels

50% of plasma calcium is ionised. Hyperventilation will induce a state of alkalosis which will lower
ionised plasma calcium levels.
Please rate this question:

Discuss and give feedback


Next question

Calcium homeostasis

Calcium ions are linked to a wide range of physiological processes. The largest store of bodily
calcium is contained within the skeleton. Calcium levels are primarily controlled by parathyroid
hormone, vitamin D and calcitonin.

Hormonal regulation of calcium


Hormone Actions

Parathyroid hormone (PTH)  Increase calcium levels and decrease phosphate


levels
 Increases bone resorption
 Immediate action on osteoblasts to increase ca2+ in
extracellular fluid
 Osteoblasts produce a protein signaling molecule
that activate osteoclasts which cause bone
resorption
Hormone Actions

 Increases renal tubular reabsorption of calcium


 Increases synthesis of 1,25(OH)2D (active form
of vitamin D) in the kidney which increases
bowel absorption of Ca2+
 Decreases renal phosphate reabsorption

1,25-dihydroxycholecalciferol (the  Increases plasma calcium and plasma phosphate


active form of vitamin D)  Increases renal tubular reabsorption and gut
absorption of calcium
 Increases osteoclastic activity
 Increases renal phosphate reabsorption

Calcitonin  Secreted by C cells of thyroid


 Inhibits intestinal calcium absorption
 Inhibits osteoclast activity
 Inhibits renal tubular absorption of calcium

Both growth hormone and thyroxine also play a small role in calcium metabolism.
Next question
Which of the following inhibits gastric acid secretion?

Histamine

Nausea

Calcium

Parasympathetic vagal stimulation

Gastrin

Nausea inhibits gastric secretion via higher cerebral activity and sympathetic innervation.
Please rate this question:

Discuss and give feedback


Next question

Gastric secretions

A working knowledge of gastric secretions is important for surgery because peptic ulcers are
common, surgeons frequently prescribe anti secretory drugs and because there are still patients
around who will have undergone acid lowering procedures (Vagotomy) in the past.

Gastric acid

 Is produced by the parietal cells in the stomach


 pH of gastric acid is around 2 with acidity being maintained by the H +/K+ ATP ase pump. As
part of the process bicarbonate ions will be secreted into the surrounding vessels.
 Sodium and chloride ions are actively secreted from the parietal cell into the canaliculus.
This sets up a negative potential across the membrane and as a result sodium and
potassium ions diffuse across into the canaliculus.
 Carbonic anhydrase forms carbonic acid which dissociates and the hydrogen ions formed by
dissociation leave the cell via the H+/K+ antiporter pump. At the same time sodium ions are
actively absorbed. This leaves hydrogen and chloride ions in the canaliculus these mix and
are secreted into the lumen of the oxyntic gland.

This is illustrated diagrammatically below:


Image sourced from Wikipedia

Phases of gastric acid secretion


There are 3 phases of gastric secretion:

1. Cephalic phase (smell / taste of food)

 30% acid produced


 Vagal cholinergic stimulation causing secretion of HCL and gastrin release from G cells

2. Gastric phase (distension of stomach )

 60% acid produced


 Stomach distension/low H+/peptides causes Gastrin release

3. Intestinal phase (food in duodenum)


 10% acid produced
 High acidity/distension/hypertonic solutions in the duodenum inhibits gastric acid secretion
via enterogastrones (CCK, secretin) and neural reflexes.

Regulation of gastric acid production


Factors increasing production include:

 Vagal nerve stimulation


 Gastrin release
 Histamine release (indirectly following gastrin release) from enterchromaffin like cells

Factors decreasing production include:

 Somatostatin (inhibits histamine release)


 Cholecystokinin
 Secretin

The diagram below illustrates some of the factors involved in regulating gastric acid secretion and
the relevant associated pharmacology

Image sourced from Wikipedia

Below is a brief summary of the major hormones involved in food digestion:

Source Stimulus Actions


Gastrin G cells in Distension of Increase HCL, pepsinogen and IF secretion,
antrum of the stomach, extrinsic increases gastric motility, trophic effect on
stomach nerves gastric mucosa
Inhibited by: low
antral pH,
somatostatin

CCK I cells in Partially digested Increases secretion of enzyme-rich fluid


upper small proteins and from pancreas, contraction of gallbladder
intestine triglycerides and relaxation of sphincter of Oddi,
decreases gastric emptying, trophic effect on
pancreatic acinar cells, induces satiety

Secretin S cells in Acidic chyme, Increases secretion of bicarbonate-rich fluid


upper small fatty acids from pancreas and hepatic duct cells,
intestine decreases gastric acid secretion, trophic
effect on pancreatic acinar cells

VIP Small Neural Stimulates secretion by pancreas and


intestine, intestines, inhibits acid and pepsinogen
pancreas secretion

Somatostatin D cells in the Fat, bile salts and Decreases acid and pepsin secretion,
pancreas and glucose in the decreases gastrin secretion, decreases
stomach intestinal lumen pancreatic enzyme secretion, decreases
insulin and glucagon secretion
inhibits trophic effects of gastrin, stimulates
gastric mucous production

Next question
A 73 year old lady is diagnosed with hyperaldosteronism. From which of the following structures is
aldosterone released?

Zona fasciculata of the adrenal gland

Juxtaglomerular apparatus of the kidney

Zona reticularis of the adrenal gland

Adrenal medulla

Zona glomerulosa of the adrenal cortex

Aldosterone serves to conserve sodium and water. It is produced in the zona glomerulosa of the
adrenal cortex.

Please rate this question:

Discuss and give feedback

Next question

Aldosterone

Aldosterone is secreted by the zona glomerulosa of the adrenal cortex. It is a mineralocorticoid


hormone. Secretion is regulated by the renin- angiotensin system, and by plasma levels of sodium
and potassium. Aldosterone conserves sodium by stimulating the reabsorption of sodium in the
distal nephron in exchange for potassium. Lack of aldosterone release will result in hyperkalaemia
and hyponatraemia.

Next question
Which of the following hormonal agents will increase secretions of water and electrolytes in
pancreatic juice?

Secretin

Aldosterone

Somatostatin

Cholecystokinin

Adrenaline

Secretin causes secretion of water and electrolytes


Cholecystokinin causes enzyme secretion

While secretin will typically increase electrolyte and water volume of secretions, the enzyme content
in increased by cholecystokinin. Somatostatin will decrease the volume of secretions. Aldosterone
will tend to conserve electrolytes.

Please rate this question:

Discuss and give feedback

Next question

Pancreas exocrine physiology

Composition of pancreatic secretions


Pancreatic secretions are usually 1000-1500ml per 24 hours and have a pH of 8.

Secretion Source Substances secreted

Enzymic Acinar cells


Trypsinogen
Procarboxylase
Amylase
Elastase

Sodium
Bicarbonate
Water
Ductal and
Aqueous Potassium
Centroacinar cells
Chloride
NB: Sodium and potassium reflect their plasma levels; chloride
and bicarbonate vary with flow rate

Regulation
The cephalic and gastric phases (neuronal and physical) are less important in regulating the
pancreatic secretions. The effect of digested material in the small bowel stimulates CCK release and
ACh which stimulate acinar and ductal cells. Of these CCK is the most potent stimulus. In the case
of the ductal cells these are potently stimulated by secretin which is released by the S cells of the
duodenum. This results in an increase in bicarbonate.

Enzyme activation
Trypsinogen is converted via enterokinase to active trypsin in the duodenum. Trypsin then activates
the other inactive enzymes

Next question
A 43 year old lady is admitted with cholestasis secondary to a stone impacted at the level of the
ampulla of vater. Which of the following tests is most likely to be predictive of bleeding diathesis at
the time of ERCP in this particular case?

Bleeding time

Prothrombin time

APTT

Platelet count

Factor I levels

PT: Vitamin K dependent factors 2, 7, 9, 10


APTT: Factors 8, 9, 11, 12

Jaundice will impair the production of vitamin K dependent clotting factors. This is most accurately
tested by measuring the prothrombin time. APTT can be affected by vitamin K deficiency (due to
factor 9 deficiency), however this occurs to a lesser extent and is normally associated with severe
liver disease. The bleeding time is a measure of platelet function.

Please rate this question:

Discuss and give feedback

Next question

Abnormal coagulation

Cause Factors affected


Cause Factors affected

Heparin Prevents activation factors 2,9,10,11

Warfarin Affects synthesis of factors 2,7,9,10

DIC Factors 1,2,5,8,11

Liver disease Factors 1,2,5,7,9,10,11

Interpretation blood clotting test results

Disorder APTT PT Bleeding time

Haemophilia Increased Normal Normal

von Willebrand's disease Increased Normal Increased

Vitamin K deficiency Increased Increased Normal

Next question
Which of the following mechanisms best accounts for the release of adrenaline?

Release from the adrenal medulla in response to increased angiotensin 1 levels

Release from the zona fasiculata from the adrenal gland in response to increased
sympathetic discharge

Release from the adrenal medulla in response to increased noradrenaline levels

Release from the adrenal medulla in response to sympathetic stimulation from the
splanchnic nerves

None of the above

The adrenal gland releases adrenaline in response to increased sympathetic discharge from
preganglionic sympathetic fibres of the splanchnic nerves. These cause the chromafin cells of the
medulla to release adrenaline (which is preformed) by exocytosis.
Please rate this question:

Discuss and give feedback


Next question

Adrenaline

 Fight or Flight response

- Catecholamine (phenylalanine and tyrosine)


- Neurotransmitter and hormone
- Released by the adrenal glands
- Effects on α 1 and 2, β 1 and 2 receptors
- Effect on β 2 receptors in skeletal muscle vessels-causing vasodilation
- Increase cardiac output and total peripheral resistance
- Vasoconstriction in the skin and kidneys causing a narrow pulse pressure

Actions
α adrenergic receptors:

 Inhibits insulin secretion by the pancreas


 Stimulates glycogenolysis in the liver and muscle
 Stimulates glycolysis in muscle
β adrenergic receptors:

 Stimulates glucagon secretion in the pancreas


 Stimulates ACTH
 Stimulates lipolysis by adipose tissue

Next question
The acute phase response to injury does not include:

Pyrexia

Decreased albumin

Hepatic sequestration of cations

Increased transferrin

Increased serum amyloid A

The acute phase response includes:

 Acute phase proteins


 Reduction of transport proteins (albumin, transferrin)
 Hepatic sequestration cations
 Pyrexia
 Neutrophil leucocytosis
 Increased muscle proteolysis
 Changes in vascular permeability

Please rate this question:

Discuss and give feedback


Next question

Stress response: Endocrine and metabolic changes

 Surgery precipitates hormonal and metabolic changes causing the stress response.
 Stress response is associated with: substrate mobilization, muscle protein loss, sodium and
water retention, suppression of anabolic hormone secretion, activation of the sympathetic
nervous system, immunological and haematological changes.
 The hypothalamic-pituitary axis and the sympathetic nervous systems are activated and
there is a failure of the normal feedback mechanisms of control of hormone secretion.

A summary of the hormonal changes associated with the stress response:


Increased Decreased No Change

Growth hormone Insulin Thyroid stimulating hormone

Cortisol Testosterone Luteinizing hormone

Renin Oestrogen Follicle stimulating hormone

Adrenocorticotrophic hormone (ACTH)

Aldosterone

Prolactin

Antidiuretic hormone

Glucagon

Sympathetic nervous system

 Stimulates catecholamine release


 Causes tachycardia and hypertension

Pituitary gland

 ACTH and growth hormone (GH) is stimulated by hypothalamic releasing factors,


corticotrophin releasing factor (CRF) and somatotrophin (or growth hormone releasing factor)
 Perioperative increased prolactin secretion occurs by release of inhibitory control
 Secretion of thyroid stimulating hormone (TSH), luteinizing hormone (LH) and follicle
stimulating hormone (FSH) does not change significantly
 ACTH stimulates cortisol production within a few minutes of the start of surgery. More ACTH
is produced than needed to produce a maximum adrenocortical response.

Cortisol

 Significant increases within 4-6 hours of surgery (>1000 nmol litre-1).


 The usual negative feedback mechanism fails and concentrations of ACTH and cortisol
remain persistently increased.
 The magnitude and duration of the increase correlate with the severity of stress and the
response is not abolished by the administration of corticosteroids.
 The metabolic effects of cortisol are enhanced:

Skeletal muscle protein breakdown to provide gluconeogenic precursors and amino acids for protein
synthesis in the liver
Stimulation of lipolysis
'Anti-insulin effect'
Mineralocorticoid effects
Anti-inflammatory effects

Growth hormone

 Increased secretion after surgery has a minor role


 Most important for preventing muscle protein breakdown and promote tissue repair by insulin
growth factors

Alpha Endorphin

 Increased

Antidiuretic hormone

 An important vasopressor and enhances haemostasis


 Renin is released causing the conversion of angiotensinogen to angiotensin I
 Angiotensin II formed by ACE on angiotensin 1, which causes the secretion of aldosterone
from the adrenal cortex. This increases sodium reabsorption at the distal convoluted tubule

Insulin

 Release inhibited by stress


 Occurs via the inhibition of the beta cells in the pancreas by the α2-adrenergic inhibitory
effects of catecholamines
 Insulin resistance by target cells occurs later
 The perioperative period is characterized by a state of functional insulin deficiency

Thyroxine (T4) and tri-iodothyronine (T3)

 Circulating concentrations are inversely correlated with sympathetic activity and after surgery
there is a reduction in thyroid hormone production, which normalises over a few days.

Metabolic effect of endocrine response

Carbohydrate metabolism

 Hyperglycaemia is a main feature of the metabolic response to surgery


 Due to increase in glucose production and a reduction in glucose utilization
 Catecholamines and cortisol promote glycogenolysis and gluconeogenesis
 Initial failure of insulin secretion followed by insulin resistance affects the normal responses
 The proportion of the hyperglycaemic response reflects the severity of surgery
 Hyperglycaemia impairs wound healing and increase infection rates

Protein metabolism

 Initially there is inhibition of protein anabolism, followed later, if the stress response is
severe, by enhanced catabolism
 The amount of protein degradation is influenced by the type of surgery and also by the
nutritional status of the patient
 Mainly skeletal muscle protein is affected
 The amino acids released form acute phase proteins (fibrinogen, C reactive protein,
complement proteins, a2-macroglobulin, amyloid A and ceruloplasmin) and are used for
gluconeogenesis
 Nutritional support has little effect on preventing catabolism

Lipid metabolism
Increased catecholamine, cortisol and glucagon secretion, and insulin deficiency, promotes lipolysis
and ketone body production.

Salt and water metabolism

 ADH causes water retention, concentrated urine, and potassium loss and may continue for 3
to 5 days after surgery
 Renin causes sodium and water retention
Cytokines

 Glycoproteins
 Interleukins (IL) 1 to 17, interferons, and tumour necrosis factor
 Synthesized by activated macrophages, fibroblasts, endothelial and glial cells in response to
tissue injury from surgery or trauma
 IL-6 main cytokine associated with surgery. Peak 12 to 24 h after surgery and increase by
the degree of tissue damage Other effects of cytokines include fever, granulocytosis,
haemostasis, tissue damage limitation and promotion of healing.

Modifying the response

 Opioids suppress hypothalamic and pituitary hormone secretion


 At high doses the hormonal response to pelvic and abdominal surgery is abolished.
However, such doses prolong recovery and increase the need for postoperative ventilatory
support
 Spinal anaesthesia can reduce the glucose, ACTH, cortisol, GH and epinephrine changes,
although cytokine responses are unaltered
 Cytokine release is reduced in less invasive surgery
 Nutrition prevents the adverse effects of the stress response. Enteral feeding improves
recovery
 Growth hormone and anabolic steroids may improve outcome
 Normothermia decreases the metabolic response

References
Deborah Burton, Grainne Nicholson, and George Hall
Endocrine and metabolic response to surgery .

Contin Educ Anaesth Crit Care Pain (2004) 4(5): 144-147 doi:10.1093/bjaceaccp/mkh040
Next question
Which of the following statements relating to blood transfusions in surgical patients is false?

Packed red cells typically have a haematocrit of between 55 and 75%

Clotting factor activity in whole blood decreases in samples stored for longer than 7 days

After 3 weeks of storage blood has a pH of 6.9

Gamma irradiated blood products are not required routinely

Patients should be transfused to achieve a target haemoglobin of 10 g/dl and a haematocrit


of 30%

Patients can generally be managed without transfusion as long as the Hb is 7 or greater. The exact
level depends upon patient factors such as co-morbidities. Old blood functions less effectively and
should not be used during massive transfusions.
Please rate this question:

Discuss and give feedback


Next question

Blood products

Whole blood fractions

Fraction Key points

Packed red cells Used for transfusion in chronic anaemia and cases where infusion of large
volumes of fluid may result in cardiovascular compromise. Product obtained
by centrifugation of whole blood.

Platelet rich Usually administered to patients who are thrombocytopaenic and are bleeding
plasma or require surgery. It is obtained by low speed centrifugation.

Platelet Prepared by high speed centrifugation and administered to patients with


concentrate thrombocytopaenia.

Fresh frozen  Prepared from single units of blood.


plasma  Contains clotting factors, albumin and immunoglobulin.
 Unit is usually 200 to 250ml.
 Usually used in correcting clotting deficiencies in patients with hepatic
synthetic failure who are due to undergo surgery.
 Usual dose is 12-15ml/Kg-1.
 It should not be used as first line therapy for hypovolaemia.

Cryoprecipitate  Formed from supernatant of FFP.


 Rich source of Factor VIII and fibrinogen.
 Allows large concentration of factor VIII to be administered in small
volume.

SAG-Mannitol Removal of all plasma from a blood unit and substitution with:
Blood

 Sodium chloride
 Adenine
 Anhydrous glucose
 Mannitol

Up to 4 units of SAG M Blood may be administered. Thereafter whole blood


is preferred. After 8 units, clotting factors and platelets should be considered.

Cell saver devices


These collect patients own blood lost during surgery and then re-infuse it. There are two main types:

 Those which wash the blood cells prior to re-infusion. These are more expensive to purchase
and more complicated to operate. However, they reduce the risk of re-infusing contaminated
blood back into the patient.
 Those which do not wash the blood prior to re-infusion.

Their main advantage is that they avoid the use of infusion of blood from donors into patients and
this may reduce risk of blood borne infection. It may be acceptable to Jehovah's witnesses. It is
contraindicated in malignant disease for risk of facilitating disease dissemination.

Blood products used in warfarin reversal


In some surgical patients the use of warfarin can pose specific problems and may require the use of
specialised blood products

Immediate or urgent surgery in patients taking warfarin(1) (2):


1. Stop warfarin

2. Vitamin K (reversal within 4-24 hours)


-IV takes 4-6h to work (at least 5mg)
-Oral can take 24 hours to be clinically effective

3. Fresh frozen plasma


Used less commonly now as 1st line warfarin reversal
-30ml/kg-1
-Need to give at least 1L fluid in 70kg person (therefore not appropriate in fluid overload)
-Need blood group
-Only use if human prothrombin complex is not available

4. Human Prothrombin Complex (reversal within 1 hour)


-Bereplex 50 u/kg
-Rapid action but factor 6 short half life, therefore give with vitamin K

References
1. Dentali, F., C. Marchesi, et al. (2011). "Safety of prothrombin complex concentrates for rapid
anticoagulation reversal of vitamin K antagonists. A meta-analysis." Thromb Haemost 106(3): 429-
438.

2. http://www.transfusionguidelines.org/docs/pdfs/bbt-03warfarin-reversal-flowchart-2006.pdf
Next question
Which of the following statements relating the fluid physiology of a physiologically normal 70 Kg
adult male is false?

He will have more water per unit of body weight than a female of similar weight

Plasma will comprise 25% of his body weight

Interstitial fluid will account for up to 24% of body weight

Approximately 65% of total body water is intracellular

60% of his body weight is composed of water

The 60-40-20 rule:


60% total body weight is water
40% of total body weight is intracellular fluids
20% of body weight is extracellular fluids

Plasma typically accounts for 4-6% of body weight in healthy individuals.


Males typically have more water per unit weight than females, as females have a higher fat content.

Please rate this question:

Discuss and give feedback

Next question

Fluid compartment physiology

Body fluid compartments comprise intracellular and extracellular compartments. The latter includes
interstitial fluid, plasma and transcellular fluid.
Typical figures are based on the 70 Kg male.

Body fluid volumes


Compartment Volume in litres Percentage of total volume

Intracellular 28 L 60-65%

Extracellular 14 L 35-40%

Plasma 3L 5%

Interstitial 10 L 24%

Transcellular 1L 3%

Figures are approximate

Next question
A 17 year old lady with long standing anorexia nervosa is due to undergo excision of a lipoma.
Which of the following nutritional deficiencies is most likely to be implicated in poor collagen
formation as the wound heals?

Deficiency of copper

Deficiency of iron

Deficiency of ascorbic acid

Deficiency of phosphate

None of the above

Theme from January 2011

Vitamin C is involved in the cross linkage of collagen and impaired wound healing is well described
in cases of vitamin C deficiency.
Please rate this question:

Discuss and give feedback


Next question

Collagen

Collagen is one of the most important structural proteins within the extracellular matrix, collagen
together with components such as elastin and glycosaminoglycans determine the properties of all
tissues.

 Composed of 3 polypeptide strands that are woven into a helix, usually a combination of
glycine with either proline or hydroxyproline plus another amino acid
 Numerous hydrogen bonds exist within molecule to provide additional strength
 Many sub types but commonest sub type is I (90% of bodily collagen), tissues with increased
levels of flexibility have increased levels of type III collagen
 Vitamin c is important in establishing cross links
 Synthesised by fibroblasts

Collagen Diseases
Disorders of collagen range from relatively common, acquired defects (typically aging), through to
rarer congenital disorders. The latter are exemplified by conditions such as osteogenesis imperfecta
and Ehlers Danlos syndromes.

Osteogenesis imperfecta:
-8 Subtypes
-Defect of type I collagen
-In type I the collagen is normal quality but insufficient quantity
-Type II- poor quantity and quality
-Type III- Collagen poorly formed, normal quantity
-Type IV- Sufficient quantity but poor quality
Patients have bones which fracture easily, loose joint and multiple other defects depending upon
which sub type they suffer from.

Ehlers Danlos:
-Multiple sub types
-Abnormality of types 1 and 3 collagen
-Patients have features of hypermobility.
-Individuals are prone to joint dislocations and pelvic organ prolapse. In addition to many other
diseases related to connective tissue defects.
Next question
A 45 year old man is undergoing a small bowel resection. The anaesthetist decides to administer an
intravenous fluid which is electrolyte rich. Which of the following most closely matches this
requirement?

Dextrose / Saline

Pentastarch

Gelofusine

Hartmans

5% Dextrose with added potassium 20mmol/ L

Theme from April 2012 Exam


Hartmans solution is the most electrolyte rich. However, both pentastarch and gelofusine have more
macromolecules.

Please rate this question:

Discuss and give feedback

Next question

Intra operative fluid management

Composition of commonly used intravenous fluids mmol-1

Na K Cl Bicarbonate Lactate

Plasma 137-147 4-5.5 95-105 22-25 -


0.9% Saline 153 - 153 - -

Dextrose / saline 30.6 - 30.6 - -

Hartmans 130 4 110 - 28

Recommendations for intra operative fluid management


The latest set of NICE guidelines produced in 2013 relating to intravenous fluids did not specifically
address the requirements of intra operative fluid administration. The reason for this is that
administration of fluids in this specific situation does not lend itself to rigid algorithms.
With the introduction of enhanced recovery programmes 10 years ago there was an increasing
emphasis of the concept of fluid restriction. Historically, patients received very large volumes of
saline rich solutions peri-operatively. Clearing the sodium load of a single litre of saline may take up
to 36 hours or more. This can have deleterious effects on the tissues including the development of
oedema. This results in poor perfusion, increased risk of ileus and wound breakdown. A tailored
approach to fluid administration is now practiced and far greater usage is made of cardiac output
monitors in providing goal directed fluid therapy.

References
British Consensus Guidelines on Intravenous Fluid Therapy for Adult Surgical Patients
GIFTASUP (2009) Revised May 2011.

Frost P. Intravenous fluid therapy in adult inpatients. BMJ 2015 (350): 31-34.

Next question
A 16 year old girl develops pyelonephritis and is admitted in a state of septic shock. Which of the
following is not typically seen in this condition?

Increased cardiac output

Increased systemic vascular resistance

Oliguria may occur

Systemic cytokine release

Tachycardia

Cardiogenic Shock:
e.g. MI, valve abnormality

increased SVR (vasoconstriction in response to low BP)


increased HR (sympathetic response)
decreased cardiac output
decreased blood pressure

Hypovolaemic shock:
blood volume depletion
e.g. haemorrhage, vomiting, diarrhoea, dehydration, third-space losses during major operations

increased SVR
increased HR
decreased cardiac output
decreased blood pressure

Septic shock:
occurs when the peripheral vascular dilatation causes a fall in SVR
similar response may occur in anaphylactic shock, neurogenic shock

reduced SVR
increased HR
normal/increased cardiac output
decreased blood pressure

Theme from January 2012 Exam


The SVR is reduced in sepsis and for this reason a vasopressor such as noradrenaline may be used
if hypotension and oliguria remain a concern despite administration of adequate amounts of
intravenous fluids.
Please rate this question:
Discuss and give feedback
Next question

Shock

 Shock occurs when there is insufficient tissue perfusion.


 The pathophysiology of shock is an important surgical topic and may be divided into the
following aetiological groups:
 Septic
 Haemorrhagic
 Neurogenic
 Cardiogenic
 Anaphylactic

Septic shock
Septic shock is a major problem and those patients with severe sepsis have a mortality rate in
excess of 40%. In those who are admitted to intensive care mortality ranges from 6% with no organ
failure to 65% in those with 4 organ failure.

Sepsis is defined as an infection that triggers a particular Systemic Inflammatory Response


Syndrome (SIRS). This is characterised by body temperature outside 36 oC - 38 o C, HR >90
beats/min, respiratory rate >20/min, WBC count >12,000/mm 3 or < 4,000/mm 3, altered mental state
or hyperglycaemia (in absence of diabetes).

Patients with infections and two or more elements of SIRS meet the diagnostic criteria for sepsis.
Those with organ failure have severe sepsis and those with refractory hypotension -septic shock.

During the septic process there is marked activation of the immune system with extensive cytokine
release. This may be coupled with or triggered by systemic circulation of bacterial toxins. These all
cause endothelial cell damage and neutrophil adhesion. The overall hallmarks are thus those
ofexcessive inflammation, coagulation and fibrinolytic suppression.

The surviving sepsis campaign (2012) highlights the following key areas for attention:

 Prompt administration of antibiotics to cover all likely pathogens coupled with a rigorous
search for the source of infection.
 Haemodynamic stabilisation. Many patients are hypovolaemic and require aggressive fluid
administration. Aim for CVP 8-12 cm H2O, MAP >65mmHg.
 Modulation of the septic response. This includes manoeuvres to counteract the changes and
includes measures such as tight glycaemic control. The routine use of steroids is not
advised.

In surgical patients, the main groups with septic shock include those with anastomotic leaks,
abscesses and extensive superficial infections such as necrotising fasciitis. When performing
surgery the aim should be to undertake the minimum necessary to restore physiology. These
patients do not fare well with prolonged surgery. Definitive surgery can be more safely undertaken
when physiology is restored and clotting in particular has been normalised.

Haemorrhagic shock
The average adult blood volume comprises 7% of body weight. Thus in the 70 Kg adult this will
equate to 5 litres. This changes in children (8-9% body weight) and is slightly lower in the elderly.

The table below outlines the 4 major classes of haemorrhagic shock and their associated
physiological sequelae:

Parameter Class I Class II Class III Class IV

Blood loss ml <750ml 750-1500ml 1500-2000ml >2000ml

Blood loss % <15% 15-30% 30-40% >40%

Pulse rate <100 >100 >120 >140

Blood pressure Normal Normal Decreased Decreased

Respiratory rate 14-20 20-30 30-40 >35

Urine output >30ml 20-30ml 5-15ml <5ml

Symptoms Normal Anxious Confused Lethargic

Decreasing blood pressure during haemorrhagic shock causes organ hypoperfusion and relative
myocardial ischaemia. The cardiac index gives a numerical value for tissue oxygen delivery and is
given by the equation: Cardiac index= Cardiac output/ body surface area. Where Hb is
haemoglobin concentration in blood and SaO2 the saturation and PaO2 the partial pressure of
oxygen. Detailed knowledge of this equation is required for the MRCS Viva but not for part A,
although you should understand the principle.

In patients suffering from trauma the most likely cause of shock is haemorrhage. However, the
following may also be the cause or occur concomitantly:

 Tension pneumothorax
 Spinal cord injury
 Myocardial contusion
 Cardiac tamponade

When assessing trauma patients it is worth remembering that in order to generate a palpable
femoral pulse an arterial pressure of >65mmHg is required.

Once bleeding is controlled and circulating volume normalised the levels of transfusion should be to
maintain a Hb of 7-8 in those with no risk factors for tissue hypoxia and Hb 10 for those who have
such risk factors.

Neurogenic shock
This occurs most often following a spinal cord transection, usually at a high level. There is
resultant interruption of the autonomic nervous system. The result is either decreased sympathetic
tone or increased parasympathetic tone, the effect of which is a decrease in peripheral vascular
resistance mediated by marked vasodilation.

This results in decreased preload and thus decreased cardiac output (Starlings law). There is
decreased peripheral tissue perfusion and shock is thus produced. In contrast with many other types
of shock peripheral vasoconstrictors are used to return vascular tone to normal.

Cardiogenic shock
In medical patients the main cause is ischaemic heart disease. In the traumatic setting direct
myocardial trauma or contusion is more likely. Evidence of ECG changes and overlying sternal
fractures or contusions should raise the suspicion of injury. Treatment is largely supportive and
transthoracic echocardiography should be used to determine evidence of pericardial fluid or direct
myocardial injury. The measurement of troponin levels in trauma patients may be undertaken but
they are less useful in delineating the extent of myocardial trauma than following MI.

When cardiac injury is of a blunt nature and is associated with cardiogenic shock the right side of the
heart is the most likely site of injury with chamber and or valve rupture. These patients require
surgery to repair these defects and will require cardiopulmonary bypass to achieve this. Some may
require intra aortic balloon pump as a bridge to surgery.

Anaphylactic shock
Anaphylaxis may be defined as a severe, life-threatening, generalised or systemic
hypersensitivity reaction.

Anaphylaxis is one of the few times when you would not have time to look up the dose of a
medication. The Resuscitation Council guidelines on anaphylaxis have recently been updated.
Adrenaline is by far the most important drug in anaphylaxis and should be given as soon as
possible. The recommended doses for adrenaline, hydrocortisone and chlorpheniramine are as
follows:

Adrenaline Hydrocortisone Chlorpheniramine

< 6 months 150 mcg (0.15ml 1 in 1,000) 25 mg 250 mcg/kg


6 months - 6 years 150 mcg (0.15ml 1 in 1,000) 50 mg 2.5 mg

6-12 years 300 mcg (0.3ml 1 in 1,000) 100 mg 5 mg

Adult and child 12 years 500 mcg (0.5ml 1 in 1,000) 200 mg 10 mg

Adrenaline can be repeated every 5 minutes if necessary. The best site for IM injection is the
anterolateral aspect of the middle third of the thigh.

Common identified causes of anaphylaxis

 food (e.g. Nuts) - the most common cause in children


 drugs
 venom (e.g. Wasp sting)

Next question
Which of the following statements are not typically true in hypokalaemia?

It may occur as a result of mechanical bowel preparation

Chronic vomiting may increase renal potassium losses

It may be associated with aciduria

It may cause hyponatraemia

It often accompanies acidosis

Potassium depletion occurs either through the gastrointestinal tract or the kidney. Chronic vomiting
in itself is less prone to induce potassium loss than diarrhoea as gastric secretions contain less
potassium than those in the lower GI tract. If vomiting produces a metabolic alkalosis then renal
potassium wasting may occur as potassium is excreted in preference to hydrogen ions. The
converse may occur in potassium depletion resulting in acid urine.

Hypokalemia is very commonly associated with metabolic alkalosis. This is due to 2 factors: 1) the
common causes of metabolic alkalosis (vomiting, diuretics) directly induce H+ and K loss (via
aldosterone) and thus also cause hypokalemia and 2) hypokalemia is a very important cause of
metabolic alkalosis by three mechanisms. The initial effect is by causing a transcellular shift in which
K leaves and H+ enters the cells, thereby raising the extracellular pH. The second effect is by
causing a transcellular shift in the cells of the proximal tubules resulting in an intracellular acidosis,
which promotes ammonium production and excretion. Thirdly, in the presence of hypokalemia,
hydrogen secretion in the proximal and distal tubules increases. This leads to further reabsorption of
HCO3-. The net effect is an increase in the net acid excretion.
Please rate this question:

Discuss and give feedback


Next question

Hypokalaemia

Potassium and hydrogen can be thought of as competitors. Hyperkalaemia tends to be associated


with acidosis because as potassium levels rise fewer hydrogen ions can enter the cells

Hypokalaemia with alkalosis

 Vomiting
 Diuretics
 Cushing's syndrome
 Conn's syndrome (primary hyperaldosteronism)

Hypokalaemia with acidosis

 Diarrhoea
 Renal tubular acidosis
 Acetazolamide
 Partially treated diabetic ketoacidosis

Next question
A man is admitted after a period of prolonged self, induced starvation. Naso gastric feeding is
planned. Which of the following is least likely to occur?

Hypokalaemia

Increased risk of cardiac arrhythmias

His haemoglobin will have decreased affinity for oxygen

Hypophosphataemia

Hypoalbuminaemia

The process of starvation may lower DPG levels, in practice this is unlikely to occur early as it is
generated during glycolysis. Altered metabolism in starvation may be more acidotic and this would
also tend to impair oxygen carriage.
Please rate this question:

Discuss and give feedback


Next question

Nutrition - Refeeding syndrome

Refeeding syndrome describes the metabolic abnormalities which occur on feeding a person
following a period of starvation. The metabolic consequences include:

 Hypophosphataemia
 Hypokalaemia
 Hypomagnesaemia
 Abnormal fluid balance

These abnormalities can lead to organ failure.

Re-feeding problems
If patient not eaten for > 5 days, aim to re-feed at < 50% energy and protein levels

High risk for re-feeding problems


If one or more of the following:

 BMI < 16 kg/m 2


 Unintentional weight loss >15% over 3-6 months
 Little nutritional intake > 10 days
 Hypokalaemia, Hypophosphataemia or hypomagnesaemia prior to feeding (unless high)

If two or more of the following:

 BMI < 18.5 kg/m2


 Unintentional weight loss > 10% over 3-6 months
 Little nutritional intake > 5 days
 History of: alcohol abuse, drug therapy including insulin, chemotherapy, diuretics and
antacids

Prescription

 Start at up to 10 kcal/kg/day increasing to full needs over 4-7 days


 Start immediately before and during feeding: oral thiamine 200-300mg/day, vitamin B co
strong 1 tds and supplements
 Give K+ (2-4 mmol/kg/day), phosphate (0.3-0.6 mmol/kg/day), magnesium (0.2-0.4
mmol/kg/day)

Next question

v
Where does spironolactone act in the kidney?

Glomerulus

Proximal convoluted tubule

Descending limb of the loop of Henle

Ascending limb of the loop of Henle

Distal convoluted tubule

Please rate this question:

Discuss and give feedback


Next question

Potassium sparing diuretics

Potassium-sparing diuretics may be divided into the epithelial sodium channel blockers (amiloride
and triamterene) and aldosterone antagonists (spironolactone and eplerenone).

{Amiloride} is a weak diuretic which blocks the epithelial sodium channel in the distal convoluted
tubule.

Usually given with thiazides or loop diuretics as an alternative to potassium supplementation.

{Spironolactone} is an aldosterone antagonist which acts act in the distal convoluted tubule.

Indications

 ascites: patients with cirrhosis develop a secondary hyperaldosteronism. Relatively large


doses such as 100 or 200mg are often used
 heart failure
 nephrotic syndrome
 Conn's syndrome

Next question
Which receptor does noradrenaline mainly bind to?

α 1 receptors

α 2 receptors

β 1 receptors

β 2 receptors

G receptors

Theme from September 2015 Exam


Noradrenaline is the precursor of adrenaline. It is a powerful α 1 stimulant (although it will increase
myocardial contractility). Infusions will produce vasoconstriction and an increase in total peripheral
resistance. It is the inotrope of choice in septic shock.

Please rate this question:

Discuss and give feedback

Next question

Inotropes and cardiovascular receptors

Inotropes are a class of drugs which work primarily by increasing cardiac output. They should be
distinguished from vasoconstrictor drugs which are used specifically when the primary problem is
peripheral vasodilatation.

Catecholamine type agents are commonly used and work by increasing cAMP levels by adenylate
cyclase stimulation. This in turn intracellular calcium ion mobilisation and thus the force of
contraction. Adrenaline works as a beta adrenergic receptor agonist at lower doses and an alpha
receptor agonist at higher doses. Dopamine causes dopamine receptor mediated renal and
mesenteric vascular dilatation and beta 1 receptor agonism at higher doses. This results in
increased cardiac output. Since both heart rate and blood pressure are raised, there is less overall
myocardial ischaemia. Dobutamine is a predominantly beta 1 receptor agonist with weak beta 2 and
alpha receptor agonist properties. Noradrenaline is a catecholamine type agent and predominantly
acts as an alpha receptor agonist and serves as a peripheral vasoconstrictor.

Phosphodiesterase inhibitors such as milrinone act specifically on the cardiac phosphodiesterase


and increase cardiac output.

Inotrope Cardiovascular receptor action

Adrenaline α-1, α-2, β-1, β-2

Noradrenaline α-1,( α-2), (β-1), (β-2)

Dobutamine β-1, (β 2)

Dopamine (α-1), (α-2), (β-1), D-1,D-2

Minor receptor effects in brackets

Effects of receptor binding

α-1, α-2 vasoconstriction

β-1 increased cardiac contractility and HR

β-2 vasodilatation

D-1 renal and spleen vasodilatation

D-2 inhibits release of noradrenaline

Next question
Adult lung volumes. Which statement is false?

In restrictive lung disease the FEV1/FVC ratio is increased

Residual volume is increased in emphysema

Functional residual capacity is measured by helium dilution test

The tidal volume is approximately 340mls in females

The vital capacity is increased in Guillain Barre syndrome

FEV1/FVC is normal or >80% in restrictive lung disease such as pulmonary fibrosis. The ratio is
reduced in obstructive airways disease.

The functional residual capacity, residual volume and the total lung capacity cannot be measured
with spirometry. They can only be measured by helium dilution.
The vital capacity is reduced in:
1. Pulmonary fibrosis/infiltration/oedema/effusions
2. Weak respiratory muscles e.g. MG, GBS, myopathies
3. Skeletal abnormalities e.g. chest wall abnormalities
Please rate this question:

Discuss and give feedback


Next question

Lung volumes

The diagram demonstrates lung volumes and capacities


Image sourced from Wikipedia

Definitions

Tidal volume (TV)  Is the volume of air inspired and expired during each
ventilatory cycle at rest.
 It is normally 500mls in males and 340mls in females.

Inspiratory reserve  Is the maximum volume of air that can be forcibly inhaled
volume (IRV) following a normal inspiration. 3000mls.

Expiratory reserve  Is the maximum volume of air that can be forcibly exhaled
volume (ERV) following a normal expiration. 1000mls.

Residual volume (RV)  Is that volume of air remaining in the lungs after a maximal
expiration.
 RV = FRC - ERV. 1500mls.

Functional residual  Is the volume of air remaining in the lungs at the end of a
capacity (FRC) normal expiration.
 FRC = RV + ERV. 2500mls.

Vital capacity (VC)  Is the maximal volume of air that can be forcibly exhaled after
a maximal inspiration.
 VC = TV + IRV + ERV. 4500mls in males, 3500mls in
females.
Total lung capacity  Is the volume of air in the lungs at the end of a maximal
(TLC) inspiration.
 TLC = FRC + TV + IRV = VC + RV. 5500-6000mls.

Forced vital capacity  The volume of air that can be maximally forcefully exhaled.
(FVC)
Next question
A 47 year old lady is diagnosed as suffering from a phaeochromocytoma. From which of the
following amino acids are catecholamines primarily derived?

Aspartime

Glutamine

Arginine

Tyrosine

Alanine

Catecholamine hormones are derived from tyrosine, it is modified by a DOPA decarboxylase


enzyme to become dopamine and thereafter via two further enzymic modifications to noradrenaline
and finally adrenaline.

Please rate this question:

Discuss and give feedback

Next question

Adrenal physiology

Adrenal medulla
The chromaffin cells of the adrenal medulla secrete the catecholamines noradrenaline and
adrenaline. The medulla is innervated by the splanchnic nerves; the preganglionic sympathetic fibres
secrete acetylcholine causing the chromaffin cells to secrete their contents by exocytosis.
Phaeochromocytomas are derived from these cells and will secrete both adrenaline and nor
adrenaline.

Adrenal cortex
Three histologically distinct zones are recognised:
Zone Location Hormone Secreted

Zona glomerulosa Outer zone Aldosterone

Zona fasiculata Middle zone Glucocorticoids

Zona reticularis Inner zone Androgens

The glucocorticoids and aldosterone are mostly bound to plasma proteins in the circulation.
Glucocorticoids are inactivated and excreted by the liver.

Next question
Theme: Events in wound healing

A. Platelets
B. Neutrophil polymorphs
C. Endothelial cells
D. Lymphocytes
E. Myofibroblasts
F. Granulomas

Please select the dominant cell type present in a wound at the specified timeframe. Each option may
be used once, more than once or not at all.

132. 10 weeks following injury

You answered Platelets

The correct answer is Myofibroblasts

These differentiated cell types facilitate wound contraction.

133. 25 seconds following injury

You answered Neutrophil polymorphs

The correct answer is Platelets

Platelet degranulation and haemostasis are some of the earliest events in wound healing.

134. 7 days following injury

You answered Neutrophil polymorphs

The correct answer is Endothelial cells

Angiogenesis which is characterised by endothelial cell proliferation and microvessel


formation is a key step in successful wound healing. It is maximal between 1 and 2 weeks
following injury and wounds may have a reddish appearance during this time.

Please rate this question:

Discuss and give feedback


Next question
Phases of wound healing

Phase Key features Cells Timeframe

Haemostasis  Vasospasm in adjacent vessels Erythrocytes and Seconds/


 Platelet plug formation and platelets Minutes
generation of fibrin rich clot

Inflammation  Neutrophils migrate into wound Neutrophils, Days


(function impaired in diabetes). fibroblasts and
 Growth factors released, including macrophages
basic fibroblast growth factor and
vascular endothelial growth factor.
 Fibroblasts replicate within the
adjacent matrix and migrate into
wound.
 Macrophages and fibroblasts
couple matrix regeneration and clot
substitution.

Regeneration  Platelet derived growth factor and Fibroblasts, Weeks


transformation growth factors endothelial cells,
stimulate fibroblasts and epithelial macrophages
cells.
 Fibroblasts produce a collagen
network.
 Angiogenesis occurs and wound
resembles granulation tissue.

Remodelling  Longest phase of the healing Myofibroblasts 6 weeks to 1


process and may last up to one year year
(or longer).
 During this phase fibroblasts
become differentiated
(myofibroblasts) and these
facilitate wound contraction.
 Collagen fibres are remodelled.
 Microvessels regress leaving a pale
scar.

Next question
Where are the arterial baroreceptors located?

Carotid sinus and aortic arch

Carotid sinus only

Superior vena cava

External carotid artery

None of the above

Theme from September 2015 Exam


They lie in the carotid sinus and aortic arch.
Please rate this question:

Discuss and give feedback


Next question

Cardiac physiology

 The heart has four chambers ejecting blood into both low pressure and high pressure
systems.
 The pumps generate pressures of between 0-25mmHg on the right side and 0-120 mmHg on
the left.
 At rest diastole comprises 2/3 of the cardiac cycle.
 The product of the frequency of heart rate and stroke volume combine to give the cardiac
output which is typically 5-6L per minute.

Detailed descriptions of the various waveforms are often not a feature of MRCS A (although they are
on the syllabus). However, they are a very popular topic for surgical physiology in the MRCS B
exam.

Electrical properties

 Intrinsic myogenic rhythm within cardiac myocytes means that even the denervated heart is
capable of contraction.
 In the normal situation the cardiac impulse is generated in the sino atrial node in the right
atrium and conveyed to the ventricles via the atrioventricular node.
 The sino atrial node is also capable of spontaneous discharge and in the absence of
background vagal tone will typically discharge around 100x per minute. Hence the higher
resting heart rate found in cardiac transplant cases. In the SA and AV nodes the resting
membrane potential is lower than in surrounding cardiac cells and will slowly depolarise from
-70mV to around -50mV at which point an action potential is generated.
 Differences in the depolarisation slopes between SA and AV nodes help to explain why the
SA node will depolarise first. The cells have a refractory period during which they cannot be
re-stimulated and this period allows for adequate ventricular filling. In pathological
tachycardic states this time period is overridden and inadequate ventricular filling may then
occur, cardiac output falls and syncope may ensue.

Parasympathetic fibres project to the heart via the vagus and will release acetylcholine. Sympathetic
fibres release nor adrenaline and circulating adrenaline comes from the adrenal medulla.
Noradrenaline binds to β 1 receptors in the SA node and increases the rate of pacemaker potential
depolarisation.

Cardiac cycle

Image sourced from Wikipedia

 Mid diastole: AV valves open. Ventricles hold 80% of final volume. Outflow valves shut.
Aortic pressure is high.

 Late diastole: Atria contract. Ventricles receive 20% to complete filling. Typical end diastolic
volume 130-160ml.
 Early systole: AV valves shut. Ventricular pressure rises. Isovolumetric ventricular
contraction. AV Valves bulge into atria (c-wave). Aortic and pulmonary pressure exceeded-
blood is ejected. Shortening of ventricles pulls atria downwards and drops intra atrial
pressure (x-descent).

 Late systole: Ventricular muscles relax and ventricular pressures drop. Although ventricular
pressure drops the aortic pressure remains constant owing to peripheral vascular resistance
and elastic property of the aorta. Brief period of retrograde flow that occurs in aortic recoil
shuts the aortic valve. Ventricles will contain 60ml end systolic volume. The average stroke
volume is 70ml (i.e. Volume ejected).

 Early diastole: All valves are closed. Isovolumetric ventricular relaxation occurs. Pressure
wave associated with closure of the aortic valve increases aortic pressure. The pressure dip
before this rise can be seen on arterial waveforms and is called the incisura. During systole
the atrial pressure increases such that it is now above zero (v- wave). Eventually atrial
pressure exceed ventricular pressure and AV valves open - atria empty passively into
ventricles and atrial pressure falls (y -descent )

The negative atrial pressures are of clinical importance as they can allow air embolization to occur if
the neck veins are exposed to air. This patient positioning is important in head and neck surgery to
avoid this occurrence if veins are inadvertently cut, or during CVP line insertion.

Mechanical properties

 Preload = end diastolic volume


 Afterload = aortic pressure

It is important to understand the principles of Laplace's law in surgery.

 It states that for hollow organs with a circular cross section, the total circumferential wall
tension depends upon the circumference of the wall, multiplied by the thickness of the wall
and on the wall tension.
 The total luminal pressure depends upon the cross sectional area of the lumen and the
transmural pressure. Transmural pressure is the internal pressure minus external pressure
and at equilibrium the total pressure must counterbalance each other.
 In terms of cardiac physiology the law explains that the rise in ventricular pressure that
occurs during the ejection phase is due to physical change in heart size. It also explains why
a dilated diseased heart will have impaired systolic function.
Starlings law

 Increase in end diastolic volume will produce larger stroke volume.


 This occurs up to a point beyond which cardiac fibres are excessively stretched and stroke
volume will fall once more. It is important for the regulation of cardiac output in cardiac
transplant patients who need to increase their cardiac output.

Baroreceptor reflexes

 Baroreceptors located in aortic arch and carotid sinus.


 Aortic baroreceptor impulses travel via the vagus and from the carotid via the
glossopharyngeal nerve.
 They are stimulated by arterial stretch.
 Even at normal blood pressures they are tonically active.
 Increase in baroreceptor discharge causes:

*Increased parasympathetic discharge to the SA node.


*Decreased sympathetic discharge to ventricular muscle causing decreased contractility and fall in
stroke volume.
*Decreased sympathetic discharge to venous system causing increased compliance.
*Decreased peripheral arterial vascular resistance

Atrial stretch receptors

 Located in atria at junction between pulmonary veins and vena cava.


 Stimulated by atrial stretch and are thus low pressure sensors.
 Increased blood volume will cause increased parasympathetic activity.
 Very rapid infusion of blood will result in increase in heart rate mediated via atrial receptors:
theBainbridge reflex.
 Decreases in receptor stimulation results in increased sympathetic activity this will decrease
renal blood flow-decreases GFR-decreases urinary sodium excretion-renin secretion by
juxtaglomerular apparatus-Increase in angiotensin II.
 Increased atrial stretch will also result in increased release of atrial natriuretic peptide.

Next question
Which one of the following cells secretes the majority of tumour necrosis factor in humans?

Neutrophils

Macrophages

Natural killer cells

Killer-T cells

Helper-T cells

Please rate this question:

Discuss and give feedback


Next question

Tumour necrosis factor

Tumour necrosis factor (TNF) is a pro-inflammatory cytokine with multiple roles in the immune
system

TNF is secreted mainly by macrophages and has a number of effects on the immune system, acting
mainly in a paracrine fashion:

 activates macrophages and neutrophils


 acts as costimulator for T cell activation
 key mediator of bodies response to Gram negative septicaemia
 similar properties to IL-1
 anti-tumour effect (e.g. phospholipase activation)

TNF-alpha binds to both the p55 and p75 receptor. These receptors can induce apoptosis. It also
cause activation of NFkB

Endothelial effects include increase expression of selectins and increased production of platelet
activating factor, IL-1 and prostaglandins

TNF promotes the proliferation of fibroblasts and their production of protease and collagenase. It is
thought fragments of receptors act as binding points in serum
Systemic effects include pyrexia, increased acute phase proteins and disordered metabolism leading
to cachexia

TNF is important in the pathogenesis of rheumatoid arthritis - TNF blockers (e.g. infliximab,
etanercept) are now licensed for treatment of severe rheumatoid

Next question
Which of the following is responsible for the rapid depolarisation phase of the myocardial action
potential?

Rapid sodium influx

Rapid sodium efflux

Slow efflux of calcium

Efflux of potassium

Rapid calcium influx

Please rate this question:

Discuss and give feedback

Next question

Electrical activity of the heart

Myocardial action potential

Phase Description Mechanism

0 Rapid depolarisation Rapid sodium influx


These channels automatically deactivate after a few ms

1 Early repolarisation Efflux of potassium


Phase Description Mechanism

2 Plateau Slow influx of calcium

3 Final repolarisation Efflux of potassium

4 Restoration of ionic Resting potential is restored by Na+/K+ ATPase


concentrations There is slow entry of Na+ into the cell decreasing the potential
difference until the threshold potential is reached, triggering a new
action potential

NB cardiac muscle remains contracted 10-15 times longer than skeletal muscle

Conduction velocity

Atrial conduction Spreads along ordinary atrial myocardial fibres at 1 m/sec

AV node 0.05 m/sec


conduction

Ventricular Purkinje fibres are of large diameter and achieve velocities of 2-4 m/sec (this allows a
conduction rapid and coordinated contraction of the ventricles

Next question
Which of the following is not a feature of normal cerebrospinal fluid?

It has a pressure of between 10 and 15 mmHg.

It usually contains a small amount of glucose.

It may normally contain up to 5 red blood cells per mm3.

It may normally contain up to 3 white blood cells per mm3.

None of the above

It should not contain red blood cells.


Please rate this question:

Discuss and give feedback


Next question

Cerebrospinal fluid

The CSF fills the space between the arachnoid mater and pia mater (covering surface of the brain).
The total volume of CSF in the brain is approximately 150ml. Approximately 500 ml is produced by
the ependymal cells in the choroid plexus (70%), or blood vessels (30%). It is reabsorbed via the
arachnoid granulations which project into the venous sinuses.

Circulation
1. Lateral ventricles (via foramen of Munro)
2. 3rd ventricle
3. Cerebral aqueduct (aqueduct of Sylvius)
4. 4th ventricle (via foramina of Magendie and Luschka)
5. Subarachnoid space
6. Reabsorbed into the venous system via arachnoid granulations into superior sagittal sinus

Composition

 Glucose: 50-80mg/dl
 Protein: 15-40 mg/dl
 Red blood cells: Nil
 White blood cells: 0-3 cells/ mm3

Next question
Which of the following is not an effect of somatostatin?

It stimulates pancreatic acinar cells to release lipase

It decreases gastric acid secretion

It deceases gastrin release

It decreases pepsin secretion

It decreases glucagon release

It inhibits pancreatic enzyme secretion.


Please rate this question:

Discuss and give feedback

Gastric secretions

A working knowledge of gastric secretions is important for surgery because peptic ulcers are
common, surgeons frequently prescribe anti secretory drugs and because there are still patients
around who will have undergone acid lowering procedures (Vagotomy) in the past.

Gastric acid

 Is produced by the parietal cells in the stomach


 pH of gastric acid is around 2 with acidity being maintained by the H +/K+ ATP ase pump. As
part of the process bicarbonate ions will be secreted into the surrounding vessels.
 Sodium and chloride ions are actively secreted from the parietal cell into the canaliculus.
This sets up a negative potential across the membrane and as a result sodium and
potassium ions diffuse across into the canaliculus.
 Carbonic anhydrase forms carbonic acid which dissociates and the hydrogen ions formed by
dissociation leave the cell via the H+/K+ antiporter pump. At the same time sodium ions are
actively absorbed. This leaves hydrogen and chloride ions in the canaliculus these mix and
are secreted into the lumen of the oxyntic gland.

This is illustrated diagrammatically below:


Image sourced from Wikipedia

Phases of gastric acid secretion


There are 3 phases of gastric secretion:

1. Cephalic phase (smell / taste of food)

 30% acid produced


 Vagal cholinergic stimulation causing secretion of HCL and gastrin release from G cells

2. Gastric phase (distension of stomach )

 60% acid produced


 Stomach distension/low H+/peptides causes Gastrin release

3. Intestinal phase (food in duodenum)


 10% acid produced
 High acidity/distension/hypertonic solutions in the duodenum inhibits gastric acid secretion
via enterogastrones (CCK, secretin) and neural reflexes.

Regulation of gastric acid production


Factors increasing production include:

 Vagal nerve stimulation


 Gastrin release
 Histamine release (indirectly following gastrin release) from enterchromaffin like cells

Factors decreasing production include:

 Somatostatin (inhibits histamine release)


 Cholecystokinin
 Secretin

The diagram below illustrates some of the factors involved in regulating gastric acid secretion and
the relevant associated pharmacology

Image sourced from Wikipedia

Below is a brief summary of the major hormones involved in food digestion:

Source Stimulus Actions


Gastrin G cells in Distension of Increase HCL, pepsinogen and IF secretion,
antrum of the stomach, extrinsic increases gastric motility, trophic effect on
stomach nerves gastric mucosa
Inhibited by: low
antral pH,
somatostatin

CCK I cells in Partially digested Increases secretion of enzyme-rich fluid


upper small proteins and from pancreas, contraction of gallbladder
intestine triglycerides and relaxation of sphincter of Oddi,
decreases gastric emptying, trophic effect on
pancreatic acinar cells, induces satiety

Secretin S cells in Acidic chyme, Increases secretion of bicarbonate-rich fluid


upper small fatty acids from pancreas and hepatic duct cells,
intestine decreases gastric acid secretion, trophic
effect on pancreatic acinar cells

VIP Small Neural Stimulates secretion by pancreas and


intestine, intestines, inhibits acid and pepsinogen
pancreas secretion

Somatostatin D cells in the Fat, bile salts and Decreases acid and pepsin secretion,
pancreas and glucose in the decreases gastrin secretion, decreases
stomach intestinal lumen pancreatic enzyme secretion, decreases
insulin and glucagon secretion
inhibits trophic effects of gastrin, stimulates
gastric mucous production
Question 1 of 347

A 40 year old women presented with a mass on her forehead. On examination, she had a fluctuant
pulsatile mass on her head. Examination of her neck revealed a mass inferior to the hyoid with a
positive Berry's sign. What is the most likely underlying diagnosis?

Follicular thyroid cancer

Medullary thyroid cancer

Papillary thyroid cancer

Anaplastic thyroid cancer

Parathyroid gland cancer

Berrys sign= Absence of carotid pulse due to malignant thyromegaly.

Theme from April 2013 Exam


Papillary thyroid cancers will tend to spread via lymphatics and present with disease that is nearly
always confined to the neck. Follicular carcinomas may metastasise haematogenously and the skull
may be the presenting site of disease in between 2 and 8% of patients.
Please rate this question:

Discuss and give feedback


Next question

Thyroid malignancy

Papillary carcinoma

 Commonest sub-type
 Accurately diagnosed on fine needle aspiration cytology
 Histologically, they may demonstrate psammoma bodies (areas of calcification) and so
called 'orphan Annie' nuclei
 They typically metastasise via the lymphatics and thus laterally located apparently ectopic
thyroid tissue is usually a metastasis from a well differentiated papillary carcinoma
Follicular carcinoma

 Are less common than papillary lesions


 Like papillary tumours, they may present as a discrete nodule. Although they appear to be
well encapsulated macroscopically there is invasion on microscopic evaluation
 Lymph node metastases are uncommon and these tumours tend to spread
haematogenously. This translates into a higher mortality rate
 Follicular lesions cannot be accurately diagnosed on fine needle aspiration cytology and thus
all follicular FNA's (THY 3f) will require at least a hemi thyroidectomy

Anaplastic carcinoma

 Less common and tend to occur in elderly females


 Disease is usually advanced at presentation and often only palliative decompression and
radiotherapy can be offered.

Medullary carcinoma

 These are tumours of the parafollicular cells ( C Cells) and are of neural crest origin.
 The serum calcitonin may be elevated which is of use when monitoring for recurrence.
 They may be familial and occur as part of the MEN -2A disease spectrum.
 Spread may be either lymphatic or haematogenous and as these tumours are not derived
primarily from thyroid cells they are not responsive to radioiodine.

Lymphoma

 These respond well to radiotherapy


 Radical surgery is unnecessary once the disease has been diagnosed on biopsy material.
Such biopsy material is not generated by an FNA and thus a core biopsy has to be obtained
(with care!).

Next question
Question 2 of 347

What is the most common cause of mesenteric infarction?

Mesenteric vein thrombosis

Acute embolism affecting the superior mesenteric artery

Acute on chronic thrombus of the superior mesenteric artery

Sub intimal dissection of the superior mesenteric artery

Proximal migration of abdominal aortic aneurysm

Theme from January 2013 Exam


Acute embolic events account for up to 50% of cases of mesenteric infarcts. These may occur as a
result of long standing atrial fibrillation, ventricular anuerysms and post myocardial infarction.
Please rate this question:

Discuss and give feedback


Next question

Mesenteric vessel disease

Mesenteric ischaemia accounts for 1 in 1000 acute surgical admissions. It is primarily caused by
arterial embolism resulting in infarction of the colon. It is more likely to occur in areas such as the
splenic flexure that are located at the borders of the territory supplied by the superior and inferior
mesenteric arteries.

Types
Acute mesenteric  Sudden onset abdominal pain followed by profuse diarrhoea.
embolus  May be associated with vomiting.
(commonest 50%)  Rapid clinical deterioration.
 Serological tests: WCC, lactate, amylase may all be abnormal
particularly in established disease. These can be normal in the early
phases.

Acute on chronic  Usually longer prodromal history.


mesenteric  Post prandial abdominal discomfort and weight loss are dominant
ischaemia features. Patients will usually present with an acute on chronic
event, but otherwise will tend not to present until mesenteric flow is
reduced by greater than 80%.
 When acute thrombosis occurs presentation may be as above. In the
chronic setting the symptoms will often be those of ischaemic
colitis (mucosa is the most sensitive area to this insult).

Mesenteric vein  Usually a history over weeks.


thrombosis  Overt abdominal signs and symptoms will not occur until venous
thrombosis has reached a stage to compromise arterial inflow.
 Thrombophilia accounts for 60% of cases.

Low flow  This occurs in patients with multiple co morbidities in whom


mesenteric mesenteric perfusion is significantly compromised by overuse of
infarction inotropes or background cardiovascular compromise.
 The end result is that the bowel is not adequately perfused and
infarcts occur from the mucosa outwards.

Diagnosis

 Serological tests: WCC, lactate, CRP, amylase (can be normal in early disease).
 Cornerstone for diagnosis of arterial AND venous mesenteric disease is CT angiography
scanning in the arterial phase with thin slices (<5mm). Venous phase contrast is not helpful.
 SMA duplex USS is useful in the evaluation of proximal SMA disease in patients with chronic
mesenteric ischaemia.
 MRI is of limited use due to gut peristalsis and movement artefact.

Management

 Overt signs of peritonism: Laparotomy


 Mesenteric vein thrombosis: If no peritonism: Medical management with IV heparin
 At operation limited resection of frankly necrotic bowel with view to relook laparotomy at 24-
48h. In the interim urgent bowel revascularisation via endovascular (preferred) or surgery.

Prognosis
Overall poor. Best outlook is from an acute ischaemia from an embolic event where surgery occurs
within 12h. Survival may be 50%. This falls to 30% with treatment delay. The other conditions carry
worse survival figures.
Next question

Question 3 of 347
Which of the following events is most classically seen in the arterioles of patients with malignant
hypertension?

Caseous necrosis

Coagulative necrosis

Fibrinoid necrosis

Apoptosis

Colliquative necrosis

Fibrinoid necrosis is seen in the arterioles of patients with malignant hypertension.

Please rate this question:

Discuss and give feedback


Next question

Cell death

Cells can die via two mechanisms; necrosis and apoptosis. These are outlined below:

Necrosis
Necrosis is characterised by bioenergetics failure. Loss of tissue perfusion results in hypoxia and an
inability to generate ATP. The integrity of the cellular membrane is lost and the loss of ATP results in
loss of energy dependent cellular transport mechanisms. There is an influx of water, ionic instability
and cellular lysis. The release of intracellular contents may stimulate an inflammatory response.
Several types of necrosis are recognised; coagulative, colliquative, caseous, gangrene, fibrinoid and
fat necrosis. The type of tissue and the underlying cause determine the predominant necrosis
pattern.

Coagulative necrosis

 The commonest type, occurs in most organs


 Tissue is initially firm, later becomes soft as tissue is digested by macrophages
 In the early phases the histological appearances may demonstrate little change
 In later stages cellular outlines are seen with loss of intracellular detail
Colliquative necrosis

 Occurs in tissues with no supporting stroma


 Dominant necrosis pattern in the CNS
 Necrotic site may eventually become encysted

Caseous necrosis

 No definable structure seen in the necrotic tissue


 Amorphous eosinophilic tissue may be seen histologically
 Classically seen in tuberculosis

Gangrene

 Necrosis with putrefaction of tissue


 May complicate ischaemia
 Haemoglobin degenerates and results in the deposition of iron sulphide (which is why the
tissue is black)
 Both wet and dry gangrene may occur, in wet gangrene there is often a liquefactive
component and super-added infection (which usually smells!)

Fibrinoid necrosis

 Classically seen in arterioles in patients with hypertension (malignant type)


 Necrosis of the smooth muscle wall occurs and plasma may extravasate into the media with
fibrin deposition

Fat necrosis

 Direct trauma to fat can result in rupture of adipocytes


 Lipids incite a local inflammatory reaction
 Inflammatory cells phagocytose the lipid with eventual fibrosis

Apoptosis

 Also known as programmed cell death


 Energy dependent pathways are activated via a number of intracellular signalling
mechanisms
 It is the result of the activation of caspases triggered by the bcl-2 family or the binding of the
FAS ligand to its receptor
 DNA fragments, mitochondrial function ceases, nuclear and cellular shrinkage occurs
 Phagocytosis of the cell does not occur, instead the cell degenerates into apoptotic bodies

Next question
Question 4 of 347

A 38 year old lady presents with a recent episode of renal colic. As part of her investigations the
following results are obtained:
Corrected Calcium 3.84 mmol/l

PTH 88pg/ml (increased)


Her serum urea and electrolytes are normal.
What is the most likely diagnosis?

Carcinoma of the bronchus

Secondary hyperparathyroidism

Primary hyperparathyroidism

Tertiary hyperparathyroidism

Carcinoma of the breast

Theme from September 2012 exam


Theme from September 2011 exam
In this situation the most likely diagnosis is primary hyperparathyroidism. The question mentions that
serum urea and electrolytes are normal, which makes tertiary hyperparathyroidism unlikely.
Please rate this question:

Discuss and give feedback


Next question

Primary hyperparathyroidism

In exams, primary hyperparathyroidism is stereotypically seen in elderly females with an


unquenchable thirst and an inappropriately normal or raised parathyroid hormone level. It is most
commonly due to a solitary adenoma

Causes of primary hyperparathyroidism

 80%: solitary adenoma


 15%: hyperplasia
 4%: multiple adenoma
 1%: carcinoma

Features - 'bones, stones, abdominal groans and psychic moans'

 Polydipsia, polyuria
 Peptic ulceration/constipation/pancreatitis
 Bone pain/fracture
 Renal stones
 Depression
 Hypertension

Associations

 Hypertension
 Multiple endocrine neoplasia: MEN I and II

Investigations

 Raised calcium, low phosphate


 PTH may be raised or normal
 Technetium-MIBI subtraction scan

Treatment

 Parathyroidectomy, if imaging suggests target gland then a focused approach may be used

Next question
Question 5 of 347

A 24 year old man is diagnosed as having Hodgkins lymphoma. Which subtype is associated with
the most favorable prognosis?

Nodular sclerosing

Lymphocyte depleted

Nodular lymphocyte predominant

Classical lymphocyte predominant

Mixed cellularity

Theme from September 2014 Exam


The classical lymphocyte predominant variant has the most favorable prognosis. Nodular
lymphocyte predominant disease is a very different disease entity and does not have the same
favorable prognosis.
Please rate this question:

Discuss and give feedback


Next question

Hodgkins lymphoma

Presenting features

 Asymptomatic lympadenopathy
 Cough, Pel Ebstein fever, haemoptysis, dyspnoea
 B Symptoms - 10% weight loss, fever, night sweats

Staging
All patients are staged with CT scanning of the chest, abdomen and pelvis
The Ann Arbor staging system is commonly used
Stage Features
Stage Features

I Single lymph node region

II Two or more regions on the same side of the diaphragm

III Involvement of lymph node regions on both sides of the diaphragm

IV Involvement of extra nodal sites

Sub types
Classical Hodgkin lymphoma is classified into the following 4 types:

Nodular sclerosing Hodgkin lymphoma (NSHL)


Mixed-cellularity Hodgkin lymphoma (MCHL)
Lymphocyte-depleted Hodgkin lymphoma (LDHL)
Lymphocyte-rich classical Hodgkin lymphoma (LRHL)

A Reed Sternberg cell may be identified histologically.

A fifth sub type, Nodular lymphocyte-predominant Hodgkin lymphoma, is characterised by a different


cell type Reed- Sternberg cells are rarely seen.

Treatment
This may be multimodal and both chemo and radiotherapy are used.

Diagnosis
This is made by excision of a complete lymph node that is then submitted for detailed histological
evaluation.

Pathogenesis
Infection with Ebstein Barr virus is linked to the condition (particularly mixed cellularity lymphoma).

Prognosis
Stage I disease is associated with survival figures of up to 85% at 5 years. The lymphocyte rich
classical lymphoma has the best prognosis. Lymphocyte depleted Hodgkins lymphoma, advancing
age, male sex and stage IV disease are all associated with a worsening of prognosis.
Next question
Question 6-8 of 347

Theme: Head and neck lumps

A. Branchial cyst

B. Cystic hygroma

C. Carotid body tumour

D. Lymphadenopathy

E. Adenolymphoma of the parotid

F. Pleomorphic adenoma of the parotid

G. Submandibular tumour

H. Thyroglossal cyst

I. Thoracic outlet syndrome

J. Submandibular gland calculus

Please select the most likely lesion to account for the clinical scenario given. Each option may be
used once, more than once or not at all.

6. A 60 year old Tibetan immigrant is referred to the surgical clinic with a painless neck swelling. On
examination, it is located on the left side immediately anterior to the sternocleidomastoid muscle.
There are no other abnormalities to find.

You answered Branchial cyst

The correct answer is Carotid body tumour

Carotid body tumours typically present as painless masses. They may compress the vagus or
hypoglossal nerves with symptoms attributable to these structures. Over 90% occur spontaneously
and are more common in people living at high altitude. In familial cases up to 30% may be bilateral.
Treatment is with excision.
7. A 40 year old women presents as an emergency with a painful mass underneath her right
mandible. The mass has appeared over the previous week with the pain worsening as the lump has
increased in size. On examination, there is a 4cm mass underneath her mandible, there is no
associated lymphadenopathy.

You answered Branchial cyst

The correct answer is Submandibular gland calculus

The sub mandibular gland is the most common site for salivary calculi. Patients will usually
complain of pain, which is worse on eating. When the lesion is located distally the duct may be laid
open and the stone excised. Otherwise the gland will require removal.

8. A 73 year old male smoker is referred to the clinic by his GP. On examination he has a 3cm soft
mass immediately anterior to his ear. It has been present for the past five years and is otherwise
associated with no symptoms.

You answered Branchial cyst

The correct answer is Adenolymphoma of the parotid

Warthins tumours (a.k.a. adenolymphoma) are commoner in older men (especially smokers). They
are the second commonest benign tumour of the parotid gland, they may be bilateral. They are
soft and slow growing and relatively easy to excise. Pleomorphic adenomas typically present in
females aged between 40 - 60 years.

Please rate this question:

Discuss and give feedback

Next question

Neck lumps
The table below gives characteristic exam question features for conditions causing neck lumps:

Reactive By far the most common cause of neck swellings. There may be a history of
lymphadenopathy local infection or a generalised viral illness

Lymphoma Rubbery, painless lymphadenopathy


The phenomenon of pain whilst drinking alcohol is very uncommon
There may be associated night sweats and splenomegaly

Thyroid swelling May be hypo-, eu- or hyperthyroid symptomatically


Moves upwards on swallowing

Thyroglossal cyst More common in patients < 20 years old


Usually midline, between the isthmus of the thyroid and the hyoid bone
Moves upwards with protrusion of the tongue
May be painful if infected

Pharyngeal pouch More common in older men


Represents a posteromedial herniation between thyropharyngeus and
cricopharyngeus muscles
Usually not seen, but if large then a midline lump in the neck that gurgles on
palpation
Typical symptoms are dysphagia, regurgitation, aspiration and chronic cough

Cystic hygroma A congenital lymphatic lesion (lymphangioma) typically found in the neck,
classically on the left side
Most are evident at birth, around 90% present before 2 years of age

Branchial cyst An oval, mobile cystic mass that develops between the sternocleidomastoid
muscle and the pharynx
Develop due to failure of obliteration of the second branchial cleft in
embryonic development
Usually present in early adulthood
Cervical rib More common in adult females
Around 10% develop thoracic outlet syndrome

Carotid aneurysm Pulsatile lateral neck mass which doesn't move on swallowing

Next question
Question 9 of 347

A 12 year old child is admitted with a 12 hour history of colicky right upper quadrant pain. On
examination the child is afebrile and is jaundiced. The abdomen is soft and non tender at the time of
examination. What is the most likely cause?

Infectious hepatitis

Acute cholecystitis

Cholangitis

Hereditary spherocytosis

Gilberts syndrome

Cholecystitis is unlikely in the absence of focal tenderness

Theme from September 2012 Exam


The child is most likely to have hereditary spherocytosis. In these individuals there may be disease
flares precipitated by acute illness. They form small pigment stones. These may cause biliary colic
and some may require cholecystectomy.
Gilbert's syndrome is an inherited condition causing unconjugated hyperbilirubinaemia. Patients may
have jaundice or be asymptomatic. The other LFTs are normal and Gilbert's may be confirmed with
a fasting test or Nicotinic acid test. There is no need for treatment as it is a benign condition.

Please rate this question:

Discuss and give feedback

Next question

Hereditary Spherocytosis
Most common disorder of the red cell membrane, it has an incidence of 1 in 5000. The abnormally
shaped erythrocytes are prone to splenic sequestration and destruction. This can result in
hyperbilirubinaemia, jaundice and splenomegaly. In older patients an intercurrent illness may
increase the rate of red cell destruction resulting in more acute symptoms.
Severe cases may benefit from splenectomy.

Next question
Question 10 of 347

A 28 year old man has a carcinoid tumour identified in his appendix. Blood testing for which of the
substances listed below is likely to be helpful during follow up?

CA19-9

Alkaline phosphatase

AFP

CEA

Chromogranin A

Theme from September 2013 Exam


It is important to distinguish between blood and urine tests for carcinoid syndrome. Blood tests
usually measure chromogranin A,neuron-specific enolase (NSE), substance P, and gastrin. Urine
tests usually measure 5 HIAA, which is a metabolite of serotonin. Sometimes blood tests for 5
hydroxytryptamine (serotonin) are also performed.
Please rate this question:

Discuss and give feedback


Next question

Carcinoid syndrome

 Carcinoid tumours secrete serotonin


 Originate in neuroendocrine cells mainly in the intestine (midgut-distal ileum/appendix)
 Can occur in the rectum, bronchi
 Hormonal symptoms mainly occur when disease spreads outside the bowel

Clinical features

 Onset: insidious over many years


 Flushing face
 Palpitations
 Pulmonary valve stenosis and tricuspid regurgitation causing dyspnoea
 Asthma
 Severe diarrhoea (secretory, persists despite fasting)

Investigation

 5-HIAA in a 24-hour urine collection


 Somatostatin receptor scintigraphy
 CT scan
 Blood testing for chromogranin A

Treatment

 Octreotide
 Surgical removal

Next question
Question 11 of 347

A 40 year old man presents with obstructive jaundice and dysphagia. Twenty years previously he
underwent a right hemicolectomy for a mucinous right sided colonic carcinoma. He was
subsequently diagnosed as having Lynch syndrome. A recent colonoscopy was normal. What is the
most likely cause of his jaundice?

Hepatocellular carcinoma

Liver metastasis from colonic cancer

Pancreatic carcinoma

Duodenal carcinoma

Gastric carcinoma

Theme from April 2013


Lynch syndrome usually results in colonic cancer which is right sided and mucinous. The next most
common site to be affected is the uterus. The stomach is at particular risk and this risk is up to 10
times greater in HNPCC (Lynch) patients than the general population. Duodenal adenomas (and
rarely carcinoma) are usually seen in association with FAP. Whilst pancreatic carcinoma is
associated with HNPCC it is far less likely to occur than gastric cancer.

Please rate this question:

Discuss and give feedback

Next question

Genetics of colorectal cancer

The lifetime risk of colorectal cancer in the UK population is 5%. Up to 5% of newly diagnosed bowel
cancers will be in those individuals who have a high genetically acquired risk of bowel cancer.
Cancers arising in the low-moderate genetic risk group comprise approximately 30% of newly
diagnosed bowel cancer.

Genetics of inherited colorectal cancer syndromes

Syndrome Features Genes implicated

FAP More than 100 adenomatous polyps affecting the colon and APC (over 90%)
rectum. Duodenal and fundic glandular polyps

Gardner As FAP but with desmoid tumours and mandibular osteomas APC
syndrome

Turcots syndrome Polyposis and colonic tumours and CNS tumours APC +MLH1 and
PMS2

HNPCC Colorectal cancer without extensive polyposis. Endometrial MSH2, MLH1, PMS2
cancer, renal and CNS and GTBP

Peutz-Jeghers Hamartomatous polyps in GI tract and increased risk of GI LKB1 andSTK11 (in
syndrome malignancy up to 70%)

Cowden disease Multiple hamartomas (see below) PTEN (85%)

MYH associated Autosomal recessive, multiple adenomatous polyps in GI tract, MYH


polyposis those in colon having somatic KRAS mutations

FAP
Autosomal dominant condition, affects 1 in 12,000. Accounts for 0.5% of all CRCs. Lifetime
incidence of colorectal cancer in untreated FAP =100%. Up to 25% cases are caused by de-novo
germ line mutations and show no prior family history. The APC tumour suppressor gene is affected
in most cases.

APC in non inherited colorectal cancer


Up to 80% of sporadic colorectal cancers will have somatic mutations that inactivate APC[1]. Both
alleles are usually affected. Although the APC protein more than likely has multiple critical cellular
functions, the best-established role for APC in the cancer process is as a major binding partner and
regulator of the β- catenin protein in the so-called canonical or β- catenin dependent Wnt signaling
pathway.

HNPCC (Lynch syndrome)


HNPCC cancers differ from conventional tumours in a number of respects. In the colon the tumours
are more likely to be right sided, histologically they are more likely to be mucinous and have dense
lymphocytic infiltrates. To be diagnosed as having HNPCC individuals must show typically HNPCC
tumours in at least three individuals, (one of whom must be a first degree relative to the other two).
In at least two successive generations. At least one cancer must be diagnosed under the age of 50.
FAP must be excluded and tumours should be verified by pathological identification (Amsterdam
criteria). The genetic changes in HNPCC stem primarily from microsatellite instability affecting DNA
mismatch repair genes. In HNPCC the mismatch repair genes most commonly implicated include;
MSH2 and MLH1 and these occur in up to 70% of people with HNPCC. The finding of microsatellite
instability is unusual in sporadic colorectal cancers. Approximately 60% of individuals who fulfill the
Amsterdam criteria will not be found to have evidence of mismatch repair gene defects on genetic
testing. The risk of developing colorectal cancer in those who have not demonstrated mutation of the
mis match repair genes is increased if they fulfill the Amsterdam criteria, but not
the extent that it is increased in those who fulfill the criteria AND have evidence of mis match repair
gene defects.

KRAS Mutations
The RAS family of small G proteins act as molecular switches downstream of growth factor
receptors. KRAS and the other two members of the family; HRAS and NRAS, are the site of
mutation in approximately 40% of colorectal cancers. When adenomas are examined the proportion
of adenomas less than 1cm showing KRAS mutations was only 10% which contrasts with 50% in
those lesions greater than 1cm.

p53 mutations
The p53 protein functions as a key transcriptional regulator of genes that encode proteins with
functions in cell-cycle checkpoints at the G1/S and G2/M boundaries, in promoting apoptosis, and in
restricting angiogenesis . As such, selection for p53 defects at the adenoma-carcinoma transition
may reflect the fact that stresses on tumor cells activate cell-cycle arrest, apoptotic, and
antiangiogenic pathways in cells with wild-type p53 function. Many colonic tumours will demonstrate
changes in the p53 gene that may facilitate tumour progression through from adenoma to
carcinoma.

Cowden syndrome
Also known as multiple hamartoma syndrome. Rare autosomal dominant condition with incidence of
1 in 200,000.. It is characterised by multiple mucocutaneous lesions, trichilemmomas, oral
papillomas and acral keratosis. Most often diagnosed in third decade of life. Breast carcinoma may
occur in up to 50% of patients and conditions such as fibrocystic disease of the breast may occur in
75% of women. Thyroid disease occurs in 75% and may include malignancy. Endoscopic screening
will identify disease in up to 85% although the small bowel is rarely involved. There is a 15-20% risk
of developing colorectal cancer and regular colonoscopic screening from age 45 is recommended.
Terminology

Oncogene Oncogenes are genes which have the potential to induce cellular proliferation and avoid
apoptosis. Oncogene mutations are general gain of function and are therefore
dominant. Increased expression of oncogenes are found in most tumours

Tumour These genes generally inhibit cellular proliferation or induce apoptosis. Mutations in
suppressor tumour suppressor genes are generally loss of function mutations, and are therefore
gene recessive. Mutations in both tumour suppressor gene alleles allow cells to proliferate
without restraint

References
1. Fearon, E.R. and B. Vogelstein, A genetic model for colorectal tumorigenesis. Cell, 1990. 61(5): p.
759-67.

Next question
Question 12-14 of 347

Theme: Breast disease

A. Tuberculosis
B. Actinomycosis
C. Duct ectasia
D. Fibroadenoma
E. Fat necrosis
F. Intraductal papilloma
G. Breast abscess
H. Breast cancer

What is the most likely diagnosis for each scenario given? Each diagnosis may be used once, more
than once or not at all.

12. A 32 year old woman presents with a tender breast lump. She has a 2 month old child.
Clinically there is a tender, fluctuant mass of the breast.

You answered Tuberculosis

The correct answer is Breast abscess

Theme from January 2013 Exam


Theme from September 2011 Exam
This lady is likely to be breast feeding and is at risk of mastitis. This may lead to an
abscess if not treated. Staphylococcus aureus is usually the causative organism.

13. A 53 year old lady presents with a creamy nipple discharge. On examination she has
discharge originating from multiple ducts and associated nipple inversion.

You answered Tuberculosis

The correct answer is Duct ectasia

Duct ectasia is common during the period of breast involution that occurs during the
menopausal period. As the ducts shorten they may contain insipiated material. The
discharge will often discharge from several ducts.

14. A 52 year old lady presents with an episode of nipple discharge. It is usually clear in
nature. On examination the discharge is seen to originate from a single duct and although it
appears clear, when the discharge is tested with a labstix it is shown to contain blood.
Imaging and examination shows no obvious mass lesion.
You answered Tuberculosis

The correct answer is Intraductal papilloma

Intraductal papilloma usually cause single duct discharge. The fluid is often clear, although
it may be blood stained. If the fluid is tested with a labstix (little point in routine practice)
then it will usually contain small amounts of blood. A microdocechtomy may be
performed.

Please rate this question:

Discuss and give feedback


Next question

Non malignant breast disease

Duct ectasia

 Mammary duct ectasia may be seen in up to 25% of normal female breasts


 Patients usually present with nipple discharge, which may be from single or multiple ducts
(usually present age >50 years)
 The discharge is often thick and green
 Duct ectasia is a normal variant of breast involution and is not the same condition as
periductal mastitis

Periductal mastitis

 Present at younger age than duct ectasia


 May present with features of inflammation, abscess or mammary duct fistula
 Strongly associated with smoking
 Usually treated with antibiotics, abscess will require drainage

Intraductal papilloma

 Growth of papilloma in a single duct


 Usually presents with clear or blood stained discharge originating from a single duct
 No increase in risk of malignancy

Breast abscess
 Lactational mastitis is common
 Infection is usually with Staphylococcus aureus
 On examination there is usually a tender fluctuant mass
 Treatment is with antibiotics and ultrasound guided aspiration
 Overlying skin necrosis is an indication for surgical debridement, which may be complicated
by the development of a subsequent mammary duct fistula.

Tuberculosis

 Rare in western countries, usually secondary TB


 Affects women later in child bearing period
 Chronic breast or axillary sinus is present in up to 50% cases
 Diagnosis is by biopsy culture and histology

Next question
Question 15 of 347

A 34 year old woman who has previously undergone a colectomy for familial adenomatous polyposis
coli presents with a firm lesion at the inferior aspect of her rectus abdominis muscle. Which cell type
is most typically associated with such tumours?

Myocytes

Proliferation of apocrine glands

Chondrocytes

Lipoblasts

Myofibroblasts

Desmoid tumours would be the most likely differential here and consist of a clonal proliferation of
myofibroblasts.
Please rate this question:

Discuss and give feedback


Next question

Desmoid tumours

 Desmoid tumours are fibrous neoplasms arising from musculoaponeurotic structures. They
are typically contain clonal proliferations of myofibroblasts.
 They are usually firm overgrowths of tissue with a propensity to local infiltration.
 They occur in up to 15% of patients with familial adenomatous polyposis coli.
 Desmoid tumours usually show bi allelic APC mutations
 They occur most commonly in women after childbirth in the rectus abdominis muscle.
 They are usually treated (regardless of site) by radical surgical resection, in some patients
radiotherapy and chemotherapy may be considered. The results of non surgical therapy is
inferior to surgical resection. In selected cases of abdominal desmoids a period of
observation may be preferred as some may spontaneously regress.
 Desmoids have a high tendency to local recurrence.

Desmoids consist of sheets of differentiated fibroblasts


Image sourced from Wikipedia

Next question
Question 16 of 347

A 2 day old baby is noted to have voiding difficulties and on closer inspection is noted to have
hypospadias. Which of the following abnormalities is most commonly associated with the condition?

Cryptorchidism

Diaphragmatic hernia

Ventricular - septal defect

Bronchogenic cyst

Atrial septal defect

Theme from January 2012 Exam

Hypospadias most commonly occurs as an isolated disorder. Associated urological abnormalities


may be seen in up to 40% of infants, of these cryptorchidism is the most frequent (10%).
Please rate this question:

Discuss and give feedback


Next question

Hypospadias

The urethral meatus opens on the ventral surface of the penis. There is also a ventral deficiency of
the foreskin. The urethral meatus may open more proximally in the more severe variants. However,
75% of the openings are distally located. The incidence is 1 in 300 male births.

Features include:

 Absent frenular artery


 Ventrally opened glans
 Skin tethering to hypoplastic urethra
 Splayed columns of spongiosum tissue distal to the meatus
 Deficiency of the foreskin ventrally
Management:

 No routine cultural circumcisions


 Urethroplasty
 Penile reconstruction

The foreskin is often utilised in the reconstructive process. In boys with very distal disease no
treatment may be needed.
Next question
Question 17-19 of 347

Theme: Liver lesions

A. Cystadenoma
B. Hydatid cyst
C. Amoebic abscess
D. Mesenchymal hamartoma
E. Liver cell adenoma
F. Cavernous haemangioma

Please select the most likely lesion for the scenario given. Each option may be used once, more
than once or not at all.

17. A 38 year old lady presents with right upper quadrant pain and nausea. She is otherwise
well and her only medical therapy is the oral contraceptive pill which she has taken for
many years with no ill effects. Her liver function tests are normal. An ultrasound
examination demonstrates a hyperechoic well defined lesion in the left lobe of the liver
which measures 14 cm in diameter.

You answered Cystadenoma

The correct answer is Cavernous haemangioma

Cavernous haemangioma often presents with vague symptoms and signs. They may grow
to considerable size. Liver function tests are usually normal. The lesions are typically well
defined and hyperechoic on ultrasound. A causative link between OCP use and
haemangiomata has yet to be established, but is possible.

18. A 37 year old lady presents with right upper quadrant pain and nausea. She is otherwise
well and her only medical therapy is the oral contraceptive pill which she has taken for
many years with no ill effects. Her liver function tests and serum alpha feto protein are
normal. An ultrasound examination demonstrates a 4cm non encapsulated lesion in the
right lobe of the liver which has a mixed echoity and heterogeneous texture.

You answered Cystadenoma

The correct answer is Liver cell adenoma

Liver cell adenomas are linked to OCP use and 90% of patients with liver cell adenomas
have used the OCP. Liver function tests are often normal. The lesions will typically have a
mixed echoity and heterogeneous texture.
19. A 38 year old shepherd presents to the clinic with a 3 month history of malaise and right
upper quadrant pain. On examination he is mildly jaundiced. His liver function tests
demonstrate a mild elevation in bilirubin and transaminases, his full blood count shows an
elevated eosinophil level. An abdominal x-ray is performed by the senior house officer and
demonstrates a calcified lesion in the right upper quadrant of the abdomen.

You answered Cystadenoma

The correct answer is Hydatid cyst

Similar theme in September 2011 Exam

Hyatid disease is more common in those who work with sheep or dogs. Liver function
tests may be abnormal and an eosinophilia is often present. Plain radiographs may reveal a
calcified cyst wall.

Please rate this question:

Discuss and give feedback


Next question

Benign liver lesions

Benign liver lesions


Haemangioma  Most common benign tumours of mesenchymal origin
 Incidence in autopsy series is 8%
 Cavernous haemangiomas may be enormous
 Clinically they are reddish purple hypervascular lesions
 Lesions are normally separated from normal liver by ring of fibrous
tissue
 On ultrasound they are typically hyperechoic

Liver cell  90% develop in women in their third to fifth decade


adenoma  Linked to use of oral contraceptive pill
 Lesions are usually solitary
 They are usually sharply demarcated from normal liver although they
usually lack a fibrous capsule
 On ultrasound the appearances are of mixed echoity and
heterogeneous texture. On CT most lesions are hypodense when
imaged prior to administration of IV contrast agents
 In patients with haemorrhage or symptoms removal of the adenoma
may be required

Mesenchymal Congential and benign, usually present in infants. May compress normal liver
hamartomas

Liver abscess  Biliary sepsis is a major predisposing factor


 Structures drained by the portal venous system form the second largest
source
 Common symptoms include fever, right upper quadrant pain. Jaundice
may be seen in 50%
 Ultrasound will usually show a fluid filled cavity, hyperechoic walls
may be seen in chronic abscesses

Amoebic abscess  Liver abscess is the most common extra intestinal manifestation of
amoebiasis
 Between 75 and 90% lesions occur in the right lobe
 Presenting complaints typically include fever and right upper quadrant
pain
 Ultrasonography will usually show a fluid filled structure with poorly
defined boundaries
 Aspiration yield sterile odourless fluid which has an anchovy paste
consistency
 Treatment is with metronidazole

Hyatid cysts  Seen in cases of Echinococcus infection


 Typically an intense fibrotic reaction occurs around sites of infection
 The cyst has no epithelial lining
 Cysts are commonly unilocular and may grow to 20cm in size. The
cyst wall is thick and has an external laminated hilar membrane and an
internal enucleated germinal layer
 Typically presents with malaise and right upper quadrant pain.
Secondary bacterial infection occurs in 10%.
 Liver function tests are usually abnormal and eosinophilia is present in
33% cases
 Ultrasound may show septa and hyatid sand or daughter cysts.
 Percutaneous aspiration is contra indicated
 Treatment is by sterilisation of the cyst with mebendazole and may be
followed by surgical resection. Hypertonic swabs are packed around
the cysts during surgery

Polycystic liver  Usually occurs in association with polycystic kidney disease


disease  Autosomal dominant disorder
 Symptoms may occur as a result of capsular stretch

Cystadenoma  Rare lesions with malignant potential


 Usually solitary multiloculated lesions
 Liver function tests usually normal
 Ultrasonography typically shows a large anechoic, fluid filled area
with irregular margins. Internal echos may result from septa
 Surgical resection is indicated in all cases

Next question
Question 20 of 347

A 5 year old boy presents to the clinic with short stature suggestive of achondroplasia. What is the
genetic basis of this condition?

X linked defect

Y linked defect

YY linked defect

Autosomal dominant defect

Autosomal recessive defect

Theme from April 2013 Exam


Theme from April 2014 exam
Achondroplasia usually occurs as a sporadic mutation. It is then transmitted in an autosomal
dominant fashion.
Please rate this question:

Discuss and give feedback


Next question

Achondroplasia

Achondroplasia is a common cause of dwarfism and is caused by defects in the fibroblast growth
factor receptor. In most cases (approximately 70%) it occurs as a sporadic mutation. The main risk
factor is advancing parental age at the time of conception. Once present it is typically inherited in an
autosomal dominant fashion.

Radiological features

 Large skull with narrow foramen magnum


 Short, flattened intervertebral bodies
 Narrow spinal canal
 Horizontal acetabular roof
 Broad, short metacarpals
Treatment
There is no specific therapy. However, some individuals benefit from limb lengthening procedures.
These usually involve application of Ilizarov frames and targeted bone fractures. A clearly defined
need and end point is the cornerstone of achieving success with such procedures.
Next question
Question 21 of 347

A 72 year old man presents with symptoms and signs of benign prostatic hyperplasia. Which of the
following structures is most likely to be enlarged?

Posterior lobe of the prostate

Median lobe of the prostate

Right lateral lobe of the prostate

Left lateral lobe of the prostate

Anterior lobe of the prostate

Carcinoma of the prostate typically occurs in the posterior lobe. The median lobe is usually enlarged
in BPH. The anterior lobe has little in the way of glandular tissue and is seldom enlarged.
Please rate this question:

Discuss and give feedback


Next question

Benign Prostatic Hyperplasia

Benign prostatic hyperplasia occurs via an increase in the epithelial and stromal cell numbers in the
peri-urethral zone of the prostate. BPH is very common and 90% of men aged over 80 will have at
least microscopic evidence of benign prostatic hyperplasia. The causes of BPH are still not well
understood, but the importance of androgens remains appreciated even if the exact role by which
they induce BPH is elusive.

Presentation
The vast majority of men will present with lower urinary tract symptoms. These will typically be:

 Poor flow
 Nocturia
 Hesitancy
 Incomplete and double voiding
 Terminal dribbling
 Urgency
 Incontinence

Investigation

 Digital rectal examination to assess prostatic size and morphology.


 Urine dipstick for infections and haematuria.
 Uroflowmetry (a flow rate of >15ml/second helps to exclude BOO)
 Bladder pressure studies may help identify detrusor failure and whilst may not form part of
first line investigations should be included in those with atypical symptoms and prior to redo
surgery.
 Bladder scanning to demonstrate residual volumes. USS if high pressure chronic retention.

Management

 Lifestyle changes such as stopping smoking and altering fluid intake may help those with
mild symptoms.
 Medical therapy includes alpha blockers and 5 α reductase inhibitors. The former work
quickly on receptor zones located at the bladder neck. Cardiovascular side effects are well
documented. The latter work on testosterone metabolising enzymes. Although they have a
slower onset of action, the 5 α reductase inhibitors may prevent acute urinary retention.
 Surgical therapy includes transurethral resection of the prostate and is the treatment of
choice in those with severe symptoms and those who fail to respond to medical therapy.
More tailored bladder neck incision procedures may be considered in those with small
prostates. Retrograde ejaculation may occur following surgery. The change in the type of
irrigation solutions used has helped to minimise the TURP syndrome of electrolyte
disturbances.

Next question
Question 22 of 347

A 38 year old man has been suffering from mechanical back pain for several years. One morning he
awakes from sleep and feels a sudden onset of pain in his back radiating down his left leg. Which of
the following events is most likely to account for his symptoms?

Prolapse of inner annulus fibrosus

Prolapse of outer annulus fibrosus

Prolapse of nucleus pulposus

Rupture of the ligamentum flavum

None of the above

Theme from 2009 Exam


Theme from September 2012 Exam
The symptoms would be most likely the result of intervertebral disk prolapse. In disk prolapse the
nucleus pulposus is the structure which usually herniates.
Please rate this question:

Discuss and give feedback


Next question

Intervertebral discs

 Consist of an outer annulus fibrosus and an inner nucleus pulposus.


 The anulus fibrosus consists of several layers of fibrocartilage.
 The nucleus pulposus contains loose fibres suspended in a mucoprotein gel with the
consistency of jelly. The nucleus of the disc acts as a shock absorber.
 Pressure on the disc causes posterior protrusion of the nucleus pulposus. Most commonly in
the lumbrosacral and lower cervical areas.
 The discs are separated by hyaline cartilage.
 There is one disc between each pair of vertebrae, except for C1/2 and the sacrococcygeal
vertebrae.

Next question
Question 23-25 of 347

Theme: Paediatric neck masses

A. Cystic hygroma
B. Thyroglossal cyst
C. Rhabdomyosarcoma
D. Branchial cyst
E. Dermoid cyst

Please select the most likely underlying diagnosis for the situation that is described. Each option
may be used once, more than once, or not at all.

23. A 2 year old boy is brought to the clinic by his mother who has noticed that he has
developed a small mass. On examination; a small smooth cyst is identified which is
located above the hyoid bone. On ultrasound the lesion appears to be a heterogenous and
multiloculated mass.

You answered Cystic hygroma

The correct answer is Dermoid cyst

Theme from September 2015 Exam


Dermoid cysts are usually multiloculated and heterogeneous. Most are located above the
hyoid, and their appearances on imaging differentiate them from thyroglossal cysts.

24. A 22 month old baby is brought to the clinic by her mother who is concerned that she has
developed a swelling in her neck. On examination; she has a soft lesion located in the
posterior triangle that transilluminates.

Cystic hygroma

Cystic hygromas are soft and transilluminate. Most are located in the posterior triangle.

25. A 3 year old boy is brought to the clinic by his mother who has noticed a mass in his neck.
On examination; he has a smooth mass located on the lateral aspect of his anterior triangle,
near to the angle of the mandible. On ultrasound; it has a fluid filled, anechoic,
appearance.

You answered Cystic hygroma

The correct answer is Branchial cyst


Branchial cysts are usually located laterally and derived from the second branchial cleft.
Unless infection has occurred they will usually have an anechoic appearance on
ultrasound.

Please rate this question:

Discuss and give feedback


Next question

Neck Masses in Children

Thyroglossal cyst  Located in the anterior triangle, usually in the midline and below
the hyoid (65% cases)
 Derived from remnants of the thyroglossal duct
 Thin walled and anechoic on USS (echogenicity suggests
infection of cyst)

Branchial cyst  Six branchial arches separated by branchial clefts


 Incomplete obliteration of the branchial apparatus may result in
cysts, sinuses or fistulae
 75% of branchial cysts originate from the second branchial cleft
 Usually located anterior to the sternocleidomastoid near the
angle of the mandible
 Unless infected the fluid of the cyst has a similar consistency to
water and is anechoic on USS

Dermoids  Derived from pleuripotent stem cells and are located in the
midline
 Most commonly in a suprahyoid location
 They have heterogeneous appearances on imaging and contain
variable amounts of calcium and fat

Thyroid gland  True thyroid lesions are rare in children and usually represent
thyroglossal cysts or tumours like lymphoma

Lymphatic  Usually located posterior to the sternocleidomastoid


malformations  Cystic hygroma result from occlusion of lymphatic channels
 The painless, fluid filled, lesions usually present prior to the age
of 2
 They are often closely linked to surrounding structures and
surgical removal is difficult
 They are typically hypoechoic on USS

Infantile  May present in either triangle of the neck


haemangioma  Grow rapidly initially and then will often spontaneously regress
 Plain x-rays will show a mass lesion, usually containing
calcified phleboliths
 As involution occurs the fat content of the lesions increases

Lymphadenopathy  Located in either triangle of the neck


 May be reactive or neoplastic
 Generalised lymphadenopathy usually secondary to infection in
children (very common)

Next question
Question 26 of 347

A 34 year old man presents to the surgical clinic 8 months following a laparotomy for a ruptured
spleen. He complains of a nodule in the centre of his laparotomy wound. This is explored surgically
and a stitch granuloma is found and excised. From which of the following cell types do granulomata
arise?

Polymorpho nucleocytes

Plasma cells

Reed- Sternberg cells

Platelets

Macrophages

Granulomas are organised collections of macrophages

Theme from 2011 Exam

Macrophages give origin to granulomas.


Please rate this question:

Discuss and give feedback


Next question

Chronic inflammation

Overview
Chronic inflammation may occur secondary to acute inflammation.In most cases chronic
inflammation occurs as a primary process. These may be broadly viewed as being one of three main
processes:

 Persisting infection with certain organisms such as Mycobacterium tuberculosis which results
in delayed type hypersensitivity reactions and inflammation.
 Prolonged exposure to non-biodegradable substances such as silica or suture materials
which may induce an inflammatory response.
 Autoimmune conditions involving antibodies formed against host antigens.
Acute vs. Chronic inflammation
Acute inflammation Chronic inflammation

Changes to existing vascular structure and increased Angiogenesis predominates


permeability of endothelial cells

Infiltration of neutrophils Macrophages, plasma cells and


lymphocytes predominate

Process may resolve with: Healing by fibrosis is the main result

 Suppuration
 Complete resolution
 Abscess formation
 Progression to chronic inflammation
 Healing by fibrosis

Granulomatous inflammation
A granuloma consists of a microscopic aggregation of macrophages (with epithelial type
arrangement =epitheliod). Large giant cells may be found at the periphery of granulomas.

Mediators
Growth factors released by activated macrophages include agents such as interferon and fibroblast
growth factor (plus many more). Some of these such as interferons may have systemic features
resulting in systemic symptoms and signs, which may be present in individuals with long standing
chronic inflammation.

The finding of granulomas is pathognomonic of chronic inflammation, as illustrated in this biopsy


from a patient with colonic Crohns disease
Image sourced from Wikipedia

Next question
Question 27 of 347

An unusually tall 43 year old lady presents to the surgical clinic with bilateral inguinal hernias. She
develops chest pain and collapses. As part of her investigations a chest x-ray shows evidence of
mediastinal widening. What is the most likely underlying diagnosis?

Pulmonary embolus

Aortic dissection

Tietze syndrome

Boerhaaves syndrome

Myocardial infarct

Theme from September 2015 Exam


Marfans syndrome may present with a variety of connective tissue disorders such as bilateral
inguinal hernia. They are at high risk of aortic dissection, as in this case.
Please rate this question:

Discuss and give feedback


Next question

Aortic dissection

 More common than rupture of the abdominal aorta


 33% of patients die within the first 24 hours, and 50% die within 48 hours if no treatment
received
 Associated with hypertension
 Features of aortic dissection: tear in the intimal layer, followed by formation and propagation
of a subintimal hematoma. Cystic medial necrosis (Marfan's)
 Most common site of dissection: 90% occurring within 10 centimetres of the aortic valve

Stanford Classification
Type Location Treatment
Type Location Treatment

A Ascending aorta/ aortic root Surgery- aortic root replacement

B Descending aorta Medical therapy with antihypertensives

DeBakey classification
Type Site affected

I Ascending aorta, aortic arch, descending aorta

II Ascending aorta only

III Descending aorta distal to left subclavian artery

Clinical features

 Tearing, sudden onset chest pain (painless 10%)


 Hypertension or Hypotension
 A blood pressure difference (in each arm) greater than 20 mm Hg
 Neurologic deficits (20%)

Investigations

 CXR: widened mediastinum, abnormal aortic knob, ring sign, deviation of the
trachea/oesophagus
 CT angiography of the thoracic aorta
 MRI angiography
 Conventional angiography (now rarely used diagnostically)

Management

 Beta-blockers: aim HR 60-80 bpm and systolic BP 100-120 mm Hg


 For type A dissections the standard of care is aortic root replacement
 Question 28 of 347

 A 72 year old man has just undergone an emergency repair for a ruptured abdominal aortic
aneurysm. Pre operatively he was taking aspirin, clopidogrel and warfarin. Intra operatively
he received 5000 units of unfractionated heparin prior to application of the aortic cross
clamp. His blood results on admission to the critical care unit are as follows:
Full blood count

Hb 8 g/dl

Platelets 40 * 109/l

WBC 7.1 * 109/l

 His fibrin degradation products are measured and found to be markedly elevated. Which of
the following accounts for these results?

Anastomotic leak

Disseminated intravascular coagulation

Heparin induced thrombocytopenia

Adverse effect of warfarin

Adverse effects of antiplatelet agents


Theme from April 2012 Exam
The combination of low platelet counts and raised FDP in this setting make DIC the most
likely diagnosis.
 Please rate this question:


 Discuss and give feedback

 Next question
 Disseminated intravascular coagulation - Diagnosis

 Under homeostatic conditions, coagulation and fibrinolysis are coupled. The activation of the
coagulation cascade yields thrombin that converts fibrinogen to fibrin; the stable fibrin clot
being the final product of hemostasis. The fibrinolytic system breaks down fibrinogen and
fibrin. Activation of the fibrinolytic system generates plasmin (in the presence of thrombin),
which is responsible for the lysis of fibrin clots. The breakdown of fibrinogen and fibrin results
in polypeptides (fibrin degradation products). In a state of homeostasis, the presence of
plasmin is critical, as it is the central proteolytic enzyme of coagulation and is also necessary
for fibrinolysis.

In DIC, the processes of coagulation and fibrinolysis are dysregulated, and the result is
widespread clotting with resultant bleeding. Regardless of the triggering event of DIC, once
initiated, the pathophysiology of DIC is similar in all conditions. One critical mediator of DIC is
the release of a transmembrane glycoprotein (tissue factor =TF). TF is present on the
surface of many cell types (including endothelial cells, macrophages, and monocytes) and is
not normally in contact with the general circulation, but is exposed to the circulation after
vascular damage. For example, TF is released in response to exposure to cytokines
(particularly interleukin 1), tumor necrosis factor, and endotoxin. This plays a major role in
the development of DIC in septic conditions. TF is also abundant in tissues of the lungs,
brain, and placenta. This helps to explain why DIC readily develops in patients with extensive
trauma. Upon activation, TF binds with coagulation factors that then triggers the extrinsic
pathway (via Factor VII) which subsequently triggers the intrinsic pathway (XII to XI to IX) of
coagulation.

Diagnosis
Fibrin degradation products are often raised.

Bleeding Platelet
Disorder Prothrombin time APTT time count

Warfarin Prolonged Normal Normal Normal


administration

Aspirin administration Normal Normal Prolonged Normal

Heparin Often normal (may be Prolonged Normal Normal


prolonged)
Bleeding Platelet
Disorder Prothrombin time APTT time count

DIC Prolonged Prolonged Prolonged Low

 Next question

Question 29 of 347

A 53 year old man from Hong Kong presents with symptoms of fatigue, weight loss and recurrent
epistaxis. Clinical examination reveals left sided cervical lymphadenopathy and oropharyngeal
examination reveals an ulcerated mass in the naso pharynx. Which of the following viral agents is
most commonly implicated in the development of this condition?

Cytomegalovirus

Epstein Barr virus

Coxsackie virus

Herpes simplex virus

None of the above

Trotter's triad (diagnosis of nasopharyngeal carcinoma)

 Unilateral conductive hearing loss


 Ipsilateral facial & ear pain
 Ipsilateral paralysis of soft palate

The clinical scenario is most typical for nasopharyngeal carcinoma. An association with previous
Epstein Barr Virus is well established. Infection with the other viruses listed is not a recognised risk
factor for the development of the condition.
Please rate this question:

Discuss and give feedback


Next question

Nasopharyngeal carcinoma

 Squamous cell carcinoma of the nasopharynx


 Rare in most parts of the world, apart from individuals from Southern China
 Associated with Epstein Barr virus infection

Presenting features
Systemic Local

Cervical lymphadenopathy Otalgia

Unilateral serous otitis media

Nasal obstruction, discharge and/ or epistaxis

Cranial nerve palsies e.g. III-VI

Imaging
Combined CT and MRI.

Treatment
Radiotherapy is first line therapy.
Next question
Question 30 of 347

An 18 year old male presents with lethargy, night sweats and on examination is found to have left
supraclavicular lymphadenopathy. A surgical registrar performs a left supraclavicular lymph node
biopsy. The pathologist identifies Reed- Sternberg cells on the subsequent histology sections, what
is the most likely diagnosis?

Metastatic gastric cancer

Hodgkins lymphoma

Non Hodgkins lymphoma

Tuberculosis

None of the above

Reed-Sternberg cells are characteristic histological cell type found in Hodgkins disease.
Please rate this question:

Discuss and give feedback


Next question

Lymphadenopathy

 Lymphadenopathy in the neck, axillae, groins and abdomen


 Need to note: solitary/multiple, defined/indistinct, hard/rubbery/soft, tender/painless

Causes of lymphadenopathy

Mnemonic: Hodgkins disease

H aematological: Hodgkins lymphoma, NHL, Leukaemia


O ncological: metastases
D ermatopathic lympadenitis
G aucher's disease
K awasaki disease
I nfections: TB, glandular fever, Syphilis
N iemann Pick disease
S erum sickness
D rug reaction (phenytoin)
I mmunological (SLE)
S arcoidosis
E ndocrinological (Hyperthyroidism)
A ngioimmunoplastic lymphadenopathy
S LE
E osinophilic granulomatosis
Next question
Question 31 of 347

A 28 year old man develops an acute paronychia and subsequent spreading sepsis. The tissue
exudate has a higher protein content than normal tissue because?

Breakdown of tissue cells release protein

Capillary walls are more permeable

Increased blood flow transports more protein into the area

Intracapillary pressure is raised

Plasma cells release gamma globulin

The increased permeability allows the exudation of plasma proteins.


Please rate this question:

Discuss and give feedback


Next question

Acute inflammation

Inflammation is the reaction of the tissue elements to injury. Vascular changes occur, resulting in the
generation of a protein rich exudate. So long as the injury does not totally destroy the existing tissue
architecture, the episode may resolve with restoration of original tissue architecture.

Vascular changes

 Vasodilation occurs and persists throughout the inflammatory phase.


 Inflammatory cells exit the circulation at the site of injury.
 The equilibrium that balances Starlings forces within capillary beds is disrupted and a protein
rich exudate will form as the vessel walls also become more permeable to proteins.
 The high fibrinogen content of the fluid may form a fibrin clot. This has several important
immunomodulatory functions.

Sequelae
Resolution  Typically occurs with minimal initial injury
 Stimulus removed and normal tissue architecture results

Organisation  Delayed removal of exudate


 Tissues undergo organisation and usually fibrosis

Suppuration  Typically formation of an abscess or an empyema


 Sequestration of large quantities of dead neutrophils

Progression to chronic  Coupled inflammatory and reparative activities


inflammation  Usually occurs when initial infection or suppuration has
been inadequately managed

Causes

 Infections e.g. Viruses, exotoxins or endotoxins released by bacteria


 Chemical agents
 Physical agents e.g. Trauma
 Hypersensitivity reactions
 Tissue necrosis

Presence of neutrophil polymorphs is a histological diagnostic feature of acute inflammation


Next question
Question 32 of 347

A 40 year old man undergoes a complex appendicectomy and the wound fails to heal satisfactorily.
The wound site itself is associated with multiple sinuses and fistulas. Pus is sent for microbiology
and shows gram positive organisms and sulphur granules. What is the most likely underlying
diagnosis?

Infection with Staphylococcus aureus

Infection with Bacteroides fragilis

Actinomycosis

Crohns disease

Ulcerative colitis

Theme from 2014 Exam


The presence of chronic sinuses together with gram positive organisms and sulphur granules is
highly suggestive of Actinomycosis. Crohns disease is associated with multiple fistulae, but not gram
positive organisms with sulphur granules.
Please rate this question:

Discuss and give feedback


Next question

Actinomycosis

Chronic, progressive granulomatous disease caused by filamentous gram positive anaerobic


bacteria from the Actinomycetaceae family.

Actinomyces are commensal bacteria that become pathogenic when a mucosal barrier is breached.

The disease most commonly occurs in the head and neck, although it may also occur in the
abdominal cavity and in the thorax.

The mass will often enlarge across tissue planes with the formation of multiple sinus tracts.

Abdominopelvic actinomycosis occurs most frequently in individuals that have had appendicitis
(65%) cases.
Pathology

 On histological examination gram positive organisms and evidence of sulphur granules.


 Sulphur granules are colonies of organisms that appear as round or oval basophilic masses.
 They are also seen in other conditions such as nocardiosis.

Treatment

 Long term antibiotic therapy usually with penicillin.


 Surgical resection is indicated for extensive necrotic tissue, non healing sinus tracts,
abscesses or where biopsy is needed to exclude malignancy.

References
Wong V, Turmezei T and Weston V. Actinomycosis. BMJ 2011;343d6099.
Next question
Question 33 of 347

As a busy surgical trainee on the colorectal unit you are given the unenviable task of reviewing the
unit's histopathology results for colonic polyps. Which of the polyp types described below has the
greatest risk of malignant transformation?

Hyperplastic polyp

Tubular adenoma

Villous adenoma

Hamartomatous polyp

Serrated polyp

Villous adenomas carry the highest risk of malignant transformation. Hyperplastic polyps carry little
in the way of increased risk. Although, patients with hamartomatous polyp syndromes may have a
high risk of malignancy, the polyps themselves have little malignant potential.

Please rate this question:

Discuss and give feedback

Next question

Colonic polyps

Colonic Polyps
May occur in isolation, or greater numbers as part of the polyposis syndromes. In FAP greater than
100 polyps are typically present. The risk of malignancy in association with adenomas is related to
size, and is the order of 10% in a 1cm adenoma. Isolated adenomas seldom give risk of symptoms
(unless large and distal). Distally sited villous lesions may produce mucous and if very large,
electrolyte disturbances may occur.
Follow up of colonic polyps

Group Features Action

Low risk 1 or 2 adenomas less than 1cm No follow up or re-colonoscopy


at 5 years

Moderate 3 or 4 small adenomas or 1 adenoma greater than 1cm Re-scope at 3 years


risk

High risk More than 5 small adenomas or more than 3 with 1 of Re scope at 1 year
them greater than 1cm

From Atkins and Saunders Gut 2002 51 (suppl V:V6-V9). It is important to stratify patients
appropriately and ensure that a complete colonoscopy with good views was performed.

Segmental resection or complete colectomy should be considered when:

1. Incomplete excision of malignant polyp


2. Malignant sessile polyp
3. Malignant pedunculated polyp with submucosal invasion
4. Polyps with poorly differentiated carcinoma
5. Familial polyposis coli
-Screening from teenager up to 40 years by 2 yearly sigmoidoscopy/colonoscopy
-Panproctocolectomy and Ileostomy or Restorative Panproctocolectomy.

Rectal polypoidal lesions may be amenable to trans anal endoscopic microsurgery.

References
Cairns S et al. Guidelines for colorectal cancer screening and surveillance in moderate and high risk
groups (update from 2002). Gut 2010;59:666-690.

Next question
Question 34 of 347

A 23 year old man presents to the surgical clinic with an inguinal hernia. On examination he has a
small direct hernia. However, you also notice that he has pigmented spots around his mouth, on his
palms and soles. In his history he underwent a reduction of an intussusception aged 12 years.
Which of the following lesions is most likely to be identified if a colonoscopy were performed?

Hamartomas

Tubulovillous adenoma

Colorectal cancer

Crohns disease

Hyperplastic polyps

Theme from April 2014 Exam


He is most likely to have Peutz-Jeghers syndrome which is associated with Hamartomas.
Please rate this question:

Discuss and give feedback


Next question

Peutz-Jeghers syndrome

Peutz-Jeghers syndrome is an autosomal dominant condition characterised by numerous benign


hamartomatous polyps in the gastrointestinal tract. It is also associated with pigmented freckles on
the lips, face, palms and soles. Around 50% of patients will have died from a gastrointestinal tract
cancer by the age of 60 years.

Genetics

 Autosomal dominant
 Responsible gene encodes serine threonine kinase LKB1 or STK11

Features
 Hamartomatous polyps in GI tract (mainly small bowel)
 Pigmented lesions on lips, oral mucosa, face, palms and soles
 Intestinal obstruction e.g. intussusception (which may lead to diagnosis)
 Gastrointestinal bleeding

Management

 Conservative unless complications develop

Next question
Question 35 of 347

A 56 year old surgeon has been successfully operating for many years. Over the past few weeks
she has begun to notice that her hands are becoming blistering and weepy. A latex allergy is
diagnosed. Which of the following pathological processes accounts for this scenario?

Type 1 hypersensitivity reaction

Type 2 hypersensitivity reaction

Type 4 hypersensitivity reaction

Type 3 hypersensitivity reaction

None of the above

Hypersensitivity reactions: ACID

type 1 --Anaphylactic
type 2 --Cytotoxic
type 3 --Immune complex
type 4 --Delayed hypersensitivity

Theme from 2012 Exam


Contact dermatitis of a chronic nature is an example of a type 4 hypersensitivity reaction. Type 4
hypersensitivity reactions are cell mediated rather than antibody mediated.

Please rate this question:

Discuss and give feedback

Next question

Hypersensitivity reactions
The Gell and Coombs classification divides hypersensitivity reactions into 4 types

Type I Type II Type III Type IV

Description Anaphylactic Cytotoxic Immune Delayed type


complex

Mediator IgE IgG, IgM IgG, Ig A, IgM T-cells

Antigen Exogenous Cell surface Soluble Tissues

Response Minutes Hours Hours 2-3 days


time

Examples Asthma Autoimmune haemolytic Serum sickness Graft versus host


Hay fever anaemia SLE disease
Pemphigus Aspergillosis Contact dermatitis
Goodpasture's

Next question
Question 36 of 347

A 56 year old motorcyclist is involved in a road traffic accident and sustains a displaced femoral
shaft fracture. No other injuries are identified on the primary or secondary surveys. The fracture is
treated with closed, antegrade intramedullary nailing. The following day the patient becomes
increasingly agitated and confused. On examination he is pyrexial, hypoxic SaO 2 90% on 6 litres O2,
tachycardic and normotensive. Systemic examination demonstrates a non blanching petechial rash
present over the torso. What is the most likely explanation for this?

Pulmonary embolism with paradoxical embolus

Fat embolism

Meningococcal sepsis

Alcohol withdrawl

Chronic sub dural haematoma

Triad of symptoms:

 Respiratory
 Neurological
 Petechial rash (tends to occur after the first 2 symptoms)

Theme from April 2014 Exam


Theme from January 2015 Exam
This man has a recent injury and physical signs that would be concordant with fat embolism
syndrome. Meningococcal sepsis is not usually associated with hypoxia initially. Pulmonary emboli
are not typically associated with pyrexia.
Please rate this question:

Discuss and give feedback


Next question

Fat embolism

Diagnosis and clinical features


System Feature

Respiratory  Early persistent tachycardia


 Tachypnoea, dyspnoea, hypoxia usually 72 hours following injury
 Pyrexia

Dermatological  Red/ brown impalpable petechial rash (usually only in 25-50%)


 Subconjunctival and oral haemorrhage/ petechiae

CNS  Confusion and agitation


 Retinal haemorrhages and intra-arterial fat globules on fundoscopy

Imaging

 May be normal
 Fat emboli tend to lodge distally and therefore CTPA may not show any vascular occlusion, a
ground glass appearance may be seen at the periphery

Treatment

 Prompt fixation of long bone fractures


 Some debate regarding benefit Vs. risk of medullary reaming in femoral shaft/ tibial fractures
in terms of increasing risk (probably does not).
 DVT prophylaxis
 General supportive care

Next question
Question 37 of 347

A 45 year old man has widespread metastatic adenocarcinoma of the colon. Which of the following
tumour markers is most likely to be elevated?

CA19-9

Carcinoembryonic antigen

Alpha Feto Protein

CA 125

Beta HCG

Screening for colonic cancer using CEA is not justified

Carcinoembryonic antigen is elevated in colonic cancer, typically in relation to disease extent with
highest serum levels noted in metastatic disease. It is falsely elevated in a number of non-malignant
disease states such as cirrhosis and colitis and for this reason it has no role in monitoring colitics for
colonic cancer[1].
Reference
1. Sturgeon, C.M., L.C. Lai, and M.J. Duffy. Serum tumour markers: how to order and interpret them.
BMJ, 2009. 339: p. b3527.
Please rate this question:

Discuss and give feedback


Next question

Colorectal cancer screening and diagnosis

Overview

 Most cancers develop from adenomatous polyps. Screening for colorectal cancer has been
shown to reduce mortality by 16%
 The NHS now has a national screening programme offering screening every 2 years to all
men and women aged 60 to 69 years. Patients aged over 70 years may request screening
 Eligible patients are sent faecal occult blood (FOB) tests through the post
 Patients with abnormal results are offered a colonoscopy
At colonoscopy, approximately:

 5 out of 10 patients will have a normal exam


 4 out of 10 patients will be found to have polyps which may be removed due to their
premalignant potential
 1 out of 10 patients will be found to have cancer

Diagnosis
Essentially the following patients need referral:
- Altered bowel habit for more than six weeks
- New onset of rectal bleeding
- Symptoms of tenesmus

Colonoscopy is the gold standard, provided it is complete and good mucosal visualisation is
achieved. Other options include double contrast barium enema and CT colonography.

Staging
Once a malignant diagnosis is made patients with colonic cancer will be staged using chest /
abdomen and pelvic CT. Patients with rectal cancer will also undergo evaluation of the mesorectum
with pelvic MRI scanning.

For examination purposes the Dukes and TNM systems are preferred.

Tumour markers
Carcinoembryonic antigen (CEA) is the main tumour marker in colorectal cancer. Not all tumours
secrete this, and it may be raised in conditions such as IBD. However, absolute levels do correlate
(roughly) with disease burden and it is once again being used routinely in follow up.
Next question
Question 38 of 347

A 43 year old man presents with haemoptysis and is diagnosed as having tuberculosis. Which of the
cell types listed below will usually internalise the tubercule bacullis?

Fibroblast

Neutrophil

Erythrocyte

Macrophage

Eosinophil

Theme from January 2013 Exam


M. Tuberculosis will reside in macrophages where it will often survive
Please rate this question:

Discuss and give feedback


Next question

Tuberculosis pathology

 Is a form of primary chronic inflammation, caused by the inability of macrophages to kill


theMycobacterium tuberculosis.
 The macrophages often migrate to regional lymph nodes, the lung lesion plus affected lymph
nodes is referred to as a Ghon complex.
 This leads to the formation of a granuloma which is a collection of epithelioid histiocytes.
 There is the presence of caseous necrosis in the centre.
 The inflammatory response is mediated by a type 4 hypersensitivity reaction.
 In healthy individuals the disease may be contained, in the immunocompromised
disseminated (miliary TB) may occur.

Diagnosis

 Waxy membrane of mycobacteria prevents binding with normal stains. Ziehl - Neelsen
staining is typically used.
 Culture based methods take far longer.
Image showing acid- alcohol fast mycobacteria stained using the Ziehl- Neelsen method

Image sourced from Wikipedia

Next question
Question 39 of 347

Which of these tumour markers is most helpful in identifying an individual with hepatocellular
carcinoma?

Serum AFP

Serum CA19-9

CEA

Beta HCG

CA125

Theme from April 2014 Exam


Hepatocellular carcinoma is commonly diagnosed with imaging and an elevated alpha fetoprotein.
Biopsy may seed the tumour and should be avoided. Up to 80% of hepatocellular carcinoma arise in
cirrhotic livers.
Please rate this question:

Discuss and give feedback


Next question

Liver tumours

Primary liver tumours


The most common primary tumours are cholangiocarcinoma and hepatocellular carcinoma. Overall
metastatic disease accounts for 95% of all liver malignancies making the primary liver tumours
comparatively rare.

Primary liver tumours include:

 Cholangiocarcinoma
 Hepatocellular carcinoma
 Hepatoblastoma
 Sarcomas (Rare)
 Lymphomas
 Carcinoids (most often secondary although primary may occur)
Hepatocellular carcinoma
These account for the bulk of primary liver tumours (75% cases). Its worldwide incidence reflects its
propensity to occur on a background of chronic inflammatory activity. Most cases arise in cirrhotic
livers or those with chronic hepatitis B infection, especially where viral replication is actively
occurring. In the UK it accounts for less than 5% of all cancers, although in parts of Asia its
incidence is 100 per 100,000.
The majority of patients (80%) present with existing liver cirrhosis, with a mass discovered on
screening ultrasound.

Diagnosis

 CT/ MRI (usually both) are the imaging modalities of choice


 a-fetoprotein is elevated in almost all cases
 Biopsy should be avoided as it seeds tumours cells through a resection plane.
 In cases of diagnostic doubt serial CT and αFP measurements are the preferred strategy.

Treatment

 Patients should be staged with liver MRI and chest, abdomen and pelvic CT scan.
 The testis should be examined in males (testicular tumours may cause raised AFP). PET CT
may be used to identify occult nodal disease.
 Surgical resection is the mainstay of treatment in operable cases. In patients with a small
primary tumour in a cirrhotic liver whose primary disease process is controlled, consideration
may be given to primary whole liver resection and transplantation.
 Liver resections are an option but since most cases occur in an already diseased liver the
operative risks and post-operative hepatic dysfunction are far greater than is seen following
metastectomy.
 These tumours are not particularly chemo or radiosensitive however, both may be used in a
palliative setting. Tumour ablation is a more popular strategy.

Survival
Poor, overall survival is 15% at 5 years.

Cholangiocarcinoma
This is the second most common type of primary liver malignancy. As its name suggests these
tumours arise in the bile ducts. Up to 80% of tumours arise in the extra hepatic biliary tree. Most
patients present with jaundice and by this stage the majority will have disease that is not resectable.
Primary sclerosing cholangitis is the main risk factor. In deprived countries typhoid and liver flukes
are also major risk factors.

Diagnosis

 Patients will typically have an obstructive picture on liver function tests.


 CA 19-9, CEA and CA 125 are often elevated
 CT/ MRI and MRCP are the imaging methods of choice.
Treatment

 Surgical resection offers the best chance of cure. Local invasion of peri hilar tumours is a
particular problem and this coupled with lobar atrophy will often contra indicate surgical
resection.
 Palliation of jaundice is important, although metallic stents should be avoided in those
considered for resection.

Survival
Is poor, approximately 5-10% 5 year survival.
Next question
Question 40 of 347

A 39 year old man has suffered from terminal ileal Crohns disease for the past 20 years. Which
condition is he least likely to develop?

Gallstones

Malabsorption

Pyoderma gangrenosum

Amyloidosis

Feltys syndrome

Felteys syndrome:

 Rheumatoid disease
 Splenomegaly
 Neutropenia

Feltys syndrome is associated with rheumatoid disease. Individuals with long standing Crohns
disease are at risk of gallstones because of impairment of the enterohepatic recycling of bile salts.
Formation of entero-enteric fistulation may produce malabsorption. Amyloidosis may complicate
chronic inflammatory states.
Please rate this question:

Discuss and give feedback


Next question

Crohns disease

Crohns disease is a chronic transmural inflammation of a segment(s) of the gastrointestinal tract and
may be associated with extra intestinal manifestations. Frequent disease patterns observed include
ileal, ileocolic and colonic disease. Peri-anal disease may occur in association with any of these. The
disease is often discontinuous in its distribution. Inflammation may cause ulceration, fissures, fistulas
and fibrosis with stricturing. Histology reveals a chronic inflammatory infiltrate that is usually patchy
and transmural.

Ulcerative colitis Vs Crohns


Crohn's disease Ulcerative colitis

Distribution Mouth to anus Rectum and colon

Macroscopic Cobblestone appearance, apthoid ulceration Contact bleeding


changes

Depth of disease Transmural inflammation Superficial inflammation

Distribution Patchy Continuous


pattern

Histological Granulomas (non caseating epithelioid cell Crypt abscesses, Inflammatory


features aggregates with Langerhans' giant cells) cells in the lamina propria

Extraintestinal manifestations of Crohns


Related to disease extent Unrelated to disease extent

Aphthous ulcers (10%) Sacroiliiitis (10-15%)

Erythema nodosum (5-10%) Ankylosing spondylitis (1-2%)

Pyoderma gangrenosum (0.5%) Primary sclerosing cholangitis (Rare)

Acute arthropathy (6-12%) Gallstones (up to 30%)

Ocular complications (up to 10%) Renal calculi (up to 10%)

Diarrhoea in Crohns
Diarrhoea in Crohns may be multifactorial since actual inflammation of the colon is not common.
Causes therefore include the following:
 Bile salt diarrhoea secondary to terminal ileal disease
 Entero-colic fistula
 Short bowel due to multiple resections
 Bacterial overgrowth

Surgical interventions in Crohns disease


The commonest disease pattern in Crohns is stricturing terminal ileal disease and this often
culminates in an ileocaecal resection. Other procedures performed include segmental small bowel
resections and stricturoplasty. Colonic involvement in patients with Crohns is not common and,
where found, distribution is often segmental. However, despite this distribution segmental resections
of the colon in patients with Crohns disease are generally not advocated because the recurrence
rate in the remaining colon is extremely high, as a result the standard options of colonic surgery in
Crohns patients are generally; sub total colectomy, panproctocolectomy and staged sub total
colectomy and proctectomy. Restorative procedures such as ileoanal pouch have no role in therapy.
Crohns disease is notorious for the developmental of intestinal fistulae; these may form between the
rectum and skin (peri anal) or the small bowel and skin. Fistulation between loops of bowel may also
occur and result in bacterial overgrowth and malabsorption. Management of enterocutaneous
fistulae involves controlling sepsis, optimising nutrition, imaging the disease and planning definitive
surgical management.
Next question
Question 41 of 347

A 23 year old women has undergone a pan proctocolectomy and ileoanal pouch because she
suffers from familial adenomatous polyposis coli. What is the commonest extra colonic lesion in this
disorder?

Gastric fundal polyps

Trichilemmomas

Duodenal polyps

Fibrocystic disease of the breast

Skull osteomas

Duodenal polyps occur in up to 100% of patients with FAP if follow up is continued for long enough.
Duodenal cancer has an incidence of 4-10%.

Theme from April 2014 exam


Duodenal polyps are the commonest extra colonic lesion in FAP. Gastric fundal polyps are seen in
50% of patients. Skull osteomas are seen in Gardeners syndrome which is a variant of FAP.

Please rate this question:

Discuss and give feedback

Next question

Polyposis syndromes
Screening and Associated
Syndrome Genetic defect Features management disorders

Familial Mutation of APC Typically over 100 If known to be at risk Gastric fundal
adenomatous gene (80%) cases, colonic adenomas then predictive polyps (50%).
polyposis dominant Cancer risk of 100% genetic testing as Duodenal polyps
20% are new teenager 90%.
mutations Annual flexible If severe
sigmoidoscopy from duodenal
15 years polyposis cancer
If no polyps found risk of 30% at 10
then 5 yearly years.
colonoscopy started Abdominal
at age 20 desmoid
Polyps found = tumours.
resectional surgery
(resection and pouch
Vs sub total
colectomy and IRA)

MYH Biallelic mutation Multiple colonic Once identified Duodenal


associated of mut Y human polyps resection and polyposis in 30%
polyposis homologue (MYH) Later onset right ileoanal pouch Associated with
on chromosome sided cancers more reconstruction is increased risk of
1p, recessive common than in FAP recommended breast cancer
100% cancer risk by Attenuated (self examination)
age 60 phenotype - regular
colonoscopy

Peutz -Jeghers STK11 (LKB1) Multiple benign Annual examination Malignancies at


syndrome mutation on intestinal Pan intestinal other sites
chromosome 19 in hamartomas endoscopy every 2-3 Classical
some (but not all) Episodic obstruction years pigmentation
cases, dominant and intussceception pattern
Increased risk of GI
cancers (colorectal
cancer 20%, gastric
5%)
Screening and Associated
Syndrome Genetic defect Features management disorders

Increased risk of
breast, ovarian,
cervical pancreatic
and testicular
cancers

Cowden Mutation of PTEN Macrocephaly Targeted Breast cancer


disease gene on Multiple intestinal individualised (81% risk)
chromosome hamartomas screening Thyroid cancer
10q22, dominant Multiple and non toxic
trichilemmomas goitre
89% risk of cancer at Uterine cancer
any site
16% risk of colorectal
cancer

HNPCC (Lynch Germline Colo rectal cancer Colonoscopy every 1- Extra colonic
syndrome) mutations of DNA 30-70% 2 years from age 25 cancers
mismatch repair Endometrial cancer Consideration of
genes 30-70% prophylactic surgery
Gastric cancer 5-10% Extra colonic
Scanty colonic polyps surveillance
may be present recommended
Colonic tumours
likely to be right
sided and mucinous

Next question
Question 42-44 of 347

Theme: Renal stones

A. Calcium oxalate

B. Uric acid

C. Cystine

D. Struvite

E. Calcium phosphate

Please select the most likely stone type for each of the following urinary tract stone scenarios. Each
option may be used once, more than once or not at all.

42. A 73 year old lady is undergoing chemotherapy for treatment of acute leukaemia. She develops
symptoms of renal colic. Her urine tests positive for blood. A KUB x-ray shows no evidence of
stones.

You answered Calcium oxalate

The correct answer is Uric acid

Chemotherapy and cell death can increase uric acid levels. In this acute setting the uric acid
stones are unlikely to be coated with calcium and will therefore be radiolucent.

43. A 16 year old boy presents with renal colic. His parents both have a similar history of the
condition. His urine tests positive for blood. A KUB style x-ray shows a relatively radiodense stone
in the region of the mid ureter.

You answered Calcium oxalate

The correct answer is Cystine


Cystine stones are associated with an inherited metabolic disorder.

44. A 43 year old lady with episodes of recurrent urinary tract sepsis presents with a staghorn
calculus of the left kidney. Her urinary pH is 7.3. A KUB x-ray shows a faint outline of the calculus.

You answered Calcium oxalate

The correct answer is Struvite

Theme from April 2012 Exam


Chronic infection with urease producing enzymes can produce an alkaline urine with formation of
struvite stone.

Please rate this question:

Discuss and give feedback

Next question

Renal stones

Type of Percentage of
stones Features all calculi

Calcium Hypercalciuria is a major risk factor (various causes) 85%


oxalate Hyperoxaluria may also increase risk
Hypocitraturia increases risk because citrate forms complexes with
calcium making it more soluble
Stones are radio-opaque (though less than calcium phosphate stones)
Hyperuricosuria may cause uric acid stones to which calcium oxalate
binds
Type of Percentage of
stones Features all calculi

Cystine Inherited recessive disorder of transmembrane cystine transport 1%


leading to decreased absorption of cystine from intestine and renal
tubule
Multiple stones may form
Relatively radiodense because they contain sulphur

Uric acid Uric acid is a product of purine metabolism 5-10%


May precipitate when urinary pH low
May be caused by diseases with extensive tissue breakdown e.g.
malignancy
More common in children with inborn errors of metabolism
Radiolucent

Calcium May occur in renal tubular acidosis, high urinary pH increases 10%
phosphate supersaturation of urine with calcium and phosphate
Renal tubular acidosis types 1 and 3 increase risk of stone formation
(types 2 and 4 do not)
Radio-opaque stones (composition similar to bone)

Struvite Stones formed from magnesium, ammonium and phosphate 2-20%


Occur as a result of urease producing bacteria (and are thus
associated with chronic infections)
Under the alkaline conditions produced, the crystals can precipitate
Slightly radio-opaque

Effect of urinary pH on stone formation


Urine pH will show individual variation (from pH 5-7). Post prandially the pH falls as purine
metabolism will produce uric acid. Then the urine becomes more alkaline (alkaline tide). When the
stone is not available for analysis the pH of urine may help to determine which stone was present.

Stone type Urine acidity Mean urine pH


Stone type Urine acidity Mean urine pH

Calcium phosphate Normal- alkaline >5.5

Calcium oxalate Variable 6

Uric acid Acid 5.5

Struvate Alkaline >7.2

Cystine Normal 6.5

Next question
Question 45 of 347

A pathologist is examining a histological section and identifies Hassall's corpuscles. With what are
they most commonly associated?

Follicular carcinoma of the thyroid

Medulla of the thymus

Medulla of the spleen

Medulla of the kidney

Fundus of the stomach

Theme from September 2015 Exam


Hassall's corpuscles are the concentric ring of epithelial cells seen in the medulla of the thymus.

Please rate this question:

Discuss and give feedback

Next question

Thymus

The thymus develops from the third and fourth pharyngeal pouches. It descends to lie in the anterior
superior mediastinum. It is encapsulated and is subdivided into lobules, these consist of a cortex and
a medulla. The cortex is composed of tightly packed lymphocytes, the medulla consists largely of
epithelial cells. The medullary epithelial cells are concentrically arranged and may surround a
keratinised centre, known as Hassall's corpuscles.
The inferior parathyroid glands also develop from the third pharyngeal pouch and may also be
located with the thymus gland.
Its arterial supply is from the internal mammary artery or pericardiophrenic arteries. Venous drainage
is to the left brachiocephalic vein.

Hassall's corpuscles stained with H+E

Image sourced from Wikipedia

Next question
Question 46 of 347

A 64 year old man presents to the clinic with right upper quadrant discomfort. He has never attended
the hospital previously and is usually well. He has just retired from full time employment as a
machinist in a PVC factory. CT scanning shows a large irregular tumour in the right lobe of his liver.
Which of the following lesions is the most likely?

Liposarcoma

Angiosarcoma

Hamartoma

Hyatid liver disease

Benign angioma

Angiosarcoma of the liver is a rare tumour. However, it is linked to working with vinyl chloride, as in
this case. Although modern factories minimise the exposure to this agent, this has not always been
the case.
Please rate this question:

Discuss and give feedback


Next question

Occupational cancers

Occupational cancers accounted for 5.3% cancer deaths in 2005.


In men the main cancers include:

 Mesothelioma
 Bladder cancer
 Non melanoma skin cancer
 Lung cancer
 Sino nasal cancer

Occupations with high levels of occupational tumours include:


 Construction industry
 Working with coal tar and pitch
 Mining
 Metalworkers
 Working with asbestos (accounts for 98% of all mesotheliomas)
 Working in rubber industry

Shift work has been linked to breast cancer in women (Health and safety executive report RR595).

The latency between exposure and disease is typically 15 years for solid tumours and 20 for
leukaemia.

Many occupational cancers are otherwise rare. For example sino nasal cancer is an uncommon
tumour, 50% will be SCC. They are linked to conditions such as wood dust exposure and unlike lung
cancer is not strongly linked to cigarette smoking. Another typical occupational tumour is
angiosarcoma of the liver which is linked to working with vinyl chloride. Again in the non occupational
context this is an extremely rare sporadic tumour.
Next question
Question 47 of 347

A 32 year old man is involved in a house fire and sustains extensive partial thickness burns to his
torso and thigh. Two weeks post operatively he develops oedema of both lower legs. The most likely
cause of this is:

Iliofemoral deep vein thrombosis

Venous obstruction due to scarring

Hypoalbuminaemia

Excessive administration of intravenous fluids

None of the above

Theme from 2009 Exam

Loss of plasma proteins is the most common cause of oedema developing in this time frame.
Please rate this question:

Discuss and give feedback


Next question

Burns pathology

Extensive burns

 Haemolysis due to damage of erythrocytes by heat and microangiopathy


 Loss of capillary membrane integrity causing plasma leakage into interstitial space
 Extravasation of fluids from the burn site causing hypovolaemic shock (up to 48h after
injury)- decreased blood volume and increased haematocrit
 Protein loss
 Secondary infection e.g. Staphylococcus aureus
 ARDS
 Risk of Curlings ulcer (acute peptic stress ulcers)
 Danger of full thickness circumferential burns in an extremity as these may develop
compartment syndrome
Healing

 Superficial burns: keratinocytes migrate to form a new layer over the burn site
 Full thickness burns: dermal scarring. Usually need keratinocytes from skin grafts to provide
optimal coverage.

Next question
Question 48 of 347

A 42 year old man from Southern India presents with chronic swelling of both lower legs, they are
brawny and indurated with marked skin trophic changes. Which of the following organisms is the
most likely origin of this disease process?

Loa loa

Wuchereria bancrofti

Trypanosoma cruzi

Trypanosoma gambiense

None of the above

W. Bancrofti is the commonest cause of filariasis leading to lymphatic obstruction. Infection with Loa
loatypically occurs in the African sub continent and usually results in generalised sub cutaneous
infections without lymphatic obstruction. Trypanosomal infections would not produce this clinical
picture.
Please rate this question:

Discuss and give feedback


Next question

Wuchereria bancrofti

 Parasitic filarial nematode


 Accounts for 90% of cases of filariasis
 Usually diagnosed by blood smears
 Usually transmitted by mosquitos
 Treatment is with diethylcarbamazine

Next question
Question 49 of 347

A 45 year old lady has recently undergone a thyroidectomy for treatment of medullary thyroid
cancer. Which of the following tumour markers is used clinically to screen for recurrence?

Free T3

Thyroglobulin

Calcitonin

Free T4

Thyroid stimulating hormone

Theme from 2011 Exam


Calcitonin is clinically utilised to screen for medullary thyroid cancer recurrence. Thyroid function
testing does not form part of either diagnosis or follow up from a malignancy perspective. However,
routine assessment of TSH may be needed in patients on thyroxine.
Please rate this question:

Discuss and give feedback


Next question

Thyroid malignancy

Papillary carcinoma

 Commonest sub-type
 Accurately diagnosed on fine needle aspiration cytology
 Histologically, they may demonstrate psammoma bodies (areas of calcification) and so
called 'orphan Annie' nuclei
 They typically metastasise via the lymphatics and thus laterally located apparently ectopic
thyroid tissue is usually a metastasis from a well differentiated papillary carcinoma

Follicular carcinoma

 Are less common than papillary lesions


 Like papillary tumours, they may present as a discrete nodule. Although they appear to be
well encapsulated macroscopically there is invasion on microscopic evaluation
 Lymph node metastases are uncommon and these tumours tend to spread
haematogenously. This translates into a higher mortality rate
 Follicular lesions cannot be accurately diagnosed on fine needle aspiration cytology and thus
all follicular FNA's (THY 3f) will require at least a hemi thyroidectomy

Anaplastic carcinoma

 Less common and tend to occur in elderly females


 Disease is usually advanced at presentation and often only palliative decompression and
radiotherapy can be offered.

Medullary carcinoma

 These are tumours of the parafollicular cells ( C Cells) and are of neural crest origin.
 The serum calcitonin may be elevated which is of use when monitoring for recurrence.
 They may be familial and occur as part of the MEN -2A disease spectrum.
 Spread may be either lymphatic or haematogenous and as these tumours are not derived
primarily from thyroid cells they are not responsive to radioiodine.

Lymphoma

 These respond well to radiotherapy


 Radical surgery is unnecessary once the disease has been diagnosed on biopsy material.
Such biopsy material is not generated by an FNA and thus a core biopsy has to be obtained
(with care!).

Next question
Question 50 of 347

A 22 year old man is kicked in the head during a rugby match. He is temporarily concussed, but then
regains consciousness. Half an hour later he develops slurred speech, ataxia and loses
consciousnesses. On arrival in hospital he is intubated and ventilated. A CT Scan is performed
which shows an extradural haematoma. What is the most likely cause?

Basilar artery laceration

Middle meningeal artery laceration

Laceration of the sigmoid sinus

Laceration of the anterior cerebral artery

Laceration of the middle cerebral artery

Theme from April 2014 Exam


The most likely vessel from those in the list to cause an acute extra dural haemorrhage is the middle
meningeal artery. The anterior and middle cerebral arteries may cause acute sub dural
haemorrhage. Acute sub dural haemorrhages usually take slightly longer to evolve than acute extra
dural haemorrhages.
Please rate this question:

Discuss and give feedback


Next question

Middle meningeal artery

 Middle meningeal artery is typically the third branch of the first part of the maxillary artery,
one of the two terminal branches of the external carotid artery. After branching off the
maxillary artery in the infratemporal fossa, it runs through the foramen spinosum to supply
the dura mater (the outermost meninges) .
 The middle meningeal artery is the largest of the three (paired) arteries which supply the
meninges, the others being the anterior meningeal artery and the posterior meningeal artery.
 The middle meningeal artery runs beneath the pterion. It is vulnerable to injury at this point,
where the skull is thin. Rupture of the artery may give rise to an extra dural hematoma.
 In the dry cranium, the middle meningeal, which runs within the dura mater surrounding the
brain, makes a deep indention in the calvarium.
 The middle meningeal artery is intimately associated with the auriculotemporal nerve which
wraps around the artery making the two easily identifiable in the dissection of human
cadavers and also easily damaged in surgery.

Next question
Question 51 of 347

Which of the following is not characteristic of a granuloma?

Altered macrophages

Fused macrophages

Epithelioid cells

Mixture of chronic inflammatory cells

Polymorphnuclear leucocytes, cellular debris and fibrin

These are typical components of an abscess cavity. Polymorphonuclear leucocytes may be found in
a granuloma if there is a focus of suppuration.
Please rate this question:

Discuss and give feedback


Next question

Chronic inflammation

Overview
Chronic inflammation may occur secondary to acute inflammation.In most cases chronic
inflammation occurs as a primary process. These may be broadly viewed as being one of three main
processes:

 Persisting infection with certain organisms such as Mycobacterium tuberculosis which results
in delayed type hypersensitivity reactions and inflammation.
 Prolonged exposure to non-biodegradable substances such as silica or suture materials
which may induce an inflammatory response.
 Autoimmune conditions involving antibodies formed against host antigens.

Acute vs. Chronic inflammation


Acute inflammation Chronic inflammation
Changes to existing vascular structure and increased Angiogenesis predominates
permeability of endothelial cells

Infiltration of neutrophils Macrophages, plasma cells and


lymphocytes predominate

Process may resolve with: Healing by fibrosis is the main result

 Suppuration
 Complete resolution
 Abscess formation
 Progression to chronic inflammation
 Healing by fibrosis

Granulomatous inflammation
A granuloma consists of a microscopic aggregation of macrophages (with epithelial type
arrangement =epitheliod). Large giant cells may be found at the periphery of granulomas.

Mediators
Growth factors released by activated macrophages include agents such as interferon and fibroblast
growth factor (plus many more). Some of these such as interferons may have systemic features
resulting in systemic symptoms and signs, which may be present in individuals with long standing
chronic inflammation.

The finding of granulomas is pathognomonic of chronic inflammation, as illustrated in this biopsy


from a patient with colonic Crohns disease

Image sourced from Wikipedia


Question 52 of 347

A 42 year old man presents with a painless lump in the left testicle that he noticed on self
examination. Clinically there is a firm nodule in the left testicle, ultrasound appearances show an
irregular mass lesion. His serum AFP and HCG levels are both within normal limits. What is the most
likely diagnosis?

Yolk sack tumour

Seminoma

Testicular teratoma

Epididymo-orchitis

Adenomatoid tumour

Seminomas typically have normal AFP and HCG. These are usually raised in teratomas and yolk
sac tumours

This man's age, presenting symptoms and normal tumour markers make a seminoma the most
likelydiagnosis. Epididymo-orchitis does not produce irregular mass lesions which are painless.
Please rate this question:

Discuss and give feedback


Next question

Testicular disorders

Testicular cancer
Testicular cancer is the most common malignancy in men aged 20-30 years. Around 95% of cases
of testicular cancer are germ-cell tumours. Germ cell tumours may essentially be divided into:

Tumour
Tumour type Key features markers Pathology

Seminoma  Commonest  AFP usually Sheet like lobular


subtype (50%) normal patterns of cells
 Average age at with substantial
Tumour
Tumour type Key features markers Pathology

diagnosis = 40  HCG elevated fibrous


 Even advanced in 10% component.
disease associated seminomas Fibrous septa
with 5 year  Lactate contain
survival of 73% dehydrogenase; lymphocytic
elevated in 10- inclusions and
20% seminomas granulomas may
(but also in many be seen.
other conditions)

Non seminomatous germ  Younger age at  AFP elevated Heterogenous


cell tumours (42%) presentation =20-30 in up to 70% of texture with
years cases occasional ectopic
 Advanced disease  HCG elevated tissue such as hair
 Teratoma carries worse prognosis in up to 40% of
 Yolk sac tumour (48% at 5 years) cases
 Choriocarcinoma  Retroperitoneal lymph  Other markers
 Mixed germ cell node dissection may be rarely helpful
tumours (10%) needed for residual
disease after
chemotherapy

Image demonstrating a classical seminoma, these tumours are typically more uniform than
teratomas
Image sourced from Wikipedia

Risk factors for testicular cancer

 Cryptorchidism
 Infertility
 Family history
 Klinefelter's syndrome
 Mumps orchitis

Features
 A painless lump is the most common presenting symptom
 Pain may also be present in a minority of men
 Other possible features include hydrocele, gynaecomastia

Diagnosis

 Ultrasound is first-line
 CT scanning of the chest/ abdomen and pelvis is used for staging
 Tumour markers (see above) should be measured

Management

 Orchidectomy (Inguinal approach)


 Chemotherapy and radiotherapy may be given depending on staging
 Abdominal lesions >1cm following chemotherapy may require retroperitoneal lymph node
dissection.

Prognosis is generally excellent

 5 year survival for seminomas is around 95% if Stage I


 5 year survival for teratomas is around 85% if Stage I

Benign disease

Epididymo-orchitis
Acute epididymitis is an acute inflammation of the epididymis, often involving the testis and usually
caused by bacterial infection.

 Infection spreads from the urethra or bladder. In men <35 years, gonorrhoea or chlamydia
are the usual infections.
 Amiodarone is a recognised non infective cause of epididymitis, which resolves on stopping
the drug.
 Tenderness is usually confined to the epididymis, which may facilitate differentiating it from
torsion where pain usually affects the entire testis.

Testicular torsion

 Twist of the spermatic cord resulting in testicular ischaemia and necrosis.


 Most common in males aged between 10 and 30 (peak incidence 13-15 years)
 Pain is usually severe and of sudden onset.
 Cremasteric reflex is lost and elevation of the testis does not ease the pain.
 Treatment is with surgical exploration. If a torted testis is identified then both testis should be
fixed as the condition of bell clapper testis is often bilateral.

Hydrocele

 Presents as a mass that transilluminates, usually possible to "get above" it on examination.


 In younger men it should be investigated with USS to exclude tumour.
 In children it may occur as a result of a patent processus vaginalis.
 Treatment in adults is with a Lords or Jabouley procedure.
 Treatment in children is with trans inguinal ligation of PPV.

Next question
Question 53 of 347

A baby is born by normal vaginal delivery at 39 weeks gestation. Initially all appears well and then
the clinical staff become concerned because the baby develops recurrent episodes of cyanosis.
These are worse during feeding and improve dramatically when the baby cries. The most likely
underlying diagnosis is:

Choanal atresia

Oesophageal reflux

Tetralogy of Fallot

Oesophageal atresia

Congenital diaphragmatic hernia

Theme from 2011 exam


Theme from April 2013 Exam
In Choanal atresia the episodes of cyanosis are usually worst during feeding. Improvement may be
seen when the baby cries as the oropharyngeal airway is used.
Please rate this question:

Discuss and give feedback


Next question

Choanal atresia

 Congenital disorder with an incidence of 1 in 7000 births.


 Posterior nasal airway occluded by soft tissue or bone.
 Associated with other congenital malformations e.g. coloboma
 Babies with unilateral disease may go unnoticed.
 Babies with bilateral disease will present early in life as they are obligate mouth breathers.
 Treatment is with fenestration procedures designed to restore patency.

Next question
Question 54 of 347

A 28 year old lady presents with a pigmented lesion on her calf. Excisional biopsy confirms a
diagnosis of melanoma measuring 1cm in diameter with a Breslow thickness of 0.1mm. The lesion is
less than 1 mm at all resection margins. Which of the following surgical resection margins is
acceptable for this lesion?

5 cm

1 cm

0.5 cm

2 cm

3 cm

Please rate this question:

Discuss and give feedback


Next question

Malignant melanoma

The main diagnostic features (major criteria): Secondary features (minor criteria)

 Change in size  Diameter >6mm


 Change in shape  Inflammation
 Change in colour  Oozing or bleeding
 Altered sensation

Treatment

 Suspicious lesions should undergo excision biopsy. The lesion should be removed in
completely as incision biopsy can make subsequent histopathological assessment difficult.
 Once the diagnosis is confirmed the pathology report should be reviewed to determine
whether further re-excision of margins is required (see below):

Margins of excision-Related to Breslow thickness


Lesions 0-1mm thick 1cm

Lesions 1-2mm thick 1- 2cm (Depending upon site and pathological features)

Lesions 2-4mm thick 2-3 cm (Depending upon site and pathological features)

Lesions >4 mm thick 3cm

Marsden J et al Revised UK guidelines for management of Melanoma. Br J Dermatol 2010 163:238-


256.

Further treatments such as sentinel lymph node mapping, isolated limb perfusion and block
dissection of regional lymph node groups should be selectively applied.
Next question
Question 55 of 347

A 20 year old man is involved in a road traffic accident. Following the incident he is unable to extend
his wrist. However, this improves over the following weeks. Which type of injury is he most likely to
have sustained?

Radial nerve neurotmesis

Radial nerve neuropraxia

Axillary nerve axonotmesis

Ulnar nerve neuropraxia

Ulnar nerve axonotmesis

Theme from April 2011 Exam

Transient loss of function makes neuropraxia the most likely injury. The wrist extensors are
innervated by the radial nerve making this the most likely site of injury.
Please rate this question:

Discuss and give feedback


Next question

Neuropraxia

 Nerve intact but electrical conduction is affected


 Myelin sheath integrity is preserved
 Full recovery
 Autonomic function preserved
 Wallerian degeneration does not occur

Next question
Question 56 of 347

A 53 year old lady has undergone a bilateral breast augmentation procedure many years previously.
The implants are tense and uncomfortable and are removed. During their removal the surgeon
encounters a dense membrane surrounding the implants, it has a coarse granular appearance. The
tissue is sent for histology and it demonstrates fibrosis with the presence of calcification. The
underlying process responsible for these changes is:

Hyperplasia

Dysplasia

Metastatic calcification

Dystrophic calcification

Necrosis

Breast implants often become surrounded by a pseudocapsule and this may secondarily then be
subjected to a process of dystrophic calcification.

Please rate this question:

Discuss and give feedback

Next question

Pathological calcification

Dystrophic Deposition of calcium deposits in tissues that have undergone degeneration, damage
calcification or disease in the presence of normal serum calcium levels
Metastatic Deposition of calcium deposits in tissues that are otherwise normal in the presence
calcification of increased serum calcium levels

Next question
Question 57 of 347

A 4 year old girl presents with symptoms of right sided loin pain, lethargy and haematuria. On
examination she is pyrexial and has a large mass in the right upper quadrant. The most likely
underlying diagnosis is:

Perinephric abscess

Nephroblastoma

Renal cortical adenoma

Grawitz tumour

Squamous cell carcinoma of the kidney

In a child of this age, with the symptoms described a nephroblastoma is the most likely diagnosis. A
perinephric abscess is most unlikely. If an abscess were to occur it would be confined to Gerotas
fascia in the first instance, and hence anterior extension would be unlikely.
Please rate this question:

Discuss and give feedback


Next question

Nephroblastoma

Nephroblastoma (Wilms tumours)

 Usually present in first 4 years of life


 May often present as a mass associated with haematuria (pyrexia may occur in 50%)
 Often metastasise early (usually to lung)
 Treated by nephrectomy
 Younger children have better prognosis (<1 year of age =80% overall 5 year survival)

Next question
Question 58-60 of 347

Theme: Thyroid neoplasms

A. Follicular carcinoma
B. Anaplastic carcinoma
C. Medullary carcinoma
D. Papillary carcinoma
E. Lymphoma
F. Hashimotos thyroiditis
G. Graves disease

For the following histological descriptions please select the most likely underlying thyroid neoplasm.
Each option may be used once, more than once or not at all.

58. A 22 year old female undergoes a thyroidectomy. The resected specimen shows a non
encapsulated tumour with papillary projections and pale empty nuclei.

You answered Follicular carcinoma

The correct answer is Papillary carcinoma

Theme from April 2012


The presence of papillary structures together with the cytoplasmic features described is
strongly suggestive of papillary carcinoma. They are seldom encapsulated.

59. A thyroidectomy specimen from a 43 year old lady shows a mass with prominent oxyphil
cells and scanty thyroid colloid.

Follicular carcinoma

Hurthle cell tumours are a variant of follicular neoplasms in which oxyphil cells
predominate. They have a poorer prognosis than conventional follicular neoplasms.

60. A 32 year old lady undergoes a thyroidectomy for a mild goitre. The resected specimen
shows an intense lymphocytic infiltrate with acinar destruction and fibrosis.

You answered Follicular carcinoma

The correct answer is Hashimotos thyroiditis

Lymphocytic infiltrates and fibrosis are typically seen in Hashimotos thyroiditis. In


Lymphoma only dense lymphatic type tissue is usually present.
Please rate this question:

Discuss and give feedback


Next question

Thyroid neoplasms

Lesion Common features

Follicular  Usually present as a solitary thyroid nodule


adenoma  Malignancy can only be excluded on formal histological assessment

Papillary  Usually contain a mixture of papillary and colloidal filled follicles


carcinoma  Histologically tumour has papillary projections and pale empty nuclei
 Seldom encapsulated
 Lymph node metastasis predominate
 Haematogenous metastasis rare
 Account for 60% of thyroid cancers

Follicular  May appear macroscopically encapsulated, microscopically capsular


carcinoma invasion is seen. Without this finding the lesion is a follicular adenoma.
 Vascular invasion predominates
 Multifocal disease rare
 Account for 20% of all thyroid cancers

Anaplastic  Most common in elderly females


carcinoma  Local invasion is a common feature
 Account for 10% of thyroid cancers
 Treatment is by resection where possible, palliation may be achieved
through isthmusectomy and radiotherapy. Chemotherapy is ineffective.

Medullary  Tumours of the parafollicular cells (C Cells)


carcinoma  C cells derived from neural crest and not thyroid tissue
 Serum calcitonin levels often raised
 Familial genetic disease accounts for up to 20% cases
 Both lymphatic and haematogenous metastasis are recognised, nodal
disease is associated with a very poor prognosis.

Next question
Question 61 of 347

From which of the following cell types do giant cells most commonly originate?

Neutrophils

Myofibroblasts

Fibroblasts

Macrophages

Goblet cells

Theme from September 2011 and 2009 Exam

Although many cell types may give rise to giant cells, macrophages remain the most common.
Please rate this question:

Discuss and give feedback


Next question

Giant cells

 A giant cell is a mass formed by the union of several distinct types of cells
 They are most commonly comprised of macrophages
 They are different to granulomas although causative agents may overlap

Section stained using haematoxylin and eosin showing giant cell reaction to suture material
Image sourced from Wikipedia

Next question
Question 62 of 347

A 43 year old lady with hypertension is suspected of having a phaeochromocytoma. Which of the
following investigations is most likely to be beneficial in this situation?

Dexamethasone suppression test

Urinary 5-Hydroxyindoleacetic Acid (5-HIAA)

Histamine provocation test

Tyramine provocation test

Urinary vanillymandelic acid measurements

Theme from September 2011 Exam


Theme from September 2012 Exam
Urinary VMA measurements are not completely specific but constitute first line assessment.
Stimulation tests of any sort are not justified in first line assessments.
Please rate this question:

Discuss and give feedback


Next question

Phaeochromocytoma and adrenal lesions

Phaeochromocytoma
Neuroendocrine tumour of the chromaffin cells of the adrenal medulla. Hypertension and
hyperglycaemia are often found.

 10% of cases are bilateral.


 10% occur in children.
 11% are malignant (higher when tumour is located outside the adrenal).
 10% will not be hypertensive.

Familial cases are usually linked to the Multiple endocrine neoplasia syndromes (considered under
its own heading).

Most tumours are unilateral (often right sided) and smaller than 10cm.
Diagnosis
Urine analysis of vanillymandelic acid (VMA) is often used (false positives may occur e.g. in patients
eating vanilla ice cream!)

Blood testing for plasma metanephrine levels.

CT and MRI scanning are both used to localise the lesion.

Treatment
Patients require medical therapy first. An irreversible alpha adrenoreceptor blocker should be given,
although minority may prefer reversible blockade(1). Labetolol may be co-administered for cardiac
chronotropic control. Isolated beta blockade should not be considered as it will lead to unopposed
alpha activity.

These patients are often volume depleted and will often require moderate volumes of intra venous
normal saline perioperatively.

Once medically optimised the phaeochromocytoma should be removed. Most adrenalectomies can
now be performed using a laparoscopic approach(2). The adrenals are highly vascular structures
and removal can be complicated by catastrophic haemorrhage in the hands of the inexperienced.
This is particularly true of right sided resections where the IVC is perilously close. Should the IVC be
damaged a laparotomy will be necessary and the defect enclosed within a Satinsky style vascular
clamp and the defect closed with prolene sutures. Attempting to interfere with the IVC using any
instruments other than vascular clamps will result in vessel trauma and make a bad situation much
worse.

Incidental adrenal lesions


Adrenal lesions may be identified on CT scanning performed for other reasons(3). Factors
suggesting benign disease on CT include(4):

 Size less than 3cm


 Homogeneous texture
 Lipid rich tissue
 Thin wall to lesion

All patients with incidental lesions should be managed jointly with an endocrinologist and full work up
as described above. Patients with functioning lesions or those with adverse radiological features
(Particularly size >3cm) should proceed to surgery.

References
1. Weingarten TN, Cata JP, O'Hara JF, Prybilla DJ, Pike TL, Thompson GB, et al. Comparison of
two preoperative medical management strategies for laparoscopic resection of pheochromocytoma.
Urology. 2010 Aug;76(2):508 e6-11.

2. Nguyen PH, Keller JE, Novitsky YW, Heniford BT, Kercher KW. Laparoscopic approach to
adrenalectomy: review of perioperative outcomes in a single center. Am Surg. 2011 May;77(5):592-
6.

3. Ng VW, Ma RC, So WY, Choi KC, Kong AP, Cockram CS, et al. Evaluation of functional and
malignant adrenal incidentalomas. Arch Intern Med. 2010 Dec 13;170(22):2017-20.

4. Muth A, Hammarstedt L, Hellstrom M, Sigurjonsdottir HA, Almqvist E, Wangberg B. Cohort study


of patients with adrenal lesions discovered incidentally. Br J Surg. 2011 May 27.
Next question
Question 63 of 347

A 46 year old lady presents with symptoms of diarrhoea, weight loss of 10 Kg and a skin rash of
erythematous blisters involving the abdomen and buttocks. The blisters have an irregular border and
both intact and ruptured vesicles. What is the most likely diagnosis?

Colonic adenocarcinoma

Pancreatic adenocarcinoma

Tropical sprue

Glucagonoma

Insulinoma

Theme from September 2011 Exam


Theme from September 2012 Exam
Theme from September 2013 Exam
Glucagonoma is strongly associated with necrolytic migratory erythema.
Please rate this question:

Discuss and give feedback


Next question

Glucagonoma

 Rare pancreatic tumours arising from the alpha cells of the pancreas.
 Glucagon levels markedly elevated.
 Symptoms include diarrhoea, weight loss and necrolytic migratory erythema.
 A serum level of glucagon >1000pg/ml usually suggests the diagnosis, imaging with CT
scanning is also required.
 Treatment is with surgical resection. However, careful staging is required for these tumours
are usually malignant and non resectable.

Next question
Question 64 of 347

A 56 year old man presents with symptoms of neuropathic facial pain and some weakness of the
muscles of facial expression on the right side. On examination he has a hard mass approximately
6cm anterior to the right external auditory meatus. What is the most likely diagnosis?

Pleomorphic adenoma

Adenocarcinoma

Mucoepidermoid carcinoma

Adenoid cystic carcinoma

Lymphoma

Theme from September 2011 Exam

The patient is most likely to have a malignant lesion within the parotid. Of the malignancies listed;
adenoid cystic carcinoma has the greatest tendency to perineural invasion.
Please rate this question:

Discuss and give feedback


Next question

Parotid gland malignancy

 Most parotid neoplasms (80%) are benign lesions


 Most commonly present with painless mass around the mandible
 Up to 30% may present with pain, when this is associated with a discrete mass lesion in the
parotid it usually indicates perineural invasion.
 Perineural invasion is very unlikely to occur in association with benign lesions
 80% of patients with facial nerve weakness caused by parotid malignancies will have nodal
metastasis and a 5 year survival of 25%

Types of malignancy
Mucoepidermoid 30% of all parotid malignancies
carcinoma Usually low potential for local invasiveness and metastasis (depends
mainly on grade)

Adenoid cystic Unpredictable growth pattern


carcinoma Tendency for perineural spread
Nerve growth may display skip lesions resulting in incomplete excision
Distant metastasis more common (visceral rather than nodal spread)
5 year survival 35%

Mixed tumours Often a malignancy occurring in a previously benign parotid lesion

Acinic cell carcinoma Intermediate grade malignancy


May show perineural invasion
Low potential for distant metastasis
5 year survival 80%

Adenocarcinoma Develops from secretory portion of gland


Risk of regional nodal and distant metastasis
5 year survival depends upon stage at presentation, may be up to 75%
with small lesions with no nodal involvement

Lymphoma Large rubbery lesion, may occur in association with Warthins tumours
Diagnosis should be based on regional nodal biopsy rather than parotid
resection Treatment is with chemotherapy (and radiotherapy)

Next question
Question 65 of 347

A 20 year old African lady undergoes an open appendicectomy. She is reviewed for an unrelated
problem 8 months later. On abdominal inspection the wound site is covered by shiny dark
protuberant scar tissue that projects beyond the limits of the skin incision. Which of the following is
the most likely underlying process?

Hypertrophic scar

Keloid scar

Marjolins ulcer

Repeated episodes of wound sepsis

Mycosis fungoides

Keloid scars extend beyond the limits of the incision. Mycosis fungoides is a cutaneous T cell
lymphoma.
Please rate this question:

Discuss and give feedback


Next question

Wound healing

Surgical wounds are either incisional or excisional and either clean, clean contaminated or dirty.
Although the stages of wound healing are broadly similar their contributions will vary according to the
wound type.

The main stages of wound healing include:

Haemostasis

 Minutes to hours following injury


 Vasospasm in adjacent vessels, platelet plug formation and generation of fibrin rich clot.

Inflammation
 Typically days 1-5
 Neutrophils migrate into wound (function impaired in diabetes).
 Growth factors released, including basic fibroblast growth factor and vascular endothelial
growth factor.
 Fibroblasts replicate within the adjacent matrix and migrate into wound.
 Macrophages and fibroblasts couple matrix regeneration and clot substitution.

Regeneration

 Typically days 7 to 56
 Platelet derived growth factor and transformation growth factors stimulate fibroblasts and
epithelial cells.
 Fibroblasts produce a collagen network.
 Angiogenesis occurs and wound resembles granulation tissue.

Remodeling

 From 6 weeks to 1 year


 Longest phase of the healing process and may last up to one year (or longer).
 During this phase fibroblasts become differentiated (myofibroblasts) and these facilitate
wound contraction.
 Collagen fibres are remodeled.
 Microvessels regress leaving a pale scar.

The above description represents an idealised scenario. A number of diseases may distort this
process. Neovascularisation is an important early process. Endothelial cells may proliferate in the
wound bed and recanalise to form a vessel. Vascular disease, shock and sepsis can all compromise
microvascular flow and impair healing.

Conditions such as jaundice will impair fibroblast synthetic function and immunity with a detrimental
effect in most parts of the healing process.

Problems with scars:

Hypertrophic scars
Excessive amounts of collagen within a scar. Nodules may be present histologically containing
randomly arranged fibrils within and parallel fibres on the surface. The tissue itself is confined to the
extent of the wound itself and is usually the result of a full thickness dermal injury. They may go on
to develop contractures.

Image of hypertrophic scarring. Note that it remains confined to the boundaries of the original
wound:
Image sourced from Wikipedia

Keloid scars
Excessive amounts of collagen within a scar. Typically a keloid scar will pass beyond the boundaries
of the original injury. They do not contain nodules and may occur following even trivial injury. They
do not regress over time and may recur following removal.

Image of a keloid scar. Note the extension beyond the boundaries of the original incision:

Image sourced from Wikipedia

Drugs which impair wound healing:


 Non steroidal anti inflammatory drugs
 Steroids
 Immunosupressive agents
 Anti neoplastic drugs

Closure
Delayed primary closure is the anatomically precise closure that is delayed for a few days but before
granulation tissue becomes macroscopically evident.

Secondary closure refers to either spontaneous closure or to surgical closure after granulation tissue
has formed.
Next question
Question 66 of 347

A 28 year old man has a long history of recurrent chest infections. On examination, he is noted to
have no palpable vas deferens. However, both testes are located within the scrotum. What is the
most likely underlying disease association?

Kleinfelters syndrome

Kallmann syndrome

Cystic fibrosis

Coeliac disease

Gardners syndrome

99% of males with cystic fibrosis will have absent vas

Please rate this question:

Discuss and give feedback


Next question

Absence of the vas deferens

 Absence of the vas may be unilateral or bilateral


 Cystic fibrosis CFTR gene mutations are the cause in 40% of cases
 Some non CF cases are due to unilateral renal agenesis
 Sperm harvesting may allow for assisted conception

Next question
Question 67 of 347

The pathogenicity of the tubercle bacillus is due to which of the following?

Necrosis caused by expanding granulomas

Ability to multiply within fibroblasts

Delayed hypersensitivity reaction against bacteria

Effect of antibody response

Direct toxic effect on host cells

Mycobacteria stimulate a specific T cell response of cell mediated immunity. This is effective in
reducing the infection, the delayed hypersensitivity also damages tissues. Necrosis occurs in TB but
is usually within the granuloma.
Please rate this question:

Discuss and give feedback


Next question

Tuberculosis pathology

 Is a form of primary chronic inflammation, caused by the inability of macrophages to kill


theMycobacterium tuberculosis.
 The macrophages often migrate to regional lymph nodes, the lung lesion plus affected lymph
nodes is referred to as a Ghon complex.
 This leads to the formation of a granuloma which is a collection of epithelioid histiocytes.
 There is the presence of caseous necrosis in the centre.
 The inflammatory response is mediated by a type 4 hypersensitivity reaction.
 In healthy individuals the disease may be contained, in the immunocompromised
disseminated (miliary TB) may occur.

Diagnosis

 Waxy membrane of mycobacteria prevents binding with normal stains. Ziehl - Neelsen
staining is typically used.
 Culture based methods take far longer.
Image showing acid- alcohol fast mycobacteria stained using the Ziehl- Neelsen method

Image sourced from Wikipedia

Next question
Question 68 of 347

A 45 year old women with a thyroid carcinoma undergoes a total thyroidectomy. The post operative
histology report shows a final diagnosis of medullary type thyroid cancer. Which of the tests below is
most likely to be of clinical use in screening for disease recurrence?

Serum CA 19-9 Levels

Serum thyroglobulin levels

Serum PTH levels

Serum calcitonin levels

Serum TSH levels

Theme from September 2012 Exam


Medullary thyroid cancers often secrete calcitonin and monitoring the serum levels of this hormone is
useful in detecting sub clinical recurrence.
Please rate this question:

Discuss and give feedback


Next question

Thyroid neoplasms

Lesion Common features

Follicular  Usually present as a solitary thyroid nodule


adenoma  Malignancy can only be excluded on formal histological assessment

Papillary  Usually contain a mixture of papillary and colloidal filled follicles


carcinoma  Histologically tumour has papillary projections and pale empty nuclei
 Seldom encapsulated
 Lymph node metastasis predominate
 Haematogenous metastasis rare
Lesion Common features

 Account for 60% of thyroid cancers

Follicular  May appear macroscopically encapsulated, microscopically capsular


carcinoma invasion is seen. Without this finding the lesion is a follicular adenoma.
 Vascular invasion predominates
 Multifocal disease rare
 Account for 20% of all thyroid cancers

Anaplastic  Most common in elderly females


carcinoma  Local invasion is a common feature
 Account for 10% of thyroid cancers
 Treatment is by resection where possible, palliation may be achieved
through isthmusectomy and radiotherapy. Chemotherapy is ineffective.

Medullary  Tumours of the parafollicular cells (C Cells)


carcinoma  C cells derived from neural crest and not thyroid tissue
 Serum calcitonin levels often raised
 Familial genetic disease accounts for up to 20% cases
 Both lymphatic and haematogenous metastasis are recognised, nodal
disease is associated with a very poor prognosis.

Next question
Question 69 of 347

A 15 year old boy undergoes an emergency splenectomy for trauma. He makes a full recovery and
is discharged home. Eight weeks post operatively the general practitioner performs a full blood count
with a blood film. Which of the following is most likely to be present?

Myofibroblasts

Howell-Jolly bodies

Multinucleate giant cells

Reed Sternberg Cells

None of the above

Post splenectomy blood film features:


Howell- Jolly bodies
Pappenheimer bodies
Target cells
Irregular contracted erythrocytes

As the filtration function is the spleen is no longer present Howell-Jolly bodies are found.

Please rate this question:

Discuss and give feedback

Next question

Post splenectomy blood film changes

The loss of splenic tissue results in the inability to readily remove immature or abnormal red blood
cells from the circulation. The red cell count does not alter significantly. However, cytoplasmic
inclusions may be seen e.g. Howell-Jolly bodies.
In the first few days after splenectomy target cells, siderocytes and reticulocytes will appear in the
circulation. Immediately following splenectomy a granulocytosis (mainly composed of neutrophils) is
seen, this is replaced by a lymphocytosis and monocytosis over the following weeks.
The platelet count is usually increased and this may be persistent, oral antiplatelet agents may be
needed in some patients.

Image showing Howell Jolly bodies (arrowed)

Image sourced from Wikipedia

Next question
Question 70 of 347

A 43 year old women is identified as being a carrier of a BRCA 1 mutation. Apart from breast cancer,
which of the following malignancies is she at greatest risk of developing?

Colonic cancer

Ovarian cancer

Follicular carcinoma of the thyroid

Pituitary adenoma

Phaeochromocytoma

BRCA 1 mutation patients are 55% more likely to get ovarian cancer. Those with BRCA 2 are 25%
more likely. The risk of developing other malignancies is slightly increased but not to the same
extent, and not enough to justify screening.
Please rate this question:

Discuss and give feedback


Next question

Genetics and surgical disease

Some of the more commonly occurring genetic conditions occurring in surgical patients are
presented here.

Li-Fraumeni Syndrome

 Autosomal dominant
 Consists of germline mutations to p53 tumour suppressor gene
 High incidence of malignancies particularly sarcomas and leukaemias
 Diagnosed when:

*Individual develops sarcoma under 45 years


*First degree relative diagnosed with any cancer below age 45 years and another family member
develops malignancy under 45 years or sarcoma at any age
BRCA 1 and 2

 Carried on chromosome 17 (BRCA 1) and Chromosome 13 (BRCA 2)


 Linked to developing breast cancer (60%) risk.
 Associated risk of developing ovarian cancer (55% with BRCA 1 and 25% with BRCA 2).

Lynch Syndrome

 Autosomal dominant
 Develop colonic cancer and endometrial cancer at young age
 80% of affected individuals will get colonic and/ or endometrial cancer
 High risk individuals may be identified using the Amsterdam criteria

Amsterdam criteria
Three or more family members with a confirmed diagnosis of colorectal cancer, one of whom is a
first degree (parent, child, sibling) relative of the other two.
Two successive affected generations.
One or more colon cancers diagnosed under age 50 years.
Familial adenomatous polyposis (FAP) has been excluded.

Gardners syndrome

 Autosomal dominant familial colorectal polyposis


 Multiple colonic polyps
 Extra colonic diseases include: skull osteoma, thyroid cancer and epidermoid cysts
 Desmoid tumours are seen in 15%
 Mutation of APC gene located on chromosome 5
 Due to colonic polyps most patients will undergo colectomy to reduce risk of colorectal
cancer
 Now considered a variant of familial adenomatous polyposis coli

Next question
Question 71 of 347

A 53 year old man is due to undergo a splenectomy as a treatment for refractory haemolytic
anaemia. The underlying pathological basis for haemolytic anaemia is thought to be a Type 2
hypersensitivity response. Which of the following mechanisms best describes this process

Deposition of immune complexes

Cell mediated immune response

IgE mediated response

Formation of autoantibodies against cell surface antigens

None of the above

Mnemonic for the reactions and the mediators involved


ACID EGG-T
Type 1 Anaphylactic
Type 2 Cytotoxic
Type 3 Immune complex
Type 4 Delayed type

EGG T (mediators)

IgE
IgG
IgG
T cells

Type 2 hypersensitivity reactions (which includes haemolytic anaemia) are associated with formation
of antibody against cell surface antigens.

Please rate this question:


Discuss and give feedback

Next question

Hypersensitivity reactions

The Gell and Coombs classification divides hypersensitivity reactions into 4 types

Type I Type II Type III Type IV

Description Anaphylactic Cytotoxic Immune Delayed type


complex

Mediator IgE IgG, IgM IgG, Ig A, IgM T-cells

Antigen Exogenous Cell surface Soluble Tissues

Response Minutes Hours Hours 2-3 days


time

Examples Asthma Autoimmune haemolytic Serum sickness Graft versus host


Hay fever anaemia SLE disease
Pemphigus Aspergillosis Contact dermatitis
Goodpasture's

Next question
Question 72 of 347

Which of the following is not an oncogene?

ras

myc

sis

Ki 67

erb-B

Ki 67 is a nuclear proliferation marker (used in immunohistochemistry). Although, Ki67 positivity is a


marker of malignancy, it is not itself, an oncogene.
Please rate this question:

Discuss and give feedback


Next question

Oncogenes

Oncogenes are cancer promoting genes that are derived from normal genes (proto-oncogenes).
Proto-oncogenes play an important physiological role in cellular growth. They are implicated in the
development of up to 20% of human cancers.

Proto-oncogenes may become oncogenes via the following processes:

 Mutation (point mutation)


 Chromosomal translocation
 Increased protein expression

Only one mutated copy of the gene is needed for cancer to occur - a dominant effect

Classification of oncogenes

 Growth factors e.g. Sis


 Transcription factors e.g. Myc
 Receptor tyrosine kinase e.g. RET
 Cytoplasmic tyrosine kinase e.g. Src
 Regulatory GTPases e.g. Ras

Tumour suppressor genes


Tumour suppressor genes restrict or repress cellular proliferation in normal cells. Their inactivation
through mutation or germ line incorporation is implicated in renal, colonic, breast, bladder and many
other cancers. One of the best known tumour suppressor genes is p53. p53 gene offers protection
by causing apoptosis of damaged cells. Other well known genes include BRCA 1 and 2.
Next question
Question 73 of 347

A 25 year old man is injured in a road traffic accident. His right tibia is fractured and is managed by
fasciotomies and application of an external fixator. Over the next 48 hours his serum creatinine rises
and urine is sent for microscopy, muddy brown casts are identified. What is the most likely
underlying diagnosis?

Acute interstitial nephritis

Acute tubular necrosis

Glomerulonephritis

IgA Nephropathy

Thin basement membrane disease

This patient is likely to have had compartment syndrome (tibial fracture + fasciotomies) which may
produce myoglobinuria. The presence of worsening renal function, together with muddy brown casts
is strongly suggestive of acute tubular necrosis. Acute interstitial nephritis usually arises from drug
toxicity and does not usually produce urinary muddy brown casts. Thin basement membrane
disease is an autosomal dominant condition that causes persistent microscopic haematuria, but not
worsening renal function.
Please rate this question:

Discuss and give feedback


Next question

Acute Renal Failure

 Final pathway is tubular cell death.


 Renal medulla is a relatively hypoxic environment making it susceptible to renal tubular
hypoxia.
 Renovascular autoregulation maintains renal blood flow across a range of arterial pressures.
 Estimates of GFR are best indices of level of renal function. Useful clinical estimates can be
obtained by considering serum creatinine, age, race, gender and body size. eGFR
calculations such as the Cockcroft and Gault equation are less reliable in populations with
high GFR's.
 Nephrotoxic stimuli such as aminoglycosides and radiological contrast media induce
apoptosis. Myoglobinuria and haemolysis result in necrosis. Overlap exists and
proinflammatory cytokines play and important role in potentiating ongoing damage.
 Post-operative renal failure is more likely to occur in patients who are elderly, have
peripheral vascular disease, high BMI, have COPD, receive vasopressors, are on
nephrotoxic medication or undergo emergency surgery.
 Avoiding hypotension will reduce risk of renal tubular damage.
 There is no evidence that administration of ACE inhibitors or dopamine reduces the
incidence of post-operative renal failure.

Next question
Question 74 of 347

A 56 year old man has undergone a radical nephrectomy. The pathologist bisects the kidney and
identifies a pink fleshy tumour in the renal pelvis. What is the most likely disease?

Renal cell carcinoma

Transitional cell carcinoma

Angiomyolipoma

Phaeochromocytoma

Renal adenoma

Most renal tumours are yellow or brown in colour. TCC's are one of the few tumours to appear pink.

Theme from April 2012


The finding of a TCC in the renal pelvis mandates a nephroureterectomy.
Please rate this question:

Discuss and give feedback


Next question

Renal lesions

Lesion Disease specific features Treatment

Renal cell  Most present with haematuria Usually radical or partial


carcinoma (50%) nephrectomy
 Common renal tumour (85%
cases)
 Paraneoplastic features include
hypertension and polycythaemia
 Most commonly has
haematogenous mestastasis
Lesion Disease specific features Treatment

Nephroblastoma  Rare childhood tumour Surgical resection combined


 It accounts for 80% of all with chemotherapy (usually
genitourinary malignancies in vincristine, actinomycin D and
those under the age of 15 years doxorubicin)
 Up to 90% will have a mass
 50% will be hypertensive
 Diagnostic work up includes
ultrasound and CT scanning

Neuroblastoma  Most common extracranial Surgical resection, radiotherapy


tumour of childhood and chemotherapy
 80% occur in those under 4 years
of age
 Tumour of neural crest origin (up
to 50% occur in the adrenal
gland)
 The tumour is usually calcified
and may be diagnosed using
MIBG scanning
 Staging is with CT

Transitional cell  Accounts for 90% of Radical nephroureterectomy


carcinoma lower urinary tract tumours, but
only 10% of renal tumours
 Males affected 3x more than
females
 Occupational exposure to
industrial dyes and rubber
chemicals may increase risk
 Up to 80% present with painless
haematuria
 Diagnosis and staging is with CT
IVU

Angiomyolipoma  80% of these hamartoma type 50% of patients with lesions


lesions occur sporadically, the >4cm will have symptoms and
remainder are seen in those with will require surgical resection
tuberous sclerosis
 Tumour is composed of blood
vessels, smooth muscle and fat
Lesion Disease specific features Treatment

 Massive bleeding may occur in


10% of cases

Next question
Question 75 of 347

A 65 year old lady presents with a lesion affecting her right breast. On examination she has a
weeping, crusting lesion overlying the right nipple, the areolar region is not involved. There is no
palpable mass lesion in the breast, there is a palpable axillary lymph node. The patient's general
practitioner has tried treating the lesion with 1% hydrocortisone cream, with no success. What is the
most likely diagnosis?

Infection with Staphylococcus aureus

Pagets disease of the nipple

Phylloides tumour

Nipple eczema

Basal cell carcinoma

A weeping, crusty lesion such as this is most likely to represent Pagets disease of the nipple
(especially since the areolar region is spared). Although no mass lesion is palpable, a proportion of
patients will still have an underlying invasive malignancy (hence the lymphadenopathy).

Please rate this question:

Discuss and give feedback

Next question

Pagets disease of the nipple

Pagets disease is an eczematoid change of the nipple associated with an underlying breast
malignancy and it is present in 1-2% of patients with breast cancer. In half of these patients, it is
associated with an underlying mass lesion and 90% of such patients will have an invasive
carcinoma. 30% of patients without a mass lesion will still be found to have an underlying carcinoma.
The remainder will have carcinoma in situ.
Pagets disease differs from eczema of the nipple in that it involves the nipple primarily and only
latterly spreads to the areolar (the opposite occurs in eczema).
Diagnosis is made by punch biopsy, mammography and ultrasound of the breast.
Treatment will depend on the underlying lesion.

Next question
Question 76 of 347

A 73 year old man presents with haemoptysis and is suspected of suffering from lung cancer. On
examination he has an enlarged supraclavicular lymph node. Which of the following features is most
likely to be present on histological examination?

Increased mitoses

Apoptosis

Barr Bodies

Multinucleate giant cells

Granuloma

Theme from 2011 Exam

Increased mitoses are commonly seen in association with malignant transformation of cells.
Apoptosis is not a common feature of metastatic cancer. Barr Bodies are formed during X
chromosome inactivation in female somatic cells.
Please rate this question:

Discuss and give feedback


Next question

Histopathology of malignancy

 Abnormal tissue architecture


 Coarse chromatin
 Invasion of basement membrane*
 Abnormal mitoses
 Angiogenesis
 De-differentiation
 Areas of necrosis
 Nuclear pleomorphism

*= Those features that distinguish invasive malignancy from in situ disease


Next question
Question 77 of 347

Which of the following pathological explanations best describes the initial pathological processes
occurring in an abdominal aortic aneurysm in an otherwise well 65 year old, hypertensive male?

Loss of elastic fibres from the adventitia

Loss of collagen from the adventitia

Loss of collagen from the media

Loss of elastic fibres from the media

Decreased matrix metalloproteinases in the adventitia

Theme from April 2012 Exam


Theme from April 2013 Exam
In established aneurysmal disease there is dilation of all layers of the arterial wall and loss of both
elastin and collagen. The primary event is loss of elastic fibres with subsequent degradation of
collagen fibres.

Please rate this question:

Discuss and give feedback

Next question

Pathology of abdominal aortic aneurysm

Abdominal aortic aneurysms occur primarily as a result of the failure of elastic proteins within the
extracellular matrix. Aneurysms typically represent dilation of all layers of the arterial wall. Most
aneurysms are caused by degenerative disease. After the age of 50 years the normal diameter of
the infrarenal aorta is 1.5cm in females and 1.7cm in males. Diameters of 3cm and greater, are
considered aneurysmal. The pathophysiology involved in the development of aneurysms is complex
and the primary event is loss of the intima with loss of elastic fibres from the media. This process is
associated with, and potentiated by, increased proteolytic activity and lymphocytic infiltration.

Major risk factors for the development of aneurysms include smoking and hypertension. Rare but
important causes include syphilis and connective tissues diseases such as Ehlers Danlos type 1 and
Marfans syndrome.

Layers of arterial wall

Image sourced from Wikipedia

Next question
Question 78 of 347

A 28 year old lady has a malignant melanoma removed from her calf. Which of the following
pathological criteria carries the greatest prognostic weighting?

Vascular invasion

Abnormal mitoses

Breslow thickness

Perineural invasion

Lymphocytic infiltrates

Theme from April 2012 exam


Theme from September 2013 exam
The Breslow thickness has considerable prognostic importance. Lymphocytic infiltrates may be
associated with an improved prognosis, but do not carry nearly the same weight as increased
thickness.
Please rate this question:

Discuss and give feedback


Next question

Malignant melanoma

The main diagnostic features (major criteria): Secondary features (minor criteria)

 Change in size  Diameter >6mm


 Change in shape  Inflammation
 Change in colour  Oozing or bleeding
 Altered sensation

Treatment
 Suspicious lesions should undergo excision biopsy. The lesion should be removed in
completely as incision biopsy can make subsequent histopathological assessment difficult.
 Once the diagnosis is confirmed the pathology report should be reviewed to determine
whether further re-excision of margins is required (see below):

Margins of excision-Related to Breslow thickness


Lesions 0-1mm thick 1cm

Lesions 1-2mm thick 1- 2cm (Depending upon site and pathological features)

Lesions 2-4mm thick 2-3 cm (Depending upon site and pathological features)

Lesions >4 mm thick 3cm

Marsden J et al Revised UK guidelines for management of Melanoma. Br J Dermatol 2010 163:238-


256.

Further treatments such as sentinel lymph node mapping, isolated limb perfusion and block
dissection of regional lymph node groups should be selectively applied.
Next question
Question 79 of 347

A 34 year old lady undergoes an elective cholecystectomy for attacks of recurrent cholecystitis due
to gallstones. Microscopic assessment of the gallbladder is most likely to show which of the
following?

Dysplasia of the fundus

Widespread necrosis

Ashoff-Rokitansky sinuses

Metaplasia of the fundus

None of the above

Aschoff-Rokitansky sinuses are the result of hyperplasia and herniation of epithelial cells through the
fibromuscular layer of the gallbladder wall. They may be macroscopic or microscopic. Ashoff-
Rokitansky sinuses may be identified in cases of chronic cholecystitis and gallstones. Although
gallstones may predispose to the development of gallbladder cancer the actual incidence of
dysplasia and metaplastic change is rare. In the elective setting described above necrosis would be
rare.
Please rate this question:

Discuss and give feedback


Next question

Gallbladder

 Fibromuscular sac with capacity of 50ml


 Columnar epithelium

Relations of the gallbladder


Anterior Liver

Posterior  Covered by peritoneum


 Transverse colon
 1st part of the duodenum

Laterally Right lobe of liver

Medially Quadrate lobe of liver

Arterial supply
Cystic artery (branch of Right hepatic artery)

Venous drainage
Directly to the liver

Nerve supply
Sympathetic- mid thoracic spinal cord, Parasympathetic- anterior vagal trunk

Common bile duct

Origin Confluence of cystic and common hepatic ducts

Relations at  Medially - Hepatic artery


origin  Posteriorly- Portal vein

Relations distally  Duodenum - anteriorly


 Pancreas - medially and laterally
 Right renal vein - posteriorly

Arterial supply Branches of hepatic artery and retroduodenal branches of gastroduodenal


artery

Hepatobiliary triangle

Medially Common hepatic duct

Inferiorly Cystic duct

Superiorly Inferior edge of liver


Contents Cystic artery

Relations of the gallbladder

© Image provided by the University of Sheffield

Next question
Question 80 of 347

Which of the following are not true of Li-Fraumeni syndrome?

It consists of mutations to the p53 tumour suppressor gene

Is likely to be present in a teenager presenting with a liposarcoma

It has an autosomal dominant inheritance pattern

Affected individuals are unlikely to develop acute myeloid leukaemia

Adrenal malignancies are more common than in normal population

They are at high risk of developing leukaemia.


Please rate this question:

Discuss and give feedback


Next question

Genetics and surgical disease

Some of the more commonly occurring genetic conditions occurring in surgical patients are
presented here.

Li-Fraumeni Syndrome

 Autosomal dominant
 Consists of germline mutations to p53 tumour suppressor gene
 High incidence of malignancies particularly sarcomas and leukaemias
 Diagnosed when:

*Individual develops sarcoma under 45 years


*First degree relative diagnosed with any cancer below age 45 years and another family member
develops malignancy under 45 years or sarcoma at any age

BRCA 1 and 2
 Carried on chromosome 17 (BRCA 1) and Chromosome 13 (BRCA 2)
 Linked to developing breast cancer (60%) risk.
 Associated risk of developing ovarian cancer (55% with BRCA 1 and 25% with BRCA 2).

Lynch Syndrome

 Autosomal dominant
 Develop colonic cancer and endometrial cancer at young age
 80% of affected individuals will get colonic and/ or endometrial cancer
 High risk individuals may be identified using the Amsterdam criteria

Amsterdam criteria
Three or more family members with a confirmed diagnosis of colorectal cancer, one of whom is a
first degree (parent, child, sibling) relative of the other two.
Two successive affected generations.
One or more colon cancers diagnosed under age 50 years.
Familial adenomatous polyposis (FAP) has been excluded.

Gardners syndrome

 Autosomal dominant familial colorectal polyposis


 Multiple colonic polyps
 Extra colonic diseases include: skull osteoma, thyroid cancer and epidermoid cysts
 Desmoid tumours are seen in 15%
 Mutation of APC gene located on chromosome 5
 Due to colonic polyps most patients will undergo colectomy to reduce risk of colorectal
cancer
 Now considered a variant of familial adenomatous polyposis coli

Next question
Question 81 of 347

A 35 year old type 1 diabetic presents with difficulty mobilising and back pain radiating to the thigh.
He has a temperature of 39 oC and has pain on extension of the hip. He is diagnosed with an
iliopsoas abscess. Which of the following statements is false in relation to his diagnosis?

Staphylococcus aureus is the most likely primary cause

Recurrence occurs in 60% cases

More common in males

Crohn's is the most likely secondary cause

CT guided drainage is preferable first line management

Classical features include: a limp, back pain and fever. Recurrence rates are about 15-
20%.Staphylococcus is the commonest primary cause, others include Streptococcus and E.coli.
Management is ideally by CT guided drainage.
Please rate this question:

Discuss and give feedback


Next question

Iliopsoas abscess

 Collection of pus in iliopsoas compartment (iliopsoas and iliacus)


 Causes:

Primary

 Haematogenous spread of bacteria


 Staphylococcus aureus: most common

Secondary
 Crohn's (commonest cause in this category)
 Diverticulitis, Colorectal cancer
 UTI, GU cancers
 Vertebral osteomyelitis
 Femoral catheter, lithotripsy
 Endocarditis

Note the mortality rate can be up to 19-20% in secondary iliopsoas abscesses compared with 2.4%
in primary abscesses.

Clinical features

 Fever
 Back/flank pain
 Limp
 Weight loss

Clinical examination

 Patient in the supine position with the knee flexed and the hip mildly externally rotated

 Specific tests to diagnose iliopsoas inflammation:

Place hand proximal to the patient's ipsilateral knee and ask patient to lift thigh against your hand.
This will cause pain due to contraction of the psoas muscle.

Lie the patient on the normal side and hyperextend the affected hip. In inflammation this should elicit
pain as the psoas muscle is stretched.

Investigation

 CT is gold standard

Management

 Antibiotics
 Percutaneous drainage
 Surgery is indicated if:

1. Failure of percutaneous drainage


2. Presence of an another intra-abdominal pathology which requires surgery

Surgical approach
The authors technique for draining these collections is given here.
Review the CT scans and plan surgical approach. An extraperitoneal approach is important.
The collection usually extends inferiorly and can be accessed from an incision at a level of L4 on the
affected side.

GA
Transverse laterally placed incision.
Incise external oblique.
Split the subsequent muscle layers.
As you approach the peritoneum use blunt dissection to pass laterally around it.
Remember the ureter and gonadal veins lie posterior at this level.
Eventually you will enter the abscess cavity, a large amount of pus is usually released at this point.
Drain the area with suction and washout with saline.
Place a corrugated drain well into the abscess cavity.
If you have made a small skin incision it is reasonable to bring the drain up through the skin wound.
Otherwise place a lateral exit site and close the skin and external oblique. If you do this ensure that
you use interrupted sutures.

Anchor the drain with strong securely tied silk sutures (it is extremely tiresome if it falls out!)

Reference

Iliopsoas abscesses
I H Mallick, M H Thoufeeq, T P Rajendran
Postgrad Med J 2004;80:459-462
Next question
Question 82 of 347

Which of the following statements relating to parathyroid neoplasms is incorrect?

15% of cases are due to parathyroid carcinoma

80% of cases are due to parathyroid adenomas

Parathyroid adenomas are often encapsulated

10% of parathyroid adenomas develop in ectopically located glands

85% of cases of primary hyperparathyroidism are due to solitary adenomas

Parathyroid carcinomas account for up to 5% of tumours. Adenomas are often encapsulated.


Lesions that are fibrotic and densely adherent to the gland may be a carcinoma. 85% cases of
primary hyperparathyroidism are due to a single adenoma and this is the reason some surgeons
favour a focused parathyroidectomy.
Please rate this question:

Discuss and give feedback


Next question

Parathyroid glands and disorders of calcium metabolism

Hyperparathyroidism
Disease type Hormone profile Clinical features Cause

Primary  PTH (Elevated)  May be Most cases due to


hyperparathyroidism  Ca2+(Elevated) asymptomatic if solitary adenoma
 Phosphate (Low) mild (80%), multifocal
 Urine calcium :  Recurrent disease occurs in 10-
creatinine clearance abdominal pain 15% and parathyroid
ratio > 0.01 (pancreatitis, renal carcinoma in 1% or
colic) less
 Changes to
emotional or
Disease type Hormone profile Clinical features Cause

cognitive state

Secondary  PTH (Elevated)  May have few Parathyroid gland


hyperparathyroidism  Ca2+ (Low or symptoms hyperplasia occurs as
normal)  Eventually may a result of low
 Phosphate develop bone calcium, almost
(Elevated) disease, osteitis always in a setting of
 Vitamin D levels fibrosa cystica and chronic renal failure
(Low) soft tissue
calcifications

Tertiary  Ca2+(Normal or  Metastatic Occurs as a result of


hyperparathyroidism high) calcification ongoing hyperplasia
 PTH (Elevated)  Bone pain and / of the parathyroid
 Phosphate levels or fracture glands after
(Decreased or  Nephrolithiasis correction of
Normal)  Pancreatitis underlying renal
 Vitamin D (Normal disorder, hyperplasia
or decreased) of all 4 glands is
 Alkaline usually the cause
phosphatase
(Elevated)

Differential diagnoses
It is important to consider the rare but relatively benign condition of benign familial hypocalciuric
hypercalcaemia, caused by an autosomal dominant genetic disorder. Diagnosis is usually made by
genetic testing and concordant biochemistry (urine calcium : creatinine clearance ratio <0.01-
distinguished from primary hyperparathyroidism).

Treatment

Primary hyperparathyroidism
Indications for surgery

 Elevated serum Calcium > 1mg/dL above normal


 Hypercalciuria > 400mg/day
 Creatinine clearance < 30% compared with normal
 Episode of life threatening hypercalcaemia
 Nephrolithiasis
 Age < 50 years
 Neuromuscular symptoms
 Reduction in bone mineral density of the femoral neck, lumbar spine, or distal radius of more
than 2.5 standard deviations below peak bone mass (T score lower than -2.5)
Secondary hyperparathyroidism
Usually managed with medical therapy.

Indications for surgery in secondary (renal) hyperparathyroidism:

 Bone pain
 Persistent pruritus
 Soft tissue calcifications

Tertiary hyperparathyroidism
Allow 12 months to elapse following transplant as many cases will resolve
The presence of an autonomously functioning parathyroid gland may require surgery. If the culprit
gland can be identified then it should be excised. Otherwise total parathyroidectomy and re-
implantation of part of the gland may be required.

References
1. Pitt S et al. Secondary and Tertiary Hyperparathyroidism, State of the Art Surgical
Management.Surg Clin North Am 2009 Oct;89(5):1227-39.

2. MacKenzie-Feder J et al. Primary Hyperparathyroidism: An Overview. Int J Endocrinol 2011;


2011: 251410.
Next question
Question 83 of 347

A 20 year old girl presents with a thyroid cancer, she is otherwise well with no significant family
history. On examination she has a nodule in the left lobe of the thyroid with a small discrete mass
separate from the gland itself. Which of the following is the most likely cause?

Follicular carcinoma

Anaplastic carcinoma

Medullary carcinoma

Papillary carcinoma

B Cell Lymphoma

Theme from September 2011 Exam

Papillary carcinoma is the most common subtype and may cause lymph node metastasis (mass
separate from the gland itself) that is rare with follicular tumours. Anaplastic carcinoma would cause
more local symptoms and would be rare in this age group.
Please rate this question:

Discuss and give feedback


Next question

Thyroid malignancy

Papillary carcinoma

 Commonest sub-type
 Accurately diagnosed on fine needle aspiration cytology
 Histologically, they may demonstrate psammoma bodies (areas of calcification) and so
called 'orphan Annie' nuclei
 They typically metastasise via the lymphatics and thus laterally located apparently ectopic
thyroid tissue is usually a metastasis from a well differentiated papillary carcinoma

Follicular carcinoma
 Are less common than papillary lesions
 Like papillary tumours, they may present as a discrete nodule. Although they appear to be
well encapsulated macroscopically there is invasion on microscopic evaluation
 Lymph node metastases are uncommon and these tumours tend to spread
haematogenously. This translates into a higher mortality rate
 Follicular lesions cannot be accurately diagnosed on fine needle aspiration cytology and thus
all follicular FNA's (THY 3f) will require at least a hemi thyroidectomy

Anaplastic carcinoma

 Less common and tend to occur in elderly females


 Disease is usually advanced at presentation and often only palliative decompression and
radiotherapy can be offered.

Medullary carcinoma

 These are tumours of the parafollicular cells ( C Cells) and are of neural crest origin.
 The serum calcitonin may be elevated which is of use when monitoring for recurrence.
 They may be familial and occur as part of the MEN -2A disease spectrum.
 Spread may be either lymphatic or haematogenous and as these tumours are not derived
primarily from thyroid cells they are not responsive to radioiodine.

Lymphoma

 These respond well to radiotherapy


 Radical surgery is unnecessary once the disease has been diagnosed on biopsy material.
Such biopsy material is not generated by an FNA and thus a core biopsy has to be obtained
(with care!).

Next question
Question 84 of 347

A 28 year old lady is breast feeding her first child. She presents with discomfort of the right breast.
Clinical examination demonstrates erythema and an area that is fluctuant. Aspiration and culture of
the fluid is most likely to demonstrate infection with which of the following organisms?

Clostridium perfringens

Staphylococcus aureus

Streptococcus pyogenes

Staphylococcus epidermidis

Actinomycosis

Theme from April 2014 Exam

Staphylococcus aureus is the commonest cause. The infants mouth is usually the source as it
damages the nipple areolar complex allowing entry of bacteria.
Please rate this question:

Discuss and give feedback


Next question

Breast abscess

 In lactational women Staphylococcus aureus is the most common cause


 Typical presentation is with a tender, fluctuant mass in a lactating women
 Diagnosis and treatment is performed using USS and associated drainage of the abscess
cavity. Antibiotics should also be administered
 Where there is necrotic skin overlying the abscess, the patient should undergo surgery

Next question
Question 85 of 347

A 20 year old male presents with a tense, swollen knee joint. There is no history of antecedent
trauma. On examination the joint is tense and swollen but there is no sign of injury. Plain x-rays
show no fracture or arthritis. What is the most likely explanation?

Rupture of the anterior cruciate ligament

Rupture of the medial collateral ligament

Tibial plateau fracture

Haemophilia A

von Willebrands disease

Haemarthrosis without trauma is typically a feature of haemophilia A and B

Theme from 2014 Exam


Without a history of trauma, ligamentous rupture or tibial plateau fractures would be unusual.

Please rate this question:

Discuss and give feedback

Next question

Abnormal coagulation

Cause Factors affected


Cause Factors affected

Heparin Prevents activation factors 2,9,10,11

Warfarin Affects synthesis of factors 2,7,9,10

DIC Factors 1,2,5,8,11

Liver disease Factors 1,2,5,7,9,10,11

Interpretation blood clotting test results

Disorder APTT PT Bleeding time

Haemophilia Increased Normal Normal

von Willebrand's disease Increased Normal Increased

Vitamin K deficiency Increased Increased Normal

Next question
Question 86 of 347

An 18 year old rock climber falls onto his left arm and sustains a large haematoma of the left upper
arm. Unfortunately the wound associated with the injury is neglected and it becomes infected. Which
of these changes is least likely to occur?

Axillary lymphadenopathy

Leucopenia

Tenderness

Mild pyrexia

Local formation of yellow pus

Leucopenia would be unusual and should prompt a search for another cause.
Please rate this question:

Discuss and give feedback


Next question

Acute inflammation

Inflammation is the reaction of the tissue elements to injury. Vascular changes occur, resulting in the
generation of a protein rich exudate. So long as the injury does not totally destroy the existing tissue
architecture, the episode may resolve with restoration of original tissue architecture.

Vascular changes

 Vasodilation occurs and persists throughout the inflammatory phase.


 Inflammatory cells exit the circulation at the site of injury.
 The equilibrium that balances Starlings forces within capillary beds is disrupted and a protein
rich exudate will form as the vessel walls also become more permeable to proteins.
 The high fibrinogen content of the fluid may form a fibrin clot. This has several important
immunomodulatory functions.

Sequelae
Resolution  Typically occurs with minimal initial injury
 Stimulus removed and normal tissue architecture results

Organisation  Delayed removal of exudate


 Tissues undergo organisation and usually fibrosis

Suppuration  Typically formation of an abscess or an empyema


 Sequestration of large quantities of dead neutrophils

Progression to chronic  Coupled inflammatory and reparative activities


inflammation  Usually occurs when initial infection or suppuration has
been inadequately managed

Causes

 Infections e.g. Viruses, exotoxins or endotoxins released by bacteria


 Chemical agents
 Physical agents e.g. Trauma
 Hypersensitivity reactions
 Tissue necrosis

Presence of neutrophil polymorphs is a histological diagnostic feature of acute inflammation


Next question
Question 87-89 of 347

Theme: Tumour markers

A. Invasive ductal carcinoma of the breast


B. Prostate cancer
C. Gastric cancer
D. Ovarian cancer
E. Colorectal cancer
F. Pancreatic adenocarcinoma
G. Seminoma testicular cancer
H. Non-seminomatous testicular cancer
I. Hepatocellular carcinoma

For each tumour marker please select the most likely underlying malignancy. Each option may be
used once, more than once or not at all.

87. Raised beta-human chorionic gonadotropin with a raised alpha-feto protein level

You answered Invasive ductal carcinoma of the breast

The correct answer is Non-seminomatous testicular cancer

Theme from April 2012 Exam


A raised alpha-feto protein level excludes a seminoma

88. Elevated CA 19-9

You answered Invasive ductal carcinoma of the breast

The correct answer is Pancreatic adenocarcinoma

89. Raised alpha-feto protein level in a 54-year-old woman

You answered Invasive ductal carcinoma of the breast

The correct answer is Hepatocellular carcinoma

Please rate this question:


Discuss and give feedback
Next question

Tumour markers

Theme from January 2013 exam


Tumour markers may be divided into:

 monoclonal antibodies against carbohydrate or glycoprotein tumour antigens


 tumour antigens
 enzymes (alkaline phosphatase, neurone specific enolase)
 hormones (e.g. calcitonin, ADH)

It should be noted that tumour markers usually have a low specificity

Monoclonal antibodies
Tumour marker Association

CA 125 Ovarian cancer

CA 19-9 Pancreatic cancer

CA 15-3 Breast cancer

NB: The breast cancer tumour marker is not specific or sensitive enough to be used routinely.

Tumour antigens
Tumour marker Association

Prostate specific antigen (PSA) Prostatic carcinoma

Alpha-feto protein (AFP) Hepatocellular carcinoma, teratoma

Carcinoembryonic antigen (CEA) Colorectal cancer

Next question
Question 90-92 of 347

Theme: Benign breast lesions

A. Fibroadenoma
B. Breast abscess
C. Cyst of Montgomery's gland
D. Galactocele
E. Lipoma
F. Duct ectasia
G. Intraductal papilloma
H. Fat necrosis

What is the most likely diagnosis for the scenario given? Each option may be used once, more than
once or not at all.

90. A 64 year old obese female presents with a breast lump. She was hit on the breast by a
cricket ball when playing with her grandson.

You answered Fibroadenoma

The correct answer is Fat necrosis

An obese, post menopausal woman, with a history of trauma points towards fat necrosis.
Trauma causes inflammation of fat cells, leading to formation of a lump. Mammography,
USS and biopsy are usually needed to differentiate it from breast malignancy.

91. A 21 year old female notices a bloody discharge from the nipple. She is otherwise well. On
examination there are no discrete lesions to feel and mammography shows dense breast
tissue but no mass lesion.

You answered Fibroadenoma

The correct answer is Intraductal papilloma

Intraductal papillomata are the commonest cause of blood stained nipple discharge in
younger women. There is seldom any palpable mass. An ultrasound is required and
possibly a galactogram.

92. A 18 year old female notices a non tender mobile breast lump. Clinically there is a smooth
lump which is not tethered to the skin.

Fibroadenoma
Also called a breast 'mouse' due to its mobility. It is a benign condition arising from the
breast lobule. May enlarge in pregnancy.

Please rate this question:

Discuss and give feedback


Next question

Benign breast lesions

Lesion Features Treatment

Fibroadenoma  Develop from a whole lobule If >3cm surgical excision is usual,


 Mobile, firm breast lumps Phyllodes tumours should be
 12% of all breast masses widely excised (mastectomy if the
 Over a 2 year period up to lesion is large)
30% will get smaller
 No increase in risk of
malignancy

Breast cyst  7% of all Western females Cysts should be aspirated, those


will present with a breast which are blood stained or
cyst persistently refill should be
 Usually presents as a smooth biopsied or excised
discrete lump (may be
fluctuant)
 Small increased risk of breast
cancer (especially if
younger)

Sclerosing adenosis,  Usually presents as a breast Lesions should be biopsied,


(radial scars and lump or breast pain excision is not mandatory
complex sclerosing  Causes mammographic
lesions) changes which may mimic
carcinoma
 Cause distortion of the distal
lobular unit, without
hyperplasia (complex lesions
will show hyperplasia)
 Considered a disorder of
Lesion Features Treatment

involution, no increase in
malignancy risk

Epithelial hyperplasia  Variable clinical presentation If no atypical features then


ranging from generalised conservative, those with atypical
lumpiness through to discrete features require either close
lump monitoring or surgical resection
 Disorder consists of
increased cellularity of
terminal lobular unit,
atypical features may be
present
 Atypical features and family
history of breast cancer
confers greatly increased risk
of malignancy

Fat necrosis  Up to 40% cases usually Imaging and core biopsy


have a traumatic aetiology
 Physical features usually
mimic carcinoma
 Mass may increase in size
initially

Duct papilloma  Usually present with nipple Microdochectomy


discharge
 Large papillomas may
present with a mass
 The discharge usually
originates from a single duct
 No increase risk of
malignancy

Next question
Question 93 of 347

A 17 year old man is identified as having a Meckels diverticulum. From which of the following
embryological structures is it derived?

Foregut

Hindgut

Urachus

Cloaca

Vitello-intestinal duct

Rule of 2's

2% of population
2 inches (5cm) long
2 feet (60 cm) from the ileocaecal valve
2 x's more common in men
2 tissue types involved

The Meckels diverticulum is a persistence of the vitello-intestinal duct.


Please rate this question:

Discuss and give feedback


Next question

Meckel's diverticulum

 Congenital abnormality resulting in incomplete obliteration of the vitello-intestinal duct


 Normally, in the foetus, there is an attachment between the vitello-intestinal duct and the yolk
sac.This disappears at 6 weeks gestation.
 The tip is free in majority of cases.
 Associated with enterocystomas, umbilical sinuses, and omphaloileal fistulas.
 Arterial supply: omphalomesenteric artery.
 2% of population, 2 inches long, 2 feet from the ileocaecal valve.
 Typically lined by ileal mucosa but ectopic gastric mucosa can occur, with the risk of peptic
ulceration. Pancreatic and jejunal mucosa can also occur.
Clinical

 Normally asymptomatic and an incidental finding.


 Complications are the result of obstruction, ectopic tissue, or inflammation.
 Removal if narrow neck or symptomatic. Options are between wedge excision or formal
small bowel resection and anastomosis.

Next question
Question 94-96 of 347

Theme: Rectal bleeding

A. Solitary rectal ulcer syndrome


B. Haemorrhoidal disease
C. Fissure in ano
D. Fistula in ano
E. Anal cancer
F. Ulcerative colitis

Please select the most likely diagnosis for the scenario given. Each option may be used once, more
than once or not at all.

94. A 22 year old man presents with a 6 day history of passage of bloody diarrhoea with
passage of mucous and slime. He is passing an average of 8 to 9 bowel movements per
day. On digital rectal examination there is no discrete abnormality to feel, but there is
some blood stained mucous on the glove.

You answered Solitary rectal ulcer syndrome

The correct answer is Ulcerative colitis

The passage of bloody diarrhoea together with mucous and a short history makes this a
likely first presentation of inflammatory bowel disease.

95. A 17 year old man presents with a 2 week history of significant pain on defecation
accompanied by the presence of a small amount of blood which is noticed on toilet paper.

You answered Solitary rectal ulcer syndrome

The correct answer is Fissure in ano

Young patients with painful rectal bleeding may have a fissure. Treatment is with stool
softeners and either GTN or Diltiazem initially.

96. A 24 year old woman presents with a long history of obstructed defecation and chronic
constipation. She often strains to open her bowels for long periods and occasionally
notices that she has passed a small amount of blood. On examination she has an indurated
area located anteriorly approximately 3cm proximal to the anal verge.

Solitary rectal ulcer syndrome


Solitary rectal ulcers are associated with chronic constipation and straining. It will need to
be biopsied to exclude malignancy (the histological appearances are characteristic).
Diagnostic work up should include endoscopy and probably defecating proctogram and
ano-rectal manometry studies.

Please rate this question:

Discuss and give feedback


Next question

Rectal bleeding

Rectal bleeding is a common cause for patients to be referred to the surgical clinic. In the clinical
history it is useful to try and localise the anatomical source of the blood. Bright red blood is usually of
rectal anal canal origin, whilst dark red blood is more suggestive of a proximally sited bleeding
source. Blood which has entered the GI tract from a gastro-duodenal source will typically resemble
malaena due to the effects of the digestive enzymes on the blood itself.

In the table below we give some typical bleeding scenarios together with physical examination
findings and causation.

Cause Type of Features in history Examination findings


bleeding

Fissure in Bright red Painful bleeding that occurs Muco-epithelial defect usually
ano rectal post defecation in small in the midline posteriorly
bleeding volumes. Usually antecedent (anterior fissures more likely to
features of constipation be due to underlying disease)

Haemorroids Bright red Post defecation bleeding noted Normal colon and rectum.
rectal both on toilet paper and drips Proctoscopy may show internal
bleeding into pan. May be alteration of haemorrhoids. Internal
bowel habit and history of haemorrhoids are usually
straining. No blood mixed with impalpable.
stool. No local pain.

Crohns Bright red or Bleeding that is accompanied Perineal inspection may show
disease mixed blood by other symptoms such as fissures or fistulae. Proctoscopy
altered bowel habit, malaise, may demonstrate indurated
history of fissures (especially mucosa and possibly strictures.
Skip lesions may be noted at
anterior) and abscesses. colonoscopy.

Ulcerative Bright red Diarrhoea, weight loss, Proctitis is the most marked
colitis bleeding nocturnal incontinence, passage finding. Peri anal disease is
often mixed of mucous PR. usually absent. Colonoscopy
with stool will show continuous mucosal
lesion.

Rectal cancer Bright red Alteration of bowel habit. Usually obvious mucosal
blood mixed Tenesmus may be present. abnormality. Lesion may be
volumes Symptoms of metastatic fixed or mobile depending upon
disease. disease extent. Surrounding
mucosa often normal, although
polyps may be present.

Image showing a fissure in ano. Typically these are located posteriorly and in the midline. Fissures
at other sites may be associated with underlying disease.

Image sourced from Wikipedia

Colonoscopic image of internal haemorroids. Note these may often be impalpable.


Image sourced from Wikipedia

Investigation

 All patients presenting with rectal bleeding require digital rectal examination and procto-
sigmoidoscopy as a minimal baseline.
 Remember that haemorrhoids are typically impalpable and to attribute bleeding to these in
the absence of accurate internal inspection is unsatisfactory.
 In young patients with no other concerning features in the history a carefully performed
sigmoidoscopy that demonstrates clear haemorrhoidal disease may be sufficient. If clear
views cannot be obtained then patients require bowel preparation with an enema and a
flexible sigmoidscopy performed.
 In those presenting with features of altered bowel habit or suspicion of inflammatory bowel
disease a colonoscopy is the best test.
 Patients with excessive pain who are suspected of having a fissure may require an
examination under general or local anaesthesia.
 In young patients with external stigmata of fissure and a compatible history it is acceptable to
treat medically and defer internal examination until the fissure is healed. If the fissure fails to
heal then internal examination becomes necessary along the lines suggested above to
exclude internal disease.

Special tests

 In patients with a malignancy of the rectum the staging investigations comprise an MRI of the
rectum to identify circumferential resection margin compromise and to identify mesorectal
nodal disease. In addition to this CT scanning of the chest abdomen and pelvis is necessary
to stage for more distant disease. Some centres will still stage the mesorectum with endo
rectal ultrasound but this is becoming far less common.

 Patients with fissure in ano who are being considered for surgical sphincterotomy and are
females who have an obstetric history should probably have ano rectal manometry testing
performed together with endo anal ultrasound. As this service is not universally available it is
not mandatory but in the absence of such information there are continence issues that may
arise following sphincterotomy.

Management

Disease Management

Fissure in ano GTN ointment 0.2% or diltiazem cream applied topically is the usual first line
treatment. Botulinum toxin for those who fail to respond. Internal
sphincterotomy for those who fail with botox, can be considered earlier in
males.

Haemorroids Lifestyle advice, for small internal haemorrhoids can consider injection
sclerotherapy or rubber band ligation. For external haemorrhoids consider
haemorrhoidectomy. Modern options include HALO procedure and stapled
haemorrhoidectomy.

Inflammatory Medical management- although surgery may be needed for fistulating Crohns
bowel disease (setons).

Rectal cancer Anterior resection or abdomino-perineal excision of the colon and rectum.
Total mesorectal excision is now standard of care. Most resections below the
peritoneal reflection will require defunctioning ileostomy. Most patients will
require preoperative radiotherapy.

Next question
Question 97 of 347

Which of the following is the most common childhood brain tumour?

Glioblastoma multiforme

Astrocytoma

Medulloblastoma

Ependymoma

Meningioma

Glioblastoma multiforme is rare in childhood. In contrast, astrocytoma is the commonest brain


tumour in children. Medulloblastoma is no longer the commonest CNS tumour in children (Cancer
research UK)
Please rate this question:

Discuss and give feedback


Next question

CNS tumours

 60% = Glioma and metastatic disease


 20% = Meningioma
 10% = Pituitary lesions

In paediatric practice medulloblastomas (neuroectodermal tumours) were the commonest lesions,


astrocytomas now account for the majority.
Tumours arising in right temporal and frontal lobe may reach considerable size before becoming
symptomatic. Whereas tumours in the speech and visual areas will typically produce early
symptoms.

Diagnosis
MRI Scanning provides the best resolution.

Treatment
Usually surgery, even if tumour cannot be completely resected conditions such as rising ICP can be
addressed with tumour debulking and survival and quality of life prolonged.
Curative surgery can usually be undertaken with lesions such as meningiomas. Gliomas have a
marked propensity to invade normal brain and resection of these lesions is nearly always
incomplete.
Next question
Question 98 of 347

A keen surgical trainee is about to embark on her first hemi arthroplasty for a fractured neck of
femur. In the anaesthetic room the patient is given 1.2g intravenous co-amoxiclav. There is a
possible history of penicillin allergy but the patient is demented and the history is not checked. The
patient then develops severe respiratory compromise and haemodynamic collapse. Which
pathological process accounts for this event?

Binding of the drug to circulating IgG class antibodies

Recognition of the drug by IgE receptors on mast cells

Drug initiated formation of hapten-protein complexes

Binding of the drug to circulating IgM class antibodies

None of the above

Anaphylactic shock:
Antigen recognised by IgE molecules on the surface of mast cells resulting in rapid degranulation
with release of histamine and other inflammatory cytokines.

This is a case of anaphylactic shock. In anaphylaxis the mast cells degranulate.


Please rate this question:

Discuss and give feedback


Next question

Anaphylactic shock

 Suspect if there has been exposure to an allergen

Management
- Remove allergen
- ABCD
- Drugs:

Adrenaline 1:1000 0.5ml INTRAMUSCULARLY (not IV). Repeat after 5 mins if no response.
Then Chlorpheniramine 10mg IV
Then Hydrocortisone 100-200mg IV
Reference
Emergency treatment of anaphylactic reactions. Guidelines for healthcare providers. Working Group
of the Resuscitation Council (UK).2008
Next question
Question 99 of 347

Which of the following hepatobiliary disorders are most classically associated with ulcerative colitis?

Gallstones

Primary sclerosing cholangitis

Bile duct stones

Liver hamartomas

Hepatocellular carcinoma

Primary sclerosing cholangitis is an idiopathic inflammation of the bile ducts. It may result in
episodes of cholestasis and cholangitis and ultimately result in the need for liver transplantation. It
carries a 10% risk of malignant transformation. Crohns disease is associated with gallstones due to
impaired entero-hepatic circulation. Apart from PSC, ulcerative colitis does not increase the risk of
other liver lesions.
Please rate this question:

Discuss and give feedback


Next question

Ulcerative colitis

Ulcerative colitis is a form of inflammatory bowel disease. Inflammation always starts at rectum,
does not spread beyond ileocaecal valve (although backwash ileitis may occur) and is continuous.
The peak incidence of ulcerative colitis is in people aged 15-25 years and in those aged 55-65
years. It is less common in smokers.

The initial presentation is usually following insidious and intermittent symptoms. Features include:

 bloody diarrhoea
 urgency
 tenesmus
 abdominal pain, particularly in the left lower quadrant
 extra-intestinal features (see below)
Questions regarding the 'extra-intestinal' features of inflammatory bowel disease are common. Extra-
intestinal features include sclerosing cholangitis, iritis and ankylosing spondylitis.

Common to both Crohn's disease Notes


(CD) and Ulcerative colitis (UC)

Related to Arthritis: pauciarticular, asymmetric Arthritis is the most common extra-


disease activity Erythema nodosum intestinal feature in both CD and UC
Episcleritis Episcleritis is more common in
Osteoporosis Crohns disease

Unrelated to Arthritis: polyarticular, symmetric Primary sclerosing cholangitis is


disease activity Uveitis much more common in UC
Pyoderma gangrenosum Uveitis is more common in UC
Clubbing
Primary sclerosing cholangitis

Pathology

 Red, raw mucosa, bleeds easily


 No inflammation beyond submucosa (unless fulminant disease)
 Widespread superficial ulceration with preservation of adjacent mucosa which has the
appearance of polyps ('pseudopolyps')
 Inflammatory cell infiltrate in lamina propria
 Neutrophils migrate through the walls of glands to form crypt abscesses
 Depletion of goblet cells and mucin from gland epithelium
 Granulomas are infrequent

Barium enema

 Loss of haustrations
 Superficial ulceration, 'pseudopolyps'
 Long standing disease: colon is narrow and short -'drainpipe colon'

Endoscopy

 Superficial inflammation of the colonic and rectal mucosa


 Continuous disease from rectum proximally
 Superficial ulceration, mucosal islands, loss of vascular definition and continuous ulceration
pattern.
Management

 Patients with long term disease are at increased risk of development of malignancy
 Acute exacerbations are generally managed with steroids, in chronic patients agents such as
azathioprine and infliximab may be used
 Individuals with medically unresponsive disease usually require surgery- in the acute phase
a sub total colectomy and end ileostomy. In the longer term a proctectomy will be required.
An ileoanal pouch is an option for selected patients

References
Ford A et al. Ulcerative colitis. BMJ 2013 (346):29-34.
Next question
Question 100 of 347

Which of the following is least associated with thrombosis?

Endothelial cell damage

Use of tourniquets in surgery

Formation of platelet aggregates

Thrombocytopenia

Carcinoma of the stomach

All the other options either act directly to promote thrombosis e.g. endothelial cell damage or via
changes in consistency or flow of blood.

Please rate this question:

Discuss and give feedback

Next question

Abnormal coagulation

Cause Factors affected

Heparin Prevents activation factors 2,9,10,11


Cause Factors affected

Warfarin Affects synthesis of factors 2,7,9,10

DIC Factors 1,2,5,8,11

Liver disease Factors 1,2,5,7,9,10,11

Interpretation blood clotting test results

Disorder APTT PT Bleeding time

Haemophilia Increased Normal Normal

von Willebrand's disease Increased Normal Increased

Vitamin K deficiency Increased Increased Normal

Next question
Question 101 of 347

A 16 year old boy develops a painful swelling of his distal femur. An osteoblastic sarcoma is
diagnosed. To which of the following sites is this lesion most likely to metastasise?

Inguinal lymph nodes

Common iliac lymph nodes

Liver

Brain

Lung

Sarcomas in which Lymphatic Metastasis is seen:

'RACE For MS'

R: Rhabdomyosarcoma
A: Angiosarcoma
C: Clear cell sarcoma
E: Epithelial cell sarcoma

For: Fibrosarcoma

M: Malignant fibrous histiocytoma


S: Synovial cell sarcoma

Or

'SCARE'

Synovial sarcoma
Clear cell sarcoma
Angiosarcoma
Rhabdomyosarcoma
Epithelioid sarcoma

Sarcomas often metastasise via the haematogenous route and the lung is a common site for
sarcoma metastasis. The liver and brain are often spared (at least initially). A smaller number may
develop lymphatic metastasis (see above).
Please rate this question:
Discuss and give feedback
Next question

Sarcomas

 Malignant tumours of mesenchymal origin

Types
May be either bone or soft tissue in origin.
Bone sarcoma include:

 Osteosarcoma
 Ewings sarcoma (although non bony sites recognised)
 Chondrosarcoma - originate from Chondrocytes

Soft tissue sarcoma are a far more heterogeneous group and include:

 Liposarcoma-adipocytes
 Rhabdomyosarcoma-striated muscle
 Leiomyosarcoma-smooth muscle
 Synovial sarcomas- close to joints (cell of origin not known but not synovium)

Malignant fibrous histiocytoma is a sarcoma that may arise in both soft tissue and bone.

Features
Certain features of a mass or swelling should raise suspicion for a sarcoma these include:

 Large >5cm soft tissue mass


 Deep tissue location or intra muscular location
 Rapid growth
 Painful lump

Assessment
Imaging of suspicious masses should utilise a combination of MRI, CT and USS. Blind biopsy should
not be performed prior to imaging and where required should be done in such a way that the biopsy
tract can be subsequently included in any resection.

Ewings sarcoma

 Commoner in males
 Incidence of 0.3 / 1, 000, 000
 Onset typically between 10 and 20 years of age
 Location by femoral diaphysis is commonest site
 Histologically it is a small round tumour
 Blood borne metastasis is common and chemotherapy is often combined with surgery

Osteosarcoma

 Mesenchymal cells with osteoblastic differentiation


 20% of all primary bone tumours
 Incidence of 5 per 1,000,000
 Peak age 15-30, commoner in males
 Limb preserving surgery may be possible and many patients will receive chemotherapy

Liposarcoma

 Malignancy of adipocytes
 Rare, approximately 2.5 per 1,000,000. They are the second most common soft tissue
sarcoma
 Typically located in deep locations such as retroperitoneum
 Affect older age group usually >40 years of age
 May be well differentiated and thus slow growing although may undergo de-differentiation
and disease progression
 Many tumours will have a pseudocapsule that can misleadingly allow surgeons to feel that
they can 'shell out' these lesions. In reality, tumour may invade at the edge of the
pseudocapsule and result in local recurrence if this strategy is adopted
 Usually resistant to radiotherapy, although this is often used in a palliative setting

Malignant Fibrous Histiocytoma

 Tumour with large number of histiocytes


 Most common sarcoma in adults
 Also described as undifferentiated pleomorphic sarcoma NOS (i.e. Cell of origin is not
known)
 Four major subtypes are recognised: storiform-pleomorphic (70% cases), myxoid (less
aggressive), giant cell and inflammatory
 Treatment is usually with surgical resection and adjuvant radiotherapy as this reduces the
likelihood of local recurrence

Next question
Question 102 of 347

Infection with which of the following micro-organisms may result in a clinical picture resembling
achalasia of the oesphagus?

Epstein Barr virus

Wuchereria Bancrofti

Candida Spp

Trypanosoma Cruzi

Helicobacter Pylori

Infection with Trypanosoma Cruzi may result in destruction of the ganglion cells of the myenteric
plexus, resulting in a clinical picture similar to achalasia.
Please rate this question:

Discuss and give feedback


Next question

Trypanosoma Cruzi

 Protozoan
 Causes Chagas disease
 Carried by bugs which infect the skin whilst feeding
 Penetrate through open wounds and mucous membranes
 Intracellular proliferation
 Major infective sites include CNS, intestinal myenteric plexus, spleen, lymph nodes and
cardiac muscle
 Chronic disease is irreversible, nifurtimox is used to treat acute infection

Next question
Question 103 of 347

A 72 year old man has been unwell for may years and following his death a post mortem is
performed. Tissue is submitted for microscopic evaluation. Evaluation of sections of the myocardium
demonstrates evidence of apple green birefringence with polarised light. What is the most likely
diagnosis?

Amyloidosis

SLE

Tuberculosis

Disseminated B cell lymphoma

Systemic sclerosis

Amyloidosis = apple green birefringence with polarised light

Theme from September 2015 Exam

Please rate this question:

Discuss and give feedback

Next question

Amyloid

Amyloid is an extracellular protein deposit which is insoluble. These deposits disrupt normal tissue
structure and if excessive may affect function. All types of amyloid consist of a major fibrillar protein
that defines the type of amyloid (approximately 90%) plus various minor components.

Amyloid is classified with the prefix A (for amyloid) and the suffix depending upon the fibrillary
protein present. The main clinical types are AA and AL amyloidosis. Systemic AA amyloidosis is a
long-term complication of several chronic inflammatory disorders - e.g. rheumatoid arthritis,
ankylosing spondylitis, Crohn's disease, malignancies and conditions predisposing to recurrent
infections. AL amyloidosis results from extra-cellular deposition of fibril-forming monoclonal
immunoglobulin light chains (most commonly of lambda isotype). Most patients have evidence of
isolated monoclonal gammopathy or asymptomatic myeloma, and the occurrence of AL amyloidosis
in patients with symptomatic multiple myeloma or other B-cell lymphoproliferative disorders is
unusual. AL type amyloidosis is the most common variant. The kidney and heart are two of the most
commonly affected sites. Diagnosis is based on surgical biopsy and characteristic histological
features which consist birefringence under polarised light. Immunohistochemistry is used to
delineate the subtype. Treatment is usually targeted at the underlying cause.

Next question
Question 104 of 347

A 45-year-old man presents to surgical outpatients with a long history of recurrent abdominal pain
and vomiting. He is noted to have a peripheral motor neuropathy on examination. What is the most
likely diagnosis?

Huntington's disease

Myeloma

Acute intermittent porphyria

Lawrence-Moon-Biedl syndrome

Friedreich's ataxia

Neurological signs combined with abdominal pain is acute intermittent porphyria or lead poisoning
until proven otherwise.
Please rate this question:

Discuss and give feedback


Next question

Acute intermittent porphyria

Acute intermittent porphyria (AIP) is a rare autosomal dominant condition caused by a defect in
porphobilinogen deaminase, an enzyme involved in the biosynthesis of haem. The results in the
toxic accumulation of delta aminolaevulinic acid and porphobilinogen. It characteristically presents
with abdominal and neuropsychiatric symptoms in 20-40 year olds. AIP is more common in females
(5:1)

Features

 abdominal: abdominal pain, vomiting


 neurological: motor neuropathy
 psychiatric: e.g. depression
 hypertension and tachycardia common
Diagnosis

 classically urine turns deep red on standing


 raised urinary porphobilinogen (elevated between attacks and to a greater extent during
acute attacks)
 assay of red cells for porphobilinogen deaminase
 raised serum levels of delta aminolaevulinic acid and porphobilinogen

Next question
Question 105 of 347

A 56 year old man presents with episodic facial pain and discomfort whilst eating. He has suffered
from halitosis recently and he frequently complains of a dry mouth. He has a smooth swelling
underneath his right mandible. What is the most likely underlying diagnosis?

Stone impacted in Whartons duct

Stone impacted in Stensens duct

Benign adenoma of the submandibular gland

Adenocarcinoma of the submandibular gland

Squamous cell carcinoma of the submandibular gland

The symptoms are typical for sialolithiasis. The stones most commonly form in the submandibular
gland and therefore may occlude Whartons duct. Stensens duct drains the parotid gland.
Please rate this question:

Discuss and give feedback


Next question

Submandibular glands- disease

Physiology
The submandibular glands secrete approximately 800- 1000ml saliva per day. They typically
produce mixed seromucinous secretions. When parasympathetic activity is dominant; the secretions
will be more serous. The parasympathetic fibres are derived from the chorda tympani nerves and the
submandibular ganglion. Sensory fibres are conveyed by the lingual branch of the mandibular nerve.

Sialolithiasis

 80% of all salivary gland calculi occur in the submandibular gland


 70% of the these calculi are radio-opaque
 Stones are usually composed of calcium phosphate or calcium carbonate
 Patients typically develop colicky pain and post prandial swelling of the gland
 Investigation involves sialography to demonstrate the site of obstruction and associated
other stones
 Stones impacted in the distal aspect of Whartons duct may be removed orally, other stones
and chronic inflammation will usually require gland excision

Sialadenitis

 Usually occurs as a result of Staphylococcus aureus infection


 Pus may be seen leaking from the duct, erythema may also be noted
 Development of a sub mandibular abscess is a serious complication as it may spread
through the other deep fascial spaces and occlude the airway

Submandibular tumours

 Only 8% of salivary gland tumours affect the sub mandibular gland


 Of these 50% are malignant (usually adenoid cystic carcinoma)
 Diagnosis usually involves fine needle aspiration cytology
 Imaging is with CT and MRI
 In view of the high prevalence of malignancy, all masses of the submandibular glands should
generally be excised.

Next question
Question 106 of 347

Which of the following cellular types or features is not seen in sarcoidosis?

Reed Sternberg Cells

T lymphocytes

Macrophages

Asteroid bodies

B lymphocytes

Reed Sternberg cells are seen in Hodgkins disease. All of the other cell types are seen in sarcoid.
Please rate this question:

Discuss and give feedback


Next question

Chronic inflammation

Overview
Chronic inflammation may occur secondary to acute inflammation.In most cases chronic
inflammation occurs as a primary process. These may be broadly viewed as being one of three main
processes:

 Persisting infection with certain organisms such as Mycobacterium tuberculosis which results
in delayed type hypersensitivity reactions and inflammation.
 Prolonged exposure to non-biodegradable substances such as silica or suture materials
which may induce an inflammatory response.
 Autoimmune conditions involving antibodies formed against host antigens.

Acute vs. Chronic inflammation


Acute inflammation Chronic inflammation
Changes to existing vascular structure and increased Angiogenesis predominates
permeability of endothelial cells

Infiltration of neutrophils Macrophages, plasma cells and


lymphocytes predominate

Process may resolve with: Healing by fibrosis is the main result

 Suppuration
 Complete resolution
 Abscess formation
 Progression to chronic inflammation
 Healing by fibrosis

Granulomatous inflammation
A granuloma consists of a microscopic aggregation of macrophages (with epithelial type
arrangement =epitheliod). Large giant cells may be found at the periphery of granulomas.

Mediators
Growth factors released by activated macrophages include agents such as interferon and fibroblast
growth factor (plus many more). Some of these such as interferons may have systemic features
resulting in systemic symptoms and signs, which may be present in individuals with long standing
chronic inflammation.

The finding of granulomas is pathognomonic of chronic inflammation, as illustrated in this biopsy


from a patient with colonic Crohns disease

Image sourced from Wikipedia


Question 107 of 347

Which of the following diseases is not considered a risk factor for gastric cancer?

Polya gastrectomy for antral ulcer

Atrophic gastritis

Intestinal metaplasia of columnar type at the gastric cardia

Patient with polyp showing medium grade dysplasia

Long term therapy with H2 blockers

Although some acid lowering procedures increase the risk of gastric cancer the use of H 2 blockers
does not, at the present time, seem to increase the risk.
Please rate this question:

Discuss and give feedback


Next question

Gastric cancer

Overview
There are 700,000 new cases of gastric cancer worldwide each year. It is most common in Japan
and less common in western countries. It is more common in men and incidence rises with
increasing age. The exact cause of many sporadic cancer is not known, however, familial cases do
occur in HNPCC families. In addition, smoking and smoked or preserved foods increase the risk.
Japanese migrants retain their increased risk (decreased in subsequent generations). The
distribution of the disease in western countries is changing towards a more proximal location
(perhaps due to rising obesity).

Pathology
There is some evidence of support a stepwise progression of the disease through intestinal
metaplasia progressing to atrophic gastritis and subsequent dysplasia, through to cancer. The
favoured staging system is TNM. The risk of lymph node involvement is related to size and depth of
invasion; early cancers confined to submucosa have a 20% incidence of lymph node metastasis.
Tumours of the gastro-oesophageal junction are classified as below:
Type True oesophageal cancers and may be associated with Barrett's oesophagus.
1

Type Carcinoma of the cardia, arising from cardiac type epithelium


2 or short segments with intestinal metaplasia at the oesophagogastric junction.

Type Sub cardial cancers that spread across the junction. Involve similar nodal stations to
3 gastric cancer.

Groups for close endoscopic monitoring

 Intestinal metaplasia of columnar type


 Atrophic gastritis
 Low to medium grade dysplasia
 Patients who have previously undergone resections for benign peptic ulcer disease (except
highly selective vagotomy).

Referral to endoscopy

Patients of any age with Patients without Worsening dyspepsia


dyspepsia and any of the dyspepsia
following

Chronic gastrointestinal bleeding Dysphagia Barretts oesophagus

Dysphagia Unexplained abdominal Intestinal metaplasia


pain or weight loss

Weight loss Vomiting Dysplasia

Iron deficiency anaemia Upper abdominal mass Atrophic gastritis

Upper abdominal mass Jaundice Patient aged over 55 years with


unexplained or persistent
dyspepsia
Upper GI endoscopy performed for dyspepsia. The addition of dye spraying (as shown in the bottom
right) may facilitate identification of smaller tumours

Image sourced from Wikipedia

Staging

 CT scanning of the chest abdomen and pelvis is the routine first line staging investigation in
most centres.
 Laparoscopy to identify occult peritoneal disease
 PET CT (particularly for junctional tumours)

Treatment

 Proximally sited disease greater than 5-10cm from the OG junction may be treated by sub
total gastrectomy
 Total gastrectomy if tumour is <5cm from OG junction
 For type 2 junctional tumours (extending into oesophagus) oesophagogastrectomy is usual
 Endoscopic sub mucosal resection may play a role in early gastric cancer confined to the
mucosa and perhaps the sub mucosa (this is debated)
 Lymphadenectomy should be performed. A D2 lymphadenectomy is widely advocated by the
Japanese, the survival advantages of extended lymphadenectomy have been debated.
However, the overall recommendation is that a D2 nodal dissection be undertaken.
 Most patients will receive chemotherapy either pre or post operatively.

Prognosis

UK Data

Disease extent Percentage 5 year survival

All RO resections 54%

Early gastric cancer 91%

Stage 1 87%

Stage 2 65%

Stage 3 18%

Operative procedure

Total Gastrectomy , lymphadenectomy and Roux en Y anastomosis

General anaesthesia
Prophylactic intravenous antibiotics
Incision: Rooftop.
Perform a thorough laparotomy to identify any occult disease.
Mobilise the left lobe of the liver off the diaphragm and place a large pack over it. Insert a large self
retaining retractor e.g. omnitract or Balfour (take time with this, the set up should be perfect). Pack
the small bowel away.
Begin by mobilising the omentum off the transverse colon.
Proceed to detach the short gastric vessels.
Mobilise the pylorus and divide it at least 2cm distally using a linear cutter stapling device.
Continue the dissection into the lesser sac taking the lesser omentum and left gastric artery flush at
its origin.
The lymph nodes should be removed en bloc with the specimen where possible.
Place 2 stay sutures either side of the distal oesophagus. Ask the anaesthetist to pull back on the
nasogastric tube. Divide the distal oesophagus and remove the stomach.
The oesphago jejunal anastomosis should be constructed. Identify the DJ flexure and bring a loop of
jejunum up to the oesophagus (to check it will reach). Divide the jejunum at this point. Bring the
divided jejunum either retrocolic or antecolic to the oesophagus. Anastamose the oesophagus to the
jejunum, using either interrupted 3/0 vicryl or a stapling device. Then create the remainder of the
Roux en Y reconstruction distally.
Place a jejunostomy feeding tube.
Wash out the abdomen and insert drains (usually the anastomosis and duodenal stump). Help the
anaesthetist insert the nasogastric tube (carefully!)
Close the abdomen and skin.
Enteral feeding may commence on the first post-operative day. However, most surgeons will leave
patients on free NG drainage for several days and keep them nil by mouth.
Next question
Question 108 of 347

A 56 year old man is diagnosed as having a glioma. From which of the following cell types do these
tumours usually originate?

Astrocytes

Oligodendrocytes

Ependymal cells

Squamous cells

Neuroglial cells

Theme from January 2012 Exam


Gliomas originate from glial (otherwise known as neuroglial) cells. These serve a structural function
in the CNS. The tumours produced may resemble a number of CNS cell types. Tumours are
therefore named according to the cells they resemble rather than the origin. Where this is not
possible they are termed gliomas.
Please rate this question:

Discuss and give feedback


Next question

Glioma

Glioma is a tumour that is typically found in the CNS. These tumours arise from glial cells. They are
sub categorised according to the cell type they most closely resemble.

Glioma sub types

 Ependymomas- Ependymal cells


 Astocytomas- Astrocytes (including glioblastoma)
 Oligodendrogliomas- Oligodendrocytes
 Mixed- e.g. oligoastrocytomas

Gliomas are categorised as being either high or low grade lesions (the former has the worse
prognosis). They may be either supra or infra tentorial. Their symptoms will typically reflect their site
of origin. Glioblastoma multiforme has the worst prognosis and few patients will survive beyond 12
months.
Next question
Question 109 of 347

A 78 year old man presents with unilateral deafness which has been present for the past 3 months.
On examination, Webers test localises to the contralateral side and a CT scan of his head shows a
thickened calvarium with areas of sclerosis and radiolucency. His blood tests show an elevated
alkaline phosphatase, normal serum calcium and normal PTH levels. Which of the following is the
most likely underlying diagnosis?

Multiple myeloma with skull involvement

Osteoporosis

Pagets disease with skull involvement

Lung cancer with skull metastasis

Osteopetrosis with skull involvement

Of the conditions listed Pagets disease is the most likely diagnosis (skull vault expansion and
sensorineural hearing loss). Multiple myeloma would typically result in multiple areas of radiolucency
and usually raised calcium in this setting. Osteopetrosis is a recognised cause of the features
described. However, it is a rare inherited disorder and usually presents in children in young adults.
Presentation at this stage with no prior symptoms would be extremely rare and therefore this is not
themost likely diagnosis.
Please rate this question:

Discuss and give feedback


Next question

Pagets disease

Paget's disease is a disease of increased but uncontrolled bone turnover and is characterised by
architecturally abnormal bones. It is thought to be primarily a disorder of osteoclasts, with excessive
osteoclastic resorption followed by increased osteoblastic activity causing areas of sclerosis and
deformity. Paget's disease is common (UK prevalence 5%) but symptomatic in only 1 in 20 patients

Predisposing factors

 increasing age
 male sex
 northern latitude
 family history

Clinical features

 bone pain (e.g. pelvis, lumbar spine, femur)


 classical, untreated features: bowing of tibia, bossing of skull
 raised alkaline phosphatase (ALP) - calcium* and phosphate are typically normal
 skull x-ray: thickened vault, osteoporosis circumscripta

Indications for treatment include bone pain, skull or long bone deformity, fracture, periarticular
Paget's

 bisphosphonate (either oral risedronate or IV zoledronate)


 calcitonin is less commonly used now

Complications

 deafness (cranial nerve entrapment)


 bone sarcoma (1% if affected for > 10 years)
 fractures
 skull thickening
 high-output cardiac failure

*usually normal in this condition but hypercalcaemia may occur with prolonged immobilisation
Next question
Question 110-112 of 347

Theme: Genetic causes of cancer

A. Multiple endocrine neoplasia type I


B. Multiple endocrine neoplasia type II
C. Gardner's syndrome
D. Lynch Syndrome
E. Kartagener's syndrome
F. Von Recklinghausen's disease

Please select the most likely condition for the disease process described. Each option may be used
once, more than once or not at all

110. A 5 year old boy presents with recurrent episodes of sinusitis. The casualty staff are
surprised to find his liver lying in the left upper quadrant of the abdomen.

You answered Multiple endocrine neoplasia type I

The correct answer is Kartagener's syndrome

This is a case of Kartagener's syndrome. The primary problem is of immotile cilia


syndrome. When associated with situs inversus Kartagener's syndrome is diagnosed.

111. A 22 year old man presents with carcinoma of the caecum. His brother died from
colorectal cancer aged 25 and his mother died from endometrial cancer aged 38.

You answered Multiple endocrine neoplasia type I

The correct answer is Lynch Syndrome

This is a case of Lynch syndrome HNPCC. It is transmitted in an autosomal dominant


fashion. Cancer of the uterine body is more common in HNPCC than Gardners syndrome

112. A tall 32 year old lady presents with a diffuse neck swelling a carcinoma of the thyroid
medullary type is diagnosed.

You answered Multiple endocrine neoplasia type I

The correct answer is Multiple endocrine neoplasia type II

This is a case MEN type IIb. It is associated with phaeochromocytomas and is


transmitted in an autosomal dominant pattern if inherited. All MEN II tend to have
medullary carcinoma of the thyroid as a presenting feature

Please rate this question:

Discuss and give feedback


Next question

Genetics and surgical disease

Some of the more commonly occurring genetic conditions occurring in surgical patients are
presented here.

Li-Fraumeni Syndrome

 Autosomal dominant
 Consists of germline mutations to p53 tumour suppressor gene
 High incidence of malignancies particularly sarcomas and leukaemias
 Diagnosed when:

*Individual develops sarcoma under 45 years


*First degree relative diagnosed with any cancer below age 45 years and another family member
develops malignancy under 45 years or sarcoma at any age

BRCA 1 and 2

 Carried on chromosome 17 (BRCA 1) and Chromosome 13 (BRCA 2)


 Linked to developing breast cancer (60%) risk.
 Associated risk of developing ovarian cancer (55% with BRCA 1 and 25% with BRCA 2).

Lynch Syndrome

 Autosomal dominant
 Develop colonic cancer and endometrial cancer at young age
 80% of affected individuals will get colonic and/ or endometrial cancer
 High risk individuals may be identified using the Amsterdam criteria

Amsterdam criteria
Three or more family members with a confirmed diagnosis of colorectal cancer, one of whom is a
first degree (parent, child, sibling) relative of the other two.
Two successive affected generations.
One or more colon cancers diagnosed under age 50 years.
Familial adenomatous polyposis (FAP) has been excluded.
Gardners syndrome

 Autosomal dominant familial colorectal polyposis


 Multiple colonic polyps
 Extra colonic diseases include: skull osteoma, thyroid cancer and epidermoid cysts
 Desmoid tumours are seen in 15%
 Mutation of APC gene located on chromosome 5
 Due to colonic polyps most patients will undergo colectomy to reduce risk of colorectal
cancer
 Now considered a variant of familial adenomatous polyposis coli

Next question
Question 113 of 347

A 45 year old man presents with symptoms of urinary colic. In the history he has suffered from
recurrent episodes of frank haematuria over the past week or so. On examination he has a left loin
mass and a varicocele. The most likely diagnosis is:

Renal adenocarcinoma

Renal cortical adenoma

Squamous cell carcinoma of the renal pelvis

Retroperitoneal fibrosis

Nephroblastoma

 Renal adenocarcinoma are the most common renal malignancy and account for 75% cases.
 Patients may develop frank haematuria and have episodes of clot colic.
 A Grawitz tumour is an eponymous name for Renal Adenocarcinoma.
 May metastasise to bone.

Please rate this question:

Discuss and give feedback


Next question

Renal tumours

Renal cell carcinoma


Renal cell carcinoma is an adenocarcinoma of the renal cortex and is believed to arise from the
proximal convoluted tubule. They are usually solid lesions, up to 20% may be multifocal, 20% may
be calcified and 20% may have either a cystic component or be wholly cystic. They are often
circumscribed by a pseudocapsule of compressed normal renal tissue. Spread may occur either by
direct extension into the adrenal gland, renal vein or surrounding fascia. More distant disease
usually occurs via the haematogenous route to lung, bone or brain.
Renal cell carcinoma comprise up to 85% of all renal malignancies. Males are more commonly
affected than females and sporadic tumours typically affect patients in their sixth decade.
Patients may present with a variety of symptoms including; haematuria (50%), loin pain (40%), mass
(30%) and up to 25% may have symptoms of metastasis.Less than 10% have the classic triad of
haematuria, pain and mass.

Investigation
Many cases will present as haematuria and be discovered during diagnostic work up. Benign renal
tumours are rare, so renal masses should be investigated with multislice CT scanning. Some units
will add and arterial and venous phase to the scan to demonstrate vascularity and evidence of caval
ingrowth.

CT scanning of the chest and abdomen to detect distant disease should also be undertaken.

Routine bone scanning is not indicated in the absence of symptoms.

Biopsy should not be performed when a nephrectomy is planned but is mandatory before any
ablative therapies are undertaken.

Assessment of the functioning of the contra lateral kidney.

Management
T1 lesions may be managed by partial nephrectomy and this gives equivalent oncological results to
total radical nephrectomy. Partial nephrectomy may also be performed when there is inadequate
reserve in the remaining kidney.

For T2 lesions and above a radical nephrectomy is standard practice and this may be performed via
a laparoscopic or open approach. Preoperative embolisation is not indicated nor is resection of
uninvolved adrenal glands. During surgery early venous control is mandatory to avoid shedding of
tumour cells into the circulation.

Patients with completely resected disease do not benefit from adjuvant therapy with either
chemotherapy or biological agents. These should not be administered outside the setting of clinical
trials.

Patients with transitional cell cancer will require a nephroureterectomy with disconnection of the
ureter at the bladder.

References
Lungberg B et al. EAU guidelines on renal cell carcinoma: The 2010 update. European Urology 2010
(58): 398-406.
Next question
Question 114 of 347

A 63 year old man finds that he has to stop walking after 100 yards due to bilateral calf pain. He
finds that bending forwards and walking up hill helps. He is able to ride a bike without any pain. What
is the most likely underlying cause?

Lumbar canal stenosis

Diabetic neuropathy

Aorto-iliac occlusion

Occlusion of the superficial femoral artery

Pelvic rheumatoid arthritis

Theme from April 2012 Exam


Theme from April 2013 Exam
The positional nature of the pain and the fact that improves with walking uphill makes an underlying
vascular aetiology far less likely.

Please rate this question:

Discuss and give feedback

Next question

Lumbar spinal stenosis

Lumbar spinal stenosis is a condition in which the central canal is narrowed by tumour, disk prolapse
or other similar degenerative changes.
Patients may present with a combination of back pain, neuropathic pain and symptoms mimicking
claudication. One of the main features that may help to differentiate it from true claudication in the
history is the positional element to the pain. Sitting is better than standing and patients may find it
easier to walk uphill rather than downhill. The neurogenic claudication type history makes lumbar
spinal stenosis a likely underlying diagnosis, the absence of such symptoms makes it far less likely.

Pathology
Degenerative disease is the commonest underlying cause. Degeneration is believed to begin in the
intervertebral disk where biochemical changes such as cell death and loss of proteoglycan and
water content lead to progressive disk bulging and collapse. This process leads to an increased
stress transfer to the posterior facet joints, which accelerates cartilaginous degeneration,
hypertrophy, and osteophyte formation; this is associated with thickening and distortion of the
ligamentum flavum. The combination of the ventral disk bulging, osteophyte formation at the dorsal
facet, and ligamentum flavum hyptertrophy combine to circumferentially narrow the spinal canal and
the space available for the neural elements. The compression of the nerve roots of the cauda equina
leads to the characteristic clinical signs and symptoms of lumbar spinal stenosis.

Diagnosis
MRI scanning is the best modality for demonstrating the canal narrowing. Historically a bicycle test
was used as true vascular claudicants could not complete the test.

Treatment
Laminectomy

Next question
Question 115 of 347

A 73 year old lady is admitted for a laparoscopic cholecystectomy. During her pre-operative
assessment it is noted that she is receiving furosemide for the treatment of hypertension.
Approximately what proportion of the sodium that is filtered at the glomerulus will be subsequently
excreted?

Up to 25%

Upt to 75%

Between 3 and 5%

<2%

Between 1 and 2%

Theme from 2010 Exam


The loop diuretics can lead to marked increases in the amount of sodium excreted. They act in the
medullary and cortical aspects of the thick ascending limb of the loop of Henle. This results in a
decreased medullary osmolal gradient and increases free water excretion (as well as loss of
sodium). Because loop diuretics result in the loss of both sodium and water they are less frequently
associated with hyponatraemia than thiazide diuretics (these latter agents act in the cortex and do
not affect urine concentrating ability).

Please rate this question:

Discuss and give feedback

Next question

Diuretic agents
The diuretic drugs are divided into three major classes, which are distinguished according to the site
at which they impair sodium reabsorption: loop diuretics in the thick ascending loop of Henle,
thiazide type diuretics in the distal tubule and connecting segment; and potassium sparing diuretics
in the aldosterone - sensitive principal cells in the cortical collecting tubule.
In the kidney, sodium is reabsorbed through Na+/ K+ ATPase pumps located on the basolateral
membrane. These pumps return reabsorbed sodium to the circulation and maintain low intracellular
sodium levels. This latter effect ensures a constant concentration gradient.

Physiological effects of commonly used diuretics

Carrier or channel Percentage of filtered sodium


Site of action Diuretic inhibited excreted

Ascending limb of loop of Frusemide Na+/K+ 2Cl - carrier Up to 25%


Henle

Distal tubule and connecting Thiazides Na+Cl- carrier Between 3 and 5%


segment

Cortical collecting tubule Spironolactone Na+/K+ ATP ase pump Between 1 and 2%

Next question
Question 116 of 347

A 59 year old man presents with recurrent episodes of urinary sepsis. In his history he mentions that
he has suffered from recurrent attacks of left iliac fossa pain over the past few months. He has also
notices bubbles in his urine. He undergoes a CT scan which shows a large inflammatory mass in the
left iliac fossa. No other abnormality is detected. The most likely diagnosis is:

Ulcerative colitis

Crohns disease

Mesenteric ischaemia

Diverticular disease

Rectal cancer

Diverticular disease is one of the commonest causes of colovesical fistula

Theme from April 2016 Exam

Recurrent attacks of diverticulitis may cause the development of local abscesses which may erode
into the bladder resulting in urinary sepsis and pneumaturia. This would be an unusual presentation
from Crohns disease and rectal cancer would be more distally sited and generally evidence of extra
colonic disease would be present if the case were malignant and this advanced.
Please rate this question:

Discuss and give feedback


Next question

Diverticular disease

Diverticular disease is a common surgical problem. It consists of herniation of colonic mucosa


through the muscular wall of the colon. The usual site is between the taenia coli where vessels
pierce the muscle to supply the mucosa. For this reason, the rectum, which lacks taenia, is often
spared.

Symptoms

 Altered bowel habit


 Bleeding
 Abdominal pain

Complications

 Diverticulitis
 Haemorrhage
 Development of fistula
 Perforation and faecal peritonitis
 Perforation and development of abscess
 Development of diverticular phlegmon

Diagnosis
Patients presenting in clinic will typically undergo either a colonoscopy, CT cologram or barium
enema as part of their diagnostic work up. All tests can identify diverticular disease. It can be far
more difficult to confidently exclude cancer, particularly in diverticular strictures.

Acutely unwell surgical patients should be investigated in a systematic way. Plain abdominal films
and an erect chest x-ray will identify perforation. An abdominal CT scan (not a CT cologram) with
oral and intravenous contrast will help to identify whether acute inflammation is present but also the
presence of local complications such as abscess formation.

Severity Classification- Hinchey

I Para-colonic abscess

II Pelvic abscess

III Purulent peritonitis

IV Faecal peritonitis

Treatment

 Increase dietary fibre intake.


 Mild attacks of diverticulitis may be managed conservatively with antibiotics.
 Peri colonic abscesses should be drained either surgically or radiologically.
 Recurrent episodes of acute diverticulitis requiring hospitalisation are a relative indication for
a segmental resection.
 Hinchey IV perforations (generalised faecal peritonitis) will require a resection and usually a
stoma. This group have a very high risk of post operative complications and usually require
HDU admission. Less severe perforations may be managed by laparoscopic washout and
drain insertion.

Next question
Question 117 of 347

Which of the following is least likely to occur in association with severe atrophic gastritis?

Gastric ulcers

Gastric cancer

Anaemia

Duodenal ulcers

Gastric polyps

Achlorhydria would make the formation of duodenal ulcers unlikely. Note the question states "least
likely".

Due to the loss of gastric acid a duodenal ulcer is unlikely. Note that gastric polyps may form (see
below).

Please rate this question:

Discuss and give feedback

Next question

Gastritis

Type of gastritis Features

Type A Autoimmune
Type of gastritis Features

Circulating antibodies to parietal cells, causes reduction in cell mass and


hypochlorhydria
Loss of parietal cells = loss of intrinsic factor = B12 malabsorption
Absence of antral involvement
Hypochlorhydria causes elevated gastrin levels- stimulating enterochromaffin cells and
adenomas may form

Type B Antral gastritis


Associated with infection with helicobacter pylori infection
Intestinal metaplasia may occur in stomach and require surveillance endoscopy
Peptic ulceration may occur

Reflux gastritis Bile refluxes into stomach, either post surgical or due to failure of pyloric function
Histologically, evidence of chronic inflammation, and foveolar hyperplasia
May respond to therapy with prokinetics

Erosive gastritis Agents disrupt the gastric mucosal barrier


Most commonly due to NSAIDs and alcohol
With NSAIDs the effects occur secondary to COX 1 inhibition

Stress ulceration This occurs as a result of mucosal ischaemia during hypotension or hypovolaemia
The stomach is the most sensitive organ in the GI tract to ischaemia following
hypovolaemia
Diffuse ulceration may occur
Prophylaxis with acid lowering therapy and sucralfate may minimise complications

Menetriers Gross hypertrophy of the gastric mucosal folds, excessive mucous production and
disease hypochlorhydria
Pre malignant condition

References
Whiting J et al. The long term results of endoscopic surveillance of premalignant gastric
lesions. Gut2002; 50 :378381.
Dixon M et al. Reflux gastritis: distinct histopathological entity?J Clin Pathol 1986; 39 : 524-530.

Next question
Question 118-120 of 347

Theme: Lung cancer

A. Adenocarcinoma
B. Small cell lung cancer
C. Large cell lung cancer
D. Squamous cell carcinoma

Please select the most likely lung cancer variant for the scenario described. Each option may be
used once, more than once or not at all.

118. A 73 year old heavy smoker presents with haemoptysis. On examination he is cachectic
and shows evidence of clubbing. Imaging shows a main bronchial tumour with massive
mediastinal lymphadenopathy together with widespread visceral metastases.

You answered Adenocarcinoma

The correct answer is Small cell lung cancer

Theme from April 2012


Small cell carcinoma is associated with disseminated disease at presentation in the
majority of cases. Most cases occur in the main airways and paraneoplastic features are
common.

119. A 68 year old female who has never smoked presents with a mass at the periphery of her
right lung.

Adenocarcinoma

Adenocarcinomas are the most common tumour type present in never smokers. They are
usually located at the periphery.

120. An 85 year old man presents with a cough and haemoptysis. He has a modest smoking
history of 15 pack years. He is found to have a tumour located in the right main bronchus,
with no evidence of metastatic disease. He decides not undergo any treatment and he
remains well for a further 12 months before developing symptomatic metastasis.

You answered Adenocarcinoma

The correct answer is Squamous cell carcinoma

Squamous cell carcinomas are reported to be more slow growing and are typically
centrally located. Small cell carcinomas are usually centrally located. However, small cell
carcinomas would seldom be associated with a survival of a year without treatment.

Please rate this question:

Discuss and give feedback


Next question

Lung cancer

Lung cancers may be classified according to histological subtypes. The main distinction is between
small cell and non small cell lung cancer. Non small cell lung cancer is the most common variant and
accounts for 80% of all lung cancers.

Non small cell lung cancer


These share common features of prognosis and management. They comprise the following
tumours:

 Squamous cell carcinoma (25% cases)


 Adenocarcinoma (40% cases)
 Large cell carcinoma (10% cases)

Paraneoplastic features and early disease dissemination are less likely than with small cell lung
carcinoma. Adenocarcinoma is the most common lung cancer type encountered in never smokers.

Small cell lung carcinoma


Small cell lung carcinomas are comprised of cells with a neuro endocrine differentiation. The
neuroendocrine hormones may be released from these cells with a wide range of paraneoplastic
associations. These tumours are strongly associated with smoking and will typically arise in the
larger airways. They disseminate early in the course of the disease and although they are usually
chemosensitive this seldom results in long lasting remissions.
Next question
Question 121 of 347

Which of the following is not found on a blood film post splenectomy?

Pappenheimer bodies

Stipple cells

Erythrocyte containing siderotic granules

Howell-Jolly bodies

Target cells

Stipple cells are found in lead poisoning/haemoglobinopathies.

Blood film in hyposplenism:

Howell-Jolly bodies
Pappenheimer bodies
Poikilocytes (Target cells)
Erythrocyte containing siderotic granules
Heinz bodies

Please rate this question:

Discuss and give feedback

Next question

Post splenectomy blood film changes

The loss of splenic tissue results in the inability to readily remove immature or abnormal red blood
cells from the circulation. The red cell count does not alter significantly. However, cytoplasmic
inclusions may be seen e.g. Howell-Jolly bodies.
In the first few days after splenectomy target cells, siderocytes and reticulocytes will appear in the
circulation. Immediately following splenectomy a granulocytosis (mainly composed of neutrophils) is
seen, this is replaced by a lymphocytosis and monocytosis over the following weeks.
The platelet count is usually increased and this may be persistent, oral antiplatelet agents may be
needed in some patients.

Image showing Howell Jolly bodies (arrowed)

Image sourced from Wikipedia

Next question
Question 122 of 347

A 45 year old man with long standing ulcerative colitis and rectal dysplasia presents with a DALM
lesion in the rectum. What is the most appropriate management option?

Snare polypectomy

Repeat endoscopy in 2 years

Discharge

Anterior resection

Panproctocolectomy

DALM lesions complicating ulcerative colitis should be managed with panproctocolectomy. An


anterior resection is inadequate since it will only remove the rectum and ulcerative colitis affects the
entire colon. Since many will be associated with invasion a snare polypectomy is not sufficient either.
Please rate this question:

Discuss and give feedback


Next question

Colonic lesions - DALM

 The term DALM lesion refers to a Dysplasia Associated Lesion or Mass.


 They may complicate dysplasia occurring in patients with longstanding ulcerative colitis.
 They have a high incidence of invasive foci.
 When they complicate longstanding ulcerative colitis, they should be treated by
panproctocolectomy.

Next question
Question 123 of 347

Which of the metastatic bone tumours described below is at the greatest risk of pathological fracture
?

Proximal humeral lesion from a prostate cancer

Vertebral body lesions from a prostate cancer

Peritrochanteric lesion from a carcinoma of the breast

Proximal humeral lesion from a carcinoma of the breast

Peritrochanteric lesion from a prostate cancer

Peritrochanteric lesions have the greatest risks of fracture (due to loading). The lesions from breast
cancer are usually lytic and therefore at higher risk rather than the sclerotic lesions from prostate
cancer.

Please rate this question:

Discuss and give feedback

Next question

Metastatic bone disease- risk of fracture

Metastatic bone tumours may be described as blastic, lytic or mixed. Osteoblastic metastatic
disease has the lowest risk of spontaneous fracture when compared to osteolytic lesions of a similar
size.
Lesions affecting the peritrochanteric region are most prone to spontaneous fracture (because of
loading forces at that site).
The factors are incorporated into the Mirel Scoring system to stratify the risk of spontaneous fracture
for bone metastasis of varying types.
Mirel Scoring system

Score Radiographic Width of bone


points Site appearance involved Pain

1 Upper Blastic Less than 1/3 Mild


extremity

2 Lower extremity Mixed 1/3 to 2/3 Moderate

3 Peritrochanteric Lytic More than 2/3 Aggravated by


function

Depending upon the score the treatment should be as follows:

Score Risk of fracture Treatment

9 or greater Impending (33%) Prophylactic fixation

8 Borderline Consider fixation

7 or less Not impending (4%) Non operative management

Next question
Question 124 of 347

A 63 year old male presents with several episodes of haematuria. He suffers from COPD secondary
to long term smoking. What is the most likely underlying cause?

Renal cortical adenoma

Renal adenocarcinoma

Nephroblastoma

Transitional cell carcinoma of the bladder

Adenocarcinoma of the bladder

Theme from 2009 Exam


TCC is the most common subtype and is strongly linked to smoking. The important point to note in
this question is the term most likely as renal adenocarcinoma may produce similar symptoms but is
less likely.
Please rate this question:

Discuss and give feedback


Next question

Bladder cancer

Bladder cancer is the second most common urological cancer. It most commonly affects males aged
between 50 and 80 years of age. Those who are current, or previous (within 20 years), smokers
have a 2-5 fold increased risk of the disease. Exposure to hydrocarbons such as 2-Naphthylamine
increases the risk. Although rare in the UK, chronic bladder inflammation arising from
Schistosomiasis infection remains a common cause of squamous cell carcinomas, in those countries
where the disease is endemic.

Benign tumours
Benign tumours of the bladder including inverted urothelial papilloma and nephrogenic adenoma are
uncommon.

Bladder malignancies

 Transitional cell carcinoma (>90% of cases)


 Squamous cell carcinoma ( 1-7% -except in regions affected by schistosomiasis)
 Adenocarcinoma (2%)

Transitional cell carcinomas may arise as solitary lesions, or may be multifocal, owing to the effect of
"field change" within the urothelium. Up to 70% of TCC's will have a papillary growth pattern. These
tumours are usually superficial in location and accordingly have a better prognosis. The remaining
tumours show either mixed papillary and solid growth or pure solid growths. These tumours are
typically more prone to local invasion and may be of higher grade, the prognosis is therefore worse.
Those with T3 disease or worse have a 30% (or higher) risk of regional or distant lymph node
metastasis.

TNM Staging
Stage Description

T0 No evidence of tumour

Ta Non invasive papillary carcinoma

T1 Tumour invades sub epithelial connective tissue

T2a Tumor invades superficial muscularis propria (inner half)

T2b Tumor invades deep muscularis propria (outer half)

T3 Tumour extends to perivesical fat

T4 Tumor invades any of the following: prostatic stroma, seminal vesicles, uterus, vagina

T4a Invasion of uterus, prostate or bowel

T4b Invasion of pelvic sidewall or abdominal wall

N0 No nodal disease
Stage Description

N1 Single regional lymph node metastasis in the true pelvis (hypogastric, obturator, external
iliac, or presacral lymph node)

N2 Multiple regional lymph node metastasis in the true pelvis (hypogastric, obturator, external
iliac, or presacral lymph node metastasis)

N3 Lymph node metastasis to the common iliac lymph nodes

M0 No distant metastasis

M1 Distant disease

Presentation
Most patients (85%) will present with painless, macroscopic haematuria. In those patients with
incidental microscopic haematuria, up to 10% of females aged over 50 will be found to have a
malignancy (once infection excluded).

Staging
Most will undergo a cystoscopy and biopsies or TURBT, this provides histological diagnosis and
information relating to depth of invasion. Locoregional spread is best determined using pelvic MRI
and distant disease CT scanning. Nodes of uncertain significance may be investigated using PET
CT.

Treatment
Those with superficial lesions may be managed using TURBT in isolation. Those with recurrences or
higher grade/ risk on histology may be offered intravesical chemotherapy. Those with T2 disease are
usually offered either surgery (radical cystectomy and ileal conduit) or radical radiotherapy.

Prognosis
T1 90%

T2 60%

T3 35%
T4a 10-25%

Any T, N1-N2 30%

Next question
Question 125-127 of 347

Theme: Neck lumps

A. Cystic hygroma

B. Bartonella infection

C. Mycobacterium tuberculosis infection

D. Branchial cyst

E. Thyroglossal cyst

F. Pharyngeal pouch

G. Follicular thyroid cyst

H. Parathyroid adenoma

I. None of the above

Please select the most likely underlying disease process for the scenario given. Each option may be
used once, more than once or not at all.

125. A 25 year old cat lover presents with symptoms of abdominal pain, lethargy and sweats. These
have been present for the past two weeks. On examination she has lymphadenopathy in the
posterior triangle.

You answered Cystic hygroma

The correct answer is Bartonella infection

Theme from January 2015 Exam


Bartonella infection may occur following a cat scratch. The organism is intracellular. Generalised
systemic symptoms may occur for a week or so prior to clinical presentation.

126. A 25 year old lady presents with an swelling located at the anterior border of the
sternocleidomastoid muscle. The swelling is intermittent and on examination it is soft and
fluctuant.

You answered Cystic hygroma

The correct answer is Branchial cyst

Branchial cysts are remnants of the branchial cleft. They may become infected.

127. A 38 year old lady presents with a mass in the midline of the neck immediately below the hyoid
bone. It moves upwards on tongue protrusion.

You answered Cystic hygroma

The correct answer is Thyroglossal cyst

Thyroglossal cysts are usually located in the midline and are linked to the foramen caecum and
will thus move upwards on tongue protrusion.

Please rate this question:

Discuss and give feedback

Next question

Neck lumps

The table below gives characteristic exam question features for conditions causing neck lumps:

Reactive By far the most common cause of neck swellings. There may be a history of
lymphadenopathy local infection or a generalised viral illness

Lymphoma Rubbery, painless lymphadenopathy


The phenomenon of pain whilst drinking alcohol is very uncommon
There may be associated night sweats and splenomegaly

Thyroid swelling May be hypo-, eu- or hyperthyroid symptomatically


Moves upwards on swallowing

Thyroglossal cyst More common in patients < 20 years old


Usually midline, between the isthmus of the thyroid and the hyoid bone
Moves upwards with protrusion of the tongue
May be painful if infected

Pharyngeal pouch More common in older men


Represents a posteromedial herniation between thyropharyngeus and
cricopharyngeus muscles
Usually not seen, but if large then a midline lump in the neck that gurgles on
palpation
Typical symptoms are dysphagia, regurgitation, aspiration and chronic cough

Cystic hygroma A congenital lymphatic lesion (lymphangioma) typically found in the neck,
classically on the left side
Most are evident at birth, around 90% present before 2 years of age

Branchial cyst An oval, mobile cystic mass that develops between the sternocleidomastoid
muscle and the pharynx
Develop due to failure of obliteration of the second branchial cleft in
embryonic development
Usually present in early adulthood

Cervical rib More common in adult females


Around 10% develop thoracic outlet syndrome

Carotid aneurysm Pulsatile lateral neck mass which doesn't move on swallowing

Next question
Question 128 of 347

A 22 year old man presents with a discharging area on his lower back. On examination there is an
epithelial defect located 6cm proximal to the tip of his coccyx and located in the midline. There are
two further defects located about 2cm superiorly in the same position. He is extremely hirsute. What
is the most likely diagnosis?

Pre sacral tumour

Sacrococcygeal teratoma

Pilonidal sinus

Fistula in ano

Occult spina bifida

Pilonidal sinuses are extremely common in hirsute individuals and typically present as midline
sinuses in the natal cleft.
Please rate this question:

Discuss and give feedback


Next question

Pilonidal sinus

 Occur as a result of hair debris creating sinuses in the skin (Bascom theory).
 Usually in the natal cleft of male patients after puberty.
 It is more common in Caucasians related to their hair type and growth patterns.
 The opening of the sinus is lined by squamous epithelium, but most of its wall consists of
granulation tissue. Up to 50 cases of squamous cell carcinoma have been described in
patients with chronic pilonidal sinus disease.
 Hairs become trapped within the sinus.
 Clinically the sinus presents when acute inflammation occurs, leading to an abscess.
Patients may describe cycles of being asymptomatic and periods of pain and discharge from
the sinus.
 Treatment is difficult and opinions differ. Definitive treatment should never be undertaken
when acute infection or abscess is present as this will result in failure.
 Definitive treatments include the Bascom procedure with excision of the pits and obliteration
of the underlying cavity. The Karydakis procedure involves wide excision of the natal cleft
such that the surface is recontoured once the wound is closed. This avoids the shearing
forces that break off the hairs and has reasonable results.

Pilonidal sinuses are most commonly located in the midline of the natal cleft, as illustrated below

Image sourced from Wikipedia

Next question
Question 129 of 347

A 43 year old man from Greece presents with colicky right upper quadrant pain, jaundice and an
urticarial rash. He is initially treated with ciprofloxacin, but does not improve. What is the most likely
diagnosis?

Infection with Wucheria bancrofti

Infection with Echinococcus granulosus

Type III hypersensitivity reaction

Allergy to ciprofloxacin

Common bile duct stones

Infection with Echinococcus granulosus will typically produce a type I hypersensitivity reaction which
is characterised by an urticarial rash. With biliary rupture a classical triad of biliary colic, jaundice and
urticaria occurs. Whilst jaundice and biliary colic may be a feature of CBD stones they do not
produce an urticarial rash. Antibiotic sensitivity with ciprofloxacin may produce jaundice and a rash,
however it was not present at the outset and does not cause biliary colic.
Please rate this question:

Discuss and give feedback


Next question

Hydatid cysts

Hydatid cysts are endemic in Mediterranean and Middle Eastern countries. They are caused by the
tapeworm parasite Echinococcus granulosus. An outer fibrous capsule is formed containing multiple
small daughter cysts. These cysts are allergens which precipitate a type 1 hypersensitivity
reaction.

Clinical features are as follows:

 Up to 90% cysts occur in the liver and lungs


 Can be asymtomatic, or symptomatic if cysts > 5cm in diameter
 Morbidity caused by cyst bursting, infection and organ dysfunction (biliary, bronchial, renal
and cerebrospinal fluid outflow obstruction)
 In biliary ruputure there may be the classical triad of; biliary colic, jaundice, and urticaria
CT is the best investigation to differentiate hydatid cysts from amoebic and pyogenic cysts.
Surgery is the mainstay of treatment (the cyst walls must not be ruptured during removal and the
contents sterilised first).
Next question
Question 130 of 347

A 22 year old lady presents with an episode of renal colic and following investigation is suspected of
suffering from MEN IIa. Which of the following abnormalities of the parathyroid glands are most often
found in this condition?

Hypertrophy

Hyperplasia

Adenoma

Carcinoma

Metaplasia

MEN IIa

 Medullary thyroid cancer


 Hyperparathyroidism (usually hyperplasia)
 Phaeochromocytoma

In MEN IIa the commonest lesion is medullary thyroid cancer, with regards to the parathyroid glands
the most common lesion is hyperplasia. In MEN I a parathyroid adenoma is the most common
lesion.
Please rate this question:

Discuss and give feedback


Next question

Multiple Endocrine Neoplasia

Multiple endocrine neoplasia (MEN) is inherited as an autosomal dominant disorder.

The table below summarises the three main types of MEN:

MEN type I MEN type IIa MEN type IIb


Mnemonic 'three P's': Phaeochromocytoma Same as MEN IIa
Medullary thyroid cancer with addition of:
Parathyroid (95%): Parathyroid adenoma (70%) Marfanoid body
Pituitary (70%): Prolactinoma/ACTH/Growth Hyperparathyroidism habitus
Hormone secreting adenoma (60%) Mucosal neuromas
Pancreas (50%): Islet cell tumours/Zollinger
Ellison syndrome

also: Adrenal (adenoma) and thyroid


(adenoma)

MENIN gene (chromosome 11) RET oncogene RET oncogene


(chromosome 10) (chromosome 10)
Most common presentation = hypercalcaemia

Next question
Question 131 of 347

A male infant is born prematurely at 34 weeks gestation by emergency cesarean section. He initially
appears to be stable. However, over the ensuing 24 hours he develops worsening neurological
function. Which of the following processes is most likely to have occurred?

Extra dural haemorrhage

Sub dural haemorrhage

Sub arachnoid haemorrhage

Intraventricular haemorrhage

Arteriovenous malformation

Premature neonates= Intra ventricular haemorrhage


Non accidental injury= Sub dural bleed

Theme from April 2012 Exam

Please rate this question:

Discuss and give feedback

Next question

Intraventricular haemorrhage

Intraventricular haemorrhage is a haemorrhage that occurs into the ventricular system of the brain. It
is relatively rare in adult surgical practice and when it does occur, it is typically associated with
severe head injuries. In premature neonates it may occur spontaneously. The blood may clot and
occlude CSF flow, hydrocephalus may result.
In neonatal practice the vast majority of IVH occur in the first 72 hours after birth, the aetiology is not
well understood and it is suggested to occur as a result of birth trauma combined with cellular
hypoxia, together the with the delicate neonatal CNS.

Treatment
Is largely supportive, therapies such as intraventricular thrombolysis and prophylactic CSF drainage
have been trialled and not demonstrated to show benefit. Hydrocephalus and rising ICP is an
indication for shunting.

Next question
Question 132 of 347

A 22 year old man is admitted to hospital with a lower respiratory chest infection. He had a
splenectomy after being involved in a car accident. What is the most likely infective organism?

Haemophilus influenzae

Staphylococcus aureus

Rhinovirus

Mycobacterium tuberculosis

Moraxella catarrhalis

Organisms causing post splenectomy sepsis:


Streptococcus pneumoniae
Haemophilus influenzae
Meningococci

Encapsulated organisms carry the greatest pathogenic risk following splenectomy. The effects of
sepsis following splenectomy are variable. This may be the result of small isolated fragments of
splenic tissue that retain some function following splenectomy. These may implant spontaneously
following splenic rupture (in trauma) or be surgically implanted at the time of splenectomy.
Please rate this question:

Discuss and give feedback


Next question

Post splenectomy sepsis

Hyposplenism may complicate certain medical conditions where splenic atrophy occurs or may be
the result of medical intervention such as splenic artery embolization and splenectomy for trauma.
Diagnosis of hyposplenism is difficult and whilst there may be peripheral markers of the
splenectomised state (e.g. Howell Jolly bodies) these are neither 100% sensitive or specific. The
most sensitive test is a radionucleotide labeled red cell scan.
Hyposplenism, by whatever mechanism it occurs dramatically increases the risk of post splenectomy
sepsis, particularly with encapsulated organisms. Since these organisms may be opsonised, but this
then goes undetected at an immunological level due to loss of the spleen. For this reason individuals
are recommended to be vaccinated and have antibiotic prophylaxis.
Key recommendations

 All those with hyposplenism or may become so (such as prior to an elective splenectomy)
should receive pneumococcal, haemophilus type b and meningococcal type C vaccines.
These should be administered 2 weeks prior to splenectomy or two weeks following
splenectomy. The vaccine schedule for meningococcal disease essentially consists of a
dose of Men C and Hib at 2 weeks and then a dose of the MenACWY vaccine one month
later. Those aged under 2 may require a booster at 2 years. A dose of pneumococcal
polyvalent polysaccharide vaccine (PPV) is given at two weeks. A conjugated vaccine (PCV)
is offered to young children. The PCV is more immunogenic but covers fewer serotypes.
Boosting PPV is either guided by serological measurements (where available) or by routine
boosting doses at 5 yearly intervals.
 Annual influenza vaccination is recommended in all cases
 Antibiotic prophylaxis is offered to all. The risk of post splenectomy sepsis is greatest
immediately following splenectomy and in those aged less than 16 years or greater than 50
years. Individuals with a poor response to pneumococcal vaccination are another high risk
group. High risk individuals should be counselled to take penicillin or macrolide prophylaxis.
Those at low risk may choose to discontinue therapy. All patients should be advised about
taking antibiotics early in the case of intercurrent infections.
 Asplenic individuals traveling to malaria endemic areas are at high risk and should have both
pharmacological and mechanical protection.

Dosing
Penicillin V 500mg BD or amoxicillin 250mg BD

References
Davies J et al. Review of guidelines for the prevention and treatment of infection in patients with an
absent or dysfunctional spleen: Prepared on behalf of the British Committee for Standards in
Haematology by a Working Party of the Haemato-Oncology Task Force. British Journal of
Haematology2011 (155): 308317.
Next question
Question 133 of 347

A 24 year old man presents with symptoms of malaise, weight loss and lymphadenopathy. A lymph
node biopsy is performed and the subsequent histology report states that there is evidence of
granuloma formation and central necrosis. What is the most likely underlying cause?

Non Hodgkins lymphoma

Churg Strauss syndrome

Epstein Barr Virus infection

Rheumatoid nodule

Infection with Mycobacterium tuberculosis

These histological features are typically seen in TB. Necrosis occurring in granulomas is usually
indicative of an underlying infective cause. Churg Strauss syndrome is a form of vasculitis, which is
the usual histological finding. Granulomas are reported in the condition, but it is rare for them to
demonstrate necrosis.
Please rate this question:

Discuss and give feedback


Next question

Tuberculosis pathology

 Is a form of primary chronic inflammation, caused by the inability of macrophages to kill


theMycobacterium tuberculosis.
 The macrophages often migrate to regional lymph nodes, the lung lesion plus affected lymph
nodes is referred to as a Ghon complex.
 This leads to the formation of a granuloma which is a collection of epithelioid histiocytes.
 There is the presence of caseous necrosis in the centre.
 The inflammatory response is mediated by a type 4 hypersensitivity reaction.
 In healthy individuals the disease may be contained, in the immunocompromised
disseminated (miliary TB) may occur.

Diagnosis
 Waxy membrane of mycobacteria prevents binding with normal stains. Ziehl - Neelsen
staining is typically used.
 Culture based methods take far longer.

Image showing acid- alcohol fast mycobacteria stained using the Ziehl- Neelsen method

Image sourced from Wikipedia

Next question
Question 134 of 347

A 20 year old man develops acute appendicitis, his appendix is removed and he makes a full
recovery. Which of the following pathological processes is least likely to be present in the acutely
inflamed tissues?

Altered Starlings forces.

Seqestration of neutrophils

Formation of fluid exudate

Formation of granulomas

None of the above

Neutrophil polymorphs=Acute inflammation.


Granuloma = Chronic inflammation.

Acute inflammation:
3 phases
1. Changes in blood vessel and flow: flush, flare, wheal
2. Fluid exudates (rich in protein i.e. Ig, coagulation factors) produced via increased vascular
permeability
3. Cellular exudates mainly containing neutrophil polymorphs pass into extravascular space.

Neutrophils are then transported to tissues via:

a. Margination of neutrophils to the peripheral plasmatic of the vessel rather than the central axial
stream
b. Pavementing: Adhesion of neutrophils to endothelial cells in venules at site of acute inflammation
c. Emigration: neutrophils pass between endothelial cells into the tissue
Please rate this question:

Discuss and give feedback


Next question

Acute inflammation

Inflammation is the reaction of the tissue elements to injury. Vascular changes occur, resulting in the
generation of a protein rich exudate. So long as the injury does not totally destroy the existing tissue
architecture, the episode may resolve with restoration of original tissue architecture.
Vascular changes

 Vasodilation occurs and persists throughout the inflammatory phase.


 Inflammatory cells exit the circulation at the site of injury.
 The equilibrium that balances Starlings forces within capillary beds is disrupted and a protein
rich exudate will form as the vessel walls also become more permeable to proteins.
 The high fibrinogen content of the fluid may form a fibrin clot. This has several important
immunomodulatory functions.

Sequelae
Resolution  Typically occurs with minimal initial injury
 Stimulus removed and normal tissue architecture results

Organisation  Delayed removal of exudate


 Tissues undergo organisation and usually fibrosis

Suppuration  Typically formation of an abscess or an empyema


 Sequestration of large quantities of dead neutrophils

Progression to chronic  Coupled inflammatory and reparative activities


inflammation  Usually occurs when initial infection or suppuration has
been inadequately managed

Causes

 Infections e.g. Viruses, exotoxins or endotoxins released by bacteria


 Chemical agents
 Physical agents e.g. Trauma
 Hypersensitivity reactions
 Tissue necrosis

Presence of neutrophil polymorphs is a histological diagnostic feature of acute inflammation


Next question
Question 135-137 of 347

Theme: Liver lesions

A. Haemangioma
B. Hepatocellular carcinoma
C. Hepatic metastasis
D. Polycystic liver disease
E. Simple liver cyst
F. Hyatid cyst
G. Amoebic abscess
H. Mesenchymal hamartoma

Please select the most likely liver lesion for the scenario given. Each option may be used once, more
than once or not at all.

135. A 42 year old lady has suffered from hepatitis C for many years and has also developed
cirrhosis. On routine follow up, an ultrasound has demonstrated a 2.5cm lesion in the
right lobe of the liver.

You answered Haemangioma

The correct answer is Hepatocellular carcinoma

In patients with cirrhosis the presence of a lesion >2cm is highly suggestive of


malignancy. The diagnosis is virtually confirmed if the AFP is >400ng/mL.

136. A 25 year old man from the far east presents with a fever and right upper quadrant pain.
As part of his investigations a CT scan shows an ill defined lesion in the right lobe of the
liver.

You answered Haemangioma

The correct answer is Amoebic abscess

Amoebic abscesses will tend to present in a similar fashion to other pyogenic liver
abscesses. They should be considered in any individual presenting from a region
whereEntamoeba histiolytica is endemic. Treatment with metronidazole usually produces
a marked clinical response.

137. A 42 year old lady presents with right upper quadrant pain and a sensation of abdominal
fullness. An ultrasound scan demonstrates a 6.5 cm hyperechoic lesion in the right lobe of
the liver. Serum AFP is normal.
Haemangioma

A large hyperechoic lesion in the presence of normal AFP is likely to be a haemangioma.


An HCC of equivalent size will almost always result in rise in AFP.

Please rate this question:

Discuss and give feedback


Next question

Benign liver lesions

Benign liver lesions


Haemangioma  Most common benign tumours of mesenchymal origin
 Incidence in autopsy series is 8%
 Cavernous haemangiomas may be enormous
 Clinically they are reddish purple hypervascular lesions
 Lesions are normally separated from normal liver by ring of fibrous
tissue
 On ultrasound they are typically hyperechoic

Liver cell  90% develop in women in their third to fifth decade


adenoma  Linked to use of oral contraceptive pill
 Lesions are usually solitary
 They are usually sharply demarcated from normal liver although they
usually lack a fibrous capsule
 On ultrasound the appearances are of mixed echoity and
heterogeneous texture. On CT most lesions are hypodense when
imaged prior to administration of IV contrast agents
 In patients with haemorrhage or symptoms removal of the adenoma
may be required

Mesenchymal Congential and benign, usually present in infants. May compress normal liver
hamartomas

Liver abscess  Biliary sepsis is a major predisposing factor


 Structures drained by the portal venous system form the second largest
source
 Common symptoms include fever, right upper quadrant pain. Jaundice
may be seen in 50%
 Ultrasound will usually show a fluid filled cavity, hyperechoic walls
may be seen in chronic abscesses

Amoebic abscess  Liver abscess is the most common extra intestinal manifestation of
amoebiasis
 Between 75 and 90% lesions occur in the right lobe
 Presenting complaints typically include fever and right upper quadrant
pain
 Ultrasonography will usually show a fluid filled structure with poorly
defined boundaries
 Aspiration yield sterile odourless fluid which has an anchovy paste
consistency
 Treatment is with metronidazole

Hyatid cysts  Seen in cases of Echinococcus infection


 Typically an intense fibrotic reaction occurs around sites of infection
 The cyst has no epithelial lining
 Cysts are commonly unilocular and may grow to 20cm in size. The
cyst wall is thick and has an external laminated hilar membrane and an
internal enucleated germinal layer
 Typically presents with malaise and right upper quadrant pain.
Secondary bacterial infection occurs in 10%.
 Liver function tests are usually abnormal and eosinophilia is present in
33% cases
 Ultrasound may show septa and hyatid sand or daughter cysts.
 Percutaneous aspiration is contra indicated
 Treatment is by sterilisation of the cyst with mebendazole and may be
followed by surgical resection. Hypertonic swabs are packed around
the cysts during surgery

Polycystic liver  Usually occurs in association with polycystic kidney disease


disease  Autosomal dominant disorder
 Symptoms may occur as a result of capsular stretch

Cystadenoma  Rare lesions with malignant potential


 Usually solitary multiloculated lesions
 Liver function tests usually normal
 Ultrasonography typically shows a large anechoic, fluid filled area
with irregular margins. Internal echos may result from septa
 Surgical resection is indicated in all cases

Next question
Question 138 of 347

Which of the following disorders is associated with massive splenomegaly?

Acute lymphoblastic leukaemia

Acute myeloblastic leukaemia

Acute myelomonocytic leukaemia

Acute monoblastic leukaemia

Chronic granulocytic leukaemia

Chronic leukaemia is more likely to be associated with splenomegaly than acute leukaemia.
Please rate this question:

Discuss and give feedback


Next question

Spleen

The spleen is located in the left upper quadrant of the abdomen and its size can vary depending
upon the amount of blood it contains. The typical adult spleen is 12.5cm long and 7.5cm wide. The
usual weight of the adult spleen is 150g.
The exact position of the spleen can vary with respiratory activity, posture and the state of
surrounding viscera. It usually lies obliquely with its long axis aligned to the 9th, 10th and 11th ribs. It
is separated from these ribs by both diaphragm and pleural cavity. The normal spleen is not
palpable.

The shape of the spleen is influenced by the state of the colon and stomach. Gastric distension will
cause the spleen to resemble the shape of an orange segment. Colonic distension will cause it to
become more tetrahedral.

The spleen is almost entirely covered by peritoneum, which adheres firmly to its capsule. Recesses
of the greater sac separate it from the stomach and kidney. It develops from the upper dorsal
mesogastrium, remaining connected to the posterior abdominal wall and stomach by two folds of
peritoneum; the lienorenal ligament and gastrosplenic ligament. The lienorenal ligament is derived
from peritoneum where the wall of the general peritoneum meets the omental bursa between the left
kidney and spleen; the splenic vessels lie in its layers. The gastrosplenic ligament also has two
layers, formed by the meeting of the walls of the greater sac and omental bursa between spleen and
stomach, the short gastric and left gastroepiploic branches of the splenic artery pass in its layers.
Laterally, the spleen is in contact with the phrenicocolic ligament.

Relations
Superiorly Diaphragm

Anteriorly Gastric impression

Posteriorly Kidney

Inferiorly Colon

Tail of pancreas and splenic vessels (splenic artery divides here, branches pass to the
Hilum
white pulp transporting plasma)

Contents
White Immune function. Contains central trabecular artery. The germinal centres are supplied
pulp by arterioles called penicilliary radicles.

Red pulp Filters abnormal red blood cells.

Function

 Filtration of abnormal blood cells and foreign bodies such as bacteria.


 Immunity: IgM. Production of properdin, and tuftsin which help target fungi and bacteria for
phagocytosis.
 Haematopoiesis: up to 5th month gestation or in haematological disorders.
 Pooling: storage of 40% platelets.
 Iron reutilisation
 Storage monocytes

Disorders of the spleen


Massive splenomegaly

 Myelofibrosis
 Chronic myeloid leukaemia
 Visceral leishmaniasis (kala-azar)
 Malaria
 Gaucher's syndrome

Other causes (as above plus)

 Portal hypertension e.g. secondary to cirrhosis


 Lymphoproliferative disease e.g. CLL, Hodgkin's
 Haemolytic anaemia
 Infection: hepatitis, glandular fever
 Infective endocarditis
 Sickle-cell*, thalassaemia
 Rheumatoid arthritis (Felty's syndrome)

*the majority of adult patients with sickle-cell will have an atrophied spleen due to repeated infarction
Next question
Question 139 of 347

Causes of primary chronic inflammation do not include which of the following?

Sarcoidosis

Tuberculosis

Ulcerative colitis

Hip prostheses

Chronic cholecystitis

Chronic cholecystitis is caused by recurrent episodes of acute inflammation.


Prosthetic implants may be the site of primary chronic inflammation. A common example clinically is
breast implants which may become encapsulated. The subsequent fibrosis then results in distortion
and may be painful.
Please rate this question:

Discuss and give feedback


Next question

Chronic inflammation

Overview
Chronic inflammation may occur secondary to acute inflammation.In most cases chronic
inflammation occurs as a primary process. These may be broadly viewed as being one of three main
processes:

 Persisting infection with certain organisms such as Mycobacterium tuberculosis which results
in delayed type hypersensitivity reactions and inflammation.
 Prolonged exposure to non-biodegradable substances such as silica or suture materials
which may induce an inflammatory response.
 Autoimmune conditions involving antibodies formed against host antigens.

Acute vs. Chronic inflammation


Acute inflammation Chronic inflammation

Changes to existing vascular structure and increased Angiogenesis predominates


permeability of endothelial cells

Infiltration of neutrophils Macrophages, plasma cells and


lymphocytes predominate

Process may resolve with: Healing by fibrosis is the main result

 Suppuration
 Complete resolution
 Abscess formation
 Progression to chronic inflammation
 Healing by fibrosis

Granulomatous inflammation
A granuloma consists of a microscopic aggregation of macrophages (with epithelial type
arrangement =epitheliod). Large giant cells may be found at the periphery of granulomas.

Mediators
Growth factors released by activated macrophages include agents such as interferon and fibroblast
growth factor (plus many more). Some of these such as interferons may have systemic features
resulting in systemic symptoms and signs, which may be present in individuals with long standing
chronic inflammation.

The finding of granulomas is pathognomonic of chronic inflammation, as illustrated in this biopsy


from a patient with colonic Crohns disease
Image sourced from Wikipedia

Next question
Question 140 of 347

A 30 year old man is trapped in a house fire and sustains 30% partial and full thickness burns to his
torso and limbs. Three days following admission he has a brisk haematemesis. Which of the
following is the most likely explanation for this event?

Dieulafoy lesion

Curlings ulcers

Mallory Weiss tear

Depletion of platelets

Depletion of clotting factors

Curlings ulcers typically occur secondary to thermal injuries and are caused by loss of GI protective
mechanisms. They are at greater risk of perforation than stress ulcers and may also haemorrhage.
Please rate this question:

Discuss and give feedback


Next question

Burns

Burns may be thermal, chemical or electrical. In the former category are burns which occur as a
result of heat. Chemical burns occur when the skin is exposed to an extremely caustic or alkaline
substance. Electrical burns occur following exposure to electrical current. The immediate
management includes removal of the burning source which usually includes irrigation of the burned
area. A detailed assessment then needs to be made of the extent of the burns and a number of
charts are available for recording this information. The degree of injury relates to the temperature
and duration of exposure. Most domestic burns are mainly scalds in young children.

Following the burn, there is a local response with progressive tissue loss and release of
inflammatory cytokines. Systemically, there are cardiovascular effects resulting from fluid loss and
sequestration of fluid into the third space. There is a marked catabolic response. Immunosupression
is common with large burns and bacterial translocation from the gut lumen is a recognised event.
Sepsis is a common cause of death following major burns.

Types of burn
Type of burn Skin layers Skin Blanching Management
affected appearance

Epidermal/Superficial Epidermis Red, moist Yes

Superficial partial Epidermis and part Pale, dry Yes Normally heals
thickness of papillary dermis with no
affected intervention

Deep partial thickness Epidermis, whole Mottled red No Needs surgical


papillary dermis colour intervention
affected (depending on site)

Full thickness Whole skin layer Dry, leathery No Burns centre


and subcutaneous hard wound
tissue affected

Depth of burn assessment

 Bleeding on needle prick


 Sensation
 Appearance
 Blanching to pressure

Percentage burn estimation


Lund Browder chart: most accurate even in children
Wallace rule of nines
Palmar surface: surface area palm = 0.8% burn

>15% body surface area burns in adults needs urgent burn fluid resuscitation

Transfer to burn centre if:

 Need burn shock resuscitation


 Face/hands/genitals affected
 Deep partial thickness or full thickness burns
 Significant electrical/chemical burns

Management
The initial aim is to stop the burning process and resuscitate the patient. Intravenous fluids will be
required for children with burns greater than 10% of total body surface area. Adults with burns
greater than 15% of total body surface area will also require IV fluids. The fluids are calculated using
the Parkland formula which is; volume of fluid= total body surface area of the burn % x weight (Kg)
x4. Half of the fluid is administered in the first 8 hours. A urinary catheter should be inserted.
Analgesia should be given. Complex burns, burns involving the hand perineum and face and burns
>10% in adults and >5% in children should be transferred to a burns unit.

Circumferential burns affecting a limb or severe torso burns impeding respiration may require
escharotomy to divide the burnt tissue.

Conservative management is appropriate for superficial burns and mixed superficial burns that will
heal in 2 weeks. More complex burns may require excision and skin grafting. Excision and primary
closure is not generally practised as there is a high risk of infection.

There is no evidence to support the use of anti microbial prophylaxis or topical antibiotics in burn
patients.

Escharotomies

 Indicated in circumferential full thickness burns to the torso or limbs.


 Careful division of the encasing band of burn tissue will potentially improve ventilation (if the
burn involves the torso), or relieve compartment syndrome and oedema (where a limb is
involved)

References
www.euroburn.org/e107files/downloads/guidelinesburncare.pdf

Barajas-Nava LA, López-Alcalde J, Roqué i Figuls M, Solà I, Bonfill Cosp X. Antibiotic prophylaxis
for preventing burn wound infection. Cochrane Database of Systematic Reviews 2013, Issue 6. Art.
No.: CD008738. DOI: 10.1002/14651858.CD008738.pub2.

Hettiaratchy S & Papini R. Initial management of a major burn: assessment and resuscitation. BMJ
2004;329:101-103
Next question
Question 141-143 of 347

Theme: Adrenal gland disorders

A. Nelsons syndrome
B. Conns syndrome
C. Cushings syndrome
D. Benign incidental adenoma
E. Malignant adrenal adenoma
F. Waterhouse- Friderichsen syndrome
G. Metastatic lesion
H. Walker - Warburg syndrome
I. Phaeochromocytoma

Please select the most appropriate adrenal disorder for the scenario given. Each disorder may be
selected once, more than once or not at all.

141. A 19 year old lady is admitted to ITU with severe meningococcal sepsis. She is on
maximal inotropic support and a CT scan of her chest and abdomen is performed. The
adrenal glands show evidence of diffuse haemorrhage.

You answered Nelsons syndrome

The correct answer is Waterhouse- Friderichsen syndrome

WaterhouseFriderichsen syndrome is defined as adrenal gland failure due to bleeding into


the adrenal glands. It is caused by severe bacterial infection (most commonly the
meningococcus Neisseria meningitidis).

The bacterial infection leads to massive hemorrhage into one or (usually) both adrenal
glands. It is characterised by overwhelming bacterial infection meningococcemia leading
to massive blood invasion, organ failure, coma, haemodynamic shock, disseminated
intravascular coagulation with widespread purpura, rapidly developing adrenocortical
insufficiency and death.

142. A 34 year old lady is admitted with recurrent episodes of non-specific abdominal pain.
On each admission all blood investigations are normal, as are her observations. On this
admission a CT scan was performed. This demonstrates a 1.5cm nodule in the right
adrenal gland. This is associated with a lipid rich core. Urinary VMA is within normal
limits. Other hormonal studies are normal.

You answered Nelsons syndrome

The correct answer is Benign incidental adenoma


This is typical for a benign adenoma. Benign adenomas often have a lipid rich core that is
readily identifiable on CT scanning. In addition the nodules are often well circumscribed.

143. A 38 year old man is noted to have a blood pressure of 175/110 on routine screening. On
examination there are no physical abnormalities of note. CT scanning shows a left sided
adrenal mass. Plasma metanephrines are elevated.

You answered Nelsons syndrome

The correct answer is Phaeochromocytoma

Hypertension in a young patient without any obvious cause should be investigated.


Urinary VMA and plasma metanephrines are typically elevated.

Please rate this question:

Discuss and give feedback


Next question

Phaeochromocytoma and adrenal lesions

Phaeochromocytoma
Neuroendocrine tumour of the chromaffin cells of the adrenal medulla. Hypertension and
hyperglycaemia are often found.

 10% of cases are bilateral.


 10% occur in children.
 11% are malignant (higher when tumour is located outside the adrenal).
 10% will not be hypertensive.

Familial cases are usually linked to the Multiple endocrine neoplasia syndromes (considered under
its own heading).

Most tumours are unilateral (often right sided) and smaller than 10cm.

Diagnosis
Urine analysis of vanillymandelic acid (VMA) is often used (false positives may occur e.g. in patients
eating vanilla ice cream!)

Blood testing for plasma metanephrine levels.

CT and MRI scanning are both used to localise the lesion.


Treatment
Patients require medical therapy first. An irreversible alpha adrenoreceptor blocker should be given,
although minority may prefer reversible blockade(1). Labetolol may be co-administered for cardiac
chronotropic control. Isolated beta blockade should not be considered as it will lead to unopposed
alpha activity.

These patients are often volume depleted and will often require moderate volumes of intra venous
normal saline perioperatively.

Once medically optimised the phaeochromocytoma should be removed. Most adrenalectomies can
now be performed using a laparoscopic approach(2). The adrenals are highly vascular structures
and removal can be complicated by catastrophic haemorrhage in the hands of the inexperienced.
This is particularly true of right sided resections where the IVC is perilously close. Should the IVC be
damaged a laparotomy will be necessary and the defect enclosed within a Satinsky style vascular
clamp and the defect closed with prolene sutures. Attempting to interfere with the IVC using any
instruments other than vascular clamps will result in vessel trauma and make a bad situation much
worse.

Incidental adrenal lesions


Adrenal lesions may be identified on CT scanning performed for other reasons(3). Factors
suggesting benign disease on CT include(4):

 Size less than 3cm


 Homogeneous texture
 Lipid rich tissue
 Thin wall to lesion

All patients with incidental lesions should be managed jointly with an endocrinologist and full work up
as described above. Patients with functioning lesions or those with adverse radiological features
(Particularly size >3cm) should proceed to surgery.

References
1. Weingarten TN, Cata JP, O'Hara JF, Prybilla DJ, Pike TL, Thompson GB, et al. Comparison of
two preoperative medical management strategies for laparoscopic resection of pheochromocytoma.
Urology. 2010 Aug;76(2):508 e6-11.

2. Nguyen PH, Keller JE, Novitsky YW, Heniford BT, Kercher KW. Laparoscopic approach to
adrenalectomy: review of perioperative outcomes in a single center. Am Surg. 2011 May;77(5):592-
6.

3. Ng VW, Ma RC, So WY, Choi KC, Kong AP, Cockram CS, et al. Evaluation of functional and
malignant adrenal incidentalomas. Arch Intern Med. 2010 Dec 13;170(22):2017-20.

4. Muth A, Hammarstedt L, Hellstrom M, Sigurjonsdottir HA, Almqvist E, Wangberg B. Cohort study


of patients with adrenal lesions discovered incidentally. Br J Surg. 2011 May 27.
Next question
Question 144 of 347

A 43 year old man presents with dyspepsia and undergoes an upper GI endoscopy. During the
procedure diffuse gastric and duodenal ulcers are identified. A Clo test confirms the presence
ofHelicobacter pylori infection. What is the most likely explanation for the ulcers?

Decreased gastric motility

Increased urease activity

Decreased release of mucous and bicarbonate

Decreased gastrin levels

Increased acid production

Theme from April 2011 Exam


H-Pylori has a number of pathological effects. In this question the main issue is by what mechanism
the organism is able to induce both gastric and duodenal ulceration. Without modestly elevated acid
levels, the duodenum would not undergo gastric metaplasia. H-Pylori cannot colonise duodenal
mucosa and therefore the development of ulcers at this site can only occur in those who have
undergone metaplastic transformation (mediated by increased acidity).

Please rate this question:

Discuss and give feedback

Next question

Helicobacter Pylori

Infection with Helicobacter Pylori is implicated in many cases of duodenal ulceration and up to 60%
of patients with gastric ulceration.
It is a gram negative, helical shaped rod with microaerophillic requirements. It has the ability to
produce a urease enzyme that will hydrolyse urea resulting in the production of ammonia. The effect
of ammonia on antral G cells is to cause release of gastrin via a negative feedback loop.
Once infection is established the organism releases enzymes that disrupt the gastric mucous layer.
Certain subtypes release cytotoxins cag A and vac A gene products. The organism incites a
classical chronic inflammatory process of the gastric epithelium. This accounts for the development
of gastric ulcers. The mildly increased acidity may induce a process of duodenal gastric metaplasia.
Whilst duodenal mucosa cannot be colonised by H-Pylori, mucosa that has undergone metaplastic
change to the gastric epithelial type may be colonised by H- Pylori with subsequent inflammation
and development of duodenitis and ulcers.
In patients who are colonised there is a 10-20% risk of peptic ulcer, 1-2% risk gastric cancer and
<1% risk MALT lymphoma.

Next question
Question 145 of 347

A 15 year old boy is admitted with colicky abdominal pain of 6 hours duration. On examination he
has a soft abdomen, on systemic examination he has brownish spots around his mouth, feet and
hands. His mother underwent surgery for intussusception, aged 12, and has similar lesions. What is
the most likely underlying diagnosis?

Li Fraumeni syndrome

Peutz-Jeghers syndrome

Addisons disease

McCune -Albright syndrome

Appendicitis

This is most likely to be Peutz-Jeghers syndrome. Addisons and McCune Albright syndrome may
produce similar skin changes but the intussusception resulting from polyps combined with the
autosomal inheritance pattern makes this the most likely diagnosis.
Please rate this question:

Discuss and give feedback


Next question

Peutz-Jeghers syndrome

Peutz-Jeghers syndrome is an autosomal dominant condition characterised by numerous benign


hamartomatous polyps in the gastrointestinal tract. It is also associated with pigmented freckles on
the lips, face, palms and soles. Around 50% of patients will have died from a gastrointestinal tract
cancer by the age of 60 years.

Genetics

 Autosomal dominant
 Responsible gene encodes serine threonine kinase LKB1 or STK11

Features
 Hamartomatous polyps in GI tract (mainly small bowel)
 Pigmented lesions on lips, oral mucosa, face, palms and soles
 Intestinal obstruction e.g. intussusception (which may lead to diagnosis)
 Gastrointestinal bleeding

Management

 Conservative unless complications develop

Next question
Question 146 of 347

What is the most likely electrolyte abnormality in a patient with diarrhoea and a soft mass felt on
digital rectal examination?

Hyperkalaemia

Hypokalaemia

Hyponatraemia

Hypernatraemia

Hypocalcaemia

Large villous adenomas of the rectum may have marked secretory activity and result in the
development of hypokalaemia as rectal secretions are rich in potassium.
Please rate this question:

Discuss and give feedback


Next question

Hypokalaemia

Potassium and hydrogen can be thought of as competitors. Hyperkalaemia tends to be associated


with acidosis because as potassium levels rise fewer hydrogen ions can enter the cells

Hypokalaemia with alkalosis

 Vomiting
 Diuretics
 Cushing's syndrome
 Conn's syndrome (primary hyperaldosteronism)

Hypokalaemia with acidosis

 Diarrhoea
 Renal tubular acidosis
 Acetazolamide
 Partially treated diabetic ketoacidosis

Next question
Question 147 of 347

Which of the following is not included in Multiple Endocrine Neoplasia Type 2b?

Phaeochromocytoma

Visceral ganglioneuromas

Thyroid medullary carcinoma

Zollinger Ellison syndrome

Marfanoid features

MEN IIB

 Medullary thyroid cancer


 Phaeochromocytoma
 Mucosal neuroma
 Marfanoid appearance

Please rate this question:

Discuss and give feedback


Next question

Multiple Endocrine Neoplasia

Multiple endocrine neoplasia (MEN) is inherited as an autosomal dominant disorder.

The table below summarises the three main types of MEN:

MEN type I MEN type IIa MEN type IIb

Mnemonic 'three P's': Phaeochromocytoma Same as MEN IIa


Medullary thyroid cancer with addition of:
Parathyroid (95%): Parathyroid adenoma (70%) Marfanoid body
Pituitary (70%): Prolactinoma/ACTH/Growth Hyperparathyroidism habitus
Hormone secreting adenoma (60%) Mucosal neuromas
Pancreas (50%): Islet cell tumours/Zollinger
Ellison syndrome

also: Adrenal (adenoma) and thyroid


(adenoma)

MENIN gene (chromosome 11) RET oncogene RET oncogene


(chromosome 10) (chromosome 10)
Most common presentation = hypercalcaemia

Next question
Question 148 of 347

A 25 year old male pedestrian is involved in a road traffic accident. He sustains multiple injuries and
is admitted to the intensive care unit, intubated and ventilated. Over the next week he develops adult
respiratory distress syndrome. What is the main reason for hypoxaemia in this condition?

Increased lung compliance

Reduced diffusion

Reduced surfactant

Reduced elastase

Left to right shunt

Theme from September 2014 Exam


The diffuse lung injury, which is associated with loss of surfactant and increased elastase release
from neutrophils, results in fluid accumulation. This leads to reduced diffusion, which is the main
reason for hypoxaemia.
Please rate this question:

Discuss and give feedback


Next question

Adult respiratory distress syndrome

Defined as an acute condition characterized by bilateral pulmonary infiltrates and severe hypoxemia
(PaO2/FiO2 ratio < 200) in the absence of evidence for cardiogenic pulmonary oedema (clinically or
pulmonary capillary wedge pressure of less than 18 mm Hg).
In is subdivided into two stages. Early stages consist of an exudative phase of injury with associated
oedema. The later stage is one of repair and consists of fibroproliferative changes. Subsequent
scarring may result in poor lung function.

Causes

 Sepsis
 Direct lung injury
 Trauma
 Acute pancreatitis
 Long bone fracture or multiple fractures (through fat embolism)
 Head injury (causes sympathetic nervous stimulation which leads to acute pulmonary
hypertension)

Clinical features

 Acute dyspnoea and hypoxaemia hours/days after event


 Multi organ failure
 Rising ventilatory pressures

Management

 Treat the underlying cause


 Antibiotics (if signs of sepsis)
 Negative fluid balance i.e. Diuretics
 Recruitment manoeuvres such as prone ventilation, use of positive end expiratory pressure
 Mechanical ventilation strategy using low tidal volumes, as conventional tidal volumes may
cause lung injury (only treatment found to improve survival rates)

Next question
Question 149 of 347

A 24 year old male was admitted with bloody diarrhea, cramping abdominal pain and weight loss.
Colonoscopy revealed a friable, diffusely red mucosa involving the rectum and sigmoid colon. The
mucosa was normal proximal to this. The disease progressed with time to involve most of the entire
colon, but not the ileum. Many years later, a colonic biopsy shows high grade epithelial dysplasia.
What is the most likely initial diagnosis?

Colonic tuberculosis

Collagenous colitis

Ulcerative colitis

Crohns disease

Ischaemic colitis

Theme from January 2015 Exam


Ulcerative colitis spreads in a progressive distal to proximal manner. Over time a dysplastic
transformation is recognised. Such endoscopic findings mandate a minimum of close endoscopic
surveillance and if they occur in association with a colonic mass then usually a
pancproctocolectomy.
Please rate this question:

Discuss and give feedback


Next question

Ulcerative colitis

Ulcerative colitis is a form of inflammatory bowel disease. Inflammation always starts at rectum,
does not spread beyond ileocaecal valve (although backwash ileitis may occur) and is continuous.
The peak incidence of ulcerative colitis is in people aged 15-25 years and in those aged 55-65
years. It is less common in smokers.

The initial presentation is usually following insidious and intermittent symptoms. Features include:

 bloody diarrhoea
 urgency
 tenesmus
 abdominal pain, particularly in the left lower quadrant
 extra-intestinal features (see below)

Questions regarding the 'extra-intestinal' features of inflammatory bowel disease are common. Extra-
intestinal features include sclerosing cholangitis, iritis and ankylosing spondylitis.

Common to both Crohn's disease Notes


(CD) and Ulcerative colitis (UC)

Related to Arthritis: pauciarticular, asymmetric Arthritis is the most common extra-


disease activity Erythema nodosum intestinal feature in both CD and UC
Episcleritis Episcleritis is more common in
Osteoporosis Crohns disease

Unrelated to Arthritis: polyarticular, symmetric Primary sclerosing cholangitis is


disease activity Uveitis much more common in UC
Pyoderma gangrenosum Uveitis is more common in UC
Clubbing
Primary sclerosing cholangitis

Pathology

 Red, raw mucosa, bleeds easily


 No inflammation beyond submucosa (unless fulminant disease)
 Widespread superficial ulceration with preservation of adjacent mucosa which has the
appearance of polyps ('pseudopolyps')
 Inflammatory cell infiltrate in lamina propria
 Neutrophils migrate through the walls of glands to form crypt abscesses
 Depletion of goblet cells and mucin from gland epithelium
 Granulomas are infrequent

Barium enema

 Loss of haustrations
 Superficial ulceration, 'pseudopolyps'
 Long standing disease: colon is narrow and short -'drainpipe colon'

Endoscopy
 Superficial inflammation of the colonic and rectal mucosa
 Continuous disease from rectum proximally
 Superficial ulceration, mucosal islands, loss of vascular definition and continuous ulceration
pattern.

Management

 Patients with long term disease are at increased risk of development of malignancy
 Acute exacerbations are generally managed with steroids, in chronic patients agents such as
azathioprine and infliximab may be used
 Individuals with medically unresponsive disease usually require surgery- in the acute phase
a sub total colectomy and end ileostomy. In the longer term a proctectomy will be required.
An ileoanal pouch is an option for selected patients

References
Ford A et al. Ulcerative colitis. BMJ 2013 (346):29-34.
Next question
Question 150 of 347

Which virus is associated with Kaposi's sarcoma?

Human herpes virus 8

Human papillomavirus 16

Human T-lymphotropic virus 1

Epstein-Barr virus

Human papillomavirus 18

Please rate this question:

Discuss and give feedback


Next question

Oncoviruses

 Viruses which cause cancer


 These may be detected on blood test and prevented by vaccine

These are the main types of oncoviruses and their diseases:

Oncovirus Cancer

Epstein-Barr virus Burkitt's lymphoma


Hodgkin's lymphoma
Post transplant lymphoma
Nasopharyngeal carcinoma

Human papillomavirus 16/18 Cervical cancer


Anal cancer
Penile cancer
Vulval cancer
Oropharyneal cancer

Human herpes virus 8 Kaposi's sarcoma

Hepatitis B virus Hepatocellular carcinoma

Hepatitis C virus Hepatocellular carcinoma

Human T-lymphotropic virus 1 Tropical spastic paraparesis


Adult T cell leukaemia

Next question
Question 151 of 347

Which of the following is not a feature of Wallerian Degeneration?

May result from an axonotmesis

Typically occurs in the peripheral nervous systems

The axon remains excitable throughout the whole process

The distal neuronal stump is affected

Is a component of the healing process following neuronal injury

The axon loses its excitability once the process is established.


Please rate this question:

Discuss and give feedback


Next question

Wallerian degeneration

- Is the process that occurs when a nerve is cut or crushed.


- It occurs when the part of the axon separated from the neuron's cell nucleus degenerates.
- It usually begins 24 hours following neuronal injury and the distal axon remains excitable up until
this time.
- The degeneration of the axon is following by breakdown of the myelin sheath, a process that
occurs by infiltration of the site with macrophages.
- Eventually regeneration of the nerve may occur although recovery will depend on the extent and
manner of injury
Next question
Question 152 of 347

A 45 year old woman complains of painful tingling in her fingers. The pain is relieved by hanging the
arm over the side of the bed. She has a positive Tinel's sign at the wrist. Which of the following is
most likely to contribute to her diagnosis?

Methotrexate use

Crohn's disease

Hyperthyroidism

Tuberculosis

Rheumatoid arthritis

This woman has a diagnosis of carpal tunnel syndrome. Rheumatological disorders are a common
cause. Clinical examination should focus on identifying stigmata of rheumatoid arthritis, such as
rheumatoid nodules, vasculitic lesions and metacarpophalangeal joint arthritis.

Please rate this question:

Discuss and give feedback

Next question

Carpal tunnel syndome

Carpal tunnel syndrome is caused by compression of median nerve in the carpal tunnel

History

 pain/pins and needles in thumb, index, middle finger e.g. at night


 patient flicks hand to obtain relief

Examination

 weakness of thumb abduction


 wasting of thenar eminence (NOT hypothenar)
 Tinel's sign: tapping causes paraesthesia
 Phalen's sign: flexion of wrist causes symptoms

Causes of carpal tunnel syndrome

MEDIAN TRAP Mnemonic

 Myxoedema
 Edema premenstrually
 Diabetes
 Idiopathic
 Acromegaly
 Neoplasm
 Trauma
 Rheumatoid arthritis
 Amyloidosis
 Pregnancy

Management

Non surgical treatment Surgery

May resolve Complete division of the flexor retinaculum and decompression of the tunnel
spontaneously (successful in approximately 80% of patients)
Avoid precipitants and
reassurance
Night-time splints
Local steroid injections

Next question
Question 153 of 347

A 30 year old male presents with a painless swelling of the testis. Histologically the stroma has a
lymphocytic infiltrate. The most likely diagnosis is :

Differentiated teratoma

Malignant undifferentiated teratoma

Classical seminoma

Spermatocytic seminoma

Anaplastic seminoma

Seminoma is the commonest type of testicular tumour and is more common in males aged between
30-40 years. Classical seminoma is the commonest subtype and histology shows lymphocytic
stromal infiltrate. Other subtypes include:
1. Spermatocytic: tumour cells resemble spermatocytes. Excellent prognosis.
2. Anaplastic
3. Syncytiotrophoblast giant cells: β HCG present in cells
A teratoma is more common in males aged 20-30 years.
Please rate this question:

Discuss and give feedback


Next question

Testicular disorders

Testicular cancer
Testicular cancer is the most common malignancy in men aged 20-30 years. Around 95% of cases
of testicular cancer are germ-cell tumours. Germ cell tumours may essentially be divided into:

Tumour
Tumour type Key features markers Pathology

Seminoma  Commonest  AFP usually Sheet like lobular


subtype (50%) normal patterns of cells
Tumour
Tumour type Key features markers Pathology

 Average age at  HCG elevated with substantial


diagnosis = 40 in 10% fibrous
 Even advanced seminomas component.
disease associated  Lactate Fibrous septa
with 5 year dehydrogenase; contain
survival of 73% elevated in 10- lymphocytic
20% seminomas inclusions and
(but also in many granulomas may
other conditions) be seen.

Non seminomatous germ  Younger age at  AFP elevated Heterogenous


cell tumours (42%) presentation =20-30 in up to 70% of texture with
years cases occasional ectopic
 Advanced disease  HCG elevated tissue such as hair
 Teratoma carries worse prognosis in up to 40% of
 Yolk sac tumour (48% at 5 years) cases
 Choriocarcinoma  Retroperitoneal lymph  Other markers
 Mixed germ cell node dissection may be rarely helpful
tumours (10%) needed for residual
disease after
chemotherapy

Image demonstrating a classical seminoma, these tumours are typically more uniform than
teratomas
Image sourced from Wikipedia

Risk factors for testicular cancer

 Cryptorchidism
 Infertility
 Family history
 Klinefelter's syndrome
 Mumps orchitis

Features
 A painless lump is the most common presenting symptom
 Pain may also be present in a minority of men
 Other possible features include hydrocele, gynaecomastia

Diagnosis

 Ultrasound is first-line
 CT scanning of the chest/ abdomen and pelvis is used for staging
 Tumour markers (see above) should be measured

Management

 Orchidectomy (Inguinal approach)


 Chemotherapy and radiotherapy may be given depending on staging
 Abdominal lesions >1cm following chemotherapy may require retroperitoneal lymph node
dissection.

Prognosis is generally excellent

 5 year survival for seminomas is around 95% if Stage I


 5 year survival for teratomas is around 85% if Stage I

Benign disease

Epididymo-orchitis
Acute epididymitis is an acute inflammation of the epididymis, often involving the testis and usually
caused by bacterial infection.

 Infection spreads from the urethra or bladder. In men <35 years, gonorrhoea or chlamydia
are the usual infections.
 Amiodarone is a recognised non infective cause of epididymitis, which resolves on stopping
the drug.
 Tenderness is usually confined to the epididymis, which may facilitate differentiating it from
torsion where pain usually affects the entire testis.

Testicular torsion

 Twist of the spermatic cord resulting in testicular ischaemia and necrosis.


 Most common in males aged between 10 and 30 (peak incidence 13-15 years)
 Pain is usually severe and of sudden onset.
 Cremasteric reflex is lost and elevation of the testis does not ease the pain.
 Treatment is with surgical exploration. If a torted testis is identified then both testis should be
fixed as the condition of bell clapper testis is often bilateral.

Hydrocele

 Presents as a mass that transilluminates, usually possible to "get above" it on examination.


 In younger men it should be investigated with USS to exclude tumour.
 In children it may occur as a result of a patent processus vaginalis.
 Treatment in adults is with a Lords or Jabouley procedure.
 Treatment in children is with trans inguinal ligation of PPV.

Next question
Question 154 of 347

A 48 year old women presents with recurrent loin pain and fevers. Investigation reveals a staghorn
calculus of the left kidney. Infection with which of the following organisms is most likely?

Staphylococcus saprophyticus

Proteus mirabilis

Klebsiella

E-Coli

Staphylococcus epidermidis

Theme from April 2012 Exam


Infection with Proteus mirabilis accounts for 90% of all proteus infections. It has a urease producing
enzyme. This will tend to favor urinary alkalinisation which is a relative prerequisite for the formation
of staghorn calculi.

Please rate this question:

Discuss and give feedback

Next question

Renal stones

Type of Percentage of
stones Features all calculi
Type of Percentage of
stones Features all calculi

Calcium Hypercalciuria is a major risk factor (various causes) 85%


oxalate Hyperoxaluria may also increase risk
Hypocitraturia increases risk because citrate forms complexes with
calcium making it more soluble
Stones are radio-opaque (though less than calcium phosphate stones)
Hyperuricosuria may cause uric acid stones to which calcium oxalate
binds

Cystine Inherited recessive disorder of transmembrane cystine transport 1%


leading to decreased absorption of cystine from intestine and renal
tubule
Multiple stones may form
Relatively radiodense because they contain sulphur

Uric acid Uric acid is a product of purine metabolism 5-10%


May precipitate when urinary pH low
May be caused by diseases with extensive tissue breakdown e.g.
malignancy
More common in children with inborn errors of metabolism
Radiolucent

Calcium May occur in renal tubular acidosis, high urinary pH increases 10%
phosphate supersaturation of urine with calcium and phosphate
Renal tubular acidosis types 1 and 3 increase risk of stone formation
(types 2 and 4 do not)
Radio-opaque stones (composition similar to bone)

Struvite Stones formed from magnesium, ammonium and phosphate 2-20%


Occur as a result of urease producing bacteria (and are thus
associated with chronic infections)
Under the alkaline conditions produced, the crystals can precipitate
Slightly radio-opaque
Effect of urinary pH on stone formation
Urine pH will show individual variation (from pH 5-7). Post prandially the pH falls as purine
metabolism will produce uric acid. Then the urine becomes more alkaline (alkaline tide). When the
stone is not available for analysis the pH of urine may help to determine which stone was present.

Stone type Urine acidity Mean urine pH

Calcium phosphate Normal- alkaline >5.5

Calcium oxalate Variable 6

Uric acid Acid 5.5

Struvate Alkaline >7.2

Cystine Normal 6.5

Next question
Question 155 of 347

Causes of granulomatous disease do not include:

Amiodarone

Allopurinol

Sulphonamides

Beryllium

Wegener's granulomatosis

Allopurinol and sulphonamides cause hepatic granulomas.


Please rate this question:

Discuss and give feedback


Next question

Chronic inflammation

Overview
Chronic inflammation may occur secondary to acute inflammation.In most cases chronic
inflammation occurs as a primary process. These may be broadly viewed as being one of three main
processes:

 Persisting infection with certain organisms such as Mycobacterium tuberculosis which results
in delayed type hypersensitivity reactions and inflammation.
 Prolonged exposure to non-biodegradable substances such as silica or suture materials
which may induce an inflammatory response.
 Autoimmune conditions involving antibodies formed against host antigens.

Acute vs. Chronic inflammation


Acute inflammation Chronic inflammation
Changes to existing vascular structure and increased Angiogenesis predominates
permeability of endothelial cells

Infiltration of neutrophils Macrophages, plasma cells and


lymphocytes predominate

Process may resolve with: Healing by fibrosis is the main result

 Suppuration
 Complete resolution
 Abscess formation
 Progression to chronic inflammation
 Healing by fibrosis

Granulomatous inflammation
A granuloma consists of a microscopic aggregation of macrophages (with epithelial type
arrangement =epitheliod). Large giant cells may be found at the periphery of granulomas.

Mediators
Growth factors released by activated macrophages include agents such as interferon and fibroblast
growth factor (plus many more). Some of these such as interferons may have systemic features
resulting in systemic symptoms and signs, which may be present in individuals with long standing
chronic inflammation.

The finding of granulomas is pathognomonic of chronic inflammation, as illustrated in this biopsy


from a patient with colonic Crohns disease

Image sourced from Wikipedia


Question 156-158 of 347

Theme: Facial nerve palsy

A. Adenoid cystic carcinoma

B. Cerebrovascular accident

C. Petrous temporal fracture

D. Warthins tumour

E. Sarcoidosis

F. Pleomorphic adenoma

G. Cholesteatoma

Please select the most likely cause of facial nerve palsy for the scenario given. Each option may be
used once, more than once or not at all.

156. A 22 year old man presents with symptoms of lethargy and bilateral facial nerve palsy. On
examination he has bilateral parotid gland enlargement.

You answered Adenoid cystic carcinoma

The correct answer is Sarcoidosis

Theme from 2011 Exam

Facial nerve palsy is the commonest neurological manifestation of sarcoid. It usually resolves.
The absence of ear discharge or discrete lesion on palpation is against the other causes.

157. A 21 year old man presents with a unilateral facial nerve palsy after being hit in the head. On
examination he has a right sided facial nerve palsy and a watery discharge from his nose.

You answered Adenoid cystic carcinoma


The correct answer is Petrous temporal fracture

Nasal discharge of clear fluid and recent head injury makes a basal skull fracture the most likely
underlying diagnosis.

158. A 43 year old lady presents with symptoms of chronic ear discharge and a right sided facial nerve
palsy. On examination she has foul smelling fluid draining from her right ear and a complete
right sided facial nerve palsy.

You answered Adenoid cystic carcinoma

The correct answer is Cholesteatoma

Foul smelling ear discharge and facial nerve weakness is likely to be due to cholesteatoma. The
presence of a neurological deficit is a sinister feature.

Please rate this question:

Discuss and give feedback

Next question

Facial nerve palsy

Sarcoid Facial nerve palsy is the most frequent neurological manifestation of sarcoid
Affects right and left side with equal frequency, may be bilateral
Typically resolves in up to 80% of cases

Cholesteatoma Destructive and expanding growth of keratinised squamous epithelium


Patients often complain of chronic ear discharge
Infection with Pseudomonas may occur resulting in foul smell to discharge
Aquired lesions usually arise from the Pars flaccida region of the tympanic
membrane
Surgical removal and mastoidectomy may be needed
Recurrence rates of 20% may be seen following surgery

Basal skull History of head injury


fracture Presence of features such as Battles sign on examination
Clinical presence of CSF leak strongly supports diagnosis
Assessment is by CT and MRI scan
Prophylactic antibiotics should be given in cases of CSF leak

Next question
Question 159 of 347

A 59 year old lady is referred from the NHS breast screening program. A recent mammogram is
reported as showing linear, branching microcalcification with coarse granules. Which disease
process is the most likely underlying cause of these appearances?

Invasive lobular cancer

Lobular carcinoma in situ

Cribriform type ductal carcinoma in situ

Comedo type ductal carcinoma in situ

Fibroadenosis

Comedo type DCIS is usually associated with microcalcifications. Cribriform lesions are usually
multifocal but less likely to form microcalcifications. Lobular cancers and in situ lesions rarely form
microcalcifications and are difficult to detect using mammography.
Please rate this question:

Discuss and give feedback


Next question

Breast cancer - In situ disease

Breast cancer that has yet to invade the basement membrane is referred to as in situ disease. Both
ductal and lobular in situ variants are recognised.

Ductal carcinoma in situ

 Sub types include; comedo, cribriform, micropapillary and solid


 Comdeo DCIS is most likely to form microcalcifications
 Cribriform and micropapillary are most likely to be multifocal
 Most lesions are mixed (composed of multiple subtypes)
 High nuclear grade DCIS is associated with more malignant characteristics (loss of p53,
increased erbB2 expression)
 Local excision of low nuclear grade DCIS will usually produce satisfactory outcomes.
 Multifocal lesions, large and high nuclear grade lesions will usually require mastectomy
 Whole breast irradiation improves locoregional control when breast conserving surgery is
performed

Lobular carcinoma in situ

 Much rarer than DCIS


 Does not form microcalcifications
 Usually single growth pattern
 When an invasive component is found it is less likely to be associated with axillary nodal
metastasis than with DCIS
 Low grade LCIS is usually treated by monitoring rather than excision

Next question
Question 160 of 347

In patients with an annular pancreas where is the most likely site of obstruction?

The first part of the duodenum

The second part of the duodenum

The fourth part of the duodenum

The third part of the duodenum

The duodeno-jejunal flexure

Theme from September 2014 Exam


The pancreas develops from two foregut outgrowths (ventral and dorsal). During rotation the ventral
bud and adjacent gallbladder and bile duct lie together and fuse. When the pancreas fails to rotate
normally it can compress the duodenum with development of obstruction. Usually occurring as a
result of associated duodenal malformation. The second part of the duodenum is the commonest
site.
Please rate this question:

Discuss and give feedback


Next question

Pancreas

The pancreas is a retroperitoneal organ and lies posterior to the stomach. It may be accessed
surgically by dividing the peritoneal reflection that connects the greater omentum to the transverse
colon. The pancreatic head sits in the curvature of the duodenum. Its tail lies close to the hilum of
the spleen, a site of potential injury during splenectomy.

Relations
Posterior to the pancreas
Pancreatic head Inferior vena cava
Common bile duct
Right and left renal veins
Superior mesenteric vein and artery
Pancreatic neck Superior mesenteric vein, portal vein

Pancreatic body- Left renal vein


Crus of diaphragm
Psoas muscle
Adrenal gland
Kidney
Aorta

Pancreatic tail Left kidney

Anterior to the pancreas


Pancreatic head 1st part of the duodenum
Pylorus
Gastroduodenal artery
SMA and SMV(uncinate process)

Pancreatic body Stomach


Duodenojejunal flexure

Pancreatic tail Splenic hilum

Superior to the pancreas


Coeliac trunk and its branches common hepatic artery and splenic artery

Grooves of the head of the pancreas


2nd and 3rd part of the duodenum

Arterial supply

 Head: pancreaticoduodenal artery


 Rest: splenic artery

Venous drainage

 Head: superior mesenteric vein


 Body and tail: splenic vein
Ampulla of Vater

 Merge of pancreatic duct and common bile duct


 Is an important landmark, halfway along the second part of the duodenum, that marks the
anatomical transition from foregut to midgut (also the site of transition between regions
supplied by coeliac trunk and SMA).

Image sourced from Wikipedia

Next question
Question 161-163 of 347

Theme: Chest pain

A. Achalasia
B. Pulmonary embolus
C. Dissection of thoracic aorta
D. Boerhaaves syndrome
E. Gastro-oesophageal reflux
F. Carcinoma of the oesophagus
G. Oesophageal candidiasis

Please select the most likely cause for chest pain for the scenario given. Each option may be used
once, more than once or not at all.

161. A 43 year old man who has a long term history of alcohol misuse is admitted with a
history of an attack of vomiting after an episode of binge drinking. After vomiting he
developed sudden onset left sided chest pain, which is pleuritic in nature. On examination
he is profoundly septic and drowsy with severe epigastric tenderness and left sided chest
pain.

You answered Achalasia

The correct answer is Boerhaaves syndrome

In patients with Boerhaaves the rupture is often on the left side. The story here is typical.
All patients should have a contrast study to confirm the diagnosis and the affected site
prior to thoracotomy.

162. A 22 year old man is admitted with severe retrosternal chest pain and recurrent episodes
of dysphagia. These occur sporadically and often resolve spontaneously. On examination
there are no physical abnormalities and the patient seems well.

Achalasia

Achalasia may produce severe chest pain and many older patients may undergo cardiac
investigations prior to endoscopy.
Endoscopic injection with botulinum toxin is a popular treatment (although the benefit is
not long lasting). Cardiomyotomy together with an antireflux procedure is a more durable
alternative.

163. An obese 53 year old man presents with symptoms of recurrent retrosternal discomfort
and dyspepsia. This is typically worse at night after eating a large meal. On examination
there is no physical abnormality to find.

You answered Achalasia

The correct answer is Gastro-oesophageal reflux

Patients with GORD often have symptoms that are worse at night. In this age group an
Upper GI endoscopy should probably be performed.

Please rate this question:

Discuss and give feedback


Next question

Surgical chest pain

Dissection of  Tearing interscapular pain


thoracic aorta  Discrepancy in arterial blood pressures taken in both arms
 May show mediastinal widening on chest x-ray

Diffuse  Spectrum of oesophageal motility disorders


oesophageal  Caused by uncoordinated contractions of oesphageal muscles
spasm  May show "nutcracker oesophagus" on barium swallow
 Symptoms include dysphagia, retrosternal discomfort and dyspepsia

Gastro-  Common cause of retrosternal discomfort


oesphageal  Usually associated with symptoms of regurgitation, odynophagia and
reflux dyspepsia
 Symptoms usually well controlled with PPI therapy
 Risk factors include obesity, smoking and excess alcohol consumption

Boerhaaves  Spontaneous rupture of the oesophagus


syndrome  Caused by episodes of repeated vomiting often in association with
alcohol excess
 Typically there is an episode of repetitive vomiting followed by severe
chest and epigastric pain
 Diagnosis is by CT and contrast studies
 Treatment is surgical; during first 12 hours primary repair, beyond this
usually creation of controlled fistula with a T Tube, delay beyond 24
hours is associated with fulminent mediastinitis and is usually fatal.

Achalasia  Difficulty swallowing, dysphagia to both liquids and solids and


sometimes chest pain
 Usually caused by failure of distal oesphageal inhibitory neurones
 Diagnosis is by pH and manometry studies together with contrast
swallow and endoscopy
 Treatment is with either botulinum toxin, pneumatic dilatation or
cardiomyotomy

Next question
Question 164 of 347

A 32 year old woman presents with an episode of haemoptysis and is found to have metastatic
tumour present within the parenchyma of the lungs. This is biopsied and subsequent histology
shows clear cells. What is the most likely primary site?

Kidney

Breast

Liver

Adrenal

Bone

Clear cell tumours are a sub type of renal cell cancer it is associated with specific genetic changes
localised to chromosome 3.
Please rate this question:

Discuss and give feedback


Next question

Renal lesions

Lesion Disease specific features Treatment

Renal cell  Most present with haematuria Usually radical or partial


carcinoma (50%) nephrectomy
 Common renal tumour (85%
cases)
 Paraneoplastic features include
hypertension and polycythaemia
 Most commonly has
haematogenous mestastasis
Lesion Disease specific features Treatment

Nephroblastoma  Rare childhood tumour Surgical resection combined


 It accounts for 80% of all with chemotherapy (usually
genitourinary malignancies in vincristine, actinomycin D and
those under the age of 15 years doxorubicin)
 Up to 90% will have a mass
 50% will be hypertensive
 Diagnostic work up includes
ultrasound and CT scanning

Neuroblastoma  Most common extracranial Surgical resection, radiotherapy


tumour of childhood and chemotherapy
 80% occur in those under 4 years
of age
 Tumour of neural crest origin (up
to 50% occur in the adrenal
gland)
 The tumour is usually calcified
and may be diagnosed using
MIBG scanning
 Staging is with CT

Transitional cell  Accounts for 90% of Radical nephroureterectomy


carcinoma lower urinary tract tumours, but
only 10% of renal tumours
 Males affected 3x more than
females
 Occupational exposure to
industrial dyes and rubber
chemicals may increase risk
 Up to 80% present with painless
haematuria
 Diagnosis and staging is with CT
IVU

Angiomyolipoma  80% of these hamartoma type 50% of patients with lesions


lesions occur sporadically, the >4cm will have symptoms and
remainder are seen in those with will require surgical resection
tuberous sclerosis
 Tumour is composed of blood
vessels, smooth muscle and fat
Lesion Disease specific features Treatment

 Massive bleeding may occur in


10% of cases

Next question
Question 165 of 347

A laceration of the wrist produces a median nerve transection. The wound is clean and seen
immediately after injury. Collateral soft tissue damage is absent. The patient asks what the
prognosis is. You indicate that the nerve should regrow at approximately:

0.1 mm per day

1 mm per day

5 mm per day

1 cm per day

None of the above

Transaction of a peripheral nerve results in hemorrhage and retraction of the severed nerve ends.
Almost immediately, degeneration of the axon distal to the injury begins. Degeneration also occurs in
the proximal fragment back to the first node of Ranvier. Phagocytosis of the degenerated axonal
fragments leaves neurilemmal sheath with empty cylindrical spaces where the axons were. Several
days following the injury, axons from the proximal fragment begin to regrow. If they make contact
with the distal neurilemmal sheath, regrowth occurs at about the rate of 1 mm/day. However, if
associated trauma, fracture, infection, or separation of neurilemmal sheath ends precludes contact
between axons, growth is haphazard and a traumatic neuroma is formed. When neural transaction is
associated with widespread soft tissue damage and hemorrhage (with increased probability of
infection), many surgeons choose to delay reapproximation of the severed nerve end for 3 to 4
weeks.
Please rate this question:

Discuss and give feedback


Next question

Nerve injury

There are 3 types of nerve injury:


Neuropraxia  Nerve intact but electrical conduction is affected
 Full recovery
 Autonomic function preserved
 Wallerian degeneration does not occur
Axonotmesis  Axon is damaged and the myelin sheath is preserved. The connective tissue
framework is not affected.
 Wallerian degeneration occurs.

Neurotmesis  Disruption of the axon, myelin sheath and surrounding connective tissue.
 Wallerian degeneration occurs.

Wallerian Degeneration

 Axonal degeneration distal to the site of injury.


 Typically begins 24-36 hours following injury.
 Axons are excitable prior to degeneration occurring.
 Myelin sheath degenerates and is phagocytosed by tissue macrophages.

Nerve repair

 Neuronal repair may only occur physiologically where nerves are in direct contact. Where a
large defect is present, the process of nerve regeneration is hampered. It may not occur at
all or result in the formation of a neuroma. Where nerve regrowth occurs it is typically at a
rate of 1mm per day.

Next question
Question 166 of 347

Which of the following statements relating to gastric cancer is untrue?

It is associated with chronic helicobacter pylori infection

5% of gastric malignancies are due to lymphoma

In the Lauren classification the diffuse type of adenocarcinoma typically presents as a


large exophytic growth in the antrum

Smoking is a risk factor

It is associated with acanthosis nigricans

Barium meal appearances of linitis plastica:


Due to the increased rigidity of the wall, the stomach cannot be adequately distended, with only a
narrow lumen identified. The normal mucosal fold pattern is absent, either distorted, thickened or
nodular.

The Lauren classification describes a diffuse type of adenocarcinoma (Linitis plastica type lesion)
and an intestinal type. The diffuse type is often deeply infiltrative and may be difficult to detect on
endoscopy. Barium meal appearances can be characteristic.
Please rate this question:

Discuss and give feedback


Next question

Gastric cancer

Overview
There are 700,000 new cases of gastric cancer worldwide each year. It is most common in Japan
and less common in western countries. It is more common in men and incidence rises with
increasing age. The exact cause of many sporadic cancer is not known, however, familial cases do
occur in HNPCC families. In addition, smoking and smoked or preserved foods increase the risk.
Japanese migrants retain their increased risk (decreased in subsequent generations). The
distribution of the disease in western countries is changing towards a more proximal location
(perhaps due to rising obesity).

Pathology
There is some evidence of support a stepwise progression of the disease through intestinal
metaplasia progressing to atrophic gastritis and subsequent dysplasia, through to cancer. The
favoured staging system is TNM. The risk of lymph node involvement is related to size and depth of
invasion; early cancers confined to submucosa have a 20% incidence of lymph node metastasis.
Tumours of the gastro-oesophageal junction are classified as below:

Type True oesophageal cancers and may be associated with Barrett's oesophagus.
1

Type Carcinoma of the cardia, arising from cardiac type epithelium


2 or short segments with intestinal metaplasia at the oesophagogastric junction.

Type Sub cardial cancers that spread across the junction. Involve similar nodal stations to
3 gastric cancer.

Groups for close endoscopic monitoring

 Intestinal metaplasia of columnar type


 Atrophic gastritis
 Low to medium grade dysplasia
 Patients who have previously undergone resections for benign peptic ulcer disease (except
highly selective vagotomy).

Referral to endoscopy

Patients of any age with Patients without Worsening dyspepsia


dyspepsia and any of the dyspepsia
following

Chronic gastrointestinal bleeding Dysphagia Barretts oesophagus

Dysphagia Unexplained abdominal Intestinal metaplasia


pain or weight loss

Weight loss Vomiting Dysplasia

Iron deficiency anaemia Upper abdominal mass Atrophic gastritis


Upper abdominal mass Jaundice Patient aged over 55 years with
unexplained or persistent
dyspepsia

Upper GI endoscopy performed for dyspepsia. The addition of dye spraying (as shown in the bottom
right) may facilitate identification of smaller tumours

Image sourced from Wikipedia

Staging

 CT scanning of the chest abdomen and pelvis is the routine first line staging investigation in
most centres.
 Laparoscopy to identify occult peritoneal disease
 PET CT (particularly for junctional tumours)

Treatment

 Proximally sited disease greater than 5-10cm from the OG junction may be treated by sub
total gastrectomy
 Total gastrectomy if tumour is <5cm from OG junction
 For type 2 junctional tumours (extending into oesophagus) oesophagogastrectomy is usual
 Endoscopic sub mucosal resection may play a role in early gastric cancer confined to the
mucosa and perhaps the sub mucosa (this is debated)
 Lymphadenectomy should be performed. A D2 lymphadenectomy is widely advocated by the
Japanese, the survival advantages of extended lymphadenectomy have been debated.
However, the overall recommendation is that a D2 nodal dissection be undertaken.
 Most patients will receive chemotherapy either pre or post operatively.

Prognosis

UK Data

Disease extent Percentage 5 year survival

All RO resections 54%

Early gastric cancer 91%

Stage 1 87%

Stage 2 65%

Stage 3 18%

Operative procedure

Total Gastrectomy , lymphadenectomy and Roux en Y anastomosis

General anaesthesia
Prophylactic intravenous antibiotics
Incision: Rooftop.
Perform a thorough laparotomy to identify any occult disease.
Mobilise the left lobe of the liver off the diaphragm and place a large pack over it. Insert a large self
retaining retractor e.g. omnitract or Balfour (take time with this, the set up should be perfect). Pack
the small bowel away.
Begin by mobilising the omentum off the transverse colon.
Proceed to detach the short gastric vessels.
Mobilise the pylorus and divide it at least 2cm distally using a linear cutter stapling device.
Continue the dissection into the lesser sac taking the lesser omentum and left gastric artery flush at
its origin.
The lymph nodes should be removed en bloc with the specimen where possible.
Place 2 stay sutures either side of the distal oesophagus. Ask the anaesthetist to pull back on the
nasogastric tube. Divide the distal oesophagus and remove the stomach.
The oesphago jejunal anastomosis should be constructed. Identify the DJ flexure and bring a loop of
jejunum up to the oesophagus (to check it will reach). Divide the jejunum at this point. Bring the
divided jejunum either retrocolic or antecolic to the oesophagus. Anastamose the oesophagus to the
jejunum, using either interrupted 3/0 vicryl or a stapling device. Then create the remainder of the
Roux en Y reconstruction distally.
Place a jejunostomy feeding tube.
Wash out the abdomen and insert drains (usually the anastomosis and duodenal stump). Help the
anaesthetist insert the nasogastric tube (carefully!)
Close the abdomen and skin.
Enteral feeding may commence on the first post-operative day. However, most surgeons will leave
patients on free NG drainage for several days and keep them nil by mouth.
Next question
Question 167 of 347

Which of the following statements relating to Gardners syndrome variant of familial adenomatous
polyposis coli is false?

It is an autosomal dominant condition

Patients may develop retroperitoneal desmoid tumours

The vast majority of the polyps are benign and thus the risk of colorectal cancer is small

Patients are at increased risk of thyroid cancer

It is characterised by a mutation in the APC gene

The multiple polyps increase the risk of malignancy and most patients should undergo a colectomy.
Please rate this question:

Discuss and give feedback


Next question

Genetics and surgical disease

Some of the more commonly occurring genetic conditions occurring in surgical patients are
presented here.

Li-Fraumeni Syndrome

 Autosomal dominant
 Consists of germline mutations to p53 tumour suppressor gene
 High incidence of malignancies particularly sarcomas and leukaemias
 Diagnosed when:

*Individual develops sarcoma under 45 years


*First degree relative diagnosed with any cancer below age 45 years and another family member
develops malignancy under 45 years or sarcoma at any age

BRCA 1 and 2
 Carried on chromosome 17 (BRCA 1) and Chromosome 13 (BRCA 2)
 Linked to developing breast cancer (60%) risk.
 Associated risk of developing ovarian cancer (55% with BRCA 1 and 25% with BRCA 2).

Lynch Syndrome

 Autosomal dominant
 Develop colonic cancer and endometrial cancer at young age
 80% of affected individuals will get colonic and/ or endometrial cancer
 High risk individuals may be identified using the Amsterdam criteria

Amsterdam criteria
Three or more family members with a confirmed diagnosis of colorectal cancer, one of whom is a
first degree (parent, child, sibling) relative of the other two.
Two successive affected generations.
One or more colon cancers diagnosed under age 50 years.
Familial adenomatous polyposis (FAP) has been excluded.

Gardners syndrome

 Autosomal dominant familial colorectal polyposis


 Multiple colonic polyps
 Extra colonic diseases include: skull osteoma, thyroid cancer and epidermoid cysts
 Desmoid tumours are seen in 15%
 Mutation of APC gene located on chromosome 5
 Due to colonic polyps most patients will undergo colectomy to reduce risk of colorectal
cancer
 Now considered a variant of familial adenomatous polyposis coli

Next question
Question 168 of 347

A 60-year-old man presents with lower urinary tract symptoms and is offered a PSA test. Which one
of the following could interfere with the PSA level?

Vigorous exercise in the past 48 hours

Poorly controlled diabetes mellitus

Drinking more than 4 units of alcohol in the past 48 hours

Smoking

Recent cholecystectomy

Please rate this question:

Discuss and give feedback


Next question

PSA testing

Prostate specific antigen (PSA) is a serine protease enzyme produced by normal and malignant
prostate epithelial cells. It has become an important tumour marker but much controversy still exists
regarding its usefulness as a screening tool.

The NHS Prostate Cancer Risk Management Programme (PCRMP) has published updated
guidelines in 2009 on how to handle requests for PSA testing in asymptomatic men. A recent
European trial (ERSPC) showed a statistically significant reduction in the rate of death prostate
cancer by 20% in men aged 55 to 69 years but this was associated with a high risk of over-diagnosis
and over-treatment. Having reviewed this and other data the National Screening Committee have
decided not to introduce a prostate cancer screening programme yet but rather allow men to make
an informed choice.

Age-adjusted upper limits for PSA were recommended by the PCRMP*:

{Age} {PSA level (ng/ml)}


50-59 years 3.0

60-69 years 4.0

> 70 years 5.0

PSA levels may also be raised by**:

 benign prostatic hyperplasia (BPH)


 prostatitis and urinary tract infection (NICE recommend to postpone the PSA test for at least
1 month after treatment)
 ejaculation (ideally not in the previous 48 hours)
 vigorous exercise (ideally not in the previous 48 hours)
 urinary retention
 instrumentation of the urinary tract

Poor specificity and sensitivity

 around 33% of men with a PSA of 4-10 ng/ml will be found to have prostate cancer. With a
PSA of 10-20 ng/ml this rises to 60% of men
 around 20% with prostate cancer have a normal PSA
 various methods are used to try and add greater meaning to a PSA level including age-
adjusted upper limits and monitoring change in PSA level with time (PSA velocity or PSA
doubling time)

*aide memoire for upper PSA limit: (age - 20) / 10

**whether digital rectal examination actually causes a rise in PSA levels is a matter of debate
Next question
Question 169 of 347

Which of the symptoms below is least typical of pancreatic cancer?

Painless jaundice

Hyperamylasaemia

Hyperglycaemia

Weight loss

Classical Courvoisier syndrome

Raised serum amylase is relatively uncommon. The typical Courvoisier syndrome typically occurs in
20% and hyperglycaemia occurs in 15-20%.
Please rate this question:

Discuss and give feedback


Next question

Pancreatic cancer

 Adenocarcinoma
 Risk factors: Smoking, diabetes, adenoma, familial adenomatous polyposis
 Mainly occur in the head of the pancreas (70%)
 Spread locally and metastasizes to the liver
 Carcinoma of the pancreas should be differentiated from other periampullary tumours with
better prognosis

Clinical features

 Weight loss
 Painless jaundice
 Epigastric discomfort (pain usually due to invasion of the coeliac plexus is a late feature)
 Pancreatitis
 Trousseau's sign: migratory superficial thrombophlebitis
Investigations

 USS: May miss small lesions


 CT Scanning (pancreatic protocol). If unresectable on CT then no further staging needed
 PET/CT for those with operable disease on CT alone
 ERCP/ MRI for bile duct assessment
 Staging laparoscopy to exclude peritoneal disease

Management

 Head of pancreas: Whipple's resection (SE dumping and ulcers). Newer techniques include
pylorus preservation and SMA/ SMV resection
 Carcinoma body and tail: poor prognosis, distal pancreatectomy, if operable
 Usually adjuvent chemotherapy for resectable disease
 ERCP and stent for jaundice and palliation
 Surgical bypass may be needed for duodenal obstruction

Next question
Question 170 of 347

A 53 year old man presents with dyspepsia. An upper GI endoscopy is performed and Helicobacter
pylori is identified. A duodenal ulcer is present in the first part of the duodenum. Duodenal biopsies
are taken and demonstrate epithelium that resembles cells of the gastric antrum. Which of the
following is the most likely explanation for this process?

Hyperplasia of the crypts of Lieberkhun

Duodenal metaplasia

Duodenal dysplasia

Duodenal carcinoma

Hyptertrophy of Brunners glands

Metaplasia = cell type conversion

Theme in January 2012 exam

The process involved is metaplasia. During metaplasia there is no direct carcinogenesis, however
the persistent presence of precipitants of metaplasia will lead to malignant changes in cells.
Metaplastic changes in the duodenal cap are frequently seen in association with H-Pylori induced
ulcers. It typically resolves after ulcer healing and eradication therapy.
Please rate this question:

Discuss and give feedback


Next question

Metaplasia

 Definition: reversible change of differentiated cells to another cell type.


 May represent an adaptive substitution of cells that are sensitive to stress by cell types better
able to withstand the adverse environment.
 Can be a normal physiological response (ossification of cartilage to form bone)
 Most common epithelial metaplasia occurs with transformation of columnar cells to
squamous cells (smoking causes ciliated columnar cells to be replaced by squamous
epithelial cells; Schistosomiasis).
 Metaplasia from squamous to columnar cells occurs in Barrett oesophagus.
 If the metaplastic stimulus is removed, the cells will return to their original pattern of
differentiation. However, if the stimulus is not removed then progression to dysplasia may
occur.
 Not considered directly carcinogenic, however the factors which predispose to metaplasia, if
persistent may induce malignant transformation.
 The pathogenesis involves a reprogramming of stem cells that are known to exist in normal
tissues, or of undifferentiated mesenchymal cells present in connective tissue. In a
metaplastic change, these precursor cells differentiate along a new pathway.

Next question
Question 171 of 347

A 22 year old lady presents with symptoms and signs of hyperthyroidism. Her diagnostic work up
results in a diagnosis of Graves disease. Which of the following best describes the pathophysiology
of the condition?

Formation of IgG antibodies to the TSH receptors on the thyroid gland

Formation of IgG antibodies to the TRH receptors on the anterior pituitary

Formation of IgM antibodies to the TSH receptors on the thyroid gland

Formation of IgA antibodies to the TSH receptors on the thyroid gland

Formation of IgM antibodies to the TRH receptors on the anterior pituitary

Usually IgG antibodies are formed against the TSH receptors on the thyroid gland. Which is why the
TSH level is often very low in Graves disease.
Please rate this question:

Discuss and give feedback


Next question

Thyroid hormones

Hormones of the thyroid gland


Triiodothyronine T3 Major hormone active in target cells

Thyroxine T4 Most prevalent form in plasma, less biologically active than T3

Calcitonin Lowers plasma calcium

Synthesis and secretion of thyroid hormones

 Thyroid actively concentrates iodide to twenty five times the plasma concentration.
 Iodide is oxidised by peroxidase in the follicular cells to atomic iodine which then iodinates
tyrosine residues contained in thyroglobulin.
 Iodinated tyrosine residues in thyroglobulin undergo coupling to either T3 or T4.
 Process is stimulated by TSH, which stimulates secretion of thyroid hormones.
 The normal thyroid has approximately 3 month reserves of thyroid hormones.

LATS and Graves disease


In Graves disease patients develop IgG antibodies to the TSH receptors on the thyroid gland. This
results in chronic and long term stimulation of the gland with release of thyroid hormones. The
typically situation is raised thyroid hormones and low TSH. Thyroid receptor autoantibodies should
be checked in individuals presenting with hyperthyroidism as they are present in up to 85% cases.
Next question
Question 172 of 347

What is the most common cause of hypercalcaemia in the UK in hospitalised patients?

Thiazide use

Metastatic malignancy

Primary hyperparathyroidism

Osteogenic sarcoma

Sarcoidosis

Metastatic cancer accounts for most cases of hypercalcaemia in hospitalised patients. In the
community primary hyperparathyroidism is the commonest cause.
Please rate this question:

Discuss and give feedback


Next question

Hypercalcaemia

Main causes

 Malignancy (most common cause in hospital in-patients)


 Primary hyperparathyroidism (commonest cause in non hospitalised patients)

Less common

 Sarcoidosis (extrarenal synthesis of calcitriol )


 Thiazides, lithium
 Immobilisation
 Pagets disease
 Vitamin A/D toxicity
 Thyrotoxicosis
 MEN
 Milk alkali syndrome
Clinical features
Stones, bones, abdominal groans, and psychic moans
High serum calcium levels result in decreased neuronal excitability. Therefore sluggish reflexes,
muscle weakness and constipation may occur.
Next question
Question 173-175 of 347

Theme: Diseases affecting the great vessels

A. Aortic coarctation
B. Cervical rib
C. Takayasu's arteritis
D. Subclavian steal syndrome
E. Patent ductus arteriosus
F. Aortic dissection

Please select the most likely underlying cause for the symptoms described. Each option may be
used once, more than once or not at all.

173. A 24 year old lady from Western India presents with symptoms of lethargy and dizziness,
worse on turning her head. On examination her systolic blood pressure is 176/128. Her
pulses are impalpable at all peripheral sites. Auscultation of her chest reveals a systolic
heart murmur.

You answered Aortic coarctation

The correct answer is Takayasu's arteritis

Takayasu's arteritis most commonly affects young Asian females. Pulseless peripheries
are a classical finding. The CNS symptoms may be variable.

174. A 48 year old man notices that he is becoming increasingly dizzy when he plays squash,
in addition he has also developed cramping pain in his left arm. One day he is inflating
his car tyre with a hand held pump, he collapses and is brought to hospital.

You answered Aortic coarctation

The correct answer is Subclavian steal syndrome

Subclavian steal syndrome is associated with a stenosis or occlusion of the subclavian


artery, proximal to the origin of the vertebral artery. As a result the increased metabolic
needs of the arm then cause retrograde flow and symptoms of CNS vascular
insufficiency.

175. A 25 year old junior doctor has a chest x-ray performed as part of a routine insurance
medical examination. The x-ray shows evidence of rib notching. Auscultation of his chest
reveals a systolic murmur which is loudest at the posterior aspect of the fourth intercostal
space.
Aortic coarctation

Coarctation of the aorta may occur due to the remnant of the ductus arteriosus acting as a
fibrous constrictive band of the aorta. Weak arm pulses may be seen, radiofemoral delay
is the classical physical finding. Collateral flow through the intercostal vessels may
produce notching of the ribs, if the disease is long standing.

Please rate this question:

Discuss and give feedback


Next question

Vascular disease

Aortic dissection  Chest pain (anterior chest pain- ascending aorta, back pain -
descending aorta)
 Widening of aorta on chest x-ray
 Diagnosis made by CT scanning
 Treatment is either medical (Type B disease) or surgical (Type A
disease)

Cervical rib  Supernumery fibrous band arising from seventh cervical vertebra
 Incidence of 1 in 500
 May cause thoracic outlet syndrome
 Treatment involves surgical division of rib

Subclavian steal  Due to proximal stenotic lesion of the subclavian artery


syndrome  Results in retrograte flow through vertebral or internal thoracic
arteries
 The result is that decrease in cerebral blood flow may occur and
produce syncopal symptoms
 A duplex scan and/ or angiogram will delineate the lesion and allow
treatment to be planned

Takayasu's  Large vessel granulomatous vasculitis


arteritis  Results in intimal narrowing
 Most commonly affects young asian females
 Patients present with features of mild systemic illness, followed by
pulseless phase with symptoms of vascular insufficiency
 Treatment is with systemic steroids

Patent ductus  Ductus arteriosus is a normal foetal vessel that closes spontaneously
arteriosus after birth
 Results in high pressure, oxygenated blood entering the pulmonary
circuit
 Untreated patients develop symptoms of congestive cardiac failure

Coarctation of the  Aortic stenosis at the site of the ductus arteriosus insertion
aorta  More prevalent in boys or females with Turners syndrome
 Patients may present with symptoms of arterial insufficiency, such
as syncope and claudication
 Blood pressure mismatch may be seen, as may mismatch of pulse
pressure in the upper and lower limbs
 Treatment is either with angioplasty or surgical resection (the former
is the most common)

Next question
Question 176 of 347

A 25 year old man is found to have carcinoid syndrome. Which of the following hormones is released
by carcinoids?

Serotonin

Dopamine

Nor adrenaline

Adrenaline

Aldosterone

Rule of thirds:

1/3 multiple
1/3 small bowel
1/3 metastasize
1/3 second tumour

Carcinoids secrete serotonin. Carcinoid syndrome will only occur in the presence of liver metastasis
as the hormone released from primary lesions will typically be metabolised by the liver.
Please rate this question:

Discuss and give feedback


Next question

Carcinoid syndrome

 Carcinoid tumours secrete serotonin


 Originate in neuroendocrine cells mainly in the intestine (midgut-distal ileum/appendix)
 Can occur in the rectum, bronchi
 Hormonal symptoms mainly occur when disease spreads outside the bowel

Clinical features

 Onset: insidious over many years


 Flushing face
 Palpitations
 Pulmonary valve stenosis and tricuspid regurgitation causing dyspnoea
 Asthma
 Severe diarrhoea (secretory, persists despite fasting)

Investigation

 5-HIAA in a 24-hour urine collection


 Somatostatin receptor scintigraphy
 CT scan
 Blood testing for chromogranin A

Treatment

 Octreotide
 Surgical removal

Next question
Question 177 of 347

Which one of the following genes protects against neoplasms?

sis

p53

ras

myc

src

p53 is a tumour supressor gene and located on chromosome 17. It plays an important role in
causing cells that are undergoing neoplastic changes to enter an apoptotic pathway.
Please rate this question:

Discuss and give feedback


Next question

Oncogenes

Oncogenes are cancer promoting genes that are derived from normal genes (proto-oncogenes).
Proto-oncogenes play an important physiological role in cellular growth. They are implicated in the
development of up to 20% of human cancers.

Proto-oncogenes may become oncogenes via the following processes:

 Mutation (point mutation)


 Chromosomal translocation
 Increased protein expression

Only one mutated copy of the gene is needed for cancer to occur - a dominant effect

Classification of oncogenes

 Growth factors e.g. Sis


 Transcription factors e.g. Myc
 Receptor tyrosine kinase e.g. RET
 Cytoplasmic tyrosine kinase e.g. Src
 Regulatory GTPases e.g. Ras

Tumour suppressor genes


Tumour suppressor genes restrict or repress cellular proliferation in normal cells. Their inactivation
through mutation or germ line incorporation is implicated in renal, colonic, breast, bladder and many
other cancers. One of the best known tumour suppressor genes is p53. p53 gene offers protection
by causing apoptosis of damaged cells. Other well known genes include BRCA 1 and 2.
Next question
Question 178 of 347

A 55 year old man with a long history of achalasia is successfully treated by a Hellers
Cardiomyotomy. Several years later he develops an oesophageal malignancy. Which of the
following lesions is most likely to be present?

Adenocarcinoma

Gastrointestinal stromal tumour

Leiomyosarcoma

Rhabdomyosarcoma

Squamous cell carcinoma

Achalasia is a rare condition. However, even once treated there is an increased risk of malignancy.
When it does occur it is most likely to be of squamous cell type.
Please rate this question:

Discuss and give feedback


Next question

Oesophageal cancer

 Incidence is increasing
 In most cases in the Western world this increase is accounted for by a rise in the number of
cases of adenocarcinoma. In the UK adenocarcinomas account for 65% of cases.
 Barretts oesophagus is a major risk factor for most cases of oesophageal adenocarcinoma.
 In other regions of the world squamous cancer is more common and is linked to smoking,
alcohol intake, diets rich in nitrosamines and achalasia.
 Surveillance of Barretts is important, as it imparts a 30 fold increase in cancer risk and if
invasive malignancy is diagnosed early then survival may approach 85% at 5 years.

Diagnosis

 Upper GI endoscopy is the first line test


 Contrast swallow may be of benefit in classifying benign motility disorders but has no place
in the assessment of tumours
 Staging is initially undertaken with CT scanning of the chest, abdomen and pelvis. If overt
metastatic disease is identified using this modality then further complex imaging is
unnecessary
 If CT does not show metastatic disease, then local stage may be more accurately assessed
by use of endoscopic ultrasound.
 Staging laparoscopy is performed to detect occult peritoneal disease. PET CT is performed
in those with negative laparoscopy. Thoracoscopy is not routinely performed.

Treatment
Operable disease is best managed by surgical resection. The most standard procedure is an Ivor-
Lewis type oesophagectomy. This procedure involves the mobilisation of the stomach and division of
the oesophageal hiatus. The abdomen is closed and a right sided thoracotomy performed. The
stomach is brought into the chest and the oesophagus mobilised further. An intrathoracic
oesophagogastric anastomosis is constructed. Alternative surgical strategies include a transhiatal
resection (for distal lesions), a left thoraco-abdominal resection (difficult access due to thoracic
aorta) and a total oesophagectomy (McKeown) with a cervical oesophagogastric anastomosis.
The biggest surgical challenge is that of anastomotic leak, with an intrathoracic anastomosis this will
result in mediastinitis. With high mortality. The McKeown technique has an intrinsically lower
systemic insult in the event of anastomotic leakage.

In addition to surgical resection many patients will be treated with adjuvant chemotherapy.
Next question
Question 179 of 347

Which of the following genes is not implicated in the adenoma-carcinoma sequence in colorectal
cancer?

src

c-myc

APC

p53

K-ras

Other genes involved are:

MCC
DCC
c-yes
bcl-2
Please rate this question:

Discuss and give feedback


Next question

Colorectal cancer

 Annually about 150,000 new cases are diagnosed and 50,000 deaths from the disease
 About 75% will have sporadic disease and 25% will have a family history
 Colorectal tumours comprise a spectrum of disease ranging from adenomas through to polyp
cancers and frank malignancy.
 Polyps may be categorised into: neoplastic polyps, adenomatous polyps and non neoplastic
polyps.
 The majority of adenomas are polypoidal lesions, although flat lesions do occur and may
prove to be dysplastic.
 Non-neoplastic polyps include hyperplastic, juvenile, hamartomatous, inflammatory, and
lymphoid polyps, which have not generally been thought of as precursors of cancer.
 Three characteristics of adenomas that correlate with malignant potential have been
characterised. These include increased size, villous architecture and dysplasia. For this
reason most polyps identified at colonoscopy should be removed.
 The transformation from polyp to cancer is described by the adenoma - carcinoma sequence
and its principles should be appreciated. Essentially genetic changes accompany the
transition from adenoma to carcinoma; key changes include APC, c-myc, K RAS mutations
and p53 deletions.

Next question
Question 180 of 347

A 63 year old lady is suspected as having sarcoidosis. She is sent to the general surgeons and a
lymph node biopsy is performed. Which histological feature is most likely to be identified in a lymph
node if sarcoid is present?

Psammoma bodies

Extensive necrosis

Dense eosinophillic infiltrates

Asteroid bodies

None of the above

Asteroid bodies are often found in the granulomas of individuals with sarcoid. Unlike the
granulomata associated with tuberculosis the granulomas of sarcoid are rarely associated with
extensive necrosis.

An Asteroid body in an individual with sarcoid


Image sourced from Wikipedia

Please rate this question:

Discuss and give feedback


Next question

Chronic inflammation

Overview
Chronic inflammation may occur secondary to acute inflammation.In most cases chronic
inflammation occurs as a primary process. These may be broadly viewed as being one of three main
processes:

 Persisting infection with certain organisms such as Mycobacterium tuberculosis which results
in delayed type hypersensitivity reactions and inflammation.
 Prolonged exposure to non-biodegradable substances such as silica or suture materials
which may induce an inflammatory response.
 Autoimmune conditions involving antibodies formed against host antigens.
Acute vs. Chronic inflammation
Acute inflammation Chronic inflammation

Changes to existing vascular structure and increased Angiogenesis predominates


permeability of endothelial cells

Infiltration of neutrophils Macrophages, plasma cells and


lymphocytes predominate

Process may resolve with: Healing by fibrosis is the main result

 Suppuration
 Complete resolution
 Abscess formation
 Progression to chronic inflammation
 Healing by fibrosis

Granulomatous inflammation
A granuloma consists of a microscopic aggregation of macrophages (with epithelial type
arrangement =epitheliod). Large giant cells may be found at the periphery of granulomas.

Mediators
Growth factors released by activated macrophages include agents such as interferon and fibroblast
growth factor (plus many more). Some of these such as interferons may have systemic features
resulting in systemic symptoms and signs, which may be present in individuals with long standing
chronic inflammation.

The finding of granulomas is pathognomonic of chronic inflammation, as illustrated in this biopsy


from a patient with colonic Crohns disease
Image sourced from Wikipedia

Next question
Question 181 of 347

Brown tumours of bone are associated with which of the following?

Hyperthyroidism

Hypothyroidism

Hyperparathyroidism

Hypoparathyroidism

Osteopetrosis

Brown tumors are tumors of bone that arise in settings of excess osteoclast activity, such as
hyperparathyroidism, and consist of fibrous tissue, woven bone and supporting vasculature, but no
matrix. They are radiolucent on x-ray. The osteoclasts consume the trabecular bone that osteoblasts
lay down and this front of reparative bone deposition followed by additional resorption can expand
beyond the usual shape of the bone, involving the periosteum thus causing bone pain. They appear
brown because haemosiderin is deposited at the site.
Please rate this question:

Discuss and give feedback


Next question

Primary hyperparathyroidism

In exams, primary hyperparathyroidism is stereotypically seen in elderly females with an


unquenchable thirst and an inappropriately normal or raised parathyroid hormone level. It is most
commonly due to a solitary adenoma

Causes of primary hyperparathyroidism

 80%: solitary adenoma


 15%: hyperplasia
 4%: multiple adenoma
 1%: carcinoma
Features - 'bones, stones, abdominal groans and psychic moans'

 Polydipsia, polyuria
 Peptic ulceration/constipation/pancreatitis
 Bone pain/fracture
 Renal stones
 Depression
 Hypertension

Associations

 Hypertension
 Multiple endocrine neoplasia: MEN I and II

Investigations

 Raised calcium, low phosphate


 PTH may be raised or normal
 Technetium-MIBI subtraction scan

Treatment

 Parathyroidectomy, if imaging suggests target gland then a focused approach may be used

Next question
Question 182 of 347

A 73 year old man is recovering following an emergency Hartmans procedure performed for an
obstructing sigmoid cancer. The pathology report shows a moderately differentiated
adenocarcinoma that invades the muscularis propria, 3 of 15 lymph nodes are involved with
metastatic disease. What is the correct stage for this?

Astler Coller Stage B2

Dukes stage A

Dukes stage B

Dukes stage C

Dukes stage D

Remember that the term metastasis simply refers to spread and can include the lymph nodes. In an
examination setting marks can be lost by incorrectly selecting Dukes D (which would be consistent
with liver metastasis) rather than nodal metastasis (Dukes C).

Theme from September 2011 Exam

The involvement of lymph nodes makes this Dukes C. In the Astler Coller system the B and C
subsets are split to B1 and B2 and C1 and C2. Where C2 denotes involvement of the nodes in
conjunction with penetration of the muscularis propria.

Please rate this question:

Discuss and give feedback

Next question

Dukes classification
Gives the extent of spread of colorectal cancer

Dukes A Tumour confined to the mucosa (95%)

Dukes B Tumour invading bowel wall (80%)

Dukes C Lymph node metastases (60%)

Dukes D Distant metastases (6%)(25% if resectable)

5 year survival in brackets

Next question
Question 183 of 347

A 55 year old man presents with a soft, fluctuant lesion overlying his right scapula. The surgeon
suspects the lesion may be a lipoma. Which of the following, if present, may be indicative of an
alternative diagnosis?

Located in superficial tissues

Size greater than 5cm

Presence of multiple similar lesions at other anatomical sites

Increased mobility of the lesion

Lobulated appearance during surgical excision


Question 184 of 347

A 55 year old man with dyspepsia undergoes an upper GI endoscopy. An irregular erythematous
area is seen to protrude proximally from the gastro-oesophageal junction. Apart from specialised
intestinal metaplasia, which of the following cell types should also be present for a diagnosis of
Barretts oesophagus to be made?

Goblet cell

Neutrophil

Lymphocytes

Epithelial cells

Macrophages

Goblet cells need to be present for a diagnosis of Barrett's oesophagus to be made.


Please rate this question:

Discuss and give feedback


Next question

Barrett's oesophagus

Barretts oesophagus is a condition characterised by the metaplastic transformation of squamous


oesophageal epithelium to columnar gastric type epithelium. Three types of this metaplastic process
are recognised; intestinal (high risk), cardiac and fundic. The latter two categories may cause
difficulties in diagnosis. The most concrete diagnosis can be made when endoscopic features of
Barretts oesophagus are present together with a deep biopsy that demonstrates not just goblet cell
metaplasia but also oesophageal glands.

Barrett's can be sub divided into short (<3cm) and long (>3cm). The length of the affected segment
correlates strongly with the chances of identifying metaplasia. The overall prevalence of Barrett's
oesophagus is difficult to determine but may be in the region of 1 in 20 and is identified in up to 12%
of those undergoing endoscopy for reflux.

A proportion of patients with metaplasia will progress to dysplasia and for this reason individuals
identified as having Barrett's should undergo endoscopic surveillance (every 2-5 years). Biopsies
should be quadrantic and taken at 1-2cm intervals. Biopsies need to be adequate. Where mass
lesions are present consideration should be given to endoscopic sub mucosal resection. Up to 40%
of patients will be upstaged from high grade dysplasia to invasive malignancy with such techniques.

Treatment

 Long term proton pump inhibitor


 Consider pH and manometry studies in younger patients who may prefer to consider an anti
reflux procedure
 Regular endoscopic monitoring (more frequently if moderate dysplasia). With quadrantic
biopsies every 2-3 cm
 If severe dysplasia be very wary of small foci of cancer

References
A consensus statement of the British approach is provided by:
Bennett C et al Consensus Statements for Management of Barrett's Dysplasia and Early-Stage
Esophageal Adenocarcinoma, Based on a Delphi Process. Gastroenterology Volume 143, Issue 2 ,
Pages 336-346, August 2012.
Next question
Question 185 of 347

Which of the following amino acids is present in all types of collagen?

Alanine

Aspartime

Glycine

Tyrosine

Cysteine

Collagen has a generic structure of Glycine- X- Y, where X and Y are variable sub units. The
relatively small size of the glycine molecule enables collagen to form a tight helical structure.
Please rate this question:

Discuss and give feedback


Next question

Collagen

Collagen is one of the most important structural proteins within the extracellular matrix, collagen
together with components such as elastin and glycosaminoglycans determine the properties of all
tissues.

 Composed of 3 polypeptide strands that are woven into a helix, usually a combination of
glycine with either proline or hydroxyproline plus another amino acid
 Numerous hydrogen bonds exist within molecule to provide additional strength
 Many sub types but commonest sub type is I (90% of bodily collagen), tissues with increased
levels of flexibility have increased levels of type III collagen
 Vitamin c is important in establishing cross links
 Synthesised by fibroblasts

Collagen Diseases
Disorders of collagen range from relatively common, acquired defects (typically aging), through to
rarer congenital disorders. The latter are exemplified by conditions such as osteogenesis imperfecta
and Ehlers Danlos syndromes.
Osteogenesis imperfecta:
-8 Subtypes
-Defect of type I collagen
-In type I the collagen is normal quality but insufficient quantity
-Type II- poor quantity and quality
-Type III- Collagen poorly formed, normal quantity
-Type IV- Sufficient quantity but poor quality
Patients have bones which fracture easily, loose joint and multiple other defects depending upon
which sub type they suffer from.

Ehlers Danlos:
-Multiple sub types
-Abnormality of types 1 and 3 collagen
-Patients have features of hypermobility.
-Individuals are prone to joint dislocations and pelvic organ prolapse. In addition to many other
diseases related to connective tissue defects.
Next question
Question 186 of 347

An 8 year old boy presented with a painless swelling on the superotemporal aspect of his orbit. It
was smooth on examination, produced no visual disturbances. Following excision it was found to be
lined by squamous epithelium and hair follicles. Which of the following lesions most closely matches
these findings?

Dermoid cyst

Desmoid tumour

Lipoma

Sebaceous cyst

Schwannoma

Dermoid cysts are embryological remnants and may be lined by hair and squamous epithelium (like
teratomas). They are often located in the midline and may be linked to deeper structures resulting in
a dumbbell shape to the lesion. Complete excision is required as they have a propensity to local
recurrence if not excised.
Desmoid tumours are a different entity, they most commonly develop in ligaments and tendons.
They are also referred to as aggressive fibromatosis and consist of fibroblast dense lesions
(resembling scar tissue). They should be managed in a similar manner to soft tissue sarcomas.
Please rate this question:

Discuss and give feedback


Next question

Skin Diseases

Skin lesions may be referred for surgical assessment, but more commonly will come via a
dermatologist for definitive surgical management.

Skin malignancies include basal cell carcinoma, squamous cell carcinoma and malignant melanoma.

Basal Cell Carcinoma

 Most common form of skin cancer.


 Commonly occur on sun exposed sites apart from the ear.
 Sub types include nodular, morphoeic, superficial and pigmented.
 Typically slow growing with low metastatic potential.
 Standard surgical excision, topical chemotherapy and radiotherapy are all successful.
 As a minimum a diagnostic punch biopsy should be taken if treatment other than standard
surgical excision is planned.

Squamous Cell Carcinoma

 Again related to sun exposure.


 May arise in pre - existing solar keratoses.
 May metastasize if left.
 Immunosupression (e.g. following transplant), increases risk.
 Wide local excision is the treatment of choice and where a diagnostic excision biopsy has
demonstrated SCC, repeat surgery to gain adequate margins may be required.

Malignant Melanoma
The main diagnostic features (major criteria): Secondary features (minor criteria)

 Change in size  Diameter >6mm


 Change in shape  Inflammation
 Change in colour  Oozing or bleeding
 Altered sensation

Treatment

 Suspicious lesions should undergo excision biopsy. The lesion should be removed in
completely as incision biopsy can make subsequent histopathological assessment difficult.
 Once the diagnosis is confirmed the pathology report should be reviewed to determine
whether further re-excision of margins is required (see below):

Margins of excision-Related to Breslow thickness


Lesions 0-1mm thick 1cm

Lesions 1-2mm thick 1- 2cm (Depending upon site and pathological features)

Lesions 2-4mm thick 2-3 cm (Depending upon site and pathological features)
Lesions >4 mm thick 3cm

Marsden J et al. Revised UK guidelines for management of Melanoma. Br J Dermatol 2010 163:238-
256.

Further treatments such as sentinel lymph node mapping, isolated limb perfusion and block
dissection of regional lymph node groups should be selectively applied.

Kaposi Sarcoma

 Tumour of vascular and lymphatic endothelium.


 Purple cutaneous nodules.
 Associated with immuno supression.
 Classical form affects elderly males and is slow growing.
 Immunosupression form is much more aggressive and tends to affect those with HIV related
disease.

Non malignant skin disease

Dermatitis Herpetiformis

 Chronic itchy clusters of blisters.


 Linked to underlying gluten enteropathy (coeliac disease).

Dermatofibroma

 Benign lesion.
 Firm elevated nodules.
 Usually history of trauma.
 Lesion consists of histiocytes, blood vessels and fibrotic changes.

Pyogenic granuloma

 Overgrowth of blood vessels.


 Red nodules.
 Usually follow trauma.
 May mimic amelanotic melanoma.

Acanthosis nigricans

 Brown to black, poorly defined, velvety hyperpigmentation of the skin.


 Usually found in body folds such as the posterior and lateral folds of the neck, the axilla,
groin, umbilicus, forehead, and other areas.
 The most common cause of acanthosis nigricans is insulin resistance, which leads to
increased circulating insulin levels. Insulin spillover into the skin results in its abnormal
increase in growth (hyperplasia of the skin).
 In the context of a malignant disease, acanthosis nigricans is a paraneoplastic syndrome and
is then commonly referred to as acanthosis nigricans maligna. Involvement of mucous
membranes is rare and suggests a coexisting malignant condition.

Next question
Question 187 of 347

A 55 year old man from Hong Kong presents with left sided otalgia and recurrent episodes of
epistaxis. On examination his pharynx appears normal. Examination of his neck reveals left sided
cervical lymphadenopathy. What is the most likely underlying diagnosis?

Antrochoanal polyp

Nasopharyngeal carcinoma

Adenocarcinoma of the tonsil

Angiofibroma

Globus syndrome

Given this mans ethnic origin and presenting features a nasopharyngeal carcinoma is the most likely
underlying diagnosis.
Please rate this question:

Discuss and give feedback


Next question

Nasopharyngeal carcinoma

 Squamous cell carcinoma of the nasopharynx


 Rare in most parts of the world, apart from individuals from Southern China
 Associated with Epstein Barr virus infection

Presenting features
Systemic Local

Cervical lymphadenopathy Otalgia

Unilateral serous otitis media


Nasal obstruction, discharge and/ or epistaxis

Cranial nerve palsies e.g. III-VI

Imaging
Combined CT and MRI.

Treatment
Radiotherapy is first line therapy.
Next question
Question 188 of 347

A women is diagnosed as having pernicious anaemia. What is the most likely underlying explanation
for this?

Autoimmune antibodies to parietal cells

Autoimmune antibodies to chief cells

Autoimmune antibodies to goblet cells

Autoimmune antibodies to Brunners glands

Autimmune antibodies to fundic cells

Parietal cell destruction is a major cause of pernicious anaemia and is usually autoimmune
mediated. Other conditions such as bacterial overgrowth may produce mixed patterns and require
more complex diagnostic evaluation.
Please rate this question:

Discuss and give feedback


Next question

Pernicious anaemia

 Pernicious anaemia is a chronic illness caused by impaired absorption of vitamin B-12


because of a lack of intrinsic factor (IF) in gastric secretions. It occurs as a relatively common
adult form of anaemia that is associated with gastric atrophy and a loss of IF production and
as a rare congenital autosomal recessive form in which IF production is lacking without
gastric atrophy.
 Classic pernicious anaemia is caused by the failure of gastric parietal cells to produce
sufficient IF (a gastric protein secreted by parietal cells) to permit the absorption of adequate
quantities of dietary vitamin B-12. Other disorders that interfere with the absorption and
metabolism of vitamin B-12 can produce cobalamin deficiency, with the development of a
macrocytic anaemia and neurologic complications. In many cases the underlying cause is
autoimmune destruction of the parietal cell mass of the gastric antrum.
 Diagnostic evaluation may include investigations to delineate the precise cause and this may
include Schilling test, trials of B12 and in some cases bone marrow sampling.
 Treatment of the disorder is with 3 monthly treatment of vitamin B12 injections. Folic acid
supplementation may also be required.
Question 189 of 347

Patients with suspected temporal arteritis are often sent for temporal artery biopsy. Which statement
is true?

Temporal artery biopsy is only diagnostic if there is visual loss

Biopsy is typically taken from the non-symptomatic side to avoid the risk of blindness

Pre-operative localisation with duplex is mandatory

Biopsies may be non diagnostic in over 50% of cases

Biopsies are usually performed under general anaesthesia

Temporal artery biopsies are frequently non diagnostic. They should be taken from the symptomatic
side and though not mandatory a duplex ultrasound is a helpful investigation, particularly if they mark
the artery. It is usually performed under local anaesthetic.
Please rate this question:

Discuss and give feedback


Next question

Temporal artery biopsy

 Superficial temporal artery is a terminal branch of the external carotid artery

Main indication

 Temporal arteritis

American College of Rheumatology guidelines recommend a temporal artery biopsy if:

 Age of onset older than 50 years


 New-onset headache or localized head pain
 Temporal artery tenderness to palpation or reduced pulsation
 ESR > 50 mm/h

Histopathology

 Vessel wall granulomatous arteritis with mononuclear cell infiltrates and giant cell formation

Procedure

 Position: supine, head 45 degrees


 USS doppler to locate the superficial temporal artery or palpate
 Local anaesthetic
 Artery within temporoparietal fascia
 Clamp and ligate the vessel
 Cut 3-5cm
 Ligate the remaining ends with absorbable suture
 Close the skin

Contraindication
Glucocorticoid therapy > 30 days

Risks
Injury to facial or auriculotemporal nerve
Next question
Question 190 of 347

Which of the following best describes the processes underpinning type IV hypersensitivity reactions?

Deposition of immune complexes of IgG and antigen at the site of inflammation

Deposition of IgA complexes at the site of inflammation

Deposition of IgM and IgG complexes at the site of inflammation

Degranulation of mast cells at the site of inflammation

T cell mediated response at the site of inflammation

Hypersensitivity reactions: ACID

type 1 --Anaphylactic
type 2 --Cytotoxic
type 3 --Immune complex
type 4 --Delayed hypersensitivity

T Cells are the mediators of type 4 hypersensitivity reactions which are characterised by the
absence of immune complex deposition.

Please rate this question:

Discuss and give feedback

Next question

Hypersensitivity reactions

The Gell and Coombs classification divides hypersensitivity reactions into 4 types
Type I Type II Type III Type IV

Description Anaphylactic Cytotoxic Immune Delayed type


complex

Mediator IgE IgG, IgM IgG, Ig A, IgM T-cells

Antigen Exogenous Cell surface Soluble Tissues

Response Minutes Hours Hours 2-3 days


time

Examples Asthma Autoimmune haemolytic Serum sickness Graft versus host


Hay fever anaemia SLE disease
Pemphigus Aspergillosis Contact dermatitis
Goodpasture's

Next question
Question 191-193 of 347

Theme: Bone tumours

A. Osteosarcoma
B. Fibrosarcoma
C. Osteoclastoma
D. Ewings sarcoma
E. Leiomyosarcoma
F. Chondrosarcoma
G. Rhabdomyosarcoma
H. Osteoid osteoma
I. Malignant fibrous histiocytoma

Please select the most appropriate lesion for the clinical scenario given. Each option may be used
once, more than once or not at all.

191. A 16 year-old boy presents to his GP with loss of weight, pain and fever. On
examination, a soft tissue mass is palpable over the mid-thigh region

You answered Osteosarcoma

The correct answer is Ewings sarcoma

Ewing's sarcoma is a malignant round cell tumour occurring in the diaphysis of the long
bones in the children. These are not confined to the ends of long bones. x Rays often
show a large soft-tissue mass with concentric layers of new bone formation ( 'onion-peel'
sign). The ESR may be elevated, thus suggesting an inflammatory or an infective cause
such as osteomyelitis; although osteomyelitis usually affects the metaphyseal region in
children. Treatment is with chemotherapy and surgical excision, an endoprothesis may be
used to conserve the limb.

192. A 75 year old lady presents with weight loss, pain and a swelling over her left knee. She
has been treated for Pagets disease of the bone for some time.

Osteosarcoma

Osteosarcoma may complicate Pagets disease of bone in up to 10% cases. Radiological


appearances include bone destruction coupled with new bone formation, periosteal
elevation may also occur. Surgical resection is the main treatment.

193. A 17-year-old girl presents with weight loss, fever and a swelling over her right knee.
Movements of her knee are restricted. A plain x-ray of the affected site shows multiple
lytic and lucent lesions with clearly defined borders.

You answered Osteosarcoma

The correct answer is Osteoclastoma

Osteoclastoma has a characteristic appearance on x-ray with multple lytic and lucent
areas (Soap bubble) appearances. Pathological fractures may occur. The disease is
usually indolent.

Please rate this question:

Discuss and give feedback


Next question

Sarcomas

 Malignant tumours of mesenchymal origin

Types
May be either bone or soft tissue in origin.
Bone sarcoma include:

 Osteosarcoma
 Ewings sarcoma (although non bony sites recognised)
 Chondrosarcoma - originate from Chondrocytes

Soft tissue sarcoma are a far more heterogeneous group and include:

 Liposarcoma-adipocytes
 Rhabdomyosarcoma-striated muscle
 Leiomyosarcoma-smooth muscle
 Synovial sarcomas- close to joints (cell of origin not known but not synovium)

Malignant fibrous histiocytoma is a sarcoma that may arise in both soft tissue and bone.

Features
Certain features of a mass or swelling should raise suspicion for a sarcoma these include:

 Large >5cm soft tissue mass


 Deep tissue location or intra muscular location
 Rapid growth
 Painful lump

Assessment
Imaging of suspicious masses should utilise a combination of MRI, CT and USS. Blind biopsy should
not be performed prior to imaging and where required should be done in such a way that the biopsy
tract can be subsequently included in any resection.

Ewings sarcoma

 Commoner in males
 Incidence of 0.3 / 1, 000, 000
 Onset typically between 10 and 20 years of age
 Location by femoral diaphysis is commonest site
 Histologically it is a small round tumour
 Blood borne metastasis is common and chemotherapy is often combined with surgery

Osteosarcoma

 Mesenchymal cells with osteoblastic differentiation


 20% of all primary bone tumours
 Incidence of 5 per 1,000,000
 Peak age 15-30, commoner in males
 Limb preserving surgery may be possible and many patients will receive chemotherapy

Liposarcoma

 Malignancy of adipocytes
 Rare, approximately 2.5 per 1,000,000. They are the second most common soft tissue
sarcoma
 Typically located in deep locations such as retroperitoneum
 Affect older age group usually >40 years of age
 May be well differentiated and thus slow growing although may undergo de-differentiation
and disease progression
 Many tumours will have a pseudocapsule that can misleadingly allow surgeons to feel that
they can 'shell out' these lesions. In reality, tumour may invade at the edge of the
pseudocapsule and result in local recurrence if this strategy is adopted
 Usually resistant to radiotherapy, although this is often used in a palliative setting

Malignant Fibrous Histiocytoma

 Tumour with large number of histiocytes


 Most common sarcoma in adults
 Also described as undifferentiated pleomorphic sarcoma NOS (i.e. Cell of origin is not
known)
 Four major subtypes are recognised: storiform-pleomorphic (70% cases), myxoid (less
aggressive), giant cell and inflammatory
 Treatment is usually with surgical resection and adjuvant radiotherapy as this reduces the
likelihood of local recurrence

Next question
Question 194 of 347

A 56 year old man is diagnosed with an abdominal aortic aneurysm and undergoes a CT scan to
asses the size of the aorta. During the course of his investigations a lesion of the adrenal gland is
identified. It measures 1.5 cm in diameter and the gland is otherwise normal. What is the most likely
diagnosis?

Adrenal gland metastasis

Adrenal gland arterio-venous malformation

Adrenal cyst

Phaeochromocytoma

Adrenal cortical adenoma

25% of all adrenal lesions >4cm in diameter are malignant

Theme from 2011 Exam


Incidentalomas of the adrenal gland are common and represent the most likely lesion in this
scenario. Clearly the other lesions are all possibilities but are unlikely.
Please rate this question:

Discuss and give feedback


Next question

Adrenal lesions- Incidental

Incidentaloma of the adrenal glands have become increasingly common as CT scanning of the
abdomen is widely undertaken. Prevalences range from 1.5-9% in autopsy studies. Overall, 75% will
be non functioning adenomas. However, a thorough diagnostic work up is required to exclude a
more significant lesion.

Investigation

 Morning and midnight plasma cortisol measurements


 Dexamethasone suppression test
 24 hour urinary cortisol excretion
 24 hour urinary excretion of catecholamines
 Serum potassium, aldosterone and renin levels

Management
The risk of malignancy is related to the size of the lesion and 25% of all masses greater than 4cm
will be malignant. Such lesions should usually be excised. Where a lesion is a suspected metastatic
deposit a biopsy may be considered. Smaller, innocent lesions are usually followed up by serial CT
scans at 6, 12 and 24 months.
Next question
Question 195 of 347

A 22 year old man is undergoing an abdominal ultrasound scan as part of a series of investigations
for abdominal pain. The radiologist notes that there is evidence of splenic atrophy. What is the most
likely cause?

Letterer-Siwe disease

Coeliac disease

Malaria

Niemann-Pick disease

Sarcoidosis

Splenic atrophy may occur in coeliac disease together with the appearance of Howell-Jolly bodies in
erythrocytes. Letterer - Siwe disease is a form of Histiocytosis X in which macrophages proliferate.
Please rate this question:

Discuss and give feedback


Next question

Spleen

The spleen is located in the left upper quadrant of the abdomen and its size can vary depending
upon the amount of blood it contains. The typical adult spleen is 12.5cm long and 7.5cm wide. The
usual weight of the adult spleen is 150g.
The exact position of the spleen can vary with respiratory activity, posture and the state of
surrounding viscera. It usually lies obliquely with its long axis aligned to the 9th, 10th and 11th ribs. It
is separated from these ribs by both diaphragm and pleural cavity. The normal spleen is not
palpable.

The shape of the spleen is influenced by the state of the colon and stomach. Gastric distension will
cause the spleen to resemble the shape of an orange segment. Colonic distension will cause it to
become more tetrahedral.

The spleen is almost entirely covered by peritoneum, which adheres firmly to its capsule. Recesses
of the greater sac separate it from the stomach and kidney. It develops from the upper dorsal
mesogastrium, remaining connected to the posterior abdominal wall and stomach by two folds of
peritoneum; the lienorenal ligament and gastrosplenic ligament. The lienorenal ligament is derived
from peritoneum where the wall of the general peritoneum meets the omental bursa between the left
kidney and spleen; the splenic vessels lie in its layers. The gastrosplenic ligament also has two
layers, formed by the meeting of the walls of the greater sac and omental bursa between spleen and
stomach, the short gastric and left gastroepiploic branches of the splenic artery pass in its layers.
Laterally, the spleen is in contact with the phrenicocolic ligament.

Relations
Superiorly Diaphragm

Anteriorly Gastric impression

Posteriorly Kidney

Inferiorly Colon

Tail of pancreas and splenic vessels (splenic artery divides here, branches pass to the
Hilum
white pulp transporting plasma)

Contents
White Immune function. Contains central trabecular artery. The germinal centres are supplied
pulp by arterioles called penicilliary radicles.

Red pulp Filters abnormal red blood cells.

Function

 Filtration of abnormal blood cells and foreign bodies such as bacteria.


 Immunity: IgM. Production of properdin, and tuftsin which help target fungi and bacteria for
phagocytosis.
 Haematopoiesis: up to 5th month gestation or in haematological disorders.
 Pooling: storage of 40% platelets.
 Iron reutilisation
 Storage monocytes

Disorders of the spleen


Massive splenomegaly

 Myelofibrosis
 Chronic myeloid leukaemia
 Visceral leishmaniasis (kala-azar)
 Malaria
 Gaucher's syndrome

Other causes (as above plus)


 Portal hypertension e.g. secondary to cirrhosis
 Lymphoproliferative disease e.g. CLL, Hodgkin's
 Haemolytic anaemia
 Infection: hepatitis, glandular fever
 Infective endocarditis
 Sickle-cell*, thalassaemia
 Rheumatoid arthritis (Felty's syndrome)

*the majority of adult patients with sickle-cell will have an atrophied spleen due to repeated infarction
Next question
Question 196 of 347

A patient undergoes excision of a lump and the pathologist then examines a histological section and
identifies a clump of Reed Sternberg cells. What is the most likely diagnosis?

Thymoma

Papillary carcinoma of the thyroid

Hodgkins lymphoma

Kaposi sarcoma

Carcinoma of the parathyroid gland

Reed Sternberg cells are diagnostic of Hodgkins lymphoma

Please rate this question:

Discuss and give feedback


Next question

Hodgkins lymphoma

Presenting features

 Asymptomatic lympadenopathy
 Cough, Pel Ebstein fever, haemoptysis, dyspnoea
 B Symptoms - 10% weight loss, fever, night sweats

Staging
All patients are staged with CT scanning of the chest, abdomen and pelvis
The Ann Arbor staging system is commonly used
Stage Features
Stage Features

I Single lymph node region

II Two or more regions on the same side of the diaphragm

III Involvement of lymph node regions on both sides of the diaphragm

IV Involvement of extra nodal sites

Sub types
Classical Hodgkin lymphoma is classified into the following 4 types:

Nodular sclerosing Hodgkin lymphoma (NSHL)


Mixed-cellularity Hodgkin lymphoma (MCHL)
Lymphocyte-depleted Hodgkin lymphoma (LDHL)
Lymphocyte-rich classical Hodgkin lymphoma (LRHL)

A Reed Sternberg cell may be identified histologically.

A fifth sub type, Nodular lymphocyte-predominant Hodgkin lymphoma, is characterised by a different


cell type Reed- Sternberg cells are rarely seen.

Treatment
This may be multimodal and both chemo and radiotherapy are used.

Diagnosis
This is made by excision of a complete lymph node that is then submitted for detailed histological
evaluation.

Pathogenesis
Infection with Ebstein Barr virus is linked to the condition (particularly mixed cellularity lymphoma).

Prognosis
Stage I disease is associated with survival figures of up to 85% at 5 years. The lymphocyte rich
classical lymphoma has the best prognosis. Lymphocyte depleted Hodgkins lymphoma, advancing
age, male sex and stage IV disease are all associated with a worsening of prognosis.
Next question
Question 197 of 347

Which statement relating to phaeochromocytoma is untrue?

They are tumours of chromaffin cells in the adrenal medulla.

They are bilateral in 10% of cases.

When located in an extra adrenal location have a higher incidence of malignancy.

May be associated with an elevated urinary VMA.

Up to 40% may have a blood pressure within the normal range.

Normotension is seen in around 10% cases. The remainder show a degree of hypertension.
Please rate this question:

Discuss and give feedback


Next question

Phaeochromocytoma and adrenal lesions

Phaeochromocytoma
Neuroendocrine tumour of the chromaffin cells of the adrenal medulla. Hypertension and
hyperglycaemia are often found.

 10% of cases are bilateral.


 10% occur in children.
 11% are malignant (higher when tumour is located outside the adrenal).
 10% will not be hypertensive.

Familial cases are usually linked to the Multiple endocrine neoplasia syndromes (considered under
its own heading).

Most tumours are unilateral (often right sided) and smaller than 10cm.

Diagnosis
Urine analysis of vanillymandelic acid (VMA) is often used (false positives may occur e.g. in patients
eating vanilla ice cream!)
Blood testing for plasma metanephrine levels.

CT and MRI scanning are both used to localise the lesion.

Treatment
Patients require medical therapy first. An irreversible alpha adrenoreceptor blocker should be given,
although minority may prefer reversible blockade(1). Labetolol may be co-administered for cardiac
chronotropic control. Isolated beta blockade should not be considered as it will lead to unopposed
alpha activity.

These patients are often volume depleted and will often require moderate volumes of intra venous
normal saline perioperatively.

Once medically optimised the phaeochromocytoma should be removed. Most adrenalectomies can
now be performed using a laparoscopic approach(2). The adrenals are highly vascular structures
and removal can be complicated by catastrophic haemorrhage in the hands of the inexperienced.
This is particularly true of right sided resections where the IVC is perilously close. Should the IVC be
damaged a laparotomy will be necessary and the defect enclosed within a Satinsky style vascular
clamp and the defect closed with prolene sutures. Attempting to interfere with the IVC using any
instruments other than vascular clamps will result in vessel trauma and make a bad situation much
worse.

Incidental adrenal lesions


Adrenal lesions may be identified on CT scanning performed for other reasons(3). Factors
suggesting benign disease on CT include(4):

 Size less than 3cm


 Homogeneous texture
 Lipid rich tissue
 Thin wall to lesion

All patients with incidental lesions should be managed jointly with an endocrinologist and full work up
as described above. Patients with functioning lesions or those with adverse radiological features
(Particularly size >3cm) should proceed to surgery.

References
1. Weingarten TN, Cata JP, O'Hara JF, Prybilla DJ, Pike TL, Thompson GB, et al. Comparison of
two preoperative medical management strategies for laparoscopic resection of pheochromocytoma.
Urology. 2010 Aug;76(2):508 e6-11.

2. Nguyen PH, Keller JE, Novitsky YW, Heniford BT, Kercher KW. Laparoscopic approach to
adrenalectomy: review of perioperative outcomes in a single center. Am Surg. 2011 May;77(5):592-
6.

3. Ng VW, Ma RC, So WY, Choi KC, Kong AP, Cockram CS, et al. Evaluation of functional and
malignant adrenal incidentalomas. Arch Intern Med. 2010 Dec 13;170(22):2017-20.

4. Muth A, Hammarstedt L, Hellstrom M, Sigurjonsdottir HA, Almqvist E, Wangberg B. Cohort study


of patients with adrenal lesions discovered incidentally. Br J Surg. 2011 May 27.
Question 198 of 347

A 69 year old man presents with a purple lesion on his forearm. It is excised and an a 3 cm Merkel
cell tumour is diagnosed. Which of the following statements relating to this diagnosis is false?

He should undergo a sentinel lymph node biopsy.

Lymphovascular invasion is typically seen histologically

They are more common in immunosupressed patients

Histologically they may resemble pyogenic granuloma

They are associated with visceral metastasis

Merkel cell tumours are rare cutaneous tumours. Histologically they consist of sheets and nodules of
hyperchromatic epithelial cells, with high rates of mitosis and apoptosis. As such they are relatively
easy to distinguish from pyogenic granuloma which has no features of malignancy and would not
show lymphovascular invasion.
Please rate this question:

Discuss and give feedback


Next question

Merkel cell tumours of the skin

 Rare but aggressive tumour.


 Develops from intra epidermal Merkel cells.
 Usually presents on elderly, sun damaged skin. The periorbital area is the commonest site.
 Histologically these tumours appear within the dermis and subcutis. The lesions consist of
sheets and nodules of small hyperchromatic epithelial cells with high rates of mitosis and
apoptosis. Lymphovascular invasion is commonly seen.
 Pre-existing infection with Merkel Cell Polyomavirus is seen in 80% cases.

Treatment
Surgical excision is first line. Margins of 1cm are required. Lesions >10mm in diameter should
undergo sentinel lymph node biopsy. Adjuvant radiotherapy is often given to reduce the risk of local
recurrence.
Prognosis

 With lymph node metastasis 5 year survival is 50% or less.


 Small lesions without nodal spread are usually associated with a 5 year survival of 80%.

Skin biopsy demonstrating a Merkel Cell cancer. Note the hyperchromatic epithelial cells and high
mitotic rate

Image sourced from Wikipedia

Next question
Question 199 of 347

A 58 year old lady undergoes a screening mammogram and appearances are suspicious for ductal
carcinoma in situ. A stereotactic core biopsy is performed. If ductal carcinoma in situ is to be
diagnosed, which of the following pathological features must not be present?

Nuclear pleomorphism

Coarse chromatin

Abnormal mitoses

Angiogenesis

Dysplastic cells infiltrating the suspensory ligaments of the breast

The presence of invasion is a hallmark of invasive disease and thus would not be a feature of DCIS.
Angiogenesis may occur in association with high grade DCIS.
Please rate this question:

Discuss and give feedback


Next question

Histopathology of malignancy

 Abnormal tissue architecture


 Coarse chromatin
 Invasion of basement membrane*
 Abnormal mitoses
 Angiogenesis
 De-differentiation
 Areas of necrosis
 Nuclear pleomorphism

*= Those features that distinguish invasive malignancy from in situ disease


Next question
Question 200 of 347

Which of the following does not occur as a pathological response to extensive burns?

Plasma leakage into interstitial space

Absolute polycythaemia

Increased haematocrit

Keratinocyte migration during healing

Cardiac output reduction by 50% in first 30 minutes

Haemolysis is the main pathological response.


Please rate this question:

Discuss and give feedback


Next question

Burns pathology

Extensive burns

 Haemolysis due to damage of erythrocytes by heat and microangiopathy


 Loss of capillary membrane integrity causing plasma leakage into interstitial space
 Extravasation of fluids from the burn site causing hypovolaemic shock (up to 48h after
injury)- decreased blood volume and increased haematocrit
 Protein loss
 Secondary infection e.g. Staphylococcus aureus
 ARDS
 Risk of Curlings ulcer (acute peptic stress ulcers)
 Danger of full thickness circumferential burns in an extremity as these may develop
compartment syndrome

Healing

 Superficial burns: keratinocytes migrate to form a new layer over the burn site
 Full thickness burns: dermal scarring. Usually need keratinocytes from skin grafts to provide
optimal coverage.

Next question
Question 1-3 of 147

Theme: Chest pain

A. Pulmonary embolism
B. Acute exacerbation asthma
C. Physiological
D. Mitral valve stenosis
E. Aortic dissection
F. Mitral regurgitation
G. Bronchopneumonia
H. Tuberculosis
I. None of the above

What is the most likely diagnosis for the scenario given? Each option may be used once, more than
once or not at all.

1. A 28 year old Indian woman, who is 18 weeks pregnant, presents with increasing shortness
of breath, chest pain and coughing clear sputum. She is apyrexial, blood pressure is 140/80
mmHg, heart rate 130 bpm and saturations 94% on 15L oxygen. On examination there is a
mid diastolic murmur, there are bibasal crepitations and mild pedal oedema. She suddenly
deteriorates and has a respiratory arrest. Her chest x-ray shows a whiteout of both of her
lungs.

You answered Pulmonary embolism

The correct answer is Mitral valve stenosis

Mitral stenosis is the commonest cause of cardiac abnormality occurring in pregnant


women. Mitral stenosis is becoming less common in the UK population, however should be
considered in women from countries where there is a higher incidence of rheumatic heart
disease. Mitral stenosis causes a mid diastolic murmur which may be difficult to auscultate
unless the patient is placed into the left lateral position. These patients are at risk of atrial
fibrillation (up to 40%), which can also contribute to rapid decompensation such as
pulmonary oedema (hence cxr 'whiteout' of lungs). Physiological changes in pregnancy may
cause an otherwise asymptomatic patient to suddenly deteriorate. Balloon valvuloplasty is
the treatment of choice.

2. A 28 year old woman, who is 30 weeks pregnant, presents with sudden onset chest pain
associated with loss of consciousness. Her blood pressure is 170/90 mmHg, saturations on
15L oxygen 93%, heart rate 120 bpm and she is apyrexial. On examination there is an early
diastolic murmur, occasional bibasal creptitations and mild peal oedema. An ECG shows
ST elevation in leads II, III and aVF.
You answered Pulmonary embolism

The correct answer is Aortic dissection

Aortic dissection is associated with the 3rd trimester of pregnancy, connective tissue
disorders (Marfan's, Ehlers- Danlos) and bicuspid valve. Patients may complain of a tearing
chest pain or syncope. Clinically they may be hypertensive. The right coronary artery may
become involved in the dissection, causing myocardial infarct in up to 2% cases (hence ST
elevation in the inferior leads). An aortic regurgitant murmur may be auscultated.

3. A 28 year old woman, who is 18 weeks pregnant, presents with sudden chest pain. Her
blood pressure is 150/70 mmHg, saturations are 92% on 15L oxygen and her heart rate is
130 bpm. There are no murmurs and her chest is clear. There is signs of thrombophlebitis in
the left leg.

Pulmonary embolism

Chest pain, hypoxia and clear chest on auscultation in pregnancy should lead to a high
suspicion of pulmonary embolism.
Theme question in September 2011 exam
Pregnant women can decompensate rapidly from cardiac compromise.

Please rate this question:

Discuss and give feedback


Next question

Chest pain in pregnancy

Aortic dissection

 Predisposing factors in pregnancy are hypertension, congenital heart disease and Marfan's
syndrome
 Mainly Stanford type A dissections
 Sudden tearing chest pain, transient syncope
 Patient may be cold and clammy, hypertensive and have an aortic regurgitation murmur
 Involvement of the right coronary artery may cause inferior myocardial infarction

Surgical management
Gestational timeframe Management

< 28/40 Aortic repair with the fetus kept in utero

28-32/40 Dependent on fetal condition

> 32/40 Primary Cesarean section followed by aortic repair at the same operation

Mitral stenosis

 Most cases associated with rheumatic heart disease


 Becoming less common in British women; suspect in Immigrant women
 Commonest cardiac condition in pregnancy
 Commonly associated with mortality
 Valve surgery; balloon valvuloplasty preferable

Pulmonary embolism

 Leading cause of mortality in pregnancy


 Half dose scintigraphy; CT chest if underlying lung disease, should aid diagnosis
 Treatment with low molecular weight heparin throughout pregnancy and 4-6 weeks after
childbirth
 Warfarin is contra indicated in pregnancy (though may be continued in women with
mechanical heart valves due to the significant risk of thromboembolism)

References
1. Bates S.M. and Ginsberg J.S. How we manage venous thromboembolism during
pregnancy. Blood2002 (100): 3470-3478.

2. Scarsbrook A.Fand Gleeson V. Investigating suspected pulmonary embolism in


pregnancy. BMJ2007 (326) : 1135 doi: 10.1136/bmj.7399.1135.

3. Morley C. A. and Lim B. A. Lesson of the Week: The risks of delay in diagnosis of breathlessness
in pregnancy. BMJ 1995 (311) : 1083.
Next question
Question 4 of 147

A 67 year old man is investigated for biliary colic and a 4.8 cm abdominal aortic aneurysm is
identified. Which of the following statements relating to this condition is untrue?

The wall will be composed of dense fibrous tissue only

The majority are located inferior to the renal arteries

They occur most often in current or former smokers

He should initially be managed by a process of active surveillance

Aortoduodenal fistula is a recognised complication following repair.

They are true aneurysms and have all 3 layers of arterial wall.
Please rate this question:

Discuss and give feedback


Next question

Abdominal aorta aneurysm

 Abdominal aortic aneurysms are a common problem in vascular surgery.


 They may occur as either true or false aneurysm. With the former all 3 layers of the arterial
wall are involved, in the latter only a single layer of fibrous tissue forms the aneurysm wall.
 True abdominal aortic aneurysms have an approximate incidence of 0.06 per 1000 people.
They are commonest in elderly men and for this reason the UK is now introducing the
aneurysm screening program with the aim of performing an abdominal aortic ultrasound
measurement in all men aged 65 years.

Causes

 Several different groups of patients suffer from aneurysmal disease.


 The commonest group is those who suffer from standard arterial disease, i.e. Those who
arehypertensive, have diabetes and have been or are smokers.
 Other patients such as those suffering from connective tissue diseases such as Marfan's
may also develop aneurysms. In patients with abdominal aortic aneurysms the extracellular
matrix becomes disrupted with a change in the balance of collagen and elastic fibres.

Management

 Most abdominal aortic aneurysms are an incidental finding.


 Symptoms most often relate to rupture or impending rupture.
 20% rupture anteriorly into the peritoneal cavity. Very poor prognosis.
 80% rupture posteriorly into the retroperitoneal space
 The risk of rupture is related to aneurysm size, only 2% of aneurysms measuring less than
4cm in diameter will rupture over a 5 year period. This contrasts with 75% of aneurysms
measuring over 7cm in diameter.
 This is well explained by Laplaces' law which relates size to transmural pressure.
 For this reason most vascular surgeons will subject patients with an aneurysm size of 5cm or
greater to CT scanning of the chest, abdomen and pelvis with the aim of delineating anatomy
and planning treatment. Depending upon co-morbidities, surgery is generally offered once
the aneurysm is between 5.5cm and 6cm.

A CT reconstruction showing an infrarenal abdominal aortic aneurysm. The walls of the sac are
calcified which may facilitate identification on plain x-rays
Image sourced from Wikipedia

Indications for surgery

 Symptomatic aneurysms (80% annual mortality if untreated)


 Increasing size above 5.5cm if asymptomatic
 Rupture (100% mortality without surgery)
Surgical procedures
Abdominal aortic aneurysm repair

Procedure:

GA
Invasive monitoring (A-line, CVP, catheter)
Incision: Midline or transverse
Bowel and distal duodenum mobilised to access aorta.
Aneurysm neck and base dissected out and prepared for cross clamp
Systemic heparinisation
Cross clamp (proximal first)
Longitudinal aortotomy
Atherectomy
Deal with back bleeding from lumbar vessels and inferior mesenteric artery
Insert graft either tube or bifurcated depending upon anatomy
Suture using Prolene (3/0 for proximal , distal anastomosis suture varies according to site)
Clamps off: End tidal CO2 will rise owing to effects of reperfusion, at this point major risk of
myocardial events.
Haemostasis
Closure of aneurysm sac to minimise risk of aorto-enteric fistula
Closure: Loop 1 PDS or Prolene to abdominal wall
Skin- surgeons preference

Post operatively:

ITU (Almost all)


Greatest risk of complications following emergency repair
Complications: Embolic- gut and foot infarcts
Cardiac - owing to premorbid states, re-perfusion injury and effects of cross clamp
Wound problems
Later risks related to graft- infection and aorto-enteric fistula

Special groups

Supra renal AAA


These patients will require a supra renal clamp and this carries a far higher risk of complications and
risk of renal failure.

Ruptured AAA
Pre-operatively the management depends upon haemodynamic instability. In patients with
symptoms of rupture (typical pain, haemodynamic compromise and risk factors) then ideally prompt
laparotomy. In those with vague symptoms and haemodynamic stability the ideal test is CT scan to
determine whether rupture has occurred or not. Most common rupture site is retroperitoneal 80%.
These patients will tend to develop retroperitoneal haematoma. This can be disrupted if Bp is
allowed to rise too high so aim for Bp 100mmHg.
Operative details are similar to elective repair although surgery should be swift, blind rushing often
makes the situation worse. Plunging vascular clamps blindly into a pool of blood at the aneurysm
neck carries the risk of injury the vena cava that these patients do not withstand. Occasionally a
supracoeliac clamp is needed to effect temporary control, although leaving this applied for more than
20 minutes tends to carry a dismal outcome.
EVAR
Increasingly patients are now being offered endovascular aortic aneurysm repair. This is undertaken
by surgeons and radiologists working jointly. The morphology of the aneurysm is important and not
all are suitable. Here is a typical list of those features favoring a suitable aneurysm:

 Long neck
 Straight iliac vessels
 Healthy groin vessels

Clearly few AAA patients possess the above and compromise has to be made. The use of
fenestrated grafts can allow supra renal AAA to be treated.

Procedure:

GA
Radiology or theatre
Bilateral groin incisions
Common femoral artery dissected out
Heparinisation
Arteriotomy and insertion of guide wire
Dilation of arteriotomy
Insertion of EVAR Device
Once in satisfactory position it is released
Arteriotomy closed once check angiogram shows good position and no endoleak

Complications:
Endoleaks depending upon site are either Type I or 2. These may necessitate re-intervention and all
EVAR patients require follow up . Details are not needed for MRCS.

References
A reasonable review is provided by:
Sakalihasan N, Limet R, Defawe O. Abdominal aortic aneurysm. Lancet 2005 (365):1577- 1589
Next question
Question 5 of 147

Which of the following statements in relation to the p53 tumour suppressor protein is false?

It may induce necrosis of cells with non repairable DNA damage

It is affected in Li Fraumeni syndrome

It can induce DNA repair

It can halt the cell cycle

It may inhibit angiogenesis

When DNA cannot be repaired it will induce cellular apoptosis (not necrosis)
Please rate this question:

Discuss and give feedback


Next question

Genetics and surgical disease

Some of the more commonly occurring genetic conditions occurring in surgical patients are
presented here.

Li-Fraumeni Syndrome

 Autosomal dominant
 Consists of germline mutations to p53 tumour suppressor gene
 High incidence of malignancies particularly sarcomas and leukaemias
 Diagnosed when:

*Individual develops sarcoma under 45 years


*First degree relative diagnosed with any cancer below age 45 years and another family member
develops malignancy under 45 years or sarcoma at any age

BRCA 1 and 2
 Carried on chromosome 17 (BRCA 1) and Chromosome 13 (BRCA 2)
 Linked to developing breast cancer (60%) risk.
 Associated risk of developing ovarian cancer (55% with BRCA 1 and 25% with BRCA 2).

Lynch Syndrome

 Autosomal dominant
 Develop colonic cancer and endometrial cancer at young age
 80% of affected individuals will get colonic and/ or endometrial cancer
 High risk individuals may be identified using the Amsterdam criteria

Amsterdam criteria
Three or more family members with a confirmed diagnosis of colorectal cancer, one of whom is a
first degree (parent, child, sibling) relative of the other two.
Two successive affected generations.
One or more colon cancers diagnosed under age 50 years.
Familial adenomatous polyposis (FAP) has been excluded.

Gardners syndrome

 Autosomal dominant familial colorectal polyposis


 Multiple colonic polyps
 Extra colonic diseases include: skull osteoma, thyroid cancer and epidermoid cysts
 Desmoid tumours are seen in 15%
 Mutation of APC gene located on chromosome 5
 Due to colonic polyps most patients will undergo colectomy to reduce risk of colorectal
cancer
 Now considered a variant of familial adenomatous polyposis coli

Next question
Question 6 of 147

Which of the following cell types is most likely to be identified in the wall of a fistula in ano?

Squamous cells

Goblet cells

Columnar cells

Ciliated columnar cells

None of the above

A fistula is an abnormal connection between two epithelial lined surfaces, in the case of a fistula in
ano it will be lined by squamous cells.
Please rate this question:

Discuss and give feedback


Next question

Fistulas

 A fistula is defined as an abnormal connection between two epithelial surfaces.


 There are many types ranging from Branchial fistulae in the neck to entero-cutaneous
fistulae abdominally.
 In general surgical practice the abdominal cavity generates the majority and most of these
arise from diverticular disease and Crohn's.
 As a general rule all fistulae will resolve spontaneously as long as there is no distal
obstruction. This is particularly true of intestinal fistulae.

The four types of fistulae are:

Enterocutaneous
These link the intestine to the skin. They may be high (>500ml) or low output (<250ml) depending
upon source. Duodenal /jejunal fistulae will tend to produce high volume, electrolyte rich secretions
which can lead to severe excoriation of the skin. Colo-cutaneous fistulae will tend to leak faeculent
material. Both fistulae may result from the spontaneous rupture of an abscess cavity onto the skin
(such as following perianal abscess drainage) or may occur as a result of iatrogenic input. In some
cases it may even be surgically desirable e.g. mucous fistula following sub total colectomy for colitis.

Suspect if there is excess fluid in the drain.

Enteroenteric or Enterocolic
This is a fistula that involves the large or small intestine. They may originate in a similar manner to
enterocutaneous fistulae. A particular problem with this fistula type is that bacterial overgrowth may
precipitate malabsorption syndromes. This may be particularly serious in inflammatory bowel
disease.

Enterovaginal
Aetiology as above.

Enterovesicular
This type of fistula goes to the bladder. These fistulas may result in frequent urinary tract infections,
or the passage of gas from the urethra during urination.

Management
Some rules relating to fistula management:

 They will heal provided there is no underlying inflammatory bowel disease and no distal
obstruction, so conservative measures may be the best option
 Where there is skin involvement, protect the overlying skin, often using a well fitted stoma
bag- skin damage is difficult to treat
 A high output fistula may be rendered more easily managed by the use of octreotide, this will
tend to reduce the volume of pancreatic secretions.
 Nutritional complications are common especially with high fistula (e.g. high jejunal or
duodenal) these may necessitate the use of TPN to provide nutritional support together with
the concomitant use of octreotide to reduce volume and protect skin.
 When managing perianal fistulae surgeons should avoid probing the fistula where acute
inflammation is present, this almost always worsens outcomes.
 When perianal fistulae occur secondary to Crohn's disease the best management option is
often to drain acute sepsis and maintain that drainage through the judicious use of setons
whilst medical management is implemented.
 Always attempt to delineate the fistula anatomy, for abscesses and fistulae that have an intra
abdominal source the use of barium and CT studies should show a track. For perianal
fistulae surgeons should recall Goodsall's rule in relation to internal and external openings.

Next question
Question 7 of 147

A 22 year old man is referred to the surgical clinic. He has been complaining of varicose veins for
many years. On examination he has extensive varicosities of the right leg, there are areas of marked
port wine staining. The saphenofemoral junction is competent on doppler assessment. The most
likely underlying diagnosis is:

Deep vein thrombosis

Klippel-Trenaunay syndrome

Varicose veins due to sapheno-popliteal junction incompetence

Sturge - Weber syndrome

Angiosarcoma

Sturge - Weber syndrome is a an arteriovenous malformation affecting the face and CNS, the
peripheral vessels are not affected. Simple varicose veins should not typically be associated with
port wine staining, nor should a DVT or angiosarcoma.
Please rate this question:

Discuss and give feedback


Next question

Klippel-Trenaunay syndrome

Klippel-Trenaunay-Weber syndrome generally affects a single extremity, although cases of multiple


affected limbs have been reported. The leg is the most common site followed by the arms, the trunk,
and rarely the head and the neck

Signs and symptoms


The birth defect is diagnosed by the presence of a combination of these symptoms:

 One or more distinctive port-wine stains with sharp borders


 Varicose veins
 Hypertrophy of bony and soft tissues, that may lead to local gigantism or shrinking.
 An improperly developed lymphatic system
In some cases, port-wine stains (capillary port wine type) may be absent. Such cases are very rare
and may be classified as "atypical Klippel-Trenaunay syndrome".

KTS can either affect blood vessels, lymph vessels, or both. The condition most commonly presents
with a mixture of the two. Those with venous involvement experience increased pain and
complications.
Next question
Question 8 of 147

A 68 year old man presents with an ulcerated lesion on his right cheek. It is excised and on
histological assessment a squamous cell carcinoma is diagnosed. It measures 25mm in diameter
and is 4mm deep. Which of the following statements relating to this condition is false?

In this particular case margins of at least 6mm are required

Use of cryosurgery to treat this patients lesion would have been unsafe

Use of radiotherapy to treat this lesion would have been unsafe

This patients local recurrence rate may approach 15%

The disease usually spreads via lymphatics

Poor prognostic factors in SCC:

 Size >20mm (local recurrence rate of up to 15%)


 Depth greater than 4mm (risk of metastasis up to 30%)

This man has an SCC with significant risk of metastasis. Although cryotherapy may be used to treat
SCC it would be most unsafe in this setting as the lesion extends deeply. However, radiotherapy is a
safe treatment modality for SCC and may be used in selected cases. It is unwise to use radiotherapy
in areas prone to radionecrosis e.g. the nose.
Please rate this question:

Discuss and give feedback


Next question

Squamous cell carcinoma of the skin

 Second most common skin malignancy


 Derived from epidermal keratinocytes
 Commonest in fair skinned individuals in sun exposed sites
 May occur in perianal and genital skin especially in association with Human Papilloma Virus
16 and 18 infections.
Groups at high risk
Renal transplant and on immunosuppression

Individuals with HIV

Those who have received psoralen UVA therapy

Chronic wounds (Marjolins ulcer)

Xeroderma pigmentosum

Oculocutaneous albinism

Prognosis
Good Prognosis Poor prognosis

Well differentiated tumours Poorly differentiated tumours

<20mm diameter >20mm in diameter

<2mm deep >4mm deep

No associated diseases Immunosupression for whatever reason

Treatment
Surgical excision with 4mm margins if lesion <20mm in diameter. If tumour >20mm then margins
should be 6mm.

Squamous cell carcinomas of the skin typically arise in areas of sun exposure as shown here
Image sourced from Wikipedia

References
The British Association of Dermatology provides guidelines for the diagnosis and treatment of SCC.
http://www. bad.org.uk
Next question
Question 9 of 147

A 23 year old man presents with weight loss fatigue and lymphadenopathy. He is diagnosed with
tuberculosis. Which of the following processes most closely matches the underlying pathological
process?

Type 1 hypersensitivity reaction

Type 2 hypersensitivity reaction

Type 3 hypersensitivity reaction

Type 4 hypersensitivity reaction

None of the above

Granulomas (which occur in tuberculosis) are a feature of Type 4 hypersensitivity reactions.

Please rate this question:

Discuss and give feedback

Next question

Hypersensitivity reactions

The Gell and Coombs classification divides hypersensitivity reactions into 4 types

Type I Type II Type III Type IV


Type I Type II Type III Type IV

Description Anaphylactic Cytotoxic Immune Delayed type


complex

Mediator IgE IgG, IgM IgG, Ig A, IgM T-cells

Antigen Exogenous Cell surface Soluble Tissues

Response Minutes Hours Hours 2-3 days


time

Examples Asthma Autoimmune haemolytic Serum sickness Graft versus host


Hay fever anaemia SLE disease
Pemphigus Aspergillosis Contact dermatitis
Goodpasture's

Next question
Question 10 of 147

A 73 year old man undergoes an emergency amputation for severe lower limb sepsis and gangrene.
Post operatively he develops disseminated intravascular coagulation. Which of the following clotting
factors will be most rapidly consumed in this process?

Factor V and VIII

Factor I

Factor I and III

Factor III and VII

Factor VI and VIII

D-I-S-S-E-M-I-N-A-T-E-D

D-Dx: D dimer
I-Immune complexes
S-Snakebite, shock, heatstroke
S-SLE
E-Eclampsia, HELLP syndrome
M-Massive tissue damage
I-Infections: viral and bacterial
N-Neoplasms
A-Acute promyelocytic leukemia
T-Tumor products: Tissue Factor (TF) and TF-like factors released by carcinomas of pancreas,
prostate, lung,
colon, stomach
E-Endotoxins (bacterial)
D-Dead fetus (retained)

DIC Will tend to consume factors five and eight intially (and platelets).
Please rate this question:

Discuss and give feedback


Next question

Disseminated intravascular coagulation


Simultaneous coagulation and haemorrhage caused by initially formation of thrombi which consume
clotting factors (factors 5,8) and platelets, ultimately leading to bleeding

Causes include:

 Infection
 Malignancy
 Trauma e.g. major surgery, burns, shock, dissecting aortic aneurysm
 Liver disease
 Obstetric complications

Key points

 Clinically bleeding is usually a dominant feature, bruising, ischaemia and organ failure
 Blood tests: prolonged clotting times, thrombocytopenia, decreased fibrinogen, increased
fibrinogen degradation products
 Treat the underlying cause and supportive management

Next question
Question 11 of 147

A pregnant women suddenly develops bilateral leg swelling. Her mother and aunt were troubled by
the same problem. What is the most likely underlying abnormality?

Anti endomysial antibodies

Anti nuclear antibodies

Anti cardiolipin antibodies

Anti thyroid antibodies

Anti mitochondrial antibodies

Anti phospholipid syndrome= following antibodies

 Lupus anticoagulant
 Anti-cardiolipin
 Anti-β2-glycoprotein

Theme from September 2014 Exam


Antiphospholipid syndrome, is an autoimmune, hypercoagulable state caused by antiphospholipid
antibodies. APS provokes blood clots (thrombosis) in both arteries and veins as well as pregnancy-
related complications such as miscarriage, stillbirth, preterm delivery, or severe preeclampsia.

The diagnostic criteria requires one clinical event, i.e. thrombosis or pregnancy complication, and
two positive blood tests spaced at least 3 months apart. These antibodies are: lupus anticoagulant,
anti-cardiolipin and anti-β2-glycoprotein.

Antiphospholipid syndrome can be primary or secondary. Primary antiphospholipid syndrome occurs


in the absence of any other related disease. Secondary antiphospholipid syndrome occurs with other
autoimmune diseases, such as systemic lupus erythematosus (SLE). In rare cases, APS leads to
rapid organ failure due to generalised thrombosis; this is termed "catastrophic antiphospholipid
syndrome" (CAPS) and is associated with a high risk of death.

Antiphospholipid syndrome often requires treatment with anticoagulant medication such as heparin
to reduce the risk of further episodes of thrombosis and improve the prognosis of pregnancy.
Warfarin is not used during pregnancy because it can cross the placenta, unlike heparin, and is
teratogenic.
Please rate this question:
Discuss and give feedback
Next question

Hypercoagulability

Type of thrombophilia Features

Antithrombin deficiency Antithrombin inactivates thrombin and factor XII a, XIa, IXa and Xa
Rare defect, inherited in autosomal dominant fashion
10x increase in risk of thrombotic events
Heparin may be ineffective because it works via antithrombin

Protein C and S These are natural anticoagulants (vitamin K dependent synthesis)


deficiency Protein C produced by liver
Protein S produced by liver, megakaryocytes, Leydig cells and
endothelial cells
Protein C and S bind to form activated complex which binds to factor
V
Deficiency accounts for up to 5% of thrombotic episodes

Factor V Leiden Resistance to anticoagulant effect of activated protein C


May account for up to 20% or more of thrombotic episodes
Prevalence of 7% in Europe
Most common genetic defect accounting for DVT

Antiphospholipid Multi organ disease


syndrome Pregnancy involvement common
Arterial and venous thromboses
Either Lupus anticoagulant or Anti cardiolipin antibodies
APTT usually prolonged
Antibodies may be elevated following surgery, drugs or malignancy
Need anticoagulation with INR between 3 and 4

Next question
Question 12 of 147

A 63 year old Japanese man presents with epigastric discomfort and iron deficiency anaemia. He
undergoes an upper GI endoscopy, where the following appearances are found:

Image sourced from Wikipedia

The most likely diagnosis is:

Squamous cell carcinoma

Linitis plastica

Leiomyosarcoma

Gastric varices

None of the above

During upper GI endoscopy, a linitis plastica lesion may prevent gastric distension.
Linitis plastica produces a diffuse infiltrating lesion, the stomach is fibrotic and rigid and will not
typically distend. This may be described as a 'leather bottle stomach'. Diagnosis is made with a
combination of pathology examination with endoscopy, radiological or surgical assessment.
Pathologically signet-ring cell proliferation occurs.
Please rate this question:

Discuss and give feedback


Next question

Gastric cancer

Overview
There are 700,000 new cases of gastric cancer worldwide each year. It is most common in Japan
and less common in western countries. It is more common in men and incidence rises with
increasing age. The exact cause of many sporadic cancer is not known, however, familial cases do
occur in HNPCC families. In addition, smoking and smoked or preserved foods increase the risk.
Japanese migrants retain their increased risk (decreased in subsequent generations). The
distribution of the disease in western countries is changing towards a more proximal location
(perhaps due to rising obesity).

Pathology
There is some evidence of support a stepwise progression of the disease through intestinal
metaplasia progressing to atrophic gastritis and subsequent dysplasia, through to cancer. The
favoured staging system is TNM. The risk of lymph node involvement is related to size and depth of
invasion; early cancers confined to submucosa have a 20% incidence of lymph node metastasis.
Tumours of the gastro-oesophageal junction are classified as below:

Type True oesophageal cancers and may be associated with Barrett's oesophagus.
1

Type Carcinoma of the cardia, arising from cardiac type epithelium


2 or short segments with intestinal metaplasia at the oesophagogastric junction.

Type Sub cardial cancers that spread across the junction. Involve similar nodal stations to
3 gastric cancer.

Groups for close endoscopic monitoring

 Intestinal metaplasia of columnar type


 Atrophic gastritis
 Low to medium grade dysplasia
 Patients who have previously undergone resections for benign peptic ulcer disease (except
highly selective vagotomy).
Referral to endoscopy

Patients of any age with Patients without Worsening dyspepsia


dyspepsia and any of the dyspepsia
following

Chronic gastrointestinal bleeding Dysphagia Barretts oesophagus

Dysphagia Unexplained abdominal Intestinal metaplasia


pain or weight loss

Weight loss Vomiting Dysplasia

Iron deficiency anaemia Upper abdominal mass Atrophic gastritis

Upper abdominal mass Jaundice Patient aged over 55 years with


unexplained or persistent
dyspepsia

Upper GI endoscopy performed for dyspepsia. The addition of dye spraying (as shown in the bottom
right) may facilitate identification of smaller tumours
Image sourced from Wikipedia

Staging

 CT scanning of the chest abdomen and pelvis is the routine first line staging investigation in
most centres.
 Laparoscopy to identify occult peritoneal disease
 PET CT (particularly for junctional tumours)

Treatment

 Proximally sited disease greater than 5-10cm from the OG junction may be treated by sub
total gastrectomy
 Total gastrectomy if tumour is <5cm from OG junction
 For type 2 junctional tumours (extending into oesophagus) oesophagogastrectomy is usual
 Endoscopic sub mucosal resection may play a role in early gastric cancer confined to the
mucosa and perhaps the sub mucosa (this is debated)
 Lymphadenectomy should be performed. A D2 lymphadenectomy is widely advocated by the
Japanese, the survival advantages of extended lymphadenectomy have been debated.
However, the overall recommendation is that a D2 nodal dissection be undertaken.
 Most patients will receive chemotherapy either pre or post operatively.

Prognosis
UK Data

Disease extent Percentage 5 year survival

All RO resections 54%

Early gastric cancer 91%

Stage 1 87%

Stage 2 65%

Stage 3 18%

Operative procedure

Total Gastrectomy , lymphadenectomy and Roux en Y anastomosis

General anaesthesia
Prophylactic intravenous antibiotics
Incision: Rooftop.
Perform a thorough laparotomy to identify any occult disease.
Mobilise the left lobe of the liver off the diaphragm and place a large pack over it. Insert a large self
retaining retractor e.g. omnitract or Balfour (take time with this, the set up should be perfect). Pack
the small bowel away.
Begin by mobilising the omentum off the transverse colon.
Proceed to detach the short gastric vessels.
Mobilise the pylorus and divide it at least 2cm distally using a linear cutter stapling device.
Continue the dissection into the lesser sac taking the lesser omentum and left gastric artery flush at
its origin.
The lymph nodes should be removed en bloc with the specimen where possible.
Place 2 stay sutures either side of the distal oesophagus. Ask the anaesthetist to pull back on the
nasogastric tube. Divide the distal oesophagus and remove the stomach.
The oesphago jejunal anastomosis should be constructed. Identify the DJ flexure and bring a loop of
jejunum up to the oesophagus (to check it will reach). Divide the jejunum at this point. Bring the
divided jejunum either retrocolic or antecolic to the oesophagus. Anastamose the oesophagus to the
jejunum, using either interrupted 3/0 vicryl or a stapling device. Then create the remainder of the
Roux en Y reconstruction distally.
Place a jejunostomy feeding tube.
Wash out the abdomen and insert drains (usually the anastomosis and duodenal stump). Help the
anaesthetist insert the nasogastric tube (carefully!)
Close the abdomen and skin.
Enteral feeding may commence on the first post-operative day. However, most surgeons will leave
patients on free NG drainage for several days and keep them nil by mouth.
Next question
0/3
Question 13-15 of 147

Theme: Genetics and cancer

A. Multiple endocrine neoplasia type I


B. Multiple endocrine neoplasia type II
C. Gardner's syndrome
D. Lynch Syndrome
E. Kartagener's syndrome
F. Neurofibromatosis Type I
G. Neurofibromatosis Type II

Please select the most likely condition for the disease process described. Each option may be used
once, more than once or not at all.

13. A 40 year old male is found to have multiple colonic polyps during a colonoscopy. He
mentions that he has extra teeth.

You answered Multiple endocrine neoplasia type I

The correct answer is Gardner's syndrome

Gardner's syndrome is an AD disorder, characterised by: Colonic polyps, supernumerary


teeth, jaw osteomas, congenital hypertrophy of retinal pigment. osteomas of the skull,
thyroid cancer, epidermoid cysts, fibromas and sebaceous cysts.

14. A 10 year old boy who has learning difficulties, reports a difference in size between his
two legs.

You answered Multiple endocrine neoplasia type I

The correct answer is Neurofibromatosis Type I

Neurofibromatosis type I. A hallmark finding is a plexiform neurofibroma, which is a


sheet of neurofibromatosis tissue which encases major nerves. In children this attracts
extra blood circulation, which can accelerate growth of the affected limb.
Other features include:
Schwannoma, > 6
Cafe au lait spots, axillary freckling, Lisch nodules, Optic glioma. Meningiomas, Glioma,
or Schwannoma.

15. A 22 year old is found to have bilateral acoustic neuromas.


You answered Multiple endocrine neoplasia type I

The correct answer is Neurofibromatosis Type II

In NF2 bilateral acoustic neuromas are characteristic with a family history of


Neurofibroma,

Please rate this question:

Discuss and give feedback


Next question

Genetics and surgical disease

Some of the more commonly occurring genetic conditions occurring in surgical patients are
presented here.

Li-Fraumeni Syndrome

 Autosomal dominant
 Consists of germline mutations to p53 tumour suppressor gene
 High incidence of malignancies particularly sarcomas and leukaemias
 Diagnosed when:

*Individual develops sarcoma under 45 years


*First degree relative diagnosed with any cancer below age 45 years and another family member
develops malignancy under 45 years or sarcoma at any age

BRCA 1 and 2

 Carried on chromosome 17 (BRCA 1) and Chromosome 13 (BRCA 2)


 Linked to developing breast cancer (60%) risk.
 Associated risk of developing ovarian cancer (55% with BRCA 1 and 25% with BRCA 2).

Lynch Syndrome

 Autosomal dominant
 Develop colonic cancer and endometrial cancer at young age
 80% of affected individuals will get colonic and/ or endometrial cancer
 High risk individuals may be identified using the Amsterdam criteria
Amsterdam criteria
Three or more family members with a confirmed diagnosis of colorectal cancer, one of whom is a
first degree (parent, child, sibling) relative of the other two.
Two successive affected generations.
One or more colon cancers diagnosed under age 50 years.
Familial adenomatous polyposis (FAP) has been excluded.

Gardners syndrome

 Autosomal dominant familial colorectal polyposis


 Multiple colonic polyps
 Extra colonic diseases include: skull osteoma, thyroid cancer and epidermoid cysts
 Desmoid tumours are seen in 15%
 Mutation of APC gene located on chromosome 5
 Due to colonic polyps most patients will undergo colectomy to reduce risk of colorectal
cancer
 Now considered a variant of familial adenomatous polyposis coli

Next question
Question 16 of 147

A 52 year old lady is referred to the breast clinic with symptoms of nipple discharge. The discharge
is usually thick and green. Which of the following statements relating to the most likely underlying
diagnosis is untrue?

The majority of patients will be smokers

Typically produces blood stained nipple discharge

It is not associated with increased risk of breast cancer

May result in development of mammary duct fistula

May require total duct excision (Hadfields operation) if it fails to resolve

Blood stained nipple discharge should always be investigated.


Nipple fluid cytology is generally unhelpful.

Discharge of this type of material is most likely to be due to duct ectasia. Green or brown discharge
is most common. Blood stained discharge should raise concern of intraductal papilloma or cancer.
Please rate this question:

Discuss and give feedback


Next question

Nipple discharge

Causes of nipple discharge


Physiological During breast feeding

Galactorrhoea Commonest cause may be response to emotional events, drugs such as


histamine receptor antagonists are also implicated

Hyperprolactinaemia  Commonest type of pituitary tumour


 Microadenomas <1cm in diameter
 Macroadenomas >1cm in diameter
 Pressure on optic chiasm may cause bitemporal hemianopia

Mammary duct  Dilatation breast ducts.


ectasia  Most common in menopausal women
 Discharge typically thick and green in colour
 Most common in smokers

Carcinoma  Often blood stained


 May be underlying mass or axillary lymphadenopathy

Intraductal papilloma  Commoner in younger patients


 May cause blood stained discharge
 There is usually no palpable lump

Assessment of patients

 Examine breast and determine whether there is mass lesion present


 All mass lesions should undergo Triple assessment.

Reporting of investigations
Where a mass lesion is suspected or investigations are requested these are prefixed using a system
that denotes the investigation type e.g. M for mammography, followed by a numerical code as
shown below:

1 No abnormality

2 Abnormality with benign features

3 Indeterminate probably benign

4 Indeterminate probably malignant

5 Malignant
Management of non malignant nipple discharge

 Exclude endocrine disease


 Nipple cytology unhelpful
 Smoking cessation advice for duct ectasia
 For duct ectasia with severe symptoms, total duct excision may be warranted.

Next question
Question 17 of 147

Which of the following statements relating to gas gangrene is untrue?

There is necrosis with putrefaction

The causative pathogens may be detected on normal perineal skin

Treatment with low dose penicillin is indicated

Hyperbaric oxygen may be beneficial

Clostridium perfringens is a recognised cause

Rapid surgery and high dose antibiotics are indicated in the treatment of gas gangrene.
Please rate this question:

Discuss and give feedback


Next question

Meleney's Gangrene and Necrotising Fasciitis

Necrotising fasciitis

 Advancing soft tissue infection associated with fascial necrosis


 Uncommon, but can be fatal
 In many cases there is underlying background immunosuppression e.g. Diabetes
 Caused by polymicrobial flora (aerobic and anaerobic) and MRSA is seen increasingly in
cases of necrotising fasciitis
 Streptococcus is the commonest organism in isolated pathogen infection (15%)

Meleneys gangrene

 Meleneys is a similar principle but the infection is more superficially sited than necrotising
fasciitis and often confined to the trunk
Fournier gangrene

 Necrotising fasciitis affecting the perineum


 Polymicrobial with E-coli and Bacteroides acting in synergy

Clinical features
Fever
Pain
Cellulitis
Oedema
Induration
Numbness

Late findings

Purple/black skin discolouration


Blistering
Haemorrhagic bullae
Crepitus
Dirty Dishwater fluid discharge
Septic shock

A typical case of gas gangrene presenting late demonstrating some of the features described above

Image sourced from Wikipedia

Diagnosis is mainly clinical

Management
 Radical surgical debridement forms the cornerstone of management
 Sterile dressing is used to dress the wound
 Reconstructive surgery is considered once the infection is completely treated

Reference

Hasham S, Matteucci P, Stanley PR, Hart NB. Necrotising fasciitis. BMJ 2005;330:830-833.
Next question
Question 18 of 147

A 30 year old man presents with abdominal distension, a laparotomy is performed, at operation the
abdomen is filled with a large amount of gelatinous fluid. What is the most likely underlying
diagnosis?

Infection with entamoeba histolytica

Pseudomxyoma peritonei

Metastatic colonic cancer

Chylous ascites

None of the above

Pseudomyxoma is associated with the deposition of large amounts of gelatinous material. The
appendix is the commonest organ or origin.
Please rate this question:

Discuss and give feedback


Next question

Pseudomyxoma Peritonei

 Rare mucinous tumour


 Most commonly arising from the appendix (other abdominal viscera are also recognised as
primary sites)
 Incidence of 1-2/1,000,000 per year
 The disease is characterised by the accumulation of large amounts of mucinous material in
the abdominal cavity

Treatment
Is usually surgical and consists of cytoreductive surgery (and often peritonectomy c.f Sugarbaker
procedure) combined with intra peritoneal chemotherapy with mitomycin C.

Survival is related to the quality of primary treatment and in Sugarbakers own centre 5 year survival
rates of 75% have been quoted. Patients with disseminated intraperitoneal malignancy from another
source fare far worse.
In selected patients a second look laparotomy is advocated and some practice this routinely.
Next question
Question 19 of 147

A 30 year old man is suspected of having appendicitis. At operation an inflamed Meckels


diverticulum is found. Which of the following vessels is responsible for the blood supply to a Meckels
diverticulum?

Right colic artery

Vitelline artery

Appendicular artery

Internal iliac artery

External iliac artery

The vitelline arteries supply a Meckels these are usually derived from the ileal arcades.
Please rate this question:

Discuss and give feedback


Next question

Meckel's diverticulum

 Congenital abnormality resulting in incomplete obliteration of the vitello-intestinal duct


 Normally, in the foetus, there is an attachment between the vitello-intestinal duct and the yolk
sac.This disappears at 6 weeks gestation.
 The tip is free in majority of cases.
 Associated with enterocystomas, umbilical sinuses, and omphaloileal fistulas.
 Arterial supply: omphalomesenteric artery.
 2% of population, 2 inches long, 2 feet from the ileocaecal valve.
 Typically lined by ileal mucosa but ectopic gastric mucosa can occur, with the risk of peptic
ulceration. Pancreatic and jejunal mucosa can also occur.

Clinical

 Normally asymptomatic and an incidental finding.


 Complications are the result of obstruction, ectopic tissue, or inflammation.
 Removal if narrow neck or symptomatic. Options are between wedge excision or formal
small bowel resection and anastomosis.

Next question
Question 20 of 147

Which of the following associations are incorrect?

Afro-Caribbean skin and keloid scarring

Extensive third degree burns and wound contraction

Chemotherapy and dehisence of healed wounds

Poor healing at the site of previous radiotherapy

Zinc deficiency and delayed healing

Please rate this question:

Discuss and give feedback


Next question

Wound healing

Surgical wounds are either incisional or excisional and either clean, clean contaminated or dirty.
Although the stages of wound healing are broadly similar their contributions will vary according to the
wound type.

The main stages of wound healing include:

Haemostasis

 Minutes to hours following injury


 Vasospasm in adjacent vessels, platelet plug formation and generation of fibrin rich clot.

Inflammation

 Typically days 1-5


 Neutrophils migrate into wound (function impaired in diabetes).
 Growth factors released, including basic fibroblast growth factor and vascular endothelial
growth factor.
 Fibroblasts replicate within the adjacent matrix and migrate into wound.
 Macrophages and fibroblasts couple matrix regeneration and clot substitution.

Regeneration

 Typically days 7 to 56
 Platelet derived growth factor and transformation growth factors stimulate fibroblasts and
epithelial cells.
 Fibroblasts produce a collagen network.
 Angiogenesis occurs and wound resembles granulation tissue.

Remodeling

 From 6 weeks to 1 year


 Longest phase of the healing process and may last up to one year (or longer).
 During this phase fibroblasts become differentiated (myofibroblasts) and these facilitate
wound contraction.
 Collagen fibres are remodeled.
 Microvessels regress leaving a pale scar.

The above description represents an idealised scenario. A number of diseases may distort this
process. Neovascularisation is an important early process. Endothelial cells may proliferate in the
wound bed and recanalise to form a vessel. Vascular disease, shock and sepsis can all compromise
microvascular flow and impair healing.

Conditions such as jaundice will impair fibroblast synthetic function and immunity with a detrimental
effect in most parts of the healing process.

Problems with scars:

Hypertrophic scars
Excessive amounts of collagen within a scar. Nodules may be present histologically containing
randomly arranged fibrils within and parallel fibres on the surface. The tissue itself is confined to the
extent of the wound itself and is usually the result of a full thickness dermal injury. They may go on
to develop contractures.

Image of hypertrophic scarring. Note that it remains confined to the boundaries of the original
wound:
Image sourced from Wikipedia

Keloid scars
Excessive amounts of collagen within a scar. Typically a keloid scar will pass beyond the boundaries
of the original injury. They do not contain nodules and may occur following even trivial injury. They
do not regress over time and may recur following removal.

Image of a keloid scar. Note the extension beyond the boundaries of the original incision:

Image sourced from Wikipedia

Drugs which impair wound healing:


 Non steroidal anti inflammatory drugs
 Steroids
 Immunosupressive agents
 Anti neoplastic drugs

Closure
Delayed primary closure is the anatomically precise closure that is delayed for a few days but before
granulation tissue becomes macroscopically evident.

Secondary closure refers to either spontaneous closure or to surgical closure after granulation tissue
has formed.
Next question
Question 21 of 147

A 45 year old women is identified as having a gastric gastro-intestinal stromal tumour. What is the
usual cell of origin of these lesions?

Brunners glands

Interstitial cells of Cajal

Primitive stem cells of the gut wall

Fundic glands

Antral goblet cells

GIST's are derived from the interstitial pacemaker cells of Cajal. This means that they are often
located extramucosally and macroscopically, demonstrate little mucosal disruption.

Please rate this question:

Discuss and give feedback

Next question

Gastrointestinal stromal tumour

GIST's are not common tumours (10 per million) and originate primarily from the interstitial
pacemaker cells (of Cajal). Up to 70% occur in the stomach, the remainder occurring in the small
intestine (20%) and the colon and rectum (5%). Up to 95% are solitary lesions and most are
sporadic. The vast majority express CD117 which is a transmembrane tyrosine kinase receptor and
in these there is a mutation of the c-KIT gene.
The goal of surgery is resection of the tumour with a 1-2cm margin of normal tissue. As a result
extensive resections are not required. Unfortunately there is a high local recurrence rate, the risk of
which is related to site, incomplete resections and high mitotic count. Salvage surgery for recurrent
disease is associated with a median survival of 15 months.

The prognosis in high risk patients is greatly improved through the use of imatinib, which in the
ACOSOG trial (imatinib vs placebo) improved relapse rates from 17% to 2%.
In the UK it is advocated by NICE for use in patients with metastatic disease or locally unresectable
disease.

Next question
Question 22 of 147

A 23 year old man fractures his right tibia in a sporting accident. At which point in the healing
process is fracture callus most likely to be visible radiologically?

1 day

7 days

8 weeks

6 weeks

3 weeks

Fracture callus is composed of fibroblasts and chondroblasts and the synthesis of fibrocartilage. It is
typically visible on radiographs approximately 3 weeks following injury. If delayed then there may be
risk of non union.
Please rate this question:

Discuss and give feedback


Next question

Fracture healing

Bone fracture
- Bleeding vessels in the bone and periosteum
- Clot and haematoma formation
- The clot organises over a week (improved structure and collagen)
- The periosteum contains osteoblasts which produce new bone
- Mesenchymal cells produce cartilage (fibrocartilage and hyaline cartilage) in the soft tissue around
the fracture
- Connective tissue + hyaline cartilage = callus
- As the new bone approaches the new cartilage, endochondral ossification occurs to bridge the gap
- Trabecular bone forms
- Trabecular bone is resorbed by osteoclasts and replaced with compact bone

Factors affecting fracture healing


 Age
 Malnutrition
 Bone disorders: osteoporosis
 Systemic disorders: diabetes, Marfan's syndrome and Ehlers-Danlos syndrome cause
abnormal musculoskeletal healing.
 Drugs: steroids, non steroidal anti inflammatory agents.
 Type of bone: Cancellous (spongy) bone fractures are usually more stable, involve greater
surface areas, and have a better blood supply than cortical (compact) bone fractures.
 Degree of Trauma: The more extensive the injury to bone and surrounding soft tissue, the
poorer the outcome.
 Vascular Injury: Especially the femoral head, talus, and scaphoid bones.
 Degree of Immobilization
 Intra-articular Fractures: These fractures communicate with synovial fluid, which contains
collagenases that retard bone healing.
 Separation of Bone Ends: Normal apposition of fracture fragments is needed for union to
occur. Inadequate reduction, excessive traction, or interposition of soft tissue will prevent
healing.
 Infection

Next question
Question 23 of 147

Of the options below, which does not cause lymphadenopathy?

Kawasaki disease

Systemic Lupus Erthematosus

Phenytoin

Hydrallazine

Amiodarone

Please rate this question:

Discuss and give feedback


Next question

Lymphadenopathy

 Lymphadenopathy in the neck, axillae, groins and abdomen


 Need to note: solitary/multiple, defined/indistinct, hard/rubbery/soft, tender/painless

Causes of lymphadenopathy

Mnemonic: Hodgkins disease

H aematological: Hodgkins lymphoma, NHL, Leukaemia


O ncological: metastases
D ermatopathic lympadenitis
G aucher's disease
K awasaki disease
I nfections: TB, glandular fever, Syphilis
N iemann Pick disease
S erum sickness
D rug reaction (phenytoin)
I mmunological (SLE)
S arcoidosis
E ndocrinological (Hyperthyroidism)
A ngioimmunoplastic lymphadenopathy
S LE
E osinophilic granulomatosis
Next question
Question 24 of 147

A 23 year old man is reviewed on the ward 10 days following a laparotomy. The wound is inspected
and is healing well. Which of the following processes is least likely to be occurring in the wound at
this stage?

Angiogenesis

Synthesis of collagen

Necrosis of fibroblasts

Secretion of matrix metalloproteinases by fibroblasts

Proliferation of fibroblasts

Vasculogenesis vs Angiogenesis

Vascu is new. Angi is pre

Vasculogenesis is new vessels developing in situ from existing mesenchyme.


Angiogenesis is vessels develop from sprouting off pre-existing arteries.

Fibroblasts are an important cell type in healing wounds. They typically proliferate in the early
phases of wound healing. They release matrix metalloproteinases and these facilitate in the
remodelling of the matrix within the healing wound. Necrosis in a healing wound would be unusual
as wounds will tend to show clinical evidence of angiognesis by this time.
Please rate this question:

Discuss and give feedback


Next question

Wound healing

Surgical wounds are either incisional or excisional and either clean, clean contaminated or dirty.
Although the stages of wound healing are broadly similar their contributions will vary according to the
wound type.

The main stages of wound healing include:


Haemostasis

 Minutes to hours following injury


 Vasospasm in adjacent vessels, platelet plug formation and generation of fibrin rich clot.

Inflammation

 Typically days 1-5


 Neutrophils migrate into wound (function impaired in diabetes).
 Growth factors released, including basic fibroblast growth factor and vascular endothelial
growth factor.
 Fibroblasts replicate within the adjacent matrix and migrate into wound.
 Macrophages and fibroblasts couple matrix regeneration and clot substitution.

Regeneration

 Typically days 7 to 56
 Platelet derived growth factor and transformation growth factors stimulate fibroblasts and
epithelial cells.
 Fibroblasts produce a collagen network.
 Angiogenesis occurs and wound resembles granulation tissue.

Remodeling

 From 6 weeks to 1 year


 Longest phase of the healing process and may last up to one year (or longer).
 During this phase fibroblasts become differentiated (myofibroblasts) and these facilitate
wound contraction.
 Collagen fibres are remodeled.
 Microvessels regress leaving a pale scar.

The above description represents an idealised scenario. A number of diseases may distort this
process. Neovascularisation is an important early process. Endothelial cells may proliferate in the
wound bed and recanalise to form a vessel. Vascular disease, shock and sepsis can all compromise
microvascular flow and impair healing.

Conditions such as jaundice will impair fibroblast synthetic function and immunity with a detrimental
effect in most parts of the healing process.

Problems with scars:

Hypertrophic scars
Excessive amounts of collagen within a scar. Nodules may be present histologically containing
randomly arranged fibrils within and parallel fibres on the surface. The tissue itself is confined to the
extent of the wound itself and is usually the result of a full thickness dermal injury. They may go on
to develop contractures.

Image of hypertrophic scarring. Note that it remains confined to the boundaries of the original
wound:

Image sourced from Wikipedia

Keloid scars
Excessive amounts of collagen within a scar. Typically a keloid scar will pass beyond the boundaries
of the original injury. They do not contain nodules and may occur following even trivial injury. They
do not regress over time and may recur following removal.

Image of a keloid scar. Note the extension beyond the boundaries of the original incision:
Image sourced from Wikipedia

Drugs which impair wound healing:

 Non steroidal anti inflammatory drugs


 Steroids
 Immunosupressive agents
 Anti neoplastic drugs

Closure
Delayed primary closure is the anatomically precise closure that is delayed for a few days but before
granulation tissue becomes macroscopically evident.

Secondary closure refers to either spontaneous closure or to surgical closure after granulation tissue
has formed.
Next question
Question 25 of 147

A 25 year old women presents with a slowly enlarging mass on the side of the face. Clinical
examination demonstrates that the mass is located in the tail of the parotid gland. There is no
evidence of facial nerve involvement. What is the most likely cause?

Sialolithiasis

Adenocarcinoma

Warthins tumour

Oncocytoma

Pleomorphic adenoma

Pleomorphic adenomas are the commonest tumours of the parotid gland and are often slow
growing, smooth and mobile. Warthins tumours are typically found in elderly males and are
composed of multiple cysts and solid components consisting of lymphoid tissue. Warthins tumours
are most often found in the tail of the parotid gland, but not in 25 year old females, where a
pleomorphic adenoma remains the most likely lesion.
Please rate this question:

Discuss and give feedback


Next question

Parotid gland clinical

Benign neoplasms
Up to 80% of all salivary gland tumours occur in the parotid gland and up to 80% of these are
benign. There is no consistent correlation between the rate of growth and the malignant potential of
the lesion. However, benign tumours should not invade structures such as the facial nerve.
With the exception of Warthins tumours, they are commoner in women than men. The median age of
developing a lesion is in the 5th decade of life.

Benign tumour types


Tumour type Features
Tumour type Features

Benign pleomorphic adenoma or Most common parotid neoplasm (80%)


benign mixed tumor Proliferation of epithelial and myoepithelial cells of the ducts
and an increase in stromal components
Slow growing, lobular, and not well encapsulated
Recurrence rate of 1-5% with appropriate excision
(parotidectomy)
Recurrence possibly secondary to capsular disruption during
surgery
Malignant degeneration occurring in 2-10% of adenomas
observed for long periods, with carcinoma ex-pleomorphic
adenoma occurring most frequently as adenocarcinoma

Warthin tumor (papillary Second most common benign parotid tumor (5%)
cystadenoma lymphoma or Most common bilateral benign neoplasm of the parotid
adenolymphoma) Marked male as compared to female predominance
Occurs later in life (sixth and seventh decades)
Presents as a lymphocytic infiltrate and cystic epithelial
proliferation
May represent heterotopic salivary gland epithelial tissue
trapped within intraparotid lymph nodes
Incidence of bilaterality and multicentricity of 10%
Malignant transformation rare (almost unheard of)

Monomorphic adenoma Account for less than 5% of tumours


Slow growing
Consist of only one morphological cell type (hence term
mono)
Include; basal cell adenoma, canalicular adenoma,
oncocytoma, myoepitheliomas

Haemangioma Should be considered in the differential of a parotid mass in a


child
Accounts for 90% of parotid tumours in children less than 1
year of age
Hypervascular on imaging
Spontaneous regression may occur and malignant
transformation is almost unheard of

Malignant salivary gland tumours


Types of malignancy

Mucoepidermoid 30% of all parotid malignancies


carcinoma Usually low potential for local invasiveness and metastasis (depends
mainly on grade)

Adenoid cystic Unpredictable growth pattern


carcinoma Tendency for perineural spread
Nerve growth may display skip lesions resulting in incomplete excision
Distant metastasis more common (visceral rather than nodal spread)
5 year survival 35%

Mixed tumours Often a malignancy occurring in a previously benign parotid lesion

Acinic cell carcinoma Intermediate grade malignancy


May show perineural invasion
Low potential for distant metastasis
5 year survival 80%

Adenocarcinoma Develops from secretory portion of gland


Risk of regional nodal and distant metastasis
5 year survival depends upon stage at presentation, may be up to 75%
with small lesions with no nodal involvement

Lymphoma Large rubbery lesion, may occur in association with Warthins tumours
Diagnosis should be based on regional nodal biopsy rather than parotid
resection
Treatment is with chemotherapy (and radiotherapy)

Diagnostic evaluation

 Plain x-rays may be used to exclude calculi


 Sialography may be used to delineate ductal anatomy
 FNAC is used in most cases
 Superficial parotidectomy may be either diagnostic of therapeutic depending upon the nature
of the lesion
 Where malignancy is suspected the primary approach should be definitive resection rather
than excisional biopsy
 CT/ MRI may be used in cases of malignancy for staging primary disease
Treatment
For nearly all lesions this consists of surgical resection, for benign disease this will usually consist of
a superficial parotidectomy. For malignant disease a radical or extended radical parotidectomy is
performed. The facial nerve is included in the resection if involved. The need for neck dissection is
determined by the potential for nodal involvement.

Other parotid disorders


HIV infection

 Lymphoepithelial cysts associated with HIV occur almost exclusively in the parotid
 Typically presents as bilateral, multicystic, symmetrical swelling
 Risk of malignant transformation is low and management usually conservative

Sjogren syndrome

 Autoimmune disorder characterised by parotid enlargement, xerostomia and


keratoconjunctivitis sicca
 90% of cases occur in females
 Second most common connective tissue disorder
 Bilateral, non tender enlargement of the gland is usual
 Histologically, the usual findings are of a lymphocytic infiltrate in acinar units and
epimyoepithelial islands surrounded by lymphoid stroma
 Treatment is supportive
 There is an increased risk of subsequent lymphoma

Sarcoid

 Parotid involvement occurs in 6% of patients with sarcoid


 Bilateral in most cases
 Gland is not tender
 Xerostomia may occur
 Management of isolated parotid disease is usually conservative

Next question
Question 26 of 147

Beta-naphthalamine is associated with which of the following malignancies?

Lung cancer

Bowel cancer

Bladder cancer

Liver cancer

Renal cancer

Beta-naphthalamine is used in the rubber industry.

The following factors are associated with the development of bladder cancer:

 smoking
 occupational: aniline dyes used in printing and textile industry, rubber manufacture
 schistosomiasis
 drugs: cyclophosphamide

Please rate this question:

Discuss and give feedback


Next question

Occupational cancers

Occupational cancers accounted for 5.3% cancer deaths in 2005.


In men the main cancers include:

 Mesothelioma
 Bladder cancer
 Non melanoma skin cancer
 Lung cancer
 Sino nasal cancer

Occupations with high levels of occupational tumours include:

 Construction industry
 Working with coal tar and pitch
 Mining
 Metalworkers
 Working with asbestos (accounts for 98% of all mesotheliomas)
 Working in rubber industry

Shift work has been linked to breast cancer in women (Health and safety executive report RR595).

The latency between exposure and disease is typically 15 years for solid tumours and 20 for
leukaemia.

Many occupational cancers are otherwise rare. For example sino nasal cancer is an uncommon
tumour, 50% will be SCC. They are linked to conditions such as wood dust exposure and unlike lung
cancer is not strongly linked to cigarette smoking. Another typical occupational tumour is
angiosarcoma of the liver which is linked to working with vinyl chloride. Again in the non occupational
context this is an extremely rare sporadic tumour.
Next question
Question 27 of 147

A 56 year old man with Wilsons disease presents with right upper quadrant discomfort. An
ultrasound scan is performed and this demonstrates a mass lesion in the right lobe of the liver. What
is the most appropriate method of establishing the underlying diagnosis?

PET CT scan

Ultrasound guided biopsy

Measurement of serum alpha feto protein

MRI scan of the liver

CT scan of the liver

High AFP + chronic liver inflammation = Hepatocellular carcinoma.

This is likely to be a hepatocellulcar carcinoma. Diagnosis is usually made by AFP measurement


(with further imaging depending on the result). Biopsy should not be performed as it may seed the
tumour. Chronic liver diseases such as Wilsons disease (Hepato-lenticular degeneration) increase
the risk.
Please rate this question:

Discuss and give feedback


Next question

Liver tumours

Primary liver tumours


The most common primary tumours are cholangiocarcinoma and hepatocellular carcinoma. Overall
metastatic disease accounts for 95% of all liver malignancies making the primary liver tumours
comparatively rare.

Primary liver tumours include:

 Cholangiocarcinoma
 Hepatocellular carcinoma
 Hepatoblastoma
 Sarcomas (Rare)
 Lymphomas
 Carcinoids (most often secondary although primary may occur)

Hepatocellular carcinoma
These account for the bulk of primary liver tumours (75% cases). Its worldwide incidence reflects its
propensity to occur on a background of chronic inflammatory activity. Most cases arise in cirrhotic
livers or those with chronic hepatitis B infection, especially where viral replication is actively
occurring. In the UK it accounts for less than 5% of all cancers, although in parts of Asia its
incidence is 100 per 100,000.
The majority of patients (80%) present with existing liver cirrhosis, with a mass discovered on
screening ultrasound.

Diagnosis

 CT/ MRI (usually both) are the imaging modalities of choice


 a-fetoprotein is elevated in almost all cases
 Biopsy should be avoided as it seeds tumours cells through a resection plane.
 In cases of diagnostic doubt serial CT and αFP measurements are the preferred strategy.

Treatment

 Patients should be staged with liver MRI and chest, abdomen and pelvic CT scan.
 The testis should be examined in males (testicular tumours may cause raised AFP). PET CT
may be used to identify occult nodal disease.
 Surgical resection is the mainstay of treatment in operable cases. In patients with a small
primary tumour in a cirrhotic liver whose primary disease process is controlled, consideration
may be given to primary whole liver resection and transplantation.
 Liver resections are an option but since most cases occur in an already diseased liver the
operative risks and post-operative hepatic dysfunction are far greater than is seen following
metastectomy.
 These tumours are not particularly chemo or radiosensitive however, both may be used in a
palliative setting. Tumour ablation is a more popular strategy.

Survival
Poor, overall survival is 15% at 5 years.

Cholangiocarcinoma
This is the second most common type of primary liver malignancy. As its name suggests these
tumours arise in the bile ducts. Up to 80% of tumours arise in the extra hepatic biliary tree. Most
patients present with jaundice and by this stage the majority will have disease that is not resectable.
Primary sclerosing cholangitis is the main risk factor. In deprived countries typhoid and liver flukes
are also major risk factors.

Diagnosis
 Patients will typically have an obstructive picture on liver function tests.
 CA 19-9, CEA and CA 125 are often elevated
 CT/ MRI and MRCP are the imaging methods of choice.

Treatment

 Surgical resection offers the best chance of cure. Local invasion of peri hilar tumours is a
particular problem and this coupled with lobar atrophy will often contra indicate surgical
resection.
 Palliation of jaundice is important, although metallic stents should be avoided in those
considered for resection.

Survival
Is poor, approximately 5-10% 5 year survival.
Next question
Question 28 of 147

A 55 year old man has suffered from reflux oesophagitis for many years. During a recent endoscopy
a biopsy is taken from the distal oesophagus. The histopathology report indicates that cells are
identified with features of coarse chromatin and abnormal mitoses. The cells are confined to the
superficial epithelial layer only. Which of the following accounts for this process?

Metaplasia

Apoptosis

Autoimmune oesophagitis

Dysplasia

Infection with Helicobacter pylori

Dysplasia = pre cancerous

Dysplasia tends to develop as a result of prolonged stimulation by precipitants. Removal of these


precipitants may possibly reverse these changes. Replacement of differentiated cells with another
cell type describes metaplasia rather than dysplasia. The absence of invasion distinguishes this from
malignancy.
Please rate this question:

Discuss and give feedback


Next question

Dysplasia

 Premalignant condition
 Disordered growth and differentiation of cells
 Alteration in size, shape, and organization of cells
 Features increased abnormal cell growth (increased number of mitoses/abnormal mitoses
and cellular differentiation)
 Underlying connective tissue is not invaded
 Causes include smoking, Helicobacter pylori, Human papilloma virus
 Main differences to metaplasia is that dysplasia is considered to be part of carcinogenesis
(pre cancerous) and is associated with a delay in maturation of cells rather than
differentiated cells replacing one another
 The absence of invasion differentiates dysplasia from invasive malignancy
 Severe dysplasia with foci of invasion are well recognised

Next question
Question 29 of 147

Which one of the following confers the least risk of developing osteoporosis?

Obesity

Long term unfractionated heparin therapy

Gastrectomy

Osteogenesis imperfecta

Diabetes

Low body weight is a risk factor for osteoporosis.


Please rate this question:

Discuss and give feedback


Next question

Osteoporosis

Risk factors

 Family history
 Female sex
 Increasing age
 Deficient diet
 Sedentary lifestyle
 Smoking
 Premature menopause
 Low body weight
 Caucasians and Asians

Diseases which predispose


 Endocrine: glucocorticoid excess (e.g. Cushing's, steroid therapy), hyperthyroidism,
hypogonadism (e.g. Turner's, testosterone deficiency), growth hormone deficiency,
hyperparathyroidism, diabetes mellitus
 Multiple myeloma, lymphoma
 Gastrointestinal problems: inflammatory bowel disease, malabsorption (e.g. Coeliacs),
gastrectomy, liver disease
 Rheumatoid arthritis
 Long term heparin therapy
 Chronic renal failure
 Osteogenesis imperfecta, homocystinuria

Next question
Question 30 of 147

A 63 year old man has a history of claudication that has been present for many years. He is recently
evaluated in the clinic and a duplex scan shows that he has an 85% stenosis of the superficial
femoral artery. Two weeks later he presents with a 1 hour history of severe pain in his leg. On
examination he has absent pulses in the affected limb and it is much cooler than the contra-lateral
limb. Which process best accounts for this presentation?

Thrombosis

Embolus

Atheroma growth

Sub intimal dissection

Anaemia

Theme from April 2012 Exam


In an existing lesion a complication such as thrombosis is more likely than embolus. These patients
should receive heparin and imaging with duplex scanning. Whilst an early surgical bypass or intra-
arterial thrombolysis may be indicated, an embolectomy should not generally be performed as the
lesion is not an embolus and the operation therefore ineffective.

Please rate this question:

Discuss and give feedback

Next question

Claudication
Claudication is a condition in which patients develop pain in a limb during periods of exercise. The
underlying disorder is usually that of arterial insufficiency. Atheroma develops in the arterial wall and
once this occludes >50-75% of the lumenal diameter the supply to metabolising tissues distally may
become compromised. The typical claudicant complains of calf pain that is worse on exercise and
relieved by rest. This typical description assumes that the SFA is the site of disease, more proximal
disease may present with other symptoms such as buttock claudication and impotence.

The history is usually a progressive one, patients presenting as an emergency with severe pain,
diminished sensation, pallor and absent pulses have critical limb ischaemia. This may complicate
claudication and usually indicates a plaque related complication, such as thrombosis.

Risk factors
Risk factors for claudication include smoking, diabetes and hyperlipidaemia.

Diagnosis
Diagnostic work -up includes measurement of ankle- brachial pressure indices, duplex scanning and
formal angiography.

Treatment
Those with long claudication distances, no ulceration or gangrene may be managed conservatively.
Patients with rest pain, ulceration or gangrene will almost always require intervention. All patients
should receive an antiplatelet agent and a statin unless there are compelling contra-indications.

Next question
Question 31 of 147

The following are true of carcinoid tumours except:

When present in the appendix tip and measure less than 2 cm have an excellent prognosis

Even when metastatic disease is present it tends to follow a protracted course

When present in the appendix body tend to present with carcinoid syndrome even when
liver metastases are not present

May be imaged using 5 HIAA radionucleotide scanning

Advanced appendiceal carcinoids may require right hemicolectomy

Rule of thirds:

1/3 multiple
1/3 small bowel
1/3 metastasize
1/3 second tumour

Liver metastases are necessary for the presence of carcinoid syndrome.


Please rate this question:

Discuss and give feedback


Next question

Carcinoid syndrome

 Carcinoid tumours secrete serotonin


 Originate in neuroendocrine cells mainly in the intestine (midgut-distal ileum/appendix)
 Can occur in the rectum, bronchi
 Hormonal symptoms mainly occur when disease spreads outside the bowel

Clinical features

 Onset: insidious over many years


 Flushing face
 Palpitations
 Pulmonary valve stenosis and tricuspid regurgitation causing dyspnoea
 Asthma
 Severe diarrhoea (secretory, persists despite fasting)

Investigation

 5-HIAA in a 24-hour urine collection


 Somatostatin receptor scintigraphy
 CT scan
 Blood testing for chromogranin A

Treatment

 Octreotide
 Surgical removal

Next question
Question 32 of 147

During a difficult femoro-popliteal bypass operation the surgeon inadvertently places a clamp across
the femoral nerve. It remains there for most of the procedure. At the end of the operation the nerve is
inspected, it is in continuity but has evidence of being crushed. Which of the following is most likely
to occur over the following months?

Wallerian degeneration

Rapid restoration of neuronal function because the axon itself is intact

Normal but delayed neuronal transmission due to disruption of the myelin

Absence of neuroma formation

None of the above

A neuronal injury such as this will result in Wallerian degeneration even though the nerve remains in
continuity. Neuromas may well form.
Please rate this question:

Discuss and give feedback


Next question

Nerve injury

There are 3 types of nerve injury:


Neuropraxia  Nerve intact but electrical conduction is affected
 Full recovery
 Autonomic function preserved
 Wallerian degeneration does not occur

Axonotmesis  Axon is damaged and the myelin sheath is preserved. The connective tissue
framework is not affected.
 Wallerian degeneration occurs.
Neurotmesis  Disruption of the axon, myelin sheath and surrounding connective tissue.
 Wallerian degeneration occurs.

Wallerian Degeneration

 Axonal degeneration distal to the site of injury.


 Typically begins 24-36 hours following injury.
 Axons are excitable prior to degeneration occurring.
 Myelin sheath degenerates and is phagocytosed by tissue macrophages.

Nerve repair

 Neuronal repair may only occur physiologically where nerves are in direct contact. Where a
large defect is present, the process of nerve regeneration is hampered. It may not occur at
all or result in the formation of a neuroma. Where nerve regrowth occurs it is typically at a
rate of 1mm per day.

Next question
Question 33 of 147

A 38 year old lady who smokes heavily presents with recurrent episodes of infection in the right
breast. On examination she has an indurated area at the lateral aspect of the nipple areaolar
complex. Imaging shows no mass lesions. What is the most likely diagnosis?

Duct ectasia

Periductal mastitis

Pagets disease of the nipple

Mondors disease of the breast

Radial scar

Periductal mastitis is common in smokers and may present with recurrent infections. Treatment is
with co-amoxyclav. Mondors disease of the breast is a localised thrombophlebitis of a breast vein.

Please rate this question:

Discuss and give feedback

Next question

Duct ectasia

Duct ectasia is a dilatation and shortening of the terminal breast ducts within 3cm of the nipple. It is
common and the incidence increases with age. It typically presents with nipple retraction and
occasionally creamy nipple discharge. It may be confused with periductal mastitis, which presents in
younger women, the vast majority of which are smokers. Periductal mastitis typically presents with
periareolar or sub areolar infections and may be recurrent.
Patients with troublesome nipple discharge may be treated by microdochectomy (if young) or total
duct excision (if older).

Next question
Question 34 of 147

Which of the following statements relating to chronic inflammation is true?

Chronic inflammation is mainly secondary to acute inflammation

Neutrophils are the predominant cells involved

Growth factors are not involved in the process

Appendicitis is mainly a form of chronic inflammation

Fibrosis is a macroscopic feature

Macroscopic features include:

 Ulcers
 Fibrosis
 Granulomatous process

It most commonly occurs as a primary event rather than as a result of acute inflammation.
Please rate this question:

Discuss and give feedback


Next question

Chronic inflammation

Overview
Chronic inflammation may occur secondary to acute inflammation.In most cases chronic
inflammation occurs as a primary process. These may be broadly viewed as being one of three main
processes:

 Persisting infection with certain organisms such as Mycobacterium tuberculosis which results
in delayed type hypersensitivity reactions and inflammation.
 Prolonged exposure to non-biodegradable substances such as silica or suture materials
which may induce an inflammatory response.
 Autoimmune conditions involving antibodies formed against host antigens.

Acute vs. Chronic inflammation


Acute inflammation Chronic inflammation

Changes to existing vascular structure and increased Angiogenesis predominates


permeability of endothelial cells

Infiltration of neutrophils Macrophages, plasma cells and


lymphocytes predominate

Process may resolve with: Healing by fibrosis is the main result

 Suppuration
 Complete resolution
 Abscess formation
 Progression to chronic inflammation
 Healing by fibrosis

Granulomatous inflammation
A granuloma consists of a microscopic aggregation of macrophages (with epithelial type
arrangement =epitheliod). Large giant cells may be found at the periphery of granulomas.

Mediators
Growth factors released by activated macrophages include agents such as interferon and fibroblast
growth factor (plus many more). Some of these such as interferons may have systemic features
resulting in systemic symptoms and signs, which may be present in individuals with long standing
chronic inflammation.

The finding of granulomas is pathognomonic of chronic inflammation, as illustrated in this biopsy


from a patient with colonic Crohns disease
Image sourced from Wikipedia

Next question
1/3

Question 35-37 of 147

Theme: Paediatric ano-rectal disorders

A. Ulcerative colitis

B. Juvenile polyps

C. Haemorroids

D. Intussceception

E. Rectal cancer

F. Anal fissure

G. Arteriovenous malformation

Please select the most likely cause for the condition described. Each option may be used once,
more than once or not at all.

35. A 4 year old boy is brought to the clinic. He gives a history of difficult, painful defecation with
bright red rectal bleeding.

You answered Ulcerative colitis

The correct answer is Anal fissure

Theme from April 2012 Exam


Painful rectal bleeding in this age group is typically due to a fissure. Treatment should include
stool softeners and lifestyle advice.

36. A 2 year old has a history of rectal bleeding. The parents notice that post defecation, a cherry red
lesion is present at the anal verge.

You answered Ulcerative colitis


The correct answer is Juvenile polyps

Theme from September 2012 Exam


These lesions are usually hamartomas and this accounts for the colour of the lesions. Although
the lesions are not themselves malignant they serve as a marker of an underlying polyposis
disorder.

37. A 12 year old is brought to the colorectal clinic with a history of rectal bleeding, altered bowel
habit, weight loss and malaise. Abdominal examination is normal.

Ulcerative colitis

The systemic features in the history are strongly suggestive of inflammatory bowel disease rather
than the other causes.

Please rate this question:

Discuss and give feedback

Next question

Paediatric proctology

Children may present with altered bowel habit and/ or rectal bleeding. Classical haemorroidal
disease is relatively rare in children. Painful bright red rectal bleeding is much more common since
constipation is a relatively common childhood disorder. The hard stool causes a tear of the ano-
rectal mucosa with subsequent fissure. The pain from the fissure must be addressed promptly or the
child will delay defecation and this fissure will worsen.

Inflammatory bowel disease may present in a similar pattern in paediatric practice with altered bowel
habit (usually diarrhoea) and bleeding. Systemic features may be present and investigation with an
endoscopy may be required.

Children with intussceception usually present at a relatively young age and the history is usually one
of colicky abdominal pain, together with a mass on clinical examination.The often cited red current
jelly type stool is a rare but classical feature.
Juvenile polyps may occur as part of the familial polyposis coli syndromes. The lesions, which are
hamartomas, are often cherry red if they protrude externally.

Next question
Question 38 of 147

You review a 42-year-old woman 8 months following a renal transplant for focal segmental
glomerulosclerosis. She is on a combination of tacrolimus, mycophenolate, and prednisolone. She
has now presented with a five day history of feeling generally unwell with jaundice, fatigue and
arthralgia. On examination she has jaundice, widespread lymphadenopathy and hepatomegaly.
What is the most likely diagnosis?

Hepatitis C

Epstein-Barr virus

HIV

Hepatitis B

Cytomegalovirus

Post transplant complications

CMV: 4 weeks to 6 months post transplant


EBV: post transplant lymphoproliferative disease. > 6 months post transplant

Post transplant lymphoproliferative disorder is most commonly associated with Epstein-Barr virus. It
typically occurs 6 months post transplant and is associated with high dose immunosupressant
therapy. Remember cytomegalovirus presents within the first 4 weeks to 6 months post transplant.
Please rate this question:

Discuss and give feedback


Next question

Renal transplant:HLA typing and graft failure

The human leucocyte antigen (HLA) system is the name given to the major histocompatibility
complex (MHC) in humans. It is coded for on chromosome 6.

Some basic points on the HLA system


 Class 1 antigens include A, B and C. Class 2 antigens include DP,DQ and DR
 When HLA matching for a renal transplant the relative importance of the HLA antigens are as
follows DR > B > A

Graft survival

 1 year = 90%, 10 years = 60% for cadaveric transplants


 1 year = 95%, 10 years = 70% for living-donor transplants

Post-op problems

 ATN of graft
 Vascular thrombosis
 Urine leakage
 UTI

Hyperacute acute rejection

 Due to antibodies against donor HLA type 1 antigens


 Rarely seen due to HLA matching

Acute graft failure (< 6 months)

 Usually due to mismatched HLA


 Other causes include cytomegalovirus infection
 Management: give steroids, if resistant use monoclonal antibodies

Causes of chronic graft failure (> 6 months)

 Chronic allograft nephropathy


 Ureteric obstruction
 Recurrence of original renal disease (MCGN > IgA > FSGS)

Next question
Question 39 of 147

Which of the following processes facilitates phagocytosis?

Apoptosis

Opsonisation

Proteolysis

Angiogenesis

Necrosis

Theme from September 2014 Exam


Opsonisation will facilitate phagocytosis. The micro-organism becomes coated with antibody, C3b
and certain acute phase proteins. The macrophages and neutrophils have up regulation of
phagocytic cell surface receptors in these circumstances, a process mediated by pro-inflammatory
cytokines. These cells then engulf the micro organism.
Please rate this question:

Discuss and give feedback


Next question

Phagocytosis

 Ingestion of pathogens or foreign materials by cells


 First step is opsonisation whereby the organism is coated by antibody
 Second step is adhesion to cell surface
 Third step is pseudopodial extension to form a phagocytic vacuole
 Lysosomes fuse with vacuole and degrade contents

Next question
Question 40 of 147

Features which are evaluated for the grading of breast cancer include all the following, except:

Tubule formation

Mitoses

Nuclear pleomorphism

Tumour necrosis

Coarse chromatin

The necrosis of a tumour may be suggestive of a high grade tumour which has out grown its blood
supply. However, the grading of breast cancer which classically follows the Bloom -Richardson
grading model will tend to favor nuclear appearances (which include mitoses, coarse chromatin and
pleomorphism). Tubule formation is an important marker of the degree of differentiation with
formation of tubular structures being associated with well differentiated tumours.

Please rate this question:

Discuss and give feedback

Next question

Tumour grading

Tumours may be graded according to their degree of differentiation, mitotic activity and other
features. Grade 1 tumours are the most differentiated and grade 3 or 4 the least. The assessment is
subjective, in most cases high grade equates to poor prognosis, or at least rapid growth.
Tumours of glandular epithelium will tend to arrange themselves into acinar type structures
containing a central lumen. Well differentiated tumours may show excellent acinar formation and
poorly differentiated tumours simply clumps of cells around a desmoplastic stroma. Sometimes
tumours demonstrate mucous production without evidence of acinar formation. Since mucous
production is evidence of a glandular function such tumours are often termed mucinous
adenocarcinoma.
Squamous cell tumours will typically produce structures resembling epithelial cell components. Well
differentiated tumours may also produce keratin (depending upon tissue of origin).

Next question
Question 41 of 147

A 34 year old man is diagnosed with an aggressive caecal adenocarcinoma. His sister died from the
same disease at 38 years of age. His mother died from endometrial cancer at the age of 41. What is
the most likely underlying abnormality?

Familial adenopolypomatosis coli

Gardeners syndrome

Mutation of mismatch repair genes

Deletion of chromosome 6

MYH gene mutation

Lynch syndrome which is characterised by aggressive right sided colonic malignancy and
endometrial cancer is caused by microsatellite instability of DNA repair genes.

Please rate this question:

Discuss and give feedback

Next question

Genetics of colorectal cancer

The lifetime risk of colorectal cancer in the UK population is 5%. Up to 5% of newly diagnosed bowel
cancers will be in those individuals who have a high genetically acquired risk of bowel cancer.
Cancers arising in the low-moderate genetic risk group comprise approximately 30% of newly
diagnosed bowel cancer.
Genetics of inherited colorectal cancer syndromes

Syndrome Features Genes implicated

FAP More than 100 adenomatous polyps affecting the colon and APC (over 90%)
rectum. Duodenal and fundic glandular polyps

Gardner As FAP but with desmoid tumours and mandibular osteomas APC
syndrome

Turcots syndrome Polyposis and colonic tumours and CNS tumours APC +MLH1 and
PMS2

HNPCC Colorectal cancer without extensive polyposis. Endometrial MSH2, MLH1, PMS2
cancer, renal and CNS and GTBP

Peutz-Jeghers Hamartomatous polyps in GI tract and increased risk of GI LKB1 andSTK11 (in
syndrome malignancy up to 70%)

Cowden disease Multiple hamartomas (see below) PTEN (85%)

MYH associated Autosomal recessive, multiple adenomatous polyps in GI tract, MYH


polyposis those in colon having somatic KRAS mutations

FAP
Autosomal dominant condition, affects 1 in 12,000. Accounts for 0.5% of all CRCs. Lifetime
incidence of colorectal cancer in untreated FAP =100%. Up to 25% cases are caused by de-novo
germ line mutations and show no prior family history. The APC tumour suppressor gene is affected
in most cases.

APC in non inherited colorectal cancer


Up to 80% of sporadic colorectal cancers will have somatic mutations that inactivate APC[1]. Both
alleles are usually affected. Although the APC protein more than likely has multiple critical cellular
functions, the best-established role for APC in the cancer process is as a major binding partner and
regulator of the β- catenin protein in the so-called canonical or β- catenin dependent Wnt signaling
pathway.

HNPCC (Lynch syndrome)


HNPCC cancers differ from conventional tumours in a number of respects. In the colon the tumours
are more likely to be right sided, histologically they are more likely to be mucinous and have dense
lymphocytic infiltrates. To be diagnosed as having HNPCC individuals must show typically HNPCC
tumours in at least three individuals, (one of whom must be a first degree relative to the other two).
In at least two successive generations. At least one cancer must be diagnosed under the age of 50.
FAP must be excluded and tumours should be verified by pathological identification (Amsterdam
criteria). The genetic changes in HNPCC stem primarily from microsatellite instability affecting DNA
mismatch repair genes. In HNPCC the mismatch repair genes most commonly implicated include;
MSH2 and MLH1 and these occur in up to 70% of people with HNPCC. The finding of microsatellite
instability is unusual in sporadic colorectal cancers. Approximately 60% of individuals who fulfill the
Amsterdam criteria will not be found to have evidence of mismatch repair gene defects on genetic
testing. The risk of developing colorectal cancer in those who have not demonstrated mutation of the
mis match repair genes is increased if they fulfill the Amsterdam criteria, but not
the extent that it is increased in those who fulfill the criteria AND have evidence of mis match repair
gene defects.

KRAS Mutations
The RAS family of small G proteins act as molecular switches downstream of growth factor
receptors. KRAS and the other two members of the family; HRAS and NRAS, are the site of
mutation in approximately 40% of colorectal cancers. When adenomas are examined the proportion
of adenomas less than 1cm showing KRAS mutations was only 10% which contrasts with 50% in
those lesions greater than 1cm.

p53 mutations
The p53 protein functions as a key transcriptional regulator of genes that encode proteins with
functions in cell-cycle checkpoints at the G1/S and G2/M boundaries, in promoting apoptosis, and in
restricting angiogenesis . As such, selection for p53 defects at the adenoma-carcinoma transition
may reflect the fact that stresses on tumor cells activate cell-cycle arrest, apoptotic, and
antiangiogenic pathways in cells with wild-type p53 function. Many colonic tumours will demonstrate
changes in the p53 gene that may facilitate tumour progression through from adenoma to
carcinoma.

Cowden syndrome
Also known as multiple hamartoma syndrome. Rare autosomal dominant condition with incidence of
1 in 200,000.. It is characterised by multiple mucocutaneous lesions, trichilemmomas, oral
papillomas and acral keratosis. Most often diagnosed in third decade of life. Breast carcinoma may
occur in up to 50% of patients and conditions such as fibrocystic disease of the breast may occur in
75% of women. Thyroid disease occurs in 75% and may include malignancy. Endoscopic screening
will identify disease in up to 85% although the small bowel is rarely involved. There is a 15-20% risk
of developing colorectal cancer and regular colonoscopic screening from age 45 is recommended.

Terminology
Oncogene Oncogenes are genes which have the potential to induce cellular proliferation and avoid
apoptosis. Oncogene mutations are general gain of function and are therefore
dominant. Increased expression of oncogenes are found in most tumours

Tumour These genes generally inhibit cellular proliferation or induce apoptosis. Mutations in
suppressor tumour suppressor genes are generally loss of function mutations, and are therefore
gene recessive. Mutations in both tumour suppressor gene alleles allow cells to proliferate
without restraint

References
1. Fearon, E.R. and B. Vogelstein, A genetic model for colorectal tumorigenesis. Cell, 1990. 61(5): p.
759-67.

Next question
Question 42 of 147

Which of the following is associated with poor wound healing?

Jaundice

Patients taking carbamazepine

General anaesthesia using thiopentone

General anaesthesia using ketamine

Multiple sclerosis

Mnemonic to remember factors affecting wound healing: DID NOT HEAL

D iabetes
I nfection, irradiation
D rugs eg steroids, chemotherapy

N utritional deficiencies (vitamin A, C & zinc, manganese), Neoplasia


O bject (foreign material)
T issue necrosis

H ypoxia
E xcess tension on wound
A nother wound
L ow temperature, Liver jaundice

Multiple sclerosis is associated with pressure sores, however the cellular healing process is not
affected.
Please rate this question:

Discuss and give feedback


Next question

Wound healing

Surgical wounds are either incisional or excisional and either clean, clean contaminated or dirty.
Although the stages of wound healing are broadly similar their contributions will vary according to the
wound type.

The main stages of wound healing include:

Haemostasis

 Minutes to hours following injury


 Vasospasm in adjacent vessels, platelet plug formation and generation of fibrin rich clot.

Inflammation

 Typically days 1-5


 Neutrophils migrate into wound (function impaired in diabetes).
 Growth factors released, including basic fibroblast growth factor and vascular endothelial
growth factor.
 Fibroblasts replicate within the adjacent matrix and migrate into wound.
 Macrophages and fibroblasts couple matrix regeneration and clot substitution.

Regeneration

 Typically days 7 to 56
 Platelet derived growth factor and transformation growth factors stimulate fibroblasts and
epithelial cells.
 Fibroblasts produce a collagen network.
 Angiogenesis occurs and wound resembles granulation tissue.

Remodeling

 From 6 weeks to 1 year


 Longest phase of the healing process and may last up to one year (or longer).
 During this phase fibroblasts become differentiated (myofibroblasts) and these facilitate
wound contraction.
 Collagen fibres are remodeled.
 Microvessels regress leaving a pale scar.

The above description represents an idealised scenario. A number of diseases may distort this
process. Neovascularisation is an important early process. Endothelial cells may proliferate in the
wound bed and recanalise to form a vessel. Vascular disease, shock and sepsis can all compromise
microvascular flow and impair healing.

Conditions such as jaundice will impair fibroblast synthetic function and immunity with a detrimental
effect in most parts of the healing process.

Problems with scars:


Hypertrophic scars
Excessive amounts of collagen within a scar. Nodules may be present histologically containing
randomly arranged fibrils within and parallel fibres on the surface. The tissue itself is confined to the
extent of the wound itself and is usually the result of a full thickness dermal injury. They may go on
to develop contractures.

Image of hypertrophic scarring. Note that it remains confined to the boundaries of the original
wound:

Image sourced from Wikipedia

Keloid scars
Excessive amounts of collagen within a scar. Typically a keloid scar will pass beyond the boundaries
of the original injury. They do not contain nodules and may occur following even trivial injury. They
do not regress over time and may recur following removal.

Image of a keloid scar. Note the extension beyond the boundaries of the original incision:
Image sourced from Wikipedia

Drugs which impair wound healing:

 Non steroidal anti inflammatory drugs


 Steroids
 Immunosupressive agents
 Anti neoplastic drugs

Closure
Delayed primary closure is the anatomically precise closure that is delayed for a few days but before
granulation tissue becomes macroscopically evident.

Secondary closure refers to either spontaneous closure or to surgical closure after granulation tissue
has formed.
Next question
Question 43 of 147

A 55 year old man undergoes a colonoscopy and a colonic polyp is identified. It has a lobular
appearance and is located on a stalk in the sigmoid colon. Which of the processes below best
accounts for this disease?

Apoptosis

Metaplasia

Dysplasia

Calcification

Degeneration

Theme from April 2012 Exam


Most colonic polyps described above are adenomas. These may have associated dysplasia. The
more high grade the dysplasia the greater the level of clinical concern.

Please rate this question:

Discuss and give feedback

Next question

Colonic polyps

Colonic Polyps
May occur in isolation, or greater numbers as part of the polyposis syndromes. In FAP greater than
100 polyps are typically present. The risk of malignancy in association with adenomas is related to
size, and is the order of 10% in a 1cm adenoma. Isolated adenomas seldom give risk of symptoms
(unless large and distal). Distally sited villous lesions may produce mucous and if very large,
electrolyte disturbances may occur.

Follow up of colonic polyps

Group Features Action

Low risk 1 or 2 adenomas less than 1cm No follow up or re-colonoscopy


at 5 years

Moderate 3 or 4 small adenomas or 1 adenoma greater than 1cm Re-scope at 3 years


risk

High risk More than 5 small adenomas or more than 3 with 1 of Re scope at 1 year
them greater than 1cm

From Atkins and Saunders Gut 2002 51 (suppl V:V6-V9). It is important to stratify patients
appropriately and ensure that a complete colonoscopy with good views was performed.

Segmental resection or complete colectomy should be considered when:

1. Incomplete excision of malignant polyp


2. Malignant sessile polyp
3. Malignant pedunculated polyp with submucosal invasion
4. Polyps with poorly differentiated carcinoma
5. Familial polyposis coli
-Screening from teenager up to 40 years by 2 yearly sigmoidoscopy/colonoscopy
-Panproctocolectomy and Ileostomy or Restorative Panproctocolectomy.

Rectal polypoidal lesions may be amenable to trans anal endoscopic microsurgery.

References
Cairns S et al. Guidelines for colorectal cancer screening and surveillance in moderate and high risk
groups (update from 2002). Gut 2010;59:666-690.

Next question
Question 44 of 147

A 56 year old lady has just undergone a colonoscopy and a 1.5cm lesion was identified in the
caecum. The histology report states that biopsies have been taken from a sessile serrated polyp with
traditional features. What is the best management option?

Perform a right hemicolectomy

List the patient for colonoscopic polypectomy

Discharge the patient

Re scope the patient in 6 months

Re scope the patient at 3 years

These polyps represent an alternative pathway to progression to carcinoma and may be


diagnostically confused with hyperplastic polyps. Hyperplastic polyps are more common in the left
colon and confer no increased risk. SSA's are more common in the right colon and are usually
larger. Those with "traditional features" on histology have dysplasia with increased risk of malignant
transformation.

Please rate this question:

Discuss and give feedback

Next question

Colonic polyps

Colonic Polyps
May occur in isolation, or greater numbers as part of the polyposis syndromes. In FAP greater than
100 polyps are typically present. The risk of malignancy in association with adenomas is related to
size, and is the order of 10% in a 1cm adenoma. Isolated adenomas seldom give risk of symptoms
(unless large and distal). Distally sited villous lesions may produce mucous and if very large,
electrolyte disturbances may occur.

Follow up of colonic polyps

Group Features Action

Low risk 1 or 2 adenomas less than 1cm No follow up or re-colonoscopy


at 5 years

Moderate 3 or 4 small adenomas or 1 adenoma greater than 1cm Re-scope at 3 years


risk

High risk More than 5 small adenomas or more than 3 with 1 of Re scope at 1 year
them greater than 1cm

From Atkins and Saunders Gut 2002 51 (suppl V:V6-V9). It is important to stratify patients
appropriately and ensure that a complete colonoscopy with good views was performed.

Segmental resection or complete colectomy should be considered when:

1. Incomplete excision of malignant polyp


2. Malignant sessile polyp
3. Malignant pedunculated polyp with submucosal invasion
4. Polyps with poorly differentiated carcinoma
5. Familial polyposis coli
-Screening from teenager up to 40 years by 2 yearly sigmoidoscopy/colonoscopy
-Panproctocolectomy and Ileostomy or Restorative Panproctocolectomy.

Rectal polypoidal lesions may be amenable to trans anal endoscopic microsurgery.

References
Cairns S et al. Guidelines for colorectal cancer screening and surveillance in moderate and high risk
groups (update from 2002). Gut 2010;59:666-690.

Next question
Question 45 of 147

A 30 year old male presents with gynaecomastia. Clinically, he is noted to have a nodule in the left
testis. What is the most likely diagnosis?

Oestrogen abuse

Seminoma with syncytiotrophoblast giant cells

Teratoma

Choriocarcinoma

Leydig cell tumour

Leydig cell tumours are rare testicular sex cord stromal tumours (which also include sertoli cell
tumours) which are associated with hormonal activity.

 Patients with Leydig cell tumours may present with gynaecomastia before they notice
testicular enlargement.
 Majority are benign
 Histology: eosinophilic cells in columns

Please rate this question:

Discuss and give feedback


Next question

Testicular disorders

Testicular cancer
Testicular cancer is the most common malignancy in men aged 20-30 years. Around 95% of cases
of testicular cancer are germ-cell tumours. Germ cell tumours may essentially be divided into:

Tumour
Tumour type Key features markers Pathology
Tumour
Tumour type Key features markers Pathology

Seminoma  Commonest  AFP usually Sheet like lobular


subtype (50%) normal patterns of cells
 Average age at  HCG elevated with substantial
diagnosis = 40 in 10% fibrous
 Even advanced seminomas component.
disease associated  Lactate Fibrous septa
with 5 year dehydrogenase; contain
survival of 73% elevated in 10- lymphocytic
20% seminomas inclusions and
(but also in many granulomas may
other conditions) be seen.

Non seminomatous germ  Younger age at  AFP elevated Heterogenous


cell tumours (42%) presentation =20-30 in up to 70% of texture with
years cases occasional ectopic
 Advanced disease  HCG elevated tissue such as hair
 Teratoma carries worse prognosis in up to 40% of
 Yolk sac tumour (48% at 5 years) cases
 Choriocarcinoma  Retroperitoneal lymph  Other markers
 Mixed germ cell node dissection may be rarely helpful
tumours (10%) needed for residual
disease after
chemotherapy

Image demonstrating a classical seminoma, these tumours are typically more uniform than
teratomas
Image sourced from Wikipedia

Risk factors for testicular cancer

 Cryptorchidism
 Infertility
 Family history
 Klinefelter's syndrome
 Mumps orchitis

Features
 A painless lump is the most common presenting symptom
 Pain may also be present in a minority of men
 Other possible features include hydrocele, gynaecomastia

Diagnosis

 Ultrasound is first-line
 CT scanning of the chest/ abdomen and pelvis is used for staging
 Tumour markers (see above) should be measured

Management

 Orchidectomy (Inguinal approach)


 Chemotherapy and radiotherapy may be given depending on staging
 Abdominal lesions >1cm following chemotherapy may require retroperitoneal lymph node
dissection.

Prognosis is generally excellent

 5 year survival for seminomas is around 95% if Stage I


 5 year survival for teratomas is around 85% if Stage I

Benign disease

Epididymo-orchitis
Acute epididymitis is an acute inflammation of the epididymis, often involving the testis and usually
caused by bacterial infection.

 Infection spreads from the urethra or bladder. In men <35 years, gonorrhoea or chlamydia
are the usual infections.
 Amiodarone is a recognised non infective cause of epididymitis, which resolves on stopping
the drug.
 Tenderness is usually confined to the epididymis, which may facilitate differentiating it from
torsion where pain usually affects the entire testis.

Testicular torsion

 Twist of the spermatic cord resulting in testicular ischaemia and necrosis.


 Most common in males aged between 10 and 30 (peak incidence 13-15 years)
 Pain is usually severe and of sudden onset.
 Cremasteric reflex is lost and elevation of the testis does not ease the pain.
 Treatment is with surgical exploration. If a torted testis is identified then both testis should be
fixed as the condition of bell clapper testis is often bilateral.

Hydrocele

 Presents as a mass that transilluminates, usually possible to "get above" it on examination.


 In younger men it should be investigated with USS to exclude tumour.
 In children it may occur as a result of a patent processus vaginalis.
 Treatment in adults is with a Lords or Jabouley procedure.
 Treatment in children is with trans inguinal ligation of PPV.

Next question
Question 46 of 147

What is the most common cause of osteolytic bone metastasis in children?

Osteosarcoma

Neuroblastoma

Leukaemia

Rhabdomyosarcoma

Nephroblastoma

Neuroblastomas are a relatively common childhood tumour and have a strong tendency to
developing widespread lytic metastasis.
Please rate this question:

Discuss and give feedback


Next question

Secondary malignant tumours of bone

Metastatic lesions affecting bone are more common than primary bone tumours.

The typical tumours that spread to bone include:

 Breast
 Bronchus
 Renal
 Thyroid
 Prostate

75% cases will affect those over the age of 50

The commonest bone sites affected are:


 Vertebrae (usually thoracic)
 Proximal femur
 Ribs
 Sternum
 Pelvis
 Skull

Pathological fracture
Osteolytic lesions are the greatest risk for pathological fracture
The risk and load required to produce fracture varies according to bone site. Bones with lesions that
occupy 50% or less will be prone to fracture under loading (Harrington). When 75% of the bone is
affected the process of torsion about a bony fulcrum may produce a fracture.

The Mirel scoring[1] system may be used to help determine the risk of fracture and is more
systematic than the Harrington system described above.

Mirel Scoring system

Score Radiographic Width of bone


points Site appearance involved Pain

1 Upper extremity Blastic Less than 1/3 Mild

2 Lower Mixed 1/3 to 2/3 Moderate


extremity

3 Peritrochanteric Lytic More than 2/3 Aggravated by


function

Depending upon the score the treatment should be as follows:

Score Risk of fracture Treatment

9 or greater Impending (33%) Prophylactic fixation

8 Borderline Consider fixation

7 or less Not impending (4%) Non operative management


Where the lesion is an isolated metastatic deposit consideration should be given to excision and
reconstruction as the outcome is better [2].

Non operative treatments


Hypercalcaemia- Treat with re hydration and bisphosphonates.
Pain- Opiate analgesics and radiotherapy.
Some tumours such as breast and prostate will benefit from chemotherapy and or hormonal agents.

References
1. Mirels, H. Metastatic disease in long bones. A proposed scoring system for diagnosing impending
pathologic fractures. Clin Orthop Relat Res, 1989(249): p. 256-64.
2. Mavrogenis, A.F. et al. Survival analysis of patients with femoral metastases. J Surg Oncol, 2011.
Next question
Question 47 of 147

Which of the following features are not typical of Crohns disease?

Complex fistula in ano

Small bowel strictures

Skip lesions

'Rose thorn ulcers' on barium studies

Pseudopolyps on colonoscopy

Pseudopolyps are a feature of ulcerative colitis and occur when there is severe mucosal ulceration.
The remaining islands of mucosa may then appear to be isolated and almost polypoidal.
Please rate this question:

Discuss and give feedback


Next question

Crohns disease

Crohns disease is a chronic transmural inflammation of a segment(s) of the gastrointestinal tract and
may be associated with extra intestinal manifestations. Frequent disease patterns observed include
ileal, ileocolic and colonic disease. Peri-anal disease may occur in association with any of these. The
disease is often discontinuous in its distribution. Inflammation may cause ulceration, fissures, fistulas
and fibrosis with stricturing. Histology reveals a chronic inflammatory infiltrate that is usually patchy
and transmural.

Ulcerative colitis Vs Crohns


Crohn's disease Ulcerative colitis

Distribution Mouth to anus Rectum and colon


Crohn's disease Ulcerative colitis

Macroscopic Cobblestone appearance, apthoid ulceration Contact bleeding


changes

Depth of disease Transmural inflammation Superficial inflammation

Distribution Patchy Continuous


pattern

Histological Granulomas (non caseating epithelioid cell Crypt abscesses, Inflammatory


features aggregates with Langerhans' giant cells) cells in the lamina propria

Extraintestinal manifestations of Crohns


Related to disease extent Unrelated to disease extent

Aphthous ulcers (10%) Sacroiliiitis (10-15%)

Erythema nodosum (5-10%) Ankylosing spondylitis (1-2%)

Pyoderma gangrenosum (0.5%) Primary sclerosing cholangitis (Rare)

Acute arthropathy (6-12%) Gallstones (up to 30%)

Ocular complications (up to 10%) Renal calculi (up to 10%)

Diarrhoea in Crohns
Diarrhoea in Crohns may be multifactorial since actual inflammation of the colon is not common.
Causes therefore include the following:

 Bile salt diarrhoea secondary to terminal ileal disease


 Entero-colic fistula
 Short bowel due to multiple resections
 Bacterial overgrowth
Surgical interventions in Crohns disease
The commonest disease pattern in Crohns is stricturing terminal ileal disease and this often
culminates in an ileocaecal resection. Other procedures performed include segmental small bowel
resections and stricturoplasty. Colonic involvement in patients with Crohns is not common and,
where found, distribution is often segmental. However, despite this distribution segmental resections
of the colon in patients with Crohns disease are generally not advocated because the recurrence
rate in the remaining colon is extremely high, as a result the standard options of colonic surgery in
Crohns patients are generally; sub total colectomy, panproctocolectomy and staged sub total
colectomy and proctectomy. Restorative procedures such as ileoanal pouch have no role in therapy.
Crohns disease is notorious for the developmental of intestinal fistulae; these may form between the
rectum and skin (peri anal) or the small bowel and skin. Fistulation between loops of bowel may also
occur and result in bacterial overgrowth and malabsorption. Management of enterocutaneous
fistulae involves controlling sepsis, optimising nutrition, imaging the disease and planning definitive
surgical management.
Next question
Question 48 of 147

At which of the following sites is the development of diverticulosis least likely?

Caecum

Ascending colon

Transverse colon

Sigmoid colon

Rectum

Rectal involvement with diverticular disease almost never occurs.

Because the rectum has a circular muscle coat (blending of of the tenia marks the recto-sigmoid
junction), diverticular disease almost never occurs here. Right sided colonic diverticular disease is
well recognised (though less common than left sided).
Please rate this question:

Discuss and give feedback


Next question

Diverticular disease

Diverticular disease is a common surgical problem. It consists of herniation of colonic mucosa


through the muscular wall of the colon. The usual site is between the taenia coli where vessels
pierce the muscle to supply the mucosa. For this reason, the rectum, which lacks taenia, is often
spared.

Symptoms

 Altered bowel habit


 Bleeding
 Abdominal pain
Complications

 Diverticulitis
 Haemorrhage
 Development of fistula
 Perforation and faecal peritonitis
 Perforation and development of abscess
 Development of diverticular phlegmon

Diagnosis
Patients presenting in clinic will typically undergo either a colonoscopy, CT cologram or barium
enema as part of their diagnostic work up. All tests can identify diverticular disease. It can be far
more difficult to confidently exclude cancer, particularly in diverticular strictures.

Acutely unwell surgical patients should be investigated in a systematic way. Plain abdominal films
and an erect chest x-ray will identify perforation. An abdominal CT scan (not a CT cologram) with
oral and intravenous contrast will help to identify whether acute inflammation is present but also the
presence of local complications such as abscess formation.

Severity Classification- Hinchey

I Para-colonic abscess

II Pelvic abscess

III Purulent peritonitis

IV Faecal peritonitis

Treatment

 Increase dietary fibre intake.


 Mild attacks of diverticulitis may be managed conservatively with antibiotics.
 Peri colonic abscesses should be drained either surgically or radiologically.
 Recurrent episodes of acute diverticulitis requiring hospitalisation are a relative indication for
a segmental resection.
 Hinchey IV perforations (generalised faecal peritonitis) will require a resection and usually a
stoma. This group have a very high risk of post operative complications and usually require
HDU admission. Less severe perforations may be managed by laparoscopic washout and
drain insertion.
Question 49 of 147

A 32 year old lady presents with a 1.5cm pigmented lesion on her back. The surgeon is concerned
that this may be a melanoma. What is the most appropriate course of action?

2mm punch biopsy from the centre of the lesion

4mm punch biopsy from the centre of the lesion

Wide excision of the lesion with 3cm margins

Excisional biopsy of the lesion

Wide excision of the lesion with 1cm margins

Suspicious naevi should NOT be partially sampled as histological interpretation is severely


compromised. Complete excision is mandatory where lesions fulfil diagnostic criteria. However, wide
excision for margins may be deferred until definitive histology is available.

Lesions that are suspicious for melanoma should be excised with complete margins. Radical
excision is not routinely undertaken for diagnostic purposes and therefore if subsequent
histopathological assessment determines that the lesion is a melanoma a re-exicision of margins
may be required. Incisional punch biopsies of potential melanomas makes histological interpretation
difficult and is best avoided.
Please rate this question:

Discuss and give feedback


Next question

Malignant melanoma

The main diagnostic features (major criteria): Secondary features (minor criteria)

 Change in size  Diameter >6mm


 Change in shape  Inflammation
 Change in colour  Oozing or bleeding
 Altered sensation

Treatment

 Suspicious lesions should undergo excision biopsy. The lesion should be removed in
completely as incision biopsy can make subsequent histopathological assessment difficult.
 Once the diagnosis is confirmed the pathology report should be reviewed to determine
whether further re-excision of margins is required (see below):

Margins of excision-Related to Breslow thickness


Lesions 0-1mm thick 1cm

Lesions 1-2mm thick 1- 2cm (Depending upon site and pathological features)

Lesions 2-4mm thick 2-3 cm (Depending upon site and pathological features)

Lesions >4 mm thick 3cm

Marsden J et al Revised UK guidelines for management of Melanoma. Br J Dermatol 2010 163:238-


256.

Further treatments such as sentinel lymph node mapping, isolated limb perfusion and block
dissection of regional lymph node groups should be selectively applied.
Next question
Question 50 of 147

A 50 year old male presents with painless frank haematuria. Clinical examination is unremarkable.
Routine blood tests reveal a haemoglobin of 18g/dl but are otherwise normal. What is the most likely
underlying diagnosis?

Squamous cell carcinoma of the bladder

Adenocarcinoma of the prostate

Adenocarcinoma of the kidney

Wilms tumour

Transitional cell carcinoma of the renal pelvis

Theme from April 2012 Exam


Polycythaemia is a recognised feature of renal cell carcinoma. Wilms tumours most commonly occur
in children.
Please rate this question:

Discuss and give feedback


Next question

Haematuria

Causes of haematuria

Trauma  Injury to renal tract


 Renal trauma commonly due to blunt injury (others penetrating
injuries)
 Ureter trauma rare: iatrogenic
 Bladder trauma: due to RTA or pelvic fractures

Infection  Remember TB
Malignancy  Renal cell carcinoma (remember paraneoplastic syndromes):
painful or painless
 Urothelial malignancies: 90% are transitional cell carcinoma, can
occur anywhere along the urinary tract. Painless haematuria.
 Squamous cell carcinoma and adenocarcinoma: rare bladder
tumours
 Prostate cancer
 Penile cancers: SCC

Renal disease  Glomerulonephritis

Stones  Microscopic haematuria common

Structural  Benign prostatic hyperplasia (BPH) causes haematuria due to


abnormalities hypervascularity of the prostate gland
 Cystic renal lesions e.g. polycystic kidney disease
 Vascular malformations
 Renal vein thrombosis due to renal cell carcinoma

Coagulopathy  Causes bleeding of underlying lesions

Drugs  Cause tubular necrosis or interstitial nephritis: aminoglycosides,


chemotherapy
 Interstitial nephritis: penicillin, sulphonamides, and NSAIDs
 Anticoagulants

Benign  Exercise

Gynaecological  Endometriosis: flank pain, dysuria, and haematuria that is cyclical

Iatrogenic  Catheterisation
 Radiotherapy; cystitis, severe haemorrhage, bladder necrosis

Pseudohaematuria For example following consumption of beetroot

References
Http://bestpractice.bmj.com/best-practice/monograph/316/overview/aetiology.html
Next question
1/3
Question 51-53 of 147

Theme: Thyroid neoplasms

A. Follicular carcinoma
B. Follicular adenoma
C. Papillary carcinoma
D. Papillary adenoma
E. Anaplastic carcinoma
F. Medullary carcinoma

Please select the most likely underlying diagnosis for the thyroid masses described. Each option
may be used once, more than once or not at all.

51. A 78 year old lady presents to the surgical clinic with symptoms of both dysphagia and
dyspnoea. On examination there is a large mass in the neck that moves on swallowing. CT
scanning of the neck shows a locally infiltrative lesion arising from the thyroid and
invading the strap muscles.

You answered Follicular carcinoma

The correct answer is Anaplastic carcinoma

Marked local invasion is a feature of anaplastic carcinoma. These tumours are more
common in elderly females.

52. A 25 year old female presents with a lump in her neck. On examination she has a discrete
nodule in the right lobe of the thyroid. A fine needle aspirate shows papillary cells. An
adjacent nodule is also sampled which shows similar well differentiated papillary cells.

You answered Follicular adenoma

The correct answer is Papillary carcinoma

Multifocal disease is a recognised feature of papillary lesions. Papillary adenomas are not
really recognised and most well differentiated lesions are papillary carcinomas.

53. A 45 year old man presents with a fracture of his right humerus. On examination there is a
lytic lesion of the proximal humerus and a mass in the neck, this moves on swallowing.

Follicular carcinoma
Follicular carcinomas are a recognised source of bone metastasis. Up to 60% will show
vascular invasion histologically.

Please rate this question:

Discuss and give feedback


Next question

Thyroid neoplasms

Lesion Common features

Follicular  Usually present as a solitary thyroid nodule


adenoma  Malignancy can only be excluded on formal histological assessment

Papillary  Usually contain a mixture of papillary and colloidal filled follicles


carcinoma  Histologically tumour has papillary projections and pale empty nuclei
 Seldom encapsulated
 Lymph node metastasis predominate
 Haematogenous metastasis rare
 Account for 60% of thyroid cancers

Follicular  May appear macroscopically encapsulated, microscopically capsular


carcinoma invasion is seen. Without this finding the lesion is a follicular adenoma.
 Vascular invasion predominates
 Multifocal disease rare
 Account for 20% of all thyroid cancers

Anaplastic  Most common in elderly females


carcinoma  Local invasion is a common feature
 Account for 10% of thyroid cancers
 Treatment is by resection where possible, palliation may be achieved
through isthmusectomy and radiotherapy. Chemotherapy is ineffective.

Medullary  Tumours of the parafollicular cells (C Cells)


carcinoma  C cells derived from neural crest and not thyroid tissue
 Serum calcitonin levels often raised
 Familial genetic disease accounts for up to 20% cases
 Both lymphatic and haematogenous metastasis are recognised, nodal
Lesion Common features

disease is associated with a very poor prognosis.

Next question
Question 54 of 147

Which of these lesions is most closely associated with Barretts oesophagus?

Squamous cell carcinoma

Gastro intestinal stromal tumours

Carcinoid tumours

Leiomyosarcoma

Adenocarcinoma

Theme from September 2014 Exam


Barretts oesophagus is most closely associated with adenocarcinoma, and it confers a 30 fold
increased risk of developing the condition.
Please rate this question:

Discuss and give feedback


Next question

Oesophageal cancer

 Incidence is increasing
 In most cases in the Western world this increase is accounted for by a rise in the number of
cases of adenocarcinoma. In the UK adenocarcinomas account for 65% of cases.
 Barretts oesophagus is a major risk factor for most cases of oesophageal adenocarcinoma.
 In other regions of the world squamous cancer is more common and is linked to smoking,
alcohol intake, diets rich in nitrosamines and achalasia.
 Surveillance of Barretts is important, as it imparts a 30 fold increase in cancer risk and if
invasive malignancy is diagnosed early then survival may approach 85% at 5 years.

Diagnosis

 Upper GI endoscopy is the first line test


 Contrast swallow may be of benefit in classifying benign motility disorders but has no place
in the assessment of tumours
 Staging is initially undertaken with CT scanning of the chest, abdomen and pelvis. If overt
metastatic disease is identified using this modality then further complex imaging is
unnecessary
 If CT does not show metastatic disease, then local stage may be more accurately assessed
by use of endoscopic ultrasound.
 Staging laparoscopy is performed to detect occult peritoneal disease. PET CT is performed
in those with negative laparoscopy. Thoracoscopy is not routinely performed.

Treatment
Operable disease is best managed by surgical resection. The most standard procedure is an Ivor-
Lewis type oesophagectomy. This procedure involves the mobilisation of the stomach and division of
the oesophageal hiatus. The abdomen is closed and a right sided thoracotomy performed. The
stomach is brought into the chest and the oesophagus mobilised further. An intrathoracic
oesophagogastric anastomosis is constructed. Alternative surgical strategies include a transhiatal
resection (for distal lesions), a left thoraco-abdominal resection (difficult access due to thoracic
aorta) and a total oesophagectomy (McKeown) with a cervical oesophagogastric anastomosis.
The biggest surgical challenge is that of anastomotic leak, with an intrathoracic anastomosis this will
result in mediastinitis. With high mortality. The McKeown technique has an intrinsically lower
systemic insult in the event of anastomotic leakage.

In addition to surgical resection many patients will be treated with adjuvant chemotherapy.
Next question
Question 55 of 147

A 56 year old man presents with epigastric discomfort and episodes of migratory thrombophlebitis.
On examination he is mildly jaundiced. A CT scan shows peri hilar lymphadenopathy and a mass in
the pancreatic head. Which of the following is the most likely underlying diagnosis?

Squamous cell carcinoma of the pancreas

Adenocarcinoma of the pancreas

Insulinoma

Glucagonoma

Gastrinoma

Adenocarcinoma of the pancreas is the most likely diagnosis and migratory thrombophlebitis is
associated with the condition. Squamous cells carcinoma is extremely uncommon in the pancreas.
Gastrinoma are extremely rare and thus not the most likely diagnosis.
Please rate this question:

Discuss and give feedback


Next question

Pancreatic cancer

 Adenocarcinoma
 Risk factors: Smoking, diabetes, adenoma, familial adenomatous polyposis
 Mainly occur in the head of the pancreas (70%)
 Spread locally and metastasizes to the liver
 Carcinoma of the pancreas should be differentiated from other periampullary tumours with
better prognosis

Clinical features

 Weight loss
 Painless jaundice
 Epigastric discomfort (pain usually due to invasion of the coeliac plexus is a late feature)
 Pancreatitis
 Trousseau's sign: migratory superficial thrombophlebitis

Investigations

 USS: May miss small lesions


 CT Scanning (pancreatic protocol). If unresectable on CT then no further staging needed
 PET/CT for those with operable disease on CT alone
 ERCP/ MRI for bile duct assessment
 Staging laparoscopy to exclude peritoneal disease

Management

 Head of pancreas: Whipple's resection (SE dumping and ulcers). Newer techniques include
pylorus preservation and SMA/ SMV resection
 Carcinoma body and tail: poor prognosis, distal pancreatectomy, if operable
 Usually adjuvent chemotherapy for resectable disease
 ERCP and stent for jaundice and palliation
 Surgical bypass may be needed for duodenal obstruction

Next question
Question 56 of 147

What is the urinary diagnostic marker for carcinoid syndrome?

B-HCG

Histamine

Chromogranin A

5-Hydroxyindoleacetic acid

5-Hydroxytryptamine

Urinary measurement of 5- HIAA is an important part of clinical follow up.


Please rate this question:

Discuss and give feedback


Next question

Carcinoid syndrome

 Carcinoid tumours secrete serotonin


 Originate in neuroendocrine cells mainly in the intestine (midgut-distal ileum/appendix)
 Can occur in the rectum, bronchi
 Hormonal symptoms mainly occur when disease spreads outside the bowel

Clinical features

 Onset: insidious over many years


 Flushing face
 Palpitations
 Pulmonary valve stenosis and tricuspid regurgitation causing dyspnoea
 Asthma
 Severe diarrhoea (secretory, persists despite fasting)
Investigation

 5-HIAA in a 24-hour urine collection


 Somatostatin receptor scintigraphy
 CT scan
 Blood testing for chromogranin A

Treatment

 Octreotide
 Surgical removal

Next question
Question 57 of 147

A 22 year old man presents with symptoms of headache, lethargy and confusion. On examination he
is febrile and has a right sided weakness. A CT scan shows a ring enhancing lesion affecting the left
motor cortex. Which of the following is the most likely diagnosis?

Arteriovenous malformation

Cerebral abscess

Herpes simplex encephalitis

Metastatic renal adenocarcinoma

Glioblastoma multiforme

The combination of rapidly progressive neurology, fever and headache is highly suggestive of
cerebral abscess. CT scanning will show a ring enhancing lesion because the intravenous contrast
cannot penetrate the centre of the abscess cavity. HSV encephalitis does not produce ring
enhancing lesions.
Please rate this question:

Discuss and give feedback


Next question

Brain abscess

 CNS abscesses may result from a number of causes including, extension of sepsis from
middle ear or sinuses, trauma or surgery to the scalp, penetrating head injuries and embolic
events from endocarditis.
 The presenting symptoms will depend upon the site of the abscess (those in critical areas
e.g.motor cortex) will present earlier. Abscesses have a considerable mass effect in the brain
and raised intra cranial pressure is common.
 Although fever, headache and focal neurology are highly suggestive of a brain abscess the
absence of one or more of these does not exclude the diagnosis, fever may be absent and
even if present, is usually not the swinging pyrexia seen with abscesses at other sites.
 Assessment of the patient includes imaging with CT scanning.
 Treatment is usually surgical, a craniotomy is performed and the abscess cavity debrided.
The abscess may reform because the head is closed following abscess drainage.
Question 58 of 147

A 56 year old man from Egypt has suffered from recurrent attacks of haematuria for many years. He
presents with suprapubic discomfort and at cystoscopy is found to have a mass lesion within the
bladder. What is the most likely diagnosis?

Squamous cell carcinoma

Transitional cell papilloma

Adenocarcinoma

Leiomyosarcoma

Rhabdomyosarcoma

Theme from April 2016 exam


In Egypt, Schistosomiasis is more common than in the UK and may cause recurrent episodes of
haematuria. In those affected with the condition who develop a bladder neoplasm, an SCC is the
most common type.
Please rate this question:

Discuss and give feedback


Next question

Bladder cancer

Bladder cancer is the second most common urological cancer. It most commonly affects males aged
between 50 and 80 years of age. Those who are current, or previous (within 20 years), smokers
have a 2-5 fold increased risk of the disease. Exposure to hydrocarbons such as 2-Naphthylamine
increases the risk. Although rare in the UK, chronic bladder inflammation arising from
Schistosomiasis infection remains a common cause of squamous cell carcinomas, in those countries
where the disease is endemic.

Benign tumours
Benign tumours of the bladder including inverted urothelial papilloma and nephrogenic adenoma are
uncommon.

Bladder malignancies
 Transitional cell carcinoma (>90% of cases)
 Squamous cell carcinoma ( 1-7% -except in regions affected by schistosomiasis)
 Adenocarcinoma (2%)

Transitional cell carcinomas may arise as solitary lesions, or may be multifocal, owing to the effect of
"field change" within the urothelium. Up to 70% of TCC's will have a papillary growth pattern. These
tumours are usually superficial in location and accordingly have a better prognosis. The remaining
tumours show either mixed papillary and solid growth or pure solid growths. These tumours are
typically more prone to local invasion and may be of higher grade, the prognosis is therefore worse.
Those with T3 disease or worse have a 30% (or higher) risk of regional or distant lymph node
metastasis.

TNM Staging
Stage Description

T0 No evidence of tumour

Ta Non invasive papillary carcinoma

T1 Tumour invades sub epithelial connective tissue

T2a Tumor invades superficial muscularis propria (inner half)

T2b Tumor invades deep muscularis propria (outer half)

T3 Tumour extends to perivesical fat

T4 Tumor invades any of the following: prostatic stroma, seminal vesicles, uterus, vagina

T4a Invasion of uterus, prostate or bowel

T4b Invasion of pelvic sidewall or abdominal wall

N0 No nodal disease
Stage Description

N1 Single regional lymph node metastasis in the true pelvis (hypogastric, obturator, external
iliac, or presacral lymph node)

N2 Multiple regional lymph node metastasis in the true pelvis (hypogastric, obturator, external
iliac, or presacral lymph node metastasis)

N3 Lymph node metastasis to the common iliac lymph nodes

M0 No distant metastasis

M1 Distant disease

Presentation
Most patients (85%) will present with painless, macroscopic haematuria. In those patients with
incidental microscopic haematuria, up to 10% of females aged over 50 will be found to have a
malignancy (once infection excluded).

Staging
Most will undergo a cystoscopy and biopsies or TURBT, this provides histological diagnosis and
information relating to depth of invasion. Locoregional spread is best determined using pelvic MRI
and distant disease CT scanning. Nodes of uncertain significance may be investigated using PET
CT.

Treatment
Those with superficial lesions may be managed using TURBT in isolation. Those with recurrences or
higher grade/ risk on histology may be offered intravesical chemotherapy. Those with T2 disease are
usually offered either surgery (radical cystectomy and ileal conduit) or radical radiotherapy.

Prognosis
T1 90%

T2 60%

T3 35%
T4a 10-25%

Any T, N1-N2 30%

Next question
Question 59 of 147

A newborn infant is noted to have a unilateral cleft lip only. What is the most likely explanation for
this process?

Incomplete fusion of the second branchial arch

Incomplete fusion of the nasolabial muscle rings

Incomplete fusion of the first branchial arch

Incomplete fusion of the third branchial arch

Incomplete fusion of the secondary palate

Theme from April 2012 Exam


Unilateral isolated cleft lip represents a failure of nasolabial ring fusion. It is not related to branchial
arch fusion. Arch disorders have a far more profound phenotype and malformation sequences.
Please rate this question:

Discuss and give feedback


Next question

Cleft lip and palate

Cleft lip and palate are the most common congenital deformity affecting the orofacial structures.
Whilst they may be an isolated developmental malformation they are also a recognised component
of more than 200 birth defects. The incidence is as high as 1 in 600 live births. The commonest
variants are:

 Isolated cleft lip (15%)


 Isolated cleft palate (40%)
 Combined cleft lip and palate (45%)

The aetiology of the disorder is multifactorial; both genetic (affected first degree relative increases
risk) and environmental factors play a role.

Cleft lip
Cleft lip occurs as a result of disruption of the muscles of the upper lip and nasolabial region. These
muscles comprise a chain of muscles viz; nasolabial, bilabial and labiomental. Defects may be
unilateral or bilateral.

Cleft palate
The primary palate consists of all anatomical structures anterior to the incisive foramen. The
secondary palate lies more posteriorly and is sub divided into the hard and soft palate. Cleft palate
occurs as a result of non fusion of the two palatine shelves. Both hard and soft palate may be
involved. Complete cases are associated with complete separation of the nasal septum and vomer
from the palatine processes.

Treatment
Surgical reconstruction is the mainstay of management. The procedures are planned according to
the extent of malformation and child age. Simple defects are managed as a single procedure.
Complex malformations are usually corrected in stages. Affected individuals have a higher incidence
of hearing and speech problems.
Next question
Question 60 of 147

A 72 year old woman with back pain and chronic renal failure has the following blood test results:

Reference range

Ca2+ 2.03 2.15-2.55 mmol/l

Parathyroid hormone 10.4 1-6.5 pmol/l

Phosphate 0.80 0.6-1.25 mmol/l

What is the most likely diagnosis?

Hypoparathyroidism

Primary hyperparathyroidism

Secondary hyperparathyroidism

Tertiary hyperparathyroidism

Pseudohypoparathyroidism

In relation to secondary hyperparathyroidism; there is a HIGH PTH and the Ca 2+ is NORMAL or


LOW. In secondary hyperparathyroidism there is hyperplasia of the parathyroid glands in response
to chronic hypocalcaemia (or hyperphosphataemia) and is a normal physiological response. Calcium
is released from bone, kidneys and the gastrointestinal system.
Please rate this question:

Discuss and give feedback


Next question

Parathyroid glands and disorders of calcium metabolism


Hyperparathyroidism
Disease type Hormone profile Clinical features Cause

Primary  PTH (Elevated)  May be Most cases due to


hyperparathyroidism  Ca2+(Elevated) asymptomatic if solitary adenoma
 Phosphate (Low) mild (80%), multifocal
 Urine calcium :  Recurrent disease occurs in 10-
creatinine clearance abdominal pain 15% and parathyroid
ratio > 0.01 (pancreatitis, renal carcinoma in 1% or
colic) less
 Changes to
emotional or
cognitive state

Secondary  PTH (Elevated)  May have few Parathyroid gland


hyperparathyroidism  Ca2+ (Low or symptoms hyperplasia occurs as
normal)  Eventually may a result of low
 Phosphate develop bone calcium, almost
(Elevated) disease, osteitis always in a setting of
 Vitamin D levels fibrosa cystica and chronic renal failure
(Low) soft tissue
calcifications

Tertiary  Ca2+(Normal or  Metastatic Occurs as a result of


hyperparathyroidism high) calcification ongoing hyperplasia
 PTH (Elevated)  Bone pain and / of the parathyroid
 Phosphate levels or fracture glands after
(Decreased or  Nephrolithiasis correction of
Normal)  Pancreatitis underlying renal
 Vitamin D (Normal disorder, hyperplasia
or decreased) of all 4 glands is
 Alkaline usually the cause
phosphatase
(Elevated)

Differential diagnoses
It is important to consider the rare but relatively benign condition of benign familial hypocalciuric
hypercalcaemia, caused by an autosomal dominant genetic disorder. Diagnosis is usually made by
genetic testing and concordant biochemistry (urine calcium : creatinine clearance ratio <0.01-
distinguished from primary hyperparathyroidism).

Treatment

Primary hyperparathyroidism
Indications for surgery

 Elevated serum Calcium > 1mg/dL above normal


 Hypercalciuria > 400mg/day
 Creatinine clearance < 30% compared with normal
 Episode of life threatening hypercalcaemia
 Nephrolithiasis
 Age < 50 years
 Neuromuscular symptoms
 Reduction in bone mineral density of the femoral neck, lumbar spine, or distal radius of more
than 2.5 standard deviations below peak bone mass (T score lower than -2.5)

Secondary hyperparathyroidism
Usually managed with medical therapy.

Indications for surgery in secondary (renal) hyperparathyroidism:

 Bone pain
 Persistent pruritus
 Soft tissue calcifications

Tertiary hyperparathyroidism
Allow 12 months to elapse following transplant as many cases will resolve
The presence of an autonomously functioning parathyroid gland may require surgery. If the culprit
gland can be identified then it should be excised. Otherwise total parathyroidectomy and re-
implantation of part of the gland may be required.

References
1. Pitt S et al. Secondary and Tertiary Hyperparathyroidism, State of the Art Surgical
Management.Surg Clin North Am 2009 Oct;89(5):1227-39.

2. MacKenzie-Feder J et al. Primary Hyperparathyroidism: An Overview. Int J Endocrinol 2011;


2011: 251410.
Next question
1/3
Question 61-63 of 147

Theme: Pharyngitis

A. Infectious mononucleosis
B. Acute bacterial tonsillitis
C. Quinsy
D. Lymphoma
E. Diptheria

Please select the most likely underlying cause for the following patients presenting with pharyngitis.
Each option may be used once, more than once or not at all.

61. An 8 year old child presents with enlarged tonsils that meet in the midline and are covered
with a white film that bleeds when you attempt to remove it. He is pyrexial but otherwise
well.

You answered Infectious mononucleosis

The correct answer is Acute bacterial tonsillitis

Theme from April 2012 Exam


In acute tonsillitis the tonsils will often meet in the midline and may be covered with a
membrane. Individuals who are systemically well are unlikely to have diptheria.

62. A 10 year old child presents with enlarged tonsils that meet in the midline. Oropharyngeal
examination confirms this finding and you also notice peticheal haemorrhages affecting
the oropharynx. On systemic examination he is noted to have splenomegaly.

Infectious mononucleosis

A combination of pharyngitis and tonsillitis is often seen in glandular fever. Antibiotics


containing penicillin may produce a rash when given in this situation, leading to a
mistaken label of allergy.

63. A 19 year old man has had a sore throat for the past 5 days. Over the past 24 hours he has
notices increasing and severe throbbing pain in the region of his right tonsil. He is pyrexial
and on examination he is noted to have a swelling of this area.

You answered Infectious mononucleosis

The correct answer is Quinsy


Unilateral swelling and fever is usually indicative of quinsy. Surgical drainage usually
produces prompt resolution of symptoms.

Please rate this question:

Discuss and give feedback


Next question

Acute tonsillitis

 Characterised by pharyngitis, fever, malaise and lymphadenopathy.


 Over half of all cases are bacterial with Streptococcus pyogenes the most common organism
 The tonsils are typically oedematous and yellow or white pustules may be present
 Infectious mononucleosis may mimic the condition.
 Treatment with penicillin type antibiotics is indicated for bacterial tonsillitis.
 Bacterial tonsillitis may result in local abscess formation (quinsy)

Acute streptococcal tonsillitis


Image sourced from Wikipedia

Next question
Question 64 of 147

A 4 year old boy is brought to the clinic by his mother who has noticed a small lesion at the external
angle of his eye. On examination there is a small cystic structure which has obviously been recently
infected. On removal of the scab, there is hair visible within the lesion. What is the most likely
diagnosis?

Dermoid cyst

Desmoid cyst

Sebaceous cyst

Epidermoid cyst

Keratoacanthoma

Dermoid cysts occur at sites of embryonic fusion and may contain multiple cell types. They occur
most often in children.

The lesion is unlikely to be a desmoid cyst as these are seldom located either at this site or in this
age group. In addition they do not contain hair. Sebaceous cysts will usually have a punctum and
contain a cheesy material. Epidermoid cysts contain keratin plugs.

Please rate this question:

Discuss and give feedback

Next question

Dermoid cysts
A cutaneous dermoid cyst may develop at sites of embryonic developmental fusion. They are most
common in the midline of the neck, external angle of the eye and posterior to the pinna of the ear.
They typically have multiple inclusions such as hair follicles that bud out from its walls. They may
develop at other sites such as the ovary and in these sites are synonymous with teratomas.

A desmoid tumour is a completely different entity and may be classified either as low grade
fibrosarcomas or non aggressive fibrous tumours. They commonly present as large infiltrative
masses. They may be divided into abdominal, extra abdominal and intra abdominal. All types share
the same biological features. Extra abdominal desmoids have an equal sex distribution and primarily
arise in the musculature of the shoulder, chest wall, back and thigh. Abdominal desmoids usually
arise in the musculoaponeurotic structures of the abdominal wall. Intra abdominal desmoids tend to
occur in the mesentery or pelvic side walls and occur most frequently in patients with familial
adenomatous polyposis coli syndrome.

Next question
Question 65 of 147

Which of the following tumours are most likely to give rise to para-aortic nodal metastasis early?

Ovarian

Bladder

Rectal

Caecal

Cervical

Theme from 2009 Exam

Ovarian tumours are supplied by the ovarian vessels, these branch directly from the aorta. The
cervix drains to the internal and external iliac nodes.
Please rate this question:

Discuss and give feedback


Next question

Para-aortic lymphadenopathy

Organ sites that may metastasise (early) to the para-aortic lymph nodes:

 Testis
 Ovary
 Uterine fundus

Many other organs may result in para-aortic nodal disease. However, these deposits will represent a
much later stage of the disease, since other nodal stations are involved earlier.
Next question
Question 66 of 147

Which of the following statements relating to malignant mesothelioma is false?

It may be treated by extrapleural pneumonectomy.

It is linked to asbestos exposure.

It is linked to cigarette smoking independent of asbestos exposure.

It may occur intra abdominally.

It is relatively resistant to radiotherapy

It is not linked to cigarette smoking. When identified at an early stage a radical resection is the
favored option. Radiotherapy is often given perioperatively. However, it is not a particularly
radiosensitive tumour. Combination chemotherapy gives some of the best results and most regimes
are cisplatin based.
Please rate this question:

Discuss and give feedback


Next question

Occupational cancers

Occupational cancers accounted for 5.3% cancer deaths in 2005.


In men the main cancers include:

 Mesothelioma
 Bladder cancer
 Non melanoma skin cancer
 Lung cancer
 Sino nasal cancer

Occupations with high levels of occupational tumours include:


 Construction industry
 Working with coal tar and pitch
 Mining
 Metalworkers
 Working with asbestos (accounts for 98% of all mesotheliomas)
 Working in rubber industry

Shift work has been linked to breast cancer in women (Health and safety executive report RR595).

The latency between exposure and disease is typically 15 years for solid tumours and 20 for
leukaemia.

Many occupational cancers are otherwise rare. For example sino nasal cancer is an uncommon
tumour, 50% will be SCC. They are linked to conditions such as wood dust exposure and unlike lung
cancer is not strongly linked to cigarette smoking. Another typical occupational tumour is
angiosarcoma of the liver which is linked to working with vinyl chloride. Again in the non occupational
context this is an extremely rare sporadic tumour.
Next question
2/3
Question 67-69 of 147

Theme: Thyroid disorders

A. Sick euthyroid
B. Hyperthyroidism
C. Hypothyroidism
D. Normal euthyroid
E. Anxiety state
F. Factitious hyperthyroidism

For each of the scenarios please match the scenario with the most likely underlying diagnosis. Each
answer may be used once, more than once or not at all.

67. A 33 year old man is recovering following a protracted stay on the intensive care unit
recovering from an anastomotic leak following a difficult trans hiatal oesophagectomy. His
progress is slow, and the intensive care doctors receive the following thyroid function test
results:
TSH 1.0 u/L

Free T4 8

T3 1.0 (1.2-3.1 normal)

Sick euthyroid

Theme from April 2012 Exam


Sick euthyroid syndrome is caused by systemic illness. With this, the patient may have an
apparently low total and free T4 and T3, with a normal or low TSH. Note that the levels
are only mildly below normal.

68. A 28 year old female presents to the general practitioner with symptoms of fever and
diarrhoea. As part of her diagnostic evaluation the following thyroid function tests are
obtained:
TSH < 0.01

Free T4 30

T3 4.0
Hyperthyroidism

The symptoms are suggestive of hyperthyroidism. This is supported by the abnormal blood
results; suppressed TSH with an elevated T3 and T4.

69. A 19 year old lady presents with palpitations. The medical officer takes a blood sample for
thyroid function tests. The following results are obtained:
TSH > 6.0

Free T4 20

T3 2.0

You answered Sick euthyroid

The correct answer is Hypothyroidism

An elevated TSH with normal T4 indicates partial thyroid failure. This is caused by
Hashimotos, drugs (lithium, antithyroids) and dyshormogenesis.

Please rate this question:

Discuss and give feedback


Next question

Hyperthyroidism

Causes of hyperthyroidism include:

 Diffuse toxic goitre (Graves Disease)


 Toxic nodular goitre
 Toxic nodule
 Rare causes

Graves disease
Graves disease is characterised by a diffuse vascular goitre that appears at the same time as the
clinical manifestations of hyperthyroidism. It is commonest in younger females and may be
associated with eye signs. Thyrotoxic symptoms will predominate. Up to 50% of patients will have a
familial history of autoimmune disorders. The glandular hypertrophy and hyperplasia occur as a
result of the thyroid stimulating effects of the TSH receptor antibodies.
Toxic nodular goitre
In this disorder the goitre is present for a long period of time prior to the development of clinical
symptoms. In most goitres the nodules are inactive and in some cases it is the internodular tissue
that is responsible for the hyperthyroidism.

Toxic nodule
Overactive, autonomously functioning nodule. It may occur as part of generalised nodularity or be a
true toxic adenoma. The TSH levels are usually low as the autonomously functioning thyroid tissue
will exert a negative feedback effect.

Signs and symptoms


Symptoms Signs

Lethargy Tachycardia

Emotionally labile Agitation

Heat intolerance Hot, moist palms

Weight loss Exopthalmos

Excessive appetite Thyroid goitre and bruit

Palpitations Lid lag/retraction

Diagnosis
The most sensitive test for diagnosing hyperthyroidism is plasma T3 (which is raised). Note in
hypothyroidism the plasma T4 and TSH are the most sensitive tests. A TSH level of <0.5U/L
suggests hyperthyroidism. TSH receptor antibodies may be tested for in the diagnosis of Graves.

Treatment
First line treatment for Graves disease is usually medical and the block and replace regime is the
favored option. Carbimazole is administered at higher doses and thyroxine is administered orally.
Patient are maintained on this regime for between 6 and 12 months. Attempts are then made to
wean off medication. Where relapse then occurs the options are between ongoing medical therapy,
radioiodine or surgery.
Next question
Question 70 of 147

Which of the following conditions is least likely to exhibit the Koebner phenomenon?

Vitiligo

Molluscum contagiosum

Lichen planus

Psoriasis

Lupus vulgaris

Lupus vulgaris is not associated with the Koebner phenomenon.


Please rate this question:

Discuss and give feedback


Next question

Koebner phenomenon

The Koebner phenomenon describes skin lesions which appear at the site of injury. It is seen in:

 Psoriasis
 Vitiligo
 Warts
 Lichen planus
 Lichen sclerosus
 Molluscum contagiosum

Next question
Question 71 of 147

Which of the tumour types listed below is found most frequently in a person with aggressive
fibromatosis?

Medullary thyroid cancer

Basal cell carcinoma of the skin

Desmoid tumours

Dermoid tumours

Malignant melanoma

Please rate this question:

Discuss and give feedback

Next question

Aggressive fibromatosis

Aggressive fibromatosis is a disorder consisting of desmoid tumours, which behave in a locally


aggressive manner. Desmoid tumours may be identified in both abdominal and extra-abdominal
locations. Metastatic disease is rare. The main risk factor (for abdominal desmoids) is having APC
variant of familial adenomatous polyposis coli. Most cases are sporadic.
Treatment is by surgical excision.

Next question
Question 72 of 147

A 20 year old man is suspected of having an inflamed Meckels diverticulum. At which of the
following locations is it most likely to be found?

Approximately 60 cm distal to the ileo-caecal valve

Approximately 60 cm proximal to the ileocaecal valve

Approximately 200cm distal to the ileocaecal valve

Approximately 200cm proximal to the ileocaecal valve

50cm distal to the DJ flexure

Rule of 2's

2% of population
2 inches (5cm) long
2 feet (60 cm) from the ileocaecal valve
2 x's more common in men
2 tissue types involved

They are typically found 2 feet proximal to the ileocaecal valve (or approximately 60cm).
Please rate this question:

Discuss and give feedback


Next question

Meckel's diverticulum

 Congenital abnormality resulting in incomplete obliteration of the vitello-intestinal duct


 Normally, in the foetus, there is an attachment between the vitello-intestinal duct and the yolk
sac.This disappears at 6 weeks gestation.
 The tip is free in majority of cases.
 Associated with enterocystomas, umbilical sinuses, and omphaloileal fistulas.
 Arterial supply: omphalomesenteric artery.
 2% of population, 2 inches long, 2 feet from the ileocaecal valve.
 Typically lined by ileal mucosa but ectopic gastric mucosa can occur, with the risk of peptic
ulceration. Pancreatic and jejunal mucosa can also occur.
Clinical

 Normally asymptomatic and an incidental finding.


 Complications are the result of obstruction, ectopic tissue, or inflammation.
 Removal if narrow neck or symptomatic. Options are between wedge excision or formal
small bowel resection and anastomosis.

Next question
Question 73 of 147

A 43 year old lady presents with severe chest pain. Investigations demonstrate a dissecting
aneurysm of the ascending aorta which originates at the aortic valve. What is the optimal long term
treatment?

Endovascular stent

Medical therapy with beta blockers

Medical therapy with ACE inhibitors

Sutured aortic repair

Aortic root replacement

Proximal aortic dissections are generally managed with surgical aortic root replacement. The
proximal origin of the dissection together with chest pain (which may occur in all types of aortic
dissection) raises concerns about the possibility of coronary ostial involvement (which precludes
stenting). There is no role for attempted suture repair in this situation.
Please rate this question:

Discuss and give feedback


Next question

Aortic dissection

 More common than rupture of the abdominal aorta


 33% of patients die within the first 24 hours, and 50% die within 48 hours if no treatment
received
 Associated with hypertension
 Features of aortic dissection: tear in the intimal layer, followed by formation and propagation
of a subintimal hematoma. Cystic medial necrosis (Marfan's)
 Most common site of dissection: 90% occurring within 10 centimetres of the aortic valve

Stanford Classification
Type Location Treatment
Type Location Treatment

A Ascending aorta/ aortic root Surgery- aortic root replacement

B Descending aorta Medical therapy with antihypertensives

DeBakey classification
Type Site affected

I Ascending aorta, aortic arch, descending aorta

II Ascending aorta only

III Descending aorta distal to left subclavian artery

Clinical features

 Tearing, sudden onset chest pain (painless 10%)


 Hypertension or Hypotension
 A blood pressure difference (in each arm) greater than 20 mm Hg
 Neurologic deficits (20%)

Investigations

 CXR: widened mediastinum, abnormal aortic knob, ring sign, deviation of the
trachea/oesophagus
 CT angiography of the thoracic aorta
 MRI angiography
 Conventional angiography (now rarely used diagnostically)

Management

 Beta-blockers: aim HR 60-80 bpm and systolic BP 100-120 mm Hg


 For type A dissections the standard of care is aortic root replacement
Question 74 of 147

A 45 year old lady presents with a pathological fracture of her femoral shaft. She is a poor historian,
but it transpires that she underwent a thyroidectomy 1 year previously. She has no other illness or
co-morbidities. What is the most likely underlying diagnosis?

Hyperparathyroidism

Metastatic papillary carcinoma of the thyroid

Metastatic medullary carcinoma of the thyroid

Metastatic follicular carcinoma of the thyroid

None of the above

Follicular carcinomas are a recognised cause of bone metastasis. Papillary lesions typically spread
via the lymphatics.
Please rate this question:

Discuss and give feedback


Next question

Thyroid malignancy

Papillary carcinoma

 Commonest sub-type
 Accurately diagnosed on fine needle aspiration cytology
 Histologically, they may demonstrate psammoma bodies (areas of calcification) and so
called 'orphan Annie' nuclei
 They typically metastasise via the lymphatics and thus laterally located apparently ectopic
thyroid tissue is usually a metastasis from a well differentiated papillary carcinoma

Follicular carcinoma

 Are less common than papillary lesions


 Like papillary tumours, they may present as a discrete nodule. Although they appear to be
well encapsulated macroscopically there is invasion on microscopic evaluation
 Lymph node metastases are uncommon and these tumours tend to spread
haematogenously. This translates into a higher mortality rate
 Follicular lesions cannot be accurately diagnosed on fine needle aspiration cytology and thus
all follicular FNA's (THY 3f) will require at least a hemi thyroidectomy

Anaplastic carcinoma

 Less common and tend to occur in elderly females


 Disease is usually advanced at presentation and often only palliative decompression and
radiotherapy can be offered.

Medullary carcinoma

 These are tumours of the parafollicular cells ( C Cells) and are of neural crest origin.
 The serum calcitonin may be elevated which is of use when monitoring for recurrence.
 They may be familial and occur as part of the MEN -2A disease spectrum.
 Spread may be either lymphatic or haematogenous and as these tumours are not derived
primarily from thyroid cells they are not responsive to radioiodine.

Lymphoma

 These respond well to radiotherapy


 Radical surgery is unnecessary once the disease has been diagnosed on biopsy material.
Such biopsy material is not generated by an FNA and thus a core biopsy has to be obtained
(with care!).

Next question
Question 75-77 of 147

Theme: Cardiac murmurs

A. Pulmonary stenosis

B. Mitral regurgitation

C. Tricuspid regurgitation

D. Aortic stenosis

E. Mitral stenosis

F. Aortic sclerosis

What is the most likely cause of the cardiac murmur in the following patients? Each option may be
used once, more than once or not at all.

75. A 35 year old Singaporean female attends a varicose vein pre operative clinic. On auscultation a
mid diastolic murmur is noted at the apex. The murmur is enhanced when the patient lies in the
left lateral position.

You answered Pulmonary stenosis

The correct answer is Mitral stenosis

Theme from September 2011 exam

A mid diastolic murmur at the apex is a classical description of a mitral stenosis murmur. The
most common cause is rheumatic heart disease. Complications of mitral stenosis include atrial
fibrillation, stroke, myocardial infarction and infective endocarditis.

76. A 22 year old intravenous drug user is found to have a femoral abscess. The nursing staff contact
the on call doctor as the patient has a temperature of 39oC. He is found to have a pan systolic
murmur loudest at the left sternal edge at the 4th intercostal space.

You answered Pulmonary stenosis


The correct answer is Tricuspid regurgitation

Intravenous drug users are at high risk of right sided cardiac valvular endocarditis. The character
of the murmur fits with a diagnosis of tricuspid valve endocarditis.

77. An 83 year old woman is admitted with a left intertrochanteric neck of femur fracture. On
examination the patient is found to have an ejection systolic murmur loudest in the aortic region.
There is no radiation of the murmur to the carotid arteries. Her ECG is normal.

You answered Pulmonary stenosis

The correct answer is Aortic sclerosis

The most likely diagnosis is aortic sclerosis. The main differential diagnosis is of aortic stenosis,
however as there is no radiation of the murmur to the carotids and the ECG is normal, this is less
likely.

Please rate this question:

Discuss and give feedback

Next question

Cardiac murmurs

Type of Murmur Conditions

Ejection systolic Aortic stenosis


Pulmonary stenosis, HOCM
ASD, Fallot's
Pan-systolic Mitral regurgitation
Tricuspid regurgitation
VSD

Late systolic Mitral valve prolapse


Coarctation of aorta

Early diastolic Aortic regurgitation


Graham-Steel murmur (pulmonary regurgitation)

Mid diastolic Mitral stenosis


Austin-Flint murmur (severe aortic regurgitation)

Next question
Question 78 of 147

A 55 year old lady presents with discomfort in the right breast. On clinical examination a small lesion
is identified and clinical appearances suggest fibroadenoma. Imaging confirms the presence of a
fibroadenoma alone. A core biopsy is taken, this confirms the presence of the fibroadenoma.
However, the pathologist notices that a small area of lobular carcinoma in situ is also present in the
biopsy. What is the best management?

Whole breast irradiation

Simple mastectomy

Mastectomy and sentinal lymph node biopsy

Wide local excision and sentinel lymph node biopsy

Breast MRI scan

Lobular carcinoma in situ has a low association with invasive malignancy. It is seldom associated
with microcalcification and therefore MRI is the best tool for determining disease extent. Resection of
in situ disease is not generally recommended and most surgeons would simply pursue a policy of
close clinical and radiological follow up.

Please rate this question:

Discuss and give feedback

Next question

Lobular carcinoma of the breast

Lobular breast cancers are less common than their ductal counterparts. They typically present
differently, the mass is usually more diffuse and less obvious on the usual imaging modalities of
ultrasound and mammography. This is significant since the disease may be understaged resulting in
inadequate treatment when wide local excision is undertaken.
In women with invasive lobular carcinoma it is usually safest to perform an MRI scan of the breast, if
breast conserving surgery is planned.
Lobular carcinomas are also more likely to be multifocal and metastasise to the contralateral breast.
Lobular carcinoma in situ is occasionally diagnosed incidentally on core biopsies. Unlike DCIS,
lobular carcinoma in situ is far less strongly associated with foci of invasion and is usually managed
by close monitoring.

Next question
Question 79 of 147

Which one of the following may be associated with an increased risk of venous thromboembolism?

Diabetes

Cannula

Hyperthyroidism

Tamoxifen

Amiodarone

Consider thromboembolism in breast cancer patients on tamoxifen!


Please rate this question:

Discuss and give feedback


Next question

Venous thromboembolism: risk factors

Common predisposing factors include malignancy, pregnancy and the period following an operation.
The comprehensive list below is partly based on the 2010 SIGN venous thromboembolism (VTE)
guidelines:

General

 increased risk with advancing age


 obesity
 family history of VTE
 pregnancy (especially puerperium)
 immobility
 hospitalisation
 anaesthesia
 central venous catheter: femoral >> subclavian

Underlying conditions
 malignancy
 thrombophilia: e.g. Activated protein C resistance, protein C and S deficiency
 heart failure
 antiphospholipid syndrome
 Behcet's
 polycythaemia
 nephrotic syndrome
 sickle cell disease
 paroxysmal nocturnal haemoglobinuria
 hyperviscosity syndrome
 homocystinuria

Medication

 combined oral contraceptive pill: 3rd generation more than 2nd generation
 hormone replacement therapy
 raloxifene and tamoxifen
 antipsychotics (especially olanzapine) have recently been shown to be a risk factor

SIGN also state that the following are risk factors for recurrent VTE:

 previous unprovoked VTE


 male sex
 obesity
 thrombophilias

Next question
Question 80 of 147

A 42 year old lady is investigated for symptoms of irritability and altered bowel habit. On examination
she is noted to have a smooth enlargement of the thyroid gland. As part of her investigations thyroid
function tests are requested, these are as follows:
TSH 0.1 mug/l

Free T4 35 pmol/l

The most likely underlying diagnosis is:

Multinodular goitre

Follicular carcinoma of the thyroid gland

Graves disease

Pregnancy

None of the above

TSH receptor antibodies will cause stimulation of the thyroid to synthesise T4. However, this will
have a negative feedback effect on the pituitary causing decrease in TSH levels.
Where hyperthyroidism occurs secondary to pregnancy the TSH is typically elevated.
Please rate this question:

Discuss and give feedback


Next question

Thyroid disease

Patients may present with a number of different manifestations of thyroid disease. They can be
broadly sub classified according to whether they are euthyroid or have clinical signs of thyroid
dysfunction. In addition it needs to be established whether they have a mass or not.

Assessment
 History
 Examination including USS
 If a nodule is identified then it should be sampled ideally via an image guided fine needle
aspiration
 Radionucleotide scanning is of limited use

Thyroid Tumours

 Papillary carcinoma
 Follicular carcinoma
 Anaplastic carcinoma
 Medullary carcinoma
 Lymphoma's

Multinodular goitre

 One of the most common reasons for presentation


 Provided the patient is euthyroid and asymptomatic and no discrete nodules are seen, they
can be reassured.
 In those with compressive symptoms surgery is required and the best operation is a total
thyroidectomy.
 Sub total resections were practised in the past and simply result in recurrent disease that
requires a difficult revisional resection.

Endocrine dysfunction

 In general these patients are managed by physicians initially.


 Surgery may be offered alongside radio iodine for patients with Graves disease that fails with
medical management or in patients who would prefer not to be irradiated (e.g. pregnant
women).
 Patients with hypothyroidism do not generally get offered a thyroidectomy. Sometimes
people inadvertently get offered resections during the early phase of Hashimotos thyroiditis,
however, with time the toxic phase passes and patients can simply be managed with
thyroxine.

Complications following surgery

 Anatomical such as recurrent laryngeal nerve damage.


 Bleeding. Owing to the confined space haematoma's may rapidly lead to respiratory
compromise owing to laryngeal oedema.
 Damage to the parathyroid glands resulting in hypocalcaemia.
Further sources of information
1. http://www.acb.org.uk/docs/TFTguidelinefinal.pdf- Association of Clinical Biochemistry guidelines
for thyroid function tests.

2. British association of endocrine surgeons website- http://www.baets.org.uk


Next question
Question 81 of 147

Which of the following is not a risk factor for developing tuberculosis?

Gastrectomy

Solid organ transplantation with immunosupression

Intravenous drug use

Haematological malignancy

Amiodarone

Risk factors for developing active tuberculosis include:

 silicosis
 chronic renal failure
 HIV positive
 solid organ transplantation with immunosuppression
 intravenous drug use
 haematological malignancy
 anti-TNF treatment
 previous gastrectomy

Please rate this question:

Discuss and give feedback


Next question

Tuberculosis

Tuberculosis (TB) is an infection caused by Mycobacterium tuberculosis that most commonly affects
the lungs. Understanding the pathophysiology of TB can be difficult - the key is to differentiate
between primary and secondary disease.

Primary tuberculosis
A non-immune host who is exposed to M. tuberculosis may develop primary infection of the lungs. A
small lung lesion known as a Ghon focus develops. The Ghon focus is composed of tubercle-laden
macrophages. The combination of a Ghon focus and hilar lymph nodes is known as a Ghon complex
In immunocompetent people the initial lesion usually heals by fibrosis. Those who are
immunocompromised may develop disseminated disease (miliary tuberculosis).

Secondary (post-primary) tuberculosis


If the host becomes immunocompromised the initial infection may become reactivated. Reactivation
generally occurs in the apex of the lungs and may spread locally or to more distant sites. Possible
causes of immunocomprise include:

 immunosuppressive drugs including steroids


 HIV
 malnutrition

The lungs remain the most common site for secondary tuberculosis. Extra-pulmonary infection may
occur in the following areas:

 central nervous system (tuberculous meningitis - the most serious complication)


 vertebral bodies (Pott's disease)
 cervical lymph nodes (scrofuloderma)
 renal
 gastrointestinal tract

Next question
Question 82 of 147

Which of the following is not true of gastric cancer?

There is an association with blood group A

Adenocarcinoma is the most common subtype

Individuals with histological evidence of signet ring cells have a lower incidence of
lymph node metastasis

Lymphomas account for 5% cases

In Western Countries a more proximal disease distribution has been noted

Signet ring cells are features of poorly differentiated gastric cancer associated with a increased risk
of metastatic disease.
Please rate this question:

Discuss and give feedback


Next question

Gastric cancer

Overview
There are 700,000 new cases of gastric cancer worldwide each year. It is most common in Japan
and less common in western countries. It is more common in men and incidence rises with
increasing age. The exact cause of many sporadic cancer is not known, however, familial cases do
occur in HNPCC families. In addition, smoking and smoked or preserved foods increase the risk.
Japanese migrants retain their increased risk (decreased in subsequent generations). The
distribution of the disease in western countries is changing towards a more proximal location
(perhaps due to rising obesity).

Pathology
There is some evidence of support a stepwise progression of the disease through intestinal
metaplasia progressing to atrophic gastritis and subsequent dysplasia, through to cancer. The
favoured staging system is TNM. The risk of lymph node involvement is related to size and depth of
invasion; early cancers confined to submucosa have a 20% incidence of lymph node metastasis.
Tumours of the gastro-oesophageal junction are classified as below:
Type True oesophageal cancers and may be associated with Barrett's oesophagus.
1

Type Carcinoma of the cardia, arising from cardiac type epithelium


2 or short segments with intestinal metaplasia at the oesophagogastric junction.

Type Sub cardial cancers that spread across the junction. Involve similar nodal stations to
3 gastric cancer.

Groups for close endoscopic monitoring

 Intestinal metaplasia of columnar type


 Atrophic gastritis
 Low to medium grade dysplasia
 Patients who have previously undergone resections for benign peptic ulcer disease (except
highly selective vagotomy).

Referral to endoscopy

Patients of any age with Patients without Worsening dyspepsia


dyspepsia and any of the dyspepsia
following

Chronic gastrointestinal bleeding Dysphagia Barretts oesophagus

Dysphagia Unexplained abdominal Intestinal metaplasia


pain or weight loss

Weight loss Vomiting Dysplasia

Iron deficiency anaemia Upper abdominal mass Atrophic gastritis

Upper abdominal mass Jaundice Patient aged over 55 years with


unexplained or persistent
dyspepsia
Upper GI endoscopy performed for dyspepsia. The addition of dye spraying (as shown in the bottom
right) may facilitate identification of smaller tumours

Image sourced from Wikipedia

Staging

 CT scanning of the chest abdomen and pelvis is the routine first line staging investigation in
most centres.
 Laparoscopy to identify occult peritoneal disease
 PET CT (particularly for junctional tumours)

Treatment

 Proximally sited disease greater than 5-10cm from the OG junction may be treated by sub
total gastrectomy
 Total gastrectomy if tumour is <5cm from OG junction
 For type 2 junctional tumours (extending into oesophagus) oesophagogastrectomy is usual
 Endoscopic sub mucosal resection may play a role in early gastric cancer confined to the
mucosa and perhaps the sub mucosa (this is debated)
 Lymphadenectomy should be performed. A D2 lymphadenectomy is widely advocated by the
Japanese, the survival advantages of extended lymphadenectomy have been debated.
However, the overall recommendation is that a D2 nodal dissection be undertaken.
 Most patients will receive chemotherapy either pre or post operatively.

Prognosis

UK Data

Disease extent Percentage 5 year survival

All RO resections 54%

Early gastric cancer 91%

Stage 1 87%

Stage 2 65%

Stage 3 18%

Operative procedure

Total Gastrectomy , lymphadenectomy and Roux en Y anastomosis

General anaesthesia
Prophylactic intravenous antibiotics
Incision: Rooftop.
Perform a thorough laparotomy to identify any occult disease.
Mobilise the left lobe of the liver off the diaphragm and place a large pack over it. Insert a large self
retaining retractor e.g. omnitract or Balfour (take time with this, the set up should be perfect). Pack
the small bowel away.
Begin by mobilising the omentum off the transverse colon.
Proceed to detach the short gastric vessels.
Mobilise the pylorus and divide it at least 2cm distally using a linear cutter stapling device.
Continue the dissection into the lesser sac taking the lesser omentum and left gastric artery flush at
its origin.
The lymph nodes should be removed en bloc with the specimen where possible.
Place 2 stay sutures either side of the distal oesophagus. Ask the anaesthetist to pull back on the
nasogastric tube. Divide the distal oesophagus and remove the stomach.
The oesphago jejunal anastomosis should be constructed. Identify the DJ flexure and bring a loop of
jejunum up to the oesophagus (to check it will reach). Divide the jejunum at this point. Bring the
divided jejunum either retrocolic or antecolic to the oesophagus. Anastamose the oesophagus to the
jejunum, using either interrupted 3/0 vicryl or a stapling device. Then create the remainder of the
Roux en Y reconstruction distally.
Place a jejunostomy feeding tube.
Wash out the abdomen and insert drains (usually the anastomosis and duodenal stump). Help the
anaesthetist insert the nasogastric tube (carefully!)
Close the abdomen and skin.
Enteral feeding may commence on the first post-operative day. However, most surgeons will leave
patients on free NG drainage for several days and keep them nil by mouth.
Next question
Question 83 of 147

What is the most common presentation of a parotid gland tumour?

Parapharyngeal mass

Mass at anterior border of masseter

Mass inferior to the angle of the mandible

Mass behind the angle of the mandible

Mass anterior to the ear

Parotid tumours may present at any region in the gland. However, most lesions will be located
behind the angle of the mandible, inferior to the ear lobe. Tumours of the deep lobe of the parotid
may present as a parapharyngeal mass and large lesions may displace the tonsil.
Please rate this question:

Discuss and give feedback


Next question

Parotid gland malignancy

 Most parotid neoplasms (80%) are benign lesions


 Most commonly present with painless mass around the mandible
 Up to 30% may present with pain, when this is associated with a discrete mass lesion in the
parotid it usually indicates perineural invasion.
 Perineural invasion is very unlikely to occur in association with benign lesions
 80% of patients with facial nerve weakness caused by parotid malignancies will have nodal
metastasis and a 5 year survival of 25%

Types of malignancy
Mucoepidermoid 30% of all parotid malignancies
carcinoma Usually low potential for local invasiveness and metastasis (depends
mainly on grade)
Adenoid cystic Unpredictable growth pattern
carcinoma Tendency for perineural spread
Nerve growth may display skip lesions resulting in incomplete excision
Distant metastasis more common (visceral rather than nodal spread)
5 year survival 35%

Mixed tumours Often a malignancy occurring in a previously benign parotid lesion

Acinic cell carcinoma Intermediate grade malignancy


May show perineural invasion
Low potential for distant metastasis
5 year survival 80%

Adenocarcinoma Develops from secretory portion of gland


Risk of regional nodal and distant metastasis
5 year survival depends upon stage at presentation, may be up to 75%
with small lesions with no nodal involvement

Lymphoma Large rubbery lesion, may occur in association with Warthins tumours
Diagnosis should be based on regional nodal biopsy rather than parotid
resection Treatment is with chemotherapy (and radiotherapy)

Next question
Question 84 of 147

A 52 year old male is referred to urology clinic with impotence. He is known to have hypertension.
He does not have any morning erections. On further questioning the patient reports pain in his
buttocks, this worsens on mobilising. On examination there is some muscle atrophy. The penis and
scrotum are normal. What is the most likely diagnosis?

Leriche syndrome

S3-S4 cord lesion

Pudendal nerve lesion

Psychological impotence

Beta blocker induced impotence

Leriche syndrome

Classically, it is described in male patients as a triad of symptoms:

1. Claudication of the buttocks and thighs


2. Atrophy of the musculature of the legs
3. Impotence (due to paralysis of the L1 nerve)

Leriche syndrome, is atherosclerotic occlusive disease involving the abdominal aorta and/or both of
the iliac arteries. Management involves correcting underlying risk factors such as
hypercholesterolaemia and stopping smoking. Investigation is usually with angiography.

Please rate this question:

Discuss and give feedback

Next question
Leriche syndrome

Atheromatous disease involving the iliac vessels. Blood flow to the pelvic viscera is compromised.
Patients may present with buttock claudication and impotence (in this particular syndrome).
Diagnostic work up will include angiography, where feasible, iliac occlusions are usually treated with
endovascular angioplasty and stent insertion.

Next question
Question 85 of 147

A 23 year old man presents with diarrhoea and passage of mucous. He is suspected of having
ulcerative colitis. Which of the following is least likely to be associated with this condition?

Superficial mucosal inflammation in the colon

Significant risk of dysplasia in long standing disease

Epsiodes of large bowel obstruction during acute attacks

Haemorrhage

Disease sparing the anal canal

Large bowel obstruction is not a feature of UC, patients may develop megacolon. However, this is a
different entity both diagnostically and clinically. Ulcerative colitis does not affect the anal canal and
the anal transitional zone. Inflammation is superficial. Dysplasia can occur in 2% overall, but
increases significantly if disease has been present over 20 years duration. Granulomas are features
of crohn's disease.

Other features:

 Disease maximal in the rectum and may spread proximally


 Contact bleeding
 Longstanding UC crypt atrophy and metaplasia/dysplasia

Please rate this question:

Discuss and give feedback


Next question

Ulcerative colitis

Ulcerative colitis is a form of inflammatory bowel disease. Inflammation always starts at rectum,
does not spread beyond ileocaecal valve (although backwash ileitis may occur) and is continuous.
The peak incidence of ulcerative colitis is in people aged 15-25 years and in those aged 55-65
years. It is less common in smokers.
The initial presentation is usually following insidious and intermittent symptoms. Features include:

 bloody diarrhoea
 urgency
 tenesmus
 abdominal pain, particularly in the left lower quadrant
 extra-intestinal features (see below)

Questions regarding the 'extra-intestinal' features of inflammatory bowel disease are common. Extra-
intestinal features include sclerosing cholangitis, iritis and ankylosing spondylitis.

Common to both Crohn's disease Notes


(CD) and Ulcerative colitis (UC)

Related to Arthritis: pauciarticular, asymmetric Arthritis is the most common extra-


disease activity Erythema nodosum intestinal feature in both CD and UC
Episcleritis Episcleritis is more common in
Osteoporosis Crohns disease

Unrelated to Arthritis: polyarticular, symmetric Primary sclerosing cholangitis is


disease activity Uveitis much more common in UC
Pyoderma gangrenosum Uveitis is more common in UC
Clubbing
Primary sclerosing cholangitis

Pathology

 Red, raw mucosa, bleeds easily


 No inflammation beyond submucosa (unless fulminant disease)
 Widespread superficial ulceration with preservation of adjacent mucosa which has the
appearance of polyps ('pseudopolyps')
 Inflammatory cell infiltrate in lamina propria
 Neutrophils migrate through the walls of glands to form crypt abscesses
 Depletion of goblet cells and mucin from gland epithelium
 Granulomas are infrequent

Barium enema

 Loss of haustrations
 Superficial ulceration, 'pseudopolyps'
 Long standing disease: colon is narrow and short -'drainpipe colon'
Endoscopy

 Superficial inflammation of the colonic and rectal mucosa


 Continuous disease from rectum proximally
 Superficial ulceration, mucosal islands, loss of vascular definition and continuous ulceration
pattern.

Management

 Patients with long term disease are at increased risk of development of malignancy
 Acute exacerbations are generally managed with steroids, in chronic patients agents such as
azathioprine and infliximab may be used
 Individuals with medically unresponsive disease usually require surgery- in the acute phase
a sub total colectomy and end ileostomy. In the longer term a proctectomy will be required.
An ileoanal pouch is an option for selected patients

References
Ford A et al. Ulcerative colitis. BMJ 2013 (346):29-34.
Next question
Question 86 of 147

A 23 year old lady undergoes a total thyroidectomy as treatment for a papillary carcinoma of the
thyroid. The pathologist examines histological sections of the thyroid gland and identifies a
psammoma body. What are these primarily composed of?

Clusters of calcification

Aggregations of neutrophils

Aggregations of macrophages

Giant cells surrounding the tumour

Clusters of oxalate crystals

Psammoma bodies consist of clusters of microcalcification. They are most commonly seen in
papillary carcinomas.
Please rate this question:

Discuss and give feedback


Next question

Thyroid malignancy

Papillary carcinoma

 Commonest sub-type
 Accurately diagnosed on fine needle aspiration cytology
 Histologically, they may demonstrate psammoma bodies (areas of calcification) and so
called 'orphan Annie' nuclei
 They typically metastasise via the lymphatics and thus laterally located apparently ectopic
thyroid tissue is usually a metastasis from a well differentiated papillary carcinoma

Follicular carcinoma

 Are less common than papillary lesions


 Like papillary tumours, they may present as a discrete nodule. Although they appear to be
well encapsulated macroscopically there is invasion on microscopic evaluation
 Lymph node metastases are uncommon and these tumours tend to spread
haematogenously. This translates into a higher mortality rate
 Follicular lesions cannot be accurately diagnosed on fine needle aspiration cytology and thus
all follicular FNA's (THY 3f) will require at least a hemi thyroidectomy

Anaplastic carcinoma

 Less common and tend to occur in elderly females


 Disease is usually advanced at presentation and often only palliative decompression and
radiotherapy can be offered.

Medullary carcinoma

 These are tumours of the parafollicular cells ( C Cells) and are of neural crest origin.
 The serum calcitonin may be elevated which is of use when monitoring for recurrence.
 They may be familial and occur as part of the MEN -2A disease spectrum.
 Spread may be either lymphatic or haematogenous and as these tumours are not derived
primarily from thyroid cells they are not responsive to radioiodine.

Lymphoma

 These respond well to radiotherapy


 Radical surgery is unnecessary once the disease has been diagnosed on biopsy material.
Such biopsy material is not generated by an FNA and thus a core biopsy has to be obtained
(with care!).

Next question
Question 87 of 147

A 52 year old woman attends clinic for investigation of abdominal pain and constipation. On
examination you note blue lines on the gum margin. She mentions that her legs have become weak
in the past few days. What is the most likely diagnosis?

Acute intermittent porphyria

Lead poisoning

Constipation

Guillan Barre syndrome

Rectal carcinoma

This would be an impressive diagnosis to make in the surgical out patient department! The
combination of abdominal pain and a motor periperal neuropathy, should indicate this diagnosis. The
blue line along the gum margin can occur in up to 20% patients with lead poisoning.
Please rate this question:

Discuss and give feedback


Next question

Lead poisoning

Along with acute intermittent porphyria, lead poisoning should be considered in questions giving a
combination of abdominal pain and neurological signs

Features

 abdominal pain
 peripheral neuropathy (mainly motor)
 fatigue
 constipation
 blue lines on gum margin (only 20% of adult patients, very rare in children)

Investigations
 The blood lead level is usually used for diagnosis. Levels greater than 10 mcg/dl are
considered significant
 Full blood count: microcytic anaemia. Blood film shows red cell abnormalities including
basophilic stippling and clover-leaf morphology
 Raised serum and urine levels of delta aminolaevulinic acid may be seen making it
sometimes difficult to differentiate from acute intermittent porphyria
 Urinary coproporphyrin is also increased (urinary porphobilinogen and uroporphyrin levels
are normal to slightly increased)

Management - various chelating agents are currently used:

 Dimercaptosuccinic acid (DMSA)


 D-penicillamine
 EDTA
 Dimercaprol

Next question
Question 88-90 of 147

Theme: Vasculitis

A. Wegeners granulomatosis
B. Polyarteritis nodosa
C. Giant cell arteritis
D. Takayasu's arteritis
E. Buergers disease

For each of the scenarios provided please select the most likely underlying diagnosis from the list
below. Each option may be used once, more than once or not at all.

88. A 20 year old lady is referred to the vascular clinic. She has been feeling generally unwell
for the past six weeks. She works as a typist and has noticed increasing pain in her
forearms whilst working. On examination she has absent upper limb pulses. Her ESR is
measured and mildly elevated.

You answered Wegeners granulomatosis

The correct answer is Takayasu's arteritis

Takayasus arteritis may be divided into acute systemic phases and the chronic pulseless
phase. In the latter part of the disease process the patient may complain of symptoms such
as upper limb claudication. In the later stages of the condition the vessels will typically
show changes of intimal proliferation, together with band fibrosis of the intima and media.

89. A 32 year old man presents to the vascular clinic with symptoms of foot pain during
exertion. He is a heavy smoker and has recently tried to stop smoking. On examination he
has normal pulses to the level of the popliteal. However, foot pulses are absent. A
diagnostic angiogram is performed which shows an abrupt cut off at the level of the
anterior tibial artery, together with the formation of corkscrew shaped collateral vessels
distally.

You answered Wegeners granulomatosis

The correct answer is Buergers disease

Buergers disease is most common in young male smokers. This demographic is changing
in those areas where young female smokers are more common. In the acute lesion the
internal elastic lamina of the vessels is usually intact. As the disease progresses the
changes progress to hypercellular occlusive thrombus. Tortuous corkscrew collaterals may
reconstitute patent segments of the distal tibial or pedal vessels.
90. A 78 year old man presents with symptoms of headaches and deteriorating vision. He
notices that there is marked pain on the right hand side of his face when he combs his hair.

You answered Wegeners granulomatosis

The correct answer is Giant cell arteritis

Temporal arteritis may present acutely with symptoms of headache and visual loss, or with
a less acute clinical picture. Sight may be threatened and treatment with
immunosupressants should be started promptly. The often requested temporal artery
biopsy (which can be the bane of many surgeons) is often non diagnostic and unhelpful.

Please rate this question:

Discuss and give feedback


Next question

Vasculitis

The vasculitides are a group of conditions characterised by inflammation of the blood vessel walls.
This may, in turn, compromise vessel integrity. Constitutional symptoms may be present. Whilst
certain disease subtypes are reported to affect specific vessels, there is often a degree of overlap
clinically.

Vessel diameter and vasculitis classification


Aorta and branches  Takayasu's arteritis
 Buergers disease
 Giant cell arteritis

Large and medium sized arteries  Buergers disease


 Giant cell arteritis
 Polyarteritis nodosa

Medium sized muscular arteries  Polyarteritis nodosa


 Wegeners granulomatosis

Small muscular arteries  Wegeners granulomatosis


 Rheumatoid vasculitis

Specific conditions
Takyasu's arteritis  Inflammatory, obliterative arteritis affecting aorta and branches
 Females> Males
 Symptoms may include upper limb claudication
 Clinical findings include diminished or absent pulses
 ESR often affected during the acute phase

Buergers disease  Segmental thrombotic occlusions of the small and medium sized
lower limb vessels
 Commonest in young male smokers
 Proximal pulses usually present, but pedal pulses are lost
 An acuter hypercellular occlusive thrombus is often present
 Tortuous corkscrew shaped collateral vessels may be seen on
angiography

Giant cell arteritis  Systemic granulomatous arteritis that usually affects large and
medium sized vessels
 Females > Males
 Temporal arteritis is commonest type
 Granulomatous lesions may be seen on biopsy (although up to
50% are normal)

Polyarteritis nodosa  Systemic necrotising vasculitis affecting small and medium sized
muscular arteries
 Most common in populations with high prevalence of hepatitis B
 Renal disease is seen in 70% cases
 Angiography may show saccular or fusiform aneurysms and
arterial stenoses

Wegeners  Predominantly affects small and medium sized arteries


granulomatosis  Systemic necrotising granulomatous vasculitis
 Cutaneous vascular lesions may be seen (ulceration, nodules and
purpura)
 Sinus imaging may show mucosal thickening and air fluid levels

Treatment
Conditions such as Buergers disease are markedly helped by smoking cessation.
Immunosupression is the main treatment for vasculitides.
Next question
Question 91 of 147

A 28 year old man presents with right upper quadrant pain and hydatid disease is suspected. Which
of the following statements relating to the disease is untrue?

First line treatment is with pentavalent antimony.

Peritoneal contamination with active daughter cysts may complicate surgery.

CT scanning of the liver may show a floating membrane.

Biliary communication with the cysts may occur.

It is caused by Echinococcus granulosus.

Drug treatment is with albendazole or mebendazole. Praziquantzel may be used in the pre operative
stages.
Please rate this question:

Discuss and give feedback


Next question

Hydatid cysts

Hydatid cysts are endemic in Mediterranean and Middle Eastern countries. They are caused by the
tapeworm parasite Echinococcus granulosus. An outer fibrous capsule is formed containing multiple
small daughter cysts. These cysts are allergens which precipitate a type 1 hypersensitivity
reaction.

Clinical features are as follows:

 Up to 90% cysts occur in the liver and lungs


 Can be asymtomatic, or symptomatic if cysts > 5cm in diameter
 Morbidity caused by cyst bursting, infection and organ dysfunction (biliary, bronchial, renal
and cerebrospinal fluid outflow obstruction)
 In biliary ruputure there may be the classical triad of; biliary colic, jaundice, and urticaria

CT is the best investigation to differentiate hydatid cysts from amoebic and pyogenic cysts.
Surgery is the mainstay of treatment (the cyst walls must not be ruptured during removal and the
contents sterilised first).
Next question
Question 92 of 147

Which of the following statements relating to neutrophil polymorphs is true?

Produce nitrogen peroxide as a microbicidal agent

Not involved in opsonisation

Deficiency leads to AIDS

Neutrophil disorders always result in chronic granulomatous diseases

Have a lifespan of 9 hours

 Neutrophils are the main cells of acute inflammation, important action against gram -ve and
+ve bacteria
 Appearance of segmented nucleus and granulated cytoplasm
 Have a lifespan of 1-3 days (shorter when consumed during septic process, though 9 hours
is unusual)
 Actions include: movement, opsonise microorganisms, phagocytosis & intracellular killing of
microorganisms via aerobic (produce HYDROGEN PEROXIDE) & anaerobic mechanisms.
 Neutrophil disorders include chronic granulomatous diseases: rare
 AIDS associated with T cell deficiency

Please rate this question:

Discuss and give feedback


Next question

Acute inflammation

Inflammation is the reaction of the tissue elements to injury. Vascular changes occur, resulting in the
generation of a protein rich exudate. So long as the injury does not totally destroy the existing tissue
architecture, the episode may resolve with restoration of original tissue architecture.

Vascular changes

 Vasodilation occurs and persists throughout the inflammatory phase.


 Inflammatory cells exit the circulation at the site of injury.
 The equilibrium that balances Starlings forces within capillary beds is disrupted and a protein
rich exudate will form as the vessel walls also become more permeable to proteins.
 The high fibrinogen content of the fluid may form a fibrin clot. This has several important
immunomodulatory functions.

Sequelae
Resolution  Typically occurs with minimal initial injury
 Stimulus removed and normal tissue architecture results

Organisation  Delayed removal of exudate


 Tissues undergo organisation and usually fibrosis

Suppuration  Typically formation of an abscess or an empyema


 Sequestration of large quantities of dead neutrophils

Progression to chronic  Coupled inflammatory and reparative activities


inflammation  Usually occurs when initial infection or suppuration has
been inadequately managed

Causes

 Infections e.g. Viruses, exotoxins or endotoxins released by bacteria


 Chemical agents
 Physical agents e.g. Trauma
 Hypersensitivity reactions
 Tissue necrosis

Presence of neutrophil polymorphs is a histological diagnostic feature of acute inflammation


Next question
Question 93 of 147

A 33 year old man is involved in a road traffic accident. He is initially stable and transferred to the
accident and emergency department. On arrival he is catheterised. One minute later he becomes
hypotensive, with evidence of angioedema surrounding his penis. What is the most likely explanation
for this event?

Type V latex hypersensitivity reaction

Type IV latex hypersensitivity reaction

Type III latex hypersensitivity reaction

Type I latex hypersensitivity reaction

Type II latex hypersensitivity reaction

Theme from January 2013 exam


Theme from April 2012 Exam
Sudden collapse and angioedema following exposure to latex (of which most urinary catheters are
manufactured) suggests a type I hypersensitivity reaction.

Please rate this question:

Discuss and give feedback

Next question

Hypersensitivity reactions

The Gell and Coombs classification divides hypersensitivity reactions into 4 types
Type I Type II Type III Type IV

Description Anaphylactic Cytotoxic Immune Delayed type


complex

Mediator IgE IgG, IgM IgG, Ig A, IgM T-cells

Antigen Exogenous Cell surface Soluble Tissues

Response Minutes Hours Hours 2-3 days


time

Examples Asthma Autoimmune haemolytic Serum sickness Graft versus host


Hay fever anaemia SLE disease
Pemphigus Aspergillosis Contact dermatitis
Goodpasture's

Next question
Question 94 of 147

A 43 year old female develops severe chest wall cellulitis following a mastectomy. On examination
the skin is markedly erythematous. Which of the acute inflammatory mediators listed below is least
likely to produce vasodilation?

Complement component C5a

Lysosomal compounds

Histamine

Serotonin

Prostaglandins

Erythema is a classical feature of acute inflammation. Potent mediators of vascular dilatation


include; histamine, prostaglandins, nitric oxide, platelet activating factor, complement C5a (and C3a)
and lysosomal compounds. Although serotonin is associated with acute inflammation it is a
vasoconstrictor. The effects of serotonin are dependent upon the state of the vessels in the tissues.
Intact and healthy tissues and vessels will respond to a serotonin infusion with vasodilation (hence
the flushing seen in carcinoid syndrome). In contrast it worsens cardiac ischaemia in myocardial
infarcts when released from damaged platelets.
Please rate this question:

Discuss and give feedback


Next question

Acute inflammation

Inflammation is the reaction of the tissue elements to injury. Vascular changes occur, resulting in the
generation of a protein rich exudate. So long as the injury does not totally destroy the existing tissue
architecture, the episode may resolve with restoration of original tissue architecture.

Vascular changes

 Vasodilation occurs and persists throughout the inflammatory phase.


 Inflammatory cells exit the circulation at the site of injury.
 The equilibrium that balances Starlings forces within capillary beds is disrupted and a protein
rich exudate will form as the vessel walls also become more permeable to proteins.
 The high fibrinogen content of the fluid may form a fibrin clot. This has several important
immunomodulatory functions.

Sequelae
Resolution  Typically occurs with minimal initial injury
 Stimulus removed and normal tissue architecture results

Organisation  Delayed removal of exudate


 Tissues undergo organisation and usually fibrosis

Suppuration  Typically formation of an abscess or an empyema


 Sequestration of large quantities of dead neutrophils

Progression to chronic  Coupled inflammatory and reparative activities


inflammation  Usually occurs when initial infection or suppuration has
been inadequately managed

Causes

 Infections e.g. Viruses, exotoxins or endotoxins released by bacteria


 Chemical agents
 Physical agents e.g. Trauma
 Hypersensitivity reactions
 Tissue necrosis

Presence of neutrophil polymorphs is a histological diagnostic feature of acute inflammation


Next question
Question 95 of 147

A 43 year old man presents with a 3 week history of malaise, sore throat, odynophagia and
dysphagia. On examination he is found to have patchy white spots in his oropharynx. An upper GI
endoscopy is performed and similar lesions are identified in the oesophagus. Which investigation is
most likely to identify the underlying pathology in this case?

Serum urea and electrolytes

Oesophageal biopsy for culture

Oesophageal biopsy for histology

Glucose tolerance testing

Viral serology

Oesophageal candidiasis is associated with immunosupression; mainly in patients on chemotherapy,


with haematological malignancy, HIV or inhaled steroids. In patients with HIV, oesophageal
candidiasis is part of the spectrum of AIDS defining illnesses and usually occurs when the CD4
count is less than 200. Others include PCP pneumonia and CMV infections.

Please rate this question:

Discuss and give feedback

Next question

Oesophageal candidiasis

Characterised by white spots in the oropharynx with extension into the oesophagus. Associated with
broad spectrum antibiotic usage, immunosupression and immunological disorders.
Patients may present with oropharyngeal symptoms, odynophagia and dysphagia.
Treatment is directed both at the underlying cause (which should be investigated for) and with oral
antifungal agents.

Next question
Question 96-98 of 147

Theme: Colonic obstruction

A. Malignant obstruction
B. Ogilvies syndrome
C. Volvulus
D. Diverticular stricture
E. Ischaemic stricture

Please select the most likely cause of obstruction for the situation described. Each option may be
used once, more than once or not at all.

96. A 78 year old man has undergone a hemi-arthroplasty for a intracapsular hip fracture. Post
operatively he develops electrolyte derangement and receives intravenous fluids. Over the
previous 24 hours he develops marked abdominal distension. On examination he has a
tense, tympanic abdomen which is not painful. A contrast enema shows flow of contrast
through to the caecum and through the ileocaecal valve.

You answered Malignant obstruction

The correct answer is Ogilvies syndrome

Patients with electrolyte disturbance and previous surgery may develop colonic pseudo-
obstruction (Ogilvies syndrome). The diagnosis is made using a contrast enema and
treatment is usually directed at the underlying cause with colonic decompression if
indicated.

97. A 67 year old man has had multiple episodes with fever and left iliac fossa pain. These
have usually resolved with courses of intravenous antibiotics. He is admitted with a history
of increasing constipation and abdominal distension. A contrast x-ray is performed which
shows flow of contrast to the sigmoid colon, here the contrast flows through a long narrow
segment of colon into dilated proximal bowel.

You answered Malignant obstruction

The correct answer is Diverticular stricture

The long history of left iliac fossa pain and development of bowel obstruction suggests a
diverticular stricture. These may contain a malignancy and most will require resection.

98. A 78 year old lady from a nursing home is admitted with a 24 hour history of absolute
constipation and abdominal pain. On examination she has a distended abdomen with a soft
mass in her left iliac fossa. An x-ray is performed which shows a large dilated loop of
bowel in the left iliac fossa which contains a fluid level.

You answered Malignant obstruction

The correct answer is Volvulus

Sigmoid volvulus may present with an asymmetrical mass in an elderly patient. It may
contain a fluid level, visible on plain films.

Please rate this question:

Discuss and give feedback


Next question

Colonic obstruction

Cause Features Treatment

Cancer  Usually insidious onset Establish diagnosis (e.g.


 History of progressive constipation contrast enema/ endoscopy)
 Systemic features (e.g. anaemia) Laparotomy and resection,
 Abdominal distension stenting, defunctioning
 Absence of bowel gas distal to site of colostomy or bypass
obstruction

Diverticular  Usually history of previous acute Once diagnosis established,


stricture diverticulitis usually surgical resection
 Long history of altered bowel habit Colonic stenting should not
 Evidence of diverticulosis on imaging or be performed for benign
endoscopy disease

Volvulus  Twisting of bowel around its mesentery Initial treatment is to


 Sigmoid colon affected in 76% cases untwist the loop, a flexible
 Patients usually present with abdominal sigmoidoscopy may be
pain, bloating and constipation needed
 Examination usually shows asymmetrical Those with clinical
distension evidence of ischaemia
 Plain X-rays usually show massively should undergo surgery
dilated sigmoid colon, loss of haustra and Patient with recurrent
"U" shape are typical, the loop may volvulus should undergo
contain fluid levels resection

Acute colonic  Symptoms and signs of large bowel Colonoscopic


pseudo- obstruction with no lesion decompression
obstruction  Usually associated with metabolic Correct metabolic disorders
disorders IV neostigmine
 Usually a cut off in the left colon (82% Surgery
cases)
 Although abdomen tense and distended,
it is usually not painful
 All patients should undergo contrast
enema (may be therapeutic!)

Next question
Question 99 of 147

A 52 year old man with dyspepsia is found to have a duodenal ulcer. A CLO test is taken and is
positive. Which statement relating to the likely causative organism is false?

It is a gram negative organism

It preferentially colonises gastric type mucosa

It may occupy areas of ectopic gastric metaplasia

In patients who are colonised there is commonly evidence of fundal gastritis on endoscopy

It produces a powerful urease that forms the basis of the Clo test

Helicobacter pylori accounts for >75% cases of duodenal ulceration. It may be diagnosed with either
serology, microbiology, histology or CLO testing.

Theme from January 2011 Exam


Helicobacter pylori rarely produces any typical features on endoscopy. Where infection is suspected
the easiest course of action is to take an antral biopsy for Clo testing in the endoscopy suite.

Please rate this question:

Discuss and give feedback

Next question

Helicobacter Pylori

Infection with Helicobacter Pylori is implicated in many cases of duodenal ulceration and up to 60%
of patients with gastric ulceration.

It is a gram negative, helical shaped rod with microaerophillic requirements. It has the ability to
produce a urease enzyme that will hydrolyse urea resulting in the production of ammonia. The effect
of ammonia on antral G cells is to cause release of gastrin via a negative feedback loop.
Once infection is established the organism releases enzymes that disrupt the gastric mucous layer.
Certain subtypes release cytotoxins cag A and vac A gene products. The organism incites a
classical chronic inflammatory process of the gastric epithelium. This accounts for the development
of gastric ulcers. The mildly increased acidity may induce a process of duodenal gastric metaplasia.
Whilst duodenal mucosa cannot be colonised by H-Pylori, mucosa that has undergone metaplastic
change to the gastric epithelial type may be colonised by H- Pylori with subsequent inflammation
and development of duodenitis and ulcers.
In patients who are colonised there is a 10-20% risk of peptic ulcer, 1-2% risk gastric cancer and
<1% risk MALT lymphoma.

Next question
Question 100 of 147

A 34 year old male presents with painful rectal bleeding and a fissure in ano is suspected. On
examination he has an epithelial defect at the mucocutaenous junction that is located anteriorly.
Approximately what proportion of patients with fissure in ano will present with this pattern of
disease?

90%

10%

50%

25%

100%

Only a minority of patients with fissure in ano will have an anteriorly sited fissure. They are
particularly rare in males and an anterior fissure in a man should prompt a search for an underlying
cause.
Please rate this question:

Discuss and give feedback


Next question

Anal fissure

Anal fissures are a common cause of painful, bright red, rectal bleeding.
Most fissures are idiopathic and present as a painful mucocutaneous defect in the posterior midline
(90% cases). Fissures are more likely to be anteriorly located in females, particularly if they are
multiparous. Multiple fissures and those which are located at other sites are more likely to be due to
an underlying cause.
Diseases associated with fissure in ano include:

 Crohns disease
 Tuberculosis
 Internal rectal prolapse

Diagnosis
In most cases the defect can be visualised as a posterior midline epithelial defect. Where symptoms
are highly suggestive of the condition and examination findings are unclear an examination under
anaesthesia may be helpful. Atypical disease presentation should be investigated with colonoscopy
and EUA with biopsies of the area.

Treatment
Stool softeners are important as the hard stools may tear the epithelium and result in recurrent
symptoms. The most effective first line agents are topically applied GTN (0.2%) or Diltiazem (2%)
paste. Side effects of diltiazem are better tolerated.
Resistant cases may benefit from injection of botulinum toxin or lateral internal sphincterotomy
(beware in females). Advancement flaps may be used to treat resistant cases.
Sphincterotomy produces the best healing rates. It is associated with incontinence to flatus in up to
10% of patients in the long term.
Next question
Question 101 of 147

The pathogenesis of osteopetrosis is best explained by a defect in which of the following?

Osteoclast function

PTH receptors

Osteoblast function

Calcium resorption in proximal tubule

Calcium absorption

Please rate this question:

Discuss and give feedback


Next question

Osteopetrosis

Overview

 Also known as marble bone disease


 Rare disorder of defective osteoclast function resulting in failure of normal bone resorption
 Stem cell transplant and interferon-gamma have been used for treatment

Next question
Question 102 of 147

A 13 month old boy is brought to the surgical clinic by his mother because his left testicle is not
located in the scrotum. At which of the following sites would the testicle be located if it were an
ectopic testis?

Canalicular

Inguinal

External inguinal ring

Superficial inguinal pouch

High scrotal

Theme from September 2013 Exam


Theme from January 2012 Exam
Ectopic testes are those that come to lie outside the normal range of embryological descent (i.e. in
the superficial inguinal pouch). Other sites of ectopic testes include; base of penis, femoral triangle
and perineum.
Please rate this question:

Discuss and give feedback


Next question

Testicular disorders-paediatric

Testicular disorders
Testicular disorders are some of the commonest conditions present in paediatric urological practice.

Cryptorchidism

 The embryological descent of the testicle from within the abdominal cavity may be subject to
a number of variations. Distinctions need to be made clinically from a non descended testis
and a testis that is retractile.
 Testis that lie outside the normal path of embryological descent are termed ectopic testis.
Undescended testis occurs in 1% of male infants. Where the testis does not lie in an intra
scrotal location, its location should be ascertained (by laparoscopy in first instance). Where
both testes are absent, the infant may be intersex.
 MRI scanning may reveal intra-abdominal testes; however a GA is often needed to perform
this investigation in this age group.
 Testes that are undescended should be placed in the scrotum after 1 year of age, as the
testosterone surge that may facilitate descent occurs at 6 months of age.
 Where the testes lie distally e.g. Superficial inguinal pouch an open orchidopexy is the
procedure of choice.
 With abdominal testes a laparoscopy should be performed. The risk of seminoma is
increased in individuals with a non descended testes and this risk is not reduced by
orchidopexy.

Testicular torsion

 Typically the patient has severe sudden onset of scrotal pain. The difficulty in paediatric
practice is the lack of clear history.
 On examination the testis is tender and enlarged.
 Management is by surgical exploration.
 Delay beyond 6 hours is associated with low salvage rates.
 A torted hyatid produces pain that is far more localised and the testis itself should feel
normal. However, diagnostic doubt often exists and in such cases surgical exploration is
warranted.

Hydrocele

 Occur secondary to patent processus vaginalis


 Present as fluid filling in scrotum or as cyst of the spermatic cord
 Communicating hydroceles are treated by a trans inguinal ligation of the PPV
 Cystic hydroceles in older children may be treated with scrotal exploration

Next question
Question 103 of 147

Which of the following is seen more commonly with Crohns disease rather than ulcerative colitis?

Mucosal islands at endoscopy

Goblet cell depletion on biopsy

Fat wrapping of the terminal ileum

Attenuated symptoms in smokers

Toxic megacolon

Crohns disease is worse in smokers and smoking is an independent risk factor for disease
recurrence following resection.

Fat wrapping of the terminal ileum is commonly seen in patients with ileal disease (the commonest
disease site). The mesenteric fat in patients with IBD is often dense, hard and prone to considerable
haemorrhage during surgery. At endoscopy, the mucosa in patients with Crohns disease is said to
resemble cobblestones, mucosal islands (pseudopolyps) are seen in ulcerative colitis.
Please rate this question:

Discuss and give feedback


Next question

Crohns disease

Crohns disease is a chronic transmural inflammation of a segment(s) of the gastrointestinal tract and
may be associated with extra intestinal manifestations. Frequent disease patterns observed include
ileal, ileocolic and colonic disease. Peri-anal disease may occur in association with any of these. The
disease is often discontinuous in its distribution. Inflammation may cause ulceration, fissures, fistulas
and fibrosis with stricturing. Histology reveals a chronic inflammatory infiltrate that is usually patchy
and transmural.

Ulcerative colitis Vs Crohns


Crohn's disease Ulcerative colitis
Crohn's disease Ulcerative colitis

Distribution Mouth to anus Rectum and colon

Macroscopic Cobblestone appearance, apthoid ulceration Contact bleeding


changes

Depth of disease Transmural inflammation Superficial inflammation

Distribution Patchy Continuous


pattern

Histological Granulomas (non caseating epithelioid cell Crypt abscesses, Inflammatory


features aggregates with Langerhans' giant cells) cells in the lamina propria

Extraintestinal manifestations of Crohns


Related to disease extent Unrelated to disease extent

Aphthous ulcers (10%) Sacroiliiitis (10-15%)

Erythema nodosum (5-10%) Ankylosing spondylitis (1-2%)

Pyoderma gangrenosum (0.5%) Primary sclerosing cholangitis (Rare)

Acute arthropathy (6-12%) Gallstones (up to 30%)

Ocular complications (up to 10%) Renal calculi (up to 10%)

Diarrhoea in Crohns
Diarrhoea in Crohns may be multifactorial since actual inflammation of the colon is not common.
Causes therefore include the following:
 Bile salt diarrhoea secondary to terminal ileal disease
 Entero-colic fistula
 Short bowel due to multiple resections
 Bacterial overgrowth

Surgical interventions in Crohns disease


The commonest disease pattern in Crohns is stricturing terminal ileal disease and this often
culminates in an ileocaecal resection. Other procedures performed include segmental small bowel
resections and stricturoplasty. Colonic involvement in patients with Crohns is not common and,
where found, distribution is often segmental. However, despite this distribution segmental resections
of the colon in patients with Crohns disease are generally not advocated because the recurrence
rate in the remaining colon is extremely high, as a result the standard options of colonic surgery in
Crohns patients are generally; sub total colectomy, panproctocolectomy and staged sub total
colectomy and proctectomy. Restorative procedures such as ileoanal pouch have no role in therapy.
Crohns disease is notorious for the developmental of intestinal fistulae; these may form between the
rectum and skin (peri anal) or the small bowel and skin. Fistulation between loops of bowel may also
occur and result in bacterial overgrowth and malabsorption. Management of enterocutaneous
fistulae involves controlling sepsis, optimising nutrition, imaging the disease and planning definitive
surgical management.
Next question
Question 104 of 147

A splenectomy increases the risk of infection from all the following organisms except?

Pneumococcus

Klebsiella

Haemophilus influenzae

Staphylococcus aureus

Neisseria meningitidis

Theme from 2010 Exam

Staphylococcus aureus infection following splenectomy is no more common than in non


splenectomised individuals. The other organisms are encapsulated, which is why they are more
likely to cause overwhelming post splenectomy sepsis.
Please rate this question:

Discuss and give feedback


Next question

Post splenectomy sepsis

Hyposplenism may complicate certain medical conditions where splenic atrophy occurs or may be
the result of medical intervention such as splenic artery embolization and splenectomy for trauma.
Diagnosis of hyposplenism is difficult and whilst there may be peripheral markers of the
splenectomised state (e.g. Howell Jolly bodies) these are neither 100% sensitive or specific. The
most sensitive test is a radionucleotide labeled red cell scan.
Hyposplenism, by whatever mechanism it occurs dramatically increases the risk of post splenectomy
sepsis, particularly with encapsulated organisms. Since these organisms may be opsonised, but this
then goes undetected at an immunological level due to loss of the spleen. For this reason individuals
are recommended to be vaccinated and have antibiotic prophylaxis.

Key recommendations

 All those with hyposplenism or may become so (such as prior to an elective splenectomy)
should receive pneumococcal, haemophilus type b and meningococcal type C vaccines.
These should be administered 2 weeks prior to splenectomy or two weeks following
splenectomy. The vaccine schedule for meningococcal disease essentially consists of a
dose of Men C and Hib at 2 weeks and then a dose of the MenACWY vaccine one month
later. Those aged under 2 may require a booster at 2 years. A dose of pneumococcal
polyvalent polysaccharide vaccine (PPV) is given at two weeks. A conjugated vaccine (PCV)
is offered to young children. The PCV is more immunogenic but covers fewer serotypes.
Boosting PPV is either guided by serological measurements (where available) or by routine
boosting doses at 5 yearly intervals.
 Annual influenza vaccination is recommended in all cases
 Antibiotic prophylaxis is offered to all. The risk of post splenectomy sepsis is greatest
immediately following splenectomy and in those aged less than 16 years or greater than 50
years. Individuals with a poor response to pneumococcal vaccination are another high risk
group. High risk individuals should be counselled to take penicillin or macrolide prophylaxis.
Those at low risk may choose to discontinue therapy. All patients should be advised about
taking antibiotics early in the case of intercurrent infections.
 Asplenic individuals traveling to malaria endemic areas are at high risk and should have both
pharmacological and mechanical protection.

Dosing
Penicillin V 500mg BD or amoxicillin 250mg BD

References
Davies J et al. Review of guidelines for the prevention and treatment of infection in patients with an
absent or dysfunctional spleen: Prepared on behalf of the British Committee for Standards in
Haematology by a Working Party of the Haemato-Oncology Task Force. British Journal of
Haematology2011 (155): 308317.
Next question
Question 105 of 147

Which of the following is not an extraintestinal feature Crohns disease?

Iritis

Clubbing

Aphthous ulcers

Erythema multiforme

Pyoderma gangrenosum

Extraintestinal manifestation of inflammatory bowel disease: A PIE SAC

Aphthous ulcers
Pyoderma gangrenosum
Iritis
Erythema nodosum
Sclerosing cholangitis
Arthritis
Clubbing

Please rate this question:

Discuss and give feedback


Next question

Crohns disease

Crohns disease is a chronic transmural inflammation of a segment(s) of the gastrointestinal tract and
may be associated with extra intestinal manifestations. Frequent disease patterns observed include
ileal, ileocolic and colonic disease. Peri-anal disease may occur in association with any of these. The
disease is often discontinuous in its distribution. Inflammation may cause ulceration, fissures, fistulas
and fibrosis with stricturing. Histology reveals a chronic inflammatory infiltrate that is usually patchy
and transmural.

Ulcerative colitis Vs Crohns


Crohn's disease Ulcerative colitis

Distribution Mouth to anus Rectum and colon

Macroscopic Cobblestone appearance, apthoid ulceration Contact bleeding


changes

Depth of disease Transmural inflammation Superficial inflammation

Distribution Patchy Continuous


pattern

Histological Granulomas (non caseating epithelioid cell Crypt abscesses, Inflammatory


features aggregates with Langerhans' giant cells) cells in the lamina propria

Extraintestinal manifestations of Crohns


Related to disease extent Unrelated to disease extent

Aphthous ulcers (10%) Sacroiliiitis (10-15%)

Erythema nodosum (5-10%) Ankylosing spondylitis (1-2%)

Pyoderma gangrenosum (0.5%) Primary sclerosing cholangitis (Rare)

Acute arthropathy (6-12%) Gallstones (up to 30%)

Ocular complications (up to 10%) Renal calculi (up to 10%)

Diarrhoea in Crohns
Diarrhoea in Crohns may be multifactorial since actual inflammation of the colon is not common.
Causes therefore include the following:
 Bile salt diarrhoea secondary to terminal ileal disease
 Entero-colic fistula
 Short bowel due to multiple resections
 Bacterial overgrowth

Surgical interventions in Crohns disease


The commonest disease pattern in Crohns is stricturing terminal ileal disease and this often
culminates in an ileocaecal resection. Other procedures performed include segmental small bowel
resections and stricturoplasty. Colonic involvement in patients with Crohns is not common and,
where found, distribution is often segmental. However, despite this distribution segmental resections
of the colon in patients with Crohns disease are generally not advocated because the recurrence
rate in the remaining colon is extremely high, as a result the standard options of colonic surgery in
Crohns patients are generally; sub total colectomy, panproctocolectomy and staged sub total
colectomy and proctectomy. Restorative procedures such as ileoanal pouch have no role in therapy.
Crohns disease is notorious for the developmental of intestinal fistulae; these may form between the
rectum and skin (peri anal) or the small bowel and skin. Fistulation between loops of bowel may also
occur and result in bacterial overgrowth and malabsorption. Management of enterocutaneous
fistulae involves controlling sepsis, optimising nutrition, imaging the disease and planning definitive
surgical management.
Next question
Question 106 of 147

Which of the following is not considered a risk factor for the development of oesophageal
malignancy?

Oesophageal metaplasia

Smoking

Excessive intake of alcoholic spirits

Achalasia

Blood group O

Blood group O is not a risk factor for oesophageal cancer. Achalasia is associated with the risk of
developing squamous cell carcinoma of the oesophagus.
Please rate this question:

Discuss and give feedback


Next question

Oesophageal cancer

 Incidence is increasing
 In most cases in the Western world this increase is accounted for by a rise in the number of
cases of adenocarcinoma. In the UK adenocarcinomas account for 65% of cases.
 Barretts oesophagus is a major risk factor for most cases of oesophageal adenocarcinoma.
 In other regions of the world squamous cancer is more common and is linked to smoking,
alcohol intake, diets rich in nitrosamines and achalasia.
 Surveillance of Barretts is important, as it imparts a 30 fold increase in cancer risk and if
invasive malignancy is diagnosed early then survival may approach 85% at 5 years.

Diagnosis

 Upper GI endoscopy is the first line test


 Contrast swallow may be of benefit in classifying benign motility disorders but has no place
in the assessment of tumours
 Staging is initially undertaken with CT scanning of the chest, abdomen and pelvis. If overt
metastatic disease is identified using this modality then further complex imaging is
unnecessary
 If CT does not show metastatic disease, then local stage may be more accurately assessed
by use of endoscopic ultrasound.
 Staging laparoscopy is performed to detect occult peritoneal disease. PET CT is performed
in those with negative laparoscopy. Thoracoscopy is not routinely performed.

Treatment
Operable disease is best managed by surgical resection. The most standard procedure is an Ivor-
Lewis type oesophagectomy. This procedure involves the mobilisation of the stomach and division of
the oesophageal hiatus. The abdomen is closed and a right sided thoracotomy performed. The
stomach is brought into the chest and the oesophagus mobilised further. An intrathoracic
oesophagogastric anastomosis is constructed. Alternative surgical strategies include a transhiatal
resection (for distal lesions), a left thoraco-abdominal resection (difficult access due to thoracic
aorta) and a total oesophagectomy (McKeown) with a cervical oesophagogastric anastomosis.
The biggest surgical challenge is that of anastomotic leak, with an intrathoracic anastomosis this will
result in mediastinitis. With high mortality. The McKeown technique has an intrinsically lower
systemic insult in the event of anastomotic leakage.

In addition to surgical resection many patients will be treated with adjuvant chemotherapy.
Next question
Question 107-109 of 147

Theme: Thyroid blood testing

A. Measurement of antibodies to TSH receptor


B. Thyroid peroxidase antibodies
C. Thyroglobulin antibodies
D. Serum calcitonin

Please select the blood test most commonly performed for the diagnosis or assessment of the
thyroid disorder described. Each answer may be used once, more than once or not at all.

107. A 32 year old lady is diagnosed with Medullary carcinoma of the thyroid and has
undergone resection of the tumour.

You answered Measurement of antibodies to TSH receptor

The correct answer is Serum calcitonin

Measurement of basal or stimulated calcitonin concentrations is used to assess the


completeness of surgical resection, and is of use in detecting diseases recurrences during
follow up.

108. A 20 year old lady has undergone a total thyroidectomy for a well differentiated papillary
carcinoma. She attends clinic and is well and the surgeon wishes to screen for disease
recurrence.

You answered Measurement of antibodies to TSH receptor

The correct answer is Thyroglobulin antibodies

Antibodies to thyroglobulin, the major constituent of colloid and precursor of thyroid


hormones may be elevated in those with metastatic or recurrent thyroid cancer. Results
may be erronoeous in those with other thyroid disorders.

109. A 33 year old lady presents with a recently diagnosed goitre and a diagnosis of
Hashimotos thyroiditis is suspected.

You answered Measurement of antibodies to TSH receptor

The correct answer is Thyroid peroxidase antibodies

Antibodies to thyroid peroxidase are found in most patients with Graves disease or
Hashimotos thyroiditis.

Please rate this question:

Discuss and give feedback


Next question

Blood testing in thyroid disease

Assay Usage

Thyroid peroxidase  Found in autoimmune disease affecting the thyroid


(microsomal) antibodies (Hashimotos 100%) and Graves (70%)

Antibodies to TSH  Individuals with Graves disease (95%)


receptor

Thyroglobulin antibodies  Not useful for clinically distinguishing between different types
of thyroid disease, may be used as part of thyroid cancer
follow up

Calcitonin  Released from the parafollicular cells


 Usually found in patients with medullary carcinoma of the
thyroid

Next question
Question 110 of 147

Which one of the following complications is least associated with ventricular septal defects?

Right heart failure

Aortic regurgitation

Eisenmenger's complex

Infective endocarditis

Atrial fibrillation

Atrial fibrillation is associated more with atrial septal defects


Please rate this question:

Discuss and give feedback


Next question

Ventricular septal defect

Ventricular septal defects are the most common cause of congenital heart disease. They close
spontaneously in around 50% of cases. Non-congenital causes include post myocardial infarction

Features

 classically a pan-systolic murmur which is louder in smaller defects

Complications

 aortic regurgitation*
 infective endocarditis
 Eisenmenger's complex
 right heart failure

*aortic regurgitation is due to a poorly supported right coronary cusp resulting in cusp prolapse
Question 111 of 147

A 24 year old man from Sub Saharan Africa presents with a lymphadenopathy and weight loss. A
diagnosis of tuberculosis is suspected and a lymph node biopsy is performed. Staining with which of
the agents below is most likely to facilitate identification of the causative organism?

Gram stain

Ziehl-Neelsen stain

Von Kossa stain

Van Gieson stain

Masson Trichrome stain

Ziehl-Neelsen stain is typically used to identify mycobacteria. They are not stained in the Gram
staining process. Van Gieson and Masson trichrome are histological staining methods for
identification of connective tissues. The Von Kossa technique is useful for identifying tissue
mineralisation.
Please rate this question:

Discuss and give feedback


Next question

Tuberculosis pathology

 Is a form of primary chronic inflammation, caused by the inability of macrophages to kill


theMycobacterium tuberculosis.
 The macrophages often migrate to regional lymph nodes, the lung lesion plus affected lymph
nodes is referred to as a Ghon complex.
 This leads to the formation of a granuloma which is a collection of epithelioid histiocytes.
 There is the presence of caseous necrosis in the centre.
 The inflammatory response is mediated by a type 4 hypersensitivity reaction.
 In healthy individuals the disease may be contained, in the immunocompromised
disseminated (miliary TB) may occur.

Diagnosis
 Waxy membrane of mycobacteria prevents binding with normal stains. Ziehl - Neelsen
staining is typically used.
 Culture based methods take far longer.

Image showing acid- alcohol fast mycobacteria stained using the Ziehl- Neelsen method

Image sourced from Wikipedia

Next question
Question 112 of 147

Which of the following is not a pathological feature of breast cancer?

Resemblance to ductal epithelial cells

Angiogenesis

Nuclear pleomorphism

Metastatic calcification

Vascular invasion

Dystrophic calcification may be present in breast malignancy and is the basis for the breast
screening programme. Metastatic calcification is calcification which occurs in otherwise normal
tissues, usually as a result of hypercalcaemia. Invasive ductal carcinoma is the most common type
of breast cancer, unless the tumour is very poorly differentiated there is usually some resemblance
to ductal epithelial cells.

Please rate this question:

Discuss and give feedback

Next question

Breast cancer pathology

The histological features of breast cancer depend upon the underlying diagnosis. The invasive
component is usually comprised of ductal cells (unless it is an invasive lobular cancer). In situ
lesions may co-exist (such as DCIS).

Typical changes seen in conjunction with invasive breast cancer include:


1. Nuclear pleomorphism
2. Coarse chromatin
3. Angiogenesis
4. Invasion of the basement membrane
5. Dystrophic calcification (may be seen on mammography)
6. Abnormal mitoses
7. Vascular invasion
8. Lymph node metastasis

The primary tumour is graded on a scale of 1-3 where 1 is the most benign lesion and 3 the most
poorly differentiated.

Immunohistochemistry for oestrogen receptor and herceptin status is routinely performed.

The grade, lymph node stage and size are combined to provide the Nottingham prognostic index.

Next question
Question 113-115 of 147

Theme: Renal lesions

A. Renal cell carcinoma


B. Renal transitional cell carcinoma
C. Nephroblastoma
D. Neuroblastoma
E. Angiomyolipoma
F. Renal squamous cell carcinoma
G. Retroperitoneal fibrosis

For each scenario please select the most likely underlying diagnosis. Each option may be used
once, more than once or not at all.

113. A 69 year old male presents with haematuria. He worked in the textile industry. He has a
left flank mass. A CT IVU shows a lesion of the left renal pelvis.

You answered Renal cell carcinoma

The correct answer is Renal transitional cell carcinoma

TCC is a rare form of renal cancer, accounting for approximately 7% of all renal
tumours. Risk factors include exposure to chemicals in the textile, plastic and rubber
industry.

114. A 2 year old boy presents with a right renal mass. On examination he has an irregular
mass arising from the right flank and is hypertensive. A CT scan shows a non calcified
irregular lesion affecting the apex of the right kidney and the right adrenal gland.

You answered Renal cell carcinoma

The correct answer is Nephroblastoma

Wilm's tumour of the kidney is the most common renal tumour in children. Both
nephroblastoma and neuroblastoma may occupy the adrenal and apex of the kidney. In
the case of neuroblastoma the lesion will have arisen from the adrenal, in the case of
nephroblastoma the lesion will have arisen from the kidney. Hypertension is more
commonly associated with nephroblastoma. Neuroblastomas are usually calcified,
whereas nephroblastomas are not and this may be of diagnostic usefulness pre
operatively.

115. A 35 year old male presents with haematuria. He is found to have bilateral masses in the
flanks. He has a history of epilepsy and learning disability.

You answered Renal cell carcinoma

The correct answer is Angiomyolipoma

This patient has tuberous sclerosis. This is associated with angiomyolipoma, which is
present in 60-80% patients. It is a benign lesion.

Please rate this question:

Discuss and give feedback


Next question

Renal lesions

Lesion Disease specific features Treatment

Renal cell  Most present with haematuria Usually radical or partial


carcinoma (50%) nephrectomy
 Common renal tumour (85%
cases)
 Paraneoplastic features include
hypertension and polycythaemia
 Most commonly has
haematogenous mestastasis

Nephroblastoma  Rare childhood tumour Surgical resection combined


 It accounts for 80% of all with chemotherapy (usually
genitourinary malignancies in vincristine, actinomycin D and
those under the age of 15 years doxorubicin)
 Up to 90% will have a mass
 50% will be hypertensive
 Diagnostic work up includes
ultrasound and CT scanning

Neuroblastoma  Most common extracranial Surgical resection, radiotherapy


tumour of childhood and chemotherapy
 80% occur in those under 4 years
of age
Lesion Disease specific features Treatment

 Tumour of neural crest origin (up


to 50% occur in the adrenal
gland)
 The tumour is usually calcified
and may be diagnosed using
MIBG scanning
 Staging is with CT

Transitional cell  Accounts for 90% of Radical nephroureterectomy


carcinoma lower urinary tract tumours, but
only 10% of renal tumours
 Males affected 3x more than
females
 Occupational exposure to
industrial dyes and rubber
chemicals may increase risk
 Up to 80% present with painless
haematuria
 Diagnosis and staging is with CT
IVU

Angiomyolipoma  80% of these hamartoma type 50% of patients with lesions


lesions occur sporadically, the >4cm will have symptoms and
remainder are seen in those with will require surgical resection
tuberous sclerosis
 Tumour is composed of blood
vessels, smooth muscle and fat
 Massive bleeding may occur in
10% of cases

Next question
Question 116 of 147

An 18 month old boy presents with recurrent urinary tract infections. As part of the diagnostic work-
up he is noted to have abnormal renal function. An ultrasound scan is performed and shows bilateral
hydronephrosis. What is the most likely underlying diagnosis?

Urethral valves

Meatal stenosis

Hydronephrosis

Pelvico-ureteric junction obstruction

Benign prostatic hyperplasia

Theme from April 2012 Exam


A posterior urethral valve is an obstructive, developmental uropathy that usually affects male infants
(incidence 1 in 8000). Diagnostic features include bladder wall hypertrophy, hydronephrosis and
bladder diverticula.

Please rate this question:

Discuss and give feedback

Next question

Urethral valves

Posterior urethral valves are the commonest cause of infravesical outflow obstruction in males. They
may be diagnosed on ante natal ultrasonography. Because the bladder has to develop high
emptying pressures in utero, the child may develop renal parenchymal damage. This translates to
renal impairment noted in 70% of boys at presentation. Treatment is with bladder catheterisation.
Endoscopic valvotomy is the definitive treatment of choice with cystoscopic and renal follow up.
Question 117 of 147

At which of the following anatomical sites does dormant tuberculosis most frequently reactivate?

Apex of the lung

Base of the lung

Brain

Terminal ileum

Lumbar spine

TB reactivation most commonly occurs at the lung apex. This site is better oxygenated than
elsewhere allowing the mycobacteria to multiply more rapidly and then spread both locally and
distantly.
Please rate this question:

Discuss and give feedback


Next question

Tuberculosis pathology

 Is a form of primary chronic inflammation, caused by the inability of macrophages to kill


theMycobacterium tuberculosis.
 The macrophages often migrate to regional lymph nodes, the lung lesion plus affected lymph
nodes is referred to as a Ghon complex.
 This leads to the formation of a granuloma which is a collection of epithelioid histiocytes.
 There is the presence of caseous necrosis in the centre.
 The inflammatory response is mediated by a type 4 hypersensitivity reaction.
 In healthy individuals the disease may be contained, in the immunocompromised
disseminated (miliary TB) may occur.

Diagnosis

 Waxy membrane of mycobacteria prevents binding with normal stains. Ziehl - Neelsen
staining is typically used.
 Culture based methods take far longer.
Image showing acid- alcohol fast mycobacteria stained using the Ziehl- Neelsen method

Image sourced from Wikipedia

Next question
Question 118 of 147

What is the commonest tumour type encountered in the colon?

Squamous cell carcinoma

Adenocarcinoma

Lymphoma

Anaplastic carcinoma

Sarcoma

Adenocarcinoma are the most common and typically arise as a result of the adenoma - carcinoma
sequence.
Please rate this question:

Discuss and give feedback


Next question

Colorectal cancer

 Annually about 150,000 new cases are diagnosed and 50,000 deaths from the disease
 About 75% will have sporadic disease and 25% will have a family history
 Colorectal tumours comprise a spectrum of disease ranging from adenomas through to polyp
cancers and frank malignancy.
 Polyps may be categorised into: neoplastic polyps, adenomatous polyps and non neoplastic
polyps.
 The majority of adenomas are polypoidal lesions, although flat lesions do occur and may
prove to be dysplastic.
 Non-neoplastic polyps include hyperplastic, juvenile, hamartomatous, inflammatory, and
lymphoid polyps, which have not generally been thought of as precursors of cancer.
 Three characteristics of adenomas that correlate with malignant potential have been
characterised. These include increased size, villous architecture and dysplasia. For this
reason most polyps identified at colonoscopy should be removed.
 The transformation from polyp to cancer is described by the adenoma - carcinoma sequence
and its principles should be appreciated. Essentially genetic changes accompany the
transition from adenoma to carcinoma; key changes include APC, c-myc, K RAS mutations
and p53 deletions.
Question 119 of 147

Which of the following changes are most likely to be identified in the aortic wall of a 38 year old lady
with a Marfans syndrome and a dissecting aortic aneurysm?

Transmural aortitis

Cystic medial necrosis

Foamy macrophages

Dense dystrophic calcification

None of the above

Cystic medial necrosis ( or cystic medial degeneration) occurs when basophils and mucoid material
lie in between the intimal elastic fibres of the aorta. It is typically found in the aortic degeneration of
Marfans syndrome, but may also be seen in aortic degeneration in older adults.
Please rate this question:

Discuss and give feedback


Next question

Aortic dissection

 More common than rupture of the abdominal aorta


 33% of patients die within the first 24 hours, and 50% die within 48 hours if no treatment
received
 Associated with hypertension
 Features of aortic dissection: tear in the intimal layer, followed by formation and propagation
of a subintimal hematoma. Cystic medial necrosis (Marfan's)
 Most common site of dissection: 90% occurring within 10 centimetres of the aortic valve

Stanford Classification
Type Location Treatment
Type Location Treatment

A Ascending aorta/ aortic root Surgery- aortic root replacement

B Descending aorta Medical therapy with antihypertensives

DeBakey classification
Type Site affected

I Ascending aorta, aortic arch, descending aorta

II Ascending aorta only

III Descending aorta distal to left subclavian artery

Clinical features

 Tearing, sudden onset chest pain (painless 10%)


 Hypertension or Hypotension
 A blood pressure difference (in each arm) greater than 20 mm Hg
 Neurologic deficits (20%)

Investigations

 CXR: widened mediastinum, abnormal aortic knob, ring sign, deviation of the
trachea/oesophagus
 CT angiography of the thoracic aorta
 MRI angiography
 Conventional angiography (now rarely used diagnostically)

Management

 Beta-blockers: aim HR 60-80 bpm and systolic BP 100-120 mm Hg


 For type A dissections the standard of care is aortic root replacement
Question 120 of 147

A 58 year old man undergoes an upper GI endoscopy for the investigation of odynophagia. At
endoscopy a reddish area is seen to protrude up into the oesophagus from the gastro-oesophageal
junction. Which of the following pathological events is most likely to explain this process?

Metaplasia

Anaplasia

Dysplasia

Hypoplasia

Hyperplasia

This is most likely to represent Barretts oesphagus and is thus metaplasia. Dysplasia is less likely in
this setting although biopsies are mandatory.
Please rate this question:

Discuss and give feedback


Next question

Barrett's oesophagus

Barretts oesophagus is a condition characterised by the metaplastic transformation of squamous


oesophageal epithelium to columnar gastric type epithelium. Three types of this metaplastic process
are recognised; intestinal (high risk), cardiac and fundic. The latter two categories may cause
difficulties in diagnosis. The most concrete diagnosis can be made when endoscopic features of
Barretts oesophagus are present together with a deep biopsy that demonstrates not just goblet cell
metaplasia but also oesophageal glands.

Barrett's can be sub divided into short (<3cm) and long (>3cm). The length of the affected segment
correlates strongly with the chances of identifying metaplasia. The overall prevalence of Barrett's
oesophagus is difficult to determine but may be in the region of 1 in 20 and is identified in up to 12%
of those undergoing endoscopy for reflux.

A proportion of patients with metaplasia will progress to dysplasia and for this reason individuals
identified as having Barrett's should undergo endoscopic surveillance (every 2-5 years). Biopsies
should be quadrantic and taken at 1-2cm intervals. Biopsies need to be adequate. Where mass
lesions are present consideration should be given to endoscopic sub mucosal resection. Up to 40%
of patients will be upstaged from high grade dysplasia to invasive malignancy with such techniques.

Treatment

 Long term proton pump inhibitor


 Consider pH and manometry studies in younger patients who may prefer to consider an anti
reflux procedure
 Regular endoscopic monitoring (more frequently if moderate dysplasia). With quadrantic
biopsies every 2-3 cm
 If severe dysplasia be very wary of small foci of cancer

References
A consensus statement of the British approach is provided by:
Bennett C et al Consensus Statements for Management of Barrett's Dysplasia and Early-Stage
Esophageal Adenocarcinoma, Based on a Delphi Process. Gastroenterology Volume 143, Issue 2 ,
Pages 336-346, August 2012.
Next question
Question 121 of 147

A male infant is born by emergency cesarean section at 39 weeks gestation for foetal distress. Soon
after the birth the baby becomes progressively hypoxic and on examination is found to have a
scaphoid abdomen. What is the most likely underlying diagnosis?

Intestinal malrotation

Hiatus hernia

Foramen of Bochdalek hernia

Foramen of Morgagni hernia

Tracheo-oesphageal fistula

The finding of a scaphoid abdomen and respiratory distress suggests extensive intra thoracic
herniation of the abdominal contents. This is seen most frequently with Bochdalek hernias. Morgagni
hernias seldom present in such a dramatic fashion. The other options do not typically present with
the symptoms and signs described.
Please rate this question:

Discuss and give feedback


Next question

Embryology of the diaphragm and diaphragmatic hernia

Embryology
The diaphragm is formed between the 5th and 7th weeks of gestation through the progressive fusion
of the septum transversum, pleuroperitoneal folds and via lateral muscular ingrowth. The muscular
origins of the diaphragm are somites located in cervical segments 3 to 5, which accounts for the long
path taken by the phrenic nerve. The components contribute to the following diaphragmatic
segments:

 Septum transversum - Central tendon


 Pleuroperitoneal membranes - Parietal membranes surrounding viscera
 Cervical somites C5 to C7 - Muscular component of the diaphragm

Diaphragmatic hernia
Type of hernia Features

Morgagni Anteriorly located


Minimal compromise on lung development
Minimal signs on antenatal ultrasound
Usually present later
Usually good prognosis

Bochdalek hernia Posteriorly located


Larger defect
Often diagnosed antenatally
Associated with pulmonary hypoplasia
Poor prognosis

The posterior hernias of Bochdalek are the most common type and if not diagnosed antenatally will
typically present soon after birth with respiratory distress. The classical finding is that of a scaphoid
abdomen on clinical examination because of herniation of the abdominal contents into the chest.
Bochdalek hernias are associated with a number of chromosomal abnormalities such as Trisomy 21
and 18. Infants have considerable respiratory distress due to hypoplasia of the developing lung.
Historically this was considered to be due to direct compression of the lung by herniated viscera.
This view over simplifies the situation and the pulmonary hypoplasia occurs concomitantly with the
hernial development, rather than as a direct result of it. The pulmonary hypoplasia is associated with
pulmonary hypertension and abnormalities of pulmonary vasculature. The pulmonary hypertension
renders infants at risk of right to left shunting (resulting in progressive and worsening hypoxia).
Diagnostic work up of these infants includes chest x-rays/ abdominal ultrasound scans and cardiac
echo.
Surgery forms the mainstay of treatment and both thoracic and abdominal approaches may be
utilised. Following reduction of the hernial contents a careful search needs to be made for a hernial
sac as failure to recognise and correct this will result in a high recurrence rate. Smaller defects may
be primarily closed, larger defects may require a patch to close the defect. Malrotation of the viscera
is a recognised association and may require surgical correct at the same procedure (favoring an
abdominal approach).
The mortality rate is 50-75% and is related to the degree of lung compromise and age at
presentation (considerably better in infants >24 hours old).
Next question
Question 122 of 147

A 72 year old lady falls and lands on her left hip. She attends the emergency department and is
given some paracetamol by the junior doctor and discharged. Several months later she presents
with ongoing pain and discomfort of the hip. Avascular necrosis of the femoral head is suspected.
Which of the following features is least likely to be present?

Non union of the fracture

Angiogenesis at the fracture site

Increased numbers of fibroblasts at the fracture site

Osteochondritis dissecans

Apoptosis of osteoblasts

Apoptosis is not a feature of necrotic cell death. By this stage there would usually be attempted
repair so angiogenesis and proliferation of fibroblasts would be expected. These cells may
differentiate further to become osteoblasts which in turn will lay down new matrix.
Please rate this question:

Discuss and give feedback


Next question

Avascular necrosis

 Cellular death of bone components due to interruption of the blood supply, causing bone
destruction
 Main joints affected are hip, scaphoid, lunate and the talus.
 It is not the same as non union. The fracture has usually united.
 Radiological evidence is slow to appear.
 Vascular ingrowth into the affected bone may occur. However, many joints will develop
secondary osteoarthritis.

Causes
P ancreatitis
L upus
A lcohol
S teroids
T rauma
I diopathic, infection
C aisson disease, collagen vascular disease
R adiation, rheumatoid arthritis
A myloid
G aucher disease
S ickle cell disease

Presentation
Usually pain. Often despite apparent fracture union.

Investigation
MRI scanning will show changes earlier than plain films.

Treatment
In fractures at high risk sites anticipation is key. Early prompt and accurate reduction is essential.

Non weight bearing may help to facilitate vascular regeneration.

Joint replacement may be necessary, or even the preferred option (e.g. Hip in the elderly).
Next question
Question 123 of 147

Which one of the following is least associated with the development of colorectal cancer in patients
with ulcerative colitis?

Unremitting disease

Disease duration > 10 years

Onset before 15 years old

Poor compliance to treatment

Disease confined to the rectum

Please rate this question:

Discuss and give feedback


Next question

Ulcerative colitis and colorectal cancer

Overview

 risk of colorectal cancer is 10-20 times that of general population


 the increased risk is mainly related to chronic inflammation
 worse prognosis than patients without ulcerative colitis (partly due to delayed diagnosis)
 lesions may be multifocal

Factors increasing risk of cancer

 disease duration > 10 years


 patients with pancolitis
 onset before 15 years old
 unremitting disease
 poor compliance to treatment

Next question
Question 124-126 of 147

Theme: Causes of chest pain

A. Pulmonary embolism
B. Anterior myocardial infarction
C. Inferior myocardial infarction
D. Proximal aortic dissection
E. Distal aortic dissection
F. Boerhaave Syndrome
G. Mallory weiss tear
H. Perforated gastric ulcer

Please select the most likely cause of chest pain for the scenario given. Each option may be used
once, more than once or not at all.

124. A 52 year old male presents with tearing central chest pain. On examination, he has an
aortic regurgitation murmur. An ECG shows ST elevation in leads II, III and aVF.

You answered Pulmonary embolism

The correct answer is Proximal aortic dissection

Theme from 2011 Exam


An inferior myocardial infarction and AR murmur should raise suspicions of an
ascending aorta dissection rather than an inferior myocardial infarction alone. Also the
history is more suggestive of a dissection. Other features may include pericardial
effusion, carotid dissection and absent subclavian pulse.

125. A 52 year old male presents with central chest pain and vomiting. He has drunk a bottle
of vodka. On examination, there is some mild crepitus in the epigastric region.

You answered Pulmonary embolism

The correct answer is Boerhaave Syndrome

The Mackler triad for Boerhaave syndrome: vomiting, thoracic pain, subcutaneous
emphysema. It commonly presents in middle aged men with a background of alcohol
abuse.

126. A 52 year old male presents with central chest pain. On examination, he has an mitral
regurgitation murmur. An ECG shows ST elevation in leads V1 to V6. There is no ST
elevation in leads II, III and aVF.
You answered Pulmonary embolism

The correct answer is Anterior myocardial infarction

The most likely diagnosis is an anterior MI. As there are no ST changes in the inferior
leads, aortic dissection is less likely.

Please rate this question:

Discuss and give feedback


Next question

Chest pain

Aortic dissection

 This occurs when there is a flap or filling defect within the aortic intima. Blood tracks into the
medial layer and splits the tissues with the subsequent creation of a false lumen. It most
commonly occurs in the ascending aorta or just distal to the left subclavian artery (less
common). It is most common in Afro-carribean males aged 50-70 years.
 Patients usually present with a tearing intrascapular pain, which may be similar to the pain of
a myocardial infarct.
 The dissection may spread either proximally or distally with subsequent disruption to the
arterial branches that are encountered.
 In the Stanford classification system the disease is classified into lesions with a proximal
origin (Type A) and those that commence distal to the left subclavian (Type B).
 Diagnosis may be suggested by a chest x-ray showing a widened mediastinum. Confirmation
of the diagnosis is usually made by use of CT angiography
 Proximal (Type A) lesions are usually treated surgically, type B lesions are usually managed
non operatively.

Pulmonary embolism

 Typically sudden onset of chest pain, haemoptysis, hypoxia and small pleural effusions may
be present.
 Most patients will have an underlying deep vein thrombosis
 Diagnosis may be suggested by various ECG findings including S waves in lead I, Q waves
in lead III and inverted T waves in lead III. Confirmation of the diagnosis is usually made
through use of CT pulmonary angiography.
 Treatment is with anticoagulation, in those patients who develop a cardiac arrest or severe
compromise from their PE, consideration may be given to thrombolysis.
Myocardial infarction

 Traditionally described as sudden onset of central, crushing chest pain. It may radiate into
the neck and down the left arm. Signs of autonomic dysfunction may be present. The
presenting features may be atypical in the elderly and those with diabetes.
 Diagnosis is made through identification of new and usually dynamic ECG changes (and
cardiac enzyme changes). Inferior and anterior infarcts may be distinguished by the
presence of specific ECG changes (usually II, III and aVF for inferior, leads V1-V5 for
anterior).
 Treatment is with oral antiplatelet agents, primary coronary angioplasty and/ or thrombolysis.

Perforated peptic ulcer

 Patients usually develop sudden onset of epigastric abdominal pain, it may be soon followed
by generalised abdominal pain.
 There may be features of antecendant abdominal discomfort, the pain of gastric ulcer is
typically worse immediately after eating.
 Diagnosis may be made by erect chest x-ray which may show a small amount of free intra-
abdominal air (very large amounts of air are more typically associated with colonic
perforation).
 Treatment is usually with a laparotomy, small defects may be excised and overlaid with an
omental patch, larger defects are best managed with a partial gastrectomy.

Boerhaaves syndrome

 Spontaneous rupture of the oesophagus that occurs as a result of repeated episodes of


vomiting.
 The rupture is usually distally sited and on the left side.
 Patients usually give a history of sudden onset of severe chest pain that may complicate
severe vomiting.
 Severe sepsis occurs secondary to mediastinitis.
 Diagnosis is CT contrast swallow.
 Treatment is with thoracotomy and lavage, if less than 12 hours after onset then primary
repair is usually feasible, surgery delayed beyond 12 hours is best managed by insertion of a
T tube to create a controlled fistula between oesophagus and skin.
 Delays beyond 24 hours are associated with a very high mortality rate.

Next question
Question 127 of 147

A 78 year old lady presents with a tender swelling in her right groin. On examination there is a tender
swelling that lies below and lateral to the pubic tubercle. It has a cough impulse. What is the most
likely underlying diagnosis?

Thrombophlebitis of the great saphenous vein

Femoral hernia

Thrombophlebitis of saphena varix

Inguinal hernia

Obturator hernia

Theme from April 2012 Exam


Whilst a thrombophlebitis of a saphena varix may cause a tender swelling at this site, it would not
usually be associated with a cough impulse.
Please rate this question:

Discuss and give feedback


Next question

Femoral canal

The femoral canal lies at the medial aspect of the femoral sheath. The femoral sheath is a fascial
tunnel containing both the femoral artery laterally and femoral vein medially. The canal lies medial to
the vein.

Borders of the femoral canal


Laterally Femoral vein

Medially Lacunar ligament

Anteriorly Inguinal ligament


Posteriorly Pectineal ligament

Image showing dissection of femoral canal

Image sourced from Wikipedia

Contents

 Lymphatic vessels
 Cloquet's lymph node

Physiological significance
Allows the femoral vein to expand to allow for increased venous return to the lower limbs.

Pathological significance
As a potential space, it is the site of femoral hernias. The relatively tight neck places these at high
risk of strangulation.
Question 128 of 147

A 3 month old boy is suspected of having hypospadias. At which of the following locations is the
urethral opening most frequently located in boys suffering from the condition?

On the distal ventral surface of the penis

On the proximal ventral surface of the penis

On the distal dorsal surface of the penis

On the proximal dorsal surface of the penis

At the base of the scrotum

The defect is located ventrally and most often distally. Proximally located urethral openings are well
recognised. Circumcision may compromise reconstruction.
Please rate this question:

Discuss and give feedback


Next question

Hypospadias

The urethral meatus opens on the ventral surface of the penis. There is also a ventral deficiency of
the foreskin. The urethral meatus may open more proximally in the more severe variants. However,
75% of the openings are distally located. The incidence is 1 in 300 male births.

Features include:

 Absent frenular artery


 Ventrally opened glans
 Skin tethering to hypoplastic urethra
 Splayed columns of spongiosum tissue distal to the meatus
 Deficiency of the foreskin ventrally

Management:
 No routine cultural circumcisions
 Urethroplasty
 Penile reconstruction

The foreskin is often utilised in the reconstructive process. In boys with very distal disease no
treatment may be needed.
Next question
Question 129 of 147

A 52 year old male attends for a preoperative assessment for an inguinal hernia repair. You notice
that the chest x-ray shows a loculated left pleural effusion. On further questioning the patient reports
that he worked as a builder 30 years ago. What is the most likely cause for the effusion?

Asbestosis

Pneumonia

Mesothelioma

Silicosis

Left ventricular failure

This patient has a risk of asbestos exposure through his occupation as a builder. As there a is latent
period of 30 years and a complicated effusion, the most likely cause is mesothelioma.
Please rate this question:

Discuss and give feedback


Next question

Mesothelioma

Features

 Dyspnoea, weight loss, chest wall pain


 Clubbing
 30% present as painless pleural effusion
 Only 20% have pre-existing asbestosis
 History of asbestos exposure in 85-90%, latent period of 30-40 years

Basics

 Malignancy of mesothelial cells of pleura


 Metastases to contralateral lung and peritoneum
 Right lung affected more often than left
Management

 Investigation: pleural biopsy, CT Scanning, (PET Scanning if surgery considered)


 Symptomatic
 Industrial compensation
 Chemotherapy, Surgery if operable
 Prognosis poor, median survival 12 months

Next question
Question 130 of 147

A 64-year-old woman is reviewed due to multiple non-healing leg ulcers. She reports feeling
generally unwell for many months. Examination findings include a blood pressure of 138/72 mmHg,
pulse 90 bpm, pale conjunctivae and poor dentition associated with bleeding gums. What is the most
likely underlying diagnosis?

Thyrotoxicosis

Vitamin B12 deficiency

Vitamin C deficiency

Diabetes mellitus

Sarcoidosis

Bleeding gums and poor healing are suggestive of vitamin C deficiency.


Please rate this question:

Discuss and give feedback


Next question

Vitamin C deficiency

Vitamin C deficiency (scurvy) leads to defective synthesis of collagen resulting in capillary fragility
(bleeding tendency) and poor wound healing

Features

 gingivitis, loose teeth


 poor wound healing
 bleeding from gums, haematuria, epistaxis
 general malaise

Next question
Question 131 of 147

Which of the following is not a typical feature of neuropraxia?

Transient delay in neuronal transmission

Axonal degeneration distal to the site of injury

Absence of neuroma formation

Preservation of autonomic function

Absence of axonal degeneration proximal to the site of injury

Full recovery may occur 6-8 weeks after nerve injury in neuropraxia.
Wallerian degeneration does not usually occur in simple neuropraxia.
Autonomic function is usually preserved.
Please rate this question:

Discuss and give feedback


Next question

Nerve injury

There are 3 types of nerve injury:


Neuropraxia  Nerve intact but electrical conduction is affected
 Full recovery
 Autonomic function preserved
 Wallerian degeneration does not occur

Axonotmesis  Axon is damaged and the myelin sheath is preserved. The connective tissue
framework is not affected.
 Wallerian degeneration occurs.

Neurotmesis  Disruption of the axon, myelin sheath and surrounding connective tissue.
 Wallerian degeneration occurs.

Wallerian Degeneration

 Axonal degeneration distal to the site of injury.


 Typically begins 24-36 hours following injury.
 Axons are excitable prior to degeneration occurring.
 Myelin sheath degenerates and is phagocytosed by tissue macrophages.

Nerve repair

 Neuronal repair may only occur physiologically where nerves are in direct contact. Where a
large defect is present, the process of nerve regeneration is hampered. It may not occur at
all or result in the formation of a neuroma. Where nerve regrowth occurs it is typically at a
rate of 1mm per day.

Next question
Question 132 of 147

A 44 year old lady presents with a pathological fracture of the left femur. She has previously
undergone a renal transplant for end stage renal failure. Her blood test results are as follows:

Serum Ca2+ 2.80

PTH 88pg/ml

Phosphate 0.30

A surgeon decides to perform a parathyroidectomy on the basis of these results. When the glands
are assessed histologically, which of the appearances is most likely to be identified?

Metaplasia the gland

Hypertrophy of the gland

Hyperplasia of the gland

Parathyroid carcinoma

Necrosis of the parathyroid gland

This is likely to be a case of tertiary hyperparathyroidism (high Calcium, high PTH, low phosphate).
Therefore the glands will be hyperplastic. Hypertrophy is not correct as this implies an increase in
size without an increase in cellularity. This mistake has cost many candidates marks in the MRCS
exams over the years!
Please rate this question:

Discuss and give feedback


Next question

Parathyroid glands and disorders of calcium metabolism

Hyperparathyroidism
Disease type Hormone profile Clinical features Cause

Primary  PTH (Elevated)  May be Most cases due to


hyperparathyroidism  Ca2+(Elevated) asymptomatic if solitary adenoma
 Phosphate (Low) mild (80%), multifocal
 Urine calcium :  Recurrent disease occurs in 10-
creatinine clearance abdominal pain 15% and parathyroid
ratio > 0.01 (pancreatitis, renal carcinoma in 1% or
colic) less
 Changes to
emotional or
cognitive state

Secondary  PTH (Elevated)  May have few Parathyroid gland


hyperparathyroidism  Ca2+ (Low or symptoms hyperplasia occurs as
normal)  Eventually may a result of low
 Phosphate develop bone calcium, almost
(Elevated) disease, osteitis always in a setting of
 Vitamin D levels fibrosa cystica and chronic renal failure
(Low) soft tissue
calcifications

Tertiary  Ca2+(Normal or  Metastatic Occurs as a result of


hyperparathyroidism high) calcification ongoing hyperplasia
 PTH (Elevated)  Bone pain and / of the parathyroid
 Phosphate levels or fracture glands after
(Decreased or  Nephrolithiasis correction of
Normal)  Pancreatitis underlying renal
 Vitamin D (Normal disorder, hyperplasia
or decreased) of all 4 glands is
 Alkaline usually the cause
phosphatase
(Elevated)

Differential diagnoses
It is important to consider the rare but relatively benign condition of benign familial hypocalciuric
hypercalcaemia, caused by an autosomal dominant genetic disorder. Diagnosis is usually made by
genetic testing and concordant biochemistry (urine calcium : creatinine clearance ratio <0.01-
distinguished from primary hyperparathyroidism).

Treatment

Primary hyperparathyroidism
Indications for surgery
 Elevated serum Calcium > 1mg/dL above normal
 Hypercalciuria > 400mg/day
 Creatinine clearance < 30% compared with normal
 Episode of life threatening hypercalcaemia
 Nephrolithiasis
 Age < 50 years
 Neuromuscular symptoms
 Reduction in bone mineral density of the femoral neck, lumbar spine, or distal radius of more
than 2.5 standard deviations below peak bone mass (T score lower than -2.5)

Secondary hyperparathyroidism
Usually managed with medical therapy.

Indications for surgery in secondary (renal) hyperparathyroidism:

 Bone pain
 Persistent pruritus
 Soft tissue calcifications

Tertiary hyperparathyroidism
Allow 12 months to elapse following transplant as many cases will resolve
The presence of an autonomously functioning parathyroid gland may require surgery. If the culprit
gland can be identified then it should be excised. Otherwise total parathyroidectomy and re-
implantation of part of the gland may be required.

References
1. Pitt S et al. Secondary and Tertiary Hyperparathyroidism, State of the Art Surgical
Management.Surg Clin North Am 2009 Oct;89(5):1227-39.

2. MacKenzie-Feder J et al. Primary Hyperparathyroidism: An Overview. Int J Endocrinol 2011;


2011: 251410.
Next question
Question 133 of 147

A 78 year old man is referred to the clinic by his general practitioner. For many years he noticed a
smooth swelling approximately 5cm anterior to the tragus of his right ear. Apart from being a heavy
smoker he has no co-morbidities. What is the most likely diagnosis?

Pleomorphic adenoma

Liposarcoma

Warthins tumour

Adenocarcinoma

None of the above

Warthins tumours are most common in elderly smokers. They have a relatively benign and indolent
course. They are usually well circumscribed as illustrated below:
Image sourced from Wikipedia

Please rate this question:

Discuss and give feedback


Next question

Parotid gland clinical

Benign neoplasms
Up to 80% of all salivary gland tumours occur in the parotid gland and up to 80% of these are
benign. There is no consistent correlation between the rate of growth and the malignant potential of
the lesion. However, benign tumours should not invade structures such as the facial nerve.
With the exception of Warthins tumours, they are commoner in women than men. The median age of
developing a lesion is in the 5th decade of life.

Benign tumour types


Tumour type Features

Benign pleomorphic adenoma or Most common parotid neoplasm (80%)


benign mixed tumor Proliferation of epithelial and myoepithelial cells of the ducts
and an increase in stromal components
Slow growing, lobular, and not well encapsulated
Recurrence rate of 1-5% with appropriate excision
(parotidectomy)
Recurrence possibly secondary to capsular disruption during
surgery
Malignant degeneration occurring in 2-10% of adenomas
observed for long periods, with carcinoma ex-pleomorphic
adenoma occurring most frequently as adenocarcinoma

Warthin tumor (papillary Second most common benign parotid tumor (5%)
cystadenoma lymphoma or Most common bilateral benign neoplasm of the parotid
adenolymphoma) Marked male as compared to female predominance
Occurs later in life (sixth and seventh decades)
Presents as a lymphocytic infiltrate and cystic epithelial
proliferation
May represent heterotopic salivary gland epithelial tissue
trapped within intraparotid lymph nodes
Incidence of bilaterality and multicentricity of 10%
Malignant transformation rare (almost unheard of)
Tumour type Features

Monomorphic adenoma Account for less than 5% of tumours


Slow growing
Consist of only one morphological cell type (hence term
mono)
Include; basal cell adenoma, canalicular adenoma,
oncocytoma, myoepitheliomas

Haemangioma Should be considered in the differential of a parotid mass in a


child
Accounts for 90% of parotid tumours in children less than 1
year of age
Hypervascular on imaging
Spontaneous regression may occur and malignant
transformation is almost unheard of

Malignant salivary gland tumours


Types of malignancy

Mucoepidermoid 30% of all parotid malignancies


carcinoma Usually low potential for local invasiveness and metastasis (depends
mainly on grade)

Adenoid cystic Unpredictable growth pattern


carcinoma Tendency for perineural spread
Nerve growth may display skip lesions resulting in incomplete excision
Distant metastasis more common (visceral rather than nodal spread)
5 year survival 35%

Mixed tumours Often a malignancy occurring in a previously benign parotid lesion

Acinic cell carcinoma Intermediate grade malignancy


May show perineural invasion
Low potential for distant metastasis
5 year survival 80%
Adenocarcinoma Develops from secretory portion of gland
Risk of regional nodal and distant metastasis
5 year survival depends upon stage at presentation, may be up to 75%
with small lesions with no nodal involvement

Lymphoma Large rubbery lesion, may occur in association with Warthins tumours
Diagnosis should be based on regional nodal biopsy rather than parotid
resection
Treatment is with chemotherapy (and radiotherapy)

Diagnostic evaluation

 Plain x-rays may be used to exclude calculi


 Sialography may be used to delineate ductal anatomy
 FNAC is used in most cases
 Superficial parotidectomy may be either diagnostic of therapeutic depending upon the nature
of the lesion
 Where malignancy is suspected the primary approach should be definitive resection rather
than excisional biopsy
 CT/ MRI may be used in cases of malignancy for staging primary disease

Treatment
For nearly all lesions this consists of surgical resection, for benign disease this will usually consist of
a superficial parotidectomy. For malignant disease a radical or extended radical parotidectomy is
performed. The facial nerve is included in the resection if involved. The need for neck dissection is
determined by the potential for nodal involvement.

Other parotid disorders


HIV infection

 Lymphoepithelial cysts associated with HIV occur almost exclusively in the parotid
 Typically presents as bilateral, multicystic, symmetrical swelling
 Risk of malignant transformation is low and management usually conservative

Sjogren syndrome

 Autoimmune disorder characterised by parotid enlargement, xerostomia and


keratoconjunctivitis sicca
 90% of cases occur in females
 Second most common connective tissue disorder
 Bilateral, non tender enlargement of the gland is usual
 Histologically, the usual findings are of a lymphocytic infiltrate in acinar units and
epimyoepithelial islands surrounded by lymphoid stroma
 Treatment is supportive
 There is an increased risk of subsequent lymphoma

Sarcoid

 Parotid involvement occurs in 6% of patients with sarcoid


 Bilateral in most cases
 Gland is not tender
 Xerostomia may occur
 Management of isolated parotid disease is usually conservative

Next question
Question 134 of 147

A 56 year old man presents with lethargy, haematuria and haemoptysis. On examination he is
hypertensive and has a right loin mass. A CT scan shows a lesion affecting the upper pole of the
right kidney, it has a small cystic centre. Which of the options below is the most likely diagnosis?

Squamous cell carcinoma of the kidney

Nephroblastoma

Renal adenocarcinoma

Transitional cell carcinoma of the kidney

Polycystic kidney disease

Renal adenocarcinoma are the most common renal tumours. These will typically affect the renal
parenchyma. Transitional cell carcinoma will usually affect urothelial surfaces. Nephroblastoma
would be very rare in this age group. Renal adenocarcinoma may produce cannon ball metastasis in
the lung which cause haemoptysis, this is not a feature of PKD.

Please rate this question:

Discuss and give feedback

Next question

Renal tumours

Renal cell carcinoma


Renal cell carcinoma is an adenocarcinoma of the renal cortex and is believed to arise from the
proximal convoluted tubule. They are usually solid lesions, up to 20% may be multifocal, 20% may
be calcified and 20% may have either a cystic component or be wholly cystic. They are often
circumscribed by a pseudocapsule of compressed normal renal tissue. Spread may occur either by
direct extension into the adrenal gland, renal vein or surrounding fascia. More distant disease
usually occurs via the haematogenous route to lung, bone or brain.
Renal cell carcinoma comprise up to 85% of all renal malignancies. Males are more commonly
affected than females and sporadic tumours typically affect patients in their sixth decade.
Patients may present with a variety of symptoms including; haematuria (50%), loin pain (40%), mass
(30%) and up to 25% may have symptoms of metastasis.Less than 10% have the classic triad of
haematuria, pain and mass.

Investigation
Many cases will present as haematuria and be discovered during diagnostic work up. Benign renal
tumours are rare, so renal masses should be investigated with multislice CT scanning. Some units
will add and arterial and venous phase to the scan to demonstrate vascularity and evidence of caval
ingrowth.

CT scanning of the chest and abdomen to detect distant disease should also be undertaken.

Routine bone scanning is not indicated in the absence of symptoms.

Biopsy should not be performed when a nephrectomy is planned but is mandatory before any
ablative therapies are undertaken.

Assessment of the functioning of the contra lateral kidney.

Management
T1 lesions may be managed by partial nephrectomy and this gives equivalent oncological results to
total radical nephrectomy. Partial nephrectomy may also be performed when there is inadequate
reserve in the remaining kidney.

For T2 lesions and above a radical nephrectomy is standard practice and this may be performed via
a laparoscopic or open approach. Preoperative embolisation is not indicated nor is resection of
uninvolved adrenal glands. During surgery early venous control is mandatory to avoid shedding of
tumour cells into the circulation.

Patients with completely resected disease do not benefit from adjuvant therapy with either
chemotherapy or biological agents. These should not be administered outside the setting of clinical
trials.

Patients with transitional cell cancer will require a nephroureterectomy with disconnection of the
ureter at the bladder.

References
Lungberg B et al. EAU guidelines on renal cell carcinoma: The 2010 update. European Urology 2010
(58): 398-406.

Next question

Question 135 of 147


A 34-year-old man is taken immediately to theatre with aortic dissection. You note he is tall with
pectus excavatum and arachnodactyly. His condition is primarily due to a defect in which one of the
following proteins?

Polycystin-1

Fibrillin

Type IV collagen

Type I collagen

Elastin

Although fibrillin is the primary protein affected (due to a defect in the fibrillin-1 gene) it should be
noted that fibrillin is used as a substrate of elastin.
Please rate this question:

Discuss and give feedback


Next question

Marfan's syndrome

Marfan's syndrome is an autosomal dominant connective tissue disorder. It is caused by a defect in


the fibrillin-1 gene on chromosome 15 and affects around 1 in 3,000 people.

Features

 tall stature with arm span to height ratio > 1.05


 high-arched palate
 arachnodactyly
 pectus excavatum
 pes planus
 scoliosis of > 20 degrees
 heart: dilation of the aortic sinuses (seen in 90%) which may lead to aortic aneurysm, aortic
dissection, aortic regurgitation, mitral valve prolapse (75%),
 lungs: repeated pneumothoraces
 eyes: upwards lens dislocation (superotemporal ectopia lentis), blue sclera, myopia
 dural ectasia (ballooning of the dural sac at the lumbosacral level)
The life expectancy of patients used to be around 40-50 years. With the advent of regular
echocardiography monitoring and beta-blocker/ACE-inhibitor therapy this has improved significantly
over recent years. Aortic dissection and other cardiovascular problems remain the leading cause of
death however.
Next question
Question 136 of 147

Which of the following are not typical of Lynch syndrome?

It is inherited in an autosomal recessive manner

Affected patients are more likely to develop right colon mucinous tumours than the
general population

Affected individuals have an 80% lifetime risk of colon cancer

Endometrial cancer is seen in 80% of women

Gastric cancers are more common

Lynch syndrome is inherited in an autosomal dominant fashion. It is characterised by microsatellite


instability in the DNA mismatch repair genes. Colonic tumours in patients with Lynch syndrome are
more likely to be right sided tumours and to be poorly differentiated.
Please rate this question:

Discuss and give feedback


Next question

Genetics and surgical disease

Some of the more commonly occurring genetic conditions occurring in surgical patients are
presented here.

Li-Fraumeni Syndrome

 Autosomal dominant
 Consists of germline mutations to p53 tumour suppressor gene
 High incidence of malignancies particularly sarcomas and leukaemias
 Diagnosed when:

*Individual develops sarcoma under 45 years


*First degree relative diagnosed with any cancer below age 45 years and another family member
develops malignancy under 45 years or sarcoma at any age
BRCA 1 and 2

 Carried on chromosome 17 (BRCA 1) and Chromosome 13 (BRCA 2)


 Linked to developing breast cancer (60%) risk.
 Associated risk of developing ovarian cancer (55% with BRCA 1 and 25% with BRCA 2).

Lynch Syndrome

 Autosomal dominant
 Develop colonic cancer and endometrial cancer at young age
 80% of affected individuals will get colonic and/ or endometrial cancer
 High risk individuals may be identified using the Amsterdam criteria

Amsterdam criteria
Three or more family members with a confirmed diagnosis of colorectal cancer, one of whom is a
first degree (parent, child, sibling) relative of the other two.
Two successive affected generations.
One or more colon cancers diagnosed under age 50 years.
Familial adenomatous polyposis (FAP) has been excluded.

Gardners syndrome

 Autosomal dominant familial colorectal polyposis


 Multiple colonic polyps
 Extra colonic diseases include: skull osteoma, thyroid cancer and epidermoid cysts
 Desmoid tumours are seen in 15%
 Mutation of APC gene located on chromosome 5
 Due to colonic polyps most patients will undergo colectomy to reduce risk of colorectal
cancer
 Now considered a variant of familial adenomatous polyposis coli

Next question
Question 137 of 147

A 23 year old man suffers a thermal injury to his left hand. It becomes red and painful. Which of the
following mediators are not involved in this process?

Histamine

Free radicals

Prostaglandins

Leukotrienes

Serotonin

Acute inflammation is not mediated by free radicals

Chemical mediators facilitate the spread of inflammation into normal tissue


Chemical mediators include:

 Lysosomal compounds
 Chemokines such as serotinin and histamine (released by platelets and mast cells)

Other enzyme cascades producing inflammatory mediators include:

 Complement, kinin, coagulation system and fibrinolytic system

Please rate this question:

Discuss and give feedback


Next question

Acute inflammation

Inflammation is the reaction of the tissue elements to injury. Vascular changes occur, resulting in the
generation of a protein rich exudate. So long as the injury does not totally destroy the existing tissue
architecture, the episode may resolve with restoration of original tissue architecture.
Vascular changes

 Vasodilation occurs and persists throughout the inflammatory phase.


 Inflammatory cells exit the circulation at the site of injury.
 The equilibrium that balances Starlings forces within capillary beds is disrupted and a protein
rich exudate will form as the vessel walls also become more permeable to proteins.
 The high fibrinogen content of the fluid may form a fibrin clot. This has several important
immunomodulatory functions.

Sequelae
Resolution  Typically occurs with minimal initial injury
 Stimulus removed and normal tissue architecture results

Organisation  Delayed removal of exudate


 Tissues undergo organisation and usually fibrosis

Suppuration  Typically formation of an abscess or an empyema


 Sequestration of large quantities of dead neutrophils

Progression to chronic  Coupled inflammatory and reparative activities


inflammation  Usually occurs when initial infection or suppuration has
been inadequately managed

Causes

 Infections e.g. Viruses, exotoxins or endotoxins released by bacteria


 Chemical agents
 Physical agents e.g. Trauma
 Hypersensitivity reactions
 Tissue necrosis

Presence of neutrophil polymorphs is a histological diagnostic feature of acute inflammation


Next question
Question 138 of 147

An enthusiastic medical student approaches you with a list of questions about blood transfusion
reactions. Which of her following points is incorrect?

Graft versus host disease involves neutrophil proliferation

Thrombocytopaenia may occur in women with a prior pregnancy

IgA antibodies may cause blood pressure compromise during transfusion

Hypocalcaemia can occur

Iron overload can be avoided by chelation therapy

Mnemonic for transfusion reactions:

Got a bad unit

G raft vs. Host disease


O verload
T hrombocytopaenia

A lloimmunization

B lood pressure unstable


A cute haemolytic reaction
D elayed haemolytic reaction

U rticaria
N eutrophilia
I nfection
T ransfusion associated lung injury

GVHD results from lymphocytic proliferation. The patient's own lymphocytes are similar to the
donor's lymphocytes, therefore don't perceive them as being foreign. The donor lymphocytes,
however, sees the recipient lymphocytes as being foreign. Therefore they proliferate causing severe
complications.

Thrombocytopaenia occurs a few days after transfusion and may resolve spontaneously.

Patients with IGA antibodies need IgA deficient blood transfusions.

Please rate this question:

Discuss and give feedback

Next question

Blood transfusion reactions

Acute transfusion reactions present as adverse signs or symptoms during or within 24 hours of a
blood transfusion. The most frequent reactions are fever, chills, pruritus, or urticaria, which typically
resolve promptly without specific treatment or complications. Other signs occurring in temporal
relationship with a blood transfusion, such as severe dyspnoea, pyrexia, or loss of consciousness
may be the first indication of a more severe potentially fatal reaction.
The causes of adverse reactions are multi-factorial. Immune mediated reactions, some of the most
feared, occur as a result of component mismatch, the commonest cause of which is clerical error.
More common, non immune mediated, complications may occur as a result of product
contamination, this may be bacterial or viral.
Transfusion related lung injury is well recognised and there are two proposed mechanisms which
underpin this. One involves the sequestration of primed neutrophils within the recipient pulmonary
capillary bed. The other proposed mechanism suggests that HLA mismatches between donor
neutrophils and recipient lung tissue is to blame.
The table below summarises the main types of transfusion reaction.

Immune mediated Non immune mediated

Pyrexia Hypocalcaemia

Alloimmunization CCF

Thrombocytopaenia Infections
Immune mediated Non immune mediated

Transfusion associated lung injury Hyperkalaemia

Graft vs Host disease

Urticaria

Acute or delayed haemolysis

ABO incompatibility

Rhesus incompatibility

Next question
Question 139 of 147

An 82 year old lady presents with a carcinoma of the caecum. Approximately what proportion of
patients presenting with this diagnosis will have synchronous lesions?

<1%

60%

50%

20%

5%

Synchronous colonic tumours are seen in 5% cases and all patients having a flexible sigmoidoscopy
should have completion colonoscopy if tumours or polyps are found

Synchronous lesions may occur in up to 5% of patients with colorectal cancer. A full and complete
lumenal study with either colonoscopy, CT cologram or barium enema is mandatory in all patients
being considered for surgery.
Please rate this question:

Discuss and give feedback


Next question

Colorectal cancer

 Annually about 150,000 new cases are diagnosed and 50,000 deaths from the disease
 About 75% will have sporadic disease and 25% will have a family history
 Colorectal tumours comprise a spectrum of disease ranging from adenomas through to polyp
cancers and frank malignancy.
 Polyps may be categorised into: neoplastic polyps, adenomatous polyps and non neoplastic
polyps.
 The majority of adenomas are polypoidal lesions, although flat lesions do occur and may
prove to be dysplastic.
 Non-neoplastic polyps include hyperplastic, juvenile, hamartomatous, inflammatory, and
lymphoid polyps, which have not generally been thought of as precursors of cancer.
 Three characteristics of adenomas that correlate with malignant potential have been
characterised. These include increased size, villous architecture and dysplasia. For this
reason most polyps identified at colonoscopy should be removed.
 The transformation from polyp to cancer is described by the adenoma - carcinoma sequence
and its principles should be appreciated. Essentially genetic changes accompany the
transition from adenoma to carcinoma; key changes include APC, c-myc, K RAS mutations
and p53 deletions.

Next question
Question 140 of 147

A 22 year old man undergoes a splenectomy for an iatrogenic splenic injury. On the second post
operative day a full blood count is performed. Which of the following components of the full blood
count is the first to be affected ?

Erythrocyte count

Reticulocyte count

Eosinophil count

Monocyte count

Lymphocyte count

Theme from January 2012 Exam


The granulocyte and platelet count are the first to be affected following splenectomy. Then
reticulocytes increase. Although a lymphocytosis and monocytosis are reported, these take several
weeks to develop.

Please rate this question:

Discuss and give feedback

Next question

Post splenectomy blood film changes

The loss of splenic tissue results in the inability to readily remove immature or abnormal red blood
cells from the circulation. The red cell count does not alter significantly. However, cytoplasmic
inclusions may be seen e.g. Howell-Jolly bodies.
In the first few days after splenectomy target cells, siderocytes and reticulocytes will appear in the
circulation. Immediately following splenectomy a granulocytosis (mainly composed of neutrophils) is
seen, this is replaced by a lymphocytosis and monocytosis over the following weeks.
The platelet count is usually increased and this may be persistent, oral antiplatelet agents may be
needed in some patients.

Image showing Howell Jolly bodies (arrowed)

Image sourced from Wikipedia

Next question
Question 141 of 147

A 28 year old lady presents with benign cyclical mastalgia. Which of the following is not a recognised
treatment for the condition?

Evening primrose oil

Bromocriptine

Methotrexate

Danazol

Tamoxifen

Surgical excision of tender breast tissue is inappropriate

Methotrexate is used for the treatment of breast cancer. Whilst the use of tamoxifen is of benefit
other agents such as flaxseed oil or evening primrose oil should be tried first. Danazol is effective,
but many women dislike the side effects.

Please rate this question:

Discuss and give feedback

Next question

Benign cyclical mastalgia

Benign cyclical mastalgia is a common cause of breast pain in younger females. It varies in intensity
according to the phase of the menstrual cycle. It is not associated with point tenderness of the chest
wall (more likely to be Tietze's syndrome).

The underlying cause is difficult to pinpoint, examination should focus on identifying focal lesions
(such as cysts) that may be treated to provide symptomatic benefit. Women should be advised to
wear a supportive bra. Conservative treatments include flax seed oil and evening primrose oil. There
is slightly more evidence in favor of flax seed oil, though neither has performed much better than
placebo in RCT's.

Hormonal agents such as bromocriptine and danazol may be more effective. However, many women
discontinue these therapies due to adverse effects.

Next question
Question 142 of 147

A 39 year old lady has undergone surgery for breast cancer. As part of the histopathology report the
pathologist provides the surgeon with a Nottingham Prognostic Index score of 6.4. He also states
that the tumour size is 2cm. Which of the following inferences can be made in relation to this
statement?

The tumour is likely to be grade 1

Vascular invasion is present

Lymph node metastasis are definitely present

The tumour is oestrogen receptor positive

None of the above

A score of this value is unlikely to be reached with a grade 1 tumour and a size of 2cm. Therefore
lymph node metastasis are definitely present. In addition since the maximal score for lymph node
metastasis is 3 the tumour is likely be of a higher grade (see below). The Nottingham Prognostic
Index provides no information about oestrogen receptor status or the presence or absence of
vascular invasion.

Please rate this question:

Discuss and give feedback

Next question

Nottingham prognostic index

The Nottingham Prognostic Index can be used to give an indication of survival following breast
cancer surgery. In this system, the tumour size is weighted less heavily than other major prognostic
parameters.
Calculation of NPI
Tumour Size x 0.2 + Lymph node score(From table below)+Grade score(From table below).

Lymph nodes involved Grade


Score

1 0 1

2 1-3 2

3 >3 3

Prognosis

Score Percentage 5 year survival

2.0 to 2.4 93%

2.5 to 3.4 85%

3.5 to 5.4 70%

>5.4 50%

This data was originally published in 1992. It should be emphasised that other factors such as
vascular invasion and receptor status also impact on survival and are not included in this data and
account for varying prognoses often cited in the literature.

Reference
Galea, M.H., et al., The Nottingham Prognostic Index in primary breast cancer. Breast Cancer Res
Treat, 1992. 22(3): p. 207-19.
Question 143 of 147

In patients with multiple endocrine neoplasia type IIb which of the following clinical appearances is
the patient most likely to display?

Acromegalic facies

Turners type features

Profound kyphoscoliosis

Multiple bony exostoses

Marfanoid features

Patients with MEN IIb may display Marfanoid features. It is unclear at the present time whether they
have discrete changes in the microfibrils of elastic fibres that are present in Marfans.

Please rate this question:

Discuss and give feedback

Next question

Multiple Endocrine Neoplasia

Multiple endocrine neoplasia (MEN) is inherited as an autosomal dominant disorder.

The table below summarises the three main types of MEN:

MEN type I MEN type IIa MEN type IIb


Mnemonic 'three P's': Phaeochromocytoma Same as MEN IIa with
Medullary thyroid cancer addition of:
Parathyroid (95%): Parathyroid adenoma (70%) Marfanoid body
Pituitary (70%): Prolactinoma/ACTH/Growth Hyperparathyroidism habitus
Hormone secreting adenoma (60%) Mucosal neuromas
Pancreas (50%): Islet cell tumours/Zollinger Ellison
syndrome

also: Adrenal (adenoma) and thyroid (adenoma)

MENIN gene (chromosome 11) RET oncogene RET oncogene


(chromosome 10) (chromosome 10)
Most common presentation = hypercalcaemia

Next question
Question 144 of 147

A 32 year old man undergoes an appendicectomy. A large carcinoid tumour is identified and a
completion right hemicolectomy is performed. He is well for several months and then develops
symptoms of palpitations and facial flushing. Which of the following diagnostic markers should be
requested?

Alpha feto protein

Urinary 5-Hydroxyindoleacetic acid measurements

Urinary catecholamines

Urinary VMA measurements

None of the above

5 HIAA is the most commonly used diagnostic marker for carcinoid syndrome, it is measured in a 24
hour urine collection.
Please rate this question:

Discuss and give feedback


Next question

Carcinoid syndrome

 Carcinoid tumours secrete serotonin


 Originate in neuroendocrine cells mainly in the intestine (midgut-distal ileum/appendix)
 Can occur in the rectum, bronchi
 Hormonal symptoms mainly occur when disease spreads outside the bowel

Clinical features

 Onset: insidious over many years


 Flushing face
 Palpitations
 Pulmonary valve stenosis and tricuspid regurgitation causing dyspnoea
 Asthma
 Severe diarrhoea (secretory, persists despite fasting)
Investigation

 5-HIAA in a 24-hour urine collection


 Somatostatin receptor scintigraphy
 CT scan
 Blood testing for chromogranin A

Treatment

 Octreotide
 Surgical removal

Next question
Question 145 of 147

A 20 year old male is referred to the clinic. He has undergone genetic testing because his father died
from colorectal cancer at the age of 21. His testing revealed a mutation of the APC gene. A
colonoscopy is proposed. What is the most likely finding?

Multiple colonic hamartomas

Carpet villous adenoma of the rectum

Caecal carcinoma

Multiple colonic adenomas

Multiple colonic hyperplastic polyps

APC mutations are found in familial adenomatous polyposis coli. These have multiple colonic
adenomas.

Please rate this question:

Discuss and give feedback

Next question

Polyposis syndromes

Screening and Associated


Syndrome Genetic defect Features management disorders

Familial Mutation of APC Typically over 100 If known to be at risk Gastric fundal
Screening and Associated
Syndrome Genetic defect Features management disorders

adenomatous gene (80%) cases, colonic adenomas then predictive polyps (50%).
polyposis dominant Cancer risk of 100% genetic testing as Duodenal polyps
20% are new teenager 90%.
mutations Annual flexible If severe
sigmoidoscopy from duodenal
15 years polyposis cancer
If no polyps found risk of 30% at 10
then 5 yearly years.
colonoscopy started Abdominal
at age 20 desmoid
Polyps found = tumours.
resectional surgery
(resection and pouch
Vs sub total
colectomy and IRA)

MYH Biallelic mutation Multiple colonic Once identified Duodenal


associated of mut Y human polyps resection and polyposis in 30%
polyposis homologue (MYH) Later onset right ileoanal pouch Associated with
on chromosome sided cancers more reconstruction is increased risk of
1p, recessive common than in FAP recommended breast cancer
100% cancer risk by Attenuated (self examination)
age 60 phenotype - regular
colonoscopy

Peutz -Jeghers STK11 (LKB1) Multiple benign Annual examination Malignancies at


syndrome mutation on intestinal Pan intestinal other sites
chromosome 19 in hamartomas endoscopy every 2-3 Classical
some (but not all) Episodic obstruction years pigmentation
cases, dominant and intussceception pattern
Increased risk of GI
cancers (colorectal
cancer 20%, gastric
5%)
Increased risk of
Screening and Associated
Syndrome Genetic defect Features management disorders

breast, ovarian,
cervical pancreatic
and testicular
cancers

Cowden Mutation of PTEN Macrocephaly Targeted Breast cancer


disease gene on Multiple intestinal individualised (81% risk)
chromosome hamartomas screening Thyroid cancer
10q22, dominant Multiple and non toxic
trichilemmomas goitre
89% risk of cancer at Uterine cancer
any site
16% risk of colorectal
cancer

HNPCC (Lynch Germline Colo rectal cancer Colonoscopy every 1- Extra colonic
syndrome) mutations of DNA 30-70% 2 years from age 25 cancers
mismatch repair Endometrial cancer Consideration of
genes 30-70% prophylactic surgery
Gastric cancer 5-10% Extra colonic
Scanty colonic polyps surveillance
may be present recommended
Colonic tumours
likely to be right
sided and mucinous

Next question
Question 146 of 147

Which of the following breast tumours is most commonly associated with a risk of metastasis to the
contralateral breast?

Invasive ductal carcinoma

Invasive lobular carcinoma

Phyllodes tumour

Pagets disease of the breast

Atypical ductal hyperplasia

Risk of metastasis to the contralateral breast is a classical feature of invasive lobular carcinoma.

Please rate this question:

Discuss and give feedback

Next question

Lobular carcinoma of the breast

Lobular breast cancers are less common than their ductal counterparts. They typically present
differently, the mass is usually more diffuse and less obvious on the usual imaging modalities of
ultrasound and mammography. This is significant since the disease may be understaged resulting in
inadequate treatment when wide local excision is undertaken.
In women with invasive lobular carcinoma it is usually safest to perform an MRI scan of the breast, if
breast conserving surgery is planned.
Lobular carcinomas are also more likely to be multifocal and metastasise to the contralateral breast.
Lobular carcinoma in situ is occasionally diagnosed incidentally on core biopsies. Unlike DCIS,
lobular carcinoma in situ is far less strongly associated with foci of invasion and is usually managed
by close monitoring.

Next question
Question 147 of 147

Which is the characteristic finding on a blood film post splenectomy?

Stipple cell

Tear drop cell

Reticulocytes

Howell-Jolly bodies

Schistocyte

Blood film in hyposplenism:

Howell-Jolly bodies
Pappenheimer bodies
Poikilocytes (Target cells)
Erythrocyte containing siderotic granules
Heinz bodies
Please rate this question:

Discuss and give feedback

Splenectomy

Indications

 Trauma: 1/4 are iatrogenic


 Spontaneous rupture: EBV
 Hypersplenism: hereditary spherocytosis or elliptocytosis etc
 Malignancy: lymphoma or leukaemia
 Splenic cysts, hydatid cysts, splenic abscesses

Splenectomy
Technique
Trauma
 GA
 Long midline incision
 If time permits insert a self retaining retractor (e.g. Balfour/ omnitract)
 Large amount of free blood is usually present. Pack all 4 quadrants of the abdomen. Allow
the anaesthetist to 'catch up'
 Remove the packs and assess the viability of the spleen. Hilar injuries and extensive
parenchymal lacerations will usually require splenectomy.
 Divide the short gastric vessels and ligate them.
 Clamp the splenic artery and vein. Two clamps on the patient side are better and allow for
double ligation and serve as a safety net if your assistant does not release the clamp
smoothly.
 Be careful not to damage the tail of the pancreas, if you do then this will need to be formally
removed and the pancreatic duct closed.
 Wash out the abdomen and place a tube drain to the splenic bed.
 Some surgeons implant a portion of spleen into the omentum, whether you decide to do this
is a matter of personal choice.
 Post operatively the patient will require prophylactic penicillin V and pneumococcal vaccine.

Elective
Elective splenectomy is a very different operation from that performed in the emergency setting. The
spleen is often large (sometimes massive). Most cases can be performed laparoscopically. The
spleen will often be macerated inside a specimen bag to facilitate extraction.

Complications

 Haemorrhage (may be early and either from short gastrics or splenic hilar vessels
 Pancreatic fistula (from iatrogenic damage to pancreatic tail)
 Thrombocytosis: prophylactic aspirin
 Encapsulated bacteria infection e.g. Strep. pneumoniae, Haemophilus
influenzae and Neisseriameningitidis

Post splenectomy changes

 Platelets will rise first (therefore in ITP should be given after splenic artery clamped)
 Blood film will change over following weeks, Howell Jolly bodies will appear
 Other blood film changes include target cells and Pappenheimer bodies
 Increased risk of post splenectomy sepsis, therefore prophylactic antibiotics and
pneumococcal vaccine should be given.

Post splenectomy sepsis

 Typically occurs with encapsulated organisms


 Opsonisation occurs but then not recognised
Question 1 of 560

Which of the following structures is not transmitted by the jugular foramen?

Hypoglossal nerve

Accessory nerve

Internal jugular vein

Inferior petrosal sinus

Vagus nerve

Contents of the jugular foramen:

Anterior: inferior petrosal sinus


Intermediate: glossopharyngeal, vagus, and accessory nerves
Posterior: sigmoid sinus (becoming the internal jugular vein) and some meningeal branches from the
occipital and ascending pharyngeal arteries

Theme from 2009 exam


The jugular foramen may be divided into three compartments:

 Anterior compartment transmits the inferior petrosal sinus


 Middle compartment transmits cranial nerves IX, X and XI
 Posterior compartment transmits the sigmoid sinus

Please rate this question:

Discuss and give feedback


Next question

Foramina of the base of the skull

Foramen Location Contents


Foramen Location Contents

Foramen ovale Sphenoid Otic ganglion


bone V3 (Mandibular nerve:3rd branch of
trigeminal)
Accessory meningeal artery
Lesser petrosal nerve
Emissary veins

Foramen spinosum Sphenoid Middle meningeal artery


bone Meningeal branch of the Mandibular nerve

Foramen rotundum Sphenoid Maxillary nerve (V2)


bone

Foramen lacerum/ Sphenoid Base of the medial pterygoid plate.


carotid canal bone Internal carotid artery*
Nerve and artery of the pterygoid canal

Jugular foramen Temporal Anterior: inferior petrosal sinus


bone Intermediate: glossopharyngeal, vagus, and accessory nerves.
Posterior: sigmoid sinus (becoming the internal jugular vein)
and some meningeal branches from the occipital and
ascending pharyngeal arteries.

Foramen magnum Occipital Anterior and posterior spinal arteries


bone Vertebral arteries
Medulla oblongata

Stylomastoid Temporal Stylomastoid artery


foramen bone Facial nerve

Superior orbital Sphenoid Oculomotor nerve (III)


fissure bone Recurrent meningeal artery
Trochlear nerve (IV)
Lacrimal, frontal and nasociliary branches of ophthalmic
nerve (V1)
Foramen Location Contents

Abducent nerve (VI)


Superior ophthalmic vein

*= In life the foramen lacerum is occluded by a cartilagenous plug. The ICA initially passes into the
carotid canal which ascends superomedially to enter the cranial cavity through the foramen lacerum.

Base of skull anatomical overview

Image sourced from Wikipedia

Next question
Question 2 of 560

A 19 year old female is admitted with suspected meningitis. The House Officer is due to perform a
lumbar puncture. What is the most likely structure first encountered when the needle is inserted?

Ligamentum flavum

Denticulate ligament

Dural sheath

Pia Mater

Supraspinous ligament

Theme from September 2012 exam


Please rate this question:

Discuss and give feedback


Next question

Lumbar puncture

Lumbar punctures are performed to obtain cerebrospinal fluid. In adults, the procedure is best
performed at the level of L3/L4 or L4/5 interspace. These regions are below the termination of the
spinal cord at L1.

During the procedure the needle passes through:

 The supraspinous ligament which connects the tips of spinous processes and the
interspinous ligaments between adjacent borders of spinous processes
 Then the needle passes through the ligamentum flavum, which may cause a give as it is
penetrated
 A second give represents penetration of the needle through the dura mater into the
subarachnoid space. Clear CSF should be obtained at this point

References
Boon et al Lumbar Puncture: Anatomical Review of a Clinical Skill. Clinical Anatomy 17:544553
(2004)
Question 3 of 560

A 45 year old motor cyclist sustains a tibial fracture and is noted to have anaesthesia of the web
space between his first and second toes. Which of the nerves listed below is most likely to be
compromised?

Superficial peroneal nerve

Deep peroneal nerve

Sural nerve

Long saphenous nerve

Tibial nerve

Theme from September 2014 Exam


The deep peroneal nerve lies in the anterior muscular compartment of the lower leg and can be
compromised by compartment syndrome affecting this area. It provides cutaneous sensation to the
first web space. The superficial peroneal nerve provides more lateral cutaneous innervation.
Please rate this question:

Discuss and give feedback


Next question

Deep peroneal nerve

Origin From the common peroneal nerve, at the lateral aspect of the fibula, deep to
peroneus longus

Nerve root values L4, L5, S1, S2

Course and  Pierces the anterior intermuscular septum to enter the anterior
relation compartment of the lower leg
 Passes anteriorly down to the ankle joint, midway between the two
malleoli

Terminates In the dorsum of the foot

Muscles  Tibialis anterior


innervated  Extensor hallucis longus
 Extensor digitorum longus
 Peroneus tertius
 Extensor digitorum brevis

Cutaneous Web space of the first and second toes


innervation

Actions  Dorsiflexion of ankle joint


 Extension of all toes (extensor hallucis longus and extensor
digitorum longus)
 Inversion of the foot

After its bifurcation past the ankle joint, the lateral branch of the deep peroneal nerve innervates the
extensor digitorum brevis and the extensor hallucis brevis
The medial branch supplies the web space between the first and second digits.
Next question
Question 4 of 560

A patient undergoes a femoral hernia repair and at operation the surgeon decides to enter the
abdominal cavity to resect small bowel. She makes a transverse incision two thirds of the way
between umbilicus and the symphysis pubis. Which of the structures listed below will not be divided?

Rectus abdominis

External oblique aponeurosis

Peritoneum

Fascia transversalis

Posterior lamina of the rectus sheath

An incision at this level lies below the arcuate line and the posterior wall of the rectus sheath is
deficient at this level.

Please rate this question:

Discuss and give feedback

Next question

Rectus abdominis muscle

The rectus sheath is formed by the aponeuroses of the lateral abdominal wall muscles. The rectus
sheath has a composition that varies according to anatomical level.

1. Above the costal margin the anterior sheath is composed of external oblique aponeurosis, the
costal cartilages are posterior to it.
2. From the costal margin to the arcuate line, the anterior rectus sheath is composed of external
oblique aponeurosis and the anterior part of the internal oblique aponeurosis. The posterior part of
the internal oblique aponeurosis and transversus abdominis form the posterior rectus sheath.
3. Below the arcuate line the aponeuroses of all the abdominal muscles lie in anterior aspect of the
rectus sheath. Posteriorly lies the transversalis fascia and peritoneum.

The arcuate line is the point at which the inferior epigastric vessels enter the rectus sheath.

Next question
Question 5 of 560

What is the lymphatic drainage of the ovaries?

Internal iliac nodes

Common iliac nodes

Para-aortic nodes

Para uterine nodes

Inguinal nodes

Theme from September 2015 Exam


The lymphatic drainage of the ovary follows the gonadal vessels and drainage is therefore to the
para-aortic nodes.
Please rate this question:

Discuss and give feedback


Next question

Lymphatic drainage of the ovaries, uterus and cervix

 The ovaries drain to the para-aortic lymphatics via the gonadal vessels.
 The uterine fundus has a lymphatic drainage that runs with the ovarian vessels and may thus
drain to the para-aortic nodes. Some drainage may also pass along the round ligament to the
inguinal nodes.
 The body of the uterus drains through lymphatics contained within the broad ligament to the
iliac lymph nodes.
 The cervix drains into three potential nodal stations; laterally through the broad ligament to
the external iliac nodes, along the lymphatics of the uterosacral fold to the presacral nodes
and posterolaterally along lymphatics lying alongside the uterine vessels to the internal iliac
nodes.

Next question
Question 6-8 of 560

Theme: Axillary anatomy

A. Medial pectoral nerve

B. Thoracodorsal nerve

C. Lateral pectoral nerve

D. Intercostobrachial nerve

E. Medial cord of the brachial plexus

F. Long thoracic nerve

G. Axillary nerve

H. Accessory nerve

Please identify the structure that is most likely to be affected in the scenarios described below. Each
structure may be used once, more than once or not at all.

6. A 44 year old lady has undergone a mastectomy and axillary node clearance. Post operatively, she
notices a patch of anaesthesia of her axillary skin when she applies an underarm deodorant.

You answered Medial pectoral nerve

The correct answer is Intercostobrachial nerve

The intercostobrachial nerves traverse the axilla and innervate the overlying skin. These can be
injured or divided during axillary surgery and the result is anaesthesia of the overlying skin.

7. A 44 year old lady has undergone a mastectomy and axillary node clearance to treat breast cancer.
Post operatively, it is noted that she has winging of the scapula.

You answered Medial pectoral nerve


The correct answer is Long thoracic nerve

Injury to the long thoracic nerve (which innervates the serratus anterior) can occur as it lies at the
medial aspect of the axilla, winging of the scapula will then result.

8. A 44 year old lady who works as an interior decorator has undergone a mastectomy and axillary
node clearance to treat breast cancer. Post operatively, she comments that her arm easily
becomes fatigued when she is painting walls.

You answered Medial pectoral nerve

The correct answer is Thoracodorsal nerve

The most likely explanation for this is that the thoracodorsal nerve has been injured. This will result
in atrophy of latissimus dorsi and this will become evident with repetitive arm movements where
the arm is elevated and moving up and down (such as in painting). Injury to the pectoral nerves
may produce a similar picture but this pattern of injury is very rare and the pectoral nerves are
seldom injured in breast surgery.

Please rate this question:

Discuss and give feedback

Next question

Axilla

Boundaries of the axilla

Medially Chest wall and Serratus anterior

Laterally Humeral head


Floor Subscapularis

Anterior aspect Lateral border of Pectoralis major

Fascia Clavipectoral fascia

Content:

Long thoracic nerve (of Derived from C5-C7 and passes behind the brachial plexus to enter the axilla.
Bell) It lies on the medial chest wall and supplies serratus anterior. Its location
puts it at risk during axillary surgery and damage will lead to winging of the
scapula.

Thoracodorsal nerve and Innervate and vascularise latissimus dorsi.


thoracodorsal trunk

Axillary vein Lies at the apex of the axilla, it is the continuation of the basilic vein.
Becomes the subclavian vein at the outer border of the first rib.

Intercostobrachial nerves Traverse the axillary lymph nodes and are often divided during axillary
surgery. They provide cutaneous sensation to the axillary skin.

Lymph nodes The axilla is the main site of lymphatic drainage for the breast.

Next question
Question 9 of 560

A 35 year old farm labourer injures the posterior aspect of his hand with a mechanical scythe. He
severs some of his extensor tendons in this injury. How many tunnels lie in the extensor retinaculum
that transmit the tendons of the extensor muscles?

One

Three

Four

Five

Six

There are six tunnels, each lined by its own synovial sheath.
Please rate this question:

Discuss and give feedback


Next question

Extensor retinaculum

The extensor rentinaculum is a thickening of the deep fascia that stretches across the back of the
wrist and holds the long extensor tendons in position.
Its attachments are:

 The pisiform and triquetral medially


 The end of the radius laterally

Structures related to the extensor retinaculum


Structures superficial to the  Basilic vein
retinaculum  Dorsal cutaneous branch of the ulnar nerve
 Cephalic vein
 Superficial branch of the radial nerve
Structures passing deep to the  Extensor carpi ulnaris tendon
extensor retinaculum  Extensor digiti minimi tendon
 Extensor digitorum and extensor indicis tendon
 Extensor pollicis longus tendon
 Extensor carpi radialis longus tendon
 Extensor carpi radialis brevis tendon
 Abductor pollicis longus and extensor pollicis
brevis tendons

Beneath the extensor retinaculum fibrous septa form six compartments that contain the extensor
muscle tendons. Each compartment has its own synovial sheath.

The radial artery


The radial artery passes between the lateral collateral ligament of the wrist joint and the tendons of
the abductor pollicis longus and extensor pollicis brevis.

Image illustrating the topography of tendons passing under the extensor retinaculum

Image sourced from Wikipedia

Next question

Question 10 of 560
A 23 year old man undergoes an orchidectomy. The right testicular vein is ligated; into which
structure does it drain?

Right renal vein

Inferior vena cava

Common iliac vein

Internal iliac vein

External iliac vein

Theme from April 2012 exam


Theme from April 2014 exam
The testicular venous drainage begins in the septa and these veins together with those of the tunica
vasculosa converge on the posterior border of the testis as the pampiniform plexus. The
pampiniform plexus drains to the testicular vein. The left testicular vein drains into the left renal vein.
The right testicular vein drains into the inferior vena cava.
Please rate this question:

Discuss and give feedback


Next question

Scrotal and testicular anatomy

Spermatic cord
Formed by the vas deferens and is covered by the following structures:
Layer Origin

Internal spermatic fascia Transversalis fascia

Cremasteric fascia From the fascial coverings of internal oblique


Layer Origin

External spermatic fascia External oblique aponeurosis

Contents of the cord


Vas deferens Transmits sperm and accessory gland secretions

Testicular artery Branch of abdominal aorta supplies testis and


epididymis

Artery of vas deferens Arises from inferior vesical artery

Cremasteric artery Arises from inferior epigastric artery

Pampiniform plexus Venous plexus, drains into right or left testicular vein

Sympathetic nerve fibres Lie on arteries, the parasympathetic fibres lie on the
vas

Genital branch of the genitofemoral Supplies cremaster


nerve

Lymphatic vessels Drain to lumbar and para-aortic nodes

Scrotum

 Composed of skin and closely attached dartos fascia.


 Arterial supply from the anterior and posterior scrotal arteries
 Lymphatic drainage to the inguinal lymph nodes
 Parietal layer of the tunica vaginalis is the innermost layer

Testes
 The testes are surrounded by the tunica vaginalis (closed peritoneal sac). The parietal layer
of the tunica vaginalis adjacent to the internal spermatic fascia.
 The testicular arteries arise from the aorta immediately inferiorly to the renal arteries.
 The pampiniform plexus drains into the testicular veins, the left drains into the left renal vein
and the right into the inferior vena cava.
 Lymphatic drainage is to the para-aortic nodes.

Next question
Question 11 of 560

A 44 year old lady is undergoing an abdominal hysterectomy and the ureter is identified during the
ligation of the uterine artery. At which site does it insert into the bladder?

Posterior

Apex

Anterior

Base

Superior aspect of the lateral side

Theme from September 2012 Exam


The ureters enter the bladder at the upper lateral aspect of the base of the bladder. They are about
5cm apart from each other in the empty bladder. Internally this aspect is contained within the bladder
trigone.
Please rate this question:

Discuss and give feedback


Next question

Ureter

 25-35 cm long
 Muscular tube lined by transitional epithelium
 Surrounded by thick muscular coat. Becomes 3 muscular layers as it crosses the bony pelvis
 Retroperitoneal structure overlying transverse processes L2-L5
 Lies anterior to bifurcation of iliac vessels
 Blood supply is segmental; renal artery, aortic branches, gonadal branches, common iliac
and internal iliac
 Lies beneath the uterine artery

Next question
Question 12 of 560

What is the correct embryological origin of the stapes?

First pharyngeal arch

Second pharyngeal arch

Third pharyngeal arch

Fourth pharyngeal arch

Fifth pharyngeal arch

Embryological origin stapes = 2nd pharyngeal arch


The ectoderm covering the outer aspect of the second arch originates from a strip of ectoderm
lateral to the metencephalic neural fold. The cartilaginous element to this, eponymously known as
Reicherts cartilage extends from the otic capsule to the midline on each side. Its dorsal end
separates and becomes enclosed in the tympanic cavity as the stapes.

Theme from September 2013 exam


Theme from April 2012 Exam
The dorsal ends of the cartilages of the first and second pharyngeal arches articulate superior to the
tubotympanic recess. These cartilages form the malleus, incus and stapes. At least part of the
malleus is formed from the first arch and the stapes from the second arch. The incus is most likely to
arise from the first arch.

Please rate this question:

Discuss and give feedback

Next question

Pharyngeal arches
These develop during the fourth week of embryonic growth from a series of mesodermal
outpouchings of the developing pharynx.
They develop and fuse in the ventral midline. Pharyngeal pouches form on the endodermal side
between the arches.
There are 6 pharyngeal arches, the fifth does not contribute any useful structures and often fuses
with the sixth arch.

Pharyngeal arches

Pharyngeal Muscular Skeletal


arch contributions contributions Endocrine Artery Nerve

First Muscles of Maxilla n/a Maxillary Mandibular


mastication Meckels External
Anterior belly of cartilage carotid
digastric Incus
Mylohyoid Malleus
Tensor tympanic
Tensor veli palatini

Second Buccinator Stapes n/a Inferior Facial


Platysma Styloid process branch of
Muscles of facial Lesser horn superior
expression and upper thyroid artery
Stylohyoid body of hyoid Stapedial
Posterior belly of artery
digastric
Stapedius

Third Stylopharyngeus Greater horn Thymus Common and Glossopharyngeal


and lower part Inferior internal
of hyoid parathyroids carotid

Fourth Cricothyroid Thyroid and Superior Right- Vagus


All intrinsic epiglottic parathyroids subclavian
muscles of the soft cartilages artery, Left-
palate aortic arch
Pharyngeal Muscular Skeletal
arch contributions contributions Endocrine Artery Nerve

Sixth All intrinsic Cricoid, n/a Right - Vagus and


muscles of the arytenoid and Pulmonary recurrent
larynx (except corniculate artery, Left- laryngeal nerve
cricothyroid) cartilages Pulmonary
artery and
ductus
arteriosus

Next question
Question 13 of 560

A 20 year old lady presents with pain on the medial aspect of her thigh. Investigations show a large
ovarian cyst. Compression of which of the nerves listed below is the most likely underlying cause?

Sciatic

Genitofemoral

Obturator

Ilioinguinal

Femoral cutaneous

Theme from April 2012 Exam


The cutaneous branch of the obturator nerve is frequently absent. However, the obturator nerve is a
recognised contributor to innervation of the medial thigh and large pelvic tumours may compress this
nerve with resultant pain radiating distally.
Please rate this question:

Discuss and give feedback


Next question

Obturator nerve

The obturator nerve arises from L2, L3 and L4 by branches from the ventral divisions of each of
these nerve roots. L3 forms the main contribution and the second lumbar branch is occasionally
absent. These branches unite in the substance of psoas major, descending vertically in its posterior
part to emerge from its medial border at the lateral margin of the sacrum. It then crosses the
sacroiliac joint to enter the lesser pelvis, it descends on obturator internus to enter the obturator
groove. In the lesser pelvis the nerve lies lateral to the internal iliac vessels and ureter, and is joined
by the obturator vessels lateral to the ovary or ductus deferens.

Supplies

 Medial compartment of thigh


 Muscles supplied: external obturator, adductor longus, adductor brevis, adductor magnus
(not the lower part-sciatic nerve), gracilis
 The cutaneous branch is often absent. When present, it passes between gracilis and
adductor longus near the middle part of the thigh, and supplies the skin and fascia of the
distal two thirds of the medial aspect.

Obturator canal

 Connects the pelvis and thigh: contains the obturator artery, vein, nerve which divides into
anterior and posterior branches.

Cadaveric cross section demonstrating relationships of the obturator nerve

Image sourced from Wikipedia

Next question
Question 14 of 560

A 73 year old man presents with a tumour at the central aspect of the posterior third of the tongue.
To which of the following lymph node groups is it most likely to metastasise?

Submental

Submandibular

Ipsilateral deep cervical nodes

Contralateral deep cervical nodes

Bilateral deep cervical nodes

Posterior third tumours of the tongue commonly metastasise to the bilateral deep cervical lymph
nodes

Tumours of the posterior third of the tongue will typically metastasise early and bilateral nodal
involvement is well recognised, this is most often true of centrally located tumours and those
adjacent to the midline as the lymph vessels may cross the median plane at this location.
Please rate this question:

Discuss and give feedback


Next question

Lymphatic drainage of the tongue

 The lymphatic drainage of the anterior two thirds of the tongue shows only minimal
communication of lymphatics across the midline, so metastasis to the ipsilateral nodes is
usual.
 The lymphatic drainage of the posterior third of the tongue have communicating networks, as
a result early bilateral nodal metastases are more common in this area.
 Lymphatics from the tip of the tongue usually pass to the sub mental nodes and from there to
the deep cervical nodes.
 Lymphatics from the mid portion of the tongue usually drain to the submandibular nodes and
then to the deep cervical nodes. Mid tongue tumours that are laterally located will usually
drain to the ipsilateral deep cervical nodes, those from more central regions may have
bilateral deep cervical nodal involvement.

Next question
Question 15 of 560

A 6 month old child is brought to the surgical clinic because of non descended testes. What is the
main structure that determines the descent path of the testicle?

Processus vaginalis

Cremaster

Mesorchium

Inguinal canal

Gubernaculum

Theme from April 2012 exam


Theme from April 2014 exam
The gubernaculum is a ridge of mesenchymal tissue that connects the testis to the inferior aspect of
the scrotum. Early in embryonic development the gubernaculum is long and the testis are located on
the posterior abdominal wall. During foetal growth the body grows relative to the gubernaculum, with
resultant descent of the testis.

Please rate this question:

Discuss and give feedback

Next question

Testicular embryology

Until the end of foetal life the testicles are located within the abdominal cavity. They are initially
located on the posterior abdominal wall on a level with the upper lumbar vertebrae (L2). Attached to
the inferior aspect of the testis is the gubernaculum testis which extends caudally to the inguinal
region, through the canal and down to the superficial skin. Both the testis and the gubernaculum are
extra-peritoneal.
As the foetus grows the gubernaculum becomes progressively shorter. It carries the peritoneum of
the anterior abdominal wall (the processus vaginalis). As the processus vaginalis descends the testis
is guided by the gubernaculum down the posterior abdominal wall and the back of the processus
vaginalis into the scrotum.
By the third month of foetal life the testes are located in the iliac fossae, by the seventh they lie at
the level of the deep inguinal ring.

The processus vaginalis usually closes after birth, but may persist and be the site of indirect hernias.
Part closure may result in development of cysts on the cord.

Next question
Question 16 of 560

A 21 year old man undergoes surgical removal of an impacted 3rd molar. Post operatively, he is
noted to have anaesthesia on the anterolateral aspect of the tongue. What is the most likely
explanation?

Injury to the hypoglossal nerve

Injury to the inferior alveolar nerve

Injury to the lingual nerve

Injury to the mandibular branch of the facial nerve

Injury to the glossopharyngeal nerve

Theme from September 2014 exam


The lingual nerve is closely related to the third molar and up to 10% of patients undergoing surgical
extraction of these teeth may subsequently develop a lingual neuropraxia. The result is anaesthesia
of the ipsilateral anterior aspect of the tongue. The inferior alveolar nerve innervates the teeth
themselves.
Please rate this question:

Discuss and give feedback


Next question

Lingual nerve

 Sensory nerve to the mucosa of the presulcal part of the tongue, floor of mouth and
mandibular lingual gingivae
 Arises from posterior trunk of the mandibular nerve
 Course runs past tensor veli palatini and lateral pterygoid (where it is joined by the chorda
tympani branch of the facial nerve). Emerging from the cover of the lateral pterygoid it
proceeds antero inferiorly lying on the surface of the medial pterygoid and lies close to the
medial aspect of the mandibular ramus. At the junction of the vertical and horizontal rami of
the mandible it is anterior to the inferior alveolar nerve. It then passes below the mandibular
attachment of the superior pharyngeal constrictor. Eventually, it lies on the periosteum of the
root of the third molar tooth. It then passes medial to the mandibular origin of mylohyoid and
then passes forwards on the inferior surface of this muscle

Next question
Question 17 of 560

What is the most important structure involved in supporting the uterus?

Round ligament

Broad ligament

Uterosacral ligaments

Cardinal ligaments

Central perineal tendon

Theme from April 2016 Exam


The central perineal tendon provides the main structural support to the uterus. Damage to this
structure is commonly associated with the development of pelvic organ prolapse, even when other
structures are intact.

Please rate this question:

Discuss and give feedback

Next question

Uterus

The non pregnant uterus resides entirely within the pelvis. The peritoneum invests the uterus and
the structure is contained within the peritoneal cavity. The blood supply to the uterine body is via the
uterine artery (branch of the internal iliac). The uterine artery passes from the inferior aspect of the
uterus (lateral to the cervix) and runs alongside the uterus. It frequently anastomoses with the
ovarian artery superiorly. Inferolaterally the ureter is a close relation and ureteric injuries are a
recognised complication when pathology brings these structures into close proximity.
The supports of the uterus include the central perineal tendon (the most important). The lateral
cervical, round and uterosacral ligaments are condensations of the endopelvic fascia and provide
additional structural support.

Topography of the uterus

Image sourced from Wikipedia

Next question
Question 18 of 560

A 34 year old lady suffers from hyperparathyroidism. The right inferior parathyroid is identified as
having an adenoma and is scheduled for resection. From which of the following embryological
structures is it derived?

Second pharyngeal pouch

Third pharyngeal pouch

Fourth pharyngeal pouch

First pharyngeal pouch

None of the above

The inferior parathyroid is a derivative of the third pharyngeal pouch. The superior parathyroid
originates from the fourth pharyngeal pouch.
Please rate this question:

Discuss and give feedback


Next question

Parathyroid glands- anatomy

 Four parathyroid glands


 Located posterior to the thyroid gland
 They lie within the pretracheal fascia

Embryology
The parathyroids develop from the extremities of the third and fourth pharyngeal pouches. The
parathyroids derived from the fourth pharyngeal pouch are located more superiorly and are
associated with the thyroid gland. Those derived from the third pharyngeal pouch lie more inferiorly
and may become associated with the thymus.

Blood supply
The blood supply to the parathyroid glands is derived from the inferior and superior thyroid
arteries[1]. There is a rich anastomosis between the two vessels. Venous drainage is into the thyroid
veins.
Relations
Laterally Common carotid

Medially Recurrent laryngeal nerve, trachea

Anterior Thyroid

Posterior Pretracheal fascia

References
1.Nobori, M., et al., Blood supply of the parathyroid gland from the superior thyroid artery. Surgery,
1994. 115(4): p. 417-23.
Next question
Question 19 of 560

A 23 year old man falls and slips at a nightclub. A shard of glass penetrates the skin at the level of
the medial epicondyle, which of the following sequelae is least likely to occur?

Atrophy of the first dorsal interosseous muscle

Difficulty in abduction of the the 2nd, 3rd, 4th and 5th fingers

Claw like appearance of the hand

Loss of sensation on the anterior aspect of the 5th finger

Partial denervation of flexor digitorum profundus

Injury to the ulnar nerve in the mid to distal forearm will typically produce a claw hand. This consists
of flexion of the 4th and 5th interphalangeal joints and extension of the metacarpophalangeal joints.
The effects are potentiated when flexor digitorum profundus is not affected, and the clawing is more
pronounced.More proximally sited ulnar nerve lesions produce a milder clinical picture owing to the
simultaneous paralysis of flexor digitorum profundus (ulnar half).

This is the 'ulnar paradox', due to the more proximal level of transection the hand will typically not
have a claw like appearance that may be seen following a more distal injury. The first dorsal
interosseous muscle will be affected as it is supplied by the ulnar nerve.
Please rate this question:

Discuss and give feedback


Next question

Ulnar nerve

Origin

 C8, T1

Supplies (no muscles in the upper arm)

 Flexor carpi ulnaris


 Flexor digitorum profundus
 Flexor digiti minimi
 Abductor digiti minimi
 Opponens digiti minimi
 Adductor pollicis
 Interossei muscle
 Third and fourth lumbricals
 Palmaris brevis

Path

 Posteromedial aspect of upper arm to flexor compartment of forearm, then along the ulnar.
Passes beneath the flexor carpi ulnaris muscle, then superficially through the flexor
retinaculum into the palm of the hand.

Image sourced from Wikipedia

Branches
Branch Supplies

Muscular branch Flexor carpi ulnaris


Medial half of the flexor digitorum profundus

Palmar cutaneous branch (Arises near the Skin on the medial part of the palm
middle of the forearm)

Dorsal cutaneous branch Dorsal surface of the medial part of the hand

Superficial branch Cutaneous fibres to the anterior surfaces of the


medial one and one-half digits

Deep branch Hypothenar muscles


All the interosseous muscles
Third and fourth lumbricals
Adductor pollicis
Medial head of the flexor pollicis brevis

Effects of injury
Damage at the wrist  Wasting and paralysis of intrinsic hand muscles (claw hand)
 Wasting and paralysis of hypothenar muscles
 Loss of sensation medial 1 and half fingers

Damage at the elbow  Radial deviation of the wrist


 Clawing less in 4th and 5th digits

Next question
Question 20 of 560

A 56 year old man is undergoing a superficial parotidectomy for a pleomorphic adenoma. During the
dissection of the parotid, which of the following structures will be encountered lying most
superficially?

Facial nerve

External carotid artery

Occipital artery

Maxillary artery

Retromandibular vein

Most superficial structure on the parotid gland = facial nerve

The facial nerve is the most superficial structure in the parotid gland. Slightly deeper to this lies the
retromandibular vein, with the arterial layer lying most deeply.
Please rate this question:

Discuss and give feedback


Next question

Parotid gland

Anatomy of the parotid gland


Location Overlying the mandibular ramus; anterior and inferior to the ear.

Salivary duct Crosses the masseter, pierces the buccinator and drains adjacent to the
2nd upper molar tooth (Stensen's duct).

Structures passing  Facial nerve (Mnemonic: The Zebra Buggered My Cat; Temporal
through the gland Zygomatic, Buccal, Mandibular, Cervical)
 External carotid artery
 Retromandibular vein
 Auriculotemporal nerve

Relations  Anterior: masseter, medial pterygoid, superficial temporal and


maxillary artery, facial nerve, stylomandibular ligament
 Posterior: posterior belly digastric muscle, sternocleidomastoid,
stylohyoid, internal carotid artery, mastoid process, styloid
process

Arterial supply Branches of external carotid artery

Venous drainage Retromandibular vein

Lymphatic drainage Deep cervical nodes

Nerve innervation  Parasympathetic-Secretomotor


 Sympathetic-Superior cervical ganglion
 Sensory- Greater auricular nerve

Parasympathetic stimulation produces a water rich, serous saliva. Sympathetic stimulation leads to
the production of a low volume, enzyme-rich saliva.
Next question
Question 21 of 560

A 43 year old man is stabbed outside a nightclub. He suffers a transection of his median nerve just
as it leaves the brachial plexus. Which of the following features is least likely to ensue?

Ulnar deviation of the wrist

Complete loss of wrist flexion

Loss of pronation

Loss of flexion at the thumb joint

Inability to oppose the thumb

Loss of the median nerve will result in loss of function of the flexor muscles. However, flexor carpi
ulnaris will still function and produce ulnar deviation and some residual wrist flexion. High median
nerve lesions result in complete loss of flexion at the thumb joint.
Please rate this question:

Discuss and give feedback


Next question

Median nerve

The median nerve is formed by the union of a lateral and medial root respectively from the lateral
(C5,6,7) and medial (C8 and T1) cords of the brachial plexus; the medial root passes anterior to the
third part of the axillary artery. The nerve descends lateral to the brachial artery, crosses to its
medial side (usually passing anterior to the artery). It passes deep to the bicipital aponeurosis and
the median cubital vein at the elbow.
It passes between the two heads of the pronator teres muscle, and runs on the deep surface of
flexor digitorum superficialis (within its fascial sheath).
Near the wrist it becomes superficial between the tendons of flexor digitorum superficialis and flexor
carpi radialis, deep to palmaris longus tendon. It passes deep to the flexor retinaculum to enter the
palm, but lies anterior to the long flexor tendons within the carpal tunnel.

Branches
Region Branch
Region Branch

Upper arm No branches, although the nerve commonly communicates with the
musculocutaneous nerve

Forearm Pronator teres


Flexor carpi radialis
Palmaris longus
Flexor digitorum superficialis
Flexor pollicis longus
Flexor digitorum profundus (only the radial half)

Distal Palmar cutaneous branch


forearm

Hand Motor supply (LOAF)


(Motor)

 Lateral 2 lumbricals
 Opponens pollicis
 Abductor pollicis brevis
 Flexor pollicis brevis

Hand  Over thumb and lateral 2 ½ fingers


(Sensory)  On the palmar aspect this projects proximally, on the dorsal aspect only the
distal regions are innervated with the radial nerve providing the more
proximal cutaneous innervation.

Patterns of damage
Damage at wrist

 e.g. carpal tunnel syndrome


 paralysis and wasting of thenar eminence muscles and opponens pollicis (ape hand
deformity)
 sensory loss to palmar aspect of lateral (radial) 2 ½ fingers

Damage at elbow, as above plus:


 unable to pronate forearm
 weak wrist flexion
 ulnar deviation of wrist

Anterior interosseous nerve (branch of median nerve)

 leaves just below the elbow


 results in loss of pronation of forearm and weakness of long flexors of thumb and index
finger

Topography of the median nerve

Image sourced from Wikipedia

Next question
Question 22 of 560

A 78 year old man is due to undergo an endarterectomy of the internal carotid artery. Which of the
following nervous structures are most at risk during the dissection?

Recurrent laryngeal nerve

Sympathetic chain

Hypoglossal nerve

Phrenic nerve

Lingual nerve

Nerves at risk during a carotid endarterectomy:

 Hypoglossal nerve
 Greater auricular nerve
 Superior laryngeal nerve

Theme from May 2011 exam


Theme from January 2013 Exam
During a carotid endarterectomy the sternocleidomastoid muscle is dissected, with ligation of the
common facial vein and then the internal jugular is dissected exposing the common and the internal
carotid arteries. The nerves at risk during the operation include:

 Hypoglossal nerve
 Greater auricular nerve
 Superior laryngeal nerve

The sympathetic chain lies posteriorly and is less prone to injury in this procedure.
Please rate this question:

Discuss and give feedback


Next question

Internal carotid artery


The internal carotid artery is formed from the common carotid opposite the upper border of the
thyroid cartilage. It extends superiorly to enter the skull via the carotid canal. From the carotid canal
it then passes through the cavernous sinus, above which it divides into the anterior and middle
cerebral arteries.

Relations in the neck


Posterior  Longus capitis
 Pre-vertebral fascia
 Sympathetic chain
 Superior laryngeal nerve

Medially  External carotid (near origin)


 Wall of pharynx
 Ascending pharyngeal artery

Laterally  Internal jugular vein (moves posteriorly at entrance to skull)


 Vagus nerve (most posterolaterally)

Anteriorly  Sternocleidomastoid
 Lingual and facial veins
 Hypoglossal nerve

Relations in the carotid canal

 Internal carotid plexus


 Cochlea and middle ear cavity
 Trigeminal ganglion (superiorly)
 Leaves canal lies above the foramen lacerum

Path and relations in the cranial cavity


The artery bends sharply forwards in the cavernous sinus, the aducens nerve lies close to its
inferolateral aspect. The oculomotor, trochlear, opthalmic and, usually, the maxillary nerves lie in the
lateral wall of the sinus. Near the superior orbital fissure it turns posteriorly and passes postero-
medially to pierce the roof of the cavernous sinus inferior to the optic nerve. It then passes between
the optic and oculomotor nerves to terminate below the anterior perforated substance by dividing
into the anterior and middle cerebral arteries.

Branches

 Anterior and middle cerebral artery


 Ophthalmic artery
 Posterior communicating artery
 Anterior choroid artery
 Meningeal arteries
 Hypophyseal arteries

Image demonstrating the internal carotid artery and its relationship to the external carotid artery

Image sourced from Wikipedia

Next question
Question 23 of 560

Which of the structures listed below articulates with the head of the radius superiorly?

Capitulum

Trochlea

Lateral epicondyle

Ulna

Medial epicondyle

Theme from September 2016 Exam


The head of the radius articulates with the capitulum of the humerus.
Please rate this question:

Discuss and give feedback


Next question

Radius

The radius is one of the two long forearm bones that extends from the lateral side of the elbow to the
thumb side of the wrist. It has two expanded ends, of which the distal end is the larger. Key points
relating to its topography and relations are outlined below;

Upper end

 Articular cartilage- covers medial > lateral side


 Articulates with radial notch of the ulna by the annular ligament
 Muscle attachment- biceps brachii at the tuberosity

Shaft
Muscle attachment
Upper third of the body Supinator
Flexor digitorum superficialis
Flexor pollicis longus

Middle third of the body Pronator teres

Lower quarter of the body Pronator quadratus


Tendon of supinator longus

Lower end

 Quadrilateral
 Anterior surface- capsule of wrist joint
 Medial surface- head of ulna
 Lateral surface- ends in the styloid process
 Posterior surface: 3 grooves containing:

1. Tendons of extensor carpi radialis longus and brevis


2. Tendon of extensor pollicis longus
3. Tendon of extensor indicis

Image sourced from Wikipedia

Next question
Question 24 of 560

Which of the following fascial structures encases the apex of the lungs?

Waldeyers fascia

Sibsons fascia

Pretracheal fascia

Clavipectoral fascia

None of the above

Sibson's fascia overlies the apices of both lungs

The suprapleural fascia (Sibson's fascia) runs from C7 to the first rib and overlies the apex of both
lungs.It lies between the parietal pleura and the thoracic cage.
Please rate this question:

Discuss and give feedback


Next question

Lung anatomy

The right lung is composed of 3 lobes divided by the oblique and transverse fissures. The left lung
has two lobes divided by the oblique fissure.The apex of both lungs is approximately 4cm superior to
the sterno-costal joint of the first rib. Immediately below this is a sulcus created by the subclavian
artery.

Peripheral contact points of the lung

 Base: diaphragm
 Costal surface: corresponds to the cavity of the chest
 Mediastinal surface: Contacts the mediastinal pleura. Has the cardiac impression. Above and
behind this concavity is a triangular depression named the hilum, where the structures which
form the root of the lung enter and leave the viscus. These structures are invested by pleura,
which, below the hilum and behind the pericardial impression, forms the pulmonary ligament
Right lung
Above the hilum is the azygos vein; Superior to this is the groove for the superior vena cava and
right innominate vein; behind this, and nearer the apex, is a furrow for the innominate artery. Behind
the hilum and the attachment of the pulmonary ligament is a vertical groove for the oesophagus; In
front and to the right of the lower part of the oesophageal groove is a deep concavity for the
extrapericardiac portion of the inferior vena cava.

The root of the right lung lies behind the superior vena cava and the right atrium, and below the
azygos vein.

The right main bronchus is shorter, wider and more vertical than the left main bronchus and
therefore the route taken by most foreign bodies.

Image sourced from Wikipedia

Left lung
Above the hilum is the furrow produced by the aortic arch, and then superiorly the groove
accommodating the left subclavian artery; Behind the hilum and pulmonary ligament is a vertical
groove produced by the descending aorta, and in front of this, near the base of the lung, is the lower
part of the oesophagus.

The root of the left lung passes under the aortic arch and in front of the descending aorta.
Image sourced from Wikipedia

Inferior borders of both lungs

 6th rib in mid clavicular line


 8th rib in mid axillary line
 10th rib posteriorly

The pleura runs two ribs lower than the corresponding lung level.

Bronchopulmonary segments
Segment number Right lung Left lung

1 Apical Apical

2 Posterior Posterior

3 Anterior Anterior

4 Lateral Superior lingular

5 Medial Inferior lingular

6 Superior (apical) Superior (apical)


Segment number Right lung Left lung

7 Medial basal Medial basal

8 Anterior basal Anterior basal

9 Lateral basal Lateral basal

10 Posterior basal Posterior basal

Next question
Question 25 of 560

Which of the structures listed below inserts into the site labelled in the image.

© Image provided by the University of Sheffield

Psoas minor

Psoas major

Sartorius

Obturator externus

Gemellus

Psoas major inserts onto the lesser trochanter.


Please rate this question:
Discuss and give feedback
Next question

Femur

 Extends from a rounded head, which articulates with the acetabulum down to the knee joint
where the two large condyles at it's inferior aspect articulate with the tibia.
 The superior aspect comprises a head and neck which pass inferolaterally to the body and
the two trochanters. These lie at the junction between the neck and the body.
 The neck meets the body of the femur at an angle of 125o.
 Developmentally, the neck is part of the body but is demarcated from it by a wide rough
intertrochanteric crest, this continues inferomedially as a spiral line that runs below the lesser
trochanter. Medially, the intertrochanteric line gives attachment to the inferior end of the
iliofemoral ligament. The neck is covered by synovial membrane up to the intertrochanteric
line. The posterior aspect of the neck is demarcated from the shaft by the intertrochanteric
crest and only it's medial aspect is covered by synovium and the joint capsule.
 The greater trochanter has discernible surfaces that form the site of attachment of the gluteal
muscles.Laterally, the greater trochanter overhangs the body and this forms part of the origin
of vastus lateralis
 Viewed anteriorly, the body of the femur appears rounded. Viewed laterally, it has an anterior
concavity which gives fullness to the anterior thigh. Posteriorly, there is a ridge of bone, the
linea aspera. The surface of the anterior aspect of the body forms the origin of the vastus
intermedius. More medially, it forms the origin of vastus medialis.
 The upper and middle aspects of the linea aspera form part of the origin of the attachments
of the thigh adductors. Inferiorly, it spans out to form the bony floor of the popliteal fossa. At
the inferior aspect of the popliteal surface the surface curves posteriorly to form the femoral
condyles.
 The structures that are attached to the inferior aspect of the linea aspera split with it as it
approaches the popliteal fossa. Thus the vastus medialis and adductor magnus continue
with the medial split and the biceps femoris and vastus intermedius along the lateral split.
© Image provided by the University of Sheffield

Image demonstrating anterior aspect of femur with muscular attachments

© Image provided by the University of Sheffield

Blood supply
The femur has a rich blood supply and numerous vascular foramina exist throughout it's length. The
blood supply to the femoral head is clinically important and is provided by the medial circumflex
femoral and lateral circumflex femoral arteries (Branches of profunda femoris). Also from the inferior
gluteal artery. These form an anastomosis and travel to up the femoral neck to supply the head.
Next question
Question 26 of 560

As regards the internal jugular vein, which of the following statements is untrue?

It lies within the carotid sheath

It is the continuation of the sigmoid sinus

The terminal part of the thoracic duct crosses anterior to it to insert into the right subclavian
vein

The hypoglossal nerve is closely related to it as it passes near the atlas

The vagus nerve is closely related to it within the carotid sheath

Theme from April 2013 Exam

Please rate this question:

Discuss and give feedback

Next question

Internal jugular vein

Each jugular vein begins in the jugular foramen, where they are the continuation of the sigmoid
sinus. They terminate at the medial end of the clavicle where they unite with the subclavian vein.

The vein lies within the carotid sheath throughout its course. Below the skull the internal carotid
artery and last four cranial nerves are anteromedial to the vein. Thereafter it is in contact medially
with the internal (then common) carotid artery. The vagus lies posteromedially.

At its superior aspect, the vein is overlapped by sternocleidomastoid and covered by it at the inferior
aspect of the vein.
Below the transverse process of the atlas it is crossed on its lateral side by the accessory nerve. At
its mid point it is crossed by the inferior root of the ansa cervicalis.
Posterior to the vein are the transverse processes of the cervical vertebrae, the phenic nerve as it
descends on the scalenus anterior, and the first part of the subclavian artery.

On the left side its also related to the thoracic duct.

Image sourced from Wikipedia

Next question
Question 27 of 560

A 28 year old man requires a urethral catheter to be inserted prior to undergoing a splenectomy.
Where is the first site of resistance to be encountered on inserting the catheter?

Bulbar urethra

Membranous urethra

Internal sphincter

Prostatic urethra

Bladder neck

Theme from January 2012 Exam


Theme from April 2014 Exam
The membranous urethra is the least distensible portion of the urethra. This is due to the fact that it
is surrounded by the external sphincter.

Please rate this question:

Discuss and give feedback

Next question

Urethral anatomy

Female urethra
The female urethra is shorter and more acutely angulated than the male urethra. It is an extra-
peritoneal structure and embedded in the endopelvic fascia. The neck of the bladder is subjected to
transmitted intra-abdominal pressure and therefore deficiency in this area may result in stress
urinary incontinence. Between the layers of the urogenital diaphragm the female urethra is
surrounded by the external urethral sphincter, this is innervated by the pudendal nerve. It ultimately
lies anterior to the vaginal orifice.

Male urethra
In males the urethra is much longer and is divided into four parts.

Pre-prostatic Extremely short and lies between the bladder and prostate gland.It has a stellate lumen
urethra and is between 1 and 1.5cm long.Innervated by sympathetic noradrenergic fibres, as
this region is composed of striated muscles bundles they may contract and prevent
retrograde ejaculation.

Prostatic This segment is wider than the membranous urethra and contains several openings for
urethra the transmission of semen (at the midpoint of the urethral crest).

Membranous Narrowest part of the urethra and surrounded by external sphincter. It traverses the
urethra perineal membrane 2.5cm postero-inferior to the symphysis pubis.

Penile urethra Travels through the corpus spongiosum on the underside of the penis. It is the longest
urethral segment.It is dilated at its origin as the infrabulbar fossa and again in the gland
penis as the navicular fossa. The bulbo-urethral glands open into the spongiose section
of the urethra 2.5cm below the perineal membrane.

The urothelium is transitional in nature near to the bladder and becomes squamous more distally.

Next question
Question 28 of 560

Which of the following anatomical structures lies within the spiral groove of the humerus?

Median nerve

Radial nerve

Tendon of triceps

Musculocutaneous nerve

Axillary nerve

Theme from April 2014 Exam


The radial nerve lies in this groove and may be compromised by fractures involving the shaft.

Please rate this question:

Discuss and give feedback

Next question

Humerus

The humerus extends from the scapula to the elbow joint. It has a body and two ends. It is almost
completely covered with muscle but can usually be palpated throughout its length. The smooth
rounded surface of the head articulates with the shallow glenoid cavity. The head is connected to the
body of the humerus by the anatomical neck. The surgical neck is the region below the head and
tubercles and where they join the shaft and is the commonest site of fracture. The capsule of the
shoulder joint is attached to the anatomical neck superiorly but extends down to 1.5cm on the
surgical neck.

The greater tubercle is the prominence on the lateral side of the upper end of the bone. It merges
with the body below and can be felt through the deltoid inferior to the acromion. The tendons of the
supraspinatus and infraspinatus are inserted into impressions on its superior aspect. The lesser
tubercle is a distinct prominence on the front of the upper end of the bone. It can be palpated
through the deltoid just lateral to the tip of the coracoid process.

The intertubercular groove passes on the body between the greater and lesser tubercles, continuing
down from the anterior borders of the tubercles to form the edges of the groove. The tendon of
biceps within its synovial sheath passes through this groove, held within it by a transverse ligament.

The posterior surface of the body is marked by a spiral groove for the radial nerve which runs
obliquely across the upper half of the body to reach the lateral border below the deltoid tuberosity.
Within this groove lie the radial nerve and brachial vessels and both may be affected by fractures
involving the shaft of the humerus.

The lower end of the humerus is wide and flattened anteroposteriorly, and inclined anteriorly. The
middle third of the distal edge forms the trochlea. Superior to this are indentations for the coronoid
fossa anteriorly and olecranon fossa posteriorly. Lateral to the trochlea is a rounded capitulum which
articulates with the radius.

The medial epicondyle is very prominent with a smooth posterior surface which contains a sulcus for
the ulnar nerve and collateral vessels. It's distal margin gives attachment for the ulnar collateral
ligament and, in front of this, the anterior surface has an impression for the common flexor tendon.

Next question
Question 29 of 560

A 24 year old man falls and sustains a fracture through his scaphoid bone. From which of the
following areas does the scaphoid derive the majority of its blood supply?

From its proximal medial border

From its proximal lateral border

From its proximal posterior surface

From the proximal end

From the distal end

Theme from April 2012 Exam


Theme from April 2014 Exam
The blood supply to the scaphoid enters from a small non articular surface near its distal end.
Transverse fractures through the scaphoid therefore carry a risk of non union.

Please rate this question:

Discuss and give feedback

Next question

Scaphoid bone

The scaphoid has a concave articular surface for the head of the capitate and at the edge of this is a
crescentic surface for the corresponding area on the lunate.
Proximally, it has a wide convex articular surface with the radius. It has a distally sited tubercle that
can be palpated. The remaining articular surface is to the lateral side of the tubercle. It faces laterally
and is associated with the trapezium and trapezoid bones.
The narrow strip between the radial and trapezial surfaces and the tubercle gives rise to the radial
collateral carpal ligament. The tubercle receives part of the flexor retinaculum. This area is the only
part of the scaphoid that is available for the entry of blood vessels. It is commonly fractured and
avascular necrosis may result.

Scaphoid bone

Image sourced from Wikipedia

Next question
Question 30 of 560

A 21 year old man has an inguinal hernia and is undergoing a surgical repair. As the surgeons
approach the inguinal canal they expose the superficial inguinal ring. Which of the following forms
the lateral edge of this structure?

Inferior epigastric artery

Conjoint tendon

Rectus abdominis muscle

External oblique aponeurosis

Transversalis fascia

The external oblique aponeurosis forms the anterior wall of the inguinal canal and also the lateral
edge of the superficial inguinal ring. The rectus abdominis lies posteromedially and the transversalis
posterior to this.
Please rate this question:

Discuss and give feedback


Next question

Inguinal canal

Location

 Above the inguinal ligament


 The inguinal canal is 4cm long
 The superficial ring is located anterior to the pubic tubercle
 The deep ring is located approximately 1.5-2cm above the half way point between the
anterior superior iliac spine and the pubic tubercle

Boundaries of the inguinal canal


Floor  External oblique aponeurosis
 Inguinal ligament
 Lacunar ligament

Roof  Internal oblique


 Transversus abdominis

Anterior wall External oblique aponeurosis

Posterior wall  Transversalis fascia


 Conjoint tendon

Laterally  Internal ring


 Fibres of internal oblique

Medially  External ring


 Conjoint tendon

Contents
Males Spermatic cord and ilioinguinal As it passes through the canal the spermatic cord
nerve has 3 coverings:

 External spermatic fascia from external


oblique aponeurosis
 Cremasteric fascia
 Internal spermatic fascia

Females Round ligament of uterus and


ilioinguinal nerve

Related anatomy of the inguinal region


The boundaries of Hesselbachs triangle are commonly tested and illustrated below:
Image sourced from Wikipedia

The image below demonstrates the close relationship of the vessels to the lower limb with the
inguinal canal. A fact to be borne in mind when repairing hernial defects in this region.

Image sourced from Wikipedia

Next question
Question 31 of 560

Which of the following cranial venous sinuses is unpaired?

Transverse sinus

Superior sagittal sinus

Cavernous sinus

Sigmoid sinus

Inferior petrosal sinus

The superior sagittal sinus is unpaired

The superior sagittal sinus is unpaired. It begins at the crista galli, where it may communicate with
the veins of the frontal sinus and sometimes with those of the nasal cavity. It arches backwards in
the falx cerebri to terminate at the internal occipital protuberance (usually into the right transverse
sinus). The parietal emissary veins link the superior sagittal sinus with the veins on the exterior of
the cranium.

Please rate this question:

Discuss and give feedback

Next question

Cranial venous sinuses

The cranial venous sinuses are located within the dura mater. They have no valves which is
important in the potential for spreading sepsis. They eventually drain into the internal jugular vein.

They are:
Superior sagittal sinus
Inferior sagittal sinus
Straight sinus
Transverse sinus
Sigmoid sinus
Confluence of sinuses
Occipital sinus
Cavernous sinus

Topography of cranial venous sinuses

Image sourced from Wikipedia

Next question
Question 32 of 560

Which of the following laryngeal tumours will not typically metastasise to the cervical lymph nodes?

Glottic

Supraglottic

Subglottic

Transglottic

Aryepiglottic fold

The vocal cords have no lymphatic drainage and therefore this region serves as a lymphatic
watershed. The supraglottic part drains to the upper deep cervical nodes through vessels piercing
the thyrohyoid membrane. The sub glottic part drains to the pre laryngeal, pre tracheal and inferior
deep cervical nodes. The aryepiglottic and vestibular folds have a rich lymphatic drainage and will
metastasise early.
Please rate this question:

Discuss and give feedback


Next question

Larynx

The larynx lies in the anterior part of the neck at the levels of C3 to C6 vertebral bodies. The
laryngeal skeleton consists of a number of cartilagenous segments. Three of these are paired;
arytenoid, corniculate and cuneiform. Three are single; thyroid, cricoid and epiglottic. The cricoid
cartilage forms a complete ring (the only one to do so).
The laryngeal cavity extends from the laryngeal inlet to the level of the inferior border of the cricoid
cartilage.

Divisions of the laryngeal cavity


Laryngeal vestibule Superior to the vestibular folds

Laryngeal ventricle Lies between vestibular folds and superior to the vocal cords
Infraglottic cavity Extends from vocal cords to inferior border of the cricoid cartilage

The vocal folds (true vocal cords) control sound production. The apex of each fold projects medially
into the laryngeal cavity. Each vocal fold includes:

 Vocal ligament
 Vocalis muscle (most medial part of thyroarytenoid muscle)

The glottis is composed of the vocal folds, processes and rima glottidis. The rima glottidis is the
narrowest potential site within the larynx, as the vocal cords may be completely opposed, forming a
complete barrier.

Muscles of the larynx


Muscle Origin Insertion Innervation Action

Posterior Posterior aspect Muscular process Recurrent Abducts vocal fold


cricoarytenoid of lamina of of arytenoid Laryngeal
cricoid

Lateral Arch of cricoid Muscular process Recurrent Adducts vocal fold


cricoarytenoid of arytenoid laryngeal

Thyroarytenoid Posterior aspect Muscular process Recurrent Relaxes vocal fold


of thyroid of arytenoid laryngeal
cartilage

Transverse and Arytenoid Contralateral Recurrent Closure of


oblique cartilage arytenoid laryngeal intercartilagenous
arytenoids part of the rima
glottidis

Vocalis Depression Vocal ligament Recurrent Relaxes posterior


between lamina and vocal process laryngeal vocal ligament, tenses
of thyroid of arytenoid anterior part
cartilage cartilage

Cricothyroid Anterolateral Inferior margin External Tenses vocal fold


and horn of
Muscle Origin Insertion Innervation Action

part of cricoid thyroid cartilage laryngeal

Blood supply
Arterial supply is via the laryngeal arteries, branches of the superior and inferior thyroid arteries. The
superior laryngeal artery is closely related to the internal laryngeal nerve. The inferior laryngeal
artery is related to the inferior laryngeal nerve. Venous drainage is via superior and inferior laryngeal
veins, the former draining into the superior thyroid vein and the latter draining into the middle thyroid
vein, or thyroid venous plexus.

Lymphatic drainage
The vocal cords have no lymphatic drainage and this site acts as a lymphatic watershed.
Supraglottic part Upper deep cervical nodes

Subglottic part Prelaryngeal and pretracheal nodes and inferior deep cervical nodes

The aryepiglottic fold and vestibular folds have a dense plexus of lymphatics associated with them
and malignancies at these sites have a greater propensity for nodal metastasis.

Topography of the larynx

Image sourced from Wikipedia


Question 33 of 560

Which of the following forms the medial wall of the femoral canal?

Pectineal ligament

Adductor longus

Sartorius

Lacunar ligament

Inguinal ligament

The femoral canal and the femoral triangle are distinct anatomical structures. Do not confuse them,
especially in the time pressured exam situation.

Theme from September 2011 exam


Theme from September 2012 exam
Theme from April 2014 exam
Please rate this question:

Discuss and give feedback


Next question

Femoral canal

The femoral canal lies at the medial aspect of the femoral sheath. The femoral sheath is a fascial
tunnel containing both the femoral artery laterally and femoral vein medially. The canal lies medial to
the vein.

Borders of the femoral canal


Laterally Femoral vein

Medially Lacunar ligament


Anteriorly Inguinal ligament

Posteriorly Pectineal ligament

Image showing dissection of femoral canal

Image sourced from Wikipedia

Contents

 Lymphatic vessels
 Cloquet's lymph node

Physiological significance
Allows the femoral vein to expand to allow for increased venous return to the lower limbs.
Pathological significance
As a potential space, it is the site of femoral hernias. The relatively tight neck places these at high
risk of strangulation.
Next question
Question 34 of 560

A 67 year old man is undergoing a transurethral resection of a bladder tumour using diathermy.
Suddenly during the procedure the patient's thigh begins to twitch. Stimulation of which of the
following nerves is the most likely cause?

Femoral

Pudendal

Sciatic

Obturator

Gluteal

Theme from January 2011 Exam


Theme from January 2013 Exam
The obturator nerve is most closely related to the bladder (see below)
Image sourced from Wikipedia

Please rate this question:

Discuss and give feedback


Next question

Obturator nerve

The obturator nerve arises from L2, L3 and L4 by branches from the ventral divisions of each of
these nerve roots. L3 forms the main contribution and the second lumbar branch is occasionally
absent. These branches unite in the substance of psoas major, descending vertically in its posterior
part to emerge from its medial border at the lateral margin of the sacrum. It then crosses the
sacroiliac joint to enter the lesser pelvis, it descends on obturator internus to enter the obturator
groove. In the lesser pelvis the nerve lies lateral to the internal iliac vessels and ureter, and is joined
by the obturator vessels lateral to the ovary or ductus deferens.

Supplies

 Medial compartment of thigh


 Muscles supplied: external obturator, adductor longus, adductor brevis, adductor magnus
(not the lower part-sciatic nerve), gracilis
 The cutaneous branch is often absent. When present, it passes between gracilis and
adductor longus near the middle part of the thigh, and supplies the skin and fascia of the
distal two thirds of the medial aspect.
Obturator canal

 Connects the pelvis and thigh: contains the obturator artery, vein, nerve which divides into
anterior and posterior branches.

Cadaveric cross section demonstrating relationships of the obturator nerve

Image sourced from Wikipedia

Next question
Question 35 of 560

A 5 year old boy is playing with some small ball bearings. Unfortunately he inhales one. To which of
the following lung regions is the ball most likely to settle?

Right lower lobe

Left main bronchus

Right upper lobe

Left lower lobe

None of the above

Theme from September 2011 Exam


Theme from January 2013 Exam
As the most dependent part of the right lung a small object is most likely to lodge here. Most objects
will preferentially enter the right lung owing to the angle the right main bronchus takes from the
trachea.
Please rate this question:

Discuss and give feedback


Next question

Lung anatomy

The right lung is composed of 3 lobes divided by the oblique and transverse fissures. The left lung
has two lobes divided by the oblique fissure.The apex of both lungs is approximately 4cm superior to
the sterno-costal joint of the first rib. Immediately below this is a sulcus created by the subclavian
artery.

Peripheral contact points of the lung

 Base: diaphragm
 Costal surface: corresponds to the cavity of the chest
 Mediastinal surface: Contacts the mediastinal pleura. Has the cardiac impression. Above and
behind this concavity is a triangular depression named the hilum, where the structures which
form the root of the lung enter and leave the viscus. These structures are invested by pleura,
which, below the hilum and behind the pericardial impression, forms the pulmonary ligament
Right lung
Above the hilum is the azygos vein; Superior to this is the groove for the superior vena cava and
right innominate vein; behind this, and nearer the apex, is a furrow for the innominate artery. Behind
the hilum and the attachment of the pulmonary ligament is a vertical groove for the oesophagus; In
front and to the right of the lower part of the oesophageal groove is a deep concavity for the
extrapericardiac portion of the inferior vena cava.

The root of the right lung lies behind the superior vena cava and the right atrium, and below the
azygos vein.

The right main bronchus is shorter, wider and more vertical than the left main bronchus and
therefore the route taken by most foreign bodies.

Image sourced from Wikipedia

Left lung
Above the hilum is the furrow produced by the aortic arch, and then superiorly the groove
accommodating the left subclavian artery; Behind the hilum and pulmonary ligament is a vertical
groove produced by the descending aorta, and in front of this, near the base of the lung, is the lower
part of the oesophagus.

The root of the left lung passes under the aortic arch and in front of the descending aorta.
Image sourced from Wikipedia

Inferior borders of both lungs

 6th rib in mid clavicular line


 8th rib in mid axillary line
 10th rib posteriorly

The pleura runs two ribs lower than the corresponding lung level.

Bronchopulmonary segments
Segment number Right lung Left lung

1 Apical Apical

2 Posterior Posterior

3 Anterior Anterior

4 Lateral Superior lingular

5 Medial Inferior lingular

6 Superior (apical) Superior (apical)


Segment number Right lung Left lung

7 Medial basal Medial basal

8 Anterior basal Anterior basal

9 Lateral basal Lateral basal

10 Posterior basal Posterior basal

Next question
Question 36 of 560

A patient presents with superior vena caval obstruction. How many collateral circulations exist as
alternative pathways of venous return?

None

One

Two

Three

Four

There are 4 collateral venous systems:

 Azygos venous system


 Internal mammary venous pathway
 Long thoracic venous system with connections to the femoral and vertebral veins (2
pathways)

Despite this, venous hypertension still occurs.


Please rate this question:

Discuss and give feedback


Next question

Superior vena cava

Drainage

 Head and neck


 Upper limbs
 Thorax
 Part of abdominal walls
Formation

 Subclavian and internal jugular veins unite to form the right and left brachiocephalic veins
 These unite to form the SVC
 Azygos vein joins the SVC before it enters the right atrium

Relations
Anterior Anterior margins of the right lung and pleura

Posteromedial Trachea and right vagus nerve

Posterolateral Posterior aspects of right lung and pleura


Pulmonary hilum is posterior

Right lateral Right phrenic nerve and pleura

Left lateral Brachiocephalic artery and ascending aorta

Developmental variations
Anomalies of the connection of the SVC are recognised. In some individuals a persistent left sided
SVC drains into the right atrium via an enlarged orifice of the coronary sinus. More rarely the left
sided vena cava may connect directly with the superior aspect of the left atrium, usually associated
with an un-roofing of the coronary sinus. The commonest lesion of the IVC is for its abdominal
course to be interrupted, with drainage achieved via the azygos venous system. This may occur in
patients with left sided atrial isomerism.
Next question
Question 37 of 560

An 18 year old man is cutting some plants when a small piece of vegetable matter enters his eye.
His eye becomes watery. Which of the following is responsible for relaying parasympathetic
neuronal signals to the lacrimal apparatus?

Pterygopalatine ganglion

Otic ganglion

Submandibular ganglion

Ciliary ganglion

None of the above

Theme from January 2013 Exam


The parasympathetic fibres to the lacrimal apparatus transit via the pterygopalatine ganglion.

Please rate this question:

Discuss and give feedback

Next question

Lacrimal system

Lacrimal gland
Consists of an orbital part and palpebral part. They are continuous posterolaterally around the
concave lateral edge of the levator palpebrae superioris muscle.
The ducts of the lacrimal gland open into the superior fornix. Those from the orbital part penetrate
the aponeurosis of levator palpebrae superioris to join those from the palpebral part. Therefore
excision of the palpebral part is functionally similar to excision of the entire gland.
Blood supply
Lacrimal branch of the opthalmic artery. Venous drainage is to the superior opthalmic vein.

Innervation
The gland is innervated by the secretomotor parasympathetic fibres from the pterygopalatine
ganglion which in turn may reach the gland via the zygomatic or lacrimal branches of the maxillary
nerve or pass directly to the gland. The preganglionic fibres travel to the ganglion in the greater
petrosal nerve (a branch of the facial nerve at the geniculate ganglion).

Nasolacrimal duct
Descends from the lacrimal sac to open anteriorly in the inferior meatus of the nose.

Lacrimation reflex
Occurs in response to conjunctival irritation (or emotional events). The conjunctiva will send signals
via the opthalmic nerve. These then pass to the superior salivary centre. The efferent signals pass
via the greater petrosal nerve (parasympathetic preganglionic fibres) and the deep petrosal nerve
which carries the post ganglionic sympathetic fibres. The parasympathetic fibres will relay in the
pterygopalatine ganglion, the sympathetic fibres do not synapse. They in turn will relay to the
lacrimal apparatus.

Next question
Question 38 of 560

Which of the nerves listed below is directly responsible for the innervation of the lateral aspect of
flexor digitorum profundus?

Ulnar nerve

Anterior interosseous nerve

Radial nerve

Median nerve

Posterior interosseous nerve

The anterior interosseous nerve is a branch of the median nerve and is responsible for innervation of
the lateral aspect of the flexor digitorum profundus.

Please rate this question:

Discuss and give feedback

Next question

Forearm flexor muscles

Muscle Origin Insertion Nerve supply Action

Flexor carpi Common flexor Front of bases of second Median Flexes and abducts the
radialis origin and and third metacarpals carpus, part flexes the
surrounding elbow and part
Muscle Origin Insertion Nerve supply Action

fascia pronates forearm

Palmaris Common flexor Apex of palmar Median Wrist flexor


longus origin aponeurosis

Flexor carpi Small humeral Pisiform and base of the Ulnar nerve Flexes and adducts the
ulnaris head arises from fifth metacarpal carpus
the common
flexor origin and
adjacent fascia.
Ulnar head
comes from
medial border of
olecranon and
posterior border
of ulna

Flexor Long linear origin Via tendons in the fibrous Median Flexor of
digitorum from common flexor sheath. At the level metacarpophalangeal
superficialis flexor tendon, of the joint and proximal
adjacent fascia metacarpophalangeal interphalangeal joint
and septa and joint each tendon split
medial border of into two, these bands
the coronoid pass distally to their
process insertions

Flexor Upper two thirds Via tendons that lie deep Medial part= Flexes the distal
digitorum of the medial and to those of flexor ulnar, lateral interphalangeal joints
profundus anterior surface digitorum superficialis to part=anterior and the wrist
of the ulna, insert into the distal interosseous
medial side of the phalanx nerve
olecranon,
medial half of the
interosseous
Muscle Origin Insertion Nerve supply Action

membrane

Next question
Question 39 of 560

A 45 year old lady is undergoing a Whipples procedure for carcinoma of the pancreatic head. The
bile duct is transected. Which of the following vessels is mainly responsible for the blood supply to
the bile duct?

Cystic artery

Hepatic artery

Portal vein

Left gastric artery

None of the above

Do not confuse the blood supply of the bile duct with that of the cystic duct.

Theme from April 2014 exam


The bile duct has an axial blood supply which is derived from the hepatic artery and from
retroduodenal branches of the gastroduodenal artery. Unlike the liver there is no contribution by the
portal vein to the blood supply of the bile duct. Damage to the hepatic artery during a difficult
cholecystectomy is a recognised cause of bile duct strictures.
Please rate this question:

Discuss and give feedback


Next question

Gallbladder

 Fibromuscular sac with capacity of 50ml


 Columnar epithelium

Relations of the gallbladder


Anterior Liver
Posterior  Covered by peritoneum
 Transverse colon
 1st part of the duodenum

Laterally Right lobe of liver

Medially Quadrate lobe of liver

Arterial supply
Cystic artery (branch of Right hepatic artery)

Venous drainage
Directly to the liver

Nerve supply
Sympathetic- mid thoracic spinal cord, Parasympathetic- anterior vagal trunk

Common bile duct

Origin Confluence of cystic and common hepatic ducts

Relations at  Medially - Hepatic artery


origin  Posteriorly- Portal vein

Relations distally  Duodenum - anteriorly


 Pancreas - medially and laterally
 Right renal vein - posteriorly

Arterial supply Branches of hepatic artery and retroduodenal branches of gastroduodenal


artery

Hepatobiliary triangle

Medially Common hepatic duct

Inferiorly Cystic duct

Superiorly Inferior edge of liver


Contents Cystic artery

Relations of the gallbladder

© Image provided by the University of Sheffield

Next question
Question 40 of 560

A 43 year old lady is undergoing a total thyroidectomy for an extremely large goitre. The surgeons
decide that access may be improved by division of the infra hyoid strap muscles. At which of the
following sites should they be divided?

In their upper half

In their lower half

In the middle

At their origin from the hyoid

At the point of their insertion

Theme from 2009 Exam


Should the strap muscles require division during surgery they should be divided in their upper half.
This is because their nerve supply from the ansa cervicalis enters in their lower half.
Please rate this question:

Discuss and give feedback


Next question

Anterior triangle of the neck

Boundaries

Anterior border of the Sternocleidomastoid


Lower border of mandible
Anterior midline

Sub triangles (divided by Digastric above and Omohyoid)

 Muscular triangle: Neck strap muscles


 Carotid triangle: Carotid sheath
 Submandibular Triangle (digastric)

Contents of the anterior triangle


Digastric triangle Submandibular gland
Submandibular nodes
Facial vessels
Hypoglossal nerve

Muscular triangle Strap muscles


External jugular vein

Carotid triangle Carotid sheath (Common carotid, vagus and internal jugular vein)
Ansa cervicalis

Nerve supply to digastric muscle

 Anterior: Mylohyoid nerve


 Posterior: Facial nerve

Image sourced from Wikipedia

Next question
Question 41 of 560

A 7 year old boy presents with right iliac fossa pain and there is a clinical suspicion that appendicitis
is present. From which of the following embryological structures is the appendix derived?

Vitello-intestinal duct

Uranchus

Foregut

Hindgut

Midgut

The appendix is derived from the midgut

Theme from April 2014 exam


It is derived from the midgut which is why early appendicitis may present with periumbilical pain.
Please rate this question:

Discuss and give feedback


Next question

Appendix

 Location: Base of caecum.


 Up to 10cm long.
 Mainly lymphoid tissue (Hence mesenteric adenitis may mimic appendicitis).
 Caecal taenia coli converge at base of appendix and form a longitudinal muscle cover over
the appendix. This convergence should facilitate its identification at surgery if it is retrocaecal
and difficult to find (which it can be when people start doing appendicectomies!)
 Arterial supply: Appendicular artery (branch of the ileocolic).
 It is intra peritoneal.

McBurney's point

 1/3 of the way along a line drawn from the Anterior Superior Iliac Spine to the Umbilicus
6 Positions:

 Retrocaecal 74%
 Pelvic 21%
 Postileal
 Subcaecal
 Paracaecal
 Preileal

Next question
Question 42 of 560

A 22 year old women has recently undergone a surgical excision of the submandibular gland. She
presents to the follow up clinic with a complaint of tongue weakness on the ipsilateral side to her
surgery. Which nerve has been damaged?

Hypoglossal nerve

Lingual nerve

Inferior alveolar nerve

Facial nerve

Lesser petrosal nerve

Three cranial nerves may be injured during submandibular gland excision.

 Marginal mandibular branch of the facial nerve


 Lingual nerve
 Hypoglossal nerve

Theme from April 2012 Exam


Hypoglossal nerve damage may result in paralysis of the ipsilateral aspect of the tongue. The nerve
itself lies deep to the capsule surrounding the gland and should not be injured during an
intracapsular dissection. The lingual nerve is probably at greater risk of injury. However, the effects
of lingual nerve injury are sensory rather than motor.
Image sourced from Wikipedia

Please rate this question:

Discuss and give feedback


Next question

Submandibular gland

Relations of the submandibular gland


Superficial Platysma, deep fascia and mandible
Submandibular lymph nodes
Facial vein (facial artery near mandible)
Marginal mandibular nerve
Cervical branch of the facial nerve

Deep Facial artery (inferior to the mandible)


Mylohoid muscle
Sub mandibular duct
Hyoglossus muscle
Lingual nerve
Submandibular ganglion
Hypoglossal nerve

Submandibular duct (Wharton's duct)


 Opens lateral to the lingual frenulum on the anterior floor of mouth.
 5 cm length
 Lingual nerve wraps around Wharton's duct. As the duct passes forwards it crosses medial
to the nerve to lie above it and then crosses back, lateral to it, to reach a position below the
nerve.

Innervation

 Sympathetic innervation- Derived from superior cervical ganglion


 Parasympathetic innervation- Submandibular ganglion via lingual nerve

Arterial supply
Branch of the facial artery. The facial artery passes through the gland to groove its deep surface. It
then emerges onto the face by passing between the gland and the mandible.

Venous drainage
Anterior facial vein (lies deep to the Marginal Mandibular nerve)

Lymphatic drainage
Deep cervical and jugular chains of nodes
Next question
Question 43 of 560

You decide to take an arterial blood gas from the femoral artery. Where should the needle be
inserted to gain the sample?

Mid point of the inguinal ligament

Mid inguinal point

2cm inferomedially to the pubic tubercle

2cm superomedially to the pubic tubercle

3cm inferolaterally to the deep inguinal ring

The mid inguinal point is midway between the anterior superior iliac spine and the symphysis pubis

Theme from April 2015 Exam


The mid inguinal point in the surface marking for the femoral artery.
Please rate this question:

Discuss and give feedback


Next question

Femoral triangle anatomy

Boundaries
Superiorly Inguinal ligament

Laterally Sartorius

Medially Adductor longus

Floor Iliopsoas, adductor longus and pectineus


Roof  Fascia lata and Superficial fascia
 Superficial inguinal lymph nodes (palpable below the inguinal ligament)
 Long saphenous vein

Image sourced from Wikipedia

Contents

 Femoral vein (medial to lateral)


 Femoral artery-pulse palpated at the mid inguinal point
 Femoral nerve
 Deep and superficial inguinal lymph nodes
 Lateral cutaneous nerve
 Great saphenous vein
 Femoral branch of the genitofemoral nerve

Next question
Question 44 of 560

A 67 year old man undergoes a carotid endarterectomy and seems to recover well following surgery.
When he is reviewed on the ward post operatively he complains that his voice is hoarse. What is the
most likely cause?

Damage to the accessory nerve

Damage to the cervical plexus

Damage to the glossopharyngeal nerve

Damage to the hypoglossal nerve

Damage to the vagus

Theme from April 2013 Exam


Many of these nerves are at risk of injury during carotid surgery. However, only damage to the vagus
would account for difficulties in speech.

Please rate this question:

Discuss and give feedback

Next question

Vagus nerve

The vagus nerve has mixed functions and supplies the structures from the fourth and sixth
pharyngeal arches. It also supplies the fore and midgut sections of the embryonic gut tube. It carries
afferent fibres from these areas (viz; pharynx, larynx, oesophagus, stomach, lungs, heart and great
vessels). The efferent fibres of the vagus are of two main types. The first are preganglionic
parasympathetic fibres distributed to the parasympathetic ganglia that innervate smooth muscle of
the innervated organs (such as gut). The second type of efferent fibres have direct skeletal muscle
innervation, these are largely to the muscles of the larynx and pharynx.

Origin and course


The vagus arises from the lateral surface of the medulla oblongata by a series of rootlets. It is
related to the glossopharyngeal nerve cranially and the accessory nerve caudally. It exits through
the jugular foramen and is contained within its own dural sheath alongside the accessory nerve. In
the neck it descends vertically in the carotid sheath where it is closely related to the internal and
common carotid arteries. It leaves the neck and enters the mediastinum. On the right it passes
anterior to the first part of the subclavian artery, on the left it lies in the interval between the common
carotid and subclavian arteries.
In the mediastinum both nerves pass postero-inferiorly and reach the posterior surface of the
corresponding lung root. These then branch into both lungs. At the inferior end of the mediastinum
these plexuses reunite to form the formal vagal trunks that pass through the oesophageal hiatus and
into the abdomen. The anterior and posterior vagal trunks are formal nerve fibres these then splay
out once again sending fibres over the stomach and posteriorly to the coeliac plexus. Branches pass
to the liver, spleen and kidney.

Communications and branches

Communication Details

Superior Located in jugular foramen


ganglion Communicates with the superior cervical sympathetic ganglion, accessory nerve
Two branches; meningeal and auricular (the latter may give rise to vagal stimulation
following instrumentation of the external auditory meatus)

Inferior ganglion Communicates with the superior cervical sympathetic ganglion, hypoglossal nerve and
loop between first and second cervical ventral rami
Two branches; pharyngeal (supplies pharyngeal muscles) and superior laryngeal nerve
(inferomedially- deep to both carotid arteries)

Branches in the neck

Branch Detail

Superior and inferior Arise at various points and descend into thorax
cervical cardiac On the right these pass posterior to the subclavian artery
branches On the left the superior branch passes between the arch of the aorta and the
trachea to connect with the deep cardiac plexus. The inferior branch descends
Branch Detail

with the vagus itself.

Right recurrent Arises from vagus anterior to the first part of the subclavian artery, hooks under
laryngeal nerve it, and ascends superomedially. It passes close to the common carotid and finally
the inferior thyroid artery to insert into the larynx

Branches in the thorax

Branch Details

Left recurrent Arises from the vagus on the aortic arch. It hooks around the inferior surface of the arch,
laryngeal posterior to the ligamentum arteriosum and passes upwards through the superior
nerve mediastinum and lower part of the neck. It lies in the groove between oesophagus and
trachea (supplies both). It passes with the inferior thyroid artery and inserts into the
larynx.

Thoracic and There are extensive branches to both the heart and lung roots. These pass throughout
cardiac both these viscera. The fibres reunite distally prior to passing into the abdomen.
branches

Abdominal branches
After entry into the abdominal cavity the nerves branch extensively. In previous years the extensive
network of the distal branches (nerves of Laterjet) over the surface of the distal stomach were
important for the operation of highly selective vagotomy. The use of modern PPI's has reduced the
need for such highly selective procedures. Branches pass to the coeliac axis and alongside the
vessels to supply the spleen, liver and kidney.

Next question
Question 45 of 560

A 25 year old man has an inguinal hernia, which of the following structures must be divided (at open
surgery) to gain access to the inguinal canal?

Transversalis fascia

External oblique aponeurosis

Conjoint tendon

Rectus abdominis

Inferior epigastric artery

Theme from January 2013 Exam


This question is asking what structure forms the anterior wall of the inguinal canal. The anterior wall
is formed by the external oblique aponeurosis. Once this is divided the canal is entered, the cord can
be mobilised and a hernia repair performed. The transversalis fascia and conjoint tendons form the
posterior wall and would not routinely be divided to gain access to the inguinal canal itself.
Please rate this question:

Discuss and give feedback


Next question

External oblique muscle

External oblique forms the outermost muscle of the three muscles comprising the anterolateral
aspect of the abdominal wall. Its aponeurosis comprises the anterior wall of the inguinal canal.

Origin Outer surfaces of the lowest eight ribs

Insertion  Anterior two thirds of the outer lip of the iliac crest.
 The remainder becomes the aponeurosis that fuses with the linea alba in the
midline.

Nerve Ventral rami of the lower six thoracic nerves


supply

Actions Contains the abdominal viscera, may contract to raise intra abdominal pressure.
Moves trunk to one side.

Next question
Question 46 of 560

Which muscle initiates abduction of the shoulder?

Infraspinatus

Latissimus dorsi

Supraspinatus

Deltoid

Teres major

Theme from September 2012 exam


Theme from April 2014 Exam
The intermediate portion of the deltoid muscle is the chief abductor of the humerus. However, it can
only do this after the movement has been initiated by supraspinatus. Damage to the tendon of
supraspinatus is a common form of rotator cuff disease.
Please rate this question:

Discuss and give feedback


Next question

Shoulder joint

 Shallow synovial ball and socket type of joint.


 It is an inherently unstable joint, but is capable to a wide range of movement.
 Stability is provided by muscles of the rotator cuff that pass from the scapula to insert in the
greater tuberosity (all except sub scapularis-lesser tuberosity).

Glenoid labrum

 Fibrocartilaginous rim attached to the free edge of the glenoid cavity


 Tendon of the long head of biceps arises from within the joint from the supraglenoid tubercle,
and is fused at this point to the labrum.
 The long head of triceps attaches to the infraglenoid tubercle
Fibrous capsule

 Attaches to the scapula external to the glenoid labrum and to the labrum itself (postero-
superiorly)
 Attaches to the humerus at the level of the anatomical neck superiorly and the surgical neck
inferiorly
 Anteriorly the capsule is in contact with the tendon of subscapularis, superiorly with the
supraspinatus tendon, and posteriorly with the tendons of infraspinatus and teres minor. All
these blend with the capsule towards their insertion.
 Two defects in the fibrous capsule; superiorly for the tendon of biceps. Anteriorly there is a
defect beneath the subscapularis tendon.
 The inferior extension of the capsule is closely related to the axillary nerve at the surgical
neck and this nerve is at risk in anteroinferior dislocations. It also means that proximally sited
osteomyelitis may progress to septic arthritis.

Movements and muscles


Flexion Anterior part of deltoid
Pectoralis major
Biceps
Coracobrachialis

Extension Posterior deltoid


Teres major
Latissimus dorsi

Adduction Pectoralis major


Latissimus dorsi
Teres major
Coracobrachialis

Abduction Mid deltoid


Supraspinatus

Medial rotation Subscapularis


Anterior deltoid
Teres major
Latissimus dorsi
Lateral rotation Posterior deltoid
Infraspinatus
Teres minor

Important anatomical relations


Anteriorly Brachial plexus
Axillary artery and vein

Posterior Suprascapular nerve


Suprascapular vessels

Inferior Axillary nerve


Circumflex humeral vessels

Next question
Question 47 of 560

A 34 year old man is shot in the postero- inferior aspect of his thigh. Which of the following lies at the
most lateral aspect of the popliteal fossa?

Popliteal artery

Popliteal vein

Common peroneal nerve

Tibial nerve

Small saphenous vein

Theme from April 2012 exam


Theme from April 2014 exam
The contents of the popliteal fossa are (from medial to lateral):
Popliteal artery
Popliteal vein
Tibial nerve
Common peroneal nerve

The sural nerve is a branch of the tibial nerve and usually arises at the inferior aspect of the popliteal
fossa. However, its anatomy is variable.
Please rate this question:

Discuss and give feedback


Next question

Popliteal fossa

Boundaries of the popliteal fossa


Laterally Biceps femoris above, lateral head of gastrocnemius and plantaris below

Medially Semimembranosus and semitendinosus above, medial head of gastrocnemius below


Floor Popliteal surface of the femur, posterior ligament of knee joint and popliteus muscle

Roof Superficial and deep fascia

Image showing the popliteal fossa

© Image provided by the University of Sheffield

Contents

 Popliteal artery and vein


 Small saphenous vein
 Common peroneal nerve
 Tibial nerve
 Posterior cutaneous nerve of the thigh
 Genicular branch of the obturator nerve
 Lymph nodes

Next question
Question 48 of 560

A 67 year old man has an abdominal aortic aneurysm which displaces the left renal vein. Which
branch of the aorta is most likely to affected at this level?

Inferior mesenteric artery

Superior mesenteric artery

Coeliac axis

Testicular artery

None of the above

Theme from April 2013 exam


Theme from April 2014 exam
The left renal vein lies behind of the SMA as it branches off the aorta. Whilst juxtarenal AAA may
sometimes require the division of the left renal vein, direct involvement of the SMA may require a
hybrid surgical bypass and subsequent endovascular occlusion.

Please rate this question:

Discuss and give feedback

Next question

Abdominal aortic branches

Branches Level Paired Type


Branches Level Paired Type

Inferior phrenic T12 (Upper border) Yes Parietal

Coeliac T12 No Visceral

Superior mesenteric L1 No Visceral

Middle suprarenal L1 Yes Visceral

Renal L1-L2 Yes Visceral

Gonadal L2 Yes Visceral

Lumbar L1-L4 Yes Parietal

Inferior mesenteric L3 No Visceral

Median sacral L4 No Parietal

Common iliac L4 Yes Terminal

Next question
Question 49 of 560

A 12 year old boy undergoes surgery for recurrent mastoid infections. Post operatively he complains
of an altered taste sensation. Which of the following nerves has been injured?

Glossopharyngeal

Greater petrosal

Olfactory

Trigeminal

Chorda tympani

Theme from April 2012 exam


Theme from April 2014 exam
The chorda tympani branch of the facial nerve passes forwards through itrs canaliculus into the
middle ear, and crosses the medial aspect of the tympanic membrane. It then passes antero-
inferiorly in the infratemporal fossa. It distributes taste fibres to the anterior two thirds of the tongue.
Image sourced from Wikipedia

Please rate this question:

Discuss and give feedback


Next question

Facial nerve

The facial nerve is the main nerve supplying the structures of the second embryonic branchial arch.
It is predominantly an efferent nerve to the muscles of facial expression, digastric muscle and also to
many glandular structures. It contains a few afferent fibres which originate in the cells of its genicular
ganglion and are concerned with taste.

Supply - 'face, ear, taste, tear'

 Face: muscles of facial expression


 Ear: nerve to stapedius
 Taste: supplies anterior two-thirds of tongue
 Tear: parasympathetic fibres to lacrimal glands, also salivary glands

Path
Subarachnoid path

 Origin: motor- pons, sensory- nervus intermedius


 Pass through the petrous temporal bone into the internal auditory meatus with the
vestibulocochlear nerve. Here they combine to become the facial nerve.

Facial canal path

 The canal passes superior to the vestibule of the inner ear


 At the medial aspect of the middle ear, it becomes wider and contains the geniculate
ganglion.

- 3 branches:
1. greater petrosal nerve
2. nerve to stapedius
3. chorda tympani

Stylomastoid foramen

 Passes through the stylomastoid foramen (tympanic cavity anterior and mastoid antrum
posteriorly)
 Posterior auricular nerve and branch to posterior belly of digastric and stylohyoid muscle
Face
Enters parotid gland and divides into 5 branches:

 Temporal branch
 Zygomatic branch
 Buccal branch
 Marginal mandibular branch
 Cervical branch

Next question
Question 50 of 560

The first root of the brachial plexus commonly arises at which of the following levels?

C6

C5

C3

C2

C8

It begins at C5 and has 5 roots. It ends with a total of 15 nerves of these 5 are the main nerves to
the upper limb (axillary, radial, ulnar, musculocutaneous and median)
Please rate this question:

Discuss and give feedback


Next question

Brachial plexus

Origin Anterior rami of C5 to T1

Sections of the  Roots, trunks, divisions, cords, branches


plexus  Mnemonic:Real Teenagers Drink Cold Beer

Roots  Located in the posterior triangle


 Pass between scalenus anterior and medius

Trunks  Located posterior to middle third of clavicle


 Upper and middle trunks related superiorly to the subclavian artery
 Lower trunk passes over 1st rib posterior to the subclavian artery
Divisions Apex of axilla

Cords Related to axillary artery

Diagram illustrating the branches of the brachial plexus

Image sourced from Wikipedia

Cutaneous sensation of the upper limb


Image sourced from Wikipedia

Next question
Question 51 of 560

What is the anatomical level of the transpyloric plane?

T11

T12

L1

L4

T10

Theme from January 2015 Exam


Please rate this question:

Discuss and give feedback


Next question

Transpyloric plane

Transpyloric plane
Level of the body of L1

 Pylorus stomach
 Left kidney hilum (L1- left one!)
 Right hilum of the kidney (1.5cm lower than the left)
 Fundus of the gallbladder
 Neck of pancreas
 Duodenojejunal flexure
 Superior mesenteric artery
 Portal vein
 Left and right colic flexure
 Root of the transverse mesocolon
 2nd part of the duodenum
 Upper part of conus medullaris
 Spleen
 Question 52 of 560

 A 35 year old man falls and sustains a fracture to the medial third of his clavicle. Which
vessel is at greatest risk of injury?

Subclavian vein

Subclavian artery

External carotid artery

Internal carotid artery

Vertebral artery


Theme from April 2013 exam
Theme from April 2014 exam
The subclavian vein lies behind subclavius and the medial part of the clavicle. It rests on the
first rib, below and in front of the third part of the subclavian artery, and then on scalenus
anterior which separates it from the second part of the artery (posteriorly).
 Please rate this question:


 Discuss and give feedback

 Next question

 Clavicle

 The clavicle extends from the sternum to the acromion and helps prevent the shoulder falling
forwards and downwards.
The inferior surface is irregular and strongly marked by ligaments at each end. Laterally, lies
the trapezoid line and this runs anterolaterally. Posteriorly, lies the conoid tubercle. These
give attachment to the conoid and trapezoid parts of the coracoclavicular ligament. The
medial part of the inferior surface has an irregular surface which marks the surface
attachment of the costoclavicular ligament. The intermediate portion is marked by a groove
for the subclavius muscle.
Medially, the superior part of the bone has a raised surface which gives attachment to the
clavicular head of sternocleidomastoid. Sternohyoid gains attachment to the posterior
surface.
Laterally there is an oval articular facet for the acromion and a disk lies between the clavicle
and acromion. The capsule of the joint is attached to the ridge on the margin of the facet.
 Next question

Question 53 of 560

Where are the greatest proportion of musculi pectinati found?

Right ventricle

Left ventricle

Right atrium

Pulmonary valve

Aortic valve

The musculi pectinati are found in the atria, hence the reason that the atrial walls in the right atrium
are irregular anteriorly.
The musculi pectinati of the atria are internal muscular ridges on the anterolateral surface of the
chambers and they are only present in the area derived from the embryological true atrium.
Please rate this question:

Discuss and give feedback


Next question

Heart anatomy

The walls of each cardiac chamber comprise:

 Epicardium
 Myocardium
 Endocardium

Cardiac muscle is attached to the cardiac fibrous skeleton.

Relations
The heart and roots of the great vessels within the pericardial sac are related anteriorly to the
sternum, medial ends of the 3rd to 5th ribs on the left and their associated costal cartilages. The
heart and pericardial sac are situated obliquely two thirds to the left and one third to the right of the
median plane.
The pulmonary valve lies at the level of the left third costal cartilage.
The mitral valve lies at the level of the fourth costal cartilage.

Coronary sinus
This lies in the posterior part of the coronary groove and receives blood from the cardiac veins. The
great cardiac vein lies at its left and the middle and small cardiac veins lie on its right. The smallest
cardiac vein (anterior cardiac vein) drains into the right atrium directly.

Aortic sinus
Right coronary artery arises from the right aortic sinus, the left is derived from the left aortic sinus,
which lies posteriorly.

Right and left ventricles

Structure Left Ventricle

A-V Valve Mitral (double leaflet)

Walls Twice as thick as right

Trabeculae carnae Much thicker and more numerous

Right coronary artery


The RCA supplies:

 Right atrium
 Diaphragmatic part of the left ventricle
 Usually the posterior third of the interventricular septum
 The sino atrial node (60% cases)
 The atrio ventricular node (80% cases)

Left coronary artery


The LCA supplies:

 Left atrium
 Most of left ventricle
 Part of the right ventricle
 Anterior two thirds of the inter ventricular septum
 The sino atrial node (remaining 40% cases)

Innervation of the heart


Autonomic nerve fibres from the superficial and deep cardiac plexus. These lie anterior to the
bifurcation of the trachea, posterior to the ascending aorta and superior to the bifurcation of the
pulmonary trunk. The parasympathetic supply to the heart is from presynaptic fibres of the vagus
nerves.

Valves of the heart


Mitral valve Aortic valve Pulmonary valve Tricuspid valve

2 cusps 3 cusps 3 cusps 3 cusps

First heart sound Second heart Second heart First heart sound
sound sound

1 anterior cusp 2 anterior cusps 2 anterior cusps 2 anterior cusps

Attached to chordae No chordae No chordae Attached to chordae


tendinae tendinae

Next question
Question 54 of 560

Which of the following structures separates the subclavian artery and vein?

Digastric muscle

Prevertebral fascia

Anterior scalene muscle

Middle scalene muscle

Omohyoid

The anterior scalene muscle is an important anatomical landmark and separates the subclavian vein
(anterior) from the subclavian artery (posterior).
Please rate this question:

Discuss and give feedback


Next question

Scalene muscles

The 3 paired muscles are:

 Scalenus anterior: Elevate 1st rib and laterally flex the neck to same side
 Scalenus medius: Same action as scalenus anterior
 Scalenus posterior: Elevate 2nd rib and tilt neck to opposite side

Innervation Spinal nerves C4-6

Origin Transverse processes C2 to C7

Insertion First and second ribs

 The brachial plexus and subclavian artery pass between the anterior and
Important
middle scalenes through a space called the scalene hiatus/fissure.
relations
 The subclavian vein and phrenic nerve pass anteriorly to the anterior
scalene as it crosses over the first rib.

Image sourced from Wikipedia

Thoracic outlet syndrome


The scalenes are at risk of adhering to the fascia surrounding the brachial plexus or shortening
causing compression of the brachial plexus when it passes between the clavicle and 1st rib causing
thoracic outlet syndrome.
Next question
Question 55 of 560

A 33 year old man is stabbed in the right chest and undergoes a thoracotomy. The right lung is
mobilised and the pleural reflection at the lung hilum is opened. Which of the structures listed below
does not lie within this region?

Pulmonary artery

Azygos vein

Pulmonary vein

Bronchus

None of the above

The pleural reflections encase the hilum of the lung and continue inferiorly as the pulmonary
ligament. It encases the pulmonary vessels and bronchus. The azygos vein is not contained within it.
Please rate this question:

Discuss and give feedback


Next question

Lung anatomy

The right lung is composed of 3 lobes divided by the oblique and transverse fissures. The left lung
has two lobes divided by the oblique fissure.The apex of both lungs is approximately 4cm superior to
the sterno-costal joint of the first rib. Immediately below this is a sulcus created by the subclavian
artery.

Peripheral contact points of the lung

 Base: diaphragm
 Costal surface: corresponds to the cavity of the chest
 Mediastinal surface: Contacts the mediastinal pleura. Has the cardiac impression. Above and
behind this concavity is a triangular depression named the hilum, where the structures which
form the root of the lung enter and leave the viscus. These structures are invested by pleura,
which, below the hilum and behind the pericardial impression, forms the pulmonary ligament
Right lung
Above the hilum is the azygos vein; Superior to this is the groove for the superior vena cava and
right innominate vein; behind this, and nearer the apex, is a furrow for the innominate artery. Behind
the hilum and the attachment of the pulmonary ligament is a vertical groove for the oesophagus; In
front and to the right of the lower part of the oesophageal groove is a deep concavity for the
extrapericardiac portion of the inferior vena cava.

The root of the right lung lies behind the superior vena cava and the right atrium, and below the
azygos vein.

The right main bronchus is shorter, wider and more vertical than the left main bronchus and
therefore the route taken by most foreign bodies.

Image sourced from Wikipedia

Left lung
Above the hilum is the furrow produced by the aortic arch, and then superiorly the groove
accommodating the left subclavian artery; Behind the hilum and pulmonary ligament is a vertical
groove produced by the descending aorta, and in front of this, near the base of the lung, is the lower
part of the oesophagus.

The root of the left lung passes under the aortic arch and in front of the descending aorta.
Image sourced from Wikipedia

Inferior borders of both lungs

 6th rib in mid clavicular line


 8th rib in mid axillary line
 10th rib posteriorly

The pleura runs two ribs lower than the corresponding lung level.

Bronchopulmonary segments
Segment number Right lung Left lung

1 Apical Apical

2 Posterior Posterior

3 Anterior Anterior

4 Lateral Superior lingular

5 Medial Inferior lingular

6 Superior (apical) Superior (apical)


Segment number Right lung Left lung

7 Medial basal Medial basal

8 Anterior basal Anterior basal

9 Lateral basal Lateral basal

10 Posterior basal Posterior basal

Next question
Question 56 of 560

A 56 year old man requires long term parenteral nutrition and the decision is made to insert a PICC
line for long term venous access. This is inserted into the basilic vein at the region of the elbow. As
the catheter is advanced, into which venous structure is the tip of the catheter most likely to pass
from the basilic vein?

Subclavian vein

Axillary vein

Posterior circumflex humeral vein

Cephalic vein

Superior vena cava

The basilic vein drains into the axillary vein and although PICC lines may end up in a variety of
fascinating locations the axillary vein is usually the commonest site following from the basilic. The
posterior circumflex humeral vein is encountered prior to the axillary vein. However, a PICC line is
unlikely to enter this structure because of its angle of entry into the basilic vein.
Please rate this question:

Discuss and give feedback


Next question

Basilic vein

The basilic and cephalic veins both provide the main pathways of venous drainage for the arm and
hand. It is continuous with the palmar venous arch distally and the axillary vein proximally.

Path

 Originates on the medial side of the dorsal venous network of the hand, and passes up the
forearm and arm.
 Most of its course is superficial.
 Near the region anterior to the cubital fossa the vein joins the cephalic vein.
 Midway up the humerus the basilic vein passes deep under the muscles.
 At the lower border of the teres major muscle, the anterior and posterior circumflex humeral
veins feed into it.
 It is often joined by the medial brachial vein before draining into the axillary vein.

Next question
Question 57 of 560

An individual is noted to have a left sided superior vena cava. By which pathway is blood from this
system most likely to enter the heart?

Via the coronary sinus

Via the azygos venous system and into the superior vena cava

Via anomalies in the pumonary vascular bed

Via the left atrium and persistent foramen ovale

Directly into the roof of the right atrium

Theme from September 2013 Exam


Persistent left superior vena cava is the most common anomaly of the thoracic venous system. It is
prevalent in 0.3% of the population and is a benign entity of failed involution during embryogenesis.
Please rate this question:

Discuss and give feedback


Next question

Superior vena cava

Drainage

 Head and neck


 Upper limbs
 Thorax
 Part of abdominal walls

Formation

 Subclavian and internal jugular veins unite to form the right and left brachiocephalic veins
 These unite to form the SVC
 Azygos vein joins the SVC before it enters the right atrium
Relations
Anterior Anterior margins of the right lung and pleura

Posteromedial Trachea and right vagus nerve

Posterolateral Posterior aspects of right lung and pleura


Pulmonary hilum is posterior

Right lateral Right phrenic nerve and pleura

Left lateral Brachiocephalic artery and ascending aorta

Developmental variations
Anomalies of the connection of the SVC are recognised. In some individuals a persistent left sided
SVC drains into the right atrium via an enlarged orifice of the coronary sinus. More rarely the left
sided vena cava may connect directly with the superior aspect of the left atrium, usually associated
with an un-roofing of the coronary sinus. The commonest lesion of the IVC is for its abdominal
course to be interrupted, with drainage achieved via the azygos venous system. This may occur in
patients with left sided atrial isomerism.
Next question
Question 58-60 of 560

Theme: Nerve injury

A. Ulnar nerve
B. Musculocutaneous nerve
C. Radial nerve
D. Median nerve
E. Axillary nerve
F. Intercostobrachial nerve

What is the most likely nerve injury for the scenario given? Each option may be used once, more
than once or not at all.

58. A 23 year old man is involved in a fight outside a nightclub and sustains a laceration to his
right arm. On examination he has lost extension of the fingers in his right hand.

You answered Ulnar nerve

The correct answer is Radial nerve

The radial nerve supplies the extensor muscle group.

59. A 40 year old lady trips and falls through a glass door and sustains a severe laceration to
her left arm. Amongst her injuries it is noticed that she has lost the ability to adduct the
fingers of her left hand.

Ulnar nerve

The interossei are supplied by the ulnar nerve.

60. A 28 year old rugby player injures his right humerus and on examination is noted to have a
minor sensory deficit overlying the point of deltoid insertion into the humerus.

You answered Ulnar nerve

The correct answer is Axillary nerve

This patch of skin is supplied by the axillary nerve


Theme from April 2014 exam

Please rate this question:


Discuss and give feedback
Next question

Brachial plexus

Origin Anterior rami of C5 to T1

Sections of the  Roots, trunks, divisions, cords, branches


plexus  Mnemonic:Real Teenagers Drink Cold Beer

Roots  Located in the posterior triangle


 Pass between scalenus anterior and medius

Trunks  Located posterior to middle third of clavicle


 Upper and middle trunks related superiorly to the subclavian artery
 Lower trunk passes over 1st rib posterior to the subclavian artery

Divisions Apex of axilla

Cords Related to axillary artery

Diagram illustrating the branches of the brachial plexus


Image sourced from Wikipedia

Cutaneous sensation of the upper limb

Image sourced from Wikipedia

Next question
Question 61 of 560

A 53 year old man is undergoing a radical gastrectomy for carcinoma of the stomach. Which of the
following structures will need to be divided to gain access to the coeliac axis?

Lesser omentum

Greater omentum

Falciform ligament

Median arcuate ligament

Gastrosplenic ligament

The lesser omentum will need to be divided. During a radical gastrectomy this forms one of the
nodal stations that will need to be taken.
Please rate this question:

Discuss and give feedback


Next question

Coeliac axis

The coeliac axis has three main branches.

 Left gastric
 Hepatic: branches-Right Gastric, Gastroduodenal, Right Gastroepiploic, Superior
Pancreaticoduodenal, Cystic (occasionally).
 Splenic: branches- Pancreatic, Short Gastric, Left Gastroepiploic

It occasionally gives off one of the inferior phrenic arteries.


Image sourced from Wikipedia

Relations
Anteriorly Lesser omentum

Right Right coeliac ganglion and caudate process of liver

Left Left coeliac ganglion and gastric cardia

Inferiorly Upper border of pancreas and renal vein

Next question
Question 62 of 560

A 76 year old man complains of symptoms of claudication. The decision is made to measure his
ankle brachial pressure index. The signal from the dorsalis pedis artery is auscultated with a hand
held doppler device. This vessel is the continuation of which of the following?

Posterior tibial artery

Anterior tibial artery

Peroneal artery

Popliteal artery

None of the above

The dorsalis pedis is a continuation of the anterior tibial artery.


Please rate this question:

Discuss and give feedback


Next question

Foot- anatomy

Arches of the foot


The foot is conventionally considered to have two arches.

 The longitudinal arch is higher on the medial than on the lateral side. The posterior part of
the calcaneum forms a posterior pillar to support the arch. The lateral part of this structure
passes via the cuboid bone and the lateral two metatarsal bones. The medial part of this
structure is more important. The head of the talus marks the summit of this arch, located
between the sustentaculum tali and the navicular bone. The anterior pillar of the medial arch
is composed of the navicular bone, the three cuneiforms and the medial three metatarsal
bones.
 The transverse arch is situated on the anterior part of the tarsus and the posterior part of the
metatarsus. The cuneiforms and metatarsal bases narrow inferiorly, which contributes to the
shape of the arch.

Intertarsal joints
Sub talar joint Formed by the cylindrical facet on the lower surface of the body of the
talus and the posterior facet on the upper surface of the calcaneus. The
facet on the talus is concave anteroposteriorly, the other is convex. The
synovial cavity of this joint does not communicate with any other joint.

Talocalcaneonavicular The anterior part of the socket is formed by the concave articular
joint surface of the navicular bone, posteriorly by the upper surface of the
sustentaculum tali. The talus sits within this socket

Calcaneocuboid joint Highest point in the lateral part of the longitudinal arch. The lower
aspect of this joint is reinforced by the long plantar and plantar
calcaneocuboid ligaments.

Transverse tarsal joint The talocalcaneonavicular joint and the calcaneocuboid joint extend
across the tarsus in an irregular transverse plane, between the talus and
calcaneus behind and the navicular and cuboid bones in front. This
plane is termed the transverse tarsal joint.

Cuneonavicular joint Formed between the convex anterior surface of the navicular bone and
the concave surface of the the posterior ends of the three cuneiforms.

Intercuneiform joints Between the three cuneiform bones.

Cuneocuboid joint Between the circular facets on the lateral cuneiform bone and the
cuboid. This joint contributes to the tarsal part of the transverse arch.

A detailed knowledge of the joints is not required for MRCS Part A. However, the contribution they
play to the overall structure of the foot should be appreciated

Ligaments of the ankle joint and foot


Image sourced from Wikipedia

Muscles of the foot


Muscle Origin Insertion Nerve Action
supply

Abductor Medial side of the calcaneus, Medial side of Medial Abducts the great toe
hallucis flexor retinaculum, plantar the base of the plantar
aponeurosis proximal nerve
phalanx

Flexor Medial process of the Via 4 tendons Medial Flexes all the joints of
digitorum calcaneus, plantar into the plantar the lateral 4 toes except
brevis eponeurosis. middle nerve for the interphalangeal
phalanges of joint.
the lateral 4
toes.

Abductor From the tubercle of the Together with Lateral Abducts the little toe at
digit calcaneus and from the flexor digit plantar the metatarsophalangeal
minimi plantar aponeurosis minimi brevis nerve joint
into the lateral
side of the
base of the
proximal
phalanx of the
little toe

Flexor From the medial side of the Into the Medial Flexes the
hallucis plantar surface of the cuboid proximal plantar metatarsophalangeal
brevis bone, from the adjacent part phalanx of the nerve joint of the great toe.
of the lateral cuneiform bone great toe, the
and from the tendon of tendon
tibialis posterior. contains a
sesamoid bone

Adductor Arises from two heads. The Lateral side of Lateral Adducts the great toe
hallucis oblique head arises from the the base of the plantar towards the second toe.
sheath of the peroneus longus proximal nerve Helps maintain the
tendon, and from the plantar phalanx of the transverse arch of the
surfaces of the bases of the great toe. foot.
2nd, 3rd and 4th metatarsal
bones. The transverse head
arises from the plantar
surface of the lateral 4
metatarsophalangeal joints
and from the deep transverse
metatarsal ligament.

Extensor On the dorsal surface of the Via four thin Deep Extend the
digitorum foot from the upper surface of tendons which peroneal metatarsophalangeal
brevis the calcaneus and its run forward joint of the medial four
associated fascia and medially toes. It is unable to
to be inserted extend the
into the interphalangeal joint
medial four without the assistance of
toes. The the lumbrical muscles.
lateral three
tendons join
with hoods of
extensor
digitorum
longus.

Nerves in the foot


Lateral plantar nerve
Passes anterolaterally towards the base of the 5th metatarsal between flexor digitorum brevis and
flexor accessorius. On the medial aspect of the lateral plantar artery. At the base of the 5th
metatarsal it splits into superficial and deep branches.

Medial plantar nerve


Passes forwards with the medial plantar artery under the cover of the flexor retinaculum to the
interval between abductor hallucis and flexor digitorum brevis on the sole of the foot.

Plantar arteries
Arise under the cover of the flexor retinaculum, midway between the tip of the medial malleolus and
the most prominent part of the medial side of the heel.

 Medial plantar artery. Passes forwards medial to medial plantar nerve in the space between
abductor hallucis and flexor digitorum brevis.Ends by uniting with a branch of the 1st plantar
metatarsal artery.
 Lateral plantar artery. Runs obliquely across the sole of the foot. It lies lateral to the lateral
plantar nerve. At the base of the 5th metatarsal bone it arches medially across the foot on
the metatarsals

Dorsalis pedis artery


This vessel is a direct continuation of the anterior tibial artery. It commences on the front of the ankle
joint and runs to the proximal end of the first metatarsal space. Here is gives off the arcuate artery
and continues forwards as the first dorsal metatarsal artery. It is accompanied by two veins
throughout its length. It is crossed by the extensor hallucis brevis
Next question
Question 63 of 560

A 67 year old man is due to undergo a revisional total hip replacement using a posterior approach.
After dividing gluteus maximus in the line of its fibres there is brisk arterial bleeding. Which of the
following vessels is likely to be responsible?

Profunda femoris artery

External iliac artery

Internal iliac artery

Obturator artery

Inferior gluteal artery

Theme from April 2014 Exam


The inferior gluteal artery runs on the deep surface of the gluteus maximus muscle. It is a branch of
the internal iliac artery. It is commonly divided during the posterior approach to the hip joint.
Please rate this question:

Discuss and give feedback


Next question

Hip joint

 Head of femur articulates with acetabulum of the pelvis


 Both covered by articular hyaline cartilage
 The acetabulum forms at the union of the ilium, pubis, and ischium
 The triradiate cartilage (Y-shaped growth plate) separates the pelvic bones
 The acetabulum holds the femoral head by the acetabular labrum
 Normal angle between femoral head and femoral shaft is 130o

Ligaments

 Transverse ligament: joints anterior and posterior ends of the articular cartilage
 Head of femur ligament (ligamentum teres): acetabular notch to the fovea. Contains arterial
supply to head of femur in children.
Image sourced from Wikipedia

Image sourced from Wikipedia

Extracapsular ligaments

 Iliofemoral ligament: inverted Y shape. Anterior iliac spine to the trochanteric line
 Pubofemoral ligament: acetabulum to lesser trochanter
 Ischiofemoral ligament: posterior support. Ischium to greater trochanter.

Blood supply
Medial circumflex femoral and lateral circumflex femoral arteries (Branches of profunda femoris).
Also from the inferior gluteal artery. These form an anastomosis and travel to up the femoral neck to
supply the head.
Next question
Question 64 of 560

A 17 year old lady presents with right iliac fossa pain and diagnosed as having acute appendicitis.
You take her to theatre to perform a laparoscopic appendicectomy. During the procedure the scrub
nurse distracts you and you inadvertently avulse the appendicular artery. The ensuing haemorrhage
is likely to be supplied directly from which vessel?

Inferior mesenteric artery

Superior mesenteric artery

Ileo-colic artery

Internal iliac artery

None of the above

Theme from April 2014 exam


The appendicular artery is a branch of the ileocolic artery.
Please rate this question:

Discuss and give feedback


Next question

Appendix

 Location: Base of caecum.


 Up to 10cm long.
 Mainly lymphoid tissue (Hence mesenteric adenitis may mimic appendicitis).
 Caecal taenia coli converge at base of appendix and form a longitudinal muscle cover over
the appendix. This convergence should facilitate its identification at surgery if it is retrocaecal
and difficult to find (which it can be when people start doing appendicectomies!)
 Arterial supply: Appendicular artery (branch of the ileocolic).
 It is intra peritoneal.

McBurney's point

 1/3 of the way along a line drawn from the Anterior Superior Iliac Spine to the Umbilicus
6 Positions:

 Retrocaecal 74%
 Pelvic 21%
 Postileal
 Subcaecal
 Paracaecal
 Preileal

Next question
Question 65 of 560

A 63 year old man who smokes heavily presents with dyspepsia. He is tested and found to be
positive for helicobacter pylori infection. One evening he has an episode of haematemesis and
collapses. What is the most likely vessel to be responsible?

Portal vein

Short gastric arteries

Superior mesenteric artery

Gastroduodenal artery

None of the above

Theme from January 2012 exam

He is most likely to have a posteriorly sited duodenal ulcer. These can invade the gastroduodenal
artery and present with major bleeding. Although gastric ulcers may invade vessels they do not tend
to produce major bleeding of this nature.

Please rate this question:

Discuss and give feedback

Next question

Gastroduodenal artery

Supplies
Pylorus, proximal part of the duodenum, and indirectly to the pancreatic head (via the anterior and
posterior superior pancreaticoduodenal arteries)
Path
Most commonly arises from the common hepatic artery of the coeliac trunk
Terminates by bifurcating into the right gastroepiploic artery and the superior pancreaticoduodenal
artery

Image showing stomach reflected superiorly to illustrate the relationship of the gastroduodenal artery
to the first part of the duodenum

Image sourced from Wikipedia

Next question
Question 66 of 560

Which of the following nerves is responsible for the cremasteric reflex?

Lateral femoral cutaneous nerve

Femoral nerve

Obturator nerve

Genitofemoral nerve

None of the above

The motor and sensory fibres of the genitofemoral nerve are tested in the cremasteric reflex. A small
contribution is also played by the ilioinguinal nerve and thus the reflex may be lost following an
inguinal hernia repair.
Please rate this question:

Discuss and give feedback


Next question

Genitofemoral nerve

Supplies
Small area of the upper medial thigh.

Path

 Arises from the first and second lumbar nerves.


 Passes obliquely through psoas major, and emerges from its medial border opposite the
fibrocartilage between the third and fourth lumbar vertebrae.
 It then descends on the surface of psoas major, under cover of the peritoneum
 Divides into genital and femoral branches.
 The genital branch passes through the inguinal canal, within the spermatic cord, to supply
the skin overlying the skin and fascia of the scrotum. The femoral branch enters the thigh
posterior to the inguinal ligament, lateral to the femoral artery. It supplies an area of skin and
fascia over the femoral triangle.
 It may be injured during abdominal or pelvic surgery, or during inguinal hernia repairs
Question 67 of 560

Which of the structures listed below are not located within the mediastinum?

Thymus

Heart

Great vessels

Arch of azygos vein

Vertebral bodies

The vertebral bodies lie outside of the mediastinum, as do the lungs.


Please rate this question:

Discuss and give feedback


Next question

Mediastinum

Region between the pulmonary cavities.


It is covered by the mediastinal pleura. It does not contain the lungs.
It extends from the thoracic inlet superiorly to the diaphragm inferiorly.

Mediastinal regions

 Superior mediastinum (between manubriosternal angle and T4/5)


 Middle mediastinum
 Posterior mediastinum
 Anterior mediastinum

Region Contents
Region Contents

Superior mediastinum  Superior vena cava


 Brachiocephalic veins
 Arch of aorta
 Thoracic duct
 Trachea
 Oesophagus
 Thymus
 Vagus nerve
 Left recurrent laryngeal nerve
 Phrenic nerve

Anterior mediastinum  Thymic remnants


 Lymph nodes
 Fat

Middle mediastinum  Pericardium


 Heart
 Aortic root
 Arch of azygos vein
 Main bronchi

Posterior mediastinum  Oesophagus


 Thoracic aorta
 Azygos vein
 Thoracic duct
 Vagus nerve
 Sympathetic nerve trunks
 Splanchnic nerves

Next question
Question 68 of 560

A 22 year old man is stabbed in the chest at the level of the junction between the sternum and
manubrium. Which structure is at greatest risk?

Left atrium

Oesophagus

Thyroid gland

Inferior vena cava

Aortic arch

Theme from 2014 Exam


At the level of the Angle of Louis (Manubriosternal angle), is the surface marking for the aortic arch.
The oesophagus is posteriorly located and at less risk.
Please rate this question:

Discuss and give feedback


Next question

Sternal angle

Anatomical structures at the level of the manubrium and upper sternum


Upper part of the manubrium  Left brachiocephalic vein
 Brachiocephalic artery
 Left common carotid
 Left subclavian artery

Lower part of the manubrium/  Costal cartilages of the 2nd ribs


manubrio-sternal angle  Transition point between superior and inferior
mediastinum
 Arch of the aorta
 Tracheal bifurcation
 Union of the azygos vein and superior vena
cava
 The thoracic duct crosses to the midline

Next question
Question 69 of 560

Which of these nerves passes through the greater and lesser sciatic foramina?

Pudendal nerve

Sciatic nerve

Superior gluteal nerve

Inferior gluteal nerve

Posterior cutaneous nerve of the thigh

Structures passing through the lesser and greater sciatic foramina (medial to lateral): PIN

 Pudendal nerve
 Internal pudendal artery
 Nerve to obturator internus

Theme from 2010 Exam


Theme from January 2013 Exam
Theme from January 2014 Exam
The pudendal nerve originates from the ventral rami of the second, third, and fourth sacral nerves
(S2, S3, S4).

It passes between the piriformis and coccygeus muscles and exits the pelvis through the the greater
sciatic foramen. It crosses the spine of the ischium and reenters the pelvis through the lesser sciatic
foramen. It passes through the pudendal canal.

The pudendal nerve gives off the inferior rectal nerves. It terminates into 2 branches: perineal nerve,
and the dorsal nerve of the penis or the dorsal nerve of the clitoris.
Please rate this question:

Discuss and give feedback


Next question

Greater sciatic foramen

Contents
Nerves  Sciatic Nerve
 Superior and Inferior Gluteal Nerves
 Pudendal Nerve
 Posterior Femoral Cutaneous Nerve
 Nerve to Quadratus Femoris
 Nerve to Obturator internus

Vessels  Superior Gluteal Artery and vein


 Inferior Gluteal Artery and vein
 Internal Pudendal Artery and vein

Piriformis
The piriformis is a landmark for identifying structures passing out of the sciatic notch

 Above piriformis: Superior gluteal vessels


 Below piriformis: Inferior gluteal vessels, sciatic nerve (10% pass through it, <1% above it),
posterior cutaneous nerve of the thigh

Greater sciatic foramen boundaries


Anterolaterally Greater sciatic notch of the ilium

Posteromedially Sacrotuberous ligament

Inferior Sacrospinous ligament and the ischial spine

Superior Anterior sacroiliac ligament

The greater sciatic foramen


Image sourced from Wikipedia

Structures passing between both foramina (Medial to lateral)

 Pudendal nerve
 Internal pudendal artery
 Nerve to obturator internus

Contents of the lesser sciatic foramen

 Tendon of the obturator internus


 Pudendal nerve
 Internal pudendal artery and vein
 Nerve to the obturator internus

Next question
Question 70 of 560

A cervical rib is due to which of the following?

Hyperplasia of the annulus fibrosus

Proliferation of the nucleus pulposus

Fusion of the transverse processes of the 6th and 7th cervical vertebrae

An accessory cervical vertebra

Elongation of the transverse processes of the 7th cervical vertebra

Cervical ribs occur as a result of the elongation of the transverse process of the 7th cervical
vertebra. It is usually a fibrous band that attaches to the first thoracic rib.
Please rate this question:

Discuss and give feedback


Next question

Cervical ribs

 0.2-0.4% incidence
 Most cases present with neurological symptoms
 Consist of an anomalous fibrous band that often originates from C7 and may arc towards,
but rarely reaches the sternum
 Congenital cases may present around the third decade, some cases are reported to occur
following trauma
 Bilateral in up to 70%
 Compression of the subclavian artery may produce absent radial pulse on clinical
examination and in particular may result in a positive Adsons test (lateral flexion of the neck
towards the symptomatic side and traction of the symptomatic arm- leads to obliteration of
radial pulse)
 Treatment is most commonly undertaken when there is evidence of neurovascular
compromise. A transaxillary approach is the traditional operative method for excision.
Image sourced from Wikipedia

3D reconstruction of a left-sided cervical rib


Next question
Question 71 of 560

Which of the structures listed below is not a content of the carotid sheath?

Internal jugular vein

Internal carotid artery

Vagus nerve

Recurrent laryngeal nerve

Common carotid artery

Contents of carotid sheath:


Common carotid artery
Internal carotid artery
Internal jugular vein
Vagus nerve

Theme from April 2014 exam


Please rate this question:

Discuss and give feedback


Next question

Common carotid artery

The right common carotid artery arises at the bifurcation of the brachiocephalic trunk, the left
common carotid arises from the arch of the aorta. Both terminate at the level of the upper border of
the thyroid cartilage (the lower border of the third cervical vertebra) by dividing into the internal and
external carotid arteries.

Left common carotid artery


This vessel arises immediately to the left and slightly behind the origin of the brachiocephalic trunk.
Its thoracic portion is 2.5- 3.5 cm in length and runs superolaterally to the sternoclavicular joint.

In the thorax
The vessel is in contact, from below upwards, with the trachea, left recurrent laryngeal nerve, left
margin of the oesophagus. Anteriorly the left brachiocephalic vein runs across the artery, and the
cardiac branches from the left vagus descend in front of it. These structures together with the
thymus and the anterior margins of the left lung and pleura separate the artery from the manubrium.
In the neck
The artery runs superiorly deep to sternocleidomastoid and then enters the anterior triangle. At this
point it lies within the carotid sheath with the vagus nerve and the internal jugular vein. Posteriorly
the sympathetic trunk lies between the vessel and the prevertebral fascia. At the level of C7 the
vertebral artery and thoracic duct lie behind it. The anterior tubercle of C6 transverse process is
prominent and the artery can be compressed against this structure (it corresponds to the level of the
cricoid).
Anteriorly at C6 the omohyoid muscle passes superficial to the artery.
Within the carotid sheath the jugular vein lies lateral to the artery.

Right common carotid artery


The right common carotid arises from the brachiocephalic artery. The right common carotid artery
corresponds with the cervical portion of the left common carotid, except that there is no thoracic duct
on the right. The oesophagus is less closely related to the right carotid than the left.

Summary points about the carotid anatomy

Path
Passes behind the sternoclavicular joint (12% patients above this level) to the upper border of the
thyroid cartilage, to divide into the external (ECA) and internal carotid arteries (ICA).

Relations

 Level of 6th cervical vertebra crossed by omohyoid


 Then passes deep to the thyrohyoid, sternohyoid, sternomastoid muscles.
 Passes anterior to the carotid tubercle (transverse process 6th cervical vertebra)-NB
compression here stops haemorrhage.
 The inferior thyroid artery passes posterior to the common carotid artery.
 Then : Left common carotid artery crosses the thoracic duct, Right common carotid artery
crossed by recurrent laryngeal nerve
Image sourced from Wikipedia

Next question
Question 72 of 560

A 22 year old man is undergoing a wedge excision of his great toenail. As the surgeon passes a
needle into the area to administer local anaesthetic, the patient notices a sharp pain. By which
pathway will this sensation be conveyed to the central nervous system?

Anterior corticospinal tract

Posterior spinocerebellar tract

Cuneate fasciculus

Vestibulospinal tract

Spinothalamic tract

Spinothalamic tract- Pain and temperature


Vestibulospinal tract- Motor neuronal signals relating to posture
Cuneate fasciculus- Fine touch, pressure and proprioception
Posterior spinocerebellar tract- Proprioceptive signals to cerebellum
Anterior corticospinal tract- Conveys motor signals from precentral gyrus to motor cells within the
cord

Theme from January 2011 Exam


Pain impulses are transmitted via the spinothalamic tract

Please rate this question:

Discuss and give feedback

Next question

Spinothalamic tract
The spinothalamic tract transmits impulses from receptors which measure crude touch, pain and
temperature. The spinothalamic tract comprises the lateral and anterior spinothalamic tracts, the
former typically transmits pain and temperature and the latter crude touch and pressure. Neurones
transmitting these signals will typically ascend by one or two vertebral levels in Lissaurs tract prior to
decussating in the spinal cord itself. Neurones then pass rostrally in the cord to connect at the
thalamus.

Next question
Question 73 of 560

A 73 year old lady is admitted with brisk rectal bleeding. Despite attempts at resuscitation the
bleeding proceeds to cause haemodynamic compromise. An upper GI endoscopy is normal. A
mesenteric angiogram is performed and a contrast blush is seen in the region of the sigmoid colon.
The radiologist decides to embolise the vessel supplying this area. At what spinal level does it leave
the aorta?

L2

L1

L4

L3

T10

Theme from 2009 Exam

The inferior mesenteric artery leaves the aorta at L3. It supplies the left colon and sigmoid. Its
proximal continuation to communicate with the middle colic artery is via the marginal artery.
Please rate this question:

Discuss and give feedback


Next question

Levels

Transpyloric plane
Level of the body of L1

 Pylorus stomach
 Left kidney hilum (L1- left one!)
 Fundus of the gallbladder
 Neck of pancreas
 Duodenojejunal flexure
 Superior mesenteric artery
 Portal vein
 Left and right colic flexure
 Root of the transverse mesocolon
 2nd part of the duodenum
 Upper part of conus medullaris
 Spleen

Can be identified by asking the supine patient to sit up without using their arms. The plane is located
where the lateral border of the rectus muscle crosses the costal margin.

Anatomical planes
Subcostal plane Lowest margin of 10th costal cartilage

Intercristal plane Level of body L4 (highest point of iliac crest)

Intertubercular plane Level of body L5

Common level landmarks


Inferior mesenteric artery L3

Bifurcation of aorta into common iliac arteries L4

Formation of IVC L5 (union of common iliac veins)

Diaphragm apertures  Vena cava T8


 Oesophagus T10
 Aortic hiatus T12

Next question
Question 74 of 560

A 23 year old lady with troublesome axillary hyperhidrosis is undergoing a thorascopic


sympathectomy to treat the condition. Which of the following structures will need to be divided to
access the sympathetic trunk?

Intercostal vein

Intercostal artery

Parietal pleura

Visceral pleura

None of the above

The sympathetic chain lies posterior to the parietal pleura. During a thorascopic sympathetomy this
structure will need to be divided. The intercostal vessels lie posteriorly. They may be damaged with
troublesome bleeding but otherwise are best left alone as deliberate division will not improve surgical
access.
Please rate this question:

Discuss and give feedback


Next question

Sympathetic nervous system- anatomy

The cell bodies of the pre-ganglionic efferent neurones lie in the lateral horn of the grey matter of the
spinal cord in the thoraco-lumbar regions.
The pre-ganglionic efferents leave the spinal cord at levels T1-L2. These pass to the sympathetic
chain.
Lateral branches of the sympathetic chain connect it to every spinal nerve. These post ganglionic
nerves will pass to structures that receive sympathetic innervation at the periphery.

Sympathetic chains
These lie on the vertebral column and run from the base of the skull to the coccyx.
Cervical Lie anterior to the transverse processes of the cervical vertebrae and posterior to
region the carotid sheath.
Thoracic Lie anterior to the neck of the upper ribs and and lateral sides of the lower thoracic
region vertebrae.They are covered by the parietal pleura

Lumbar Enter by passing posterior to the medial arcuate ligament. Lie anteriorly to the
region vertebrae and medial to psoas major.

Sympathetic ganglia

 Superior cervical ganglion lies anterior to C2 and C3.


 Middle cervical ganglion (if present) C6
 Stellate ganglion- anterior to transverse process of C7, lies posterior to the subclavian artery,
vertebral artery and cervical pleura.
 Thoracic ganglia are segmentally arranged.
 There are usually 4 lumbar ganglia.

Clinical importance

 Interruption of the head and neck supply of the sympathetic nerves will result in an ipsilateral
Horners syndrome.
 For treatment of hyperhidrosis the sympathetic denervation can be achieved by removing the
second and third thoracic ganglia with their rami. Removal of T1 will cause a Horners
syndrome and is therefore not performed.
 In patients with vascular disease of the lower limbs a lumbar sympathetomy may be
performed, either radiologically or (more rarely now) surgically. The ganglia of L2 and below
are disrupted. If L1 is removed then ejaculation may be compromised (and little additional
benefit conferred as the preganglionic fibres do not arise below L2.

Next question
Question 75 of 560

In which space is a lumbar puncture performed?

Subdural space

Epidural space

Subarachnoid space

Extradural space

Intraventricular space

Theme from January 2013 exam


Samples of CSF are normally obtained by inserting a needle between the third and fourth lumbar
vertebrae. The tip of the needle lies in the sub arachnoid space, the spinal cord terminates at L1 and
is not at risk of injury. Clinical evidence of raised intracranial pressure is a contraindication to lumbar
puncture.
Please rate this question:

Discuss and give feedback


Next question

Cerebrospinal fluid

The CSF fills the space between the arachnoid mater and pia mater (covering surface of the brain).
The total volume of CSF in the brain is approximately 150ml. Approximately 500 ml is produced by
the ependymal cells in the choroid plexus (70%), or blood vessels (30%). It is reabsorbed via the
arachnoid granulations which project into the venous sinuses.

Circulation
1. Lateral ventricles (via foramen of Munro)
2. 3rd ventricle
3. Cerebral aqueduct (aqueduct of Sylvius)
4. 4th ventricle (via foramina of Magendie and Luschka)
5. Subarachnoid space
6. Reabsorbed into the venous system via arachnoid granulations into superior sagittal sinus

Composition
 Glucose: 50-80mg/dl
 Protein: 15-40 mg/dl
 Red blood cells: Nil
 White blood cells: 0-3 cells/ mm3

Next question
Question 76 of 560

A 21 year old man is stabbed in the antecubital fossa. A decision is made to surgically explore the
wound. At operation the surgeon dissects down onto the brachial artery. A nerve is identified
medially, which nerve is it likely to be?

Radial

Recurrent branch of median

Anterior interosseous

Ulnar

Median

Theme from September 2012 Exam


Please rate this question:

Discuss and give feedback


Next question

Median nerve

The median nerve is formed by the union of a lateral and medial root respectively from the lateral
(C5,6,7) and medial (C8 and T1) cords of the brachial plexus; the medial root passes anterior to the
third part of the axillary artery. The nerve descends lateral to the brachial artery, crosses to its
medial side (usually passing anterior to the artery). It passes deep to the bicipital aponeurosis and
the median cubital vein at the elbow.
It passes between the two heads of the pronator teres muscle, and runs on the deep surface of
flexor digitorum superficialis (within its fascial sheath).
Near the wrist it becomes superficial between the tendons of flexor digitorum superficialis and flexor
carpi radialis, deep to palmaris longus tendon. It passes deep to the flexor retinaculum to enter the
palm, but lies anterior to the long flexor tendons within the carpal tunnel.

Branches
Region Branch
Region Branch

Upper arm No branches, although the nerve commonly communicates with the
musculocutaneous nerve

Forearm Pronator teres


Flexor carpi radialis
Palmaris longus
Flexor digitorum superficialis
Flexor pollicis longus
Flexor digitorum profundus (only the radial half)

Distal Palmar cutaneous branch


forearm

Hand Motor supply (LOAF)


(Motor)

 Lateral 2 lumbricals
 Opponens pollicis
 Abductor pollicis brevis
 Flexor pollicis brevis

Hand  Over thumb and lateral 2 ½ fingers


(Sensory)  On the palmar aspect this projects proximally, on the dorsal aspect only the
distal regions are innervated with the radial nerve providing the more
proximal cutaneous innervation.

Patterns of damage
Damage at wrist

 e.g. carpal tunnel syndrome


 paralysis and wasting of thenar eminence muscles and opponens pollicis (ape hand
deformity)
 sensory loss to palmar aspect of lateral (radial) 2 ½ fingers

Damage at elbow, as above plus:


 unable to pronate forearm
 weak wrist flexion
 ulnar deviation of wrist

Anterior interosseous nerve (branch of median nerve)

 leaves just below the elbow


 results in loss of pronation of forearm and weakness of long flexors of thumb and index
finger

Topography of the median nerve

Image sourced from Wikipedia

Next question
Question 77 of 560

A 65 year old man with long standing atrial fibrillation develops an embolus to the lower leg. The
decision is made to perform an embolectomy, utilising a trans popliteal approach. After incising the
deep fascia, which of the following structures will the surgeons encounter first on exploring the
central region of the popliteal fossa?

Popliteal vein

Common peroneal nerve

Popliteal artery

Tibial nerve

None of the above

Theme from April 2015 Exam


Theme from April 2016 Exam
The tibial nerve lies superior to the vessels in the inferior aspect of the popliteal fossa. In the upper
part of the fossa the tibial nerve lies lateral to the vessels, it then passes superficial to them to lie
medially. The popliteal artery is the deepest structure in the popliteal fossa.
Please rate this question:

Discuss and give feedback


Next question

Popliteal fossa

Boundaries of the popliteal fossa


Laterally Biceps femoris above, lateral head of gastrocnemius and plantaris below

Medially Semimembranosus and semitendinosus above, medial head of gastrocnemius below

Floor Popliteal surface of the femur, posterior ligament of knee joint and popliteus muscle
Roof Superficial and deep fascia

Image showing the popliteal fossa

© Image provided by the University of Sheffield

Contents

 Popliteal artery and vein


 Small saphenous vein
 Common peroneal nerve
 Tibial nerve
 Posterior cutaneous nerve of the thigh
 Genicular branch of the obturator nerve
 Lymph nodes

Next question
Question 78 of 560

A 43 year old man is undergoing a right hemicolectomy and the ileo-colic artery is ligated. From
which of the following vessels is is derived?

Inferior mesenteric artery

Superior mesenteric artery

Coeliac axis

Aorta

None of the above

The ileocolic artery is a branch of the SMA and supplies the right colon and terminal ileum. The
transverse colon is supplied by the middle colic artery. As veins accompany arteries in the
mesentery and are lined by lymphatics, high ligation is the norm in cancer resections. The ileo-colic
artery branches off the SMA near the duodenum.

Please rate this question:

Discuss and give feedback

Next question

Colon anatomy

The colon commences with the caecum. This represents the most dilated segment of the human
colon and its base (which is intraperitoneal) is marked by the convergence of teniae coli. At this point
is located the vermiform appendix. The colon continues as the ascending colon, the posterior aspect
of which is retroperitoneal. The line of demarcation between the intra and retro peritoneal right colon
is visible as a white line, in the living, and forms the line of incision for colonic resections.
The ascending colon becomes the transverse colon after passing the hepatic flexure. At this located
the colon becomes wholly intra peritoneal once again. The superior aspect of the transverse colon is
the point of attachment of the transverse colon to the greater omentum. This is an important
anatomical site since division of these attachments permits entry into the lesser sac. Separation of
the greater omentum from the transverse colon is a routine operative step in both gastric and colonic
resections.

At the left side of the abdomen the transverse colon passes to the left upper quadrant and makes an
oblique inferior turn at the splenic flexure. Following this, the posterior aspect becomes
retroperitoneal once again.

At the level of approximately L4 the descending colon becomes wholly intraperitoneal and becomes
the sigmoid colon. Whilst the sigmoid is wholly intraperitoneal there are usually attachments laterally
between the sigmoid and the lateral pelvic sidewall. These small congenital adhesions are not formal
anatomical attachments but frequently require division during surgical resections.

At its distal end the sigmoid passes to the midline and at the region around the sacral promontary it
becomes the upper rectum. This transition is visible macroscopically as the point where the teniae
fuse. More distally the rectum passes through the peritoneum at the region of the peritoneal
reflection and becomes extraperitoneal.

Arterial supply
Superior mesenteric artery and inferior mesenteric artery: linked by the marginal artery.
Ascending colon: ileocolic and right colic arteries
Transverse colon: middle colic artery
Descending and sigmoid colon: inferior mesenteric artery

Venous drainage
From regional veins (that accompany arteries) to superior and inferior mesenteric vein

Lymphatic drainage
Initially along nodal chains that accompany supplying arteries, then para-aortic nodes.

Embryology
Midgut- Second part of duodenum to 2/3 transverse colon
Hindgut- Distal 1/3 transverse colon to anus

Peritoneal location
The right and left colon are part intraperitoneal and part extraperitoneal. The sigmoid and transverse
colon are generally wholly intraperitoneal. This has implications for the sequelae of perforations,
which will tend to result in generalised peritonitis in the wholly intra peritoneal segments.

Colonic relations
Region of colon Relation

Caecum/ right colon Right ureter, gonadal vessels

Hepatic flexure Gallbladder (medially)

Splenic flexure Spleen and tail of pancreas

Distal sigmoid/ upper rectum Left ureter

Rectum Ureters, autonomic nerves, seminal vesicles, prostate, urethra (distally)

Next question
Question 79 of 560

A 53 year old man is undergoing a distal pancreatectomy for trauma. Which of the following vessels
is responsible for the arterial supply to the tail of the pancreas?

Splenic artery

Pancreaticoduodenal artery

Gastric artery

Hepatic artery

Superior mesenteric artery

Pancreatic head is supplied by the pancreaticoduodenal artery


Pancreatic tail is supplied by branches of the splenic artery

Theme from April 2014 Exam


There is an arterial "watershed" in the supply between the head and tail of the pancreas. The head is
supplied by the pancreaticoduodenal artery and the tail is supplied by branches of the splenic artery.
Please rate this question:

Discuss and give feedback


Next question

Pancreas

The pancreas is a retroperitoneal organ and lies posterior to the stomach. It may be accessed
surgically by dividing the peritoneal reflection that connects the greater omentum to the transverse
colon. The pancreatic head sits in the curvature of the duodenum. Its tail lies close to the hilum of
the spleen, a site of potential injury during splenectomy.

Relations
Posterior to the pancreas
Pancreatic head Inferior vena cava
Common bile duct
Right and left renal veins
Superior mesenteric vein and artery
Pancreatic neck Superior mesenteric vein, portal vein

Pancreatic body- Left renal vein


Crus of diaphragm
Psoas muscle
Adrenal gland
Kidney
Aorta

Pancreatic tail Left kidney

Anterior to the pancreas


Pancreatic head 1st part of the duodenum
Pylorus
Gastroduodenal artery
SMA and SMV(uncinate process)

Pancreatic body Stomach


Duodenojejunal flexure

Pancreatic tail Splenic hilum

Superior to the pancreas


Coeliac trunk and its branches common hepatic artery and splenic artery

Grooves of the head of the pancreas


2nd and 3rd part of the duodenum

Arterial supply

 Head: pancreaticoduodenal artery


 Rest: splenic artery

Venous drainage

 Head: superior mesenteric vein


 Body and tail: splenic vein
Ampulla of Vater

 Merge of pancreatic duct and common bile duct


 Is an important landmark, halfway along the second part of the duodenum, that marks the
anatomical transition from foregut to midgut (also the site of transition between regions
supplied by coeliac trunk and SMA).

Image sourced from Wikipedia

Next question
Question 80 of 560

A 43 year old lady presents with varicose veins and undergoes a saphenofemoral disconnection,
long saphenous vein stripping to the ankle and isolated hook phlebectomies. Post operatively she
notices an area of numbness superior to her ankle. What is the most likely cause for this?

Sural nerve injury

Femoral nerve injury

Saphenous nerve injury

Common peroneal nerve injury

Superficial peroneal nerve injury

The sural nerve is related to the short saphenous vein. The saphenous nerve is related to the long
saphenous vein below the knee and for this reason full length stripping of the vein is no longer
advocated.
Please rate this question:

Discuss and give feedback


Next question

Saphenous vein

Long saphenous vein


This vein may be harvested for bypass surgery, or removed as treatment for varicose veins with
saphenofemoral junction incompetence.

 Originates at the 1st digit where the dorsal vein merges with the dorsal venous arch of the
foot
 Passes anterior to the medial malleolus and runs up the medial side of the leg
 At the knee, it runs over the posterior border of the medial epicondyle of the femur bone
 Then passes laterally to lie on the anterior surface of the thigh before entering an opening in
the fascia lata called the saphenous opening
 It joins with the femoral vein in the region of the femoral triangle at the saphenofemoral
junction
Tributaries

 Medial marginal
 Superficial epigastric
 Superficial iliac circumflex
 Superficial external pudendal veins

Short saphenous vein

 Originates at the 5th digit where the dorsal vein merges with the dorsal venous arch of the
foot, which attaches to the great saphenous vein.
 It passes around the lateral aspect of the foot (inferior and posterior to the lateral malleolus)
and runs along the posterior aspect of the leg (with the sural nerve)
 Passes between the heads of the gastrocnemius muscle, and drains into the popliteal vein,
approximately at or above the level of the knee joint.

Next question
Question 81 of 560

Which of the following muscles does not attach to the radius?

Pronator quadratus

Biceps

Brachioradialis

Supinator

Brachialis

The brachialis muscle inserts into the ulna. The other muscles are all inserted onto the radius.
Please rate this question:

Discuss and give feedback


Next question

Radius

The radius is one of the two long forearm bones that extends from the lateral side of the elbow to the
thumb side of the wrist. It has two expanded ends, of which the distal end is the larger. Key points
relating to its topography and relations are outlined below;

Upper end

 Articular cartilage- covers medial > lateral side


 Articulates with radial notch of the ulna by the annular ligament
 Muscle attachment- biceps brachii at the tuberosity

Shaft
Muscle attachment
Upper third of the body Supinator
Flexor digitorum superficialis
Flexor pollicis longus

Middle third of the body Pronator teres

Lower quarter of the body Pronator quadratus


Tendon of supinator longus

Lower end

 Quadrilateral
 Anterior surface- capsule of wrist joint
 Medial surface- head of ulna
 Lateral surface- ends in the styloid process
 Posterior surface: 3 grooves containing:

1. Tendons of extensor carpi radialis longus and brevis


2. Tendon of extensor pollicis longus
3. Tendon of extensor indicis

Image sourced from Wikipedia

Next question
Question 82 of 560

A 25 year old man is stabbed in the upper arm. The brachial artery is lacerated at the level of the
proximal humerus, and is being repaired. A nerve lying immediately lateral to the brachial artery is
also lacerated. Which of the following is the nerve most likely to be?

Ulnar nerve

Median nerve

Radial nerve

Intercostobrachial nerve

Axillary nerve

Theme from April 2015 exam


The brachial artery begins at the lower border of teres major and terminates in the cubital fossa by
branching into the radial and ulnar arteries. In the upper arm the median nerve lies closest to it in the
lateral position. In the cubital fossa it lies medial to it.
Image sourced from Wikipedia

Please rate this question:

Discuss and give feedback

Next question

Brachial artery

The brachial artery begins at the lower border of teres major as a continuation of the axillary artery. It
terminates in the cubital fossa at the level of the neck of the radius by dividing into the radial and
ulnar arteries.

Relations
Posterior relations include the long head of triceps with the radial nerve and profunda vessels
intervening. Anteriorly it is overlapped by the medial border of biceps.
It is crossed by the median nerve in the middle of the arm.
In the cubital fossa it is separated from the median cubital vein by the bicipital aponeurosis.
The basilic vein is in contact at the most proximal aspect of the cubital fossa and lies medially.

Next question
Question 83 of 560

What is the course of the median nerve relative to the brachial artery in the upper arm?

Medial to anterior to lateral

Lateral to posterior to medial

Medial to posterior to lateral

Medial to anterior to medial

Lateral to anterior to medial

Relations of median nerve to the brachial artery:


Lateral -> Anterior -> Medial

Theme from 2009, 2012 and 2014 Exams

The median nerve descends lateral to the brachial artery, it usually passes anterior to the artery to
lie on its medial side. It passes deep to the bicipital aponeurosis and the median cubital vein at the
elbow. It enters the forearm between the two heads of the pronator teres muscle.

Image sourced from Wikipedia


Please rate this question:

Discuss and give feedback

Next question

Brachial artery

The brachial artery begins at the lower border of teres major as a continuation of the axillary artery. It
terminates in the cubital fossa at the level of the neck of the radius by dividing into the radial and
ulnar arteries.

Relations
Posterior relations include the long head of triceps with the radial nerve and profunda vessels
intervening. Anteriorly it is overlapped by the medial border of biceps.
It is crossed by the median nerve in the middle of the arm.
In the cubital fossa it is separated from the median cubital vein by the bicipital aponeurosis.
The basilic vein is in contact at the most proximal aspect of the cubital fossa and lies medially.

Next question
Question 84 of 560

Which of the following is not a content of the cavernous sinus?

Oculomotor nerve

Internal carotid artery

Opthalmic nerve

Abducens nerve

Optic nerve

Mnemonic for contents of cavernous sinus:


O TOM CAT

Occulomotor nerve (III)


Trochlear nerve (IV)
Ophthalmic nerve (V1)
Maxillary nerve (V2)
Carotid artery
Abducent nerve (VI)
T

OTOM=lateral wall components


CA= components within sinus

The optic nerve lies above and outside the cavernous sinus.

Please rate this question:

Discuss and give feedback


Next question

Cavernous sinus

The cavernous sinuses are paired and are situated on the body of the sphenoid bone. It runs from
the superior orbital fissure to the petrous temporal bone.

Relations

Medial Lateral

Pituitary fossa
Temporal lobe
Sphenoid sinus

Contents

Lateral wall components (from top to bottom:)


Oculomotor nerve
Trochlear nerve
Ophthalmic nerve
Maxillary nerve

Contents of the sinus (from medial to lateral:)


Internal carotid artery (and sympathetic plexus)
Abducens nerve

Blood supply
Ophthalmic vein, superficial cortical veins, basilar plexus of veins posteriorly.

Drains into the internal jugular vein via: the superior and inferior petrosal sinuses
Image sourced from Wikipedia

Next question
Question 85 of 560

Surgical occlusion of which of these structures, will result in the greatest reduction in hepatic blood
flow?

Portal vein

Common hepatic artery

Right hepatic artery

Coeliac axis

Left hepatic artery

The portal vein transports 70% of the blood supply to the liver, while the hepatic artery provides
30%. The portal vein contains the products of digestion. The arterial and venous blood is dispersed
by sinusoids to the central veins of the liver lobules; these drain into the hepatic veins and then into
the IVC. The caudate lobe drains directly into the IVC rather than into other hepatic veins.
Please rate this question:

Discuss and give feedback


Next question

Liver

Structure of the liver


Right lobe  Supplied by right hepatic artery
 Contains Couinaud segments V to VIII (-/+Sg I)

Left lobe  Supplied by the left hepatic artery


 Contains Couinaud segments II to IV (+/- Sg1)

Quadrate lobe  Part of the right lobe anatomically, functionally is part of the left
 Couinaud segment IV
 Porta hepatis lies behind
 On the right lies the gallbladder fossa
 On the left lies the fossa for the umbilical vein

Caudate lobe  Supplied by both right and left hepatic arteries


 Couinaud segment I
 Lies behind the plane of the porta hepatis
 Anterior and lateral to the inferior vena cava
 Bile from the caudate lobe drains into both right and left hepatic ducts

Detailed knowledge of Couinaud segments is not required for MRCS

 Between the liver lobules are portal canals which contain the portal triad: Hepatic Artery,
Portal Vein, tributary of Bile Duct.

Relations of the liver


Anterior Postero inferiorly

Diaphragm Oesophagus

Xiphoid process Stomach

Duodenum

Hepatic flexure of colon

Right kidney

Gallbladder

Inferior vena cava

Porta hepatis
Location Postero inferior surface, it joins nearly at right angles with the left sagittal fossa, and
separates the caudate lobe behind from the quadrate lobe in front

Transmits  Common hepatic duct


 Hepatic artery
 Portal vein
 Sympathetic and parasympathetic nerve fibres
 Lymphatic drainage of the liver (and nodes)

Ligaments
Falciform ligament  2 layer fold peritoneum from the umbilicus to anterior liver surface
 Contains ligamentum teres (remnant umbilical vein)
 On superior liver surface it splits into the coronary and left
triangular ligaments

Ligamentum teres Joins the left branch of the portal vein in the porta hepatis

Ligamentum Remnant of ductus venosus


venosum

Arterial supply

 Hepatic artery

Venous

 Hepatic veins
 Portal vein

Nervous supply

 Sympathetic and parasympathetic trunks of coeliac plexus

Next question
Question 86 of 560

A 43 year old man is due to undergo an excision of the sub mandibular gland. Which of the following
incisions is the most appropriate for this procedure?

A transversely orientated incision 4cm below the mandible

A transversely orientated incision immediately inferior to the mandible

A vertical incision 3 cm anterior to the angle of the mandible and extending inferiorly

A transversely orientated incision 2cm above the mandible

A transversely orientated incision 12cm below the mandible

Theme from 2009 Exam


To access the sub mandibular gland a transverse incision 4cm below the mandible should be made.
Incisions located higher than this may damage the marginal mandibular branch of the facial nerve.
Please rate this question:

Discuss and give feedback


Next question

Anterior triangle of the neck

Boundaries

Anterior border of the Sternocleidomastoid


Lower border of mandible
Anterior midline

Sub triangles (divided by Digastric above and Omohyoid)

 Muscular triangle: Neck strap muscles


 Carotid triangle: Carotid sheath
 Submandibular Triangle (digastric)
Contents of the anterior triangle
Digastric triangle Submandibular gland
Submandibular nodes
Facial vessels
Hypoglossal nerve

Muscular triangle Strap muscles


External jugular vein

Carotid triangle Carotid sheath (Common carotid, vagus and internal jugular vein)
Ansa cervicalis

Nerve supply to digastric muscle

 Anterior: Mylohyoid nerve


 Posterior: Facial nerve

Image sourced from Wikipedia

Next question
Question 87 of 560

A 5 year old boy presents with recurrent headaches. As part of his assessment he undergoes an
MRI scan of his brain. This demonstrates enlargement of the lateral and third ventricles. Where is
the most likely site of obstruction?

Foramen of Luschka

Foramen of Magendie

Foramen of Munro

Aqueduct of Sylvius

None of the above

Theme from September 2013 Exam


Theme from April 2014 exam
The CSF flows from the 3rd to the 4th ventricle via the Aqueduct of Sylvius.
Please rate this question:

Discuss and give feedback


Next question

Cerebrospinal fluid

The CSF fills the space between the arachnoid mater and pia mater (covering surface of the brain).
The total volume of CSF in the brain is approximately 150ml. Approximately 500 ml is produced by
the ependymal cells in the choroid plexus (70%), or blood vessels (30%). It is reabsorbed via the
arachnoid granulations which project into the venous sinuses.

Circulation
1. Lateral ventricles (via foramen of Munro)
2. 3rd ventricle
3. Cerebral aqueduct (aqueduct of Sylvius)
4. 4th ventricle (via foramina of Magendie and Luschka)
5. Subarachnoid space
6. Reabsorbed into the venous system via arachnoid granulations into superior sagittal sinus

Composition
 Glucose: 50-80mg/dl
 Protein: 15-40 mg/dl
 Red blood cells: Nil
 White blood cells: 0-3 cells/ mm3

Next question
Question 88 of 560

A 23 year old man presents with appendicitis. A decision is made to perform an appendicectomy.
The operation commences with a 5cm incision centered on McBurneys point. Which of the following
structures will be encountered first during the dissection?

External oblique aponeurosis

Internal oblique muscle

Transversalis fascia

Rectus sheath

Peritoneum

Theme from April 2014 exam


The external oblique will be encountered first in this location. The rectus sheath lies more medially.
The external oblique muscle is the most superficial of the abdominal wall muscles. It originates from
the 5th to 12th ribs and passes inferomedially to insert into the linea alba, pubic tubercle and anterior
half of the iliac crest. It is innervated by the thoracoabdominal nerves (T7-T11) and sub costal
nerves.
Please rate this question:

Discuss and give feedback


Next question

Abdominal wall

The 2 main muscles of the abdominal wall are the rectus abdominis (anterior) and the quadratus
lumborum (posterior).
The remaining abdominal wall consists of 3 muscular layers. Each muscle passes from the lateral
aspect of the quadratus lumborum posteriorly to the lateral margin of the rectus sheath anteriorly.
Each layer is muscular posterolaterally and aponeurotic anteriorly.
Image sourced from Wikipedia

Muscles of abdominal wall


External  Lies most superficially
oblique  Originates from 5th to 12th ribs
 Inserts into the anterior half of the outer aspect of the iliac crest, linea
alba and pubic tubercle
 More medially and superiorly to the arcuate line, the aponeurotic layer
overlaps the rectus abdominis muscle
 The lower border forms the inguinal ligament
 The triangular expansion of the medial end of the inguinal ligament is
the lacunar ligament.

Internal  Arises from the thoracolumbar fascia, the anterior 2/3 of the iliac crest
oblique and the lateral 2/3 of the inguinal ligament
 The muscle sweeps upwards to insert into the cartilages of the lower 3
ribs
 The lower fibres form an aponeurosis that runs from the tenth costal
cartilage to the body of the pubis
 At its lowermost aspect it joins the fibres of the aponeurosis of
transversus abdominis to form the conjoint tendon.

Transversus  Innermost muscle


abdominis  Arises from the inner aspect of the costal cartilages of the lower 6 ribs ,
from the anterior 2/3 of the iliac crest and lateral 1/3 of the inguinal
ligament
 Its fibres run horizontally around the abdominal wall ending in an
aponeurosis. The upper part runs posterior to the rectus abdominis.
Lower down the fibres run anteriorly only.
 The rectus abdominis lies medially; running from the pubic crest and
symphysis to insert into the xiphoid process and 5th, 6th and 7th costal
cartilages. The muscles lies in a aponeurosis as described above.
 Nerve supply: anterior primary rami of T7-12

Surgical notes
During abdominal surgery it is usually necessary to divide either the muscles or their aponeuroses.
During a midline laparotomy it is desirable to divide the aponeurosis. This will leave the rectus
sheath intact above the arcuate line and the muscles intact below it. Straying off the midline will
often lead to damage to the rectus muscles, particularly below the arcuate line where they may often
be in close proximity to each other.
Next question
Question 89 of 560

A 23 year old man is undergoing an inguinal hernia repair. The surgeons mobilise the spermatic cord
and place it in a hernia ring. A small slender nerve is identified superior to the cord. Which nerve is it
most likely to be?

Iliohypogastric nerve

Pudendal nerve

Femoral branch of the genitofemoral nerve

Ilioinguinal nerve

Obturator nerve

Theme from April 2014 exam


The ilioinguinal nerve passes through the inguinal canal and is the nerve most commonly identified
during hernia surgery. The genitofemoral nerve splits into two branches, the genital branch passes
through the inguinal canal within the cord structures. The femoral branch of the genitofemoral nerve
enters the thigh posterior to the inguinal ligament, lateral to the femoral artery. The iliohypogastric
nerve pierces the external oblique aponeurosis above the superficial inguinal ring.
Please rate this question:

Discuss and give feedback


Next question

Ilioinguinal nerve

Arises from the first lumbar ventral ramus with the iliohypogastric nerve. It passes inferolaterally
through the substance of psoas major and over the anterior surface of quadratus lumborum. It
pierces the internal oblique muscle and passes deep to the aponeurosis of the external oblique
muscle. It enters the inguinal canal and then passes through the superficial inguinal ring to reach the
skin.

Branches

 To supply those muscles of the abdominal wall through which it passes.


 Skin and fascia over the pubic symphysis, superomedial part of the femoral triangle, surface
of the scrotum, root and dorsum of penis or labum majus in females.
Question 90 of 560

A 34 year old man undergoes excision of a sarcoma from the right buttock. During the procedure the
sciatic nerve is sacrificed. Which of the following will not occur as a result of this process?

Loss of extension at the knee joint

Foot drop

Inability to extend extensor hallucis longus

Loss of sensation to the posterior aspect of the thigh

Loss of sensation to the posterior aspect of the lower leg

Extension of the knee joint is caused by the obturator and femoral nerves.
Please rate this question:

Discuss and give feedback


Next question

Sciatic nerve

The sciatic nerve is formed from the sacral plexus and is the largest nerve in the body. It is the
continuation of the main part of the plexus arising from ventral rami of L4 to S3. These rami
converge at the inferior border of piriformis to form the nerve itself. It passes through the inferior part
of the greater sciatic foramen and emerges beneath piriformis. Medially, lie the inferior gluteal nerve
and vessels and the pudendal nerve and vessels. It runs inferolaterally under the cover of gluteus
maximus midway between the greater trochanter and ischial tuberosity. It receives its blood supply
from the inferior gluteal artery. The nerve provides cutaneous sensation to the skin of the foot and
the leg. It also innervates the posterior thigh muscles and the lower leg and foot muscles. The nerve
splits into the tibial and common peroneal nerves approximately half way down the posterior thigh.
The tibial nerve supplies the flexor muscles and the common peroneal nerve supplies the extensor
muscles and the abductor muscles.

Summary points
Origin Spinal nerves L4 - S3
Articular Branches Hip joint

Muscular branches in  Semitendinosus


upper leg  Semimembranosus
 Biceps femoris
 Part of adductor magnus

Cutaneous sensation  Posterior aspect of thigh (via cutaneous nerves)


 Gluteal region
 Entire lower leg (except the medial aspect)

Terminates At the upper part of the popliteal fossa by dividing into the tibial and
peroneal nerves

 The nerve to the short head of the biceps femoris comes from the common peroneal part of
the sciatic and the other muscular branches arise from the tibial portion.
 The tibial nerve goes on to innervate all muscles of the foot except the extensor digitorum
brevis (which is innervated by the common peroneal nerve).

Next question
Question 91 of 560

Where does the spinal cord terminate in neonates?

L1

L2

L3

L4

L5

Theme from 2009 Exam


Theme from January 2013 Exam
At the 3rd month the foetus's spinal cord occupies the entire length of the vertebral canal. The
vertebral column then grows longer exceeding the growth rate of the spinal cord. This results with
the cord being at L3 at birth and L1-2 by adulthood.
Please rate this question:

Discuss and give feedback


Next question

Spinal cord

 Located in a canal within the vertebral column that affords it structural support.
 Rostrally it continues to the medulla oblongata of the brain and caudally it tapers at a level
corresponding to the L1-2 interspace (in the adult), a central structure, the filum terminale
anchors the cord to the first coccygeal vertebra.
 The spinal cord is characterised by cervico-lumbar enlargements and these, broadly
speaking, are the sites which correspond to the brachial and lumbar plexuses respectively.

There are some key points to note when considering the surgical anatomy of the spinal cord:

* During foetal growth the spinal cord becomes shorter than the spinal canal, hence the adult site of
cord termination at the L1-2 level.

* Due to growth of the vertebral column the spine segmental levels may not always correspond to
bony landmarks as they do in the cervical spine.
* The spinal cord is incompletely divided into two symmetrical halves by a dorsal median
sulcus andventral median fissure. Grey matter surrounds a central canal that is continuous
rostrally with the ventricular system of the CNS.

* The grey matter is sub divided cytoarchitecturally into Rexeds laminae.

* Afferent fibres entering through the dorsal roots usually terminate near their point of entry but may
travel for varying distances in Lissauers tract. In this way they may establish synaptic connections
over several levels

* At the tip of the dorsal horn are afferents associated with nociceptive stimuli. The ventral horn
contains neurones that innervate skeletal muscle.

The key point to remember when revising CNS anatomy is to keep a clinical perspective in mind. So
it is worth classifying the ways in which the spinal cord may become injured. These include:

 Trauma either direct or as a result of disc protrusion


 Neoplasia either by direct invasion (rare) or as a result of pathological vertebral fracture
 Inflammatory diseases such as Rheumatoid disease, or OA (formation of osteophytes
compressing nerve roots etc.
 Vascular either as a result of stroke (rare in cord) or as complication of aortic dissection
 Infection historically diseases such as TB, epidural abscesses.

The anatomy of the cord will, to an extent dictate the clinical presentation. Some points/ conditions to
remember:

 Brown- Sequard syndrome-Hemisection of the cord producing ipsilateral loss of


proprioception and upper motor neurone signs, plus contralateral loss of pain and
temperature sensation. The explanation of this is that the fibres decussate at different levels.
 Lesions below L1 will tend to present with lower motor neurone signs

Next question
Question 92 of 560

A 45 year old man is undergoing a low anterior resection for a carcinoma of the rectum. Which of the
following fascial structures will need to be divided to mobilise the mesorectum from the sacrum and
coccyx?

Denonvilliers fascia

Colles fascia

Sibsons fascia

Waldeyers fascia

None of the above

Fascial layers surrounding the rectum:

 Anteriorly lies the fascia of Denonvilliers


 Posteriorly lies Waldeyers fascia

Waldeyers fascia separates the mesorectum from the sacrum and will need to be divided.
Please rate this question:

Discuss and give feedback


Next question

Rectum

The rectum is approximately 12 cm long. It is a capacitance organ. It has both intra and
extraperitoneal components. The transition between the sigmoid colon is marked by the
disappearance of the tenia coli.The extra peritoneal rectum is surrounded by mesorectal fat that also
contains lymph nodes. This mesorectal fatty layer is removed surgically during rectal cancer surgery
(Total Mesorectal Excision). The fascial layers that surround the rectum are important clinical
landmarks, anteriorly lies the fascia of Denonvilliers. Posteriorly lies Waldeyers fascia.

Extra peritoneal rectum

 Posterior upper third


 Posterior and lateral middle third
 Whole lower third

Relations
Anteriorly (Males) Rectovesical pouch
Bladder
Prostate
Seminal vesicles

Anteriorly (Females) Recto-uterine pouch (Douglas)


Cervix
Vaginal wall

Posteriorly Sacrum
Coccyx
Middle sacral artery

Laterally Levator ani


Coccygeus

Arterial supply
Superior rectal artery

Venous drainage
Superior rectal vein

Lymphatic drainage

 Mesorectal lymph nodes (superior to dentate line)


 Internal iliac and then para-aortic nodes
 Inguinal nodes (inferior to dentate line)

Next question
Question 93 of 560

What is the nerve supply to the muscle indicated by an X on the prosection below.

© Image provided by the University of Sheffield

CN XI

CN IX

Median pectoral nerve

Lateral pectoral nerve


Suprascapular nerve

Be careful not to confuse CN IX with CN XI

The muscle indicated is trapezius and it is innervated by the accessory nerve (CN XI)

Please rate this question:

Discuss and give feedback

Next question

Trapezius

Origin Medial third of the superior nuchal line of the occiput


External occipital protruberance
Ligamentum nuchae
Spines of C7 and all thoracic vertebrae and all intervening interspinous ligaments

Insertion Posterior border of the lateral third of the clavicle


Medial border of the acromion
Upper border of the crest of the spine of the scapula

Nerve supply Spinal portion of the accessory nerve

Actions Elevation of the shoulder girdle


Lateral rotation of the scapula

Next question

v
Question 94 of 560

A 10 year old child has a grommet inserted for a glue ear. What type of epithelium is present on the
external aspect of the tympanic membrane?

Stratified squamous

Ciliated columnar

Non ciliated columnar

Non stratified squamous

None of the above

The external aspect of the tympanic membrane is lined by stratified squamous epithelium. This is
significant clinically in the development of middle ear infections when this type of epithelium may
migrate inside the middle ear.
Please rate this question:

Discuss and give feedback


Next question

Ear- anatomy

The ear is composed of three anatomically distinct regions.

External ear
Auricle is composed of elastic cartilage covered by skin. The lobule has no cartilage and contains fat
and fibrous tissue.

External auditory meatus is approximately 2.5cm long.


Lateral third of the external auditory meatus is cartilaginous and the medial two thirds is bony.

The region is innervated by the greater auricular nerve. The auriculotemporal branch of the
trigeminal nerve supplies most the of external auditory meatus and the lateral surface of the auricle.

Middle ear
Space between the tympanic membrane and cochlea. The aditus leads to the mastoid air cells is the
route through which middle ear infections may cause mastoiditis. Anteriorly the eustacian tube
connects the middle ear to the naso pharynx.
The tympanic membrane consists of:

 Outer layer of stratified squamous epithelium.


 Middle layer of fibrous tissue.
 Inner layer of mucous membrane continuous with the middle ear.

The tympanic membrane is approximately 1cm in diameter.


The chorda tympani nerve passes on the medial side of the pars flaccida.

The middle ear is innervated by the glossopharyngeal nerve and pain may radiate to the middle ear
following tonsillectomy.

Ossicles
Malleus attaches to the tympanic membrane (the Umbo).
Malleus articulates with the incus (synovial joint).
Incus attaches to stapes (another synovial joint).

Internal ear
Cochlea, semi circular canals and vestibule

Organ of corti is the sense organ of hearing and is located on the inside of the cochlear duct on the
basilar membrane.

Vestibule accommodates the utricule and the saccule. These structures contain endolymph and are
surrounded by perilymph within the vestibule.

The semicircular canals lie at various angles to the petrous temporal bone. All share a common
opening into the vestibule.
Next question
Question 95 of 560

A 73 year old lady is admitted with acute mesenteric ischaemia. A CT angiogram is performed and a
stenotic lesion is noted at the origin of the superior mesenteric artery. At which of the following levels
does this branch from the aorta?

L1

L2

L3

L4

L5

Theme from January 2012 Exam


Theme from April 2014 exam
The SMA leaves the aorta at L1. It passes under the neck of the pancreas prior to giving its first
branch the inferior pancreatico-duodenal artery.
Please rate this question:

Discuss and give feedback


Next question

Superior mesenteric artery

 Branches off aorta at L1


 Supplies small bowel from duodenum (distal to ampulla of vater) through to mid transverse
colon
 Takes more oblique angle from aorta and thus more likely to recieve emboli than coeliac axis

Relations of superior mesenteric artery


Superiorly Neck of pancreas

Third part of duodenum


Postero-inferiorly
Uncinate process

Posteriorly Left renal vein


Right Superior mesenteric vein

Branches of the superior mesenteric artery

 Inferior pancreatico-duodenal artery


 Jejunal and ileal arcades
 Ileo-colic artery
 Right colic artery
 Middle colic artery

Overview of SMA and branches

Image sourced from Wikipedia

Next question
Question 96 of 560

The following statements relating to the musculocutaneous nerve are true except?

It arises from the lateral cord of the brachial plexus

It provides cutaneous innervation to the lateral side of the forearm

If damaged, then extension of the elbow joint will be impaired

It supplies the biceps muscle

It runs beneath biceps

It supplies biceps, brachialis and coracobrachialis. If damaged then elbow flexion will be impaired.
Please rate this question:

Discuss and give feedback


Next question

Musculocutaneous nerve

 Branch of lateral cord of brachial plexus

Path

 It penetrates the coracobrachialis muscle


 Passes obliquely between the biceps brachii and the brachialis to the lateral side of the arm
 Above the elbow it pierces the deep fascia lateral to the tendon of the biceps brachii
 Continues into the forearm as the lateral cutaneous nerve of the forearm

Innervates

 Coracobrachialis
 Biceps brachii
 Brachialis
 Question 97 of 560

 Which of the following structures does not pass through the foramen ovale?

Lesser petrosal nerve

Accessory meningeal artery

Maxillary nerve

Emissary veins

Otic ganglion

 Mnemonic: OVALE

O tic ganglion
V3 (Mandibular nerve:3rd branch of trigeminal)
A ccessory meningeal artery
L esser petrosal nerve
E missary veins

 Please rate this question:


 Discuss and give feedback

 Next question

 Foramina of the base of the skull

Foramen Location Contents

Foramen ovale Sphenoid Otic ganglion


bone V3 (Mandibular nerve:3rd branch of
trigeminal)
Accessory meningeal artery
Foramen Location Contents

Lesser petrosal nerve


Emissary veins

Foramen spinosum Sphenoid Middle meningeal artery


bone Meningeal branch of the Mandibular nerve

Foramen rotundum Sphenoid Maxillary nerve (V2)


bone

Foramen lacerum/ Sphenoid Base of the medial pterygoid plate.


carotid canal bone Internal carotid artery*
Nerve and artery of the pterygoid canal

Jugular foramen Temporal Anterior: inferior petrosal sinus


bone Intermediate: glossopharyngeal, vagus, and accessory nerves.
Posterior: sigmoid sinus (becoming the internal jugular vein) and
some meningeal branches from the occipital and ascending
pharyngeal arteries.

Foramen magnum Occipital Anterior and posterior spinal arteries


bone Vertebral arteries
Medulla oblongata

Stylomastoid Temporal Stylomastoid artery


foramen bone Facial nerve

Superior orbital Sphenoid Oculomotor nerve (III)


fissure bone Recurrent meningeal artery
Trochlear nerve (IV)
Lacrimal, frontal and nasociliary branches of ophthalmic nerve (V1)
Abducent nerve (VI)
Foramen Location Contents

Superior ophthalmic vein


*= In life the foramen lacerum is occluded by a cartilagenous plug. The ICA initially passes
into the carotid canal which ascends superomedially to enter the cranial cavity through the
foramen lacerum.

Base of skull anatomical overview

Image sourced from Wikipedia

 Next question

Question 98 of 560

Which of the cranial nerves listed below is least likely to carry parasympathetic fibres?

III

VII

IX

II

Cranial nerves carrying parasympathetic fibres


X IX VII III (1973)

The parasympathetic functions served by the cranial nerves include:

III (oculomotor) Pupillary constriction and accommodation

VII (facial) Lacrimal gland, submandibular and sublingual glands

IX (glossopharyngeal) Parotid

X (vagus) Heart and abdominal viscera

The optic nerve carries no parasympathetic fibres.

The cranial preganglionic parasympathetic nerves arise from specific nuclei in the CNS. These
synapse at one of four parasympathetic ganglia; otic, pterygopalatine, ciliary and submandibular.
From these ganglia the parasympathetic nerves complete their journey to their target tissues via CN
V (trigeminal) branches (ophthalmic nerve CNV branch 1, Maxillary nerve CN V branch2, mandibular
nerve CN V branch 3)

Please rate this question:


Discuss and give feedback

Next question

Cranial nerves

Cranial nerve lesions

Olfactory nerve May be injured in basal skull fractures or involved in frontal lobe tumour extension.
Loss of olfactory nerve function in relation to major CNS pathology is seldom an
isolated event and thus it is poor localiser of CNS pathology.

Optic nerve Problems with visual acuity may result from intra ocular disorders. Problems with
the blood supply such as amaurosis fugax may produce temporary visual distortion.
More important surgically is the pupillary response to light. The pupillary size may
be altered in a number of disorders. Nerves involved in the resizing of the pupil
connect to the pretectal nucleus of the high midbrain, bypassing the lateral
geniculate nucleus and the primary visual cortex. From the pretectal nucleus
neurones pass to the Edinger - Westphal nucleus, motor axons from here pass along
with the oculomotor nerve. They synapse with ciliary ganglion neurones; the
parasympathetic axons from this then innervate the iris and produce miosis. The
miotic pupil is seen in disorders such as Horner's syndrome or opiate overdose.
Mydriasis is the dilatation of the pupil in response to disease, trauma, drugs (or the
dark!). It is pathological when light fails to induce miosis. The radial muscle is
innervated by the sympathetic nervous system. Because the parasympathetic fibres
travel with the oculomotor nerve they will be damaged by lesions affecting this
nerve (e.g. cranial trauma).
The response to light shone in one eye is usually a constriction of both pupils. This
indicates intact direct and consensual light reflexes. When the optic nerve has an
afferent defect the light shining on the affected eye will produce a diminished
pupillary response in both eyes. Whereas light shone on the unaffected eye will
produce a normal pupillary response in both eyes. This is referred to as the Marcus
Gunn pupil and is seen in conditions such as optic neuritis. In a total CN II lesion
shining the light in the affected eye will produce no response.

Oculomotor nerve The pupillary effects are described above. In addition it supplies all ocular muscles
apart from lateral rectus and superior oblique. Thus the affected eye will be
deviated inferolaterally. Levator palpebrae superioris may also be impaired resulting
in impaired ability to open the eye.

Trochlear nerve The eye will not be able to look down.

Trigeminal nerve Largest cranial nerve. Exits the brainstem at the pons. Branches are ophthalmic,
maxillary and mandibular. Only the mandibular branch has both sensory and motor
fibres. Branches converge to form the trigeminal ganglion (located in Meckels cave).
It supplies the muscles of mastication and also tensor veli palatine, mylohyoid,
anterior belly of digastric and tensor tympani. The detailed descriptions of the
various sensory functions are described in other areas of the website. The corneal
reflex is important and is elicited by applying a small tip of cotton wool to the
cornea, a reflex blink should occur if it is intact. It is mediated by: the naso ciliary
branch of the ophthalmic branch of the trigeminal (sensory component) and the
facial nerve producing the motor response. Lesions of the afferent arc will produce
bilateral absent blink and lesions of the efferent arc will result in a unilateral absent
blink.

Abducens nerve The affected eye will have a deficit of abduction. This cranial nerve exits the
brainstem between the pons and medulla. It thus has a relatively long intra cranial
course which renders it susceptible to damage in raised intra cranial pressure.

Facial nerve Emerges from brainstem between pons and medulla. It controls muscles of facial
expression and taste from the anterior 2/3 of the tongue. The nerve passes into the
petrous temporal bone and into the internal auditory meatus. It then passes
through the facial canal and exits at the stylomastoid foramen. It passes through the
parotid gland and divides at this point. It does not innervate the parotid gland. Its
divisions are considered in other parts of the website. Its motor fibres innervate
orbicularis oculi to produce the efferent arm of the corneal reflex. In surgical
practice it may be injured during parotid gland surgery or invaded by malignancies
of the gland and a lower motor neurone on the ipsilateral side will result.

Vestibulo-cochlear Exits from the pons and then passes through the internal auditory meatus. It is
nerve implicated in sensorineural hearing loss. Individuals with sensorineural hearing loss
will localise the sound in webers test to the normal ear. Rinnes test will be reduced
on the affected side but should still work. These two tests will distinguish
sensorineural hearing loss from conductive deafness. In the latter condition webers
test will localise to the affected ear and Rinnes test will be impaired on the affected
side. Surgical lesions affecting this nerve include CNS tumours and basal skull
fractures. It may also be damaged by the administration of ototoxic drugs (of which
gentamicin is the most commonly used in surgical practice).

Glossopharyngeal Exits the pons just above the vagus. Receives sensory fibres from posterior 1/3
nerve tongue, tonsils, pharynx and middle ear (otalgia may occur following tonsillectomy).
It receives visceral afferents from the carotid bodies. It supplies parasympathetic
fibres to the parotid gland via the otic ganglion and motor function to
stylopharyngeaus muscle. The sensory function of the nerve is tested using the gag
reflex.

Vagus nerve Leaves the medulla between the olivary nucleus and the inferior cerebellar
peduncle. Passes through the jugular foramen and into the carotid sheath. Details of
the functions of the vagus nerve are covered in the website under relevant organ
sub headings.

Accessory nerve Exists from the caudal aspect of the brainstem (multiple branches) supplies
trapezius and sternocleidomastoid muscles. The distal portion of this nerve is most
prone to injury during surgical procedures.

Hypoglossal nerve Emerges from the medulla at the preolivary sulcus, passes through the hypoglossal
canal. It lies on the carotid sheath and passes deep to the posterior belly of digastric
to supply muscles of the tongue (except palatoglossus). Its location near the carotid
sheath makes it vulnerable during carotid endarterectomy surgery and damage will
produce ipsilateral defect in muscle function.

Next question
Question 99 of 560

A 72 year old man is undergoing an open abdominal aortic aneurysm repair. The aneurysm is
located in a juxtarenal location and surgical access to the neck of aneurysm is difficult. Which of the
following structures may be divided to improve access?

Cisterna chyli

Transverse colon

Left renal vein

Superior mesenteric artery

Coeliac axis

The left renal vein will be stretched over the neck of the anuerysm in this location and is not
infrequently divided. This adds to the nephrotoxic insult of juxtarenal aortic surgery as a supra renal
clamp is also often applied. Deliberate division of the Cisterna Chyli will not improve access and will
result in a chyle leak. Division of the transverse colon will not help at all and would result in a high
risk of graft infection. Division of the SMA is pointless for a juxtarenal procedure.

Please rate this question:

Discuss and give feedback

Next question

Abdominal aorta

Abdominal aortic topography


Origin T12

Termination L4

Posterior relations L1-L4 Vertebral bodies

Anterior relations Lesser omentum


Liver
Left renal vein
Inferior mesenteric vein
Third part of duodenum
Pancreas
Parietal peritoneum
Peritoneal cavity

Right lateral relations Right crus of the diaphragm


Cisterna chyli
Azygos vein
IVC (becomes posterior distally)

Left lateral relations 4th part of duodenum


Duodenal-jejunal flexure
Left sympathetic trunk

The abdominal aorta


Image sourced from Wikipedia

Next question
Question 100 of 560

An occlusion of the anterior cerebral artery may compromise the blood supply to the following
structures except:

Medial inferior surface of the frontal lobe

Corpus callosum

Medial surface of the frontal lobe

Olfactory bulb

Brocas area

Brocas area is usually supplied by branches from the middle cerebral artery.
Please rate this question:

Discuss and give feedback


Next question

Circle of Willis

The two internal carotid arteries and two vertebral arteries form an anastomosis known as the Circle
of Willis on the inferior surface of the brain. Each half of the circle is formed by:
1. Anterior communicating artery
2. Anterior cerebral artery
3. Internal carotid artery
4. Posterior communicating artery
5. Posterior cerebral arteries and the termination of the basilar artery

The circle and its branches supply; the corpus striatum, internal capsule, diencephalon and
midbrain.
Image sourced from Wikipedia

Vertebral arteries

 Enter the cranial cavity via foramen magnum


 Lie in the subarachnoid space
 Ascend on anterior surface of medulla oblongata
 Unite to form the basilar artery at the base of the pons

Branches:

 Posterior spinal artery


 Anterior spinal artery
 Posterior inferior cerebellar artery

Basilar artery
Branches:
 Anterior inferior cerebellar artery
 Labyrinthine artery
 Pontine arteries
 Superior cerebellar artery
 Posterior cerebral artery

Internal carotid arteries


Branches:

 Posterior communicating artery


 Anterior cerebral artery
 Middle cerebral artery
 Anterior choroid artery

Next question
Question 101 of 560

Parasympathetic fibres innervating the parotid gland originate from which of the following?

Submandibular ganglion

Otic ganglion

Ciliary ganglion

Pterygopalatine ganglion

None of the above

Theme from April 2014


Secretion of saliva by the parotid gland is controlled by postsynaptic parasympathetic fibres
originating in the inferior salivatory nucleus; these leave the brain via the tympanic nerve (branch of
glossopharyngeal nerve (CN IX), travel through the tympanic plexus (located in the middle ear), and
then form the lesser petrosal nerve until reaching the otic ganglion. After synapsing in the Otic
ganglion, the postganglionic (postsynaptic) fibres travel as part of the auriculotemporal nerve (a
branch of the mandibular nerve (V3) to reach the parotid gland.
Please rate this question:

Discuss and give feedback


Next question

Parotid gland

Anatomy of the parotid gland


Location Overlying the mandibular ramus; anterior and inferior to the ear.

Salivary duct Crosses the masseter, pierces the buccinator and drains adjacent to the
2nd upper molar tooth (Stensen's duct).

Structures passing  Facial nerve (Mnemonic: The Zebra Buggered My Cat; Temporal
through the gland Zygomatic, Buccal, Mandibular, Cervical)
 External carotid artery
 Retromandibular vein
 Auriculotemporal nerve

Relations  Anterior: masseter, medial pterygoid, superficial temporal and


maxillary artery, facial nerve, stylomandibular ligament
 Posterior: posterior belly digastric muscle, sternocleidomastoid,
stylohyoid, internal carotid artery, mastoid process, styloid
process

Arterial supply Branches of external carotid artery

Venous drainage Retromandibular vein

Lymphatic drainage Deep cervical nodes

Nerve innervation  Parasympathetic-Secretomotor


 Sympathetic-Superior cervical ganglion
 Sensory- Greater auricular nerve

Parasympathetic stimulation produces a water rich, serous saliva. Sympathetic stimulation leads to
the production of a low volume, enzyme-rich saliva.
Next question
Question 102 of 560

Following an oesophagogastrectomy the surgeons will anastomose the oesophageal remnant to the
stomach, which of the following is not part of the layers that comprise the oesophageal wall?

Serosa

Adventitia

Muscularis propria

Submucosa

Mucosa

The oesophageal wall lacks the serosa layer

Theme from April 2010 exam


The wall lacks a serosa which can make the wall hold sutures less securely.
Please rate this question:

Discuss and give feedback


Next question

Oesophagus

 25cm long
 Starts at C6 vertebra, pierces diaphragm at T10 and ends at T11
 Squamous epithelium

Constrictions of the oesophagus


Structure Distance from incisors

Cricoid cartilage 15cm


Arch of the Aorta 22.5cm

Left principal bronchus 27cm

Diaphragmatic hiatus 40cm

Relations
Anteriorly  Trachea to T4
 Recurrent laryngeal nerve
 Left bronchus, Left atrium
 Diaphragm

Posteriorly  Thoracic duct to left at T5


 Hemiazygos to the left T8
 Descending aorta
 First 2 intercostal branches of aorta

Left  Thoracic duct


 Left subclavian artery

Right  Azygos vein

Arterial, venous and lymphatic drainage of the oesophagus


Artery Vein Lymphatics Muscularis externa

Upper Inferior Inferior thyroid Deep Striated muscle


third thyroid cervical

Mid third Aortic Azygos branches Mediastinal Smooth & striated


branches muscle

Lower Left gastric Posterior mediastinal and Gastric Smooth muscle


third coeliac
Nerve supply

 Upper half is supplied by recurrent laryngeal nerve


 Lower half by oesophageal plexus (vagus)

Histology

 Mucosa :Non-keratinized stratified squamous epithelium


 Submucosa: glandular tissue
 Muscularis externa (muscularis): composition varies. See table
 Adventitia

Next question
Question 103 of 560

Which of the following structures suspends the spinal cord in the dural sheath?

Filum terminale

Conus medullaris

Ligamentum flavum

Denticulate ligaments

Anterior longitudinal ligament

The spinal cord is approximately 45cm in men and 43cm in women. The denticulate ligament is a
continuation of the pia mater (innermost covering of the spinal cord) which has intermittent lateral
projections attaching the spinal cord to the dura mater.
Please rate this question:

Discuss and give feedback


Next question

Spinal cord

 Located in a canal within the vertebral column that affords it structural support.
 Rostrally it continues to the medulla oblongata of the brain and caudally it tapers at a level
corresponding to the L1-2 interspace (in the adult), a central structure, the filum terminale
anchors the cord to the first coccygeal vertebra.
 The spinal cord is characterised by cervico-lumbar enlargements and these, broadly
speaking, are the sites which correspond to the brachial and lumbar plexuses respectively.

There are some key points to note when considering the surgical anatomy of the spinal cord:

* During foetal growth the spinal cord becomes shorter than the spinal canal, hence the adult site of
cord termination at the L1-2 level.

* Due to growth of the vertebral column the spine segmental levels may not always correspond to
bony landmarks as they do in the cervical spine.

* The spinal cord is incompletely divided into two symmetrical halves by a dorsal median
sulcus andventral median fissure. Grey matter surrounds a central canal that is continuous
rostrally with the ventricular system of the CNS.

* The grey matter is sub divided cytoarchitecturally into Rexeds laminae.

* Afferent fibres entering through the dorsal roots usually terminate near their point of entry but may
travel for varying distances in Lissauers tract. In this way they may establish synaptic connections
over several levels

* At the tip of the dorsal horn are afferents associated with nociceptive stimuli. The ventral horn
contains neurones that innervate skeletal muscle.

The key point to remember when revising CNS anatomy is to keep a clinical perspective in mind. So
it is worth classifying the ways in which the spinal cord may become injured. These include:

 Trauma either direct or as a result of disc protrusion


 Neoplasia either by direct invasion (rare) or as a result of pathological vertebral fracture
 Inflammatory diseases such as Rheumatoid disease, or OA (formation of osteophytes
compressing nerve roots etc.
 Vascular either as a result of stroke (rare in cord) or as complication of aortic dissection
 Infection historically diseases such as TB, epidural abscesses.

The anatomy of the cord will, to an extent dictate the clinical presentation. Some points/ conditions to
remember:

 Brown- Sequard syndrome-Hemisection of the cord producing ipsilateral loss of


proprioception and upper motor neurone signs, plus contralateral loss of pain and
temperature sensation. The explanation of this is that the fibres decussate at different levels.
 Lesions below L1 will tend to present with lower motor neurone signs

Next question
Question 104 of 560

Where is the 'safe triangle' for chest drain insertion located?

4th intercostal space, mid axillary line

5th intercostal space, mid axillary line

4th intercostal space, mid scapular line

5th intercostal space, mid scapular line

4th intercostal space, mid clavicular line

'Safe Triangle' for chest drain insertion:

5th intercostal space, mid axillary line

Theme from April 2012 exam

Please rate this question:

Discuss and give feedback

Next question

Chest drains

There are a number of different indications for chest drain insertion. In general terms large bore
chest drains are preferred for trauma and haemothorax drainage. Smaller diameter chest drains can
be used for pneumothorax or pleural effusion drainage.

Insertion can be performed either using anatomical guidance or through ultrasound guidance. In the
exam, the anatomical method is usually tested.
It is advised that chest drains are placed in the 'safe triangle'. The triangle is located in the mid
axillary line of the 5th intercostal space. It is bordered by:
Anterior edge latissimus dorsi, the lateral border of pectoralis major, a line superior to the horizontal
level of the nipple, and the apex below the axilla.

Another triangle is situated behind the scapula. It is bounded above by the trapezius, below by the
latissimus dorsi, and laterally by the vertebral border of the scapula; the floor is partly formed by the
rhomboid major. If the scapula is drawn forward by folding the arms across the chest, and the trunk
bent forward, parts of the sixth and seventh ribs and the interspace between them become
subcutaneous and available for auscultation. The space is therefore known as the triangle of
auscultation.

References
Prof Harold Ellis. The applied anatomy of chest drains insertions. British Journal of hospital medicine
2007; (68): 44-45.

Laws D, Neville E, Duffy J. BTS guidelines for insertion of chest drains. Thorax, 2003; (58): 53-59.

Next question
Question 105 of 560

Your consultant decides to perform an open inguinal hernia repair under local anaesthesia. Which of
the following dermatomal levels will require blockade?

T10

T12

T11

S1

S2

Theme from April 2012 Exam

Please rate this question:

Discuss and give feedback

Next question

Dermatomes

The common dermatomal levels and cutaneous nerves responsible for them is illustrated below.
Image sourced from Wikipedia

Next question
Question 106 of 560

A 44 year old man is undergoing a parotidectomy and the surgeon is carefully preserving the facial
nerve. Unfortunately his trainee then proceeds to divide it. Which of the following will not be affected
as a result?

Taste sensation from anterior two thirds of the tongue

Closing the ipsilateral eyelid

Raising the ipsilateral side of the lip

Ipsilateral corneal reflex

None of the above

Theme from April 2014 exam


The chorda tympani branches inside the facial canal and will therefore be unaffected by this most
unfortunate event! The corneal reflex is mediated by the opthalmic branch of the trigeminal nerve
sensing the stimulus on the cornea, lid or conjunctiva; the facial nerve initiates the motor response of
the reflex.
Please rate this question:

Discuss and give feedback


Next question

Facial nerve

The facial nerve is the main nerve supplying the structures of the second embryonic branchial arch.
It is predominantly an efferent nerve to the muscles of facial expression, digastric muscle and also to
many glandular structures. It contains a few afferent fibres which originate in the cells of its genicular
ganglion and are concerned with taste.

Supply - 'face, ear, taste, tear'

 Face: muscles of facial expression


 Ear: nerve to stapedius
 Taste: supplies anterior two-thirds of tongue
 Tear: parasympathetic fibres to lacrimal glands, also salivary glands
Path
Subarachnoid path

 Origin: motor- pons, sensory- nervus intermedius


 Pass through the petrous temporal bone into the internal auditory meatus with the
vestibulocochlear nerve. Here they combine to become the facial nerve.

Facial canal path

 The canal passes superior to the vestibule of the inner ear


 At the medial aspect of the middle ear, it becomes wider and contains the geniculate
ganglion.

- 3 branches:
1. greater petrosal nerve
2. nerve to stapedius
3. chorda tympani

Stylomastoid foramen

 Passes through the stylomastoid foramen (tympanic cavity anterior and mastoid antrum
posteriorly)
 Posterior auricular nerve and branch to posterior belly of digastric and stylohyoid muscle

Face
Enters parotid gland and divides into 5 branches:

 Temporal branch
 Zygomatic branch
 Buccal branch
 Marginal mandibular branch
 Cervical branch

Next question
Question 107 of 560

A 45 year old lady develops severe back pain and on examination is found to have clinical evidence
of an L5/ S1 radiculopathy. Her symptoms deteriorate and eventually a laminectomy is performed.
During a posterior surgical approach the surgeons encounter a tough ligamentous structure lying
anterior to the spinous processes. This structure is most likely to be the

Transverse spinal ligament

Supraspinal ligament

Anterior longitudinal ligament

Ligamentum flavum

Posterior longitudinal ligament

The ligamentum lies in this position, as illustrated below:


Image sourced from Wikipedia

Please rate this question:

Discuss and give feedback

Next question

Vertebral column

 There are 7 cervical, 12 thoracic, 5 lumbar, and 5 sacral vertebrae.


 The spinal cord segmental levels do not necessarily correspond to the vertebral segments.
For example, while the C1 cord is located at the C1 vertebra, the C8 cord is situated at the
C7 vertebra. While the T1 cord is situated at the T1 vertebra, the T12 cord is situated at the
T8 vertebra. The lumbar cord is situated between T9 and T11 vertebrae. The sacral cord is
situated between the T12 to L2 vertebrae.

Cervical vertebrae
The interface between the first and second vertebra is called the atlanto-axis junction. The C3 cord
contains the phrenic nucleus.
Muscle Nerve root value
Muscle Nerve root value

Deltoid C5,6

Biceps C5,6

Wrist extensors C6-8

Triceps C6-8

Wrist flexors C6-T1

Hand muscles C8-T1

Thoracic vertebrae
The thoracic vertebral segments are defined by those that have a rib. The spinal roots form the
intercostal nerves that run on the bottom side of the ribs and these nerves control the intercostal
muscles and associated dermatomes.

Lumbosacral vertebrae
Form the remainder of the segments below the vertebrae of the thorax. The lumbosacral spinal cord,
however, starts at about T9 and continues only to L2. It contains most of the segments that innervate
the hip and legs, as well as the buttocks and anal regions.

Cauda Equina
The spinal cord ends at L1-L2 vertebral level. The tip of the spinal cord is called the conus. Below
the conus, there is a spray of spinal roots that is called the cauda equina. Injuries below L2
represent injuries to spinal roots rather than the spinal cord proper.

Next question
Question 108 of 560

Which of the following does not pass through the superior orbital fissure?

Lacrimal nerve

Abducens nerve

Opthalmic artery

Trochlear nerve

Superior opthalmic vein

Mnemonic for the nerves passing through the supraorbital fissure:

Live Frankly To See Absolutely No Insult

Lacrimal
Frontal
Trochlear
Superior Division of Oculomotor
Abducens
Nasociliary
Inferior Division of Oculomotor nerve

Theme from September 2015 Exam


Theme from April 2014 exam
The opthalmic artery arises from the internal carotid immediately after it has pierced the dura and
arachnoid. It runs through the optic canal below the optic nerve and within its dural and arachnoid
sheaths. It terminates as the supratrochlear and dorsal nasal arteries.
Please rate this question:

Discuss and give feedback


Next question

Foramina of the base of the skull

Foramen Location Contents


Foramen Location Contents

Foramen ovale Sphenoid Otic ganglion


bone V3 (Mandibular nerve:3rd branch of
trigeminal)
Accessory meningeal artery
Lesser petrosal nerve
Emissary veins

Foramen spinosum Sphenoid Middle meningeal artery


bone Meningeal branch of the Mandibular nerve

Foramen rotundum Sphenoid Maxillary nerve (V2)


bone

Foramen lacerum/ Sphenoid Base of the medial pterygoid plate.


carotid canal bone Internal carotid artery*
Nerve and artery of the pterygoid canal

Jugular foramen Temporal Anterior: inferior petrosal sinus


bone Intermediate: glossopharyngeal, vagus, and accessory nerves.
Posterior: sigmoid sinus (becoming the internal jugular vein)
and some meningeal branches from the occipital and
ascending pharyngeal arteries.

Foramen magnum Occipital Anterior and posterior spinal arteries


bone Vertebral arteries
Medulla oblongata

Stylomastoid Temporal Stylomastoid artery


foramen bone Facial nerve

Superior orbital Sphenoid Oculomotor nerve (III)


fissure bone Recurrent meningeal artery
Trochlear nerve (IV)
Lacrimal, frontal and nasociliary branches of ophthalmic
nerve (V1)
Foramen Location Contents

Abducent nerve (VI)


Superior ophthalmic vein

*= In life the foramen lacerum is occluded by a cartilagenous plug. The ICA initially passes into the
carotid canal which ascends superomedially to enter the cranial cavity through the foramen lacerum.

Base of skull anatomical overview

Image sourced from Wikipedia

Next question
Question 109 of 560

An 18 year old man undergoes a tonsillectomy for attacks of recurrent acute tonsillitis. Whilst in
recovery he develops a post operative haemorrhage. Which of the following vessels is the most
likely culprit?

Facial vein

External palatine vein

External carotid artery

Internal jugular vein

None of the above

The external palatine vein lies immediately lateral to the tonsil and if damaged may be a cause of
reactionary haemorrhage following tonsillectomy.

Please rate this question:

Discuss and give feedback

Next question

Tonsil

Anatomy

 Each palatine tonsil has two surfaces, a medial surface which projects into the pharynx and a
lateral surface that is embedded in the wall of the pharynx.
 They are usually 25mm tall by 15mm wide, although this varies according to age and may be
almost completely atrophied in the elderly.
 Their arterial supply is from the tonsillar artery, a branch of the facial artery.
 Its veins pierce the constrictor muscle to join the external palatine or facial veins. The
external palatine vein is immediately lateral to the tonsil, which may result in haemorrhage
during tonsillectomy.
 Lymphatic drainage is the jugulodigastric node and the deep cervical nodes.

Tonsillitis

 Usually bacterial (50%)- group A Streptococcus. Remainder viral.


 May be complicated by development of abscess (quinsy). This may distort the uvula.

- Indications for tonsillectomy include recurrent acute tonsillitis, suspected malignancy, enlargement
causing sleep apnoea.
- Dissection tonsillectomy is the preferred technique with haemorrhage being the commonest
complication. Delayed otalgia may occur owing to irritation of the glossopharyngeal nerve.
Next question
Question 110 of 560

A patient is found to have an ischaemic left colon. Which artery arising from the aorta at around the
level of L3 is most likely to account for this situation?

Superior mesenteric artery

Inferior mesenteric artery

Superior rectal artery

Ileocolic artery

Middle colic artery

Theme from January 2013 Exam


Theme from April 2014 exam
Only the IMA is likely to affect the left side of the colon and originate at L3.
Please rate this question:

Discuss and give feedback


Next question

Inferior mesenteric artery

The IMA is the main arterial supply of the embryonic hindgut and originates approximately 3-4 cm
superior to the aortic bifurcation. From its aortic origin it passes immediately inferiorly across the
anterior aspect of the aorta to eventually lie on its left hand side. At the level of the left common iliac
artery it becomes the superior rectal artery.

Branches
The left colic artery arises from the IMA near its origin. More distally up to three sigmoid arteries will
exit the IMA to supply the sigmoid colon.
Next question
Question 111 of 560

At which level does the aorta perforate the diaphragm?

T10

T9

T8

T11

T12

Memory aid:
T8 (8 letters) = vena cava
T10 (10 letters) = oesophagus
T12 (12 letters) = aortic hiatus

Theme from April 2012 exam


Theme from April 2014 exam
Please rate this question:

Discuss and give feedback


Next question

Diaphragm apertures

Diaphragm aperture levels

Vena cava T8
Oesophagus T10
Aortic hiatus T12
Next question
Question 112 of 560

A 24 year old lady is stabbed in the buttock. Following the injury the wound is sutured in the
emergency department. Eight weeks later she attends the clinic, as she walks into the clinic room
she has a waddling gait and difficulty with thigh abduction. On examination she has buttock muscle
wasting. Which nerve has been injured?

Superior gluteal nerve

Obturator nerve

Sciatic nerve

Femoral nerve

Inferior gluteal nerve

Theme from April 2016 Exam


Damage to the superior gluteal nerve will result in a Trendelenburg gait.
Please rate this question:

Discuss and give feedback


Next question

Trendelenburg test

Injury or division of the superior gluteal nerve results in a motor deficit that consists of weakened
abduction of the thigh by gluteus medius, a disabling gluteus medius limp and a compensatory list of
the body to the weakened gluteal side. The compensation results in a gravitational shift so that the
body is supported on the unaffected limb.

When a person is asked to stand on one leg, the gluteus medius usually contracts as soon as the
contralateral leg leaves the floor, preventing the pelvis from dipping towards the unsupported side.
When a person with paralysis of the superior gluteal nerve is asked to stand on one leg, the pelvis
on the unsupported side descends, indicating that the gluteus medius on the affected side is weak or
non functional ( a positive Trendelenburg test).

This eponymous test also refers to a vascular investigation in which tourniquets are placed around
the upper thigh, these can help determine whether saphenofemoral incompetence is present.
Next question

Question 113 of 560


At which level is the hilum of the left kidney located?

L1

L2

T12

T11

L3

Remember L1 ('left one') is the level of the hilum of the left kidney

Theme from April 2012 exam


Theme from April 2014
Please rate this question:

Discuss and give feedback


Next question

Renal anatomy

Each kidney is about 11cm long, 5cm wide and 3cm thick. They are located in a deep gutter
alongside the projecting vertebral bodies, on the anterior surface of psoas major. In most cases the
left kidney lies approximately 1.5cm higher than the right. The upper pole of both kidneys
approximates with the 11th rib (beware pneumothorax during nephrectomy). On the left hand side
the hilum is located at the L1 vertebral level and the right kidney at level L1-2. The lower border of
the kidneys is usually alongside L3.

The table below shows the anatomical relations of the kidneys:

Relations
Relations Right Kidney Left Kidney

Posterior Quadratus lumborum, diaphragm, psoas Quadratus lumborum, diaphragm, psoas


major, transversus abdominis major, transversus abdominis
Relations Right Kidney Left Kidney

Anterior Hepatic flexure of colon Stomach, Pancreatic tail

Superior Liver, adrenal gland Spleen, adrenal gland

Fascial covering
Each kidney and suprarenal gland is enclosed within a common layer of investing fascia, derived
from the transversalis fascia. It is divided into anterior and posterior layers (Gerotas fascia).

Renal structure
Kidneys are surrounded by an outer cortex and an inner medulla which usually contains between 6
and 10 pyramidal structures. The papilla marks the innermost apex of these. They terminate at the
renal pelvis, into the ureter.
Lying in a hollow within the kidney is the renal sinus. This contains:
1. Branches of the renal artery
2. Tributaries of the renal vein
3. Major and minor calyces's
4. Fat

Structures at the renal hilum


The renal vein lies most anteriorly, then renal artery (it is an end artery) and the ureter lies most
posterior.
Next question
Question 114 of 560

During a radical neck dissection, division of which of the following fascial layers will expose the ansa
cervicalis?

Pretracheal fascia

Carotid sheath

Prevertebral fascia

Investing layer of fascia

Sibsons fascia

The ansa cervicalis lies anterior to the carotid sheath. It may be exposed by division of the
pretracheal fascia at the posterolateral aspect of the thyroid gland. The pre vertebral fascia lies more
posteriorly and division of the investing layer of fascia will not expose this nerve.
Please rate this question:

Discuss and give feedback


Next question

Ansa cervicalis

Superior Branch of C1 anterolateral to carotid sheath


root

Inferior root Derived from C2 and C3 roots, passes posterolateral to the internal jugular vein
(may lie either deep or superficial to it)

Innervation Sternohyoid
Sternothyroid
Omohyoid
The ansa cervicalis lies anterior to the carotid sheath. The nerve supply to the inferior strap muscles
enters at their inferior aspect. Therefore when dividing these muscles to expose a large goitre, the
muscles should be divided in their upper half.

Image sourced from Wikipedia

Next question
Question 115 of 560

A 73 year old lady presents with symptoms of faecal incontinence. On examination she has weak
anal sphincter muscles. What are the main nerve root values of the nerves supplying the external
anal sphincter?

S2,3

L5, S1

S4,5

S5

S2,3,4

S2, 3, 4 Keeps the poo off the floor

Theme from April 2015Exam


The external anal sphincter is innervated by the inferior rectal branch of the pudendal nerve, this has
root values of S2, 3 and the perineal branch of S4.

Please rate this question:

Discuss and give feedback

Next question

Anal sphincter

 Internal anal sphincter composed of smooth muscle continuous with the circular muscle of
the rectum. It surrounds the upper two- thirds of the anal canal and is supplied by
sympathetic nerves.
 External anal sphincter is composed of striated muscle which surrounds the internal
sphincter but extends more distally.
 The nerve supply of the external anal sphincter is from the inferior rectal branch of the
pudendal nerve (S2 and S3) and the perineal branch of the S4 nerve roots.

Image showing relationship of internal and external anal sphincters

Image sourced from Wikipedia

Next question
Question 116 of 560

A 22 year old falls over and lands on a shard of glass. It penetrates the palmar aspect of his hand,
immediately lateral to the pisiform bone. Which of the following structures is most likely to be
injured?

Palmar cutaneous branch of the median nerve

Lateral tendons of flexor digitorum superficialis

Ulnar artery

Flexor carpi radialis tendons

Lateral tendons of flexor digitorum profundus

The ulnar nerve and artery are at most immediate risk in this injury. This is illustrated in the image
below:
Image sourced from Wikipedia

Please rate this question:

Discuss and give feedback

Next question

Hand

Anatomy of the hand

Bones  8 Carpal bones


 5 Metacarpals
 14 phalanges

Intrinsic Muscles 7 Interossei - Supplied by ulnar nerve

 3 palmar-adduct fingers
 4 dorsal- abduct fingers

Intrinsic muscles Lumbricals

 Flex MCPJ and extend the IPJ.


 Origin deep flexor tendon and insertion dorsal extensor hood mechanism.
 Innervation: 1st and 2nd- median nerve, 3rd and 4th- deep branch of the
ulnar nerve.

Thenar eminence  Abductor pollicis brevis


 Opponens pollicis
 Flexor pollicis brevis

Hypothenar  Opponens digiti minimi


eminence  Flexor digiti minimi brevis
 Abductor digiti minimi
Image sourced from Wikipedia

Fascia and compartments of the palm


The fascia of the palm is continuous with the antebrachial fascia and the fascia of the dorsum of the
hand. The palmar fascia is thin over the thenar and hypothenar eminences. In contrast the palmar
fascia is relatively thick. The palmar aponeurosis covers the soft tissues and overlies the flexor
tendons. The apex of the palmar aponeurosis is continuous with the flexor retinaculum and the
palmaris longus tendon. Distally, it forms four longitudinal digital bands that attach to the bases of
the proximal phalanges, blending with the fibrous digital sheaths.
A medial fibrous septum extends deeply from the medial border of the palmar aponeurosis to the 5th
metacarpal. Lying medial to this are the hypothenar muscles. In a similar fashion, a lateral fibrous
septum extends deeply from the lateral border of the palmar aponeurosis to the 3rd metacarpal. The
thenar compartment lies lateral to this area.
Lying between the thenar and hypothenar compartments is the central compartment. It contains the
flexor tendons and their sheaths, the lumbricals, the superficial palmar arterial arch and the digital
vessels and nerves.
The deepest muscular plane is the adductor compartment, which contains adductor pollicis.

Short muscles of the hand


These comprise the lumbricals and interossei. The four slender lumbrical muscles flex the fingers at
the metacarpophalangeal joints and extend the interphalangeal joint. The four dorsal interossei are
located between the metacarpals and the four palmar interossei lie on the palmar surface of the
metacarpals in the interosseous compartment of the hand.
Long flexor tendons and sheaths in the hand
The tendons of FDS and FDP enter the common flexor sheath deep to the flexor retinaculum. The
tendons enter the central compartment of the hand and fan out to their respective digital synovial
sheaths. Near the base of the proximal phalanx, the tendon of FDS splits to permit the passage of
FDP. The FDP tendons are attached to the margins of the anterior aspect of the base of the distal
phalanx.
The fibrous digital sheaths contain the flexor tendons and their synovial sheaths. These extend from
the heads of the metacarpals to the base of the distal phalanges.

Next question
Question 117 of 560

A 72 year old man has a fall. He is found to have a fractured neck of femur and goes on to have a
left hip hemiarthroplasty. Two months post operatively he is found to have an odd gait. When
standing on his left leg his pelvis dips on the right side. There is no foot drop. What is the cause?

Sciatic nerve damage

L5 radiculopathy

Inferior gluteal nerve damage

Previous poliomyelitis

Superior gluteal nerve damage

Theme from 2010 exam


Theme from April 2014 exam

This patient has a trendelenburg gait caused by damage to the superior gluteal nerve causing
weakness of the abductor muscles. Classically a patient is asked to stand on one leg and the pelvis
dips on the opposite side. The absence of a foot drop excludes the possibility of polio or L5
radiculopathy.

Please rate this question:

Discuss and give feedback

Next question

Gluteal region

Gluteal muscles
 Gluteus maximus: inserts to gluteal tuberosity of the femur and iliotibial tract
 Gluteus medius: attach to lateral greater trochanter
 Gluteus minimis: attach to anterior greater trochanter
 All extend and abduct the hip

Deep lateral hip rotators

 Piriformis
 Gemelli
 Obturator internus
 Quadratus femoris

Nerves

Superior gluteal nerve (L5, S1)  Gluteus medius


 Gluteus minimis
 Tensor fascia lata

Inferior gluteal nerve Gluteus maximus

Damage to the superior gluteal nerve will result in the patient developing a Trendelenberg gait.
Affected patients are unable to abduct the thigh at the hip joint. During the stance phase, the
weakened abductor muscles allow the pelvis to tilt down on the opposite side. To compensate, the
trunk lurches to the weakened side to attempt to maintain a level pelvis throughout the gait cycle.
The pelvis sags on the opposite side of the lesioned superior gluteal nerve.

Next question
Question 118 of 560

Which of the following structures lies posterior to the femoral nerve in the femoral triangle?

Adductor longus

Pectineus

Psoas major

Iliacus

None of the above

The iliacus lies posterior to the femoral nerve in the femoral triangle. The femoral sheath lies anterior
to the iliacus and pectineus muscles.

Please rate this question:

Discuss and give feedback

Next question

Femoral nerve

Root values L2, 3, 4

Innervates  Pectineus
 Sartorius
 Quadriceps femoris
 Vastus lateralis/medialis/intermedius

Branches  Medial cutaneous nerve of thigh


 Saphenous nerve
 Intermediate cutaneous nerve of thigh

Path
Penetrates psoas major and exits the pelvis by passing under the inguinal ligament to enter the
femoral triangle, lateral to the femoral artery and vein.

Image sourced from Wikipedia

Mnemonic for femoral nerve supply

(don't) M I S V Q Scan for PE


M edial cutaneous nerve of the thigh
I ntermediate cutaneous nerve of the thigh
S aphenous nerve

V astus
Q uadriceps femoris
S artorius

PE ectineus

Next question
Question 119 of 560

You are assisting in an open right adrenalectomy for a large adrenal adenoma. The consultant is
distracted and you helpfully pull the adrenal into the wound to improve the view. Unfortunately this is
followed by brisk bleeding. The vessel responsible for this is most likely to be:

Portal vein

Phrenic vein

Right renal vein

Superior mesenteric vein

Inferior vena cava

Theme from April 2014 exam


It drains directly via a very short vessel. If the sutures are not carefully tied then it may be avulsed off
the IVC. An injury best managed using a Satinsky clamp and a 6/0 prolene suture.
Please rate this question:

Discuss and give feedback


Next question

Adrenal gland anatomy

Anatomy

Location Superomedially to the upper pole of each kidney

Relationships of the Diaphragm-Posteriorly, Kidney-Inferiorly, Vena Cava-Medially,


right adrenal Hepato-renal pouch and bare area of the liver-Anteriorly

Relationships of the left Crus of the diaphragm-Postero- medially, Pancreas and splenic
adrenal vessels-Inferiorly, Lesser sac and stomach-Anteriorly
Superior adrenal arteries- from inferior phrenic artery, Middle adrenal
Arterial supply arteries - from aorta, Inferior adrenal arteries -from renal arteries

Venous drainage of the Via one central vein directly into the IVC
right adrenal

Venous drainage of the Via one central vein into the left renal vein
left adrenal

Next question
Question 120 of 560

A 28 year old lady requires an episiotomy for a ventouse vaginal delivery. Which of the nerves listed
below will usually be anaesthetised to allow the episiotomy?

Femoral

Ilioinguinal

Pudendal

Genitofemoral

Sacral plexus

Theme from April 2015 Exam


The pudendal nerve innervates the posterior vulval area and is routinely blocked in procedures such
as episiotomy.
Please rate this question:

Discuss and give feedback


Next question

Pudendal nerve

The pudendal nerve arises from nerve roots S2, S3 and S4 and exits the pelvis through the greater
sciatic foramen. It re-enters the perineum through the lesser sciatic foramen. It travels inferior to give
innervation to the anal sphincters and external urethral sphincter. It also provides cutaneous
innervation to the region of perineum surrounding the anus and posterior vulva.

Traction and compression of the pudendal nerve by the foetus in late pregnancy may result in late
onset pudendal neuropathy which may be part of the process involved in the development of faecal
incontinence.
Next question
Question 121 of 560

A motorcyclist is involved in a road traffic accident. He suffers a complex humeral shaft fracture
which is plated. Post operatively he complains of an inability to extend his fingers. Which of the
following structures is most likely to have been injured?

Ulnar nerve

Radial nerve

Median nerve

Axillary nerve

None of the above

Mnemonic for radial nerve muscles: BEST

B rachioradialis
E xtensors
S upinator
T riceps

The radial nerve is responsible for innervation of the extensor compartment of the forearm.

Please rate this question:

Discuss and give feedback

Next question

Radial nerve
Continuation of posterior cord of the brachial plexus (root values C5 to T1)

Path

 In the axilla: lies posterior to the axillary artery on subscapularis, latissimus dorsi and teres
major.
 Enters the arm between the brachial artery and the long head of triceps (medial to humerus).
 Spirals around the posterior surface of the humerus in the groove for the radial nerve.
 At the distal third of the lateral border of the humerus it then pierces the intermuscular
septum and descends in front of the lateral epicondyle.
 At the lateral epicondyle it lies deeply between brachialis and brachioradialis where it then
divides into a superficial and deep terminal branch.
 Deep branch crosses the supinator to become the posterior interosseous nerve.

In the image below the relationships of the radial nerve can be appreciated

Image sourced from Wikipedia

Regions innervated

 Triceps
Motor (main nerve)
 Anconeus
 Brachioradialis
 Extensor carpi radialis

 Supinator
 Extensor carpi ulnaris
 Extensor digitorum
Motor (posterior  Extensor indicis
interosseous branch)  Extensor digiti minimi
 Extensor pollicis longus and brevis
 Abductor pollicis longus

The area of skin supplying the proximal phalanges on the dorsal aspect of the
Sensory hand is supplied by the radial nerve (this does not apply to the little finger and
part of the ring finger)

Muscular innervation and effect of denervation

Anatomical
location Muscle affected Effect of paralysis

Shoulder Long head of triceps Minor effects on shoulder stability in abduction

Arm Triceps Loss of elbow extension

Forearm Supinator Weakening of supination of prone hand and elbow


Brachioradialis flexion in mid prone position
Extensor carpi radialis longus
and brevis

The cutaneous sensation of the upper limb- illustrating the contribution of the radial nerve
Image sourced from Wikipedia

Next question
Question 122 of 560

An enthusiastic surgical registrar undertakes his first solo splenectomy. The operation is far more
difficult than anticipated and the registrar leaves a tube drain to the splenic bed at the end of the
procedure. Over the following 24 hours approximately 500ml of clear fluid has entered the drain.
Biochemical testing of the fluid is most likely to reveal:

Elevated creatinine

Elevated triglycerides

Elevated glucagon

Elevated amylase

None of the above

During splenectomy the tail of the pancreas may be damaged. The pancreatic duct will then drain
into the splenic bed, amylase is the most likely biochemical finding. Glucagon is not secreted into the
pancreatic duct.

Please rate this question:

Discuss and give feedback

Next question

Splenic anatomy

The spleen is the largest lymphoid organ in the body. It is an intraperitoneal organ, the peritoneal
attachments condense at the hilum where the vessels enter the spleen. Its blood supply is from the
splenic artery (derived from the coeliac axis) and the splenic vein (which is joined by the IMV and
unites with the SMV).
 Embryology: derived from mesenchymal tissue
 Shape: clenched fist
 Position: below 9th-12th ribs
 Weight: 75-150g

Relations

 Superiorly- diaphragm
 Anteriorly- gastric impression
 Posteriorly- kidney
 Inferiorly- colon
 Hilum: tail of pancreas and splenic vessels
 Forms apex of lesser sac (containing short gastric vessels)

Next question
Question 123 of 560

A 48 year old lady is undergoing an axillary node clearance for breast cancer. Which of the
structures listed below are most likely to be encountered during the axillary dissection?

Cords of the brachial plexus

Thoracodorsal trunk

Internal mammary artery

Thoracoacromial artery

None of the above

Beware of damaging the thoracodorsal trunk if a latissimus dorsi flap reconstruction is planned.

Theme from 2009 Exam


Theme from 2014 Exam
The thoracodorsal trunk runs through the nodes in the axilla. If injured it may compromise the
function and blood supply to latissimus dorsi, which is significant if it is to be used as a flap for a
reconstructive procedure.
Please rate this question:

Discuss and give feedback


Next question

Axilla

Boundaries of the axilla


Medially Chest wall and Serratus anterior

Laterally Humeral head

Floor Subscapularis
Anterior aspect Lateral border of Pectoralis major

Fascia Clavipectoral fascia

Content:
Long thoracic nerve (of Derived from C5-C7 and passes behind the brachial plexus to enter
Bell) the axilla. It lies on the medial chest wall and supplies serratus
anterior. Its location puts it at risk during axillary surgery and
damage will lead to winging of the scapula.

Thoracodorsal nerve and Innervate and vascularise latissimus dorsi.


thoracodorsal trunk

Axillary vein Lies at the apex of the axilla, it is the continuation of the basilic vein.
Becomes the subclavian vein at the outer border of the first rib.

Intercostobrachial Traverse the axillary lymph nodes and are often divided during
nerves axillary surgery. They provide cutaneous sensation to the axillary
skin.

Lymph nodes The axilla is the main site of lymphatic drainage for the breast.

Next question
Question 124 of 560

A 53 year old lady is recovering following a difficult mastectomy and axillary nodal clearance for
carcinoma of the breast. She complains of shoulder pain and on examination has obvious winging of
the scapula. Loss of innervation to which of the following is the most likely underlying cause?

Latissimus dorsi

Serratus anterior

Pectoralis minor

Pectoralis major

Rhomboids

Theme from April 2012 exam


Theme from April 2014 exam
Winging of the scapula is most commonly the result of long thoracic nerve injury or dysfunction.
Iatrogenic damage during the course of the difficult axillary dissection is the most likely cause in this
scenario. Damage to the rhomboids may produce winging of the scapula but would be rare in the
scenario given.

Please rate this question:

Discuss and give feedback

Next question

Long thoracic nerve

 Derived from ventral rami of C5, C6, and C7 (close to their emergence from intervertebral
foramina)
 It runs downward and passes either anterior or posterior to the middle scalene muscle
 It reaches upper tip of serratus anterior muscle and descends on outer surface of this
muscle, giving branches into it
 Winging of Scapula occurs in long thoracic nerve injury (most common) or from spinal
accessory nerve injury (which denervates the trapezius) or a dorsal scapular nerve injury

Next question
Question 125 of 560

A 56 year old lady is referred to the colorectal clinic with symptoms of pruritus ani. On examination a
polypoidal mass is identified inferior to the dentate line. A biopsy confirms squamous cell carcinoma.
To which of the following lymph node groups will the lesion potentially metastasise?

Internal iliac

External iliac

Mesorectal

Inguinal

None of the above

Theme from September 2011 Exam


Theme from April 2012 Exam
Theme from April 2014 exam
Lesions distal to the dentate line drain to the inguinal nodes. Occasionally this will result in the need
for a block dissection of the groin.

Please rate this question:

Discuss and give feedback

Next question

Rectum

The rectum is approximately 12 cm long. It is a capacitance organ. It has both intra and
extraperitoneal components. The transition between the sigmoid colon is marked by the
disappearance of the tenia coli.The extra peritoneal rectum is surrounded by mesorectal fat that also
contains lymph nodes. This mesorectal fatty layer is removed surgically during rectal cancer surgery
(Total Mesorectal Excision). The fascial layers that surround the rectum are important clinical
landmarks, anteriorly lies the fascia of Denonvilliers. Posteriorly lies Waldeyers fascia.

Extra peritoneal rectum

 Posterior upper third


 Posterior and lateral middle third
 Whole lower third

Relations

Anteriorly (Males) Rectovesical pouch


Bladder
Prostate
Seminal vesicles

Anteriorly (Females) Recto-uterine pouch (Douglas)


Cervix
Vaginal wall

Posteriorly Sacrum
Coccyx
Middle sacral artery

Laterally Levator ani


Coccygeus

Arterial supply
Superior rectal artery

Venous drainage
Superior rectal vein

Lymphatic drainage

 Mesorectal lymph nodes (superior to dentate line)


 Internal iliac and then para-aortic nodes
 Inguinal nodes (inferior to dentate line)
Question 126 of 560

A 20 year old man is hit with a hammer on the right side of the head. He dies on arrival in the
emergency department. Which of these features is most likely to be found at post mortem?

Hydrocephalus

Supra tentorial herniation

Laceration of the middle meningeal artery

Sub dural haematoma

Posterior fossa haematoma

Theme based on 2014 exam


This will account for the scenario given where there is a brief delay prior to death. The other options
are less acute and a supratentorial herniation would not occur in this setting.

Please rate this question:

Discuss and give feedback

Next question

Head injury

Patients who suffer head injuries should be managed according to ATLS principles and extra cranial
injuries should be managed alongside cranial trauma. Inadequate cardiac output will compromise
CNS perfusion irrespective of the nature of the cranial injury.

Types of traumatic brain injury


Bleeding into the space between the dura mater and the skull. Often results from
acceleration-deceleration trauma or a blow to the side of the head. The majority
of extradural haematomas occur in the temporal region where skull fractures
cause a rupture of the middle meningeal artery.
Extradural
haematoma Features

 Raised intracranial pressure


 Some patients may exhibit a lucid interval

Bleeding into the outermost meningeal layer. Most commonly occur around the
frontal and parietal lobes. May be either acute or chronic.
Subdural
haematoma Risk factors include old age and alcoholism.

Slower onset of symptoms than a extradural haematoma.

Usually occurs spontaneously in the context of a ruptured cerebral aneurysm, but


Subarachnoid
may be seen in association with other injuries when a patient has sustained a
haemorrhage
traumatic brain injury.

Pathophysiology

 Primary brain injury may be focal (contusion/ haematoma) or diffuse (diffuse axonal injury)
 Diffuse axonal injury occurs as a result of mechanical shearing following deceleration,
causing disruption and tearing of axons
 Intra-cranial haematomas can be extradural, subdural or intracerebral, while contusions may
occur adjacent to (coup) or contralateral (contre-coup) to the side of impact
 Secondary brain injury occurs when cerebral oedema, ischaemia, infection, tonsillar or
tentorial herniation exacerbates the original injury. The normal cerebral auto regulatory
processes are disrupted following trauma rendering the brain more susceptible to blood flow
changes and hypoxia
 The Cushings reflex (hypertension and bradycardia) often occurs late and is usually a pre
terminal event

Management

 Where there is life threatening rising ICP such as in extra dural haematoma and whilst
theatre is prepared or transfer arranged use of IV mannitol/ frusemide may be required.
 Diffuse cerebral oedema may require decompressive craniotomy
 Exploratory Burr Holes have little management in modern practice except where scanning
may be unavailable and to thus facilitate creation of formal craniotomy flap
 Depressed skull fractures that are open require formal surgical reduction and debridement,
closed injuries may be managed non operatively if there is minimal displacement.
 ICP monitoring is appropriate in those who have GCS 3-8 and normal CT scan.
 ICP monitoring is mandatory in those who have GCS 3-8 and abnormal CT scan.
 Hyponatraemia is most likely to be due to syndrome of inappropriate ADH secretion.
 Minimum of cerebral perfusion pressure of 70mmHg in adults.
 Minimum cerebral perfusion pressure of between 40 and 70 mmHg in children.

Interpretation of pupillary findings in head injuries

Pupil size Light response Interpretation

Unilaterally dilated Sluggish or fixed 3rd nerve compression secondary to tentorial


herniation

Bilaterally dilated Sluggish or fixed  Poor CNS perfusion


 Bilateral 3rd nerve palsy

Unilaterally dilated or Cross reactive (Marcus - Optic nerve injury


equal Gunn)

Bilaterally constricted May be difficult to assess  Opiates


 Pontine lesions
 Metabolic encephalopathy

Unilaterally constricted Preserved Sympathetic pathway disruption

Next question
Question 127 of 560

Which of the following ligaments contains the artery supplying the head of femur in children?

Transverse ligament

Ligamentum teres

Iliofemoral ligament

Ischiofemoral ligament

Pubofemoral ligament

Theme from 2010 Exam

Please rate this question:

Discuss and give feedback

Next question

Hip joint

 Head of femur articulates with acetabulum of the pelvis


 Both covered by articular hyaline cartilage
 The acetabulum forms at the union of the ilium, pubis, and ischium
 The triradiate cartilage (Y-shaped growth plate) separates the pelvic bones
 The acetabulum holds the femoral head by the acetabular labrum
 Normal angle between femoral head and femoral shaft is 130o

Ligaments
 Transverse ligament: joints anterior and posterior ends of the articular cartilage
 Head of femur ligament (ligamentum teres): acetabular notch to the fovea. Contains arterial
supply to head of femur in children.

Image sourced from Wikipedia


Image sourced from Wikipedia

Extracapsular ligaments

 Iliofemoral ligament: inverted Y shape. Anterior iliac spine to the trochanteric line
 Pubofemoral ligament: acetabulum to lesser trochanter
 Ischiofemoral ligament: posterior support. Ischium to greater trochanter.

Blood supply
Medial circumflex femoral and lateral circumflex femoral arteries (Branches of profunda femoris).
Also from the inferior gluteal artery. These form an anastomosis and travel to up the femoral neck to
supply the head.
Next question
Question 128 of 560

A 72 year old man develops a hydrocele which is being surgically managed. As part of the
procedure the surgeons divide the tunica vaginalis. From which of the following is this structure
derived?

Peritoneum

External oblique aponeurosis

Internal oblique aponeurosis

Transversalis fascia

Rectus sheath

The tunica vaginalis is derived from peritoneum, it secretes the fluid that fills the hydrocele cavity.

Please rate this question:

Discuss and give feedback

Next question

Scrotal and testicular anatomy

Spermatic cord
Formed by the vas deferens and is covered by the following structures:

Layer Origin
Layer Origin

Internal spermatic fascia Transversalis fascia

Cremasteric fascia From the fascial coverings of internal oblique

External spermatic fascia External oblique aponeurosis

Contents of the cord

Vas deferens Transmits sperm and accessory gland secretions

Testicular artery Branch of abdominal aorta supplies testis and epididymis

Artery of vas deferens Arises from inferior vesical artery

Cremasteric artery Arises from inferior epigastric artery

Pampiniform plexus Venous plexus, drains into right or left testicular vein

Sympathetic nerve fibres Lie on arteries, the parasympathetic fibres lie on the vas

Genital branch of the genitofemoral nerve Supplies cremaster

Lymphatic vessels Drain to lumbar and para-aortic nodes

Scrotum

 Composed of skin and closely attached dartos fascia.


 Arterial supply from the anterior and posterior scrotal arteries
 Lymphatic drainage to the inguinal lymph nodes
 Parietal layer of the tunica vaginalis is the innermost layer

Testes

 The testes are surrounded by the tunica vaginalis (closed peritoneal sac). The parietal layer
of the tunica vaginalis adjacent to the internal spermatic fascia.
 The testicular arteries arise from the aorta immediately inferiorly to the renal arteries.
 The pampiniform plexus drains into the testicular veins, the left drains into the left renal vein
and the right into the inferior vena cava.
 Lymphatic drainage is to the para-aortic nodes.

Next question
Question 129 of 560

A 43 year old lady is donating her left kidney to her sister and the surgeons are harvesting the left
kidney. Which of the following structures will lie most anteriorly at the hilum of the left kidney?

Left renal artery

Left renal vein

Left ureter

Left ovarian vein

Left ovarian artery

The renal veins lie most anteriorly, then artery and ureter lies posteriorly.

Please rate this question:

Discuss and give feedback

Next question

Renal arteries

 The right renal artery is longer than the left renal artery
 The renal vein/artery/pelvis enter the kidney at the hilum

Relations

Right Anterior- IVC, right renal vein, the head of the pancreas, and the descending part of the
duodenum

Left Anterior- left renal vein, the tail of the pancreas

Branches

 The renal arteries are direct branches off the aorta (upper border of L2- right side and L1 -
left side)
 In 30% there may be accessory arteries (mainly left side). Instead of entering the kidney at
the hilum, they usually pierce the upper or lower part of the organ.
 Before reaching the hilum of the kidney, each artery divides into four or five segmental
branches (renal vein anterior and ureter posterior); which then divide within the sinus into
lobar arteries supplying each pyramid and cortex.
 Each vessel gives off some small inferior suprarenal branches to the suprarenal gland, the
ureter, and the surrounding cellular tissue and muscles.

Next question
Question 130 of 560

What is the sensory nerve supply to the angle of the jaw?

Maxillary branch of the trigeminal nerve

Mandibular branch of the trigeminal nerve

C3-C4

Greater auricular nerve (C2-C3)

Buccal branch of the facial nerve

The trigeminal nerve is the major sensory nerve to the face except over the angle of the jaw. The
angle of the jaw is innervated by the greater auricular nerve.

Please rate this question:

Discuss and give feedback

Next question

Trigeminal nerve

The trigeminal nerve is the main sensory nerve of the head. In addition to its major sensory role, it
also innervates the muscles of mastication.

Distribution of the trigeminal nerve

Sensory  Scalp
 Face
 Oral cavity (and teeth)
 Nose and sinuses
 Dura mater

Motor  Muscles of mastication


 Mylohyoid
 Anterior belly of digastric
 Tensor tympani
 Tensor palati

Autonomic connections (ganglia)  Ciliary


 Sphenopalatine
 Otic
 Submandibular

Path

 Originates at the pons


 Sensory root forms the large, crescentic trigeminal ganglion within Meckel's cave, and
contains the cell bodies of incoming sensory nerve fibres. Here the 3 branches exit.
 The motor root cell bodies are in the pons and the motor fibres are distributed via the
mandibular nerve. The motor root is not part of the trigeminal ganglion.

Branches of the trigeminal nerve

Ophthalmic nerve Sensory only

Maxillary nerve Sensory only

Mandibular nerve Sensory and motor

Sensory

Ophthalmic Exits skull via the superior orbital fissure


Sensation of: scalp and forehead, the upper eyelid, the conjunctiva and cornea of the eye,
the nose (including the tip of the nose, except alae nasi), the nasal mucosa, the frontal
sinuses, and parts of the meninges (the dura and blood vessels).

Maxillary Exit skull via the foramen rotundum


nerve Sensation: lower eyelid and cheek, the nares and upper lip, the upper teeth and gums,
the nasal mucosa, the palate and roof of the pharynx, the maxillary, ethmoid and
sphenoid sinuses, and parts of the meninges.

Mandibular Exit skull via the foramen ovale


nerve Sensation: lower lip, the lower teeth and gums, the chin and jaw (except the angle of the
jaw), parts of the external ear, and parts of the meninges.

Motor
Distributed via the mandibular nerve.
The following muscles of mastication are innervated:

 Masseter
 Temporalis
 Medial pterygoid
 Lateral pterygoid

Other muscles innervated include:

 Tensor veli palatini


 Mylohyoid
 Anterior belly of digastric
 Tensor tympani

Next question
Question 131 of 560

A 63 year old man is undergoing a coronary artery bypass procedure. During the median sternotomy
which structure would routinely require division?

Parietal pleura

Interclavicular ligament

Internal mammary artery

Brachiocephalic vein

Left vagus nerve

Theme from January 2011 Exam


The interclavicular ligament lies at the upper end of a median sternotomy and is routinely divided to
provide access. The pleural reflections are often encountered and should not be intentionally
divided, if they are, then a chest drain will need to be inserted on the affected side as collections
may then accumulate in the pleural cavity. Other structures encountered include the pectoralis major
muscles, again if the incision is truly midline then these should not require formal division. The close
relationship of the brachiocephalic vein should be borne in mind and it should be avoided, iatrogenic
injury to this structure will result in considerable haemorrhage.
Please rate this question:

Discuss and give feedback


Next question

Sternotomy

A median sternotomy is the commonest incision utilised to access the heart and great vessels.
A midline incision is made from the interclavicular fossa to the xiphoid process and the fat and
subcutaneous tissues are divided to the level of the sternum. The periosteum may be gently
mobilised off the midline, although vigorous periosteal stripping is best avoided. A bone saw is used
to divide the bone itself. Posteriorly the reflections of the parietal pleura should be identified and
avoided (unless surgery to the lung is planned). The fibrous pericardium is incised and the heart
brought into view. Bleeding from the bony edges of the cut sternum is stopped using roller ball
diathermy or bone wax. The left brachiocephalic vein is an important posterior relation at the
superior aspect of the sternotomy incision and should be avoided. More inferior the thymic remnants
may be identified. At the inferior aspect of the incision the abdominal cavity may be entered (though
this is seldom troublesome).
Question 132 of 560

Which of the following structures separates the subclavian artery from the subclavian vein?

Scalenus anterior

Scalenus medius

Sternocleidomastoid

Pectoralis major

Pectoralis minor

The artery and vein are separated by scalenus anterior. This muscle runs from the transverse
processes of C3,4,5 and 6 to insert onto the scalene tubercle of the first rib.

Please rate this question:

Discuss and give feedback

Next question

Subclavian artery

Path

 The left subclavian comes directly off the arch of aorta


 The right subclavian arises from the brachiocephalic artery (trunk) when it bifurcates into the
subclavian and the right common carotid artery.
 From its origin, the subclavian artery travels laterally, passing between anterior and middle
scalene muscles, deep to scalenus anterior and anterior to scalenus medius. As the
subclavian artery crosses the lateral border of the first rib, it becomes the axillary artery. At
this point it is superficial and within the subclavian triangle.

Image sourced from Wikipedia

Branches

 Vertebral artery
 Internal thoracic artery
 Thyrocervical trunk
 Costocervical trunk
 Dorsal scapular artery

Next question
Question 133 of 560

A 56 year old lady is due to undergo a left hemicolectomy for carcinoma of the splenic flexure. The
surgeons decide to perform a high ligation of the inferior mesenteric vein. Into which of the following
does this structure usually drain?

Portal vein

Inferior vena cava

Left renal vein

Left iliac vein

Splenic vein

Beware of ureteric injury in colonic surgery.

The inferior mesenteric vein drains into the splenic vein, this point of union lies close to the
duodenum and this surgical maneouvre is a recognised cause of ileus.

Please rate this question:

Discuss and give feedback

Next question

Left colon

Position
 As the left colon passes inferiorly its posterior aspect becomes extraperitoneal, and the
ureter and gonadal vessels are close posterior relations that may become involved in
disease processes
 At a level of L3-4 (variable) the left colon becomes the sigmoid colon and wholly
intraperitoneal once again
 The sigmoid colon is a highly mobile structure and may even lie on the right side of the
abdomen
 It passes towards the midline, the taenia blend and this marks the transition between sigmoid
colon and upper rectum

Blood supply

 Inferior mesenteric artery


 However, the marginal artery (from the right colon) contributes, this contribution becomes
clinically significant when the IMA is divided surgically (e.g. During AAA repair)

Next question
Question 134 of 560

A man undergoes a high anterior resection for carcinoma of the upper rectum. Which of the following
vessels will require ligation?

Superior mesenteric artery

Inferior mesenteric artery

Coeliac axis

Perineal artery

Middle colic artery

Theme from April 2013 Exam


Theme from September 2013 Exam
The IMA is usually divided during anterior resection. Not only is this borne out of oncological
necessity but it also permits sufficient colonic mobilisation for anastomosis.
Please rate this question:

Discuss and give feedback


Next question

Colon anatomy

The colon commences with the caecum. This represents the most dilated segment of the human
colon and its base (which is intraperitoneal) is marked by the convergence of teniae coli. At this point
is located the vermiform appendix. The colon continues as the ascending colon, the posterior aspect
of which is retroperitoneal. The line of demarcation between the intra and retro peritoneal right colon
is visible as a white line, in the living, and forms the line of incision for colonic resections.

The ascending colon becomes the transverse colon after passing the hepatic flexure. At this located
the colon becomes wholly intra peritoneal once again. The superior aspect of the transverse colon is
the point of attachment of the transverse colon to the greater omentum. This is an important
anatomical site since division of these attachments permits entry into the lesser sac. Separation of
the greater omentum from the transverse colon is a routine operative step in both gastric and colonic
resections.

At the left side of the abdomen the transverse colon passes to the left upper quadrant and makes an
oblique inferior turn at the splenic flexure. Following this, the posterior aspect becomes
retroperitoneal once again.

At the level of approximately L4 the descending colon becomes wholly intraperitoneal and becomes
the sigmoid colon. Whilst the sigmoid is wholly intraperitoneal there are usually attachments laterally
between the sigmoid and the lateral pelvic sidewall. These small congenital adhesions are not formal
anatomical attachments but frequently require division during surgical resections.

At its distal end the sigmoid passes to the midline and at the region around the sacral promontary it
becomes the upper rectum. This transition is visible macroscopically as the point where the teniae
fuse. More distally the rectum passes through the peritoneum at the region of the peritoneal
reflection and becomes extraperitoneal.

Arterial supply
Superior mesenteric artery and inferior mesenteric artery: linked by the marginal artery.
Ascending colon: ileocolic and right colic arteries
Transverse colon: middle colic artery
Descending and sigmoid colon: inferior mesenteric artery

Venous drainage
From regional veins (that accompany arteries) to superior and inferior mesenteric vein

Lymphatic drainage
Initially along nodal chains that accompany supplying arteries, then para-aortic nodes.

Embryology
Midgut- Second part of duodenum to 2/3 transverse colon
Hindgut- Distal 1/3 transverse colon to anus

Peritoneal location
The right and left colon are part intraperitoneal and part extraperitoneal. The sigmoid and transverse
colon are generally wholly intraperitoneal. This has implications for the sequelae of perforations,
which will tend to result in generalised peritonitis in the wholly intra peritoneal segments.

Colonic relations
Region of colon Relation

Caecum/ right colon Right ureter, gonadal vessels

Hepatic flexure Gallbladder (medially)

Splenic flexure Spleen and tail of pancreas

Distal sigmoid/ upper Left ureter


rectum
Region of colon Relation

Rectum Ureters, autonomic nerves, seminal vesicles, prostate, urethra


(distally)

Next question
Question 135 of 560

A 43 year old lady is due to undergo an axillary node clearance as part of treatment for carcinoma of
the breast. Which of the following fascial layers will be divided during the surgical approach to the
axilla?

Sibsons fascia

Pre tracheal fascia

Waldayers fascia

Clavipectoral fascia

None of the above

The clavipectoral fascia is situated under the clavicular portion of pectoralis major. It protects both
the axillary vessels and nodes. During an axillary node clearance for breast cancer the clavipectoral
fascia is incised and this allows access to the nodal stations. The nodal stations are; level 1 nodes
inferior to pectoralis minor, level 2 lie behind it and level 3 above it. During a Patey Mastectomy
surgeons divide pectoralis minor to gain access to level 3 nodes. The use of sentinel node biopsy
(and stronger assistants!) have made this procedure far less common.
Please rate this question:

Discuss and give feedback


Next question

Axilla

Boundaries of the axilla


Medially Chest wall and Serratus anterior

Laterally Humeral head

Floor Subscapularis
Anterior aspect Lateral border of Pectoralis major

Fascia Clavipectoral fascia

Content:
Long thoracic nerve (of Derived from C5-C7 and passes behind the brachial plexus to enter
Bell) the axilla. It lies on the medial chest wall and supplies serratus
anterior. Its location puts it at risk during axillary surgery and
damage will lead to winging of the scapula.

Thoracodorsal nerve and Innervate and vascularise latissimus dorsi.


thoracodorsal trunk

Axillary vein Lies at the apex of the axilla, it is the continuation of the basilic vein.
Becomes the subclavian vein at the outer border of the first rib.

Intercostobrachial Traverse the axillary lymph nodes and are often divided during
nerves axillary surgery. They provide cutaneous sensation to the axillary
skin.

Lymph nodes The axilla is the main site of lymphatic drainage for the breast.

Next question
Question 136 of 560

What are the boundaries of the 'safe triangle' for chest drain insertion?

Bounded by trapezius, latissimus dorsi, and laterally by the vertebral border of the scapula

Bounded by latissimus dorsi, pectoralis major, line superior to the nipple and apex at the
axilla

Bounded by latissimus dorsi, serratus anterior, line superior to the nipple and apex at the
axilla

Bounded by trapezius, deltoid, rhomboid major and teres minor

Bounded by trapezius, deltoid and latissimus dorsi

Theme from April 2012 Exam


Please rate this question:

Discuss and give feedback


Next question

Chest drains

There are a number of different indications for chest drain insertion. In general terms large bore
chest drains are preferred for trauma and haemothorax drainage. Smaller diameter chest drains can
be used for pneumothorax or pleural effusion drainage.

Insertion can be performed either using anatomical guidance or through ultrasound guidance. In the
exam, the anatomical method is usually tested.

It is advised that chest drains are placed in the 'safe triangle'. The triangle is located in the mid
axillary line of the 5th intercostal space. It is bordered by:
Anterior edge latissimus dorsi, the lateral border of pectoralis major, a line superior to the horizontal
level of the nipple, and the apex below the axilla.

Another triangle is situated behind the scapula. It is bounded above by the trapezius, below by the
latissimus dorsi, and laterally by the vertebral border of the scapula; the floor is partly formed by the
rhomboid major. If the scapula is drawn forward by folding the arms across the chest, and the trunk
bent forward, parts of the sixth and seventh ribs and the interspace between them become
subcutaneous and available for auscultation. The space is therefore known as the triangle of
auscultation.
References
Prof Harold Ellis. The applied anatomy of chest drains insertions. British Journal of hospital medicine
2007; (68): 44-45.

Laws D, Neville E, Duffy J. BTS guidelines for insertion of chest drains. Thorax, 2003; (58): 53-59.
Next question
Question 137 of 560

The vertebral artery traverses all of the following except?

Transverse process of C6

Transverse process of the axis

Vertebral canal

Foramen magnum

Intervertebral foramen

The vertebral artery passes through the foramina which are located in the transverse processes of
the cervical vertebra, it does not traverse the intervertebral foramen.

Please rate this question:

Discuss and give feedback

Next question

Vertebral artery

The vertebral artery is the first branch of the subclavian artery. Anatomically it is divisible into 4
regions:

 The first part runs to the foramen in the transverse process of C6. Anterior to this part lies the
vertebral and internal jugular veins. On the left side the thoracic duct is also an anterior
relation.
 The second part runs superiorly through the foramina of the the transverse processes of the
upper 6 cervical vertebrae. Once it has passed through the transverse process of the axis it
then turns superolaterally to the atlas. It is accompanied by a venous plexus and the inferior
cervical sympathetic ganglion.
 The third part runs posteromedially on the lateral mass of the atlas. It enters the sub occipital
triangle, in the groove of the upper surface of the posterior arch of the atlas. It then passes
anterior to the edge of the posterior atlanto-occipital membrane to enter the vertebral canal.
 The fourth part passes through the spinal dura and arachnoid, running superiorly and
anteriorly at the lateral aspect of the medulla oblongata. At the lower border of the pons it
unites to form the basilar artery.

Next question
Question 138 of 560

A 60 year old female attends the preoperative hernia clinic. She reports some visual difficulty. On
examination she is noted to have a homonymous hemianopia. Where is the lesion most likely to be?

Frontal lobe

Pituitary gland

Parietal lobe

Optic chiasm

Optic tract

Lesions before optic chiasm:


Monocular vision loss = Optic nerve lesion
Bitemporal hemianopia = Optic chiasm lesion

Lesions after the optic chiasm:


Homonymous hemianopia = Optic tract lesion
Upper quadranopia = Temporal lobe lesion
Lower quadranopia = Parietal lobe lesion

Theme from April 2012 exam


Theme from April 2014 exam
Unfortunately we thought as surgeons we could forget about visual field defects! However the
college seem to like them. Remember a homonymous hemianopia is indicative of an optic tract
lesion. Parietal lobe lesions tend to cause inferior quadranopias and there is a bitemporal
hemianopia with optic chiasm lesion or pituitary tumours.

Please rate this question:


Discuss and give feedback

Next question

Visual field defects

Theme from January 2012 exam


Theme from April 2012 exam

 left homonymous hemianopia means visual field defect to the left, i.e. Lesion of right optic
tract
 homonymous quadrantanopias: PITS (Parietal-Inferior, Temporal-Superior)
 incongruous defects = optic tract lesion; congruous defects = optic radiation lesion or
occipital cortex

Homonymous hemianopia

 Incongruous defects: lesion of optic tract


 Congruous defects: lesion of optic radiation or occipital cortex
 Macula sparing: lesion of occipital cortex

Homonymous quadrantanopias

 Superior: lesion of temporal lobe


 Inferior: lesion of parietal lobe
 Mnemonic = PITS (Parietal-Inferior, Temporal-Superior)

Bitemporal hemianopia

 Lesion of optic chiasm


 Upper quadrant defect > lower quadrant defect = inferior chiasmal compression, commonly a
pituitary tumour
 Lower quadrant defect > upper quadrant defect = superior chiasmal compression, commonly
a craniopharyngioma

Next question
Question 139 of 560

A 34 year old male is being examined in the pre-operative assessment clinic. A murmur is identified
in the 4th intercostal space just next to the left side of the sternum. From where is it most likely to
have originated?

Mitral valve

Aortic valve

Pulmonary valve

Right ventricular aneurysm

Tricuspid valve

Theme from September 2012 Exam


The tricuspid valve is generally referred to being best auscultated adjacent to the sternum. The
plane of projected sound from the mitral area is best heard in the region of the cadiac apex.
Please rate this question:

Discuss and give feedback


Next question

Heart sounds

Sites of auscultation
Valve Site

Pulmonary valve Left second intercostal space, at the upper sternal border

Aortic valve Right second intercostal space, at the upper sternal border

Mitral valve Left fifth intercostal space, just medial to mid clavicular line
Valve Site

Tricuspid valve Left fourth intercostal space, at the lower left sternal border

The diagram below demonstrates where the various cardiac valves are best heard.

Image sourced from Wikipedia

Next question
Question 140 of 560

During an Ivor Lewis Oesophagectomy for carcinoma of the lower third of the oesophagus which
structure is divided to allow mobilisation of the oesophagus?

Vagus nerve

Azygos vein

Right inferior lobar bronchus

Phrenic nerve

Pericardiophrenic artery

Theme from April 2014 exam


The azygos vein is routinely divided during an oesophagectomy to allow mobilisation. It arches
anteriorly to insert into the SVC on the right hand side.

Please rate this question:

Discuss and give feedback

Next question

Treatment of oesophageal cancer

 In general resections are not offered to those patients with distant metastasis, and usually
not to those with N2 disease.
 Local nodal involvement is not in itself a contra indication to resection.
 Surgical resection is the mainstay of treatment.
 Neoadjuvent chemotherapy is given in most cases prior to surgery.
 In situ disease may be managed by endoscopic mucosal resection, although this is still
debated.
 In patients with lower third lesions an Ivor - Lewis type procedure is most commonly
performed. Very distal tumours may be suitable to a transhiatal procedure. Which is an
attractive option as the penetration of two visceral cavities required for an Ivor- Lewis type
procedure increases the morbidity considerably.
 More proximal lesions will require a total oesphagectomy (Mckeown type) with anastomosis
to the cervical oesophagus.
 Patients with unresectable disease may derive benefit from local ablative procedures,
palliative chemotherapy or stent insertion.

Operative details of Ivor- Lewis procedure

 Combined laparotomy and right thoracotomy

Indication

 Lower and middle third oesophageal tumours

Preparation

 Staging with a combination of CT chest abdomen and pelvis- if no metastatic disease


detected then patients will undergo a staging laparoscopy to detect peritoneal disease.
 If both these modalities are negative then patients will finally undergo a PET CT scan to
detect occult metastatic disease. Only in those whom no evidence of advanced disease is
detected will proceed to resection.
 Patients receive a GA, double lumen endotracheal tube to allow for lung deflation, CVP and
arterial monitoring.

Procedure

 A rooftop incision is made to access the stomach and duodenum.

Laparotomy To mobilize the stomach

 The greater omentum is incised away from its attachment to the right gastroepiploic vessels
along the greater curvature of the stomach.
 Then the short gastric vessels are ligated and detached from the greater curvature from the
spleen.
 The lesser omentum is incised, preserving the right gastric artery.
 The retroperitoneal attachments of the duodenum in its second and third portions are
incised, allowing the pylorus to reach the oesophageal hiatus. Some surgeons perform a
pyloroplasty at this point to facilitate gastric emptying.
 The left gastric vessels are then ligated, avoiding any injury to the common hepatic or splenic
arteries. Care must be taken to avoid inadvertently devascularising the liver owing to
variations in anatomy.

Right Thoracotomy Oesophageal resection and oesophagogastric anastomosis

 Through 5th intercostal space


 Dissection performed 10cm above the tumour
 This may involve transection of the azygos vein.
 The oesophagus is then removed with the stomach creating a gastric tube.
 An anastomosis is created.

The chest is closed with underwater seal drainage and tube drains to the abdominal cavity.

Post operatively

 Patients will typically recover in ITU initially.


 A nasogastric tube will have been inserted intraoperatively and must remain in place during
the early phases of recovery.
 Post operatively these patients are at relatively high risk of developing complications:

* Atelectasis- due to the effects of thoracotomy and lung collapse


* Anastomotic leakage. The risk is relatively high owing to the presence of a relatively
devascularised stomach. Often the only blood supply is from the gastroepiploic artery as all others
will have been divided. If a leak does occur then many will attempt to manage conservatively with
prolonged nasogastric tube drainage and TPN. The reality is that up to 50% of patients developing
an anastomotic leak will not survive to discharge.
* Delayed gastric emptying (may be avoided by performing a pyloroplasty).
Next question
Question 141 of 560

Which of the following statements relating to quadratus lumborum is false?

Causes flexion of the thoracic spine

Causes the rib cage to be pulled down

Innervated by anterior primary rami of T12 and L1-3

Attached to the iliac crest

Inserts into the 12th rib

Quadratus lumborum
Origin: Medial aspect of iliac crest and iliolumbar ligament
Insertion: 12th rib
Action: Pulls the rib cage inferiorly. Lateral flexion.
Nerve supply: Anterior primary rami of T12 and L1-3

The rectus abdominis causes flexion of the thoracic spine and therefore the statement suggesting
that quaratus lumborum does so is incorrect.

Please rate this question:

Discuss and give feedback

Next question

Abdominal wall

The 2 main muscles of the abdominal wall are the rectus abdominis (anterior) and the quadratus
lumborum (posterior).
The remaining abdominal wall consists of 3 muscular layers. Each muscle passes from the lateral
aspect of the quadratus lumborum posteriorly to the lateral margin of the rectus sheath anteriorly.
Each layer is muscular posterolaterally and aponeurotic anteriorly.

Image sourced from Wikipedia

Muscles of abdominal wall

External  Lies most superficially


oblique  Originates from 5th to 12th ribs
 Inserts into the anterior half of the outer aspect of the iliac crest, linea alba and
pubic tubercle
 More medially and superiorly to the arcuate line, the aponeurotic layer
overlaps the rectus abdominis muscle
 The lower border forms the inguinal ligament
 The triangular expansion of the medial end of the inguinal ligament is the
lacunar ligament.

Internal  Arises from the thoracolumbar fascia, the anterior 2/3 of the iliac crest and the
oblique lateral 2/3 of the inguinal ligament
 The muscle sweeps upwards to insert into the cartilages of the lower 3 ribs
 The lower fibres form an aponeurosis that runs from the tenth costal cartilage
to the body of the pubis
 At its lowermost aspect it joins the fibres of the aponeurosis of transversus
abdominis to form the conjoint tendon.

Transversus  Innermost muscle


abdominis  Arises from the inner aspect of the costal cartilages of the lower 6 ribs , from
the anterior 2/3 of the iliac crest and lateral 1/3 of the inguinal ligament
 Its fibres run horizontally around the abdominal wall ending in an aponeurosis.
The upper part runs posterior to the rectus abdominis. Lower down the fibres
run anteriorly only.
 The rectus abdominis lies medially; running from the pubic crest and symphysis
to insert into the xiphoid process and 5th, 6th and 7th costal cartilages. The
muscles lies in a aponeurosis as described above.
 Nerve supply: anterior primary rami of T7-12

Surgical notes
During abdominal surgery it is usually necessary to divide either the muscles or their aponeuroses.
During a midline laparotomy it is desirable to divide the aponeurosis. This will leave the rectus
sheath intact above the arcuate line and the muscles intact below it. Straying off the midline will
often lead to damage to the rectus muscles, particularly below the arcuate line where they may often
be in close proximity to each other.

Next question
Question 142 of 560

A 23 year old climber falls and fractures his humerus. The surgeons decide upon a posterior
approach to the middle third of the bone. Which of the following nerves is at greatest risk in this
approach?

Ulnar

Antebrachial

Musculocutaneous

Radial

Intercostobrachial

Theme from April 2012 Exam


The radial nerve wraps around the humerus and may be injured during a posterior approach. An IM
nail may be preferred as it avoids the complex dissection needed for direct bone exposure.

Please rate this question:

Discuss and give feedback

Next question

Radial nerve

Continuation of posterior cord of the brachial plexus (root values C5 to T1)

Path
 In the axilla: lies posterior to the axillary artery on subscapularis, latissimus dorsi and teres
major.
 Enters the arm between the brachial artery and the long head of triceps (medial to humerus).
 Spirals around the posterior surface of the humerus in the groove for the radial nerve.
 At the distal third of the lateral border of the humerus it then pierces the intermuscular
septum and descends in front of the lateral epicondyle.
 At the lateral epicondyle it lies deeply between brachialis and brachioradialis where it then
divides into a superficial and deep terminal branch.
 Deep branch crosses the supinator to become the posterior interosseous nerve.

In the image below the relationships of the radial nerve can be appreciated

Image sourced from Wikipedia

Regions innervated

 Triceps
 Anconeus
Motor (main nerve)  Brachioradialis
 Extensor carpi radialis

 Supinator
Motor (posterior
 Extensor carpi ulnaris
interosseous branch)
 Extensor digitorum
 Extensor indicis
 Extensor digiti minimi
 Extensor pollicis longus and brevis
 Abductor pollicis longus

The area of skin supplying the proximal phalanges on the dorsal aspect of the
Sensory hand is supplied by the radial nerve (this does not apply to the little finger and
part of the ring finger)

Muscular innervation and effect of denervation

Anatomical
location Muscle affected Effect of paralysis

Shoulder Long head of triceps Minor effects on shoulder stability in abduction

Arm Triceps Loss of elbow extension

Forearm Supinator Weakening of supination of prone hand and elbow


Brachioradialis flexion in mid prone position
Extensor carpi radialis longus
and brevis

The cutaneous sensation of the upper limb- illustrating the contribution of the radial nerve
Image sourced from Wikipedia

Next question
Question 143-145 of 560

Theme: Nerve injury

A. Median nerve

B. Ulnar nerve

C. Radial nerve

D. Posterior interosseous nerve

E. Anterior interosseous nerve

F. Musculocutaneous nerve

G. Axillary nerve

H. Brachial Trunks C5-6

I. Brachial trunks C6-7

J. Brachial Trunks C8-T1

Please select the most likely lesion site for each scenario. Each option may be used once, more
than once or not at all.

143. A 42 year old teacher is admitted with a fall. An x-ray confirms a fracture of the surgical neck of
the humerus. Which nerve is at risk?

You answered Median nerve

The correct answer is Axillary nerve

The Axillary nerve winds around the bone at the neck of the humerus. The axillary nerve is also
at risk during shoulder dislocation.

144. A 32 year old window cleaner is admitted after falling off the roof. He reports that he had
slipped off the top of the roof and was able to cling onto the gutter for a few seconds. The
patient has Horner's syndrome.

You answered Median nerve

The correct answer is Brachial Trunks C8-T1

The patient has a Klumpke's paralysis involving brachial trunks C8-T1. Classically there is
weakness of the hand intrinsic muscles. Involvement of T1 may cause a Horner's syndrome. It
occurs as a result of traction injuries or during delivery.

145. A 32 year old rugby player is hit hard on the shoulder during a rough tackle. Clinically his arm is
hanging loose on the side. It is pronated and medially rotated.

You answered Median nerve

The correct answer is Brachial Trunks C5-6

The patient has an Erb's palsy involving brachial trunks C5-6.

Please rate this question:

Discuss and give feedback

Next question

Brachial plexus

Origin Anterior rami of C5 to T1

Sections of the plexus  Roots, trunks, divisions, cords, branches


 Mnemonic:Real Teenagers Drink Cold Beer
Roots  Located in the posterior triangle
 Pass between scalenus anterior and medius

Trunks  Located posterior to middle third of clavicle


 Upper and middle trunks related superiorly to the subclavian artery
 Lower trunk passes over 1st rib posterior to the subclavian artery

Divisions Apex of axilla

Cords Related to axillary artery

Diagram illustrating the branches of the brachial plexus

Image sourced from Wikipedia

Cutaneous sensation of the upper limb


Image sourced from Wikipedia

Next question
Question 146 of 560

A 68 year old man with critical limb ischaemia is undergoing a femoro-distal bypass graft. During
mobilisation of the proximal part of the posterior tibial artery which of the following is at greatest risk
of injury?

Tibial nerve

Sciatic nerve

Saphenous nerve

Common peroneal nerve

Medial superior genicular artery

The tibial nerve is closely related to the posterior tibial artery. The tibial nerve crosses the vessel
posteriorly approximately 2.5cm distal to its origin. At its origin the nerve lies medial and then lateral
after it crosses the vessel as described.

Please rate this question:

Discuss and give feedback

Next question

Posterior tibial artery

 Larger terminal branch of the popliteal artery


 Terminates by dividing into the medial and lateral plantar arteries
 Accompanied by two veins throughout its length
 Position of the artery corresponds to a line drawn from the lower angle of the popliteal fossa,
at the level of the neck of the fibula, to a point midway between the medial malleolus and the
most prominent part of the heel
Relations of the posterior tibial artery
Proximal to distal

Anteriorly Tibialis posterior


Flexor digitorum longus
Posterior surface of tibia and ankle joint

Posterior Tibial nerve 2.5 cm distal to its origin


Fascia overlying the deep muscular layer
Proximal part covered by gastrocnemius and soleus
Distal part covered by skin and fascia

Next question
Question 147 of 560

A 67 year old man is undergoing an angiogram for gastro intestinal bleeding. The radiologist
advances the catheter into the coeliac axis. At what spinal level does this vessel typically arise from
the aorta?

T10

L3

L4

T12

None of the above

Theme from April 2014 exam


The coeliac axis lies at T12, it takes an almost horizontal angle off the aorta. It has three major
branches.
Please rate this question:

Discuss and give feedback


Next question

Abdominal aortic branches

Branches Level Paired Type

Inferior phrenic T12 (Upper border) Yes Parietal

Coeliac T12 No Visceral

Superior mesenteric L1 No Visceral


Branches Level Paired Type

Middle suprarenal L1 Yes Visceral

Renal L1-L2 Yes Visceral

Gonadal L2 Yes Visceral

Lumbar L1-L4 Yes Parietal

Inferior mesenteric L3 No Visceral

Median sacral L4 No Parietal

Common iliac L4 Yes Terminal

Next question
Question 148 of 560

Which muscle does not insert on the medial surface of the greater trochanter?

Gemelli

Obturator internus

Piriformis

Quadratus femoris

Obturator externus

Mnemonic for muscle attachment on greater trochanter is POGO:

 Piriformis
 Obturator internus
 Gemelli
 Obturator externus

The quadratus femoris fibres pass laterally to be inserted into the quadrate tubercle on the
intertrochanteric crest of the femur. The other muscles all insert on the trochanteric fossa lying
medial to the greater trochanter.
Please rate this question:

Discuss and give feedback

Next question

Gluteal region
Gluteal muscles

 Gluteus maximus: inserts to gluteal tuberosity of the femur and iliotibial tract
 Gluteus medius: attach to lateral greater trochanter
 Gluteus minimis: attach to anterior greater trochanter
 All extend and abduct the hip

Deep lateral hip rotators

 Piriformis
 Gemelli
 Obturator internus
 Quadratus femoris

Nerves

Superior gluteal nerve (L5, S1)  Gluteus medius


 Gluteus minimis
 Tensor fascia lata

Inferior gluteal nerve Gluteus maximus

Damage to the superior gluteal nerve will result in the patient developing a Trendelenberg gait.
Affected patients are unable to abduct the thigh at the hip joint. During the stance phase, the
weakened abductor muscles allow the pelvis to tilt down on the opposite side. To compensate, the
trunk lurches to the weakened side to attempt to maintain a level pelvis throughout the gait cycle.
The pelvis sags on the opposite side of the lesioned superior gluteal nerve.

Next question
Question 149 of 560

During a radical gastrectomy for carcinoma of the stomach the surgeons remove the omentum.
What is the main source of its blood supply?

Ileocolic artery

Superior mesenteric artery

Gastroepiploic artery

Middle colic artery

Inferior mesenteric artery

The vessels supplying the omentum are the omental branches of the right and left gastro-epiploic
arteries. The colonic vessels are not responsible for the arterial supply to the omentum. The left
gastro-epiploic artery is a branch of the splenic artery and the right gastro-epiploic artery is a
terminal branch of the gastroduodenal artery.

Please rate this question:

Discuss and give feedback

Next question

Omentum

 The omentum is divided into two parts which invest the stomach. Giving rise to the greater
and lesser omentum. The greater omentum is attached to the inferolateral border of the
stomach and houses the gastro-epiploic arteries.
 It is of variable size but is less well developed in children. This is important as the omentum
confers protection against visceral perforation (e.g. Appendicitis).
 Inferiorly between the omentum and transverse colon is one potential entry point into the
lesser sac.
 Several malignant processes may involve the omentum of which ovarian cancer is the most
notable.

Next question
Question 150 of 560

A 38 year old lady is due to undergo a parathyroidectomy for hyperparathyroidism. At operation the
inferior parathyroid gland is identified as being enlarged. A vessel is located adjacent to the gland
laterally. This vessel is most likely to be the:

External carotid artery

Common carotid artery

Internal carotid artery

External jugular vein

None of the above

The common carotid artery is a lateral relation of the inferior parathyroid.

Please rate this question:

Discuss and give feedback

Next question

Parathyroid glands- anatomy

 Four parathyroid glands


 Located posterior to the thyroid gland
 They lie within the pretracheal fascia

Embryology
The parathyroids develop from the extremities of the third and fourth pharyngeal pouches. The
parathyroids derived from the fourth pharyngeal pouch are located more superiorly and are
associated with the thyroid gland. Those derived from the third pharyngeal pouch lie more inferiorly
and may become associated with the thymus.

Blood supply
The blood supply to the parathyroid glands is derived from the inferior and superior thyroid
arteries[1]. There is a rich anastomosis between the two vessels. Venous drainage is into the thyroid
veins.

Relations

Laterally Common carotid

Medially Recurrent laryngeal nerve, trachea

Anterior Thyroid

Posterior Pretracheal fascia

References
1.Nobori, M., et al., Blood supply of the parathyroid gland from the superior thyroid artery. Surgery,
1994. 115(4): p. 417-23.

Next question
Question 151 of 560

A 45 year old man has a long femoral line inserted to provide CVP measurements. The catheter
passes from the common iliac vein into the inferior vena cava. At which of the following vertebral
levels will this occur?

L5

L4

S1

L3

L2

The common iliac veins fuse with the IVC at L5.


Please rate this question:

Discuss and give feedback


Next question

Inferior vena cava

Origin

 L5

Path

 Left and right common iliac veins merge to form the IVC.
 Passes right of midline
 Paired segmental lumbar veins drain into the IVC throughout its length
 The right gonadal vein empties directly into the cava and the left gonadal vein generally
empties into the left renal vein.
 The next major veins are the renal veins and the hepatic veins
 Pierces the central tendon of diaphragm at T8
 Right atrium
Image sourced from Wikipedia

Relations
Anteriorly Small bowel, first and third part of duodenum, head of pancreas, liver and bile duct,
right common iliac artery, right gonadal artery

Posteriorly Right renal artery, right psoas, right sympathetic chain, coeliac ganglion

Levels
Level Vein

T8 Hepatic vein, inferior phrenic vein, pierces diaphragm

L1 Suprarenal veins, renal vein

L2 Gonadal vein

L1-5 Lumbar veins


L5 Common iliac vein, formation of IVC

Next question
Question 152 of 560

Following a carotid endarterectomy a man notices that he has a weakness of his tongue. Damage to
which of the following nerves is the most likely explanation for this process?

Hypoglossal

Accessory

Ansa cervicalis

Vagus

Cervical plexus

Theme from April 2013


The hypoglossal nerve innervates the tongue and is one of the structures more commonly at risk in
carotid surgery.
Please rate this question:

Discuss and give feedback


Next question

Internal carotid artery

The internal carotid artery is formed from the common carotid opposite the upper border of the
thyroid cartilage. It extends superiorly to enter the skull via the carotid canal. From the carotid canal
it then passes through the cavernous sinus, above which it divides into the anterior and middle
cerebral arteries.

Relations in the neck


Posterior  Longus capitis
 Pre-vertebral fascia
 Sympathetic chain
 Superior laryngeal nerve

Medially  External carotid (near origin)


 Wall of pharynx
 Ascending pharyngeal artery

Laterally  Internal jugular vein (moves posteriorly at entrance to skull)


 Vagus nerve (most posterolaterally)

Anteriorly  Sternocleidomastoid
 Lingual and facial veins
 Hypoglossal nerve

Relations in the carotid canal

 Internal carotid plexus


 Cochlea and middle ear cavity
 Trigeminal ganglion (superiorly)
 Leaves canal lies above the foramen lacerum

Path and relations in the cranial cavity


The artery bends sharply forwards in the cavernous sinus, the aducens nerve lies close to its
inferolateral aspect. The oculomotor, trochlear, opthalmic and, usually, the maxillary nerves lie in the
lateral wall of the sinus. Near the superior orbital fissure it turns posteriorly and passes postero-
medially to pierce the roof of the cavernous sinus inferior to the optic nerve. It then passes between
the optic and oculomotor nerves to terminate below the anterior perforated substance by dividing
into the anterior and middle cerebral arteries.

Branches

 Anterior and middle cerebral artery


 Ophthalmic artery
 Posterior communicating artery
 Anterior choroid artery
 Meningeal arteries
 Hypophyseal arteries

Image demonstrating the internal carotid artery and its relationship to the external carotid artery
Image sourced from Wikipedia

Next question
Question 153 of 560

At which of the following levels does the inferior vena cava exit the abdominal cavity?

T6

T7

T10

T8

T12

Theme from April 2012 exam


Theme from April 2014 exam
Please rate this question:

Discuss and give feedback


Next question

Levels

Transpyloric plane
Level of the body of L1

 Pylorus stomach
 Left kidney hilum (L1- left one!)
 Fundus of the gallbladder
 Neck of pancreas
 Duodenojejunal flexure
 Superior mesenteric artery
 Portal vein
 Left and right colic flexure
 Root of the transverse mesocolon
 2nd part of the duodenum
 Upper part of conus medullaris
 Spleen
Can be identified by asking the supine patient to sit up without using their arms. The plane is located
where the lateral border of the rectus muscle crosses the costal margin.

Anatomical planes
Subcostal plane Lowest margin of 10th costal cartilage

Intercristal plane Level of body L4 (highest point of iliac crest)

Intertubercular plane Level of body L5

Common level landmarks


Inferior mesenteric artery L3

Bifurcation of aorta into common iliac arteries L4

Formation of IVC L5 (union of common iliac veins)

Diaphragm apertures  Vena cava T8


 Oesophagus T10
 Aortic hiatus T12

Next question
Question 154 of 560

Which of the following structures lies deepest in the popliteal fossa?

Popliteal artery

Popliteal vein

Tibial nerve

Common peroneal nerve

Popliteal lymph nodes

Theme from January 2012 exam


Theme from April 2014 exam

From superficial to deep:


The common peroneal nerve exits the popliteal fossa along the medial border of the biceps tendon.
Then the tibial nerve lies lateral to the popliteal vessels to pass posteriorly and then medially to
them. The popliteal vein lies superficial to the popliteal artery, which is the deepest structure in the
fossa.
Please rate this question:

Discuss and give feedback


Next question

Popliteal fossa

Boundaries of the popliteal fossa


Laterally Biceps femoris above, lateral head of gastrocnemius and plantaris below

Medially Semimembranosus and semitendinosus above, medial head of gastrocnemius below

Floor Popliteal surface of the femur, posterior ligament of knee joint and popliteus muscle
Roof Superficial and deep fascia

Image showing the popliteal fossa

© Image provided by the University of Sheffield

Contents

 Popliteal artery and vein


 Small saphenous vein
 Common peroneal nerve
 Tibial nerve
 Posterior cutaneous nerve of the thigh
 Genicular branch of the obturator nerve
 Lymph nodes

Next question
Question 155 of 560

Which of the following nerves is responsible for innervation of the triceps muscle?

Radial

Ulnar

Axillary

Median

None of the above

To remember nerve roots and their reflexes:

1-2 Ankle (S1-S2)


3-4 Knee (L3-L4)
5-6 Biceps (C5-C6)
7-8 Triceps (C7-C8)

The radial nerve innervates all three heads of triceps, with a separate branch to each head.
Please rate this question:

Discuss and give feedback


Next question

Triceps

Origin  Long head- infraglenoid tubercle of the scapula.


 Lateral head- dorsal surface of the humerus, lateral and proximal to the
groove of the radial nerve
 Medial head- posterior surface of the humerus on the inferomedial side of
the radial groove and both of the intermuscular septae

Insertion  Olecranon process of the ulna. Here the olecranon bursa is between the
triceps tendon and olecranon.
 Some fibres insert to the deep fascia of the forearm, posterior capsule of the
elbow (preventing the capsule from being trapped between olecranon and
olecranon fossa during extension)

Innervation Radial nerve

Blood Profunda brachii artery


supply

Action Elbow extension. The long head can adduct the humerus and and extend it from a
flexed position

Relations The radial nerve and profunda brachii vessels lie between the lateral and medial
heads
Next question
Question 156 of 560

Which of the positions listed below best describes the location of the coeliac autonomic plexus?

Anterolateral to the aorta

Posterolateral to the aorta

Anterolateral to the sympathetic chain

Anteromedial to the sympathetic chain

Posterior to L1

Please rate this question:

Discuss and give feedback

Next question

Coeliac plexus

The coeliac plexus is the largest of the autonomic plexuses. It is located on a level of the last
thoracic and first lumbar vertebrae. It surrounds the coeliac axis and the SMA. It lies posterior to the
stomach and the lesser sac. It lies anterior to the crura of the diaphragm and the aorta. The plexus
and ganglia are joined are joined by the greater and lesser splanchnic nerves on both sides and
branches from both the vagus and phrenic nerves.
Image sourced from Wikipedia

Next question
Question 157 of 560

An intravenous drug user develops a false aneurysm and requires emergency surgery. The
procedure is difficult and the femoral nerve is inadvertently transected. Which of the following
muscles is least likely to be affected as a result?

Sartorius

Vastus medialis

Pectineus

Quadriceps femoris

Adductor magnus

Mnemonic for femoral nerve supply

(don't) M I S V Q Scan for PE


M edial cutaneous nerve of the thigh
I ntermediate cutaneous nerve of the thigh
S aphenous nerve

V astus
Q uadriceps femoris
S artorius

PE ectineus

Adductor magnus is innervated by the obturator and sciatic nerve. The pectineus muscle is
sometimes supplied by the obturator nerve but this is variable. Since the question states least likely,
the correct answer is adductor magnus
Please rate this question:

Discuss and give feedback


Next question

Femoral nerve
Root values L2, 3, 4

Innervates  Pectineus
 Sartorius
 Quadriceps femoris
 Vastus lateralis/medialis/intermedius

Branches  Medial cutaneous nerve of thigh


 Saphenous nerve
 Intermediate cutaneous nerve of thigh

Path
Penetrates psoas major and exits the pelvis by passing under the inguinal ligament to enter the
femoral triangle, lateral to the femoral artery and vein.

Image sourced from Wikipedia


Mnemonic for femoral nerve supply

(don't) M I S V Q Scan for PE


M edial cutaneous nerve of the thigh
I ntermediate cutaneous nerve of the thigh
S aphenous nerve

V astus
Q uadriceps femoris
S artorius

PE ectineus

Next question
Question 158 of 560

What is the nerve root value of the external urethral sphincter?

S4

S1, S2, S3

S2, S3, S4

L3, L4, L5

L5, S1, S2

Theme from April 2012 exam


Theme from April 2013 exam
Theme from April 2014 exam
The external urethral sphincter is innervated by branches of the pudendal nerve, therefore the root
values are S2, S3, S4.

Please rate this question:

Discuss and give feedback

Next question

Urethral anatomy

Female urethra
The female urethra is shorter and more acutely angulated than the male urethra. It is an extra-
peritoneal structure and embedded in the endopelvic fascia. The neck of the bladder is subjected to
transmitted intra-abdominal pressure and therefore deficiency in this area may result in stress
urinary incontinence. Between the layers of the urogenital diaphragm the female urethra is
surrounded by the external urethral sphincter, this is innervated by the pudendal nerve. It ultimately
lies anterior to the vaginal orifice.

Male urethra
In males the urethra is much longer and is divided into four parts.

Pre-prostatic Extremely short and lies between the bladder and prostate gland.It has a stellate lumen
urethra and is between 1 and 1.5cm long.Innervated by sympathetic noradrenergic fibres, as
this region is composed of striated muscles bundles they may contract and prevent
retrograde ejaculation.

Prostatic This segment is wider than the membranous urethra and contains several openings for
urethra the transmission of semen (at the midpoint of the urethral crest).

Membranous Narrowest part of the urethra and surrounded by external sphincter. It traverses the
urethra perineal membrane 2.5cm postero-inferior to the symphysis pubis.

Penile urethra Travels through the corpus spongiosum on the underside of the penis. It is the longest
urethral segment.It is dilated at its origin as the infrabulbar fossa and again in the gland
penis as the navicular fossa. The bulbo-urethral glands open into the spongiose section
of the urethra 2.5cm below the perineal membrane.

The urothelium is transitional in nature near to the bladder and becomes squamous more distally.

Next question
Question 159 of 560

A 45 year old man is stabbed in the abdomen and the inferior vena cava is injured. How many
functional valves does this vessel usually have?

Mnemonic for the Inferior vena cava tributaries: I Like To Rise So High:

Iliacs
Lumbar
Testicular
Renal
Suprarenal
Hepatic vein

The lack of valves in the IVC is important clinically when it is cannulated during cardiopulmonary
bypass, using separate SVC and IVC catheters, such as when the right atrium is to be opened. Note
that there is a non functional valve between the right atrium and inferior vena cava.
Please rate this question:

Discuss and give feedback


Next question

Inferior vena cava

Origin

 L5
Path

 Left and right common iliac veins merge to form the IVC.
 Passes right of midline
 Paired segmental lumbar veins drain into the IVC throughout its length
 The right gonadal vein empties directly into the cava and the left gonadal vein generally
empties into the left renal vein.
 The next major veins are the renal veins and the hepatic veins
 Pierces the central tendon of diaphragm at T8
 Right atrium

Image sourced from Wikipedia

Relations
Anteriorly Small bowel, first and third part of duodenum, head of pancreas, liver and bile duct,
right common iliac artery, right gonadal artery

Posteriorly Right renal artery, right psoas, right sympathetic chain, coeliac ganglion

Levels
Level Vein
T8 Hepatic vein, inferior phrenic vein, pierces diaphragm

L1 Suprarenal veins, renal vein

L2 Gonadal vein

L1-5 Lumbar veins

L5 Common iliac vein, formation of IVC

Next question
Question 160 of 560

Which of the following structures does not pass posteriorly to the medial malleolus?

Posterior tibial artery

Tibial nerve

Tibialis anterior tendon

Tendon of flexor digitorum longus

Tendon of flexor hallucis longus

Mnemonic for structures posterior to the medial malleolus:

Tom Dick And Nervous Harry

T ibialis posterior tendon


flexor Digitorum longus
A rtery
N erve
H allucis longus

Theme from April 2014 exam


Please rate this question:

Discuss and give feedback


Next question

Medial malleolus

The following structures pass posterior to medial malleolus (in order):

 Tibialis posterior tendon


 Flexor digitorum longus tendon
 Posterior tibial artery
 Tibial nerve
 Tendon of flexor hallucis longus
Question 161 of 560

Which of the following statements relating to the root of the neck is false?

The lung projects into the neck beyond the first rib and is constrained by Sibson's fascia

The subclavian artery arches over the first rib anterior to scalenus anterior

The roots and trunks of the Brachial plexus lie posterior to the subclavian artery on the
first rib

The roots and trunks of the Brachial plexus lie between scalenus anterior and scalenus
medius muscles

The thyrocervical trunk is a branch of the subclavian artery

The subclavian artery lies posterior to scalenus anterior, the vein lies in front. Sibson's fascia is
another name for the suprapleural membrane.
Please rate this question:

Discuss and give feedback


Next question

Root of the neck

Thoracic Outlet

 Where the subclavian artery and vein and the brachial plexus exit the thorax and enter the
arm.
 They pass over the 1st rib and under the clavicle.
 The subclavian vein is the most anterior structure and is immediately anterior to scalenus
anterior and its attachment to the first rib.
 Scalenus anterior has 2 parts, the subclavian artery leaves the thorax by passing over the
first rib and between these 2 portions of the muscle.
 At the level of the first rib, the lower cervical nerve roots combine to form the 3 trunks of the
brachial plexus. The lowest trunk is formed by the union of C8 and T1, and this trunk lies
directly posterior to the artery and is in contact with the superior surface of the first rib.
Thoracic outlet obstruction causes neurovascular compromise.
Next question
Question 162 of 560

A patient presents to the clinic following a surgical procedure. She complains that she is unable to
shrug her shoulder. What is the most likely underlying nerve injury?

Accessory nerve

Cervical plexus

Ansa cervicalis

Long thoracic nerve

Axillary nerve

Theme from April 2013 exam


Theme from April 2014 exam
The accessory nerve may be injured in operations in the posterior triangle. Injury will affect
trapezius.

Please rate this question:

Discuss and give feedback

Next question

Cranial nerves

Cranial nerve lesions

Olfactory nerve May be injured in basal skull fractures or involved in frontal lobe tumour extension.
Loss of olfactory nerve function in relation to major CNS pathology is seldom an
isolated event and thus it is poor localiser of CNS pathology.

Optic nerve Problems with visual acuity may result from intra ocular disorders. Problems with
the blood supply such as amaurosis fugax may produce temporary visual distortion.
More important surgically is the pupillary response to light. The pupillary size may
be altered in a number of disorders. Nerves involved in the resizing of the pupil
connect to the pretectal nucleus of the high midbrain, bypassing the lateral
geniculate nucleus and the primary visual cortex. From the pretectal nucleus
neurones pass to the Edinger - Westphal nucleus, motor axons from here pass along
with the oculomotor nerve. They synapse with ciliary ganglion neurones; the
parasympathetic axons from this then innervate the iris and produce miosis. The
miotic pupil is seen in disorders such as Horner's syndrome or opiate overdose.
Mydriasis is the dilatation of the pupil in response to disease, trauma, drugs (or the
dark!). It is pathological when light fails to induce miosis. The radial muscle is
innervated by the sympathetic nervous system. Because the parasympathetic fibres
travel with the oculomotor nerve they will be damaged by lesions affecting this
nerve (e.g. cranial trauma).
The response to light shone in one eye is usually a constriction of both pupils. This
indicates intact direct and consensual light reflexes. When the optic nerve has an
afferent defect the light shining on the affected eye will produce a diminished
pupillary response in both eyes. Whereas light shone on the unaffected eye will
produce a normal pupillary response in both eyes. This is referred to as the Marcus
Gunn pupil and is seen in conditions such as optic neuritis. In a total CN II lesion
shining the light in the affected eye will produce no response.

Oculomotor nerve The pupillary effects are described above. In addition it supplies all ocular muscles
apart from lateral rectus and superior oblique. Thus the affected eye will be
deviated inferolaterally. Levator palpebrae superioris may also be impaired resulting
in impaired ability to open the eye.

Trochlear nerve The eye will not be able to look down.

Trigeminal nerve Largest cranial nerve. Exits the brainstem at the pons. Branches are ophthalmic,
maxillary and mandibular. Only the mandibular branch has both sensory and motor
fibres. Branches converge to form the trigeminal ganglion (located in Meckels cave).
It supplies the muscles of mastication and also tensor veli palatine, mylohyoid,
anterior belly of digastric and tensor tympani. The detailed descriptions of the
various sensory functions are described in other areas of the website. The corneal
reflex is important and is elicited by applying a small tip of cotton wool to the
cornea, a reflex blink should occur if it is intact. It is mediated by: the naso ciliary
branch of the ophthalmic branch of the trigeminal (sensory component) and the
facial nerve producing the motor response. Lesions of the afferent arc will produce
bilateral absent blink and lesions of the efferent arc will result in a unilateral absent
blink.

Abducens nerve The affected eye will have a deficit of abduction. This cranial nerve exits the
brainstem between the pons and medulla. It thus has a relatively long intra cranial
course which renders it susceptible to damage in raised intra cranial pressure.

Facial nerve Emerges from brainstem between pons and medulla. It controls muscles of facial
expression and taste from the anterior 2/3 of the tongue. The nerve passes into the
petrous temporal bone and into the internal auditory meatus. It then passes
through the facial canal and exits at the stylomastoid foramen. It passes through the
parotid gland and divides at this point. It does not innervate the parotid gland. Its
divisions are considered in other parts of the website. Its motor fibres innervate
orbicularis oculi to produce the efferent arm of the corneal reflex. In surgical
practice it may be injured during parotid gland surgery or invaded by malignancies
of the gland and a lower motor neurone on the ipsilateral side will result.

Vestibulo-cochlear Exits from the pons and then passes through the internal auditory meatus. It is
nerve implicated in sensorineural hearing loss. Individuals with sensorineural hearing loss
will localise the sound in webers test to the normal ear. Rinnes test will be reduced
on the affected side but should still work. These two tests will distinguish
sensorineural hearing loss from conductive deafness. In the latter condition webers
test will localise to the affected ear and Rinnes test will be impaired on the affected
side. Surgical lesions affecting this nerve include CNS tumours and basal skull
fractures. It may also be damaged by the administration of ototoxic drugs (of which
gentamicin is the most commonly used in surgical practice).

Glossopharyngeal Exits the pons just above the vagus. Receives sensory fibres from posterior 1/3
nerve tongue, tonsils, pharynx and middle ear (otalgia may occur following tonsillectomy).
It receives visceral afferents from the carotid bodies. It supplies parasympathetic
fibres to the parotid gland via the otic ganglion and motor function to
stylopharyngeaus muscle. The sensory function of the nerve is tested using the gag
reflex.

Vagus nerve Leaves the medulla between the olivary nucleus and the inferior cerebellar
peduncle. Passes through the jugular foramen and into the carotid sheath. Details of
the functions of the vagus nerve are covered in the website under relevant organ
sub headings.

Accessory nerve Exists from the caudal aspect of the brainstem (multiple branches) supplies
trapezius and sternocleidomastoid muscles. The distal portion of this nerve is most
prone to injury during surgical procedures.

Hypoglossal nerve Emerges from the medulla at the preolivary sulcus, passes through the hypoglossal
canal. It lies on the carotid sheath and passes deep to the posterior belly of digastric
to supply muscles of the tongue (except palatoglossus). Its location near the carotid
sheath makes it vulnerable during carotid endarterectomy surgery and damage will
produce ipsilateral defect in muscle function.

Next question
Question 163 of 560

Which of the following are not generally supplied by the right coronary artery?

The sino atrial node

The circumflex artery

The atrioventricular node

Most of the right ventricle

The right atrium

The circumflex artery is generally a branch of the left coronary artery.


Please rate this question:

Discuss and give feedback


Next question

Heart anatomy

The walls of each cardiac chamber comprise:

 Epicardium
 Myocardium
 Endocardium

Cardiac muscle is attached to the cardiac fibrous skeleton.

Relations
The heart and roots of the great vessels within the pericardial sac are related anteriorly to the
sternum, medial ends of the 3rd to 5th ribs on the left and their associated costal cartilages. The
heart and pericardial sac are situated obliquely two thirds to the left and one third to the right of the
median plane.

The pulmonary valve lies at the level of the left third costal cartilage.
The mitral valve lies at the level of the fourth costal cartilage.
Coronary sinus
This lies in the posterior part of the coronary groove and receives blood from the cardiac veins. The
great cardiac vein lies at its left and the middle and small cardiac veins lie on its right. The smallest
cardiac vein (anterior cardiac vein) drains into the right atrium directly.

Aortic sinus
Right coronary artery arises from the right aortic sinus, the left is derived from the left aortic sinus,
which lies posteriorly.

Right and left ventricles

Structure Left Ventricle

A-V Valve Mitral (double leaflet)

Walls Twice as thick as right

Trabeculae carnae Much thicker and more numerous

Right coronary artery


The RCA supplies:

 Right atrium
 Diaphragmatic part of the left ventricle
 Usually the posterior third of the interventricular septum
 The sino atrial node (60% cases)
 The atrio ventricular node (80% cases)

Left coronary artery


The LCA supplies:

 Left atrium
 Most of left ventricle
 Part of the right ventricle
 Anterior two thirds of the inter ventricular septum
 The sino atrial node (remaining 40% cases)

Innervation of the heart


Autonomic nerve fibres from the superficial and deep cardiac plexus. These lie anterior to the
bifurcation of the trachea, posterior to the ascending aorta and superior to the bifurcation of the
pulmonary trunk. The parasympathetic supply to the heart is from presynaptic fibres of the vagus
nerves.
Valves of the heart
Mitral valve Aortic valve Pulmonary valve Tricuspid valve

2 cusps 3 cusps 3 cusps 3 cusps

First heart sound Second heart Second heart First heart sound
sound sound

1 anterior cusp 2 anterior cusps 2 anterior cusps 2 anterior cusps

Attached to chordae No chordae No chordae Attached to chordae


tendinae tendinae

Next question
Question 164 of 560

A 44 year old man has a malignant melanoma and is undergoing a block dissection of the groin. The
femoral triangle is being explored for intra operative bleeding. Which of the following forms the
medial border of the femoral triangle?

Femoral artery

Biceps femoris

Adductor longus

Sartorius

Adductor magnus

Vastus medialis forms the lateral border of the adductor canal. The sartorius muscles forms the roof
of the adductor canal.

Adductor longus forms the medial boundary of the femoral triangle (see below).
Please rate this question:

Discuss and give feedback


Next question

Femoral triangle anatomy

Boundaries
Superiorly Inguinal ligament

Laterally Sartorius

Medially Adductor longus

Floor Iliopsoas, adductor longus and pectineus


Roof  Fascia lata and Superficial fascia
 Superficial inguinal lymph nodes (palpable below the inguinal ligament)
 Long saphenous vein

Image sourced from Wikipedia

Contents

 Femoral vein (medial to lateral)


 Femoral artery-pulse palpated at the mid inguinal point
 Femoral nerve
 Deep and superficial inguinal lymph nodes
 Lateral cutaneous nerve
 Great saphenous vein
 Femoral branch of the genitofemoral nerve

Next question
Question 165 of 560

The foramen marking the termination of the adductor canal is located in which of the following?

Adductor longus

Adductor magnus

Adductor brevis

Sartorius

Semimembranosus

The foramen marking the distal limit of the adductor canal is contained within adductor magnus. The
vessel passes through this region to enter the popliteal fossa.
Please rate this question:

Discuss and give feedback


Next question

Adductor canal

 Also called Hunter's or subsartorial canal


 Immediately distal to the apex of the femoral triangle, lying in the middle third of the thigh.
Canal terminates at the adductor hiatus.

Borders Contents

Laterally Vastus medialis muscle Saphenous nerve

Posteriorly Adductor longus, adductor magnus Superficial femoral artery


Borders Contents

Roof Sartorius Superficial femoral vein

In the image below the sartorius muscle is removed to expose the canal contents

Image sourced from Wikipedia

Next question
Question 166 of 560

Which of the following is the first vessel to branch from the external carotid artery?

Superior thyroid artery

Inferior thyroid artery

Lingual artery

Facial artery

Occipital artery

Mnemonic
(Order in which they branch off)Some (sup thyroid)Attendings (Ascending Pharyngeal)Like
(Lingual)Freaking (Facial)Out (Occipital)Potential (Post auricular)Medical (Maxillary)Students (Sup
temporal)

The first branch of the external carotid artery is the superior thyroid artery. The inferior thyroid artery
is derived from the thyrocervical trunk. The other branches are illustrated below.

Please rate this question:

Discuss and give feedback

Next question

External carotid artery

The external carotid commences immediately lateral to the pharyngeal side wall. It ascends and lies
anterior to the internal carotid and posterior to the posterior belly of digastric and stylohyoid. More
inferiorly it is covered by sternocleidomastoid, passed by hypoglossal nerves, lingual and facial
veins.
It then pierces the fascia of the parotid gland finally dividing into its terminal branches within the
gland itself.

Surface marking of the carotid


This is an imaginary line drawn from the bifurcation of the common carotid passing behind the angle
of the jaw to a point immediately anterior to the tragus of the ear.

Branches of the external carotid artery


It has six branches, three in front, two behind and one deep.

Three in front Superior thyroid


Lingual
Facial

Two behind Occipital


Posterior auricular

Deep Ascending pharyngeal

It terminates by dividing into the superficial temporal and maxillary arteries in the parotid gland.
Image sourced from Wikipedia

Next question
Question 167 of 560

A motorcyclist is injured in a road traffic accident and is not wearing a helmet. He suffers a severe
closed head injury and develops raised intracranial pressure. The first cranial nerve to be affected by
this process is likely to be:

Oculomotor

Hypoglossal

Motor branch of the trigeminal

Abducens

Sensory branch of the trigeminal

Theme from April 2014 exam


The abducens nerve (CN VI) has a long intra cranial course and is thus susceptible to raised intra
cranial pressure. It also passes over the petrous temporal bone and 6th nerve palsies are also seen
in mastoiditis.

Please rate this question:

Discuss and give feedback

Next question

Cranial nerves

Cranial nerve lesions

Olfactory nerve May be injured in basal skull fractures or involved in frontal lobe tumour extension.
Loss of olfactory nerve function in relation to major CNS pathology is seldom an
isolated event and thus it is poor localiser of CNS pathology.

Optic nerve Problems with visual acuity may result from intra ocular disorders. Problems with
the blood supply such as amaurosis fugax may produce temporary visual distortion.
More important surgically is the pupillary response to light. The pupillary size may
be altered in a number of disorders. Nerves involved in the resizing of the pupil
connect to the pretectal nucleus of the high midbrain, bypassing the lateral
geniculate nucleus and the primary visual cortex. From the pretectal nucleus
neurones pass to the Edinger - Westphal nucleus, motor axons from here pass along
with the oculomotor nerve. They synapse with ciliary ganglion neurones; the
parasympathetic axons from this then innervate the iris and produce miosis. The
miotic pupil is seen in disorders such as Horner's syndrome or opiate overdose.
Mydriasis is the dilatation of the pupil in response to disease, trauma, drugs (or the
dark!). It is pathological when light fails to induce miosis. The radial muscle is
innervated by the sympathetic nervous system. Because the parasympathetic fibres
travel with the oculomotor nerve they will be damaged by lesions affecting this
nerve (e.g. cranial trauma).
The response to light shone in one eye is usually a constriction of both pupils. This
indicates intact direct and consensual light reflexes. When the optic nerve has an
afferent defect the light shining on the affected eye will produce a diminished
pupillary response in both eyes. Whereas light shone on the unaffected eye will
produce a normal pupillary response in both eyes. This is referred to as the Marcus
Gunn pupil and is seen in conditions such as optic neuritis. In a total CN II lesion
shining the light in the affected eye will produce no response.

Oculomotor nerve The pupillary effects are described above. In addition it supplies all ocular muscles
apart from lateral rectus and superior oblique. Thus the affected eye will be
deviated inferolaterally. Levator palpebrae superioris may also be impaired resulting
in impaired ability to open the eye.

Trochlear nerve The eye will not be able to look down.

Trigeminal nerve Largest cranial nerve. Exits the brainstem at the pons. Branches are ophthalmic,
maxillary and mandibular. Only the mandibular branch has both sensory and motor
fibres. Branches converge to form the trigeminal ganglion (located in Meckels cave).
It supplies the muscles of mastication and also tensor veli palatine, mylohyoid,
anterior belly of digastric and tensor tympani. The detailed descriptions of the
various sensory functions are described in other areas of the website. The corneal
reflex is important and is elicited by applying a small tip of cotton wool to the
cornea, a reflex blink should occur if it is intact. It is mediated by: the naso ciliary
branch of the ophthalmic branch of the trigeminal (sensory component) and the
facial nerve producing the motor response. Lesions of the afferent arc will produce
bilateral absent blink and lesions of the efferent arc will result in a unilateral absent
blink.

Abducens nerve The affected eye will have a deficit of abduction. This cranial nerve exits the
brainstem between the pons and medulla. It thus has a relatively long intra cranial
course which renders it susceptible to damage in raised intra cranial pressure.

Facial nerve Emerges from brainstem between pons and medulla. It controls muscles of facial
expression and taste from the anterior 2/3 of the tongue. The nerve passes into the
petrous temporal bone and into the internal auditory meatus. It then passes
through the facial canal and exits at the stylomastoid foramen. It passes through the
parotid gland and divides at this point. It does not innervate the parotid gland. Its
divisions are considered in other parts of the website. Its motor fibres innervate
orbicularis oculi to produce the efferent arm of the corneal reflex. In surgical
practice it may be injured during parotid gland surgery or invaded by malignancies
of the gland and a lower motor neurone on the ipsilateral side will result.

Vestibulo-cochlear Exits from the pons and then passes through the internal auditory meatus. It is
nerve implicated in sensorineural hearing loss. Individuals with sensorineural hearing loss
will localise the sound in webers test to the normal ear. Rinnes test will be reduced
on the affected side but should still work. These two tests will distinguish
sensorineural hearing loss from conductive deafness. In the latter condition webers
test will localise to the affected ear and Rinnes test will be impaired on the affected
side. Surgical lesions affecting this nerve include CNS tumours and basal skull
fractures. It may also be damaged by the administration of ototoxic drugs (of which
gentamicin is the most commonly used in surgical practice).

Glossopharyngeal Exits the pons just above the vagus. Receives sensory fibres from posterior 1/3
nerve tongue, tonsils, pharynx and middle ear (otalgia may occur following tonsillectomy).
It receives visceral afferents from the carotid bodies. It supplies parasympathetic
fibres to the parotid gland via the otic ganglion and motor function to
stylopharyngeaus muscle. The sensory function of the nerve is tested using the gag
reflex.

Vagus nerve Leaves the medulla between the olivary nucleus and the inferior cerebellar
peduncle. Passes through the jugular foramen and into the carotid sheath. Details of
the functions of the vagus nerve are covered in the website under relevant organ
sub headings.

Accessory nerve Exists from the caudal aspect of the brainstem (multiple branches) supplies
trapezius and sternocleidomastoid muscles. The distal portion of this nerve is most
prone to injury during surgical procedures.

Hypoglossal nerve Emerges from the medulla at the preolivary sulcus, passes through the hypoglossal
canal. It lies on the carotid sheath and passes deep to the posterior belly of digastric
to supply muscles of the tongue (except palatoglossus). Its location near the carotid
sheath makes it vulnerable during carotid endarterectomy surgery and damage will
produce ipsilateral defect in muscle function.

Next question
Question 168 of 560

A 32 year old man is undergoing a splenectomy. Division of which of the following will be necessary
during the procedure?

Left crus of diaphragm

Short gastric vessels

Gerotas fascia

Splenic flexure of colon

Marginal artery

Theme from 2011 Exam


During a splenectomy the short gastric vessels which lie within the gastrosplenic ligament will need
to be divided. The splenic flexure of the colon may need to be mobilised. However, it will almost
never need to be divided, as this is watershed area that would necessitate a formal colonic resection
in the event of division.
Please rate this question:

Discuss and give feedback


Next question

Splenic anatomy

The spleen is the largest lymphoid organ in the body. It is an intraperitoneal organ, the peritoneal
attachments condense at the hilum where the vessels enter the spleen. Its blood supply is from the
splenic artery (derived from the coeliac axis) and the splenic vein (which is joined by the IMV and
unites with the SMV).

 Embryology: derived from mesenchymal tissue


 Shape: clenched fist
 Position: below 9th-12th ribs
 Weight: 75-150g
Relations

 Superiorly- diaphragm
 Anteriorly- gastric impression
 Posteriorly- kidney
 Inferiorly- colon
 Hilum: tail of pancreas and splenic vessels
 Forms apex of lesser sac (containing short gastric vessels)

Next question
Question 169 of 560

Which structure separates the cephalic vein and the brachial artery in the antecubital fossa?

Brachioradialis muscle

Biceps muscle

Origin of flexor digitorum profundus muscle

Pronator quadratus muscle

Origin of flexor digitorum superficialis muscle

Please rate this question:

Discuss and give feedback

Next question

Antecubital fossa
© Image provided by the University of Sheffield

Next question
Question 170 of 560

A 24 year old motor cyclist is involved in a road traffic accident. He suffers a tibial fracture which is
treated with an intra medullary nail. Post operatively he develops a compartment syndrome. Surgical
decompression of the anterior compartment will relieve pressure on all of the following muscles
except?

Peroneus brevis

Peroneus tertius

Extensor digitorum longus

Tibialis anterior

None of the above

The anterior compartment contains:


Tibialis anterior
Extensor digitorum longus
Peroneus tertius
Extensor hallucis longus
Anterior tibial artery
All the muscles are innervated by the deep peroneal nerve.

Please rate this question:

Discuss and give feedback

Next question

Lower limb- Muscular compartments

Anterior compartment
Muscle Nerve Action

Tibialis anterior Deep peroneal nerve Dorsiflexes ankle joint, inverts foot

Extensor digitorum longus Deep peroneal nerve Extends lateral four toes, dorsiflexes ankle joint

Peroneus tertius Deep peroneal nerve Dorsiflexes ankle, everts foot

Extensor hallucis longus Deep peroneal nerve Dorsiflexes ankle joint, extends big toe

Peroneal compartment

Muscle Nerve Action

Peroneus longus Superficial peroneal nerve Everts foot, assists in plantar flexion

Peroneus brevis Superficial peroneal nerve Plantar flexes the ankle joint

Superficial posterior compartment

<muscle< b="" style="box-sizing: border-


box;"></muscle<> Nerve Action

Gastrocnemius Tibial Plantar flexes the foot, may also flex


nerve the knee

Soleus Tibial Plantar flexor


nerve

Deep posterior compartment


Muscle Nerve Action

Flexor digitorum longus Tibial Flexes the lateral four toes

Flexor hallucis longus Tibial Flexes the great toe

Tibialis posterior Tibial Plantar flexor, inverts the foot

Next question
Question 171 of 560

A 43 year old lady underwent an attempted placement of a central line into the internal jugular vein.
Unfortunately, the doctor damaged the carotid artery and this necessitated surgical exploration. As
the surgeons incise the carotid sheath a nerve is identified lying between the internal jugular vein
and the carotid artery. Which of the following is this nerve most likely to be?

Glossopharyngeal nerve

Hypoglossal nerve

Superior laryngeal nerve

Recurrent laryngeal nerve

Vagus

The vagus lies in the carotid sheath. The hypoglossal nerve crosses the sheath, but does not lie
within it.
Please rate this question:

Discuss and give feedback


Next question

Common carotid artery

The right common carotid artery arises at the bifurcation of the brachiocephalic trunk, the left
common carotid arises from the arch of the aorta. Both terminate at the level of the upper border of
the thyroid cartilage (the lower border of the third cervical vertebra) by dividing into the internal and
external carotid arteries.

Left common carotid artery


This vessel arises immediately to the left and slightly behind the origin of the brachiocephalic trunk.
Its thoracic portion is 2.5- 3.5 cm in length and runs superolaterally to the sternoclavicular joint.

In the thorax
The vessel is in contact, from below upwards, with the trachea, left recurrent laryngeal nerve, left
margin of the oesophagus. Anteriorly the left brachiocephalic vein runs across the artery, and the
cardiac branches from the left vagus descend in front of it. These structures together with the
thymus and the anterior margins of the left lung and pleura separate the artery from the manubrium.
In the neck
The artery runs superiorly deep to sternocleidomastoid and then enters the anterior triangle. At this
point it lies within the carotid sheath with the vagus nerve and the internal jugular vein. Posteriorly
the sympathetic trunk lies between the vessel and the prevertebral fascia. At the level of C7 the
vertebral artery and thoracic duct lie behind it. The anterior tubercle of C6 transverse process is
prominent and the artery can be compressed against this structure (it corresponds to the level of the
cricoid).
Anteriorly at C6 the omohyoid muscle passes superficial to the artery.
Within the carotid sheath the jugular vein lies lateral to the artery.

Right common carotid artery


The right common carotid arises from the brachiocephalic artery. The right common carotid artery
corresponds with the cervical portion of the left common carotid, except that there is no thoracic duct
on the right. The oesophagus is less closely related to the right carotid than the left.

Summary points about the carotid anatomy

Path
Passes behind the sternoclavicular joint (12% patients above this level) to the upper border of the
thyroid cartilage, to divide into the external (ECA) and internal carotid arteries (ICA).

Relations

 Level of 6th cervical vertebra crossed by omohyoid


 Then passes deep to the thyrohyoid, sternohyoid, sternomastoid muscles.
 Passes anterior to the carotid tubercle (transverse process 6th cervical vertebra)-NB
compression here stops haemorrhage.
 The inferior thyroid artery passes posterior to the common carotid artery.
 Then : Left common carotid artery crosses the thoracic duct, Right common carotid artery
crossed by recurrent laryngeal nerve
Image sourced from Wikipedia

Next question
Question 172 of 560

A patient has a chest drain insertion. There is fresh blood at the chest drain insertion area. Which
vessel has been damaged?

Pericardiophrenic artery

Intercostal vein

Right ventricle

Vagus artery

Intercostal artery

The intercostal vein is more superior than the artery and is thus slightly less susceptible to injury.

Theme from 2009 Exam


Within the intercostal spaces there are thin, strong muscles, intercostal vessels, nerves and
lymphatics. There are 3 intercostal muscle layers corresponding to the lateral abdominal wall;
external, internal, innermost intercostals. At the mid axillary line there are thin intracostals which is
an extension of the internal intercostal muscle. In each intercostal space lies the neurovascular
bundle, comprising, from superior to inferiorly; the posterior intercostal vein, artery and nerve, lying
protected in the subcostal groove of the rib above and situated between the second and third layer
of the intercostal muscles. These blood vessels anastomose anteriorly with the anterior intercostal
vessels, which arise from the internal thoracic artery and vein.

Please rate this question:

Discuss and give feedback

Next question

Chest drains
There are a number of different indications for chest drain insertion. In general terms large bore
chest drains are preferred for trauma and haemothorax drainage. Smaller diameter chest drains can
be used for pneumothorax or pleural effusion drainage.

Insertion can be performed either using anatomical guidance or through ultrasound guidance. In the
exam, the anatomical method is usually tested.

It is advised that chest drains are placed in the 'safe triangle'. The triangle is located in the mid
axillary line of the 5th intercostal space. It is bordered by:
Anterior edge latissimus dorsi, the lateral border of pectoralis major, a line superior to the horizontal
level of the nipple, and the apex below the axilla.

Another triangle is situated behind the scapula. It is bounded above by the trapezius, below by the
latissimus dorsi, and laterally by the vertebral border of the scapula; the floor is partly formed by the
rhomboid major. If the scapula is drawn forward by folding the arms across the chest, and the trunk
bent forward, parts of the sixth and seventh ribs and the interspace between them become
subcutaneous and available for auscultation. The space is therefore known as the triangle of
auscultation.

References
Prof Harold Ellis. The applied anatomy of chest drains insertions. British Journal of hospital medicine
2007; (68): 44-45.

Laws D, Neville E, Duffy J. BTS guidelines for insertion of chest drains. Thorax, 2003; (58): 53-59.

Next question
Question 173 of 560

Two teenagers are playing with an airgun when one accidentally shoots his friend in the abdomen.
He is brought to the emergency department. On examination there is a bullet entry point immediately
to the right of the rectus sheath at the level of the 1st lumbar vertebra. Which of the following
structures is most likely to be injured by the bullet?

Head of pancreas

Right ureter

Right adrenal gland

Fundus of the gallbladder

Gastric antrum

Theme from September 2011 Exam


Theme from April 2014 exam

The fundus of the gallbladder lies at this level and is the most superficially located structure.
Please rate this question:

Discuss and give feedback


Next question

Levels

Transpyloric plane
Level of the body of L1

 Pylorus stomach
 Left kidney hilum (L1- left one!)
 Fundus of the gallbladder
 Neck of pancreas
 Duodenojejunal flexure
 Superior mesenteric artery
 Portal vein
 Left and right colic flexure
 Root of the transverse mesocolon
 2nd part of the duodenum
 Upper part of conus medullaris
 Spleen

Can be identified by asking the supine patient to sit up without using their arms. The plane is located
where the lateral border of the rectus muscle crosses the costal margin.

Anatomical planes
Subcostal plane Lowest margin of 10th costal cartilage

Intercristal plane Level of body L4 (highest point of iliac crest)

Intertubercular plane Level of body L5

Common level landmarks


Inferior mesenteric artery L3

Bifurcation of aorta into common iliac arteries L4

Formation of IVC L5 (union of common iliac veins)

Diaphragm apertures  Vena cava T8


 Oesophagus T10
 Aortic hiatus T12

Next question
Question 174 of 560

Which of the following muscles inserts onto the lesser tuberostiy of the the humerus?

Subscapularis

Deltoid

Supraspinatus

Teres minor

Infraspinatus

With the exception of subscapularis which inserts into the lesser tuberosity, the muscles of the
rotator cuff insert into the greater tuberosity.
Please rate this question:

Discuss and give feedback


Next question

Shoulder joint

 Shallow synovial ball and socket type of joint.


 It is an inherently unstable joint, but is capable to a wide range of movement.
 Stability is provided by muscles of the rotator cuff that pass from the scapula to insert in the
greater tuberosity (all except sub scapularis-lesser tuberosity).

Glenoid labrum

 Fibrocartilaginous rim attached to the free edge of the glenoid cavity


 Tendon of the long head of biceps arises from within the joint from the supraglenoid tubercle,
and is fused at this point to the labrum.
 The long head of triceps attaches to the infraglenoid tubercle

Fibrous capsule
 Attaches to the scapula external to the glenoid labrum and to the labrum itself (postero-
superiorly)
 Attaches to the humerus at the level of the anatomical neck superiorly and the surgical neck
inferiorly
 Anteriorly the capsule is in contact with the tendon of subscapularis, superiorly with the
supraspinatus tendon, and posteriorly with the tendons of infraspinatus and teres minor. All
these blend with the capsule towards their insertion.
 Two defects in the fibrous capsule; superiorly for the tendon of biceps. Anteriorly there is a
defect beneath the subscapularis tendon.
 The inferior extension of the capsule is closely related to the axillary nerve at the surgical
neck and this nerve is at risk in anteroinferior dislocations. It also means that proximally sited
osteomyelitis may progress to septic arthritis.

Movements and muscles


Flexion Anterior part of deltoid
Pectoralis major
Biceps
Coracobrachialis

Extension Posterior deltoid


Teres major
Latissimus dorsi

Adduction Pectoralis major


Latissimus dorsi
Teres major
Coracobrachialis

Abduction Mid deltoid


Supraspinatus

Medial rotation Subscapularis


Anterior deltoid
Teres major
Latissimus dorsi

Lateral rotation Posterior deltoid


Infraspinatus
Teres minor
Important anatomical relations
Anteriorly Brachial plexus
Axillary artery and vein

Posterior Suprascapular nerve


Suprascapular vessels

Inferior Axillary nerve


Circumflex humeral vessels

Next question
Question 175 of 560

Which of the following nerves is not contained within the posterior triangle of the neck?

Accessory nerve

Phrenic nerve

Greater auricular nerve

Ansa cervicalis

Lesser occiptal nerve

Theme from September 2012 Exam


Ansa cervicalis is a content of the anterior triangle of the neck.
Please rate this question:

Discuss and give feedback


Next question

Posterior triangle of the neck

Boundaries
Apex Sternocleidomastoid and the Trapezius muscles at the Occipital bone

Anterior Posterior border of the Sternocleidomastoid

Posterior Anterior border of the Trapezius

Base Middle third of the clavicle


Image sourced from Wikipedia

Contents
Nerves  Accessory nerve
 Phrenic nerve
 Three trunks of the brachial plexus
 Branches of the cervical plexus: Supraclavicular nerve, transverse cervical
nerve, great auricular nerve, lesser occipital nerve

Vessels  External jugular vein


 Subclavian artery

Muscles  Inferior belly of omohyoid


 Scalene

Lymph  Supraclavicular
nodes  Occipital

Next question
Question 176 of 560

A 42 year old lady is reviewed in the outpatient clinic following a routine surgical procedure. She
complains of diminished sensation at the dorso-lateral aspect of her foot. Which of the following
nerves is most likely to be affected?

Sural

Superficial peroneal

Deep peroneal

Medial plantar

Lateral plantar

Theme from April 2012 Exam


The sural nerve supplies the lateral aspect of the foot. It runs alongside the short saphenous vein
and may be injured in short saphenous vein surgery.

Please rate this question:

Discuss and give feedback

Next question

Foot- Cutaneous sensation

Region Nerve
Lateral plantar Sural

Dorsum (not 1st web space) Superficial peroneal

1st Web space Deep peroneal

Extremities of toes Medial and lateral plantar nerves

Proximal plantar Tibial

Medial plantar Medial plantar nerve

Lateral plantar Lateral plantar nerve


Image sourced from Wikipedia

Next question
Question 177 of 560

Which of the following anatomical planes separates the prostate from the rectum?

Sibsons fascia

Denonvilliers fascia

Levator ani muscle

Waldeyers fascia

None of the above

Theme from April 2014 exam


The Denonvilliers fascia separates the rectum from the prostate. Waldeyers fascia separates the
rectum from the sacrum
Please rate this question:

Discuss and give feedback


Next question

Prostate gland

The prostate gland is approximately the shape and size of a walnut and is located inferior to the
bladder. It is separated from the rectum by Denonvilliers fascia and its blood supply is derived from
the internal iliac vessels (via inferior vesical artery). The internal sphincter lies at the apex of the
gland and may be damaged during prostatic surgery, affected individuals may complain of
retrograde ejaculation.

Summary of prostate gland


Arterial supply Inferior vesical artery (from internal iliac)

Venous drainage Prostatic venous plexus (to paravertebral veins)

Lymphatic Internal iliac nodes


drainage

Innervation Inferior hypogastric plexus

Dimensions  Transverse diameter (4cm)


 AP diameter (2cm)
 Height (3cm)

Lobes  Posterior lobe: posterior to urethra


 Median lobe: posterior to urethra, in between ejaculatory ducts
 Lateral lobes x 2
 Isthmus

Zones  Peripheral zone: subcapsular portion of posterior prostate. Most


prostate cancers are here
 Central zone
 Transition zone
 Stroma

Relations
Pubic symphysis
Anterior Prostatic venous plexus

Posterior Denonvilliers fascia


Rectum
Ejaculatory ducts

Lateral Venous plexus (lies on prostate)


Levator ani (immediately below the puboprostatic ligaments)
Image sourced from Wikipedia

Next question
Question 178 of 560

A 56 year old lady is undergoing an adrenalectomy for Conns syndrome. During the operation the
surgeon damages the middle adrenal artery and haemorrhage ensues. From which of the following
structures does this vessel originate?

Aorta

Renal artery

Splenic artery

Coeliac axis

Superior mesenteric artery

The middle adrenal artery is usually a branch of the aorta, the lower adrenal artery typically arises
from the renal vessels.

Please rate this question:

Discuss and give feedback

Next question

Adrenal gland anatomy

Anatomy

Location Superomedially to the upper pole of each kidney


Relationships of the right Diaphragm-Posteriorly, Kidney-Inferiorly, Vena Cava-Medially, Hepato-renal
adrenal pouch and bare area of the liver-Anteriorly

Relationships of the left Crus of the diaphragm-Postero- medially, Pancreas and splenic vessels-
adrenal Inferiorly, Lesser sac and stomach-Anteriorly

Superior adrenal arteries- from inferior phrenic artery, Middle adrenal


Arterial supply arteries - from aorta, Inferior adrenal arteries -from renal arteries

Venous drainage of the Via one central vein directly into the IVC
right adrenal

Venous drainage of the Via one central vein into the left renal vein
left adrenal

Next question
Question 179 of 560

A 73 year old lady suffers a fracture at the surgical neck of the humerus. The decision is made to
operate. There are difficulties in reducing the fracture and a vessel lying posterior to the surgical
neck is injured. Which of the following is this vessel most likely to be?

Axillary artery

Brachial artery

Thoracoacromial artery

Transverse scapular artery

Posterior circumflex humeral artery

The circumflex humeral arteries lie at the surgical neck and is this scenario the posterior circumflex
is likely to be injured. The thoracoacromial and transverse scapular arteries lie more superomedially.
The posterior circumflex humeral artery is a branch of the axillary artery.
Please rate this question:

Discuss and give feedback


Next question

Shoulder joint

 Shallow synovial ball and socket type of joint.


 It is an inherently unstable joint, but is capable to a wide range of movement.
 Stability is provided by muscles of the rotator cuff that pass from the scapula to insert in the
greater tuberosity (all except sub scapularis-lesser tuberosity).

Glenoid labrum

 Fibrocartilaginous rim attached to the free edge of the glenoid cavity


 Tendon of the long head of biceps arises from within the joint from the supraglenoid tubercle,
and is fused at this point to the labrum.
 The long head of triceps attaches to the infraglenoid tubercle
Fibrous capsule

 Attaches to the scapula external to the glenoid labrum and to the labrum itself (postero-
superiorly)
 Attaches to the humerus at the level of the anatomical neck superiorly and the surgical neck
inferiorly
 Anteriorly the capsule is in contact with the tendon of subscapularis, superiorly with the
supraspinatus tendon, and posteriorly with the tendons of infraspinatus and teres minor. All
these blend with the capsule towards their insertion.
 Two defects in the fibrous capsule; superiorly for the tendon of biceps. Anteriorly there is a
defect beneath the subscapularis tendon.
 The inferior extension of the capsule is closely related to the axillary nerve at the surgical
neck and this nerve is at risk in anteroinferior dislocations. It also means that proximally sited
osteomyelitis may progress to septic arthritis.

Movements and muscles


Flexion Anterior part of deltoid
Pectoralis major
Biceps
Coracobrachialis

Extension Posterior deltoid


Teres major
Latissimus dorsi

Adduction Pectoralis major


Latissimus dorsi
Teres major
Coracobrachialis

Abduction Mid deltoid


Supraspinatus

Medial rotation Subscapularis


Anterior deltoid
Teres major
Latissimus dorsi
Lateral rotation Posterior deltoid
Infraspinatus
Teres minor

Important anatomical relations


Anteriorly Brachial plexus
Axillary artery and vein

Posterior Suprascapular nerve


Suprascapular vessels

Inferior Axillary nerve


Circumflex humeral vessels

Next question
Question 180 of 560

Which of the structures listed below lies posterior to the carotid sheath at the level of the 6th cervical
vertebra?

Hypoglossal nerve

Vagus nerve

Cervical sympathetic chain

Ansa cervicalis

Glossopharyngeal nerve

The carotid sheath is crossed anteriorly by the hypoglossal nerves and the ansa cervicalis. The
vagus lies within it. The cervical sympathetic chain lies posteriorly between the sheath and the
prevertebral fascia.
Please rate this question:

Discuss and give feedback


Next question

Common carotid artery

The right common carotid artery arises at the bifurcation of the brachiocephalic trunk, the left
common carotid arises from the arch of the aorta. Both terminate at the level of the upper border of
the thyroid cartilage (the lower border of the third cervical vertebra) by dividing into the internal and
external carotid arteries.

Left common carotid artery


This vessel arises immediately to the left and slightly behind the origin of the brachiocephalic trunk.
Its thoracic portion is 2.5- 3.5 cm in length and runs superolaterally to the sternoclavicular joint.

In the thorax
The vessel is in contact, from below upwards, with the trachea, left recurrent laryngeal nerve, left
margin of the oesophagus. Anteriorly the left brachiocephalic vein runs across the artery, and the
cardiac branches from the left vagus descend in front of it. These structures together with the
thymus and the anterior margins of the left lung and pleura separate the artery from the manubrium.

In the neck
The artery runs superiorly deep to sternocleidomastoid and then enters the anterior triangle. At this
point it lies within the carotid sheath with the vagus nerve and the internal jugular vein. Posteriorly
the sympathetic trunk lies between the vessel and the prevertebral fascia. At the level of C7 the
vertebral artery and thoracic duct lie behind it. The anterior tubercle of C6 transverse process is
prominent and the artery can be compressed against this structure (it corresponds to the level of the
cricoid).
Anteriorly at C6 the omohyoid muscle passes superficial to the artery.
Within the carotid sheath the jugular vein lies lateral to the artery.

Right common carotid artery


The right common carotid arises from the brachiocephalic artery. The right common carotid artery
corresponds with the cervical portion of the left common carotid, except that there is no thoracic duct
on the right. The oesophagus is less closely related to the right carotid than the left.

Summary points about the carotid anatomy

Path
Passes behind the sternoclavicular joint (12% patients above this level) to the upper border of the
thyroid cartilage, to divide into the external (ECA) and internal carotid arteries (ICA).

Relations

 Level of 6th cervical vertebra crossed by omohyoid


 Then passes deep to the thyrohyoid, sternohyoid, sternomastoid muscles.
 Passes anterior to the carotid tubercle (transverse process 6th cervical vertebra)-NB
compression here stops haemorrhage.
 The inferior thyroid artery passes posterior to the common carotid artery.
 Then : Left common carotid artery crosses the thoracic duct, Right common carotid artery
crossed by recurrent laryngeal nerve
Image sourced from Wikipedia

Next question
Question 181 of 560

A sprinter attends A&E with severe leg pain. He had forgotten to warm up and ran a 100m sprint
race. Towards the end of the race he experienced pain in the posterior aspect of his thigh. The pain
worsens, localising to the lateral aspect of the knee. The sprinter is unable to flex the knee. What
structure has been injured?

Anterior cruciate ligament

Posterior cruciate ligament

Semimembranosus tendon

Semitendinosus tendon

Biceps femoris tendon

Theme from 2009 Exam

The biceps femoris is commonly injured in sports that require explosive bending of the knee as seen
in sprinting, especially if the athlete has not warmed up first. Avulsion most commonly occurs where
the long head attaches to the ischial tuberosity. Injuries to biceps femoris are more common than to
the other hamstrings.

Please rate this question:

Discuss and give feedback

Next question

Biceps femoris

The biceps femoris is one of the hamstring group of muscles located in the posterior upper thigh. It
has two heads.
Long head

Origin Ischial tuberosity

Insertion Fibular head

Action Knee flexion, lateral rotation tibia, extension hip

Innervation Tibial division of sciatic nerve (L5, S1, S2)

Arterial Profunda femoris artery, inferior gluteal artery, and the superior muscular branches of
supply popliteal artery

Image demonstrating the biceps femoris muscle, with the long head outlined

Image sourced from Wikipedia

Short head
Origin Lateral lip of linea aspera, lateral supracondylar ridge of femur

Insertion Fibular head

Action Knee flexion, lateral rotation tibia

Innervation Common peroneal division of sciatic nerve (L5, S1, S2)

Arterial Profunda femoris artery, inferior gluteal artery, and the superior muscular branches of
supply popliteal artery

Next question
Question 182 of 560

A 24 year old man falls and lands astride a manhole cover. He suffers from an injury to the anterior
bulbar urethra. Where will the extravasated urine tend to collect?

Lesser pelvis

Connective tissue of the scrotum

Deep perineal space

Ischiorectal fossa

Posterior abdominal wall

This portion of the urethra is contained between the perineal membrane and the membranous layer
of the superficial fascia. As these are densely adherent to the ischiopubic rami, extravasated urine
cannot pass posteriorly because the 2 layers are continuous around the superficial transverse
perineal muscles.
Please rate this question:

Discuss and give feedback


Next question

Lower genitourinary tract trauma

 Most bladder injuries occur due to blunt trauma


 85% associated with pelvic fractures
 Easily overlooked during assessment in trauma
 Up to 10% of male pelvic fractures are associated with urethral or bladder injuries

Types of injury

Urethral injury  Mainly in males


 Blood at the meatus (50% cases)
 There are 2 types:

i.Bulbar rupture
- most common
- straddle type injury e.g. bicycles
- triad signs: urinary retention, perineal haematoma, blood at the
meatus
ii. Membranous rupture
- can be extra or intraperitoneal
- commonly due to pelvic fracture
- Penile or perineal oedema/ hematoma
- PR: prostate displaced upwards (beware co-existing
retroperitoneal haematomas as they may make examination
difficult)

- Investigation: ascending urethrogram


- Management: suprapubic catheter (surgical placement, not
percutaneously)

External genitalia injuries  Secondary to injuries caused by penetration, blunt trauma,


(i.e., the penis and the continence- or sexual pleasure-enhancing devices, and
scrotum) mutilation

Bladder injury  rupture is intra or extraperitoneal


 presents with haematuria or suprapubic pain
 history of pelvic fracture and inability to void: always
suspect bladder or urethral injury
 inability to retrieve all fluid used to irrigate the bladder
through a Foley catheter indicates bladder injury
 investigation- IVU or cystogram
 management: laparotomy if intraperitoneal, conservative if
extraperitoneal

Next question
Question 183 of 560

A 73 year old man presents with symptoms of mesenteric ischaemia. As part of his diagnostic work
up a diagnostic angiogram is performed .The radiologist is attempting to cannulate the coeliac axis
from the aorta. At which of the following vertebral levels does this is usually originate?

T10

L2

L3

T8

T12

Coeliac trunk branches:

Left Hand Side (LHS)

Left gastric
Hepatic
Splenic

The coeliac axis branches off the aorta at T12.


Please rate this question:

Discuss and give feedback


Next question

Coeliac axis

The coeliac axis has three main branches.

 Left gastric
 Hepatic: branches-Right Gastric, Gastroduodenal, Right Gastroepiploic, Superior
Pancreaticoduodenal, Cystic (occasionally).
 Splenic: branches- Pancreatic, Short Gastric, Left Gastroepiploic
It occasionally gives off one of the inferior phrenic arteries.

Image sourced from Wikipedia

Relations
Anteriorly Lesser omentum

Right Right coeliac ganglion and caudate process of liver

Left Left coeliac ganglion and gastric cardia

Inferiorly Upper border of pancreas and renal vein

Next question
Question 184 of 560

A 43 year old man is diagnosed as having a malignancy of the right adrenal gland. The decision is
made to resect this via an open anterior approach. Which of the following will be most useful during
the surgery?

Division of the coronary ligaments of the liver

Mobilisation of the colonic hepatic flexure

Division of the right renal vein

Division of the ligament of Trietz

Division of the right colic artery

Mobilisation of the hepatic flexure and right colon are standard steps in open adrenal surgery from
an anterior approach. Mobilisation of the liver is seldom required.

Please rate this question:

Discuss and give feedback

Next question

Adrenal gland anatomy

Anatomy

Location Superomedially to the upper pole of each kidney


Relationships of the right Diaphragm-Posteriorly, Kidney-Inferiorly, Vena Cava-Medially, Hepato-renal
adrenal pouch and bare area of the liver-Anteriorly

Relationships of the left Crus of the diaphragm-Postero- medially, Pancreas and splenic vessels-
adrenal Inferiorly, Lesser sac and stomach-Anteriorly

Superior adrenal arteries- from inferior phrenic artery, Middle adrenal


Arterial supply arteries - from aorta, Inferior adrenal arteries -from renal arteries

Venous drainage of the Via one central vein directly into the IVC
right adrenal

Venous drainage of the Via one central vein into the left renal vein
left adrenal

Next question
Question 185 of 560

A 45 year old man presents with a lipoma located posterior to the posterior border of the
sternocleidomastoid muscle, approximately 4cm superior to the middle third of the clavicle. During
surgical excision of the lesion troublesome bleeding is encountered. Which of the following is the
most likely source?

Internal jugular vein

External jugular vein

Common carotid artery

Vertebral artery

Second part of the subclavian artery

The external jugular vein runs obliquely in the superficial fascia of the posterior triangle. It drains into
the subclavian vein. During surgical exploration of this area the external jugular vein may be injured
and troublesome bleeding may result. The internal jugular vein and carotid arteries are located in the
anterior triangle. The third, and not the second, part of the subclavian artery is also a content of the
posterior triangle
Please rate this question:

Discuss and give feedback


Next question

Posterior triangle of the neck

Boundaries
Apex Sternocleidomastoid and the Trapezius muscles at the Occipital bone

Anterior Posterior border of the Sternocleidomastoid

Posterior Anterior border of the Trapezius


Base Middle third of the clavicle

Image sourced from Wikipedia

Contents
Nerves  Accessory nerve
 Phrenic nerve
 Three trunks of the brachial plexus
 Branches of the cervical plexus: Supraclavicular nerve, transverse cervical
nerve, great auricular nerve, lesser occipital nerve

Vessels  External jugular vein


 Subclavian artery

Muscles  Inferior belly of omohyoid


 Scalene

Lymph  Supraclavicular
nodes  Occipital

Next question
Question 186-188 of 560

Theme: Levels of spinal injury

A. C2
B. C3
C. C4
D. C5
E. C6
F. L1
G. L2
H. L3
I. L4
J. L5

Please select the most likely spinal level for the injury described. Each option may be used once,
more than once or not at all.

186. A 62 year old male complains of back pain. He has had a recent fall. Walking causes pain
of the left lower leg. On examination he is noted to have reduced sensation over the knee.

You answered C2

The correct answer is L3

Sensation over the knee is equivalent to the L3 dermatome. The four nerves involved
include the infrapatellar branch of the saphenous nerve, the lateral cutaneous nerve of the
thigh, anterior cutaneous nerve of the thigh (both lateral and medial branches).

187. A 42 year old woman is found to have a burst fracture of the C5 vertebral body. After a
few months where would the level of injury be?

You answered C2

The correct answer is C6

A C5 burst fracture usually injures the C6 spinal cord situated at the C5 vertebrae and
also the C4 spinal roots that exit the spinal column between the C4 and C5 vertebra. Such
an injury should cause a loss of sensations in C4 dermatome and weak deltoids. Due to
oedema , the biceps (C5) may be initially weak but should recover. The wrist extensors
(C6), however, should remain weak and sensation at and below C6 should be severely
compromised. A neurosurgeon would conclude that there is a burst fracture at C5 from
the x-rays, an initial sensory level at C4 (the first abnormal sensory dermatome) and the
partial loss of deltoids and biceps would imply a motor level at C4 (the highest abnormal
muscle level). Over time, as the patient recovers the C4 roots and the C5 spinal cord, both
the sensory level and motor level should end up at C6. Such recovery is often attributed
to 'root' recovery.

188. A 56 year old man suddenly develops severe back pain. His pain has a radicular pattern.
On examination he is unable to extend his great toe.

You answered C2

The correct answer is L5

Extensor hallucis longus is derived from L5 and loss of EHL function is a useful test to
determine whether this level is involved.

Please rate this question:

Discuss and give feedback


Next question

Spinal disorders

Dorsal column lesion  Loss vibration and proprioception


 Tabes dorsalis, SACD

Spinothalamic tract  Loss of pain, sensation and temperature


lesion

Central cord lesion  Flaccid paralysis of the upper limbs

Osteomyelitis  Normally progressive


 Staph aureus in IVDU, normally cervical region affected
 Fungal infections in immunocompromised
 Thoracic region affected in TB

Infarction spinal cord  Dorsal column signs (loss of proprioception and fine
discrimination)

Cord compression  UMN signs


 Malignancy
 Haematoma
 Fracture

Brown-sequard  Hemisection of the spinal cord


syndrome  Ipsilateral paralysis
 Ipsilateral loss of proprioception and fine discrimination
 Contralateral loss of pain and temperature
Image sourced from Wikipedia
Image sourced from Wikipedia

Dermatomes
 C2 to C4 The C2 dermatome covers the occiput and the top part of the neck. C3 covers the
lower part of the neck to the clavicle. C4 covers the area just below the clavicle.
 C5 to T1 Situated in the arms. C5 covers the lateral arm at and above the elbow. C6 covers
the forearm and the radial (thumb) side of the hand. C7 is the middle finger, C8 is the medial
aspect of the hand, and T1 covers the medial side of the forearm.
 T2 to T12 The thoracic covers the axillary and chest region. T3 to T12 covers the chest and
back to the hip girdle. The nipples are situated in the middle of T4. T10 is situated at the
umbilicus. T12 ends just above the hip girdle.
 L1 to L5 The cutaneous dermatome representing the hip girdle and groin area is innervated
by L1 spinal cord. L2 and 3 cover the front part of the thighs. L4 and L5 cover medial and
lateral aspects of the lower leg.
 S1 to S5 S1 covers the heel and the middle back of the leg. S2 covers the back of the
thighs. S3 cover the medial side of the buttocks and S4-5 covers the perineal region. S5 is of
course the lowest dermatome and represents the skin immediately at and adjacent to the
anus.

Myotomes

Upper limb
Elbow flexors/Biceps C5

Wrist extensors C6

Elbow extensors/Triceps C7

Long finger flexors C8

Small finger abductors T1

Lower limb
Hip flexors (psoas) L1 and L2

Knee extensors (quadriceps) L3

Ankle dorsiflexors (tibialis anterior) L4 and L5

Toe extensors (hallucis longus) L5


Ankle plantar flexors (gastrocnemius) S1

The anal sphincter is innervated by S2,3,4


Next question
Question 189 of 560

The sciatic nerve lies deep to the following structures except:

Gluteus maximus

The femoral cutaneous nerve

Long head of biceps femoris

Gluteus medius

Branch of the inferior gluteal artery

The gluteus medius does not extend around to the sciatic nerve.
Please rate this question:

Discuss and give feedback


Next question

Sciatic nerve

The sciatic nerve is formed from the sacral plexus and is the largest nerve in the body. It is the
continuation of the main part of the plexus arising from ventral rami of L4 to S3. These rami
converge at the inferior border of piriformis to form the nerve itself. It passes through the inferior part
of the greater sciatic foramen and emerges beneath piriformis. Medially, lie the inferior gluteal nerve
and vessels and the pudendal nerve and vessels. It runs inferolaterally under the cover of gluteus
maximus midway between the greater trochanter and ischial tuberosity. It receives its blood supply
from the inferior gluteal artery. The nerve provides cutaneous sensation to the skin of the foot and
the leg. It also innervates the posterior thigh muscles and the lower leg and foot muscles. The nerve
splits into the tibial and common peroneal nerves approximately half way down the posterior thigh.
The tibial nerve supplies the flexor muscles and the common peroneal nerve supplies the extensor
muscles and the abductor muscles.

Summary points
Origin Spinal nerves L4 - S3
Articular Branches Hip joint

Muscular branches in  Semitendinosus


upper leg  Semimembranosus
 Biceps femoris
 Part of adductor magnus

Cutaneous sensation  Posterior aspect of thigh (via cutaneous nerves)


 Gluteal region
 Entire lower leg (except the medial aspect)

Terminates At the upper part of the popliteal fossa by dividing into the tibial and
peroneal nerves

 The nerve to the short head of the biceps femoris comes from the common peroneal part of
the sciatic and the other muscular branches arise from the tibial portion.
 The tibial nerve goes on to innervate all muscles of the foot except the extensor digitorum
brevis (which is innervated by the common peroneal nerve).

Next question
Question 190 of 560

Which of the following upper limb muscles is not innervated by the radial nerve?

Extensor carpi ulnaris

Abductor digiti minimi

Anconeus

Supinator

Brachioradialis

Mnemonic for radial nerve muscles: BEST

B rachioradialis
E xtensors
S upinator
T riceps

Abductor digiti minimi is innervated by the ulnar nerve.


Please rate this question:

Discuss and give feedback


Next question

Radial nerve

Continuation of posterior cord of the brachial plexus (root values C5 to T1)

Path

 In the axilla: lies posterior to the axillary artery on subscapularis, latissimus dorsi and teres
major.
 Enters the arm between the brachial artery and the long head of triceps (medial to humerus).
 Spirals around the posterior surface of the humerus in the groove for the radial nerve.
 At the distal third of the lateral border of the humerus it then pierces the intermuscular
septum and descends in front of the lateral epicondyle.
 At the lateral epicondyle it lies deeply between brachialis and brachioradialis where it then
divides into a superficial and deep terminal branch.
 Deep branch crosses the supinator to become the posterior interosseous nerve.

In the image below the relationships of the radial nerve can be appreciated

Image sourced from Wikipedia

Regions innervated
 Triceps
 Anconeus
Motor (main nerve)  Brachioradialis
 Extensor carpi radialis

 Supinator
 Extensor carpi ulnaris
 Extensor digitorum
Motor (posterior  Extensor indicis
interosseous branch)  Extensor digiti minimi
 Extensor pollicis longus and brevis
 Abductor pollicis longus

The area of skin supplying the proximal phalanges on the dorsal aspect of the
Sensory hand is supplied by the radial nerve (this does not apply to the little finger and
part of the ring finger)
Muscular innervation and effect of denervation
Anatomical
location Muscle affected Effect of paralysis

Shoulder Long head of triceps Minor effects on shoulder stability in abduction

Arm Triceps Loss of elbow extension

Forearm Supinator Weakening of supination of prone hand and


Brachioradialis elbow flexion in mid prone position
Extensor carpi radialis
longus and brevis

The cutaneous sensation of the upper limb- illustrating the contribution of the radial nerve

Image sourced from Wikipedia

Next question

Question 191 of 560


Which of the following forms the floor of the anatomical snuffbox?

Radial artery

Cephalic vein

Extensor pollicis brevis

Scaphoid bone

Cutaneous branch of the radial nerve

Theme from April 2014 exam


The scaphoid bone forms the floor of the anatomical snuffbox. The cutaneous branch of the radial
nerve is much more superficially and proximally located.

Please rate this question:

Discuss and give feedback

Next question

Anatomical snuffbox

Posterior border Tendon of extensor pollicis longus

Anterior border Tendons of extensor pollicis brevis and abductor pollicis longus

Proximal border Styloid process of the radius


Distal border Apex of snuffbox triangle

Floor Trapezium and scaphoid

Content Radial artery

Image showing the anatomical snuffbox

Image sourced from Wikipedia

Next question
Question 192 of 560

During a liver resection a surgeon performs a pringles manoeuvre to control bleeding. Which of the
following structures will lie posterior to the epiploic foramen at this level?

Hepatic artery

Cystic duct

Greater omentum

Superior mesenteric artery

Inferior vena cava

Bleeding from liver trauma or a difficult cholecystectomy can be controlled with a vascular clamp
applied at the epiploic foramen.

The epiploic foramen has the following boundaries:


Anteriorly (in the free edge of the lesser omentum): Bile duct to the right, portal vein behind and
hepatic artery to the left.
PosteriorlyInferior vena cava
Inferiorly1st part of the duodenum
SuperiorlyCaudate process of the liver
Please rate this question:

Discuss and give feedback


Next question

Liver

Structure of the liver


Right lobe  Supplied by right hepatic artery
 Contains Couinaud segments V to VIII (-/+Sg I)

Left lobe  Supplied by the left hepatic artery


 Contains Couinaud segments II to IV (+/- Sg1)
Quadrate lobe  Part of the right lobe anatomically, functionally is part of the left
 Couinaud segment IV
 Porta hepatis lies behind
 On the right lies the gallbladder fossa
 On the left lies the fossa for the umbilical vein

Caudate lobe  Supplied by both right and left hepatic arteries


 Couinaud segment I
 Lies behind the plane of the porta hepatis
 Anterior and lateral to the inferior vena cava
 Bile from the caudate lobe drains into both right and left hepatic ducts

Detailed knowledge of Couinaud segments is not required for MRCS

 Between the liver lobules are portal canals which contain the portal triad: Hepatic Artery,
Portal Vein, tributary of Bile Duct.

Relations of the liver


Anterior Postero inferiorly

Diaphragm Oesophagus

Xiphoid process Stomach

Duodenum

Hepatic flexure of colon

Right kidney

Gallbladder
Inferior vena cava

Porta hepatis
Location Postero inferior surface, it joins nearly at right angles with the left sagittal fossa, and
separates the caudate lobe behind from the quadrate lobe in front

Transmits  Common hepatic duct


 Hepatic artery
 Portal vein
 Sympathetic and parasympathetic nerve fibres
 Lymphatic drainage of the liver (and nodes)

Ligaments
Falciform ligament  2 layer fold peritoneum from the umbilicus to anterior liver surface
 Contains ligamentum teres (remnant umbilical vein)
 On superior liver surface it splits into the coronary and left
triangular ligaments

Ligamentum teres Joins the left branch of the portal vein in the porta hepatis

Ligamentum Remnant of ductus venosus


venosum

Arterial supply

 Hepatic artery

Venous

 Hepatic veins
 Portal vein

Nervous supply

 Sympathetic and parasympathetic trunks of coeliac plexus

Next question
Question 193 of 560

A 72 year old lady is suspected of having a femoral hernia. At which of the following sites is it most
likely to be identifiable clinically?

Mid inguinal point

Above and medial to the pubic tubercle

Below and lateral to the pubic tubercle

Mid point of the inguinal ligament

3 cm superomedially to the superficial inguinal ring

Femoral hernias exit the femoral canal below and lateral to the pubic tubercle. Femoral hernia occur
mainly in women due to their difference in pelvic anatomy. They are at high risk of strangulation and
therefore should be repaired.
Please rate this question:

Discuss and give feedback


Next question

Femoral canal

The femoral canal lies at the medial aspect of the femoral sheath. The femoral sheath is a fascial
tunnel containing both the femoral artery laterally and femoral vein medially. The canal lies medial to
the vein.

Borders of the femoral canal


Laterally Femoral vein

Medially Lacunar ligament

Anteriorly Inguinal ligament


Posteriorly Pectineal ligament

Image showing dissection of femoral canal

Image sourced from Wikipedia

Contents

 Lymphatic vessels
 Cloquet's lymph node

Physiological significance
Allows the femoral vein to expand to allow for increased venous return to the lower limbs.

Pathological significance
As a potential space, it is the site of femoral hernias. The relatively tight neck places these at high
risk of strangulation.
Question 194 of 560

Which muscle is responsible for causing flexion of the distal interphalangeal joint of the ring finger?

Flexor digitorum superficialis

Lumbricals

Palmar interossei

Flexor digitorum profundus

Flexor digiti minimi brevis

Theme from April 2014 exam


Flexor digitorum superficialis and flexor digitorum profundus are responsible for causing flexion. The
superficialis tendons insert on the bases of the middle phalanges; the profundus tendons insert on
the bases of the distal phalanges. Both tendons flex the wrist, MCP and PIP joints; however, only the
profundus tendons flex the DIP joints.
Please rate this question:

Discuss and give feedback


Next question

Hand

Anatomy of the hand


Bones  8 Carpal bones
 5 Metacarpals
 14 phalanges

Intrinsic Muscles 7 Interossei - Supplied by ulnar nerve

 3 palmar-adduct fingers
 4 dorsal- abduct fingers
Intrinsic muscles Lumbricals

 Flex MCPJ and extend the IPJ.


 Origin deep flexor tendon and insertion dorsal extensor hood
mechanism.
 Innervation: 1st and 2nd- median nerve, 3rd and 4th- deep branch of
the ulnar nerve.

Thenar eminence  Abductor pollicis brevis


 Opponens pollicis
 Flexor pollicis brevis

Hypothenar  Opponens digiti minimi


eminence  Flexor digiti minimi brevis
 Abductor digiti minimi

Image sourced from Wikipedia

Fascia and compartments of the palm


The fascia of the palm is continuous with the antebrachial fascia and the fascia of the dorsum of the
hand. The palmar fascia is thin over the thenar and hypothenar eminences. In contrast the palmar
fascia is relatively thick. The palmar aponeurosis covers the soft tissues and overlies the flexor
tendons. The apex of the palmar aponeurosis is continuous with the flexor retinaculum and the
palmaris longus tendon. Distally, it forms four longitudinal digital bands that attach to the bases of
the proximal phalanges, blending with the fibrous digital sheaths.
A medial fibrous septum extends deeply from the medial border of the palmar aponeurosis to the 5th
metacarpal. Lying medial to this are the hypothenar muscles. In a similar fashion, a lateral fibrous
septum extends deeply from the lateral border of the palmar aponeurosis to the 3rd metacarpal. The
thenar compartment lies lateral to this area.
Lying between the thenar and hypothenar compartments is the central compartment. It contains the
flexor tendons and their sheaths, the lumbricals, the superficial palmar arterial arch and the digital
vessels and nerves.
The deepest muscular plane is the adductor compartment, which contains adductor pollicis.

Short muscles of the hand


These comprise the lumbricals and interossei. The four slender lumbrical muscles flex the fingers at
the metacarpophalangeal joints and extend the interphalangeal joint. The four dorsal interossei are
located between the metacarpals and the four palmar interossei lie on the palmar surface of the
metacarpals in the interosseous compartment of the hand.

Long flexor tendons and sheaths in the hand


The tendons of FDS and FDP enter the common flexor sheath deep to the flexor retinaculum. The
tendons enter the central compartment of the hand and fan out to their respective digital synovial
sheaths. Near the base of the proximal phalanx, the tendon of FDS splits to permit the passage of
FDP. The FDP tendons are attached to the margins of the anterior aspect of the base of the distal
phalanx.
The fibrous digital sheaths contain the flexor tendons and their synovial sheaths. These extend from
the heads of the metacarpals to the base of the distal phalanges.
Next question
Question 195 of 560

A 34 year old lady undergoes a thyroidectomy for Graves disease. Post operatively she develops a
tense haematoma in the neck. In which of the following fascial planes will it be contained?

Gerotas fascia

Waldeyers fascia

Pretracheal fascia

Sibsons fascia

Clavipectoral fascia

The pretracheal fascia encloses the thyroid and is unyielding. Therefore tense haematomas can
develop.
Please rate this question:

Discuss and give feedback


Next question

Thyroid gland

 Right and left lobes connected by isthmus


 Surrounded by sheath from pretracheal layer of deep fascia
 Apex: Lamina of thyroid cartilage
 Base: 4th-5th tracheal ring
 Pyramidal lobe: from isthmus
 May be attached to foramen caecum at the base of the tongue

Relations
Anteromedially  Sternothyroid
 Superior belly of omohyoid
 Sternohyoid
 Anterior aspect of sternocleidomastoid
Posterolaterally Carotid sheath

Medially  Larynx
 Trachea
 Pharynx
 Oesophagus
 Cricothyroid muscle
 External laryngeal nerve (near superior thyroid artery)
 Recurrent laryngeal nerve (near inferior thyroid artery)

Posterior  Parathyroid glands


 Anastomosis of superior and inferior thyroid arteries

Isthmus  Anteriorly: Sternothyroids, sternohyoids, anterior jugular veins


 Posteriorly: 2nd, 3rd, 4th tracheal rings (attached via Ligament of
Berry)

Blood Supply
Arterial  Superior thyroid artery (1st branch of external carotid)
 Inferior thyroid artery (from thyrocervical trunk)
 Thyroidea ima (in 10% of population -from brachiocephalic artery or aorta)

Venous  Superior and middle thyroid veins - into the IJV


 Inferior thyroid vein - into the brachiocephalic veins

Next question
Question 196 of 560

A 32 year old lady complains of carpal tunnel syndrome. The carpal tunnel is explored surgically.
Which of the following structures will lie in closest proximity to the hamate bone within the carpal
tunnel?

The tendon of abductor pollicis longus

The tendons of flexor digitorum profundus

The tendons of flexor carpi radialis longus

Median nerve

Radial artery

The carpal tunnel contains nine flexor tendons:

 Flexor digitorum profundus


 Flexor digitorum superficialis
 Flexor pollicis longus

The tendon of flexor digitorum profundus lies deepest in the tunnel and will thus lie nearest to the
hamate bone.
Please rate this question:

Discuss and give feedback


Next question

Carpal bones

The wrist is comprised of 8 carpal bones, these are arranged in two rows of 4. It is convex from side
to side posteriorly and concave anteriorly.
Diagrammatic image of carpal bones
Image sourced from Wikipedia

Key to image
A Scaphoid

B Lunate

C Triquetrum

D Pisiform

E Trapezium

F Trapezoid

G Capitate

H Hamate

1 Radius
2 Ulna

3 Metacarpals

 No tendons attach to: Scaphoid, lunate, triquetrum (stabilised by ligaments)

Next question
Question 197 of 560

A 45 year old man sustains a significant head injury and a craniotomy is performed. The sigmoid
sinus is bleeding profusely, into which of the following structures does it drain?

Internal jugular vein

Straight sinus

Petrosal sinus

Inferior sagittal sinus

External jugular vein

The sigmoid sinus is joined by the inferior petrosal sinus to drain into the internal jugular vein.

Please rate this question:

Discuss and give feedback

Next question

Cranial venous sinuses

The cranial venous sinuses are located within the dura mater. They have no valves which is
important in the potential for spreading sepsis. They eventually drain into the internal jugular vein.

They are:

Superior sagittal sinus


Inferior sagittal sinus
Straight sinus
Transverse sinus
Sigmoid sinus
Confluence of sinuses
Occipital sinus
Cavernous sinus

Topography of cranial venous sinuses

Image sourced from Wikipedia

Next question
Question 198 of 560

Which nerve supplies the interossei of the fourth finger?

Radial

Median

Superficial ulnar

Deep ulnar

Posterior interosseous

Mnemonic:
PAD and DAB
Palmer interossei ADduct
Dorsal interossei ABduct

Theme from April 2013 exam


Theme from April 2014 exam

Please rate this question:

Discuss and give feedback

Next question

Interossei

Origin and insertion Nerve supply Actions


Origin and insertion Nerve supply Actions

Three palmar and four dorsal interossei occupy the spaces They are all Dorsal interossei
between the metacarpal bones. Each palmar interossei innervated by abduct the fingers,
originates from the metacarpal of the digit on which it acts. the ulnar nerve palmar interossei
Each dorsal interossei comes from the surface of the adjacent adduct the fingers
metacarpal on which it acts. As a result the dorsal interossei are
twice the size of the palmar ones.
The interossei tendons, except the first palmar, pass to one or
other side of the metacarpophalangeal joint posterior to the
deep transverse metacarpal ligament. They become inserted
into the base of the proximal phalanx and partly into the
extensor hood

Clinical notes
Along with the lumbricals the interossei flex the metacarpophalangeal joints and extend the proximal
and distal interphalangeal joints. They are responsible for fine tuning these movements. When the
interossei and lumbricals are paralysed the digits are pulled into hyperextension by extensor
digitorum and a claw hand is seen.

Next question
Question 199 of 560

In which of the following cranial bones does the foramen spinosum lie?

Sphenoid bone

Frontal bone

Temporal bone

Occipital bone

Parietal bone

The foramen spinosum (which transmits the middle meningeal artery and vein) lies in the sphenoid
bone.

Please rate this question:

Discuss and give feedback

Next question

Foramina of the base of the skull

Foramen Location Contents

Foramen ovale Sphenoid Otic ganglion


bone V3 (Mandibular nerve:3rd branch of
trigeminal)
Accessory meningeal artery
Foramen Location Contents

Lesser petrosal nerve


Emissary veins

Foramen spinosum Sphenoid Middle meningeal artery


bone Meningeal branch of the Mandibular nerve

Foramen rotundum Sphenoid Maxillary nerve (V2)


bone

Foramen lacerum/ Sphenoid Base of the medial pterygoid plate.


carotid canal bone Internal carotid artery*
Nerve and artery of the pterygoid canal

Jugular foramen Temporal Anterior: inferior petrosal sinus


bone Intermediate: glossopharyngeal, vagus, and accessory nerves.
Posterior: sigmoid sinus (becoming the internal jugular vein) and
some meningeal branches from the occipital and ascending
pharyngeal arteries.

Foramen magnum Occipital Anterior and posterior spinal arteries


bone Vertebral arteries
Medulla oblongata

Stylomastoid Temporal Stylomastoid artery


foramen bone Facial nerve

Superior orbital Sphenoid Oculomotor nerve (III)


fissure bone Recurrent meningeal artery
Trochlear nerve (IV)
Lacrimal, frontal and nasociliary branches of ophthalmic nerve (V1)
Abducent nerve (VI)
Foramen Location Contents

Superior ophthalmic vein

*= In life the foramen lacerum is occluded by a cartilagenous plug. The ICA initially passes into the
carotid canal which ascends superomedially to enter the cranial cavity through the foramen lacerum.

Base of skull anatomical overview

Image sourced from Wikipedia

Next question
Question 200 of 560

Which of the following is not considered a major branch of the descending thoracic aorta?

Bronchial artery

Mediastinal artery

Inferior thyroid artery

Posterior intercostal artery

Oesophageal artery

The inferior thyroid artery is usually derived from the thyrocervical trunk, a branch of the subclavian
artery.
Please rate this question:

Discuss and give feedback


Next question

Thoracic aorta

Origin T4

Terminates T12

Relations  Anteriorly (from top to bottom)-root of the left lung, the pericardium, the
oesophagus, and the diaphragm
 Posteriorly-vertebral column, azygos vein
 Right- hemiazygos veins, thoracic duct
 Left- left pleura and lung

Branches  Lateral segmental branches: Posterior intercostal arteries


 Lateral visceral: Bronchial arteries supply bronchial walls and lung
excluding the alveoli
 Midline branches: Oesophageal arteries

Next question
Question 201 of 560

An 18 year old lady with troublesome hyperhidrosis of the hands and arms is due to undergo a
sympathectomy to treat the condition. Which of the following should the surgeons divide to most
effectively treat her condition?

Sympathetic ganglia at T1, T2 and T3

Sympathetic ganglia at T2 and T3

Sympathetic ganglia at T1 and T2

Stellate ganglion

Superior cervical ganglion

To treat hyperhidrosis the sympathetic ganglia at T2 and T3 should be divided. Dividing the other
structures listed would either carry a risk of Horners syndrome or be ineffective.
Please rate this question:

Discuss and give feedback


Next question

Sympathetic nervous system- anatomy

The cell bodies of the pre-ganglionic efferent neurones lie in the lateral horn of the grey matter of the
spinal cord in the thoraco-lumbar regions.
The pre-ganglionic efferents leave the spinal cord at levels T1-L2. These pass to the sympathetic
chain.
Lateral branches of the sympathetic chain connect it to every spinal nerve. These post ganglionic
nerves will pass to structures that receive sympathetic innervation at the periphery.

Sympathetic chains
These lie on the vertebral column and run from the base of the skull to the coccyx.
Cervical Lie anterior to the transverse processes of the cervical vertebrae and posterior to
region the carotid sheath.

Thoracic Lie anterior to the neck of the upper ribs and and lateral sides of the lower thoracic
region vertebrae.They are covered by the parietal pleura

Lumbar Enter by passing posterior to the medial arcuate ligament. Lie anteriorly to the
region vertebrae and medial to psoas major.

Sympathetic ganglia

 Superior cervical ganglion lies anterior to C2 and C3.


 Middle cervical ganglion (if present) C6
 Stellate ganglion- anterior to transverse process of C7, lies posterior to the subclavian artery,
vertebral artery and cervical pleura.
 Thoracic ganglia are segmentally arranged.
 There are usually 4 lumbar ganglia.

Clinical importance

 Interruption of the head and neck supply of the sympathetic nerves will result in an ipsilateral
Horners syndrome.
 For treatment of hyperhidrosis the sympathetic denervation can be achieved by removing the
second and third thoracic ganglia with their rami. Removal of T1 will cause a Horners
syndrome and is therefore not performed.
 In patients with vascular disease of the lower limbs a lumbar sympathetomy may be
performed, either radiologically or (more rarely now) surgically. The ganglia of L2 and below
are disrupted. If L1 is removed then ejaculation may be compromised (and little additional
benefit conferred as the preganglionic fibres do not arise below L2.

Next question
Question 202 of 560

A 44 year old lady is recovering following a transphenoidal hypophysectomy. Unfortunately there is a


post operative haemorrhage. Which of the following features is most likely to occur initially?

Cavernous sinus thrombosis

Abducens nerve palsy

Bi-temporal hemianopia

Inferior homonymous hemianopia

Central retinal vein occlusion

Theme from April 2014 exam


The pituitary is covered by a sheath of dura and an expanding haematoma at this site may compress
the optic chiasm in the same manner as an expanding pituitary tumour.
Please rate this question:

Discuss and give feedback


Next question

Pituitary Gland

The pituitary gland is located within the sella turcica within the sphenoid bone in the middle cranial
fossa. It is covered by a dural fold and weighs around 0.5g. It is attached to the hypothalamus by the
infundibulum. The anterior pituitary receives hormonal stimuli from the hypothalamus by way of the
hypothalamo-pituitary portal system. It develops from a depression in the wall of the pharynx
(Rathkes pouch).

Anterior pituitary hormones

 Growth hormone
 Thyroid stimulating hormone
 ACTH
 Prolactin
 LH and FSH
 Melanocyte releasing hormone
Posterior pituitary hormones

 Oxytocin
 Anti diuretic hormone

Next question
Question 203 of 560

During a right hemicolectomy the caecum is mobilised. As the bowel is retracted medially a vessel is
injured, posterior to the colon. Which of the following is the most likely vessel?

Right colic artery

Inferior vena cava

Aorta

External iliac artery

Gonadal vessels

The gonadal vessels and ureter are important posterior relations that are at risk during a right
hemicolectomy.
Please rate this question:

Discuss and give feedback


Next question

Caecum

Location  Proximal right colon below the ileocaecal valve


 Intraperitoneal

Posterior relations  Psoas


 Iliacus
 Femoral nerve
 Genitofemoral nerve
 Gonadal vessels

Anterior relations Greater omentum

Arterial supply Ileocolic artery


Lymphatic drainage Mesenteric nodes accompany the venous drainage

 The caecum is the most distensible part of the colon and in complete large bowel obstruction
with a competent ileocaecal valve the most likely site of eventual perforation.

Next question
Question 204 of 560

A 53 year old man with a carcinoma of the lower third of the oesophagus is undergoing an
oesophagogastrectomy. As the surgeons mobilise the lower part of the oesophagus, where are they
most likely to encounter the thoracic duct?

Anterior to the oesophagus

On the left side of the oesophagus

On the right side of the oesophagus

Immediately anterior to the azygos vein

Posterior to the oesophagus

The thoracic duct lies posterior to the oesophagus and passes to the left at the level of the Angle of
Louis. It enters the thorax at T12 together with the aorta.
Please rate this question:

Discuss and give feedback


Next question

Thoracic duct

 Continuation of the cisterna chyli in the abdomen.


 Enters the thorax at T12.
 Lies posterior to the oesophagus for most of its intrathoracic course. Passes to the left at T5.
 Lymphatics draining the left side of the head and neck join the thoracic duct prior to its
insertion into the left brachiocephalic vein.
 Lymphatics draining the right side of the head and neck drain via the subclavian and jugular
trunks into the right lymphatic duct and thence into the mediastinal trunk and eventually the
right brachiocephalic vein.
 Its location in the thorax makes it prone to injury during oesophageal surgery. Some
surgeons administer cream to patients prior to oesophagectomy so that it is easier to identify
the cut ends of the duct.

Next question
Question 205 of 560

Which of the following represents the root values of the sciatic nerve?

L4 to S3

L1 to L4

L3 to S1

S1 to S4

L5 to S1

Theme from April 2014 exam


The sciatic nerve most commonly arises from L4 to S3.
Please rate this question:

Discuss and give feedback


Next question

Sciatic nerve

The sciatic nerve is formed from the sacral plexus and is the largest nerve in the body. It is the
continuation of the main part of the plexus arising from ventral rami of L4 to S3. These rami
converge at the inferior border of piriformis to form the nerve itself. It passes through the inferior part
of the greater sciatic foramen and emerges beneath piriformis. Medially, lie the inferior gluteal nerve
and vessels and the pudendal nerve and vessels. It runs inferolaterally under the cover of gluteus
maximus midway between the greater trochanter and ischial tuberosity. It receives its blood supply
from the inferior gluteal artery. The nerve provides cutaneous sensation to the skin of the foot and
the leg. It also innervates the posterior thigh muscles and the lower leg and foot muscles. The nerve
splits into the tibial and common peroneal nerves approximately half way down the posterior thigh.
The tibial nerve supplies the flexor muscles and the common peroneal nerve supplies the extensor
muscles and the abductor muscles.

Summary points
Origin Spinal nerves L4 - S3
Articular Branches Hip joint

Muscular branches in  Semitendinosus


upper leg  Semimembranosus
 Biceps femoris
 Part of adductor magnus

Cutaneous sensation  Posterior aspect of thigh (via cutaneous nerves)


 Gluteal region
 Entire lower leg (except the medial aspect)

Terminates At the upper part of the popliteal fossa by dividing into the tibial and
peroneal nerves

 The nerve to the short head of the biceps femoris comes from the common peroneal part of
the sciatic and the other muscular branches arise from the tibial portion.
 The tibial nerve goes on to innervate all muscles of the foot except the extensor digitorum
brevis (which is innervated by the common peroneal nerve).

Next question
Question 206 of 560

The common peroneal nerve, or its branches, supply the following muscles except:

Peroneus longus

Tibialis anterior

Extensor hallucis longus

Flexor digitorum brevis

Extensor digitorum longus

Flexor digitorum is supplied by the tibial nerve.

Please rate this question:

Discuss and give feedback

Next question

Common peroneal nerve

Derived from the dorsal divisions of the sacral plexus (L4, L5, S1 and S2).

This nerve supplies the skin and fascia of the anterolateral surface of the leg and the dorsum of the
foot. It also innervates the muscles of the anterior and peroneal compartments of the leg, extensor
digitorum brevis as well as the knee, ankle and foot joints.

It is laterally placed within the sciatic nerve. From the bifurcation of the sciatic nerve it passes
inferolaterally in the lateral and proximal part of the popliteal fossa, under the cover of biceps femoris
and its tendon. To reach the posterior aspect of the fibular head. It ends by dividing into the deep
and superficial peroneal nerves at the point where it winds around the lateral surface of the neck of
the fibula in the body of peroneus longus, approximately 2cm distal to the apex of the head of the
fibula. It is palpable posterior to the head of the fibula.

Branches

In the thigh Nerve to the short head of biceps


Articular branch (knee)

In the popliteal fossa Lateral cutaneous nerve of the calf

Neck of fibula Superficial and deep peroneal nerves

Next question
Question 207 of 560

An 83 year old lady presents with a femoral hernia and undergoes a femoral hernia repair. Which of
the following forms the posterior wall of the femoral canal?

Pectineal ligament

Lacunar ligament

Inguinal ligament

Adductor longus

Sartorius

Please rate this question:

Discuss and give feedback


Next question

Femoral canal

The femoral canal lies at the medial aspect of the femoral sheath. The femoral sheath is a fascial
tunnel containing both the femoral artery laterally and femoral vein medially. The canal lies medial to
the vein.

Borders of the femoral canal


Laterally Femoral vein

Medially Lacunar ligament

Anteriorly Inguinal ligament

Posteriorly Pectineal ligament


Image showing dissection of femoral canal

Image sourced from Wikipedia

Contents

 Lymphatic vessels
 Cloquet's lymph node

Physiological significance
Allows the femoral vein to expand to allow for increased venous return to the lower limbs.

Pathological significance
As a potential space, it is the site of femoral hernias. The relatively tight neck places these at high
risk of strangulation.
Next question
Question 208 of 560

A 45 year man presents with hand weakness. He is given a piece of paper to hold between his
thumb and index finger. When the paper is pulled, the patient has difficulty maintaining a grip. Grip
pressure is maintained by flexing the thumb at the interphalangeal joint. What is the most likely
nerve lesion?

Posterior interosseous nerve

Deep branch of ulnar nerve

Anterior interosseous nerve

Superficial branch of the ulnar nerve

Radial nerve

Theme from January 2012 exam

This is a description of Froment's sign, which tests for ulnar nerve palsy. It mainly tests for the
function of adductor pollicis. This is supplied by the deep branch of the ulnar nerve. Remember the
anterior interosseous branch (of the median nerve), which innervates the flexor pollicis longus
(hence causing flexion of the thumb IP joint), branches off more proximally to the wrist.
Please rate this question:

Discuss and give feedback


Next question

Ulnar nerve

Origin

 C8, T1

Supplies (no muscles in the upper arm)

 Flexor carpi ulnaris


 Flexor digitorum profundus
 Flexor digiti minimi
 Abductor digiti minimi
 Opponens digiti minimi
 Adductor pollicis
 Interossei muscle
 Third and fourth lumbricals
 Palmaris brevis

Path

 Posteromedial aspect of upper arm to flexor compartment of forearm, then along the ulnar.
Passes beneath the flexor carpi ulnaris muscle, then superficially through the flexor
retinaculum into the palm of the hand.

Image sourced from Wikipedia

Branches
Branch Supplies

Muscular branch Flexor carpi ulnaris


Medial half of the flexor digitorum profundus

Palmar cutaneous branch (Arises near the Skin on the medial part of the palm
middle of the forearm)

Dorsal cutaneous branch Dorsal surface of the medial part of the hand

Superficial branch Cutaneous fibres to the anterior surfaces of the


medial one and one-half digits

Deep branch Hypothenar muscles


All the interosseous muscles
Third and fourth lumbricals
Adductor pollicis
Medial head of the flexor pollicis brevis

Effects of injury
Damage at the wrist  Wasting and paralysis of intrinsic hand muscles (claw hand)
 Wasting and paralysis of hypothenar muscles
 Loss of sensation medial 1 and half fingers

Damage at the elbow  Radial deviation of the wrist


 Clawing less in 4th and 5th digits

Next question
Question 209 of 560

Which of the following statements relating to the right phrenic nerve is false?

It lies deep to the prevertebral layer of deep cervical fascia

Crosses posterior to the 2nd part of the subclavian artery

It runs on the anterior surface of the scalene muscle

On the right side it leaves the mediastinum via the vena cava hiatus at a level of T8

The right phrenic nerve passes over the right atrium

Please rate this question:

Discuss and give feedback


Next question

Phrenic nerve

Origin

 C3,4,5

Supplies

 Diaphragm, sensation central diaphragm and pericardium

Path

 The phrenic nerve passes with the internal jugular vein across scalenus anterior. It passes
deep to prevertebral fascia of deep cervical fascia.
 Left: crosses anterior to the 1st part of the subclavian artery.
 Right: Anterior to scalenus anterior and crosses anterior to the 2nd part of the subclavian
artery.
 On both sides, the phrenic nerve runs posterior to the subclavian vein and posterior to the
internal thoracic artery as it enters the thorax.

Right phrenic nerve

 In the superior mediastinum: anterior to right vagus and laterally to superior vena cava
 Middle mediastinum: right of pericardium
 It passes over the right atrium to exit the diaphragm at T8

Left phrenic nerve

 Passes lateral to the left subclavian artery, aortic arch and left ventricle
 Passes anterior to the root of the lung
 Pierces the diaphragm alone

Image showing the passage of the phrenic nerve in the neck

Image sourced from Wikipedia

Next question
Question 210 of 560

Which of the following cranial foramina pairings are incorrect?

Carotid canal and internal carotid artery.

Foramen ovale and mandibular nerve.

Optic canal and ophthalmic artery.

Optic canal and ophthalmic nerve.

Foramen rotundum and maxillary nerve.

Question derived from 2010 and 2011 exams

The optic canal transmits the optic nerve. The ophthalmic nerve traverses the superior orbital
fissure.

Please rate this question:

Discuss and give feedback

Next question

Foramina of the base of the skull

Foramen Location Contents

Foramen ovale Sphenoid Otic ganglion


V3 (Mandibular nerve:3rd branch of
Foramen Location Contents

bone trigeminal)
Accessory meningeal artery
Lesser petrosal nerve
Emissary veins

Foramen spinosum Sphenoid Middle meningeal artery


bone Meningeal branch of the Mandibular nerve

Foramen rotundum Sphenoid Maxillary nerve (V2)


bone

Foramen lacerum/ Sphenoid Base of the medial pterygoid plate.


carotid canal bone Internal carotid artery*
Nerve and artery of the pterygoid canal

Jugular foramen Temporal Anterior: inferior petrosal sinus


bone Intermediate: glossopharyngeal, vagus, and accessory nerves.
Posterior: sigmoid sinus (becoming the internal jugular vein) and
some meningeal branches from the occipital and ascending
pharyngeal arteries.

Foramen magnum Occipital Anterior and posterior spinal arteries


bone Vertebral arteries
Medulla oblongata

Stylomastoid Temporal Stylomastoid artery


foramen bone Facial nerve

Superior orbital Sphenoid Oculomotor nerve (III)


fissure bone Recurrent meningeal artery
Trochlear nerve (IV)
Lacrimal, frontal and nasociliary branches of ophthalmic nerve (V1)
Foramen Location Contents

Abducent nerve (VI)


Superior ophthalmic vein

*= In life the foramen lacerum is occluded by a cartilagenous plug. The ICA initially passes into the
carotid canal which ascends superomedially to enter the cranial cavity through the foramen lacerum.

Base of skull anatomical overview

Image sourced from Wikipedia

Next question
Question 211 of 560

A 22 year old man is involved in a fight and sustains a stab wound in his upper forearm. On
examination there is a small, but deep laceration. There is an obvious loss of pincer movement
involving the thumb and index finger with minimal loss of sensation. The most likely nerve injury is to
the:

Ulnar nerve

Radial nerve

Anterior interosseous nerve

Axillary nerve

Median nerve

The anterior interosseous nerve is a motor branch of the median nerve just below the elbow. When
damaged it classically causes:

 Pain in the forearm


 Loss of pincer movement of the thumb and index finger (innervates the long flexor muscles
of flexor pollicis longus & flexor digitorum profundus of the index and middle finger)
 Minimal loss of sensation due to lack of a cutaneous branch

Please rate this question:

Discuss and give feedback


Next question

Median nerve

The median nerve is formed by the union of a lateral and medial root respectively from the lateral
(C5,6,7) and medial (C8 and T1) cords of the brachial plexus; the medial root passes anterior to the
third part of the axillary artery. The nerve descends lateral to the brachial artery, crosses to its
medial side (usually passing anterior to the artery). It passes deep to the bicipital aponeurosis and
the median cubital vein at the elbow.
It passes between the two heads of the pronator teres muscle, and runs on the deep surface of
flexor digitorum superficialis (within its fascial sheath).
Near the wrist it becomes superficial between the tendons of flexor digitorum superficialis and flexor
carpi radialis, deep to palmaris longus tendon. It passes deep to the flexor retinaculum to enter the
palm, but lies anterior to the long flexor tendons within the carpal tunnel.

Branches
Region Branch

Upper arm No branches, although the nerve commonly communicates with the
musculocutaneous nerve

Forearm Pronator teres


Flexor carpi radialis
Palmaris longus
Flexor digitorum superficialis
Flexor pollicis longus
Flexor digitorum profundus (only the radial half)

Distal Palmar cutaneous branch


forearm

Hand Motor supply (LOAF)


(Motor)

 Lateral 2 lumbricals
 Opponens pollicis
 Abductor pollicis brevis
 Flexor pollicis brevis

Hand  Over thumb and lateral 2 ½ fingers


(Sensory)  On the palmar aspect this projects proximally, on the dorsal aspect only the
distal regions are innervated with the radial nerve providing the more
proximal cutaneous innervation.

Patterns of damage
Damage at wrist

 e.g. carpal tunnel syndrome


 paralysis and wasting of thenar eminence muscles and opponens pollicis (ape hand
deformity)
 sensory loss to palmar aspect of lateral (radial) 2 ½ fingers
Damage at elbow, as above plus:

 unable to pronate forearm


 weak wrist flexion
 ulnar deviation of wrist

Anterior interosseous nerve (branch of median nerve)

 leaves just below the elbow


 results in loss of pronation of forearm and weakness of long flexors of thumb and index
finger

Topography of the median nerve

Image sourced from Wikipedia

Next question
Question 212 of 560

A 66 year old man is undergoing a left nephro-ureterectomy. The surgeons remove the ureter, which
of the following is responsible for the blood supply to the proximal ureter?

Branches of the renal artery

External iliac artery

Internal iliac artery

Direct branches from the aorta

Common iliac artery

Theme from April 2014 exam


The proximal ureter is supplied by branches from the renal artery. For the other feeding vessels -
see below.
Please rate this question:

Discuss and give feedback


Next question

Ureter

 25-35 cm long
 Muscular tube lined by transitional epithelium
 Surrounded by thick muscular coat. Becomes 3 muscular layers as it crosses the bony pelvis
 Retroperitoneal structure overlying transverse processes L2-L5
 Lies anterior to bifurcation of iliac vessels
 Blood supply is segmental; renal artery, aortic branches, gonadal branches, common iliac
and internal iliac
 Lies beneath the uterine artery

Next question
Question 213 of 560

Which of the following structures does not pass behind the lateral malleolus?

Peroneus brevis tendon

Sural nerve

Short saphenous vein

Peroneus longus tendon

Tibialis anterior tendon

Theme from April 2014 exam


Tibialis anterior tendon passes anterior to the medial malleolus.
Please rate this question:

Discuss and give feedback


Next question

Lateral malleolus

Structures posterior to the lateral malleolus and superficial to superior peroneal retinaculum

 Sural nerve
 Short saphenous vein

Structures posterior to the lateral malleolus and deep to superior peroneal retinaculum

 Peroneus longus tendon


 Peroneus brevis tendon

The calcaneofibular ligament is attached at the lateral malleolus


Next question

Question 214 of 560


A 78 year old man presents with symptoms consistent with intermittent claudication. To assess the
severity of his disease you decide to measure his ankle brachial pressure index. To do this you will
identify the dorsalis pedis artery. Which of the following statements relating to this vessel is false?

It originates from the peroneal artery

It is crossed by the tendon of extensor hallucis brevis

Two veins are usually closely related to it

It passes under the inferior extensor retinaculum

The tendon of extensor hallucis longus lies medial to it.

The dorsalis pedis artery is a direct continuation of the anterior tibial artery.
Please rate this question:

Discuss and give feedback


Next question

Foot- anatomy

Arches of the foot


The foot is conventionally considered to have two arches.

 The longitudinal arch is higher on the medial than on the lateral side. The posterior part of
the calcaneum forms a posterior pillar to support the arch. The lateral part of this structure
passes via the cuboid bone and the lateral two metatarsal bones. The medial part of this
structure is more important. The head of the talus marks the summit of this arch, located
between the sustentaculum tali and the navicular bone. The anterior pillar of the medial arch
is composed of the navicular bone, the three cuneiforms and the medial three metatarsal
bones.
 The transverse arch is situated on the anterior part of the tarsus and the posterior part of the
metatarsus. The cuneiforms and metatarsal bases narrow inferiorly, which contributes to the
shape of the arch.

Intertarsal joints
Sub talar joint Formed by the cylindrical facet on the lower surface of the body of the
talus and the posterior facet on the upper surface of the calcaneus. The
facet on the talus is concave anteroposteriorly, the other is convex. The
synovial cavity of this joint does not communicate with any other joint.

Talocalcaneonavicular The anterior part of the socket is formed by the concave articular
joint surface of the navicular bone, posteriorly by the upper surface of the
sustentaculum tali. The talus sits within this socket

Calcaneocuboid joint Highest point in the lateral part of the longitudinal arch. The lower
aspect of this joint is reinforced by the long plantar and plantar
calcaneocuboid ligaments.

Transverse tarsal joint The talocalcaneonavicular joint and the calcaneocuboid joint extend
across the tarsus in an irregular transverse plane, between the talus and
calcaneus behind and the navicular and cuboid bones in front. This
plane is termed the transverse tarsal joint.

Cuneonavicular joint Formed between the convex anterior surface of the navicular bone and
the concave surface of the the posterior ends of the three cuneiforms.

Intercuneiform joints Between the three cuneiform bones.

Cuneocuboid joint Between the circular facets on the lateral cuneiform bone and the
cuboid. This joint contributes to the tarsal part of the transverse arch.

A detailed knowledge of the joints is not required for MRCS Part A. However, the contribution they
play to the overall structure of the foot should be appreciated

Ligaments of the ankle joint and foot


Image sourced from Wikipedia

Muscles of the foot


Muscle Origin Insertion Nerve Action
supply

Abductor Medial side of the calcaneus, Medial side of Medial Abducts the great toe
hallucis flexor retinaculum, plantar the base of the plantar
aponeurosis proximal nerve
phalanx

Flexor Medial process of the Via 4 tendons Medial Flexes all the joints of
digitorum calcaneus, plantar into the plantar the lateral 4 toes except
brevis eponeurosis. middle nerve for the interphalangeal
phalanges of joint.
the lateral 4
toes.

Abductor From the tubercle of the Together with Lateral Abducts the little toe at
digit calcaneus and from the flexor digit plantar the metatarsophalangeal
minimi plantar aponeurosis minimi brevis nerve joint
into the lateral
side of the
base of the
proximal
phalanx of the
little toe

Flexor From the medial side of the Into the Medial Flexes the
hallucis plantar surface of the cuboid proximal plantar metatarsophalangeal
brevis bone, from the adjacent part phalanx of the nerve joint of the great toe.
of the lateral cuneiform bone great toe, the
and from the tendon of tendon
tibialis posterior. contains a
sesamoid bone

Adductor Arises from two heads. The Lateral side of Lateral Adducts the great toe
hallucis oblique head arises from the the base of the plantar towards the second toe.
sheath of the peroneus longus proximal nerve Helps maintain the
tendon, and from the plantar phalanx of the transverse arch of the
surfaces of the bases of the great toe. foot.
2nd, 3rd and 4th metatarsal
bones. The transverse head
arises from the plantar
surface of the lateral 4
metatarsophalangeal joints
and from the deep transverse
metatarsal ligament.

Extensor On the dorsal surface of the Via four thin Deep Extend the
digitorum foot from the upper surface of tendons which peroneal metatarsophalangeal
brevis the calcaneus and its run forward joint of the medial four
associated fascia and medially toes. It is unable to
to be inserted extend the
into the interphalangeal joint
medial four without the assistance of
toes. The the lumbrical muscles.
lateral three
tendons join
with hoods of
extensor
digitorum
longus.

Nerves in the foot


Lateral plantar nerve
Passes anterolaterally towards the base of the 5th metatarsal between flexor digitorum brevis and
flexor accessorius. On the medial aspect of the lateral plantar artery. At the base of the 5th
metatarsal it splits into superficial and deep branches.

Medial plantar nerve


Passes forwards with the medial plantar artery under the cover of the flexor retinaculum to the
interval between abductor hallucis and flexor digitorum brevis on the sole of the foot.

Plantar arteries
Arise under the cover of the flexor retinaculum, midway between the tip of the medial malleolus and
the most prominent part of the medial side of the heel.

 Medial plantar artery. Passes forwards medial to medial plantar nerve in the space between
abductor hallucis and flexor digitorum brevis.Ends by uniting with a branch of the 1st plantar
metatarsal artery.
 Lateral plantar artery. Runs obliquely across the sole of the foot. It lies lateral to the lateral
plantar nerve. At the base of the 5th metatarsal bone it arches medially across the foot on
the metatarsals

Dorsalis pedis artery


This vessel is a direct continuation of the anterior tibial artery. It commences on the front of the ankle
joint and runs to the proximal end of the first metatarsal space. Here is gives off the arcuate artery
and continues forwards as the first dorsal metatarsal artery. It is accompanied by two veins
throughout its length. It is crossed by the extensor hallucis brevis
Next question
Question 215 of 560

Which of the following is not a content of the anterior triangle of the neck?

Vagus nerve

Submandibular gland

Phrenic nerve

Internal jugular vein

Hypoglossal nerve

The phrenic nerve is a content of the posterior triangle. The anterior triangle contains the carotid
sheath and its contents.
Please rate this question:

Discuss and give feedback


Next question

Anterior triangle of the neck

Boundaries

Anterior border of the Sternocleidomastoid


Lower border of mandible
Anterior midline

Sub triangles (divided by Digastric above and Omohyoid)

 Muscular triangle: Neck strap muscles


 Carotid triangle: Carotid sheath
 Submandibular Triangle (digastric)
Contents of the anterior triangle
Digastric triangle Submandibular gland
Submandibular nodes
Facial vessels
Hypoglossal nerve

Muscular triangle Strap muscles


External jugular vein

Carotid triangle Carotid sheath (Common carotid, vagus and internal jugular vein)
Ansa cervicalis

Nerve supply to digastric muscle

 Anterior: Mylohyoid nerve


 Posterior: Facial nerve

Image sourced from Wikipedia

Next question
Question 216 of 560

A 32 year old attends neurology clinic complaining of tingling in his hand. He has radial deviation of
his wrist and there is mild clawing of his fingers, with the 4th and 5th digits being relatively spared.
What is the most likely lesion?

Ulnar nerve damage at the wrist

Ulnar nerve damage at the elbow

Radial nerve damage at the elbow

Median nerve damage at the wrist

Median nerve damage at the elbow

The ulnar paradox- the higher the lesion, the less the clawing of the fingers seen clinically.

At the elbow the ulnar nerve lesion affects the flexor carpi ulnaris and flexor digitorum profundus.
Please rate this question:

Discuss and give feedback


Next question

Ulnar nerve

Origin

 C8, T1

Supplies (no muscles in the upper arm)

 Flexor carpi ulnaris


 Flexor digitorum profundus
 Flexor digiti minimi
 Abductor digiti minimi
 Opponens digiti minimi
 Adductor pollicis
 Interossei muscle
 Third and fourth lumbricals
 Palmaris brevis

Path

 Posteromedial aspect of upper arm to flexor compartment of forearm, then along the ulnar.
Passes beneath the flexor carpi ulnaris muscle, then superficially through the flexor
retinaculum into the palm of the hand.

Image sourced from Wikipedia

Branches
Branch Supplies
Branch Supplies

Muscular branch Flexor carpi ulnaris


Medial half of the flexor digitorum profundus

Palmar cutaneous branch (Arises near the Skin on the medial part of the palm
middle of the forearm)

Dorsal cutaneous branch Dorsal surface of the medial part of the hand

Superficial branch Cutaneous fibres to the anterior surfaces of the


medial one and one-half digits

Deep branch Hypothenar muscles


All the interosseous muscles
Third and fourth lumbricals
Adductor pollicis
Medial head of the flexor pollicis brevis

Effects of injury
Damage at the wrist  Wasting and paralysis of intrinsic hand muscles (claw hand)
 Wasting and paralysis of hypothenar muscles
 Loss of sensation medial 1 and half fingers

Damage at the elbow  Radial deviation of the wrist


 Clawing less in 4th and 5th digits

Next question
Question 217 of 560

A 22 year old man is undergoing an endotracheal intubation. Which of the following vertebral levels
is consistent with the origin of the trachea?

C2

T1

C6

C4

C3

The trachea commences at C6. It terminates at the level of T5 (or T6 in tall subjects in deep
inspiration).
Please rate this question:

Discuss and give feedback


Next question

Trachea

Trachea
Location C6 vertebra to the upper border of T5 vertebra (bifurcation)

Arterial and venous supply Inferior thyroid arteries and the thyroid venous plexus.

Nerve Branches of vagus, sympathetic and the recurrent nerves

Relations in the neck


Anterior(Superior to  Isthmus of the thyroid gland
 Inferior thyroid veins
inferior)  Arteria thyroidea ima (when that vessel exists)
 Sternothyroid
 Sternohyoid
 Cervical fascia
 Anastomosing branches between the anterior jugular
veins

Posterior Oesophagus.

Laterally  Common carotid arteries


 Right and left lobes of the thyroid gland
 Inferior thyroid arteries
 Recurrent laryngeal nerves

Relations in the thorax

Anterior

 Manubrium, the remains of the thymus, the aortic arch, left common carotid arteries, and the
deep cardiac plexus

Lateral

 In the superior mediastinum, on the right side is the pleura and right vagus; on its left side
are the left recurrent nerve, the aortic arch, and the left common carotid and subclavian
arteries.

Next question
Question 218 of 560

A young child undergoes a difficult craniotomy for fulminant mastoiditis and associated abscess.
During the procedure the trigeminal nerve is severely damaged within Meckels cave. Which deficit is
least likely to be present?

Anaesthesia over the ipsilateral anterior aspect of the scalp

Loss of the corneal reflex

Weakness of the ipsilateral masseter muscle

Anaesthesia of the anterior aspect of the lip

Anaesthesia over the entire ipsilateral side of the face

The angle of the jaw is not innervated by sensory fibres of the trigeminal nerve and is spared in this
type of injury.
Remember the trigeminal nerve provides motor innervation to the muscles of mastication. The close
proximity of the site of injury to the motor fibres is likely to result in at least some compromise of
motor muscle function.
Please rate this question:

Discuss and give feedback


Next question

Trigeminal nerve

The trigeminal nerve is the main sensory nerve of the head. In addition to its major sensory role, it
also innervates the muscles of mastication.

Distribution of the trigeminal nerve


Sensory  Scalp
 Face
 Oral cavity (and teeth)
 Nose and sinuses
 Dura mater
Motor  Muscles of mastication
 Mylohyoid
 Anterior belly of digastric
 Tensor tympani
 Tensor palati

Autonomic connections (ganglia)  Ciliary


 Sphenopalatine
 Otic
 Submandibular

Path

 Originates at the pons


 Sensory root forms the large, crescentic trigeminal ganglion within Meckel's cave, and
contains the cell bodies of incoming sensory nerve fibres. Here the 3 branches exit.
 The motor root cell bodies are in the pons and the motor fibres are distributed via the
mandibular nerve. The motor root is not part of the trigeminal ganglion.

Branches of the trigeminal nerve


Ophthalmic nerve Sensory only

Maxillary nerve Sensory only

Mandibular nerve Sensory and motor

Sensory
Ophthalmic Exits skull via the superior orbital fissure
Sensation of: scalp and forehead, the upper eyelid, the conjunctiva and cornea of
the eye, the nose (including the tip of the nose, except alae nasi), the nasal
mucosa, the frontal sinuses, and parts of the meninges (the dura and blood
vessels).

Maxillary Exit skull via the foramen rotundum


nerve Sensation: lower eyelid and cheek, the nares and upper lip, the upper teeth and
gums, the nasal mucosa, the palate and roof of the pharynx, the maxillary,
ethmoid and sphenoid sinuses, and parts of the meninges.

Mandibular Exit skull via the foramen ovale


nerve Sensation: lower lip, the lower teeth and gums, the chin and jaw (except the angle
of the jaw), parts of the external ear, and parts of the meninges.

Motor
Distributed via the mandibular nerve.
The following muscles of mastication are innervated:

 Masseter
 Temporalis
 Medial pterygoid
 Lateral pterygoid

Other muscles innervated include:

 Tensor veli palatini


 Mylohyoid
 Anterior belly of digastric
 Tensor tympani

Next question
Question 219-221 of 560

Theme: Nerve lesions

A. Iliohypogastric nerve
B. Ilioinguinal nerve
C. Lateral cutaneous nerve of the thigh
D. Femoral nerve
E. Saphenous nerve
F. Genitofemoral nerve

Please select the most likely nerve implicated in the situation described. Each option may be used
once, more than once or not at all.

219. A 42 year old woman complains of a burning pain of her anterior thigh which worsens on
walking. There is a positive tinel sign over the inguinal ligament.

You answered Iliohypogastric nerve

The correct answer is Lateral cutaneous nerve of the thigh

The lateral cutaneous nerve supplies sensation to the anterior and lateral aspect of the
thigh. Entrapment is commonly due to intra and extra pelvic causes. Treatment involves
local anaesthetic injections.

220. A 29 year old woman has had a Pfannenstiel incision. She has pain over the inguinal
ligament which radiates to the lower abdomen. There is tenderness when the inguinal
canal is compressed.

You answered Iliohypogastric nerve

The correct answer is Ilioinguinal nerve

221. A 22 year man is shot in the groin. On examination he has weak hip flexion, weak knee
extension, and impaired quadriceps tendon reflex, as well as sensory deficit in the
anteromedial aspect of the thigh.

You answered Iliohypogastric nerve

The correct answer is Femoral nerve

This is a classical description of a femoral nerve injury.


Please rate this question:

Discuss and give feedback


Next question

Nerve lesions during surgery

A variety of different procedures carry the risk of iatrogenic nerve injury. These are important not
only from the patients perspective but also from a medicolegal standpoint.

The following operations and their associated nerve lesions are listed here:

 Posterior triangle lymph node biopsy and accessory nerve lesion.


 Lloyd Davies stirrups and common peroneal nerve.
 Thyroidectomy and laryngeal nerve.
 Anterior resection of rectum and hypogastric autonomic nerves.
 Axillary node clearance; long thoracic nerve, thoracodorsal nerve and intercostobrachial
nerve.
 Inguinal hernia surgery and ilioinguinal nerve.
 Varicose vein surgery- sural and saphenous nerves.
 Posterior approach to the hip and sciatic nerve.
 Carotid endarterectomy and hypoglossal nerve.

There are many more, with sound anatomical understanding of the commonly performed procedures
the incidence of nerve lesions can be minimised. They commonly occur when surgeons operate in
an unfamiliar tissue plane or by blind placement of haemostats (not recommended).
Next question
Question 222 of 560

Which of the following is not a branch of the external carotid artery?

Facial artery

Lingual artery

Superior thyroid artery

Mandibular artery

Maxillary artery

External carotid artery branches mnemonic:

'Some Angry Lady Figured Out PMS'

Superior thyroid (superior laryngeal artery branch)


Ascending pharyngeal
Lingual
Facial (tonsillar and labial artery)
Occipital
Posterior auricular
Maxillary (inferior alveolar artery, middle meningeal artery)
Superficial temporal

Please rate this question:

Discuss and give feedback

Next question
External carotid artery

The external carotid commences immediately lateral to the pharyngeal side wall. It ascends and lies
anterior to the internal carotid and posterior to the posterior belly of digastric and stylohyoid. More
inferiorly it is covered by sternocleidomastoid, passed by hypoglossal nerves, lingual and facial
veins.
It then pierces the fascia of the parotid gland finally dividing into its terminal branches within the
gland itself.

Surface marking of the carotid


This is an imaginary line drawn from the bifurcation of the common carotid passing behind the angle
of the jaw to a point immediately anterior to the tragus of the ear.

Branches of the external carotid artery


It has six branches, three in front, two behind and one deep.

Three in front Superior thyroid


Lingual
Facial

Two behind Occipital


Posterior auricular

Deep Ascending pharyngeal

It terminates by dividing into the superficial temporal and maxillary arteries in the parotid gland.
Image sourced from Wikipedia

Next question
Question 223 of 560

A 23 year old man is stabbed in the groin, several structures are injured and the adductor longus
muscle has been lacerated. Which of the following nerves is responsible for the innervation of
adductor longus?

Femoral nerve

Obturator nerve

Sciatic nerve

Common peroneal nerve

Ilioinguinal nerve

The adductors are innervated by the obturator nerve

Please rate this question:

Discuss and give feedback

Next question

Adductor longus

Origin Anterior body of pubis

Insertion Middle third of linea aspera


Action Adducts and flexes the thigh, medially rotate the hip

Innervation Anterior division of obturator nerve (L2, L3, L4)

The schematic image below demonstrates the relationship of the adductor muscles

Image sourced from Wikipedia

Next question
Question 224 of 560

Which of the following statements relating to the basilar artery and its branches is false?

The superior cerebellar artery may be decompressed to treat trigeminal neuralgia

Occlusion of the posterior cerebral artery causes contralateral loss of the visual field

The oculomotor nerve lies between the superior cerebellar and posterior cerebral arteries

The posterior inferior cerebellar artery is the largest of the cerebellar arteries arising from
the basilar artery

The labyrinthine branch is accompanied by the facial nerve

The posterior inferior cerebellar artery is the largest of the cerebellar arteries arising from the
vertebral artery. The labyrinthine artery is long and slender and may arise from the lower part of the
basilar artery. It accompanies the facial and vestibulocochlear nerves into the internal auditory
meatus. The posterior cerebral artery is often larger than the superior cerebellar artery and it is
separated from the vessel, near it's origin, by the oculomotor nerve. Arterial decompression is a well
established therapy for trigeminal neuralgia.
Please rate this question:

Discuss and give feedback


Next question

Circle of Willis

The two internal carotid arteries and two vertebral arteries form an anastomosis known as the Circle
of Willis on the inferior surface of the brain. Each half of the circle is formed by:
1. Anterior communicating artery
2. Anterior cerebral artery
3. Internal carotid artery
4. Posterior communicating artery
5. Posterior cerebral arteries and the termination of the basilar artery

The circle and its branches supply; the corpus striatum, internal capsule, diencephalon and
midbrain.
Image sourced from Wikipedia

Vertebral arteries

 Enter the cranial cavity via foramen magnum


 Lie in the subarachnoid space
 Ascend on anterior surface of medulla oblongata
 Unite to form the basilar artery at the base of the pons

Branches:

 Posterior spinal artery


 Anterior spinal artery
 Posterior inferior cerebellar artery

Basilar artery
Branches:
 Anterior inferior cerebellar artery
 Labyrinthine artery
 Pontine arteries
 Superior cerebellar artery
 Posterior cerebral artery

Internal carotid arteries


Branches:

 Posterior communicating artery


 Anterior cerebral artery
 Middle cerebral artery
 Anterior choroid artery

Next question
Question 225 of 560

Which of the following muscles does not recieve any innervation from the sciatic nerve?

Semimembranosus

Quadriceps femoris

Biceps femoris

Semitendinosus

Adductor magnus

The sciatic nerve is traditionally viewed as being a nerve of the posterior compartment. It is known to
contribute to the innervation of adductor magnus (although the main innervation to this muscle is
from the obturator nerve). The quadriceps femoris is nearly always innervated by the femoral nerve.
Please rate this question:

Discuss and give feedback


Next question

Sciatic nerve

The sciatic nerve is formed from the sacral plexus and is the largest nerve in the body. It is the
continuation of the main part of the plexus arising from ventral rami of L4 to S3. These rami
converge at the inferior border of piriformis to form the nerve itself. It passes through the inferior part
of the greater sciatic foramen and emerges beneath piriformis. Medially, lie the inferior gluteal nerve
and vessels and the pudendal nerve and vessels. It runs inferolaterally under the cover of gluteus
maximus midway between the greater trochanter and ischial tuberosity. It receives its blood supply
from the inferior gluteal artery. The nerve provides cutaneous sensation to the skin of the foot and
the leg. It also innervates the posterior thigh muscles and the lower leg and foot muscles. The nerve
splits into the tibial and common peroneal nerves approximately half way down the posterior thigh.
The tibial nerve supplies the flexor muscles and the common peroneal nerve supplies the extensor
muscles and the abductor muscles.

Summary points
Origin Spinal nerves L4 - S3
Articular Branches Hip joint

Muscular branches in  Semitendinosus


upper leg  Semimembranosus
 Biceps femoris
 Part of adductor magnus

Cutaneous sensation  Posterior aspect of thigh (via cutaneous nerves)


 Gluteal region
 Entire lower leg (except the medial aspect)

Terminates At the upper part of the popliteal fossa by dividing into the tibial and
peroneal nerves

 The nerve to the short head of the biceps femoris comes from the common peroneal part of
the sciatic and the other muscular branches arise from the tibial portion.
 The tibial nerve goes on to innervate all muscles of the foot except the extensor digitorum
brevis (which is innervated by the common peroneal nerve).

Next question
Question 226 of 560

A 23 year old man is involved in a fight and is stabbed in his upper arm. The ulnar nerve is
transected. Which of the following muscles will not demonstrate compromised function as a result?

Flexor carpi ulnaris

Medial half of flexor digitorum profundus

Palmaris brevis

Hypothenar muscles

Pronator teres

M edial lumbricals
A dductor pollicis
F lexor digitorum profundus/Flexor digiti minimi
I nterossei
A bductor digiti minimi and opponens

Innervates all intrinsic muscles of the hand (EXCEPT 2: thenar muscles & first two lumbricals -
supplied by median nerve)

Pronator teres is innervated by the median nerve. Palmaris brevis is innervated by the ulnar nerve
Please rate this question:

Discuss and give feedback


Next question

Ulnar nerve

Origin

 C8, T1

Supplies (no muscles in the upper arm)


 Flexor carpi ulnaris
 Flexor digitorum profundus
 Flexor digiti minimi
 Abductor digiti minimi
 Opponens digiti minimi
 Adductor pollicis
 Interossei muscle
 Third and fourth lumbricals
 Palmaris brevis

Path

 Posteromedial aspect of upper arm to flexor compartment of forearm, then along the ulnar.
Passes beneath the flexor carpi ulnaris muscle, then superficially through the flexor
retinaculum into the palm of the hand.

Image sourced from Wikipedia


Branches
Branch Supplies

Muscular branch Flexor carpi ulnaris


Medial half of the flexor digitorum profundus

Palmar cutaneous branch (Arises near the Skin on the medial part of the palm
middle of the forearm)

Dorsal cutaneous branch Dorsal surface of the medial part of the hand

Superficial branch Cutaneous fibres to the anterior surfaces of the


medial one and one-half digits

Deep branch Hypothenar muscles


All the interosseous muscles
Third and fourth lumbricals
Adductor pollicis
Medial head of the flexor pollicis brevis

Effects of injury
Damage at the wrist  Wasting and paralysis of intrinsic hand muscles (claw hand)
 Wasting and paralysis of hypothenar muscles
 Loss of sensation medial 1 and half fingers

Damage at the elbow  Radial deviation of the wrist


 Clawing less in 4th and 5th digits

Next question
Question 227 of 560

Which of the structures listed below overlies the cephalic vein?

Extensor retinaculum

Bicipital aponeurosis

Biceps muscle

Antebrachial fascia

None of the above

The cephalic vein is superficially located in the upper limb and overlies most the fascial planes. It
pierces the coracoid membrane (continuation of the clavipectoral fascia) to terminate in the axillary
vein. It lies anterolaterally to biceps.
Please rate this question:

Discuss and give feedback


Next question

Cephalic vein

Path

 Dorsal venous arch drains laterally into the cephalic vein


 Crosses the anatomical snuffbox and travels laterally up the arm
 At the antecubital fossa connected to the basilic vein by the median cubital vein
 Pierces deep fascia of deltopectoral groove to join axillary vein

Next question
Question 228 of 560

Which of the following pairings are incorrect?

Aortic bifurcation and L4

Transpyloric plane and L1

Termination of dural sac and L4

Oesophageal passage through diaphragm and T10

Transition between pharynx and oesophagus at C6

Vena cava T8 (eight letters)


Oesophagus T10 (ten letters)
Aortic hiatus T12 (twelve letters)

It terminates at S2, which is why it is safe to undertake an LP at L4/5 levels. The spinal cord itself
terminates at L1.
Please rate this question:

Discuss and give feedback


Next question

Levels

Transpyloric plane
Level of the body of L1

 Pylorus stomach
 Left kidney hilum (L1- left one!)
 Fundus of the gallbladder
 Neck of pancreas
 Duodenojejunal flexure
 Superior mesenteric artery
 Portal vein
 Left and right colic flexure
 Root of the transverse mesocolon
 2nd part of the duodenum
 Upper part of conus medullaris
 Spleen

Can be identified by asking the supine patient to sit up without using their arms. The plane is located
where the lateral border of the rectus muscle crosses the costal margin.

Anatomical planes
Subcostal plane Lowest margin of 10th costal cartilage

Intercristal plane Level of body L4 (highest point of iliac crest)

Intertubercular plane Level of body L5

Common level landmarks


Inferior mesenteric artery L3

Bifurcation of aorta into common iliac arteries L4

Formation of IVC L5 (union of common iliac veins)

Diaphragm apertures  Vena cava T8


 Oesophagus T10
 Aortic hiatus T12

Next question
Question 229 of 560

A 22 year old man is involved in a fight. He sustains a laceration to the posterior aspect of his wrist.
In the emergency department the wound is explored and the laceration is found to be transversely
orientated and overlies the region of the extensor retinaculum, which is intact. Which of the following
structures is least likely to be injured in this scenario?

Dorsal cutaneous branch of the ulnar nerve

Tendon of extensor indicis

Basilic vein

Superficial branch of the radial nerve

Cephalic vein

The extensor retinaculum attaches to the radius proximal to the styloid, thereafter it runs obliquely
and distally to wind around the ulnar styloid (but does not attach to it). The extensor tendons lie deep
to the extensor retinaculum and would therefore be less susceptible to injury than the superficial
structures.
Please rate this question:

Discuss and give feedback


Next question

Extensor retinaculum

The extensor rentinaculum is a thickening of the deep fascia that stretches across the back of the
wrist and holds the long extensor tendons in position.
Its attachments are:

 The pisiform and triquetral medially


 The end of the radius laterally

Structures related to the extensor retinaculum


Structures superficial to the  Basilic vein
 Dorsal cutaneous branch of the ulnar nerve
retinaculum  Cephalic vein
 Superficial branch of the radial nerve

Structures passing deep to the  Extensor carpi ulnaris tendon


extensor retinaculum  Extensor digiti minimi tendon
 Extensor digitorum and extensor indicis tendon
 Extensor pollicis longus tendon
 Extensor carpi radialis longus tendon
 Extensor carpi radialis brevis tendon
 Abductor pollicis longus and extensor pollicis
brevis tendons

Beneath the extensor retinaculum fibrous septa form six compartments that contain the extensor
muscle tendons. Each compartment has its own synovial sheath.

The radial artery


The radial artery passes between the lateral collateral ligament of the wrist joint and the tendons of
the abductor pollicis longus and extensor pollicis brevis.

Image illustrating the topography of tendons passing under the extensor retinaculum
Image sourced from Wikipedia

Next question
Question 230 of 560

Which of the following is not a content of the porta hepatis?

Portal vein

Hepatic artery

Cystic duct

Lymph nodes

None of the above

The cystic duct lies outside the porta hepatis and is an important landmark in laparoscopic
cholecystectomy. The structures in the porta hepatis are:

 Portal vein
 Hepatic artery
 Common hepatic duct

These structures divide immediately after or within the porta hepatis to supply the functional left and
right lobes of the liver.
The porta hepatis is also surrounded by lymph nodes, that may enlarge to produce obstructive
jaundice and parasympathetic nervous fibres that travel along vessels to enter the liver.
Please rate this question:

Discuss and give feedback


Next question

Liver

Structure of the liver


Right lobe  Supplied by right hepatic artery
 Contains Couinaud segments V to VIII (-/+Sg I)

Left lobe  Supplied by the left hepatic artery


 Contains Couinaud segments II to IV (+/- Sg1)

Quadrate lobe  Part of the right lobe anatomically, functionally is part of the left
 Couinaud segment IV
 Porta hepatis lies behind
 On the right lies the gallbladder fossa
 On the left lies the fossa for the umbilical vein

Caudate lobe  Supplied by both right and left hepatic arteries


 Couinaud segment I
 Lies behind the plane of the porta hepatis
 Anterior and lateral to the inferior vena cava
 Bile from the caudate lobe drains into both right and left hepatic ducts

Detailed knowledge of Couinaud segments is not required for MRCS

 Between the liver lobules are portal canals which contain the portal triad: Hepatic Artery,
Portal Vein, tributary of Bile Duct.

Relations of the liver


Anterior Postero inferiorly

Diaphragm Oesophagus

Xiphoid process Stomach

Duodenum

Hepatic flexure of colon

Right kidney
Gallbladder

Inferior vena cava

Porta hepatis
Location Postero inferior surface, it joins nearly at right angles with the left sagittal fossa, and
separates the caudate lobe behind from the quadrate lobe in front

Transmits  Common hepatic duct


 Hepatic artery
 Portal vein
 Sympathetic and parasympathetic nerve fibres
 Lymphatic drainage of the liver (and nodes)

Ligaments
Falciform ligament  2 layer fold peritoneum from the umbilicus to anterior liver surface
 Contains ligamentum teres (remnant umbilical vein)
 On superior liver surface it splits into the coronary and left
triangular ligaments

Ligamentum teres Joins the left branch of the portal vein in the porta hepatis

Ligamentum Remnant of ductus venosus


venosum

Arterial supply

 Hepatic artery

Venous

 Hepatic veins
 Portal vein

Nervous supply
 Sympathetic and parasympathetic trunks of coeliac plexus

Next question
Question 231 of 560

Which of the following structures is not closely related to the carotid sheath?

Sternothyroid muscle

Sternohyoid muscle

Hypoglossal nerve

Superior belly of omohyoid muscle

Anterior belly of digastric muscle

At its lower end the carotid sheath is related to sternohyoid and sternothyroid. Opposite the cricoid
cartilage the sheath is crossed by the superior belly of omohyoid. Above this level the sheath is
covered by the sternocleidomastoid muscle. Above the level of the hyoid the vessels pass deep to
the posterior belly of digastric and stylohyoid. Opposite the hyoid bone the sheath is crossed
obliquely by the hypoglossal nerve.
Please rate this question:

Discuss and give feedback


Next question

Common carotid artery

The right common carotid artery arises at the bifurcation of the brachiocephalic trunk, the left
common carotid arises from the arch of the aorta. Both terminate at the level of the upper border of
the thyroid cartilage (the lower border of the third cervical vertebra) by dividing into the internal and
external carotid arteries.

Left common carotid artery


This vessel arises immediately to the left and slightly behind the origin of the brachiocephalic trunk.
Its thoracic portion is 2.5- 3.5 cm in length and runs superolaterally to the sternoclavicular joint.

In the thorax
The vessel is in contact, from below upwards, with the trachea, left recurrent laryngeal nerve, left
margin of the oesophagus. Anteriorly the left brachiocephalic vein runs across the artery, and the
cardiac branches from the left vagus descend in front of it. These structures together with the
thymus and the anterior margins of the left lung and pleura separate the artery from the manubrium.
In the neck
The artery runs superiorly deep to sternocleidomastoid and then enters the anterior triangle. At this
point it lies within the carotid sheath with the vagus nerve and the internal jugular vein. Posteriorly
the sympathetic trunk lies between the vessel and the prevertebral fascia. At the level of C7 the
vertebral artery and thoracic duct lie behind it. The anterior tubercle of C6 transverse process is
prominent and the artery can be compressed against this structure (it corresponds to the level of the
cricoid).
Anteriorly at C6 the omohyoid muscle passes superficial to the artery.
Within the carotid sheath the jugular vein lies lateral to the artery.

Right common carotid artery


The right common carotid arises from the brachiocephalic artery. The right common carotid artery
corresponds with the cervical portion of the left common carotid, except that there is no thoracic duct
on the right. The oesophagus is less closely related to the right carotid than the left.

Summary points about the carotid anatomy

Path
Passes behind the sternoclavicular joint (12% patients above this level) to the upper border of the
thyroid cartilage, to divide into the external (ECA) and internal carotid arteries (ICA).

Relations

 Level of 6th cervical vertebra crossed by omohyoid


 Then passes deep to the thyrohyoid, sternohyoid, sternomastoid muscles.
 Passes anterior to the carotid tubercle (transverse process 6th cervical vertebra)-NB
compression here stops haemorrhage.
 The inferior thyroid artery passes posterior to the common carotid artery.
 Then : Left common carotid artery crosses the thoracic duct, Right common carotid artery
crossed by recurrent laryngeal nerve
Image sourced from Wikipedia

Next question
Question 232 of 560

A 21 year old develops tonsillitis. He is in considerable pain. Which of the following nerves is
responsible for the sensory innervation of the tonsillar fossa?

Facial nerve

Trigeminal nerve

Glossopharyngeal nerve

Hypoglossal nerve

Vagus

The glossopharyngeal nerve is the main sensory nerve for the tonsillar fossa. A lesser contribution is
made by the lesser palatine nerve. Because of this otalgia may occur following tonsillectomy.
Please rate this question:

Discuss and give feedback


Next question

Tonsil

Anatomy

 Each palatine tonsil has two surfaces, a medial surface which projects into the pharynx and a
lateral surface that is embedded in the wall of the pharynx.
 They are usually 25mm tall by 15mm wide, although this varies according to age and may be
almost completely atrophied in the elderly.
 Their arterial supply is from the tonsillar artery, a branch of the facial artery.
 Its veins pierce the constrictor muscle to join the external palatine or facial veins. The
external palatine vein is immediately lateral to the tonsil, which may result in haemorrhage
during tonsillectomy.
 Lymphatic drainage is the jugulodigastric node and the deep cervical nodes.

Tonsillitis
 Usually bacterial (50%)- group A Streptococcus. Remainder viral.
 May be complicated by development of abscess (quinsy). This may distort the uvula.

- Indications for tonsillectomy include recurrent acute tonsillitis, suspected malignancy, enlargement
causing sleep apnoea.
- Dissection tonsillectomy is the preferred technique with haemorrhage being the commonest
complication. Delayed otalgia may occur owing to irritation of the glossopharyngeal nerve.
Next question
Question 233 of 560

A man has an incision sited that runs 8cm from the deltopectoral groove to the midline. Which of the
following is not at risk of injury?

Cephalic vein

Shoulder joint capsule

Axillary artery

Pectoralis major

Trunk of the brachial plexus

Theme from April 2012 Exam


This region will typically lie medial to the joint capsule. The diagram below illustrates the plane that
this would transect and as it can be appreciated the other structures are all at risk of injury.
Image sourced from Wikipedia

Please rate this question:

Discuss and give feedback

Next question

Pectoralis major muscle


Origin From the medial two thirds of the clavicle, manubrium and sternocostal angle

Insertion Lateral edge of the bicipital groove of the humerus

Nerve supply Lateral pectoral nerve

Actions Adductor and medial rotator of the humerus

Next question
Question 234 of 560

A surgeon is due to perform a laparotomy for perforated duodenal ulcer. An upper midline incision is
to be performed. Which of the following structures is the incision most likely to divide?

Rectus abdominis muscle

External oblique muscle

Linea alba

Internal oblique muscle

None of the above

Theme from September 2011 Exam


Upper midline abdominal incisions will involve the division of the linea alba. Division of muscles will
not usually improve access in this approach and they would not be routinely encountered during this
incision.
Please rate this question:

Discuss and give feedback


Next question

Abdominal incisions

Midline incision  Commonest approach to the abdomen


 Structures divided: linea alba, transversalis fascia, extraperitoneal fat,
peritoneum (avoid falciform ligament above the umbilicus)
 Bladder can be accessed via an extraperitoneal approach through the
space of Retzius

Paramedian  Parallel to the midline (about 3-4cm)


incision  Structures divided/retracted: anterior rectus sheath, rectus (retracted),
posterior rectus sheath, transversalis fascia, extraperitoneal fat,
peritoneum
 Incision is closed in layers

Battle  Similar location to paramedian but rectus displaced medially (and thus
denervated)
 Now seldom used

Kocher's Incision under right subcostal margin e.g. Cholecystectomy (open)

Lanz Incision in right iliac fossa e.g. Appendicectomy

Gridiron Oblique incision centered over McBurneys point- usually appendicectomy


(less cosmetically acceptable than Lanz

Gable Rooftop incision

Pfannenstiel's Transverse supra pubic, primarily used to access pelvic organs

McEvedy's Groin incision e.g. Emergency repair strangulated femoral hernia

Rutherford Extraperitoneal approach to left or right lower quadrants. Gives excellent


Morrison access to iliac vessels and is the approach of choice for first time renal
transplantation.

Image sourced from Wikipedia

Next question
Question 235 of 560

A 59 year old man is undergoing an extended right hemicolectomy for a carcinoma of the splenic
flexure of the colon. The surgeons divide the middle colic vein close to its origin. Into which of the
following structures does this vessel primarily drain?

Superior mesenteric vein

Portal vein

Inferior mesenteric vein

Inferior vena cava

Ileocolic vein

The middle colonic vein drains into the SMV, if avulsed during mobilisation then dramatic
haemorrhage can occur and be difficult to control.
Please rate this question:

Discuss and give feedback


Next question

Transverse colon

 The right colon undergoes a sharp turn at the level of the hepatic flexure to become the
transverse colon.
 At this point it also becomes intraperitoneal.
 It is connected to the inferior border of the pancreas by the transverse mesocolon.
 The greater omentum is attached to the superior aspect of the transverse colon from which it
can easily be separated. The mesentery contains the middle colic artery and vein. The
greater omentum remains attached to the transverse colon up to the splenic flexure. At this
point the colon undergoes another sharp turn.

Relations
Liver and gall-bladder, the greater curvature of the stomach, and the lower end of the
Superior
spleen

Inferior Small intestine


Anterior Greater omentum

From right to left with the descending portion of the duodenum, the head of the pancreas,
Posterior
convolutions of the jejunum and ileum, spleen
Next question
Question 236-238 of 560

Theme: Nerve Injury

A. Median nerve
B. Ulnar nerve
C. Radial nerve
D. Musculocutaneous nerve
E. Axillary nerve
F. Anterior interosseous nerve
G. Posterior interosseous nerve

For each scenario please select the most likely underlying nerve injury. Each option may be used
once, more than once or not at all.

236. A 10 year old boy is admitted to casualty following a fall. On examination there is
deformity and swelling of the upper arm. The ability to flex the fingers of the affected
limb is impaired. However, there is not sensory impairment. Imaging confirms a
displaced supra condylar fracture

You answered Median nerve

The correct answer is Anterior interosseous nerve

Supracondylar fractures may be complicated by neurovascular compromise. The anterior


interosseous nerve is most commonly affected. It has no sensory supply so the defect is
motor alone.

237. A well toned weight lifter attends clinic reporting weakness of his left arm. There is
weakness of flexion and supination of the forearm.

You answered Median nerve

The correct answer is Musculocutaneous nerve

Musculocutaneous nerve compression due to entrapment of the nerve between biceps and
brachialis. Elbow flexion and supination of the arm are affected. This is a rare isolated
injury.

238. An 18 year old girl sustains an Holstein-Lewis fracture. Which nerve is at risk?

You answered Median nerve


The correct answer is Radial nerve

Proximal lesions affect the triceps. Also paralysis of wrist extensors and forearm
supinators occur. Reduced sensation of dorsoradial aspect of hand and dorsal 31/2
fingers. Holstein-Lewis fractures are fractures of the distal humerus with radial nerve
entrapment.

Please rate this question:

Discuss and give feedback


Next question

Brachial plexus

Origin Anterior rami of C5 to T1

Sections of the  Roots, trunks, divisions, cords, branches


plexus  Mnemonic:Real Teenagers Drink Cold Beer

Roots  Located in the posterior triangle


 Pass between scalenus anterior and medius

Trunks  Located posterior to middle third of clavicle


 Upper and middle trunks related superiorly to the subclavian artery
 Lower trunk passes over 1st rib posterior to the subclavian artery

Divisions Apex of axilla

Cords Related to axillary artery

Diagram illustrating the branches of the brachial plexus


Image sourced from Wikipedia

Cutaneous sensation of the upper limb

Image sourced from Wikipedia

Next question
Question 239 of 560

A 23 year old man is stabbed in the chest approximately 10cm below the right nipple. In the
emergency department a abdominal ultrasound scan shows a large amount of intraperitoneal blood.
Which of the following statements relating to the likely site of injury is untrue?

Part of its posterior surface is devoid of peritoneum.

The quadrate lobe is contained within the functional right lobe.

Its nerve supply is from the coeliac plexus.

The hepatic flexure of the colon lies posterio-inferiorly.

The right kidney is closely related posteriorly.

The right lobe of the liver is the most likely site of injury. Therefore the answer is B as the quadrate
lobe is functionally part of the left lobe of the liver. The liver is largely covered in peritoneum.
Posteriorly there is an area devoid of peritoneum (the bare area of the liver). The right lobe of the
liver has the largest bare area (and is larger than the left lobe).
Please rate this question:

Discuss and give feedback


Next question

Liver

Structure of the liver


Right lobe  Supplied by right hepatic artery
 Contains Couinaud segments V to VIII (-/+Sg I)

Left lobe  Supplied by the left hepatic artery


 Contains Couinaud segments II to IV (+/- Sg1)

Quadrate lobe  Part of the right lobe anatomically, functionally is part of the left
 Couinaud segment IV
 Porta hepatis lies behind
 On the right lies the gallbladder fossa
 On the left lies the fossa for the umbilical vein

Caudate lobe  Supplied by both right and left hepatic arteries


 Couinaud segment I
 Lies behind the plane of the porta hepatis
 Anterior and lateral to the inferior vena cava
 Bile from the caudate lobe drains into both right and left hepatic ducts

Detailed knowledge of Couinaud segments is not required for MRCS

 Between the liver lobules are portal canals which contain the portal triad: Hepatic Artery,
Portal Vein, tributary of Bile Duct.

Relations of the liver


Anterior Postero inferiorly

Diaphragm Oesophagus

Xiphoid process Stomach

Duodenum

Hepatic flexure of colon

Right kidney

Gallbladder

Inferior vena cava

Porta hepatis
Location Postero inferior surface, it joins nearly at right angles with the left sagittal fossa, and
separates the caudate lobe behind from the quadrate lobe in front

Transmits  Common hepatic duct


 Hepatic artery
 Portal vein
 Sympathetic and parasympathetic nerve fibres
 Lymphatic drainage of the liver (and nodes)

Ligaments
Falciform ligament  2 layer fold peritoneum from the umbilicus to anterior liver surface
 Contains ligamentum teres (remnant umbilical vein)
 On superior liver surface it splits into the coronary and left
triangular ligaments

Ligamentum teres Joins the left branch of the portal vein in the porta hepatis

Ligamentum Remnant of ductus venosus


venosum

Arterial supply

 Hepatic artery

Venous

 Hepatic veins
 Portal vein

Nervous supply

 Sympathetic and parasympathetic trunks of coeliac plexus

Next question
Question 240 of 560

A 22 year old man is involved in a fight and sustains a skull fracture with an injury to the middle
meningeal artery. A craniotomy is performed, and with considerable difficulty the haemorrhage from
the middle meningeal artery is controlled by ligating it close to its origin. What is the most likely
sensory impairment that the patient may notice post operatively?

Parasthesia of the ipsilateral external ear

Loss of taste sensation from the anterior two thirds of the tongue

Parasthesia overlying the angle of the jaw

Loss of sensation from the ipsilateral side of the tongue

Loss of taste from the posterior two thirds of the tongue

The auriculotemporal nerve is closely related to the middle meningeal artery and may be damaged
in this scenario. The nerve supplied sensation to the external ear and outermost part of the tympanic
membrane. The angle of the jaw is innervated by C2,3 roots and would not be affected. The tongue
is supplied by the glossopharyngeal nerve.
Please rate this question:

Discuss and give feedback


Next question

Middle meningeal artery

 Middle meningeal artery is typically the third branch of the first part of the maxillary artery,
one of the two terminal branches of the external carotid artery. After branching off the
maxillary artery in the infratemporal fossa, it runs through the foramen spinosum to supply
the dura mater (the outermost meninges) .
 The middle meningeal artery is the largest of the three (paired) arteries which supply the
meninges, the others being the anterior meningeal artery and the posterior meningeal artery.
 The middle meningeal artery runs beneath the pterion. It is vulnerable to injury at this point,
where the skull is thin. Rupture of the artery may give rise to an extra dural hematoma.
 In the dry cranium, the middle meningeal, which runs within the dura mater surrounding the
brain, makes a deep indention in the calvarium.
 The middle meningeal artery is intimately associated with the auriculotemporal nerve which
wraps around the artery making the two easily identifiable in the dissection of human
cadavers and also easily damaged in surgery.
Question 241 of 560

A 72 year old man presents with haemoptysis and undergoes a bronchoscopy. The carina is noted
to be widened. At which level does the trachea bifurcate?

T3

T5

T7

T2

T8

The trachea bifurcates at the level of the fifth thoracic vertebra. Or the sixth in tall subjects.
Please rate this question:

Discuss and give feedback


Next question

Trachea

Trachea
Location C6 vertebra to the upper border of T5 vertebra (bifurcation)

Arterial and venous supply Inferior thyroid arteries and the thyroid venous plexus.

Nerve Branches of vagus, sympathetic and the recurrent nerves

Relations in the neck


Anterior(Superior to  Isthmus of the thyroid gland
inferior)  Inferior thyroid veins
 Arteria thyroidea ima (when that vessel exists)
 Sternothyroid
 Sternohyoid
 Cervical fascia
 Anastomosing branches between the anterior jugular
veins

Posterior Oesophagus.

Laterally  Common carotid arteries


 Right and left lobes of the thyroid gland
 Inferior thyroid arteries
 Recurrent laryngeal nerves

Relations in the thorax

Anterior

 Manubrium, the remains of the thymus, the aortic arch, left common carotid arteries, and the
deep cardiac plexus

Lateral

 In the superior mediastinum, on the right side is the pleura and right vagus; on its left side
are the left recurrent nerve, the aortic arch, and the left common carotid and subclavian
arteries.

Next question
Question 242 of 560

A 23 year old man is injured during a game of rugby. He suffers a fracture of the distal third of his
clavicle, it is a compound fracture and there is evidence of arterial haemorrhage. Which of the
following vessels is most likely to be encountered first during subsequent surgical exploration?

Posterior circumflex humeral artery

Axillary artery

Thoracoacromial artery

Sub scapular artery

Lateral thoracic artery

Similar theme in September 2011 Exam


The thoracoacromial artery arises from the second part of the axillary artery. It is a short, wide trunk,
which pierces the clavipectoral fascia, and ends, deep to pectoralis major by dividing into four
branches.

Please rate this question:

Discuss and give feedback

Next question

Thoracoacromial artery

The thoracoacromial artery (acromiothoracic artery; thoracic axis) is a short trunk, which arises from
the forepart of the axillary artery, its origin being generally overlapped by the upper edge of the
Pectoralis minor.

Projecting forward to the upper border of the Pectoralis minor, it pierces the coracoclavicular fascia
and divides into four branches: pectoral, acromial, clavicular, and deltoid.

Branch Description

Pectoral Descends between the two Pectoral muscles, and is distributed to them and to the breast,
branch anastomosing with the intercostal branches of the internal thoracic artery and with the
lateral thoracic.

Acromial Runs laterally over the coracoid process and under the Deltoid, to which it gives branches; it
branch then pierces that muscle and ends on the acromion in an arterial network formed by
branches from the suprascapular, thoracoacromial, and posterior humeral circumflex
arteries.

Clavicular Runs upwards and medially to the sternoclavicular joint, supplying this articulation, and the
branch Subclavius.

Deltoid Arising with the acromial, it crosses over the Pectoralis minor and passes in the same
branch groove as the cephalic vein, between the Pectoralis major and Deltoid, and gives branches
to both muscles.

Next question
Question 243 of 560

The following are true of the femoral nerve, except:

It is derived from L2, L3 and L4 nerve roots

It supplies sartorius

It supplies quadriceps femoris

It gives cutaneous innervations via the saphenous nerve

It supplies adductor longus

Adductor longus is supplied by the obturator nerve.


Please rate this question:

Discuss and give feedback


Next question

Femoral nerve

Root values L2, 3, 4

Innervates  Pectineus
 Sartorius
 Quadriceps femoris
 Vastus lateralis/medialis/intermedius

Branches  Medial cutaneous nerve of thigh


 Saphenous nerve
 Intermediate cutaneous nerve of thigh

Path
Penetrates psoas major and exits the pelvis by passing under the inguinal ligament to enter the
femoral triangle, lateral to the femoral artery and vein.

Image sourced from Wikipedia

Mnemonic for femoral nerve supply

(don't) M I S V Q Scan for PE


M edial cutaneous nerve of the thigh
I ntermediate cutaneous nerve of the thigh
S aphenous nerve

V astus
Q uadriceps femoris
S artorius

PE ectineus

Next question
Question 244 of 560

Where is the vomiting centre located?

Medulla oblongata

Substantia nigra

Antrum of stomach

Pons

Midbrain

ABC's of Non- GI causes of vomiting

Acute renal failure


Brain (Increased ICP)
Cardiac (Inferior MI)
DKA
Ears (labyrinthitis)
Foreign substances (Tylenol, theo, etc)
Glaucoma
Hyperemesis Gravidarum
Infections (pyelonephritis, meningitis)

Please rate this question:

Discuss and give feedback


Next question

Vomiting

Reflex oral expulsion of gastric (and sometimes intestinal) contents - reverse peristalsis and
abdominal contraction

The vomiting centre is in part of the medulla oblongata and is triggered by receptors in several
locations:
 Labyrinthine receptors of ear (motion sickness)
 Over distention receptors of duodenum and stomach
 Trigger zone of CNS - many drugs (e.g., opiates) act here
 Touch receptors in throat

Next question
Question 245 of 560

Which of the following nerves conveys sensory information from the laryngeal mucosa?

Glossopharyngeal

Laryngeal branches of the vagus

Ansa cervicalis

Laryngeal branches of the trigeminal

None of the above

The laryngeal branches of the vagus supply sensory information from the larynx.
Please rate this question:

Discuss and give feedback


Next question

Larynx

The larynx lies in the anterior part of the neck at the levels of C3 to C6 vertebral bodies. The
laryngeal skeleton consists of a number of cartilagenous segments. Three of these are paired;
arytenoid, corniculate and cuneiform. Three are single; thyroid, cricoid and epiglottic. The cricoid
cartilage forms a complete ring (the only one to do so).
The laryngeal cavity extends from the laryngeal inlet to the level of the inferior border of the cricoid
cartilage.

Divisions of the laryngeal cavity


Laryngeal vestibule Superior to the vestibular folds

Laryngeal ventricle Lies between vestibular folds and superior to the vocal cords

Infraglottic cavity Extends from vocal cords to inferior border of the cricoid cartilage
The vocal folds (true vocal cords) control sound production. The apex of each fold projects medially
into the laryngeal cavity. Each vocal fold includes:

 Vocal ligament
 Vocalis muscle (most medial part of thyroarytenoid muscle)

The glottis is composed of the vocal folds, processes and rima glottidis. The rima glottidis is the
narrowest potential site within the larynx, as the vocal cords may be completely opposed, forming a
complete barrier.

Muscles of the larynx


Muscle Origin Insertion Innervation Action

Posterior Posterior aspect Muscular process Recurrent Abducts vocal fold


cricoarytenoid of lamina of of arytenoid Laryngeal
cricoid

Lateral Arch of cricoid Muscular process Recurrent Adducts vocal fold


cricoarytenoid of arytenoid laryngeal

Thyroarytenoid Posterior aspect Muscular process Recurrent Relaxes vocal fold


of thyroid of arytenoid laryngeal
cartilage

Transverse and Arytenoid Contralateral Recurrent Closure of


oblique cartilage arytenoid laryngeal intercartilagenous
arytenoids part of the rima
glottidis

Vocalis Depression Vocal ligament Recurrent Relaxes posterior


between lamina and vocal process laryngeal vocal ligament, tenses
of thyroid of arytenoid anterior part
cartilage cartilage

Cricothyroid Anterolateral Inferior margin External Tenses vocal fold


part of cricoid and horn of laryngeal
thyroid cartilage
Blood supply
Arterial supply is via the laryngeal arteries, branches of the superior and inferior thyroid arteries. The
superior laryngeal artery is closely related to the internal laryngeal nerve. The inferior laryngeal
artery is related to the inferior laryngeal nerve. Venous drainage is via superior and inferior laryngeal
veins, the former draining into the superior thyroid vein and the latter draining into the middle thyroid
vein, or thyroid venous plexus.

Lymphatic drainage
The vocal cords have no lymphatic drainage and this site acts as a lymphatic watershed.
Supraglottic part Upper deep cervical nodes

Subglottic part Prelaryngeal and pretracheal nodes and inferior deep cervical nodes

The aryepiglottic fold and vestibular folds have a dense plexus of lymphatics associated with them
and malignancies at these sites have a greater propensity for nodal metastasis.

Topography of the larynx

Image sourced from Wikipedia

Next question
Question 246 of 560

Which of the following nerves passes through the greater sciatic foramen and innervates the
perineum?

Pudendal

Sciatic

Superior gluteal

Inferior gluteal

Posterior cutaneous nerve of the thigh

3 divisions of the pudendal nerve:

 Rectal nerve
 Perineal nerve
 Dorsal nerve of penis/ clitoris

All these pass through the greater sciatic foramen.

The pudendal nerve innervates the perineum. It passes between piriformis and coccygeus medial to
the sciatic nerve.
Please rate this question:

Discuss and give feedback


Next question

Gluteal region

Gluteal muscles

 Gluteus maximus: inserts to gluteal tuberosity of the femur and iliotibial tract
 Gluteus medius: attach to lateral greater trochanter
 Gluteus minimis: attach to anterior greater trochanter
 All extend and abduct the hip
Deep lateral hip rotators

 Piriformis
 Gemelli
 Obturator internus
 Quadratus femoris

Nerves
Superior gluteal nerve (L5, S1)  Gluteus medius
 Gluteus minimis
 Tensor fascia lata

Inferior gluteal nerve Gluteus maximus


Damage to the superior gluteal nerve will result in the patient developing a Trendelenberg gait.
Affected patients are unable to abduct the thigh at the hip joint. During the stance phase, the
weakened abductor muscles allow the pelvis to tilt down on the opposite side. To compensate, the
trunk lurches to the weakened side to attempt to maintain a level pelvis throughout the gait cycle.
The pelvis sags on the opposite side of the lesioned superior gluteal nerve.
Next question
Question 247 of 560

Which of the following is true in relation to the sartorius muscle?

Innervated by the deep branch of the femoral nerve

Inserts at the fibula

It is the shortest muscle in the body

Forms the Pes anserinus with Gracilis and semitendinous muscle

Causes extension of the knee

It is innervated by the superficial branch of the femoral nerve. It is a component of the pes anserinus.
Please rate this question:

Discuss and give feedback


Next question

Sartorius

 Longest strap muscle in the body


 Most superficial muscle in the anterior compartment of the thigh

Origin Anterior superior iliac spine

Insertion Medial surface of the of the body of the tibia (upper part). It inserts anterior to
gracilis and semitendinosus

Nerve Supply Femoral nerve (L2,3)

Action  Flexor of the hip and knee, slight abducts the thigh and rotates it laterally
 It assists with medial rotation of the tibia on the femur. For example it
would play a pivotal role in placing the right heel onto the left knee ( and
vice versa)

Important The middle third of this muscle, and its strong underlying fascia forms the roof of
relations the adductor canal , in which lie the femoral vessels, the saphenous nerve and the
nerve to vastus medialis.
Next question
Question 248-250 of 560

Theme: Nerve lesions

A. Sciatic nerve
B. Peroneal nerve
C. Tibial Nerve
D. Obturator nerve
E. Ilioinguinal nerve
F. Femoral nerve
G. None of the above

Please select the most likely nerve injury for the scenario given. Each option may be used once,
more than once or not at all

248. A 56 year old man undergoes a low anterior resection with legs in the Lloyd-Davies
position. Post operatively he complains of foot drop.

You answered Sciatic nerve

The correct answer is Peroneal nerve

Positioning legs in Lloyd- Davies stirrups can carry the risk of peroneal nerve
neuropraxia if not done carefully.

249. A 23 year old man complains of severe groin pain several weeks after a difficult inguinal
hernia repair.

You answered Sciatic nerve

The correct answer is Ilioinguinal nerve

The ilioinguinal nerve may have been entrapped in the mesh causing a neuroma.

250. A 72 year old man develops a foot drop after a revision total hip replacement.

Sciatic nerve

This may be done by a number of approaches, in this scenario a posterior approach is the
most likely culprit.

Please rate this question:


Discuss and give feedback
Next question

Lower limb- Muscular compartments

Anterior compartment
Muscle Nerve Action

Tibialis anterior Deep peroneal Dorsiflexes ankle joint, inverts foot


nerve

Extensor digitorum Deep peroneal Extends lateral four toes, dorsiflexes ankle
longus nerve joint

Peroneus tertius Deep peroneal Dorsiflexes ankle, everts foot


nerve

Extensor hallucis longus Deep peroneal Dorsiflexes ankle joint, extends big toe
nerve

Peroneal compartment
Muscle Nerve Action

Peroneus longus Superficial peroneal nerve Everts foot, assists in plantar flexion

Peroneus brevis Superficial peroneal nerve Plantar flexes the ankle joint

Superficial posterior compartment


<muscle< b="" style="box-sizing: border-
box;"></muscle<> Nerve Action

Gastrocnemius Tibial Plantar flexes the foot, may also


nerve flex the knee
<muscle< b="" style="box-sizing: border-
box;"></muscle<> Nerve Action

Soleus Tibial Plantar flexor


nerve

Deep posterior compartment


Muscle Nerve Action

Flexor digitorum longus Tibial Flexes the lateral four toes

Flexor hallucis longus Tibial Flexes the great toe

Tibialis posterior Tibial Plantar flexor, inverts the foot

Next question
Question 251 of 560

A 38 year old man falls onto an outstretched hand. Following the accident he is examined in the
emergency department. On palpating his anatomical snuffbox there is tenderness noted in the base.
What is the most likely injury in this scenario?

Rupture of the tendon of flexor pollicis

Scaphoid fracture

Distal radius fracture

Rupture of flexor carpi ulnaris tendon

None of the above

A fall onto an outstretched hand is a common mechanism of injury for a scaphoid fracture. This
should be suspected clinically if there is tenderness in the base of the anatomical snuffbox. A tendon
rupture would not result in bony tenderness.
Please rate this question:

Discuss and give feedback


Next question

Scaphoid bone

The scaphoid has a concave articular surface for the head of the capitate and at the edge of this is a
crescentic surface for the corresponding area on the lunate.
Proximally, it has a wide convex articular surface with the radius. It has a distally sited tubercle that
can be palpated. The remaining articular surface is to the lateral side of the tubercle. It faces laterally
and is associated with the trapezium and trapezoid bones.

The narrow strip between the radial and trapezial surfaces and the tubercle gives rise to the radial
collateral carpal ligament. The tubercle receives part of the flexor retinaculum. This area is the only
part of the scaphoid that is available for the entry of blood vessels. It is commonly fractured and
avascular necrosis may result.

Scaphoid bone
Image sourced from Wikipedia

Next question
Question 252 of 560

A 25 year old man sustains a severe middle cranial fossa basal skull fracture. Once he has
recovered it is noticed that he has impaired tear secretion. This is most likely to be the result of
damage to which of the following?

Stellate ganglion

Ciliary ganglion

Otic ganglion

Trigeminal nerve

Greater petrosal nerve

The greater petrosal nerve may be injured and carries fibres for lacrimation (see below).
Please rate this question:

Discuss and give feedback


Next question

Lacrimal system

Lacrimal gland
Consists of an orbital part and palpebral part. They are continuous posterolaterally around the
concave lateral edge of the levator palpebrae superioris muscle.
The ducts of the lacrimal gland open into the superior fornix. Those from the orbital part penetrate
the aponeurosis of levator palpebrae superioris to join those from the palpebral part. Therefore
excision of the palpebral part is functionally similar to excision of the entire gland.

Blood supply
Lacrimal branch of the opthalmic artery. Venous drainage is to the superior opthalmic vein.

Innervation
The gland is innervated by the secretomotor parasympathetic fibres from the pterygopalatine
ganglion which in turn may reach the gland via the zygomatic or lacrimal branches of the maxillary
nerve or pass directly to the gland. The preganglionic fibres travel to the ganglion in the greater
petrosal nerve (a branch of the facial nerve at the geniculate ganglion).

Nasolacrimal duct
Descends from the lacrimal sac to open anteriorly in the inferior meatus of the nose.

Lacrimation reflex
Occurs in response to conjunctival irritation (or emotional events). The conjunctiva will send signals
via the opthalmic nerve. These then pass to the superior salivary centre. The efferent signals pass
via the greater petrosal nerve (parasympathetic preganglionic fibres) and the deep petrosal nerve
which carries the post ganglionic sympathetic fibres. The parasympathetic fibres will relay in the
pterygopalatine ganglion, the sympathetic fibres do not synapse. They in turn will relay to the
lacrimal apparatus.
Next question
Question 253 of 560

Which of the following structures passes through the quadrangular space near the humeral head?

Axillary artery

Radial nerve

Axillary nerve

Median nerve

Transverse scapular artery

The quadrangular space is bordered by the humerus laterally, subscapularis superiorly, teres major
inferiorly and the long head of triceps medially. It lies lateral to the triangular space. It transmits the
axillary nerve and posterior circumflex humeral artery.
Image sourced from Wikipedia

Please rate this question:

Discuss and give feedback

Next question

Shoulder joint

 Shallow synovial ball and socket type of joint.


 It is an inherently unstable joint, but is capable to a wide range of movement.
 Stability is provided by muscles of the rotator cuff that pass from the scapula to insert in the
greater tuberosity (all except sub scapularis-lesser tuberosity).

Glenoid labrum

 Fibrocartilaginous rim attached to the free edge of the glenoid cavity


 Tendon of the long head of biceps arises from within the joint from the supraglenoid tubercle,
and is fused at this point to the labrum.
 The long head of triceps attaches to the infraglenoid tubercle

Fibrous capsule

 Attaches to the scapula external to the glenoid labrum and to the labrum itself (postero-
superiorly)
 Attaches to the humerus at the level of the anatomical neck superiorly and the surgical neck
inferiorly
 Anteriorly the capsule is in contact with the tendon of subscapularis, superiorly with the
supraspinatus tendon, and posteriorly with the tendons of infraspinatus and teres minor. All
these blend with the capsule towards their insertion.
 Two defects in the fibrous capsule; superiorly for the tendon of biceps. Anteriorly there is a
defect beneath the subscapularis tendon.
 The inferior extension of the capsule is closely related to the axillary nerve at the surgical
neck and this nerve is at risk in anteroinferior dislocations. It also means that proximally sited
osteomyelitis may progress to septic arthritis.

Movements and muscles


Flexion Anterior part of deltoid
Pectoralis major
Biceps
Coracobrachialis

Extension Posterior deltoid


Teres major
Latissimus dorsi

Adduction Pectoralis major


Latissimus dorsi
Teres major
Coracobrachialis

Abduction Mid deltoid


Supraspinatus

Medial rotation Subscapularis


Anterior deltoid
Teres major
Latissimus dorsi

Lateral rotation Posterior deltoid


Infraspinatus
Teres minor

Important anatomical relations

Anteriorly Brachial plexus


Axillary artery and vein

Posterior Suprascapular nerve


Suprascapular vessels
Inferior Axillary nerve
Circumflex humeral vessels

Next question
Question 254 of 560

Which of the following pairings of foramina and their contents is not correct?

Superior orbital fissure and the oculomotor nerve

Foramina rotundum and the maxillary nerve

Jugular foramen and the hypoglossal nerve

Foramina spinosum and the middle meningeal artery

Carotid canal and the internal carotid artery

The hypoglossal nerve passes through the hypoglossal canal.

Please rate this question:

Discuss and give feedback

Next question

Foramina of the base of the skull

Foramen Location Contents

Foramen ovale Sphenoid Otic ganglion


bone V3 (Mandibular nerve:3rd branch of
trigeminal)
Accessory meningeal artery
Lesser petrosal nerve
Foramen Location Contents

Emissary veins

Foramen spinosum Sphenoid Middle meningeal artery


bone Meningeal branch of the Mandibular nerve

Foramen rotundum Sphenoid Maxillary nerve (V2)


bone

Foramen lacerum/ Sphenoid Base of the medial pterygoid plate.


carotid canal bone Internal carotid artery*
Nerve and artery of the pterygoid canal

Jugular foramen Temporal Anterior: inferior petrosal sinus


bone Intermediate: glossopharyngeal, vagus, and accessory nerves.
Posterior: sigmoid sinus (becoming the internal jugular vein) and
some meningeal branches from the occipital and ascending
pharyngeal arteries.

Foramen magnum Occipital Anterior and posterior spinal arteries


bone Vertebral arteries
Medulla oblongata

Stylomastoid Temporal Stylomastoid artery


foramen bone Facial nerve

Superior orbital Sphenoid Oculomotor nerve (III)


fissure bone Recurrent meningeal artery
Trochlear nerve (IV)
Lacrimal, frontal and nasociliary branches of ophthalmic nerve (V1)
Abducent nerve (VI)
Superior ophthalmic vein
*= In life the foramen lacerum is occluded by a cartilagenous plug. The ICA initially passes into the
carotid canal which ascends superomedially to enter the cranial cavity through the foramen lacerum.

Base of skull anatomical overview

Image sourced from Wikipedia

Next question
Question 255 of 560

A 55 year old man with carcinoma of the larynx is undergoing a difficult laryngectomy. The surgeons
divide the thyrocervical trunk, from which of the following vessels does this structure most commonly
originate?

Subclavian artery

Common carotid artery

Vertebral artery

External carotid artery

Internal carotid artery

The thyrocervical trunk is a branch of the subclavian artery. It arises from the first part between the
subclavian artery and the inner border of scalenus anterior. It branches off the subclavian distal to
the vertebral artery.
Please rate this question:

Discuss and give feedback


Next question

Root of the neck

Thoracic Outlet

 Where the subclavian artery and vein and the brachial plexus exit the thorax and enter the
arm.
 They pass over the 1st rib and under the clavicle.
 The subclavian vein is the most anterior structure and is immediately anterior to scalenus
anterior and its attachment to the first rib.
 Scalenus anterior has 2 parts, the subclavian artery leaves the thorax by passing over the
first rib and between these 2 portions of the muscle.
 At the level of the first rib, the lower cervical nerve roots combine to form the 3 trunks of the
brachial plexus. The lowest trunk is formed by the union of C8 and T1, and this trunk lies
directly posterior to the artery and is in contact with the superior surface of the first rib.
Thoracic outlet obstruction causes neurovascular compromise.
Next question
Question 256 of 560

The following structures are closely related to the brachiocephalic artery except:

Trachea posteriorly

Right brachiocephalic vein

Inferior thyroid vein

Right recurrent laryngeal nerve

None of the above

There is no brachiocephalic artery on the left, however the left brachiocephalic vein lies anteriorly to
the roots of all the 3 great arteries (including the brachiocephalic artery). The right recurrent
laryngeal nerve has no relation to the brachiocephalic artery.
Please rate this question:

Discuss and give feedback


Next question

Brachiocephalic artery

The brachiocephalic artery is the largest branch of the aortic arch. From its aortic origin it ascends
superiorly, it initially lies anterior to the trachea and then on its right hand side. It branches into the
common carotid and right subclavian arteries at the level of the sternoclavicular joint.

Path
Origin- apex of the midline of the aortic arch
Passes superiorly and posteriorly to the right
Divides into the right subclavian and right common carotid artery

Relations
Anterior  Sternohyoid
 Sternothyroid
 Thymic remnants
 Left brachiocephalic vein
 Right inferior thyroid veins

Posterior  Trachea
 Right pleura

Right lateral  Right brachiocephalic vein


 Superior part of SVC

Left lateral  Thymic remnants


 Origin of left common carotid
 Inferior thyroid veins
 Trachea (higher level)

Branches
Normally none but may have the thyroidea ima artery

Image sourced from Wikipedia

Next question
Question 257 of 560

Which of the following structures separates the ulnar artery from the median nerve?

Brachioradialis

Pronator teres

Tendon of biceps brachii

Flexor carpi ulnaris

Brachialis

It lies deep to pronator teres and this separates it from the median nerve.
Please rate this question:

Discuss and give feedback


Next question

Ulnar artery

Path

 Starts: middle of antecubital fossa


 Passes obliquely downward, reaching the ulnar side of the forearm at a point about midway
between the elbow and the wrist. It follows the ulnar border to the wrist, crossing over the
flexor retinaculum. It then divides into the superficial and deep volar arches.

Relations
Deep to- Pronator teres, Flexor carpi radialis, Palmaris longus
Lies on- Brachialis and Flexor digitorum profundus
Superficial to the flexor retinaculum at the wrist

The median nerve is in relation with the medial side of the artery for about 2.5 cm. And then crosses
the vessel, being separated from it by the ulnar head of the Pronator teres

The ulnar nerve lies medially to the lower two-thirds of the artery
Branch

 Anterior interosseous artery

Image sourced from Wikipedia

Next question
Question 258 of 560

Which muscle is supplied by the superficial peroneal nerve?

Peroneus tertius

Sartorius

Adductor magnus

Peroneus brevis

Gracilis

Please rate this question:

Discuss and give feedback


Next question

Superficial peroneal nerve

Supplies

 Lateral compartment of leg: peroneus longus, peroneus brevis (action: eversion and plantar
flexion)
 Sensation over dorsum of the foot (except the first web space, which is innervated by the
deep peroneal nerve)

Path

 Passes between peroneus longus and peroneus brevis along the length of the proximal one
third of the fibula
 10-12 cm above the tip of the lateral malleolus, the superficial peroneal nerve pierces the
fascia
 6-7 cm distal to the fibula, the superficial peroneal nerve bifurcates into intermediate and
medial dorsal cutaneous nerves

Next question
Question 259 of 560

A 32 year old motorcyclist is involved in a road traffic accident. His humerus is fractured and
severely displaced. At the time of surgical repair the surgeon notes that the radial nerve has been
injured. Which of the following muscles is least likely to be affected by an injury at this site?

Extensor carpi radialis brevis

Brachioradialis

Abductor pollicis longus

Extensor pollicis brevis

None of the above

Muscles supplied by the radial nerve

BEST
Brachioradialis
Extensors
Supinator
Triceps

The radial nerve supplies the extensor muscles, abductor pollicis longus and extensor pollicis brevis
(the latter two being innervated by the posterior interosseous branch of the radial nerve).
Please rate this question:

Discuss and give feedback


Next question

Radial nerve

Continuation of posterior cord of the brachial plexus (root values C5 to T1)

Path

 In the axilla: lies posterior to the axillary artery on subscapularis, latissimus dorsi and teres
major.
 Enters the arm between the brachial artery and the long head of triceps (medial to humerus).
 Spirals around the posterior surface of the humerus in the groove for the radial nerve.
 At the distal third of the lateral border of the humerus it then pierces the intermuscular
septum and descends in front of the lateral epicondyle.
 At the lateral epicondyle it lies deeply between brachialis and brachioradialis where it then
divides into a superficial and deep terminal branch.
 Deep branch crosses the supinator to become the posterior interosseous nerve.

In the image below the relationships of the radial nerve can be appreciated

Image sourced from Wikipedia

Regions innervated
 Triceps
 Anconeus
Motor (main nerve)  Brachioradialis
 Extensor carpi radialis

 Supinator
 Extensor carpi ulnaris
 Extensor digitorum
Motor (posterior  Extensor indicis
interosseous branch)  Extensor digiti minimi
 Extensor pollicis longus and brevis
 Abductor pollicis longus

Sensory The area of skin supplying the proximal phalanges on the dorsal aspect of the
hand is supplied by the radial nerve (this does not apply to the little finger and
part of the ring finger)

Muscular innervation and effect of denervation


Anatomical
location Muscle affected Effect of paralysis

Shoulder Long head of triceps Minor effects on shoulder stability in abduction

Arm Triceps Loss of elbow extension

Forearm Supinator Weakening of supination of prone hand and


Brachioradialis elbow flexion in mid prone position
Extensor carpi radialis
longus and brevis

The cutaneous sensation of the upper limb- illustrating the contribution of the radial nerve

Image sourced from Wikipedia


Question 260 of 560

A man develops an infection in his external auditory meatus. The infection is extremely painful.
Which of the following nerves conveys sensation from this region?

Occipital branch of the trigeminal nerve

Vestibulocochlear nerve

Facial nerve

Auriculotemporal nerve

Maxillary branch of the trigeminal nerve

Tensor tympania and stapedius are the only two muscles of the middle ear. Contraction of tensor
tympani will tend to dampen the vibrations produced by loud sounds, it is innervated by a branch of
the trigeminal nerve. The stapedius dampens movements of the ossicles in response to loud sounds
and is innervated by a branch of the facial nerve.

The auriculotemporal nerve, which is derived from the mandibular branch of the trigeminal nerve
supplies this area.
Please rate this question:

Discuss and give feedback


Next question

Ear- anatomy

The ear is composed of three anatomically distinct regions.

External ear
Auricle is composed of elastic cartilage covered by skin. The lobule has no cartilage and contains fat
and fibrous tissue.

External auditory meatus is approximately 2.5cm long.


Lateral third of the external auditory meatus is cartilaginous and the medial two thirds is bony.

The region is innervated by the greater auricular nerve. The auriculotemporal branch of the
trigeminal nerve supplies most the of external auditory meatus and the lateral surface of the auricle.
Middle ear
Space between the tympanic membrane and cochlea. The aditus leads to the mastoid air cells is the
route through which middle ear infections may cause mastoiditis. Anteriorly the eustacian tube
connects the middle ear to the naso pharynx.
The tympanic membrane consists of:

 Outer layer of stratified squamous epithelium.


 Middle layer of fibrous tissue.
 Inner layer of mucous membrane continuous with the middle ear.

The tympanic membrane is approximately 1cm in diameter.


The chorda tympani nerve passes on the medial side of the pars flaccida.

The middle ear is innervated by the glossopharyngeal nerve and pain may radiate to the middle ear
following tonsillectomy.

Ossicles
Malleus attaches to the tympanic membrane (the Umbo).
Malleus articulates with the incus (synovial joint).
Incus attaches to stapes (another synovial joint).

Internal ear
Cochlea, semi circular canals and vestibule

Organ of corti is the sense organ of hearing and is located on the inside of the cochlear duct on the
basilar membrane.

Vestibule accommodates the utricule and the saccule. These structures contain endolymph and are
surrounded by perilymph within the vestibule.

The semicircular canals lie at various angles to the petrous temporal bone. All share a common
opening into the vestibule.
Next question
Question 261 of 560

Which muscle is responsible for causing flexion of the interphalangeal joint of the thumb?

Flexor pollicis longus

Flexor pollicis brevis

Flexor digitorum superficialis

Flexor digitorum profundus

Adductor pollicis

There are 8 muscles:


1. Two flexors (flexor pollicis brevis and flexor pollicis longus)
2. Two extensors (extensor pollicis brevis and longus)
3. Two abductors (abductor pollicis brevis and longus)
4. One adductor (adductor pollicis)
5. One muscle that opposes the thumb by rotating the CMC joint (opponens pollicis).

Flexor and extensor longus insert on the distal phalanx moving both the MCP and IP joints.
Please rate this question:

Discuss and give feedback


Next question

Hand

Anatomy of the hand


Bones  8 Carpal bones
 5 Metacarpals
 14 phalanges

Intrinsic Muscles 7 Interossei - Supplied by ulnar nerve

 3 palmar-adduct fingers
 4 dorsal- abduct fingers

Intrinsic muscles Lumbricals

 Flex MCPJ and extend the IPJ.


 Origin deep flexor tendon and insertion dorsal extensor hood
mechanism.
 Innervation: 1st and 2nd- median nerve, 3rd and 4th- deep branch of
the ulnar nerve.

Thenar eminence  Abductor pollicis brevis


 Opponens pollicis
 Flexor pollicis brevis

Hypothenar  Opponens digiti minimi


eminence  Flexor digiti minimi brevis
 Abductor digiti minimi
Image sourced from Wikipedia

Fascia and compartments of the palm


The fascia of the palm is continuous with the antebrachial fascia and the fascia of the dorsum of the
hand. The palmar fascia is thin over the thenar and hypothenar eminences. In contrast the palmar
fascia is relatively thick. The palmar aponeurosis covers the soft tissues and overlies the flexor
tendons. The apex of the palmar aponeurosis is continuous with the flexor retinaculum and the
palmaris longus tendon. Distally, it forms four longitudinal digital bands that attach to the bases of
the proximal phalanges, blending with the fibrous digital sheaths.
A medial fibrous septum extends deeply from the medial border of the palmar aponeurosis to the 5th
metacarpal. Lying medial to this are the hypothenar muscles. In a similar fashion, a lateral fibrous
septum extends deeply from the lateral border of the palmar aponeurosis to the 3rd metacarpal. The
thenar compartment lies lateral to this area.
Lying between the thenar and hypothenar compartments is the central compartment. It contains the
flexor tendons and their sheaths, the lumbricals, the superficial palmar arterial arch and the digital
vessels and nerves.
The deepest muscular plane is the adductor compartment, which contains adductor pollicis.

Short muscles of the hand


These comprise the lumbricals and interossei. The four slender lumbrical muscles flex the fingers at
the metacarpophalangeal joints and extend the interphalangeal joint. The four dorsal interossei are
located between the metacarpals and the four palmar interossei lie on the palmar surface of the
metacarpals in the interosseous compartment of the hand.

Long flexor tendons and sheaths in the hand


The tendons of FDS and FDP enter the common flexor sheath deep to the flexor retinaculum. The
tendons enter the central compartment of the hand and fan out to their respective digital synovial
sheaths. Near the base of the proximal phalanx, the tendon of FDS splits to permit the passage of
FDP. The FDP tendons are attached to the margins of the anterior aspect of the base of the distal
phalanx.
The fibrous digital sheaths contain the flexor tendons and their synovial sheaths. These extend from
the heads of the metacarpals to the base of the distal phalanges.
Next question
Question 262 of 560

Which of the following structures separates the posterior cruciate ligament from the popliteal artery?

Oblique popliteal ligament

Transverse ligament

Popliteus tendon

Biceps femoris

Semitendinosus

The posterior cruciate ligament is separated from the popliteal vessels at its origin by the oblique
popliteal ligament. The transverse ligament is located anteriorly.
Please rate this question:

Discuss and give feedback


Next question

Knee joint

The knee joint is a synovial joint, the largest and most complicated. It consists of two condylar joints
between the femur and tibia and a sellar joint between the patella and the femur. The tibiofemoral
articular surfaces are incongruent, however, this is improved by the presence of the menisci. The
degree of congruence is related to the anatomical position of the knee joint and is greatest in full
extension.

Knee joint compartments


 Comprised of the patella/femur joint, lateral and medial compartments
(between femur condyles and tibia)
Tibiofemoral  Synovial membrane and cruciate ligaments partially separate the medial
and lateral compartments

 Ligamentum patellae
Patellofemoral  Actions: provides joint stability in full extension
Fibrous capsule
The capsule of the knee joint is a complex, composite structure with contributions from adjacent
tendons.
Anterior The capsule does not pass proximal to the patella. It blends with the tendinous
fibres expansions of vastus medialis and lateralis

Posterior These fibres are vertical and run from the posterior surface of the femoral condyles
fibres to the posterior aspect of the tibial condyle

Attach to the femoral and tibial condyles beyond their articular margins, blending
Medial fibres
with the tibial collateral ligament

Lateral Attach to the femur superior to popliteus, pass over its tendon to head of fibula and
fibres tibial condyle

Bursae
 Subcutaneous prepatellar bursa; between patella and skin
 Deep infrapatellar bursa; between tibia and patellar ligament
Anterior
 Subcutaneous infrapatellar bursa; between distal tibial tuberosity and skin

 Bursa between lateral head of gastrocnemius and joint capsule


 Bursa between fibular collateral ligament and tendon of biceps femoris
Laterally
 Bursa between fibular collateral ligament and tendon of popliteus

 Bursa between medial head of gastrocnemius and the fibrous capsule


 Bursa between tibial collateral ligament and tendons of sartorius, gracilis and
semitendinosus
Medially
 Bursa between the tendon of semimembranosus and medial tibial condyle and
medial head of gastrocnemius

Posterior Highly variable and inconsistent

Ligaments
Medial collateral Medial epicondyle femur to medial tibial condyle: valgus stability
ligament

Lateral collateral Lateral epicondyle femur to fibula head: varus stability


ligament

Anterior cruciate Anterior tibia to lateral intercondylar notch femur: prevents tibia sliding
ligament anteriorly

Posterior cruciate Posterior tibia to medial intercondylar notch femur: prevents tibia
ligament sliding posteriorly

Patellar ligament Central band of the tendon of quadriceps femoris, extends from patella
to tibial tuberosity

Image sourced from Wikipedia


Image sourced from Wikipedia

© Image provided by the University of Sheffield

Menisci
Medial and lateral menisci compensate for the incongruence of the femoral and tibial condyles.
Composed of fibrous tissue.
Medial meniscus is attached to the tibial collateral ligament.
Lateral meniscus is attached to the loose fibres at the lateral edge of the joint and is separate from
the fibular collateral ligament. The lateral meniscus is crossed by the popliteus tendon.
Nerve supply
The knee joint is supplied by the femoral, tibial and common peroneal divisions of the sciatic and by
a branch from the obturator nerve. Hip pathology pain may be referred to the knee.

Blood supply
Genicular branches of the femoral artery, popliteal and anterior tibial arteries all supply the knee
joint.
Next question
Question 263 of 560

How many compartments are there in the lower leg?

The posterior compartment of the lower leg has both superficial and deep posterior layers, together
with the anterior and lateral compartments this allows for four compartments. Decompression of the
deep posterior compartment during fasciotomy may be overlooked with significant sequelae.
Please rate this question:

Discuss and give feedback


Next question

Fascial compartments of the leg

Compartments of the thigh

Formed by septae passing from the femur to the fascia lata.


Compartment Nerve Muscles Blood supply

Anterior compartment Femoral  Iliacus Femoral artery


 Tensor fasciae latae
 Sartorius
 Quadriceps femoris

Medial compartment Obturator  Adductor Profunda femoris artery


longus/magnus/brevis and obturator artery
 Gracilis
Compartment Nerve Muscles Blood supply

 Obturator externus

Posterior Sciatic  Semimembranosus Branches of Profunda


compartment (2  Semitendinosus femoris artery
layers)  Biceps femoris

Compartments of the lower leg


Separated by the interosseous membrane (anterior and posterior compartments), anterior fascial
septum (separate anterior and lateral compartments) and posterior fascial septum (separate lateral
and posterior compartments)

Blood
Compartment Nerve Muscles supply

Anterior Deep  Tibialis anterior Anterior


compartment peroneal  Extensor digitorum longus tibial artery
nerve  Extensor hallucis longus
 Peroneus tertius

Posterior Tibial  Muscles: deep and superficial Posterior


compartment compartments (separated by deep tibial
transverse fascia)
 Deep: Flexor hallucis longus, Flexor
digitalis longus, Tibialis posterior,
Popliteus
 Superficial: Gastrocnemius, Soleus,
Plantaris

Lateral Superficial  Peroneus longus/brevis Peroneal


compartment peroneal artery
Next question
Question 264 of 560

Which structure is least likely to be found at the level of the sternal angle?

Left brachiocephalic vein

Intervertebral discs T4-T5

Start of aortic arch

2nd pair of costal cartilages

Bifurcation of the trachea into left and right bronchi

The left brachiocephalic vein lies posterior to the manubrium, at the level of its upper border. The
sternal angle refers to the transition between manubrium and sternum and therefore will not include
the left brachiocephalic vein.
Please rate this question:

Discuss and give feedback


Next question

Sternal angle

Anatomical structures at the level of the manubrium and upper sternum


Upper part of the manubrium  Left brachiocephalic vein
 Brachiocephalic artery
 Left common carotid
 Left subclavian artery

Lower part of the manubrium/  Costal cartilages of the 2nd ribs


manubrio-sternal angle  Transition point between superior and inferior
mediastinum
 Arch of the aorta
 Tracheal bifurcation
 Union of the azygos vein and superior vena
cava
 The thoracic duct crosses to the midline

Next question
Question 265 of 560

A 53 year old man is undergoing a left hemicolectomy for carcinoma of the descending colon. From
which embryological structure is this region of the gastrointestinal tract derived?

Vitellino-intestinal duct

Hind gut

Mid gut

Fore gut

Woolffian duct

The left colon is embryologically part of the hind gut. Which accounts for its separate blood supply
via the IMA.

Please rate this question:

Discuss and give feedback

Next question

Colon anatomy

The colon commences with the caecum. This represents the most dilated segment of the human
colon and its base (which is intraperitoneal) is marked by the convergence of teniae coli. At this point
is located the vermiform appendix. The colon continues as the ascending colon, the posterior aspect
of which is retroperitoneal. The line of demarcation between the intra and retro peritoneal right colon
is visible as a white line, in the living, and forms the line of incision for colonic resections.

The ascending colon becomes the transverse colon after passing the hepatic flexure. At this located
the colon becomes wholly intra peritoneal once again. The superior aspect of the transverse colon is
the point of attachment of the transverse colon to the greater omentum. This is an important
anatomical site since division of these attachments permits entry into the lesser sac. Separation of
the greater omentum from the transverse colon is a routine operative step in both gastric and colonic
resections.

At the left side of the abdomen the transverse colon passes to the left upper quadrant and makes an
oblique inferior turn at the splenic flexure. Following this, the posterior aspect becomes
retroperitoneal once again.

At the level of approximately L4 the descending colon becomes wholly intraperitoneal and becomes
the sigmoid colon. Whilst the sigmoid is wholly intraperitoneal there are usually attachments laterally
between the sigmoid and the lateral pelvic sidewall. These small congenital adhesions are not formal
anatomical attachments but frequently require division during surgical resections.

At its distal end the sigmoid passes to the midline and at the region around the sacral promontary it
becomes the upper rectum. This transition is visible macroscopically as the point where the teniae
fuse. More distally the rectum passes through the peritoneum at the region of the peritoneal
reflection and becomes extraperitoneal.

Arterial supply
Superior mesenteric artery and inferior mesenteric artery: linked by the marginal artery.
Ascending colon: ileocolic and right colic arteries
Transverse colon: middle colic artery
Descending and sigmoid colon: inferior mesenteric artery

Venous drainage
From regional veins (that accompany arteries) to superior and inferior mesenteric vein

Lymphatic drainage
Initially along nodal chains that accompany supplying arteries, then para-aortic nodes.

Embryology
Midgut- Second part of duodenum to 2/3 transverse colon
Hindgut- Distal 1/3 transverse colon to anus

Peritoneal location
The right and left colon are part intraperitoneal and part extraperitoneal. The sigmoid and transverse
colon are generally wholly intraperitoneal. This has implications for the sequelae of perforations,
which will tend to result in generalised peritonitis in the wholly intra peritoneal segments.

Colonic relations

Region of colon Relation


Region of colon Relation

Caecum/ right colon Right ureter, gonadal vessels

Hepatic flexure Gallbladder (medially)

Splenic flexure Spleen and tail of pancreas

Distal sigmoid/ upper rectum Left ureter

Rectum Ureters, autonomic nerves, seminal vesicles, prostate, urethra (distally)

Next question
Question 266 of 560

What is the most useful test to clinically distinguish between an upper and lower motor neurone
lesion of the facial nerve?

Blow cheeks out

Loss of chin reflex

Close eye

Raise eyebrow

Open mouth against resistance

Upper motor neurone lesions of the facial nerve- Paralysis of the lower half of face.
Lower motor neurone lesion- Paralysis of the entire ipsilateral face.

Theme from April 2012 Exam

Please rate this question:

Discuss and give feedback

Next question

Upper Vs Lower motor neurone lesions - Facial nerve

The nucleus of the facial nerve is located in the caudal aspect of the ventrolateral pontine
tegmentum. Its axons exit the ventral pons medial to the spinal trigeminal nucleus.

Any lesion occurring within or affecting the corticobulbar tract is known as an upper motor neuron
lesion. Any lesion affecting the individual branches (temporal, zygomatic, buccal, mandibular and
cervical) is known as a lower motor neuron lesion.
Branches of the facial nerve leaving the facial motor nucleus (FMN) for the muscles do so via both
left and right posterior (dorsal) and anterior (ventral) routes. In other words, this means lower motor
neurons of the facial nerve can leave either from the left anterior, left posterior, right anterior or right
posterior facial motor nucleus. The temporal branch travels out from the left and right posterior
components. The inferior four branches do so via the left and right anterior components. The left and
right branches supply their respective sides of the face (ipsilateral innervation). Accordingly, the
posterior components receive motor input from both hemispheres of the cerebral cortex (bilaterally),
whereas the anterior components receive strictly contra-lateral input. This means that the temporal
branch of the facial nerve receives motor input from both hemispheres of the cerebral cortex
whereas the zygomatic, buccal, mandibular and cervical branches receive information from only
contralateral hemispheres.

Now, because the anterior FMN receives only contralateral cortical input whereas the posterior
receives that which is bilateral, a corticobulbar lesion (UMN lesion) occurring in the left hemisphere
would eliminate motor input to the right anterior FMN component, thus removing signaling to the
inferior four facial nerve branches, thereby paralyzing the right mid- and lower-face. The posterior
component, however, although now only receiving input from the right hemisphere, is still able to
allow the temporal branch to sufficiently innervate the entire forehead. This means that the forehead
will not be paralyzed.

The same mechanism applies for an upper motor neuron lesion in the right hemisphere. The left
anterior FMN component no longer receives cortical motor input due to its strict contralateral
innervation, whereas the posterior component is still sufficiently supplied by the left hemisphere. The
result is paralysis of the left mid- and lower-face with an unaffected forehead.

On the other hand, a lower motor neuron lesion is a bit different.

A lesion on either the left or right side would affect both the anterior and posterior routes on that side
because of their close physical proximity to one another. So, a lesion on the left side would inhibit
muscle innervation from both the left posterior and anterior routes, thus paralyzing the whole left side
of the face (Bells Palsy). With this type of lesion, the bilateral and contalateral inputs of the posterior
and anterior routes, respectively, become irrelevant because the lesion is below the level of the
medulla and the facial motor nucleus. Whereas at a level above the medulla a lesion occurring in
one hemisphere would mean that the other hemisphere could still sufficiently innervate the posterior
facial motor nucleus, a lesion affecting a lower motor neuron would eliminate innervation altogether
because the nerves no longer have a means to receive compensatory contralateral input at a
downstream decussation.

Next question
Question 267 of 560

An 18 year old man is stabbed in the axilla during a fight. His axillary artery is lacerated and
repaired. However, the surgeon neglects to repair an associated injury to the upper trunk of the
brachial plexus. Which of the following muscles is least likely to demonstrate impaired function as a
result?

Palmar interossei

Infraspinatus

Brachialis

Supinator brevis

None of the above

The palmar interossei are supplied by the ulnar nerve. Which lies inferiorly and is therefore less
likely to be injured.
Please rate this question:

Discuss and give feedback


Next question

Brachial plexus

Origin Anterior rami of C5 to T1

Sections of the  Roots, trunks, divisions, cords, branches


plexus  Mnemonic:Real Teenagers Drink Cold Beer

Roots  Located in the posterior triangle


 Pass between scalenus anterior and medius

Trunks  Located posterior to middle third of clavicle


 Upper and middle trunks related superiorly to the subclavian artery
 Lower trunk passes over 1st rib posterior to the subclavian artery

Divisions Apex of axilla

Cords Related to axillary artery

Diagram illustrating the branches of the brachial plexus

Image sourced from Wikipedia

Cutaneous sensation of the upper limb


Image sourced from Wikipedia

Next question
Question 268 of 560

A 23 year old man is involved in a fight, during the dispute he sustains a laceration to the posterior
aspect of his right arm, approximately 2cm proximal to the olecranon process. On assessment in the
emergency department he is unable to extend his elbow joint. Which of the following tendons is most
likely to have been cut?

Triceps

Pronator teres

Brachioradialis

Brachialis

Biceps

Theme from 2009 Exam

The triceps muscle extends the elbow joint. The other muscles listed all produce flexion of the elbow
joint.
Please rate this question:

Discuss and give feedback


Next question

Triceps

Origin  Long head- infraglenoid tubercle of the scapula.


 Lateral head- dorsal surface of the humerus, lateral and proximal to the
groove of the radial nerve
 Medial head- posterior surface of the humerus on the inferomedial side of
the radial groove and both of the intermuscular septae

Insertion  Olecranon process of the ulna. Here the olecranon bursa is between the
triceps tendon and olecranon.
 Some fibres insert to the deep fascia of the forearm, posterior capsule of the
elbow (preventing the capsule from being trapped between olecranon and
olecranon fossa during extension)

Innervation Radial nerve

Blood Profunda brachii artery


supply

Action Elbow extension. The long head can adduct the humerus and and extend it from a
flexed position

Relations The radial nerve and profunda brachii vessels lie between the lateral and medial
heads
Next question
Question 269 of 560

A 25 year old man undergoes an excision of a pelvic chondrosarcoma, during the operation the
obturator nerve is sacrificed. Which of the following muscles is least likely to be affected as a result?

Adductor longus

Pectineus

Adductor magnus

Sartorius

Gracilis

Sartorius is supplied by the femoral nerve. In approximately 20% of the population, pectineus is
supplied by the accessory obturator nerve.
Please rate this question:

Discuss and give feedback


Next question

Obturator nerve

The obturator nerve arises from L2, L3 and L4 by branches from the ventral divisions of each of
these nerve roots. L3 forms the main contribution and the second lumbar branch is occasionally
absent. These branches unite in the substance of psoas major, descending vertically in its posterior
part to emerge from its medial border at the lateral margin of the sacrum. It then crosses the
sacroiliac joint to enter the lesser pelvis, it descends on obturator internus to enter the obturator
groove. In the lesser pelvis the nerve lies lateral to the internal iliac vessels and ureter, and is joined
by the obturator vessels lateral to the ovary or ductus deferens.

Supplies

 Medial compartment of thigh


 Muscles supplied: external obturator, adductor longus, adductor brevis, adductor magnus
(not the lower part-sciatic nerve), gracilis
 The cutaneous branch is often absent. When present, it passes between gracilis and
adductor longus near the middle part of the thigh, and supplies the skin and fascia of the
distal two thirds of the medial aspect.
Obturator canal

 Connects the pelvis and thigh: contains the obturator artery, vein, nerve which divides into
anterior and posterior branches.

Cadaveric cross section demonstrating relationships of the obturator nerve

Image sourced from Wikipedia

Next question
Question 270 of 560

You excitedly embark on your first laparoscopic cholecystectomy and during the operation the
anatomy of Calots triangle is more hostile than anticipated. Whilst trying to apply a haemostatic clip
you avulse the cystic artery. This is followed by brisk haemorrhage. From which source is this most
likely to originate ?

Right hepatic artery

Portal vein

Gastroduodenal artery

Liver bed

Common hepatic artery

The cystic artery is a branch of the right hepatic artery. There are recognised variations in the
anatomy of the blood supply to the gallbladder. However, the commonest situation is for the cystic
artery to branch from the right hepatic artery.
Please rate this question:

Discuss and give feedback


Next question

Gallbladder

 Fibromuscular sac with capacity of 50ml


 Columnar epithelium

Relations of the gallbladder


Anterior Liver

Posterior  Covered by peritoneum


 Transverse colon
 1st part of the duodenum
Laterally Right lobe of liver

Medially Quadrate lobe of liver

Arterial supply
Cystic artery (branch of Right hepatic artery)

Venous drainage
Directly to the liver

Nerve supply
Sympathetic- mid thoracic spinal cord, Parasympathetic- anterior vagal trunk

Common bile duct

Origin Confluence of cystic and common hepatic ducts

Relations at  Medially - Hepatic artery


origin  Posteriorly- Portal vein

Relations distally  Duodenum - anteriorly


 Pancreas - medially and laterally
 Right renal vein - posteriorly

Arterial supply Branches of hepatic artery and retroduodenal branches of gastroduodenal


artery

Hepatobiliary triangle

Medially Common hepatic duct

Inferiorly Cystic duct

Superiorly Inferior edge of liver

Contents Cystic artery

Relations of the gallbladder


© Image provided by the University of Sheffield

Next question
Question 271 of 560

A 43 year old man suffers a pelvic fracture which is complicated by an injury to the junction of the
membranous urethra to the bulbar urethra. In which of the following directions is the extravasated
urine most likely to pass?

Posteriorly into extra peritoneal tissues

Laterally into the buttocks

Into the abdomen

Anteriorly into the connective tissues surrounding the scrotum

None of the above

The superficial perineal pouch is a compartment bounded superficially by the superficial perineal
fascia, deep by the perineal membrane (inferior fascia of the urogenital diaphragm), and laterally by
the ischiopubic ramus. It contains the crura of the penis or clitoris, muscles, viscera, blood vessels,
nerves, the proximal part of the spongy urethra in males, and the greater vestibular glands in
females.
When urethral rupture occurs as in this case the urine will tend to pass anteriorly because the fascial
condensations will prevent lateral and posterior passage of the urine.
Please rate this question:

Discuss and give feedback


Next question

Urogenital triangle

The urogenital triangle is formed by the:

 Ischiopubic inferior rami


 Ischial tuberosities

A fascial sheet is attached to the sides, forming the inferior fascia of the urogenital diaphragm.

It transmits the urethra in males and both the urethra and vagina in females. The membranous
urethra lies deep to this structure and is surrounded by the external urethral sphincter.
Superficial to the urogenital diaphragm lies the superficial perineal pouch. In males this contains:

 Bulb of penis
 Crura of the penis
 Superficial transverse perineal muscle
 Posterior scrotal arteries
 Posterior scrotal nerves

In females the internal pudendal artery branches to become the posterior labial arteries in the
superficial perineal pouch.
Next question
Question 272 of 560

Which of the following does not pass through the superior orbital fissure?

Oculomotor nerve

Abducens nerve

Ophthalmic artery

Ophthalmic division of the trigeminal nerve

Ophthalmic veins

The ophthalmic artery, a branch of the internal carotid enters the orbit with the optic nerve in the
canal.

Please rate this question:

Discuss and give feedback

Next question

Foramina of the base of the skull

Foramen Location Contents

Foramen ovale Sphenoid Otic ganglion


bone V3 (Mandibular nerve:3rd branch of
trigeminal)
Accessory meningeal artery
Foramen Location Contents

Lesser petrosal nerve


Emissary veins

Foramen spinosum Sphenoid Middle meningeal artery


bone Meningeal branch of the Mandibular nerve

Foramen rotundum Sphenoid Maxillary nerve (V2)


bone

Foramen lacerum/ Sphenoid Base of the medial pterygoid plate.


carotid canal bone Internal carotid artery*
Nerve and artery of the pterygoid canal

Jugular foramen Temporal Anterior: inferior petrosal sinus


bone Intermediate: glossopharyngeal, vagus, and accessory nerves.
Posterior: sigmoid sinus (becoming the internal jugular vein) and
some meningeal branches from the occipital and ascending
pharyngeal arteries.

Foramen magnum Occipital Anterior and posterior spinal arteries


bone Vertebral arteries
Medulla oblongata

Stylomastoid Temporal Stylomastoid artery


foramen bone Facial nerve

Superior orbital Sphenoid Oculomotor nerve (III)


fissure bone Recurrent meningeal artery
Trochlear nerve (IV)
Lacrimal, frontal and nasociliary branches of ophthalmic nerve (V1)
Abducent nerve (VI)
Foramen Location Contents

Superior ophthalmic vein

*= In life the foramen lacerum is occluded by a cartilagenous plug. The ICA initially passes into the
carotid canal which ascends superomedially to enter the cranial cavity through the foramen lacerum.

Base of skull anatomical overview

Image sourced from Wikipedia

Next question
Question 273 of 560

Which nerve supplies the 1st web space of the foot?

Popliteal nerve

Superficial peroneal nerve

Deep peroneal nerve

Tibial nerve

Saphenous nerve

The first web space is innervated by the deep peroneal nerve. See diagram below:
Image sourced from Wikipedia

Please rate this question:

Discuss and give feedback


Next question

Deep peroneal nerve

Origin From the common peroneal nerve, at the lateral aspect of the fibula, deep to
peroneus longus
Nerve root values L4, L5, S1, S2

Course and  Pierces the anterior intermuscular septum to enter the anterior
relation compartment of the lower leg
 Passes anteriorly down to the ankle joint, midway between the two
malleoli

Terminates In the dorsum of the foot

Muscles  Tibialis anterior


innervated  Extensor hallucis longus
 Extensor digitorum longus
 Peroneus tertius
 Extensor digitorum brevis

Cutaneous Web space of the first and second toes


innervation

Actions  Dorsiflexion of ankle joint


 Extension of all toes (extensor hallucis longus and extensor
digitorum longus)
 Inversion of the foot

After its bifurcation past the ankle joint, the lateral branch of the deep peroneal nerve innervates the
extensor digitorum brevis and the extensor hallucis brevis
The medial branch supplies the web space between the first and second digits.
Next question
Question 274 of 560

During the course of a radical gastrectomy the surgeons detach the omentum and ligate the right
gastro-epiploic artery. From which vessel does it originate?

Superior mesenteric artery

Inferior mesenteric artery

Coeliac axis

Common hepatic artery

Gastroduodenal artery

Theme from January 2013 Exam


The gastroduodenal artery arises at the superior part of the duodenum and descends behind it to
terminate at its lower border. It terminates by dividing into the right gastro-epiploic artery and the
superior pancreaticoduodenal artery. The right gastro-opiploic artery passes to the left and passes
between the layers of the greater omentum to anastomose with the left gastro-epiploic artery.

Please rate this question:

Discuss and give feedback

Next question

Gastroduodenal artery

Supplies
Pylorus, proximal part of the duodenum, and indirectly to the pancreatic head (via the anterior and
posterior superior pancreaticoduodenal arteries)

Path
Most commonly arises from the common hepatic artery of the coeliac trunk
Terminates by bifurcating into the right gastroepiploic artery and the superior pancreaticoduodenal
artery

Image showing stomach reflected superiorly to illustrate the relationship of the gastroduodenal artery
to the first part of the duodenum

Image sourced from Wikipedia

Next question
Question 275 of 560

Which of the following is not an intrinsic muscle of the hand?

Opponens pollicis

Palmaris longus

Flexor pollicis brevis

Flexor digiti minimi brevis

Opponens digiti minimi

Mnemonic for intrinsic hand muscles


'A OF A OF A'

A bductor pollicis brevis


O pponens pollicis
F lexor pollicis brevis
A dductor pollicis (thenar muscles)
O pponens digiti minimi
F lexor digiti minimi brevis
A bductor digiti minimi (hypothenar muscles)

Palmaris longus originates in the forearm.


Please rate this question:

Discuss and give feedback


Next question

Hand

Anatomy of the hand


Bones  8 Carpal bones
 5 Metacarpals
 14 phalanges

Intrinsic Muscles 7 Interossei - Supplied by ulnar nerve


 3 palmar-adduct fingers
 4 dorsal- abduct fingers

Intrinsic muscles Lumbricals

 Flex MCPJ and extend the IPJ.


 Origin deep flexor tendon and insertion dorsal extensor hood
mechanism.
 Innervation: 1st and 2nd- median nerve, 3rd and 4th- deep branch of
the ulnar nerve.

Thenar eminence  Abductor pollicis brevis


 Opponens pollicis
 Flexor pollicis brevis

Hypothenar  Opponens digiti minimi


eminence  Flexor digiti minimi brevis
 Abductor digiti minimi
Image sourced from Wikipedia

Fascia and compartments of the palm


The fascia of the palm is continuous with the antebrachial fascia and the fascia of the dorsum of the
hand. The palmar fascia is thin over the thenar and hypothenar eminences. In contrast the palmar
fascia is relatively thick. The palmar aponeurosis covers the soft tissues and overlies the flexor
tendons. The apex of the palmar aponeurosis is continuous with the flexor retinaculum and the
palmaris longus tendon. Distally, it forms four longitudinal digital bands that attach to the bases of
the proximal phalanges, blending with the fibrous digital sheaths.
A medial fibrous septum extends deeply from the medial border of the palmar aponeurosis to the 5th
metacarpal. Lying medial to this are the hypothenar muscles. In a similar fashion, a lateral fibrous
septum extends deeply from the lateral border of the palmar aponeurosis to the 3rd metacarpal. The
thenar compartment lies lateral to this area.
Lying between the thenar and hypothenar compartments is the central compartment. It contains the
flexor tendons and their sheaths, the lumbricals, the superficial palmar arterial arch and the digital
vessels and nerves.
The deepest muscular plane is the adductor compartment, which contains adductor pollicis.

Short muscles of the hand


These comprise the lumbricals and interossei. The four slender lumbrical muscles flex the fingers at
the metacarpophalangeal joints and extend the interphalangeal joint. The four dorsal interossei are
located between the metacarpals and the four palmar interossei lie on the palmar surface of the
metacarpals in the interosseous compartment of the hand.

Long flexor tendons and sheaths in the hand


The tendons of FDS and FDP enter the common flexor sheath deep to the flexor retinaculum. The
tendons enter the central compartment of the hand and fan out to their respective digital synovial
sheaths. Near the base of the proximal phalanx, the tendon of FDS splits to permit the passage of
FDP. The FDP tendons are attached to the margins of the anterior aspect of the base of the distal
phalanx.
The fibrous digital sheaths contain the flexor tendons and their synovial sheaths. These extend from
the heads of the metacarpals to the base of the distal phalanges.
Next question
Question 276 of 560

A man with lung cancer and bone metastasis in the thoracic spinal vertebral bodies, sustains a
pathological fracture at the level of T4. The fracture is unstable and the spinal cord is severely
compressed at this level. Which of the findings below will not be present 6 weeks after injury?

Extensor plantar reflexes

Spasticity of the lower limbs

Diminished patellar tendon reflex

Urinary incontinence

Sensory ataxia

A thoracic cord lesion causes spastic paraperesis, hyperrflexia and extensor plantar responses
(UMN lesion), incontinence, sensory loss below the lesion and 'sensory' ataxia.These features
typically manifest several weeks later, once spinal shock (in which areflexia predominates) has
resolved.
Please rate this question:

Discuss and give feedback


Next question

Spinal cord

 Located in a canal within the vertebral column that affords it structural support.
 Rostrally it continues to the medulla oblongata of the brain and caudally it tapers at a level
corresponding to the L1-2 interspace (in the adult), a central structure, the filum terminale
anchors the cord to the first coccygeal vertebra.
 The spinal cord is characterised by cervico-lumbar enlargements and these, broadly
speaking, are the sites which correspond to the brachial and lumbar plexuses respectively.

There are some key points to note when considering the surgical anatomy of the spinal cord:

* During foetal growth the spinal cord becomes shorter than the spinal canal, hence the adult site of
cord termination at the L1-2 level.

* Due to growth of the vertebral column the spine segmental levels may not always correspond to
bony landmarks as they do in the cervical spine.

* The spinal cord is incompletely divided into two symmetrical halves by a dorsal median
sulcus andventral median fissure. Grey matter surrounds a central canal that is continuous
rostrally with the ventricular system of the CNS.

* The grey matter is sub divided cytoarchitecturally into Rexeds laminae.

* Afferent fibres entering through the dorsal roots usually terminate near their point of entry but may
travel for varying distances in Lissauers tract. In this way they may establish synaptic connections
over several levels

* At the tip of the dorsal horn are afferents associated with nociceptive stimuli. The ventral horn
contains neurones that innervate skeletal muscle.

The key point to remember when revising CNS anatomy is to keep a clinical perspective in mind. So
it is worth classifying the ways in which the spinal cord may become injured. These include:

 Trauma either direct or as a result of disc protrusion


 Neoplasia either by direct invasion (rare) or as a result of pathological vertebral fracture
 Inflammatory diseases such as Rheumatoid disease, or OA (formation of osteophytes
compressing nerve roots etc.
 Vascular either as a result of stroke (rare in cord) or as complication of aortic dissection
 Infection historically diseases such as TB, epidural abscesses.

The anatomy of the cord will, to an extent dictate the clinical presentation. Some points/ conditions to
remember:

 Brown- Sequard syndrome-Hemisection of the cord producing ipsilateral loss of


proprioception and upper motor neurone signs, plus contralateral loss of pain and
temperature sensation. The explanation of this is that the fibres decussate at different levels.
 Lesions below L1 will tend to present with lower motor neurone signs

Next question
Question 277 of 560

Through which of the following foramina does the genital branch of the genitofemoral nerve exit the
abdominal cavity?

Superficial inguinal ring

Sciatic notch

Obturator foramen

Femoral canal

Deep inguinal ring

The genitofemoral nerve divides into two branches as it approaches the inguinal ligament. The
genital branch passes anterior to the external iliac artery through the deep inguinal ring into the
inguinal canal. It communicates with the ilioinguinal nerve in the inguinal canal (though this is seldom
of clinical significance).
Please rate this question:

Discuss and give feedback


Next question

Genitofemoral nerve

Supplies
Small area of the upper medial thigh.

Path

 Arises from the first and second lumbar nerves.


 Passes obliquely through psoas major, and emerges from its medial border opposite the
fibrocartilage between the third and fourth lumbar vertebrae.
 It then descends on the surface of psoas major, under cover of the peritoneum
 Divides into genital and femoral branches.
 The genital branch passes through the inguinal canal, within the spermatic cord, to supply
the skin overlying the skin and fascia of the scrotum. The femoral branch enters the thigh
posterior to the inguinal ligament, lateral to the femoral artery. It supplies an area of skin and
fascia over the femoral triangle.
 It may be injured during abdominal or pelvic surgery, or during inguinal hernia repairs.

Next question
Question 278 of 560

A 28 year old man lacerates the posterolateral aspect of his wrist with a knife in an attempted
suicide. On arrival in the emergency department the wound is inspected and found to be located
over the lateral aspect of the extensor retinaculum (which is intact). Which of the following structures
is at greatest risk of injury?

Superficial branch of the radial nerve

Radial artery

Dorsal branch of the ulnar nerve

Tendon of extensor carpi radialis brevis

Tendon of extensor digiti minimi

The superficial branch of the radial nerve passes superior to the extensor retinaculum in the position
of this laceration and is at greatest risk of injury. The dorsal branch of the ulnar nerve and artery also
pass superior to the extensor retinaculum n but are located medially.
Please rate this question:

Discuss and give feedback


Next question

Extensor retinaculum

The extensor rentinaculum is a thickening of the deep fascia that stretches across the back of the
wrist and holds the long extensor tendons in position.
Its attachments are:

 The pisiform and triquetral medially


 The end of the radius laterally

Structures related to the extensor retinaculum


Structures superficial to the  Basilic vein
 Dorsal cutaneous branch of the ulnar nerve
retinaculum  Cephalic vein
 Superficial branch of the radial nerve

Structures passing deep to the  Extensor carpi ulnaris tendon


extensor retinaculum  Extensor digiti minimi tendon
 Extensor digitorum and extensor indicis tendon
 Extensor pollicis longus tendon
 Extensor carpi radialis longus tendon
 Extensor carpi radialis brevis tendon
 Abductor pollicis longus and extensor pollicis
brevis tendons

Beneath the extensor retinaculum fibrous septa form six compartments that contain the extensor
muscle tendons. Each compartment has its own synovial sheath.

The radial artery


The radial artery passes between the lateral collateral ligament of the wrist joint and the tendons of
the abductor pollicis longus and extensor pollicis brevis.

Image illustrating the topography of tendons passing under the extensor retinaculum
Image sourced from Wikipedia

Next question
Question 279 of 560

A 43 year old man is reviewed in the clinic following a cardiac operation. A chest x-ray is performed
and a circular radio-opaque structure is noted medial to the 4th interspace on the left. Which of the
following procedures is the patient most likely to have undergone?

Aortic valve replacement with metallic valve

Tricuspid valve replacement with metallic valve

Tricuspid valve replacement with porcine valve

Pulmonary valve replacement with porcine valve

Mitral valve replacement with metallic valve

Theme from April 2012 Exam

Please rate this question:

Discuss and give feedback

Next question

Prosthetic heart valves on Chest X-rays

The aortic and mitral valves are most commonly replaced and when a metallic valve is used, can be
most readily identified on plain x-rays.
The presence of cardiac disease (such as cardiomegaly) may affect the figures quoted here.

Aortic
Usually located medial to the 3rd interspace on the right.

Mitral
Usually located medial to the 4th interspace on the left.

Tricuspid
Usually located medial to the 5th interspace on the right.

Please note that these are the sites at which an artificial valve may be located and are NOT the sites
of auscultation.

Next question
Question 280 of 560

A 63 year old lady is diagnosed as having an endometrial carcinoma arising from the uterine body.
To which nodal region will the tumour initially metastasise?

Para aortic nodes

Iliac lymph nodes

Inguinal nodes

Pre sacral nodes

Mesorectal lymph nodes

Theme from 2011 exam


Tumours of the uterine body will tend to spread to the iliac nodes initially. Tumour expansion
crossing different nodal margins this is of considerable clinical significance, if nodal clearance is
performed during a Wertheims type hysterectomy.
Please rate this question:

Discuss and give feedback


Next question

Lymphatic drainage of the ovaries, uterus and cervix

 The ovaries drain to the para-aortic lymphatics via the gonadal vessels.
 The uterine fundus has a lymphatic drainage that runs with the ovarian vessels and may thus
drain to the para-aortic nodes. Some drainage may also pass along the round ligament to the
inguinal nodes.
 The body of the uterus drains through lymphatics contained within the broad ligament to the
iliac lymph nodes.
 The cervix drains into three potential nodal stations; laterally through the broad ligament to
the external iliac nodes, along the lymphatics of the uterosacral fold to the presacral nodes
and posterolaterally along lymphatics lying alongside the uterine vessels to the internal iliac
nodes.

Next question
Question 281 of 560

Transection of the radial nerve at the level of the axilla will result in all of the following except:

Loss of elbow extension.

Loss of extension of the interphalangeal joints.

Loss of metacarpophalangeal extension.

Loss of triceps reflex.

Loss of sensation overlying the first dorsal interosseous.

These may still extend by virtue of retained lumbrical muscle function.


Please rate this question:

Discuss and give feedback


Next question

Radial nerve

Continuation of posterior cord of the brachial plexus (root values C5 to T1)

Path

 In the axilla: lies posterior to the axillary artery on subscapularis, latissimus dorsi and teres
major.
 Enters the arm between the brachial artery and the long head of triceps (medial to humerus).
 Spirals around the posterior surface of the humerus in the groove for the radial nerve.
 At the distal third of the lateral border of the humerus it then pierces the intermuscular
septum and descends in front of the lateral epicondyle.
 At the lateral epicondyle it lies deeply between brachialis and brachioradialis where it then
divides into a superficial and deep terminal branch.
 Deep branch crosses the supinator to become the posterior interosseous nerve.

In the image below the relationships of the radial nerve can be appreciated
Image sourced from Wikipedia

Regions innervated
 Triceps
 Anconeus
Motor (main nerve)  Brachioradialis
 Extensor carpi radialis

 Supinator
 Extensor carpi ulnaris
 Extensor digitorum
Motor (posterior  Extensor indicis
interosseous branch)  Extensor digiti minimi
 Extensor pollicis longus and brevis
 Abductor pollicis longus

The area of skin supplying the proximal phalanges on the dorsal aspect of the
Sensory hand is supplied by the radial nerve (this does not apply to the little finger and
part of the ring finger)

Muscular innervation and effect of denervation


Anatomical
location Muscle affected Effect of paralysis
Anatomical
location Muscle affected Effect of paralysis

Shoulder Long head of triceps Minor effects on shoulder stability in abduction

Arm Triceps Loss of elbow extension

Forearm Supinator Weakening of supination of prone hand and


Brachioradialis elbow flexion in mid prone position
Extensor carpi radialis
longus and brevis

The cutaneous sensation of the upper limb- illustrating the contribution of the radial nerve

Image sourced from Wikipedia

Next question
Question 282 of 560

Which of the following structures is not located in the superficial perineal space in females?

Posterior labial arteries

Pudendal nerve

Superficial transverse perineal muscle

Greater vestibular glands

None of the above

The pudendal nerve is located in the deep perineal space and then branches to innervate more
superficial structures.
Please rate this question:

Discuss and give feedback


Next question

Urogenital triangle

The urogenital triangle is formed by the:

 Ischiopubic inferior rami


 Ischial tuberosities

A fascial sheet is attached to the sides, forming the inferior fascia of the urogenital diaphragm.

It transmits the urethra in males and both the urethra and vagina in females. The membranous
urethra lies deep to this structure and is surrounded by the external urethral sphincter.

Superficial to the urogenital diaphragm lies the superficial perineal pouch. In males this contains:

 Bulb of penis
 Crura of the penis
 Superficial transverse perineal muscle
 Posterior scrotal arteries
 Posterior scrotal nerves

In females the internal pudendal artery branches to become the posterior labial arteries in the
superficial perineal pouch.
Next question
Question 283 of 560

Which of the following is not a branch of the hepatic artery?

Pancreatic artery

Cystic artery

Right gastric artery

Right hepatic artery

Gastroduodenal artery

The pancreatic artery is a branch of the splenic artery.


Please rate this question:

Discuss and give feedback


Next question

Coeliac axis

The coeliac axis has three main branches.

 Left gastric
 Hepatic: branches-Right Gastric, Gastroduodenal, Right Gastroepiploic, Superior
Pancreaticoduodenal, Cystic (occasionally).
 Splenic: branches- Pancreatic, Short Gastric, Left Gastroepiploic

It occasionally gives off one of the inferior phrenic arteries.


Image sourced from Wikipedia

Relations
Anteriorly Lesser omentum

Right Right coeliac ganglion and caudate process of liver

Left Left coeliac ganglion and gastric cardia

Inferiorly Upper border of pancreas and renal vein

Next question
Question 284 of 560

Which of the following structures does not pass behind the piriformis muscle in the greater sciatic
foramen?

Sciatic nerve

Posterior cutaneous nerve of the thigh

Inferior gluteal artery

Obturator nerve

None of the above

The obturator nerve does not pass through the greater sciatic foramen.
Please rate this question:

Discuss and give feedback


Next question

Greater sciatic foramen

Contents
Nerves  Sciatic Nerve
 Superior and Inferior Gluteal Nerves
 Pudendal Nerve
 Posterior Femoral Cutaneous Nerve
 Nerve to Quadratus Femoris
 Nerve to Obturator internus

Vessels  Superior Gluteal Artery and vein


 Inferior Gluteal Artery and vein
 Internal Pudendal Artery and vein

Piriformis
The piriformis is a landmark for identifying structures passing out of the sciatic notch

 Above piriformis: Superior gluteal vessels


 Below piriformis: Inferior gluteal vessels, sciatic nerve (10% pass through it, <1% above it),
posterior cutaneous nerve of the thigh

Greater sciatic foramen boundaries


Anterolaterally Greater sciatic notch of the ilium

Posteromedially Sacrotuberous ligament

Inferior Sacrospinous ligament and the ischial spine

Superior Anterior sacroiliac ligament

The greater sciatic foramen


Image sourced from Wikipedia

Structures passing between both foramina (Medial to lateral)

 Pudendal nerve
 Internal pudendal artery
 Nerve to obturator internus

Contents of the lesser sciatic foramen

 Tendon of the obturator internus


 Pudendal nerve
 Internal pudendal artery and vein
 Nerve to the obturator internus

Next question
Question 285 of 560

A 56 year old man is undergoing a right nephrectomy. The surgeons divide the renal artery. At what
level does this usually branch off the abdominal aorta?

T9

L2

L3

T10

L4

The renal arteries usually branch off the aorta on a level with L2.
Please rate this question:

Discuss and give feedback


Next question

Renal arteries

 The right renal artery is longer than the left renal artery
 The renal vein/artery/pelvis enter the kidney at the hilum

Relations
Right Anterior- IVC, right renal vein, the head of the pancreas, and the descending part of the
duodenum

Left Anterior- left renal vein, the tail of the pancreas

Branches
 The renal arteries are direct branches off the aorta (upper border of L2- right side and L1 -
left side)
 In 30% there may be accessory arteries (mainly left side). Instead of entering the kidney at
the hilum, they usually pierce the upper or lower part of the organ.
 Before reaching the hilum of the kidney, each artery divides into four or five segmental
branches (renal vein anterior and ureter posterior); which then divide within the sinus into
lobar arteries supplying each pyramid and cortex.
 Each vessel gives off some small inferior suprarenal branches to the suprarenal gland, the
ureter, and the surrounding cellular tissue and muscles.

Next question
Question 286 of 560

A 23 year old man is shot in the chest during a robbery. The left lung is lacerated and is bleeding. An
emergency thoracotomy is performed. The surgeons place a clamp over the hilum of the left lung.
Which of the following structures lies most anteriorly at this level?

Vagus nerve

Oesophagus

Descending aorta

Phrenic nerve

Azygos vein

The phrenic nerve lies anteriorly at this point. The vagus passes anteriorly and then arches
backwards immediately superior to the root of the left bronchus, giving off the recurrent laryngeal
nerve as it does so.
Please rate this question:

Discuss and give feedback


Next question

Lung anatomy

The right lung is composed of 3 lobes divided by the oblique and transverse fissures. The left lung
has two lobes divided by the oblique fissure.The apex of both lungs is approximately 4cm superior to
the sterno-costal joint of the first rib. Immediately below this is a sulcus created by the subclavian
artery.

Peripheral contact points of the lung

 Base: diaphragm
 Costal surface: corresponds to the cavity of the chest
 Mediastinal surface: Contacts the mediastinal pleura. Has the cardiac impression. Above and
behind this concavity is a triangular depression named the hilum, where the structures which
form the root of the lung enter and leave the viscus. These structures are invested by pleura,
which, below the hilum and behind the pericardial impression, forms the pulmonary ligament
Right lung
Above the hilum is the azygos vein; Superior to this is the groove for the superior vena cava and
right innominate vein; behind this, and nearer the apex, is a furrow for the innominate artery. Behind
the hilum and the attachment of the pulmonary ligament is a vertical groove for the oesophagus; In
front and to the right of the lower part of the oesophageal groove is a deep concavity for the
extrapericardiac portion of the inferior vena cava.

The root of the right lung lies behind the superior vena cava and the right atrium, and below the
azygos vein.

The right main bronchus is shorter, wider and more vertical than the left main bronchus and
therefore the route taken by most foreign bodies.

Image sourced from Wikipedia

Left lung
Above the hilum is the furrow produced by the aortic arch, and then superiorly the groove
accommodating the left subclavian artery; Behind the hilum and pulmonary ligament is a vertical
groove produced by the descending aorta, and in front of this, near the base of the lung, is the lower
part of the oesophagus.

The root of the left lung passes under the aortic arch and in front of the descending aorta.
Image sourced from Wikipedia

Inferior borders of both lungs

 6th rib in mid clavicular line


 8th rib in mid axillary line
 10th rib posteriorly

The pleura runs two ribs lower than the corresponding lung level.

Bronchopulmonary segments
Segment number Right lung Left lung

1 Apical Apical

2 Posterior Posterior

3 Anterior Anterior

4 Lateral Superior lingular

5 Medial Inferior lingular

6 Superior (apical) Superior (apical)


Segment number Right lung Left lung

7 Medial basal Medial basal

8 Anterior basal Anterior basal

9 Lateral basal Lateral basal

10 Posterior basal Posterior basal

Next question
Question 287 of 560

A 22 year old man presents with appendicitis. At operation the appendix is retrocaecal and difficult to
access. Division of which of the following anatomical structures should be undertaken?

Ileocolic artery

Mesentery of the caecum

Gonadal vessels

Lateral peritoneal attachments of the caecum

Right colic artery

The commonest appendiceal location is retrocaecal. Those struggling to find it at operation should
trace the tenia to the caecal pole where the appendix is located. If it cannot be mobilised easily then
division of the lateral caecal peritoneal attachments (as for a right hemicolectomy) will allow caecal
mobilisation and facilitate the procedure.
Please rate this question:

Discuss and give feedback


Next question

Appendix

 Location: Base of caecum.


 Up to 10cm long.
 Mainly lymphoid tissue (Hence mesenteric adenitis may mimic appendicitis).
 Caecal taenia coli converge at base of appendix and form a longitudinal muscle cover over
the appendix. This convergence should facilitate its identification at surgery if it is retrocaecal
and difficult to find (which it can be when people start doing appendicectomies!)
 Arterial supply: Appendicular artery (branch of the ileocolic).
 It is intra peritoneal.

McBurney's point

 1/3 of the way along a line drawn from the Anterior Superior Iliac Spine to the Umbilicus
6 Positions:

 Retrocaecal 74%
 Pelvic 21%
 Postileal
 Subcaecal
 Paracaecal
 Preileal

Next question
Question 288 of 560

Which of the following muscles does not adduct the shoulder?

Teres major

Pectoralis major

Coracobrachialis

Supraspinatus

Latissimus dorsi

Supraspinatus is an abductor of the shoulder.


Please rate this question:

Discuss and give feedback


Next question

Shoulder joint

 Shallow synovial ball and socket type of joint.


 It is an inherently unstable joint, but is capable to a wide range of movement.
 Stability is provided by muscles of the rotator cuff that pass from the scapula to insert in the
greater tuberosity (all except sub scapularis-lesser tuberosity).

Glenoid labrum

 Fibrocartilaginous rim attached to the free edge of the glenoid cavity


 Tendon of the long head of biceps arises from within the joint from the supraglenoid tubercle,
and is fused at this point to the labrum.
 The long head of triceps attaches to the infraglenoid tubercle

Fibrous capsule
 Attaches to the scapula external to the glenoid labrum and to the labrum itself (postero-
superiorly)
 Attaches to the humerus at the level of the anatomical neck superiorly and the surgical neck
inferiorly
 Anteriorly the capsule is in contact with the tendon of subscapularis, superiorly with the
supraspinatus tendon, and posteriorly with the tendons of infraspinatus and teres minor. All
these blend with the capsule towards their insertion.
 Two defects in the fibrous capsule; superiorly for the tendon of biceps. Anteriorly there is a
defect beneath the subscapularis tendon.
 The inferior extension of the capsule is closely related to the axillary nerve at the surgical
neck and this nerve is at risk in anteroinferior dislocations. It also means that proximally sited
osteomyelitis may progress to septic arthritis.

Movements and muscles


Flexion Anterior part of deltoid
Pectoralis major
Biceps
Coracobrachialis

Extension Posterior deltoid


Teres major
Latissimus dorsi

Adduction Pectoralis major


Latissimus dorsi
Teres major
Coracobrachialis

Abduction Mid deltoid


Supraspinatus

Medial rotation Subscapularis


Anterior deltoid
Teres major
Latissimus dorsi

Lateral rotation Posterior deltoid


Infraspinatus
Teres minor
Important anatomical relations
Anteriorly Brachial plexus
Axillary artery and vein

Posterior Suprascapular nerve


Suprascapular vessels

Inferior Axillary nerve


Circumflex humeral vessels

Next question
Question 289 of 560

Which of these muscles is innervated by the cervical branch of the facial nerve?

Masseter

Sternocleidomastoid

Platysma

Geniohyoid

Sternothyroid

The cervical branch of the facial nerve innervates platysma.


Please rate this question:

Discuss and give feedback


Next question

Facial nerve

The facial nerve is the main nerve supplying the structures of the second embryonic branchial arch.
It is predominantly an efferent nerve to the muscles of facial expression, digastric muscle and also to
many glandular structures. It contains a few afferent fibres which originate in the cells of its genicular
ganglion and are concerned with taste.

Supply - 'face, ear, taste, tear'

 Face: muscles of facial expression


 Ear: nerve to stapedius
 Taste: supplies anterior two-thirds of tongue
 Tear: parasympathetic fibres to lacrimal glands, also salivary glands

Path
Subarachnoid path

 Origin: motor- pons, sensory- nervus intermedius


 Pass through the petrous temporal bone into the internal auditory meatus with the
vestibulocochlear nerve. Here they combine to become the facial nerve.

Facial canal path

 The canal passes superior to the vestibule of the inner ear


 At the medial aspect of the middle ear, it becomes wider and contains the geniculate
ganglion.

- 3 branches:
1. greater petrosal nerve
2. nerve to stapedius
3. chorda tympani

Stylomastoid foramen

 Passes through the stylomastoid foramen (tympanic cavity anterior and mastoid antrum
posteriorly)
 Posterior auricular nerve and branch to posterior belly of digastric and stylohyoid muscle

Face
Enters parotid gland and divides into 5 branches:

 Temporal branch
 Zygomatic branch
 Buccal branch
 Marginal mandibular branch
 Cervical branch

Next question
Question 290 of 560

During a thyroidectomy the surgeons ligate the inferior thyroid artery. From which vessel does this
structure usually originate?

External carotid artery

Thyrocervical trunk

Internal carotid artery

Subclavian artery

Vertebral artery

The inferior thyroid artery originates from the thyrocervical trunk. This is a branch of the subclavian
artery.
Please rate this question:

Discuss and give feedback


Next question

Thyroid gland

 Right and left lobes connected by isthmus


 Surrounded by sheath from pretracheal layer of deep fascia
 Apex: Lamina of thyroid cartilage
 Base: 4th-5th tracheal ring
 Pyramidal lobe: from isthmus
 May be attached to foramen caecum at the base of the tongue

Relations
Anteromedially  Sternothyroid
 Superior belly of omohyoid
 Sternohyoid
 Anterior aspect of sternocleidomastoid
Posterolaterally Carotid sheath

Medially  Larynx
 Trachea
 Pharynx
 Oesophagus
 Cricothyroid muscle
 External laryngeal nerve (near superior thyroid artery)
 Recurrent laryngeal nerve (near inferior thyroid artery)

Posterior  Parathyroid glands


 Anastomosis of superior and inferior thyroid arteries

Isthmus  Anteriorly: Sternothyroids, sternohyoids, anterior jugular veins


 Posteriorly: 2nd, 3rd, 4th tracheal rings (attached via Ligament of
Berry)

Blood Supply
Arterial  Superior thyroid artery (1st branch of external carotid)
 Inferior thyroid artery (from thyrocervical trunk)
 Thyroidea ima (in 10% of population -from brachiocephalic artery or aorta)

Venous  Superior and middle thyroid veins - into the IJV


 Inferior thyroid vein - into the brachiocephalic veins

Next question
Question 291 of 560

A 56 year old man is left impotent following an abdomino-perineal excision of the colon and rectum.
What is the most likely explanation?

Psychosexual issues related to an end colostomy

Damage to the sacral venous plexus during total mesorectal excision

Damage to the left ureter during sigmoid mobilisation

Damage to the hypogastric plexus during mobilisation of the inferior mesenteric artery

Damage to the internal iliac artery during total mesorectal excision

Autonomic nerve injury is the most common cause.


Please rate this question:

Discuss and give feedback


Next question

Nerve lesions during surgery

A variety of different procedures carry the risk of iatrogenic nerve injury. These are important not
only from the patients perspective but also from a medicolegal standpoint.

The following operations and their associated nerve lesions are listed here:

 Posterior triangle lymph node biopsy and accessory nerve lesion.


 Lloyd Davies stirrups and common peroneal nerve.
 Thyroidectomy and laryngeal nerve.
 Anterior resection of rectum and hypogastric autonomic nerves.
 Axillary node clearance; long thoracic nerve, thoracodorsal nerve and intercostobrachial
nerve.
 Inguinal hernia surgery and ilioinguinal nerve.
 Varicose vein surgery- sural and saphenous nerves.
 Posterior approach to the hip and sciatic nerve.
 Carotid endarterectomy and hypoglossal nerve.
There are many more, with sound anatomical understanding of the commonly performed procedures
the incidence of nerve lesions can be minimised. They commonly occur when surgeons operate in
an unfamiliar tissue plane or by blind placement of haemostats (not recommended).
Next question
Question 292 of 560

A 73 year old man is due to undergo a radical prostatectomy for carcinoma of the prostate gland. To
which of the following lymph nodes will the tumour drain primarily?

Para aortic

Internal iliac

Superficial inguinal

Meso rectal

None of the above

The prostate lymphatic drainage is primarily to the internal iliac nodes and also the sacral nodes.
Although internal iliac is the first site.
Please rate this question:

Discuss and give feedback


Next question

Prostate gland

The prostate gland is approximately the shape and size of a walnut and is located inferior to the
bladder. It is separated from the rectum by Denonvilliers fascia and its blood supply is derived from
the internal iliac vessels (via inferior vesical artery). The internal sphincter lies at the apex of the
gland and may be damaged during prostatic surgery, affected individuals may complain of
retrograde ejaculation.

Summary of prostate gland


Arterial supply Inferior vesical artery (from internal iliac)

Venous drainage Prostatic venous plexus (to paravertebral veins)

Lymphatic Internal iliac nodes


drainage

Innervation Inferior hypogastric plexus

Dimensions  Transverse diameter (4cm)


 AP diameter (2cm)
 Height (3cm)

Lobes  Posterior lobe: posterior to urethra


 Median lobe: posterior to urethra, in between ejaculatory ducts
 Lateral lobes x 2
 Isthmus

Zones  Peripheral zone: subcapsular portion of posterior prostate. Most


prostate cancers are here
 Central zone
 Transition zone
 Stroma

Relations
Pubic symphysis
Anterior Prostatic venous plexus

Posterior Denonvilliers fascia


Rectum
Ejaculatory ducts

Lateral Venous plexus (lies on prostate)


Levator ani (immediately below the puboprostatic ligaments)
Image sourced from Wikipedia

Next question
Question 293 of 560

Which of the following statements relating to the vertebral column is false?

There are 7 cervical vertebrae

The cervical and lumbar lordosis are secondary curves developing after birth due to
change in shape of the intervertebral discs

The lumbar vertebrae do not have a transverse process foramina

The lumbar vertebrae receive blood directly from the aorta

The spinous process is formed by the junction of the pedicles posteriorly

The spinous process is formed by 2 laminae posteriorly.


Please rate this question:

Discuss and give feedback


Next question

Vertebral column

 There are 7 cervical, 12 thoracic, 5 lumbar, and 5 sacral vertebrae.


 The spinal cord segmental levels do not necessarily correspond to the vertebral segments.
For example, while the C1 cord is located at the C1 vertebra, the C8 cord is situated at the
C7 vertebra. While the T1 cord is situated at the T1 vertebra, the T12 cord is situated at the
T8 vertebra. The lumbar cord is situated between T9 and T11 vertebrae. The sacral cord is
situated between the T12 to L2 vertebrae.

Cervical vertebrae
The interface between the first and second vertebra is called the atlanto-axis junction. The C3 cord
contains the phrenic nucleus.
Muscle Nerve root value

Deltoid C5,6
Muscle Nerve root value

Biceps C5,6

Wrist extensors C6-8

Triceps C6-8

Wrist flexors C6-T1

Hand muscles C8-T1

Thoracic vertebrae
The thoracic vertebral segments are defined by those that have a rib. The spinal roots form the
intercostal nerves that run on the bottom side of the ribs and these nerves control the intercostal
muscles and associated dermatomes.

Lumbosacral vertebrae
Form the remainder of the segments below the vertebrae of the thorax. The lumbosacral spinal cord,
however, starts at about T9 and continues only to L2. It contains most of the segments that innervate
the hip and legs, as well as the buttocks and anal regions.

Cauda Equina
The spinal cord ends at L1-L2 vertebral level. The tip of the spinal cord is called the conus. Below
the conus, there is a spray of spinal roots that is called the cauda equina. Injuries below L2
represent injuries to spinal roots rather than the spinal cord proper.
Next question
Question 294 of 560

A 78 year old lady falls over in her nursing home and sustains a displaced intracapsular fracture of
the femoral neck. A decision is made to perform a hemi arthroplasty through a lateral approach.
Which of these vessels will be divided to facilitate access?

Saphenous vein

Superior gluteal artery

Superficial circumflex iliac artery

Profunda femoris artery

Transverse branch of the lateral circumflex artery

During the Hardinge style lateral approach the transverse branch of the lateral circumflex artery is
divided to gain access. The vessels and its branches are illustrated below:
Image sourced from Wikipedia

Please rate this question:

Discuss and give feedback


Next question

Hip joint

 Head of femur articulates with acetabulum of the pelvis


 Both covered by articular hyaline cartilage
 The acetabulum forms at the union of the ilium, pubis, and ischium
 The triradiate cartilage (Y-shaped growth plate) separates the pelvic bones
 The acetabulum holds the femoral head by the acetabular labrum
 Normal angle between femoral head and femoral shaft is 130o

Ligaments

 Transverse ligament: joints anterior and posterior ends of the articular cartilage
 Head of femur ligament (ligamentum teres): acetabular notch to the fovea. Contains arterial
supply to head of femur in children.

Image sourced from Wikipedia


Image sourced from Wikipedia

Extracapsular ligaments

 Iliofemoral ligament: inverted Y shape. Anterior iliac spine to the trochanteric line
 Pubofemoral ligament: acetabulum to lesser trochanter
 Ischiofemoral ligament: posterior support. Ischium to greater trochanter.

Blood supply
Medial circumflex femoral and lateral circumflex femoral arteries (Branches of profunda femoris).
Also from the inferior gluteal artery. These form an anastomosis and travel to up the femoral neck to
supply the head.
Next question
Question 295 of 560

A 73 year old man undergoes a sub total oesophagectomy with anastomosis of the stomach to the
cervical oesophagus. Which vessel will be primarily responsible for the arterial supply to the
oesophageal portion of the anastomosis?

Superior thyroid artery

Internal carotid artery

Direct branches from the thoracic aorta

Inferior thyroid artery

Subclavian artery

The cervical oesophagus is supplied by the inferior thyroid artery. The thoracic oesophagus
(removed in this case) is supplied by direct branches from the thoracic aorta.
Please rate this question:

Discuss and give feedback


Next question

Oesophagus

 25cm long
 Starts at C6 vertebra, pierces diaphragm at T10 and ends at T11
 Squamous epithelium

Constrictions of the oesophagus


Structure Distance from incisors

Cricoid cartilage 15cm

Arch of the Aorta 22.5cm


Left principal bronchus 27cm

Diaphragmatic hiatus 40cm

Relations
Anteriorly  Trachea to T4
 Recurrent laryngeal nerve
 Left bronchus, Left atrium
 Diaphragm

Posteriorly  Thoracic duct to left at T5


 Hemiazygos to the left T8
 Descending aorta
 First 2 intercostal branches of aorta

Left  Thoracic duct


 Left subclavian artery

Right  Azygos vein

Arterial, venous and lymphatic drainage of the oesophagus


Artery Vein Lymphatics Muscularis externa

Upper Inferior Inferior thyroid Deep Striated muscle


third thyroid cervical

Mid third Aortic Azygos branches Mediastinal Smooth & striated


branches muscle

Lower Left gastric Posterior mediastinal and Gastric Smooth muscle


third coeliac

Nerve supply
 Upper half is supplied by recurrent laryngeal nerve
 Lower half by oesophageal plexus (vagus)

Histology

 Mucosa :Non-keratinized stratified squamous epithelium


 Submucosa: glandular tissue
 Muscularis externa (muscularis): composition varies. See table
 Adventitia

Next question
Question 296 of 560

Which of the following structures is not closely related to the brachial artery?

Ulnar nerve

Median nerve

Cephalic vein

Long head of biceps

Median cubital vein

The cephalic vein lies superficially and on the contralateral side of the arm to the brachial artery. The
relation of the ulnar nerves and others are demonstrated in the image below:

Image sourced from Wikipedia

Please rate this question:


Discuss and give feedback

Next question

Brachial artery

The brachial artery begins at the lower border of teres major as a continuation of the axillary artery. It
terminates in the cubital fossa at the level of the neck of the radius by dividing into the radial and
ulnar arteries.

Relations
Posterior relations include the long head of triceps with the radial nerve and profunda vessels
intervening. Anteriorly it is overlapped by the medial border of biceps.
It is crossed by the median nerve in the middle of the arm.
In the cubital fossa it is separated from the median cubital vein by the bicipital aponeurosis.
The basilic vein is in contact at the most proximal aspect of the cubital fossa and lies medially.

Next question
Question 297-299 of 560

Theme: Anatomy of the abdominal aorta

A. Common iliac artery

B. Median sacral artery

C. Left renal artery

D. Right renal artery

E. Inferior mesenteric artery

F. Superior mesenteric artery

G. Coeliac axis

H. Lumbar arteries

Please select the branch of the abdominal aorta that most closely matches the description provided.
Each option may be used once, more than once or not at all.

297. An aortic branch that leaves the aorta approximately 1cm below the coeliac axis.

You answered Common iliac artery

The correct answer is Superior mesenteric artery

The SMA leaves the aorta approximately 1cm below the coeliac axis. This is usually a level of L1
to L 2. It's crossed anteriorly by the splenic vein and the body of the pancreas. It runs
downwards and forwards anterior to the uncinate process.

298. The most inferior single branch of the aorta.

You answered Common iliac artery

The correct answer is Median sacral artery


The median sacral artery leaves the aorta a little above its bifurcation. It descends in the midline
anterior to L4 and L5.

299. The most inferior anterior branch of the aorta.

You answered Common iliac artery

The correct answer is Inferior mesenteric artery

The IMA leaves the front of of the aorta usually about 3 to 4cm superior to its bifurcation.

Please rate this question:

Discuss and give feedback

Next question

Abdominal aortic branches

Branches Level Paired Type

Inferior phrenic T12 (Upper border) Yes Parietal

Coeliac T12 No Visceral

Superior mesenteric L1 No Visceral

Middle suprarenal L1 Yes Visceral


Branches Level Paired Type

Renal L1-L2 Yes Visceral

Gonadal L2 Yes Visceral

Lumbar L1-L4 Yes Parietal

Inferior mesenteric L3 No Visceral

Median sacral L4 No Parietal

Common iliac L4 Yes Terminal

Next question
Question 300 of 560

Which ligament keeps the head of the radius connected to the radial notch of the ulna?

Annular (orbicular) ligament

Quadrate ligament

Radial collateral ligament of the elbow

Ulnar collateral ligament

Radial collateral ligament

Theme from September 2013


The annular ligament connects the radial head to the radial notch of the ulna. This is illustrated
below:

Image sourced from Wikipedia


Please rate this question:

Discuss and give feedback


Next question

Radius

The radius is one of the two long forearm bones that extends from the lateral side of the elbow to the
thumb side of the wrist. It has two expanded ends, of which the distal end is the larger. Key points
relating to its topography and relations are outlined below;

Upper end

 Articular cartilage- covers medial > lateral side


 Articulates with radial notch of the ulna by the annular ligament
 Muscle attachment- biceps brachii at the tuberosity

Shaft
Muscle attachment
Upper third of the body Supinator
Flexor digitorum superficialis
Flexor pollicis longus

Middle third of the body Pronator teres

Lower quarter of the body Pronator quadratus


Tendon of supinator longus

Lower end

 Quadrilateral
 Anterior surface- capsule of wrist joint
 Medial surface- head of ulna
 Lateral surface- ends in the styloid process
 Posterior surface: 3 grooves containing:

1. Tendons of extensor carpi radialis longus and brevis


2. Tendon of extensor pollicis longus
3. Tendon of extensor indicis
Image sourced from Wikipedia

Next question
Question 301 of 560

A 38 year old man presents to the clinic with shoulder weakness. On examination he has an inability
to initiate shoulder abduction. Which of the nerves listed below is least likely to be functioning
normally?

Suprascapular nerve

Medial pectoral nerve

Axillary nerve

Median nerve

Radial nerve

Theme from April 2012 Exam


Please rate this question:

Discuss and give feedback


Next question

Suprascapular nerve

The suprascapular nerve arises from the upper trunk of the brachial plexus. It lies superior to the
trunks of the brachial plexus and passes inferolaterally parallel to them. It passes through the
scapular notch, deep to trapezius. It innervates both supraspinatus and infraspinatus and initiates
abduction of the shoulder. If damaged, patients may be able to abduct the shoulder by leaning over
the affected side and deltoid can then continue to abduct the shoulder.
Image sourced from Wikipedia

Next question
Question 302 of 560

Which of the following statements relating to the Cavernous Sinus is false?

The pituitary gland lies medially

The internal carotid artery passes through it

The temporal lobe of the brain is a lateral relation

The mandibular branch of the trigeminal and optic nerve lie on the lateral wall

The ophthalmic veins drain into the anterior aspect of the sinus

The veins that drain into the sinus are important as sepsis can cause cavernous sinus thrombosis.
The maxillary branch of the trigeminal and not the mandibular branches pass through the sinus
Please rate this question:

Discuss and give feedback


Next question

Cavernous sinus

The cavernous sinuses are paired and are situated on the body of the sphenoid bone. It runs from
the superior orbital fissure to the petrous temporal bone.

Relations
Medial Lateral

Pituitary fossa
Temporal lobe
Sphenoid sinus

Contents
Lateral wall components (from top to bottom:)
Oculomotor nerve
Trochlear nerve
Ophthalmic nerve
Maxillary nerve

Contents of the sinus (from medial to lateral:)


Internal carotid artery (and sympathetic plexus)
Abducens nerve

Blood supply
Ophthalmic vein, superficial cortical veins, basilar plexus of veins posteriorly.

Drains into the internal jugular vein via: the superior and inferior petrosal sinuses

Image sourced from Wikipedia

Next question
Question 303 of 560

Which of the following is not a branch of the subclavian artery?

Superior thyroid artery

Vertebral artery

Thyrocervical trunk

Internal thoracic artery

Dorsal scapular artery

Mnemonic for the branches of the subclavian artery: VIT C & D

V ertebral artery
I nternal thoracic
T hyrocervical trunk

C ostalcervical trunk
D orsal scapular

Superior thyroid artery is a branch of the external carotid artery.

Please rate this question:

Discuss and give feedback

Next question

Subclavian artery
Path

 The left subclavian comes directly off the arch of aorta


 The right subclavian arises from the brachiocephalic artery (trunk) when it bifurcates into the
subclavian and the right common carotid artery.
 From its origin, the subclavian artery travels laterally, passing between anterior and middle
scalene muscles, deep to scalenus anterior and anterior to scalenus medius. As the
subclavian artery crosses the lateral border of the first rib, it becomes the axillary artery. At
this point it is superficial and within the subclavian triangle.

Image sourced from Wikipedia

Branches

 Vertebral artery
 Internal thoracic artery
 Thyrocervical trunk
 Costocervical trunk
 Dorsal scapular artery

Next question
Question 304 of 560

During the repair of an atrial septal defect the surgeons note that blood starts to leak from the
coronary sinus. Which structure forms the largest tributary of the coronary sinus?

Thebesian veins

Great cardiac vein

Oblique vein

Small cardiac veins

None of the above

The great cardiac vein runs in the anterior interventricular groove, and is the largest tributary of the
coronary sinus. The thebesian veins drain into the heart directly.
Please rate this question:

Discuss and give feedback


Next question

Heart anatomy

The walls of each cardiac chamber comprise:

 Epicardium
 Myocardium
 Endocardium

Cardiac muscle is attached to the cardiac fibrous skeleton.

Relations
The heart and roots of the great vessels within the pericardial sac are related anteriorly to the
sternum, medial ends of the 3rd to 5th ribs on the left and their associated costal cartilages. The
heart and pericardial sac are situated obliquely two thirds to the left and one third to the right of the
median plane.

The pulmonary valve lies at the level of the left third costal cartilage.
The mitral valve lies at the level of the fourth costal cartilage.

Coronary sinus
This lies in the posterior part of the coronary groove and receives blood from the cardiac veins. The
great cardiac vein lies at its left and the middle and small cardiac veins lie on its right. The smallest
cardiac vein (anterior cardiac vein) drains into the right atrium directly.

Aortic sinus
Right coronary artery arises from the right aortic sinus, the left is derived from the left aortic sinus,
which lies posteriorly.

Right and left ventricles

Structure Left Ventricle

A-V Valve Mitral (double leaflet)

Walls Twice as thick as right

Trabeculae carnae Much thicker and more numerous

Right coronary artery


The RCA supplies:

 Right atrium
 Diaphragmatic part of the left ventricle
 Usually the posterior third of the interventricular septum
 The sino atrial node (60% cases)
 The atrio ventricular node (80% cases)

Left coronary artery


The LCA supplies:

 Left atrium
 Most of left ventricle
 Part of the right ventricle
 Anterior two thirds of the inter ventricular septum
 The sino atrial node (remaining 40% cases)

Innervation of the heart


Autonomic nerve fibres from the superficial and deep cardiac plexus. These lie anterior to the
bifurcation of the trachea, posterior to the ascending aorta and superior to the bifurcation of the
pulmonary trunk. The parasympathetic supply to the heart is from presynaptic fibres of the vagus
nerves.

Valves of the heart


Mitral valve Aortic valve Pulmonary valve Tricuspid valve

2 cusps 3 cusps 3 cusps 3 cusps

First heart sound Second heart Second heart First heart sound
sound sound

1 anterior cusp 2 anterior cusps 2 anterior cusps 2 anterior cusps

Attached to chordae No chordae No chordae Attached to chordae


tendinae tendinae

Next question
Question 305 of 560

Which of the following vessels provides the greatest contribution to the arterial supply of the breast?

External mammary artery

Thoracoacromial artery

Internal mammary artery

Lateral thoracic artery

Subclavian artery

60% of the arterial supply to the breast is derived from the internal mammary artery. The external
mammary and lateral thoracic arteries also make a significant (but lesser) contribution. This is of
importance clinically in performing reduction mammoplasty procedures.
Please rate this question:

Discuss and give feedback


Next question

Breast

The breast itself lies on a layer of pectoral fascia and the following muscles:
1. Pectoralis major
2. Serratus anterior
3. External oblique

Image showing the topography of the female breast


Image sourced from Wikipedia

Breast anatomy
Nerve supply Branches of intercostal nerves from T4-T6.

Arterial supply  Internal mammary (thoracic) artery


 External mammary artery (laterally)
 Anterior intercostal arteries
 Thoraco-acromial artery

Venous drainage Superficial venous plexus to subclavian, axillary and intercostal veins.

Lymphatic  70% Axillary nodes


drainage  Internal mammary chain
 Other lymphatic sites such as deep cervical and supraclavicular fossa
(later in disease)

Next question
Question 306 of 560

Which of the following muscles is supplied by the external laryngeal nerve?

Transverse arytenoid

Cricothyroid

Thyro-arytenoid

Posterior crico-arytenoid

Oblique arytenoid

The others are all supplied by the recurrent laryngeal nerve.


Please rate this question:

Discuss and give feedback


Next question

Larynx

The larynx lies in the anterior part of the neck at the levels of C3 to C6 vertebral bodies. The
laryngeal skeleton consists of a number of cartilagenous segments. Three of these are paired;
arytenoid, corniculate and cuneiform. Three are single; thyroid, cricoid and epiglottic. The cricoid
cartilage forms a complete ring (the only one to do so).
The laryngeal cavity extends from the laryngeal inlet to the level of the inferior border of the cricoid
cartilage.

Divisions of the laryngeal cavity


Laryngeal vestibule Superior to the vestibular folds

Laryngeal ventricle Lies between vestibular folds and superior to the vocal cords

Infraglottic cavity Extends from vocal cords to inferior border of the cricoid cartilage
The vocal folds (true vocal cords) control sound production. The apex of each fold projects medially
into the laryngeal cavity. Each vocal fold includes:

 Vocal ligament
 Vocalis muscle (most medial part of thyroarytenoid muscle)

The glottis is composed of the vocal folds, processes and rima glottidis. The rima glottidis is the
narrowest potential site within the larynx, as the vocal cords may be completely opposed, forming a
complete barrier.

Muscles of the larynx


Muscle Origin Insertion Innervation Action

Posterior Posterior aspect Muscular process Recurrent Abducts vocal fold


cricoarytenoid of lamina of of arytenoid Laryngeal
cricoid

Lateral Arch of cricoid Muscular process Recurrent Adducts vocal fold


cricoarytenoid of arytenoid laryngeal

Thyroarytenoid Posterior aspect Muscular process Recurrent Relaxes vocal fold


of thyroid of arytenoid laryngeal
cartilage

Transverse and Arytenoid Contralateral Recurrent Closure of


oblique cartilage arytenoid laryngeal intercartilagenous
arytenoids part of the rima
glottidis

Vocalis Depression Vocal ligament Recurrent Relaxes posterior


between lamina and vocal process laryngeal vocal ligament, tenses
of thyroid of arytenoid anterior part
cartilage cartilage

Cricothyroid Anterolateral Inferior margin External Tenses vocal fold


part of cricoid and horn of laryngeal
thyroid cartilage
Blood supply
Arterial supply is via the laryngeal arteries, branches of the superior and inferior thyroid arteries. The
superior laryngeal artery is closely related to the internal laryngeal nerve. The inferior laryngeal
artery is related to the inferior laryngeal nerve. Venous drainage is via superior and inferior laryngeal
veins, the former draining into the superior thyroid vein and the latter draining into the middle thyroid
vein, or thyroid venous plexus.

Lymphatic drainage
The vocal cords have no lymphatic drainage and this site acts as a lymphatic watershed.
Supraglottic part Upper deep cervical nodes

Subglottic part Prelaryngeal and pretracheal nodes and inferior deep cervical nodes

The aryepiglottic fold and vestibular folds have a dense plexus of lymphatics associated with them
and malignancies at these sites have a greater propensity for nodal metastasis.

Topography of the larynx

Image sourced from Wikipedia

Next question
Question 307 of 560

A 28 year old man has sustained a non salvageable testicular injury to his left testicle. The surgeon
decides to perform an orchidectomy and divides the left testicular artery. From which of the following
does this vessel originate?

Abdominal aorta

Internal iliac artery

Inferior epigastric artery

Inferior vesical artery

External iliac artery

The testicular artery is a branch of the abdominal aorta.


Please rate this question:

Discuss and give feedback


Next question

Scrotal and testicular anatomy

Spermatic cord
Formed by the vas deferens and is covered by the following structures:
Layer Origin

Internal spermatic fascia Transversalis fascia

Cremasteric fascia From the fascial coverings of internal oblique

External spermatic fascia External oblique aponeurosis

Contents of the cord


Vas deferens Transmits sperm and accessory gland secretions

Testicular artery Branch of abdominal aorta supplies testis and


epididymis

Artery of vas deferens Arises from inferior vesical artery

Cremasteric artery Arises from inferior epigastric artery

Pampiniform plexus Venous plexus, drains into right or left testicular vein

Sympathetic nerve fibres Lie on arteries, the parasympathetic fibres lie on the
vas

Genital branch of the genitofemoral Supplies cremaster


nerve

Lymphatic vessels Drain to lumbar and para-aortic nodes

Scrotum

 Composed of skin and closely attached dartos fascia.


 Arterial supply from the anterior and posterior scrotal arteries
 Lymphatic drainage to the inguinal lymph nodes
 Parietal layer of the tunica vaginalis is the innermost layer

Testes

 The testes are surrounded by the tunica vaginalis (closed peritoneal sac). The parietal layer
of the tunica vaginalis adjacent to the internal spermatic fascia.
 The testicular arteries arise from the aorta immediately inferiorly to the renal arteries.
 The pampiniform plexus drains into the testicular veins, the left drains into the left renal vein
and the right into the inferior vena cava.
 Lymphatic drainage is to the para-aortic nodes.

Next question
Question 308 of 560

During a carotid endarterectomy the internal carotid artery is cross clamped. Assuming that no shunt
is inserted, which of the following vessels will not have diminished or absent flow as a result?

Anterior cerebral artery

Ophthalmic artery

Middle cerebral artery

Maxillary artery

None of the above

Mnemonic for branches of the cerebral portion of the internal carotid artery 'Only Press Carotid
Arteries Momentarily'

Only = Opthalmic
Press = Posterior communicating
Carotid = Choroidal
Arteries = Anterior cerebral
Momentarily = Middle cerebral

The maxillary artery is a branch of the external carotid artery.


Please rate this question:

Discuss and give feedback


Next question

Internal carotid artery

The internal carotid artery is formed from the common carotid opposite the upper border of the
thyroid cartilage. It extends superiorly to enter the skull via the carotid canal. From the carotid canal
it then passes through the cavernous sinus, above which it divides into the anterior and middle
cerebral arteries.

Relations in the neck


Posterior  Longus capitis
 Pre-vertebral fascia
 Sympathetic chain
 Superior laryngeal nerve

Medially  External carotid (near origin)


 Wall of pharynx
 Ascending pharyngeal artery

Laterally  Internal jugular vein (moves posteriorly at entrance to skull)


 Vagus nerve (most posterolaterally)

Anteriorly  Sternocleidomastoid
 Lingual and facial veins
 Hypoglossal nerve

Relations in the carotid canal

 Internal carotid plexus


 Cochlea and middle ear cavity
 Trigeminal ganglion (superiorly)
 Leaves canal lies above the foramen lacerum

Path and relations in the cranial cavity


The artery bends sharply forwards in the cavernous sinus, the aducens nerve lies close to its
inferolateral aspect. The oculomotor, trochlear, opthalmic and, usually, the maxillary nerves lie in the
lateral wall of the sinus. Near the superior orbital fissure it turns posteriorly and passes postero-
medially to pierce the roof of the cavernous sinus inferior to the optic nerve. It then passes between
the optic and oculomotor nerves to terminate below the anterior perforated substance by dividing
into the anterior and middle cerebral arteries.

Branches

 Anterior and middle cerebral artery


 Ophthalmic artery
 Posterior communicating artery
 Anterior choroid artery
 Meningeal arteries
 Hypophyseal arteries

Image demonstrating the internal carotid artery and its relationship to the external carotid artery
Image sourced from Wikipedia

Next question
Question 309 of 560

A 72 year old lady with osteoporosis falls and sustains an intracapsular femoral neck fracture. The
fracture is completely displaced. Which of the following vessels is the main contributor to the arterial
supply of the femoral head?

Deep external pudendal artery

Superficial femoral artery

External iliac artery

Circumflex femoral arteries

Superficial external pudendal artery

Theme from 2010 Exam


The vessels which form the anastomoses around the femoral head are derived from the medial and
lateral circumflex femoral arteries. These are usually derived from the profunda femoris artery.
Please rate this question:

Discuss and give feedback


Next question

Hip joint

 Head of femur articulates with acetabulum of the pelvis


 Both covered by articular hyaline cartilage
 The acetabulum forms at the union of the ilium, pubis, and ischium
 The triradiate cartilage (Y-shaped growth plate) separates the pelvic bones
 The acetabulum holds the femoral head by the acetabular labrum
 Normal angle between femoral head and femoral shaft is 130o

Ligaments

 Transverse ligament: joints anterior and posterior ends of the articular cartilage
 Head of femur ligament (ligamentum teres): acetabular notch to the fovea. Contains arterial
supply to head of femur in children.
Image sourced from Wikipedia

Image sourced from Wikipedia

Extracapsular ligaments

 Iliofemoral ligament: inverted Y shape. Anterior iliac spine to the trochanteric line
 Pubofemoral ligament: acetabulum to lesser trochanter
 Ischiofemoral ligament: posterior support. Ischium to greater trochanter.

Blood supply
Medial circumflex femoral and lateral circumflex femoral arteries (Branches of profunda femoris).
Also from the inferior gluteal artery. These form an anastomosis and travel to up the femoral neck to
supply the head.
Next question
Question 310 of 560

A 21 year old man is hit with a hammer and sustains a depressed skull fracture at the vertex. Which
of the following sinuses is at risk in this injury?

Superior sagittal sinus

Inferior petrosal sinus

Transverse sinus

Inferior sagittal sinus

Straight sinus

Theme in September 2011 Exam

The superior sagittal sinus is at greatest risk in this pattern of injury. This sinus begins at the front of
the crista galli and courses backwards along the falx cerebri. It becomes continuous with the right
transverse sinus near the internal occipital protuberance.

Please rate this question:

Discuss and give feedback

Next question

Cranial venous sinuses

The cranial venous sinuses are located within the dura mater. They have no valves which is
important in the potential for spreading sepsis. They eventually drain into the internal jugular vein.

They are:
Superior sagittal sinus
Inferior sagittal sinus
Straight sinus
Transverse sinus
Sigmoid sinus
Confluence of sinuses
Occipital sinus
Cavernous sinus

Topography of cranial venous sinuses

Image sourced from Wikipedia

Next question
Question 311 of 560

A 44 year old man is stabbed in the back and the left kidney is injured. A haematoma forms, which of
the following fascial structures will contain the haematoma?

Waldeyers fascia

Sibsons fascia

Bucks fascia

Gerotas fascia

Denonvilliers fascia

Waldeyers fascia- Posterior ano-rectum


Sibsons fascia- Lung apex
Bucks fascia- Base of penis
Gerotas fascia- Surrounding kidney
Denonvilliers fascia- Between rectum and prostate

Please rate this question:

Discuss and give feedback

Next question

Renal anatomy

Each kidney is about 11cm long, 5cm wide and 3cm thick. They are located in a deep gutter
alongside the projecting vertebral bodies, on the anterior surface of psoas major. In most cases the
left kidney lies approximately 1.5cm higher than the right. The upper pole of both kidneys
approximates with the 11th rib (beware pneumothorax during nephrectomy). On the left hand side
the hilum is located at the L1 vertebral level and the right kidney at level L1-2. The lower border of
the kidneys is usually alongside L3.

The table below shows the anatomical relations of the kidneys:

Relations

Relations Right Kidney Left Kidney

Posterior Quadratus lumborum, diaphragm, psoas Quadratus lumborum, diaphragm, psoas


major, transversus abdominis major, transversus abdominis

Anterior Hepatic flexure of colon Stomach, Pancreatic tail

Superior Liver, adrenal gland Spleen, adrenal gland

Fascial covering
Each kidney and suprarenal gland is enclosed within a common layer of investing fascia, derived
from the transversalis fascia. It is divided into anterior and posterior layers (Gerotas fascia).

Renal structure
Kidneys are surrounded by an outer cortex and an inner medulla which usually contains between 6
and 10 pyramidal structures. The papilla marks the innermost apex of these. They terminate at the
renal pelvis, into the ureter.
Lying in a hollow within the kidney is the renal sinus. This contains:
1. Branches of the renal artery
2. Tributaries of the renal vein
3. Major and minor calyces's
4. Fat

Structures at the renal hilum


The renal vein lies most anteriorly, then renal artery (it is an end artery) and the ureter lies most
posterior.

Next question
Question 312 of 560

A baby is found to have a Klumpke's palsy post delivery. Which of the following is most likely to be
present?

Loss of flexors of the wrist

Weak elbow flexion

Pronation of the forearm

Adducted shoulder

Shoulder medially rotated

Features of Klumpkes Paralysis

 Claw hand (MCP joints extended and IP joints flexed)


 Loss of sensation over medial aspect of forearm and hand
 Horner's syndrome
 Loss of flexors of the wrist

A C8, T1 root lesion is called Klumpke's paralysis and is caused by delivery with the arm extended.
Please rate this question:

Discuss and give feedback


Next question

Brachial plexus

Origin Anterior rami of C5 to T1

Sections of the  Roots, trunks, divisions, cords, branches


plexus  Mnemonic:Real Teenagers Drink Cold Beer

Roots  Located in the posterior triangle


 Pass between scalenus anterior and medius

Trunks  Located posterior to middle third of clavicle


 Upper and middle trunks related superiorly to the subclavian artery
 Lower trunk passes over 1st rib posterior to the subclavian artery

Divisions Apex of axilla

Cords Related to axillary artery

Diagram illustrating the branches of the brachial plexus

Image sourced from Wikipedia

Cutaneous sensation of the upper limb


Image sourced from Wikipedia

Next question
Question 313 of 560

A 22 year old man undergoes a superficial parotidectomy for a pleomorphic adenoma. The operation
does not proceed well and a diathermy malfunction results in division of the buccal branch of the
facial nerve. Which of the following muscles will not demonstrate impaired function as a result?

Zygomaticus minor

Mentalis

Buccinator

Levator anguli oris

Risorius

Buccal branch supplies


Zygomaticus minor Elevates upper lip

Risorius Aids smile

Pulls corner of mouth backward and compresses cheek


Buccinator

Levator anguli oris Pulls angles of mouth upward and toward midline

Orbicularis Closes and tightens lips together

Nasalis Flares nostrils and compresses nostrils

Please rate this question:

Discuss and give feedback


Next question

Facial nerve

The facial nerve is the main nerve supplying the structures of the second embryonic branchial arch.
It is predominantly an efferent nerve to the muscles of facial expression, digastric muscle and also to
many glandular structures. It contains a few afferent fibres which originate in the cells of its genicular
ganglion and are concerned with taste.

Supply - 'face, ear, taste, tear'

 Face: muscles of facial expression


 Ear: nerve to stapedius
 Taste: supplies anterior two-thirds of tongue
 Tear: parasympathetic fibres to lacrimal glands, also salivary glands

Path
Subarachnoid path

 Origin: motor- pons, sensory- nervus intermedius


 Pass through the petrous temporal bone into the internal auditory meatus with the
vestibulocochlear nerve. Here they combine to become the facial nerve.

Facial canal path

 The canal passes superior to the vestibule of the inner ear


 At the medial aspect of the middle ear, it becomes wider and contains the geniculate
ganglion.

- 3 branches:
1. greater petrosal nerve
2. nerve to stapedius
3. chorda tympani

Stylomastoid foramen

 Passes through the stylomastoid foramen (tympanic cavity anterior and mastoid antrum
posteriorly)
 Posterior auricular nerve and branch to posterior belly of digastric and stylohyoid muscle

Face
Enters parotid gland and divides into 5 branches:
 Temporal branch
 Zygomatic branch
 Buccal branch
 Marginal mandibular branch
 Cervical branch

Next question
Question 314 of 560

At which of the following vertebral body levels does the common carotid artery typically bifurcate into
the external and internal carotid arteries?

C4

C2

C1

C6

C7

It terminates at the upper border of the thyroid cartilege, Which is usually located at C4.
Please rate this question:

Discuss and give feedback


Next question

Common carotid artery

The right common carotid artery arises at the bifurcation of the brachiocephalic trunk, the left
common carotid arises from the arch of the aorta. Both terminate at the level of the upper border of
the thyroid cartilage (the lower border of the third cervical vertebra) by dividing into the internal and
external carotid arteries.

Left common carotid artery


This vessel arises immediately to the left and slightly behind the origin of the brachiocephalic trunk.
Its thoracic portion is 2.5- 3.5 cm in length and runs superolaterally to the sternoclavicular joint.

In the thorax
The vessel is in contact, from below upwards, with the trachea, left recurrent laryngeal nerve, left
margin of the oesophagus. Anteriorly the left brachiocephalic vein runs across the artery, and the
cardiac branches from the left vagus descend in front of it. These structures together with the
thymus and the anterior margins of the left lung and pleura separate the artery from the manubrium.

In the neck
The artery runs superiorly deep to sternocleidomastoid and then enters the anterior triangle. At this
point it lies within the carotid sheath with the vagus nerve and the internal jugular vein. Posteriorly
the sympathetic trunk lies between the vessel and the prevertebral fascia. At the level of C7 the
vertebral artery and thoracic duct lie behind it. The anterior tubercle of C6 transverse process is
prominent and the artery can be compressed against this structure (it corresponds to the level of the
cricoid).
Anteriorly at C6 the omohyoid muscle passes superficial to the artery.
Within the carotid sheath the jugular vein lies lateral to the artery.

Right common carotid artery


The right common carotid arises from the brachiocephalic artery. The right common carotid artery
corresponds with the cervical portion of the left common carotid, except that there is no thoracic duct
on the right. The oesophagus is less closely related to the right carotid than the left.

Summary points about the carotid anatomy

Path
Passes behind the sternoclavicular joint (12% patients above this level) to the upper border of the
thyroid cartilage, to divide into the external (ECA) and internal carotid arteries (ICA).

Relations

 Level of 6th cervical vertebra crossed by omohyoid


 Then passes deep to the thyrohyoid, sternohyoid, sternomastoid muscles.
 Passes anterior to the carotid tubercle (transverse process 6th cervical vertebra)-NB
compression here stops haemorrhage.
 The inferior thyroid artery passes posterior to the common carotid artery.
 Then : Left common carotid artery crosses the thoracic duct, Right common carotid artery
crossed by recurrent laryngeal nerve
Image sourced from Wikipedia

Next question
Question 315 of 560

A man is stabbed in the chest to the right of the manubriosternal angle. Which structure is least likely
to be injured in this case?

Right pleura

The trachea

Right phrenic nerve

Right recurrent laryngeal nerve

Brachiocephalic vein

The right recurrent laryngeal nerve branches off the right vagus more proximally and arches
posteriorly round the subclavian artery. So of the structures given it is the least likely to be injured.
Please rate this question:

Discuss and give feedback


Next question

Mediastinum

Region between the pulmonary cavities.


It is covered by the mediastinal pleura. It does not contain the lungs.
It extends from the thoracic inlet superiorly to the diaphragm inferiorly.

Mediastinal regions

 Superior mediastinum (between manubriosternal angle and T4/5)


 Middle mediastinum
 Posterior mediastinum
 Anterior mediastinum

Region Contents
Region Contents

Superior mediastinum  Superior vena cava


 Brachiocephalic veins
 Arch of aorta
 Thoracic duct
 Trachea
 Oesophagus
 Thymus
 Vagus nerve
 Left recurrent laryngeal nerve
 Phrenic nerve

Anterior mediastinum  Thymic remnants


 Lymph nodes
 Fat

Middle mediastinum  Pericardium


 Heart
 Aortic root
 Arch of azygos vein
 Main bronchi

Posterior mediastinum  Oesophagus


 Thoracic aorta
 Azygos vein
 Thoracic duct
 Vagus nerve
 Sympathetic nerve trunks
 Splanchnic nerves

Next question
Question 316 of 560

An 18 year old man is stabbed in the neck and has to undergo repair of a laceration to the internal
carotid artery. Post operatively he is noted to have a Horners syndrome. Which of the following will
not be present?

Apparent enopthalmos

Loss of sweating on the entire ipsilateral side of the face

Constricted pupil

Mild ptosis

Normal sympathetic activity in the torso

The anhidrosis will be mild as this is a distal lesion and at worst only a very limited area of the
ipsilateral face will be anhidrotic.
Please rate this question:

Discuss and give feedback


Next question

Horners syndrome

Horners syndrome, clinical features:

 Ptosis
 Miosis
 Enopthalmos
 Anhydrosis

Primarily a disorder of the sympathetic nervous system. Extent of symptoms depends upon the
anatomical site of the lesion.

Proximal lesions occur along the hypothalamospinal tract

Distal lesions are usually post ganglionic e.g. at level of internal carotid artery or beyond.
Next question
Question 317 of 560

Which of the following types of epithelium lines the lumenal surface of the normal oesophagus?

Non keratinised stratified squamous epithelium

Ciliated columnar epithelium

Keratinised stratified squamous epithelium

Non ciliated columnar epithelium

None of the above

The oesphagus is lined by non keratinised stratified squamous epithelium. Changes to glandular
type epithelium occur as part of metaplastic processes in reflux.
Please rate this question:

Discuss and give feedback


Next question

Oesophagus

 25cm long
 Starts at C6 vertebra, pierces diaphragm at T10 and ends at T11
 Squamous epithelium

Constrictions of the oesophagus


Structure Distance from incisors

Cricoid cartilage 15cm

Arch of the Aorta 22.5cm


Left principal bronchus 27cm

Diaphragmatic hiatus 40cm

Relations
Anteriorly  Trachea to T4
 Recurrent laryngeal nerve
 Left bronchus, Left atrium
 Diaphragm

Posteriorly  Thoracic duct to left at T5


 Hemiazygos to the left T8
 Descending aorta
 First 2 intercostal branches of aorta

Left  Thoracic duct


 Left subclavian artery

Right  Azygos vein

Arterial, venous and lymphatic drainage of the oesophagus


Artery Vein Lymphatics Muscularis externa

Upper Inferior Inferior thyroid Deep Striated muscle


third thyroid cervical

Mid third Aortic Azygos branches Mediastinal Smooth & striated


branches muscle

Lower Left gastric Posterior mediastinal and Gastric Smooth muscle


third coeliac

Nerve supply
 Upper half is supplied by recurrent laryngeal nerve
 Lower half by oesophageal plexus (vagus)

Histology

 Mucosa :Non-keratinized stratified squamous epithelium


 Submucosa: glandular tissue
 Muscularis externa (muscularis): composition varies. See table
 Adventitia

Next question
Question 318 of 560

A 23 year old man is stabbed in the neck, in the region between the omohyoid and digastric
muscles, the injury is explored surgically. At operation a nerve injury is identified immediately
superior to the lingual artery as is branches off the external carotid artery. Which of the following is
the most likely result of this injury?

Paralysis of the ipsilateral side of the tongue

Abduction of the ipsilateral vocal cord

Winging of the scapula

Paralysis of the ipsilateral hemi diaphragm

Inability to abduct the shoulder

The hypoglossal nerve runs anterior to the external carotid, above the lingual arterial branch. If
damaged then ipsilateral paralysis of the genioglossus, hyoglossus and styloglossus muscles will
occur. If the patient is asked to protrude their tongue then it will tend to point to the affected side.
Please rate this question:

Discuss and give feedback


Next question

Anterior triangle of the neck

Boundaries

Anterior border of the Sternocleidomastoid


Lower border of mandible
Anterior midline

Sub triangles (divided by Digastric above and Omohyoid)

 Muscular triangle: Neck strap muscles


 Carotid triangle: Carotid sheath
 Submandibular Triangle (digastric)

Contents of the anterior triangle


Digastric triangle Submandibular gland
Submandibular nodes
Facial vessels
Hypoglossal nerve

Muscular triangle Strap muscles


External jugular vein

Carotid triangle Carotid sheath (Common carotid, vagus and internal jugular vein)
Ansa cervicalis

Nerve supply to digastric muscle

 Anterior: Mylohyoid nerve


 Posterior: Facial nerve

Image sourced from Wikipedia

Next question
Question 319 of 560

Which of the following structures is not directly related to the right adrenal gland?

Diaphragm posteriorly

Bare area of the liver anteriorly

Right renal vein

Inferior vena cava

Hepato-renal pouch

The right renal vein is very short and lies more inferiorly.

Please rate this question:

Discuss and give feedback

Next question

Adrenal gland anatomy

Anatomy

Location Superomedially to the upper pole of each kidney

Relationships of the right Diaphragm-Posteriorly, Kidney-Inferiorly, Vena Cava-Medially, Hepato-renal


adrenal pouch and bare area of the liver-Anteriorly

Relationships of the left Crus of the diaphragm-Postero- medially, Pancreas and splenic vessels-
adrenal Inferiorly, Lesser sac and stomach-Anteriorly

Superior adrenal arteries- from inferior phrenic artery, Middle adrenal


Arterial supply arteries - from aorta, Inferior adrenal arteries -from renal arteries

Venous drainage of the Via one central vein directly into the IVC
right adrenal

Venous drainage of the Via one central vein into the left renal vein
left adrenal

Next question
Question 320 of 560

With respect to the basilic vein, which statement is false?

Its deep anatomical location makes it unsuitable for use as an arteriovenous access site in
fistula surgery

It originates from the dorsal venous network on the hand

It travels up the medial aspect of the forearm

Halfway between the shoulder and the elbow it lies deep to muscle

It joins the brachial vein to form the axillary vein

It is used in arteriovenous fistula surgery during a procedure known as a basilic vein transposition.
Please rate this question:

Discuss and give feedback


Next question

Basilic vein

The basilic and cephalic veins both provide the main pathways of venous drainage for the arm and
hand. It is continuous with the palmar venous arch distally and the axillary vein proximally.

Path

 Originates on the medial side of the dorsal venous network of the hand, and passes up the
forearm and arm.
 Most of its course is superficial.
 Near the region anterior to the cubital fossa the vein joins the cephalic vein.
 Midway up the humerus the basilic vein passes deep under the muscles.
 At the lower border of the teres major muscle, the anterior and posterior circumflex humeral
veins feed into it.
 It is often joined by the medial brachial vein before draining into the axillary vein.

Next question
Question 321 of 560

Mobilisation of the left lobe of the liver will facilitate surgical access to which of the following?

Abdominal oesophagus

Duodenum

Right colic flexure

Right kidney

Pylorus of stomach

The fundus of the stomach is a posterior relation. The pylorus lies more inferolaterally. During a total
gastrectomy division of the ligaments holding the left lobe of the liver will facilitate access to the
proximal stomach and abdominal oesophagus. This manoeuvre is seldom beneficial during a distal
gastrectomy.
Please rate this question:

Discuss and give feedback


Next question

Liver

Structure of the liver


Right lobe  Supplied by right hepatic artery
 Contains Couinaud segments V to VIII (-/+Sg I)

Left lobe  Supplied by the left hepatic artery


 Contains Couinaud segments II to IV (+/- Sg1)

Quadrate lobe  Part of the right lobe anatomically, functionally is part of the left
 Couinaud segment IV
 Porta hepatis lies behind
 On the right lies the gallbladder fossa
 On the left lies the fossa for the umbilical vein

Caudate lobe  Supplied by both right and left hepatic arteries


 Couinaud segment I
 Lies behind the plane of the porta hepatis
 Anterior and lateral to the inferior vena cava
 Bile from the caudate lobe drains into both right and left hepatic ducts

Detailed knowledge of Couinaud segments is not required for MRCS

 Between the liver lobules are portal canals which contain the portal triad: Hepatic Artery,
Portal Vein, tributary of Bile Duct.

Relations of the liver


Anterior Postero inferiorly

Diaphragm Oesophagus

Xiphoid process Stomach

Duodenum

Hepatic flexure of colon

Right kidney

Gallbladder

Inferior vena cava

Porta hepatis
Location Postero inferior surface, it joins nearly at right angles with the left sagittal fossa, and
separates the caudate lobe behind from the quadrate lobe in front

Transmits  Common hepatic duct


 Hepatic artery
 Portal vein
 Sympathetic and parasympathetic nerve fibres
 Lymphatic drainage of the liver (and nodes)

Ligaments
Falciform ligament  2 layer fold peritoneum from the umbilicus to anterior liver surface
 Contains ligamentum teres (remnant umbilical vein)
 On superior liver surface it splits into the coronary and left
triangular ligaments

Ligamentum teres Joins the left branch of the portal vein in the porta hepatis

Ligamentum Remnant of ductus venosus


venosum

Arterial supply

 Hepatic artery

Venous

 Hepatic veins
 Portal vein

Nervous supply

 Sympathetic and parasympathetic trunks of coeliac plexus

Next question
Question 322 of 560

The following statements relating to the ankle joint are true except?

Three groups of ligaments provide mechanical stability

The sural nerve lies medial to the Achilles tendon at its point of insertion

Eversion of the foot occurs at the sub talar joint

The flexor hallucis longus tendon is the most posterior structure at the medial malleolus

The saphenous nerve crosses the ankle joint.

The sural nerve lies behind the distal fibula. Inversion and eversion are sub talar movements. The
structures passing behind the medial malleolus from anterior to posterior include: tibialis posterior,
flexor digitorum longus, posterior tibia vein, posterior tibial artery, nerve, flexor hallucis longus.
Please rate this question:

Discuss and give feedback


Next question

Ankle joint

The ankle joint is a synovial joint composed of the tibia and fibula superiorly and the talus inferiorly.

Ligaments of the ankle joint

 Deltoid ligament (medially)


 Lateral collateral ligament
 Talofibular ligaments (both anteriorly and posteriorly)

The calcaneofibular ligament is separate from the fibrous capsule of the joint. The two talofibular
ligaments are fused with it.

The components of the syndesmosis are

 Antero-inferior tibiofibular ligament


 Postero-inferior tibiofibular ligament
 Inferior transverse tibiofibular ligament
 Interosseous ligament

Movements at the ankle joint

 Plantar flexion (55 degrees)


 Dorsiflexion (35 degrees)
 Inversion and eversion movements occur at the level of the sub talar joint

Nerve supply
Branches of deep peroneal and tibial nerves.

References
Golano P et al. Anatomy of the ankle ligaments: a pictorial essay. Knee Surg Sports Traumatol
Arthrosc. 2010 May;18(5):557-69
Next question
Question 323 of 560

The oesophagus is constricted at the following levels apart from:

Cricoid cartilage

Arch of the aorta

Lower oesophageal sphincter

Left main stem bronchus

Diaphragmatic hiatus

The oesophagus is not constricted at the level of the lower oesophageal sphincter.
Please rate this question:

Discuss and give feedback


Next question

Oesophagus

 25cm long
 Starts at C6 vertebra, pierces diaphragm at T10 and ends at T11
 Squamous epithelium

Constrictions of the oesophagus


Structure Distance from incisors

Cricoid cartilage 15cm

Arch of the Aorta 22.5cm


Left principal bronchus 27cm

Diaphragmatic hiatus 40cm

Relations
Anteriorly  Trachea to T4
 Recurrent laryngeal nerve
 Left bronchus, Left atrium
 Diaphragm

Posteriorly  Thoracic duct to left at T5


 Hemiazygos to the left T8
 Descending aorta
 First 2 intercostal branches of aorta

Left  Thoracic duct


 Left subclavian artery

Right  Azygos vein

Arterial, venous and lymphatic drainage of the oesophagus


Artery Vein Lymphatics Muscularis externa

Upper Inferior Inferior thyroid Deep Striated muscle


third thyroid cervical

Mid third Aortic Azygos branches Mediastinal Smooth & striated


branches muscle

Lower Left gastric Posterior mediastinal and Gastric Smooth muscle


third coeliac

Nerve supply
 Upper half is supplied by recurrent laryngeal nerve
 Lower half by oesophageal plexus (vagus)

Histology

 Mucosa :Non-keratinized stratified squamous epithelium


 Submucosa: glandular tissue
 Muscularis externa (muscularis): composition varies. See table
 Adventitia

Next question
Question 324 of 560

A 19 year old man is playing rugby when he suddenly notices a severe pain at the posterolateral
aspect of his right thigh. Which of the following muscle groups is most likely to have been injured?

Semimembranosus

Semitendinosus

Long head of biceps femoris

Gastrocnemius

Soleus

Theme from April 2012 Exam


The biceps femoris is the laterally located hamstring muscle. The semitendinosus and
semimembranosus are located medially. Rupture of gastrocnemius and soleus may occur but is less
common.

Please rate this question:

Discuss and give feedback

Next question

Biceps femoris

The biceps femoris is one of the hamstring group of muscles located in the posterior upper thigh. It
has two heads.

Long head
Origin Ischial tuberosity

Insertion Fibular head

Action Knee flexion, lateral rotation tibia, extension hip

Innervation Tibial division of sciatic nerve (L5, S1, S2)

Arterial Profunda femoris artery, inferior gluteal artery, and the superior muscular branches of
supply popliteal artery

Image demonstrating the biceps femoris muscle, with the long head outlined

Image sourced from Wikipedia

Short head

Origin Lateral lip of linea aspera, lateral supracondylar ridge of femur


Insertion Fibular head

Action Knee flexion, lateral rotation tibia

Innervation Common peroneal division of sciatic nerve (L5, S1, S2)

Arterial Profunda femoris artery, inferior gluteal artery, and the superior muscular branches of
supply popliteal artery

Next question
Question 325 of 560

Which of the following is a branch of the third part of the axillary artery?

Superior thoracic

Lateral thoracic

Dorsal scapular

Thoracoacromial

Posterior circumflex humeral

The other branches include:

 Subscapular
 Anterior circumflex humeral

Please rate this question:

Discuss and give feedback


Next question

Axilla

Boundaries of the axilla


Medially Chest wall and Serratus anterior

Laterally Humeral head

Floor Subscapularis
Anterior aspect Lateral border of Pectoralis major

Fascia Clavipectoral fascia

Content:
Long thoracic nerve (of Derived from C5-C7 and passes behind the brachial plexus to enter
Bell) the axilla. It lies on the medial chest wall and supplies serratus
anterior. Its location puts it at risk during axillary surgery and
damage will lead to winging of the scapula.

Thoracodorsal nerve and Innervate and vascularise latissimus dorsi.


thoracodorsal trunk

Axillary vein Lies at the apex of the axilla, it is the continuation of the basilic vein.
Becomes the subclavian vein at the outer border of the first rib.

Intercostobrachial Traverse the axillary lymph nodes and are often divided during
nerves axillary surgery. They provide cutaneous sensation to the axillary
skin.

Lymph nodes The axilla is the main site of lymphatic drainage for the breast.

Next question
Question 326 of 560

Which of the following structures separates the intervertebral disks from the spinal cord?

Anterior longitudinal ligament

Posterior longitudinal ligament

Supraspinous ligament

Interspinous ligament

Ligamentum flavum

The posterior longitudinal ligament overlies the posterior aspect of the vertebral bodies. It also
overlies the posterior aspect of the intervertebral disks.
Please rate this question:

Discuss and give feedback


Next question

Intervertebral discs

 Consist of an outer annulus fibrosus and an inner nucleus pulposus.


 The anulus fibrosus consists of several layers of fibrocartilage.
 The nucleus pulposus contains loose fibres suspended in a mucoprotein gel with the
consistency of jelly. The nucleus of the disc acts as a shock absorber.
 Pressure on the disc causes posterior protrusion of the nucleus pulposus. Most commonly in
the lumbrosacral and lower cervical areas.
 The discs are separated by hyaline cartilage.
 There is one disc between each pair of vertebrae, except for C1/2 and the sacrococcygeal
vertebrae.

Next question
Question 327 of 560

At what level does the aorta bifurcate into the left and right common iliac arteries?

L1

L2

L3

L4

L5

Theme from September 2013 Exam


The aorta typically bifurcates at L4. This level is usually fairly constant and is often tested in the
exam.
Please rate this question:

Discuss and give feedback


Next question

Levels

Transpyloric plane
Level of the body of L1

 Pylorus stomach
 Left kidney hilum (L1- left one!)
 Fundus of the gallbladder
 Neck of pancreas
 Duodenojejunal flexure
 Superior mesenteric artery
 Portal vein
 Left and right colic flexure
 Root of the transverse mesocolon
 2nd part of the duodenum
 Upper part of conus medullaris
 Spleen
Can be identified by asking the supine patient to sit up without using their arms. The plane is located
where the lateral border of the rectus muscle crosses the costal margin.

Anatomical planes
Subcostal plane Lowest margin of 10th costal cartilage

Intercristal plane Level of body L4 (highest point of iliac crest)

Intertubercular plane Level of body L5

Common level landmarks


Inferior mesenteric artery L3

Bifurcation of aorta into common iliac arteries L4

Formation of IVC L5 (union of common iliac veins)

Diaphragm apertures  Vena cava T8


 Oesophagus T10
 Aortic hiatus T12

Next question
Question 328 of 560

A 23 year old man is due to undergo a mitral valve repair for mitral regurgitation. Which of the
following is a feature of the mitral valve?

Its closure is marked by the first heart sound

It has two anterior cusps

The chordae tendinae attach to the anterior cusps only

The chordae tendinae anchor the valve directly to the wall of the left ventricle

It is best auscultated in the left third interspace

The mitral valve is best auscultated over the cardiac apex, where its closure marks the first heart
sound. It has only two cusps. These are attached to chordae tendinae which themselves are linked
to the wall of the ventricle by the papillary muscles.
Please rate this question:

Discuss and give feedback


Next question

Heart anatomy

The walls of each cardiac chamber comprise:

 Epicardium
 Myocardium
 Endocardium

Cardiac muscle is attached to the cardiac fibrous skeleton.

Relations
The heart and roots of the great vessels within the pericardial sac are related anteriorly to the
sternum, medial ends of the 3rd to 5th ribs on the left and their associated costal cartilages. The
heart and pericardial sac are situated obliquely two thirds to the left and one third to the right of the
median plane.
The pulmonary valve lies at the level of the left third costal cartilage.
The mitral valve lies at the level of the fourth costal cartilage.

Coronary sinus
This lies in the posterior part of the coronary groove and receives blood from the cardiac veins. The
great cardiac vein lies at its left and the middle and small cardiac veins lie on its right. The smallest
cardiac vein (anterior cardiac vein) drains into the right atrium directly.

Aortic sinus
Right coronary artery arises from the right aortic sinus, the left is derived from the left aortic sinus,
which lies posteriorly.

Right and left ventricles

Structure Left Ventricle

A-V Valve Mitral (double leaflet)

Walls Twice as thick as right

Trabeculae carnae Much thicker and more numerous

Right coronary artery


The RCA supplies:

 Right atrium
 Diaphragmatic part of the left ventricle
 Usually the posterior third of the interventricular septum
 The sino atrial node (60% cases)
 The atrio ventricular node (80% cases)

Left coronary artery


The LCA supplies:

 Left atrium
 Most of left ventricle
 Part of the right ventricle
 Anterior two thirds of the inter ventricular septum
 The sino atrial node (remaining 40% cases)

Innervation of the heart


Autonomic nerve fibres from the superficial and deep cardiac plexus. These lie anterior to the
bifurcation of the trachea, posterior to the ascending aorta and superior to the bifurcation of the
pulmonary trunk. The parasympathetic supply to the heart is from presynaptic fibres of the vagus
nerves.

Valves of the heart


Mitral valve Aortic valve Pulmonary valve Tricuspid valve

2 cusps 3 cusps 3 cusps 3 cusps

First heart sound Second heart Second heart First heart sound
sound sound

1 anterior cusp 2 anterior cusps 2 anterior cusps 2 anterior cusps

Attached to chordae No chordae No chordae Attached to chordae


tendinae tendinae

Next question
Question 329-331 of 560

Theme: Nerve lesions

A. Intercostobrachial
B. Median
C. Axillary
D. Radial
E. Ulnar
F. Musculocutaneous
G. Brachial plexus upper cord
H. Brachial plexus lower cord

Please select the most likely nerve injury for the scenarios given. Each option may be used once,
more than once or not at all.

329. A 23 year old rugby player sustains a Smiths Fracture. On examination opposition of the
thumb is markedly weakened.

You answered Intercostobrachial

The correct answer is Median

This high velocity injury can often produce significant angulation and displacement. Both
of these may impair the function of the median nerve with loss of function of the muscles
of the thenar eminence

330. A 45 year old lady recovering from a mastectomy and axillary node clearance notices that
sensation in her armpit is impaired.

Intercostobrachial

The intercostobrachial nerves are frequently injured during axillary dissection. These
nerves traverse the axilla and supply cutaneous sensation.

331. An 8 year old boy falls onto an outstretched hand and sustains a supracondylar fracture.
In addition to a weak radial pulse the child is noted to have loss of pronation of the
affected hand.

You answered Intercostobrachial

The correct answer is Median


This is a common injury in children. In this case the angulation and displacement have
resulted in median nerve injury.

Please rate this question:

Discuss and give feedback


Next question

Brachial plexus

Origin Anterior rami of C5 to T1

Sections of the  Roots, trunks, divisions, cords, branches


plexus  Mnemonic:Real Teenagers Drink Cold Beer

Roots  Located in the posterior triangle


 Pass between scalenus anterior and medius

Trunks  Located posterior to middle third of clavicle


 Upper and middle trunks related superiorly to the subclavian artery
 Lower trunk passes over 1st rib posterior to the subclavian artery

Divisions Apex of axilla

Cords Related to axillary artery

Diagram illustrating the branches of the brachial plexus


Image sourced from Wikipedia

Cutaneous sensation of the upper limb

Image sourced from Wikipedia

Next question
Question 332 of 560

A 23 year old lady with sialolithiasis of the submandibular gland is undergoing excision of the gland.
Which of the following nerves is at risk as the duct is mobilised?

Lingual nerve

Buccal nerve

Facial nerve

Glossopharyngeal

Vagus

The lingual nerve wraps around Whartons duct. The lingual nerve provides sensory supply to the
anterior 2/3 of the tongue.
Please rate this question:

Discuss and give feedback


Next question

Submandibular gland

Relations of the submandibular gland


Superficial Platysma, deep fascia and mandible
Submandibular lymph nodes
Facial vein (facial artery near mandible)
Marginal mandibular nerve
Cervical branch of the facial nerve

Deep Facial artery (inferior to the mandible)


Mylohoid muscle
Sub mandibular duct
Hyoglossus muscle
Lingual nerve
Submandibular ganglion
Hypoglossal nerve

Submandibular duct (Wharton's duct)

 Opens lateral to the lingual frenulum on the anterior floor of mouth.


 5 cm length
 Lingual nerve wraps around Wharton's duct. As the duct passes forwards it crosses medial
to the nerve to lie above it and then crosses back, lateral to it, to reach a position below the
nerve.

Innervation

 Sympathetic innervation- Derived from superior cervical ganglion


 Parasympathetic innervation- Submandibular ganglion via lingual nerve

Arterial supply
Branch of the facial artery. The facial artery passes through the gland to groove its deep surface. It
then emerges onto the face by passing between the gland and the mandible.

Venous drainage
Anterior facial vein (lies deep to the Marginal Mandibular nerve)

Lymphatic drainage
Deep cervical and jugular chains of nodes
Next question
Question 333 of 560

Which of the following is true in connection with the phrenic nerves?

They both lie anterior to the hilum of the lungs

They are derived from spinal roots C 2,3,4

They pierce the diaphragm at the level of T7

They consist of motor fibres only

None of the above

C3,4,5
Keeps the diaphragm alive!

They both lie anterior to the hilum of the lung. The phrenic nerves have both motor and sensory
functions. For this reason sub diaphragmatic pathology may cause referred pain to the shoulder.
Please rate this question:

Discuss and give feedback


Next question

Phrenic nerve

Origin

 C3,4,5

Supplies

 Diaphragm, sensation central diaphragm and pericardium

Path
 The phrenic nerve passes with the internal jugular vein across scalenus anterior. It passes
deep to prevertebral fascia of deep cervical fascia.
 Left: crosses anterior to the 1st part of the subclavian artery.
 Right: Anterior to scalenus anterior and crosses anterior to the 2nd part of the subclavian
artery.
 On both sides, the phrenic nerve runs posterior to the subclavian vein and posterior to the
internal thoracic artery as it enters the thorax.

Right phrenic nerve

 In the superior mediastinum: anterior to right vagus and laterally to superior vena cava
 Middle mediastinum: right of pericardium
 It passes over the right atrium to exit the diaphragm at T8

Left phrenic nerve

 Passes lateral to the left subclavian artery, aortic arch and left ventricle
 Passes anterior to the root of the lung
 Pierces the diaphragm alone

Image showing the passage of the phrenic nerve in the neck

Image sourced from Wikipedia

Next question
Question 334 of 560

A 32 year old man presents with an inguinal hernia and undergoes an open surgical repair. The
surgeons decide to place a mesh on the posterior wall of the inguinal canal to complete the repair,
which of the following structures will lie posterior to the mesh?

Transversalis fascia

External oblique

Rectus abdominis

Obturator nerve

None of the above

Inguinal canal walls: 'MALT: 2M, 2A, 2L, 2T':

Starting from superior, moving around in order to posterior:


Superior wall (roof): 2 Muscles:Internal oblique, transversus abdominis
Anterior wall: 2 Aponeuroses: Aponeurosis of external oblique, Aponeurosis of internal oblique
Lower wall (floor): 2 Ligaments: Inguinal Ligament, Lacunar Ligament Posterior wall: 2Ts:
Transversalis fascia, Conjoint Tendon

This is actually quite a straightforward question. It is simply asking for the structure that forms the
posterior wall of the inguinal canal. This is composed of the transversalis fascia, the conjoint tendon
and more laterally the deep inguinal ring.
Please rate this question:

Discuss and give feedback


Next question

Inguinal canal

Location

 Above the inguinal ligament


 The inguinal canal is 4cm long
 The superficial ring is located anterior to the pubic tubercle
 The deep ring is located approximately 1.5-2cm above the half way point between the
anterior superior iliac spine and the pubic tubercle

Boundaries of the inguinal canal


Floor  External oblique aponeurosis
 Inguinal ligament
 Lacunar ligament

Roof  Internal oblique


 Transversus abdominis

Anterior wall External oblique aponeurosis

Posterior wall  Transversalis fascia


 Conjoint tendon

Laterally  Internal ring


 Fibres of internal oblique

Medially  External ring


 Conjoint tendon

Contents
Males Spermatic cord and ilioinguinal As it passes through the canal the spermatic cord
nerve has 3 coverings:

 External spermatic fascia from external


oblique aponeurosis
 Cremasteric fascia
 Internal spermatic fascia

Females Round ligament of uterus and


ilioinguinal nerve

Related anatomy of the inguinal region


The boundaries of Hesselbachs triangle are commonly tested and illustrated below:
Image sourced from Wikipedia

The image below demonstrates the close relationship of the vessels to the lower limb with the
inguinal canal. A fact to be borne in mind when repairing hernial defects in this region.

Image sourced from Wikipedia

Next question
Question 335 of 560

A 22 year old man is involved in a fight and is stabbed in the posterior aspect of his right leg. The
knife passes into the popliteal fossa. He sustains an injury to his tibial nerve. Which muscle is least
likely to be compromised as a result?

Tibialis posterior

Flexor hallucis longus

Flexor digitorum brevis

Soleus

Peroneus tertius

Peroneus tertius is innervated by the deep peroneal nerve.


Please rate this question:

Discuss and give feedback


Next question

Tibial nerve

Begins at the upper border of the popliteal fossa and is a branch of the sciatic nerve.

 Root values: L4, L5, S1, S2, S3

Muscles innervated

 Popliteus
 Gastrocnemius
 Soleus
 Plantaris
 Tibialis posterior
 Flexor hallucis longus
 Flexor digitorum brevis
Terminates by dividing into the medial and lateral plantar nerves.
Next question
Question 336 of 560

Which of the following overlies the outer muscular layer of the intrathoracic oesophagus?

Serosa

Meissners plexus

Auerbach's plexus

Loose connective tissue

None of the above

The oesophagus has no serosal covering and hence holds sutures poorly. The Auerbach's and
Meissner's nerve plexuses lie in between the longitudinal and circular muscle layers and
submucosally. The sub mucosal location of the Meissner's nerve plexus facilitates its sensory role.
Please rate this question:

Discuss and give feedback


Next question

Oesophagus

 25cm long
 Starts at C6 vertebra, pierces diaphragm at T10 and ends at T11
 Squamous epithelium

Constrictions of the oesophagus


Structure Distance from incisors

Cricoid cartilage 15cm

Arch of the Aorta 22.5cm


Left principal bronchus 27cm

Diaphragmatic hiatus 40cm

Relations
Anteriorly  Trachea to T4
 Recurrent laryngeal nerve
 Left bronchus, Left atrium
 Diaphragm

Posteriorly  Thoracic duct to left at T5


 Hemiazygos to the left T8
 Descending aorta
 First 2 intercostal branches of aorta

Left  Thoracic duct


 Left subclavian artery

Right  Azygos vein

Arterial, venous and lymphatic drainage of the oesophagus


Artery Vein Lymphatics Muscularis externa

Upper Inferior Inferior thyroid Deep Striated muscle


third thyroid cervical

Mid third Aortic Azygos branches Mediastinal Smooth & striated


branches muscle

Lower Left gastric Posterior mediastinal and Gastric Smooth muscle


third coeliac

Nerve supply
 Upper half is supplied by recurrent laryngeal nerve
 Lower half by oesophageal plexus (vagus)

Histology

 Mucosa :Non-keratinized stratified squamous epithelium


 Submucosa: glandular tissue
 Muscularis externa (muscularis): composition varies. See table
 Adventitia

Next question
Question 337 of 560

Which nerve lies medially on the thyroid gland, in the groove between the oesophagus and trachea?

Vagus nerve

External laryngeal nerve

Recurrent laryngeal nerve

Ansa cervicalis

Phrenic nerve

The recurrent laryngeal nerve may be injured at this site during ligation of the inferior thyroid artery.
Please rate this question:

Discuss and give feedback


Next question

Thyroid gland

 Right and left lobes connected by isthmus


 Surrounded by sheath from pretracheal layer of deep fascia
 Apex: Lamina of thyroid cartilage
 Base: 4th-5th tracheal ring
 Pyramidal lobe: from isthmus
 May be attached to foramen caecum at the base of the tongue

Relations
Anteromedially  Sternothyroid
 Superior belly of omohyoid
 Sternohyoid
 Anterior aspect of sternocleidomastoid

Posterolaterally Carotid sheath


Medially  Larynx
 Trachea
 Pharynx
 Oesophagus
 Cricothyroid muscle
 External laryngeal nerve (near superior thyroid artery)
 Recurrent laryngeal nerve (near inferior thyroid artery)

Posterior  Parathyroid glands


 Anastomosis of superior and inferior thyroid arteries

Isthmus  Anteriorly: Sternothyroids, sternohyoids, anterior jugular veins


 Posteriorly: 2nd, 3rd, 4th tracheal rings (attached via Ligament of
Berry)

Blood Supply
Arterial  Superior thyroid artery (1st branch of external carotid)
 Inferior thyroid artery (from thyrocervical trunk)
 Thyroidea ima (in 10% of population -from brachiocephalic artery or aorta)

Venous  Superior and middle thyroid veins - into the IJV


 Inferior thyroid vein - into the brachiocephalic veins

Next question
Question 338 of 560

At which of the following spinal levels does the oesophagus pass through the diaphragm into the
abdominal cavity?

L2

L1

T10

T5

T12

The oesophagus passes into the abdomen at T10.


Please rate this question:

Discuss and give feedback


Next question

Oesophagus

 25cm long
 Starts at C6 vertebra, pierces diaphragm at T10 and ends at T11
 Squamous epithelium

Constrictions of the oesophagus


Structure Distance from incisors

Cricoid cartilage 15cm

Arch of the Aorta 22.5cm


Left principal bronchus 27cm

Diaphragmatic hiatus 40cm

Relations
Anteriorly  Trachea to T4
 Recurrent laryngeal nerve
 Left bronchus, Left atrium
 Diaphragm

Posteriorly  Thoracic duct to left at T5


 Hemiazygos to the left T8
 Descending aorta
 First 2 intercostal branches of aorta

Left  Thoracic duct


 Left subclavian artery

Right  Azygos vein

Arterial, venous and lymphatic drainage of the oesophagus


Artery Vein Lymphatics Muscularis externa

Upper Inferior Inferior thyroid Deep Striated muscle


third thyroid cervical

Mid third Aortic Azygos branches Mediastinal Smooth & striated


branches muscle

Lower Left gastric Posterior mediastinal and Gastric Smooth muscle


third coeliac

Nerve supply
 Upper half is supplied by recurrent laryngeal nerve
 Lower half by oesophageal plexus (vagus)

Histology

 Mucosa :Non-keratinized stratified squamous epithelium


 Submucosa: glandular tissue
 Muscularis externa (muscularis): composition varies. See table
 Adventitia

Next question
Question 339 of 560

Which of the following nerve roots contribute nerve fibres to the ansa cervicalis?

C1 only

C1, C2 and C3

C2, C3 and C6

C2, C4 and C5

C4, C5 and C6

Ansa cervicalis muscles:

GHost THought SOmeone Stupid Shot Irene

GenioHyoid
ThyroidHyoid
Superior Omohyoid
SternoThyroid
SternoHyoid
Inferior Omohyoid

The ansa cervicalis is composed of a superior and inferior root, derived from C1, C2 and C3. The
superior root arises where the nerve crosses the internal carotid artery. It descends anterior to the
carotid sheath in the anterior triangle. It is joined in the region of the mid neck by the inferior root.
The inferior root may pass either superficially or deep to the internal jugular vein.

Please rate this question:

Discuss and give feedback


Next question

Ansa cervicalis

Superior Branch of C1 anterolateral to carotid sheath


root

Inferior root Derived from C2 and C3 roots, passes posterolateral to the internal jugular vein (may lie
either deep or superficial to it)

Innervation Sternohyoid
Sternothyroid
Omohyoid

The ansa cervicalis lies anterior to the carotid sheath. The nerve supply to the inferior strap muscles
enters at their inferior aspect. Therefore when dividing these muscles to expose a large goitre, the
muscles should be divided in their upper half.
Image sourced from Wikipedia

Next question
Question 340 of 560

The anterior interosseous nerve is a branch of which of the following?

Ulnar nerve

Superficial branch of the radial nerve

Axillary nerve

Deep branch of the radial nerve

Median nerve

Please rate this question:

Discuss and give feedback


Next question

Anterior interosseous nerve

The anterior interosseous nerve (volar interosseous nerve) is a branch of the median nerve that
supplies the deep muscles on the front of the forearm, except the ulnar half of the flexor digitorum
profundus.

It accompanies the anterior interosseous artery along the anterior of the interosseous membrane of
the forearm, in the interval between the flexor pollicis longus and flexor digitorum profundus,
supplying the whole of the former and (most commonly) the radial half of the latter, and ending
below in the pronator quadratus and wrist joint.

Innervation
The anterior interosseous nerve classically innervates 2.5 muscles:

 Flexor pollicis longus


 Pronator quadratus
 The radial half of flexor digitorum profundus (the lateral two out of the four tendons).

These muscles are in the deep level of the anterior compartment of the forearm.
Question 341 of 560

At which of the following anatomical locations does the common peroneal nerve bifurcate into the
superficial and deep peroneal nerves?

Immediately anterior to the linea aspera

At the lateral aspect of the neck of the fibula

Within the substance of tibialis anterior muscle

At the inferomedial aspect of the popliteal fossa

Under the medial head of gastrocnemius

The common peroneal nerve bifurcates at the neck of the fibula (where it is most likely to be injured).

Please rate this question:

Discuss and give feedback

Next question

Common peroneal nerve

Derived from the dorsal divisions of the sacral plexus (L4, L5, S1 and S2).

This nerve supplies the skin and fascia of the anterolateral surface of the leg and the dorsum of the
foot. It also innervates the muscles of the anterior and peroneal compartments of the leg, extensor
digitorum brevis as well as the knee, ankle and foot joints.

It is laterally placed within the sciatic nerve. From the bifurcation of the sciatic nerve it passes
inferolaterally in the lateral and proximal part of the popliteal fossa, under the cover of biceps femoris
and its tendon. To reach the posterior aspect of the fibular head. It ends by dividing into the deep
and superficial peroneal nerves at the point where it winds around the lateral surface of the neck of
the fibula in the body of peroneus longus, approximately 2cm distal to the apex of the head of the
fibula. It is palpable posterior to the head of the fibula.

Branches

In the thigh Nerve to the short head of biceps


Articular branch (knee)

In the popliteal fossa Lateral cutaneous nerve of the calf

Neck of fibula Superficial and deep peroneal nerves

Next question
Question 342 of 560

A 48 year old motor cyclist sustains a complex lower limb fracture in a motor accident. For a time the
popliteal artery is occluded and eventually repaired. Subsequently he develops a compartment
syndrome and the anterior and superficial posterior compartments of the lower leg are
decompressed. Unfortunately, the operating surgeon neglects to decompress the deep posterior
compartment. Which of the following muscles is least likely to be affected as a result?

Flexor digitorum longus

Plantaris

Tibialis posterior

Flexor hallucis longus

None of the above

Muscles of the deep posterior compartment:

 Tibialis posterior
 Flexor hallucis longus
 Flexor digitorum longus
 Popliteus

The plantaris muscle lies within the superficial posterior compartment of the lower leg.
Please rate this question:

Discuss and give feedback


Next question

Lower limb- Muscular compartments

Anterior compartment
Muscle Nerve Action
Muscle Nerve Action

Tibialis anterior Deep peroneal Dorsiflexes ankle joint, inverts foot


nerve

Extensor digitorum Deep peroneal Extends lateral four toes, dorsiflexes ankle
longus nerve joint

Peroneus tertius Deep peroneal Dorsiflexes ankle, everts foot


nerve

Extensor hallucis longus Deep peroneal Dorsiflexes ankle joint, extends big toe
nerve

Peroneal compartment
Muscle Nerve Action

Peroneus longus Superficial peroneal nerve Everts foot, assists in plantar flexion

Peroneus brevis Superficial peroneal nerve Plantar flexes the ankle joint

Superficial posterior compartment


<muscle< b="" style="box-sizing: border-
box;"></muscle<> Nerve Action

Gastrocnemius Tibial Plantar flexes the foot, may also


nerve flex the knee

Soleus Tibial Plantar flexor


nerve

Deep posterior compartment


Muscle Nerve Action

Flexor digitorum longus Tibial Flexes the lateral four toes

Flexor hallucis longus Tibial Flexes the great toe

Tibialis posterior Tibial Plantar flexor, inverts the foot

Next question
Question 343 of 560

A 23 year old lady is undergoing a trendelenberg procedure for varicose veins. During the dissection
of the saphenofemoral junction, which of the structures listed below is most liable to injury?

Superficial circumflex iliac artery

Superficial circumflex iliac vein

Femoral artery

Femoral nerve

Deep external pudendal artery

Theme from September 2011 exam

The deep external pudendal artery runs under the long saphenous vein close to its origin and may
be injured. It is at greatest risk of injury during the flush ligation of the saphenofemoral junction.
Provided an injury is identified and vessel ligated, injury is seldom associated with any serious
adverse sequelae.
Please rate this question:

Discuss and give feedback


Next question

Saphenous vein

Long saphenous vein


This vein may be harvested for bypass surgery, or removed as treatment for varicose veins with
saphenofemoral junction incompetence.

 Originates at the 1st digit where the dorsal vein merges with the dorsal venous arch of the
foot
 Passes anterior to the medial malleolus and runs up the medial side of the leg
 At the knee, it runs over the posterior border of the medial epicondyle of the femur bone
 Then passes laterally to lie on the anterior surface of the thigh before entering an opening in
the fascia lata called the saphenous opening
 It joins with the femoral vein in the region of the femoral triangle at the saphenofemoral
junction

Tributaries

 Medial marginal
 Superficial epigastric
 Superficial iliac circumflex
 Superficial external pudendal veins

Short saphenous vein

 Originates at the 5th digit where the dorsal vein merges with the dorsal venous arch of the
foot, which attaches to the great saphenous vein.
 It passes around the lateral aspect of the foot (inferior and posterior to the lateral malleolus)
and runs along the posterior aspect of the leg (with the sural nerve)
 Passes between the heads of the gastrocnemius muscle, and drains into the popliteal vein,
approximately at or above the level of the knee joint.

Next question
Question 344 of 560

Considering the pituitary gland, which of the following is false?

The anterior pituitary secretes thyroid stimulating hormone

The anterior pituitary develops from Rathkes pouch

Patients with craniopharyngioma may develop bi temporal hemianopia

The pituitary is in direct contact with the optic chiasm

The posterior pituitary secretes oxytocin via a positive feedback loop

Although the optic chiasm is closely related to the pituitary, and craniopharyngiomas may compress
this structure leading to bitemporal hemianopia, it is separated from the chiasm itself by a dural fold.
Please rate this question:

Discuss and give feedback


Next question

Pituitary Gland

The pituitary gland is located within the sella turcica within the sphenoid bone in the middle cranial
fossa. It is covered by a dural fold and weighs around 0.5g. It is attached to the hypothalamus by the
infundibulum. The anterior pituitary receives hormonal stimuli from the hypothalamus by way of the
hypothalamo-pituitary portal system. It develops from a depression in the wall of the pharynx
(Rathkes pouch).

Anterior pituitary hormones

 Growth hormone
 Thyroid stimulating hormone
 ACTH
 Prolactin
 LH and FSH
 Melanocyte releasing hormone
Posterior pituitary hormones

 Oxytocin
 Anti diuretic hormone

Next question
Question 345 of 560

A 24 year old man is involved in a fight and his face is cut with a knife. The wound lies immediately
anterior to the tragus of the ear and extends anteriorly. The wound is surgically explored and the
laceration is found to be mainly superficial. It extends slightly more deeply immediately inferior to the
main trunk of the facial nerve. Bleeding is observed, from which of the following is it most likely to
originate?

External carotid artery

Retromandibular vein

Occipital artery

Maxillary artery

Ascending pharyngeal artery

The retromandibular vein lies slightly more deeply than the facial nerve in the parotid gland. It is
formed from the maxillary and superficial temporal vein.
Please rate this question:

Discuss and give feedback


Next question

Retromandibular vein

 Formed by a union of the maxillary vein and superficial temporal vein


 It descends through the parotid gland and bifurcates within it
 The anterior division passes forwards to join the facial vein, the posterior division is one of
the tributaries of the external jugular vein

Next question
Question 346 of 560

A 52 year female post hysterectomy attends clinic. She reports pain and reduced sensation over the
medial aspect of her thigh. Clinically thigh adduction is weak. What is the most likely nerve injury?

Obturator nerve

Sciatic nerve

Femoral nerve

L3 cord compression

Deep peroneal nerve

The obturator nerve supplies sensation to the medial aspect of the thigh and causes adduction and
internal rotation of the thigh.
Injury occurs during pelvic or abdominal surgery.
L3 cord compression is unlikely.
Please rate this question:

Discuss and give feedback


Next question

Obturator nerve

The obturator nerve arises from L2, L3 and L4 by branches from the ventral divisions of each of
these nerve roots. L3 forms the main contribution and the second lumbar branch is occasionally
absent. These branches unite in the substance of psoas major, descending vertically in its posterior
part to emerge from its medial border at the lateral margin of the sacrum. It then crosses the
sacroiliac joint to enter the lesser pelvis, it descends on obturator internus to enter the obturator
groove. In the lesser pelvis the nerve lies lateral to the internal iliac vessels and ureter, and is joined
by the obturator vessels lateral to the ovary or ductus deferens.

Supplies

 Medial compartment of thigh


 Muscles supplied: external obturator, adductor longus, adductor brevis, adductor magnus
(not the lower part-sciatic nerve), gracilis
 The cutaneous branch is often absent. When present, it passes between gracilis and
adductor longus near the middle part of the thigh, and supplies the skin and fascia of the
distal two thirds of the medial aspect.

Obturator canal

 Connects the pelvis and thigh: contains the obturator artery, vein, nerve which divides into
anterior and posterior branches.

Cadaveric cross section demonstrating relationships of the obturator nerve

Image sourced from Wikipedia

Next question
Question 347 of 560

An ENT surgeon is performing a radical neck dissection. She wishes to fully expose the external
carotid artery. To do so she inserts a self retaining retractor close to its origin. Which of the following
structures lies posterolaterally to the external carotid at this point?

Superior thyroid artery

Internal carotid artery

Lingual artery

Facial artery

None of the above

The internal carotid artery lies posterolaterally to the external carotid artery at it's origin from the
common carotid. The superior thyroid, lingual and facial arteries all arise from its anterior surface.

Please rate this question:

Discuss and give feedback

Next question

External carotid artery

The external carotid commences immediately lateral to the pharyngeal side wall. It ascends and lies
anterior to the internal carotid and posterior to the posterior belly of digastric and stylohyoid. More
inferiorly it is covered by sternocleidomastoid, passed by hypoglossal nerves, lingual and facial
veins.
It then pierces the fascia of the parotid gland finally dividing into its terminal branches within the
gland itself.
Surface marking of the carotid
This is an imaginary line drawn from the bifurcation of the common carotid passing behind the angle
of the jaw to a point immediately anterior to the tragus of the ear.

Branches of the external carotid artery


It has six branches, three in front, two behind and one deep.

Three in front Superior thyroid


Lingual
Facial

Two behind Occipital


Posterior auricular

Deep Ascending pharyngeal

It terminates by dividing into the superficial temporal and maxillary arteries in the parotid gland.

Image sourced from Wikipedia


Question 348 of 560

In Froment's test which muscle function is tested?

Flexor pollicis longus

Abductor pollicis longus

Abductor pollicis brevis

Adductor pollicis

Opponens pollicis

Please rate this question:

Discuss and give feedback


Next question

Nerve signs

Froment's sign

 Assess for ulnar nerve palsy


 Adductor pollicis muscle function tested
 Hold a piece of paper between their thumb and index finger. The object is then pulled away.
If ulnar nerve palsy, unable to hold the paper and will flex the flexor pollicis longus to
compensate (flexion of thumb at interphalangeal joint).

Phalen's test

 Assess carpal tunnel syndrome


 More sensitive than Tinel's sign
 Hold wrist in maximum flexion and the test is positive if there is numbness in the median
nerve distribution.
Tinel's sign

 Assess for carpal tunnel syndrome


 Tap the median nerve at the wrist and the test is positive if there is tingling/electric-like
sensations over the distribution of the median nerve.

Next question
Question 349 of 560

A 22 year old man is involved in a fight outside a nightclub. He is stabbed in the back, on the left
side, approximately 3cm below the 12th rib in the mid scapular line. The structure most likely to be
injured first as a result is the:

Spleen

Left kidney

Left adrenal gland

Left ureter

None of the above

The left kidney lies in this location and is the most likely structure to be injured. The Spleen lies more
superiorly, and the left adrenal and ureter are unlikely to be injured in isolation.
Please rate this question:

Discuss and give feedback


Next question

Levels

Transpyloric plane
Level of the body of L1

 Pylorus stomach
 Left kidney hilum (L1- left one!)
 Fundus of the gallbladder
 Neck of pancreas
 Duodenojejunal flexure
 Superior mesenteric artery
 Portal vein
 Left and right colic flexure
 Root of the transverse mesocolon
 2nd part of the duodenum
 Upper part of conus medullaris
 Spleen

Can be identified by asking the supine patient to sit up without using their arms. The plane is located
where the lateral border of the rectus muscle crosses the costal margin.

Anatomical planes
Subcostal plane Lowest margin of 10th costal cartilage

Intercristal plane Level of body L4 (highest point of iliac crest)

Intertubercular plane Level of body L5

Common level landmarks


Inferior mesenteric artery L3

Bifurcation of aorta into common iliac arteries L4

Formation of IVC L5 (union of common iliac veins)

Diaphragm apertures  Vena cava T8


 Oesophagus T10
 Aortic hiatus T12

Next question
Question 350-352 of 560

Theme: Cutaneous innervation

A. Ulnar nerve
B. Fifth cervical spinal segment
C. Radial nerve
D. Musculocutaneous nerve
E. Median nerve
F. None of these

Please select the source of innervation for the region described. Each option may be used once,
more than once or not at all.

350. The skin on the palmar aspect of the thumb

You answered Ulnar nerve

The correct answer is Median nerve

The median nerve supplies cutaneous sensation to this region.


See diagram below

351. The nail bed of the index finger

You answered Ulnar nerve

The correct answer is Median nerve

352. The skin overlying the medial aspect of the palm

Ulnar nerve

This area is innervated by the ulnar nerve.

Please rate this question:

Discuss and give feedback


Next question

Brachial plexus
Origin Anterior rami of C5 to T1

Sections of the  Roots, trunks, divisions, cords, branches


plexus  Mnemonic:Real Teenagers Drink Cold Beer

Roots  Located in the posterior triangle


 Pass between scalenus anterior and medius

Trunks  Located posterior to middle third of clavicle


 Upper and middle trunks related superiorly to the subclavian artery
 Lower trunk passes over 1st rib posterior to the subclavian artery

Divisions Apex of axilla

Cords Related to axillary artery

Diagram illustrating the branches of the brachial plexus

Image sourced from Wikipedia

Cutaneous sensation of the upper limb


Image sourced from Wikipedia

Next question
Question 353 of 560

At what level does the sciatic nerve usually bifurcate into the tibial and common peroneal nerves?

At the superior aspect of the popliteal fossa

At the inferior aspect of the popliteal fossa

At the inferior border of gluteus maximus

At the inferior border of the piriformis muscle

In the pelvis

The sciatic nerve passes vertically downwards over the posterior surface of the obturator internus
and quadratus femoris to the hamstring compartment of the thigh, here it is crossed posteriorly by
the long head of biceps femoris. In the buttock it lies under the cover of gluteus maximus. It
separates into its tibial and common peroneal components at the upper aspect of the popliteal fossa.
Please rate this question:

Discuss and give feedback


Next question

Sciatic nerve

The sciatic nerve is formed from the sacral plexus and is the largest nerve in the body. It is the
continuation of the main part of the plexus arising from ventral rami of L4 to S3. These rami
converge at the inferior border of piriformis to form the nerve itself. It passes through the inferior part
of the greater sciatic foramen and emerges beneath piriformis. Medially, lie the inferior gluteal nerve
and vessels and the pudendal nerve and vessels. It runs inferolaterally under the cover of gluteus
maximus midway between the greater trochanter and ischial tuberosity. It receives its blood supply
from the inferior gluteal artery. The nerve provides cutaneous sensation to the skin of the foot and
the leg. It also innervates the posterior thigh muscles and the lower leg and foot muscles. The nerve
splits into the tibial and common peroneal nerves approximately half way down the posterior thigh.
The tibial nerve supplies the flexor muscles and the common peroneal nerve supplies the extensor
muscles and the abductor muscles.

Summary points
Origin Spinal nerves L4 - S3
Articular Branches Hip joint

Muscular branches in  Semitendinosus


upper leg  Semimembranosus
 Biceps femoris
 Part of adductor magnus

Cutaneous sensation  Posterior aspect of thigh (via cutaneous nerves)


 Gluteal region
 Entire lower leg (except the medial aspect)

Terminates At the upper part of the popliteal fossa by dividing into the tibial and
peroneal nerves

 The nerve to the short head of the biceps femoris comes from the common peroneal part of
the sciatic and the other muscular branches arise from the tibial portion.
 The tibial nerve goes on to innervate all muscles of the foot except the extensor digitorum
brevis (which is innervated by the common peroneal nerve).

Next question
Question 354 of 560

A 28 year old man has a pleomorphic adenoma and the decision is made to resect this surgically.
Which of the following structures is least likely to be encountered during surgical resection of the
parotid gland?

External carotid artery

Retromandibular vein

Auriculotemporal nerve

Mandibular nerve

Zygomatic branch of the facial nerve

Structures passing through the parotid gland

 Facial nerve and branches


 External carotid artery (and its branches; the maxillary and superficial temporal)
 Retromandibular vein
 Auriculotemporal nerve

The mandibular nerve is well separated from the parotid gland.


The maxillary vein joins to the superficial temporal vein and they form the retromandibular vein which
then runs through the parotid gland.
The auriculotemporal nerve runs through the gland. Following a parotidectomy this nerve may be
damaged and during neuronal regrowth may then attach to sweat glands in this region. This can
then cause gustatory sweating (Freys Syndrome).
The facial nerve branch is the marginal mandibular branch and this is related to the gland.
Please rate this question:

Discuss and give feedback


Next question

Parotid gland

Anatomy of the parotid gland


Location Overlying the mandibular ramus; anterior and inferior to the ear.

Salivary duct Crosses the masseter, pierces the buccinator and drains adjacent to the
2nd upper molar tooth (Stensen's duct).

Structures passing  Facial nerve (Mnemonic: The Zebra Buggered My Cat; Temporal
through the gland Zygomatic, Buccal, Mandibular, Cervical)
 External carotid artery
 Retromandibular vein
 Auriculotemporal nerve

Relations  Anterior: masseter, medial pterygoid, superficial temporal and


maxillary artery, facial nerve, stylomandibular ligament
 Posterior: posterior belly digastric muscle, sternocleidomastoid,
stylohyoid, internal carotid artery, mastoid process, styloid
process

Arterial supply Branches of external carotid artery

Venous drainage Retromandibular vein

Lymphatic drainage Deep cervical nodes

Nerve innervation  Parasympathetic-Secretomotor


 Sympathetic-Superior cervical ganglion
 Sensory- Greater auricular nerve

Parasympathetic stimulation produces a water rich, serous saliva. Sympathetic stimulation leads to
the production of a low volume, enzyme-rich saliva.
Next question
Question 355 of 560

A 23 year old man is undergoing a hernia repair and the mesh is to be sutured to the inguinal
ligament. From which of the following does the inguinal ligament arise?

Transversus abdominis fascia

Internal oblique

Rectus sheath

Rectus abdominis muscle

External oblique aponeurosis

The inguinal ligament is formed by the external oblique aponeurosis. It runs from the pubic tubercle
to the anterior superior iliac spine.
Please rate this question:

Discuss and give feedback


Next question

Abdominal wall

The 2 main muscles of the abdominal wall are the rectus abdominis (anterior) and the quadratus
lumborum (posterior).
The remaining abdominal wall consists of 3 muscular layers. Each muscle passes from the lateral
aspect of the quadratus lumborum posteriorly to the lateral margin of the rectus sheath anteriorly.
Each layer is muscular posterolaterally and aponeurotic anteriorly.

Image sourced from Wikipedia


Muscles of abdominal wall
External  Lies most superficially
oblique  Originates from 5th to 12th ribs
 Inserts into the anterior half of the outer aspect of the iliac crest, linea
alba and pubic tubercle
 More medially and superiorly to the arcuate line, the aponeurotic layer
overlaps the rectus abdominis muscle
 The lower border forms the inguinal ligament
 The triangular expansion of the medial end of the inguinal ligament is
the lacunar ligament.

Internal  Arises from the thoracolumbar fascia, the anterior 2/3 of the iliac crest
oblique and the lateral 2/3 of the inguinal ligament
 The muscle sweeps upwards to insert into the cartilages of the lower 3
ribs
 The lower fibres form an aponeurosis that runs from the tenth costal
cartilage to the body of the pubis
 At its lowermost aspect it joins the fibres of the aponeurosis of
transversus abdominis to form the conjoint tendon.

Transversus  Innermost muscle


abdominis  Arises from the inner aspect of the costal cartilages of the lower 6 ribs ,
from the anterior 2/3 of the iliac crest and lateral 1/3 of the inguinal
ligament
 Its fibres run horizontally around the abdominal wall ending in an
aponeurosis. The upper part runs posterior to the rectus abdominis.
Lower down the fibres run anteriorly only.
 The rectus abdominis lies medially; running from the pubic crest and
symphysis to insert into the xiphoid process and 5th, 6th and 7th costal
cartilages. The muscles lies in a aponeurosis as described above.
 Nerve supply: anterior primary rami of T7-12

Surgical notes
During abdominal surgery it is usually necessary to divide either the muscles or their aponeuroses.
During a midline laparotomy it is desirable to divide the aponeurosis. This will leave the rectus
sheath intact above the arcuate line and the muscles intact below it. Straying off the midline will
often lead to damage to the rectus muscles, particularly below the arcuate line where they may often
be in close proximity to each other.
Next question
Question 356 of 560

A 56 year old man is undergoing a carotid endarterectomy. The internal carotid artery is mobilised.
How many branches does this vessel give off in the neck?

The internal carotid does not have any branches in the neck.
Please rate this question:

Discuss and give feedback


Next question

Internal carotid artery

The internal carotid artery is formed from the common carotid opposite the upper border of the
thyroid cartilage. It extends superiorly to enter the skull via the carotid canal. From the carotid canal
it then passes through the cavernous sinus, above which it divides into the anterior and middle
cerebral arteries.

Relations in the neck


Posterior  Longus capitis
 Pre-vertebral fascia
 Sympathetic chain
 Superior laryngeal nerve

Medially  External carotid (near origin)


 Wall of pharynx
 Ascending pharyngeal artery
Laterally  Internal jugular vein (moves posteriorly at entrance to skull)
 Vagus nerve (most posterolaterally)

Anteriorly  Sternocleidomastoid
 Lingual and facial veins
 Hypoglossal nerve

Relations in the carotid canal

 Internal carotid plexus


 Cochlea and middle ear cavity
 Trigeminal ganglion (superiorly)
 Leaves canal lies above the foramen lacerum

Path and relations in the cranial cavity


The artery bends sharply forwards in the cavernous sinus, the aducens nerve lies close to its
inferolateral aspect. The oculomotor, trochlear, opthalmic and, usually, the maxillary nerves lie in the
lateral wall of the sinus. Near the superior orbital fissure it turns posteriorly and passes postero-
medially to pierce the roof of the cavernous sinus inferior to the optic nerve. It then passes between
the optic and oculomotor nerves to terminate below the anterior perforated substance by dividing
into the anterior and middle cerebral arteries.

Branches

 Anterior and middle cerebral artery


 Ophthalmic artery
 Posterior communicating artery
 Anterior choroid artery
 Meningeal arteries
 Hypophyseal arteries

Image demonstrating the internal carotid artery and its relationship to the external carotid artery
Image sourced from Wikipedia

Next question
Question 357 of 560

Which of the following is a content of the adductor canal?

Saphenous nerve

Sural nerve

Femoral nerve

Profunda branch of the femoral artery

Saphenous vein

It contains the saphenous nerve and the superficial branch of the femoral artery.
Please rate this question:

Discuss and give feedback


Next question

Adductor canal

 Also called Hunter's or subsartorial canal


 Immediately distal to the apex of the femoral triangle, lying in the middle third of the thigh.
Canal terminates at the adductor hiatus.

Borders Contents

Laterally Vastus medialis muscle Saphenous nerve

Posteriorly Adductor longus, adductor magnus Superficial femoral artery

Roof Sartorius Superficial femoral vein


In the image below the sartorius muscle is removed to expose the canal contents

Image sourced from Wikipedia

Next question
Question 358 of 560

A 56 year old man is undergoing a high anterior resection. Which of the following structures is at
greatest risk of injury in this procedure?

Superior mesenteric artery

Left ureter

External iliac vein

External iliac artery

Inferior vena cava

A careless surgeon may damage all of these structures. However, the structure at greatest risk and
most frequently encountered is the left ureter.

Please rate this question:

Discuss and give feedback

Next question

Colon anatomy

The colon commences with the caecum. This represents the most dilated segment of the human
colon and its base (which is intraperitoneal) is marked by the convergence of teniae coli. At this point
is located the vermiform appendix. The colon continues as the ascending colon, the posterior aspect
of which is retroperitoneal. The line of demarcation between the intra and retro peritoneal right colon
is visible as a white line, in the living, and forms the line of incision for colonic resections.

The ascending colon becomes the transverse colon after passing the hepatic flexure. At this located
the colon becomes wholly intra peritoneal once again. The superior aspect of the transverse colon is
the point of attachment of the transverse colon to the greater omentum. This is an important
anatomical site since division of these attachments permits entry into the lesser sac. Separation of
the greater omentum from the transverse colon is a routine operative step in both gastric and colonic
resections.

At the left side of the abdomen the transverse colon passes to the left upper quadrant and makes an
oblique inferior turn at the splenic flexure. Following this, the posterior aspect becomes
retroperitoneal once again.

At the level of approximately L4 the descending colon becomes wholly intraperitoneal and becomes
the sigmoid colon. Whilst the sigmoid is wholly intraperitoneal there are usually attachments laterally
between the sigmoid and the lateral pelvic sidewall. These small congenital adhesions are not formal
anatomical attachments but frequently require division during surgical resections.

At its distal end the sigmoid passes to the midline and at the region around the sacral promontary it
becomes the upper rectum. This transition is visible macroscopically as the point where the teniae
fuse. More distally the rectum passes through the peritoneum at the region of the peritoneal
reflection and becomes extraperitoneal.

Arterial supply
Superior mesenteric artery and inferior mesenteric artery: linked by the marginal artery.
Ascending colon: ileocolic and right colic arteries
Transverse colon: middle colic artery
Descending and sigmoid colon: inferior mesenteric artery

Venous drainage
From regional veins (that accompany arteries) to superior and inferior mesenteric vein

Lymphatic drainage
Initially along nodal chains that accompany supplying arteries, then para-aortic nodes.

Embryology
Midgut- Second part of duodenum to 2/3 transverse colon
Hindgut- Distal 1/3 transverse colon to anus

Peritoneal location
The right and left colon are part intraperitoneal and part extraperitoneal. The sigmoid and transverse
colon are generally wholly intraperitoneal. This has implications for the sequelae of perforations,
which will tend to result in generalised peritonitis in the wholly intra peritoneal segments.

Colonic relations

Region of colon Relation


Region of colon Relation

Caecum/ right colon Right ureter, gonadal vessels

Hepatic flexure Gallbladder (medially)

Splenic flexure Spleen and tail of pancreas

Distal sigmoid/ upper rectum Left ureter

Rectum Ureters, autonomic nerves, seminal vesicles, prostate, urethra (distally)

Next question
Question 359 of 560

From which of these foraminae does the opthalmic branch of the trigeminal nerve exit the skull?

Foramen ovale

Foramen rotundum

Foramen spinosum

Superior orbital fissure

Foramen magnum

Mnemonic:

Standing Room Only -Exit of branches of trigeminal nerve from the skull

V1 -Superior orbital fissure


V2 -foramen Rotundum
V3 -foramen Ovale

The opthalmic branch of the trigeminal nerve exits the skull through the superior orbital fissure.
Please rate this question:

Discuss and give feedback


Next question

Trigeminal nerve

The trigeminal nerve is the main sensory nerve of the head. In addition to its major sensory role, it
also innervates the muscles of mastication.

Distribution of the trigeminal nerve


Sensory  Scalp
 Face
 Oral cavity (and teeth)
 Nose and sinuses
 Dura mater
Motor  Muscles of mastication
 Mylohyoid
 Anterior belly of digastric
 Tensor tympani
 Tensor palati

Autonomic connections (ganglia)  Ciliary


 Sphenopalatine
 Otic
 Submandibular

Path

 Originates at the pons


 Sensory root forms the large, crescentic trigeminal ganglion within Meckel's cave, and
contains the cell bodies of incoming sensory nerve fibres. Here the 3 branches exit.
 The motor root cell bodies are in the pons and the motor fibres are distributed via the
mandibular nerve. The motor root is not part of the trigeminal ganglion.

Branches of the trigeminal nerve


Ophthalmic nerve Sensory only

Maxillary nerve Sensory only

Mandibular nerve Sensory and motor

Sensory
Ophthalmic Exits skull via the superior orbital fissure
Sensation of: scalp and forehead, the upper eyelid, the conjunctiva and cornea of
the eye, the nose (including the tip of the nose, except alae nasi), the nasal
mucosa, the frontal sinuses, and parts of the meninges (the dura and blood
vessels).

Maxillary Exit skull via the foramen rotundum


nerve Sensation: lower eyelid and cheek, the nares and upper lip, the upper teeth and
gums, the nasal mucosa, the palate and roof of the pharynx, the maxillary,
ethmoid and sphenoid sinuses, and parts of the meninges.

Mandibular Exit skull via the foramen ovale


nerve Sensation: lower lip, the lower teeth and gums, the chin and jaw (except the angle
of the jaw), parts of the external ear, and parts of the meninges.

Motor
Distributed via the mandibular nerve.
The following muscles of mastication are innervated:

 Masseter
 Temporalis
 Medial pterygoid
 Lateral pterygoid

Other muscles innervated include:

 Tensor veli palatini


 Mylohyoid
 Anterior belly of digastric
 Tensor tympani

Next question
Question 360 of 560

A 56 year old lady with metastatic breast cancer develops an oestolytic deposit in the proximal
femur. One morning whilst getting out of bed she notices severe groin pain. X-rays show that the
lesser trochanter has been avulsed. Which muscle is the most likely culprit?

Vastus lateralis

Psoas major

Piriformis

Gluteus maximus

Gluteus medius

The psoas major inserts into the lesser trochanter and contracts when raising the trunk from the
supine position. When oestolytic lesions are present in the femur the lesser trochanter may be
avulsed.

Please rate this question:

Discuss and give feedback

Next question

Psoas Muscle

Origin
The deep part originates from the transverse processes of the five lumbar vertebrae, the superficial
part originates from T12 and the first 4 lumbar vertebrae.

Insertion
Lesser trochanter of the femur.
Innervation
Anterior rami of L1 to L3.

Action
Flexion and external rotation of the hip. Bilateral contraction can raise the trunk from the supine
position.

Next question
Question 361 of 560

Which of the following nerves is responsible for the motor innervation of the sternocleidomastoid
muscle?

Ansa cervicalis

Accessory nerve

Hypoglossal nerve

Facial nerve

Vagus nerve

Theme from January 2013 Exam


The motor supply to the sternocleidomastoid is from the accessory nerve. The ansa cervicalis
supplies sensory information from the muscle.
Please rate this question:

Discuss and give feedback


Next question

Sternocleidomastoid

Anatomy
Origin Rounded tendon attached to upper manubrium sterni and muscular head attached to
medial third of the clavicle

Insertion Mastoid process of the temporal bone and lateral area of the superior nuchal line of
the occipital bone

Innervation Spinal part of accessory nerve and anterior rami of C2 and C3 (proprioception)
Action  Both: extend the head at atlanto-occipital joint and flex the cervical
vertebral column. Accessory muscles of inspiration.
 Single: lateral flexion of neck, rotates head so face looks upward to the
opposite side

Sternocleidomastoid divides the anterior and posterior triangles of the neck.


Next question
Question 362 of 560

A 42 year old lady undergoes a difficult cholecystectomy and significant bleeding is occurring. The
surgeons place a vascular clamp transversely across the anterior border of the epiploic foramen.
Which of the following structures will be occluded in this manoeuvre?

Cystic artery

Cystic duct

Left gastric artery

Portal vein

None of the above

The portal vein, hepatic artery and common bile duct are occluded.
Please rate this question:

Discuss and give feedback


Next question

Epiploic Foramen

The epiploic foramen has the following boundaries:


Anteriorly (in the free edge of the lesser Bile duct to the right, portal vein behind and hepatic
omentum) artery to the left.

Posteriorly Inferior vena cava

Inferiorly 1st part of the duodenum

Superiorly Caudate process of the liver


During liver surgery bleeding may be controlled using a Pringles manoeuvre, this involves placing a
vascular clamp across the anterior aspect of the epiploic foramen. Thereby occluding:

 Common bile duct


 Hepatic artery
 Portal vein

Next question
Question 363 of 560

A 34 year old man is injured by farm machinery and sustains a laceration at the superolateral aspect
of the popliteal fossa. The medial aspect of biceps femoris is lacerated. Which of the following
underlying structures is at greatest risk of injury?

Gracilis

Sural nerve

Nerve to semimembranosus

Popliteal artery

Common peroneal nerve

The common peroneal nerve lies under the medial aspect of biceps femoris and is therefore at
greatest risk of injury. The tibial nerve may also be damaged in such an injury (but is not listed here).
The sural nerve branches off more inferiorly.

Please rate this question:

Discuss and give feedback

Next question

Common peroneal nerve

Derived from the dorsal divisions of the sacral plexus (L4, L5, S1 and S2).

This nerve supplies the skin and fascia of the anterolateral surface of the leg and the dorsum of the
foot. It also innervates the muscles of the anterior and peroneal compartments of the leg, extensor
digitorum brevis as well as the knee, ankle and foot joints.
It is laterally placed within the sciatic nerve. From the bifurcation of the sciatic nerve it passes
inferolaterally in the lateral and proximal part of the popliteal fossa, under the cover of biceps femoris
and its tendon. To reach the posterior aspect of the fibular head. It ends by dividing into the deep
and superficial peroneal nerves at the point where it winds around the lateral surface of the neck of
the fibula in the body of peroneus longus, approximately 2cm distal to the apex of the head of the
fibula. It is palpable posterior to the head of the fibula.

Branches

In the thigh Nerve to the short head of biceps


Articular branch (knee)

In the popliteal fossa Lateral cutaneous nerve of the calf

Neck of fibula Superficial and deep peroneal nerves

Next question
Question 364 of 560

A 56 year old lady undergoes a Hartmans style resection of the sigmoid colon, with ligation of the
vessels close to the colon. Which of the following vessels will be responsible to supplying the rectal
stump directly?

Superior mesenteric artery

Middle colic artery

Superior rectal artery

Inferior mesenteric artery

External iliac artery

This question is addressing the blood supply to the rectum. Which is supplied by the superior rectal
artery. High ligation of the IMA may compromise this structure. However, the question states that
during the Hartmans procedure the vessels were ligated close to the bowel. Implying that the
superior rectal was preserved.
Please rate this question:

Discuss and give feedback


Next question

Rectum

The rectum is approximately 12 cm long. It is a capacitance organ. It has both intra and
extraperitoneal components. The transition between the sigmoid colon is marked by the
disappearance of the tenia coli.The extra peritoneal rectum is surrounded by mesorectal fat that also
contains lymph nodes. This mesorectal fatty layer is removed surgically during rectal cancer surgery
(Total Mesorectal Excision). The fascial layers that surround the rectum are important clinical
landmarks, anteriorly lies the fascia of Denonvilliers. Posteriorly lies Waldeyers fascia.

Extra peritoneal rectum

 Posterior upper third


 Posterior and lateral middle third
 Whole lower third
Relations
Anteriorly (Males) Rectovesical pouch
Bladder
Prostate
Seminal vesicles

Anteriorly (Females) Recto-uterine pouch (Douglas)


Cervix
Vaginal wall

Posteriorly Sacrum
Coccyx
Middle sacral artery

Laterally Levator ani


Coccygeus

Arterial supply
Superior rectal artery

Venous drainage
Superior rectal vein

Lymphatic drainage

 Mesorectal lymph nodes (superior to dentate line)


 Internal iliac and then para-aortic nodes
 Inguinal nodes (inferior to dentate line)

Next question
Question 365 of 560

Which of the nerves listed below is at greatest risk of injury with a laceration to the upper lateral
margin of the popliteal fossa?

Common peroneal nerve

Sural nerve

Sciatic nerve

Saphenous nerve

Tibial nerve

The sural nerve exits at the lower infero-lateral aspect of the fossa and is more at risk in short
saphenous vein surgery. The tibial nerve lies more medially and is even less likely to be injured in
this location.
Please rate this question:

Discuss and give feedback


Next question

Popliteal fossa

Boundaries of the popliteal fossa


Laterally Biceps femoris above, lateral head of gastrocnemius and plantaris below

Medially Semimembranosus and semitendinosus above, medial head of gastrocnemius below

Floor Popliteal surface of the femur, posterior ligament of knee joint and popliteus muscle

Roof Superficial and deep fascia


Image showing the popliteal fossa

© Image provided by the University of Sheffield

Contents

 Popliteal artery and vein


 Small saphenous vein
 Common peroneal nerve
 Tibial nerve
 Posterior cutaneous nerve of the thigh
 Genicular branch of the obturator nerve
 Lymph nodes

Next question
Question 366 of 560

Which option is false in relation to the trigeminal nerve?

The nerve originates at the pons

The posterior scalp is supplied by the trigeminal nerve

The maxillary nerve exits via the foramen rotundum

The maxillary nerve is purely sensory

The motor root is not in the trigeminal ganglion

The posterior scalp is supplied by C2-C3.


Please rate this question:

Discuss and give feedback


Next question

Trigeminal nerve

The trigeminal nerve is the main sensory nerve of the head. In addition to its major sensory role, it
also innervates the muscles of mastication.

Distribution of the trigeminal nerve


Sensory  Scalp
 Face
 Oral cavity (and teeth)
 Nose and sinuses
 Dura mater

Motor  Muscles of mastication


 Mylohyoid
 Anterior belly of digastric
 Tensor tympani
 Tensor palati

Autonomic connections (ganglia)  Ciliary


 Sphenopalatine
 Otic
 Submandibular

Path

 Originates at the pons


 Sensory root forms the large, crescentic trigeminal ganglion within Meckel's cave, and
contains the cell bodies of incoming sensory nerve fibres. Here the 3 branches exit.
 The motor root cell bodies are in the pons and the motor fibres are distributed via the
mandibular nerve. The motor root is not part of the trigeminal ganglion.

Branches of the trigeminal nerve


Ophthalmic nerve Sensory only

Maxillary nerve Sensory only

Mandibular nerve Sensory and motor

Sensory
Ophthalmic Exits skull via the superior orbital fissure
Sensation of: scalp and forehead, the upper eyelid, the conjunctiva and cornea of
the eye, the nose (including the tip of the nose, except alae nasi), the nasal
mucosa, the frontal sinuses, and parts of the meninges (the dura and blood
vessels).

Maxillary Exit skull via the foramen rotundum


nerve Sensation: lower eyelid and cheek, the nares and upper lip, the upper teeth and
gums, the nasal mucosa, the palate and roof of the pharynx, the maxillary,
ethmoid and sphenoid sinuses, and parts of the meninges.

Mandibular Exit skull via the foramen ovale


nerve Sensation: lower lip, the lower teeth and gums, the chin and jaw (except the angle
of the jaw), parts of the external ear, and parts of the meninges.

Motor
Distributed via the mandibular nerve.
The following muscles of mastication are innervated:

 Masseter
 Temporalis
 Medial pterygoid
 Lateral pterygoid

Other muscles innervated include:

 Tensor veli palatini


 Mylohyoid
 Anterior belly of digastric
 Tensor tympani

Next question
Question 367 of 560

A 45 year old man is undergoing a lymph node biopsy from the posterior triangle of his neck. Which
structure forms the posterior border of this region?

Trapezius muscle

Diagastric muscle

External jugular vein

Omohyoid muscle

Sternocleidomastoid muscle

The borders are described below.


Please rate this question:

Discuss and give feedback


Next question

Posterior triangle of the neck

Boundaries
Apex Sternocleidomastoid and the Trapezius muscles at the Occipital bone

Anterior Posterior border of the Sternocleidomastoid

Posterior Anterior border of the Trapezius

Base Middle third of the clavicle


Image sourced from Wikipedia

Contents
Nerves  Accessory nerve
 Phrenic nerve
 Three trunks of the brachial plexus
 Branches of the cervical plexus: Supraclavicular nerve, transverse cervical
nerve, great auricular nerve, lesser occipital nerve

Vessels  External jugular vein


 Subclavian artery

Muscles  Inferior belly of omohyoid


 Scalene

Lymph  Supraclavicular
nodes  Occipital

Next question
Question 368 of 560

On inspecting the caecum, which of the following structures is most likely to be identified at the point
at which all the tenia coli converge?

Gonadal vessels

Appendix base

Appendix tip

Ileocaecal valve

Ileocolic artery

The tenia coli converge at the base of the appendix.


Please rate this question:

Discuss and give feedback


Next question

Caecum

Location  Proximal right colon below the ileocaecal valve


 Intraperitoneal

Posterior relations  Psoas


 Iliacus
 Femoral nerve
 Genitofemoral nerve
 Gonadal vessels

Anterior relations Greater omentum

Arterial supply Ileocolic artery


Lymphatic drainage Mesenteric nodes accompany the venous drainage

 The caecum is the most distensible part of the colon and in complete large bowel obstruction
with a competent ileocaecal valve the most likely site of eventual perforation.

Next question
Question 369 of 560

A 42 year old lady has had an axillary node clearance for breast malignancy. Post operatively she
reports weakness of the shoulder. She is unable to push herself forwards from a wall with the right
arm and the scapula is pushed out medially from the chest wall. What is the most likely nerve injury?

C5, C6

C8, T1

Axillary nerve

Long thoracic nerve

Spinal accessory nerve

Theme from January 2012 and 2009 Exam

The patient has a winged scapula caused by damage to the long thoracic nerve (C5,6,7) during
surgery. The long thoracic nerve innervates serratus anterior. Serratus anterior causes pushing out
of the scapula during a punch.

NB winging of the scapular laterally may indicate trapezius muscle weakness. Innervated by the
spinal accessory nerve.
Please rate this question:

Discuss and give feedback


Next question

Long thoracic nerve

 Derived from ventral rami of C5, C6, and C7 (close to their emergence from intervertebral
foramina)
 It runs downward and passes either anterior or posterior to the middle scalene muscle
 It reaches upper tip of serratus anterior muscle and descends on outer surface of this
muscle, giving branches into it
 Winging of Scapula occurs in long thoracic nerve injury (most common) or from spinal
accessory nerve injury (which denervates the trapezius) or a dorsal scapular nerve injury

Next question
Question 370 of 560

A 36 year old male is admitted for elective surgery for a lymph node biopsy in the supraclavicular
region. Post operatively the patient has difficulty shrugging his left shoulder. What is the most likely
reason?

Phrenic nerve lesion

Axillary nerve lesion

C5, C6 root lesion

C8, T1 root lesion

Accessory nerve lesion

Theme from September 2011 Exam


Theme from September 2013 Exam
The accessory nerve lies in the posterior triangle and may be injured in this region. Apart from
problems with shrugging the shoulder, he may also have difficulty lifting his arm above his head.
Please rate this question:

Discuss and give feedback


Next question

Posterior triangle of the neck

Boundaries
Apex Sternocleidomastoid and the Trapezius muscles at the Occipital bone

Anterior Posterior border of the Sternocleidomastoid

Posterior Anterior border of the Trapezius

Base Middle third of the clavicle


Image sourced from Wikipedia

Contents
Nerves  Accessory nerve
 Phrenic nerve
 Three trunks of the brachial plexus
 Branches of the cervical plexus: Supraclavicular nerve, transverse cervical
nerve, great auricular nerve, lesser occipital nerve

Vessels  External jugular vein


 Subclavian artery

Muscles  Inferior belly of omohyoid


 Scalene

Lymph  Supraclavicular
nodes  Occipital

Next question
Question 371 of 560

How many fissures are present within the right lung?

One

Three

Two

Four

Five

The right lung has an oblique and horizontal fissure. The upper oblique fissure separates the inferior
from the middle and upper lobes. The short horizontal fissure separates the superior and middle
lobes.
Please rate this question:

Discuss and give feedback


Next question

Lung anatomy

The right lung is composed of 3 lobes divided by the oblique and transverse fissures. The left lung
has two lobes divided by the oblique fissure.The apex of both lungs is approximately 4cm superior to
the sterno-costal joint of the first rib. Immediately below this is a sulcus created by the subclavian
artery.

Peripheral contact points of the lung

 Base: diaphragm
 Costal surface: corresponds to the cavity of the chest
 Mediastinal surface: Contacts the mediastinal pleura. Has the cardiac impression. Above and
behind this concavity is a triangular depression named the hilum, where the structures which
form the root of the lung enter and leave the viscus. These structures are invested by pleura,
which, below the hilum and behind the pericardial impression, forms the pulmonary ligament

Right lung
Above the hilum is the azygos vein; Superior to this is the groove for the superior vena cava and
right innominate vein; behind this, and nearer the apex, is a furrow for the innominate artery. Behind
the hilum and the attachment of the pulmonary ligament is a vertical groove for the oesophagus; In
front and to the right of the lower part of the oesophageal groove is a deep concavity for the
extrapericardiac portion of the inferior vena cava.

The root of the right lung lies behind the superior vena cava and the right atrium, and below the
azygos vein.

The right main bronchus is shorter, wider and more vertical than the left main bronchus and
therefore the route taken by most foreign bodies.

Image sourced from Wikipedia

Left lung
Above the hilum is the furrow produced by the aortic arch, and then superiorly the groove
accommodating the left subclavian artery; Behind the hilum and pulmonary ligament is a vertical
groove produced by the descending aorta, and in front of this, near the base of the lung, is the lower
part of the oesophagus.

The root of the left lung passes under the aortic arch and in front of the descending aorta.

Image sourced from Wikipedia


Inferior borders of both lungs

 6th rib in mid clavicular line


 8th rib in mid axillary line
 10th rib posteriorly

The pleura runs two ribs lower than the corresponding lung level.

Bronchopulmonary segments
Segment number Right lung Left lung

1 Apical Apical

2 Posterior Posterior

3 Anterior Anterior

4 Lateral Superior lingular

5 Medial Inferior lingular

6 Superior (apical) Superior (apical)

7 Medial basal Medial basal

8 Anterior basal Anterior basal

9 Lateral basal Lateral basal

10 Posterior basal Posterior basal

Next question
Question 372 of 560

Which of the following muscles is supplied by the musculocutaneous nerve?

Brachialis

Latissimus dorsi

Flexor carpi ulnaris

Teres minor

Triceps

Mnemonic

Muscles innervated by the musculocutaneous nerve BBC:

Biceps brachii
Brachialis
Coracobrachialis

Please rate this question:

Discuss and give feedback


Next question

Musculocutaneous nerve

 Branch of lateral cord of brachial plexus

Path

 It penetrates the coracobrachialis muscle


 Passes obliquely between the biceps brachii and the brachialis to the lateral side of the arm
 Above the elbow it pierces the deep fascia lateral to the tendon of the biceps brachii
 Continues into the forearm as the lateral cutaneous nerve of the forearm
Innervates

 Coracobrachialis
 Biceps brachii
 Brachialis

Next question
Question 373 of 560

Which of the following statements relating to the posterior cerebral artery is false?

It supplies the visual cortex

It is closely related to the 3rd cranial nerve

It is a branch of the basilar artery

It is connected to the circle of Willis via the superior cerebellar artery

When occluded may result in contralateral loss of field of vision

The posterior cerebral arteries are formed by the bifurcation of the basilar artery and is connected to
the circle of Willis via the posterior communicating artery.

The posterior cerebral arteries supply the occipital lobe and part of the temporal lobe.
Please rate this question:

Discuss and give feedback


Next question

Circle of Willis

The two internal carotid arteries and two vertebral arteries form an anastomosis known as the Circle
of Willis on the inferior surface of the brain. Each half of the circle is formed by:
1. Anterior communicating artery
2. Anterior cerebral artery
3. Internal carotid artery
4. Posterior communicating artery
5. Posterior cerebral arteries and the termination of the basilar artery

The circle and its branches supply; the corpus striatum, internal capsule, diencephalon and
midbrain.
Image sourced from Wikipedia

Vertebral arteries

 Enter the cranial cavity via foramen magnum


 Lie in the subarachnoid space
 Ascend on anterior surface of medulla oblongata
 Unite to form the basilar artery at the base of the pons

Branches:

 Posterior spinal artery


 Anterior spinal artery
 Posterior inferior cerebellar artery

Basilar artery
Branches:
 Anterior inferior cerebellar artery
 Labyrinthine artery
 Pontine arteries
 Superior cerebellar artery
 Posterior cerebral artery

Internal carotid arteries


Branches:

 Posterior communicating artery


 Anterior cerebral artery
 Middle cerebral artery
 Anterior choroid artery

Next question
Question 374 of 560

An elderly lady falls and lands on her hip. On examination, her hip is tender to palpation and x-rays
are taken. There are concerns that she may have an intertrochanteric fracture. What is the normal
angle between the femoral neck and the femoral shaft?

90o

105o

80o

130o

180o

The normal angle between the femoral head and shaft is 130o. Changes to this angle may occur as a
result of disease or pathology and should be investigated.
Please rate this question:

Discuss and give feedback


Next question

Hip joint

 Head of femur articulates with acetabulum of the pelvis


 Both covered by articular hyaline cartilage
 The acetabulum forms at the union of the ilium, pubis, and ischium
 The triradiate cartilage (Y-shaped growth plate) separates the pelvic bones
 The acetabulum holds the femoral head by the acetabular labrum
 Normal angle between femoral head and femoral shaft is 130o

Ligaments

 Transverse ligament: joints anterior and posterior ends of the articular cartilage
 Head of femur ligament (ligamentum teres): acetabular notch to the fovea. Contains arterial
supply to head of femur in children.
Image sourced from Wikipedia

Image sourced from Wikipedia

Extracapsular ligaments

 Iliofemoral ligament: inverted Y shape. Anterior iliac spine to the trochanteric line
 Pubofemoral ligament: acetabulum to lesser trochanter
 Ischiofemoral ligament: posterior support. Ischium to greater trochanter.

Blood supply
Medial circumflex femoral and lateral circumflex femoral arteries (Branches of profunda femoris).
Also from the inferior gluteal artery. These form an anastomosis and travel to up the femoral neck to
supply the head.
Next question
Question 375 of 560

A 22 year old man suffers a compound fracture of the tibia. During attempted surgical repair the
deep peroneal nerve is divided. Which of the following muscles will not be affected as a result?

Tibialis anterior

Peroneus longus

Extensor hallucis longus

Extensor digitorum longus

Peroneus tertius

Peroneus longus is innervated by the superficial peroneal nerve (L4, L5, S1).
Image sourced from Wikipedia

Please rate this question:

Discuss and give feedback


Next question

Deep peroneal nerve


Origin From the common peroneal nerve, at the lateral aspect of the fibula, deep to
peroneus longus

Nerve root values L4, L5, S1, S2

Course and  Pierces the anterior intermuscular septum to enter the anterior
relation compartment of the lower leg
 Passes anteriorly down to the ankle joint, midway between the two
malleoli

Terminates In the dorsum of the foot

Muscles  Tibialis anterior


innervated  Extensor hallucis longus
 Extensor digitorum longus
 Peroneus tertius
 Extensor digitorum brevis

Cutaneous Web space of the first and second toes


innervation

Actions  Dorsiflexion of ankle joint


 Extension of all toes (extensor hallucis longus and extensor
digitorum longus)
 Inversion of the foot

After its bifurcation past the ankle joint, the lateral branch of the deep peroneal nerve innervates the
extensor digitorum brevis and the extensor hallucis brevis
The medial branch supplies the web space between the first and second digits.
Next question
Question 376 of 560

A 17 year old male presents to the clinic. He complains of difficulty using his left hand. It has been a
persistent problem since he sustained a distal humerus fracture as a child. On examination there is
diminished sensation overlying the hypothenar eminence and medial one and half fingers. What is
the most likely nerve lesion?

Anterior interosseous nerve

Posterior interosseous nerve

Ulnar nerve

Median nerve

Radial nerve

Theme from April 2012 Exam


This sensory deficit pattern is most consistent with ulnar nerve injury.
Image sourced from Wikipedia

Please rate this question:

Discuss and give feedback


Next question

Ulnar nerve

Origin

 C8, T1

Supplies (no muscles in the upper arm)

 Flexor carpi ulnaris


 Flexor digitorum profundus
 Flexor digiti minimi
 Abductor digiti minimi
 Opponens digiti minimi
 Adductor pollicis
 Interossei muscle
 Third and fourth lumbricals
 Palmaris brevis

Path

 Posteromedial aspect of upper arm to flexor compartment of forearm, then along the ulnar.
Passes beneath the flexor carpi ulnaris muscle, then superficially through the flexor
retinaculum into the palm of the hand.

Image sourced from Wikipedia

Branches
Branch Supplies

Muscular branch Flexor carpi ulnaris


Medial half of the flexor digitorum profundus

Palmar cutaneous branch (Arises near the Skin on the medial part of the palm
middle of the forearm)

Dorsal cutaneous branch Dorsal surface of the medial part of the hand

Superficial branch Cutaneous fibres to the anterior surfaces of the


medial one and one-half digits

Deep branch Hypothenar muscles


All the interosseous muscles
Third and fourth lumbricals
Adductor pollicis
Medial head of the flexor pollicis brevis

Effects of injury
Damage at the wrist  Wasting and paralysis of intrinsic hand muscles (claw hand)
 Wasting and paralysis of hypothenar muscles
 Loss of sensation medial 1 and half fingers

Damage at the elbow  Radial deviation of the wrist


 Clawing less in 4th and 5th digits

Next question
Question 377 of 560

An 18 year old athlete attends orthopaedic clinic reporting pain and swelling over the medial aspect
of the knee joint. The pain occurs when climbing the stairs, but is not present when walking on flat
ground. Clinically there is pain over the medial, proximal tibia and the McMurray test is negative.
What is the most likely cause of this patient's symptoms?

Anterior cruciate ligament tear

Prepatellar bursitis

Medial meniscus injury

Pes Anserinus Bursitis

Fracture of tibia

Pes anserinus: GOOSE'S FOOT

Combination of sartorius, gracilis and semitendinous tendons inserting into the anteromedial
proximal tibia.

Pes Anserinus Bursitis is common in sportsmen due to overuse injuries. The main sign is of pain in
the medial proximal tibia. As the McMurray test is negative, medial meniscal injury is excluded.
Please rate this question:

Discuss and give feedback


Next question

Sartorius

 Longest strap muscle in the body


 Most superficial muscle in the anterior compartment of the thigh

Origin Anterior superior iliac spine


Insertion Medial surface of the of the body of the tibia (upper part). It inserts anterior to
gracilis and semitendinosus

Nerve Supply Femoral nerve (L2,3)

Action  Flexor of the hip and knee, slight abducts the thigh and rotates it laterally
 It assists with medial rotation of the tibia on the femur. For example it
would play a pivotal role in placing the right heel onto the left knee ( and
vice versa)

Important The middle third of this muscle, and its strong underlying fascia forms the roof of
relations the adductor canal , in which lie the femoral vessels, the saphenous nerve and the
nerve to vastus medialis.
Next question
Question 378 of 560

Which of the following structures lies most posteriorly at the porta hepatis?

Cystic artery

Common hepatic artery

Left hepatic artery

Portal vein

Common bile duct

The portal vein is the most posterior structure at the porta hepatis.The common bile duct is a
continuation of the common hepatic duct and is formed by the union of the common hepatic duct and
the cystic duct.
Please rate this question:

Discuss and give feedback


Next question

Liver

Structure of the liver


Right lobe  Supplied by right hepatic artery
 Contains Couinaud segments V to VIII (-/+Sg I)

Left lobe  Supplied by the left hepatic artery


 Contains Couinaud segments II to IV (+/- Sg1)

Quadrate lobe  Part of the right lobe anatomically, functionally is part of the left
 Couinaud segment IV
 Porta hepatis lies behind
 On the right lies the gallbladder fossa
 On the left lies the fossa for the umbilical vein

Caudate lobe  Supplied by both right and left hepatic arteries


 Couinaud segment I
 Lies behind the plane of the porta hepatis
 Anterior and lateral to the inferior vena cava
 Bile from the caudate lobe drains into both right and left hepatic ducts

Detailed knowledge of Couinaud segments is not required for MRCS

 Between the liver lobules are portal canals which contain the portal triad: Hepatic Artery,
Portal Vein, tributary of Bile Duct.

Relations of the liver


Anterior Postero inferiorly

Diaphragm Oesophagus

Xiphoid process Stomach

Duodenum

Hepatic flexure of colon

Right kidney

Gallbladder

Inferior vena cava

Porta hepatis
Location Postero inferior surface, it joins nearly at right angles with the left sagittal fossa, and
separates the caudate lobe behind from the quadrate lobe in front

Transmits  Common hepatic duct


 Hepatic artery
 Portal vein
 Sympathetic and parasympathetic nerve fibres
 Lymphatic drainage of the liver (and nodes)

Ligaments
Falciform ligament  2 layer fold peritoneum from the umbilicus to anterior liver surface
 Contains ligamentum teres (remnant umbilical vein)
 On superior liver surface it splits into the coronary and left
triangular ligaments

Ligamentum teres Joins the left branch of the portal vein in the porta hepatis

Ligamentum Remnant of ductus venosus


venosum

Arterial supply

 Hepatic artery

Venous

 Hepatic veins
 Portal vein

Nervous supply

 Sympathetic and parasympathetic trunks of coeliac plexus

Next question
Question 379 of 560

A 76 year old man is undergoing an abdominal aortic aneurysm repair. The surgeons occlude the
aorta with two clamps, the inferior clamp being placed at the point of aortic bifurcation. Which of the
following vertebral bodies will lie posterior to the clamp at this level?

L1

T10

L4

L5

L2

Theme from September 2013 Exam


The aorta bifurcates at L4. An important landmark that is tested frequently.

Please rate this question:

Discuss and give feedback

Next question

Abdominal aorta

Abdominal aortic topography

Origin T12
Termination L4

Posterior relations L1-L4 Vertebral bodies

Anterior relations Lesser omentum


Liver
Left renal vein
Inferior mesenteric vein
Third part of duodenum
Pancreas
Parietal peritoneum
Peritoneal cavity

Right lateral relations Right crus of the diaphragm


Cisterna chyli
Azygos vein
IVC (becomes posterior distally)

Left lateral relations 4th part of duodenum


Duodenal-jejunal flexure
Left sympathetic trunk

The abdominal aorta


Image sourced from Wikipedia

Next question
Question 380 of 560

Which of the following statements relating to the greater omentum is false?

It is less well developed in children under 5.

It has no relationship to the lesser sac.

It contains the gastroepiploic arteries.

Has an attachment to the transverse colon.

It may be a site of metastatic disease in ovarian cancer.

It is connected with the lesser sac and the transverse colon. This plane is entered when performing
a colonic resection. It is a common site of metastasis in many visceral malignancies.
Please rate this question:

Discuss and give feedback


Next question

Omentum

 The omentum is divided into two parts which invest the stomach. Giving rise to the greater
and lesser omentum. The greater omentum is attached to the inferolateral border of the
stomach and houses the gastro-epiploic arteries.
 It is of variable size but is less well developed in children. This is important as the omentum
confers protection against visceral perforation (e.g. Appendicitis).
 Inferiorly between the omentum and transverse colon is one potential entry point into the
lesser sac.
 Several malignant processes may involve the omentum of which ovarian cancer is the most
notable.

Next question
Question 381 of 560

A 48 year old man with newly diagnosed hypertension is found to have a phaeochromocytoma of the
left adrenal gland and is due to undergo a laparoscopic left adrenalectomy. Which of the following
structures is not directly related to the left adrenal gland?

Crus of the diaphragm

Lesser curvature of the stomach

Kidney

Pancreas

Splenic artery

The left adrenal gland is slightly larger than the right. It is crescent in shape and its concavity is
adapted to the medial border of the upper part of the left kidney. The upper area is covered by
peritoneum of the omental bursa which separates it from the cardia of the stomach. The lower area
is in contact with the pancreas and splenic artery and is not covered by peritoneum. On the anterior
surface is a hilum from which the suprarenal vein emerges. The lateral aspect rests on the kidney.
The medial is small and is on the left crus of the diaphragm.

Please rate this question:

Discuss and give feedback

Next question

Adrenal gland anatomy

Anatomy
Location Superomedially to the upper pole of each kidney

Relationships of the right Diaphragm-Posteriorly, Kidney-Inferiorly, Vena Cava-Medially, Hepato-renal


adrenal pouch and bare area of the liver-Anteriorly

Relationships of the left Crus of the diaphragm-Postero- medially, Pancreas and splenic vessels-
adrenal Inferiorly, Lesser sac and stomach-Anteriorly

Superior adrenal arteries- from inferior phrenic artery, Middle adrenal


Arterial supply arteries - from aorta, Inferior adrenal arteries -from renal arteries

Venous drainage of the Via one central vein directly into the IVC
right adrenal

Venous drainage of the Via one central vein into the left renal vein
left adrenal

Next question
Question 382 of 560

Which of the following nerves innervates the long head of the biceps femoris muscle?

Inferior gluteal nerve

Tibial division of sciatic nerve

Superior gluteal nerve

Common peroneal division of sciatic nerve

Obturator nerve

The short head of biceps femoris, which may occasionally be absent, is innervated by the common
peroneal component of the sciatic nerve. The long head is innervated by the tibial division of the
sciatic nerve.

Please rate this question:

Discuss and give feedback

Next question

Biceps femoris

The biceps femoris is one of the hamstring group of muscles located in the posterior upper thigh. It
has two heads.

Long head

Origin Ischial tuberosity


Insertion Fibular head

Action Knee flexion, lateral rotation tibia, extension hip

Innervation Tibial division of sciatic nerve (L5, S1, S2)

Arterial Profunda femoris artery, inferior gluteal artery, and the superior muscular branches of
supply popliteal artery

Image demonstrating the biceps femoris muscle, with the long head outlined

Image sourced from Wikipedia

Short head

Origin Lateral lip of linea aspera, lateral supracondylar ridge of femur

Insertion Fibular head


Action Knee flexion, lateral rotation tibia

Innervation Common peroneal division of sciatic nerve (L5, S1, S2)

Arterial Profunda femoris artery, inferior gluteal artery, and the superior muscular branches of
supply popliteal artery

Next question
Question 383 of 560

A 72 year old male with end stage critical ischaemia is undergoing an axillo-femoral bypass. What
structure is not closely related to the axillary artery?

Posterior cord of the brachial plexus

Scalenus anterior muscle

Pectoralis minor muscle

Axillary vein

Lateral cord of the brachial plexus

The axillary artery is the continuation of the subclavian artery. It is surrounded by the cords of the
brachial plexus (from which they are named). The axillary vein runs alongside the axillary artery
throughout its length.

Please rate this question:

Discuss and give feedback

Next question

Axilla

Boundaries of the axilla

Medially Chest wall and Serratus anterior


Laterally Humeral head

Floor Subscapularis

Anterior aspect Lateral border of Pectoralis major

Fascia Clavipectoral fascia

Content:

Long thoracic nerve (of Derived from C5-C7 and passes behind the brachial plexus to enter the axilla.
Bell) It lies on the medial chest wall and supplies serratus anterior. Its location
puts it at risk during axillary surgery and damage will lead to winging of the
scapula.

Thoracodorsal nerve and Innervate and vascularise latissimus dorsi.


thoracodorsal trunk

Axillary vein Lies at the apex of the axilla, it is the continuation of the basilic vein.
Becomes the subclavian vein at the outer border of the first rib.

Intercostobrachial nerves Traverse the axillary lymph nodes and are often divided during axillary
surgery. They provide cutaneous sensation to the axillary skin.

Lymph nodes The axilla is the main site of lymphatic drainage for the breast.

Next question
Question 384 of 560

A 28 year old man is shot in the right chest and develops a right haemothorax necessitating a
thoracotomy. The surgeons decide to place a vascular clamp across the hilum of the right lung.
Which of the following structures will lie most anteriorly at this point?

Thoracic duct

Phrenic nerve

Vagus nerve

Pulmonary artery

Pulmonary vein

The phrenic nerve lies anteriorly at the root of the right lung.
Please rate this question:

Discuss and give feedback


Next question

Lung anatomy

The right lung is composed of 3 lobes divided by the oblique and transverse fissures. The left lung
has two lobes divided by the oblique fissure.The apex of both lungs is approximately 4cm superior to
the sterno-costal joint of the first rib. Immediately below this is a sulcus created by the subclavian
artery.

Peripheral contact points of the lung

 Base: diaphragm
 Costal surface: corresponds to the cavity of the chest
 Mediastinal surface: Contacts the mediastinal pleura. Has the cardiac impression. Above and
behind this concavity is a triangular depression named the hilum, where the structures which
form the root of the lung enter and leave the viscus. These structures are invested by pleura,
which, below the hilum and behind the pericardial impression, forms the pulmonary ligament

Right lung
Above the hilum is the azygos vein; Superior to this is the groove for the superior vena cava and
right innominate vein; behind this, and nearer the apex, is a furrow for the innominate artery. Behind
the hilum and the attachment of the pulmonary ligament is a vertical groove for the oesophagus; In
front and to the right of the lower part of the oesophageal groove is a deep concavity for the
extrapericardiac portion of the inferior vena cava.

The root of the right lung lies behind the superior vena cava and the right atrium, and below the
azygos vein.

The right main bronchus is shorter, wider and more vertical than the left main bronchus and
therefore the route taken by most foreign bodies.

Image sourced from Wikipedia

Left lung
Above the hilum is the furrow produced by the aortic arch, and then superiorly the groove
accommodating the left subclavian artery; Behind the hilum and pulmonary ligament is a vertical
groove produced by the descending aorta, and in front of this, near the base of the lung, is the lower
part of the oesophagus.

The root of the left lung passes under the aortic arch and in front of the descending aorta.

Image sourced from Wikipedia


Inferior borders of both lungs

 6th rib in mid clavicular line


 8th rib in mid axillary line
 10th rib posteriorly

The pleura runs two ribs lower than the corresponding lung level.

Bronchopulmonary segments
Segment number Right lung Left lung

1 Apical Apical

2 Posterior Posterior

3 Anterior Anterior

4 Lateral Superior lingular

5 Medial Inferior lingular

6 Superior (apical) Superior (apical)

7 Medial basal Medial basal

8 Anterior basal Anterior basal

9 Lateral basal Lateral basal

10 Posterior basal Posterior basal

Next question
Question 385 of 560

An 18 year old boy is undergoing an appendicectomy for appendicitis. At which of the following
locations is the appendix most likely to be found?

Pre ileal

Pelvic

Retrocaecal

Post ileal

None of the above

Most appendixes lie in the retrocaecal position. If a retrocaecal appendix is difficult to remove then
mobilisation of the right colon significantly improves access.
Please rate this question:

Discuss and give feedback


Next question

Appendix

 Location: Base of caecum.


 Up to 10cm long.
 Mainly lymphoid tissue (Hence mesenteric adenitis may mimic appendicitis).
 Caecal taenia coli converge at base of appendix and form a longitudinal muscle cover over
the appendix. This convergence should facilitate its identification at surgery if it is retrocaecal
and difficult to find (which it can be when people start doing appendicectomies!)
 Arterial supply: Appendicular artery (branch of the ileocolic).
 It is intra peritoneal.

McBurney's point

 1/3 of the way along a line drawn from the Anterior Superior Iliac Spine to the Umbilicus
6 Positions:

 Retrocaecal 74%
 Pelvic 21%
 Postileal
 Subcaecal
 Paracaecal
 Preileal

Next question
Question 386 of 560

A 56 year old man is undergoing a pancreatectomy for carcinoma. During resection of the gland
which of the following structures will the surgeon not encounter posterior to the pancreas itself?

Left crus of the diaphragm

Superior mesenteric vein

Common bile duct

Portal vein

Gastroduodenal artery

Theme from 2010 Exam


The gastroduodenal artery divides into the gastro-epiploic and pancreaticoduodenal arteries at the
superior aspect of the pancreas.
Please rate this question:

Discuss and give feedback


Next question

Pancreas

The pancreas is a retroperitoneal organ and lies posterior to the stomach. It may be accessed
surgically by dividing the peritoneal reflection that connects the greater omentum to the transverse
colon. The pancreatic head sits in the curvature of the duodenum. Its tail lies close to the hilum of
the spleen, a site of potential injury during splenectomy.

Relations
Posterior to the pancreas
Pancreatic head Inferior vena cava
Common bile duct
Right and left renal veins
Superior mesenteric vein and artery
Pancreatic neck Superior mesenteric vein, portal vein

Pancreatic body- Left renal vein


Crus of diaphragm
Psoas muscle
Adrenal gland
Kidney
Aorta

Pancreatic tail Left kidney

Anterior to the pancreas


Pancreatic head 1st part of the duodenum
Pylorus
Gastroduodenal artery
SMA and SMV(uncinate process)

Pancreatic body Stomach


Duodenojejunal flexure

Pancreatic tail Splenic hilum

Superior to the pancreas


Coeliac trunk and its branches common hepatic artery and splenic artery

Grooves of the head of the pancreas


2nd and 3rd part of the duodenum

Arterial supply

 Head: pancreaticoduodenal artery


 Rest: splenic artery

Venous drainage

 Head: superior mesenteric vein


 Body and tail: splenic vein
Ampulla of Vater

 Merge of pancreatic duct and common bile duct


 Is an important landmark, halfway along the second part of the duodenum, that marks the
anatomical transition from foregut to midgut (also the site of transition between regions
supplied by coeliac trunk and SMA).

Image sourced from Wikipedia

Next question
Question 387 of 560

Which of the following bones is related to the cuboid's distal articular surface?

All metatarsals

5th metatarsal

Calcaneum

Medial cuneiform

3rd metatarsal

Theme from April 2012 Exam


The cuboid is located at the lateral aspect of the foot between the calcaneus posteriorly and the 4th
and 5th metatarsals distally.
Please rate this question:

Discuss and give feedback


Next question

Foot- anatomy

Arches of the foot


The foot is conventionally considered to have two arches.

 The longitudinal arch is higher on the medial than on the lateral side. The posterior part of
the calcaneum forms a posterior pillar to support the arch. The lateral part of this structure
passes via the cuboid bone and the lateral two metatarsal bones. The medial part of this
structure is more important. The head of the talus marks the summit of this arch, located
between the sustentaculum tali and the navicular bone. The anterior pillar of the medial arch
is composed of the navicular bone, the three cuneiforms and the medial three metatarsal
bones.
 The transverse arch is situated on the anterior part of the tarsus and the posterior part of the
metatarsus. The cuneiforms and metatarsal bases narrow inferiorly, which contributes to the
shape of the arch.

Intertarsal joints
Sub talar joint Formed by the cylindrical facet on the lower surface of the body of the
talus and the posterior facet on the upper surface of the calcaneus. The
facet on the talus is concave anteroposteriorly, the other is convex. The
synovial cavity of this joint does not communicate with any other joint.

Talocalcaneonavicular The anterior part of the socket is formed by the concave articular
joint surface of the navicular bone, posteriorly by the upper surface of the
sustentaculum tali. The talus sits within this socket

Calcaneocuboid joint Highest point in the lateral part of the longitudinal arch. The lower
aspect of this joint is reinforced by the long plantar and plantar
calcaneocuboid ligaments.

Transverse tarsal joint The talocalcaneonavicular joint and the calcaneocuboid joint extend
across the tarsus in an irregular transverse plane, between the talus and
calcaneus behind and the navicular and cuboid bones in front. This
plane is termed the transverse tarsal joint.

Cuneonavicular joint Formed between the convex anterior surface of the navicular bone and
the concave surface of the the posterior ends of the three cuneiforms.

Intercuneiform joints Between the three cuneiform bones.

Cuneocuboid joint Between the circular facets on the lateral cuneiform bone and the
cuboid. This joint contributes to the tarsal part of the transverse arch.

A detailed knowledge of the joints is not required for MRCS Part A. However, the contribution they
play to the overall structure of the foot should be appreciated

Ligaments of the ankle joint and foot


Image sourced from Wikipedia

Muscles of the foot


Muscle Origin Insertion Nerve Action
supply

Abductor Medial side of the calcaneus, Medial side of Medial Abducts the great toe
hallucis flexor retinaculum, plantar the base of the plantar
aponeurosis proximal nerve
phalanx

Flexor Medial process of the Via 4 tendons Medial Flexes all the joints of
digitorum calcaneus, plantar into the plantar the lateral 4 toes except
brevis eponeurosis. middle nerve for the interphalangeal
phalanges of joint.
the lateral 4
toes.

Abductor From the tubercle of the Together with Lateral Abducts the little toe at
digit calcaneus and from the flexor digit plantar the metatarsophalangeal
minimi plantar aponeurosis minimi brevis nerve joint
into the lateral
side of the
base of the
proximal
phalanx of the
little toe

Flexor From the medial side of the Into the Medial Flexes the
hallucis plantar surface of the cuboid proximal plantar metatarsophalangeal
brevis bone, from the adjacent part phalanx of the nerve joint of the great toe.
of the lateral cuneiform bone great toe, the
and from the tendon of tendon
tibialis posterior. contains a
sesamoid bone

Adductor Arises from two heads. The Lateral side of Lateral Adducts the great toe
hallucis oblique head arises from the the base of the plantar towards the second toe.
sheath of the peroneus longus proximal nerve Helps maintain the
tendon, and from the plantar phalanx of the transverse arch of the
surfaces of the bases of the great toe. foot.
2nd, 3rd and 4th metatarsal
bones. The transverse head
arises from the plantar
surface of the lateral 4
metatarsophalangeal joints
and from the deep transverse
metatarsal ligament.

Extensor On the dorsal surface of the Via four thin Deep Extend the
digitorum foot from the upper surface of tendons which peroneal metatarsophalangeal
brevis the calcaneus and its run forward joint of the medial four
associated fascia and medially toes. It is unable to
to be inserted extend the
into the interphalangeal joint
medial four without the assistance of
toes. The the lumbrical muscles.
lateral three
tendons join
with hoods of
extensor
digitorum
longus.

Nerves in the foot


Lateral plantar nerve
Passes anterolaterally towards the base of the 5th metatarsal between flexor digitorum brevis and
flexor accessorius. On the medial aspect of the lateral plantar artery. At the base of the 5th
metatarsal it splits into superficial and deep branches.

Medial plantar nerve


Passes forwards with the medial plantar artery under the cover of the flexor retinaculum to the
interval between abductor hallucis and flexor digitorum brevis on the sole of the foot.

Plantar arteries
Arise under the cover of the flexor retinaculum, midway between the tip of the medial malleolus and
the most prominent part of the medial side of the heel.

 Medial plantar artery. Passes forwards medial to medial plantar nerve in the space between
abductor hallucis and flexor digitorum brevis.Ends by uniting with a branch of the 1st plantar
metatarsal artery.
 Lateral plantar artery. Runs obliquely across the sole of the foot. It lies lateral to the lateral
plantar nerve. At the base of the 5th metatarsal bone it arches medially across the foot on
the metatarsals

Dorsalis pedis artery


This vessel is a direct continuation of the anterior tibial artery. It commences on the front of the ankle
joint and runs to the proximal end of the first metatarsal space. Here is gives off the arcuate artery
and continues forwards as the first dorsal metatarsal artery. It is accompanied by two veins
throughout its length. It is crossed by the extensor hallucis brevis
Next question
Question 388 of 560

From which structure is the central tendon of the diaphragm derived?

Septum transversum

Pleuroperitoneal folds

Diaphragmatic crura

Dorsal mesocardium

Oropharyngeal membrane

The septum transversum is a thick ridge of mesodermal tissue in the developing embryo that
separates the thoracic and abdominal cavities and forms the central tendon of the diaphragm.
Please rate this question:

Discuss and give feedback


Next question

Embryology of the diaphragm and diaphragmatic hernia

Embryology
The diaphragm is formed between the 5th and 7th weeks of gestation through the progressive fusion
of the septum transversum, pleuroperitoneal folds and via lateral muscular ingrowth. The muscular
origins of the diaphragm are somites located in cervical segments 3 to 5, which accounts for the long
path taken by the phrenic nerve. The components contribute to the following diaphragmatic
segments:

 Septum transversum - Central tendon


 Pleuroperitoneal membranes - Parietal membranes surrounding viscera
 Cervical somites C5 to C7 - Muscular component of the diaphragm

Diaphragmatic hernia
Type of hernia Features
Type of hernia Features

Morgagni Anteriorly located


Minimal compromise on lung development
Minimal signs on antenatal ultrasound
Usually present later
Usually good prognosis

Bochdalek hernia Posteriorly located


Larger defect
Often diagnosed antenatally
Associated with pulmonary hypoplasia
Poor prognosis

The posterior hernias of Bochdalek are the most common type and if not diagnosed antenatally will
typically present soon after birth with respiratory distress. The classical finding is that of a scaphoid
abdomen on clinical examination because of herniation of the abdominal contents into the chest.
Bochdalek hernias are associated with a number of chromosomal abnormalities such as Trisomy 21
and 18. Infants have considerable respiratory distress due to hypoplasia of the developing lung.
Historically this was considered to be due to direct compression of the lung by herniated viscera.
This view over simplifies the situation and the pulmonary hypoplasia occurs concomitantly with the
hernial development, rather than as a direct result of it. The pulmonary hypoplasia is associated with
pulmonary hypertension and abnormalities of pulmonary vasculature. The pulmonary hypertension
renders infants at risk of right to left shunting (resulting in progressive and worsening hypoxia).
Diagnostic work up of these infants includes chest x-rays/ abdominal ultrasound scans and cardiac
echo.
Surgery forms the mainstay of treatment and both thoracic and abdominal approaches may be
utilised. Following reduction of the hernial contents a careful search needs to be made for a hernial
sac as failure to recognise and correct this will result in a high recurrence rate. Smaller defects may
be primarily closed, larger defects may require a patch to close the defect. Malrotation of the viscera
is a recognised association and may require surgical correct at the same procedure (favoring an
abdominal approach).
The mortality rate is 50-75% and is related to the degree of lung compromise and age at
presentation (considerably better in infants >24 hours old).
Next question
Question 389 of 560

Where does Stensens duct primarily open?

Immediately lateral to the foramen caecum

Floor of mouth

Opposite the second molar tooth

Opposite the fifth molar tooth

Into the post nasal space

Stensens duct conveys secretions from the parotid gland and these enter the oral cavity at the level
of the second molar tooth.
Please rate this question:

Discuss and give feedback


Next question

Parotid gland

Anatomy of the parotid gland


Location Overlying the mandibular ramus; anterior and inferior to the ear.

Salivary duct Crosses the masseter, pierces the buccinator and drains adjacent to the
2nd upper molar tooth (Stensen's duct).

Structures passing  Facial nerve (Mnemonic: The Zebra Buggered My Cat; Temporal
through the gland Zygomatic, Buccal, Mandibular, Cervical)
 External carotid artery
 Retromandibular vein
 Auriculotemporal nerve
Relations  Anterior: masseter, medial pterygoid, superficial temporal and
maxillary artery, facial nerve, stylomandibular ligament
 Posterior: posterior belly digastric muscle, sternocleidomastoid,
stylohyoid, internal carotid artery, mastoid process, styloid
process

Arterial supply Branches of external carotid artery

Venous drainage Retromandibular vein

Lymphatic drainage Deep cervical nodes

Nerve innervation  Parasympathetic-Secretomotor


 Sympathetic-Superior cervical ganglion
 Sensory- Greater auricular nerve

Parasympathetic stimulation produces a water rich, serous saliva. Sympathetic stimulation leads to
the production of a low volume, enzyme-rich saliva.
Next question
Question 390 of 560

Which of the following nerves is responsible for the innervation of the posterior belly of the digastric
muscle?

Facial nerve

Hypoglossal nerve

Trigeminal nerve

Ansa cervicalis

Mylohoid nerve

The posterior belly of digastric is innervated by the facial nerve and the anterior belly by the
mylohoid nerve.
Please rate this question:

Discuss and give feedback


Next question

Anterior triangle of the neck

Boundaries

Anterior border of the Sternocleidomastoid


Lower border of mandible
Anterior midline

Sub triangles (divided by Digastric above and Omohyoid)

 Muscular triangle: Neck strap muscles


 Carotid triangle: Carotid sheath
 Submandibular Triangle (digastric)
Contents of the anterior triangle
Digastric triangle Submandibular gland
Submandibular nodes
Facial vessels
Hypoglossal nerve

Muscular triangle Strap muscles


External jugular vein

Carotid triangle Carotid sheath (Common carotid, vagus and internal jugular vein)
Ansa cervicalis

Nerve supply to digastric muscle

 Anterior: Mylohyoid nerve


 Posterior: Facial nerve

Image sourced from Wikipedia

Next question
Question 391 of 560

Which of the following carpal bones is a sesamoid bone in the tendon of flexor carpi ulnaris?

Triquetrum

Lunate

Pisiform

Scaphoid

Capitate

This small bone has a single articular facet. It projects from the triquetral bone at the ulnar aspect of
the wrist where most regard it as a sesamoid bone lying within the tendon of flexor carpi ulnaris.
Please rate this question:

Discuss and give feedback


Next question

Carpal bones

The wrist is comprised of 8 carpal bones, these are arranged in two rows of 4. It is convex from side
to side posteriorly and concave anteriorly.
Diagrammatic image of carpal bones
Image sourced from Wikipedia

Key to image
A Scaphoid

B Lunate

C Triquetrum

D Pisiform

E Trapezium

F Trapezoid

G Capitate

H Hamate

1 Radius
2 Ulna

3 Metacarpals

 No tendons attach to: Scaphoid, lunate, triquetrum (stabilised by ligaments)

Next question
Question 392 of 560

A 70 year old man falls and fractures his scaphoid bone. The fracture is displaced and the decision
is made to insert a screw to fix the fracture. Which of the following structures lies directly medial to
the scaphoid?

Lunate

Pisiform

Trapezoid

Trapezium

None of the above

The lunate lies medially in the anatomical plane. Fractures of the scaphoid that are associated with
high velocity injuries may cause associated lunate dislocation.

Please rate this question:

Discuss and give feedback

Next question

Scaphoid bone

The scaphoid has a concave articular surface for the head of the capitate and at the edge of this is a
crescentic surface for the corresponding area on the lunate.
Proximally, it has a wide convex articular surface with the radius. It has a distally sited tubercle that
can be palpated. The remaining articular surface is to the lateral side of the tubercle. It faces laterally
and is associated with the trapezium and trapezoid bones.

The narrow strip between the radial and trapezial surfaces and the tubercle gives rise to the radial
collateral carpal ligament. The tubercle receives part of the flexor retinaculum. This area is the only
part of the scaphoid that is available for the entry of blood vessels. It is commonly fractured and
avascular necrosis may result.

Scaphoid bone

Image sourced from Wikipedia

Next question
Question 393 of 560

A 55 year old man is admitted with a brisk haematemesis. He is taken to the endoscopy department
and an upper GI endoscopy is performed by the gastroenterologist. He identifies an ulcer on the
posterior duodenal wall and spends an eternity trying to control the bleeding with all the latest
haemostatic techniques. He eventually asks the surgeons for help. A laparotomy and anterior
duodenotomy are performed, as the surgeon opens the duodenum a vessel is spurting blood into the
duodenal lumen. From which of the following does this vessel arise?

Left gastric artery

Common hepatic artery

Right hepatic artery

Superior mesenteric artery

Splenic artery

The vessel will be the gastroduodenal artery, this arises from the common hepatic artery.

Please rate this question:

Discuss and give feedback

Next question

Gastroduodenal artery

Supplies
Pylorus, proximal part of the duodenum, and indirectly to the pancreatic head (via the anterior and
posterior superior pancreaticoduodenal arteries)

Path
Most commonly arises from the common hepatic artery of the coeliac trunk
Terminates by bifurcating into the right gastroepiploic artery and the superior pancreaticoduodenal
artery

Image showing stomach reflected superiorly to illustrate the relationship of the gastroduodenal artery
to the first part of the duodenum

Image sourced from Wikipedia

Next question
Question 394 of 560

A 73 year old lady is hit by a car. She suffers a complex fracture of the distal aspect of her humerus
with associated injury to the radial nerve. Which of the following movements will be most impaired as
a result?

Elbow extension

Elbow flexion

Shoulder abduction

Wrist extension

None of the above

The triceps will not be affected so elbow extension will be preserved. Loss of wrist extension will be
the most obvious effect.
Please rate this question:

Discuss and give feedback


Next question

Radial nerve

Continuation of posterior cord of the brachial plexus (root values C5 to T1)

Path

 In the axilla: lies posterior to the axillary artery on subscapularis, latissimus dorsi and teres
major.
 Enters the arm between the brachial artery and the long head of triceps (medial to humerus).
 Spirals around the posterior surface of the humerus in the groove for the radial nerve.
 At the distal third of the lateral border of the humerus it then pierces the intermuscular
septum and descends in front of the lateral epicondyle.
 At the lateral epicondyle it lies deeply between brachialis and brachioradialis where it then
divides into a superficial and deep terminal branch.
 Deep branch crosses the supinator to become the posterior interosseous nerve.
In the image below the relationships of the radial nerve can be appreciated

Image sourced from Wikipedia

Regions innervated
 Triceps
 Anconeus
Motor (main nerve)  Brachioradialis
 Extensor carpi radialis

 Supinator
 Extensor carpi ulnaris
 Extensor digitorum
Motor (posterior  Extensor indicis
interosseous branch)  Extensor digiti minimi
 Extensor pollicis longus and brevis
 Abductor pollicis longus

The area of skin supplying the proximal phalanges on the dorsal aspect of the
Sensory hand is supplied by the radial nerve (this does not apply to the little finger and
part of the ring finger)

Muscular innervation and effect of denervation


Anatomical
location Muscle affected Effect of paralysis

Shoulder Long head of triceps Minor effects on shoulder stability in abduction

Arm Triceps Loss of elbow extension

Forearm Supinator Weakening of supination of prone hand and


Brachioradialis elbow flexion in mid prone position
Extensor carpi radialis
longus and brevis

The cutaneous sensation of the upper limb- illustrating the contribution of the radial nerve

Image sourced from Wikipedia

Next question
Question 395 of 560

Which of the following is not a content of the rectus sheath?

Pyramidalis

Superior epigastric artery

Inferior epigastric vein

Internal iliac artery

Rectus abdominis

The rectus sheath also contains:


superior epigastric vein
inferior epigastric artery
Please rate this question:

Discuss and give feedback


Next question

Abdominal wall

The 2 main muscles of the abdominal wall are the rectus abdominis (anterior) and the quadratus
lumborum (posterior).
The remaining abdominal wall consists of 3 muscular layers. Each muscle passes from the lateral
aspect of the quadratus lumborum posteriorly to the lateral margin of the rectus sheath anteriorly.
Each layer is muscular posterolaterally and aponeurotic anteriorly.

Image sourced from Wikipedia


Muscles of abdominal wall
External  Lies most superficially
oblique  Originates from 5th to 12th ribs
 Inserts into the anterior half of the outer aspect of the iliac crest, linea
alba and pubic tubercle
 More medially and superiorly to the arcuate line, the aponeurotic layer
overlaps the rectus abdominis muscle
 The lower border forms the inguinal ligament
 The triangular expansion of the medial end of the inguinal ligament is
the lacunar ligament.

Internal  Arises from the thoracolumbar fascia, the anterior 2/3 of the iliac crest
oblique and the lateral 2/3 of the inguinal ligament
 The muscle sweeps upwards to insert into the cartilages of the lower 3
ribs
 The lower fibres form an aponeurosis that runs from the tenth costal
cartilage to the body of the pubis
 At its lowermost aspect it joins the fibres of the aponeurosis of
transversus abdominis to form the conjoint tendon.

Transversus  Innermost muscle


abdominis  Arises from the inner aspect of the costal cartilages of the lower 6 ribs ,
from the anterior 2/3 of the iliac crest and lateral 1/3 of the inguinal
ligament
 Its fibres run horizontally around the abdominal wall ending in an
aponeurosis. The upper part runs posterior to the rectus abdominis.
Lower down the fibres run anteriorly only.
 The rectus abdominis lies medially; running from the pubic crest and
symphysis to insert into the xiphoid process and 5th, 6th and 7th costal
cartilages. The muscles lies in a aponeurosis as described above.
 Nerve supply: anterior primary rami of T7-12

Surgical notes
During abdominal surgery it is usually necessary to divide either the muscles or their aponeuroses.
During a midline laparotomy it is desirable to divide the aponeurosis. This will leave the rectus
sheath intact above the arcuate line and the muscles intact below it. Straying off the midline will
often lead to damage to the rectus muscles, particularly below the arcuate line where they may often
be in close proximity to each other.
Next question
Question 396 of 560

Which of the following vessels does not drain directly into the inferior vena cava?

Superior mesenteric vein

Right common iliac

Right hepatic vein

Left hepatic vein

Right testicular vein

The superior mesenteric vein drains into the portal vein. The right and left hepatic veins drain into it
directly, this can account for major bleeding in more extensive liver shearing type injuries.
Please rate this question:

Discuss and give feedback


Next question

Inferior vena cava

Origin

 L5

Path

 Left and right common iliac veins merge to form the IVC.
 Passes right of midline
 Paired segmental lumbar veins drain into the IVC throughout its length
 The right gonadal vein empties directly into the cava and the left gonadal vein generally
empties into the left renal vein.
 The next major veins are the renal veins and the hepatic veins
 Pierces the central tendon of diaphragm at T8
 Right atrium
Image sourced from Wikipedia

Relations
Anteriorly Small bowel, first and third part of duodenum, head of pancreas, liver and bile duct,
right common iliac artery, right gonadal artery

Posteriorly Right renal artery, right psoas, right sympathetic chain, coeliac ganglion

Levels
Level Vein

T8 Hepatic vein, inferior phrenic vein, pierces diaphragm

L1 Suprarenal veins, renal vein

L2 Gonadal vein

L1-5 Lumbar veins


L5 Common iliac vein, formation of IVC

Next question
Question 397 of 560

A 17 year old male has a suspected testicular torsion and the scrotum is to be explored surgically.
The surgeon incises the skin and then the dartos muscle. What is the next tissue layer that will be
encountered during the dissection?

Visceral layer of the tunica vaginalis

Cremasteric fascia

Parietal layer of the tunica vaginalis

External spermatic fascia

Internal spermatic fascia

The layers that will be encountered are (in order):


1. Skin
2. Dartos fascia and muscle
3. External spermatic fascia
4. Cremasteric muscle and fascia
5. Internal spermatic fascia
6. Parietal layer of the tunica vaginalis

The layers of the spermatic cord and scrotum are a popular topic in the MRCS exam.
Please rate this question:

Discuss and give feedback


Next question

Scrotal and testicular anatomy

Spermatic cord
Formed by the vas deferens and is covered by the following structures:
Layer Origin

Internal spermatic fascia Transversalis fascia


Layer Origin

Cremasteric fascia From the fascial coverings of internal oblique

External spermatic fascia External oblique aponeurosis

Contents of the cord


Vas deferens Transmits sperm and accessory gland secretions

Testicular artery Branch of abdominal aorta supplies testis and


epididymis

Artery of vas deferens Arises from inferior vesical artery

Cremasteric artery Arises from inferior epigastric artery

Pampiniform plexus Venous plexus, drains into right or left testicular vein

Sympathetic nerve fibres Lie on arteries, the parasympathetic fibres lie on the
vas

Genital branch of the genitofemoral Supplies cremaster


nerve

Lymphatic vessels Drain to lumbar and para-aortic nodes

Scrotum

 Composed of skin and closely attached dartos fascia.


 Arterial supply from the anterior and posterior scrotal arteries
 Lymphatic drainage to the inguinal lymph nodes
 Parietal layer of the tunica vaginalis is the innermost layer
Testes

 The testes are surrounded by the tunica vaginalis (closed peritoneal sac). The parietal layer
of the tunica vaginalis adjacent to the internal spermatic fascia.
 The testicular arteries arise from the aorta immediately inferiorly to the renal arteries.
 The pampiniform plexus drains into the testicular veins, the left drains into the left renal vein
and the right into the inferior vena cava.
 Lymphatic drainage is to the para-aortic nodes.

Next question
Question 398 of 560

A 25 year old man is stabbed in the groin and the area, which lies within the femoral triangle is
explored. Which structure forms the lateral wall of the femoral triangle?

Adductor longus

Pectineus

Adductor magnus

Sartorius

Conjoint tendon

The sartorius forms the lateral wall of the femoral triangle (see below).
Please rate this question:

Discuss and give feedback


Next question

Femoral triangle anatomy

Boundaries
Superiorly Inguinal ligament

Laterally Sartorius

Medially Adductor longus

Floor Iliopsoas, adductor longus and pectineus

Roof  Fascia lata and Superficial fascia


 Superficial inguinal lymph nodes (palpable below the inguinal ligament)
 Long saphenous vein

Image sourced from Wikipedia

Contents

 Femoral vein (medial to lateral)


 Femoral artery-pulse palpated at the mid inguinal point
 Femoral nerve
 Deep and superficial inguinal lymph nodes
 Lateral cutaneous nerve
 Great saphenous vein
 Femoral branch of the genitofemoral nerve

Next question
Question 399 of 560

An 18 year old man develops a severe spreading sepsis of the hand. The palm is explored surgically
and the flexor digiti minimi brevis muscle is mobilised to facilitate drainage of the infection. Which of
the following structures is not closely related to this muscle?

The hook of hamate

Median nerve

Superficial palmar arterial arch

Digital nerves arising from the ulnar nerve

None of the above

The flexor digiti minimi brevis originates from the Hamate, on its under- surface lie the ulnar
contribution to the superficial palmar arterial arch and digital nerves derived from the ulnar nerve.
The median nerve overlies the flexor tendons.
Please rate this question:

Discuss and give feedback


Next question

Hand

Anatomy of the hand


Bones  8 Carpal bones
 5 Metacarpals
 14 phalanges

Intrinsic Muscles 7 Interossei - Supplied by ulnar nerve

 3 palmar-adduct fingers
 4 dorsal- abduct fingers
Intrinsic muscles Lumbricals

 Flex MCPJ and extend the IPJ.


 Origin deep flexor tendon and insertion dorsal extensor hood
mechanism.
 Innervation: 1st and 2nd- median nerve, 3rd and 4th- deep branch of
the ulnar nerve.

Thenar eminence  Abductor pollicis brevis


 Opponens pollicis
 Flexor pollicis brevis

Hypothenar  Opponens digiti minimi


eminence  Flexor digiti minimi brevis
 Abductor digiti minimi

Image sourced from Wikipedia

Fascia and compartments of the palm


The fascia of the palm is continuous with the antebrachial fascia and the fascia of the dorsum of the
hand. The palmar fascia is thin over the thenar and hypothenar eminences. In contrast the palmar
fascia is relatively thick. The palmar aponeurosis covers the soft tissues and overlies the flexor
tendons. The apex of the palmar aponeurosis is continuous with the flexor retinaculum and the
palmaris longus tendon. Distally, it forms four longitudinal digital bands that attach to the bases of
the proximal phalanges, blending with the fibrous digital sheaths.
A medial fibrous septum extends deeply from the medial border of the palmar aponeurosis to the 5th
metacarpal. Lying medial to this are the hypothenar muscles. In a similar fashion, a lateral fibrous
septum extends deeply from the lateral border of the palmar aponeurosis to the 3rd metacarpal. The
thenar compartment lies lateral to this area.
Lying between the thenar and hypothenar compartments is the central compartment. It contains the
flexor tendons and their sheaths, the lumbricals, the superficial palmar arterial arch and the digital
vessels and nerves.
The deepest muscular plane is the adductor compartment, which contains adductor pollicis.

Short muscles of the hand


These comprise the lumbricals and interossei. The four slender lumbrical muscles flex the fingers at
the metacarpophalangeal joints and extend the interphalangeal joint. The four dorsal interossei are
located between the metacarpals and the four palmar interossei lie on the palmar surface of the
metacarpals in the interosseous compartment of the hand.

Long flexor tendons and sheaths in the hand


The tendons of FDS and FDP enter the common flexor sheath deep to the flexor retinaculum. The
tendons enter the central compartment of the hand and fan out to their respective digital synovial
sheaths. Near the base of the proximal phalanx, the tendon of FDS splits to permit the passage of
FDP. The FDP tendons are attached to the margins of the anterior aspect of the base of the distal
phalanx.
The fibrous digital sheaths contain the flexor tendons and their synovial sheaths. These extend from
the heads of the metacarpals to the base of the distal phalanges.
Next question
Question 400 of 560

A 19 year old man undergoes an open inguinal hernia repair. The cord is mobilised and the deep
inguinal ring identified. Which of the following structures forms its lateral wall?

External oblique aponeurosis

Transversalis fascia

Conjoint tendon

Inferior epigastric artery

Inferior epigastric vein

The transversalis fascia forms the superolateral edge of the deep inguinal ring. The epigastric
vessels form its inferomedial wall.
Please rate this question:

Discuss and give feedback


Next question

Inguinal canal

Location

 Above the inguinal ligament


 The inguinal canal is 4cm long
 The superficial ring is located anterior to the pubic tubercle
 The deep ring is located approximately 1.5-2cm above the half way point between the
anterior superior iliac spine and the pubic tubercle

Boundaries of the inguinal canal


Floor  External oblique aponeurosis
 Inguinal ligament
 Lacunar ligament
Roof  Internal oblique
 Transversus abdominis

Anterior wall External oblique aponeurosis

Posterior wall  Transversalis fascia


 Conjoint tendon

Laterally  Internal ring


 Fibres of internal oblique

Medially  External ring


 Conjoint tendon

Contents
Males Spermatic cord and ilioinguinal As it passes through the canal the spermatic cord
nerve has 3 coverings:

 External spermatic fascia from external


oblique aponeurosis
 Cremasteric fascia
 Internal spermatic fascia

Females Round ligament of uterus and


ilioinguinal nerve

Related anatomy of the inguinal region


The boundaries of Hesselbachs triangle are commonly tested and illustrated below:
Image sourced from Wikipedia

The image below demonstrates the close relationship of the vessels to the lower limb with the
inguinal canal. A fact to be borne in mind when repairing hernial defects in this region.

Image sourced from Wikipedia

Next question
Question 401 of 560

A 22 year old man develops an infection in the pulp of his little finger. What is the most proximal site
to which this infection may migrate?

The metacarpophalangeal joint

The distal interphalangeal joint

The proximal interphalangeal joint

Proximal to the flexor retinaculum

Immediately distal to the carpal tunnel

The 5th tendon sheath extends from the little finger to the proximal aspect of the carpal tunnel. This
carries a significant risk of allowing infections to migrate proximally.
Please rate this question:

Discuss and give feedback


Next question

Hand

Anatomy of the hand


Bones  8 Carpal bones
 5 Metacarpals
 14 phalanges

Intrinsic Muscles 7 Interossei - Supplied by ulnar nerve

 3 palmar-adduct fingers
 4 dorsal- abduct fingers

Intrinsic muscles Lumbricals


 Flex MCPJ and extend the IPJ.
 Origin deep flexor tendon and insertion dorsal extensor hood
mechanism.
 Innervation: 1st and 2nd- median nerve, 3rd and 4th- deep branch of
the ulnar nerve.

Thenar eminence  Abductor pollicis brevis


 Opponens pollicis
 Flexor pollicis brevis

Hypothenar  Opponens digiti minimi


eminence  Flexor digiti minimi brevis
 Abductor digiti minimi

Image sourced from Wikipedia

Fascia and compartments of the palm


The fascia of the palm is continuous with the antebrachial fascia and the fascia of the dorsum of the
hand. The palmar fascia is thin over the thenar and hypothenar eminences. In contrast the palmar
fascia is relatively thick. The palmar aponeurosis covers the soft tissues and overlies the flexor
tendons. The apex of the palmar aponeurosis is continuous with the flexor retinaculum and the
palmaris longus tendon. Distally, it forms four longitudinal digital bands that attach to the bases of
the proximal phalanges, blending with the fibrous digital sheaths.
A medial fibrous septum extends deeply from the medial border of the palmar aponeurosis to the 5th
metacarpal. Lying medial to this are the hypothenar muscles. In a similar fashion, a lateral fibrous
septum extends deeply from the lateral border of the palmar aponeurosis to the 3rd metacarpal. The
thenar compartment lies lateral to this area.
Lying between the thenar and hypothenar compartments is the central compartment. It contains the
flexor tendons and their sheaths, the lumbricals, the superficial palmar arterial arch and the digital
vessels and nerves.
The deepest muscular plane is the adductor compartment, which contains adductor pollicis.

Short muscles of the hand


These comprise the lumbricals and interossei. The four slender lumbrical muscles flex the fingers at
the metacarpophalangeal joints and extend the interphalangeal joint. The four dorsal interossei are
located between the metacarpals and the four palmar interossei lie on the palmar surface of the
metacarpals in the interosseous compartment of the hand.

Long flexor tendons and sheaths in the hand


The tendons of FDS and FDP enter the common flexor sheath deep to the flexor retinaculum. The
tendons enter the central compartment of the hand and fan out to their respective digital synovial
sheaths. Near the base of the proximal phalanx, the tendon of FDS splits to permit the passage of
FDP. The FDP tendons are attached to the margins of the anterior aspect of the base of the distal
phalanx.
The fibrous digital sheaths contain the flexor tendons and their synovial sheaths. These extend from
the heads of the metacarpals to the base of the distal phalanges.
Next question
Question 402 of 560

Which of the following muscles is not innervated by the deep branch of the ulnar nerve?

Adductor pollicis

Hypothenar muscles

All the interosseous muscles

Opponens pollicis

Third and fourth lumbricals

Please rate this question:

Discuss and give feedback


Next question

Ulnar nerve

Origin

 C8, T1

Supplies (no muscles in the upper arm)

 Flexor carpi ulnaris


 Flexor digitorum profundus
 Flexor digiti minimi
 Abductor digiti minimi
 Opponens digiti minimi
 Adductor pollicis
 Interossei muscle
 Third and fourth lumbricals
 Palmaris brevis
Path

 Posteromedial aspect of upper arm to flexor compartment of forearm, then along the ulnar.
Passes beneath the flexor carpi ulnaris muscle, then superficially through the flexor
retinaculum into the palm of the hand.

Image sourced from Wikipedia

Branches
Branch Supplies

Muscular branch Flexor carpi ulnaris


Medial half of the flexor digitorum profundus
Branch Supplies

Palmar cutaneous branch (Arises near the Skin on the medial part of the palm
middle of the forearm)

Dorsal cutaneous branch Dorsal surface of the medial part of the hand

Superficial branch Cutaneous fibres to the anterior surfaces of the


medial one and one-half digits

Deep branch Hypothenar muscles


All the interosseous muscles
Third and fourth lumbricals
Adductor pollicis
Medial head of the flexor pollicis brevis

Effects of injury
Damage at the wrist  Wasting and paralysis of intrinsic hand muscles (claw hand)
 Wasting and paralysis of hypothenar muscles
 Loss of sensation medial 1 and half fingers

Damage at the elbow  Radial deviation of the wrist


 Clawing less in 4th and 5th digits

Next question
Question 403 of 560

During an inguinal hernia repair the surgeon identifies a small nerve whilst mobilising the cord
structures at the level of the superficial inguinal ring. Which nerve is this most likely to be?

Subcostal

Iliohypogastric

Ilioinguinal

Obturator

Pudendal

Ilioinguinal nerve entrapment may be a cause of neuropathic pain following inguinal hernia surgery.

The ilioinguinal nerve passes through the superfical inguinal ring and is routinely encountered when
exploring the inguinal canal during hernia surgery. The iliohypogastric nerve pierces the aponeurosis
of the external oblique muscle superior to the superficial inguinal ring.
Please rate this question:

Discuss and give feedback


Next question

Ilioinguinal nerve

Arises from the first lumbar ventral ramus with the iliohypogastric nerve. It passes inferolaterally
through the substance of psoas major and over the anterior surface of quadratus lumborum. It
pierces the internal oblique muscle and passes deep to the aponeurosis of the external oblique
muscle. It enters the inguinal canal and then passes through the superficial inguinal ring to reach the
skin.

Branches

 To supply those muscles of the abdominal wall through which it passes.


 Skin and fascia over the pubic symphysis, superomedial part of the femoral triangle, surface
of the scrotum, root and dorsum of penis or labum majus in females.

Next question
Question 404 of 560

From which of the following structures does the anterior cruciate ligament originate?

Posterolateral aspect of the lateral femoral condyle

Anterior intercondylar area of tibia

Posterolateral aspect of the medial femoral condyle

Posteromedial aspect of the medial femoral condyle

Posterior intercondylar area of tibia

The anterior cruciate ligament is attached to the anterior intercondylar area of the tibia. Is then
passes posterolaterally to insert into the posteromedial aspect of the lateral femoral condyle.
Please rate this question:

Discuss and give feedback


Next question

Knee joint

The knee joint is a synovial joint, the largest and most complicated. It consists of two condylar joints
between the femur and tibia and a sellar joint between the patella and the femur. The tibiofemoral
articular surfaces are incongruent, however, this is improved by the presence of the menisci. The
degree of congruence is related to the anatomical position of the knee joint and is greatest in full
extension.

Knee joint compartments


 Comprised of the patella/femur joint, lateral and medial compartments
(between femur condyles and tibia)
Tibiofemoral  Synovial membrane and cruciate ligaments partially separate the medial
and lateral compartments

 Ligamentum patellae
Patellofemoral  Actions: provides joint stability in full extension
Fibrous capsule
The capsule of the knee joint is a complex, composite structure with contributions from adjacent
tendons.
Anterior The capsule does not pass proximal to the patella. It blends with the tendinous
fibres expansions of vastus medialis and lateralis

Posterior These fibres are vertical and run from the posterior surface of the femoral condyles
fibres to the posterior aspect of the tibial condyle

Attach to the femoral and tibial condyles beyond their articular margins, blending
Medial fibres
with the tibial collateral ligament

Lateral Attach to the femur superior to popliteus, pass over its tendon to head of fibula and
fibres tibial condyle

Bursae
 Subcutaneous prepatellar bursa; between patella and skin
 Deep infrapatellar bursa; between tibia and patellar ligament
Anterior
 Subcutaneous infrapatellar bursa; between distal tibial tuberosity and skin

 Bursa between lateral head of gastrocnemius and joint capsule


 Bursa between fibular collateral ligament and tendon of biceps femoris
Laterally
 Bursa between fibular collateral ligament and tendon of popliteus

 Bursa between medial head of gastrocnemius and the fibrous capsule


 Bursa between tibial collateral ligament and tendons of sartorius, gracilis and
semitendinosus
Medially
 Bursa between the tendon of semimembranosus and medial tibial condyle and
medial head of gastrocnemius

Posterior Highly variable and inconsistent

Ligaments
Medial collateral Medial epicondyle femur to medial tibial condyle: valgus stability
ligament

Lateral collateral Lateral epicondyle femur to fibula head: varus stability


ligament

Anterior cruciate Anterior tibia to lateral intercondylar notch femur: prevents tibia sliding
ligament anteriorly

Posterior cruciate Posterior tibia to medial intercondylar notch femur: prevents tibia
ligament sliding posteriorly

Patellar ligament Central band of the tendon of quadriceps femoris, extends from patella
to tibial tuberosity

Image sourced from Wikipedia


Image sourced from Wikipedia

© Image provided by the University of Sheffield

Menisci
Medial and lateral menisci compensate for the incongruence of the femoral and tibial condyles.
Composed of fibrous tissue.
Medial meniscus is attached to the tibial collateral ligament.
Lateral meniscus is attached to the loose fibres at the lateral edge of the joint and is separate from
the fibular collateral ligament. The lateral meniscus is crossed by the popliteus tendon.
Nerve supply
The knee joint is supplied by the femoral, tibial and common peroneal divisions of the sciatic and by
a branch from the obturator nerve. Hip pathology pain may be referred to the knee.

Blood supply
Genicular branches of the femoral artery, popliteal and anterior tibial arteries all supply the knee
joint.
Next question
Question 405 of 560

During an arch aortogram the brachiocephalic artery is entered with an angiography catheter. The
radiologist continues to advance the catheter. Into which of the following vessels is it likely to enter?

Left subclavian artery

Left axillary artery

Right subclavian artery

Right axillary artery

None of the above

The axillary artery is a branch of the subclavian artery and although developmental anomalies may
occur they are rare. The catheter may also enter the right carotid. There is no brachiocephalic artery
on the left side.
Please rate this question:

Discuss and give feedback


Next question

Brachiocephalic artery

The brachiocephalic artery is the largest branch of the aortic arch. From its aortic origin it ascends
superiorly, it initially lies anterior to the trachea and then on its right hand side. It branches into the
common carotid and right subclavian arteries at the level of the sternoclavicular joint.

Path
Origin- apex of the midline of the aortic arch
Passes superiorly and posteriorly to the right
Divides into the right subclavian and right common carotid artery

Relations
Anterior  Sternohyoid
 Sternothyroid
 Thymic remnants
 Left brachiocephalic vein
 Right inferior thyroid veins

Posterior  Trachea
 Right pleura

Right lateral  Right brachiocephalic vein


 Superior part of SVC

Left lateral  Thymic remnants


 Origin of left common carotid
 Inferior thyroid veins
 Trachea (higher level)

Branches
Normally none but may have the thyroidea ima artery

Image sourced from Wikipedia

Next question
Question 406 of 560

Which of the following structures lie between the lateral and medial heads of the triceps muscle?

Radial nerve

Median nerve

Ulnar nerve

Axillary nerve

Medial cutaneous nerve of the forearm

The radial nerve runs in its groove on between the two heads. The ulnar nerve lies anterior to the
medial head. The axillary nerve passes through the quadrangular space. This lies superior to lateral
head of the triceps muscle and thus the lateral border of the quadrangular space is the humerus.
Therefore the correct answer is the radial nerve.
Please rate this question:

Discuss and give feedback


Next question

Triceps

Origin  Long head- infraglenoid tubercle of the scapula.


 Lateral head- dorsal surface of the humerus, lateral and proximal to the
groove of the radial nerve
 Medial head- posterior surface of the humerus on the inferomedial side of
the radial groove and both of the intermuscular septae

Insertion  Olecranon process of the ulna. Here the olecranon bursa is between the
triceps tendon and olecranon.
 Some fibres insert to the deep fascia of the forearm, posterior capsule of the
elbow (preventing the capsule from being trapped between olecranon and
olecranon fossa during extension)
Innervation Radial nerve

Blood Profunda brachii artery


supply

Action Elbow extension. The long head can adduct the humerus and and extend it from a
flexed position

Relations The radial nerve and profunda brachii vessels lie between the lateral and medial
heads
Next question
Question 407 of 560

Into which of the following structures does the superior part of the fibrous capsule of the shoulder
joint insert?

The surgical neck of the humerus

The body of the humerus

The bicipital groove

Immediately distal to the greater tuberosity

The anatomical neck of the humerus

The shoulder joint is a shallow joint, hence its great mobility. However, this comes at the expense of
stability. The fibrous capsule attaches to the anatomical neck superiorly and the surgical neck
inferiorly
Please rate this question:

Discuss and give feedback


Next question

Shoulder joint

 Shallow synovial ball and socket type of joint.


 It is an inherently unstable joint, but is capable to a wide range of movement.
 Stability is provided by muscles of the rotator cuff that pass from the scapula to insert in the
greater tuberosity (all except sub scapularis-lesser tuberosity).

Glenoid labrum

 Fibrocartilaginous rim attached to the free edge of the glenoid cavity


 Tendon of the long head of biceps arises from within the joint from the supraglenoid tubercle,
and is fused at this point to the labrum.
 The long head of triceps attaches to the infraglenoid tubercle
Fibrous capsule

 Attaches to the scapula external to the glenoid labrum and to the labrum itself (postero-
superiorly)
 Attaches to the humerus at the level of the anatomical neck superiorly and the surgical neck
inferiorly
 Anteriorly the capsule is in contact with the tendon of subscapularis, superiorly with the
supraspinatus tendon, and posteriorly with the tendons of infraspinatus and teres minor. All
these blend with the capsule towards their insertion.
 Two defects in the fibrous capsule; superiorly for the tendon of biceps. Anteriorly there is a
defect beneath the subscapularis tendon.
 The inferior extension of the capsule is closely related to the axillary nerve at the surgical
neck and this nerve is at risk in anteroinferior dislocations. It also means that proximally sited
osteomyelitis may progress to septic arthritis.

Movements and muscles


Flexion Anterior part of deltoid
Pectoralis major
Biceps
Coracobrachialis

Extension Posterior deltoid


Teres major
Latissimus dorsi

Adduction Pectoralis major


Latissimus dorsi
Teres major
Coracobrachialis

Abduction Mid deltoid


Supraspinatus

Medial rotation Subscapularis


Anterior deltoid
Teres major
Latissimus dorsi
Lateral rotation Posterior deltoid
Infraspinatus
Teres minor

Important anatomical relations


Anteriorly Brachial plexus
Axillary artery and vein

Posterior Suprascapular nerve


Suprascapular vessels

Inferior Axillary nerve


Circumflex humeral vessels

Next question
Question 408 of 560

A 34 year old lady presents with symptoms of faecal incontinence. Ten years previously she gave
birth to a child by normal vaginal delivery. Injury to which of the following nerves is most likely to
account for this process?

Genitofemoral

Ilioinguinal

Pudendal

Hypogastric autonomic nerve

Obturator

S2,3,4 keeps the poo up off the floor - POOdendal nerve

Theme from April 2013 Exam


Damage to the pudendal nerve is classically associated with faecal incontinence and it is for this
reason that sacral neuromodulation is a popular treatment for the condition. Injury to the hypogastric
autonomic nerves is an aetiological factor in the development of constipation.

Please rate this question:

Discuss and give feedback

Next question

Pudendal nerve

The pudendal nerve arises from nerve roots S2, S3 and S4 and exits the pelvis through the greater
sciatic foramen. It re-enters the perineum through the lesser sciatic foramen. It travels inferior to give
innervation to the anal sphincters and external urethral sphincter. It also provides cutaneous
innervation to the region of perineum surrounding the anus and posterior vulva.

Traction and compression of the pudendal nerve by the foetus in late pregnancy may result in late
onset pudendal neuropathy which may be part of the process involved in the development of faecal
incontinence.

Next question
Question 409 of 560

During a difficult thyroidectomy haemorrhage is noted from the thyroidea ima vessel. From which
structure does this vessel usually arise?

External carotid artery

Internal carotid artery

Brachiocephalic artery

Axillary artery

Superior thyroid artery

Rhyme isthmus location:

Rings 2,3,4 make the isthmus floor

This accessory vessel which usually lies at the inferior aspect of the gland is derived either from the
brachiocephalic artery or the arch of the aorta.
Please rate this question:

Discuss and give feedback


Next question

Thyroid gland

 Right and left lobes connected by isthmus


 Surrounded by sheath from pretracheal layer of deep fascia
 Apex: Lamina of thyroid cartilage
 Base: 4th-5th tracheal ring
 Pyramidal lobe: from isthmus
 May be attached to foramen caecum at the base of the tongue

Relations
Anteromedially  Sternothyroid
 Superior belly of omohyoid
 Sternohyoid
 Anterior aspect of sternocleidomastoid

Posterolaterally Carotid sheath

Medially  Larynx
 Trachea
 Pharynx
 Oesophagus
 Cricothyroid muscle
 External laryngeal nerve (near superior thyroid artery)
 Recurrent laryngeal nerve (near inferior thyroid artery)

Posterior  Parathyroid glands


 Anastomosis of superior and inferior thyroid arteries

Isthmus  Anteriorly: Sternothyroids, sternohyoids, anterior jugular veins


 Posteriorly: 2nd, 3rd, 4th tracheal rings (attached via Ligament of
Berry)

Blood Supply
Arterial  Superior thyroid artery (1st branch of external carotid)
 Inferior thyroid artery (from thyrocervical trunk)
 Thyroidea ima (in 10% of population -from brachiocephalic artery or aorta)

Venous  Superior and middle thyroid veins - into the IJV


 Inferior thyroid vein - into the brachiocephalic veins

Next question
Question 410 of 560

A 49 year old man undergoes a low anterior resection for cancer. He is assessed in the outpatient
clinic post operatively. His wounds are well healed. However, he complains of impotence. Which of
the following best explains this problem?

Sciatic nerve injury

Damage to the internal iliac artery

Damage to the nervi erigentes

Damage to the vas

Damage to the genitofemoral nerve

Theme from 2012 Exam


The penis takes autonomic nerves from the nervi erigentes that lie near the seminal vesicles. These
may be compromised by direct surgical trauma (such as use of diathermy in this area) and also by
radiotherapy that is used in these patients pre operatively. The result is that up to 50% of patients
may develop impotence following rectal cancer surgery.
Please rate this question:

Discuss and give feedback


Next question

Penile erection

Physiology of erection
Autonomic  Sympathetic nerves originate from T11-L2 and parasympathetic nerves
from S2-4 join to form pelvic plexus.
 Parasympathetic discharge causes erection, sympathetic discharge causes
ejaculation and detumescence.

Somatic Supplied by dorsal penile and pudendal nerves. Efferent signals are relayed from
nerves Onufs nucleus (S2-4) to innervate ischiocavernosus and bulbocavernosus muscles.

Autonomic discharge to the penis will trigger the veno-occlusive mechanism which triggers the flow
of arterial blood into the penile sinusoidal spaces. As the inflow increases the increased volume in
this space will secondarily lead to compression of the subtunical venous plexus with reduced venous
return. During the detumesence phase the arteriolar constriction will reduce arterial inflow and
thereby allow venous return to normalise.

Priapism
Prolonged unwanted erection, in the absence of sexual desire, lasting more than 4 hours.

Classification of priaprism
Low flow priaprism Due to veno-occlusion (high intracavernosal pressures).

 Most common type


 Often painful
 Often low cavernosal flow
 If present for >4 hours requires emergency treatment

High flow priaprism Due to unregulated arterial blood flow.

 Usually presents as semi rigid painless erection

Recurrent priaprism Typically seen in sickle cell disease, most commonly of high flow type.

Causes

 Intracavernosal drug therapies (e.g. for erectile dysfunction>


 Blood disorders such as leukaemia and sickle cell disease
 Neurogenic disorders such as spinal cord transection
 Trauma to penis resulting in arterio-venous malformations

Tests

 Exclude sickle cell/ leukaemia


 Consider blood sampling from cavernosa to determine whether high or low flow (low flow is
often hypoxic)

Management

 Ice packs/ cold showers


 If due to low flow then blood may be aspirated from copora or try intracavernosal alpha
adrenergic agonists.
 Delayed therapy of low flow priaprism may result in erectile dysfunction.

Next question
Question 411 of 560

The cephalic vein pierces the clavipectoral fascia to terminate in which of the veins listed below?

External jugular

Axillary

Internal jugular

Azygos

Brachial

Please rate this question:

Discuss and give feedback


Next question

Cephalic vein

Path

 Dorsal venous arch drains laterally into the cephalic vein


 Crosses the anatomical snuffbox and travels laterally up the arm
 At the antecubital fossa connected to the basilic vein by the median cubital vein
 Pierces deep fascia of deltopectoral groove to join axillary vein

Next question
Question 412 of 560

A 43 year old lady develops a cerebello-pontine angle lesion. Which of the nerves listed below is
likely to be affected first?

CN X

CN III

CN V

CN IX

CN XII

Theme from September 2013 Exam


The most likely lesion to occur in the cerebello-pontine angle is an acoustic neuroma.
The trigeminal nerve has a broad base and involvement of at least part of this nerve is the most
likely initial finding. The defect may be subtle such as loss of the ipsilateral corneal reflex. Ipsilateral
hearing loss will also occur. Untreated, progressive lesions, may ultimately affect may cranial nerve
roots in this region.

Please rate this question:

Discuss and give feedback

Next question

Cranial nerves

Cranial nerve lesions


Olfactory nerve May be injured in basal skull fractures or involved in frontal lobe tumour extension.
Loss of olfactory nerve function in relation to major CNS pathology is seldom an
isolated event and thus it is poor localiser of CNS pathology.

Optic nerve Problems with visual acuity may result from intra ocular disorders. Problems with
the blood supply such as amaurosis fugax may produce temporary visual distortion.
More important surgically is the pupillary response to light. The pupillary size may
be altered in a number of disorders. Nerves involved in the resizing of the pupil
connect to the pretectal nucleus of the high midbrain, bypassing the lateral
geniculate nucleus and the primary visual cortex. From the pretectal nucleus
neurones pass to the Edinger - Westphal nucleus, motor axons from here pass along
with the oculomotor nerve. They synapse with ciliary ganglion neurones; the
parasympathetic axons from this then innervate the iris and produce miosis. The
miotic pupil is seen in disorders such as Horner's syndrome or opiate overdose.
Mydriasis is the dilatation of the pupil in response to disease, trauma, drugs (or the
dark!). It is pathological when light fails to induce miosis. The radial muscle is
innervated by the sympathetic nervous system. Because the parasympathetic fibres
travel with the oculomotor nerve they will be damaged by lesions affecting this
nerve (e.g. cranial trauma).
The response to light shone in one eye is usually a constriction of both pupils. This
indicates intact direct and consensual light reflexes. When the optic nerve has an
afferent defect the light shining on the affected eye will produce a diminished
pupillary response in both eyes. Whereas light shone on the unaffected eye will
produce a normal pupillary response in both eyes. This is referred to as the Marcus
Gunn pupil and is seen in conditions such as optic neuritis. In a total CN II lesion
shining the light in the affected eye will produce no response.

Oculomotor nerve The pupillary effects are described above. In addition it supplies all ocular muscles
apart from lateral rectus and superior oblique. Thus the affected eye will be
deviated inferolaterally. Levator palpebrae superioris may also be impaired resulting
in impaired ability to open the eye.

Trochlear nerve The eye will not be able to look down.

Trigeminal nerve Largest cranial nerve. Exits the brainstem at the pons. Branches are ophthalmic,
maxillary and mandibular. Only the mandibular branch has both sensory and motor
fibres. Branches converge to form the trigeminal ganglion (located in Meckels cave).
It supplies the muscles of mastication and also tensor veli palatine, mylohyoid,
anterior belly of digastric and tensor tympani. The detailed descriptions of the
various sensory functions are described in other areas of the website. The corneal
reflex is important and is elicited by applying a small tip of cotton wool to the
cornea, a reflex blink should occur if it is intact. It is mediated by: the naso ciliary
branch of the ophthalmic branch of the trigeminal (sensory component) and the
facial nerve producing the motor response. Lesions of the afferent arc will produce
bilateral absent blink and lesions of the efferent arc will result in a unilateral absent
blink.

Abducens nerve The affected eye will have a deficit of abduction. This cranial nerve exits the
brainstem between the pons and medulla. It thus has a relatively long intra cranial
course which renders it susceptible to damage in raised intra cranial pressure.

Facial nerve Emerges from brainstem between pons and medulla. It controls muscles of facial
expression and taste from the anterior 2/3 of the tongue. The nerve passes into the
petrous temporal bone and into the internal auditory meatus. It then passes
through the facial canal and exits at the stylomastoid foramen. It passes through the
parotid gland and divides at this point. It does not innervate the parotid gland. Its
divisions are considered in other parts of the website. Its motor fibres innervate
orbicularis oculi to produce the efferent arm of the corneal reflex. In surgical
practice it may be injured during parotid gland surgery or invaded by malignancies
of the gland and a lower motor neurone on the ipsilateral side will result.

Vestibulo-cochlear Exits from the pons and then passes through the internal auditory meatus. It is
nerve implicated in sensorineural hearing loss. Individuals with sensorineural hearing loss
will localise the sound in webers test to the normal ear. Rinnes test will be reduced
on the affected side but should still work. These two tests will distinguish
sensorineural hearing loss from conductive deafness. In the latter condition webers
test will localise to the affected ear and Rinnes test will be impaired on the affected
side. Surgical lesions affecting this nerve include CNS tumours and basal skull
fractures. It may also be damaged by the administration of ototoxic drugs (of which
gentamicin is the most commonly used in surgical practice).

Glossopharyngeal Exits the pons just above the vagus. Receives sensory fibres from posterior 1/3
nerve tongue, tonsils, pharynx and middle ear (otalgia may occur following tonsillectomy).
It receives visceral afferents from the carotid bodies. It supplies parasympathetic
fibres to the parotid gland via the otic ganglion and motor function to
stylopharyngeaus muscle. The sensory function of the nerve is tested using the gag
reflex.

Vagus nerve Leaves the medulla between the olivary nucleus and the inferior cerebellar
peduncle. Passes through the jugular foramen and into the carotid sheath. Details of
the functions of the vagus nerve are covered in the website under relevant organ
sub headings.

Accessory nerve Exists from the caudal aspect of the brainstem (multiple branches) supplies
trapezius and sternocleidomastoid muscles. The distal portion of this nerve is most
prone to injury during surgical procedures.

Hypoglossal nerve Emerges from the medulla at the preolivary sulcus, passes through the hypoglossal
canal. It lies on the carotid sheath and passes deep to the posterior belly of digastric
to supply muscles of the tongue (except palatoglossus). Its location near the carotid
sheath makes it vulnerable during carotid endarterectomy surgery and damage will
produce ipsilateral defect in muscle function.

Next question
Question 413 of 560

Which of the following is not a branch of the abdominal aorta?

Inferior mesenteric artery

Inferior phrenic artery

Superior mesenteric artery

Superior phrenic artery

Renal artery

Mnemonic for the Descending abdominal aorta branches from diaphragm to iliacs:

'Prostitutes Cause Sagging Swollen Red Testicles [in men] Living In Sin':

Phrenic [inferior]
Celiac
Superior mesenteric
Suprarenal [middle]
Renal
Testicular ['in men' only]
Lumbars
Inferior mesenteric
Sacral

The superior phrenic artery branches from the aorta in the thorax.

Please rate this question:

Discuss and give feedback


Next question

Abdominal aortic branches

Branches Level Paired Type

Inferior phrenic T12 (Upper border) Yes Parietal

Coeliac T12 No Visceral

Superior mesenteric L1 No Visceral

Middle suprarenal L1 Yes Visceral

Renal L1-L2 Yes Visceral

Gonadal L2 Yes Visceral

Lumbar L1-L4 Yes Parietal

Inferior mesenteric L3 No Visceral

Median sacral L4 No Parietal

Common iliac L4 Yes Terminal

Next question
Question 414 of 560

A 40 year old lady presents with varicose veins, these are found to originate from the short
saphenous vein. As the vein is mobilised close to its origin which structure is at greatest risk of
injury?

Sciatic nerve

Sural nerve

Common peroneal nerve

Tibial nerve

Popliteal artery

The sural nerve is closely related and damage to this structure is a major cause of litigation. The
other structures may all be injured but the risks are lower.
Please rate this question:

Discuss and give feedback


Next question

Popliteal fossa

Boundaries of the popliteal fossa


Laterally Biceps femoris above, lateral head of gastrocnemius and plantaris below

Medially Semimembranosus and semitendinosus above, medial head of gastrocnemius below

Floor Popliteal surface of the femur, posterior ligament of knee joint and popliteus muscle

Roof Superficial and deep fascia


Image showing the popliteal fossa

© Image provided by the University of Sheffield

Contents

 Popliteal artery and vein


 Small saphenous vein
 Common peroneal nerve
 Tibial nerve
 Posterior cutaneous nerve of the thigh
 Genicular branch of the obturator nerve
 Lymph nodes

Next question
Question 415 of 560

A 23 year old man is admitted with a suspected ureteric colic. A KUB style x-ray is obtained. In
which of the following locations is the stone most likely to be visualised?

The tips of the transverse processes between L2 and L5

The tips of transverse processes between T10-L1

At the crest of the ilium

Over the S3 foramina

Over the sacrococcygeal joint

The ureter lies anterior to L2 to L5 and stones may be visualised at these points, they may also be
identified over the sacro-iliac joints.
Please rate this question:

Discuss and give feedback


Next question

Ureter

 25-35 cm long
 Muscular tube lined by transitional epithelium
 Surrounded by thick muscular coat. Becomes 3 muscular layers as it crosses the bony pelvis
 Retroperitoneal structure overlying transverse processes L2-L5
 Lies anterior to bifurcation of iliac vessels
 Blood supply is segmental; renal artery, aortic branches, gonadal branches, common iliac
and internal iliac
 Lies beneath the uterine artery

Next question
Question 416 of 560

A 72 year old man with non reconstructible arterial disease is undergoing an above knee
amputation. The posterior compartment muscles are divided. Which of the following muscles does
not lie in the posterior compartment of the thigh?

Biceps femoris

Quadriceps femoris

Semitendinosus

Semimembranosus

None of the above

The quadriceps femoris lies in the anterior compartment.


Please rate this question:

Discuss and give feedback


Next question

Fascial compartments of the leg

Compartments of the thigh

Formed by septae passing from the femur to the fascia lata.


Compartment Nerve Muscles Blood supply

Anterior compartment Femoral  Iliacus Femoral artery


 Tensor fasciae latae
 Sartorius
 Quadriceps femoris

Medial compartment Obturator  Adductor Profunda femoris artery


longus/magnus/brevis and obturator artery
 Gracilis
Compartment Nerve Muscles Blood supply

 Obturator externus

Posterior Sciatic  Semimembranosus Branches of Profunda


compartment (2  Semitendinosus femoris artery
layers)  Biceps femoris

Compartments of the lower leg


Separated by the interosseous membrane (anterior and posterior compartments), anterior fascial
septum (separate anterior and lateral compartments) and posterior fascial septum (separate lateral
and posterior compartments)

Blood
Compartment Nerve Muscles supply

Anterior Deep  Tibialis anterior Anterior


compartment peroneal  Extensor digitorum longus tibial artery
nerve  Extensor hallucis longus
 Peroneus tertius

Posterior Tibial  Muscles: deep and superficial Posterior


compartment compartments (separated by deep tibial
transverse fascia)
 Deep: Flexor hallucis longus, Flexor
digitalis longus, Tibialis posterior,
Popliteus
 Superficial: Gastrocnemius, Soleus,
Plantaris

Lateral Superficial  Peroneus longus/brevis Peroneal


compartment peroneal artery
Next question
Question 417 of 560

A woman develops winging of the scapula following a Patey mastectomy. What is the most likely
cause?

Division of pectoralis minor to access level 3 axillary nodes

Damage to the brachial plexus during axillary dissection

Damage to the long thoracic nerve during axillary dissection

Division of the thoracodorsal trunk during axillary dissection

Damage to the thoracodorsal trunk during axillary dissection

Theme from January 2012 exam


The serratus anterior muscle is supplied by the long thoracic nerve which runs along the surface of
serratus anterior and is liable to injury during nodal dissection. Although pectoralis minor is divided
during a Patey mastectomy (now seldom performed) it is rare for this alone to produce winging of the
scapula.
Please rate this question:

Discuss and give feedback


Next question

Long thoracic nerve

 Derived from ventral rami of C5, C6, and C7 (close to their emergence from intervertebral
foramina)
 It runs downward and passes either anterior or posterior to the middle scalene muscle
 It reaches upper tip of serratus anterior muscle and descends on outer surface of this
muscle, giving branches into it
 Winging of Scapula occurs in long thoracic nerve injury (most common) or from spinal
accessory nerve injury (which denervates the trapezius) or a dorsal scapular nerve injury

Next question
Question 418 of 560

In a patient with an ectopic kidney where is the adrenal gland most likely to be located?

In the pelvis

On the contralateral side

In its usual position

Superior to the spleen

It will be absent

Because the kidney is present, rather than absent, the adrenal will usual develop and in the normal
location.

Please rate this question:

Discuss and give feedback

Next question

Adrenal gland embryology

First detected at 6 weeks' gestation, the adrenal cortex is derived from the mesoderm of the
posterior abdominal wall. Steroid secretion from the fetal cortex begins shortly thereafter. Adult-type
zona glomerulosa and fasciculata are detected in fetal life but make up only a small proportion of the
gland, and the zona reticularis is not present at all. The fetal cortex predominates throughout fetal
life. The adrenal medulla is of ectodermal origin, arising from neural crest cells that migrate to the
medial aspect of the developing cortex.

The fetal adrenal gland is relatively large. At 4 months' gestation, it is 4 times the size of the kidney;
however, at birth, it is a third of the size of the kidney. This occurs because of the rapid regression of
the fetal cortex at birth. It disappears almost completely by age 1 year; by age 4-5 years, the
permanent adult-type adrenal cortex has fully developed.

Anatomic anomalies of the adrenal gland may occur. Because the development of the adrenals is
closely associated with that of the kidneys, agenesis of an adrenal gland is usually associated with
ipsilateral agenesis of the kidney, and fused adrenal glands (whereby the 2 glands join across the
midline posterior to the aorta) are also associated with a fused kidney.

Adrenal hypoplasia occurs in the following 2 forms: (1) hypoplasia or absence of the fetal cortex with
a poorly formed medulla and (2) disorganized fetal cortex and medulla with no permanent cortex
present. Adrenal heterotopia describes a normal adrenal gland in an abnormal location, such as
within the renal or hepatic capsules. Accessory adrenal tissue (adrenal rests), which is usually
comprised only of cortex but seen combined with medulla in some cases, is most commonly located
in the broad ligament or spermatic cord but can be found anywhere within the abdomen. Even
intracranial adrenal rests have been reported

Next question
Question 419 of 560

Which of the following structures is not closely related to the posterior tibial artery?

Soleus posteriorly

Tibial nerve medially

Deep peroneal nerve laterally

Flexor hallucis longus postero-inferiorly

Popliteus

The deep peroneal nerve lies in the anterior compartment. The tibial nerve lies medially. At its
termination it lies deep to the flexor retinaculum.
Please rate this question:

Discuss and give feedback


Next question

Posterior tibial artery

 Larger terminal branch of the popliteal artery


 Terminates by dividing into the medial and lateral plantar arteries
 Accompanied by two veins throughout its length
 Position of the artery corresponds to a line drawn from the lower angle of the popliteal fossa,
at the level of the neck of the fibula, to a point midway between the medial malleolus and the
most prominent part of the heel

Relations of the posterior tibial artery


Proximal to distal
Anteriorly Tibialis posterior
Flexor digitorum longus
Posterior surface of tibia and ankle joint
Posterior Tibial nerve 2.5 cm distal to its origin
Fascia overlying the deep muscular layer
Proximal part covered by gastrocnemius and soleus
Distal part covered by skin and fascia

Next question
Question 420 of 560

Which of the following is not closely related to the capitate bone?

Lunate bone

Scaphoid bone

Ulnar nerve

Hamate bone

Trapezoid bone

The ulnar nerve and artery lie adjacent to the pisiform bone. The capitate bone articulates with the
lunate, scaphoid, hamate and trapezoid bones, which are therefore closely related to it.

Please rate this question:

Discuss and give feedback

Next question

Capitate bone

This is the largest of the carpal bones. It is centrally placed with a rounded head set into the cavities
of the lunate and scaphoid bones. Flatter articular surfaces are present for the hamate medially and
the trapezoid laterally. Distally the bone articulates predominantly with the middle metacarpal.

Next question
Question 421 of 560

Which of the following statements relating to the tympanic membrane is false?

The umbo marks the point of attachment of the handle of the malleus to the tympanic
membrane

The lateral aspect of the tympanic membrane is lined by stratified squamous epithelium

The chorda tympani nerve runs medial to the pars tensa

The medial aspect of the tympanic membrane is lined by mucous membrane

The tympanic membrane is approximately 1cm in diameter

The chorda tympani runs medially to the pars flaccida. The relationship is shown from the medial
aspect in the dissection below.

Image sourced from Wikipedia

Please rate this question:


Discuss and give feedback
Next question

Ear- anatomy

The ear is composed of three anatomically distinct regions.

External ear
Auricle is composed of elastic cartilage covered by skin. The lobule has no cartilage and contains fat
and fibrous tissue.

External auditory meatus is approximately 2.5cm long.


Lateral third of the external auditory meatus is cartilaginous and the medial two thirds is bony.

The region is innervated by the greater auricular nerve. The auriculotemporal branch of the
trigeminal nerve supplies most the of external auditory meatus and the lateral surface of the auricle.

Middle ear
Space between the tympanic membrane and cochlea. The aditus leads to the mastoid air cells is the
route through which middle ear infections may cause mastoiditis. Anteriorly the eustacian tube
connects the middle ear to the naso pharynx.
The tympanic membrane consists of:

 Outer layer of stratified squamous epithelium.


 Middle layer of fibrous tissue.
 Inner layer of mucous membrane continuous with the middle ear.

The tympanic membrane is approximately 1cm in diameter.


The chorda tympani nerve passes on the medial side of the pars flaccida.

The middle ear is innervated by the glossopharyngeal nerve and pain may radiate to the middle ear
following tonsillectomy.

Ossicles
Malleus attaches to the tympanic membrane (the Umbo).
Malleus articulates with the incus (synovial joint).
Incus attaches to stapes (another synovial joint).

Internal ear
Cochlea, semi circular canals and vestibule

Organ of corti is the sense organ of hearing and is located on the inside of the cochlear duct on the
basilar membrane.

Vestibule accommodates the utricule and the saccule. These structures contain endolymph and are
surrounded by perilymph within the vestibule.

The semicircular canals lie at various angles to the petrous temporal bone. All share a common
opening into the vestibule.
Question 422 of 560

An injury to the spinal accessory nerve will affect which of the following movements?

Lateral rotation of the arm

Adduction of the arm at the glenohumeral joint

Protraction of the scapula

Upward rotation of the scapula

Depression of the scapula

The spinal accessory nerve innervates trapezius. The entire muscle will retract the scapula.
However, its upper and lower fibres act together to upwardly rotate it.
Please rate this question:

Discuss and give feedback


Next question

Shoulder joint

 Shallow synovial ball and socket type of joint.


 It is an inherently unstable joint, but is capable to a wide range of movement.
 Stability is provided by muscles of the rotator cuff that pass from the scapula to insert in the
greater tuberosity (all except sub scapularis-lesser tuberosity).

Glenoid labrum

 Fibrocartilaginous rim attached to the free edge of the glenoid cavity


 Tendon of the long head of biceps arises from within the joint from the supraglenoid tubercle,
and is fused at this point to the labrum.
 The long head of triceps attaches to the infraglenoid tubercle

Fibrous capsule
 Attaches to the scapula external to the glenoid labrum and to the labrum itself (postero-
superiorly)
 Attaches to the humerus at the level of the anatomical neck superiorly and the surgical neck
inferiorly
 Anteriorly the capsule is in contact with the tendon of subscapularis, superiorly with the
supraspinatus tendon, and posteriorly with the tendons of infraspinatus and teres minor. All
these blend with the capsule towards their insertion.
 Two defects in the fibrous capsule; superiorly for the tendon of biceps. Anteriorly there is a
defect beneath the subscapularis tendon.
 The inferior extension of the capsule is closely related to the axillary nerve at the surgical
neck and this nerve is at risk in anteroinferior dislocations. It also means that proximally sited
osteomyelitis may progress to septic arthritis.

Movements and muscles


Flexion Anterior part of deltoid
Pectoralis major
Biceps
Coracobrachialis

Extension Posterior deltoid


Teres major
Latissimus dorsi

Adduction Pectoralis major


Latissimus dorsi
Teres major
Coracobrachialis

Abduction Mid deltoid


Supraspinatus

Medial rotation Subscapularis


Anterior deltoid
Teres major
Latissimus dorsi

Lateral rotation Posterior deltoid


Infraspinatus
Teres minor
Important anatomical relations
Anteriorly Brachial plexus
Axillary artery and vein

Posterior Suprascapular nerve


Suprascapular vessels

Inferior Axillary nerve


Circumflex humeral vessels

Next question
Question 423 of 560

Which of the following is not contained within the middle mediastinum?

Main bronchi

Arch of the azygos vein

Thoracic duct

Pericardium

Aortic root

The thoracic duct lies within the posterior and superior mediastinum.
Please rate this question:

Discuss and give feedback


Next question

Mediastinum

Region between the pulmonary cavities.


It is covered by the mediastinal pleura. It does not contain the lungs.
It extends from the thoracic inlet superiorly to the diaphragm inferiorly.

Mediastinal regions

 Superior mediastinum (between manubriosternal angle and T4/5)


 Middle mediastinum
 Posterior mediastinum
 Anterior mediastinum

Region Contents
Region Contents

Superior mediastinum  Superior vena cava


 Brachiocephalic veins
 Arch of aorta
 Thoracic duct
 Trachea
 Oesophagus
 Thymus
 Vagus nerve
 Left recurrent laryngeal nerve
 Phrenic nerve

Anterior mediastinum  Thymic remnants


 Lymph nodes
 Fat

Middle mediastinum  Pericardium


 Heart
 Aortic root
 Arch of azygos vein
 Main bronchi

Posterior mediastinum  Oesophagus


 Thoracic aorta
 Azygos vein
 Thoracic duct
 Vagus nerve
 Sympathetic nerve trunks
 Splanchnic nerves

Next question
Question 424 of 560

A 55 year old man is due to undergo a radical prostatectomy for carcinoma of the prostate gland.
Which of the following vessels directly supplies the prostate?

External iliac artery

Common iliac artery

Internal iliac artery

Inferior vesical artery

None of the above

The arterial supply to the prostate gland is from the inferior vesical artery, it is a branch of the
prostatovesical artery. The prostatovesical artery usually arises from the internal pudendal and
inferior gluteal arterial branches of the internal iliac artery.
Please rate this question:

Discuss and give feedback


Next question

Prostate gland

The prostate gland is approximately the shape and size of a walnut and is located inferior to the
bladder. It is separated from the rectum by Denonvilliers fascia and its blood supply is derived from
the internal iliac vessels (via inferior vesical artery). The internal sphincter lies at the apex of the
gland and may be damaged during prostatic surgery, affected individuals may complain of
retrograde ejaculation.

Summary of prostate gland


Arterial supply Inferior vesical artery (from internal iliac)

Venous drainage Prostatic venous plexus (to paravertebral veins)


Lymphatic Internal iliac nodes
drainage

Innervation Inferior hypogastric plexus

Dimensions  Transverse diameter (4cm)


 AP diameter (2cm)
 Height (3cm)

Lobes  Posterior lobe: posterior to urethra


 Median lobe: posterior to urethra, in between ejaculatory ducts
 Lateral lobes x 2
 Isthmus

Zones  Peripheral zone: subcapsular portion of posterior prostate. Most


prostate cancers are here
 Central zone
 Transition zone
 Stroma

Relations
Pubic symphysis
Anterior Prostatic venous plexus

Posterior Denonvilliers fascia


Rectum
Ejaculatory ducts

Lateral Venous plexus (lies on prostate)


Levator ani (immediately below the puboprostatic ligaments)
Image sourced from Wikipedia

Next question
Question 425 of 560

Which nerve directly innervates the sinoatrial node?

Superior cardiac nerve

Right vagus nerve

Left vagus nerve

Inferior cardiac nerve

None of the above

Theme from September 2012 Exam


Theme from April 2016 Exam
No single one of the above nerves is responsible for direct cardiac innervation (which those who
have handled the heart surgically will appreciate).
The heart receives its nerves from the superficial and deep cardiac plexuses. The cardiac plexuses
send small branches to the heart along the major vessels, continuing with the right and left coronary
arteries. The vagal efferent fibres emerge from the brainstem in the roots of the vagus and
accessory nerves, and run to ganglia in the cardiac plexuses and within the heart itself.

The background vagal discharge serves to limit heart rate, and loss of this background vagal tone
accounts for the higher resting heart rate seen following cardiac transplant.
Please rate this question:

Discuss and give feedback


Next question

Sinoatrial node

 Located in the wall of the right atrium in the upper part of the sulcus terminalis from which it
extends anteriorly over the opening of the superior vena cava.
 In most cases it is supplied by the right coronary artery.
 It has a complicated nerve supply from the cardiac nerve plexus that takes both sympathetic
and parasympathetic fibres that run alongside the main vessels.

Next question
Question 426 of 560

A 30 year old man presents with back pain and the surgeon tests the ankle reflex. Which of the
following nerve roots are tested in this manoeuvre?

S3 and S4

L4 and L5

L3 and L4

S1 and S2

S4 only

Theme from April 2012 Exam

Please rate this question:

Discuss and give feedback

Next question

Ankle reflex

The ankle reflex is elicited by tapping the Achilles tendon with a tendon hammer. It tests the S1 and
S2 nerve roots. It is typically delayed in L5 and S1 disk prolapses.

Next question
Question 427 of 560

Which of the following structures is not closely related to the piriformis muscle?

Superior gluteal nerve

Sciatic nerve

Inferior gluteal artery

Inferior gluteal nerve

Medial femoral circumflex artery

Nerve supply of lateral hip rotators

Piriformis: ventral rami S1, S2


Obturator internus: nerve to obturator internus
Superior gemellus: nerve to obturator internus
Inferior gemellus: nerve to quadratus femoris
Quadrator femoris: nerve to quadrator femoris

The piriformis muscle is an important anatomical landmark in the gluteal region. The following
structures are closely related:

 Sciatic nerve
 Inferior gluteal artery and nerve
 Superior gluteal artery and nerve

The medial femoral circumflex artery runs deep to quadratus femoris.


Please rate this question:

Discuss and give feedback


Next question

Gluteal region

Gluteal muscles
 Gluteus maximus: inserts to gluteal tuberosity of the femur and iliotibial tract
 Gluteus medius: attach to lateral greater trochanter
 Gluteus minimis: attach to anterior greater trochanter
 All extend and abduct the hip

Deep lateral hip rotators

 Piriformis
 Gemelli
 Obturator internus
 Quadratus femoris

Nerves
Superior gluteal nerve (L5, S1)  Gluteus medius
 Gluteus minimis
 Tensor fascia lata

Inferior gluteal nerve Gluteus maximus


Damage to the superior gluteal nerve will result in the patient developing a Trendelenberg gait.
Affected patients are unable to abduct the thigh at the hip joint. During the stance phase, the
weakened abductor muscles allow the pelvis to tilt down on the opposite side. To compensate, the
trunk lurches to the weakened side to attempt to maintain a level pelvis throughout the gait cycle.
The pelvis sags on the opposite side of the lesioned superior gluteal nerve.
Next question
Question 428 of 560

An 18 year old male presents to casualty with a depressed skull fracture. This is managed surgically.
Over the next few days he complains of double vision on walking down stairs and reading. On
testing ocular convergence, the left eye faces downwards and medially, but the right side does not
do so. Which of the nerves listed below is most likely to be responsible?

Facial

Oculomotor

Abducens

Trochlear

Trigeminal nerve

Theme from September 2012 Exam


The trochlear nerve has a relatively long intracranial course and this makes it vulnerable to injury in
head trauma. Head trauma is the commonest cause of an acute fourth nerve palsy. A 4th nerve
palsy is the commonest cause of a vertical diplopia. The diplopia is at its worst when the eye looks
medially which it usually does as part of the accommodation reflex when walking down stairs.
Please rate this question:

Discuss and give feedback


Next question

Disorders of the oculomotor system

Nerve Path Nerve palsy features

Oculomotor  Large nucleus at the midbrain Ptosis


nerve  Fibres pass through the red nucleus and the Eye down and out
pyramidal tract; through the cavernous sinus Unable to move the eye
into the orbit superiorly, inferiorly,
medially
Pupil fixed and dilated
Nerve Path Nerve palsy features

Trochlear  Longest intracranial course Vertical diplopia


nerve  Only nerve to exit the dorsal aspect of (diplopia on descending
brainstem the stairs)
 Nucleus at midbrain, passes between the Unable to look down
posterior cerebral and superior cerebellar and in
arteries, through the cavernous sinus into the
orbit

Abducens Nucleus lies in the mid pons Convergence of eyes in


nerve primary position
Lateral diplopia towards
side of lesion
Eye deviates medially
Next question
Question 429 of 560

A 77 year old man with symptoms of intermittent claudication is due to have his ankle brachial
pressure indices measured. The dorsalis pedis artery is impalpable. Which of the following tendinous
structures lies medial to it, that may facilitate its identification?

Extensor digitorum longus tendon

Peroneus tertius tendon

Extensor hallucis longus tendon

Extensor digitorum brevis tendon

Flexor digitorum longus tendon

The extensor hallucis longus tendon lies medial to the dorsalis pedis artery.
Please rate this question:

Discuss and give feedback


Next question

Foot- anatomy

Arches of the foot


The foot is conventionally considered to have two arches.

 The longitudinal arch is higher on the medial than on the lateral side. The posterior part of
the calcaneum forms a posterior pillar to support the arch. The lateral part of this structure
passes via the cuboid bone and the lateral two metatarsal bones. The medial part of this
structure is more important. The head of the talus marks the summit of this arch, located
between the sustentaculum tali and the navicular bone. The anterior pillar of the medial arch
is composed of the navicular bone, the three cuneiforms and the medial three metatarsal
bones.
 The transverse arch is situated on the anterior part of the tarsus and the posterior part of the
metatarsus. The cuneiforms and metatarsal bases narrow inferiorly, which contributes to the
shape of the arch.

Intertarsal joints
Sub talar joint Formed by the cylindrical facet on the lower surface of the body of the
talus and the posterior facet on the upper surface of the calcaneus. The
facet on the talus is concave anteroposteriorly, the other is convex. The
synovial cavity of this joint does not communicate with any other joint.

Talocalcaneonavicular The anterior part of the socket is formed by the concave articular
joint surface of the navicular bone, posteriorly by the upper surface of the
sustentaculum tali. The talus sits within this socket

Calcaneocuboid joint Highest point in the lateral part of the longitudinal arch. The lower
aspect of this joint is reinforced by the long plantar and plantar
calcaneocuboid ligaments.

Transverse tarsal joint The talocalcaneonavicular joint and the calcaneocuboid joint extend
across the tarsus in an irregular transverse plane, between the talus and
calcaneus behind and the navicular and cuboid bones in front. This
plane is termed the transverse tarsal joint.

Cuneonavicular joint Formed between the convex anterior surface of the navicular bone and
the concave surface of the the posterior ends of the three cuneiforms.

Intercuneiform joints Between the three cuneiform bones.

Cuneocuboid joint Between the circular facets on the lateral cuneiform bone and the
cuboid. This joint contributes to the tarsal part of the transverse arch.

A detailed knowledge of the joints is not required for MRCS Part A. However, the contribution they
play to the overall structure of the foot should be appreciated

Ligaments of the ankle joint and foot


Image sourced from Wikipedia

Muscles of the foot


Muscle Origin Insertion Nerve Action
supply

Abductor Medial side of the calcaneus, Medial side of Medial Abducts the great toe
hallucis flexor retinaculum, plantar the base of the plantar
aponeurosis proximal nerve
phalanx

Flexor Medial process of the Via 4 tendons Medial Flexes all the joints of
digitorum calcaneus, plantar into the plantar the lateral 4 toes except
brevis eponeurosis. middle nerve for the interphalangeal
phalanges of joint.
the lateral 4
toes.

Abductor From the tubercle of the Together with Lateral Abducts the little toe at
digit calcaneus and from the flexor digit plantar the metatarsophalangeal
minimi plantar aponeurosis minimi brevis nerve joint
into the lateral
side of the
base of the
proximal
phalanx of the
little toe

Flexor From the medial side of the Into the Medial Flexes the
hallucis plantar surface of the cuboid proximal plantar metatarsophalangeal
brevis bone, from the adjacent part phalanx of the nerve joint of the great toe.
of the lateral cuneiform bone great toe, the
and from the tendon of tendon
tibialis posterior. contains a
sesamoid bone

Adductor Arises from two heads. The Lateral side of Lateral Adducts the great toe
hallucis oblique head arises from the the base of the plantar towards the second toe.
sheath of the peroneus longus proximal nerve Helps maintain the
tendon, and from the plantar phalanx of the transverse arch of the
surfaces of the bases of the great toe. foot.
2nd, 3rd and 4th metatarsal
bones. The transverse head
arises from the plantar
surface of the lateral 4
metatarsophalangeal joints
and from the deep transverse
metatarsal ligament.

Extensor On the dorsal surface of the Via four thin Deep Extend the
digitorum foot from the upper surface of tendons which peroneal metatarsophalangeal
brevis the calcaneus and its run forward joint of the medial four
associated fascia and medially toes. It is unable to
to be inserted extend the
into the interphalangeal joint
medial four without the assistance of
toes. The the lumbrical muscles.
lateral three
tendons join
with hoods of
extensor
digitorum
longus.

Nerves in the foot


Lateral plantar nerve
Passes anterolaterally towards the base of the 5th metatarsal between flexor digitorum brevis and
flexor accessorius. On the medial aspect of the lateral plantar artery. At the base of the 5th
metatarsal it splits into superficial and deep branches.

Medial plantar nerve


Passes forwards with the medial plantar artery under the cover of the flexor retinaculum to the
interval between abductor hallucis and flexor digitorum brevis on the sole of the foot.

Plantar arteries
Arise under the cover of the flexor retinaculum, midway between the tip of the medial malleolus and
the most prominent part of the medial side of the heel.

 Medial plantar artery. Passes forwards medial to medial plantar nerve in the space between
abductor hallucis and flexor digitorum brevis.Ends by uniting with a branch of the 1st plantar
metatarsal artery.
 Lateral plantar artery. Runs obliquely across the sole of the foot. It lies lateral to the lateral
plantar nerve. At the base of the 5th metatarsal bone it arches medially across the foot on
the metatarsals

Dorsalis pedis artery


This vessel is a direct continuation of the anterior tibial artery. It commences on the front of the ankle
joint and runs to the proximal end of the first metatarsal space. Here is gives off the arcuate artery
and continues forwards as the first dorsal metatarsal artery. It is accompanied by two veins
throughout its length. It is crossed by the extensor hallucis brevis
Next question
Question 430 of 560

A 23 year old man falls over whilst intoxicated and a shard of glass transects his median nerve at the
proximal border of the flexor retinaculum. His tendons escape injury. Which of the following features
is least likely to be present?

Weakness of thumb abduction

Loss of sensation on the dorsal aspect of the thenar eminence

Loss of power of opponens pollicis

Adduction and lateral rotation of the thumb at rest

Loss of power of abductor pollicis brevis

The median nerve may be injured proximal to the flexor retinaculum. This will result in loss of
abductor pollicis brevis, flexor pollicis brevis, opponens pollicis and the first and second lumbricals.
When the patient is asked to close the hand slowly there is a lag of the index and middle fingers
reflecting the impaired lumbrical muscle function. The sensory changes are minor and do not extend
to the dorsal aspect of the thenar eminence.
Abductor pollicis longus will contribute to thumb abduction (and is innervated by the posterior
interosseous nerve) and therefore abduction will be weaker than prior to the injury.
Please rate this question:

Discuss and give feedback


Next question

Median nerve

The median nerve is formed by the union of a lateral and medial root respectively from the lateral
(C5,6,7) and medial (C8 and T1) cords of the brachial plexus; the medial root passes anterior to the
third part of the axillary artery. The nerve descends lateral to the brachial artery, crosses to its
medial side (usually passing anterior to the artery). It passes deep to the bicipital aponeurosis and
the median cubital vein at the elbow.
It passes between the two heads of the pronator teres muscle, and runs on the deep surface of
flexor digitorum superficialis (within its fascial sheath).
Near the wrist it becomes superficial between the tendons of flexor digitorum superficialis and flexor
carpi radialis, deep to palmaris longus tendon. It passes deep to the flexor retinaculum to enter the
palm, but lies anterior to the long flexor tendons within the carpal tunnel.
Branches
Region Branch

Upper arm No branches, although the nerve commonly communicates with the
musculocutaneous nerve

Forearm Pronator teres


Flexor carpi radialis
Palmaris longus
Flexor digitorum superficialis
Flexor pollicis longus
Flexor digitorum profundus (only the radial half)

Distal Palmar cutaneous branch


forearm

Hand Motor supply (LOAF)


(Motor)

 Lateral 2 lumbricals
 Opponens pollicis
 Abductor pollicis brevis
 Flexor pollicis brevis

Hand  Over thumb and lateral 2 ½ fingers


(Sensory)  On the palmar aspect this projects proximally, on the dorsal aspect only the
distal regions are innervated with the radial nerve providing the more
proximal cutaneous innervation.

Patterns of damage
Damage at wrist

 e.g. carpal tunnel syndrome


 paralysis and wasting of thenar eminence muscles and opponens pollicis (ape hand
deformity)
 sensory loss to palmar aspect of lateral (radial) 2 ½ fingers
Damage at elbow, as above plus:

 unable to pronate forearm


 weak wrist flexion
 ulnar deviation of wrist

Anterior interosseous nerve (branch of median nerve)

 leaves just below the elbow


 results in loss of pronation of forearm and weakness of long flexors of thumb and index
finger

Topography of the median nerve

Image sourced from Wikipedia

Next question
Question 431 of 560

The following muscles are supplied by the recurrent laryngeal nerve except:

Transverse arytenoid

Posterior crico-arytenoid

Cricothyroid

Oblique arytenoid

Thyroarytenoid

Innervates: all intrinsic larynx muscles (excluding cricothyroid)

The external branch of the superior laryngeal nerve innervates the cricothyroid muscle.
Please rate this question:

Discuss and give feedback


Next question

Recurrent laryngeal nerve

 Branch of the vagus nerve

Path

Right

 Arises anterior to the subclavian artery and ascends obliquely next to the trachea, behind the
common carotid artery
 It is either anterior or posterior to the inferior thyroid artery

Left

 Arises left to the arch of the aorta


 Winds below the aorta
 Ascends along the side of the trachea

Then both

 Pass in a groove between the trachea and oesophagus


 Enters the larynx behind the articulation between the thyroid cartilage and cricoid
 Distributed to larynx muscles

Branches to

 Cardiac plexus
 Mucous membrane and muscular coat of the oesophagus and trachea

Innervates

 Intrinsic larynx muscles (excluding cricothyroid)

Image sourced from Wikipedia

Next question
Question 432 of 560

From which embryological structure is the ureter derived?

Uranchus

Cloaca

Vitello-intestinal duct

Mesonephric duct

None of the above

The ureter develops from the mesonephric duct. The mesonephric duct is associated with the
metanephric duct that develops within the metenephrogenic blastema. This forms the site of the
ureteric bud which branches off the mesonephric duct.
Please rate this question:

Discuss and give feedback


Next question

Ureter

 25-35 cm long
 Muscular tube lined by transitional epithelium
 Surrounded by thick muscular coat. Becomes 3 muscular layers as it crosses the bony pelvis
 Retroperitoneal structure overlying transverse processes L2-L5
 Lies anterior to bifurcation of iliac vessels
 Blood supply is segmental; renal artery, aortic branches, gonadal branches, common iliac
and internal iliac
 Lies beneath the uterine artery

Next question
Question 433 of 560

A 16 year old boy is hit by a car and sustains a blow to the right side of his head. He is initially
conscious but on arrival in the emergency department is comatose. On examination his right pupil is
fixed and dilated. The neurosurgeons plan immediate surgery. What type of initial approach should
be made?

Left parieto-temporal craniotomy

Right parieto-temporal craniotomy

Posterior fossa craniotomy

Left parieto-temporal burr holes

None of the above

Theme from April 2012 Exam


A unilateral dilated pupil is a classic sign of transtentorial herniation. The medial aspect of the
temporal lobe (uncus) herniates across the tentorium and causes pressure on the ipsilateral
oculomotor nerve, interrupting parasympathetic input to the eye and resulting in a dilated pupil. In
addition the brainstem is compressed. As the ipsilateral oculomotor nerve is being compressed,
craniotomy (rather than Burr Holes) should be made on the ipsilateral side.
Please rate this question:

Discuss and give feedback


Next question

Head injury

Patients who suffer head injuries should be managed according to ATLS principles and extra cranial
injuries should be managed alongside cranial trauma. Inadequate cardiac output will compromise
CNS perfusion irrespective of the nature of the cranial injury.

Types of traumatic brain injury


Bleeding into the space between the dura mater and the skull. Often results from
acceleration-deceleration trauma or a blow to the side of the head. The majority
Extradural
of extradural haematomas occur in the temporal region where skull fractures
haematoma
cause a rupture of the middle meningeal artery.
Features

 Raised intracranial pressure


 Some patients may exhibit a lucid interval

Bleeding into the outermost meningeal layer. Most commonly occur around the
frontal and parietal lobes. May be either acute or chronic.
Subdural
haematoma Risk factors include old age and alcoholism.

Slower onset of symptoms than a extradural haematoma.

Usually occurs spontaneously in the context of a ruptured cerebral aneurysm, but


Subarachnoid
may be seen in association with other injuries when a patient has sustained a
haemorrhage
traumatic brain injury.

Pathophysiology

 Primary brain injury may be focal (contusion/ haematoma) or diffuse (diffuse axonal injury)
 Diffuse axonal injury occurs as a result of mechanical shearing following deceleration,
causing disruption and tearing of axons
 Intra-cranial haematomas can be extradural, subdural or intracerebral, while contusions may
occur adjacent to (coup) or contralateral (contre-coup) to the side of impact
 Secondary brain injury occurs when cerebral oedema, ischaemia, infection, tonsillar or
tentorial herniation exacerbates the original injury. The normal cerebral auto regulatory
processes are disrupted following trauma rendering the brain more susceptible to blood flow
changes and hypoxia
 The Cushings reflex (hypertension and bradycardia) often occurs late and is usually a pre
terminal event

Management

 Where there is life threatening rising ICP such as in extra dural haematoma and whilst
theatre is prepared or transfer arranged use of IV mannitol/ frusemide may be required.
 Diffuse cerebral oedema may require decompressive craniotomy
 Exploratory Burr Holes have little management in modern practice except where scanning
may be unavailable and to thus facilitate creation of formal craniotomy flap
 Depressed skull fractures that are open require formal surgical reduction and debridement,
closed injuries may be managed non operatively if there is minimal displacement.
 ICP monitoring is appropriate in those who have GCS 3-8 and normal CT scan.
 ICP monitoring is mandatory in those who have GCS 3-8 and abnormal CT scan.
 Hyponatraemia is most likely to be due to syndrome of inappropriate ADH secretion.
 Minimum of cerebral perfusion pressure of 70mmHg in adults.
 Minimum cerebral perfusion pressure of between 40 and 70 mmHg in children.
Interpretation of pupillary findings in head injuries
Pupil size Light response Interpretation

Unilaterally dilated Sluggish or fixed 3rd nerve compression secondary to tentorial


herniation

Bilaterally dilated Sluggish or fixed  Poor CNS perfusion


 Bilateral 3rd nerve palsy

Unilaterally dilated or Cross reactive (Marcus - Optic nerve injury


equal Gunn)

Bilaterally constricted May be difficult to  Opiates


assess  Pontine lesions
 Metabolic encephalopathy

Unilaterally Preserved Sympathetic pathway disruption


constricted
Next question
Question 434 of 560

Which of the following relationship descriptions regarding the scalene muscles is incorrect?

The brachial plexus passes anterior to the middle scalene muscle

The phrenic nerve lies anterior to the anterior scalene muscle

The subclavian artery passes posterior to the middle scalene

The subclavian vein lies anterior to the anterior scalene muscle at the level of the first rib

The anterior scalene inserts into the first rib

The subclavian artery passes anterior to the middle scalene.


Please rate this question:

Discuss and give feedback


Next question

Scalene muscles

The 3 paired muscles are:

 Scalenus anterior: Elevate 1st rib and laterally flex the neck to same side
 Scalenus medius: Same action as scalenus anterior
 Scalenus posterior: Elevate 2nd rib and tilt neck to opposite side

Innervation Spinal nerves C4-6

Origin Transverse processes C2 to C7

Insertion First and second ribs

 The brachial plexus and subclavian artery pass between the anterior and
Important
middle scalenes through a space called the scalene hiatus/fissure.
relations
 The subclavian vein and phrenic nerve pass anteriorly to the anterior
scalene as it crosses over the first rib.

Image sourced from Wikipedia

Thoracic outlet syndrome


The scalenes are at risk of adhering to the fascia surrounding the brachial plexus or shortening
causing compression of the brachial plexus when it passes between the clavicle and 1st rib causing
thoracic outlet syndrome.
Next question
Question 435 of 560

A 56 year old man is having a long venous line inserted via the femoral vein into the right atrium for
CVP measurements. The catheter is advanced through the IVC. At which of the following levels
does this vessel enter the thorax?

L2

T10

L1

T8

T6

Theme from 2010 Exam


Theme from September 2012 Exam
The IVC passes through the diaphragm at T8.
Please rate this question:

Discuss and give feedback


Next question

Inferior vena cava

Origin

 L5

Path

 Left and right common iliac veins merge to form the IVC.
 Passes right of midline
 Paired segmental lumbar veins drain into the IVC throughout its length
 The right gonadal vein empties directly into the cava and the left gonadal vein generally
empties into the left renal vein.
 The next major veins are the renal veins and the hepatic veins
 Pierces the central tendon of diaphragm at T8
 Right atrium

Image sourced from Wikipedia

Relations
Anteriorly Small bowel, first and third part of duodenum, head of pancreas, liver and bile duct,
right common iliac artery, right gonadal artery

Posteriorly Right renal artery, right psoas, right sympathetic chain, coeliac ganglion

Levels
Level Vein

T8 Hepatic vein, inferior phrenic vein, pierces diaphragm

L1 Suprarenal veins, renal vein

L2 Gonadal vein
L1-5 Lumbar veins

L5 Common iliac vein, formation of IVC

Next question
Question 436 of 560

A 23 year old man falls and injures his hand. There are concerns that he may have a scaphoid
fracture as there is tenderness in his anatomical snuffbox on clinical examination. Which of the
following forms the posterior border of this structure?

Basilic vein

Radial artery

Extensor pollicis brevis

Abductor pollicis longus

Extensor pollicis longus

Theme from 2009 Exam


Theme from September 2012 Exam
Its boundaries are extensor pollicis longus, medially (posterior border) and laterally (anterior border)
by the tendons of abductor pollicis longus and extensor pollicis brevis.

Please rate this question:

Discuss and give feedback

Next question

Anatomical snuffbox

Posterior border Tendon of extensor pollicis longus


Anterior border Tendons of extensor pollicis brevis and abductor pollicis longus

Proximal border Styloid process of the radius

Distal border Apex of snuffbox triangle

Floor Trapezium and scaphoid

Content Radial artery

Image showing the anatomical snuffbox

Image sourced from Wikipedia

Next question
Question 437 of 560

Which of the following structures attaches periosteum to bone?

Sharpeys fibres

Peripheral lamellae

Elastic fibres

Fibrolamellar bundles

Purkinje fibres

Periosteum is attached to bone by strong collagenous fibers called Sharpey's fibres, which extend to
the outer circumferential and interstitial lamellae. It also provides an attachment for muscles and
tendons.

Please rate this question:

Discuss and give feedback

Next question

Periosteum

Periosteum is a membrane that covers the outer surface of all bones, except at the joints of long
bones. Endosteum lines the inner surface of all bones.

Periosteum consists of dense irregular connective tissue. Periosteum is divided into an outer "fibrous
layer" and inner "cambium layer" (or "osteogenic layer"). The fibrous layer contains fibroblasts, while
the cambium layer contains progenitor cells that develop into osteoblasts. These osteoblasts are
responsible for increasing the width of a long bone and the overall size of the other bone types. After
a bone fracture the progenitor cells develop into osteoblasts and chondroblasts, which are essential
to the healing process.

As opposed to osseous tissue, periosteum has nociceptive nerve endings, making it very sensitive to
manipulation. It also provides nourishment by providing the blood supply. Periosteum is attached to
bone by strong collagenous fibers called Sharpey's fibres, which extend to the outer circumferential
and interstitial lamellae. It also provides an attachment for muscles and tendons.

Periosteum that covers the outer surface of the bones of the skull is known as "pericranium" except
when in reference to the layers of the scalp.

Next question
Question 438 of 560

A 62 year old man is undergoing a left hemicolectomy for carcinoma of the descending colon. The
registrar commences mobilisation of the left colon by pulling downwards and medially. Blood soon
appears in the left paracolic gutter. The most likely source of bleeding is the:

Marginal artery

Left testicular artery

Spleen

Left renal vein

None of the above

The spleen is commonly torn by traction injuries in colonic surgery. The other structures are
associated with bleeding during colonic surgery but would not manifest themselves as blood in the
paracolic gutter prior to incision of the paracolonic peritoneal edge.
Please rate this question:

Discuss and give feedback


Next question

Left colon

Position

 As the left colon passes inferiorly its posterior aspect becomes extraperitoneal, and the
ureter and gonadal vessels are close posterior relations that may become involved in
disease processes
 At a level of L3-4 (variable) the left colon becomes the sigmoid colon and wholly
intraperitoneal once again
 The sigmoid colon is a highly mobile structure and may even lie on the right side of the
abdomen
 It passes towards the midline, the taenia blend and this marks the transition between sigmoid
colon and upper rectum
Blood supply

 Inferior mesenteric artery


 However, the marginal artery (from the right colon) contributes, this contribution becomes
clinically significant when the IMA is divided surgically (e.g. During AAA repair)

Next question
Question 439 of 560

A man is undergoing excision of a sub mandibular gland. As the gland is mobilised, a vessel is
injured lying between the gland and the mandible. Which of the following is this vessel most likely to
be?

Lingual artery

Occipital artery

Superior thyroid artery

Facial artery

External jugular vein

The high salivary viscosity of submandibular gland secretions favors stone formation.
Most stones are radio-opaque.

The facial artery lies between the gland and mandible and is often ligated during excision of the
gland. The lingual artery may be encountered but this is usually later in the operative process as
Whartons duct is mobilised.
Please rate this question:

Discuss and give feedback


Next question

Submandibular gland

Relations of the submandibular gland


Superficial Platysma, deep fascia and mandible
Submandibular lymph nodes
Facial vein (facial artery near mandible)
Marginal mandibular nerve
Cervical branch of the facial nerve

Deep Facial artery (inferior to the mandible)


Mylohoid muscle
Sub mandibular duct
Hyoglossus muscle
Lingual nerve
Submandibular ganglion
Hypoglossal nerve

Submandibular duct (Wharton's duct)

 Opens lateral to the lingual frenulum on the anterior floor of mouth.


 5 cm length
 Lingual nerve wraps around Wharton's duct. As the duct passes forwards it crosses medial
to the nerve to lie above it and then crosses back, lateral to it, to reach a position below the
nerve.

Innervation

 Sympathetic innervation- Derived from superior cervical ganglion


 Parasympathetic innervation- Submandibular ganglion via lingual nerve

Arterial supply
Branch of the facial artery. The facial artery passes through the gland to groove its deep surface. It
then emerges onto the face by passing between the gland and the mandible.

Venous drainage
Anterior facial vein (lies deep to the Marginal Mandibular nerve)

Lymphatic drainage
Deep cervical and jugular chains of nodes
Next question
Question 440-442 of 560

Theme: Cranial nerves

A. Facial

B. Trigeminal

C. Vagus

D. Hypoglossal

E. Glossopharyngeal

For each of the following functions please select the most likely responsible cranial nerve. Each
option may be used once, more than once or not at all.

440. Supplies the motor fibres of styloglossus.

You answered Facial

The correct answer is Hypoglossal

The hypoglossal nerve supplies motor innervation to all extrinsic and intrinsic muscles of the
tongue. The only possible exception to this is palatoglossus (which is jointly innervated by the
vagus and accessory nerves.

441. Provides general sensation to the anterior two thirds of the tongue.

You answered Facial

The correct answer is Trigeminal

Taste to the anterior two thirds of the tongue is supplied by the facial nerve, the trigeminal
supplies general sensation, this is mediated by the mandibular branch of the trigeminal nerve
(via the lingual nerve).
442. Supplies general sensation to the posterior third of the tongue.

You answered Facial

The correct answer is Glossopharyngeal

The glossopharyngeal nerve supplies general sensation to the posterior third of the tongue and
contributes to the gag reflex.

Please rate this question:

Discuss and give feedback

Next question

Cranial nerves

Cranial nerve lesions

Olfactory nerve May be injured in basal skull fractures or involved in frontal lobe tumour extension.
Loss of olfactory nerve function in relation to major CNS pathology is seldom an
isolated event and thus it is poor localiser of CNS pathology.

Optic nerve Problems with visual acuity may result from intra ocular disorders. Problems with
the blood supply such as amaurosis fugax may produce temporary visual distortion.
More important surgically is the pupillary response to light. The pupillary size may
be altered in a number of disorders. Nerves involved in the resizing of the pupil
connect to the pretectal nucleus of the high midbrain, bypassing the lateral
geniculate nucleus and the primary visual cortex. From the pretectal nucleus
neurones pass to the Edinger - Westphal nucleus, motor axons from here pass along
with the oculomotor nerve. They synapse with ciliary ganglion neurones; the
parasympathetic axons from this then innervate the iris and produce miosis. The
miotic pupil is seen in disorders such as Horner's syndrome or opiate overdose.
Mydriasis is the dilatation of the pupil in response to disease, trauma, drugs (or the
dark!). It is pathological when light fails to induce miosis. The radial muscle is
innervated by the sympathetic nervous system. Because the parasympathetic fibres
travel with the oculomotor nerve they will be damaged by lesions affecting this
nerve (e.g. cranial trauma).
The response to light shone in one eye is usually a constriction of both pupils. This
indicates intact direct and consensual light reflexes. When the optic nerve has an
afferent defect the light shining on the affected eye will produce a diminished
pupillary response in both eyes. Whereas light shone on the unaffected eye will
produce a normal pupillary response in both eyes. This is referred to as the Marcus
Gunn pupil and is seen in conditions such as optic neuritis. In a total CN II lesion
shining the light in the affected eye will produce no response.

Oculomotor nerve The pupillary effects are described above. In addition it supplies all ocular muscles
apart from lateral rectus and superior oblique. Thus the affected eye will be
deviated inferolaterally. Levator palpebrae superioris may also be impaired resulting
in impaired ability to open the eye.

Trochlear nerve The eye will not be able to look down.

Trigeminal nerve Largest cranial nerve. Exits the brainstem at the pons. Branches are ophthalmic,
maxillary and mandibular. Only the mandibular branch has both sensory and motor
fibres. Branches converge to form the trigeminal ganglion (located in Meckels cave).
It supplies the muscles of mastication and also tensor veli palatine, mylohyoid,
anterior belly of digastric and tensor tympani. The detailed descriptions of the
various sensory functions are described in other areas of the website. The corneal
reflex is important and is elicited by applying a small tip of cotton wool to the
cornea, a reflex blink should occur if it is intact. It is mediated by: the naso ciliary
branch of the ophthalmic branch of the trigeminal (sensory component) and the
facial nerve producing the motor response. Lesions of the afferent arc will produce
bilateral absent blink and lesions of the efferent arc will result in a unilateral absent
blink.

Abducens nerve The affected eye will have a deficit of abduction. This cranial nerve exits the
brainstem between the pons and medulla. It thus has a relatively long intra cranial
course which renders it susceptible to damage in raised intra cranial pressure.

Facial nerve Emerges from brainstem between pons and medulla. It controls muscles of facial
expression and taste from the anterior 2/3 of the tongue. The nerve passes into the
petrous temporal bone and into the internal auditory meatus. It then passes
through the facial canal and exits at the stylomastoid foramen. It passes through the
parotid gland and divides at this point. It does not innervate the parotid gland. Its
divisions are considered in other parts of the website. Its motor fibres innervate
orbicularis oculi to produce the efferent arm of the corneal reflex. In surgical
practice it may be injured during parotid gland surgery or invaded by malignancies
of the gland and a lower motor neurone on the ipsilateral side will result.

Vestibulo-cochlear Exits from the pons and then passes through the internal auditory meatus. It is
nerve implicated in sensorineural hearing loss. Individuals with sensorineural hearing loss
will localise the sound in webers test to the normal ear. Rinnes test will be reduced
on the affected side but should still work. These two tests will distinguish
sensorineural hearing loss from conductive deafness. In the latter condition webers
test will localise to the affected ear and Rinnes test will be impaired on the affected
side. Surgical lesions affecting this nerve include CNS tumours and basal skull
fractures. It may also be damaged by the administration of ototoxic drugs (of which
gentamicin is the most commonly used in surgical practice).

Glossopharyngeal Exits the pons just above the vagus. Receives sensory fibres from posterior 1/3
nerve tongue, tonsils, pharynx and middle ear (otalgia may occur following tonsillectomy).
It receives visceral afferents from the carotid bodies. It supplies parasympathetic
fibres to the parotid gland via the otic ganglion and motor function to
stylopharyngeaus muscle. The sensory function of the nerve is tested using the gag
reflex.

Vagus nerve Leaves the medulla between the olivary nucleus and the inferior cerebellar
peduncle. Passes through the jugular foramen and into the carotid sheath. Details of
the functions of the vagus nerve are covered in the website under relevant organ
sub headings.

Accessory nerve Exists from the caudal aspect of the brainstem (multiple branches) supplies
trapezius and sternocleidomastoid muscles. The distal portion of this nerve is most
prone to injury during surgical procedures.

Hypoglossal nerve Emerges from the medulla at the preolivary sulcus, passes through the hypoglossal
canal. It lies on the carotid sheath and passes deep to the posterior belly of digastric
to supply muscles of the tongue (except palatoglossus). Its location near the carotid
sheath makes it vulnerable during carotid endarterectomy surgery and damage will
produce ipsilateral defect in muscle function.

Next question
Question 443 of 560

The integrity of which muscle is assessed by the Trendelenburg test?

Sartorius

Quadratus femoris

Semimembranosus

Gluteus medius

Piriformis

Theme from 2011 Exam

Please rate this question:

Discuss and give feedback

Next question

Trendelenburg test

Injury or division of the superior gluteal nerve results in a motor deficit that consists of weakened
abduction of the thigh by gluteus medius, a disabling gluteus medius limp and a compensatory list of
the body to the weakened gluteal side. The compensation results in a gravitational shift so that the
body is supported on the unaffected limb.

When a person is asked to stand on one leg, the gluteus medius usually contracts as soon as the
contralateral leg leaves the floor, preventing the pelvis from dipping towards the unsupported side.
When a person with paralysis of the superior gluteal nerve is asked to stand on one leg, the pelvis
on the unsupported side descends, indicating that the gluteus medius on the affected side is weak or
non functional ( a positive Trendelenburg test).
This eponymous test also refers to a vascular investigation in which tourniquets are placed around
the upper thigh, these can help determine whether saphenofemoral incompetence is present.

Next question
Question 444 of 560

A 52 year old female renal patient needs a femoral catheter to allow for haemodialysis. Which of the
structures listed below is least likely to be encountered during its insertion?

Great saphenous vein

Deep circumflex iliac artery

Superficial circumflex iliac artery

Femoral vein

Femoral branch of the genitofemoral nerve

Femoral access catheters are typically inserted in the region of the femoral triangle. Therefore the
physician may encounter the femoral, vein, nerve, branches of the femoral artery and tributaries of
the femoral vein. The deep circumflex iliac artery arises above the inguinal ligament and is therefore
less likely to be encountered than the superficial circumflex iliac artery which arises below the
inguinal ligament.
Please rate this question:

Discuss and give feedback


Next question

Femoral triangle anatomy

Boundaries
Superiorly Inguinal ligament

Laterally Sartorius

Medially Adductor longus

Floor Iliopsoas, adductor longus and pectineus


Roof  Fascia lata and Superficial fascia
 Superficial inguinal lymph nodes (palpable below the inguinal ligament)
 Long saphenous vein

Image sourced from Wikipedia

Contents

 Femoral vein (medial to lateral)


 Femoral artery-pulse palpated at the mid inguinal point
 Femoral nerve
 Deep and superficial inguinal lymph nodes
 Lateral cutaneous nerve
 Great saphenous vein
 Femoral branch of the genitofemoral nerve

Next question
Question 445 of 560

A 53 year old man with a chronically infected right kidney is due to undergo a nephrectomy. Which of
the following structures would be encountered first during a posterior approach to the hilum of the
right kidney?

Right renal artery

Ureter

Right renal vein

Inferior vena cava

Right testicular vein

The ureter is the most posterior structure at the hilum of the right kidney and would therefore be
encountered first during a posterior approach.
Please rate this question:

Discuss and give feedback


Next question

Renal arteries

 The right renal artery is longer than the left renal artery
 The renal vein/artery/pelvis enter the kidney at the hilum

Relations
Right Anterior- IVC, right renal vein, the head of the pancreas, and the descending part of the
duodenum

Left Anterior- left renal vein, the tail of the pancreas

Branches
 The renal arteries are direct branches off the aorta (upper border of L2- right side and L1 -
left side)
 In 30% there may be accessory arteries (mainly left side). Instead of entering the kidney at
the hilum, they usually pierce the upper or lower part of the organ.
 Before reaching the hilum of the kidney, each artery divides into four or five segmental
branches (renal vein anterior and ureter posterior); which then divide within the sinus into
lobar arteries supplying each pyramid and cortex.
 Each vessel gives off some small inferior suprarenal branches to the suprarenal gland, the
ureter, and the surrounding cellular tissue and muscles.

Next question
Question 446 of 560

A 28 year old man is stabbed outside a nightclub in the upper arm. The median nerve is transected.
Which of the following muscles will demonstrate impaired function as a result?

Palmaris brevis

Second and third interossei

Adductor pollicis

Abductor pollicis longus

Abductor pollicis brevis

Palmaris brevis - Ulnar nerve


Palmar interossei- Ulnar nerve
Adductor pollicis - Ulnar nerve
Abductor pollicis longus - Posterior interosseous nerve
Abductor pollicis brevis - Median nerve

The median nerve innervates all the short muscles of the thumb except the adductor and the deep
head of the short flexor. Palmaris and the interossei are innervated by the ulnar nerve.
Please rate this question:

Discuss and give feedback


Next question

Median nerve

The median nerve is formed by the union of a lateral and medial root respectively from the lateral
(C5,6,7) and medial (C8 and T1) cords of the brachial plexus; the medial root passes anterior to the
third part of the axillary artery. The nerve descends lateral to the brachial artery, crosses to its
medial side (usually passing anterior to the artery). It passes deep to the bicipital aponeurosis and
the median cubital vein at the elbow.
It passes between the two heads of the pronator teres muscle, and runs on the deep surface of
flexor digitorum superficialis (within its fascial sheath).
Near the wrist it becomes superficial between the tendons of flexor digitorum superficialis and flexor
carpi radialis, deep to palmaris longus tendon. It passes deep to the flexor retinaculum to enter the
palm, but lies anterior to the long flexor tendons within the carpal tunnel.

Branches
Region Branch

Upper arm No branches, although the nerve commonly communicates with the
musculocutaneous nerve

Forearm Pronator teres


Flexor carpi radialis
Palmaris longus
Flexor digitorum superficialis
Flexor pollicis longus
Flexor digitorum profundus (only the radial half)

Distal Palmar cutaneous branch


forearm

Hand Motor supply (LOAF)


(Motor)

 Lateral 2 lumbricals
 Opponens pollicis
 Abductor pollicis brevis
 Flexor pollicis brevis

Hand  Over thumb and lateral 2 ½ fingers


(Sensory)  On the palmar aspect this projects proximally, on the dorsal aspect only the
distal regions are innervated with the radial nerve providing the more
proximal cutaneous innervation.

Patterns of damage
Damage at wrist

 e.g. carpal tunnel syndrome


 paralysis and wasting of thenar eminence muscles and opponens pollicis (ape hand
deformity)
 sensory loss to palmar aspect of lateral (radial) 2 ½ fingers

Damage at elbow, as above plus:


 unable to pronate forearm
 weak wrist flexion
 ulnar deviation of wrist

Anterior interosseous nerve (branch of median nerve)

 leaves just below the elbow


 results in loss of pronation of forearm and weakness of long flexors of thumb and index
finger

Topography of the median nerve

Image sourced from Wikipedia

Next question
Question 447 of 560

A 22 year old man sustains a blow to the side of his head with a baseball bat during a fight. He is
initially conscious. However, he subsequently loses consciousness and then dies. Post mortem
examination shows an extradural haematoma. The most likely culprit vessel is a branch of which of
the following?

Middle cerebral artery

Internal carotid artery

Anterior cerebral artery

Maxillary artery

Mandibular artery

The middle meningeal artery is the most likely source of the extradural haematoma in this setting. It
is a branch of the maxillary artery. The middle cerebral artery does not give rise to the middle
meningeal artery. Note that the question is asking for the vessel which gives rise to the middle
meningeal artery ("the likely culprit vessel is a branch of which of the following")
Please rate this question:

Discuss and give feedback


Next question

Middle meningeal artery

 Middle meningeal artery is typically the third branch of the first part of the maxillary artery,
one of the two terminal branches of the external carotid artery. After branching off the
maxillary artery in the infratemporal fossa, it runs through the foramen spinosum to supply
the dura mater (the outermost meninges) .
 The middle meningeal artery is the largest of the three (paired) arteries which supply the
meninges, the others being the anterior meningeal artery and the posterior meningeal artery.
 The middle meningeal artery runs beneath the pterion. It is vulnerable to injury at this point,
where the skull is thin. Rupture of the artery may give rise to an extra dural hematoma.
 In the dry cranium, the middle meningeal, which runs within the dura mater surrounding the
brain, makes a deep indention in the calvarium.
 The middle meningeal artery is intimately associated with the auriculotemporal nerve which
wraps around the artery making the two easily identifiable in the dissection of human
cadavers and also easily damaged in surgery.
Question 448 of 560

A 72 year old man with carcinoma of the lung is undergoing a left pneumonectomy. The left main
bronchus is divided. Which of the following thoracic vertebrae lies posterior to this structure?

T3

T7

T6

T10

T1

The left main bronchus lies at T6. Topographical anatomy of the thorax is important as it helps
surgeons to predict the likely structures to be injured in trauma scenarios (so popular with
examiners)
Please rate this question:

Discuss and give feedback


Next question

Lung anatomy

The right lung is composed of 3 lobes divided by the oblique and transverse fissures. The left lung
has two lobes divided by the oblique fissure.The apex of both lungs is approximately 4cm superior to
the sterno-costal joint of the first rib. Immediately below this is a sulcus created by the subclavian
artery.

Peripheral contact points of the lung

 Base: diaphragm
 Costal surface: corresponds to the cavity of the chest
 Mediastinal surface: Contacts the mediastinal pleura. Has the cardiac impression. Above and
behind this concavity is a triangular depression named the hilum, where the structures which
form the root of the lung enter and leave the viscus. These structures are invested by pleura,
which, below the hilum and behind the pericardial impression, forms the pulmonary ligament
Right lung
Above the hilum is the azygos vein; Superior to this is the groove for the superior vena cava and
right innominate vein; behind this, and nearer the apex, is a furrow for the innominate artery. Behind
the hilum and the attachment of the pulmonary ligament is a vertical groove for the oesophagus; In
front and to the right of the lower part of the oesophageal groove is a deep concavity for the
extrapericardiac portion of the inferior vena cava.

The root of the right lung lies behind the superior vena cava and the right atrium, and below the
azygos vein.

The right main bronchus is shorter, wider and more vertical than the left main bronchus and
therefore the route taken by most foreign bodies.

Image sourced from Wikipedia

Left lung
Above the hilum is the furrow produced by the aortic arch, and then superiorly the groove
accommodating the left subclavian artery; Behind the hilum and pulmonary ligament is a vertical
groove produced by the descending aorta, and in front of this, near the base of the lung, is the lower
part of the oesophagus.

The root of the left lung passes under the aortic arch and in front of the descending aorta.
Image sourced from Wikipedia

Inferior borders of both lungs

 6th rib in mid clavicular line


 8th rib in mid axillary line
 10th rib posteriorly

The pleura runs two ribs lower than the corresponding lung level.

Bronchopulmonary segments
Segment number Right lung Left lung

1 Apical Apical

2 Posterior Posterior

3 Anterior Anterior

4 Lateral Superior lingular

5 Medial Inferior lingular

6 Superior (apical) Superior (apical)


Segment number Right lung Left lung

7 Medial basal Medial basal

8 Anterior basal Anterior basal

9 Lateral basal Lateral basal

10 Posterior basal Posterior basal

Next question
Question 449 of 560

Which of the following regions of the male urethra is entirely surrounded by Bucks fascia?

Preprostatic part

Prostatic part

Membranous part

Spongiose part

None of the above

Theme from 2010 Exam


Bucks fascia is a layer of deep fascia that covers the penis it is continuous with the external
spermatic fascia and the penile suspensory ligament. The membranous part of the urethra may
partially pass through Bucks fascia as it passes into the penis. However, the spongiose part of the
urethra is contained wholly within Bucks fascia.

Image of penile cross section


Bucks fascia corresponds to the layer of deep fascia

Image sourced from Wikipedia

Please rate this question:


Discuss and give feedback

Next question

Urethral anatomy

Female urethra
The female urethra is shorter and more acutely angulated than the male urethra. It is an extra-
peritoneal structure and embedded in the endopelvic fascia. The neck of the bladder is subjected to
transmitted intra-abdominal pressure and therefore deficiency in this area may result in stress
urinary incontinence. Between the layers of the urogenital diaphragm the female urethra is
surrounded by the external urethral sphincter, this is innervated by the pudendal nerve. It ultimately
lies anterior to the vaginal orifice.

Male urethra
In males the urethra is much longer and is divided into four parts.

Pre-prostatic Extremely short and lies between the bladder and prostate gland.It has a stellate lumen
urethra and is between 1 and 1.5cm long.Innervated by sympathetic noradrenergic fibres, as
this region is composed of striated muscles bundles they may contract and prevent
retrograde ejaculation.

Prostatic This segment is wider than the membranous urethra and contains several openings for
urethra the transmission of semen (at the midpoint of the urethral crest).

Membranous Narrowest part of the urethra and surrounded by external sphincter. It traverses the
urethra perineal membrane 2.5cm postero-inferior to the symphysis pubis.

Penile urethra Travels through the corpus spongiosum on the underside of the penis. It is the longest
urethral segment.It is dilated at its origin as the infrabulbar fossa and again in the gland
penis as the navicular fossa. The bulbo-urethral glands open into the spongiose section
of the urethra 2.5cm below the perineal membrane.

The urothelium is transitional in nature near to the bladder and becomes squamous more distally.

Next question
Question 450 of 560

Which of the following statements relating to the knee joint is false?

It is the largest synovial joint in the body

When the knee is fully extended all ligaments of the knee joint are taut

Rupture of the anterior cruciate ligament may result in haemarthrosis

The posterior aspect of the patella is extrasynovial

The joint is innervated by the femoral, sciatic and obturator nerves

The posterior aspect is intrasynovial and the knee itself comprises the largest synovial joint in the
body. It may swell considerably following trauma such as ACL injury. Which may be extremely
painful owing to rich innervation from femoral, sciatic and ( a smaller) contribution from the obturator
nerve. During full extension all ligaments are taut and the knee is locked.
Please rate this question:

Discuss and give feedback


Next question

Knee joint

The knee joint is a synovial joint, the largest and most complicated. It consists of two condylar joints
between the femur and tibia and a sellar joint between the patella and the femur. The tibiofemoral
articular surfaces are incongruent, however, this is improved by the presence of the menisci. The
degree of congruence is related to the anatomical position of the knee joint and is greatest in full
extension.

Knee joint compartments


 Comprised of the patella/femur joint, lateral and medial compartments
(between femur condyles and tibia)
Tibiofemoral  Synovial membrane and cruciate ligaments partially separate the medial
and lateral compartments

Patellofemoral  Ligamentum patellae


 Actions: provides joint stability in full extension

Fibrous capsule
The capsule of the knee joint is a complex, composite structure with contributions from adjacent
tendons.
Anterior The capsule does not pass proximal to the patella. It blends with the tendinous
fibres expansions of vastus medialis and lateralis

Posterior These fibres are vertical and run from the posterior surface of the femoral condyles
fibres to the posterior aspect of the tibial condyle

Attach to the femoral and tibial condyles beyond their articular margins, blending
Medial fibres
with the tibial collateral ligament

Lateral Attach to the femur superior to popliteus, pass over its tendon to head of fibula and
fibres tibial condyle

Bursae
 Subcutaneous prepatellar bursa; between patella and skin
 Deep infrapatellar bursa; between tibia and patellar ligament
Anterior
 Subcutaneous infrapatellar bursa; between distal tibial tuberosity and skin

 Bursa between lateral head of gastrocnemius and joint capsule


 Bursa between fibular collateral ligament and tendon of biceps femoris
Laterally
 Bursa between fibular collateral ligament and tendon of popliteus

 Bursa between medial head of gastrocnemius and the fibrous capsule


 Bursa between tibial collateral ligament and tendons of sartorius, gracilis and
semitendinosus
Medially
 Bursa between the tendon of semimembranosus and medial tibial condyle and
medial head of gastrocnemius

Posterior Highly variable and inconsistent

Ligaments
Medial collateral Medial epicondyle femur to medial tibial condyle: valgus stability
ligament

Lateral collateral Lateral epicondyle femur to fibula head: varus stability


ligament
Anterior cruciate Anterior tibia to lateral intercondylar notch femur: prevents tibia sliding
ligament anteriorly

Posterior cruciate Posterior tibia to medial intercondylar notch femur: prevents tibia
ligament sliding posteriorly

Patellar ligament Central band of the tendon of quadriceps femoris, extends from patella
to tibial tuberosity

Image sourced from Wikipedia


Image sourced from Wikipedia

© Image provided by the University of Sheffield

Menisci
Medial and lateral menisci compensate for the incongruence of the femoral and tibial condyles.
Composed of fibrous tissue.
Medial meniscus is attached to the tibial collateral ligament.
Lateral meniscus is attached to the loose fibres at the lateral edge of the joint and is separate from
the fibular collateral ligament. The lateral meniscus is crossed by the popliteus tendon.
Nerve supply
The knee joint is supplied by the femoral, tibial and common peroneal divisions of the sciatic and by
a branch from the obturator nerve. Hip pathology pain may be referred to the knee.

Blood supply
Genicular branches of the femoral artery, popliteal and anterior tibial arteries all supply the knee
joint.
Next question
Question 451 of 560

In the distal third of the upper arm, where is the musculocutaneous nerve located?

Between the biceps brachii and brachialis muscles

Between the brachialis and brachioradialis muscles

Between the brachioradialis and triceps muscles

Between the brachialis and triceps muscles

Between the humerus and brachialis muscles

The musculocutaneous nerve lies between the biceps and brachialis muscles.
Please rate this question:

Discuss and give feedback


Next question

Musculocutaneous nerve

 Branch of lateral cord of brachial plexus

Path

 It penetrates the coracobrachialis muscle


 Passes obliquely between the biceps brachii and the brachialis to the lateral side of the arm
 Above the elbow it pierces the deep fascia lateral to the tendon of the biceps brachii
 Continues into the forearm as the lateral cutaneous nerve of the forearm

Innervates

 Coracobrachialis
 Biceps brachii
 Brachialis
 Question 452 of 560

 A 48 year old lady is undergoing a left sided adrenalectomy for an adrenal adenoma. The
superior adrenal artery is injured and starts to bleed, from which of the following does this
vessel arise?

Left renal artery

Inferior phrenic artery

Aorta

Splenic

None of the above


The superior adrenal artery is a branch of the inferior phrenic artery.
 Please rate this question:


 Discuss and give feedback

 Next question

 Adrenal gland anatomy

 Anatomy

Location Superomedially to the upper pole of each kidney

Relationships of the right Diaphragm-Posteriorly, Kidney-Inferiorly, Vena Cava-Medially, Hepato-renal


adrenal pouch and bare area of the liver-Anteriorly

Relationships of the left Crus of the diaphragm-Postero- medially, Pancreas and splenic vessels-
adrenal Inferiorly, Lesser sac and stomach-Anteriorly

Superior adrenal arteries- from inferior phrenic artery, Middle adrenal


Arterial supply arteries - from aorta, Inferior adrenal arteries -from renal arteries

Venous drainage of the Via one central vein directly into the IVC
right adrenal

Venous drainage of the Via one central vein into the left renal vein
left adrenal

 Next question

Question 453 of 560

Which of the following does not exit the pelvis through the greater sciatic foramen?

Superior gluteal artery

Internal pudendal vessels

Sciatic nerve

Obturator nerve

Inferior gluteal nerve

The obturator nerve exits through the obturator foramen.


Please rate this question:

Discuss and give feedback


Next question

Greater sciatic foramen

Contents
Nerves  Sciatic Nerve
 Superior and Inferior Gluteal Nerves
 Pudendal Nerve
 Posterior Femoral Cutaneous Nerve
 Nerve to Quadratus Femoris
 Nerve to Obturator internus

Vessels  Superior Gluteal Artery and vein


 Inferior Gluteal Artery and vein
 Internal Pudendal Artery and vein

Piriformis
The piriformis is a landmark for identifying structures passing out of the sciatic notch
 Above piriformis: Superior gluteal vessels
 Below piriformis: Inferior gluteal vessels, sciatic nerve (10% pass through it, <1% above it),
posterior cutaneous nerve of the thigh

Greater sciatic foramen boundaries


Anterolaterally Greater sciatic notch of the ilium

Posteromedially Sacrotuberous ligament

Inferior Sacrospinous ligament and the ischial spine

Superior Anterior sacroiliac ligament

The greater sciatic foramen


Image sourced from Wikipedia

Structures passing between both foramina (Medial to lateral)

 Pudendal nerve
 Internal pudendal artery
 Nerve to obturator internus

Contents of the lesser sciatic foramen

 Tendon of the obturator internus


 Pudendal nerve
 Internal pudendal artery and vein
 Nerve to the obturator internus

Next question
Question 454 of 560

Which statement is false about the foramina of the skull?

The hypoglossal canal transmits the hypoglossal nerve

The foramen spinosum is at the base of the medial pterygoid plate.

The jugular foramen transmits the accessory nerve

The foramen lacerum is located in the sphenoid bone

The stylomastoid foramen transmits the facial nerve

Please rate this question:

Discuss and give feedback

Next question

Foramina of the base of the skull

Foramen Location Contents

Foramen ovale Sphenoid Otic ganglion


bone V3 (Mandibular nerve:3rd branch of
trigeminal)
Accessory meningeal artery
Lesser petrosal nerve
Foramen Location Contents

Emissary veins

Foramen spinosum Sphenoid Middle meningeal artery


bone Meningeal branch of the Mandibular nerve

Foramen rotundum Sphenoid Maxillary nerve (V2)


bone

Foramen lacerum/ Sphenoid Base of the medial pterygoid plate.


carotid canal bone Internal carotid artery*
Nerve and artery of the pterygoid canal

Jugular foramen Temporal Anterior: inferior petrosal sinus


bone Intermediate: glossopharyngeal, vagus, and accessory nerves.
Posterior: sigmoid sinus (becoming the internal jugular vein) and
some meningeal branches from the occipital and ascending
pharyngeal arteries.

Foramen magnum Occipital Anterior and posterior spinal arteries


bone Vertebral arteries
Medulla oblongata

Stylomastoid Temporal Stylomastoid artery


foramen bone Facial nerve

Superior orbital Sphenoid Oculomotor nerve (III)


fissure bone Recurrent meningeal artery
Trochlear nerve (IV)
Lacrimal, frontal and nasociliary branches of ophthalmic nerve (V1)
Abducent nerve (VI)
Superior ophthalmic vein
*= In life the foramen lacerum is occluded by a cartilagenous plug. The ICA initially passes into the
carotid canal which ascends superomedially to enter the cranial cavity through the foramen lacerum.

Base of skull anatomical overview

Image sourced from Wikipedia

Next question
Question 455 of 560

An 80 year old lady with a caecal carcinoma is undergoing a right hemicolectomy performed through
a transverse incision. The procedure is difficult and the incision is extended medially by dividing the
rectus sheath. Brisk arterial haemorrhage ensues. From which of the following does the damaged
vessel originate?

Internal iliac artery

External iliac artery

Superior vesical artery

Inferior vesical artery

None of the above

The vessel damaged is the epigastric artery. This originates from the external iliac artery (see
below).

Please rate this question:

Discuss and give feedback

Next question

Epigastric artery

The inferior epigastric artery arises from the external iliac artery immediately above the inguinal
ligament. It then passes along the medial margin of the deep inguinal ring. From here it continues
superiorly to lie behind the rectus abdominis muscle.

This is illustrated below:


Image sourced from Wikipedia

Next question
Question 456 of 560

A 73 year old man has a large abdominal aortic aneurysm. During a laparotomy for planned surgical
repair the surgeons find the aneurysm is far more proximally located and lies near the origin of the
SMA. During the dissection a vessel lying transversely across the aorta is injured. What is this
vessel most likely to be?

Left renal vein

Right renal vein

Inferior mesenteric artery

Ileocolic artery

Middle colic artery

Theme from April 2012 Exam


The left renal vein runs across the surface of the aorta and may require deliberate ligation during
juxtarenal aneurysm repair.

Please rate this question:

Discuss and give feedback

Next question

Abdominal aorta

Abdominal aortic topography

Origin T12
Termination L4

Posterior relations L1-L4 Vertebral bodies

Anterior relations Lesser omentum


Liver
Left renal vein
Inferior mesenteric vein
Third part of duodenum
Pancreas
Parietal peritoneum
Peritoneal cavity

Right lateral relations Right crus of the diaphragm


Cisterna chyli
Azygos vein
IVC (becomes posterior distally)

Left lateral relations 4th part of duodenum


Duodenal-jejunal flexure
Left sympathetic trunk

The abdominal aorta


Image sourced from Wikipedia

Next question
Question 457 of 560

Which of the following is not a branch of the posterior cord of the brachial plexus?

Thoracodorsal nerve

Axillary nerve

Radial nerve

Lower subscapular nerve

Musculocutaneous nerve

Mnemonic branches off the posterior cord

S ubscapular (upper and lower)


T horacodorsal
A xillary
R adial

The musculocutaneous nerve is a branch off the lateral cord.


Please rate this question:

Discuss and give feedback


Next question

Brachial plexus

Origin Anterior rami of C5 to T1

Sections of the  Roots, trunks, divisions, cords, branches


plexus  Mnemonic:Real Teenagers Drink Cold Beer

Roots  Located in the posterior triangle


 Pass between scalenus anterior and medius
Trunks  Located posterior to middle third of clavicle
 Upper and middle trunks related superiorly to the subclavian artery
 Lower trunk passes over 1st rib posterior to the subclavian artery

Divisions Apex of axilla

Cords Related to axillary artery

Diagram illustrating the branches of the brachial plexus

Image sourced from Wikipedia

Cutaneous sensation of the upper limb


Image sourced from Wikipedia

Next question
Question 458 of 560

A 18 year old man presents with an indirect inguinal hernia and undergoes surgery. The deep
inguinal ring is exposed and held with a retractor at its medial aspect. Which structure is most likely
to lie under the retractor?

Ureter

Inferior epigastric artery

Internal iliac vein

Femoral artery

Lateral border of rectus abdominis

Boundaries of the deep inguinal ring:

 Superolaterally - transversalis fascia


 Inferomedially - inferior epigastric artery

The deep inguinal ring is closely related to the inferior epigastric artery. The inferior epigastric artery
forms part of the structure referred to as Hesselbach's triangle.
Image sourced from Wikipedia

Please rate this question:

Discuss and give feedback


Next question

Inguinal canal

Location

 Above the inguinal ligament


 The inguinal canal is 4cm long
 The superficial ring is located anterior to the pubic tubercle
 The deep ring is located approximately 1.5-2cm above the half way point between the
anterior superior iliac spine and the pubic tubercle

Boundaries of the inguinal canal


Floor  External oblique aponeurosis
 Inguinal ligament
 Lacunar ligament
Roof  Internal oblique
 Transversus abdominis

Anterior wall External oblique aponeurosis

Posterior wall  Transversalis fascia


 Conjoint tendon

Laterally  Internal ring


 Fibres of internal oblique

Medially  External ring


 Conjoint tendon

Contents
Males Spermatic cord and ilioinguinal As it passes through the canal the spermatic cord
nerve has 3 coverings:

 External spermatic fascia from external


oblique aponeurosis
 Cremasteric fascia
 Internal spermatic fascia

Females Round ligament of uterus and


ilioinguinal nerve

Related anatomy of the inguinal region


The boundaries of Hesselbachs triangle are commonly tested and illustrated below:
Image sourced from Wikipedia

The image below demonstrates the close relationship of the vessels to the lower limb with the
inguinal canal. A fact to be borne in mind when repairing hernial defects in this region.

Image sourced from Wikipedia

Next question
Question 459 of 560

A 73 year old man presents with a tumour at the tip of his tongue. To which of the following regions
will the tumour initially metastasise?

Sub mental nodes

Ipsilateral deep cervical nodes

Tonsil

Ipsilateral superficial cervical nodes

Contralateral deep cervical nodes

Please rate this question:

Discuss and give feedback


Next question

Lymphatic drainage of the tongue

 The lymphatic drainage of the anterior two thirds of the tongue shows only minimal
communication of lymphatics across the midline, so metastasis to the ipsilateral nodes is
usual.
 The lymphatic drainage of the posterior third of the tongue have communicating networks, as
a result early bilateral nodal metastases are more common in this area.
 Lymphatics from the tip of the tongue usually pass to the sub mental nodes and from there to
the deep cervical nodes.
 Lymphatics from the mid portion of the tongue usually drain to the submandibular nodes and
then to the deep cervical nodes. Mid tongue tumours that are laterally located will usually
drain to the ipsilateral deep cervical nodes, those from more central regions may have
bilateral deep cervical nodal involvement.

Next question
Question 460 of 560

A 78 year old man is undergoing a femoro-popliteal bypass graft. The operation is not progressing
well and the surgeon is complaining of poor access. Retraction of which of the following structures
will improve access to the femoral artery in the groin?

Quadriceps

Adductor longus

Adductor magnus

Pectineus

Sartorius

At the lower border of the femoral triangle the femoral artery passes under the sartorius muscle. This
can be retracted to improve access.
Please rate this question:

Discuss and give feedback


Next question

Femoral triangle anatomy

Boundaries
Superiorly Inguinal ligament

Laterally Sartorius

Medially Adductor longus

Floor Iliopsoas, adductor longus and pectineus


Roof  Fascia lata and Superficial fascia
 Superficial inguinal lymph nodes (palpable below the inguinal ligament)
 Long saphenous vein

Image sourced from Wikipedia

Contents

 Femoral vein (medial to lateral)


 Femoral artery-pulse palpated at the mid inguinal point
 Femoral nerve
 Deep and superficial inguinal lymph nodes
 Lateral cutaneous nerve
 Great saphenous vein
 Femoral branch of the genitofemoral nerve

Next question
Question 461 of 560

A builder falls off a ladder whilst laying roof tiles. He sustains a burst fracture of L3. The MRI scan
shows complete nerve transection at this level, as a result of the injury. Which clinical sign will not be
present initially?

Flaccid paralysis of the legs

Extensor plantar response

Sensory loss in the legs

Incontinence

Areflexia

In lower motor neuron lesions everything is reduced

The main purpose of this question is to differentiate the features of an UMN lesion and a LMN lesion.
The features of a LMN lesion include:

 Flaccid paralysis of muscles supplied


 Atrophy of muscles supplied.
 Loss of reflexes of muscles supplied.
 Muscles fasciculation

For lesions below L1 LMN signs will occur. Hence in an L3 lesion, there will be loss of the patella
reflex but there will be no extensor plantar reflex.
Please rate this question:

Discuss and give feedback


Next question

Spinal cord

 Located in a canal within the vertebral column that affords it structural support.
 Rostrally it continues to the medulla oblongata of the brain and caudally it tapers at a level
corresponding to the L1-2 interspace (in the adult), a central structure, the filum terminale
anchors the cord to the first coccygeal vertebra.
 The spinal cord is characterised by cervico-lumbar enlargements and these, broadly
speaking, are the sites which correspond to the brachial and lumbar plexuses respectively.

There are some key points to note when considering the surgical anatomy of the spinal cord:

* During foetal growth the spinal cord becomes shorter than the spinal canal, hence the adult site of
cord termination at the L1-2 level.

* Due to growth of the vertebral column the spine segmental levels may not always correspond to
bony landmarks as they do in the cervical spine.

* The spinal cord is incompletely divided into two symmetrical halves by a dorsal median
sulcus andventral median fissure. Grey matter surrounds a central canal that is continuous
rostrally with the ventricular system of the CNS.

* The grey matter is sub divided cytoarchitecturally into Rexeds laminae.

* Afferent fibres entering through the dorsal roots usually terminate near their point of entry but may
travel for varying distances in Lissauers tract. In this way they may establish synaptic connections
over several levels

* At the tip of the dorsal horn are afferents associated with nociceptive stimuli. The ventral horn
contains neurones that innervate skeletal muscle.

The key point to remember when revising CNS anatomy is to keep a clinical perspective in mind. So
it is worth classifying the ways in which the spinal cord may become injured. These include:

 Trauma either direct or as a result of disc protrusion


 Neoplasia either by direct invasion (rare) or as a result of pathological vertebral fracture
 Inflammatory diseases such as Rheumatoid disease, or OA (formation of osteophytes
compressing nerve roots etc.
 Vascular either as a result of stroke (rare in cord) or as complication of aortic dissection
 Infection historically diseases such as TB, epidural abscesses.

The anatomy of the cord will, to an extent dictate the clinical presentation. Some points/ conditions to
remember:

 Brown- Sequard syndrome-Hemisection of the cord producing ipsilateral loss of


proprioception and upper motor neurone signs, plus contralateral loss of pain and
temperature sensation. The explanation of this is that the fibres decussate at different levels.
 Lesions below L1 will tend to present with lower motor neurone signs

Next question
Question 462 of 560

A 56 year old machinist has his arm entrapped in a steel grinder and is brought to the emergency
department. On examination, he is unable to extend his metacarpophalangeal joints and abduct his
shoulder. He has weakness of his elbow and wrist. What has been injured?

Ulnar nerve

Axillary nerve

Medial cord of brachial plexus

Lateral cord of brachial plexus

Posterior cord of brachial plexus

The posterior cord gives rise to:

 Radial nerve ((innervates the triceps, brachioradialis, wrist extensors, and finger extensors)
 Axillary nerve (innervates deltoid and teres minor)
 Upper subscapular nerve (innervates subscapularis)
 Lower subscapular nerve (innervates teres major and subscapularis)
 Thoracodorsal nerve (innervates latissimus dorsi)

Theme from September 2012 exam


This is a description of a posterior cord lesion. Remember that the posterior cord gives rise to the
axillary and radial nerve.
Please rate this question:

Discuss and give feedback


Next question

Cords of the brachial plexus

The brachial plexus cords are described according to their relationship with the axillary artery. The
cords pass over the 1st rib near to the dome of the lung and pass beneath the clavicle immediately
posterior to the subclavian artery.

Lateral cord
 Anterior divisions of the upper and middle trunks form the lateral cord
 Origin of the lateral pectoral nerve (C5, C6, C7)

Medial cord

 Anterior division of the lower trunk forms the medial cord


 Origin of the medial pectoral nerve (C8, T1), the medial brachial cutaneous nerve (T1), and
the medial antebrachial cutaneous nerve (C8, T1)

Posterior cord

 Formed by the posterior divisions of the 3 trunks (C5-T1)


 Origin of the upper and lower subscapular nerves (C7, C8 and C5, C6, respectively) and the
thoracodorsal nerve to the latissimus dorsi (also known as the middle subscapular nerve, C6,
C7, C8), axillary and radial nerve

Next question
Question 463 of 560

A 66 year old man with peripheral vascular disease is undergoing a below knee amputation. In which
of the lower leg compartments does peroneus brevis lie?

Lateral compartment

Anterior compartment

Superficial posterior compartment

Deep posterior compartment

None of the above

The interosseous membrane separates the anterior and posterior compartments. The deep and
superficial compartments are separated by the deep transverse fascia. The peroneus brevis is part
of the lateral compartment.
Please rate this question:

Discuss and give feedback


Next question

Fascial compartments of the leg

Compartments of the thigh

Formed by septae passing from the femur to the fascia lata.


Compartment Nerve Muscles Blood supply

Anterior compartment Femoral  Iliacus Femoral artery


 Tensor fasciae latae
 Sartorius
 Quadriceps femoris

Medial compartment Obturator  Adductor Profunda femoris artery


longus/magnus/brevis and obturator artery
Compartment Nerve Muscles Blood supply

 Gracilis
 Obturator externus

Posterior Sciatic  Semimembranosus Branches of Profunda


compartment (2  Semitendinosus femoris artery
layers)  Biceps femoris

Compartments of the lower leg


Separated by the interosseous membrane (anterior and posterior compartments), anterior fascial
septum (separate anterior and lateral compartments) and posterior fascial septum (separate lateral
and posterior compartments)

Blood
Compartment Nerve Muscles supply

Anterior Deep  Tibialis anterior Anterior


compartment peroneal  Extensor digitorum longus tibial artery
nerve  Extensor hallucis longus
 Peroneus tertius

Posterior Tibial  Muscles: deep and superficial Posterior


compartment compartments (separated by deep tibial
transverse fascia)
 Deep: Flexor hallucis longus, Flexor
digitalis longus, Tibialis posterior,
Popliteus
 Superficial: Gastrocnemius, Soleus,
Plantaris

Lateral Superficial  Peroneus longus/brevis Peroneal


compartment peroneal artery
Next question
Question 464 of 560

A 70 year old man is due to undergo an arterial bypass procedure for claudication and foot
ulceration. The anterior tibial artery will form the site of the distal arterial anastomosis. Which of the
following structures is not closely related to it?

Interosseous membrane

Deep peroneal nerve

Tibialis posterior

Extensor hallucis longus

Dorsalis pedis artery

As an artery of the anterior compartment, the anterior tibial artery is closely related to tibialis anterior.
Please rate this question:

Discuss and give feedback


Next question

Anterior tibial artery

 Begins opposite the distal border of popliteus


 Terminates in front of the ankle, continuing as the dorsalis pedis artery
 As it descends it lies on the interosseous membrane, distal part of the tibia and front of the
ankle joint
 Passes between the tendons of extensor digitorum and extensor hallucis longus distally
 It is related to the deep peroneal nerve, it lies anterior to the middle third of the vessel and
lateral to it in the lower third

Next question
Question 465 of 560

Which of the muscles below does not cause lateral rotation of the hip?

Obturator internus

Quadratus femoris

Gemellus inferior

Piriformis

Pectineus

Mnemonic lateral hip rotators: P-GO-GO-Q (top to bottom)

Piriformis
Gemellus superior
Obturator internus
Gemellus inferior
Obturator externus
Quadratus femoris

Pectineus adducts and medially rotates the femur.


Please rate this question:

Discuss and give feedback


Next question

Hip joint

 Head of femur articulates with acetabulum of the pelvis


 Both covered by articular hyaline cartilage
 The acetabulum forms at the union of the ilium, pubis, and ischium
 The triradiate cartilage (Y-shaped growth plate) separates the pelvic bones
 The acetabulum holds the femoral head by the acetabular labrum
 Normal angle between femoral head and femoral shaft is 130o

Ligaments
 Transverse ligament: joints anterior and posterior ends of the articular cartilage
 Head of femur ligament (ligamentum teres): acetabular notch to the fovea. Contains arterial
supply to head of femur in children.

Image sourced from Wikipedia

Image sourced from Wikipedia


Extracapsular ligaments

 Iliofemoral ligament: inverted Y shape. Anterior iliac spine to the trochanteric line
 Pubofemoral ligament: acetabulum to lesser trochanter
 Ischiofemoral ligament: posterior support. Ischium to greater trochanter.

Blood supply
Medial circumflex femoral and lateral circumflex femoral arteries (Branches of profunda femoris).
Also from the inferior gluteal artery. These form an anastomosis and travel to up the femoral neck to
supply the head.
Next question
Question 466 of 560

Which of the following is not a content of the posterior triangle of the neck?

Spinal accessory nerve

Phrenic nerve

External jugular vein

Occipital lymph nodes

Internal jugular vein

The IJV does not lie in the posterior triangle. However, the terminal branches of the external jugular
vein do.
Please rate this question:

Discuss and give feedback


Next question

Posterior triangle of the neck

Boundaries
Apex Sternocleidomastoid and the Trapezius muscles at the Occipital bone

Anterior Posterior border of the Sternocleidomastoid

Posterior Anterior border of the Trapezius

Base Middle third of the clavicle


Image sourced from Wikipedia

Contents
Nerves  Accessory nerve
 Phrenic nerve
 Three trunks of the brachial plexus
 Branches of the cervical plexus: Supraclavicular nerve, transverse cervical
nerve, great auricular nerve, lesser occipital nerve

Vessels  External jugular vein


 Subclavian artery

Muscles  Inferior belly of omohyoid


 Scalene

Lymph  Supraclavicular
nodes  Occipital

Next question
Question 467 of 560

Which nerve is at risk during submandibular gland excision?

Maxillary nerve

Buccal nerve

Zygomatic nerve

Marginal mandibular nerve

Cervical nerve

The marginal mandibular nerve lies deep to platysma. It supplies the depressor anguli oris and the
depressor labii inferioris. If injured it may lead to facial asymmetry and dribbling.
Please rate this question:

Discuss and give feedback


Next question

Submandibular gland

Relations of the submandibular gland


Superficial Platysma, deep fascia and mandible
Submandibular lymph nodes
Facial vein (facial artery near mandible)
Marginal mandibular nerve
Cervical branch of the facial nerve

Deep Facial artery (inferior to the mandible)


Mylohoid muscle
Sub mandibular duct
Hyoglossus muscle
Lingual nerve
Submandibular ganglion
Hypoglossal nerve

Submandibular duct (Wharton's duct)

 Opens lateral to the lingual frenulum on the anterior floor of mouth.


 5 cm length
 Lingual nerve wraps around Wharton's duct. As the duct passes forwards it crosses medial
to the nerve to lie above it and then crosses back, lateral to it, to reach a position below the
nerve.

Innervation

 Sympathetic innervation- Derived from superior cervical ganglion


 Parasympathetic innervation- Submandibular ganglion via lingual nerve

Arterial supply
Branch of the facial artery. The facial artery passes through the gland to groove its deep surface. It
then emerges onto the face by passing between the gland and the mandible.

Venous drainage
Anterior facial vein (lies deep to the Marginal Mandibular nerve)

Lymphatic drainage
Deep cervical and jugular chains of nodes
Next question
Question 468 of 560

In a patient with a carcinoma of the distal sigmoid colon, what is the most likely source of its blood
supply?

Ileocolic artery

External iliac artery

Internal iliac artery

Superior mesenteric artery

Inferior mesenteric artery

Theme from September 2013 Exam


During a high anterior resection of such tumours, the inferior mesenteric artery is ligated. Note that
the branches (mainly middle rectal branch) of the internal iliac artery are important in maintaining
vascularity of the rectal stump and hence the integrity of the anastomoses.
Please rate this question:

Discuss and give feedback


Next question

Rectum

The rectum is approximately 12 cm long. It is a capacitance organ. It has both intra and
extraperitoneal components. The transition between the sigmoid colon is marked by the
disappearance of the tenia coli.The extra peritoneal rectum is surrounded by mesorectal fat that also
contains lymph nodes. This mesorectal fatty layer is removed surgically during rectal cancer surgery
(Total Mesorectal Excision). The fascial layers that surround the rectum are important clinical
landmarks, anteriorly lies the fascia of Denonvilliers. Posteriorly lies Waldeyers fascia.

Extra peritoneal rectum

 Posterior upper third


 Posterior and lateral middle third
 Whole lower third
Relations
Anteriorly (Males) Rectovesical pouch
Bladder
Prostate
Seminal vesicles

Anteriorly (Females) Recto-uterine pouch (Douglas)


Cervix
Vaginal wall

Posteriorly Sacrum
Coccyx
Middle sacral artery

Laterally Levator ani


Coccygeus

Arterial supply
Superior rectal artery

Venous drainage
Superior rectal vein

Lymphatic drainage

 Mesorectal lymph nodes (superior to dentate line)


 Internal iliac and then para-aortic nodes
 Inguinal nodes (inferior to dentate line)

Next question
Question 469 of 560

Which of these openings transmits the facial nerve into the temporal bone?

Internal acoustic meatus

Foramen lacerum

Foramen spinosum

Stylomastoid foramen

Jugular foramen

It enters the temporal bone through the internal acoustic meatus and exits through the stylomastoid
foramen.
Please rate this question:

Discuss and give feedback


Next question

Facial nerve

The facial nerve is the main nerve supplying the structures of the second embryonic branchial arch.
It is predominantly an efferent nerve to the muscles of facial expression, digastric muscle and also to
many glandular structures. It contains a few afferent fibres which originate in the cells of its genicular
ganglion and are concerned with taste.

Supply - 'face, ear, taste, tear'

 Face: muscles of facial expression


 Ear: nerve to stapedius
 Taste: supplies anterior two-thirds of tongue
 Tear: parasympathetic fibres to lacrimal glands, also salivary glands

Path
Subarachnoid path
 Origin: motor- pons, sensory- nervus intermedius
 Pass through the petrous temporal bone into the internal auditory meatus with the
vestibulocochlear nerve. Here they combine to become the facial nerve.

Facial canal path

 The canal passes superior to the vestibule of the inner ear


 At the medial aspect of the middle ear, it becomes wider and contains the geniculate
ganglion.

- 3 branches:
1. greater petrosal nerve
2. nerve to stapedius
3. chorda tympani

Stylomastoid foramen

 Passes through the stylomastoid foramen (tympanic cavity anterior and mastoid antrum
posteriorly)
 Posterior auricular nerve and branch to posterior belly of digastric and stylohyoid muscle

Face
Enters parotid gland and divides into 5 branches:

 Temporal branch
 Zygomatic branch
 Buccal branch
 Marginal mandibular branch
 Cervical branch

Next question
Question 470 of 560

A motor cyclist is involved in a road traffic accident causing severe right shoulder injuries. He is
found to have an adducted, medially rotated shoulder. The elbow is fully extended and the forearm
pronated. Which is the most likely diagnosis?

C8, T1 root lesion

C5, C6 root lesion

Radial nerve lesion

Ulnar nerve lesion

Axillary nerve lesion

Erbs Palsy C5, C6 lesion


The features include:

 Waiter's tip position


 Loss of shoulder abduction (deltoid and supraspinatus paralysis)
 Loss of external rotation of the shoulder (paralysis of infraspinatus)
 Loss of elbow flexion (paralysis of biceps, brachialis and brachioradialis)
 Loss of forearm supination (paralysis of Biceps)

The motorcyclist has had an Erb's palsy (C5, C6 root lesion). This is commonly known to be
associated with birth injury when a baby has a shoulder dystocia.
Please rate this question:

Discuss and give feedback


Next question

Brachial plexus

Origin Anterior rami of C5 to T1

Sections of the  Roots, trunks, divisions, cords, branches


plexus  Mnemonic:Real Teenagers Drink Cold Beer

Roots  Located in the posterior triangle


 Pass between scalenus anterior and medius

Trunks  Located posterior to middle third of clavicle


 Upper and middle trunks related superiorly to the subclavian artery
 Lower trunk passes over 1st rib posterior to the subclavian artery

Divisions Apex of axilla

Cords Related to axillary artery

Diagram illustrating the branches of the brachial plexus

Image sourced from Wikipedia

Cutaneous sensation of the upper limb


Image sourced from Wikipedia

Next question
Question 471 of 560

A patient is due to undergo a right hemicolectomy for a carcinoma of the caecum. Which of the
following vessels will require high ligation to provide optimal oncological control?

Middle colic artery

Inferior mesenteric artery

Superior mesenteric artery

Ileo-colic artery

None of the above

The ileo - colic artery supplies the caecum and would require high ligation during a right
hemicolectomy. The middle colic artery should generally be preserved when resecting a caecal
lesion.
This question is essentially asking you to name the vessel supplying the caecum. The SMA does not
directly supply the caecum, it is the ileocolic artery which does this.
Please rate this question:

Discuss and give feedback


Next question

Caecum

Location  Proximal right colon below the ileocaecal valve


 Intraperitoneal

Posterior relations  Psoas


 Iliacus
 Femoral nerve
 Genitofemoral nerve
 Gonadal vessels

Anterior relations Greater omentum


Arterial supply Ileocolic artery

Lymphatic drainage Mesenteric nodes accompany the venous drainage

 The caecum is the most distensible part of the colon and in complete large bowel obstruction
with a competent ileocaecal valve the most likely site of eventual perforation.

Next question
Question 472 of 560

A 40-year-old man presents with pain in his lower back and 'sciatica' for the past three days. He
describes bending down to pick up a washing machine when he felt 'something go'. He now has
severe pain radiating from his back down the right leg. On examination he describes paraesthesia
over the anterior aspect of the right knee and the medial aspect of his calf. Power is intact and the
right knee reflex is diminished. The femoral stretch test is positive on the right side. Which nerve or
nerve root is most likely to be affected?

Common peroneal nerve

Lateral cutaneous nerve of the thigh

L5

L3

L1

Please rate this question:

Discuss and give feedback


Next question

Prolapsed disc

A prolapsed lumbar disc usually produces clear dermatomal leg pain associated with neurological
deficits.

Features

 Leg pain usually worse than back


 Pain often worse when sitting

The table below demonstrates the expected features according to the level of compression:

L3 nerve root compression Sensory loss over anterior thigh/knee


Weak quadriceps
Reduced knee reflex
Positive femoral stretch test

L5 nerve root compression Sensory loss dorsum of foot


Weakness in foot and big toe dorsiflexion
Reflexes intact
Positive sciatic nerve stretch test

S1 nerve root compression Sensory loss posterolateral aspect of leg and lateral aspect of foot
Weakness in plantar flexion of foot
Reduced ankle reflex
Positive sciatic nerve stretch test

Management

 Similar to that of other musculoskeletal lower back pain: analgesia, physiotherapy, exercises
 Persistent symptoms, muscular weakness, bladder or bowel dysfunction are indications for
urgent MRI scanning to delineate the disease extent to allow surgical planning
 Plain spinal x-rays have no useful role in establishing the extent of disk disease

Next question
Question 473 of 560

A 72 year old man is undergoing a repair of an abdominal aortic aneurysm. The aorta is cross
clamped both proximally and distally. The proximal clamp is applied immediately inferior to the renal
arteries. Both common iliac arteries are clamped distally. A longitudinal aortotomy is performed.
After evacuating the contents of the aneurysm sac a significant amount of ongoing bleeding is
encountered. This is most likely to originate from:

The coeliac axis

Testicular artery

Splenic artery

Superior mesenteric artery

Lumbar arteries

The lumbar arteries are posteriorly sited and are a common cause of back bleeding during aortic
surgery. The other vessels cited all exit the aorta in the regions that have been cross clamped.

Please rate this question:

Discuss and give feedback

Next question

Abdominal aortic branches

Branches Level Paired Type


Branches Level Paired Type

Inferior phrenic T12 (Upper border) Yes Parietal

Coeliac T12 No Visceral

Superior mesenteric L1 No Visceral

Middle suprarenal L1 Yes Visceral

Renal L1-L2 Yes Visceral

Gonadal L2 Yes Visceral

Lumbar L1-L4 Yes Parietal

Inferior mesenteric L3 No Visceral

Median sacral L4 No Parietal

Common iliac L4 Yes Terminal

Next question
Question 474 of 560

Which of the following statements relating to sartorius is untrue?

It is supplied by the femoral nerve

It forms the lateral boundary of the femoral triangle

The middle third forms the roof of the adductor canal

It is a flexor of the hip and knee

It inserts into the medial femoral condyle

It inserts into the medial aspect of the upper part of the tibia.
Please rate this question:

Discuss and give feedback


Next question

Sartorius

 Longest strap muscle in the body


 Most superficial muscle in the anterior compartment of the thigh

Origin Anterior superior iliac spine

Insertion Medial surface of the of the body of the tibia (upper part). It inserts anterior to
gracilis and semitendinosus

Nerve Supply Femoral nerve (L2,3)

Action  Flexor of the hip and knee, slight abducts the thigh and rotates it laterally
 It assists with medial rotation of the tibia on the femur. For example it
would play a pivotal role in placing the right heel onto the left knee ( and
vice versa)

Important The middle third of this muscle, and its strong underlying fascia forms the roof of
relations the adductor canal , in which lie the femoral vessels, the saphenous nerve and the
nerve to vastus medialis.
Next question
Question 475 of 560

A 63 year old man undergoes a radical cystectomy for carcinoma of the bladder. During the
procedure there is considerable venous bleeding. What is the primary site of venous drainage of the
urinary bladder?

Vesicoprostatic venous plexus

Internal iliac vein

External iliac vein

Gonadal vein

Common iliac vein

The urinary bladder has a rich venous plexus surrounding it, this drains subsequently into the
internal iliac vein. The vesicoprostatic plexus may be a site of considerable venous bleeding during
cystectomy.

Please rate this question:

Discuss and give feedback

Next question

Bladder

The empty bladder is contained within the pelvic cavity. It is usually a three sided pyramid. The apex
of the bladder points forwards towards the symphysis pubis and the base lies immediately anterior to
the rectum or vagina. Continuous with the apex is the median umbilical ligament, during
development this was the site of the uranchus.
The inferior aspect of the bladder is retroperitoneal and the superior aspect covered by peritoneum.
As the bladder distends it will tend to separate the peritoneum from the fascia of transversalis. For
this reason a bladder that is distended due to acute urinary retention may be approached with a
suprapubic catheter that avoids entry into the peritoneal cavity.
The trigone is the least mobile part of the bladder and forms the site of the ureteric orifices and
internal urethral orifice. In the empty bladder the ureteric orifices are approximately 2-3cm apart, this
distance may increase to 5cm in the distended bladder.

Arterial supply
The superior and inferior vesical arteries provide the main blood supply to the bladder. These are
branches of the internal iliac artery.

Venous drainage
In males the bladder is drained by the vesicoprostatic venous plexus. In females the bladder is
drained by the vesicouterine venous plexus. In both sexes this venous plexus will ultimately drain to
the internal iliac veins.

Lymphatic drainage
Lymphatic drainage is predominantly to the external iliac nodes, internal iliac and obturator nodes
also form sites of bladder lymphatic drainage.

Innervation
Parasympathetic nerve fibres innervate the bladder from the pelvic splanchnic nerves. Sympathetic
nerve fibres are derived from L1 and L2 via the hypogastric nerve plexuses. The parasympathetic
nerve fibres will typically cause detrusor muscle contraction and result in voiding. The muscle of the
trigone is innervated by the sympathetic nervous system. The external urethral sphincter is under
conscious control. During bladder filling the rate of firing of nerve impulses to the detrusor muscle is
low and receptive relaxation occurs. At higher volumes and increased intra vesical pressures the
rate of neuronal firing will increase and eventually voiding will occur.

Next question
Question 476 of 560

A man sustains a laceration between the base of the little finger and wrist. Several weeks after the
injury there is loss of thumb adduction power. Which nerve is most likely to have been injured?

Superficial ulnar nerve

Deep ulnar nerve

Median nerve

Radial nerve

Recurrent branch of median nerve

Theme from 2009 Exam


Theme from April 2014 Exam

Please rate this question:

Discuss and give feedback

Next question

Ulnar nerve injury at wrist

Branches of the ulnar nerve in the wrist and hand


At the wrist the ulnar nerve divides into superficial and deep branches. The superficial branch lies
deep to the palmaris brevis. It divides into two; to produce digital nerves, which innervate the skin of
the medial third of the palm and the palmar surface of one and a half fingers.
The deep branch arises from the nerve on the flexor retinaculum lateral to the pisiform bone. It
passes posteriorly between the abductor and short flexor of the little finger supplying them, and
supplying and piercing the opponens digiti minimi near its origin from the flexor retinaculum, turns
laterally over the distal surface of the Hook of the Hamate bone. It eventually passes between the
two heads of adductor pollicis with the deep palmar arch and ends in the first dorsal interosseous
muscle. In the palm the deep branch also innervates the lumbricals and interosseous muscles.

Next question
Question 477 of 560

A 60 year old female is undergoing a Whipples procedure for adenocarcinoma of the pancreas. As
the surgeons begin to mobilise the pancreatic head they identify a large vessel passing inferiorly
over the anterior aspect of the uncinate process. What is it likely to be?

Superior mesenteric artery

Coeliac axis

Inferior mesenteric artery

Aorta

Left gastric artery

Theme from January 2012 Exam


The superior mesenteric artery arises from the aorta and passes anterior to the lower part of the
pancreas. Invasion of this structure is a relative contra indication to resectional surgery.
Please rate this question:

Discuss and give feedback


Next question

Pancreas

The pancreas is a retroperitoneal organ and lies posterior to the stomach. It may be accessed
surgically by dividing the peritoneal reflection that connects the greater omentum to the transverse
colon. The pancreatic head sits in the curvature of the duodenum. Its tail lies close to the hilum of
the spleen, a site of potential injury during splenectomy.

Relations
Posterior to the pancreas
Pancreatic head Inferior vena cava
Common bile duct
Right and left renal veins
Superior mesenteric vein and artery
Pancreatic neck Superior mesenteric vein, portal vein

Pancreatic body- Left renal vein


Crus of diaphragm
Psoas muscle
Adrenal gland
Kidney
Aorta

Pancreatic tail Left kidney

Anterior to the pancreas


Pancreatic head 1st part of the duodenum
Pylorus
Gastroduodenal artery
SMA and SMV(uncinate process)

Pancreatic body Stomach


Duodenojejunal flexure

Pancreatic tail Splenic hilum

Superior to the pancreas


Coeliac trunk and its branches common hepatic artery and splenic artery

Grooves of the head of the pancreas


2nd and 3rd part of the duodenum

Arterial supply

 Head: pancreaticoduodenal artery


 Rest: splenic artery

Venous drainage

 Head: superior mesenteric vein


 Body and tail: splenic vein
Ampulla of Vater

 Merge of pancreatic duct and common bile duct


 Is an important landmark, halfway along the second part of the duodenum, that marks the
anatomical transition from foregut to midgut (also the site of transition between regions
supplied by coeliac trunk and SMA).

Image sourced from Wikipedia

Next question
Question 478 of 560

A 23 year old man has a cannula inserted into his cephalic vein. Through which structure does the
cephalic vein pass?

Interosseous membrane

Triceps

Pectoralis major

Clavipectoral fascia

Tendon of biceps

The cephalic vein is a favored vessel for arteriovenous fistula formation and should be preserved in
patients with end stage renal failure

The cephalic vein penetrates the calvipectoral fascia (but not the pectoralis major) prior to
terminating in the axillary vein.
Please rate this question:

Discuss and give feedback


Next question

Cephalic vein

Path

 Dorsal venous arch drains laterally into the cephalic vein


 Crosses the anatomical snuffbox and travels laterally up the arm
 At the antecubital fossa connected to the basilic vein by the median cubital vein
 Pierces deep fascia of deltopectoral groove to join axillary vein

Next question
Question 479 of 560

An 18 year old man is undergoing an orchidectomy via a scrotal approach. The surgeons mobilise
the spermatic cord. From which of the following is the outermost layer of this structure derived?

Internal oblique aponeurosis

External oblique aponeurosis

Transversalis fascia

Rectus sheath

Campers fascia

The outermost covering of the spermatic cord is derived from the external oblique aponeurosis.This
layer is added as the cord passes through the superficial inguinal ring.
Please rate this question:

Discuss and give feedback


Next question

Scrotal and testicular anatomy

Spermatic cord
Formed by the vas deferens and is covered by the following structures:
Layer Origin

Internal spermatic fascia Transversalis fascia

Cremasteric fascia From the fascial coverings of internal oblique

External spermatic fascia External oblique aponeurosis

Contents of the cord


Vas deferens Transmits sperm and accessory gland secretions

Testicular artery Branch of abdominal aorta supplies testis and


epididymis

Artery of vas deferens Arises from inferior vesical artery

Cremasteric artery Arises from inferior epigastric artery

Pampiniform plexus Venous plexus, drains into right or left testicular vein

Sympathetic nerve fibres Lie on arteries, the parasympathetic fibres lie on the
vas

Genital branch of the genitofemoral Supplies cremaster


nerve

Lymphatic vessels Drain to lumbar and para-aortic nodes

Scrotum

 Composed of skin and closely attached dartos fascia.


 Arterial supply from the anterior and posterior scrotal arteries
 Lymphatic drainage to the inguinal lymph nodes
 Parietal layer of the tunica vaginalis is the innermost layer

Testes

 The testes are surrounded by the tunica vaginalis (closed peritoneal sac). The parietal layer
of the tunica vaginalis adjacent to the internal spermatic fascia.
 The testicular arteries arise from the aorta immediately inferiorly to the renal arteries.
 The pampiniform plexus drains into the testicular veins, the left drains into the left renal vein
and the right into the inferior vena cava.
 Lymphatic drainage is to the para-aortic nodes.

Next question
Question 480 of 560

Which of the following is not a carpal bone?

Trapezium

Triquetrum

Trapezoid

Trapezius

Lunate

Mnemonic for the Carpal Bones

Sally Likes To Play The Tiny Chrome Harmonica


She Looks Too Pretty Try To Catch Her
Scared Lovers Try Positions That They Can't Handle

Trapezius is a muscle of the back.

Please rate this question:

Discuss and give feedback


Next question

Carpal bones

The wrist is comprised of 8 carpal bones, these are arranged in two rows of 4. It is convex from side
to side posteriorly and concave anteriorly.
Diagrammatic image of carpal bones
Image sourced from Wikipedia

Key to image
A Scaphoid

B Lunate

C Triquetrum

D Pisiform

E Trapezium

F Trapezoid

G Capitate

H Hamate

1 Radius
2 Ulna

3 Metacarpals

 No tendons attach to: Scaphoid, lunate, triquetrum (stabilised by ligaments)

Next question
Question 481 of 560

A 53 year old male presents with a carcinoma of the transverse colon. Which of the following
structures should be ligated close to their origin to maximise clearance of the tumour?

Superior mesenteric artery

Inferior mesenteric artery

Middle colic artery

Ileo-colic artery

Superior rectal artery

The middle colic artery supplies the transverse colon and requires high ligation during cancer
resections. It is a branch of the superior mesenteric artery.
Please rate this question:

Discuss and give feedback


Next question

Transverse colon

 The right colon undergoes a sharp turn at the level of the hepatic flexure to become the
transverse colon.
 At this point it also becomes intraperitoneal.
 It is connected to the inferior border of the pancreas by the transverse mesocolon.
 The greater omentum is attached to the superior aspect of the transverse colon from which it
can easily be separated. The mesentery contains the middle colic artery and vein. The
greater omentum remains attached to the transverse colon up to the splenic flexure. At this
point the colon undergoes another sharp turn.

Relations
Liver and gall-bladder, the greater curvature of the stomach, and the lower end of the
Superior
spleen

Inferior Small intestine


Anterior Greater omentum

From right to left with the descending portion of the duodenum, the head of the pancreas,
Posterior
convolutions of the jejunum and ileum, spleen
Next question
Question 482 of 560

Which of the following structures are not closely related to the adductor longus muscle?

Long saphenous vein

Tendon of iliacus

The profunda branch of the femoral artery

Pectineus muscle

Femoral nerve

Femoral triangle:
Adductor longus medially
Inguinal ligament superiorly
Sartorius muscle laterally

Adductor longus forms the medial border of the femoral triangle. It is closely related to the long
saphenous vein which overlies it and the profunda branch of the femoral artery. The femoral nerve is
related to it inferiorly. However, the tendon of iliacus inserts proximally and is not in contact with
adductor longus.

Please rate this question:

Discuss and give feedback

Next question

Adductor longus
Origin Anterior body of pubis

Insertion Middle third of linea aspera

Action Adducts and flexes the thigh, medially rotate the hip

Innervation Anterior division of obturator nerve (L2, L3, L4)

The schematic image below demonstrates the relationship of the adductor muscles

Image sourced from Wikipedia

Next question
Question 483 of 560

Which of the following structures does not lie posterior to the right kidney?

Psoas major

12th rib

Quadratus lumborum

Medial arcuate ligament

10th rib

Theme from April 2012 Exam


The 8th and10th ribs lie more superiorly. The 12th rib is a closer relation posteriorly.
Quadratus lumborum runs between the posterior part of the iliac crest, iliolumbar ligament and the
transverse processes of the lower lumbar vertebrae to the medial part of the lower border of the last
rib and transverse process of the upper lumbar vertebrae. In these last two locations it is posterior to
the kidney.

Please rate this question:

Discuss and give feedback

Next question

Renal anatomy

Each kidney is about 11cm long, 5cm wide and 3cm thick. They are located in a deep gutter
alongside the projecting vertebral bodies, on the anterior surface of psoas major. In most cases the
left kidney lies approximately 1.5cm higher than the right. The upper pole of both kidneys
approximates with the 11th rib (beware pneumothorax during nephrectomy). On the left hand side
the hilum is located at the L1 vertebral level and the right kidney at level L1-2. The lower border of
the kidneys is usually alongside L3.

The table below shows the anatomical relations of the kidneys:

Relations

Relations Right Kidney Left Kidney

Posterior Quadratus lumborum, diaphragm, psoas Quadratus lumborum, diaphragm, psoas


major, transversus abdominis major, transversus abdominis

Anterior Hepatic flexure of colon Stomach, Pancreatic tail

Superior Liver, adrenal gland Spleen, adrenal gland

Fascial covering
Each kidney and suprarenal gland is enclosed within a common layer of investing fascia, derived
from the transversalis fascia. It is divided into anterior and posterior layers (Gerotas fascia).

Renal structure
Kidneys are surrounded by an outer cortex and an inner medulla which usually contains between 6
and 10 pyramidal structures. The papilla marks the innermost apex of these. They terminate at the
renal pelvis, into the ureter.
Lying in a hollow within the kidney is the renal sinus. This contains:
1. Branches of the renal artery
2. Tributaries of the renal vein
3. Major and minor calyces's
4. Fat

Structures at the renal hilum


The renal vein lies most anteriorly, then renal artery (it is an end artery) and the ureter lies most
posterior.

Next question
Question 484 of 560

A 56 year old man is undergoing a radical nephrectomy via a posterior approach. Which of the
following structures is most likely to be encountered during the operative approach?

8th rib

10th rib

6th rib

12th rib

9th rib

The 11th and 12th ribs lie posterior to the kidneys and may be encountered during a posterior
approach. A pneumothorax is a recognised complication of this type of surgery.

Please rate this question:

Discuss and give feedback

Next question

Renal anatomy

Each kidney is about 11cm long, 5cm wide and 3cm thick. They are located in a deep gutter
alongside the projecting vertebral bodies, on the anterior surface of psoas major. In most cases the
left kidney lies approximately 1.5cm higher than the right. The upper pole of both kidneys
approximates with the 11th rib (beware pneumothorax during nephrectomy). On the left hand side
the hilum is located at the L1 vertebral level and the right kidney at level L1-2. The lower border of
the kidneys is usually alongside L3.

The table below shows the anatomical relations of the kidneys:


Relations

Relations Right Kidney Left Kidney

Posterior Quadratus lumborum, diaphragm, psoas Quadratus lumborum, diaphragm, psoas


major, transversus abdominis major, transversus abdominis

Anterior Hepatic flexure of colon Stomach, Pancreatic tail

Superior Liver, adrenal gland Spleen, adrenal gland

Fascial covering
Each kidney and suprarenal gland is enclosed within a common layer of investing fascia, derived
from the transversalis fascia. It is divided into anterior and posterior layers (Gerotas fascia).

Renal structure
Kidneys are surrounded by an outer cortex and an inner medulla which usually contains between 6
and 10 pyramidal structures. The papilla marks the innermost apex of these. They terminate at the
renal pelvis, into the ureter.
Lying in a hollow within the kidney is the renal sinus. This contains:
1. Branches of the renal artery
2. Tributaries of the renal vein
3. Major and minor calyces's
4. Fat

Structures at the renal hilum


The renal vein lies most anteriorly, then renal artery (it is an end artery) and the ureter lies most
posterior.

Next question
Question 485 of 560

A 73 year old lady presents with a femoral hernia. Which of the following structures forms the lateral
wall of the femoral canal?

Pubic tubercle

Femoral vein

Femoral artery

Conjoint tendon

Femoral nerve

The canal exists to allow for the physiological expansion of the femoral vein, which lies lateral to it.
Please rate this question:

Discuss and give feedback


Next question

Femoral canal

The femoral canal lies at the medial aspect of the femoral sheath. The femoral sheath is a fascial
tunnel containing both the femoral artery laterally and femoral vein medially. The canal lies medial to
the vein.

Borders of the femoral canal


Laterally Femoral vein

Medially Lacunar ligament

Anteriorly Inguinal ligament

Posteriorly Pectineal ligament


Image showing dissection of femoral canal

Image sourced from Wikipedia

Contents

 Lymphatic vessels
 Cloquet's lymph node

Physiological significance
Allows the femoral vein to expand to allow for increased venous return to the lower limbs.

Pathological significance
As a potential space, it is the site of femoral hernias. The relatively tight neck places these at high
risk of strangulation.
Next question
Question 486 of 560

Which of these muscles is not a component of the rotator cuff?

Subscapularis

Teres minor

Supraspinatus

Infraspinatus

Deltoid

Deltoid may abduct the shoulder and is not a rotator cuff muscle.

Please rate this question:

Discuss and give feedback

Next question

Muscles of the rotator cuff

Muscle Innervation

Supraspinatus muscle Suprascapular nerve

Infraspinatus muscle Suprascapular nerve


Teres minor muscle Axillary nerve

Subscapularis muscle Superior and inferior subscapular nerves

Next question
Question 487 of 560

Which of the following muscles is not within the posterior compartment of the lower leg?

Peroneus brevis

Flexor digitorum longus

Soleus

Popliteus

Flexor hallucis longus

Theme from 2007 Exam

Peroneus brevis lies in the lateral compartment.


Please rate this question:

Discuss and give feedback


Next question

Fascial compartments of the leg

Compartments of the thigh

Formed by septae passing from the femur to the fascia lata.


Compartment Nerve Muscles Blood supply

Anterior compartment Femoral  Iliacus Femoral artery


 Tensor fasciae latae
 Sartorius
 Quadriceps femoris

Medial compartment Obturator  Adductor Profunda femoris artery


longus/magnus/brevis and obturator artery
 Gracilis
Compartment Nerve Muscles Blood supply

 Obturator externus

Posterior Sciatic  Semimembranosus Branches of Profunda


compartment (2  Semitendinosus femoris artery
layers)  Biceps femoris

Compartments of the lower leg


Separated by the interosseous membrane (anterior and posterior compartments), anterior fascial
septum (separate anterior and lateral compartments) and posterior fascial septum (separate lateral
and posterior compartments)

Blood
Compartment Nerve Muscles supply

Anterior Deep  Tibialis anterior Anterior


compartment peroneal  Extensor digitorum longus tibial artery
nerve  Extensor hallucis longus
 Peroneus tertius

Posterior Tibial  Muscles: deep and superficial Posterior


compartment compartments (separated by deep tibial
transverse fascia)
 Deep: Flexor hallucis longus, Flexor
digitalis longus, Tibialis posterior,
Popliteus
 Superficial: Gastrocnemius, Soleus,
Plantaris

Lateral Superficial  Peroneus longus/brevis Peroneal


compartment peroneal artery
Next question
Question 488 of 560

How many unpaired branches leave the abdominal aorta to supply the abdominal viscera?

One

Two

Three

Four

Five

There are three unpaired branches to the abdominal viscera. These include the coeliac axis, the
SMA and IMA. Branches to the adrenals, renal arteries and gonadal vessels are paired. The fourth
unpaired branch of the abdominal aorta, the median sacral artery, does not directly supply the
abdominal viscera.

Please rate this question:

Discuss and give feedback

Next question

Abdominal aortic branches

Branches Level Paired Type

Inferior phrenic T12 (Upper border) Yes Parietal


Branches Level Paired Type

Coeliac T12 No Visceral

Superior mesenteric L1 No Visceral

Middle suprarenal L1 Yes Visceral

Renal L1-L2 Yes Visceral

Gonadal L2 Yes Visceral

Lumbar L1-L4 Yes Parietal

Inferior mesenteric L3 No Visceral

Median sacral L4 No Parietal

Common iliac L4 Yes Terminal

Next question
Question 489 of 560

A 34 year old man with a submandibular gland stone is undergoing excision of the submandibular
gland. The incision is sited transversely approximately 4cm below the mandible. After incising the
skin, platysma and deep fascia which of the following structures is most likely to be encountered.

Facial artery

Facial vein

Lingual nerve

Hypoglossal nerve

Glossopharyngeal nerve

When approaching the submandibular gland the facial vein and submandibular lymph nodes are the
most superficially encountered structures. Each sub mandibular gland has a superficial and deep
part, separated by the mylohyoid muscle. The facial artery passes in a groove on the superficial
aspect of the gland. It then emerges onto the surface of the face by passing between the gland and
the mandible. The facial vein is encountered first in this surgical approach because the incision is
made 4cm below the mandible (to avoid injury to the marginal mandibular nerve).
Please rate this question:

Discuss and give feedback


Next question

Submandibular gland

Relations of the submandibular gland


Superficial Platysma, deep fascia and mandible
Submandibular lymph nodes
Facial vein (facial artery near mandible)
Marginal mandibular nerve
Cervical branch of the facial nerve

Deep Facial artery (inferior to the mandible)


Mylohoid muscle
Sub mandibular duct
Hyoglossus muscle
Lingual nerve
Submandibular ganglion
Hypoglossal nerve

Submandibular duct (Wharton's duct)

 Opens lateral to the lingual frenulum on the anterior floor of mouth.


 5 cm length
 Lingual nerve wraps around Wharton's duct. As the duct passes forwards it crosses medial
to the nerve to lie above it and then crosses back, lateral to it, to reach a position below the
nerve.

Innervation

 Sympathetic innervation- Derived from superior cervical ganglion


 Parasympathetic innervation- Submandibular ganglion via lingual nerve

Arterial supply
Branch of the facial artery. The facial artery passes through the gland to groove its deep surface. It
then emerges onto the face by passing between the gland and the mandible.

Venous drainage
Anterior facial vein (lies deep to the Marginal Mandibular nerve)

Lymphatic drainage
Deep cervical and jugular chains of nodes
Next question
Question 490 of 560

You are working as an anatomy demonstrator and the medical students decide to test your
knowledge on the Circle of Willis. Which of the following comments is false?

Does not include the middle cerebral artery

Asymmetry of the circle of willis is a risk factor for the development of intracranial
aneurysms

Majority of blood passing through the vessels mix together

Includes the anterior communicating arteries

The circle surrounds the stalk of the pituitary gland

There is minimum mixing of blood passing through the vessels.


Please rate this question:

Discuss and give feedback


Next question

Circle of Willis

The two internal carotid arteries and two vertebral arteries form an anastomosis known as the Circle
of Willis on the inferior surface of the brain. Each half of the circle is formed by:
1. Anterior communicating artery
2. Anterior cerebral artery
3. Internal carotid artery
4. Posterior communicating artery
5. Posterior cerebral arteries and the termination of the basilar artery

The circle and its branches supply; the corpus striatum, internal capsule, diencephalon and
midbrain.
Image sourced from Wikipedia

Vertebral arteries

 Enter the cranial cavity via foramen magnum


 Lie in the subarachnoid space
 Ascend on anterior surface of medulla oblongata
 Unite to form the basilar artery at the base of the pons

Branches:

 Posterior spinal artery


 Anterior spinal artery
 Posterior inferior cerebellar artery

Basilar artery
Branches:
 Anterior inferior cerebellar artery
 Labyrinthine artery
 Pontine arteries
 Superior cerebellar artery
 Posterior cerebral artery

Internal carotid arteries


Branches:

 Posterior communicating artery


 Anterior cerebral artery
 Middle cerebral artery
 Anterior choroid artery

Next question

Question 491-493 of 560

Theme: Surgical nerve lesions

A. Recurrent laryngeal nerve


B. Accessory nerve
C. Hypoglossal nerve
D. Vagus nerve
E. Common peroneal nerve
F. Tibial nerve
G. Long saphenous nerve
H. Phrenic nerve
I. Thoracodorsal nerve

Please choose which of the listed nerves is at greatest risk for the procedures given. Each option
may be used once, more than once or not at all.

491. A 64 year old man has a suspected lymphoma and lymph node biopsy from the
posterolateral aspect of the right neck is planned.

You answered Recurrent laryngeal nerve

The correct answer is Accessory nerve

The accessory nerve has a superficial course and is easily injured. It lies under platysma
and may be divided during the early part of the procedure.
492. A 43 year old woman is due to undergo an axillary node clearance following a positive
sentinel node biopsy.

You answered Recurrent laryngeal nerve

The correct answer is Thoracodorsal nerve

The long thoracic nerve is also at risk. The thoracodorsal nerve traverses the level 2
axillary nodes to supply latissimus dorsi and may be divided or damaged with diathermy.

493. A 53 year old man is to undergo a thyroidectomy.

Recurrent laryngeal nerve

Recurrent laryngeal nerve injury may complicate thyroid surgery in up to 2% of cases.

Please rate this question:

Discuss and give feedback


Next question

Nerve lesions during surgery

A variety of different procedures carry the risk of iatrogenic nerve injury. These are important not
only from the patients perspective but also from a medicolegal standpoint.

The following operations and their associated nerve lesions are listed here:

 Posterior triangle lymph node biopsy and accessory nerve lesion.


 Lloyd Davies stirrups and common peroneal nerve.
 Thyroidectomy and laryngeal nerve.
 Anterior resection of rectum and hypogastric autonomic nerves.
 Axillary node clearance; long thoracic nerve, thoracodorsal nerve and intercostobrachial
nerve.
 Inguinal hernia surgery and ilioinguinal nerve.
 Varicose vein surgery- sural and saphenous nerves.
 Posterior approach to the hip and sciatic nerve.
 Carotid endarterectomy and hypoglossal nerve.

There are many more, with sound anatomical understanding of the commonly performed procedures
the incidence of nerve lesions can be minimised. They commonly occur when surgeons operate in
an unfamiliar tissue plane or by blind placement of haemostats (not recommended).
Next question
Question 494 of 560

A 45 year old man presents with bilateral inguinal hernias. The surgical team decide to repair these
laparoscopically through an extraperitoneal approach. Through an infraumbilical incision the
surgeons displace the inferior aspect of the rectus abdominis muscle anteriorly and place a
prosthetic mesh into the area to repair the hernias. Which structure will lie posterior to the mesh?

Peritoneum

Internal oblique aponeurosis

External oblique aponeurosis

Posterior aspect of the rectus sheath

Bucks fascia

During a TEP repair of inguinal hernia the only structure to lie posterior to the mesh is peritoneum.
The question is really only asking which structure lies posterior to the rectus abdominis muscle.
Since this region is below the arcuate line the transversalis fascia and peritoneum lie posterior to it.
Bucks fascia lies in the penis.

Please rate this question:

Discuss and give feedback

Next question

Rectus abdominis muscle

The rectus sheath is formed by the aponeuroses of the lateral abdominal wall muscles. The rectus
sheath has a composition that varies according to anatomical level.

1. Above the costal margin the anterior sheath is composed of external oblique aponeurosis, the
costal cartilages are posterior to it.
2. From the costal margin to the arcuate line, the anterior rectus sheath is composed of external
oblique aponeurosis and the anterior part of the internal oblique aponeurosis. The posterior part of
the internal oblique aponeurosis and transversus abdominis form the posterior rectus sheath.
3. Below the arcuate line the aponeuroses of all the abdominal muscles lie in anterior aspect of the
rectus sheath. Posteriorly lies the transversalis fascia and peritoneum.

The arcuate line is the point at which the inferior epigastric vessels enter the rectus sheath.

Next question
Question 495 of 560

A 20 year old man undergoes an open appendicectomy performed via a lanz incision. This surgeon
places the incision on a level of the anterior superior iliac spine in an attempt to improve cosmesis.
During the procedure the appendix is found to be retrocaecal and the incision is extended laterally.
Which of the following nerves is at greatest risk of injury?

Genitofemoral

Ilioinguinal

Obturator

Lateral femoral cutaneous

Femoral

Theme from April 2012 Exam


Please rate this question:

Discuss and give feedback


Next question

Ilioinguinal nerve

Arises from the first lumbar ventral ramus with the iliohypogastric nerve. It passes inferolaterally
through the substance of psoas major and over the anterior surface of quadratus lumborum. It
pierces the internal oblique muscle and passes deep to the aponeurosis of the external oblique
muscle. It enters the inguinal canal and then passes through the superficial inguinal ring to reach the
skin.

Branches

 To supply those muscles of the abdominal wall through which it passes.


 Skin and fascia over the pubic symphysis, superomedial part of the femoral triangle, surface
of the scrotum, root and dorsum of penis or labum majus in females.

Next question
Question 496 of 560

The femoral nerve is transected by a rather careless surgeon during a botched femoro-popliteal
bypass operation. Which of the following actions will be impaired?

Extension of the great toe

Adduction of the thigh

Flexion of the knee joint

Extension of the knee joint

Eversion of the foot

The femoral nerve supplies the quadriceps muscle which is responsible for extension at the knee
joint.
Please rate this question:

Discuss and give feedback


Next question

Femoral nerve

Root values L2, 3, 4

Innervates  Pectineus
 Sartorius
 Quadriceps femoris
 Vastus lateralis/medialis/intermedius

Branches  Medial cutaneous nerve of thigh


 Saphenous nerve
 Intermediate cutaneous nerve of thigh
Path
Penetrates psoas major and exits the pelvis by passing under the inguinal ligament to enter the
femoral triangle, lateral to the femoral artery and vein.

Image sourced from Wikipedia

Mnemonic for femoral nerve supply

(don't) M I S V Q Scan for PE


M edial cutaneous nerve of the thigh
I ntermediate cutaneous nerve of the thigh
S aphenous nerve

V astus
Q uadriceps femoris
S artorius

PE ectineus
Question 497 of 560

Which of the following is not a direct branch of the facial nerve?

Greater petrosal nerve

Nerve to stapedius

Auriculotemporal

Chorda tympani

Buccal

The auriculotemporal nerve is a direct branch of the mandibular nerve.


Other branches of the mandibular nerve include:
Lingual
Inferior alveolar
Nerve to the mylohyoid
Mental
Please rate this question:

Discuss and give feedback


Next question

Facial nerve

The facial nerve is the main nerve supplying the structures of the second embryonic branchial arch.
It is predominantly an efferent nerve to the muscles of facial expression, digastric muscle and also to
many glandular structures. It contains a few afferent fibres which originate in the cells of its genicular
ganglion and are concerned with taste.

Supply - 'face, ear, taste, tear'

 Face: muscles of facial expression


 Ear: nerve to stapedius
 Taste: supplies anterior two-thirds of tongue
 Tear: parasympathetic fibres to lacrimal glands, also salivary glands
Path
Subarachnoid path

 Origin: motor- pons, sensory- nervus intermedius


 Pass through the petrous temporal bone into the internal auditory meatus with the
vestibulocochlear nerve. Here they combine to become the facial nerve.

Facial canal path

 The canal passes superior to the vestibule of the inner ear


 At the medial aspect of the middle ear, it becomes wider and contains the geniculate
ganglion.

- 3 branches:
1. greater petrosal nerve
2. nerve to stapedius
3. chorda tympani

Stylomastoid foramen

 Passes through the stylomastoid foramen (tympanic cavity anterior and mastoid antrum
posteriorly)
 Posterior auricular nerve and branch to posterior belly of digastric and stylohyoid muscle

Face
Enters parotid gland and divides into 5 branches:

 Temporal branch
 Zygomatic branch
 Buccal branch
 Marginal mandibular branch
 Cervical branch

Next question
Question 498 of 560

A 32 year old man is stabbed in the neck and the inferior trunk of his brachial plexus is injured.
Which of the modalities listed below is least likely to be affected?

Initiating abduction of the shoulder

Abduction of the fingers

Flexion of the little finger

Sensation on the palmar aspect of the little finger

Gripping a screwdriver

Inferior trunk of brachial plexus.

 C8 and T1 roots
 Contributes to ulnar nerve and part of median nerve

Theme from September 2012 Exam


The inferior trunk of the brachial plexus is rarely injured. Nerve roots C8 and T1 are the main
contributors to this trunk. Therefore an injury to this site will most consistently affect the ulnar nerve.
The inferior trunk also contributes to the median nerve by way of the medial cord and therefore some
impairment of grip is almost inevitable.
Please rate this question:

Discuss and give feedback


Next question

Brachial plexus

Origin Anterior rami of C5 to T1

Sections of the  Roots, trunks, divisions, cords, branches


plexus  Mnemonic:Real Teenagers Drink Cold Beer
Roots  Located in the posterior triangle
 Pass between scalenus anterior and medius

Trunks  Located posterior to middle third of clavicle


 Upper and middle trunks related superiorly to the subclavian artery
 Lower trunk passes over 1st rib posterior to the subclavian artery

Divisions Apex of axilla

Cords Related to axillary artery

Diagram illustrating the branches of the brachial plexus

Image sourced from Wikipedia

Cutaneous sensation of the upper limb


Image sourced from Wikipedia

Next question
Question 499 of 560

During embryological development, which of the following represent the correct origin of the
pancreas?

Ventral and dorsal endodermal outgrowths of the duodenum

Ventral and dorsal outgrowths of mesenchymal tissue from the posterior abdominal wall

Ventral and dorsal outgrowths of the vitellointestinal duct

Ventral and dorsal biliary tract diverticulae

Buds from the inferior aspect of the caudate lobe

The pancreas develops from a ventral and dorsal endodermal outgrowth of the duodenum. The
ventral arises close to, or in common with the hepatic diverticulum, and the larger, dorsal outgrowth
arises slightly cranial to the ventral extending into the mesoduodenum and mesogastrium. When the
buds eventually fuse the duct of the ventral rudiment becomes the main pancreatic duct.
Please rate this question:

Discuss and give feedback


Next question

Pancreas

The pancreas is a retroperitoneal organ and lies posterior to the stomach. It may be accessed
surgically by dividing the peritoneal reflection that connects the greater omentum to the transverse
colon. The pancreatic head sits in the curvature of the duodenum. Its tail lies close to the hilum of
the spleen, a site of potential injury during splenectomy.

Relations
Posterior to the pancreas
Pancreatic head Inferior vena cava
Common bile duct
Right and left renal veins
Superior mesenteric vein and artery
Pancreatic neck Superior mesenteric vein, portal vein

Pancreatic body- Left renal vein


Crus of diaphragm
Psoas muscle
Adrenal gland
Kidney
Aorta

Pancreatic tail Left kidney

Anterior to the pancreas


Pancreatic head 1st part of the duodenum
Pylorus
Gastroduodenal artery
SMA and SMV(uncinate process)

Pancreatic body Stomach


Duodenojejunal flexure

Pancreatic tail Splenic hilum

Superior to the pancreas


Coeliac trunk and its branches common hepatic artery and splenic artery

Grooves of the head of the pancreas


2nd and 3rd part of the duodenum

Arterial supply

 Head: pancreaticoduodenal artery


 Rest: splenic artery

Venous drainage

 Head: superior mesenteric vein


 Body and tail: splenic vein
Ampulla of Vater

 Merge of pancreatic duct and common bile duct


 Is an important landmark, halfway along the second part of the duodenum, that marks the
anatomical transition from foregut to midgut (also the site of transition between regions
supplied by coeliac trunk and SMA).

Image sourced from Wikipedia

Next question
Question 500 of 560

Which of the following is not a content of the anterior triangle of the neck?

Ansa cervicalis

Vagus nerve

Anterior jugular vein

Transverse cervical nerve

Hypoglossal nerve

The transverse cervical nerve lies within the posterior triangle. The anterior jugular vein is formed in
the submental region and descends in the superficial fascia near the median plane. It passes inferior
to enter the suprasternal space, it is linked to the contralateral anterior jugular vein by the jugular
venous arch.
Please rate this question:

Discuss and give feedback


Next question

Anterior triangle of the neck

Boundaries

Anterior border of the Sternocleidomastoid


Lower border of mandible
Anterior midline

Sub triangles (divided by Digastric above and Omohyoid)

 Muscular triangle: Neck strap muscles


 Carotid triangle: Carotid sheath
 Submandibular Triangle (digastric)
Contents of the anterior triangle
Digastric triangle Submandibular gland
Submandibular nodes
Facial vessels
Hypoglossal nerve

Muscular triangle Strap muscles


External jugular vein

Carotid triangle Carotid sheath (Common carotid, vagus and internal jugular vein)
Ansa cervicalis

Nerve supply to digastric muscle

 Anterior: Mylohyoid nerve


 Posterior: Facial nerve

Image sourced from Wikipedia

Next question
Question 501 of 560

A 23 year old man presents with delayed diagnosis of appendicitis. The appendix is retrocaecal and
has perforated causing a psoas abscess. Into which structure does the psoas major muscle insert?

Greater trochanter of the femur

Linea aspera of the femur

Lesser trochanter of the femur

Iliac crest

None of the above

Theme based on 2011 exam


The psoas major inserts into the lesser trochanter.
Please rate this question:

Discuss and give feedback


Next question

Psoas Muscle

Origin
The deep part originates from the transverse processes of the five lumbar vertebrae, the superficial
part originates from T12 and the first 4 lumbar vertebrae.

Insertion
Lesser trochanter of the femur.

Innervation
Anterior rami of L1 to L3.

Action
Flexion and external rotation of the hip. Bilateral contraction can raise the trunk from the supine
position.
Next question
Question 502 of 560

A 63 year old man is due to undergo a splenectomy. Which splenic structure lies most posteriorly?

Gastrosplenic ligament

Splenic vein

Splenic artery

Splenic notch

Lienorenal ligament

Theme from 2011 Exam


The lienorenal ligament lies most posteriorly. The antero-lateral connection is via the phrenicocolic
ligament. Anteriorly the gastro splenic ligament. These structures condense around the vessels at
the splenic hilum.

Please rate this question:

Discuss and give feedback

Next question

Splenic anatomy

The spleen is the largest lymphoid organ in the body. It is an intraperitoneal organ, the peritoneal
attachments condense at the hilum where the vessels enter the spleen. Its blood supply is from the
splenic artery (derived from the coeliac axis) and the splenic vein (which is joined by the IMV and
unites with the SMV).
 Embryology: derived from mesenchymal tissue
 Shape: clenched fist
 Position: below 9th-12th ribs
 Weight: 75-150g

Relations

 Superiorly- diaphragm
 Anteriorly- gastric impression
 Posteriorly- kidney
 Inferiorly- colon
 Hilum: tail of pancreas and splenic vessels
 Forms apex of lesser sac (containing short gastric vessels)

Next question
Question 503 of 560

Which of these statements relating to the external carotid is false?

It ends by bifurcating into the superficial temporal and ascending pharyngeal artery

Its first branch is the superior thyroid artery

The superior thyroid, lingual and facial arteries all arise from its anterior surface

The ascending pharyngeal artery is a medial branch

Initially it lies anteromedial to the internal carotid

It terminates by dividing into the superficial temporal and maxillary branches. The external carotid
has eight branches, 3 from its anterior surface ; thyroid, lingual and facial. The pharyngeal artery is a
medial branch. The posterior auricular and occipital are posterior branches.

Please rate this question:

Discuss and give feedback

Next question

External carotid artery

The external carotid commences immediately lateral to the pharyngeal side wall. It ascends and lies
anterior to the internal carotid and posterior to the posterior belly of digastric and stylohyoid. More
inferiorly it is covered by sternocleidomastoid, passed by hypoglossal nerves, lingual and facial
veins.
It then pierces the fascia of the parotid gland finally dividing into its terminal branches within the
gland itself.

Surface marking of the carotid


This is an imaginary line drawn from the bifurcation of the common carotid passing behind the angle
of the jaw to a point immediately anterior to the tragus of the ear.

Branches of the external carotid artery


It has six branches, three in front, two behind and one deep.

Three in front Superior thyroid


Lingual
Facial

Two behind Occipital


Posterior auricular

Deep Ascending pharyngeal

It terminates by dividing into the superficial temporal and maxillary arteries in the parotid gland.

Image sourced from Wikipedia

Next question
Question 504 of 560

Which of the following statements about the spleen is false?

The hilum contains the splenic vessels.

The spleen is derived from endodermal tissue.

The white pulp has immune function.

The colon lies inferiorly.

Weighs 150g.

1,3,5,7,9,11 (odd numbers up to 11)

The spleen is: 1 inch thick, 3 inches wide, 5 inches long, weighs 7oz (150-200g), lies between the
9th and 11th ribs

Most of the gut is derived endodermally except for the spleen which is from mesenchymal tissue.
Please rate this question:

Discuss and give feedback


Next question

Spleen

The spleen is located in the left upper quadrant of the abdomen and its size can vary depending
upon the amount of blood it contains. The typical adult spleen is 12.5cm long and 7.5cm wide. The
usual weight of the adult spleen is 150g.
The exact position of the spleen can vary with respiratory activity, posture and the state of
surrounding viscera. It usually lies obliquely with its long axis aligned to the 9th, 10th and 11th ribs. It
is separated from these ribs by both diaphragm and pleural cavity. The normal spleen is not
palpable.

The shape of the spleen is influenced by the state of the colon and stomach. Gastric distension will
cause the spleen to resemble the shape of an orange segment. Colonic distension will cause it to
become more tetrahedral.

The spleen is almost entirely covered by peritoneum, which adheres firmly to its capsule. Recesses
of the greater sac separate it from the stomach and kidney. It develops from the upper dorsal
mesogastrium, remaining connected to the posterior abdominal wall and stomach by two folds of
peritoneum; the lienorenal ligament and gastrosplenic ligament. The lienorenal ligament is derived
from peritoneum where the wall of the general peritoneum meets the omental bursa between the left
kidney and spleen; the splenic vessels lie in its layers. The gastrosplenic ligament also has two
layers, formed by the meeting of the walls of the greater sac and omental bursa between spleen and
stomach, the short gastric and left gastroepiploic branches of the splenic artery pass in its layers.
Laterally, the spleen is in contact with the phrenicocolic ligament.

Relations
Superiorly Diaphragm

Anteriorly Gastric impression

Posteriorly Kidney

Inferiorly Colon

Tail of pancreas and splenic vessels (splenic artery divides here, branches pass to the
Hilum
white pulp transporting plasma)

Contents
White Immune function. Contains central trabecular artery. The germinal centres are supplied
pulp by arterioles called penicilliary radicles.

Red pulp Filters abnormal red blood cells.

Function

 Filtration of abnormal blood cells and foreign bodies such as bacteria.


 Immunity: IgM. Production of properdin, and tuftsin which help target fungi and bacteria for
phagocytosis.
 Haematopoiesis: up to 5th month gestation or in haematological disorders.
 Pooling: storage of 40% platelets.
 Iron reutilisation
 Storage monocytes

Disorders of the spleen


Massive splenomegaly

 Myelofibrosis
 Chronic myeloid leukaemia
 Visceral leishmaniasis (kala-azar)
 Malaria
 Gaucher's syndrome
Other causes (as above plus)

 Portal hypertension e.g. secondary to cirrhosis


 Lymphoproliferative disease e.g. CLL, Hodgkin's
 Haemolytic anaemia
 Infection: hepatitis, glandular fever
 Infective endocarditis
 Sickle-cell*, thalassaemia
 Rheumatoid arthritis (Felty's syndrome)

*the majority of adult patients with sickle-cell will have an atrophied spleen due to repeated infarction
Next question
Question 505 of 560

As it exits the axilla the radial nerve lies on which of the following muscles?

Supraspinatus

Infraspinatus

Teres major

Deltoid

Pectoralis major

The radial nerve passes through the triangular space to leave the axilla. The superior border of this
is bounded by the teres major muscle to which the radial nerve is closely related.
Please rate this question:

Discuss and give feedback


Next question

Radial nerve

Continuation of posterior cord of the brachial plexus (root values C5 to T1)

Path

 In the axilla: lies posterior to the axillary artery on subscapularis, latissimus dorsi and teres
major.
 Enters the arm between the brachial artery and the long head of triceps (medial to humerus).
 Spirals around the posterior surface of the humerus in the groove for the radial nerve.
 At the distal third of the lateral border of the humerus it then pierces the intermuscular
septum and descends in front of the lateral epicondyle.
 At the lateral epicondyle it lies deeply between brachialis and brachioradialis where it then
divides into a superficial and deep terminal branch.
 Deep branch crosses the supinator to become the posterior interosseous nerve.

In the image below the relationships of the radial nerve can be appreciated
Image sourced from Wikipedia

Regions innervated
 Triceps
 Anconeus
Motor (main nerve)  Brachioradialis
 Extensor carpi radialis

 Supinator
 Extensor carpi ulnaris
 Extensor digitorum
Motor (posterior  Extensor indicis
interosseous branch)  Extensor digiti minimi
 Extensor pollicis longus and brevis
 Abductor pollicis longus

The area of skin supplying the proximal phalanges on the dorsal aspect of the
Sensory hand is supplied by the radial nerve (this does not apply to the little finger and
part of the ring finger)

Muscular innervation and effect of denervation


Anatomical
location Muscle affected Effect of paralysis
Anatomical
location Muscle affected Effect of paralysis

Shoulder Long head of triceps Minor effects on shoulder stability in abduction

Arm Triceps Loss of elbow extension

Forearm Supinator Weakening of supination of prone hand and


Brachioradialis elbow flexion in mid prone position
Extensor carpi radialis
longus and brevis

The cutaneous sensation of the upper limb- illustrating the contribution of the radial nerve

Image sourced from Wikipedia

Next question
Question 506 of 560

Into which of the following veins does the middle thyroid vein drain?

Vertebral

External jugular

Internal jugular

Subclavian

Anterior jugular

It drains to the internal jugular vein. Which is one of the reasons why it bleeds so copiously if a
ligature slips.
Please rate this question:

Discuss and give feedback


Next question

Thyroid gland

 Right and left lobes connected by isthmus


 Surrounded by sheath from pretracheal layer of deep fascia
 Apex: Lamina of thyroid cartilage
 Base: 4th-5th tracheal ring
 Pyramidal lobe: from isthmus
 May be attached to foramen caecum at the base of the tongue

Relations
Anteromedially  Sternothyroid
 Superior belly of omohyoid
 Sternohyoid
 Anterior aspect of sternocleidomastoid

Posterolaterally Carotid sheath


Medially  Larynx
 Trachea
 Pharynx
 Oesophagus
 Cricothyroid muscle
 External laryngeal nerve (near superior thyroid artery)
 Recurrent laryngeal nerve (near inferior thyroid artery)

Posterior  Parathyroid glands


 Anastomosis of superior and inferior thyroid arteries

Isthmus  Anteriorly: Sternothyroids, sternohyoids, anterior jugular veins


 Posteriorly: 2nd, 3rd, 4th tracheal rings (attached via Ligament of
Berry)

Blood Supply
Arterial  Superior thyroid artery (1st branch of external carotid)
 Inferior thyroid artery (from thyrocervical trunk)
 Thyroidea ima (in 10% of population -from brachiocephalic artery or aorta)

Venous  Superior and middle thyroid veins - into the IJV


 Inferior thyroid vein - into the brachiocephalic veins

Next question
Question 507 of 560

Which of the following structures is not at the level of the transpyloric plane?

Hilum left kidney

Superior mesenteric artery

Fundus of the gallbladder

Cardioesophageal junction

Root of transverse mesocolon

Cardiooesophageal junction level = T11

A knowledge of this anatomic level is commonly tested.


The oesophagus extends from C6 (the lower border of the cricoid cartilage) to T11 at the
cardioesophageal junction. Note that in the neonate the oesophagus extends from C4 or C5 to T9.
Please rate this question:

Discuss and give feedback


Next question

Levels

Transpyloric plane
Level of the body of L1

 Pylorus stomach
 Left kidney hilum (L1- left one!)
 Fundus of the gallbladder
 Neck of pancreas
 Duodenojejunal flexure
 Superior mesenteric artery
 Portal vein
 Left and right colic flexure
 Root of the transverse mesocolon
 2nd part of the duodenum
 Upper part of conus medullaris
 Spleen

Can be identified by asking the supine patient to sit up without using their arms. The plane is located
where the lateral border of the rectus muscle crosses the costal margin.

Anatomical planes
Subcostal plane Lowest margin of 10th costal cartilage

Intercristal plane Level of body L4 (highest point of iliac crest)

Intertubercular plane Level of body L5

Common level landmarks


Inferior mesenteric artery L3

Bifurcation of aorta into common iliac arteries L4

Formation of IVC L5 (union of common iliac veins)

Diaphragm apertures  Vena cava T8


 Oesophagus T10
 Aortic hiatus T12

Next question
Question 508 of 560

A 62 year old man presents with arm weakness. On examination he has a weakness of elbow
extension and loss of sensation on the dorsal aspect of the first digit. What is the site of the most
likely underlying defect?

Axillary nerve

Median nerve

Ulnar nerve

Radial nerve

Musculocutaneous nerve

Theme from April 2012 Exam


The long head of the triceps muscle may be innervated by the axillary nerve and therefore complete
loss of triceps muscles function may not be present even with proximally sited nerve lesions.
Please rate this question:

Discuss and give feedback


Next question

Radial nerve

Continuation of posterior cord of the brachial plexus (root values C5 to T1)

Path

 In the axilla: lies posterior to the axillary artery on subscapularis, latissimus dorsi and teres
major.
 Enters the arm between the brachial artery and the long head of triceps (medial to humerus).
 Spirals around the posterior surface of the humerus in the groove for the radial nerve.
 At the distal third of the lateral border of the humerus it then pierces the intermuscular
septum and descends in front of the lateral epicondyle.
 At the lateral epicondyle it lies deeply between brachialis and brachioradialis where it then
divides into a superficial and deep terminal branch.
 Deep branch crosses the supinator to become the posterior interosseous nerve.
In the image below the relationships of the radial nerve can be appreciated

Image sourced from Wikipedia

Regions innervated
 Triceps
 Anconeus
Motor (main nerve)  Brachioradialis
 Extensor carpi radialis

 Supinator
 Extensor carpi ulnaris
 Extensor digitorum
Motor (posterior  Extensor indicis
interosseous branch)  Extensor digiti minimi
 Extensor pollicis longus and brevis
 Abductor pollicis longus

The area of skin supplying the proximal phalanges on the dorsal aspect of the
Sensory hand is supplied by the radial nerve (this does not apply to the little finger and
part of the ring finger)

Muscular innervation and effect of denervation


Anatomical
location Muscle affected Effect of paralysis

Shoulder Long head of triceps Minor effects on shoulder stability in abduction

Arm Triceps Loss of elbow extension

Forearm Supinator Weakening of supination of prone hand and


Brachioradialis elbow flexion in mid prone position
Extensor carpi radialis
longus and brevis

The cutaneous sensation of the upper limb- illustrating the contribution of the radial nerve

Image sourced from Wikipedia

Next question
Question 509 of 560

From which of the following structures does the long head of the triceps muscle arise?

Coracoid process

Acromion

Infraglenoid tubercle

Coraco-acromial ligament

Coraco-humeral ligament

The long head arises from the infraglenoid tubercle. The fleshy lateral and medial heads are
attached to the posterior aspect of the humerus between the insertion of the teres minor and the
olecranon fossa.
Please rate this question:

Discuss and give feedback


Next question

Triceps

Origin  Long head- infraglenoid tubercle of the scapula.


 Lateral head- dorsal surface of the humerus, lateral and proximal to the
groove of the radial nerve
 Medial head- posterior surface of the humerus on the inferomedial side of
the radial groove and both of the intermuscular septae

Insertion  Olecranon process of the ulna. Here the olecranon bursa is between the
triceps tendon and olecranon.
 Some fibres insert to the deep fascia of the forearm, posterior capsule of the
elbow (preventing the capsule from being trapped between olecranon and
olecranon fossa during extension)
Innervation Radial nerve

Blood Profunda brachii artery


supply

Action Elbow extension. The long head can adduct the humerus and and extend it from a
flexed position

Relations The radial nerve and profunda brachii vessels lie between the lateral and medial
heads
Next question
Question 510 of 560

A 45 year old man is undergoing a left hemicolectomy. As the surgeons mobilise the left colon they
identify a tubular structure lying at the inferior aspect of psoas major. What is it most likely to be?

Left ureter

Left common iliac vein

Left common iliac artery

Left external iliac artery

Left external iliac vein

The left ureter lies posterior to the left colon. The sigmoid colon and upper rectum may be more
closely related to the iliac vessels. These are not typically found above L4.
Please rate this question:

Discuss and give feedback


Next question

Ureter

 25-35 cm long
 Muscular tube lined by transitional epithelium
 Surrounded by thick muscular coat. Becomes 3 muscular layers as it crosses the bony pelvis
 Retroperitoneal structure overlying transverse processes L2-L5
 Lies anterior to bifurcation of iliac vessels
 Blood supply is segmental; renal artery, aortic branches, gonadal branches, common iliac
and internal iliac
 Lies beneath the uterine artery

Next question
Question 511 of 560

Which muscle is not innervated by the trigeminal nerve?

Medial pterygoid

Mylohyoid

Stylohyoid

Masseter

Temporalis

Stylohyoid is innervated by the facial nerve.


Please rate this question:

Discuss and give feedback


Next question

Trigeminal nerve

The trigeminal nerve is the main sensory nerve of the head. In addition to its major sensory role, it
also innervates the muscles of mastication.

Distribution of the trigeminal nerve


Sensory  Scalp
 Face
 Oral cavity (and teeth)
 Nose and sinuses
 Dura mater

Motor  Muscles of mastication


 Mylohyoid
 Anterior belly of digastric
 Tensor tympani
 Tensor palati

Autonomic connections (ganglia)  Ciliary


 Sphenopalatine
 Otic
 Submandibular

Path

 Originates at the pons


 Sensory root forms the large, crescentic trigeminal ganglion within Meckel's cave, and
contains the cell bodies of incoming sensory nerve fibres. Here the 3 branches exit.
 The motor root cell bodies are in the pons and the motor fibres are distributed via the
mandibular nerve. The motor root is not part of the trigeminal ganglion.

Branches of the trigeminal nerve


Ophthalmic nerve Sensory only

Maxillary nerve Sensory only

Mandibular nerve Sensory and motor

Sensory
Ophthalmic Exits skull via the superior orbital fissure
Sensation of: scalp and forehead, the upper eyelid, the conjunctiva and cornea of
the eye, the nose (including the tip of the nose, except alae nasi), the nasal
mucosa, the frontal sinuses, and parts of the meninges (the dura and blood
vessels).

Maxillary Exit skull via the foramen rotundum


nerve Sensation: lower eyelid and cheek, the nares and upper lip, the upper teeth and
gums, the nasal mucosa, the palate and roof of the pharynx, the maxillary,
ethmoid and sphenoid sinuses, and parts of the meninges.

Mandibular Exit skull via the foramen ovale


nerve Sensation: lower lip, the lower teeth and gums, the chin and jaw (except the angle
of the jaw), parts of the external ear, and parts of the meninges.

Motor
Distributed via the mandibular nerve.
The following muscles of mastication are innervated:

 Masseter
 Temporalis
 Medial pterygoid
 Lateral pterygoid

Other muscles innervated include:

 Tensor veli palatini


 Mylohyoid
 Anterior belly of digastric
 Tensor tympani

Next question
Question 512 of 560

A 42 year old woman is due to undergo a left nephroureterectomy for a transitional cell carcinoma
involving the ureter. Which of the following structures is not related to the left ureter?

Round ligament of the uterus

Internal iliac artery

Ovarian artery

Peritoneum

Sigmoid mesocolon

The ureter is not related to the round ligament of the uterus, it is related to the broad ligament and is
within 1.5cm of the supravaginal part of the cervix.
Please rate this question:

Discuss and give feedback


Next question

Ureter

 25-35 cm long
 Muscular tube lined by transitional epithelium
 Surrounded by thick muscular coat. Becomes 3 muscular layers as it crosses the bony pelvis
 Retroperitoneal structure overlying transverse processes L2-L5
 Lies anterior to bifurcation of iliac vessels
 Blood supply is segmental; renal artery, aortic branches, gonadal branches, common iliac
and internal iliac
 Lies beneath the uterine artery

Next question
Question 513 of 560

Which of the following most commonly arises from the brachiocephalic artery?

Vertebral artery

Subscapular artery

Thyroidea ima artery

Left Subclavian artery

None of the above

Other occasional branches include the thymic and bronchial branch.


Please rate this question:

Discuss and give feedback


Next question

Brachiocephalic artery

The brachiocephalic artery is the largest branch of the aortic arch. From its aortic origin it ascends
superiorly, it initially lies anterior to the trachea and then on its right hand side. It branches into the
common carotid and right subclavian arteries at the level of the sternoclavicular joint.

Path
Origin- apex of the midline of the aortic arch
Passes superiorly and posteriorly to the right
Divides into the right subclavian and right common carotid artery

Relations
Anterior  Sternohyoid
 Sternothyroid
 Thymic remnants
 Left brachiocephalic vein
 Right inferior thyroid veins
Posterior  Trachea
 Right pleura

Right lateral  Right brachiocephalic vein


 Superior part of SVC

Left lateral  Thymic remnants


 Origin of left common carotid
 Inferior thyroid veins
 Trachea (higher level)

Branches
Normally none but may have the thyroidea ima artery

Image sourced from Wikipedia

Next question
Question 514 of 560

A 28 year old man is undergoing an appendicectomy. The external oblique aponeurosis is incised
and the underlying muscle split in the line of its fibres. At the medial edge of the wound is a tough
fibrous structure. Entry to this structure will most likely encounter which of the following?

Internal oblique

Rectus abdominis

Transversus abdominis

Linea alba

Peritoneum

This structure will be the rectus sheath and when entered the rectus abdominis muscle will be
encountered.
Please rate this question:

Discuss and give feedback


Next question

Abdominal incisions

Midline incision  Commonest approach to the abdomen


 Structures divided: linea alba, transversalis fascia, extraperitoneal fat,
peritoneum (avoid falciform ligament above the umbilicus)
 Bladder can be accessed via an extraperitoneal approach through the
space of Retzius

Paramedian  Parallel to the midline (about 3-4cm)


incision  Structures divided/retracted: anterior rectus sheath, rectus (retracted),
posterior rectus sheath, transversalis fascia, extraperitoneal fat,
peritoneum
 Incision is closed in layers
Battle  Similar location to paramedian but rectus displaced medially (and thus
denervated)
 Now seldom used

Kocher's Incision under right subcostal margin e.g. Cholecystectomy (open)

Lanz Incision in right iliac fossa e.g. Appendicectomy

Gridiron Oblique incision centered over McBurneys point- usually appendicectomy


(less cosmetically acceptable than Lanz

Gable Rooftop incision

Pfannenstiel's Transverse supra pubic, primarily used to access pelvic organs

McEvedy's Groin incision e.g. Emergency repair strangulated femoral hernia

Rutherford Extraperitoneal approach to left or right lower quadrants. Gives excellent


Morrison access to iliac vessels and is the approach of choice for first time renal
transplantation.

Image sourced from Wikipedia

Next question
Question 515 of 560

A 35 year old man presents to the surgical clinic with a suspected direct inguinal hernia. These will
pass through Hesselbach's triangle. Which of the following forms the medial edge of this structure?

External oblique aponeurosis

Inferior epigastric artery

Rectus abdominis muscle

Inferior epigastric vein

Obturator nerve

Direct inguinal hernias pass through Hesselbachs triangle (although this is of minimal clinical
significance!). Its medial boundary is the rectus muscle.

Please rate this question:

Discuss and give feedback

Next question

Hesselbach's triangle

Direct hernias pass through Hesselbachs triangle.

Superolaterally Epigastric vessels


Medially Lateral edge of rectus muscle

Inferiorly Inguinal ligament

The boundaries of Hesselbachs triangle are commonly tested and illustrated below

Image sourced from Wikipedia

Next question
Question 516 of 560

Which of the following muscles is not innervated by the ansa cervicalis?

Sternohyoid

Mylohyoid

Omohyoid

Sternothyroid

None of the above

Ansa cervicalis muscles:

GHost THought SOmeone Stupid Shot Irene

GenioHyoid
ThyroidHyoid
Superior Omohyoid
SternoThyroid
SternoHyoid
Inferior Omohyoid

Mylohyoid is innervated by the mylohyoid branch of the inferior alveolar nerve.

Please rate this question:

Discuss and give feedback

Next question

Ansa cervicalis
Superior Branch of C1 anterolateral to carotid sheath
root

Inferior root Derived from C2 and C3 roots, passes posterolateral to the internal jugular vein (may lie
either deep or superficial to it)

Innervation Sternohyoid
Sternothyroid
Omohyoid

The ansa cervicalis lies anterior to the carotid sheath. The nerve supply to the inferior strap muscles
enters at their inferior aspect. Therefore when dividing these muscles to expose a large goitre, the
muscles should be divided in their upper half.
Image sourced from Wikipedia
Question 517 of 560

A 58 year old lady presents with a mass in the upper outer quadrant of the right breast. Which of the
following statements relating to the breast is untrue?

The internal mammary artery provides the majority of its arterial supply

Nipple retraction may occur as a result of tumour infiltration of the clavipectoral fascia

The internal mammary artery is a branch of the subclavian artery

Up to 70% of lymphatic drainage is to the ipsilateral axillary nodes

None of the above

Both skin dimpling and nipple retraction are features of breast malignancy. However, they usually
occur as a result of tumour infiltration of the breast ligaments and ducts respectively. The
clavipectoral fascia encases the axillary contents. The lymphatic drainage of the breast is to the
axilla and also to the internal mammary chain. The breast is well vascularised and the internal
mammary artery is a branch of the subclavian artery.
Please rate this question:

Discuss and give feedback


Next question

Breast

The breast itself lies on a layer of pectoral fascia and the following muscles:
1. Pectoralis major
2. Serratus anterior
3. External oblique

Image showing the topography of the female breast


Image sourced from Wikipedia

Breast anatomy
Nerve supply Branches of intercostal nerves from T4-T6.

Arterial supply  Internal mammary (thoracic) artery


 External mammary artery (laterally)
 Anterior intercostal arteries
 Thoraco-acromial artery

Venous drainage Superficial venous plexus to subclavian, axillary and intercostal veins.

Lymphatic  70% Axillary nodes


drainage  Internal mammary chain
 Other lymphatic sites such as deep cervical and supraclavicular fossa
(later in disease)

Next question
Question 518 of 560

Where are accessory spleens not found?

Gonads

Tail of pancreas

Greater omentum

Splenorenal ligament

Ureter

Accessory spleens

- 10% population
- 1 cm size
- locations: hilum of the spleen, tail of the pancreas, along the splenic vessels, in the gastrosplenic
ligament, the splenorenal ligament, the walls of the stomach or intestines, the greater omentum, the
mesentery, the gonads
Please rate this question:

Discuss and give feedback


Next question

Spleen

The spleen is located in the left upper quadrant of the abdomen and its size can vary depending
upon the amount of blood it contains. The typical adult spleen is 12.5cm long and 7.5cm wide. The
usual weight of the adult spleen is 150g.
The exact position of the spleen can vary with respiratory activity, posture and the state of
surrounding viscera. It usually lies obliquely with its long axis aligned to the 9th, 10th and 11th ribs. It
is separated from these ribs by both diaphragm and pleural cavity. The normal spleen is not
palpable.

The shape of the spleen is influenced by the state of the colon and stomach. Gastric distension will
cause the spleen to resemble the shape of an orange segment. Colonic distension will cause it to
become more tetrahedral.

The spleen is almost entirely covered by peritoneum, which adheres firmly to its capsule. Recesses
of the greater sac separate it from the stomach and kidney. It develops from the upper dorsal
mesogastrium, remaining connected to the posterior abdominal wall and stomach by two folds of
peritoneum; the lienorenal ligament and gastrosplenic ligament. The lienorenal ligament is derived
from peritoneum where the wall of the general peritoneum meets the omental bursa between the left
kidney and spleen; the splenic vessels lie in its layers. The gastrosplenic ligament also has two
layers, formed by the meeting of the walls of the greater sac and omental bursa between spleen and
stomach, the short gastric and left gastroepiploic branches of the splenic artery pass in its layers.
Laterally, the spleen is in contact with the phrenicocolic ligament.

Relations
Superiorly Diaphragm

Anteriorly Gastric impression

Posteriorly Kidney

Inferiorly Colon

Tail of pancreas and splenic vessels (splenic artery divides here, branches pass to the
Hilum
white pulp transporting plasma)

Contents
White Immune function. Contains central trabecular artery. The germinal centres are supplied
pulp by arterioles called penicilliary radicles.

Red pulp Filters abnormal red blood cells.

Function

 Filtration of abnormal blood cells and foreign bodies such as bacteria.


 Immunity: IgM. Production of properdin, and tuftsin which help target fungi and bacteria for
phagocytosis.
 Haematopoiesis: up to 5th month gestation or in haematological disorders.
 Pooling: storage of 40% platelets.
 Iron reutilisation
 Storage monocytes

Disorders of the spleen


Massive splenomegaly

 Myelofibrosis
 Chronic myeloid leukaemia
 Visceral leishmaniasis (kala-azar)
 Malaria
 Gaucher's syndrome
Other causes (as above plus)

 Portal hypertension e.g. secondary to cirrhosis


 Lymphoproliferative disease e.g. CLL, Hodgkin's
 Haemolytic anaemia
 Infection: hepatitis, glandular fever
 Infective endocarditis
 Sickle-cell*, thalassaemia
 Rheumatoid arthritis (Felty's syndrome)

*the majority of adult patients with sickle-cell will have an atrophied spleen due to repeated infarction
Next question
Question 519-521 of 560

Theme: Nerve injury

A. Median nerve
B. Ulnar nerve
C. Radial nerve
D. Anterior interosseous nerve
E. Posterior interosseous nerve
F. Axillary nerve
G. Musculocutaneous nerve

Please select the nerve at risk of injury in each scenario. Each option may be used once, more than
once or not at all.

519. A 43 year old typist presents with pain at the dorsal aspect of the upper part of her
forearm. She also complains of weakness when extending her fingers. On examination
triceps and supinator are both functioning normally. There is weakness of most of the
extensor muscles. However, there is no sensory deficit.

You answered Median nerve

The correct answer is Posterior interosseous nerve

The radial nerve may become entrapped in the "arcade of Frohse" which is a superficial
part of the supinator muscle which overlies the posterior interosseous nerve. This nerve is
entirely muscular and articular in its distribution. It passes postero-inferiorly and gives
branches to extensor carpi radialis brevis and supinator. It enters supinator and curves
around the lateral and posterior surfaces of the radius. On emerging from the supinator
the posterior interosseous nerve lies between the superficial extensor muscles and the
lowermost fibres of supinator. It then gives branches to the extensors.

520. A 28 year teacher reports difficulty with writing. There is no sensory loss. She is known
to have an aberrant Gantzer muscle.

You answered Median nerve

The correct answer is Anterior interosseous nerve

Anterior interosseous lesions occur due to fracture, or rarely due to compression. The
Gantzer muscle is an aberrant accessory of the flexor pollicis longus and is a risk factor
for anterior interosseous nerve compression. Remember loss of pincer grip and normal
sensation indicates an interosseous nerve lesion.
521. A 35 year tennis player attends reporting tingling down his arm. He says that his 'funny
bone' was hit very hard by a tennis ball. There is weakness of abduction and adduction of
his extended fingers.

You answered Median nerve

The correct answer is Ulnar nerve

Theme from September 2012 exam


The ulnar nerve arises from the medial cord of the brachial plexus (C8, T1 and
contribution from C7). The nerve descends between the axillary artery and vein, posterior
to the cutaneous nerve of the forearm and then lies anterior to triceps on the medial side
of the brachial artery. In the distal half of the arm it passes through the medial
intermuscular septum, and continues between this structure and the medial head of triceps
to enter the forearm between the medial epicondyle of the humerus and the olecranon. It
may be injured at this site in this scenario.

Please rate this question:

Discuss and give feedback


Next question

Brachial plexus

Origin Anterior rami of C5 to T1

Sections of the  Roots, trunks, divisions, cords, branches


plexus  Mnemonic:Real Teenagers Drink Cold Beer

Roots  Located in the posterior triangle


 Pass between scalenus anterior and medius

Trunks  Located posterior to middle third of clavicle


 Upper and middle trunks related superiorly to the subclavian artery
 Lower trunk passes over 1st rib posterior to the subclavian artery

Divisions Apex of axilla

Cords Related to axillary artery


Diagram illustrating the branches of the brachial plexus

Image sourced from Wikipedia

Cutaneous sensation of the upper limb


Image sourced from Wikipedia

Next question
Question 522 of 560

A 72 year old man is undergoing a left pneumonectomy for carcinoma of the bronchus. As the
surgeons approach the root of the lung, which structure will lie most anteriorly (in the anatomical
plane)?

Vagus nerve

Phrenic nerve

Bronchus

Pulmonary vein

Pulmonary artery

The phrenic nerve is the most anteriorly located structure in the lung root. The vagus nerve lies most
posteriorly.
Please rate this question:

Discuss and give feedback


Next question

Lung anatomy

The right lung is composed of 3 lobes divided by the oblique and transverse fissures. The left lung
has two lobes divided by the oblique fissure.The apex of both lungs is approximately 4cm superior to
the sterno-costal joint of the first rib. Immediately below this is a sulcus created by the subclavian
artery.

Peripheral contact points of the lung

 Base: diaphragm
 Costal surface: corresponds to the cavity of the chest
 Mediastinal surface: Contacts the mediastinal pleura. Has the cardiac impression. Above and
behind this concavity is a triangular depression named the hilum, where the structures which
form the root of the lung enter and leave the viscus. These structures are invested by pleura,
which, below the hilum and behind the pericardial impression, forms the pulmonary ligament
Right lung
Above the hilum is the azygos vein; Superior to this is the groove for the superior vena cava and
right innominate vein; behind this, and nearer the apex, is a furrow for the innominate artery. Behind
the hilum and the attachment of the pulmonary ligament is a vertical groove for the oesophagus; In
front and to the right of the lower part of the oesophageal groove is a deep concavity for the
extrapericardiac portion of the inferior vena cava.

The root of the right lung lies behind the superior vena cava and the right atrium, and below the
azygos vein.

The right main bronchus is shorter, wider and more vertical than the left main bronchus and
therefore the route taken by most foreign bodies.

Image sourced from Wikipedia

Left lung
Above the hilum is the furrow produced by the aortic arch, and then superiorly the groove
accommodating the left subclavian artery; Behind the hilum and pulmonary ligament is a vertical
groove produced by the descending aorta, and in front of this, near the base of the lung, is the lower
part of the oesophagus.

The root of the left lung passes under the aortic arch and in front of the descending aorta.
Image sourced from Wikipedia

Inferior borders of both lungs

 6th rib in mid clavicular line


 8th rib in mid axillary line
 10th rib posteriorly

The pleura runs two ribs lower than the corresponding lung level.

Bronchopulmonary segments
Segment number Right lung Left lung

1 Apical Apical

2 Posterior Posterior

3 Anterior Anterior

4 Lateral Superior lingular

5 Medial Inferior lingular

6 Superior (apical) Superior (apical)


Segment number Right lung Left lung

7 Medial basal Medial basal

8 Anterior basal Anterior basal

9 Lateral basal Lateral basal

10 Posterior basal Posterior basal

Next question
Question 523 of 560

A 56 year old man is undergoing an anterior resection for a carcinoma of the rectum. Which of the
structures below is least likely to be encountered during the mobilisation of the anterior rectum?

Denonvilliers' fascia

Middle sacral artery

Bladder

Rectovesical pouch

Seminal vesicles

With the exception of the middle sacral artery all of the other structures lie anterior to the rectum.
They may all be palpated during digital rectal examination.
Please rate this question:

Discuss and give feedback


Next question

Rectum

The rectum is approximately 12 cm long. It is a capacitance organ. It has both intra and
extraperitoneal components. The transition between the sigmoid colon is marked by the
disappearance of the tenia coli.The extra peritoneal rectum is surrounded by mesorectal fat that also
contains lymph nodes. This mesorectal fatty layer is removed surgically during rectal cancer surgery
(Total Mesorectal Excision). The fascial layers that surround the rectum are important clinical
landmarks, anteriorly lies the fascia of Denonvilliers. Posteriorly lies Waldeyers fascia.

Extra peritoneal rectum

 Posterior upper third


 Posterior and lateral middle third
 Whole lower third

Relations
Anteriorly (Males) Rectovesical pouch
Bladder
Prostate
Seminal vesicles

Anteriorly (Females) Recto-uterine pouch (Douglas)


Cervix
Vaginal wall

Posteriorly Sacrum
Coccyx
Middle sacral artery

Laterally Levator ani


Coccygeus

Arterial supply
Superior rectal artery

Venous drainage
Superior rectal vein

Lymphatic drainage

 Mesorectal lymph nodes (superior to dentate line)


 Internal iliac and then para-aortic nodes
 Inguinal nodes (inferior to dentate line)

Next question
Question 524 of 560

In relation to the middle cranial fossa, which of the following statements relating to the foramina is
incorrect?

The foramen rotundum transmits the maxillary nerve

The foramen lacerum is closely related to the internal carotid artery

The foramen spinosum lies posterolateral to the foramen ovale

The foramen ovale transmits the middle meningeal artery

The foramen rotundum lies anteromedial to the foramen ovale

Theme addressed in 2010 and 2011 exam


The foramen spinosum transmits the middle meningeal artery. The foramen ovale transmits the
mandibular nerve. As the foramina weaken the bone, a fracture at this site is not uncommon.

Please rate this question:

Discuss and give feedback

Next question

Foramina of the base of the skull

Foramen Location Contents

Foramen ovale Sphenoid Otic ganglion


V3 (Mandibular nerve:3rd branch of
Foramen Location Contents

bone trigeminal)
Accessory meningeal artery
Lesser petrosal nerve
Emissary veins

Foramen spinosum Sphenoid Middle meningeal artery


bone Meningeal branch of the Mandibular nerve

Foramen rotundum Sphenoid Maxillary nerve (V2)


bone

Foramen lacerum/ Sphenoid Base of the medial pterygoid plate.


carotid canal bone Internal carotid artery*
Nerve and artery of the pterygoid canal

Jugular foramen Temporal Anterior: inferior petrosal sinus


bone Intermediate: glossopharyngeal, vagus, and accessory nerves.
Posterior: sigmoid sinus (becoming the internal jugular vein) and
some meningeal branches from the occipital and ascending
pharyngeal arteries.

Foramen magnum Occipital Anterior and posterior spinal arteries


bone Vertebral arteries
Medulla oblongata

Stylomastoid Temporal Stylomastoid artery


foramen bone Facial nerve

Superior orbital Sphenoid Oculomotor nerve (III)


fissure bone Recurrent meningeal artery
Trochlear nerve (IV)
Lacrimal, frontal and nasociliary branches of ophthalmic nerve (V1)
Foramen Location Contents

Abducent nerve (VI)


Superior ophthalmic vein

*= In life the foramen lacerum is occluded by a cartilagenous plug. The ICA initially passes into the
carotid canal which ascends superomedially to enter the cranial cavity through the foramen lacerum.

Base of skull anatomical overview

Image sourced from Wikipedia

Next question
Question 525 of 560

During an operation for varicose veins the surgeons are mobilising the long saphenous vein. Near its
point of entry to the femoral vein an artery is injured and bleeding is encountered. From where is the
bleeding most likely to originate?

Femoral artery

Profunda femoris artery

Superficial circumflex iliac artery

Superficial epigastric artery

Deep external pudendal artery

Theme from 2011 Exam


The deep external pudendal artery is a branch of the SFA and it runs medially under the long
saphenous vein near its point of union with the femoral vein. The superficial external pudendal artery
lies superior to the SFJ. Neither vessel is functionally important and if injured they are best ligated.
Please rate this question:

Discuss and give feedback


Next question

Femoral triangle anatomy

Boundaries
Superiorly Inguinal ligament

Laterally Sartorius

Medially Adductor longus

Floor Iliopsoas, adductor longus and pectineus


Roof  Fascia lata and Superficial fascia
 Superficial inguinal lymph nodes (palpable below the inguinal ligament)
 Long saphenous vein

Image sourced from Wikipedia

Contents

 Femoral vein (medial to lateral)


 Femoral artery-pulse palpated at the mid inguinal point
 Femoral nerve
 Deep and superficial inguinal lymph nodes
 Lateral cutaneous nerve
 Great saphenous vein
 Femoral branch of the genitofemoral nerve

Next question
Question 526 of 560

A 78 year old man is lifting a heavy object when a feels a pain in his forearm and is unable to
continue. He has a swelling over his upper forearm. An MRI scan shows a small cuff of tendon still
attached to the radial tuberosity consistent with a recent tear. Which of the following muscles has
been injured?

Pronator teres

Supinator

Aconeus

Brachioradialis

Biceps brachii

Biceps inserts into the radial tuberosity. Distal injuries of this muscle are rare but are reported and
are clinically more important than more proximal ruptures.
Please rate this question:

Discuss and give feedback


Next question

Radius

The radius is one of the two long forearm bones that extends from the lateral side of the elbow to the
thumb side of the wrist. It has two expanded ends, of which the distal end is the larger. Key points
relating to its topography and relations are outlined below;

Upper end

 Articular cartilage- covers medial > lateral side


 Articulates with radial notch of the ulna by the annular ligament
 Muscle attachment- biceps brachii at the tuberosity

Shaft
Muscle attachment
Upper third of the body Supinator
Flexor digitorum superficialis
Flexor pollicis longus

Middle third of the body Pronator teres

Lower quarter of the body Pronator quadratus


Tendon of supinator longus

Lower end

 Quadrilateral
 Anterior surface- capsule of wrist joint
 Medial surface- head of ulna
 Lateral surface- ends in the styloid process
 Posterior surface: 3 grooves containing:

1. Tendons of extensor carpi radialis longus and brevis


2. Tendon of extensor pollicis longus
3. Tendon of extensor indicis

Image sourced from Wikipedia

Next question
Question 527 of 560

What is embryological origin of the pulmonary artery?

First pharyngeal arch

Second pharyngeal arch

Fourth pharyngeal arch

Fifth pharyngeal arch

Sixth pharyngeal arch

Theme from September 2011 Exam


Theme from September 2012 Exam
The proximal part of the sixth right pharyngeal arch gives origin to the right pulmonary artery. The
distal part gives origin to the left pulmonary artery and the ductus arteriosus.

Please rate this question:

Discuss and give feedback

Next question

Pharyngeal arches

These develop during the fourth week of embryonic growth from a series of mesodermal
outpouchings of the developing pharynx.
They develop and fuse in the ventral midline. Pharyngeal pouches form on the endodermal side
between the arches.
There are 6 pharyngeal arches, the fifth does not contribute any useful structures and often fuses
with the sixth arch.
Pharyngeal arches

Pharyngeal Muscular Skeletal


arch contributions contributions Endocrine Artery Nerve

First Muscles of Maxilla n/a Maxillary Mandibular


mastication Meckels External
Anterior belly of cartilage carotid
digastric Incus
Mylohyoid Malleus
Tensor tympanic
Tensor veli palatini

Second Buccinator Stapes n/a Inferior Facial


Platysma Styloid process branch of
Muscles of facial Lesser horn superior
expression and upper thyroid artery
Stylohyoid body of hyoid Stapedial
Posterior belly of artery
digastric
Stapedius

Third Stylopharyngeus Greater horn Thymus Common and Glossopharyngeal


and lower part Inferior internal
of hyoid parathyroids carotid

Fourth Cricothyroid Thyroid and Superior Right- Vagus


All intrinsic epiglottic parathyroids subclavian
muscles of the soft cartilages artery, Left-
palate aortic arch

Sixth All intrinsic Cricoid, n/a Right - Vagus and


muscles of the arytenoid and Pulmonary recurrent
larynx (except corniculate artery, Left- laryngeal nerve
cricothyroid) cartilages Pulmonary
artery and
Pharyngeal Muscular Skeletal
arch contributions contributions Endocrine Artery Nerve

ductus
arteriosus

Next question
Question 528 of 560

A 53 year old lady presents with pain and discomfort in her hand. She works as a typist and notices
that the pain is worst when she is working. She also suffers symptoms at night. Her little finger is
less affected by the pain. Which of the nerves listed below is most likely to be affected?

Radial

Median

Ulnar

Anterior interosseous nerve

Posterior interosseous nerve

Motor supply: LOAF

L ateral 2 lumbricals
O pponens pollicis
A bductor pollicisbrevis
F lexor pollicis brevis

Theme from September 2015 Exam


The most likely diagnosis here is carpal tunnel syndrome, the median nerve is compressed in the
wrist and symptoms usually affect the fingers and wrist either at night or when the hand is being
used (e.g. as a typist).
Please rate this question:

Discuss and give feedback


Next question

Median nerve

The median nerve is formed by the union of a lateral and medial root respectively from the lateral
(C5,6,7) and medial (C8 and T1) cords of the brachial plexus; the medial root passes anterior to the
third part of the axillary artery. The nerve descends lateral to the brachial artery, crosses to its
medial side (usually passing anterior to the artery). It passes deep to the bicipital aponeurosis and
the median cubital vein at the elbow.
It passes between the two heads of the pronator teres muscle, and runs on the deep surface of
flexor digitorum superficialis (within its fascial sheath).
Near the wrist it becomes superficial between the tendons of flexor digitorum superficialis and flexor
carpi radialis, deep to palmaris longus tendon. It passes deep to the flexor retinaculum to enter the
palm, but lies anterior to the long flexor tendons within the carpal tunnel.

Branches
Region Branch

Upper arm No branches, although the nerve commonly communicates with the
musculocutaneous nerve

Forearm Pronator teres


Flexor carpi radialis
Palmaris longus
Flexor digitorum superficialis
Flexor pollicis longus
Flexor digitorum profundus (only the radial half)

Distal Palmar cutaneous branch


forearm

Hand Motor supply (LOAF)


(Motor)

 Lateral 2 lumbricals
 Opponens pollicis
 Abductor pollicis brevis
 Flexor pollicis brevis

Hand  Over thumb and lateral 2 ½ fingers


(Sensory)  On the palmar aspect this projects proximally, on the dorsal aspect only the
distal regions are innervated with the radial nerve providing the more
proximal cutaneous innervation.

Patterns of damage
Damage at wrist

 e.g. carpal tunnel syndrome


 paralysis and wasting of thenar eminence muscles and opponens pollicis (ape hand
deformity)
 sensory loss to palmar aspect of lateral (radial) 2 ½ fingers
Damage at elbow, as above plus:

 unable to pronate forearm


 weak wrist flexion
 ulnar deviation of wrist

Anterior interosseous nerve (branch of median nerve)

 leaves just below the elbow


 results in loss of pronation of forearm and weakness of long flexors of thumb and index
finger

Topography of the median nerve

Image sourced from Wikipedia

Next question
Question 529 of 560

Which of the following muscles lies medial to the long thoracic nerve?

Serratus anterior

Latissimus dorsi

Pectoralis major

Pectoralis minor

None of the above

Theme from 2009 Exam


Please rate this question:

Discuss and give feedback


Next question

Long thoracic nerve

 Derived from ventral rami of C5, C6, and C7 (close to their emergence from intervertebral
foramina)
 It runs downward and passes either anterior or posterior to the middle scalene muscle
 It reaches upper tip of serratus anterior muscle and descends on outer surface of this
muscle, giving branches into it
 Winging of Scapula occurs in long thoracic nerve injury (most common) or from spinal
accessory nerve injury (which denervates the trapezius) or a dorsal scapular nerve injury

Next question
Question 530 of 560

The thebesian veins contribute to the venous drainage of the heart. Into which of the following
structures do they primarily drain?

Great cardiac vein

Atrium

Superior vena cava

Oblique vein

Small cardiac vein

The thebesian veins are numerous small veins running over the surface of the heart they drain into
the heart itself. Usually this is to the atrium directly.
Please rate this question:

Discuss and give feedback


Next question

Heart anatomy

The walls of each cardiac chamber comprise:

 Epicardium
 Myocardium
 Endocardium

Cardiac muscle is attached to the cardiac fibrous skeleton.

Relations
The heart and roots of the great vessels within the pericardial sac are related anteriorly to the
sternum, medial ends of the 3rd to 5th ribs on the left and their associated costal cartilages. The
heart and pericardial sac are situated obliquely two thirds to the left and one third to the right of the
median plane.

The pulmonary valve lies at the level of the left third costal cartilage.
The mitral valve lies at the level of the fourth costal cartilage.

Coronary sinus
This lies in the posterior part of the coronary groove and receives blood from the cardiac veins. The
great cardiac vein lies at its left and the middle and small cardiac veins lie on its right. The smallest
cardiac vein (anterior cardiac vein) drains into the right atrium directly.

Aortic sinus
Right coronary artery arises from the right aortic sinus, the left is derived from the left aortic sinus,
which lies posteriorly.

Right and left ventricles

Structure Left Ventricle

A-V Valve Mitral (double leaflet)

Walls Twice as thick as right

Trabeculae carnae Much thicker and more numerous

Right coronary artery


The RCA supplies:

 Right atrium
 Diaphragmatic part of the left ventricle
 Usually the posterior third of the interventricular septum
 The sino atrial node (60% cases)
 The atrio ventricular node (80% cases)

Left coronary artery


The LCA supplies:

 Left atrium
 Most of left ventricle
 Part of the right ventricle
 Anterior two thirds of the inter ventricular septum
 The sino atrial node (remaining 40% cases)

Innervation of the heart


Autonomic nerve fibres from the superficial and deep cardiac plexus. These lie anterior to the
bifurcation of the trachea, posterior to the ascending aorta and superior to the bifurcation of the
pulmonary trunk. The parasympathetic supply to the heart is from presynaptic fibres of the vagus
nerves.

Valves of the heart


Mitral valve Aortic valve Pulmonary valve Tricuspid valve

2 cusps 3 cusps 3 cusps 3 cusps

First heart sound Second heart Second heart First heart sound
sound sound

1 anterior cusp 2 anterior cusps 2 anterior cusps 2 anterior cusps

Attached to chordae No chordae No chordae Attached to chordae


tendinae tendinae

Next question
Question 531 of 560

Which of the following is not contained within the deep posterior compartment of the lower leg?

Tibialis posterior muscle

Posterior tibial artery

Tibial nerve

Sural nerve

Flexor hallucis longus

The deep posterior compartment lies anterior to soleus. The sural nerve is superficially sited and
therefore not contained within it.

Please rate this question:

Discuss and give feedback

Next question

Lower limb- Muscular compartments

Anterior compartment

Muscle Nerve Action

Tibialis anterior Deep peroneal nerve Dorsiflexes ankle joint, inverts foot
Muscle Nerve Action

Extensor digitorum longus Deep peroneal nerve Extends lateral four toes, dorsiflexes ankle joint

Peroneus tertius Deep peroneal nerve Dorsiflexes ankle, everts foot

Extensor hallucis longus Deep peroneal nerve Dorsiflexes ankle joint, extends big toe

Peroneal compartment

Muscle Nerve Action

Peroneus longus Superficial peroneal nerve Everts foot, assists in plantar flexion

Peroneus brevis Superficial peroneal nerve Plantar flexes the ankle joint

Superficial posterior compartment

<muscle< b="" style="box-sizing: border-


box;"></muscle<> Nerve Action

Gastrocnemius Tibial Plantar flexes the foot, may also flex


nerve the knee

Soleus Tibial Plantar flexor


nerve

Deep posterior compartment

Muscle Nerve Action


Muscle Nerve Action

Flexor digitorum longus Tibial Flexes the lateral four toes

Flexor hallucis longus Tibial Flexes the great toe

Tibialis posterior Tibial Plantar flexor, inverts the foot

Next question
Question 532 of 560

When performing minor surgery in the scalp, which of the following regions is considered a danger
area as regards spread of infection into the CNS?

Aponeurosis epicranialis

Skin

Pericranium

Connective tissue

Loose areolar tissue

This area is most dangerous as infections can spread easily. The emissary veins that drain this area
may allow sepsis to spread to the cranial cavity.
Please rate this question:

Discuss and give feedback


Next question

Head injury

Patients who suffer head injuries should be managed according to ATLS principles and extra cranial
injuries should be managed alongside cranial trauma. Inadequate cardiac output will compromise
CNS perfusion irrespective of the nature of the cranial injury.

Types of traumatic brain injury


Bleeding into the space between the dura mater and the skull. Often results from
acceleration-deceleration trauma or a blow to the side of the head. The majority
of extradural haematomas occur in the temporal region where skull fractures
cause a rupture of the middle meningeal artery.

Extradural
Features
haematoma

 Raised intracranial pressure


 Some patients may exhibit a lucid interval
Bleeding into the outermost meningeal layer. Most commonly occur around the
frontal and parietal lobes. May be either acute or chronic.
Subdural
haematoma Risk factors include old age and alcoholism.

Slower onset of symptoms than a extradural haematoma.

Usually occurs spontaneously in the context of a ruptured cerebral aneurysm, but


Subarachnoid
may be seen in association with other injuries when a patient has sustained a
haemorrhage
traumatic brain injury.

Pathophysiology

 Primary brain injury may be focal (contusion/ haematoma) or diffuse (diffuse axonal injury)
 Diffuse axonal injury occurs as a result of mechanical shearing following deceleration,
causing disruption and tearing of axons
 Intra-cranial haematomas can be extradural, subdural or intracerebral, while contusions may
occur adjacent to (coup) or contralateral (contre-coup) to the side of impact
 Secondary brain injury occurs when cerebral oedema, ischaemia, infection, tonsillar or
tentorial herniation exacerbates the original injury. The normal cerebral auto regulatory
processes are disrupted following trauma rendering the brain more susceptible to blood flow
changes and hypoxia
 The Cushings reflex (hypertension and bradycardia) often occurs late and is usually a pre
terminal event

Management

 Where there is life threatening rising ICP such as in extra dural haematoma and whilst
theatre is prepared or transfer arranged use of IV mannitol/ frusemide may be required.
 Diffuse cerebral oedema may require decompressive craniotomy
 Exploratory Burr Holes have little management in modern practice except where scanning
may be unavailable and to thus facilitate creation of formal craniotomy flap
 Depressed skull fractures that are open require formal surgical reduction and debridement,
closed injuries may be managed non operatively if there is minimal displacement.
 ICP monitoring is appropriate in those who have GCS 3-8 and normal CT scan.
 ICP monitoring is mandatory in those who have GCS 3-8 and abnormal CT scan.
 Hyponatraemia is most likely to be due to syndrome of inappropriate ADH secretion.
 Minimum of cerebral perfusion pressure of 70mmHg in adults.
 Minimum cerebral perfusion pressure of between 40 and 70 mmHg in children.

Interpretation of pupillary findings in head injuries


Pupil size Light response Interpretation
Pupil size Light response Interpretation

Unilaterally dilated Sluggish or fixed 3rd nerve compression secondary to tentorial


herniation

Bilaterally dilated Sluggish or fixed  Poor CNS perfusion


 Bilateral 3rd nerve palsy

Unilaterally dilated or Cross reactive (Marcus - Optic nerve injury


equal Gunn)

Bilaterally constricted May be difficult to  Opiates


assess  Pontine lesions
 Metabolic encephalopathy

Unilaterally Preserved Sympathetic pathway disruption


constricted
Next question
Question 533 of 560

Which of the following structures are at risk of direct injury following a fracture dislocation of the
femoral condyles?

Popliteal artery

Sciatic nerve

Plantaris muscle

Tibial artery

Tibial nerve

The heads of gastrocnemius will contract to pull the fracture segment posteriorly. The popliteal artery
lies against the bone and may be damaged or compressed.
Please rate this question:

Discuss and give feedback


Next question

Popliteal fossa

Boundaries of the popliteal fossa


Laterally Biceps femoris above, lateral head of gastrocnemius and plantaris below

Medially Semimembranosus and semitendinosus above, medial head of gastrocnemius below

Floor Popliteal surface of the femur, posterior ligament of knee joint and popliteus muscle

Roof Superficial and deep fascia

Image showing the popliteal fossa


© Image provided by the University of Sheffield

Contents

 Popliteal artery and vein


 Small saphenous vein
 Common peroneal nerve
 Tibial nerve
 Posterior cutaneous nerve of the thigh
 Genicular branch of the obturator nerve
 Lymph nodes

Next question
Question 534 of 560

A 25 year old man is being catheterised, prior to a surgical procedure. As the catheter enters the
prostatic urethra which of the following changes will occur?

Resistance will increase significantly

Resistance will increase slightly

It will lie horizontally

Resistance will decrease

It will deviate laterally

Theme from September 2011 Exam


The prostatic urethra is much wider than the membranous urethra and therefore resistance will
decrease. The prostatic urethra is inclined vertically.
Please rate this question:

Discuss and give feedback


Next question

Prostate gland

The prostate gland is approximately the shape and size of a walnut and is located inferior to the
bladder. It is separated from the rectum by Denonvilliers fascia and its blood supply is derived from
the internal iliac vessels (via inferior vesical artery). The internal sphincter lies at the apex of the
gland and may be damaged during prostatic surgery, affected individuals may complain of
retrograde ejaculation.

Summary of prostate gland


Arterial supply Inferior vesical artery (from internal iliac)

Venous drainage Prostatic venous plexus (to paravertebral veins)


Lymphatic Internal iliac nodes
drainage

Innervation Inferior hypogastric plexus

Dimensions  Transverse diameter (4cm)


 AP diameter (2cm)
 Height (3cm)

Lobes  Posterior lobe: posterior to urethra


 Median lobe: posterior to urethra, in between ejaculatory ducts
 Lateral lobes x 2
 Isthmus

Zones  Peripheral zone: subcapsular portion of posterior prostate. Most


prostate cancers are here
 Central zone
 Transition zone
 Stroma

Relations
Pubic symphysis
Anterior Prostatic venous plexus

Posterior Denonvilliers fascia


Rectum
Ejaculatory ducts

Lateral Venous plexus (lies on prostate)


Levator ani (immediately below the puboprostatic ligaments)
Image sourced from Wikipedia

Next question
Question 535 of 560

A 24 year female is admitted to A&E with tingling of her hand after a fall. She is found to have a
fracture of the medial epicondyle. What is the most likely nerve lesion?

Ulnar nerve

Radial nerve

Median nerve

Axillary nerve

Cutaneous nerve

The radial nerve is located near the lateral epicondyle.


Please rate this question:

Discuss and give feedback


Next question

Ulnar nerve

Origin

 C8, T1

Supplies (no muscles in the upper arm)

 Flexor carpi ulnaris


 Flexor digitorum profundus
 Flexor digiti minimi
 Abductor digiti minimi
 Opponens digiti minimi
 Adductor pollicis
 Interossei muscle
 Third and fourth lumbricals
 Palmaris brevis
Path

 Posteromedial aspect of upper arm to flexor compartment of forearm, then along the ulnar.
Passes beneath the flexor carpi ulnaris muscle, then superficially through the flexor
retinaculum into the palm of the hand.

Image sourced from Wikipedia

Branches
Branch Supplies

Muscular branch Flexor carpi ulnaris


Medial half of the flexor digitorum profundus
Branch Supplies

Palmar cutaneous branch (Arises near the Skin on the medial part of the palm
middle of the forearm)

Dorsal cutaneous branch Dorsal surface of the medial part of the hand

Superficial branch Cutaneous fibres to the anterior surfaces of the


medial one and one-half digits

Deep branch Hypothenar muscles


All the interosseous muscles
Third and fourth lumbricals
Adductor pollicis
Medial head of the flexor pollicis brevis

Effects of injury
Damage at the wrist  Wasting and paralysis of intrinsic hand muscles (claw hand)
 Wasting and paralysis of hypothenar muscles
 Loss of sensation medial 1 and half fingers

Damage at the elbow  Radial deviation of the wrist


 Clawing less in 4th and 5th digits

Next question
Question 536 of 560

During a gangland gunfight a man is shot in the chest. The bullet passes through the posterior
mediastinum (from left to right). Which of the following structures is least likely to be injured

Thoracic duct

Oesophagus

Vagus nerve

Descending thoracic aorta

Arch of the azygos vein

The arch of the azygos vein lies in the middle mediastinum.


Please rate this question:

Discuss and give feedback


Next question

Mediastinum

Region between the pulmonary cavities.


It is covered by the mediastinal pleura. It does not contain the lungs.
It extends from the thoracic inlet superiorly to the diaphragm inferiorly.

Mediastinal regions

 Superior mediastinum (between manubriosternal angle and T4/5)


 Middle mediastinum
 Posterior mediastinum
 Anterior mediastinum

Region Contents
Region Contents

Superior mediastinum  Superior vena cava


 Brachiocephalic veins
 Arch of aorta
 Thoracic duct
 Trachea
 Oesophagus
 Thymus
 Vagus nerve
 Left recurrent laryngeal nerve
 Phrenic nerve

Anterior mediastinum  Thymic remnants


 Lymph nodes
 Fat

Middle mediastinum  Pericardium


 Heart
 Aortic root
 Arch of azygos vein
 Main bronchi

Posterior mediastinum  Oesophagus


 Thoracic aorta
 Azygos vein
 Thoracic duct
 Vagus nerve
 Sympathetic nerve trunks
 Splanchnic nerves

Next question
Question 537 of 560

The space between the vocal cords is referred to as which of the following?

Piriform recess

Rima vestibuli

Vestibule

Glottis

Rima glottidis

The rima glottidis is the narrowest part of the laryngeal cavity.


Please rate this question:

Discuss and give feedback


Next question

Larynx

The larynx lies in the anterior part of the neck at the levels of C3 to C6 vertebral bodies. The
laryngeal skeleton consists of a number of cartilagenous segments. Three of these are paired;
arytenoid, corniculate and cuneiform. Three are single; thyroid, cricoid and epiglottic. The cricoid
cartilage forms a complete ring (the only one to do so).
The laryngeal cavity extends from the laryngeal inlet to the level of the inferior border of the cricoid
cartilage.

Divisions of the laryngeal cavity


Laryngeal vestibule Superior to the vestibular folds

Laryngeal ventricle Lies between vestibular folds and superior to the vocal cords

Infraglottic cavity Extends from vocal cords to inferior border of the cricoid cartilage
The vocal folds (true vocal cords) control sound production. The apex of each fold projects medially
into the laryngeal cavity. Each vocal fold includes:

 Vocal ligament
 Vocalis muscle (most medial part of thyroarytenoid muscle)

The glottis is composed of the vocal folds, processes and rima glottidis. The rima glottidis is the
narrowest potential site within the larynx, as the vocal cords may be completely opposed, forming a
complete barrier.

Muscles of the larynx


Muscle Origin Insertion Innervation Action

Posterior Posterior aspect Muscular process Recurrent Abducts vocal fold


cricoarytenoid of lamina of of arytenoid Laryngeal
cricoid

Lateral Arch of cricoid Muscular process Recurrent Adducts vocal fold


cricoarytenoid of arytenoid laryngeal

Thyroarytenoid Posterior aspect Muscular process Recurrent Relaxes vocal fold


of thyroid of arytenoid laryngeal
cartilage

Transverse and Arytenoid Contralateral Recurrent Closure of


oblique cartilage arytenoid laryngeal intercartilagenous
arytenoids part of the rima
glottidis

Vocalis Depression Vocal ligament Recurrent Relaxes posterior


between lamina and vocal process laryngeal vocal ligament, tenses
of thyroid of arytenoid anterior part
cartilage cartilage

Cricothyroid Anterolateral Inferior margin External Tenses vocal fold


part of cricoid and horn of laryngeal
thyroid cartilage
Blood supply
Arterial supply is via the laryngeal arteries, branches of the superior and inferior thyroid arteries. The
superior laryngeal artery is closely related to the internal laryngeal nerve. The inferior laryngeal
artery is related to the inferior laryngeal nerve. Venous drainage is via superior and inferior laryngeal
veins, the former draining into the superior thyroid vein and the latter draining into the middle thyroid
vein, or thyroid venous plexus.

Lymphatic drainage
The vocal cords have no lymphatic drainage and this site acts as a lymphatic watershed.
Supraglottic part Upper deep cervical nodes

Subglottic part Prelaryngeal and pretracheal nodes and inferior deep cervical nodes

The aryepiglottic fold and vestibular folds have a dense plexus of lymphatics associated with them
and malignancies at these sites have a greater propensity for nodal metastasis.

Topography of the larynx

Image sourced from Wikipedia

Next question
Question 538 of 560

A 78 year old man develops a carcinoma of the scrotum. To which of the following lymph node
groups may the tumour initially metastasise?

Para aortic

Obturator

Inguinal

Meso rectal

None of the above

The scrotum is drained by the inguinal nodes.


Please rate this question:

Discuss and give feedback


Next question

Scrotal and testicular anatomy

Spermatic cord
Formed by the vas deferens and is covered by the following structures:
Layer Origin

Internal spermatic fascia Transversalis fascia

Cremasteric fascia From the fascial coverings of internal oblique

External spermatic fascia External oblique aponeurosis

Contents of the cord


Vas deferens Transmits sperm and accessory gland secretions

Testicular artery Branch of abdominal aorta supplies testis and


epididymis

Artery of vas deferens Arises from inferior vesical artery

Cremasteric artery Arises from inferior epigastric artery

Pampiniform plexus Venous plexus, drains into right or left testicular vein

Sympathetic nerve fibres Lie on arteries, the parasympathetic fibres lie on the
vas

Genital branch of the genitofemoral Supplies cremaster


nerve

Lymphatic vessels Drain to lumbar and para-aortic nodes

Scrotum

 Composed of skin and closely attached dartos fascia.


 Arterial supply from the anterior and posterior scrotal arteries
 Lymphatic drainage to the inguinal lymph nodes
 Parietal layer of the tunica vaginalis is the innermost layer

Testes

 The testes are surrounded by the tunica vaginalis (closed peritoneal sac). The parietal layer
of the tunica vaginalis adjacent to the internal spermatic fascia.
 The testicular arteries arise from the aorta immediately inferiorly to the renal arteries.
 The pampiniform plexus drains into the testicular veins, the left drains into the left renal vein
and the right into the inferior vena cava.
 Lymphatic drainage is to the para-aortic nodes.

Next question
Question 539 of 560

A 63 year old man is undergoing an upper GI endoscopy for dysphagia. At 33 cm (from the incisors)
a malignant looking stricture is encountered. The endoscopist attempts a balloon
dilatation.Unfortunately the tumour splits through the oesophageal wall. Into which region will the
oesophageal contents now drain?

Superior mediastinum

Posterior mediastinum

Middle mediastinum

Anterior mediastinum

Peritoneal cavity

At this position the oesophagus is still likely to be intrathoracic and located in the posterior
mediastinum.
Please rate this question:

Discuss and give feedback


Next question

Mediastinum

Region between the pulmonary cavities.


It is covered by the mediastinal pleura. It does not contain the lungs.
It extends from the thoracic inlet superiorly to the diaphragm inferiorly.

Mediastinal regions

 Superior mediastinum (between manubriosternal angle and T4/5)


 Middle mediastinum
 Posterior mediastinum
 Anterior mediastinum
Region Contents

Superior mediastinum  Superior vena cava


 Brachiocephalic veins
 Arch of aorta
 Thoracic duct
 Trachea
 Oesophagus
 Thymus
 Vagus nerve
 Left recurrent laryngeal nerve
 Phrenic nerve

Anterior mediastinum  Thymic remnants


 Lymph nodes
 Fat

Middle mediastinum  Pericardium


 Heart
 Aortic root
 Arch of azygos vein
 Main bronchi

Posterior mediastinum  Oesophagus


 Thoracic aorta
 Azygos vein
 Thoracic duct
 Vagus nerve
 Sympathetic nerve trunks
 Splanchnic nerves

Next question
Question 540 of 560

During a tricuspid valve repair the right atrium is opened, following establishment of cardiopulmonary
bypass. Which of the following structures do not lie within the right atrium?

Crista terminalis

Tricuspid valve

Fossa ovalis

Trabeculae carnae

Musculi pectinati

Structures within the right atrium:

 Musculi pectinati
 Crista terminalis
 Opening of the coronary sinus
 Fossa ovalis

The trabeculae carnae are located in the right ventricle.


Please rate this question:

Discuss and give feedback


Next question

Heart anatomy

The walls of each cardiac chamber comprise:

 Epicardium
 Myocardium
 Endocardium

Cardiac muscle is attached to the cardiac fibrous skeleton.


Relations
The heart and roots of the great vessels within the pericardial sac are related anteriorly to the
sternum, medial ends of the 3rd to 5th ribs on the left and their associated costal cartilages. The
heart and pericardial sac are situated obliquely two thirds to the left and one third to the right of the
median plane.

The pulmonary valve lies at the level of the left third costal cartilage.
The mitral valve lies at the level of the fourth costal cartilage.

Coronary sinus
This lies in the posterior part of the coronary groove and receives blood from the cardiac veins. The
great cardiac vein lies at its left and the middle and small cardiac veins lie on its right. The smallest
cardiac vein (anterior cardiac vein) drains into the right atrium directly.

Aortic sinus
Right coronary artery arises from the right aortic sinus, the left is derived from the left aortic sinus,
which lies posteriorly.

Right and left ventricles

Structure Left Ventricle

A-V Valve Mitral (double leaflet)

Walls Twice as thick as right

Trabeculae carnae Much thicker and more numerous

Right coronary artery


The RCA supplies:

 Right atrium
 Diaphragmatic part of the left ventricle
 Usually the posterior third of the interventricular septum
 The sino atrial node (60% cases)
 The atrio ventricular node (80% cases)

Left coronary artery


The LCA supplies:

 Left atrium
 Most of left ventricle
 Part of the right ventricle
 Anterior two thirds of the inter ventricular septum
 The sino atrial node (remaining 40% cases)

Innervation of the heart


Autonomic nerve fibres from the superficial and deep cardiac plexus. These lie anterior to the
bifurcation of the trachea, posterior to the ascending aorta and superior to the bifurcation of the
pulmonary trunk. The parasympathetic supply to the heart is from presynaptic fibres of the vagus
nerves.

Valves of the heart


Mitral valve Aortic valve Pulmonary valve Tricuspid valve

2 cusps 3 cusps 3 cusps 3 cusps

First heart sound Second heart Second heart First heart sound
sound sound

1 anterior cusp 2 anterior cusps 2 anterior cusps 2 anterior cusps

Attached to chordae No chordae No chordae Attached to chordae


tendinae tendinae

Next question
Question 541 of 560

Which of the following is a recognised tributary of the retromandibular vein?

Internal jugular vein

External jugular vein

Anterior temporal diploic vein

Maxillary vein

Inferior opthalmic vein

The retromandibular vein is formed from the union of the maxillary and superficial temporal veins.
Please rate this question:

Discuss and give feedback


Next question

Retromandibular vein

 Formed by a union of the maxillary vein and superficial temporal vein


 It descends through the parotid gland and bifurcates within it
 The anterior division passes forwards to join the facial vein, the posterior division is one of
the tributaries of the external jugular vein

Next question
Question 542 of 560

An 22 year old soldier is shot in the abdomen and amongst his various injuries is a major disruption
to the abdominal aorta. There is torrential haemorrhage and the surgeons decide to control the aorta
by placement of a vascular clamp immediately inferior to the diaphragm. Which of the following
vessels may be injured in this maneouvre?

Inferior phrenic arteries

Superior phrenic arteries

Splenic artery

Renal arteries

Superior mesenteric artery

As the first branches of the abdominal aorta the inferior phrenic arteries are at greatest risk. The
superior phrenic arteries lie in the thorax. The potential space at the level of the diaphragmatic hiatus
is a potentially useful site for aortic occlusion. However, leaving the clamp applied for more than
about 10 -15 minutes usually leads to poor outcomes.

Please rate this question:

Discuss and give feedback

Next question

Abdominal aorta

Abdominal aortic topography


Origin T12

Termination L4

Posterior relations L1-L4 Vertebral bodies

Anterior relations Lesser omentum


Liver
Left renal vein
Inferior mesenteric vein
Third part of duodenum
Pancreas
Parietal peritoneum
Peritoneal cavity

Right lateral relations Right crus of the diaphragm


Cisterna chyli
Azygos vein
IVC (becomes posterior distally)

Left lateral relations 4th part of duodenum


Duodenal-jejunal flexure
Left sympathetic trunk

The abdominal aorta


Image sourced from Wikipedia

Next question
Question 543 of 560

Which of the following statements relating to the gallbladder is untrue?

The fundus is usually intra peritoneal

Arterial supply is from the cystic artery

The cystic artery is usually located in Calots triangle

Calots triangle may rarely contain an aberrant hepatic artery

Cholecystokinin causes relaxation of the gallbladder

CCK causes gallbladder contraction.


Please rate this question:

Discuss and give feedback


Next question

Gallbladder

 Fibromuscular sac with capacity of 50ml


 Columnar epithelium

Relations of the gallbladder


Anterior Liver

Posterior  Covered by peritoneum


 Transverse colon
 1st part of the duodenum

Laterally Right lobe of liver

Medially Quadrate lobe of liver


Arterial supply
Cystic artery (branch of Right hepatic artery)

Venous drainage
Directly to the liver

Nerve supply
Sympathetic- mid thoracic spinal cord, Parasympathetic- anterior vagal trunk

Common bile duct

Origin Confluence of cystic and common hepatic ducts

Relations at  Medially - Hepatic artery


origin  Posteriorly- Portal vein

Relations distally  Duodenum - anteriorly


 Pancreas - medially and laterally
 Right renal vein - posteriorly

Arterial supply Branches of hepatic artery and retroduodenal branches of gastroduodenal


artery

Hepatobiliary triangle

Medially Common hepatic duct

Inferiorly Cystic duct

Superiorly Inferior edge of liver

Contents Cystic artery

Relations of the gallbladder


© Image provided by the University of Sheffield

Next question
Question 544 of 560

Which of the following nerves is the primary source of innervation to the anterior scrotal skin?

Iliohypogastric nerve

Pudendal nerve

Ilioinguinal nerve

Femoral branch of the genitofemoral nerve

Obturator nerve

Theme from April 2012 Exam


The pudendal nerve may innervate the posterior skin of the scrotum. The anterior innervation of the
scrotum is primarily provided by the ilioinguinal nerve. The genital branch of the genitofemoral nerve
provides a smaller contribution.

Please rate this question:

Discuss and give feedback

Next question

Scrotal sensation

The scrotum is innervated by the ilioinguinal nerve and the pudendal nerve. The ilioinguinal nerve
arises from L1 and pierces the internal oblique muscle. It eventually passes through the superficial
inguinal ring to innervate the anterior skin of the scrotum.

The pudendal nerve is the principal nerve of the perineum. It arises in the pelvis from 3 nerve roots.
It passes through both greater and lesser sciatic foramina to enter the perineal region. The perineal
branches pass anteromedially and divide into posterior scrotal branches. The posterior scrotal
branches pass superficially to supply the skin and fascia of the perineum. It cross communicates
with the inferior rectal nerve.

Next question
Question 545 of 560

The transversalis fascia contributes to which of the following?

Pectineal ligament

Deep inguinal ring

Cremaster muscle and fascia

Inguinal ligament

External spermatic fascia

The internal spermatic fascia (derived from transversalis fascia) invests:


Ducuts deferens
Testicular vessels

The principal outpouching of the transversalis fascia is the internal spermatic fascia. The mouth of
the outpouching is the deep inguinal ring.
Please rate this question:

Discuss and give feedback


Next question

Abdominal wall

The 2 main muscles of the abdominal wall are the rectus abdominis (anterior) and the quadratus
lumborum (posterior).
The remaining abdominal wall consists of 3 muscular layers. Each muscle passes from the lateral
aspect of the quadratus lumborum posteriorly to the lateral margin of the rectus sheath anteriorly.
Each layer is muscular posterolaterally and aponeurotic anteriorly.
Image sourced from Wikipedia

Muscles of abdominal wall


External  Lies most superficially
oblique  Originates from 5th to 12th ribs
 Inserts into the anterior half of the outer aspect of the iliac crest, linea
alba and pubic tubercle
 More medially and superiorly to the arcuate line, the aponeurotic layer
overlaps the rectus abdominis muscle
 The lower border forms the inguinal ligament
 The triangular expansion of the medial end of the inguinal ligament is
the lacunar ligament.

Internal  Arises from the thoracolumbar fascia, the anterior 2/3 of the iliac crest
oblique and the lateral 2/3 of the inguinal ligament
 The muscle sweeps upwards to insert into the cartilages of the lower 3
ribs
 The lower fibres form an aponeurosis that runs from the tenth costal
cartilage to the body of the pubis
 At its lowermost aspect it joins the fibres of the aponeurosis of
transversus abdominis to form the conjoint tendon.

Transversus  Innermost muscle


abdominis  Arises from the inner aspect of the costal cartilages of the lower 6 ribs ,
from the anterior 2/3 of the iliac crest and lateral 1/3 of the inguinal
ligament
 Its fibres run horizontally around the abdominal wall ending in an
aponeurosis. The upper part runs posterior to the rectus abdominis.
Lower down the fibres run anteriorly only.
 The rectus abdominis lies medially; running from the pubic crest and
symphysis to insert into the xiphoid process and 5th, 6th and 7th costal
cartilages. The muscles lies in a aponeurosis as described above.
 Nerve supply: anterior primary rami of T7-12

Surgical notes
During abdominal surgery it is usually necessary to divide either the muscles or their aponeuroses.
During a midline laparotomy it is desirable to divide the aponeurosis. This will leave the rectus
sheath intact above the arcuate line and the muscles intact below it. Straying off the midline will
often lead to damage to the rectus muscles, particularly below the arcuate line where they may often
be in close proximity to each other.
Next question
Question 546 of 560

A 63 year old man is undergoing a right pneumonectomy for carcinoma of the bronchus. As the
surgeons approach the root of the lung, which structure will lie most posteriorly (in the anatomical
plane)?

Phrenic nerve

Main bronchus

Vagus nerve

Pulmonary vein

Pulmonary artery

The vagus nerve is the most posteriorly located structure at the lung root. The phrenic nerve lies
most anteriorly.
Please rate this question:

Discuss and give feedback


Next question

Lung anatomy

The right lung is composed of 3 lobes divided by the oblique and transverse fissures. The left lung
has two lobes divided by the oblique fissure.The apex of both lungs is approximately 4cm superior to
the sterno-costal joint of the first rib. Immediately below this is a sulcus created by the subclavian
artery.

Peripheral contact points of the lung

 Base: diaphragm
 Costal surface: corresponds to the cavity of the chest
 Mediastinal surface: Contacts the mediastinal pleura. Has the cardiac impression. Above and
behind this concavity is a triangular depression named the hilum, where the structures which
form the root of the lung enter and leave the viscus. These structures are invested by pleura,
which, below the hilum and behind the pericardial impression, forms the pulmonary ligament
Right lung
Above the hilum is the azygos vein; Superior to this is the groove for the superior vena cava and
right innominate vein; behind this, and nearer the apex, is a furrow for the innominate artery. Behind
the hilum and the attachment of the pulmonary ligament is a vertical groove for the oesophagus; In
front and to the right of the lower part of the oesophageal groove is a deep concavity for the
extrapericardiac portion of the inferior vena cava.

The root of the right lung lies behind the superior vena cava and the right atrium, and below the
azygos vein.

The right main bronchus is shorter, wider and more vertical than the left main bronchus and
therefore the route taken by most foreign bodies.

Image sourced from Wikipedia

Left lung
Above the hilum is the furrow produced by the aortic arch, and then superiorly the groove
accommodating the left subclavian artery; Behind the hilum and pulmonary ligament is a vertical
groove produced by the descending aorta, and in front of this, near the base of the lung, is the lower
part of the oesophagus.

The root of the left lung passes under the aortic arch and in front of the descending aorta.
Image sourced from Wikipedia

Inferior borders of both lungs

 6th rib in mid clavicular line


 8th rib in mid axillary line
 10th rib posteriorly

The pleura runs two ribs lower than the corresponding lung level.

Bronchopulmonary segments
Segment number Right lung Left lung

1 Apical Apical

2 Posterior Posterior

3 Anterior Anterior

4 Lateral Superior lingular

5 Medial Inferior lingular

6 Superior (apical) Superior (apical)


Segment number Right lung Left lung

7 Medial basal Medial basal

8 Anterior basal Anterior basal

9 Lateral basal Lateral basal

10 Posterior basal Posterior basal

Next question
Question 547 of 560

A 43 year old lady is undergoing an axillary node clearance for breast cancer. The nodal disease is
bulky. During clearance of the level 3 nodes there is suddenly brisk haemorrhage. The most likely
vessel responsible is:

Thoracoacromial artery

Cephalic vein

Thoracodorsal trunk

Internal mammary artery

Posterior circumflex humeral artery

The thoracoacromial artery pierces the pectoralis major and gives off branches within this space.
The level 3 axillary nodes lie between pectoralis major and minor.Although the thoracodorsal trunk
may be injured during an axillary dissection it does not lie within the level 3 nodes.

Please rate this question:

Discuss and give feedback

Next question

Thoracoacromial artery

The thoracoacromial artery (acromiothoracic artery; thoracic axis) is a short trunk, which arises from
the forepart of the axillary artery, its origin being generally overlapped by the upper edge of the
Pectoralis minor.

Projecting forward to the upper border of the Pectoralis minor, it pierces the coracoclavicular fascia
and divides into four branches: pectoral, acromial, clavicular, and deltoid.

Branch Description

Pectoral Descends between the two Pectoral muscles, and is distributed to them and to the breast,
branch anastomosing with the intercostal branches of the internal thoracic artery and with the
lateral thoracic.

Acromial Runs laterally over the coracoid process and under the Deltoid, to which it gives branches; it
branch then pierces that muscle and ends on the acromion in an arterial network formed by
branches from the suprascapular, thoracoacromial, and posterior humeral circumflex
arteries.

Clavicular Runs upwards and medially to the sternoclavicular joint, supplying this articulation, and the
branch Subclavius.

Deltoid Arising with the acromial, it crosses over the Pectoralis minor and passes in the same
branch groove as the cephalic vein, between the Pectoralis major and Deltoid, and gives branches
to both muscles.

Next question
Question 548 of 560

A 73 year old lady with long standing atrial fibrillation develops a cold and pulseless white arm. A
brachial embolus is suspected and a brachial embolectomy is performed. Which of the following
structures is at greatest risk of injury during this procedure?

Radial nerve

Cephalic vein

Ulnar nerve

Median nerve

None of the above

The median nerve lies close to the brachial artery in the antecubital fossa. This is the usual site of
surgical access to the brachial artery for an embolectomy procedure. The median nerve may be
damaged during clumsy application of vascular clamps to the artery.

Please rate this question:

Discuss and give feedback

Next question

Brachial artery

The brachial artery begins at the lower border of teres major as a continuation of the axillary artery. It
terminates in the cubital fossa at the level of the neck of the radius by dividing into the radial and
ulnar arteries.

Relations
Posterior relations include the long head of triceps with the radial nerve and profunda vessels
intervening. Anteriorly it is overlapped by the medial border of biceps.
It is crossed by the median nerve in the middle of the arm.
In the cubital fossa it is separated from the median cubital vein by the bicipital aponeurosis.
The basilic vein is in contact at the most proximal aspect of the cubital fossa and lies medially.

Next question
Question 549 of 560

A 73 year old lady is admitted with right iliac fossa pain. A plain abdominal x-ray is taken and the
caecal diameter measured. Which of the following caecal diameters is pathological?

4cm

5cm

6cm

7cm

10cm

8 cm is still within normal limits. However, caecal diameters of 9 and 10 are pathological and should
prompt further investigation.
Please rate this question:

Discuss and give feedback


Next question

Right colon

Ileocaecal valve

 Entry point of the terminal ileum to the caecum


 An important colonoscopic landmark
 The ileocaecal valve is not always competent and this may allow partial decompression of an
obstructed colon

Appendix

 At the base of the caecum the taenia coalesce to mark the base of the appendix
 This is a reliable way of locating the appendix surgically and is a constant landmark
 The appendix has a small mesentery (the mesoappendix) and in this runs the appendiceal
artery, a branch of the ileocolic artery.
The posterior aspect of the right colon is extra peritoneal and the anterior aspect intraperitoneal.

Relations

 Posterior

Iliacus, Iliolumbar ligament, Quadratus lumborum, Transverse abdominis, Diaphragm at the tip of the
last rib; Lateral cutaneous, ilioinguinal, and iliohypogastric nerves; the iliac branches of the iliolumbar
vessels, the fourth lumbar artery, gonadal vessels, ureter and the right kidney.

 Superior

Right kidney which is embedded in the perinephric fat

 Medial

Mesentery which contains the ileocolic artery that supplies the right colon and terminal ileum. A
further branch , the right colic artery, also contributes to supply the hepatic flexure and proximal
transverse colon. Medially these pass through the mesentery to join the SMA. This occurs near to
the head of the pancreas and care has to be taken when ligating the ileocolic artery near to its origin
in cancer cases for fear of impinging on the SMA.

- Anterior
Coils of small intestine, the right edge of the greater omentum, and the anterior abdominal wall.

Nerve supply

 Parasympathetic fibres of the vagus nerve (CN X)

Arterial supply

 Ileocolic artery and right colic artery, both branches of the SMA. While the ileocolic artery is
almost always present, the right colic can be absent in 5-15% of individuals.

Next question
Question 550 of 560

Which of the following fingers is not a point of attachment for the palmar interossei?

Middle finger

Little finger

Ring finger

Index finger

None of the above

The middle finger has no attachment of the palmar interosseous.

Image sourced from Wikipedia

Please rate this question:


Discuss and give feedback
Next question

Hand

Anatomy of the hand


Bones  8 Carpal bones
 5 Metacarpals
 14 phalanges

Intrinsic Muscles 7 Interossei - Supplied by ulnar nerve

 3 palmar-adduct fingers
 4 dorsal- abduct fingers

Intrinsic muscles Lumbricals

 Flex MCPJ and extend the IPJ.


 Origin deep flexor tendon and insertion dorsal extensor hood
mechanism.
 Innervation: 1st and 2nd- median nerve, 3rd and 4th- deep branch of
the ulnar nerve.

Thenar eminence  Abductor pollicis brevis


 Opponens pollicis
 Flexor pollicis brevis

Hypothenar  Opponens digiti minimi


eminence  Flexor digiti minimi brevis
 Abductor digiti minimi
Image sourced from Wikipedia

Fascia and compartments of the palm


The fascia of the palm is continuous with the antebrachial fascia and the fascia of the dorsum of the
hand. The palmar fascia is thin over the thenar and hypothenar eminences. In contrast the palmar
fascia is relatively thick. The palmar aponeurosis covers the soft tissues and overlies the flexor
tendons. The apex of the palmar aponeurosis is continuous with the flexor retinaculum and the
palmaris longus tendon. Distally, it forms four longitudinal digital bands that attach to the bases of
the proximal phalanges, blending with the fibrous digital sheaths.
A medial fibrous septum extends deeply from the medial border of the palmar aponeurosis to the 5th
metacarpal. Lying medial to this are the hypothenar muscles. In a similar fashion, a lateral fibrous
septum extends deeply from the lateral border of the palmar aponeurosis to the 3rd metacarpal. The
thenar compartment lies lateral to this area.
Lying between the thenar and hypothenar compartments is the central compartment. It contains the
flexor tendons and their sheaths, the lumbricals, the superficial palmar arterial arch and the digital
vessels and nerves.
The deepest muscular plane is the adductor compartment, which contains adductor pollicis.

Short muscles of the hand


These comprise the lumbricals and interossei. The four slender lumbrical muscles flex the fingers at
the metacarpophalangeal joints and extend the interphalangeal joint. The four dorsal interossei are
located between the metacarpals and the four palmar interossei lie on the palmar surface of the
metacarpals in the interosseous compartment of the hand.

Long flexor tendons and sheaths in the hand


The tendons of FDS and FDP enter the common flexor sheath deep to the flexor retinaculum. The
tendons enter the central compartment of the hand and fan out to their respective digital synovial
sheaths. Near the base of the proximal phalanx, the tendon of FDS splits to permit the passage of
FDP. The FDP tendons are attached to the margins of the anterior aspect of the base of the distal
phalanx.
The fibrous digital sheaths contain the flexor tendons and their synovial sheaths. These extend from
the heads of the metacarpals to the base of the distal phalanges.
Next question
Question 551 of 560

A 33 year old man sustains an injury to his forearm and wrist. When examined in clinic he is unable
to adduct his thumb. What is the most likely underlying nerve lesion?

Radial nerve

Superficial branch of the ulnar nerve

Median nerve

Posterior interosseous nerve

Deep branch of the ulnar nerve

Theme from April 2013 Exam


Theme from April 2014 Exam
Damage to the deep branch of the ulnar nerve may result in an inability to adduct the thumb. This is
tested clinically by trying to withdraw a piece of paper from a patients hand grasped between thumb
and index finger.

Please rate this question:

Discuss and give feedback

Next question

Adductor pollicis

Nerve
Origin Insertion supply Actions
Nerve
Origin Insertion supply Actions

Tendon sheath of flexor Fibres of the two heads converge Deep branch Adducts the thumb
carpi radialis on insertion into the ulnar aspect of the ulnar into the plane of the
Bases of second, third and of the base of the proximal (C8, T1) palm and draws it to
fourth metacarpals phalanx of the thumb the midline
Anterior aspect of the
trapezoid and capitate
bones
Transverse head comes
from the longitudinal ride
of the third metacarpal

Next question
Question 552 of 560

A 6 year old sustains a supracondylar fracture of the distal humerus. There are concerns that the
radial nerve may have been injured. What is the relationship of the radial nerve to the humerus at
this point?

Anterolateral

Anteromedial

Posterolateral

Posteromedial

Immediately anterior

The radial nerve lies anterolateral to the humerus in the supracondylar area.
Please rate this question:

Discuss and give feedback


Next question

Radial nerve

Continuation of posterior cord of the brachial plexus (root values C5 to T1)

Path

 In the axilla: lies posterior to the axillary artery on subscapularis, latissimus dorsi and teres
major.
 Enters the arm between the brachial artery and the long head of triceps (medial to humerus).
 Spirals around the posterior surface of the humerus in the groove for the radial nerve.
 At the distal third of the lateral border of the humerus it then pierces the intermuscular
septum and descends in front of the lateral epicondyle.
 At the lateral epicondyle it lies deeply between brachialis and brachioradialis where it then
divides into a superficial and deep terminal branch.
 Deep branch crosses the supinator to become the posterior interosseous nerve.

In the image below the relationships of the radial nerve can be appreciated
Image sourced from Wikipedia

Regions innervated
 Triceps
 Anconeus
Motor (main nerve)  Brachioradialis
 Extensor carpi radialis

 Supinator
 Extensor carpi ulnaris
 Extensor digitorum
Motor (posterior  Extensor indicis
interosseous branch)  Extensor digiti minimi
 Extensor pollicis longus and brevis
 Abductor pollicis longus

The area of skin supplying the proximal phalanges on the dorsal aspect of the
Sensory hand is supplied by the radial nerve (this does not apply to the little finger and
part of the ring finger)

Muscular innervation and effect of denervation


Anatomical
location Muscle affected Effect of paralysis
Anatomical
location Muscle affected Effect of paralysis

Shoulder Long head of triceps Minor effects on shoulder stability in abduction

Arm Triceps Loss of elbow extension

Forearm Supinator Weakening of supination of prone hand and


Brachioradialis elbow flexion in mid prone position
Extensor carpi radialis
longus and brevis

The cutaneous sensation of the upper limb- illustrating the contribution of the radial nerve

Image sourced from Wikipedia

Next question
Question 553 of 560

Which of the following muscles is penetrated by the parotid duct?

Medial pterygoid

Buccinator

Levator anguli oris

Temporalis

Masseter

The duct crosses the masseter muscle and buccal fat pad and then penetrates the buccinator
muscle to enter the oral cavity opposite the second upper molar tooth.
Please rate this question:

Discuss and give feedback


Next question

Parotid gland

Anatomy of the parotid gland


Location Overlying the mandibular ramus; anterior and inferior to the ear.

Salivary duct Crosses the masseter, pierces the buccinator and drains adjacent to the
2nd upper molar tooth (Stensen's duct).

Structures passing  Facial nerve (Mnemonic: The Zebra Buggered My Cat; Temporal
through the gland Zygomatic, Buccal, Mandibular, Cervical)
 External carotid artery
 Retromandibular vein
 Auriculotemporal nerve
Relations  Anterior: masseter, medial pterygoid, superficial temporal and
maxillary artery, facial nerve, stylomandibular ligament
 Posterior: posterior belly digastric muscle, sternocleidomastoid,
stylohyoid, internal carotid artery, mastoid process, styloid
process

Arterial supply Branches of external carotid artery

Venous drainage Retromandibular vein

Lymphatic drainage Deep cervical nodes

Nerve innervation  Parasympathetic-Secretomotor


 Sympathetic-Superior cervical ganglion
 Sensory- Greater auricular nerve

Parasympathetic stimulation produces a water rich, serous saliva. Sympathetic stimulation leads to
the production of a low volume, enzyme-rich saliva.
Next question
Question 554 of 560

The following are true of the ulnar nerve except:

It innervates the palmar interossei

Derived from the medial cord of the brachial plexus

Supplies the muscles of the thenar eminence

Supplies the medial half of flexor digitorum profundus

Passes superficial to the flexor retinaculum

The muscles of the thenar eminence are supplied by the median nerve and atrophy of these is a
feature of carpal tunnel syndrome.
Please rate this question:

Discuss and give feedback


Next question

Ulnar nerve

Origin

 C8, T1

Supplies (no muscles in the upper arm)

 Flexor carpi ulnaris


 Flexor digitorum profundus
 Flexor digiti minimi
 Abductor digiti minimi
 Opponens digiti minimi
 Adductor pollicis
 Interossei muscle
 Third and fourth lumbricals
 Palmaris brevis
Path

 Posteromedial aspect of upper arm to flexor compartment of forearm, then along the ulnar.
Passes beneath the flexor carpi ulnaris muscle, then superficially through the flexor
retinaculum into the palm of the hand.

Image sourced from Wikipedia

Branches
Branch Supplies

Muscular branch Flexor carpi ulnaris


Medial half of the flexor digitorum profundus
Branch Supplies

Palmar cutaneous branch (Arises near the Skin on the medial part of the palm
middle of the forearm)

Dorsal cutaneous branch Dorsal surface of the medial part of the hand

Superficial branch Cutaneous fibres to the anterior surfaces of the


medial one and one-half digits

Deep branch Hypothenar muscles


All the interosseous muscles
Third and fourth lumbricals
Adductor pollicis
Medial head of the flexor pollicis brevis

Effects of injury
Damage at the wrist  Wasting and paralysis of intrinsic hand muscles (claw hand)
 Wasting and paralysis of hypothenar muscles
 Loss of sensation medial 1 and half fingers

Damage at the elbow  Radial deviation of the wrist


 Clawing less in 4th and 5th digits

Next question
Question 555 of 560

How many valves lie between the superior vena cava and the right atrium?

None

One

Two

Three

Four

There are no valves which is why it is relatively easy to insert a CVP line from the internal jugular
vein into the right atrium.
Please rate this question:

Discuss and give feedback


Next question

Superior vena cava

Drainage

 Head and neck


 Upper limbs
 Thorax
 Part of abdominal walls

Formation

 Subclavian and internal jugular veins unite to form the right and left brachiocephalic veins
 These unite to form the SVC
 Azygos vein joins the SVC before it enters the right atrium

Relations
Anterior Anterior margins of the right lung and pleura

Posteromedial Trachea and right vagus nerve

Posterolateral Posterior aspects of right lung and pleura


Pulmonary hilum is posterior

Right lateral Right phrenic nerve and pleura

Left lateral Brachiocephalic artery and ascending aorta

Developmental variations
Anomalies of the connection of the SVC are recognised. In some individuals a persistent left sided
SVC drains into the right atrium via an enlarged orifice of the coronary sinus. More rarely the left
sided vena cava may connect directly with the superior aspect of the left atrium, usually associated
with an un-roofing of the coronary sinus. The commonest lesion of the IVC is for its abdominal
course to be interrupted, with drainage achieved via the azygos venous system. This may occur in
patients with left sided atrial isomerism.
Next question
Question 556 of 560

Which of the following options in relation to the liver is true?

Ligamentum venosum is an anterior relation of the liver

The portal triad comprises the hepatic artery, hepatic vein and tributary of the bile duct

The liver is completely covered by peritoneum

There are no nerves within the porta hepatis

The caudate lobe is superior to the porta hepatis

'VC goes with VC'

The ligamentun Venosum and Caudate is on same side as Vena Cava [posterior].

Ligamentum venosum is posterior to the liver. The portal triad contains the portal vein rather than the
hepatic vein. There is the 'bare area of the liver' created by a void due to the coronary ligament
layers being widely separated. There are sympathetic and parasympathetic nerves in the porta
hepatis.
Please rate this question:

Discuss and give feedback


Next question

Liver

Structure of the liver


Right lobe  Supplied by right hepatic artery
 Contains Couinaud segments V to VIII (-/+Sg I)

Left lobe  Supplied by the left hepatic artery


 Contains Couinaud segments II to IV (+/- Sg1)

Quadrate lobe  Part of the right lobe anatomically, functionally is part of the left
 Couinaud segment IV
 Porta hepatis lies behind
 On the right lies the gallbladder fossa
 On the left lies the fossa for the umbilical vein

Caudate lobe  Supplied by both right and left hepatic arteries


 Couinaud segment I
 Lies behind the plane of the porta hepatis
 Anterior and lateral to the inferior vena cava
 Bile from the caudate lobe drains into both right and left hepatic ducts

Detailed knowledge of Couinaud segments is not required for MRCS

 Between the liver lobules are portal canals which contain the portal triad: Hepatic Artery,
Portal Vein, tributary of Bile Duct.

Relations of the liver


Anterior Postero inferiorly

Diaphragm Oesophagus

Xiphoid process Stomach

Duodenum

Hepatic flexure of colon

Right kidney

Gallbladder

Inferior vena cava


Porta hepatis
Location Postero inferior surface, it joins nearly at right angles with the left sagittal fossa, and
separates the caudate lobe behind from the quadrate lobe in front

Transmits  Common hepatic duct


 Hepatic artery
 Portal vein
 Sympathetic and parasympathetic nerve fibres
 Lymphatic drainage of the liver (and nodes)

Ligaments
Falciform ligament  2 layer fold peritoneum from the umbilicus to anterior liver surface
 Contains ligamentum teres (remnant umbilical vein)
 On superior liver surface it splits into the coronary and left
triangular ligaments

Ligamentum teres Joins the left branch of the portal vein in the porta hepatis

Ligamentum Remnant of ductus venosus


venosum

Arterial supply

 Hepatic artery

Venous

 Hepatic veins
 Portal vein

Nervous supply

 Sympathetic and parasympathetic trunks of coeliac plexus

Next question
Question 557 of 560

Which of the following structures does not pass anterior to the lateral malleolus?

Anterior tibial artery

Extensor digitorum longus

Tibialis anterior

Peroneus brevis

Peroneus tertius

Peroneus brevis passes posterior to the lateral malleolus.


Please rate this question:

Discuss and give feedback


Next question

Lateral malleolus

Structures posterior to the lateral malleolus and superficial to superior peroneal retinaculum

 Sural nerve
 Short saphenous vein

Structures posterior to the lateral malleolus and deep to superior peroneal retinaculum

 Peroneus longus tendon


 Peroneus brevis tendon

The calcaneofibular ligament is attached at the lateral malleolus


Next question
Question 558 of 560

The following statements regarding the rectus abdominis muscle are true except:

It runs from the symphysis pubis to the xiphoid process

Its nerve supply is from the ventral rami of the lower 6 thoracic nerves

It has collateral supply from both superior and inferior epigastric vessels

It lies in a muscular aponeurosis throughout its length

It has a number of tendinous intersections that penetrate through the anterior layer of the
muscle

Rectus abdominis

 Arises from the pubis.


 Inserts into 5th, 6th, 7th costal cartilages.
 The muscle lies in the rectal sheath, which also contains the superior and inferior epigastric
artery and vein.
 Action: flexion of thoracic and lumbar spine.
 Nerve supply: anterior primary rami of T7-12.

The aponeurosis is deficient below the arcuate line.


Please rate this question:

Discuss and give feedback


Next question

Abdominal wall

The 2 main muscles of the abdominal wall are the rectus abdominis (anterior) and the quadratus
lumborum (posterior).
The remaining abdominal wall consists of 3 muscular layers. Each muscle passes from the lateral
aspect of the quadratus lumborum posteriorly to the lateral margin of the rectus sheath anteriorly.
Each layer is muscular posterolaterally and aponeurotic anteriorly.
Image sourced from Wikipedia

Muscles of abdominal wall


External  Lies most superficially
oblique  Originates from 5th to 12th ribs
 Inserts into the anterior half of the outer aspect of the iliac crest, linea
alba and pubic tubercle
 More medially and superiorly to the arcuate line, the aponeurotic layer
overlaps the rectus abdominis muscle
 The lower border forms the inguinal ligament
 The triangular expansion of the medial end of the inguinal ligament is
the lacunar ligament.

Internal  Arises from the thoracolumbar fascia, the anterior 2/3 of the iliac crest
oblique and the lateral 2/3 of the inguinal ligament
 The muscle sweeps upwards to insert into the cartilages of the lower 3
ribs
 The lower fibres form an aponeurosis that runs from the tenth costal
cartilage to the body of the pubis
 At its lowermost aspect it joins the fibres of the aponeurosis of
transversus abdominis to form the conjoint tendon.

Transversus  Innermost muscle


abdominis  Arises from the inner aspect of the costal cartilages of the lower 6 ribs ,
from the anterior 2/3 of the iliac crest and lateral 1/3 of the inguinal
ligament
 Its fibres run horizontally around the abdominal wall ending in an
aponeurosis. The upper part runs posterior to the rectus abdominis.
Lower down the fibres run anteriorly only.
 The rectus abdominis lies medially; running from the pubic crest and
symphysis to insert into the xiphoid process and 5th, 6th and 7th costal
cartilages. The muscles lies in a aponeurosis as described above.
 Nerve supply: anterior primary rami of T7-12

Surgical notes
During abdominal surgery it is usually necessary to divide either the muscles or their aponeuroses.
During a midline laparotomy it is desirable to divide the aponeurosis. This will leave the rectus
sheath intact above the arcuate line and the muscles intact below it. Straying off the midline will
often lead to damage to the rectus muscles, particularly below the arcuate line where they may often
be in close proximity to each other.
Next question
Question 559 of 560

Which of the following statements relating to sternocleidomastoid is untrue?

The external jugular vein lies posteromedially.

It is supplied by the accessory nerve.

It has two heads of origin

It inserts into the lateral aspect of the mastoid process.

It marks the anterior border of the posterior triangle.

The external jugular vein lies lateral (i.e. superficial) to the sternocleidomastoid.
Please rate this question:

Discuss and give feedback


Next question

Sternocleidomastoid

Anatomy
Origin Rounded tendon attached to upper manubrium sterni and muscular head attached to
medial third of the clavicle

Insertion Mastoid process of the temporal bone and lateral area of the superior nuchal line of
the occipital bone

Innervation Spinal part of accessory nerve and anterior rami of C2 and C3 (proprioception)

Action  Both: extend the head at atlanto-occipital joint and flex the cervical
vertebral column. Accessory muscles of inspiration.
 Single: lateral flexion of neck, rotates head so face looks upward to the
opposite side

Sternocleidomastoid divides the anterior and posterior triangles of the neck.


Next question
Question 560 of 560

During liver mobilisation for a cadaveric liver transplant the hepatic ligaments will require
mobilisation. Which of the following statements relating to these structures is untrue?

Lesser omentum arises from the porta hepatis and passes the lesser curvature of the
stomach

The falciform ligament divides into the left triangular ligament and coronary ligament

The liver has an area devoid of peritoneum

The coronary ligament is attached to the liver

The right triangular ligament is an early branch of the left triangular ligament

The right triangular ligament is a continuation of the coronary ligament.


Please rate this question:

Discuss and give feedback

Liver

Structure of the liver


Right lobe  Supplied by right hepatic artery
 Contains Couinaud segments V to VIII (-/+Sg I)

Left lobe  Supplied by the left hepatic artery


 Contains Couinaud segments II to IV (+/- Sg1)

Quadrate lobe  Part of the right lobe anatomically, functionally is part of the left
 Couinaud segment IV
 Porta hepatis lies behind
 On the right lies the gallbladder fossa
 On the left lies the fossa for the umbilical vein
Caudate lobe  Supplied by both right and left hepatic arteries
 Couinaud segment I
 Lies behind the plane of the porta hepatis
 Anterior and lateral to the inferior vena cava
 Bile from the caudate lobe drains into both right and left hepatic ducts

Detailed knowledge of Couinaud segments is not required for MRCS

 Between the liver lobules are portal canals which contain the portal triad: Hepatic Artery,
Portal Vein, tributary of Bile Duct.

Relations of the liver


Anterior Postero inferiorly

Diaphragm Oesophagus

Xiphoid process Stomach

Duodenum

Hepatic flexure of colon

Right kidney

Gallbladder

Inferior vena cava

Porta hepatis
Location Postero inferior surface, it joins nearly at right angles with the left sagittal fossa, and
separates the caudate lobe behind from the quadrate lobe in front
Transmits  Common hepatic duct
 Hepatic artery
 Portal vein
 Sympathetic and parasympathetic nerve fibres
 Lymphatic drainage of the liver (and nodes)

Ligaments
Falciform ligament  2 layer fold peritoneum from the umbilicus to anterior liver surface
 Contains ligamentum teres (remnant umbilical vein)
 On superior liver surface it splits into the coronary and left
triangular ligaments

Ligamentum teres Joins the left branch of the portal vein in the porta hepatis

Ligamentum Remnant of ductus venosus


venosum

Arterial supply

 Hepatic artery

Venous

 Hepatic veins
 Portal vein

Nervous supply

 Sympathetic and parasympathetic trunks of coeliac plexus


Theme: Skin disease

A. Squamous cell carcinoma


B. Bowens disease
C. Actinic keratosis
D. Basal cell carcinoma
E. Malignant melanoma
F. Keratoacanthoma
G. Apthous ulcer
H. Pyogenic granuloma

Please select the most likely underlying diagnosis for each of the following skin lesions. Each option
may be used once, more than once or not at all.

1. A 53 year old man presents with a nodule on his chin. He is concerned because it has grown
extremely rapidly over the course of the preceding week. On examination he has a swollen,
red, dome shaped lesion with a central defect that contains a keratinous type material.

You answered Squamous cell carcinoma

The correct answer is Keratoacanthoma

Theme from April 2013 Exam


Keratoacanthomas are characterised by a rapid growth phase. This may mimic amelanotic
melanoma (although such rapid growth is rare even in these lesions). The keratin core is the
clue as to the true nature of the lesion.

2. A 68 year old farmer presents with a skin lesion on his forehead. It has been present for the
past 6 months and has grown slightly in size during that time. On examination he has an
ulcerated lesion with pearly white raised edges that measures 2cm in diameter.

You answered Squamous cell carcinoma

The correct answer is Basal cell carcinoma

The raised pearly edges in an ulcerated lesion at a sun exposed site makes BCC most likely.

3. A 34 year old gardener presents with a lesion affecting the dorsum of his right hand. It has
been present for the past 10 days and occurred after he had been pruning rose bushes. On
examination he has a raised ulcerated lesion which bleeds easily on contact.

You answered Squamous cell carcinoma


The correct answer is Pyogenic granuloma

Trauma is a common precipitant of pyogenic granuloma and contact bleeding and ulceration
are common.

Please rate this question:

Discuss and give feedback


Next question

Skin disorders in surgery- malignancy and related lesions

Non melanoma skin cancer (BCC and SCC) are some of the commonest types of human
malignancy. Up to 80% of these are BCC's with approximately 20% comprising SCC's. The
incidence of NMSC's increases with age and whilst there is a female preponderance in those under
40 years of age, in latter life the sex incidence is roughly equal.
The vast majority of NMSC's are related to UV light exposure. For SCC's the major pattern is is
chronic long term exposure. For BCC's, the pattern of sporadic exposure with episodes of burning is
more important. Organ transplant recipients have a markedly increased incidence of SCC, risk
factors include length of immunosuppression, ethnic origin and associated sunlight exposure.
Human papilloma virus DNA is found in the majority of transplant recipient SCC's. In addition to this
increased risk, transplant recipients are also more likely to develop locoregional recurrences
following treatment.

Actinic keratosis and SCC


Actinic keratosis is viewed as a premalignant lesion because there are atypical keratinocytes present
in the epidermis. In a person with 7 actinic keratosis the risks of subsequent SCC is of the order of
10% at 10 years. The primary lesion is a rough erythematous papule with a white to yellow scale.
Lesions are typically clustered at sites of chronic sun exposure.

Squamous cell carcinoma in situ


Also known as Bowens disease the commonest presentation of in situ SCC is with an erythematous
scaling patch or elevated plaque arising on sun exposed skin in an elderly patient. Lesions may
arisede novo or from pre-existing actinic keratosis.
Pathologically there is full thickness atypia of dermal keratinocytes over a broad zone. Nuclear
pleomorphism, apoptosis and abnormal mitoses are all seen.

Invasive SCC
The commonest clinical presentation of SCC is with an erythematous keratotic papule or nodule on a
background of sun exposure. Ulceration may occur and both exophytic and endophytic areas may
be seen. Regional lymphadenopathy may be present.
Pathologically there is downward proliferation of malignant cells and invasion of the basement
membrane. Poorly differentiated lesions may show perineural invasion and require
immunohistochemistry with S100 to distinguish them from melanomas (which stain strongly positive
with this marker).

Basal cell carcinoma


Nodular BCC Commonest variant (60%)
Raised translucent papule
Usually affect the face
Large nodular BCC's are locally destructive

Superficial BCC Usually appears as superficial erythematous macule affecting the trunk
Younger age at presentation (mean 57)
May show areas of spontaneous regression
Horizontal growth pattern predominates
High recurrence rate (due to sub clinical lateral spread)

Morpheaform BCC Macroscopically resembles flat, slightly atrophic lesion or plaque


without well defined borders
Tumour has sub clinical lateral spread which increases recurrence rates

Cystic BCC Often have clear or blue - grey appearance


Cystic degeneration may not be clinically obvious and tumour may
resemble nodular BCC

Basosquamous Atypical BCC


carcinoma Basaloid histological BCC features with eosinophillic squamoid
features of SCC
Biologically more aggressive and are more locally destructive
Rare lesion accounts for 1% of all non melanoma skin cancers
Metastatic disease may occur in 9-10% of cases and resemble an SCC

Keratoacanthoma
Dome shaped erythematous lesions that develop over a period of days and grow rapidly. They often
contain a central pit of keratin. They then begin to necrose and slough off. They are generally benign
lesions although some do view them as precursors of malignancy. They may be treated by curettage
and cautery. If there is diagnostic doubt (they can mimic malignancy) then formal excision biopsy is
warranted.

Pyogenic granuloma
These present as friable overgrowths of granulation at sites of minor trauma. They may be ulcerated
and bleeding on contact is common. They may be treated with curretage and cautery, formal
excision may be used if there is diagnostic doubt.
Next question
Theme: Skin disorders

A. Basal cell carcinoma

B. Dermatofibroma

C. Pilar cyst

D. Epidermoid cyst

E. Spitz naevus

F. Seborrhoeic keratosis

G. Atypical naevus

H. Capillary cavernous haemangioma

Please select the most likely underlying nature of the skin lesion described. Each option may be
used once, more than once or not at all.

4. A 70 year old lady presents with a number of skin lesions that she describes as unsightly. On
examination she has a number of raised lesions with a greasy surface located over her trunk. Apart
from having a greasy surface the lesions also seem to have scattered keratin plugs located within
them.

You answered Basal cell carcinoma

The correct answer is Seborrhoeic keratosis

Theme from September 2012 Exam


Seborrhoeic keratosis may have a number of appearances. However, the scaly, thick, greasy
surface with scattered keratin plugs makes this the most likely diagnosis.

5. A 28 year old female presents with a small nodule located on the back of her neck. It is excised for
cosmetic reasons and the histology report states that the lesion consists of a sebum filled lesion
surrounded by the outer root sheath of a hair follicle.

You answered Basal cell carcinoma


The correct answer is Pilar cyst

Pilar cysts may contain foul smelling cheesy material and are surrounded by the outer part of a hair
follicle. Because of their histological appearances they are more correctly termed pilar cysts than
sebaceous cysts.

6. A 21 year old lady presents with a nodule on the posterior aspect of her right calf. It has been
present at the site for the past 6 months and occurred at the site of a previous insect bite.
Although the nodule appears small, on palpation it appears to be nearly twice the size it appears
on examination. The overlying skin is faintly pigmented.

You answered Basal cell carcinoma

The correct answer is Dermatofibroma

Dermatofibromas may be pigmented and are often larger than they appear. They frequently occur
at sites of previous trauma.

Please rate this question:

Discuss and give feedback

Next question

Benign skin diseases

Seborrhoeic keratosis

 Most commonly arise in patients over the age of 50 years, often idiopathic
 Equal sex incidence and prevalence
 Usually multiple lesions over face and trunk
 Flat, raised, filiform and pedunculated subtypes are recognised
 Variable colours and surface may have greasy scale overlying it
 Treatment options consist of leaving alone or simple shave excision
Melanocytic naevi

Congenital  Typically appear at, or soon after, birth


melanocytic naevi  Usually greater than 1cm diameter
 Increased risk of malignant transformation (increased risk greatest for
large lesions)

Junctional  Circular macules


melanocytic naevi  May have heterogeneous colour even within same lesion
 Most naevi of the palms, soles and mucous membranes are of this type

Compound naevi  Domed pigmented nodules up to 1cm in diameter


 Arise from junctional naevi, usually have uniform colour and are smooth

Spitz naevus  Usually develop over a few months in children


 May be pink or red in colour, most common on face and legs
 May grow up to 1cm and growth can be rapid, this usually results in
excision

Atypical naevus  Atypical melanocytic naevi that may be autosomally dominantly


syndrome inherited
 Some individuals are at increased risk of melanoma (usually have
mutations of CDKN2A gene)
 Many people with atypical naevus syndrome AND a parent sibling with
melanoma will develop melanoma

Epidermoid cysts

 Common and affect face and trunk


 They have a central punctum, they may contain small quantities of sebum
 The cyst lining is either normal epidermis (epidermoid cyst) or outer root sheath of hair
follicle (pilar cyst)

Dermatofibroma

 Solitary dermal nodules


 Usually affect extremities of young adults
 Lesions feel larger than they appear visually
 Histologically they consist of proliferating fibroblasts merging with sparsely cellular dermal
tissues

Painful skin lesions

 Eccrine spiradenoma
 Neuroma
 Glomus tumour
 Leimyoma
 Angiolipoma
 Neurofibroma (rarely painful) and dermatofibroma (rarely painful)
Theme: Skin lesion diagnosis

A. Pyogenic granuloma
B. Amelanotic melanoma
C. Dermatitis herpetiformis
D. Scabies
E. Basal cell carcinoma
F. Squamous cell carcinoma
G. Keratoacanthoma

Please select the most likely underlying diagnosis for the scenario given. Each option may be used
once, more than once or not at all.

7. A 72 year old man presents with a large nodule on his face. It is friable. There is no regional
lymphadenopathy. He is lost to follow up and re-attends several months later. On this
occasion the lesion has been noted to resolve with scarring.

You answered Pyogenic granuloma

The correct answer is Keratoacanthoma

Keratoacanthomas may reach a considerable size prior to sloughing off and scarring.

8. A 22 year old girl is troubled by intensely itchy crops of blisters on her arms and legs. On
examination she is malnourished and she has papulovesicular eruptions over her elbows and
knees.

You answered Pyogenic granuloma

The correct answer is Dermatitis herpetiformis

Dermatitis herpetiformis is seen in association with coeliac disease.

9. A 30 year old man cuts the corner of his lip whilst shaving. Over the next few days a large
purplish lesion appears at the site which bleeds on contact.

Pyogenic granuloma

Pyogenic granulomas often appear at sites of trauma.

Please rate this question:


Discuss and give feedback
Next question

Skin Diseases

Skin lesions may be referred for surgical assessment, but more commonly will come via a
dermatologist for definitive surgical management.

Skin malignancies include basal cell carcinoma, squamous cell carcinoma and malignant melanoma.

Basal Cell Carcinoma

 Most common form of skin cancer.


 Commonly occur on sun exposed sites apart from the ear.
 Sub types include nodular, morphoeic, superficial and pigmented.
 Typically slow growing with low metastatic potential.
 Standard surgical excision, topical chemotherapy and radiotherapy are all successful.
 As a minimum a diagnostic punch biopsy should be taken if treatment other than standard
surgical excision is planned.

Squamous Cell Carcinoma

 Again related to sun exposure.


 May arise in pre - existing solar keratoses.
 May metastasize if left.
 Immunosupression (e.g. following transplant), increases risk.
 Wide local excision is the treatment of choice and where a diagnostic excision biopsy has
demonstrated SCC, repeat surgery to gain adequate margins may be required.

Malignant Melanoma
The main diagnostic features (major criteria): Secondary features (minor criteria)

 Change in size  Diameter >6mm


 Change in shape  Inflammation
 Change in colour  Oozing or bleeding
 Altered sensation

Treatment

 Suspicious lesions should undergo excision biopsy. The lesion should be removed in
completely as incision biopsy can make subsequent histopathological assessment difficult.
 Once the diagnosis is confirmed the pathology report should be reviewed to determine
whether further re-excision of margins is required (see below):

Margins of excision-Related to Breslow thickness


Lesions 0-1mm thick 1cm

Lesions 1-2mm thick 1- 2cm (Depending upon site and pathological features)

Lesions 2-4mm thick 2-3 cm (Depending upon site and pathological features)

Lesions >4 mm thick 3cm

Marsden J et al. Revised UK guidelines for management of Melanoma. Br J Dermatol 2010 163:238-
256.

Further treatments such as sentinel lymph node mapping, isolated limb perfusion and block
dissection of regional lymph node groups should be selectively applied.

Kaposi Sarcoma

 Tumour of vascular and lymphatic endothelium.


 Purple cutaneous nodules.
 Associated with immuno supression.
 Classical form affects elderly males and is slow growing.
 Immunosupression form is much more aggressive and tends to affect those with HIV related
disease.

Non malignant skin disease

Dermatitis Herpetiformis

 Chronic itchy clusters of blisters.


 Linked to underlying gluten enteropathy (coeliac disease).

Dermatofibroma

 Benign lesion.
 Firm elevated nodules.
 Usually history of trauma.
 Lesion consists of histiocytes, blood vessels and fibrotic changes.
Pyogenic granuloma

 Overgrowth of blood vessels.


 Red nodules.
 Usually follow trauma.
 May mimic amelanotic melanoma.

Acanthosis nigricans

 Brown to black, poorly defined, velvety hyperpigmentation of the skin.


 Usually found in body folds such as the posterior and lateral folds of the neck, the axilla,
groin, umbilicus, forehead, and other areas.
 The most common cause of acanthosis nigricans is insulin resistance, which leads to
increased circulating insulin levels. Insulin spillover into the skin results in its abnormal
increase in growth (hyperplasia of the skin).
 In the context of a malignant disease, acanthosis nigricans is a paraneoplastic syndrome and
is then commonly referred to as acanthosis nigricans maligna. Involvement of mucous
membranes is rare and suggests a coexisting malignant condition
A 22 year old man presents with an infected sebaceous cyst. The cyst itself is swollen, discharging
pus and has some surrounding erythema. What is the most appropriate treatment?

Excision of the cyst of closure of the defect with interrupted 3/0 silk

Excision of the cyst and closure of the defect with subcuticular 4/0 undyed nylon

Incision and drainage with excision of the cyst wall and packing of the defect

Incision and drainage with conservation of the cyst wall and packing of the defect

Administration of oral co-amoxyclav and definitive surgery once the infection has cleared

Similar theme in January 2013 Exam


The correct treatment for an infected sebaceous cyst is incision and drainage with removal of the
cyst wall. Conservation of the cyst wall will invariably lead to recurrence. Under no circumstances
should an infected wound like this be primarily closed. The administration of antibiotics without
drainage of sepsis is futile.
Please rate this question:

Discuss and give feedback


Next question

Sebaceous cysts

 Originate from sebaceous glands and contain sebum.


 Location: anywhere but most common scalp, ears, back, face, and upper arm (not palms of
the hands and soles of the feet).
 They will typically contain a punctum.
 Excision of the cyst wall needs to be complete to prevent recurrence.
 A Cock's 'Peculiar' Tumour is a suppurating and ulcerated sebaceous cyst. It may resemble
a squamous cell carcinoma- hence its name.
Theme: Management of skin lesions

A. Excision biopsy
B. Excision with 0.5 cm margin
C. Excision with 2 cm margin
D. Shave biopsy and cautery
E. Punch biopsy
F. Excision and full thickness skin graft
G. Discharge

For each skin lesion please select the most appropriate management option. Each option may be
used once, more than once, or not at all.

11. A 22 year old women presents with a newly pigmented lesion on her right shin, it has
regular borders and normal appearing dermal appendages. However, she reports a recent
increase in size.

Excision biopsy

Lesion bearing normal dermal appendages and regular borders are likely to be a benign
pigmented naevi. Therefore diagnostic and not radical excision is indicated.

12. A 58 year old lady presents with changes that are suspicious of lichen sclerosis of the
perineum.

You answered Excision biopsy

The correct answer is Punch biopsy

Punch biopsies are a useful option for obtaining a full thickness tissues sample with
minimal tissue disruption. In this situation the other differential would be AIN or VIN and
punch biopsies would be useful in distinguishing these.

13. A 73 year old man presents with a 1.5cm ulcerated basal cell carcinoma on his back.

You answered Excision biopsy

The correct answer is Excision with 0.5 cm margin

A small lesion such as this is adequately treated by local excision. The British Association
of Dermatology guidelines suggest that excision of conventional BCC (<2cm) with
margins of 3-5mm have locoregional control rates of 85%. Morpoeic lesions have higher
local recurrence rates.
Please rate this question:

Discuss and give feedback


Next question

Skin Diseases

Skin lesions may be referred for surgical assessment, but more commonly will come via a
dermatologist for definitive surgical management.

Skin malignancies include basal cell carcinoma, squamous cell carcinoma and malignant melanoma.

Basal Cell Carcinoma

 Most common form of skin cancer.


 Commonly occur on sun exposed sites apart from the ear.
 Sub types include nodular, morphoeic, superficial and pigmented.
 Typically slow growing with low metastatic potential.
 Standard surgical excision, topical chemotherapy and radiotherapy are all successful.
 As a minimum a diagnostic punch biopsy should be taken if treatment other than standard
surgical excision is planned.

Squamous Cell Carcinoma

 Again related to sun exposure.


 May arise in pre - existing solar keratoses.
 May metastasize if left.
 Immunosupression (e.g. following transplant), increases risk.
 Wide local excision is the treatment of choice and where a diagnostic excision biopsy has
demonstrated SCC, repeat surgery to gain adequate margins may be required.

Malignant Melanoma
The main diagnostic features (major criteria): Secondary features (minor criteria)

 Change in size  Diameter >6mm


 Change in shape  Inflammation
 Change in colour  Oozing or bleeding
 Altered sensation

Treatment
 Suspicious lesions should undergo excision biopsy. The lesion should be removed in
completely as incision biopsy can make subsequent histopathological assessment difficult.
 Once the diagnosis is confirmed the pathology report should be reviewed to determine
whether further re-excision of margins is required (see below):

Margins of excision-Related to Breslow thickness


Lesions 0-1mm thick 1cm

Lesions 1-2mm thick 1- 2cm (Depending upon site and pathological features)

Lesions 2-4mm thick 2-3 cm (Depending upon site and pathological features)

Lesions >4 mm thick 3cm

Marsden J et al. Revised UK guidelines for management of Melanoma. Br J Dermatol 2010 163:238-
256.

Further treatments such as sentinel lymph node mapping, isolated limb perfusion and block
dissection of regional lymph node groups should be selectively applied.

Kaposi Sarcoma

 Tumour of vascular and lymphatic endothelium.


 Purple cutaneous nodules.
 Associated with immuno supression.
 Classical form affects elderly males and is slow growing.
 Immunosupression form is much more aggressive and tends to affect those with HIV related
disease.

Non malignant skin disease

Dermatitis Herpetiformis

 Chronic itchy clusters of blisters.


 Linked to underlying gluten enteropathy (coeliac disease).

Dermatofibroma

 Benign lesion.
 Firm elevated nodules.
 Usually history of trauma.
 Lesion consists of histiocytes, blood vessels and fibrotic changes.

Pyogenic granuloma

 Overgrowth of blood vessels.


 Red nodules.
 Usually follow trauma.
 May mimic amelanotic melanoma.

Acanthosis nigricans

 Brown to black, poorly defined, velvety hyperpigmentation of the skin.


 Usually found in body folds such as the posterior and lateral folds of the neck, the axilla,
groin, umbilicus, forehead, and other areas.
 The most common cause of acanthosis nigricans is insulin resistance, which leads to
increased circulating insulin levels. Insulin spillover into the skin results in its abnormal
increase in growth (hyperplasia of the skin).
 In the context of a malignant disease, acanthosis nigricans is a paraneoplastic syndrome and
is then commonly referred to as acanthosis nigricans maligna. Involvement of mucous
membranes is rare and suggests a coexisting malignant condition.
Theme: Dermatological manifestations of disease

A. Pyoderma gangrenosum
B. Erythroderma
C. Dermatitis herpetiformis
D. Acanthosis nigricans
E. Multiple lipomata
F. Multiple neurofibromata
G. Multiple telangectasia
H. None of the above

Please select the skin disease associated with the condition described. Each option may be used
once, more than once or not at all.

14. A 22 year old man is investigated for weight loss. A duodenal biopsy taken as part of his
investigations shows total villous atrophy and lymphocytic infiltrate. He has a skin lesion
that has small itchy papules.

You answered Pyoderma gangrenosum

The correct answer is Dermatitis herpetiformis

Theme from September 2012 Exam


The patient has coeliac disease and this is associated with dermatitis herpetiformis.

15. A 72 year old man is investigated for weight loss. On examination he is deeply jaundiced
and cachectic. He also has a dark velvety lesion coating his tongue.

You answered Pyoderma gangrenosum

The correct answer is Acanthosis nigricans

Acanthosis nigricans may be associated with GI malignancies such as gastric and


pancreatic cancer.

16. A lesion that may occur in a 32 year old man with long standing Crohns disease.

Pyoderma gangrenosum

Pyoderma gangrenosum may occur in Crohns disease.

Please rate this question:


Discuss and give feedback
Next question

Skin Diseases

Skin lesions may be referred for surgical assessment, but more commonly will come via a
dermatologist for definitive surgical management.

Skin malignancies include basal cell carcinoma, squamous cell carcinoma and malignant melanoma.

Basal Cell Carcinoma

 Most common form of skin cancer.


 Commonly occur on sun exposed sites apart from the ear.
 Sub types include nodular, morphoeic, superficial and pigmented.
 Typically slow growing with low metastatic potential.
 Standard surgical excision, topical chemotherapy and radiotherapy are all successful.
 As a minimum a diagnostic punch biopsy should be taken if treatment other than standard
surgical excision is planned.

Squamous Cell Carcinoma

 Again related to sun exposure.


 May arise in pre - existing solar keratoses.
 May metastasize if left.
 Immunosupression (e.g. following transplant), increases risk.
 Wide local excision is the treatment of choice and where a diagnostic excision biopsy has
demonstrated SCC, repeat surgery to gain adequate margins may be required.

Malignant Melanoma
The main diagnostic features (major criteria): Secondary features (minor criteria)

 Change in size  Diameter >6mm


 Change in shape  Inflammation
 Change in colour  Oozing or bleeding
 Altered sensation

Treatment

 Suspicious lesions should undergo excision biopsy. The lesion should be removed in
completely as incision biopsy can make subsequent histopathological assessment difficult.
 Once the diagnosis is confirmed the pathology report should be reviewed to determine
whether further re-excision of margins is required (see below):

Margins of excision-Related to Breslow thickness


Lesions 0-1mm thick 1cm

Lesions 1-2mm thick 1- 2cm (Depending upon site and pathological features)

Lesions 2-4mm thick 2-3 cm (Depending upon site and pathological features)

Lesions >4 mm thick 3cm

Marsden J et al. Revised UK guidelines for management of Melanoma. Br J Dermatol 2010 163:238-
256.

Further treatments such as sentinel lymph node mapping, isolated limb perfusion and block
dissection of regional lymph node groups should be selectively applied.

Kaposi Sarcoma

 Tumour of vascular and lymphatic endothelium.


 Purple cutaneous nodules.
 Associated with immuno supression.
 Classical form affects elderly males and is slow growing.
 Immunosupression form is much more aggressive and tends to affect those with HIV related
disease.

Non malignant skin disease

Dermatitis Herpetiformis

 Chronic itchy clusters of blisters.


 Linked to underlying gluten enteropathy (coeliac disease).

Dermatofibroma

 Benign lesion.
 Firm elevated nodules.
 Usually history of trauma.
 Lesion consists of histiocytes, blood vessels and fibrotic changes.
Pyogenic granuloma

 Overgrowth of blood vessels.


 Red nodules.
 Usually follow trauma.
 May mimic amelanotic melanoma.

Acanthosis nigricans

 Brown to black, poorly defined, velvety hyperpigmentation of the skin.


 Usually found in body folds such as the posterior and lateral folds of the neck, the axilla,
groin, umbilicus, forehead, and other areas.
 The most common cause of acanthosis nigricans is insulin resistance, which leads to
increased circulating insulin levels. Insulin spillover into the skin results in its abnormal
increase in growth (hyperplasia of the skin).
 In the context of a malignant disease, acanthosis nigricans is a paraneoplastic syndrome and
is then commonly referred to as acanthosis nigricans maligna. Involvement of mucous
membranes is rare and suggests a coexisting malignant condition
Which of the following statements relating to sebaceous cysts is false?

When infected are also known as Cocks peculiar tumour

Typically contain pus

Are usually associated with a central punctum

Most commonly occur on the scalp

They will typically have a cyst wall

Sebaceous cysts usually contain sebum, pus is only present in infected sebaceous cysts which
should then be treated by surgical incision and drainage.
Please rate this question:

Discuss and give feedback


Next question

Sebaceous cysts

 Originate from sebaceous glands and contain sebum.


 Location: anywhere but most common scalp, ears, back, face, and upper arm (not palms of
the hands and soles of the feet).
 They will typically contain a punctum.
 Excision of the cyst wall needs to be complete to prevent recurrence.
 A Cock's 'Peculiar' Tumour is a suppurating and ulcerated sebaceous cyst. It may resemble
a squamous cell carcinoma- hence its name.

Next question
Which of the following statements relating to Keloid scars is untrue?

They have a predilection for sternal , mandibular and deltiod area wounds

They are confined to the margins of the original injury

They often recur following excision

May occur even after superficial injury

They may be treated by injection of triamcinolone

Hypertrophic scars remain confined to the wound edges.

Keloids (by definition) will tend to extend beyond the margins of the wound and in wounds of any
depth.
Please rate this question:

Discuss and give feedback


Next question

Wound healing

Surgical wounds are either incisional or excisional and either clean, clean contaminated or dirty.
Although the stages of wound healing are broadly similar their contributions will vary according to the
wound type.

The main stages of wound healing include:

Haemostasis

 Minutes to hours following injury


 Vasospasm in adjacent vessels, platelet plug formation and generation of fibrin rich clot.

Inflammation

 Typically days 1-5


 Neutrophils migrate into wound (function impaired in diabetes).
 Growth factors released, including basic fibroblast growth factor and vascular endothelial
growth factor.
 Fibroblasts replicate within the adjacent matrix and migrate into wound.
 Macrophages and fibroblasts couple matrix regeneration and clot substitution.

Regeneration

 Typically days 7 to 56
 Platelet derived growth factor and transformation growth factors stimulate fibroblasts and
epithelial cells.
 Fibroblasts produce a collagen network.
 Angiogenesis occurs and wound resembles granulation tissue.

Remodeling

 From 6 weeks to 1 year


 Longest phase of the healing process and may last up to one year (or longer).
 During this phase fibroblasts become differentiated (myofibroblasts) and these facilitate
wound contraction.
 Collagen fibres are remodeled.
 Microvessels regress leaving a pale scar.

The above description represents an idealised scenario. A number of diseases may distort this
process. Neovascularisation is an important early process. Endothelial cells may proliferate in the
wound bed and recanalise to form a vessel. Vascular disease, shock and sepsis can all compromise
microvascular flow and impair healing.

Conditions such as jaundice will impair fibroblast synthetic function and immunity with a detrimental
effect in most parts of the healing process.

Problems with scars:

Hypertrophic scars
Excessive amounts of collagen within a scar. Nodules may be present histologically containing
randomly arranged fibrils within and parallel fibres on the surface. The tissue itself is confined to the
extent of the wound itself and is usually the result of a full thickness dermal injury. They may go on
to develop contractures.

Image of hypertrophic scarring. Note that it remains confined to the boundaries of the original
wound:
Image sourced from Wikipedia

Keloid scars
Excessive amounts of collagen within a scar. Typically a keloid scar will pass beyond the boundaries
of the original injury. They do not contain nodules and may occur following even trivial injury. They
do not regress over time and may recur following removal.

Image of a keloid scar. Note the extension beyond the boundaries of the original incision:

Image sourced from Wikipedia

Drugs which impair wound healing:


 Non steroidal anti inflammatory drugs
 Steroids
 Immunosupressive agents
 Anti neoplastic drugs

Closure
Delayed primary closure is the anatomically precise closure that is delayed for a few days but before
granulation tissue becomes macroscopically evident.

Secondary closure refers to either spontaneous closure or to surgical closure after granulation tissue
has formed.
A 72 year old man presents with a lesion on his back. Its appearances are as shown below:

Image sourced from Wikipedia

What is the most likely diagnosis?

Amelanotic malignant melanoma

Squamous cell carcinoma

Merkel Cell tumour

Basal cell carcinoma

None of the above

The lesion has all the characteristic features of a basal cell carcinoma. Including raised surface and
overlying telangectasia. Amelanotic melanomas are rare lesions and usually have a more ulcerated
appearance. Since the question is directed towards the most likely diagnosis the correct answer is
basal cell carcinoma.

Clinical images are not currently part of the MRCS Part A


Please rate this question:

Discuss and give feedback


Next question
Skin Diseases

Skin lesions may be referred for surgical assessment, but more commonly will come via a
dermatologist for definitive surgical management.

Skin malignancies include basal cell carcinoma, squamous cell carcinoma and malignant melanoma.

Basal Cell Carcinoma

 Most common form of skin cancer.


 Commonly occur on sun exposed sites apart from the ear.
 Sub types include nodular, morphoeic, superficial and pigmented.
 Typically slow growing with low metastatic potential.
 Standard surgical excision, topical chemotherapy and radiotherapy are all successful.
 As a minimum a diagnostic punch biopsy should be taken if treatment other than standard
surgical excision is planned.

Squamous Cell Carcinoma

 Again related to sun exposure.


 May arise in pre - existing solar keratoses.
 May metastasize if left.
 Immunosupression (e.g. following transplant), increases risk.
 Wide local excision is the treatment of choice and where a diagnostic excision biopsy has
demonstrated SCC, repeat surgery to gain adequate margins may be required.

Malignant Melanoma
The main diagnostic features (major criteria): Secondary features (minor criteria)

 Change in size  Diameter >6mm


 Change in shape  Inflammation
 Change in colour  Oozing or bleeding
 Altered sensation

Treatment

 Suspicious lesions should undergo excision biopsy. The lesion should be removed in
completely as incision biopsy can make subsequent histopathological assessment difficult.
 Once the diagnosis is confirmed the pathology report should be reviewed to determine
whether further re-excision of margins is required (see below):
Margins of excision-Related to Breslow thickness
Lesions 0-1mm thick 1cm

Lesions 1-2mm thick 1- 2cm (Depending upon site and pathological features)

Lesions 2-4mm thick 2-3 cm (Depending upon site and pathological features)

Lesions >4 mm thick 3cm

Marsden J et al. Revised UK guidelines for management of Melanoma. Br J Dermatol 2010 163:238-
256.

Further treatments such as sentinel lymph node mapping, isolated limb perfusion and block
dissection of regional lymph node groups should be selectively applied.

Kaposi Sarcoma

 Tumour of vascular and lymphatic endothelium.


 Purple cutaneous nodules.
 Associated with immuno supression.
 Classical form affects elderly males and is slow growing.
 Immunosupression form is much more aggressive and tends to affect those with HIV related
disease.

Non malignant skin disease

Dermatitis Herpetiformis

 Chronic itchy clusters of blisters.


 Linked to underlying gluten enteropathy (coeliac disease).

Dermatofibroma

 Benign lesion.
 Firm elevated nodules.
 Usually history of trauma.
 Lesion consists of histiocytes, blood vessels and fibrotic changes.
Pyogenic granuloma

 Overgrowth of blood vessels.


 Red nodules.
 Usually follow trauma.
 May mimic amelanotic melanoma.

Acanthosis nigricans

 Brown to black, poorly defined, velvety hyperpigmentation of the skin.


 Usually found in body folds such as the posterior and lateral folds of the neck, the axilla,
groin, umbilicus, forehead, and other areas.
 The most common cause of acanthosis nigricans is insulin resistance, which leads to
increased circulating insulin levels. Insulin spillover into the skin results in its abnormal
increase in growth (hyperplasia of the skin).
 In the context of a malignant disease, acanthosis nigricans is a paraneoplastic syndrome and
is then commonly referred to as acanthosis nigricans maligna. Involvement of mucous
membranes is rare and suggests a coexisting malignant condition.

Next question
A 29 year old man presents with a lump in his scalp. It is located approximately 4cm superior to the
external occipital protuberance. It feels smooth and slightly fluctuant and has a centrally located
small epithelial defect. What is the most likely underlying diagnosis?

Cocks peculiar tumour

Dermoid cyst

Sebaceous cyst

Merkel cell tumour

Seborrhoeic wart

Sebaceous cysts are most frequently located in the scalp and have an associated central punctum.
They may become infected and develop superficial ulceration in which case they are known as
"Cocks Peculiar Tumour". The presence of a punctum is highly suggestive of a sebaceous cyst and
are not typically found in the other lesions described.
Please rate this question:

Discuss and give feedback


Next question

Sebaceous cysts

 Originate from sebaceous glands and contain sebum.


 Location: anywhere but most common scalp, ears, back, face, and upper arm (not palms of
the hands and soles of the feet).
 They will typically contain a punctum.
 Excision of the cyst wall needs to be complete to prevent recurrence.
 A Cock's 'Peculiar' Tumour is a suppurating and ulcerated sebaceous cyst. It may resemble
a squamous cell carcinoma- hence its name.
Theme: Management of skin diseases

A. Excision biopsy
B. Excision with 1 cm margin
C. Excision with 5 cm margin
D. Shave biopsy and cautery
E. Punch biopsy
F. Excision and full thickness skin graft
G. Discharge

For each scenario please select the most appropriate management option. Each option may be used
once, more than once or not at all.

21. A 89 year old women presents with long standing seborrhoeic warts of her abdominal wall
, they have caused troublesome itching.

You answered Excision biopsy

The correct answer is Shave biopsy and cautery

These lesions are often extensive and superficial. Shave excision will suffice, material
must be sent for histology.

22. A 22 year old man has an excision biopsy of a pigmented lesion from his back, histology
shows a 1mm depth nodular melanoma, all resection margins are clear of tumour and the
nearest is 0.5cm.

You answered Excision biopsy

The correct answer is Excision with 1 cm margin

This man will require re-excision of margins so that a 1cm margin around the lesion is
achieved. This can usually be achieved without skin grafting.

23. A 73 year old lady presents to the breast clinic with a weeping crusty skin lesion of the left
nipple. There are no masses to feel in the breast itself and imaging is normal.

You answered Excision biopsy

The correct answer is Punch biopsy

This is likely to represent Pagets disease of the nipple and is best diagnosed on punch
biopsy.
Please rate this question:

Discuss and give feedback

Skin Diseases

Skin lesions may be referred for surgical assessment, but more commonly will come via a
dermatologist for definitive surgical management.

Skin malignancies include basal cell carcinoma, squamous cell carcinoma and malignant melanoma.

Basal Cell Carcinoma

 Most common form of skin cancer.


 Commonly occur on sun exposed sites apart from the ear.
 Sub types include nodular, morphoeic, superficial and pigmented.
 Typically slow growing with low metastatic potential.
 Standard surgical excision, topical chemotherapy and radiotherapy are all successful.
 As a minimum a diagnostic punch biopsy should be taken if treatment other than standard
surgical excision is planned.

Squamous Cell Carcinoma

 Again related to sun exposure.


 May arise in pre - existing solar keratoses.
 May metastasize if left.
 Immunosupression (e.g. following transplant), increases risk.
 Wide local excision is the treatment of choice and where a diagnostic excision biopsy has
demonstrated SCC, repeat surgery to gain adequate margins may be required.

Malignant Melanoma
The main diagnostic features (major criteria): Secondary features (minor criteria)

 Change in size  Diameter >6mm


 Change in shape  Inflammation
 Change in colour  Oozing or bleeding
 Altered sensation

Treatment
 Suspicious lesions should undergo excision biopsy. The lesion should be removed in
completely as incision biopsy can make subsequent histopathological assessment difficult.
 Once the diagnosis is confirmed the pathology report should be reviewed to determine
whether further re-excision of margins is required (see below):

Margins of excision-Related to Breslow thickness


Lesions 0-1mm thick 1cm

Lesions 1-2mm thick 1- 2cm (Depending upon site and pathological features)

Lesions 2-4mm thick 2-3 cm (Depending upon site and pathological features)

Lesions >4 mm thick 3cm

Marsden J et al. Revised UK guidelines for management of Melanoma. Br J Dermatol 2010 163:238-
256.

Further treatments such as sentinel lymph node mapping, isolated limb perfusion and block
dissection of regional lymph node groups should be selectively applied.

Kaposi Sarcoma

 Tumour of vascular and lymphatic endothelium.


 Purple cutaneous nodules.
 Associated with immuno supression.
 Classical form affects elderly males and is slow growing.
 Immunosupression form is much more aggressive and tends to affect those with HIV related
disease.

Non malignant skin disease

Dermatitis Herpetiformis

 Chronic itchy clusters of blisters.


 Linked to underlying gluten enteropathy (coeliac disease).

Dermatofibroma

 Benign lesion.
 Firm elevated nodules.
 Usually history of trauma.
 Lesion consists of histiocytes, blood vessels and fibrotic changes.

Pyogenic granuloma

 Overgrowth of blood vessels.


 Red nodules.
 Usually follow trauma.
 May mimic amelanotic melanoma.

Acanthosis nigricans

 Brown to black, poorly defined, velvety hyperpigmentation of the skin.


 Usually found in body folds such as the posterior and lateral folds of the neck, the axilla,
groin, umbilicus, forehead, and other areas.
 The most common cause of acanthosis nigricans is insulin resistance, which leads to
increased circulating insulin levels. Insulin spillover into the skin results in its abnormal
increase in growth (hyperplasia of the skin).
 In the context of a malignant disease, acanthosis nigricans is a paraneoplastic syndrome and
is then commonly referred to as acanthosis nigricans maligna. Involvement of mucous
membranes is rare and suggests a coexisting malignant condition.
A 48 year old lady undergoes a laparotomy and a retroperitoneal tumour is identified. The surgeons
suspect that the lesion is a liposarcoma. Which of the following is not typical of liposarcomas?

They are the most common variant of sarcoma in adults

Core biopsies in low grade liposarcomas may be normal

May have a pseudocapsule

It is unlikely in a lesion measuring less than 5cm

Pulmonary metastasis are more likely than regional nodal involvement

Malignant fibrous histiocytoma is the commonest variant of sarcoma and liposarcoma the second
most common. The presence of a pseudocapsule should be borne in mind when performing surgery,
as incomplete removal will result in local recurrence.
Please rate this question:

Discuss and give feedback


Next question

Sarcomas

 Malignant tumours of mesenchymal origin

Types
May be either bone or soft tissue in origin.
Bone sarcoma include:

 Osteosarcoma
 Ewings sarcoma (although non bony sites recognised)
 Chondrosarcoma - originate from Chondrocytes

Soft tissue sarcoma are a far more heterogeneous group and include:

 Liposarcoma-adipocytes
 Rhabdomyosarcoma-striated muscle
 Leiomyosarcoma-smooth muscle
 Synovial sarcomas- close to joints (cell of origin not known but not synovium)

Malignant fibrous histiocytoma is a sarcoma that may arise in both soft tissue and bone.

Features
Certain features of a mass or swelling should raise suspicion for a sarcoma these include:

 Large >5cm soft tissue mass


 Deep tissue location or intra muscular location
 Rapid growth
 Painful lump

Assessment
Imaging of suspicious masses should utilise a combination of MRI, CT and USS. Blind biopsy should
not be performed prior to imaging and where required should be done in such a way that the biopsy
tract can be subsequently included in any resection.

Ewings sarcoma

 Commoner in males
 Incidence of 0.3 / 1, 000, 000
 Onset typically between 10 and 20 years of age
 Location by femoral diaphysis is commonest site
 Histologically it is a small round tumour
 Blood borne metastasis is common and chemotherapy is often combined with surgery

Osteosarcoma

 Mesenchymal cells with osteoblastic differentiation


 20% of all primary bone tumours
 Incidence of 5 per 1,000,000
 Peak age 15-30, commoner in males
 Limb preserving surgery may be possible and many patients will receive chemotherapy

Liposarcoma

 Malignancy of adipocytes
 Rare, approximately 2.5 per 1,000,000. They are the second most common soft tissue
sarcoma
 Typically located in deep locations such as retroperitoneum
 Affect older age group usually >40 years of age
 May be well differentiated and thus slow growing although may undergo de-differentiation
and disease progression
 Many tumours will have a pseudocapsule that can misleadingly allow surgeons to feel that
they can 'shell out' these lesions. In reality, tumour may invade at the edge of the
pseudocapsule and result in local recurrence if this strategy is adopted
 Usually resistant to radiotherapy, although this is often used in a palliative setting

Malignant Fibrous Histiocytoma

 Tumour with large number of histiocytes


 Most common sarcoma in adults
 Also described as undifferentiated pleomorphic sarcoma NOS (i.e. Cell of origin is not
known)
 Four major subtypes are recognised: storiform-pleomorphic (70% cases), myxoid (less
aggressive), giant cell and inflammatory
 Treatment is usually with surgical resection and adjuvant radiotherapy as this reduces the
likelihood of local recurrence

Next question
A 63 year old lady with metastatic breast cancer presents with bone pain. Radiological tests show a
metastatic lytic deposit to her femoral shaft. The lesion occupies 75% of the bone diameter. What is
the most appropriate management?

Surgical fixation with a dynamic compression plate

Hemi-arthroplasty

Fixation with intramedullary nail

Radical radiotherapy

Chemotherapy

Even with surgical fixation only 30% of pathological fractures unite. The type of fixation should be
chosen accordingly.

A lesion of this nature is at high risk of spontaneous fracture. Whilst radiotherapy may palliate her
symptoms of pain it will not reduce the risk of fracture. In fit patients, an intramedullary nail should be
inserted. Very proximal lesions may be best managed by a total hip replacement
Please rate this question:

Discuss and give feedback


Next question

Secondary malignant tumours of bone

Metastatic lesions affecting bone are more common than primary bone tumours.

The typical tumours that spread to bone include:

 Breast
 Bronchus
 Renal
 Thyroid
 Prostate

75% cases will affect those over the age of 50


The commonest bone sites affected are:

 Vertebrae (usually thoracic)


 Proximal femur
 Ribs
 Sternum
 Pelvis
 Skull

Pathological fracture
Osteolytic lesions are the greatest risk for pathological fracture
The risk and load required to produce fracture varies according to bone site. Bones with lesions that
occupy 50% or less will be prone to fracture under loading (Harrington). When 75% of the bone is
affected the process of torsion about a bony fulcrum may produce a fracture.

The Mirel scoring[1] system may be used to help determine the risk of fracture and is more
systematic than the Harrington system described above.

Mirel Scoring system

Score Radiographic Width of bone


points Site appearance involved Pain

1 Upper extremity Blastic Less than 1/3 Mild

2 Lower Mixed 1/3 to 2/3 Moderate


extremity

3 Peritrochanteric Lytic More than 2/3 Aggravated by


function

Depending upon the score the treatment should be as follows:

Score Risk of fracture Treatment

9 or greater Impending (33%) Prophylactic fixation

8 Borderline Consider fixation


Score Risk of fracture Treatment

7 or less Not impending (4%) Non operative management

Where the lesion is an isolated metastatic deposit consideration should be given to excision and
reconstruction as the outcome is better [2].

Non operative treatments


Hypercalcaemia- Treat with re hydration and bisphosphonates.
Pain- Opiate analgesics and radiotherapy.
Some tumours such as breast and prostate will benefit from chemotherapy and or hormonal agents.

References
1. Mirels, H. Metastatic disease in long bones. A proposed scoring system for diagnosing impending
pathologic fractures. Clin Orthop Relat Res, 1989(249): p. 256-64.
2. Mavrogenis, A.F. et al. Survival analysis of patients with femoral metastases. J Surg Oncol, 2011.
Next question
Theme: Familial polyposis syndromes

A. Peutz-Jeghers syndrome

B. Cowden disease

C. Familial adenomatous polyposis coli

D. Lynch syndrome

E. MYH associated polyposis

Please select the familial cancer syndrome that most closely matches the description provided. Each
option may be used once, more than once or not at all.

3. A syndrome consisting of a PTEN mutation and intestinal hamartomas.

You answered Peutz-Jeghers syndrome

The correct answer is Cowden disease

PTEN is a tumour supressor gene and loss of function mutations result in up regulation of the
mTOR pathway.

4. A syndrome which may be present in a patient with multiple intestinal hamartomas and
pigmentation spots around the mouth.

Peutz-Jeghers syndrome

5. A syndrome likely to be present in a 28 year old man who presents with a locally advanced
mucinous carcinoma of the caecum. There are scanty polyps in the remaining colon. His father died
from colorectal cancer aged 34.

You answered Peutz-Jeghers syndrome

The correct answer is Lynch syndrome


Lynch syndrome is likely when right sided colonic cancers occur at a young age. These tumours are
often poorly differentiated and mucinous. The Amsterdam criteria can be used to identify families
at risk who may benefit from genetic testing.

Please rate this question:

Discuss and give feedback

Next question

Polyposis syndromes

Screening and Associated


Syndrome Genetic defect Features management disorders

Familial Mutation of APC Typically over 100 If known to be at risk Gastric fundal
adenomatous gene (80%) cases, colonic adenomas then predictive polyps (50%).
polyposis dominant Cancer risk of 100% genetic testing as Duodenal polyps
20% are new teenager 90%.
mutations Annual flexible If severe
sigmoidoscopy from duodenal
15 years polyposis cancer
If no polyps found risk of 30% at 10
then 5 yearly years.
colonoscopy started Abdominal
at age 20 desmoid
Polyps found = tumours.
resectional surgery
(resection and pouch
Vs sub total
colectomy and IRA)

MYH Biallelic mutation Multiple colonic Once identified Duodenal


Screening and Associated
Syndrome Genetic defect Features management disorders

associated of mut Y human polyps resection and polyposis in 30%


polyposis homologue (MYH) Later onset right ileoanal pouch Associated with
on chromosome sided cancers more reconstruction is increased risk of
1p, recessive common than in FAP recommended breast cancer
100% cancer risk by Attenuated (self examination)
age 60 phenotype - regular
colonoscopy

Peutz -Jeghers STK11 (LKB1) Multiple benign Annual examination Malignancies at


syndrome mutation on intestinal Pan intestinal other sites
chromosome 19 in hamartomas endoscopy every 2-3 Classical
some (but not all) Episodic obstruction years pigmentation
cases, dominant and intussceception pattern
Increased risk of GI
cancers (colorectal
cancer 20%, gastric
5%)
Increased risk of
breast, ovarian,
cervical pancreatic
and testicular
cancers

Cowden Mutation of PTEN Macrocephaly Targeted Breast cancer


disease gene on Multiple intestinal individualised (81% risk)
chromosome hamartomas screening Thyroid cancer
10q22, dominant Multiple and non toxic
trichilemmomas goitre
89% risk of cancer at Uterine cancer
any site
16% risk of colorectal
cancer

HNPCC (Lynch Germline Colo rectal cancer Colonoscopy every 1- Extra colonic
Screening and Associated
Syndrome Genetic defect Features management disorders

syndrome) mutations of DNA 30-70% 2 years from age 25 cancers


mismatch repair Endometrial cancer Consideration of
genes 30-70% prophylactic surgery
Gastric cancer 5-10% Extra colonic
Scanty colonic polyps surveillance
may be present recommended
Colonic tumours
likely to be right
sided and mucinous

Next question
A 45 year old women with breast cancer is started on a chemotherapy regime containing epirubicin.
What is the primary mode of action of this drug?

Intercalation of DNA

Antimetabolite

Monoclonal antibody to epidermal growth factor

Inhibition of DNA gyrase

Inhibition of topoisomerase 1

Class Example Mode of action

Antimetabolites 5 FU S Phase specific drug, mimics uracil and is


incorporated into RNA

Anthracyclines* Doxorubicin Inhibits DNA and RNA synthesis by intercalating


base pairs

Topoisomerase Etoposide Inhibits topoisomerase II, prevents efficient DNA


inhibitors** coiling

Platinum Cisplatin Crosslinks DNA, this then distorts molecule and


induces apoptosis (similar to alkylating agents)

Alkylating agent Cyclophosphamide Phosphoramide mustard forms DNA crosslinks and


then cell death

Taxanes Docetaxal Disrupts microtubule formation


*=Main adverse effect cardiotoxicity
**=Irinotecan is a similar drug which works by inhibition of topoisomerase I
Please rate this question:

Discuss and give feedback


Next question

Breast cancer treatment

Treatment Indication

Endocrine  Oestrogen receptor positive tumours


therapy  Downstaging primary lesions
 Definitive treatment in old, infirm patients

Irradiation  Wide local excision


 Large lesion, high grade or marked vascular invasion following
mastectomy

Chemotherapy  Downstaging advanced lesions to facilitate breast conserving surgery


 Patients with grade 3 lesions or axillary nodal disease

Endocrine agents
Tamoxifen is used and works as a partial oestrogen receptor agonist. It will typically block activity at
the breast. It does, however, stimulate the receptors at other sites and it is this that accounts for its
association with endometrial cancer. In post menopausal women the process of aromatisation
accounts for most oestrogen production. Therefore in this group aromatase inhibitors are the
preferred agents. Women who are perimenopausal start on tamoxifen and switch at 3 years.

Chemotherapy
The FEC regime is most commonly used (Fluorouracil, epirubicin and cyclophosphamide). This was
found to be superior to the older CMF regime. The Taxanes are commonly used in high risk patients
and in this setting a regime of docetaxal, doxorubicin and cyclophosphamide may be used. The
anthracycline class drugs have marked cardiotoxicity (a property that they share with trastuzumab)
and this can limit their use.
Next question
A 50 year old lady presents with pain in her proximal femur. Imaging demonstrates a bone
metastasis from an unknown primary site. CT scanning with arterial phase contrast shows that the
lesion is hypervascular. From which of the following primary sites is the lesion most likely to have
originated?

Breast

Renal

Bronchus

Thyroid

Colon

Renal metastases have a tendency to be hypervascular. This is of considerable importance if


surgical fixation is planned.
Please rate this question:

Discuss and give feedback


Next question

Secondary malignant tumours of bone

Metastatic lesions affecting bone are more common than primary bone tumours.

The typical tumours that spread to bone include:

 Breast
 Bronchus
 Renal
 Thyroid
 Prostate

75% cases will affect those over the age of 50

The commonest bone sites affected are:

 Vertebrae (usually thoracic)


 Proximal femur
 Ribs
 Sternum
 Pelvis
 Skull

Pathological fracture
Osteolytic lesions are the greatest risk for pathological fracture
The risk and load required to produce fracture varies according to bone site. Bones with lesions that
occupy 50% or less will be prone to fracture under loading (Harrington). When 75% of the bone is
affected the process of torsion about a bony fulcrum may produce a fracture.

The Mirel scoring[1] system may be used to help determine the risk of fracture and is more
systematic than the Harrington system described above.

Mirel Scoring system

Score Radiographic Width of bone


points Site appearance involved Pain

1 Upper extremity Blastic Less than 1/3 Mild

2 Lower Mixed 1/3 to 2/3 Moderate


extremity

3 Peritrochanteric Lytic More than 2/3 Aggravated by


function

Depending upon the score the treatment should be as follows:

Score Risk of fracture Treatment

9 or greater Impending (33%) Prophylactic fixation

8 Borderline Consider fixation

7 or less Not impending (4%) Non operative management


Where the lesion is an isolated metastatic deposit consideration should be given to excision and
reconstruction as the outcome is better [2].

Non operative treatments


Hypercalcaemia- Treat with re hydration and bisphosphonates.
Pain- Opiate analgesics and radiotherapy.
Some tumours such as breast and prostate will benefit from chemotherapy and or hormonal agents.

References
1. Mirels, H. Metastatic disease in long bones. A proposed scoring system for diagnosing impending
pathologic fractures. Clin Orthop Relat Res, 1989(249): p. 256-64.
2. Mavrogenis, A.F. et al. Survival analysis of patients with femoral metastases. J Surg Oncol, 2011.
Next question
Which of the following group of patients are not screened for colorectal cancer?

Peutz Jeghers syndrome

Asymptomatic patients aged 45 years

Acromegaly

Ureterosigmoidostomy

Inflammatory bowel disease

Other disorders which are screened for colorectal malignancy include:


Familial adenomatous polyposis, Hereditary non polyposis colorectal cancer. The NHS screening
programme starts at 60.
Please rate this question:

Discuss and give feedback


Next question

Colorectal cancer screening and diagnosis

Overview

 Most cancers develop from adenomatous polyps. Screening for colorectal cancer has been
shown to reduce mortality by 16%
 The NHS now has a national screening programme offering screening every 2 years to all
men and women aged 60 to 69 years. Patients aged over 70 years may request screening
 Eligible patients are sent faecal occult blood (FOB) tests through the post
 Patients with abnormal results are offered a colonoscopy

At colonoscopy, approximately:

 5 out of 10 patients will have a normal exam


 4 out of 10 patients will be found to have polyps which may be removed due to their
premalignant potential
 1 out of 10 patients will be found to have cancer
Diagnosis
Essentially the following patients need referral:
- Altered bowel habit for more than six weeks
- New onset of rectal bleeding
- Symptoms of tenesmus

Colonoscopy is the gold standard, provided it is complete and good mucosal visualisation is
achieved. Other options include double contrast barium enema and CT colonography.

Staging
Once a malignant diagnosis is made patients with colonic cancer will be staged using chest /
abdomen and pelvic CT. Patients with rectal cancer will also undergo evaluation of the mesorectum
with pelvic MRI scanning.

For examination purposes the Dukes and TNM systems are preferred.

Tumour markers
Carcinoembryonic antigen (CEA) is the main tumour marker in colorectal cancer. Not all tumours
secrete this, and it may be raised in conditions such as IBD. However, absolute levels do correlate
(roughly) with disease burden and it is once again being used routinely in follow up.
Next question
A 56-year-old man with metastatic prostate cancer comes for review. He is known to have spinal
metastases but until now has not had any significant problems with pain control. Unfortunately he is
now getting regular back pain despite taking paracetamol 1g qds. Neurological examination is
unremarkable. What is the most appropriate next step?

Switch to co-codamol 30/500

Refer for radiotherapy

Add oral bisphosphonate

Add non steroidal anti inflammatory drug

Add dexamethasone

Metastatic bone pain may respond to NSAIDs, bisphosphonates or radiotherapy

Bone pain often responds well to NSAIDs. Both radiotherapy and bisphosphonates have a role in
managing bony pain but these are not first-line treatments.
Please rate this question:

Discuss and give feedback


Next question

Palliative care prescribing: pain

SIGN issued guidance on the control of pain in adults with cancer in 2008. Selected points

 the breakthrough dose of morphine is one-sixth the daily dose of morphine


 all patients who receive opioids should be prescribed a laxative
 opioids should be used with caution in patients with chronic kidney disease. Alfentanil,
buprenorphine and fentanyl are preferred
 metastatic bone pain may respond to NSAIDs, bisphosphonates or radiotherapy

When increasing the dose of opioids the next dose should be increased by 30-50%.

Conversion between opioids


From To
From To

Oral codeine Oral morphine Divide by 10

Oral tramadol Oral morphine Divide by 5

From To

Oral morphine Oral oxycodone Divide by 2

The BNF states that oral morphine sulphate 80-90mg over 24 hours is approximately equivalent to
one '25 mcg/hour' fentanyl patch, therefore product literature should be consulted.

From To

Oral morphine Subcutaneous diamorphine Divide by 3

Oral oxycodone Subcutaneous diamorphine Divide by 1.5

Next question
A 62 year old male is found to have colorectal cancer. He has Dukes C disease. What is his 5 year
prognosis?

100%

90%

80%

70%

60%

Theme from September 2011 Exam

Please rate this question:

Discuss and give feedback

Next question

Dukes classification

Gives the extent of spread of colorectal cancer

Dukes A Tumour confined to the mucosa (95%)

Dukes B Tumour invading bowel wall (80%)

Dukes C Lymph node metastases (60%)


Dukes D Distant metastases (6%)(25% if resectable)

5 year survival in brackets

Next question
Chordoma may typically occur at the following sites, except?

Ribs

Clivus

Sacrum

Lumbar vertebra

Cervical vertebra

Chordoma is a neoplasm originating from ectopic cellular remnants of the notochord and therefore
arises from the midline of the axial skeleton. It accounts for 24% of all primary malignant bone
tumours. Chordoma is the second commonest primary malignancy of the spine and accounts for
over 50% of primary sacral tumours. The neoplasm has a predilection for the sacrococcygeal (50%)
and clival (40%) regions, with other areas of the spine rarely involved. More than one vertebral body
can be affected in half the cases. Chordomas most commonly present between 50 and 70 years of
age. Sex incidence is equal below 40 years, but men are affected twice as often at older ages,
particularly in the sacral region.

The most frequent radiographic appearance of chordoma is that of a destructive lesion of a vertebral
body centered in the midline, with a large, associated soft-tissue mass.
Please rate this question:

Discuss and give feedback


Next question

Chordoma

Chordoma is a rare slow-growing bone tumour. Their favored origin is remnants of the notochord.

Chordomas can arise anywhere from the skull base to the sacrum. The two most common locations
are the skull base and sacrum.

There are three histological variants of chordoma: classical (or "conventional"), chondroid and de-
differentiated.

 The histological appearance of classical chordoma is of a lobulated tumor composed of


groups of cells separated by fibrous septa. The cells have small round nuclei and abundant
vacuolated cytoplasm.
 Chondroid chordomas histologically show features of both chordoma and chondrosarcoma.

The 10-year tumor free survival rate for sacral chordoma was 46%. Chondroid chordomas appear to
have a more indolent clinical course.

In most cases, complete surgical resection followed by radiation therapy offers the best chance of
long-term control. Unfortunately, the lesion has a close proximity to the spine itself and this can
compromise resection margins.

Chordomas are relatively radioresistant, requiring high doses of radiation to be controlled. The
proximity of chordomas to vital neurological structures such as the brain stem and nerves limits the
dose of radiation that can safely be delivered. Therefore, highly focused radiation such as proton
therapy and carbon ion therapy are more effective than conventional x-ray radiation.
Next question
In examining a biopsy of a primary tumour, the clearest evidence of malignancy is provided by:

Absence of a capsule

Basophilia of the cytoplasm

Invasion of surrounding structures

Excess of mitoses

Nuclear aberrations

Invasion is the hallmark of malignancy. The others may occur in insitu disease or dysplastic lesions.

Please rate this question:

Discuss and give feedback

Next question

Tissue sampling

Tissue sampling is an important surgical process. Biopsy modalities vary according to the site,
experience and subsequent planned therapeutic outcome

The modalities comprise:


-Fine needle aspiration cytology
-Core biopsy
-Excision biopsy
-Tru cut biopsy
-Punch biopsy
-Cytological smears
-Endoscopic or laparoscopic biopsy

When the lesion is superficial the decision needs to be taken as to whether complete excision is
desirable or whether excision biopsy is acceptable. In malignant melanoma for example the need for
safe margins will mean that a more radical surgical approach needs to be adopted after diagnostic
confirmation from excision biopsy than would be the case in basal cell carcinoma. Punch biopsies
are useful in gaining histological diagnosis of unclear skin lesions where excision biopsy is
undesirable such as in establishing whether a skin lesion is vasculitic or not.

Fine needle aspiration cytology (FNAC) is an operator dependent procedure that may or may not be
image guided and essentially involves passing a needle through a lesion whilst suction is applied to
a syringe. The material thus obtained is expressed onto a slide and sent for cytological assessment.
This test can be limited by operator inexperience and also by the lack of histological architectural
information (e.g. Follicular carcinoma of the thyroid). Where a discharge is present a sample may be
sent for cytology although in some sites (e.g. Nipple discharge ) the information gleaned may be
meaningless.

Tissue samples may be obtained by both core and tru cut biopsy. A core biopsy is obtained by use
of a spring loaded gun with a needle passing quickly through the lesion of interest. A tru cut biopsy
achieves the same objective but the needle moved by hand. When performing these techniques
image guidance may be desirable (e.g. In breast lesions). Consideration needs to be given to any
planned surgical resection as it may be necessary to resect the biopsy tract along with the specimen
(e.g. In sarcoma surgery).

Visceral lesions may be accessed percutaneously under image guidance such as ultrasound guided
biopsy of liver metastases. Or under direct vision such as a colonoscopic biopsy.

Next question
A 45 year old male is referred to clinic for consideration of resection of a lung malignancy. He reports
shortness of breath and haemoptysis. Investigations reveal a corrected calcium of 2.84 mmol/l, an
FEV 1 of 1.9L and histology of a squamous cell carcinoma. The patient is noted to have a hoarse
voice. Which one of the following is a contraindication to surgical resection in lung cancer?

Haemoptysis

FEV 1 of 1.9 litres

Histology shows squamous cell cancer

Vocal cord paralysis

Calcium = 2.84 mmol/L

Contraindications to lung cancer surgery include SVC obstruction, FEV < 1.5, MALIGNANT pleural
effusion, and vocal cord paralysis

Paralysis of a vocal cord implies extracapsular spread to mediastinal nodes and is an indication of
inoperability.
Please rate this question:

Discuss and give feedback


Next question

Lung cancer: non-small cell management

Management

 Only 20% suitable for surgery


 Mediastinoscopy performed prior to surgery as CT does not always show mediastinal lymph
node involvement
 Curative or palliative radiotherapy
 Poor response to chemotherapy

Surgery contraindications

 Assess general health


 Stage IIIb or IV (i.e. metastases present)
 FEV1 < 1.5 litres is considered a general cut-off point*
 Malignant pleural effusion
 Tumour near hilum
 Vocal cord paralysis
 SVC obstruction

* However if FEV1 < 1.5 for lobectomy or < 2.0 for pneumonectomy then some authorities advocate
further lung function tests as operations may still go ahead based on the results
Next question
A 43 year old lady is receiving chemotherapy for the treatment of metastatic breast cancer. You are
called because it has become apparent that her doxorubicin infusion has extravasated. What is the
most appropriate course of action?

Stop the infusion and administer dexamethasone through the infusion device

Stop the infusion and administer hyaluronidase through the infusion device

Stop the infusion and apply a cold compress to the site

Stop the infusion and apply a warm compress to the site

Stop the infusion and administer sodium bicarbonate through the infusion device

The application of cold compresses is indicated in doxorubicin extravasation. Warm compresses


increase the risk of doxorubicin ulceration. Hyaluronidase is indicated in the extravasation of contrast
media, TPN and vinca alkaloids. However, if administered following doxorubicin extravasation it will
dramatically worsen the situation and is contra indicated.
Up to 50% of those sustaining severe injuries will require delayed surgical reconstruction.

Please rate this question:

Discuss and give feedback

Next question

Extravasation injury

Chemotherapy may be complicated by extravasation reactions in up to 6% of cases. The following


chemotherapy agents are recognised causes of extravasation reactions; doxorubicin, vincristine,
vinblastine, adriamycin, cisplatin, mitomycin and mithramycin.
Up to 30% of extravasation reactions may be complicated by the development of ulceration.
When an extravasation reaction is suspected, the infusion should be stopped and the infusing device
aspirated. The extremity should be elevated. As a general rule cold compresses have been shown to
reduce the incidence of subsequent ulceration with doxorubicin. Warm compresses have been found
to be beneficial in extravasation of vinca alkaloids. Dimethylsulfoxide may be infused in some cases,
ideally within 5 hours of the event occurring. No conclusive evidence exists to support the use of
corticosteroids or sodium bicarbonate for extravasation injuries.

Extravasation of total parenteral nutrition solutions is usually managed by the local administration of
hyaluronidase to the infusion site.

Next question
A 56 year old lady presents with a pathological fracture of the proximal femur. Which of the following
primary sites is the most likely source of her disease?

Thyroid

Breast

Kidney

Endometrium

None of the above

Primary site= BBRTP

 Breast
 Bronchus
 Renal
 Thyroid
 Prostate

Theme from September 2011 exam

The correct answer is breast, because the question asks for the most likely primary site. Breast
cancer is the commonest cause of lytic bone metastasis in women of this age, especially from
amongst those options given.
Please rate this question:

Discuss and give feedback


Next question

Secondary malignant tumours of bone

Metastatic lesions affecting bone are more common than primary bone tumours.

The typical tumours that spread to bone include:

 Breast
 Bronchus
 Renal
 Thyroid
 Prostate

75% cases will affect those over the age of 50

The commonest bone sites affected are:

 Vertebrae (usually thoracic)


 Proximal femur
 Ribs
 Sternum
 Pelvis
 Skull

Pathological fracture
Osteolytic lesions are the greatest risk for pathological fracture
The risk and load required to produce fracture varies according to bone site. Bones with lesions that
occupy 50% or less will be prone to fracture under loading (Harrington). When 75% of the bone is
affected the process of torsion about a bony fulcrum may produce a fracture.

The Mirel scoring[1] system may be used to help determine the risk of fracture and is more
systematic than the Harrington system described above.

Mirel Scoring system

Score Radiographic Width of bone


points Site appearance involved Pain

1 Upper extremity Blastic Less than 1/3 Mild

2 Lower Mixed 1/3 to 2/3 Moderate


extremity

3 Peritrochanteric Lytic More than 2/3 Aggravated by


function

Depending upon the score the treatment should be as follows:

Score Risk of fracture Treatment


Score Risk of fracture Treatment

9 or greater Impending (33%) Prophylactic fixation

8 Borderline Consider fixation

7 or less Not impending (4%) Non operative management

Where the lesion is an isolated metastatic deposit consideration should be given to excision and
reconstruction as the outcome is better [2].

Non operative treatments


Hypercalcaemia- Treat with re hydration and bisphosphonates.
Pain- Opiate analgesics and radiotherapy.
Some tumours such as breast and prostate will benefit from chemotherapy and or hormonal agents.

References
1. Mirels, H. Metastatic disease in long bones. A proposed scoring system for diagnosing impending
pathologic fractures. Clin Orthop Relat Res, 1989(249): p. 256-64.
2. Mavrogenis, A.F. et al. Survival analysis of patients with femoral metastases. J Surg Oncol, 2011.
Next question
A 67-year-old man with colorectal cancer is currently taking MST 30mg bd for pain relief. What dose
of oral morphine solution should he be prescribed for breakthrough pain?

5 mg

10 mg

15 mg

20 mg

30 mg

Breakthrough dose = 1/6th of daily morphine dose

The total daily morphine dose is 30 * 2 = 60 mg, therefore the breakthrough dose should be one-
sixth of this, 10 mg.
Please rate this question:

Discuss and give feedback

Palliative care prescribing: pain

SIGN issued guidance on the control of pain in adults with cancer in 2008. Selected points

 the breakthrough dose of morphine is one-sixth the daily dose of morphine


 all patients who receive opioids should be prescribed a laxative
 opioids should be used with caution in patients with chronic kidney disease. Alfentanil,
buprenorphine and fentanyl are preferred
 metastatic bone pain may respond to NSAIDs, bisphosphonates or radiotherapy

When increasing the dose of opioids the next dose should be increased by 30-50%.

Conversion between opioids


From To
From To

Oral codeine Oral morphine Divide by 10

Oral tramadol Oral morphine Divide by 5

From To

Oral morphine Oral oxycodone Divide by 2

The BNF states that oral morphine sulphate 80-90mg over 24 hours is approximately equivalent to
one '25 mcg/hour' fentanyl patch, therefore product literature should be consulted.

From To

Oral morphine Subcutaneous diamorphine Divide by 3

Oral oxycodone Subcutaneous diamorphine Divide by 1.5


An orthopaedic surgeon makes a modification to an operative approach for total knee arthroplasty.
After he has completed 25 cases, he stops and reviews his patient outcomes. He publishes the data.
What level of evidence is supplied by this type of data?

II

IV

III

Case series that are non randomised and lack concurrent controls at best supply level IV evidence
only. To qualify for level I and II evidence a prospective randomised controlled trial with appropriate
blinding, control matching and power calculations is needed.
Please rate this question:

Discuss and give feedback


Next question

Levels of evidence

The level of evidence refers to the study design used by investigators to minimise bias.

Level of Source
evidence

I Evidence obtained from systematic review of all relevant randomised controlled


trials

II Evidence derived from at least one properly designed randomised controlled trial

III Evidence derived from well designed pseudo-randomised controlled trials (e.g.
alternate allocation) or historical controls

IV Evidence derived from case series or case reports

V Panel or expert opinion

Many of the categories contain sub groups, detailed knowledge of these are not required for MRCS
Part A.

References
Interested users will find further information at www.cebm.net
Next question
Which of the following is not utilised as a descriptive statistic?

Mean

Median

Mode

Z score

Standard deviation

The z score is determined using the normal distribution and is not a descriptive statistic.
Please rate this question:

Discuss and give feedback


Next question

Descriptive statistics

Descriptive statistics include a point estimate of the measured variable as well as a measure of the
variability of the data around that point estimate. Typical examples of point estimates include; mean,
median and mode. The two most commonly employed measurements of variability include standard
deviation and the inter quartile range. The standard deviation is usually considered in association
with the mean, while the inter quartile range is used alongside the median. Other measures of data
variability include the standard error of the mean and confidence interval. The standard error of the
mean represents the measure of variation around the point estimate of the mean of a group of
sample means, as such it should only be used when describing the characteristics of more than one
sample.
Next question
Theme: Use of suture materials and closure devices

A. Silk 3/0

B. Polyglactin 3/0

C. Polydioxanone 1/0

D. Stainless steel skin clips

E. Stainless steel wire 1/0

F. 6/0 Polypropylene

G. 3/0 Undyed polyglactin

H. Polypropylene 3/0

Please select the most appropriate suture material for the situation described. Each option may be
used once, more than once or not at all.

3. Mass closure of abdominal wall following elective right hemicolectomy through a midline incision.

You answered Silk 3/0

The correct answer is Polydioxanone 1/0

PDS or polydioxanone is the ideal suture material. Non absorbable sutures have higher incidence of
incisional herniae.

4. Closure of the sternum following coronary artery bypass grafting.

You answered Silk 3/0

The correct answer is Stainless steel wire 1/0

Stainless steel wire is typically used.


5. Application of vein patch to femoral artery following endarterectomy.

You answered Silk 3/0

The correct answer is 6/0 Polypropylene

Polypropylene is the suture of choice. Fine sutures are preferred.

Please rate this question:

Discuss and give feedback

Next question

Suture material

Suture materials

Agent Classification Durability Uses Special points

Silk Braided Theoretically Anchoring devices, skin Knots easily, poor


Biological permanent closure cosmesis
although
strength not
preserved

Catgut Braided 5-7 days Short term wound Poor cosmesis


Biological approximation Degrades rapidly
Not available in UK

Chromic catgut Braided Up to 12 weeks Apposition of deeply Unpredictable


Biological sited tissues degradation pattern
Agent Classification Durability Uses Special points

Not in use in UK

Polydiaxonone Synthetic Up to 3 months Widespread surgical Used in most surgical


(PDS) Monofilament (longer with applications including specialties (avoid dyed
thicker sutures) visceral anastomoses, form in dermal
dermal closure, mass closure)
closure of abdominal
wall

Polyglycolic acid Braided Up to 6 weeks Most tissues can be It has good handling
(Vicryl, Dexon) Synthetic apposed using properties, the dyed
polyglycolic acid form of this suture
should not be used for
skin closure

Polypropylene Synthetic Permanent Widely used, agent of Poor handling


(Prolene) Monofilament choice for vascular properties
anastomoses

Polyester Synthetic Permanent Its combination of It is more expensive


(Ethibond) Braided permanency and and has considerable
braiding makes it useful tissue drag
for laparoscopic surgery

Absorbable vs Non absorbable

 Time taken to degrade absorbable materials varies


 Usually by macrophages hydrolysing material
 Consider absorbable sutures in situations where long term tissue apposition is not required.
In cardiac and vascular surgery non absorbable sutures are usually used.

Suture size
 The higher the index number the smaller the suture i.e. : 6/0 prolene is finer than 1/0 prolene.
 Finer sutures have less tensile strength. For example 6/0 prolene would not be a suture
suitable for abdominal mass closure but would be ideal for small calibre distal arterial
anastomoses.

Braided vs monofilament
Generally speaking braided sutures have better handling characteristics than non braided. However,
they are associated with higher bacterial counts. Braided materials are unsuitable for use in vascular
surgery as they are potentially thrombogenic.
Next question
Which of the following visceral anastomoses has the lowest risk of anastomotic leak? You may
assume that all are constructed in ideal circumstances.

Stapled ileocolic anastomosis

Hand sewn anastomosis of the proximal ileum

Stapled colorectal anastomosis defunctioned with loop ileostomy

Stapled colorectal anastomosis defunctioned with loop colostomy

Hand sewn oesophagojejunal anastomosis

Rectal and oesophageal surgery have some of the highest rates of anastomotic leakage. Following
anterior resection leaks are quoted to occur in up to 10% of cases. Small bowel anastomoses are
the most technically forgiving. Factors increasing the risk of anastamotic leakage include previous
irradiation, sepsis, malnutrition, poor blood supply and poor technique.
The defunctioning of rectal anastomoses may reduce the clinical impact of anastomotic leak and
make it amenable to percutaneous drainage, but does not necessarily reduce the incidence of leaks
themselves.
Please rate this question:

Discuss and give feedback


Next question

Anastomoses

 A wide variety of anastomoses are constructed in surgical practice. Essentially the term
refers to the restoration of luminal continuity. As such they are a feature of both abdominal
and vascular surgery.

Visceral anastomoses

For an anastomosis to heal three criteria need to be fulfilled:

 Adequate blood supply


 Mucosal apposition
 Minimal tension
When these are compromised the anastomosis may break down. Even in the best surgical hands
some anastomoses are more prone to dehiscence than others. Oesophageal and rectal
anastomoses are more prone to leakage and reported leak rates following oesophageal and rectal
surgery can be as high as 20%. This figure includes radiological leaks and those with a clinically
significant leak will be of a lower order of magnitude. As a rule small bowel anastomoses heal most
reliably.

The decision as to how best to achieve mucosal apposition is one for each surgeon. Some will prefer
the use of stapling devices as they are quicker to use, others will prefer to perform a sutured
anastomosis. The attention to surgical technique is more important than the method chosen and a
poorly constructed stapled anastomosis in thickened tissue is far more prone to leakage than a hand
sewn anastomosis in the same circumstances.

If an anastomosis looks unsafe then it may be best not to construct one at all. In colonic surgery this
is relatively clear cut and most surgeons would bring out an end colostomy. In situations such as
oesophageal surgery this is far more problematic and colonic interposition may be required in this
situation.

Vascular anastomoses

Most arterial surgery involving bypasses or aneurysm repairs will require construction of an arterial
anastomosis. Technique is important and for small diameter distal arterial surgery the intimal
hyperplasia resulting from a badly constructed anastomosis may render the whole operation futile
before the patient leaves hospital.

Some key points about vascular anastomoses:

 Always use non absorbable monofilament suture (e.g. Polypropylene).


 Round bodied needle.
 Correct size for anastamosis ( i.e. 6/0 prolene for bottom end of a femoro-distal bypass).
 Suture should be continuous and from inside to outside of artery to avoid raising an intimal
flap.

Next question
Theme: Management of skin wounds

A. Immediate split thickness skin graft


B. Delayed split thickness skin graft
C. Primary closure
D. Delayed primary closure
E. Compression bandages
F. Myocutaneous flap
G. Random free flap

For each of the following injury scenarios please select the most appropriate management. Each
option may be used once, more than once or not at all.

7. A 63 year old male is gardening when he trips and lands on a scythe. He sustains a deep
laceration of his lateral thigh, it measures 3cm depth by 7cm length, it penetrates down to
the bone, but no fracture is evident on imaging or examination. His co- morbidities include
type II diabetes mellitus (diet controlled) and polymyalgia rheumatica (takes regular low
dose prednisolone).

You answered Immediate split thickness skin graft

The correct answer is Delayed primary closure

Theme from September 2012 Exam


Wounds which are contaminated or have the potential to become so are unsafe for
immediate primary closure. The combination of diabetes and steroids makes wound
complications more likely. Despite his high risk a primary skin graft or flap is unlikely to be
a safer option. Either may be used at a later date in the event that delayed primary closure is
unsuccessful.

8. A 71 year old lady trips over and falls landing on her left shin. She sustains a large pretibial
laceration of her leg.

You answered Immediate split thickness skin graft

The correct answer is Delayed split thickness skin graft

Pretibial lacerations do not heal well. Simple apposition of skin edges almost always fails
due to poor quality dermal tissues and underlying haematoma. Debridement of devitalised
tissues prior to grafting usually gives the best results.

9. A 73 year old lady presents with an ulcer overlying her medial malleolus. It is painless and
has been present for 4 months. She has oedema of the lower limbs and her ABPI measures
0.9.
You answered Immediate split thickness skin graft

The correct answer is Compression bandages

This is likely to be a venous leg ulcer. These are typically managed using compression
bandages. Contra indications to this technique include peripheral vascular disease (not
present here).

Please rate this question:

Discuss and give feedback


Next question

Methods of wound closure

Method of Indication
closure

Primary closure  Clean wound, usually surgically created or following minor trauma
 Standard suturing methods will usually suffice
 Wound heals by primary intention

Delayed primary  Similar methods of actual closure to primary closure


closure  May be used in situations where primary closure is either not
achievable or not advisable e.g. infection

Vacuum assisted  Uses negative pressure therapy to facilitate wound closure


closure  Sponge is inserted into wound cavity and then negative pressure
applied
 Advantages include removal of exudate and versatility
 Disadvantages include cost and risk of fistulation if used incorrectly
on sites such as bowel

Split thickness  Superficial dermis removed with Watson knife or dermatome


skin grafts (commonly from thigh)
 Remaining epithelium regenerates from dermal appendages
 Coverage may be increased by meshing
Full thickness skin  Whole dermal thickness is removed
grafts  Sub dermal fat is then removed and graft placed over donor site
 Better cosmesis and flexibility at recipient site
 Donor site "cost"

Flaps  Viable tissue with a blood supply


 May be pedicled or free
 Pedicled flaps are more reliable, but limited in range
 Free flaps have greater range but carry greater risk of breakdown as
they require vascular anastomosis

Next question
Theme: Surgical energy devices

A. Monopolar diathermy
B. Bipolar diathermy
C. CUSA device
D. Argon plasma coagulation device
E. Ligasure device
F. Monopolar device in cutting mode
G. Monopolar device in coagulation mode
H. Monopolar device in blend mode

Please select the most appropriate surgical energy device for the procedure described. Each option
may be used once, more than once or not at all.

10. Posterior dissection of the thyroid gland during total thyroid lobectomy

You answered Monopolar diathermy

The correct answer is Bipolar diathermy

This will minimise thermal trauma to the recurrent laryngeal nerve

11. Undertaking a snare polypectomy for a villous adenoma of the descending colon

You answered Monopolar diathermy

The correct answer is Monopolar device in blend mode

Blend applies a mixture of coagulation and cutting modes to achieve smooth polypectomy

12. Dissection of temporal lobe for tumour

You answered Monopolar diathermy

The correct answer is CUSA device

The ultrasonic dissector is the preferred tool for this. It is also extensively used in liver
resections

Please rate this question:

Discuss and give feedback


Next question

Diathermy

 Diathermy devices are used by surgeons in all branches of surgery.


 Use electric currents to produce local heat and thereby facilitate haemostasis or surgical
dissection.
 Consist of a generator unit that is located outside the patient and can be set to the level of
power required by the surgeon.
 There are two major types of diathermy machine;

Monopolar
The current flows through the diathermy unit into a handheld device that is controlled by the
surgeon. Electricity can flow from the tip of the device into the patient. The earth electrode is located
some distance away. The relatively narrow tip of the diathermy device produces local heat and this
can be used to vaporise and fulgurate tissues. The current can be adjusted in terms of frequency so
that different actions can be effected. In cutting mode sufficient power is applied to the tissues to
vaporise their water content. In coagulation mode the power level is reduced so that a coagulum is
formed instead. Some diathermy machines can utilise a setting known as blend that alternates
cutting and coagulation functions, these tend to be used during procedures such as colonoscopic
polypectomy.

Bipolar
The electric current flows from one electrode to another however, both electrodes are usually
contained within the same device e.g. a pair of forceps. The result is that heating is localised to the
area between the two electrodes and surrounding tissue damage is minimised.

Ultrasound based devices


These include CUSA and Harmonic scalpel. They generate high frequency oscillations that seal and
coagulate tissues. They have different energy settings that allow them to dissect and simultaneously
seal vessels if required. The CUSA device leaves vessels intact that may then be divided.

Ligasure device
Delivers tailored energy levels to allow simultaneous haemostasis and dissection. The device
senses the impedance of the tissues and tailors energy levels accordingly.

Hazards of diathermy

 Inadvertent patient burn. This may result of careless handling of the device or in the case of
monopolar devices forgetting to apply a return electrode plate, In this situation patients may
develop a contact burn when electricity flows to earth
 Explosion or fire. This may occur when volatile anaesthetic gases or skin preparation fluid
have been used

Next question
Theme: Surgical drains

A. Redivac suction drain


B. Corrugated drain
C. Wallace Robinson drain (non suction)
D. Penrose tubing
E. Latex T Tube drain
F. Silastic T Tube drain
G. No drain

Please select the most appropriate surgical drainage system for the indication given. Each option
may be used once, more than once or not at all.

13. A 56 year old lady undergoes an open cholecystectomy and exploration of common bile
duct. The bile duct is closed over a drain.

You answered Redivac suction drain

The correct answer is Latex T Tube drain

This will elicit a fibrotic response and encourage a track to form.

14. A 48 year old lady undergoes a mastectomy and axillary node clearance for an invasive
ductal cancer of the breast with lymph node metastasis.

Redivac suction drain

The raw tissue exposed from the mastectomy site will often ooze serous fluid and may
result in seroma formation when the drain is removed.

15. A 75 year old man undergoes a Hartman's procedure for sigmoid diverticular disease with
pericolic abscess and colovesical fistula.

You answered Redivac suction drain

The correct answer is Wallace Robinson drain (non suction)

A non suction drain is the preferred option here.

Please rate this question:

Discuss and give feedback


Next question

Surgical drains

 Drains are inserted in many surgical procedures and are of many types.
 As a broad rule they can be divided into those using suction and those which do not.
 The diameter of the drain will depend upon the substance being drained, for example smaller
lumen drain for pneumothoraces vs haemothorax.
 Drains can be associated with complications and these begin with insertion when there may
be iatrogenic damage. When in situ they serve as a route for infections. In some specific
situations they may cause other complications, for example suction drains left in contact with
bowel for long periods may carry a risk of inducing fistulation.
 Drains should be inserted for a defined purpose and removed once the need has passed.

A brief overview of types of drain and sites is given below

CNS

 Low suction drain or free drainage systems may be used for situations such as drainage of
sub dural haematomas.

CVS

 Following cardiothoracic procedures of thoracic trauma underwater seal drains are often
placed. These should be carefully secured. When an air leak is present they may be placed
on suction whilst the air leak settles

Orthopaedics and trauma

 In this setting drains are usually used to prevent haematoma formation (with associated risk
of infection). Some orthopaedic drains may also be specially adapted to allow the drained
blood to be auto transfused.

Gastro-intestinal surgery

 Surgeons often place abdominal drains either to prevent or drain abscesses, or to turn an
anticipated complication into one that can be easily controlled such as a bile leak following
cholecystectomy. The type of drain used will depend upon the indication.

Drain types
Type of drain Features

Redivac  Suction type of drain


 Closed drainage system
 High pressure vacuum system

Low pressure  Consist of small systems such as the lantern style drain that may be
drainage systems used for short term drainage of small wounds and cavities
 Larger systems are sometimes used following abdominal surgery,
they have a lower pressure than the redivac system, which decreases
the risks of fistulation
 May be emptied and re-pressurised

Latex tube drains  May be shaped (e.g. T Tube) or straight


 Usually used in non pressurised systems and act as sump drains
 Most often used when it is desirable to generate fibrosis along the
drain track (e.g. following exploration of the CBD)

Chest drains  May be large or small diameter (depending on the indication)


 Connected to underwater seal system to ensure one way flow of air

Corrugated drain  Thin, wide sheet of plastic, usually soft


 Contains corrugations, along which fluids can track

Next question
Which of the following local anaesthetics is not an amino amide type?

Lignocaine

Xylocaine

Procaine

Bupivicaine

Prilocaine

All local anaesthetics have a chemical bond linking an amine to either an amide or an ester. Most
local anaesthetics are of the amino- amide types, these have a more favorable side effect profile and
are more stable in solution. Procaine and benzocaine have amino - ester groups, these are
metabolised by pseudocholinesterases.
Please rate this question:

Discuss and give feedback


Next question

Local anaesthetic agents

Lidocaine

 An amide
 Local anaesthetic and a less commonly used antiarrhythmic (affects Na channels in the
axon)
 Hepatic metabolism, protein bound, renally excreted
 Toxicity: due to IV or excess administration. Increased risk if liver dysfunction or low protein
states. Note acidosis causes lidocaine to detach from protein binding.
 Drug interactions: Beta blockers, ciprofloxacin, phenytoin
 Features of toxicity: Initial CNS over activity then depression as lidocaine initially blocks
inhibitory pathways then blocks both inhibitory and activating pathways. Cardiac arrhythmias.
 Increased doses may be used when combined with adrenaline to limit systemic absorption.

Cocaine
 Pure cocaine is a salt, usually cocaine hydrochloride. It is supplied for local anaesthetic
purposes as a paste.
 It is supplied for clinical use in concentrations of 4 and 10%. It may be applied topically to the
nasal mucosa. It has a rapid onset of action and has the additional advantage of causing
marked vasoconstriction.
 It is lipophillic and will readily cross the blood brain barrier. Its systemic effects also include
cardiac arrhythmias and tachycardia.
 Apart from its limited use in ENT surgery it is otherwise used rarely in mainstream surgical
practice.

Bupivacaine

 Bupivacaine binds to the intracellular portion of sodium channels and blocks sodium influx
into nerve cells, which prevents depolarization.
 It has a much longer duration of action than lignocaine and this is of use in that it may be
used for topical wound infiltration at the conclusion of surgical procedures with long duration
analgesic effect.
 It is cardiotoxic and is therefore contra indicated in regional blockage in case the tourniquet
fails.
 Levobupivicaine (Chirocaine) is less cardiotoxic and causes less vasodilation.

Prilocaine

 Similar mechanism of action to other local anaesthetic agents. However, it is far less
cardiotoxic and is therefore the agent of choice for intravenous regional anaesthesia e.g.
Biers Block.

All local anaesthetic agents dissociate in tissues and this contributes to their therapeutic effect. The
dissociation constant shifts in tissues that are acidic e.g. where an abscess is present, and this
reduces the efficacy.

Doses of local anaesthetics


Agent Dose plain Dose with adrenaline

Lignocaine 3mg/Kg 7mg/Kg

Bupivacaine 2mg/Kg 2mg/Kg

Prilocaine 6mg/Kg 9mg/Kg


These are a guide only as actual doses depend on site of administration, tissue vascularity and co-
morbidities.

Maximum total local anaesthetic doses

 Lignocaine 1% plain - 3mg/ Kg - 200mg (20ml)


 Lignocaine 1% with 1 in 200,000 adrenaline - 7mg/Kg - 500mg (50ml)
 Bupivicaine 0.5% - 2mg/kg- 150mg (30ml)

Maximum doses are based on ideal body weight

Effects of adrenaline
Adrenaline may be added to local anaesthetic drugs. It prolongs the duration of action at the site of
injection and permits usage of higher doses (see above). It is contra indicated in patients taking
MAOI's or tricyclic antidepressants. The toxicity of bupivacaine is related to protein binding and
addition of adrenaline to this drug does not permit increases in the total dose of bupivacaine, in
contrast to the situation with lignocaine.

References
An excellent review is provided by:
French J and Sharp L. Local Anaesthetics. Ann R Coll Surg Engl 2012; 94: 76-80.
Next question
Theme: Biological therapies

A. Bevacizumab

B. Infliximab

C. Trastuzumab

D. Basiliximab

E. Imatinib

F. Cetuximab

Please select the most appropriate biological agent for the situation described. Each option may be
used once, more than once or not at all.

17. A 32 year old lady has previously undergone a wide local excision and axillary node clearance (5
nodes positive) for an invasive ductal carcinoma. It is oestrogen receptor negative, HER 2 positive,
vascular invasion is present. She has a lesion suspicious for metastatic disease in the left lobe of
her liver.

You answered Bevacizumab

The correct answer is Trastuzumab

This ladies young age, coupled with ER negativity and extensive nodal disease with suspicion of
metastatic disease makes her a candidate for treatment with trastuzumab (herceptin).

18. A 22 year old lady has severe peri anal crohns disease with multiple anal fistulae, the acute sepsis
has been drained and setons are in place. She is already receiving standard non biological
therapy.

You answered Bevacizumab

The correct answer is Infliximab

Infliximab is a popular choice in managing complex peri anal crohns. It is absolutely vital that all
sepsis is drained prior to starting therapy.

19. A 63 year old man presents with a locally unresectable gastrointestinal stromal tumour. Biopsies
confirm that it is KIT positive.

You answered Bevacizumab

The correct answer is Imatinib

Imatinib is licensed for treatment of GIST in the United Kingdom for this situation. The guidance
from the National Institute of Clinical evidence is that patients be reviewed at 12 weeks after
initiating therapy.

Please rate this question:

Discuss and give feedback

Next question

Biological agents

Agents Target Uses

Adalimumab TNF alpha inhibitor Crohns disease


Infliximab Rheumatoid disease
Etanercept

Bevacizumab Anti VEGF (anti angiogenic) Colorectal cancer


Renal
Glioblastoma
Agents Target Uses

Trastuzumab HER receptor Breast cancer

Imatinib Tyrosine kinase inhibitor Gastrointestinal stromal tumours


Chronic myeloid leukaemia

Basiliximab IL2 binding site Renal transplants

Cetuximab Epidermal growth factor inhibitor EGF positive colorectal cancers

Detailed understanding of the actions of biological agents is well beyond the scope of the MRCS
syllabus. However, many of these drugs are being frequently encountered in surgical patients.

Next question
Which of the following sutures has the largest diameter?

6/0 Polypropylene

5/0 Silk

3/0 Nylon

1 Polypropylene

0 Polydiaxone

The sizes of suture material are not related to the composition of the suture material.

Please rate this question:

Discuss and give feedback

Next question

Suture sizes

USP Suture size and corresponding suture diameter

USP Size Diameter in mm

11-0 0.01

10-0 0.02
6-0 0.07

3-0 0.2

0 0.35

1 0.4

Next question
Theme: Management of bleeding

A. Ligate vessel

B. Underrun vessel

C. Use of diathermy

D. Application of surgicell

E. Digital pressure

In each of the following scenarios the surgeon has encountered bleeding. Please select the most
appropriate immediate management of the situation from the list below. Each option may be used
once, more than once or not at all.

21. A 23 year old man is undergoing an open appendicectomy. The surgeons extend the incision
medially and suddenly encounter troublesome bleeding.

Ligate vessel

Theme from April 2012 Exam


Medial extension of an appendicectomy incision carries the risk of injury to the inferior epigastric
artery. This can bleed briskly and is best managed by ligation.

22. A 45 year old man is undergoing a laparotomy and following incision of the skin multiple bleeding
points are identified in the dermis and sub dermal tissues.

You answered Ligate vessel

The correct answer is Use of diathermy

Multiple bleeding points are best managed through the use of diathermy.

23. A 38 year old lady is undergoing a laparotomy when the surgeons damage the common iliac vein
whilst commencing a pelvic dissection.
You answered Ligate vessel

The correct answer is Digital pressure

Major venous bleeding such as this should be controlled with digital pressure in the first instance.
The definitive management will usually consist of suturing the defect closed with prolene sutures.
Transection of the common iliac vein will necessitate a major venous reconstruction.

Please rate this question:

Discuss and give feedback

Next question

Management of bleeding

Bleeding is a process that is encountered in all branches of surgery. The decision as to how best to
manage bleeding depends upon the site, vessel and circumstances.

Management of superficial dermal bleeding


This will usually cease spontaneously. If it is troublesome then direct use of monopolar or bipolar
cautery devices will usually control the situation. Scalp wounds are a notable exception and the
bleeding from these may be brisk. In this situation the use of mattress sutures as a wound closure
method will usually address the problem.

Superficial arterial bleeding


If the vessel can be safely identified then the easiest method is to apply a haemostatic clip and ligate
the vessel.

Major arterial bleeding


If the vessel can be clearly identified and is accessible then it may be possible to apply a clip and
ligate the vessel. If the vessel is located in a pool of blood then blind application of haemostatic clips
is highly dangerous and may result in collateral injury. In this situation evacuating the clot and
packing the area is often safer. The pack can then be carefully removed when the required
instruments are available. Some vessels may retract and bleeding may then be controlled by
dissection of surrounding structures or under-running the bleeding point.

Major venous bleeding


The safest initial course of action is to apply digital pressure to the bleeding point. To control the
bleeding the surgeon will need a working suction device. Divided veins may require ligation.
Incomplete lacerations of major veins (e.g. IVC) are best repaired. In order to do this it is safest to
apply a Satinsky type vascular clamp and repair the defect with 5/0 prolene.

Bleeding from raw surfaces


This may be mixed bleeding and can be troublesome. Spray diathermy and argon plasma
coagulation are both useful agents. Certain topical haemostatic agents such as surgicell are useful
in encouraging clot formation and may be used in conjunction with, or instead of, the above agents.

Next question
A 7 year old boy is due to undergo a circumcision for phimosis. Which of the following devices would
be the most appropriate agent to use for achieving haemostasis?

Monopolar unit in cutting mode

Bipolar unit

Monopolar unit in coagulation mode

Monopolar unit in blend mode

Monopolar unit configured to spray mode

The danger with the use of any source other than bipolar diathermy in this setting is the risk of
causing trauma to end vessels. All the monopolar units, regardless of the setting will carry this risk.
Please rate this question:

Discuss and give feedback


Next question

Diathermy

 Diathermy devices are used by surgeons in all branches of surgery.


 Use electric currents to produce local heat and thereby facilitate haemostasis or surgical
dissection.
 Consist of a generator unit that is located outside the patient and can be set to the level of
power required by the surgeon.
 There are two major types of diathermy machine;

Monopolar
The current flows through the diathermy unit into a handheld device that is controlled by the
surgeon. Electricity can flow from the tip of the device into the patient. The earth electrode is located
some distance away. The relatively narrow tip of the diathermy device produces local heat and this
can be used to vaporise and fulgurate tissues. The current can be adjusted in terms of frequency so
that different actions can be effected. In cutting mode sufficient power is applied to the tissues to
vaporise their water content. In coagulation mode the power level is reduced so that a coagulum is
formed instead. Some diathermy machines can utilise a setting known as blend that alternates
cutting and coagulation functions, these tend to be used during procedures such as colonoscopic
polypectomy.

Bipolar
The electric current flows from one electrode to another however, both electrodes are usually
contained within the same device e.g. a pair of forceps. The result is that heating is localised to the
area between the two electrodes and surrounding tissue damage is minimised.

Ultrasound based devices


These include CUSA and Harmonic scalpel. They generate high frequency oscillations that seal and
coagulate tissues. They have different energy settings that allow them to dissect and simultaneously
seal vessels if required. The CUSA device leaves vessels intact that may then be divided.

Ligasure device
Delivers tailored energy levels to allow simultaneous haemostasis and dissection. The device
senses the impedance of the tissues and tailors energy levels accordingly.

Hazards of diathermy

 Inadvertent patient burn. This may result of careless handling of the device or in the case of
monopolar devices forgetting to apply a return electrode plate, In this situation patients may
develop a contact burn when electricity flows to earth
 Explosion or fire. This may occur when volatile anaesthetic gases or skin preparation fluid
have been used

Next question
If a 2 x 2 cm autologus skin graft is placed on an area of healthy granulation tissue. After about a
week, a thin bluish - white margin appears around the graft and spreads at a rate of 1mm per day.
What is it?

Epidermis alone

Epidermis and dermis

Dermis alone

Inflammatory exudate

Fibrin

This is the process of re-epithelialisation.

Please rate this question:

Discuss and give feedback

Next question

Wound healing

Surgical wounds are either incisional or excisional and either clean, clean contaminated or dirty.
Although the stages of wound healing are broadly similar their contributions will vary according to the
wound type.

The main stages of wound healing include:

Haemostasis

 Minutes to hours following injury


 Vasospasm in adjacent vessels, platelet plug formation and generation of fibrin rich clot.
Inflammation

 Typically days 1-5


 Neutrophils migrate into wound (function impaired in diabetes).
 Growth factors released, including basic fibroblast growth factor and vascular endothelial
growth factor.
 Fibroblasts replicate within the adjacent matrix and migrate into wound.
 Macrophages and fibroblasts couple matrix regeneration and clot substitution.

Regeneration

 Typically days 7 to 56
 Platelet derived growth factor and transformation growth factors stimulate fibroblasts and
epithelial cells.
 Fibroblasts produce a collagen network.
 Angiogenesis occurs and wound resembles granulation tissue.

Remodeling

 From 6 weeks to 1 year


 Longest phase of the healing process and may last up to one year (or longer).
 During this phase fibroblasts become differentiated (myofibroblasts) and these facilitate
wound contraction.
 Collagen fibres are remodeled.
 Microvessels regress leaving a pale scar.

The above description represents an idealised scenario. A number of diseases may distort this
process. Neovascularisation is an important early process. Endothelial cells may proliferate in the
wound bed and recanalise to form a vessel. Vascular disease, shock and sepsis can all compromise
microvascular flow and impair healing.

Conditions such as jaundice will impair fibroblast synthetic function and immunity with a detrimental
effect in most parts of the healing process.

Problems with scars:

Hypertrophic scars
Excessive amounts of collagen within a scar. Nodules may be present histologically containing
randomly arranged fibrils within and parallel fibres on the surface. The tissue itself is confined to the
extent of the wound itself and is usually the result of a full thickness dermal injury. They may go on
to develop contractures.
Image of hypertrophic scarring. Note that it remains confined to the boundaries of the original
wound:

Image sourced from Wikipedia

Keloid scars
Excessive amounts of collagen within a scar. Typically a keloid scar will pass beyond the boundaries
of the original injury. They do not contain nodules and may occur following even trivial injury. They
do not regress over time and may recur following removal.

Image of a keloid scar. Note the extension beyond the boundaries of the original incision:
Image sourced from Wikipedia

Drugs which impair wound healing:

 Non steroidal anti inflammatory drugs


 Steroids
 Immunosupressive agents
 Anti neoplastic drugs

Closure
Delayed primary closure is the anatomically precise closure that is delayed for a few days but before
granulation tissue becomes macroscopically evident.

Secondary closure refers to either spontaneous closure or to surgical closure after granulation tissue
has formed.
Next question
Which of the following methods is most effective at destroying spores of the tubercle bacilli?

Immersion in 0.5% chlorhexidine in alcohol

Immersion in aqueous iodine

Heating in a hot air oven

Immersion in 0.1% sodium hypochlorite

Autoclaving

The tubercle bacilli has a waxy outer membrane that renders it more resistant to sterilisation and
cleaning methods. Whilst 0.1% sodium hypochlorite will destroy many microbes it is less reliable in
destroying tubercle bacilli. Hot air ovens provide less reliable pathogen destruction than autoclaving,
but may be indicated in situations where the equipment is sensitive to the autoclaving process. From
the list of options above, autoclaving will most reliably destroy tubercle bacilli.

Please rate this question:

Discuss and give feedback

Next question

Sterilisation

Surgical equipment has to be cleaned and sterilised prior to use. The extent to which these
processes will be required varies according to the type of equipment and the purpose for which it will
be used. In general, the three processes are relevant; cleaning, disinfection and sterilisation.

 Cleaning refers to removal of physical debris.


 Disinfection refers to reduction in numbers of viable organisms.
 Sterilisation is removal of all organisms and spores.
Methods

Method Details Indication

Autoclaving Air removed and high pressure Most reusable surgical equipment, must be
steam used (usually 134 oC for physically cleaned prior to autoclaving, unsuitable
3 minutes) for fragile items

Glutaraldehyde Colourless oily liquid, directly Specifically used for endoscopes and some
solution (2%) cytocidal and virucidal even at laparoscopic items, staff can rapidly develop
low temperatures allergy to this substance which has limited its more
widespread use

Ethylene oxide 3% mixture of gas with carbon Used for packaged materials that cannot be
dioxide used heated, the gas is explosive and environmentally
toxic, it is used mainly in the industrial setting

Gamma irradiation Gamma rays emitted from Suitable for batch treatment of relatively
radioactive substance such as thermostable items, typically an industrial process
cobalt 60 or caesium 137

Next question
Which of the following is a permanent suture material best suited for interrupted mattress dermal
closure?

2/0 Polydiaxone

3/0 Polydiaxone

4/0 Polyglycolic acid

1/0 Dexon

3/0 Polypropylene

Of the sutures listed only prolene is a permanent suture material. It is a good agent for skin closure
as it does not incite an inflammatory response and thus provides good cosmesis.

Please rate this question:

Discuss and give feedback

Next question

Suture material

Suture materials

Agent Classification Durability Uses Special points

Silk Braided Theoretically Anchoring devices, skin Knots easily, poor


Biological permanent closure cosmesis
although
strength not
Agent Classification Durability Uses Special points

preserved

Catgut Braided 5-7 days Short term wound Poor cosmesis


Biological approximation Degrades rapidly
Not available in UK

Chromic catgut Braided Up to 12 weeks Apposition of deeply Unpredictable


Biological sited tissues degradation pattern
Not in use in UK

Polydiaxonone Synthetic Up to 3 months Widespread surgical Used in most surgical


(PDS) Monofilament (longer with applications including specialties (avoid dyed
thicker sutures) visceral anastomoses, form in dermal
dermal closure, mass closure)
closure of abdominal
wall

Polyglycolic acid Braided Up to 6 weeks Most tissues can be It has good handling
(Vicryl, Dexon) Synthetic apposed using properties, the dyed
polyglycolic acid form of this suture
should not be used for
skin closure

Polypropylene Synthetic Permanent Widely used, agent of Poor handling


(Prolene) Monofilament choice for vascular properties
anastomoses

Polyester Synthetic Permanent Its combination of It is more expensive


(Ethibond) Braided permanency and and has considerable
braiding makes it useful tissue drag
for laparoscopic surgery
Absorbable vs Non absorbable

 Time taken to degrade absorbable materials varies


 Usually by macrophages hydrolysing material
 Consider absorbable sutures in situations where long term tissue apposition is not required.
In cardiac and vascular surgery non absorbable sutures are usually used.

Suture size

 The higher the index number the smaller the suture i.e. : 6/0 prolene is finer than 1/0 prolene.
 Finer sutures have less tensile strength. For example 6/0 prolene would not be a suture
suitable for abdominal mass closure but would be ideal for small calibre distal arterial
anastomoses.

Braided vs monofilament
Generally speaking braided sutures have better handling characteristics than non braided. However,
they are associated with higher bacterial counts. Braided materials are unsuitable for use in vascular
surgery as they are potentially thrombogenic.
Next question
Theme: Suture materials

A. Silk 3/0

B. Polyglactin 3/0

C. Polydiaxone 1/0

D. Stainless steel skin clips

E. Stainless steel wire 1/0

F. 6/0 Polypropylene

G. 1/0 Undyed polyglactin

H. Polypropylene 3/0

Please select the most appropriate suture for the scenario given. Each option may be used once,
more than once or not at all.

28. Anastomosis of ileum to transverse colon following right hemicolectomy.

You answered Silk 3/0

The correct answer is Polyglactin 3/0

3/0 PDS would be an alternative, as would linear stapler but those are not in the list.

29. Distal anastomosis in a femorodistal bypass using vein.

You answered Silk 3/0

The correct answer is 6/0 Polypropylene

Arterial anastomoses should be constructed using polypropylene. In this case a fine suture
material such as 6/0 is indicated.
30. Closure of skin following thyroidectomy for Graves disease.

You answered Silk 3/0

The correct answer is Stainless steel skin clips

Although some use sub cuticular stitches skin clips remain the standard of many. In the event of
post operative haematoma causing respiratory obstruction, they are easier to remove.

Please rate this question:

Discuss and give feedback

Next question

Suture material

Suture materials

Agent Classification Durability Uses Special points

Silk Braided Theoretically Anchoring devices, skin Knots easily, poor


Biological permanent closure cosmesis
although
strength not
preserved

Catgut Braided 5-7 days Short term wound Poor cosmesis


Biological approximation Degrades rapidly
Not available in UK

Chromic catgut Braided Up to 12 weeks Apposition of deeply Unpredictable


degradation pattern
Agent Classification Durability Uses Special points

Biological sited tissues Not in use in UK

Polydiaxonone Synthetic Up to 3 months Widespread surgical Used in most surgical


(PDS) Monofilament (longer with applications including specialties (avoid dyed
thicker sutures) visceral anastomoses, form in dermal
dermal closure, mass closure)
closure of abdominal
wall

Polyglycolic acid Braided Up to 6 weeks Most tissues can be It has good handling
(Vicryl, Dexon) Synthetic apposed using properties, the dyed
polyglycolic acid form of this suture
should not be used for
skin closure

Polypropylene Synthetic Permanent Widely used, agent of Poor handling


(Prolene) Monofilament choice for vascular properties
anastomoses

Polyester Synthetic Permanent Its combination of It is more expensive


(Ethibond) Braided permanency and and has considerable
braiding makes it useful tissue drag
for laparoscopic surgery

Absorbable vs Non absorbable

 Time taken to degrade absorbable materials varies


 Usually by macrophages hydrolysing material
 Consider absorbable sutures in situations where long term tissue apposition is not required.
In cardiac and vascular surgery non absorbable sutures are usually used.

Suture size
 The higher the index number the smaller the suture i.e. : 6/0 prolene is finer than 1/0 prolene.
 Finer sutures have less tensile strength. For example 6/0 prolene would not be a suture
suitable for abdominal mass closure but would be ideal for small calibre distal arterial
anastomoses.

Braided vs monofilament
Generally speaking braided sutures have better handling characteristics than non braided. However,
they are associated with higher bacterial counts. Braided materials are unsuitable for use in vascular
surgery as they are potentially thrombogenic.
Next question
Theme: Management of skin lesions

A. 5mm punch biopsy


B. Shave biopsy
C. Excisional biopsy
D. Wide excision of 5cm
E. Tru cut biopsy
F. Incisional biopsy

For the skin lesions described please select the most appropriate management option. Each option
may be used once, more than once or not at all.

31. An 83 year old lady presents with multiple patches of pigmented irregular, superficial
lesions over the torso. They do not bleed but have become increasingly itchy.

You answered 5mm punch biopsy

The correct answer is Shave biopsy

Theme from April 2012 Exam


This is most likely to be seborrhoeic warts. These are usually superficially sited and are
best managed with shave biopsy and cautery.

32. A 65 year old man presents with a 5cm ulcerated area over his medial malleolus.

5mm punch biopsy

This is likely to be a venous ulcer and should usually be managed with compression
bandaging if there is no arterial compromise. Long standing lesions may be complicated
by the development of malignancy and for this reason a punch biopsy of long standing or
non healing lesions is advisable.

33. A 23 year old lady presents with an itchy, bleeding pigmented lesion on her right thigh.

You answered 5mm punch biopsy

The correct answer is Excisional biopsy

This may represent a malignant melanoma. Complete excision is required to allow


accurate histological assessment. If the diagnosis is confirmed then re-excision of margins
may be required. Clearly if the lesion is benign then no further action is required.

Please rate this question:


Discuss and give feedback
Next question

Treatment of suspicious skin lesions

Skin lesions may be referred to surgeons for treatment or discovered incidentally. The table below
outlines the various therapeutic options:

Method Indication

Tru-cut Most often used for percutaneous sampling of deep seated lesions or used intra
biopsy operatively for visceral lesions

5mm punch Used for diagnostic confirmation of lesions that are suspected to be benign or
biopsy where the definitive management is unlikely to be surgical. Of limited usefulness in
pigmented lesions where they do not include sufficient tissue for accurate
diagnosis. May be used in non melanoma type skin disease to establish diagnosis
prior to more extensive resection.

Wide Where the complete excision of the lesion (with healthy margins) is the main
excision objective. In cosmetically sensitive sites, or where the defect is large, this may need
to be complemented with plastic surgical techniques

Incisional Used mainly for deep seated or extensive lesions where there is diagnostic doubt
biopsy (usually following core or tru-cut biopsy). Used rarely for skin lesions.

Diagnostic Primarily used for lesions that are suspicious for melanoma, the lesion is excised
excision with a rim of normal tissue. Excision of margins may be required subsequently.

Next question
Theme: Tissue sampling

A. Fine needle aspiration cytology


B. Surgical excision biopsy
C. Smear cytology
D. Core cut biopsy
E. Conventional surgical excision
F. Tru cut biopsy
G. Punch biopsy

Please select the most appropriate sampling method for the situation given. Each option may be
used once, more than once or not at all.

34. A 45 year old patient undergoes a CT scan of the abdomen and is noted to have a 6cm
mass in the right adrenal gland. Urinary catecholamines and other endocrine investigations
are negative. CT of the chest and remainder of the abdomen is otherwise normal.

You answered Fine needle aspiration cytology

The correct answer is Conventional surgical excision

Most surgeons would excise a mass of this size rather than attempt biopsy. Further
information relating to adrenal masses is covered under this topic.

35. A 67 year old lady is suspected of having Pagets disease of the nipple.

You answered Fine needle aspiration cytology

The correct answer is Punch biopsy

This is a relatively clear indication for a punch biopsy. If cellular atypia is present on
punch biopsy then any in situ malignancy should be considered. FNAC would be
unsuitable.

36. A 23 year old lady presents with a nodule in the right lobe of the thyroid. Examination of
the neck is otherwise unremarkable and clinically she is euthyroid. Imaging shows a solid
nodule at the site.

Fine needle aspiration cytology

FNAC is the first line investigation in this setting.Where as FNAC has declined in
popularity recently, it remain a very popular option in the investigation of thyroid masses.
It cannot reliably diagnose a follicular tumour.
Please rate this question:

Discuss and give feedback


Next question

Tissue sampling

Tissue sampling is an important surgical process. Biopsy modalities vary according to the site,
experience and subsequent planned therapeutic outcome

The modalities comprise:


-Fine needle aspiration cytology
-Core biopsy
-Excision biopsy
-Tru cut biopsy
-Punch biopsy
-Cytological smears
-Endoscopic or laparoscopic biopsy

When the lesion is superficial the decision needs to be taken as to whether complete excision is
desirable or whether excision biopsy is acceptable. In malignant melanoma for example the need for
safe margins will mean that a more radical surgical approach needs to be adopted after diagnostic
confirmation from excision biopsy than would be the case in basal cell carcinoma. Punch biopsies
are useful in gaining histological diagnosis of unclear skin lesions where excision biopsy is
undesirable such as in establishing whether a skin lesion is vasculitic or not.

Fine needle aspiration cytology (FNAC) is an operator dependent procedure that may or may not be
image guided and essentially involves passing a needle through a lesion whilst suction is applied to
a syringe. The material thus obtained is expressed onto a slide and sent for cytological assessment.
This test can be limited by operator inexperience and also by the lack of histological architectural
information (e.g. Follicular carcinoma of the thyroid). Where a discharge is present a sample may be
sent for cytology although in some sites (e.g. Nipple discharge ) the information gleaned may be
meaningless.

Tissue samples may be obtained by both core and tru cut biopsy. A core biopsy is obtained by use
of a spring loaded gun with a needle passing quickly through the lesion of interest. A tru cut biopsy
achieves the same objective but the needle moved by hand. When performing these techniques
image guidance may be desirable (e.g. In breast lesions). Consideration needs to be given to any
planned surgical resection as it may be necessary to resect the biopsy tract along with the specimen
(e.g. In sarcoma surgery).

Visceral lesions may be accessed percutaneously under image guidance such as ultrasound guided
biopsy of liver metastases. Or under direct vision such as a colonoscopic biopsy.
Next question
Theme: Management of wounds

A. Split thickness skin graft

B. Full thickness skin graft

C. Insertion of tissue expander at donor site and delayed split thickness skin
graft

D. Myocutaneous flap reconstruction (pedicled)

E. Direct primary closure

F. Delayed primary closure

Please select the most appropriate management for the wound described. Each option may be used
once, more than once or not at all.

37. A 34 year old man has a tissue defect measuring 3 cm by 1 cm following an excision of a lipoma
from the scapula.

You answered Split thickness skin graft

The correct answer is Direct primary closure

This wound should be amenable to primary closure. There is minimal associated tissue loss and
the surgery is minor and uncontaminated.

38. A 72 year old lady has a 4cm basal cell carcinoma excised from her right cheek. There is a
rhomboid defect measuring 4cm by 4cm.

You answered Split thickness skin graft

The correct answer is Full thickness skin graft

Facial wounds that are large and irregularly shaped are best managed with full thickness skin
grafts.
39. A 5 year old suffers 20% burns to the torso. On examination there is fixed pigmentation and the
affected area has a white and dry appearance.

Split thickness skin graft

This is a full thickness burn and will require split thickness skin grafting. Meshing the graft may
increase the donor site yield. However, this is at the expense of cosmesis.The burn itself must be
debrided first to ensure an adequate wound bed.

Please rate this question:

Discuss and give feedback

Next question

Tissue reconstruction

Skin Grafts and Flaps


Skin flaps or grafts may be required where primary wound closure cannot be achieved or would
entail either significant cosmetic defect or considerable functional disturbance as a result of wound
contraction.

Reconstructive ladder

Method Types

Direct closure The simplest option where possible

Grafting techniques  Split thickness


 Full thickness
 Skin Substitute
 Composite

Flap technique Local:


Method Types

 Transposition
 Pivot
 Alphabetplasty (e.g. Z-Y)

Regional:

 Myocutaneous
 Fasciocutaneous
 Neurocutaneous

Distant:

 Free tissue transfer

Prelamination Allows creation of specialised flaps e.g. buccal mucosa


techniques

Tissue expansion Involves placement of tissue expanders to increase amount of tissue at donor
sites

Skin Grafts Vs. Flaps

Skin Grafts Flaps

No size limit (Split)/ Relative size limit (full thickness) Size limited by territory of blood supply

Rely on wound bed for blood supply Tissue has its own blood supply

Take better on clean well vascularised wound beds Will survive independent of the wound bed
Skin Grafts Flaps

Split skin graft donor site typically heals in 12 days Direct closure of donor site or secondary skin graft

Donor site may be reused Donor site cannot be reused

Split thickness skin grafts

 Available in range of thicknesses.


 Thigh is the commonest donor site
 Size may be increased by meshing the graft. However this comes with compromise on
cosmesis.
 Donor sites, especially if thin grafts are taken can be reused following re-epithelialisation

Full thickness grafts

 Most commonly used for facial reconstruction


 Include dermal appendages
 Provide superior cosmetic result

Composite grafts
These are grafts containing more than one tissue type, such as skin and fat. They are usually used
to cover small defects in cosmetically important areas.

Flaps

 Flaps have their own blood supply and may be pedicled or free.
 May have multiple components e.g. skin, skin + fat, skin + fat + muscle.
 They will have the ability to take regardless of the underlying tissue bed.
 The type of intrinsic blood supply is important. For example in breast surgery pedicled
latissimus dorsi flaps will be less prone to failure than microsvascular anastomosed free Diep
flaps.

Next question
You have just completed a laparotomy for peritonitis due to a perforated peptic ulcer. What is the
best surgical strategy for avoidance of a complete abdominal wound dehisence?

Use of skin clips to close the skin rather than sub cuticular sutures

Careful approximation of the peritonum with non absorbable sutures

Mass closure of the midline wound using a 1/0 polydiaxone suture

Direct apposition of the rectus muscle rather than linea alba aponeurosis

Mass closure of the midline wound using a 3/0 polypropylene suture

The incidence of post operative wound dehisence is minimise by following Jenkins rule which
advocates mass closure of the midline wound. However, the suture strength is an important
consideration and 3/0 sutures do not have sufficient tensile strength. Both polydiaxone (PDS) and
polypropylene (Prolene) or nylon (Ethilon) are all equally suitable. Although separate closure of the
peritoneum was practised it has no bearing on the incidence of abdominal wound dehisence.

Please rate this question:

Discuss and give feedback

Next question

Abdominal wound dehiscence

 This is a significant problem facing all surgeons who undertake abdominal surgery on a
regular basis. Traditionally, it is said to occur when all layers of an abdominal mass closure
fail and the viscera protrude externally (associated with 30% mortality).
 It can be subdivided into superficial, in which the skin wound alone fails and complete,
implying failure of all layers.

Factors which increase the risk are:


* Malnutrition
* Vitamin deficiencies
* Jaundice
* Steroid use
* Major wound contamination (e.g. faecal peritonitis)
* Poor surgical technique (Mass closure technique is the preferred method-Jenkins Rule)

When sudden full dehiscence occurs the management is as follows:


* Analgesia
* Intravenous fluids
* Intravenous broad spectrum antibiotics
* Coverage of the wound with saline impregnated gauze (on the ward)
* Arrangements made for a return to theatre

Surgical strategy

 Correct the underlying cause (e.g. TPN or NG feed if malnourished)


 Determine the most appropriate strategy for managing the wound

Options

Resuturing of the This may be an option if the wound edges are healthy and there is enough tissue for
wound sufficient coverage. Deep tension sutures are traditionally used for this purpose.

Application of a This is a clear dressing with removable front. Particularly suitable when some
wound manager granulation tissue is present over the viscera or where there is a high output bowel
fistula present in the dehisced wound.

Application of a This is a clear plastic bag that is cut and sutured to the wound edges and is only a
'Bogota bag' temporary measure to be adopted when the wound cannot be closed and will
necessitate a return to theatre for definitive management.

Application of a These can be safely used BUT ONLY if the correct layer is interposed between the
VAC dressing suction device and the bowel. Failure to adhere to this absolute rule will almost
system invariably result in the development of multiple bowel fistulae and create an
extremely difficult management problem.

Next question
Theme: Instrument cleaning

A. Immersion in glutaraldehyde

B. Gamma irradiation

C. Autoclaving

D. Ethylene chloride

E. Phenolic lavage

F. Disposal of instrument

Please select the most appropriate cleaning method of instrument for the situation described. Each
option may be used once, more than once or not at all.

41. A company manufacturing scalpel blades wishes to sterilise them in bulk before use.

You answered Immersion in glutaraldehyde

The correct answer is Gamma irradiation

Theme from September 2013 Exam


Industry often uses gamma irradiation. It is not routinely used in hospitals

42. For sterilisation and cleaning of a colonoscope.

Immersion in glutaraldehyde

Washing systems using glutaraldehyde are often used although development of sensitivity in staff
is well known and it is used in closed systems

43. For cleaning instruments following a tonsillectomy in a patient who recieved human growth
hormone extract in 1981

You answered Immersion in glutaraldehyde


The correct answer is Disposal of instrument

High risk of prion disease mandates disposal on instruments which is often undertaken following
all tonsillectomy procedures regardless of level of percieved risk

Please rate this question:

Discuss and give feedback

Next question

Sterilisation

Surgical equipment has to be cleaned and sterilised prior to use. The extent to which these
processes will be required varies according to the type of equipment and the purpose for which it will
be used. In general, the three processes are relevant; cleaning, disinfection and sterilisation.

 Cleaning refers to removal of physical debris.


 Disinfection refers to reduction in numbers of viable organisms.
 Sterilisation is removal of all organisms and spores.

Methods
Method Details Indication

Autoclaving Air removed and high pressure Most reusable surgical equipment, must be
steam used (usually 134 oC for physically cleaned prior to autoclaving, unsuitable
3 minutes) for fragile items

Glutaraldehyde Colourless oily liquid, directly Specifically used for endoscopes and some
solution (2%) cytocidal and virucidal even at laparoscopic items, staff can rapidly develop
low temperatures allergy to this substance which has limited its more
widespread use
Method Details Indication

Ethylene oxide 3% mixture of gas with carbon Used for packaged materials that cannot be
dioxide used heated, the gas is explosive and environmentally
toxic, it is used mainly in the industrial setting

Gamma irradiation Gamma rays emitted from Suitable for batch treatment of relatively
radioactive substance such as thermostable items, typically an industrial process
cobalt 60 or caesium 137

Next question
Which of the following is not an absorbable suture material?

Chromic catgut

Nylon

Vicryl

Dexon

Poly diaxone (PDS).

Please rate this question:

Discuss and give feedback

Next question

Suture material

Suture materials

Agent Classification Durability Uses Special points

Silk Braided Theoretically Anchoring devices, skin Knots easily, poor


Biological permanent closure cosmesis
although
strength not
preserved
Agent Classification Durability Uses Special points

Catgut Braided 5-7 days Short term wound Poor cosmesis


Biological approximation Degrades rapidly
Not available in UK

Chromic catgut Braided Up to 12 weeks Apposition of deeply Unpredictable


Biological sited tissues degradation pattern
Not in use in UK

Polydiaxonone Synthetic Up to 3 months Widespread surgical Used in most surgical


(PDS) Monofilament (longer with applications including specialties (avoid dyed
thicker sutures) visceral anastomoses, form in dermal
dermal closure, mass closure)
closure of abdominal
wall

Polyglycolic acid Braided Up to 6 weeks Most tissues can be It has good handling
(Vicryl, Dexon) Synthetic apposed using properties, the dyed
polyglycolic acid form of this suture
should not be used for
skin closure

Polypropylene Synthetic Permanent Widely used, agent of Poor handling


(Prolene) Monofilament choice for vascular properties
anastomoses

Polyester Synthetic Permanent Its combination of It is more expensive


(Ethibond) Braided permanency and and has considerable
braiding makes it useful tissue drag
for laparoscopic surgery

Absorbable vs Non absorbable


 Time taken to degrade absorbable materials varies
 Usually by macrophages hydrolysing material
 Consider absorbable sutures in situations where long term tissue apposition is not required.
In cardiac and vascular surgery non absorbable sutures are usually used.

Suture size

 The higher the index number the smaller the suture i.e. : 6/0 prolene is finer than 1/0 prolene.
 Finer sutures have less tensile strength. For example 6/0 prolene would not be a suture
suitable for abdominal mass closure but would be ideal for small calibre distal arterial
anastomoses.

Braided vs monofilament
Generally speaking braided sutures have better handling characteristics than non braided. However,
they are associated with higher bacterial counts. Braided materials are unsuitable for use in vascular
surgery as they are potentially thrombogenic.
Next question
A 73 year old lady sustains a distal radius fracture and this is manipulated using a Biers block with
prilocaine as the local anaesthetic agent. During the procedure the occlusion cuff deflates and the
patient becomes progressively cyanosed. What is the treatment of choice?

Intravenous calcium gluconate

Exchange transfusion

Intravenous methylene blue

Intravenous sodium thiosulphate

Intravenous gelofusine

Prilocaine is a recognised cause of methaemoglobinaemia, this is characterised by the development


of cyanosis and dyspnoea. This disorder occurs because of the change haemoglobin to a ferric
subtype rather than ferrous (Fe2+). This type of change shifts the oxygen dissociation curve to the left
and tissue hypoxia occurs. Methylene blue will revert the haemoglobin to the ferrous type and
reverse this effect.

Please rate this question:

Discuss and give feedback

Next question

Local anaesthetic toxicity

Toxicity results from either accidental intravascular injection (rapid onset of symptoms-usually
correct dose), or from excessive dosage (slower onset). Local anaesthetic agents not only exert a
membrane stabilising effect on peripheral nerves but will also act on excitable membranes within the
CNS and Heart. The sensory neurones in the CNS are suppressed before the motor ones. As a
result the early symptoms will typically be those of circumoral paraesthesia and tinnitus, followed by
falling GCS and eventually coma.
Management of toxicity

 Stop injecting the anaesthetic agent


 High flow 100% oxygen via face mask
 Cardiovascular monitoring
 Administer lipid emulsion (Intralipid 20%) at 1.5ml/Kg over 1 minute as a bolus
 Consider lipid emulsion infusion, at 0.25ml/ Kg/ minute
 If toxicity due to prilocaine then administer methylene blue

Safe doses
10ml of lignocaine 1% contains 100mg of drug, this would constitute 70% of the maximum safe dose
in a 50 kg patient. Up to 7mg / kg can be administered if adrenaline is added to the solution.

Doses of local anaesthetics


Agent Dose plain Dose with adrenaline

Lignocaine 3mg/Kg 7mg/Kg

Bupivicane 2mg/Kg 2mg/Kg

Prilocaine 6mg/Kg 9mg/Kg

These are a guide only as actual doses depend on site of administration, tissue vascularity and co-
morbidities.

Next question
A 43 year old man has symptoms of carcinoid syndrome. Which of the following is the most effective
therapeutic agent in controlling the symptoms?

Atenolol

Octreotide

Glucagon

Somatostatin

Spironolactone

Theme from April 2012 Exam


Octreotide is the usual treatment for carcinoid syndrome. Somatostatin inhibits the release of a
number of gut hormones. Octreotide is the synthetic alternative to somatostatin and thus the most
appropriate therapeutic agent.

Please rate this question:

Discuss and give feedback

Next question

Carcinoid syndrome

 Carcinoid tumours secrete serotonin


 Originate in neuroendocrine cells mainly in the intestine (midgut-distal ileum/appendix)
 Can occur in the rectum, bronchi
 Hormonal symptoms mainly occur when disease spreads outside the bowel

Clinical features
 Onset: insidious over many years
 Flushing face
 Palpitations
 Pulmonary valve stenosis and tricuspid regurgitation causing dyspnoea
 Asthma
 Severe diarrhoea (secretory, persists despite fasting)

Investigation

 5-HIAA in a 24-hour urine collection


 Somatostatin receptor scintigraphy
 CT scan
 Blood testing for chromogranin A

Treatment

 Octreotide
 Surgical removal

Next question
A 43 year old lady is due to undergo a diagnostic laparoscopy. Which of the agents listed below
should be used for inducing pneumoperitoneum?

Argon

Helium

Air

Carbon dioxide

Nitrogen

Carbon dioxide is the agent of choice. It is rapidly re-absorbed, does not support combustion and is
cheap. It is rapidly cleared from the lungs and so effects on pH are unusual.
Please rate this question:

Discuss and give feedback


Next question

Gases for laparoscopic surgery

Laparoscopic surgery may be performed in a number of body cavities. In some areas irrigation
solutions are preferred. In the abdomen insufflation with carbon dioxide gas is commonly used. The
amount of gas delivered is adjusted to maintain a constant intra-abdominal pressure of between 12
and 15 mmHg. Excessive intra-abdominal pressure may reduce venous return and lead to
hypotension. Too little insufflation will risk obscuring the surgical view.
Next question
Theme: Levels of evidence

A. I
B. II
C. III
D. IV
E. V

Please select the level of evidence which is supplied by the following. Each option may be used
once, more than once or not at all.

48. One of the senior surgeons in the hospital advises as to the best management of Merkel
cell tumours of the skin in which she has a special interest.

You answered I

The correct answer is V

Personal expert opinion qualifies for level V evidence.

49. A group of surgeons review a meta-analysis of a series of randomised controlled trials on


the Cochrane database and decide that one type of hip replacement is superior to another.

A meta- analysis of more than one well designed trials will typically represent level I
evidence. It does, of course, depend on how well the trials were conducted and reported.

50. A group of surgeons are trying to decide which type of mesh to use for incisional hernia
repair. Their assimilated evidence includes two case series and one randomised controlled
trial.

You answered I

The correct answer is II

Data which includes at least one RCT will usually qualify for level II evidence.

Please rate this question:

Discuss and give feedback


Next question
Levels of evidence

The level of evidence refers to the study design used by investigators to minimise bias.

Level of Source
evidence

I Evidence obtained from systematic review of all relevant randomised controlled


trials

II Evidence derived from at least one properly designed randomised controlled trial

III Evidence derived from well designed pseudo-randomised controlled trials (e.g.
alternate allocation) or historical controls

IV Evidence derived from case series or case reports

V Panel or expert opinion

Many of the categories contain sub groups, detailed knowledge of these are not required for MRCS
Part A.

References
Interested users will find further information at www.cebm.net
Next question
A surgeon wishes to determine whether different methods of perioperative shaving have an effect on
post operative wound infection rates. Which of the following is the best method for assessing
whether one method is better than the other?

Cohort study

Retrospective study

Case controlled study

Cross over study

Randomised controlled study

A randomised controlled study is the best method for assessing this relationship. It is important to
analyse data from RCT's on an intention to treat basis.
Please rate this question:

Discuss and give feedback


Next question

Randomised controlled trials

Randomised controlled trials are an established method of comparing two variables. These may
consist of comparison of treatments or treatment versus placebo. Ideally the trials should be blinded,
usually to the patient and those treating them.
In most cases a power calculation should be performed to determine the sample size required to
detect a difference.
Next question
Theme: Electrosurgery

A. Cutting current

B. Coagulation current

C. Blended current

D. Fulguration

E. Desiccation

For each of the following electrosurgical applications please select the most likely modality used.
Each option may be used once, more than once or not at all.

52. In this modality the active electrode is placed in direct contact with the tissue and is characterised
by low current and high voltage over a broad area.

You answered Cutting current

The correct answer is Desiccation

In desiccation the device is placed in direct contact with the tissues (unlike fulguration). Because it
is applied over a broad area it tends not to cause protein damage (unlike coagulation).

53. An electrosurgical mode whereby the electrode is held away from the tissue. The current utilises
a low amplitude and high voltage.

You answered Cutting current

The correct answer is Fulguration

Fulguration typically avoids contact between the electrode and the tissue with the current
configured to favor arc formation.

54. A modality in which a sinusoidal, non modulated waveform is produced and vaporises the tissues.
Cutting current

The high energy levels result in tissue vaporisation and cleavage of tissues.

Please rate this question:

Discuss and give feedback

Next question

Electrosurgery

Electrosurgery utilises the heat generated by the passage of high frequency alternating electrical
current through living tissues. The application of a voltage across human tissue results in the
formation of an electrical circuit between the voltage source and the tissue. The tissue acts as a
resistor and the level of resistance is determined by the water content of the tissue. It is this
resistance that results in the formation of heat.

An alternating current constantly changes the direction in which the current flows, the speed with
which this occurs is measured in Hertz. Most diathermy units operate at a frequency of between
200,000 kHZ to 5MHz. This means that tissue such as nerves and muscles will not depolarise (since
this seldom occurs at frequencies above 10,000Hz). The current waveform can be adjusted to
deliver three main therapeutic modalities; cutting, coagulation and blend.

Types of current

Cutting  Sinusoidal and non modulated waveform


 High average power and current density
 Precise cutting without thermal damage

Coagulation  Modulated current with intermittent dampened sine waves of high peak voltage
 Evaporation, rather than vaporisation of intracellular fluid occurs
 Results in formation of coagulum

Desication  Active electrode in direct contact with tissue


 Low current and high voltage system
 Results in loss of cellular water but no protein damage

Fulguration  Electrode probe is held away from tissue


 Produces spray effect with local, superficial tissue destruction
 Low amplitude and high voltage system

Blend  Alternating cutting and coagulation modes


 Total average power is less than with cutting

Next question
Theme: Surgical site infections

A. Glutaraldehyde 2% applied to the skin

B. Sodium hypochlorite solution applied to the skin

C. Aqueous iodine applied to the skin

D. Perform surgery in a lamninar flow theatre

E. Surgeon to wear exhaust suit

F. Administration of clindamycin

G. Administration of gentamicin

H. Pre operative shaving

Please select the most appropriate modality to reduce the risk of developing a surgical site infection
for the scenario given. Each option may be used once, more than once or not at all.

55. A 42 year old man is due to undergo a Mayo repair of a paraumbilical hernia. He is otherwise
well.

You answered Glutaraldehyde 2% applied to the skin

The correct answer is Aqueous iodine applied to the skin

The patient will require skin preparation. However, use of glutaraldehyde or sodium hypochlorite
would be an inappropriate choice. As the Mayo repair does not involve implantation of prosthetic
mesh the use of antibiotics is not appropriate.

56. A 63 year old man with end stage oestoarthritis of the hip is due to undergo a total hip
replacement. The skin has been prepared and antibiotics given.

You answered Glutaraldehyde 2% applied to the skin

The correct answer is Perform surgery in a lamninar flow theatre


Laminar flow is more important than an exhaust suit although use of both is ideal.

57. A 22 year old man is undergoing an appendicectomy. At operation there is copious pus around
the appendix.

You answered Glutaraldehyde 2% applied to the skin

The correct answer is Administration of gentamicin

Gentamicin is the preferred agent. Clindamycin is associated with high rate of clostridium dificile
infection.

Please rate this question:

Discuss and give feedback

Next question

Surgical site infection

 Surgical site infections may occur following a breach in tissue surfaces and allow normal
commensals and other pathogens to initiate infection. They are a major cause of morbidity
and mortality.
 Surgical site infections (SSI) comprise up to 20% of all healthcare associated infections and
at least 5% of patients undergoing surgery will develop an SSI as a result.
 In many cases the organisms are derived from the patient's own body. Measures that may
increase the risk of SSI include:
 Shaving the wound using a razor (disposable clipper preferred)
 Using a non iodine impregnated incise drape if one is deemed to be necessary
 Tissue hypoxia
 Delayed administration of prophylactic antibiotics in tourniquet surgery

Preoperatively

 Don't remove body hair routinely


 If hair needs removal, use electrical clippers with single use head (razors increase infection
risk)
 Antibiotic prophylaxis if:

- placement of prosthesis or valve


- clean-contaminated surgery
- contaminated surgery

 Use local formulary


 Aim to give single dose IV antibiotic on anaesthesia
 If a tourniquet is to be used, give prophylactic antibiotics earlier

Intraoperatively

 Prepare the skin with alcoholic chlorhexidine (Lowest incidence of SSI)


 Cover surgical site with dressing
 A recent meta analysis has confirmed that administration of supplementary oxygen does not
reduce the risk of wound infection. In contrast to previous individual RCT's(1)
 Wound edge protectors do not appear to confer benefit (2)

Post operatively
Tissue viability advice for management of surgical wounds healing by secondary intention

Use of diathermy for skin incisions


In the NICE guidelines the use of diathermy for skin incisions is not advocated(3). Several
randomised controlled trials have been undertaken and demonstrated no increase in risk of SSI
when diathermy is used(4).

References
1. Brar M et al.. Perioperative supplemental oxygen in colorectal patients: a meta analysis. J Surg
Res2011 (166): 227 -235.
2. Pinkney T et al. Impact of wound edge protection devices on surgical site infection after
laparotomy: impact of a multicentre randomised controlled trial (ROSSINI Trial). BMJ 2013 (347):10.
3. http://www.nice.org.uk/CG74
4. Ahmad N and Ahmed A. Meta-analysis of the effectiveness of surgical scalpel or diathermy in
making abdominal skin incisions. Ann Surg 2011, 253(1):8-13.
Next question
Theme: Suture materials

A. Silk 3/0

B. Polyglactin 3/0

C. Polydioxanone 1/0

D. Stainless steel skin clips

E. Stainless steel wire 1/0

F. 6/0 Polypropylene

G. 3/0 Undyed polyglactin

H. 3/0 Polypropylene

Please select the most appropriate suture for the situation described. Each option may be used
once, more than once or not at all.

58. Anchoring a RediVac drain to the skin following a mastectomy.

Silk 3/0

Silk is traditionally used for this purpose because of its reliable knotting.

59. A surgeon wishes to closure the linea alba of the abdominal wall following a laparotomy

You answered Silk 3/0

The correct answer is Polydioxanone 1/0

A large suture such as 1/0 PDS or 1/0 polypropylene is the standard material for this indication.
From the list 1/0 PDS is the most appropriate.

60. Anastomosis of Dacron graft to proximal abdominal aorta during abdominal aortic aneurysm
repair.
You answered Silk 3/0

The correct answer is 3/0 Polypropylene

3/0 polypropylene is the suture of choice in this setting. 6/0 is too fine and will not withstand the
tensile forces.

Please rate this question:

Discuss and give feedback

Next question

Suture material

Suture materials

Agent Classification Durability Uses Special points

Silk Braided Theoretically Anchoring devices, skin Knots easily, poor


Biological permanent closure cosmesis
although
strength not
preserved

Catgut Braided 5-7 days Short term wound Poor cosmesis


Biological approximation Degrades rapidly
Not available in UK

Chromic catgut Braided Up to 12 weeks Apposition of deeply Unpredictable


Biological sited tissues degradation pattern
Not in use in UK
Agent Classification Durability Uses Special points

Polydiaxonone Synthetic Up to 3 months Widespread surgical Used in most surgical


(PDS) Monofilament (longer with applications including specialties (avoid dyed
thicker sutures) visceral anastomoses, form in dermal
dermal closure, mass closure)
closure of abdominal
wall

Polyglycolic acid Braided Up to 6 weeks Most tissues can be It has good handling
(Vicryl, Dexon) Synthetic apposed using properties, the dyed
polyglycolic acid form of this suture
should not be used for
skin closure

Polypropylene Synthetic Permanent Widely used, agent of Poor handling


(Prolene) Monofilament choice for vascular properties
anastomoses

Polyester Synthetic Permanent Its combination of It is more expensive


(Ethibond) Braided permanency and and has considerable
braiding makes it useful tissue drag
for laparoscopic surgery

Absorbable vs Non absorbable

 Time taken to degrade absorbable materials varies


 Usually by macrophages hydrolysing material
 Consider absorbable sutures in situations where long term tissue apposition is not required.
In cardiac and vascular surgery non absorbable sutures are usually used.

Suture size

 The higher the index number the smaller the suture i.e. : 6/0 prolene is finer than 1/0 prolene.
 Finer sutures have less tensile strength. For example 6/0 prolene would not be a suture
suitable for abdominal mass closure but would be ideal for small calibre distal arterial
anastomoses.

Braided vs monofilament
Generally speaking braided sutures have better handling characteristics than non braided. However,
they are associated with higher bacterial counts. Braided materials are unsuitable for use in vascular
surgery as they are potentially thrombogenic.
Next question
A 19 year old male presents with axillary lymphadenopathy and symptoms suggestive of Hodgkins
lymphoma. What is the most appropriate investigation?

Fine needle aspiration of the lymph nodes

Freehand needle core biopsy of the lymph nodes

Image guided core biopsy of the lymph nodes

Excision biopsy of a lymph node

Axillary node clearance

When a diagnosis of lymphoma is suspected, the correct investigation is excision biopsy of a


complete lymph node to confirm the diagnosis.

Excision of a single node is appropriate. Lymphoma is usually treated with chemotherapy and
axillary node clearance is therefore inappropriate. FNA and core biopsy will not allow accurate
diagnosis and are therefore not appropriate.

Please rate this question:

Discuss and give feedback

Next question

Hodgkins lymphoma

Presenting features

 Asymptomatic lympadenopathy
 Cough, Pel Ebstein fever, haemoptysis, dyspnoea
 B Symptoms - 10% weight loss, fever, night sweats
Staging
All patients are staged with CT scanning of the chest, abdomen and pelvis
The Ann Arbor staging system is commonly used
Stage Features

I Single lymph node region

II Two or more regions on the same side of the diaphragm

III Involvement of lymph node regions on both sides of the diaphragm

IV Involvement of extra nodal sites

Sub types
Classical Hodgkin lymphoma is classified into the following 4 types:

Nodular sclerosing Hodgkin lymphoma (NSHL)


Mixed-cellularity Hodgkin lymphoma (MCHL)
Lymphocyte-depleted Hodgkin lymphoma (LDHL)
Lymphocyte-rich classical Hodgkin lymphoma (LRHL)

A Reed Sternberg cell may be identified histologically.

A fifth sub type, Nodular lymphocyte-predominant Hodgkin lymphoma, is characterised by a different


cell type Reed- Sternberg cells are rarely seen.

Treatment
This may be multimodal and both chemo and radiotherapy are used.

Diagnosis
This is made by excision of a complete lymph node that is then submitted for detailed histological
evaluation.

Pathogenesis
Infection with Ebstein Barr virus is linked to the condition (particularly mixed cellularity lymphoma).

Prognosis
Stage I disease is associated with survival figures of up to 85% at 5 years. The lymphocyte rich
classical lymphoma has the best prognosis. Lymphocyte depleted Hodgkins lymphoma, advancing
age, male sex and stage IV disease are all associated with a worsening of prognosis.

Next question
What is the mechanism of action of ciprofloxacin?

Inhibition of DNA gyrase

Direct injury to the bacterial cell wall

Osmotic damage to the cell

Inhibition of reverse transcriptase

Destruction of bacterial aquaporin proteins

Please rate this question:

Discuss and give feedback

Next question

Antibiotics: mechanism of action

The lists below summarise the site of action of the commonly used antibiotics

Inhibit cell wall formation

 penicillins
 cephalosporins

Inhibit protein synthesis

 aminoglycosides (cause misreading of mRNA)


 chloramphenicol
 macrolides (e.g. erythromycin)
 tetracyclines
 fusidic acid

Inhibit DNA synthesis

 quinolones (e.g. ciprofloxacin)


 metronidazole
 sulphonamides
 trimethoprim

Inhibit RNA synthesis

 rifampicin

Next question
A 73 year old lady with gallstones is about the undergo a laparoscopic cholecystectomy. The
surgeon inserts a Verress needle and performs a successful drop test prior to establishing a
pneumoperitoneum. A 5 minute delay ensues before a 10mm infraumbilical trocar is inserted. The
surgeon performs a diagnostic laparoscopy which shows a thickened gallbladder but is otherwise
normal. The anaesthetist complains that the patient has become hypotensive with a blood pressure
of 80/40 mmHg. Of the options below, which is the most appropriate course of action?

Release of pneumoperitoneum

Perform a laparotomy

Administration of intravenous adrenaline

Administration of intravenous amiodarone

End the operation

Excessive intra-abdominal pressure may cause decreased venous return and hypotension. Since
the preliminary laparoscopy did not show any major vascular catastrophe an emergency laparotomy
would not be indicated. In most cases the release of pressure is often sufficient. In cases of a vaso-
vagal episode (which may be induced by peritoneal stretching) a dose of atropine may be required.
Please rate this question:

Discuss and give feedback


Next question

Pneumoperitoneum- therapeutic

During a laparoscopic procedure a surgeon will need to create a pneumoperitoneum. This can be
achieved by use of a Verress needle (risk of visceral injury). An alternative is the open "Hassan"
style technique. Once access to the abdominal cavity is secured carbon dioxide gas is insufflated to
induce a working space. Higher intra-abdominal pressures may compromise venous return and
reduce cardiac output. If the blood pressure is seen to drop in this way then release of air, will often
improve matters. Should this not be the case then a laparotomy may be necessary to exclude a
more significant internal injury.
Next question
Which of the following is least likely to reduce the risk of post operative wound infection?

Electrical clippers to remove body hair

Use of poviodone impregnated drapes

Antibiotic prophylaxis for prosthesis placement

Routine use of mechanical bowel preparation

Chlorhexidine to prepare the skin

The routine use of mechanical bowel preparation is not recommended. There is some recent
evidence to support the use of selective gut decontamination. However, this is not in mainstream
practice at present.

Please rate this question:

Discuss and give feedback

Next question

Surgical site infection

 Surgical site infections may occur following a breach in tissue surfaces and allow normal
commensals and other pathogens to initiate infection. They are a major cause of morbidity
and mortality.
 Surgical site infections (SSI) comprise up to 20% of all healthcare associated infections and
at least 5% of patients undergoing surgery will develop an SSI as a result.
 In many cases the organisms are derived from the patient's own body. Measures that may
increase the risk of SSI include:
 Shaving the wound using a razor (disposable clipper preferred)
 Using a non iodine impregnated incise drape if one is deemed to be necessary
 Tissue hypoxia
 Delayed administration of prophylactic antibiotics in tourniquet surgery
Preoperatively

 Don't remove body hair routinely


 If hair needs removal, use electrical clippers with single use head (razors increase infection
risk)
 Antibiotic prophylaxis if:

- placement of prosthesis or valve


- clean-contaminated surgery
- contaminated surgery

 Use local formulary


 Aim to give single dose IV antibiotic on anaesthesia
 If a tourniquet is to be used, give prophylactic antibiotics earlier

Intraoperatively

 Prepare the skin with alcoholic chlorhexidine (Lowest incidence of SSI)


 Cover surgical site with dressing
 A recent meta analysis has confirmed that administration of supplementary oxygen does not
reduce the risk of wound infection. In contrast to previous individual RCT's(1)
 Wound edge protectors do not appear to confer benefit (2)

Post operatively
Tissue viability advice for management of surgical wounds healing by secondary intention

Use of diathermy for skin incisions


In the NICE guidelines the use of diathermy for skin incisions is not advocated(3). Several
randomised controlled trials have been undertaken and demonstrated no increase in risk of SSI
when diathermy is used(4).

References
1. Brar M et al.. Perioperative supplemental oxygen in colorectal patients: a meta analysis. J Surg
Res2011 (166): 227 -235.
2. Pinkney T et al. Impact of wound edge protection devices on surgical site infection after
laparotomy: impact of a multicentre randomised controlled trial (ROSSINI Trial). BMJ 2013 (347):10.
3. http://www.nice.org.uk/CG74
4. Ahmad N and Ahmed A. Meta-analysis of the effectiveness of surgical scalpel or diathermy in
making abdominal skin incisions. Ann Surg 2011, 253(1):8-13.
Next question
A 67 year old women is undergoing a femoral hernia repair and the surgeon is using a bipolar
diathermy unit for haemostasis. Which of the following is a recognised risk with the use of bipolar
diathermy?

Patient burns at the site of the contact plate

Fires when used near alcoholic skin preparations that have pooled

Coupling injuries

Risk of thermal injury to regional vessels as a result of tissue heating

Capacitance injuries

In bipolar units the flow of electricity is from one electrode to the other over a small area. As a result
a contact plate is not used and coupling and capacitance injuries are uncommon. They have a low
risk of thermal injury to adjacent structures and are preferred for this reason. However, they may
cause sparks and ignite inflammable solutions.

Please rate this question:

Discuss and give feedback

Next question

Diathermy

 Diathermy devices are used by surgeons in all branches of surgery.


 Use electric currents to produce local heat and thereby facilitate haemostasis or surgical
dissection.
 Consist of a generator unit that is located outside the patient and can be set to the level of
power required by the surgeon.
 There are two major types of diathermy machine;

Monopolar
The current flows through the diathermy unit into a handheld device that is controlled by the
surgeon. Electricity can flow from the tip of the device into the patient. The earth electrode is located
some distance away. The relatively narrow tip of the diathermy device produces local heat and this
can be used to vaporise and fulgurate tissues. The current can be adjusted in terms of frequency so
that different actions can be effected. In cutting mode sufficient power is applied to the tissues to
vaporise their water content. In coagulation mode the power level is reduced so that a coagulum is
formed instead. Some diathermy machines can utilise a setting known as blend that alternates
cutting and coagulation functions, these tend to be used during procedures such as colonoscopic
polypectomy.

Bipolar
The electric current flows from one electrode to another however, both electrodes are usually
contained within the same device e.g. a pair of forceps. The result is that heating is localised to the
area between the two electrodes and surrounding tissue damage is minimised.

Ultrasound based devices


These include CUSA and Harmonic scalpel. They generate high frequency oscillations that seal and
coagulate tissues. They have different energy settings that allow them to dissect and simultaneously
seal vessels if required. The CUSA device leaves vessels intact that may then be divided.

Ligasure device
Delivers tailored energy levels to allow simultaneous haemostasis and dissection. The device
senses the impedance of the tissues and tailors energy levels accordingly.

Hazards of diathermy

 Inadvertent patient burn. This may result of careless handling of the device or in the case of
monopolar devices forgetting to apply a return electrode plate, In this situation patients may
develop a contact burn when electricity flows to earth
 Explosion or fire. This may occur when volatile anaesthetic gases or skin preparation fluid
have been used

Next question
A 34 year old lady is due to undergo a laparoscopic cholecystectomy. Which of the following
intrabdominal pressures should typically be set on the gas insufflation system?

4mm Hg

10mm Hg

20mm Hg

40mm Hg

60mm Hg

Pressures lower than 7mm Hg are not usually compatible with satisfactory views. Pressures >15mm
Hg are usually associated with decreased venous return and hypotension.
Please rate this question:

Discuss and give feedback


Next question

Gases for laparoscopic surgery

Laparoscopic surgery may be performed in a number of body cavities. In some areas irrigation
solutions are preferred. In the abdomen insufflation with carbon dioxide gas is commonly used. The
amount of gas delivered is adjusted to maintain a constant intra-abdominal pressure of between 12
and 15 mmHg. Excessive intra-abdominal pressure may reduce venous return and lead to
hypotension. Too little insufflation will risk obscuring the surgical view.
Next question
A 53 year old man undergoes an elective right hemicolectomy. A stapled ileo-colic anastomosis is
constructed. Eight hours later he becomes tachycardic and passes approximately 600ml of dark red
blood per rectum. Which of the following processes is the most likely explanation for this
occurrence?

Anastomotic leak

Discharging mesenteric haematoma

Bleeding peptic ulcer

Anastomotic staple line bleeding

Mesenteric infarct

Safe visceral anastamosis requires:

 Mucosal to mucosal apposition


 Adequate vascularity
 Minimal tension

Stapled anastomoses are associated with staple line bleeding and this may typically occur in the
early post operative phase. They should be managed conservatively as most will settle.
Stapled anastomoses are quicker to perform. Ironically, although they may appear easy they can
carry considerably more potential pitfalls than their hand sewn equivalent and should be used with
caution by the inexperienced, this is especially true if the bowel is very thick walled.
Please rate this question:

Discuss and give feedback

Anastomoses
 A wide variety of anastomoses are constructed in surgical practice. Essentially the term
refers to the restoration of luminal continuity. As such they are a feature of both abdominal
and vascular surgery.

Visceral anastomoses

For an anastomosis to heal three criteria need to be fulfilled:

 Adequate blood supply


 Mucosal apposition
 Minimal tension

When these are compromised the anastomosis may break down. Even in the best surgical hands
some anastomoses are more prone to dehiscence than others. Oesophageal and rectal
anastomoses are more prone to leakage and reported leak rates following oesophageal and rectal
surgery can be as high as 20%. This figure includes radiological leaks and those with a clinically
significant leak will be of a lower order of magnitude. As a rule small bowel anastomoses heal most
reliably.

The decision as to how best to achieve mucosal apposition is one for each surgeon. Some will prefer
the use of stapling devices as they are quicker to use, others will prefer to perform a sutured
anastomosis. The attention to surgical technique is more important than the method chosen and a
poorly constructed stapled anastomosis in thickened tissue is far more prone to leakage than a hand
sewn anastomosis in the same circumstances.

If an anastomosis looks unsafe then it may be best not to construct one at all. In colonic surgery this
is relatively clear cut and most surgeons would bring out an end colostomy. In situations such as
oesophageal surgery this is far more problematic and colonic interposition may be required in this
situation.

Vascular anastomoses

Most arterial surgery involving bypasses or aneurysm repairs will require construction of an arterial
anastomosis. Technique is important and for small diameter distal arterial surgery the intimal
hyperplasia resulting from a badly constructed anastomosis may render the whole operation futile
before the patient leaves hospital.

Some key points about vascular anastomoses:

 Always use non absorbable monofilament suture (e.g. Polypropylene).


 Round bodied needle.
 Correct size for anastamosis ( i.e. 6/0 prolene for bottom end of a femoro-distal bypass).
 Suture should be continuous and from inside to outside of artery to avoid raising an intimal
flap.
A 53 year old man undergoes a reversal of a loop colostomy. He recovers well and is discharged
home. He is readmitted 10 days later with symptoms of vomiting and colicky abdominal pain. On
examination he has a swelling of the loop colostomy site and it is tender. What is the most likely
underlying diagnosis?

Haematoma

Intra abdominal adhesions

Anastomotic leak

Anastomotic stricture

Obstructed incisional hernia

Theme from January 2016 Exam


In this scenario the most likely diagnosis would be obstructed incisional hernia. The tender swelling
coupled with symptoms of obstruction point to this diagnosis. Prompt surgical exploration is
warranted. Loop colostomy reversals are at high risk of this complication as the operative site is at
increased risk of the development of post operative wound infections.
Please rate this question:

Discuss and give feedback


Next question

Acute incisional hernia

 Any surgical procedure involving entry into a cavity containing viscera may be complicated
by post operative hernia
 The abdomen is the commonest site
 The deep layer of the wound has usually broken down, allowing internal viscera to protrude
through
 Management is dictated by the patients clinical status and the timing of the hernia in relation
to recent surgery
 Bowel obstruction or tenderness at the hernia site both mandate early surgical intervention to
reduce the risk of bowel necrosis
 Mature incisional hernias with a wide neck, and no symptoms, may be either left or listed for
elective repair
 Risk factors for the development of post operative incisional hernias include; post operative
wound infections, long term steroid use, obesity and chronic cough
Theme: Abdominal stomas

A. End ileostomy
B. End colostomy
C. Loop ileostomy
D. Loop colostomy
E. End jejunostomy
F. Loop jejunostomy
G. Caecostomy

For each of the following scenarios, please select the most appropriate type of stoma to be
constructed. Each option may be selected once, more than once or not at all.

2. A 56 year old man is undergoing a low anterior resection for carcinoma of the rectum. A
primary anastomosis is planned.

You answered End ileostomy

The correct answer is Loop ileostomy

Theme from April 2014 Exam


Colonic resections with an anastomosis below the peritoneal reflection may have an
anastomotic leak rate (both clinical and radiological) of up to 15%. Therefore most surgeons
will defunction such an anastomosis to reduce the clinical severity of an anastomotic leak.
A loop ileostomy will achieve this end point and is relatively easy to reverse.

3. A 23 year old man with uncontrolled ulcerative colitis is undergoing an emergency sub total
colectomy.

End ileostomy

Following a sub total colectomy the immediate surgical options include an end ileostomy or
ileorectal anastomosis. In the emergency setting an ileorectal anastomosis would be unsafe.

4. A 63 year old women presents with large bowel obstruction. On examination she has a
carcinoma 10cm from the anal verge.

You answered End ileostomy

The correct answer is Loop colostomy

Large bowel obstruction resulting from carcinoma should be resected, stented or


defunctioned. The first two options typically apply to tumours above the peritoneal
reflection. Lower tumours should be defunctioned with a loop colostomy and then formal
staging undertaken prior to definitive surgery. An emergency attempted rectal resection
carries a high risk of involvement of the circumferential resection margin and is not
recommended.

Please rate this question:

Discuss and give feedback


Next question

Abdominal stomas

Stomas may be sited during a range of abdominal procedures and involve bringing the lumen or
visceral contents onto the skin. In most cases this applies to the bowel. However, other organs or
their contents may be diverted in case of need.

With bowel stomas the type method of construction and to a lesser extent the site will be determined
by the contents of the bowel. In practice, small bowel stomas should be spouted so that their irritant
contents are not in contact with the skin. Colonic stomas do not need to be spouted as their contents
are less irritant.

In the ideal situation the site of the stoma should be marked with the patient prior to surgery. Stoma
siting is important as it will ultimately influence the ability of the patient to manage their stoma and
also reduce the risk of leakage. Leakage of stoma contents and subsequent maceration of the
surrounding skin can rapidly progress into a spiraling loss of control of stoma contents.

Types of stomas
Name of stoma Use Common sites

Gastrostomy  Gastric decompression or fixation Epigastrium


 Feeding

Loop  Seldom used as very high output Any location according to


jejunostomy  May be used following emergency need
laparotomy with planned early closure

Percutaneous  Usually performed for feeding Usually left upper quadrant


jejunostomy purposes and site in the proximal
bowel

Loop ileostomy  Defunctioning of colon e.g. following Usually right iliac fossa
rectal cancer surgery
Name of stoma Use Common sites

 Does not decompress colon (if


ileocaecal valve competent)

End ilestomy  Usually following complete excision Usually right iliac fossa
of colon or where ileo-colic
anastomosis is not planned
 May be used to defunction colon, but
reversal is more difficult

End colostomy Where a colon is diverted or resected and Either left or right iliac fossa
anastomosis is not primarily achievable or
desirable

Loop colostomy  To defunction a distal segment of May be located in any region


colon of the abdomen, depending
 Since both lumens are present the upon colonic segment used
distal lumen acts as a vent

Caecostomy Stoma of last resort where loop colostomy is Right iliac fossa
not possible

Mucous fistula  To decompress a distal segment of May be located in any region


bowel following colonic division or of the abdomen according to
resection clinical need
 Where closure of a distal resection
margin is not safe or achievable

Next question
A 34 year old man undergoes a sub total colectomy to treat fulminant ulcerative colitis. What type of
stoma is most likely to be fashioned?

End colostomy

Loop colostomy

End ileostomy

Loop ileostomy

End jejunostomy

Theme from January 2016 Exam


A sub total colectomy involves the removal of the entire right, transverse, left and part of the sigmoid
colon. The rectal stump is closed and an end ileostomy fashioned in the right iliac fossa.
Please rate this question:

Discuss and give feedback


Next question

Abdominal stomas

Stomas may be sited during a range of abdominal procedures and involve bringing the lumen or
visceral contents onto the skin. In most cases this applies to the bowel. However, other organs or
their contents may be diverted in case of need.

With bowel stomas the type method of construction and to a lesser extent the site will be determined
by the contents of the bowel. In practice, small bowel stomas should be spouted so that their irritant
contents are not in contact with the skin. Colonic stomas do not need to be spouted as their contents
are less irritant.

In the ideal situation the site of the stoma should be marked with the patient prior to surgery. Stoma
siting is important as it will ultimately influence the ability of the patient to manage their stoma and
also reduce the risk of leakage. Leakage of stoma contents and subsequent maceration of the
surrounding skin can rapidly progress into a spiraling loss of control of stoma contents.

Types of stomas
Name of stoma Use Common sites
Name of stoma Use Common sites

Gastrostomy  Gastric decompression or fixation Epigastrium


 Feeding

Loop  Seldom used as very high output Any location according to


jejunostomy  May be used following emergency need
laparotomy with planned early closure

Percutaneous  Usually performed for feeding Usually left upper quadrant


jejunostomy purposes and site in the proximal
bowel

Loop ileostomy  Defunctioning of colon e.g. following Usually right iliac fossa
rectal cancer surgery
 Does not decompress colon (if
ileocaecal valve competent)

End ilestomy  Usually following complete excision Usually right iliac fossa
of colon or where ileo-colic
anastomosis is not planned
 May be used to defunction colon, but
reversal is more difficult

End colostomy Where a colon is diverted or resected and Either left or right iliac fossa
anastomosis is not primarily achievable or
desirable

Loop colostomy  To defunction a distal segment of May be located in any region


colon of the abdomen, depending
 Since both lumens are present the upon colonic segment used
distal lumen acts as a vent

Caecostomy Stoma of last resort where loop colostomy is Right iliac fossa
not possible

Mucous fistula  To decompress a distal segment of May be located in any region


bowel following colonic division or of the abdomen according to
resection clinical need
Name of stoma Use Common sites

 Where closure of a distal resection


margin is not safe or achievable

Next question
Theme: Appendicitis

A. Colonoscopy
B. MRI Abdomen
C. Appendicectomy
D. Abdominal CT scan
E. Barium enema
F. Exploratory laparotomy
G. Conservative management with intravenous antibiotics
H. Re-assure and discharge
I. Abdominal ultrasound scan

Please select the most appropriate management option for the following patients. Each option may
be used once, more than once or not at all.

6. A 24 year old man presents with a 10 day history of right sided abdominal pain. Prior to this
he was well. On examination he has a low grade fever and a mass palpable in the right iliac
fossa. The rest of his abdomen is soft. An abdominal USS demonstrates matted bowel loops
surrounding a thickened appendix.

You answered Colonoscopy

The correct answer is Conservative management with intravenous antibiotics

This man is likely to have an appendix mass. There is no history suggestive of inflammatory
bowel disease. These are usually managed without surgery, especially in the absence of
peritoneal signs. Broad spectrum antibiotics are required. In the past an interval
appendicectomy was performed. This is rare now and in most cases the process resolves
with fibrosis of the appendix.

7. A 22 year old man presents with a 48 hour history of right iliac fossa pain. On examination
he has a low grade pyrexia and is tender with voluntary guarding in the right iliac fossa. His
blood tests reveal a WCC of 13 and a CRP of 6. A urine dipstick is positive for leucocytes.

You answered Colonoscopy

The correct answer is Appendicectomy

This is a typical history for acute appendicitis and in a young male, few differentials would
be compatible with this history and signs. Whilst inflammatory markers may be raised this
is by no means universal. Further imaging will delay treatment and is unlikely to alter the
eventual surgical outcome.

8. A 63 year old man presents with a 48 hour history of right iliac fossa pain. On examination
he has a low grade pyrexia and is tender with some voluntary guarding in the right iliac
fossa. Some of his blood tests are reproduced below:
Hb 8.1

WCC 13.8

Platelets 438

Albumin 22

CRP 24

You answered Colonoscopy

The correct answer is Abdominal CT scan

This man's investigations point to a more longstanding disease process (Hb and albumin),
right sided colonic cancer being the most likely. For this reason a CT scan is a sensible
option as it will adjust the surgical planning.

Please rate this question:

Discuss and give feedback


Next question

Appendicitis

History

 Peri umbilical abdominal pain (visceral stretching of appendix lumen and appendix is mid gut
structure) radiating to the right iliac fossa due to localised parietal peritoneal inflammation.
 Vomit once or twice but marked and persistent vomiting is unusual.
 Diarrhoea is rare. However, pelvic appendicitis may cause localised rectal irritation and some
loose stools. A pelvic abscess may also cause diarrhoea.
 Mild pyrexia is common - temperature is usually 37.5 -38oC. Higher temperatures are more
typical of conditions like mesenteric adenitis.
 Anorexia is very common. It is very unusual for patients with appendicitis to be hungry.

Examination

 Generalised peritonitis if perforation has occurred or localised peritonism.


 Retrocaecal appendicitis may have relatively few signs.
 Digital rectal examination may reveal boggy sensation if pelvic abscess is present, or even
tenderness with a pelvic appendix.

Diagnosis

 Typically raised inflammatory markers coupled with compatible history and examination
findings should be enough to justify appendicectomy.
 Urine analysis may show mild leucocytosis but no nitrites.
 Ultrasound is useful in females where pelvic organ pathology is suspected. Although it is not
always possible to visualise the appendix on ultrasound, the presence of free fluid (always
pathological in males) should raise suspicion.

Ultrasound examination may show evidence of luminal obstruction and thickening of the appendiceal
wall as shown below

Image sourced from Wikipedia

Treatment

 Appendicectomy which can be performed via either an open or laparoscopic approach.


 Administration of metronidazole reduces wound infection rates.
 Patients with perforated appendicitis require copious abdominal lavage.
 Patients without peritonitis who have an appendix mass should receive broad spectrum
antibiotics and consideration given to performing an interval appendicectomy.
 Be wary in the older patients who may have either an underlying caecal malignancy or
perforated sigmoid diverticular disease.
Laparoscopic appendicectomy is becoming increasing popular as demonstrated below

Image sourced from Wikipedia


Theme: Acute abdominal pain

A. Ruptured abdominal aortic aneurysm

B. Perforated peptic ulcer

C. Perforated appendicitis

D. Mesenteric infarction

E. Small bowel obstruction

F. Large bowel obstruction

G. Pelvic inflammatory disease

H. Mesenteric adenitis

I. Pancreatitis

J. None of the above

Please select the most likely cause of abdominal pain for the scenario given. Each option may be
used once, more than once or not at all.

9. A 75 year old man is admitted with sudden onset severe generalised abdominal pain, vomiting
and a single episode of bloody diarrhoea. On examination he looks unwell and is in uncontrolled
atrial fibrillation. Although diffusely tender his abdomen is soft.

You answered Ruptured abdominal aortic aneurysm

The correct answer is Mesenteric infarction

In mesenteric infarction there is sudden onset of pain together with vomiting and occasionally
passage of bloody diarrhoea. The pain present is usually out of proportion to the physical signs.

10. A 19 year old lady is admitted with lower abdominal pain. On examination she is diffusely tender.
A laparoscopy is performed and at operation multiple fine adhesions are noted between the liver
and abdominal wall. Her appendix is normal.
You answered Ruptured abdominal aortic aneurysm

The correct answer is Pelvic inflammatory disease

This is Fitz Hugh Curtis syndrome in which pelvic inflammatory disease (usually Chlamydia) causes
the formation of fine peri hepatic adhesions.

11. A 78 year old man is walking to the bus stop when he suddenly develops severe back pain and
collapses. On examination he has a blood pressure of 90/40 and pulse rate of 110. His abdomen is
distended and he is obese. Though tender his abdomen itself is soft.

Ruptured abdominal aortic aneurysm

This will be a retroperitoneal rupture (anterior ones generally don't survive to hospital). The
debate regarding CT varies, it is the authors opinion that a systolic BP of <100mmHg at
presentation mandates immediate laparotomy.

Acute mesenteric ischaemia- Pain out of proportion to the physical signs.


Atrial fibrillation is often present.

Fitz Hugh Curtis = Fine Hepatic Connections

Please rate this question:

Discuss and give feedback

Next question

Acute abdominal pain-diagnoses

Conditions presenting with acute abdominal pain

Condition Features Investigations Management


Condition Features Investigations Management

Appendicitis History of migratory Differential white cell count Appendicectomy


pain. Pregnancy test
Fever. C-Reactive protein
Anorexia. Amylase
Evidence of right iliac Urine dipstick testing
fossa tenderness.
Mild pyrexia.

Mesenteric Usually recent upper Full blood count- may show Conservative management-
adenitis respiratory tract slightly raised white cell appendicectomy if
infection. count diagnostic doubt
High fever. Urine dipstick often normal
Generalised abdominal Abdominal ultrasound scan
discomfort- true - usually no free fluid
localised pain and signs
are rare.

Mittelschmerz Only seen in females Full blood count- normal Manage conservatively if
Mid cycle pain Urine dipstick- normal doubt or symptoms fail to
Usually occurs two Abdominal and pelvic settle then laparoscopy
weeks after last ultrasound- may show a
menstrual period trace of pelvic free fluid
Pain usually has a supra-
pubic location
Usually subsides over a
24-48 hour period.

Fitz-Hugh Curtis Disseminated infection Abdominal ultrasound scan- Usually medically managed-
syndrome with Chlamydia. may show free fluid doxycycline or azithromycin
Usually seen in females. High vaginal swabs - may
Consists of evidence of show evidence of sexually
pelvic inflammatory transmitted infections
disease together with
peri-hepatic
inflammation and
Condition Features Investigations Management

subsequent adhesion
formation.

Abdominal Sudden onset of Patients who are Unstable patients should


aortic aneurysm abdominal pain haemodynamically stable undergo immediate surgery
(ruptured) radiating to the back in should have a CT scan (unless it is not in their best
older adults (look for interests).
risk factors). Those with evidence of
Collapse. contained leak on CT should
May be moribund on undergo immediate surgery
arrival in casualty, more Increasing unruptured
stable if contained aneurysmal size is an
haematoma. indication for urgent
Careful clinical surgical intervention (that
assessment may reveal can wait until the next
pulsatile mass. working day)

Perforated Sudden onset of pain Erect CXR may show free Laparotomy (laparoscopic
peptic ulcer (usually epigastric). air. A CT scan may be surgery for perforated
Often preceding history indicated where there is peptic ulcers is both safe
of upper abdominal diagnostic doubt and feasible in experienced
pain. hands)
Soon develop
generalised abdominal
pain.
On examination may
have clinical evidence of
peritonitis.

Intestinal Colicky abdominal pain A plain abdominal film may In those with a virgin
obstruction and vomiting (the help with making the abdomen a lower and
nature of which diagnosis. A CT scan may be earlier threshold for
depends on the level of useful where diagnostic laparotomy should exist
the obstruction). uncertainty exists than in those who may have
Abdominal distension
Condition Features Investigations Management

and constipation (again adhesional obstruction


depending upon site of
obstruction).
Features of peritonism
may occur where local
necrosis of bowel loops
is occurring.

Mesenteric Embolic events present Arterial pH and lactate Immediate laparotomy and
infarction with sudden pain and Arterial phase CT scanning resection of affected
forceful evacuation. is the most sensitive test segments, in acute embolic
Acute on chronic events events SMA embolectomy
usually have a longer may be needed.
history and previous
weight loss.
On examination the
pain is typically greater
than the physical signs
would suggest.

Next question
Theme: Management of splenic trauma

A. Splenectomy
B. Angiography
C. CT Scan
D. Admit for bed rest and observation
E. Ultrasound scan
F. Splenic conservation
G. MRI of the abdomen

Please select the most appropriate intervention for the scenario given. Each option may be used
once, more than once or not at all.

12. A 7 year old boy falls off a wall the distance is 7 feet. He lands on his left side and there is
left flank bruising. There is no haematuria. He is otherwise stable and haemoglobin is
within normal limits.

You answered Splenectomy

The correct answer is Ultrasound scan

This will demonstrate any overt splenic injury. A CT scan carries a significant dose of
radiation. In the absence of haemodynamic instability or other major associated injuries
the use of USS to exclude intraabdominal free fluid (blood) would seem safe when
coupled with active observation. An USS will also show splenic haematomas.

13. A 42 year old motorcyclist is involved in a road traffic accident. A FAST scan in the
emergency department shows free intrabdominal fluid and a laparotomy is performed. At
operation there is evidence of small liver laceration that has stopped bleeding and a tear to
the inferior pole of the spleen.

You answered Splenectomy

The correct answer is Splenic conservation

As minimum damage, attempt conservation.

14. An 18 year old man is involved in a road traffic accident. A CT scan shows disruption of
the splenic hilum and a moderate sized perisplenic haematoma.

Splenectomy

Hilar injuries usually mandate splenectomy. The main risk with conservative management
here is that he will rebleed and with hilar injuries this can be dramatic.

Please rate this question:

Discuss and give feedback


Next question

Splenic trauma

 The spleen is one of the more commonly injured intra abdominal organs
 In most cases the spleen can be conserved. The management is dictated by the associated
injuries, haemodynamic status and extent of direct splenic injury.

Management of splenic trauma


Conservative Small subcapsular haematoma
Minimal intra abdominal blood
No hilar disruption

Laparotomy with conservation Increased amounts of intraabdominal blood


Moderate haemodynamic compromise
Tears or lacerations affecting <50%

Resection Hilar injuries


Major haemorrhage
Major associated injuries

Splenectomy
Technique
Trauma

 GA
 Long midline incision
 If time permits insert a self retaining retractor (e.g. Balfour/ omnitract)
 Large amount of free blood is usually present. Pack all 4 quadrants of the abdomen. Allow
the anaesthetist to 'catch up'
 Remove the packs and assess the viability of the spleen. Hilar injuries and extensive
parenchymal lacerations will usually require splenectomy.
 Divide the short gastric vessels and ligate them.
 Clamp the splenic artery and vein. Two clamps on the patient side are better and allow for
double ligation and serve as a safety net if your assistant does not release the clamp
smoothly.
 Be careful not to damage the tail of the pancreas, if you do then this will need to be formally
removed and the pancreatic duct closed.
 Wash out the abdomen and place a tube drain to the splenic bed.
 Some surgeons implant a portion of spleen into the omentum, whether you decide to do this
is a matter of personal choice.
 Post operatively the patient will require prophylactic penicillin V and pneumococcal vaccine.

Elective
Elective splenectomy is a very different operation from that performed in the emergency setting. The
spleen is often large (sometimes massive). Most cases can be performed laparoscopically. The
spleen will often be macerated inside a specimen bag to facilitate extraction.

Complications

 Haemorrhage (may be early and either from short gastrics or splenic hilar vessels
 Pancreatic fistula (from iatrogenic damage to pancreatic tail)
 Thrombocytosis: prophylactic aspirin
 Encapsulated bacteria infection e.g. Strep. pneumoniae, Haemophilus
influenzae and Neisseriameningitidis

Next question
Theme: Gastrointestinal bleeding

A. Haemorrhoids
B. Meckels diverticulum
C. Angiodysplasia
D. Colonic cancer
E. Diverticular bleed
F. Ulcerative colitis
G. Ischaemic colitis

Please select the most likely cause of colonic bleeding for the scenario given. Each option may be
used once, more than once or not at all

15. A 73 year old lady is admitted with a brisk rectal bleed. She is otherwise well and the
bleed settles. On examination her abdomen is soft and non tender. Elective colonoscopy
shows a small erythematous lesion in the right colon, but no other abnormality.

You answered Haemorrhoids

The correct answer is Angiodysplasia

Angiodysplasia can be difficult to identify and treat. The colonoscopic stigmata are easily
missed by poor bowel preparation.

16. A 23 year old man complains of passing bright red blood rectally. It has been occurring
over the past week and tends to occur post defecation. He also suffers from pruritus ani.

Haemorrhoids

Classical haemorrhoidal symptoms include bright red rectal bleeding, it typically occurs
post defecation and is noticed on the toilet paper and in the toilet pan. It is usually painless,
however, thrombosed external haemorrhoids may be very painful.

17. A 63 year old man presents with episodic rectal bleeding the blood tends to be dark in
colour and may be mixed with stool. His bowel habit has been erratic since an abdominal
aortic aneurysm repair 6 weeks previously.

You answered Haemorrhoids

The correct answer is Ischaemic colitis

The inferior mesenteric artery may have been ligated and being an arteriopath collateral
flow through the marginal may be imperfect.
Please rate this question:

Discuss and give feedback


Next question

Lower Gastrointestinal bleeding

Colonic bleeding
This typically presents as bright red or dark red blood per rectum. Colonic bleeding rarely presents
as malaena type stool, this is because blood in the colon has a powerful laxative effect and is rarely
retained long enough for transformation to occur and because the digestive enzymes present in the
small bowel are not present in the colon. Up to 15% of patients presenting with haemochezia will
have an upper gastrointestinal source of haemorrhage.

As a general rule right sided bleeds tend to present with darker coloured blood than left sided
bleeds. Haemorrhoidal bleeding typically presents as bright red rectal bleeding that occurs post
defecation either onto toilet paper or into the toilet pan. It is very unusual for haemorrhoids alone to
cause any degree of haemodynamic compromise.

Causes
Cause Presenting features

Colitis Bleeding may be brisk in advanced cases, diarrhoea is commonly present.


Abdominal x-ray may show featureless colon.

Diverticular Acute diverticulitis often is not complicated by major bleeding and


disease diverticular bleeds often occur sporadically. 75% all will cease spontaneously
within 24-48 hours. Bleeding is often dark and of large volume.

Cancer Colonic cancers often bleed and for many patients this may be the first sign of
the disease. Major bleeding from early lesions is uncommon

Haemorrhoidal Typically bright red bleeding occurring post defecation. Although patients
bleeding may give graphic descriptions bleeding of sufficient volume to cause
haemodynamic compromise is rare.

Angiodysplasia Apart from bleeding, which may be massive, these arteriovenous lesions
cause little in the way of symptoms. The right side of the colon is more
commonly affected.
Management

 Prompt correction of any haemodynamic compromise is required. Unlike upper


gastrointestinal bleeding the first line management is usually supportive. This is because in
the acute setting endoscopy is rarely helpful.
 When haemorrhoidal bleeding is suspected a proctosigmoidoscopy is reasonable as
attempts at full colonoscopy are usually time consuming and often futile.
 In the unstable patient the usual procedure would be an angiogram (either CT or
percutaneous), when these are performed during a period of haemodynamic instability they
may show a bleeding point and may be the only way of identifying a patch of angiodysplasia.
 In others who are more stable the standard procedure would be a colonoscopy in the
elective setting. In patients undergoing angiography attempts can be made to address the
lesion in question such as coiling. Otherwise surgery will be necessary.
 In patients with ulcerative colitis who have significant haemorrhage the standard approach
would be a sub total colectomy, particularly if medical management has already been tried
and is not effective.

Indications for surgery


Patients > 60 years
Continued bleeding despite endoscopic intervention
Recurrent bleeding
Known cardiovascular disease with poor response to hypotension

Surgery
Selective mesenteric embolisation if life threatening bleeding. This is most helpful if conducted
during a period of relative haemodynamic instability. If all haemodynamic parameters are normal
then the bleeding is most likely to have stopped and any angiography normal in appearance. In
many units a CT angiogram will replace selective angiography but the same caveats will apply.

If the source of colonic bleeding is unclear; perform a laparotomy, on table colonic lavage and
following this attempt a resection. A blind sub total colectomy is most unwise, for example bleeding
from an small bowel arterio-venous malformation will not be treated by this manoeuvre.

Summary of Acute Lower GI bleeding recommendations


Consider admission if:
* Over 60 years
* Haemodynamically unstable/profuse PR bleeding
* On aspirin or NSAID
* Significant co morbidity

Management

 All patients should have a history and examination, PR and proctoscopy


 Colonoscopic haemostasis aimed for in post polypectomy or diverticular bleeding
Theme: Surgical signs

A. Rovsing's sign
B. Boas' sign
C. Psoas stretch sign
D. Cullen's sign
E. Grey-Turner's sign
F. Murphy's sign
G. None of the above

Please select the most appropriate eponymous abdominal sign for the scenario given. Each option
may be used once, more than once or not at all.

18. Severe acute peri-umbilical bruising in the setting of acute pancreatitis.

You answered Rovsing's sign

The correct answer is Cullen's sign

Cullens sign occurs when there has been intraabdominal haemorrage. It is seen in cases of
severe haemorrhagic pancreatitis and is associated with a poor prognosis. It is also seen in
other cases of intraabdominal haemorrhage (such as ruptured ectopic pregnancy).

19. In acute cholecystitis there is hyperaesthesia beneath the right scapula.

You answered Rovsing's sign

The correct answer is Boas' sign

Boas sign refers to this hyperaesthesia. It occurs because the abdominal wall innervation of
this region is from the spinal roots that lie at this level.

20. In appendicitis palpation of the left iliac fossa causes pain in the right iliac fossa.

Rovsing's sign

Rovsings sign elicits tenderness because the deep palpation induces shift of the appendix
(which is inflamed) against the peritoneal surface. This has somatic innervation and will
therefore localise the pain. It is less reliable in pelvic appendicitis and when the appendix
is truly retrocaecal

Please rate this question:


Discuss and give feedback
Next question

Abdominal signs

A number of eponymous abdominal signs are noted. These include:

 Rovsings sign- appendicitis


 Boas sign -cholecystitis
 Murphys sign- cholecystitis
 Cullens sign- pancreatitis (other intraabdominal haemorrhage)
 Grey-Turners sign- pancreatitis (or other retroperitoneal haemorrhage)

In clinical practice haemorrhagic pancreatitis is thankfully rare. The signs are important and thus
shown below:

Cullen's sign

Image sourced from Wikipedia

Grey Turner's sign


Theme: Surgical access

A. Gridiron
B. Lanz
C. McEvedy
D. Midline abdominal
E. Rutherford Morrison
F. Battle (abdominal)
G. Lower midline

Please select the most appropriate incision for the procedure required. Each option may be used
once, more than once or not at all.

21. A 78 year old lady is admitted with a tender lump in her right groin. It is within the
femoral triangle and there is concern that there may be small bowel obstruction
developing.

You answered Gridiron

The correct answer is McEvedy

This is one approach to an obstructed femoral hernia. It is possible to undertake a small


bowel resection through this approach. Recourse to laparotomy may be needed if access is
difficult.

22. A 45 year old woman with end stage renal failure is due to undergo a cadaveric renal
transplant. This will be her first transplant.

You answered Gridiron

The correct answer is Rutherford Morrison

This is the incision of choice for the extraperitoneal approach to the iliac vessels which
will be required for a renal transplant.

23. A slim 20 year old lady is suffering from appendicitis and requires an appendicectomy.

You answered Gridiron

The correct answer is Lanz

Either a Lanz or Gridiron incision will give access for appendicectomy. However, in the
case described a Lanz incision will give better cosmesis and can be extended should pelvic
surgery be required eg for gynaecological disease.

Please rate this question:

Discuss and give feedback


Next question

Abdominal incisions

Midline incision  Commonest approach to the abdomen


 Structures divided: linea alba, transversalis fascia, extraperitoneal fat,
peritoneum (avoid falciform ligament above the umbilicus)
 Bladder can be accessed via an extraperitoneal approach through the
space of Retzius

Paramedian  Parallel to the midline (about 3-4cm)


incision  Structures divided/retracted: anterior rectus sheath, rectus (retracted),
posterior rectus sheath, transversalis fascia, extraperitoneal fat,
peritoneum
 Incision is closed in layers

Battle  Similar location to paramedian but rectus displaced medially (and thus
denervated)
 Now seldom used

Kocher's Incision under right subcostal margin e.g. Cholecystectomy (open)

Lanz Incision in right iliac fossa e.g. Appendicectomy

Gridiron Oblique incision centered over McBurneys point- usually appendicectomy


(less cosmetically acceptable than Lanz

Gable Rooftop incision

Pfannenstiel's Transverse supra pubic, primarily used to access pelvic organs

McEvedy's Groin incision e.g. Emergency repair strangulated femoral hernia

Rutherford Extraperitoneal approach to left or right lower quadrants. Gives excellent


Morrison access to iliac vessels and is the approach of choice for first time renal
transplantation.
Image sourced from Wikipedia

Next question
Theme: Hernias

A. Littres hernia
B. Richters hernia
C. Bochdalek hernia
D. Morgagni hernia
E. Spigelian hernia
F. Lumbar hernia
G. Obturator hernia

Please select the type of hernia that most closely matches the description given. Each option may
be used once, more than once or not at all.

24. A 73 year old lady presents with peritonitis and tenderness of the left groin. At operation
she has a left femoral hernia with perforation of the anti mesenteric border of ileum
associated with the hernia.

You answered Littres hernia

The correct answer is Richters hernia

When part of the bowel wall is trapped in a hernia such as this it is termed a Richters
hernia and may complicate any hernia although femoral and obturator hernias are most
typically implicated.

25. A 22 year old man is operated on for a left inguinal hernia, at operation the sac is opened
to reveal a large Meckels diverticulum.

Littres hernia

Hernia containing Meckels diverticulum is termed a Littres hernia.

26. A 45 year old man has recurrent colicky abdominal pain. As part of a series of
investigations he undergoes a CT scan and this demonstrates a hernia lateral to the rectus
muscle at the level of the arcuate line.

You answered Littres hernia

The correct answer is Spigelian hernia

This is the site for a spigelian hernia.

Please rate this question:


Discuss and give feedback
Next question

Hernia

Hernias occur when a viscus or part of it protrudes from within its normal anatomical cavity. Specific
hernias are covered under their designated titles the remainder are addressed here.

Spigelian hernia

 Interparietal hernia occurring at the level of the arcuate line


 Rare
 May lie beneath internal oblique muscle. Usually between internal and external oblique
 Equal sex distribution
 Position is lateral to rectus abdominis
 Both open and laparoscopic repair are possible, the former in cases of strangulation

Lumbar hernia
The lumbar triangle (through which these may occur) is bounded by:
Crest of ilium (inferiorly)
External oblique (laterally)
Latissimus dorsi (medially)

 Primary lumbar herniae are rare, most are incisional hernias following renal surgery

- Direct anatomical repair with or without mesh re-enforcement is the procedure of choice

Obturator hernia

 Herniation through the obturator canal


 Commoner in females
 Usually lies behind pectineus muscle
 Elective diagnosis is unusual most will present acutely with obstruction
 When presenting acutely most cases with require laparotomy or laparoscopy (and small
bowel resection if indicated)

Richters hernia

 Condition in which part of the wall of the small bowel (usually the anti mesenteric border) is
strangulated within a hernia (of any type)
 They do not present with typical features of intestinal obstruction as lumenal patency is
preserved
 Where vomiting is prominent it usually occurs as a result of paralytic ileus from peritonitis (as
these hernias may perforate)

Incisional hernia

 Occur through sites of surgical access into the abdominal cavity


 Most common following surgical wound infection
 To minimise following midline laparotomy Jenkins Rule should be followed and this
necessitates a suture length 4x length of incision with bites taken at 1cm intervals, 1 cm from
the wound edge
 Repair may be performed either at open surgery or laparoscopically and a wide variety of
techniques are described

Bochdalek hernia

 Typically congenital diaphragmatic hernia


 85% cases are located in the left hemi diaphragm
 Associated with lung hypoplasia on the affected side
 More common in males
 Associated with other birth defects
 May contain stomach
 May be treated by direct anatomical apposition or placement of mesh. In infants that have
severe respiratory compromise mechanical ventilation may be needed and mortality rate is
high

Morgagni Hernia

 Rare type of diaphragmatic hernia (approx 2% cases)


 Herniation through foramen of Morgagni
 Usually located on the right and tend to be less symptomatic
 More advanced cases may contain transverse colon
 As defects are small pulmonary hypoplasia is less common
 Direct anatomical repair is performed

Umbilical hernia

 Hernia through weak umbilicus


 Usually presents in childhood
 Often symptomatic
 Equal sex incidence
 95% will resolve by the age of 2 years
 Surgery performed after the third birthday
Paraumbilical hernia

 Usually a condition of adulthood


 Defect is in the linea alba
 More common in females
 Multiparity and obesity are risk factors
 Traditionally repaired using Mayos technique - overlapping repair, mesh may be used though
not if small bowel resection is required owing to acute strangulation

Littres hernia

 Hernia containing Meckels diverticulum


 Resection of the diverticulum is usually required and this will preclude a mesh repair
Theme: Groin masses

A. Femoral hernia
B. Lymphadenitis
C. Inguinal hernia
D. Psoas abscess
E. Saphenous varix
F. Femoral artery aneurysm
G. Metastatic lymphadenopathy
H. Lymphoma
I. False femoral artery aneurysm

What is the likely diagnosis for groin mass described? Each option may be used once, more than
once, or not at all.

27. A 52 year old obese lady reports a painless grape sized mass in her groin area. She has no
medical conditions apart from some varicose veins. There is a cough impulse and the mass
disappears on lying down.

You answered Femoral hernia

The correct answer is Saphenous varix

The history of varicose veins should indicate a more likely diagnosis of a varix. The varix
can enlarge during coughing/sneezing. A blue discolouration may be noted.

28. A 32 year old male is noted to have a tender mass in the right groin area. There are also
red streaks on the thigh, extending from a small abrasion.

You answered Femoral hernia

The correct answer is Lymphadenitis

The red streaks are along the line of the lymphatics, indicating infection of the lymphatic
vessels. Lymphadenitis is infection of the local lymph nodes.

29. A 23 year old male suffering from hepatitis C presents with right groin pain and swelling.
On examination there is a large abscess in the groin. Adjacent to this is a pulsatile
swelling. There is no cough impulse.

You answered Femoral hernia

The correct answer is False femoral artery aneurysm


False aneurysms may occur following arterial trauma in IVDU. They may have associated
blood borne virus infections and should undergo duplex scanning prior to surgery. False
aneurysms do not contain all layers of the arterial wall.

Please rate this question:

Discuss and give feedback


Next question

Groin masses clinical

Groin masses are common and include:

 Herniae
 Lipomas
 Lymph nodes
 Undescended testis
 Femoral aneurysm
 Saphena varix (more a swelling than a mass!)

In the history, features relating to systemic illness and tempo of onset will often give a clue as to the
most likely underlying diagnosis.

Groin lumps- some key questions

 Is there a cough impulse


 Is it pulsatile AND is it expansile (to distinguish between false and true aneurysm)
 Are both testes intra scrotal
 Any lesions in the legs such as malignancy or infections (?lymph nodes)
 Examine the ano rectum as anal cancer may metastasise to the groin
 Is the lump soft, small and very superficial (?lipoma)

Scrotal lumps - some key questions

 Is the lump entirely intra scrotal


 Does it transilluminate (?hydrocele)
 Is there a cough impulse (?hernia)

In most cases a diagnosis can be made clinically. Where it is not clear an ultrasound scan is often
the most convenient next investigation.
Theme: Right iliac fossa pain

A. Urinary tract infection


B. Appendicitis
C. Mittelschmerz
D. Mesenteric adenitis
E. Crohns disease
F. Ulcerative colitis
G. Meckels diverticulum

Please select the most likely cause for right iliac fossa pain for the scenario given. Each option may
be used once, more than once or not at all.

30. A 17 year old male is admitted with lower abdominal discomfort. He has been suffering
from intermittent right iliac fossa pain for the past few months. His past medical history
includes a negative colonoscopy and gastroscopy for iron deficiency anaemia. The pain is
worse after meals. Inflammatory markers are normal.

You answered Urinary tract infection

The correct answer is Meckels diverticulum

This scenario should raise suspicion for Meckels as these may contain ectopic gastric
mucosa which may secrete acid with subsequent bleeding and ulceration.

31. A 14 year old female is admitted with sudden onset right iliac fossa pain. She is otherwise
well and on examination has some right iliac fossa tenderness but no guarding. She is
afebrile. Urinary dipstick is normal. Her previous menstrual period two weeks ago was
normal and pregnancy test is negative.

You answered Urinary tract infection

The correct answer is Mittelschmerz

Typical story and timing for mid cycle pain. Mid cycle pain typically occurs because a
small amount of fluid is released at the time of ovulation. It will usually resolve over 24-48
hours.

32. A 21 year old male is admitted with a 3 month history of intermittent right iliac fossa pain.
He suffers from episodic diarrhoea and has lost 2 kilos in weight. On examination he has
some right iliac fossa tenderness and is febrile.

You answered Urinary tract infection


The correct answer is Crohns disease

Weight loss and chronic symptoms coupled with change in bowel habit should raise
suspicion for Crohns. The presence of intermittent right iliac fossa pain is far more typical
of terminal ileal Crohns disease. Both UC and Crohns may be associated with a low grade
pyrexia. The main concern here would be locally perforated Crohns disease with a small
associated abscess.

Please rate this question:

Discuss and give feedback


Next question

Right iliac fossa pain

Differential diagnosis

Appendicitis  Pain radiating to right iliac fossa


 Anorexia (very common)
 Short history
 Diarrhoea and profuse vomiting rare

Crohn's disease  Often long history


 Signs of malnutrition
 Change in bowel habit, especially diarrhoea

Mesenteric adenitis  Mainly affects children


 Causes include Adenoviruses, Epstein Barr Virus, beta-
haemolytic Streptococcus, Staphylococcus spp., Escherichia
coli,Streptococcus viridans and Yersinia spp.
 Patients have a higher temperature than those with appendicitis
 If laparotomy is performed, enlarged mesenteric lymph nodes will
be present

Diverticulitis  Both left and right sided disease may present with right iliac fossa
pain
 Clinical history may be similar, although some change in bowel
habit is usual
 When suspected, a CT scan may help in refining the diagnosis
Meckel's  A Meckel's diverticulum is a congenital abnormality that is present
diverticulitis in about 2% of the population
 Typically 2 feet proximal to the ileocaecal valve
 May be lined by ectopic gastric mucosal tissue and produce
bleeding

Perforated peptic  This usually produces upper quadrant pain but pain may be lower
ulcer  Perforations typically have a sharp sudden onset of pain in the
history

Incarcerated right  Usually only right iliac fossa pain if right sided or bowel
inguinal or femoral obstruction.
hernia

Bowel perforation  Seldom localised to right iliac fossa, although complete large bowel
secondary to caecal obstruction with caecal distension may cause pain prior to
or colon carcinoma perforation.

Gynaecological  Pelvic inflammatory disease/salpingitis/pelvic abscess/Ectopic


causes pregnancy/Ovarian torsion/Threatened or complete
abortion/Mittelschmerz

Urological causes  Ureteric colic/UTI/Testicular torsion

Other causes  TB/Typhoid/Herpes Zoster/AAA/Situs inversus

Next question
A 78 year old lady presents with colicky abdominal pain and a tender mass in her groin. On
examination; there is a small firm mass below and lateral to the pubic tubercle. Which of the
following is the most likely underlying diagnosis?

Incarcerated inguinal hernia

Thrombophlebitis of a saphena varix

Incarcerated femoral hernia

Incarcerated obturator hernia

Deep vein thrombosis

Femoral hernia = High risk of strangulation (repair urgently)

Femoral herniae account for <10% of all groin hernias. In the scenario the combination of symptoms
of intestinal compromise with a mass in the region of the femoral canal points to femoral hernia as
the most likely cause.
Please rate this question:

Discuss and give feedback


Next question

Femoral canal

The femoral canal lies at the medial aspect of the femoral sheath. The femoral sheath is a fascial
tunnel containing both the femoral artery laterally and femoral vein medially. The canal lies medial to
the vein.

Borders of the femoral canal


Laterally Femoral vein

Medially Lacunar ligament

Anteriorly Inguinal ligament


Posteriorly Pectineal ligament

Image showing dissection of femoral canal

Image sourced from Wikipedia

Contents

 Lymphatic vessels
 Cloquet's lymph node

Physiological significance
Allows the femoral vein to expand to allow for increased venous return to the lower limbs.

Pathological significance
As a potential space, it is the site of femoral hernias. The relatively tight neck places these at high
risk of strangulation.
Which of the following is not a typical feature of acute appendicitis?

Neutrophilia

Profuse vomiting

Anorexia

Low grade pyrexia

Small amounts of protein on urine analysis

Profuse vomiting and diarrhoea are rare in early appendicitis

Whilst patients may vomit once or twice, profuse vomiting is unusual, and would fit more with
gastroenteritis or an ileus. A trace of protein is not an uncommon occurrence in acute appendicitis. A
free lying pelvic appendix may result in localised bladder irritation, with inflammation occurring as a
secondary phenomena. This latter feature may result in patients being incorrectly diagnosed as
having a urinary tract infection. A urine dipstick test is useful in differentiating between the two
conditions.
Please rate this question:

Discuss and give feedback


Next question

Appendicitis

History

 Peri umbilical abdominal pain (visceral stretching of appendix lumen and appendix is mid gut
structure) radiating to the right iliac fossa due to localised parietal peritoneal inflammation.
 Vomit once or twice but marked and persistent vomiting is unusual.
 Diarrhoea is rare. However, pelvic appendicitis may cause localised rectal irritation and some
loose stools. A pelvic abscess may also cause diarrhoea.
 Mild pyrexia is common - temperature is usually 37.5 -38oC. Higher temperatures are more
typical of conditions like mesenteric adenitis.
 Anorexia is very common. It is very unusual for patients with appendicitis to be hungry.

Examination
 Generalised peritonitis if perforation has occurred or localised peritonism.
 Retrocaecal appendicitis may have relatively few signs.
 Digital rectal examination may reveal boggy sensation if pelvic abscess is present, or even
tenderness with a pelvic appendix.

Diagnosis

 Typically raised inflammatory markers coupled with compatible history and examination
findings should be enough to justify appendicectomy.
 Urine analysis may show mild leucocytosis but no nitrites.
 Ultrasound is useful in females where pelvic organ pathology is suspected. Although it is not
always possible to visualise the appendix on ultrasound, the presence of free fluid (always
pathological in males) should raise suspicion.

Ultrasound examination may show evidence of luminal obstruction and thickening of the appendiceal
wall as shown below

Image sourced from Wikipedia

Treatment

 Appendicectomy which can be performed via either an open or laparoscopic approach.


 Administration of metronidazole reduces wound infection rates.
 Patients with perforated appendicitis require copious abdominal lavage.
 Patients without peritonitis who have an appendix mass should receive broad spectrum
antibiotics and consideration given to performing an interval appendicectomy.
 Be wary in the older patients who may have either an underlying caecal malignancy or
perforated sigmoid diverticular disease.

Laparoscopic appendicectomy is becoming increasing popular as demonstrated below

Image sourced from Wikipedia


An 28 year old man presents with a direct inguinal hernia. A decision is made to perform an open
inguinal hernia repair. Which of the following is the best option for abdominal wall reconstruction in
this case?

Suture plication of the transversalis fascia using PDS only

Suture plication of the hernial defect with nylon and placement of prolene mesh anterior
to external oblique

Suture plication of the hernia defect using nylon and re-enforcing with a sutured repair of
the abdominal wall

Sutured repair of the hernial defect with prolene and placement of prolene mesh over the
cord structures in the inguinal canal

Sutured repair of the hernial defect using nylon and placement of a prolene mesh posterior
to the cord structures

Laparoscopic repair- bilateral and recurrent cases

During an inguinal hernia repair in males the cord structures will always lie anterior to the mesh. In
the conventional open repairs the cord structures are mobilised and the mesh placed behind them,
with a slit made to allow passage of the cord structures through the deep inguinal ring. Placement of
the mesh over the cord structures results in chronic pain and usually a higher risk of recurrence.

Laparoscopic inguinal hernia repair is the procedure of choice for bilateral inguinal hernias.

Types of surgery include:

 Onlay mesh repair (Lichtenstein style)


 Inguinal herniorrhaphy
 Shouldice repair
 Darn repair
 Laparoscopic mesh repair

Open mesh repair and laparoscopic repair are the two main procedures in mainstream use. The
Shouldice repair is a useful procedure in cases where a mesh repair would be associated with
increased risk of infection, e.g. repair of case with strangulated bowel, as it avoids the use of mesh.
It is, however, far more technically challenging to perform.
Please rate this question:
Discuss and give feedback
Next question

Inguinal hernia surgery

Inguinal hernias occur when the abdominal viscera protrude through the anterior abdominal wall into
the inguinal canal. They may be classified as being either direct or indirect. The distinction between
these two rests on their relation to Hesselbach's triangle.

Boundaries of Hesselbach's Triangle

 Medial: Rectus abdominis


 Lateral: Inferior epigastric vessels
 Inferior: Inguinal ligament

Image sourced from Wikipedia

Hernias occurring within the triangle tend to be direct and those outside - indirect.

Diagnosis
Most cases are diagnosed clinically, a reducible swelling may be located at the level of the inguinal
canal. Large hernia's may extend down into the male scrotum, these will not trans-illuminate and it is
not possible to "get above" the swelling.
Cases that are unclear on examination, but suspected from the history, may be further investigated
using ultrasound or by performing a herniogram.

Treatment
Hernias associated with few symptoms may be managed conservatively. Symptomatic hernias or
those which are at risk of developing complications are usually treated surgically.
First time hernias may be treated by performing an open inguinal hernia repair; the inguinal canal is
opened, the hernia reduced and the defect repaired. A prosthetic mesh may be placed posterior to
the cord structures to re-enforce the repair and reduce the risk of recurrence.
Recurrent hernias and those which are bilateral are generally managed with a laparoscopic
approach. This may be via an intra or extra peritoneal route. As in open surgery a mesh is deployed.
However, it will typically lie posterior to the deep ring.

Inguinal hernia in children


Inguinal hernias in children are almost always of an indirect type and therefore are usually dealt with
by herniotomy, rather than herniorraphy. Neonatal hernias especially in those children born
prematurely are at highest risk of strangulation and should be repaired urgently. Other hernias may
be repaired on an elective basis.

References
The UK Based National Institute of Clinical Excellence has published guidelines relating to the
choice between open and laparoscopic inguinal hernia repair. Which users may find interesting:

http://guidance.nice.org.uk/TA83/Guidance/pdf/English
Theme: Abdominal closure methods

A. Looped 1 PDS (polydiaxone)


B. Looped 1/0 silk
C. 1/0 Vicryl (polyglactin)
D. 1/0 Vicryl rapide
E. 2/0 Prolene (Polypropylene)
F. Re-inforced 1 Nylon
G. Re-inforced 1/0 Silk
H. Application of VAC system without separation film
I. Application of VAC System with separation film
J. Application of a 'Bogota Bag'

Please select the most appropriate wound closure method (for the deep layer) for the abdominal
surgery described.

36. A 59 year old man with morbid obesity undergoes a laparotomy and Hartmans procedure
for perforated sigmoid diverticular disease. At the conclusion of the procedure the
abdomen cannot be primarily closed. The Vac system is not available for use.

You answered Looped 1 PDS (polydiaxone)

The correct answer is Application of a 'Bogota Bag'

Application of a Bogota bag is safest as attempted closure will almost certainly fail.
Repeat look at 48 hours to determine the best definitive option is needed.

37. A 73 year old lady undergoes a low anterior resection for carcinoma of the rectum.

Looped 1 PDS (polydiaxone)

Mass closure obeying Jenkins rule is required and this states that the suture must be 4
times the length of the wound with tissue bites 1cm deep and 1 cm apart.

38. A 67 year old is returned to theatre after developing a burst abdomen on the ward. She has
originally undergone a right hemicolectomy and the SHO who closed the wound had failed
to tie the midline suture correctly. The wound edges appear healthy.

You answered Looped 1 PDS (polydiaxone)

The correct answer is Re-inforced 1 Nylon

Attempt at re-closing the wound is reasonable, in which case 1 nylon is often used in
preference to a dissolvable suture.

Please rate this question:

Discuss and give feedback


Next question

Abdominal wound dehiscence

 This is a significant problem facing all surgeons who undertake abdominal surgery on a
regular basis. Traditionally, it is said to occur when all layers of an abdominal mass closure
fail and the viscera protrude externally (associated with 30% mortality).
 It can be subdivided into superficial, in which the skin wound alone fails and complete,
implying failure of all layers.

Factors which increase the risk are:


* Malnutrition
* Vitamin deficiencies
* Jaundice
* Steroid use
* Major wound contamination (e.g. faecal peritonitis)
* Poor surgical technique (Mass closure technique is the preferred method-Jenkins Rule)

When sudden full dehiscence occurs the management is as follows:


* Analgesia
* Intravenous fluids
* Intravenous broad spectrum antibiotics
* Coverage of the wound with saline impregnated gauze (on the ward)
* Arrangements made for a return to theatre

Surgical strategy

 Correct the underlying cause (e.g. TPN or NG feed if malnourished)


 Determine the most appropriate strategy for managing the wound

Options
Resuturing of the This may be an option if the wound edges are healthy and there is enough
wound tissue for sufficient coverage. Deep tension sutures are traditionally used for
this purpose.

Application of a This is a clear dressing with removable front. Particularly suitable when some
wound manager granulation tissue is present over the viscera or where there is a high output
bowel fistula present in the dehisced wound.

Application of a This is a clear plastic bag that is cut and sutured to the wound edges and is
'Bogota bag' only a temporary measure to be adopted when the wound cannot be closed
and will necessitate a return to theatre for definitive management.

Application of a These can be safely used BUT ONLY if the correct layer is interposed
VAC dressing between the suction device and the bowel. Failure to adhere to this absolute
system rule will almost invariably result in the development of multiple bowel
fistulae and create an extremely difficult management problem.
Theme: Hernias

A. Umbilical hernia
B. Para umbilical hernia
C. Morgagni hernia
D. Littres hernia
E. Bochdalek hernia
F. Richters hernia
G. Obturator hernia

Please select the hernia that most closely matches the description given. Each option may be used
once, more than once or not at all.

39. A 1 day old infant is born with severe respiratory compromise. On examination he has a
scaphoid abdomen and an absent apex beat.

You answered Umbilical hernia

The correct answer is Bochdalek hernia

Theme from 2011 exam


The large hernia may displace the heart although true dextrocardia is not present. The
associated pulmonary hypoplasia will compromise lung development.

40. A 2 month old infant is troubled by recurrent colicky abdominal pain and intermittent
intestinal obstruction. On imaging the transverse colon is herniated into the thoracic cavity,
through a mid line defect.

You answered Umbilical hernia

The correct answer is Morgagni hernia

Morgagni hernia may contain the transverse colon. Unless there is substantial herniation,
pulmonary hypoplasia is uncommon. As a result, major respiratory compromise is often
absent.

41. A 78 year old lady is admitted with small bowel obstruction, on examination she has a
distended abdomen and the leg is held semi flexed. She has some groin pain radiating to
the ipsilateral knee.

You answered Umbilical hernia

The correct answer is Obturator hernia


The groin swelling in obturator hernia is subtle and hard to elicit clinically. There may be
pain in the region of sensory distribution of the obturator nerve. The defect is usually
repaired from within the abdomen.

Please rate this question:

Discuss and give feedback


Next question

Hernia

Hernias occur when a viscus or part of it protrudes from within its normal anatomical cavity. Specific
hernias are covered under their designated titles the remainder are addressed here.

Spigelian hernia

 Interparietal hernia occurring at the level of the arcuate line


 Rare
 May lie beneath internal oblique muscle. Usually between internal and external oblique
 Equal sex distribution
 Position is lateral to rectus abdominis
 Both open and laparoscopic repair are possible, the former in cases of strangulation

Lumbar hernia
The lumbar triangle (through which these may occur) is bounded by:
Crest of ilium (inferiorly)
External oblique (laterally)
Latissimus dorsi (medially)

 Primary lumbar herniae are rare, most are incisional hernias following renal surgery

- Direct anatomical repair with or without mesh re-enforcement is the procedure of choice

Obturator hernia

 Herniation through the obturator canal


 Commoner in females
 Usually lies behind pectineus muscle
 Elective diagnosis is unusual most will present acutely with obstruction
 When presenting acutely most cases with require laparotomy or laparoscopy (and small
bowel resection if indicated)
Richters hernia

 Condition in which part of the wall of the small bowel (usually the anti mesenteric border) is
strangulated within a hernia (of any type)
 They do not present with typical features of intestinal obstruction as lumenal patency is
preserved
 Where vomiting is prominent it usually occurs as a result of paralytic ileus from peritonitis (as
these hernias may perforate)

Incisional hernia

 Occur through sites of surgical access into the abdominal cavity


 Most common following surgical wound infection
 To minimise following midline laparotomy Jenkins Rule should be followed and this
necessitates a suture length 4x length of incision with bites taken at 1cm intervals, 1 cm from
the wound edge
 Repair may be performed either at open surgery or laparoscopically and a wide variety of
techniques are described

Bochdalek hernia

 Typically congenital diaphragmatic hernia


 85% cases are located in the left hemi diaphragm
 Associated with lung hypoplasia on the affected side
 More common in males
 Associated with other birth defects
 May contain stomach
 May be treated by direct anatomical apposition or placement of mesh. In infants that have
severe respiratory compromise mechanical ventilation may be needed and mortality rate is
high

Morgagni Hernia

 Rare type of diaphragmatic hernia (approx 2% cases)


 Herniation through foramen of Morgagni
 Usually located on the right and tend to be less symptomatic
 More advanced cases may contain transverse colon
 As defects are small pulmonary hypoplasia is less common
 Direct anatomical repair is performed

Umbilical hernia
 Hernia through weak umbilicus
 Usually presents in childhood
 Often symptomatic
 Equal sex incidence
 95% will resolve by the age of 2 years
 Surgery performed after the third birthday

Paraumbilical hernia

 Usually a condition of adulthood


 Defect is in the linea alba
 More common in females
 Multiparity and obesity are risk factors
 Traditionally repaired using Mayos technique - overlapping repair, mesh may be used though
not if small bowel resection is required owing to acute strangulation

Littres hernia

 Hernia containing Meckels diverticulum


 Resection of the diverticulum is usually required and this will preclude a mesh repair

Next question
Theme: Right iliac fossa pain

A. Open Appendicectomy
B. Laparoscopic appendicectomy
C. Laparotomy
D. CT Scan
E. Colonoscopy
F. Ultrasound scan abdomen/pelvis
G. Active observation

For each scenario please select the most appropriate management option from the list. Each option
may be used once, more than once or not at all.

42. A 21 year old women is admitted with a 48 hour history of worsening right iliac fossa
pain. She has been nauseated and vomited twice. On examination, she is markedly tender
in the right iliac fossa with localised guarding. Vaginal examination is unremarkable.
Urine dipstick (including beta HCG) is negative. Blood tests show a WCC of 13.5 and
CRP 70.

You answered Open Appendicectomy

The correct answer is Laparoscopic appendicectomy

She is likely to have appendicitis. In women of this age there is always diagnostic
uncertainty. With a normal vaginal exam laparoscopy would be preferred over USS.

43. An 8 year old boy presents with a 4 hour history of right iliac fossa pain with nausea and
vomiting. He has been back at school for two days after being kept home with a flu like
illness. On examination he is tender in the right iliac fossa, although his abdomen is soft.
Temperature is 38.3oc. Blood tests show a CRP of 40 and a WCC of 8.1.

You answered Open Appendicectomy

The correct answer is Active observation

This is mesenteric adenitis. Note history of flu like illness and temp > 38 o c.
The decision as to how to manage this situation is based on the abdominal findings.
Patients with localising signs such as guarding or peritonism should undergo surgery.

44. A 21 year old women presents with right iliac fossa pain. She reports some bloodstained
vaginal discharge. She has a HR of 65 bpm.

You answered Open Appendicectomy


The correct answer is Ultrasound scan abdomen/pelvis

This patient is suspected of having an ectopic pregnancy. She needs an urgent β HCG and
USS of the pelvis. If she were haemodynamically unstable then laparotomy would be
indicated.

Please rate this question:

Discuss and give feedback


Next question

Right iliac fossa pain

Differential diagnosis

Appendicitis  Pain radiating to right iliac fossa


 Anorexia (very common)
 Short history
 Diarrhoea and profuse vomiting rare

Crohn's disease  Often long history


 Signs of malnutrition
 Change in bowel habit, especially diarrhoea

Mesenteric adenitis  Mainly affects children


 Causes include Adenoviruses, Epstein Barr Virus, beta-
haemolytic Streptococcus, Staphylococcus spp., Escherichia
coli,Streptococcus viridans and Yersinia spp.
 Patients have a higher temperature than those with appendicitis
 If laparotomy is performed, enlarged mesenteric lymph nodes will
be present

Diverticulitis  Both left and right sided disease may present with right iliac fossa
pain
 Clinical history may be similar, although some change in bowel
habit is usual
 When suspected, a CT scan may help in refining the diagnosis

Meckel's  A Meckel's diverticulum is a congenital abnormality that is present


diverticulitis in about 2% of the population
 Typically 2 feet proximal to the ileocaecal valve
 May be lined by ectopic gastric mucosal tissue and produce
bleeding

Perforated peptic  This usually produces upper quadrant pain but pain may be lower
ulcer  Perforations typically have a sharp sudden onset of pain in the
history

Incarcerated right  Usually only right iliac fossa pain if right sided or bowel
inguinal or femoral obstruction.
hernia

Bowel perforation  Seldom localised to right iliac fossa, although complete large bowel
secondary to caecal obstruction with caecal distension may cause pain prior to
or colon carcinoma perforation.

Gynaecological  Pelvic inflammatory disease/salpingitis/pelvic abscess/Ectopic


causes pregnancy/Ovarian torsion/Threatened or complete
abortion/Mittelschmerz

Urological causes  Ureteric colic/UTI/Testicular torsion

Other causes  TB/Typhoid/Herpes Zoster/AAA/Situs inversus


Which of the following is not a typical feature of irritable bowel syndrome?

A change in the consistency of stools

Abdominal pain relieved with defecation

A change in frequency of defecation

Abdominal bloating

Pain at a single fixed site

The pain or discomfort of IBS is typically migratory and variable in intensity. Pain at a fixed site is
suggestive of malignancy.
Abdominal bloating is an extremely common feature.
Please rate this question:

Discuss and give feedback


Next question

Irritable bowel syndrome

The diagnosis of irritable bowel syndrome is made according to the ROME III diagnostic criteria
which state:

Recurrent abdominal pain or discomfort at 3 days per month for the past 3 months
associated with two or more of the following:

 Improvement with defecation.


 Onset associated with a change in the frequency of stool.
 Onset associated with a change in the form of the stool.

Features such as lethargy, nausea, backache and bladder symptoms may also support the
diagnosis

Red flag features should be inquired about:

 Rectal bleeding
 Unexplained/unintentional weight loss
 Family history of bowel or ovarian cancer
 Onset after 60 years of age

Suggested investigations are:

 Full blood count


 ESR/CRP
 Coeliac disease screen (tissue transglutaminase antibodies)
 Colonoscopy (if worrying symptoms, positive family history)
 Thyroid function tests
 Glucose (ensure not diabetic)

The NICE criteria state that blood tests alone will suffice in people fulfilling the diagnostic criteria. We
would point out that luminal colonic studies should be considered early in patients with altered bowel
habit referred to hospital and a diagnosis of IBS should still be largely one of exclusion.

Treatment

 Usually reduce fibre intake.


 Tailored prescriptions of laxatives or loperamide according to clinical picture.
 Dietary modification (caffeine avoidance, less carbonated drinks).
 Consider low dose tricyclic antidepressants if pain is a dominant symptom.
 Biofeedback may help.
Theme: Causes of abdominal pain

A. Acute on chronic mesenteric ischaemia


B. Ruptured aortic aneurysm
C. Acute Pancreatitis
D. Acute mesenteric embolus
E. Acute appendicitis
F. Chronic pancreatitis
G. Mesenteric vein thrombosis

Please select the most likely underlying diagnosis from the list above. Each option may be used
once, more than once or not at all.

46. A 41 year old man is admitted with peritonitis secondary to a perforated appendix. He is
treated with a laparoscopic appendicectomy but has a stormy post operative course. He is
now developing increasing abdominal pain and has been vomiting. A laparotomy is
performed and at operation a large amount of small bowel shows evidence of patchy areas
of infarction.

You answered Acute on chronic mesenteric ischaemia

The correct answer is Mesenteric vein thrombosis

Mesenteric vein thrombosis may complicate severe intra abdominal sepsis and when it
progresses may impair bowel perfusion. The serosa is quite resistant to ischaemia so in this
case the appearances are usually patchy.

47. A 68 year old man is admitted with abdominal pain and vomiting of 48 hours duration, the
pain radiates to his back and he has required a considerable amount of volume
replacement. Amylase is 741.

You answered Acute on chronic mesenteric ischaemia

The correct answer is Acute Pancreatitis

Although back pain and abdominal pain coupled with haemodynamic compromise may
suggest ruptured AAA the 48 hour history and amylase >3 times normal go against this
diagnosis.

48. A 79 year old lady develops sudden onset of abdominal pain and collapses, she has passed
a large amount of diarrhoea. In casualty her pH is 7.35 and WCC is 18.

You answered Acute on chronic mesenteric ischaemia


The correct answer is Acute mesenteric embolus

Although mesenteric infarct may raise the lactate the pH may be raised often secondary to
vomiting.

Please rate this question:

Discuss and give feedback


Next question

Mesenteric vessel disease

Mesenteric ischaemia accounts for 1 in 1000 acute surgical admissions. It is primarily caused by
arterial embolism resulting in infarction of the colon. It is more likely to occur in areas such as the
splenic flexure that are located at the borders of the territory supplied by the superior and inferior
mesenteric arteries.

Types
Acute mesenteric  Sudden onset abdominal pain followed by profuse diarrhoea.
embolus  May be associated with vomiting.
(commonest 50%)  Rapid clinical deterioration.
 Serological tests: WCC, lactate, amylase may all be abnormal
particularly in established disease. These can be normal in the early
phases.

Acute on chronic  Usually longer prodromal history.


mesenteric  Post prandial abdominal discomfort and weight loss are dominant
ischaemia features. Patients will usually present with an acute on chronic
event, but otherwise will tend not to present until mesenteric flow is
reduced by greater than 80%.
 When acute thrombosis occurs presentation may be as above. In the
chronic setting the symptoms will often be those of ischaemic
colitis (mucosa is the most sensitive area to this insult).

Mesenteric vein  Usually a history over weeks.


thrombosis  Overt abdominal signs and symptoms will not occur until venous
thrombosis has reached a stage to compromise arterial inflow.
 Thrombophilia accounts for 60% of cases.

Low flow  This occurs in patients with multiple co morbidities in whom


mesenteric mesenteric perfusion is significantly compromised by overuse of
infarction inotropes or background cardiovascular compromise.
 The end result is that the bowel is not adequately perfused and
infarcts occur from the mucosa outwards.

Diagnosis

 Serological tests: WCC, lactate, CRP, amylase (can be normal in early disease).
 Cornerstone for diagnosis of arterial AND venous mesenteric disease is CT angiography
scanning in the arterial phase with thin slices (<5mm). Venous phase contrast is not helpful.
 SMA duplex USS is useful in the evaluation of proximal SMA disease in patients with chronic
mesenteric ischaemia.
 MRI is of limited use due to gut peristalsis and movement artefact.

Management

 Overt signs of peritonism: Laparotomy


 Mesenteric vein thrombosis: If no peritonism: Medical management with IV heparin
 At operation limited resection of frankly necrotic bowel with view to relook laparotomy at 24-
48h. In the interim urgent bowel revascularisation via endovascular (preferred) or surgery.

Prognosis
Overall poor. Best outlook is from an acute ischaemia from an embolic event where surgery occurs
within 12h. Survival may be 50%. This falls to 30% with treatment delay. The other conditions carry
worse survival figures.
Theme: Groin masses

A. Femoral aneurysm
B. Lymphadenitis
C. Saphena varix
D. Femoral hernia
E. Indirect inguinal hernia
F. Direct inguinal hernia
G. Psoas abscess

What is the likely diagnosis for the groin mass described? Each option may be used once, more than
once or not at all.

49. A 3 year old boy is referred to the clinic with a scrotal swelling. On examination the mass
does not transilluminate and it is impossible to palpate normal cord above it.

You answered Femoral aneurysm

The correct answer is Indirect inguinal hernia

This is likely to be an indirect hernia. In children these arise from persistent processus
vaginalis and require herniotomy.

50. A 52 year old obese lady reports a painless mass in the groin area. A mass is noted on
coughing. It is below and lateral to the pubic tubercle.

You answered Femoral aneurysm

The correct answer is Femoral hernia

A mass below and lateral to the pubic tubercle is indicative of a femoral hernia.

51. A 21 year old man is admitted with a tender mass in the right groin, fevers and sweats. He
is on multiple medical therapy for HIV infection. On examination he has a swelling in his
right groin, hip extension exacerbates the pain.

You answered Femoral aneurysm

The correct answer is Psoas abscess

Psoas abscesses may be either primary or secondary. Primary cases often occur in the
immunosuppressed and may occur as a result of haematogenous spread. Secondary cases
may complicated intra abdominal diseases such as Crohns. Patients usually present with
low back pain and if the abscess is extensive a mass that may be localised to the inguinal
region or femoral triangle . Smaller collections may be percutaneously drained. If the
collection is larger, or the percutaneous route fails, then surgery (via a retroperitoneal
approach) should be performed.

Please rate this question:

Discuss and give feedback


Next question

Groin masses clinical

Groin masses are common and include:

 Herniae
 Lipomas
 Lymph nodes
 Undescended testis
 Femoral aneurysm
 Saphena varix (more a swelling than a mass!)

In the history, features relating to systemic illness and tempo of onset will often give a clue as to the
most likely underlying diagnosis.

Groin lumps- some key questions

 Is there a cough impulse


 Is it pulsatile AND is it expansile (to distinguish between false and true aneurysm)
 Are both testes intra scrotal
 Any lesions in the legs such as malignancy or infections (?lymph nodes)
 Examine the ano rectum as anal cancer may metastasise to the groin
 Is the lump soft, small and very superficial (?lipoma)

Scrotal lumps - some key questions

 Is the lump entirely intra scrotal


 Does it transilluminate (?hydrocele)
 Is there a cough impulse (?hernia)

In most cases a diagnosis can be made clinically. Where it is not clear an ultrasound scan is often
the most convenient next investigation.
A 56 year old lady is admitted with colicky abdominal pain. A plain x-ray is performed. Which of the
following should not show fluid levels on a plain abdominal film?

Stomach

Jejunum

Ileum

Caecum

Descending colon

Fluid levels in the distal colon are nearly always pathological. In general contents of the left colon
transit quickly and are seldom held in situ for long periods, the content is also more solid.
Please rate this question:

Discuss and give feedback


Next question

Abdominal radiology

Plain abdominal x-rays are often used as a first line investigation in patients with acute abdominal
pain. A plain abdominal film may demonstrate free air, evidence of bowel obstruction and possibly
other causes of pain (e.g. renal or gallbladder stones).
Investigation of potential visceral perforation is usually best performed by obtaining an erect chest x-
ray, as this is a more sensitive investigation for suspected visceral perforation.

Features which are usually abnormal

 Large amounts of free air (colonic perforation), smaller volumes seen with more proximal
perforations.
 A positive Riglers sign (gas on both sides of the bowel wall).
 Caecal diameter of >8cm
 Fluid levels in the colon
 Ground glass appearance to film (usually due to large amounts of free fluid).
 Sentinel loop in patients with inflammation of other organs (e.g. pancreatitis).

Features which should be expected/ or occur without pathology


 In Chiladitis sign, a loop of bowel may be interposed between the liver and diaphragm, giving
the mistaken impression that free air is present.
 Following ERCP (and sphincterotomy) air may be identified in the biliary tree.
 Free intra abdominal air following laparoscopy / laparotomy, although usually dissipates after
48-72 hours.
A 56 year old lady presents with a large bowel obstruction and abdominal distension. Which of the
following confirmatory tests should be performed prior to surgery?

Abdominal ultrasound scan

Barium enema

Rectal MRI Scan

Endoanal ultrasound scan

Gastrograffin enema

Patients with suspected large bowel obstruction due to tumour should have this confirmed with
gastrograffin enema, sigmoidoscopy or CT scanning prior to surgery.

Patients with clinical evidence of large bowel obstruction, should have the presence or absence of
an obstructing lesion confirmed prior to surgery. This is because colonic pseudo-obstruction may
produce a similar radiological picture. A gastrograffin enema is the traditional test, as barium is too
toxic if it spills into the abdominal cavity. An MRI scan will not provide the relevant information,
unless the lesion is rectal and below the peritoneal reflection.A CT scan would be an acceptable
alternative.
Please rate this question:

Discuss and give feedback


Next question

Abdominal radiology

Plain abdominal x-rays are often used as a first line investigation in patients with acute abdominal
pain. A plain abdominal film may demonstrate free air, evidence of bowel obstruction and possibly
other causes of pain (e.g. renal or gallbladder stones).
Investigation of potential visceral perforation is usually best performed by obtaining an erect chest x-
ray, as this is a more sensitive investigation for suspected visceral perforation.

Features which are usually abnormal

 Large amounts of free air (colonic perforation), smaller volumes seen with more proximal
perforations.
 A positive Riglers sign (gas on both sides of the bowel wall).
 Caecal diameter of >8cm
 Fluid levels in the colon
 Ground glass appearance to film (usually due to large amounts of free fluid).
 Sentinel loop in patients with inflammation of other organs (e.g. pancreatitis).

Features which should be expected/ or occur without pathology

 In Chiladitis sign, a loop of bowel may be interposed between the liver and diaphragm, giving
the mistaken impression that free air is present.
 Following ERCP (and sphincterotomy) air may be identified in the biliary tree.
 Free intra abdominal air following laparoscopy / laparotomy, although usually dissipates after
48-72 hours.
Which of these factors does not increase the risk of abdominal wound dehiscence following
laparotomy?

Jaundice

Abdominal compartment syndrome

Poorly controlled diabetes mellitus

Administration of intravenous steroids

Use of Ketamine as an anaesthetic agent

Ketamine does not affect healing. All the other situations in the list carry a strong association with
poor healing and risk of dehisence.
Please rate this question:

Discuss and give feedback


Next question

Abdominal wound dehiscence

 This is a significant problem facing all surgeons who undertake abdominal surgery on a
regular basis. Traditionally, it is said to occur when all layers of an abdominal mass closure
fail and the viscera protrude externally (associated with 30% mortality).
 It can be subdivided into superficial, in which the skin wound alone fails and complete,
implying failure of all layers.

Factors which increase the risk are:


* Malnutrition
* Vitamin deficiencies
* Jaundice
* Steroid use
* Major wound contamination (e.g. faecal peritonitis)
* Poor surgical technique (Mass closure technique is the preferred method-Jenkins Rule)

When sudden full dehiscence occurs the management is as follows:


* Analgesia
* Intravenous fluids
* Intravenous broad spectrum antibiotics
* Coverage of the wound with saline impregnated gauze (on the ward)
* Arrangements made for a return to theatre

Surgical strategy

 Correct the underlying cause (e.g. TPN or NG feed if malnourished)


 Determine the most appropriate strategy for managing the wound

Options
Resuturing of the This may be an option if the wound edges are healthy and there is enough
wound tissue for sufficient coverage. Deep tension sutures are traditionally used for
this purpose.

Application of a This is a clear dressing with removable front. Particularly suitable when some
wound manager granulation tissue is present over the viscera or where there is a high output
bowel fistula present in the dehisced wound.

Application of a This is a clear plastic bag that is cut and sutured to the wound edges and is
'Bogota bag' only a temporary measure to be adopted when the wound cannot be closed
and will necessitate a return to theatre for definitive management.

Application of a These can be safely used BUT ONLY if the correct layer is interposed
VAC dressing between the suction device and the bowel. Failure to adhere to this absolute
system rule will almost invariably result in the development of multiple bowel
fistulae and create an extremely difficult management problem.

Next question
Theme: Causes of diarrhoea

A. Campylobacter jejuni infection


B. Salmonella gastroenteritis infection
C. Crohns disease
D. Ulcerative colitis
E. Irritable bowel syndrome
F. Ischaemic colitis
G. Laxative abuse
H. Clostridium difficile infection

Please select the most likely cause of diarrhoea for each scenario given. Each option may be used
once, more than once or not at all.

55. A 23 year old lady has suffered from diarrhoea for 8 months, she has also lost 2 Kg in
weight. At colonoscopy appearances of melanosis coli are identified and confirmed on
biopsy

You answered Campylobacter jejuni infection

The correct answer is Laxative abuse

This may occur as a result of laxative abuse and consists of lipofuschin laden marcophages
that appear brown.

56. A 68 year old lady has recently undergone an abdominal aortic aneurysm repair. The
operation was performed electively and was uncomplicated. Since surgery she has had
repeated episodes of diarrhoea.

You answered Campylobacter jejuni infection

The correct answer is Ischaemic colitis

The IMA is commonly ligated during an AAA repair and this may then render the left
colon relatively ischaemic, thereby causing mesenteric colitis. Treatment is supportive and
most cases will settle with conservative management.

57. A 23 year old man is admitted to hospital with diarrhoea and severe abdominal pain. He
was previously well and his illness has lasted 18 hours.

Campylobacter jejuni infection

Severe abdominal pain tends to favour Campylobacter infection.


Please rate this question:

Discuss and give feedback


Next question

Diarrhoea

World Health Organisation definitions


Diarrhoea: > 3 loose or watery stool per day
Acute diarrhoea < 14 days
Chronic diarrhoea > 14 days

Acute Diarrhoea

Gastroenteritis May be accompanied by abdominal pain or nausea/vomiting

Diverticulitis Classically causes left lower quadrant pain, diarrhoea and fever

Antibiotic therapy More common with broad spectrum antibiotics


Clostridium difficile is also seen with antibiotic use

Constipation causing A history of alternating diarrhoea and constipation may be


overflow given
May lead to faecal incontinence in the elderly

Chronic
Diarrhoea

Irritable bowel Extremely common. The most consistent features are abdominal pain, bloating
syndrome and change in bowel habit. Patients may be divided into those with diarrhoea
predominant IBS and those with constipation predominant IBS.
Features such as lethargy, nausea, backache and bladder symptoms may also be
present

Ulcerative Bloody diarrhoea may be seen. Crampy abdominal pain and weight loss are also
Chronic
Diarrhoea

colitis common. Faecal urgency and tenesmus may occur

Crohn's Crampy abdominal pains and diarrhoea. Bloody diarrhoea less common than in
disease ulcerative colitis. Other features include malabsorption, mouth ulcers perianal
disease and intestinal obstruction

Colorectal Symptoms depend on the site of the lesion but include diarrhoea, rectal
cancer bleeding, anaemia and constitutional symptoms e.g. Weight loss and anorexia

Coeliac disease  In children may present with failure to thrive, diarrhoea and abdominal
distension
 In adults lethargy, anaemia, diarrhoea and weight loss are seen. Other
autoimmune conditions may coexist

Other conditions associated with diarrhoea include:

 Thyrotoxicosis
 Laxative abuse
 Appendicitis with pelvic abscess or pelvic appendix
 Radiation enteritis

Diagnosis
Stool culture
Abdominal and digital rectal examination
Consider colonoscopy (radiological studies unhelpful)
Thyroid function tests, serum calcium, anti endomysial antibodies, glucose
Next question
A 6 year old child presents with colicky abdominal pain, vomiting and the passage of red current jelly
stool per rectum. On examination the child has a tender abdomen and a palpable mass in the right
upper quadrant. Imaging shows an intussusception. Which of the conditions below is least
recognised as a precipitant?

Inflammation of Payers patches

Cystic fibrosis

Meckels diverticulum

Mesenteric cyst

Mucosal polyps

Mesenteric cysts may be associated with intra abdominal catastrophes where these occur they are
typically either intestinal volvulus or intestinal infarction. They seldom cause intussusception. Cystic
fibrosis may lead to the formation of meconium ileus equivalent and plugs may occasionally serve as
the lead points for an intussusception.

Please rate this question:

Discuss and give feedback

Next question

Intussusception- Paediatric

Intussusception typically presents with colicky abdominal pain and vomiting. The telescoping of the
bowel produces mucosal ischaemia and bleeding may occur resulting in the passage of "red current
jelly" stools. Recognised causes include lumenal pathologies such as polyps, lymphadenopathy and
diseases such as cystic fibrosis. Idiopathic intussceception of the ileocaecal valve and terminal ileum
is the most common variant and typically affects young children and toddlers.
The diagnosis is usually made by abdominal ultrasound investigation. The decision as to the optimal
treatment is dictated by the patients physiological status and abdominal signs. In general, children
who are unstable with localising peritoneal signs should undergo laparotomy, as should those in
whom attempted radiological reduction has failed.
In relatively well children without localising signs attempted pneumatic reduction under fluroscopic
guidance is the usual treatment.
Which one of the following is least likely to cause malabsorption?

Primary biliary cirrhosis

Ileo-colic bypass

Chronic pancreatitis

Whipples disease

Hartmans procedure

In a Hartmans procedure the sigmoid colon is removed and an end colostomy is fashioned. The
bowel remains in continuity and no absorptive ability is lost.
An ileo-colic bypass leaves a redundant loop of small bowel in continuity, where the contents will
stagnate and bacterial overgrowth will occur. Therefore this is recognised cause of malabsorption.
Please rate this question:

Discuss and give feedback


Next question

Malabsorption

Malabsorption is characterised by diarrhoea, steatorrhoea and weight loss. Causes may be broadly
divided into intestinal (e.g. villous atrophy), pancreatic (deficiency of pancreatic enzyme production
or secretion) and biliary (deficiency of bile-salts needed for emulsification of fats)

Intestinal causes of malabsorption

 coeliac disease
 Crohn's disease
 tropical sprue
 Whipple's disease
 Giardiasis
 brush border enzyme deficiencies (e.g. lactase insufficiency)

Pancreatic causes of malabsorption

 chronic pancreatitis
 cystic fibrosis
 pancreatic cancer

Biliary causes of malabsorption

 biliary obstruction
 primary biliary cirrhosis

Other causes

 bacterial overgrowth (e.g. systemic sclerosis, diverticulae, blind loop)


 short bowel syndrome
 lymphoma
Theme: Intra abdominal malignancies

A. Metastatic adenocarcinoma of the pancreas


B. Metastatic appendiceal carcinoid
C. Metastatic colonic cancer
D. Pseudomyxoma peritonei
E. MALT lymphoma
F. Retroperitoneal liposarcoma
G. Retroperitoneal fibrosis

For the disease given please give the most likely primary disease process. Each option may be used
once, more than once or not at all.

60. A 32 year old man is admitted with a distended tense abdomen. He previously underwent a
difficult appendicectomy 1 year previously and was discharged. At laparotomy the
abdomen is filled with a gelatinous substance.

You answered Metastatic adenocarcinoma of the pancreas

The correct answer is Pseudomyxoma peritonei

Pseudomyxoma is classically associated with mucin production and the appendix is the
commonest source.

61. A 62 year old man is admitted with dull lower back pain and abdominal discomfort. On
examination he is hypertensive and a lower abdominal fullness is elicited on examination.
An abdominal ultrasound demonstrates hydronephrosis and intravenous urography
demonstrated medially displaced ureters. A CT scan shows a periaortic mass.

You answered Metastatic adenocarcinoma of the pancreas

The correct answer is Retroperitoneal fibrosis

Retroperitoneal fibrosis is an uncommon condition and its aetiology is poorly understood.


In a significant proportion the ureters are displaced medially. In most retroperitoneal
malignancies they are displaced laterally. Hypertension is another common finding. A CT
scan will often show a para-aortic mass

62. A 48 year old lady is admitted with abdominal distension. On examination she is cachectic
and has ascites. Her CA19-9 returns highly elevated.

Metastatic adenocarcinoma of the pancreas


Although not specific CA 19-9 in the context of this history is highly suggestive of
pancreatic cancer over the other scenarios.
Pseudomyxoma peritoneii- Curative treatment is peritonectomy (Sugarbaker procedure) and heated
intra peritoneal chemotherapy.

Please rate this question:

Discuss and give feedback


Next question

Pseudomyxoma Peritonei

 Rare mucinous tumour


 Most commonly arising from the appendix (other abdominal viscera are also recognised as
primary sites)
 Incidence of 1-2/1,000,000 per year
 The disease is characterised by the accumulation of large amounts of mucinous material in
the abdominal cavity

Treatment
Is usually surgical and consists of cytoreductive surgery (and often peritonectomy c.f Sugarbaker
procedure) combined with intra peritoneal chemotherapy with mitomycin C.

Survival is related to the quality of primary treatment and in Sugarbakers own centre 5 year survival
rates of 75% have been quoted. Patients with disseminated intraperitoneal malignancy from another
source fare far worse.
In selected patients a second look laparotomy is advocated and some practice this routinely.
Theme: Abdominal pain

A. Acute mesenteric embolus


B. Acute on chronic mesenteric ischaemia
C. Mesenteric vein thrombosis
D. Ruptured abdominal aortic aneurysm
E. Pancreatitis
F. Appendicitis
G. Acute cholecystitis

Please select the most likely underlying diagnosis from the list above. Each option may be used
once, more than once or not at all.

63. A 72 year old man collapses with sudden onset abdominal pain. He has been suffering
from back pain recently and has been taking ibuprofen.

You answered Acute mesenteric embolus

The correct answer is Ruptured abdominal aortic aneurysm

Back pain is a common feature with expanding aneurysms and may be miss classified as
being of musculoskeletal origin.

64. A 73 year old women collapses with sudden onset of abdominal pain and the passes a large
amount of diarrhoea. On admission she is vomiting repeatedly. She has recently been
discharged from hospital following a myocardial infarct but recovered well.

Acute mesenteric embolus

Sudden onset of abdominal pain and forceful bowel evacuation are features of acute
mesenteric infarct.

65. A 66 year old man has been suffering from weight loss and develops severe abdominal
pain. He is admitted to hospital and undergoes a laparotomy. At operation the entire small
bowel is infarcted and only the left colon is viable.

You answered Acute mesenteric embolus

The correct answer is Acute on chronic mesenteric ischaemia

This man is likely to have underlying chronic mesenteric vascular disease. Only 15% of
emboli will occlude SMA orifice leading to entire small bowel infarct. The background
history of weight loss also favours an acute on chronic event.
Please rate this question:

Discuss and give feedback


Next question

Mesenteric vessel disease

Mesenteric ischaemia accounts for 1 in 1000 acute surgical admissions. It is primarily caused by
arterial embolism resulting in infarction of the colon. It is more likely to occur in areas such as the
splenic flexure that are located at the borders of the territory supplied by the superior and inferior
mesenteric arteries.

Types
Acute mesenteric  Sudden onset abdominal pain followed by profuse diarrhoea.
embolus  May be associated with vomiting.
(commonest 50%)  Rapid clinical deterioration.
 Serological tests: WCC, lactate, amylase may all be abnormal
particularly in established disease. These can be normal in the early
phases.

Acute on chronic  Usually longer prodromal history.


mesenteric  Post prandial abdominal discomfort and weight loss are dominant
ischaemia features. Patients will usually present with an acute on chronic
event, but otherwise will tend not to present until mesenteric flow is
reduced by greater than 80%.
 When acute thrombosis occurs presentation may be as above. In the
chronic setting the symptoms will often be those of ischaemic
colitis (mucosa is the most sensitive area to this insult).

Mesenteric vein  Usually a history over weeks.


thrombosis  Overt abdominal signs and symptoms will not occur until venous
thrombosis has reached a stage to compromise arterial inflow.
 Thrombophilia accounts for 60% of cases.

Low flow  This occurs in patients with multiple co morbidities in whom


mesenteric mesenteric perfusion is significantly compromised by overuse of
infarction inotropes or background cardiovascular compromise.
 The end result is that the bowel is not adequately perfused and
infarcts occur from the mucosa outwards.
Diagnosis

 Serological tests: WCC, lactate, CRP, amylase (can be normal in early disease).
 Cornerstone for diagnosis of arterial AND venous mesenteric disease is CT angiography
scanning in the arterial phase with thin slices (<5mm). Venous phase contrast is not helpful.
 SMA duplex USS is useful in the evaluation of proximal SMA disease in patients with chronic
mesenteric ischaemia.
 MRI is of limited use due to gut peristalsis and movement artefact.

Management

 Overt signs of peritonism: Laparotomy


 Mesenteric vein thrombosis: If no peritonism: Medical management with IV heparin
 At operation limited resection of frankly necrotic bowel with view to relook laparotomy at 24-
48h. In the interim urgent bowel revascularisation via endovascular (preferred) or surgery.

Prognosis
Overall poor. Best outlook is from an acute ischaemia from an embolic event where surgery occurs
within 12h. Survival may be 50%. This falls to 30% with treatment delay. The other conditions carry
worse survival figures.
Theme: Surgical incisions

A. Lanz incision
B. Gridiron incision
C. Kochers incision
D. Rutherford Morrison
E. Rooftop incision
F. McEvedy Incision
G. Lotheissen Incision

Please select the most appropriate incision for the procedure described. Each option may be used
once, more than once or not at all.

66. A 78 year old lady is admitted with an incarcerated femoral hernia. Abdominal signs are
absent and there are no symptoms of obstruction. AXR is normal.

You answered Lanz incision

The correct answer is McEvedy Incision

From the list, the McEvedy approach is the most appropriate. The Lotheissen incision may
compromise the posterior wall of the inguinal canal and is best avoided. The author prefers
a limited pfannenstial type incision for this procedure, as it gives better control of the
hernia, but this is not on the list.

67. A 15 year old girl presents with right iliac fossa pain and guarding, pregnancy test is
negative and WCC is 16.

Lanz incision

She requires an appendicectomy. Although there is an increasing vogue for performing


this procedure laparoscopically, an open procedure is entirely suitable. However, although
both a Gridiron and Lanz incision are suitable for appendicectomy a Lanz will give a
superior cosmetic result and would be the preferred option for most young females.

68. A 45 year old man is due to undergo a live donor renal transplant. This will be his first
procedure.

You answered Lanz incision

The correct answer is Rutherford Morrison

The Rutherford Morrison incision will typically give access to the iliac vessels and bladder
for the procedure.

Please rate this question:

Discuss and give feedback


Next question

Abdominal incisions

Midline incision  Commonest approach to the abdomen


 Structures divided: linea alba, transversalis fascia, extraperitoneal fat,
peritoneum (avoid falciform ligament above the umbilicus)
 Bladder can be accessed via an extraperitoneal approach through the
space of Retzius

Paramedian  Parallel to the midline (about 3-4cm)


incision  Structures divided/retracted: anterior rectus sheath, rectus (retracted),
posterior rectus sheath, transversalis fascia, extraperitoneal fat,
peritoneum
 Incision is closed in layers

Battle  Similar location to paramedian but rectus displaced medially (and thus
denervated)
 Now seldom used

Kocher's Incision under right subcostal margin e.g. Cholecystectomy (open)

Lanz Incision in right iliac fossa e.g. Appendicectomy

Gridiron Oblique incision centered over McBurneys point- usually appendicectomy


(less cosmetically acceptable than Lanz

Gable Rooftop incision

Pfannenstiel's Transverse supra pubic, primarily used to access pelvic organs

McEvedy's Groin incision e.g. Emergency repair strangulated femoral hernia

Rutherford Extraperitoneal approach to left or right lower quadrants. Gives excellent


Morrison access to iliac vessels and is the approach of choice for first time renal
transplantation.
Image sourced from Wikipedia

Next question
Theme: Acute abdominal pain

A. Appendicitis

B. Henoch Schonlein purpura

C. Diabetes mellitus

D. Intussusception

E. Mittelschmerz

F. Pneumonia

G. Sickle cell crisis

H. Spontaneous bacterial peritonitis

I. Rupure of follicular cyst

Please select the most likely cause of abdominal pain for the scenario given. Each option may be
used once, more than once or not at all.

69. An 11 month-old girl develops sudden onset abdominal pain. She has a high pitched scream and
draws up her legs. Her BP is 90/40 mm/Hg, her pulse 118/min and abdominal examination is
normal.

You answered Appendicitis

The correct answer is Intussusception

Intussusception should be considered in toddlers and infants presenting with screaming attacks.
The child often has a history of being unwell for one to three days prior to presentation. The child
may pass bloody mucus stool, which is a late sign. Examination of the abdomen is often normal as
the sausage mass in the right upper quadrant is difficult to feel.

70. An 8 year-old West Indian boy presents with periumbilical abdominal pain. He has vomited twice
and is refusing fluids. His temperature is 38.1oC and blood tests are as follows: Haemoglobin 8
g/dl, WCC 13 x 109/l, with a neutrophilia.
You answered Appendicitis

The correct answer is Sickle cell crisis

Sickle cell anaemia is characterised by severe chronic haemolytic anaemia resulting from poorly
formed erythrocytes. Painful crises result from vaso-occlusive episodes, which may occur
spontaneously or may be precipitated by infection. Consider this diagnosis in all children of
appropriate ethnic background.

71. A 15-month-old girl presents with a three day history of periorbital oedema. She is brought to
hospital. On examination she has facial oedema and a tender distended abdomen. Her
temperature is 39oC and her blood pressure is 90/45 mmHg. There is clinical evidence of poor
peripheral perfusion.

You answered Appendicitis

The correct answer is Spontaneous bacterial peritonitis

The 15-month-old girl is a patient with nephrotic syndrome. Patients with this condition are at risk
of septicaemia and peritonitis from Streptococcus pneumoniae, due to the loss of
immunoglobulins and opsonins in the urine.

Please rate this question:

Discuss and give feedback

Next question

Acute abdominal pain-diagnoses

Conditions presenting with acute abdominal pain

Condition Features Investigations Management


Condition Features Investigations Management

Appendicitis History of migratory Differential white cell count Appendicectomy


pain. Pregnancy test
Fever. C-Reactive protein
Anorexia. Amylase
Evidence of right iliac Urine dipstick testing
fossa tenderness.
Mild pyrexia.

Mesenteric Usually recent upper Full blood count- may show Conservative management-
adenitis respiratory tract slightly raised white cell appendicectomy if
infection. count diagnostic doubt
High fever. Urine dipstick often normal
Generalised abdominal Abdominal ultrasound scan
discomfort- true - usually no free fluid
localised pain and signs
are rare.

Mittelschmerz Only seen in females Full blood count- normal Manage conservatively if
Mid cycle pain Urine dipstick- normal doubt or symptoms fail to
Usually occurs two Abdominal and pelvic settle then laparoscopy
weeks after last ultrasound- may show a
menstrual period trace of pelvic free fluid
Pain usually has a supra-
pubic location
Usually subsides over a
24-48 hour period.

Fitz-Hugh Curtis Disseminated infection Abdominal ultrasound scan- Usually medically managed-
syndrome with Chlamydia. may show free fluid doxycycline or azithromycin
Usually seen in females. High vaginal swabs - may
Consists of evidence of show evidence of sexually
pelvic inflammatory transmitted infections
disease together with
peri-hepatic
inflammation and
Condition Features Investigations Management

subsequent adhesion
formation.

Abdominal Sudden onset of Patients who are Unstable patients should


aortic aneurysm abdominal pain haemodynamically stable undergo immediate surgery
(ruptured) radiating to the back in should have a CT scan (unless it is not in their best
older adults (look for interests).
risk factors). Those with evidence of
Collapse. contained leak on CT should
May be moribund on undergo immediate surgery
arrival in casualty, more Increasing unruptured
stable if contained aneurysmal size is an
haematoma. indication for urgent
Careful clinical surgical intervention (that
assessment may reveal can wait until the next
pulsatile mass. working day)

Perforated Sudden onset of pain Erect CXR may show free Laparotomy (laparoscopic
peptic ulcer (usually epigastric). air. A CT scan may be surgery for perforated
Often preceding history indicated where there is peptic ulcers is both safe
of upper abdominal diagnostic doubt and feasible in experienced
pain. hands)
Soon develop
generalised abdominal
pain.
On examination may
have clinical evidence of
peritonitis.

Intestinal Colicky abdominal pain A plain abdominal film may In those with a virgin
obstruction and vomiting (the help with making the abdomen a lower and
nature of which diagnosis. A CT scan may be earlier threshold for
depends on the level of useful where diagnostic laparotomy should exist
the obstruction). uncertainty exists than in those who may have
Abdominal distension
Condition Features Investigations Management

and constipation (again adhesional obstruction


depending upon site of
obstruction).
Features of peritonism
may occur where local
necrosis of bowel loops
is occurring.

Mesenteric Embolic events present Arterial pH and lactate Immediate laparotomy and
infarction with sudden pain and Arterial phase CT scanning resection of affected
forceful evacuation. is the most sensitive test segments, in acute embolic
Acute on chronic events events SMA embolectomy
usually have a longer may be needed.
history and previous
weight loss.
On examination the
pain is typically greater
than the physical signs
would suggest.

Next question
Which of the following interventions is most likely to reduce the incidence of intra abdominal
adhesions?

Peritoneal lavage with cetrimide following elective right hemicolectomy

Use of a laparoscopic approach over open surgery

Use of talc to coat surgical gloves

Performing a Nobles plication of the small bowel

Using stapled rather than a hand sewn anastamosis

Laparoscopy results in fewer adhesions. When talc was used to coat surgical gloves it was a major
cause of adhesion formation and withdrawn for that reason. A Nobles plication is an old fashioned
operation which has no place in the prevention of adhesion formation. Use of an anastamotic
stapling device will not influence the development of adhesions per se although clearly an
anastamotic leak will result in more adhesion formation
Please rate this question:

Discuss and give feedback


Next question

Surgical complications

Complications occur in all branches of surgery and require vigilance in their detection. In many
cases anticipating the likely complications and appropriate avoidance will minimise their occurrence.
For the purposes of the MRCS the important principles to appreciate are:

 The anatomical principles that underpin complications


 The physiological and biochemical derangements that occur
 The most appropriate diagnostic modalities to utilise
 The principles which underpin their management

This is clearly a very broad area and impossible to cover comprehensively. There is considerable
overlap with other topic areas within the website.

Avoiding complications
Some points to hopefully avert complications:

 World Health Organisation checklist- now mandatory prior to all operations


 Prophylactic antibiotics - right dose, right drug, right time.
 Assess DVT/ PE risk and ensure adequate prophylaxis
 MARK site of surgery
 Use tourniquets with caution and with respect for underlying structures
 Remember the danger of end arteries and in situations where they occur avoid using
adrenaline containing solutions and monopolar diathermy.
 Handle tissues with care- devitalised tissue serves as a nidus for infection
 Be very wary of the potential for coupling injuries when using diathermy during laparoscopic
surgery
 The inferior epigastric artery is a favorite target for laparoscopic ports and surgical drains!

Anatomical principles
Understanding the anatomy of a surgical field will allow appreciation of local and systemic
complications that may occur. For example, nerve injuries may occur following surgery in specific
regions. The table below lists some of the more important nerves to consider and mechanisms of
injury

Nerve Mechanism

Accessory Posterior triangle lymph node biopsy

Sciatic Posterior approach to hip

Common peroneal Legs in Lloyd Davies position

Long thoracic Axillary node clearance

Pelvic autonomic nerves Pelvic cancer surgery

Recurrent laryngeal nerves During thyroid surgery

Hypoglossal nerve During carotid endarterectomy


Nerve Mechanism

Ulnar and median nerves During upper limb fracture repairs

These are just a few. The detailed functional sequelae are particularly important and will often be
tested. In addition to nerve injuries certain procedures carry risks of visceral or structural injury.
Again some particular favorites are given below:

Structure Mechanism

Thoracic duct During thoracic surgery e.g. Pneumonectomy, oesphagectomy

Parathyroid glands During difficult thyroid surgery

Ureters During colonic resections/ gynaecological surgery

Bowel perforation Use of Verres Needle to establish pneumoperitoneum

Bile duct injury Failure to delineate Calots triangle carefully and careless use of diathermy

Facial nerve Always at risk during Parotidectomy

Tail of pancreas When ligating splenic hilum

Testicular vessels During re-do open hernia surgery

Hepatic veins During liver mobilisation

Again many could be predicted from the anatomy of the procedure.

Physiological derangements
A very common complication is bleeding and this is covered under the section of haemorrhagic
shock. Another variant is infection either superficial or deep seated. The organisms are covered
under microbiology and the features of sepsis covered under shock. Do not forget that
immunocompromised and elderly patients may present will atypical physiological parameters.

Selected physiological and biochemical issues are given below:

Complication Physiological/ Biochemical Problem

Arrhythmias following Susceptibility to hypokalaemia (K+ <4.0 in cardiac patients)


cardiac surgery

Neurosurgical electrolyte SIADH following cranial surgery causing hyponatraemia


disturbance

Ileus following Fluid sequestration and loss of electrolytes


gastrointestinal surgery

Pulmonary oedema following Loss of lung volume makes these patients very sensitive to fluid
pneumonectomy overload

Anastamotic leak Generalised sepsis causing mediastinitis or peritonitis depending


on site of leak

Myocardial infarct May follow any type of surgery and in addition to direct cardiac
effects the decreased cardiac output may well compromise grafts
etc.

Try making a short list of problems and causes specific to your own clinical area.

Diagnostic modalities
Depends largely on the suspected complication. In the acutely unwell surgical patient the following
baseline investigations are often helpful:

 Full blood count, urea and electrolytes, C- reactive protein (trend rather than absolute value),
serum calcium, liver function tests, clotting (don't forget to repeat if on-going bleeding)
 Arterial blood gases
 ECG (+cardiac enzymes if MI suspected)
 Chest x-ray to identify collapse/ consolidation
 Urine analysis for UTI
These will often identify the most common complications.

Special tests

 CT scanning for identification of intra-abdominal abscesses, air and if luminal contrast is


used an anastomotic leak
 Doppler USS of leg veins- for identification of DVT
 CTPA for PE
 Sending peritoneal fluid for U+E (if ureteric injury suspected) or amylase (if pancreatic injury
suspected)
 Echocardiogram if pericardial effusion suspected post cardiac surgery and no pleural window
made.

Management of complications
The guiding principal should be safe and timely intervention. Patients should be stabilised and if an
operation needs to occur in tandem with resuscitation then generally this should be of a damage
limitation type procedure rather than definitive surgery (which can be more safely undertaken in a
stable patient the following day).

Remember that recent surgery is a contra indication to thrombolysis and that in some patients IV
heparin may be preferable to a low molecular weight heparin (easier to reverse).

As a general rule laparotomies for bleeding should follow the core principle of quadrant packing and
then subsequent pack removal rather than plunging large clamps into pools of blood. The latter
approach invariable worsens the situation is often accompanied by significant visceral injury
particularly when done by the inexperienced. If packing controls a situation it is entirely acceptable
practice to leave packs in situ and return the patient to ITU for pack removal the subsequent day.
Next question
Theme: Abdominal pain

A. Appendicitis

B. Threatened miscarriage

C. Ectopic pregnancy

D. Irritable bowel syndrome

E. Mittelschmerz

F. Pelvic inflammatory disease

G. Adnexial torsion

H. Endometriosis

I. Degenerating fibroid

Please select the most likely cause of abdominal pain for the clinical scenario given. Each option
may be used once, more than once or not at all.

73. An 18 year-old girl presents to the Emergency Department with sudden onset sharp, tearing
pelvic pain associated with a small amount of vaginal bleeding. She also complains of shoulder tip
pain. On examination she is hypotensive, tachycardic and has marked cervical excitation.

You answered Appendicitis

The correct answer is Ectopic pregnancy

The history of tearing pain and haemodynamic compromise in a women of child bearing years
should prompt a diagnosis of ectopic pregnancy.

74. A 25 year-old lady presents to her GP complaining of a two day history of right upper quadrant
pain, fever and a white vaginal discharge. She has seen the GP twice in 12 weeks complaining of
pelvic pain and dyspareunia.

You answered Appendicitis


The correct answer is Pelvic inflammatory disease

The most likely diagnosis is pelvic inflammatory disease. Right upper quadrant pain occurs as part
of the Fitz Hugh Curtis syndrome in which peri hepatic inflammation occurs.

75. A 16 year old female presents to the emergency department with a 12 hour history of pelvic
discomfort. She is otherwise well and her last normal menstrual period was 2 weeks ago. On
examination she has a soft abdomen with some mild supra pubic discomfort.

You answered Appendicitis

The correct answer is Mittelschmerz

Mid cycle pain is very common and is due to the small amount of fluid released during ovulation.
Inflammatory markers are usually normal and the pain typically subsides over the next 24-48
hours.

Please rate this question:

Discuss and give feedback

Next question

Gynaecological causes of abdominal pain

A number of women will present with abdominal pain and subsequently be diagnosed with a
gynaecological disorder. In addition to routine diagnostic work up of abdominal pain, all female
patients should also undergo a bimanual vaginal examination, urine pregnancy test and
consideration given to abdominal and pelvic ultrasound scanning.
When diagnostic doubt persists a laparoscopy provides a reliable method of assessing suspected
tubulo-ovarian pathology.

Differential diagnoses of abdominal pain in females

Diagnosis Features Investigation Treatment


Mittelschmerz Usually mid cycle pain. Full blood count- Conservative
Often sharp onset. usually normal
Little systemic disturbance. Ultrasound- may
May have recurrent show small
episodes. quantity of free
Usually settles over 24-48 fluid
hours.

Endometriosis 25% asymptomatic, in a Ultrasound- may Usually managed medically,


further 25% associated with show free fluid complex disease will often
other pelvic organ Laparoscopy will require surgery and some
pathology. usually show patients will even require formal
Remaining 50% may have lesions colonic and rectal resections if
menstrual irregularity, these areas are involved
infertility, pain and deep
dyspareurina.
Complex disease may result
in pelvic adhesional
formation with episodes of
intermittent small bowel
obstruction.
Intra-abdominal bleeding
may produce localised
peritoneal inflammation.
Recurrent episodes are
common.

Ovarian torsion Usually sudden onset of Ultrasound may Laparoscopy


deep seated colicky show free fluid
abdominal pain. Laparoscopy is
Associated with vomiting usually both
and distress. diagnostic and
Vaginal examination may therapeutic
reveal adnexial tenderness.

Ectopic Symptoms of pregnancy Ultrasound Laparoscopy or laparotomy is


gestation without evidence of intra showing no intra haemodynamically unstable. A
uterine gestation. uterine pregnancy salphingectomy is usually
Present as an emergency and beta HCG that performed.
with evidence of rupture or is elevated
impending rupture. May show intra
Open tubular ruptures may abdominal free
have sudden onset of fluid
abdominal pain and
circulatory collapse, in other
the symptoms may be more
prolonged and less marked.
Small amount of vaginal
discharge is common.
There is usually adnexial
tenderness.

Pelvic Bilateral lower abdominal Full blood count- Usually medical management
inflammatory pain associated with vaginal Leucocytosis
disease discharge. Pregnancy test
Dysuria may also be present. negative (Although
Peri-hepatic inflammation infection and
secondary to Chlamydia (Fitz pregnancy may co-
Hugh Curtis Syndrome) may exist)
produce right upper Amylase - usually
quadrant discomfort. normal or slightly
Fever >38o raised
High vaginal and
urethral swabs
Which of the following statements relating to a burst abdomen is false?

Is seen in 1-2% of modern laparotomies

Is more common in faecal peritonitis

Is less common when a 'mass closure' technique is used

When it does occur is most common at 15 days

Is similar in incidence regardless of whether 1/0 polydiaxone or 1/0 polypropylene are


used

When it does occur a burst abdomen is most common at 6 days and is usually the result of technical
error when Jenkins rule is not followed and sutures are placed in the zone of collagenolysis. The
choice of materials given above does not influence dehisence rates.
Please rate this question:

Discuss and give feedback


Next question

Abdominal wound dehiscence

 This is a significant problem facing all surgeons who undertake abdominal surgery on a
regular basis. Traditionally, it is said to occur when all layers of an abdominal mass closure
fail and the viscera protrude externally (associated with 30% mortality).
 It can be subdivided into superficial, in which the skin wound alone fails and complete,
implying failure of all layers.

Factors which increase the risk are:


* Malnutrition
* Vitamin deficiencies
* Jaundice
* Steroid use
* Major wound contamination (e.g. faecal peritonitis)
* Poor surgical technique (Mass closure technique is the preferred method-Jenkins Rule)

When sudden full dehiscence occurs the management is as follows:


* Analgesia
* Intravenous fluids
* Intravenous broad spectrum antibiotics
* Coverage of the wound with saline impregnated gauze (on the ward)
* Arrangements made for a return to theatre

Surgical strategy

 Correct the underlying cause (e.g. TPN or NG feed if malnourished)


 Determine the most appropriate strategy for managing the wound

Options
Resuturing of the This may be an option if the wound edges are healthy and there is enough
wound tissue for sufficient coverage. Deep tension sutures are traditionally used for
this purpose.

Application of a This is a clear dressing with removable front. Particularly suitable when some
wound manager granulation tissue is present over the viscera or where there is a high output
bowel fistula present in the dehisced wound.

Application of a This is a clear plastic bag that is cut and sutured to the wound edges and is
'Bogota bag' only a temporary measure to be adopted when the wound cannot be closed
and will necessitate a return to theatre for definitive management.

Application of a These can be safely used BUT ONLY if the correct layer is interposed
VAC dressing between the suction device and the bowel. Failure to adhere to this absolute
system rule will almost invariably result in the development of multiple bowel
fistulae and create an extremely difficult management problem.
A 35 year old women who is a heavy smoker has long standing stool frequency and crampy
abdominal pain. A colonoscopy is performed (which is macroscopically normal) and pan colonic
biopsies are taken. Histological analysis of the biopsies demonstrates a thickened sub apical
collagen layer together with an increase in lymphocytes in the lamina propria. What is the most likely
diagnosis?

Microscopic colitis

Crohns disease

Ulcerative colitis

Pseudomembranous colitis

Irritable bowel syndrome

Microscopic colitis is a common condition characterised by normal endoscopic appearances,


microscopic features of colonic inflammation and thickening of the sub epithelial collagen layer.
Features such as granulomas are absent. It is the normal endoscopic appearance that makes the
other options less likely. IBS is a diagnosis of exclusion and the features of inflammation activity
would favor a different diagnosis. Severe cases can be treated with steroids. Other agents such as
cholestyramine may be beneficial. It is important to exclude infection with a stool culture if this has
not been done, not least because the precipitant of many cases of microscopic colitis is an episode
of infective diarrhoea.
Please rate this question:

Discuss and give feedback


Next question

Diarrhoea

World Health Organisation definitions


Diarrhoea: > 3 loose or watery stool per day
Acute diarrhoea < 14 days
Chronic diarrhoea > 14 days

Acute Diarrhoea
Acute Diarrhoea

Gastroenteritis May be accompanied by abdominal pain or nausea/vomiting

Diverticulitis Classically causes left lower quadrant pain, diarrhoea and fever

Antibiotic therapy More common with broad spectrum antibiotics


Clostridium difficile is also seen with antibiotic use

Constipation causing A history of alternating diarrhoea and constipation may be


overflow given
May lead to faecal incontinence in the elderly

Chronic
Diarrhoea

Irritable bowel Extremely common. The most consistent features are abdominal pain, bloating
syndrome and change in bowel habit. Patients may be divided into those with diarrhoea
predominant IBS and those with constipation predominant IBS.
Features such as lethargy, nausea, backache and bladder symptoms may also be
present

Ulcerative Bloody diarrhoea may be seen. Crampy abdominal pain and weight loss are also
colitis common. Faecal urgency and tenesmus may occur

Crohn's Crampy abdominal pains and diarrhoea. Bloody diarrhoea less common than in
disease ulcerative colitis. Other features include malabsorption, mouth ulcers perianal
disease and intestinal obstruction

Colorectal Symptoms depend on the site of the lesion but include diarrhoea, rectal
cancer bleeding, anaemia and constitutional symptoms e.g. Weight loss and anorexia

Coeliac disease  In children may present with failure to thrive, diarrhoea and abdominal
distension
 In adults lethargy, anaemia, diarrhoea and weight loss are seen. Other
Chronic
Diarrhoea

autoimmune conditions may coexist

Other conditions associated with diarrhoea include:

 Thyrotoxicosis
 Laxative abuse
 Appendicitis with pelvic abscess or pelvic appendix
 Radiation enteritis

Diagnosis
Stool culture
Abdominal and digital rectal examination
Consider colonoscopy (radiological studies unhelpful)
Thyroid function tests, serum calcium, anti endomysial antibodies, glucose
Next question
A 40 year old man presents with a long standing inguinal hernia. On examination he has a small,
direct inguinal hernia. He inquires as to the risk of strangulation over the next twelve months should
he decide not to undergo surgery. Which of the following most closely matches the likely risk of
strangulation over the next 12 months?

50%

40%

25%

15%

<5%

The annual probability of strangulation is up to 3% and is more common in indirect hernias. Elective
repair poses few risks. However, emergency repair is associated with increased mortality,
particularly in the elderly.
Please rate this question:

Discuss and give feedback


Next question

Inguinal hernia surgery

Inguinal hernias occur when the abdominal viscera protrude through the anterior abdominal wall into
the inguinal canal. They may be classified as being either direct or indirect. The distinction between
these two rests on their relation to Hesselbach's triangle.

Boundaries of Hesselbach's Triangle

 Medial: Rectus abdominis


 Lateral: Inferior epigastric vessels
 Inferior: Inguinal ligament
Image sourced from Wikipedia

Hernias occurring within the triangle tend to be direct and those outside - indirect.

Diagnosis
Most cases are diagnosed clinically, a reducible swelling may be located at the level of the inguinal
canal. Large hernia's may extend down into the male scrotum, these will not trans-illuminate and it is
not possible to "get above" the swelling.
Cases that are unclear on examination, but suspected from the history, may be further investigated
using ultrasound or by performing a herniogram.

Treatment
Hernias associated with few symptoms may be managed conservatively. Symptomatic hernias or
those which are at risk of developing complications are usually treated surgically.
First time hernias may be treated by performing an open inguinal hernia repair; the inguinal canal is
opened, the hernia reduced and the defect repaired. A prosthetic mesh may be placed posterior to
the cord structures to re-enforce the repair and reduce the risk of recurrence.
Recurrent hernias and those which are bilateral are generally managed with a laparoscopic
approach. This may be via an intra or extra peritoneal route. As in open surgery a mesh is deployed.
However, it will typically lie posterior to the deep ring.

Inguinal hernia in children


Inguinal hernias in children are almost always of an indirect type and therefore are usually dealt with
by herniotomy, rather than herniorraphy. Neonatal hernias especially in those children born
prematurely are at highest risk of strangulation and should be repaired urgently. Other hernias may
be repaired on an elective basis.
A 60 year old women has fully recovered from an attack of pancreatitis. Over the following 12
months she develops episodic epigastric discomfort. Un upper GI endoscopy shows gastric varices
only. An abdominal CT scan demonstrates a splenic vein thrombosis. What is the treatment of
choice?

Splenectomy

Insertion of transjugular porto-systemic shunt

Surgical bypass of the splenic vein

Gastrectomy

Stapling of the gastro-oesophgeal junction

Please rate this question:

Discuss and give feedback

Next question

Splenic vein thrombosis

Thrombosis of the splenic vein may complicate pancreatitis, pancreatic carcinoma, iatrogenic trauma
and hypercoagulable diseases. The condition may predispose to the development of gastric varices,
oesophageal varices are uncommon in splenic vein thrombosis alone.
Diagnosis is made by CT angiography.
Treatment is with splenectomy.
Theme: Surgical signs

A. Rovsing's sign
B. Boas' sign
C. Psoas stretch sign
D. Cullen's sign
E. Grey-Turner's sign
F. Murphy's sign
G. None of the above

Please match the clinical sign to the clinical scenario described. Each option may be used once,
more than once or not at all.

80. Acute retrocaecal appendicitis is indicated when the right thigh is passively extended with
the patient lying on their side with their knees extended.

You answered Rovsing's sign

The correct answer is Psoas stretch sign

81. In acute pancreatitis there is bruising in the flanks.

You answered Rovsing's sign

The correct answer is Grey-Turner's sign

Grey-Turners sign occurs in patients with severe haemorrhagic pancreatitis. In this


situation the major vessels surrounding the pancreas bleed. The pancreatitis process also
results in local fat destruction, this results in blood tracking in the tissue planes of the
retroperitoneum and appearing as flank bruising.

82. In cholecystitis there is pain/catch of breath elicited on palpation of the right


hypochondrium during inspiration.

You answered Rovsing's sign

The correct answer is Murphy's sign

Invariably present when patients are assessed in the emergency department! This sign
occurs because the inflamed gallbladder irritates the parietal peritoneum in this
manoeuvre.
Please rate this question:

Discuss and give feedback


Next question

Abdominal signs

A number of eponymous abdominal signs are noted. These include:

 Rovsings sign- appendicitis


 Boas sign -cholecystitis
 Murphys sign- cholecystitis
 Cullens sign- pancreatitis (other intraabdominal haemorrhage)
 Grey-Turners sign- pancreatitis (or other retroperitoneal haemorrhage)

In clinical practice haemorrhagic pancreatitis is thankfully rare. The signs are important and thus
shown below:

Cullen's sign

Image sourced from Wikipedia

Grey Turner's sign


Image sourced from Wikipedia
Which of the following is commonest cause of acute abdominal pain in acute unselected surgical
'take'?

Non specific abdominal pain

Biliary colic

Acute appendicitis

Ureteric colic

Pancreatitis

Non specific abdominal pain is a commonly recorded diagnosis for patients presenting with acute
abdominal pain. Following careful diagnostic work up, a proportion of patients may be identified with
disorders such as coeliac disease and the diagnosis of non specific abdominal pain should not be
used lightly.
Please rate this question:

Discuss and give feedback


Next question

Acute abdominal pain

Acute abdominal pain is a common cause of admission to hospital. The relative proportions of
conditions presenting with abdominal pain is given below:

 Non specific abdominal pain (35%)


 Appendicitis (17%)
 Intestinal obstruction (15%)
 Urological disease (6%)
 Gallstone disease (5%)
 Colonic diverticular disease (4%)
 Abdominal trauma (3%)
 Perforated peptic ulcer (3%)
 Pancreatitis (2%)

(Data derived from Irvin T. Br. J. Surg 1989 76:1121-1125)

Non specific abdominal pain should really be a diagnosis of exclusion and if care is taken in
excluding organic disease the proportion of cases labeled such should decline. It should also be
appreciated that a proportion of patients may have an underlying medical cause for their symptoms
such as pneumonia or diabetic ketoacidosis.

Key points in management

 Early administration of adequate analgesia (including opiates).


 Abdominal ultrasound is safe, non invasive and cheap and yields significantly more
information than plain radiology. However, plain radiology is still the main test for suspected
perforated viscus, especially out of hours.
 In up to 50% cases with perforated peptic ulcer, the plain x-rays may show no evidence of
free air. If clinical signs suggest otherwise, then a CT scan may be a more accurate
investigation, if plain films are normal.
 Plain film radiology usually cannot detect <1mm free air, and is 33% sensitive for detection of
1-13mm pockets of free air (Stoker et al. Radiology 2009 253: 31-46).
 Think of strangulated intestine when there is fever, raised white cell count, tachycardia and
peritonism.
 In suspected large bowel obstruction a key investigation is either a water soluble contrast
enema or CT scan.
 Where need for surgery is difficult to define and imaging is inconclusive the use of
laparoscopy as a definitive diagnostic test is both safe and sensible.

Next question
A 72 year old obese man undergoes an emergency repair of a ruptured abdominal aortic aneurysm.
The wound is closed with an onlay prolene mesh to augment the closure. Post operatively he is
taken to the intensive care unit. Over the following twenty four hours his nasogastric aspirates
increase, his urine output falls and he has a metabolic acidosis. What is the most likely underlying
cause?

Colonic ischaemia

Abdominal compartment syndrome

Peritonitis

Reactionary haemorrhage

Aorto-duodenal fistula

Obese patients with ileus following major abdominal surgery are at increased risk of intra abdominal
compartment syndrome.The risk is increased by the use of prosthetic meshes, which some
surgeons favor following a major vascular case as they may reduce the incidence of incisional
hernia. They prevent abdominal distension and may increase the risk of intra abdominal
hypertension in the short term. Although colonic ischaemia may occur following major aortic surgery
it would not typically present in this way.
Please rate this question:

Discuss and give feedback


Next question

Abdominal compartment syndrome

Background
Intra-abdominal pressure is the steady state pressure concealed within the abdominal cavity.

 In critically ill adults the normal intra abdominal pressure = 5-7mmHg


 Intra abdominal hypertension has pressures of 12-25mmHg
 Changes >15mmHg are associated with microvascular hypoperfusion
 Abdominal compartment syndrome is defined as sustained intra abdominal pressure
>20mmHg coupled with new organ dysfunction / failure
 It may occur either primarily without previous surgical intervention e.g. Following intestinal
ischaemia or secondarily following a surgical procedure
 Diagnosis is typically made by transvesical pressure measurements coupled with an index of
clinical suspicion.
Management
Once the diagnosis is made non operative measures should be instituted including:

 Gastric decompression
 Improve abdominal wall compliance e.g. muscle relaxants/ sedation
 Drain abdominal fluid collections.
 Consider fluid restriction/ diuretics if clinically indicated.

In those whom non operative treatment is failing; the correct treatment is laparotomy and
laparostomy. Options for laparostomy are many although the Bogota bag or VAC techniques are the
most widely practised. Re-look laparotomy and attempts at delayed closure will follow in due course.
Next question
Theme: Surgical drains

A. Redivac suction drain


B. Corrugated drain
C. Wallace Robinson drain
D. Penrose tubing
E. Latex T Tube drain
F. Silastic T Tube drain

Please select the most appropriate surgical drainage system for the indication given. Each option
may be used once, more than once or not at all.

85. A 56 year old lady undergoes and open cholecystectomy and exploration of common bile
duct. The bile duct is closed over a drain.

You answered Redivac suction drain

The correct answer is Latex T Tube drain

Latex is used for this indication as it will encourage track formation.

86. A 48 year old lady undergoes a mastectomy and axillary node clearance for an invasive
ductal cancer of the breast with lymph node metastasis.

Redivac suction drain

Suction drains are commonly used following mastectomy and axillary surgery to prevent
haematoma formation. Not all surgeons routinely drain the axilla.

87. A 75 year old man undergoes a hartmans procedure for sigmoid diverticular disease with
pericolic abscess and colovesical fistula.

You answered Redivac suction drain

The correct answer is Wallace Robinson drain

These tube drains are often used in abdominal surgery to drain abscess cavities. Debate
might occur around the use of low pressure vs no suction in this setting so this option is
deliberately omitted.

Please rate this question:


Discuss and give feedback
Next question

Surgical drains

 Drains are inserted in many surgical procedures and are of many types.
 As a broad rule they can be divided into those using suction and those which do not.
 The diameter of the drain will depend upon the substance being drained, for example smaller
lumen drain for pneumothoraces vs haemothorax.
 Drains can be associated with complications and these begin with insertion when there may
be iatrogenic damage. When in situ they serve as a route for infections. In some specific
situations they may cause other complications, for example suction drains left in contact with
bowel for long periods may carry a risk of inducing fistulation.
 Drains should be inserted for a defined purpose and removed once the need has passed.

A brief overview of types of drain and sites is given below

CNS

 Low suction drain or free drainage systems may be used for situations such as drainage of
sub dural haematomas.

CVS

 Following cardiothoracic procedures of thoracic trauma underwater seal drains are often
placed. These should be carefully secured. When an air leak is present they may be placed
on suction whilst the air leak settles

Orthopaedics and trauma

 In this setting drains are usually used to prevent haematoma formation (with associated risk
of infection). Some orthopaedic drains may also be specially adapted to allow the drained
blood to be auto transfused.

Gastro-intestinal surgery

 Surgeons often place abdominal drains either to prevent or drain abscesses, or to turn an
anticipated complication into one that can be easily controlled such as a bile leak following
cholecystectomy. The type of drain used will depend upon the indication.

Drain types
Type of drain Features

Redivac  Suction type of drain


 Closed drainage system
 High pressure vacuum system

Low pressure  Consist of small systems such as the lantern style drain that may be
drainage systems used for short term drainage of small wounds and cavities
 Larger systems are sometimes used following abdominal surgery,
they have a lower pressure than the redivac system, which decreases
the risks of fistulation
 May be emptied and re-pressurised

Latex tube drains  May be shaped (e.g. T Tube) or straight


 Usually used in non pressurised systems and act as sump drains
 Most often used when it is desirable to generate fibrosis along the
drain track (e.g. following exploration of the CBD)

Chest drains  May be large or small diameter (depending on the indication)


 Connected to underwater seal system to ensure one way flow of air

Corrugated drain  Thin, wide sheet of plastic, usually soft


 Contains corrugations, along which fluids can track

Next question
What is the commonest site in the abdomen for fluid to collect following a perforated appendix?

Pelvis

Hepatorenal pouch

Between small bowel loops

Right iliac fossa

Lesser sac

Following perforated appendicitis fluid is most likely to accumulate in the pelvis. Fluid may
accumulate in the hepatorenal pouch although this is less common. Gravity favors the pelvis as the
site of most collections. The incidence of these is higher with laparoscopic rather than open surgery.
Please rate this question:

Discuss and give feedback


Next question

Appendicitis

History

 Peri umbilical abdominal pain (visceral stretching of appendix lumen and appendix is mid gut
structure) radiating to the right iliac fossa due to localised parietal peritoneal inflammation.
 Vomit once or twice but marked and persistent vomiting is unusual.
 Diarrhoea is rare. However, pelvic appendicitis may cause localised rectal irritation and some
loose stools. A pelvic abscess may also cause diarrhoea.
 Mild pyrexia is common - temperature is usually 37.5 -38oC. Higher temperatures are more
typical of conditions like mesenteric adenitis.
 Anorexia is very common. It is very unusual for patients with appendicitis to be hungry.

Examination

 Generalised peritonitis if perforation has occurred or localised peritonism.


 Retrocaecal appendicitis may have relatively few signs.
 Digital rectal examination may reveal boggy sensation if pelvic abscess is present, or even
tenderness with a pelvic appendix.
Diagnosis

 Typically raised inflammatory markers coupled with compatible history and examination
findings should be enough to justify appendicectomy.
 Urine analysis may show mild leucocytosis but no nitrites.
 Ultrasound is useful in females where pelvic organ pathology is suspected. Although it is not
always possible to visualise the appendix on ultrasound, the presence of free fluid (always
pathological in males) should raise suspicion.

Ultrasound examination may show evidence of luminal obstruction and thickening of the appendiceal
wall as shown below

Image sourced from Wikipedia

Treatment

 Appendicectomy which can be performed via either an open or laparoscopic approach.


 Administration of metronidazole reduces wound infection rates.
 Patients with perforated appendicitis require copious abdominal lavage.
 Patients without peritonitis who have an appendix mass should receive broad spectrum
antibiotics and consideration given to performing an interval appendicectomy.
 Be wary in the older patients who may have either an underlying caecal malignancy or
perforated sigmoid diverticular disease.

Laparoscopic appendicectomy is becoming increasing popular as demonstrated below


Image sourced from Wikipedia
Theme: Right iliac fossa pathology

A. Appendicitis
B. Mesenteric adenitis
C. Inflammatory bowel disease
D. Irritable bowel syndrome
E. Mesenteric cyst
F. Campylobacter infection
G. Appendix abscess

Please select the most likely diagnosis for the scenario given. Each option may be used once, more
than once or not at all.

89. An 8 year old boy is examined by his doctor as part of a routine clinical examination. The
doctor notices a smooth swelling in the right iliac fossa. It is mobile and the patient is
otherwise well.

You answered Appendicitis

The correct answer is Mesenteric cyst

Theme from April 2012 Exam


Mesenteric cysts are often smooth. Imaging with ultrasound and CT is usually sufficient.
Although rare, they most often occur in young children (up to 30% present before the age
of 15). Many are asymptomatic and discovered incidentally. Acute presentations are
recognised and may occur following cyst torsion, infarction or rupture. Most cysts will be
surgically resected.

90. An 8 year old boy presents with abdominal pain,a twelve hour history of vomiting, a fever
of 38.3 oC and four day history of diarrhoea. His abdominal pain has been present for the
past week.

You answered Appendicitis

The correct answer is Appendix abscess

The high fever and diarrhoea together with vomiting all point to a pelvic abscess. The
presence of pelvic pus is highly irritant to the rectum, and many patients in this situation
will complain of diarrhoea.

91. A 7 year old boy presents with a three day history of right iliac fossa pain and fever. On
examination he has a temperature of 39.9o C. His abdomen is soft and mildly tender in the
right iliac fossa.
You answered Appendicitis

The correct answer is Mesenteric adenitis

High fever and mild abdominal signs in a younger child should raise suspicion for
mesenteric adenitis. The condition may mimic appendicitis and many may require surgery.

Please rate this question:

Discuss and give feedback


Next question

Right iliac fossa pain

Differential diagnosis

Appendicitis  Pain radiating to right iliac fossa


 Anorexia (very common)
 Short history
 Diarrhoea and profuse vomiting rare

Crohn's disease  Often long history


 Signs of malnutrition
 Change in bowel habit, especially diarrhoea

Mesenteric adenitis  Mainly affects children


 Causes include Adenoviruses, Epstein Barr Virus, beta-
haemolytic Streptococcus, Staphylococcus spp., Escherichia
coli,Streptococcus viridans and Yersinia spp.
 Patients have a higher temperature than those with appendicitis
 If laparotomy is performed, enlarged mesenteric lymph nodes will
be present

Diverticulitis  Both left and right sided disease may present with right iliac fossa
pain
 Clinical history may be similar, although some change in bowel
habit is usual
 When suspected, a CT scan may help in refining the diagnosis

Meckel's  A Meckel's diverticulum is a congenital abnormality that is present


diverticulitis in about 2% of the population
 Typically 2 feet proximal to the ileocaecal valve
 May be lined by ectopic gastric mucosal tissue and produce
bleeding

Perforated peptic  This usually produces upper quadrant pain but pain may be lower
ulcer  Perforations typically have a sharp sudden onset of pain in the
history

Incarcerated right  Usually only right iliac fossa pain if right sided or bowel
inguinal or femoral obstruction.
hernia

Bowel perforation  Seldom localised to right iliac fossa, although complete large bowel
secondary to caecal obstruction with caecal distension may cause pain prior to
or colon carcinoma perforation.

Gynaecological  Pelvic inflammatory disease/salpingitis/pelvic abscess/Ectopic


causes pregnancy/Ovarian torsion/Threatened or complete
abortion/Mittelschmerz

Urological causes  Ureteric colic/UTI/Testicular torsion

Other causes  TB/Typhoid/Herpes Zoster/AAA/Situs inversus


A 56 year old man undergoes a difficult splenectomy and is left with a pancreatic fistula. There are
ongoing problems with very high fistula output. Which of the following agents may be administered to
reduce the fistula output?

Metoclopramide

Erthyromycin

Octreotide

Loperamide

Omeprazole

Octreotide is a useful agent in reducing the output from pancreatic fistulae. Prokinetic agents will
increase fistula output and should be avoided.
Please rate this question:

Discuss and give feedback


Next question

Fistulas

 A fistula is defined as an abnormal connection between two epithelial surfaces.


 There are many types ranging from Branchial fistulae in the neck to entero-cutaneous
fistulae abdominally.
 In general surgical practice the abdominal cavity generates the majority and most of these
arise from diverticular disease and Crohn's.
 As a general rule all fistulae will resolve spontaneously as long as there is no distal
obstruction. This is particularly true of intestinal fistulae.

The four types of fistulae are:

Enterocutaneous
These link the intestine to the skin. They may be high (>500ml) or low output (<250ml) depending
upon source. Duodenal /jejunal fistulae will tend to produce high volume, electrolyte rich secretions
which can lead to severe excoriation of the skin. Colo-cutaneous fistulae will tend to leak faeculent
material. Both fistulae may result from the spontaneous rupture of an abscess cavity onto the skin
(such as following perianal abscess drainage) or may occur as a result of iatrogenic input. In some
cases it may even be surgically desirable e.g. mucous fistula following sub total colectomy for colitis.
Suspect if there is excess fluid in the drain.

Enteroenteric or Enterocolic
This is a fistula that involves the large or small intestine. They may originate in a similar manner to
enterocutaneous fistulae. A particular problem with this fistula type is that bacterial overgrowth may
precipitate malabsorption syndromes. This may be particularly serious in inflammatory bowel
disease.

Enterovaginal
Aetiology as above.

Enterovesicular
This type of fistula goes to the bladder. These fistulas may result in frequent urinary tract infections,
or the passage of gas from the urethra during urination.

Management
Some rules relating to fistula management:

 They will heal provided there is no underlying inflammatory bowel disease and no distal
obstruction, so conservative measures may be the best option
 Where there is skin involvement, protect the overlying skin, often using a well fitted stoma
bag- skin damage is difficult to treat
 A high output fistula may be rendered more easily managed by the use of octreotide, this will
tend to reduce the volume of pancreatic secretions.
 Nutritional complications are common especially with high fistula (e.g. high jejunal or
duodenal) these may necessitate the use of TPN to provide nutritional support together with
the concomitant use of octreotide to reduce volume and protect skin.
 When managing perianal fistulae surgeons should avoid probing the fistula where acute
inflammation is present, this almost always worsens outcomes.
 When perianal fistulae occur secondary to Crohn's disease the best management option is
often to drain acute sepsis and maintain that drainage through the judicious use of setons
whilst medical management is implemented.
 Always attempt to delineate the fistula anatomy, for abscesses and fistulae that have an intra
abdominal source the use of barium and CT studies should show a track. For perianal
fistulae surgeons should recall Goodsall's rule in relation to internal and external openings.
Theme: Inguinal hernia management

A. Bassini repair
B. Inguinal herniotomy
C. Lichtenstein repair
D. Laparoscopic hernia repair
E. Shouldice repair
F. McVey repair

For the herniae described please select the most appropriate procedure from the list. Each option
may be used once, more than once or not at all.

93. A 11 month old child presents with intermittent groin swelling, it has a cough impulse and
is easily reducible.

You answered Bassini repair

The correct answer is Inguinal herniotomy

Infants usually suffer from a patent processus vaginalis (a congential problem). As a result
a simple herniotomy is all that is required. A mesh is not required as there is not specific
muscle weakness.

94. A 25 year old builder presents with a reducible swelling in the right groin, it is becoming
larger and has not been operated on previously.

You answered Bassini repair

The correct answer is Lichtenstein repair

An open Lichtenstein repair using mesh is appropriate. There is a 0.77% recurrence rate
with this technique. A Shouldice repair is an acceptable alternative if the surgeon is
experienced

95. A 28 year old man presents with a recurrent inguinal hernia on the left side of his abdomen
and a newly diagnosed inguinal hernia on the right side.

You answered Bassini repair

The correct answer is Laparoscopic hernia repair

Laparoscopic hernia repairs are specifically indicated where there are bilateral hernias or
recurrence of a previous open repair.
Please rate this question:

Discuss and give feedback


Next question

Inguinal hernia surgery

Inguinal hernias occur when the abdominal viscera protrude through the anterior abdominal wall into
the inguinal canal. They may be classified as being either direct or indirect. The distinction between
these two rests on their relation to Hesselbach's triangle.

Boundaries of Hesselbach's Triangle

 Medial: Rectus abdominis


 Lateral: Inferior epigastric vessels
 Inferior: Inguinal ligament

Image sourced from Wikipedia

Hernias occurring within the triangle tend to be direct and those outside - indirect.

Diagnosis
Most cases are diagnosed clinically, a reducible swelling may be located at the level of the inguinal
canal. Large hernia's may extend down into the male scrotum, these will not trans-illuminate and it is
not possible to "get above" the swelling.
Cases that are unclear on examination, but suspected from the history, may be further investigated
using ultrasound or by performing a herniogram.

Treatment
Hernias associated with few symptoms may be managed conservatively. Symptomatic hernias or
those which are at risk of developing complications are usually treated surgically.
First time hernias may be treated by performing an open inguinal hernia repair; the inguinal canal is
opened, the hernia reduced and the defect repaired. A prosthetic mesh may be placed posterior to
the cord structures to re-enforce the repair and reduce the risk of recurrence.
Recurrent hernias and those which are bilateral are generally managed with a laparoscopic
approach. This may be via an intra or extra peritoneal route. As in open surgery a mesh is deployed.
However, it will typically lie posterior to the deep ring.

Inguinal hernia in children


Inguinal hernias in children are almost always of an indirect type and therefore are usually dealt with
by herniotomy, rather than herniorraphy. Neonatal hernias especially in those children born
prematurely are at highest risk of strangulation and should be repaired urgently. Other hernias may
be repaired on an elective basis.

References
The UK Based National Institute of Clinical Excellence has published guidelines relating to the
choice between open and laparoscopic inguinal hernia repair. Which users may find interesting:

http://guidance.nice.org.uk/TA83/Guidance/pdf/English
Theme: Surgical access

A. Kocher's
B. Lanz
C. Rooftop
D. Pfannenstiel's
E. Midline
F. Paramedian incision
G. Mcevedy

Please select the most appropriate incision for the procedure described. Each option may be used
once, more than once or not at all.

96. A 19 year old girl who is 39 weeks pregnant goes into labour. The labour is prolonged and
she is found to have an undiagnosed breech baby.

You answered Kocher's

The correct answer is Pfannenstiel's

This patient needs an emergency cesarean section.

97. A 49 year old woman presents with jaundice and abdominal pain. She is
haemodynamically unstable. An USS shows a dilated common bile duct and gallstones in
the gallbladder.

Kocher's

This lady needs a cholecystectomy and bile duct exploration.

98. A 42 year old man with history of alcohol abuse is diagnosed with pancreatic cancer and
requires a Whipples resection.

You answered Kocher's

The correct answer is Rooftop

A pancreatectomy is usually performed through a roof top incision. This provides


excellent access to the upper abdomen.

Please rate this question:


Discuss and give feedback

Abdominal incisions

Midline incision  Commonest approach to the abdomen


 Structures divided: linea alba, transversalis fascia, extraperitoneal fat,
peritoneum (avoid falciform ligament above the umbilicus)
 Bladder can be accessed via an extraperitoneal approach through the
space of Retzius

Paramedian  Parallel to the midline (about 3-4cm)


incision  Structures divided/retracted: anterior rectus sheath, rectus (retracted),
posterior rectus sheath, transversalis fascia, extraperitoneal fat,
peritoneum
 Incision is closed in layers

Battle  Similar location to paramedian but rectus displaced medially (and thus
denervated)
 Now seldom used

Kocher's Incision under right subcostal margin e.g. Cholecystectomy (open)

Lanz Incision in right iliac fossa e.g. Appendicectomy

Gridiron Oblique incision centered over McBurneys point- usually appendicectomy


(less cosmetically acceptable than Lanz

Gable Rooftop incision

Pfannenstiel's Transverse supra pubic, primarily used to access pelvic organs

McEvedy's Groin incision e.g. Emergency repair strangulated femoral hernia

Rutherford Extraperitoneal approach to left or right lower quadrants. Gives excellent


Morrison access to iliac vessels and is the approach of choice for first time renal
transplantation.
Image sourced from Wikipedia
Theme: Causes of dysphagia

A. Benign oesophageal stricture


B. Globus
C. Carcinoma of the oesophagus
D. Achalasia
E. Hiatus hernia
F. Pharyngeal pouch
G. Oesophageal web
H. Oesophageal atresia

Please select the the most likely cause of swallowing difficulties for the scenarios described. Each
option may be used once, more than once or not at all.

1. A 55 year old man presents with a history of progressive dysphagia over the past 4 weeks.
For the preceding 5 years he had regularly attended his general practitioner with symptoms
of dyspepsia and reflux.

You answered Benign oesophageal stricture

The correct answer is Carcinoma of the oesophagus

Theme from January 2013 Exam


A short history of progressive dysphagia in a middle aged man who has a background
history of reflux is strongly suggestive of malignancy. Long standing reflux symptoms may
be suggestive of a increased risk of developing Barretts oesophagus. Note that not all
patients with Barretts transformation alone are symptomatic.

2. A 40 year old man presents with symptoms of dysphagia that have been present for many
months. His investigations demonstrate lack of relaxation of the lower oesophageal
sphincter during swallowing.

You answered Benign oesophageal stricture

The correct answer is Achalasia

Patients with dysphagia will usually undergo an upper GI endoscopy as a first line
investigation. Where this investigation is normal, the next stage is to perform studies
assessing oesophageal motility. These comprise fluroscopic barium swallows and
oesophageal manometry and pH studies. Lack of sphincter relaxation suggests achalasia
(pressures are usually high).

3. A 4 year old presents with sudden onset of dysphagia. He undergoes an upper GI endoscopy
and a large bolus of food is identified in the mid oesophagus. He has no significant history,
other than a tracheo-oesophageal fistula repair soon after birth.

Benign oesophageal stricture

Children with tracheo-oesophageal fistulas will commonly develop oesophageal strictures


following repair. These may require regular dilations throughout childhood.

Please rate this question:

Discuss and give feedback


Next question

Dysphagia

Causes of dysphagia

Extrinsic  Mediastinal masses


 Cervical spondylosis

Oesophageal wall  Achalasia


 Diffuse oesophageal spasm
 Hypertensive lower oesophageal sphincter

Intrinsic  Tumours
 Strictures
 Oesophageal web
 Schatzki rings

Neurological  CVA
 Parkinson's disease
 Multiple Sclerosis
 Brainstem pathology
 Myasthenia Gravis

Investigation
All patients require an upper GI endoscopy unless there are compelling reasons for this not to be
performed. Motility disorders may be best appreciated by undertaking fluoroscopic swallowing
studies.

A full blood count should be performed.


Ambulatory oesophageal pH and manometry studies will be required to evaluate conditions such as
achalasia and patients with GORD being considered for fundoplication surgery.
Next question
A 45 year old man undergoes an upper gastrointestinal endoscopy for a benign oesophageal
stricture. This is dilated and he suffers an iatrogenic perforation at the site. His imaging shows a
small contained leak and a small amount of surgical emphysema. What is the most appropriate
nutritional option?

Nil by mouth and intravenous fluids alone

Intravenous fluids and sips orally

Total parenteral nutrition

Nasogastric feeding

PEG tube feeding

Theme from April 2013 Exam


Iatrogenic perforations of the oesophagus may be managed non operatively. This usually involves a
nil by mouth regime, tube thoracostomy may be needed. Total parenteral nutrition is the safest
option. Insertion of NG feeding tubes and PEG tubes may complicate the process or allow feed to
enter the perforation site.
Please rate this question:

Discuss and give feedback


Next question

Nutrition options in surgical patients

Oral intake  Easiest option


 May be supplemented by calorie rich dietary supplements
 May contra indicated following certain procedures

Naso gastric feeding  Usually administered via fine bore naso gastric feeding tube
 Complications relate to aspiration of feed or misplaced tube
 May be safe to use in patients with impaired swallow
 Often contra indicated following head injury due to risks
associated with tube insertion
Naso jejunal feeding  Avoids problems of feed pooling in stomach (and risk of
aspiration)
 Insertion of feeding tube more technically complicated (easiest
if done intra operatively)
 Safe to use following oesophagogastric surgery

Feeding jejunostomy  Surgically sited feeding tube


 May be used for long term feeding
 Low risk of aspiration and thus safe for long term feeding
following upper GI surgery
 Main risks are those of tube displacement and peritubal leakage
immediately following insertion, which carries a risk of
peritonitis

Percutaneous  Combined endoscopic and percutaneous tube insertion


endoscopic  May not be technically possible in those patients who cannot
gastrostomy undergo successful endoscopy
 Risks include aspiration and leakage at the insertion site

Total parenteral  The definitive option in those patients in whom enteral feeding
nutrition is contra indicated
 Individualised prescribing and monitoring needed
 Should be administered via a central vein as it is strongly
phlebitic
 Long term use is associated with fatty liver and deranged LFT's

Next question
A 38 year old women undergoes a gastric bypass procedure. Post operatively she attends the clinic
and complains that following a meal she develops vertigo and develops crampy abdominal pain.
What is the most likely underlying explanation?

Insulin resistance

Irritable bowel syndrome

Biliary colic

Dumping syndrome

Enterogastric reflux

Dumping syndrome, which can be divided into early and late, may occur following gastric surgery. It
occurs as a result of a hyperosmolar load rapidly entering the proximal jejunum. Osmosis drags
water into the lumen, this results in lumen distension (pain) and then diarrhoea. Excessive insulin
release also occurs and results in hypoglycaemic symptoms.
Please rate this question:

Discuss and give feedback


Next question

Post gastrectomy syndromes

Post gastrectomy syndromes may vary slightly depending upon whether a total of partial
gastrectomy is performed. A Roux en Y reconstruction generally gives the best functional outcomes.
Where a gastrojejunostomy is performed as reconstruction following a distal gastrectomy the gastric
emptying is generally better if the jejunal limbs are tunneled in the retrocolic plane.

The following may occur following gastrectomy:

 Small capacity (early satiety)


 Dumping syndrome
 Bile gastritis
 Afferent loop syndrome
 Efferent loop syndrome
 Anaemia (B12 deficiency)
 Metabolic bone disease

Next question
A 63 year old man undergoes an upper GI endoscopy and adrenaline injection for a large actively
bleeding duodenal ulcer. He remains stable for 6 hours and the nurses then call because he has
passed 400ml malaena and has become tachycardic (pulse rate 120) and hypotensive (Bp 80/40).
What is the best option?

Reassure that blood trapped in the upper portion of the gastrointestinal system will pass
and that this episode will resolve with phosphate enema

Perform a repeat upper GI endoscopy

Perform a laparotomy and under-running of the ulcer

Administer tranexamic acid and intravenous proton pump inhibitors

Insert a Minnesota tube

The decision as to how best to manage patients with re-bleeding is difficult. Whilst it is tempting to
offer repeat endoscopy, this intervention is best used on those with small ulcers. Large posteriorly
sited duodenal ulcers are at high risk for re-bleeding and the timeframe of this event suggests that
primary endoscopic haemostasis was inadequate. Surgery thus represents the safest way forward.
Please rate this question:

Discuss and give feedback


Next question

Upper gastrointestinal bleeding

Patients may present with the following:

 Haematemesis and/ or malaena


 Epigastric discomfort
 Sudden collapse

The extent to which these will occur will depend upon the source. Mortality is higher in patients
presenting with haematemesis than malaena alone.

Oesophageal bleeding
Cause Presenting features
Oesophagitis Small volume of fresh blood, often streaking vomit. Malaena rare. Often ceases
spontaneously. Usually history of antecedent GORD type symptoms.

Cancer Usually small volume of blood, except as pre terminal event with erosion of
major vessels. Often associated symptoms of dysphagia and constitutional
symptoms such as weight loss. May be recurrent until malignancy managed.

Mallory Weiss Typically brisk small to moderate volume of bright red blood following bout of
Tear repeated vomiting. Malaena rare. Usually ceases spontaneously.

Varices Usually large volume of fresh blood. Swallowed blood may cause malaena.
Often associated with haemodynamic compromise. May stop spontaneously but
re-bleeds are common until appropriately managed.

Gastric Bleeding
Cause Presenting features

Gastric May be frank haematemesis or altered blood mixed with vomit. Usually
cancer prodromal features of dyspepsia and may have constitutional symptoms. Amount
of bleeding variable but erosion of major vessel may produce considerable
haemorrhage.

Dieulafoy Often no prodromal features prior to haematemesis and malaena, but this
Lesion arteriovenous malformation may produce quite considerable haemorrhage and
may be difficult to detect endoscopically.

Diffuse Usually haematemesis and epigastric discomfort. Usually there is an underlying


erosive cause such as recent NSAID usage. Large volume haemorrhage may occur with
gastritis considerable haemodynamic compromise.

Gastric ulcer Small low volume bleeds more common so would tend to present as iron
deficiency anaemia. Erosion into a significant vessel may produce considerable
haemorrhage and haematemesis.

Duodenum
Most common cause of major haemorrhage is a posteriorly sited duodenal ulcer. However, ulcers at
any site in the duodenum may present with haematemesis, malaena and epigastric discomfort. The
pain of duodenal ulcer is slightly different to that of gastric ulcers and often occurs several hours
after eating. Peri ampullary tumours may bleed but these are rare. In patients with previous
abdominal aortic aneurysm surgery aorto-enteric fistulation remains a rare but important cause of
major haemorrhage associated with high mortality.

Management

 Admission to hospital careful monitoring, cross match blood, check FBC, LFTs, U+E and
Clotting (as a minimum)
 Patients with on-going bleeding and haemodynamic instability are likely to require O negative
blood pending cross matched blood
 Early control of airway is vital (e.g. Drowsy patient with liver failure)
 Patients with suspected varices should receive terlipressin prior to endoscopy
 Ideally all patients admitted with upper gastrointestinal haemorrhage should undergo Upper
GI endoscopy within 24 hours of admission. In those who are unstable this should occur
immediately after resuscitation or in tandem with it. The endoscopy department is a
potentially dangerous place for unstable patients and it may be safer to perform the
endoscopy in theatre with an anaesthetist present.
 Varices should be banded or subjected to sclerotherapy. If this is not possible owing to active
bleeding then a Sengaksten- Blakemore tube (or Minnesota tube) should be inserted. This
should be done with care; gastric balloon should be inflated first and oesophageal balloon
second. Remember the balloon will need deflating after 12 hours (ideally sooner) to prevent
necrosis. Portal pressure should be lowered by combination of medical therapy +/- TIPSS.
 Patients with erosive oesophagitis / gastritis should receive a proton pump inhibitor.
 Mallory Weiss tears will typically resolve spontaneously
 Identifiable bleeding points should receive combination therapy of injection of adrenaline and
either a thermal or mechanical treatment. All who have received intervention should receive
a continuous infusion of a proton pump inhibitor (IV omeprazole for 72 hours) to reduce the
re-bleeding rate.
 Patients with diffuse erosive gastritis who cannot be managed endoscopically and continue
to bleed may require gastrectomy
 Bleeding ulcers that cannot be controlled endoscopically may require laparotomy and ulcer
underruning

Indications for surgery

 Patients > 60 years


 Continued bleeding despite endoscopic intervention
 Recurrent bleeding
 Known cardiovascular disease with poor response to hypotension

Surgery
Duodenal ulcer
Laparotomy, duodenotomy and under running of the ulcer. If bleeding is brisk then the ulcer is
almost always posteriorly sited and will have invaded the gastroduodenal artery. Large bites using 0
Vicryl are taken above and below the ulcer base to occlude the vessel. The duodenotomy should be
longitudinal but closed transversely to avoid stenosis.
For gastric ulcer
Under-running of the bleeding site
Partial gastrectomy-antral ulcer
Partial gastrectomy or under running the ulcer- lesser curve ulcer (involving left gastric artery)
Total gastrectomy if bleeding persists

Summary of Acute Upper GI bleeding recommendations:


The need for admission and timing of endoscopic intervention may be predicted by using the
Blatchford score. This considers a patients Hb, serum urea, pulse rate and blood pressure. Those
patients with a score of 0 are low risk, all others are considered high risk and require admission and
endoscopy.
The requirement for pre endoscopic proton pump inhibition is contentious. In the UK the National
Institute of Clinical Excellence guidelines suggest the pre endoscopic PPI therapy is unnecessary.
Whilst it is accepted that such treatment has no impact on mortality or morbidity a Cochrane review
of this practice in 2007 did suggest that it reduced the stigmata of recent haemorrhage at
endoscopy. As a result many will still administer PPI to patients prior to endoscopic intervention.
Following endoscopy it is important to calculate the Rockall score for patients to determine their risk
of rebleeding and mortality. A score of 3 or less is associated with a rebleeding rate of 4% and a
very low risk of mortality and identifies a group of patients suitable for early discharge.

References
1. http://www.sign.ac.uk/guidelines/fulltext/105/index.html
2. Joint Advisory Group on Endoscopy (JAG) Guidelines - http://www.thejag.org.uk
3. NICE Guideline: Management of acute upper GI bleeding. July 2012.
Next question
Theme: Oesophgeal disease

A. Schatzki ring

B. Plummer Vinson syndrome

C. Squamous cell carcinoma

D. Barretts oesophagus

E. Pharyngeal pouch

F. Adenocarcinoma

G. Leiomyoma

H. Oesophageal rupture

I. Diffuse oesophageal spasm

J. Hiatus hernia

Please select the most likely underlying diagnosis for the scenario described. Each option may be
used once, more than once or not at all.

7. A 56 year old man who drinks heavily is found collapsed by friends at his house. He was out
drinking the previous night and following this was noted to have vomited repeatedly so his friends
brought him home.

You answered Schatzki ring

The correct answer is Oesophageal rupture

Spontaneous rupture of the oesophagus may occur following an episode of vomiting. The
subsequent mediastinitis can produce severe sepsis and death if not treated promptly. Adequate
drainage of sepsis and early surgery are the cornerstones of management.

8. A 43 year old man has been troubled with dysphagia for many years. He is known to have achalasia
and has had numerous dilatations. Over the past 6 weeks his dysphagia has worsened. At
endoscopy a friable mass is noted in the oesophagus.
You answered Schatzki ring

The correct answer is Squamous cell carcinoma

The risk of squamous cell carcinoma of the oesophagus is increased in people with achalasia. The
condition often presents late and has a poor prognosis.

9. A 73 year old lady is troubled by episodic swallowing difficulty and halitosis. An upper GI
endoscopy is attempted and abandoned due to difficulty in achieving intubation.

You answered Schatzki ring

The correct answer is Pharyngeal pouch

Pharyngeal pouches occur when a defect occurs in killians dehiscence. Difficulty in intubation is a
well recognised consequence and care must be taken to take the correct track during OGD to avoid
perforation. Most cases are now treated with endoscopic stapling.

Please rate this question:

Discuss and give feedback

Next question

Oesophageal disease

Disorder Features

Mallory-Weiss Tear Usually history of antecedent vomiting. This is then followed by the vomiting of a
small amount of blood. There is usually little in the way of systemic disturbance
or prior symptoms.
Disorder Features

Hiatus hernia of Often longstanding history of dyspepsia, patients are often overweight.
gastric cardia Uncomplicated hiatus hernias should not be associated with dysphagia or
haematemesis.

Oesophageal rupture Complete disruption of the oesophageal wall in absence of pre-existing


pathology. Left postero-lateral oesophageal is commonest site (2-3cm from OG
junction). Suspect in patients with severe chest pain without cardiac diagnosis
and signs suggestive of pneumonia without convincing history, where there is
history of vomiting. Erect CXR shows infiltrate or effusion in 90% of cases(1).

Squamous cell History of progressive dysphagia. Often signs of weight loss. Usually little or no
carcinoma of the history of previous GORD type symptoms.
oesophagus

Adenocarcinoma of Progressive dysphagia, may have previous symptoms of GORD or Barretts


the oesophagus oesophagus.

Peptic stricture Longer history of dysphagia, often not progressive. Usually symptoms of GORD.
Often lack systemic features seen with malignancy

Dysmotility disorder May have dysphagia that is episodic and non progressive. Retrosternal pain may
accompany the episodes.

Diagnosis
Most of the differential diagnoses listed above can be accurately categorised by upper GI endoscopy
(usually most patients). Where this fails to demonstrate a mechanical stricture the use of pH and
manometry studies together with radiological contrast swallows will facilitate the diagnosis.

References
Blencowe N et al. Spontaneous oesophageal rupture. BMJ 2013 (346):38-39.

Next question
A 55 year old man presents with symptoms of dyspepsia and on upper GI endoscopy an area of
patchy erythematous tissue is identified protruding proximally from the gastro oesophageal junction.
Biopsies are diagnostic of Barretts oesophagus with low grade dysplasia. Which of the following is
the most appropriate management?

Distal oesophagectomy

Upper GI endoscopy with quadrantic biopsies from the region

Photodynamic therapy

Endoscopic sub mucosal resection of the area

Argon plasma coagulation

In Barrett's surveillance the safest option is quadrantic (i.e. 4 biopsies, one from each quarter of the
oesophagus at 2cm intervals)

Low grade dysplasia in conjunction with Barretts oesphagus should be monitored with regular (6
monthly) upper GI endoscopy and quadrantic biopsies. If the disease remains static at 2 years then
the screening frequency may be decreased.
Please rate this question:

Discuss and give feedback


Next question

Barrett's oesophagus

Barretts oesophagus is a condition characterised by the metaplastic transformation of squamous


oesophageal epithelium to columnar gastric type epithelium. Three types of this metaplastic process
are recognised; intestinal (high risk), cardiac and fundic. The latter two categories may cause
difficulties in diagnosis. The most concrete diagnosis can be made when endoscopic features of
Barretts oesophagus are present together with a deep biopsy that demonstrates not just goblet cell
metaplasia but also oesophageal glands.

Barrett's can be sub divided into short (<3cm) and long (>3cm). The length of the affected segment
correlates strongly with the chances of identifying metaplasia. The overall prevalence of Barrett's
oesophagus is difficult to determine but may be in the region of 1 in 20 and is identified in up to 12%
of those undergoing endoscopy for reflux.

A proportion of patients with metaplasia will progress to dysplasia and for this reason individuals
identified as having Barrett's should undergo endoscopic surveillance (every 2-5 years). Biopsies
should be quadrantic and taken at 1-2cm intervals. Biopsies need to be adequate. Where mass
lesions are present consideration should be given to endoscopic sub mucosal resection. Up to 40%
of patients will be upstaged from high grade dysplasia to invasive malignancy with such techniques.

Treatment

 Long term proton pump inhibitor


 Consider pH and manometry studies in younger patients who may prefer to consider an anti
reflux procedure
 Regular endoscopic monitoring (more frequently if moderate dysplasia). With quadrantic
biopsies every 2-3 cm
 If severe dysplasia be very wary of small foci of cancer

References
A consensus statement of the British approach is provided by:
Bennett C et al Consensus Statements for Management of Barrett's Dysplasia and Early-Stage
Esophageal Adenocarcinoma, Based on a Delphi Process. Gastroenterology Volume 143, Issue 2 ,
Pages 336-346, August 2012.
Next question
Theme: Causes of dysphagia

A. Adenocarcinoma of the oesophagus


B. Plummer Vinson syndrome
C. Squamous cell carcinoma of the oesophagus
D. Achalasia
E. Benign oesophageal stricture
F. Pharyngeal pouch
G. Tracheo-oesophageal fistula

Please select the most likely underlying cause for dysphagia from the scenario given. Each option
may be used once, more than once or not at all.

11. A 78 year old lady presents with episodic dysphagia and halitosis, occasionally she
complains of regurgitation. A recent attempted upper GI endoscopy was poorly tolerated
and abandoned.

You answered Adenocarcinoma of the oesophagus

The correct answer is Pharyngeal pouch

This is a pulsion type diverticulum through Killians dehiscence. Patients often complain of
regurgitation of foul smelling liquid. Upper GI endoscopy is potentially hazardous and
may result in iatrogenic perforation.

12. An overweight 56 year old man with longstanding Barrett's oesophagus complains of
worsening dysphagia to solids over the past 6 weeks.

Adenocarcinoma of the oesophagus

Barrett's metaplasia places patients at increased risk of adenocarcinoma of the oesophagus.


Small localised lesions and in situ disease has been treated by endoscopic mucosal
resection and photodynamic therapy. In many centres oesophagectomy is still offered as
the safest and standard treatment. Where dysphagia is present, a significant lesion
(requiring resection if operable) is invariably found.

13. A 24 year old man complains of occasional retrosternal chest pain and dysphagia that
occurs to both liquids and solids. He is otherwise well.

You answered Adenocarcinoma of the oesophagus

The correct answer is Achalasia


This is an atypical cause of these symptoms. Diagnosis is made by upper GI endoscopy,
barium swallow and manometry/ pH studies. Treatments range from botulinum toxin
injection through to Hellers Cardiomyotomy that may be performed laparoscopically.

Please rate this question:

Discuss and give feedback


Next question

Dysphagia

Causes of dysphagia

Extrinsic  Mediastinal masses


 Cervical spondylosis

Oesophageal wall  Achalasia


 Diffuse oesophageal spasm
 Hypertensive lower oesophageal sphincter

Intrinsic  Tumours
 Strictures
 Oesophageal web
 Schatzki rings

Neurological  CVA
 Parkinson's disease
 Multiple Sclerosis
 Brainstem pathology
 Myasthenia Gravis

Investigation
All patients require an upper GI endoscopy unless there are compelling reasons for this not to be
performed. Motility disorders may be best appreciated by undertaking fluoroscopic swallowing
studies.

A full blood count should be performed.

Ambulatory oesophageal pH and manometry studies will be required to evaluate conditions such as
achalasia and patients with GORD being considered for fundoplication surgery.
Next question
A 58 year old lady has a two year history of worsening dysphagia. In addition to this she has a
nocturnal cough. Over this time she has lost a total of 8kg in weight. Which of the processes below is
the most likely explanation for this?

Gastro-oesophageal reflux disease

Barretts oesophagus

Carcinoma

Mallory Weiss tear

Achalasia

Theme from April 2013 Exam


Such marked weight loss should arouse suspicion of cancer. She is most likely to have a
longstanding stricture associated with oesophagitis and Barretts oesophagus. This may progress to
carcinoma which will tend to occur in association with worsening dysphagia and weight loss.
Please rate this question:

Discuss and give feedback


Next question

Oesophageal cancer

 Incidence is increasing
 In most cases in the Western world this increase is accounted for by a rise in the number of
cases of adenocarcinoma. In the UK adenocarcinomas account for 65% of cases.
 Barretts oesophagus is a major risk factor for most cases of oesophageal adenocarcinoma.
 In other regions of the world squamous cancer is more common and is linked to smoking,
alcohol intake, diets rich in nitrosamines and achalasia.
 Surveillance of Barretts is important, as it imparts a 30 fold increase in cancer risk and if
invasive malignancy is diagnosed early then survival may approach 85% at 5 years.

Diagnosis

 Upper GI endoscopy is the first line test


 Contrast swallow may be of benefit in classifying benign motility disorders but has no place
in the assessment of tumours
 Staging is initially undertaken with CT scanning of the chest, abdomen and pelvis. If overt
metastatic disease is identified using this modality then further complex imaging is
unnecessary
 If CT does not show metastatic disease, then local stage may be more accurately assessed
by use of endoscopic ultrasound.
 Staging laparoscopy is performed to detect occult peritoneal disease. PET CT is performed
in those with negative laparoscopy. Thoracoscopy is not routinely performed.

Treatment
Operable disease is best managed by surgical resection. The most standard procedure is an Ivor-
Lewis type oesophagectomy. This procedure involves the mobilisation of the stomach and division of
the oesophageal hiatus. The abdomen is closed and a right sided thoracotomy performed. The
stomach is brought into the chest and the oesophagus mobilised further. An intrathoracic
oesophagogastric anastomosis is constructed. Alternative surgical strategies include a transhiatal
resection (for distal lesions), a left thoraco-abdominal resection (difficult access due to thoracic
aorta) and a total oesophagectomy (McKeown) with a cervical oesophagogastric anastomosis.
The biggest surgical challenge is that of anastomotic leak, with an intrathoracic anastomosis this will
result in mediastinitis. With high mortality. The McKeown technique has an intrinsically lower
systemic insult in the event of anastomotic leakage.

In addition to surgical resection many patients will be treated with adjuvant chemotherapy.
Next question
Theme: Management of oesophageal cancer

A. Endo lumenal brachytherapy


B. Chemo-radiotherapy
C. Radiotherapy alone
D. Insertion of expanding metallic stent
E. Ivor-Lewis oesophagectomy
F. Total oesophagectomy
G. Segmental resection of mid oesophagus
H. Endoscopic mucosal resection

Please select the most appropriate intervention for the following patients with oesophageal cancer.
Each option may be used once, more than once or not at all.

15. A 58 year old man with long standing Barretts oesophagus is found to have a nodule on
endoscopic surveillence. Biopsies and endoscopic USS suggest this is at most a 1cm foci
of T1 disease in the distal oesophagus 4 cm proximal to the oesophagogastric junction.

You answered Endo lumenal brachytherapy

The correct answer is Endoscopic mucosal resection

EMR is an reasonable option for small areas of malignancy occurring on a background of


Barretts change. Segmental resections of the oesophagus are not practised and the only
resectional strategy in this scenario would be an Ivor- Lewis type resection. The morbidity
such a strategy in T1 disease is probably not justified.

16. An 82 year old man presents with dysphagia and on investigation is found to have a
stenosing tumour of the mid oesophagus with a single mestastasis in the right lobe of the
liver (segment VI).

You answered Endo lumenal brachytherapy

The correct answer is Insertion of expanding metallic stent

Distant disease in patients with oesophageal cancer is a contra indication to a resectional


strategy and downstaging with chemotherapy is not routinely undertaken in this age group
as the results are poor. An expanding stent will provide rapid and durable palliation.

17. A 56 year old man presents with odynophagia and on investigation is found to have a
squamous cell carcinoma of the upper third of the oesophagus. Staging investigations are
negative for metastatic disease.
You answered Endo lumenal brachytherapy

The correct answer is Chemo-radiotherapy

SCC of the oesophagus is treated with chemo-radiotherapy in the first instance.

Please rate this question:

Discuss and give feedback


Next question

Oesophageal cancer - treatment

Treatments for SCC's and adenocarcinomas of the oesophagus differ. This is primarily due to the
positive outcomes that are observed when localised SCC's (particularly of the proximal oesophagus
are treated with radical chemoradiotherapy (obviating the need for surgery).
Only those patients whose staging investigations are negative for metastatic disease should be
considered for surgery.

Surgical options
Endoscopic mucosal Treatment for early localised adenocarcinoma of the distal oesophagus.
resection Survival mirrors that of surgical resection for Tis and T1 disease

Transhiatal Most commonly used for junctional (type II) (1) tumours where limited
oeosphagectomy thoracic oesophageal resection is required. Less morbidity than two field
oesophagectomy

Ivor Lewis Two stage approach for middle and distal tumours. Very commonly
oesophagectomy performed, intrathoracic anastomosis will result in mediastinitis in event of
anastomotic leak. Lower incidence of recurrent laryngeal nerve injury

McKeown Three field approach, may be useful for proximal tumours. Anastomotic
oesophagectomy leakage is less serious. Higher incidence of recurrent laryngeal nerve
injury

Neoadjuvent and adjuvent treatment

 Neoadjuvent radiotherapy alone prior to resection confers little benefit and is not routinely
performed (2)
 Preoperative chemotherapy is associated with a survival advantage (OE02 trial)
 Peri operative (pre and post operative) chemotherapy confers a survival advantage in
junctional tumours
 Post operative chemotherapy is not generally recommended following oesophageal
resections outside clinical trials

Palliation strategies

 Combination chemotherapy improves quality of life and survival in non operable disease (3)
 Trastuzumab may improve survival in patients with HER 2 positive tumours
 Oesophageal intubation with self expanding metal stents is the treatment of choice in
patients with occluding tumours >2cm from the cricopharyngeus
 Covered metal stents are useful in cases of malignant fistulas
 Laser therapy and argon plasma coagulation may be useful as therapies for tumour
overgrowth and bleeding
 Photodynamic therapy and ethanol injections confer little benefit and should not be routinely
used

References
1. Hulscher JB, van Sandick JW, de Boer AG, et al. Extended transthoracic resection compared with
limited transhiatal resection for adenocarcinoma of the esophagus. N Engl J Med 2002;347:1662-9
2. Arnott SJ, Duncan W, Gignoux M, et al. Preoperative radiotherapy for esophageal carcinoma.
Cochrane Database Syst Rev 2005;(4):CD001799.
3.Allum W et al. Guidelines for the management of oesophageal and gastric cancer. Gut 2011;60:
1449-1472
Next question
Theme: Oesophageal disease

A. Oesophagectomy
B. Endoscopic sub mucosal dissection
C. Photodynamic therapy
D. Insertion of oesophageal stent
E. Chemotherapy
F. Radiotherapy

Please select the most appropriate treatment modality for the scenario given. Each option may be
used once, more than once or not at all.

18. A 52 year old man with long standing Barretts oesophagus is diagnosed with high grade
dysplasia on recent endoscopy. The lesions are multifocal and mainly distally sited.

Oesophagectomy

Similar theme in 2011 exams

Some may argue for local therapy. However, in young patients who are otherwise fit,
multifocal disease such as this should probably be resected.

19. A 82 year old man presents with dysphagia. He is investigated and found to have an
adenocarcinoma of the distal oesophagus. His staging investigations have revealed a
solitary metastatic lesion in the right lobe of his liver.

You answered Oesophagectomy

The correct answer is Insertion of oesophageal stent

Similar theme in 2011 exam


Although he may be palliated with chemotherapy a stent will produce the quickest clinical
response. Metastatic disease is usually a contra indication to oesophageal resection.

20. An 83 year old lady with long standing Barretts oesophagus is diagnosed with a 1cm focus
of high grade dysplasia 3cm from the gastrooesophageal junction.

You answered Oesophagectomy

The correct answer is Endoscopic sub mucosal dissection

As she is elderly and the disease localised EMR is an appropriate first line step.
The technique involves raising the mucosa containing the lesion and then using an
endoscopic snare to remove it. This technique is therefore minimally invasive. However, it
is only suitable for early superficial lesions. Deeper invasion would carry a high risk of
recurrence.

Please rate this question:

Discuss and give feedback


Next question

Treatment of oesophageal cancer

 In general resections are not offered to those patients with distant metastasis, and usually
not to those with N2 disease.
 Local nodal involvement is not in itself a contra indication to resection.
 Surgical resection is the mainstay of treatment.
 Neoadjuvent chemotherapy is given in most cases prior to surgery.
 In situ disease may be managed by endoscopic mucosal resection, although this is still
debated.
 In patients with lower third lesions an Ivor - Lewis type procedure is most commonly
performed. Very distal tumours may be suitable to a transhiatal procedure. Which is an
attractive option as the penetration of two visceral cavities required for an Ivor- Lewis type
procedure increases the morbidity considerably.
 More proximal lesions will require a total oesphagectomy (Mckeown type) with anastomosis
to the cervical oesophagus.
 Patients with unresectable disease may derive benefit from local ablative procedures,
palliative chemotherapy or stent insertion.

Operative details of Ivor- Lewis procedure

 Combined laparotomy and right thoracotomy

Indication

 Lower and middle third oesophageal tumours

Preparation

 Staging with a combination of CT chest abdomen and pelvis- if no metastatic disease


detected then patients will undergo a staging laparoscopy to detect peritoneal disease.
 If both these modalities are negative then patients will finally undergo a PET CT scan to
detect occult metastatic disease. Only in those whom no evidence of advanced disease is
detected will proceed to resection.
 Patients receive a GA, double lumen endotracheal tube to allow for lung deflation, CVP and
arterial monitoring.

Procedure

 A rooftop incision is made to access the stomach and duodenum.

Laparotomy To mobilize the stomach

 The greater omentum is incised away from its attachment to the right gastroepiploic vessels
along the greater curvature of the stomach.
 Then the short gastric vessels are ligated and detached from the greater curvature from the
spleen.
 The lesser omentum is incised, preserving the right gastric artery.
 The retroperitoneal attachments of the duodenum in its second and third portions are
incised, allowing the pylorus to reach the oesophageal hiatus. Some surgeons perform a
pyloroplasty at this point to facilitate gastric emptying.
 The left gastric vessels are then ligated, avoiding any injury to the common hepatic or splenic
arteries. Care must be taken to avoid inadvertently devascularising the liver owing to
variations in anatomy.

Right Thoracotomy Oesophageal resection and oesophagogastric anastomosis

 Through 5th intercostal space


 Dissection performed 10cm above the tumour
 This may involve transection of the azygos vein.
 The oesophagus is then removed with the stomach creating a gastric tube.
 An anastomosis is created.

The chest is closed with underwater seal drainage and tube drains to the abdominal cavity.

Post operatively

 Patients will typically recover in ITU initially.


 A nasogastric tube will have been inserted intraoperatively and must remain in place during
the early phases of recovery.
 Post operatively these patients are at relatively high risk of developing complications:

* Atelectasis- due to the effects of thoracotomy and lung collapse


* Anastomotic leakage. The risk is relatively high owing to the presence of a relatively
devascularised stomach. Often the only blood supply is from the gastroepiploic artery as all others
will have been divided. If a leak does occur then many will attempt to manage conservatively with
prolonged nasogastric tube drainage and TPN. The reality is that up to 50% of patients developing
an anastomotic leak will not survive to discharge.
* Delayed gastric emptying (may be avoided by performing a pyloroplasty).
Next question
Theme: Pancreatic tumours

A. Lymphoma
B. Gastrinoma
C. Insulinoma
D. Glucagonoma
E. Phaeochromocytoma
F. Carcinoid syndrome
G. Vasoactive Intestinal Peptide secreting tumour
H. Pancreatic adenocarcinoma

Please select the most likely diagnosis for the scenario given. Each option may be used once, more
than once or not at all.

21. A 65 year old male attends surgical out patients with epigastric discomfort. He has
recently been diagnosed with diabetes by the GP and is a heavy smoker. An OGD is
normal.

You answered Lymphoma

The correct answer is Pancreatic adenocarcinoma

The dominant differential diagnosis should be of pancreatic adenocarcinoma in this


setting. Glucagonomas are very rare and may be associated with a bullous rash.

22. A 50 year old male presents with recurrent episodes of abdominal pain and diarrhoea.
Blood tests reveal mild iron deficiency anaemia and an upper GI endoscopy demonstrates
multiple ulcers in the first part of the duodenum.

You answered Lymphoma

The correct answer is Gastrinoma

Diarrhoea, abdominal pain and multiple ulcers should raise the suspicion of Zollinger
Ellison syndrome caused by gastrinoma.

23. An obese 40 year old male presents with episodes of anxiety, confusion and one
convulsive episode. CT brain is normal. An abdominal CT scan shows a small 1.5cm
lesion in the head of the pancreas.

You answered Lymphoma

The correct answer is Insulinoma


These episodes are due to hypoglycaemia. Insulinomas are normally solitary tumours and
may not be seen by radiological imaging. Resection is the treatment of choice.

Please rate this question:

Discuss and give feedback


Next question

Pancreatic cancer

 Adenocarcinoma
 Risk factors: Smoking, diabetes, adenoma, familial adenomatous polyposis
 Mainly occur in the head of the pancreas (70%)
 Spread locally and metastasizes to the liver
 Carcinoma of the pancreas should be differentiated from other periampullary tumours with
better prognosis

Clinical features

 Weight loss
 Painless jaundice
 Epigastric discomfort (pain usually due to invasion of the coeliac plexus is a late feature)
 Pancreatitis
 Trousseau's sign: migratory superficial thrombophlebitis

Investigations

 USS: May miss small lesions


 CT Scanning (pancreatic protocol). If unresectable on CT then no further staging needed
 PET/CT for those with operable disease on CT alone
 ERCP/ MRI for bile duct assessment
 Staging laparoscopy to exclude peritoneal disease

Management

 Head of pancreas: Whipple's resection (SE dumping and ulcers). Newer techniques include
pylorus preservation and SMA/ SMV resection
 Carcinoma body and tail: poor prognosis, distal pancreatectomy, if operable
 Usually adjuvent chemotherapy for resectable disease
 ERCP and stent for jaundice and palliation
 Surgical bypass may be needed for duodenal obstruction
A 45 year old man has a 4 week history of epigastric discomfort which is relieved by eating. He
develops haematemesis and undergoes an upper GI endoscopy. An actively bleeding ulcer is noted
in the first part of the duodenum. What is the best management?

Whipples procedure

Truncal vagotomy and drainage

Distal gastrectomy

Injection with tranexamic acid

Injection with adrenaline

Current guidance is that bleeding peptic ulcers should be treated with dual therapeutic modalities.
Adrenaline injection should be augmented with an additional therapy such as endoscopic clipping
where this is available.

Bleeding duodenal ulcers will usually undergo adrenaline injection. This may be augmented by the
placement of endoscopic clips or heat therapy with endoscopic heater probes. Following these
interventions patients should receive a proton pump inhibitor infusion. Those who re-bleed, may
require surgery. For ulcers in this location, laparotomy, duodenotomy and under-running of the ulcer
is usually performed.
Please rate this question:

Discuss and give feedback


Next question

Upper gastrointestinal bleeding

Patients may present with the following:

 Haematemesis and/ or malaena


 Epigastric discomfort
 Sudden collapse

The extent to which these will occur will depend upon the source. Mortality is higher in patients
presenting with haematemesis than malaena alone.
Oesophageal bleeding
Cause Presenting features

Oesophagitis Small volume of fresh blood, often streaking vomit. Malaena rare. Often ceases
spontaneously. Usually history of antecedent GORD type symptoms.

Cancer Usually small volume of blood, except as pre terminal event with erosion of
major vessels. Often associated symptoms of dysphagia and constitutional
symptoms such as weight loss. May be recurrent until malignancy managed.

Mallory Weiss Typically brisk small to moderate volume of bright red blood following bout of
Tear repeated vomiting. Malaena rare. Usually ceases spontaneously.

Varices Usually large volume of fresh blood. Swallowed blood may cause malaena.
Often associated with haemodynamic compromise. May stop spontaneously but
re-bleeds are common until appropriately managed.

Gastric Bleeding
Cause Presenting features

Gastric May be frank haematemesis or altered blood mixed with vomit. Usually
cancer prodromal features of dyspepsia and may have constitutional symptoms. Amount
of bleeding variable but erosion of major vessel may produce considerable
haemorrhage.

Dieulafoy Often no prodromal features prior to haematemesis and malaena, but this
Lesion arteriovenous malformation may produce quite considerable haemorrhage and
may be difficult to detect endoscopically.

Diffuse Usually haematemesis and epigastric discomfort. Usually there is an underlying


erosive cause such as recent NSAID usage. Large volume haemorrhage may occur with
gastritis considerable haemodynamic compromise.

Gastric ulcer Small low volume bleeds more common so would tend to present as iron
deficiency anaemia. Erosion into a significant vessel may produce considerable
haemorrhage and haematemesis.

Duodenum
Most common cause of major haemorrhage is a posteriorly sited duodenal ulcer. However, ulcers at
any site in the duodenum may present with haematemesis, malaena and epigastric discomfort. The
pain of duodenal ulcer is slightly different to that of gastric ulcers and often occurs several hours
after eating. Peri ampullary tumours may bleed but these are rare. In patients with previous
abdominal aortic aneurysm surgery aorto-enteric fistulation remains a rare but important cause of
major haemorrhage associated with high mortality.

Management

 Admission to hospital careful monitoring, cross match blood, check FBC, LFTs, U+E and
Clotting (as a minimum)
 Patients with on-going bleeding and haemodynamic instability are likely to require O negative
blood pending cross matched blood
 Early control of airway is vital (e.g. Drowsy patient with liver failure)
 Patients with suspected varices should receive terlipressin prior to endoscopy
 Ideally all patients admitted with upper gastrointestinal haemorrhage should undergo Upper
GI endoscopy within 24 hours of admission. In those who are unstable this should occur
immediately after resuscitation or in tandem with it. The endoscopy department is a
potentially dangerous place for unstable patients and it may be safer to perform the
endoscopy in theatre with an anaesthetist present.
 Varices should be banded or subjected to sclerotherapy. If this is not possible owing to active
bleeding then a Sengaksten- Blakemore tube (or Minnesota tube) should be inserted. This
should be done with care; gastric balloon should be inflated first and oesophageal balloon
second. Remember the balloon will need deflating after 12 hours (ideally sooner) to prevent
necrosis. Portal pressure should be lowered by combination of medical therapy +/- TIPSS.
 Patients with erosive oesophagitis / gastritis should receive a proton pump inhibitor.
 Mallory Weiss tears will typically resolve spontaneously
 Identifiable bleeding points should receive combination therapy of injection of adrenaline and
either a thermal or mechanical treatment. All who have received intervention should receive
a continuous infusion of a proton pump inhibitor (IV omeprazole for 72 hours) to reduce the
re-bleeding rate.
 Patients with diffuse erosive gastritis who cannot be managed endoscopically and continue
to bleed may require gastrectomy
 Bleeding ulcers that cannot be controlled endoscopically may require laparotomy and ulcer
underruning

Indications for surgery

 Patients > 60 years


 Continued bleeding despite endoscopic intervention
 Recurrent bleeding
 Known cardiovascular disease with poor response to hypotension
Surgery
Duodenal ulcer
Laparotomy, duodenotomy and under running of the ulcer. If bleeding is brisk then the ulcer is
almost always posteriorly sited and will have invaded the gastroduodenal artery. Large bites using 0
Vicryl are taken above and below the ulcer base to occlude the vessel. The duodenotomy should be
longitudinal but closed transversely to avoid stenosis.

For gastric ulcer


Under-running of the bleeding site
Partial gastrectomy-antral ulcer
Partial gastrectomy or under running the ulcer- lesser curve ulcer (involving left gastric artery)
Total gastrectomy if bleeding persists

Summary of Acute Upper GI bleeding recommendations:


The need for admission and timing of endoscopic intervention may be predicted by using the
Blatchford score. This considers a patients Hb, serum urea, pulse rate and blood pressure. Those
patients with a score of 0 are low risk, all others are considered high risk and require admission and
endoscopy.
The requirement for pre endoscopic proton pump inhibition is contentious. In the UK the National
Institute of Clinical Excellence guidelines suggest the pre endoscopic PPI therapy is unnecessary.
Whilst it is accepted that such treatment has no impact on mortality or morbidity a Cochrane review
of this practice in 2007 did suggest that it reduced the stigmata of recent haemorrhage at
endoscopy. As a result many will still administer PPI to patients prior to endoscopic intervention.
Following endoscopy it is important to calculate the Rockall score for patients to determine their risk
of rebleeding and mortality. A score of 3 or less is associated with a rebleeding rate of 4% and a
very low risk of mortality and identifies a group of patients suitable for early discharge.

References
1. http://www.sign.ac.uk/guidelines/fulltext/105/index.html
2. Joint Advisory Group on Endoscopy (JAG) Guidelines - http://www.thejag.org.uk
3. NICE Guideline: Management of acute upper GI bleeding. July 2012.
Next question
Which of the following patients should be referred for 1st line bariatric surgery?

BMI 35 kg/m2 and hypertension

BMI 40 kg/m2

BMI 40 kg/m2 and type 2 diabetes

BMI 52kg/m2

BMI 40kg/m2, COPD and type 2 diabetes

With all the other options the patient should have conservative management for a minimum of 6
months first.
Please rate this question:

Discuss and give feedback


Next question

Bariatric surgery

Obesity is a major health problem in the Western world. Surgical solutions to the problem have
evolved dramatically over the past few years. Randomised controlled trials have shown that dramatic
weight loss can be achieved following surgical interventions compared with standard medical
therapy. The weight loss process is also more durable following surgery than with non surgical
interventions.

Case selection
BMI >/= 40 kg/m 2 or between 35-40 kg/m 2 and other significant disease (for example, type 2
diabetes, hypertension) that could be improved with weight loss.

Pre-requisites to surgery (NICE UK Guidelines)

 All non-surgical measures have failed to achieve or maintain adequate clinically beneficial
weight loss for at least 6 months.
 Will receive intensive specialist management
 They are generally fit for anaesthesia and surgery
 They commit to the need for long-term follow-up
 First-line option for adults with a BMI > 50 kg/m 2 in whom surgical intervention is considered
appropriate; consider orlistat if there is a long waiting list.
Surgical options
Adjustable gastric  Laparoscopic placement of adjustable band around proximal
band stomach.
 Contains an adjustable filling port
 Effective method for lifestyle control
 Reversible
 Takes longer to achieve target weight
 Complications such as band erosion (rare), slippage or loss of
efficacy may require re-intervention

Gastric bypass  Combines changes to reservoir size with malabsorptive procedure


for more enduring weight loss.
 Technically more challenging
 Risks related to anastomoses (2% leak rate)
 Irreversible
 Up to 50% may become B12 deficient

Sleeve gastrectomy  Resection of stomach using stapling devices


 Less popular now as initial promising results not sustained

References
1. National Institute for Health and Clinical Excellence. Obesity: the prevention, identification,
assessment and management of overweight and obesity in adults and children.
www.nice.org.uk/CG43

2. Leff DR, Heath D. Surgery for obesity in adulthood. BMJ 2009;339:b3402


Next question
Theme: Management of gastric cancer

A. Radical radiotherapy
B. Endoscopic submucosal resection
C. Polya Gastrectomy
D. Distal gastrectomy and anterior gastrojejunostomy
E. Distal gastrectomy and posterior gastrojejunostomy
F. Belsey Mark IV procedure
G. Sub total gastrectomy and Roux and Y reconstruction
H. Total gastrectomy and Roux en Y reconstruction

Please select the most appropriate management for the gastric cancer case described. Each option
may be used once, more than once or not at all.

26. An otherwise fit 73 year old man presents with gastric outlet obstruction. An upper GI
endoscopy shows a prepyloric tumour occluding the pylorus. Staging investigations show
nodal disease at D2 and an involved paraaortic lymph node.

You answered Radical radiotherapy

The correct answer is Distal gastrectomy and anterior gastrojejunostomy

This man does not have disease amenable to curative surgical resection. However, good
palliation can be achieved with a resection and chemotherapy. He is likely to have
recurrent disease in the gastric bed and an anterior gastrojejunostomy is therefore
preferred.

27. A 40 year old lady presents with a gastric carcinoma of the greater curvature of the
stomach. Her staging investigations are negative for metastatic disease.

You answered Radical radiotherapy

The correct answer is Sub total gastrectomy and Roux and Y reconstruction

This is amenable to potentially curative resection. The proximal stomach can be


conserved.

28. A 62 year old man presents with dyspepsia and a tumour of the gastric cardia is diagnosed.
He has no evidence of metastatic disease.

You answered Radical radiotherapy

The correct answer is Total gastrectomy and Roux en Y reconstruction


This will require a total gastrectomy. Retention of a gastric remnant is unlikely to achieve
acceptable resection margins

Please rate this question:

Discuss and give feedback


Next question

Gastric cancer

Overview
There are 700,000 new cases of gastric cancer worldwide each year. It is most common in Japan
and less common in western countries. It is more common in men and incidence rises with
increasing age. The exact cause of many sporadic cancer is not known, however, familial cases do
occur in HNPCC families. In addition, smoking and smoked or preserved foods increase the risk.
Japanese migrants retain their increased risk (decreased in subsequent generations). The
distribution of the disease in western countries is changing towards a more proximal location
(perhaps due to rising obesity).

Pathology
There is some evidence of support a stepwise progression of the disease through intestinal
metaplasia progressing to atrophic gastritis and subsequent dysplasia, through to cancer. The
favoured staging system is TNM. The risk of lymph node involvement is related to size and depth of
invasion; early cancers confined to submucosa have a 20% incidence of lymph node metastasis.
Tumours of the gastro-oesophageal junction are classified as below:

Type True oesophageal cancers and may be associated with Barrett's oesophagus.
1

Type Carcinoma of the cardia, arising from cardiac type epithelium


2 or short segments with intestinal metaplasia at the oesophagogastric junction.

Type Sub cardial cancers that spread across the junction. Involve similar nodal stations to
3 gastric cancer.

Groups for close endoscopic monitoring

 Intestinal metaplasia of columnar type


 Atrophic gastritis
 Low to medium grade dysplasia
 Patients who have previously undergone resections for benign peptic ulcer disease (except
highly selective vagotomy).

Referral to endoscopy

Patients of any age with Patients without Worsening dyspepsia


dyspepsia and any of the dyspepsia
following

Chronic gastrointestinal bleeding Dysphagia Barretts oesophagus

Dysphagia Unexplained abdominal Intestinal metaplasia


pain or weight loss

Weight loss Vomiting Dysplasia

Iron deficiency anaemia Upper abdominal mass Atrophic gastritis

Upper abdominal mass Jaundice Patient aged over 55 years with


unexplained or persistent
dyspepsia

Upper GI endoscopy performed for dyspepsia. The addition of dye spraying (as shown in the bottom
right) may facilitate identification of smaller tumours
Image sourced from Wikipedia

Staging

 CT scanning of the chest abdomen and pelvis is the routine first line staging investigation in
most centres.
 Laparoscopy to identify occult peritoneal disease
 PET CT (particularly for junctional tumours)

Treatment

 Proximally sited disease greater than 5-10cm from the OG junction may be treated by sub
total gastrectomy
 Total gastrectomy if tumour is <5cm from OG junction
 For type 2 junctional tumours (extending into oesophagus) oesophagogastrectomy is usual
 Endoscopic sub mucosal resection may play a role in early gastric cancer confined to the
mucosa and perhaps the sub mucosa (this is debated)
 Lymphadenectomy should be performed. A D2 lymphadenectomy is widely advocated by the
Japanese, the survival advantages of extended lymphadenectomy have been debated.
However, the overall recommendation is that a D2 nodal dissection be undertaken.
 Most patients will receive chemotherapy either pre or post operatively.

Prognosis
UK Data

Disease extent Percentage 5 year survival

All RO resections 54%

Early gastric cancer 91%

Stage 1 87%

Stage 2 65%

Stage 3 18%

Operative procedure

Total Gastrectomy , lymphadenectomy and Roux en Y anastomosis

General anaesthesia
Prophylactic intravenous antibiotics
Incision: Rooftop.
Perform a thorough laparotomy to identify any occult disease.
Mobilise the left lobe of the liver off the diaphragm and place a large pack over it. Insert a large self
retaining retractor e.g. omnitract or Balfour (take time with this, the set up should be perfect). Pack
the small bowel away.
Begin by mobilising the omentum off the transverse colon.
Proceed to detach the short gastric vessels.
Mobilise the pylorus and divide it at least 2cm distally using a linear cutter stapling device.
Continue the dissection into the lesser sac taking the lesser omentum and left gastric artery flush at
its origin.
The lymph nodes should be removed en bloc with the specimen where possible.
Place 2 stay sutures either side of the distal oesophagus. Ask the anaesthetist to pull back on the
nasogastric tube. Divide the distal oesophagus and remove the stomach.
The oesphago jejunal anastomosis should be constructed. Identify the DJ flexure and bring a loop of
jejunum up to the oesophagus (to check it will reach). Divide the jejunum at this point. Bring the
divided jejunum either retrocolic or antecolic to the oesophagus. Anastamose the oesophagus to the
jejunum, using either interrupted 3/0 vicryl or a stapling device. Then create the remainder of the
Roux en Y reconstruction distally.
Place a jejunostomy feeding tube.
Wash out the abdomen and insert drains (usually the anastomosis and duodenal stump). Help the
anaesthetist insert the nasogastric tube (carefully!)
Close the abdomen and skin.
Enteral feeding may commence on the first post-operative day. However, most surgeons will leave
patients on free NG drainage for several days and keep them nil by mouth.
Next question
Theme: Causes of upper gastrointestinal haemorrhage

A. Antral gastric ulcer


B. Mallory Weiss tear
C. Oesphageal varices
D. Dieulafoy lesion
E. Gastritis
F. Duodenal ulcer anterior wall
G. Duodenal ulcer posterior wall

Please select the most appropriate likely diagnosis for the scenario given. Each option may be used
once, more than once or not at all.

29. A 35 year old man is admitted with an episode of collapse and passage of malaena. He has
been suffering from post prandial abdominal pain for 5 weeks and this is most marked
several hours after eating.

You answered Antral gastric ulcer

The correct answer is Duodenal ulcer posterior wall

Patients with duodenal ulcers will usually have a history of epigastric pain that occurs
several hours after eating. The pain is often improved by eating food. They are most
frequently located i nthe first part of the duodenum. Anteriorly sited ulcers may perforate
and result in peritonitis, posteriorly sited ulcers may erode the gastroduodenal artery and
present with haematemesis and/ or malaena.

30. A 72 year old man is admitted with an episode of brisk haematemesis. Following
resuscitation an upper GI endoscopy is performed and a prominent blood vessel is
identified in the mucosa approximately 6 cm from the O-G junction on the lesser curve of
the stomach.

You answered Antral gastric ulcer

The correct answer is Dieulafoy lesion

These small arterial lesions are a rare cause of bleeding and are characterised by a single
large tortuous arteriole in the sub mucosa. Extra gastric lesions may occur.

31. A 56 year old man is admitted with a profuse upper gastro intestinal haemorrhage. He is
relatively malnourished and has evidence of gynaecomastia.

You answered Antral gastric ulcer


The correct answer is Oesphageal varices

Patients presenting with gastrointestinal bleeding and evidence of established liver disease
may have portal hypertension and develop variceal haemorrhage. The patient may have
evidence of jaundice, gynaecomastia, spider naevia, caput medusae and ascites. The
bleeding is usually profuse and painless.

Please rate this question:

Discuss and give feedback


Next question

Upper gastrointestinal bleeding

Patients may present with the following:

 Haematemesis and/ or malaena


 Epigastric discomfort
 Sudden collapse

The extent to which these will occur will depend upon the source. Mortality is higher in patients
presenting with haematemesis than malaena alone.

Oesophageal bleeding
Cause Presenting features

Oesophagitis Small volume of fresh blood, often streaking vomit. Malaena rare. Often ceases
spontaneously. Usually history of antecedent GORD type symptoms.

Cancer Usually small volume of blood, except as pre terminal event with erosion of
major vessels. Often associated symptoms of dysphagia and constitutional
symptoms such as weight loss. May be recurrent until malignancy managed.

Mallory Weiss Typically brisk small to moderate volume of bright red blood following bout of
Tear repeated vomiting. Malaena rare. Usually ceases spontaneously.

Varices Usually large volume of fresh blood. Swallowed blood may cause malaena.
Often associated with haemodynamic compromise. May stop spontaneously but
re-bleeds are common until appropriately managed.

Gastric Bleeding
Cause Presenting features

Gastric May be frank haematemesis or altered blood mixed with vomit. Usually
cancer prodromal features of dyspepsia and may have constitutional symptoms. Amount
of bleeding variable but erosion of major vessel may produce considerable
haemorrhage.

Dieulafoy Often no prodromal features prior to haematemesis and malaena, but this
Lesion arteriovenous malformation may produce quite considerable haemorrhage and
may be difficult to detect endoscopically.

Diffuse Usually haematemesis and epigastric discomfort. Usually there is an underlying


erosive cause such as recent NSAID usage. Large volume haemorrhage may occur with
gastritis considerable haemodynamic compromise.

Gastric ulcer Small low volume bleeds more common so would tend to present as iron
deficiency anaemia. Erosion into a significant vessel may produce considerable
haemorrhage and haematemesis.

Duodenum
Most common cause of major haemorrhage is a posteriorly sited duodenal ulcer. However, ulcers at
any site in the duodenum may present with haematemesis, malaena and epigastric discomfort. The
pain of duodenal ulcer is slightly different to that of gastric ulcers and often occurs several hours
after eating. Peri ampullary tumours may bleed but these are rare. In patients with previous
abdominal aortic aneurysm surgery aorto-enteric fistulation remains a rare but important cause of
major haemorrhage associated with high mortality.

Management

 Admission to hospital careful monitoring, cross match blood, check FBC, LFTs, U+E and
Clotting (as a minimum)
 Patients with on-going bleeding and haemodynamic instability are likely to require O negative
blood pending cross matched blood
 Early control of airway is vital (e.g. Drowsy patient with liver failure)
 Patients with suspected varices should receive terlipressin prior to endoscopy
 Ideally all patients admitted with upper gastrointestinal haemorrhage should undergo Upper
GI endoscopy within 24 hours of admission. In those who are unstable this should occur
immediately after resuscitation or in tandem with it. The endoscopy department is a
potentially dangerous place for unstable patients and it may be safer to perform the
endoscopy in theatre with an anaesthetist present.
 Varices should be banded or subjected to sclerotherapy. If this is not possible owing to active
bleeding then a Sengaksten- Blakemore tube (or Minnesota tube) should be inserted. This
should be done with care; gastric balloon should be inflated first and oesophageal balloon
second. Remember the balloon will need deflating after 12 hours (ideally sooner) to prevent
necrosis. Portal pressure should be lowered by combination of medical therapy +/- TIPSS.
 Patients with erosive oesophagitis / gastritis should receive a proton pump inhibitor.
 Mallory Weiss tears will typically resolve spontaneously
 Identifiable bleeding points should receive combination therapy of injection of adrenaline and
either a thermal or mechanical treatment. All who have received intervention should receive
a continuous infusion of a proton pump inhibitor (IV omeprazole for 72 hours) to reduce the
re-bleeding rate.
 Patients with diffuse erosive gastritis who cannot be managed endoscopically and continue
to bleed may require gastrectomy
 Bleeding ulcers that cannot be controlled endoscopically may require laparotomy and ulcer
underruning

Indications for surgery

 Patients > 60 years


 Continued bleeding despite endoscopic intervention
 Recurrent bleeding
 Known cardiovascular disease with poor response to hypotension

Surgery
Duodenal ulcer
Laparotomy, duodenotomy and under running of the ulcer. If bleeding is brisk then the ulcer is
almost always posteriorly sited and will have invaded the gastroduodenal artery. Large bites using 0
Vicryl are taken above and below the ulcer base to occlude the vessel. The duodenotomy should be
longitudinal but closed transversely to avoid stenosis.

For gastric ulcer


Under-running of the bleeding site
Partial gastrectomy-antral ulcer
Partial gastrectomy or under running the ulcer- lesser curve ulcer (involving left gastric artery)
Total gastrectomy if bleeding persists

Summary of Acute Upper GI bleeding recommendations:


The need for admission and timing of endoscopic intervention may be predicted by using the
Blatchford score. This considers a patients Hb, serum urea, pulse rate and blood pressure. Those
patients with a score of 0 are low risk, all others are considered high risk and require admission and
endoscopy.
The requirement for pre endoscopic proton pump inhibition is contentious. In the UK the National
Institute of Clinical Excellence guidelines suggest the pre endoscopic PPI therapy is unnecessary.
Whilst it is accepted that such treatment has no impact on mortality or morbidity a Cochrane review
of this practice in 2007 did suggest that it reduced the stigmata of recent haemorrhage at
endoscopy. As a result many will still administer PPI to patients prior to endoscopic intervention.
Following endoscopy it is important to calculate the Rockall score for patients to determine their risk
of rebleeding and mortality. A score of 3 or less is associated with a rebleeding rate of 4% and a
very low risk of mortality and identifies a group of patients suitable for early discharge.

References
1. http://www.sign.ac.uk/guidelines/fulltext/105/index.html
2. Joint Advisory Group on Endoscopy (JAG) Guidelines - http://www.thejag.org.uk
3. NICE Guideline: Management of acute upper GI bleeding. July 2012.
Next question
Theme: Gastro intestinal haemorrhage.

A. Proctoscopy and injection sclerotherapy


B. IV terlipressin
C. Pan Proctocolectomy
D. Sub total colectomy
E. Colonscopy and bleeding therapy
F. Barium enema
G. Angiography of mesenteric artery

Please select the most appropriate management option for the scenario given. Each option may be
used once more than once or not at all

32. A 56 year old man is admitted with passage of a large volume of blood per rectum. On
examination he is tachycardic, his abdomen is soft, although he has marked dilated veins
on his abdominal wall. Proctoscopy reveals large dilated veins with stigmata of recent
haemorrhage.

You answered Proctoscopy and injection sclerotherapy

The correct answer is IV terlipressin

Rectal varices are a recognised complication of portal hypertension. In the first instance
they can be managed with medical therapy to lower pressure in the portal venous system.
TIPSS may be considered. Whilst band ligation is an option, attempting to inject these in
same way as haemorroids would carry a high risk of precipitating further haemorrhage.

33. A 73 year old lady is admitted with dark red PR bleeding. She undergoes an OGD which is
normal. Digital rectal examination shows blood but no masses. She becomes tachycardic
and BP is 95/40.

You answered Proctoscopy and injection sclerotherapy

The correct answer is Angiography of mesenteric artery

This women is actively bleeding and mesenteric angiography may localise the bleeding.
Colonoscopy in this situation is seldom helpful or successful.

34. A 68 year old man with ulcerative colitis is admitted with an exacerbation. You are called
to see him because he is having brisk dark PR bleeding. He has been on intravenous
hydrocortisone for 5 days. The gastroenterologists have done an OGD to exclude a
duodenal ulcer, this was normal.
You answered Proctoscopy and injection sclerotherapy

The correct answer is Sub total colectomy

This man requires surgery to remove the bleeding segment of bowel. Medical management
has failed here. Note that a pan proctocolectomy is not a suitable option in the emergency
setting because there is increased morbidity from the pelvic dissection. In the unlikely
event that a sub total colectomy did not address the bleeding then consideration may have
to be given to removal of the rectum but this would not usually be the case.

Please rate this question:

Discuss and give feedback


Next question

Lower Gastrointestinal bleeding

Colonic bleeding
This typically presents as bright red or dark red blood per rectum. Colonic bleeding rarely presents
as malaena type stool, this is because blood in the colon has a powerful laxative effect and is rarely
retained long enough for transformation to occur and because the digestive enzymes present in the
small bowel are not present in the colon. Up to 15% of patients presenting with haemochezia will
have an upper gastrointestinal source of haemorrhage.

As a general rule right sided bleeds tend to present with darker coloured blood than left sided
bleeds. Haemorrhoidal bleeding typically presents as bright red rectal bleeding that occurs post
defecation either onto toilet paper or into the toilet pan. It is very unusual for haemorrhoids alone to
cause any degree of haemodynamic compromise.

Causes
Cause Presenting features

Colitis Bleeding may be brisk in advanced cases, diarrhoea is commonly present.


Abdominal x-ray may show featureless colon.

Diverticular Acute diverticulitis often is not complicated by major bleeding and


disease diverticular bleeds often occur sporadically. 75% all will cease spontaneously
within 24-48 hours. Bleeding is often dark and of large volume.

Cancer Colonic cancers often bleed and for many patients this may be the first sign of
the disease. Major bleeding from early lesions is uncommon
Cause Presenting features

Haemorrhoidal Typically bright red bleeding occurring post defecation. Although patients
bleeding may give graphic descriptions bleeding of sufficient volume to cause
haemodynamic compromise is rare.

Angiodysplasia Apart from bleeding, which may be massive, these arteriovenous lesions
cause little in the way of symptoms. The right side of the colon is more
commonly affected.

Management

 Prompt correction of any haemodynamic compromise is required. Unlike upper


gastrointestinal bleeding the first line management is usually supportive. This is because in
the acute setting endoscopy is rarely helpful.
 When haemorrhoidal bleeding is suspected a proctosigmoidoscopy is reasonable as
attempts at full colonoscopy are usually time consuming and often futile.
 In the unstable patient the usual procedure would be an angiogram (either CT or
percutaneous), when these are performed during a period of haemodynamic instability they
may show a bleeding point and may be the only way of identifying a patch of angiodysplasia.
 In others who are more stable the standard procedure would be a colonoscopy in the
elective setting. In patients undergoing angiography attempts can be made to address the
lesion in question such as coiling. Otherwise surgery will be necessary.
 In patients with ulcerative colitis who have significant haemorrhage the standard approach
would be a sub total colectomy, particularly if medical management has already been tried
and is not effective.

Indications for surgery


Patients > 60 years
Continued bleeding despite endoscopic intervention
Recurrent bleeding
Known cardiovascular disease with poor response to hypotension

Surgery
Selective mesenteric embolisation if life threatening bleeding. This is most helpful if conducted
during a period of relative haemodynamic instability. If all haemodynamic parameters are normal
then the bleeding is most likely to have stopped and any angiography normal in appearance. In
many units a CT angiogram will replace selective angiography but the same caveats will apply.

If the source of colonic bleeding is unclear; perform a laparotomy, on table colonic lavage and
following this attempt a resection. A blind sub total colectomy is most unwise, for example bleeding
from an small bowel arterio-venous malformation will not be treated by this manoeuvre.

Summary of Acute Lower GI bleeding recommendations


Consider admission if:
* Over 60 years
* Haemodynamically unstable/profuse PR bleeding
* On aspirin or NSAID
* Significant co morbidity

Management

 All patients should have a history and examination, PR and proctoscopy


 Colonoscopic haemostasis aimed for in post polypectomy or diverticular bleeding

References
http://www.sign.ac.uk/guidelines/fulltext/105/index.html
Next question
Theme: Dysphagia

A. Mallory Weiss Tear

B. Hiatus hernia

C. Squamous cell carcinoma of the oesophagus

D. Adenocarcinoma of the oesophagus

E. Peptic stricture

F. Motility disorder

For each of the following scenarios please select the most likely underlying disorder. Each option
may be used once, more than once or not at all.

35. A 56 year old lady presents with a 6 month history of dysphagia to solids. She has a long history of
retrosternal chest pain that is worse on lying flat and bending forwards. She undergoes an upper
GI endoscopy where a smooth stricture is identified.

You answered Mallory Weiss Tear

The correct answer is Peptic stricture

Theme from April 2012 Exam


A six month history of dysphagia is a relatively long history and makes malignancy less likely. The
lesion should be biopsied for histological confirmation. Long standing oesophagitis may be
complicated by the development of strictures, Barretts oesophagus or both.

36. A 76 year old man presents with a 5 week history of progressive dysphagia. An upper GI
endoscopy is performed and the surgeon notices changes that are compatible with Barretts
oesophagus. The oesophagus is filled with food debris that cannot be cleared and the endoscope
encounters a resistance that cannot be passed.

You answered Mallory Weiss Tear

The correct answer is Adenocarcinoma of the oesophagus


A short history of dysphagia together with food debris and Barretts changes makes
adenocarcinoma the most likely diagnosis.

37. A 22 year old man presents with a 5 month history of episodic retrosternal chest pain together
with episodes of dysphagia to liquids. An upper GI endoscopy is performed and no mucosal
abnormality is seen.

You answered Mallory Weiss Tear

The correct answer is Motility disorder

Dysphagia that is episodic and varies between solids and liquids is more likely to represent a
motility disorder.

Please rate this question:

Discuss and give feedback

Next question

Oesophageal disease

Disorder Features

Mallory-Weiss Tear Usually history of antecedent vomiting. This is then followed by the vomiting of a
small amount of blood. There is usually little in the way of systemic disturbance
or prior symptoms.

Hiatus hernia of Often longstanding history of dyspepsia, patients are often overweight.
gastric cardia Uncomplicated hiatus hernias should not be associated with dysphagia or
haematemesis.
Disorder Features

Oesophageal rupture Complete disruption of the oesophageal wall in absence of pre-existing


pathology. Left postero-lateral oesophageal is commonest site (2-3cm from OG
junction). Suspect in patients with severe chest pain without cardiac diagnosis
and signs suggestive of pneumonia without convincing history, where there is
history of vomiting. Erect CXR shows infiltrate or effusion in 90% of cases(1).

Squamous cell History of progressive dysphagia. Often signs of weight loss. Usually little or no
carcinoma of the history of previous GORD type symptoms.
oesophagus

Adenocarcinoma of Progressive dysphagia, may have previous symptoms of GORD or Barretts


the oesophagus oesophagus.

Peptic stricture Longer history of dysphagia, often not progressive. Usually symptoms of GORD.
Often lack systemic features seen with malignancy

Dysmotility disorder May have dysphagia that is episodic and non progressive. Retrosternal pain may
accompany the episodes.

Diagnosis
Most of the differential diagnoses listed above can be accurately categorised by upper GI endoscopy
(usually most patients). Where this fails to demonstrate a mechanical stricture the use of pH and
manometry studies together with radiological contrast swallows will facilitate the diagnosis.

References
Blencowe N et al. Spontaneous oesophageal rupture. BMJ 2013 (346):38-39.

Next question
A 73 year old lady presents with progressive dysphagia and is diagnosed with oesophageal cancer
and liver metastases, it is located 8cm proximal to the gastro-oesophageal junction. Which of the
following treatment options would be the the most appropriate management?

Insertion of Celestin tube

Insertion of Minnesota tube

Insertion of self expanding metal stent

Photodynamic therapy

Trans hiatal oesphagectomy

Most cases of malignant oesophageal obstruction can be managed by the placement of self
expanding metal stents. The Celestin tube requires a laparotomy and is therefore obsolete. A
resectional procedure would be inappropriate in the presence of liver metastasis. The main contra
indication to metallic stent placement are very proximal tumours as it can be difficult to get proximal
control in this situation and chemotherapy may be more appropriate.
Please rate this question:

Discuss and give feedback


Next question

Treatment of oesophageal cancer

 In general resections are not offered to those patients with distant metastasis, and usually
not to those with N2 disease.
 Local nodal involvement is not in itself a contra indication to resection.
 Surgical resection is the mainstay of treatment.
 Neoadjuvent chemotherapy is given in most cases prior to surgery.
 In situ disease may be managed by endoscopic mucosal resection, although this is still
debated.
 In patients with lower third lesions an Ivor - Lewis type procedure is most commonly
performed. Very distal tumours may be suitable to a transhiatal procedure. Which is an
attractive option as the penetration of two visceral cavities required for an Ivor- Lewis type
procedure increases the morbidity considerably.
 More proximal lesions will require a total oesphagectomy (Mckeown type) with anastomosis
to the cervical oesophagus.
 Patients with unresectable disease may derive benefit from local ablative procedures,
palliative chemotherapy or stent insertion.
Operative details of Ivor- Lewis procedure

 Combined laparotomy and right thoracotomy

Indication

 Lower and middle third oesophageal tumours

Preparation

 Staging with a combination of CT chest abdomen and pelvis- if no metastatic disease


detected then patients will undergo a staging laparoscopy to detect peritoneal disease.
 If both these modalities are negative then patients will finally undergo a PET CT scan to
detect occult metastatic disease. Only in those whom no evidence of advanced disease is
detected will proceed to resection.
 Patients receive a GA, double lumen endotracheal tube to allow for lung deflation, CVP and
arterial monitoring.

Procedure

 A rooftop incision is made to access the stomach and duodenum.

Laparotomy To mobilize the stomach

 The greater omentum is incised away from its attachment to the right gastroepiploic vessels
along the greater curvature of the stomach.
 Then the short gastric vessels are ligated and detached from the greater curvature from the
spleen.
 The lesser omentum is incised, preserving the right gastric artery.
 The retroperitoneal attachments of the duodenum in its second and third portions are
incised, allowing the pylorus to reach the oesophageal hiatus. Some surgeons perform a
pyloroplasty at this point to facilitate gastric emptying.
 The left gastric vessels are then ligated, avoiding any injury to the common hepatic or splenic
arteries. Care must be taken to avoid inadvertently devascularising the liver owing to
variations in anatomy.

Right Thoracotomy Oesophageal resection and oesophagogastric anastomosis

 Through 5th intercostal space


 Dissection performed 10cm above the tumour
 This may involve transection of the azygos vein.
 The oesophagus is then removed with the stomach creating a gastric tube.
 An anastomosis is created.

The chest is closed with underwater seal drainage and tube drains to the abdominal cavity.

Post operatively

 Patients will typically recover in ITU initially.


 A nasogastric tube will have been inserted intraoperatively and must remain in place during
the early phases of recovery.
 Post operatively these patients are at relatively high risk of developing complications:

* Atelectasis- due to the effects of thoracotomy and lung collapse


* Anastomotic leakage. The risk is relatively high owing to the presence of a relatively
devascularised stomach. Often the only blood supply is from the gastroepiploic artery as all others
will have been divided. If a leak does occur then many will attempt to manage conservatively with
prolonged nasogastric tube drainage and TPN. The reality is that up to 50% of patients developing
an anastomotic leak will not survive to discharge.
* Delayed gastric emptying (may be avoided by performing a pyloroplasty).
Next question
Which of the following statements relating to gastric banding for obesity is false?

It is one of the safest anti obesity operations

If successful up to 55% of excess weight may be lost over 2 years

Excessively tight gastric bands have increased risk of long term complications

It is associated with early satiety

It is contra indicated in patients with polycystic ovaries who are trying to conceive

Adjustable gastric bands are one of the most widely performed anti obesity procedures in the UK.
They are relatively easy to insert. Weight loss is slightly slower than with some of the other weight
loss procedures. Up to 15% patients may require revisional surgery.
Please rate this question:

Discuss and give feedback


Next question

Bariatric surgery

Obesity is a major health problem in the Western world. Surgical solutions to the problem have
evolved dramatically over the past few years. Randomised controlled trials have shown that dramatic
weight loss can be achieved following surgical interventions compared with standard medical
therapy. The weight loss process is also more durable following surgery than with non surgical
interventions.

Case selection
BMI >/= 40 kg/m 2 or between 35-40 kg/m 2 and other significant disease (for example, type 2
diabetes, hypertension) that could be improved with weight loss.

Pre-requisites to surgery (NICE UK Guidelines)

 All non-surgical measures have failed to achieve or maintain adequate clinically beneficial
weight loss for at least 6 months.
 Will receive intensive specialist management
 They are generally fit for anaesthesia and surgery
 They commit to the need for long-term follow-up
 First-line option for adults with a BMI > 50 kg/m 2 in whom surgical intervention is considered
appropriate; consider orlistat if there is a long waiting list.
Surgical options
Adjustable gastric  Laparoscopic placement of adjustable band around proximal
band stomach.
 Contains an adjustable filling port
 Effective method for lifestyle control
 Reversible
 Takes longer to achieve target weight
 Complications such as band erosion (rare), slippage or loss of
efficacy may require re-intervention

Gastric bypass  Combines changes to reservoir size with malabsorptive procedure


for more enduring weight loss.
 Technically more challenging
 Risks related to anastomoses (2% leak rate)
 Irreversible
 Up to 50% may become B12 deficient

Sleeve gastrectomy  Resection of stomach using stapling devices


 Less popular now as initial promising results not sustained

References
1. National Institute for Health and Clinical Excellence. Obesity: the prevention, identification,
assessment and management of overweight and obesity in adults and children.
www.nice.org.uk/CG43

2. Leff DR, Heath D. Surgery for obesity in adulthood. BMJ 2009;339:b3402


Next question
Which of the following procedures is not performed for obesity?

Sleeve gastrectomy

Gastric band

Intra gastric balloon

Mckeown procedure

Small bowel bypass

A McKeown procedure is a total oesophagectomy.


Please rate this question:

Discuss and give feedback


Next question

Bariatric surgery

Obesity is a major health problem in the Western world. Surgical solutions to the problem have
evolved dramatically over the past few years. Randomised controlled trials have shown that dramatic
weight loss can be achieved following surgical interventions compared with standard medical
therapy. The weight loss process is also more durable following surgery than with non surgical
interventions.

Case selection
BMI >/= 40 kg/m 2 or between 35-40 kg/m 2 and other significant disease (for example, type 2
diabetes, hypertension) that could be improved with weight loss.

Pre-requisites to surgery (NICE UK Guidelines)

 All non-surgical measures have failed to achieve or maintain adequate clinically beneficial
weight loss for at least 6 months.
 Will receive intensive specialist management
 They are generally fit for anaesthesia and surgery
 They commit to the need for long-term follow-up
 First-line option for adults with a BMI > 50 kg/m 2 in whom surgical intervention is considered
appropriate; consider orlistat if there is a long waiting list.
Surgical options
Adjustable gastric  Laparoscopic placement of adjustable band around proximal
band stomach.
 Contains an adjustable filling port
 Effective method for lifestyle control
 Reversible
 Takes longer to achieve target weight
 Complications such as band erosion (rare), slippage or loss of
efficacy may require re-intervention

Gastric bypass  Combines changes to reservoir size with malabsorptive procedure


for more enduring weight loss.
 Technically more challenging
 Risks related to anastomoses (2% leak rate)
 Irreversible
 Up to 50% may become B12 deficient

Sleeve gastrectomy  Resection of stomach using stapling devices


 Less popular now as initial promising results not sustained

References
1. National Institute for Health and Clinical Excellence. Obesity: the prevention, identification,
assessment and management of overweight and obesity in adults and children.
www.nice.org.uk/CG43

2. Leff DR, Heath D. Surgery for obesity in adulthood. BMJ 2009;339:b3402


Next question
Which of the following variables is not included in the Rockall score?

Congestive cardiac failure

Liver failure

Systolic blood pressure < 100mmHg

Aspirin usage

Age

A patients should have their Rockall score calculated following endoscopy for upper GI haemorrhage

Mnemonic for Rockall score


ABCDE
A: Age
B: Blood pressure drop (Shock)
C: Co-morbidity
D: Diagnosis
E: Evidence of bleeding

Rockall Score

Applies to upper gastrointestinal bleeding

Variable Score 0 Score 1 Score 2 Score 3

Age <60 60-79 >80

Shock None Pulse Hypotension (systolic


>100 <100mmHg)

Nil or Major organ disease e.g. Renal or liver failure,


Co-Morbidity minimal IHD, CCF metastatic cancer
Variable Score 0 Score 1 Score 2 Score 3

Diagnosis Mallory- All GI Cancer


Weiss

Evidence of None Clot, Blood, spurting


Bleeding vessel

Score <3 = Good prognosis (mortality approx. 2%)

Score >8= High mortality (Mortality approx. 40%)


Please rate this question:

Discuss and give feedback


Next question

Upper gastrointestinal bleeding

Patients may present with the following:

 Haematemesis and/ or malaena


 Epigastric discomfort
 Sudden collapse

The extent to which these will occur will depend upon the source. Mortality is higher in patients
presenting with haematemesis than malaena alone.

Oesophageal bleeding
Cause Presenting features

Oesophagitis Small volume of fresh blood, often streaking vomit. Malaena rare. Often ceases
spontaneously. Usually history of antecedent GORD type symptoms.

Cancer Usually small volume of blood, except as pre terminal event with erosion of
major vessels. Often associated symptoms of dysphagia and constitutional
symptoms such as weight loss. May be recurrent until malignancy managed.
Mallory Weiss Typically brisk small to moderate volume of bright red blood following bout of
Tear repeated vomiting. Malaena rare. Usually ceases spontaneously.

Varices Usually large volume of fresh blood. Swallowed blood may cause malaena.
Often associated with haemodynamic compromise. May stop spontaneously but
re-bleeds are common until appropriately managed.

Gastric Bleeding
Cause Presenting features

Gastric May be frank haematemesis or altered blood mixed with vomit. Usually
cancer prodromal features of dyspepsia and may have constitutional symptoms. Amount
of bleeding variable but erosion of major vessel may produce considerable
haemorrhage.

Dieulafoy Often no prodromal features prior to haematemesis and malaena, but this
Lesion arteriovenous malformation may produce quite considerable haemorrhage and
may be difficult to detect endoscopically.

Diffuse Usually haematemesis and epigastric discomfort. Usually there is an underlying


erosive cause such as recent NSAID usage. Large volume haemorrhage may occur with
gastritis considerable haemodynamic compromise.

Gastric ulcer Small low volume bleeds more common so would tend to present as iron
deficiency anaemia. Erosion into a significant vessel may produce considerable
haemorrhage and haematemesis.

Duodenum
Most common cause of major haemorrhage is a posteriorly sited duodenal ulcer. However, ulcers at
any site in the duodenum may present with haematemesis, malaena and epigastric discomfort. The
pain of duodenal ulcer is slightly different to that of gastric ulcers and often occurs several hours
after eating. Peri ampullary tumours may bleed but these are rare. In patients with previous
abdominal aortic aneurysm surgery aorto-enteric fistulation remains a rare but important cause of
major haemorrhage associated with high mortality.

Management
 Admission to hospital careful monitoring, cross match blood, check FBC, LFTs, U+E and
Clotting (as a minimum)
 Patients with on-going bleeding and haemodynamic instability are likely to require O negative
blood pending cross matched blood
 Early control of airway is vital (e.g. Drowsy patient with liver failure)
 Patients with suspected varices should receive terlipressin prior to endoscopy
 Ideally all patients admitted with upper gastrointestinal haemorrhage should undergo Upper
GI endoscopy within 24 hours of admission. In those who are unstable this should occur
immediately after resuscitation or in tandem with it. The endoscopy department is a
potentially dangerous place for unstable patients and it may be safer to perform the
endoscopy in theatre with an anaesthetist present.
 Varices should be banded or subjected to sclerotherapy. If this is not possible owing to active
bleeding then a Sengaksten- Blakemore tube (or Minnesota tube) should be inserted. This
should be done with care; gastric balloon should be inflated first and oesophageal balloon
second. Remember the balloon will need deflating after 12 hours (ideally sooner) to prevent
necrosis. Portal pressure should be lowered by combination of medical therapy +/- TIPSS.
 Patients with erosive oesophagitis / gastritis should receive a proton pump inhibitor.
 Mallory Weiss tears will typically resolve spontaneously
 Identifiable bleeding points should receive combination therapy of injection of adrenaline and
either a thermal or mechanical treatment. All who have received intervention should receive
a continuous infusion of a proton pump inhibitor (IV omeprazole for 72 hours) to reduce the
re-bleeding rate.
 Patients with diffuse erosive gastritis who cannot be managed endoscopically and continue
to bleed may require gastrectomy
 Bleeding ulcers that cannot be controlled endoscopically may require laparotomy and ulcer
underruning

Indications for surgery

 Patients > 60 years


 Continued bleeding despite endoscopic intervention
 Recurrent bleeding
 Known cardiovascular disease with poor response to hypotension

Surgery
Duodenal ulcer
Laparotomy, duodenotomy and under running of the ulcer. If bleeding is brisk then the ulcer is
almost always posteriorly sited and will have invaded the gastroduodenal artery. Large bites using 0
Vicryl are taken above and below the ulcer base to occlude the vessel. The duodenotomy should be
longitudinal but closed transversely to avoid stenosis.

For gastric ulcer


Under-running of the bleeding site
Partial gastrectomy-antral ulcer
Partial gastrectomy or under running the ulcer- lesser curve ulcer (involving left gastric artery)
Total gastrectomy if bleeding persists

Summary of Acute Upper GI bleeding recommendations:


The need for admission and timing of endoscopic intervention may be predicted by using the
Blatchford score. This considers a patients Hb, serum urea, pulse rate and blood pressure. Those
patients with a score of 0 are low risk, all others are considered high risk and require admission and
endoscopy.
The requirement for pre endoscopic proton pump inhibition is contentious. In the UK the National
Institute of Clinical Excellence guidelines suggest the pre endoscopic PPI therapy is unnecessary.
Whilst it is accepted that such treatment has no impact on mortality or morbidity a Cochrane review
of this practice in 2007 did suggest that it reduced the stigmata of recent haemorrhage at
endoscopy. As a result many will still administer PPI to patients prior to endoscopic intervention.
Following endoscopy it is important to calculate the Rockall score for patients to determine their risk
of rebleeding and mortality. A score of 3 or less is associated with a rebleeding rate of 4% and a
very low risk of mortality and identifies a group of patients suitable for early discharge.

References
1. http://www.sign.ac.uk/guidelines/fulltext/105/index.html
2. Joint Advisory Group on Endoscopy (JAG) Guidelines - http://www.thejag.org.uk
3. NICE Guideline: Management of acute upper GI bleeding. July 2012.
Next question
Which of the following criteria is not an indication for bariatric surgery to be performed in the UK
National Institute of Clinical Excellence Guidelines?

Patients must be non smoking for at least one year

Patients must have tried conservative management for at least 6 months

Commitment to long-term follow up

Surgery to be performed in a specialist unit

BMI > 40 kg/m2

Being a non smoker is not included in the criteria, however poor respiratory function due to smoking
may affect fitness for surgery.
Please rate this question:

Discuss and give feedback


Next question

Bariatric surgery

Obesity is a major health problem in the Western world. Surgical solutions to the problem have
evolved dramatically over the past few years. Randomised controlled trials have shown that dramatic
weight loss can be achieved following surgical interventions compared with standard medical
therapy. The weight loss process is also more durable following surgery than with non surgical
interventions.

Case selection
BMI >/= 40 kg/m 2 or between 35-40 kg/m 2 and other significant disease (for example, type 2
diabetes, hypertension) that could be improved with weight loss.

Pre-requisites to surgery (NICE UK Guidelines)

 All non-surgical measures have failed to achieve or maintain adequate clinically beneficial
weight loss for at least 6 months.
 Will receive intensive specialist management
 They are generally fit for anaesthesia and surgery
 They commit to the need for long-term follow-up
 First-line option for adults with a BMI > 50 kg/m 2 in whom surgical intervention is considered
appropriate; consider orlistat if there is a long waiting list.
Surgical options
Adjustable gastric  Laparoscopic placement of adjustable band around proximal
band stomach.
 Contains an adjustable filling port
 Effective method for lifestyle control
 Reversible
 Takes longer to achieve target weight
 Complications such as band erosion (rare), slippage or loss of
efficacy may require re-intervention

Gastric bypass  Combines changes to reservoir size with malabsorptive procedure


for more enduring weight loss.
 Technically more challenging
 Risks related to anastomoses (2% leak rate)
 Irreversible
 Up to 50% may become B12 deficient

Sleeve gastrectomy  Resection of stomach using stapling devices


 Less popular now as initial promising results not sustained

References
1. National Institute for Health and Clinical Excellence. Obesity: the prevention, identification,
assessment and management of overweight and obesity in adults and children.
www.nice.org.uk/CG43

2. Leff DR, Heath D. Surgery for obesity in adulthood. BMJ 2009;339:b3402


Next question
A 34-year-old HIV positive man is referred to the surgical out patient department with jaundice and
abnormal liver function tests. Liver function tests are as follows:

Albumin 34 g/l

ALP 540 iu/l

Bilirubin 67 µmol/l

ALT 45 iu/l

What is the most likely diagnosis?

Hepatic abscess

Fungal obstruction of the bile duct

Duodenal adenoma

Primary biliary cirrhosis

Sclerosing cholangitis

The LFTs clearly show a cholestatic picture. Given the background of HIV the most likely cause is
sclerosing cholangitis.

Please rate this question:

Discuss and give feedback

Next question
HIV: biliary and pancreatic disease

The most common cause of biliary disease in patients with HIV is sclerosing cholangitis due to
infections such as CMV, Cryptosporidium and Microsporidia

Pancreatitis in the context of HIV infection may be secondary to anti-retroviral treatment (especially
didanosine) or by opportunistic infections e.g. CMV

Next question
A 42 year old woman with known multiple gastric ulcers attends the surgical out patient unit. She has
not improved despite 2 months of proton pump inhibitor treatment. She is found to have a
gastrinoma. Which of the following is false in relation to her diagnosis?

Most commonly found in the pancreas

Associated with multiple endocrine neoplasia I

Somatostatin sensitive scintigraphy is the most senstive non invasive test for localizing
primary tumours

Primary tumours can occur in the ovary

Secretory diarrhoea is a feature

Most commonly found in the duodenum (in up to 50% patients), then the pancreas (approximately
20%). Other ectopic areas include stomach, spleen, gallbladder and ovary

 Greater than 4/5 of gastrinomas are found within the triangle bounded by:

Cystic and common bile duct (Top)


2nd and 3rd part of the duodenum (Bottom)
Neck and body of pancreas (Medial)

 Pancreatic gastrinomas are normally solitary and highly malignant.


 Zollinger Ellison syndrome is composed of the triad of:

1. Non beta islet cell tumours of the pancreas


2. Hypergastrinaemia
3. Severe ulcer disease

Clinical features related to peptic ulcer disease. Diagnosis is based on 3 criteria:


1. Fasting hypergastrinaemia
2. Increased basal acid output
3. Secretin stimulation test positive

Management

 Resection if localised disease

Please rate this question:


Discuss and give feedback
Next question

Pancreatic cancer

 Adenocarcinoma
 Risk factors: Smoking, diabetes, adenoma, familial adenomatous polyposis
 Mainly occur in the head of the pancreas (70%)
 Spread locally and metastasizes to the liver
 Carcinoma of the pancreas should be differentiated from other periampullary tumours with
better prognosis

Clinical features

 Weight loss
 Painless jaundice
 Epigastric discomfort (pain usually due to invasion of the coeliac plexus is a late feature)
 Pancreatitis
 Trousseau's sign: migratory superficial thrombophlebitis

Investigations

 USS: May miss small lesions


 CT Scanning (pancreatic protocol). If unresectable on CT then no further staging needed
 PET/CT for those with operable disease on CT alone
 ERCP/ MRI for bile duct assessment
 Staging laparoscopy to exclude peritoneal disease

Management

 Head of pancreas: Whipple's resection (SE dumping and ulcers). Newer techniques include
pylorus preservation and SMA/ SMV resection
 Carcinoma body and tail: poor prognosis, distal pancreatectomy, if operable
 Usually adjuvent chemotherapy for resectable disease
 ERCP and stent for jaundice and palliation
 Surgical bypass may be needed for duodenal obstruction

Next question
A 45 year old man is admitted with haematemesis. An upper gastrointestinal endoscopy is
performed. A large ulcer in the first part of the duodenum is noted. Attempts are made to
endoscopically clip and inject the ulcer which is bleeding profusely. These efforts are unsuccessful.
What is the most appropriate management option?

Therapeutic angiogram

Diagnostic angiography

Laparotomy and underrunning of the ulcer

Laparotomy and distal gastrectomy

Duodenal resection and gastro jejunostomy

The standard surgical option for bleeding peptic ulcers is to underrun them. Resectional surgery is
very much the option of last resort and is seldom helpful or easy. An isolated duodenal resection
would almost never be performed.
Please rate this question:

Discuss and give feedback


Next question

Upper gastrointestinal bleeding

Patients may present with the following:

 Haematemesis and/ or malaena


 Epigastric discomfort
 Sudden collapse

The extent to which these will occur will depend upon the source. Mortality is higher in patients
presenting with haematemesis than malaena alone.

Oesophageal bleeding
Cause Presenting features
Oesophagitis Small volume of fresh blood, often streaking vomit. Malaena rare. Often ceases
spontaneously. Usually history of antecedent GORD type symptoms.

Cancer Usually small volume of blood, except as pre terminal event with erosion of
major vessels. Often associated symptoms of dysphagia and constitutional
symptoms such as weight loss. May be recurrent until malignancy managed.

Mallory Weiss Typically brisk small to moderate volume of bright red blood following bout of
Tear repeated vomiting. Malaena rare. Usually ceases spontaneously.

Varices Usually large volume of fresh blood. Swallowed blood may cause malaena.
Often associated with haemodynamic compromise. May stop spontaneously but
re-bleeds are common until appropriately managed.

Gastric Bleeding
Cause Presenting features

Gastric May be frank haematemesis or altered blood mixed with vomit. Usually
cancer prodromal features of dyspepsia and may have constitutional symptoms. Amount
of bleeding variable but erosion of major vessel may produce considerable
haemorrhage.

Dieulafoy Often no prodromal features prior to haematemesis and malaena, but this
Lesion arteriovenous malformation may produce quite considerable haemorrhage and
may be difficult to detect endoscopically.

Diffuse Usually haematemesis and epigastric discomfort. Usually there is an underlying


erosive cause such as recent NSAID usage. Large volume haemorrhage may occur with
gastritis considerable haemodynamic compromise.

Gastric ulcer Small low volume bleeds more common so would tend to present as iron
deficiency anaemia. Erosion into a significant vessel may produce considerable
haemorrhage and haematemesis.

Duodenum
Most common cause of major haemorrhage is a posteriorly sited duodenal ulcer. However, ulcers at
any site in the duodenum may present with haematemesis, malaena and epigastric discomfort. The
pain of duodenal ulcer is slightly different to that of gastric ulcers and often occurs several hours
after eating. Peri ampullary tumours may bleed but these are rare. In patients with previous
abdominal aortic aneurysm surgery aorto-enteric fistulation remains a rare but important cause of
major haemorrhage associated with high mortality.

Management

 Admission to hospital careful monitoring, cross match blood, check FBC, LFTs, U+E and
Clotting (as a minimum)
 Patients with on-going bleeding and haemodynamic instability are likely to require O negative
blood pending cross matched blood
 Early control of airway is vital (e.g. Drowsy patient with liver failure)
 Patients with suspected varices should receive terlipressin prior to endoscopy
 Ideally all patients admitted with upper gastrointestinal haemorrhage should undergo Upper
GI endoscopy within 24 hours of admission. In those who are unstable this should occur
immediately after resuscitation or in tandem with it. The endoscopy department is a
potentially dangerous place for unstable patients and it may be safer to perform the
endoscopy in theatre with an anaesthetist present.
 Varices should be banded or subjected to sclerotherapy. If this is not possible owing to active
bleeding then a Sengaksten- Blakemore tube (or Minnesota tube) should be inserted. This
should be done with care; gastric balloon should be inflated first and oesophageal balloon
second. Remember the balloon will need deflating after 12 hours (ideally sooner) to prevent
necrosis. Portal pressure should be lowered by combination of medical therapy +/- TIPSS.
 Patients with erosive oesophagitis / gastritis should receive a proton pump inhibitor.
 Mallory Weiss tears will typically resolve spontaneously
 Identifiable bleeding points should receive combination therapy of injection of adrenaline and
either a thermal or mechanical treatment. All who have received intervention should receive
a continuous infusion of a proton pump inhibitor (IV omeprazole for 72 hours) to reduce the
re-bleeding rate.
 Patients with diffuse erosive gastritis who cannot be managed endoscopically and continue
to bleed may require gastrectomy
 Bleeding ulcers that cannot be controlled endoscopically may require laparotomy and ulcer
underruning

Indications for surgery

 Patients > 60 years


 Continued bleeding despite endoscopic intervention
 Recurrent bleeding
 Known cardiovascular disease with poor response to hypotension

Surgery
Duodenal ulcer
Laparotomy, duodenotomy and under running of the ulcer. If bleeding is brisk then the ulcer is
almost always posteriorly sited and will have invaded the gastroduodenal artery. Large bites using 0
Vicryl are taken above and below the ulcer base to occlude the vessel. The duodenotomy should be
longitudinal but closed transversely to avoid stenosis.
For gastric ulcer
Under-running of the bleeding site
Partial gastrectomy-antral ulcer
Partial gastrectomy or under running the ulcer- lesser curve ulcer (involving left gastric artery)
Total gastrectomy if bleeding persists

Summary of Acute Upper GI bleeding recommendations:


The need for admission and timing of endoscopic intervention may be predicted by using the
Blatchford score. This considers a patients Hb, serum urea, pulse rate and blood pressure. Those
patients with a score of 0 are low risk, all others are considered high risk and require admission and
endoscopy.
The requirement for pre endoscopic proton pump inhibition is contentious. In the UK the National
Institute of Clinical Excellence guidelines suggest the pre endoscopic PPI therapy is unnecessary.
Whilst it is accepted that such treatment has no impact on mortality or morbidity a Cochrane review
of this practice in 2007 did suggest that it reduced the stigmata of recent haemorrhage at
endoscopy. As a result many will still administer PPI to patients prior to endoscopic intervention.
Following endoscopy it is important to calculate the Rockall score for patients to determine their risk
of rebleeding and mortality. A score of 3 or less is associated with a rebleeding rate of 4% and a
very low risk of mortality and identifies a group of patients suitable for early discharge.

References
1. http://www.sign.ac.uk/guidelines/fulltext/105/index.html
2. Joint Advisory Group on Endoscopy (JAG) Guidelines - http://www.thejag.org.uk
3. NICE Guideline: Management of acute upper GI bleeding. July 2012.
Next question
Which of the procedures listed below is most strongly associated with delayed gastric emptying?

Posterior gastrojejunostomy

Pyloromyotomy

Pyloroplasty

Anterior gastrojejunostomy

Roux en Y gastrojejunostomy

Anterior gastrojejunostomy is one of the easiest gastric bypass procedures to perform and is still
often used for reconstruction following distal gastrectomy. It is associated with impairment of gastric
emptying and patients may have considerable problems with flatulent dyspepsia. Roux en Y
methods provide the best function. Pyloroplasty enhances gastric emptying (and was historically
performed following vagotomy for this reason). Pyloromyotomy is not associated with an increase in
gastric transit times (though failed procedures may be).
Please rate this question:

Discuss and give feedback


Next question

Gastric emptying

 The stomach serves both a mechanical and immunological function. Solid and liquid are
retained in the stomach during which time repeated peristaltic activity against a closed
pyloric sphincter will cause fragmentation of food bolus material. Contact with gastric acid will
help to neutralise any pathogens present.
 The amount of time material spends in the stomach is related to its composition and volume.
For example a glass of water will empty more quickly than a large meal. The presence of
amino acids and fat will all serve to delay gastric emptying.

Controlling factors
Neuronal stimulation of the stomach is mediated via the vagus and the parasympathetic nervous
system will tend to favor an increase in gastric motility. It is for this reason that individuals who have
undergone truncal vagotomy will tend to routinely require either a pyloroplasty or gastro-enterostomy
as they would otherwise have delayed gastric emptying.

The following hormonal factors are all involved:


Delay emptying Increase emptying

Gastric inhibitory peptide Gastrin

Cholecystokinin

Enteroglucagon

Diseases affecting gastric emptying


All diseases that affect gastric emptying may result in bacterial overgrowth, retained food and
eventually the formation of bezoars that may occlude the pylorus and make gastric emptying even
worse. Fermentation of food may cause dyspepsia, reflux and foul smelling belches of gas.

Iatrogenic
Gastric surgery can have profound effects on gastric emptying. As stated above any procedure that
disrupts the vagus can cause delayed emptying. Whilst this is particularly true of vagotomy, this
operation is now rarely performed. Surgeons are divided on the importance of vagal disruption that
occurs during an oesophagectomy, some will routinely perform a pyloroplasty and others will not.

When a distal gastrectomy is performed, the type of anastomosis performed will impact on emptying.
When a gastro-enterostomy is constructed, a posterior, retrocolic gastroenterostomy will empty
better than an anterior one.

Diabetic gastroparesis
This is predominantly due to neuropathy affecting the vagus nerve. The stomach empties poorly and
patients may have episodes of repeated and protracted vomiting. Diagnosis is made by upper GI
endoscopy and contrast studies, in some cases a radio nucleotide scan is needed to demonstrate
the abnormality more clearly. In treating these conditions, drugs such as metoclopramide will be less
effective as they exert their effect via the vagus nerve. One of the few prokinetic drugs that do not
work in this way is the antibiotic erythromycin.

Malignancies
Obviously a distal gastric cancer may obstruct the pylorus and delay emptying. In addition,
malignancies of the pancreas may cause extrinsic compression of the duodenum and delay
emptying. Treatment in these cases is by gastric decompression using a wide bore nasogastric tube
and insertion of a stent or, if that is not possible, by a surgical gastroenterostomy. As a general rule
gastroenterostomies constructed for bypass of malignancy are usually placed on the anterior wall of
the stomach (in spite of the fact that they empty less well). A Roux en Y bypass may also be
undertaken, but the increased number of anastomoses for this, in malignant disease that is being
palliated, is probably not justified.

Congenital Hypertrophic Pyloric Stenosis


This is typically a disease of infancy. Most babies will present around 6 weeks of age with projectile
non bile stained vomiting. It has an incidence of 2.4 per 1000 live births and is more common in
males. Diagnosis is usually made by careful history and examination and a mass may be palpable in
the epigastrium (often cited seldom felt!). The most important diagnostic test is an ultrasound that
usually demonstrates the hypertrophied pylorus. Blood tests may reveal a hypochloraemic metabolic
alkalosis if the vomiting is long standing. Once the diagnosis is made the infant is resuscitated and a
pyloromyotomy is performed (either open or laparoscopically). Once treated there are no long term
sequelae.
Next question
A 63 year old man undergoes a total gastrectomy for carcinoma of the stomach. Which of the
sequelae below is least likely to occur?

Metabolic bone disease

Bile reflux

Dumping syndrome

Zinc deficiency

B12 deficiency

Zinc is mainly absorbed in the duodenum and jejunum. Bile reflux may occur post gastrectomy. The
risk of bile reflux is lower if a Roux en Y reconstruction is used.
Please rate this question:

Discuss and give feedback


Next question

Post gastrectomy syndromes

Post gastrectomy syndromes may vary slightly depending upon whether a total of partial
gastrectomy is performed. A Roux en Y reconstruction generally gives the best functional outcomes.
Where a gastrojejunostomy is performed as reconstruction following a distal gastrectomy the gastric
emptying is generally better if the jejunal limbs are tunneled in the retrocolic plane.

The following may occur following gastrectomy:

 Small capacity (early satiety)


 Dumping syndrome
 Bile gastritis
 Afferent loop syndrome
 Efferent loop syndrome
 Anaemia (B12 deficiency)
 Metabolic bone disease

Next question
A 34 year old women with morbid obesity is referred for consideration of bariatric surgery. Which of
the following options is associated with the highest long term failure rates?

Gastric band

Intra gastric balloon

Roux en Y bypass

Sleeve gastrectomy

Duodenal switch

Intragastric balloon is really only suitable as a bridge to a more definitive surgical solution.

Bariatric surgery: the main operations

Gastric banding: band applied to upper stomach which can be inflated or deflated with normal
saline. This affects satiety. Over a 5 year period complications requiring further surgery occur in up
to 15% cases.

Roux-en-Y gastric bypass: a gastric pouch is formed and connected to the jejunum. Patients
achieve greater and more longterm weight loss than gastric banding.

Sleeve gastrectomy: body and fundus resected to leave a small section of stomach

Biliopancreatic diversion +/- duodenal switch: bypass the small bowel. Greatest weight loss but a
very complex procedure associated with malnutrition and diarrhoea.

Vertical banded gastroplasty (stomach stapling): rarely performed due to longterm failure rate.
Please rate this question:

Discuss and give feedback


Next question

Bariatric surgery

Obesity is a major health problem in the Western world. Surgical solutions to the problem have
evolved dramatically over the past few years. Randomised controlled trials have shown that dramatic
weight loss can be achieved following surgical interventions compared with standard medical
therapy. The weight loss process is also more durable following surgery than with non surgical
interventions.
Case selection
BMI >/= 40 kg/m 2 or between 35-40 kg/m 2 and other significant disease (for example, type 2
diabetes, hypertension) that could be improved with weight loss.

Pre-requisites to surgery (NICE UK Guidelines)

 All non-surgical measures have failed to achieve or maintain adequate clinically beneficial
weight loss for at least 6 months.
 Will receive intensive specialist management
 They are generally fit for anaesthesia and surgery
 They commit to the need for long-term follow-up
 First-line option for adults with a BMI > 50 kg/m 2 in whom surgical intervention is considered
appropriate; consider orlistat if there is a long waiting list.

Surgical options
Adjustable gastric  Laparoscopic placement of adjustable band around proximal
band stomach.
 Contains an adjustable filling port
 Effective method for lifestyle control
 Reversible
 Takes longer to achieve target weight
 Complications such as band erosion (rare), slippage or loss of
efficacy may require re-intervention

Gastric bypass  Combines changes to reservoir size with malabsorptive procedure


for more enduring weight loss.
 Technically more challenging
 Risks related to anastomoses (2% leak rate)
 Irreversible
 Up to 50% may become B12 deficient

Sleeve gastrectomy  Resection of stomach using stapling devices


 Less popular now as initial promising results not sustained

References
1. National Institute for Health and Clinical Excellence. Obesity: the prevention, identification,
assessment and management of overweight and obesity in adults and children.
www.nice.org.uk/CG43

2. Leff DR, Heath D. Surgery for obesity in adulthood. BMJ 2009;339:b3402


Next question
Theme: Gastric ulceration

A. Acute peptic ulcer


B. Adenocarcinoma
C. Carcinoid Tumour
D. Gastrointestinal stromal tumour
E. Chronic peptic ulcer
F. Lymphosarcoma
G. Leiomyoma

Please select the most likely cause of gastric ulceration for the scenario given. Each option may be
used once, more than once or not at all.

49. A 53 year old man presents with dyspepsia. At upper GI endoscopy he has a punched out
ulcer on the lesser curve of the stomach. It measures approximately 2cm in diameter and is
seen to penetrate muscle with fibrosis present at the base.

You answered Acute peptic ulcer

The correct answer is Chronic peptic ulcer

Fibrosis is usually a sign of chronic ulcer. It should be biopsied carefully, a proton pump
inhibitor started and re endoscopy should occur at 6 weeks.

50. A 42 year old man presents with epigastric pain. At endoscopy, he is found to have a
punched out ulcer on the anterior wall of the stomach. It is shallow and measures 1cm in
diameter.

Acute peptic ulcer

The absence of fibrosis coupled with small size suggests a more acute ulcer. Management
should include biopsy, PPI and repeat endoscopy at 6 weeks.

51. A 65 year old man presents with epigastric discomfort. At endoscopy he is found to have
an ulcer at the antrum, is has thick rolled edges and measures 3cm in diameter.

You answered Acute peptic ulcer

The correct answer is Adenocarcinoma

Rolled edges and location favor tumour. Careful biopsies should be taken and staging CT
performed.
Please rate this question:

Discuss and give feedback


Next question

Upper gastrointestinal bleeding

Patients may present with the following:

 Haematemesis and/ or malaena


 Epigastric discomfort
 Sudden collapse

The extent to which these will occur will depend upon the source. Mortality is higher in patients
presenting with haematemesis than malaena alone.

Oesophageal bleeding
Cause Presenting features

Oesophagitis Small volume of fresh blood, often streaking vomit. Malaena rare. Often ceases
spontaneously. Usually history of antecedent GORD type symptoms.

Cancer Usually small volume of blood, except as pre terminal event with erosion of
major vessels. Often associated symptoms of dysphagia and constitutional
symptoms such as weight loss. May be recurrent until malignancy managed.

Mallory Weiss Typically brisk small to moderate volume of bright red blood following bout of
Tear repeated vomiting. Malaena rare. Usually ceases spontaneously.

Varices Usually large volume of fresh blood. Swallowed blood may cause malaena.
Often associated with haemodynamic compromise. May stop spontaneously but
re-bleeds are common until appropriately managed.

Gastric Bleeding
Cause Presenting features

Gastric May be frank haematemesis or altered blood mixed with vomit. Usually
cancer prodromal features of dyspepsia and may have constitutional symptoms. Amount
of bleeding variable but erosion of major vessel may produce considerable
haemorrhage.

Dieulafoy Often no prodromal features prior to haematemesis and malaena, but this
Lesion arteriovenous malformation may produce quite considerable haemorrhage and
may be difficult to detect endoscopically.

Diffuse Usually haematemesis and epigastric discomfort. Usually there is an underlying


erosive cause such as recent NSAID usage. Large volume haemorrhage may occur with
gastritis considerable haemodynamic compromise.

Gastric ulcer Small low volume bleeds more common so would tend to present as iron
deficiency anaemia. Erosion into a significant vessel may produce considerable
haemorrhage and haematemesis.

Duodenum
Most common cause of major haemorrhage is a posteriorly sited duodenal ulcer. However, ulcers at
any site in the duodenum may present with haematemesis, malaena and epigastric discomfort. The
pain of duodenal ulcer is slightly different to that of gastric ulcers and often occurs several hours
after eating. Peri ampullary tumours may bleed but these are rare. In patients with previous
abdominal aortic aneurysm surgery aorto-enteric fistulation remains a rare but important cause of
major haemorrhage associated with high mortality.

Management

 Admission to hospital careful monitoring, cross match blood, check FBC, LFTs, U+E and
Clotting (as a minimum)
 Patients with on-going bleeding and haemodynamic instability are likely to require O negative
blood pending cross matched blood
 Early control of airway is vital (e.g. Drowsy patient with liver failure)
 Patients with suspected varices should receive terlipressin prior to endoscopy
 Ideally all patients admitted with upper gastrointestinal haemorrhage should undergo Upper
GI endoscopy within 24 hours of admission. In those who are unstable this should occur
immediately after resuscitation or in tandem with it. The endoscopy department is a
potentially dangerous place for unstable patients and it may be safer to perform the
endoscopy in theatre with an anaesthetist present.
 Varices should be banded or subjected to sclerotherapy. If this is not possible owing to active
bleeding then a Sengaksten- Blakemore tube (or Minnesota tube) should be inserted. This
should be done with care; gastric balloon should be inflated first and oesophageal balloon
second. Remember the balloon will need deflating after 12 hours (ideally sooner) to prevent
necrosis. Portal pressure should be lowered by combination of medical therapy +/- TIPSS.
 Patients with erosive oesophagitis / gastritis should receive a proton pump inhibitor.
 Mallory Weiss tears will typically resolve spontaneously
 Identifiable bleeding points should receive combination therapy of injection of adrenaline and
either a thermal or mechanical treatment. All who have received intervention should receive
a continuous infusion of a proton pump inhibitor (IV omeprazole for 72 hours) to reduce the
re-bleeding rate.
 Patients with diffuse erosive gastritis who cannot be managed endoscopically and continue
to bleed may require gastrectomy
 Bleeding ulcers that cannot be controlled endoscopically may require laparotomy and ulcer
underruning

Indications for surgery

 Patients > 60 years


 Continued bleeding despite endoscopic intervention
 Recurrent bleeding
 Known cardiovascular disease with poor response to hypotension

Surgery
Duodenal ulcer
Laparotomy, duodenotomy and under running of the ulcer. If bleeding is brisk then the ulcer is
almost always posteriorly sited and will have invaded the gastroduodenal artery. Large bites using 0
Vicryl are taken above and below the ulcer base to occlude the vessel. The duodenotomy should be
longitudinal but closed transversely to avoid stenosis.

For gastric ulcer


Under-running of the bleeding site
Partial gastrectomy-antral ulcer
Partial gastrectomy or under running the ulcer- lesser curve ulcer (involving left gastric artery)
Total gastrectomy if bleeding persists

Summary of Acute Upper GI bleeding recommendations:


The need for admission and timing of endoscopic intervention may be predicted by using the
Blatchford score. This considers a patients Hb, serum urea, pulse rate and blood pressure. Those
patients with a score of 0 are low risk, all others are considered high risk and require admission and
endoscopy.
The requirement for pre endoscopic proton pump inhibition is contentious. In the UK the National
Institute of Clinical Excellence guidelines suggest the pre endoscopic PPI therapy is unnecessary.
Whilst it is accepted that such treatment has no impact on mortality or morbidity a Cochrane review
of this practice in 2007 did suggest that it reduced the stigmata of recent haemorrhage at
endoscopy. As a result many will still administer PPI to patients prior to endoscopic intervention.
Following endoscopy it is important to calculate the Rockall score for patients to determine their risk
of rebleeding and mortality. A score of 3 or less is associated with a rebleeding rate of 4% and a
very low risk of mortality and identifies a group of patients suitable for early discharge.

References
1. http://www.sign.ac.uk/guidelines/fulltext/105/index.html
2. Joint Advisory Group on Endoscopy (JAG) Guidelines - http://www.thejag.org.uk
3. NICE Guideline: Management of acute upper GI bleeding. July 2012.
Next question
Theme: Causes of dysphagia

A. Oesophageal cancer
B. Post radiotherapy fibrosis
C. Benign stricture
D. Plummer Vinson syndrome
E. Oesophageal candidiasis
F. Neuropathy
G. Globus

Please select the most likely cause of dysphagia for the scenarios given. Each option may be used
once, more than once or not at all.

52. A cachectic 32 year old man with severe perineal Crohns disease is receiving treatment
with intravenous antibiotics. Over the past 72 hours he has complained of intermittent
dysphagia and odynophagia.

You answered Oesophageal cancer

The correct answer is Oesophageal candidiasis

Treatment with systemic antibiotics may result in development of candidiasis. Patients


may present with odynophagia and episodic dysphagia. Endoscopic appearances are
usually diagnostic and treatment is with an oral anti fungal agent.

53. A 78 year old lady presents 6 years following a successfully treated squamous cell
carcinoma of the oesophagus. She has a long history of dysphagia but it is not progressive.

You answered Oesophageal cancer

The correct answer is Post radiotherapy fibrosis

SCC of the oesophagus is commonly treated with chemoradiotherapy. Fibrosis and


dysphagia may occur in survivors.

54. A 32 year old lady presents with dysphagia. She has a 10 year history of anaemia
secondary to menorrhagia and has been strongly resistant to treatment.

You answered Oesophageal cancer

The correct answer is Plummer Vinson syndrome

Plummer Vinson syndrome (oesophageal web) may occur in association with iron
deficiency anaemia (although rare!).

Please rate this question:

Discuss and give feedback


Next question

Dysphagia

Causes of dysphagia

Extrinsic  Mediastinal masses


 Cervical spondylosis

Oesophageal wall  Achalasia


 Diffuse oesophageal spasm
 Hypertensive lower oesophageal sphincter

Intrinsic  Tumours
 Strictures
 Oesophageal web
 Schatzki rings

Neurological  CVA
 Parkinson's disease
 Multiple Sclerosis
 Brainstem pathology
 Myasthenia Gravis

Investigation
All patients require an upper GI endoscopy unless there are compelling reasons for this not to be
performed. Motility disorders may be best appreciated by undertaking fluoroscopic swallowing
studies.

A full blood count should be performed.

Ambulatory oesophageal pH and manometry studies will be required to evaluate conditions such as
achalasia and patients with GORD being considered for fundoplication surgery.
Next question
Which of the following is false during the pre operative preparation for surgery in pancreatic cancer?

IV antibiotics should be given intra operatively

Endotoxamia can be reduced with lactulose

Subcutaneous heparin should be avoided

Endotoxaemia can be reduced with IV mannitol

There is a higher risk of complications if the bilirubin is greater than 150

Vitamin K should be given to correct abnormal clotting initially, however there is still a risk of
thrombosis so low molecular weight heparin should be used. Bile salts can also be given to reduce
endotoxaemia. Biliary obstruction should be relieved. In the case of biliary obstruction, if a stent is
used it should be a made of plastic. Metallic stents will become embedded and may compromise
attempts at resection.
Please rate this question:

Discuss and give feedback

Pancreatic cancer

 Adenocarcinoma
 Risk factors: Smoking, diabetes, adenoma, familial adenomatous polyposis
 Mainly occur in the head of the pancreas (70%)
 Spread locally and metastasizes to the liver
 Carcinoma of the pancreas should be differentiated from other periampullary tumours with
better prognosis

Clinical features

 Weight loss
 Painless jaundice
 Epigastric discomfort (pain usually due to invasion of the coeliac plexus is a late feature)
 Pancreatitis
 Trousseau's sign: migratory superficial thrombophlebitis
Investigations

 USS: May miss small lesions


 CT Scanning (pancreatic protocol). If unresectable on CT then no further staging needed
 PET/CT for those with operable disease on CT alone
 ERCP/ MRI for bile duct assessment
 Staging laparoscopy to exclude peritoneal disease

Management

 Head of pancreas: Whipple's resection (SE dumping and ulcers). Newer techniques include
pylorus preservation and SMA/ SMV resection
 Carcinoma body and tail: poor prognosis, distal pancreatectomy, if operable
 Usually adjuvent chemotherapy for resectable disease
 ERCP and stent for jaundice and palliation
 Surgical bypass may be needed for duodenal obstruction
A 39 year old man notices a swelling in his left hemiscrotum. On examination he has a left sided
varicocele. The ipsilateral testis is normal on palpation. What is the most appropriate course of
action?

Scrotal exploration and ligation of the varicocele

Abdominal ultrasound

Scrotal ultrasound

Left orchidectomy

Discharge

A left sided varicocele is a recognised presenting sign of a renal tumour occluding the renal vein
(into which the left testicular vein drains). An abdominal ultrasound should be undertaken to exclude
this. Surgery for uncomplicated varicocele is usually unnecessary.
Please rate this question:

Discuss and give feedback


Next question

Renal tumours

Renal cell carcinoma


Renal cell carcinoma is an adenocarcinoma of the renal cortex and is believed to arise from the
proximal convoluted tubule. They are usually solid lesions, up to 20% may be multifocal, 20% may
be calcified and 20% may have either a cystic component or be wholly cystic. They are often
circumscribed by a pseudocapsule of compressed normal renal tissue. Spread may occur either by
direct extension into the adrenal gland, renal vein or surrounding fascia. More distant disease
usually occurs via the haematogenous route to lung, bone or brain.
Renal cell carcinoma comprise up to 85% of all renal malignancies. Males are more commonly
affected than females and sporadic tumours typically affect patients in their sixth decade.
Patients may present with a variety of symptoms including; haematuria (50%), loin pain (40%), mass
(30%) and up to 25% may have symptoms of metastasis.Less than 10% have the classic triad of
haematuria, pain and mass.

Investigation
Many cases will present as haematuria and be discovered during diagnostic work up. Benign renal
tumours are rare, so renal masses should be investigated with multislice CT scanning. Some units
will add and arterial and venous phase to the scan to demonstrate vascularity and evidence of caval
ingrowth.
CT scanning of the chest and abdomen to detect distant disease should also be undertaken.

Routine bone scanning is not indicated in the absence of symptoms.

Biopsy should not be performed when a nephrectomy is planned but is mandatory before any
ablative therapies are undertaken.

Assessment of the functioning of the contra lateral kidney.

Management
T1 lesions may be managed by partial nephrectomy and this gives equivalent oncological results to
total radical nephrectomy. Partial nephrectomy may also be performed when there is inadequate
reserve in the remaining kidney.

For T2 lesions and above a radical nephrectomy is standard practice and this may be performed via
a laparoscopic or open approach. Preoperative embolisation is not indicated nor is resection of
uninvolved adrenal glands. During surgery early venous control is mandatory to avoid shedding of
tumour cells into the circulation.

Patients with completely resected disease do not benefit from adjuvant therapy with either
chemotherapy or biological agents. These should not be administered outside the setting of clinical
trials.

Patients with transitional cell cancer will require a nephroureterectomy with disconnection of the
ureter at the bladder.

References
Lungberg B et al. EAU guidelines on renal cell carcinoma: The 2010 update. European Urology 2010
(58): 398-406.
Next question
Theme: Management of urinary tract calculi

A. Nephrectomy
B. Open ureteric exploration
C. Extra corporeal shock wave lithotripsy
D. Percutaneous nephrostomy
E. Pyeloplasty
F. Conservative management
G. Percutaneous nephrolithotomy

Please select the most appropriate management for the scenario given. Each option may be used
once, more than once or not at all.

2. A 23 year old male is admitted with left sided loin pain and fever. His investigations
demonstrate a left sided ureteric calculi that measures 0.7cm in diameter and associated
hydronephrosis.

You answered Nephrectomy

The correct answer is Percutaneous nephrostomy

Theme from January 2013 Exam


An obstructed, infected system is an indication for urgent decompression. This may be
achieved by ureteroscopy or nephrostomy. In addition to this the patient should also receive
broad spectrum, intravenous antibiotics.

3. A 23 year old man is admitted with left sided loin pain that radiates to his groin. His
investigations demonstrate a 1cm left sided ureteric calculus with no associated
hydronephrosis.

You answered Nephrectomy

The correct answer is Extra corporeal shock wave lithotripsy

Stones with a total volume of less than 2cm can be considered for lithotripsy. If it is
impacted in the upper ureter then some may consider a ureteroscopy.

4. A 30 year old male presents with left sided loin pain. His investigations demonstrate a large
left sided staghorn calculus that measures 2.3cm in diameter.

You answered Nephrectomy

The correct answer is Percutaneous nephrolithotomy


Large, proximal stones are generally best managed with a percutaneous nephrolithotomy.
The use of lithotripsy has low clearance rates. Where stones remain after the initial
procedure a repeat percutaneous nephrolithotomy is generally preferred over follow up
lithotripsy.

Please rate this question:

Discuss and give feedback


Next question

Management of renal stones

Urolithiasis will affect up to 15% of the worldwide population. The development of sudden onset loin
to groin pain which is colicky in nature is a classic feature in the history. It is nearly always
associated with haematuria that is either micro or macroscopic.
Where the diagnosis is suspected the most sensitive and specific diagnostic test is helical, non
contrast, computerised tomographic (CT) scanning.

Management
Most renal stones measuring less than 5mm in maximum diameter will typically pass within 4 weeks
of symptom onset. More intensive and urgent treatment is indicated in the presence of ureteric
obstruction, renal developmental abnormality such as horseshoe kidney and previous renal
transplant. Ureteric obstruction due to stones together with infection is a surgical emergency and the
system must be decompressed. Options include nephrostomy tube placement, insertion of ureteric
catheters and ureteric stent placement.
In the non emergency setting the preferred options for treatment of stone disease include extra
corporeal shock wave lithotripsy, percutaneous nephrolithotomy, ureteroscopy, open surgery
remains an option for selected cases. However, minimally invasive options are the most popular first
line treatment.

Shock wave lithotripsy


A shock wave is generated external to the patient, internally cavitation bubbles and mechanical
stress lead to stone fragmentation. The passage of shock waves can result in the development of
solid organ injury. Fragmentation of larger stones may result in the development of ureteric
obstruction. The procedure is uncomfortable for patients and analgesia is required during the
procedure and afterwards.

Ureteroscopy
A ureteroscope is passed retrograde through the ureter and into the renal pelvis. It is indicated in
individuals (e.g. pregnant females) where lithotripsy is contraindicated and in complex stone
disease. In most cases a stent is left in situ for 4 weeks after the procedure.

Percutaneous nephrolithotomy
In this procedure access is gained to the renal collecting system. Once access is achieved, intra
corporeal lithotripsy or stone fragmentation is performed and stone fragments removed.

Therapeutic selection
Disease Option

Stone burden of less than 2cm in aggregate Lithotripsy

Stone burden of less than 2cm in pregnant females Ureteroscopy

Complex renal calculi and staghorn calculi Percutaneous nephrolithotomy

Ureteric calculi less than 5mm Manage expectantly

Reference
Miller N and Lingeman J. Clinical review- Management of kidney stones. BMJ 2007;334:468- 472.
Next question
Theme: Testicular disorders

A. Antibiotics

B. Aspiration

C. Testicular exploration after 6 hours

D. Testicular exploration within 6 hours

E. Orchidectomy via inguinal approach

F. Orchidectomy via scrotal approach

G. No treatment needed

H. Ligation of patent processus vaginalis via inguinal approach

I. Jaboulay procedure via scrotal approach

For each scenario please select the most appropriate management. Each option may be used once,
more than once or not at all.

5. A parent brings her 4 year old child to the surgical clinic. She has noticed an intermittent swelling in
the right scrotum that is worse in the evening. On examination he has a soft fluctuant swelling in
the right scrotum that cannot be separated from the testis. It transilluminates when a pen torch is
held against it.

You answered Antibiotics

The correct answer is Ligation of patent processus vaginalis via inguinal approach

In children, a hydrocele is most commonly due to a persistent processus vaginalis. The swelling is
intermittent and in most cases that are diagnosed in infancy the hydrocele resolves. Cases that
persist beyond two years of age are best managed surgically and the surgical approach is usually
made via the inguinal canal where the patent processus is identified and ligated.

6. A 20 year old complains of severe pain in the right scrotal area after jumping onto his moped. He
has noticed discomfort intermittently in this area over the past few months. Clinically the testis is
tender.
You answered Antibiotics

The correct answer is Testicular exploration within 6 hours

Testicular torsion: Severe pain which can be spontaneous or precipitated by minor trauma. The
patient may have noticed pain previously. Surgical intervention is needed as soon as possible to
prevent the risk of loss of the testis.

7. A 44 year old man is referred to the clinic because of an swelling in the right scrotum. This is
present most of the time and he is otherwise well with no urinary symptoms. On examination he
has a soft, fluctuant swelling in the right scrotum that transilluminates easily. An ultrasound is
performed that confirms that the underlying testicle is structurally normal.

You answered Antibiotics

The correct answer is Jaboulay procedure via scrotal approach

Adult hydroceles are less commonly due to the persistence of embryonic remnants and therefore
can be managed via a scrotal approach. Both the Lords and Jaboulay procedures are reasonable
options.

Please rate this question:

Discuss and give feedback

Next question

Scrotal swelling

Differential diagnosis

Inguinal hernia If inguinoscrotal swelling; cannot "get above it" on examination


Cough impulse may be present
May be reducible

Testicular tumours Often discrete testicular nodule (may have associated hydrocele)
Symptoms of metastatic disease may be present
USS scrotum and serum AFP and β HCG required

Acute epididymo- Often history of dysuria and urethral discharge


orchitis Swelling may be tender and eased by elevating testis
Most cases due to Chlamydia
Infections with other gram negative organisms may be associated with underlying
structural abnormality

Epididymal cysts Single or multiple cysts


May contain clear or opalescent fluid (spermatoceles)
Usually occur over 40 years of age
Painless
Lie above and behind testis
It is usually possible to "get above the lump" on examination

Hydrocele Non painful, soft fluctuant swelling


Often possible to "get above it" on examination
Usually contain clear fluid
Will often transilluminate
May be presenting feature of testicular cancer in young men

Testicular torsion Severe, sudden onset testicular pain


Risk factors include abnormal testicular lie
Typically affects adolescents and young males
On examination testis is tender and pain not eased by elevation
Urgent surgery is indicated, the contra lateral testis should also be fixed

Varicocele Varicosities of the pampiniform plexus


Typically occur on left (because testicular vein drains into renal vein)
May be presenting feature of renal cell carcinoma
Affected testis may be smaller and bilateral varicoceles may affect fertility
Management

 Testicular malignancy is always treated with orchidectomy via an inguinal approach. This
allows high ligation of the testicular vessels and avoids exposure of another lymphatic field to
the tumour.
 Torsion is commonest in young teenagers and the history in older children can be difficult to
elicit. Intermittent torsion is a recognised problem. The treatment is prompt surgical
exploration and testicular fixation. This can be achieved using sutures or by placement of the
testis in a Dartos pouch.
 Varicoceles are usually managed conservatively. If there are concerns about testicular
function of infertility then surgery or radiological management can be considered.
 Epididymal cysts can be excised using a scrotal approach
 Hydroceles are managed differently in children where the underlying pathology is a patent
processus vaginalis and therefore an inguinal approach is used in children so that the
processus can be ligated. In adults a scrotal approach is preferred and the hydrocele sac
excised or plicated.

Next question
Theme: Urinary tract trauma

A. Urinary tract infection


B. Bladder outlet obstruction
C. Bulbar urethral rupture
D. Membranous urethral rupture
E. Bladder rupture
F. Bladder contusion

For the scenario given please select the most likely injury. Each option may be used once, more
than once or not at all.

8. A 56 year old man is involved in a road traffic accident. He is found to have a pelvic
fracture. He reports that he has some lower abdominal pain. He has peritonism in the lower
abdomen. The nursing staff report that he has not passed any urine. A CT scan shows
evidence of free fluid.

You answered Urinary tract infection

The correct answer is Bladder rupture

A pelvic fracture and lower abdominal peritonism should raise suspicions of bladder
rupture (especially as this man cannot pass urine).

9. A 52 year old man falls off his bike. He is found to have a pelvic fracture. On examination
he is found to have perineal oedema and on PR the prostate is not palpable. A urine
dipstick shows blood.

You answered Urinary tract infection

The correct answer is Membranous urethral rupture

A pelvic fracture and highly displaced prostate should indicate a diagnosis of membranous
urethral rupture.

10. A 52 year old woman falls out of a tree while rescuing a cat. Imaging shows no bony
injury. She has suprapubic tenderness and complains of dysuria. Her abdomen is soft and
non tender. A urine dipstick shows blood, nitrites and leucocytes.

Urinary tract infection

There is no indication of a more sinister diagnosis here. The patient's abdomen is normal
and she is able to pass urine. Her dipstick confirms an infection. Also in women urethral
injury is rare.
Similar theme questions in September 2009 and April 2010

Please rate this question:

Discuss and give feedback


Next question

Lower genitourinary tract trauma

 Most bladder injuries occur due to blunt trauma


 85% associated with pelvic fractures
 Easily overlooked during assessment in trauma
 Up to 10% of male pelvic fractures are associated with urethral or bladder injuries

Types of injury

Urethral injury  Mainly in males


 Blood at the meatus (50% cases)
 There are 2 types:

i.Bulbar rupture
- most common
- straddle type injury e.g. bicycles
- triad signs: urinary retention, perineal haematoma, blood at the
meatus
ii. Membranous rupture
- can be extra or intraperitoneal
- commonly due to pelvic fracture
- Penile or perineal oedema/ hematoma
- PR: prostate displaced upwards (beware co-existing
retroperitoneal haematomas as they may make examination
difficult)

- Investigation: ascending urethrogram


- Management: suprapubic catheter (surgical placement, not
percutaneously)

External genitalia injuries  Secondary to injuries caused by penetration, blunt trauma,


(i.e., the penis and the continence- or sexual pleasure-enhancing devices, and
scrotum) mutilation
Bladder injury  rupture is intra or extraperitoneal
 presents with haematuria or suprapubic pain
 history of pelvic fracture and inability to void: always
suspect bladder or urethral injury
 inability to retrieve all fluid used to irrigate the bladder
through a Foley catheter indicates bladder injury
 investigation- IVU or cystogram
 management: laparotomy if intraperitoneal, conservative if
extraperitoneal

Next question
Which of the following renal stone types is most radiodense on a plain x-ray?

Calcium phosphate

Calcium oxalate

Uric acid

Struvite

Cystine

Calcium phosphate stones are the most radiodense stones, calcium oxalate stones slightly less so.
Uric acid stones are radiolucent (unless they have calcium contained within them).

Please rate this question:

Discuss and give feedback

Next question

Renal stones

Type of Percentage of
stones Features all calculi

Calcium Hypercalciuria is a major risk factor (various causes) 85%


oxalate Hyperoxaluria may also increase risk
Hypocitraturia increases risk because citrate forms complexes with
calcium making it more soluble
Stones are radio-opaque (though less than calcium phosphate stones)
Hyperuricosuria may cause uric acid stones to which calcium oxalate
Type of Percentage of
stones Features all calculi

binds

Cystine Inherited recessive disorder of transmembrane cystine transport 1%


leading to decreased absorption of cystine from intestine and renal
tubule
Multiple stones may form
Relatively radiodense because they contain sulphur

Uric acid Uric acid is a product of purine metabolism 5-10%


May precipitate when urinary pH low
May be caused by diseases with extensive tissue breakdown e.g.
malignancy
More common in children with inborn errors of metabolism
Radiolucent

Calcium May occur in renal tubular acidosis, high urinary pH increases 10%
phosphate supersaturation of urine with calcium and phosphate
Renal tubular acidosis types 1 and 3 increase risk of stone formation
(types 2 and 4 do not)
Radio-opaque stones (composition similar to bone)

Struvite Stones formed from magnesium, ammonium and phosphate 2-20%


Occur as a result of urease producing bacteria (and are thus
associated with chronic infections)
Under the alkaline conditions produced, the crystals can precipitate
Slightly radio-opaque

Effect of urinary pH on stone formation


Urine pH will show individual variation (from pH 5-7). Post prandially the pH falls as purine
metabolism will produce uric acid. Then the urine becomes more alkaline (alkaline tide). When the
stone is not available for analysis the pH of urine may help to determine which stone was present.
Stone type Urine acidity Mean urine pH

Calcium phosphate Normal- alkaline >5.5

Calcium oxalate Variable 6

Uric acid Acid 5.5

Struvate Alkaline >7.2

Cystine Normal 6.5

Next question
Theme: Scrotal swellings

A. Haematocele
B. Epididymal cyst
C. Hydrocele
D. Testicular torsion
E. Orchitis
F. Epididymo-orchitis

For each case please select the most likely underlying diagnosis from the list. Each option may be
used once, more than once or not at all.

12. A 32 year old male presents with a swollen right scrotum which has developed over 3
weeks after being kicked in the groin area. There is a non tense swelling of the right
scrotum and the underlying testis cannot be easily palpated. A dipstick is positive for
nitrates only.

You answered Haematocele

The correct answer is Hydrocele

This is a secondary hydrocele which occurs in patients aged 20-40 years. It develops
rapidly and there may not be a tense swelling. The underlying testis is NOT palpated
therefore indicating a hydrocele. Causes include trauma, infection and tumour.

13. A 40 year old male presents with a non painful, bilateral scrotal swellings over 3 years.
The testis is felt separately and the swelling transilluminates.

You answered Haematocele

The correct answer is Epididymal cyst

The testis is palpated therefore this differentiates it from a hydrocele.

14. A 32 year old male presents with a swollen, painful right scrotum after being kicked in the
groin area 1 hour ago. There is a painful swelling of the right scrotum and the underlying
testis cannot be easily palpated.

Haematocele

Acute haematocele: tense, tender and non transilluminating mass post trauma. A chronic
haematoma causes a blood clot to surround the testis. The blood clot hardens and contracts
causing a hard mass which may be indistinguishable from a tumour. Therefore the testis
will need surgical exploration.

Please rate this question:

Discuss and give feedback


Next question

Scrotal swelling

Differential diagnosis
Inguinal hernia If inguinoscrotal swelling; cannot "get above it" on examination
Cough impulse may be present
May be reducible

Testicular tumours Often discrete testicular nodule (may have associated hydrocele)
Symptoms of metastatic disease may be present
USS scrotum and serum AFP and β HCG required

Acute epididymo- Often history of dysuria and urethral discharge


orchitis Swelling may be tender and eased by elevating testis
Most cases due to Chlamydia
Infections with other gram negative organisms may be associated with
underlying structural abnormality

Epididymal cysts Single or multiple cysts


May contain clear or opalescent fluid (spermatoceles)
Usually occur over 40 years of age
Painless
Lie above and behind testis
It is usually possible to "get above the lump" on examination

Hydrocele Non painful, soft fluctuant swelling


Often possible to "get above it" on examination
Usually contain clear fluid
Will often transilluminate
May be presenting feature of testicular cancer in young men
Testicular torsion Severe, sudden onset testicular pain
Risk factors include abnormal testicular lie
Typically affects adolescents and young males
On examination testis is tender and pain not eased by elevation
Urgent surgery is indicated, the contra lateral testis should also be fixed

Varicocele Varicosities of the pampiniform plexus


Typically occur on left (because testicular vein drains into renal vein)
May be presenting feature of renal cell carcinoma
Affected testis may be smaller and bilateral varicoceles may affect fertility

Management

 Testicular malignancy is always treated with orchidectomy via an inguinal approach. This
allows high ligation of the testicular vessels and avoids exposure of another lymphatic field to
the tumour.
 Torsion is commonest in young teenagers and the history in older children can be difficult to
elicit. Intermittent torsion is a recognised problem. The treatment is prompt surgical
exploration and testicular fixation. This can be achieved using sutures or by placement of the
testis in a Dartos pouch.
 Varicoceles are usually managed conservatively. If there are concerns about testicular
function of infertility then surgery or radiological management can be considered.
 Epididymal cysts can be excised using a scrotal approach
 Hydroceles are managed differently in children where the underlying pathology is a patent
processus vaginalis and therefore an inguinal approach is used in children so that the
processus can be ligated. In adults a scrotal approach is preferred and the hydrocele sac
excised or plicated.

Next question
Theme: Management of testicular disorders

A. Fine needle aspiration cytology


B. Excision biopsy
C. Orchidectomy via an inguinal approach
D. Lords procedure
E. Orchidectomy via a scrotal approach
F. Division of patent processus vaginalis via an inguinal approach
G. Division of patent processus vaginalis via a scrotal approach
H. Fowler Stephens procedure
I. Immediate scrotal exploration
J. Conservative management

Please select the most appropriate management for the following testicular disorders. Each option
may be used once, more than once or not at all.

15. An 85 year old man is diagnosed as having prostate cancer and is considered suitable for
hormonal ablation. However, he does not want the repeated injections of GnRH
analogues.

You answered Fine needle aspiration cytology

The correct answer is Orchidectomy via a scrotal approach

At one time bilateral orchidectomy was performed routinely when prostate cancer was
diagnosed (sometimes under the same anaesthetic). A combination of modern agents
(GnRH analogues) and better consent processes have made this almost obsolete. Where
required a scrotal approach should be used.

16. A 33 year old man presents with a painless lump in his left testes. USS and blood tests are
suspicious for teratoma.

You answered Fine needle aspiration cytology

The correct answer is Orchidectomy via an inguinal approach

Oncological orchidectomy is routinely performed via an inguinal approach to avoid


contamination of another lymphatic field.

17. A 4 year old boy is brought to the clinic by his mother. He has a swelling in his right
hemiscrotum. On examination is transilluminates brilliantly.

You answered Fine needle aspiration cytology


The correct answer is Division of patent processus vaginalis via an inguinal approach

Ligation of the patent processus vaginalis is performed via an inguinal approach. There is
no indication for scrotal surgery for hydrocele in young children.

Please rate this question:

Discuss and give feedback


Next question

Testicular disorders

Testicular cancer
Testicular cancer is the most common malignancy in men aged 20-30 years. Around 95% of cases
of testicular cancer are germ-cell tumours. Germ cell tumours may essentially be divided into:

Tumour
Tumour type Key features markers Pathology

Seminoma  Commonest  AFP usually Sheet like lobular


subtype (50%) normal patterns of cells
 Average age at  HCG elevated with substantial
diagnosis = 40 in 10% fibrous
 Even advanced seminomas component.
disease associated  Lactate Fibrous septa
with 5 year dehydrogenase; contain
survival of 73% elevated in 10- lymphocytic
20% seminomas inclusions and
(but also in many granulomas may
other conditions) be seen.

Non seminomatous germ  Younger age at  AFP elevated Heterogenous


cell tumours (42%) presentation =20-30 in up to 70% of texture with
years cases occasional ectopic
 Advanced disease  HCG elevated tissue such as hair
 Teratoma carries worse prognosis in up to 40% of
 Yolk sac tumour (48% at 5 years) cases
 Choriocarcinoma  Retroperitoneal lymph  Other markers
 Mixed germ cell node dissection may be rarely helpful
tumours (10%) needed for residual
disease after
chemotherapy
Image demonstrating a classical seminoma, these tumours are typically more uniform than
teratomas

Image sourced from Wikipedia

Risk factors for testicular cancer

 Cryptorchidism
 Infertility
 Family history
 Klinefelter's syndrome
 Mumps orchitis
Features

 A painless lump is the most common presenting symptom


 Pain may also be present in a minority of men
 Other possible features include hydrocele, gynaecomastia

Diagnosis

 Ultrasound is first-line
 CT scanning of the chest/ abdomen and pelvis is used for staging
 Tumour markers (see above) should be measured

Management

 Orchidectomy (Inguinal approach)


 Chemotherapy and radiotherapy may be given depending on staging
 Abdominal lesions >1cm following chemotherapy may require retroperitoneal lymph node
dissection.

Prognosis is generally excellent

 5 year survival for seminomas is around 95% if Stage I


 5 year survival for teratomas is around 85% if Stage I

Benign disease

Epididymo-orchitis
Acute epididymitis is an acute inflammation of the epididymis, often involving the testis and usually
caused by bacterial infection.

 Infection spreads from the urethra or bladder. In men <35 years, gonorrhoea or chlamydia
are the usual infections.
 Amiodarone is a recognised non infective cause of epididymitis, which resolves on stopping
the drug.
 Tenderness is usually confined to the epididymis, which may facilitate differentiating it from
torsion where pain usually affects the entire testis.

Testicular torsion
 Twist of the spermatic cord resulting in testicular ischaemia and necrosis.
 Most common in males aged between 10 and 30 (peak incidence 13-15 years)
 Pain is usually severe and of sudden onset.
 Cremasteric reflex is lost and elevation of the testis does not ease the pain.
 Treatment is with surgical exploration. If a torted testis is identified then both testis should be
fixed as the condition of bell clapper testis is often bilateral.

Hydrocele

 Presents as a mass that transilluminates, usually possible to "get above" it on examination.


 In younger men it should be investigated with USS to exclude tumour.
 In children it may occur as a result of a patent processus vaginalis.
 Treatment in adults is with a Lords or Jabouley procedure.
 Treatment in children is with trans inguinal ligation of PPV.

Next question
Theme: Management of testicular disorders

A. Antibiotics
B. Aspiration
C. Testicular exploration after 6 hours
D. Testicular exploration within 6 hours
E. Orchidectomy via inguinal approach
F. Orchidectomy via scrotal approach
G. No treatment needed

Please select the most appropriate management for the scenario given. Each option may be used
once, more than once or not at all.

18. A 20 year old male notices a mild painful swelling of his right scrotum. He also complains
of abdominal pain. Clinically the patient is found to have a swollen right testicle. Apart
from a supraclavicular node, there is no obvious lymphadenopathy.

You answered Antibiotics

The correct answer is Orchidectomy via inguinal approach

The patient is likely to have a teratoma which has metastasized to the supraclavicular
nodes. There is suspicion of spread to the para-aortic nodes due to the abdominal pain. He
will need orchidectomy and combination chemotherapy. There is no role for orchidectomy
via scrotal approach in malignancy.

19. A 40 year old male presents with a non painful, bilateral scrotal swellings over 3 years.
The testis is felt separately and the swelling transilluminates.

You answered Antibiotics

The correct answer is No treatment needed

This is an epididymal cyst, the testis is palpated therefore this differentiates it from a
hydrocele.

20. A 32 year old male presents with a swollen, painful right scrotum after being kicked in the
groin area. There is a painful swelling of the right scrotum and the underlying testis cannot
be easily palpated.

You answered Antibiotics

The correct answer is Testicular exploration within 6 hours


Acute haematocele: tense, tender and non transilluminating mass. The testis will need
surgical exploration to evacuate the blood and repair any damage.

Please rate this question:

Discuss and give feedback


Next question

Scrotal swelling

Differential diagnosis
Inguinal hernia If inguinoscrotal swelling; cannot "get above it" on examination
Cough impulse may be present
May be reducible

Testicular tumours Often discrete testicular nodule (may have associated hydrocele)
Symptoms of metastatic disease may be present
USS scrotum and serum AFP and β HCG required

Acute epididymo- Often history of dysuria and urethral discharge


orchitis Swelling may be tender and eased by elevating testis
Most cases due to Chlamydia
Infections with other gram negative organisms may be associated with
underlying structural abnormality

Epididymal cysts Single or multiple cysts


May contain clear or opalescent fluid (spermatoceles)
Usually occur over 40 years of age
Painless
Lie above and behind testis
It is usually possible to "get above the lump" on examination

Hydrocele Non painful, soft fluctuant swelling


Often possible to "get above it" on examination
Usually contain clear fluid
Will often transilluminate
May be presenting feature of testicular cancer in young men
Testicular torsion Severe, sudden onset testicular pain
Risk factors include abnormal testicular lie
Typically affects adolescents and young males
On examination testis is tender and pain not eased by elevation
Urgent surgery is indicated, the contra lateral testis should also be fixed

Varicocele Varicosities of the pampiniform plexus


Typically occur on left (because testicular vein drains into renal vein)
May be presenting feature of renal cell carcinoma
Affected testis may be smaller and bilateral varicoceles may affect fertility

Management

 Testicular malignancy is always treated with orchidectomy via an inguinal approach. This
allows high ligation of the testicular vessels and avoids exposure of another lymphatic field to
the tumour.
 Torsion is commonest in young teenagers and the history in older children can be difficult to
elicit. Intermittent torsion is a recognised problem. The treatment is prompt surgical
exploration and testicular fixation. This can be achieved using sutures or by placement of the
testis in a Dartos pouch.
 Varicoceles are usually managed conservatively. If there are concerns about testicular
function of infertility then surgery or radiological management can be considered.
 Epididymal cysts can be excised using a scrotal approach
 Hydroceles are managed differently in children where the underlying pathology is a patent
processus vaginalis and therefore an inguinal approach is used in children so that the
processus can be ligated. In adults a scrotal approach is preferred and the hydrocele sac
excised or plicated.

Next question
A 75 year old man presents with locally advanced carcinoma of the prostate and vertebral body
metastasis with impending spinal cord compression. Which of the following agents (if used in
isolation) carries the greatest risk of worsening his symptoms in the short term?

Surgical orchidectomy

Cyproterone acetate

Luetenising hormone releasing hormone analogues

Flutamide

None of the above

Theme from April 2016 Exam


LHRH analogues may cause flare of metastatic disease and anti androgens should be administered
to counter this. Surgical orchidectomy reduces testosterone levels within 8 hours (but fails to reduce
adrenal androgen release). Cyproterone and flutamide are androgen blockers that may be
considered as add on therapy to reduce the risk of tumour flare when commencing treatment with
LH RH analogues.
Please rate this question:

Discuss and give feedback


Next question

Prostate Cancer

Prostate Cancer
This is a common condition and up to 30,000 men are diagnosed with the condition each year. Up to
9,000 will die in in the UK from the condition per year.

Diagnosis
Early prostate cancers have few symptoms.
Metastatic disease may present as bone pain.
Locally advanced disease may present as pelvic pain or with urinary symptoms.
Prostate specific antigen measurement
Digital rectal examination
Trans rectal USS (+/- biopsy)
MRI/ CT and bone scan for staging.

PSA Test
The normal upper limit for PSA is 4ng/ml. However, in this group will lie patients with benign disease
and some with localised prostate cancer. False positives may be due to prostatitis, UTI, BPH,
vigorous DRE.
The percentage of free: total PSA may help to distinguish benign disease from cancer. Values of
<20% are suggestive of cancer and biopsy is advised.

Pathology

 95% adenocarcinoma
 In situ malignancy is sometimes found in areas adjacent to cancer. Multiple biopsies needed
to call true in situ disease.
 Often multifocal- 70% lie in the peripheral zone.
 Graded using the Gleason grading system, two grades awarded 1 for most dominant grade
(on scale of 1-5) and 2 for second most dominant grade (scale 1-5). The two added together
give the Gleason score. Where 2 is best prognosis and 10 the worst.
 Lymphatic spread occurs first to the obturator nodes and local extra prostatic spread to the
seminal vesicles is associated with distant disease.

Treatment

 Watch and wait- Elderly, multiple co-morbidities, low Gleason score


 Radiotherapy (External)- Both potentially curative and palliative therapy possible. However,
radiation proctitis and rectal malignancy are late problems. Brachytherapy is a modification
allowing internal radiotherapy.
 Surgery- Radical prostatectomy. Surgical removal of the prostate is the standard treatment
for localised disease. The robot is being used increasingly for this procedure. As well as the
prostate the obturator nodes are also removed to complement the staging process. Erectile
dysfunction is a common side effect. Survival may be better than with radiotherapy (see
references).
 Hormonal therapy- Testosterone stimulates prostate tissue and prostatic cancers usually
show some degree of testosterone dependence. 95% of testosterone is derived from the
testis and bilateral orchidectomy may be used for this reason. Pharmacological alternatives
include LHRH analogues and anti androgens (which may be given in combination).
 In the UK the National Institute for Clinical Excellence (NICE) suggests that active
surveillance is the preferred option for low risk men. It is particularly suitable for men with
clinical stage T1c, Gleason score 3+3 and PSA density < 0.15 ng/ml/ml who have cancer in
less than 50% of their biopsy cores, with < 10 mm of any core involved.

Candidates for active surveillance should:

 have had at least 10 biopsy cores taken


 have at least one re-biopsy.

If men on active surveillance show evidence of disease progression, offer radical treatment.
Treatment decisions should be made with the man, taking into account co-morbidities and life
expectancy.

References
1. Prostate cancer pathway. NICE.(http://guidance.nice.org.uk/IPG424)
2. Sooriakumaran P et al. Comparative effectiveness of radical prostatectomy and radiotherapy in
prostate cancer: observational study of mortality outcomes. BMJ 2014 (348):13. This study shows
that in men with localised disease survival was greater in those offered surgery.
Next question
A 42 year old man undergoes a vasectomy at the surgical clinic. He is reviewed at the request of his
general practitioner. On examination he has a small rounded nodule adjacent to the vas. What is the
most likely underlying diagnosis?

Haematoma

Sperm granuloma

Varicocele

Hydrocele

Epididymal cyst

Sperm granulomas are a common sequelae of vasectomy and are smooth round lumps adherant to
the vas. They may be safely left alone.

Please rate this question:

Discuss and give feedback

Next question

Vasectomy

Vasectomy is a commonly performed technique for achieving permanent sterilisation. It has a failure
rate of 1 in 2000 and is conveniently performed under local anaesthesia.
Reversal success rates are approximately 55% if performed within 10 years of the procedure. For
the purposes of counseling the procedure should be deemed permanent.

Procedure
Small bilateral incisions and formal dissection of the vas is the standard technique. A technique
involving the use of haemostats for skin puncture is used in the "no scalpel"
technique. It is not necessary to routinely send the vas for histology.
Controversies
Be wary of performing the procedure in childless, single men under age 30.

Risks
Following the procedure between 12 and 52% of men reported chronic scrotal pain. Of which 5.2%
sought help. Haematomas and sperm granulomas may also occur.

Follow up
Viable sperm may persist following surgery. Clearance should not be granted until a negative sperm
sample is available. This is usually taken after 12-16 weeks post procedure (and preferably after 24
ejaculates). Recanalisation may occur in 0.04% of cases.In a small minority of men, non-motile
sperm persist after vasectomy. In such cases, "special clearance" to stop contraception may be
given when less than 10,000 non-motile sperm/mL are found in a fresh specimen examined at least
7 months after vasectomy.
The risks of STI's are unchanged.

Next question
Theme: Haematuria

A. Retroperitoneal liposarcoma
B. Transitional cell carcinoma
C. Retroperitoneal fibrosis
D. Renal squamous cell carcinoma
E. Renal adenocarcinoma
F. Nephroblastoma

Please select the most likely cause of haematuria for the scenarios given. Each option may be used
once, more than once or not at all.

23. A 28 year old man presents with hypertension and haematuria. Haematological
investigations show polycythaemia but otherwise no abnormality. CT scanning shows a
left renal mass.

You answered Retroperitoneal liposarcoma

The correct answer is Renal adenocarcinoma

Renal adenocarcinoma is the most common variant and is associated with polycythaemia.

24. A 68 year of man presents with recurrent episodes of left sided ureteric colic and
haematuria. Investigations show some dilatation of the renal pelvis but the outline is
irregular.

You answered Retroperitoneal liposarcoma

The correct answer is Transitional cell carcinoma

These arise from urothelium and necessitate a nephroureterectomy.

25. A 4 year old boy presents with haematuria and on examination is found to have a right
sided renal mass.

You answered Retroperitoneal liposarcoma

The correct answer is Nephroblastoma

Wilms tumours (nephroblastoma) usually present in the first 4 years of life and may cause
lung metastases.

Please rate this question:


Discuss and give feedback
Next question

Haematuria

Causes of haematuria

Trauma  Injury to renal tract


 Renal trauma commonly due to blunt injury (others penetrating
injuries)
 Ureter trauma rare: iatrogenic
 Bladder trauma: due to RTA or pelvic fractures

Infection  Remember TB

Malignancy  Renal cell carcinoma (remember paraneoplastic syndromes):


painful or painless
 Urothelial malignancies: 90% are transitional cell carcinoma, can
occur anywhere along the urinary tract. Painless haematuria.
 Squamous cell carcinoma and adenocarcinoma: rare bladder
tumours
 Prostate cancer
 Penile cancers: SCC

Renal disease  Glomerulonephritis

Stones  Microscopic haematuria common

Structural  Benign prostatic hyperplasia (BPH) causes haematuria due to


abnormalities hypervascularity of the prostate gland
 Cystic renal lesions e.g. polycystic kidney disease
 Vascular malformations
 Renal vein thrombosis due to renal cell carcinoma

Coagulopathy  Causes bleeding of underlying lesions

Drugs  Cause tubular necrosis or interstitial nephritis: aminoglycosides,


chemotherapy
 Interstitial nephritis: penicillin, sulphonamides, and NSAIDs
 Anticoagulants

Benign  Exercise

Gynaecological  Endometriosis: flank pain, dysuria, and haematuria that is cyclical

Iatrogenic  Catheterisation
 Radiotherapy; cystitis, severe haemorrhage, bladder necrosis

Pseudohaematuria For example following consumption of beetroot

References
Http://bestpractice.bmj.com/best-practice/monograph/316/overview/aetiology.html
Next question
A 22 year old man is involved in a road traffic accident. He is found to have a pelvic fracture. While
on the ward the nursing staff report that he is complaining of lower abdominal pain. On examination
you find a distended tender bladder. What is the most likely diagnosis?

Bladder rupture

Ureter injury

Urethral injury

Clot retention

Prostate rupture

Theme from 2009 Exam

Pelvic fractures may cause laceration of the urethra. Urinary retention, blood at the urethral meatus
and a high riding prostate on digital rectal examination are the typical features.
Please rate this question:

Discuss and give feedback


Next question

Lower genitourinary tract trauma

 Most bladder injuries occur due to blunt trauma


 85% associated with pelvic fractures
 Easily overlooked during assessment in trauma
 Up to 10% of male pelvic fractures are associated with urethral or bladder injuries

Types of injury

Urethral injury  Mainly in males


 Blood at the meatus (50% cases)
 There are 2 types:

i.Bulbar rupture
- most common
- straddle type injury e.g. bicycles
- triad signs: urinary retention, perineal haematoma, blood at the
meatus
ii. Membranous rupture
- can be extra or intraperitoneal
- commonly due to pelvic fracture
- Penile or perineal oedema/ hematoma
- PR: prostate displaced upwards (beware co-existing
retroperitoneal haematomas as they may make examination
difficult)

- Investigation: ascending urethrogram


- Management: suprapubic catheter (surgical placement, not
percutaneously)

External genitalia injuries  Secondary to injuries caused by penetration, blunt trauma,


(i.e., the penis and the continence- or sexual pleasure-enhancing devices, and
scrotum) mutilation

Bladder injury  rupture is intra or extraperitoneal


 presents with haematuria or suprapubic pain
 history of pelvic fracture and inability to void: always
suspect bladder or urethral injury
 inability to retrieve all fluid used to irrigate the bladder
through a Foley catheter indicates bladder injury
 investigation- IVU or cystogram
 management: laparotomy if intraperitoneal, conservative if
extraperitoneal

Next question
Theme: Haematuria

A. Benign prostatic hyperplasia


B. Ureteric calculus
C. Pyelonephritis
D. Prostatitis
E. Cystitis
F. Prostate cancer

Please select the most likely source of haematuria for the scenarios given. Each option may be used
once, more than once or not at all.

27. A 67 year old man presents with recurrent episodes of haematuria, typically at the end of
the urinary stream, he has been suffering from occasional fevers and has noticed pus on the
urethral meatus on occasion. On examination the prostate has no discernable masses but is
tender.

You answered Benign prostatic hyperplasia

The correct answer is Prostatitis

This is most likely prostatitis and the bleeding at the end of micturition suggests a distal
problem. Treatment is usually with prolonged courses of antibiotics.

28. A 23 year old girl is admitted with loin pain and a fever, she has noticed haematuria for the
past week accompanied by dysuria, this was treated empirically with trimethoprim.

You answered Benign prostatic hyperplasia

The correct answer is Pyelonephritis

This is most likely pyelonephritis and partially treated cystitis is a common cause.

29. A 56 year old man is admitted with severe loin to groin pain associated with haematuria.
He was well until 1 week ago when he was unwell with diarrhoea and vomiting.

You answered Benign prostatic hyperplasia

The correct answer is Ureteric calculus

Ureteric stones may develop in a background of dehydration.

Please rate this question:


Discuss and give feedback
Next question

Haematuria

Causes of haematuria

Trauma  Injury to renal tract


 Renal trauma commonly due to blunt injury (others penetrating
injuries)
 Ureter trauma rare: iatrogenic
 Bladder trauma: due to RTA or pelvic fractures

Infection  Remember TB

Malignancy  Renal cell carcinoma (remember paraneoplastic syndromes):


painful or painless
 Urothelial malignancies: 90% are transitional cell carcinoma, can
occur anywhere along the urinary tract. Painless haematuria.
 Squamous cell carcinoma and adenocarcinoma: rare bladder
tumours
 Prostate cancer
 Penile cancers: SCC

Renal disease  Glomerulonephritis

Stones  Microscopic haematuria common

Structural  Benign prostatic hyperplasia (BPH) causes haematuria due to


abnormalities hypervascularity of the prostate gland
 Cystic renal lesions e.g. polycystic kidney disease
 Vascular malformations
 Renal vein thrombosis due to renal cell carcinoma

Coagulopathy  Causes bleeding of underlying lesions

Drugs  Cause tubular necrosis or interstitial nephritis: aminoglycosides,


chemotherapy
 Interstitial nephritis: penicillin, sulphonamides, and NSAIDs
 Anticoagulants

Benign  Exercise

Gynaecological  Endometriosis: flank pain, dysuria, and haematuria that is cyclical

Iatrogenic  Catheterisation
 Radiotherapy; cystitis, severe haemorrhage, bladder necrosis

Pseudohaematuria For example following consumption of beetroot

References
Http://bestpractice.bmj.com/best-practice/monograph/316/overview/aetiology.html
Next question
Which of the following would be most consistent with a histologically aggressive form of prostate
cancer?

FIGO stage 1 disease

FIGO stage IV disease

EuroQOL score of 5

Gleason score of 2

Gleason score of 10

Prostate cancer is histologically graded using the Gleason score (see below). A score of 10 is
consistent with a histologically aggressive form of the disease. The FIGO staging system is used to
stage gynaecological malignancy. The EuroQOL score is a quality of life measurement tool.
Please rate this question:

Discuss and give feedback


Next question

Prostate Cancer

Prostate Cancer
This is a common condition and up to 30,000 men are diagnosed with the condition each year. Up to
9,000 will die in in the UK from the condition per year.

Diagnosis
Early prostate cancers have few symptoms.
Metastatic disease may present as bone pain.
Locally advanced disease may present as pelvic pain or with urinary symptoms.
Prostate specific antigen measurement
Digital rectal examination
Trans rectal USS (+/- biopsy)
MRI/ CT and bone scan for staging.

PSA Test
The normal upper limit for PSA is 4ng/ml. However, in this group will lie patients with benign disease
and some with localised prostate cancer. False positives may be due to prostatitis, UTI, BPH,
vigorous DRE.
The percentage of free: total PSA may help to distinguish benign disease from cancer. Values of
<20% are suggestive of cancer and biopsy is advised.
Pathology

 95% adenocarcinoma
 In situ malignancy is sometimes found in areas adjacent to cancer. Multiple biopsies needed
to call true in situ disease.
 Often multifocal- 70% lie in the peripheral zone.
 Graded using the Gleason grading system, two grades awarded 1 for most dominant grade
(on scale of 1-5) and 2 for second most dominant grade (scale 1-5). The two added together
give the Gleason score. Where 2 is best prognosis and 10 the worst.
 Lymphatic spread occurs first to the obturator nodes and local extra prostatic spread to the
seminal vesicles is associated with distant disease.

Treatment

 Watch and wait- Elderly, multiple co-morbidities, low Gleason score


 Radiotherapy (External)- Both potentially curative and palliative therapy possible. However,
radiation proctitis and rectal malignancy are late problems. Brachytherapy is a modification
allowing internal radiotherapy.
 Surgery- Radical prostatectomy. Surgical removal of the prostate is the standard treatment
for localised disease. The robot is being used increasingly for this procedure. As well as the
prostate the obturator nodes are also removed to complement the staging process. Erectile
dysfunction is a common side effect. Survival may be better than with radiotherapy (see
references).
 Hormonal therapy- Testosterone stimulates prostate tissue and prostatic cancers usually
show some degree of testosterone dependence. 95% of testosterone is derived from the
testis and bilateral orchidectomy may be used for this reason. Pharmacological alternatives
include LHRH analogues and anti androgens (which may be given in combination).
 In the UK the National Institute for Clinical Excellence (NICE) suggests that active
surveillance is the preferred option for low risk men. It is particularly suitable for men with
clinical stage T1c, Gleason score 3+3 and PSA density < 0.15 ng/ml/ml who have cancer in
less than 50% of their biopsy cores, with < 10 mm of any core involved.

Candidates for active surveillance should:

 have had at least 10 biopsy cores taken


 have at least one re-biopsy.

If men on active surveillance show evidence of disease progression, offer radical treatment.
Treatment decisions should be made with the man, taking into account co-morbidities and life
expectancy.

References
1. Prostate cancer pathway. NICE.(http://guidance.nice.org.uk/IPG424)
2. Sooriakumaran P et al. Comparative effectiveness of radical prostatectomy and radiotherapy in
prostate cancer: observational study of mortality outcomes. BMJ 2014 (348):13. This study shows
that in men with localised disease survival was greater in those offered surgery.
Next question
A 13 month old boy is brought to the paediatric clinic by his mother who is concerned that his testis
are not palpable. On examination his testis are not palpable either in the scrotum or inguinal region
and cannot be visualised on ultrasound either. What is the most appropriate next stage in
management?

Laparoscopy

Re-assess at 5 years of age

Re-assess at 13 years of age

Administration of testosterone

Administration of cyproterone acetate

Impalpable testes are an indication for laparoscopy. Ultrasound is a relatively unhelpful tool in
evaluating cryptorchid patients and most experienced paediatric surgeons would not use it pre-
operatively. They may be associated with an intra-abdominal location. Whilst it is reasonable to defer
orchidopexy for retractile testis completely absent testes should be investigated further.
Please rate this question:

Discuss and give feedback


Next question

Cryptorchidism

A congenital undescended testis is one that has failed to reach the bottom of the scrotum by 3
months of age. At birth up to 5% of boys will have an undescended testis, post natal descent occurs
in most and by 3 months the incidence of cryptorchidism falls to 1-2%. In the vast majority of cases
the cause of the maldescent is unknown. A proportion may be associated with other congenital
defects including:

Patent processus vaginalis


Abnormal epididymis
Cerebral palsy
Mental retardation
Wilms tumour
Abdominal wall defects (e.g. gastroschisis, prune belly syndrome)

Differential diagnosis
These include retractile testes and, in the case of absent bilateral testes the possibility of intersex
conditions. A retractile testis can be brought into the scrotum by the clinician and when released
remains in the scrotum. If the examining clinician notes the testis to return rapidly into the inguinal
canal when released then surgery is probably indicated.

Reasons for correction of cryptorchidism

 Reduce risk of infertility


 Allows the testes to be examined for testicular cancer
 Avoid testicular torsion
 Cosmetic appearance

Males with undescended testis are 40 times as likely to develop testicular cancer (seminoma) as
males without undescended testis
The location of the undescended testis affects the relative risk of testicular cancer (50% intra-
abdominal testes)

Treatment

 Orchidopexy at 6- 18 months of age. The operation usually consists of inguinal exploration,


mobilisation of the testis and implantation into a dartos pouch.
 Intra-abdominal testis should be evaluated laparoscopically and mobilised. Whether this is a
single stage or two stage procedure depends upon the exact location.
 After the age of 2 years in untreated individuals the Sertoli cells will degrade and those
presenting late in teenage years may be better served by orchidectomy than to try and
salvage a non functioning testis with an increased risk of malignancy.

Next question
A 34-year-old man from Zimbabwe is admitted with abdominal pain to the Emergency Department.
An abdominal x-ray reveals urinary bladder calcification. What is the most likely cause?

Schistosoma mansoni

Sarcoidosis

Leishmaniasis

Tuberculosis

Schistosoma haematobium

Schistosoma haematobium causes haematuria

Theme from April 2016 exam


Schistosomiasis is the most common cause of bladder calcification worldwide. Schistosoma
mansoni typically resided in the colon from where it is excreted.
Please rate this question:

Discuss and give feedback


Next question

Schistosomiasis

Schistosomiasis, or bilharzia, is a parasitic flatworm infection. The following types of schistosomiasis


are recognised:

 Schistosoma mansoni and Schistosoma intercalatum: intestinal schistosomiasis


 Schistosoma haematobium: urinary schistosomiasis

Schistosoma haematobium
This typically presents as a 'swimmer's itch' in patients who have recently returned from Africa.
Schistosoma haematobium is a risk factor for squamous cell bladder cancer

Features

 Frequency
 Haematuria
 Bladder calcification

Management

 Single oral dose of praziquantel

Next question
Theme: Testicular disorders

A. Testicular tumour
B. Torsion of the spermatic cord
C. Acute infective epididymo-orchitis
D. Non infective epididymo- orchitis
E. Torsion of testicular appendage
F. Hydrocele
G. Haematocele

Please select the most likely cause for the testicular disorder described. Each option may be used
once, more than once or not at all.

33. An 28 year old man presents with pain in the testis and scrotum. It began 10 hours
previously and has worsened during that time. On examination he is pyrexial, the testis is
swollen and tender and there is an associated hydrocele.

You answered Testicular tumour

The correct answer is Acute infective epididymo-orchitis

The onset is relatively slow for torsion and the presence of fever favors epididymo-
orchitis.

34. A 15 year old boy develops sudden onset of pain in the left hemiscrotum. He has no other
urinary symptoms. On examination the superior pole of the testis is tender and the
cremasteric reflex is particularly marked.

You answered Testicular tumour

The correct answer is Torsion of testicular appendage

The cremasteric reflex is usually preserved when the torsion affects the appendage only.

35. A 14 year old boy develops sudden onset severe pain in the left testicle radiating to the left
groin. He is distressed and vomits. On examination the testis is very tender and the
cremasteric reflex is absent.

You answered Testicular tumour

The correct answer is Torsion of the spermatic cord

The cremasteric reflex is usually absent in torsion.


Similar theme question in September 2011 exam

Please rate this question:

Discuss and give feedback


Next question

Testicular disorders

Testicular cancer
Testicular cancer is the most common malignancy in men aged 20-30 years. Around 95% of cases
of testicular cancer are germ-cell tumours. Germ cell tumours may essentially be divided into:

Tumour
Tumour type Key features markers Pathology

Seminoma  Commonest  AFP usually Sheet like lobular


subtype (50%) normal patterns of cells
 Average age at  HCG elevated with substantial
diagnosis = 40 in 10% fibrous
 Even advanced seminomas component.
disease associated  Lactate Fibrous septa
with 5 year dehydrogenase; contain
survival of 73% elevated in 10- lymphocytic
20% seminomas inclusions and
(but also in many granulomas may
other conditions) be seen.

Non seminomatous germ  Younger age at  AFP elevated Heterogenous


cell tumours (42%) presentation =20-30 in up to 70% of texture with
years cases occasional ectopic
 Advanced disease  HCG elevated tissue such as hair
 Teratoma carries worse prognosis in up to 40% of
 Yolk sac tumour (48% at 5 years) cases
 Choriocarcinoma  Retroperitoneal lymph  Other markers
 Mixed germ cell node dissection may be rarely helpful
tumours (10%) needed for residual
disease after
chemotherapy

Image demonstrating a classical seminoma, these tumours are typically more uniform than
teratomas
Image sourced from Wikipedia

Risk factors for testicular cancer

 Cryptorchidism
 Infertility
 Family history
 Klinefelter's syndrome
 Mumps orchitis

Features
 A painless lump is the most common presenting symptom
 Pain may also be present in a minority of men
 Other possible features include hydrocele, gynaecomastia

Diagnosis

 Ultrasound is first-line
 CT scanning of the chest/ abdomen and pelvis is used for staging
 Tumour markers (see above) should be measured

Management

 Orchidectomy (Inguinal approach)


 Chemotherapy and radiotherapy may be given depending on staging
 Abdominal lesions >1cm following chemotherapy may require retroperitoneal lymph node
dissection.

Prognosis is generally excellent

 5 year survival for seminomas is around 95% if Stage I


 5 year survival for teratomas is around 85% if Stage I

Benign disease

Epididymo-orchitis
Acute epididymitis is an acute inflammation of the epididymis, often involving the testis and usually
caused by bacterial infection.

 Infection spreads from the urethra or bladder. In men <35 years, gonorrhoea or chlamydia
are the usual infections.
 Amiodarone is a recognised non infective cause of epididymitis, which resolves on stopping
the drug.
 Tenderness is usually confined to the epididymis, which may facilitate differentiating it from
torsion where pain usually affects the entire testis.

Testicular torsion

 Twist of the spermatic cord resulting in testicular ischaemia and necrosis.


 Most common in males aged between 10 and 30 (peak incidence 13-15 years)
 Pain is usually severe and of sudden onset.
 Cremasteric reflex is lost and elevation of the testis does not ease the pain.
 Treatment is with surgical exploration. If a torted testis is identified then both testis should be
fixed as the condition of bell clapper testis is often bilateral.

Hydrocele

 Presents as a mass that transilluminates, usually possible to "get above" it on examination.


 In younger men it should be investigated with USS to exclude tumour.
 In children it may occur as a result of a patent processus vaginalis.
 Treatment in adults is with a Lords or Jabouley procedure.
 Treatment in children is with trans inguinal ligation of PPV.

Next question
Which of the following statements is false in relation to renal adenocarcinoma?

They account for over 75% cases of renal tumours

Renal biopsy should be performed in all cases considered for radical nephrectomy

They typically spread via the haematogenous route

Patients with completely resected T2 disease should not receive adjuvant chemotherapy

Partial nephrectomy gives equivalent oncological outcomes in patients with T1 disease

Routine chemotherapy is not effective in patients with renal adenocarcinoma and should not be used
following R0 resections.

Routine renal biopsy should not be performed in cases for nephrectomy. Most cases of malignancy
can be accurately classified on imaging.

Please rate this question:

Discuss and give feedback

Next question

Renal tumours

Renal cell carcinoma


Renal cell carcinoma is an adenocarcinoma of the renal cortex and is believed to arise from the
proximal convoluted tubule. They are usually solid lesions, up to 20% may be multifocal, 20% may
be calcified and 20% may have either a cystic component or be wholly cystic. They are often
circumscribed by a pseudocapsule of compressed normal renal tissue. Spread may occur either by
direct extension into the adrenal gland, renal vein or surrounding fascia. More distant disease
usually occurs via the haematogenous route to lung, bone or brain.
Renal cell carcinoma comprise up to 85% of all renal malignancies. Males are more commonly
affected than females and sporadic tumours typically affect patients in their sixth decade.
Patients may present with a variety of symptoms including; haematuria (50%), loin pain (40%), mass
(30%) and up to 25% may have symptoms of metastasis.Less than 10% have the classic triad of
haematuria, pain and mass.

Investigation
Many cases will present as haematuria and be discovered during diagnostic work up. Benign renal
tumours are rare, so renal masses should be investigated with multislice CT scanning. Some units
will add and arterial and venous phase to the scan to demonstrate vascularity and evidence of caval
ingrowth.

CT scanning of the chest and abdomen to detect distant disease should also be undertaken.

Routine bone scanning is not indicated in the absence of symptoms.

Biopsy should not be performed when a nephrectomy is planned but is mandatory before any
ablative therapies are undertaken.

Assessment of the functioning of the contra lateral kidney.

Management
T1 lesions may be managed by partial nephrectomy and this gives equivalent oncological results to
total radical nephrectomy. Partial nephrectomy may also be performed when there is inadequate
reserve in the remaining kidney.

For T2 lesions and above a radical nephrectomy is standard practice and this may be performed via
a laparoscopic or open approach. Preoperative embolisation is not indicated nor is resection of
uninvolved adrenal glands. During surgery early venous control is mandatory to avoid shedding of
tumour cells into the circulation.

Patients with completely resected disease do not benefit from adjuvant therapy with either
chemotherapy or biological agents. These should not be administered outside the setting of clinical
trials.

Patients with transitional cell cancer will require a nephroureterectomy with disconnection of the
ureter at the bladder.

References
Lungberg B et al. EAU guidelines on renal cell carcinoma: The 2010 update. European Urology 2010
(58): 398-406.

Next question
Theme: Haematuria

A. Benign prostatic hyperplasia


B. Transitional cell carcinoma of the bladder
C. Renal cell carcinoma
D. Ureteric calculus
E. Staghorn calculus
F. Uncomplicated urinary tract infection
G. Adenocarcinoma of the bladder

Please select the most likely cause of haematuria from the scenarios given. Each option may be
used once, more than once or not at all.

37. A 58 year old man has an episode of painless frank haematuria whilst undergoing a 24
urine collection for investigation of hypertension.

You answered Benign prostatic hyperplasia

The correct answer is Renal cell carcinoma

These tumours may often have paraneoplastic effects such as hypertension.

38. A 73 year old lady has an episode of haematuria whilst receiving a course of intravesical
BCG therapy.

You answered Benign prostatic hyperplasia

The correct answer is Transitional cell carcinoma of the bladder

Transitional cell carcinoma of the bladder may be treated with intravesical BCG therapy.

39. A 32 year old lady suffers from severe left sided abdominal pain, that radiates to her groin.
As part of her evaluation the nurses identify microscopic haematuria on dipstick.

You answered Benign prostatic hyperplasia

The correct answer is Ureteric calculus

Ureteric calculi will often present with loin pain radiating to the groin. It is usually severe.
There may be macroscopic or microscopic haematuria. The absence of haematuria on
dipstick testing should prompt investigations for alternative diagnoses. The best
investigation is a non contrast CT scan. CT changes consistent with stone or recent stone
passage include evidence of stone, perinephric stranding, ureteric oedema or
hydronephrosis.

Please rate this question:

Discuss and give feedback


Next question

Haematuria

Causes of haematuria

Trauma  Injury to renal tract


 Renal trauma commonly due to blunt injury (others penetrating
injuries)
 Ureter trauma rare: iatrogenic
 Bladder trauma: due to RTA or pelvic fractures

Infection  Remember TB

Malignancy  Renal cell carcinoma (remember paraneoplastic syndromes):


painful or painless
 Urothelial malignancies: 90% are transitional cell carcinoma, can
occur anywhere along the urinary tract. Painless haematuria.
 Squamous cell carcinoma and adenocarcinoma: rare bladder
tumours
 Prostate cancer
 Penile cancers: SCC

Renal disease  Glomerulonephritis

Stones  Microscopic haematuria common

Structural  Benign prostatic hyperplasia (BPH) causes haematuria due to


abnormalities hypervascularity of the prostate gland
 Cystic renal lesions e.g. polycystic kidney disease
 Vascular malformations
 Renal vein thrombosis due to renal cell carcinoma
Coagulopathy  Causes bleeding of underlying lesions

Drugs  Cause tubular necrosis or interstitial nephritis: aminoglycosides,


chemotherapy
 Interstitial nephritis: penicillin, sulphonamides, and NSAIDs
 Anticoagulants

Benign  Exercise

Gynaecological  Endometriosis: flank pain, dysuria, and haematuria that is cyclical

Iatrogenic  Catheterisation
 Radiotherapy; cystitis, severe haemorrhage, bladder necrosis

Pseudohaematuria For example following consumption of beetroot

References
Http://bestpractice.bmj.com/best-practice/monograph/316/overview/aetiology.html
Next question
A 73 year old man has previously undergone a prostatectomy to treat prostate cancer. On review,
his PSA has risen to 55 and he has developed pain in his lower back. Imaging shows osteosclerotic
lesions in L4 and L3. What is the best treatment strategy?

Posterior spinal fusion

Vertebral body reconstruction

Bisphosphonates and radiotherapy

Androgen suppression, bisphosphonates and radiotherapy

Radiotherapy alone

Theme from April 2015 Exam


In men with metastatic bone lesions from prostate cancer, the best outcomes are achieved with
androgen suppression. Radiotherapy can also produced marked palliation. A 2010 Cochrane review
has clearly demonstrated added benefit, in terms of symptom control, from the addition of a
bisphosphonate.
Please rate this question:

Discuss and give feedback


Next question

Prostate Cancer

Prostate Cancer
This is a common condition and up to 30,000 men are diagnosed with the condition each year. Up to
9,000 will die in in the UK from the condition per year.

Diagnosis
Early prostate cancers have few symptoms.
Metastatic disease may present as bone pain.
Locally advanced disease may present as pelvic pain or with urinary symptoms.
Prostate specific antigen measurement
Digital rectal examination
Trans rectal USS (+/- biopsy)
MRI/ CT and bone scan for staging.

PSA Test
The normal upper limit for PSA is 4ng/ml. However, in this group will lie patients with benign disease
and some with localised prostate cancer. False positives may be due to prostatitis, UTI, BPH,
vigorous DRE.
The percentage of free: total PSA may help to distinguish benign disease from cancer. Values of
<20% are suggestive of cancer and biopsy is advised.

Pathology

 95% adenocarcinoma
 In situ malignancy is sometimes found in areas adjacent to cancer. Multiple biopsies needed
to call true in situ disease.
 Often multifocal- 70% lie in the peripheral zone.
 Graded using the Gleason grading system, two grades awarded 1 for most dominant grade
(on scale of 1-5) and 2 for second most dominant grade (scale 1-5). The two added together
give the Gleason score. Where 2 is best prognosis and 10 the worst.
 Lymphatic spread occurs first to the obturator nodes and local extra prostatic spread to the
seminal vesicles is associated with distant disease.

Treatment

 Watch and wait- Elderly, multiple co-morbidities, low Gleason score


 Radiotherapy (External)- Both potentially curative and palliative therapy possible. However,
radiation proctitis and rectal malignancy are late problems. Brachytherapy is a modification
allowing internal radiotherapy.
 Surgery- Radical prostatectomy. Surgical removal of the prostate is the standard treatment
for localised disease. The robot is being used increasingly for this procedure. As well as the
prostate the obturator nodes are also removed to complement the staging process. Erectile
dysfunction is a common side effect. Survival may be better than with radiotherapy (see
references).
 Hormonal therapy- Testosterone stimulates prostate tissue and prostatic cancers usually
show some degree of testosterone dependence. 95% of testosterone is derived from the
testis and bilateral orchidectomy may be used for this reason. Pharmacological alternatives
include LHRH analogues and anti androgens (which may be given in combination).
 In the UK the National Institute for Clinical Excellence (NICE) suggests that active
surveillance is the preferred option for low risk men. It is particularly suitable for men with
clinical stage T1c, Gleason score 3+3 and PSA density < 0.15 ng/ml/ml who have cancer in
less than 50% of their biopsy cores, with < 10 mm of any core involved.

Candidates for active surveillance should:

 have had at least 10 biopsy cores taken


 have at least one re-biopsy.

If men on active surveillance show evidence of disease progression, offer radical treatment.
Treatment decisions should be made with the man, taking into account co-morbidities and life
expectancy.

References
1. Prostate cancer pathway. NICE.(http://guidance.nice.org.uk/IPG424)
2. Sooriakumaran P et al. Comparative effectiveness of radical prostatectomy and radiotherapy in
prostate cancer: observational study of mortality outcomes. BMJ 2014 (348):13. This study shows
that in men with localised disease survival was greater in those offered surgery.
Next question
A 24 year old man presents with a persistent and unwanted erection that has been present for the
previous 6 hours. On examination the penis is rigid and tender. Aspiration of blood from the corpus
cavernosa shows dark blood. Which of the following is the most appropriate initial management?

Discharge the patient home and review in 12 hours

Admit the patient to hospital and review in 12 hours

Aspirate further blood from the corpus cavernosa in an attempt to decompress

Use a trucut needle to induce an arteriovenous shunt

Administer intracavernosal adrenaline 1 in 500 concentration

Low flow priaprism is a urological emergency. Aspiration of bright red blood is more reassuring and
may indicate high flow priaprism that may be actively monitored. Low flow priaprism should be
decompressed with aspiration of blood from the corpus caveronsum.
Please rate this question:

Discuss and give feedback


Next question

Penile erection

Physiology of erection
Autonomic  Sympathetic nerves originate from T11-L2 and parasympathetic nerves
from S2-4 join to form pelvic plexus.
 Parasympathetic discharge causes erection, sympathetic discharge causes
ejaculation and detumescence.

Somatic Supplied by dorsal penile and pudendal nerves. Efferent signals are relayed from
nerves Onufs nucleus (S2-4) to innervate ischiocavernosus and bulbocavernosus muscles.

Autonomic discharge to the penis will trigger the veno-occlusive mechanism which triggers the flow
of arterial blood into the penile sinusoidal spaces. As the inflow increases the increased volume in
this space will secondarily lead to compression of the subtunical venous plexus with reduced venous
return. During the detumesence phase the arteriolar constriction will reduce arterial inflow and
thereby allow venous return to normalise.
Priapism
Prolonged unwanted erection, in the absence of sexual desire, lasting more than 4 hours.

Classification of priaprism
Low flow priaprism Due to veno-occlusion (high intracavernosal pressures).

 Most common type


 Often painful
 Often low cavernosal flow
 If present for >4 hours requires emergency treatment

High flow priaprism Due to unregulated arterial blood flow.

 Usually presents as semi rigid painless erection

Recurrent priaprism Typically seen in sickle cell disease, most commonly of high flow type.

Causes

 Intracavernosal drug therapies (e.g. for erectile dysfunction>


 Blood disorders such as leukaemia and sickle cell disease
 Neurogenic disorders such as spinal cord transection
 Trauma to penis resulting in arterio-venous malformations

Tests

 Exclude sickle cell/ leukaemia


 Consider blood sampling from cavernosa to determine whether high or low flow (low flow is
often hypoxic)

Management

 Ice packs/ cold showers


 If due to low flow then blood may be aspirated from copora or try intracavernosal alpha
adrenergic agonists.
 Delayed therapy of low flow priaprism may result in erectile dysfunction.

Next question
Theme: Management of urinary obstruction

A. Discharge
B. Start oxybutynin
C. Intravenous antibiotics
D. Urethral catheter
E. Emergency nephrostomy
F. Antegrade ureteric stent
G. Retrograde ureteric stent

What is the best management for the scenario given? Each option may be used once, more than
once or not at all.

42. A 68 year old man has a TCC of the bladder. He has a right hydronephrosis detected on
ultrasound and deteriorating renal function. A DMSA scan shows a non functioning left
kidney.

You answered Discharge

The correct answer is Antegrade ureteric stent

A TCC occluding the ureteric orifice will obscure its identification during surgery, so that
passage of a retrograde stent is difficult. Therefore passage of a stent from the renal pelvis
is preferable.

43. A 52 year old male with hypercalcaemia secondary to primary hyperparathyroidism


presents with renal colic. USS demonstrates ureteric obstruction due to a stone. Multiple
attempts at stone extraction are performed. However, the stone could not be removed. He
is now septic with a pyrexia of 39.5 oc.

You answered Discharge

The correct answer is Emergency nephrostomy

The likely scenario is that this man has developed a calculus causing ureteric obstruction.
The stagnant column of urine can become colonised and infected. An infected obstructed
system is one of the few true urological emergencies. A nephrostomy is needed as the
stone could not be removed.

44. A 56 year old man is admitted with acute retention of urine. He has had a recent urinary
tract infection. An USS shows bilateral hydronephrosis.

You answered Discharge


The correct answer is Urethral catheter

Establishing bladder drainage will often correct the situation. These patients often have a
significant diuresis with associated electrolyte disturbance.
Antegrade ureteric stents pass from the kidney to the bladder.
Retrograde stents pass from the bladder to the kidney

Please rate this question:

Discuss and give feedback


Next question

Hydronephrosis

Causes of hydronephrosis

Unilateral: PACT

 Pelvic-ureteric obstruction (congenital or acquired)


 Aberrant renal vessels
 Calculi
 Tumours of renal pelvis

Bilateral: SUPER

 Stenosis of the urethra


 Urethral valve
 Prostatic enlargement
 Extensive bladder tumour
 Retro-peritoneal fibrosis

Investigation

 USS- identifies presence of hydronephrosis and can assess the kidneys


 IVU- assess the position of the obstruction
 Antegrade or retrograde pyelography- allows treatment
 If renal colic suspected: non contrast CT scan (majority of stones are detected this way)

Management
 Remove the obstruction and drainage of urine
 Acute upper urinary tract obstruction: Nephrostomy tube
 Chronic upper urinary tract obstruction: Ureteric stent or a pyeloplasty

Next question
Theme: Haematuria

A. Transitional cell carcinoma of the bladder


B. Squamous cell carcinoma of the bladder
C. Renal cell carcinoma
D. Polycystic kidney disease
E. Wilms tumour
F. Neuroblastoma
G. Benign prostatic hyperplasia

Please select the most likely cause of haematuria for the scenarios given. Each option may be used
once, more than once or not at all.

45. A 40 year old women is being investigated for haematuria. She was living with her sister
who has just died from a sub arachnoid haemorrhage. The haematuria is painless and she
has mild renal impairment.

You answered Transitional cell carcinoma of the bladder

The correct answer is Polycystic kidney disease

This is likely to be polycystic kidney disease as she has renal failure and family history of
sub arachnoid haemorrhage.

46. A 75 year old lady is investigated for episodes of painless haematuria. Apart from COPD
from long term smoking she is otherwise well. She has no other urinary symptoms.

Transitional cell carcinoma of the bladder

TCC commonly presents with painless haematuria that may be detected during testing
carried out for other reasons.

47. A 78 year old man has a long history of nocturia, urinary frequency and terminal dribbling.
He was admitted with urinary retention and was catheterised. On removal of the catheter
he has noticed some haematuria.

You answered Transitional cell carcinoma of the bladder

The correct answer is Benign prostatic hyperplasia

The symptoms are typical for prostatic disease.

Please rate this question:


Discuss and give feedback
Next question

Haematuria

Causes of haematuria

Trauma  Injury to renal tract


 Renal trauma commonly due to blunt injury (others penetrating
injuries)
 Ureter trauma rare: iatrogenic
 Bladder trauma: due to RTA or pelvic fractures

Infection  Remember TB

Malignancy  Renal cell carcinoma (remember paraneoplastic syndromes):


painful or painless
 Urothelial malignancies: 90% are transitional cell carcinoma, can
occur anywhere along the urinary tract. Painless haematuria.
 Squamous cell carcinoma and adenocarcinoma: rare bladder
tumours
 Prostate cancer
 Penile cancers: SCC

Renal disease  Glomerulonephritis

Stones  Microscopic haematuria common

Structural  Benign prostatic hyperplasia (BPH) causes haematuria due to


abnormalities hypervascularity of the prostate gland
 Cystic renal lesions e.g. polycystic kidney disease
 Vascular malformations
 Renal vein thrombosis due to renal cell carcinoma

Coagulopathy  Causes bleeding of underlying lesions

Drugs  Cause tubular necrosis or interstitial nephritis: aminoglycosides,


chemotherapy
 Interstitial nephritis: penicillin, sulphonamides, and NSAIDs
 Anticoagulants

Benign  Exercise

Gynaecological  Endometriosis: flank pain, dysuria, and haematuria that is cyclical

Iatrogenic  Catheterisation
 Radiotherapy; cystitis, severe haemorrhage, bladder necrosis

Pseudohaematuria For example following consumption of beetroot

References
Http://bestpractice.bmj.com/best-practice/monograph/316/overview/aetiology.html
Next question
A 65 year old man presents with significant lower urinary tract symptoms and is diagnosed as having
benign prostatic hyperplasia. Which of the following drug treatments will produce the slowest clinical
response?

Tamsulosin

Alfuzosin

Doxazosin

Finasteride

Terazosin

5 alpha reductase inhibitors have a more favorable side effect profile than α blockers.

Alpha blockers have a faster onset of action (but lower reduction of complications from BPH) than 5
α reductase inhibitors.
Please rate this question:

Discuss and give feedback


Next question

Benign Prostatic Hyperplasia

Benign prostatic hyperplasia occurs via an increase in the epithelial and stromal cell numbers in the
peri-urethral zone of the prostate. BPH is very common and 90% of men aged over 80 will have at
least microscopic evidence of benign prostatic hyperplasia. The causes of BPH are still not well
understood, but the importance of androgens remains appreciated even if the exact role by which
they induce BPH is elusive.

Presentation
The vast majority of men will present with lower urinary tract symptoms. These will typically be:

 Poor flow
 Nocturia
 Hesitancy
 Incomplete and double voiding
 Terminal dribbling
 Urgency
 Incontinence

Investigation

 Digital rectal examination to assess prostatic size and morphology.


 Urine dipstick for infections and haematuria.
 Uroflowmetry (a flow rate of >15ml/second helps to exclude BOO)
 Bladder pressure studies may help identify detrusor failure and whilst may not form part of
first line investigations should be included in those with atypical symptoms and prior to redo
surgery.
 Bladder scanning to demonstrate residual volumes. USS if high pressure chronic retention.

Management

 Lifestyle changes such as stopping smoking and altering fluid intake may help those with
mild symptoms.
 Medical therapy includes alpha blockers and 5 α reductase inhibitors. The former work
quickly on receptor zones located at the bladder neck. Cardiovascular side effects are well
documented. The latter work on testosterone metabolising enzymes. Although they have a
slower onset of action, the 5 α reductase inhibitors may prevent acute urinary retention.
 Surgical therapy includes transurethral resection of the prostate and is the treatment of
choice in those with severe symptoms and those who fail to respond to medical therapy.
More tailored bladder neck incision procedures may be considered in those with small
prostates. Retrograde ejaculation may occur following surgery. The change in the type of
irrigation solutions used has helped to minimise the TURP syndrome of electrolyte
disturbances.

Next question
Theme: Testicular disorders

A. Haematocele
B. Epididymal cyst
C. Hydrocele
D. Testicular torsion
E. Orchitis
F. Epididymo-orchitis

Please select the most likely diagnosis for the scenario given. Each option may be used once, more
than once or not at all.

49. A 20 year old complains of severe pain and swelling of the scrotum after a cystoscopy. He
had mumps as a child. The testis is tender. The urine dipstick is positive for leucocytes.

You answered Haematocele

The correct answer is Epididymo-orchitis

Epididymo-orchitis: acute pain and swelling after urological intervention. To differentiate


from testicular torsion there is usually pyrexia and positive urine dipstick.

50. A 20 year old complains of severe pain in the right scrotal area after jumping onto his
moped. He has also noticed discomfort in this area over the past few months. On
examination there is a swollen, painful testis that is drawn up into the groin.

You answered Haematocele

The correct answer is Testicular torsion

Testicular torsion: Severe pain which can be spontaneous or precipitated by minor trauma.
There is usually severe pain and the patient will often not tolerate the testis being touched.
Urgent scrotal exploration is indicated. It is associated with a high investment of the the
tunica vaginalis with horizontal testicular lie, or when the epididymis and testis are
separated by a mesorchium, in which case the twist occurs at that point.

51. An 8 year old presents with scrotal swelling. He has just recovered from an acute viral
illness with swelling of the parotid glands. On examination both testes are tender and
slightly swollen.

You answered Haematocele

The correct answer is Orchitis


Orchitis may be associated with mumps viral infections.

Please rate this question:

Discuss and give feedback


Next question

Scrotal swelling

Differential diagnosis
Inguinal hernia If inguinoscrotal swelling; cannot "get above it" on examination
Cough impulse may be present
May be reducible

Testicular tumours Often discrete testicular nodule (may have associated hydrocele)
Symptoms of metastatic disease may be present
USS scrotum and serum AFP and β HCG required

Acute epididymo- Often history of dysuria and urethral discharge


orchitis Swelling may be tender and eased by elevating testis
Most cases due to Chlamydia
Infections with other gram negative organisms may be associated with
underlying structural abnormality

Epididymal cysts Single or multiple cysts


May contain clear or opalescent fluid (spermatoceles)
Usually occur over 40 years of age
Painless
Lie above and behind testis
It is usually possible to "get above the lump" on examination

Hydrocele Non painful, soft fluctuant swelling


Often possible to "get above it" on examination
Usually contain clear fluid
Will often transilluminate
May be presenting feature of testicular cancer in young men
Testicular torsion Severe, sudden onset testicular pain
Risk factors include abnormal testicular lie
Typically affects adolescents and young males
On examination testis is tender and pain not eased by elevation
Urgent surgery is indicated, the contra lateral testis should also be fixed

Varicocele Varicosities of the pampiniform plexus


Typically occur on left (because testicular vein drains into renal vein)
May be presenting feature of renal cell carcinoma
Affected testis may be smaller and bilateral varicoceles may affect fertility

Management

 Testicular malignancy is always treated with orchidectomy via an inguinal approach. This
allows high ligation of the testicular vessels and avoids exposure of another lymphatic field to
the tumour.
 Torsion is commonest in young teenagers and the history in older children can be difficult to
elicit. Intermittent torsion is a recognised problem. The treatment is prompt surgical
exploration and testicular fixation. This can be achieved using sutures or by placement of the
testis in a Dartos pouch.
 Varicoceles are usually managed conservatively. If there are concerns about testicular
function of infertility then surgery or radiological management can be considered.
 Epididymal cysts can be excised using a scrotal approach
 Hydroceles are managed differently in children where the underlying pathology is a patent
processus vaginalis and therefore an inguinal approach is used in children so that the
processus can be ligated. In adults a scrotal approach is preferred and the hydrocele sac
excised or plicated.

Next question
Theme: Renal imaging

A. Non contrast abdominal CT scan

B. DMSA scan

C. PET/CT scan

D. MAG 3 Renogram

E. Renal ultrasound scan

F. DTPA Scan

G. Micturating cystourethrogram

H. Intra venous urography

Please select the most appropriate imaging modality for the scenario descrived. Each agent may be
used once, more than once or not at all.

52. A 43 year old female has undergone a renal transplant 12 months previously. Over the past few
weeks there have been concerns about deteriorating renal function.

You answered Non contrast abdominal CT scan

The correct answer is MAG 3 Renogram

Theme from 2010 Exam


Similar theme September 2012 Exam
Because it is excreted by renal tubular cells a MAG 3 renogram provides excellent imaging of renal
function and is often used in investigating failing transplants.

53. A 5 year old boy presents with recurrent urinary tract infections and left sided loin pain. On
investigation he is found to have a left sided PUJ obstruction, there are concerns that he may
have developed renal scarring.

You answered Non contrast abdominal CT scan


The correct answer is DMSA scan

Although MAG 3 renograms may provide some information relating to the structural integrity of
the kidney, many still consider a DMSA scan to be the gold standard for the detection of renal
scarring (which is the main concern in PUJ obstruction and infections).

54. A 17 year old man is referred to the urology clinic. As a child he was diagnosed as having a right
sided PUJ obstruction. However, he was lost to follow up. Over the past 7 months he has been
complaining of recurrent episodes of right loin pain. A CT scan shows considerable renal scarring.

You answered Non contrast abdominal CT scan

The correct answer is MAG 3 Renogram

In patients with long standing PUJ obstruction and renal scarring the main diagnostic question is
whether the individual has sufficient renal function to consider a pyeloplasty or whether a
primary nephrectomy is preferable. Since the CT has demonstrated scarring there is no use in
obtaining a DMSA scan. Of the investigations listed both a DTPA and MAG 3 renogram will allow
assessment of renal function. However, MAG 3 is superior in the assessment of renal function in
damaged kidneys (as it is subjected to tubular secretion).

Please rate this question:

Discuss and give feedback

Next question

Functional renal imaging

DMSA scan
Dimercaptosuccinic acid (DMSA) scintigraphy
DMSA localises to the renal cortex with little accumulation in the renal papilla and medulla. It is
useful for the identification of cortical defects and ectopic or aberrant kidneys. It does not provide
useful information on the ureter of collecting system.

Diethylene-triamine-penta-acetic acid (DTPA)


This is primarily a glomerular filtration agent. It is most useful for the assessment of renal function.
Because it is filtered at the level of the glomerulus it provides useful information about the GFR.
Image quality may be degraded in patients with chronic renal impairment and derangement of GFR.

MAG 3 renogram
Mercaptoacetyle triglycine is an is extensively protein bound and is primarily secreted by tubular
cells rather than filtered at the glomerulus. This makes it the agent of choice for imaging the kidneys
of patients with existing renal impairment (where GFR is impaired).

Micturating cystourethrogram (MCUG scan)


This scan provides information relating to bladder reflux and is obtained by filling the bladder with
contrast media (via a catheter) and asking the child to void. Images are taken during this phase and
the degree of reflux can be calculated

Intra venous urography


This examination is conducted by the administration of intravenous iodinated contrast media. The
agent is filtered by the kidneys and excreted and may provide evidence of renal stones or other
structural lesions. A rough approximation of renal function may be obtained using the technique. But
it is not primarily a technique to be used for this purpose. With the advent of widespread non
contrast CT scan protocols for the detection of urinary tract calculi it is now rarely used.

PET/CT
This may be used to evaluate structurally indeterminate lesions in the staging of malignancy.

References
Davis A et al. Investigating urinary tract infections in children. BMJ 2013 (346):35-37.

Next question
A 35-year-old female is admitted to hospital with hypovolaemic shock. CT abdomen reveals a
haemorrhagic lesion in the right kidney. Following surgery and biopsy this is shown to be an
angiomyolipomata. What is the most likely underlying diagnosis?

Neurofibromatosis

Budd-Chiari syndrome

Hereditary haemorrhagic telangiectasia

Von Hippel-Lindau syndrome

Tuberous sclerosis

Please rate this question:

Discuss and give feedback


Next question

Tuberous sclerosis

Tuberous sclerosis (TS) is a genetic condition of autosomal dominant inheritance. Like


neurofibromatosis, the majority of features seen in TS are neuro-cutaneous

Cutaneous features

 depigmented 'ash-leaf' spots which fluoresce under UV light


 roughened patches of skin over lumbar spine (Shagreen patches)
 adenoma sebaceum: butterfly distribution over nose
 fibromata beneath nails (subungual fibromata)
 café-au-lait spots* may be seen

Neurological features

 developmental delay
 epilepsy (infantile spasms or partial)
 intellectual impairment
Also

 retinal hamartomas: dense white areas on retina (phakomata)


 rhabdomyomas of the heart
 gliomatous changes can occur in the brain lesions
 polycystic kidneys, renal angiomyolipomata

*these of course are more commonly associated with neurofibromatosis. However a 1998 study of
106 children with TS found café-au-lait spots in 28% of patients
Next question
A 22 year old man is participating in vigorous intercourse and suddenly feels a snap and his penis
becomes swollen and painful. The admitting surgeon suspects a penile fracture. Which of the
following is the most appropriate initial management?

MRI scan of the penis

Immediate surgical exploration

CT scan of the penis

USS of the penis

Cystogram

Suspected penile fractures should be surgically explored and the injury repaired.

Please rate this question:

Discuss and give feedback

Next question

Penile fracture

Penile fractures are a rare type of urological trauma that may be encountered. The injury is usually in
the proximal part of the penile shaft and may involve the urethra. A classically history of a snapping
sensation followed by immediate pain is usually given by the patient (usually during vigorous
intercourse). On examination there is usually a tense haematoma and blood may be seen at the
meatus if the urethra is injured.
When there is a a strong suspicion of the diagnosis the correct management is surgical and a
circumferential incision made immediately inferior to the glans. The skin and superficial tissues are
stripped back and the penile shaft inspected. Injuries are usually sutured and the urethra repaired
over a catheter.

Next question
Which of the following does not cause red urine?

Rifampicin

Phosphaturia

Beetroot

Rhubarb

Blackberries

Phosphaturia causes cloudy urine.


Please rate this question:

Discuss and give feedback


Next question

Haematuria

Causes of haematuria

Trauma  Injury to renal tract


 Renal trauma commonly due to blunt injury (others penetrating
injuries)
 Ureter trauma rare: iatrogenic
 Bladder trauma: due to RTA or pelvic fractures

Infection  Remember TB

Malignancy  Renal cell carcinoma (remember paraneoplastic syndromes):


painful or painless
 Urothelial malignancies: 90% are transitional cell carcinoma, can
occur anywhere along the urinary tract. Painless haematuria.
 Squamous cell carcinoma and adenocarcinoma: rare bladder
tumours
 Prostate cancer
 Penile cancers: SCC

Renal disease  Glomerulonephritis

Stones  Microscopic haematuria common

Structural  Benign prostatic hyperplasia (BPH) causes haematuria due to


abnormalities hypervascularity of the prostate gland
 Cystic renal lesions e.g. polycystic kidney disease
 Vascular malformations
 Renal vein thrombosis due to renal cell carcinoma

Coagulopathy  Causes bleeding of underlying lesions

Drugs  Cause tubular necrosis or interstitial nephritis: aminoglycosides,


chemotherapy
 Interstitial nephritis: penicillin, sulphonamides, and NSAIDs
 Anticoagulants

Benign  Exercise

Gynaecological  Endometriosis: flank pain, dysuria, and haematuria that is cyclical

Iatrogenic  Catheterisation
 Radiotherapy; cystitis, severe haemorrhage, bladder necrosis

Pseudohaematuria For example following consumption of beetroot

References
Http://bestpractice.bmj.com/best-practice/monograph/316/overview/aetiology.html
Next question
From the list below, which drug is known to cause haemorrhagic cystitis?

Rifampicin

Methotrexate

Dexamethasone

Leflunomide

Cyclophosphamide

Cyclophosphamide is metabolised into a toxic metabolite acrolein. The effects may be attenuated by
administration of large volumes of intravenous fluids and mesna (which neutralises the metabolite).
The condition may be managed initially by bladder catheterisation and irrigation.
Please rate this question:

Discuss and give feedback


Next question

Haematuria

Causes of haematuria

Trauma  Injury to renal tract


 Renal trauma commonly due to blunt injury (others penetrating
injuries)
 Ureter trauma rare: iatrogenic
 Bladder trauma: due to RTA or pelvic fractures

Infection  Remember TB

Malignancy  Renal cell carcinoma (remember paraneoplastic syndromes):


painful or painless
 Urothelial malignancies: 90% are transitional cell carcinoma, can
occur anywhere along the urinary tract. Painless haematuria.
 Squamous cell carcinoma and adenocarcinoma: rare bladder
tumours
 Prostate cancer
 Penile cancers: SCC

Renal disease  Glomerulonephritis

Stones  Microscopic haematuria common

Structural  Benign prostatic hyperplasia (BPH) causes haematuria due to


abnormalities hypervascularity of the prostate gland
 Cystic renal lesions e.g. polycystic kidney disease
 Vascular malformations
 Renal vein thrombosis due to renal cell carcinoma

Coagulopathy  Causes bleeding of underlying lesions

Drugs  Cause tubular necrosis or interstitial nephritis: aminoglycosides,


chemotherapy
 Interstitial nephritis: penicillin, sulphonamides, and NSAIDs
 Anticoagulants

Benign  Exercise

Gynaecological  Endometriosis: flank pain, dysuria, and haematuria that is cyclical

Iatrogenic  Catheterisation
 Radiotherapy; cystitis, severe haemorrhage, bladder necrosis

Pseudohaematuria For example following consumption of beetroot

References
Http://bestpractice.bmj.com/best-practice/monograph/316/overview/aetiology.html
Next question
Theme: Management of prostatic disease

A. Commence alpha blocker


B. Commence 5 alpha reductase inhibitor
C. Transurethral resection of the prostate
D. Commence LnRH analogue
E. Radical prostatectomy
F. Transvesical prostatectomy
G. Unilateral orchidectomy
H. Watch and wait
I. Radical radiotherapy

For the prostatic disorders described please select the most appropriate management option. Each
option may be used once, more than once or not at all.

59. A 49 year old man presents with a single episode of haematuria. Investigations
demonstrate adenocarcinoma of the prostate gland. Imaging shows T2 disease and no
evidence of metastasis.

You answered Commence alpha blocker

The correct answer is Radical prostatectomy

In a young patient with local disease only a radical prostatectomy is the best chance of
cure. Radiotherapy may be given instead but has long term sequelae (and inferior survival
outcomes). A transvesical prostatectomy is a largely historical operation performed for
BPH before TURP was established.

60. A 72 year old man is admitted with acute urinary retention. On examination he has a small
but palpable bladder. Digital rectal examination identifies a benign feeling enlarged
prostate gland. He has been treated with finasteride for the past 9 months.

You answered Commence alpha blocker

The correct answer is Transurethral resection of the prostate

Medical therapy has failed and although an alpha blocker may help his symptoms he
would fare better with a TURP.

61. A 73 year old man presents with haematuria. Investigations demonstrate a localised, high
risk, prostatic cancer. His co-morbidities include COPD and ischaemic heart disease. His
staging investigations show no evidence of metastatic disease
You answered Commence alpha blocker

The correct answer is Radical radiotherapy

The co-morbidities of this patient make a surgical approach a less favorable option.
Radical radiotherapy offers a more favorable alternative.

Please rate this question:

Discuss and give feedback


Next question

Prostate Cancer

Prostate Cancer
This is a common condition and up to 30,000 men are diagnosed with the condition each year. Up to
9,000 will die in in the UK from the condition per year.

Diagnosis
Early prostate cancers have few symptoms.
Metastatic disease may present as bone pain.
Locally advanced disease may present as pelvic pain or with urinary symptoms.
Prostate specific antigen measurement
Digital rectal examination
Trans rectal USS (+/- biopsy)
MRI/ CT and bone scan for staging.

PSA Test
The normal upper limit for PSA is 4ng/ml. However, in this group will lie patients with benign disease
and some with localised prostate cancer. False positives may be due to prostatitis, UTI, BPH,
vigorous DRE.
The percentage of free: total PSA may help to distinguish benign disease from cancer. Values of
<20% are suggestive of cancer and biopsy is advised.

Pathology

 95% adenocarcinoma
 In situ malignancy is sometimes found in areas adjacent to cancer. Multiple biopsies needed
to call true in situ disease.
 Often multifocal- 70% lie in the peripheral zone.
 Graded using the Gleason grading system, two grades awarded 1 for most dominant grade
(on scale of 1-5) and 2 for second most dominant grade (scale 1-5). The two added together
give the Gleason score. Where 2 is best prognosis and 10 the worst.
 Lymphatic spread occurs first to the obturator nodes and local extra prostatic spread to the
seminal vesicles is associated with distant disease.
Treatment

 Watch and wait- Elderly, multiple co-morbidities, low Gleason score


 Radiotherapy (External)- Both potentially curative and palliative therapy possible. However,
radiation proctitis and rectal malignancy are late problems. Brachytherapy is a modification
allowing internal radiotherapy.
 Surgery- Radical prostatectomy. Surgical removal of the prostate is the standard treatment
for localised disease. The robot is being used increasingly for this procedure. As well as the
prostate the obturator nodes are also removed to complement the staging process. Erectile
dysfunction is a common side effect. Survival may be better than with radiotherapy (see
references).
 Hormonal therapy- Testosterone stimulates prostate tissue and prostatic cancers usually
show some degree of testosterone dependence. 95% of testosterone is derived from the
testis and bilateral orchidectomy may be used for this reason. Pharmacological alternatives
include LHRH analogues and anti androgens (which may be given in combination).
 In the UK the National Institute for Clinical Excellence (NICE) suggests that active
surveillance is the preferred option for low risk men. It is particularly suitable for men with
clinical stage T1c, Gleason score 3+3 and PSA density < 0.15 ng/ml/ml who have cancer in
less than 50% of their biopsy cores, with < 10 mm of any core involved.

Candidates for active surveillance should:

 have had at least 10 biopsy cores taken


 have at least one re-biopsy.

If men on active surveillance show evidence of disease progression, offer radical treatment.
Treatment decisions should be made with the man, taking into account co-morbidities and life
expectancy.

References
1. Prostate cancer pathway. NICE.(http://guidance.nice.org.uk/IPG424)
2. Sooriakumaran P et al. Comparative effectiveness of radical prostatectomy and radiotherapy in
prostate cancer: observational study of mortality outcomes. BMJ 2014 (348):13. This study shows
that in men with localised disease survival was greater in those offered surgery.
Next question
Theme: Haematuria

A. Squamous cell carcinoma of the renal pelvis


B. Renal adenocarcinoma
C. Nephroblastoma
D. Retroperitoneal fibrosis
E. Transitional cell carcinoma of the renal pelvis
F. Retroperitoneal sarcoma

Please select the most likely lesion for the scenario given. Each option may be used once, more
than once or not at all.

62. A 72 year old man presents with haematuria which is recurrent. On investigation a
retrograde pyelogram shows multiple ureteric filling defects and the renal pelvis is
irregular.

You answered Squamous cell carcinoma of the renal pelvis

The correct answer is Transitional cell carcinoma of the renal pelvis

TCC of the renal pelvis may seed down the ureter.

63. An 83 year old man with a long standing staghorn calculus presents with recurrent
haematuria and investigation shows a mass of the left renal pelvis.

Squamous cell carcinoma of the renal pelvis

SCC of the kidney usually arises in an area of chronic inflammation such as a staghorn
calculus.

64. A 28 year old man presents to his GP with haematuria and on examination is noted to have
a varicocele. He was noted to have renal colic 8 weeks ago which was secondary to
hypercalcaemia.

You answered Squamous cell carcinoma of the renal pelvis

The correct answer is Renal adenocarcinoma

Renal adenocarcinoma on the left side may invade the gonadal vein and produce
varicocele. They also have paraneoplastic phenomena such as hypercalcaemia.

Please rate this question:


Discuss and give feedback
Next question

Haematuria

Causes of haematuria

Trauma  Injury to renal tract


 Renal trauma commonly due to blunt injury (others penetrating
injuries)
 Ureter trauma rare: iatrogenic
 Bladder trauma: due to RTA or pelvic fractures

Infection  Remember TB

Malignancy  Renal cell carcinoma (remember paraneoplastic syndromes):


painful or painless
 Urothelial malignancies: 90% are transitional cell carcinoma, can
occur anywhere along the urinary tract. Painless haematuria.
 Squamous cell carcinoma and adenocarcinoma: rare bladder
tumours
 Prostate cancer
 Penile cancers: SCC

Renal disease  Glomerulonephritis

Stones  Microscopic haematuria common

Structural  Benign prostatic hyperplasia (BPH) causes haematuria due to


abnormalities hypervascularity of the prostate gland
 Cystic renal lesions e.g. polycystic kidney disease
 Vascular malformations
 Renal vein thrombosis due to renal cell carcinoma

Coagulopathy  Causes bleeding of underlying lesions

Drugs  Cause tubular necrosis or interstitial nephritis: aminoglycosides,


chemotherapy
 Interstitial nephritis: penicillin, sulphonamides, and NSAIDs
 Anticoagulants

Benign  Exercise

Gynaecological  Endometriosis: flank pain, dysuria, and haematuria that is cyclical

Iatrogenic  Catheterisation
 Radiotherapy; cystitis, severe haemorrhage, bladder necrosis

Pseudohaematuria For example following consumption of beetroot

References
Http://bestpractice.bmj.com/best-practice/monograph/316/overview/aetiology.html
Next question
Theme: Urinary incontinence

A. Bladder diary for 3 days


B. Urodynamic studies
C. Bladder drill training for 6 weeks
D. Pelvic floor exercises 3 months
E. Oxybutynin
F. IV urography
G. Urinary dye studies
H. None of the above

Choose the best management option for each clinical scenario. Each option may be used once,
more than once or not at all.

65. A 34 year old woman from Africa presents with continuous dribbling incontinence after
having her 2nd child. Apart from prolonged labour the woman denies any complications
related to her pregnancies. She is normally fit and well.

You answered Bladder diary for 3 days

The correct answer is Urinary dye studies

Vesicovaginal fistulae should be suspected in patients with continuous dribbling


incontinence after prolonged labour and from a country with poor obstetric services. A dye
stains the urine and hence identifies the presence of a fistula.

66. A 53 year old lady complains of involuntary passage of urine when she coughs or sneezes.
She is multiparous G2, P2 with a forceps delivery of her second child.

You answered Bladder diary for 3 days

The correct answer is Pelvic floor exercises 3 months

The first line management of stress urinary incontinence, in this case arising for pelvic
floor trauma is pelvic floor exercises. These will help a proportion of patients. Non
responders should have urodynamics performed to confirm the diagnosis.

67. A 56 year old lady reports incontinence mainly when walking the dog. A bladder diary is
inconclusive.

You answered Bladder diary for 3 days

The correct answer is Urodynamic studies


Urodynamic studies are indicated when there is diagnostic uncertainty or plans for surgery.

Please rate this question:

Discuss and give feedback


Next question

Urinary incontinence

Involuntary passage of urine. Most cases are female (80%). It has a prevalence of 11% in those
aged greater than 65 years. The commonest variants include:

 Stress urinary incontinence (50%)


 Urge incontinence (15%)
 Mixed (35%)

Males
Males may also suffer from incontinence although it is a much rarer condition in men. A number of
anatomical factors contribute to this. Males have 2 powerful sphincters; one at the bladder neck and
the other in the urethra. Damage to the bladder neck mechanism is a factor in causing retrograde
ejaculation following prostatectomy. The short segment of urethra passing through the urogenital
diaphragm consists of striated muscle fibres (the external urethral sphincter) and smooth muscle
capable of more sustained contraction. It is the latter mechanism that maintains continence following
prostatectomy.

Females
The sphincter complex at the level of bladder neck is poorly developed in females. As a result the
external sphincter complex is functionally more important, its composition being similar to that of
males. Innervation is via the pudendal nerve and the neuropathy that may accompany obstetric
events may compromise this and lead to stress urinary incontinence.

Innervation
Somatic innervation to the bladder is via the pudendal, hypogastric and pelvic nerves. Autonomic
nerves travel in these nerve fibres too. Bladder filling leads to detrusor relaxation (sympathetic)
coupled with sphincter contraction. The parasympathetic system causes detrusor contraction and
sphincter relaxation. Overall control of micturition is centrally mediated via centres in the Pons.

Stress urinary incontinence

 50% of cases, especially in females.


 Damage (often obstetric) to the supporting structures surrounding the bladder may lead to
urethral hypermobility.
 Other cases due to sphincter dysfunction, usually from neurological disorders (e.g. Pudendal
neuropathy, multiple sclerosis).
Urethral mobility:
Pressure not transmitted appropriately to the urethra resulting in involuntary passage of urine during
episodes of raised intra-abdominal pressure.

Sphincter dysfunction:
Sphincter fails to adapt to compress urethra resulting in involuntary passage of urine. When the
sphincter completely fails there is often to continuous passage of urine.

Urge incontinence
In these patients there is sense of urgency followed by incontinence. The detrusor muscle in these
patients is unstable and urodynamic investigation will demonstrate overactivity of the detrusor
muscle at inappropriate times (e.g. Bladder filling). Urgency may be seen in patients with overt
neurological disorders and those without. The pathophysiology is not well understood but poor
central and peripheral co-ordination of the events surrounding bladder filling are the main
processes.

Assessment
Careful history and examination including vaginal examination for cystocele.
Bladder diary for at least 3 days
Consider flow cystometry if unclear symptomatology or surgery considered and diagnosis is unclear.
Exclusion of other organic disease (e.g. Stones, UTI, Cancer)

Management
Conservative measures should be tried first; Stress urinary incontinence or mixed symptoms should
undergo 3 months of pelvic floor exercise. Over active bladder should have 6 weeks of bladder
retraining.
Drug therapy for women with overactive bladder should be offered oxybutynin (or solifenacin if
elderly) if conservative measures fail.
In women with detrusor instability who fail non operative therapy a trial of sacral neuromodulation
may be considered, with conversion to permanent implant if good response. Augmentation
cystoplasty is an alternative but will involve long term intermittent self catheterisation.
In women with stress urinary incontinence a urethral sling type procedure may be undertaken.
Where cystocele is present in association with incontinence it should be repaired particularly if it lies
at the introitus.

NICE guidelines

 Initial assessment urinary incontinence should be classified as stress/urge/mixed.


 At least 3/7 bladder diary if unable to classify easily.
 Start conservative treatment before urodynamic studies if a diagnosis is obvious from the
history
 Urodynamic studies if plans for surgery.
 Stress incontinence: Pelvic floor exercises 3/12, if fails consider surgery.
 Urge incontinence: Bladder training >6/52, if fails for oxybutynin (antimuscarinic drugs) then
sacral nerve stimulation.
 Pelvic floor exercises offered to all women in their 1st pregnancy.

Next question
A 72 year old man presents with lower urinary tract symptoms. On digital rectal examination, benign
prostatic hyperplasia is suspected. Which of the following treatments is associated with a reduction
in the risk of urinary retention?

Alfuzosin

Finasteride

Prazosin

Tamsulosin

Terazosin

5 alpha reductase inhibitors reduce the risk of urinary retention.


In the PLESS study, data show a reduction in the risk of urinary retention although the absolute risk
reduction was small.

Reference
McConnell J et al. The effect of finasteride on the risk of urinary retention and the need for surgical
intervention amongst men with benign prostatic hyperplasia. N Engl J Med 338:557-563
Please rate this question:

Discuss and give feedback


Next question

Benign Prostatic Hyperplasia

Benign prostatic hyperplasia occurs via an increase in the epithelial and stromal cell numbers in the
peri-urethral zone of the prostate. BPH is very common and 90% of men aged over 80 will have at
least microscopic evidence of benign prostatic hyperplasia. The causes of BPH are still not well
understood, but the importance of androgens remains appreciated even if the exact role by which
they induce BPH is elusive.

Presentation
The vast majority of men will present with lower urinary tract symptoms. These will typically be:

 Poor flow
 Nocturia
 Hesitancy
 Incomplete and double voiding
 Terminal dribbling
 Urgency
 Incontinence

Investigation

 Digital rectal examination to assess prostatic size and morphology.


 Urine dipstick for infections and haematuria.
 Uroflowmetry (a flow rate of >15ml/second helps to exclude BOO)
 Bladder pressure studies may help identify detrusor failure and whilst may not form part of
first line investigations should be included in those with atypical symptoms and prior to redo
surgery.
 Bladder scanning to demonstrate residual volumes. USS if high pressure chronic retention.

Management

 Lifestyle changes such as stopping smoking and altering fluid intake may help those with
mild symptoms.
 Medical therapy includes alpha blockers and 5 α reductase inhibitors. The former work
quickly on receptor zones located at the bladder neck. Cardiovascular side effects are well
documented. The latter work on testosterone metabolising enzymes. Although they have a
slower onset of action, the 5 α reductase inhibitors may prevent acute urinary retention.
 Surgical therapy includes transurethral resection of the prostate and is the treatment of
choice in those with severe symptoms and those who fail to respond to medical therapy.
More tailored bladder neck incision procedures may be considered in those with small
prostates. Retrograde ejaculation may occur following surgery. The change in the type of
irrigation solutions used has helped to minimise the TURP syndrome of electrolyte
disturbances.

Next question
A 22 year old man is involved in a road traffic accident. He is found to have a pelvic fracture. While
on the ward the nursing staff report that he is complaining of lower abdominal pain. On examination
you find a distended tender bladder. What is the best management?

10 Ch foley urethral catheter

Suprapubic catheter

16 Ch foley urethral catheter

18 Ch coude tip urethral catheter

Pain relief and review in 1 hour

This patient has possible urethral injury based on the history. Urethral catheterisation is
contraindicated in this situation.
Please rate this question:

Discuss and give feedback


Next question

Lower genitourinary tract trauma

 Most bladder injuries occur due to blunt trauma


 85% associated with pelvic fractures
 Easily overlooked during assessment in trauma
 Up to 10% of male pelvic fractures are associated with urethral or bladder injuries

Types of injury

Urethral injury  Mainly in males


 Blood at the meatus (50% cases)
 There are 2 types:

i.Bulbar rupture
- most common
- straddle type injury e.g. bicycles
- triad signs: urinary retention, perineal haematoma, blood at the
meatus
ii. Membranous rupture
- can be extra or intraperitoneal
- commonly due to pelvic fracture
- Penile or perineal oedema/ hematoma
- PR: prostate displaced upwards (beware co-existing
retroperitoneal haematomas as they may make examination
difficult)

- Investigation: ascending urethrogram


- Management: suprapubic catheter (surgical placement, not
percutaneously)

External genitalia injuries  Secondary to injuries caused by penetration, blunt trauma,


(i.e., the penis and the continence- or sexual pleasure-enhancing devices, and
scrotum) mutilation

Bladder injury  rupture is intra or extraperitoneal


 presents with haematuria or suprapubic pain
 history of pelvic fracture and inability to void: always
suspect bladder or urethral injury
 inability to retrieve all fluid used to irrigate the bladder
through a Foley catheter indicates bladder injury
 investigation- IVU or cystogram
 management: laparotomy if intraperitoneal, conservative if
extraperitoneal

Next question
Which of the following procedures represents the optimal operative procedure for testicular cancer?

Lords procedure

Orchidectomy via a scrotal approach

Orchidectomy via inguinal approach

Orchidectomy via a combined inguino-scrotal approach

None of the above

Testicular tumours metastasise to Para aortic nodes and thus an inguinal rather than scrotal
approach should be used. There are two main operations that are termed Lords procedure; one is
for fissure in ano and the other is a procedure for hydrocele.
Please rate this question:

Discuss and give feedback


Next question

Testicular disorders

Testicular cancer
Testicular cancer is the most common malignancy in men aged 20-30 years. Around 95% of cases
of testicular cancer are germ-cell tumours. Germ cell tumours may essentially be divided into:

Tumour
Tumour type Key features markers Pathology

Seminoma  Commonest  AFP usually Sheet like lobular


subtype (50%) normal patterns of cells
 Average age at  HCG elevated with substantial
diagnosis = 40 in 10% fibrous
 Even advanced seminomas component.
disease associated  Lactate Fibrous septa
with 5 year dehydrogenase; contain
survival of 73% elevated in 10- lymphocytic
20% seminomas inclusions and
Tumour
Tumour type Key features markers Pathology

(but also in many granulomas may


other conditions) be seen.

Non seminomatous germ  Younger age at  AFP elevated Heterogenous


cell tumours (42%) presentation =20-30 in up to 70% of texture with
years cases occasional ectopic
 Advanced disease  HCG elevated tissue such as hair
 Teratoma carries worse prognosis in up to 40% of
 Yolk sac tumour (48% at 5 years) cases
 Choriocarcinoma  Retroperitoneal lymph  Other markers
 Mixed germ cell node dissection may be rarely helpful
tumours (10%) needed for residual
disease after
chemotherapy

Image demonstrating a classical seminoma, these tumours are typically more uniform than
teratomas
Image sourced from Wikipedia

Risk factors for testicular cancer

 Cryptorchidism
 Infertility
 Family history
 Klinefelter's syndrome
 Mumps orchitis

Features
 A painless lump is the most common presenting symptom
 Pain may also be present in a minority of men
 Other possible features include hydrocele, gynaecomastia

Diagnosis

 Ultrasound is first-line
 CT scanning of the chest/ abdomen and pelvis is used for staging
 Tumour markers (see above) should be measured

Management

 Orchidectomy (Inguinal approach)


 Chemotherapy and radiotherapy may be given depending on staging
 Abdominal lesions >1cm following chemotherapy may require retroperitoneal lymph node
dissection.

Prognosis is generally excellent

 5 year survival for seminomas is around 95% if Stage I


 5 year survival for teratomas is around 85% if Stage I

Benign disease

Epididymo-orchitis
Acute epididymitis is an acute inflammation of the epididymis, often involving the testis and usually
caused by bacterial infection.

 Infection spreads from the urethra or bladder. In men <35 years, gonorrhoea or chlamydia
are the usual infections.
 Amiodarone is a recognised non infective cause of epididymitis, which resolves on stopping
the drug.
 Tenderness is usually confined to the epididymis, which may facilitate differentiating it from
torsion where pain usually affects the entire testis.

Testicular torsion

 Twist of the spermatic cord resulting in testicular ischaemia and necrosis.


 Most common in males aged between 10 and 30 (peak incidence 13-15 years)
 Pain is usually severe and of sudden onset.
 Cremasteric reflex is lost and elevation of the testis does not ease the pain.
 Treatment is with surgical exploration. If a torted testis is identified then both testis should be
fixed as the condition of bell clapper testis is often bilateral.

Hydrocele

 Presents as a mass that transilluminates, usually possible to "get above" it on examination.


 In younger men it should be investigated with USS to exclude tumour.
 In children it may occur as a result of a patent processus vaginalis.
 Treatment in adults is with a Lords or Jabouley procedure.
 Treatment in children is with trans inguinal ligation of PPV.

Next question
Theme: Management of testicular disorders

A. Fine needle aspiration cytology


B. Tru Cut biopsy
C. Orchidectomy via inguinal approach
D. Orchidectomy via scrotal approach
E. Administration of antibiotics
F. Exploration of scrotum via scrotal approach
G. Reassure and discharge

Please select the most appropriate management option for the scenario given. Each option may be
used once, more than once or not at all.

71. A 22 year old man presents with an aching pain and discomfort in his right testicle. He has
felt systemically unwell for the preceding 48 hours. On examination there is tenderness of
the right testicle. He has an exaggerated cremasteric reflex.

You answered Fine needle aspiration cytology

The correct answer is Administration of antibiotics

This is likely to represent epididymo-orchitis, this is usually due to infection with


gonorrhoea or chlamydia in this age group. In addition to treatment with antibiotics contact
tracing and appropriate swabs should also be performed.

72. A 25 year old man presents with aching and discomfort of his right testicle. He has felt
generally unwell and lethargic over the past few weeks. On examination there is a small
nodule palpable in the testis, on ultrasound this is hypoechoic. Systematic examination
demonstrates supraclavicular lymphadenopathy.

You answered Fine needle aspiration cytology

The correct answer is Orchidectomy via inguinal approach

Hypoechoic masses within the testicle in the context are most likely to represent
malignancy. He should have a staging CT scan, thereafter an orchidectomy should be
performed via an inguinal approach. Percutaneous biopsy has no role in the management
of testicular malignancy.

73. A 15 year old boy is woken from sleep by a severe pain in his left testicle. He was
previously fit and well. On examination he has a tender left testicle with an absent
cremasteric reflex.
You answered Fine needle aspiration cytology

The correct answer is Exploration of scrotum via scrotal approach

This is likely to represent torsion, surgical exploration is warranted.

Please rate this question:

Discuss and give feedback


Next question

Testicular disorders

Testicular cancer
Testicular cancer is the most common malignancy in men aged 20-30 years. Around 95% of cases
of testicular cancer are germ-cell tumours. Germ cell tumours may essentially be divided into:

Tumour
Tumour type Key features markers Pathology

Seminoma  Commonest  AFP usually Sheet like lobular


subtype (50%) normal patterns of cells
 Average age at  HCG elevated with substantial
diagnosis = 40 in 10% fibrous
 Even advanced seminomas component.
disease associated  Lactate Fibrous septa
with 5 year dehydrogenase; contain
survival of 73% elevated in 10- lymphocytic
20% seminomas inclusions and
(but also in many granulomas may
other conditions) be seen.

Non seminomatous germ  Younger age at  AFP elevated Heterogenous


cell tumours (42%) presentation =20-30 in up to 70% of texture with
years cases occasional ectopic
 Advanced disease  HCG elevated tissue such as hair
 Teratoma carries worse prognosis in up to 40% of
 Yolk sac tumour (48% at 5 years) cases
 Choriocarcinoma  Retroperitoneal lymph  Other markers
 Mixed germ cell node dissection may be rarely helpful
tumours (10%) needed for residual
disease after
Tumour
Tumour type Key features markers Pathology

chemotherapy

Image demonstrating a classical seminoma, these tumours are typically more uniform than
teratomas

Image sourced from Wikipedia

Risk factors for testicular cancer


 Cryptorchidism
 Infertility
 Family history
 Klinefelter's syndrome
 Mumps orchitis

Features

 A painless lump is the most common presenting symptom


 Pain may also be present in a minority of men
 Other possible features include hydrocele, gynaecomastia

Diagnosis

 Ultrasound is first-line
 CT scanning of the chest/ abdomen and pelvis is used for staging
 Tumour markers (see above) should be measured

Management

 Orchidectomy (Inguinal approach)


 Chemotherapy and radiotherapy may be given depending on staging
 Abdominal lesions >1cm following chemotherapy may require retroperitoneal lymph node
dissection.

Prognosis is generally excellent

 5 year survival for seminomas is around 95% if Stage I


 5 year survival for teratomas is around 85% if Stage I

Benign disease

Epididymo-orchitis
Acute epididymitis is an acute inflammation of the epididymis, often involving the testis and usually
caused by bacterial infection.

 Infection spreads from the urethra or bladder. In men <35 years, gonorrhoea or chlamydia
are the usual infections.
 Amiodarone is a recognised non infective cause of epididymitis, which resolves on stopping
the drug.
 Tenderness is usually confined to the epididymis, which may facilitate differentiating it from
torsion where pain usually affects the entire testis.

Testicular torsion

 Twist of the spermatic cord resulting in testicular ischaemia and necrosis.


 Most common in males aged between 10 and 30 (peak incidence 13-15 years)
 Pain is usually severe and of sudden onset.
 Cremasteric reflex is lost and elevation of the testis does not ease the pain.
 Treatment is with surgical exploration. If a torted testis is identified then both testis should be
fixed as the condition of bell clapper testis is often bilateral.

Hydrocele

 Presents as a mass that transilluminates, usually possible to "get above" it on examination.


 In younger men it should be investigated with USS to exclude tumour.
 In children it may occur as a result of a patent processus vaginalis.
 Treatment in adults is with a Lords or Jabouley procedure.
 Treatment in children is with trans inguinal ligation of PPV.

Next question
Theme: Haematuria

A. Interstitial nephritis
B. Membranous glomerulonephritis
C. Endometriosis
D. Placenta percreta
E. Adult polycystic kidney disease
F. Renal vein thrombosis
G. Urinary tract infection

Please select the most likely cause for haematuria for the scenario described. Each option may be
used once, more than once or not at all.

74. A 22 year female who is 24 weeks pregnant presents with frank haematuria. She is
sexually active. She has had a previous pregnancy resulting in caesarean section.

You answered Interstitial nephritis

The correct answer is Placenta percreta

Pregnancy and frank haematuria, especially if there is a history of placenta previa or prior
caesarean section, should indicate this diagnosis. There is invasive placental implantation
into the myometrium, which can rarely extend into the bladder causing severe bleeding.

75. A 22 year old woman presents with macroscopic haematuria. She is sexually active. She is
known to have renal calculi and had a berry aneurysm clipped.

You answered Interstitial nephritis

The correct answer is Adult polycystic kidney disease

APKD is associated with liver cysts (70%), berry aneurysms (25%) and pancreatic cysts
(10%). Patients may have a renal mass, hypertension, renal calculi and macroscopic
haematuria.

76. A 45 year woman presents with haematuria. She has a temperature of 38 oC and is found to
have a Hb 17. Her urine dipstick shows nitrites and 3+ blood. Blood and urine cultures are
negative.

You answered Interstitial nephritis

The correct answer is Renal vein thrombosis


Renal vein thrombosis is a common feature of renal cell carcinoma as it invades the renal
vein. Other features include PUO, left varicocele and paraneoplastic endocrine effects due
to erythropoietin factor, renin, ACTH and PTH like substance.

Please rate this question:

Discuss and give feedback


Next question

Haematuria

Causes of haematuria

Trauma  Injury to renal tract


 Renal trauma commonly due to blunt injury (others penetrating
injuries)
 Ureter trauma rare: iatrogenic
 Bladder trauma: due to RTA or pelvic fractures

Infection  Remember TB

Malignancy  Renal cell carcinoma (remember paraneoplastic syndromes):


painful or painless
 Urothelial malignancies: 90% are transitional cell carcinoma, can
occur anywhere along the urinary tract. Painless haematuria.
 Squamous cell carcinoma and adenocarcinoma: rare bladder
tumours
 Prostate cancer
 Penile cancers: SCC

Renal disease  Glomerulonephritis

Stones  Microscopic haematuria common

Structural  Benign prostatic hyperplasia (BPH) causes haematuria due to


abnormalities hypervascularity of the prostate gland
 Cystic renal lesions e.g. polycystic kidney disease
 Vascular malformations
 Renal vein thrombosis due to renal cell carcinoma

Coagulopathy  Causes bleeding of underlying lesions

Drugs  Cause tubular necrosis or interstitial nephritis: aminoglycosides,


chemotherapy
 Interstitial nephritis: penicillin, sulphonamides, and NSAIDs
 Anticoagulants

Benign  Exercise

Gynaecological  Endometriosis: flank pain, dysuria, and haematuria that is cyclical

Iatrogenic  Catheterisation
 Radiotherapy; cystitis, severe haemorrhage, bladder necrosis

Pseudohaematuria For example following consumption of beetroot

References
Http://bestpractice.bmj.com/best-practice/monograph/316/overview/aetiology.html
Next question
A 55 year old man presents with an episode of frank haematuria and on investigation is found to
have a T2 transitional cell carcinoma of the bladder. His staging investigations are negative for
metastatic disease. What is the most appropriate treatment?

Radical cystectomy

Palliative radiotherapy

Intravesical BCG

Intravesical mitomycin C

Intravesical cisplatin

T2 lesions in a young fit patient are best managed surgically. Up to 25 % patients may develop
perioperative complications. However, palliative treatments and intravesical chemotherapy (which
does NOT include cisplatin) are not used curatively in this situation.
Please rate this question:

Discuss and give feedback


Next question

Bladder cancer

Bladder cancer is the second most common urological cancer. It most commonly affects males aged
between 50 and 80 years of age. Those who are current, or previous (within 20 years), smokers
have a 2-5 fold increased risk of the disease. Exposure to hydrocarbons such as 2-Naphthylamine
increases the risk. Although rare in the UK, chronic bladder inflammation arising from
Schistosomiasis infection remains a common cause of squamous cell carcinomas, in those countries
where the disease is endemic.

Benign tumours
Benign tumours of the bladder including inverted urothelial papilloma and nephrogenic adenoma are
uncommon.

Bladder malignancies

 Transitional cell carcinoma (>90% of cases)


 Squamous cell carcinoma ( 1-7% -except in regions affected by schistosomiasis)
 Adenocarcinoma (2%)
Transitional cell carcinomas may arise as solitary lesions, or may be multifocal, owing to the effect of
"field change" within the urothelium. Up to 70% of TCC's will have a papillary growth pattern. These
tumours are usually superficial in location and accordingly have a better prognosis. The remaining
tumours show either mixed papillary and solid growth or pure solid growths. These tumours are
typically more prone to local invasion and may be of higher grade, the prognosis is therefore worse.
Those with T3 disease or worse have a 30% (or higher) risk of regional or distant lymph node
metastasis.

TNM Staging
Stage Description

T0 No evidence of tumour

Ta Non invasive papillary carcinoma

T1 Tumour invades sub epithelial connective tissue

T2a Tumor invades superficial muscularis propria (inner half)

T2b Tumor invades deep muscularis propria (outer half)

T3 Tumour extends to perivesical fat

T4 Tumor invades any of the following: prostatic stroma, seminal vesicles, uterus, vagina

T4a Invasion of uterus, prostate or bowel

T4b Invasion of pelvic sidewall or abdominal wall

N0 No nodal disease

N1 Single regional lymph node metastasis in the true pelvis (hypogastric, obturator, external
iliac, or presacral lymph node)
Stage Description

N2 Multiple regional lymph node metastasis in the true pelvis (hypogastric, obturator, external
iliac, or presacral lymph node metastasis)

N3 Lymph node metastasis to the common iliac lymph nodes

M0 No distant metastasis

M1 Distant disease

Presentation
Most patients (85%) will present with painless, macroscopic haematuria. In those patients with
incidental microscopic haematuria, up to 10% of females aged over 50 will be found to have a
malignancy (once infection excluded).

Staging
Most will undergo a cystoscopy and biopsies or TURBT, this provides histological diagnosis and
information relating to depth of invasion. Locoregional spread is best determined using pelvic MRI
and distant disease CT scanning. Nodes of uncertain significance may be investigated using PET
CT.

Treatment
Those with superficial lesions may be managed using TURBT in isolation. Those with recurrences or
higher grade/ risk on histology may be offered intravesical chemotherapy. Those with T2 disease are
usually offered either surgery (radical cystectomy and ileal conduit) or radical radiotherapy.

Prognosis
T1 90%

T2 60%

T3 35%

T4a 10-25%

Any T, N1-N2 30%


Theme: Urinary incontinence

A. Bladder diary for 3 days


B. Urodynamic studies
C. Bladder drill training for 6 weeks
D. Pelvic floor exercises for 3 months
E. Oxybutynin
F. IV urography
G. Dyes studies including phenazopyridine
H. None of the above

Choose the best management option for each clinical scenario. Each option may be used once,
more than once or not at all.

78. A 75 year old lady reports urinary incontinence when coughing and sneezing. She has had
2 children with no complications. She has no significant past medical history and is on no
medications. What is the most appropriate initial management?

You answered Bladder diary for 3 days

The correct answer is Pelvic floor exercises for 3 months

A diagnosis of stress incontinence is obvious from the history, therefore there is no need
for a bladder diary or urodynamic studies.
Pelvic floor exercises would be the first line management.

79. A 26 year old pregnant woman is having her 1st child and has never had problems with
incontinence.

You answered Bladder diary for 3 days

The correct answer is Pelvic floor exercises for 3 months

Pregnant women should receive instructions as to how to perform pelvic floor exercises
during pregnancy as this may help to decrease subsequent risk of stress urinary
incontinence.

80. A 67 year old lady reports urinary incontinence. She describes the sensation of needing to
pass urine immediately. She has had 2 children and is on no medications. What is the most
appropriate initial management?

Bladder diary for 3 days


The patient appears to be describing urge incontinence. A bladder diary is needed to
establish the baseline frequency of micturition and amounts of urine passed. Then bladder
training can be initiated to increase the volume of urine passed at reduced frequencies.

Please rate this question:

Discuss and give feedback


Next question

Urinary incontinence

Involuntary passage of urine. Most cases are female (80%). It has a prevalence of 11% in those
aged greater than 65 years. The commonest variants include:

 Stress urinary incontinence (50%)


 Urge incontinence (15%)
 Mixed (35%)

Males
Males may also suffer from incontinence although it is a much rarer condition in men. A number of
anatomical factors contribute to this. Males have 2 powerful sphincters; one at the bladder neck and
the other in the urethra. Damage to the bladder neck mechanism is a factor in causing retrograde
ejaculation following prostatectomy. The short segment of urethra passing through the urogenital
diaphragm consists of striated muscle fibres (the external urethral sphincter) and smooth muscle
capable of more sustained contraction. It is the latter mechanism that maintains continence following
prostatectomy.

Females
The sphincter complex at the level of bladder neck is poorly developed in females. As a result the
external sphincter complex is functionally more important, its composition being similar to that of
males. Innervation is via the pudendal nerve and the neuropathy that may accompany obstetric
events may compromise this and lead to stress urinary incontinence.

Innervation
Somatic innervation to the bladder is via the pudendal, hypogastric and pelvic nerves. Autonomic
nerves travel in these nerve fibres too. Bladder filling leads to detrusor relaxation (sympathetic)
coupled with sphincter contraction. The parasympathetic system causes detrusor contraction and
sphincter relaxation. Overall control of micturition is centrally mediated via centres in the Pons.

Stress urinary incontinence

 50% of cases, especially in females.


 Damage (often obstetric) to the supporting structures surrounding the bladder may lead to
urethral hypermobility.
 Other cases due to sphincter dysfunction, usually from neurological disorders (e.g. Pudendal
neuropathy, multiple sclerosis).
Urethral mobility:
Pressure not transmitted appropriately to the urethra resulting in involuntary passage of urine during
episodes of raised intra-abdominal pressure.

Sphincter dysfunction:
Sphincter fails to adapt to compress urethra resulting in involuntary passage of urine. When the
sphincter completely fails there is often to continuous passage of urine.

Urge incontinence
In these patients there is sense of urgency followed by incontinence. The detrusor muscle in these
patients is unstable and urodynamic investigation will demonstrate overactivity of the detrusor
muscle at inappropriate times (e.g. Bladder filling). Urgency may be seen in patients with overt
neurological disorders and those without. The pathophysiology is not well understood but poor
central and peripheral co-ordination of the events surrounding bladder filling are the main
processes.

Assessment
Careful history and examination including vaginal examination for cystocele.
Bladder diary for at least 3 days
Consider flow cystometry if unclear symptomatology or surgery considered and diagnosis is unclear.
Exclusion of other organic disease (e.g. Stones, UTI, Cancer)

Management
Conservative measures should be tried first; Stress urinary incontinence or mixed symptoms should
undergo 3 months of pelvic floor exercise. Over active bladder should have 6 weeks of bladder
retraining.
Drug therapy for women with overactive bladder should be offered oxybutynin (or solifenacin if
elderly) if conservative measures fail.
In women with detrusor instability who fail non operative therapy a trial of sacral neuromodulation
may be considered, with conversion to permanent implant if good response. Augmentation
cystoplasty is an alternative but will involve long term intermittent self catheterisation.
In women with stress urinary incontinence a urethral sling type procedure may be undertaken.
Where cystocele is present in association with incontinence it should be repaired particularly if it lies
at the introitus.

NICE guidelines

 Initial assessment urinary incontinence should be classified as stress/urge/mixed.


 At least 3/7 bladder diary if unable to classify easily.
 Start conservative treatment before urodynamic studies if a diagnosis is obvious from the
history
 Urodynamic studies if plans for surgery.
 Stress incontinence: Pelvic floor exercises 3/12, if fails consider surgery.
 Urge incontinence: Bladder training >6/52, if fails for oxybutynin (antimuscarinic drugs) then
sacral nerve stimulation.
 Pelvic floor exercises offered to all women in their 1st pregnancy.

Next question
A 47-year-old woman presents with loin pain and haematuria. Urine dipstick demonstrates:

Blood ++++

Nitrites POS

Leucocytes +++

Protein ++

Urine culture shows a Proteus infection. An x-ray demonstrates a stag-horn calculus in the left renal
pelvis. What is the most likely composition of the renal stone?

Xanthine

Calcium oxalate

Struvite

Cystine

Urate

Stag-horn calculi are composed of struvite and form in alkaline urine (ammonia producing bacteria
therefore predispose)

Renal stones on x-ray

 cystine stones: semi-opaque


 urate + xanthine stones: radio-lucent

Please rate this question:

Discuss and give feedback

Renal stones: imaging

The table below summarises the appearance of different types of renal stone on x-ray
Type Frequency Radiograph appearance

Calcium oxalate 40% Opaque

Mixed calcium oxalate/phosphate stones 25% Opaque

Triple phosphate stones* 10% Opaque

Calcium phosphate 10% Opaque

Urate stones 5-10% Radio-lucent

Cystine stones 1% Semi-opaque, 'ground-glass' appearance

Xanthine stones <1% Radio-lucent

*stag-horn calculi involve the renal pelvis and extend into at least 2 calyces. They develop in alkaline
urine and are composed of struvite (ammonium magnesium phosphate, triple phosphate).
Ureaplasma urealyticum and Proteus infections predispose to their formation
Theme: Amputations

A. Transfemoral amputation
B. Gritti - Stokes amputation
C. Digital amputation
D. Syme's amputation
E. Hindquarter amputation
F. Below knee amputation
G. Trans metatarsal amputation
H. Amputation of digit

Please select the most appropriate procedure for the scenario given. Each option may be used
once, more than once or not at all.

1. The operation of choice for a 90 year old lady with infected gangrene of the mid foot
secondary to diabetes. She has fixed flexion deformity of the knee.

Transfemoral amputation

An elderly patient with diabetes and peripheral vascular disease is a high risk surgical
candidate. It is important that the chances of a successful outcome are maximised at the first
operation. SInce above knee amputations usually heal more reliably than below knee
amputations this is a preferable option, especially since she has a fixed deformity.

2. An operation in which Skew flaps are created.

You answered Transfemoral amputation

The correct answer is Below knee amputation

This is one variant of a below knee amputation. The Burgess flap is the other commonly
practised approach.

3. An amputation of the lower limb in which the femoral condyles are removed and the patella
retained.

You answered Transfemoral amputation

The correct answer is Gritti - Stokes amputation

This is a Gritti - Stokes amputation. During a Gritti - Stokes operation the patella is
conserved and swung posteriorly to cover the distal femoral surface.
Beware performing amputations in patients with peripheral vascular disease without optimising
inflow first!

Please rate this question:

Discuss and give feedback


Next question

Amputations

Amputations are indicated when the affected limb is one of the following:

 Dead non viable


 Deadly where it is posing a major threat to life
 Dead useless where it is viable but a prosthesis would be preferable

Orthopaedic surgery

 Amputation is often undertaken as an option of last resort e.g. Limb salvage has failed and
the limb is so non functional that mobility needs would be best met with prosthesis.
 Chronic fracture non union or significant limb shortening following trauma would fit into this
category. Occasionally following major trauma a primary amputation is preferable. This
would be the case in an open fracture with major distal neurovascular compromise and other
more life threatening injuries are present.

Vascular surgery

 The first two categories are the most prevalent.


 Diabetic foot sepsis is often a major cause of sepsis which can spread rapidly in the
presence of established peripheral vascular disease.
 As a general rule the main issue in vascular surgery is to optimise vascular inflow prior to
surgery. The more distal the planned amputation is to be, the more important this rule
becomes.
 In other situations there has been something such as an embolic event that has not been
revascularised in time. In this case the limb shows fixed mottling and an amputation will be
needed.

Types of amputations
As the vast majority of commonly performed amputations affect the lower limbs these will be covered
here.

The main categories of amputations are:


 Pelvic disarticulation (hindquarter)
 Above knee amputation
 Gritti Stokes (through knee amputation)
 Below knee amputation (using either Skew or Burgess flaps)
 Syme's amputation (through ankle)
 Amputations of mid foot and digits

Choosing a level of amputation depends on:

 The disease process being treated


 Desired functional outcome
 Co-morbidities of the patient

Above knee amputations

 Quick to perform
 Heal reliably
 Patients regain their general health quickly
 For this benefit, a functional price has to be paid and many patients over the age of 70 will
never walk on an above knee prosthesis.
 Above knee amputations use equal anterior-posterior flaps

Below knee amputations

 Technically more challenging to perform


 Heal less reliably than their above knee counterparts.
 However, many more patients are able to walk using a below knee prosthesis.
 In below knee amputations the two main flaps are Skew flaps or the Burgess long posterior
flap. Skew flaps result in a less bulky limb that is easier to attach a prosthesis to.

It is worth remembering that whilst it may be technically feasible to offer a below knee amputation
there may be circumstances where an above knee option is preferable. For example, in fixed flexion
deformities of the lower limb, little functional benefit would be gained from below knee amputation
surgery.
Next question
Theme: Acute limb ischaemia

A. Primary amputation
B. Transfemoral embolectomy with prophylactic fasciotomy
C. Transpopliteal embolectomy without prophylactic fasciotomy
D. Transfemoral embolectomy without prophylactic fasciotomy
E. Transpopliteal embolectomy with prophylactic fasciotomy
F. Angiogram
G. Systemic heparin infusion
H. Peripheral thrombolysis

Please select the most appropriate management for the following patients presenting with acutely
ischaemic limbs. Each option may be used once, more than once or not at all.

4. A 76 year old man presents with a painful left leg. The pain began suddenly and with no
previous history. On examination he has a white left leg with no palpable femoral pulse and
loss of sensation. The pulses in the contra lateral limb are normal. It is now three hours
since the pain first started.

You answered Primary amputation

The correct answer is Transfemoral embolectomy without prophylactic fasciotomy

A limb which is acutely ischaemic and with normal contralateral pulses normally indicates
an acute embolus. Whilst intra arterial thrombolysis may be an option there is a reasonable
argument for immediate surgery. A fasciotomy is unlikely to be required.

5. A 56 year old man presents with a painful left leg. The pain has been present for the past 8
hours although it has also been present (though less severe) about a week ago. At that stage
he noted that his hallux had turned blue. This resolved spontaneously. On examination he
has a weakly palpable femoral pulse on the affected side but no pulses palpable distal to
this. His sensation is mildly impaired.

You answered Primary amputation

The correct answer is Angiogram

The history favors a more chronic process and the great toe cyanotic spell may be indicative
of previous embolism from pathology such as an aneurysm. In the ideal scenario a duplex
scan would be performed. However, an angiogram would probably supply sufficient
information and allow appropriate endovascular therapy.

6. A 78 year old lady is found by carers with a severely painful left leg. On examination she
has no palpable pulses and the limb is cold, insensate and mottled. The mottling does not
blanch with pressure.

Primary amputation

This is an unsalvagable limb and is best amputated primarily.

Please rate this question:

Discuss and give feedback


Next question

Acute limb ischaemia

 Thrombosis of a pre-existing site of atherosclerosis if the commonest cause of acute limb


ischaemia
 Acute thrombosis of popliteal aneurysms poses the greatest threat to the limb
 Sudden occlusion of a large proximal vessel results in the typical appearances of acute limb
ischaemia

Clinical appearances

 Less than 6 hours = White leg


 At 6 -12 hours = Mottled limb with blanching on pressure
 More than 12-24 hours = Fixed mottling

Management of acutely ischaemic leg


Clinical picture Treatment

White leg with sensorimotor deficit Surgery and embolectomy

Dusky leg, mild anaesthesia Angiography

Fixed mottling Primary amputation

Role of thrombolysis

 Intra arterial thrombolysis is better than peripheral thrombolysis


 Mainly indicated in acute on chronic thrombosis
 Avoid if within 2 months of CVA or 2 weeks of surgery
 Aspiration of clot may improve success rate if the thrombosis is large

Surgery

 Both groins should be prepared


 Transverse arteriotomy is easier to close
 Poor inflow should be managed with iliac trawl- if this fails to improve then consider a
femoro-femoral cross over or axillo-femoral cross over.
 A check angiogram should be performed on table and prior to closure
 Systemic heparinisation should follow surgery
 Fasciotomy should be considered if the time between onset and surgery exceeds 6 hours

Next question
Theme: Vascular disorders affecting the upper limb

A. Proximal brachial artery occlusion secondary to atheroma


B. Distal brachial artery occlusion secondary to atheroma
C. Axillary artery embolus
D. Axillary vein thrombosis
E. Cervical rib
F. Raynaud's disease
G. Rheumatoid disease

Please select the most likely cause for the presenting scenario described. Each option may be used
once, more than once or not at all.

7. A 73 year old male presents with a collapse and is brought to the emergency department.
On examination he has a cold, painful left hand and forearm.

You answered Proximal brachial artery occlusion secondary to atheroma

The correct answer is Axillary artery embolus

Theme from September 2012 Exam


Sudden arterial embolus will affect the axillary artery in up to 30% cases. Because of the
acute nature of the condition there is no time for the development of a collateral circulation
so the limb is usually pale and painful. Emboli occur usually occur as a result of atrial
fibrillation. Fast atrial fibrillation can cause syncope and an acute embolus.

8. A 23 year old man presents with intermittent symptoms of altered sensation in his arm and
discomfort when he uses his hands. He works as an electrician and his symptoms are worst
when he is fitting light fixtures.

You answered Proximal brachial artery occlusion secondary to atheroma

The correct answer is Cervical rib

Compression of the thoracic outlet by the fibrous band of the "rib" can result in both
neurological and circulatory compromise. When manual tasks are performed in which the
hand works overhead the signs and symptoms will be maximal and this is the basis of
Adsons test.

9. A 19 year old lady presents with recurrent episodes of pain in her hands. She notices that
her symptoms are worst in cold weather. When she gets the pain she notices that her hands
are very pale, they then become dark blue in colour.
You answered Proximal brachial artery occlusion secondary to atheroma

The correct answer is Raynaud's disease

Raynauds disease is characterised by a series of colour changes and discomfort is often


present. The young age at presentation coupled with the absence of a smoking history (in
most cases) makes occlusive disease unlikely.

Please rate this question:

Discuss and give feedback


Next question

Vascular disorders of the upper limb

Upper limb arterial disease is less common than lesions causing symptoms in the lower limb. The
upper limb circulation may be affected by embolic events, stenotic lesions (both internal and
extrinsic), inflammatory disorders and venous diseases.
The anatomy of the collateral circulation of the arterial inflow may impact on the history and nature of
disease presentation. In the region of the subclavian and axillary arteries the collateral vessels
passing around the shoulder joint may provide pathways for flow if the main vessels are stenotic or
occluded. During periods of increased metabolic demand the collateral flow is not sufficient and the
vertebral arteries may have diminished flow. This may result in diminished flow to the brain with
neurological sequelae such as syncope.

Vascular disease of the upper limb


Condition Features

Axillary/  50% of upper limb emboli will lodge in the brachial artery
brachial  30% of upper limb emboli will lodge in the axillary artery
embolus  Sudden onset of symptoms; pain, pallor, paresis, pulselessness,
paraesthesia
 Sources are left atrium with cardiac arrhythmia (mainly AF), mural
thrombus
 Cardiac arrhythmias may result in impaired consciousness in addition to
the embolus

Arterial  Those resulting from atheroma are the most common, trauma may result
occlusions in vascular changes and long term occlusion but this is rare
 Features may include claudication, ulceration and gangrene. Proximally
sited lesions may result in subclavian steal syndrome
 The progressive nature of the disease allows development of collaterals,
acute ischaemia may occur as a result of acute thrombosis

Raynaud's  Idiopathic condition affecting young females


disease  Usually affects hands > feet
 Digits become: white →blue →red
 Treatment is with calcium antagonists

Upper limb  Gradual onset of upper limb swelling and discomfort.


venous  Sensation and motor function are normal
thrombosis  Condition may complicate pre-existing malignancy (especially breast
cancer) or arise as a result of repetitive use of the limb in a task such as
painting a ceiling
 The condition is diagnosed with duplex ultrasound and treatment is with
anticoagulation

Cervical rib  0.2-0.4% incidence


 Consist of an anomalous fibrous band that often originates from C7 and
may arc towards, but rarely reaches the sternum
 Congenital cases may present around the third decade, some cases are
reported to occur following trauma
 Bilateral in up to 70%
 Compression of the subclavian artery may produce absent radial pulse on
clinical examination and in particular may result in a positive Adsons test
(lateral flexion of the neck away from symptomatic side and traction of
the symptomatic arm- leads to obliteration of radial pulse)
 Treatment is most commonly undertaken when there is evidence of
neurovascular compromise. A transaxillary approach is the traditional
operative method for excision

Next question
Theme: Management of occlusive vascular disease

A. Aorto-bifemoral bypass graft


B. Femoro-femoral cross over graft
C. Femoro-popliteal bypass graft
D. Femoro-distal bypass graft
E. Axillo-bifemoral bypass graft
F. Bilateral above knee amputation

Please select the most appropriate arterial bypass method for the scenario described. Each option
may be used once, more than once or not at all.

10. An 83 year old lady with a significant cardiac history is admitted with rest pain and
bilateral leg ulcers. Imaging demonstrates bilateral occlusion of both common iliac arteries
that are unsuitable for stenting.

You answered Aorto-bifemoral bypass graft

The correct answer is Axillo-bifemoral bypass graft

Theme from January 2012 Exam


In patients with major cardiac co-morbidities the safest option is to choose an axillo-
bifemoral bypass graft. The long term patency rates are less good than with aorto-
bifemoral bypass grafts, however, the operation is less major.

11. A 54 year old man presents to the vascular clinic with severe rest pain and an ulcer on his
right foot that is not healing. On examination he has bilateral absent femoral pulses.
Imaging demonstrates a bilateral occlusion of the common iliac arteries that is not suitable
for stenting.

Aorto-bifemoral bypass graft

In a young patient consideration should be given to aorto-bifemoral bypass grafts as these


have the best long term functional outcome compared with an axillo-bifemoral bypass
graft.

12. A 78 year old man presents with left sided rest pain in his leg and a non healing arterial leg
ulcer on the same leg. Imaging shows normal right leg vessels, on the left side there is a
long occlusion of the external iliac artery that is unsuitable for stenting. He has a
significant cardiac history.

You answered Aorto-bifemoral bypass graft


The correct answer is Femoro-femoral cross over graft

Femoro-femoral cross over grafts are an option for treatment of iliac occlusions in patients
with significant co-morbidities and healthy contralateral vessels. In reality the idealised
situation presented here seldom applies and the opposite vessels usually have some disease
and one must be careful not to damage the "healthy" side.

Please rate this question:

Discuss and give feedback


Next question

Peripheral vascular disease

Indications for surgery to revascularise the lower limb

 Intermittent claudication
 Critical ischaemia
 Ulceration
 Gangrene

Intermittent claudication that is not disabling may provide a relative indication, whilst the other
complaints are often absolute indications depending upon the frailty of the patient.

Assessment

 Clinical examination
 Ankle brachial pressure index measurement
 Duplex arterial ultrasound
 Angiography (standard, CT or MRI): usually performed only if intervention being considered.

Angioplasty
In order for angioplasty to be undertaken successfully the artery has to be accessible. The lesion
relatively short and reasonable distal vessel runoff. Longer lesions may be amenable to sub-intimal
angioplasty.

Surgery
Surgery will be undertaken where attempts at angioplasty have either failed or are unsuitable.
Bypass essentially involves bypassing the affected arterial segment by utilising a graft to run from
above the disease to below the disease. As with angioplasty good runoff improves the outcome.

Some key concepts with bypass surgery

Superficial femoral artery occlusion to the above knee popliteal


In the ideal scenario, vein (either in situ or reversed LSV) would the used as a conduit. However,
prosthetic material has reasonable 5 year patency rates and some would advocate using this in
preference to vein so that vein can be used for other procedures in the future. In general terms either
technique is usually associated with an excellent outcome (if run off satisfactory).

Procedure

 Artery dissected out, IV heparin 3,000 units given and then the vessels are cross clamped
 Longitudinal arteriotomy
 Graft cut to size and tunneled to arteriotomy sites
 Anastomosis to femoral artery usually with 5/0 'double ended' Prolene suture
 Distal anastomosis usually using 6/0 'double ended' Prolene

Distal disease

 Femoro-distal bypass surgery takes longer to perform, is more technically challenging and
has higher failure rates.
 In elderly diabetic patients with poor runoff a primary amputation may well be a safer and
more effective option. There is no point in embarking on this type of surgery in patients who
are wheelchair bound.
 In femorodistal bypasses vein gives superior outcomes to PTFE.

Rules

 Vein mapping 1st to see whether there is suitable vein (the preferred conduit). Sub intimal
hyperplasia occurs early when PTFE is used for the distal anastomosis and will lead to early
graft occlusion and failure.
 Essential operative procedure as for above knee fem-pop.
 If there is insufficient vein for the entire conduit then vein can be attached to the end of the
PTFE graft and then used for the distal anastomosis. This type of 'vein boot' is technically
referred to as a Miller Cuff and is associated with better patency rates than PTFE alone.
 Remember the more distal the arterial anastomosis the lower the success rate.

References
Peach G et al. Diagnosis and management of peripheral arterial disease. BMJ 2012; 345: 36-41.
Next question
Theme: Causes of arterial occlusion

A. Vasculitis
B. Steal syndrome
C. Thrombosis
D. Foreign body embolus
E. Clot embolus
F. Vasospasm
G. Direct arterial injury

Please select the most likely underlying cause for the scenario provided. Each option may be used
once, more than once or not at all.

13. A 73 year old lady develops a cold, pulseless hand 3 days following a myocardial
infarction.

You answered Vasculitis

The correct answer is Clot embolus

The development of mural or atrial appendage thrombi may occur following a myocardial
infarct and co-existing atrial fibrillation may contribute to the formation. They tend to
present with classical features of an embolic event.

14. A 6 year old child has suffered a displaced supracondylar humeral fracture. On
examination they have a cold and insensate hand with absent pulses.

You answered Vasculitis

The correct answer is Direct arterial injury

Both vasospasm and arterial injury may complicate supracondylar fractures and are seen in
1% of all cases. Vasospasm is usually transient and more likely when the injury is minor
and reduced early. Severely displaced injuries and those with more advanced signs are
usually associated with direct arterial injury.

15. A 26 year old man who smokes heavily develops aching, crampy pains in his legs. On
examination distal limb pulses are diminished.

Vasculitis

This is likely to represent Buergers disease. It is commonest in young males who smoke
heavily.
Please rate this question:

Discuss and give feedback


Next question

Arterial occlusions/ insufficiency

Arterial occlusions may occur as a result of a number of processes. The typical clinical scenarios are
outlined below.

Cause of
occlusion Typical picture

Embolus Sudden onset


Depending upon level of occlusion; limb may show typical features of pain, loss
of pulses and pallor. Sensory perceptive changes may also be present

Thrombosis Usually known disease and prodromal symptoms e.g. claudication


Disruption to flow may be incomplete
If background disease process present then collaterals may be present and
picture less dramatic

Vasospasm May be due to Raynauds and affect extremities


Symptoms are often temperature related
Discolouration of the hands may occur (pale, dark, red)
Symptoms improve during pregnancy (hyperdynamic circulation)

Steal Occur secondary to arteriovenous fistula, or partial arterial occlusions (e.g.


syndromes cervical rib)
Pain and diminished pulses distal to fistula are seen

Vasculitis
Vessel diameter and vasculitis classification
Aorta and branches  Takayasu's arteritis
 Buergers disease
 Giant cell arteritis
Large and medium sized arteries  Buergers disease
 Giant cell arteritis
 Polyarteritis nodosa

Medium sized muscular arteries  Polyarteritis nodosa


 Wegeners granulomatosis

Small muscular arteries  Wegeners granulomatosis


 Rheumatoid vasculitis

Specific conditions
Takyasu's arteritis  Inflammatory, obliterative arteritis affecting aorta and branches
 Females> Males
 Symptoms may include upper limb claudication
 Clinical findings include diminished or absent pulses
 ESR often affected during the acute phase

Buergers disease  Segmental thrombotic occlusions of the small and medium sized
lower limb vessels
 Commonest in young male smokers
 Proximal pulses usually present, but pedal pulses are lost
 An acuter hypercellular occlusive thrombus is often present
 Tortuous corkscrew shaped collateral vessels may be seen on
angiography

Giant cell arteritis  Systemic granulomatous arteritis that usually affects large and
medium sized vessels
 Females > Males
 Temporal arteritis is commonest type
 Granulomatous lesions may be seen on biopsy (although up to
50% are normal)

Polyarteritis nodosa  Systemic necrotising vasculitis affecting small and medium sized
muscular arteries
 Most common in populations with high prevalence of hepatitis B
 Renal disease is seen in 70% cases
 Angiography may show saccular or fusiform aneurysms and
arterial stenoses
Wegeners  Predominantly affects small and medium sized arteries
granulomatosis  Systemic necrotising granulomatous vasculitis
 Cutaneous vascular lesions may be seen (ulceration, nodules and
purpura)
 Sinus imaging may show mucosal thickening and air fluid levels

Next question
Theme: Ankle brachial pressure index

A. >1.2
B. 1.0
C. 0.8
D. 0.5
E. 0.3

Please select the ankle brachial pressure index that is most likely to be present for the scenario
given. Each value may be used once, more than once or not at all.

16. A 73 year old lifelong heavy smoker presents to the vascular clinic with symptoms of foot
ulceration and rest pain. On examination her foot has areas of gangrene and pulses are
impalpable.

You answered >1.2

The correct answer is 0.3

This is critical limb ischaemia. Values of 0.3 are typical in this setting and urgent further
imaging is needed. Debridement of necrosis prior to improving arterial inflow carries a
high risk of limb loss.

17. A 63 year old man presents with a claudication distance of 15 yards. He is a lifelong heavy
smoker. On examination his foot is hyperaemic and there is a small ulcer at the tip of his
great toe.

You answered >1.2

The correct answer is 0.5

Hyperaemia may occur in association with severe vascular disease and is referred to
surgically as a "sunset foot". ABPI is usually higher than 0.3, but seldom greater than 0.5.
Especially when associated with hyperaemic changes and ulceration. Urgent further
imaging and risk factor modification is needed.

18. A 77 year old morbidly obese man with type 2 diabetes presents with leg pain at rest. His
symptoms are worst at night and sometimes improve during the day. He has no areas of
ulceration.

>1.2

Type 2 diabetics may have vessel calcification. This will result in abnormally high ABPI
readings. Pain of this nature in diabetics is usually neuropathic and if a duplex scan is
normal then treatment with an agent such as duloxetine is sometimes helpful.
Theme from September 2011 and September 2012 exam

Please rate this question:

Discuss and give feedback


Next question

Ankle-Brachial pressure index

 Measurement of ankle- brachial pressure index (ABPI) is a commonly performed vascular


investigation.
 Calculated by dividing lower limb pressure by the highest upper limb pressure.

Results of ABPI
1.2 or greater Usually due to vessel calcification

1.0- 1.2 Normal

Minor stenotic lesion


0.8-1.0
Initiate risk factor management

Moderate stenotic lesion


Consider duplex
0.50-0.8
Risk factor management
If mixed ulcers present then avoid tight compression bandages

Likely significant stenosis


0.5- 0.3 Duplex scanning to delineate lesions needed
Compression bandaging contra indicated

Indicative of critical ischaemia


Less than 0.3
Urgent detailed imaging required
Next question
A 67 year old patient is due to undergo a femoro-popliteal bypass graft. Which heparin regime
should the surgeon ask for prior to cross clamping the femoral artery?

Single therapeutic dose of low molecular weight heparin on the ward prior to coming to
theatre

Single therapeutic dose of low molecular weight heparin the night before surgery

Dose of 10,000 units of unfractionated heparin prior to induction of anaesthesia

Dose of 3,000 units of unfractionated heparin, 3 minutes prior to cross clamping

Dose of 30,000 units of unfractionated heparin, 3 minutes prior to cross clamping

As a rule most vascular surgeons will administer approximately 3,000 units of systemic heparin 3-5
minutes prior to cross clamping to help prevent further intra arterial thromboses. A dose of 30,000
units is given prior to going on cardiopulmonary bypass. Heparin given at induction will cause
bleeding during routine dissection.
Please rate this question:

Discuss and give feedback


Next question

Heparin

 Causes the formation of complexes between antithrombin and activated thrombin/factors


7,9,10,11 & 12

Advantages of low molecular weight heparin

 Better bioavailability
 Lower risk of bleeding
 Longer half life
 Little effect on APTT at prophylactic dosages
 Less risk of HIT

Complications
 Bleeding
 Osteoporosis
 Heparin induced thrombocytopenia (HIT): occurs 5-14 days after 1st exposure
 Anaphylaxis

In surgical patients that may need a rapid return to theatre, administration of unfractionated heparin
is preferred; as low molecular weight heparins have a longer duration of action and are harder to
reverse.
Next question
An 18 year old lady presents with extensive varicose veins of her left leg. There is associated port
wine staining. What is the most likely diagnosis?

Type 1 diabetes

Osler syndrome

Gardner's syndrome

Proteus syndrome

Klippel-Trenaunay-Weber syndrome

A less common cause of venous insufficiency is Klippel-Trenaunay-Weber (KTW) syndrome, which


involves port-wine stains, varicose veins, and bony or soft-tissue hypertrophy.
Please rate this question:

Discuss and give feedback


Next question

Lower leg ulcers

Venous leg ulcers

 Most due to venous hypertension, secondary to chronic venous insufficiency (other causes
include calf pump dysfunction or neuromuscular disorders)
 Ulcers form due to capillary fibrin cuff or leucocyte sequestration
 Features of venous insufficiency include oedema, brown pigmentation, lipodermatosclerosis,
eczema
 Location above the ankle, painless
 Deep venous insufficiency is related to previous DVT and superficial venous insufficiency is
associated with varicose veins
 Doppler ultrasound looks for presence of reflux and duplex ultrasound looks at the anatomy/
flow of the vein
 Management: 4 layer compression banding after exclusion of arterial disease or surgery
 If fail to heal after 12 weeks or >10cm 2 skin grafting may be needed

Marjolin's ulcer
Image sourced from Wikipedia

 Squamous cell carcinoma


 Occurring at sites of chronic inflammation e.g; burns, osteomyelitis after 10-20 years
 Mainly occur on the lower limb

Arterial ulcers

 Occur on the toes and heel


 Painful
 There may be areas of gangrene
 Cold with no palpable pulses
 Low ABPI measurements

Neuropathic ulcers

 Commonly over plantar surface of metatarsal head and plantar surface of hallux
 The plantar neuropathic ulcer is the condition that most commonly leads to amputation in
diabetic patients
 Due to pressure
 Management includes cushioned shoes to reduce callous formation

Pyoderma gangrenosum
Image sourced from Wikipedia

 Associated with inflammatory bowel disease/RA


 Can occur at stoma sites
 Erythematous nodules or pustules which ulcerate

Next question
Theme: Management of peripheral arterial disease

A. Primary amputation
B. Angioplasty
C. Arterial bypass surgery using vein
D. Arterial bypass surgery using PTFE
E. Conservative management with medical therapy and exercise
F. Watch and wait
G. Duplex scanning

Please select the most appropriate management for the scenario given. Each option may be used
once, more than once or not at all.

21. A 63 year old man is admitted with rest pain and foot ulceration. An angiogram shows a 3
cm area of occlusion of the distal superficial femoral artery with 3 vessel run off. His ankle
- brachial pressure index is 0.4.

You answered Primary amputation

The correct answer is Angioplasty

Short segment disease and good run off with tissue loss is a compelling indication for
angioplasty. He should receive aspirin and a statin if not already taking them.

22. A 72 year old man present in the vascular clinic with calf pain present on walking 100
yards. He is an ex-smoker and lives alone. On examination he has reasonable leg pulses.
His right dorsalis pedis pulse gives a monophasic doppler signal with an ankle brachial
pressure index measurement of 0.7. All other pressures are acceptable. There is no
evidence of ulceration or gangrene.

You answered Primary amputation

The correct answer is Conservative management with medical therapy and exercise

Structured exercise programmes combined with medical therapy will improve many
patients. Should his symptoms worsen or fail to improve then imaging with duplex
scanning would be required.

23. An 83 year old lady is admitted from a nursing home with infected lower leg ulcers. She
underwent an attempted long superficial femoral artery sub initimal angioplasty 2 weeks
previously. This demonstrated poor runoff below the knee.

Primary amputation
Poor runoff and sepsis would equate to poor outcome with attempted bypass surgery.

Please rate this question:

Discuss and give feedback


Next question

Peripheral vascular disease

Indications for surgery to revascularise the lower limb

 Intermittent claudication
 Critical ischaemia
 Ulceration
 Gangrene

Intermittent claudication that is not disabling may provide a relative indication, whilst the other
complaints are often absolute indications depending upon the frailty of the patient.

Assessment

 Clinical examination
 Ankle brachial pressure index measurement
 Duplex arterial ultrasound
 Angiography (standard, CT or MRI): usually performed only if intervention being considered.

Angioplasty
In order for angioplasty to be undertaken successfully the artery has to be accessible. The lesion
relatively short and reasonable distal vessel runoff. Longer lesions may be amenable to sub-intimal
angioplasty.

Surgery
Surgery will be undertaken where attempts at angioplasty have either failed or are unsuitable.
Bypass essentially involves bypassing the affected arterial segment by utilising a graft to run from
above the disease to below the disease. As with angioplasty good runoff improves the outcome.

Some key concepts with bypass surgery

Superficial femoral artery occlusion to the above knee popliteal


In the ideal scenario, vein (either in situ or reversed LSV) would the used as a conduit. However,
prosthetic material has reasonable 5 year patency rates and some would advocate using this in
preference to vein so that vein can be used for other procedures in the future. In general terms either
technique is usually associated with an excellent outcome (if run off satisfactory).
Procedure

 Artery dissected out, IV heparin 3,000 units given and then the vessels are cross clamped
 Longitudinal arteriotomy
 Graft cut to size and tunneled to arteriotomy sites
 Anastomosis to femoral artery usually with 5/0 'double ended' Prolene suture
 Distal anastomosis usually using 6/0 'double ended' Prolene

Distal disease

 Femoro-distal bypass surgery takes longer to perform, is more technically challenging and
has higher failure rates.
 In elderly diabetic patients with poor runoff a primary amputation may well be a safer and
more effective option. There is no point in embarking on this type of surgery in patients who
are wheelchair bound.
 In femorodistal bypasses vein gives superior outcomes to PTFE.

Rules

 Vein mapping 1st to see whether there is suitable vein (the preferred conduit). Sub intimal
hyperplasia occurs early when PTFE is used for the distal anastomosis and will lead to early
graft occlusion and failure.
 Essential operative procedure as for above knee fem-pop.
 If there is insufficient vein for the entire conduit then vein can be attached to the end of the
PTFE graft and then used for the distal anastomosis. This type of 'vein boot' is technically
referred to as a Miller Cuff and is associated with better patency rates than PTFE alone.
 Remember the more distal the arterial anastomosis the lower the success rate.

References
Peach G et al. Diagnosis and management of peripheral arterial disease. BMJ 2012; 345: 36-41.
Next question
A 32 year old woman attends clinic for assessment of varicose veins. She has suffered for varicose
veins for many years and can trace their development back to when she suffered a complex tibial
fracture. On examination she has marked truncal varicosities with a long tortuous long saphenous
vein. What is the most appropriate next step?

Arrange a venogram

Arrange a venous duplex scan

List her for a trendelenberg procedure

List her for injection foam sclerotherapy

List her for multiple avulsion phlebectomies

This lady is likely to have deep venous incompetence as she will have been immobilised for her tibial
fracture and may well have had a DVT. A duplex scan is mandatory prior to any form of surgical
intervention. A venogram would provide similar information but is more invasive.
Please rate this question:

Discuss and give feedback


Next question

Chronic venous insufficiency and varicose veins

Wide spectrum of disease ranging from minor cosmetic problem through to ulceration and disability.
It is commoner in women than men and is worse during pregnancy. Varicose veins are best
considered as being a saccular dilation of veins (WHO). Chronic venous insufficiency is a series of
tissue changes which occur in relation to pooling of blood in the extremities with associated venous
hypertension occurring as a result of incompetent deep vein valves.

The veins of the lower limb consist of an interconnected network of superficial and deep venous
systems. Varices occur because of localised weakness in the vein wall resulting in dilatation and
reflux of blood due to non union of valve cusps. Histologically the typical changes include fibrous
scar tissue dividing smooth muscle within media in the vessel wall.

Tissue damage in chronic venous insufficiency occurs because of perivascular cytokine leakage
resulting in localised tissue damage coupled with impaired lymphatic flow.

Diagnosis
Typical symptoms of varicose veins include:
 Cosmetic appearance
 Aching
 Ankle swelling that worsens as the day progresses
 Episodic thrombophlebitis
 Bleeding
 Itching

Symptoms of chronic venous insufficiency include:

 Dependant leg pain


 Prominent leg swelling
 Oedema extending beyond the ankle
 Venous stasis ulcers

The typical venous stasis ulcer is:

 Located above the medial malleolus


 Indolent appearance with basal granulation tissue
 Variable degree of scarring
 Non ischaemic edges
 Haemosiderin deposition in the gaiter area (and also lipodermatosclerosis).

Differential diagnosis

 Lower limb arterial disease


 Marjolins ulcer
 Claudication
 Spinal stenosis
 Swelling due to medical causes e.g. CCF.

Exclusion of these differentials is by means of physical examination and ankle brachial pressure
index measurement.

Examination

 Assess for dilated short saphenous vein (popliteal fossa) and palpate for saphena varix
medial to the femoral artery
 Brodie-Trendelenburg test: to assess level of incompetence
 Perthes' walking test: assess if deep venous system competent

Investigation
 Doppler exam: if incompetent a biphasic signal due to retrograde flow is detected
 Duplex scanning: to ensure patent deep venous system (do if DVT or trauma)

All patients should have a Doppler assessment to assess for venous reflux and should be classified
as having uncomplicated varicose veins or varicose veins with associated chronic venous
insufficiency. In the history establishing a previous thrombotic event (DVT/ lower limb fracture) is
important and patients with such a history and all who have evidence of chronic venous insufficiency
should have a duplex scan performed.

Owing to litigation patients with saphenopopliteal incompetence should have a duplex scan
performed and the site marked by scan on the day of surgery.

Treatment
Indications for surgery:

 Cosmetic: majority
 Lipodermatosclerosis causing venous ulceration
 Recurrent superficial thrombophlebitis
 Bleeding from ruptured varix

Condition Therapy

Minor varicose veins - Reassure/ cosmetic therapy


no complications

Symptomatic In those without deep venous insufficiency options include; endothermal


uncomplicated varicose ablation, foam sclerotherapy, saphenofemoral / popliteal disconnection,
veins stripping and avulsions, compression stockings

Varicose veins with Therapy as above (if compression minimum is formal class I stockings)
skin changes

Chronic venous Class 2-3 compression stockings (ensure no arterial disease).


insufficiency or ulcers

 Application of formal compression stockings (usually class II/III). In patients who have
suffered ulceration, compression stockings should be worn long term. Where ulceration is
present and established saphenofemoral reflux exists this should be addressed surgically for
durable relief of symptoms, either at the outset or following ulcer healing.
 Injection sclerotherapy (5% Ethanolamine oleate), foam is increasingly popular, though
transient blindness has been reported. Endo venous laser therapy is another minimally
invasive option
 Sapheno-femoral or sapheno-popliteal ligation, in the case of the LSV; stripping and multiple
phlebectomies

Current best practice guidance


In the United Kingdom the National Institute of Clinical Excellence guidance on varicose veins
suggests that for patients with symptomatic varicose veins the first line procedure of choice should
be endothermal ablation (see reference for more information). Where this is unavailable or
unsuitable then foam sclerotherapy should be the second line option. Surgery is currently the third
line treatment option.

Trendelenburg procedure (sapheno-femoral junction ligation)

 Head tilt 15 degrees and legs abducted


 Oblique incision 1cm medial from artery
 Tributaries ligated (Superficial circumflex iliac vein, Superficial inferior epigastric vein,
Superficial and deep external pudendal vein)
 SF junction double ligated
 Saphenous vein stripped to level of knee/upper calf. NB increased risk of saphenous
neuralgia if stripped more distally

References
Marsden G et al. Diagnosis of management of varicose veins in the the legs: summary of NICE
guidance. BMJ 2013 (347): 30-31.
Next question
Theme: Management of abdominal aortic aneurysms

A. Immediate laparotomy
B. Immediate CT
C. AAA repair during next 48 hours
D. USS in 6 months
E. CT scan during next 4 weeks
F. Endovascular aortic aneurysm repair
G. Discharge
H. Palliate
I. None of the above

Please select the most appropriate management for the scenario given. Each option may be used
once, more than once or not at all.

25. A 66 year old man is referred via the aneurysm screening programme with an abdominal
aortic aneurysm measuring 4.4 cm. Apart from well controlled type 2 DM he is otherwise
well

You answered Immediate laparotomy

The correct answer is USS in 6 months

At this point continue with ultrasound surveillance

26. A 72 year old man has a CT scan for abdominal discomfort and the surgeon suspects
AAA. This shows a 6.6cm aneurysm with a 3.5cm neck and it continues to involve the
right common iliac. The left iliac is occluded. He is hypertensive and has Type 2 DM
which is well controlled.

You answered Immediate laparotomy

The correct answer is AAA repair during next 48 hours

Assuming he is fit enough. This would be a typical 'open ' case as the marked iliac disease
would make EVAR difficult

27. An 89 year old man presents with hypotension and collapse and is found by the staff in the
toilet of his care home. He is moribund and unable to give a clear history. He had suffered
a cardiac arrest in the ambulance but has since been resuscitated and now has a Bp of 95
systolic. He has an obviously palpable AAA.

You answered Immediate laparotomy


The correct answer is Palliate

He will not survive aortic surgery and whilst some may disagree, I would argue that taking
this case to theatre would be futile

Please rate this question:

Discuss and give feedback


Next question

Abdominal aorta aneurysm

 Abdominal aortic aneurysms are a common problem in vascular surgery.


 They may occur as either true or false aneurysm. With the former all 3 layers of the arterial
wall are involved, in the latter only a single layer of fibrous tissue forms the aneurysm wall.
 True abdominal aortic aneurysms have an approximate incidence of 0.06 per 1000 people.
They are commonest in elderly men and for this reason the UK is now introducing the
aneurysm screening program with the aim of performing an abdominal aortic ultrasound
measurement in all men aged 65 years.

Causes

 Several different groups of patients suffer from aneurysmal disease.


 The commonest group is those who suffer from standard arterial disease, i.e. Those who
arehypertensive, have diabetes and have been or are smokers.
 Other patients such as those suffering from connective tissue diseases such as Marfan's
may also develop aneurysms. In patients with abdominal aortic aneurysms the extracellular
matrix becomes disrupted with a change in the balance of collagen and elastic fibres.

Management

 Most abdominal aortic aneurysms are an incidental finding.


 Symptoms most often relate to rupture or impending rupture.
 20% rupture anteriorly into the peritoneal cavity. Very poor prognosis.
 80% rupture posteriorly into the retroperitoneal space
 The risk of rupture is related to aneurysm size, only 2% of aneurysms measuring less than
4cm in diameter will rupture over a 5 year period. This contrasts with 75% of aneurysms
measuring over 7cm in diameter.
 This is well explained by Laplaces' law which relates size to transmural pressure.
 For this reason most vascular surgeons will subject patients with an aneurysm size of 5cm or
greater to CT scanning of the chest, abdomen and pelvis with the aim of delineating anatomy
and planning treatment. Depending upon co-morbidities, surgery is generally offered once
the aneurysm is between 5.5cm and 6cm.
A CT reconstruction showing an infrarenal abdominal aortic aneurysm. The walls of the sac are
calcified which may facilitate identification on plain x-rays

Image sourced from Wikipedia

Indications for surgery


 Symptomatic aneurysms (80% annual mortality if untreated)
 Increasing size above 5.5cm if asymptomatic
 Rupture (100% mortality without surgery)

Surgical procedures
Abdominal aortic aneurysm repair

Procedure:

GA
Invasive monitoring (A-line, CVP, catheter)
Incision: Midline or transverse
Bowel and distal duodenum mobilised to access aorta.
Aneurysm neck and base dissected out and prepared for cross clamp
Systemic heparinisation
Cross clamp (proximal first)
Longitudinal aortotomy
Atherectomy
Deal with back bleeding from lumbar vessels and inferior mesenteric artery
Insert graft either tube or bifurcated depending upon anatomy
Suture using Prolene (3/0 for proximal , distal anastomosis suture varies according to site)
Clamps off: End tidal CO2 will rise owing to effects of reperfusion, at this point major risk of
myocardial events.
Haemostasis
Closure of aneurysm sac to minimise risk of aorto-enteric fistula
Closure: Loop 1 PDS or Prolene to abdominal wall
Skin- surgeons preference

Post operatively:

ITU (Almost all)


Greatest risk of complications following emergency repair
Complications: Embolic- gut and foot infarcts
Cardiac - owing to premorbid states, re-perfusion injury and effects of cross clamp
Wound problems
Later risks related to graft- infection and aorto-enteric fistula

Special groups

Supra renal AAA


These patients will require a supra renal clamp and this carries a far higher risk of complications and
risk of renal failure.

Ruptured AAA
Pre-operatively the management depends upon haemodynamic instability. In patients with
symptoms of rupture (typical pain, haemodynamic compromise and risk factors) then ideally prompt
laparotomy. In those with vague symptoms and haemodynamic stability the ideal test is CT scan to
determine whether rupture has occurred or not. Most common rupture site is retroperitoneal 80%.
These patients will tend to develop retroperitoneal haematoma. This can be disrupted if Bp is
allowed to rise too high so aim for Bp 100mmHg.
Operative details are similar to elective repair although surgery should be swift, blind rushing often
makes the situation worse. Plunging vascular clamps blindly into a pool of blood at the aneurysm
neck carries the risk of injury the vena cava that these patients do not withstand. Occasionally a
supracoeliac clamp is needed to effect temporary control, although leaving this applied for more than
20 minutes tends to carry a dismal outcome.

EVAR
Increasingly patients are now being offered endovascular aortic aneurysm repair. This is undertaken
by surgeons and radiologists working jointly. The morphology of the aneurysm is important and not
all are suitable. Here is a typical list of those features favoring a suitable aneurysm:

 Long neck
 Straight iliac vessels
 Healthy groin vessels

Clearly few AAA patients possess the above and compromise has to be made. The use of
fenestrated grafts can allow supra renal AAA to be treated.

Procedure:

GA
Radiology or theatre
Bilateral groin incisions
Common femoral artery dissected out
Heparinisation
Arteriotomy and insertion of guide wire
Dilation of arteriotomy
Insertion of EVAR Device
Once in satisfactory position it is released
Arteriotomy closed once check angiogram shows good position and no endoleak

Complications:
Endoleaks depending upon site are either Type I or 2. These may necessitate re-intervention and all
EVAR patients require follow up . Details are not needed for MRCS.

References
A reasonable review is provided by:
Sakalihasan N, Limet R, Defawe O. Abdominal aortic aneurysm. Lancet 2005 (365):1577- 1589
Next question
During short saphenous vein surgery for varicose veins which of the following nerves is particularly
at risk?

Sural nerve

Popliteal nerve

Tibial nerve

Femoral nerve

Saphenous nerve

Please rate this question:

Discuss and give feedback


Next question

Saphenous vein

Long saphenous vein


This vein may be harvested for bypass surgery, or removed as treatment for varicose veins with
saphenofemoral junction incompetence.

 Originates at the 1st digit where the dorsal vein merges with the dorsal venous arch of the
foot
 Passes anterior to the medial malleolus and runs up the medial side of the leg
 At the knee, it runs over the posterior border of the medial epicondyle of the femur bone
 Then passes laterally to lie on the anterior surface of the thigh before entering an opening in
the fascia lata called the saphenous opening
 It joins with the femoral vein in the region of the femoral triangle at the saphenofemoral
junction

Tributaries

 Medial marginal
 Superficial epigastric
 Superficial iliac circumflex
 Superficial external pudendal veins

Short saphenous vein

 Originates at the 5th digit where the dorsal vein merges with the dorsal venous arch of the
foot, which attaches to the great saphenous vein.
 It passes around the lateral aspect of the foot (inferior and posterior to the lateral malleolus)
and runs along the posterior aspect of the leg (with the sural nerve)
 Passes between the heads of the gastrocnemius muscle, and drains into the popliteal vein,
approximately at or above the level of the knee joint.

Next question
A 21 year old badminton player attends A&E with a painful, swollen right arm. He is right handed.
Clinically he has dusky fingers and his upper limb pulses are present. An axillary vein thrombosis is
confirmed. What is the best acute treatment to achieve vein patency?

Intravenous heparin

Warfarin

Catheter directed tPA

Low molecular weight heparin

Aspirin

Heparin and warfarin prevent propagation of the clot.


Please rate this question:

Discuss and give feedback


Next question

Axillary vein thrombosis

 1-2% of all deep venous thrombosis


 Primary cause is associated with trauma, thoracic outlet obstruction or repeated effort in a
dominant arm (young active individuals)
 Secondary causes include central line insertion, malignancy, pacemakers

Clinical features

 Pain and swelling (non pitting)


 Numbness
 Discolouration: mottling, dusky
 Pulses present
 Congested veins

Investigations
 FBC: viscosity, platelet function
 Clotting
 Liver function tests
 D-dimer
 Duplex scan: investigation of choice
 CT scan: thoracic outlet obstruction

Treatment

 Local catheter directed TPA


 Heparin
 Warfarin

Next question
A 23 year old man presents with a brachial artery embolus. A cervical rib is suspected as being the
underlying cause. From which of the following vertebral levels do they most often arise?

C7

C5

C4

C3

C2

They usually arise from C7.


Please rate this question:

Discuss and give feedback


Next question

Cervical ribs

 0.2-0.4% incidence
 Most cases present with neurological symptoms
 Consist of an anomalous fibrous band that often originates from C7 and may arc towards,
but rarely reaches the sternum
 Congenital cases may present around the third decade, some cases are reported to occur
following trauma
 Bilateral in up to 70%
 Compression of the subclavian artery may produce absent radial pulse on clinical
examination and in particular may result in a positive Adsons test (lateral flexion of the neck
towards the symptomatic side and traction of the symptomatic arm- leads to obliteration of
radial pulse)
 Treatment is most commonly undertaken when there is evidence of neurovascular
compromise. A transaxillary approach is the traditional operative method for excision.
Image sourced from Wikipedia

3D reconstruction of a left-sided cervical rib


Next question
A 73 year old man with rest pain and ulceration of the foot undergoes a femoro-distal bypass graft
with a PTFE graft. At the end of the procedure there are good distal foot pulses and a warm pink
foot. Over the ensuing 60 days the foot becomes progressively cooler and the pulses diminish. What
is the most likely underlying explanation for this process?

Embolus

Neo-intimal flap

Neo-intimal hyperplasia

Polyarteritis

Steal syndrome

Neo-intimal hyperplasia in distal arterial anastamoses may be reduced by use of a Miller Cuff when
PTFE is the bypass conduit.

PTFE may induce neo-intimal hyperplasia with subsequent occlusion of the distal anastomosis. In
more proximal arterial bypass surgery the process of neo-intimal hyperplasia is not sufficient to
cause anastomotic occlusion. However, distal bypasses are at greater risk and if vein cannot be
used as a conduit then the distal end of the PTFE should anastomosed to a vein cuff to minimise the
risk of neo-intimal hyperplasia.
Please rate this question:

Discuss and give feedback


Next question

Anastomoses

 A wide variety of anastomoses are constructed in surgical practice. Essentially the term
refers to the restoration of luminal continuity. As such they are a feature of both abdominal
and vascular surgery.

Visceral anastomoses

For an anastomosis to heal three criteria need to be fulfilled:

 Adequate blood supply


 Mucosal apposition
 Minimal tension

When these are compromised the anastomosis may break down. Even in the best surgical hands
some anastomoses are more prone to dehiscence than others. Oesophageal and rectal
anastomoses are more prone to leakage and reported leak rates following oesophageal and rectal
surgery can be as high as 20%. This figure includes radiological leaks and those with a clinically
significant leak will be of a lower order of magnitude. As a rule small bowel anastomoses heal most
reliably.

The decision as to how best to achieve mucosal apposition is one for each surgeon. Some will prefer
the use of stapling devices as they are quicker to use, others will prefer to perform a sutured
anastomosis. The attention to surgical technique is more important than the method chosen and a
poorly constructed stapled anastomosis in thickened tissue is far more prone to leakage than a hand
sewn anastomosis in the same circumstances.

If an anastomosis looks unsafe then it may be best not to construct one at all. In colonic surgery this
is relatively clear cut and most surgeons would bring out an end colostomy. In situations such as
oesophageal surgery this is far more problematic and colonic interposition may be required in this
situation.

Vascular anastomoses

Most arterial surgery involving bypasses or aneurysm repairs will require construction of an arterial
anastomosis. Technique is important and for small diameter distal arterial surgery the intimal
hyperplasia resulting from a badly constructed anastomosis may render the whole operation futile
before the patient leaves hospital.

Some key points about vascular anastomoses:

 Always use non absorbable monofilament suture (e.g. Polypropylene).


 Round bodied needle.
 Correct size for anastamosis ( i.e. 6/0 prolene for bottom end of a femoro-distal bypass).
 Suture should be continuous and from inside to outside of artery to avoid raising an intimal
flap.

Next question
Theme: Lymphoedema Management

A. Homans operation
B. Charles operation
C. Frusemide at high doses
D. Frusemide at low doses
E. Multilayer compression bandaging
F. Lymphovenous anastomosis

Please select the most appropriate management for the lymphoedema scenario given. Each option
may be used once, more than once or not at all.

32. A 52 year old lady develops lower leg swelling following redo varicose vein surgery.
There is evidence of swelling of the left leg up to the knee. The overlying skin appears
healthy.

You answered Homans operation

The correct answer is Multilayer compression bandaging

Unfortunately lymphoedema may complicate redo varicose vein surgery (in 0.5% of
cases). As the presentation is mild, she should be managed using compression hosiery.
Diuretics do not help in cases of true lymphoedema and a dramatic response suggests an
alternative underlying cause.

33. A 57 year old lady has suffered from lymphoedema for many years. The left leg is swollen
to the mid thigh. Severe limb deformity has developed as a result of process and in spite of
compression hosiery. Lymphoscintography shows no patent lymphatics in the proximal
leg. The overlying skin is healthy.

Homans operation

Surgery is indicated in less than 10% of cases. However, severe deformity is one of the
indications for surgery. Lymphovenous anastomosis is indicated where the proximal
lymphatics are not patent. When the overlying skin is healthy (and limb deformity a
problem), a Homans procedure is a reasonable first line operative option.

34. A 38 year old lady is troubled by lymphoedema that occurred following a block dissection
of the groin for malignant melanoma many years previously. Despite therapy with
compression bandages she has persistent lower leg swelling impairing her activities of
daily living. She has no evidence of recurrent malignancy. Lymphoscintography
demonstrates occlusion of the groin lymphatics. However, the distal lymphatic system
appears healthy.
You answered Homans operation

The correct answer is Lymphovenous anastomosis

In young patients with proximal disease and healthy distal lymphatics a lymphovenous
anastomosis may be considered. Such cases are rare.

Please rate this question:

Discuss and give feedback


Next question

Lymphoedema

 Due to impaired lymphatic drainage in the presence of normal capillary function.


 Lymphoedema causes the accumulation of protein rich fluid, subdermal fibrosis and dermal
thickening.
 Characteristically fluid is confined to the epifascial space (skin and subcutaneous tissues);
muscle compartments are free of oedema. It involves the foot, unlike other forms of oedema.
There may be a 'buffalo hump' on the dorsum of the foot and the skin cannot be pinched due
to subcutaneous fibrosis.

Causes of lymphoedema

Primary  Congenital < 1 year: sporadic, Milroy's disease


 Onset 1-35 years: sporadic, Meige's disease
 > 35 years: Tarda

Secondary  Bacterial/fungal/parasitic infection (filariasis)


 Lymphatic malignancy
 Radiotherapy to lymph nodes
 Surgical resection of lymph nodes
 DVT
 Thrombophlebitis

Indications for surgery

 Marked disability or deformity from limb swelling


 Lymphoedema caused by proximal lymphatic obstruction with patent distal lymphatics
suitable for a lymphatic drainage procedure
 Lymphocutaneous fistulae and megalymphatics
Procedures
Homans operation Reduction procedure with preservation of overlying skin (which must be in
good condition). Skin flaps are raised and the underlying tissue excised.
Limb circumference typically reduced by a third.

Charles operation All skin and subcutaneous tissue around the calf are excised down to the
deep fascia. Split skin grafts are placed over the site. May be performed if
overlying skin is not in good condition. Larger reduction in size than with
Homans procedure.

Lymphovenous Identifiable lymphatics are anastomosed to sub dermal venules. Usually


anastamosis indicated in 2% of patients with proximal lymphatic obstruction and normal
distal lymphatics.

Next question
Which of the following is not a typical feature of a chronic venous leg ulcer?

Heaped raised borders if the ulcer has been present more than 5 years

Evidence of surrounding lipodermatosclerosis

Irregular shape to the ulcer

20% of cases will have a previous history of deep vein thrombosis

Haemosiderin deposits in surrounding skin

The borders of the ulcer are often well defined even though they may be irregular. Heaped or raised
borders should raise suspicion of a marjolins ulcer.
Please rate this question:

Discuss and give feedback


Next question

Lower leg ulcers

Venous leg ulcers

 Most due to venous hypertension, secondary to chronic venous insufficiency (other causes
include calf pump dysfunction or neuromuscular disorders)
 Ulcers form due to capillary fibrin cuff or leucocyte sequestration
 Features of venous insufficiency include oedema, brown pigmentation, lipodermatosclerosis,
eczema
 Location above the ankle, painless
 Deep venous insufficiency is related to previous DVT and superficial venous insufficiency is
associated with varicose veins
 Doppler ultrasound looks for presence of reflux and duplex ultrasound looks at the anatomy/
flow of the vein
 Management: 4 layer compression banding after exclusion of arterial disease or surgery
 If fail to heal after 12 weeks or >10cm 2 skin grafting may be needed

Marjolin's ulcer
Image sourced from Wikipedia

 Squamous cell carcinoma


 Occurring at sites of chronic inflammation e.g; burns, osteomyelitis after 10-20 years
 Mainly occur on the lower limb

Arterial ulcers

 Occur on the toes and heel


 Painful
 There may be areas of gangrene
 Cold with no palpable pulses
 Low ABPI measurements

Neuropathic ulcers

 Commonly over plantar surface of metatarsal head and plantar surface of hallux
 The plantar neuropathic ulcer is the condition that most commonly leads to amputation in
diabetic patients
 Due to pressure
 Management includes cushioned shoes to reduce callous formation

Pyoderma gangrenosum
Image sourced from Wikipedia

 Associated with inflammatory bowel disease/RA


 Can occur at stoma sites
 Erythematous nodules or pustules which ulcerate

Next question
Which of the following is not a typical feature of an arterial leg ulcer?

Well demarcated edges

A grey - white base to the ulcer

Men are affected more than women

Painful

Ankle swelling

Ankle swelling is often absent. If present it may be due to mixed arteriovenous disease. With mixed
disease the arterial component is treated first.
Please rate this question:

Discuss and give feedback


Next question

Peripheral vascular disease

Indications for surgery to revascularise the lower limb

 Intermittent claudication
 Critical ischaemia
 Ulceration
 Gangrene

Intermittent claudication that is not disabling may provide a relative indication, whilst the other
complaints are often absolute indications depending upon the frailty of the patient.

Assessment

 Clinical examination
 Ankle brachial pressure index measurement
 Duplex arterial ultrasound
 Angiography (standard, CT or MRI): usually performed only if intervention being considered.
Angioplasty
In order for angioplasty to be undertaken successfully the artery has to be accessible. The lesion
relatively short and reasonable distal vessel runoff. Longer lesions may be amenable to sub-intimal
angioplasty.

Surgery
Surgery will be undertaken where attempts at angioplasty have either failed or are unsuitable.
Bypass essentially involves bypassing the affected arterial segment by utilising a graft to run from
above the disease to below the disease. As with angioplasty good runoff improves the outcome.

Some key concepts with bypass surgery

Superficial femoral artery occlusion to the above knee popliteal


In the ideal scenario, vein (either in situ or reversed LSV) would the used as a conduit. However,
prosthetic material has reasonable 5 year patency rates and some would advocate using this in
preference to vein so that vein can be used for other procedures in the future. In general terms either
technique is usually associated with an excellent outcome (if run off satisfactory).

Procedure

 Artery dissected out, IV heparin 3,000 units given and then the vessels are cross clamped
 Longitudinal arteriotomy
 Graft cut to size and tunneled to arteriotomy sites
 Anastomosis to femoral artery usually with 5/0 'double ended' Prolene suture
 Distal anastomosis usually using 6/0 'double ended' Prolene

Distal disease

 Femoro-distal bypass surgery takes longer to perform, is more technically challenging and
has higher failure rates.
 In elderly diabetic patients with poor runoff a primary amputation may well be a safer and
more effective option. There is no point in embarking on this type of surgery in patients who
are wheelchair bound.
 In femorodistal bypasses vein gives superior outcomes to PTFE.

Rules

 Vein mapping 1st to see whether there is suitable vein (the preferred conduit). Sub intimal
hyperplasia occurs early when PTFE is used for the distal anastomosis and will lead to early
graft occlusion and failure.
 Essential operative procedure as for above knee fem-pop.
 If there is insufficient vein for the entire conduit then vein can be attached to the end of the
PTFE graft and then used for the distal anastomosis. This type of 'vein boot' is technically
referred to as a Miller Cuff and is associated with better patency rates than PTFE alone.
 Remember the more distal the arterial anastomosis the lower the success rate.
Theme: Lower limb ulceration

A. Mixed ulcer
B. Chronic obliterative arterial disease
C. Superficial venous insufficiency
D. Deep venous insufficiency
E. Neuropathic ulcer
F. Basal cell carcinoma
G. Squamous cell carcinoma

Please select the most likely cause of ulceration for the scenario given. Each option may be used
once, more than once or not at all.

37. A 65 year old diabetic female presents with a painless ulcer at the medial malleolus, it has
been present for the past 16 years. On examination she has evidence of truncal varicosities
and a brownish discolouration of the skin overlying the affected area.

You answered Mixed ulcer

The correct answer is Superficial venous insufficiency

Theme from September 2012 Exam


Venous ulcers are usually associated with features of venous insufficiency. These include
haemosiderin deposition and varicose veins. Neuropathic ulcers will tend to present at sites
of pressure, which is not typically at the medial malleolus.

38. A 71 year old man presents with a painful lower calf ulcer, mild pitting oedema and an
ABPI of 0.3.

You answered Mixed ulcer

The correct answer is Chronic obliterative arterial disease

Painful ulcers associated with a low ABPI are usually arterial in nature. The question does
not indicate that features of chronic venous insufficiency are present. Patients may have
mild pitting oedema as many vascular patients will also have ischaemic heart disease and
elevated right heart pressures. The absence of more compelling signs of venous
insufficiency makes a mixed ulcer less likely.

39. A 79 year old retired teacher has had an ulcer for 15 years. It is at the medial malleolus and
has associated lipodermatosclerosis of the lower limb. The ulcer base is heaped up and
irregular.
You answered Mixed ulcer

The correct answer is Squamous cell carcinoma

If after many years an ulcer becomes heaped up and irregular, with rolled edges then
suspect a
squamous cell carcinoma.

Please rate this question:

Discuss and give feedback


Next question

Lower leg ulcers

Venous leg ulcers

 Most due to venous hypertension, secondary to chronic venous insufficiency (other causes
include calf pump dysfunction or neuromuscular disorders)
 Ulcers form due to capillary fibrin cuff or leucocyte sequestration
 Features of venous insufficiency include oedema, brown pigmentation, lipodermatosclerosis,
eczema
 Location above the ankle, painless
 Deep venous insufficiency is related to previous DVT and superficial venous insufficiency is
associated with varicose veins
 Doppler ultrasound looks for presence of reflux and duplex ultrasound looks at the anatomy/
flow of the vein
 Management: 4 layer compression banding after exclusion of arterial disease or surgery
 If fail to heal after 12 weeks or >10cm 2 skin grafting may be needed

Marjolin's ulcer
Image sourced from Wikipedia

 Squamous cell carcinoma


 Occurring at sites of chronic inflammation e.g; burns, osteomyelitis after 10-20 years
 Mainly occur on the lower limb

Arterial ulcers

 Occur on the toes and heel


 Painful
 There may be areas of gangrene
 Cold with no palpable pulses
 Low ABPI measurements

Neuropathic ulcers

 Commonly over plantar surface of metatarsal head and plantar surface of hallux
 The plantar neuropathic ulcer is the condition that most commonly leads to amputation in
diabetic patients
 Due to pressure
 Management includes cushioned shoes to reduce callous formation

Pyoderma gangrenosum
Image sourced from Wikipedia

 Associated with inflammatory bowel disease/RA


 Can occur at stoma sites
 Erythematous nodules or pustules which ulcerate

Next question
A 66 year old man is admitted with severe angina. There is a lesion of the proximal left anterior
descending coronary artery. Which of the following would be the most suitable conduit for bypass?

Long saphenous vein

Short saphenous vein

Cephalic vein

Internal mammary artery

Thoraco-acromial artery

The internal mammary artery is an excellent conduit for coronary artery bypass. It has better long
term patency rates than venous grafts. The thoraco-acromial artery is seldom used.
Please rate this question:

Discuss and give feedback


Next question

Cardiopulmonary bypass

Indications for surgery

 Left main stem stenosis or equivalent (proximal LAD and proximal circumflex)
 Triple vessel disease
 Diffuse disease unsuitable for PCI

The guidelines state that CABG is the preferred treatment in high-risk patients with severe
ventricular dysfunction or diabetes mellitus.

Technique
General anaesthesia
Central and arterial lines
Midline sternotomy or left sub mammary incision
Aortic root and pericardium dissected
Heart inspected

Bypass grafting may be performed using a cardiopulmonary bypass circuit with cardiac arrest or
using a number of novel 'off pump' techniques.
Procedure cardiopulmonary bypass

 Aortic root cannulated


 Right atrial cannula
 Circuit primed and patient fully heparinised (30,000 Units unfractionated heparin) as the
circuit is highly thrombogenic
 Flow established through circuit
 Aortic cross clamp applied
 Cardioplegia solution instilled into the aortic root below cross clamp
 Heart now asystolic and ready for surgery

Off pump techniques are evolving on a constant basis and details are beyond the scope of the
MRCS.

Conduits for bypass


> Internal mammary artery is best. Use of both is associated with increased risk of sternal wound
dehiscence. However, many surgeons will use both especially for redo surgery.
> Radial artery harvested from forearm. Ensure ulnar collateral working first!
> Reversed long saphenous vein grafts
Typically anastamosed using 7/0-8/0 prolene sutures (distally) and 6/0 prolene for top end.

Once flow established


Anticoagulation reversed using protamine
Patient is taken off bypass
Inotropes given if needed
Sternum closed using sternal closure device or stainless steel wire

Complications

 Post perfusion syndrome: transient cognitive impairment


 Non union of the sternum; due to loss of the internal thoracic artery
 Myocardial infarction
 Late graft stenosis
 Acute renal failure
 Stroke
 Gastrointestinal

Perioperative risk is quantified using the Parsonnet and Euroscores and unit outcomes are audited
using this data.

Reference
Eagle KA, Guyton RA, Davidoff R, et al: ACC/AHA 2004 guideline update for coronary artery bypass
graft surgery: A report of the American College of Cardiology/American Heart Association Task
Force on Practice Guidelines (Committee to Revise the 1999 Guidelines for Coronary Artery Bypass
Graft Surgery). Circulation 2004; 110
Next question
Concerning proximal aortic dissection (Debakey types 1 and 2/ Stanford type A) which statement is
false?

The intimal tear is typically >50% of the aortic circumference.

It is usually treated using an endovascular approach.

They have a 50% mortality in the first 2 days.

Arch reconstructions may require deep hypothermic circulatory arrest.

Target systolic pressure of <110mmHg should be maintained.

Usually open surgery is required for these lesions as customised grafts are not usually available for
this type of repair yet.
Please rate this question:

Discuss and give feedback


Next question

Aortic dissection

 More common than rupture of the abdominal aorta


 33% of patients die within the first 24 hours, and 50% die within 48 hours if no treatment
received
 Associated with hypertension
 Features of aortic dissection: tear in the intimal layer, followed by formation and propagation
of a subintimal hematoma. Cystic medial necrosis (Marfan's)
 Most common site of dissection: 90% occurring within 10 centimetres of the aortic valve

Stanford Classification
Type Location Treatment

A Ascending aorta/ aortic root Surgery- aortic root replacement

B Descending aorta Medical therapy with antihypertensives


DeBakey classification
Type Site affected

I Ascending aorta, aortic arch, descending aorta

II Ascending aorta only

III Descending aorta distal to left subclavian artery

Clinical features

 Tearing, sudden onset chest pain (painless 10%)


 Hypertension or Hypotension
 A blood pressure difference (in each arm) greater than 20 mm Hg
 Neurologic deficits (20%)

Investigations

 CXR: widened mediastinum, abnormal aortic knob, ring sign, deviation of the
trachea/oesophagus
 CT angiography of the thoracic aorta
 MRI angiography
 Conventional angiography (now rarely used diagnostically)

Management

 Beta-blockers: aim HR 60-80 bpm and systolic BP 100-120 mm Hg


 For type A dissections the standard of care is aortic root replacement

Next question
A 67 year old male is diagnosed as having a 7cm infra renal abdominal aortic aneurysm. What is the
likely risk of rupture over the next 5 years?

<10%

20%

25%

75%

35%

Risks of abdominal aortic aneurysm rupture (over 5 years):

 5-5.9cm = 25%
 6-6.9cm= 35%
 7cm and over = 75%

Aneuryms greater than 5cm in diameter on USS should be formally assessed using CT scanning
with arterial phases to delineate anatomy and facilitate surgical planning.
Please rate this question:

Discuss and give feedback


Next question

Abdominal aorta aneurysm

 Abdominal aortic aneurysms are a common problem in vascular surgery.


 They may occur as either true or false aneurysm. With the former all 3 layers of the arterial
wall are involved, in the latter only a single layer of fibrous tissue forms the aneurysm wall.
 True abdominal aortic aneurysms have an approximate incidence of 0.06 per 1000 people.
They are commonest in elderly men and for this reason the UK is now introducing the
aneurysm screening program with the aim of performing an abdominal aortic ultrasound
measurement in all men aged 65 years.

Causes
 Several different groups of patients suffer from aneurysmal disease.
 The commonest group is those who suffer from standard arterial disease, i.e. Those who
arehypertensive, have diabetes and have been or are smokers.
 Other patients such as those suffering from connective tissue diseases such as Marfan's
may also develop aneurysms. In patients with abdominal aortic aneurysms the extracellular
matrix becomes disrupted with a change in the balance of collagen and elastic fibres.

Management

 Most abdominal aortic aneurysms are an incidental finding.


 Symptoms most often relate to rupture or impending rupture.
 20% rupture anteriorly into the peritoneal cavity. Very poor prognosis.
 80% rupture posteriorly into the retroperitoneal space
 The risk of rupture is related to aneurysm size, only 2% of aneurysms measuring less than
4cm in diameter will rupture over a 5 year period. This contrasts with 75% of aneurysms
measuring over 7cm in diameter.
 This is well explained by Laplaces' law which relates size to transmural pressure.
 For this reason most vascular surgeons will subject patients with an aneurysm size of 5cm or
greater to CT scanning of the chest, abdomen and pelvis with the aim of delineating anatomy
and planning treatment. Depending upon co-morbidities, surgery is generally offered once
the aneurysm is between 5.5cm and 6cm.

A CT reconstruction showing an infrarenal abdominal aortic aneurysm. The walls of the sac are
calcified which may facilitate identification on plain x-rays
Image sourced from Wikipedia

Indications for surgery

 Symptomatic aneurysms (80% annual mortality if untreated)


 Increasing size above 5.5cm if asymptomatic
 Rupture (100% mortality without surgery)
Surgical procedures
Abdominal aortic aneurysm repair

Procedure:

GA
Invasive monitoring (A-line, CVP, catheter)
Incision: Midline or transverse
Bowel and distal duodenum mobilised to access aorta.
Aneurysm neck and base dissected out and prepared for cross clamp
Systemic heparinisation
Cross clamp (proximal first)
Longitudinal aortotomy
Atherectomy
Deal with back bleeding from lumbar vessels and inferior mesenteric artery
Insert graft either tube or bifurcated depending upon anatomy
Suture using Prolene (3/0 for proximal , distal anastomosis suture varies according to site)
Clamps off: End tidal CO2 will rise owing to effects of reperfusion, at this point major risk of
myocardial events.
Haemostasis
Closure of aneurysm sac to minimise risk of aorto-enteric fistula
Closure: Loop 1 PDS or Prolene to abdominal wall
Skin- surgeons preference

Post operatively:

ITU (Almost all)


Greatest risk of complications following emergency repair
Complications: Embolic- gut and foot infarcts
Cardiac - owing to premorbid states, re-perfusion injury and effects of cross clamp
Wound problems
Later risks related to graft- infection and aorto-enteric fistula

Special groups

Supra renal AAA


These patients will require a supra renal clamp and this carries a far higher risk of complications and
risk of renal failure.

Ruptured AAA
Pre-operatively the management depends upon haemodynamic instability. In patients with
symptoms of rupture (typical pain, haemodynamic compromise and risk factors) then ideally prompt
laparotomy. In those with vague symptoms and haemodynamic stability the ideal test is CT scan to
determine whether rupture has occurred or not. Most common rupture site is retroperitoneal 80%.
These patients will tend to develop retroperitoneal haematoma. This can be disrupted if Bp is
allowed to rise too high so aim for Bp 100mmHg.
Operative details are similar to elective repair although surgery should be swift, blind rushing often
makes the situation worse. Plunging vascular clamps blindly into a pool of blood at the aneurysm
neck carries the risk of injury the vena cava that these patients do not withstand. Occasionally a
supracoeliac clamp is needed to effect temporary control, although leaving this applied for more than
20 minutes tends to carry a dismal outcome.
EVAR
Increasingly patients are now being offered endovascular aortic aneurysm repair. This is undertaken
by surgeons and radiologists working jointly. The morphology of the aneurysm is important and not
all are suitable. Here is a typical list of those features favoring a suitable aneurysm:

 Long neck
 Straight iliac vessels
 Healthy groin vessels

Clearly few AAA patients possess the above and compromise has to be made. The use of
fenestrated grafts can allow supra renal AAA to be treated.

Procedure:

GA
Radiology or theatre
Bilateral groin incisions
Common femoral artery dissected out
Heparinisation
Arteriotomy and insertion of guide wire
Dilation of arteriotomy
Insertion of EVAR Device
Once in satisfactory position it is released
Arteriotomy closed once check angiogram shows good position and no endoleak

Complications:
Endoleaks depending upon site are either Type I or 2. These may necessitate re-intervention and all
EVAR patients require follow up . Details are not needed for MRCS.

References
A reasonable review is provided by:
Sakalihasan N, Limet R, Defawe O. Abdominal aortic aneurysm. Lancet 2005 (365):1577- 1589
Next question
Theme: Venous disease

A. No further management needed


B. Injection sclerotherapy using 0.5% Sodium tetradecyl sulphate
C. Injection sclerotherapy using 5% phenol
D. Long saphenous vein ligation
E. Long saphenous vein stripped to the ankle
F. Long saphenous vein stripped to the knee
G. Doppler scan
H. Duplex scan
I. Endothermal ablation

Please select the most appropriate management plan for the scenario given. Each option may be
used once, more than once or not at all.

43. A 42 year old teacher presents with an ulcer associated with varicose veins in the long
saphenous vein territory. Apart from a DVT 1 year ago, she has no other past medical
history.

You answered No further management needed

The correct answer is Duplex scan

This patient needs a duplex scan to assess the patency of her deep venous system before
surgery can be undertaken. Other indications for duplex scan include recurrent varicose
veins or complications.

44. A 42 year old accountant presents with thrombophlebitis of a long standing varicosity of
the inner thigh. His past medical history is unremarkable apart from a conservatively
managed tibial fracture of the ipsilateral limb 10 years ago. Doppler and clinical
assessment demonstrate saphenofemoral junction incompetence.

You answered No further management needed

The correct answer is Duplex scan

Tibial fractures are a well recognised risk factor for occult lower limb deep venous
thrombosis and most surgeons would perform a duplex scan to exclude deep venous
insufficiency prior to surgery.

45. A 43 year old lady presents with a thigh varicosity in the territory of the long saphenous
vein. She underwent endovenous laser therapy 5 years previously. On duplex assessment
she has a patent deep venous system and sapheno-femoral junction incompetence.
You answered No further management needed

The correct answer is Long saphenous vein stripped to the knee

In the UK, NICE, suggest the use non operative measures such as endothermal ablation for
first time varicose veins. Recurrences respond far less favourably and are best managed
with surgery.

Please rate this question:

Discuss and give feedback


Next question

Chronic venous insufficiency and varicose veins

Wide spectrum of disease ranging from minor cosmetic problem through to ulceration and disability.
It is commoner in women than men and is worse during pregnancy. Varicose veins are best
considered as being a saccular dilation of veins (WHO). Chronic venous insufficiency is a series of
tissue changes which occur in relation to pooling of blood in the extremities with associated venous
hypertension occurring as a result of incompetent deep vein valves.

The veins of the lower limb consist of an interconnected network of superficial and deep venous
systems. Varices occur because of localised weakness in the vein wall resulting in dilatation and
reflux of blood due to non union of valve cusps. Histologically the typical changes include fibrous
scar tissue dividing smooth muscle within media in the vessel wall.

Tissue damage in chronic venous insufficiency occurs because of perivascular cytokine leakage
resulting in localised tissue damage coupled with impaired lymphatic flow.

Diagnosis
Typical symptoms of varicose veins include:

 Cosmetic appearance
 Aching
 Ankle swelling that worsens as the day progresses
 Episodic thrombophlebitis
 Bleeding
 Itching

Symptoms of chronic venous insufficiency include:

 Dependant leg pain


 Prominent leg swelling
 Oedema extending beyond the ankle
 Venous stasis ulcers
The typical venous stasis ulcer is:

 Located above the medial malleolus


 Indolent appearance with basal granulation tissue
 Variable degree of scarring
 Non ischaemic edges
 Haemosiderin deposition in the gaiter area (and also lipodermatosclerosis).

Differential diagnosis

 Lower limb arterial disease


 Marjolins ulcer
 Claudication
 Spinal stenosis
 Swelling due to medical causes e.g. CCF.

Exclusion of these differentials is by means of physical examination and ankle brachial pressure
index measurement.

Examination

 Assess for dilated short saphenous vein (popliteal fossa) and palpate for saphena varix
medial to the femoral artery
 Brodie-Trendelenburg test: to assess level of incompetence
 Perthes' walking test: assess if deep venous system competent

Investigation

 Doppler exam: if incompetent a biphasic signal due to retrograde flow is detected


 Duplex scanning: to ensure patent deep venous system (do if DVT or trauma)

All patients should have a Doppler assessment to assess for venous reflux and should be classified
as having uncomplicated varicose veins or varicose veins with associated chronic venous
insufficiency. In the history establishing a previous thrombotic event (DVT/ lower limb fracture) is
important and patients with such a history and all who have evidence of chronic venous insufficiency
should have a duplex scan performed.

Owing to litigation patients with saphenopopliteal incompetence should have a duplex scan
performed and the site marked by scan on the day of surgery.

Treatment
Indications for surgery:
 Cosmetic: majority
 Lipodermatosclerosis causing venous ulceration
 Recurrent superficial thrombophlebitis
 Bleeding from ruptured varix

Condition Therapy

Minor varicose veins - Reassure/ cosmetic therapy


no complications

Symptomatic In those without deep venous insufficiency options include; endothermal


uncomplicated varicose ablation, foam sclerotherapy, saphenofemoral / popliteal disconnection,
veins stripping and avulsions, compression stockings

Varicose veins with Therapy as above (if compression minimum is formal class I stockings)
skin changes

Chronic venous Class 2-3 compression stockings (ensure no arterial disease).


insufficiency or ulcers

 Application of formal compression stockings (usually class II/III). In patients who have
suffered ulceration, compression stockings should be worn long term. Where ulceration is
present and established saphenofemoral reflux exists this should be addressed surgically for
durable relief of symptoms, either at the outset or following ulcer healing.
 Injection sclerotherapy (5% Ethanolamine oleate), foam is increasingly popular, though
transient blindness has been reported. Endo venous laser therapy is another minimally
invasive option
 Sapheno-femoral or sapheno-popliteal ligation, in the case of the LSV; stripping and multiple
phlebectomies

Current best practice guidance


In the United Kingdom the National Institute of Clinical Excellence guidance on varicose veins
suggests that for patients with symptomatic varicose veins the first line procedure of choice should
be endothermal ablation (see reference for more information). Where this is unavailable or
unsuitable then foam sclerotherapy should be the second line option. Surgery is currently the third
line treatment option.

Trendelenburg procedure (sapheno-femoral junction ligation)


 Head tilt 15 degrees and legs abducted
 Oblique incision 1cm medial from artery
 Tributaries ligated (Superficial circumflex iliac vein, Superficial inferior epigastric vein,
Superficial and deep external pudendal vein)
 SF junction double ligated
 Saphenous vein stripped to level of knee/upper calf. NB increased risk of saphenous
neuralgia if stripped more distally

References
Marsden G et al. Diagnosis of management of varicose veins in the the legs: summary of NICE
guidance. BMJ 2013 (347): 30-31.
Next question
A 24-year-old female is referred to the acute surgical team as she is noted to have an absent left
radial pulse. Apart from some dizziness and lethargy, the patient does not have any features
suggestive of an acute ischaemic limb. Blood tests are as follows:

Na+ 136 mmol/l

K+ 4.1 mmol/l

Urea 2.3 mmol/l

Creatinine 77 µmol/l

ESR 66 mm/hr

What is the most likely diagnosis?

Turner's syndrome

Takayasu's arteritis

Kawasaki disease

Coarctation of the aorta

Breast carcinoma with local spread

 Inflammatory, obliterative arteritis affecting aorta and branches


 Females> Males
 Symptoms may include upper limb claudication
 Clinical findings include diminished or absent pulses
 ESR often affected during the acute phase

Please rate this question:

Discuss and give feedback


Next question

Takayasu's arteritis
Takayasu's arteritis is a large vessel vasculitis. It typically causes occlusion of the aorta and
questions commonly refer to an absent limb pulse. It is more common in females and Asian people

Associations

 renal artery stenosis

Management

 steroids

Next question
Theme: Leg swelling

A. Milroy's disease
B. Meige's disease
C. Lymphoedema tarda
D. Filariasis
E. Tuberculosis
F. Locally advanced bladder carcinoma
G. Malaria

Which is the most likely diagnosis for the scenario given? Each option may be used once, more than
once or not at all.

47. The medical team refer a 72 year old lady with a bilateral swollen legs. Deep vein
thrombosis has been excluded and there is no response to diuretics. On further questioning,
the patient reveals that she was born with the swelling in both of her legs.

Milroy's disease

Milroy's disease is present from birth and is due to failure of the lymphatic vessels to
develop. Note that Meige's disease develops AFTER birth.

48. A 52 year old woman presents with rapid swelling of the left leg. The swelling is greater in
the thigh compared to the calf.

You answered Milroy's disease

The correct answer is Locally advanced bladder carcinoma

Always consider a malignancy in an older adult with new lymphoedema in a limb,


especially if the swelling is greater proximally than distally. If malignancy is excluded
consider the diagnosis of lymphoedema tarda.

49. A 34 year old African teacher attends A&E with a swollen leg. She has been in England
for 2 weeks. She lives in an area prevalent with mosquitoes and where there is poor
sanitation.

You answered Milroy's disease

The correct answer is Filariasis

Filariasis is caused by the nematode Wuchereria bancrofti, which is mainly spread by


mosquito. The oedema can be gross leading to elephantitis. Treatment is with
diethylcarbamazine.

Please rate this question:

Discuss and give feedback


Next question

Lymphoedema

 Due to impaired lymphatic drainage in the presence of normal capillary function.


 Lymphoedema causes the accumulation of protein rich fluid, subdermal fibrosis and dermal
thickening.
 Characteristically fluid is confined to the epifascial space (skin and subcutaneous tissues);
muscle compartments are free of oedema. It involves the foot, unlike other forms of oedema.
There may be a 'buffalo hump' on the dorsum of the foot and the skin cannot be pinched due
to subcutaneous fibrosis.

Causes of lymphoedema

Primary  Congenital < 1 year: sporadic, Milroy's disease


 Onset 1-35 years: sporadic, Meige's disease
 > 35 years: Tarda

Secondary  Bacterial/fungal/parasitic infection (filariasis)


 Lymphatic malignancy
 Radiotherapy to lymph nodes
 Surgical resection of lymph nodes
 DVT
 Thrombophlebitis

Indications for surgery

 Marked disability or deformity from limb swelling


 Lymphoedema caused by proximal lymphatic obstruction with patent distal lymphatics
suitable for a lymphatic drainage procedure
 Lymphocutaneous fistulae and megalymphatics

Procedures
Homans operation Reduction procedure with preservation of overlying skin (which must be in
good condition). Skin flaps are raised and the underlying tissue excised.
Limb circumference typically reduced by a third.

Charles operation All skin and subcutaneous tissue around the calf are excised down to the
deep fascia. Split skin grafts are placed over the site. May be performed if
overlying skin is not in good condition. Larger reduction in size than with
Homans procedure.

Lymphovenous Identifiable lymphatics are anastomosed to sub dermal venules. Usually


anastamosis indicated in 2% of patients with proximal lymphatic obstruction and normal
distal lymphatics.

Next question
A 21 year old post man notices leg pain after 5 minutes walking during his round. It improves 3
minutes after stopping. Clinically he is noted to have reduced hair of the lower limbs and his calf
muscles appear atrophied. There is a weak popliteal pulse, but it is still present when the knee is
fully extended. What is the most likely diagnosis?

Occlusive arterial disease caused by atherosclerosis

Popliteal fossa entrapment

Cerebral vascular accident

Diabetes mellitus

Adductor canal compression syndrome

Adductor canal compression syndrome most commonly presents in young males and is an important
differential diagnosis in men presenting with symptoms of acute limb ischaemia on exertion. It is
caused by compression of the femoral artery by the musculotendinous band from adductor magnus
muscle.
The treatment consists of the division of the abnormal band and restoration of the arterial circulation.
Popliteal fossa entrapment is the main differential diagnosis, however the pulse disappears when the
knee is fully extended.
Please rate this question:

Discuss and give feedback


Next question

Adductor canal

 Also called Hunter's or subsartorial canal


 Immediately distal to the apex of the femoral triangle, lying in the middle third of the thigh.
Canal terminates at the adductor hiatus.

Borders Contents

Laterally Vastus medialis muscle Saphenous nerve


Borders Contents

Posteriorly Adductor longus, adductor magnus Superficial femoral artery

Roof Sartorius Superficial femoral vein

In the image below the sartorius muscle is removed to expose the canal contents

Image sourced from Wikipedia

Next question
Theme: Peripheral arterial disease

A. Femoro-above knee popliteal bypass graft using PTFE


B. Femoro-above knee popliteal bypass graft using long saphenous vein
C. Femoro-distal bypass graft using PTFE
D. Femoro-distal bypass graft using PTFE with Miller Cuff
E. Femoro-distal bypass graft using long saphenous vein
F. Above knee amputation
G. Below knee amputation
H. Angioplasty

Please select the most appropriate management for the vascular scenario given. Each option may
be used once, more than once or not at all.

51. A 63 year old man presents with a non healing ulcer on his foot. His ABPI measurements
are 0.35. A duplex scan shows a 4 cm 90% stenotic lesion of the superficial femoral artery
with no distal disease.

You answered Femoro-above knee popliteal bypass graft using PTFE

The correct answer is Angioplasty

Short occlusions are generally reasonable candidates for primary attempts at angioplasty.

52. A 63 year old man who previously smoked 20 cigarettes a day and has newly diagnosed
type II diabetes. He presents with rest pain. A diagnostic angiogram demonstrates
occlusion of the distal superficial femoral artery continuing below the knee. He has
reasonable posterior tibial artery below this level which branches to give good runoff to
the foot. He has varicose veins.

You answered Femoro-above knee popliteal bypass graft using PTFE

The correct answer is Femoro-distal bypass graft using PTFE with Miller Cuff

This man needs a bypass operation. Using PTFE alone will not give a good result as sub
intimal hyperplasia will give poor outcome early. Using a vein cuff (Miller cuff) at the end
of a PTFE graft will improve the situation.

53. A wheelchair bound 78 year old women with ischaemic heart disease secondary to long
smoking history and longstanding type II diabetes presents with rest pain and a non
healing ulcer on the dorsum of her foot. Angiogram shows reasonable superficial femoral
artery and iliacs. At the level of the popliteal there is an occlusion. Below this there is a
short area of patent posterior tibial and this reconstitutes lower down the leg to flow to the
foot.
You answered Femoro-above knee popliteal bypass graft using PTFE

The correct answer is Above knee amputation

A femoro-distal bypass graft would carry a high risk of failure and risk of peri-operative
myocardial infarct. This lady would be well suited to primary amputation.

Please rate this question:

Discuss and give feedback


Next question

Peripheral vascular disease

Indications for surgery to revascularise the lower limb

 Intermittent claudication
 Critical ischaemia
 Ulceration
 Gangrene

Intermittent claudication that is not disabling may provide a relative indication, whilst the other
complaints are often absolute indications depending upon the frailty of the patient.

Assessment

 Clinical examination
 Ankle brachial pressure index measurement
 Duplex arterial ultrasound
 Angiography (standard, CT or MRI): usually performed only if intervention being considered.

Angioplasty
In order for angioplasty to be undertaken successfully the artery has to be accessible. The lesion
relatively short and reasonable distal vessel runoff. Longer lesions may be amenable to sub-intimal
angioplasty.

Surgery
Surgery will be undertaken where attempts at angioplasty have either failed or are unsuitable.
Bypass essentially involves bypassing the affected arterial segment by utilising a graft to run from
above the disease to below the disease. As with angioplasty good runoff improves the outcome.

Some key concepts with bypass surgery


Superficial femoral artery occlusion to the above knee popliteal
In the ideal scenario, vein (either in situ or reversed LSV) would the used as a conduit. However,
prosthetic material has reasonable 5 year patency rates and some would advocate using this in
preference to vein so that vein can be used for other procedures in the future. In general terms either
technique is usually associated with an excellent outcome (if run off satisfactory).

Procedure

 Artery dissected out, IV heparin 3,000 units given and then the vessels are cross clamped
 Longitudinal arteriotomy
 Graft cut to size and tunneled to arteriotomy sites
 Anastomosis to femoral artery usually with 5/0 'double ended' Prolene suture
 Distal anastomosis usually using 6/0 'double ended' Prolene

Distal disease

 Femoro-distal bypass surgery takes longer to perform, is more technically challenging and
has higher failure rates.
 In elderly diabetic patients with poor runoff a primary amputation may well be a safer and
more effective option. There is no point in embarking on this type of surgery in patients who
are wheelchair bound.
 In femorodistal bypasses vein gives superior outcomes to PTFE.

Rules

 Vein mapping 1st to see whether there is suitable vein (the preferred conduit). Sub intimal
hyperplasia occurs early when PTFE is used for the distal anastomosis and will lead to early
graft occlusion and failure.
 Essential operative procedure as for above knee fem-pop.
 If there is insufficient vein for the entire conduit then vein can be attached to the end of the
PTFE graft and then used for the distal anastomosis. This type of 'vein boot' is technically
referred to as a Miller Cuff and is associated with better patency rates than PTFE alone.
 Remember the more distal the arterial anastomosis the lower the success rate.

References
Peach G et al. Diagnosis and management of peripheral arterial disease. BMJ 2012; 345: 36-41.
Next question
Which of the following is not a feature of a Charcot foot?

Bounding foot pulses in the early phases

Often occurs in the complete absence of trauma

Erythema of the foot in the early phase

Autonomic neuropathy

Peripheral neuropathy

Do not confuse the early phase of Charcot foot with cellulitis

Trauma (even if only minor) is a prerequisite. Patients cannot usually recall the traumatic event. The
associated neuropathy means that patients continue to walk on the affected foot with subsequent
deformity developing over time.
Please rate this question:

Discuss and give feedback


Next question

Peripheral vascular disease

Indications for surgery to revascularise the lower limb

 Intermittent claudication
 Critical ischaemia
 Ulceration
 Gangrene

Intermittent claudication that is not disabling may provide a relative indication, whilst the other
complaints are often absolute indications depending upon the frailty of the patient.

Assessment

 Clinical examination
 Ankle brachial pressure index measurement
 Duplex arterial ultrasound
 Angiography (standard, CT or MRI): usually performed only if intervention being considered.

Angioplasty
In order for angioplasty to be undertaken successfully the artery has to be accessible. The lesion
relatively short and reasonable distal vessel runoff. Longer lesions may be amenable to sub-intimal
angioplasty.

Surgery
Surgery will be undertaken where attempts at angioplasty have either failed or are unsuitable.
Bypass essentially involves bypassing the affected arterial segment by utilising a graft to run from
above the disease to below the disease. As with angioplasty good runoff improves the outcome.

Some key concepts with bypass surgery

Superficial femoral artery occlusion to the above knee popliteal


In the ideal scenario, vein (either in situ or reversed LSV) would the used as a conduit. However,
prosthetic material has reasonable 5 year patency rates and some would advocate using this in
preference to vein so that vein can be used for other procedures in the future. In general terms either
technique is usually associated with an excellent outcome (if run off satisfactory).

Procedure

 Artery dissected out, IV heparin 3,000 units given and then the vessels are cross clamped
 Longitudinal arteriotomy
 Graft cut to size and tunneled to arteriotomy sites
 Anastomosis to femoral artery usually with 5/0 'double ended' Prolene suture
 Distal anastomosis usually using 6/0 'double ended' Prolene

Distal disease

 Femoro-distal bypass surgery takes longer to perform, is more technically challenging and
has higher failure rates.
 In elderly diabetic patients with poor runoff a primary amputation may well be a safer and
more effective option. There is no point in embarking on this type of surgery in patients who
are wheelchair bound.
 In femorodistal bypasses vein gives superior outcomes to PTFE.

Rules

 Vein mapping 1st to see whether there is suitable vein (the preferred conduit). Sub intimal
hyperplasia occurs early when PTFE is used for the distal anastomosis and will lead to early
graft occlusion and failure.
 Essential operative procedure as for above knee fem-pop.
 If there is insufficient vein for the entire conduit then vein can be attached to the end of the
PTFE graft and then used for the distal anastomosis. This type of 'vein boot' is technically
referred to as a Miller Cuff and is associated with better patency rates than PTFE alone.
 Remember the more distal the arterial anastomosis the lower the success rate.

References
Peach G et al. Diagnosis and management of peripheral arterial disease. BMJ 2012; 345: 36-41.
Next question
A 34 year old man presents with varicose veins and it is suspected that these are part of the Klippel-
Trenaunay syndrome. Which of the following is not a characteristic of this condition?

Presence of varicose veins

Gigantism of a limb

Long saphenous vein involvement

Port wine stains with clear borders

Arteriovenous fistulae

The Klippel-Trenaunay vein is a large, lateral, superficial vein sometimes seen at birth. This vein
begins in the foot or the lower leg and travels proximally until it enters the thigh or the gluteal area.
Otherwise, varicosities may not be clinically evident until the child begins to ambulate.
Varicosities may be extensive, though they often spare the saphenous distribution. They are seen
below the knee, laterally above the knee, and occasionally in the pelvic region. Varicosities may
affect the superficial, deep, and perforating venous systems.
Surgical exploration has demonstrated atresia and agenesis of deep veins, compression due to
fibrous bands, aberrant arteries, abnormal muscles, or venous sheaths.
Rarely, varicosities have been found in the bladder, the colon, and the pulmonary vessels
Please rate this question:

Discuss and give feedback


Next question

Klippel-Trenaunay syndrome

Klippel-Trenaunay-Weber syndrome generally affects a single extremity, although cases of multiple


affected limbs have been reported. The leg is the most common site followed by the arms, the trunk,
and rarely the head and the neck

Signs and symptoms


The birth defect is diagnosed by the presence of a combination of these symptoms:

 One or more distinctive port-wine stains with sharp borders


 Varicose veins
 Hypertrophy of bony and soft tissues, that may lead to local gigantism or shrinking.
 An improperly developed lymphatic system
In some cases, port-wine stains (capillary port wine type) may be absent. Such cases are very rare
and may be classified as "atypical Klippel-Trenaunay syndrome".

KTS can either affect blood vessels, lymph vessels, or both. The condition most commonly presents
with a mixture of the two. Those with venous involvement experience increased pain and
complications.
Next question
Theme: Diabetic foot sepsis

A. IV broad spectrum antibiotics


B. Incision and drainage of pus
C. Ray amputation
D. Below knee amputation
E. Above knee amputation
F. Vacuum Assisted Closure device (VAC)
G. Discharge home
H. Application of 4 layer bandages

Please select the most appropriate management for the scenario given. Each option may be used
once, more than once or not at all.

56. A 68 year old man with type II diabetes has a non healing ulcer following a ray amputation
2 weeks ago. An x-ray shows no osteomyelitis and the ABPI is >1.

You answered IV broad spectrum antibiotics

The correct answer is Vacuum Assisted Closure device (VAC)

A VAC dressing may avoid the need for further surgery.

57. A 48 year old woman is admitted with sepsis secondary to an infected diabetic foot ulcer.
She has a necrotic and infected forefoot with necrosis of the heel. There is a boggy
indurated swelling anterior to the ankle joint. Pulses are normal.

You answered IV broad spectrum antibiotics

The correct answer is Below knee amputation

A below knee amputation is the best option here. The foot is non salvageable. However,
she may ambulate with a prosthesis.

58. An 84 year old lady is admitted with an infected diabetic foot. An x-ray shows
osteomyelitis of her calcaneum. She has a fixed flexion deformity of her knee, but normal
pulses.

You answered IV broad spectrum antibiotics

The correct answer is Above knee amputation

This patient will not be able to walk with a below knee amputation, therefore an above
knee amputation would be preferable, as it guarantees better healing the short term.
Consider above knee amputation in patients with fixed flexion deformity.

Please rate this question:

Discuss and give feedback


Next question

Amputations

Amputations are indicated when the affected limb is one of the following:

 Dead non viable


 Deadly where it is posing a major threat to life
 Dead useless where it is viable but a prosthesis would be preferable

Orthopaedic surgery

 Amputation is often undertaken as an option of last resort e.g. Limb salvage has failed and
the limb is so non functional that mobility needs would be best met with prosthesis.
 Chronic fracture non union or significant limb shortening following trauma would fit into this
category. Occasionally following major trauma a primary amputation is preferable. This
would be the case in an open fracture with major distal neurovascular compromise and other
more life threatening injuries are present.

Vascular surgery

 The first two categories are the most prevalent.


 Diabetic foot sepsis is often a major cause of sepsis which can spread rapidly in the
presence of established peripheral vascular disease.
 As a general rule the main issue in vascular surgery is to optimise vascular inflow prior to
surgery. The more distal the planned amputation is to be, the more important this rule
becomes.
 In other situations there has been something such as an embolic event that has not been
revascularised in time. In this case the limb shows fixed mottling and an amputation will be
needed.

Types of amputations
As the vast majority of commonly performed amputations affect the lower limbs these will be covered
here.
The main categories of amputations are:

 Pelvic disarticulation (hindquarter)


 Above knee amputation
 Gritti Stokes (through knee amputation)
 Below knee amputation (using either Skew or Burgess flaps)
 Syme's amputation (through ankle)
 Amputations of mid foot and digits

Choosing a level of amputation depends on:

 The disease process being treated


 Desired functional outcome
 Co-morbidities of the patient

Above knee amputations

 Quick to perform
 Heal reliably
 Patients regain their general health quickly
 For this benefit, a functional price has to be paid and many patients over the age of 70 will
never walk on an above knee prosthesis.
 Above knee amputations use equal anterior-posterior flaps

Below knee amputations

 Technically more challenging to perform


 Heal less reliably than their above knee counterparts.
 However, many more patients are able to walk using a below knee prosthesis.
 In below knee amputations the two main flaps are Skew flaps or the Burgess long posterior
flap. Skew flaps result in a less bulky limb that is easier to attach a prosthesis to.

It is worth remembering that whilst it may be technically feasible to offer a below knee amputation
there may be circumstances where an above knee option is preferable. For example, in fixed flexion
deformities of the lower limb, little functional benefit would be gained from below knee amputation
surgery.
Next question
Which of the following statements related to coronary artery bypass surgery is true?

Late graft stenosis is mainly associated with saphenous vein grafts

Is indicated if there is stenosis > 70% of the right coronary artery

The left atrium is cannulated during the procedure

The CHADS score is used to assess peri operative risk

Cardioplegia is always undertaken at a 37 degrees

Indications are:

1. Left main stem stenosis or equivalent (proximal LAD and proximal circumflex)
2. Triple vessel disease
3. Diffuse disease unsuitable for PCI
The right atrium is cannulated. The CHADS score assesses whether a patient should be
warfarinised if they have atrial fibrillation. Cardioplegia can be undertaken at cold or warm
temperatures.
Please rate this question:

Discuss and give feedback


Next question

Cardiopulmonary bypass

Indications for surgery

 Left main stem stenosis or equivalent (proximal LAD and proximal circumflex)
 Triple vessel disease
 Diffuse disease unsuitable for PCI

The guidelines state that CABG is the preferred treatment in high-risk patients with severe
ventricular dysfunction or diabetes mellitus.

Technique
General anaesthesia
Central and arterial lines
Midline sternotomy or left sub mammary incision
Aortic root and pericardium dissected
Heart inspected

Bypass grafting may be performed using a cardiopulmonary bypass circuit with cardiac arrest or
using a number of novel 'off pump' techniques.

Procedure cardiopulmonary bypass

 Aortic root cannulated


 Right atrial cannula
 Circuit primed and patient fully heparinised (30,000 Units unfractionated heparin) as the
circuit is highly thrombogenic
 Flow established through circuit
 Aortic cross clamp applied
 Cardioplegia solution instilled into the aortic root below cross clamp
 Heart now asystolic and ready for surgery

Off pump techniques are evolving on a constant basis and details are beyond the scope of the
MRCS.

Conduits for bypass


> Internal mammary artery is best. Use of both is associated with increased risk of sternal wound
dehiscence. However, many surgeons will use both especially for redo surgery.
> Radial artery harvested from forearm. Ensure ulnar collateral working first!
> Reversed long saphenous vein grafts
Typically anastamosed using 7/0-8/0 prolene sutures (distally) and 6/0 prolene for top end.

Once flow established


Anticoagulation reversed using protamine
Patient is taken off bypass
Inotropes given if needed
Sternum closed using sternal closure device or stainless steel wire

Complications

 Post perfusion syndrome: transient cognitive impairment


 Non union of the sternum; due to loss of the internal thoracic artery
 Myocardial infarction
 Late graft stenosis
 Acute renal failure
 Stroke
 Gastrointestinal

Perioperative risk is quantified using the Parsonnet and Euroscores and unit outcomes are audited
using this data.

Reference
Eagle KA, Guyton RA, Davidoff R, et al: ACC/AHA 2004 guideline update for coronary artery bypass
graft surgery: A report of the American College of Cardiology/American Heart Association Task
Force on Practice Guidelines (Committee to Revise the 1999 Guidelines for Coronary Artery Bypass
Graft Surgery). Circulation 2004; 110
Next question
A 73 year old man develops sudden onset abdominal pain and collapses. On examination he has a
tender pulsatile mass in his upper abdomen. He has a blood pressure of 90/60mmHg and pulse rate
of 105 beats per minute. Which of the following intravenous fluid regimens is most appropriate,
whilst waiting for operative repair?

1 Litre of pentastarch over 15 minutes

1 litre of Hartmans solution over 4 hours

1 litre of gelofusin over 30 minutes

1 litre of Hartmans solution over 30 minutes

1 litre of blood over 15 minutes

This man will have a contained haematoma and is awaiting surgery. Rapid, high volume infusions
may cause this to dislodge with disastrous consequences
Please rate this question:

Discuss and give feedback


Next question

Abdominal aorta aneurysm

 Abdominal aortic aneurysms are a common problem in vascular surgery.


 They may occur as either true or false aneurysm. With the former all 3 layers of the arterial
wall are involved, in the latter only a single layer of fibrous tissue forms the aneurysm wall.
 True abdominal aortic aneurysms have an approximate incidence of 0.06 per 1000 people.
They are commonest in elderly men and for this reason the UK is now introducing the
aneurysm screening program with the aim of performing an abdominal aortic ultrasound
measurement in all men aged 65 years.

Causes

 Several different groups of patients suffer from aneurysmal disease.


 The commonest group is those who suffer from standard arterial disease, i.e. Those who
arehypertensive, have diabetes and have been or are smokers.
 Other patients such as those suffering from connective tissue diseases such as Marfan's
may also develop aneurysms. In patients with abdominal aortic aneurysms the extracellular
matrix becomes disrupted with a change in the balance of collagen and elastic fibres.
Management

 Most abdominal aortic aneurysms are an incidental finding.


 Symptoms most often relate to rupture or impending rupture.
 20% rupture anteriorly into the peritoneal cavity. Very poor prognosis.
 80% rupture posteriorly into the retroperitoneal space
 The risk of rupture is related to aneurysm size, only 2% of aneurysms measuring less than
4cm in diameter will rupture over a 5 year period. This contrasts with 75% of aneurysms
measuring over 7cm in diameter.
 This is well explained by Laplaces' law which relates size to transmural pressure.
 For this reason most vascular surgeons will subject patients with an aneurysm size of 5cm or
greater to CT scanning of the chest, abdomen and pelvis with the aim of delineating anatomy
and planning treatment. Depending upon co-morbidities, surgery is generally offered once
the aneurysm is between 5.5cm and 6cm.

A CT reconstruction showing an infrarenal abdominal aortic aneurysm. The walls of the sac are
calcified which may facilitate identification on plain x-rays
Image sourced from Wikipedia

Indications for surgery

 Symptomatic aneurysms (80% annual mortality if untreated)


 Increasing size above 5.5cm if asymptomatic
 Rupture (100% mortality without surgery)
Surgical procedures
Abdominal aortic aneurysm repair

Procedure:

GA
Invasive monitoring (A-line, CVP, catheter)
Incision: Midline or transverse
Bowel and distal duodenum mobilised to access aorta.
Aneurysm neck and base dissected out and prepared for cross clamp
Systemic heparinisation
Cross clamp (proximal first)
Longitudinal aortotomy
Atherectomy
Deal with back bleeding from lumbar vessels and inferior mesenteric artery
Insert graft either tube or bifurcated depending upon anatomy
Suture using Prolene (3/0 for proximal , distal anastomosis suture varies according to site)
Clamps off: End tidal CO2 will rise owing to effects of reperfusion, at this point major risk of
myocardial events.
Haemostasis
Closure of aneurysm sac to minimise risk of aorto-enteric fistula
Closure: Loop 1 PDS or Prolene to abdominal wall
Skin- surgeons preference

Post operatively:

ITU (Almost all)


Greatest risk of complications following emergency repair
Complications: Embolic- gut and foot infarcts
Cardiac - owing to premorbid states, re-perfusion injury and effects of cross clamp
Wound problems
Later risks related to graft- infection and aorto-enteric fistula

Special groups

Supra renal AAA


These patients will require a supra renal clamp and this carries a far higher risk of complications and
risk of renal failure.

Ruptured AAA
Pre-operatively the management depends upon haemodynamic instability. In patients with
symptoms of rupture (typical pain, haemodynamic compromise and risk factors) then ideally prompt
laparotomy. In those with vague symptoms and haemodynamic stability the ideal test is CT scan to
determine whether rupture has occurred or not. Most common rupture site is retroperitoneal 80%.
These patients will tend to develop retroperitoneal haematoma. This can be disrupted if Bp is
allowed to rise too high so aim for Bp 100mmHg.
Operative details are similar to elective repair although surgery should be swift, blind rushing often
makes the situation worse. Plunging vascular clamps blindly into a pool of blood at the aneurysm
neck carries the risk of injury the vena cava that these patients do not withstand. Occasionally a
supracoeliac clamp is needed to effect temporary control, although leaving this applied for more than
20 minutes tends to carry a dismal outcome.
EVAR
Increasingly patients are now being offered endovascular aortic aneurysm repair. This is undertaken
by surgeons and radiologists working jointly. The morphology of the aneurysm is important and not
all are suitable. Here is a typical list of those features favoring a suitable aneurysm:

 Long neck
 Straight iliac vessels
 Healthy groin vessels

Clearly few AAA patients possess the above and compromise has to be made. The use of
fenestrated grafts can allow supra renal AAA to be treated.

Procedure:

GA
Radiology or theatre
Bilateral groin incisions
Common femoral artery dissected out
Heparinisation
Arteriotomy and insertion of guide wire
Dilation of arteriotomy
Insertion of EVAR Device
Once in satisfactory position it is released
Arteriotomy closed once check angiogram shows good position and no endoleak

Complications:
Endoleaks depending upon site are either Type I or 2. These may necessitate re-intervention and all
EVAR patients require follow up . Details are not needed for MRCS.

References
A reasonable review is provided by:
Sakalihasan N, Limet R, Defawe O. Abdominal aortic aneurysm. Lancet 2005 (365):1577- 1589
Next question
Theme: Mesenteric vascular disease

A. Abdominal ultrasound

B. Abdominal CT with venous phase contrast

C. CT angiogram

D. Magnetic resonance angiogram

E. Duplex ultrasound

F. Abdominal x-ray

G. Mesenteric angiography via brachial artery

Please select the most appropriate investigation for the scenario given. Each option may be used
once, more than once or not at all.

61. A thin 72 year old lady has a 3 week history of postprandial abdominal pain that is centrally
located. She has episodic diarrhoea and occasionally has passed blood PR. She has a history of
ischaemic heart disease and marked renal impairment from ACE inhibitor usage.

You answered Abdominal ultrasound

The correct answer is Duplex ultrasound

She is likely to have mesenteric vascular disease. Proximal SMA disease would be the most serious
variant. Ideally a CT angiogram would be the best test but with her impaired renal function and
low BMI, make a duplex of the SMA is a reasonable first line investigation. Gut peristalsis may
impair acquisition of magnetic resonance images.

62. A 78 year old man develops sudden onset abdominal pain and almost immediately afterwards
passes a large amount of diarrhoea.

You answered Abdominal ultrasound

The correct answer is CT angiogram


Sudden onset of abdominal pain followed by forceful evacuation are the classical presenting
features of acute mesenteric infarction. This is best investigated by CT angiography, which has a
sensitivity of 95% for the diagnosis.

63. A 28 year old female has suffered from abdominal pain for the past 2 weeks since she was started
on the contraceptive pill. The pain has increased significantly over the post 10 hours and has been
associated with vomiting.

You answered Abdominal ultrasound

The correct answer is Abdominal CT with venous phase contrast

Mesenteric venous thrombosis is the likely underlying cause and an angiogram is the sensible step
as it will also facilitate the identification of areas of infarcted bowel , similar to that which may
occur in the leg when massive DVT is present.

Please rate this question:

Discuss and give feedback

Next question

Vascular investigations

Venous disease

Venous Doppler
The simplest investigation for assessment of venous junctional incompetence is a Doppler
assessment. This involves the patient standing and manual compression of the limb distal to the
junction of interest. Flow should normally occur in one direction only. Where junctional incompetence
is present reverse flow will occur and is relatively easy to identify.

Venograms and duplex scans


Structural venous information is historically obtained using a venogram. This is an invasive test and
rarely required in modern clinical practice. The most helpful test is a venous duplex scan which will
provide information relating to flow and vessel characteristics. Duplex is also useful in providing vein
maps for bypass surgery.
Arterial disease

Ankle-brachial pressure
The ankle brachial pressure index measurement is an important investigation as it will allow
classification of the severity of the flow compromise present. False readings may occur in those with
calcified vessels such as diabetics and results in such settings should be interpreted with caution.
When auscultating the vessel note should be made of the character of the signal. Monophasic
signals are associated with a proximal stenosis and reduction in flow. Triphasic signals provide
reassurance of a healthy vessel.

Arterial Duplex
As with the vein the duplex scan can provide a substantial amount of information about arterial
patency and flow patterns. In skilled hands they can provide insight as to the state of proximal
vessels that are anatomically inaccessible to duplex (e.g. Iliacs). Through assessment of distal flow
patterns. It is an operator dependent test.

Conventional angiogram
Vessel puncture and catheter angiography is the gold standard method of assessing arteries. High
quality information can usually be obtained. Limitations of the technique include the risk of contrast
toxicity and risks of vessel damage. Severely calcified vessels may be difficult to puncture and in this
situation a remote access site (e.g. brachial) may be used. This technique is particularly useful in
providing a distal arterial roadmap prior to femoro-distal bypass.

CT angiography
These tests provide a considerable amount of structural and flow information. They require contrast
and thus carry the risks associated with this. They are particularly useful in the setting of GI bleeding
as they are rapidly available and can be performed by a non vascular radiologist. However, they lack
the facility for endovascular intervention. In general they do not provide high enough resolution for
distal arterial surgery.

Magnetic resonance angiography


This has the advantage of being non-invasive and not using nephrotoxic contrast. Movement artifact
remains a problem in some sites and distal arterial resolution is imperfect.

Next question
A 52 year old male attends the stroke unit with dizziness and vertigo while playing tennis. He is
known to have hypertension and a previous myocardial infarct. He now complains of right arm pain.
What is the most likely diagnosis?

Posterior circulation infarct

Vertebrobasilar aneurysm

Dissection of thoracic aorta

Subclavian steal syndrome

Left middle cerebral artery infarct

Subclavian steal syndrome characteristically presents with posterior circulation symptoms, such as
dizziness and vertigo, during exertion of an arm. There is subclavian artery steno-occlusive disease
proximal to the origin of the vertebral artery and is associated with flow reversal in the vertebral
artery. Management involves percutaneous transluminal angioplasty or a stent.
Please rate this question:

Discuss and give feedback


Next question

Subclavian artery

Path

 The left subclavian comes directly off the arch of aorta


 The right subclavian arises from the brachiocephalic artery (trunk) when it bifurcates into the
subclavian and the right common carotid artery.
 From its origin, the subclavian artery travels laterally, passing between anterior and middle
scalene muscles, deep to scalenus anterior and anterior to scalenus medius. As the
subclavian artery crosses the lateral border of the first rib, it becomes the axillary artery. At
this point it is superficial and within the subclavian triangle.
Image sourced from Wikipedia

Branches

 Vertebral artery
 Internal thoracic artery
 Thyrocervical trunk
 Costocervical trunk
 Dorsal scapular artery

Next question
A 74 year old lady has a long standing venous leg ulcer overlying her medial malleolus. Which of the
following statements relating to the management of this condition is false?

Pentoxifylline may speed ulcer healing

Treatment with daily low dose flucloxacillin may speed ulcer healing

Multilayer bandages may provide compression equivalent to 40mmHg

Large ulcers may be considered for skin grafting

They should not be treated with compression stockings if the ankle / brachial pressure
index is 0.6

Routine use of antibiotics is not advised as this may predispose to resistant organisms.
Pentoxifylline was subjected to a Cochrane review in 2007 and shown to improve healing rates.
Please rate this question:

Discuss and give feedback


Next question

Lower leg ulcers

Venous leg ulcers

 Most due to venous hypertension, secondary to chronic venous insufficiency (other causes
include calf pump dysfunction or neuromuscular disorders)
 Ulcers form due to capillary fibrin cuff or leucocyte sequestration
 Features of venous insufficiency include oedema, brown pigmentation, lipodermatosclerosis,
eczema
 Location above the ankle, painless
 Deep venous insufficiency is related to previous DVT and superficial venous insufficiency is
associated with varicose veins
 Doppler ultrasound looks for presence of reflux and duplex ultrasound looks at the anatomy/
flow of the vein
 Management: 4 layer compression banding after exclusion of arterial disease or surgery
 If fail to heal after 12 weeks or >10cm 2 skin grafting may be needed

Marjolin's ulcer
Image sourced from Wikipedia

 Squamous cell carcinoma


 Occurring at sites of chronic inflammation e.g; burns, osteomyelitis after 10-20 years
 Mainly occur on the lower limb

Arterial ulcers

 Occur on the toes and heel


 Painful
 There may be areas of gangrene
 Cold with no palpable pulses
 Low ABPI measurements

Neuropathic ulcers

 Commonly over plantar surface of metatarsal head and plantar surface of hallux
 The plantar neuropathic ulcer is the condition that most commonly leads to amputation in
diabetic patients
 Due to pressure
 Management includes cushioned shoes to reduce callous formation

Pyoderma gangrenosum
Image sourced from Wikipedia

 Associated with inflammatory bowel disease/RA


 Can occur at stoma sites
 Erythematous nodules or pustules which ulcerate

Next question
Theme: Investigation of vascular disease

A. Angiography

B. Arterial duplex scan

C. Arch aortogram

D. CXR

E. CT scan

F. Venous duplex scan

Please select the most appropriate investigation for the scenario given. Each option may be used
once, more than once or not at all.

66. A 22 year old professional tennis player attends A&E with a swollen painful right arm. His fingers
are dusky.

You answered Angiography

The correct answer is Venous duplex scan

This patient has an axillary vein thrombosis. It classically presents with pain and swelling of an
effort induced limb. Duplex scan is needed to exclude a thombus.

67. A 65 year old man presents with expressive dysphasia and left sided weakness over 4 hours. His
symptoms have now completely resolved. A CT scan of his head is normal.

You answered Angiography

The correct answer is Arterial duplex scan

This patient has had a transient ischaemic attack. He urgently needs carotid duplex scanning to
assess if he needs a carotid endarterectomy.
68. A 65 year old man presents, for the first time, with pain at the back of his calves when he
mobilises 10 metres. He is known to have hypertension.

You answered Angiography

The correct answer is Arterial duplex scan

An arterial duplex should be performed first, before progression to an angiography.

Please rate this question:

Discuss and give feedback

Next question

Vascular investigations

Venous disease

Venous Doppler
The simplest investigation for assessment of venous junctional incompetence is a Doppler
assessment. This involves the patient standing and manual compression of the limb distal to the
junction of interest. Flow should normally occur in one direction only. Where junctional incompetence
is present reverse flow will occur and is relatively easy to identify.

Venograms and duplex scans


Structural venous information is historically obtained using a venogram. This is an invasive test and
rarely required in modern clinical practice. The most helpful test is a venous duplex scan which will
provide information relating to flow and vessel characteristics. Duplex is also useful in providing vein
maps for bypass surgery.

Arterial disease

Ankle-brachial pressure
The ankle brachial pressure index measurement is an important investigation as it will allow
classification of the severity of the flow compromise present. False readings may occur in those with
calcified vessels such as diabetics and results in such settings should be interpreted with caution.
When auscultating the vessel note should be made of the character of the signal. Monophasic
signals are associated with a proximal stenosis and reduction in flow. Triphasic signals provide
reassurance of a healthy vessel.

Arterial Duplex
As with the vein the duplex scan can provide a substantial amount of information about arterial
patency and flow patterns. In skilled hands they can provide insight as to the state of proximal
vessels that are anatomically inaccessible to duplex (e.g. Iliacs). Through assessment of distal flow
patterns. It is an operator dependent test.

Conventional angiogram
Vessel puncture and catheter angiography is the gold standard method of assessing arteries. High
quality information can usually be obtained. Limitations of the technique include the risk of contrast
toxicity and risks of vessel damage. Severely calcified vessels may be difficult to puncture and in this
situation a remote access site (e.g. brachial) may be used. This technique is particularly useful in
providing a distal arterial roadmap prior to femoro-distal bypass.

CT angiography
These tests provide a considerable amount of structural and flow information. They require contrast
and thus carry the risks associated with this. They are particularly useful in the setting of GI bleeding
as they are rapidly available and can be performed by a non vascular radiologist. However, they lack
the facility for endovascular intervention. In general they do not provide high enough resolution for
distal arterial surgery.

Magnetic resonance angiography


This has the advantage of being non-invasive and not using nephrotoxic contrast. Movement artifact
remains a problem in some sites and distal arterial resolution is imperfect.

Next question
Theme: Ankle Brachial pressure index measurements

A. >1
B. 0.6- 0.8
C. 1
D. 0.4-0.6
E. <0.4

For the scenarios described below, please select the most likely ankle brachial pressure index
measurement. Each option may be used once, more than once or not at all.

69. An 83 year old male with rest pain.

You answered >1

The correct answer is <0.4

Theme from September 2015 Exam


Rest pain is typically associated with low ABPI values.

70. A 45 year old man who develops calf pain after walking 600 yards. It resolves during
periods of rest.

You answered >1

The correct answer is 0.6- 0.8

Since this is a long claudication distance it may be that only a minor lesion is present.
Whilst resting ABPI may be normal, they are usually abnormal following exercise.

71. A 43 year old lady with long standing diabetes who complains of calf pain. It is worse at
night and during minor exercise.

>1

Diabetes may be complicated by vessel calcification and neuropathic pain. Therefore


individuals may present with pain which is atypical for claudation both in terms of its
tempo of onset and location.

Please rate this question:

Discuss and give feedback


Next question

Ankle-Brachial pressure index

 Measurement of ankle- brachial pressure index (ABPI) is a commonly performed vascular


investigation.
 Calculated by dividing lower limb pressure by the highest upper limb pressure.

Results of ABPI
1.2 or greater Usually due to vessel calcification

1.0- 1.2 Normal

Minor stenotic lesion


0.8-1.0
Initiate risk factor management

Moderate stenotic lesion


Consider duplex
0.50-0.8
Risk factor management
If mixed ulcers present then avoid tight compression bandages

Likely significant stenosis


0.5- 0.3 Duplex scanning to delineate lesions needed
Compression bandaging contra indicated

Indicative of critical ischaemia


Less than 0.3
Urgent detailed imaging required
Next question
Theme: Causes of ulceration

A. Marjolin's ulcer
B. Neuropathic ulcer
C. Arterial ulcer
D. Deep venous dysfunction
E. Superficial venous dysfunction
F. Rheumatoid arthritis
G. Pyoderma gangrenosum
H. Pressure ulcer

Please select the most likely cause of ulceration for the scenario given. Each option may be used
once, more than once or not at all.

72. A 62 year old diabetic man presents with long standing plantar ulcer he has clinical
evidence of a charcot foot.

You answered Marjolin's ulcer

The correct answer is Neuropathic ulcer

Plantar ulcers in association with peripheral neuropathy are often neuropathic. They
classically occur at pressure points.

73. A 66 year old female has long standing mixed arteriovenous ulcers of the lower leg. Over
the past 6 months one of the ulcers has become much worse and despite a number of
different topical therapies is increasing in size.

Marjolin's ulcer

Marjolin's ulcer is a squamous cell carcinoma occurring at sites of chronic inflammation or


previous injury.

74. A 28 year old man undergoes a ileocaecal resection and end ileostomy for Crohn's disease.
One year later he presents with a deep painful ulcer at his stoma site.

You answered Marjolin's ulcer

The correct answer is Pyoderma gangrenosum

Pyoderma gangrenosum is associated with inflammatory bowel disease (this patient had a
stoma for crohns!). It is commonly found on lower limbs and described as being painful,
the size of an insect bite and growing. It looks like a margarita pizza (with a red base and
yellow topping!) Treatment involves steroids.

Please rate this question:

Discuss and give feedback

Lower leg ulcers

Venous leg ulcers

 Most due to venous hypertension, secondary to chronic venous insufficiency (other causes
include calf pump dysfunction or neuromuscular disorders)
 Ulcers form due to capillary fibrin cuff or leucocyte sequestration
 Features of venous insufficiency include oedema, brown pigmentation, lipodermatosclerosis,
eczema
 Location above the ankle, painless
 Deep venous insufficiency is related to previous DVT and superficial venous insufficiency is
associated with varicose veins
 Doppler ultrasound looks for presence of reflux and duplex ultrasound looks at the anatomy/
flow of the vein
 Management: 4 layer compression banding after exclusion of arterial disease or surgery
 If fail to heal after 12 weeks or >10cm 2 skin grafting may be needed

Marjolin's ulcer

Image sourced from Wikipedia


 Squamous cell carcinoma
 Occurring at sites of chronic inflammation e.g; burns, osteomyelitis after 10-20 years
 Mainly occur on the lower limb

Arterial ulcers

 Occur on the toes and heel


 Painful
 There may be areas of gangrene
 Cold with no palpable pulses
 Low ABPI measurements

Neuropathic ulcers

 Commonly over plantar surface of metatarsal head and plantar surface of hallux
 The plantar neuropathic ulcer is the condition that most commonly leads to amputation in
diabetic patients
 Due to pressure
 Management includes cushioned shoes to reduce callous formation

Pyoderma gangrenosum

Image sourced from Wikipedia

 Associated with inflammatory bowel disease/RA


 Can occur at stoma sites
 Erythematous nodules or pustules which ulcerate
A 43 year old lady is diagnosed as having a malignant lesion in the inferior aspect of her left breast.
There is palpable axillary lymphadenopathy. What is the most appropriate course of action?

Mastectomy and axillary node clearance

Wide local excision and axillary node clearance

Wide local excision and sentinel lymph node biopsy

Image guided fine needle aspiration of the axillary nodes

CT scanning of the chest, abdomen and pelvis

Where axillary nodal involvement is suspected from the outset it is important to establish whether
this is the case prior to surgery. This is because, if axillary metastatic disease is present then the
correct management would be an axillary node clearance and this is irrespective of the surgical
plans for the breast primary. In the case of breast cancer, image guided FNAC is acceptable as it is
accurate and if carcinoma cells are identified at FNA then axillary node clearance can be performed.
If FNAC is negative then a sentinel node biopsy should accompany excision of the primary tumour.
Where the axilla is clinically clear on palpation and imaging then a sentinel lymph node biopsy
should accompany excision of the primary tumour.
Please rate this question:

Discuss and give feedback


Next question

Management of the axilla- breast cancer

Lymph node stage


Accurate staging of the axilla is an essential component of breast cancer management. Involvement
of the axillary nodes has an adverse effect on prognosis with 10 year survival reduced from 75% to
25%[1]. Involvement of level 3 nodes carries the worst prognosis[2]. Historically, management of the
axilla ranged from limited level 1 axillary node excision through to full level 3 axillary nodal
clearances. Attempts to mimimise the morbidity of axillary node clearance led to targeted operations
including axillary nodal sampling and sentinel lymph node biopsy. The focus on sentinel lymph node
biopsy has led to more detailed pathological analysis of excised lymph nodes (e.g. using
immunohistochemistry). This has led to increasing focus on the develop of axillary nodal
micrometastasis. The presence of micrometastasis and its impact on survival is debated. In some
studies it seems to confer an increased risk of locoregional recurrence [3] and a reduction of disease
free survival [4], whilst in others it shows no overall impact[5]. It is important to distinguish between
micrometastasis and isolated tumour cells, as the latter do not have an adverse impact on
prognosis[6]. The need for definitive treatment of the axilla in women with positive sentinel nodes
was addressed by the ASCOG Z0011trial. In this trial women were randomised to either undergo
axillary node clearance or observation, groups were adjusted for other prognostic factors and
treatments. The investigators found no survival benefit in routinely undertaking axillary node
clearance where axillary nodal disease was limited in its extent.
Regardless of the options in women with a low risk axilla, those individuals who have overt evidence
of axillary nodal involvement either through positive SLNB or preoperative USS and FNA, should still
receive axillary clearance as a standard of care.

References
1. Carter, C.L., C. Allen, and D.E. Henson, Relation of tumor size, lymph node status, and survival in
24,740 breast cancer cases. Cancer, 1989. 63(1): p. 181-7.
2. Clark, G.M. Integrating prognostic factors. Breast Cancer Res Treat, 1992. 22(3): p. 187-91.
3. Lupe, K., et al. Ten-year locoregional recurrence risks in women with nodal micrometastatic
breast cancer staged with axillary dissection. Int J Radiat Oncol Biol Phys, 2011. 81(5): 681-8.
4. Park, D. et al. The prognostic impact of occult nodal metastasis in early breast carcinoma. Breast
Cancer Res Treat, 2009. 118(1): 57-66.
5. Galimberti, V., et al. Positive axillary sentinel lymph node: is axillary dissection always necessary?
Breast, 2011. 20 Suppl 3: S96-8.
6. Ahmad, N. and J. Park, Defining Semantic Structure Features for Content-Based Visual Object
Class Recognition. Journal of Imaging Science and Technology, 2011. 55(2).
Next question
Theme: Breast conditions

A. Mondors disease
B. Duct ectasia
C. Periductal mastitis
D. Lactational breast abscess
E. Fibroadenoma
F. Breast cyst
G. Intraductal papilloma
H. Atypical ductal hyperplasia
I. Radial scar

Please select the most likely underlying diagnosis for the scenario given. Each option may be used
once, more than once or not at all.

2. A 20 year old lady presents with a mobile lump in the upper outer aspect of her right breast.
On examination she has a firm mobile mass in the upper outer quadrant of her right breast.

You answered Mondors disease

The correct answer is Fibroadenoma

Fibroadenomas account for 60% of discrete breast lesions in the 18-25 year age group. They
are mobile lesions. Core biopsy should be performed in lesions measuring more than 4cm in
diameter.

3. A 55 year old women presents with nipple discharge. On examination she has a slit like
retraction of the nipple in the centre of this area is a small amount of cheese like material.
No discrete mass lesion is palpable in the underlying breast.

You answered Mondors disease

The correct answer is Duct ectasia

Duct ectasia is a common alteration in the breast that occurs with aging. As the ducts
shorten and dilate a degree of symmetrical slit like retraction occurs. A small amount of
cheese like discharge may occur.

4. A 48 year old lady presents with discomfort in the right breast. On examination she has a
discrete soft fluctuant area in the upper outer quadrant of her right breast. A mammogram is
performed and a "halo sign" is seen by the radiologist.

You answered Mondors disease


The correct answer is Breast cyst

Lesions such as breast cysts compress the underlying fat and produce a radiolucent area
(halo sign). If symptomatic, these cysts should be aspirated.
Theme January 2014 exam

Please rate this question:

Discuss and give feedback


Next question

Abberations of normal development and involution- breast

Fibroadenoma
Under the age of 25 years the breast is usually classified as undergoing development. Lobular units
are being formed and a dense stroma is formed within the breast tissue. This may result in the
development of fibroadenomas.
As a group, fibroadenomas account for 13% of all palpable breast lesions. However, in women aged
18-25 they constitute up to 60% of all palpable breast lesions. The are classified as juvenile,
common and giant. The former occur in early adolescence and the latter are characterised by a size
greater than 4cm. In young females with small fibroadenomas (less than 3cm on imaging) a policy of
watchful waiting without biopsy may be adopted. A size of greater than 4cm attracts a
recommendation for core biopsy to exclude a phyllodes tumour. The natural history of
fibroadenomas is that 10% will increase in size, 30% regress and the remainder stay the same. This
does not apply during pregnancy and lactation when they may increase in size substantially and
subsequently sequester milk.
Some women may wish to have their fibroadenomas excised, they can usually be shelled out
through a circumareolar incision. Smaller lesions may be removed using a mammotome.

Breast cysts
Palpable cysts constitute 15% of all breast lumps. They occur most frequently in perimenopausal
females and are caused by distended and involuted lobules.
They may be readily apparent on clinical examination as soft, fluctuant swellings. It is important to
exclude the presence of an underlying mass lesion. On imaging they will usually show a "halo
appearance" on mammography. Ultrasound will confirm the fluid filled nature of the cyst.
Symptomatic cysts may be aspirated and following aspiration the breast re-examined to ensure that
the lump has gone.

Duct ectasia
As women progress through the menopause the breast ducts shorten and dilate. In some women
this may cause a cheese like nipple discharge and slit like retraction of the nipple. No specific
treatment is required.
Next question
A 43 year old lady has recently undergone a wide local excision and sentinel lymph node biopsy for
carcinoma of the breast. Of the factors listed below, which will provide the most important prognostic
information?

Mitotic number

Grade

Nodal status

Size

Oestrogen receptor status

Nodal status is the single most important prognostic factor in breast cancer.

Theme from April 2012 Exam


Theme from January 2013 Exam
Theme from April 2014 Exam
Nodal status is important because it serves as a marker of tumour metastatic potential. This
translates to survival advantages of up to 40% at five years. Both grade and size are of secondary
importance as they both less concerning in the absence of nodal involvement.

Please rate this question:

Discuss and give feedback

Next question

Breast cancer

 Commoner in the older age group


 Invasive ductal carcinomas are the most common type. Some may arise as a result
of ductal carcinoma in situ (DCIS). There are associated carcinomas of special type e.g.
Tubular that may carry better prognosis.
 The pathological assessment involves assessment of the tumour and lymph nodes, sentinel
lymph node biopsy is often used to minimise the morbidity of an axillary dissection.
 Treatment, typically this is either wide local excision or mastectomy. There are many sub
types of both of these that fall outside of the MRCS. Some key rules to bear in mind.
 Whatever operation is contemplated the final cosmetic outcome does have a bearing. A
woman with small breasts and a large tumour will tend to fare better with mastectomy, even
if clear pathological and clinical margins can be obtained. Conversely a women with larger
breasts may be able to undergo breast conserving surgery even with a relatively large
primary lesion (NB tumours >4cm used to attract recommendation for mastectomy). For
screen detected and impalpable tumour image guidance will be necessary.
 Reconstruction is always an option following any resectional procedure. However, its exact
type must be tailored to age and co-morbidities of the patient. The main operations in
common use include latissimus dorsi myocutaneous flap and sub pectoral implants. Women
wishing to avoid a prosthesis may be offered TRAM or DIEP flaps.

Surgical options
Mastectomy vs Wide local excision

Mastectomy Wide Local Excision

Multifocal tumour Solitary lesion

Central tumour Peripheral tumour

Large lesion in small breast Small lesion in large breast

DCIS >4cm DCIS <4cm

Patient Choice Patient choice

Central lesions may be managed using breast conserving surgery where an acceptable cosmetic
result may be obtained, this is rarely the case in small breasts

A compelling indication for mastectomy, a larger tumour that would be unsuitable for breast
conserving surgery
Image sourced from Wikipedia

Whatever surgical option is chosen the aim should be to have a local recurrence rate of 5% or less
at 5 years [1].

Nottingham Prognostic Index


The Nottingham Prognostic Index can be used to give an indication of survival. In this system the
tumour size is weighted less heavily than other major prognostic parameters.

Calculation of NPI
Tumour Size x 0.2 + Lymph node score(From table below)+Grade score(From table below).

Score Lymph nodes involved Grade

1 0 1

2 1-3 2

3 >3 3
Prognosis

Score Percentage 5 year survival

2.0 to 2.4 93%

2.5 to 3.4 85%

3.5 to 5.4 70%

>5.4 50%

This data was originally published in 1992. It should be emphasised that other factors such as
vascular invasion and receptor status also impact on survival and are not included in this data and
account for varying prognoses often cited in the literature.

References
Surgical guidelines for the management of breast cancer, Association of Breast Surgery at BASO
2009, Eur J Surg Oncol (2009), doi:10.1016/j.ejso.2009.01.008

Next question
Theme: Management of nipple discharge

A. Prescribe danazol
B. Microdochectomy
C. Total duct excision
D. Cytology of duct fluid
E. Core biopsy
F. Prescribe co-amoxiclav
G. Reassure and discharge
H. Mastectomy

What is the best management for each nipple discharge presentation? Each option may be used
once, more than once or not at all.

6. A 23 year old women with greenish nipple discharge on one occasion. Clinical examination
of the breast is normal. Ultrasound report is U1.

You answered Prescribe danazol

The correct answer is Reassure and discharge

Theme from April 2016 Exam


This is likely to be simple duct ectasia and U1 (normal USS) coupled with normal
examination would favor discharge from clinic. Mammography is generally unhelpful in
this age group

7. A 43 year old women has had recurrent episodes of periductal mastitis. She has received
multiple courses of antibiotics and is troubled by persisting green nipple discharge. Clinical
examination reveals green nipple discharge, but no discrete lump. Imaging with
mammography and ultrasound is reassuring (U2, M2)

You answered Prescribe danazol

The correct answer is Total duct excision

This woman has troublesome duct ectasia and total duct excision is warranted.

8. A 55 year old women complains of nipple discharge. This was blood stained on one
occasion. But not subsequently. Clinical examination shows clear fluid but no discrete
lump. Imaging with ultrasound and mammography is normal.

You answered Prescribe danazol


The correct answer is Microdochectomy

Although this is likely to be benign disease, her age coupled with an episode of blood
stained discharge would attract a recommendation for microdochectomy. She may have an
intraductal papilloma. But the concern would be DCIS.

Please rate this question:

Discuss and give feedback


Next question

Nipple discharge

Causes of nipple discharge


Physiological During breast feeding

Galactorrhoea Commonest cause may be response to emotional events, drugs such as


histamine receptor antagonists are also implicated

Hyperprolactinaemia  Commonest type of pituitary tumour


 Microadenomas <1cm in diameter
 Macroadenomas >1cm in diameter
 Pressure on optic chiasm may cause bitemporal hemianopia

Mammary duct  Dilatation breast ducts.


ectasia  Most common in menopausal women
 Discharge typically thick and green in colour
 Most common in smokers

Carcinoma  Often blood stained


 May be underlying mass or axillary lymphadenopathy

Intraductal papilloma  Commoner in younger patients


 May cause blood stained discharge
 There is usually no palpable lump

Assessment of patients
 Examine breast and determine whether there is mass lesion present
 All mass lesions should undergo Triple assessment.

Reporting of investigations
Where a mass lesion is suspected or investigations are requested these are prefixed using a system
that denotes the investigation type e.g. M for mammography, followed by a numerical code as
shown below:

1 No abnormality

2 Abnormality with benign features

3 Indeterminate probably benign

4 Indeterminate probably malignant

5 Malignant

Management of non malignant nipple discharge

 Exclude endocrine disease


 Nipple cytology unhelpful
 Smoking cessation advice for duct ectasia
 For duct ectasia with severe symptoms, total duct excision may be warranted.

Next question
Theme: Breast disease

A. Ductal carcinoma in situ


B. Lobular carcinoma in situ
C. Invasive ductal carcinoma
D. Invasive lobular carcinoma
E. Inflammatory carcinoma
F. Phyllodes tumour
G. Paget's disease of the nipple
H. Fibroadenoma
I. Mucinous breast carcinoma

From the list please select the most likely diagnosis for the scenario given. Each diagnosis may be
used once, more than once or not at all.

9. A 32 year old Indian lady presents with breast lump. She has a 4 month old child.
Clinically she has jaundice and there is erythema of the left breast.

You answered Ductal carcinoma in situ

The correct answer is Inflammatory carcinoma

Inflammatory breast cancers have an aggressive nature. Dissemination occurs early and is
more resistant to adjuvent treatments than other types of breast cancer. Often occurs in
pregnancy or lactation.

10. A 72 year old female presents with a painless breast lump. Clinically she has a 4cm
diameter irregular breast mass, with no other palpable masses.

You answered Ductal carcinoma in situ

The correct answer is Invasive ductal carcinoma

A post menopausal woman is more likely to have a ductal carcinoma and they tend to
occur at a single focus within the breast.

11. A 72 year old woman presents with 2 breast lumps. She has a history of breast cancer in
the opposite breast 5 years ago.

You answered Ductal carcinoma in situ

The correct answer is Invasive lobular carcinoma


This is likely to be an invasive lobular carcinoma, mainly due to the multifocal lesions and
the history of previous breast cancer in the opposite breast.

Please rate this question:

Discuss and give feedback


Next question

Breast cancer

 Commoner in the older age group


 Invasive ductal carcinomas are the most common type. Some may arise as a result
of ductal carcinoma in situ (DCIS). There are associated carcinomas of special type e.g.
Tubular that may carry better prognosis.
 The pathological assessment involves assessment of the tumour and lymph nodes, sentinel
lymph node biopsy is often used to minimise the morbidity of an axillary dissection.
 Treatment, typically this is either wide local excision or mastectomy. There are many sub
types of both of these that fall outside of the MRCS. Some key rules to bear in mind.
 Whatever operation is contemplated the final cosmetic outcome does have a bearing. A
woman with small breasts and a large tumour will tend to fare better with mastectomy, even
if clear pathological and clinical margins can be obtained. Conversely a women with larger
breasts may be able to undergo breast conserving surgery even with a relatively large
primary lesion (NB tumours >4cm used to attract recommendation for mastectomy). For
screen detected and impalpable tumour image guidance will be necessary.
 Reconstruction is always an option following any resectional procedure. However, its exact
type must be tailored to age and co-morbidities of the patient. The main operations in
common use include latissimus dorsi myocutaneous flap and sub pectoral implants. Women
wishing to avoid a prosthesis may be offered TRAM or DIEP flaps.

Surgical options
Mastectomy vs Wide local excision

Mastectomy Wide Local Excision

Multifocal tumour Solitary lesion

Central tumour Peripheral tumour

Large lesion in small breast Small lesion in large breast


Mastectomy Wide Local Excision

DCIS >4cm DCIS <4cm

Patient Choice Patient choice

Central lesions may be managed using breast conserving surgery where an acceptable cosmetic
result may be obtained, this is rarely the case in small breasts

A compelling indication for mastectomy, a larger tumour that would be unsuitable for breast
conserving surgery

Image sourced from Wikipedia

Whatever surgical option is chosen the aim should be to have a local recurrence rate of 5% or less
at 5 years [1].

Nottingham Prognostic Index


The Nottingham Prognostic Index can be used to give an indication of survival. In this system the
tumour size is weighted less heavily than other major prognostic parameters.

Calculation of NPI
Tumour Size x 0.2 + Lymph node score(From table below)+Grade score(From table below).
Score Lymph nodes involved Grade

1 0 1

2 1-3 2

3 >3 3

Prognosis

Score Percentage 5 year survival

2.0 to 2.4 93%

2.5 to 3.4 85%

3.5 to 5.4 70%

>5.4 50%

This data was originally published in 1992. It should be emphasised that other factors such as
vascular invasion and receptor status also impact on survival and are not included in this data and
account for varying prognoses often cited in the literature.

References
Surgical guidelines for the management of breast cancer, Association of Breast Surgery at BASO
2009, Eur J Surg Oncol (2009), doi:10.1016/j.ejso.2009.01.008
Next question
A 45 year old man is referred to the breast clinic with gynaecomastia. He takes the drugs listed
below. Which is least likely to be the cause of his symptoms?

Spironolactone

Carbimazole

Chlorpromazine

Cimetidine

Methyldopa

Mnemonic for drugs causing gynaecomastia: DISCO

D igitalis
I soniazid
S pironolactone
C imetidine
O estrogen

Mnemonic for causes of gynaecomastia: METOCLOPRAMIDE

M etoclopramide
E ctopic oestrogen
T rauma skull/tumour breast, testes
O rchitis
C imetidine, Cushings
L iver cirrhosis
O besity
P araplegia
RA
A cromegaly
M ethyldopa
I soniazid
D igoxin
E thionamide

Carbimazole is not associated with gynaecomastia.


Please rate this question:

Discuss and give feedback


Next question
Gynaecomastia

Gynaecomastia describes an abnormal amount of breast tissue in males and is usually caused by
an increased oestrogen:androgen ratio. It is important to differentiate the causes of galactorrhoea
(due to the actions of prolactin on breast tissue) from those of gynaecomastia

Causes of gynaecomastia

 physiological: normal in puberty


 syndromes with androgen deficiency: Kallman's, Klinefelter's
 testicular failure: e.g. Mumps
 liver disease
 testicular cancer e.g. Seminoma secreting HCG
 ectopic tumour secretion
 hyperthyroidism
 haemodialysis
 drugs: see below

Drug causes of gynaecomastia

 spironolactone (most common drug cause)


 cimetidine
 digoxin
 cannabis
 finasteride
 oestrogens, anabolic steroids

Very rare drug causes of gynaecomastia

 tricyclics
 isoniazid
 calcium channel blockers
 heroin
 busulfan
 methyldopa

Treatment options

 identify and manage any overt underlying cause


 liposuction provides the best cosmetic outcome

Next question
A 72 year old female is found to have a malignant lesion in her left arm. She had a mastectomy of
the left breast 10 years ago and has chronic lymph oedema of the left arm. What is the most likely
cause of the malignancy?

Lymphangiosarcoma

Lymphoma

Myeloma

Angiomyolipoma

Giant cell tumour

Lymphangiosarcoma is a rare condition arising as a result of chronic oedema. It is an aggressive


malignancy.
Please rate this question:

Discuss and give feedback


Next question

Lymphoedema

 Due to impaired lymphatic drainage in the presence of normal capillary function.


 Lymphoedema causes the accumulation of protein rich fluid, subdermal fibrosis and dermal
thickening.
 Characteristically fluid is confined to the epifascial space (skin and subcutaneous tissues);
muscle compartments are free of oedema. It involves the foot, unlike other forms of oedema.
There may be a 'buffalo hump' on the dorsum of the foot and the skin cannot be pinched due
to subcutaneous fibrosis.

Causes of lymphoedema

Primary  Congenital < 1 year: sporadic, Milroy's disease


 Onset 1-35 years: sporadic, Meige's disease
 > 35 years: Tarda

Secondary  Bacterial/fungal/parasitic infection (filariasis)


 Lymphatic malignancy
 Radiotherapy to lymph nodes
 Surgical resection of lymph nodes
 DVT
 Thrombophlebitis

Indications for surgery

 Marked disability or deformity from limb swelling


 Lymphoedema caused by proximal lymphatic obstruction with patent distal lymphatics
suitable for a lymphatic drainage procedure
 Lymphocutaneous fistulae and megalymphatics

Procedures
Homans operation Reduction procedure with preservation of overlying skin (which must be in
good condition). Skin flaps are raised and the underlying tissue excised.
Limb circumference typically reduced by a third.

Charles operation All skin and subcutaneous tissue around the calf are excised down to the
deep fascia. Split skin grafts are placed over the site. May be performed if
overlying skin is not in good condition. Larger reduction in size than with
Homans procedure.

Lymphovenous Identifiable lymphatics are anastomosed to sub dermal venules. Usually


anastamosis indicated in 2% of patients with proximal lymphatic obstruction and normal
distal lymphatics.

Next question
A 58 year old male is referred to endocrinology clinic for a parathyroidectomy by the F1 in medicine.
His corrected calcium is 2.85 (2.2-2.6), PTH 7.5 (3-7) and 24h urinary calcium is 1.5 (2.5-7.5). What
is the diagnosis?

Primary hyperparathyroidism

Secondary hyperparathyroidism

Tertiary hyperparathyroidism

Familial hypocalciuric hypercalcaemia

Hypercalacemia associated with malignancy

This F1 should have spoken to his senior. This patient has familial hypocalciuric hypercalcaemia,
which requires no further action. A calcium to creatinine clearance ratio of <0.01 will confirm this
diagnosis.
Please rate this question:

Discuss and give feedback


Next question

Parathyroid glands and disorders of calcium metabolism

Hyperparathyroidism
Disease type Hormone profile Clinical features Cause

Primary  PTH (Elevated)  May be Most cases due to


hyperparathyroidism  Ca2+(Elevated) asymptomatic if solitary adenoma
 Phosphate (Low) mild (80%), multifocal
 Urine calcium :  Recurrent disease occurs in 10-
creatinine clearance abdominal pain 15% and parathyroid
ratio > 0.01 (pancreatitis, renal carcinoma in 1% or
colic) less
 Changes to
emotional or
cognitive state
Disease type Hormone profile Clinical features Cause

Secondary  PTH (Elevated)  May have few Parathyroid gland


hyperparathyroidism  Ca2+ (Low or symptoms hyperplasia occurs as
normal)  Eventually may a result of low
 Phosphate develop bone calcium, almost
(Elevated) disease, osteitis always in a setting of
 Vitamin D levels fibrosa cystica and chronic renal failure
(Low) soft tissue
calcifications

Tertiary  Ca2+(Normal or  Metastatic Occurs as a result of


hyperparathyroidism high) calcification ongoing hyperplasia
 PTH (Elevated)  Bone pain and / of the parathyroid
 Phosphate levels or fracture glands after
(Decreased or  Nephrolithiasis correction of
Normal)  Pancreatitis underlying renal
 Vitamin D (Normal disorder, hyperplasia
or decreased) of all 4 glands is
 Alkaline usually the cause
phosphatase
(Elevated)

Differential diagnoses
It is important to consider the rare but relatively benign condition of benign familial hypocalciuric
hypercalcaemia, caused by an autosomal dominant genetic disorder. Diagnosis is usually made by
genetic testing and concordant biochemistry (urine calcium : creatinine clearance ratio <0.01-
distinguished from primary hyperparathyroidism).

Treatment

Primary hyperparathyroidism
Indications for surgery

 Elevated serum Calcium > 1mg/dL above normal


 Hypercalciuria > 400mg/day
 Creatinine clearance < 30% compared with normal
 Episode of life threatening hypercalcaemia
 Nephrolithiasis
 Age < 50 years
 Neuromuscular symptoms
 Reduction in bone mineral density of the femoral neck, lumbar spine, or distal radius of more
than 2.5 standard deviations below peak bone mass (T score lower than -2.5)
Secondary hyperparathyroidism
Usually managed with medical therapy.

Indications for surgery in secondary (renal) hyperparathyroidism:

 Bone pain
 Persistent pruritus
 Soft tissue calcifications

Tertiary hyperparathyroidism
Allow 12 months to elapse following transplant as many cases will resolve
The presence of an autonomously functioning parathyroid gland may require surgery. If the culprit
gland can be identified then it should be excised. Otherwise total parathyroidectomy and re-
implantation of part of the gland may be required.

References
1. Pitt S et al. Secondary and Tertiary Hyperparathyroidism, State of the Art Surgical
Management.Surg Clin North Am 2009 Oct;89(5):1227-39.

2. MacKenzie-Feder J et al. Primary Hyperparathyroidism: An Overview. Int J Endocrinol 2011;


2011: 251410.
Next question
Theme: Management of breast cancer

A. Simple mastectomy alone


B. Radical mastectomy alone
C. Simple mastectomy and sentinel lymph node biopsy
D. Wide local excision and sentinel lymph node biopsy
E. Simple mastectomy and axillary node clearance
F. Radical mastectomy and axillary node clearance
G. Wide local excision and axillary node clearance
H. Wide local excision alone

Please select the most appropriate treatment for the situation described. Each option may be used
once, more than once or not at all.

15. A 44 year old lady presents with a mass in the upper outer quadrant of her right breast.
Imaging, histology and clinical examination confirm a 1.5cm malignant mass lesion with
no clinical evidence of axillary nodal disease.

You answered Simple mastectomy alone

The correct answer is Wide local excision and sentinel lymph node biopsy

A small peripheral lesion such as this would usually be suitable for breast conserving
surgery. Since imaging and clinical examination is not suspicious for axillary disease, a
sentinel lymph node biopsy should be performed.

16. A 44 year old lady presents with a mass lesion in the upper outer quadrant of the left
breast. On clinical examination she has a 2cm mass lesion which on core biopsy is
demonstrated to have invasive ductal carcinoma. An FNA of a bulky axillary lymph node
contains malignant cells.

You answered Simple mastectomy alone

The correct answer is Wide local excision and axillary node clearance

Although the primary lesion is small enough for breast conserving surgery, the presence of
overt axillary lymph node metastasis will attract a recommendation for axillary node
clearance.

17. A 39 year old lady presents with a mass lesion in her right breast. Clinical examination,
biopsy and imaging confirm a 2.5 cm lesion in the upper inner quadrant of her right breast
and a 1.5 cm lesion at the central aspect of the same breast. Her axilla shows
lymphadenopathy and a fine needle aspirate from the node shows malignant cells.
You answered Simple mastectomy alone

The correct answer is Simple mastectomy and axillary node clearance

A combination of established axillary disease and multifocal invasive lesions attracts an


indication for mastectomy and axillary clearance. A radical mastectomy is less frequently
indicated in modern surgical practice, disease that is locally advanced is often best
downstaged using medical therapy, rather than embarking on the operations for breast
cancer that were first popularised over 100 years ago.

Please rate this question:

Discuss and give feedback


Next question

Breast cancer management

 Surgery is performed in most patients suffering from breast cancer.


 Chemotherapy may be used to downstage tumours and allow breast conserving surgery.
Hormonal therapy may also be used for the same purposes.
 Radiotherapy is given to all patients who have undergone breast conserving surgery.
 Patients who have undergone mastectomy may be offered a reconstructive procedure either
in conjunction with their primary resection or as a staged procedure at a later date.

Surgical options
Mastectomy vs Wide local excision

Mastectomy Wide Local Excision

Multifocal tumour Solitary lesion

Central tumour Peripheral tumour

Large lesion in small breast Small lesion in large breast

DCIS >4cm DCIS <4cm


Patient Choice Patient choice

Central lesions may be managed using breast conserving surgery, where an acceptable cosmetic
result may be obtained, this is rarely the case in small breasts

Axillary disease

 As a minimum, all patients with invasive breast cancer should have their axilla staged. In
those who do not have overt evidence of axillary nodal involvement this can be undertaken
using sentinel lymph node biopsy.
 Patients with a positive sentinel lymph node biopsy or who have imaging and cytological or
histological evidence of axillary nodal metastasis should undergo axillary node clearance.
 Axillary node clearance is associated with the development of lymphoedema, increased risk
of cellulitis and frozen shoulder.

Next question
A 50 year old lady is commenced on tamoxifen for the treatment of an oestrogen receptor positive
breast cancer. Which of the following malignancies are associated with tamoxifen use?

Adenocarcinoma of the colon

Hodgkins lymphoma

Adenocarcinoma of the lung

Ovarian cancer

Endometrial cancer

Tamoxifen is an oestrogen receptor antagonist in breast tissues. However, at other sites, such as
the endometrium it may act as an agonist. Hence the reason for increasing risk of endometrial
cancer.
Please rate this question:

Discuss and give feedback


Next question

Tamoxifen

 Synthetic partial oestrogen agonist, acts primarily by binding to the oestrogen receptor.
 Half life of 7 days, takes 4 weeks for drug to reach plasma steady state.
 Should usually be considered in patients with oestrogen receptor positive tumours
(alternative agents may be preferred in some groups).
 Although antagonistic with respects to breast tissue tamoxifen may serve as an agonist at
other sites. Therefore risk of endometrial cancer is increased, preservation of bone density
and decreased cardiovascular risks.
 Climateric side effects are common, 3% stop taking the drug because of these.
 Aromatase inhibitors are an alternative class of drugs, these work by blocking the peripheral
aromatization of androgens (post menopausal women produce oestrogens in this way). They
may treat cancers for which tamoxifen is no longer effective.

Next question
Theme: Thyroid nodules

A. Toxic adenoma
B. Anaplastic carcinoma of thyroid
C. Follicular carcinoma of thyroid
D. Papillary carcinoma of thyroid
E. Medullary carcinoma of thyroid
F. Thyroid lymphoma
G. Multinodular goitre
H. Parathyroid gland tumour

For each scenario please select the most likely underlying diagnosis. Each option may be used
once, more than once or not at all.

19. A 52 year old woman with known Hashimotos thyroiditis presents with a neck swelling.
She describes it as rapidly increasing in size over 3 months and she complains of
dysphagia to solids. On examination there is an asymmetrical swelling of the thyroid
gland.

You answered Toxic adenoma

The correct answer is Thyroid lymphoma

Thyroid lymphoma (Non Hodgkin's B cell lymphoma) is rare. It should be considered in


patients with a background of Hashimoto's thyroiditis and a rapid growth in size of the
thyroid gland. Diagnosis can be made with core needle biopsy; however an incisional
biopsy may be needed. Radiotherapy is the main treatment option.

20. A 52 year old woman presents with a neck swelling. On examination she is noted to have
single nodule on the thyroid gland. A CXR shows two mass lesions.

You answered Toxic adenoma

The correct answer is Follicular carcinoma of thyroid

A solitary nodule with signs of haematogenous spread indicates a follicular tumour. Note
that papillary tumours tend to be multinodular and spread via the lymphatic
system.Lymphatic spread from a papillary thyroid cancer is nearly always to neck nodes in
the first instance and mediastinal lymphadenopathy is vanishingly rare. Lung lesions are
typically synonymous with haematogenous metastasis of which a follicular lesion is the
most likely culprit.

21. A 52 year old woman presents with a neck swelling. Her GP reports that her TSH value is
low at 0.01. A scintigraphy demonstrates a hot nodule.
Toxic adenoma

This lady has thyrotoxicosis (low TSH) and a hot solitary nodule indicating a toxic
adenoma. Thyroid cancer rarely causes thyrotoxicosis or hot nodules.

Please rate this question:

Discuss and give feedback


Next question

Thyroid disease

Patients may present with a number of different manifestations of thyroid disease. They can be
broadly sub classified according to whether they are euthyroid or have clinical signs of thyroid
dysfunction. In addition it needs to be established whether they have a mass or not.

Assessment

 History
 Examination including USS
 If a nodule is identified then it should be sampled ideally via an image guided fine needle
aspiration
 Radionucleotide scanning is of limited use

Thyroid Tumours

 Papillary carcinoma
 Follicular carcinoma
 Anaplastic carcinoma
 Medullary carcinoma
 Lymphoma's

Multinodular goitre

 One of the most common reasons for presentation


 Provided the patient is euthyroid and asymptomatic and no discrete nodules are seen, they
can be reassured.
 In those with compressive symptoms surgery is required and the best operation is a total
thyroidectomy.
 Sub total resections were practised in the past and simply result in recurrent disease that
requires a difficult revisional resection.
Endocrine dysfunction

 In general these patients are managed by physicians initially.


 Surgery may be offered alongside radio iodine for patients with Graves disease that fails with
medical management or in patients who would prefer not to be irradiated (e.g. pregnant
women).
 Patients with hypothyroidism do not generally get offered a thyroidectomy. Sometimes
people inadvertently get offered resections during the early phase of Hashimotos thyroiditis,
however, with time the toxic phase passes and patients can simply be managed with
thyroxine.

Complications following surgery

 Anatomical such as recurrent laryngeal nerve damage.


 Bleeding. Owing to the confined space haematoma's may rapidly lead to respiratory
compromise owing to laryngeal oedema.
 Damage to the parathyroid glands resulting in hypocalcaemia.

Further sources of information


1. http://www.acb.org.uk/docs/TFTguidelinefinal.pdf- Association of Clinical Biochemistry guidelines
for thyroid function tests.

2. British association of endocrine surgeons website- http://www.baets.org.uk


Next question
Which investigation is best for initial assessment of recurrence of follicular carcinoma of the thyroid?

Free T4

Thyroid stimulating hormone

Scintigraphy

Serum thyroglobulin

USS thyroid gland

Elevated thyroglobulin levels raises suspicion of recurrence.


Please rate this question:

Discuss and give feedback


Next question

Thyroid malignancy

Papillary carcinoma

 Commonest sub-type
 Accurately diagnosed on fine needle aspiration cytology
 Histologically, they may demonstrate psammoma bodies (areas of calcification) and so
called 'orphan Annie' nuclei
 They typically metastasise via the lymphatics and thus laterally located apparently ectopic
thyroid tissue is usually a metastasis from a well differentiated papillary carcinoma

Follicular carcinoma

 Are less common than papillary lesions


 Like papillary tumours, they may present as a discrete nodule. Although they appear to be
well encapsulated macroscopically there is invasion on microscopic evaluation
 Lymph node metastases are uncommon and these tumours tend to spread
haematogenously. This translates into a higher mortality rate
 Follicular lesions cannot be accurately diagnosed on fine needle aspiration cytology and thus
all follicular FNA's (THY 3f) will require at least a hemi thyroidectomy
Anaplastic carcinoma

 Less common and tend to occur in elderly females


 Disease is usually advanced at presentation and often only palliative decompression and
radiotherapy can be offered.

Medullary carcinoma

 These are tumours of the parafollicular cells ( C Cells) and are of neural crest origin.
 The serum calcitonin may be elevated which is of use when monitoring for recurrence.
 They may be familial and occur as part of the MEN -2A disease spectrum.
 Spread may be either lymphatic or haematogenous and as these tumours are not derived
primarily from thyroid cells they are not responsive to radioiodine.

Lymphoma

 These respond well to radiotherapy


 Radical surgery is unnecessary once the disease has been diagnosed on biopsy material.
Such biopsy material is not generated by an FNA and thus a core biopsy has to be obtained
(with care!).

Next question
A 33 year old lady attends the clinic with a 3 month history of palpitations and irritability. Her thyroid
function, PTH and calcium are measured:
Thyroid function
Free T4 40 pmol/L

TSH < 0.1 miu/L

Free T3 25 p mol/L (normal 3.5-7.7 p mol/L)

PTH 10pg/ml (normal 10-55pg/ml)


(Normal values listed in reference range link)
What is the most likely diagnosis?

Hypothyroidism

Hyperthyroidism

Hypoparathyroidism

Hyperparathyroidism

Euthyroid

Theme from April 2012 Exam


Elevated T4 and suppressed TSH makes this the most likely diagnosis. The PTH level is normal.
Please rate this question:

Discuss and give feedback


Next question

Hyperthyroidism

Causes of hyperthyroidism include:

 Diffuse toxic goitre (Graves Disease)


 Toxic nodular goitre
 Toxic nodule
 Rare causes
Graves disease
Graves disease is characterised by a diffuse vascular goitre that appears at the same time as the
clinical manifestations of hyperthyroidism. It is commonest in younger females and may be
associated with eye signs. Thyrotoxic symptoms will predominate. Up to 50% of patients will have a
familial history of autoimmune disorders. The glandular hypertrophy and hyperplasia occur as a
result of the thyroid stimulating effects of the TSH receptor antibodies.

Toxic nodular goitre


In this disorder the goitre is present for a long period of time prior to the development of clinical
symptoms. In most goitres the nodules are inactive and in some cases it is the internodular tissue
that is responsible for the hyperthyroidism.

Toxic nodule
Overactive, autonomously functioning nodule. It may occur as part of generalised nodularity or be a
true toxic adenoma. The TSH levels are usually low as the autonomously functioning thyroid tissue
will exert a negative feedback effect.

Signs and symptoms


Symptoms Signs

Lethargy Tachycardia

Emotionally labile Agitation

Heat intolerance Hot, moist palms

Weight loss Exopthalmos

Excessive appetite Thyroid goitre and bruit

Palpitations Lid lag/retraction

Diagnosis
The most sensitive test for diagnosing hyperthyroidism is plasma T3 (which is raised). Note in
hypothyroidism the plasma T4 and TSH are the most sensitive tests. A TSH level of <0.5U/L
suggests hyperthyroidism. TSH receptor antibodies may be tested for in the diagnosis of Graves.

Treatment
First line treatment for Graves disease is usually medical and the block and replace regime is the
favored option. Carbimazole is administered at higher doses and thyroxine is administered orally.
Patient are maintained on this regime for between 6 and 12 months. Attempts are then made to
wean off medication. Where relapse then occurs the options are between ongoing medical therapy,
radioiodine or surgery.
Next question
A 23 year old lady has Graves disease that has relapsed on stopping anti thyroid drugs, radioiodine
is offered as the next treatment by the endocrinologists. Which statement is false?

Close contact with children is not permitted for up to 4 weeks following treatment

15% of patients with opthalmopathy will see worsening of eye signs

Symptomatic improvement takes 6-8 weeks

Up to 80% of patients will become hypothyroid

It increases the risk of parathyroid carcinoma

Radio-iodine- may worsen opthalmopathy, contraindicated in pregnancy and those wishing to


concieve within 6 months.

Radioiodine vs. Surgery


Surgery Radioiodine

Symptomatic improvement within 10 days Symptomatic improvement takes up to 2


months

No effect on opthalmopathy Eye signs may worsen

Risk of damage to adjacent anatomical No risk of anatomical damage


structures

No restrictions on contact No contact with children for 4 weeks

Please rate this question:

Discuss and give feedback


Next question

Thyroid disease
Patients may present with a number of different manifestations of thyroid disease. They can be
broadly sub classified according to whether they are euthyroid or have clinical signs of thyroid
dysfunction. In addition it needs to be established whether they have a mass or not.

Assessment

 History
 Examination including USS
 If a nodule is identified then it should be sampled ideally via an image guided fine needle
aspiration
 Radionucleotide scanning is of limited use

Thyroid Tumours

 Papillary carcinoma
 Follicular carcinoma
 Anaplastic carcinoma
 Medullary carcinoma
 Lymphoma's

Multinodular goitre

 One of the most common reasons for presentation


 Provided the patient is euthyroid and asymptomatic and no discrete nodules are seen, they
can be reassured.
 In those with compressive symptoms surgery is required and the best operation is a total
thyroidectomy.
 Sub total resections were practised in the past and simply result in recurrent disease that
requires a difficult revisional resection.

Endocrine dysfunction

 In general these patients are managed by physicians initially.


 Surgery may be offered alongside radio iodine for patients with Graves disease that fails with
medical management or in patients who would prefer not to be irradiated (e.g. pregnant
women).
 Patients with hypothyroidism do not generally get offered a thyroidectomy. Sometimes
people inadvertently get offered resections during the early phase of Hashimotos thyroiditis,
however, with time the toxic phase passes and patients can simply be managed with
thyroxine.

Complications following surgery


 Anatomical such as recurrent laryngeal nerve damage.
 Bleeding. Owing to the confined space haematoma's may rapidly lead to respiratory
compromise owing to laryngeal oedema.
 Damage to the parathyroid glands resulting in hypocalcaemia.

Further sources of information


1. http://www.acb.org.uk/docs/TFTguidelinefinal.pdf- Association of Clinical Biochemistry guidelines
for thyroid function tests.

2. British association of endocrine surgeons website- http://www.baets.org.uk


Next question
Theme: Thyroid disease

A. Papillary carcinoma
B. Follicular carcinoma
C. Multinodular goitre
D. Parathyroid adenoma
E. Anaplastic thyroid carcinoma
F. Medullary carcinoma
G. Toxic nodule
H. Graves disease

Please select the most likely thyroid lesion for the scenario given. Each option may be used once,
more than once or not at all.

25. A 34 year old female presents with a thyroid nodule. She has a family history of thyroid
disease and both her sisters have undergone total thyroidectomies. Her past medical
history includes hypertension which has been difficult to manage.

You answered Papillary carcinoma

The correct answer is Medullary carcinoma

This is a typical scenario for medullary carcinoma in which a phaeochromocytoma may


also be present. It may be inherited in an autosomal dominant fashion and affected family
members may be offered prophylactic thyroidectomy.

26. A 46 year old man is admitted to hospital with a femoral shaft fracture that occurred
suddenly whilst he was out walking his dog. On examination there is no neurovascular
deficit distal to the fracture site. He has a large firm nodule in the left lobe of the thyroid,
there is no associated lymphadenopathy.

You answered Papillary carcinoma

The correct answer is Follicular carcinoma

Follicular carcinomas may metastasise haematogenously (often to bone) where they may
give rise to pathological fractures as in this case.

27. An 18 year old female presents with 3 nodules in the right lobe of the thyroid. Clinically
she is euthyroid and there is associated cervical lymphadenopathy. She has no family
history of thyroid disease.

Papillary carcinoma
Papillary thyroid cancers are the most common type of thyroid cancer and are the more
common in females (M:F=1:3). Papillary tumours are more likely to develop lymphatic
spread than follicular tumours.

Please rate this question:

Discuss and give feedback


Next question

Thyroid disease

Patients may present with a number of different manifestations of thyroid disease. They can be
broadly sub classified according to whether they are euthyroid or have clinical signs of thyroid
dysfunction. In addition it needs to be established whether they have a mass or not.

Assessment

 History
 Examination including USS
 If a nodule is identified then it should be sampled ideally via an image guided fine needle
aspiration
 Radionucleotide scanning is of limited use

Thyroid Tumours

 Papillary carcinoma
 Follicular carcinoma
 Anaplastic carcinoma
 Medullary carcinoma
 Lymphoma's

Multinodular goitre

 One of the most common reasons for presentation


 Provided the patient is euthyroid and asymptomatic and no discrete nodules are seen, they
can be reassured.
 In those with compressive symptoms surgery is required and the best operation is a total
thyroidectomy.
 Sub total resections were practised in the past and simply result in recurrent disease that
requires a difficult revisional resection.
Endocrine dysfunction

 In general these patients are managed by physicians initially.


 Surgery may be offered alongside radio iodine for patients with Graves disease that fails with
medical management or in patients who would prefer not to be irradiated (e.g. pregnant
women).
 Patients with hypothyroidism do not generally get offered a thyroidectomy. Sometimes
people inadvertently get offered resections during the early phase of Hashimotos thyroiditis,
however, with time the toxic phase passes and patients can simply be managed with
thyroxine.

Complications following surgery

 Anatomical such as recurrent laryngeal nerve damage.


 Bleeding. Owing to the confined space haematoma's may rapidly lead to respiratory
compromise owing to laryngeal oedema.
 Damage to the parathyroid glands resulting in hypocalcaemia.

Further sources of information


1. http://www.acb.org.uk/docs/TFTguidelinefinal.pdf- Association of Clinical Biochemistry guidelines
for thyroid function tests.

2. British association of endocrine surgeons website- http://www.baets.org.uk


Next question
A 19 year old male presents with bilateral gynaecomastia, poor vision and nipple discharge. Which
of the following blood tests is most likely to be abnormal?

Oestrogen

Testosterone

β HCG

Prolactin

Calcitonin

A combination of nipple discharge, gynaecomastia and poor vision may well be associated with a
prolactinoma. The poor vision results from compression of the optic chiasm resulting in bi temporal
hemianopia.
Please rate this question:

Discuss and give feedback


Next question

Gynaecomastia

Gynaecomastia describes an abnormal amount of breast tissue in males and is usually caused by
an increased oestrogen:androgen ratio. It is important to differentiate the causes of galactorrhoea
(due to the actions of prolactin on breast tissue) from those of gynaecomastia

Causes of gynaecomastia

 physiological: normal in puberty


 syndromes with androgen deficiency: Kallman's, Klinefelter's
 testicular failure: e.g. Mumps
 liver disease
 testicular cancer e.g. Seminoma secreting HCG
 ectopic tumour secretion
 hyperthyroidism
 haemodialysis
 drugs: see below
Drug causes of gynaecomastia

 spironolactone (most common drug cause)


 cimetidine
 digoxin
 cannabis
 finasteride
 oestrogens, anabolic steroids

Very rare drug causes of gynaecomastia

 tricyclics
 isoniazid
 calcium channel blockers
 heroin
 busulfan
 methyldopa

Treatment options

 identify and manage any overt underlying cause


 liposuction provides the best cosmetic outcome

Next question
Theme: Management of calcium metabolic disorders

A. No action needed
B. Intravenous fluid (0.9% N.Saline)
C. Risedronate and calcium supplements
D. Calcium supplements
E. Exploration and parathyroidectomy
F. DEXA bone scan
G. Pamidronate IV

For each scenario please select the most appropriate management plan. Each option may be used
once, more than once or not at all.

29. An 80 year old woman has a hip fracture. Her calcium is normal. She has never been given
a diagnosis of osteoporosis.

You answered No action needed

The correct answer is Risedronate and calcium supplements

The osteoporosis guidelines state if a postmenopausal woman has a fracture she should be
put on bisphosphonates (there is no need for a DEXA scan).

30. A 60 year old man presents with recurrent renal stones. He is found to have a calcium of
2.72 (elevated) and a PTH of 12 (elevated).

You answered No action needed

The correct answer is Exploration and parathyroidectomy

This patient has primary hyperparathyroidism and nephrolithiasis, which is an indication


for parathyroidectomy.

31. An 82 year old woman from a nursing home is admitted to the orthopaedic ward with a hip
fracture. She is acutely confused and agitated. Her Calcium is 2.95 (elevated).

You answered No action needed

The correct answer is Intravenous fluid (0.9% N.Saline)

This patient needs rehydration due to hypercalcaemia. An intravenous bisphosphonate is


indicated if the Ca is above 3.
Please rate this question:

Discuss and give feedback


Next question

Parathyroid glands and disorders of calcium metabolism

Hyperparathyroidism
Disease type Hormone profile Clinical features Cause

Primary  PTH (Elevated)  May be Most cases due to


hyperparathyroidism  Ca2+(Elevated) asymptomatic if solitary adenoma
 Phosphate (Low) mild (80%), multifocal
 Urine calcium :  Recurrent disease occurs in 10-
creatinine clearance abdominal pain 15% and parathyroid
ratio > 0.01 (pancreatitis, renal carcinoma in 1% or
colic) less
 Changes to
emotional or
cognitive state

Secondary  PTH (Elevated)  May have few Parathyroid gland


hyperparathyroidism  Ca2+ (Low or symptoms hyperplasia occurs as
normal)  Eventually may a result of low
 Phosphate develop bone calcium, almost
(Elevated) disease, osteitis always in a setting of
 Vitamin D levels fibrosa cystica and chronic renal failure
(Low) soft tissue
calcifications

Tertiary  Ca2+(Normal or  Metastatic Occurs as a result of


hyperparathyroidism high) calcification ongoing hyperplasia
 PTH (Elevated)  Bone pain and / of the parathyroid
 Phosphate levels or fracture glands after
(Decreased or  Nephrolithiasis correction of
Normal)  Pancreatitis underlying renal
 Vitamin D (Normal disorder, hyperplasia
or decreased) of all 4 glands is
 Alkaline usually the cause
phosphatase
(Elevated)

Differential diagnoses
It is important to consider the rare but relatively benign condition of benign familial hypocalciuric
hypercalcaemia, caused by an autosomal dominant genetic disorder. Diagnosis is usually made by
genetic testing and concordant biochemistry (urine calcium : creatinine clearance ratio <0.01-
distinguished from primary hyperparathyroidism).

Treatment

Primary hyperparathyroidism
Indications for surgery

 Elevated serum Calcium > 1mg/dL above normal


 Hypercalciuria > 400mg/day
 Creatinine clearance < 30% compared with normal
 Episode of life threatening hypercalcaemia
 Nephrolithiasis
 Age < 50 years
 Neuromuscular symptoms
 Reduction in bone mineral density of the femoral neck, lumbar spine, or distal radius of more
than 2.5 standard deviations below peak bone mass (T score lower than -2.5)

Secondary hyperparathyroidism
Usually managed with medical therapy.

Indications for surgery in secondary (renal) hyperparathyroidism:

 Bone pain
 Persistent pruritus
 Soft tissue calcifications

Tertiary hyperparathyroidism
Allow 12 months to elapse following transplant as many cases will resolve
The presence of an autonomously functioning parathyroid gland may require surgery. If the culprit
gland can be identified then it should be excised. Otherwise total parathyroidectomy and re-
implantation of part of the gland may be required.

References
1. Pitt S et al. Secondary and Tertiary Hyperparathyroidism, State of the Art Surgical
Management.Surg Clin North Am 2009 Oct;89(5):1227-39.

2. MacKenzie-Feder J et al. Primary Hyperparathyroidism: An Overview. Int J Endocrinol 2011;


2011: 251410.
Next question
Theme: Management of thyroid disease

A. Total Thyroidectomy
B. Thyroid lobectomy
C. Sub total thyroidectomy
D. Radioactive iodine
E. Carbimazole
F. Tru cut biopsy
G. Further fine needle aspiration
H. Observation

For each scenario please select the most appropriate management option. Each option may be used
once, more than once or not at all.

32. A 59 year old man is referred with symptoms of dysphagia. On examination he has a large
goitre and on imaging there is significant retrosternal extension and features of a
multinodular goitre.

Total Thyroidectomy

Sub total thyroidectomy is no longer routinely undertaken in this group.

33. A 48 year old lady with thyrotoxicosis is referred to the clinic, she was poorly controlled
on carbimazole and has received orbital radiotherapy for severe proptosis. This has
improved matters but she relapsed on stopping her carbimazole.

Total Thyroidectomy

Eye signs worsen with radioiodine.

34. A 23 year old lady has re attended the clinic on three occasions with a cyst in her thyroid
that refills. Cytology on each occasion is reassuring.

You answered Total Thyroidectomy

The correct answer is Thyroid lobectomy

Persist refilling cysts may be associated with a well differentiated tumour and should be
removed by lobectomy.

Please rate this question:


Discuss and give feedback
Next question

Thyroid disease

Patients may present with a number of different manifestations of thyroid disease. They can be
broadly sub classified according to whether they are euthyroid or have clinical signs of thyroid
dysfunction. In addition it needs to be established whether they have a mass or not.

Assessment

 History
 Examination including USS
 If a nodule is identified then it should be sampled ideally via an image guided fine needle
aspiration
 Radionucleotide scanning is of limited use

Thyroid Tumours

 Papillary carcinoma
 Follicular carcinoma
 Anaplastic carcinoma
 Medullary carcinoma
 Lymphoma's

Multinodular goitre

 One of the most common reasons for presentation


 Provided the patient is euthyroid and asymptomatic and no discrete nodules are seen, they
can be reassured.
 In those with compressive symptoms surgery is required and the best operation is a total
thyroidectomy.
 Sub total resections were practised in the past and simply result in recurrent disease that
requires a difficult revisional resection.

Endocrine dysfunction

 In general these patients are managed by physicians initially.


 Surgery may be offered alongside radio iodine for patients with Graves disease that fails with
medical management or in patients who would prefer not to be irradiated (e.g. pregnant
women).
 Patients with hypothyroidism do not generally get offered a thyroidectomy. Sometimes
people inadvertently get offered resections during the early phase of Hashimotos thyroiditis,
however, with time the toxic phase passes and patients can simply be managed with
thyroxine.

Complications following surgery

 Anatomical such as recurrent laryngeal nerve damage.


 Bleeding. Owing to the confined space haematoma's may rapidly lead to respiratory
compromise owing to laryngeal oedema.
 Damage to the parathyroid glands resulting in hypocalcaemia.

Further sources of information


1. http://www.acb.org.uk/docs/TFTguidelinefinal.pdf- Association of Clinical Biochemistry guidelines
for thyroid function tests.

2. British association of endocrine surgeons website- http://www.baets.org.uk


Next question
Which of the following are not true of follicular thyroid cancer?

They often appear to be encapsulated.

Those with a Hurthle cell subtype have an excellent prognosis.

Haematogenous metastasis is more common than in Papillary carcinoma.

The overall mortality rate is 24%.

Vascular invasion is seen in up to 60% of cases.

The Hurthle cell subtype have a worse prognosis.


Please rate this question:

Discuss and give feedback


Next question

Thyroid malignancy

Papillary carcinoma

 Commonest sub-type
 Accurately diagnosed on fine needle aspiration cytology
 Histologically, they may demonstrate psammoma bodies (areas of calcification) and so
called 'orphan Annie' nuclei
 They typically metastasise via the lymphatics and thus laterally located apparently ectopic
thyroid tissue is usually a metastasis from a well differentiated papillary carcinoma

Follicular carcinoma

 Are less common than papillary lesions


 Like papillary tumours, they may present as a discrete nodule. Although they appear to be
well encapsulated macroscopically there is invasion on microscopic evaluation
 Lymph node metastases are uncommon and these tumours tend to spread
haematogenously. This translates into a higher mortality rate
 Follicular lesions cannot be accurately diagnosed on fine needle aspiration cytology and thus
all follicular FNA's (THY 3f) will require at least a hemi thyroidectomy
Anaplastic carcinoma

 Less common and tend to occur in elderly females


 Disease is usually advanced at presentation and often only palliative decompression and
radiotherapy can be offered.

Medullary carcinoma

 These are tumours of the parafollicular cells ( C Cells) and are of neural crest origin.
 The serum calcitonin may be elevated which is of use when monitoring for recurrence.
 They may be familial and occur as part of the MEN -2A disease spectrum.
 Spread may be either lymphatic or haematogenous and as these tumours are not derived
primarily from thyroid cells they are not responsive to radioiodine.

Lymphoma

 These respond well to radiotherapy


 Radical surgery is unnecessary once the disease has been diagnosed on biopsy material.
Such biopsy material is not generated by an FNA and thus a core biopsy has to be obtained
(with care!).

Next question
Theme: Thyroid function tests

A. Thyrotoxicosis

B. Sick euthyroid syndrome

C. Hypothyroidism

D. Poor compliance with thyroid medication

E. Hashimotos thyroiditis

F. Multinodular goitre

Please select the most likely diagnosis for the scenario given. Each option may be used once, more
than once or not at all.

36. A 52 year old woman presents with an acute ischaemic right arm. She is found to have fast atrial
fibrillation. Her blood results reveal a free T4 level of 20 and a TSH of < 0.01.

Thyrotoxicosis

Theme from January 2013


The diagnosis is thyrotoxicosis. An elevated T4 and a low TSH should indicate this diagnosis.
Remember atrial fibrillation and its complications i.e acute ischaemic limbs can be precipitated by
hyperthyroid disorders.

37. A 42 year old woman presents with a goitre. On examination the goitre feels 'lumpy'. The blood
results reveal a TSH of 12 and a free T4 of 2. Antithyroid peroxidase antibodies are high.

You answered Thyrotoxicosis

The correct answer is Hashimotos thyroiditis

Hashimotos usually presents in women aged between 30- 50 years. They are normally associated
with a goitre. To differentiate from hypothyroidism, the antithyroid peroxidase antibodies will be
elevated.
38. A 55 year old man is on the intensive care unit for many months after open aortic surgery. He is
maintained on total parenteral nutrition. Clinically he is euthyroid, but his thyroid function tests
reveal a low TSH and low T4.

You answered Thyrotoxicosis

The correct answer is Sick euthyroid syndrome

Sick euthyroid syndrome is most commonly seen in chronically ill patients or those with
starvation. The thyroid function tests are often low and the patient clinically euthyroid.

Please rate this question:

Discuss and give feedback

Next question

Thyroid function tests

The interpretation of thyroid function tests is usually straightforward:

Disorder TSH Free T4

Thyrotoxicosis (e.g. Graves' disease) Low High In T3 thyrotoxicosis the free T4 will be
normal

Primary hypothyroidism (primary High Low


atrophic hypothyroidism)

Secondary hypothyroidism Low Low Replacement steroid therapy is


required prior to thyroxine
Disorder TSH Free T4

Sick euthyroid syndrome* Low** Low Common in hospital inpatients

Poor compliance with thyroxine High Normal /


high

Steroid therapy Low Normal

*now referred to as non-thyroidal illness

**TSH may be normal in some cases

Next question
Theme: Nottingham prognostic index

A. <2.5

B. <3.4

C. 3.4-5.4

D. >5.4

Please match the prognosis of patients who have undergone breast cancer surgery to the most
appropriate Nottingham Prognostic Index score. Each option may be used once, more than once or
not at all.

39. Worst prognostic group

You answered <2.5

The correct answer is >5.4

The Nottingham prognostic index may be used to stratify patients into various prognostic groups
(see below). An excellent prognosis is seen with a score of <2.4. Scores of over 5 equate to a
greatly reduced survival rate.

40. Intermediate prognosis

You answered <2.5

The correct answer is 3.4-5.4

41. Excellent prognosis

<2.5
Please rate this question:

Discuss and give feedback

Next question

Nottingham prognostic index

The Nottingham Prognostic Index can be used to give an indication of survival following breast
cancer surgery. In this system, the tumour size is weighted less heavily than other major prognostic
parameters.

Calculation of NPI
Tumour Size x 0.2 + Lymph node score(From table below)+Grade score(From table below).

Lymph nodes involved Grade


Score

1 0 1

2 1-3 2

3 >3 3

Prognosis

Score Percentage 5 year survival

2.0 to 2.4 93%

2.5 to 3.4 85%


3.5 to 5.4 70%

>5.4 50%

This data was originally published in 1992. It should be emphasised that other factors such as
vascular invasion and receptor status also impact on survival and are not included in this data and
account for varying prognoses often cited in the literature.

Reference
Galea, M.H., et al., The Nottingham Prognostic Index in primary breast cancer. Breast Cancer Res
Treat, 1992. 22(3): p. 207-19.

Next question
A 56 year old lady undergoes a mastectomy as treatment for multifocal ductal carcinoma in situ. Two
weeks post operatively she attends the clinic and complains of a diffuse swelling at the surgical site.
On examination she has a large, fluctuant area underlying the mastectomy skin flaps. She is
otherwise well. What is the most likely cause?

Abscess

Seroma

Haematoma

Disease recurrence

Arteriovenous malformation

Seromas are very common after breast surgery. The exposed raw surfaces created during the
elevation of the skin flaps are a common cause. Treatment usually involves percutaneous drainage
under aseptic conditions.
Please rate this question:

Discuss and give feedback


Next question

Complications of breast surgery

Breast surgery may be associated with the following complications:

 Long thoracic nerve injury. This may occur during the axillary dissection and result in winging
of the scapula.
 Intercostobrachial nerve injury. These nerves traverse the axilla. When they are divided
(which they often are) the patient will notice an area of parasthesia in the armpit.
 Injury to the thoracodorsal trunk. This nerve and vessels supply latissimus dorsi. If they are
damaged the functional effects are not too serious, the greatest setback is that a latissimus
dorsi flap cannot be used for reconstruction purposes.
 Infections. Cellulitis of the chest wall and arm may be a major problem if axillary nodal
clearance is undertaken. Infections may run a protracted course and require polytherapy for
treatment.
 Lymphoedema. Usually complicates axillary node clearance or irradiation. Treatment is with
manual lymphatic drainage and compression sleeves.
 Seroma. This is an accumulation of fluid at the site of surgery. The fluid is usually straw
coloured and may re-accumulate despite drainage. Most will resolve with time.
You are the specialist trainee in endocrinology clinic. The medical team have referred a man for a
parathyroidectomy who has a corrected calcium of 2.82 (elevated) and a PTH of 11 (elevated).
Which of the following is not an indication for parathyroidectomy?

Nephrolithiasis

Reduction in bone mineral density of the femoral neck, lumbar spine, or distal radius of
more than 2.5 standard deviations below peak bone mass

Age < 50 years

Episode of life threatening hypercalcaemia

None of the above

All of the situations listed are indications for parathyroidectomy. See below for more information.
Please rate this question:

Discuss and give feedback


Next question

Parathyroid glands and disorders of calcium metabolism

Hyperparathyroidism
Disease type Hormone profile Clinical features Cause

Primary  PTH (Elevated)  May be Most cases due to


hyperparathyroidism  Ca2+(Elevated) asymptomatic if solitary adenoma
 Phosphate (Low) mild (80%), multifocal
 Urine calcium :  Recurrent disease occurs in 10-
creatinine clearance abdominal pain 15% and parathyroid
ratio > 0.01 (pancreatitis, renal carcinoma in 1% or
colic) less
 Changes to
emotional or
cognitive state

Secondary  PTH (Elevated)  May have few Parathyroid gland


Disease type Hormone profile Clinical features Cause

hyperparathyroidism  Ca2+ (Low or symptoms hyperplasia occurs as


normal)  Eventually may a result of low
 Phosphate develop bone calcium, almost
(Elevated) disease, osteitis always in a setting of
 Vitamin D levels fibrosa cystica and chronic renal failure
(Low) soft tissue
calcifications

Tertiary  Ca2+(Normal or  Metastatic Occurs as a result of


hyperparathyroidism high) calcification ongoing hyperplasia
 PTH (Elevated)  Bone pain and / of the parathyroid
 Phosphate levels or fracture glands after
(Decreased or  Nephrolithiasis correction of
Normal)  Pancreatitis underlying renal
 Vitamin D (Normal disorder, hyperplasia
or decreased) of all 4 glands is
 Alkaline usually the cause
phosphatase
(Elevated)

Differential diagnoses
It is important to consider the rare but relatively benign condition of benign familial hypocalciuric
hypercalcaemia, caused by an autosomal dominant genetic disorder. Diagnosis is usually made by
genetic testing and concordant biochemistry (urine calcium : creatinine clearance ratio <0.01-
distinguished from primary hyperparathyroidism).

Treatment

Primary hyperparathyroidism
Indications for surgery

 Elevated serum Calcium > 1mg/dL above normal


 Hypercalciuria > 400mg/day
 Creatinine clearance < 30% compared with normal
 Episode of life threatening hypercalcaemia
 Nephrolithiasis
 Age < 50 years
 Neuromuscular symptoms
 Reduction in bone mineral density of the femoral neck, lumbar spine, or distal radius of more
than 2.5 standard deviations below peak bone mass (T score lower than -2.5)

Secondary hyperparathyroidism
Usually managed with medical therapy.

Indications for surgery in secondary (renal) hyperparathyroidism:

 Bone pain
 Persistent pruritus
 Soft tissue calcifications

Tertiary hyperparathyroidism
Allow 12 months to elapse following transplant as many cases will resolve
The presence of an autonomously functioning parathyroid gland may require surgery. If the culprit
gland can be identified then it should be excised. Otherwise total parathyroidectomy and re-
implantation of part of the gland may be required.

References
1. Pitt S et al. Secondary and Tertiary Hyperparathyroidism, State of the Art Surgical
Management.Surg Clin North Am 2009 Oct;89(5):1227-39.

2. MacKenzie-Feder J et al. Primary Hyperparathyroidism: An Overview. Int J Endocrinol 2011;


2011: 251410.
Next question
Theme: Breast disease treatment

A. Mastectomy
B. Sentinel lymph node biopsy
C. Axillary node clearance
D. Wide local excision
E. Breast lump excision biopsy
F. Image guided wide local excision
G. Radiotherapy
H. Chemotherapy
I. Excision of margins
J. Discharge to routine follow-up

Please select the most appropriate management option from the list for the scenario given. Each
option may be used once, more than once or not at all.

44. A 35 year old woman has undergone a wide local excision. The histology shows an
invasive lobular carcinoma present at 3 of the resection margins. Cavity shavings taken at
the original operation are also involved. Sentinel node biopsy was negative.

Mastectomy

This patient has an extensive disease process and lobular cancers are notorious for being
multifocal. In this case a mastectomy is the safest next step.

45. A 56 year old woman has undergone a wide local excision and axillary node sample. The
histology shows a 2.5cm invasive ductal carcinoma grade 1 which is completely excised.
None of the axillary lymph nodes show evidence of metastatic disease.

You answered Mastectomy

The correct answer is Radiotherapy

This woman has a good prognosis. However, irradiation of the breast is mandatory after
breast cancer has been treated using breast conserving surgery.

46. A 28 year old female presents with a painless lump in the upper outer quadrant of her left
breast. Imaging using ultrasound is indeterminate (U3). Two core biopsies have now been
performed and both show normal breast tissue (B1).

You answered Mastectomy

The correct answer is Breast lump excision biopsy


The imaging and biopsy results are not concordant. At this stage an excision biopsy is the
safest option.

Please rate this question:

Discuss and give feedback


Next question

Breast cancer management

 Surgery is performed in most patients suffering from breast cancer.


 Chemotherapy may be used to downstage tumours and allow breast conserving surgery.
Hormonal therapy may also be used for the same purposes.
 Radiotherapy is given to all patients who have undergone breast conserving surgery.
 Patients who have undergone mastectomy may be offered a reconstructive procedure either
in conjunction with their primary resection or as a staged procedure at a later date.

Surgical options
Mastectomy vs Wide local excision

Mastectomy Wide Local Excision

Multifocal tumour Solitary lesion

Central tumour Peripheral tumour

Large lesion in small breast Small lesion in large breast

DCIS >4cm DCIS <4cm

Patient Choice Patient choice

Central lesions may be managed using breast conserving surgery, where an acceptable cosmetic
result may be obtained, this is rarely the case in small breasts

Axillary disease
 As a minimum, all patients with invasive breast cancer should have their axilla staged. In
those who do not have overt evidence of axillary nodal involvement this can be undertaken
using sentinel lymph node biopsy.
 Patients with a positive sentinel lymph node biopsy or who have imaging and cytological or
histological evidence of axillary nodal metastasis should undergo axillary node clearance.
 Axillary node clearance is associated with the development of lymphoedema, increased risk
of cellulitis and frozen shoulder.

Next question
Theme: Non operative treatment of breast cancer

A. Radiotherapy
B. Cytotoxic chemotherapy
C. Surveillence alone
D. Endocrine therapy using tamoxifen
E. Endocrine therapy using letrozole
F. Endocrine therapy using medroxyprogesterone acetate

Please select the most appropriate agent for the situation described. Each option may be used once,
more than once or not at all.

47. A 55 year old lady has undergone a wide local excision and sentinel lymph node biopsy
for breast cancer. The histology report shows a completely excised 1.3cm grade 1 invasive
ductal carcinoma. The sentinel node contained no evidence of metastatic disease. The
tumour is oestrogen receptor negative.

Radiotherapy

Radiotherapy is routine following breast conserving surgery. Without irradiation the local
recurrence rates are approximately 40%.

48. An 88 year old lady presents with a large mass in the upper inner quadrant of her right
breast. Investigations confirm an oestrogen receptor positive, invasive ductal carcinoma.
She has declined operative treatment.

You answered Radiotherapy

The correct answer is Endocrine therapy using letrozole

Elderly patients may be managed using endocrine therapy alone. Eventually most will
escape hormonal control. In post menopausal women oestrogens are produced by the
peripheral aromatization of androgens and aromatase inhibitors are therefore the most
popular agent in this age group.

49. A 38 year old lady has undergone a mastectomy and axillary node clearance for invasive
ductal carcinoma. The histology report shows a completely excised 3.5cm lesion which is
grade 3. Two of the axillary lymph nodes contain metastatic disease. The tumour is
oestrogen receptor negative.

You answered Radiotherapy

The correct answer is Cytotoxic chemotherapy


The combination of a grade 3 tumour and axillary nodal metastasis in a young female
would attract a recommendation for chemotherapy. Some may also add herceptin (if they
are HER 2 positive).

Please rate this question:

Discuss and give feedback


Next question

Breast cancer treatment

Treatment Indication

Endocrine  Oestrogen receptor positive tumours


therapy  Downstaging primary lesions
 Definitive treatment in old, infirm patients

Irradiation  Wide local excision


 Large lesion, high grade or marked vascular invasion following
mastectomy

Chemotherapy  Downstaging advanced lesions to facilitate breast conserving surgery


 Patients with grade 3 lesions or axillary nodal disease

Endocrine agents
Tamoxifen is used and works as a partial oestrogen receptor agonist. It will typically block activity at
the breast. It does, however, stimulate the receptors at other sites and it is this that accounts for its
association with endometrial cancer. In post menopausal women the process of aromatisation
accounts for most oestrogen production. Therefore in this group aromatase inhibitors are the
preferred agents. Women who are perimenopausal start on tamoxifen and switch at 3 years.

Chemotherapy
The FEC regime is most commonly used (Fluorouracil, epirubicin and cyclophosphamide). This was
found to be superior to the older CMF regime. The Taxanes are commonly used in high risk patients
and in this setting a regime of docetaxal, doxorubicin and cyclophosphamide may be used. The
anthracycline class drugs have marked cardiotoxicity (a property that they share with trastuzumab)
and this can limit their use.
Next question
Which of the following statements regarding papillary carcinoma of the thyroid is false?

They account for the majority of thyroid carcinomas

Spread predominantly via the lymphatics

May be diagnosed using fine needle aspiration cytology

When viewed microscopically may demonstrate 'orphan Annie 'nuclei

Have a five year survival of 65% if confined to the thyroid alone

The prognosis for localised papillary carcinomas is excellent. Survival rates at 5 years approach
90%.
Please rate this question:

Discuss and give feedback


Next question

Thyroid malignancy

Papillary carcinoma

 Commonest sub-type
 Accurately diagnosed on fine needle aspiration cytology
 Histologically, they may demonstrate psammoma bodies (areas of calcification) and so
called 'orphan Annie' nuclei
 They typically metastasise via the lymphatics and thus laterally located apparently ectopic
thyroid tissue is usually a metastasis from a well differentiated papillary carcinoma

Follicular carcinoma

 Are less common than papillary lesions


 Like papillary tumours, they may present as a discrete nodule. Although they appear to be
well encapsulated macroscopically there is invasion on microscopic evaluation
 Lymph node metastases are uncommon and these tumours tend to spread
haematogenously. This translates into a higher mortality rate
 Follicular lesions cannot be accurately diagnosed on fine needle aspiration cytology and thus
all follicular FNA's (THY 3f) will require at least a hemi thyroidectomy
Anaplastic carcinoma

 Less common and tend to occur in elderly females


 Disease is usually advanced at presentation and often only palliative decompression and
radiotherapy can be offered.

Medullary carcinoma

 These are tumours of the parafollicular cells ( C Cells) and are of neural crest origin.
 The serum calcitonin may be elevated which is of use when monitoring for recurrence.
 They may be familial and occur as part of the MEN -2A disease spectrum.
 Spread may be either lymphatic or haematogenous and as these tumours are not derived
primarily from thyroid cells they are not responsive to radioiodine.

Lymphoma

 These respond well to radiotherapy


 Radical surgery is unnecessary once the disease has been diagnosed on biopsy material.
Such biopsy material is not generated by an FNA and thus a core biopsy has to be obtained
(with care!).

Next question
Theme: Breast malignancy

A. Eczema of the nipple


B. Ductal carcinoma in situ
C. Mucinous carcinoma
D. Invasive ductal carcinoma
E. Invasive lobular carcinoma
F. Paget's disease of the nipple
G. Lobular carcinoma in situ.

What is the likely diagnosis for the scenario given? Each option may be used once, more than once,
or not at all.

51. A 74 year old woman presents with a breast lump. On examination is has a soft
consistency. The lump is removed and sliced apart. Macroscopically there is a grey,
gelatinous surface.

You answered Eczema of the nipple

The correct answer is Mucinous carcinoma

Mucinous carcinomas comprise 2-3% of all breast cancers. They are one of the special
type of carcinomas. These have a better prognosis that tumours of Non Special Type
(NST) and axillary nodal disease is rare in this group.

52. A 74 year old woman presents with an erythematous rash originating in the nipple. It is
spreading to the surrounding areolar area and the associated normal tissue.

You answered Eczema of the nipple

The correct answer is Paget's disease of the nipple

Paget's is associated with DCIS or invasive carcinoma.Unlike eczema of the nipple which
predominantly affects the areolar region, Pagets will usually affect the nipple first and then
spread to the areolar area. Diagnosis is made by punch biopsy.

53. A 53 year old woman presents with a bloody nipple discharge. On mammography there is
calcification behind the nipple areolar complex. A core biopsy shows background benign
change, but cells that show comedo necrosis which have not breached the basement
membrane.

You answered Eczema of the nipple


The correct answer is Ductal carcinoma in situ

Comedo necrosis is a feature of high nuclear grade ductal carcinoma in situ. It is has a high
risk of being associated with foci of invasion.

Please rate this question:

Discuss and give feedback

Breast cancer

 Commoner in the older age group


 Invasive ductal carcinomas are the most common type. Some may arise as a result
of ductal carcinoma in situ (DCIS). There are associated carcinomas of special type e.g.
Tubular that may carry better prognosis.
 The pathological assessment involves assessment of the tumour and lymph nodes, sentinel
lymph node biopsy is often used to minimise the morbidity of an axillary dissection.
 Treatment, typically this is either wide local excision or mastectomy. There are many sub
types of both of these that fall outside of the MRCS. Some key rules to bear in mind.
 Whatever operation is contemplated the final cosmetic outcome does have a bearing. A
woman with small breasts and a large tumour will tend to fare better with mastectomy, even
if clear pathological and clinical margins can be obtained. Conversely a women with larger
breasts may be able to undergo breast conserving surgery even with a relatively large
primary lesion (NB tumours >4cm used to attract recommendation for mastectomy). For
screen detected and impalpable tumour image guidance will be necessary.
 Reconstruction is always an option following any resectional procedure. However, its exact
type must be tailored to age and co-morbidities of the patient. The main operations in
common use include latissimus dorsi myocutaneous flap and sub pectoral implants. Women
wishing to avoid a prosthesis may be offered TRAM or DIEP flaps.

Surgical options
Mastectomy vs Wide local excision

Mastectomy Wide Local Excision

Multifocal tumour Solitary lesion

Central tumour Peripheral tumour

Large lesion in small breast Small lesion in large breast


Mastectomy Wide Local Excision

DCIS >4cm DCIS <4cm

Patient Choice Patient choice

Central lesions may be managed using breast conserving surgery where an acceptable cosmetic
result may be obtained, this is rarely the case in small breasts

A compelling indication for mastectomy, a larger tumour that would be unsuitable for breast
conserving surgery

Image sourced from Wikipedia

Whatever surgical option is chosen the aim should be to have a local recurrence rate of 5% or less
at 5 years [1].

Nottingham Prognostic Index


The Nottingham Prognostic Index can be used to give an indication of survival. In this system the
tumour size is weighted less heavily than other major prognostic parameters.

Calculation of NPI
Tumour Size x 0.2 + Lymph node score(From table below)+Grade score(From table below).
Score Lymph nodes involved Grade

1 0 1

2 1-3 2

3 >3 3

Prognosis

Score Percentage 5 year survival

2.0 to 2.4 93%

2.5 to 3.4 85%

3.5 to 5.4 70%

>5.4 50%

This data was originally published in 1992. It should be emphasised that other factors such as
vascular invasion and receptor status also impact on survival and are not included in this data and
account for varying prognoses often cited in the literature.

References
Surgical guidelines for the management of breast cancer, Association of Breast Surgery at BASO
2009, Eur J Surg Oncol (2009), doi:10.1016/j.ejso.2009.01.008
A 68 year old man with poorly controlled diabetes presents with severe otalgia and headaches. On
examination, there is granulation tissue within the external auditory meatus. What is the most likely
underlying infective agent?

Pseudomonas aeruginosa

Streptococcus pyogenes

Staphylococcus aureus

Actinomyces

Bacteroides fragilis

Malignant otitis externa is caused by Pseudomonas aeruginosa

Severe pain, headaches and granulation tissue within the external auditory meatus are key features
of malignant otitis externa. Diabetes mellitus is one of the commonest risk factors.
Please rate this question:

Discuss and give feedback


Next question

Malignant otitis externa

 Uncommon type of otitis externa that is found in immunocompromised individuals (90%


cases found in diabetics)
 Infective organism is usually Pseudomonas aeruginosa
 Infection commences in the soft tissues of the external auditory meatus, then progresses to
involve the soft tissues and into the bony ear canal
 Progresses to temporal bone osteomyelitis

Key features in history


Diabetes (90%) or immunosuppression (illness or treatment related)
Severe, unrelenting, deep-seated otalgia
Temporal headaches
Purulent otorrhea
Possibly dysphagia, hoarseness, and/or facial nerve dysfunction

Treatment
Anti pseudomonal antimicrobial agents
Topical agents
Hyperbaric oxygen is sometimes used in refractory cases
Next question
A 53 year old man presents with an ulcerated mass at the anal verge. A biopsy is taken and the
histology demonstrates as squamous cell carcinoma. Infection with which of the viruses below is
most likely to have contributed to the development of the condition?

Human papillomavirus 7

Human immunodeficiency virus 1

Human immunodeficiency virus 2

Human papillomavirus 16

Human T-lymphotropic virus 1

Theme from January 2013 exam


Infection with human papilloma virus 16 is a risk factor for the development of intra epithelial
dysplasia of the anal skin with subsequent increased risk of invasive malignancy.
Please rate this question:

Discuss and give feedback


Next question

Oncoviruses

 Viruses which cause cancer


 These may be detected on blood test and prevented by vaccine

These are the main types of oncoviruses and their diseases:

Oncovirus Cancer

Epstein-Barr virus Burkitt's lymphoma


Hodgkin's lymphoma
Post transplant lymphoma
Nasopharyngeal carcinoma
Human papillomavirus 16/18 Cervical cancer
Anal cancer
Penile cancer
Vulval cancer
Oropharyneal cancer

Human herpes virus 8 Kaposi's sarcoma

Hepatitis B virus Hepatocellular carcinoma

Hepatitis C virus Hepatocellular carcinoma

Human T-lymphotropic virus 1 Tropical spastic paraparesis


Adult T cell leukaemia

Next question
Theme: Parasitic infections

A. Giardia Infection
B. Cryptosporidium infection
C. Clonorchis sinensis infection
D. Ancylostoma duodenale infection
E. Ascaris lumbricoides infection
F. Echinococcus granulosus infection
G. Enterobius vermicularis infection

Please select the most likely infective organism for the scenario given. Each option may be used
once, more than once or not at all.

3. A 6 year old boy presents with symptoms of recurrent pruritus ani. On examination there is
evidence of a small worm like structure protruding from the anus.

You answered Giardia Infection

The correct answer is Enterobius vermicularis infection

Similar theme to September 2011 Exam


Infection with enterobius is extremely common. Pruritus is the main symptom, as there is a
lack of tissue invasion it is rare for individuals to have any signs of systemic sepsis.

4. A 58 year old man is reviewed in the clinic following a successful cadaveric renal transplant
the previous year. He has been able to return to work as a swimming instructor. Over the
past week he reports that he has been suffering from recurrent episodes of diarrhoea. It has
made him feel lethargic and exhausted. Stool microscopy shows evidence of cysts.

You answered Giardia Infection

The correct answer is Cryptosporidium infection

Cryptosporidium is associated with infection, particularly in those who are


immunocompromised. Diarrhoea is the main disease. The cysts are typically identified on
stool microscopy.

5. A 25 year old man returns from a backpacking holiday in India. He presents with symptoms
of coughing and also of episodic abdominal discomfort. Peri anal examination is normal.
Stool microscopy demonstrates both worms and eggs within the faeces.

You answered Giardia Infection


The correct answer is Ascaris lumbricoides infection

Infection with Ascaris lumbricoides usually occurs after individuals have visited places like
sub Saharan Africa or the far east. Unlike ancylostoma duodenale infection there is usually
evidence of both worms and eggs in the stool. The absence of pruritus makes enterobius less
likely. The presence of coughing may be due to the migration of the larva through the lungs.

Please rate this question:

Discuss and give feedback


Next question

Gastro intestinal parasitic infections

Common infections

Enterobiasis  Due to organism Enterobius vermicularis


 Common cause of pruritus ani
 Diagnosis usually made by placing scotch tape at the anus, this will
trap eggs that can then be viewed microscopically
 Treatment is with mebendazole

Ancylostoma  Hookworms that anchor in proximal small bowel


duodenale  Most infections are asymptomatic although may cause iron
deficiency anaemia
 Larvae may be found in stools left at ambient temperature,
otherwise infection is difficult to diagnose
 Infection occurs as a result of cutaneous penetration, migrates to
lungs, coughed up and then swallowed
 Treatment is with mebendazole

Ascariasis  Due to infection with roundworm Ascaris lumbricoides


 Infections begin in gut following ingestion, then penetrate duodenal
wall to migrate to lungs, coughed up and swallowed, cycle begins
again
 Diagnosis is made by identification of worm or eggs within faeces
 Treatment is with mebendazole

Strongyloidiasis  Due to infection with Strongyloides stercoralis


 Rare in west
 Organism is a nematode living in duodenum of host
 Initial infection is via skin penetration. They then migrate to lungs
and are coughed up and swallowed. Then mature in small bowel are
excreted and cycle begins again
 An auto infective cycle is also recognised where larvae will
penetrate colonic wall
 Individuals may be asymptomatic, although they may also have
respiratory disease and skin lesions
 Diagnosis is usually made by stool microscopy
 In the UK mebendazole is used for treatment

Cryptosporidium  Protozoal infection


 Organisms produce cysts which are excreted and thereby cause new
infections
 Symptoms consist of diarrhoea and cramping abdominal pains.
Symptoms are worse in immunosuppressed people
 Cysts may be identified in stools
 Treatment is with metronidazole

Giardiasis  Diarrhoeal infection caused by Giardia lamblia (protozoan)


 Infections occur as a result of ingestion of cysts
 Symptoms are usually gastrointestinal with abdominal pain,
bloating and passage of soft or loose stools
 Diagnosis is by serology or stool microscopy
 First line treatment is with metronidazole

Next question
A 29 year old female undergoes a sub total thyroidectomy. Five days post operatively the wound
becomes erythematous and discharges pus. What is the most likely causative organism?

Streptococcus pyogenes

Haemophilus influenzae

Pseudomonas aeruginosa

Staphylococcus aureus

Proteus mirabilis

Theme from 2011 Exam


In this setting Staphylococcus aureus Infection is the most likely cause. In the UK between 2010 and
2011 the commonest cause of wound infection was enterobacter infections (usually following cardiac
or colonic surgery). 23% of infections were due to Staph aureus, which fits the scenario above.
Infection with the other organisms including strep pyogenes was much rarer.
Please rate this question:

Discuss and give feedback


Next question

Surgical site infection

 Surgical site infections may occur following a breach in tissue surfaces and allow normal
commensals and other pathogens to initiate infection. They are a major cause of morbidity
and mortality.
 Surgical site infections (SSI) comprise up to 20% of all healthcare associated infections and
at least 5% of patients undergoing surgery will develop an SSI as a result.
 In many cases the organisms are derived from the patient's own body. Measures that may
increase the risk of SSI include:
 Shaving the wound using a razor (disposable clipper preferred)
 Using a non iodine impregnated incise drape if one is deemed to be necessary
 Tissue hypoxia
 Delayed administration of prophylactic antibiotics in tourniquet surgery

Preoperatively

 Don't remove body hair routinely


 If hair needs removal, use electrical clippers with single use head (razors increase infection
risk)
 Antibiotic prophylaxis if:

- placement of prosthesis or valve


- clean-contaminated surgery
- contaminated surgery

 Use local formulary


 Aim to give single dose IV antibiotic on anaesthesia
 If a tourniquet is to be used, give prophylactic antibiotics earlier

Intraoperatively

 Prepare the skin with alcoholic chlorhexidine (Lowest incidence of SSI)


 Cover surgical site with dressing
 A recent meta analysis has confirmed that administration of supplementary oxygen does not
reduce the risk of wound infection. In contrast to previous individual RCT's(1)
 Wound edge protectors do not appear to confer benefit (2)

Post operatively
Tissue viability advice for management of surgical wounds healing by secondary intention

Use of diathermy for skin incisions


In the NICE guidelines the use of diathermy for skin incisions is not advocated(3). Several
randomised controlled trials have been undertaken and demonstrated no increase in risk of SSI
when diathermy is used(4).

References
1. Brar M et al.. Perioperative supplemental oxygen in colorectal patients: a meta analysis. J Surg
Res2011 (166): 227 -235.
2. Pinkney T et al. Impact of wound edge protection devices on surgical site infection after
laparotomy: impact of a multicentre randomised controlled trial (ROSSINI Trial). BMJ 2013 (347):10.
3. http://www.nice.org.uk/CG74
4. Ahmad N and Ahmed A. Meta-analysis of the effectiveness of surgical scalpel or diathermy in
making abdominal skin incisions. Ann Surg 2011, 253(1):8-13.
Next question
Which of the following cancers is not associated with the human papillomavirus?

Anal cancer

Oropharyngeal cancer

Tracheal cancer

Vulval cancer

Penile cancer

HPV is associated with:

1. Cervical cancer (HPV 16/18 most common)


2. Anal cancer
3. Penile cancer
4. Vulval cancer
5. Oropharyngeal cancer
Please rate this question:

Discuss and give feedback


Next question

Oncoviruses

 Viruses which cause cancer


 These may be detected on blood test and prevented by vaccine

These are the main types of oncoviruses and their diseases:

Oncovirus Cancer

Epstein-Barr virus Burkitt's lymphoma


Hodgkin's lymphoma
Post transplant lymphoma
Nasopharyngeal carcinoma
Human papillomavirus 16/18 Cervical cancer
Anal cancer
Penile cancer
Vulval cancer
Oropharyneal cancer

Human herpes virus 8 Kaposi's sarcoma

Hepatitis B virus Hepatocellular carcinoma

Hepatitis C virus Hepatocellular carcinoma

Human T-lymphotropic virus 1 Tropical spastic paraparesis


Adult T cell leukaemia

Next question
Which of the following is not associated with Epstein-Barr virus?

Burkitt's lymphoma

Post-transplantation lymphoma

Hodgkin's lymphoma

Nasopharyngeal carcinoma

Mycosis fungoides

Please rate this question:

Discuss and give feedback


Next question

Oncoviruses

 Viruses which cause cancer


 These may be detected on blood test and prevented by vaccine

These are the main types of oncoviruses and their diseases:

Oncovirus Cancer

Epstein-Barr virus Burkitt's lymphoma


Hodgkin's lymphoma
Post transplant lymphoma
Nasopharyngeal carcinoma

Human papillomavirus 16/18 Cervical cancer


Anal cancer
Penile cancer
Vulval cancer
Oropharyneal cancer

Human herpes virus 8 Kaposi's sarcoma

Hepatitis B virus Hepatocellular carcinoma

Hepatitis C virus Hepatocellular carcinoma

Human T-lymphotropic virus 1 Tropical spastic paraparesis


Adult T cell leukaemia

Next question
Which organism is most likely to cause osteomyelitis in children with sickle cell disease?

Group A Streptococcus species

Staphylococcus aureus

Enterobacter

Salmonella species

Group B Streptococcus species

Theme from January 2013 Exam


Worldwide the Salmonella species remains the most common infecting organism in children with
osteomyelitis. Reference: "Etiology of Osteomyelitis Complicating Sickle Cell Disease (Burnett, et
al.Pediatrics 1998; 101:2 296-297)"
Please rate this question:

Discuss and give feedback


Next question

Osteomyelitis

Infection of the bone

Causes

 S aureus and occasionally Enterobacter or Streptococcus species


 In sickle cell: Salmonella species

Clinical features

 Erythema
 Pain
 Fever

Investigation
 X-ray: lytic centre with a ring of sclerosis
 Bone biopsy and culture

Treatment

 Prolonged antibiotics
 Sequestra may need surgical removal

Next question
A 22 year old man has undergone an inguinal hernia repair. Seven days later he presents with an
erythematous and tender wound that is discharging a purulent material. What is the most likely
cause?

Infection with Staphylococcus aureus

Discharging haematoma

Infection with Pseudomonas

Infection with Streptococcus pyogenes

Infection with Bacteroides

In this setting Staphylococcus aureus Infection is the most likely cause. In the UK between 2010 and
2011 the commonest cause of wound infection was enterobacter infections (usually following cardiac
or colonic surgery). 23% of infections were due to Staph aureus, which fits the scenario above.
Infection with the other organisms including strep pyogenes was much rarer.
Please rate this question:

Discuss and give feedback


Next question

Surgical site infection

 Surgical site infections may occur following a breach in tissue surfaces and allow normal
commensals and other pathogens to initiate infection. They are a major cause of morbidity
and mortality.
 Surgical site infections (SSI) comprise up to 20% of all healthcare associated infections and
at least 5% of patients undergoing surgery will develop an SSI as a result.
 In many cases the organisms are derived from the patient's own body. Measures that may
increase the risk of SSI include:
 Shaving the wound using a razor (disposable clipper preferred)
 Using a non iodine impregnated incise drape if one is deemed to be necessary
 Tissue hypoxia
 Delayed administration of prophylactic antibiotics in tourniquet surgery

Preoperatively

 Don't remove body hair routinely


 If hair needs removal, use electrical clippers with single use head (razors increase infection
risk)
 Antibiotic prophylaxis if:

- placement of prosthesis or valve


- clean-contaminated surgery
- contaminated surgery

 Use local formulary


 Aim to give single dose IV antibiotic on anaesthesia
 If a tourniquet is to be used, give prophylactic antibiotics earlier

Intraoperatively

 Prepare the skin with alcoholic chlorhexidine (Lowest incidence of SSI)


 Cover surgical site with dressing
 A recent meta analysis has confirmed that administration of supplementary oxygen does not
reduce the risk of wound infection. In contrast to previous individual RCT's(1)
 Wound edge protectors do not appear to confer benefit (2)

Post operatively
Tissue viability advice for management of surgical wounds healing by secondary intention

Use of diathermy for skin incisions


In the NICE guidelines the use of diathermy for skin incisions is not advocated(3). Several
randomised controlled trials have been undertaken and demonstrated no increase in risk of SSI
when diathermy is used(4).

References
1. Brar M et al.. Perioperative supplemental oxygen in colorectal patients: a meta analysis. J Surg
Res2011 (166): 227 -235.
2. Pinkney T et al. Impact of wound edge protection devices on surgical site infection after
laparotomy: impact of a multicentre randomised controlled trial (ROSSINI Trial). BMJ 2013 (347):10.
3. http://www.nice.org.uk/CG74
4. Ahmad N and Ahmed A. Meta-analysis of the effectiveness of surgical scalpel or diathermy in
making abdominal skin incisions. Ann Surg 2011, 253(1):8-13.
Next question
A 22 year old female is admitted to A&E with a splenic rupture. She has not been involved in any
trauma. What infection can cause spontaneous splenic rupture?

Measles

Mumps

Influenza

Epstein-Barr virus

Rubella

Theme from 2009 and 2011 Exams

Epstein - Barr Virus may cause generalised lymphadenopathy. This may be associated with
splenomegaly. This enlargement has been associated with spontaneous rupture.
Please rate this question:

Discuss and give feedback


Next question

Spleen

The spleen is located in the left upper quadrant of the abdomen and its size can vary depending
upon the amount of blood it contains. The typical adult spleen is 12.5cm long and 7.5cm wide. The
usual weight of the adult spleen is 150g.
The exact position of the spleen can vary with respiratory activity, posture and the state of
surrounding viscera. It usually lies obliquely with its long axis aligned to the 9th, 10th and 11th ribs. It
is separated from these ribs by both diaphragm and pleural cavity. The normal spleen is not
palpable.

The shape of the spleen is influenced by the state of the colon and stomach. Gastric distension will
cause the spleen to resemble the shape of an orange segment. Colonic distension will cause it to
become more tetrahedral.

The spleen is almost entirely covered by peritoneum, which adheres firmly to its capsule. Recesses
of the greater sac separate it from the stomach and kidney. It develops from the upper dorsal
mesogastrium, remaining connected to the posterior abdominal wall and stomach by two folds of
peritoneum; the lienorenal ligament and gastrosplenic ligament. The lienorenal ligament is derived
from peritoneum where the wall of the general peritoneum meets the omental bursa between the left
kidney and spleen; the splenic vessels lie in its layers. The gastrosplenic ligament also has two
layers, formed by the meeting of the walls of the greater sac and omental bursa between spleen and
stomach, the short gastric and left gastroepiploic branches of the splenic artery pass in its layers.
Laterally, the spleen is in contact with the phrenicocolic ligament.

Relations
Superiorly Diaphragm

Anteriorly Gastric impression

Posteriorly Kidney

Inferiorly Colon

Tail of pancreas and splenic vessels (splenic artery divides here, branches pass to the
Hilum
white pulp transporting plasma)

Contents
White Immune function. Contains central trabecular artery. The germinal centres are supplied
pulp by arterioles called penicilliary radicles.

Red pulp Filters abnormal red blood cells.

Function

 Filtration of abnormal blood cells and foreign bodies such as bacteria.


 Immunity: IgM. Production of properdin, and tuftsin which help target fungi and bacteria for
phagocytosis.
 Haematopoiesis: up to 5th month gestation or in haematological disorders.
 Pooling: storage of 40% platelets.
 Iron reutilisation
 Storage monocytes

Disorders of the spleen


Massive splenomegaly

 Myelofibrosis
 Chronic myeloid leukaemia
 Visceral leishmaniasis (kala-azar)
 Malaria
 Gaucher's syndrome

Other causes (as above plus)

 Portal hypertension e.g. secondary to cirrhosis


 Lymphoproliferative disease e.g. CLL, Hodgkin's
 Haemolytic anaemia
 Infection: hepatitis, glandular fever
 Infective endocarditis
 Sickle-cell*, thalassaemia
 Rheumatoid arthritis (Felty's syndrome)

*the majority of adult patients with sickle-cell will have an atrophied spleen due to repeated infarction
Next question
A group of consultant surgeons are meeting at a symposium. The chef preparing the canapes has
an infection on his finger. Approximately 40 minutes after eating the canapes the group are struck
down with severe vomiting. What is the most likely underlying explanation for this process?

Presence of enterotoxin from Staphylococcus aureus in the food

Presence of enterotoxin from Streptococcus pyogenes in the food

Infection with Campylobacter jejuni

Presence of enterotoxin from Clostridium perfringens in the food

Infection with Shigella soneii

Staphylococcus aureus may release an enterotoxin, this is preformed and thus will typically result in
rapid onset of symptoms in affected individuals.
Please rate this question:

Discuss and give feedback


Next question

Surgical Microbiology

An extensive topic so an overview is given here. Organisms causing common surgical infections are
reasonable topics in the examination. However, microbiology is less rigorously tested than anatomy,
for example.

Common organisms

Staphylococcus aureus

 Facultative anaerobe
 Gram positive coccus
 Haemolysis on blood agar plates
 Catalase positive
 20% population are long term carriers
 Exo and entero toxin may result in toxic shock syndrome and gastroenteritis respectively
 Ideally treated with penicillin although many strains now resistant through beta Lactamase
production. In the UK less than 5% of isolates are sensitive to penicillin.
 Resistance to methicillin (and other antibiotics) is mediated by the mec operon , essentially
penicillin binding protein is altered and resistance to this class of antibiotics ensues
 Common cause of cutaneous infections and abscesses

Streptococcus pyogenes

 Gram positive, forms chain like colonies, Lancefield Group A Streptococcus


 Produces beta haemolysis on blood agar plates
 Rarely part of normal skin microflora
 Catalase negative
 Releases a number of proteins/ virulence factors into host including hyaluronidase,
streptokinase which allow rapid tissue destruction
 Releases superantigens such as pyogenic exotoxin A which results in scarlet fever
 Remains sensitive to penicillin, macrolides may be used as an alternative.

Escherichia coli

 Gram negative rod


 Facultative anaerobe, non sporing
 Wide range of subtypes and some are normal gut commensals
 Some subtypes such as 0157 may produce lethal toxins resulting in haemolytic-uraemic
syndrome
 Enterotoxigenic E-Coli produces an enterotoxin (ST enterotoxin) that results in large
volume fluid secretion into the gut lumen (Via cAMP activation)
 Enteropathogenic E-Coli binds to intestinal cells and cause structural damage, this coupled
with a moderate (or in case of enteroinvasive E-Coli significant) invasive component
produces enteritis and large volume diarrhoea together with fever.
 They are resistant to many antibiotics used to treat gram positive infections and acquire
resistance rapidly and are recognised as producing beta lactamases

Campylobacter jejuni

 Curved, gram negative, non sporulating bacteria


 One of the commonest causes of diarrhoea worldwide
 Produces enteritis which is often diffuse and blood may be passed
 Remains a differential for right iliac fossa pain with diarrhoea
 Self limiting infection so antibiotics are not usually advised. However, the quinolones are
often rapidly effective.

Helicobacter pylori

 Gram negative, helix shaped rod, microaerophillic


 Produces hydrogenase that can derive energy from hydrogen released by intestinal bacteria
 Flagellated and mobile
 Those carrying the cag A gene may cause ulcers
 It secretes urease that breaks down gastric urea> Carbon dioxide and ammonia>
ammonium>bicarbonate (simplified!) The bicarbonate can neutralise the gastric acid.
 Usually colonises the gastric antrum and irritates resulting in increased gastrin release and
higher levels of gastric acid. These patients will develop duodenal ulcers. In those with more
diffuse H-Pylori infection gastric acid levels are lower and ulcers develop by local tissue
damage from H-Pylori- these patients get gastric ulcers.
 Diagnosis may be made by serology (approx. 75% sensitive). Biopsy urease test during
endoscopy probably the most sensitive.
 In patients who are colonised 10-20% risk of peptic ulcer, 1-2% risk gastric cancer, <1% risk
MALT lymphoma.

Next question
Theme: Infective organisms

A. Clostridium difficle
B. Cryptosporidium
C. Escherichia coli
D. Streptococcus bovis
E. Staphylococcus epidermidis
F. Clostridium botulinum
G. Staphylococcus aureus
H. Streptococcus pyogenes

Please select the most likely microorganism for the infection scenarios given. Each option may be
used once, more than once or not at all.

13. A 56 year old man undergoes a difficult colonoscopy for assessment of a caecal cancer. 48
hours after the procedure he is admitted with septicaemia. His abdomen is soft and non
tender. Blood cultures grow gram positive cocci.

You answered Clostridium difficle

The correct answer is Streptococcus bovis

Streptococcus bovis septicaemia is associated with carcinoma of the colon. It also can also
cause endocarditis.

14. A 23 year old woman is admitted with sepsis and right loin pain. She has a history of a
UTI that was treated by the GP with a course of trimethoprim that was commenced 24
hours previously.

You answered Clostridium difficle

The correct answer is Escherichia coli

Ascending infection is most likely to be caused by E-Coli (from the list).

15. A 34 year old homosexual is admitted with diarrhoea of 3 months duration. He is found to
by HIV positive with a CD4 count <50.

You answered Clostridium difficle

The correct answer is Cryptosporidium

Although a self limiting diarrhoea is the norm, this is not the case in immunocompromised
individuals who can develop severe illness. Other organisms may
include Salmonella,Shigella, and Campylobacter.

Please rate this question:

Discuss and give feedback


Next question

Surgical Microbiology

An extensive topic so an overview is given here. Organisms causing common surgical infections are
reasonable topics in the examination. However, microbiology is less rigorously tested than anatomy,
for example.

Common organisms

Staphylococcus aureus

 Facultative anaerobe
 Gram positive coccus
 Haemolysis on blood agar plates
 Catalase positive
 20% population are long term carriers
 Exo and entero toxin may result in toxic shock syndrome and gastroenteritis respectively
 Ideally treated with penicillin although many strains now resistant through beta Lactamase
production. In the UK less than 5% of isolates are sensitive to penicillin.
 Resistance to methicillin (and other antibiotics) is mediated by the mec operon , essentially
penicillin binding protein is altered and resistance to this class of antibiotics ensues
 Common cause of cutaneous infections and abscesses

Streptococcus pyogenes

 Gram positive, forms chain like colonies, Lancefield Group A Streptococcus


 Produces beta haemolysis on blood agar plates
 Rarely part of normal skin microflora
 Catalase negative
 Releases a number of proteins/ virulence factors into host including hyaluronidase,
streptokinase which allow rapid tissue destruction
 Releases superantigens such as pyogenic exotoxin A which results in scarlet fever
 Remains sensitive to penicillin, macrolides may be used as an alternative.

Escherichia coli
 Gram negative rod
 Facultative anaerobe, non sporing
 Wide range of subtypes and some are normal gut commensals
 Some subtypes such as 0157 may produce lethal toxins resulting in haemolytic-uraemic
syndrome
 Enterotoxigenic E-Coli produces an enterotoxin (ST enterotoxin) that results in large
volume fluid secretion into the gut lumen (Via cAMP activation)
 Enteropathogenic E-Coli binds to intestinal cells and cause structural damage, this coupled
with a moderate (or in case of enteroinvasive E-Coli significant) invasive component
produces enteritis and large volume diarrhoea together with fever.
 They are resistant to many antibiotics used to treat gram positive infections and acquire
resistance rapidly and are recognised as producing beta lactamases

Campylobacter jejuni

 Curved, gram negative, non sporulating bacteria


 One of the commonest causes of diarrhoea worldwide
 Produces enteritis which is often diffuse and blood may be passed
 Remains a differential for right iliac fossa pain with diarrhoea
 Self limiting infection so antibiotics are not usually advised. However, the quinolones are
often rapidly effective.

Helicobacter pylori

 Gram negative, helix shaped rod, microaerophillic


 Produces hydrogenase that can derive energy from hydrogen released by intestinal bacteria
 Flagellated and mobile
 Those carrying the cag A gene may cause ulcers
 It secretes urease that breaks down gastric urea> Carbon dioxide and ammonia>
ammonium>bicarbonate (simplified!) The bicarbonate can neutralise the gastric acid.
 Usually colonises the gastric antrum and irritates resulting in increased gastrin release and
higher levels of gastric acid. These patients will develop duodenal ulcers. In those with more
diffuse H-Pylori infection gastric acid levels are lower and ulcers develop by local tissue
damage from H-Pylori- these patients get gastric ulcers.
 Diagnosis may be made by serology (approx. 75% sensitive). Biopsy urease test during
endoscopy probably the most sensitive.
 In patients who are colonised 10-20% risk of peptic ulcer, 1-2% risk gastric cancer, <1% risk
MALT lymphoma.

Next question
What is the mechanism of action of macrolides?

Causes misreading of mRNA

Interferes with cell wall formation

Inhibits DNA synthesis

Inhibits RNA synthesis

Inhibits protein synthesis

Macrolides act by inhibiting bacterial protein synthesis. If pushed to give an answer they are
bacteriostatic in nature, but in reality this depends on the dose and type of organism being treated.
Erythromycin was the first macrolide used clinically. Newer examples include clarithromycin and
azithromycin.

Adverse effects of erythromycin

 gastrointestinal side-effects are common


 cholestatic jaundice: risk may be reduced if erythromycin stearate is used
 P450 inhibitor

Please rate this question:

Discuss and give feedback


Next question

Antibiotics: mechanism of action

The lists below summarise the site of action of the commonly used antibiotics

Inhibit cell wall formation

 penicillins
 cephalosporins

Inhibit protein synthesis


 aminoglycosides (cause misreading of mRNA)
 chloramphenicol
 macrolides (e.g. erythromycin)
 tetracyclines
 fusidic acid

Inhibit DNA synthesis

 quinolones (e.g. ciprofloxacin)


 metronidazole
 sulphonamides
 trimethoprim

Inhibit RNA synthesis

 rifampicin

Next question
Theme: Diarrhoea

A. Vibrio cholera
B. Yersinia enterocolitica
C. Clostridium difficile
D. Campylobacter jejuni
E. Salmonella spp.
F. E. coli (Enterohaemorrhagic)
G. E.Coli (Enteroinvasive)

What is the most likely infective organism for the scenario given? Each option may be used once,
more than once or not at all.

17. A 22 year old chef presents to the medical team with profuse bloody diarrhoea. On further
questioning he describes tenesmus. They arrange a sigmoidoscopy which reveals necrosis
and ulceration of the descending colon mucosa.

You answered Vibrio cholera

The correct answer is E.Coli (Enteroinvasive)

Necrosis and ulcers of the large bowel are a feature of enteroinvasive E.coli. It presents
with a dysentery type illness similar to shigellosis.

18. A 22 year old Chef presents with abdominal pain in the right iliac fossa. There is an
associated temperature and diarrhoea. The SHO takes the patient to theatre for an
appendicectomy, but the appendix appears normal. The terminal ileum appears thickened
and engorged.

You answered Vibrio cholera

The correct answer is Yersinia enterocolitica

Yersinia can be mistaken for acute appendicitis due to mesenteric lymphadenitis and
ileitis. Yersinia infection of the terminal ileum typically produces more marked clinical
changes of this segment of bowel than infection with campylobacter.

19. A 30 year old aid worker becomes unwell whilst helping at the scene of a recent
earthquake. He develops vomiting and soon afterwards a diarrhoea that is loose and
extremely watery.

Vibrio cholera
The passage of very loose and watery stools distinguishes cholera. Most gastroenteric
infections do not produce such watery motions.

Please rate this question:

Discuss and give feedback


Next question

Bacterial Gastroenteritis

Causative organisms Features

Campylobacterjejuni  Most common cause of acute infective diarrhoea


 Spiral, gram negative rods
 Usually infects caecum and terminal ileum. Local
lymphadenopathy is common
 May mimic appendicitis as it has marked right iliac fossa pain
 Reactive arthritis is seen in 1-2% of cases

Shigella spp.  Members of the enterobacteriaceae


 Gram negative bacilli
 Clinically causes dysentery
 Shigella soneii is the commonest infective organism (mild illness)
 Usually self limiting, ciprofloxacin may be required if individual
is in a high risk group

Salmonellaspp  Facultatively anaerobic, gram negative, enterobacteriaceae


 Infective dose varies according to subtype
 Salmonellosis: usually transmitted by infected meat (especially
poultry) and eggs

E. coli  Enteropathogenic
 Enteroinvasive: dysentery, large bowel necrosis/ulcers
 Enterotoxigenic: small intestine, travelers diarrhoea
 Enterohaemorrhagic: 0157, cause a haemorrhagic colitis,
haemolytic uraemic syndrome and thrombotic thrombocytopaenic
purpura

Yersinia  Gram negative, coccobacilli


Causative organisms Features

enterocolitica  Typically produces a protracted terminal ileitis that may mimic


Crohns disease
 Differential diagnosis acute appendicitis
 May progress to septicaemia in susceptible individuals
 Usually sensitive to quinolone or tetracyclines

Vibrio cholera  Short, gram negative rods


 Transmitted by contaminated water, seafood
 Symptoms include sudden onset of effortless vomiting and profuse
watery diarrhoea
 Correction of fluid and electrolyte losses are the mainstay of
treatment
 Most cases will resolve, antibiotics are not generally indicated

Next question
Which of the following is true regarding the Salmonella species?

Rose spots appear in all patients with typhoid

They are normally present in the gut as commensals

Subsequent chronic biliary infection occurs in 75% of cases

A relative bradycardia is often seen in typhoid fever

Salmonella typhi can be categorised into type A, B and C

Please rate this question:

Discuss and give feedback


Next question

Salmonella

The Salmonella group contains many members, most of which cause diarrhoeal diseases. They are
facultative anaerobes, Gram negative rods which are not normally present as commensals in the
gut.

Typhoid and paratyphoid are caused by Salmonella typhi and Salmonella paratyphi (types A, B & C)
respectively. They are often termed enteric fevers, producing systemic symptoms such as
headache, fever, arthralgia

Features

 initially systemic upset as above


 relative bradycardia
 abdominal pain, distension
 constipation: although Salmonella is a recognised cause of diarrhoea, constipation is more
common in typhoid
 rose spots: present on the trunk in 40% of patients, and are more common in paratyphoid

Possible complications include


 osteomyelitis (especially in sickle cell disease where Salmonella is one of the most common
pathogens)
 GI bleed/perforation
 meningitis
 cholecystitis
 chronic carriage (1%, more likely if adult females)

Next question
A 77-year-old female presents with a non-healing ulcer on her right foot. Blood cultures grow MRSA.
Which antibiotic would you consider in addition to vancomycin to cover this?

Flucloxacillin

Ceftazidime

Ciprofloxacin

Metronidazole

Rifampicin

The MRSA would or may be resistant to Other antibiotics. Rifampicin is normally given in
combination with another antibiotic.
Please rate this question:

Discuss and give feedback


Next question

MRSA

Methicillin-resistant Staphylococcus aureus (MRSA) was one of the first organisms which highlighted
the dangers of hospital-acquired infections.

Who should be screened for MRSA?

 all patients awaiting elective admissions (exceptions include day patients having terminations
of pregnancy and ophthalmic surgery. Patients admitted to mental health trusts are also
excluded)
 in the UK all emergency admissions are currently screened

How should a patient be screened for MRSA?

 nasal swab and skin lesions or wounds


 the swab should be wiped around the inside rim of a patient's nose for 5 seconds
 the microbiology form must be labelled 'MRSA screen'
Suppression of MRSA from a carrier once identified

 nose: mupirocin 2% in white soft paraffin, tds for 5 days


 skin: chlorhexidine gluconate, od for 5 days. Apply all over but particularly to the axilla, groin
and perineum

The following antibiotics are commonly used in the treatment of MRSA infections:

 vancomycin
 teicoplanin

Some strains may be sensitive to the antibiotics listed below but they should not generally be used
alone because resistance may develop:

 rifampicin
 macrolides
 tetracyclines
 aminoglycosides
 clindamycin

Relatively new antibiotics such as linezolid, quinupristin/dalfopristin combinations and tigecycline


have activity against MRSA but should be reserved for resistant cases
Next question
Theme: Infective organisms

A. E-coli and bacteroides


B. Staphylococcus aureus
C. Streptococcus viridians
D. Staphylococcus epidermidis
E. Klebsiella
F. Clostridium tetani
G. Clostridium difficile
H. None of the above

Which is the most likely infective organism for the scenario given? Each option may be used once,
more than once or not at all.

22. A 32 year old women undergoes mastectomy and latissimus dorsi flap reconstruction for
breast cancer, to provide optimal cosmesis a McGhan implant is placed under the
myocutaneous flap. Three weeks post operatively the patient continues to suffer from
recurrent wound infections that have proved resistant to multiple courses of antibiotics.

You answered E-coli and bacteroides

The correct answer is Staphylococcus epidermidis

This tends to colonise plastic devices and forms a biofilm which allows colonisation with
other bacterial agents. It is notoriously difficult to eradicate once established and the usual
treatment is removal of the device.

23. A 68 year old man with diabetes presents with an area of necrosis of the perineum at the
base of the scrotum, there is some surrounding erythema. He is systemically unwell and
hypotensive.

E-coli and bacteroides

This is likely to be Fournier's Gangrene. A number of agents are implicated. E-coli and
bacteroides are the most commonly isolated organisms. The key point is that both aerobic
and anaerobic organisms must be present and only A has this option.

24. A 68 year old women with previous rheumatic fever is admitted with pyrexia of unknown
origin. Her blood cultures are unhelpful but transoesophageal echocardiography reveals
vegetations on the mitral valve.

You answered E-coli and bacteroides


The correct answer is Streptococcus viridians

This is the most common organism affecting previously abnormal heart valves.

Please rate this question:

Discuss and give feedback


Next question

Surgical Microbiology

An extensive topic so an overview is given here. Organisms causing common surgical infections are
reasonable topics in the examination. However, microbiology is less rigorously tested than anatomy,
for example.

Common organisms

Staphylococcus aureus

 Facultative anaerobe
 Gram positive coccus
 Haemolysis on blood agar plates
 Catalase positive
 20% population are long term carriers
 Exo and entero toxin may result in toxic shock syndrome and gastroenteritis respectively
 Ideally treated with penicillin although many strains now resistant through beta Lactamase
production. In the UK less than 5% of isolates are sensitive to penicillin.
 Resistance to methicillin (and other antibiotics) is mediated by the mec operon , essentially
penicillin binding protein is altered and resistance to this class of antibiotics ensues
 Common cause of cutaneous infections and abscesses

Streptococcus pyogenes

 Gram positive, forms chain like colonies, Lancefield Group A Streptococcus


 Produces beta haemolysis on blood agar plates
 Rarely part of normal skin microflora
 Catalase negative
 Releases a number of proteins/ virulence factors into host including hyaluronidase,
streptokinase which allow rapid tissue destruction
 Releases superantigens such as pyogenic exotoxin A which results in scarlet fever
 Remains sensitive to penicillin, macrolides may be used as an alternative.

Escherichia coli
 Gram negative rod
 Facultative anaerobe, non sporing
 Wide range of subtypes and some are normal gut commensals
 Some subtypes such as 0157 may produce lethal toxins resulting in haemolytic-uraemic
syndrome
 Enterotoxigenic E-Coli produces an enterotoxin (ST enterotoxin) that results in large
volume fluid secretion into the gut lumen (Via cAMP activation)
 Enteropathogenic E-Coli binds to intestinal cells and cause structural damage, this coupled
with a moderate (or in case of enteroinvasive E-Coli significant) invasive component
produces enteritis and large volume diarrhoea together with fever.
 They are resistant to many antibiotics used to treat gram positive infections and acquire
resistance rapidly and are recognised as producing beta lactamases

Campylobacter jejuni

 Curved, gram negative, non sporulating bacteria


 One of the commonest causes of diarrhoea worldwide
 Produces enteritis which is often diffuse and blood may be passed
 Remains a differential for right iliac fossa pain with diarrhoea
 Self limiting infection so antibiotics are not usually advised. However, the quinolones are
often rapidly effective.

Helicobacter pylori

 Gram negative, helix shaped rod, microaerophillic


 Produces hydrogenase that can derive energy from hydrogen released by intestinal bacteria
 Flagellated and mobile
 Those carrying the cag A gene may cause ulcers
 It secretes urease that breaks down gastric urea> Carbon dioxide and ammonia>
ammonium>bicarbonate (simplified!) The bicarbonate can neutralise the gastric acid.
 Usually colonises the gastric antrum and irritates resulting in increased gastrin release and
higher levels of gastric acid. These patients will develop duodenal ulcers. In those with more
diffuse H-Pylori infection gastric acid levels are lower and ulcers develop by local tissue
damage from H-Pylori- these patients get gastric ulcers.
 Diagnosis may be made by serology (approx. 75% sensitive). Biopsy urease test during
endoscopy probably the most sensitive.
 In patients who are colonised 10-20% risk of peptic ulcer, 1-2% risk gastric cancer, <1% risk
MALT lymphoma.

Next question
A 48 year old lady undergoes an ERCP for jaundice. 36 hours following the procedure she develops
a fever and rigors. A blood culture is taken, which of the following organisms is most likely to be
cultured?

Pseudomonas aeruginosa

Streptococcus

Enterobacter

Staphylococcus

Escherichia coli

Charcots triad = Surgical emergency.


Patients need: Biliary decompression and broad spectrum antibiotics.

E Coli is the most common organism implicated in cholangitis infections. Whilst enterobacter is
occasionally cultured it is not the most common organism.
Please rate this question:

Discuss and give feedback


Next question

Cholangitis

 Combination of bacterial infection and biliary obstruction


 Most common organisms are: (most frequent at top of list)

Escherichia coli
Klebsiella species
Enterococcus species
Streptococcus species

Clinical features
Charcot's triad:
Fever (90% cases)
Right upper quadrant pain
Jaundice

Reynolds pentad: Above plus confusion and hypotension


Investigations
USS 1st line
CT scan
ERCP: may be 1st line if high clinical suspicion and suitable for treatment

Treatment
ERCP -usually after 72 hours of antibiotics
Percutaneous transhepatic cholangiogram and biliary drain
Next question
In a 72 year old man undergoing a sigmoid colectomy for diverticular disease, which of the following
interventions is most likely to reduce his risk of developing a post operative wound infection?

Using a plain clear incise type drape to cover the operative field

Administering mechanical bowel preparation pre operatively

Shaving his abdominal wall one day prior to surgery

Administration of single dose of broad spectrum antibiotics prior to the procedure

None of the above

Administration of prophylactic antibiotics will reduce the risk of wound infection. Plain incise drapes
increase the risk of wound infections and should not be used. Iodophor impregnated drapes have
been demonstrated to reduce the risk of wound infection. Shaving one day prior to surgery will
increase the risk.
Please rate this question:

Discuss and give feedback


Next question

Surgical site infection

 Surgical site infections may occur following a breach in tissue surfaces and allow normal
commensals and other pathogens to initiate infection. They are a major cause of morbidity
and mortality.
 Surgical site infections (SSI) comprise up to 20% of all healthcare associated infections and
at least 5% of patients undergoing surgery will develop an SSI as a result.
 In many cases the organisms are derived from the patient's own body. Measures that may
increase the risk of SSI include:
 Shaving the wound using a razor (disposable clipper preferred)
 Using a non iodine impregnated incise drape if one is deemed to be necessary
 Tissue hypoxia
 Delayed administration of prophylactic antibiotics in tourniquet surgery

Preoperatively

 Don't remove body hair routinely


 If hair needs removal, use electrical clippers with single use head (razors increase infection
risk)
 Antibiotic prophylaxis if:

- placement of prosthesis or valve


- clean-contaminated surgery
- contaminated surgery

 Use local formulary


 Aim to give single dose IV antibiotic on anaesthesia
 If a tourniquet is to be used, give prophylactic antibiotics earlier

Intraoperatively

 Prepare the skin with alcoholic chlorhexidine (Lowest incidence of SSI)


 Cover surgical site with dressing
 A recent meta analysis has confirmed that administration of supplementary oxygen does not
reduce the risk of wound infection. In contrast to previous individual RCT's(1)
 Wound edge protectors do not appear to confer benefit (2)

Post operatively
Tissue viability advice for management of surgical wounds healing by secondary intention

Use of diathermy for skin incisions


In the NICE guidelines the use of diathermy for skin incisions is not advocated(3). Several
randomised controlled trials have been undertaken and demonstrated no increase in risk of SSI
when diathermy is used(4).

References
1. Brar M et al.. Perioperative supplemental oxygen in colorectal patients: a meta analysis. J Surg
Res2011 (166): 227 -235.
2. Pinkney T et al. Impact of wound edge protection devices on surgical site infection after
laparotomy: impact of a multicentre randomised controlled trial (ROSSINI Trial). BMJ 2013 (347):10.
3. http://www.nice.org.uk/CG74
4. Ahmad N and Ahmed A. Meta-analysis of the effectiveness of surgical scalpel or diathermy in
making abdominal skin incisions. Ann Surg 2011, 253(1):8-13.
Next question
Which one of the following statements regarding hepatitis C is correct?

Cannot be transmitted vertically from mother to child

Interferon-alpha and ribavirin are the treatments of choice

It is more infectious than hepatitis B following a needle stick injury

Breast feeding is contraindicated in mothers with hepatitis C

HCV RNA is the initial investigation of choice for at-risk groups

Please rate this question:

Discuss and give feedback


Next question

Hepatitis C

Hepatitis C is likely to become a significant public health problem in the UK in the next decade. It is
thought around 200,000 people are chronically infected with the virus. At risk groups include
intravenous drug users and patients who received a blood transfusion prior to 1991 (e.g.
haemophiliacs).

Transmission

 the risk of transmission during a needle stick injury is about 2%


 the vertical transmission rate from mother to child is about 6%
 breast feeding is not contraindicated in mothers with hepatitis C
 the risk of transmitting the virus during sexual intercourse is probably less than 5%

Features

 after exposure to the hepatitis C virus less than 20% of patients develop an acute hepatitis

Complications
 chronic infection (80-85%) - only 15-20% of patients will clear the virus after an acute
infection and hence the majority will develop chronic hepatitis C
 cirrhosis (20-30% of those with chronic disease)
 hepatocellular cancer
 cryoglobulinaemia

Management of chronic infection

 currently a combination of pegylated interferon-alpha and ribavirin are used


 up to 55% of patients successfully clear the virus, with success rates of around 80% for
some strains

Complications of treatment

 ribavirin - side-effects: haemolytic anaemia, cough. Women should not become pregnant
within 6 months of stopping ribavirin as it is teratogenic
 interferon alpha - side-effects: flu-like symptoms, depression, fatigue, leukopenia,
thrombocytopenia

Next question
A 22 year old man presents with a 5 day history of sore throat, malaise and fatigue. On examination
he has a large peritonsillar abscess. What is the most likely underlying infective organism?

Epstein Barr Virus

Streptococcus pyogenes

Cytomegalovirus

Moraxella catarrhalis

Streptococcus viridans

Similar theme in September 2011 Exam

Quinsy usually occurs as a result of bacterial tonsillitis and the most common cause of bacterial
tonsillitis is streptococcal organisms.
Please rate this question:

Discuss and give feedback


Next question

Acute tonsillitis

 Characterised by pharyngitis, fever, malaise and lymphadenopathy.


 Over half of all cases are bacterial with Streptococcus pyogenes the most common organism
 The tonsils are typically oedematous and yellow or white pustules may be present
 Infectious mononucleosis may mimic the condition.
 Treatment with penicillin type antibiotics is indicated for bacterial tonsillitis.
 Bacterial tonsillitis may result in local abscess formation (quinsy)

Acute streptococcal tonsillitis


Image sourced from Wikipedia

Next question
Which of the following statements relating to osteomyelitis is false?

Is the result of haematogenous spread in most cases

Is due to Staphylococcus aureus in 50% cases

Should be treated by aggressive surgical debridement initially

Plain radiographs may be normal in the early stages

The presence of associated septic joint involvement will significantly alter management

It is managed medically in the first instance (with an antistaphylococcal antibiotic). This differs from
the situation in septic joints where early joint washout is mandatory.
Please rate this question:

Discuss and give feedback


Next question

Osteomyelitis

Infection of the bone

Causes

 S aureus and occasionally Enterobacter or Streptococcus species


 In sickle cell: Salmonella species

Clinical features

 Erythema
 Pain
 Fever

Investigation
 X-ray: lytic centre with a ring of sclerosis
 Bone biopsy and culture

Treatment

 Prolonged antibiotics
 Sequestra may need surgical removal

Next question
A 48 year old lady is admitted with crampy abdominal pain and diarrhoea. She has been unwell for
the past 12 hours. In the history she complains that her milk bottles have been pecked repeatedly by
birds, she otherwise has had no dietary changes. Which of the following is the most likely causative
organism?

Staphylococcus aureus

Campylobacter jejuni

Clostridium difficile

Norovirus

Clostridium botulinum

Birds are a recognised reservoir of campylobacter.


Please rate this question:

Discuss and give feedback


Next question

Bacterial Gastroenteritis

Causative organisms Features

Campylobacterjejuni  Most common cause of acute infective diarrhoea


 Spiral, gram negative rods
 Usually infects caecum and terminal ileum. Local
lymphadenopathy is common
 May mimic appendicitis as it has marked right iliac fossa pain
 Reactive arthritis is seen in 1-2% of cases

Shigella spp.  Members of the enterobacteriaceae


 Gram negative bacilli
 Clinically causes dysentery
 Shigella soneii is the commonest infective organism (mild illness)
 Usually self limiting, ciprofloxacin may be required if individual
Causative organisms Features

is in a high risk group

Salmonellaspp  Facultatively anaerobic, gram negative, enterobacteriaceae


 Infective dose varies according to subtype
 Salmonellosis: usually transmitted by infected meat (especially
poultry) and eggs

E. coli  Enteropathogenic
 Enteroinvasive: dysentery, large bowel necrosis/ulcers
 Enterotoxigenic: small intestine, travelers diarrhoea
 Enterohaemorrhagic: 0157, cause a haemorrhagic colitis,
haemolytic uraemic syndrome and thrombotic thrombocytopaenic
purpura

Yersinia  Gram negative, coccobacilli


enterocolitica  Typically produces a protracted terminal ileitis that may mimic
Crohns disease
 Differential diagnosis acute appendicitis
 May progress to septicaemia in susceptible individuals
 Usually sensitive to quinolone or tetracyclines

Vibrio cholera  Short, gram negative rods


 Transmitted by contaminated water, seafood
 Symptoms include sudden onset of effortless vomiting and profuse
watery diarrhoea
 Correction of fluid and electrolyte losses are the mainstay of
treatment
 Most cases will resolve, antibiotics are not generally indicated

Next question
A 50-year-old female with a history of rheumatoid presents with a suspected septic knee joint. A
diagnostic aspiration is performed and sent to microbiology. Which of the organisms below is most
likely to be responsible?

Staphylococcus aureus

Staphylococcus epidermidis

Escherichia coli

Neisseria gonorrhoeae

Streptococcus pneumoniae

Septic arthritis - most common organism: Staphylococcus aureus

Please rate this question:

Discuss and give feedback


Next question

Septic arthritis

Overview

 Most common organism overall is Staphylococcus aureus


 In young adults who are sexually active Neisseria gonorrhoeae should also be considered

Management

 Synovial fluid should be obtained before starting treatment


 Intravenous antibiotics which cover Gram-positive cocci are indicated. The BNF currently
recommends flucloxacillin or clindamycin if penicillin allergic
 Antibiotic treatment is normally be given for several weeks (BNF states 6-12 weeks)
 Needle aspiration should be used to decompress the joint
 Arthroscopic lavage may be required

Next question
A 22 year old man presents with crampy abdominal pain diarrhoea and bloating. He has just
returned from a holiday in Egypt. He had been swimming in the local pool a few days ago. He
reports that he is opening his bowels 5 times a day. The stool floats in the toilet water, but there is no
blood. What is the most likely cause?

Cryptosporidium

Salmonella sp

E.coli sp

Chronic pancreatitis

Giardia lamblia

Giardia causes fat malabsorption, therefore greasy stool can occur. It is resistant to chlorination,
hence risk of transfer in swimming pools.
Please rate this question:

Discuss and give feedback


Next question

Diarrhoea

World Health Organisation definitions


Diarrhoea: > 3 loose or watery stool per day
Acute diarrhoea < 14 days
Chronic diarrhoea > 14 days

Acute Diarrhoea

Gastroenteritis May be accompanied by abdominal pain or nausea/vomiting

Diverticulitis Classically causes left lower quadrant pain, diarrhoea and fever
Acute Diarrhoea

Antibiotic therapy More common with broad spectrum antibiotics


Clostridium difficile is also seen with antibiotic use

Constipation causing A history of alternating diarrhoea and constipation may be


overflow given
May lead to faecal incontinence in the elderly

Chronic
Diarrhoea

Irritable bowel Extremely common. The most consistent features are abdominal pain, bloating
syndrome and change in bowel habit. Patients may be divided into those with diarrhoea
predominant IBS and those with constipation predominant IBS.
Features such as lethargy, nausea, backache and bladder symptoms may also be
present

Ulcerative Bloody diarrhoea may be seen. Crampy abdominal pain and weight loss are also
colitis common. Faecal urgency and tenesmus may occur

Crohn's Crampy abdominal pains and diarrhoea. Bloody diarrhoea less common than in
disease ulcerative colitis. Other features include malabsorption, mouth ulcers perianal
disease and intestinal obstruction

Colorectal Symptoms depend on the site of the lesion but include diarrhoea, rectal
cancer bleeding, anaemia and constitutional symptoms e.g. Weight loss and anorexia

Coeliac disease  In children may present with failure to thrive, diarrhoea and abdominal
distension
 In adults lethargy, anaemia, diarrhoea and weight loss are seen. Other
autoimmune conditions may coexist

Other conditions associated with diarrhoea include:


 Thyrotoxicosis
 Laxative abuse
 Appendicitis with pelvic abscess or pelvic appendix
 Radiation enteritis

Diagnosis
Stool culture
Abdominal and digital rectal examination
Consider colonoscopy (radiological studies unhelpful)
Thyroid function tests, serum calcium, anti endomysial antibodies, glucose
Next question
A 54-year-old female is admitted one week following a cholecystectomy with profuse diarrhoea.
Apart from a minor intra-operative bile spillage incurred during removal of the gallbladder, the
procedure was uncomplicated. What is the most likely diagnosis?

Campylobacter infection

E. coli infection

Clostridium difficile infection

Salmonella infection

Pelvic abscess

Antibiotics are not routinely administered during an uncomplicated cholecystectomy. Indications for
administration of broad spectrum antibiotics include intraoperative bile spillage. Delayed pelvic
abscesses following bile spills are extremely rare since most surgeons will manage these intra-
operatively.
Please rate this question:

Discuss and give feedback


Next question

Clostridium difficile

Clostridium difficile is a Gram positive rod often encountered in hospital practice. In the UK it can be
found in 3% of normal adults and up to 66% of babies. It produces an exotoxin which causes
intestinal damage leading to a syndrome called pseudomembranous colitis.

Risk factors

 Broad spectrum antibiotics


 Use of PPI and H2 receptor antagonists
 Contacted with persons infected with c.difficile

Features

 Diarrhoea
 Abdominal pain
 A raised white blood cell count is characteristic
 If severe, toxic megacolon may develop

Diagnosis is made by detecting Clostridium difficile toxin (CDT) in the stool

Management

 First-line therapy is oral metronidazole for 10-14 days


 If severe, or not responding to metronidazole, then oral vancomycin may be used
 Patients who do not respond to vancomycin may respond to oral fidaxomicin
 Patients with severe and unremitting colitis should be considered for colectomy

Next question
Which of the following is not a feature of Campylobacter jejuni infection?

Infection may present in a similar manner to acute appendicitis

Pyrexia is unusual

They are gram negative organisms

Infection accounts for 26% case of Guillain-Barre syndrome

It is the commonest cause of infective diarrhoea arising from non viral causes

A prodromal period of fever and generalised malaise precedes abdominal pain (which may mimic
appendicitis) and diarrhoea.
Please rate this question:

Discuss and give feedback


Next question

Bacterial Gastroenteritis

Causative organisms Features

Campylobacterjejuni  Most common cause of acute infective diarrhoea


 Spiral, gram negative rods
 Usually infects caecum and terminal ileum. Local
lymphadenopathy is common
 May mimic appendicitis as it has marked right iliac fossa pain
 Reactive arthritis is seen in 1-2% of cases

Shigella spp.  Members of the enterobacteriaceae


 Gram negative bacilli
 Clinically causes dysentery
 Shigella soneii is the commonest infective organism (mild illness)
 Usually self limiting, ciprofloxacin may be required if individual
is in a high risk group
Causative organisms Features

Salmonellaspp  Facultatively anaerobic, gram negative, enterobacteriaceae


 Infective dose varies according to subtype
 Salmonellosis: usually transmitted by infected meat (especially
poultry) and eggs

E. coli  Enteropathogenic
 Enteroinvasive: dysentery, large bowel necrosis/ulcers
 Enterotoxigenic: small intestine, travelers diarrhoea
 Enterohaemorrhagic: 0157, cause a haemorrhagic colitis,
haemolytic uraemic syndrome and thrombotic thrombocytopaenic
purpura

Yersinia  Gram negative, coccobacilli


enterocolitica  Typically produces a protracted terminal ileitis that may mimic
Crohns disease
 Differential diagnosis acute appendicitis
 May progress to septicaemia in susceptible individuals
 Usually sensitive to quinolone or tetracyclines

Vibrio cholera  Short, gram negative rods


 Transmitted by contaminated water, seafood
 Symptoms include sudden onset of effortless vomiting and profuse
watery diarrhoea
 Correction of fluid and electrolyte losses are the mainstay of
treatment
 Most cases will resolve, antibiotics are not generally indicated

Next question
A 53-year-old woman is diagnosed with cellulitis surrounding her leg ulcer. A swab is taken and oral
flucloxacillin is started. The following result is obtained:

Skin swab: Group A streptococcus

How should the antibiotic therapy be changed?

No change

Add topical fusidic acid

Add clindamycin

Add penicillin

Add erythromycin

Penicillin is the antibiotic of choice for group A streptococcal infections. The BNF suggests stopping
flucloxacillin if streptococcal infection is confirmed in patients with cellulitis, due to the high
sensitivity. This should be balanced however with the variable absorption of phenoxymethylpenicillin.
Please rate this question:

Discuss and give feedback


Next question

Streptococci

Streptococci may be divided into alpha and beta haemolytic types

Alpha haemolytic streptococci

The most important alpha haemolytic streptococcus is Streptococcus pneumoniae (pneumococcus).


Pneumococcus is a common cause of pneumonia, meningitis and otitis media. Another clinical
example is Streptococcus viridans

Beta haemolytic streptococci

These can be subdivided into group A and B

Group A
 most important organism is Streptococcus pyogenes
 responsible for erysipelas, impetigo, cellulitis, type 2 necrotizing fasciitis and
pharyngitis/tonsillitis
 immunological reactions can cause rheumatic fever or post-streptococcal glomerulonephritis
 erythrogenic toxins cause scarlet fever

Group B

 Streptococcus agalactiae may lead to neonatal meningitis and septicaemia

Next question
Theme: Infectious disease

A. Clostridium difficile
B. Clostridium perfringens
C. Clostridium tetani
D. Streptococcus pyogenes
E. Steptococcus Bovis
F. Staphylococcus aureus
G. Staphylococcus epidermidis
H. Bacteroides fragilis
I. None of the above

Please select the most likely infective organism for the scenario given. Each option may be used
once, more than once or not at all.

36. A 23 year old man is readmitted following a difficult appendicectomy. His wound is
erythematous and, on incision, foul smelling pus is drained.

You answered Clostridium difficile

The correct answer is Bacteroides fragilis

Bacteroides is commonly present in severe peritoneal infections and as it is facultatively


anaerobic may be present in pus. It smells foul!

37. A 62 year old lady is unwell following a difficult acute cholecystectomy for acute
cholecystitis. Her gallbladder spilled stones intraoperatively and she has been on
ciprofloxacin intravenously for this for the past 4 days. She now has colicky abdominal
pain and profuse, foul smelling diarrhoea.

Clostridium difficile

C. difficile may complicate administration of broad spectrum antibiotics.

38. A 21 year old man is admitted with crampy abdominal pain and diarrhoea. He attended a
large wedding earlier in the day. Several other guests are also affected with the same
illness.

You answered Clostridium difficile

The correct answer is Clostridium perfringens

C. Perfringens is a common cause of food borne illness and its ability to form spores may
make it relatively resistant to cooking. The timing of onset would favor C. Perfringens
which typically evolves over several hours, rather than staphylococcus aureus poisening
which may occur sooner.

Please rate this question:

Discuss and give feedback


Next question

Surgical Microbiology

An extensive topic so an overview is given here. Organisms causing common surgical infections are
reasonable topics in the examination. However, microbiology is less rigorously tested than anatomy,
for example.

Common organisms

Staphylococcus aureus

 Facultative anaerobe
 Gram positive coccus
 Haemolysis on blood agar plates
 Catalase positive
 20% population are long term carriers
 Exo and entero toxin may result in toxic shock syndrome and gastroenteritis respectively
 Ideally treated with penicillin although many strains now resistant through beta Lactamase
production. In the UK less than 5% of isolates are sensitive to penicillin.
 Resistance to methicillin (and other antibiotics) is mediated by the mec operon , essentially
penicillin binding protein is altered and resistance to this class of antibiotics ensues
 Common cause of cutaneous infections and abscesses

Streptococcus pyogenes

 Gram positive, forms chain like colonies, Lancefield Group A Streptococcus


 Produces beta haemolysis on blood agar plates
 Rarely part of normal skin microflora
 Catalase negative
 Releases a number of proteins/ virulence factors into host including hyaluronidase,
streptokinase which allow rapid tissue destruction
 Releases superantigens such as pyogenic exotoxin A which results in scarlet fever
 Remains sensitive to penicillin, macrolides may be used as an alternative.

Escherichia coli
 Gram negative rod
 Facultative anaerobe, non sporing
 Wide range of subtypes and some are normal gut commensals
 Some subtypes such as 0157 may produce lethal toxins resulting in haemolytic-uraemic
syndrome
 Enterotoxigenic E-Coli produces an enterotoxin (ST enterotoxin) that results in large
volume fluid secretion into the gut lumen (Via cAMP activation)
 Enteropathogenic E-Coli binds to intestinal cells and cause structural damage, this coupled
with a moderate (or in case of enteroinvasive E-Coli significant) invasive component
produces enteritis and large volume diarrhoea together with fever.
 They are resistant to many antibiotics used to treat gram positive infections and acquire
resistance rapidly and are recognised as producing beta lactamases

Campylobacter jejuni

 Curved, gram negative, non sporulating bacteria


 One of the commonest causes of diarrhoea worldwide
 Produces enteritis which is often diffuse and blood may be passed
 Remains a differential for right iliac fossa pain with diarrhoea
 Self limiting infection so antibiotics are not usually advised. However, the quinolones are
often rapidly effective.

Helicobacter pylori

 Gram negative, helix shaped rod, microaerophillic


 Produces hydrogenase that can derive energy from hydrogen released by intestinal bacteria
 Flagellated and mobile
 Those carrying the cag A gene may cause ulcers
 It secretes urease that breaks down gastric urea> Carbon dioxide and ammonia>
ammonium>bicarbonate (simplified!) The bicarbonate can neutralise the gastric acid.
 Usually colonises the gastric antrum and irritates resulting in increased gastrin release and
higher levels of gastric acid. These patients will develop duodenal ulcers. In those with more
diffuse H-Pylori infection gastric acid levels are lower and ulcers develop by local tissue
damage from H-Pylori- these patients get gastric ulcers.
 Diagnosis may be made by serology (approx. 75% sensitive). Biopsy urease test during
endoscopy probably the most sensitive.
 In patients who are colonised 10-20% risk of peptic ulcer, 1-2% risk gastric cancer, <1% risk
MALT lymphoma.

Next question
Which of the following statements related to necrotising fasciitis is false?

Mainly polymicrobial

A feature may include 'dirty dishwater fluid' in the wound

The presence of crepitus is needed to make the diagnosis

Further surgery is mandatory 24-48h after initial surgery to review extension of infection

The muscles are relatively spared

Never attempt primary closure after the initial debridement of necrotising fasciitis.

Crepitus may be present in only 35% of cases, therefore its absence should not exclude a diagnosis
of necrotising fasciitis.
Please rate this question:

Discuss and give feedback


Next question

Meleney's Gangrene and Necrotising Fasciitis

Necrotising fasciitis

 Advancing soft tissue infection associated with fascial necrosis


 Uncommon, but can be fatal
 In many cases there is underlying background immunosuppression e.g. Diabetes
 Caused by polymicrobial flora (aerobic and anaerobic) and MRSA is seen increasingly in
cases of necrotising fasciitis
 Streptococcus is the commonest organism in isolated pathogen infection (15%)

Meleneys gangrene

 Meleneys is a similar principle but the infection is more superficially sited than necrotising
fasciitis and often confined to the trunk
Fournier gangrene

 Necrotising fasciitis affecting the perineum


 Polymicrobial with E-coli and Bacteroides acting in synergy

Clinical features
Fever
Pain
Cellulitis
Oedema
Induration
Numbness

Late findings

Purple/black skin discolouration


Blistering
Haemorrhagic bullae
Crepitus
Dirty Dishwater fluid discharge
Septic shock

A typical case of gas gangrene presenting late demonstrating some of the features described above

Image sourced from Wikipedia

Diagnosis is mainly clinical

Management
 Radical surgical debridement forms the cornerstone of management
 Sterile dressing is used to dress the wound
 Reconstructive surgery is considered once the infection is completely treated

Reference

Hasham S, Matteucci P, Stanley PR, Hart NB. Necrotising fasciitis. BMJ 2005;330:830-833.
Next question
A surgical trainee is incising a groin "abscess" in an intravenous drug abuser. Unfortunately the
"abscess" is a false aneurysm and torrential bleeding ensues. In the panic of the situation the doctor
then stabs himself in the finger. It transpires that the patient is a Hepatitis B carrier and the doctor is
not immunised! What type of virus is Hepatitis B?

Double stranded DNA virus

Single stranded DNA virus

Double stranded RNA virus

Single stranded RNA virus

Retrovirus

Please rate this question:

Discuss and give feedback


Next question

Hepatitis B

Hepatitis B is a double-stranded DNA virus and is spread through exposure to infected blood or body
fluids, including vertical transmission from mother to child. The incubation period is 6-20 weeks.

Immunisation against hepatitis B

 Contains HBsAg absorbed onto aluminium hydroxide adjuvant and is prepared from yeast
cells using recombinant DNA technology
 Most schedules give 3 doses of the vaccine with a recommendation for a one-off booster 5
years following the initial primary vaccination
 At risk groups who should be vaccinated include: healthcare workers, intravenous drug
users, sex workers, close family contacts of an individual with hepatitis B, individuals
receiving blood transfusions regularly, chronic kidney disease patients who may soon require
renal replacement therapy, prisoners, chronic liver disease patients
 Around 10-15% of adults fail to respond or respond poorly to 3 doses of the vaccine. Risk
factors include age over 40 years, obesity, smoking, alcohol excess and immunosuppression
 Testing for anti-HBs is only recommended for those at risk of occupational exposure (i.e.
Healthcare workers) and patients with chronic kidney disease. In these patients anti-HBs
levels should be checked 1-4 months after primary immunisation
 The table below shows how to interpret anti-HBs levels:
Anti-HBs Response
level
(mIU/ml)

> 100 Indicates adequate response, no further testing required. Should still receive
booster at 5 years

10 - 100 Suboptimal response - one additional vaccine dose should be given. If


immunocompetent no further testing is required

< 10 Non-responder. Test for current or past infection. Give further vaccine course
(i.e. 3 doses again) with testing following. If still fails to respond then HBIG
would be required for protection if exposed to the virus

Complications of hepatitis B infection

 Chronic hepatitis (5-10%)


 Fulminant liver failure (1%)
 Hepatocellular carcinoma
 Glomerulonephritis
 Polyarteritis nodosa
 Cryoglobulinaemia

Management of hepatitis B

 Pegylated interferon-alpha used to be the only treatment available. It reduces viral replication
in up to 30% of chronic carriers. A better response is predicted by being female, < 50 years
old, low HBV DNA levels, non-Asian, HIV negative, high degree of inflammation on liver
biopsy
 However, due to the side-effects of pegylated interferon it is now used less commonly in
clinical practice. Oral antiviral medication is increasingly used with an aim to suppress viral
replication (not in dissimilar way to treating HIV patients)
 Examples include lamivudine, tenofovir and entecavir

Next question
Which virus is implicated in cervical carcinoma?

Human papilloma virus 16

Human papilloma virus 2

Human herpes virus 8

Human herpes virus 2

Epstein-Barr virus

Theme from September 2013


Please rate this question:

Discuss and give feedback


Next question

Oncoviruses

 Viruses which cause cancer


 These may be detected on blood test and prevented by vaccine

These are the main types of oncoviruses and their diseases:

Oncovirus Cancer

Epstein-Barr virus Burkitt's lymphoma


Hodgkin's lymphoma
Post transplant lymphoma
Nasopharyngeal carcinoma

Human papillomavirus 16/18 Cervical cancer


Anal cancer
Penile cancer
Vulval cancer
Oropharyneal cancer

Human herpes virus 8 Kaposi's sarcoma

Hepatitis B virus Hepatocellular carcinoma

Hepatitis C virus Hepatocellular carcinoma

Human T-lymphotropic virus 1 Tropical spastic paraparesis


Adult T cell leukaemia

Next question
A young woman is admitted to hospital with E-coli 0157 after visiting Germany during an outbreak.
Which of the following is not true of the condition?

It may be complicated by micro-angiopathic haemolytic anaemia.

Adults typically develop haemolytic uraemic syndome.

It is most commonly transmitted by consumption of contaminated food.

Plasmids typically confer antibiotic resistance.

E-Coli is a gram negative organism.

Children typically develop this complication.


Please rate this question:

Discuss and give feedback


Next question

Bacterial Gastroenteritis

Causative organisms Features

Campylobacterjejuni  Most common cause of acute infective diarrhoea


 Spiral, gram negative rods
 Usually infects caecum and terminal ileum. Local
lymphadenopathy is common
 May mimic appendicitis as it has marked right iliac fossa pain
 Reactive arthritis is seen in 1-2% of cases

Shigella spp.  Members of the enterobacteriaceae


 Gram negative bacilli
 Clinically causes dysentery
 Shigella soneii is the commonest infective organism (mild illness)
 Usually self limiting, ciprofloxacin may be required if individual
is in a high risk group
Causative organisms Features

Salmonellaspp  Facultatively anaerobic, gram negative, enterobacteriaceae


 Infective dose varies according to subtype
 Salmonellosis: usually transmitted by infected meat (especially
poultry) and eggs

E. coli  Enteropathogenic
 Enteroinvasive: dysentery, large bowel necrosis/ulcers
 Enterotoxigenic: small intestine, travelers diarrhoea
 Enterohaemorrhagic: 0157, cause a haemorrhagic colitis,
haemolytic uraemic syndrome and thrombotic thrombocytopaenic
purpura

Yersinia  Gram negative, coccobacilli


enterocolitica  Typically produces a protracted terminal ileitis that may mimic
Crohns disease
 Differential diagnosis acute appendicitis
 May progress to septicaemia in susceptible individuals
 Usually sensitive to quinolone or tetracyclines

Vibrio cholera  Short, gram negative rods


 Transmitted by contaminated water, seafood
 Symptoms include sudden onset of effortless vomiting and profuse
watery diarrhoea
 Correction of fluid and electrolyte losses are the mainstay of
treatment
 Most cases will resolve, antibiotics are not generally indicated

Next question
Theme: Infections

A. Staphylococcus aureus
B. Streptococcus bovis
C. Clostridium perfringens
D. Clostridium difficile
E. Clostridium tetani
F. Klebsiella
G. Streptococcus pyogenes
H. Yersinia enterocolitica
I. None of the above

Please select the most likely pathogen to account for the scenario given. Each option may be used
once, more than once or not at all.

43. A 72 year old man with peripheral vascular disease develops a gangrenous toe. This
becomes infected and there is evidence of infection in the surrounding tissues. On clinical
palpation there is crepitus present within the tissues.

You answered Staphylococcus aureus

The correct answer is Clostridium perfringens

Theme from January 2013 Exam


Clostridium perfringens is the most likely pathogen to be associated with gangrene.

44. A 22 year old lady is breastfeeding her first child. One week post partum she presents with
a tender indurated mass in the right breast.

Staphylococcus aureus

Staphylococcus aureus is the commonest cause of lactational mastitis.

45. A 45 year old man is recovering in hospital following a total hip replacement. He develops
a profuse and watery diarrhoea. Several other patients have been suffering from similar
symptoms.

You answered Staphylococcus aureus

The correct answer is Clostridium difficile

Clostridium difficile can spread rapidly on surgical wards. The use of broad spectrum
prophylactic antibiotics during arthroplasty surgery can increase the risk.
Please rate this question:

Discuss and give feedback


Next question

Surgical Microbiology

An extensive topic so an overview is given here. Organisms causing common surgical infections are
reasonable topics in the examination. However, microbiology is less rigorously tested than anatomy,
for example.

Common organisms

Staphylococcus aureus

 Facultative anaerobe
 Gram positive coccus
 Haemolysis on blood agar plates
 Catalase positive
 20% population are long term carriers
 Exo and entero toxin may result in toxic shock syndrome and gastroenteritis respectively
 Ideally treated with penicillin although many strains now resistant through beta Lactamase
production. In the UK less than 5% of isolates are sensitive to penicillin.
 Resistance to methicillin (and other antibiotics) is mediated by the mec operon , essentially
penicillin binding protein is altered and resistance to this class of antibiotics ensues
 Common cause of cutaneous infections and abscesses

Streptococcus pyogenes

 Gram positive, forms chain like colonies, Lancefield Group A Streptococcus


 Produces beta haemolysis on blood agar plates
 Rarely part of normal skin microflora
 Catalase negative
 Releases a number of proteins/ virulence factors into host including hyaluronidase,
streptokinase which allow rapid tissue destruction
 Releases superantigens such as pyogenic exotoxin A which results in scarlet fever
 Remains sensitive to penicillin, macrolides may be used as an alternative.

Escherichia coli

 Gram negative rod


 Facultative anaerobe, non sporing
 Wide range of subtypes and some are normal gut commensals
 Some subtypes such as 0157 may produce lethal toxins resulting in haemolytic-uraemic
syndrome
 Enterotoxigenic E-Coli produces an enterotoxin (ST enterotoxin) that results in large
volume fluid secretion into the gut lumen (Via cAMP activation)
 Enteropathogenic E-Coli binds to intestinal cells and cause structural damage, this coupled
with a moderate (or in case of enteroinvasive E-Coli significant) invasive component
produces enteritis and large volume diarrhoea together with fever.
 They are resistant to many antibiotics used to treat gram positive infections and acquire
resistance rapidly and are recognised as producing beta lactamases

Campylobacter jejuni

 Curved, gram negative, non sporulating bacteria


 One of the commonest causes of diarrhoea worldwide
 Produces enteritis which is often diffuse and blood may be passed
 Remains a differential for right iliac fossa pain with diarrhoea
 Self limiting infection so antibiotics are not usually advised. However, the quinolones are
often rapidly effective.

Helicobacter pylori

 Gram negative, helix shaped rod, microaerophillic


 Produces hydrogenase that can derive energy from hydrogen released by intestinal bacteria
 Flagellated and mobile
 Those carrying the cag A gene may cause ulcers
 It secretes urease that breaks down gastric urea> Carbon dioxide and ammonia>
ammonium>bicarbonate (simplified!) The bicarbonate can neutralise the gastric acid.
 Usually colonises the gastric antrum and irritates resulting in increased gastrin release and
higher levels of gastric acid. These patients will develop duodenal ulcers. In those with more
diffuse H-Pylori infection gastric acid levels are lower and ulcers develop by local tissue
damage from H-Pylori- these patients get gastric ulcers.
 Diagnosis may be made by serology (approx. 75% sensitive). Biopsy urease test during
endoscopy probably the most sensitive.
 In patients who are colonised 10-20% risk of peptic ulcer, 1-2% risk gastric cancer, <1% risk
MALT lymphoma.

Next question
A 27-year-old male presents to urology for investigation of pyelonephritis. He reports malaise,
pyrexia, lymphadenopathy and a maculopapular rash. The Monospot test is negative. Given a
history of high-risk sexual behaviour you are asked to exclude a HIV seroconversion illness. What is
the most appropriate investigation?

Antibodies to HIV-2

gp120 polymerase chain reaction

p24 antigen test

CCR5 polymerase chain reaction

Antibodies to HIV-1

Please rate this question:

Discuss and give feedback


Next question

HIV testing

HIV seroconversion is symptomatic in 60-80% of patients and typically presents as a glandular fever
type illness. Increased symptomatic severity is associated with poorer long term prognosis. It
typically occurs 3-12 weeks after infection

Features

 sore throat
 lymphadenopathy
 malaise, myalgia, arthralgia
 diarrhoea
 maculopapular rash
 mouth ulcers
 rarely meningoencephalitis

Diagnosis

 antibodies to HIV may not be present


 HIV PCR and p24 antigen tests can confirm diagnosis

HIV antibody test

 most common and accurate test


 usually consists of both a screening ELISA (Enzyme Linked Immuno-Sorbent Assay) test
and a confirmatory Western Blot Assay
 most people develop antibodies to HIV at 4-6 weeks but 99% do by 3 months

p24 antigen test

 usually positive from about 1 week to 3 - 4 weeks after infection with HIV
 sometimes used as an additional screening test in blood banks

Next question
Which statement relating to actinomycosis is false?

They are gram positive bacilli

They are strict aerobes

It may be a cause of chronic multiple abscesses

Abdominal cases may develop in the appendix

Open biopsy of the lesions is the best diagnostic test

They are facultative anaerobes and may be difficult to culture. Direct visualisation of organisms and
sulphur granules from lesions themselves is the easiest way to make a diagnosis. It remains a
differential of conditions such as hydradenitis supprativa, particularly if it is occurring in odd locations
and with deeper abscesses than usual.
Please rate this question:

Discuss and give feedback

Surgical Microbiology

An extensive topic so an overview is given here. Organisms causing common surgical infections are
reasonable topics in the examination. However, microbiology is less rigorously tested than anatomy,
for example.

Common organisms

Staphylococcus aureus

 Facultative anaerobe
 Gram positive coccus
 Haemolysis on blood agar plates
 Catalase positive
 20% population are long term carriers
 Exo and entero toxin may result in toxic shock syndrome and gastroenteritis respectively
 Ideally treated with penicillin although many strains now resistant through beta Lactamase
production. In the UK less than 5% of isolates are sensitive to penicillin.
 Resistance to methicillin (and other antibiotics) is mediated by the mec operon , essentially
penicillin binding protein is altered and resistance to this class of antibiotics ensues
 Common cause of cutaneous infections and abscesses
Streptococcus pyogenes

 Gram positive, forms chain like colonies, Lancefield Group A Streptococcus


 Produces beta haemolysis on blood agar plates
 Rarely part of normal skin microflora
 Catalase negative
 Releases a number of proteins/ virulence factors into host including hyaluronidase,
streptokinase which allow rapid tissue destruction
 Releases superantigens such as pyogenic exotoxin A which results in scarlet fever
 Remains sensitive to penicillin, macrolides may be used as an alternative.

Escherichia coli

 Gram negative rod


 Facultative anaerobe, non sporing
 Wide range of subtypes and some are normal gut commensals
 Some subtypes such as 0157 may produce lethal toxins resulting in haemolytic-uraemic
syndrome
 Enterotoxigenic E-Coli produces an enterotoxin (ST enterotoxin) that results in large
volume fluid secretion into the gut lumen (Via cAMP activation)
 Enteropathogenic E-Coli binds to intestinal cells and cause structural damage, this coupled
with a moderate (or in case of enteroinvasive E-Coli significant) invasive component
produces enteritis and large volume diarrhoea together with fever.
 They are resistant to many antibiotics used to treat gram positive infections and acquire
resistance rapidly and are recognised as producing beta lactamases

Campylobacter jejuni

 Curved, gram negative, non sporulating bacteria


 One of the commonest causes of diarrhoea worldwide
 Produces enteritis which is often diffuse and blood may be passed
 Remains a differential for right iliac fossa pain with diarrhoea
 Self limiting infection so antibiotics are not usually advised. However, the quinolones are
often rapidly effective.

Helicobacter pylori

 Gram negative, helix shaped rod, microaerophillic


 Produces hydrogenase that can derive energy from hydrogen released by intestinal bacteria
 Flagellated and mobile
 Those carrying the cag A gene may cause ulcers
 It secretes urease that breaks down gastric urea> Carbon dioxide and ammonia>
ammonium>bicarbonate (simplified!) The bicarbonate can neutralise the gastric acid.
 Usually colonises the gastric antrum and irritates resulting in increased gastrin release and
higher levels of gastric acid. These patients will develop duodenal ulcers. In those with more
diffuse H-Pylori infection gastric acid levels are lower and ulcers develop by local tissue
damage from H-Pylori- these patients get gastric ulcers.
 Diagnosis may be made by serology (approx. 75% sensitive). Biopsy urease test during
endoscopy probably the most sensitive.
 In patients who are colonised 10-20% risk of peptic ulcer, 1-2% risk gastric cancer, <1% risk
MALT lymphoma.
What is the earliest complication that can occur following construction of an ileostomy?

Prolapse

Retraction

Necrosis

Parastomal hernia

Dermatitis

Construction of a stoma may be complicated by several factors. Necrosis may occur because of
technical errors in mesenteric division, excessive tension or failure to construct a fascial defect of
adequate size to permit safe passage of the mesentery and the bowel.
Please rate this question:

Discuss and give feedback


Next question

Ileostomy

Ileostomies are generally fashioned in the right iliac fossa in a triangle between the anterior superior
iliac spine, symphysis pubis and umbilicus. They should lie one-third of the distance between the
umbilicus and anterior superior iliac spine. A 2cm skin incision is made and dissection continued
through the rectus muscle. A cruciate incision should be made, and generally dilated to admit two
fingers. The ileum is brought through the incisions and should generally be spouted to a final length
of 2.5cm. Ileostomies that are too short may cause problems with appliance fixation and those which
are too long may cause problems with tension and subsequent ulceration or prolapse.

Complications following ileostomy construction include dermatitis (most common), bowel obstruction
(usually adhesional) and prolapse.

Ileostomy output is roughly in the range of 5-10ml/Kg/ 24 hours. Output in excess of 20ml/Kg/24
hours usually requires supplementary intravenous fluids. Excessive fluid losses are generally
managed by administration of oral loperamide (up to 4mg QDS) to try and slow the output. Foods
containing gelatine may also thicken output. Early high output is not uncommon and most patients
(50%) will respond to conservative management.
Next question
Theme: Proctology

A. Fissure in ano

B. Fistula in ano

C. Rectal prolapse

D. Juvenile polyps

E. Rectal adenoma

F. Intersphincteric abscess

G. Haemorrhoids

Please select the most likely underlying cause for the presentations described. Each option may be
used once, more than once or not at all.

2. A 21 year old female presents with a 24 hour history of increasingly severe ano-rectal pain. On
examination she is febrile and the skin surrounding the anus looks normal. She did not tolerate an
attempted digital rectal examination.

You answered Fissure in ano

The correct answer is Intersphincteric abscess

Theme from September 2012 Exam


The presence of fever and severe pain makes an abscess more likely than a fissure. Although
fissures may be painful they do not, in themselves, cause fever. The usual management for this
condition is examination of the ano-rectum under general anaesthesia and drainage of the sepsis.

3. A 21 year old male presents with a 4 week history of frank, bright red, rectal bleeding. This typically
occurs post defecation into the toilet pan. He has a long standing history of constipation and a
previous fissure in ano. On examination the skin surrounding the anus is normal and digital rectal
examination is normal.

You answered Fissure in ano


The correct answer is Haemorrhoids

Haemorrhoids are a common cause of bright red rectal bleeding. The bleeding is typically painless.
A history of constipation is usual and may have been previously associated with a fissure (though
this is less common). Haemorrhoids are not always associated with external features and digital
rectal examination is usually unremarkable.

4. A 21 year old lady presents with a 6 month history of an offensive discharge from the anus. She is
otherwise well, but is increasingly annoyed at the need to wear pads. On examination she has a
small epithelial defect in the 5 o'clock position, approximately 3cm from the anal verge.

You answered Fissure in ano

The correct answer is Fistula in ano

Fistulas usually occur following previous ano-rectal sepsis. The discharge may be foul smelling and
troublesome. Patients should be listed for examination under anaesthesia. Fistulas which are low
and have little or no sphincter involvement are usually laid open.

Please rate this question:

Discuss and give feedback

Next question

Ano rectal disease

Location: 3, 7, 11 o'clock position


Haemorrhoids Internal or external
Treatment: Conservative, Rubber band ligation, Haemorrhoidectomy

Fissure in ano Location: midline 6 (posterior midline 90%) and 12 o'clock position. Distal to the
dentate line
Chronic fissure > 6/52: triad: Ulcer, sentinel pile, enlarged anal papillae

Proctitis Causes: Crohn's, ulcerative colitis, Clostridium difficile

Ano rectal E.coli, staph aureus


abscess Positions: Perianal, Ischiorectal, Pelvirectal, Intersphincteric

Anal fistula Usually due to previous ano-rectal abscess


Intersphincteric, transsphincteric, suprasphincteric, and extrasphincteric. Goodsalls
rule determines location

Rectal prolapse Associated with childbirth and rectal intussceception. May be internal or external

Pruritus ani Systemic and local causes

Anal neoplasm Squamous cell carcinoma commonest unlike adenocarcinoma in rectum

Solitary rectal Associated with chronic straining and constipation. Histology shows mucosal
ulcer thickening, lamina propria replaced with collagen and smooth muscle (fibromuscular
obliteration)

Rectal prolapse

 Common especially in multiparous women.


 May be internal or external.
 Internal rectal prolapse can present insidiously.
 External prolapse can ulcerate and in long term impair continence.
 Diagnostic work up includes colonoscopy, defecating proctogram, ano rectal manometry
studies and if doubt exists an examination under anaesthesia.

Treatments for prolapse

 In the acute setting reduce it (covering it with sugar may reduce swelling.
 Delormes procedure which excises mucosa and plicates the rectum (high recurrence rates)
may be used for external prolapse.
 Altmeirs procedure which resects the colon via the perineal route has lower recurrence rates
but carries the risk of anastamotic leak.
 Rectopexy is an abdominal procedure in which the rectum is elevated and usually supported
at the level of the sacral promontory. Post operative constipation may be reduced by limiting
the dissection to the anterior plane (laparoscopic ventral mesh rectopexy).

Pruritus ani

 Extremely common.
 Check not secondary to altered bowel habits (e.g. Diarrhoea)
 Associated with underlying diseases such as haemorrhoids.
 Examine to look for causes such as worms.
 Proctosigmoidoscopy to identify associated haemorrhoids and exclude cancer.
 Treatment is largely supportive and patients should avoid using perfumed products around
the area.

Fissure in ano

 Typically painful PR bleeding (bright red).


 Nearly always in the posterior midline.
 Usually solitary.

Treatment

 Stool softeners.
 Topical diltiazem (or GTN).
 If topical treatments fail then botulinum toxin should be injected.
 If botulinum toxin fails then males should probably undergo lateral internal sphincterotomy.
 Females who do not respond to botulinum toxin should undergo ano rectal manometry
studies and endo anal USS prior to being offered surgery such as sphincterotomy.

Next question
Theme: Management of ano-rectal disease

A. Excision and primary closure

B. Incision and drainage

C. Topical steroids

D. Topical diltiazem

E. Steroid injections

F. Haemorroidectomy

G. Manual anal dilation

H. Injection with 88% aqueous phenol

I. Sphincterotomy

Please select the most appropriate management for the situation described. Each option may be
used once, more than once or not at all.

5. A 19 year old female presents with severe anal pain and bleeding which typically occurs post
defecation. On examination she has a large posteriorly sited fissure in ano.

You answered Excision and primary closure

The correct answer is Topical diltiazem

Theme from January 2013 Exam


Theme from April 2014 Exam
Initial therapy should be with pharmacological agents to relax the sphincter and facilitate healing.
This is particularly true in females presenting for the first time.

6. A 43 year old male has been troubled with symptoms of post defecation bleeding for many years.
On examination he has large prolapsed haemorroids, colonoscopy shows no other disease.

You answered Excision and primary closure


The correct answer is Haemorroidectomy

Prolapsed haemorroids are best managed surgically if symptomatic. Note that phenol injections
are usually only used for minor internal haemorroids. Where used low concentration phenol in oil
is used, the phenolic solution above is used to ablate the nail bed in toe nail surgery!

7. A 20 year old man presents with a 24 hour history of anal pain. On examination he has a peri anal
abscess.

You answered Excision and primary closure

The correct answer is Incision and drainage

Abscesses require incision and drainage as a first line treatment.

Please rate this question:

Discuss and give feedback

Next question

Benign proctology

Condition Features Treatment

Fissure in ano Painful, bright red rectal bleeding Stool softeners, topical diltiazem or GTN,
botulinum toxin, Sphincterotomy

Haemorroids Painless, bright red rectal bleeding Stool softeners, avoid straining, surgery (see
occurs following defecation and bleeds below)
onto the toilet paper and into the toilet
pan
Fistula in ano May initially present with an abscess and Lay open if low, no sphincter involvement or
then persisting discharge onto the IBD, if complex, high or IBD insert seton and
perineum, separate from the anus consider other options (see below)

Peri anal Peri anal swelling and surrounding Incision and drainage, leave the cavity open
abscess erythema to heal by secondary intention

Pruritus ani Peri anal itching, occasional mild Avoid scented products, use wet wipes rather
bleeding (if severe skin damage) than tissue, avoidance of scratching, ensure
no underlying faecal incontinence

Overview of surgical therapies


Haemorroidal disease
The treatment of haemorroids is usually conservative. Acutely thrombosed haemorroids may be
extremely painful. Treatment of this acute condition is usually conservative and consists of stool
softeners, ice compressions and topical GTN or diltiazem to reduce sphincter spasm. Most cases
managed with this approach will settle over the next 5-7 days. After this period there may be residual
skin tags that merit surgical excision or indeed residual haemorroidal disease that may necessitate
haemorroidectomy.
Patients with more chronic symptoms are managed according to the stage of their disease, small
mild internal haemorroids causing little symptoms are best managed conservatively. More marked
symptoms of bleeding and occasional prolapse, where the haemorroidal complex is largely internal
may benefit from stapled haemorroidopexy. This procedure excises rectal tissue above the dentate
line and disrupts the haemorroidal blood supply. At the same time the excisional component of the
procedure means that the haemorroids are less prone to prolapse. Adverse effects of this procedure
include urgency, which can affect up to 40% of patients (but settles over 6-12 months) and
recurrence. The procedure does not address skin tags and therefore this procedure is unsuitable if
this is the dominant symptom.
Large haemorroids with a substantial external component may be best managed with a Milligan
Morgan style conventional haemorroidectomy. In this procedure three haemorroidal cushions are
excised, together with their vascular pedicle. Excision of excessive volumes of tissue may result in
anal stenosis. The procedure is quite painful and most surgeons prescribe metronidazole post
operatively as it decreases post operative pain.

Fissure in ano
Probably the most efficient and definitive treatment for fissure in ano is lateral internal
sphincterotomy. The treatment is permanent and nearly all patients will recover. Up to 30% will
develop incontinence to flatus. There are justifiable concerns about using this procedure in females
as pregnancy and pelvic floor damage together with a sphincterotomy may result in faecal
incontinence. The usual first line therapy is relaxation of the internal sphincter with either GTN or
diltiazem (the latter being better tolerated) applied topically for 6 weeks. Treatment failures with
topical therapy will usually go on to have treatment with botulinum toxin. This leads to more
permanent changes in the sphincter and this may facilitate healing.
Typical fissures usually present in the posterior midline, multiple or unusually located fissures should
prompt a search for an underlying cause such as inflammatory bowel disease or internal prolapse.
Refractory cases where the above treatments have failed may be considered for advancement
flaps.

Fistula in ano
The most effective treatment for fistula is laying it open (fistulotomy). When the fistula is below the
sphincter and uncomplicated, this is a reasonable option. Sphincter involvement and complex
underlying disease should be assessed both surgically and ideally with imaging (either MRI or
endoanal USS). Surgery is then usually staged, in the first instance a draining seton suture may be
inserted. This avoids the development of recurrent sepsis and may allow resolution. In patients with
Crohns disease the seton should be left in situ long term and the patient managed medically, as in
these cases attempts at complex surgical repair nearly always fail. Fistulas not associated with IBD
may be managed by advancement flaps, instillation of plugs and glue is generally unsuccessful. A
newer technique of ligation of intersphincteric tract (LIFT procedure) is reported to have good results
in selected centres.

Next question
Theme: Management of colonic polypoidal lesions

A. Reassure and discharge

B. Pan proctocolectomy

C. Hot biopsy

D. Snare polypectomy

E. Segmental colonic resection

F. Repeat endoscopy at 3 years

G. Repeat endoscopy at 1 year

H. Repeat endoscopy at 5 years

Please select the most appropriate management for the scenario given. Each option may be used
once, more than once or not at all.

8. A 43 year old man is investigated for altered bowel habit. At colonoscopy he is found to have a
2cm polyp on a long stalk in the proximal sigmoid colon. The rest of the colonoscopy is normal. It
bears no macroscopic features of malignancy.

You answered Reassure and discharge

The correct answer is Snare polypectomy

Polyps on long stalks are best managed by snare excision. It is important to retrieve the polyp for
histology.

9. A 60 year old lady is investigated for abdominal pain. A polyp is identified at the proximal
descending colon, three small polyps are also noted in the sigmoid colon. The largest lesion is
removed by snare polypectomy and the pathology report states that this polyp is a low grade
dysplastic adenoma measuring 3cm in diameter. The remaining lesions are ablated using
diathermy.

You answered Reassure and discharge


The correct answer is Repeat endoscopy at 1 year

She is at high risk of malignancy and should be closely followed up. Fulguration of polyps without
histology is unhelpful.

10. A 73 year old lady is investigated for anaemia. At colonoscopy she is found to have a flat broad
based lesion in the caecum. This is biopsied and the histology report states that these have
diagnostic features of an adenoma with high grade dysplasia.

You answered Reassure and discharge

The correct answer is Segmental colonic resection

Management of right sided colonic polyps such as this is controversial. There is a high liklihood
that this lesion will harbor an focus on invasive malignancy (which should generally be managed
with resection). This is further suggested by the background history of anaemia. It is for this
reason that resection over right sided colonic endoscopic mucosal resection is chosen.

Polypectomy of flat broad lesions in the right colon is difficult and where concern arises a right
hemicolectomy is probably the safest option.

Please rate this question:

Discuss and give feedback

Next question

Colonic polyps

Colonic Polyps
May occur in isolation, or greater numbers as part of the polyposis syndromes. In FAP greater than
100 polyps are typically present. The risk of malignancy in association with adenomas is related to
size, and is the order of 10% in a 1cm adenoma. Isolated adenomas seldom give risk of symptoms
(unless large and distal). Distally sited villous lesions may produce mucous and if very large,
electrolyte disturbances may occur.
Follow up of colonic polyps

Group Features Action

Low risk 1 or 2 adenomas less than 1cm No follow up or re-colonoscopy


at 5 years

Moderate 3 or 4 small adenomas or 1 adenoma greater than 1cm Re-scope at 3 years


risk

High risk More than 5 small adenomas or more than 3 with 1 of Re scope at 1 year
them greater than 1cm

From Atkins and Saunders Gut 2002 51 (suppl V:V6-V9). It is important to stratify patients
appropriately and ensure that a complete colonoscopy with good views was performed.

Segmental resection or complete colectomy should be considered when:

1. Incomplete excision of malignant polyp


2. Malignant sessile polyp
3. Malignant pedunculated polyp with submucosal invasion
4. Polyps with poorly differentiated carcinoma
5. Familial polyposis coli
-Screening from teenager up to 40 years by 2 yearly sigmoidoscopy/colonoscopy
-Panproctocolectomy and Ileostomy or Restorative Panproctocolectomy.

Rectal polypoidal lesions may be amenable to trans anal endoscopic microsurgery.

References
Cairns S et al. Guidelines for colorectal cancer screening and surveillance in moderate and high risk
groups (update from 2002). Gut 2010;59:666-690.

Next question
Theme: Colonic resections

A. End ileostomy

B. Loop ileostomy

C. Ileo anal pouch

D. Loop colostomy

E. Pan proctocelectomy

F. Extended right hemicolectomy

G. Right hemicolectomy

H. Anterior resection

I. Anterior resection with covering loop ileostomy

Please select the most appropriate procedure from the list, each option may be used once, more
than once or not at all.

11. A 75 year old man requires resection of an obstructing carcinoma of the splenic flexure.

You answered End ileostomy

The correct answer is Extended right hemicolectomy

Carcinoma of the splenic flexure requires extended right hemicolectomy. Or a left


hemicolectomy. The ileocolic anastomosis has a lower leak rate, particularly when the bowel is
obstructed.

12. A patient presenting with a large bowel obstruction from a low rectal cancer.

You answered End ileostomy

The correct answer is Loop colostomy

This patient should be defunctioned, definitive surgery should wait until staging is completed. A
loop ileostomy will not satisfactorily decompress an acutely obstructed colon. Low rectal cancers
that are obstructed should not usually be primarily resected. The obstructed colon that would be
used for anastomosis would carry a high risk of anastomotic dehisence. In addition, as this is an
emergency presentation, staging may not be completed, an attempted resection may therefore
compromise the circumferential resection margin, with an associated risk of local recurrence.

13. A 45 year old man presents with a carcinoma 10cm from the anal verge, he has completed a long
course of chemoradiotherapy and has achieved downstaging with no evidence of threatened
circumferential margin on MRI scanning.

You answered End ileostomy

The correct answer is Anterior resection with covering loop ileostomy

Low rectal cancer is usually treated with a low anterior resection. Contraindications to this include
involvement of the sphincters (unlikely here) and poor sphincter function that would lead to
unsatisfactory function post resection. Most colorectal surgeons defunction resections below the
peritoneal reflection as they have an intrinsically high risk of anastomotic leak. A loop ileostomy
provides a safe an satisfactory method of defunctioning these patients. A contrast enema should
be performed prior to stoma reversal.

Please rate this question:

Discuss and give feedback

Next question

Colorectal cancer treatment

Patients diagnosed as having colorectal cancer should be completely staged using CT of the chest/
abdomen and pelvis. Their entire colon should have been evaluated with colonoscopy or CT
colonography. Patients whose tumours lie below the peritoneal reflection should have their
mesorectum evaluated with MRI.

Once their staging is complete patients should be discussed within a dedicated colorectal MDT
meeting and a treatment plan formulated.
Treatment of colonic cancer
Cancer of the colon is nearly always treated with surgery. Stents, surgical bypass and diversion
stomas may all be used as palliative adjuncts. Resectional surgery is the only option for cure in
patients with colon cancer. The procedure is tailored to the patient and the tumour location. The
lymphatic drainage of the colon follows the arterial supply and therefore most resections are tailored
around the resection of particular lymphatic chains (e.g. ileo-colic pedicle for right sided tumours).
Some patients may have confounding factors that will govern the choice of procedure, for example a
tumour in a patient from a HNPCC family may be better served with a panproctocolectomy rather
than segmental resection. Following resection the decision has to be made regarding restoration of
continuity. For an anastomosis to heal the key technical factors include; adequate blood supply,
mucosal apposition and no tissue tension. Surrounding sepsis, unstable patients and inexperienced
surgeons may compromise these key principles and in such circumstances it may be safer to
construct an end stoma rather than attempting an anastomosis.
When a colonic cancer presents with an obstructing lesion; the options are to either stent it or resect.
In modern practice it is unusual to simply defunction a colonic tumour with a proximal loop stoma.
This differs from the situation in the rectum (see below).
Following resection patients with risk factors for disease recurrence are usually offered
chemotherapy, a combination of 5FU and oxaliplatin is common.

Rectal cancer
The management of rectal cancer is slightly different to that of colonic cancer. This reflects the
rectum's anatomical location and the challenges posed as a result. Tumours located in the rectum
can be surgically resected with either an anterior resection or an abdomino - perineal resection. The
technical aspects governing the choice between these two procedures can be complex to appreciate
and the main point to appreciate for the MRCS is that involvement of the sphincter complex or very
low tumours require APER. In the rectum a 2cm distal clearance margin is required and this may
also impact on the procedure chosen. In addition to excision of the rectal tube an integral part of the
procedure is a meticulous dissection of the mesorectal fat and lymph nodes (total mesorectal
excision/ TME). In rectal cancer surgery invovlement of the cirumferential resection margin carries a
high risk of disease recurrence. Because the rectum is an extraperitoneal structure (until you remove
it that is!) it is possible to irradiate it, something which cannot be offered for colonic tumours. This
has a major impact in rectal cancer treatment and many patients will be offered neoadjuvent
radiotherapy (both long and short course) prior to resectional surgery. Patients with T1 and 2 /N0
disease on imaging do not require irradiation and should proceed straight to surgery. Patients with
T4 disease will typically have long course chemo radiotherapy. Those with T3 , N0 tumours may be
offered short course radiotherapy prior to surgery. Patients presenting with large bowel obstruction
from rectal cancer should not undergo resectional surgery without staging as primary treatment (very
different from colonic cancer). This is because rectal surgery is more technically demanding, the
anastomotic leak rate is higher and the danger of a positive resection margin in an unstaged patient
is high. Therefore patients with obstructing rectal cancer should have a defunctioning loop
colostomy.

Summary of procedures
The operations for cancer are segmental resections based on blood supply and lymphatic drainage.
These commonly performed procedures are core knowledge for the MRCS and should be
understood.

Risk of
Site of cancer Type of resection Anastomosis leak

Right colon Right hemicolectomy Ileo-colic Low <5%

Transverse Extended right hemicolectomy Ileo-colic Low <5%

Splenic Extended right hemicolectomy Ileo-colic Low <5%


flexure

Splenic Left hemicolectomy Colo-colon 2-5%


flexure

Left colon Left hemicolectomy Colo-colon 2-5%

Sigmoid colon High anterior resection Colo-rectal 5%

Upper rectum Anterior resection (TME) Colo-rectal 5%

Low rectum Anterior resection (Low TME) Colo-rectal 10%


(+/- Defunctioning
stoma)

Anal verge Abdomino-perineal excision of colon and None n/a


rectum

In the emergency setting, where the bowel has perforated, the risk of an anastomosis is much
greater, particularly when the anastomosis is colon-colon. In this situation, an end colostomy is often
safer and can be reversed later. When resection of the sigmoid colon is performed and an end
colostomy is fashioned the operation is referred to as a Hartmans procedure. Whilst left sided
resections are more risky, ileo-colic anastomoses are relatively safe even in the emergency setting
and do not need to be defunctioned.

References
A review of the diagnosis and management of colorectal cancer and a summary of the UK National
Institute of Clinical Excellence guidelines is provided in:
Poston G, et al . Diagnosis and management of colorectal cancer:summary of NICE
guidance. BMJ2011: 343: d 6751.

Next question
A 62 year old man has previously undergone a left hemicolectomy for carcinoma of the descending
colon. On follow up imaging he is found to have two deposits of metastatic disease located in the
right lobe of his liver. What is the best treatment strategy?

Chemotherapy alone

Chemotherapy followed by surgical resection

Radiofrequency ablation

Chemoradiotherapy

Palliation

Theme from April 2016 Exam


Liver metastasis from colorectal cancer is still potentially curable. Without resection, survival at 5
years is around 5%. With resection, this figure rises to around 20%. The best outcomes are seen
where chemotherapy is given, followed by resection. Radiofrequency ablation is an option for those
patients who lack the physiological reserve for surgery. However, there is longer term recurrence
rates will all the non resectional strategies. There is no role for radiotherapy.
Please rate this question:

Discuss and give feedback


Next question

Colorectal cancer

 Annually about 150,000 new cases are diagnosed and 50,000 deaths from the disease
 About 75% will have sporadic disease and 25% will have a family history
 Colorectal tumours comprise a spectrum of disease ranging from adenomas through to polyp
cancers and frank malignancy.
 Polyps may be categorised into: neoplastic polyps, adenomatous polyps and non neoplastic
polyps.
 The majority of adenomas are polypoidal lesions, although flat lesions do occur and may
prove to be dysplastic.
 Non-neoplastic polyps include hyperplastic, juvenile, hamartomatous, inflammatory, and
lymphoid polyps, which have not generally been thought of as precursors of cancer.
 Three characteristics of adenomas that correlate with malignant potential have been
characterised. These include increased size, villous architecture and dysplasia. For this
reason most polyps identified at colonoscopy should be removed.
 The transformation from polyp to cancer is described by the adenoma - carcinoma sequence
and its principles should be appreciated. Essentially genetic changes accompany the
transition from adenoma to carcinoma; key changes include APC, c-myc, K RAS mutations
and p53 deletions.

Next question
A 70 year old female is admitted with a history of passing brown coloured urine and abdominal
distension. Clinically she has features of large bowel obstruction with central abdominal tenderness.
She is maximally tender in the left iliac fossa. There is no evidence of haemodynamic instability.
What is the most appropriate investigation?

Cystogram

Abdominal X-ray of the kidney, ureters and bladder

Computerised tomogram of the abdomen and pelvis

Flexible sigmoidoscopy

Barium enema

Theme from April 2013 Exam


Theme from April 2014 exam
This lady is most likely to have a colovesical fistula complicating diverticular disease of the sigmoid
colon. In addition she may also have developed a diverticular stricture resulting in large bowel
obstruction. A locally advanced tumour of the sigmoid colon may produce a similar clinical picture.
The best investigation of this acute surgical patient is an abdominal CT scan, this will demonstrate
the site of the disease and also supply regional information such as organ involvement and other
local complications such as a pericolic abscess. A barium enema would require formal bowel
preparation and this is contra indicated where large bowel obstruction is suspected. A flexible
sigmoidoscopy is unlikely to be helpful and the air insufflated at the time of endoscopy may make the
colonic distension worse. A cystogram would provide only very limited information.
Please rate this question:

Discuss and give feedback


Next question

Diverticular disease

Diverticular disease is a common surgical problem. It consists of herniation of colonic mucosa


through the muscular wall of the colon. The usual site is between the taenia coli where vessels
pierce the muscle to supply the mucosa. For this reason, the rectum, which lacks taenia, is often
spared.

Symptoms

 Altered bowel habit


 Bleeding
 Abdominal pain

Complications

 Diverticulitis
 Haemorrhage
 Development of fistula
 Perforation and faecal peritonitis
 Perforation and development of abscess
 Development of diverticular phlegmon

Diagnosis
Patients presenting in clinic will typically undergo either a colonoscopy, CT cologram or barium
enema as part of their diagnostic work up. All tests can identify diverticular disease. It can be far
more difficult to confidently exclude cancer, particularly in diverticular strictures.

Acutely unwell surgical patients should be investigated in a systematic way. Plain abdominal films
and an erect chest x-ray will identify perforation. An abdominal CT scan (not a CT cologram) with
oral and intravenous contrast will help to identify whether acute inflammation is present but also the
presence of local complications such as abscess formation.

Severity Classification- Hinchey

I Para-colonic abscess

II Pelvic abscess

III Purulent peritonitis

IV Faecal peritonitis

Treatment

 Increase dietary fibre intake.


 Mild attacks of diverticulitis may be managed conservatively with antibiotics.
 Peri colonic abscesses should be drained either surgically or radiologically.
 Recurrent episodes of acute diverticulitis requiring hospitalisation are a relative indication for
a segmental resection.
 Hinchey IV perforations (generalised faecal peritonitis) will require a resection and usually a
stoma. This group have a very high risk of post operative complications and usually require
HDU admission. Less severe perforations may be managed by laparoscopic washout and
drain insertion.

Next question
Which of the following statements in relation to fistula in ano is untrue?

High fistulae are safest treated with a seton insertion

Low fistulae may be laid open

They are typically probed with Lockhart Mummary probes

When discovered during incision and drainage of peri anal abscess; should always be
probed to locate the internal opening

When complicating Crohns disease, may respond to infliximab

Probing fistulae during acute sepsis is associated with a high complication rate and should not be
undertaken routinely.
Please rate this question:

Discuss and give feedback


Next question

Fistulas

 A fistula is defined as an abnormal connection between two epithelial surfaces.


 There are many types ranging from Branchial fistulae in the neck to entero-cutaneous
fistulae abdominally.
 In general surgical practice the abdominal cavity generates the majority and most of these
arise from diverticular disease and Crohn's.
 As a general rule all fistulae will resolve spontaneously as long as there is no distal
obstruction. This is particularly true of intestinal fistulae.

The four types of fistulae are:

Enterocutaneous
These link the intestine to the skin. They may be high (>500ml) or low output (<250ml) depending
upon source. Duodenal /jejunal fistulae will tend to produce high volume, electrolyte rich secretions
which can lead to severe excoriation of the skin. Colo-cutaneous fistulae will tend to leak faeculent
material. Both fistulae may result from the spontaneous rupture of an abscess cavity onto the skin
(such as following perianal abscess drainage) or may occur as a result of iatrogenic input. In some
cases it may even be surgically desirable e.g. mucous fistula following sub total colectomy for colitis.

Suspect if there is excess fluid in the drain.


Enteroenteric or Enterocolic
This is a fistula that involves the large or small intestine. They may originate in a similar manner to
enterocutaneous fistulae. A particular problem with this fistula type is that bacterial overgrowth may
precipitate malabsorption syndromes. This may be particularly serious in inflammatory bowel
disease.

Enterovaginal
Aetiology as above.

Enterovesicular
This type of fistula goes to the bladder. These fistulas may result in frequent urinary tract infections,
or the passage of gas from the urethra during urination.

Management
Some rules relating to fistula management:

 They will heal provided there is no underlying inflammatory bowel disease and no distal
obstruction, so conservative measures may be the best option
 Where there is skin involvement, protect the overlying skin, often using a well fitted stoma
bag- skin damage is difficult to treat
 A high output fistula may be rendered more easily managed by the use of octreotide, this will
tend to reduce the volume of pancreatic secretions.
 Nutritional complications are common especially with high fistula (e.g. high jejunal or
duodenal) these may necessitate the use of TPN to provide nutritional support together with
the concomitant use of octreotide to reduce volume and protect skin.
 When managing perianal fistulae surgeons should avoid probing the fistula where acute
inflammation is present, this almost always worsens outcomes.
 When perianal fistulae occur secondary to Crohn's disease the best management option is
often to drain acute sepsis and maintain that drainage through the judicious use of setons
whilst medical management is implemented.
 Always attempt to delineate the fistula anatomy, for abscesses and fistulae that have an intra
abdominal source the use of barium and CT studies should show a track. For perianal
fistulae surgeons should recall Goodsall's rule in relation to internal and external openings.

Next question
Theme: Large bowel obstruction

A. Ileocolic bypass

B. Loop ileostomy

C. High anterior resection

D. Insertion of self expanding metallic stent

E. Left hemicolectomy and on table colonic lavage and primary anastomosis

F. Extended right hemicolectomy and ileocolic anastomosis

G. Low anterior resection

H. Loop colostomy of the transverse colon

I. Loop colostomy of the sigmoid colon

J. Right hemicolectomy

Please select the most appropriate initial procedure for the following patients with large bowel
obstruction. Each option may be used once, more than once or not at all.

17. A 49 year old lady presents with an 18 hour history of absolute constipation. A CT scan is
performed and demonstrates an obstructing T3 tumour of the distal descending colon. There is
no evidence of distant visceral metastatic disease.

You answered Ileocolic bypass

The correct answer is Left hemicolectomy and on table colonic lavage and primary anastomosis

Resection of the distal descending colon will provide the best method of oncological control.
Careful bowel washout will hopefully create the optimal circumstances for anastomosis. A self
expanding metallic stent is not likely to be beneficial (see below).

18. A 65 year old man presents with absolute constipation and abdominal pain. On examination he
has marked abdominal distension. A digital rectal examination reveals an empty rectum. A rectal
contrast study shows an obstructing lesion of the proximal rectum.
You answered Ileocolic bypass

The correct answer is Loop colostomy of the sigmoid colon

Rectal cancers should not be primarily resected prior to definitive staging and a tumour of this
nature is likely to have circumferential margin involvement. Whilst a sigmoid and transverse loop
colostomy would both provide an equal relief of obstruction the former procedure has the added
benefit of making a subsequent resection safer, since a transverse colostomy would have to be
taken down and closed during the course of subsequent surgery.

19. A 70 year old lady presents with a two day history of constipation and vomiting. On examination
she has right iliac fossa tenderness and little abdominal distension. A CT scan is performed and is
suggestive of an obstructing carcinoma of the colonic hepatic flexure (stage T3).

You answered Ileocolic bypass

The correct answer is Right hemicolectomy

This lesion should be amenable to standard right hemicolectomy. Extending the resection to take
the middle colic vessels and distal transverse colon is unlikely to provide additional oncological
benefit.

Please rate this question:

Discuss and give feedback

Next question

Large bowel obstruction

Colonic obstruction remains a common surgical problem. It is most commonly due to malignancy
(60%) and diverticular disease (20%). Volvulus affecting the colon accounts for 5% of cases. Acute
colonic pseudo-obstruction remains a potential differential diagnosis in all cases. Intussusception
affecting the colon (most often due to tumours in the adult population) remains a rare but recognised
cause.
The typical patient will present with gradual onset of progressive abdominal distension, colicky
abdominal pain and either obstipation or absolute constipation.
On examination abdominal distension is present, the presence of caecal tenderness (assuming no
overt evidence of peritonitis) is a useful sign to elicit. A digital rectal examination and rigid
sigmoidoscopy should be performed.
A plain abdominal x-ray is the usual first line test and; the caecal diameter and ileocaecal valve
competency should be assessed on this film.

Imaging modalities
Debate long surrounds the use of CT versus gastrograffin enemas. The latter investigation has
always been the traditional method of determining whether a structural lesion is indeed present.
However, in the UK the use of this technique has declined and in most units a CT scan will be
offered as the first line investigation by the majority of radiologists (and is advocated by the
ACPGBI). In most cases this will provide sufficient detail to allow operative planning, and since
malignancy accounts for most presentations may also stage the disease. In the event that the
radiologist cannot provide a clear statement of lesion site, the surgeon should have no hesitation in
requesting a contrast enema.

Surgical options
The decision as to when to operate or not is determined firstly by the patients physiological status.
Unstable patients require resuscitation prior to surgery and admission to a critical care unit for
invasive monitoring and potential inotropic support may be needed. In patients who are otherwise
stable the decision then rests on the radiological and clinical findings. As a general rule the old
adage that the sun should not rise and set on unrelieved large bowel obstruction still holds true. A
caecal diameter of 12cm or more in the presence of complete obstruction with a competent
ileocaecal valve and caecal tenderness is a sign of impending perforation and a relative indication
for prompt surgery.

Right sided and transverse lesions


Right sided lesions producing large bowel obstruction should generally be treated by right
hemicolectomy or its extended variant if the lesion lies in the distal transverse colon or splenic
flexure. In these cases an ileocolic anastomosis may be easily constructed and even in the
emergency setting has a low risk of anastomotic leak.

Left sided lesions


The options here lie between sub total colectomy and anastomosis, left hemicolectomy with on table
lavage and primary anastomosis, left hemicolectomy and end colostomy formation and finally colonic
stent insertion.
The usefulness of colonic stents was the subject of a Cochrane review in 2011. The authors
concluded that on the basis of the data that they reviewed, there was no benefit from the use of
colonic stents over conventional surgical resection with a tendency to better outcomes seen in the
surgical group (1). A more recently conducted meta analysis met with the same conclusion (2).
There remains some enthusiasm for use of stents as a bridge to surgery but this may too pass with
the passage of time. They do remain an option in selected cases such as patients who are unfit for
anaesthesia. However, in this group the development of complications at the time of stent insertion
would have dire consequences.
Rectosigmoid lesions
Lesions below the peritoneal reflection that are causing obstruction should generally be treated with
a loop colostomy. Primary resection of unstaged rectal cancer would most likely carry a high CRM
positivity rate and cannot be condoned. Where the lesion occupies the distal sigmoid colon the usual
practice would be to perform a high anterior resection. The decision surrounding restoration of
intestinal continuity would lie with the operating surgeon.

References
1. Sagar J. Colorectal stents for the management of malignant colonic obstructions. Cochrane
Database of Systematic Reviews 2011, Issue 11. Art. No.: CD007378. DOI:
10.1002/14651858.CD007378.pub2.
2. Cirrochi et al Safety and efficacy of endoscopic colonic stenting as a bridge to surgery in the
management of intestinal obstruction due to left colon and rectal cancer: A systematic review and
meta-analysis. Surg Oncol. 2013 Mar;22(1):14-21.

Next question
A 48 year old lady has previously undergone a sigmoid colectomy for carcinoma. On follow up
imaging she is found to have a 3cm foci of metastatic disease in segment IV of the liver. What is the
most appropriate course of action?

Palliative chemotherapy

External beam radiotherapy

Brachytherapy

Surgical resection alone

Chemotherapy followed by surgical resection

Theme from January 2015 Exam


The treatment of colorectal liver metastasis is usually with chemotherapy followed by surgical
resection. Where surgery is performed for liver metastasis with curative intent, the 5 year survival is
20%. Palliation would generally only be considered if the patient were frail or widespread disease
found on imaging. Radiotherapy is not part of the treatment of liver metastasis.

Please rate this question:

Discuss and give feedback

Next question

Colorectal cancer treatment

Patients diagnosed as having colorectal cancer should be completely staged using CT of the chest/
abdomen and pelvis. Their entire colon should have been evaluated with colonoscopy or CT
colonography. Patients whose tumours lie below the peritoneal reflection should have their
mesorectum evaluated with MRI.

Once their staging is complete patients should be discussed within a dedicated colorectal MDT
meeting and a treatment plan formulated.
Treatment of colonic cancer
Cancer of the colon is nearly always treated with surgery. Stents, surgical bypass and diversion
stomas may all be used as palliative adjuncts. Resectional surgery is the only option for cure in
patients with colon cancer. The procedure is tailored to the patient and the tumour location. The
lymphatic drainage of the colon follows the arterial supply and therefore most resections are tailored
around the resection of particular lymphatic chains (e.g. ileo-colic pedicle for right sided tumours).
Some patients may have confounding factors that will govern the choice of procedure, for example a
tumour in a patient from a HNPCC family may be better served with a panproctocolectomy rather
than segmental resection. Following resection the decision has to be made regarding restoration of
continuity. For an anastomosis to heal the key technical factors include; adequate blood supply,
mucosal apposition and no tissue tension. Surrounding sepsis, unstable patients and inexperienced
surgeons may compromise these key principles and in such circumstances it may be safer to
construct an end stoma rather than attempting an anastomosis.
When a colonic cancer presents with an obstructing lesion; the options are to either stent it or resect.
In modern practice it is unusual to simply defunction a colonic tumour with a proximal loop stoma.
This differs from the situation in the rectum (see below).
Following resection patients with risk factors for disease recurrence are usually offered
chemotherapy, a combination of 5FU and oxaliplatin is common.

Rectal cancer
The management of rectal cancer is slightly different to that of colonic cancer. This reflects the
rectum's anatomical location and the challenges posed as a result. Tumours located in the rectum
can be surgically resected with either an anterior resection or an abdomino - perineal resection. The
technical aspects governing the choice between these two procedures can be complex to appreciate
and the main point to appreciate for the MRCS is that involvement of the sphincter complex or very
low tumours require APER. In the rectum a 2cm distal clearance margin is required and this may
also impact on the procedure chosen. In addition to excision of the rectal tube an integral part of the
procedure is a meticulous dissection of the mesorectal fat and lymph nodes (total mesorectal
excision/ TME). In rectal cancer surgery invovlement of the cirumferential resection margin carries a
high risk of disease recurrence. Because the rectum is an extraperitoneal structure (until you remove
it that is!) it is possible to irradiate it, something which cannot be offered for colonic tumours. This
has a major impact in rectal cancer treatment and many patients will be offered neoadjuvent
radiotherapy (both long and short course) prior to resectional surgery. Patients with T1 and 2 /N0
disease on imaging do not require irradiation and should proceed straight to surgery. Patients with
T4 disease will typically have long course chemo radiotherapy. Those with T3 , N0 tumours may be
offered short course radiotherapy prior to surgery. Patients presenting with large bowel obstruction
from rectal cancer should not undergo resectional surgery without staging as primary treatment (very
different from colonic cancer). This is because rectal surgery is more technically demanding, the
anastomotic leak rate is higher and the danger of a positive resection margin in an unstaged patient
is high. Therefore patients with obstructing rectal cancer should have a defunctioning loop
colostomy.

Summary of procedures
The operations for cancer are segmental resections based on blood supply and lymphatic drainage.
These commonly performed procedures are core knowledge for the MRCS and should be
understood.

Risk of
Site of cancer Type of resection Anastomosis leak

Right colon Right hemicolectomy Ileo-colic Low <5%

Transverse Extended right hemicolectomy Ileo-colic Low <5%

Splenic Extended right hemicolectomy Ileo-colic Low <5%


flexure

Splenic Left hemicolectomy Colo-colon 2-5%


flexure

Left colon Left hemicolectomy Colo-colon 2-5%

Sigmoid colon High anterior resection Colo-rectal 5%

Upper rectum Anterior resection (TME) Colo-rectal 5%

Low rectum Anterior resection (Low TME) Colo-rectal 10%


(+/- Defunctioning
stoma)

Anal verge Abdomino-perineal excision of colon and None n/a


rectum

In the emergency setting, where the bowel has perforated, the risk of an anastomosis is much
greater, particularly when the anastomosis is colon-colon. In this situation, an end colostomy is often
safer and can be reversed later. When resection of the sigmoid colon is performed and an end
colostomy is fashioned the operation is referred to as a Hartmans procedure. Whilst left sided
resections are more risky, ileo-colic anastomoses are relatively safe even in the emergency setting
and do not need to be defunctioned.

References
A review of the diagnosis and management of colorectal cancer and a summary of the UK National
Institute of Clinical Excellence guidelines is provided in:
Poston G, et al . Diagnosis and management of colorectal cancer:summary of NICE
guidance. BMJ2011: 343: d 6751.

Next question
What is the commonest type of fistula in ano?

Trans-sphincteric

Supra levator

Complex supra levator

Intersphincteric

Suprasphincteric

Intersphincteric fistulas are the commonest type and the external opening may be internal or
external. These are the classical type of fistula and will have an internal opening near the anal verge
and obey Goodsalls rule. Primary fistulotomy in this situation usually poses little risk to continence.

Please rate this question:

Discuss and give feedback

Next question

Anal fistula

Fistula in ano is the most common form of ano rectal sepsis. Fistulae will have both an internal
opening and external opening, these will be connected by tract(s). Complexity arises because of the
potential for multiple entry and exit sites, together with multiple tracts. Fistulae are classified into four
main groups according to anatomical location and the degree of sphincter involvement. Simple
uncomplicated fistulae are low and do not involve more than 30% of the external sphincter. Complex
fistulae involve the sphincter, have multiple branches or are non cryptoglandular in origin[1]

Assessment
Examination of the perineum for signs of trauma, external openings or the stigmata of IBD is
important. Digital rectal examination may reveal the cord linking the internal and external openings.
At the same time the integrity of the sphincter mechanism can be assessed. Low, uncomplicated
fistulas may not require any further assessment, other groups will usually require more detailed
investigation. For the fistula, the use of endo-anal USS with instillation of hydrogen peroxide into the
fistula tract may be helpful. Ano-rectal MRI scanning is also a useful tool, it is sensitive and specific
for the identification of fistula anatomy, branching tracts and identifying occult sphincter
involvement[2].

Identification of the internal opening


Fistulas with an external opening less than 3cm from the anal verge will typically obey Goodsalls rule
(see below).

Image sourced from Wikipedia

Therapies
Seton suture
A seton is a piece of material that is passed through the fistula between the internal and external
openings that allows the drainage of sepsis. This is important as undrained septic foci may drain
along the path of least resistance, which may result in the development of accessory tracts and
openings. Their main use is in treating complex fistula. Two types of seton are recognised, simple
and cutting. Simple setons lie within the fistula tract and encourage both drainage and fibrosis. A
cutting seton is inserted and the skin incised. The suture is tightened and re-tightened at regular
intervals. This may convert a high fistula to a low fistula. Since the tissue will scar surrounding the
fistula it is hoped that this technique will minimise incontinence[3]. Unfortunately, a large
retrospective review of the literature related to the use of cutting setons has found that they are
associated with a 12% long term incontinence rate [4]

Fistulotomy
Low fistulas, that are simple should be treated by fistulotomy once the acute sepsis has been
controlled. Fistulotomy (where safe) provides the highest healing rates [5]. Because fistulotomy is
regarded as having a high cure rate, there are some who prefer to use this technique with more
extensive sphincter involvement. In these patients the fistulotomy is performed as for a low fistula.
However, the muscle that is encountered is then divided and reconstructed with an overlapping
sphincter repair. A price is paid in terms of incontinence with this technique and up to 12.5% of
patients who were continent pre-operatively will have issues relating to continence post
procedure[6]. The same group also randomised between fistulotomy and sphincter reconstruction
and ano-rectal advancement flaps for the treatment of complex cryptoglandular fistulas and reported
similar outcomes in terms of recurrence (>90%) and disturbances to continence (20%)[7].
Other authors have found adverse outcomes following fistulotomy in patients who have undergone
previous surgery, are of female gender or who have high internal openings [8], in these patients
careful assessment of pre-operative sphincter function should be considered mandatory prior to
fistulotomy.

Anal fistula plugs and fibrin glue


The desire to avoid injury to the sphincter complex has led to surgeons using both fibrin glue and
plugs to try and improve fistula healing. Meticulous preparation of the tract and prior use of a
draining seton is likely to improve chances of success.
The use of anal fistula plugs in high transphincteric fistula of cryptoglandular origin is to be
discouraged because of the high incidence of non response in patients treated with such devices
[9]In most patients septic complications are the reasons for failure [10]. Fibrin glue is a popular
option for the treatment of fistula. There is variability of reported healing rates In some cases initial
success rates of up to 50% healing at six months are reported (in patients with complex cryptogenic
fistula). Of these successes 25% suffer a long term recurrence of fistula [11]. There are, however, no
obvious cases of damage to the sphincter complex and the use of the devices does not appear to
adversely impact on subsequent surgical options.

Ano-rectal advancement flaps


This procedure is primarily directed at high fistulae, and is considered attractive as a sphincter
saving operation. The procedure is performed either with the patient in the prone jack knife position
or in lithotomy (depending upon the site of the fistula). The dissection is commenced in the sub
mucosal plane (which may be infiltrated with dilute adrenaline solution to ease dissection). The
dissection is continued into healthy proximal tissue. This is brought down and sutured over the
defect.
Follow up of patients with cryptoglandular fistulas treated with advancement flaps shows a success
in up to 80% patients[12-14]. With most recurrences occurring in the first 6 months following
surgery[12]. Continence was affected in some patients, with up to 10% describing major continence
issues post operatively.

Ligation of the intersphincteric tract procedure


In this procedure an incision is made in the intersphincteric groove and the fistula tract dissected out
in this plane and divided. A greater than 90% cure rate within 4 weeks was initially reported[15].
Others have subsequently performed similar studies on larger numbers of patients with similar
success rates.

Fistulotomy at the time of abscess drainage?


A Cochrane review conducted in 2010 suggests that primary fistulotomy for low, uncomplicated
fistula in ano may be safe and associated with better outcomes in relation to long term chronic
sepsis[16]. However, there is a danger that such surgery performed by non specialists may result in
a higher complication rate and therefore the traditional teaching is that primary treatment of acute
sepsis is incision and drainage only. All agree that high/ complex fistulae should never be subject to
primary fistulotomy in the acute setting.

References
1. Parks, A.G., P.H. Gordon, and J.D. Hardcastle, A classification of fistula-in-ano. Br J Surg, 1976.
63(1): p. 1-12.
2. Lunniss, P.J., et al., Magnetic resonance imaging of fistula-in-ano. Dis Colon Rectum, 1994.
37(7): p. 708-18.
3. Misra, M.C. and B.M. Kapur, A new non-operative approach to fistula in ano. Br J Surg, 1988.
75(11): p. 1093-4.
4. Ritchie, R.D., J.M. Sackier, and J.P. Hodde, Incontinence rates after cutting seton treatment for
anal fistula. Colorectal Dis, 2009. 11(6): p. 564-71.
5. Tyler, K.M., C.B. Aarons, and S.M. Sentovich, Successful sphincter-sparing surgery for all anal
fistulas. Dis Colon Rectum, 2007. 50(10): p. 1535-9.
6. Perez, F., et al., Prospective clinical and manometric study of fistulotomy with primary sphincter
reconstruction in the management of recurrent complex fistula-in-ano. Int J Colorectal Dis, 2006.
21(6): p. 522-6.
7. Perez, F., et al., Randomized clinical and manometric study of advancement flap versus
fistulotomy with sphincter reconstruction in the management of complex fistula-in-ano. Am J Surg,
2006. 192(1): p. 34-40.
8. Garcia-Aguilar, J., et al., Anal fistula surgery. Factors associated with recurrence and
incontinence. Dis Colon Rectum, 1996. 39(7): p. 723-9.
9. Ortiz, H., et al., Randomized clinical trial of anal fistula plug versus endorectal advancement flap
for the treatment of high cryptoglandular fistula in ano. Br J Surg, 2009. 96(6): p. 608-12.
10. El-Gazzaz, G., M. Zutshi, and T. Hull, A retrospective review of chronic anal fistulae treated by
anal fistulae plug. Colorectal Dis, 2010. 12(5): p. 442-7.
11. Haim, N., et al., Long-term results of fibrin glue treatment for cryptogenic perianal fistulas: a
multicenter study. Dis Colon Rectum, 2011. 54(10): p. 1279-83.
12. Ortiz, H., et al., Length of follow-up after fistulotomy and fistulectomy associated with endorectal
advancement flap repair for fistula in ano. Br J Surg, 2008. 95(4): p. 484-7.
13. Kodner, I.J., et al., Endorectal advancement flap repair of rectovaginal and other complicated
anorectal fistulas. Surgery, 1993. 114(4): p. 682-9; discussion 689-90.
14. Abbas, M.A., R. Lemus-Rangel, and A. Hamadani, Long-term outcome of endorectal
advancement flap for complex anorectal fistulae. Am Surg, 2008. 74(10): p. 921-4.
15. Rojanasakul, A., et al., Total anal sphincter saving technique for fistula-in-ano; the ligation of
intersphincteric fistula tract. J Med Assoc Thai, 2007. 90(3): p. 581-6.
16. Malik, A.I., R.L. Nelson, and S. Tou, Incision and drainage of perianal abscess with or without
treatment of anal fistula. Cochrane Database Syst Rev, 2010(7): p. CD006827.

Next question
You embark on a laparoscopic appendicectomy and find an appendix mass. There is no free fluid
and the patient has no evidence of peritonitis. Which is the best option?

Convert to a midline laparotomy and perform a limited right hemicolectomy and end
ileostomy

Convert to midline laparotomy and perform and appendicectomy after taking down the
adhesions

Place a drain laparoscopically and administer parenteral antibiotics

Send the patient for CT guided drainage

Wrap omentum around the area and avoid drainage

Attempt conservative management for appendix mass without peritonitis.

Dissection of appendix masses can be associated with a considerable degree of morbidity, the gains
of formally dissecting them over simple drainage and antibiotics are minimal.
This was initially described as the Ochsner-Sherren regime and was based on the teachings of
Albert Ochsner of Chicago and James Sherren of the London hospital. The key facts of both
methods (which essentially consisted of non surgical management and careful observation) were
combined and published by Hamilton Bailey in 1930 (Bailey H. The Oschner- Sherren treatment of
acute appendicitis.BMJ 1930 Jan 25; 1(3603): 140143.)
Please rate this question:

Discuss and give feedback


Next question

Appendicitis

History

 Peri umbilical abdominal pain (visceral stretching of appendix lumen and appendix is mid gut
structure) radiating to the right iliac fossa due to localised parietal peritoneal inflammation.
 Vomit once or twice but marked and persistent vomiting is unusual.
 Diarrhoea is rare. However, pelvic appendicitis may cause localised rectal irritation and some
loose stools. A pelvic abscess may also cause diarrhoea.
 Mild pyrexia is common - temperature is usually 37.5 -38oC. Higher temperatures are more
typical of conditions like mesenteric adenitis.
 Anorexia is very common. It is very unusual for patients with appendicitis to be hungry.
Examination

 Generalised peritonitis if perforation has occurred or localised peritonism.


 Retrocaecal appendicitis may have relatively few signs.
 Digital rectal examination may reveal boggy sensation if pelvic abscess is present, or even
tenderness with a pelvic appendix.

Diagnosis

 Typically raised inflammatory markers coupled with compatible history and examination
findings should be enough to justify appendicectomy.
 Urine analysis may show mild leucocytosis but no nitrites.
 Ultrasound is useful in females where pelvic organ pathology is suspected. Although it is not
always possible to visualise the appendix on ultrasound, the presence of free fluid (always
pathological in males) should raise suspicion.

Ultrasound examination may show evidence of luminal obstruction and thickening of the appendiceal
wall as shown below

Image sourced from Wikipedia

Treatment

 Appendicectomy which can be performed via either an open or laparoscopic approach.


 Administration of metronidazole reduces wound infection rates.
 Patients with perforated appendicitis require copious abdominal lavage.
 Patients without peritonitis who have an appendix mass should receive broad spectrum
antibiotics and consideration given to performing an interval appendicectomy.
 Be wary in the older patients who may have either an underlying caecal malignancy or
perforated sigmoid diverticular disease.

Laparoscopic appendicectomy is becoming increasing popular as demonstrated below

Image sourced from Wikipedia

Next question
A 28 year old male presents with painful, bright red, rectal bleeding. On examination he is found to
have a posteriorly sited, midline, fissure in ano. What is the most appropriate treatment?

Topical GTN paste

Sub lingual GTN paste

Anal stretch

Advancement flap

Tailored division of the external anal sphincter

Theme from January 2015 Exam


Topical vasodilator therapy is the most commonly utilised treatment for fissure in ano. Surgical
division of the internal anal sphincter is a reasonable treatment option in a young male. Division of
the external sphincter will almost certainly result in incontinence and is not performed. Anal stretches
were associated with a high rate of external sphincter injuries and have been discontinued for this
reason.
Please rate this question:

Discuss and give feedback


Next question

Anal fissure

Anal fissures are a common cause of painful, bright red, rectal bleeding.
Most fissures are idiopathic and present as a painful mucocutaneous defect in the posterior midline
(90% cases). Fissures are more likely to be anteriorly located in females, particularly if they are
multiparous. Multiple fissures and those which are located at other sites are more likely to be due to
an underlying cause.
Diseases associated with fissure in ano include:

 Crohns disease
 Tuberculosis
 Internal rectal prolapse

Diagnosis
In most cases the defect can be visualised as a posterior midline epithelial defect. Where symptoms
are highly suggestive of the condition and examination findings are unclear an examination under
anaesthesia may be helpful. Atypical disease presentation should be investigated with colonoscopy
and EUA with biopsies of the area.

Treatment
Stool softeners are important as the hard stools may tear the epithelium and result in recurrent
symptoms. The most effective first line agents are topically applied GTN (0.2%) or Diltiazem (2%)
paste. Side effects of diltiazem are better tolerated.
Resistant cases may benefit from injection of botulinum toxin or lateral internal sphincterotomy
(beware in females). Advancement flaps may be used to treat resistant cases.
Sphincterotomy produces the best healing rates. It is associated with incontinence to flatus in up to
10% of patients in the long term.
Next question
A 73 year old lady presents with constipation and no organic disease is identified on investigation.
Which of the following types of laxatives works by direct bowel stimulation?

Magnesium sulphate

Lactulose

Potassium sodium tatrate

Methylcellulose

Senna

Senna contains glycosides. It passes unchanged into the colon where bacteria hydrolyse the
glycosidic bond, releasing the anthracene derivatives. These stimulate the myenteric plexus.

Please rate this question:

Discuss and give feedback

Next question

Laxatives

Bulk forming laxatives

Bran
Psyllium
Methylcellulose
Osmotic laxatives

Magnesium sulphate
Magnesium citrate
Sodium phosphate
Sodium sulphate
Potassium sodium tatrate
Polyethylene glycol

Stimulant laxatives

Docusates
Bisacodyl
Sodium picosulphate
Senna
Ricinoleic acid

Next question
A 32 year old man is diagnosed as having a carcinoma of the caecum. On questioning, his mother
developed uterine cancer at the age of 39 and his maternal uncle died from colonic cancer aged 38.
His older brother developed a colonic cancer with micro satellite instability aged 37. What is the most
appropriate operative treatment?

Limited ileocaecal resection

Right hemicolectomy

Extended right hemicolectomy

Panproctocolectomy

Sub total colectomy

The likely diagnosis is one of a familial cancer syndrome and now that he has developed a colonic
cancer the safest operative strategy is a total colectomy and end ileostomy.

Please rate this question:

Discuss and give feedback

Next question

Polyposis syndromes

Screening and Associated


Syndrome Genetic defect Features management disorders

Familial Mutation of APC Typically over 100 If known to be at risk Gastric fundal
adenomatous gene (80%) cases, colonic adenomas then predictive polyps (50%).
Cancer risk of 100% genetic testing as Duodenal polyps
Screening and Associated
Syndrome Genetic defect Features management disorders

polyposis dominant 20% are new teenager 90%.


mutations Annual flexible If severe
sigmoidoscopy from duodenal
15 years polyposis cancer
If no polyps found risk of 30% at 10
then 5 yearly years.
colonoscopy started Abdominal
at age 20 desmoid
Polyps found = tumours.
resectional surgery
(resection and pouch
Vs sub total
colectomy and IRA)

MYH Biallelic mutation Multiple colonic Once identified Duodenal


associated of mut Y human polyps resection and polyposis in 30%
polyposis homologue (MYH) Later onset right ileoanal pouch Associated with
on chromosome sided cancers more reconstruction is increased risk of
1p, recessive common than in FAP recommended breast cancer
100% cancer risk by Attenuated (self examination)
age 60 phenotype - regular
colonoscopy

Peutz -Jeghers STK11 (LKB1) Multiple benign Annual examination Malignancies at


syndrome mutation on intestinal Pan intestinal other sites
chromosome 19 in hamartomas endoscopy every 2-3 Classical
some (but not all) Episodic obstruction years pigmentation
cases, dominant and intussceception pattern
Increased risk of GI
cancers (colorectal
cancer 20%, gastric
5%)
Increased risk of
breast, ovarian,
cervical pancreatic
Screening and Associated
Syndrome Genetic defect Features management disorders

and testicular
cancers

Cowden Mutation of PTEN Macrocephaly Targeted Breast cancer


disease gene on Multiple intestinal individualised (81% risk)
chromosome hamartomas screening Thyroid cancer
10q22, dominant Multiple and non toxic
trichilemmomas goitre
89% risk of cancer at Uterine cancer
any site
16% risk of colorectal
cancer

HNPCC (Lynch Germline Colo rectal cancer Colonoscopy every 1- Extra colonic
syndrome) mutations of DNA 30-70% 2 years from age 25 cancers
mismatch repair Endometrial cancer Consideration of
genes 30-70% prophylactic surgery
Gastric cancer 5-10% Extra colonic
Scanty colonic polyps surveillance
may be present recommended
Colonic tumours
likely to be right
sided and mucinous

Next question
A 53 year old man has a 1.5cm polyp identified and completely removed during a colonoscopy.
Histology confirms a low grade adenoma. What is the correct follow up?

Discharge.

Repeat endoscopy in 5 years.

Repeat endoscopy in 3 years.

Segmental resection of the affected area.

Barium enema at 5 years.

It would be unsafe to discharge. Follow up with barium enemas for polyps is counter intuitive. In the
UK NICE guidance (2011) this patient would only be classified as high risk if other adenomas were
present, or the removal incomplete, in which case a repeat endoscopy at 1 year would be required.
Otherwise the patient is at intermediate risk and repeat endoscopy at 3 years is warranted.

Please rate this question:

Discuss and give feedback

Next question

Colonic polyps

Colonic Polyps
May occur in isolation, or greater numbers as part of the polyposis syndromes. In FAP greater than
100 polyps are typically present. The risk of malignancy in association with adenomas is related to
size, and is the order of 10% in a 1cm adenoma. Isolated adenomas seldom give risk of symptoms
(unless large and distal). Distally sited villous lesions may produce mucous and if very large,
electrolyte disturbances may occur.

Follow up of colonic polyps


Group Features Action

Low risk 1 or 2 adenomas less than 1cm No follow up or re-colonoscopy


at 5 years

Moderate 3 or 4 small adenomas or 1 adenoma greater than 1cm Re-scope at 3 years


risk

High risk More than 5 small adenomas or more than 3 with 1 of Re scope at 1 year
them greater than 1cm

From Atkins and Saunders Gut 2002 51 (suppl V:V6-V9). It is important to stratify patients
appropriately and ensure that a complete colonoscopy with good views was performed.

Segmental resection or complete colectomy should be considered when:

1. Incomplete excision of malignant polyp


2. Malignant sessile polyp
3. Malignant pedunculated polyp with submucosal invasion
4. Polyps with poorly differentiated carcinoma
5. Familial polyposis coli
-Screening from teenager up to 40 years by 2 yearly sigmoidoscopy/colonoscopy
-Panproctocolectomy and Ileostomy or Restorative Panproctocolectomy.

Rectal polypoidal lesions may be amenable to trans anal endoscopic microsurgery.

References
Cairns S et al. Guidelines for colorectal cancer screening and surveillance in moderate and high risk
groups (update from 2002). Gut 2010;59:666-690.

Next question
Theme: Bowel cancer management

A. Loop colostomy

B. Loop ileostomy

C. Ileo-colic bypass

D. Hartman's procedure

E. Sub total colectomy

F. Right hemicolectomy

G. Left hemicolectomy

H. Abdomino-perineal excision of the colon and rectum

I. Anterior resection

Please select the most appropriate management option for the scenario given. Each option may be
used once, more than once or not at all.

27. A 67 year old man is admitted with acute abdominal pain. He has features of large bowel
obstruction. At laparotomy he has a carcinoma of the sigmoid colon and perforation of the
caecum.

You answered Loop colostomy

The correct answer is Sub total colectomy

Large bowel obstruction will typically result in caecal perforation once the caecal diameter
exceeds 10cm. Once this has occurred the only realistic option is a sub total colectomy and end
ileostomy.

28. A 89 year old lady is admitted with large bowel obstruction. She has tenderness of the right side
of her abdomen and CT scanning shows a sigmoid lesion with liver metastasis. Her caecum
measures 11cm.

Loop colostomy
A loop colostomy is the safest option. A stent would be ideal (but is not on the list).

29. A patient has a tumour 10cm from the anal verge. Staging investigations show localised disease
only.

You answered Loop colostomy

The correct answer is Anterior resection

This should be manageable with a low anterior resection. A covering loop ileostomy should be
constructed to mitigate the effects of any anastomotic leakage. The functional effects of low
anterior resection can be variable and some patients with poor pre-operative anal function (e.g.
faecal incontinence) may be better served with a non restorative procedure (such as a low
Hartmans type resection/ low anterior resection and end colostomy).

Loop colostomy remains the traditional method for relieving inoperable large bowel obstruction.
Colonic stents are becoming increasing popular alternatives, especially as a bridge to surgery.

Please rate this question:

Discuss and give feedback

Next question

Colorectal cancer treatment

Patients diagnosed as having colorectal cancer should be completely staged using CT of the chest/
abdomen and pelvis. Their entire colon should have been evaluated with colonoscopy or CT
colonography. Patients whose tumours lie below the peritoneal reflection should have their
mesorectum evaluated with MRI.

Once their staging is complete patients should be discussed within a dedicated colorectal MDT
meeting and a treatment plan formulated.

Treatment of colonic cancer


Cancer of the colon is nearly always treated with surgery. Stents, surgical bypass and diversion
stomas may all be used as palliative adjuncts. Resectional surgery is the only option for cure in
patients with colon cancer. The procedure is tailored to the patient and the tumour location. The
lymphatic drainage of the colon follows the arterial supply and therefore most resections are tailored
around the resection of particular lymphatic chains (e.g. ileo-colic pedicle for right sided tumours).
Some patients may have confounding factors that will govern the choice of procedure, for example a
tumour in a patient from a HNPCC family may be better served with a panproctocolectomy rather
than segmental resection. Following resection the decision has to be made regarding restoration of
continuity. For an anastomosis to heal the key technical factors include; adequate blood supply,
mucosal apposition and no tissue tension. Surrounding sepsis, unstable patients and inexperienced
surgeons may compromise these key principles and in such circumstances it may be safer to
construct an end stoma rather than attempting an anastomosis.
When a colonic cancer presents with an obstructing lesion; the options are to either stent it or resect.
In modern practice it is unusual to simply defunction a colonic tumour with a proximal loop stoma.
This differs from the situation in the rectum (see below).
Following resection patients with risk factors for disease recurrence are usually offered
chemotherapy, a combination of 5FU and oxaliplatin is common.

Rectal cancer
The management of rectal cancer is slightly different to that of colonic cancer. This reflects the
rectum's anatomical location and the challenges posed as a result. Tumours located in the rectum
can be surgically resected with either an anterior resection or an abdomino - perineal resection. The
technical aspects governing the choice between these two procedures can be complex to appreciate
and the main point to appreciate for the MRCS is that involvement of the sphincter complex or very
low tumours require APER. In the rectum a 2cm distal clearance margin is required and this may
also impact on the procedure chosen. In addition to excision of the rectal tube an integral part of the
procedure is a meticulous dissection of the mesorectal fat and lymph nodes (total mesorectal
excision/ TME). In rectal cancer surgery invovlement of the cirumferential resection margin carries a
high risk of disease recurrence. Because the rectum is an extraperitoneal structure (until you remove
it that is!) it is possible to irradiate it, something which cannot be offered for colonic tumours. This
has a major impact in rectal cancer treatment and many patients will be offered neoadjuvent
radiotherapy (both long and short course) prior to resectional surgery. Patients with T1 and 2 /N0
disease on imaging do not require irradiation and should proceed straight to surgery. Patients with
T4 disease will typically have long course chemo radiotherapy. Those with T3 , N0 tumours may be
offered short course radiotherapy prior to surgery. Patients presenting with large bowel obstruction
from rectal cancer should not undergo resectional surgery without staging as primary treatment (very
different from colonic cancer). This is because rectal surgery is more technically demanding, the
anastomotic leak rate is higher and the danger of a positive resection margin in an unstaged patient
is high. Therefore patients with obstructing rectal cancer should have a defunctioning loop
colostomy.

Summary of procedures
The operations for cancer are segmental resections based on blood supply and lymphatic drainage.
These commonly performed procedures are core knowledge for the MRCS and should be
understood.
Risk of
Site of cancer Type of resection Anastomosis leak

Right colon Right hemicolectomy Ileo-colic Low <5%

Transverse Extended right hemicolectomy Ileo-colic Low <5%

Splenic Extended right hemicolectomy Ileo-colic Low <5%


flexure

Splenic Left hemicolectomy Colo-colon 2-5%


flexure

Left colon Left hemicolectomy Colo-colon 2-5%

Sigmoid colon High anterior resection Colo-rectal 5%

Upper rectum Anterior resection (TME) Colo-rectal 5%

Low rectum Anterior resection (Low TME) Colo-rectal 10%


(+/- Defunctioning
stoma)

Anal verge Abdomino-perineal excision of colon and None n/a


rectum

In the emergency setting, where the bowel has perforated, the risk of an anastomosis is much
greater, particularly when the anastomosis is colon-colon. In this situation, an end colostomy is often
safer and can be reversed later. When resection of the sigmoid colon is performed and an end
colostomy is fashioned the operation is referred to as a Hartmans procedure. Whilst left sided
resections are more risky, ileo-colic anastomoses are relatively safe even in the emergency setting
and do not need to be defunctioned.

References
A review of the diagnosis and management of colorectal cancer and a summary of the UK National
Institute of Clinical Excellence guidelines is provided in:
Poston G, et al . Diagnosis and management of colorectal cancer:summary of NICE
guidance. BMJ2011: 343: d 6751.

Next question
Theme: Causes of rectal bleeding

A. Ulcerative colitis proctitis


B. Diversion proctitis
C. Haemorrhoidal disease
D. Fissure in ano
E. Crohns Proctitis
F. Diverticular bleed
G. Ischaemic colitis
H. Rectal intussceception

Please select the most likely cause of bleeding for the scenario given. Each option may be used
once, more than once or not at all.

30. A previously well 21 year old man is admitted with 2 week history of diarrhoea and
passage of blood and mucous rectally. He has previously undergone an ileocaecal
resection in the past for an inflammatory bowel disorder and takes mesalazine.

You answered Ulcerative colitis proctitis

The correct answer is Crohns Proctitis

His previous right sided resection makes crohns disease the most likely scenario.

31. A 56 year old lady has undergone a Hartman's procedure for diverticulitis. 6 months post
operatively she complains of painless passage of blood stained mucous per rectum.

You answered Ulcerative colitis proctitis

The correct answer is Diversion proctitis

Rectal diversion may result in proctitis.

32. A 74 year old lady has been admitted with sudden onset profuse dark red rectal bleeding.
She was previously well. At the time of assessment her bleeding had stopped but
haemoglobin was 10.5.

You answered Ulcerative colitis proctitis

The correct answer is Diverticular bleed

This pattern of sudden onset profuse bleeding is typical of diverticular bleeding. This often
ceases spontaneously.
Please rate this question:

Discuss and give feedback


Next question

Rectal bleeding

Rectal bleeding is a common cause for patients to be referred to the surgical clinic. In the clinical
history it is useful to try and localise the anatomical source of the blood. Bright red blood is usually of
rectal anal canal origin, whilst dark red blood is more suggestive of a proximally sited bleeding
source. Blood which has entered the GI tract from a gastro-duodenal source will typically resemble
malaena due to the effects of the digestive enzymes on the blood itself.

In the table below we give some typical bleeding scenarios together with physical examination
findings and causation.

Cause Type of Features in history Examination findings


bleeding

Fissure in Bright red Painful bleeding that occurs Muco-epithelial defect usually
ano rectal post defecation in small in the midline posteriorly
bleeding volumes. Usually antecedent (anterior fissures more likely to
features of constipation be due to underlying disease)

Haemorroids Bright red Post defecation bleeding noted Normal colon and rectum.
rectal both on toilet paper and drips Proctoscopy may show internal
bleeding into pan. May be alteration of haemorrhoids. Internal
bowel habit and history of haemorrhoids are usually
straining. No blood mixed with impalpable.
stool. No local pain.

Crohns Bright red or Bleeding that is accompanied Perineal inspection may show
disease mixed blood by other symptoms such as fissures or fistulae. Proctoscopy
altered bowel habit, malaise, may demonstrate indurated
history of fissures (especially mucosa and possibly strictures.
anterior) and abscesses. Skip lesions may be noted at
colonoscopy.

Ulcerative Bright red Diarrhoea, weight loss, Proctitis is the most marked
colitis bleeding nocturnal incontinence, passage finding. Peri anal disease is
often mixed usually absent. Colonoscopy
with stool of mucous PR. will show continuous mucosal
lesion.

Rectal cancer Bright red Alteration of bowel habit. Usually obvious mucosal
blood mixed Tenesmus may be present. abnormality. Lesion may be
volumes Symptoms of metastatic fixed or mobile depending upon
disease. disease extent. Surrounding
mucosa often normal, although
polyps may be present.

Image showing a fissure in ano. Typically these are located posteriorly and in the midline. Fissures
at other sites may be associated with underlying disease.

Image sourced from Wikipedia

Colonoscopic image of internal haemorroids. Note these may often be impalpable.


Image sourced from Wikipedia

Investigation

 All patients presenting with rectal bleeding require digital rectal examination and procto-
sigmoidoscopy as a minimal baseline.
 Remember that haemorrhoids are typically impalpable and to attribute bleeding to these in
the absence of accurate internal inspection is unsatisfactory.
 In young patients with no other concerning features in the history a carefully performed
sigmoidoscopy that demonstrates clear haemorrhoidal disease may be sufficient. If clear
views cannot be obtained then patients require bowel preparation with an enema and a
flexible sigmoidscopy performed.
 In those presenting with features of altered bowel habit or suspicion of inflammatory bowel
disease a colonoscopy is the best test.
 Patients with excessive pain who are suspected of having a fissure may require an
examination under general or local anaesthesia.
 In young patients with external stigmata of fissure and a compatible history it is acceptable to
treat medically and defer internal examination until the fissure is healed. If the fissure fails to
heal then internal examination becomes necessary along the lines suggested above to
exclude internal disease.

Special tests

 In patients with a malignancy of the rectum the staging investigations comprise an MRI of the
rectum to identify circumferential resection margin compromise and to identify mesorectal
nodal disease. In addition to this CT scanning of the chest abdomen and pelvis is necessary
to stage for more distant disease. Some centres will still stage the mesorectum with endo
rectal ultrasound but this is becoming far less common.

 Patients with fissure in ano who are being considered for surgical sphincterotomy and are
females who have an obstetric history should probably have ano rectal manometry testing
performed together with endo anal ultrasound. As this service is not universally available it is
not mandatory but in the absence of such information there are continence issues that may
arise following sphincterotomy.

Management

Disease Management

Fissure in ano GTN ointment 0.2% or diltiazem cream applied topically is the usual first line
treatment. Botulinum toxin for those who fail to respond. Internal
sphincterotomy for those who fail with botox, can be considered earlier in
males.

Haemorroids Lifestyle advice, for small internal haemorrhoids can consider injection
sclerotherapy or rubber band ligation. For external haemorrhoids consider
haemorrhoidectomy. Modern options include HALO procedure and stapled
haemorrhoidectomy.

Inflammatory Medical management- although surgery may be needed for fistulating Crohns
bowel disease (setons).

Rectal cancer Anterior resection or abdomino-perineal excision of the colon and rectum.
Total mesorectal excision is now standard of care. Most resections below the
peritoneal reflection will require defunctioning ileostomy. Most patients will
require preoperative radiotherapy.

Next question
Theme: Diverticular disease management

A. Active observation
B. Colonoscopy acutely
C. Intravenous antibiotics
D. Abdominal CT Scan
E. Ultrasound scan
F. Defecating proctogram
G. Flexible sigmoidoscopy
H. Laparotomy

Please select the most appropriate immediate management for the diverticular presentations given.
Each option may be used once, more than once or not at all.

33. A 40 year old man with known diverticular disease diagnosed on colonoscopy 1 year
previously is admitted with acute abdominal pain. His abdomen is maximally tender in the
left iliac fossa and he describes pneumaturia. His GP has been giving him metronidazole
for 2 days.

You answered Active observation

The correct answer is Abdominal CT Scan

A colovesical fistula has formed and CT will help to delineate the other complications
which may have occurred.

34. An 83 year old lady with known diverticular disease is admitted with a brisk PR bleed. On
assessment the bleeding is settling and her abdomen is soft. Hb 10.2, other blood tests are
normal

Active observation

Diverticular bleeds often settle spontaneously. Acute colonoscopy is rarely helpful. She
may require an elective endoscopy. Isolated diverticular bleeds without evidence of
infection do not necessarily require antibiotics.

35. A 72 year old man is admitted with large bowel obstruction and CT scan suggests
diverticular stricture in the sigmoid colon.

You answered Active observation

The correct answer is Laparotomy


The stricture could be benign or malignant and although a luminal study would establish
aetiology the opportunity for that intervention has passed.
70% of diverticular bleeds will settle with conservative management.

Please rate this question:

Discuss and give feedback


Next question

Diverticular disease

Diverticular disease is a common surgical problem. It consists of herniation of colonic mucosa


through the muscular wall of the colon. The usual site is between the taenia coli where vessels
pierce the muscle to supply the mucosa. For this reason, the rectum, which lacks taenia, is often
spared.

Symptoms

 Altered bowel habit


 Bleeding
 Abdominal pain

Complications

 Diverticulitis
 Haemorrhage
 Development of fistula
 Perforation and faecal peritonitis
 Perforation and development of abscess
 Development of diverticular phlegmon

Diagnosis
Patients presenting in clinic will typically undergo either a colonoscopy, CT cologram or barium
enema as part of their diagnostic work up. All tests can identify diverticular disease. It can be far
more difficult to confidently exclude cancer, particularly in diverticular strictures.

Acutely unwell surgical patients should be investigated in a systematic way. Plain abdominal films
and an erect chest x-ray will identify perforation. An abdominal CT scan (not a CT cologram) with
oral and intravenous contrast will help to identify whether acute inflammation is present but also the
presence of local complications such as abscess formation.
Severity Classification- Hinchey

I Para-colonic abscess

II Pelvic abscess

III Purulent peritonitis

IV Faecal peritonitis

Treatment

 Increase dietary fibre intake.


 Mild attacks of diverticulitis may be managed conservatively with antibiotics.
 Peri colonic abscesses should be drained either surgically or radiologically.
 Recurrent episodes of acute diverticulitis requiring hospitalisation are a relative indication for
a segmental resection.
 Hinchey IV perforations (generalised faecal peritonitis) will require a resection and usually a
stoma. This group have a very high risk of post operative complications and usually require
HDU admission. Less severe perforations may be managed by laparoscopic washout and
drain insertion.

Next question
A 34 year old man presents with symptoms attributable to a fistula in ano. He is examined in the
lithotomy position and the external opening of the fistula is identified in the 7 o'clock position. At
which of the following locations is the internal opening most likely to be identified?

7 o'clock

12 o'clock

9 o'clock

3 o'clock

6 o'clock

Goodsals rule:
Anterior fistulae will tend to have an internal opening opposite the external opening.
Posterior fistulae will tend to have a curved track that passes towards the midline.

According to Goodsalls rule the track of a posteriorly sited fistula will track to the posterior midline
(i.e. 6 o'clock)
Please rate this question:

Discuss and give feedback


Next question

Fistulas

 A fistula is defined as an abnormal connection between two epithelial surfaces.


 There are many types ranging from Branchial fistulae in the neck to entero-cutaneous
fistulae abdominally.
 In general surgical practice the abdominal cavity generates the majority and most of these
arise from diverticular disease and Crohn's.
 As a general rule all fistulae will resolve spontaneously as long as there is no distal
obstruction. This is particularly true of intestinal fistulae.

The four types of fistulae are:

Enterocutaneous
These link the intestine to the skin. They may be high (>500ml) or low output (<250ml) depending
upon source. Duodenal /jejunal fistulae will tend to produce high volume, electrolyte rich secretions
which can lead to severe excoriation of the skin. Colo-cutaneous fistulae will tend to leak faeculent
material. Both fistulae may result from the spontaneous rupture of an abscess cavity onto the skin
(such as following perianal abscess drainage) or may occur as a result of iatrogenic input. In some
cases it may even be surgically desirable e.g. mucous fistula following sub total colectomy for colitis.

Suspect if there is excess fluid in the drain.

Enteroenteric or Enterocolic
This is a fistula that involves the large or small intestine. They may originate in a similar manner to
enterocutaneous fistulae. A particular problem with this fistula type is that bacterial overgrowth may
precipitate malabsorption syndromes. This may be particularly serious in inflammatory bowel
disease.

Enterovaginal
Aetiology as above.

Enterovesicular
This type of fistula goes to the bladder. These fistulas may result in frequent urinary tract infections,
or the passage of gas from the urethra during urination.

Management
Some rules relating to fistula management:

 They will heal provided there is no underlying inflammatory bowel disease and no distal
obstruction, so conservative measures may be the best option
 Where there is skin involvement, protect the overlying skin, often using a well fitted stoma
bag- skin damage is difficult to treat
 A high output fistula may be rendered more easily managed by the use of octreotide, this will
tend to reduce the volume of pancreatic secretions.
 Nutritional complications are common especially with high fistula (e.g. high jejunal or
duodenal) these may necessitate the use of TPN to provide nutritional support together with
the concomitant use of octreotide to reduce volume and protect skin.
 When managing perianal fistulae surgeons should avoid probing the fistula where acute
inflammation is present, this almost always worsens outcomes.
 When perianal fistulae occur secondary to Crohn's disease the best management option is
often to drain acute sepsis and maintain that drainage through the judicious use of setons
whilst medical management is implemented.
 Always attempt to delineate the fistula anatomy, for abscesses and fistulae that have an intra
abdominal source the use of barium and CT studies should show a track. For perianal
fistulae surgeons should recall Goodsall's rule in relation to internal and external openings.

Next question
Theme: Proctology

A. Haemorrhoids
B. Rectal intussceception
C. Fistula in ano
D. Fissure in ano
E. Peri-anal abscess
F. Solitary rectal ulcer
G. Marjolins ulcer

Please select the most likely disorder for the scenario given. Each option may be used once, more
than once or not at all.

37. A 38 year old lady presents with symptoms of obstructed defecation that date back to the
birth of her second child by use of ventouse. She passes mucous and suffers from pelvic
pain. Digital rectal examination and barium enema are normal.

You answered Haemorrhoids

The correct answer is Rectal intussceception

Rectal intussceception (internal rectal prolapse) typically presents with symptoms of


obstructed defecation. The pathology is best demonstrated by a defecating procotogram
rather than barium enema.

38. A 23 year old male presents with bright red rectal bleeding that occurs post defecation
onto the toilet paper. He has been suffering from severe pain associated with this. On
external anal examination there is a skin tag located at the 6 O'clock position.

You answered Haemorrhoids

The correct answer is Fissure in ano

This is a typical story for fissure and should be treated with laxatives and topical
vasodilator (eg GTN) in the first instance.

39. A 19 year old male presents with bright red rectal bleeding that occurs post defecation
onto the paper and into the pan. Apart from constipation his bowel habit is normal. Digital
rectal examination is normal.

Haemorrhoids

This is likely to be haemorrhoidal disease. A sigmoidoscopy should always be performed


to exclude more sinister pathology.

Please rate this question:

Discuss and give feedback


Next question

Ano rectal disease

Location: 3, 7, 11 o'clock position


Haemorrhoids Internal or external
Treatment: Conservative, Rubber band ligation, Haemorrhoidectomy

Fissure in ano Location: midline 6 (posterior midline 90%) and 12 o'clock position. Distal to
the dentate line
Chronic fissure > 6/52: triad: Ulcer, sentinel pile, enlarged anal papillae

Proctitis Causes: Crohn's, ulcerative colitis, Clostridium difficile

Ano rectal E.coli, staph aureus


abscess Positions: Perianal, Ischiorectal, Pelvirectal, Intersphincteric

Anal fistula Usually due to previous ano-rectal abscess


Intersphincteric, transsphincteric, suprasphincteric, and extrasphincteric.
Goodsalls rule determines location

Rectal prolapse Associated with childbirth and rectal intussceception. May be internal or
external

Pruritus ani Systemic and local causes

Anal neoplasm Squamous cell carcinoma commonest unlike adenocarcinoma in rectum

Solitary rectal Associated with chronic straining and constipation. Histology shows mucosal
thickening, lamina propria replaced with collagen and smooth muscle
ulcer (fibromuscular obliteration)

Rectal prolapse

 Common especially in multiparous women.


 May be internal or external.
 Internal rectal prolapse can present insidiously.
 External prolapse can ulcerate and in long term impair continence.
 Diagnostic work up includes colonoscopy, defecating proctogram, ano rectal manometry
studies and if doubt exists an examination under anaesthesia.

Treatments for prolapse

 In the acute setting reduce it (covering it with sugar may reduce swelling.
 Delormes procedure which excises mucosa and plicates the rectum (high recurrence rates)
may be used for external prolapse.
 Altmeirs procedure which resects the colon via the perineal route has lower recurrence rates
but carries the risk of anastamotic leak.
 Rectopexy is an abdominal procedure in which the rectum is elevated and usually supported
at the level of the sacral promontory. Post operative constipation may be reduced by limiting
the dissection to the anterior plane (laparoscopic ventral mesh rectopexy).

Pruritus ani

 Extremely common.
 Check not secondary to altered bowel habits (e.g. Diarrhoea)
 Associated with underlying diseases such as haemorrhoids.
 Examine to look for causes such as worms.
 Proctosigmoidoscopy to identify associated haemorrhoids and exclude cancer.
 Treatment is largely supportive and patients should avoid using perfumed products around
the area.

Fissure in ano

 Typically painful PR bleeding (bright red).


 Nearly always in the posterior midline.
 Usually solitary.

Treatment
 Stool softeners.
 Topical diltiazem (or GTN).
 If topical treatments fail then botulinum toxin should be injected.
 If botulinum toxin fails then males should probably undergo lateral internal sphincterotomy.
 Females who do not respond to botulinum toxin should undergo ano rectal manometry
studies and endo anal USS prior to being offered surgery such as sphincterotomy.

Next question
Theme: Management of inflammatory bowel disease

A. Ileo-anal pouch

B. Panproctocolectomy

C. Sub total colectomy

D. Hartmans procedure

E. Right hemicolectomy

F. Intravenous steroids

G. Infliximab

H. Proctectomy

Please select the most appropriate management option from the list. Each option may be used once,
more than once or not at all.

40. A 20 year old man is admitted with bloody diarrhoea. He has been passing 10 stools per day, Hb-
8.0, albumin-20. Stool culture negative. Evidence of colitis on endoscopy. He has been on
intravenous steroids for 5 days and has now developed megacolon. His haemoglobin is falling and
inflammatory markers are static.

You answered Ileo-anal pouch

The correct answer is Sub total colectomy

This man requires a sub total colectomy. Conservative management has failed. Patients with
ulcerative colitis should undergo colectomy if there is no significant improvement in 5-7 days after
initiating medical therapy if they have a severe attack of the disease.

41. A 19 year old lady has a long standing history of diarrhoea and weight loss. She is investigated
with an upper gastro intestinal endoscopy which is normal. A small bowel contrast study shows a
terminal ileal stricture. A colonoscopy was performed which was normal but the endoscopist was
unable to intubate the terminal ileum. One week after the colonoscopy she is admitted with small
bowel obstruction. Steroids are administered but despite this she fails to improve.
You answered Ileo-anal pouch

The correct answer is Right hemicolectomy

It is likely that this lady has terminal ileal disease. Although , first presentation of Crohns disease is
usually managed with IV steroids, these have been trialled here and failed. A resection will
remove the stricturing disease. If proximal small bowel disease has not been excluded pre-
operatively then this must be evaluated "on table" during surgery to exclude other small bowel
strictures.

42. A 28 year old man is reviewed in the clinic. He has suffered from Crohns disease for many years,
he has recently undergone a sub total colectomy. However, he has residual Crohns in his rectum
and this is the cause of ongoing symptoms. Medical therapy is proving ineffective.

You answered Ileo-anal pouch

The correct answer is Proctectomy

In Crohns patients who have rectal disease and a previous sub total colectomy, a proctectomy is
the best option. An ileo-anal pouch is contra indicated in Crohns as they may fistulate and have
major post operative complications.

Please rate this question:

Discuss and give feedback

Next question

IBD

Ulcerative colitis Vs Crohns

Crohn's disease Ulcerative colitis


Crohn's disease Ulcerative colitis

Distribution Mouth to anus Rectum and colon

Macroscopic Cobblestone appearance, apthoid ulceration Contact bleeding


changes

Depth of disease Transmural inflammation Superficial inflammation

Distribution Patchy Continuous


pattern

Histological Granulomas (non caseating epithelioid cell Crypt abscesses, Inflammatory cells
features aggregates with Langerhans' giant cells) in the lamina propria

Surgical treatment

Ulcerative colitis
In UC the main place for surgery is when medical treatment has failed, in the emergency setting this
will be a sub total colectomy, end ileostomy and a mucous fistula. Electively it will be a pan
proctocolectomy, an ileoanal pouch may be a selected option for some. Remember that
longstanding UC increases colorectal cancer risk.
Image sourced from Wikipedia

Crohn's disease
Unlike UC Crohn's patients need to avoid surgeons, minimal resections are the rule. They should not
have ileoanal pouches as they will do poorly with them. Management of Crohn's ano rectal sepsis is
with a minimal approach, simply drain sepsis and use setons to facilitate drainage. Definitive fistula
surgery should be avoided.
Image sourced from Wikipedia

Next question
A 19 year old man presents with painful rectal bleeding and is found to have an anal fissure. Which
of the following is least associated with this condition?

Leukaemia

Syphilis

Tuberculosis

Sickle cell disease

Crohn's disease

Anal fissures are associated with:

 Sexually transmitted diseases (syphilis, HIV)


 Inflammatory bowel disease (Crohn's up to 50%)
 Leukaemia (25% of patients)
 Tuberculosis
 Previous anal surgery

Please rate this question:

Discuss and give feedback


Next question

Ano rectal disease

Location: 3, 7, 11 o'clock position


Haemorrhoids Internal or external
Treatment: Conservative, Rubber band ligation, Haemorrhoidectomy

Fissure in ano Location: midline 6 (posterior midline 90%) and 12 o'clock position. Distal to
the dentate line
Chronic fissure > 6/52: triad: Ulcer, sentinel pile, enlarged anal papillae
Proctitis Causes: Crohn's, ulcerative colitis, Clostridium difficile

Ano rectal E.coli, staph aureus


abscess Positions: Perianal, Ischiorectal, Pelvirectal, Intersphincteric

Anal fistula Usually due to previous ano-rectal abscess


Intersphincteric, transsphincteric, suprasphincteric, and extrasphincteric.
Goodsalls rule determines location

Rectal prolapse Associated with childbirth and rectal intussceception. May be internal or
external

Pruritus ani Systemic and local causes

Anal neoplasm Squamous cell carcinoma commonest unlike adenocarcinoma in rectum

Solitary rectal Associated with chronic straining and constipation. Histology shows mucosal
ulcer thickening, lamina propria replaced with collagen and smooth muscle
(fibromuscular obliteration)

Rectal prolapse

 Common especially in multiparous women.


 May be internal or external.
 Internal rectal prolapse can present insidiously.
 External prolapse can ulcerate and in long term impair continence.
 Diagnostic work up includes colonoscopy, defecating proctogram, ano rectal manometry
studies and if doubt exists an examination under anaesthesia.

Treatments for prolapse

 In the acute setting reduce it (covering it with sugar may reduce swelling.
 Delormes procedure which excises mucosa and plicates the rectum (high recurrence rates)
may be used for external prolapse.
 Altmeirs procedure which resects the colon via the perineal route has lower recurrence rates
but carries the risk of anastamotic leak.
 Rectopexy is an abdominal procedure in which the rectum is elevated and usually supported
at the level of the sacral promontory. Post operative constipation may be reduced by limiting
the dissection to the anterior plane (laparoscopic ventral mesh rectopexy).

Pruritus ani

 Extremely common.
 Check not secondary to altered bowel habits (e.g. Diarrhoea)
 Associated with underlying diseases such as haemorrhoids.
 Examine to look for causes such as worms.
 Proctosigmoidoscopy to identify associated haemorrhoids and exclude cancer.
 Treatment is largely supportive and patients should avoid using perfumed products around
the area.

Fissure in ano

 Typically painful PR bleeding (bright red).


 Nearly always in the posterior midline.
 Usually solitary.

Treatment

 Stool softeners.
 Topical diltiazem (or GTN).
 If topical treatments fail then botulinum toxin should be injected.
 If botulinum toxin fails then males should probably undergo lateral internal sphincterotomy.
 Females who do not respond to botulinum toxin should undergo ano rectal manometry
studies and endo anal USS prior to being offered surgery such as sphincterotomy.

Next question
Theme: Treatment of bowel cancer

A. Active observation

B. Long course chemoirradiation

C. Chemotherapy with oxaliplatin

D. Short course chemotherapy (5 days)

E. Proceed straight to surgery

F. Post operative radiotherapy

G. Chemotherapy with methotrexate

H. Brachytherapy

I. Anterior pelvic exenteration

Please select the most appropriate management option from the list below. Each option may be
used once, more than once or not at all.

44. A 45 year old female is diagnosed as having a carcinoma of the caecum. She undergoes a CT scan
which shows a tumour invading the muscularis propria with some regional lymphadenopathy.

You answered Active observation

The correct answer is Proceed straight to surgery

Right sided colonic cancers should proceed straight to surgery. Radiotherapy to this area is poorly
tolerated and almost never offered as first line treatment. The decision as to whether or not
chemotherapy is given is dependent upon the final histology.

45. A 55 year old man presents with tenesmus and rectal bleeding. On examination he has a large
bulky rectal cancer at 5cm with tethering to the prostate gland. Imaging shows no distant disease.

You answered Active observation

The correct answer is Long course chemoirradiation


T4 rectal cancers are managed with long course chemoradiotherapy. A dramatic response is not
uncommon. To embark on attempted resection at this stage is to court failure.

46. A 43 year old women undergoes a sigmoid colectomy for carcinoma. The histology report shows
Dukes C disease. She is otherwise well.

You answered Active observation

The correct answer is Chemotherapy with oxaliplatin

Chemotherapy for colonic cancer is usually with oxaliplatin. Neuropathy is a recognised side effect
of this treatment.

Patients with suspected large bowel obstruction should have pseudobstruction excluded with CT
scan, gastrograffin enema or endoscopy prior to intervention.

Please rate this question:

Discuss and give feedback

Next question

Colorectal cancer treatment

Patients diagnosed as having colorectal cancer should be completely staged using CT of the chest/
abdomen and pelvis. Their entire colon should have been evaluated with colonoscopy or CT
colonography. Patients whose tumours lie below the peritoneal reflection should have their
mesorectum evaluated with MRI.

Once their staging is complete patients should be discussed within a dedicated colorectal MDT
meeting and a treatment plan formulated.

Treatment of colonic cancer


Cancer of the colon is nearly always treated with surgery. Stents, surgical bypass and diversion
stomas may all be used as palliative adjuncts. Resectional surgery is the only option for cure in
patients with colon cancer. The procedure is tailored to the patient and the tumour location. The
lymphatic drainage of the colon follows the arterial supply and therefore most resections are tailored
around the resection of particular lymphatic chains (e.g. ileo-colic pedicle for right sided tumours).
Some patients may have confounding factors that will govern the choice of procedure, for example a
tumour in a patient from a HNPCC family may be better served with a panproctocolectomy rather
than segmental resection. Following resection the decision has to be made regarding restoration of
continuity. For an anastomosis to heal the key technical factors include; adequate blood supply,
mucosal apposition and no tissue tension. Surrounding sepsis, unstable patients and inexperienced
surgeons may compromise these key principles and in such circumstances it may be safer to
construct an end stoma rather than attempting an anastomosis.
When a colonic cancer presents with an obstructing lesion; the options are to either stent it or resect.
In modern practice it is unusual to simply defunction a colonic tumour with a proximal loop stoma.
This differs from the situation in the rectum (see below).
Following resection patients with risk factors for disease recurrence are usually offered
chemotherapy, a combination of 5FU and oxaliplatin is common.

Rectal cancer
The management of rectal cancer is slightly different to that of colonic cancer. This reflects the
rectum's anatomical location and the challenges posed as a result. Tumours located in the rectum
can be surgically resected with either an anterior resection or an abdomino - perineal resection. The
technical aspects governing the choice between these two procedures can be complex to appreciate
and the main point to appreciate for the MRCS is that involvement of the sphincter complex or very
low tumours require APER. In the rectum a 2cm distal clearance margin is required and this may
also impact on the procedure chosen. In addition to excision of the rectal tube an integral part of the
procedure is a meticulous dissection of the mesorectal fat and lymph nodes (total mesorectal
excision/ TME). In rectal cancer surgery invovlement of the cirumferential resection margin carries a
high risk of disease recurrence. Because the rectum is an extraperitoneal structure (until you remove
it that is!) it is possible to irradiate it, something which cannot be offered for colonic tumours. This
has a major impact in rectal cancer treatment and many patients will be offered neoadjuvent
radiotherapy (both long and short course) prior to resectional surgery. Patients with T1 and 2 /N0
disease on imaging do not require irradiation and should proceed straight to surgery. Patients with
T4 disease will typically have long course chemo radiotherapy. Those with T3 , N0 tumours may be
offered short course radiotherapy prior to surgery. Patients presenting with large bowel obstruction
from rectal cancer should not undergo resectional surgery without staging as primary treatment (very
different from colonic cancer). This is because rectal surgery is more technically demanding, the
anastomotic leak rate is higher and the danger of a positive resection margin in an unstaged patient
is high. Therefore patients with obstructing rectal cancer should have a defunctioning loop
colostomy.

Summary of procedures
The operations for cancer are segmental resections based on blood supply and lymphatic drainage.
These commonly performed procedures are core knowledge for the MRCS and should be
understood.
Risk of
Site of cancer Type of resection Anastomosis leak

Right colon Right hemicolectomy Ileo-colic Low <5%

Transverse Extended right hemicolectomy Ileo-colic Low <5%

Splenic Extended right hemicolectomy Ileo-colic Low <5%


flexure

Splenic Left hemicolectomy Colo-colon 2-5%


flexure

Left colon Left hemicolectomy Colo-colon 2-5%

Sigmoid colon High anterior resection Colo-rectal 5%

Upper rectum Anterior resection (TME) Colo-rectal 5%

Low rectum Anterior resection (Low TME) Colo-rectal 10%


(+/- Defunctioning
stoma)

Anal verge Abdomino-perineal excision of colon and None n/a


rectum

In the emergency setting, where the bowel has perforated, the risk of an anastomosis is much
greater, particularly when the anastomosis is colon-colon. In this situation, an end colostomy is often
safer and can be reversed later. When resection of the sigmoid colon is performed and an end
colostomy is fashioned the operation is referred to as a Hartmans procedure. Whilst left sided
resections are more risky, ileo-colic anastomoses are relatively safe even in the emergency setting
and do not need to be defunctioned.

References
A review of the diagnosis and management of colorectal cancer and a summary of the UK National
Institute of Clinical Excellence guidelines is provided in:
Poston G, et al . Diagnosis and management of colorectal cancer:summary of NICE
guidance. BMJ2011: 343: d 6751.

Next question
A 53 year old man presents with a full thickness external rectal prolapse. Which of the following
procedures would be the most suitable surgical option?

Rectopexy

Delormes

Altmeirs

Thirsch tape

Abdomino-perineal excision of the rectum

Theme from 2012 Exam


As this man is young and has full thickness prolapse a rectopexy is the most appropriate procedure.
It will give the lowest recurrence rates.
Please rate this question:

Discuss and give feedback


Next question

Rectal prolapse

Rectal prolapse may be divided into internal and external prolapse. Patients with the former
condition may have internal intussceception of the rectum and present with constipation, obstructed
defecation and occasionally faecal incontinence. Patients with external rectal prolapse have a full
thickness external protrusion of the rectum. Risk factors for the condition include multiparity, pelvic
floor trauma and connective tissue disorders.

Diagnosis
External prolapse is usually evident. Internal prolapse may be identified by defecating proctography
and examination under anaesthesia.
Sinister pathology should be excluded with endoscopy

Treatment

 Perineal approaches include the Delormes operation, this avoids resection and is relatively
safe but is associated with high recurrence rates. An Altmeirs operation involves a perineal
excision of the sigmoid colon and rectum, it may be a more effective procedure than a
Delormes but carries the risk of anastomotic leak.
 Rectopexy - this is an abdominal procedure. The rectum is mobilised and fixed onto the
sacral promontary. A prosthetic mesh may be inserted. The recurrence rates are low and the
procedure is well tolerated (particularly if performed laparoscopically).
 Thirsch tape- this is a largely historical procedure and involves encircling the rectum with
tape or wire. It may be of use in a palliative setting.

Next question
A patient has an appendicectomy and a 1.2cm carcinoid tumour is identified in the tip of the
appendix. What is the most appropriate management?

Watchful waiting

Discharge

Right hemicolectomy

Limited ileocaecal resection

Radioisotope scan

Individuals with small carcinoids can be discharged (<2cm and limited to the appendix). Larger
tumours should have a radioisotope scan. Where the resection margin is positive or where the
isotope scan suggests lymphatic metastasis a right hemicolectomy should be performed.
Please rate this question:

Discuss and give feedback


Next question

Carcinoid syndrome

 Carcinoid tumours secrete serotonin


 Originate in neuroendocrine cells mainly in the intestine (midgut-distal ileum/appendix)
 Can occur in the rectum, bronchi
 Hormonal symptoms mainly occur when disease spreads outside the bowel

Clinical features

 Onset: insidious over many years


 Flushing face
 Palpitations
 Pulmonary valve stenosis and tricuspid regurgitation causing dyspnoea
 Asthma
 Severe diarrhoea (secretory, persists despite fasting)

Investigation
 5-HIAA in a 24-hour urine collection
 Somatostatin receptor scintigraphy
 CT scan
 Blood testing for chromogranin A

Treatment

 Octreotide
 Surgical removal

Next question
Theme: Management of colonic disease

A. Reassure and discharge

B. Right hemicolectomy

C. Left hemicolectomy

D. Hot biopsy

E. Snare polypectomy

F. Single colonoscopy

G. Annual colonoscopy

H. Colonoscopy every 3-5 years

I. Panproctocolectomy

For each scenario given please select the most appropriate management option. Each option may
be used once, more than once or not at all.

49. A 25 year old male presents with altered bowel habit. He is known to have familial polyposis coli.
A colonoscopy shows widespread polyps, with high grade dysplasia in a polyp removed from the
rectum.

You answered Reassure and discharge

The correct answer is Panproctocolectomy

There is a high risk of conversion to malignancy, therefore panproctocolectomy is the safest


option.

50. A 19 year old female presents with colicky abdominal pain, bloating and alternating
constipation/diarrhoea. Her grandmother died from colon cancer at the age of 77 years. A digital
rectal examination and general physical examination are normal.

Reassure and discharge


This girl fulfills the Rome criteria for irritable bowel syndrome. Examination is normal, therefore
there is no indication for further investigation.

51. A 62 year old man is being investigated for iron deficiency anaemia. During a colonoscopy a flat
polypoidal lesion is identified in the caecum. Biopsies of this lesion demonstrate high grade
dysplasia.

You answered Reassure and discharge

The correct answer is Right hemicolectomy

High grade dysplasia in a flat villous lesion of the right colon is highly likely to be associated with
an invasive lesion at this site. Hot biopsy of right sided colonic lesions is unwise an snare
polypectomy would be unlikely to remove the entire lesion.

Please rate this question:

Discuss and give feedback

Next question

Colonic polyps

Colonic Polyps
May occur in isolation, or greater numbers as part of the polyposis syndromes. In FAP greater than
100 polyps are typically present. The risk of malignancy in association with adenomas is related to
size, and is the order of 10% in a 1cm adenoma. Isolated adenomas seldom give risk of symptoms
(unless large and distal). Distally sited villous lesions may produce mucous and if very large,
electrolyte disturbances may occur.

Follow up of colonic polyps

Group Features Action


Group Features Action

Low risk 1 or 2 adenomas less than 1cm No follow up or re-colonoscopy


at 5 years

Moderate 3 or 4 small adenomas or 1 adenoma greater than 1cm Re-scope at 3 years


risk

High risk More than 5 small adenomas or more than 3 with 1 of Re scope at 1 year
them greater than 1cm

From Atkins and Saunders Gut 2002 51 (suppl V:V6-V9). It is important to stratify patients
appropriately and ensure that a complete colonoscopy with good views was performed.

Segmental resection or complete colectomy should be considered when:

1. Incomplete excision of malignant polyp


2. Malignant sessile polyp
3. Malignant pedunculated polyp with submucosal invasion
4. Polyps with poorly differentiated carcinoma
5. Familial polyposis coli
-Screening from teenager up to 40 years by 2 yearly sigmoidoscopy/colonoscopy
-Panproctocolectomy and Ileostomy or Restorative Panproctocolectomy.

Rectal polypoidal lesions may be amenable to trans anal endoscopic microsurgery.

References
Cairns S et al. Guidelines for colorectal cancer screening and surveillance in moderate and high risk
groups (update from 2002). Gut 2010;59:666-690.

Next question
Theme: Fistula management

A. No further action needed


B. Intravenous fluids
C. Intravenous fluids and nasogastric tube
D. Total parenteral nutrition and octreotide
E. Defunctioning stoma
F. Insertion of seton
G. Intravenous octreotide
H. Lay open fistula

What is the best management for the following types of fistula? Each option may be used once,
more than once or not at all.

52. A 45 year old man develops a colocutaneous fistulae following reversal of a loop
colostomy fashioned for the defunctioning of an anterior resection. Pre-operative
gastrograffin enema showed no distal obstruction or anastamotic stricture.

No further action needed

Colocutaneous fistulae may occur as a result of anastomotic leakage following loop


colostomy reversal. In the absence of abdominal signs a laparotomy is not necessarily
required. Signs of wound sepsis may require antibiotics. Because there is not any distal
obstruction (note normal pre-operative gastrograffin enema) these fistulae will usually
close spontaneously.

53. A 43 year old man has suffered from small bowel Crohns disease for 15 years. Following a
recent stricturoplasty he develops an enterocutaneous fistula which is high output. Small
bowel follow through shows it to be 15 cm from the DJ flexure. His overlying skin is
becoming excoriated.

You answered No further action needed

The correct answer is Total parenteral nutrition and octreotide

This man has a high output and anatomically high fistula. Drying up the fistula with
octreotide will not suffice, his nutrition is compromised and TPN will help.

54. A 33 year old lady presented with jaundice secondary to common bile duct stones. A
cholecystectomy and common bile duct exploration is performed and the bile duct closed
over a T tube. Six weeks post operatively a T tube cholangiogram is performed and shows
no residual stones. The T tube is removed and five hours after removal a small amount of
bile is noted to be draining from the T tube site.
No further action needed

When the bile duct is closed over a T Tube the latex in the T tube encourages tract fibrosis.
This actually encourages a fistula to develop. The result is that when the tube is removed
any bile which leaks will usually drain through the tract. Provided that there are no
residual stones in the duct the fistula will slowly close. Persistent high volume drainage
may be managed with ERCP and sphincterotomy.
Fistulas are more likely to heal in the absence of distal lumenal obstruction

Please rate this question:

Discuss and give feedback


Next question

Fistulas

 A fistula is defined as an abnormal connection between two epithelial surfaces.


 There are many types ranging from Branchial fistulae in the neck to entero-cutaneous
fistulae abdominally.
 In general surgical practice the abdominal cavity generates the majority and most of these
arise from diverticular disease and Crohn's.
 As a general rule all fistulae will resolve spontaneously as long as there is no distal
obstruction. This is particularly true of intestinal fistulae.

The four types of fistulae are:

Enterocutaneous
These link the intestine to the skin. They may be high (>500ml) or low output (<250ml) depending
upon source. Duodenal /jejunal fistulae will tend to produce high volume, electrolyte rich secretions
which can lead to severe excoriation of the skin. Colo-cutaneous fistulae will tend to leak faeculent
material. Both fistulae may result from the spontaneous rupture of an abscess cavity onto the skin
(such as following perianal abscess drainage) or may occur as a result of iatrogenic input. In some
cases it may even be surgically desirable e.g. mucous fistula following sub total colectomy for colitis.

Suspect if there is excess fluid in the drain.

Enteroenteric or Enterocolic
This is a fistula that involves the large or small intestine. They may originate in a similar manner to
enterocutaneous fistulae. A particular problem with this fistula type is that bacterial overgrowth may
precipitate malabsorption syndromes. This may be particularly serious in inflammatory bowel
disease.

Enterovaginal
Aetiology as above.

Enterovesicular
This type of fistula goes to the bladder. These fistulas may result in frequent urinary tract infections,
or the passage of gas from the urethra during urination.

Management
Some rules relating to fistula management:

 They will heal provided there is no underlying inflammatory bowel disease and no distal
obstruction, so conservative measures may be the best option
 Where there is skin involvement, protect the overlying skin, often using a well fitted stoma
bag- skin damage is difficult to treat
 A high output fistula may be rendered more easily managed by the use of octreotide, this will
tend to reduce the volume of pancreatic secretions.
 Nutritional complications are common especially with high fistula (e.g. high jejunal or
duodenal) these may necessitate the use of TPN to provide nutritional support together with
the concomitant use of octreotide to reduce volume and protect skin.
 When managing perianal fistulae surgeons should avoid probing the fistula where acute
inflammation is present, this almost always worsens outcomes.
 When perianal fistulae occur secondary to Crohn's disease the best management option is
often to drain acute sepsis and maintain that drainage through the judicious use of setons
whilst medical management is implemented.
 Always attempt to delineate the fistula anatomy, for abscesses and fistulae that have an intra
abdominal source the use of barium and CT studies should show a track. For perianal
fistulae surgeons should recall Goodsall's rule in relation to internal and external openings.

Next question
Theme: Surgery for inflammatory bowel disease

A. Proctectomy
B. Anterior resection
C. Panproctocolectomy
D. Panproctocolectomy and ileoanal pouch
E. Sub total colectomy
F. Right hemicolectomy

Please select the most appropriate surgical modality for treating the inflammatory bowel disease
scenarios described. Each option may be used once, more than once or not at all.

55. A 22 year old man presents with his first presentation of ulcerative colitis. Despite
aggressive medical management with steroids, azathioprine and infliximab his symptoms
remain unchanged and he has developed a megacolon.

You answered Proctectomy

The correct answer is Sub total colectomy

In patients with fulminant UC a sub total colectomy is the safest treatment option. The
rectum will be left in situ as resection of the rectum in these acutely unwell patients carries
an extremely high risk of complications.

56. A 22 year old lady has a long history of severe perianal Crohns disease with multiple
fistulae. She is keen to avoid a stoma. However, she has progressive disease and multiple
episodes of rectal bleeding. A colonoscopy shows rectal disease only and a small bowel
study shows no involvement with Crohns.

Proctectomy

Severe rectal Crohns that has developed complications such as haemorrhage and multiple
fistulae is usually best managed with proctectomy. Although a diverting stoma may reduce
the risk of local sepsis it is unlikely to reduce the bleeding. She is keen to conserve a
rectum, however, an ileoanal pouch in this setting is unwise.

57. A 22 year old man has a long history of ulcerative colitis. His symptoms are well
controlled with steroids. However, attempts at steroid weaning and use of steroid sparing
drugs have repeatedly failed. He wishes to avoid a permanent stoma.

You answered Proctectomy

The correct answer is Panproctocolectomy and ileoanal pouch


In patients with UC where medical management is not successful, surgical resection may
offer a chance of cure. Those patients wishing to avoid a permanent stoma may be
considered for an ileoanal pouch. However, this procedure is only offered in the elective
setting.

Please rate this question:

Discuss and give feedback


Next question

Surgery for inflammatory bowel disease

Patients with inflammatory bowel disease (UC and Crohns) frequently present in surgical practice.
Ulcerative colitis may be cured by surgical resection (Proctocolectomy), this is not the case in
Crohns disease which may recur and affect other areas of the gastrointestinal tract.

Ulcerative colitis

 Elective indications for surgery include disease that is requiring maximal therapy, or
prolonged courses of steroids.
 Longstanding UC is associated with a risk of malignant transformation. Dysplastic
transformation of the colonic epithelium with associated mass lesions is an absolute
indication for a proctocolectomy.
 Emergency presentations of poorly controlled colitis that fails to respond to medical therapy
should usually be managed with a sub total colectomy. Excision of the rectum is a procedure
with a higher morbidity and is not generally performed in the emergency setting. An end
ileostomy is usually created and the rectum either stapled off and left in situ, or, if the bowel
is very oedematous, may be brought to the surface as a mucous fistula.
 Patients with IBD have a high incidence of DVT and appropriate thromboprophylaxis is
mandatory.
 Restorative options in UC include an ileoanal pouch. This procedure can only be performed
whilst the rectum is in situ and cannot usually be undertaken as a delayed procedure
following proctectomy.
 Ileoanal pouch complications include, anastomotic dehiscence, pouchitis and poor
physiological function with seepage and soiling.

Crohns disease

 Surgical resection of Crohns disease does not equate with cure, but may produce substantial
symptomatic improvement.
 Indications for surgery include complications such as fistulae, abscess formation and
strictures.
 Extensive small bowel resections may result in short bowel syndrome and localised
stricturoplasty may allow preservation of intestinal length.
 Staging of Crohns will usually involve colonoscopy and a small bowel study (e.g. MRI
enteroclysis).
 Complex perianal fistulae are best managed with long term draining seton sutures, complex
attempts at fistula closure e.g. advancement flaps, may be complicated by non healing and
fistula recurrence.
 Severe perianal and / or rectal Crohns may require proctectomy. Ileoanal pouch
reconstruction in Crohns carries a high risk of fistula formation and pouch failure and is not
recommended.
 Terminal ileal Crohns remains the commonest disease site and these patients may be
treated with limited ileocaecal resections.
 Terminal ileal Crohns may affect enterohepatic bile salt recycling and increase the risk of
gallstones.

Next question
A 55 year old man develops an acute colonic pseudo-obstruction following a laminectomy. Despite
correction of his electrolytes and ongoing supportive care he fails to settle. Which of the drugs listed
below may improve the situation?

Buscopan

Neostigmine

Metoclopramide

Mebevrine

Sodium picosulphate

Neostigmine affects the degradation of acetylcholine and will therefore stimulate both nicotinic and
muscarinic receptors. It may produce symptomatic bradycardia and should therefore only be
administered in a monitored environment. In colonic pseudo-obstruction it produces generalised
colonic contractions and its onset is usually rapid.

Please rate this question:

Discuss and give feedback

Next question

Colonic pseudo-obstruction

Colonic pseudo-obstruction is characterised by the progressive and painless dilation of the colon.
The abdomen may become grossly distended and tympanic. Unless a complication such as
impending bowel necrosis or perforation occurs, there is usually little pain.
Diagnosis involves excluding a mechanical bowel obstruction with a plain film and contrast enema.
The underlying cause is usually electrolyte imbalance and the condition will resolve with correction of
this and supportive care.
Patients who do not respond to supportive measures should be treated with attempted colonoscopic
decompression and/ or the drug neostigmine. In rare cases surgery may be required.
A 55 year old man is found to have a carcinoma of the sigmoid colon on screening colonoscopy.
How should this be staged?

MRI of the abdomen and CT of the chest

Liver MRI and Chest CT

CT scanning of the chest, abdomen and pelvis alone

MRI of the rectum and CT of the abdomen and chest

Endoluminal USS and CT scanning of the abdomen

Colonic cancers are staged with CT scanning of the chest, abdomen and pelvis.

Theme from January 2015 Exam


Rectal cancer is staged with MRI rectum (and sometimes endolumenal USS for low T1 lesions)
together with CT scanning of the chest, abdomen and pelvis. Historically, colonic cancer was staged
with liver USS and CXR. However, modern imaging has made this practice obsolete.
Please rate this question:

Discuss and give feedback


Next question

Colorectal cancer screening and diagnosis

Overview

 Most cancers develop from adenomatous polyps. Screening for colorectal cancer has been
shown to reduce mortality by 16%
 The NHS now has a national screening programme offering screening every 2 years to all
men and women aged 60 to 69 years. Patients aged over 70 years may request screening
 Eligible patients are sent faecal occult blood (FOB) tests through the post
 Patients with abnormal results are offered a colonoscopy

At colonoscopy, approximately:

 5 out of 10 patients will have a normal exam


 4 out of 10 patients will be found to have polyps which may be removed due to their
premalignant potential
 1 out of 10 patients will be found to have cancer

Diagnosis
Essentially the following patients need referral:
- Altered bowel habit for more than six weeks
- New onset of rectal bleeding
- Symptoms of tenesmus

Colonoscopy is the gold standard, provided it is complete and good mucosal visualisation is
achieved. Other options include double contrast barium enema and CT colonography.

Staging
Once a malignant diagnosis is made patients with colonic cancer will be staged using chest /
abdomen and pelvic CT. Patients with rectal cancer will also undergo evaluation of the mesorectum
with pelvic MRI scanning.

For examination purposes the Dukes and TNM systems are preferred.

Tumour markers
Carcinoembryonic antigen (CEA) is the main tumour marker in colorectal cancer. Not all tumours
secrete this, and it may be raised in conditions such as IBD. However, absolute levels do correlate
(roughly) with disease burden and it is once again being used routinely in follow up.
Next question
Theme: Causes of rectal bleeding

A. Fissure in ano
B. Intersphincteric abscess
C. Haemorroidal disease
D. Proctitis
E. Solitary rectal ulcer syndrome
F. Rectal cancer
G. Anal cancer

Please select the most likely cause of rectal bleeding from the list above. Each option may be used
once, more than once or not at all.

60. An 18 year old man with a previous history of constipation presents with bright red rectal
bleeding and diarrhoea. He has suffered episodes of faecal incontinence, which have
occurred randomly throughout the day and night.

You answered Fissure in ano

The correct answer is Proctitis

Nocturnal diarrhoea and incontinence are typical of inflammatory bowel disease.

61. A 56 year old man presents with episodes of pruritus ani and bright red rectal bleeding. On
examination there is a mass in the ano rectal region and biopsies confirm squamous cell
cancer.

You answered Fissure in ano

The correct answer is Anal cancer

These are features of anal cancer. Anal cancers arise from the cutaneous epithelium and
are therefore typically squamous cell. They are usually sensitive to chemoradiotherapy.

62. A 19 year old man presents with bright red rectal bleeding. He has a longstanding history
of irritable bowel syndrome. At flexible sigmoidoscopy a lesion is biopsied and reported as
showing 'fibromuscular obliteration'.

You answered Fissure in ano

The correct answer is Solitary rectal ulcer syndrome

This is the typical presentation of SRUS. These patients require careful diagnostic work up
to elicit the underlying cause of their altered bowel habit. The histological appearances of
solitary rectal ulcers are characteristic and extensive collagenous deposits are often seen.
This is usually termed fibromuscular obliteration.

Please rate this question:

Discuss and give feedback


Next question

Rectal bleeding

Rectal bleeding is a common cause for patients to be referred to the surgical clinic. In the clinical
history it is useful to try and localise the anatomical source of the blood. Bright red blood is usually of
rectal anal canal origin, whilst dark red blood is more suggestive of a proximally sited bleeding
source. Blood which has entered the GI tract from a gastro-duodenal source will typically resemble
malaena due to the effects of the digestive enzymes on the blood itself.

In the table below we give some typical bleeding scenarios together with physical examination
findings and causation.

Cause Type of Features in history Examination findings


bleeding

Fissure in Bright red Painful bleeding that occurs Muco-epithelial defect usually
ano rectal post defecation in small in the midline posteriorly
bleeding volumes. Usually antecedent (anterior fissures more likely to
features of constipation be due to underlying disease)

Haemorroids Bright red Post defecation bleeding noted Normal colon and rectum.
rectal both on toilet paper and drips Proctoscopy may show internal
bleeding into pan. May be alteration of haemorrhoids. Internal
bowel habit and history of haemorrhoids are usually
straining. No blood mixed with impalpable.
stool. No local pain.

Crohns Bright red or Bleeding that is accompanied Perineal inspection may show
disease mixed blood by other symptoms such as fissures or fistulae. Proctoscopy
altered bowel habit, malaise, may demonstrate indurated
history of fissures (especially mucosa and possibly strictures.
anterior) and abscesses. Skip lesions may be noted at
colonoscopy.

Ulcerative Bright red Diarrhoea, weight loss, Proctitis is the most marked
colitis bleeding nocturnal incontinence, passage finding. Peri anal disease is
often mixed of mucous PR. usually absent. Colonoscopy
with stool will show continuous mucosal
lesion.

Rectal cancer Bright red Alteration of bowel habit. Usually obvious mucosal
blood mixed Tenesmus may be present. abnormality. Lesion may be
volumes Symptoms of metastatic fixed or mobile depending upon
disease. disease extent. Surrounding
mucosa often normal, although
polyps may be present.

Image showing a fissure in ano. Typically these are located posteriorly and in the midline. Fissures
at other sites may be associated with underlying disease.

Image sourced from Wikipedia

Colonoscopic image of internal haemorroids. Note these may often be impalpable.


Image sourced from Wikipedia

Investigation

 All patients presenting with rectal bleeding require digital rectal examination and procto-
sigmoidoscopy as a minimal baseline.
 Remember that haemorrhoids are typically impalpable and to attribute bleeding to these in
the absence of accurate internal inspection is unsatisfactory.
 In young patients with no other concerning features in the history a carefully performed
sigmoidoscopy that demonstrates clear haemorrhoidal disease may be sufficient. If clear
views cannot be obtained then patients require bowel preparation with an enema and a
flexible sigmoidscopy performed.
 In those presenting with features of altered bowel habit or suspicion of inflammatory bowel
disease a colonoscopy is the best test.
 Patients with excessive pain who are suspected of having a fissure may require an
examination under general or local anaesthesia.
 In young patients with external stigmata of fissure and a compatible history it is acceptable to
treat medically and defer internal examination until the fissure is healed. If the fissure fails to
heal then internal examination becomes necessary along the lines suggested above to
exclude internal disease.

Special tests

 In patients with a malignancy of the rectum the staging investigations comprise an MRI of the
rectum to identify circumferential resection margin compromise and to identify mesorectal
nodal disease. In addition to this CT scanning of the chest abdomen and pelvis is necessary
to stage for more distant disease. Some centres will still stage the mesorectum with endo
rectal ultrasound but this is becoming far less common.

 Patients with fissure in ano who are being considered for surgical sphincterotomy and are
females who have an obstetric history should probably have ano rectal manometry testing
performed together with endo anal ultrasound. As this service is not universally available it is
not mandatory but in the absence of such information there are continence issues that may
arise following sphincterotomy.

Management

Disease Management

Fissure in ano GTN ointment 0.2% or diltiazem cream applied topically is the usual first line
treatment. Botulinum toxin for those who fail to respond. Internal
sphincterotomy for those who fail with botox, can be considered earlier in
males.

Haemorroids Lifestyle advice, for small internal haemorrhoids can consider injection
sclerotherapy or rubber band ligation. For external haemorrhoids consider
haemorrhoidectomy. Modern options include HALO procedure and stapled
haemorrhoidectomy.

Inflammatory Medical management- although surgery may be needed for fistulating Crohns
bowel disease (setons).

Rectal cancer Anterior resection or abdomino-perineal excision of the colon and rectum.
Total mesorectal excision is now standard of care. Most resections below the
peritoneal reflection will require defunctioning ileostomy. Most patients will
require preoperative radiotherapy.

Next question
A 56 year old man presents with his first attack of diverticulitis. Which of these complications is least
likely to ensue?

Formation of colonic strictures

Malignant transformation

Development of colovesical fistula

Formation of a pericolic abscess

Formation of a phlegmon

Diverticulitis may result in a number of complications. However, whilst malignant disease may
coincide with diverticulitis it is not, in itself, a risk factor for colonic cancer.
Please rate this question:

Discuss and give feedback


Next question

Diverticular disease

Diverticular disease is a common surgical problem. It consists of herniation of colonic mucosa


through the muscular wall of the colon. The usual site is between the taenia coli where vessels
pierce the muscle to supply the mucosa. For this reason, the rectum, which lacks taenia, is often
spared.

Symptoms

 Altered bowel habit


 Bleeding
 Abdominal pain

Complications

 Diverticulitis
 Haemorrhage
 Development of fistula
 Perforation and faecal peritonitis
 Perforation and development of abscess
 Development of diverticular phlegmon

Diagnosis
Patients presenting in clinic will typically undergo either a colonoscopy, CT cologram or barium
enema as part of their diagnostic work up. All tests can identify diverticular disease. It can be far
more difficult to confidently exclude cancer, particularly in diverticular strictures.

Acutely unwell surgical patients should be investigated in a systematic way. Plain abdominal films
and an erect chest x-ray will identify perforation. An abdominal CT scan (not a CT cologram) with
oral and intravenous contrast will help to identify whether acute inflammation is present but also the
presence of local complications such as abscess formation.

Severity Classification- Hinchey

I Para-colonic abscess

II Pelvic abscess

III Purulent peritonitis

IV Faecal peritonitis

Treatment

 Increase dietary fibre intake.


 Mild attacks of diverticulitis may be managed conservatively with antibiotics.
 Peri colonic abscesses should be drained either surgically or radiologically.
 Recurrent episodes of acute diverticulitis requiring hospitalisation are a relative indication for
a segmental resection.
 Hinchey IV perforations (generalised faecal peritonitis) will require a resection and usually a
stoma. This group have a very high risk of post operative complications and usually require
HDU admission. Less severe perforations may be managed by laparoscopic washout and
drain insertion.

Next question
Theme: Cancer management

A. Right hemicolectomy and ileocolic anastomosis

B. Extended right hemicolectomy and ileocolic anastomosis

C. Left hemicolectomy and colon-colon anastomosis

D. High anterior resection and colo-rectal anastomosis

E. Anterior resection and colo-rectal anastomosis

F. Abdomino-perineal excision of colon and rectum

G. Hartman's procedure

H. Pan proctocolectomy

Please select the most appropriate operation for the scenario given. Each option may be used once,
more than once or not at all.

64. A 28 year old man with a large (>5cm) appendiceal carcinoid tumour.

Right hemicolectomy and ileocolic anastomosis

This will require formal resection.

65. A 68 year old lady has an adenocarcinoma of the rectum that invades to the dentate line distally.

You answered Right hemicolectomy and ileocolic anastomosis

The correct answer is Abdomino-perineal excision of colon and rectum

Intestinal continuity cannot be restored here.

66. Emergency resection of perforated sigmoid tumour.

You answered Right hemicolectomy and ileocolic anastomosis


The correct answer is Hartman's procedure

Please rate this question:

Discuss and give feedback

Next question

Colorectal cancer treatment

Patients diagnosed as having colorectal cancer should be completely staged using CT of the chest/
abdomen and pelvis. Their entire colon should have been evaluated with colonoscopy or CT
colonography. Patients whose tumours lie below the peritoneal reflection should have their
mesorectum evaluated with MRI.

Once their staging is complete patients should be discussed within a dedicated colorectal MDT
meeting and a treatment plan formulated.

Treatment of colonic cancer


Cancer of the colon is nearly always treated with surgery. Stents, surgical bypass and diversion
stomas may all be used as palliative adjuncts. Resectional surgery is the only option for cure in
patients with colon cancer. The procedure is tailored to the patient and the tumour location. The
lymphatic drainage of the colon follows the arterial supply and therefore most resections are tailored
around the resection of particular lymphatic chains (e.g. ileo-colic pedicle for right sided tumours).
Some patients may have confounding factors that will govern the choice of procedure, for example a
tumour in a patient from a HNPCC family may be better served with a panproctocolectomy rather
than segmental resection. Following resection the decision has to be made regarding restoration of
continuity. For an anastomosis to heal the key technical factors include; adequate blood supply,
mucosal apposition and no tissue tension. Surrounding sepsis, unstable patients and inexperienced
surgeons may compromise these key principles and in such circumstances it may be safer to
construct an end stoma rather than attempting an anastomosis.
When a colonic cancer presents with an obstructing lesion; the options are to either stent it or resect.
In modern practice it is unusual to simply defunction a colonic tumour with a proximal loop stoma.
This differs from the situation in the rectum (see below).
Following resection patients with risk factors for disease recurrence are usually offered
chemotherapy, a combination of 5FU and oxaliplatin is common.

Rectal cancer
The management of rectal cancer is slightly different to that of colonic cancer. This reflects the
rectum's anatomical location and the challenges posed as a result. Tumours located in the rectum
can be surgically resected with either an anterior resection or an abdomino - perineal resection. The
technical aspects governing the choice between these two procedures can be complex to appreciate
and the main point to appreciate for the MRCS is that involvement of the sphincter complex or very
low tumours require APER. In the rectum a 2cm distal clearance margin is required and this may
also impact on the procedure chosen. In addition to excision of the rectal tube an integral part of the
procedure is a meticulous dissection of the mesorectal fat and lymph nodes (total mesorectal
excision/ TME). In rectal cancer surgery invovlement of the cirumferential resection margin carries a
high risk of disease recurrence. Because the rectum is an extraperitoneal structure (until you remove
it that is!) it is possible to irradiate it, something which cannot be offered for colonic tumours. This
has a major impact in rectal cancer treatment and many patients will be offered neoadjuvent
radiotherapy (both long and short course) prior to resectional surgery. Patients with T1 and 2 /N0
disease on imaging do not require irradiation and should proceed straight to surgery. Patients with
T4 disease will typically have long course chemo radiotherapy. Those with T3 , N0 tumours may be
offered short course radiotherapy prior to surgery. Patients presenting with large bowel obstruction
from rectal cancer should not undergo resectional surgery without staging as primary treatment (very
different from colonic cancer). This is because rectal surgery is more technically demanding, the
anastomotic leak rate is higher and the danger of a positive resection margin in an unstaged patient
is high. Therefore patients with obstructing rectal cancer should have a defunctioning loop
colostomy.

Summary of procedures
The operations for cancer are segmental resections based on blood supply and lymphatic drainage.
These commonly performed procedures are core knowledge for the MRCS and should be
understood.

Risk of
Site of cancer Type of resection Anastomosis leak

Right colon Right hemicolectomy Ileo-colic Low <5%

Transverse Extended right hemicolectomy Ileo-colic Low <5%

Splenic Extended right hemicolectomy Ileo-colic Low <5%


flexure

Splenic Left hemicolectomy Colo-colon 2-5%


flexure
Risk of
Site of cancer Type of resection Anastomosis leak

Left colon Left hemicolectomy Colo-colon 2-5%

Sigmoid colon High anterior resection Colo-rectal 5%

Upper rectum Anterior resection (TME) Colo-rectal 5%

Low rectum Anterior resection (Low TME) Colo-rectal 10%


(+/- Defunctioning
stoma)

Anal verge Abdomino-perineal excision of colon and None n/a


rectum

In the emergency setting, where the bowel has perforated, the risk of an anastomosis is much
greater, particularly when the anastomosis is colon-colon. In this situation, an end colostomy is often
safer and can be reversed later. When resection of the sigmoid colon is performed and an end
colostomy is fashioned the operation is referred to as a Hartmans procedure. Whilst left sided
resections are more risky, ileo-colic anastomoses are relatively safe even in the emergency setting
and do not need to be defunctioned.

References
A review of the diagnosis and management of colorectal cancer and a summary of the UK National
Institute of Clinical Excellence guidelines is provided in:
Poston G, et al . Diagnosis and management of colorectal cancer:summary of NICE
guidance. BMJ2011: 343: d 6751.

Next question
A 34 year old lady presents to her general practitioner with peri anal discomfort. The general
practitioner diagnoses pruritus ani, which of the following is least associated with the condition?

Hyperbilirubinaemia

Anal fissure

Leukaemia

Syphilis

Tuberculosis

Causes:

 Systemic (DM, Hyperbilirubinaemia, aplastic anaemia)


 Mechanical (diarrhoea, constipation, anal fissure)
 Infections (STDs)
 Dermatological
 Drugs (quinidine, colchicine)
 Topical agents

Please rate this question:

Discuss and give feedback


Next question

Ano rectal disease

Location: 3, 7, 11 o'clock position


Haemorrhoids Internal or external
Treatment: Conservative, Rubber band ligation, Haemorrhoidectomy

Fissure in ano Location: midline 6 (posterior midline 90%) and 12 o'clock position. Distal to
the dentate line
Chronic fissure > 6/52: triad: Ulcer, sentinel pile, enlarged anal papillae

Proctitis Causes: Crohn's, ulcerative colitis, Clostridium difficile

Ano rectal E.coli, staph aureus


abscess Positions: Perianal, Ischiorectal, Pelvirectal, Intersphincteric

Anal fistula Usually due to previous ano-rectal abscess


Intersphincteric, transsphincteric, suprasphincteric, and extrasphincteric.
Goodsalls rule determines location

Rectal prolapse Associated with childbirth and rectal intussceception. May be internal or
external

Pruritus ani Systemic and local causes

Anal neoplasm Squamous cell carcinoma commonest unlike adenocarcinoma in rectum

Solitary rectal Associated with chronic straining and constipation. Histology shows mucosal
ulcer thickening, lamina propria replaced with collagen and smooth muscle
(fibromuscular obliteration)

Rectal prolapse

 Common especially in multiparous women.


 May be internal or external.
 Internal rectal prolapse can present insidiously.
 External prolapse can ulcerate and in long term impair continence.
 Diagnostic work up includes colonoscopy, defecating proctogram, ano rectal manometry
studies and if doubt exists an examination under anaesthesia.

Treatments for prolapse

 In the acute setting reduce it (covering it with sugar may reduce swelling.
 Delormes procedure which excises mucosa and plicates the rectum (high recurrence rates)
may be used for external prolapse.
 Altmeirs procedure which resects the colon via the perineal route has lower recurrence rates
but carries the risk of anastamotic leak.
 Rectopexy is an abdominal procedure in which the rectum is elevated and usually supported
at the level of the sacral promontory. Post operative constipation may be reduced by limiting
the dissection to the anterior plane (laparoscopic ventral mesh rectopexy).

Pruritus ani

 Extremely common.
 Check not secondary to altered bowel habits (e.g. Diarrhoea)
 Associated with underlying diseases such as haemorrhoids.
 Examine to look for causes such as worms.
 Proctosigmoidoscopy to identify associated haemorrhoids and exclude cancer.
 Treatment is largely supportive and patients should avoid using perfumed products around
the area.

Fissure in ano

 Typically painful PR bleeding (bright red).


 Nearly always in the posterior midline.
 Usually solitary.

Treatment

 Stool softeners.
 Topical diltiazem (or GTN).
 If topical treatments fail then botulinum toxin should be injected.
 If botulinum toxin fails then males should probably undergo lateral internal sphincterotomy.
 Females who do not respond to botulinum toxin should undergo ano rectal manometry
studies and endo anal USS prior to being offered surgery such as sphincterotomy.

Next question
Theme: Fistula

A. Recurrent urinary tract infection secondary to catheter


B. Nephroenteric fistula
C. Enterovesical fistula
D. High Enterocutaneous fistula
E. Low Enterocutaneous fistula
F. Enteroenteric fistula
G. Colocutaneous fistula
H. Wound infection

Please select the most likely diagnosis for the scenario given. Each option may be used once, more
than once or not at all.

68. You are the specialist trainee asked to review a 39 year old man post gastrectomy for
bleeding duodenal ulcers. He is hypotensive and tachycardic. His drain has increased
output, contains pus and has bubbles. There is excoriated skin around the drain site.

You answered Recurrent urinary tract infection secondary to catheter

The correct answer is High Enterocutaneous fistula

Suspect an enterocutaneous fistula if there is excessive drainage and bubbles. Pus may
confuse surgeons, leading them to make a diagnosis of wound infection. If there is any
uncertainty, methylene blue can be given. If methylene blue is found in the drain, this
confirms a fistula.

69. A 43 year old female presents with recurrent urinary tract infections. She describes blood
and frothy urine. She is 6 weeks post operative for a left hemicolectomy for crohn's
disease.

You answered Recurrent urinary tract infection secondary to catheter

The correct answer is Enterovesical fistula

A nephroenteric fistula causes a chronic urinary tract infection, hence develops over a
longer time frame.

70. A 2 week infant has foul smelling material discharging from the umbilicus.

You answered Recurrent urinary tract infection secondary to catheter

The correct answer is Low Enterocutaneous fistula


This baby has an enterocutaneous fistula at the umbilicus due to complete failure of the
omphalomesenteric duct to obliterate. This is treated with resection.

Please rate this question:

Discuss and give feedback


Next question

Fistulas

 A fistula is defined as an abnormal connection between two epithelial surfaces.


 There are many types ranging from Branchial fistulae in the neck to entero-cutaneous
fistulae abdominally.
 In general surgical practice the abdominal cavity generates the majority and most of these
arise from diverticular disease and Crohn's.
 As a general rule all fistulae will resolve spontaneously as long as there is no distal
obstruction. This is particularly true of intestinal fistulae.

The four types of fistulae are:

Enterocutaneous
These link the intestine to the skin. They may be high (>500ml) or low output (<250ml) depending
upon source. Duodenal /jejunal fistulae will tend to produce high volume, electrolyte rich secretions
which can lead to severe excoriation of the skin. Colo-cutaneous fistulae will tend to leak faeculent
material. Both fistulae may result from the spontaneous rupture of an abscess cavity onto the skin
(such as following perianal abscess drainage) or may occur as a result of iatrogenic input. In some
cases it may even be surgically desirable e.g. mucous fistula following sub total colectomy for colitis.

Suspect if there is excess fluid in the drain.

Enteroenteric or Enterocolic
This is a fistula that involves the large or small intestine. They may originate in a similar manner to
enterocutaneous fistulae. A particular problem with this fistula type is that bacterial overgrowth may
precipitate malabsorption syndromes. This may be particularly serious in inflammatory bowel
disease.

Enterovaginal
Aetiology as above.

Enterovesicular
This type of fistula goes to the bladder. These fistulas may result in frequent urinary tract infections,
or the passage of gas from the urethra during urination.

Management
Some rules relating to fistula management:
 They will heal provided there is no underlying inflammatory bowel disease and no distal
obstruction, so conservative measures may be the best option
 Where there is skin involvement, protect the overlying skin, often using a well fitted stoma
bag- skin damage is difficult to treat
 A high output fistula may be rendered more easily managed by the use of octreotide, this will
tend to reduce the volume of pancreatic secretions.
 Nutritional complications are common especially with high fistula (e.g. high jejunal or
duodenal) these may necessitate the use of TPN to provide nutritional support together with
the concomitant use of octreotide to reduce volume and protect skin.
 When managing perianal fistulae surgeons should avoid probing the fistula where acute
inflammation is present, this almost always worsens outcomes.
 When perianal fistulae occur secondary to Crohn's disease the best management option is
often to drain acute sepsis and maintain that drainage through the judicious use of setons
whilst medical management is implemented.
 Always attempt to delineate the fistula anatomy, for abscesses and fistulae that have an intra
abdominal source the use of barium and CT studies should show a track. For perianal
fistulae surgeons should recall Goodsall's rule in relation to internal and external openings.

Next question
A 28 year old male presents with a discharging sinus in his natal cleft. He is found to have a pilonidal
sinus. Which statement is false?

Can occur in webs of fingers and the axilla

After drainage pilonidal abscesses should not be primarily closed

A rare complication is squamous cell carcinoma

In a patient with an acute abscess the Bascoms procedure is the treatment of choice.

Treatment involves excising or laying open the sinus tract.

When performing incision and drainage for pilonidal abscess try to avoid making the incision in the
midline of the natal cleft.

Acute pilonidal abscesses should receive simple incision and drainage. Definitive treatments such as
a Bascoms procedure should not be undertaken when acute sepsis is present.
Please rate this question:

Discuss and give feedback


Next question

Pilonidal sinus

 Occur as a result of hair debris creating sinuses in the skin (Bascom theory).
 Usually in the natal cleft of male patients after puberty.
 It is more common in Caucasians related to their hair type and growth patterns.
 The opening of the sinus is lined by squamous epithelium, but most of its wall consists of
granulation tissue. Up to 50 cases of squamous cell carcinoma have been described in
patients with chronic pilonidal sinus disease.
 Hairs become trapped within the sinus.
 Clinically the sinus presents when acute inflammation occurs, leading to an abscess.
Patients may describe cycles of being asymptomatic and periods of pain and discharge from
the sinus.
 Treatment is difficult and opinions differ. Definitive treatment should never be undertaken
when acute infection or abscess is present as this will result in failure.
 Definitive treatments include the Bascom procedure with excision of the pits and obliteration
of the underlying cavity. The Karydakis procedure involves wide excision of the natal cleft
such that the surface is recontoured once the wound is closed. This avoids the shearing
forces that break off the hairs and has reasonable results.
Pilonidal sinuses are most commonly located in the midline of the natal cleft, as illustrated below

Image sourced from Wikipedia

Next question
Theme: Colonic resections

A. Right hemicolectomy and ileocolic anastomosis

B. Extended right hemicolectomy and ileocolic anastomosis

C. Hartman's procedure

D. Defunctioning loop ileostomy only

E. Anterior resection and colo-rectal anastomosis

F. Abdomino-perineal excision of colon and rectum

Please select the most appropriate resection for the scenario given. Each option may be used once,
more than once or not at all.

72. A 58 year old man with a tumour of the splenic flexure that requires resection.

You answered Right hemicolectomy and ileocolic anastomosis

The correct answer is Extended right hemicolectomy and ileocolic anastomosis

Beware of the anatomy at the base of the middle colic which will require high ligation.

73. A 63 year old man presents with a carcinoma of the upper rectum. Staging investigations
demonstrate localised disease and he is not deemed to require and neo adjuvent therapy.

You answered Right hemicolectomy and ileocolic anastomosis

The correct answer is Anterior resection and colo-rectal anastomosis

Upper rectal tumours are usually amenable to restoration of intestinal continuity and therefore
an anterior resection is a reasonable treatment option.

74. A 66 year old lady presents with a tumour of the low rectum. There is a projection inferior to
within 1cm of the dentate line.
You answered Right hemicolectomy and ileocolic anastomosis

The correct answer is Abdomino-perineal excision of colon and rectum

Low rectal tumours such as this will require APER.

Please rate this question:

Discuss and give feedback

Next question

Colorectal cancer treatment

Patients diagnosed as having colorectal cancer should be completely staged using CT of the chest/
abdomen and pelvis. Their entire colon should have been evaluated with colonoscopy or CT
colonography. Patients whose tumours lie below the peritoneal reflection should have their
mesorectum evaluated with MRI.

Once their staging is complete patients should be discussed within a dedicated colorectal MDT
meeting and a treatment plan formulated.

Treatment of colonic cancer


Cancer of the colon is nearly always treated with surgery. Stents, surgical bypass and diversion
stomas may all be used as palliative adjuncts. Resectional surgery is the only option for cure in
patients with colon cancer. The procedure is tailored to the patient and the tumour location. The
lymphatic drainage of the colon follows the arterial supply and therefore most resections are tailored
around the resection of particular lymphatic chains (e.g. ileo-colic pedicle for right sided tumours).
Some patients may have confounding factors that will govern the choice of procedure, for example a
tumour in a patient from a HNPCC family may be better served with a panproctocolectomy rather
than segmental resection. Following resection the decision has to be made regarding restoration of
continuity. For an anastomosis to heal the key technical factors include; adequate blood supply,
mucosal apposition and no tissue tension. Surrounding sepsis, unstable patients and inexperienced
surgeons may compromise these key principles and in such circumstances it may be safer to
construct an end stoma rather than attempting an anastomosis.
When a colonic cancer presents with an obstructing lesion; the options are to either stent it or resect.
In modern practice it is unusual to simply defunction a colonic tumour with a proximal loop stoma.
This differs from the situation in the rectum (see below).
Following resection patients with risk factors for disease recurrence are usually offered
chemotherapy, a combination of 5FU and oxaliplatin is common.

Rectal cancer
The management of rectal cancer is slightly different to that of colonic cancer. This reflects the
rectum's anatomical location and the challenges posed as a result. Tumours located in the rectum
can be surgically resected with either an anterior resection or an abdomino - perineal resection. The
technical aspects governing the choice between these two procedures can be complex to appreciate
and the main point to appreciate for the MRCS is that involvement of the sphincter complex or very
low tumours require APER. In the rectum a 2cm distal clearance margin is required and this may
also impact on the procedure chosen. In addition to excision of the rectal tube an integral part of the
procedure is a meticulous dissection of the mesorectal fat and lymph nodes (total mesorectal
excision/ TME). In rectal cancer surgery invovlement of the cirumferential resection margin carries a
high risk of disease recurrence. Because the rectum is an extraperitoneal structure (until you remove
it that is!) it is possible to irradiate it, something which cannot be offered for colonic tumours. This
has a major impact in rectal cancer treatment and many patients will be offered neoadjuvent
radiotherapy (both long and short course) prior to resectional surgery. Patients with T1 and 2 /N0
disease on imaging do not require irradiation and should proceed straight to surgery. Patients with
T4 disease will typically have long course chemo radiotherapy. Those with T3 , N0 tumours may be
offered short course radiotherapy prior to surgery. Patients presenting with large bowel obstruction
from rectal cancer should not undergo resectional surgery without staging as primary treatment (very
different from colonic cancer). This is because rectal surgery is more technically demanding, the
anastomotic leak rate is higher and the danger of a positive resection margin in an unstaged patient
is high. Therefore patients with obstructing rectal cancer should have a defunctioning loop
colostomy.

Summary of procedures
The operations for cancer are segmental resections based on blood supply and lymphatic drainage.
These commonly performed procedures are core knowledge for the MRCS and should be
understood.

Risk of
Site of cancer Type of resection Anastomosis leak

Right colon Right hemicolectomy Ileo-colic Low <5%

Transverse Extended right hemicolectomy Ileo-colic Low <5%

Splenic Extended right hemicolectomy Ileo-colic Low <5%


flexure
Risk of
Site of cancer Type of resection Anastomosis leak

Splenic Left hemicolectomy Colo-colon 2-5%


flexure

Left colon Left hemicolectomy Colo-colon 2-5%

Sigmoid colon High anterior resection Colo-rectal 5%

Upper rectum Anterior resection (TME) Colo-rectal 5%

Low rectum Anterior resection (Low TME) Colo-rectal 10%


(+/- Defunctioning
stoma)

Anal verge Abdomino-perineal excision of colon and None n/a


rectum

In the emergency setting, where the bowel has perforated, the risk of an anastomosis is much
greater, particularly when the anastomosis is colon-colon. In this situation, an end colostomy is often
safer and can be reversed later. When resection of the sigmoid colon is performed and an end
colostomy is fashioned the operation is referred to as a Hartmans procedure. Whilst left sided
resections are more risky, ileo-colic anastomoses are relatively safe even in the emergency setting
and do not need to be defunctioned.

References
A review of the diagnosis and management of colorectal cancer and a summary of the UK National
Institute of Clinical Excellence guidelines is provided in:
Poston G, et al . Diagnosis and management of colorectal cancer:summary of NICE
guidance. BMJ2011: 343: d 6751.

Next question
A 23 year old lady presents with a posteriorly sited fissure in ano. Treatment with stool softeners and
topical GTN has failed to improve matters. Which of the following would be the most appropriate
next management step?

Lords procedure

Fissurectomy and injection of botulinum toxin

Lateral internal sphincterotomy

Endoanal advancement flap

Surgical division of the external anal sphincter

The next most appropriate management option when GTN or other topical nitrates has failed is to
consider botulinum toxin injection. In males a lateral internal sphincterotomy would be an acceptable
alternative. In a female who has yet to conceive this may predispose to delayed increased risk of
sphincter dysfunction. Division of the external sphincter will result in faecal incontinence and is not a
justified treatment for fissure.
Please rate this question:

Discuss and give feedback

Rectal bleeding

Rectal bleeding is a common cause for patients to be referred to the surgical clinic. In the clinical
history it is useful to try and localise the anatomical source of the blood. Bright red blood is usually of
rectal anal canal origin, whilst dark red blood is more suggestive of a proximally sited bleeding
source. Blood which has entered the GI tract from a gastro-duodenal source will typically resemble
malaena due to the effects of the digestive enzymes on the blood itself.

In the table below we give some typical bleeding scenarios together with physical examination
findings and causation.

Cause Type of Features in history Examination findings


bleeding

Fissure in Bright red Painful bleeding that occurs Muco-epithelial defect usually
rectal post defecation in small in the midline posteriorly
ano bleeding volumes. Usually antecedent (anterior fissures more likely to
features of constipation be due to underlying disease)

Haemorroids Bright red Post defecation bleeding noted Normal colon and rectum.
rectal both on toilet paper and drips Proctoscopy may show internal
bleeding into pan. May be alteration of haemorrhoids. Internal
bowel habit and history of haemorrhoids are usually
straining. No blood mixed with impalpable.
stool. No local pain.

Crohns Bright red or Bleeding that is accompanied Perineal inspection may show
disease mixed blood by other symptoms such as fissures or fistulae. Proctoscopy
altered bowel habit, malaise, may demonstrate indurated
history of fissures (especially mucosa and possibly strictures.
anterior) and abscesses. Skip lesions may be noted at
colonoscopy.

Ulcerative Bright red Diarrhoea, weight loss, Proctitis is the most marked
colitis bleeding nocturnal incontinence, passage finding. Peri anal disease is
often mixed of mucous PR. usually absent. Colonoscopy
with stool will show continuous mucosal
lesion.

Rectal cancer Bright red Alteration of bowel habit. Usually obvious mucosal
blood mixed Tenesmus may be present. abnormality. Lesion may be
volumes Symptoms of metastatic fixed or mobile depending upon
disease. disease extent. Surrounding
mucosa often normal, although
polyps may be present.

Image showing a fissure in ano. Typically these are located posteriorly and in the midline. Fissures
at other sites may be associated with underlying disease.
Image sourced from Wikipedia

Colonoscopic image of internal haemorroids. Note these may often be impalpable.

Image sourced from Wikipedia

Investigation

 All patients presenting with rectal bleeding require digital rectal examination and procto-
sigmoidoscopy as a minimal baseline.
 Remember that haemorrhoids are typically impalpable and to attribute bleeding to these in
the absence of accurate internal inspection is unsatisfactory.
 In young patients with no other concerning features in the history a carefully performed
sigmoidoscopy that demonstrates clear haemorrhoidal disease may be sufficient. If clear
views cannot be obtained then patients require bowel preparation with an enema and a
flexible sigmoidscopy performed.
 In those presenting with features of altered bowel habit or suspicion of inflammatory bowel
disease a colonoscopy is the best test.
 Patients with excessive pain who are suspected of having a fissure may require an
examination under general or local anaesthesia.
 In young patients with external stigmata of fissure and a compatible history it is acceptable to
treat medically and defer internal examination until the fissure is healed. If the fissure fails to
heal then internal examination becomes necessary along the lines suggested above to
exclude internal disease.

Special tests

 In patients with a malignancy of the rectum the staging investigations comprise an MRI of the
rectum to identify circumferential resection margin compromise and to identify mesorectal
nodal disease. In addition to this CT scanning of the chest abdomen and pelvis is necessary
to stage for more distant disease. Some centres will still stage the mesorectum with endo
rectal ultrasound but this is becoming far less common.

 Patients with fissure in ano who are being considered for surgical sphincterotomy and are
females who have an obstetric history should probably have ano rectal manometry testing
performed together with endo anal ultrasound. As this service is not universally available it is
not mandatory but in the absence of such information there are continence issues that may
arise following sphincterotomy.

Management

Disease Management

Fissure in ano GTN ointment 0.2% or diltiazem cream applied topically is the usual first line
treatment. Botulinum toxin for those who fail to respond. Internal
sphincterotomy for those who fail with botox, can be considered earlier in
males.

Haemorroids Lifestyle advice, for small internal haemorrhoids can consider injection
sclerotherapy or rubber band ligation. For external haemorrhoids consider
haemorrhoidectomy. Modern options include HALO procedure and stapled
haemorrhoidectomy.

Inflammatory Medical management- although surgery may be needed for fistulating Crohns
bowel disease (setons).

Rectal cancer Anterior resection or abdomino-perineal excision of the colon and rectum.
Total mesorectal excision is now standard of care. Most resections below the
peritoneal reflection will require defunctioning ileostomy. Most patients will
require preoperative radiotherapy.
A 21 year old man is involved in a road traffic accident. After a transient period of concussion he is
found to have a GCS of 15 by the paramedics. On arrival at hospital he is monitored in a side room
of the emergency department. When he is next observed he is noted to have a GCS of 3 and a
blown right pupil. Which of the processes below best accounts for this deterioration?

Hydrocephalus

Intraventricular bleed

Sub dural bleed

Trans tentorial herniation

Sub arachnoid haemorrhage

Theme from April 2012 Exam


The presence of a blown right pupil is a sign of a third cranial nerve compression. The most likely
cause is an extradural bleed. However, since this option is not listed the process of trans tentorial
herniation would be the most applicable answer. Intraventricular bleeds are typically more common
in premature neonates, deterioration due to hydrocephalus is more chronic.
Please rate this question:

Discuss and give feedback


Next question

Head injuries

Head injury is the commonest cause of death and disability in people aged 1-40 years in the UK. In
the UK 1.4 million people will attend emergency departments each year with a recent head injury.
The typical patterns are described below:

Extradural Bleeding into the space between the dura mater and the skull. Often results
haematoma from acceleration-deceleration trauma or a blow to the side of the head. The
majority of extradural haematomas occur in the temporal region where skull
fractures cause a rupture of the middle meningeal artery.

Features

 Raised intracranial pressure


 Some patients may exhibit a lucid interval

Subdural Bleeding into the outermost meningeal layer. Most commonly occur around
haematoma the frontal and parietal lobes. May be either acute or chronic.

Risk factors include old age and alcoholism.

Slower onset of symptoms than a extradural haematoma.

Subarachnoid Usually occurs spontaneously in the context of a ruptured cerebral aneurysm


haemorrhage but may be seen in association with other injuries when a patient has
sustained a traumatic brain injury
Theme: Intracranial bleeds

A. Intraventricular haemorrhage
B. Chronic sub dural bleed
C. Acute sub dural bleed
D. Extra dural haemorrhage
E. Sub arachnoid haemorrhage

Please select the most likely underlying diagnosis for the scenario described. Each option may be
used once, more than once or not at all.

2. A 50 year old alcoholic man attends the emergency department. His main reason for
presenting is that he has no home to go to. On examination he has no evidence of
involvement in recent trauma, a skull x-ray fails to show any evidence of skull fracture. He
is admitted and twelve hours following admission he develops sudden onset headache,
becomes comatose and then dies.

You answered Intraventricular haemorrhage

The correct answer is Sub arachnoid haemorrhage

Theme from April 2013 Exam


The absence of trauma here makes an acute sub dural and extra dural bleed unlikely.
Chronic sub dural bleeds would usually cause a more gradual deterioration than is seen
here. The absence of any skull fracture also makes an underlying intra cranial bleed less
likely. Sudden onset headaches, together with sudden deterioration in neurological function
are typical of a sub arachnoid haemorrhage.

3. A 50 year old lady is admitted having fallen down some stairs sustaining multiple rib
fractures 36 hours previously. On examination she is confused and agitated and has clinical
evidence of lateralising signs. She deteriorates further and then dies with no response to
resuscitation.

You answered Intraventricular haemorrhage

The correct answer is Acute sub dural bleed

The time frame of deterioration of an acute sub dural bleed would fit with this scenario.
They are highly lethal and not uncommon injuries. As the bleed enlarges, lateralising signs
may be seen and eventually coning and death will occur.

4. A male infant is born at 28 weeks gestation by emergency cesarean section. He is taken to


theatre for a colostomy due to an imperforate anus. He initially seems to be progressing
well. However, he begins to develop decerebrate posturing and is becoming increasingly
obtunded.

Intraventricular haemorrhage

Acute neurological deterioration in premature neonates is usually due to intraventricular


haemorrhage. Diagnosis is made by cranial ultrasound. Development of hydrocephalus may
necessitate surgery.

Please rate this question:

Discuss and give feedback


Next question

Intra cranial haemorrhage

Extradural Bleeding into the space between the dura mater and the skull. Often results
haematoma from acceleration-deceleration trauma or a blow to the side of the head. The
majority of extradural haematomas occur in the temporal region where skull
fractures cause a rupture of the middle meningeal artery.

Features

 Raised intracranial pressure


 Some patients may exhibit a lucid interval

Subdural Bleeding into the outermost meningeal layer. Most commonly occur around
haematoma the frontal and parietal lobes. May be either acute or chronic.

Risk factors include old age and alcoholism.

Slower onset of symptoms than a extradural haematoma.

Intracerebral Usually hyperdense lesions on CT scanning. Arise in areas of traumatic


haematoma contusion with fuse to become a haematoma. Areas of clot and fresh blood
may co-exist on the same CT scan (Swirl sign). Large haematomas and
those associated with mass effect should be evacuated.

Subarachnoid Usually occurs spontaneously in the context of a ruptured cerebral


haemorrhage aneurysm but may be seen in association with other injuries when a patient
has sustained a traumatic brain injury
Intraventricular Haemorrhage that occurs into the ventricular system of the brain. It is
haemorrhage relatively rare in adult surgical practice and when it does occur, it is
typically associated with severe head injuries. In premature neonates it may
occur spontaneously. The blood may clot and occlude CSF flow,
hydrocephalus may result.
In neonatal practice the vast majority of IVH occur in the first 72 hours
after birth, the aetiology is not well understood and it is suggested to occur
as a result of birth trauma combined with cellular hypoxia, together with the
delicate neonatal CNS.
Theme: Cranial nerve lesions

A. Optic nerve

B. Oculomotor nerve

C. Trigeminal nerve

D. Facial nerve

E. Abducens nerve

F. Glossopharyngeal nerve

G. Vestibulocochlear nerve

H. Accessory nerve

I. Hypoglossal nerve

For each of the scenarios given please give the most likely cranial nerve responsible for the
symptom or lesion described. Each nerve may be used once, more than once or not at all.

5. A 63 year old man is admitted with severe headache, nausea and recent epileptic fit. Fundoscopy
shows papilloedema. He is also noted to have diplopia.

You answered Optic nerve

The correct answer is Abducens nerve

The long intracranial course of this nerve makes it susceptible to damage early in the course of
raised ICP.

6. A 32 year old lady is admitted with weakness, visual disturbance and peri orbital pain. On
examination, she is noted to have mydriasis and diminished direct response to light shone into the
affected eye. The consensual response is preserved when light is shone into the unaffected eye.

Optic nerve

This describes a relative afferent pupillary defect (RAPD). RAPD is a defect in the direct response to
light. It is due to damage in optic nerve or severe retinal disease. If an optic nerve lesion is present
the affected pupil will not constrict to light when light is shone in the that pupil during the swinging
flashlight test. However, it will constrict if light is shone in the other eye (consensual response).

The most likely cause for this is an optic neuritis (not really surgical!). Other causes include
ischemic optic disease or retinal disease, severe glaucoma causing trauma to optic nerve and direct
optic nerve damage (trauma, radiation, tumor).

7. An 18 year old boy undergoes an uncomplicated tonsillectomy for recurrent attacks of tonsillitis.
Post operatively he complains of otalgia.

You answered Optic nerve

The correct answer is Glossopharyngeal nerve

The glossopharyngeal nerve supplies this area and the ear and otalgia may be the result of referred
pain.

Please rate this question:

Discuss and give feedback

Next question

Cranial nerves

Cranial nerve lesions

Olfactory nerve May be injured in basal skull fractures or involved in frontal lobe tumour extension.
Loss of olfactory nerve function in relation to major CNS pathology is seldom an
isolated event and thus it is poor localiser of CNS pathology.

Optic nerve Problems with visual acuity may result from intra ocular disorders. Problems with
the blood supply such as amaurosis fugax may produce temporary visual distortion.
More important surgically is the pupillary response to light. The pupillary size may
be altered in a number of disorders. Nerves involved in the resizing of the pupil
connect to the pretectal nucleus of the high midbrain, bypassing the lateral
geniculate nucleus and the primary visual cortex. From the pretectal nucleus
neurones pass to the Edinger - Westphal nucleus, motor axons from here pass along
with the oculomotor nerve. They synapse with ciliary ganglion neurones; the
parasympathetic axons from this then innervate the iris and produce miosis. The
miotic pupil is seen in disorders such as Horner's syndrome or opiate overdose.
Mydriasis is the dilatation of the pupil in response to disease, trauma, drugs (or the
dark!). It is pathological when light fails to induce miosis. The radial muscle is
innervated by the sympathetic nervous system. Because the parasympathetic fibres
travel with the oculomotor nerve they will be damaged by lesions affecting this
nerve (e.g. cranial trauma).
The response to light shone in one eye is usually a constriction of both pupils. This
indicates intact direct and consensual light reflexes. When the optic nerve has an
afferent defect the light shining on the affected eye will produce a diminished
pupillary response in both eyes. Whereas light shone on the unaffected eye will
produce a normal pupillary response in both eyes. This is referred to as the Marcus
Gunn pupil and is seen in conditions such as optic neuritis. In a total CN II lesion
shining the light in the affected eye will produce no response.

Oculomotor nerve The pupillary effects are described above. In addition it supplies all ocular muscles
apart from lateral rectus and superior oblique. Thus the affected eye will be
deviated inferolaterally. Levator palpebrae superioris may also be impaired resulting
in impaired ability to open the eye.

Trochlear nerve The eye will not be able to look down.

Trigeminal nerve Largest cranial nerve. Exits the brainstem at the pons. Branches are ophthalmic,
maxillary and mandibular. Only the mandibular branch has both sensory and motor
fibres. Branches converge to form the trigeminal ganglion (located in Meckels cave).
It supplies the muscles of mastication and also tensor veli palatine, mylohyoid,
anterior belly of digastric and tensor tympani. The detailed descriptions of the
various sensory functions are described in other areas of the website. The corneal
reflex is important and is elicited by applying a small tip of cotton wool to the
cornea, a reflex blink should occur if it is intact. It is mediated by: the naso ciliary
branch of the ophthalmic branch of the trigeminal (sensory component) and the
facial nerve producing the motor response. Lesions of the afferent arc will produce
bilateral absent blink and lesions of the efferent arc will result in a unilateral absent
blink.

Abducens nerve The affected eye will have a deficit of abduction. This cranial nerve exits the
brainstem between the pons and medulla. It thus has a relatively long intra cranial
course which renders it susceptible to damage in raised intra cranial pressure.

Facial nerve Emerges from brainstem between pons and medulla. It controls muscles of facial
expression and taste from the anterior 2/3 of the tongue. The nerve passes into the
petrous temporal bone and into the internal auditory meatus. It then passes
through the facial canal and exits at the stylomastoid foramen. It passes through the
parotid gland and divides at this point. It does not innervate the parotid gland. Its
divisions are considered in other parts of the website. Its motor fibres innervate
orbicularis oculi to produce the efferent arm of the corneal reflex. In surgical
practice it may be injured during parotid gland surgery or invaded by malignancies
of the gland and a lower motor neurone on the ipsilateral side will result.

Vestibulo-cochlear Exits from the pons and then passes through the internal auditory meatus. It is
nerve implicated in sensorineural hearing loss. Individuals with sensorineural hearing loss
will localise the sound in webers test to the normal ear. Rinnes test will be reduced
on the affected side but should still work. These two tests will distinguish
sensorineural hearing loss from conductive deafness. In the latter condition webers
test will localise to the affected ear and Rinnes test will be impaired on the affected
side. Surgical lesions affecting this nerve include CNS tumours and basal skull
fractures. It may also be damaged by the administration of ototoxic drugs (of which
gentamicin is the most commonly used in surgical practice).

Glossopharyngeal Exits the pons just above the vagus. Receives sensory fibres from posterior 1/3
nerve tongue, tonsils, pharynx and middle ear (otalgia may occur following tonsillectomy).
It receives visceral afferents from the carotid bodies. It supplies parasympathetic
fibres to the parotid gland via the otic ganglion and motor function to
stylopharyngeaus muscle. The sensory function of the nerve is tested using the gag
reflex.

Vagus nerve Leaves the medulla between the olivary nucleus and the inferior cerebellar
peduncle. Passes through the jugular foramen and into the carotid sheath. Details of
the functions of the vagus nerve are covered in the website under relevant organ
sub headings.

Accessory nerve Exists from the caudal aspect of the brainstem (multiple branches) supplies
trapezius and sternocleidomastoid muscles. The distal portion of this nerve is most
prone to injury during surgical procedures.

Hypoglossal nerve Emerges from the medulla at the preolivary sulcus, passes through the hypoglossal
canal. It lies on the carotid sheath and passes deep to the posterior belly of digastric
to supply muscles of the tongue (except palatoglossus). Its location near the carotid
sheath makes it vulnerable during carotid endarterectomy surgery and damage will
produce ipsilateral defect in muscle function.
Theme: Head injury- assessment

A. 3
B. 4
C. 6
D. 8
E. 10
F. 15
G. 12
H. 13
I. 5
J. 7

For each of the scenarios given please determine the most likely Glasgow Coma Score. Each option
may be used once, more than once or not at all.

8. A 20 year old man is hit over the head with a mallet. On arrival in the accident and
emergency department he opens his eyes to pain and groans or grunts. On application of a
painful stimulus to his hands, he extends his arm at the elbow.

You answered 3

The correct answer is 6

Theme from 2011 Exam


Theme from September 2012 Exam
E=2, V= 2, M=2.

9. A 20 year old man falls over and bangs his head whilst intoxicated. On arrival in the
emergency department he opens his eyes in response to speech, and is able to speak,
although he is disorientated. He obeys motor commands.

You answered 3

The correct answer is 13

E=3, V=4, M=6.

10. A 20 year old man is hit over the head with an iron bar. On arrival in the emergency
department he opens his eyes in response to pain, his only verbal responses are in the form
of groans and grunts. On application of a painful stimulus to his hands, he flexes his
forearms away from the painful stimuli.
You answered 3

The correct answer is 8

E=2, V=2, M=4.


Appropriate flexion to pain carries a higher score than decorticate posturing or
inappropriate flexion.

Please rate this question:

Discuss and give feedback


Next question

Glasgow coma scale

Modality Options

Eye opening  Spontaneous


 To speech
 To pain
 None

Verbal response  Orientated


 Confused
 Words
 Sounds
 None

Motor response  Obeys commands


 Localises to pain
 Withdraws from pain
 Abnormal flexion to pain (decorticate posture)
 Extending to pain
 None

Responses are taken from each category (marks in descending order) to produce an overall score.
Severe brain injuries are generally associated with GCS <8.
A 33 year old lady develops a thunderclap headache and collapses. A CT scan shows that she has
developed a subarachnoid haemorrhage. She currently has no evidence of raised intracranial
pressure. Which of the following drugs should be administered?

None

Atenotol

Labetolol

Nimodipine

Mannitol

Theme from 2007 Exam


Nimodipine is a calcium channel blocker. It reduces cerebral vasospasm and improves outcomes. It
is administered to most cases of sub arachnoid haemorrhage.

Please rate this question:

Discuss and give feedback

Next question

Sub arachnoid haemorrhage

Spontaneous intracranial haemorrhage


Most commonly sub arachnoid haemorrhage. It is due to intra cranial aneurysm in 85% cases.
Approximately 10% of cases will have normal angiography and the cause will remain unclear.
Patients with inherited connective tissue disorders are at higher risk although most cases are
sporadic.
>95% cases will have headache (often thunderclap)
>15% will have coma

Investigation
CT scan for all (although as CSF blood clears the sensitivity declines)
Lumbar puncture if CT normal (very unlikely if normal)
CT angiogram to look for aneurysms.

Management
Supportive treatment, optimising BP (not too high if untreated aneurysm) and ventilation if needed.
Nimodipine reduces cerebral vasospasm and reduces poor outcomes.
Untreated patients most likely to rebleed in first 2 weeks.
Patients developing hydrocephalus will need a V-P shunt (external ventricular drain acutely).
Electrolytes require careful monitoring and hyponatraemia is common.

Treatment of aneurysm
>80% aneuryms arise from the anterior circulation
Craniotomy and clipping of aneurysm is standard treatment, alternatively suitable lesions may be
coiled using an endovascular approach. Where both options are suitable data suggests that
outcomes are better with coiling than surgery.
A 65 year old male with known nasopharyngeal carcinoma presents with double vision over a few
weeks. On examination he is found to have left eye proptosis and it is down and out. He reports pain
on attempting to move the eye. There is an absent corneal reflex. What is the most likely diagnosis?

Posterior communicating artery aneurysm

Cavernous sinus syndrome

Optic nerve tumour

Migraine

Cerebral metastases

Cavernous sinus syndrome is most commonly caused by cavernous sinus tumours. In this case, the
nasopharyngeal malignancy has locally invaded the left cavernous sinus. Diagnosis is based on
signs of pain, opthalmoplegia, proptosis, trigeminal nerve lesion (opthalmic branch) and Horner's
syndrome.

Please rate this question:

Discuss and give feedback

Next question

Cavernous sinus

The cavernous sinuses are paired and are situated on the body of the sphenoid bone. It runs from
the superior orbital fissure to the petrous temporal bone.

Relations

Medial Lateral
Pituitary fossa
Temporal lobe
Sphenoid sinus

Contents

Lateral wall components (from top to bottom:)


Oculomotor nerve
Trochlear nerve
Ophthalmic nerve
Maxillary nerve

Contents of the sinus (from medial to lateral:)


Internal carotid artery (and sympathetic plexus)
Abducens nerve

Blood supply
Ophthalmic vein, superficial cortical veins, basilar plexus of veins posteriorly.

Drains into the internal jugular vein via: the superior and inferior petrosal sinuses

Image sourced from Wikipedia


Theme: Head injury

A. Subdural haematoma
B. Extradural haematoma
C. Subarachnoid haemorrhage
D. Basal skull fracture
E. Intracerebral haematoma
F. Le fort 1 fracture of maxilla
G. Le fort fracture 3 of maxilla
H. Mandibular fracture

What is the most likely diagnosis for the scenario given? Each option may be used once, more than
once or not at all.

13. A 32 year old female hits her head on the steering wheel during a collision with another
car. She has periorbital swelling and a flattened appearance of the face.

You answered Subdural haematoma

The correct answer is Le fort fracture 3 of maxilla

The flattened appearance of the face is a classical description of the dish/pan face
associated with Le fort fracture 2 or 3 of the maxilla.

14. A 29 year bouncer is hit on the side of the head with a bat. He now presents to A&E with
odd behaviour and complaining of a headache. Whilst waiting for a CT scan he becomes
drowsy and unresponsive.

You answered Subdural haematoma

The correct answer is Extradural haematoma

The middle meningeal artery is prone to damage when the temporal side of the head is hit.
Note that there may NOT be any initial LOC or lucid interval.

15. A 40 year old alcoholic presents with worsening confusion over 2 weeks. He has weakness
of the left side of the body.

Subdural haematoma

Subdural haematomas can have a history over weeks/months. It is common in alcoholics


due to cerebral atrophy causing increased stretching of veins.
Please rate this question:

Discuss and give feedback


Next question

Head injury

Patients who suffer head injuries should be managed according to ATLS principles and extra cranial
injuries should be managed alongside cranial trauma. Inadequate cardiac output will compromise
CNS perfusion irrespective of the nature of the cranial injury.

Types of traumatic brain injury


Bleeding into the space between the dura mater and the skull. Often results from
acceleration-deceleration trauma or a blow to the side of the head. The majority
of extradural haematomas occur in the temporal region where skull fractures
cause a rupture of the middle meningeal artery.

Extradural
Features
haematoma

 Raised intracranial pressure


 Some patients may exhibit a lucid interval

Bleeding into the outermost meningeal layer. Most commonly occur around the
frontal and parietal lobes. May be either acute or chronic.
Subdural
haematoma Risk factors include old age and alcoholism.

Slower onset of symptoms than a extradural haematoma.

Usually occurs spontaneously in the context of a ruptured cerebral aneurysm, but


Subarachnoid
may be seen in association with other injuries when a patient has sustained a
haemorrhage
traumatic brain injury.

Pathophysiology

 Primary brain injury may be focal (contusion/ haematoma) or diffuse (diffuse axonal injury)
 Diffuse axonal injury occurs as a result of mechanical shearing following deceleration,
causing disruption and tearing of axons
 Intra-cranial haematomas can be extradural, subdural or intracerebral, while contusions may
occur adjacent to (coup) or contralateral (contre-coup) to the side of impact
 Secondary brain injury occurs when cerebral oedema, ischaemia, infection, tonsillar or
tentorial herniation exacerbates the original injury. The normal cerebral auto regulatory
processes are disrupted following trauma rendering the brain more susceptible to blood flow
changes and hypoxia
 The Cushings reflex (hypertension and bradycardia) often occurs late and is usually a pre
terminal event
Management

 Where there is life threatening rising ICP such as in extra dural haematoma and whilst
theatre is prepared or transfer arranged use of IV mannitol/ frusemide may be required.
 Diffuse cerebral oedema may require decompressive craniotomy
 Exploratory Burr Holes have little management in modern practice except where scanning
may be unavailable and to thus facilitate creation of formal craniotomy flap
 Depressed skull fractures that are open require formal surgical reduction and debridement,
closed injuries may be managed non operatively if there is minimal displacement.
 ICP monitoring is appropriate in those who have GCS 3-8 and normal CT scan.
 ICP monitoring is mandatory in those who have GCS 3-8 and abnormal CT scan.
 Hyponatraemia is most likely to be due to syndrome of inappropriate ADH secretion.
 Minimum of cerebral perfusion pressure of 70mmHg in adults.
 Minimum cerebral perfusion pressure of between 40 and 70 mmHg in children.

Interpretation of pupillary findings in head injuries


Pupil size Light response Interpretation

Unilaterally dilated Sluggish or fixed 3rd nerve compression secondary to tentorial


herniation

Bilaterally dilated Sluggish or fixed  Poor CNS perfusion


 Bilateral 3rd nerve palsy

Unilaterally dilated or Cross reactive (Marcus - Optic nerve injury


equal Gunn)

Bilaterally constricted May be difficult to  Opiates


assess  Pontine lesions
 Metabolic encephalopathy

Unilaterally Preserved Sympathetic pathway disruption


constricted
Next question
A 28 year old women collapses and is found to be deeply comatose with a GCS of 3. She is
admitted to hospital, intubated and ventilated. A CT scan shows an extensive sub arachnoid
haemorrhage. A urinary catheter is inserted. What type of urine output would be expected?

85ml concentrated urine per hour

30ml diluted urine per hour

60ml diluted urine per hour

300ml concentrated urine per hour

300ml diluted urine per hour

Theme from 2011 Exam


An extensive CNS insult is likely to result in cerebral salt wasting (low Na, low plasma osmolality and
high urine output) or centrally mediated diabetes insipidus (high Na, high plasma osmolality and high
urine output). This may either be the result of hypothalamic injury directly or the result of disruption
of the hypothalamic-pituitary connections. The result is a large volume diuresis with diluted urine.

Please rate this question:

Discuss and give feedback

Next question

Sub arachnoid haemorrhage

Spontaneous intracranial haemorrhage


Most commonly sub arachnoid haemorrhage. It is due to intra cranial aneurysm in 85% cases.
Approximately 10% of cases will have normal angiography and the cause will remain unclear.
Patients with inherited connective tissue disorders are at higher risk although most cases are
sporadic.
>95% cases will have headache (often thunderclap)
>15% will have coma
Investigation
CT scan for all (although as CSF blood clears the sensitivity declines)
Lumbar puncture if CT normal (very unlikely if normal)
CT angiogram to look for aneurysms.

Management
Supportive treatment, optimising BP (not too high if untreated aneurysm) and ventilation if needed.
Nimodipine reduces cerebral vasospasm and reduces poor outcomes.
Untreated patients most likely to rebleed in first 2 weeks.
Patients developing hydrocephalus will need a V-P shunt (external ventricular drain acutely).
Electrolytes require careful monitoring and hyponatraemia is common.

Treatment of aneurysm
>80% aneuryms arise from the anterior circulation
Craniotomy and clipping of aneurysm is standard treatment, alternatively suitable lesions may be
coiled using an endovascular approach. Where both options are suitable data suggests that
outcomes are better with coiling than surgery.

Next question
A patient is referred due to the development of a third nerve palsy associated with a headache. On
examination meningism is present. Which one of the following diagnoses needs to be urgently
excluded?

Weber's syndrome

Internal carotid artery aneurysm

Multiple sclerosis

Posterior communicating artery aneurysm

Anterior communicating artery aneurysm

Painful third nerve palsy = posterior communicating artery aneurysm

Given the combination of a headache and third nerve palsy it is important to exclude a posterior
communicating artery aneurysm
Please rate this question:

Discuss and give feedback


Next question

Third nerve palsy

Features

 eye is deviated 'down and out'


 ptosis
 pupil may be dilated (sometimes called a 'surgical' third nerve palsy)

Causes

 diabetes mellitus
 vasculitis e.g. temporal arteritis, SLE
 false localizing sign* due to uncal herniation through tentorium if raised ICP
 posterior communicating artery aneurysm (pupil dilated)
 cavernous sinus thrombosis
 Weber's syndrome: ipsilateral third nerve palsy with contralateral hemiplegia -caused by
midbrain strokes
 other possible causes: amyloid, multiple sclerosis

*this term is usually associated with sixth nerve palsies but it may be used for a variety of
neurological presentations
A 23 year old man was driving a car at high speed whilst intoxicated, he was wearing a seat belt.
The car collides with a brick wall at around 140km/h. When he arrives in the emergency department
he is comatose. His CT scan appears to be normal. He remains in a persistent vegetative state.
What is the most likely underlying cause?

Extradural haemorrhage

Sub dural haemorrhage

Sub arachnoid haemorrhage

Intracerebral haemorrhage

Diffuse axonal injury

Diffuse axonal injury occurs when the head is rapidly accelerated or decelerated. There are 2
components:

1. Multiple haemorrhages
2. Diffuse axonal damage in the white matter

Up to 2/3 occur at the junction of grey/white matter due to the different densities of the tissue. The
changes are mainly histological and axonal damage is secondary to biochemical cascades. Often
there are no signs of a fracture or contusion.
Please rate this question:

Discuss and give feedback


Next question

Head injury

Patients who suffer head injuries should be managed according to ATLS principles and extra cranial
injuries should be managed alongside cranial trauma. Inadequate cardiac output will compromise
CNS perfusion irrespective of the nature of the cranial injury.

Types of traumatic brain injury


Bleeding into the space between the dura mater and the skull. Often results from
acceleration-deceleration trauma or a blow to the side of the head. The majority
Extradural
of extradural haematomas occur in the temporal region where skull fractures
haematoma
cause a rupture of the middle meningeal artery.
Features

 Raised intracranial pressure


 Some patients may exhibit a lucid interval

Bleeding into the outermost meningeal layer. Most commonly occur around the
frontal and parietal lobes. May be either acute or chronic.
Subdural
haematoma Risk factors include old age and alcoholism.

Slower onset of symptoms than a extradural haematoma.

Usually occurs spontaneously in the context of a ruptured cerebral aneurysm, but


Subarachnoid
may be seen in association with other injuries when a patient has sustained a
haemorrhage
traumatic brain injury.

Pathophysiology

 Primary brain injury may be focal (contusion/ haematoma) or diffuse (diffuse axonal injury)
 Diffuse axonal injury occurs as a result of mechanical shearing following deceleration,
causing disruption and tearing of axons
 Intra-cranial haematomas can be extradural, subdural or intracerebral, while contusions may
occur adjacent to (coup) or contralateral (contre-coup) to the side of impact
 Secondary brain injury occurs when cerebral oedema, ischaemia, infection, tonsillar or
tentorial herniation exacerbates the original injury. The normal cerebral auto regulatory
processes are disrupted following trauma rendering the brain more susceptible to blood flow
changes and hypoxia
 The Cushings reflex (hypertension and bradycardia) often occurs late and is usually a pre
terminal event

Management

 Where there is life threatening rising ICP such as in extra dural haematoma and whilst
theatre is prepared or transfer arranged use of IV mannitol/ frusemide may be required.
 Diffuse cerebral oedema may require decompressive craniotomy
 Exploratory Burr Holes have little management in modern practice except where scanning
may be unavailable and to thus facilitate creation of formal craniotomy flap
 Depressed skull fractures that are open require formal surgical reduction and debridement,
closed injuries may be managed non operatively if there is minimal displacement.
 ICP monitoring is appropriate in those who have GCS 3-8 and normal CT scan.
 ICP monitoring is mandatory in those who have GCS 3-8 and abnormal CT scan.
 Hyponatraemia is most likely to be due to syndrome of inappropriate ADH secretion.
 Minimum of cerebral perfusion pressure of 70mmHg in adults.
 Minimum cerebral perfusion pressure of between 40 and 70 mmHg in children.
Interpretation of pupillary findings in head injuries
Pupil size Light response Interpretation

Unilaterally dilated Sluggish or fixed 3rd nerve compression secondary to tentorial


herniation

Bilaterally dilated Sluggish or fixed  Poor CNS perfusion


 Bilateral 3rd nerve palsy

Unilaterally dilated or Cross reactive (Marcus - Optic nerve injury


equal Gunn)

Bilaterally constricted May be difficult to  Opiates


assess  Pontine lesions
 Metabolic encephalopathy

Unilaterally Preserved Sympathetic pathway disruption


constricted
Next question
Theme: Visual defects

A. Right homonymous hemianopia


B. Left homonymous hemianopia
C. Right superior quadranopia
D. Left superior quadranopia
E. Right inferior quadranopia
F. Left inferior quadranopia
G. Upper bitemporal hemianopia
H. Lower bitemporal hemianopia

What is the most likely visual field defect for the scenario given? Each option may be used once,
more than once or not at all.

19. A 42 year old woman is admitted to the vascular ward for an endarterectomy. Her CT
report confirms a left temporal lobe infarct.

You answered Right homonymous hemianopia

The correct answer is Right superior quadranopia

Temporal lesions cause a contralateral superior quadranopia. Think temporal area is at the
top of the head i.e. superior quadranopia.

20. A 22 year old man is referred to urology with possible urinary retention. He is passing
huge amounts of urine. Post void bladder ultrasound is normal.

You answered Right homonymous hemianopia

The correct answer is Lower bitemporal hemianopia

Theme from January 2012 exam


This patient has diabetes insipidus due to a craniopharyngioma. Lesions at the optic
chiasm classically produce a bitemporal hemianopia, however note lesions that spread up
from below ie pituitary tumours, the defect is worse in the upper fields and if a lesion
spreads down from above ie craniopharyngiomas, the visual defect is worse in the lower
quadrants. Therefore this patient is likely to have a lower bitemporal hemianopia.

21. A 53 year old man is admitted to the vascular ward for a carotid endarterectomy. His CT
head report confirms a left parietal lobe infarct.

You answered Right homonymous hemianopia


The correct answer is Right inferior quadranopia

Parietal lesions cause a contralateral inferior quadranopia.


Superior quadranopia = temporal lobe lesion
Inferior quadranopia = parietal lobe lesion

Please rate this question:

Discuss and give feedback


Next question

Visual field defects

Theme from January 2012 exam


Theme from April 2012 exam

 left homonymous hemianopia means visual field defect to the left, i.e. Lesion of right optic
tract
 homonymous quadrantanopias: PITS (Parietal-Inferior, Temporal-Superior)
 incongruous defects = optic tract lesion; congruous defects = optic radiation lesion or
occipital cortex

Homonymous hemianopia

 Incongruous defects: lesion of optic tract


 Congruous defects: lesion of optic radiation or occipital cortex
 Macula sparing: lesion of occipital cortex

Homonymous quadrantanopias

 Superior: lesion of temporal lobe


 Inferior: lesion of parietal lobe
 Mnemonic = PITS (Parietal-Inferior, Temporal-Superior)

Bitemporal hemianopia

 Lesion of optic chiasm


 Upper quadrant defect > lower quadrant defect = inferior chiasmal compression, commonly a
pituitary tumour
 Lower quadrant defect > upper quadrant defect = superior chiasmal compression, commonly
a craniopharyngioma
A middle aged lady is brought to the clinic by her husband who has noted a change in her
appearance. She finds removal of rings difficult, her shoe size has changed and photographs show a
marked change in her appearance. Which of the following is most likely to be identified on
neurological examination?

Bi nasal hemianopia

Bi temporal hemianopia

Inferior quadrantanopia

Homonymous hemianopia

Unilateral loss of vision

Theme from 2011 Exam


The patient is most likely to have developed acromegaly. Since a pituitary lesion is likely to be
present; compression of the optic chiasm may occur.
Please rate this question:

Discuss and give feedback


Next question

Visual field defects

Theme from January 2012 exam


Theme from April 2012 exam

 left homonymous hemianopia means visual field defect to the left, i.e. Lesion of right optic
tract
 homonymous quadrantanopias: PITS (Parietal-Inferior, Temporal-Superior)
 incongruous defects = optic tract lesion; congruous defects = optic radiation lesion or
occipital cortex

Homonymous hemianopia

 Incongruous defects: lesion of optic tract


 Congruous defects: lesion of optic radiation or occipital cortex
 Macula sparing: lesion of occipital cortex

Homonymous quadrantanopias

 Superior: lesion of temporal lobe


 Inferior: lesion of parietal lobe
 Mnemonic = PITS (Parietal-Inferior, Temporal-Superior)

Bitemporal hemianopia

 Lesion of optic chiasm


 Upper quadrant defect > lower quadrant defect = inferior chiasmal compression, commonly a
pituitary tumour
 Lower quadrant defect > upper quadrant defect = superior chiasmal compression, commonly
a craniopharyngioma
Theme: Intra cranial haemorrhage

A. Acute sub dural haematoma


B. Chronic sub dural haematoma
C. Acute extradural haematoma
D. Chronic extradural haematoma
E. Intraventricular haemorrhage
F. Sub arachnoid haemorrhage

Please select the most likely intra cranial bleeding event to account for the scenario described. Each
option may be used once, more than once or not at all.

23. A 28 year old man is playing tennis when he suddenly collapses and has a GCS of 4 when
examined.

You answered Acute sub dural haematoma

The correct answer is Sub arachnoid haemorrhage

Theme from April 2012 Exam


A sudden collapse and loss of consciousness is most likely to be due to a sub arachnoid
haemorrhage. The other potential causes in the list usually occur as a sequel to a traumatic
event, which has not occurred here.

24. A 2 day old premature neonate is born by emergency cesarean section for maternal illness.
The baby is noted to become floppy and unresponsive.

You answered Acute sub dural haematoma

The correct answer is Intraventricular haemorrhage

Neonatal deterioration in premature babies is not infrequently due to intra ventricular


haemorrhage. In extreme prematurity the prognosis can be very poor.

25. A 78 year old man is brought to the emergency department by the police. He is found
wandering around the town centre and is confused. His family report that he is usually well
apart from a simple mechanical fall 3 weeks previously from which he sustained no
obvious injuries.

You answered Acute sub dural haematoma

The correct answer is Chronic sub dural haematoma


The injuries that are responsible for chronic sub dural haematomas are usually fairly trivial
and forgotten by the patient and their families. The onset of symptoms can be insidious
with vague symptomatology and confusion predominating.

Please rate this question:

Discuss and give feedback


Next question

Intra cranial haemorrhage

Extradural Bleeding into the space between the dura mater and the skull. Often results
haematoma from acceleration-deceleration trauma or a blow to the side of the head. The
majority of extradural haematomas occur in the temporal region where skull
fractures cause a rupture of the middle meningeal artery.

Features

 Raised intracranial pressure


 Some patients may exhibit a lucid interval

Subdural Bleeding into the outermost meningeal layer. Most commonly occur around
haematoma the frontal and parietal lobes. May be either acute or chronic.

Risk factors include old age and alcoholism.

Slower onset of symptoms than a extradural haematoma.

Intracerebral Usually hyperdense lesions on CT scanning. Arise in areas of traumatic


haematoma contusion with fuse to become a haematoma. Areas of clot and fresh blood
may co-exist on the same CT scan (Swirl sign). Large haematomas and
those associated with mass effect should be evacuated.

Subarachnoid Usually occurs spontaneously in the context of a ruptured cerebral


haemorrhage aneurysm but may be seen in association with other injuries when a patient
has sustained a traumatic brain injury

Intraventricular Haemorrhage that occurs into the ventricular system of the brain. It is
haemorrhage relatively rare in adult surgical practice and when it does occur, it is
typically associated with severe head injuries. In premature neonates it may
occur spontaneously. The blood may clot and occlude CSF flow,
hydrocephalus may result.
In neonatal practice the vast majority of IVH occur in the first 72 hours
after birth, the aetiology is not well understood and it is suggested to occur
as a result of birth trauma combined with cellular hypoxia, together with the
delicate neonatal CNS.
Theme: Management of head injuries

A. Intravenous mannitol
B. Parietotemporal craniotomy
C. Burr Hole decompression
D. Posterior fossa craniotomy
E. Insertion of intracranial bolt monitor
F. Discharge
G. Intravenous frusemide

What is the best immediate management plan for the injury described? Each option may be used
once, more than once or not at all.

26. A 25 year old cyclist is hit by a bus traveling at 30mph. He is not wearing a helmet. He
arrives with a GCS of 3/15 and is intubated. A CT scan shows evidence of cerebral
contusion but no localising clinical signs are present

You answered Intravenous mannitol

The correct answer is Insertion of intracranial bolt monitor

This patient may well develop raised ICP over the next few days and Intracranial pressure
monitoring will help with management.

27. A 32 year old rugby player is crushed in a scrum. He is briefly concussed but then regains
consciousness. He then collapses and is brought to A+E. His GCS on arrival is 6/15 and
his left pupil is dilated.

You answered Intravenous mannitol

The correct answer is Parietotemporal craniotomy

This man needs urgent decompression and extradural haematoma is the most likely event,
from a lacerated middle meningeal artery. The debate as to whether Burr Holes or
craniotomy is the best option continues. Most neurosurgeons would perform a craniotomy.
However, rural units and those units without neurosurgical kit facing this emergency may
resort to Burr Holes.

28. A 30 year old women is injured in a skiing accident. She suffers a blow to the occiput and
is concussed for 5 minutes. On arrival in A+E she is confused with GCS 10/15. A CT scan
shows no evidence of acute bleed or fracture but some evidence of oedema with the
beginnings of mass effect
Intravenous mannitol

This women has raised ICP and mannitol will help reduce this in the acute phase.

Please rate this question:

Discuss and give feedback


Next question

Head injury management- NICE Guidelines

Summary of guidelines

 All patients should be assessed within 15 minutes on arrival to A&E


 Document all 3 components of the GCS
 If GCS <8 or = to 8, consider stabilising the airway
 Treat pain with low dose IV opiates (if safe)
 Full spine immobilisation until assessment if:

- GCS < 15
- neck pain/tenderness
- paraesthesia extremities
- focal neurological deficit
- suspected c-spine injury

If a c-spine injury is suspected a 3 view c-spine x-ray is indicated. CT c-spine is preferred if:
- Intubated
- GCS <13
- Normal x-ray but continued concerns regarding c-spine injury
- Any focal neurology
- A CT head scan is being performed
- Initial plain films are abnormal

Immediate CT head (within 1 hour) if:

 GCS < 13 on admission


 GCS < 15 2 hours after admission
 Suspected open or depressed skull fracture
 Suspected skull base fracture (panda eyes, Battle's sign, CSF from nose/ear, bleeding ear)
 Focal neurology
 Vomiting > 1 episode
 Post traumatic seizure
 Coagulopathy
Contact neurosurgeon if:

 Persistent GCS < 8 or = 8


 Unexplained confusion > 4h
 Reduced GCS after admission
 Progressive neurological signs
 Incomplete recovery post seizure
 Penetrating injury
 Cerebrospinal fluid leak

Observations

 1/2 hourly GCS until 15


A 25-year-old female with a history of bilateral vitreous haemorrhage is referred with bilateral lesions
in the cerebellar region. What is the likely diagnosis?

Neurofibromatosis type I

Neurofibromatosis type II

Tuberose sclerosis

Von Hippel-Lindau syndrome

Sarcoidosis

Retinal and cerebellar haemangiomas are key features of Von Hippel-Lindau syndrome. Retinal
haemangiomas are bilateral in 25% of patients and may lead to vitreous haemorrhage
Please rate this question:

Discuss and give feedback


Next question

Von Hippel-Lindau syndrome

Von Hippel-Lindau (VHL) syndrome is an autosomal dominant condition predisposing to neoplasia. It


is due to an abnormality in the VHL gene located on short arm of chromosome 3

Features

 cerebellar haemangiomas
 retinal haemangiomas: vitreous haemorrhage
 renal cysts (premalignant)
 phaeochromocytoma
 extra-renal cysts: epididymal, pancreatic, hepatic
 endolymphatic sac tumours
Theme: Head injury

A. Acute sub dural haematoma


B. Intra cerebral haematoma
C. Extra dural haematoma
D. Chronic sub dural haematoma
E. Basal skull fracture
F. Subarachnoid haemorrhage
G. Diffuse axonal injury

What is the most likely diagnosis for the scenario given? Each option may be used once, more than
once or not at all.

30. A 18 year old boy is involved in a fall from a balcony whilst intoxicated. He has bruising
over the mastoid area and is unconscious.

You answered Acute sub dural haematoma

The correct answer is Basal skull fracture

Bruising over the mastoid process of the temporal bone is battle's sign caused by a basal
skull fracture.

31. A 18 year old boy falls off a balcony and hits the right side of the head. He is admitted to
the emergency department and has a GCS of 8. He is admitted for observation, and over
the following twelve hours develops an increasing headache and confusion. A CT scan
shows a crescent shaped collection of fluid between the brain and the dura with associated
midline shift.

Acute sub dural haematoma

Sub dural haematomas are the commonest intracranial mass lesions resulting from trauma.
They are classified as acute, sub acute or chronic according to tempo of onset. Acute sub
dural haematomas will present within 72 hours of the original injury and have hyperdense,
crescent shaped appearances on CT scanning.

32. A baby is brought to casualty unconscious and in a vegetative state. She has cigarette
burns on her legs.

You answered Acute sub dural haematoma

The correct answer is Diffuse axonal injury


The baby is likely to be a victim of shaken baby syndrome. This may result in diffuse
axonal injury causing extensive lesions in the white matter.

Please rate this question:

Discuss and give feedback


Next question

Head injury

Patients who suffer head injuries should be managed according to ATLS principles and extra cranial
injuries should be managed alongside cranial trauma. Inadequate cardiac output will compromise
CNS perfusion irrespective of the nature of the cranial injury.

Types of traumatic brain injury


Bleeding into the space between the dura mater and the skull. Often results from
acceleration-deceleration trauma or a blow to the side of the head. The majority
of extradural haematomas occur in the temporal region where skull fractures
cause a rupture of the middle meningeal artery.

Extradural
Features
haematoma

 Raised intracranial pressure


 Some patients may exhibit a lucid interval

Bleeding into the outermost meningeal layer. Most commonly occur around the
frontal and parietal lobes. May be either acute or chronic.
Subdural
haematoma Risk factors include old age and alcoholism.

Slower onset of symptoms than a extradural haematoma.

Usually occurs spontaneously in the context of a ruptured cerebral aneurysm, but


Subarachnoid
may be seen in association with other injuries when a patient has sustained a
haemorrhage
traumatic brain injury.

Pathophysiology

 Primary brain injury may be focal (contusion/ haematoma) or diffuse (diffuse axonal injury)
 Diffuse axonal injury occurs as a result of mechanical shearing following deceleration,
causing disruption and tearing of axons
 Intra-cranial haematomas can be extradural, subdural or intracerebral, while contusions may
occur adjacent to (coup) or contralateral (contre-coup) to the side of impact
 Secondary brain injury occurs when cerebral oedema, ischaemia, infection, tonsillar or
tentorial herniation exacerbates the original injury. The normal cerebral auto regulatory
processes are disrupted following trauma rendering the brain more susceptible to blood flow
changes and hypoxia
 The Cushings reflex (hypertension and bradycardia) often occurs late and is usually a pre
terminal event

Management

 Where there is life threatening rising ICP such as in extra dural haematoma and whilst
theatre is prepared or transfer arranged use of IV mannitol/ frusemide may be required.
 Diffuse cerebral oedema may require decompressive craniotomy
 Exploratory Burr Holes have little management in modern practice except where scanning
may be unavailable and to thus facilitate creation of formal craniotomy flap
 Depressed skull fractures that are open require formal surgical reduction and debridement,
closed injuries may be managed non operatively if there is minimal displacement.
 ICP monitoring is appropriate in those who have GCS 3-8 and normal CT scan.
 ICP monitoring is mandatory in those who have GCS 3-8 and abnormal CT scan.
 Hyponatraemia is most likely to be due to syndrome of inappropriate ADH secretion.
 Minimum of cerebral perfusion pressure of 70mmHg in adults.
 Minimum cerebral perfusion pressure of between 40 and 70 mmHg in children.

Interpretation of pupillary findings in head injuries


Pupil size Light response Interpretation

Unilaterally dilated Sluggish or fixed 3rd nerve compression secondary to tentorial


herniation

Bilaterally dilated Sluggish or fixed  Poor CNS perfusion


 Bilateral 3rd nerve palsy

Unilaterally dilated or Cross reactive (Marcus - Optic nerve injury


equal Gunn)

Bilaterally constricted May be difficult to  Opiates


assess  Pontine lesions
 Metabolic encephalopathy

Unilaterally Preserved Sympathetic pathway disruption


constricted
Next question
A 28 year old man falls and hits his head against a wall. There is a brief loss of consciousness.
When assessed in accident and emergency he is alert and orientated with a GCS of 15, imaging
shows no fracture of the skull. What is his risk of having an intracranial haematoma requiring
removal?

1 in 6000

1 in 40

1 in 4

1 in 50,000

1 in 120

Please rate this question:

Discuss and give feedback

Next question

Head injury and haematoma

Risk of haematoma (requiring removal) in adults attending accident and emergency units following
head injury.

Injury Conscious level Risk of haematoma requiring removal

Concussion, no skull fracture Orientated 1 in 6000

Concussion, no skull fracture Not orientated 1 in 120


Skull fracture Orientated 1 in 32

Skull fracture Not orientated 1 in 4

Next question

The term signature fracture is synonymous with which of the following injuries?
Depressed skull fracture

Le Fort II fracture

Orbital blow out

Oblique fracture of the tibia

Supracondylar fracture

Signature fractures are synonymous with depressed skull fractures, they are usually low velocity
injuries where the fracture impression resembles the injurious source.
Please rate this question:

Discuss and give feedback


Next question

Depressed skull fracture

 Depressed skull fractures are also referred to as signature fractures.


 Results from the focal impact of a moving object on the cranial vault.
 High velocity objects will not only disrupt bone, but may also drive the fracture fragments into
the brain.
 Blunt objects moving at low velocity may produce a defect in the skull that is of similar
dimensions to the object (c.f. signature)
 Injuries may affect the outer table alone or extend to involve the inner table.
 Open fractures and those associated with intracranial haematomas may require surgery,
uncomplicated fractures without significant cosmetic deformities may be managed
conservatively.
 CT scanning is the initial imaging modality of choice.

Which of the following is not a form of primary brain injury?


Sub dural haemorrhage after being hit in the head with a hammer

Meningitis resulting from infected CSF rhinorrhoea after a basal skull fracture

A truck driver is involved in a road traffic accident and suffers an axonal stretch injury

A man is hit with a baseball bat and suffers a cerebral contusion

A man suffers an intraparenchymal haemorrhage after being hit in head during a car crash

Primary brain damage occurs at the point of injury. It includes contusions and diffuse axonal injury.
Non reversible.
Secondary brain damage occurs after the injury. Complications include:
1. Haemorrhage
2. Meningitis
3. Herniation
4. Hypoxia
5. Oedema
6. Arterial damage: internal carotid, vertebral artery common
Please rate this question:

Discuss and give feedback

Head injury

Patients who suffer head injuries should be managed according to ATLS principles and extra cranial
injuries should be managed alongside cranial trauma. Inadequate cardiac output will compromise
CNS perfusion irrespective of the nature of the cranial injury.

Types of traumatic brain injury


Bleeding into the space between the dura mater and the skull. Often results from
acceleration-deceleration trauma or a blow to the side of the head. The majority
of extradural haematomas occur in the temporal region where skull fractures
cause a rupture of the middle meningeal artery.

Extradural
Features
haematoma

 Raised intracranial pressure


 Some patients may exhibit a lucid interval
Bleeding into the outermost meningeal layer. Most commonly occur around the
frontal and parietal lobes. May be either acute or chronic.
Subdural
haematoma Risk factors include old age and alcoholism.

Slower onset of symptoms than a extradural haematoma.

Usually occurs spontaneously in the context of a ruptured cerebral aneurysm, but


Subarachnoid
may be seen in association with other injuries when a patient has sustained a
haemorrhage
traumatic brain injury.

Pathophysiology

 Primary brain injury may be focal (contusion/ haematoma) or diffuse (diffuse axonal injury)
 Diffuse axonal injury occurs as a result of mechanical shearing following deceleration,
causing disruption and tearing of axons
 Intra-cranial haematomas can be extradural, subdural or intracerebral, while contusions may
occur adjacent to (coup) or contralateral (contre-coup) to the side of impact
 Secondary brain injury occurs when cerebral oedema, ischaemia, infection, tonsillar or
tentorial herniation exacerbates the original injury. The normal cerebral auto regulatory
processes are disrupted following trauma rendering the brain more susceptible to blood flow
changes and hypoxia
 The Cushings reflex (hypertension and bradycardia) often occurs late and is usually a pre
terminal event

Management

 Where there is life threatening rising ICP such as in extra dural haematoma and whilst
theatre is prepared or transfer arranged use of IV mannitol/ frusemide may be required.
 Diffuse cerebral oedema may require decompressive craniotomy
 Exploratory Burr Holes have little management in modern practice except where scanning
may be unavailable and to thus facilitate creation of formal craniotomy flap
 Depressed skull fractures that are open require formal surgical reduction and debridement,
closed injuries may be managed non operatively if there is minimal displacement.
 ICP monitoring is appropriate in those who have GCS 3-8 and normal CT scan.
 ICP monitoring is mandatory in those who have GCS 3-8 and abnormal CT scan.
 Hyponatraemia is most likely to be due to syndrome of inappropriate ADH secretion.
 Minimum of cerebral perfusion pressure of 70mmHg in adults.
 Minimum cerebral perfusion pressure of between 40 and 70 mmHg in children.

Interpretation of pupillary findings in head injuries


Pupil size Light response Interpretation
Pupil size Light response Interpretation

Unilaterally dilated Sluggish or fixed 3rd nerve compression secondary to tentorial


herniation

Bilaterally dilated Sluggish or fixed  Poor CNS perfusion


 Bilateral 3rd nerve palsy

Unilaterally dilated or Cross reactive (Marcus - Optic nerve injury


equal Gunn)

Bilaterally constricted May be difficult to  Opiates


assess  Pontine lesions
 Metabolic encephalopathy

Unilaterally Preserved Sympathetic pathway disruption


constricted
A 25 year old man is shot in the abdomen and is transferred to the operating theatre following arrival
in the emergency department, as he is unstable and a FAST scan is positive. At operation there is
an extensive laceration to the right lobe of the liver and involvement of the IVC. There is massive
haemorrhage. What is the most appropriate approach to blood component therapy?

Use Factor VIII concentrates early

Avoid use of "o" negative blood

Transfuse packed cells, FFP and platelets in fixed ratios of 1:1:1

Transfuse packed cells and FFP in a fixed ratio of 4:1

Perform goal directed transfusion based on the Hb, PT and TEG studies

There is strong evidence to support the use of haemostatic transfusion in the setting of major
haemorrhage due to trauma. This advocates the use of 1:1:1 ratios.
Please rate this question:

Discuss and give feedback


Next question

Transfusions in major trauma

Uncontrolled haemorrhage accounts for up to 39% of all trauma related death. In the UK
approximately 2% of all trauma patients will need massive transfusion.Massive transfusion is defined
as the replacement of a patient's total blood volume in less than 24 hours, or as the acute
administration of more than half the patient's estimated blood volume per hour. In haemorrhaging
patients following trauma there is evidence to support the initial administration of tranexamic acid
(CRASH study). During acute bleeding the practice of haemostatic resuscitation has been shown to
reduce mortality rates. The principle of haemostatic resuscitation is that blood components are
transfused in fixed ratios. For example; packed red cells, FFP and platelets are administered in a
ratio of 1:1:1.

The typical therapeutic end points include:

 Hb: 8-10 g/dl


 Platelets > 100
 PT and APTT (INR)< 1.5
 Fibrinogen > 1.0 g/l
 Ca2+ > 1 mmol/l
 pH: 7.35-7.45
 BE: +/- 2
 ToC > 36 °C

Next question
A 76 year old woman with a body weight of 50 kg is undergoing an excision of a lipoma from her
forehead. It is the first time the senior house officer has performed the procedure. He administers
30ml of 2% lignocaine to the area. The procedure is complicated by bleeding and the patient
experiences discomfort, a further 10ml of the same anaesthetic formulation is then administered.
Over the following 5 minutes the patient complains of tinnitus and becomes drowsy. Which of the
drugs listed below should be administered?

Temazepam

Lorazepam

Naloxone

Intralipid 20%

Sodium bicarbonate 20%

Local anaesthetic toxicity treatment = Intralipid

Intralipid is indicated for the treatment of local anaesthetic toxicity. In this case the safe dose of local
anaesthetic has been exceeded and is thus this lady's symptoms are likely to represent toxicity.
Please rate this question:

Discuss and give feedback


Next question

Local anaesthetic toxicity

Toxicity results from either accidental intravascular injection (rapid onset of symptoms-usually
correct dose), or from excessive dosage (slower onset). Local anaesthetic agents not only exert a
membrane stabilising effect on peripheral nerves but will also act on excitable membranes within the
CNS and Heart. The sensory neurones in the CNS are suppressed before the motor ones. As a
result the early symptoms will typically be those of circumoral paraesthesia and tinnitus, followed by
falling GCS and eventually coma.

Management of toxicity

 Stop injecting the anaesthetic agent


 High flow 100% oxygen via face mask
 Cardiovascular monitoring
 Administer lipid emulsion (Intralipid 20%) at 1.5ml/Kg over 1 minute as a bolus
 Consider lipid emulsion infusion, at 0.25ml/ Kg/ minute
 If toxicity due to prilocaine then administer methylene blue

Safe doses
10ml of lignocaine 1% contains 100mg of drug, this would constitute 70% of the maximum safe dose
in a 50 kg patient. Up to 7mg / kg can be administered if adrenaline is added to the solution.

Doses of local anaesthetics


Agent Dose plain Dose with adrenaline

Lignocaine 3mg/Kg 7mg/Kg

Bupivicane 2mg/Kg 2mg/Kg

Prilocaine 6mg/Kg 9mg/Kg

These are a guide only as actual doses depend on site of administration, tissue vascularity and co-
morbidities.
Next question
A 27 year old man is involved in a road traffic accident. He is seen in the emergency department
with chest pain. Clinical examination is essentially unremarkable and he is discharged. He
subsequently is found dead at home. What is the most likely underlying injury?

Tracheobronchial tree injury

Traumatic aortic disruption

Cardiac laceration

Diaphragmatic rupture

Rupture of the oesophagus

Theme from January 2013 Exam


Aortic injuries that do not die at the scene may have a contained haematoma. Clinical signs are
subtle and the diagnosis may not be apparent on clinical examination. Without prompt treatment the
haematoma usually bursts and the patient dies.
Please rate this question:

Discuss and give feedback


Next question

Thoracic aorta rupture

 Mechanism of injury: Decelerating force i.e. RTA, fall from a great height
 Most people die at scene
 Survivors may have an incomplete laceration at the ligamentum arteriosum of the aorta.

Clinical features

 Contained haematoma: persistent hypotension


 Detected mainly by history, CXR changes

CXR changes

 Widened mediastinum
 Trachea/Oesophagus to right
 Depression of left main stem bronchus
 Widened paratracheal stripe/paraspinal interfaces
 Space between aorta and pulmonary artery obliterated
 Rib fracture/left haemothorax

Diagnosis
Angiography, usually CT aortogram.

Treatment
Repair or replacement. Ideally they should undergo endovascular repair.
Next question
A patient is brought to the emergency department following a motor vehicle accident. He is
unconscious and has a deep scalp laceration. His heart rate is 120/min, blood pressure is 80/40
mmHg, and respiratory rate is 35/min. Despite rapid administration of 2 litres of Hartmans solution,
the patient's vital signs do not change significantly. The injury likely to explain this patient's
hypotension is:

Epidural haematoma

Sub dural haematoma

Intra parenchymal brain haemorrhage

Base of skull fracture

None of the above

Raised intracranial pressure (Cushing response)

 Hypertension
 Bradycardia
 Respiratory depression

In the patient described, hypotension and tachycardia should not be uncritically attributed to the
head injury, since these findings in the setting of blunt trauma are suggestive of serious thoracic,
abdominal, or pelvic hemorrhage. When cardiovascular collapse occurs as a result of rising
intracranial pressure, it is generally accompanied by hypertension, bradycardia, and respiratory
depression.
Please rate this question:

Discuss and give feedback


Next question

Head injury

Patients who suffer head injuries should be managed according to ATLS principles and extra cranial
injuries should be managed alongside cranial trauma. Inadequate cardiac output will compromise
CNS perfusion irrespective of the nature of the cranial injury.

Types of traumatic brain injury


Extradural Bleeding into the space between the dura mater and the skull. Often results from
haematoma acceleration-deceleration trauma or a blow to the side of the head. The majority
of extradural haematomas occur in the temporal region where skull fractures
cause a rupture of the middle meningeal artery.

Features

 Raised intracranial pressure


 Some patients may exhibit a lucid interval

Bleeding into the outermost meningeal layer. Most commonly occur around the
frontal and parietal lobes. May be either acute or chronic.
Subdural
haematoma Risk factors include old age and alcoholism.

Slower onset of symptoms than a extradural haematoma.

Usually occurs spontaneously in the context of a ruptured cerebral aneurysm, but


Subarachnoid
may be seen in association with other injuries when a patient has sustained a
haemorrhage
traumatic brain injury.

Pathophysiology

 Primary brain injury may be focal (contusion/ haematoma) or diffuse (diffuse axonal injury)
 Diffuse axonal injury occurs as a result of mechanical shearing following deceleration,
causing disruption and tearing of axons
 Intra-cranial haematomas can be extradural, subdural or intracerebral, while contusions may
occur adjacent to (coup) or contralateral (contre-coup) to the side of impact
 Secondary brain injury occurs when cerebral oedema, ischaemia, infection, tonsillar or
tentorial herniation exacerbates the original injury. The normal cerebral auto regulatory
processes are disrupted following trauma rendering the brain more susceptible to blood flow
changes and hypoxia
 The Cushings reflex (hypertension and bradycardia) often occurs late and is usually a pre
terminal event

Management

 Where there is life threatening rising ICP such as in extra dural haematoma and whilst
theatre is prepared or transfer arranged use of IV mannitol/ frusemide may be required.
 Diffuse cerebral oedema may require decompressive craniotomy
 Exploratory Burr Holes have little management in modern practice except where scanning
may be unavailable and to thus facilitate creation of formal craniotomy flap
 Depressed skull fractures that are open require formal surgical reduction and debridement,
closed injuries may be managed non operatively if there is minimal displacement.
 ICP monitoring is appropriate in those who have GCS 3-8 and normal CT scan.
 ICP monitoring is mandatory in those who have GCS 3-8 and abnormal CT scan.
 Hyponatraemia is most likely to be due to syndrome of inappropriate ADH secretion.
 Minimum of cerebral perfusion pressure of 70mmHg in adults.
 Minimum cerebral perfusion pressure of between 40 and 70 mmHg in children.

Interpretation of pupillary findings in head injuries


Pupil size Light response Interpretation

Unilaterally dilated Sluggish or fixed 3rd nerve compression secondary to tentorial


herniation

Bilaterally dilated Sluggish or fixed  Poor CNS perfusion


 Bilateral 3rd nerve palsy

Unilaterally dilated or Cross reactive (Marcus - Optic nerve injury


equal Gunn)

Bilaterally constricted May be difficult to  Opiates


assess  Pontine lesions
 Metabolic encephalopathy

Unilaterally Preserved Sympathetic pathway disruption


constricted
Next question
A 42 year old man is admitted to surgery with acute appendicitis. He is known to have hypertension,
psoriatic arthropathy and polymyalgia rheumatica. His medical therapy includes:
Paracetamol 1g qds
Codeine phosphate 30mg qds
Bendrofluazide 2.5 mg od
Ramipril 10mg od
Methotrexate 7.5mg once a week
Prednisolone 5mg od
You are called by the Senior House Officer to assess this man as he has become delirious and
hypotensive 2 hours after surgery. His blood results reveal:

Na+ 132 mmol/l

K+ 5.2 mmol/l

Urea 10 mmol/l

Creatinine 111 µmol/l

Glucose 3.5

CRP 158

Hb 10.2 g/dl

Platelets 156 * 109/l

WBC 14 * 109/l

What is the most likely diagnosis?

Septic shock secondary to appendicitis

Neutropenic sepsis

Phaeochromocytoma

Perforated bowel
Addisonian crisis

Features of an addisonian crisis:

 Hyponatraemia
 Hyperkalaemia
 Hypoglycaemia

This man is on steroids for polymyalgia rheumatica. Surgery can precipitate acute adrenal
deficiency. The diagnosis is further confirmed by the blood results of hyponatraemia, hyperkalaemia
and hypoglycaemia. This patient urgently needs hydrocortisone.
Please rate this question:

Discuss and give feedback


Next question

Addisonian crisis

Causes

 Sepsis or surgery causing an acute exacerbation of chronic insufficiency (Addison's,


Hypopituitarism)
 Adrenal haemorrhage eg Waterhouse-Friderichsen syndrome (fulminant meningococcemia)
 Steroid withdrawal

Management

 Hydrocortisone 100 mg im or iv
 1 litre normal saline infused over 30-60 mins or with dextrose if hypoglycaemic
 Continue hydrocortisone 6 hourly until the patient is stable. No fludrocortisone is required
because high cortisol exerts weak mineralocorticoid action
 Oral replacement may begin after 24 hours and be reduced to maintenance over 3-4 days

Next question
A 32 year old man is involved in a motorcycle accident and sustains a closed unstable spiral tibial
fracture. This is managed with an intramedullary nail. On return to the ward he is noted to have
increasing pain in the limb and on examination the limb is swollen and tender with pain on passive
stretching of the toes. The most likely diagnosis is:

Tibial nerve neuropraxia

Displaced tibial nail

Compartment syndrome

Deep vein thrombosis

Sciatic nerve injury

Theme from September 2014 Exam


Severe pain in a limb should raise suspicions of compartment syndrome especially in tibial fractures
following fixation with intra medullary devices.
Please rate this question:

Discuss and give feedback


Next question

Compartment syndrome

 This is a particular complication that may occur following fractures (or following ischaemia re-
perfusion injury in vascular patients). It is characterised by raised pressure within a closed
anatomical space.
 The raised pressure within the compartment will eventually compromise tissue perfusion
resulting in necrosis. The two main fractures carrying this complication include supracondylar
fractures and tibial shaft injuries.

Symptoms and signs

 Pain, especially on movement (even passive)


 Parasthesiae
 Pallor may be present
 Arterial pulsation may still be felt as the necrosis occurs as a result of microvascular
compromise
 Paralysis of the muscle group may occur
Diagnosis

 Is made by measurement of intracompartmental pressure measurements. Pressures in


excess of 20mmHg are abnormal and >40mmHg is diagnostic.

Treatment

 This is essentially prompt and extensive fasciotomies


 In the lower limb the deep muscles may be inadequately decompressed by the
inexperienced operator when smaller incisions are performed
 Myoglobinuria may occur following fasciotomy and result in renal failure and for this reason
these patients require aggressive IV fluids
 Where muscle groups are frankly necrotic at fasciotomy they should be debrided and
amputation may have to be considered
 Death of muscle groups may occur within 4-6 hours

Next question
A 28 year old man is involved in a road traffic accident and sustains a flail chest injury. On arrival in
the emergency department he is hypotensive. On examination; he has an elevated jugular venous
pulse and auscultation of the heart reveals quiet heard sounds. What is the most likely diagnosis?

Pneumothorax

Myocardial contusion

Cardiac tamponade

Haemothorax

Ventricular septal defect

Theme from 2010 exam


The presence of a cardiac tamponade is suggested by Becks Triad:

 Hypotension
 Muffled heart sounds
 Raised JVP

Please rate this question:

Discuss and give feedback


Next question

Thoracic trauma

Types of thoracic trauma

Tension  Often laceration to lung parenchyma with flap


pneumothorax  Pressure develops in thorax
 Most common cause is mechanical ventilation in patient with
pleural injury
 Symptoms overlap with cardiac tamponade, hyper-resonant
percussion note is more likely in tension pnemothorax

Flail chest  Chest wall disconnects from thoracic cage


 Multiple rib fractures (at least two fractures per rib in at least two
ribs)
 Associated with pulmonary contusion
 Abnormal chest motion
 Avoid over hydration and fluid overload

Pneumothorax  Most common cause is lung laceration with air leakage


 Most traumatic pneumothoraces should have a chest drain
 Patients with traumatic pneumothorax should never be
mechanically ventilated until a chest drain is inserted

Haemothorax  Most commonly due to laceration of lung, intercostal vessel or


internal mammary artery
 Haemothoraces large enough to appear on CXR are treated with
large bore chest drain
 Surgical exploration is warranted if >1500ml blood drained
immediately

Cardiac tamponade  Beck's triad: elevated venous pressure, reduced arterial pressure,
reduced heart sounds
 Pulsus paradoxus
 May occur with as little as 100ml blood

Pulmonary contusion  Most common potentially lethal chest injury


 Arterial blood gases and pulse oximetry important
 Early intubation within an hour if significant hypoxia

Blunt cardiac injury  Usually occurs secondary to chest wall injury


 ECG may show features of myocardial infarction
 Sequelae: hypotension, arrhythmias, cardiac wall motion
abnormalities

Aorta disruption  Deceleration injuries


 Contained haematoma
 Widened mediastinum

Diaphragm  Most due to motor vehicle accidents and blunt trauma causing
disruption large radial tears (laceration injuries result in small tears)
 More common on left side
 Insert gastric tube, may pass into intrathoracic stomach

Mediastinal  Entrance wound in one hemithorax and exit wound/foreign body


traversing wounds in opposite hemithorax
 Mediastinal haematoma or pleural cap suggests great vessel
injury
 Mortality is 20%

References
ATLS Manual 8th Edition
Next question
A 31 year old lady is struck by a car and is 32 weeks pregnant. On arrival in the emergency
department she has a systolic blood pressure of 105mmHg and a pulse rate of 126 beats per
minute. Abdominal examination demonstrates a diffusely tender abdomen and some left sided flank
bruising. A FAST scan is normal. What is the most appropriate course of action?

Arrange a departmental abdominal USS scan

Arrange an urgent abdominal MRI scan

Perform a laparotomy

Perform diagnostic peritoneal lavage

Arrange an urgent abdominal CT scan

The patient's mechanism of injury makes a solid organ injury likely. FAST scanning is associated
with a false negative rate in pregnancy which makes the negative result less reassuring. CT
scanning remains the gold standard.

Please rate this question:

Discuss and give feedback

Next question

Imaging in the pregnant trauma patient

Sonography and FAST scanning are established in pregnancy and have the advantage of avoiding
ionising radiation. However, the sensitivity of the FAST scan is reduced in pregnancy especially with
advanced gestational age. Sensitivity of FAST scanning is 60-80% across all trimesters and 90% in
the first. CT scanning remains the first line investigation in major trauma where significant visceral
injury is suspected. The maximum permitted safe dose of radiation in pregnancy is 5mSv. A pelvic
CT scan would fall below this level. That said, early exposure to radiation will increase the risk of
developmental anomalies and foetal loss. Late exposure increases the risk of childhood cancer
twofold. CT scanning remains the most sensitive test for identifying complications such as placental
abruption in this group.

Next question
Theme: Trauma

A. Tension pneumothorax
B. Haemopericardium
C. Haemothorax
D. Aortic transection
E. Ruptured spleen
F. Duodeno-jejunal flexure disruption
G. Aorto iliac disruption
H. Ileo-colic junction disruption

For each scenario please select the most likely injury. Each option may be used once, more than
once or not at all.

9. A 24 year old motorist is involved in a road traffic accident in which he collides with the
wall of a tunnel in a head on car crash, speed 85mph. He is wearing a seatbelt and the
airbags have deployed. When rescuers arrive he is lucid and conscious and then dies
suddenly.

You answered Tension pneumothorax

The correct answer is Aortic transection

Aortic transections typically occur distal to the ligamentum arteriosum. A temporary


haematoma may prevent the immediate death that usually occurs. This is a deceleration
injury. A widened mediastinum may be seen on x-ray.

10. A 30 year old women is involved in a road traffic accident she is a passenger in a car
involved in a head on collision with another vehicle. Her car is travelling at 60mph. She
has been haemodynamically stable throughout with only minimal tachycardia. On
examination she has marked abdominal tenderness and a large amount of intra abdominal
fluid on CT scan

You answered Tension pneumothorax

The correct answer is Duodeno-jejunal flexure disruption

This is another site of sudden deceleration injury. Given the large amount of free fluid, if it
were blood, then a greater degree of haemodynamic instability would be expected.

11. A 17 year old boy is involved in a motorcycle accident in which he is thrown from his
motorcycle. On admission he has distended neck veins and a weak pulse. The trachea is
central.
You answered Tension pneumothorax

The correct answer is Haemopericardium

This is most likely a cardiac tamponade produced by haemopericardium. As little as 100ml


of blood may result in tamponade as the pericardial sac is not distensible. Diagnosis is
suggested by muffled heart sounds, paradoxical pulse and jugular vein distension.

Please rate this question:

Discuss and give feedback


Next question

Thoracic trauma

Types of thoracic trauma

Tension  Often laceration to lung parenchyma with flap


pneumothorax  Pressure develops in thorax
 Most common cause is mechanical ventilation in patient with
pleural injury
 Symptoms overlap with cardiac tamponade, hyper-resonant
percussion note is more likely in tension pnemothorax

Flail chest  Chest wall disconnects from thoracic cage


 Multiple rib fractures (at least two fractures per rib in at least two
ribs)
 Associated with pulmonary contusion
 Abnormal chest motion
 Avoid over hydration and fluid overload

Pneumothorax  Most common cause is lung laceration with air leakage


 Most traumatic pneumothoraces should have a chest drain
 Patients with traumatic pneumothorax should never be
mechanically ventilated until a chest drain is inserted

Haemothorax  Most commonly due to laceration of lung, intercostal vessel or


internal mammary artery
 Haemothoraces large enough to appear on CXR are treated with
large bore chest drain
 Surgical exploration is warranted if >1500ml blood drained
immediately

Cardiac tamponade  Beck's triad: elevated venous pressure, reduced arterial pressure,
reduced heart sounds
 Pulsus paradoxus
 May occur with as little as 100ml blood

Pulmonary contusion  Most common potentially lethal chest injury


 Arterial blood gases and pulse oximetry important
 Early intubation within an hour if significant hypoxia

Blunt cardiac injury  Usually occurs secondary to chest wall injury


 ECG may show features of myocardial infarction
 Sequelae: hypotension, arrhythmias, cardiac wall motion
abnormalities

Aorta disruption  Deceleration injuries


 Contained haematoma
 Widened mediastinum

Diaphragm  Most due to motor vehicle accidents and blunt trauma causing
disruption large radial tears (laceration injuries result in small tears)
 More common on left side
 Insert gastric tube, may pass into intrathoracic stomach

Mediastinal  Entrance wound in one hemithorax and exit wound/foreign body


traversing wounds in opposite hemithorax
 Mediastinal haematoma or pleural cap suggests great vessel
injury
 Mortality is 20%

References
ATLS Manual 8th Edition
Next question
A 14-year-old boy is admitted to the acute surgical unit with appendicitis. He is normally fit and well.
Apart from metoclopramide, the patient has had no other medications. The nursing staff contact you
as the patient is acting strange. On examination he is agitated, has a clenched jaw and his eyes are
deviated upwards. What is the most likely diagnosis?

Functional disorder

Malignant hyperthermia

Oculogyric crisis

Epilepsy

Serotonin syndrome

This is a classic description of an oculogyric crisis, a form of extrapyramidal disorder. An oculogyric


crisis is an acute dystonic reaction. This is precipitated by antipsychotics (haloperidol) and
metoclopramide in susceptible individuals with a genetic predisposition to this. Treatment is with
procyclidine IM.
Please rate this question:

Discuss and give feedback


Next question

Oculogyric crisis

An oculogyric crisis is a dystonic reaction to certain drugs or medical conditions

Features

 Restlessness, agitation
 Involuntary upward deviation of the eyes

Causes

 Phenothiazines
 Haloperidol
 Metoclopramide
 Postencephalitic Parkinson's disease
Management

 Procyclidine

Next question
A 6 year old boy pulls over a kettle and suffers superficial partial thickness burns to his legs. Which
of the following will not occur?

Preservation of hair follicles

Formation of vesicles or bullae

Damage to sweat glands

Healing by re-epithelialisation

Pain at the burn site

Partial thickness burns are divided into superficial and deep burns, however, this is often not
possible on initial assessment and it may be a week or more before the distinction is clear cut.
Dermal appendages are, by definition, intact. Superficial partial thickness burns will typically heal by
re-epithelialisation, deeper burns will heal with scarring.
Please rate this question:

Discuss and give feedback


Next question

Burns

Burns may be thermal, chemical or electrical. In the former category are burns which occur as a
result of heat. Chemical burns occur when the skin is exposed to an extremely caustic or alkaline
substance. Electrical burns occur following exposure to electrical current. The immediate
management includes removal of the burning source which usually includes irrigation of the burned
area. A detailed assessment then needs to be made of the extent of the burns and a number of
charts are available for recording this information. The degree of injury relates to the temperature
and duration of exposure. Most domestic burns are mainly scalds in young children.

Following the burn, there is a local response with progressive tissue loss and release of
inflammatory cytokines. Systemically, there are cardiovascular effects resulting from fluid loss and
sequestration of fluid into the third space. There is a marked catabolic response. Immunosupression
is common with large burns and bacterial translocation from the gut lumen is a recognised event.
Sepsis is a common cause of death following major burns.

Types of burn
Type of burn Skin layers Skin Blanching Management
affected appearance

Epidermal/Superficial Epidermis Red, moist Yes

Superficial partial Epidermis and part Pale, dry Yes Normally heals
thickness of papillary dermis with no
affected intervention

Deep partial thickness Epidermis, whole Mottled red No Needs surgical


papillary dermis colour intervention
affected (depending on site)

Full thickness Whole skin layer Dry, leathery No Burns centre


and subcutaneous hard wound
tissue affected

Depth of burn assessment

 Bleeding on needle prick


 Sensation
 Appearance
 Blanching to pressure

Percentage burn estimation


Lund Browder chart: most accurate even in children
Wallace rule of nines
Palmar surface: surface area palm = 0.8% burn

>15% body surface area burns in adults needs urgent burn fluid resuscitation

Transfer to burn centre if:

 Need burn shock resuscitation


 Face/hands/genitals affected
 Deep partial thickness or full thickness burns
 Significant electrical/chemical burns

Management
The initial aim is to stop the burning process and resuscitate the patient. Intravenous fluids will be
required for children with burns greater than 10% of total body surface area. Adults with burns
greater than 15% of total body surface area will also require IV fluids. The fluids are calculated using
the Parkland formula which is; volume of fluid= total body surface area of the burn % x weight (Kg)
x4. Half of the fluid is administered in the first 8 hours. A urinary catheter should be inserted.
Analgesia should be given. Complex burns, burns involving the hand perineum and face and burns
>10% in adults and >5% in children should be transferred to a burns unit.

Circumferential burns affecting a limb or severe torso burns impeding respiration may require
escharotomy to divide the burnt tissue.

Conservative management is appropriate for superficial burns and mixed superficial burns that will
heal in 2 weeks. More complex burns may require excision and skin grafting. Excision and primary
closure is not generally practised as there is a high risk of infection.

There is no evidence to support the use of anti microbial prophylaxis or topical antibiotics in burn
patients.

Escharotomies

 Indicated in circumferential full thickness burns to the torso or limbs.


 Careful division of the encasing band of burn tissue will potentially improve ventilation (if the
burn involves the torso), or relieve compartment syndrome and oedema (where a limb is
involved)

References
www.euroburn.org/e107files/downloads/guidelinesburncare.pdf

Barajas-Nava LA, López-Alcalde J, Roqué i Figuls M, Solà I, Bonfill Cosp X. Antibiotic prophylaxis
for preventing burn wound infection. Cochrane Database of Systematic Reviews 2013, Issue 6. Art.
No.: CD008738. DOI: 10.1002/14651858.CD008738.pub2.

Hettiaratchy S & Papini R. Initial management of a major burn: assessment and resuscitation. BMJ
2004;329:101-103
Next question
You are called to the acute surgical unit. A patient who has short gut syndrome has developed a
broad complex tachycardia. You suspect a diagnosis of ventricular tachycardia. What is the most
likely precipitant?

Hypoglycaemia

Bisoprolol

Hypomagnesaemia

Dehydration

Hyperthyroidism

Please rate this question:

Discuss and give feedback


Next question

Ventricular tachcardia

Ventricular tachycardia (VT)is broad-complex tachycardia originating from a ventricular ectopic


focus. It has the potential to precipitate ventricular fibrillation and hence requires urgent treatment.

There are two main types of VT:

 monomorphic VT: most commonly caused by myocardial infarction


 polymorphic VT: A subtype of polymorphic VT is torsades de pointes which is precipitated by
prolongation of the QT interval. The causes of a long QT interval are listed below

Causes of a prolonged QT interval

Congenital Drugs Other

 Jervell-Lange-Nielsen  amiodarone, sotalol,  electrolyte:


syndrome (includes class 1a hypocalcaemia,
deafness and is due to an antiarrhythmic drugs hypokalaemia,
abnormal potassium  tricyclic hypomagnesaemia
channel) antidepressants,  acute myocardial
 Romano-Ward syndrome fluoxetine infarction
(no deafness)  chloroquine  myocarditis
 terfenadine  hypothermia
 erythromycin  subarachnoid
haemorrhage

Next question
Based on the current guidelines, which option regarding management of head injuries is false?

Opiates should be avoided

Consider intubation if the GCS is <8 or = 8

Immediate CT head if there is > 1 episode of vomiting

Half hourly GCS assessment until GCS is 15

Contact neurosurgeons if suspected penetrating injury

Once eight, intubate!

Pain should be controlled, with opiates preferably, as this avoids distress and hypertension post
injury.
Please rate this question:

Discuss and give feedback


Next question

Head injury management- NICE Guidelines

Summary of guidelines

 All patients should be assessed within 15 minutes on arrival to A&E


 Document all 3 components of the GCS
 If GCS <8 or = to 8, consider stabilising the airway
 Treat pain with low dose IV opiates (if safe)
 Full spine immobilisation until assessment if:

- GCS < 15
- neck pain/tenderness
- paraesthesia extremities
- focal neurological deficit
- suspected c-spine injury

If a c-spine injury is suspected a 3 view c-spine x-ray is indicated. CT c-spine is preferred if:
- Intubated
- GCS <13
- Normal x-ray but continued concerns regarding c-spine injury
- Any focal neurology
- A CT head scan is being performed
- Initial plain films are abnormal

Immediate CT head (within 1 hour) if:

 GCS < 13 on admission


 GCS < 15 2 hours after admission
 Suspected open or depressed skull fracture
 Suspected skull base fracture (panda eyes, Battle's sign, CSF from nose/ear, bleeding ear)
 Focal neurology
 Vomiting > 1 episode
 Post traumatic seizure
 Coagulopathy

Contact neurosurgeon if:

 Persistent GCS < 8 or = 8


 Unexplained confusion > 4h
 Reduced GCS after admission
 Progressive neurological signs
 Incomplete recovery post seizure
 Penetrating injury
 Cerebrospinal fluid leak

Observations

 1/2 hourly GCS until 15

Reference
1. http://guidance.nice.org.uk/CG56/QuickRefGuide/pdf/English
2. Hodgkinson S et al. Early management of head injury: summary of NICE guidance. BMJ 2014
(348):34-37.
Next question
A 22 year old man suffers 20% partial and full thickness burns in a house fire. There is an
associated inhalational injury. It is decided to administer intravenous fluids to replace fluid losses.
Which of the intravenous fluids listed below should be used for initial resuscitation?

Dextran 40

5% Dextrose

Fresh frozen plasma

Hartmans solution

Blood

In most units a crystalloid such as Hartmans (Ringers lactate) is administered initially. Controversy
does remain and some units do prefer colloid. Should this leak in the interstial tissues this may
increase the risk of oedema.
Please rate this question:

Discuss and give feedback


Next question

Fluid resuscitation burns

Indication: >15% total body area burns in adults (>10% children)

 The main aim of resuscitation is to prevent the burn deepening


 Most fluid is lost 24 hours after injury
 First 8-12 hour fluid shifts are from intravascular to interstitial fluid compartments
 Therefore circulatory volume can be compromised. However fluid resuscitation causes more
fluid into the interstitial compartment especially colloid (therefore avoided in first 8-24 hours)
 Protein loss occurs

Fluid resuscitation formula


Parkland formula
(Crystalloid only e.g. Hartman's solution/Ringers' lactate)

Total fluid requirement in 24 hours =


4 ml x (total burn surface area (%)) x (body weight (kg))
 50% given in first 8 hours
 50% given in next 16 hours

Resuscitation endpoint:Urine output of 0.5-1.0 ml/kg/hour in adults (increase rate of fluid to


achieve this)

Points to note:

 Starting point of resuscitation is time of injury


 Deduct fluids already given

After 24 hours

 Maintenance crystalloid (usually dextrose-saline) is continued at a rate of 1.5 ml x(burn


area)x(body weight)
 Colloids are rarely used (e.g. albumin)
 Antioxidants, such as vitamin C, can be used to minimize oxidant-mediated contributions to
the inflammatory cascade in burns
 High tension electrical injuries and inhalation injuries require more fluid
 Monitor: packed cell volume, plasma sodium, base excess, and lactate

Next question
A 23 year old man sustains a severe facial fracture and reconstruction is planned. Which of the
following investigations will facilitate pre-operative planning?

Mandibular tomography

Magnetic resonance scan of face

Skull X-ray

Computerised tomography of the head

Orthopantomogram

Theme from 2011 Exam


Significant facial fractures may have intracranial communication. CT scanning will allow delineation
of injury extent and 3D reconstruction images can be created. An Orthopantomogram (OPT) will
provide good images of mandible and surrounding bony structures but will not give intracranial
detail. A skull x-ray lacks the detail for modern practice.
Please rate this question:

Discuss and give feedback


Next question

Craniomaxillofacial injuries

Craniomaxillofacial injuries in the UK are due to:

 Interpersonal violence (52%)


 Motor vehicle accidents (16%)
 Sporting injuries (19%)
 Falls (11%)

Le Fort Fractures
Grade Feature

Le The fracture extends from the nasal septum to the lateral pyriform rims, travels
horizontally above the teeth apices, crosses below the zygomaticomaxillary junction, and
Grade Feature

Fort 1 traverses the pterygomaxillary junction to interrupt the pterygoid plates.

Le These fractures have a pyramidal shape and extend from the nasal bridge at or below the
Fort 2 nasofrontal suture through the frontal process of the maxilla, inferolaterally through the
lacrimal bones and inferior orbital floor and rim through or near the inferior orbital
foramen, and inferiorly through the anterior wall of the maxillary sinus; it then travels
under the zygoma, across the pterygomaxillary fissure, and through the pterygoid plates.

Le These fractures start at the nasofrontal and frontomaxillary sutures and extend posteriorly
Fort 3 along the medial wall of the orbit through the nasolacrimal groove and ethmoid bones.
The thicker sphenoid bone posteriorly usually prevents continuation of the fracture into
the optic canal. Instead, the fracture continues along the floor of the orbit along the
inferior orbital fissure and continues superolaterally through the lateral orbital wall,
through the zygomaticofrontal junction and the zygomatic arch. Intranasally, a branch of
the fracture extends through the base of the perpendicular plate of the ethmoid, through
the vomer, and through the interface of the pterygoid plates to the base of the sphenoid.
This type of fracture predisposes the patient to CSF rhinorrhea more commonly than the
other types.

Ocular injuries
Superior orbital fissure syndrome
Severe force to the lateral wall of the orbit resulting in compression of neurovascular structures.
Results in :

 Complete opthalmoplegia and ptosis (Cranial nerves 3, 4, 6 and nerve to levator palpebrae
superioris)
 Relative afferent pupillary defect
 Dilatation of the pupil and loss of accommodation and corneal reflexes
 Altered sensation from forehead to vertex (frontal branch of trigeminal nerve)

Orbital blow out fracture


Typically occurs when an object of slightly larger diameter than the orbital rim strikes the
incompressible eyeball. The bone fragment is displaced downwards into the antral cavity, remaining
attached to the orbital periosteum. Periorbital fat may be herniated through the defect, interfering
with the inferior rectus and inferior oblique muscles which are contained within the same fascial
sheath. This prevents upward movement and outward rotation of the eye and the patient
experiences diplopia on upward gaze. The initial bruising and swelling may make assessment
difficult and patients should usually be reviewed 5 days later. Residual defects may require orbital
floor reconstruction.
Nasal Fractures

 Common injury
 Ensure new and not old deformity
 Control epistaxis
 CSF rhinorrhoea implies that the cribriform plate has been breached and antibiotics will be
required.
 Usually best to allow bruising and swelling to settle and then review patient clinically. Major
persistent deformity requires fracture manipulation, best performed within 10 days of injury.

Retrobulbar haemorrhage
Rare but important ocular emergency. Presents with:

 Pain (usually sharp and within the globe)


 Proptosis
 Pupil reactions are lost
 Paralysis (eye movements lost)
 Visual acuity is lost (colour vision is lost first)

May be the result of Le Fort type facial fractures.

Management:

 Mannitol 1g/Kg as 20% infusion, Osmotic diuretic, Contra-indicated in congestive heart


failure and pulmonary oedema
 Acetazolamide 500mg IV, (Monitor FBC/U+E) Reduces aqueous pressure by inhibition of
carbonic anhydrase (used in glaucoma)
 Dexamethasone 8mg orally or intravenously
 In a traumatic setting an urgent cantholysis may be needed prior to definitive surgery.

Consider
Papaverine 40mg smooth muscle relaxant
Dextran 40 500mls IV improves perfusion
Next question
A 23 year old man is stabbed in the right upper quadrant and is haemodynamically unstable. A
laparotomy is performed and the liver has some extensive superficial lacerations and is bleeding
profusely. The patient becomes progressively more haemodynamically unstable. What is the best
management option?

Pack the liver and close the abdomen

Occlude the hepatic inflow with a pringles manoeuvre and suture the defects

Occlude vascular inflow and resect the most severely affected area anatomically

Perform a portosystemic shunt procedure

Suture the defects without vascular occlusion

Packing of the liver is the safest option and resection or repair considered later when the physiology
is normalised. Often when the packs are removed all the bleeding has ceased and the abdomen can
be closed without further action. Definitive attempts at suturing or resection at the primary
laparotomy are often complicated by severe bleeding.
Please rate this question:

Discuss and give feedback


Next question

Trauma management

The cornerstone of trauma management is embodied in the principles of ATLS.

Following trauma there is a trimodal death distribution:

 Immediately following injury. Typically as result of brain or high spinal injuries, cardiac or
great vessel damage. Salvage rate is low.
 In early hours following injury. In this group deaths are due to phenomena such as splenic
rupture, sub dural haematomas and haemopneumothoraces
 In the days following injury. Usually due to sepsis or multi organ failure.

Aspects of trauma management

 ABCDE approach.
 Tension pneumothoraces will deteriorate with vigorous ventilation attempts.
 External haemorrhage is managed as part of the primary survey. As a rule tourniquets
should not be used. Blind application of clamps will tend to damage surrounding structures
and packing is the preferred method of haemorrhage control.
 Urinary catheters and naso gastric tubes may need inserting. Be wary of basal skull fractures
and urethral injuries.
 Patients with head and neck trauma should be assumed to have a cervical spine injury until
proven otherwise.

Thoracic injuries

 Simple pneumothorax
 Mediastinal traversing wounds
 Tracheobronchial tree injury
 Haemothorax
 Blunt cardiac injury
 Diaphragmatic injury
 Aortic disruption
 Pulmonary contusion

Management of thoracic trauma

 Simple pneumothorax insert chest drain. Aspiration is risky in trauma as pneumothorax


may be from lung laceration and convert to tension pneumothorax.
 Mediastinal traversing wounds These result from situations like stabbings. Exit and entry
wounds in separate hemithoraces. The presence of a mediastinal haematoma indicates the
likelihood of a great vessel injury. All patients should undergo CT angiogram and
oesophageal contrast swallow. Indications for thoracotomy are largely related to blood loss
and will be addressed below.
 Tracheobronchial tree injury Unusual injuries. In blunt trauma most injuries occur within
4cm of the carina. Features suggesting this injury include haemoptysis and surgical
emphysema. These injuries have a very large air leak and may have tension pneumothorax.
 Haemothorax Usually caused by laceration of lung vessel or internal mammary artery by rib
fracture. Patients should all have a wide bore 36F chest drain. Indications for thoracotomy
include loss of more than 1.5L blood initially or ongoing losses of >200ml per hour for >2
hours.
 Cardiac contusions Usually cardiac arrhythmias, often overlying sternal fracture. Perform
echocardiography to exclude pericardial effusions and tamponade. Risk of arrhythmias falls
after 24 hours.
 Diaphragmatic injury Usually left sided. Direct surgical repair is performed.
 Traumatic aortic disruption Commonest cause of death after RTA or falls. Usually
incomplete laceration near ligamentum arteriosum. All survivors will have contained
haematoma. Only 1-2% of patients with this injury will have a normal chest x-ray.
 Pulmonary contusion Common and lethal. Insidious onset. Early intubation and ventilation.

Abdominal trauma
 Deceleration injuries are common.
 In blunt trauma requiring laparotomy the spleen is most commonly injured (40%)
 Stab wounds traverse structures most commonly liver (40%)
 Gunshot wounds have variable effects depending upon bullet type. Small bowel is most
commonly injured (50%)
 Patients with stab wounds and no peritoneal signs up to 25% will not enter the peritoneal
cavity
 Blood at urethral meatus suggests a urethral tear
 High riding prostate on PR = urethral disruption
 Mechanical testing for pelvic stability should only be performed once

Investigations in abdominal trauma

Diagnostic Peritoneal Abdominal CT scan USS


Lavage

Indication Document bleeding if Document organ injury if Document fluid if


hypotensive normotensive hypotensive

Advantages Early diagnosis and Most specific for Early diagnosis, non
sensitive; 98% accurate localising injury; 92 to invasive and repeatable;
98% accurate 86 to 95% accurate

Disadvantages Invasive and may miss Location of scanner away Operator dependent and
retroperitoneal and from facilities, time taken may miss
diaphragmatic injury for reporting, need for retroperitoneal injury
contrast

 Amylase may be normal following pancreatic trauma


 Urethrography if suspected urethral injury

Next question
A 62 year old woman presents with acute bowel obstruction. She has been vomiting up to 15 times a
day and is taking erythromycin. She suddenly complains of dizziness. Her ECG shows torsades de
pointes. What is the management of choice?

IV Atropine

IV Potassium

IV Magnesium sulphate

IV Bicarbonate

IV Adrenaline

Torsades de pointes: Treatment IV magnesium sulphate

This woman is likely to have hypokalaemia and hypomagnasaemia as a result of vomiting. In


addition to this, the erythromycin will predispose her to torsades de pointes. The patient needs
Magnesium 2g over 10 minutes. Knowledge of the management of this peri arrest diagnosis is
hence important in surgical practice.
Please rate this question:

Discuss and give feedback


Next question

Torsades de pointes

Torsades de pointes ('twisting of the points') is a rare arrhythmia associated with a long QT interval.
It may deteriorate into ventricular fibrillation and hence lead to sudden death

Causes of long QT interval

 congenital: Jervell-Lange-Nielsen syndrome, Romano-Ward syndrome


 antiarrhythmics: amiodarone, sotalol, class 1a antiarrhythmic drugs
 tricyclic antidepressants
 antipsychotics
 chloroquine
 terfenadine
 erythromycin
 electrolyte: hypocalcaemia, hypokalaemia, hypomagnesaemia
 myocarditis
 hypothermia
 subarachnoid haemorrhage

Management

 IV magnesium sulphate

Next question
A 27 year old man sustains a single gunshot wound to the left thigh. In the emergency department,
he is noted to have a large haematoma of his medial thigh. He complains of parasthesia in his foot.
On examination, there are weak pulses palpable distal to the injury and the patient is unable to move
his foot. The appropriate initial management of this patient is:

Conventional angiography

Immediate exploration and repair

Fasciotomy of the anterior compartment

Observation for resolution of spasm

Local wound exploration

The five P's of arterial injury include pain, parasthesias, pallor, pulselessness and paralysis. In the
extremities, the tissues most sensitive to anoxia are the peripheral nerves and striated muscle. The
early developments of paresthesias and paralysis are signals that there is significant ischemia
present, and immediate exploration and repair are warranted. The presence of palpable pulse does
not exclude an arterial injury because this presence may represent a transmitted pulsation through a
blood clot. When severe ischemia is present, the repair must be completed within 6 to 8 h to prevent
irreversible muscle ischemia and loss of limb function. Delay to obtain a conventional angiogram or
to observe for change needlessly prolongs the ischemic time. A CT angiogram may be a reasonable
alternative. Fasciotomy may be required but should be done in conjunction with and after re-
establishment of arterial flow. Local wound exploration is not recommended because brisk
hemorrhage may be encountered without the securing of prior vascular control.
Please rate this question:

Discuss and give feedback


Next question

Vascular trauma

Peripheral and central vessels may be injured by blunt, penetrating or shearing injuries. Fractures of
bones close to vessels may also be associated with vascular injury or vessel occlusion.

Assessment

 Check for signs of distal perfusion


 Doppler signal distally (monophasic/ biphasic or triphasic)
 Anatomical location (which vessel is likely to be involved)
 Duplex scanning and angiography are "gold standard" tests but may not be immediately
available in the trauma setting

Management

 Almost always operative.


 Obtaining proximal and distal control of affected vessels is crucial.
 Simple lacerations of arteries may be directly closed, or a vein patch applied if there is a risk
of subsequent stenosis.
 Transection of the vessel should be treated by either end to end anastomosis (often not
possible) or an interposition vein graft.
 Use of PTFE in traumatic open injuries will invariably result in infection.

Next question
Theme: Paediatric emergencies

A. Manage conservatively
B. Immediate emergency theatre
C. Treat in emergency department
D. Treat in emergency department under sedation
E. Operate on next emergency list

Please select the most appropriate intervention for the scenario given. Each option may be used
once, more than once or not at all.

21. A 3 year old child inserts a crayon into their external auditory meatus. Attempts to remove
it have not been successful.

You answered Manage conservatively

The correct answer is Operate on next emergency list

Theme from September 2011 Exam


They would not tolerate removal in the emergency department. The tympanic membrane
should be carefully inspected and again this will be easier under general anaesthesia.

22. A 2 year old accidentally inhales a peanut. They arrive in the emergency department
extremely distressed and cyanotic. Imaging shows it to be lodged in the left main
bronchus.

You answered Manage conservatively

The correct answer is Immediate emergency theatre

As they are cyanosed it requires immediate removal and this should be undertaken in a
fully staffed theatre. Ideally a rigid bronchoscopy should be performed.

23. A 10 year old boy is shot in the head with an airgun pellet. He is concerned that he will get
into trouble and the injury remains concealed for 10 days. Imaging using CT scanning
shows it to be lodged in the frontal lobe.

Manage conservatively

The pellet is small and no serious injury has occurred at this stage. This should therefore
be managed conservatively.

Please rate this question:


Discuss and give feedback
Next question

Management of acute cases- Paediatric

 Children will often insert objects into orifices such as the nose and external auditory meatus
 Assessment includes assessment of airway and haemodynamic status
 Where the airway is not immediately threatened decisions can be made as to whether to
manage in the emergency department or transfer to theatre
 In general children do not tolerate procedures well and it is usually safer to remove objects in
theatre and under general anaesthesia with a secure airway
 A chest x-ray is required to ensure that no object is present in the chest, not all objects are
radioopaque. However, signs such as focal consolidation may indicate small airway
obstruction
 In the case of small bore missile injuries the decision relating to surgery depends on the size
of the missile and its location. Airgun pellets are a common culprit, if there is a long time
interval between the incident and presentation and the object has not caused any significant
problems then it may be best left alone
 Airgun pellets (and glass) lodged in the soft tissues are usually notoriously difficult to localise
and extract, no matter how superficial. Removal in theatre is usually the best option. If the
object is radioopaque then an image intensifier should be used

Next question
A 32 year old male is receiving a blood transfusion after being involved in a road traffic accident. A
few minutes after the transfusion he complains of loin pain. His observations show temperature
39 oC, HR 130bpm and blood pressure is 95/40mmHg. What is the best test to confirm his
diagnosis?

USS abdomen

Direct Coomb's test

Blood cultures

Blood film

Sickle cell test

Mnemonic for transfusion reactions:

Got a bad unit

G raft vs. Host disease


O verload
T hrombocytopaenia

A lloimmunization

B lood pressure unstable


A cute haemolytic reaction
D elayed haemolytic reaction

U rticaria
N eutrophilia
I nfection
T ransfusion associated lung injury

The diagnosis is of an acute haemolytic transfusion reaction, normally due to ABO incompatibility.
Haemolysis of the transfused cells occurs causing the combination of shock, haemoglobinaemia and
loin pain. This may subsequently lead to disseminated intravascular coagulation. A Coomb's test
should confirm haemolysis. Other tests for haemolysis include: unconjugated bilirubin, haptoglobin,
serum and urine free haemoglobin.
Note that delayed haemolytic reactions are normally associated with antibodies to the Rh system
and occur 5-10 days after transfusion.

Please rate this question:

Discuss and give feedback

Next question

Blood transfusion reactions

Acute transfusion reactions present as adverse signs or symptoms during or within 24 hours of a
blood transfusion. The most frequent reactions are fever, chills, pruritus, or urticaria, which typically
resolve promptly without specific treatment or complications. Other signs occurring in temporal
relationship with a blood transfusion, such as severe dyspnoea, pyrexia, or loss of consciousness
may be the first indication of a more severe potentially fatal reaction.
The causes of adverse reactions are multi-factorial. Immune mediated reactions, some of the most
feared, occur as a result of component mismatch, the commonest cause of which is clerical error.
More common, non immune mediated, complications may occur as a result of product
contamination, this may be bacterial or viral.
Transfusion related lung injury is well recognised and there are two proposed mechanisms which
underpin this. One involves the sequestration of primed neutrophils within the recipient pulmonary
capillary bed. The other proposed mechanism suggests that HLA mismatches between donor
neutrophils and recipient lung tissue is to blame.
The table below summarises the main types of transfusion reaction.

Immune mediated Non immune mediated

Pyrexia Hypocalcaemia

Alloimmunization CCF

Thrombocytopaenia Infections

Transfusion associated lung injury Hyperkalaemia


Immune mediated Non immune mediated

Graft vs Host disease

Urticaria

Acute or delayed haemolysis

ABO incompatibility

Rhesus incompatibility

Next question
A 45 year old man complains of sharp chest pain. He is due to have elective surgery to replace his
left hip. He has been bed bound for 3 months. He suddenly collapses; his blood pressue is
70/40mmHg, heart rate 120 bpm and his saturations are 74% on air. He is deteriorating in front of
you. What is the next best management plan?

Aspirin

Thrombolysis with Alteplase

Unfractionated heparin

Thrombolysis with streptokinase

Clopidogrel

This man is peri arrest with the diagnosis of pulmonary embolism (chest pain,bedbound, collapse,
low saturations). He needs urgent thrombolysis with alteplase (he may not survive if you wait for the
medical Spr/ITU to arrive!).
Please rate this question:

Discuss and give feedback


Next question

Pulmonary embolism: management

A summary of the British Thoracic Society guidelines

 Heparin should be given if intermediate or high clinical probability before imaging.


 Unfractionated heparin (UFH) should be considered (a) as a first dose bolus, (b) in massive
PE, or (c) where rapid reversal of effect may be needed.
 Otherwise, low molecular weight heparin (LMWH) should be considered as preferable to
UFH, having equal efficacy and safety and being easier to use.
 Oral anticoagulation should only be commenced once VTE has been reliably confirmed.
 The target INR should be 2.0-3.0; when this is achieved, heparin can be discontinued.
 The standard duration of oral anticoagulation is: 4 to 6 weeks for temporary risk factors, 3
months for first idiopathic, and at least 6 months for other; the risk of bleeding should be
balanced with that of further VTE.

Massive PE
 CTPA or echocardiography will reliably diagnose clinically massive PE.
 Thrombolysis is 1st line for massive PE (ie circulatory failure) and may be instituted on
clinical grounds alone if cardiac arrest is imminent; a 50 mg bolus of alteplase is
recommended.
 Invasive approaches (thrombus fragmentation and IVC filter insertion) should be considered
where facilities and expertise are readily available.

Next question
A 30 year old woman, who is 30 weeks pregnant, attends the varicose vein clinic. The patient is
normally fit and well, with no lung disorders. She suddenly complains of shortness of breath and
chest pain. She has no underlying lung condition. Her saturations are 92 % air, blood pressure
150/80 mmHg and her chest sounds clear. What would be the main investigation recommended to
establish her diagnosis?

Lung spirometry

Half dose scintigraphy

CTPA

Echocardiogram

Full dose scintigraphy

The main differential diagnosis is pulmonary embolism. A CXR should be performed first in second
and third trimester to exclude other diagnoses such as pneumothorax or pneumonia. Concerns
surrounding radiation exposure by CXR have been discounted at this stage, as the need to establish
a diagnosis is the major priority. If the chest x-ray is normal, then half dose scintigraphy should be
performed. If there is underlying lung pathology, then a CTPA is performed.

This is a controversial topic area, however the answer is related to guidance from the Royal College
of Obstetricians and Gynaecologists. The official line is that half dose scintigraphy is undertaken in
those with normal CXR (or no chest signs) and no pre- existing lung disease. The reason for this is
the significant subsequent increase in risk of maternal breast cancer with CTPA (lifetime risk
increased by up to 13.6% with CTPA, background risk of 1/200 for study population) vs scintigraphy.
The vast majority of female patients are not found to have a PE which is also an important
component. However, we recognise that in most hospitals a CTPA is still performed first line.
Reference
The acute management of thrombosis and embolism during pregnancy and the puerperium RCOG
guidelines February 2007

Scarsbrook A.Fand Gleeson V. Investigating suspected pulmonary embolism in


pregnancy. BMJ 2007 (326) : 1135 doi: 10.1136/bmj.7399.1135.
Please rate this question:

Discuss and give feedback


Next question

Chest pain in pregnancy


Aortic dissection

 Predisposing factors in pregnancy are hypertension, congenital heart disease and Marfan's
syndrome
 Mainly Stanford type A dissections
 Sudden tearing chest pain, transient syncope
 Patient may be cold and clammy, hypertensive and have an aortic regurgitation murmur
 Involvement of the right coronary artery may cause inferior myocardial infarction

Surgical management
Gestational timeframe Management

< 28/40 Aortic repair with the fetus kept in utero

28-32/40 Dependent on fetal condition

> 32/40 Primary Cesarean section followed by aortic repair at the same operation

Mitral stenosis

 Most cases associated with rheumatic heart disease


 Becoming less common in British women; suspect in Immigrant women
 Commonest cardiac condition in pregnancy
 Commonly associated with mortality
 Valve surgery; balloon valvuloplasty preferable

Pulmonary embolism

 Leading cause of mortality in pregnancy


 Half dose scintigraphy; CT chest if underlying lung disease, should aid diagnosis
 Treatment with low molecular weight heparin throughout pregnancy and 4-6 weeks after
childbirth
 Warfarin is contra indicated in pregnancy (though may be continued in women with
mechanical heart valves due to the significant risk of thromboembolism)

References
1. Bates S.M. and Ginsberg J.S. How we manage venous thromboembolism during
pregnancy. Blood2002 (100): 3470-3478.
2. Scarsbrook A.Fand Gleeson V. Investigating suspected pulmonary embolism in
pregnancy. BMJ2007 (326) : 1135 doi: 10.1136/bmj.7399.1135.

3. Morley C. A. and Lim B. A. Lesson of the Week: The risks of delay in diagnosis of breathlessness
in pregnancy. BMJ 1995 (311) : 1083.
Next question
Theme: Bleeding disorders

A. Vitamin K deficiency

B. von Willebrand's disease

C. Acquired haemophilia

D. Haemophilia B

E. Protein C deficiency

F. Disseminated intravascular coagulation

G. Factor V Leiden

H. Excess heparin

I. Warfarin overdose

J. Antiphospholipid syndrome

What is the most likely diagnosis for the scenario given? Each option may be used once, more than
once or not at all.

27. A 33 year old female is admitted for varicose vein surgery. She is fit and well. After the procedure
she is persistently bleeding. She is known to have menorrhagia. Investigations show a prolonged
bleeding time and increased APTT. She has a normal PT and platelet count.

You answered Vitamin K deficiency

The correct answer is von Willebrand's disease

Bleeding post operatively, epistaxis and menorrhagia may indicate a diagnosis of vWD.
Haemoarthroses are rare. The bleeding time is usually normal in haemophilia (X-linked) and
vitamin K deficiency.

28. A 70 year old heavy smoker presents with 3 weeks of haematuria and bruising. He is normally fit
and well. He is on no medications. His results reveal:
Hb 9.0
WCC 11
Pl 255
PT 16 (normal)
APTT 58 (increased)
Thrombin time 20 (normal).

You answered Vitamin K deficiency

The correct answer is Acquired haemophilia

This patient has Factor 8 acquired disorder. He is likely to have developed a lung malignancy
(smoker) and as a result aquired a haemophilia disorder. The elderly, pregnancy, malignancy and
autoimmune conditions are associated with acquired haemophilia. Prolonged APTT is key to the
diagnosis. Management involves steroids.

29. A 28 year old female is attends the gynaecology unit for a D+C following an incomplete
miscarriage. She has previously had recurrent pulmonary embolic events. After the procedure she
is persistently bleeding. Her APTT is 52 (increased).

You answered Vitamin K deficiency

The correct answer is Antiphospholipid syndrome

A combination of thromboembolism and bleeding in a young woman should raise the possibility
of antiphospholipid syndrome. Other features may include foetal loss, venous and arterial
thrombosis and thrombocytopenia. A Lupus anticoagulant may be present and the APTT is
prolonged.

Theme from January 2012 exam

Please rate this question:

Discuss and give feedback

Next question
Abnormal coagulation

Cause Factors affected

Heparin Prevents activation factors 2,9,10,11

Warfarin Affects synthesis of factors 2,7,9,10

DIC Factors 1,2,5,8,11

Liver disease Factors 1,2,5,7,9,10,11

Interpretation blood clotting test results

Disorder APTT PT Bleeding time

Haemophilia Increased Normal Normal

von Willebrand's disease Increased Normal Increased

Vitamin K deficiency Increased Increased Normal

Next question
A 54-year-old man is brought to the Emergency Department after being found collapsed in the
street. He is known to have a history of alcoholic liver disease. Blood tests reveal the following:

Calcium 1.62 mmol/l

Albumin 33 g/l

Which one of the following is the most appropriate management of the calcium result?

10ml of 10% calcium chloride over 10 minutes

20% albumin infusion

10ml of 50% calcium gluconate over 10 minutes

No action

10ml of 10% calcium chloride over 4 hours

Current UK ALS guidance is to use calcium chloride

Even after correction for the low albumin level this patient has significant hypocalcaemia which
should be corrected.
Please rate this question:

Discuss and give feedback


Next question

Hypocalcaemia: causes and management

The clinical history combined with parathyroid hormone levels will reveal the cause of
hypocalcaemia in the majority of cases

Causes

 Vitamin D deficiency (osteomalacia)


 Acute pancreatitis
 Chronic renal failure
 Hypoparathyroidism (e.g. post thyroid/parathyroid surgery)
 Pseudohypoparathyroidism (target cells insensitive to PTH)
 Rhabdomyolysis (initial stages)
 Magnesium deficiency (due to end organ PTH resistance)

Management

 Acute management of severe hypocalcaemia is with intravenous replacement. The preferred


method is with intravenous calcium chloride, 10ml of 10% solution over 10 minutes
 ECG monitoring is recommended
 Further management depends on the underlying cause
 Calcium and bicarbonate should not be administered via the same route

Next question
A 25 year old male pedestrian is hit by a van on a busy road. He is brought to the Emergency
Department by ambulance. On examination he is dyspneoic, and hypoxic despite administration of
high flow 100% oxygen. His blood pressure is 110/70 and pulse rate is 115 bpm. The right side of
his chest is hyper-resonant on percussion and has decreased breath sounds. The trachea is
deviated to the left. What is the most likely underlying diagnosis?

Fat embolism

Tension pneumothorax

Rupture of the right main bronchus

Rupture of the diaphragm

Pulmonary contusion

Blunt or penetrating chest trauma that creates a flap type defect on the surface of the lung can result
in a tension pneumothorax. Typical features include dyspnoea, progressive hypoxia,
hyperresonance and tracheal deviation. Treatment is with needle decompression and chest tube
insertion.
Please rate this question:

Discuss and give feedback


Next question

Thoracic trauma

Types of thoracic trauma

Tension  Often laceration to lung parenchyma with flap


pneumothorax  Pressure develops in thorax
 Most common cause is mechanical ventilation in patient with
pleural injury
 Symptoms overlap with cardiac tamponade, hyper-resonant
percussion note is more likely in tension pnemothorax

Flail chest  Chest wall disconnects from thoracic cage


 Multiple rib fractures (at least two fractures per rib in at least two
ribs)
 Associated with pulmonary contusion
 Abnormal chest motion
 Avoid over hydration and fluid overload

Pneumothorax  Most common cause is lung laceration with air leakage


 Most traumatic pneumothoraces should have a chest drain
 Patients with traumatic pneumothorax should never be
mechanically ventilated until a chest drain is inserted

Haemothorax  Most commonly due to laceration of lung, intercostal vessel or


internal mammary artery
 Haemothoraces large enough to appear on CXR are treated with
large bore chest drain
 Surgical exploration is warranted if >1500ml blood drained
immediately

Cardiac tamponade  Beck's triad: elevated venous pressure, reduced arterial pressure,
reduced heart sounds
 Pulsus paradoxus
 May occur with as little as 100ml blood

Pulmonary contusion  Most common potentially lethal chest injury


 Arterial blood gases and pulse oximetry important
 Early intubation within an hour if significant hypoxia

Blunt cardiac injury  Usually occurs secondary to chest wall injury


 ECG may show features of myocardial infarction
 Sequelae: hypotension, arrhythmias, cardiac wall motion
abnormalities

Aorta disruption  Deceleration injuries


 Contained haematoma
 Widened mediastinum

Diaphragm  Most due to motor vehicle accidents and blunt trauma causing
disruption large radial tears (laceration injuries result in small tears)
 More common on left side
 Insert gastric tube, may pass into intrathoracic stomach
Mediastinal  Entrance wound in one hemithorax and exit wound/foreign body
traversing wounds in opposite hemithorax
 Mediastinal haematoma or pleural cap suggests great vessel
injury
 Mortality is 20%

References
ATLS Manual 8th Edition
Next question
Theme: Management of skin injuries

A. Wound excision and primary closure


B. Simple primary closure
C. Delayed primary closure
D. Debridement and healing by secondary intention
E. Split thickness skin graft
F. Full thickness skin graft
G. Free flap
H. Pedicled flap
I. Debridement and rotational flap

For the injuries described please select the most appropriate management. Each option may be
used once, more than once or not at all.

32. A 32 year old man is involved in a road traffic accident and sustains a significant
laceration to the lateral aspect of the nose which is associated with tissue loss.

You answered Wound excision and primary closure

The correct answer is Debridement and rotational flap

Theme from April 2011 Exam


Nasal injuries can be challenging to manage and where there is tissue loss, it can be
difficult to primarily close them and still obtain a satisfactory aesthetic result. Debridement
together with a rotational flap would obtain the best results here.

33. A 7 year old boy falls over and sustains a 6cm laceration to his head. On inspection his
wound contains some dirt in it.

Wound excision and primary closure

By debriding the wound, the area can then be primarily closed. Prophylactic antibiotics
should be administered.

34. A 45 year old man is gardening and damages his foot with a fork. On examination there
are cutaneous defects and the surrounding skin looks dusky.

You answered Wound excision and primary closure

The correct answer is Debridement and healing by secondary intention

The skin changes described here should be debrided. Closure would not be safe with the
skin changes documented and the wound should be left open.

Please rate this question:

Discuss and give feedback


Next question

Methods of wound closure

Method of Indication
closure

Primary closure  Clean wound, usually surgically created or following minor trauma
 Standard suturing methods will usually suffice
 Wound heals by primary intention

Delayed primary  Similar methods of actual closure to primary closure


closure  May be used in situations where primary closure is either not
achievable or not advisable e.g. infection

Vacuum assisted  Uses negative pressure therapy to facilitate wound closure


closure  Sponge is inserted into wound cavity and then negative pressure
applied
 Advantages include removal of exudate and versatility
 Disadvantages include cost and risk of fistulation if used incorrectly
on sites such as bowel

Split thickness  Superficial dermis removed with Watson knife or dermatome


skin grafts (commonly from thigh)
 Remaining epithelium regenerates from dermal appendages
 Coverage may be increased by meshing

Full thickness skin  Whole dermal thickness is removed


grafts  Sub dermal fat is then removed and graft placed over donor site
 Better cosmesis and flexibility at recipient site
 Donor site "cost"

Flaps  Viable tissue with a blood supply


 May be pedicled or free
 Pedicled flaps are more reliable, but limited in range
 Free flaps have greater range but carry greater risk of breakdown as
they require vascular anastomosis

Next question
A 19 year old student is involved in a head on car collision. He complains of severe chest pain. A
Chest x-ray performed as part of a trauma series shows widening of the mediastinum. Which is the
most likely injury in this scenario?

Rupture of the distal oesophagus

Rupture of the left main bronchus

Rupture of the aorta proximal to the left subclavian artery

Rupture of the aorta distal to the left subclavian artery

Rupture of the inferior vena cava

The aorta may be injured in deceleration accidents. In the setting of deceleration injury, chest pain
and mediastinal widening the most likely problem is aortic rupture. This will typically occur distal to
the left subclavian artery. Rupture of the proximal aorta may occur. However, survival is unlikely. It is
important to note that the question uses the term Most likely injury as this is the component that
distinguishes an ascending rupture from a descending rupture.
Please rate this question:

Discuss and give feedback


Next question

Thoracic aorta rupture

 Mechanism of injury: Decelerating force i.e. RTA, fall from a great height
 Most people die at scene
 Survivors may have an incomplete laceration at the ligamentum arteriosum of the aorta.

Clinical features

 Contained haematoma: persistent hypotension


 Detected mainly by history, CXR changes

CXR changes
 Widened mediastinum
 Trachea/Oesophagus to right
 Depression of left main stem bronchus
 Widened paratracheal stripe/paraspinal interfaces
 Space between aorta and pulmonary artery obliterated
 Rib fracture/left haemothorax

Diagnosis
Angiography, usually CT aortogram.

Treatment
Repair or replacement. Ideally they should undergo endovascular repair.
Next question
Theme: Management of head and neck trauma

A. Observation
B. CT head within 1 hour
C. CT head within 8 hours
D. Urgent neurosurgical review (even before CT head performed)
E. 3 view c-spine xray
F. 2 view c-spine xray
G. CT c-spine

What is the best initial management plan for the injuries described? Each option may be used once,
more than once or not at all.

36. A 22 year old mechanic is involved in a fight. He is hit on the head with a hammer. On
examination he had clinical evidence of an open depressed skull fracture and a GCS of
6/15.

You answered Observation

The correct answer is Urgent neurosurgical review (even before CT head performed)

A patient with GCS <8 or = to 8 needs urgent neurosurgical review. Especially when an
open fracture is present.

37. A 67 year old retired lawyer falls down the stairs. His GCS is 15/15 and he has some
bruising over the mastoid.

You answered Observation

The correct answer is CT head within 1 hour

This patient has a basal skull fracture, which is indicated by a positive Battle's sign. He
should have a CT head within 1h.

38. A 52 year old secretary falls down the stairs. She complains of neck pain. She has a GCS
of 15/15 and no neurology. She is unable to rotate her c-spine 45 degrees to the left and
right.

You answered Observation

The correct answer is 3 view c-spine xray

In the January 2014 NICE guidance relating to the diagnosis of head and spinal injury the
evidence for initial imaging of the C spine was reviewed. The current UK practice is that
"low risk" patients with pain but no neurology undergo a 3 view C spine x-ray. The more
detailed 5 view x-ray was not found to be any better than 3 view films. In patients with
high risk injuries (which the patient in the scenario does not have) there is a likelihood that
1 in 6 injuries would be missed on plain films alone and therefore CT scanning of the C
spine is recommended in this group.

Please rate this question:

Discuss and give feedback


Next question

Head injury management- NICE Guidelines

Summary of guidelines

 All patients should be assessed within 15 minutes on arrival to A&E


 Document all 3 components of the GCS
 If GCS <8 or = to 8, consider stabilising the airway
 Treat pain with low dose IV opiates (if safe)
 Full spine immobilisation until assessment if:

- GCS < 15
- neck pain/tenderness
- paraesthesia extremities
- focal neurological deficit
- suspected c-spine injury

If a c-spine injury is suspected a 3 view c-spine x-ray is indicated. CT c-spine is preferred if:
- Intubated
- GCS <13
- Normal x-ray but continued concerns regarding c-spine injury
- Any focal neurology
- A CT head scan is being performed
- Initial plain films are abnormal

Immediate CT head (within 1 hour) if:

 GCS < 13 on admission


 GCS < 15 2 hours after admission
 Suspected open or depressed skull fracture
 Suspected skull base fracture (panda eyes, Battle's sign, CSF from nose/ear, bleeding ear)
 Focal neurology
 Vomiting > 1 episode
 Post traumatic seizure
 Coagulopathy

Contact neurosurgeon if:

 Persistent GCS < 8 or = 8


 Unexplained confusion > 4h
 Reduced GCS after admission
 Progressive neurological signs
 Incomplete recovery post seizure
 Penetrating injury
 Cerebrospinal fluid leak

Observations

 1/2 hourly GCS until 15

Reference
1. http://guidance.nice.org.uk/CG56/QuickRefGuide/pdf/English
2. Hodgkinson S et al. Early management of head injury: summary of NICE guidance. BMJ 2014
(348):34-37.
Next question
A 28 year old African man is admitted with acute severe abdominal pain. He has just flown into the
UK long haul and the pain developed whilst in flight. On examination he is tender in the left upper
quadrant. His blood tests are as shown.

Hb 6 g/dl
Reticulocyte count 15%.
Ultrasound shows a spleen with a heterogeous texture and a few small gallstones but is otherwise
normal.
What is the most likely diagnosis?

Pancreatitis

Parvovirus infection

Sickle cell anaemia

Pulmonary embolism

Beta Thalassaemia minor

A combination of a high reticulocyte count and severe anaemia indicates sickle cell anaemia,
however another differential can be of a transient aplastic crisis due to parvovirus. This is less likely
as this causes a reticulocytopenia rather than a reticulocytosis.

Parvovirus B19 infects erythroid progenitor cells in the bone marrow and causes temporary
cessation of red blood cell production, patients who have underlying hematologic abnormalities are
at risk of cessation of red blood cell production if they become infected. This can result in a transient
aplastic crisis. Thus, patients with sickle cell anaemia are at risk. Typically, these patients have a
viral prodrome followed by anaemia, often with haemoglobin concentrations falling below 5.0 g/dL
and reticulocytosis.
Please rate this question:

Discuss and give feedback


Next question

Sickle cell anaemia

 Autosomal recessive
 Single base mutation
 Deoxygenated cells become sickle in shape
 Causes: short red cell survival, obstruction of microvessels and infarction
 Sickling is precipitated by: dehydration, infection, hypoxia
 Manifest at 6 months age
 Africans, Middle East, Indian
 Diagnosis: Hb electrophoresis

Sickle crises

 Bone pain
 Pleuritic chest pain: acute sickle chest syndrome commonest cause of death
 CVA, seizures
 Papillary necrosis
 Splenic infarcts
 Priapism
 Hepatic pain

Hb does not fall during a crisis, unless there is

 Aplasia: parvovirus
 Acute sequestration
 Haemolysis

Long-term complications

 Infections: Streptococcus pnemoniae


 Chronic leg ulcers
 Gallstones: haemolysis
 Aseptic necrosis of bone
 Chronic renal disease
 Retinal detachment, proliferative retinopathy

Surgical complications

 Bowel ischaemia
 Cholecystitis
 Avascular necrosis

Management
 Supportive
 Hydroxyurea
 Repeated transfusions pre operatively
 Exchange transfusion in emergencies

Sickle cell trait

 Heterozygous state
 Asymptomatic
 Symptoms associated with extreme situations ie anaesthesia complications
 Protective against Plasmodium falciparum

Next question
Theme: Blood transfusion reactions

A. Neutrophilic febrile reaction

B. Acute haemolytic transfusion reaction

C. Delayed haemolytic transfusion reaction

D. Pulmonary oedema

E. Sickle cell crisis

F. Transfusion associated lung injury

G. Graft vs. Host disease

Please select the most likely underlying cause for each scenario. Each option may be used once,
more than once or not at all.

40. A 22 year old man is having a blood transfusion after losing blood from a peptic ulcer. He is
normally fit and well. Four hours after the transfusion; he complains of sudden onset shortness of
breath and chest pain. On examination his temperature is 37.2, saturations are 88% on air, blood
pressure 100/55 mmHg and HR 110 bpm. He has crepitations bilaterally up to the midzones. He is
given IV frusemide, but deteriorates and is admitted to ITU. A pulmonary catheter is inserted and
the PCWP is 10 mmHg.

You answered Neutrophilic febrile reaction

The correct answer is Transfusion associated lung injury

The pulmonary catheter reading indicates that this is not a case of fluid overload (the PCWP
should be high, normal values PCWP systolic 7mmHg, diastolic 10mmHg). Transfusion associated
lung injury is a rare reaction causing neutrophilic mediated allergic pulmonary oedema. Patient's
have antibodies to donor leukocytes. It is important to consider this as a diagnosis when patients
don't respond to treatment for pulmonary oedema. Patients normally respond to supportive
therapy including fluids and oxygen.

41. A 32 year male with leukaemia attends the day unit for a blood transfusion. Five days after the
transfusion he attends A&E with a temperature of 38.5, erythroderma and desquamation.
You answered Neutrophilic febrile reaction

The correct answer is Graft vs. Host disease

This is associated with transfusion of unirradiated blood in immunosupressed patients.


Transfusion associated GVHD can occur 4-30 days after a transfusion and follows a sub acute
pathway. Patients may also have diarrhoea and abnormal liver function tests. Management
involves steroid therapy.

42. A 22 year old man is having a blood transfusion after losing blood from haemorrhoids. He is
normally fit and well. 3h during the transfusion he complains of sudden onset abdominal pain and
nausea. His temperature is 39 degrees, Blood pressure 98/42 mmHg, HR 105 bpm and saturations
94% air. His urine appears dark.

You answered Neutrophilic febrile reaction

The correct answer is Acute haemolytic transfusion reaction

Rapid intravascular haemolysis leading to shock, DIC and death can occur with this reaction.

Please rate this question:

Discuss and give feedback

Next question

Blood transfusion reactions

Acute transfusion reactions present as adverse signs or symptoms during or within 24 hours of a
blood transfusion. The most frequent reactions are fever, chills, pruritus, or urticaria, which typically
resolve promptly without specific treatment or complications. Other signs occurring in temporal
relationship with a blood transfusion, such as severe dyspnoea, pyrexia, or loss of consciousness
may be the first indication of a more severe potentially fatal reaction.
The causes of adverse reactions are multi-factorial. Immune mediated reactions, some of the most
feared, occur as a result of component mismatch, the commonest cause of which is clerical error.
More common, non immune mediated, complications may occur as a result of product
contamination, this may be bacterial or viral.
Transfusion related lung injury is well recognised and there are two proposed mechanisms which
underpin this. One involves the sequestration of primed neutrophils within the recipient pulmonary
capillary bed. The other proposed mechanism suggests that HLA mismatches between donor
neutrophils and recipient lung tissue is to blame.
The table below summarises the main types of transfusion reaction.

Immune mediated Non immune mediated

Pyrexia Hypocalcaemia

Alloimmunization CCF

Thrombocytopaenia Infections

Transfusion associated lung injury Hyperkalaemia

Graft vs Host disease

Urticaria

Acute or delayed haemolysis

ABO incompatibility

Rhesus incompatibility

Next question
A 49-year-old male sustained a severe blunt injury just below the bridge of the nose with industrial
machinery. Imaging demonstrates a fracture involving the superior orbital fissure. On examination an
ipsilateral pupillary defect is present and loss of the corneal reflexes. In addition to these
examination findings, which of the following will not be present?

Altered cutaneous sensation from the forehead to the vertex

Ptosis

Complete opthalmoplegia

Nystagmus

Enopthalmos

Orbital apex syndrome


This is an extension of superior orbital fissure syndrome and includes compression of the optic nerve
passing through the optic foramen. It is indicated by features of superior orbital fissure syndrome
and ipsilateral afferent pupillary defect.

This type of injury will result in the orbital apex syndrome (See above). As such opthalmoplegia will
be present and nystagmus cannot occur.
Please rate this question:

Discuss and give feedback


Next question

Craniomaxillofacial injuries

Craniomaxillofacial injuries in the UK are due to:

 Interpersonal violence (52%)


 Motor vehicle accidents (16%)
 Sporting injuries (19%)
 Falls (11%)

Le Fort Fractures
Grade Feature
Grade Feature

Le The fracture extends from the nasal septum to the lateral pyriform rims, travels
Fort 1 horizontally above the teeth apices, crosses below the zygomaticomaxillary junction, and
traverses the pterygomaxillary junction to interrupt the pterygoid plates.

Le These fractures have a pyramidal shape and extend from the nasal bridge at or below the
Fort 2 nasofrontal suture through the frontal process of the maxilla, inferolaterally through the
lacrimal bones and inferior orbital floor and rim through or near the inferior orbital
foramen, and inferiorly through the anterior wall of the maxillary sinus; it then travels
under the zygoma, across the pterygomaxillary fissure, and through the pterygoid plates.

Le These fractures start at the nasofrontal and frontomaxillary sutures and extend posteriorly
Fort 3 along the medial wall of the orbit through the nasolacrimal groove and ethmoid bones.
The thicker sphenoid bone posteriorly usually prevents continuation of the fracture into
the optic canal. Instead, the fracture continues along the floor of the orbit along the
inferior orbital fissure and continues superolaterally through the lateral orbital wall,
through the zygomaticofrontal junction and the zygomatic arch. Intranasally, a branch of
the fracture extends through the base of the perpendicular plate of the ethmoid, through
the vomer, and through the interface of the pterygoid plates to the base of the sphenoid.
This type of fracture predisposes the patient to CSF rhinorrhea more commonly than the
other types.

Ocular injuries
Superior orbital fissure syndrome
Severe force to the lateral wall of the orbit resulting in compression of neurovascular structures.
Results in :

 Complete opthalmoplegia and ptosis (Cranial nerves 3, 4, 6 and nerve to levator palpebrae
superioris)
 Relative afferent pupillary defect
 Dilatation of the pupil and loss of accommodation and corneal reflexes
 Altered sensation from forehead to vertex (frontal branch of trigeminal nerve)

Orbital blow out fracture


Typically occurs when an object of slightly larger diameter than the orbital rim strikes the
incompressible eyeball. The bone fragment is displaced downwards into the antral cavity, remaining
attached to the orbital periosteum. Periorbital fat may be herniated through the defect, interfering
with the inferior rectus and inferior oblique muscles which are contained within the same fascial
sheath. This prevents upward movement and outward rotation of the eye and the patient
experiences diplopia on upward gaze. The initial bruising and swelling may make assessment
difficult and patients should usually be reviewed 5 days later. Residual defects may require orbital
floor reconstruction.

Nasal Fractures

 Common injury
 Ensure new and not old deformity
 Control epistaxis
 CSF rhinorrhoea implies that the cribriform plate has been breached and antibiotics will be
required.
 Usually best to allow bruising and swelling to settle and then review patient clinically. Major
persistent deformity requires fracture manipulation, best performed within 10 days of injury.

Retrobulbar haemorrhage
Rare but important ocular emergency. Presents with:

 Pain (usually sharp and within the globe)


 Proptosis
 Pupil reactions are lost
 Paralysis (eye movements lost)
 Visual acuity is lost (colour vision is lost first)

May be the result of Le Fort type facial fractures.

Management:

 Mannitol 1g/Kg as 20% infusion, Osmotic diuretic, Contra-indicated in congestive heart


failure and pulmonary oedema
 Acetazolamide 500mg IV, (Monitor FBC/U+E) Reduces aqueous pressure by inhibition of
carbonic anhydrase (used in glaucoma)
 Dexamethasone 8mg orally or intravenously
 In a traumatic setting an urgent cantholysis may be needed prior to definitive surgery.

Consider
Papaverine 40mg smooth muscle relaxant
Dextran 40 500mls IV improves perfusion
Next question
Which option is not recommended during the management of compartment syndrome?

Anticoagulation

Keep limb level with the body

Intravenous fluids

Pain control

Fasciotomy

Anticoagulation will worsen compartment syndrome.


Please rate this question:

Discuss and give feedback


Next question

Compartment syndrome

 This is a particular complication that may occur following fractures (or following ischaemia re-
perfusion injury in vascular patients). It is characterised by raised pressure within a closed
anatomical space.
 The raised pressure within the compartment will eventually compromise tissue perfusion
resulting in necrosis. The two main fractures carrying this complication include supracondylar
fractures and tibial shaft injuries.

Symptoms and signs

 Pain, especially on movement (even passive)


 Parasthesiae
 Pallor may be present
 Arterial pulsation may still be felt as the necrosis occurs as a result of microvascular
compromise
 Paralysis of the muscle group may occur

Diagnosis
 Is made by measurement of intracompartmental pressure measurements. Pressures in
excess of 20mmHg are abnormal and >40mmHg is diagnostic.

Treatment

 This is essentially prompt and extensive fasciotomies


 In the lower limb the deep muscles may be inadequately decompressed by the
inexperienced operator when smaller incisions are performed
 Myoglobinuria may occur following fasciotomy and result in renal failure and for this reason
these patients require aggressive IV fluids
 Where muscle groups are frankly necrotic at fasciotomy they should be debrided and
amputation may have to be considered
 Death of muscle groups may occur within 4-6 hours

Next question
Theme: Complications of burns

A. Deep vein thrombosis


B. Curlings Ulcer
C. Contracture
D. Type I respiratory failure
E. Type II respiratory failure
F. Toxic shock syndrome
G. Compartment syndrome
H. Rhabdomyolysis
I. Disseminated intravascular coagulation

For each clinical scenario please select the most likely complication to have occurred. Each option
may be used once, more than once or not at all.

45. A 10 year old child is admitted with severe 30% burns following a house fire. After wound
cleaning and dressings he is admitted to critical care. 1 day following skin grafts he
becomes tachycardic and hypotensive. He vomits twice and this shows evidence of
haematemesis

You answered Deep vein thrombosis

The correct answer is Curlings Ulcer

Stress ulcers may occur in the duodenum of burns patients and are more common in
children.

46. A 26 year old electrician suffers a full thickness high voltage burn to his leg. On routine
urine analysis he has + blood. His U+E's show mild hyperkalaemia and a CK of 3000

You answered Deep vein thrombosis

The correct answer is Rhabdomyolysis

Electrical high voltage burns are associated with rhabdomyolysis. Acute tubular necrosis
may occur. Aggressive IV fluids should be given

47. A 45 year old man is admitted after his clothing caught fire. He suffers a full thickness
circumferential burn to his lower thigh. He complains of increasing pain in lower leg and
on examination there is parasthesia and severe pain in the lower leg. Foot pulses are
normal

You answered Deep vein thrombosis


The correct answer is Compartment syndrome

Circumferential burns may constrict the limb and cause a compartment syndrome to
develop. Eshcarotomy is required, and compartmental decompression.

Please rate this question:

Discuss and give feedback


Next question

Burns

Burns may be thermal, chemical or electrical. In the former category are burns which occur as a
result of heat. Chemical burns occur when the skin is exposed to an extremely caustic or alkaline
substance. Electrical burns occur following exposure to electrical current. The immediate
management includes removal of the burning source which usually includes irrigation of the burned
area. A detailed assessment then needs to be made of the extent of the burns and a number of
charts are available for recording this information. The degree of injury relates to the temperature
and duration of exposure. Most domestic burns are mainly scalds in young children.

Following the burn, there is a local response with progressive tissue loss and release of
inflammatory cytokines. Systemically, there are cardiovascular effects resulting from fluid loss and
sequestration of fluid into the third space. There is a marked catabolic response. Immunosupression
is common with large burns and bacterial translocation from the gut lumen is a recognised event.
Sepsis is a common cause of death following major burns.

Types of burn

Type of burn Skin layers Skin Blanching Management


affected appearance

Epidermal/Superficial Epidermis Red, moist Yes

Superficial partial Epidermis and part Pale, dry Yes Normally heals
thickness of papillary dermis with no
affected intervention

Deep partial thickness Epidermis, whole Mottled red No Needs surgical


papillary dermis colour intervention
affected (depending on site)
Full thickness Whole skin layer Dry, leathery No Burns centre
and subcutaneous hard wound
tissue affected

Depth of burn assessment

 Bleeding on needle prick


 Sensation
 Appearance
 Blanching to pressure

Percentage burn estimation


Lund Browder chart: most accurate even in children
Wallace rule of nines
Palmar surface: surface area palm = 0.8% burn

>15% body surface area burns in adults needs urgent burn fluid resuscitation

Transfer to burn centre if:

 Need burn shock resuscitation


 Face/hands/genitals affected
 Deep partial thickness or full thickness burns
 Significant electrical/chemical burns

Management
The initial aim is to stop the burning process and resuscitate the patient. Intravenous fluids will be
required for children with burns greater than 10% of total body surface area. Adults with burns
greater than 15% of total body surface area will also require IV fluids. The fluids are calculated using
the Parkland formula which is; volume of fluid= total body surface area of the burn % x weight (Kg)
x4. Half of the fluid is administered in the first 8 hours. A urinary catheter should be inserted.
Analgesia should be given. Complex burns, burns involving the hand perineum and face and burns
>10% in adults and >5% in children should be transferred to a burns unit.

Circumferential burns affecting a limb or severe torso burns impeding respiration may require
escharotomy to divide the burnt tissue.

Conservative management is appropriate for superficial burns and mixed superficial burns that will
heal in 2 weeks. More complex burns may require excision and skin grafting. Excision and primary
closure is not generally practised as there is a high risk of infection.

There is no evidence to support the use of anti microbial prophylaxis or topical antibiotics in burn
patients.

Escharotomies
 Indicated in circumferential full thickness burns to the torso or limbs.
 Careful division of the encasing band of burn tissue will potentially improve ventilation (if the
burn involves the torso), or relieve compartment syndrome and oedema (where a limb is
involved)

References
www.euroburn.org/e107files/downloads/guidelinesburncare.pdf

Barajas-Nava LA, López-Alcalde J, Roqué i Figuls M, Solà I, Bonfill Cosp X. Antibiotic prophylaxis
for preventing burn wound infection. Cochrane Database of Systematic Reviews 2013, Issue 6. Art.
No.: CD008738. DOI: 10.1002/14651858.CD008738.pub2.

Hettiaratchy S & Papini R. Initial management of a major burn: assessment and resuscitation. BMJ
2004;329:101-103
Next question
A 23 year old man who plays rugby for a hobby presents with recurrent anterior dislocation of the
shoulder. Which of the following abnormalities is most likely to be present to account for this?

Rotator cuff tear

Biceps tendon rupture

Bankart lesion

Axillary nerve injury

Infraspinatus tendinitis

A Bankart lesion is an injury of the anterior (inferior) glenoid labrum of the shoulder due to anterior
shoulder dislocation. When this happens, a pocket at the front of the glenoid forms that allows the
humeral head to dislocate into it.

Anterior dislocations are the most common. When recurrent, a Bankart lesion is the most common
underlying abnormality. This is usually visualised by CT and MRI scanning and often repaired
arthroscopically.
Please rate this question:

Discuss and give feedback


Next question

Shoulder disorders

Shoulder fractures and dislocations


Fractures
Proximal humerus
Background

 Third most common fragility fracture in the elderly.


 Results from low energy fall in predominantly elderly females, or from high energy trauma in
young males.
 Can be associated with nerve injury (commonly axillary), and fracture-dislocation of the
humeral head. Detailed neurological assessment is essential for all upper limb injuries.

Anatomy
Osteology
Consists of articular head, greater tuberosity, lesser tuberosity, metaphysis and diaphysis. Between
the articular head and the tuberosities is the anatomical neck (previous physis). Between the
tuberosities and the metaphysis is the surgical neck.
The supraspinatus, infraspinatus and teres minor muscles attach to the greater tuberosity. The
subscapularis muscle attaches to the lesser tuberosity.

Vascular Supply
Humeral head is supplied by the anterior and posterior humeral circumflex arteries. Anatomical neck
fractures are at greatest risk of osteonecrosis.

Imaging
Imaging aims to both delineate the fracture pattern, and confirm/exlude the presence of an
associated dislocation.

 Radiographs - True anteroposterior (AP), axillary lateral and/or scapula Y view.


 CT - indicated to better define intra-articular involvement and to aid pre-operative planning.
MRI is not useful for fracture imaging.

Classification
Description of the fracture is often more useful than classification. Particular attention should be paid
to humeral alignment, fracture displacement, and greater tuberosity position (rotator cuff will pull the
GT supero-posterioly, which can cause impingement problems with malunion).
- Neer Classification: Most commonly used. Describes fracture as 2,3,or 4 part depending upon the
number main fragments. Also comments on the degree of displacement. Fragments:
-greater tuberosity
-lesser tuberosity
- articular surface
- shaft
Displacement: >1cm or angulation >45 degrees.

Treatment
The vast majority of proximal humeral fractures are minimally displaced, and therefore can be
managed conservatively. This involves immobilisation in a polysling, and progressive mobilisation.
Pendular exercise can commence at 14 days, and active abduction from 4-6 weeks.

Irreducible fracture dislocation is an indication for operative management. Other indications include
large displacement, younger patient, head splitting (intra-articular fractures). However, the recent
PROFHER trial (1) has suggested no benefit to operative intervention on patient outcome (it must be
applied cautiously as majority of patients were elderly with extraarticular fractures). Options available
for surgical management include:

ORIF Most commonly used. Plate and screw fixation. Can reconstruct
complex fractures.

Intramedullary nail Suitable for extra-articular configuration, predominantly surgical neck


+/- GT fractures.
Used for un-reconstructable fractures in the older patient who has good
Hemiarthroplasty glenoid quality.

Total shoulder Unconstructable fractures where high functioning shoulder is required


arthroplasty (hemiarthroplasty will cause glenoid erosion)

Reverse shoulder Total shoulder arthroplasty that provides better functional outcome than
arthroplasty conventional total shoulder replacement.

Scapula
Background
Uncommon fractures usually associated with high energy trauma. Most commonly involve scapula
body or spine (50%), glenoid fossa and glenoid neck. Important to exclude associated life
threatening injury.

Imaging
Plain radiographs should include true anteroposterior (AP), axillary lateral and/or scapula Y view. CT
scanning is useful for defining intra-articular involvement, displacement and for three dimensional
reconstruction.

Classification
Based on the location of the fracture (coracoid, acromion, glenoid neck, glenoid fossa, scapula
body). Beware of ipsilateral glenoid neck and clavicle fracture -floating shoulder - where limb is
effectively dissociated from axial skeleton.

Treatment
The vast majority of scapula fractures are amenable to conservative management, consisting of
sling immobilisation for two weeks followed by early rehabilitation. Floating shoulder will usually
require fixation, and consideration of surgery should also be given to intra-articular and
displaced/angulated glenoid fractures.

Dislocations

Types
Dislocations around the shoulder joint include glenohumeral dislocation, acromioclavicular joint
disruption and sternoclavicular dislocation. Only glenohumeral dislocation will be covered here.

Glenohumeral dislocation
Diagnosis, classification and management are covered here.

Background
Shoulder dislocation is commonly seen in A&E. It has a high recurrence rate that is as high as 80%
in teenagers. Initial management requires emergent reduction to prevent lasting chondral damage.

Early assessment and management


Usually a traumatic cause (multi-directional instability in frequent dislocations requires discussion
with orthopaedics and is not covered here). Careful history, examination and documentation of
neurovascular status of the limb, in particular the axillary nerve (regimental badge sensation). This
should be re-assessed post manipulation. Early radiographs to confirm direction of dislocation.

Initial management consists of emergent closed reduction under under entanox and analgesia, but
often requires conscious sedation. Arm should then be immobilised in a polysling, and XR to confirm
relocation.

Imaging - True anteroposterior (AP), axillary lateral and/or scapula Y view. Reduced humeral head
should lie between acromion and coracoid on lateral/scapula view.

Types
Reduction
Direction Features Cause Examination techniques

Anterior Most Usually traumatic - anterior Loss of shoulder


Common force on arm when shoulder contour - sulcus sign. Hippocratic.
>90% is abducted, eternally Humeral head can be Milch.
rotated felt anteriorly. Stimson.

Kocher not
advised due to
complication of
fracture

Posterior 50% 50% traumatic, but Shoulder locked in Gentle lateral


missed in classically post seizure or internal rotation. XR traction to
A&E electrocution may show lightbulb adducted arm.
appearance.

Inferior Rare Associated with pectorals As for primary injury Management of


and rotator cuff tears, and primary injury
glenoid fracture

Rare Associated with As for primary injury Management of


Superior acrominon/clavicle fracture primary injury

Associated injuries
 Bankart lesion - avulsion of the anterior glenoid labrum with an anterior shoulder dislocation
(reverse Bankart if poster labrum in posterior dislocation).
 Hill Sachs defect - chondral impaction on posteriosuperior humeral head from contact with
gleonoid rim. Can be large enough to lock shoulder, requiring open reduction. (Reverse Hill
Sachs in posterior dislocation).
 Rotator cuff tear - increases with age.
 Greater or lesser tuberosity fracture - increases with age.
 Humeral neck fracture - shoulder fracture dislocation. More common in high energy trauma
and elderly. Should be discussed with orthopaedics prior to any attempted reduction.

Rotator Cuff Disease

Rotator cuff disease is a spectrum of conditions that ranges from subacromial impingement to rotator
cuff tears and eventually to rotator cuff arthropathy (arthritis).

Anatomy
The rotator cuff is a group of four muscles that are important in shoulder movements, and
maintenance of glenohumeral stability.
Scapular Humeral
Muscle attachment attachment Action Innervation

Supraspinatus Supraspinatus Superior facet of Initiation of Suprascapular nerve


fossa greater tuberosity abduction of
humerus

Infraspinatus Infraspinatus Posterior facet of External rotation Suprascapular nerve


fossa greater tuberosity of humerus

Teres Minor Lateral border Inferior facet of External rotation Axillary Nerve
greater tuberosity of humerus

Subscapularis Subscapular Lesser tuberosity Internal rotation Upper and lower


fossa of humerus subscapular nerve

 The inferior rotator cuff muscles (infraspinatus, teres minor, and subscapularis) balance the
superior pull of the deltoid. Injury/tear results in upward migration of the humeral head on the
glenoid (can be seen on AP radiograph).
 Likewise, the anterior muscles (subscapularis) are balanced with the posterior muscles
(infraspinatus, teres minor).
Subacromial Impingement

 The most common cause of shoulder pain, which results from impingement of the superior
cuff on the undersurface of the acromion, and an inflammatory bursitis.
 Associated with certain types of acromial morphology (Bigliani classification).
 Presents as insidious pain which is exacerbated by overhead activities.

Rotator Cuff Tear

 Often presents as an acute event on the background of chronic subacromial impingement in


the older patient, but can present as an avulsion injury in younger patients.
 Majority of tears are to the superior cuff (supraspinatus, infraspinatus, teres minor), though a
tear to subscapularis is associated with subcoracoid impingement.
 Tears present as pain and weakness when using the muscles in question.

Rotator Cuff Arthropathy

 Defined as shoulder arthritis in the setting of rotator cuff dysfunction. Results from superior
migration due to the loss of rotator cuff function and integrity. Unopposed deltoid pulls the
humeral head superiorly.
 Associated with massive chronic cuff tears.

Imaging

Plain radiographs

 AP of the shoulder may show superior migration of the humerus with a cuff tear, and features
of arthritis with arthropathy. Other causes of pain may also be identified (e.g. calcific
tendonitis/fracture)
 Outlet view is useful for defining the acromial morphology

USS

 Allows dynamic imaging of the cuff, and is inexpensive. However, it is very user dependent.

MRI
 Best imaging modality for cuff pathology.
 Also allows imaging of the rest of the shoulder. When intra-articular pathology is suspected,
can be combined with an arthrogram for improved sensitivity and specificity.

Treatment

Subacromial impingement

 Physiotherapy, oral anti-inflammatory medication


 Subacromial steroid injection can settle inflammation
 Arthroscopic subacromial decompression by shaving away the undersurface of the
acromion, more space is created for the rotator cuff. Cuff integrity is assessed also at time of
surgery, and can be repaired if necessary.

Rotator cuff tear

 When considering repair of a cuff tear, the age and activity of the patient, the nature of the
tear (degenerative vs. acute traumatic), and the size and retraction of the tear should be
considered when making a surgical plan.
 Mild tears or tears in the elderly can be managed conservatively, as outlined above.
 Moderate tears can be repaired arthroscopically. Massive or retracted tears will often require
an open repair (occasionally with a tendon transfer). Subacromial decompression is
performed at the same time to reduce impingement, symptoms and recurrence.

Calcific tendonitis
Calcific tendonitis involves calcific deposits within tendons anywhere in the body, but most
commonly in the rotator cuff (specifically the supraspinatus tendon). When present in the shoulder, it
is associated with subacromial impingement and pain.

Pathology

 More common in women aged 30-60 years.


 Association with diabetes and hypothyroidism

There are three stages of calcification

 Formative phase characterized by calcific deposits


 Resting phase deposit is stable, but presents with impingement problems
 Resorptive phase phagocytic resorption. Most painful stage.

Presentation
 Similar in presentation to subacromial impingement, with pain especially with over head
activities. Atraumatic in nature.

Imaging

 Plain radiographs show calcification of the rotator cuff, usually within 1.5cm of its insertion on
the humerus. Supraspinatus outlet views can show level of impingment. Further imaging is
rarely needed.

Treatment

 Non-operative NSAIDS, steroid injection (controversial, but practiced) and physiotherapy.


Approximately 75% will resolve by 6 months with conservative management.
 Ultrasound guided or surgical needle barbotage can break down deposits and resolve
symptoms. Occasionally surgical excision is required.

Adhesive capsulitis (Frozen Shoulder)

 Pain and loss of movement of shoulder joint, which involves fibroplastic proliferation of
capsular tissue, causing soft tissue scarring and contracture. Patients present with a painful
and decreased arc of motion.
 Associated with prolonged immobilization, previous surgery, thyroid disorders (AI) and
diabetes
 Classically three stages which can take up to two years to resolve:

Stage one the freezing and painful stage


Stage two the frozen and stiff stage
Stage three the thawing stage, where shoulder movement slowly improves

Imaging

 Plain radiographs to exclude other causes of a painful shoulder


 MRI arthrogram may show capsular contracture, and again may be used to exclude cuff
pathology. However, often not performed as diagnosis is largely clinical.

Treatment

 Non-operative NSAIDS, steroid injection and physiotherapy. Patience is required as


condition can take up to 2 years to improve.
 Operative MUA or arthroscopic adhesiolysis (release of adhesions) can expedite recovery,
followed by intensive physiotherapy.
Glenohumeral Arthritis
Shoulder arthritis presents with the normal symptoms of arthritis, however primary osteoarthritis is
not as commonly the primary cause as seen in other large joints.. A large proportion of shoulder
arthritis is rheumatoid (RA), post traumatic, or secondary to rotator cuff arthropathy (discussed
above).

Treatment

 Surgical options include hemiarthroplasty, total shoulder replacement and reverse geometry
total shoulder replacement (used when rotator cuff function is absent).

References
1. JAMA. 2015;313(10):1037-1047. doi:10.1001/jama.2015.1629
Next question
A 44 year old man is involved in a road traffic accident. He suffers significant injuries to his thorax,
he has bilateral haemopneumothoraces and a suspected haemopericardium. He is to undergo
surgery, what is the best method of accessing these injuries?

Bilateral thoracoscopy and mediastinoscopy

Midline sternotomy

Bilateral posterolateral thoracotomy

Clam shell thoracotomy

None of the above

Patients with significant mediastinal and lung injuries are best operated on using a Clam shell
thoracotomy. All modes of access involve a degree of compromise. A sternotomy would give good
access to the heart. However, it takes longer to perform and does not provide good access to the
lungs. Trauma should not be managed using laparoscopy.
Please rate this question:

Discuss and give feedback


Next question

Trauma management

The cornerstone of trauma management is embodied in the principles of ATLS.

Following trauma there is a trimodal death distribution:

 Immediately following injury. Typically as result of brain or high spinal injuries, cardiac or
great vessel damage. Salvage rate is low.
 In early hours following injury. In this group deaths are due to phenomena such as splenic
rupture, sub dural haematomas and haemopneumothoraces
 In the days following injury. Usually due to sepsis or multi organ failure.

Aspects of trauma management

 ABCDE approach.
 Tension pneumothoraces will deteriorate with vigorous ventilation attempts.
 External haemorrhage is managed as part of the primary survey. As a rule tourniquets
should not be used. Blind application of clamps will tend to damage surrounding structures
and packing is the preferred method of haemorrhage control.
 Urinary catheters and naso gastric tubes may need inserting. Be wary of basal skull fractures
and urethral injuries.
 Patients with head and neck trauma should be assumed to have a cervical spine injury until
proven otherwise.

Thoracic injuries

 Simple pneumothorax
 Mediastinal traversing wounds
 Tracheobronchial tree injury
 Haemothorax
 Blunt cardiac injury
 Diaphragmatic injury
 Aortic disruption
 Pulmonary contusion

Management of thoracic trauma

 Simple pneumothorax insert chest drain. Aspiration is risky in trauma as pneumothorax


may be from lung laceration and convert to tension pneumothorax.
 Mediastinal traversing wounds These result from situations like stabbings. Exit and entry
wounds in separate hemithoraces. The presence of a mediastinal haematoma indicates the
likelihood of a great vessel injury. All patients should undergo CT angiogram and
oesophageal contrast swallow. Indications for thoracotomy are largely related to blood loss
and will be addressed below.
 Tracheobronchial tree injury Unusual injuries. In blunt trauma most injuries occur within
4cm of the carina. Features suggesting this injury include haemoptysis and surgical
emphysema. These injuries have a very large air leak and may have tension pneumothorax.
 Haemothorax Usually caused by laceration of lung vessel or internal mammary artery by rib
fracture. Patients should all have a wide bore 36F chest drain. Indications for thoracotomy
include loss of more than 1.5L blood initially or ongoing losses of >200ml per hour for >2
hours.
 Cardiac contusions Usually cardiac arrhythmias, often overlying sternal fracture. Perform
echocardiography to exclude pericardial effusions and tamponade. Risk of arrhythmias falls
after 24 hours.
 Diaphragmatic injury Usually left sided. Direct surgical repair is performed.
 Traumatic aortic disruption Commonest cause of death after RTA or falls. Usually
incomplete laceration near ligamentum arteriosum. All survivors will have contained
haematoma. Only 1-2% of patients with this injury will have a normal chest x-ray.
 Pulmonary contusion Common and lethal. Insidious onset. Early intubation and ventilation.

Abdominal trauma
 Deceleration injuries are common.
 In blunt trauma requiring laparotomy the spleen is most commonly injured (40%)
 Stab wounds traverse structures most commonly liver (40%)
 Gunshot wounds have variable effects depending upon bullet type. Small bowel is most
commonly injured (50%)
 Patients with stab wounds and no peritoneal signs up to 25% will not enter the peritoneal
cavity
 Blood at urethral meatus suggests a urethral tear
 High riding prostate on PR = urethral disruption
 Mechanical testing for pelvic stability should only be performed once

Investigations in abdominal trauma

Diagnostic Peritoneal Abdominal CT scan USS


Lavage

Indication Document bleeding if Document organ injury if Document fluid if


hypotensive normotensive hypotensive

Advantages Early diagnosis and Most specific for Early diagnosis, non
sensitive; 98% accurate localising injury; 92 to invasive and repeatable;
98% accurate 86 to 95% accurate

Disadvantages Invasive and may miss Location of scanner away Operator dependent and
retroperitoneal and from facilities, time taken may miss
diaphragmatic injury for reporting, need for retroperitoneal injury
contrast

 Amylase may be normal following pancreatic trauma


 Urethrography if suspected urethral injury

Next question
A 63 year old male is admitted to the surgical ward for an elective femoral popliteal bypass. He
suddenly starts complaining of central, crushing chest pain which is radiating to the left arm. The
Nursing staff start high flow oxygen and give a spray of glyceryl trinitrate spray. Unfortunately there
is no relief of symptoms. What is the next agent to be administered?

Aspirin 75mg

Clopidogrel 75mg

Aspirin 300mg

Clopidogrel 300mg

Direct admission to angiography suite

Aspirin 300mg should be given as soon as possible. If the patient has a moderate to high risk of
myocardial infarction, then Clopidogrel should be given with a low molecular weight heparin.
Thromboloysis or urgent percutaneous intervention should be given if there are significant ECG
changes.

Please rate this question:

Discuss and give feedback

Next question

Management of acute coronary syndrome

NICE produced guidelines in 2010 on the management of unstable angina and non-ST elevation
myocardial infarction (NSTEMI). They advocate managing patients based on the early risk
assessment using a recognised scoring system such as GRACE (Global Registry of Acute Cardiac
Events) to calculate a predicted 6 month mortality.

All patients should receive


- aspirin 300mg
- nitrates or morphine to relieve chest pain if required

Whilst it is common that non-hypoxic patients receive oxygen therapy there is little evidence to
support this approach. The 2008 British Thoracic Society oxygen therapy guidelines advise not
giving oxygen unless the patient is hypoxic.

Antithrombin treatment. Low molecular weight heparin should be offered to patients who are not at
a high risk of bleeding and who are not having angiography within the next 24 hours. If angiography
is likely within 24 hours or a patients creatinine is > 265 umol/l unfractionated heparin should be
given.

Clopidogrel 300mg should be given to patients with a predicted 6 month mortality of more than
1.5% or patients who may undergo percutaneous coronary intervention within 24 hours of admission
to hospital. Clopidogrel should be continued for 12 months.

Intravenous glycoprotein IIb/IIIa receptor antagonists (eptifibatide or tirofiban) should be given to


patients who have an intermediate or higher risk of adverse cardiovascular events (predicted 6-
month mortality above 3.0%), and who are scheduled to undergo angiography within 96 hours of
hospital admission.

Coronary angiography should be considered within 96 hours of first admission


to hospital to patients who have a predicted 6-month mortality above 3.0%. It should also be
performed as soon as possible in patients who are clinically unstable.

Next question
Which of the following is not a change found on an ECG in acute pulmonary embolism?

No changes

J waves

P pulmonale

Right ventricular strain

T wave inversion in the inferior leads

S1, Q3, T3

J waves are pathognomonic of hypothermia.


Please rate this question:

Discuss and give feedback


Next question

Pulmonary Embolism: ECG changes

 No changes
 S1, Q3, T3
 Tall R waves: V1
 P pulmonale (peaked P waves): inferior leads
 Right axis deviation, Right bundle branch block
 Atrial arrhythmias
 T wave inversion: V1, V2, V3
 Right ventricular strain: if identified is associated with adverse short-term outcome and adds
prognostic value to echocardiographic evidence of right ventricular dysfunction in patients
with acute pulmonary embolism and normal blood pressure.

References
Vanni S et al. Prognostic value of ECG among patients with acute pulmonary embolism and normal
blood pressure. Am J Med. 2009 Mar;122(3):257-64.
Next question
A 21 year old man falls down a ravine whilst skiing and is trapped for several hours. He is finally
brought to the emergency department profoundly hypothermic with a core temperature of 29 oC.
Which method is most effective at raising the core temperature?

Re-warming with electric blankets

Increasing the room temperature

Instillation of warm intravesical fluid

Instillation of warmed rectal fluid

Instillation of warmed intra peritoneal fluid

Theme from 2011 Examination


Visceral cavity re-warming be it lung or abdomen (or both) provides rapid rewarming. Only
extracorporeal circulatory devices provide faster rates of re-warming.

Please rate this question:

Discuss and give feedback

Next question

Hypothermia

Core body temperature below 35oC. Severe hypothermia is present when the core temperature is
below 30oC. Hypothermia is associated with a reduction in both respiratory and cardiac activity.

Management
An organised cardiac rhythm may be converted to fibrillation if CPR is attempted inappropriately so
ECG should be analysed with care. The rewarming technique used depends upon the degree of
hypothermia and the physiological state of the patient. Mild hypothermia may respond to external
rewarming devices. Severe hypothermia may require active core rewarming techniques such as
peritoneal lavage, haemodialysis or cardiac bypass. Patients who develop cardiac arrhythmias who
are severely hypothermic may respond to bretylium toslyte (sadly no longer available in most
centres), but do not generally respond to standard therapies or DC cardioversion.

Next question
A 42 year old woman is admitted to surgery with acute cholecystitis. She is known to have
hypertension, rheumatoid arthritis and polymyalgia rheumatica. Her medical therapy includes:
Paracetamol 1g qds
Codeine phosphate 30mg qds
Bendrofluazide 2.5 mg od
Ramipril 10mg od
Methotrexate 7.5mg once a week
Prednisolone 5mg od
You are called by the Senior House Officer to assess this lady as she has become delirious and
hypotensive 2 hours after surgery. Her blood results reveal:

Na+ 132 mmol/l

K+ 5.3 mmol/l

Urea 7 mmol/l

Creatinine 108 µmol/l

Hb 12.4 g/dl

Platelets 178 * 109/l

WBC 15.4 * 109/l

What management is needed immediately?

Ceftriaxone IV

Hydrocortisone 50mg IV

CT scan abdomen

Urgent exploratory laparotomy

Hydrocortisone 100mg IV

This patient has acute adrenal insufficiency and urgently needs steroid replacement.
Please rate this question:
Discuss and give feedback
Next question

Addisonian crisis

Causes

 Sepsis or surgery causing an acute exacerbation of chronic insufficiency (Addison's,


Hypopituitarism)
 Adrenal haemorrhage eg Waterhouse-Friderichsen syndrome (fulminant meningococcemia)
 Steroid withdrawal

Management

 Hydrocortisone 100 mg im or iv
 1 litre normal saline infused over 30-60 mins or with dextrose if hypoglycaemic
 Continue hydrocortisone 6 hourly until the patient is stable. No fludrocortisone is required
because high cortisol exerts weak mineralocorticoid action
 Oral replacement may begin after 24 hours and be reduced to maintenance over 3-4 days

Next question
Theme: Thoracic trauma

A. Thoracotomy
B. Manage conservatively
C. Intercostal tube drain insertion
D. CT scanning
E. Bronchoscopy
F. Negative pressure intercostal tube drainage
G. Video assisted thoracoscopy and pleurectomy

For each of the following scenarios please select the most appropriate management option from the
list. Each option may be used once, more than once or not at all.

54. A 28 year old male is involved in a road traffic accident he is thrown from his motorbike
onto the pavement and sustains a haemopneumothorax and flail segment of the right chest.

You answered Thoracotomy

The correct answer is Intercostal tube drain insertion

He requires a chest drain and analgesia. In general all haemopneumothoraces should be


managed by intercostal chest drain insertion as they have a risk of becoming a tension
pneumothorax until the lung laceration has sealed.

55. A 19 year old man is stabbed in the chest at a nightclub. He develops a cardiac arrest in
casualty following an attempted transfer to the CT scanning room.

Thoracotomy

This is one indication for an 'emergency room' thoracotomy, there are not many others!
Typical injuries include ventricular penetration, great vessel disruption and hilar lung
injuries.

56. A 32 year old male falls over and sustains a small pneumothorax following a simple rib
fracture. He has no physiological compromise.

You answered Thoracotomy

The correct answer is Intercostal tube drain insertion

Unlike spontaneous pneumothoraces most would advocate chest tube drainage in the
context of pneumothorax resulting from trauma. This is because of the risk of the lung
laceration developing a tension. Once there is no further evidence of air leak the chest
drain may be removed and a check x-ray performed to check there is no re-accumulation
prior to discharge.

Please rate this question:

Discuss and give feedback


Next question

Thoracic trauma

Types of thoracic trauma

Tension  Often laceration to lung parenchyma with flap


pneumothorax  Pressure develops in thorax
 Most common cause is mechanical ventilation in patient with
pleural injury
 Symptoms overlap with cardiac tamponade, hyper-resonant
percussion note is more likely in tension pnemothorax

Flail chest  Chest wall disconnects from thoracic cage


 Multiple rib fractures (at least two fractures per rib in at least two
ribs)
 Associated with pulmonary contusion
 Abnormal chest motion
 Avoid over hydration and fluid overload

Pneumothorax  Most common cause is lung laceration with air leakage


 Most traumatic pneumothoraces should have a chest drain
 Patients with traumatic pneumothorax should never be
mechanically ventilated until a chest drain is inserted

Haemothorax  Most commonly due to laceration of lung, intercostal vessel or


internal mammary artery
 Haemothoraces large enough to appear on CXR are treated with
large bore chest drain
 Surgical exploration is warranted if >1500ml blood drained
immediately

Cardiac tamponade  Beck's triad: elevated venous pressure, reduced arterial pressure,
reduced heart sounds
 Pulsus paradoxus
 May occur with as little as 100ml blood

Pulmonary contusion  Most common potentially lethal chest injury


 Arterial blood gases and pulse oximetry important
 Early intubation within an hour if significant hypoxia

Blunt cardiac injury  Usually occurs secondary to chest wall injury


 ECG may show features of myocardial infarction
 Sequelae: hypotension, arrhythmias, cardiac wall motion
abnormalities

Aorta disruption  Deceleration injuries


 Contained haematoma
 Widened mediastinum

Diaphragm  Most due to motor vehicle accidents and blunt trauma causing
disruption large radial tears (laceration injuries result in small tears)
 More common on left side
 Insert gastric tube, may pass into intrathoracic stomach

Mediastinal  Entrance wound in one hemithorax and exit wound/foreign body


traversing wounds in opposite hemithorax
 Mediastinal haematoma or pleural cap suggests great vessel
injury
 Mortality is 20%

References
ATLS Manual 8th Edition
Next question
A 21 year old man is undergoing an inguinal hernia repair and receives a dose of intravenous co-
amoxiclav. He is reported to have a penicillin allergy. Over the next few minutes his vital signs are:
Pulse - 130bpm, blood pressure- 60/40mmHg. What is the first line treatment?

Hydrocortisone 100mg IV

Adrenaline 1:1000 IV

Chlorpheniramine 10mg IV

Adrenaline 1:1000 IM

Adrenaline 1:10000 IV

Theme from 2009 Exam


Theme from September 2013 Exam
The first line treatment of anaphylactic shock is intra muscular adrenaline.
Please rate this question:

Discuss and give feedback


Next question

Anaphylactic shock

 Suspect if there has been exposure to an allergen

Management
- Remove allergen
- ABCD
- Drugs:

Adrenaline 1:1000 0.5ml INTRAMUSCULARLY (not IV). Repeat after 5 mins if no response.
Then Chlorpheniramine 10mg IV
Then Hydrocortisone 100-200mg IV

Reference
Emergency treatment of anaphylactic reactions. Guidelines for healthcare providers. Working Group
of the Resuscitation Council (UK).2008
Next question
Theme: Types of stroke

A. Anterior cerebral artery infarct


B. Middle cerebral artery infarct
C. Posterior cerebral artery infarct
D. Pituitary mass
E. Lacunar infarct
F. Lateral medullary syndrome
G. Pontine infarct
H. Horner's syndrome
I. Cerebellar infarct

Please select the most likely cause for the symptoms given. Each option may be used once, more
than once or not at all.

58. A 53 year old teacher is admitted to the vascular ward for a carotid endarterectomy. Your
house officer does a preoperative assessment and notes that there is a right homonymous
hemianopia. There is no other neurology.

You answered Anterior cerebral artery infarct

The correct answer is Posterior cerebral artery infarct

This patient has had a left occipital infarct, as there is only a homonymous hemianopia. If
this patient had a temporal or parietal lobe infarct, there would be associated hemiparesis
and higher cortical dysfunction. This is important to differentiate, as the carotid
endarterectomy is inappropriate in this patient as the lesion is in the posterior cerebral
artery.

59. A 52 year man is admitted to the vascular ward for an amputation. The patient complains
of unsteadiness. On further examination you detect right facial numbness and right sided
nystagmus. There is sensory loss of the left side of the body.

You answered Anterior cerebral artery infarct

The correct answer is Lateral medullary syndrome

A combination of ipsilateral ataxia, nystagmus, dysphagia, facial numbness, cranial nerve


palsy with contralateral hemisensory loss indicates this diagnosis.

60. A 48 year old type 2 diabetic complains of numbness in his left arm and leg. Otherwise
there is no other neurological signs.
You answered Anterior cerebral artery infarct

The correct answer is Lacunar infarct

Isolated hemisensory loss is a feature of a lacunar infarct.

Please rate this question:

Discuss and give feedback


Next question

Stroke: types

Primary intracerebral  Presents with headache, vomiting, loss of


haemorrhage (PICH, c. 10%) consciousness

Total anterior circulation  Involves middle and anterior cerebral arteries


infarcts (TACI, c. 15%)  Hemiparesis/hemisensory loss
 Homonymous hemianopia
 Higher cognitive dysfunction e.g. Dysphasia

Partial anterior circulation  Involves smaller arteries of anterior circulation e.g.


infarcts (PACI, c. 25%) upper or lower division of middle cerebral artery
 Higher cognitive dysfunction or two of the three TACI
features

Lacunar infarcts (LACI, c.  Involves perforating arteries around the internal


25%) capsule, thalamus and basal ganglia
 Present with either isolated hemiparesis, hemisensory
loss or hemiparesis with limb ataxia

Posterior circulation infarcts  Vertebrobasilar arteries


(POCI, c. 25%)  Presents with features of brainstem damage
 Ataxia, disorders of gaze and vision, cranial nerve
lesions

Lateral medullary syndrome  Wallenberg's syndrome


(posterior inferior cerebellar  Ipsilateral: ataxia, nystagmus, dysphagia, facial
artery) numbness, cranial nerve palsy
 Contralateral: limb sensory loss

Weber's syndrome  Ipsilateral III palsy


 Contralateral weakness

--------------------------------------

Anterior cerebral artery

 Contralateral hemiparesis and sensory loss, lower extremity > upper


 Disconnection syndrome

Middle cerebral artery

 Contralateral hemiparesis and sensory loss, upper extremity > lower


 Contralateral hemianopia
 Aphasia (Wernicke's)
 Gaze abnormalities

Posterior cerebral artery

 Contralateral hemianopia with macular sparing


 Disconnection syndrome

Lacunar

 Present with either isolated hemiparesis, hemisensory loss or hemiparesis with limb ataxia

Lateral medulla (posterior inferior cerebellar artery)

 Ipsilateral: ataxia, nystagmus, dysphagia, facial numbness, cranial nerve palsy e.g.

Horner's
 Contralateral: limb sensory loss

Pontine

 VI nerve: horizontal gaze palsy


 VII nerve
 Contralateral hemiparesis

Next question
A 22 year old man has a full thickness burn on his chest. It is well circumscribed. In A&E his
saturations are reduced to 92% on 15L Oxygen, Blood pressure 102/66 mmHg and HR 105bpm.
What is the best management?

Haemodialysis

Escharotomy

Fasciotomy

Cardiac bypass

Non invasive ventilation

The chest burn and its associated oedema is limiting respiration. Therefore an escharotomy of the
chest is indicated, this will remove the constriction on the chest wall and improve ventilation.
Please rate this question:

Discuss and give feedback


Next question

Burns

Burns may be thermal, chemical or electrical. In the former category are burns which occur as a
result of heat. Chemical burns occur when the skin is exposed to an extremely caustic or alkaline
substance. Electrical burns occur following exposure to electrical current. The immediate
management includes removal of the burning source which usually includes irrigation of the burned
area. A detailed assessment then needs to be made of the extent of the burns and a number of
charts are available for recording this information. The degree of injury relates to the temperature
and duration of exposure. Most domestic burns are mainly scalds in young children.

Following the burn, there is a local response with progressive tissue loss and release of
inflammatory cytokines. Systemically, there are cardiovascular effects resulting from fluid loss and
sequestration of fluid into the third space. There is a marked catabolic response. Immunosupression
is common with large burns and bacterial translocation from the gut lumen is a recognised event.
Sepsis is a common cause of death following major burns.

Types of burn

Type of burn Skin layers Skin Blanching Management


affected appearance

Epidermal/Superficial Epidermis Red, moist Yes

Superficial partial Epidermis and part Pale, dry Yes Normally heals
thickness of papillary dermis with no
affected intervention

Deep partial thickness Epidermis, whole Mottled red No Needs surgical


papillary dermis colour intervention
affected (depending on site)

Full thickness Whole skin layer Dry, leathery No Burns centre


and subcutaneous hard wound
tissue affected

Depth of burn assessment

 Bleeding on needle prick


 Sensation
 Appearance
 Blanching to pressure

Percentage burn estimation


Lund Browder chart: most accurate even in children
Wallace rule of nines
Palmar surface: surface area palm = 0.8% burn

>15% body surface area burns in adults needs urgent burn fluid resuscitation

Transfer to burn centre if:

 Need burn shock resuscitation


 Face/hands/genitals affected
 Deep partial thickness or full thickness burns
 Significant electrical/chemical burns

Management
The initial aim is to stop the burning process and resuscitate the patient. Intravenous fluids will be
required for children with burns greater than 10% of total body surface area. Adults with burns
greater than 15% of total body surface area will also require IV fluids. The fluids are calculated using
the Parkland formula which is; volume of fluid= total body surface area of the burn % x weight (Kg)
x4. Half of the fluid is administered in the first 8 hours. A urinary catheter should be inserted.
Analgesia should be given. Complex burns, burns involving the hand perineum and face and burns
>10% in adults and >5% in children should be transferred to a burns unit.

Circumferential burns affecting a limb or severe torso burns impeding respiration may require
escharotomy to divide the burnt tissue.

Conservative management is appropriate for superficial burns and mixed superficial burns that will
heal in 2 weeks. More complex burns may require excision and skin grafting. Excision and primary
closure is not generally practised as there is a high risk of infection.

There is no evidence to support the use of anti microbial prophylaxis or topical antibiotics in burn
patients.

Escharotomies

 Indicated in circumferential full thickness burns to the torso or limbs.


 Careful division of the encasing band of burn tissue will potentially improve ventilation (if the
burn involves the torso), or relieve compartment syndrome and oedema (where a limb is
involved)

References
www.euroburn.org/e107files/downloads/guidelinesburncare.pdf

Barajas-Nava LA, López-Alcalde J, Roqué i Figuls M, Solà I, Bonfill Cosp X. Antibiotic prophylaxis
for preventing burn wound infection. Cochrane Database of Systematic Reviews 2013, Issue 6. Art.
No.: CD008738. DOI: 10.1002/14651858.CD008738.pub2.

Hettiaratchy S & Papini R. Initial management of a major burn: assessment and resuscitation. BMJ
2004;329:101-103
Next question
A 16 year old man sustains a basal skull fracture and is suspected of having CSF rhinorrhoea.
Which of the following laboratory tests would most accurately identify whether CSF is present or
not?

Microscopy to identify red blood cells

Lab stix testing for glucose

Lab stix testing for protein

Beta 2 transferrin assay

Microscopy, gram stain and culture

Beta 2 transferrin is a carbohydrate free form of transferrin that is almost exclusively found in the
CSF. Although lab stix testing for glucose is traditional it is associated with false positive results
secondary to contamination with other glucose containing bodily secretions.
Please rate this question:

Discuss and give feedback


Next question

Head injury management- NICE Guidelines

Summary of guidelines

 All patients should be assessed within 15 minutes on arrival to A&E


 Document all 3 components of the GCS
 If GCS <8 or = to 8, consider stabilising the airway
 Treat pain with low dose IV opiates (if safe)
 Full spine immobilisation until assessment if:

- GCS < 15
- neck pain/tenderness
- paraesthesia extremities
- focal neurological deficit
- suspected c-spine injury

If a c-spine injury is suspected a 3 view c-spine x-ray is indicated. CT c-spine is preferred if:
- Intubated
- GCS <13
- Normal x-ray but continued concerns regarding c-spine injury
- Any focal neurology
- A CT head scan is being performed
- Initial plain films are abnormal

Immediate CT head (within 1 hour) if:

 GCS < 13 on admission


 GCS < 15 2 hours after admission
 Suspected open or depressed skull fracture
 Suspected skull base fracture (panda eyes, Battle's sign, CSF from nose/ear, bleeding ear)
 Focal neurology
 Vomiting > 1 episode
 Post traumatic seizure
 Coagulopathy

Contact neurosurgeon if:

 Persistent GCS < 8 or = 8


 Unexplained confusion > 4h
 Reduced GCS after admission
 Progressive neurological signs
 Incomplete recovery post seizure
 Penetrating injury
 Cerebrospinal fluid leak

Observations

 1/2 hourly GCS until 15

Reference
1. http://guidance.nice.org.uk/CG56/QuickRefGuide/pdf/English
2. Hodgkinson S et al. Early management of head injury: summary of NICE guidance. BMJ 2014
(348):34-37.
Next question
A 66 year old male is admitted to the vascular ward for an amputation. He reports episodes of
vertigo and dysarthria to the house officer. He suddenly collapses with a Glasgow Coma Score of 3.
What is the most likely diagnosis?

Cerebral haemorrhage in left temporal parietal area

Opiate overdose

Cerebral haemorrhage in right temporal parietal area

Diazepam overdose

Basilar artery occlusion

Vertigo and dysarthria suggest a posterior circulation event. In the scenario of a patient complaining
of posterior symptoms and a sudden deterioration in consciousness, the main differential diagnosis
is of a basilar artery occlusion.
Please rate this question:

Discuss and give feedback


Next question

Stroke: types

Primary intracerebral  Presents with headache, vomiting, loss of


haemorrhage (PICH, c. 10%) consciousness

Total anterior circulation  Involves middle and anterior cerebral arteries


infarcts (TACI, c. 15%)  Hemiparesis/hemisensory loss
 Homonymous hemianopia
 Higher cognitive dysfunction e.g. Dysphasia

Partial anterior circulation  Involves smaller arteries of anterior circulation e.g.


infarcts (PACI, c. 25%) upper or lower division of middle cerebral artery
 Higher cognitive dysfunction or two of the three TACI
features
Lacunar infarcts (LACI, c.  Involves perforating arteries around the internal
25%) capsule, thalamus and basal ganglia
 Present with either isolated hemiparesis, hemisensory
loss or hemiparesis with limb ataxia

Posterior circulation infarcts  Vertebrobasilar arteries


(POCI, c. 25%)  Presents with features of brainstem damage
 Ataxia, disorders of gaze and vision, cranial nerve
lesions

Lateral medullary syndrome  Wallenberg's syndrome


(posterior inferior cerebellar  Ipsilateral: ataxia, nystagmus, dysphagia, facial
artery) numbness, cranial nerve palsy
 Contralateral: limb sensory loss

Weber's syndrome  Ipsilateral III palsy


 Contralateral weakness

--------------------------------------

Anterior cerebral artery

 Contralateral hemiparesis and sensory loss, lower extremity > upper


 Disconnection syndrome

Middle cerebral artery

 Contralateral hemiparesis and sensory loss, upper extremity > lower


 Contralateral hemianopia
 Aphasia (Wernicke's)
 Gaze abnormalities

Posterior cerebral artery

 Contralateral hemianopia with macular sparing


 Disconnection syndrome
Lacunar

 Present with either isolated hemiparesis, hemisensory loss or hemiparesis with limb ataxia

Lateral medulla (posterior inferior cerebellar artery)

 Ipsilateral: ataxia, nystagmus, dysphagia, facial numbness, cranial nerve palsy e.g.

Horner's

 Contralateral: limb sensory loss

Pontine

 VI nerve: horizontal gaze palsy


 VII nerve
 Contralateral hemiparesis

Next question
A 22 year old man is brought to the emergency department. He was found lying unconscious on his
right arm and it is evident that he has taken a temazepam overdose. His right arm is mottled in
colour and swollen, his hand is insensate and stiff. What substance is most likely to be present in the
urine in increased quantities?

Protein

Haemoglobin

Myoglobin

Erythrocytes

Lymphocytes

Theme from 2011 Exam


This man is likely to have muscle death secondary to compartment syndrome. This will result in
muscle breakdown and release of myoglobin. This may accumulate in the kidney and result in renal
failure.
Please rate this question:

Discuss and give feedback


Next question

Compartment syndrome

 This is a particular complication that may occur following fractures (or following ischaemia re-
perfusion injury in vascular patients). It is characterised by raised pressure within a closed
anatomical space.
 The raised pressure within the compartment will eventually compromise tissue perfusion
resulting in necrosis. The two main fractures carrying this complication include supracondylar
fractures and tibial shaft injuries.

Symptoms and signs

 Pain, especially on movement (even passive)


 Parasthesiae
 Pallor may be present
 Arterial pulsation may still be felt as the necrosis occurs as a result of microvascular
compromise
 Paralysis of the muscle group may occur

Diagnosis

 Is made by measurement of intracompartmental pressure measurements. Pressures in


excess of 20mmHg are abnormal and >40mmHg is diagnostic.

Treatment

 This is essentially prompt and extensive fasciotomies


 In the lower limb the deep muscles may be inadequately decompressed by the
inexperienced operator when smaller incisions are performed
 Myoglobinuria may occur following fasciotomy and result in renal failure and for this reason
these patients require aggressive IV fluids
 Where muscle groups are frankly necrotic at fasciotomy they should be debrided and
amputation may have to be considered
 Death of muscle groups may occur within 4-6 hours

Next question
A 19 year intravenous drug abuser is recovering following a surgical drainage of a psoas abscess.
He is found collapsed in the ward toilet unresponsive and with pinpoint pupils. What is the most
appropriate immediate management?

Intravenous flumazenil

Intravenous nalaxone

Intravenous benxhexol

No further management

Intravenous glycopyrolate

Intravenous nalaxone is needed to treat the patient who has had an overdose of opiate. Naloxone
has the quickest onset of action, however it is important to be aware of its short acting duration and
the need for further administration. There is also the risk of rebound pain once naloxone is given.
Please rate this question:

Discuss and give feedback


Next question

Opioid misuse

Opioids are substances which bind to opioid receptors. This includes both naturally occurring opiates
such as morphine and synthetic opioids such as buprenorphine and methadone.

Features of opioid misuse

 Rhinorrhoea
 Needle track marks
 Pinpoint pupils
 Drowsiness

Complications of intravenous opioid misuse

 Viral infection secondary to sharing needles: HIV, hepatitis B & C


 Bacterial infection secondary to injection: infective endocarditis, septic arthritis, septicaemia,
necrotising fasciitis, groin abscess
 Pseudoaneurysm
 Venous thromboembolism
 Osteomyelitis
 Overdose may lead to respiratory depression and death

Emergency management of opioid overdose

 IV or IM naloxone: has a rapid onset and relatively short duration of action

Next question
A 68 year old male is admitted to the surgical ward for assessment of severe epigastric pain. His
abdomen is soft and non tender. However the Nurse forces you to look at the ECG. It looks
abnormal. Which of the following features is an indication for urgent coronary thrombolysis or
percutaneous intervention?

Right bundle branch block

ST elevation of 1mm in leads V1 to V6

Ventricular tachycardia

Q waves in leads V1 to V6

ST elevation of 1mm in leads II, III and aVF

ECG changes for thrombolysis or percutaneous intervention:


ST elevation of > 2mm (2 small squares) in 2 or more consecutive anterior leads (V1-V6) OR

ST elevation of greater than 1mm (1 small square) in greater than 2 consecutive inferior leads (II, III,
avF, avL) OR

New Left bundle branch block

ST elevation of 1mm in leads II, III and aVF reflects significant cardiac ischaemia due to the right
coronary artery occlusion. The medical registrar should be contacted to urgently assess the patient.
Note right coronary artery occlusions puts the patient at risk of cardiac arrhythmias (due to blood
supply to the sino atrial node).
Please rate this question:

Discuss and give feedback


Next question

Thrombolysis or percutaneous intervention in myocardial infarction

Thrombolytic drugs activate plasminogen to form plasmin. This in turn degrades fibrin and help
breaks up thrombi. They in primarily used in patients who present with a ST elevation myocardial
infarction. Other indications include acute ischaemic stroke and pulmonary embolism, although strict
inclusion criteria apply.

Examples
 alteplase
 tenecteplase
 streptokinase

Contraindications to thrombolysis

 active internal bleeding


 recent haemorrhage, trauma or surgery (including dental extraction)
 coagulation and bleeding disorders
 intracranial neoplasm
 stroke < 3 months
 aortic dissection
 recent head injury
 pregnancy
 severe hypertension

Side-effects

 haemorrhage
 hypotension - more common with streptokinase
 allergic reactions may occur with streptokinase

Next question
Which of the following is not typically associated with a degloving injury?

Overlying pallor of the skin

Abnormal motility of the overlying skin

History of friction type injury

Improved results when the degloved segment is left in situ as a temporary closure

Poor results when primary compression treatment is used in preference to skin grafting

Degloving injuries typically involve extremities and are usually friction injuries e.g. arm being run
over. There is abnormal motility of the overlying skin, pallor, loss of sensation. Early treatment is key
and should involve skin grafting which may use the degloved segment. This however, should be
formally prepared for the role and simple compression bandaging gives poor results.
Please rate this question:

Discuss and give feedback


Next question

Wound healing

Surgical wounds are either incisional or excisional and either clean, clean contaminated or dirty.
Although the stages of wound healing are broadly similar their contributions will vary according to the
wound type.

The main stages of wound healing include:

Haemostasis

 Minutes to hours following injury


 Vasospasm in adjacent vessels, platelet plug formation and generation of fibrin rich clot.

Inflammation

 Typically days 1-5


 Neutrophils migrate into wound (function impaired in diabetes).
 Growth factors released, including basic fibroblast growth factor and vascular endothelial
growth factor.
 Fibroblasts replicate within the adjacent matrix and migrate into wound.
 Macrophages and fibroblasts couple matrix regeneration and clot substitution.

Regeneration

 Typically days 7 to 56
 Platelet derived growth factor and transformation growth factors stimulate fibroblasts and
epithelial cells.
 Fibroblasts produce a collagen network.
 Angiogenesis occurs and wound resembles granulation tissue.

Remodeling

 From 6 weeks to 1 year


 Longest phase of the healing process and may last up to one year (or longer).
 During this phase fibroblasts become differentiated (myofibroblasts) and these facilitate
wound contraction.
 Collagen fibres are remodeled.
 Microvessels regress leaving a pale scar.

The above description represents an idealised scenario. A number of diseases may distort this
process. Neovascularisation is an important early process. Endothelial cells may proliferate in the
wound bed and recanalise to form a vessel. Vascular disease, shock and sepsis can all compromise
microvascular flow and impair healing.

Conditions such as jaundice will impair fibroblast synthetic function and immunity with a detrimental
effect in most parts of the healing process.

Problems with scars:

Hypertrophic scars
Excessive amounts of collagen within a scar. Nodules may be present histologically containing
randomly arranged fibrils within and parallel fibres on the surface. The tissue itself is confined to the
extent of the wound itself and is usually the result of a full thickness dermal injury. They may go on
to develop contractures.

Image of hypertrophic scarring. Note that it remains confined to the boundaries of the original
wound:
Image sourced from Wikipedia

Keloid scars
Excessive amounts of collagen within a scar. Typically a keloid scar will pass beyond the boundaries
of the original injury. They do not contain nodules and may occur following even trivial injury. They
do not regress over time and may recur following removal.

Image of a keloid scar. Note the extension beyond the boundaries of the original incision:

Image sourced from Wikipedia

Drugs which impair wound healing:


 Non steroidal anti inflammatory drugs
 Steroids
 Immunosupressive agents
 Anti neoplastic drugs

Closure
Delayed primary closure is the anatomically precise closure that is delayed for a few days but before
granulation tissue becomes macroscopically evident.

Secondary closure refers to either spontaneous closure or to surgical closure after granulation tissue
has formed.
Next question
Which of the following statements relating to large volume blood loss in trauma is incorrect?

Tranexamic acid reduces the incidence of rebleeding following surgery

Hypocalcaemia may complicate resuscitation

Colloids are preferred initially as they reduce the incidence of coagulopathy

When patients receive over 5 units of whole blood mortality increases when blood
products greater than 3 weeks old are utilised

In the battlefield setting a ratio of 1:1:1 for blood, plasma and platelets is used

Fresh blood is the fluid of choice when large volume blood loss complicates trauma. Mortality is
doubled when blood >3 weeks old is used.
Please rate this question:

Discuss and give feedback


Next question

Trauma management

The cornerstone of trauma management is embodied in the principles of ATLS.

Following trauma there is a trimodal death distribution:

 Immediately following injury. Typically as result of brain or high spinal injuries, cardiac or
great vessel damage. Salvage rate is low.
 In early hours following injury. In this group deaths are due to phenomena such as splenic
rupture, sub dural haematomas and haemopneumothoraces
 In the days following injury. Usually due to sepsis or multi organ failure.

Aspects of trauma management

 ABCDE approach.
 Tension pneumothoraces will deteriorate with vigorous ventilation attempts.
 External haemorrhage is managed as part of the primary survey. As a rule tourniquets
should not be used. Blind application of clamps will tend to damage surrounding structures
and packing is the preferred method of haemorrhage control.
 Urinary catheters and naso gastric tubes may need inserting. Be wary of basal skull fractures
and urethral injuries.
 Patients with head and neck trauma should be assumed to have a cervical spine injury until
proven otherwise.

Thoracic injuries

 Simple pneumothorax
 Mediastinal traversing wounds
 Tracheobronchial tree injury
 Haemothorax
 Blunt cardiac injury
 Diaphragmatic injury
 Aortic disruption
 Pulmonary contusion

Management of thoracic trauma

 Simple pneumothorax insert chest drain. Aspiration is risky in trauma as pneumothorax


may be from lung laceration and convert to tension pneumothorax.
 Mediastinal traversing wounds These result from situations like stabbings. Exit and entry
wounds in separate hemithoraces. The presence of a mediastinal haematoma indicates the
likelihood of a great vessel injury. All patients should undergo CT angiogram and
oesophageal contrast swallow. Indications for thoracotomy are largely related to blood loss
and will be addressed below.
 Tracheobronchial tree injury Unusual injuries. In blunt trauma most injuries occur within
4cm of the carina. Features suggesting this injury include haemoptysis and surgical
emphysema. These injuries have a very large air leak and may have tension pneumothorax.
 Haemothorax Usually caused by laceration of lung vessel or internal mammary artery by rib
fracture. Patients should all have a wide bore 36F chest drain. Indications for thoracotomy
include loss of more than 1.5L blood initially or ongoing losses of >200ml per hour for >2
hours.
 Cardiac contusions Usually cardiac arrhythmias, often overlying sternal fracture. Perform
echocardiography to exclude pericardial effusions and tamponade. Risk of arrhythmias falls
after 24 hours.
 Diaphragmatic injury Usually left sided. Direct surgical repair is performed.
 Traumatic aortic disruption Commonest cause of death after RTA or falls. Usually
incomplete laceration near ligamentum arteriosum. All survivors will have contained
haematoma. Only 1-2% of patients with this injury will have a normal chest x-ray.
 Pulmonary contusion Common and lethal. Insidious onset. Early intubation and ventilation.

Abdominal trauma

 Deceleration injuries are common.


 In blunt trauma requiring laparotomy the spleen is most commonly injured (40%)
 Stab wounds traverse structures most commonly liver (40%)
 Gunshot wounds have variable effects depending upon bullet type. Small bowel is most
commonly injured (50%)
 Patients with stab wounds and no peritoneal signs up to 25% will not enter the peritoneal
cavity
 Blood at urethral meatus suggests a urethral tear
 High riding prostate on PR = urethral disruption
 Mechanical testing for pelvic stability should only be performed once

Investigations in abdominal trauma

Diagnostic Peritoneal Abdominal CT scan USS


Lavage

Indication Document bleeding if Document organ injury if Document fluid if


hypotensive normotensive hypotensive

Advantages Early diagnosis and Most specific for Early diagnosis, non
sensitive; 98% accurate localising injury; 92 to invasive and repeatable;
98% accurate 86 to 95% accurate

Disadvantages Invasive and may miss Location of scanner away Operator dependent and
retroperitoneal and from facilities, time taken may miss
diaphragmatic injury for reporting, need for retroperitoneal injury
contrast

 Amylase may be normal following pancreatic trauma


 Urethrography if suspected urethral injury

Next question
The following features are typical of superficial partial dermal burns except:

Erythema

Absence of blisters

Spontaneous healing in most cases

No extension beyond proximal dermal papillae

Good capillary refill at the burn site

Superficial dermal burns are typically erythematous, do not extend beyond the upper part of the
dermal papillae, capillary return and blisters are both usually present.

A Typical example of a superficial dermal burn is shown below

Image sourced from Wikipedia

Please rate this question:

Discuss and give feedback


Next question
Burns

Burns may be thermal, chemical or electrical. In the former category are burns which occur as a
result of heat. Chemical burns occur when the skin is exposed to an extremely caustic or alkaline
substance. Electrical burns occur following exposure to electrical current. The immediate
management includes removal of the burning source which usually includes irrigation of the burned
area. A detailed assessment then needs to be made of the extent of the burns and a number of
charts are available for recording this information. The degree of injury relates to the temperature
and duration of exposure. Most domestic burns are mainly scalds in young children.

Following the burn, there is a local response with progressive tissue loss and release of
inflammatory cytokines. Systemically, there are cardiovascular effects resulting from fluid loss and
sequestration of fluid into the third space. There is a marked catabolic response. Immunosupression
is common with large burns and bacterial translocation from the gut lumen is a recognised event.
Sepsis is a common cause of death following major burns.

Types of burn

Type of burn Skin layers Skin Blanching Management


affected appearance

Epidermal/Superficial Epidermis Red, moist Yes

Superficial partial Epidermis and part Pale, dry Yes Normally heals
thickness of papillary dermis with no
affected intervention

Deep partial thickness Epidermis, whole Mottled red No Needs surgical


papillary dermis colour intervention
affected (depending on site)

Full thickness Whole skin layer Dry, leathery No Burns centre


and subcutaneous hard wound
tissue affected

Depth of burn assessment

 Bleeding on needle prick


 Sensation
 Appearance
 Blanching to pressure
Percentage burn estimation
Lund Browder chart: most accurate even in children
Wallace rule of nines
Palmar surface: surface area palm = 0.8% burn

>15% body surface area burns in adults needs urgent burn fluid resuscitation

Transfer to burn centre if:

 Need burn shock resuscitation


 Face/hands/genitals affected
 Deep partial thickness or full thickness burns
 Significant electrical/chemical burns

Management
The initial aim is to stop the burning process and resuscitate the patient. Intravenous fluids will be
required for children with burns greater than 10% of total body surface area. Adults with burns
greater than 15% of total body surface area will also require IV fluids. The fluids are calculated using
the Parkland formula which is; volume of fluid= total body surface area of the burn % x weight (Kg)
x4. Half of the fluid is administered in the first 8 hours. A urinary catheter should be inserted.
Analgesia should be given. Complex burns, burns involving the hand perineum and face and burns
>10% in adults and >5% in children should be transferred to a burns unit.

Circumferential burns affecting a limb or severe torso burns impeding respiration may require
escharotomy to divide the burnt tissue.

Conservative management is appropriate for superficial burns and mixed superficial burns that will
heal in 2 weeks. More complex burns may require excision and skin grafting. Excision and primary
closure is not generally practised as there is a high risk of infection.

There is no evidence to support the use of anti microbial prophylaxis or topical antibiotics in burn
patients.

Escharotomies

 Indicated in circumferential full thickness burns to the torso or limbs.


 Careful division of the encasing band of burn tissue will potentially improve ventilation (if the
burn involves the torso), or relieve compartment syndrome and oedema (where a limb is
involved)

References
www.euroburn.org/e107files/downloads/guidelinesburncare.pdf

Barajas-Nava LA, López-Alcalde J, Roqué i Figuls M, Solà I, Bonfill Cosp X. Antibiotic prophylaxis
for preventing burn wound infection. Cochrane Database of Systematic Reviews 2013, Issue 6. Art.
No.: CD008738. DOI: 10.1002/14651858.CD008738.pub2.
Which of the following is not a feature found on a CXR in traumatic aortic disruption?

Widened mediastinum

Trachea deviated to the left

Depression of the left main stem bronchus

Obliteration of the aortic knob

Widened paraspinal interfaces

The trachea is normally deviated to the right.


Please rate this question:

Discuss and give feedback


Next question

Thoracic aorta rupture

 Mechanism of injury: Decelerating force i.e. RTA, fall from a great height
 Most people die at scene
 Survivors may have an incomplete laceration at the ligamentum arteriosum of the aorta.

Clinical features

 Contained haematoma: persistent hypotension


 Detected mainly by history, CXR changes

CXR changes

 Widened mediastinum
 Trachea/Oesophagus to right
 Depression of left main stem bronchus
 Widened paratracheal stripe/paraspinal interfaces
 Space between aorta and pulmonary artery obliterated
 Rib fracture/left haemothorax

Diagnosis
Angiography, usually CT aortogram.

Treatment
Repair or replacement. Ideally they should undergo endovascular repair.
Next question
Theme: Head injury management

A. Observation
B. CT head within 1 hour
C. CT head within 8 hours
D. Urgent neurosurgical review (even before CT head performed)
E. 3 view c-spine xray
F. 2 view c-spine xray
G. CT c-spine
H. MRI c-spine

What is the best initial management plan for the scenario given? Each option may be used once,
more than once or not at all.

3. A 22 year old male falls of a ladder. He complains of neck pain and cannot feel his legs. His
GCS suddenly deteriorates and a CT head confirms an extradural haematoma. What is the
best imaging for his neck?

You answered Observation

The correct answer is CT c-spine

This man needs a CT scan of his c-spine. A CT scan will give the best resolution of any
bony injury.

4. A 25 year old teacher falls down the stairs. She complains of a headache and has vomited 3
times. She has a GCS of 15/15.

You answered Observation

The correct answer is CT head within 1 hour

This lady has a head injury and vomiting > 1, therefore an urgent CT head is indicated.

5. An 18 year old student is shot in the back of the head.

You answered Observation

The correct answer is Urgent neurosurgical review (even before CT head performed)

A penetrating injury needs urgent neurosurgical review.

Please rate this question:


Discuss and give feedback
Next question

Head injury management- NICE Guidelines

Summary of guidelines

 All patients should be assessed within 15 minutes on arrival to A&E


 Document all 3 components of the GCS
 If GCS <8 or = to 8, consider stabilising the airway
 Treat pain with low dose IV opiates (if safe)
 Full spine immobilisation until assessment if:

- GCS < 15
- neck pain/tenderness
- paraesthesia extremities
- focal neurological deficit
- suspected c-spine injury

If a c-spine injury is suspected a 3 view c-spine x-ray is indicated. CT c-spine is preferred if:
- Intubated
- GCS <13
- Normal x-ray but continued concerns regarding c-spine injury
- Any focal neurology
- A CT head scan is being performed
- Initial plain films are abnormal

Immediate CT head (within 1 hour) if:

 GCS < 13 on admission


 GCS < 15 2 hours after admission
 Suspected open or depressed skull fracture
 Suspected skull base fracture (panda eyes, Battle's sign, CSF from nose/ear, bleeding ear)
 Focal neurology
 Vomiting > 1 episode
 Post traumatic seizure
 Coagulopathy

Contact neurosurgeon if:

 Persistent GCS < 8 or = 8


 Unexplained confusion > 4h
 Reduced GCS after admission
 Progressive neurological signs
 Incomplete recovery post seizure
 Penetrating injury
 Cerebrospinal fluid leak

Observations

 1/2 hourly GCS until 15

Reference
1. http://guidance.nice.org.uk/CG56/QuickRefGuide/pdf/English
2. Hodgkinson S et al. Early management of head injury: summary of NICE guidance. BMJ 2014
(348):34-37.
Next question
A 60-year-old man develops palpitations while on the acute surgical unit. An ECG shows a broad
complex tachycardia at a rate of 150 bpm. His blood pressure is 124/82 mmHg and there is no
evidence of heart failure. The surgical consultant wants to give rate control (the medical team are not
answering their bleeps). Which one of the following is it least appropriate to give?

Procainamide

Lidocaine

Amiodarone

Adenosine

Verapamil

Ventricular tachycardia - verapamil is contraindicated

Verapamil should never be given to a patient with a broad complex tachycardia as it may precipitate
ventricular fibrillation in patients with ventricular tachycardia. Adenosine is sometimes given in this
situation as a 'trial' if there is a strong suspicion the underlying rhythm is a supraventricular
tachycardia with aberrant conduction
Please rate this question:

Discuss and give feedback


Next question

Ventricular tachycardia: management

Whilst a broad complex tachycardia may result from a supraventricular rhythm with aberrant
conduction, the European Resuscitation Council advise that in a peri-arrest situation it is assumed to
be ventricular in origin

If the patient has adverse signs (systolic BP < 90 mmHg, chest pain, heart failure or rate > 150
beats/min) then immediate cardioversion is indicated. In the absence of such signs antiarrhythmics
may be used. If these fail, then electrical cardioversion may be needed with synchronised DC
shocks

Drug therapy

 amiodarone: ideally administered through a central line


 lidocaine: use with caution in severe left ventricular impairment
 procainamide

Verapamil should NOT be used in VT

If drug therapy fails

 electrophysiological study (EPS)


 implant able cardioverter-defibrillator (ICD) - this is particularly indicated in patients with
significantly impaired LV function

Next question
A 62 year old male attends the hernia clinic. He suddenly develops speech problems, left facial
weakness and left sided arm and leg weakness lasting longer than 5 minutes. A CT head shows no
intracerebral bleed. What is the next line of management?

Aspirin 300mg

Aspirin 75 mg

Clopidogrel 300mg

Urgent referral for thrombolysis

Carotid endarterectomy

This patient is within 3h of symptom onset of a stroke. Therefore he should be urgently referred to
the medical team for thrombolysis, before Aspirin is given. There are concerns that high dose aspirin
would increase the risk of intracerebral haemorrhage if thrombolysis is undertaken.
This is an example of the type of medical problem you should be aware of as a surgeon, as
ultimately you can make a difference by referring QUICKLY to the correct specialty for management.
Please rate this question:

Discuss and give feedback


Next question

Stroke: types

Primary intracerebral  Presents with headache, vomiting, loss of


haemorrhage (PICH, c. 10%) consciousness

Total anterior circulation  Involves middle and anterior cerebral arteries


infarcts (TACI, c. 15%)  Hemiparesis/hemisensory loss
 Homonymous hemianopia
 Higher cognitive dysfunction e.g. Dysphasia

Partial anterior circulation  Involves smaller arteries of anterior circulation e.g.


infarcts (PACI, c. 25%) upper or lower division of middle cerebral artery
 Higher cognitive dysfunction or two of the three TACI
features

Lacunar infarcts (LACI, c.  Involves perforating arteries around the internal


25%) capsule, thalamus and basal ganglia
 Present with either isolated hemiparesis, hemisensory
loss or hemiparesis with limb ataxia

Posterior circulation infarcts  Vertebrobasilar arteries


(POCI, c. 25%)  Presents with features of brainstem damage
 Ataxia, disorders of gaze and vision, cranial nerve
lesions

Lateral medullary syndrome  Wallenberg's syndrome


(posterior inferior cerebellar  Ipsilateral: ataxia, nystagmus, dysphagia, facial
artery) numbness, cranial nerve palsy
 Contralateral: limb sensory loss

Weber's syndrome  Ipsilateral III palsy


 Contralateral weakness

--------------------------------------

Anterior cerebral artery

 Contralateral hemiparesis and sensory loss, lower extremity > upper


 Disconnection syndrome

Middle cerebral artery

 Contralateral hemiparesis and sensory loss, upper extremity > lower


 Contralateral hemianopia
 Aphasia (Wernicke's)
 Gaze abnormalities

Posterior cerebral artery

 Contralateral hemianopia with macular sparing


 Disconnection syndrome

Lacunar

 Present with either isolated hemiparesis, hemisensory loss or hemiparesis with limb ataxia

Lateral medulla (posterior inferior cerebellar artery)

 Ipsilateral: ataxia, nystagmus, dysphagia, facial numbness, cranial nerve palsy e.g.

Horner's

 Contralateral: limb sensory loss

Pontine

 VI nerve: horizontal gaze palsy


 VII nerve
 Contralateral hemiparesis

Next question
A 45-year-old man is seen in the Emergency Department with nausea, pallor and lethargy. He has
no past medical history of note. A cannula is inserted and serum urea and electrolytes show the
following

Na+ 140 mmol/l

K+ 6.7 mmol/l

Bicarbonate 14 mmol/l

Urea 18.2 mmol/l

Creatinine 230 µmol/l

What is the most appropriate initial management?

Nebulised salbutamol

Intravenous bicarbonate

Haemodialysis

Insulin/dextrose infusion

Intravenous calcium gluconate

The first priority in this patient is to stabilise the myocardium with intravenous calcium gluconate.
Please rate this question:

Discuss and give feedback


Next question

Management of hyperkalaemia

Untreated hyperkalaemia may cause life-threatening arrhythmias. Precipitating factors should be


addressed (e.g. acute renal failure) and aggravating drugs stopped (e.g. ACE inhibitors).
Management may be categorised by the aims of treatment

Stabilisation of the cardiac membrane


 Intravenous calcium gluconate

Short-term shift in potassium from extracellular to intracellular fluid compartment

 Combined insulin/dextrose infusion


 Nebulised salbutamol

Removal of potassium from the body

 Calcium resonium (orally or enema)


 Loop diuretics
 Dialysis

Next question
Theme: Visceral injury

A. Ruptured spleen
B. Ileum injury
C. Duodenal injury
D. Urethral injury
E. Rectal injury
F. Oesophageal injury
G. Liver laceration

Please select the most likely injury for the scenario given. Each option may be used once, more than
once or not at all.

9. A motorcyclist is involved in a head on road traffic accident with a lorry. He is comatose


at scene and trauma series x-rays confirm a pelvic fracture. On rectal examination he has a
high riding prostate.

You answered Ruptured spleen

The correct answer is Urethral injury

This is classical for urethral injury. Features of a urethral injury include; pelvic fracture,
high riding prostate on digital rectal examination and blood at the urethral meatus. Where
this is the suspected diagnosis a suprapubic catheter and urethral contrast studies
performed.

10. A cyclist loses control and falls off the side of a road landing on the bicycle handlebars.
CT scanning shows a large amount of retroperitoneal air.

You answered Ruptured spleen

The correct answer is Duodenal injury

Theme from 2012 exam


Retroperitoneal air is more likely with a duodenal injury. As it is largely retroperitoneal. A
handlebar type injury is the commonest cause and the pancreas should be carefully
inspected as it too may be injured. It would be unusual for the ileum to be injured in this
type of scenario as it is mobile.

11. A 23 year old man is shot in the abdomen. He is haemodynamically stable but on
ultrasound he has a large amount of intra abdominal free fluid.

You answered Ruptured spleen


The correct answer is Ileum injury

Small bowel injury is the most common type of injury in this scenario. The enteric
contents will tend to result in a large amount of intra abdominal fluid.

Please rate this question:

Discuss and give feedback


Next question

Trauma management

The cornerstone of trauma management is embodied in the principles of ATLS.

Following trauma there is a trimodal death distribution:

 Immediately following injury. Typically as result of brain or high spinal injuries, cardiac or
great vessel damage. Salvage rate is low.
 In early hours following injury. In this group deaths are due to phenomena such as splenic
rupture, sub dural haematomas and haemopneumothoraces
 In the days following injury. Usually due to sepsis or multi organ failure.

Aspects of trauma management

 ABCDE approach.
 Tension pneumothoraces will deteriorate with vigorous ventilation attempts.
 External haemorrhage is managed as part of the primary survey. As a rule tourniquets
should not be used. Blind application of clamps will tend to damage surrounding structures
and packing is the preferred method of haemorrhage control.
 Urinary catheters and naso gastric tubes may need inserting. Be wary of basal skull fractures
and urethral injuries.
 Patients with head and neck trauma should be assumed to have a cervical spine injury until
proven otherwise.

Thoracic injuries

 Simple pneumothorax
 Mediastinal traversing wounds
 Tracheobronchial tree injury
 Haemothorax
 Blunt cardiac injury
 Diaphragmatic injury
 Aortic disruption
 Pulmonary contusion

Management of thoracic trauma

 Simple pneumothorax insert chest drain. Aspiration is risky in trauma as pneumothorax


may be from lung laceration and convert to tension pneumothorax.
 Mediastinal traversing wounds These result from situations like stabbings. Exit and entry
wounds in separate hemithoraces. The presence of a mediastinal haematoma indicates the
likelihood of a great vessel injury. All patients should undergo CT angiogram and
oesophageal contrast swallow. Indications for thoracotomy are largely related to blood loss
and will be addressed below.
 Tracheobronchial tree injury Unusual injuries. In blunt trauma most injuries occur within
4cm of the carina. Features suggesting this injury include haemoptysis and surgical
emphysema. These injuries have a very large air leak and may have tension pneumothorax.
 Haemothorax Usually caused by laceration of lung vessel or internal mammary artery by rib
fracture. Patients should all have a wide bore 36F chest drain. Indications for thoracotomy
include loss of more than 1.5L blood initially or ongoing losses of >200ml per hour for >2
hours.
 Cardiac contusions Usually cardiac arrhythmias, often overlying sternal fracture. Perform
echocardiography to exclude pericardial effusions and tamponade. Risk of arrhythmias falls
after 24 hours.
 Diaphragmatic injury Usually left sided. Direct surgical repair is performed.
 Traumatic aortic disruption Commonest cause of death after RTA or falls. Usually
incomplete laceration near ligamentum arteriosum. All survivors will have contained
haematoma. Only 1-2% of patients with this injury will have a normal chest x-ray.
 Pulmonary contusion Common and lethal. Insidious onset. Early intubation and ventilation.

Abdominal trauma

 Deceleration injuries are common.


 In blunt trauma requiring laparotomy the spleen is most commonly injured (40%)
 Stab wounds traverse structures most commonly liver (40%)
 Gunshot wounds have variable effects depending upon bullet type. Small bowel is most
commonly injured (50%)
 Patients with stab wounds and no peritoneal signs up to 25% will not enter the peritoneal
cavity
 Blood at urethral meatus suggests a urethral tear
 High riding prostate on PR = urethral disruption
 Mechanical testing for pelvic stability should only be performed once

Investigations in abdominal trauma

Diagnostic Peritoneal Abdominal CT scan USS


Lavage

Indication Document bleeding if Document organ injury if Document fluid if


hypotensive normotensive hypotensive

Advantages Early diagnosis and Most specific for Early diagnosis, non
sensitive; 98% accurate localising injury; 92 to invasive and repeatable;
98% accurate 86 to 95% accurate

Disadvantages Invasive and may miss Location of scanner away Operator dependent and
retroperitoneal and from facilities, time taken may miss
diaphragmatic injury for reporting, need for retroperitoneal injury
contrast

 Amylase may be normal following pancreatic trauma


 Urethrography if suspected urethral injury

Next question
A 22 year old man has a full thickness burn of his leg after being trapped in a burning car. There are
no fractures of the limb. There burn is well circumscribed. After 2 hours he complains of tingling of
his leg and it appears dusky. What is the best management for this?

Fasciotomy

Escharotomy

Angioplasty

Pain control

Anticoagulation

The full thickness burn has oedema which is affecting the peripheral circulation. Therefore the burn
needs to be divided (not the fascia) to allow normal circulation to return.
Please rate this question:

Discuss and give feedback


Next question

Burns

Burns may be thermal, chemical or electrical. In the former category are burns which occur as a
result of heat. Chemical burns occur when the skin is exposed to an extremely caustic or alkaline
substance. Electrical burns occur following exposure to electrical current. The immediate
management includes removal of the burning source which usually includes irrigation of the burned
area. A detailed assessment then needs to be made of the extent of the burns and a number of
charts are available for recording this information. The degree of injury relates to the temperature
and duration of exposure. Most domestic burns are mainly scalds in young children.

Following the burn, there is a local response with progressive tissue loss and release of
inflammatory cytokines. Systemically, there are cardiovascular effects resulting from fluid loss and
sequestration of fluid into the third space. There is a marked catabolic response. Immunosupression
is common with large burns and bacterial translocation from the gut lumen is a recognised event.
Sepsis is a common cause of death following major burns.

Types of burn

Type of burn Skin layers Skin Blanching Management


affected appearance

Epidermal/Superficial Epidermis Red, moist Yes

Superficial partial Epidermis and part Pale, dry Yes Normally heals
thickness of papillary dermis with no
affected intervention

Deep partial thickness Epidermis, whole Mottled red No Needs surgical


papillary dermis colour intervention
affected (depending on site)

Full thickness Whole skin layer Dry, leathery No Burns centre


and subcutaneous hard wound
tissue affected

Depth of burn assessment

 Bleeding on needle prick


 Sensation
 Appearance
 Blanching to pressure

Percentage burn estimation


Lund Browder chart: most accurate even in children
Wallace rule of nines
Palmar surface: surface area palm = 0.8% burn

>15% body surface area burns in adults needs urgent burn fluid resuscitation

Transfer to burn centre if:

 Need burn shock resuscitation


 Face/hands/genitals affected
 Deep partial thickness or full thickness burns
 Significant electrical/chemical burns

Management
The initial aim is to stop the burning process and resuscitate the patient. Intravenous fluids will be
required for children with burns greater than 10% of total body surface area. Adults with burns
greater than 15% of total body surface area will also require IV fluids. The fluids are calculated using
the Parkland formula which is; volume of fluid= total body surface area of the burn % x weight (Kg)
x4. Half of the fluid is administered in the first 8 hours. A urinary catheter should be inserted.
Analgesia should be given. Complex burns, burns involving the hand perineum and face and burns
>10% in adults and >5% in children should be transferred to a burns unit.

Circumferential burns affecting a limb or severe torso burns impeding respiration may require
escharotomy to divide the burnt tissue.

Conservative management is appropriate for superficial burns and mixed superficial burns that will
heal in 2 weeks. More complex burns may require excision and skin grafting. Excision and primary
closure is not generally practised as there is a high risk of infection.

There is no evidence to support the use of anti microbial prophylaxis or topical antibiotics in burn
patients.

Escharotomies

 Indicated in circumferential full thickness burns to the torso or limbs.


 Careful division of the encasing band of burn tissue will potentially improve ventilation (if the
burn involves the torso), or relieve compartment syndrome and oedema (where a limb is
involved)

References
www.euroburn.org/e107files/downloads/guidelinesburncare.pdf

Barajas-Nava LA, López-Alcalde J, Roqué i Figuls M, Solà I, Bonfill Cosp X. Antibiotic prophylaxis
for preventing burn wound infection. Cochrane Database of Systematic Reviews 2013, Issue 6. Art.
No.: CD008738. DOI: 10.1002/14651858.CD008738.pub2.

Hettiaratchy S & Papini R. Initial management of a major burn: assessment and resuscitation. BMJ
2004;329:101-103
Next question
A 28 year old man is in the surgical intensive care unit. He has suffered a flail chest injury several
hours earlier and he was intubated and ventilated. Over the past few minutes he has become
increasingly hypoxic and is now needing increased ventilation pressures. What is the most common
cause?

Pulmonary embolism

Cardiac tamponade

Fat embolism

Tension pneumothorax

Adult respiratory distress syndrome

Theme from April 2011 Exam

A flail chest segment may lacerate the underlying lung and create a flap valve. A tension
pneumothorax can be created by intubation and ventilation in this situation. Sudden hypoxia and
increased ventilation pressure are clues.
Please rate this question:

Discuss and give feedback


Next question

Tension Pneumothorax

Tension pneumothorax is a state of positive pressure within a pneumothorax throughout the


respiratory cycle. A breach in the pleura allows air into the intra pleural space via a one way valve.
The initial pressure pneumothorax expands until positive pressure is present throughout the
respiratory cycle. The risk is greatest in the ventilated trauma patient as positive pressure is used.
Undiagnosed tension pneumothorax accounts for 3.8% of trauma deaths.

Clinically, the classic features include chest pain, dyspnoea, hypoxia, hypotension, tracheal
deviation, ipsilateral hyperpercussion note, decreased air entry. In ventilated patients, cardiovascular
disturbance and sub cutaneous emphysema are relatively common and more so than in a case
where the patient is breathing spontaneously.

Chest x-ray features

 Lung collapse towards the hilum


 Diaphragmatic depression Increased rib separation
 Increased thoracic volume
 Ipsilateral flattening of the heart border
 Contra lateral mediastinal deviation

Management

Immediate needle decompression followed by definitive wide bore chest drain insertion

Image showing mediastinal shift with a tension pneumothorax

Image sourced from Wikipedia

Next question
A 10 year old boy is playing with a firework which explodes and he sustains a full thickness burn to
his left arm. Which of the following statements is not characteristic of this situation?

They have a leathery appearance

The burn area is extremely painful until skin grafted

They always heal with scarring

Blanching does not occur under pressure

Absence of,or few, blisters

Full thickness burns involve complete injury to the dermis and sub dermal appendages. They have a
leathery, often white appearance. They are initially insensate although pain often occurs during
healing following skin grafting. They do not blanch under pressure.
Please rate this question:

Discuss and give feedback


Next question

Burns

Burns may be thermal, chemical or electrical. In the former category are burns which occur as a
result of heat. Chemical burns occur when the skin is exposed to an extremely caustic or alkaline
substance. Electrical burns occur following exposure to electrical current. The immediate
management includes removal of the burning source which usually includes irrigation of the burned
area. A detailed assessment then needs to be made of the extent of the burns and a number of
charts are available for recording this information. The degree of injury relates to the temperature
and duration of exposure. Most domestic burns are mainly scalds in young children.

Following the burn, there is a local response with progressive tissue loss and release of
inflammatory cytokines. Systemically, there are cardiovascular effects resulting from fluid loss and
sequestration of fluid into the third space. There is a marked catabolic response. Immunosupression
is common with large burns and bacterial translocation from the gut lumen is a recognised event.
Sepsis is a common cause of death following major burns.

Types of burn

Type of burn Skin layers Skin Blanching Management


affected appearance

Epidermal/Superficial Epidermis Red, moist Yes

Superficial partial Epidermis and part Pale, dry Yes Normally heals
thickness of papillary dermis with no
affected intervention

Deep partial thickness Epidermis, whole Mottled red No Needs surgical


papillary dermis colour intervention
affected (depending on site)

Full thickness Whole skin layer Dry, leathery No Burns centre


and subcutaneous hard wound
tissue affected

Depth of burn assessment

 Bleeding on needle prick


 Sensation
 Appearance
 Blanching to pressure

Percentage burn estimation


Lund Browder chart: most accurate even in children
Wallace rule of nines
Palmar surface: surface area palm = 0.8% burn

>15% body surface area burns in adults needs urgent burn fluid resuscitation

Transfer to burn centre if:

 Need burn shock resuscitation


 Face/hands/genitals affected
 Deep partial thickness or full thickness burns
 Significant electrical/chemical burns

Management
The initial aim is to stop the burning process and resuscitate the patient. Intravenous fluids will be
required for children with burns greater than 10% of total body surface area. Adults with burns
greater than 15% of total body surface area will also require IV fluids. The fluids are calculated using
the Parkland formula which is; volume of fluid= total body surface area of the burn % x weight (Kg)
x4. Half of the fluid is administered in the first 8 hours. A urinary catheter should be inserted.
Analgesia should be given. Complex burns, burns involving the hand perineum and face and burns
>10% in adults and >5% in children should be transferred to a burns unit.

Circumferential burns affecting a limb or severe torso burns impeding respiration may require
escharotomy to divide the burnt tissue.

Conservative management is appropriate for superficial burns and mixed superficial burns that will
heal in 2 weeks. More complex burns may require excision and skin grafting. Excision and primary
closure is not generally practised as there is a high risk of infection.

There is no evidence to support the use of anti microbial prophylaxis or topical antibiotics in burn
patients.

Escharotomies

 Indicated in circumferential full thickness burns to the torso or limbs.


 Careful division of the encasing band of burn tissue will potentially improve ventilation (if the
burn involves the torso), or relieve compartment syndrome and oedema (where a limb is
involved)

References
www.euroburn.org/e107files/downloads/guidelinesburncare.pdf

Barajas-Nava LA, López-Alcalde J, Roqué i Figuls M, Solà I, Bonfill Cosp X. Antibiotic prophylaxis
for preventing burn wound infection. Cochrane Database of Systematic Reviews 2013, Issue 6. Art.
No.: CD008738. DOI: 10.1002/14651858.CD008738.pub2.

Hettiaratchy S & Papini R. Initial management of a major burn: assessment and resuscitation. BMJ
2004;329:101-103
Next question
Theme: Thoracic injuries

A. Pneumothorax
B. Tension pneumothorax
C. Flail chest
D. Cardiac tamponade
E. Aorta rupture
F. Cardiac contusion
G. Diaphragmatic rupture
H. Acute phrenic nerve injury

For each of the scenarios given, please select the most likely underlying injury. Each option may be
used once, more than once or not at all.

15. An 18 year old student is involved in a car crash, with another car crashing into the side of
the car.
A CXR shows an indistinct left hemidiaphragm.

You answered Pneumothorax

The correct answer is Diaphragmatic rupture

A lateral blunt injury during a road traffic accident is a common cause of diaphragmatic
rupture. Diagnosis is usually evident on chest x-ray. CXR changes include non visible
diaphragm, bowel loops in the hemithorax and displacement of the mediastinum. In most
cases direct surgical repair is the best option.

16. A 19 year old motorcyclist is involved in a road traffic accident. His chest movements are
irregular. He is found to have multiple rib fractures, with 2 fractures in the 3rd rib and 3
fractures in the 4th rib.

You answered Pneumothorax

The correct answer is Flail chest

Multiple rib fractures with > or = 2 rib fractures in more than 2 ribs is diagnosed as a flail
chest. This is associated with pulmonary contusion.

17. A 19 year old student falls from a 2nd floor window. He is persistently hypotensive. A
CXR shows depression of the left main bronchus and deviation of the trachea to the right.

You answered Pneumothorax


The correct answer is Aorta rupture

He has a deceleration injury, with persistent hypotension (contained haematoma). This


should indicate aorta rupture. Widened mediastinum may not always be present on a CXR.
A CT angiogram will provide clearer evidence of the extent of injury. The presence of
persistent hypotension, from a early stage is more consistent with haematoma than a
tension pneumothorax in which it occurs as a final periarrest phenomena.
CXR findings in diaphragmatic rupture:
Hemidiaphragm is not visible
Bowel loops in the lower half of the hemi-thorax
Mediastinum is displaced

Please rate this question:

Discuss and give feedback


Next question

Thoracic trauma

Types of thoracic trauma

Tension  Often laceration to lung parenchyma with flap


pneumothorax  Pressure develops in thorax
 Most common cause is mechanical ventilation in patient with
pleural injury
 Symptoms overlap with cardiac tamponade, hyper-resonant
percussion note is more likely in tension pnemothorax

Flail chest  Chest wall disconnects from thoracic cage


 Multiple rib fractures (at least two fractures per rib in at least two
ribs)
 Associated with pulmonary contusion
 Abnormal chest motion
 Avoid over hydration and fluid overload

Pneumothorax  Most common cause is lung laceration with air leakage


 Most traumatic pneumothoraces should have a chest drain
 Patients with traumatic pneumothorax should never be
mechanically ventilated until a chest drain is inserted

Haemothorax  Most commonly due to laceration of lung, intercostal vessel or


internal mammary artery
 Haemothoraces large enough to appear on CXR are treated with
large bore chest drain
 Surgical exploration is warranted if >1500ml blood drained
immediately

Cardiac tamponade  Beck's triad: elevated venous pressure, reduced arterial pressure,
reduced heart sounds
 Pulsus paradoxus
 May occur with as little as 100ml blood

Pulmonary contusion  Most common potentially lethal chest injury


 Arterial blood gases and pulse oximetry important
 Early intubation within an hour if significant hypoxia

Blunt cardiac injury  Usually occurs secondary to chest wall injury


 ECG may show features of myocardial infarction
 Sequelae: hypotension, arrhythmias, cardiac wall motion
abnormalities

Aorta disruption  Deceleration injuries


 Contained haematoma
 Widened mediastinum

Diaphragm  Most due to motor vehicle accidents and blunt trauma causing
disruption large radial tears (laceration injuries result in small tears)
 More common on left side
 Insert gastric tube, may pass into intrathoracic stomach

Mediastinal  Entrance wound in one hemithorax and exit wound/foreign body


traversing wounds in opposite hemithorax
 Mediastinal haematoma or pleural cap suggests great vessel
injury
 Mortality is 20%

References
ATLS Manual 8th Edition
Next question
Which of the features below, following a head injury, is not an indication for an immediate CT head
scan in children?

Drowsiness

A single, discrete episode of vomiting

A 9 month old child with a 6cm haematoma on the head

Numb left arm

Suspicion of a non accidental head injury

Whilst not an indication for immediate CT there should be a low threshold for admission and
observation.

Please rate this question:

Discuss and give feedback

Next question

Head injury paediatrics

Criteria for immediate request for CT scan of the head (children)


* Loss of consciousness lasting more than 5 minutes (witnessed)
* Amnesia (antegrade or retrograde) lasting more than 5 minutes
* Abnormal drowsiness
* Three or more discrete episodes of vomiting
* Clinical suspicion of non-accidental injury
* Post-traumatic seizure but no history of epilepsy
* GCS less than 14, or for a baby under 1 year GCS (paediatric) less than 15, on assessment in the
emergency department
* Suspicion of open or depressed skull injury or tense fontanelle
* Any sign of basal skull fracture (haemotympanum, panda' eyes, cerebrospinal fluid leakage from
the ear or nose, Battle's sign)
* Focal neurological deficit
* If under 1 year, presence of bruise, swelling or laceration of more than 5 cm on the head
* Dangerous mechanism of injury (high-speed road traffic accident either as pedestrian, cyclist or
vehicle occupant, fall from a height of greater than 3 m, high-speed injury from a projectile or an
object)

Next question
Theme: Management of osteomyelitis

A. Lautenbach regime
B. Below knee amputation
C. Hindquater amputation
D. Above knee amputation
E. Removal of metalwork and implantation of local antibiotics
F. Removal of metalwork and bone grafting
G. Intravenous antibiotics

Which option is the best management plan? Each option may be used once, more than once or not
at all

19. A 65 year old type 2 diabetic with poor glycaemic control is admitted with forefoot
cellulitis. X-ray of the foot shows some evidence of osteomyelitis of the 2nd ray but
overlying skin is healthy.

You answered Lautenbach regime

The correct answer is Intravenous antibiotics

It is worth attempting to try and resolve this situation with antibiotics at first presentation.
A primary amputation will not heal well and may result in progressive surgery.

20. A 28 year old infantryman is shot in the leg during combat. Primary debridement and
lavage of the wound is undertaken. Several months post surgery there is ongoing discharge
from a sinus originating in the proximal femur, X-ray and MRI shows evidence of
osteomyelitis of the proximal femur. There are no obvious sequestra.

Lautenbach regime

This involves local administration of antibiotics via intramedullary lines and is an


intensive regime. However, the morbidity of a high above knee or hindquater amputation
makes conservative management an attractive option.

21. A 70 year old man undergoes a revision total hip replacement. 10 days post operatively the
hip dislocates and pus is discharging from the wound. He is systemically unwell with a
temperature of 38.5 and WCC 19.

You answered Lautenbach regime

The correct answer is Removal of metalwork and implantation of local antibiotics


Removal of metal work implantation of gentamicin beads and delayed revision is the
mainstay of managing this complication.

Please rate this question:

Discuss and give feedback


Next question

Osteomyelitis

Infection of the bone

Causes

 S aureus and occasionally Enterobacter or Streptococcus species


 In sickle cell: Salmonella species

Clinical features

 Erythema
 Pain
 Fever

Investigation

 X-ray: lytic centre with a ring of sclerosis


 Bone biopsy and culture

Treatment

 Prolonged antibiotics
 Sequestra may need surgical removal

Next question
What is the least likely examination finding in patients with Le Fort II fractures?

Excessive mobility of the palate

Paraesthesia in the region supplied by the inferior alveolar nerve

Malocclusion of the teeth

Enopthalmos

Parasthesia in the region supplied by the infraorbital nerve

Le Fort II fractures have a pyramidal shape. The fracture line involves the orbit and extends to
involve the bridge of the nose and the ethmoids. In continues to involve the infraorbital rim and
usually through the infraorbital foramen. As a result infraorbital parasthesia, palatal mobility and
malocclusion are common findings. Severe fractures may result in endopthalmos. However, the
fracture does not, by definition, involve the inferior alveolar nerve.
Please rate this question:

Discuss and give feedback


Next question

Craniomaxillofacial injuries

Craniomaxillofacial injuries in the UK are due to:

 Interpersonal violence (52%)


 Motor vehicle accidents (16%)
 Sporting injuries (19%)
 Falls (11%)

Le Fort Fractures
Grade Feature

Le The fracture extends from the nasal septum to the lateral pyriform rims, travels
Fort 1 horizontally above the teeth apices, crosses below the zygomaticomaxillary junction, and
Grade Feature

traverses the pterygomaxillary junction to interrupt the pterygoid plates.

Le These fractures have a pyramidal shape and extend from the nasal bridge at or below the
Fort 2 nasofrontal suture through the frontal process of the maxilla, inferolaterally through the
lacrimal bones and inferior orbital floor and rim through or near the inferior orbital
foramen, and inferiorly through the anterior wall of the maxillary sinus; it then travels
under the zygoma, across the pterygomaxillary fissure, and through the pterygoid plates.

Le These fractures start at the nasofrontal and frontomaxillary sutures and extend posteriorly
Fort 3 along the medial wall of the orbit through the nasolacrimal groove and ethmoid bones.
The thicker sphenoid bone posteriorly usually prevents continuation of the fracture into
the optic canal. Instead, the fracture continues along the floor of the orbit along the
inferior orbital fissure and continues superolaterally through the lateral orbital wall,
through the zygomaticofrontal junction and the zygomatic arch. Intranasally, a branch of
the fracture extends through the base of the perpendicular plate of the ethmoid, through
the vomer, and through the interface of the pterygoid plates to the base of the sphenoid.
This type of fracture predisposes the patient to CSF rhinorrhea more commonly than the
other types.

Ocular injuries
Superior orbital fissure syndrome
Severe force to the lateral wall of the orbit resulting in compression of neurovascular structures.
Results in :

 Complete opthalmoplegia and ptosis (Cranial nerves 3, 4, 6 and nerve to levator palpebrae
superioris)
 Relative afferent pupillary defect
 Dilatation of the pupil and loss of accommodation and corneal reflexes
 Altered sensation from forehead to vertex (frontal branch of trigeminal nerve)

Orbital blow out fracture


Typically occurs when an object of slightly larger diameter than the orbital rim strikes the
incompressible eyeball. The bone fragment is displaced downwards into the antral cavity, remaining
attached to the orbital periosteum. Periorbital fat may be herniated through the defect, interfering
with the inferior rectus and inferior oblique muscles which are contained within the same fascial
sheath. This prevents upward movement and outward rotation of the eye and the patient
experiences diplopia on upward gaze. The initial bruising and swelling may make assessment
difficult and patients should usually be reviewed 5 days later. Residual defects may require orbital
floor reconstruction.
Nasal Fractures

 Common injury
 Ensure new and not old deformity
 Control epistaxis
 CSF rhinorrhoea implies that the cribriform plate has been breached and antibiotics will be
required.
 Usually best to allow bruising and swelling to settle and then review patient clinically. Major
persistent deformity requires fracture manipulation, best performed within 10 days of injury.

Retrobulbar haemorrhage
Rare but important ocular emergency. Presents with:

 Pain (usually sharp and within the globe)


 Proptosis
 Pupil reactions are lost
 Paralysis (eye movements lost)
 Visual acuity is lost (colour vision is lost first)

May be the result of Le Fort type facial fractures.

Management:

 Mannitol 1g/Kg as 20% infusion, Osmotic diuretic, Contra-indicated in congestive heart


failure and pulmonary oedema
 Acetazolamide 500mg IV, (Monitor FBC/U+E) Reduces aqueous pressure by inhibition of
carbonic anhydrase (used in glaucoma)
 Dexamethasone 8mg orally or intravenously
 In a traumatic setting an urgent cantholysis may be needed prior to definitive surgery.

Consider
Papaverine 40mg smooth muscle relaxant
Dextran 40 500mls IV improves perfusion
Next question
A Medical F1 phones you as he is concerned his patient has had a major internal bleed. The patient
is 42 years old and is known to have sickle cell anaemia. His blood results are:

Hb 3.7 g /dl
Reticulocyte count 0.4%

His Hb is normally 7g/dl. What is the diagnosis?

Psoas haemorrhage

Acute sequestration

Parvovirus

Splenic haemorrhage

Acute haemolysis

A sudden anemia and a LOW reticulocute count indicates parvovirus. Acute sequestration and
haemolysis causes a high reticulocyte count. There is no clinical indication to suspect a bleed,
therefore you can advise the F1 not to panic and to speak to the haematologists!
Please rate this question:

Discuss and give feedback


Next question

Sickle cell anaemia

 Autosomal recessive
 Single base mutation
 Deoxygenated cells become sickle in shape
 Causes: short red cell survival, obstruction of microvessels and infarction
 Sickling is precipitated by: dehydration, infection, hypoxia
 Manifest at 6 months age
 Africans, Middle East, Indian
 Diagnosis: Hb electrophoresis

Sickle crises
 Bone pain
 Pleuritic chest pain: acute sickle chest syndrome commonest cause of death
 CVA, seizures
 Papillary necrosis
 Splenic infarcts
 Priapism
 Hepatic pain

Hb does not fall during a crisis, unless there is

 Aplasia: parvovirus
 Acute sequestration
 Haemolysis

Long-term complications

 Infections: Streptococcus pnemoniae


 Chronic leg ulcers
 Gallstones: haemolysis
 Aseptic necrosis of bone
 Chronic renal disease
 Retinal detachment, proliferative retinopathy

Surgical complications

 Bowel ischaemia
 Cholecystitis
 Avascular necrosis

Management

 Supportive
 Hydroxyurea
 Repeated transfusions pre operatively
 Exchange transfusion in emergencies

Sickle cell trait


 Heterozygous state
 Asymptomatic
 Symptoms associated with extreme situations ie anaesthesia complications
 Protective against Plasmodium falciparum

Next question
Theme: Management of chest trauma

A. Thoracotomy in operating theatre


B. 36F intercostal chest drain
C. 14F intercostal chest drain
D. Active observation
E. Thoracotomy in the emergency room
F. MRI of aortic arch
G. Bronchoscopy
H. Pericardiocentesis
I. Further transfusion

For each of the following scenarios please select the most appropriate next stage of management.
Each option may be used once, more than once or not at all.

24. A 30 year old male is stabbed outside a nightclub he has a brisk haemoptysis and in
casualty has a chest drain inserted into the left chest. This drained 750ml frank blood. He
fails to improve with this intervention. He has received 4 units of blood. His CVP is now
13.

You answered Thoracotomy in operating theatre

The correct answer is Pericardiocentesis

This man has cardiac tamponade. The raised CVP in the setting of haemodynamic
compromise is the pointer to this. Whilst he will almost certainly require surgery, he
requires urgent decompression of his heart first.

25. A 26 year old male falls from a cliff. He suffers from multiple fractures and has a right
sided pneumothorax that has collapsed a 1/3 of his lung. He has no respiratory
compromise.

You answered Thoracotomy in operating theatre

The correct answer is 14F intercostal chest drain

Simple observation is unsafe as he will almost certainly have suffered an oblique


laceration to his lung. These can become tension pneumothoraces. In the absence of blood
a 36 F drain is probably not required

26. An 18 year old male is shot in the left chest he was unstable but his blood pressure has
improved with 1 litre of colloid. His chest x-ray shows a left sided pneumothorax with no
lung visible.
You answered Thoracotomy in operating theatre

The correct answer is 36F intercostal chest drain

This man requires wide bore intercostal tube drainage. Smaller intercostal chest drains can
become occluded with blood clot and fail to function adequately.

Please rate this question:

Discuss and give feedback


Next question

Thoracic trauma

Types of thoracic trauma

Tension  Often laceration to lung parenchyma with flap


pneumothorax  Pressure develops in thorax
 Most common cause is mechanical ventilation in patient with
pleural injury
 Symptoms overlap with cardiac tamponade, hyper-resonant
percussion note is more likely in tension pnemothorax

Flail chest  Chest wall disconnects from thoracic cage


 Multiple rib fractures (at least two fractures per rib in at least two
ribs)
 Associated with pulmonary contusion
 Abnormal chest motion
 Avoid over hydration and fluid overload

Pneumothorax  Most common cause is lung laceration with air leakage


 Most traumatic pneumothoraces should have a chest drain
 Patients with traumatic pneumothorax should never be
mechanically ventilated until a chest drain is inserted

Haemothorax  Most commonly due to laceration of lung, intercostal vessel or


internal mammary artery
 Haemothoraces large enough to appear on CXR are treated with
large bore chest drain
 Surgical exploration is warranted if >1500ml blood drained
immediately

Cardiac tamponade  Beck's triad: elevated venous pressure, reduced arterial pressure,
reduced heart sounds
 Pulsus paradoxus
 May occur with as little as 100ml blood

Pulmonary contusion  Most common potentially lethal chest injury


 Arterial blood gases and pulse oximetry important
 Early intubation within an hour if significant hypoxia

Blunt cardiac injury  Usually occurs secondary to chest wall injury


 ECG may show features of myocardial infarction
 Sequelae: hypotension, arrhythmias, cardiac wall motion
abnormalities

Aorta disruption  Deceleration injuries


 Contained haematoma
 Widened mediastinum

Diaphragm  Most due to motor vehicle accidents and blunt trauma causing
disruption large radial tears (laceration injuries result in small tears)
 More common on left side
 Insert gastric tube, may pass into intrathoracic stomach

Mediastinal  Entrance wound in one hemithorax and exit wound/foreign body


traversing wounds in opposite hemithorax
 Mediastinal haematoma or pleural cap suggests great vessel
injury
 Mortality is 20%

References
ATLS Manual 8th Edition
Next question
Theme: Management of burns

A. Escharotomy
B. Endotracheal intubation
C. Broad spectrum intravenous antibiotics
D. Intravenous fluids calculated according to extent of burned area
E. Discharge with review in outpatients
F. Transfer to regional burn centre once stabilised
G. Split thickness skin graft
H. Full thickness skin graft

What is the best management for the scenario given? Each option may be used once, more than
once or not at all.

27. A 34 year old women trips over and falls into a bonfire whilst intoxicated at a party. She
suffers burns to her arms, torso and face. These are calculated to be 25% body surface
area. She is otherwise stable. The burns to the torso are superficial, her left forearm has a
full thickness burn and the burns to her face are superficial. There is no airway
compromise. She has received 1000ml of intravenous Hartman's solution, with a further
1000ml prescribed to run over 4 hours.

You answered Escharotomy

The correct answer is Transfer to regional burn centre once stabilised

This women has been resuscitated and requires transfer for specialist management.

28. A 20 year old man is trapped in a warehouse fire. He has sustained 60% burns to his torso
and limbs. The limb burns are partial thickness but the torso burns are full thickness. He
was intubated by paramedics at the scene and is receiving intravenous fluids. His
ventilation pressure requirements are rising.

Escharotomy

He requires an escharotomy as this will be contributing to impaired ventilation.

29. An 18 year old man accidentally pours boiling water onto his left arm. The area is
erythematous and has a blister measuring 5cm. The wound is extremely painful.

You answered Escharotomy

The correct answer is Discharge with review in outpatients


This is a superficial burn and should recover with no further input than simple dressings,
an alternative would be deroofing the blister and applying dressings prior to outpatient
review.

Please rate this question:

Discuss and give feedback


Next question

Burns

Burns may be thermal, chemical or electrical. In the former category are burns which occur as a
result of heat. Chemical burns occur when the skin is exposed to an extremely caustic or alkaline
substance. Electrical burns occur following exposure to electrical current. The immediate
management includes removal of the burning source which usually includes irrigation of the burned
area. A detailed assessment then needs to be made of the extent of the burns and a number of
charts are available for recording this information. The degree of injury relates to the temperature
and duration of exposure. Most domestic burns are mainly scalds in young children.

Following the burn, there is a local response with progressive tissue loss and release of
inflammatory cytokines. Systemically, there are cardiovascular effects resulting from fluid loss and
sequestration of fluid into the third space. There is a marked catabolic response. Immunosupression
is common with large burns and bacterial translocation from the gut lumen is a recognised event.
Sepsis is a common cause of death following major burns.

Types of burn

Type of burn Skin layers Skin Blanching Management


affected appearance

Epidermal/Superficial Epidermis Red, moist Yes

Superficial partial Epidermis and part Pale, dry Yes Normally heals
thickness of papillary dermis with no
affected intervention

Deep partial thickness Epidermis, whole Mottled red No Needs surgical


papillary dermis colour intervention
affected (depending on site)
Full thickness Whole skin layer Dry, leathery No Burns centre
and subcutaneous hard wound
tissue affected

Depth of burn assessment

 Bleeding on needle prick


 Sensation
 Appearance
 Blanching to pressure

Percentage burn estimation


Lund Browder chart: most accurate even in children
Wallace rule of nines
Palmar surface: surface area palm = 0.8% burn

>15% body surface area burns in adults needs urgent burn fluid resuscitation

Transfer to burn centre if:

 Need burn shock resuscitation


 Face/hands/genitals affected
 Deep partial thickness or full thickness burns
 Significant electrical/chemical burns

Management
The initial aim is to stop the burning process and resuscitate the patient. Intravenous fluids will be
required for children with burns greater than 10% of total body surface area. Adults with burns
greater than 15% of total body surface area will also require IV fluids. The fluids are calculated using
the Parkland formula which is; volume of fluid= total body surface area of the burn % x weight (Kg)
x4. Half of the fluid is administered in the first 8 hours. A urinary catheter should be inserted.
Analgesia should be given. Complex burns, burns involving the hand perineum and face and burns
>10% in adults and >5% in children should be transferred to a burns unit.

Circumferential burns affecting a limb or severe torso burns impeding respiration may require
escharotomy to divide the burnt tissue.

Conservative management is appropriate for superficial burns and mixed superficial burns that will
heal in 2 weeks. More complex burns may require excision and skin grafting. Excision and primary
closure is not generally practised as there is a high risk of infection.

There is no evidence to support the use of anti microbial prophylaxis or topical antibiotics in burn
patients.

Escharotomies
 Indicated in circumferential full thickness burns to the torso or limbs.
 Careful division of the encasing band of burn tissue will potentially improve ventilation (if the
burn involves the torso), or relieve compartment syndrome and oedema (where a limb is
involved)

References
www.euroburn.org/e107files/downloads/guidelinesburncare.pdf

Barajas-Nava LA, López-Alcalde J, Roqué i Figuls M, Solà I, Bonfill Cosp X. Antibiotic prophylaxis
for preventing burn wound infection. Cochrane Database of Systematic Reviews 2013, Issue 6. Art.
No.: CD008738. DOI: 10.1002/14651858.CD008738.pub2.

Hettiaratchy S & Papini R. Initial management of a major burn: assessment and resuscitation. BMJ
2004;329:101-103
Next question
A 52 year old male type 2 diabetic is admitted to the vascular ward for a femoral popliteal bypass.
He suddenly develops expressive dysphasia and marked right sided weakness. The Senior house
officer arranges a CT head scan which shows a 60% left middle cerebral artery territory infarct.
There are no beds on the stroke unit. Overnight the patient becomes unresponsive and a CT head
confirms no bleed. What is the next best management option?

IV heparin

Clopidogrel

Burr hole surgery

Aspirin

Hemicranieotomy

The likely cause for the reduced consciousness is raised intracranial pressure due to increasing
cerebral oedema related to the infarct. In this situation, urgent neurosurgical review is needed for
possible decompressive hemicranieotomy to relieve the pressure. Ideally no further antiplatelet or
anticoagulation therapy should be given until a plan for surgery is confirmed.

Indications for hemicranieotomy include:

 Age under 60 years


 Clinical deficit in middle cerebral artery territory
 Decreased consciousness
 >50% territory infarct

Please rate this question:

Discuss and give feedback


Next question

Stroke: types

Primary intracerebral  Presents with headache, vomiting, loss of


haemorrhage (PICH, c. 10%) consciousness
Total anterior circulation  Involves middle and anterior cerebral arteries
infarcts (TACI, c. 15%)  Hemiparesis/hemisensory loss
 Homonymous hemianopia
 Higher cognitive dysfunction e.g. Dysphasia

Partial anterior circulation  Involves smaller arteries of anterior circulation e.g.


infarcts (PACI, c. 25%) upper or lower division of middle cerebral artery
 Higher cognitive dysfunction or two of the three TACI
features

Lacunar infarcts (LACI, c.  Involves perforating arteries around the internal


25%) capsule, thalamus and basal ganglia
 Present with either isolated hemiparesis, hemisensory
loss or hemiparesis with limb ataxia

Posterior circulation infarcts  Vertebrobasilar arteries


(POCI, c. 25%)  Presents with features of brainstem damage
 Ataxia, disorders of gaze and vision, cranial nerve
lesions

Lateral medullary syndrome  Wallenberg's syndrome


(posterior inferior cerebellar  Ipsilateral: ataxia, nystagmus, dysphagia, facial
artery) numbness, cranial nerve palsy
 Contralateral: limb sensory loss

Weber's syndrome  Ipsilateral III palsy


 Contralateral weakness

--------------------------------------

Anterior cerebral artery

 Contralateral hemiparesis and sensory loss, lower extremity > upper


 Disconnection syndrome

Middle cerebral artery


 Contralateral hemiparesis and sensory loss, upper extremity > lower
 Contralateral hemianopia
 Aphasia (Wernicke's)
 Gaze abnormalities

Posterior cerebral artery

 Contralateral hemianopia with macular sparing


 Disconnection syndrome

Lacunar

 Present with either isolated hemiparesis, hemisensory loss or hemiparesis with limb ataxia

Lateral medulla (posterior inferior cerebellar artery)

 Ipsilateral: ataxia, nystagmus, dysphagia, facial numbness, cranial nerve palsy e.g.

Horner's

 Contralateral: limb sensory loss

Pontine

 VI nerve: horizontal gaze palsy


 VII nerve
 Contralateral hemiparesis

Next question
A 56-year-old female is admitted to ITU with a severe pancreatitis. Thyroid function tests show:

TSH = 0.5 Low


Thyroxine = 1.0 Low
T3 = 0.5 Low

What is the most likely cause?

Sick euthyroid syndrome

Graves disease

Hashimotos thyroiditis

Levothyroxine

None of the above

This patient has sick euthyroid syndrome as all thyroid parameters are reduced. Graves disease and
levothyroxine will cause hyperthyroidism (low TSH and elevated thyroxine/T3). Hashimotos
thyroiditis is associated with hypothyroidism (high TSH and low thyroxine/T3).

Please rate this question:

Discuss and give feedback

Sick euthyroid syndrome

In sick euthyroid syndrome (now referred to as non-thyroidal illness) it is often said that everything
(TSH, thyroxine and T3) is low. In the majority of cases however the TSH level is within the normal
range (inappropriately normal given the low thyroxine and T3).

Changes are reversible upon recovery from the systemic illness.


Theme: Hand disorders

A. de Quervain's tenosynovitis
B. Dupuytren's contracture
C. Bouchard's nodes
D. Ganglion
E. Carpal tunnel syndrome
F. Radial nerve injury
G. Ulnar nerve injury
H. Heberden's nodes
I. Tendon sheath infection

Please select the most likely diagnosis to account for the scenario given. Each option may be used
once, more than once or not at all.

1. A 49 -year-old male presents with discomfort in the fingers of his left hand. On examination
the ring and little fingers of his left hand are flexed and unable to extend completely. He is
able to make a fist with the hand. Palpation reveals thickened nodules on the medial half of
the palm.

You answered de Quervain's tenosynovitis

The correct answer is Dupuytren's contracture

Discomfort of the hand is not uncommon in Dupuytrens contracture, true pain is unusual.
The disease most commonly affects the ring and little fingers.

2. A 62 year old man presents after his wife commented on the unusual shape of his fingers.
On examination he has a hard swelling adjacent to the distal interphalangeal joint of his
index finger of the right hand with lateral deviation of the finger tip. There is no sensory
disturbance and the swelling is not tender.

You answered de Quervain's tenosynovitis

The correct answer is Heberden's nodes

These are bony outgrowths that occur in the distal interphalangeal joint in association with
osteoarthritis. They may skew the finger tip sideways. Bouchards nodes are similar, but
affect the proximal interphalangeal joint.

3. A 57 year - old lady presents with a three month history of pins and needles in the fingers of
the right hand, particularly at night. On examination, there is some loss of the sensation over
the palmar aspect of the lateral three fingers and wasting of the thenar eminence.
You answered de Quervain's tenosynovitis

The correct answer is Carpal tunnel syndrome

Carpal tunnel syndrome commonly produces pain at night as the wrists are flexed during
sleep. Compromise of the median nerve may produce wasting of the thenar eminence
muscles.

Please rate this question:

Discuss and give feedback


Next question

Hand diseases

Dupuytrens contracture

 Fixed flexion contracture of the hand where the fingers bend towards the palm and cannot be
fully extended.
 Caused by underlying contractures of the palmar aponeurosis . The ring finger and little
finger are the fingers most commonly affected. The middle finger may be affected in
advanced cases, but the index finger and the thumb are nearly always spared.
 Progresses slowly and is usually painless. In patients with this condition, the tissues under
the skin on the palm of the hand thicken and shorten so that the tendons connected to the
fingers cannot move freely. The palmar aponeurosis becomes hyperplastic and undergoes
contracture.
 Commonest in males over 40 years of age.
 Association with liver cirrhosis and alcoholism. However, many cases are idiopathic.
 Treatment is surgical and involves fasciectomy. However, the condition may recur and many
surgical therapies are associated with risk of neurovascular damage to the digital nerves and
arteries.

Carpel tunnel syndrome

 Idiopathic median neuropathy at the carpal tunnel.


 Characterised by altered sensation of the lateral 3 fingers.
 The condition is commoner in females and is associated with other connective tissue
disorders such as rheumatoid disease. It may also occur following trauma to the distal
radius.
 Symptoms occur mainly at night in early stages of the condition.
 Examination may demonstrate wasting of the muscles of the thenar eminence and
symptoms may be reproduced by Tinels test (compression of the contents of the carpal
tunnel).
 Formal diagnosis is usually made by electrophysiological studies.
 Treatment is by surgical decompression of the carpal tunnel, a procedure achieved by
division of the flexor retinaculum. Non - surgical options include splinting and bracing.

Miscellaneous hand lumps


Osler's Osler's nodes are painful, red, raised lesions found on the hands and feet. They are
nodes the result of the deposition of immune complexes.

Bouchards Hard, bony outgrowths or gelatinous cysts on the proximal interphalangeal joints
nodes (the middle joints of fingers or toes.) They are a sign of osteoarthritis, and are
caused by formation of calcific spurs of the articular cartilage.

Heberdens Typically develop in middle age, beginning either with a chronic swelling of the
nodes affected joints or the sudden painful onset of redness, numbness, and loss of
manual dexterity. This initial inflammation and pain eventually subsides, and the
patient is left with a permanent bony outgrowth that often skews the fingertip
sideways. It typically affects the DIP joint.

Ganglion Swelling in association with a tendon sheath commonly near a joint. They are
common lesions in the wrist and hand. Usually they are asymptomatic and cause
little in the way of functional compromise. They are fluid filled although the fluid
is similar to synovial fluid it is slightly more viscous. When the cysts are
troublesome they may be excised.
Theme: Hand disorders

A. Malignant fibrous histiocytoma


B. Oslers nodes
C. Heberdens nodes
D. Bouchards nodes
E. Carpal tunnel syndrome
F. Complex regional pain syndrome
G. Osteoclastoma
H. Osteosarcoma
I. Ganglion

Please select the most likely diagnosis for the lesion described. Each option may be used once,
more than once or not at all.

4. A 42 year old lady who has systemic lupus erythematosus presents to the clinic with a 5 day
history of a painful purple lesion on her index finger. On examination she has a tender red
lesion on the index finger.

You answered Malignant fibrous histiocytoma

The correct answer is Oslers nodes

Osler nodes are normally described as tender, purple/red raised lesions with a pale centre.
These lesions occur as a result of immune complex deposition. These occur most often in
association with endocarditis. However, other causes include SLE, gonorrhoea, typhoid and
haemolytic anaemia.

5. A 62 year old lady presents with an non tender lump overlying the distal interphalangeal
joint of the index finger. On examination she has a hard, non tender lump overlying the joint
and deviation of the tip of the finger.

You answered Malignant fibrous histiocytoma

The correct answer is Heberdens nodes

Heberdens nodes may produce swelling of the distal interphalangeal joint with deviation of
the finger tip.

6. A 17 year old boy is brought to the clinic by his mother who is concerned about a lesion
that has developed on the dorsal surface of his left hand. On examination he has a soft
fluctuant swelling on the dorsal aspect of the hand, it is most obvious on making a fist.
You answered Malignant fibrous histiocytoma

The correct answer is Ganglion

Ganglions commonly occur in the hand and are usually associated with tendons. They are
typically soft and fluctuant. They do not require removal unless they are atypical or causing
symptoms.

Please rate this question:

Discuss and give feedback

Hand diseases

Dupuytrens contracture

 Fixed flexion contracture of the hand where the fingers bend towards the palm and cannot be
fully extended.
 Caused by underlying contractures of the palmar aponeurosis . The ring finger and little
finger are the fingers most commonly affected. The middle finger may be affected in
advanced cases, but the index finger and the thumb are nearly always spared.
 Progresses slowly and is usually painless. In patients with this condition, the tissues under
the skin on the palm of the hand thicken and shorten so that the tendons connected to the
fingers cannot move freely. The palmar aponeurosis becomes hyperplastic and undergoes
contracture.
 Commonest in males over 40 years of age.
 Association with liver cirrhosis and alcoholism. However, many cases are idiopathic.
 Treatment is surgical and involves fasciectomy. However, the condition may recur and many
surgical therapies are associated with risk of neurovascular damage to the digital nerves and
arteries.

Carpel tunnel syndrome

 Idiopathic median neuropathy at the carpal tunnel.


 Characterised by altered sensation of the lateral 3 fingers.
 The condition is commoner in females and is associated with other connective tissue
disorders such as rheumatoid disease. It may also occur following trauma to the distal
radius.
 Symptoms occur mainly at night in early stages of the condition.
 Examination may demonstrate wasting of the muscles of the thenar eminence and
symptoms may be reproduced by Tinels test (compression of the contents of the carpal
tunnel).
 Formal diagnosis is usually made by electrophysiological studies.
 Treatment is by surgical decompression of the carpal tunnel, a procedure achieved by
division of the flexor retinaculum. Non - surgical options include splinting and bracing.
Miscellaneous hand lumps
Osler's Osler's nodes are painful, red, raised lesions found on the hands and feet. They are
nodes the result of the deposition of immune complexes.

Bouchards Hard, bony outgrowths or gelatinous cysts on the proximal interphalangeal joints
nodes (the middle joints of fingers or toes.) They are a sign of osteoarthritis, and are
caused by formation of calcific spurs of the articular cartilage.

Heberdens Typically develop in middle age, beginning either with a chronic swelling of the
nodes affected joints or the sudden painful onset of redness, numbness, and loss of
manual dexterity. This initial inflammation and pain eventually subsides, and the
patient is left with a permanent bony outgrowth that often skews the fingertip
sideways. It typically affects the DIP joint.

Ganglion Swelling in association with a tendon sheath commonly near a joint. They are
common lesions in the wrist and hand. Usually they are asymptomatic and cause
little in the way of functional compromise. They are fluid filled although the fluid
is similar to synovial fluid it is slightly more viscous. When the cysts are
troublesome they may be excised.
Theme: Parotid gland disease

A. Pleomorphic adenoma
B. Adenoid cystic carcinoma
C. Sarcoid
D. Sjogrens syndrome
E. Sialolithiasis
F. Wharthins tumour

Please select the most likely explanation for the following patients with parotid gland symptoms.
Each option may be used once, more than once or not at all.

1. A 50 year old female presents with bilateral parotid gland swelling and symptoms of a dry
mouth. On examination she has bilateral facial nerve palsies. This improved following
steroid treatment.

You answered Pleomorphic adenoma

The correct answer is Sarcoid

Theme from April 2013 exam


Sarcoid occurs bilaterally in 70% of cases and facial nerve involvement is recognised.
Treatment is conservative in most cases although individuals with facial nerve palsy will
usually receive steroids with good effect.

2. A 50 year old women presents with a diffuse swelling in the region of her right parotid
together with facial pain. On examination she has a right sided facial nerve palsy.

You answered Pleomorphic adenoma

The correct answer is Adenoid cystic carcinoma

Theme from April 2015 exam


Adenoid cystic carcinoma commonly infiltrates the facial nerve and may cause neuropathy
and facial pain.

3. A 50 year old lady presents with symptoms of a dry mouth that has been present for the past
few months. She also has a sensation of grittiness in her eyes. On examination she has a
diffuse swelling of her parotid gland. There is no evidence of facial nerve palsy.

You answered Pleomorphic adenoma

The correct answer is Sjogrens syndrome


Theme from April 2014 exam
Most patients with Sjogrens present in the post menopausal years. Multi system
involvement is common.

Please rate this question:

Discuss and give feedback


Next question

Parotid gland clinical

Benign neoplasms
Up to 80% of all salivary gland tumours occur in the parotid gland and up to 80% of these are
benign. There is no consistent correlation between the rate of growth and the malignant potential of
the lesion. However, benign tumours should not invade structures such as the facial nerve.
With the exception of Warthins tumours, they are commoner in women than men. The median age of
developing a lesion is in the 5th decade of life.

Benign tumour types


Tumour type Features

Benign pleomorphic adenoma or Most common parotid neoplasm (80%)


benign mixed tumor Proliferation of epithelial and myoepithelial cells of the ducts
and an increase in stromal components
Slow growing, lobular, and not well encapsulated
Recurrence rate of 1-5% with appropriate excision
(parotidectomy)
Recurrence possibly secondary to capsular disruption during
surgery
Malignant degeneration occurring in 2-10% of adenomas
observed for long periods, with carcinoma ex-pleomorphic
adenoma occurring most frequently as adenocarcinoma

Warthin tumor (papillary Second most common benign parotid tumor (5%)
cystadenoma lymphoma or Most common bilateral benign neoplasm of the parotid
adenolymphoma) Marked male as compared to female predominance
Occurs later in life (sixth and seventh decades)
Presents as a lymphocytic infiltrate and cystic epithelial
proliferation
May represent heterotopic salivary gland epithelial tissue
trapped within intraparotid lymph nodes
Incidence of bilaterality and multicentricity of 10%
Tumour type Features

Malignant transformation rare (almost unheard of)

Monomorphic adenoma Account for less than 5% of tumours


Slow growing
Consist of only one morphological cell type (hence term
mono)
Include; basal cell adenoma, canalicular adenoma,
oncocytoma, myoepitheliomas

Haemangioma Should be considered in the differential of a parotid mass in a


child
Accounts for 90% of parotid tumours in children less than 1
year of age
Hypervascular on imaging
Spontaneous regression may occur and malignant
transformation is almost unheard of

Malignant salivary gland tumours


Types of malignancy

Mucoepidermoid 30% of all parotid malignancies


carcinoma Usually low potential for local invasiveness and metastasis (depends
mainly on grade)

Adenoid cystic Unpredictable growth pattern


carcinoma Tendency for perineural spread
Nerve growth may display skip lesions resulting in incomplete excision
Distant metastasis more common (visceral rather than nodal spread)
5 year survival 35%

Mixed tumours Often a malignancy occurring in a previously benign parotid lesion

Acinic cell carcinoma Intermediate grade malignancy


May show perineural invasion
Low potential for distant metastasis
5 year survival 80%

Adenocarcinoma Develops from secretory portion of gland


Risk of regional nodal and distant metastasis
5 year survival depends upon stage at presentation, may be up to 75%
with small lesions with no nodal involvement

Lymphoma Large rubbery lesion, may occur in association with Warthins tumours
Diagnosis should be based on regional nodal biopsy rather than parotid
resection
Treatment is with chemotherapy (and radiotherapy)

Diagnostic evaluation

 Plain x-rays may be used to exclude calculi


 Sialography may be used to delineate ductal anatomy
 FNAC is used in most cases
 Superficial parotidectomy may be either diagnostic of therapeutic depending upon the nature
of the lesion
 Where malignancy is suspected the primary approach should be definitive resection rather
than excisional biopsy
 CT/ MRI may be used in cases of malignancy for staging primary disease

Treatment
For nearly all lesions this consists of surgical resection, for benign disease this will usually consist of
a superficial parotidectomy. For malignant disease a radical or extended radical parotidectomy is
performed. The facial nerve is included in the resection if involved. The need for neck dissection is
determined by the potential for nodal involvement.

Other parotid disorders


HIV infection

 Lymphoepithelial cysts associated with HIV occur almost exclusively in the parotid
 Typically presents as bilateral, multicystic, symmetrical swelling
 Risk of malignant transformation is low and management usually conservative

Sjogren syndrome

 Autoimmune disorder characterised by parotid enlargement, xerostomia and


keratoconjunctivitis sicca
 90% of cases occur in females
 Second most common connective tissue disorder
 Bilateral, non tender enlargement of the gland is usual
 Histologically, the usual findings are of a lymphocytic infiltrate in acinar units and
epimyoepithelial islands surrounded by lymphoid stroma
 Treatment is supportive
 There is an increased risk of subsequent lymphoma

Sarcoid

 Parotid involvement occurs in 6% of patients with sarcoid


 Bilateral in most cases
 Gland is not tender
 Xerostomia may occur
 Management of isolated parotid disease is usually conservative

Next question
A 53 year old man presents with a mass lesion slightly inferior to the tragus of his right ear. An FNA
and USS are performed and a 2cm pleomorphic adenoma is diagnosed. What is the most
appropriate course of action?

USS guided core biopsy

Radical parotidectomy

Superficial parotidectomy

Discharge

MRI scanning of the region

Pleomorphic adenoma of the parotid= surgical excision

Theme from September 2014 Exam


Pleomorphic adenomas a usually benign tumours. However, they will enlarge over time and a
proportion can undergo malignant transformation. Therefore, all pleomorphic adenomas are excised
and a superficial parotidectomy is generally the procedure of choice. The facial nerve is preserved.
More recently, there has been a move towards partial superficial parotidectomy. However, complete
resection of the lesion is still madatory.
Please rate this question:

Discuss and give feedback


Next question

Parotid gland clinical

Benign neoplasms
Up to 80% of all salivary gland tumours occur in the parotid gland and up to 80% of these are
benign. There is no consistent correlation between the rate of growth and the malignant potential of
the lesion. However, benign tumours should not invade structures such as the facial nerve.
With the exception of Warthins tumours, they are commoner in women than men. The median age of
developing a lesion is in the 5th decade of life.

Benign tumour types


Tumour type Features

Benign pleomorphic adenoma or Most common parotid neoplasm (80%)


Tumour type Features

benign mixed tumor Proliferation of epithelial and myoepithelial cells of the ducts
and an increase in stromal components
Slow growing, lobular, and not well encapsulated
Recurrence rate of 1-5% with appropriate excision
(parotidectomy)
Recurrence possibly secondary to capsular disruption during
surgery
Malignant degeneration occurring in 2-10% of adenomas
observed for long periods, with carcinoma ex-pleomorphic
adenoma occurring most frequently as adenocarcinoma

Warthin tumor (papillary Second most common benign parotid tumor (5%)
cystadenoma lymphoma or Most common bilateral benign neoplasm of the parotid
adenolymphoma) Marked male as compared to female predominance
Occurs later in life (sixth and seventh decades)
Presents as a lymphocytic infiltrate and cystic epithelial
proliferation
May represent heterotopic salivary gland epithelial tissue
trapped within intraparotid lymph nodes
Incidence of bilaterality and multicentricity of 10%
Malignant transformation rare (almost unheard of)

Monomorphic adenoma Account for less than 5% of tumours


Slow growing
Consist of only one morphological cell type (hence term
mono)
Include; basal cell adenoma, canalicular adenoma,
oncocytoma, myoepitheliomas

Haemangioma Should be considered in the differential of a parotid mass in a


child
Accounts for 90% of parotid tumours in children less than 1
year of age
Hypervascular on imaging
Spontaneous regression may occur and malignant
transformation is almost unheard of

Malignant salivary gland tumours


Types of malignancy

Mucoepidermoid 30% of all parotid malignancies


carcinoma Usually low potential for local invasiveness and metastasis (depends
mainly on grade)

Adenoid cystic Unpredictable growth pattern


carcinoma Tendency for perineural spread
Nerve growth may display skip lesions resulting in incomplete excision
Distant metastasis more common (visceral rather than nodal spread)
5 year survival 35%

Mixed tumours Often a malignancy occurring in a previously benign parotid lesion

Acinic cell carcinoma Intermediate grade malignancy


May show perineural invasion
Low potential for distant metastasis
5 year survival 80%

Adenocarcinoma Develops from secretory portion of gland


Risk of regional nodal and distant metastasis
5 year survival depends upon stage at presentation, may be up to 75%
with small lesions with no nodal involvement

Lymphoma Large rubbery lesion, may occur in association with Warthins tumours
Diagnosis should be based on regional nodal biopsy rather than parotid
resection
Treatment is with chemotherapy (and radiotherapy)

Diagnostic evaluation

 Plain x-rays may be used to exclude calculi


 Sialography may be used to delineate ductal anatomy
 FNAC is used in most cases
 Superficial parotidectomy may be either diagnostic of therapeutic depending upon the nature
of the lesion
 Where malignancy is suspected the primary approach should be definitive resection rather
than excisional biopsy
 CT/ MRI may be used in cases of malignancy for staging primary disease
Treatment
For nearly all lesions this consists of surgical resection, for benign disease this will usually consist of
a superficial parotidectomy. For malignant disease a radical or extended radical parotidectomy is
performed. The facial nerve is included in the resection if involved. The need for neck dissection is
determined by the potential for nodal involvement.

Other parotid disorders


HIV infection

 Lymphoepithelial cysts associated with HIV occur almost exclusively in the parotid
 Typically presents as bilateral, multicystic, symmetrical swelling
 Risk of malignant transformation is low and management usually conservative

Sjogren syndrome

 Autoimmune disorder characterised by parotid enlargement, xerostomia and


keratoconjunctivitis sicca
 90% of cases occur in females
 Second most common connective tissue disorder
 Bilateral, non tender enlargement of the gland is usual
 Histologically, the usual findings are of a lymphocytic infiltrate in acinar units and
epimyoepithelial islands surrounded by lymphoid stroma
 Treatment is supportive
 There is an increased risk of subsequent lymphoma

Sarcoid

 Parotid involvement occurs in 6% of patients with sarcoid


 Bilateral in most cases
 Gland is not tender
 Xerostomia may occur
 Management of isolated parotid disease is usually conservative

Next question
Theme: Disorders affecting the ear

A. Acoustic neuroma
B. Otosclerosis
C. Preauricular sinus
D. Acute suppurative otitis media
E. Cholesteatoma
F. Long standing perforation of the pars tensa
G. Otitis externa

Please select the most likely underlying explanation for the disorder described. Each option may be
used once, more than once or not at all.

5. A 34 year old lady presents with a long standing offensive discharge from the ear and on
examination is noted to have a reduction in her hearing of 40 decibels compared to the
opposite side.

You answered Acoustic neuroma

The correct answer is Cholesteatoma

Theme from January 2013 Exam


The combination of offensive discharge and hearing loss is strongly suggestive of
cholesteatoma.

6. A 4 year old is brought to the general practitioner by her mother. She has been distressed
with ear pain for the past 14 hours. She is constantly touching and pulling at her ear. Whilst
she is sat in the waiting room her mother notices a discharge of foul smelling fluid from the
ear, following which the pain resolves.

You answered Acoustic neuroma

The correct answer is Acute suppurative otitis media

In young children acute suppurative otitis media is a common condition. Rupture of the
tympanic membrane is a rare, but recognised complication.

7. A 4 year old child is brought to the clinic by his father. They are concerned because the
child has been noted to have a small epithelial defect anterior to the left ear and is has been
noted to discharge foul smelling material for the past 2 days.

You answered Acoustic neuroma


The correct answer is Preauricular sinus

Pre auricular sinuses that a deeper may accumulate secretions and produce foul smelling
discharge.

Please rate this question:

Discuss and give feedback


Next question

Disorders affecting the ear

Otitis externa
Variant Cause Features Treatment

Acute Boil in external auditory meatus Acute pain on Ear packs may be used
otitis moving the pinna Topical antibiotics
externa Conductive hearing Operative debridement
loss if lesion is large may be needed in severe
When rupture occurs cases
pus will flow from
ear

Chronic Chronic combined infection in Chronic discharge Cleansing of the external


otitis the external auditory meatus from affected ear, ear and treatment with
externa usually combined hearing loss and antifungal and
staphylococcal and fungal severe pain rare antibacterial ear drops
infection

Otitis media
Variant Cause Features Treatment

Acute Viral induced middle Most common in Antibiotics (usually


suppurative ear effusions children and rare in amoxycillin)
otitis media secondary to eustacian adults
tube dysfunction May present with
symptoms elsewhere
(e.g. vomiting) in
children
Variant Cause Features Treatment

Severe pain and


sometimes fever
May present with
discharge is tympanic
rupture occurs

Chronic May occur with or Those without Simple pars tensa


suppurative without cholesteatoma cholesteatoma may perforations may be managed
otitis media Those without complain of non operatively or a
cholesteatoma have a intermittent discharge myringoplasty considered if
perforation of the pars (non offensive) symptoms troublesome.
tensa Those with Pars flaccida perforations
Those with cholesteatoma have will usually require a radical
cholesteatoma have a impaired hearing and mastoidectomy
perforation of the pars foul smelling discharge
flaccida

Otosclerosis

 Progressive conductive deafness


 Secondary to fixation of the stapes in the oval window
 Treatment is with stapedectomy and insertion of a prosthesis

Acoustic neuroma

 Symptoms of gradually progressive unilateral perceptive deafness and tinnitus


 Involvement of the vestibular nerve may cause vertigo
 Extension to involve the facial nerve may cause weakness and then paralysis.

Pre auricular sinus

 Common congenital condition in which an epithelial defect forms around the external ear
 Small sinuses require no treatment
 Deeper sinuses may become blocked and develop episodes of infection, they may be closely
related to the facial nerve and are challenging to excise

Next question
A 46-year-old woman is referred to endocrine surgery for a possible thyroidectomy. She has a
tender neck swelling. Blood results are as follows:

TSH <0.1 mU/l

T4 188 nmol/l

Hb 14.2 g/dl

Plt 377 * 10^9/l

WBC 6.4 * 10^9/l

ESR 65 mm/hr

Technetium thyroid scan shows decreased uptake globally

What is the most likely diagnosis?

Sick thyroid syndrome

Acute bacterial thyroiditis

Hashimoto's thyroiditis

Subacute thyroiditis

Toxic multinodular goitre

This patient does not need surgery! Subacute thyroiditis is suggested by the tender goitre,
hyperthyroidism and raised ESR. The globally reduced uptake on technetium thyroid scan is also
typical. This should resolve without any active intervention.
Please rate this question:

Discuss and give feedback


Next question

Thyroiditis
Sub acute thyroiditis
Subacute thyroiditis (also known as De Quervain's thyroiditis) is thought to occur following viral
infection and typically presents with hyperthyroidism

Features

 Hyperthyroidism
 Painful goitre
 Raised ESR
 Globally reduced uptake on iodine-131 scan

Management

 Usually self-limiting - most patients do not require treatment


 Thyroid pain may respond to aspirin or other NSAIDs
 In more severe cases steroids are used, particularly if hypothyroidism develops

Hashimotos thyroiditis
Hashimotos thyroiditis is an immunological disorder in which lymphocytes become sensitised to
thyroidal antigens. The three most important antibodies include; thyroglobulin, TPO and TSH-R.
During the early phase of Hashimotos the the thyroglobulin antibody is markedly elevated and then
declines.

Features

 Goitre and either euthyroid or mild hypothyroidism


 Progressive hypothyroidism (and associated symptoms)

Management

 During the hyperthyroid phase of illness beta blockers may manage symptoms
 As hypothyroidism develops patients may require thyroxine

Next question
Theme: Voice disorders

A. Vagus nerve injury


B. Thyroid nerve injury
C. Superior laryngeal nerve injury
D. Unilateral inferior laryngeal nerve injury
E. Bilateral inferior laryngeal nerves injuries
F. Stroke
G. Lacunar infarcts
H. None of the above

Please select the most likely reason for the scenarios given. Each option may be used once, more
than once or not at all.

9. A 42 year old singer is admitted for a thyroidectomy. Post operatively she is only able to
make a gargling noise. Her voice sounds breathy.

You answered Vagus nerve injury

The correct answer is Unilateral inferior laryngeal nerve injury

This patient has diplophonia which causes a gargling sound. This is associated with
dysphagia. This can also be caused by a vagus nerve lesion, but the recurrent laryngeal
nerve is more at risk of damage.

10. A 42 year old singer is admitted for a thyroidectomy. Post operatively she is unable to sing
high pitched notes.

You answered Vagus nerve injury

The correct answer is Superior laryngeal nerve injury

Theme from September 2011 Exam

SLN lesions cause difficulty in voice pitch.

11. A 42 year old singer is admitted for a thyroidectomy. Post operatively the patient develops
dyspnoea and is unable to speak.

You answered Vagus nerve injury

The correct answer is Bilateral inferior laryngeal nerves injuries


This patient has aphonia due to bilateral damage to the recurrent laryngeal nerve.

Please rate this question:

Discuss and give feedback


Next question

Voice production

There are 2 main nerves involved:

Superior laryngeal nerve (SLN)

 Innervates the cricothyroid muscle

Since the cricothyroid muscle is involved in adjusting the tension of the vocal fold for high notes
during singing, SLN paresis and paralysis result in:

a. Abnormalities in pitch
b. Inability to sing with smooth change to each higher note (glissando or pitch glide)

Recurrent laryngeal nerve (RLN)/Inferior laryngeal nerve

 Innervates intrinsic larynx muscles

a. Opening vocal folds (as in breathing, coughing)


b. Closing vocal folds for vocal fold vibration during voice use
c. Closing vocal folds during swallowing
Next question
Theme: Nasal diseases

A. Ethmoid sinus cancer


B. Maxillary sinus cancer
C. Ethmoid adenoma
D. Maxillary adenoma
E. Ethmoidal fracture
F. Nasal polyps
G. Sphenoid osteoma
H. Ethmoidal sinusitis
I. Maxillary sinusitis

Please select the most likely diagnosis for the scenario given. Each option may be used once, more
than once or not at all.

12. A 56 year old man presents with symptoms of nasal pain, anosmia and rhinorrhea. He has
been well until recently and has worked as a wood carver for many years.

Ethmoid sinus cancer

Theme from September 2012 Exam


Theme from September 2013 Exam
Nasopharyngeal cancer is strongly associated with wood work. Most cases require an
occupational exposure of greater than 10 years and are adenocarcinomas on histology.
Most cases are ethmoidal in origin (Hadfield E. Ann R Coll Surg Engl. 1970 June; 46(6):
301319)

13. A 32 year old female presents with recurrent episodes of rhinorrhoea, the discharge is
watery. She has a medical history of asthma and intolerance of aspirin. On examination
she has multiple soft, semi- transparent polyps within her nasal cavity.

You answered Ethmoid sinus cancer

The correct answer is Nasal polyps

The combination of nasal polyps and atopy is well described. Some cases will respond
favourably to systemic steroids and avoid surgery.

14. A child is brought to casualty complaining of a headache and a sensation of pressure


between the eyes. On examination she is febrile with a smooth swelling overlying the
superomedial aspect of the right eye. The eye is uncomfortable and there is a purulent
discharge from the inner canthus.
You answered Ethmoid sinus cancer

The correct answer is Ethmoidal sinusitis

Ethmoidal sinusitis may spread to the periorbital tissues resulting in periorbital cellulitis.
The superomedial distribution makes a maxillary sinusitis less likely.

Please rate this question:

Discuss and give feedback


Next question

Diseases of nose and sinuses

Benign Tumours

 Simple papillomas may be an incidental finding or present with obstructive symptoms.


Excision under general anaesthesia is sufficient management.
 Transitional cell papillomas may be more extensive and produce obstructive symptoms.
Erosion of local structures is a recognised complication. These lesions may rarely undergo
malignant transformation and therefore careful and complete excision is required, some
cases may require partial or total maxillectomy.
 Pleomorphic adenomas of the maxillary sinuses are reported but are extremely rare, their
symptoms typically include nasal obstruction and pain if the sinus is obstructed. Treatment is
by complete surgical excision, the diagnosis is not infrequently made post operatively.
 Benign osteomas may develop in the paranasal sinuses, the frontal sinus is the most
frequent location of such lesions. Symptoms include; pain, rhinorrhoea and anosmia. Most
osteomas may be observed if asymptomatic, sphenoid osteomas should be resected soon
after diagnosis as enlargement may compromise visual fields. Many sinus osteomas can
now be resected endoscopically, complete surgical resection is required.
 Nasal polyps are benign lesions of the ethmoid sinus mucosa. Many patients may also have
asthma, cystic fibrosis and a sensitivity to aspirin. Symptoms include watery rhinorrhoea,
infection and anosmia. The polyps are usually a semi transparent grey mass. They are rare
in childhood. Treatment is either with systemic steroids or surgical resection. The latter
should be combined with antral washout. Low dose, nasal, steroid drops may reduce the risk
of recurrence.

Malignant disease

 Malignancies encountered in the nose and paranasal sinuses include; adenoid cystic
carcinoma, squamous cell carcinoma and adenocarcinoma.
 Adenocarcinoma of the paranasal sinuses and nasopharynx is strongly linked to exposure to
hard wood dust (after >10 years exposure).
 Adenoid cystic carcinoma usually originate in the smaller salivary glands.
 The majority of cancers (50%) arise from the lateral nasal wall, a smaller number (33%) arise
from the maxillary antrum, ethmoid and sphenoid cancers comprise only 7%.
 Signs of malignancy on clinical examination include loose teeth, cranial nerve palsies and
lymphadenopathy.
 Nasopharyngeal cancers are most common in individuals presenting from China and Asia
and are linked to viral infection with Epstein Barr Virus. Radiotherapy and chemotherapy are
the most commonly used modalities.

Maxillary sinusitis

 Common symptoms include post nasal discharge, pain, headache and toothache.
 Imaging may show a fluid level in the antrum.
 Common organisms include Haemophilus influenzae or Streptococcus pneumoniae.
 Treatment with antral lavage may facilitate diagnosis and relieve symptoms. Antimicrobial
therapy has to be continued for long periods. Antrostomy may be needed.

Frontoethmoidal sinusitis

 Usually presents with frontal headache, nasal obstruction and altered sense of smell.
 Inflammation may progress to involve periorbital tissues. Ocular symptoms may occur and
secondary CNS involvement brought about by infection entering via emissary veins.
 CT scanning is the imaging modality of choice. Early cases may be managed with antibiotics.
More severe cases usually require surgical drainage.

Next question
Which of the following statements relating to branchial cysts is untrue?

The greater auricular nerve may be divided during excision

They typically occur in young adults

They move upwards on swallowing

They are rare over the age of 40 years

They are usually located in the anterior triangle of the neck

Nerves at risk during branchial cyst excision: Mandibular branch of facial nerve, greater auricular
nerve and accessory nerve.

They do not move on swallowing. They should be diagnosed with caution in those aged >40 years,
as lumps in this age group may in fact be metastatic disease from oropharyngeal cancer.

Please rate this question:

Discuss and give feedback

Next question

Neck lumps

The table below gives characteristic exam question features for conditions causing neck lumps:

Reactive By far the most common cause of neck swellings. There may be a history of
lymphadenopathy local infection or a generalised viral illness
Lymphoma Rubbery, painless lymphadenopathy
The phenomenon of pain whilst drinking alcohol is very uncommon
There may be associated night sweats and splenomegaly

Thyroid swelling May be hypo-, eu- or hyperthyroid symptomatically


Moves upwards on swallowing

Thyroglossal cyst More common in patients < 20 years old


Usually midline, between the isthmus of the thyroid and the hyoid bone
Moves upwards with protrusion of the tongue
May be painful if infected

Pharyngeal pouch More common in older men


Represents a posteromedial herniation between thyropharyngeus and
cricopharyngeus muscles
Usually not seen, but if large then a midline lump in the neck that gurgles on
palpation
Typical symptoms are dysphagia, regurgitation, aspiration and chronic cough

Cystic hygroma A congenital lymphatic lesion (lymphangioma) typically found in the neck,
classically on the left side
Most are evident at birth, around 90% present before 2 years of age

Branchial cyst An oval, mobile cystic mass that develops between the sternocleidomastoid
muscle and the pharynx
Develop due to failure of obliteration of the second branchial cleft in
embryonic development
Usually present in early adulthood

Cervical rib More common in adult females


Around 10% develop thoracic outlet syndrome

Carotid aneurysm Pulsatile lateral neck mass which doesn't move on swallowing

Next question
A 22 year old female attends clinic after noticing a painless neck lump. On examination she is noted
to have bilateral thyroid masses and multicentric nodes near the base of the thyroid. Her corrected
Ca is 2.18. What is the most likely diagnosis?

Sporadic medullary carcinoma of the thyroid

Medullary carcinoma of the thyroid associated with multiple endocrine neoplasia

Follicular thyroid carcinoma

Anaplastic thyroid carcinoma

Toxic nodular goitre

Medullary thyroid cancer is a tumour of the parafollicular cells of the thyroid. Less than 10% of
thyroid cancers are of this type. Patients typically present in children or young adults. Diarrhoea
occurs in 30% of cases. Toxic nodular goitre are very rare. In sporadic medullary thyroid cancer,
patients typically present with a unilateral solitary nodule and it tends to spread early to the neck
lymph nodes. In association with multiple endocrine neoplasia (MEN) syndromes, medullary thyroid
cancers are always bilateral and multicentric. It may be the presenting feature in MEN 2a and 2b;
almost all MEN 2a patients develop medullary thyroid carcinoma.

Please rate this question:

Discuss and give feedback


Next question

Thyroid malignancy

Papillary carcinoma

 Commonest sub-type
 Accurately diagnosed on fine needle aspiration cytology
 Histologically, they may demonstrate psammoma bodies (areas of calcification) and so
called 'orphan Annie' nuclei
 They typically metastasise via the lymphatics and thus laterally located apparently ectopic
thyroid tissue is usually a metastasis from a well differentiated papillary carcinoma

Follicular carcinoma
 Are less common than papillary lesions
 Like papillary tumours, they may present as a discrete nodule. Although they appear to be
well encapsulated macroscopically there is invasion on microscopic evaluation
 Lymph node metastases are uncommon and these tumours tend to spread
haematogenously. This translates into a higher mortality rate
 Follicular lesions cannot be accurately diagnosed on fine needle aspiration cytology and thus
all follicular FNA's (THY 3f) will require at least a hemi thyroidectomy

Anaplastic carcinoma

 Less common and tend to occur in elderly females


 Disease is usually advanced at presentation and often only palliative decompression and
radiotherapy can be offered.

Medullary carcinoma

 These are tumours of the parafollicular cells ( C Cells) and are of neural crest origin.
 The serum calcitonin may be elevated which is of use when monitoring for recurrence.
 They may be familial and occur as part of the MEN -2A disease spectrum.
 Spread may be either lymphatic or haematogenous and as these tumours are not derived
primarily from thyroid cells they are not responsive to radioiodine.

Lymphoma

 These respond well to radiotherapy


 Radical surgery is unnecessary once the disease has been diagnosed on biopsy material.
Such biopsy material is not generated by an FNA and thus a core biopsy has to be obtained
(with care!).

Next question
Theme: Parotid gland disorders

A. Sialectasis
B. Pleomorphic adenoma
C. Bacterial parotitis
D. Viral parotitis
E. Sjogren's syndrome
F. Adenoid cystic carcinoma
G. Mucoepidermoid carcinoma
H. Warthins tumour

Please select the most likely diagnosis for the scenario given. Each option may be used once, more
than once or not at all.

17. A 40 year old lady with longstanding rheumatoid arthritis presents with a dry mouth and
parotid gland swelling. Her symptoms transiently improved following administration of a
course of steroids for her rheumatoid disease.

You answered Sialectasis

The correct answer is Sjogren's syndrome

Theme from April 2012 Exam


Sjogrens syndrome is linked to other autoimmune conditions such as rheumatoid disease.
A transient improvement may be seen with steroid administration. Most patients are
treated with artificial saliva.

18. A 77 year old lady presents with facial pain and on examination is found to have clinical
evidence of a facial nerve palsy and a parotid mass. Following surgical excision of the
lesion the histological report comments on extensive perineural invasion.

You answered Sialectasis

The correct answer is Adenoid cystic carcinoma

Extensive perineural invasion is most commonly seen in patients with adenoid cystic
carcinoma. Both Warthins tumours and mucoepidermoid carcinoma rarely show such
marked perineural infiltration.

19. An 18 year old boy presents with pancreatitis. He has bilateral painful parotid
enlargement.

You answered Sialectasis


The correct answer is Viral parotitis

In a young adult with parotid swelling and pancreatitis/orchitis/reduced


hearing/meningoencephalitis suspect mumps.

Please rate this question:

Discuss and give feedback


Next question

Parotid gland clinical

Benign neoplasms
Up to 80% of all salivary gland tumours occur in the parotid gland and up to 80% of these are
benign. There is no consistent correlation between the rate of growth and the malignant potential of
the lesion. However, benign tumours should not invade structures such as the facial nerve.
With the exception of Warthins tumours, they are commoner in women than men. The median age of
developing a lesion is in the 5th decade of life.

Benign tumour types


Tumour type Features

Benign pleomorphic adenoma or Most common parotid neoplasm (80%)


benign mixed tumor Proliferation of epithelial and myoepithelial cells of the ducts
and an increase in stromal components
Slow growing, lobular, and not well encapsulated
Recurrence rate of 1-5% with appropriate excision
(parotidectomy)
Recurrence possibly secondary to capsular disruption during
surgery
Malignant degeneration occurring in 2-10% of adenomas
observed for long periods, with carcinoma ex-pleomorphic
adenoma occurring most frequently as adenocarcinoma

Warthin tumor (papillary Second most common benign parotid tumor (5%)
cystadenoma lymphoma or Most common bilateral benign neoplasm of the parotid
adenolymphoma) Marked male as compared to female predominance
Occurs later in life (sixth and seventh decades)
Presents as a lymphocytic infiltrate and cystic epithelial
proliferation
May represent heterotopic salivary gland epithelial tissue
trapped within intraparotid lymph nodes
Tumour type Features

Incidence of bilaterality and multicentricity of 10%


Malignant transformation rare (almost unheard of)

Monomorphic adenoma Account for less than 5% of tumours


Slow growing
Consist of only one morphological cell type (hence term
mono)
Include; basal cell adenoma, canalicular adenoma,
oncocytoma, myoepitheliomas

Haemangioma Should be considered in the differential of a parotid mass in a


child
Accounts for 90% of parotid tumours in children less than 1
year of age
Hypervascular on imaging
Spontaneous regression may occur and malignant
transformation is almost unheard of

Malignant salivary gland tumours


Types of malignancy

Mucoepidermoid 30% of all parotid malignancies


carcinoma Usually low potential for local invasiveness and metastasis (depends
mainly on grade)

Adenoid cystic Unpredictable growth pattern


carcinoma Tendency for perineural spread
Nerve growth may display skip lesions resulting in incomplete excision
Distant metastasis more common (visceral rather than nodal spread)
5 year survival 35%

Mixed tumours Often a malignancy occurring in a previously benign parotid lesion

Acinic cell carcinoma Intermediate grade malignancy


May show perineural invasion
Low potential for distant metastasis
5 year survival 80%

Adenocarcinoma Develops from secretory portion of gland


Risk of regional nodal and distant metastasis
5 year survival depends upon stage at presentation, may be up to 75%
with small lesions with no nodal involvement

Lymphoma Large rubbery lesion, may occur in association with Warthins tumours
Diagnosis should be based on regional nodal biopsy rather than parotid
resection
Treatment is with chemotherapy (and radiotherapy)

Diagnostic evaluation

 Plain x-rays may be used to exclude calculi


 Sialography may be used to delineate ductal anatomy
 FNAC is used in most cases
 Superficial parotidectomy may be either diagnostic of therapeutic depending upon the nature
of the lesion
 Where malignancy is suspected the primary approach should be definitive resection rather
than excisional biopsy
 CT/ MRI may be used in cases of malignancy for staging primary disease

Treatment
For nearly all lesions this consists of surgical resection, for benign disease this will usually consist of
a superficial parotidectomy. For malignant disease a radical or extended radical parotidectomy is
performed. The facial nerve is included in the resection if involved. The need for neck dissection is
determined by the potential for nodal involvement.

Other parotid disorders


HIV infection

 Lymphoepithelial cysts associated with HIV occur almost exclusively in the parotid
 Typically presents as bilateral, multicystic, symmetrical swelling
 Risk of malignant transformation is low and management usually conservative

Sjogren syndrome

 Autoimmune disorder characterised by parotid enlargement, xerostomia and


keratoconjunctivitis sicca
 90% of cases occur in females
 Second most common connective tissue disorder
 Bilateral, non tender enlargement of the gland is usual
 Histologically, the usual findings are of a lymphocytic infiltrate in acinar units and
epimyoepithelial islands surrounded by lymphoid stroma
 Treatment is supportive
 There is an increased risk of subsequent lymphoma

Sarcoid

 Parotid involvement occurs in 6% of patients with sarcoid


 Bilateral in most cases
 Gland is not tender
 Xerostomia may occur
 Management of isolated parotid disease is usually conservative

Next question
Theme: Neck lumps

A. Dermoid cyst

B. Thyroglossal cyst

C. Sjogren's syndrome

D. Mikulicz's syndrome

E. Pleomorphic adenoma of the parotid

F. Carcinoma of the parotid

G. Cystic hygroma

H. Branchial cyst

I. Pharyngeal pouch

Please select the most likely diagnosis for the scenario given. Each option may be used once, more
than once or not at all.

20. A dentist treating a women with rheumatoid arthritis for recurrent episodes of dental sepsis
notices that both parotid and submandibular glands are symmetrically enlarged.

You answered Dermoid cyst

The correct answer is Sjogren's syndrome

Sjogren's is associated with autoimmune disorders. Mikulicz's is similar but there is no sicca or
arthritis.

21. A patient presents with a facial nerve palsy. This occurred following repeat excision of a facial
lump. The histology report remarks on the biphasic appearance of the lesion and mucinous
connective tissue.

You answered Dermoid cyst

The correct answer is Pleomorphic adenoma of the parotid


The histological features are as described with a classic biphasic (mixed stromal and epithelial
elements), although benign local recurrence can complicate incomplete excision. As this is a
benign lesion direct extension into the facial nerve is unlikely to occur. Facial nerve injury can
happen during repeat parotid surgery.

22. A patient is recovering from a Sistrunk's procedure, what lesion was treated with this operation?

You answered Dermoid cyst

The correct answer is Thyroglossal cyst

This is the procedure for excision of the cyst and its associated track. Excision must be complete
and thus a small segment of the hyoid bone is removed to gain access to the upper part of the
cyst tract.

Please rate this question:

Discuss and give feedback

Next question

Neck lumps

The table below gives characteristic exam question features for conditions causing neck lumps:

Reactive By far the most common cause of neck swellings. There may be a history of
lymphadenopathy local infection or a generalised viral illness

Lymphoma Rubbery, painless lymphadenopathy


The phenomenon of pain whilst drinking alcohol is very uncommon
There may be associated night sweats and splenomegaly
Thyroid swelling May be hypo-, eu- or hyperthyroid symptomatically
Moves upwards on swallowing

Thyroglossal cyst More common in patients < 20 years old


Usually midline, between the isthmus of the thyroid and the hyoid bone
Moves upwards with protrusion of the tongue
May be painful if infected

Pharyngeal pouch More common in older men


Represents a posteromedial herniation between thyropharyngeus and
cricopharyngeus muscles
Usually not seen, but if large then a midline lump in the neck that gurgles on
palpation
Typical symptoms are dysphagia, regurgitation, aspiration and chronic cough

Cystic hygroma A congenital lymphatic lesion (lymphangioma) typically found in the neck,
classically on the left side
Most are evident at birth, around 90% present before 2 years of age

Branchial cyst An oval, mobile cystic mass that develops between the sternocleidomastoid
muscle and the pharynx
Develop due to failure of obliteration of the second branchial cleft in
embryonic development
Usually present in early adulthood

Cervical rib More common in adult females


Around 10% develop thoracic outlet syndrome

Carotid aneurysm Pulsatile lateral neck mass which doesn't move on swallowing

Next question
A 10 year old girl presents with epistaxis. From which of the following regions is the bleeding most
likely to originate?

Posterior nasal space

Alar rim

Kiesselbach's plexus

Cribriform plate

None of the above

Kiesselbachs plexus has an arterial supply derived from both the internal and external carotid
arteries and is the commonest area for bleeding in idiopathic epistaxis.
Please rate this question:

Discuss and give feedback


Next question

Epistaxis

Usually trivial and insignificant but severe haemorrhage may compromise airway and pose a risk to
life.

Anatomy:

Arterial supply

 From internal and external carotid


 An arterial plexus exists at Little's area and is the source of bleeding in 90% cases
 Major arterial supply is from the sphenopalatine and greater palatine arteries (branches of
the maxillary artery)
 The facial artery supplies the more anterior aspect of the nose
 Ethmoidal arteries are branches of the ophthalmic artery. They supply the posterosuperior
nasal cavity

Venous drainage follows the arterial pattern


Classification

 Primary idiopathic epistaxis accounts for 75% of all cases


 Secondary cases arise as a result of events such as anticoagulants, trauma and
coagulopathy
 Classification into anterior and posterior epistaxis may help to locate the source and
becomes more important when invasive treatment is required

Management

 Resuscitate if required
 Subject should sit upright and pinch nose firmly
 Nasal cavity should be examined using a headlight
 Simple anterior epistaxis may be managed using silver nitrate cautery. If difficult to manage
then custom manufactured packs may be inserted
 Posterior packing or tamponade may be achieved by passing a balloon tamponade device
and inflating it. This is indicated where anterior packing alone has failed to achieve
haemostasis.
 Post nasal pack patients should receive antibiotics
 Failure of these methods will require more invasive therapy. Where a vascular radiology
suite is available, consideration may be given to angiographic techniques. Direct ligation of
the nasal arterial supply may also be undertaken. Of the arterial ligation techniques
available, the endo nasal sphenopalatine arterial ligation procedure is most popular.

Next question
A 56 year old man presents with a painless swelling in the upper part of the anterior triangle of his
neck. On examination a mass lesion involving the sub mandibular gland is identified. On CT
scanning this is shown to be a solid lesion. There is no regional lymphadenopathy. Two fine needle
aspirates have failed to be diagnostic. Which of the following is the most appropriate management
option?

Sub mandibular gland excision

Incisional biopsy of the mass

Manage conservatively and repeat the CT scan in 6 months

Sub mandibular gland excision and radical neck dissection

Diagnostic excision of the superficial lobe of the submandibular gland

There is a 50% risk that this lesion is malignant (in some series up to 70%). Therefore the gland
should be excised entirely. At this stage a radical neck dissection is not justified.
Please rate this question:

Discuss and give feedback


Next question

Submandibular glands- disease

Physiology
The submandibular glands secrete approximately 800- 1000ml saliva per day. They typically
produce mixed seromucinous secretions. When parasympathetic activity is dominant; the secretions
will be more serous. The parasympathetic fibres are derived from the chorda tympani nerves and the
submandibular ganglion. Sensory fibres are conveyed by the lingual branch of the mandibular nerve.

Sialolithiasis

 80% of all salivary gland calculi occur in the submandibular gland


 70% of the these calculi are radio-opaque
 Stones are usually composed of calcium phosphate or calcium carbonate
 Patients typically develop colicky pain and post prandial swelling of the gland
 Investigation involves sialography to demonstrate the site of obstruction and associated
other stones
 Stones impacted in the distal aspect of Whartons duct may be removed orally, other stones
and chronic inflammation will usually require gland excision
Sialadenitis

 Usually occurs as a result of Staphylococcus aureus infection


 Pus may be seen leaking from the duct, erythema may also be noted
 Development of a sub mandibular abscess is a serious complication as it may spread
through the other deep fascial spaces and occlude the airway

Submandibular tumours

 Only 8% of salivary gland tumours affect the sub mandibular gland


 Of these 50% are malignant (usually adenoid cystic carcinoma)
 Diagnosis usually involves fine needle aspiration cytology
 Imaging is with CT and MRI
 In view of the high prevalence of malignancy, all masses of the submandibular glands should
generally be excised.

Next question
A 36-year-old woman who presented with a goitre is diagnosed with autoimmune thyroiditis. Which
one of the following types of thyroid cancer is she predisposed to developing?

Anaplastic

Lymphoma

Medullary

Follicular

None of the above

Hashimoto's thyroiditis is associated with thyroid lymphoma

Rarely, a patient with Hashimotos thyroiditis may develop a lymphoma of the thyroid gland. The
exact aetiology of thyroid gland lymphoma is unknown. Hashimotos thyroiditis is a definite risk factor.
It is possible that the lymphoma may represent the expansion of a clone of immortalised
intrathyroidal lymphocytes. Lymphoma of the gland is characterised by rapid thyroid growth despite
T4 therapy. Whilst adenocarcinoma of the thyroid may occur in association with thyroiditis there are
no studies, to date, showing a correlation between these two conditions. The commonest sequelae
of thyroiditis is hypothyroidism.
Please rate this question:

Discuss and give feedback


Next question

Thyroiditis

Sub acute thyroiditis


Subacute thyroiditis (also known as De Quervain's thyroiditis) is thought to occur following viral
infection and typically presents with hyperthyroidism

Features

 Hyperthyroidism
 Painful goitre
 Raised ESR
 Globally reduced uptake on iodine-131 scan
Management

 Usually self-limiting - most patients do not require treatment


 Thyroid pain may respond to aspirin or other NSAIDs
 In more severe cases steroids are used, particularly if hypothyroidism develops

Hashimotos thyroiditis
Hashimotos thyroiditis is an immunological disorder in which lymphocytes become sensitised to
thyroidal antigens. The three most important antibodies include; thyroglobulin, TPO and TSH-R.
During the early phase of Hashimotos the the thyroglobulin antibody is markedly elevated and then
declines.

Features

 Goitre and either euthyroid or mild hypothyroidism


 Progressive hypothyroidism (and associated symptoms)

Management

 During the hyperthyroid phase of illness beta blockers may manage symptoms
 As hypothyroidism develops patients may require thyroxine

Next question
An elderly diabetic male presents with a severe deep seated otalgia and a facial nerve palsy, he has
completed a course of amoxycillin with no benefit. What is the most likely diagnosis?

Malignant otitis externa

Otosclerosis

Acoustic neuroma

Meniers disease

Viral illness

A combination of severe otalgia and facial nerve palsy in a diabetic should raise suspicion of
malignant otitis externa. This is a condition caused by pseudomonas. It commences as otitis externa
and then progresses to involve the temporal bone. Spread of the disease outside the external
auditory canal occurs through the fissures of Santorini and the osseocartilaginous junction.
Please rate this question:

Discuss and give feedback


Next question

Disorders affecting the ear

Otitis externa
Variant Cause Features Treatment

Acute Boil in external auditory meatus Acute pain on Ear packs may be used
otitis moving the pinna Topical antibiotics
externa Conductive hearing Operative debridement
loss if lesion is large may be needed in severe
When rupture occurs cases
pus will flow from
ear

Chronic Chronic combined infection in Chronic discharge Cleansing of the external


otitis the external auditory meatus from affected ear, ear and treatment with
Variant Cause Features Treatment

externa usually combined hearing loss and antifungal and


staphylococcal and fungal severe pain rare antibacterial ear drops
infection

Otitis media
Variant Cause Features Treatment

Acute Viral induced middle Most common in Antibiotics (usually


suppurative ear effusions children and rare in amoxycillin)
otitis media secondary to eustacian adults
tube dysfunction May present with
symptoms elsewhere
(e.g. vomiting) in
children
Severe pain and
sometimes fever
May present with
discharge is tympanic
rupture occurs

Chronic May occur with or Those without Simple pars tensa


suppurative without cholesteatoma cholesteatoma may perforations may be managed
otitis media Those without complain of non operatively or a
cholesteatoma have a intermittent discharge myringoplasty considered if
perforation of the pars (non offensive) symptoms troublesome.
tensa Those with Pars flaccida perforations
Those with cholesteatoma have will usually require a radical
cholesteatoma have a impaired hearing and mastoidectomy
perforation of the pars foul smelling discharge
flaccida

Otosclerosis

 Progressive conductive deafness


 Secondary to fixation of the stapes in the oval window
 Treatment is with stapedectomy and insertion of a prosthesis
Acoustic neuroma

 Symptoms of gradually progressive unilateral perceptive deafness and tinnitus


 Involvement of the vestibular nerve may cause vertigo
 Extension to involve the facial nerve may cause weakness and then paralysis.

Pre auricular sinus

 Common congenital condition in which an epithelial defect forms around the external ear
 Small sinuses require no treatment
 Deeper sinuses may become blocked and develop episodes of infection, they may be closely
related to the facial nerve and are challenging to excise

Next question
Which of the following does not cause parotid enlargement?

Liver cirrhosis

Myxoedema

Amiodarone

Tuberculosis

Sjogrens syndrome

Drugs commonly implicated in parotid gland enlargement include:


Thiouracil, isoprenaline, phenylbutazone, high oestrogen dose contraceptive pills.
Please rate this question:

Discuss and give feedback


Next question

Parotid gland clinical

Benign neoplasms
Up to 80% of all salivary gland tumours occur in the parotid gland and up to 80% of these are
benign. There is no consistent correlation between the rate of growth and the malignant potential of
the lesion. However, benign tumours should not invade structures such as the facial nerve.
With the exception of Warthins tumours, they are commoner in women than men. The median age of
developing a lesion is in the 5th decade of life.

Benign tumour types


Tumour type Features

Benign pleomorphic adenoma or Most common parotid neoplasm (80%)


benign mixed tumor Proliferation of epithelial and myoepithelial cells of the ducts
and an increase in stromal components
Slow growing, lobular, and not well encapsulated
Recurrence rate of 1-5% with appropriate excision
(parotidectomy)
Recurrence possibly secondary to capsular disruption during
surgery
Tumour type Features

Malignant degeneration occurring in 2-10% of adenomas


observed for long periods, with carcinoma ex-pleomorphic
adenoma occurring most frequently as adenocarcinoma

Warthin tumor (papillary Second most common benign parotid tumor (5%)
cystadenoma lymphoma or Most common bilateral benign neoplasm of the parotid
adenolymphoma) Marked male as compared to female predominance
Occurs later in life (sixth and seventh decades)
Presents as a lymphocytic infiltrate and cystic epithelial
proliferation
May represent heterotopic salivary gland epithelial tissue
trapped within intraparotid lymph nodes
Incidence of bilaterality and multicentricity of 10%
Malignant transformation rare (almost unheard of)

Monomorphic adenoma Account for less than 5% of tumours


Slow growing
Consist of only one morphological cell type (hence term
mono)
Include; basal cell adenoma, canalicular adenoma,
oncocytoma, myoepitheliomas

Haemangioma Should be considered in the differential of a parotid mass in a


child
Accounts for 90% of parotid tumours in children less than 1
year of age
Hypervascular on imaging
Spontaneous regression may occur and malignant
transformation is almost unheard of

Malignant salivary gland tumours


Types of malignancy

Mucoepidermoid 30% of all parotid malignancies


carcinoma Usually low potential for local invasiveness and metastasis (depends
mainly on grade)
Adenoid cystic Unpredictable growth pattern
carcinoma Tendency for perineural spread
Nerve growth may display skip lesions resulting in incomplete excision
Distant metastasis more common (visceral rather than nodal spread)
5 year survival 35%

Mixed tumours Often a malignancy occurring in a previously benign parotid lesion

Acinic cell carcinoma Intermediate grade malignancy


May show perineural invasion
Low potential for distant metastasis
5 year survival 80%

Adenocarcinoma Develops from secretory portion of gland


Risk of regional nodal and distant metastasis
5 year survival depends upon stage at presentation, may be up to 75%
with small lesions with no nodal involvement

Lymphoma Large rubbery lesion, may occur in association with Warthins tumours
Diagnosis should be based on regional nodal biopsy rather than parotid
resection
Treatment is with chemotherapy (and radiotherapy)

Diagnostic evaluation

 Plain x-rays may be used to exclude calculi


 Sialography may be used to delineate ductal anatomy
 FNAC is used in most cases
 Superficial parotidectomy may be either diagnostic of therapeutic depending upon the nature
of the lesion
 Where malignancy is suspected the primary approach should be definitive resection rather
than excisional biopsy
 CT/ MRI may be used in cases of malignancy for staging primary disease

Treatment
For nearly all lesions this consists of surgical resection, for benign disease this will usually consist of
a superficial parotidectomy. For malignant disease a radical or extended radical parotidectomy is
performed. The facial nerve is included in the resection if involved. The need for neck dissection is
determined by the potential for nodal involvement.

Other parotid disorders


HIV infection

 Lymphoepithelial cysts associated with HIV occur almost exclusively in the parotid
 Typically presents as bilateral, multicystic, symmetrical swelling
 Risk of malignant transformation is low and management usually conservative

Sjogren syndrome

 Autoimmune disorder characterised by parotid enlargement, xerostomia and


keratoconjunctivitis sicca
 90% of cases occur in females
 Second most common connective tissue disorder
 Bilateral, non tender enlargement of the gland is usual
 Histologically, the usual findings are of a lymphocytic infiltrate in acinar units and
epimyoepithelial islands surrounded by lymphoid stroma
 Treatment is supportive
 There is an increased risk of subsequent lymphoma

Sarcoid

 Parotid involvement occurs in 6% of patients with sarcoid


 Bilateral in most cases
 Gland is not tender
 Xerostomia may occur
 Management of isolated parotid disease is usually conservative

Next question
At which of the following time frames is secondary haemorrhage most likely to occur following
tonsillectomy?

Between 5 and 10 days after surgery

During the first 6 hours after surgery

Between 6 and 12 hours after surgery

Upon resumption of normal feeding

Between 2 and 3 days post operatively

Haemorrhage in the first 6 hours after surgery is termed reactionary haemorrhage. Feeding does not
increase the risk and may actually lower the risks of infection developing.

Please rate this question:

Discuss and give feedback

Next question

Secondary haemorrhage after tonsillectomy

Haemorrhage is a feared complication following tonsillectomy. Primary, or reactionary haemorrhage


most commonly occurs in the first 6-8 hours following surgery. It is managed by immediate return to
theatre.
Secondary haemorrhage occurs between 5 and 10 days after surgery, it is often associated with a
wound infection. Treatment is usually with admission and antibiotics. Severe bleeding may require
surgery. Secondary haemorrhage occurs in 3% of all tonsillectomies.

Next question
A 8 year old boy with recurrent attacks of otitis media is suspected of developing a glue ear. If his
sound conduction is tested, which of the following is most consistent with a unilateral middle ear
effusion?

Negative Rinne's test on the ipsilateral side

Positive Rinne's test on the ipsilateral side

Positive Webers and Rinnes tests on the ipsilateral side

Positive Rinne's test on the contralateral side

Negative Webers test only on the contralateral side

Theme from 2008 Exam


Rinne's test will localise to the affected side (i.e. it is negative in conductive deafness). In a positive
Rinne's test sound heard by air conduction is better than that conveyed by bone conduction.
Reduction of both air and bone conduction in equal measure is a feature of sensorineural hearing
loss.

Please rate this question:

Discuss and give feedback

Next question

Hearing Loss

Hearing loss may be conductive or sensorineural. To determine which is present patients will often
require a formal assessment with pure tone audiometry. In the clinical setting Webers and Rinnes
tests may be helpful in categorising various types of hearing loss.

Webers and Rinnes Tests


In a normal patient, the Weber tuning fork sound is heard equally loud in both ears with no one ear
hearing the sound louder than the other. A patient with symmetrical hearing loss will hear the Weber
tuning fork sound equally well with diagnostic utility only in asymmetric (one-sided) hearing losses. In
a patient with asymmetrical hearing loss, the Weber tuning fork sound is heard louder in one ear
versus the other. This clinical finding should be confirmed by repeating the procedure and having the
patient occlude one ear with a finger; the sound should be heard best in the occluded ear.

Weber without
Rinne Test lateralisation Weber lateralises to left Weber lateralises to right

Both ears Normal Sensorineural loss on right Sensorineural loss on left


Air>Bone

Left Bone > Conductive loss on left Combined loss on left


Air

Right Bone> Combined loss on right Conductive loss on right


Air

Both Bone > Combined loss on right and Combined loss on left and
Air conductive on left conductive on right

Next question
Theme: Head and neck lesions

A. Dermoid cyst

B. Thyroglossal cyst

C. Sjogren's syndrome

D. Mikulicz's syndrome

E. Pleomorphic adenoma of the parotid

F. Carcinoma of the parotid

G. Cystic hygroma

H. Branchial cyst

I. Pharyngeal pouch

Please select the most likely diagnosis for the scenario given. Each option may be used once, more
than once or not at all.

30. The parents of a 2 year old child are concerned after he develops a lump in his neck. On
examination there is a swelling in the subcutaneous tissue of the posterior triangle which
transilluminates.

You answered Dermoid cyst

The correct answer is Cystic hygroma

This is a classical description. Collection of dilated lymphatic sacs which are fluctuant and
brilliantly transilluminable. Recurrence is common.

31. A 40 year old female presents with a painless neck lump. There is a mass noted beneath the
sternocleidomastoid muscle. There is a long history and somewhat unkindly her husband
remarked on her rather noticeable halitosis.

You answered Dermoid cyst


The correct answer is Pharyngeal pouch

Usual history of regurgitation of undigested food or coughing at night. Associated with halitosis
and throat infections.

32. A 32 year old lady is referred to the clinic with recurrent infections and abscesses in the neck. On
examination she has a midline defect with an overlying scab, which moves upwards on tongue
protrusion.

You answered Dermoid cyst

The correct answer is Thyroglossal cyst

Thyroglossal cyst is always located in the midline as it is this route that the thyroid takes during its
embryological descent. Its connection with the foramen caecum means it will move on tongue
protrusion.

Similar theme question in September 2011 exam

Please rate this question:

Discuss and give feedback

Neck lumps

The table below gives characteristic exam question features for conditions causing neck lumps:

Reactive By far the most common cause of neck swellings. There may be a history of
lymphadenopathy local infection or a generalised viral illness

Lymphoma Rubbery, painless lymphadenopathy


The phenomenon of pain whilst drinking alcohol is very uncommon
There may be associated night sweats and splenomegaly

Thyroid swelling May be hypo-, eu- or hyperthyroid symptomatically


Moves upwards on swallowing

Thyroglossal cyst More common in patients < 20 years old


Usually midline, between the isthmus of the thyroid and the hyoid bone
Moves upwards with protrusion of the tongue
May be painful if infected

Pharyngeal pouch More common in older men


Represents a posteromedial herniation between thyropharyngeus and
cricopharyngeus muscles
Usually not seen, but if large then a midline lump in the neck that gurgles on
palpation
Typical symptoms are dysphagia, regurgitation, aspiration and chronic cough

Cystic hygroma A congenital lymphatic lesion (lymphangioma) typically found in the neck,
classically on the left side
Most are evident at birth, around 90% present before 2 years of age

Branchial cyst An oval, mobile cystic mass that develops between the sternocleidomastoid
muscle and the pharynx
Develop due to failure of obliteration of the second branchial cleft in
embryonic development
Usually present in early adulthood

Cervical rib More common in adult females


Around 10% develop thoracic outlet syndrome

Carotid aneurysm Pulsatile lateral neck mass which doesn't move on swallowing
Theme: Management of biliary disease

A. Acute laparoscopic cholecystectomy


B. Delayed laparoscopic cholecystectomy
C. Percutaneous cholecystostomy
D. Elective cholecystectomy and intra operative cholangiogram
E. Endoscopic retrograde cholangiopancreatography
F. Choledochoduodenostomy
G. Bile duct excision and hepatico-jejunostomy
H. Operative cholecystostomy

For each scenario please select the most appropriate management option. Each option may be used
once, more than once or not at all.

1. A 72 year old lady underwent an open cholecystectomy 12 years previously. She has been
admitted since with 2 episodes of cholangitis and stones were retrieved at ERCP. She has
just recovered from a further episode of sepsis and MRCP has shown further biliary stones.

You answered Acute laparoscopic cholecystectomy

The correct answer is Choledochoduodenostomy

A patient with long standing common bile duct stones is at risk of developing duct fibrosis
and ductal disproportion. This can result in impaired biliary drainage. Not only may further
stones be formed in the bile that is present, but because of the ductal disproportion the
tendency will be for the stones to accumulate (rather than pass spontaneously, as would
usually be the case post ERCP and sphincterotomy). A biliary bypass procedure is the
standard method dealing with this and a choledochoduodenstomy is one procedure that can
be used.

2. A 26 year old women is admitted with acute cholecystitis of 24 hours duration. LFT's are
normal and Ultrasound shows a thick walled gallbladder containing stones.

Acute laparoscopic cholecystectomy

This is an ideal case for an acute cholecystectomy, provided that surgery can be undertaken
promptly. After 48 -72 hours the patient should receive parenteral antibiotics and delayed
cholecystectomy performed.

3. A 32 year old lady is seen in the outpatients. She has had multiple episodes of biliary colic
and ultrasound shows thin walled gallbladder with multiple calculi. Her ALT is slightly
raised but other parameters are normal.
You answered Acute laparoscopic cholecystectomy

The correct answer is Elective cholecystectomy and intra operative cholangiogram

The easiest option is to perform an intraoperative cholangiogram. It is unlikely to reveal any


stones. If is does then either laparoscopic bile duct exploration or urgent ERCP can be
performed. An MRCP pre op is an alternative strategy.

Please rate this question:

Discuss and give feedback


Next question

Biliary disease

Diagnosis Typical features Pathogenesis

Gallstones Typically history of Usually small calibre gallstones which can


biliary colic or pass through the cystic duct. In Mirizzi
episodes of syndrome the stone may compress the bile
chlolecystitis. duct directly- one of the rare times that
Obstructive type cholecystitis may present with jaundice
history and test results.

Cholangitis Usually obstructive Ascending infection of the bile ducts usually


and will have Charcot's by E. coliand by definition occurring in a pool
triad of symptoms of stagnant bile.
(pain, fever, jaundice)

Pancreatic cancer Typically painless Direct occlusion of distal bile duct or


jaundice with palpable pancreatic duct by tumour. Sometimes nodal
gallbladder disease at the portal hepatis may be the culprit
(Courvoisier's Law) in which case the bile duct may be of normal
calibre.

TPN (total parenteral Usually follows long Often due to hepatic dysfunction and fatty
nutrition) associated term use and is usually liver which may occur with long term TPN
jaundice painless with non usage.
obstructive features

Bile duct injury Depending upon the Often due to a difficult laparoscopic•
type of injury may be cholecystectomy when anatomy in Calots
of sudden or gradual triangle is not appreciated. In the worst
onset and is usually of scenario the bile duct is excised and jaundice
obstructive type develops rapidly post operatively. More
insidious is that of bile duct stenosis which
may be caused by clips or diathermy injury.

Cholangiocarcinoma Gradual onset Direct occlusion by disease and also extrinsic


obstructive pattern compression by nodal disease at the porta
hepatis.

Septic surgical patient Usually hepatic Combination of impaired biliary excretion and
features drugs such as ciprofloxacin which may cause
cholestasis.

Metastatic disease Mixed hepatic and post Combination of liver synthetic failure (late)
hepatic and extrinsic compression by nodal disease
and anatomical compression of intra hepatic
structures (earlier)

A gallbladder may develop a thickened wall in chronic cholecystitis, microscopically Roikitansky-


Aschoff Sinuses may be seen

Image sourced from Wikipedia

Next question
Theme: Management of pancreatic malignancy

A. Gastrojejunostomy

B. Pancreatoduodenectomy

C. MRI guided pancreatic stent

D. Endoscopic pancreatic stent

E. Duodenoduodenostomy

F. Pancreatic radiotherapy

Please select the most appropriate treatment for these patients with pancreatic cancer. Each option
may be used once, more than once or not at all.

4. A 40 year old lady presents with new onset dyspepsia. She is diagnosed as having a localised
cacinoma of the pancreatic head.

You answered Gastrojejunostomy

The correct answer is Pancreatoduodenectomy

Theme from January 2013 exam


Localised carcinoma of the pancreas is treated with a pancreatoduodenectomy, the eponymous
name for this is a Whipples procedure. Newer variants of the procedure include pylorus
preservation. Adjuvent chemotherapy is often used.

5. A 67 year old lady presents with jaundice and abdominal pain. Her investigations show a dilated
common bile duct, a carcinoma of the pancreatic head compressing the pancreatic duct. Her liver
contains bi-lobar metastasis.

You answered Gastrojejunostomy

The correct answer is Endoscopic pancreatic stent

Jaundice associated with pancreatic cancer is best managed with a stent. These are usually
inserted at the time of ERCP. Consideration here should be given to the use of a metallic stent
(which is contra indicated where resection is contemplated).

6. A 67 year old lady presents with symptoms of persistent vomiting. Her investigations show gastric
outlet obstruction from a carcinoma of the pancreatic head. Her liver contains bi-lobar metastases.

Gastrojejunostomy

Gastric outlet obstruction from pancreatic cancer is best managed with a surgical bypass procedure
or a duodenal stent (if the disease is not resectable or curable).

Please rate this question:

Discuss and give feedback

Next question

Pancreatic cancer

 Adenocarcinoma
 Risk factors: Smoking, diabetes, adenoma, familial adenomatous polyposis
 Mainly occur in the head of the pancreas (70%)
 Spread locally and metastasizes to the liver
 Carcinoma of the pancreas should be differentiated from other periampullary tumours with
better prognosis

Clinical features

 Weight loss
 Painless jaundice
 Epigastric discomfort (pain usually due to invasion of the coeliac plexus is a late feature)
 Pancreatitis
 Trousseau's sign: migratory superficial thrombophlebitis

Investigations
 USS: May miss small lesions
 CT Scanning (pancreatic protocol). If unresectable on CT then no further staging needed
 PET/CT for those with operable disease on CT alone
 ERCP/ MRI for bile duct assessment
 Staging laparoscopy to exclude peritoneal disease

Management

 Head of pancreas: Whipple's resection (SE dumping and ulcers). Newer techniques include
pylorus preservation and SMA/ SMV resection
 Carcinoma body and tail: poor prognosis, distal pancreatectomy, if operable
 Usually adjuvent chemotherapy for resectable disease
 ERCP and stent for jaundice and palliation
 Surgical bypass may be needed for duodenal obstruction

Next question
Theme: Management of jaundice

A. ERCP

B. MRCP

C. Percutaneous transhepatic cholangiogram

D. Laparotomy

E. Laparotomy and formation of hepatico-jejunostomy

F. Laparoscopic biliary bypass

G. CT scan

For each of the following scenarios please select the most appropriate next stage of management.
Each option may be used once, more than once or not at all.

7. A 65 year old man is admitted with jaundice and investigations demonstrate a carcinoma of the
pancreatic head. An ERCP is attempted but the surgeon is unable to cannulate the ampulla.

You answered ERCP

The correct answer is Percutaneous transhepatic cholangiogram

Theme from September 2012 Exam


Cancer of the pancreatic head will cause obstructive jaundice and intrahepatic duct dilatation.
When an ERCP has failed the most appropriate option is to attempt a PTC. This procedure is always
preceded by an ultrasound (which presumably this patient has already had or they would not be
undergoing an ERCP). Prior to performing the PTC it is important to stage the disease and establish
resectability or not. This is because the PTC drains frequently dislodge and fall out. It is usually
desirable to pass a stent at the time of doing the PTC to mitigate the effects of this problem.

8. A 48 year old lady is admitted with attacks of biliary colic and investigations show gallstones. A
laparoscopic cholecystectomy is performed. The operation is technically challenging due to a large
stone impacted in Hartmans pouch. Following the operation she fails to settle and becomes
jaundiced and has bile draining into a drain placed at the surgical site.
ERCP

In this scenario it must be assumed that the bile duct has been damaged. In most cases an ERCP is
the most appropriate investigation. This can also allow the passage of a stent if this is deemed to
be safe and sensible.

9. A 34 year old lady is admitted with jaundice and undergoes an ERCP. The procedure is technically
difficult and she is returned to the ward still jaundiced. Unfortunately she now has severe
generalised abdominal pain.

You answered ERCP

The correct answer is CT scan

There are two main differential diagnoses here. One is pancreatitis, repeated trauma to the
ampulla and duct (if partially cannulated) is a major risk factor for pancreatitis. The second is the
possibility that the duodenum has been perforated. ERCP is performed using a side viewing
endoscope, the manipulation of which can be technically challenging for the inexperienced
operator in a patient with abnormal anatomy. A CT scan is the best investigation to distinguish
between these two differential diagnoses.

Please rate this question:

Discuss and give feedback

Next question

Surgical jaundice

Jaundice can present in a manner of different surgical situations. As with all types of jaundice a
careful history and examination will often give clues as to the most likely underlying cause. Liver
function tests whilst conveying little in the way of information about liver synthetic function, will often
facilitate classification as to whether the jaundice is pre hepatic, hepatic or post hepatic. The typical
LFT patterns are given below:
Location Bilirubin ALT/ AST Alkaline phosphatase

Pre hepatic Normal or high Normal Normal

Hepatic High Elevated (often very high) Elevated but seldom to very high levels

Post hepatic High-very high Moderate elevation High- very high

In post hepatic jaundice the stools are often of pale colour and this feature should be specifically
addressed in the history.

Modes of presentation
These are addressed in the table below:

Diagnosis Typical features Pathogenesis

Gallstones Typically history of biliary Usually small calibre gallstones which can pass
colic or episodes of through the cystic duct. In Mirizzi syndrome the stone
chlolecystitis. Obstructive may compress the bile duct directly- one of the rare
type history and test times that cholecystitis may present with jaundice
results.

Cholangitis Usually obstructive and Ascending infection of the bile ducts usually by E.
will have Charcots triad of coliand by definition occurring in a pool of stagnant
symptoms (pain, fever, bile.
jaundice)

Pancreatic cancer Typically painless jaundice Direct occlusion of distal bile duct or pancreatic duct
with palpable gallbladder by tumour. Sometimes nodal disease at the portal
(Courvoisier's Law) hepatis may be the culprit in which case the bile duct
may be of normal calibre.

TPN associated Usually follows long term Often due to hepatic dysfunction and fatty liver
Diagnosis Typical features Pathogenesis

jaundice use and is usually painless which may occur with long term TPN usage.
with non obstructive
features

Bile duct injury Depending upon the type Often due to a difficult cholecystectomy when
of injury may be of anatomy in Calots triangle is not appreciated. In the
sudden or gradual onset worst scenario the bile duct is excised and jaundice
and is usually of offers rapidly post operatively. More insidious is that
obstructive type of bile duct stenosis which may be caused by clips or
diathermy injury.

Cholangiocarcinoma Gradual onset obstructive Direct occlusion by disease and also extrinsic
pattern compression by nodal disease at the porta hepatis.

Septic surgical Usually hepatic features Combination of impaired biliary excretion and drugs
patient such as ciprofloxacin which may cause cholestasis.

Metastatic disease Mixed hepatic and post Combination of liver synthetic failure (late) and
hepatic extrinsic compression by nodal disease and
anatomical compression of intra hepatic structures
(earlier)

Diagnosis
An ultrasound of the liver and biliary tree is the most commonly used first line test. This will establish
bile duct calibre, often ascertain the presence of gallstones, may visualise pancreatic masses and
other lesions. The most important clinical question is essentially the extent of biliary dilatation and its
distribution.

Where pancreatic neoplasia is suspected, the next test should be a pancreatic protocol CT scan.
With liver tumours and cholangiocarcinoma an MRI/ MRCP is often the preferred option. PET scans
may be used to stage a number of malignancies but do not routinely form part of first line testing.

Where MRCP fails to give adequate information an ERCP may be necessary. In many cases this
may form part of patient management. It is however, invasive and certainly not without risk and
highly operator dependent.
Management
Clearly this will depend to an extent upon the underlying cause but relief of jaundice is important,
even if surgery forms part of the planned treatment. Patients with unrelieved jaundice have a much
higher incidence of septic complications, bleeding and death.

Screen for and address any clotting irregularities

In patients with malignancy a stent will need to be inserted. These come in two main types; metal
and plastic. Plastic stents are cheap and easy to replace and should be used if any surgical
intervention (e.g. Whipples) is planned. However, they are prone to displacement and blockage.
Metal stents are much more expensive and may compromise a surgical resection. However, they
are far less prone to displacement and to a lesser extent blockage than their plastic counterparts.

If malignancy is in bile duct/ pancreatic head and stenting has been attempted and has failed, then
an alternative strategy is to drain the biliary system percutaneously via a transhepatic route. It may
also be possible to insert a stent in this way. One of the main problems with temporary PTC's is their
propensity to displacement, which may result in a bile leak.

In patients who have a bile duct injury surgery will be required to repair the defect. If the bile duct
has been inadvertently excised then a hepatico-jejunostomy will need to be created (difficult!)

If gallstones are the culprit, then these may be removed by ERCP and a cholecystectomy
performed. Where there is doubt about the efficacy of the ERCP an operative cholangiogram should
be performed and bile duct exploration undertaken where stones remain. When the bile duct has
been formally opened the options are between closure over a T tube, a choledochoduodenostomy or
choledochojejunostomy.

Patients with cholangitis should receive high dose broad spectrum antibiotics via the intravenous
route. Biliary decompression should follow soon afterwards, instrumenting the bile duct of these
patients will often provoke a septic episode (but should be done anyway).

Next question
Theme: Gallstone disease

A. Uncomplicated biliary colic


B. Acute cholecystitis
C. Cholangitis
D. Gallbladder abscess
E. Acalculous cholecystitis
F. Pancreatitis
G. Gallstone ileus

Please select the most likely underlying diagnosis for the scenario given. Each option may be used
once, more than once or not at all.

10. A 68 year old man with type 2 diabetes is admitted to hospital unwell. On examination he
has features of septic shock and right upper quadrant tenderness. He is not jaundiced.
Imaging shows a normal calibre bile duct and no stones in the gallbladder.

You answered Uncomplicated biliary colic

The correct answer is Acalculous cholecystitis

Theme from April 2013 Exam


Acalculous cholecystitis is more common in patients with an underlying co-morbidity. The
morbidity and mortality following intervention are higher than in conventional gallstone
disease.

11. A 43 year old lady with known gallstones is admitted with a high fever and jaundice. On
examination, she looks extremely unwell. Her abdomen is generally soft although there is
some mild tenderness in the right upper quadrant.

You answered Uncomplicated biliary colic

The correct answer is Cholangitis

Features of jaundice, fever and systemic sepsis are typical of cholangitis.

12. A 34 year old lady is admitted with a 3 day history of colicky right upper quadrant pain
which radiates to her back. The pain is now more constant. On examination she is not
jaundiced, but has a temperature of 38.5oC. She has localised peritonism in the right upper
quadrant.

You answered Uncomplicated biliary colic


The correct answer is Acute cholecystitis

The features of pain and fever with right upper quadrant pain are suggestive of acute
cholecystitis. The short nature of the history makes an abscess less likely.

Please rate this question:

Discuss and give feedback


Next question

Gallstones

Up to 24% of women and 12% of men may have gallstones. Of these up to 30% may develop local
infection and cholecystitis. In patients subjected to surgery 12% will have stones contained within the
common bile duct. The majority of gallstones are of a mixed composition (50%) with pure cholesterol
stones accounting for 20% of cases.
The aetiology of CBD stones differs in the world, in the West most CBD stones are the result of
migration. In the East a far higher proportion arise in the CBD de novo.
The classical symptoms are of colicky right upper quadrant pain that occurs post prandially. The
symptoms are usually worst following a fatty meal when cholecystokinin levels are highest and
gallbladder contraction is maximal.

Investigation
In almost all suspected cases the standard diagnostic work up consists of abdominal ultrasound and
liver function tests. Of patients who have stones within the bile duct, 60% will have at least one
abnormal result on LFT's. Ultrasound is an important test, but is operator dependent and therefore
may occasionally need to be repeated if a negative result is at odds with the clinical picture. Where
stones are suspected in the bile duct, the options lie between magnetic resonance cholangiography
and intraoperative imaging. The choice between these two options is determined by the skills and
experience of the surgeon. The advantages of intra operative imaging are less useful in making
therapeutic decisions if the operator is unhappy about proceeding the bile duct exploration, and in
such circumstances pre operative MRCP is probably a better option.

Specific gallstone and gallbladder related disease


Disease Features Management

Biliary colic Colicky abdominal pain, worse If imaging shows gallstones and history
post prandially, worse after fatty compatible then laparoscopic cholecystectomy
foods

Acute  Right upper quadrant Imaging (USS) and cholecystectomy (ideally


cholecystitis pain within 48 hours of presentation) (2)
 Fever
 Murphys sign on
Disease Features Management

examination
 Occasionally mildly
deranged LFT's
(especially if Mirizzi
syndrome)

Gallbladder  Usually prodromal Imaging with USS +/- CT Scanning


abscess illness and right upper Ideally surgery, sub total cholecystectomy may
quadrant pain be needed if Calots triangle is hostile
 Swinging pyrexia In unfit patients percutaneous drainage may be
 Patient may be considered
systemically unwell
 Generalised peritonism
not present

Cholangitis  Patient severely septic  Fluid resuscitation


and unwell  Broad spectrum intravenous antibiotics
 Jaundice  Correct any coagulopathy
 Right upper quadrant  Early ERCP
pain

Gallstone  Patients may have a Laparotomy and removal of gallstone from


ileus history of previous small bowel, the enterotomy must be made
cholecystitis and known proximal to the site of obstruction and not at
gallstones the site of obstruction. The fistula between the
 Small bowel obstruction gallbladder and duodenum should not be
(may be intermittent) interfered with.

Acalculous  Patients with inter If patient fit then cholecystectomy, if unfit then
cholecystitis current illness (e.g. percutaneous cholecystostomy
diabetes, organ failure)
 Patient of systemically
unwell
 Gallbladder
inflammation in absence
of stones
 High fever
Treatment
Patients with asymptomatic gallstones rarely develop symptoms related to them (less than 2% per
year) and may therefore be managed expectantly. In almost all cases of symptomatic gallstones the
treatment of choice is cholecystectomy performed via the laparoscopic route. In the very frail patient
there is sometimes a role for selective use of ultrasound guided cholecystostomy.
During the course of the procedure some surgeons will routinely perform either intra operative
cholangiography to either confirm anatomy or to exclude CBD stones. The latter may be more easily
achieved by use of laparoscopic ultrasound. If stones are found then the options lie between early
ERCP in the day or so following surgery or immediate surgical exploration of the bile duct. When
performed via the trans cystic route this adds little in the way of morbidity and certainly results in
faster recovery. Where transcystic exploration fails the alternative strategy is that of formal
choledochotomy. The exploration of a small duct is challenging and ducts of less than 8mm should
not be explored. Small stones that measure less than 5mm may be safely left and most will pass
spontaneously.

Risks of ERCP(1)

 Bleeding 0.9% (rises to 1.5% if sphincterotomy performed)


 Duodenal perforation 0.4%
 Cholangitis 1.1%
 Pancreatitis 1.5%

References
1. Williams E et al. Guidelines on the management of common bile duct stones
(CBDS)Gut2008;57:10041021

2. Gurusamy KS, Samraj K. Early versus delayed laparoscopic cholecystectomy for acute
cholecystitis. Cochrane Database Syst Rev. 2006 Oct 18;(4):CD005440.

3. Gurusamy K and Davidson B. Gallstones. BMJ 2014 (348):27-30.


Next question
Which of the following is the most sensitive blood test for diagnosis of acute pancreatitis?

Amylase

Lipase

C-peptide

Trypsin

Trysinogen

The serum amylase may rise and fall quite quickly and lead to a false negative result. Should the
clinical picture not be concordant with the amylase level then serum lipase or a CT Scan should be
performed.
Please rate this question:

Discuss and give feedback


Next question

Management of Pancreatitis

Management of Acute Pancreatitis in the UK

Diagnosis

 Traditionally hyperamylasaemia has been utilised with amylase being elevated three times
the normal range.
 However, amylase may give both false positive and negative results.
 Serum lipase is both more sensitive and specific than serum amylase. It also has a longer
half life.
 Serum amylase levels do not correlate with disease severity.

Differential causes of hyperamylasaemia


Acute pancreatitis
Pancreatic pseudocyst

Mesenteric infarct

Perforated viscus

Acute cholecystitis

Diabetic ketoacidosis

Assessment of severity

 Glasgow, Ranson scoring systems and APACHE II


 Biochemical scoring e.g. using CRP

Features that may predict a severe attack within 48 hours of admission to hospital
Initial assessment  Clinical impression of severity
 Body mass index >30
 Pleural effusion
 APACHE score >8

24 hours after admission  Clinical impression of severity


 APACHE II >8
 Glasgow score of 3 or more
 Persisting multiple organ failure
 CRP>150

48 hours after admission  Glasgow Score of >3


 CRP >150
 Persisting or progressive organ failure

Table adapted from UK guidelines for management of acute pancreatitis. GUT 2005, 54 suppl III

Management

Nutrition
 There is reasonable evidence to suggest that the use of enteral nutrition does not worsen the
outcome in pancreatitis
 Most trials to date were underpowered to demonstrate a conclusive benefit.
 The rationale behind feeding is that it helps to prevent bacterial translocation from the gut,
thereby contributing to the development of infected pancreatic necrosis.

Use of antibiotic therapy

 Many UK surgeons administer antibiotics to patients with acute pancreatitis. However, there
is very little evidence to support this practice.
 A recent Cochrane review highlights the potential benefits of administering Imipenem to
patients with established pancreatic necrosis in the hope of averting the progression to
infection.
 There are concerns that the administration of antibiotics in mild attacks of pancreatitis will not
affect outcome and may contribute to antibiotic resistance and increase the risks of antibiotic
associated diarrhoea.

Surgery

 Patients with acute pancreatitis due to gallstones should undergo early cholecystectomy.
 Patients with obstructed biliary system due to stones should undergo early ERCP.
 Patients with extensive necrosis where infection is suspected should usually undergo FNA
for culture.
 Patients with infected necrosis should undergo either radiological drainage or surgical
necrosectomy. The choice of procedure depends upon local expertise.

References
www.bsg.org.uk/pdfworddocs/pancreatic.pdf

Antibiotic therapy for prophylaxis against infection of pancreatic necrosis in acute


pancreatitis. Villatoro et al. Cochrane Library DOI: 10.1002/14651858.CD002941.pub3. 2010
version.
Next question
Theme: Jaundice

A. Gilberts syndrome

B. Crigler Najjar syndrome

C. Hepatocellular carcinoma

D. Mirizzi syndrome

E. Hepatitis A

F. Hepatitis E

G. Bile duct stones

H. Multi cystic liver disease

Please select the most likely cause of jaundice for the scenario given. Each option may be used
once, more than once or not at all.

14. A 22 year old man returns to the UK from holiday in India. He presents with painless jaundice. On
examination he is not deeply jaundiced and there is no organomegaly.

You answered Gilberts syndrome

The correct answer is Hepatitis A

Infective hepatitis is the most likely cause. In the UK, foreign travel is a common cause of
developing infectious hepatitis, of which hepatitis A is the most common.

15. A 56 year old man presents with jaundice. He has a long history of alcohol misuse. On
examination he is jaundiced and ultrasound shows multiple echo dense lesions in both lobes of
the liver. His alpha feto protein is elevated 6 times the normal range

You answered Gilberts syndrome

The correct answer is Hepatocellular carcinoma


HCC may complicate cirrhosis. AFP is often raised in HCC.

16. A 32 year old man who has suffered from Crohns disease for many years presents with
intermittent jaundice. When it occurs it is obstructive in nature. It then usually resolves
spontaneously.

You answered Gilberts syndrome

The correct answer is Bile duct stones

Bile salts are absorbed in the terminal ileum. When this process is impaired as in Crohns the
patient may develop gallstones, if these pass into the CBD then obstructive jaundice will result.

Please rate this question:

Discuss and give feedback

Next question

Surgical jaundice

Jaundice can present in a manner of different surgical situations. As with all types of jaundice a
careful history and examination will often give clues as to the most likely underlying cause. Liver
function tests whilst conveying little in the way of information about liver synthetic function, will often
facilitate classification as to whether the jaundice is pre hepatic, hepatic or post hepatic. The typical
LFT patterns are given below:

Location Bilirubin ALT/ AST Alkaline phosphatase

Pre hepatic Normal or high Normal Normal

Hepatic High Elevated (often very high) Elevated but seldom to very high levels
Location Bilirubin ALT/ AST Alkaline phosphatase

Post hepatic High-very high Moderate elevation High- very high

In post hepatic jaundice the stools are often of pale colour and this feature should be specifically
addressed in the history.

Modes of presentation
These are addressed in the table below:

Diagnosis Typical features Pathogenesis

Gallstones Typically history of biliary Usually small calibre gallstones which can pass
colic or episodes of through the cystic duct. In Mirizzi syndrome the stone
chlolecystitis. Obstructive may compress the bile duct directly- one of the rare
type history and test times that cholecystitis may present with jaundice
results.

Cholangitis Usually obstructive and Ascending infection of the bile ducts usually by E.
will have Charcots triad of coliand by definition occurring in a pool of stagnant
symptoms (pain, fever, bile.
jaundice)

Pancreatic cancer Typically painless jaundice Direct occlusion of distal bile duct or pancreatic duct
with palpable gallbladder by tumour. Sometimes nodal disease at the portal
(Courvoisier's Law) hepatis may be the culprit in which case the bile duct
may be of normal calibre.

TPN associated Usually follows long term Often due to hepatic dysfunction and fatty liver
jaundice use and is usually painless which may occur with long term TPN usage.
with non obstructive
features
Diagnosis Typical features Pathogenesis

Bile duct injury Depending upon the type Often due to a difficult cholecystectomy when
of injury may be of anatomy in Calots triangle is not appreciated. In the
sudden or gradual onset worst scenario the bile duct is excised and jaundice
and is usually of offers rapidly post operatively. More insidious is that
obstructive type of bile duct stenosis which may be caused by clips or
diathermy injury.

Cholangiocarcinoma Gradual onset obstructive Direct occlusion by disease and also extrinsic
pattern compression by nodal disease at the porta hepatis.

Septic surgical Usually hepatic features Combination of impaired biliary excretion and drugs
patient such as ciprofloxacin which may cause cholestasis.

Metastatic disease Mixed hepatic and post Combination of liver synthetic failure (late) and
hepatic extrinsic compression by nodal disease and
anatomical compression of intra hepatic structures
(earlier)

Diagnosis
An ultrasound of the liver and biliary tree is the most commonly used first line test. This will establish
bile duct calibre, often ascertain the presence of gallstones, may visualise pancreatic masses and
other lesions. The most important clinical question is essentially the extent of biliary dilatation and its
distribution.

Where pancreatic neoplasia is suspected, the next test should be a pancreatic protocol CT scan.
With liver tumours and cholangiocarcinoma an MRI/ MRCP is often the preferred option. PET scans
may be used to stage a number of malignancies but do not routinely form part of first line testing.

Where MRCP fails to give adequate information an ERCP may be necessary. In many cases this
may form part of patient management. It is however, invasive and certainly not without risk and
highly operator dependent.

Management
Clearly this will depend to an extent upon the underlying cause but relief of jaundice is important,
even if surgery forms part of the planned treatment. Patients with unrelieved jaundice have a much
higher incidence of septic complications, bleeding and death.
Screen for and address any clotting irregularities

In patients with malignancy a stent will need to be inserted. These come in two main types; metal
and plastic. Plastic stents are cheap and easy to replace and should be used if any surgical
intervention (e.g. Whipples) is planned. However, they are prone to displacement and blockage.
Metal stents are much more expensive and may compromise a surgical resection. However, they
are far less prone to displacement and to a lesser extent blockage than their plastic counterparts.

If malignancy is in bile duct/ pancreatic head and stenting has been attempted and has failed, then
an alternative strategy is to drain the biliary system percutaneously via a transhepatic route. It may
also be possible to insert a stent in this way. One of the main problems with temporary PTC's is their
propensity to displacement, which may result in a bile leak.

In patients who have a bile duct injury surgery will be required to repair the defect. If the bile duct
has been inadvertently excised then a hepatico-jejunostomy will need to be created (difficult!)

If gallstones are the culprit, then these may be removed by ERCP and a cholecystectomy
performed. Where there is doubt about the efficacy of the ERCP an operative cholangiogram should
be performed and bile duct exploration undertaken where stones remain. When the bile duct has
been formally opened the options are between closure over a T tube, a choledochoduodenostomy or
choledochojejunostomy.

Patients with cholangitis should receive high dose broad spectrum antibiotics via the intravenous
route. Biliary decompression should follow soon afterwards, instrumenting the bile duct of these
patients will often provoke a septic episode (but should be done anyway).

Next question
Theme: Surgical jaundice

A. Carcinoma of the head of the pancreas

B. Bile duct stricture

C. Mirizzi syndrome

D. Bile duct stones

E. Chronic cholecystitis

F. Peri hilar lymphadenopathy

G. Fitz - Hugh Curtis syndrome

Please select the most appropriate cause of the jaundice scenario given. Each option may be used
once, more than once or not at all.

17. A 63 year old man is admitted with obstructive jaundice that has developed over the past 3
weeks. He was previously well and on examination has a smooth mass in his right upper quadrant.

Carcinoma of the head of the pancreas

Carcinoma of the pancreas (Courvoisiers law!). The development of jaundice in association with a
smooth right upper quadrant mass is typical of distal biliary obstruction secondary to pancreatic
malignancy. A bile duct stricture would not present in this way, all the other choices are related to
gallstones and Fitz Hugh Curtis syndrome is a complication of pelvic inflammatory disease.

18. A 41 year old lady is admitted with colicky right upper quadrant pain. On clinical examination she
has a mild pyrexia and is clinically jaundiced. An ultrasound scan is reported as showing gallstones
and the patient is taken to theatre for an open cholecystectomy. At operation, Calots triangle is
almost completely impossible to delineate.

You answered Carcinoma of the head of the pancreas

The correct answer is Mirizzi syndrome

In Mirizzi syndrome the gallstone becomes impacted in Hartmans pouch. Episodes of recurrent
inflammation occur and this causes compression of the bile duct. In severe cases this then
progresses to fistulation. Surgery is extremely difficult as Calots triangle is often completely
obliterated and the risks of causing injury to the CBD are high.

19. A 72 year old man undergoes a distal gastrectomy for carcinoma of the stomach. He presents with
jaundice approximately 8 months post operatively. Ultrasound of the liver and bile ducts shows
no focal liver lesion and normal calibre common bile duct with intra hepatic duct dilatation.

You answered Carcinoma of the head of the pancreas

The correct answer is Peri hilar lymphadenopathy

Unfortunately metastatic disease is the most likely event. Peri hilar lymphadenopathy would be a
common culprit.

Courvoisiers Law:
Obstructive jaundice in the presence of a palpable gallbladder is unlikely to be due to stones.
This is due to the fibrotic effect that stones have on the gallbladder. Like all these laws there are
numerous exceptions and many cases will not present in the typical manner.

Bile duct injury


Inadvertent bile duct injury during laparoscopic surgery should be referred to a specialist
hepatobiliary surgeon. Outcomes are far worse when repair is undertaken by a non specialist
surgeon in a district hospital.

Please rate this question:

Discuss and give feedback

Next question

Surgical jaundice

Jaundice can present in a manner of different surgical situations. As with all types of jaundice a
careful history and examination will often give clues as to the most likely underlying cause. Liver
function tests whilst conveying little in the way of information about liver synthetic function, will often
facilitate classification as to whether the jaundice is pre hepatic, hepatic or post hepatic. The typical
LFT patterns are given below:

Location Bilirubin ALT/ AST Alkaline phosphatase

Pre hepatic Normal or high Normal Normal

Hepatic High Elevated (often very high) Elevated but seldom to very high levels

Post hepatic High-very high Moderate elevation High- very high

In post hepatic jaundice the stools are often of pale colour and this feature should be specifically
addressed in the history.

Modes of presentation
These are addressed in the table below:

Diagnosis Typical features Pathogenesis

Gallstones Typically history of biliary Usually small calibre gallstones which can pass
colic or episodes of through the cystic duct. In Mirizzi syndrome the stone
chlolecystitis. Obstructive may compress the bile duct directly- one of the rare
type history and test times that cholecystitis may present with jaundice
results.

Cholangitis Usually obstructive and Ascending infection of the bile ducts usually by E.
will have Charcots triad of coliand by definition occurring in a pool of stagnant
symptoms (pain, fever, bile.
jaundice)

Pancreatic cancer Typically painless jaundice Direct occlusion of distal bile duct or pancreatic duct
with palpable gallbladder by tumour. Sometimes nodal disease at the portal
(Courvoisier's Law) hepatis may be the culprit in which case the bile duct
may be of normal calibre.
Diagnosis Typical features Pathogenesis

TPN associated Usually follows long term Often due to hepatic dysfunction and fatty liver
jaundice use and is usually painless which may occur with long term TPN usage.
with non obstructive
features

Bile duct injury Depending upon the type Often due to a difficult cholecystectomy when
of injury may be of anatomy in Calots triangle is not appreciated. In the
sudden or gradual onset worst scenario the bile duct is excised and jaundice
and is usually of offers rapidly post operatively. More insidious is that
obstructive type of bile duct stenosis which may be caused by clips or
diathermy injury.

Cholangiocarcinoma Gradual onset obstructive Direct occlusion by disease and also extrinsic
pattern compression by nodal disease at the porta hepatis.

Septic surgical Usually hepatic features Combination of impaired biliary excretion and drugs
patient such as ciprofloxacin which may cause cholestasis.

Metastatic disease Mixed hepatic and post Combination of liver synthetic failure (late) and
hepatic extrinsic compression by nodal disease and
anatomical compression of intra hepatic structures
(earlier)

Diagnosis
An ultrasound of the liver and biliary tree is the most commonly used first line test. This will establish
bile duct calibre, often ascertain the presence of gallstones, may visualise pancreatic masses and
other lesions. The most important clinical question is essentially the extent of biliary dilatation and its
distribution.

Where pancreatic neoplasia is suspected, the next test should be a pancreatic protocol CT scan.
With liver tumours and cholangiocarcinoma an MRI/ MRCP is often the preferred option. PET scans
may be used to stage a number of malignancies but do not routinely form part of first line testing.

Where MRCP fails to give adequate information an ERCP may be necessary. In many cases this
may form part of patient management. It is however, invasive and certainly not without risk and
highly operator dependent.

Management
Clearly this will depend to an extent upon the underlying cause but relief of jaundice is important,
even if surgery forms part of the planned treatment. Patients with unrelieved jaundice have a much
higher incidence of septic complications, bleeding and death.

Screen for and address any clotting irregularities

In patients with malignancy a stent will need to be inserted. These come in two main types; metal
and plastic. Plastic stents are cheap and easy to replace and should be used if any surgical
intervention (e.g. Whipples) is planned. However, they are prone to displacement and blockage.
Metal stents are much more expensive and may compromise a surgical resection. However, they
are far less prone to displacement and to a lesser extent blockage than their plastic counterparts.

If malignancy is in bile duct/ pancreatic head and stenting has been attempted and has failed, then
an alternative strategy is to drain the biliary system percutaneously via a transhepatic route. It may
also be possible to insert a stent in this way. One of the main problems with temporary PTC's is their
propensity to displacement, which may result in a bile leak.

In patients who have a bile duct injury surgery will be required to repair the defect. If the bile duct
has been inadvertently excised then a hepatico-jejunostomy will need to be created (difficult!)

If gallstones are the culprit, then these may be removed by ERCP and a cholecystectomy
performed. Where there is doubt about the efficacy of the ERCP an operative cholangiogram should
be performed and bile duct exploration undertaken where stones remain. When the bile duct has
been formally opened the options are between closure over a T tube, a choledochoduodenostomy or
choledochojejunostomy.

Patients with cholangitis should receive high dose broad spectrum antibiotics via the intravenous
route. Biliary decompression should follow soon afterwards, instrumenting the bile duct of these
patients will often provoke a septic episode (but should be done anyway).

Next question
A 59 year old man is diagnosed as having carcinoma of the pancreas with two malignant deposits in
the right lobe of the liver. What is the most appropriate treatment?

Palliative chemotherapy

Liver resection followed by chemotherapy

Simultaneous resection of liver metastasis and en bloc segmental pancreatic resection

Pancreatic resection followed by liver resection once recovered

Radical radiotherapy followed by surgery

Pancreatic cancer has a poor prognosis and most cases have metastatic disease at presentation.
There is no role in pancreatic cancer for liver resection together with pancreatic surgery as there is
no survival benefit. Most centres will offer palliative chemotherapy which has improved both
longevity and quality of life.
Please rate this question:

Discuss and give feedback


Next question

Pancreatic cancer

 Adenocarcinoma
 Risk factors: Smoking, diabetes, adenoma, familial adenomatous polyposis
 Mainly occur in the head of the pancreas (70%)
 Spread locally and metastasizes to the liver
 Carcinoma of the pancreas should be differentiated from other periampullary tumours with
better prognosis

Clinical features

 Weight loss
 Painless jaundice
 Epigastric discomfort (pain usually due to invasion of the coeliac plexus is a late feature)
 Pancreatitis
 Trousseau's sign: migratory superficial thrombophlebitis
Investigations

 USS: May miss small lesions


 CT Scanning (pancreatic protocol). If unresectable on CT then no further staging needed
 PET/CT for those with operable disease on CT alone
 ERCP/ MRI for bile duct assessment
 Staging laparoscopy to exclude peritoneal disease

Management

 Head of pancreas: Whipple's resection (SE dumping and ulcers). Newer techniques include
pylorus preservation and SMA/ SMV resection
 Carcinoma body and tail: poor prognosis, distal pancreatectomy, if operable
 Usually adjuvent chemotherapy for resectable disease
 ERCP and stent for jaundice and palliation
 Surgical bypass may be needed for duodenal obstruction

Next question
A 41 year old lady with colicky right upper quadrant pain is identified as having gallstones on an
abdominal ultrasound scan. What is the most appropriate course of action?

Laparoscopic cholecystectomy

Open cholecystectomy

Liver function tests

MRCP

ERCP

Liver function testing is part of the core diagnostic work up of biliary colic and surgical planning
cannot proceed until this (and the diameter of the CBD on USS) are known.
Please rate this question:

Discuss and give feedback


Next question

Gallstones

Up to 24% of women and 12% of men may have gallstones. Of these up to 30% may develop local
infection and cholecystitis. In patients subjected to surgery 12% will have stones contained within the
common bile duct. The majority of gallstones are of a mixed composition (50%) with pure cholesterol
stones accounting for 20% of cases.
The aetiology of CBD stones differs in the world, in the West most CBD stones are the result of
migration. In the East a far higher proportion arise in the CBD de novo.
The classical symptoms are of colicky right upper quadrant pain that occurs post prandially. The
symptoms are usually worst following a fatty meal when cholecystokinin levels are highest and
gallbladder contraction is maximal.

Investigation
In almost all suspected cases the standard diagnostic work up consists of abdominal ultrasound and
liver function tests. Of patients who have stones within the bile duct, 60% will have at least one
abnormal result on LFT's. Ultrasound is an important test, but is operator dependent and therefore
may occasionally need to be repeated if a negative result is at odds with the clinical picture. Where
stones are suspected in the bile duct, the options lie between magnetic resonance cholangiography
and intraoperative imaging. The choice between these two options is determined by the skills and
experience of the surgeon. The advantages of intra operative imaging are less useful in making
therapeutic decisions if the operator is unhappy about proceeding the bile duct exploration, and in
such circumstances pre operative MRCP is probably a better option.
Specific gallstone and gallbladder related disease
Disease Features Management

Biliary colic Colicky abdominal pain, worse If imaging shows gallstones and history
post prandially, worse after fatty compatible then laparoscopic cholecystectomy
foods

Acute  Right upper quadrant Imaging (USS) and cholecystectomy (ideally


cholecystitis pain within 48 hours of presentation) (2)
 Fever
 Murphys sign on
examination
 Occasionally mildly
deranged LFT's
(especially if Mirizzi
syndrome)

Gallbladder  Usually prodromal Imaging with USS +/- CT Scanning


abscess illness and right upper Ideally surgery, sub total cholecystectomy may
quadrant pain be needed if Calots triangle is hostile
 Swinging pyrexia In unfit patients percutaneous drainage may be
 Patient may be considered
systemically unwell
 Generalised peritonism
not present

Cholangitis  Patient severely septic  Fluid resuscitation


and unwell  Broad spectrum intravenous antibiotics
 Jaundice  Correct any coagulopathy
 Right upper quadrant  Early ERCP
pain

Gallstone  Patients may have a Laparotomy and removal of gallstone from


ileus history of previous small bowel, the enterotomy must be made
cholecystitis and known proximal to the site of obstruction and not at
gallstones the site of obstruction. The fistula between the
 Small bowel obstruction gallbladder and duodenum should not be
(may be intermittent) interfered with.
Disease Features Management

Acalculous  Patients with inter If patient fit then cholecystectomy, if unfit then
cholecystitis current illness (e.g. percutaneous cholecystostomy
diabetes, organ failure)
 Patient of systemically
unwell
 Gallbladder
inflammation in absence
of stones
 High fever

Treatment
Patients with asymptomatic gallstones rarely develop symptoms related to them (less than 2% per
year) and may therefore be managed expectantly. In almost all cases of symptomatic gallstones the
treatment of choice is cholecystectomy performed via the laparoscopic route. In the very frail patient
there is sometimes a role for selective use of ultrasound guided cholecystostomy.
During the course of the procedure some surgeons will routinely perform either intra operative
cholangiography to either confirm anatomy or to exclude CBD stones. The latter may be more easily
achieved by use of laparoscopic ultrasound. If stones are found then the options lie between early
ERCP in the day or so following surgery or immediate surgical exploration of the bile duct. When
performed via the trans cystic route this adds little in the way of morbidity and certainly results in
faster recovery. Where transcystic exploration fails the alternative strategy is that of formal
choledochotomy. The exploration of a small duct is challenging and ducts of less than 8mm should
not be explored. Small stones that measure less than 5mm may be safely left and most will pass
spontaneously.

Risks of ERCP(1)

 Bleeding 0.9% (rises to 1.5% if sphincterotomy performed)


 Duodenal perforation 0.4%
 Cholangitis 1.1%
 Pancreatitis 1.5%

References
1. Williams E et al. Guidelines on the management of common bile duct stones
(CBDS)Gut2008;57:10041021

2. Gurusamy KS, Samraj K. Early versus delayed laparoscopic cholecystectomy for acute
cholecystitis. Cochrane Database Syst Rev. 2006 Oct 18;(4):CD005440.

3. Gurusamy K and Davidson B. Gallstones. BMJ 2014 (348):27-30.


Next question
A 42 year old female presents with symptoms of biliary colic and on investigation is identified as
having gallstones. Of the procedures listed below, which is most likely to increase the risk of
gallstone formation?

Partial gastrectomy

Jejunal resection

Liver lobectomy

Ileal resection

Left hemicolectomy

Bile salt reabsorption occurs at the ileum. Therefore cholesterol gallstones form as a result of ileal
resection.

Please rate this question:

Discuss and give feedback

Next question

Biliary disease

Diagnosis Typical features Pathogenesis

Gallstones Typically history of Usually small calibre gallstones which can pass
biliary colic or episodes through the cystic duct. In Mirizzi syndrome the
of chlolecystitis. stone may compress the bile duct directly- one of
Obstructive type history the rare times that cholecystitis may present with
and test results. jaundice

Cholangitis Usually obstructive and Ascending infection of the bile ducts usually by E.
will have Charcot's triad coliand by definition occurring in a pool of stagnant
of symptoms (pain, bile.
fever, jaundice)

Pancreatic cancer Typically painless Direct occlusion of distal bile duct or pancreatic
jaundice with palpable duct by tumour. Sometimes nodal disease at the
gallbladder portal hepatis may be the culprit in which case the
(Courvoisier's Law) bile duct may be of normal calibre.

TPN (total parenteral Usually follows long Often due to hepatic dysfunction and fatty liver
nutrition) associated term use and is usually which may occur with long term TPN usage.
jaundice painless with non
obstructive features

Bile duct injury Depending upon the Often due to a difficult laparoscopic•
type of injury may be of cholecystectomy when anatomy in Calots triangle is
sudden or gradual onset not appreciated. In the worst scenario the bile duct
and is usually of is excised and jaundice develops rapidly post
obstructive type operatively. More insidious is that of bile duct
stenosis which may be caused by clips or diathermy
injury.

Cholangiocarcinoma Gradual onset Direct occlusion by disease and also extrinsic


obstructive pattern compression by nodal disease at the porta hepatis.

Septic surgical patient Usually hepatic features Combination of impaired biliary excretion and
drugs such as ciprofloxacin which may cause
cholestasis.

Metastatic disease Mixed hepatic and post Combination of liver synthetic failure (late) and
hepatic extrinsic compression by nodal disease and
anatomical compression of intra hepatic structures
(earlier)

A gallbladder may develop a thickened wall in chronic cholecystitis, microscopically Roikitansky-


Aschoff Sinuses may be seen

Image sourced from Wikipedia

Next question
Theme: Management of pancreatitis

A. Non Contrast enhanced CT scan


B. USS abdomen
C. ERCP alone
D. ERCP with Sphincterotomy and biliary drainage
E. Fine needle aspiration of necrosis
F. Pancreatic necrosectomy
G. Contrast enhanced CT scan

What is the next best step in management for the scenario given? Each option may be used once,
more than once or not at all.

23. A 58 year old woman is admitted with an attack of severe acute pancreatitis. She is
managed on the intensive care unit and is making progress. She then deteriorates and a CT
scan shows extensive pancreatic necrosis (>40%). There are concerns that this may have
become infected.

You answered Non Contrast enhanced CT scan

The correct answer is Fine needle aspiration of necrosis

When there are concerns that pancreatic necrosis may have become infected the usual
approach is to perform an image guided FNA for culture. There is always the risk of
seeding infection with such a strategy so it must be performed with care. Pancreatic
necrosectomy is not usually undertaken until the presence of infection is proven.

24. A 22 year old teacher is admitted with severe epigastric pain. Serum amylase is normal.
You wish to exclude a perforated viscus, and determine whether pancreatitis is present.

You answered Non Contrast enhanced CT scan

The correct answer is Contrast enhanced CT scan

An ultrasound will not accurately answer this question. Therefore a CT scan is required.
Oral and IV contrast would usually be given.

25. A 55 year old accountant has jaundice and a temperature of 39 oC. He is known to have
gallstones. Blood cultures have grown a gram negative bacilli. Imaging shows a bile duct
measuring 1.2cm in diameter.

You answered Non Contrast enhanced CT scan


The correct answer is ERCP with Sphincterotomy and biliary drainage

You should suspect cholangitis in a patient with fevers and jaundice. Charcot's triad may
only be present in 20% of patients. This patient needs biliary drainage with an ERCP.
Infected pancreatic necrosis is one of the few indications for surgery in pancreatitis

Please rate this question:

Discuss and give feedback


Next question

Management of Pancreatitis

Management of Acute Pancreatitis in the UK

Diagnosis

 Traditionally hyperamylasaemia has been utilised with amylase being elevated three times
the normal range.
 However, amylase may give both false positive and negative results.
 Serum lipase is both more sensitive and specific than serum amylase. It also has a longer
half life.
 Serum amylase levels do not correlate with disease severity.

Differential causes of hyperamylasaemia


Acute pancreatitis

Pancreatic pseudocyst

Mesenteric infarct

Perforated viscus

Acute cholecystitis

Diabetic ketoacidosis
Assessment of severity

 Glasgow, Ranson scoring systems and APACHE II


 Biochemical scoring e.g. using CRP

Features that may predict a severe attack within 48 hours of admission to hospital
Initial assessment  Clinical impression of severity
 Body mass index >30
 Pleural effusion
 APACHE score >8

24 hours after admission  Clinical impression of severity


 APACHE II >8
 Glasgow score of 3 or more
 Persisting multiple organ failure
 CRP>150

48 hours after admission  Glasgow Score of >3


 CRP >150
 Persisting or progressive organ failure

Table adapted from UK guidelines for management of acute pancreatitis. GUT 2005, 54 suppl III

Management

Nutrition

 There is reasonable evidence to suggest that the use of enteral nutrition does not worsen the
outcome in pancreatitis
 Most trials to date were underpowered to demonstrate a conclusive benefit.
 The rationale behind feeding is that it helps to prevent bacterial translocation from the gut,
thereby contributing to the development of infected pancreatic necrosis.

Use of antibiotic therapy

 Many UK surgeons administer antibiotics to patients with acute pancreatitis. However, there
is very little evidence to support this practice.
 A recent Cochrane review highlights the potential benefits of administering Imipenem to
patients with established pancreatic necrosis in the hope of averting the progression to
infection.
 There are concerns that the administration of antibiotics in mild attacks of pancreatitis will not
affect outcome and may contribute to antibiotic resistance and increase the risks of antibiotic
associated diarrhoea.

Surgery

 Patients with acute pancreatitis due to gallstones should undergo early cholecystectomy.
 Patients with obstructed biliary system due to stones should undergo early ERCP.
 Patients with extensive necrosis where infection is suspected should usually undergo FNA
for culture.
 Patients with infected necrosis should undergo either radiological drainage or surgical
necrosectomy. The choice of procedure depends upon local expertise.

References
www.bsg.org.uk/pdfworddocs/pancreatic.pdf

Antibiotic therapy for prophylaxis against infection of pancreatic necrosis in acute


pancreatitis. Villatoro et al. Cochrane Library DOI: 10.1002/14651858.CD002941.pub3. 2010
version.
Next question
A 43 year old male with long standing chronic hepatitis is being followed up. Recently his AFP is
noted to be increased and an abdominal USS demonstrates a 2cm lesion in segment V of the liver.
What is the most appropriate course of action?

PET CT scan

Liver MRI

USS guided liver biopsy

Laparoscopic biopsy

Segmental resection of segment V

Liver lesions that are suspicious of HCC should be scanned prior to resection as there is a risk of
multifocal lesions that would either preclude or otherwise affect the decision to proceed with
segmental resection.

Please rate this question:

Discuss and give feedback

Next question

Hepatocellular carcinoma

Hepatocellular carcinoma is the second leading cause of cancer deaths globally. Up to 750,000
cases are reported annually. Unfortunately the incidence approximates to the death rate so there are
few long term survivors[1]. The disease occurs most commonly in those with chronic hepatitis and
established liver cirrhosis. Therefore, these individuals should be closely screened for the
development of HCC with serum AFP and liver USS every 6-12 months. Rising AFP and liver USS
showing a nodule greater than 1cm in diameter makes HCC much more likely and such patients
should then undergo MRI scanning.
The presence of adenomas in an otherwise healthy liver is a recognised risk factor for HCC [2, 3]
and many surgeons will remove liver adenomas for this reason[4].
Diagnosis
The aim is to avoid unnecessary percutaneous biopsy. Radiologically on CT the classical feature is a
suspicious lesion which is highlighted during the arterial phase with washout during the venous
phase, this reflects the hypervascularity of the lesions.The risk of tumour seeding as a result of a
liver biopsy is 2.7% with a median time interval between biopsy and seeding of 17 months[5].

Barcelona Clinic Liver Classification


There are many classification systems for addressing the management and prognosis, the BCLC
system has the convenience of categorising disease extent with treatment and prognostic outcomes.
In determining the ideal treatment modality for HCC the key points are not just disease extent, but
also the functional state of the liver and patient.

Prognosis-5 yr
Stage Features Treatment survival

Stage Child-Pugh A Resection 40-70%


0 Single lesion (less than 2cm)
Normal portal pressures

Stage Single nodule greater than 3cm or If associated disease then May be up to 70%
A multiple nodules (no more than 3) radiofrequency ablation in some
Child Pugh A/ B If no associated disease then
transplantation

Stage Multiple nodules Trans arterial chemo-embolisation 26% at 3 years


B Child Pugh A/B (usually with doxorubicin)

Stage Advanced tumours Sorafenib Usually survive


C Invasion of portal vein 10.7 months
Child Pugh A/B

Stage Child Pugh stage C Best supportive care Less than 6


D Advanced tumours months survival

[6]
In selected patients the best outcomes are achieved with surgical resection, or transplantation where
surgical resection is precluded. Anatomical resections with minimum 2cm margins provide the best
outcomes.
At the present time there is no evidence to recommend treatment with adjuvant chemotherapy[6].

Sorafenib
This is an oral multi tyrosine kinase inhibitor. It is the only drug that has been currently demonstrated
to extend survival in individuals with advanced hepatocellular cancer[7]. The improvement in survival
is from a median of 7 months to 10 months.

References
1. Jemal, A., et al., Global cancer statistics. CA Cancer J Clin, 2011. 61(2): p. 69-90.
2. Leese, T., O. Farges, and H. Bismuth, Liver cell adenomas. A 12-year surgical experience from a
specialist hepato-biliary unit. Ann Surg, 1988. 208(5): p. 558-64.
3. Farges, O. and S. Dokmak, Malignant transformation of liver adenoma: an analysis of the
literature. Dig Surg, 2010. 27(1): p. 32-8.
4. Ehrl, D., et al., "Incidentaloma" of the liver: management of a diagnostic and therapeutic dilemma.
HPB Surg, 2012. 2012: p. 891787.
5. Silva, M.A., et al., Needle track seeding following biopsy of liver lesions in the diagnosis of
hepatocellular cancer: a systematic review and meta-analysis. Gut, 2008. 57(11): p. 1592-6.
6. EASL-EORTC clinical practice guidelines: management of hepatocellular carcinoma. J Hepatol,
2012. 56(4): p. 908-43.
7. Abou-Alfa, G.K., et al., Phase II study of sorafenib in patients with advanced hepatocellular
carcinoma. J Clin Oncol, 2006. 24(26): p. 4293-300.

Next question
Theme: Management of biliary diseases

A. Acute laparoscopic cholecystectomy

B. Delayed laparoscopic cholecystectomy

C. Percutaneous cholecystostomy

D. Lithotripsy

E. Endoscopic retrograde cholangiopancreatography

F. Choledochoduodenostomy

G. Bile duct excision and hepatico-jejunostomy

H. Operative cholecystostomy

Please select the most appropriate management option for the scenario given. Each option may be
used once, more than once or not at all.

27. A 43 year old women is admitted with acute cholecystitis and fails to settle. A laparoscopic
cholecystectomy is being performed, at operation the gallbladder has evidence of an empyema
and Calots triangle is inflamed and the surgeon suspects that a Mirizzi syndrome has occurred.

You answered Acute laparoscopic cholecystectomy

The correct answer is Operative cholecystostomy

This will address the acute sepsis and resolve the situation. Attempts at completing the surgery at
this stage, even in expert hands carries a very high risk of bile duct injury.

28. Following a difficult cholecystectomy a surgeon leaves a drain. 24 hours later bile is seen to be
accumulating in the drain and this fails to resolve over the next 48 hours. The patient is otherwise
well.

You answered Acute laparoscopic cholecystectomy

The correct answer is Endoscopic retrograde cholangiopancreatography


This will delineate the presence of potential bile duct injury. Usually this is result of leakage from
the cystic duct and placement of a stent will allow free biliary drainage and the leak should settle.

29. A 40 year old woman is admitted with abdominal pain. She has suffered from repeated episodes
of this colicky right upper quadrant pain. On examination she is pyrexial with right upper quadrant
peritonism. Her blood tests show a white cell count of 23. However, the liver function tests are
normal. An abdominal ultrasound scan shows multiple gallstones in a thick walled gallbladder, the
bile duct measures 4mm.

Acute laparoscopic cholecystectomy

This lady has acute cholecystitis and needs an acute cholecystectomy. This operation should
usually be performed within 48 hours of admission. Delay beyond this timeframe will usually
result in increased operative complications and most surgeons would administer antibiotics and
perform and interval cholecystectomy if the early window for an acute procedure is missed. A bile
duct measuring 4mm is usually normal.

Please rate this question:

Discuss and give feedback

Next question

Biliary disease

Diagnosis Typical features Pathogenesis

Gallstones Typically history of Usually small calibre gallstones which can pass
biliary colic or episodes through the cystic duct. In Mirizzi syndrome the
of chlolecystitis. stone may compress the bile duct directly- one of
Obstructive type history the rare times that cholecystitis may present with
and test results. jaundice
Cholangitis Usually obstructive and Ascending infection of the bile ducts usually by E.
will have Charcot's triad coliand by definition occurring in a pool of stagnant
of symptoms (pain, bile.
fever, jaundice)

Pancreatic cancer Typically painless Direct occlusion of distal bile duct or pancreatic
jaundice with palpable duct by tumour. Sometimes nodal disease at the
gallbladder portal hepatis may be the culprit in which case the
(Courvoisier's Law) bile duct may be of normal calibre.

TPN (total parenteral Usually follows long Often due to hepatic dysfunction and fatty liver
nutrition) associated term use and is usually which may occur with long term TPN usage.
jaundice painless with non
obstructive features

Bile duct injury Depending upon the Often due to a difficult laparoscopic•
type of injury may be of cholecystectomy when anatomy in Calots triangle is
sudden or gradual onset not appreciated. In the worst scenario the bile duct
and is usually of is excised and jaundice develops rapidly post
obstructive type operatively. More insidious is that of bile duct
stenosis which may be caused by clips or diathermy
injury.

Cholangiocarcinoma Gradual onset Direct occlusion by disease and also extrinsic


obstructive pattern compression by nodal disease at the porta hepatis.

Septic surgical patient Usually hepatic features Combination of impaired biliary excretion and
drugs such as ciprofloxacin which may cause
cholestasis.

Metastatic disease Mixed hepatic and post Combination of liver synthetic failure (late) and
hepatic extrinsic compression by nodal disease and
anatomical compression of intra hepatic structures
(earlier)
A gallbladder may develop a thickened wall in chronic cholecystitis, microscopically Roikitansky-
Aschoff Sinuses may be seen

Image sourced from Wikipedia

Next question
Theme: Liver tumours

A. Rhabdomyosarcoma
B. Yolk sac tumour
C. Hepatocellular carcinoma
D. Metastatic lesion
E. Haemangioendothelioma
F. Cholangiocarcinoma
G. Hepatoblastoma
H. Angiosarcoma

Please select the most likely diagnosis for the scenario given. Each answer may be used once, more
than once or not at all.

30. A 56 year old man with long standing ulcerative colitis and a DALM lesion in the rectum
is admitted with jaundice. On CT scanning the liver has 3 nodules in the right lobe and 1
nodule in the left lobe. Carcinoembryonic antigen levels are elevated.

You answered Rhabdomyosarcoma

The correct answer is Metastatic lesion

This is likely to be due to metastatic lesions from a colonic primary. DALM lesions should
be excised by oncological colectomy for this reason. This burden of metastatic disease is
unlikely to precipitate jaundice directly and nodal disease at the porta hepatis is the most
likely cause in this case.

31. A 48 year old lady with chronic hepatitis B infection is noted to have worsening liver
function tests and progressive jaundice. Her alpha feto protein levels are grossly elevated.

You answered Rhabdomyosarcoma

The correct answer is Hepatocellular carcinoma

This is most likely to be hepatocellular carcinoma and markedly elevated AFP levels in
association with a compatible risk factor should make this the diagnosis.

32. A 55 year old man with long standing ulcerative colitis is admitted with cholangitis and
weight loss. Blood tests reveal a markedly elevated CA 19-9.

You answered Rhabdomyosarcoma

The correct answer is Cholangiocarcinoma


This is most likely a cholangiocarcinoma. UC with sclerosing cholangitis increases the risk
of cholangiocarcinoma. CA19-9 is elevated in approximately 80% cases.

Please rate this question:

Discuss and give feedback


Next question

Liver tumours

Primary liver tumours


The most common primary tumours are cholangiocarcinoma and hepatocellular carcinoma. Overall
metastatic disease accounts for 95% of all liver malignancies making the primary liver tumours
comparatively rare.

Primary liver tumours include:

 Cholangiocarcinoma
 Hepatocellular carcinoma
 Hepatoblastoma
 Sarcomas (Rare)
 Lymphomas
 Carcinoids (most often secondary although primary may occur)

Hepatocellular carcinoma
These account for the bulk of primary liver tumours (75% cases). Its worldwide incidence reflects its
propensity to occur on a background of chronic inflammatory activity. Most cases arise in cirrhotic
livers or those with chronic hepatitis B infection, especially where viral replication is actively
occurring. In the UK it accounts for less than 5% of all cancers, although in parts of Asia its
incidence is 100 per 100,000.
The majority of patients (80%) present with existing liver cirrhosis, with a mass discovered on
screening ultrasound.

Diagnosis

 CT/ MRI (usually both) are the imaging modalities of choice


 a-fetoprotein is elevated in almost all cases
 Biopsy should be avoided as it seeds tumours cells through a resection plane.
 In cases of diagnostic doubt serial CT and αFP measurements are the preferred strategy.

Treatment
 Patients should be staged with liver MRI and chest, abdomen and pelvic CT scan.
 The testis should be examined in males (testicular tumours may cause raised AFP). PET CT
may be used to identify occult nodal disease.
 Surgical resection is the mainstay of treatment in operable cases. In patients with a small
primary tumour in a cirrhotic liver whose primary disease process is controlled, consideration
may be given to primary whole liver resection and transplantation.
 Liver resections are an option but since most cases occur in an already diseased liver the
operative risks and post-operative hepatic dysfunction are far greater than is seen following
metastectomy.
 These tumours are not particularly chemo or radiosensitive however, both may be used in a
palliative setting. Tumour ablation is a more popular strategy.

Survival
Poor, overall survival is 15% at 5 years.

Cholangiocarcinoma
This is the second most common type of primary liver malignancy. As its name suggests these
tumours arise in the bile ducts. Up to 80% of tumours arise in the extra hepatic biliary tree. Most
patients present with jaundice and by this stage the majority will have disease that is not resectable.
Primary sclerosing cholangitis is the main risk factor. In deprived countries typhoid and liver flukes
are also major risk factors.

Diagnosis

 Patients will typically have an obstructive picture on liver function tests.


 CA 19-9, CEA and CA 125 are often elevated
 CT/ MRI and MRCP are the imaging methods of choice.

Treatment

 Surgical resection offers the best chance of cure. Local invasion of peri hilar tumours is a
particular problem and this coupled with lobar atrophy will often contra indicate surgical
resection.
 Palliation of jaundice is important, although metallic stents should be avoided in those
considered for resection.

Survival
Is poor, approximately 5-10% 5 year survival.
Next question
A 45 year old man presents with an episode of alcoholic pancreatitis. He makes slow but steady
progress. He is reviewed clinically at 6 weeks following admission. He has a diffuse fullness of his
upper abdomen and on imaging a collection of fluid is found to be located behind the stomach. His
serum amylase is mildly elevated. Which of the following is the most likely explanation?

Early fluid collection

Pancreatic abscess

Peripancreatic necrosis

Pseudocyst

Sterile necrosis

Psuedocysts are unlikely to be present less than 4 weeks after an attack of acute pancreatitis.
However, they are more common at this stage and are associated with a raised amylase.
Please rate this question:

Discuss and give feedback


Next question

Pancreatitis: sequelae

Peripancreatic fluid collections

 Occur in 25% cases


 Located in or near the pancreas and lack a wall of granulation or fibrous tissue
 May resolve or develop into pseudocysts or abscesses
 Since most resolve aspiration and drainage is best avoided as it may precipitate infection

Pseudocysts

 In acute pancreatitis result from organisation of peripancreatic fluid collection. They may or
may not communicate with the ductal system.
 The collection is walled by fibrous or granulation tissue and typically occurs 4 weeks or more
after an attack of acute pancreatitis
 Most are retrogastric
 75% are associated with persistent mild elevation of amylase
 Investigation is with CT, ERCP and MRI or Endoscopic USS
 Symptomatic cases may be observed for 12 weeks as up to 50% resolve
 Treatment is either with endoscopic or surgical cystogastrostomy or aspiration

Pancreatic necrosis

 Pancreatic necrosis may involve both the pancreatic parenchyma and surrounding fat
 Complications are directly linked to extent of parenchymal necrosis and extent of necrosis
overall
 Early necrosectomy is associated with a high mortality rate (and should be avoided unless
compelling indications for surgery exist)
 Sterile necrosis should be managed conservatively (at least initially)
 Some centres will perform fine needle aspiration sampling of necrotic tissue if infection is
suspected. False negatives may occur. The extent of sepsis and organ dysfunction may be a
better guide to surgery

Pancreatic abscess

 Intra abdominal collection of pus associated with pancreas but in the absence of necrosis
 Typically occur as a result of infected pseudocyst
 They are usually managed by placement of percutaneous drains

Haemorrhage

 Infected necrosis may involve vascular structures with resultant haemorrhage that may occur
de novo or as a result of surgical necrosectomy.
 When retroperitoneal haemorrhage occurs Grey Turners sign may be identified

Next question
A 34 year old lady is admitted with pancreatitis. The aetiology is unclear and it is classified as an
attack of moderate severity according to the Glasgow criteria. Her imaging shows no gallstones and
fluid around the pancreas. Which of the following is the most appropriate initial management option?

Laparotomy

Laparoscopy

Radiological aspiration of the fluid

Active observation

Administration of octreotide

LEARN THIS!

Mnemonic for the assessment of the severity of pancreatitis: PANCREAS


(Ann R Coll Surg Engl 2000; 82: 16-17

P a02 < 60 mmHg


A ge > 55 years
N eutrophils > 15 x 10/l
C alcium < 2 mmol/l
R aised urea > 16 mmol/l
E nzyme (lactate dehydrogenase) > 600 units/l
A lbumin < 32 g/l
S ugar (glucose) > 10 mmol/l

> 3 positive criteria indicates severe pancreatitis.

Acute early fluid collections are seen in 25% of patients with pancreatitis and require no specific
treatment. Attempts at drainage may introduce infection and result in pancreatic abscess formation.
Please rate this question:

Discuss and give feedback


Next question

Management of Pancreatitis

Management of Acute Pancreatitis in the UK


Diagnosis

 Traditionally hyperamylasaemia has been utilised with amylase being elevated three times
the normal range.
 However, amylase may give both false positive and negative results.
 Serum lipase is both more sensitive and specific than serum amylase. It also has a longer
half life.
 Serum amylase levels do not correlate with disease severity.

Differential causes of hyperamylasaemia


Acute pancreatitis

Pancreatic pseudocyst

Mesenteric infarct

Perforated viscus

Acute cholecystitis

Diabetic ketoacidosis

Assessment of severity

 Glasgow, Ranson scoring systems and APACHE II


 Biochemical scoring e.g. using CRP

Features that may predict a severe attack within 48 hours of admission to hospital
Initial assessment  Clinical impression of severity
 Body mass index >30
 Pleural effusion
 APACHE score >8

24 hours after admission  Clinical impression of severity


 APACHE II >8
 Glasgow score of 3 or more
 Persisting multiple organ failure
 CRP>150

48 hours after admission  Glasgow Score of >3


 CRP >150
 Persisting or progressive organ failure

Table adapted from UK guidelines for management of acute pancreatitis. GUT 2005, 54 suppl III

Management

Nutrition

 There is reasonable evidence to suggest that the use of enteral nutrition does not worsen the
outcome in pancreatitis
 Most trials to date were underpowered to demonstrate a conclusive benefit.
 The rationale behind feeding is that it helps to prevent bacterial translocation from the gut,
thereby contributing to the development of infected pancreatic necrosis.

Use of antibiotic therapy

 Many UK surgeons administer antibiotics to patients with acute pancreatitis. However, there
is very little evidence to support this practice.
 A recent Cochrane review highlights the potential benefits of administering Imipenem to
patients with established pancreatic necrosis in the hope of averting the progression to
infection.
 There are concerns that the administration of antibiotics in mild attacks of pancreatitis will not
affect outcome and may contribute to antibiotic resistance and increase the risks of antibiotic
associated diarrhoea.

Surgery

 Patients with acute pancreatitis due to gallstones should undergo early cholecystectomy.
 Patients with obstructed biliary system due to stones should undergo early ERCP.
 Patients with extensive necrosis where infection is suspected should usually undergo FNA
for culture.
 Patients with infected necrosis should undergo either radiological drainage or surgical
necrosectomy. The choice of procedure depends upon local expertise.

References
www.bsg.org.uk/pdfworddocs/pancreatic.pdf
Antibiotic therapy for prophylaxis against infection of pancreatic necrosis in acute
pancreatitis. Villatoro et al. Cochrane Library DOI: 10.1002/14651858.CD002941.pub3. 2010
version.
Next question
A 43 year old lady presents with an attack of acute pancreatitis. It is classified as a mild attack on
severity scoring. Imaging identifies gallstones but a normal calibre bile duct, and a peripancreatic
fluid collection. Which of the following management options is most appropriate?

Intravenous octreotide

Cholecystectomy within 4 weeks

Nasogastric tube drainage of the stomach

Insertion of a radiological drain

Avoidance of enteral feeding

Patients with gallstone pancreatitis should undergo early cholecystectomy.


Enteral feeding helps minimise gut bacterial translocation and should be given to most patients with
pancreatitis. Many studies have evaluated the role of octreotide in reducing pancreatic secretions
and shown no benefit (Uhl W et al Gut 1999 45:97-104, McKay C et al. Int J Pancreatol 1997; 21:
13-19).
The use of antibiotics in pancreatitis is controversial. However, a recent Cochrane review has
presented reasonable evidence in favor of administration of imipenem to prevent infection in
established necrosis.

Please rate this question:

Discuss and give feedback

Next question

Management of Pancreatitis

Management of Acute Pancreatitis in the UK

Diagnosis
 Traditionally hyperamylasaemia has been utilised with amylase being elevated three times
the normal range.
 However, amylase may give both false positive and negative results.
 Serum lipase is both more sensitive and specific than serum amylase. It also has a longer
half life.
 Serum amylase levels do not correlate with disease severity.

Differential causes of hyperamylasaemia

Acute pancreatitis

Pancreatic pseudocyst

Mesenteric infarct

Perforated viscus

Acute cholecystitis

Diabetic ketoacidosis

Assessment of severity

 Glasgow, Ranson scoring systems and APACHE II


 Biochemical scoring e.g. using CRP

Features that may predict a severe attack within 48 hours of admission to hospital

Initial assessment  Clinical impression of severity


 Body mass index >30
 Pleural effusion
 APACHE score >8

24 hours after admission  Clinical impression of severity


 APACHE II >8
 Glasgow score of 3 or more
 Persisting multiple organ failure
 CRP>150

48 hours after admission  Glasgow Score of >3


 CRP >150
 Persisting or progressive organ failure

Table adapted from UK guidelines for management of acute pancreatitis. GUT 2005, 54 suppl III

Management

Nutrition

 There is reasonable evidence to suggest that the use of enteral nutrition does not worsen the
outcome in pancreatitis
 Most trials to date were underpowered to demonstrate a conclusive benefit.
 The rationale behind feeding is that it helps to prevent bacterial translocation from the gut,
thereby contributing to the development of infected pancreatic necrosis.

Use of antibiotic therapy

 Many UK surgeons administer antibiotics to patients with acute pancreatitis. However, there
is very little evidence to support this practice.
 A recent Cochrane review highlights the potential benefits of administering Imipenem to
patients with established pancreatic necrosis in the hope of averting the progression to
infection.
 There are concerns that the administration of antibiotics in mild attacks of pancreatitis will not
affect outcome and may contribute to antibiotic resistance and increase the risks of antibiotic
associated diarrhoea.

Surgery

 Patients with acute pancreatitis due to gallstones should undergo early cholecystectomy.
 Patients with obstructed biliary system due to stones should undergo early ERCP.
 Patients with extensive necrosis where infection is suspected should usually undergo FNA
for culture.
 Patients with infected necrosis should undergo either radiological drainage or surgical
necrosectomy. The choice of procedure depends upon local expertise.

References
www.bsg.org.uk/pdfworddocs/pancreatic.pdf
Antibiotic therapy for prophylaxis against infection of pancreatic necrosis in acute
pancreatitis. Villatoro et al. Cochrane Library DOI: 10.1002/14651858.CD002941.pub3. 2010
version.
Next question
A 43 year old lady with repeated episodes of abdominal pain is admitted with small bowel
obstruction. A laparotomy is performed and at surgery a gallstone ileus is identified. What is the
most appropriate course of action?

Remove the gallstone via a proximally sited terminal ileal enterotomy and decompress the small
bowel. Leave the gallbladder in situ.

Remove the gallstone via a proximally sited terminal ileal enterotomy and decompress the small
bowel. Remove the gallbladder.

Remove the gallstone via a proximally sited terminal ileal enterotomy and decompress the small
bowel. Perform a choledochoduodenostomy.

Remove the gallstone from an enterotomy at the site of the obstruction and leave the
gallbladder in situ.

Remove the gallstone from an enterotomy at the site of the obstruction and remove the
gallbladder.

Gallstone ileus occurs as a result of the fistula developing between the gallbladder and the
duodenum. These tend to become impacted somewhat proximal to the ileocaecal valve and cause
small bowel obstruction. The correct management is to remove the gallstone from an enterotomy
proximal to the site of stone impaction. The bowel at the site of impaction itself may not heal well and
an enterotomy performed at this site may well result in the need for a resection. The standard
surgical teaching is that under almost all circumstances the gallbladder should be left in situ, as the
anatomy in this area is often hostile and unpredictable. Disconnecting it from the duodenum leaves a
large defect that is difficult to close.

Please rate this question:

Discuss and give feedback

Next question

Gallstones
Up to 24% of women and 12% of men may have gallstones. Of these up to 30% may develop local
infection and cholecystitis. In patients subjected to surgery 12% will have stones contained within the
common bile duct. The majority of gallstones are of a mixed composition (50%) with pure cholesterol
stones accounting for 20% of cases.
The aetiology of CBD stones differs in the world, in the West most CBD stones are the result of
migration. In the East a far higher proportion arise in the CBD de novo.
The classical symptoms are of colicky right upper quadrant pain that occurs post prandially. The
symptoms are usually worst following a fatty meal when cholecystokinin levels are highest and
gallbladder contraction is maximal.

Investigation
In almost all suspected cases the standard diagnostic work up consists of abdominal ultrasound and
liver function tests. Of patients who have stones within the bile duct, 60% will have at least one
abnormal result on LFT's. Ultrasound is an important test, but is operator dependent and therefore
may occasionally need to be repeated if a negative result is at odds with the clinical picture. Where
stones are suspected in the bile duct, the options lie between magnetic resonance cholangiography
and intraoperative imaging. The choice between these two options is determined by the skills and
experience of the surgeon. The advantages of intra operative imaging are less useful in making
therapeutic decisions if the operator is unhappy about proceeding the bile duct exploration, and in
such circumstances pre operative MRCP is probably a better option.

Specific gallstone and gallbladder related disease

Disease Features Management

Biliary colic Colicky abdominal pain, worse If imaging shows gallstones and history compatible
post prandially, worse after fatty then laparoscopic cholecystectomy
foods

Acute  Right upper quadrant pain Imaging (USS) and cholecystectomy (ideally within
cholecystitis  Fever 48 hours of presentation) (2)
 Murphys sign on
examination
 Occasionally mildly
deranged LFT's (especially
if Mirizzi syndrome)

Gallbladder  Usually prodromal illness Imaging with USS +/- CT Scanning


abscess and right upper quadrant Ideally surgery, sub total cholecystectomy may be
pain needed if Calots triangle is hostile
 Swinging pyrexia
In unfit patients percutaneous drainage may be
 Patient may be
systemically unwell
Disease Features Management

 Generalised peritonism considered


not present

Cholangitis  Patient severely septic  Fluid resuscitation


and unwell
 Jaundice  Broad spectrum intravenous antibiotics
 Right upper quadrant pain
 Correct any coagulopathy

 Early ERCP

Gallstone  Patients may have a Laparotomy and removal of gallstone from small
ileus history of previous bowel, the enterotomy must be made proximal to
cholecystitis and known the site of obstruction and not at the site of
gallstones
obstruction. The fistula between the gallbladder and
 Small bowel obstruction
(may be intermittent) duodenum should not be interfered with.

Acalculous  Patients with inter If patient fit then cholecystectomy, if unfit then
cholecystitis current illness (e.g. percutaneous cholecystostomy
diabetes, organ failure)
 Patient of systemically
unwell
 Gallbladder inflammation
in absence of stones
 High fever

Treatment
Patients with asymptomatic gallstones rarely develop symptoms related to them (less than 2% per
year) and may therefore be managed expectantly. In almost all cases of symptomatic gallstones the
treatment of choice is cholecystectomy performed via the laparoscopic route. In the very frail patient
there is sometimes a role for selective use of ultrasound guided cholecystostomy.
During the course of the procedure some surgeons will routinely perform either intra operative
cholangiography to either confirm anatomy or to exclude CBD stones. The latter may be more easily
achieved by use of laparoscopic ultrasound. If stones are found then the options lie between early
ERCP in the day or so following surgery or immediate surgical exploration of the bile duct. When
performed via the trans cystic route this adds little in the way of morbidity and certainly results in
faster recovery. Where transcystic exploration fails the alternative strategy is that of formal
choledochotomy. The exploration of a small duct is challenging and ducts of less than 8mm should
not be explored. Small stones that measure less than 5mm may be safely left and most will pass
spontaneously.

Risks of ERCP(1)

 Bleeding 0.9% (rises to 1.5% if sphincterotomy performed)


 Duodenal perforation 0.4%
 Cholangitis 1.1%
 Pancreatitis 1.5%

References
1. Williams E et al. Guidelines on the management of common bile duct stones
(CBDS)Gut2008;57:10041021

2. Gurusamy KS, Samraj K. Early versus delayed laparoscopic cholecystectomy for acute
cholecystitis. Cochrane Database Syst Rev. 2006 Oct 18;(4):CD005440.

3. Gurusamy K and Davidson B. Gallstones. BMJ 2014 (348):27-30.


Next question
A 43 year old lady presents with jaundice and is diagnosed as having a carcinoma of the head of the
pancreas. Although she is deeply jaundiced, her staging investigations are negative for metastatic
disease. What is the best method of biliary decompression in this case?

ERCP and placement of metallic stent

ERCP alone

ERCP and placement of plastic stent

Cholecystostomy

Choledochoduodenostomy

Metallic stents are contraindicated in resectable biliary disease

A plastic stent is the best option for biliary decompression in resectable disease. Surgical bypasses
have no place in the management of operable malignancy as a bridge to definitive surgery.
Please rate this question:

Discuss and give feedback


Next question

Pancreatic stents

Both benign and malignant biliary obstruction may be treated by placement of stents. These may be
either plastic tubes or self expanding metallic stents. They can be placed either percutaneously, at
ERCP, or, less commonly now, open surgery. Complications include blockage, displacement and
those related to the method of insertion.

Metallic Vs Plastic stents


Metallic stents Plastic stents

Expensive Cheap

Embed in surrounding tissues Do not usually embed


Displacement rare Displacement common

Blockage rare Blockage common

Contraindicated in resectable malignant disease May be used as a bridge to resectional surgery

Next question
Theme: Pancreatitis management

A. Pancreatic necrosectomy

B. Staging laparotomy to assess severity

C. Endoscopic retrograde cholangiopancreatography

D. Emergency cystogastrostomy

E. Cholecystectomy within 4 weeks

F. Elective cystogastrostomy

G. Parenteral nutrition

Please select the most appropriate next stage in management for the scenario given. Each option
may be used once, more than once or not at all.

38. A 34 year old women is admitted with cholangitis. Her bilirubin is 180 and alkaline phosphatase is
348. She becomes progressively more unwell and develops abdominal pain. The houseman checks
her amylase which is elevated at 1080. Standard treatment is initiated and her Glasgow score is 3.

You answered Pancreatic necrosectomy

The correct answer is Endoscopic retrograde cholangiopancreatography

She requires urgent decompression of her biliary system. An ERCP is the conventional method of
performing this. It is important to ensure that her coagulation status is normalised prior to
performing this procedure.

39. A 63 year old man is admitted to ITU with an attack of severe gallstone pancreatitis. He requires
ventillatory support for ARDS. Over the past few days he has become more unwell and a CT scan
is organised. This demonstrates an area of necrosis, culture from this area shows a gram negative
bacillus. His CRP is 400 and WCC 25.1.

Pancreatic necrosectomy

This man requires necrosectomy as he has infected pancreatic necrosis and is haemodynamically
unstable. A radiological drainage procedure is unlikely to be sufficient.

40. A 53 year old alcoholic develops acute pancreatitis and is making slow but reasonable progress.
He is troubled by persisting ileus and for this reason a CT scan is undertaken. This demonstrates a
large pancreatic pseudocyst. This is monitored by repeat CT scanning which shows no resolution
and he is now complaining of early satiety.

You answered Pancreatic necrosectomy

The correct answer is Elective cystogastrostomy

Drainage of this man's pseudocyst is required. This could be accomplished radiologically or


endoscopically or surgically. As the other options are not on the list this is the best option from
those available.

Please rate this question:

Discuss and give feedback

Next question

Management of Pancreatitis

Management of Acute Pancreatitis in the UK

Diagnosis

 Traditionally hyperamylasaemia has been utilised with amylase being elevated three times
the normal range.
 However, amylase may give both false positive and negative results.
 Serum lipase is both more sensitive and specific than serum amylase. It also has a longer
half life.
 Serum amylase levels do not correlate with disease severity.

Differential causes of hyperamylasaemia


Acute pancreatitis

Pancreatic pseudocyst

Mesenteric infarct

Perforated viscus

Acute cholecystitis

Diabetic ketoacidosis

Assessment of severity

 Glasgow, Ranson scoring systems and APACHE II


 Biochemical scoring e.g. using CRP

Features that may predict a severe attack within 48 hours of admission to hospital

Initial assessment  Clinical impression of severity


 Body mass index >30
 Pleural effusion
 APACHE score >8

24 hours after admission  Clinical impression of severity


 APACHE II >8
 Glasgow score of 3 or more
 Persisting multiple organ failure
 CRP>150

48 hours after admission  Glasgow Score of >3


 CRP >150
 Persisting or progressive organ failure
Table adapted from UK guidelines for management of acute pancreatitis. GUT 2005, 54 suppl III

Management

Nutrition

 There is reasonable evidence to suggest that the use of enteral nutrition does not worsen the
outcome in pancreatitis
 Most trials to date were underpowered to demonstrate a conclusive benefit.
 The rationale behind feeding is that it helps to prevent bacterial translocation from the gut,
thereby contributing to the development of infected pancreatic necrosis.

Use of antibiotic therapy

 Many UK surgeons administer antibiotics to patients with acute pancreatitis. However, there
is very little evidence to support this practice.
 A recent Cochrane review highlights the potential benefits of administering Imipenem to
patients with established pancreatic necrosis in the hope of averting the progression to
infection.
 There are concerns that the administration of antibiotics in mild attacks of pancreatitis will not
affect outcome and may contribute to antibiotic resistance and increase the risks of antibiotic
associated diarrhoea.

Surgery

 Patients with acute pancreatitis due to gallstones should undergo early cholecystectomy.
 Patients with obstructed biliary system due to stones should undergo early ERCP.
 Patients with extensive necrosis where infection is suspected should usually undergo FNA
for culture.
 Patients with infected necrosis should undergo either radiological drainage or surgical
necrosectomy. The choice of procedure depends upon local expertise.

References
www.bsg.org.uk/pdfworddocs/pancreatic.pdf

Antibiotic therapy for prophylaxis against infection of pancreatic necrosis in acute


pancreatitis. Villatoro et al. Cochrane Library DOI: 10.1002/14651858.CD002941.pub3. 2010
version.
Next question
What proportion of patients presenting for cholecystectomy for treatment of biliary colic due to
gallstones will have stones in the common bile duct?

10%

30%

2%

50%

25%

Up to 10% of all patients may have stones in the CBD. Therefore, all patients should have their liver
function tests checked prior to embarking on a cholecystectomy.
Please rate this question:

Discuss and give feedback

Biliary disease

Diagnosis Typical features Pathogenesis

Gallstones Typically history of Usually small calibre gallstones which can


biliary colic or pass through the cystic duct. In Mirizzi
episodes of syndrome the stone may compress the bile
chlolecystitis. duct directly- one of the rare times that
Obstructive type cholecystitis may present with jaundice
history and test results.

Cholangitis Usually obstructive Ascending infection of the bile ducts usually


and will have Charcot's by E. coliand by definition occurring in a pool
triad of symptoms of stagnant bile.
(pain, fever, jaundice)
Pancreatic cancer Typically painless Direct occlusion of distal bile duct or
jaundice with palpable pancreatic duct by tumour. Sometimes nodal
gallbladder disease at the portal hepatis may be the culprit
(Courvoisier's Law) in which case the bile duct may be of normal
calibre.

TPN (total parenteral Usually follows long Often due to hepatic dysfunction and fatty
nutrition) associated term use and is usually liver which may occur with long term TPN
jaundice painless with non usage.
obstructive features

Bile duct injury Depending upon the Often due to a difficult laparoscopic•
type of injury may be cholecystectomy when anatomy in Calots
of sudden or gradual triangle is not appreciated. In the worst
onset and is usually of scenario the bile duct is excised and jaundice
obstructive type develops rapidly post operatively. More
insidious is that of bile duct stenosis which
may be caused by clips or diathermy injury.

Cholangiocarcinoma Gradual onset Direct occlusion by disease and also extrinsic


obstructive pattern compression by nodal disease at the porta
hepatis.

Septic surgical patient Usually hepatic Combination of impaired biliary excretion and
features drugs such as ciprofloxacin which may cause
cholestasis.

Metastatic disease Mixed hepatic and post Combination of liver synthetic failure (late)
hepatic and extrinsic compression by nodal disease
and anatomical compression of intra hepatic
structures (earlier)

A gallbladder may develop a thickened wall in chronic cholecystitis, microscopically Roikitansky-


Aschoff Sinuses may be seen
Image sourced from Wikipedia
Theme: Consent

A. Consent the patient


B. Consent the parents
C. Proceed without consent
D. Refer the matter to a court
E. Do not proceed with treatment
F. Consent by proxy

For each of the scenarios given please select the most appropriate course of action. Each option
may be used once, more than once or not at all.

1. A 6 year old is hit by a car and is brought to the emergency department. He is


haemodynamically unstable with bilateral femoral shaft fractures and concerns that he may
have a ruptured spleen. The parents have refused blood transfusions on religious grounds.

You answered Consent the patient

The correct answer is Proceed without consent

In the UK the GMC and common law advises that emergency life saving treatment can be
given to a child irrespective of the parents views. There is clearly insufficient time here to
apply to a court.

2. A 63 year old lady with advanced motor neurone disease is developing progressive feeding
difficulties. Her daughter and the team treating her would like a PEG inserted. However, the
patients partner produces an advanced directive which the patient had signed 15 years
previously and registered with their solicitor at the time the couple wrote their wills. In it
she states she would not wish such treatment.

You answered Consent the patient

The correct answer is Do not proceed with treatment

A properly completed and registered advanced medical directive signed by a mentally


competent adult is legally binding and cannot be overridden.

3. A 15 year old girl presents with right iliac fossa pain. She becomes progressively
tachycardic and a ruptured ectopic pregnancy is suspected. She is deemed to be mentally
competent and will agree to surgery, but not if her parents are informed.

Consent the patient


Since she is likely to be Gillick competent she can consent for herself.

Please rate this question:

Discuss and give feedback


Next question

Consent

There are 3 types of consent:

1. Informed
2. Expressed
3. Implied

Consent forms used in UK NHS


Consent For competent adults who are able to consent for themselves where
Form 1 consciousness may be impaired (e.g. GA)

Consent For an adult consenting on behalf of a child where consciousness is impaired


Form 2

Consent For an adult or child where consciousness is not impaired


Form 3

Consent For adults who lack capacity to provide informed consent


Form 4

Capacity
Key points include:
1. Understand and retain information
2. Patient believes the information to be true
3. Patient is able to weigh the information to make a decision
All patients must be assumed to have capacity

Consent in minors
Young children and older children who are not Gillick competent cannot consent for themselves. In
British law the patients biological mother can always provide consent. The child's father can consent
if the parents are married (and the father is the biological father), or if the father is named on the
birth certificate (irrespective of marital status). If parents are not married and the father is not named
on the birth certificate then the father cannot consent.
Next question
What is the reciprocal of absolute risk reduction?

Odds ratio

Number needed to treat

False positive

False negative

None of the above

Similar theme to September 2011 Exam

In epidemiology, the absolute risk reduction, or risk difference is the decrease in risk of a given
activity or treatment in relation to a control activity or treatment. It is the inverse of the number
needed to treat.
Please rate this question:

Discuss and give feedback


Next question

Absolute risk reduction

The absolute risk reduction is the decrease in risk of a given activity or treatment in relation to a
control activity or treatment. It is the inverse of the number needed to treat.

The absolute risk reduction is usually calculated for two different treatments. For example, consider
surgical resection (X) versus watchful waiting (Y) for prostate cancer. A defined end point, such as 5
year survival is required. If the probabilities pX and pY of this end point are known then the absolute
risk reduction is calculated (pX-pY).

The inverse of absolute risk reduction is the Number Needed to Treat . This is useful in determining
the cost Vs benefit of many treatments.

Number needed to treat


Definition: how many patients would need to receive a treatment to prevent one event. It is the
absolute difference between two treatments.
Next question
You have been asked to investigate the potential benefit of setting up a service to help patients with
stomas in the local area. What is the most important factor when determining how many resources
will be required?

Incidence

Bayesian factor

Prevalence

Denominator data

P value

Please rate this question:

Discuss and give feedback

Next question

Incidence and prevalence

These two terms are used to describe the frequency of a condition in a population.

The incidence is the number of new cases per population in a given time period.

For example, if condition X has caused 40 new cases over the past 12 months per 1,000 of the
population the annual incidence is 0.04 or 4%.

The prevalence is the total number of cases per population at a particular point in time.

For example, imagine a questionnaire is sent to 2,500 adults asking them how much they weigh. If
from this sample population, 500 of the adults were obese then the prevalence of obesity would be
0.2 or 20%.
Relationship

 prevalence = incidence * duration of condition


 in chronic diseases the prevalence is much greater than the incidence
 in acute diseases the prevalence and incidence are similar. For conditions such as the
common cold the incidence may be greater than the prevalence

Next question
In medical statistics, which of the following does a p value of 0.04 represent?

Risk of type 1 Error

Risk of type 2 Error

Size of power of the study

Sample size

Number of degrees of freedom

P values are related to the significance levels of a statistical test and therefore are in effect
measuring the risk of a type 1 error.
Please rate this question:

Discuss and give feedback


Next question

Statistical error

Type 1 Error  Test rejects true null hypothesis


 Rate of type 1 error is the given the value of α
 It usually equals the significance level of a test

Type 2 Error  Test fails to reject a false null hypothesis


 Rate of type 2 errors is given the value of β
 It is related to the power of the test

Next question
Which of the following statements relating to quantitative data is false?

Discrete data cannot be sub divided

The median is less susceptible to extreme outliers than the mean

The mean is susceptible to extreme outliers

Data that fits the standard distribution perfectly will have a mode that is half the value of the
mean

Values obtained have a numerical scale

Data that fits the standard distribution perfectly will have a mean, median and mode that are all the
same value.

Please rate this question:

Discuss and give feedback

Next question

Qualitative and quantitative data

Qualitative and quantitative data


Qualitative (categorical) data refers to different descriptions of a characteristic, although it may be
possible to allocate a number it has no scale.
Quantitative data is associated with numerical values on a numerical scale.

Since quantitative data is based on a numerical scale it can be organised to create a distribution
curve. The central tendency may be estimated using the mode, median and mean. The standard
deviation gives an estimation of the spread of data.

Next question
A surgical team wish to conduct a meta analysis of randomised controlled trials of the use of low
molecular weight heparins in the prevention of post operative deep vein thrombosis. How would
these results be best displayed graphically?

Forest plot

Box Whisker plot

Violin plot

Kaplan Meier graph

None of the above

Data from multiple RCT's are best displayed using Forest plots. Funnel plots may be used to
determine the effect of small studies and their overall effect on the data. Violin plots and Box
Whisker plots are often used to graphically display non parametric data from single studies and are
not generally used to display data from meta analyses.

Please rate this question:

Discuss and give feedback

Next question

Forest plots

A Forest plot is a graphical display designed to illustrate the relative strength of treatment effects in
multiple quantitative scientific studies, addressing the same question. It is often used to graphically
display meta analyses of randomised controlled trials.

The graph may be plotted on a natural logarithmic scale when using odds ratios or other ratio-based
effect measures, so that the confidence intervals are symmetrical about the means from each study
and to ensure undue emphasis is not given to odds ratios greater than 1 when compared to those
less than 1. The area of each square is proportional to the study's weight in the meta-analysis. The
overall meta-analysed measure of effect is often represented on the plot as a vertical line. This
meta-analysed measure of effect is commonly plotted as a diamond, the lateral points of which
indicate confidence intervals for this estimate.

A vertical line representing no effect is also plotted. If the confidence intervals for individual studies
overlap with this line, it demonstrates that at the given level of confidence their effect sizes do not
differ from no effect for the individual study. The same applies for the meta-analysed measure of
effect: if the points of the diamond overlap the line of no effect the overall meta-analysed result
cannot be said to differ from no effect at the given level of confidence.

Next question
A rapid finger-prick blood test to help diagnosis deep vein thrombosis is developed. Comparing the
test to current standard techniques a study is done on 1,000 patients:

DVT present DVT absent

New test positive 200 100

New test negative 20 680

What is the specificity of the new test?

680/880

200/220

680/780

680/700

200/300

Specificity = true negatives / (true negatives + false positives)

= 680 / (680 + 100)

Please rate this question:

Discuss and give feedback

Next question
Screening test statistics

It would be unusual for a medical exam not to feature a question based around screening test
statistics. The available data should be used to construct a contingency table as below:

TP = true positive; FP = false positive; TN = true negative; FN = false negative

Disease present Disease absent

Test positive TP FP

Test negative FN TN

The table below lists the main statistical terms used in relation to screening tests:

Sensitivity TP / (TP + FN ) Proportion of patients with the condition who have a


positive test result

Specificity TN / (TN + FP) Proportion of patients without the condition who


have a negative test result

Positive predictive value TP / (TP + FP) The chance that the patient has the condition if the
diagnostic test is positive

Negative predictive value TN / (TN + FN) The chance that the patient does not have the
condition if the diagnostic test is negative

Likelihood ratio for a sensitivity / (1 - How much the odds of the disease increase when a
positive test result specificity) test is positive

Likelihood ratio for a (1 - sensitivity) / How much the odds of the disease decrease when a
negative test result specificity test is negative
Positive and negative predictive values are prevalence dependent. Likelihood ratios are not
prevalence dependent

Next question
Theme: Statistics

A. LSD post hoc test

B. Bonferroni test

C. Mann Whitney U test

D. Paired T test

E. Chi squared test

F. Fishers exact test

G. Unpaired T Test

Please select the statistical test that is most appropriate for the scenario provided. Each option may
be used once, more than once or not at all.

10. A surgeon has conducted a piece of research and is try to make his data appear interesting for
publication. To do this he is conducting multiple analyses of sub group data using multiple tests.

You answered LSD post hoc test

The correct answer is Bonferroni test

This is a process referred to as "data dredging" and can lead to erroneous results. Post hoc testing
in general can be a problem in research and to try and minimise the potential for error some
advocate the use of the Bonferroni method. This adjusts the test to take account of the number
of tests that have been performed on the data.

11. A surgical unit are conducting a study to determine whether patients who have bowel
preparation have a lower risk of colonic anastomotic leakage than those having none. The
planned sample size is 25.

You answered LSD post hoc test

The correct answer is Fishers exact test


It is likely to be underpowered with the number provided. However, it would be possible to
classify such data into a 2x2 contingency table. However, when the sample size is small the Chi
squared test is not suitable and in these situations the Fishers exact test is used.

12. A surgeon wishes to conduct a national study relating patient weight to the length of inpatient
stay following all major operations.

You answered LSD post hoc test

The correct answer is Unpaired T Test

Weight is likely to be normally distributed and when a large size is used it is possible that this will
be suitable for testing using a parametric method. The T Test is a powerful test providing it is used
correctly and would probably be best suited for analysis of this data set.

Please rate this question:

Discuss and give feedback

Next question

Statistics

Statistics is a topic that generally strikes fear and dread into most surgeons hearts. The MRCS is not
an examination designed to test mathematical skill but the examiners do expect you to have working
knowledge of commonly used tests so that you can appraise the literature properly.

Data types
Before selecting a method of statistical analysis it is imperative that the type of data to be analysed
is correctly categorised. Commonly used terms include nominal, ordinal, interval and continuous.

Term Interpretation

Nominal Data can be allocated a numerical code that is arbitrary. For example allocating people as
Term Interpretation

alive or dead using codes of 0 or 1

Ordinal data Data using numbers that can be used on a scale. Severity of pain is often measured in this
way

Interval Data is measured numerically. However, the zero point is arbitrary


scale

Continuous Data is measured numerically where the numerical value is a real number and may be any
value. Examples include height and weight

Analysing data
Having ascribed the data it is then possible to begin the process of analysis. Nominal data is often
tabulated into categories because of the nature of the underlying data sets. Continuous data may be
displayed graphically often as individual data points. When the sample size is large enough,
continuous data can be analysed to determine the distribution of the data points. Often, but not
always these will be in the form of a gaussian distribution. Determining whether data is normally
distributed or not is key to making sense of the subsequent statistical tests. Parametric tests are
used to test normally distributed data, the T Test is one of the best examples. Data which is not
normally distributed cannot be analysed in this way and a non parametric test must be used.
Examples of such tests include Chi Squared and Mann Whitney U tests. Chi squared tests often
appear in the medical literature. There are some assumptions that are made in relation to Chi
squared tests; these include the need to use 2 degrees of freedom (usually) and the minimum
sample size. Where the sample size is small then a different test is appropriate and the Fishers
exact test is often used.
In situations where data is normally distributed and paired samples are taken from the same
individuals (such as following an intervention) then the paired T Test may be used.

Multiple testing and post hoc analysis


In the ideal world statistical analysis is conducted on data that is collected prospectively according to
pre set power calculations and defined end points. Occasionally, data does not produce an expected
outcome or a certain type of patient appears to have a different result. Subsequent analysis of such
groups is termed a post hoc analysis. This can be perfectly legitimate, alternatively it can represent
the last ditch attempt of a researcher to try and find any aspect of the data that is worthwhile. This
can lead to errors and false rejection of a null hypothesis. A statistically significant result is more
likely to occur if the same dataset is subjected to multiple analyses. To counteract this problem some
researchers will apply a Bonferroni correction, this adjusts the analysis to allow for multiple testing.
A new test to screen for pulmonary embolism (PE) is used in 100 patients who present to the
Emergency Department. The test is positive in 30 of the 40 patients who are proven to have a PE.
Of the remaining 60 patients, only 5 have a positive test. What is the sensitivity of the new test?

8.33%

30%

40%

66.66%

75%

A contingency table can be constructed from the above data, as shown below:

{PE diagnosed}{No PE} {Test positive}305 {Test negative}1055


The sensitivity is therefore 30 / (30 + 10) = 75%

Please rate this question:

Discuss and give feedback

Next question

Screening test statistics

It would be unusual for a medical exam not to feature a question based around screening test
statistics. The available data should be used to construct a contingency table as below:

TP = true positive; FP = false positive; TN = true negative; FN = false negative


Disease present Disease absent

Test positive TP FP

Test negative FN TN

The table below lists the main statistical terms used in relation to screening tests:

Sensitivity TP / (TP + FN ) Proportion of patients with the condition who have a


positive test result

Specificity TN / (TN + FP) Proportion of patients without the condition who


have a negative test result

Positive predictive value TP / (TP + FP) The chance that the patient has the condition if the
diagnostic test is positive

Negative predictive value TN / (TN + FN) The chance that the patient does not have the
condition if the diagnostic test is negative

Likelihood ratio for a sensitivity / (1 - How much the odds of the disease increase when a
positive test result specificity) test is positive

Likelihood ratio for a (1 - sensitivity) / How much the odds of the disease decrease when a
negative test result specificity test is negative

Positive and negative predictive values are prevalence dependent. Likelihood ratios are not
prevalence dependent

Next question
Theme: Consent processes

A. Consent form 1 (competent adult)

B. Consent form 2 (procedure on a child)

C. Consent form 3 (procedure on a child or adult where conciousness not


impaired)

D. Consent form 4 (Undertaking procedure where patient lacks capacity)

E. Verbal consent will suffice

F. Research consent form

Please select the most appropriate consent modality for the procedure described. Each option may
be used once, more than once or not at all

14. A 65 year old patient has a wound which is discharging following a laparotomy. The wound is
closed with clips and underlying this is a small superficial collection measuring 2 x 3cm.

You answered Consent form 1 (competent adult)

The correct answer is Verbal consent will suffice

This is done on the ward and is a straightforward procedure. Written consent is not required.

15. An 18 year old male has appendicitis and requires an appendicectomy.

Consent form 1 (competent adult)

The patient is 18 so technically an adult where consent is concerned.

16. An 85 year old lady with severe vascular dementia is admitted from a nursing home with a
displaced femoral neck fracture and it is proposed to undertake a hemi arthroplasty.

You answered Consent form 1 (competent adult)


The correct answer is Consent form 4 (Undertaking procedure where patient lacks capacity)

Where an individual lacks capacity it is necessary to act in the patients best interest. In the case of
a femoral neck fracture this can be managed using form 4. Where treatment is more controversial
and less immediately required an IMCA can be considered (under UK law).

Please rate this question:

Discuss and give feedback

Next question

Consent

There are 3 types of consent:

1. Informed
2. Expressed
3. Implied

Consent forms used in UK NHS

Consent Form For competent adults who are able to consent for themselves where consciousness may
1 be impaired (e.g. GA)

Consent Form For an adult consenting on behalf of a child where consciousness is impaired
2

Consent Form For an adult or child where consciousness is not impaired


3

Consent Form For adults who lack capacity to provide informed consent
4
Capacity
Key points include:
1. Understand and retain information
2. Patient believes the information to be true
3. Patient is able to weigh the information to make a decision
All patients must be assumed to have capacity

Consent in minors
Young children and older children who are not Gillick competent cannot consent for themselves. In
British law the patients biological mother can always provide consent. The child's father can consent
if the parents are married (and the father is the biological father), or if the father is named on the
birth certificate (irrespective of marital status). If parents are not married and the father is not named
on the birth certificate then the father cannot consent.

Next question
Which one of the following statements best describes a type II statistical error?

The p value fails to reach statistical significance

The alternative hypothesis is rejected when it is false

The null hypothesis is rejected when it is true

The null hypothesis is accepted when it is false

None of the above

Please rate this question:

Discuss and give feedback


Next question

Significance tests

A null hypothesis (H0) states that two treatments are equally effective (and is hence negatively
phrased). A significance test uses the sample data to assess how likely the null hypothesis is to be
correct.

For example:

 'there is no difference in the prevalence of colorectal cancer in patients taking low-dose


aspirin compared to those who are not'

The alternative hypothesis (H1) is the opposite of the null hypothesis, i.e. There is a difference
between the two treatments

The p value is the probability of obtaining a result by chance at least as extreme as the one that was
actually observed, assuming that the null hypothesis is true. It is therefore equal to the chance of
making a type I error (see below).

Two types of errors may occur when testing the null hypothesis

 type I: the null hypothesis is rejected when it is true - i.e. Showing a difference between two
groups when it doesn't exist, a false positive. This is determined against a preset significance
level (termed alpha). As the significance level is determined in advance the chance of
making a type I error is not affected by sample size. It is however increased if the number of
end-points are increased. For example if a study has 20 end-points it is likely one of these
will be reached, just by chance.
 type II: the null hypothesis is accepted when it is false - i.e. Failing to spot a difference when
one really exists, a false negative. The probability of making a type II error is termed beta. It
is determined by both sample size and alpha

Study accepts H0 Study rejects H0

Reality H0 Type 1 error (alpha)

Reality H1 Type 2 error (beta) Power (1 - beta)

The power of a study is the probability of (correctly) rejecting the null hypothesis when it is false

 power = 1 - the probability of a type II error


 power can be increased by increasing the sample size

Next question
As part of a research project you are trying to ascertain whether the use of dummies in infants is
linked to sudden infant death syndrome. What is the most appropriate form of study design?

Randomised controlled trial

Cross-over trial

Cross-sectional survey

Case-control study

Cohort study

As sudden infant death syndrome is relatively rare a case-control design is more appropriate than a
cohort study.

Please rate this question:

Discuss and give feedback

Next question

Study design

The following table highlights the main features of the main types of study:

Randomised Participants randomly allocated to intervention or control group (e.g. standard


controlled trial treatment or placebo)

Practical or ethical problems may limit use


Cohort study Observational and prospective. Two (or more) are selected according to their
exposure to a particular agent (e.g. medicine, toxin) and followed up to see how
many develop a disease or other outcome.

The usual outcome measure is the relative risk.

Examples include Framingham Heart Study

Case-control Observational and retrospective. Patients with a particular condition (cases) are
study identified and matched with controls. Data is then collected on past exposure to a
possible causal agent for the condition.

The usual outcome measure is the odds ratio.

Inexpensive, produce quick results


Useful for studying rare conditions
Prone to confounding

Cross-sectional Provide a 'snapshot', sometimes called prevalence studies


survey
Provide weak evidence of cause and effect

Next question
Which of the following has the greatest impact on the positive predictive value of a test?

Prevalence

Subjects who are true negatives

Specificity

Relative risk

None of the above

The positive predictive value (PPV) is the probability that an individual with a positive screening
result has the disease. The sensitivity is the probability that an individual with the disease is
screened positive and the specificity is the probability that an individual without the disease is
screened negative.
Its value depends upon the prevalence of the condition being tested for and the sensitivity of the test
used.
It may be calculated by dividing the number of true positives by the number of true positives and the
number of false positives.
Please rate this question:

Discuss and give feedback


Next question

Positive predictive values

Screening tests

 Sensitivity: proportion of true positives identified by a test


 Specificity: proportion of true negatives correctly identified by a test
 Positive predictive value: proportion of those who have a positive test who actually have the
disease
 Negative predictive value: proportion of those who test negative who do not have the disease

Predictive values are dependent on the prevalence

 Likelihood ratio for a positive test result = sensitivity/(1-specificity)


 Likelihood ratio for a negative test result = (1-sensitivity)/specificity

Likelihood ratios are not prevalence dependent


Next question
Considering cluster randomised trials, which of the following statements is false?

They consider interventions targeted at groups

They require increased recruitment to achieve the same level of statistical power as
individual trials

If results are analysed on an individual basis a lower P value may be obtained

They are less prone to unit of analyses errors than trials involving individual observations

The statistical analyses for these trials is more complex than that required for trials based
on individuals

Cluster randomised trials are more prone to unit of analyses errors than individual based trials.
Clustering needs to be considered in trial design and data analysis. One of the commonest errors is
where a study is a cluster study but researchers have failed to recognise this fact. This will then
result in the incorrect analysis being pursued. A lower P value will then result and a false positive
error will occur.
Please rate this question:

Discuss and give feedback


Next question

Cluster randomised controlled trials

 Groups are randomised rather than individuals


 Avoids cross contamination amongst participants
 Participants in any one cluster are more likely to respond in a similar fashion
 Higher risk of unit of analysis error as these studies should be analysed as clusters rather
than on an individual basis. This leads to a higher false positive rate.
 It is possible to adjust for clustering in statistical analyses

Next question
Theme: Timing of surgery

A. Immediate surgery
B. Surgery within 2 hours
C. Surgery within 6 hours
D. Surgery within 24 hours
E. Surgery within same hospital admission
F. Urgent elective surgery within 4 weeks
G. True elective surgery

For each procedure please select the most appropriate time interval for surgery. Each option may be
selected once, more than once or not at all.

21. A 43 year old women is admitted with acute cholecystitis, her USS confirms the diagnosis
and LFT's are normal. It is now 10 hours since admission.

You answered Immediate surgery

The correct answer is Surgery within 24 hours

Ideal case for acute cholecystectomy. This will enable prompt discharge and facilitate
recovery. Whilst expedient surgery is desirable an emergency procedure is not justified.

22. A 5 year old boy is admitted with a suspected acute appendicitis. He has tenderness but no
guarding as yet. He requires appendicectomy.

You answered Surgery within 2 hours

The correct answer is Surgery within 6 hours

The kind of case that can wait till the following day if presenting out of hours.
Appendicectomy may be deferred where peritoneal signs are absent. Where tenderness and
guarding are present a more urgent approach is warranted.

23. A 72 year old man is admitted with large bowel obstruction. He has been vomiting for 24
hours and his caecum is tender and measures 11cm.

You answered Surgery within 2 hours

The correct answer is Surgery within 6 hours

The sun should not rise and set on unrelieved large bowel obstruction! This patient has a
competent ileocaecal valve. As a result lack of surgery would result in caecal perforation
leading to faecal peritonitis with and associated high mortality rate.

Please rate this question:

Discuss and give feedback


Next question

Preparation for surgery

Elective and emergency patients require different preparation.

Elective cases

 Consider pre admission clinic to address medical issues.


 Blood tests including FBC, U+E, LFT's, Clotting, Group and Save
 Urine analysis
 Pregnancy test
 Sickle cell test
 ECG/ Chest x-ray

Exact tests to be performed will depend upon the proposed procedure and patient fitness.

Risk factors for development of deep vein thrombosis should be assessed and a plan for
thromboprophylaxis formulated.

Diabetes
Diabetic patients have greater risk of complications.
Poorly controlled diabetes carries high risk of wound infections.
Patients with diet or tablet controlled diabetes may be managed using a policy of omitting medication
and checking blood glucose levels regularly. Diabetics who are poorly controlled or who take insulin
will require a intravenous sliding scale. Potassium supplementation should also be given.
Diabetic cases should be operated on first.

Emergency cases
Stabilise and resuscitate where needed.
Consider whether antibiotics are needed and when and how they should be administered.
Inform blood bank if major procedures planned particularly where coagulopathies are present at the
outset or anticipated (e.g. Ruptured AAA repair)
Don't forget to consent and inform relatives.

Special preparation
Some procedures require special preparation:

 Thyroid surgery; vocal cord check.


 Parathyroid surgery; consider methylene blue to identify gland.
 Sentinel node biopsy; radioactive marker/ patent blue dye.
 Surgery involving the thoracic duct; consider administration of cream.
 Pheochromocytoma surgery; will need alpha and beta blockade.
 Surgery for carcinoid tumours; will need covering with octreotide.
 Colorectal cases; bowel preparation (especially left sided surgery)
 Thyrotoxicosis; lugols iodine/ medical therapy.

References
Management of adults with diabetes undergoing surgery and elective procedures. NHS Diabetes.
April 2011.
Next question
In a randomized study of chemotherapy drugs for bowel cancer, a group receiving treatment A had a
recurrence rate of 12.5% and a group receiving treatment B had a recurrence rate of 15%. Both
groups are matched for size and length of follow up. What is the number needed to treat to prevent a
recurrence?

2.5

25

40

There is an absolute risk reduction of 15-12.5%= 2.5% for treatment A


Therefore the NNT = 1/0.025 = 40

Please rate this question:

Discuss and give feedback

Next question

Absolute risk reduction

The absolute risk reduction is the decrease in risk of a given activity or treatment in relation to a
control activity or treatment. It is the inverse of the number needed to treat.

The absolute risk reduction is usually calculated for two different treatments. For example, consider
surgical resection (X) versus watchful waiting (Y) for prostate cancer. A defined end point, such as 5
year survival is required. If the probabilities pX and pY of this end point are known then the absolute
risk reduction is calculated (pX-pY).

The inverse of absolute risk reduction is the Number Needed to Treat . This is useful in determining
the cost Vs benefit of many treatments.

Number needed to treat


Definition: how many patients would need to receive a treatment to prevent one event. It is the
absolute difference between two treatments.

Next question
A new blood test to screen patients for colorectal cancer is trialled on 500 patients. The test was
positive in 40 of the 50 patients shown to have colorectal cancer by colonscopy. It was also positive
in 20 patients who were shown not to have colorectal cancer. What is the positive predictive value of
the test?

0.8

0.66

0.33

0.1

Cannot be calculated

A contingency table can be constructed from the above data, as shown below:

Colorectal cancer No colorectal cancer

Test positive 40 20

Test negative 10 430

Positive predictive value = TP / (TP + FP) = 40 / (40 + 20) = 0.66

Please rate this question:

Discuss and give feedback

Next question
Screening test statistics

It would be unusual for a medical exam not to feature a question based around screening test
statistics. The available data should be used to construct a contingency table as below:

TP = true positive; FP = false positive; TN = true negative; FN = false negative

Disease present Disease absent

Test positive TP FP

Test negative FN TN

The table below lists the main statistical terms used in relation to screening tests:

Sensitivity TP / (TP + FN ) Proportion of patients with the condition who have a


positive test result

Specificity TN / (TN + FP) Proportion of patients without the condition who


have a negative test result

Positive predictive value TP / (TP + FP) The chance that the patient has the condition if the
diagnostic test is positive

Negative predictive value TN / (TN + FN) The chance that the patient does not have the
condition if the diagnostic test is negative

Likelihood ratio for a sensitivity / (1 - How much the odds of the disease increase when a
positive test result specificity) test is positive

Likelihood ratio for a (1 - sensitivity) / How much the odds of the disease decrease when a
negative test result specificity test is negative
Positive and negative predictive values are prevalence dependent. Likelihood ratios are not
prevalence dependent

Next question
Which of the following statements relating to randomised controlled trials is false?

Consist of a control group recruited during the same time interval as the treatment group.

Are not applicable to retrospectively analysed data even if captured on a prospectively created
database.

They require concealment of treatment throughout the duration of the study.

They require concealment of treatment until after randomisation.

They are less susceptible to researcher bias than non-randomised controlled trials.

An RCT does not have to include concealment although many medical trials may do so. Indeed in
the case of surgical research it may not be practicable or possible to include concealment in the
protocol. This does not mean that the trial is not an RCT, simply that it is not blinded.

Please rate this question:

Discuss and give feedback

Next question

Audit and Research

Clinical audit
Quality improvement process that seeks to improve patient care and outcomes through systematic
review of care against explicit criteria and the implementation of change. Aspects of the structure,
processes, and outcomes of care are selected and systematically evaluated against explicit criteria.
Where indicated, changes are implemented at an individual, team, or service level and further
monitoring is used to confirm improvement in healthcare delivery. (NICE).

Research
Aims to derive new knowledge which is potentially generalisable or transferable.

Next question
Which of the following most closely describes the risk of a type I statistical error?

Power calculation

P value

Odds ratio

Relative risk

None of the above

Type 1 errors occur when a test rejects a true null hypothesis and is therefore related to the
significance level of the test result. To explain consider the following arbitrary example.

We hypothesise that bowel preparation vs no bowel preparation has no effect on anastomotic leak
rates following left hemicolectomy. If we compare the rates of anastomotic leak and perform a Chi
Squared test and obtained a P value of 0.95 we should conclude that we unable to reject the null
hypothesis. Should we choose to do so then we are at risk of committing a type 1 error. In reality the
knowledge that a type 1 error was committed is usually some time after the event. When other
studies have been performed that have shown an effect.
Power calculations are related to type 2 errors.

Please rate this question:

Discuss and give feedback

Next question

Power calculations and statistical error

Statistical error

Type 1 A test rejects a true null hypothesis. Analogus to false positive. It usually equates to the
Error significance level assigned to a test.

Type 2 A test fails to reject a false null hypothesis. It is related to the power of a test.
Error

Statistical power
The power of a test is the probability that the test will reject the null hypothesis when it is false
(thereby avoiding a type 2 error). Increasing the power of a test will reduce the probability of a type 2
error. Usually a value of 0.8 is selected.

Next question
A cohort study is being designed to look at the relationship between smoking and breast cancer.
What is the usual outcome measure in a cohort study?

Odds ratio

Experimental event rate

Relative risk

Absolute risk increase

Numbers needed to harm

Cohort studies - relative risk

Please rate this question:

Discuss and give feedback

Next question

Study design

The following table highlights the main features of the main types of study:

Randomised Participants randomly allocated to intervention or control group (e.g. standard


controlled trial treatment or placebo)

Practical or ethical problems may limit use


Cohort study Observational and prospective. Two (or more) are selected according to their
exposure to a particular agent (e.g. medicine, toxin) and followed up to see how
many develop a disease or other outcome.

The usual outcome measure is the relative risk.

Examples include Framingham Heart Study

Case-control Observational and retrospective. Patients with a particular condition (cases) are
study identified and matched with controls. Data is then collected on past exposure to a
possible causal agent for the condition.

The usual outcome measure is the odds ratio.

Inexpensive, produce quick results


Useful for studying rare conditions
Prone to confounding

Cross-sectional Provide a 'snapshot', sometimes called prevalence studies


survey
Provide weak evidence of cause and effect

Next question
Which of the following statements relating to consenting patients for surgery is false?

Consent should be taken by a person who has sufficient knowledge of the procedure

All risks with a frequency of 1 in 500 or greater must be disclosed

Patients who have received sedating pre medication may no longer be able to provide informed
consent

Written consent is required for operative procedures performed under local anaesthesia

Where a procedure (or part thereof) consists of research this should be recorded on a separate
research consent form

Generally risks with an incidence of 1% or greater are disclosed. Exceptions to this are where a rarer
complication is particularly serious.

Please rate this question:

Discuss and give feedback

Next question

Consent

There are 3 types of consent:

1. Informed
2. Expressed
3. Implied

Consent forms used in UK NHS

Consent Form For competent adults who are able to consent for themselves where consciousness may
1 be impaired (e.g. GA)

Consent Form For an adult consenting on behalf of a child where consciousness is impaired
2

Consent Form For an adult or child where consciousness is not impaired


3

Consent Form For adults who lack capacity to provide informed consent
4

Capacity
Key points include:
1. Understand and retain information
2. Patient believes the information to be true
3. Patient is able to weigh the information to make a decision
All patients must be assumed to have capacity

Consent in minors
Young children and older children who are not Gillick competent cannot consent for themselves. In
British law the patients biological mother can always provide consent. The child's father can consent
if the parents are married (and the father is the biological father), or if the father is named on the
birth certificate (irrespective of marital status). If parents are not married and the father is not named
on the birth certificate then the father cannot consent.

Next question
A group of surgeons conduct a meta analysis of randomised controlled trials comparing the use of
analgesic regimes following laparoscopic cholecystectomy. What level of evidence is provided by
such an analysis?

II

III

IV

Please rate this question:

Discuss and give feedback


Next question

Study design: evidence and recommendations

Levels of evidence

 I - evidence from meta-analysis of randomised controlled trials


 II - evidence from at least one well designed controlled trial which is not randomised
 III - evidence from correlation and comparative studies or use of historical controls
 IV - evidence from case series
 V - Expert opinion or founded on basic principles

Knowledge of the sub groups of the levels of evidence are not routinely tested in MRCS Part A.

Grading of recommendation

 Grade A - based on evidence from at least one randomised controlled trial (i.e. Ia or Ib)
 Grade B - based on evidence from non-randomised controlled trials (i.e. IIa, IIb or III)
 Grade C - based on evidence from a panel of experts (i.e. IV)

Next question
Theme: Audit

A. Standards based audit

B. Departmental review

C. Systems based audit

D. Operational audit

E. Financial audit

F. Peer review

Please select the most appropriate type of audit method for the situation described. Each option may
be used once, more than once or not at all.

31. A surgical department wishes to determine whether it is using types of prosthetic mesh material
for incisional hernia surgery in the most effective manner. Recently there have been cases of non
mesh usage and loss of material as a result of the implants being "out of date".

You answered Standards based audit

The correct answer is Systems based audit

Theme from April 2015 Exam


Theme from April 2013 Exam
This is primarily an issue of stock control. However, the system by which the materials are used
within the theatre will need evaluation. Because it is the usage and stock that are a problem,
rather than the sourcing the systems based audit will be more effective than an operational audit.

32. A group of surgeons wish to determine whether patients are receiving adequate deep vein
thrombosis prophylaxis following surgery.

Standards based audit

This type of audit is widely undertaken in most trusts in the UK.


33. Surgeons are becoming increasing concerned about the adverse results of Mrs X performing a
new an innovative operative procedure not widely practised elsewhere.

You answered Standards based audit

The correct answer is Peer review

In the situation where a surgeon performs an unfamiliar procedure a peer review is often the best
way to evaluate the problem. This does not have to be externally based, but often is.

Please rate this question:

Discuss and give feedback

Next question

Audit categories

Audits may be used in a variety of clinical settings. These range from standards based audits, which
will be familiar to most clinicians, through to systems based audits which focus more on the
processes within an organisation.

Types of audit

Financial audit A historically oriented, independent evaluation performed for the purpose of attesting
to the fairness, accuracy, and reliability of financial data

Operational A future-oriented, systematic, and independent evaluation of organizational activities.


audit Financial data may be used, but the primary sources of evidence are the operational
policies and achievements related to organizational objectives. Internal controls and
efficiencies may be evaluated during this type of review.

Departmental A current period analysis of administrative functions, to evaluate the adequacy of


controls, safeguarding of assets, efficient use of resources, compliance with related
review laws, regulations and institutional policy and integrity of financial information.

Standards Comparison of care or passage of care against set and widely agreed standards or
based audit outcomes.

Systems based Evaluation of processes occurring within an institution.


audit

Systems based audits are an integral part of the process of clinical governance.

Next question
Theme: Statistics in surgery

A. Mann Whitney U test

B. Analysis of variance

C. LSR post hoc test

D. Bonferroni test

E. Kruskall Wallis test

F. T Test

Please select the most appropriate statistical test for the situation described. Each option may be
used once, more than once or not at all.

34. We wish to determine whether there are significantly more patients in a surgical unit presenting
with post appendicectomy wound complications than there were one year previously. Review of
the dataset suggests a normal distribution.

You answered Mann Whitney U test

The correct answer is T Test

This will involve the comparison of absolute numbers of patients and therefore this can be
assessed using a T -Test. It does make the assumption that the data is normally distributed.
However, the other tests would not be suitable.

35. We want to make multiple comparisons of different types of side effects of a new drug.

You answered Mann Whitney U test

The correct answer is Bonferroni test

As more types of side effects are considered, it becomes more likely that the new drug will appear
to be less safe than existing drugs in terms of at least one side effect. Methods are available to
adjust the p value to reflect the multiple comparisons being made, the aim being to avoid
spurious results. A frequently applied correction is the Bonferroni Method in which the observed
p values are multiplied by the number of tests performed, any resulting p value which is greater
than 1 is set to 1 and any which remains at less than 0.05 can be considered significant at the 5%
level.

36. 5 surgeons in a colorectal unit wish to determine whether there is a significant difference in their
individual leak rates for anterior resection of the rectum.

You answered Mann Whitney U test

The correct answer is Kruskall Wallis test

In this scenario the data is derived from 5 groups of surgeons. If the data were normally
distributed then an ANOVA could be considered. Since these assumptions cannot be met, or
satisfied by transforming the data then the Kruskall-Wallis test provides a non parametric
alternative. This is essentially an extension of the Wilcoxon Rank sum test and detects differences
in median values between each group. To compare more accurately differences between two
individual surgeons a Mann Whitney U test may be a more acceptable alternative.

Please rate this question:

Discuss and give feedback

Next question

Qualitative and quantitative data

Qualitative and quantitative data


Qualitative (categorical) data refers to different descriptions of a characteristic, although it may be
possible to allocate a number it has no scale.
Quantitative data is associated with numerical values on a numerical scale.

Since quantitative data is based on a numerical scale it can be organised to create a distribution
curve. The central tendency may be estimated using the mode, median and mean. The standard
deviation gives an estimation of the spread of data.

Next question
Theme: Governance issues

A. Clinical audit

B. Service evaluation

C. Refer to research ethics committee

D. Implement procedure with no further monitoring

E. Cease activity and refer individual to GMC

F. Cease activity and undertake full service evaluation

G. Obtain written consent from each participant

For the following research/ audit scenarios please select the most appropriate governance modality.
Each option may be used once, more than once or not at all.

37. A general surgical unit has become increasingly concerned about the behaviour of consultant
Slasher. Over the past 48 months he has persisted in performing neonatal tracheoesphageal
fistula repairs. Unfortunately he has resisted efforts to prevent him from undertaking these
unsupervised. 2 more babies die and the Chief Executive would like your guidance.

You answered Clinical audit

The correct answer is Cease activity and refer individual to GMC

While most surgical and departmental problems can be handled in house is can be seen that this
approach has been tried and failed. Given the deaths there is no other option than E

38. As the SpR in general surgery you wish to determine whether your breast cancer unit is complying
with the British Association of Surgical Oncology guidelines for management of high grade ductal
carcinoma in situ

Clinical audit

Where there are clear guidelines, an audit is the best measure.


39. A surgeon wishes to undertake some laboratory research into the migratory behaviour patterns
of metastatic colorectal cancer cells. These will be harvested from patients who are undergoing
hepatic resection of metastatic colorectal cancer; apart from diseased tissue no other samples
will be taken.

You answered Clinical audit

The correct answer is Refer to research ethics committee

Whenever patient tissue is taken for research it is necessary practice to gain ethics approval.
Some units may have blanket policies in place for taking tissue for research to tissue banks but as
a general rule most people should seek ethics approval PRIOR to starting research.

Please rate this question:

Discuss and give feedback

Next question

Audit and Research

Clinical audit
Quality improvement process that seeks to improve patient care and outcomes through systematic
review of care against explicit criteria and the implementation of change. Aspects of the structure,
processes, and outcomes of care are selected and systematically evaluated against explicit criteria.
Where indicated, changes are implemented at an individual, team, or service level and further
monitoring is used to confirm improvement in healthcare delivery. (NICE).

Research
Aims to derive new knowledge which is potentially generalisable or transferable.

Next question
Which one of the following is equivalent to the pre-test probability?

Post test odds / (1 + post-test odds)

Pre-test odds x likelihood ratio

The prevalence of a condition

The incidence of a condition

Post-test odds / likelihood ratio

The prevalence is the proportion of a population that have the condition at a point in time whilst the
incidence is the rate at which new cases occur in a population during a specified time period.

Please rate this question:

Discuss and give feedback

Next question

Pre- and post- test odds and probability

Pre-test probability
The proportion of people with the target disorder in the population at risk at a specific time (point
prevalence) or time interval (period prevalence)

For example, the prevalence of rheumatoid arthritis in the UK is 1%

Post-test probability
The proportion of patients with that particular test result who have the target disorder

Post-test probability = post test odds / (1 + post-test odds)

Pre-test odds
The odds that the patient has the target disorder before the test is carried out

Pre-test odds = pre-test probability / (1 - pre-test probability)

Post-test odds
The odds that the patient has the target disorder after the test is carried out

Post-test odds = pre-test odds x likelihood ratio

where the likelihood ratio for a positive test result = sensitivity / (1 - specificity)

Next question
You are performing a study of weight in patients attending pre operative assessment clinic for
elective knee replacement. Assuming that the results are normally distributed, what percentage of
values lie within two standard deviations of the mean weight?

95.4%

5.3%

98.3%

10%

97.5%

In statistics, the 68-95-99.7 rule or three-sigma rule, or empirical rule states that for a normal
distribution, nearly all values lie within 3 standard deviations of the mean.
About 68.27% of the values lie within 1 standard deviation of the mean. Similarly, about 95.45% of
the values lie within 2 standard deviations of the mean. Nearly all (99.73%) of the values lie within 3
standard deviations of the mean

95.4% of values lie within 2 SD of the mean.


Please rate this question:

Discuss and give feedback


Next question

Normal distribution

The normal distribution is also known as the Gaussian distribution or 'bell-shaped' distribution. It
describes the spread of many biological and clinical measurements

Properties of the Normal distribution

 symmetrical i.e. Mean = mode = median


 68.3% of values lie within 1 SD of the mean
 95.4% of values lie within 2 SD of the mean
 99.7% of values lie within 3 SD of the mean
 this is often reversed, so that within 1.96 SD of the mean lie 95% of the sample values
 the range of the mean - (1.96 *SD) to the mean + (1.96 * SD) is called the 95% confidence
interval, i.e. If a repeat sample of 100 observations are taken from the same group 95 of
them would be expected to lie in that range
Standard deviation

 the standard deviation (SD) represents the average difference each observation in a sample
lies from the sample mean
 SD = square root (variance)

Next question
In Intention to treat analysis, which statement is untrue?

It is based on the initial treatment intended.

Excludes those who cross over to alternative treatment arms of a trial.

Is different from a per protocol analysis.

Will affect the statistical power calculation to compare outcomes of treatment.

Helps to minimise observer bias.

An intention to treat analysis considers data according to the treatment to which the patient was
randomised, rather than the treatment which was recieved.

It includes those who cross over which is how it helps provide additional information relating to those
groups.

Please rate this question:

Discuss and give feedback

Next question

Audit and Research

Clinical audit
Quality improvement process that seeks to improve patient care and outcomes through systematic
review of care against explicit criteria and the implementation of change. Aspects of the structure,
processes, and outcomes of care are selected and systematically evaluated against explicit criteria.
Where indicated, changes are implemented at an individual, team, or service level and further
monitoring is used to confirm improvement in healthcare delivery. (NICE).

Research
Aims to derive new knowledge which is potentially generalisable or transferable.
Which of the following is not usually required to make a power calculation?

The specificity of the intervention being tested

The desired significance level

The size of the effect being measured

The desired power value

Sample size used to detect the effect

The specificity of a test is related to type 1 errors.

The components that are nearly always needed for power calculations are :
Size of effect
Significance level
Sample size used to detect the effect
Desired power value

Please rate this question:

Discuss and give feedback

Next question

Power calculations and statistical error

Statistical error

Type 1 A test rejects a true null hypothesis. Analogus to false positive. It usually equates to the
Error significance level assigned to a test.
Type 2 A test fails to reject a false null hypothesis. It is related to the power of a test.
Error

Statistical power
The power of a test is the probability that the test will reject the null hypothesis when it is false
(thereby avoiding a type 2 error). Increasing the power of a test will reduce the probability of a type 2
error. Usually a value of 0.8 is selected.

Next question
A new hernia mesh designed to prevent the risk of infection undergoes clinical trials. One hundred
patients are given the new mesh. During a three month period 10 of the patients have an episode of
infection. In the control group there are 300 patients who are given a placebo. In this group 50
people have an infection during the same time period. What is the relative risk of having an infection
when the new mesh is used?

0.8

0.2

1.66

0.6

0.06

Experimental event rate, EER = 10 / 100 = 0.10

Control event rate, CER = 50 / 300 = 0.166

Therefore the relative risk = EER / CER = 0.1 / 0.166 = 0.6


Please rate this question:

Discuss and give feedback


Next question

Relative risk

Relative risk (RR) is the ratio of risk in the experimental group (experimental event rate, EER) to
risk in the control group (control event rate, CER)

To recap

 EER = rate at which events occur in the experimental group


 CER = rate at which events occur in the control group

For example, if we look at a trial comparing the use of paracetamol for back pain compared to
placebo we may get the following results
Total number of patients Experienced significant pain relief

Paracetamol 100 60

Placebo 80 20

Experimental event rate, EER = 60 / 100 = 0.6

Control event rate, CER = 20 / 80 = 0.25

Therefore the relative risk = EER / CER = 0.6 / 0.25 = 2.4

If the risk ratio is > 1 then the rate of an event (in this case experiencing significant pain relief) is
increased compared to controls. It is therefore appropriate to calculate the relative risk increase if
necessary (see below).

If the risk ratio is < 1 then the rate of an event is decreased compared to controls. The relative risk
reduction should therefore be calculated (see below).

Relative risk reduction (RRR) or relative risk increase (RRI) is calculated by dividing the absolute
risk change by the control event rate

Using the above data, RRI = (EER - CER) / CER = (0.6 - 0.25) / 0.25 = 1.4 = 140%
Next question
Which of the following statements relating to audit and governance is untrue?

An audit standard is a threshold of compliance with an audit criterion

Sample size calculations are an important part of audit planning

Clinical audit is part of clinical governance

Audits should be performed regularly when a novel surgical technique is introduced and where
there is little knowledge of anticipated complications or outcomes

An audit criterion is a measurable outcome of care, aspect of practice or capacity

6 pillars of clinical governance:


Clinical effectiveness
Research and development
Openness
Risk management
Education and training
Clinical audit

Audits should compare performance against known standards. Where a novel technique is being
introduced standards are unlikely to exist, sample sizes cannot therefore be accurately calculated.
This is an example of research, which is not an audit.

Please rate this question:

Discuss and give feedback

Next question

Audit and Research

Clinical audit
Quality improvement process that seeks to improve patient care and outcomes through systematic
review of care against explicit criteria and the implementation of change. Aspects of the structure,
processes, and outcomes of care are selected and systematically evaluated against explicit criteria.
Where indicated, changes are implemented at an individual, team, or service level and further
monitoring is used to confirm improvement in healthcare delivery. (NICE).

Research
Aims to derive new knowledge which is potentially generalisable or transferable.

Next question
Which of the following statements relating to qualitative data is false?

The data has no true numerical scale

It may comprise multiple data groups

May be reported using odds ratios

May be reported using frequency histograms

It is best analysed statistically using a students T test when multiple factors are present

The students T test should be performed if two sets of data have a normal distribution, the T test
cannot be used to analyse multiple data sets.

Please rate this question:

Discuss and give feedback

Next question

Qualitative and quantitative data

Qualitative and quantitative data


Qualitative (categorical) data refers to different descriptions of a characteristic, although it may be
possible to allocate a number it has no scale.
Quantitative data is associated with numerical values on a numerical scale.

Since quantitative data is based on a numerical scale it can be organised to create a distribution
curve. The central tendency may be estimated using the mode, median and mean. The standard
deviation gives an estimation of the spread of data.

Next question
Which of the following statements is false in relation to consent?

All adults by law are assumed to be competent

The Bolam test defines if a patient has capacity or not

Consent 2 is the form signed by parents on behalf of their children

Implied consent is a form of consent

Consent 4 is the form signed when a patient is unable to consent for a treatment or investigation

The Bolam test defines if a decision made by a doctor is in agreement with the professional standard
of medical practise.

Please rate this question:

Discuss and give feedback

Consent

There are 3 types of consent:

1. Informed
2. Expressed
3. Implied

Consent forms used in UK NHS

Consent Form For competent adults who are able to consent for themselves where consciousness may
1 be impaired (e.g. GA)
Consent Form For an adult consenting on behalf of a child where consciousness is impaired
2

Consent Form For an adult or child where consciousness is not impaired


3

Consent Form For adults who lack capacity to provide informed consent
4

Capacity
Key points include:
1. Understand and retain information
2. Patient believes the information to be true
3. Patient is able to weigh the information to make a decision
All patients must be assumed to have capacity

Consent in minors
Young children and older children who are not Gillick competent cannot consent for themselves. In
British law the patients biological mother can always provide consent. The child's father can consent
if the parents are married (and the father is the biological father), or if the father is named on the
birth certificate (irrespective of marital status). If parents are not married and the father is not named
on the birth certificate then the father cannot consent.
Theme: Transplantation

A. Xenograft
B. Allograft
C. Autograft
D. Isograft

Please select the type of transplantation that has occurred in the situation described. Each option
may be used once, more than once or not at all.

1. A 38 year old lady donates her kidney to her identical twin sibling.

You answered Xenograft

The correct answer is Isograft

Theme from 2015 Exam


Identical twin- twin donations are usually genetically identical and are therefore isografts.

2. A 53 year old man with severe angina undergoes a coronary artery bypass procedure and his
long saphenous vein is used as a bypass conduit.

You answered Xenograft

The correct answer is Autograft

The long saphenous vein is one of the commonest autografts in surgery.

3. A 38 year old lady donates her kidney to her niece.

You answered Xenograft

The correct answer is Allograft

Though related this donor will not be genetically identical and thus this will be an allograft.

Please rate this question:

Discuss and give feedback


Next question

Transplant types
Graft Features Uses

Allograft Transplant of tissue from genetically non identical Solid organ transplant from
donor from the same species non related donor

Isograft Graft of tissue between two individuals who are Solid organ transplant in
genetically identical identical twins

Autograft Transplantation of organs or tissues from one part of Skin graft


the body to another in the same individual

Xenograft Tissue transplanted from another species Porcine heart valve

Next question
In matching donated kidneys to the most appropriate recipient, apart from ABO matching, which of
the following is most important?

HLA DR

Rhesus

HLA A

HLA B

Duffy antigen

Theme from January 2013 Exam


The rhesus group is not important in matching donor and recipient kidneys.

Please rate this question:

Discuss and give feedback

Next question

Renal transplant:HLA typing and graft failure

The human leucocyte antigen (HLA) system is the name given to the major histocompatibility
complex (MHC) in humans. It is coded for on chromosome 6.

Some basic points on the HLA system

 Class 1 antigens include A, B and C. Class 2 antigens include DP,DQ and DR


 When HLA matching for a renal transplant the relative importance of the HLA antigens are as
follows DR > B > A
Graft survival

 1 year = 90%, 10 years = 60% for cadaveric transplants


 1 year = 95%, 10 years = 70% for living-donor transplants

Post-op problems

 ATN of graft
 Vascular thrombosis
 Urine leakage
 UTI

Hyperacute acute rejection

 Due to antibodies against donor HLA type 1 antigens


 Rarely seen due to HLA matching

Acute graft failure (< 6 months)

 Usually due to mismatched HLA


 Other causes include cytomegalovirus infection
 Management: give steroids, if resistant use monoclonal antibodies

Causes of chronic graft failure (> 6 months)

 Chronic allograft nephropathy


 Ureteric obstruction
 Recurrence of original renal disease (MCGN > IgA > FSGS)

Next question
A 43 year old lady undergoes a live donor related renal transplant. Over the next few years it is
noted that her renal function progressively deteriorates. What is the most likely underlying
explanation?

Type I hypersensitivity reaction

Type III hypersensitivity reaction

Type II hypersensitivity reaction

Type IV hypersensitivity reaction

None of the above

Chronic rejection of renal transplants is mediated via T lymphocytes and is therefore a type IV
hypersensitivity reaction. This process can be mitigated by immunosupression.
Please rate this question:

Discuss and give feedback


Next question

Organ Transplant

A number of different organ and tissue transplants are now available. In many cases an allograft is
performed, where an organ is transplanted from one individual to another. Allografts will elicit an
immune response and this is one of the main reasons for organ rejection.

Graft rejection occurs because allografts have allelic differences at genes that code
immunohistocompatability complex genes. The main antigens that give rise to rejection are:

 ABO blood group


 Human leucocyte antigens (HLA)
 Minor histocompatability antigens

ABO Matching
ABO incompatibility will result in early organ rejection (hyperacute) because of pre existing
antibodies to other groups. Group O donors can give organs to any type of ABO recipient whereas
group AB donor can only donate to AB recipient.

HLA System
The four most important HLA alleles are:

 HLA A
 HLA B
 HLA C
 HLA DR

An ideal organ match would be one in which all 8 alleles are matched (remember 2 from each
parent, four each = 8 alleles). Modern immunosuppressive regimes help to manage the potential
rejection due to HLA mismatching. However, the greater the number of mismatches the worse the
long term outcome will be. T lymphocytes will recognise antigens bound to HLA molecules and will
then become activated. Clonal expansion then occurs with a response directed against that antigen.

Types of organ rejection

 Hyperacute. This occurs immediately through presence of pre formed antibodies (such as
ABO incompatibility).
 Acute. Occurs during the first 6 months and is usually T cell mediated. Usually tissue
infiltrates and vascular lesions.
 Chronic. Occurs after the first 6 months. Vascular changes predominate.

Hyperacute
Renal transplants at greatest risk and liver transplants at least risk. Although ABO incompatibility
and HLA Class I incompatible transplants will all fare worse in long term.

Acute
All organs may undergo acute rejection. Mononuclear cell infiltrates predominate. All types of
transplanted organ are susceptible and it may occur in up to 50% cases.

Chronic
Again all transplants with HLA mismatch may suffer this fate. Previous acute rejections and other
immunosensitising events all increase the risk. Vascular changes are most prominent with
myointimal proliferation leading to organ ischaemia. Organ specific changes are also seen such as
loss of acinar cells in pancreas transplants and rapidly progressive coronary artery disease in
cardiac transplants.

Surgical overview-Renal transplantation


A brief overview of the steps involved in renal transplantation is given.
Patients with end stage renal failure who are dialysis dependent or likely to become so in the
immediate future are considered for transplant. Exclusion criteria include; active malignancy, old age
(due to limited organ availability). Patients are medically optimised.
Donor kidneys, these may be taken from live related donors and close family, members may have
less HLA mismatch than members of the general population. Laparoscopic donor nephrectomy
further minimises the operative morbidity for the donor. Other organs are typically taken from brain
dead or dying patients who have a cardiac arrest and in whom resuscitation is futile. The key event
is to minimise the warm ischaemic time in the donor phase.
The kidney once removed is usually prepared on the bench in theatre by the transplant surgeon
immediately prior to implantation and factors such as accessory renal arteries and vessel length are
assessed and managed.

For first time recipients the operation is performed under general anaesthesia. A Rutherford-
Morrison incision is made on the preferred side. This provides excellent extraperitoneal access to
the iliac vessels. The external iliac artery and vein are dissected out and following systemic
heparinisation are cross clamped. The vein and artery are anastamosed to the iliacs and the clamps
removed. The ureter is then implanted into the bladder and a stent is usually placed to maintain
patency. The wounds are then closed and the patient recovered from surgery.

In the immediate phase a common problem encountered in cadaveric kidneys is acute tubular
necrosis and this tends to resolve.

Graft survival times from cadaveric donors are typically of the order of 9 years and monozygotic twin
transplant (live donor) may survive as long as 25 years.
Next question
Theme: Renal transplant complications

A. Acute tubular necrosis


B. Renal artery thrombosis
C. Bladder occlusion
D. Ureteric occlusion
E. Acute rejection
F. Acute on chronic rejection
G. Hyperacute rejection

For each of the scenarios given please select the most likely underlying process from the list below.
Each option may be used once, more than once or not at all.

6. A 45 year old man with end stage renal failure undergoes a cadaveric renal transplant. The
transplanted organ has a cold ischaemic time of 26 hours and a warm ischaemic time of 54
minutes. Post operatively the patient receives immunosuppressive therapy. Ten days later
the patient has gained weight, becomes oliguric and feels systemically unwell. He also
complains of swelling over the transplant site that is painful.

You answered Acute tubular necrosis

The correct answer is Acute rejection

Theme from April 2012 Exam


The features described are those of worsening graft function and acute rejection. The fact
that there is a 10 day delay goes against hyperacute rejection. Cold ischaemic times are a
major factor for delayed graft function. However, even 26 hours is not incompatible with
graft survival.

7. A 44 year old man with end stage renal failure undergoes a live donor renal transplant.
During the immediate post operative period a good urine output is recorded. However, on
return to the ward the nursing staff notice that the urinary catheter is no longer draining.
However, the urostomy is continuing to drain urine.

You answered Acute tubular necrosis

The correct answer is Bladder occlusion

The most likely explanation for this event is a blocked catheter. This may be the result of
blood clot from the ureteric anastomosis. Bladder irrigation will usually resolve the
problem.

8. A 43 year old man undergoes a live donor renal transplant. The donor's right kidney is
anastomosed to the recipient. On removal of the arterial clamps there is good urinary flow
noted and the wounds are closed. On return to the ward the nurses notice that the patient
suddenly becomes anuric and irrigation of the bladder does not improve the situation.

You answered Acute tubular necrosis

The correct answer is Renal artery thrombosis

Right sided live donor transplants are extremely rare. This is because the vena cava
precludes mobilisation of the right renal artery. The short right renal artery that is produced
therefore presents a major challenge. The sudden cessation of urine output in this context is
highly suggestive of an acute thrombosis. Delay in thrombectomy beyond 1 hour almost
inevitably results in graft loss.

Please rate this question:

Discuss and give feedback


Next question

Complications following renal transplant

Renal transplantation is widely practised. The commonest technical related complications are related
to the ureteric anastomosis. The warm ischaemic time is also of considerable importance and graft
survival is directly related to this. Long warm ischaemic times increase the risk of acute tubular
necrosis which may occur in all types of renal transplanation and provided other insults are
minimised, will usually recover. Organ rejection may occur at any phase following the transplantation
process.

Immunological complications
Types of organ rejection

 Hyperacute. This occurs immediately through presence of pre formed antibody (such as
ABO incompatibility).
 Acute. Occurs during the first 6 months and is usually T cell mediated. Usually tissue
infiltrates and vascular lesions.
 Chronic. Occurs after the first 6 months. Vascular changes predominate.

Hyperacute
Renal transplants are most susceptible to this process. Risk factors include major HLA mismatch
and ABO incompatibility. The rejection occurs almost immediately and the macroscopic features
may become manifest following completion of the vascular anastomosis and removal of clamps. The
kidney becomes mottled, dusky and the vessels will thrombose. The only treatment is removal of the
graft, if left in situ it will result in abscess formation.

Acute
All organs may undergo acute rejection. Mononuclear cell infiltrates predominate. All types of
transplanted organ are susceptible and it may occur in up to 50% cases. Most cases can be
managed medically.

Chronic
Again all transplants with HLA mismatch may suffer this fate. Previous acute rejections and other
immunosensitising events all increase the risk. Vascular changes are most prominent with
myointimal proliferation leading to organ ischaemia. Organ specific changes are also seen such as
loss of acinar cells in pancreas transplants and rapidly progressive coronary artery disease in
cardiac transplants.

Technical complications
Complication Presenting features Treatment

Renal artery Sudden complete loss of urine Immediate surgery may salvage the graft,
thrombosis output delays beyond 30 minutes are associated
with a high rate of graft loss

Renal artery Uncontrolled hypertension, Angioplasty is the treatment of choice


stenosis allograft dysfunction and oedema

Renal vein Pain and swelling over the graft The graft is usually lost
thrombosis site, haematuria and oliguria

Urine leaks Diminished urine output, rising USS shows perigraft collection, necrosis
creatinine, fever and abdominal of ureter tip is the commonest cause and
pain the anastomosis may need revision

Lymphocele Common complication (occurs in May be drained with percutaneous


15%), may present as a mass, if technique and sclerotherapy, or
large may compress ureter intraperitoneal drainage

Next question
A 43 year old man undergoes a cadaveric renal transplant. The operation is uncomplicated. On
removal of the vascular clamps the transplanted kidney immediately turns dusky and over the
ensuing hours appears non viable. Which of the following best explains this event?

Chronic rejection

Hyper acute rejection

Acute rejection

Sub chronic rejection

Infection of the graft

Immediate rejection is due to the presence of pre-existing antibodies e.g. ABO mismatch. The
transplanted organ should be removed.
Please rate this question:

Discuss and give feedback


Next question

Organ Transplant

A number of different organ and tissue transplants are now available. In many cases an allograft is
performed, where an organ is transplanted from one individual to another. Allografts will elicit an
immune response and this is one of the main reasons for organ rejection.

Graft rejection occurs because allografts have allelic differences at genes that code
immunohistocompatability complex genes. The main antigens that give rise to rejection are:

 ABO blood group


 Human leucocyte antigens (HLA)
 Minor histocompatability antigens

ABO Matching
ABO incompatibility will result in early organ rejection (hyperacute) because of pre existing
antibodies to other groups. Group O donors can give organs to any type of ABO recipient whereas
group AB donor can only donate to AB recipient.

HLA System
The four most important HLA alleles are:

 HLA A
 HLA B
 HLA C
 HLA DR

An ideal organ match would be one in which all 8 alleles are matched (remember 2 from each
parent, four each = 8 alleles). Modern immunosuppressive regimes help to manage the potential
rejection due to HLA mismatching. However, the greater the number of mismatches the worse the
long term outcome will be. T lymphocytes will recognise antigens bound to HLA molecules and will
then become activated. Clonal expansion then occurs with a response directed against that antigen.

Types of organ rejection

 Hyperacute. This occurs immediately through presence of pre formed antibodies (such as
ABO incompatibility).
 Acute. Occurs during the first 6 months and is usually T cell mediated. Usually tissue
infiltrates and vascular lesions.
 Chronic. Occurs after the first 6 months. Vascular changes predominate.

Hyperacute
Renal transplants at greatest risk and liver transplants at least risk. Although ABO incompatibility
and HLA Class I incompatible transplants will all fare worse in long term.

Acute
All organs may undergo acute rejection. Mononuclear cell infiltrates predominate. All types of
transplanted organ are susceptible and it may occur in up to 50% cases.

Chronic
Again all transplants with HLA mismatch may suffer this fate. Previous acute rejections and other
immunosensitising events all increase the risk. Vascular changes are most prominent with
myointimal proliferation leading to organ ischaemia. Organ specific changes are also seen such as
loss of acinar cells in pancreas transplants and rapidly progressive coronary artery disease in
cardiac transplants.

Surgical overview-Renal transplantation


A brief overview of the steps involved in renal transplantation is given.
Patients with end stage renal failure who are dialysis dependent or likely to become so in the
immediate future are considered for transplant. Exclusion criteria include; active malignancy, old age
(due to limited organ availability). Patients are medically optimised.
Donor kidneys, these may be taken from live related donors and close family, members may have
less HLA mismatch than members of the general population. Laparoscopic donor nephrectomy
further minimises the operative morbidity for the donor. Other organs are typically taken from brain
dead or dying patients who have a cardiac arrest and in whom resuscitation is futile. The key event
is to minimise the warm ischaemic time in the donor phase.
The kidney once removed is usually prepared on the bench in theatre by the transplant surgeon
immediately prior to implantation and factors such as accessory renal arteries and vessel length are
assessed and managed.

For first time recipients the operation is performed under general anaesthesia. A Rutherford-
Morrison incision is made on the preferred side. This provides excellent extraperitoneal access to
the iliac vessels. The external iliac artery and vein are dissected out and following systemic
heparinisation are cross clamped. The vein and artery are anastamosed to the iliacs and the clamps
removed. The ureter is then implanted into the bladder and a stent is usually placed to maintain
patency. The wounds are then closed and the patient recovered from surgery.

In the immediate phase a common problem encountered in cadaveric kidneys is acute tubular
necrosis and this tends to resolve.

Graft survival times from cadaveric donors are typically of the order of 9 years and monozygotic twin
transplant (live donor) may survive as long as 25 years.
Next question
Theme: Complications following renal transplant

A. Ureteric anastomotic leak


B. Renal vein thrombosis
C. Acute rejection
D. Chronic allograft nephropathy
E. Renal artery thrombosis
F. Renal artery stenosis
G. Lymphocele
H. Hyperacute rejection

For each of the patients described below, please select the most appropriate underlying explanation
for the situation described. Each option may be used once, more than once or not at all.

10. A 45 year old lady undergoes a renal transplant from a living related donor. She is well for
several months but on review in the outpatient department is noted to have persistent
hypertension and a slight deterioration in renal function.

You answered Ureteric anastomotic leak

The correct answer is Renal artery stenosis

Renal artery stenosis typically occurs over several months and will usually result in the
development of hypertension. Most cases can be assessed using duplex scanning and
managed with angioplasty.

11. A 43 year old lady undergoes a live related renal transplant. At the conclusion of the
operation she has a good urine output and the graft appeared well perfused. On the ward
she suddenly becomes anuric.

You answered Ureteric anastomotic leak

The correct answer is Renal artery thrombosis

Sudden loss of urine output is most commonly due to a blocked catheter. However, if this
is excluded (and is not included in the options) the most worrisome cause is arterial
thrombosis. This will often be a delayed diagnosis and the rate of graft loss is high.

12. A 39 year old lady undergoes a live related renal transplant. She progresses well. Two
weeks following the transplant she is noted to have swelling overlying the transplant site
and swelling of the ipsilateral limb.Urine output is acceptable and creatinine unchanged.

You answered Ureteric anastomotic leak


The correct answer is Lymphocele

Swelling over the graft site is often due to a lymphocele and this is further suggested by
the normal renal function. They cause symptoms through mass effect and limb swelling
may occur. Treatment is often surgical.

Please rate this question:

Discuss and give feedback


Next question

Complications of renal transplantation

A number of complications may occur following renal transplantation. A critical aspect of post
operative care is evaluation of graft function. Post operatively, urine output is the most readily
available, and easily measured, indicator of graft function. If an individual was relatively anuric pre-
transplant and has a good urine output following surgery then this is more useful than it would be in
someone who had a higher volume diuresis prior to transplantation. Recipients can be divided into
three main groups following renal transplantation, with regard to their graft function:

 Immediate graft function; brisk diuresis and falling serum creatinine


 Slow graft function; modest urine output and slowly falling creatinine levels
 Delayed graft function; defined as need for dialysis post transplant

Decreased urine output following surgery can be the result of hypovolvaemia or a blocked catheter
(commonest causes). Other important causes include rejection, or a vascular complication.

Vascular complications
These may involve the donor vessels, those of the recipient or both. Renal artery thrombosis usually
occurs early post transplant, but is uncommon with an incidence of less than 1%. It typically results
in graft loss. It usually occurs as a result of a technical problem such a vessel torsion or sub intimal
flaps. The usual presenting feature is a sudden cessation of urine output. When suspected, the
occlusion is usually well demonstrated with duplex scanning. Ideally immediate surgical re-
exploration should occur. Sadly, the graft has usually been lost by this stage and will require graft
nephrectomy. Renal vein thrombosis is not as common as arterial graft thrombosis and the usual
presenting features include discomfort at the graft site and swelling of the graft associated with loss
of urine output. Again, duplex scanning is indicated. Unfortunately, this complication is also
associated with a high incidence of graft loss.
Over a longer time frame (typically months) some individuals will develop renal artery stenosis.
These individuals will typically develop hypertension and over time graft function will decline as
hypertensive nephropathy occurs. It is usually demonstrated by duplex scanning and is usually
amenable to endovascular intervention.

Urological complications
Urinary tract complications manifesting as leakage or obstruction are common complications
following renal transplantation and occur in up to 10% of patients. The main underlying cause is the
relatively poor blood supply to the transplanted ureter. Patients typically present relatively early in
the first 5 weeks following transplantation with pain and swelling at the graft site. Imaging with USS
is often the initial test. Therapeutic options include surgical re-implantation of the ureter for large
leaks and stent insertion and nephrostomy placement for smaller leaks.

Lymphocele
These do not generally occur until 2 weeks or longer after surgery. They are, however, relatively
common and may be seen in up to 18% of patients. Symptoms usually occur as a result of mass
effect with compression of adjacent structures. These include the vessels supplying both the graft,
with deterioration in graft function, the ureter, with alteration in urine output and the recipients lower
limb vessels, with development of leg swelling. Creation of a laparoscopic or open peritoneal window
is a favored treatment.

Rejection
Four types of graft rejection are recognised; hyperacute, accelerated acute, acute and chronic.

Type of
rejection Key features

Hyperacute Occurs within minutes of clamp release


Due to pre formed antibodies
Immediate loss of graft occurs

Accelerated Occurs in first few days following surgery


acute Involved both cellular and antibody mediated injury
Pre-sensitisation of the donor is a common cause

Acute Traditionally the most common type of rejection


Seen days to weeks after surgery
Predominantly a cell mediated process mediated by lymphocytes
Organ biopsy demonstrates cellular infiltrates and graft cell apoptosis

Chronic Increasingly common problem


Typically; graft atrophy and atherosclerosis are seen. Fibrosis often occurs as a
late event

Next question
A 48 year old lady with end stage renal failure receives a cadaveric renal transplant. The organ is
ABO group matched only. On completion of the vascular anastomoses the surgeons remove the
clamps. Over the course of the next twelve minutes the donated kidney becomes dusky and swollen
and appears non viable. Which of the following is the most likely process that has caused this event?

IgG anti HLA Class I antibodies in the recipient

IgM anti HLA Class I antibodies in the recipient

IgG anti HLA Class I antibodies from the donor

IgM anti HLA Class I antibodies from the donor

IgM anti HLA Class II antibodies from the recipient

Episodes of hyperacute rejection are typically due to preformed antibodies. ABO mismatch is the
best example. However, IgG anti HLA Class I antibodies are another potential cause. These events
are now seen less commonly because the cross matching process generally takes this possibility
into account.
Please rate this question:

Discuss and give feedback


Next question

Organ Transplant

A number of different organ and tissue transplants are now available. In many cases an allograft is
performed, where an organ is transplanted from one individual to another. Allografts will elicit an
immune response and this is one of the main reasons for organ rejection.

Graft rejection occurs because allografts have allelic differences at genes that code
immunohistocompatability complex genes. The main antigens that give rise to rejection are:

 ABO blood group


 Human leucocyte antigens (HLA)
 Minor histocompatability antigens

ABO Matching
ABO incompatibility will result in early organ rejection (hyperacute) because of pre existing
antibodies to other groups. Group O donors can give organs to any type of ABO recipient whereas
group AB donor can only donate to AB recipient.

HLA System
The four most important HLA alleles are:

 HLA A
 HLA B
 HLA C
 HLA DR

An ideal organ match would be one in which all 8 alleles are matched (remember 2 from each
parent, four each = 8 alleles). Modern immunosuppressive regimes help to manage the potential
rejection due to HLA mismatching. However, the greater the number of mismatches the worse the
long term outcome will be. T lymphocytes will recognise antigens bound to HLA molecules and will
then become activated. Clonal expansion then occurs with a response directed against that antigen.

Types of organ rejection

 Hyperacute. This occurs immediately through presence of pre formed antibodies (such as
ABO incompatibility).
 Acute. Occurs during the first 6 months and is usually T cell mediated. Usually tissue
infiltrates and vascular lesions.
 Chronic. Occurs after the first 6 months. Vascular changes predominate.

Hyperacute
Renal transplants at greatest risk and liver transplants at least risk. Although ABO incompatibility
and HLA Class I incompatible transplants will all fare worse in long term.

Acute
All organs may undergo acute rejection. Mononuclear cell infiltrates predominate. All types of
transplanted organ are susceptible and it may occur in up to 50% cases.

Chronic
Again all transplants with HLA mismatch may suffer this fate. Previous acute rejections and other
immunosensitising events all increase the risk. Vascular changes are most prominent with
myointimal proliferation leading to organ ischaemia. Organ specific changes are also seen such as
loss of acinar cells in pancreas transplants and rapidly progressive coronary artery disease in
cardiac transplants.

Surgical overview-Renal transplantation


A brief overview of the steps involved in renal transplantation is given.
Patients with end stage renal failure who are dialysis dependent or likely to become so in the
immediate future are considered for transplant. Exclusion criteria include; active malignancy, old age
(due to limited organ availability). Patients are medically optimised.
Donor kidneys, these may be taken from live related donors and close family, members may have
less HLA mismatch than members of the general population. Laparoscopic donor nephrectomy
further minimises the operative morbidity for the donor. Other organs are typically taken from brain
dead or dying patients who have a cardiac arrest and in whom resuscitation is futile. The key event
is to minimise the warm ischaemic time in the donor phase.

The kidney once removed is usually prepared on the bench in theatre by the transplant surgeon
immediately prior to implantation and factors such as accessory renal arteries and vessel length are
assessed and managed.

For first time recipients the operation is performed under general anaesthesia. A Rutherford-
Morrison incision is made on the preferred side. This provides excellent extraperitoneal access to
the iliac vessels. The external iliac artery and vein are dissected out and following systemic
heparinisation are cross clamped. The vein and artery are anastamosed to the iliacs and the clamps
removed. The ureter is then implanted into the bladder and a stent is usually placed to maintain
patency. The wounds are then closed and the patient recovered from surgery.

In the immediate phase a common problem encountered in cadaveric kidneys is acute tubular
necrosis and this tends to resolve.

Graft survival times from cadaveric donors are typically of the order of 9 years and monozygotic twin
transplant (live donor) may survive as long as 25 years.
Next question
A 54-year-old man who has end stage diabetic nephropathy is being assessed for a renal transplant.
When assessing the HLA matching between donor and recipient what is the most important HLA
antigen to match?

DP

DR

Renal transplant HLA matching - DR is the most important

Please rate this question:

Discuss and give feedback


Next question

Renal transplant:HLA typing and graft failure

The human leucocyte antigen (HLA) system is the name given to the major histocompatibility
complex (MHC) in humans. It is coded for on chromosome 6.

Some basic points on the HLA system

 Class 1 antigens include A, B and C. Class 2 antigens include DP,DQ and DR


 When HLA matching for a renal transplant the relative importance of the HLA antigens are as
follows DR > B > A

Graft survival

 1 year = 90%, 10 years = 60% for cadaveric transplants


 1 year = 95%, 10 years = 70% for living-donor transplants
Post-op problems

 ATN of graft
 Vascular thrombosis
 Urine leakage
 UTI

Hyperacute acute rejection

 Due to antibodies against donor HLA type 1 antigens


 Rarely seen due to HLA matching

Acute graft failure (< 6 months)

 Usually due to mismatched HLA


 Other causes include cytomegalovirus infection
 Management: give steroids, if resistant use monoclonal antibodies

Causes of chronic graft failure (> 6 months)

 Chronic allograft nephropathy


 Ureteric obstruction
 Recurrence of original renal disease (MCGN > IgA > FSGS)

Next question
Which of the following transplants is most susceptible to donor- recipient HLA mismatches?

Autologous skin graft

Renal allograft

Liver allograft

Corneal allograft

Cardiac valve allograft

Autologous transplant- same individual (genetically identical)


Allograft - Genetically different

The kidney is highly susceptible to HLA mismatches and hyperacute rejection may occur in patients
with IgG anti HLA Class I antibodies. The liver is at far lower risk of rejection of this nature. Although
the heart is sensitive to HLA mismatches this is less than the kidney. Cardiac valves and the cornea
incite little immunological response.
Please rate this question:

Discuss and give feedback


Next question

Organ Transplant

A number of different organ and tissue transplants are now available. In many cases an allograft is
performed, where an organ is transplanted from one individual to another. Allografts will elicit an
immune response and this is one of the main reasons for organ rejection.

Graft rejection occurs because allografts have allelic differences at genes that code
immunohistocompatability complex genes. The main antigens that give rise to rejection are:

 ABO blood group


 Human leucocyte antigens (HLA)
 Minor histocompatability antigens

ABO Matching
ABO incompatibility will result in early organ rejection (hyperacute) because of pre existing
antibodies to other groups. Group O donors can give organs to any type of ABO recipient whereas
group AB donor can only donate to AB recipient.

HLA System
The four most important HLA alleles are:

 HLA A
 HLA B
 HLA C
 HLA DR

An ideal organ match would be one in which all 8 alleles are matched (remember 2 from each
parent, four each = 8 alleles). Modern immunosuppressive regimes help to manage the potential
rejection due to HLA mismatching. However, the greater the number of mismatches the worse the
long term outcome will be. T lymphocytes will recognise antigens bound to HLA molecules and will
then become activated. Clonal expansion then occurs with a response directed against that antigen.

Types of organ rejection

 Hyperacute. This occurs immediately through presence of pre formed antibodies (such as
ABO incompatibility).
 Acute. Occurs during the first 6 months and is usually T cell mediated. Usually tissue
infiltrates and vascular lesions.
 Chronic. Occurs after the first 6 months. Vascular changes predominate.

Hyperacute
Renal transplants at greatest risk and liver transplants at least risk. Although ABO incompatibility
and HLA Class I incompatible transplants will all fare worse in long term.

Acute
All organs may undergo acute rejection. Mononuclear cell infiltrates predominate. All types of
transplanted organ are susceptible and it may occur in up to 50% cases.

Chronic
Again all transplants with HLA mismatch may suffer this fate. Previous acute rejections and other
immunosensitising events all increase the risk. Vascular changes are most prominent with
myointimal proliferation leading to organ ischaemia. Organ specific changes are also seen such as
loss of acinar cells in pancreas transplants and rapidly progressive coronary artery disease in
cardiac transplants.

Surgical overview-Renal transplantation


A brief overview of the steps involved in renal transplantation is given.
Patients with end stage renal failure who are dialysis dependent or likely to become so in the
immediate future are considered for transplant. Exclusion criteria include; active malignancy, old age
(due to limited organ availability). Patients are medically optimised.
Donor kidneys, these may be taken from live related donors and close family, members may have
less HLA mismatch than members of the general population. Laparoscopic donor nephrectomy
further minimises the operative morbidity for the donor. Other organs are typically taken from brain
dead or dying patients who have a cardiac arrest and in whom resuscitation is futile. The key event
is to minimise the warm ischaemic time in the donor phase.

The kidney once removed is usually prepared on the bench in theatre by the transplant surgeon
immediately prior to implantation and factors such as accessory renal arteries and vessel length are
assessed and managed.

For first time recipients the operation is performed under general anaesthesia. A Rutherford-
Morrison incision is made on the preferred side. This provides excellent extraperitoneal access to
the iliac vessels. The external iliac artery and vein are dissected out and following systemic
heparinisation are cross clamped. The vein and artery are anastamosed to the iliacs and the clamps
removed. The ureter is then implanted into the bladder and a stent is usually placed to maintain
patency. The wounds are then closed and the patient recovered from surgery.

In the immediate phase a common problem encountered in cadaveric kidneys is acute tubular
necrosis and this tends to resolve.

Graft survival times from cadaveric donors are typically of the order of 9 years and monozygotic twin
transplant (live donor) may survive as long as 25 years.
Next question
Which of the following is not true of hyper acute solid organ transplant rejection?

It may occur during the surgical procedure itself.

May occur as a result of blood group A, B or O incompatibility.

May be due to pre existing anti HLA antibodies.

On biopsy will typically show neo intimal hyperplasia of donor arterioles.

Complement system activation is one of the key mediators.

These changes are more often seen in the chronic setting. Thrombosis is more commonly seen in
the hyperacute phase.
Please rate this question:

Discuss and give feedback


Next question

Organ Transplant

A number of different organ and tissue transplants are now available. In many cases an allograft is
performed, where an organ is transplanted from one individual to another. Allografts will elicit an
immune response and this is one of the main reasons for organ rejection.

Graft rejection occurs because allografts have allelic differences at genes that code
immunohistocompatability complex genes. The main antigens that give rise to rejection are:

 ABO blood group


 Human leucocyte antigens (HLA)
 Minor histocompatability antigens

ABO Matching
ABO incompatibility will result in early organ rejection (hyperacute) because of pre existing
antibodies to other groups. Group O donors can give organs to any type of ABO recipient whereas
group AB donor can only donate to AB recipient.

HLA System
The four most important HLA alleles are:
 HLA A
 HLA B
 HLA C
 HLA DR

An ideal organ match would be one in which all 8 alleles are matched (remember 2 from each
parent, four each = 8 alleles). Modern immunosuppressive regimes help to manage the potential
rejection due to HLA mismatching. However, the greater the number of mismatches the worse the
long term outcome will be. T lymphocytes will recognise antigens bound to HLA molecules and will
then become activated. Clonal expansion then occurs with a response directed against that antigen.

Types of organ rejection

 Hyperacute. This occurs immediately through presence of pre formed antibodies (such as
ABO incompatibility).
 Acute. Occurs during the first 6 months and is usually T cell mediated. Usually tissue
infiltrates and vascular lesions.
 Chronic. Occurs after the first 6 months. Vascular changes predominate.

Hyperacute
Renal transplants at greatest risk and liver transplants at least risk. Although ABO incompatibility
and HLA Class I incompatible transplants will all fare worse in long term.

Acute
All organs may undergo acute rejection. Mononuclear cell infiltrates predominate. All types of
transplanted organ are susceptible and it may occur in up to 50% cases.

Chronic
Again all transplants with HLA mismatch may suffer this fate. Previous acute rejections and other
immunosensitising events all increase the risk. Vascular changes are most prominent with
myointimal proliferation leading to organ ischaemia. Organ specific changes are also seen such as
loss of acinar cells in pancreas transplants and rapidly progressive coronary artery disease in
cardiac transplants.

Surgical overview-Renal transplantation


A brief overview of the steps involved in renal transplantation is given.
Patients with end stage renal failure who are dialysis dependent or likely to become so in the
immediate future are considered for transplant. Exclusion criteria include; active malignancy, old age
(due to limited organ availability). Patients are medically optimised.
Donor kidneys, these may be taken from live related donors and close family, members may have
less HLA mismatch than members of the general population. Laparoscopic donor nephrectomy
further minimises the operative morbidity for the donor. Other organs are typically taken from brain
dead or dying patients who have a cardiac arrest and in whom resuscitation is futile. The key event
is to minimise the warm ischaemic time in the donor phase.
The kidney once removed is usually prepared on the bench in theatre by the transplant surgeon
immediately prior to implantation and factors such as accessory renal arteries and vessel length are
assessed and managed.

For first time recipients the operation is performed under general anaesthesia. A Rutherford-
Morrison incision is made on the preferred side. This provides excellent extraperitoneal access to
the iliac vessels. The external iliac artery and vein are dissected out and following systemic
heparinisation are cross clamped. The vein and artery are anastamosed to the iliacs and the clamps
removed. The ureter is then implanted into the bladder and a stent is usually placed to maintain
patency. The wounds are then closed and the patient recovered from surgery.

In the immediate phase a common problem encountered in cadaveric kidneys is acute tubular
necrosis and this tends to resolve.

Graft survival times from cadaveric donors are typically of the order of 9 years and monozygotic twin
transplant (live donor) may survive as long as 25 years.
Next question
You review a 42-year-old woman six weeks following a renal transplant for focal segmental
glomerulosclerosis. Following the procedure she was discharged on a combination of tacrolimus,
mycophenolate, and prednisolone. She has now presented with a five day history of feeling
generally unwell with anorexia, fatigue and arthralgia. On examination she has a temperature of 37.9
and has widespread lymphadenopathy. What is the most likely diagnosis?

Hepatitis C

Epstein-Barr virus

HIV

Hepatitis B

Cytomegalovirus

Cytomegalovirus is the most common and important viral infection in solid organ transplant
recipients

Primary infection with CMV typically occurs 6 weeks post transplantation in a seronegative individual
who receives an organ from a seropositive donor. Symptoms may occur as early as 20 days but can
occur up to 6 months post transplant . Symptoms are often vague, retinitis can be pathognomonic,
but is rarely seen in the transplant population. CMV disease is seen in 8% of renal transplant
patients. Intravenous ganciclovir is the treatment of choice in such patients. Unfortunately, relapses
are not uncommon.
Please rate this question:

Discuss and give feedback


Next question

Renal transplant:HLA typing and graft failure

The human leucocyte antigen (HLA) system is the name given to the major histocompatibility
complex (MHC) in humans. It is coded for on chromosome 6.

Some basic points on the HLA system

 Class 1 antigens include A, B and C. Class 2 antigens include DP,DQ and DR


 When HLA matching for a renal transplant the relative importance of the HLA antigens are as
follows DR > B > A
Graft survival

 1 year = 90%, 10 years = 60% for cadaveric transplants


 1 year = 95%, 10 years = 70% for living-donor transplants

Post-op problems

 ATN of graft
 Vascular thrombosis
 Urine leakage
 UTI

Hyperacute acute rejection

 Due to antibodies against donor HLA type 1 antigens


 Rarely seen due to HLA matching

Acute graft failure (< 6 months)

 Usually due to mismatched HLA


 Other causes include cytomegalovirus infection
 Management: give steroids, if resistant use monoclonal antibodies

Causes of chronic graft failure (> 6 months)

 Chronic allograft nephropathy


 Ureteric obstruction
 Recurrence of original renal disease (MCGN > IgA > FSGS)

Next question
A 43 year old lady is recovering following a live donor related renal transplant. She has significant
abdominal pain. Which of the following analgesic drugs should be avoided?

Paracetamol

Morphine

Nefopam

Diclofenac

Co-codamol

Non steroidal anti inflammatory drugs may be nephrotoxic and therefore are usually avoided in
patients who have undergone renal transplants. Paracetamol and morphine are metabolised
predominantly in the liver. There is some renal contribution to morphine metabolism and excretion
and the drug should be administered in reduced doses or avoided if the transplanted kidney stops
functioning.
Please rate this question:

Discuss and give feedback


Next question

Organ Transplant

A number of different organ and tissue transplants are now available. In many cases an allograft is
performed, where an organ is transplanted from one individual to another. Allografts will elicit an
immune response and this is one of the main reasons for organ rejection.

Graft rejection occurs because allografts have allelic differences at genes that code
immunohistocompatability complex genes. The main antigens that give rise to rejection are:

 ABO blood group


 Human leucocyte antigens (HLA)
 Minor histocompatability antigens

ABO Matching
ABO incompatibility will result in early organ rejection (hyperacute) because of pre existing
antibodies to other groups. Group O donors can give organs to any type of ABO recipient whereas
group AB donor can only donate to AB recipient.
HLA System
The four most important HLA alleles are:

 HLA A
 HLA B
 HLA C
 HLA DR

An ideal organ match would be one in which all 8 alleles are matched (remember 2 from each
parent, four each = 8 alleles). Modern immunosuppressive regimes help to manage the potential
rejection due to HLA mismatching. However, the greater the number of mismatches the worse the
long term outcome will be. T lymphocytes will recognise antigens bound to HLA molecules and will
then become activated. Clonal expansion then occurs with a response directed against that antigen.

Types of organ rejection

 Hyperacute. This occurs immediately through presence of pre formed antibodies (such as
ABO incompatibility).
 Acute. Occurs during the first 6 months and is usually T cell mediated. Usually tissue
infiltrates and vascular lesions.
 Chronic. Occurs after the first 6 months. Vascular changes predominate.

Hyperacute
Renal transplants at greatest risk and liver transplants at least risk. Although ABO incompatibility
and HLA Class I incompatible transplants will all fare worse in long term.

Acute
All organs may undergo acute rejection. Mononuclear cell infiltrates predominate. All types of
transplanted organ are susceptible and it may occur in up to 50% cases.

Chronic
Again all transplants with HLA mismatch may suffer this fate. Previous acute rejections and other
immunosensitising events all increase the risk. Vascular changes are most prominent with
myointimal proliferation leading to organ ischaemia. Organ specific changes are also seen such as
loss of acinar cells in pancreas transplants and rapidly progressive coronary artery disease in
cardiac transplants.

Surgical overview-Renal transplantation


A brief overview of the steps involved in renal transplantation is given.
Patients with end stage renal failure who are dialysis dependent or likely to become so in the
immediate future are considered for transplant. Exclusion criteria include; active malignancy, old age
(due to limited organ availability). Patients are medically optimised.
Donor kidneys, these may be taken from live related donors and close family, members may have
less HLA mismatch than members of the general population. Laparoscopic donor nephrectomy
further minimises the operative morbidity for the donor. Other organs are typically taken from brain
dead or dying patients who have a cardiac arrest and in whom resuscitation is futile. The key event
is to minimise the warm ischaemic time in the donor phase.

The kidney once removed is usually prepared on the bench in theatre by the transplant surgeon
immediately prior to implantation and factors such as accessory renal arteries and vessel length are
assessed and managed.

For first time recipients the operation is performed under general anaesthesia. A Rutherford-
Morrison incision is made on the preferred side. This provides excellent extraperitoneal access to
the iliac vessels. The external iliac artery and vein are dissected out and following systemic
heparinisation are cross clamped. The vein and artery are anastamosed to the iliacs and the clamps
removed. The ureter is then implanted into the bladder and a stent is usually placed to maintain
patency. The wounds are then closed and the patient recovered from surgery.

In the immediate phase a common problem encountered in cadaveric kidneys is acute tubular
necrosis and this tends to resolve.

Graft survival times from cadaveric donors are typically of the order of 9 years and monozygotic twin
transplant (live donor) may survive as long as 25 years.
Next question
A 52 year old male attends renal transplant clinic for a post operative assessment. You note that he
is on ciclosporin and that a recent blood test shows that the ciclosporin level is elevated. Which of
the following is a recognised side effect of ciclosporin?

Hyperthyroidism

Diabetes

Alopecia

Hypothermia

Nephrotoxicity

Ciclosporin- nephrotoxicity

This patient is at risk of nephrotoxicity and should be referred to the renal team as soon as possible.
Alopecia is associated with azathioprine and diabetes is associated with tacrolimus.
Please rate this question:

Discuss and give feedback


Next question

Organ transplantation: immunosupressants

A number of drugs are available which help to mitigate the processes resulting in acute rejection.
Cyclosporin and tacrolimus are commonly used drugs.

Example regime

 Initial: ciclosporin/tacrolimus with a monoclonal antibody


 Maintenance: ciclosporin/tacrolimus with MMF or sirolimus
 Add steroids if more than one steroid responsive acute rejection episode

Ciclosporin

 Inhibits calcineurin, a phosphatase involved in T cell activation


 Nephrotoxic
 Monitor levels
Azathioprine

 Metabolised to form 6 mercaptopurine which inhibits DNA synthesis and cell division
 Side effects include myelosupression, alopecia and nausea

Tacrolimus

 Lower incidence of acute rejection compared to ciclosporin


 Also less hypertension and hyperlipidaemia
 However, high incidence of impaired glucose tolerance and diabetes

Mycophenolate mofetil (MMF)

 Blocks purine synthesis by inhibition of IMPDH


 Therefore inhibits proliferation of B and T cells
 Side-effects: GI and marrow suppression

Sirolimus (rapamycin)

 Blocks T cell proliferation by blocking the IL-2 receptor


 Can cause hyperlipidaemia

Monoclonal antibodies

 Selective inhibitors of IL-2 receptor


 Daclizumab
 Basilximab

Next question
A 48 year old woman with end stage renal failure is undergoing a live donor renal transplant. The
surgeon decides to implant the kidney in the left iliac fossa via a Rutherford Morrison incision. To
which of the following vessels should the transplanted kidney be anastomosed?

Aorta and inferior vena cava

Internal iliac artery and vein

Common iliac artery and vein

External iliac artery and vein

Inferior epigastric artery and vein

First time renal tranplants and typically implanted in the left or right iliac fossae. The vessels are
usually joined to the external iliac artery and vein as these are the most easily accessible. The
Rutherford Morrison incision provides access to the external iliac vessels.
Please rate this question:

Discuss and give feedback


Next question

Organ Transplant

A number of different organ and tissue transplants are now available. In many cases an allograft is
performed, where an organ is transplanted from one individual to another. Allografts will elicit an
immune response and this is one of the main reasons for organ rejection.

Graft rejection occurs because allografts have allelic differences at genes that code
immunohistocompatability complex genes. The main antigens that give rise to rejection are:

 ABO blood group


 Human leucocyte antigens (HLA)
 Minor histocompatability antigens

ABO Matching
ABO incompatibility will result in early organ rejection (hyperacute) because of pre existing
antibodies to other groups. Group O donors can give organs to any type of ABO recipient whereas
group AB donor can only donate to AB recipient.
HLA System
The four most important HLA alleles are:

 HLA A
 HLA B
 HLA C
 HLA DR

An ideal organ match would be one in which all 8 alleles are matched (remember 2 from each
parent, four each = 8 alleles). Modern immunosuppressive regimes help to manage the potential
rejection due to HLA mismatching. However, the greater the number of mismatches the worse the
long term outcome will be. T lymphocytes will recognise antigens bound to HLA molecules and will
then become activated. Clonal expansion then occurs with a response directed against that antigen.

Types of organ rejection

 Hyperacute. This occurs immediately through presence of pre formed antibodies (such as
ABO incompatibility).
 Acute. Occurs during the first 6 months and is usually T cell mediated. Usually tissue
infiltrates and vascular lesions.
 Chronic. Occurs after the first 6 months. Vascular changes predominate.

Hyperacute
Renal transplants at greatest risk and liver transplants at least risk. Although ABO incompatibility
and HLA Class I incompatible transplants will all fare worse in long term.

Acute
All organs may undergo acute rejection. Mononuclear cell infiltrates predominate. All types of
transplanted organ are susceptible and it may occur in up to 50% cases.

Chronic
Again all transplants with HLA mismatch may suffer this fate. Previous acute rejections and other
immunosensitising events all increase the risk. Vascular changes are most prominent with
myointimal proliferation leading to organ ischaemia. Organ specific changes are also seen such as
loss of acinar cells in pancreas transplants and rapidly progressive coronary artery disease in
cardiac transplants.

Surgical overview-Renal transplantation


A brief overview of the steps involved in renal transplantation is given.
Patients with end stage renal failure who are dialysis dependent or likely to become so in the
immediate future are considered for transplant. Exclusion criteria include; active malignancy, old age
(due to limited organ availability). Patients are medically optimised.
Donor kidneys, these may be taken from live related donors and close family, members may have
less HLA mismatch than members of the general population. Laparoscopic donor nephrectomy
further minimises the operative morbidity for the donor. Other organs are typically taken from brain
dead or dying patients who have a cardiac arrest and in whom resuscitation is futile. The key event
is to minimise the warm ischaemic time in the donor phase.
The kidney once removed is usually prepared on the bench in theatre by the transplant surgeon
immediately prior to implantation and factors such as accessory renal arteries and vessel length are
assessed and managed.

For first time recipients the operation is performed under general anaesthesia. A Rutherford-
Morrison incision is made on the preferred side. This provides excellent extraperitoneal access to
the iliac vessels. The external iliac artery and vein are dissected out and following systemic
heparinisation are cross clamped. The vein and artery are anastamosed to the iliacs and the clamps
removed. The ureter is then implanted into the bladder and a stent is usually placed to maintain
patency. The wounds are then closed and the patient recovered from surgery.

In the immediate phase a common problem encountered in cadaveric kidneys is acute tubular
necrosis and this tends to resolve.

Graft survival times from cadaveric donors are typically of the order of 9 years and monozygotic twin
transplant (live donor) may survive as long as 25 years.
Next question
A 28-year-old female undergoes a renal transplant for focal segmental glomerulosclerosis. Within
hours of the operation the patient becomes unwell with features consistent with severe systemic
inflammatory response syndrome. The patient is immediately taken back to theatre and the
transplanted kidney is removed. What type of immunoglobulins are responsible for the graft
rejection?

IgE

IgM

IgG

IgD

IgA

Hyperacute graft rejection is due to pre-existent antibodies to HLA antigens and is therefore IgG
mediated
Please rate this question:

Discuss and give feedback


Next question

Renal transplant:HLA typing and graft failure

The human leucocyte antigen (HLA) system is the name given to the major histocompatibility
complex (MHC) in humans. It is coded for on chromosome 6.

Some basic points on the HLA system

 Class 1 antigens include A, B and C. Class 2 antigens include DP,DQ and DR


 When HLA matching for a renal transplant the relative importance of the HLA antigens are as
follows DR > B > A

Graft survival

 1 year = 90%, 10 years = 60% for cadaveric transplants


 1 year = 95%, 10 years = 70% for living-donor transplants
Post-op problems

 ATN of graft
 Vascular thrombosis
 Urine leakage
 UTI

Hyperacute acute rejection

 Due to antibodies against donor HLA type 1 antigens


 Rarely seen due to HLA matching

Acute graft failure (< 6 months)

 Usually due to mismatched HLA


 Other causes include cytomegalovirus infection
 Management: give steroids, if resistant use monoclonal antibodies

Causes of chronic graft failure (> 6 months)

 Chronic allograft nephropathy


 Ureteric obstruction
 Recurrence of original renal disease (MCGN > IgA > FSGS)
A 38 year old man is recovering following a live donor related renal transplant. The surgeon
prescribes corticosteroids to reduce the risk of graft rejection. Which of the following will not occur as
a result of their administration?

Suppression of macrophage activation

Reduction of expression of major histocompatability complex antigens on the graft

Reduction in the proliferation of lymphocytes

Necrosis of activated lymphocytes

Reduction of expression of endothelial cell adhesion molecules

Corticosteroids at higher doses are able to induce apoptosis of activated lymphocytes. Necrosis is a
different process and not induced by steroids.
Please rate this question:

Discuss and give feedback


Next question

Organ Transplant

A number of different organ and tissue transplants are now available. In many cases an allograft is
performed, where an organ is transplanted from one individual to another. Allografts will elicit an
immune response and this is one of the main reasons for organ rejection.

Graft rejection occurs because allografts have allelic differences at genes that code
immunohistocompatability complex genes. The main antigens that give rise to rejection are:

 ABO blood group


 Human leucocyte antigens (HLA)
 Minor histocompatability antigens

ABO Matching
ABO incompatibility will result in early organ rejection (hyperacute) because of pre existing
antibodies to other groups. Group O donors can give organs to any type of ABO recipient whereas
group AB donor can only donate to AB recipient.
HLA System
The four most important HLA alleles are:

 HLA A
 HLA B
 HLA C
 HLA DR

An ideal organ match would be one in which all 8 alleles are matched (remember 2 from each
parent, four each = 8 alleles). Modern immunosuppressive regimes help to manage the potential
rejection due to HLA mismatching. However, the greater the number of mismatches the worse the
long term outcome will be. T lymphocytes will recognise antigens bound to HLA molecules and will
then become activated. Clonal expansion then occurs with a response directed against that antigen.

Types of organ rejection

 Hyperacute. This occurs immediately through presence of pre formed antibodies (such as
ABO incompatibility).
 Acute. Occurs during the first 6 months and is usually T cell mediated. Usually tissue
infiltrates and vascular lesions.
 Chronic. Occurs after the first 6 months. Vascular changes predominate.

Hyperacute
Renal transplants at greatest risk and liver transplants at least risk. Although ABO incompatibility
and HLA Class I incompatible transplants will all fare worse in long term.

Acute
All organs may undergo acute rejection. Mononuclear cell infiltrates predominate. All types of
transplanted organ are susceptible and it may occur in up to 50% cases.

Chronic
Again all transplants with HLA mismatch may suffer this fate. Previous acute rejections and other
immunosensitising events all increase the risk. Vascular changes are most prominent with
myointimal proliferation leading to organ ischaemia. Organ specific changes are also seen such as
loss of acinar cells in pancreas transplants and rapidly progressive coronary artery disease in
cardiac transplants.

Surgical overview-Renal transplantation


A brief overview of the steps involved in renal transplantation is given.
Patients with end stage renal failure who are dialysis dependent or likely to become so in the
immediate future are considered for transplant. Exclusion criteria include; active malignancy, old age
(due to limited organ availability). Patients are medically optimised.
Donor kidneys, these may be taken from live related donors and close family, members may have
less HLA mismatch than members of the general population. Laparoscopic donor nephrectomy
further minimises the operative morbidity for the donor. Other organs are typically taken from brain
dead or dying patients who have a cardiac arrest and in whom resuscitation is futile. The key event
is to minimise the warm ischaemic time in the donor phase.

The kidney once removed is usually prepared on the bench in theatre by the transplant surgeon
immediately prior to implantation and factors such as accessory renal arteries and vessel length are
assessed and managed.

For first time recipients the operation is performed under general anaesthesia. A Rutherford-
Morrison incision is made on the preferred side. This provides excellent extraperitoneal access to
the iliac vessels. The external iliac artery and vein are dissected out and following systemic
heparinisation are cross clamped. The vein and artery are anastamosed to the iliacs and the clamps
removed. The ureter is then implanted into the bladder and a stent is usually placed to maintain
patency. The wounds are then closed and the patient recovered from surgery.

In the immediate phase a common problem encountered in cadaveric kidneys is acute tubular
necrosis and this tends to resolve.

Graft survival times from cadaveric donors are typically of the order of 9 years and monozygotic twin
transplant (live donor) may survive as long as 25 years.
Next question
A 52 year old female underwent a cadaveric renal transplant and recovers well post operatively. Her
immunosupression regime consists of tacrolimus. Which of the substances listed below should be
avoided?

Paracetamol

Apple juice

Penicillin

Prune juice

Grapefruit juice

Tacrolimus is metabolised by the P450 enzyme system. This is inhibited by a number of naturally
occurring substances, these include grapefruit, watercress and St.Johns Wort. These should all be
avoided in immunosupressed patients taking tacrolimus.
Please rate this question:

Discuss and give feedback

Organ transplantation: immunosupressants

A number of drugs are available which help to mitigate the processes resulting in acute rejection.
Cyclosporin and tacrolimus are commonly used drugs.

Example regime

 Initial: ciclosporin/tacrolimus with a monoclonal antibody


 Maintenance: ciclosporin/tacrolimus with MMF or sirolimus
 Add steroids if more than one steroid responsive acute rejection episode

Ciclosporin

 Inhibits calcineurin, a phosphatase involved in T cell activation


 Nephrotoxic
 Monitor levels
Azathioprine

 Metabolised to form 6 mercaptopurine which inhibits DNA synthesis and cell division
 Side effects include myelosupression, alopecia and nausea

Tacrolimus

 Lower incidence of acute rejection compared to ciclosporin


 Also less hypertension and hyperlipidaemia
 However, high incidence of impaired glucose tolerance and diabetes

Mycophenolate mofetil (MMF)

 Blocks purine synthesis by inhibition of IMPDH


 Therefore inhibits proliferation of B and T cells
 Side-effects: GI and marrow suppression

Sirolimus (rapamycin)

 Blocks T cell proliferation by blocking the IL-2 receptor


 Can cause hyperlipidaemia

Monoclonal antibodies

 Selective inhibitors of IL-2 receptor


 Daclizumab
 Basilximab
Theme: Spinal disorders

A. Osteomyelitis
B. Potts disease of the spine
C. Scheuermanns disease
D. Transverse myelitis
E. Tabes dorsalis
F. Subacute degeneration of the cord
G. Brown-Sequard syndrome
H. Syringomyelia
I. Epidural haematoma

Which is the most likely diagnosis for the scenario given. Each option may be used once, more than
once or not at all.

1. A 68 year old man presents to the plastics team with severe burns to his hands. He is not
distressed by the burns. He has bilateral charcot joints. On examination; there is loss of pain
and temperature sensation of the upper limbs.

You answered Osteomyelitis

The correct answer is Syringomyelia

This patient has syringomyelia which selectively affects the spinotholamic tracts.
Syringomyelia is a disorder in which a cystic cavity forms within the spinal cord. The
commonest variant is the Arnold- Chiari malformation in which the cavity connects with a
congenital malformation affecting the cerebellum. Acquired forms of the condition may
occur as a result of previous meningitis, surgery or tumours. Many neurological
manifestations have been reported, although the classical variety spares the dorsal columns
and medial lemniscus and affecting only the spinothalamic tract with loss of pain and
temperature sensation. The bilateral distribution of this patients symptoms would therefore
favor syringomyelia over SCID or Brown Sequard syndrome. Osteomyelitis would tend to
present with back pain and fever in addition to any neurological signs. Epidural haematoma
large enough to produce neurological impairment will usually have motor symptoms in
addition to any selective sensory loss, and the history is usually shorter.

2. A 24 year old man presents with localised spinal pain over 2 months which is worsened on
movement. He is known to be an IVDU. He has no history suggestive of tuberculosis. The
pain is now excruciating at rest and not improving with analgesia. He has a temperature of
39 oC.

Osteomyelitis

In an IVDU with back pain and pyrexia have a high suspicion for osteomylelitis. The most
likely organism is staph aureus and the cervical spine is the most common region affected.
TB tends to affect the thoracic spine and in other causes of osteomyelitis the lumbar spine is
affected.

3. A 22 year man is shot in the back, in the lumbar region. He has increased tone and hyper-
reflexia of his right leg. He cannot feel his left leg.

You answered Osteomyelitis

The correct answer is Brown-Sequard syndrome

Theme from January 2012 exam


Brown -Sequard syndrome is caused by hemisection of the spinal cord. It may result from
stab injuries or lateral vertebral fractures. It results in ipsilateral paralysis (pyramidal tract) ,
and also loss of proprioception and fine discrimination (dorsal columns). Pain and
temperature sensation are lost on the contra-lateral side. This is because the fibres of the
spinothalamic tract have decussated below the level of the cord transection.

Please rate this question:

Discuss and give feedback


Next question

Spinal disorders

Dorsal column lesion  Loss vibration and proprioception


 Tabes dorsalis, SACD

Spinothalamic tract  Loss of pain, sensation and temperature


lesion

Central cord lesion  Flaccid paralysis of the upper limbs

Osteomyelitis  Normally progressive


 Staph aureus in IVDU, normally cervical region affected
 Fungal infections in immunocompromised
 Thoracic region affected in TB

Infarction spinal cord  Dorsal column signs (loss of proprioception and fine
discrimination)

Cord compression  UMN signs


 Malignancy
 Haematoma
 Fracture

Brown-sequard  Hemisection of the spinal cord


syndrome  Ipsilateral paralysis
 Ipsilateral loss of proprioception and fine discrimination
 Contralateral loss of pain and temperature
Image sourced from Wikipedia
Image sourced from Wikipedia

Dermatomes
 C2 to C4 The C2 dermatome covers the occiput and the top part of the neck. C3 covers the
lower part of the neck to the clavicle. C4 covers the area just below the clavicle.
 C5 to T1 Situated in the arms. C5 covers the lateral arm at and above the elbow. C6 covers
the forearm and the radial (thumb) side of the hand. C7 is the middle finger, C8 is the medial
aspect of the hand, and T1 covers the medial side of the forearm.
 T2 to T12 The thoracic covers the axillary and chest region. T3 to T12 covers the chest and
back to the hip girdle. The nipples are situated in the middle of T4. T10 is situated at the
umbilicus. T12 ends just above the hip girdle.
 L1 to L5 The cutaneous dermatome representing the hip girdle and groin area is innervated
by L1 spinal cord. L2 and 3 cover the front part of the thighs. L4 and L5 cover medial and
lateral aspects of the lower leg.
 S1 to S5 S1 covers the heel and the middle back of the leg. S2 covers the back of the
thighs. S3 cover the medial side of the buttocks and S4-5 covers the perineal region. S5 is of
course the lowest dermatome and represents the skin immediately at and adjacent to the
anus.

Myotomes

Upper limb
Elbow flexors/Biceps C5

Wrist extensors C6

Elbow extensors/Triceps C7

Long finger flexors C8

Small finger abductors T1

Lower limb
Hip flexors (psoas) L1 and L2

Knee extensors (quadriceps) L3

Ankle dorsiflexors (tibialis anterior) L4 and L5

Toe extensors (hallucis longus) L5


Ankle plantar flexors (gastrocnemius) S1

The anal sphincter is innervated by S2,3,4


Next question
A 24 year old man is brought to the emergency department have suffered a crush injury to his
forearm. Assessment demonstrates that the arm is tender, red and swollen. There is clinical
evidence of an ulnar fracture and the patient cannot move their fingers. Which is the most
appropriate course of action?

Application of an external fixation device

Closed reduction

Debridement

Discharge and review in fracture clinic

Fasciotomy

Theme from April 2012


The combination of a crush injury, limb swelling and inability to move digits should raise suspicion of
a compartment syndrome that will require a fasciotomy. Paralysis is a very late sign.
Please rate this question:

Discuss and give feedback


Next question

Compartment syndrome

 This is a particular complication that may occur following fractures (or following ischaemia re-
perfusion injury in vascular patients). It is characterised by raised pressure within a closed
anatomical space.
 The raised pressure within the compartment will eventually compromise tissue perfusion
resulting in necrosis. The two main fractures carrying this complication include supracondylar
fractures and tibial shaft injuries.

Symptoms and signs

 Pain, especially on movement (even passive)


 Parasthesiae
 Pallor may be present
 Arterial pulsation may still be felt as the necrosis occurs as a result of microvascular
compromise
 Paralysis of the muscle group may occur
Diagnosis

 Is made by measurement of intracompartmental pressure measurements. Pressures in


excess of 20mmHg are abnormal and >40mmHg is diagnostic.

Treatment

 This is essentially prompt and extensive fasciotomies


 In the lower limb the deep muscles may be inadequately decompressed by the
inexperienced operator when smaller incisions are performed
 Myoglobinuria may occur following fasciotomy and result in renal failure and for this reason
these patients require aggressive IV fluids
 Where muscle groups are frankly necrotic at fasciotomy they should be debrided and
amputation may have to be considered
 Death of muscle groups may occur within 4-6 hours

Next question
Theme: Disorders of the knee

A. Chondromalacia patellae
B. Dislocated patella
C. Undisplaced fracture patella
D. Displaced patella fracture
E. Avulsion fracture of the tibial tubercle
F. Quadriceps tendon rupture
G. Osgood Schlatters disease

Please select the most likely explanation for the scenario given. Each option may be used once,
more than once or not at all.

5. A 19 year old sportswoman presents with knee pain which is worse on walking down the
stairs and when sitting still. On examination there is wasting of the quadriceps and
pseudolocking of the knee.

Chondromalacia patellae

A teenage girl with knee pain on walking down the stairs is characteristic for
chondromalacia patellae (anterior knee pain). Most cases are managed with physiotherapy.

6. A tall 18 year old male athlete is admitted to the emergency room after being hit in the knee
by a hockey stick. On examination his knee is tense and swollen. X-ray shows no fractures.

You answered Chondromalacia patellae

The correct answer is Dislocated patella

A patella dislocation is a common cause of haemarthrosis and many will spontaneously


reduce when the leg is straightened. In the chronic setting physiotherapy is used to
strengthen the quadriceps muscles.

7. An athletic 15 year old boy presents with knee pain of 3 weeks duration. It is worst during
activity and settles with rest. On examination there is tenderness overlying the tibial
tuberosity and an associated swelling at this site.

You answered Chondromalacia patellae

The correct answer is Osgood Schlatters disease

Athletic boys and girls may develop this condition in their teenage years. It is caused by
multiple micro fractures at the point of insertion of the tendon into the tibial tuberosity.
Most cases settle with physiotherapy and rest.

Please rate this question:

Discuss and give feedback


Next question

Knee injury

Types of injury

Ruptured anterior  Sport injury


cruciate ligament  Mechanism: high twisting force applied to a bent knee
 Typically presents with: loud crack, pain and RAPID joint
swelling (haemoarthrosis)
 Poor healing
 Management: intense physiotherapy or surgery

Ruptured posterior  Mechanism: hyperextension injuries


cruciate ligament  Tibia lies back on the femur
 Paradoxical anterior draw test

Rupture of medial  Mechanism: leg forced into valgus via force outside the leg
collateral ligament  Knee unstable when put into valgus position

Menisceal tear  Rotational sporting injuries


 Delayed knee swelling
 Joint locking (Patient may develop skills to "unlock" the knee
 Recurrent episodes of pain and effusions are common, often
following minor trauma

Chondromalacia  Teenage girls, following an injury to knee e.g. Dislocation


patellae patella
 Typical history of pain on going downstairs or at rest
 Tenderness, quadriceps wasting

Dislocation of the  Most commonly occurs as a traumatic primary event, either


patella through direct trauma or through severe contraction of
quadriceps with knee stretched in valgus and external rotation
 Genu valgum, tibial torsion and high riding patella are risk
factors
 Skyline x-ray views of patella are required, although displaced
patella may be clinically obvious
 An osteochondral fracture is present in 5%
 The condition has a 20% recurrence rate

Fractured patella  2 types:

i. Direct blow to patella causing undisplaced fragments


ii. Avulsion fracture

Tibial plateau fracture  Occur in the elderly (or following significant trauma in young)
 Mechanism: knee forced into valgus or varus, but the knee
fractures before the ligaments rupture
 Varus injury affects medial plateau and if valgus injury, lateral
plateau depressed fracture occurs
 Classified using the Schatzker system (see below)

Schatzker Classification system for tibial plateau fractures


Type Anatomical description Features

1 Vertical split of lateral Fracture through dense bone, usually in the young. It may be
condyle virtually undisplaced, or the condylar fragment may be
pushed inferiorly and tilted

2 Vertical split of the lateral The wedge fragment (which may be of variable size), is
condyle combined with an displaced laterally; the joint is widened. Untreated, a valgus
adjacent load bearing part deformity may develop
of the condyle

3 Depression of the articular The split does not extend to the edge of the plateau.
surface with intact condylar Depressed fragments may be firmly embedded in
rim subchondral bone, the joint is stable

4 Fragment of the medial Two injuries are seen in this category; (1) a depressed
tibial condyle fracture of osteoporotic bone in the elderly. (2) a high
energy fracture resulting in a condylar split that runs from
the intercondylar eminence to the medial cortex. Associated
Type Anatomical description Features

ligamentous injury may be severe

5 Fracture of both condyles Both condyles fractured but the column of the metaphysis
remains in continuity with the tibial shaft

6 Combined condylar and High energy fracture with marked comminution


subcondylar fractures

Next question
An 8 year old boy presents with symptoms of right knee pain. The pain has been present on most
occasions for the past three months and the pain typically lasts for several hours at a time. On
examination; he walks with an antalgic gait and has apparent right leg shortening. What is the most
likely diagnosis?

Perthes Disease

Osteosarcoma of the femur

Osteoarthritis of the hip

Transient synovitis of the hip

Torn medial meniscus

Theme from September 2012 Exam


There are many causes of the irritable hip in the 10-14 year age group. Many of these may cause
both hip pain or knee pain. Transient synovitis of the hip the commonest disorder but does not
typically last for 3 months. An osteosarcoma would not usually present with apparent limb shortening
unless pathological fracture had occurred. A slipped upper femoral epiphysis can cause a similar
presentation although it typically presents later and with different patient characteristics.
Please rate this question:

Discuss and give feedback


Next question

Perthes disease

Perthes disease

 Idiopathic avascular necrosis of the femoral epiphysis of the femoral head


 Impaired blood supply to femoral head, causing bone infarction. New vessels develop and
ossification occurs. The bone either heals or a subchondral fracture occurs.

Clinical features

 Males 4x's greater than females


 Age between 2-12 years (the younger the age of onset, the better the prognosis)
 Limp
 Hip pain
 Bilateral in 20%

Diagnosis
Plain x-ray, Technetium bone scan or magnetic resonance imaging if normal x-ray and symptoms
persist.

Catterall staging
Stage Features

Stage 1 Clinical and histological features only

Stage 2 Sclerosis with or without cystic changes and preservation of the articular surface

Stage 3 Loss of structural integrity of the femoral head

Stage 4 Loss of acetabular integrity

Management

 To keep the femoral head within the acetabulum: cast, braces


 If less than 6 years: observation
 Older: surgical management with moderate results
 Operate on severe deformities

Prognosis
Most cases will resolve with conservative management. Early diagnosis improves outcomes.
Next question
Which of the following types of growth plate fractures may have similar radiological appearances?

Salter Harris types 1 and 5

Salter Harris types 4 and 5

Salter Harris types 3 and 5

Salter Harris types 1 and 2

Salter Harris types 1 and 3

Mnemonic: SALTER

S (Type 1): Straight through the growth plate


A (Type 2): Above - through growth plate and Above involving the metaphysis
L (Type 3): Lower -through growth plate and beLow involving the epiphysis
T (Type 4):Through - Through both metaphysis, epiphysis and growth plate
E (Type 5): Everything - Crush / compression injury
R (Type 5): Ruined
As recommended by one of our users

Salter Harris injury types 1 and 5 (transverse fracture through growth plate Vs. Compression
fracture) may mimic each other radiologically. Type 5 injuries have the worst outcomes. Radiological
signs of type 5 injuries are subtle and may include narrowing of the growth plate.

Please rate this question:

Discuss and give feedback

Next question

Epiphyseal fractures
Fractures involving the growth plate in children are classified using the Salter - Harris system.
There are 5 main types.

Salter Harris Classification

Type Description

Type 1 Transverse fracture through the growth plate

Type 2 Fracture through the growth plate to the metaphysis (commonest type)

Type 3 Fracture through the growth plate and the epiphysis with metaphysis spared

Type 4 Fracture involving the growth plate, metaphysis and epiphysis

Type 5 Compression fracture of the growth plate (worst outcome)

Management
Non displaced type 1 injuries can generally be managed conservatively. Unstable or more extensive
injuries will usually require surgical reduction and/ or fixation, as proper alignment is crucial.

Next question
Theme: Pathological fractures

A. Osteosarcoma
B. Osteomalacia
C. Osteoporosis
D. Metastatic carcinoma
E. Osteoblastoma
F. Giant cell tumour
G. Ewing's sarcoma

For each pathological fracture please select the most likely aetiology for the scenario given. Each
option may be used once, more than once or not at all.

10. A 30 year old woman presents with pain and swelling of the left shoulder. There is a large
radiolucent lesion in the head of the humerus extending to the subchondral plate.

You answered Osteosarcoma

The correct answer is Giant cell tumour

Giant cell tumours on x-ray have a 'soap bubble' appearance. They present as pain or
pathological fractures. They commonly metastasize to the lungs.

11. A 72 year old woman has a lumbar vertebral crush fracture. She has hypocalcaemia and a
low urinary calcium.

You answered Osteosarcoma

The correct answer is Osteomalacia

Hypocalcemia and low urinary calcium are biochemical features of osteomalacia.


Unfortunately surgeons do need to look at some blood results!

12. A 16 year old boy presents with severe groin pain after kicking a football. Imaging
confirms a pelvic fracture. A previous pelvic x-ray performed 2 weeks ago shows a lytic
lesion with 'onion type' periosteal reaction.

You answered Osteosarcoma

The correct answer is Ewing's sarcoma

A Ewings sarcoma is most common in males between 10-20 years. It can occur in girls. A
lytic lesion with a lamellated or onion type periosteal reaction is a classical finding on x-
rays. Most patients present with metastatic disease with a 5 year prognosis between 5-
10%.

Please rate this question:

Discuss and give feedback


Next question

Pathological fractures

 A pathological fracture occurs in abnormal bone due to insignificant injury

Causes

Metastatic tumours  Breast


 Lung
 Thyroid
 Renal
 Prostate

Bone disease  Osteogenesis imperfecta


 Osteoporosis
 Metabolic bone disease
 Paget's disease

Local benign conditions  Chronic osteomyelitis


 Solitary bone cyst

Primary malignant tumours  Chondrosarcoma


 Osteosarcoma
 Ewing's tumour

Next question
An 8 year old boy falls onto an outstretched hand and is brought to the emergency department. He is
examined by a doctor and a bony injury is cleared clinically. He re-presents a week later with pain in
his hand. What is the most likely underlying injury?

Fracture of the distal radius

Fracture of the scaphoid

Dislocation of the lunate

Rupture of flexor pollicis longus tendon

Bennett's fracture

Theme from January 2013 Exam


Scaphoid fractures in children are rare, will usually involve the distal pole and are easily missed. The
initial clinical examination (and sometimes x-rays) may be normal and repeated clinical examination
and imaging is advised for this reason. Whilst the other injuries may be sustained from a fall onto an
outstretched hand they are less likely to be overlooked on clinical examination. In the case of a
Bennetts fracture, the injury mechanism is less compatible with this type of injury.
Please rate this question:

Discuss and give feedback


Next question

Scaphoid fractures

 Scaphoid fractures are the commonest carpal fractures.


 Surface of scaphoid is covered by articular cartilage with small area available for blood
vessels (fracture risks blood supply)
 Forms floor of anatomical snuffbox
 Risk of fracture associated with fall onto outstretched hand (tubercle, waist, or proximal third)
 Ulnar deviation AP needed for visualization of scaphoid
 Immobilization of scaphoid fractures difficult

Management
Non-displaced fractures - Casts or splints
- Percutaneous scaphoid fixation
Displaced fracture Surgical fixation, usually with a screw

Complications

 Non union of scaphoid


 Avascular necrosis of the scaphoid
 Scapholunate disruption and wrist collapse
 Degenerative changes of the adjacent joint

Next question
Theme: Shoulder injuries

A. Glenohumeral dislocation
B. Acromioclavicular dislocation
C. Sternoclavicular dislocation
D. Biceps tendon tear
E. Supraspinatus tear
F. Fracture of the surgical neck of the humerus
G. Infra spinatus tear

For each scenario please select the most likely underlying diagnosis. Each option may be used
once, more than once or not at all.

14. A 23 year old rugby player falls directly onto his shoulder. There is pain and swelling of
the shoulder joint. The clavicle is prominent and there appears to be a step deformity.

You answered Glenohumeral dislocation

The correct answer is Acromioclavicular dislocation

Acromioclavicular joint (ACJ) dislocation normally occurs secondary to direct injury to


the superior aspect of the acromion. Loss of shoulder contour and prominent clavicle are
key features. Note; rotator cuff tears rarely occur in the second decade.

15. A 22 year old man falls over and presents to casualty. A shoulder x-ray is performed, the
radiologist comments that a Hill-Sachs lesion is present.

Glenohumeral dislocation

A Hill-Sachs lesion occurs when the cartilage surface of the humerus is in contact with the
rim of the glenoid. About 50% of anterior glenohumeral dislocations are associated with
this lesion.

16. An 82 year old female presents to A&E after tripping on a step. She complains of shoulder
pain. On examination there is pain to 90o on abduction.

You answered Glenohumeral dislocation

The correct answer is Supraspinatus tear

A supraspinatus tear is the most common of rotator cuff tears. It occurs as a result of
degeneration and is rare in younger adults.
Please rate this question:

Discuss and give feedback


Next question

Shoulder disorders

Shoulder fractures and dislocations


Fractures
Proximal humerus
Background

 Third most common fragility fracture in the elderly.


 Results from low energy fall in predominantly elderly females, or from high energy trauma in
young males.
 Can be associated with nerve injury (commonly axillary), and fracture-dislocation of the
humeral head. Detailed neurological assessment is essential for all upper limb injuries.

Anatomy
Osteology
Consists of articular head, greater tuberosity, lesser tuberosity, metaphysis and diaphysis. Between
the articular head and the tuberosities is the anatomical neck (previous physis). Between the
tuberosities and the metaphysis is the surgical neck.
The supraspinatus, infraspinatus and teres minor muscles attach to the greater tuberosity. The
subscapularis muscle attaches to the lesser tuberosity.

Vascular Supply
Humeral head is supplied by the anterior and posterior humeral circumflex arteries. Anatomical neck
fractures are at greatest risk of osteonecrosis.

Imaging
Imaging aims to both delineate the fracture pattern, and confirm/exlude the presence of an
associated dislocation.

 Radiographs - True anteroposterior (AP), axillary lateral and/or scapula Y view.


 CT - indicated to better define intra-articular involvement and to aid pre-operative planning.
MRI is not useful for fracture imaging.

Classification
Description of the fracture is often more useful than classification. Particular attention should be paid
to humeral alignment, fracture displacement, and greater tuberosity position (rotator cuff will pull the
GT supero-posterioly, which can cause impingement problems with malunion).
- Neer Classification: Most commonly used. Describes fracture as 2,3,or 4 part depending upon the
number main fragments. Also comments on the degree of displacement. Fragments:
-greater tuberosity
-lesser tuberosity
- articular surface
- shaft
Displacement: >1cm or angulation >45 degrees.

Treatment
The vast majority of proximal humeral fractures are minimally displaced, and therefore can be
managed conservatively. This involves immobilisation in a polysling, and progressive mobilisation.
Pendular exercise can commence at 14 days, and active abduction from 4-6 weeks.

Irreducible fracture dislocation is an indication for operative management. Other indications include
large displacement, younger patient, head splitting (intra-articular fractures). However, the recent
PROFHER trial (1) has suggested no benefit to operative intervention on patient outcome (it must be
applied cautiously as majority of patients were elderly with extraarticular fractures). Options available
for surgical management include:

ORIF Most commonly used. Plate and screw fixation. Can reconstruct
complex fractures.

Intramedullary nail Suitable for extra-articular configuration, predominantly surgical neck


+/- GT fractures.

Used for un-reconstructable fractures in the older patient who has good
Hemiarthroplasty glenoid quality.

Total shoulder Unconstructable fractures where high functioning shoulder is required


arthroplasty (hemiarthroplasty will cause glenoid erosion)

Reverse shoulder Total shoulder arthroplasty that provides better functional outcome than
arthroplasty conventional total shoulder replacement.

Scapula
Background
Uncommon fractures usually associated with high energy trauma. Most commonly involve scapula
body or spine (50%), glenoid fossa and glenoid neck. Important to exclude associated life
threatening injury.

Imaging
Plain radiographs should include true anteroposterior (AP), axillary lateral and/or scapula Y view. CT
scanning is useful for defining intra-articular involvement, displacement and for three dimensional
reconstruction.

Classification
Based on the location of the fracture (coracoid, acromion, glenoid neck, glenoid fossa, scapula
body). Beware of ipsilateral glenoid neck and clavicle fracture -floating shoulder - where limb is
effectively dissociated from axial skeleton.

Treatment
The vast majority of scapula fractures are amenable to conservative management, consisting of
sling immobilisation for two weeks followed by early rehabilitation. Floating shoulder will usually
require fixation, and consideration of surgery should also be given to intra-articular and
displaced/angulated glenoid fractures.

Dislocations

Types
Dislocations around the shoulder joint include glenohumeral dislocation, acromioclavicular joint
disruption and sternoclavicular dislocation. Only glenohumeral dislocation will be covered here.

Glenohumeral dislocation
Diagnosis, classification and management are covered here.

Background
Shoulder dislocation is commonly seen in A&E. It has a high recurrence rate that is as high as 80%
in teenagers. Initial management requires emergent reduction to prevent lasting chondral damage.

Early assessment and management


Usually a traumatic cause (multi-directional instability in frequent dislocations requires discussion
with orthopaedics and is not covered here). Careful history, examination and documentation of
neurovascular status of the limb, in particular the axillary nerve (regimental badge sensation). This
should be re-assessed post manipulation. Early radiographs to confirm direction of dislocation.

Initial management consists of emergent closed reduction under under entanox and analgesia, but
often requires conscious sedation. Arm should then be immobilised in a polysling, and XR to confirm
relocation.

Imaging - True anteroposterior (AP), axillary lateral and/or scapula Y view. Reduced humeral head
should lie between acromion and coracoid on lateral/scapula view.

Types
Reduction
Direction Features Cause Examination techniques

Anterior Most Usually traumatic - anterior Loss of shoulder


Common force on arm when shoulder contour - sulcus sign. Hippocratic.
>90% is abducted, eternally Humeral head can be Milch.
rotated felt anteriorly. Stimson.

Kocher not
advised due to
complication of
fracture
Reduction
Direction Features Cause Examination techniques

Posterior 50% 50% traumatic, but Shoulder locked in Gentle lateral


missed in classically post seizure or internal rotation. XR traction to
A&E electrocution may show lightbulb adducted arm.
appearance.

Inferior Rare Associated with pectorals As for primary injury Management of


and rotator cuff tears, and primary injury
glenoid fracture

Rare Associated with As for primary injury Management of


Superior acrominon/clavicle fracture primary injury

Associated injuries

 Bankart lesion - avulsion of the anterior glenoid labrum with an anterior shoulder dislocation
(reverse Bankart if poster labrum in posterior dislocation).
 Hill Sachs defect - chondral impaction on posteriosuperior humeral head from contact with
gleonoid rim. Can be large enough to lock shoulder, requiring open reduction. (Reverse Hill
Sachs in posterior dislocation).
 Rotator cuff tear - increases with age.
 Greater or lesser tuberosity fracture - increases with age.
 Humeral neck fracture - shoulder fracture dislocation. More common in high energy trauma
and elderly. Should be discussed with orthopaedics prior to any attempted reduction.

Rotator Cuff Disease

Rotator cuff disease is a spectrum of conditions that ranges from subacromial impingement to rotator
cuff tears and eventually to rotator cuff arthropathy (arthritis).

Anatomy
The rotator cuff is a group of four muscles that are important in shoulder movements, and
maintenance of glenohumeral stability.
Scapular Humeral
Muscle attachment attachment Action Innervation
Scapular Humeral
Muscle attachment attachment Action Innervation

Supraspinatus Supraspinatus Superior facet of Initiation of Suprascapular nerve


fossa greater tuberosity abduction of
humerus

Infraspinatus Infraspinatus Posterior facet of External rotation Suprascapular nerve


fossa greater tuberosity of humerus

Teres Minor Lateral border Inferior facet of External rotation Axillary Nerve
greater tuberosity of humerus

Subscapularis Subscapular Lesser tuberosity Internal rotation Upper and lower


fossa of humerus subscapular nerve

 The inferior rotator cuff muscles (infraspinatus, teres minor, and subscapularis) balance the
superior pull of the deltoid. Injury/tear results in upward migration of the humeral head on the
glenoid (can be seen on AP radiograph).
 Likewise, the anterior muscles (subscapularis) are balanced with the posterior muscles
(infraspinatus, teres minor).

Subacromial Impingement

 The most common cause of shoulder pain, which results from impingement of the superior
cuff on the undersurface of the acromion, and an inflammatory bursitis.
 Associated with certain types of acromial morphology (Bigliani classification).
 Presents as insidious pain which is exacerbated by overhead activities.

Rotator Cuff Tear

 Often presents as an acute event on the background of chronic subacromial impingement in


the older patient, but can present as an avulsion injury in younger patients.
 Majority of tears are to the superior cuff (supraspinatus, infraspinatus, teres minor), though a
tear to subscapularis is associated with subcoracoid impingement.
 Tears present as pain and weakness when using the muscles in question.

Rotator Cuff Arthropathy

 Defined as shoulder arthritis in the setting of rotator cuff dysfunction. Results from superior
migration due to the loss of rotator cuff function and integrity. Unopposed deltoid pulls the
humeral head superiorly.
 Associated with massive chronic cuff tears.

Imaging

Plain radiographs

 AP of the shoulder may show superior migration of the humerus with a cuff tear, and features
of arthritis with arthropathy. Other causes of pain may also be identified (e.g. calcific
tendonitis/fracture)
 Outlet view is useful for defining the acromial morphology

USS

 Allows dynamic imaging of the cuff, and is inexpensive. However, it is very user dependent.

MRI

 Best imaging modality for cuff pathology.


 Also allows imaging of the rest of the shoulder. When intra-articular pathology is suspected,
can be combined with an arthrogram for improved sensitivity and specificity.

Treatment

Subacromial impingement

 Physiotherapy, oral anti-inflammatory medication


 Subacromial steroid injection can settle inflammation
 Arthroscopic subacromial decompression by shaving away the undersurface of the
acromion, more space is created for the rotator cuff. Cuff integrity is assessed also at time of
surgery, and can be repaired if necessary.

Rotator cuff tear


 When considering repair of a cuff tear, the age and activity of the patient, the nature of the
tear (degenerative vs. acute traumatic), and the size and retraction of the tear should be
considered when making a surgical plan.
 Mild tears or tears in the elderly can be managed conservatively, as outlined above.
 Moderate tears can be repaired arthroscopically. Massive or retracted tears will often require
an open repair (occasionally with a tendon transfer). Subacromial decompression is
performed at the same time to reduce impingement, symptoms and recurrence.

Calcific tendonitis
Calcific tendonitis involves calcific deposits within tendons anywhere in the body, but most
commonly in the rotator cuff (specifically the supraspinatus tendon). When present in the shoulder, it
is associated with subacromial impingement and pain.

Pathology

 More common in women aged 30-60 years.


 Association with diabetes and hypothyroidism

There are three stages of calcification

 Formative phase characterized by calcific deposits


 Resting phase deposit is stable, but presents with impingement problems
 Resorptive phase phagocytic resorption. Most painful stage.

Presentation

 Similar in presentation to subacromial impingement, with pain especially with over head
activities. Atraumatic in nature.

Imaging

 Plain radiographs show calcification of the rotator cuff, usually within 1.5cm of its insertion on
the humerus. Supraspinatus outlet views can show level of impingment. Further imaging is
rarely needed.

Treatment

 Non-operative NSAIDS, steroid injection (controversial, but practiced) and physiotherapy.


Approximately 75% will resolve by 6 months with conservative management.
 Ultrasound guided or surgical needle barbotage can break down deposits and resolve
symptoms. Occasionally surgical excision is required.
Adhesive capsulitis (Frozen Shoulder)

 Pain and loss of movement of shoulder joint, which involves fibroplastic proliferation of
capsular tissue, causing soft tissue scarring and contracture. Patients present with a painful
and decreased arc of motion.
 Associated with prolonged immobilization, previous surgery, thyroid disorders (AI) and
diabetes
 Classically three stages which can take up to two years to resolve:

Stage one the freezing and painful stage


Stage two the frozen and stiff stage
Stage three the thawing stage, where shoulder movement slowly improves

Imaging

 Plain radiographs to exclude other causes of a painful shoulder


 MRI arthrogram may show capsular contracture, and again may be used to exclude cuff
pathology. However, often not performed as diagnosis is largely clinical.

Treatment

 Non-operative NSAIDS, steroid injection and physiotherapy. Patience is required as


condition can take up to 2 years to improve.
 Operative MUA or arthroscopic adhesiolysis (release of adhesions) can expedite recovery,
followed by intensive physiotherapy.

Glenohumeral Arthritis
Shoulder arthritis presents with the normal symptoms of arthritis, however primary osteoarthritis is
not as commonly the primary cause as seen in other large joints.. A large proportion of shoulder
arthritis is rheumatoid (RA), post traumatic, or secondary to rotator cuff arthropathy (discussed
above).

Treatment

 Surgical options include hemiarthroplasty, total shoulder replacement and reverse geometry
total shoulder replacement (used when rotator cuff function is absent).

References
1. JAMA. 2015;313(10):1037-1047. doi:10.1001/jama.2015.1629
Next question
Which of the following statements relating to menisceal tears is false?

The medial meniscus is most often affected

True locking of the knee joint may occur

Most established tears will heal with conservative management

In the chronic setting there is typically little to find on examination if the knee is not
locked

An arthroscopic approach may be used to treat most lesions

Menisci have no nerve or blood supply and thus heal poorly. Established tears with associated
symptoms are best managed by arthroscopic menisectomy.
Please rate this question:

Discuss and give feedback


Next question

Knee injury

Types of injury

Ruptured anterior  Sport injury


cruciate ligament  Mechanism: high twisting force applied to a bent knee
 Typically presents with: loud crack, pain and RAPID joint
swelling (haemoarthrosis)
 Poor healing
 Management: intense physiotherapy or surgery

Ruptured posterior  Mechanism: hyperextension injuries


cruciate ligament  Tibia lies back on the femur
 Paradoxical anterior draw test

Rupture of medial  Mechanism: leg forced into valgus via force outside the leg
collateral ligament
 Knee unstable when put into valgus position

Menisceal tear  Rotational sporting injuries


 Delayed knee swelling
 Joint locking (Patient may develop skills to "unlock" the knee
 Recurrent episodes of pain and effusions are common, often
following minor trauma

Chondromalacia  Teenage girls, following an injury to knee e.g. Dislocation


patellae patella
 Typical history of pain on going downstairs or at rest
 Tenderness, quadriceps wasting

Dislocation of the  Most commonly occurs as a traumatic primary event, either


patella through direct trauma or through severe contraction of
quadriceps with knee stretched in valgus and external rotation
 Genu valgum, tibial torsion and high riding patella are risk
factors
 Skyline x-ray views of patella are required, although displaced
patella may be clinically obvious
 An osteochondral fracture is present in 5%
 The condition has a 20% recurrence rate

Fractured patella  2 types:

i. Direct blow to patella causing undisplaced fragments


ii. Avulsion fracture

Tibial plateau fracture  Occur in the elderly (or following significant trauma in young)
 Mechanism: knee forced into valgus or varus, but the knee
fractures before the ligaments rupture
 Varus injury affects medial plateau and if valgus injury, lateral
plateau depressed fracture occurs
 Classified using the Schatzker system (see below)

Schatzker Classification system for tibial plateau fractures


Type Anatomical description Features
Type Anatomical description Features

1 Vertical split of lateral Fracture through dense bone, usually in the young. It may be
condyle virtually undisplaced, or the condylar fragment may be
pushed inferiorly and tilted

2 Vertical split of the lateral The wedge fragment (which may be of variable size), is
condyle combined with an displaced laterally; the joint is widened. Untreated, a valgus
adjacent load bearing part deformity may develop
of the condyle

3 Depression of the articular The split does not extend to the edge of the plateau.
surface with intact condylar Depressed fragments may be firmly embedded in
rim subchondral bone, the joint is stable

4 Fragment of the medial Two injuries are seen in this category; (1) a depressed
tibial condyle fracture of osteoporotic bone in the elderly. (2) a high
energy fracture resulting in a condylar split that runs from
the intercondylar eminence to the medial cortex. Associated
ligamentous injury may be severe

5 Fracture of both condyles Both condyles fractured but the column of the metaphysis
remains in continuity with the tibial shaft

6 Combined condylar and High energy fracture with marked comminution


subcondylar fractures

Next question
Theme: Developmental bone disorders

A. Rickets
B. Craniocleidodysostosis
C. Achondroplasia
D. Scurvy
E. Pagets disease
F. Multiple myeloma
G. Osteogenesis imperfecta
H. Osteomalacia
I. Osteopetrosis
J. None of the above

Please select the most likely disease process to account for the clinical scenario. Each option may
be used once, more than once or not at all.

18. A 15 year-old boy presents to the out-patient clinic with tiredness, recurrent throat and
chest infections, and gradual loss of vision. Multiple x-rays show brittle bones with no
differentiation between the cortex and the medulla.

You answered Rickets

The correct answer is Osteopetrosis

Osteopetrosis is an autosomal recessive condition. It is commonest in young adults. They


may present with symptoms of anaemia or thrombocytopaenia due to decreased marrow
space. Radiology reveals a lack of differentiation between the cortex and the medulla
described as marble bone. These bones are very dense and brittle.

19. A 12 year-old boy who is small for his age presents to the clinic with poor muscular
development and hyper-mobile fingers. His x-rays show multiple fractures of the long
bones and irregular patches of ossification.

You answered Rickets

The correct answer is Osteogenesis imperfecta

Osteogenesis imperfecta is caused by defective osteoid formation due to congenital


inability to produce adequate intercellular substances like osteoid, collagen and dentine.
There is a failure of maturation of collagen in all the connective tissues.Radiology may
show translucent bones, multiple fractures, particularly of the long bones, wormian bones
(irregular patches of ossification) and a trefoil pelvis.

20. A 1 year-old is brought to the Emergency Department with a history of failure to thrive.
On examination, the child is small for age and has a large head. X-ray shows a cupped
appearance of the epiphysis of the wrist.

Rickets

Rickets is the childhood form of osteomalacia. It is due to the failure of the osteoid to
ossify due to vitamin D deficiency. Symptoms start about the age of one. The child is
small for age and there is a history of failure to thrive. Bony deformities include bowing of
the femur and tibia, a large head, deformity of the chest wall with thickening of the
costochondral junction (rickettary rosary), and a transverse sulcus in the chest caused by
the pull of the diaphragm (Harrison's sulcus). X- Rays show widening and cupping of the
epiphysis of the long bones, most readily apparent in the wrist.

Please rate this question:

Discuss and give feedback


Next question

Paediatric fractures

Paediatric fracture types


Type Injury pattern

Complete fracture Both sides of cortex are breached

Toddlers fracture Oblique tibial fracture in infants

Plastic deformity Stress on bone resulting in deformity without cortical disruption

Greenstick fracture Unilateral cortical breach only

Buckle fracture Incomplete cortical disruption resulting in periosteal haematoma only

Growth plate fractures


In paediatric practice fractures may also involve the growth plate and these injuries are classified
according to the Salter- Harris system (given below):
Type Injury pattern

I Fracture through the physis only (x-ray often normal)

II Fracture through the physis and metaphysis

III Fracture through the physis and epiphyisis to include the joint

IV Fracture involving the physis, metaphysis and epiphysis

V Crush injury involving the physis (x-ray may resemble type I, and appear normal)

As a general rule it is safer to assume that growth plate tenderness is indicative of an underlying
fracture even if the x-ray appears normal. Injuries of Types III, IV and V will usually require surgery.
Type V injuries are often associated with disruption to growth.

Non accidental injury

 Delayed presentation
 Delay in attaining milestones
 Lack of concordance between proposed and actual mechanism of injury
 Multiple injuries
 Injuries at sites not commonly exposed to trauma
 Children on the at risk register

Pathological fractures
Genetic conditions, such as osteogenesis imperfecta, may cause pathological fractures.

Osteogenesis imperfecta

 Defective osteoid formation due to congenital inability to produce adequate intercellular


substances like osteoid, collagen and dentine.
 Failure of maturation of collagen in all the connective tissues.
 Radiology may show translucent bones, multiple fractures, particularly of the long bones,
wormian bones (irregular patches of ossification) and a trefoil pelvis.

Subtypes
 Type I The collagen is normal quality but insufficient quantity.
 Type II- Poor collagen quantity and quality.
 Type III- Collagen poorly formed. Normal quantity.
 Type IV- Sufficient collagen quantity but poor quality.

Osteopetrosis

 Bones become harder and more dense.


 Autosomal recessive condition.
 It is commonest in young adults.
 Radiology reveals a lack of differentiation between the cortex and the medulla described as
marble bone.

Next question
Theme: Hip fractures

A. Conservative management

B. Percutaneous pinning

C. Fracture reduction and internal fixation

D. Hemiarthroplasty

E. Total hip replacement

F. Dynamic hip screw

G. Intramedullary femoral nail

For each scenario please select the most appropriate management option. Each option may be used
once, more than once or not at all.

21. A 60 year old male is admitted to A&E with a fall. He lives with his wife and still works as a
restaurant manager. He has a past history of benign prostatic hypertrophy and is currently taking
tamsulosin. He is otherwise fit and healthy. On examination there is right hip tenderness on
movement in all directions. A hip x-ray confirms an intertrochanteric fracture.

You answered Conservative management

The correct answer is Dynamic hip screw

The blood supply to the femoral head may be intact and the fracture should heal with
compression type devices such as gamma nails or dynamic hip screws. The latter device being the
most commonly performed therapeutic intervention.

22. An 86 year old retired pharmacist is admitted to A&E following a fall. She complains of right hip
pain. She is known to have hypertension and is currently on bendrofluazide. She lives alone and
mobilises with a Zimmer frame. Her right leg is shortened and externally rotated. A hip x-ray
confirms a displaced intracapsular fracture.

You answered Conservative management


The correct answer is Hemiarthroplasty

Hemiarthroplasty is offered to older, less mobile individuals compared to fracture reduction and
fixation in younger patients.

23. A 74 year old male is admitted to A&E with a fall. He is known to have rheumatoid arthritis and is
on methotrexate and paracetamol. He lives alone in a bungalow and enjoys playing golf. He is
independent with his ADLs. He complains of left groin pain, therefore has a hip x-ray which
confirms a displaced intracapsular fracture.

You answered Conservative management

The correct answer is Total hip replacement

This patient has pre-existing joint disease, good level of activity and a relatively high life
expectancy, therefore THR is preferable to hemiarthroplasty.

Please rate this question:

Discuss and give feedback

Next question

Hip fractures

Background
Neck of femur (NOF) fracture is a common orthopaedic presentation, with over 65000 fractures in
the UK per year. Like many orthopaedic injuries, there is a bimodal age distribution. It is imperative
to distinguish between the high energy injury in a young patient, and the low energy osteoporotic
fracture in the elderly, as their management aims are very different:

Young patient - Usually high energy trauma (e.g road traffic accident, horse riding) and needs
treating in accordance with Advanced Trauma Life Support (ATLS) principles. Will often have
associated injuries. Aim is to retain the patients own anatomy, and optimise their function.

Elderly patient - Predominantly female, fall from standing height (fragility fracture). Often patients
have multiple comorbidities that will ultimately dictate their prognosis. Aim of orthopaedic treatment
is to immediately regain patient mobility so that morbidity (infection, thromboembolic events,
pressure sores etc) and mortality associated with prolonged bed rest is avoided. Left untreated, a
neck of femur fracture can be considered a terminal event. Historically, mortality associated with
elderly hip fracture is 10% at one month, and 30% at one year. However, this has been improved in
the UK with the introduction of multidisciplinary, orthogeriatric lead care and the National Hip
Fracture Database and Best Practice Tariff.

Pertinent anatomy
Osteology - normal neck-shaft angle is 130 +/- 7 degrees, and 10 +/- 7 degrees of neck anteversion.
Vascular supply - The predominant blood supply to the femoral head and neck is from the medial
and lateral femoral circumflex arteries (branches of profunda femoris). These anastomose and
pierce the joint capsule at the base of the neck, mainly posteriorly. There is a small vascular
contribution from the artery of the ligament teres. Understanding the blood supply is fundamental to
the decision making process in treating NOF fractures.

Presentation and initial management


Typically, patients present with pain in the hip/groin, a shortened, abducted, externally rotated leg
(due to the unopposed pull of the muscles that act across the hip joint) and the inability to straight-
leg-raise. With undisplaced fractures, signs are more subtle.
High energy injuries should be treated in line with ATLS principles. All patients should be fluid
resuscitated, have adequate pain relief (often with a fascio-iliiaca nerve block), and be optimised for
surgery. In addition, elderly patients should be assessed by an orthogeriatrician.

Imaging
Anteroposterior and cross-table lateral plain radiographs are sufficient to diagnose the majority of
NOF fractures. If the fracture extends below the level of the lesser trochanter, or there is any
possibility of pathological fracture, full length femur views are essential to plan surgery.

Where there is a high index of suspicion of fracture, but plain radiographs are inconclusive, gold
standard investigation is MRI. However, if unavailable within 24 hours, or if the patient will not
tolerate MRI, CT is appropriate. The majority of fractures can be seen with modern CT techniques,
and so this is becoming first line in many hospitals.

Classification
There has been a move away from named classification systems towards descriptive classification
systems.
Two main types of NOF exist: Intra-capsular, and extra-capsular. Extra-capsular fractures are further
divided into pertrochanteric or subtrochanteric (within 5cm distal to the lesser trochanter). All
fractures are then described as undisplaced, minimally displaced, or displaced.
Femoral neck and head blood supply disruption is common with intracapsular NOF fractures, and
rare with extracapsular fractures. This fundamental principle underpins the practise of arthroplasty
for intracapsular fractures, and fixation for extracapsular fractures.

If you wish to use a named classification system, the most commonly used are below:
Elderly intracapsular - Garden Classification
Young intrasapsular - Pauvels Classification
Intertrochanteric - Evans
Subtrochanteric - Russell Taylor

Treatment
In general, NOF fractures are treated operatively except if the patient is deemed unlikely to survive
an anaesthetic. Best Practice Tarif (BPT) dictates that surgery should happen within 36 hours, as
delay of greater than 48 hours is associated with increased morbidity and mortality. Below are
suggested algorithms for the treatment of NOF. There are some areas of debate/controversy which
are detailed below.

Image sourced from Wikipedia

* The priority with the young patient is to retain the femoral head if possible, even with a displaced
intracapsular fracture. The risk of avascular necrosis and non-union (and therefore revision surgery)
associated with internal fixation needs weighing up against the sequelae of total hip replacement in
the young (wear, dislocation, revision). Discussion is necessary with the patient, on a case by case
basis.

** Undisplaced fractures in the elderly can be treated with internal fixation, often with cannulated
screws. This is appropriate for valgus impacted subcapital fractures which are inherently stable, to
prevent secondary displacement. This does still carry the risk of AVN or non-union, and therefore a
future revision. For this reason, many surgeons advocate arthroplasty as a single surgery.

*** NICE guidance - patients who fulfil these criteria should be offered total hip replacement which
conveys better function and prosthetic survivorship, compared with hemiarthroplasty, but at an
increased risk of dislocation.
Image sourced from Wikipedia

* Intertrochanteric fractures vary greatly in their stability. If the trochanter (and therefore lateral wall),
and medial calcar is in tact, then the fracture configuration bears stability. This can be treated with a
DHS, as collapse of the fracture is predictable. Where either or both structures are involved in the
fracture, stability becomes compromised and many surgeons will favour using an intramedullary
device. This is an ongoing debate, and difficult to test in an exam setting.

Post operative management


Patients should be mobilised fully weight bearing where possible. Care is multidisciplinary in its
delivery. Elderly patients should have orthogeriatrician assessment of comorbidity, and bone health
with secondary prevention measures if appropriate. There should be early involvement of
physiotherapy and occupational therapy services. For further guidance see sources listed below.

NICE clinical guidance on hip fracture: https://www.nice.org.uk/guidance/cg124


Best Practice Tarif:
www.nhfd.co.uk/20/hipfractureR.../Best%20Practice%20Tariff%20User%20Guide.pdf
National Hip Fracture Database: www.nhfd.co.uk/

Next question
Of the list below, which is not a cause of avascular necrosis?

Steroids

Sickle cell disease

Radiotherapy

Myeloma

Caisson disease

Causes of avascular necrosis


P ancreatitis
L upus
A lcohol
S teroids
T rauma
I diopathic, infection
C aisson disease, collagen vascular disease
R adiation, rheumatoid arthritis
A myloid
G aucher disease
S ickle cell disease

Steroid containing therapy for myeloma may induce avascular necrosis, however the disease itself
does not cause it. Caisson disease as may occur in deep sea divers is a recognised cause.
Please rate this question:

Discuss and give feedback


Next question

Avascular necrosis

 Cellular death of bone components due to interruption of the blood supply, causing bone
destruction
 Main joints affected are hip, scaphoid, lunate and the talus.
 It is not the same as non union. The fracture has usually united.
 Radiological evidence is slow to appear.
 Vascular ingrowth into the affected bone may occur. However, many joints will develop
secondary osteoarthritis.
Causes
P ancreatitis
L upus
A lcohol
S teroids
T rauma
I diopathic, infection
C aisson disease, collagen vascular disease
R adiation, rheumatoid arthritis
A myloid
G aucher disease
S ickle cell disease

Presentation
Usually pain. Often despite apparent fracture union.

Investigation
MRI scanning will show changes earlier than plain films.

Treatment
In fractures at high risk sites anticipation is key. Early prompt and accurate reduction is essential.

Non weight bearing may help to facilitate vascular regeneration.

Joint replacement may be necessary, or even the preferred option (e.g. Hip in the elderly).
Next question
Which of the following is the first radiological change likely to be apparent in a plain radiograph of a
12 year old presenting with suspected Perthes disease

Multiple bone cysts

Sclerosis of the femoral head

Loss of bone density

Joint space narrowing

Collapse of the femoral head

In Catterall stage I disease there may be no radiological abnormality at all. In Stage II disease there
may be sclerosis of the femoral head.

Indication for treatment (aide memoire):Half a dozen, half a head


Those aged greater than 6 years with >50% involvement of the femoral head should almost always
be treated.
Please rate this question:

Discuss and give feedback


Next question

Perthes disease

Perthes disease

 Idiopathic avascular necrosis of the femoral epiphysis of the femoral head


 Impaired blood supply to femoral head, causing bone infarction. New vessels develop and
ossification occurs. The bone either heals or a subchondral fracture occurs.

Clinical features

 Males 4x's greater than females


 Age between 2-12 years (the younger the age of onset, the better the prognosis)
 Limp
 Hip pain
 Bilateral in 20%
Diagnosis
Plain x-ray, Technetium bone scan or magnetic resonance imaging if normal x-ray and symptoms
persist.

Catterall staging
Stage Features

Stage 1 Clinical and histological features only

Stage 2 Sclerosis with or without cystic changes and preservation of the articular surface

Stage 3 Loss of structural integrity of the femoral head

Stage 4 Loss of acetabular integrity

Management

 To keep the femoral head within the acetabulum: cast, braces


 If less than 6 years: observation
 Older: surgical management with moderate results
 Operate on severe deformities

Prognosis
Most cases will resolve with conservative management. Early diagnosis improves outcomes.
Next question
A footballer is injured in a match and is being assessed in the outpatient department. On
examination he has a positive valgus stress test and minimal joint effusion. What is the most likely
underlying injury?

Injury to the lateral collateral ligament

Injury to the medial collateral ligament

Injury to the anterior cruciate ligament

Injury to the posterior cruciate ligament

Injury to the patellar tendon

Theme from January 2013 Exam


A knee injury in the footballer with a positive valgus stress test is usually associated with MCL injury.

Please rate this question:

Discuss and give feedback

Next question

Knee collateral ligament

Anatomy
The tibial collateral ligament is a broad, flat band. Its upper end has an extensive attachment to the
medial epicondyle of the femur with some fibres projecting onto the adductor magnus tendon. The
ligament passes downwards and forwards to the medial side of the tibia. The deepest fibres are
fused with the medial meniscus.
The fibular collateral ligament is round and cord like and stands clear of the thin, lateral part of the
fibrous capsule. It is enclosed within the fascia lata. It passes from the lateral epicondyle of the
femur to the head of the fibula in front of its highest point and splits the tendon of biceps femoris. On
the lateral side of the joint the fibres are short and weak and bridge the interval between the femoral
and tibial condyles. The popliteus tendon intervenes between the lateral meniscus and the capsule.
The tibial and fibular collateral ligaments prevent disruption of the joint at the sides. They are most
tightly stretched in extension, and then their direction- the fibular ligament downwards and
backwards, the tibial downwards and forwards- prevents rotation of the tibia laterally or the femur
medially. Rotation may be demonstrated in the flexed knee.

Injury
The collateral ligaments are commonly injured, the medial is most often affected. It requires a
significant force such as sporting tackle or motor vehicle to strike the side of the leg. Associated
injuries to both the tibial plateau or menisci are not uncommon.

Grading and treatment

Grade of
injury Features Treatment

1 Minor tearing of ligament fibres Conservative (analgesia and


Negative instability tests physiotherapy)

2 Ligament laxity (seen with knee in Usually splinting or casting for 4-6 weeks
30oflexion)
Knee stable when joint extended

3 Ligament completely torn Surgical ligament reconstruction


Joint instability

Next question
Theme: Upper limb injuries

A. Pulled elbow
B. Fracture of the coronoid process
C. Scaphoid fracture
D. Fracture of the distal humerus
E. Bennets fracture
F. Fracture of the shaft of the radius and ulnar
G. Galeazzi fracture
H. Fracture of the olecranon
I. Fracture of the radial head

Please select the most likely injury for the scenario given. Each option may be used once, more than
once or not at all.

27. A 32 year old man presents with a painful swelling over the volar aspect of his hand after
receiving a hard blow to his palm. On examination, he experiences pain on moving the
wrist and on longitudinal compression of the thumb.

You answered Pulled elbow

The correct answer is Scaphoid fracture

Scaphoid fractures usually occur as a result of direct hard blow to the palm or following a
fall on the out-stretched hand. The main physical signs are swelling and tenderness in the
anatomical snuff box, and pain on wrist movements and on longitudinal compression of
the thumb

28. A 26 year old man presents to the emergency department with a swelling over his left
elbow after a fall on an outstretched hand. On examination, he has tenderness over the
proximal part of his forearm, and has severely restricted supination and pronation
movements.

You answered Pulled elbow

The correct answer is Fracture of the radial head

Fracture of the radial head is common in young adults. It is usually caused by a fall on the
outstretched hand. On examination, there is marked local tenderness over the head of the
radius, impaired movements at the elbow, and a sharp pain at the lateral side of the elbow
at the extremes of rotation (pronation and supination).

29. A 56 year old lady presents with a painful swelling over the lower end of the forearm
following a fall. Imaging reveals a distal radial fracture with disruption of the distal radio-
ulnar joint.

You answered Pulled elbow

The correct answer is Galeazzi fracture

Galeazzi fractures occur after a fall on the hand with a rotational force superimposed on it.
On examination, there is bruising, swelling and tenderness over the lower end of the
forearm. X- Rays reveal a displaced fracture of the radius and a prominent ulnar head due
to dislocation of the inferior radio-ulnar joint.

Please rate this question:

Discuss and give feedback


Next question

Upper limb fractures

Colles' fracture

 Fall onto extended outstretched hands


 Described as a dinner fork type deformity
 Classical Colles' fractures have the following 3 features:

Features of the injury


1. Transverse fracture of the radius
2. 1 inch proximal to the radio-carpal joint
3. Dorsal displacement and angulation

Smith's fracture (reverse Colles' fracture)

 Volar angulation of distal radius fragment (Garden spade deformity)


 Caused by falling backwards onto the palm of an outstretched hand or falling with wrists
flexed

Bennett's fracture

 Intra-articular fracture of the first carpometacarpal joint


 Impact on flexed metacarpal, caused by fist fights
 X-ray: triangular fragment at ulnar base of metacarpal
Monteggia's fracture

 Dislocation of the proximal radioulnar joint in association with an ulna fracture


 Fall on outstretched hand with forced pronation
 Needs prompt diagnosis to avoid disability

Galeazzi fracture

 Radial shaft fracture with associated dislocation of the distal radioulnar joint
 Occur after a fall on the hand with a rotational force superimposed on it.
 On examination, there is bruising, swelling and tenderness over the lower end of the
forearm.
 X Rays reveal the displaced fracture of the radius and a prominent ulnar head due to
dislocation of the inferior radio-ulnar joint.

Barton's fracture

 Distal radius fracture (Colles'/Smith's) with associated radiocarpal dislocation


 Fall onto extended and pronated wrist

Scaphoid fractures

 Scaphoid fractures are the commonest carpal fractures.


 Surface of scaphoid is covered by articular cartilage with small area available for blood
vessels (fracture risks blood supply)
 Forms floor of anatomical snuffbox
 Risk of fracture associated with fall onto outstretched hand (tubercle, waist, or proximal 1/3)
 The main physical signs are swelling and tenderness in the anatomical snuff box, and pain
on wrist movements and on longitudinal compression of the thumb.
 Ulnar deviation AP needed for visualization of scaphoid
 Immobilization of scaphoid fractures difficult

Radial head fracture

 Fracture of the radial head is common in young adults.


 It is usually caused by a fall on the outstretched hand.
 On examination, there is marked local tenderness over the head of the radius, impaired
movements at the elbow, and a sharp pain at the lateral side of the elbow at the extremes of
rotation (pronation and supination).

Next question
Theme: Hand injuries

A. Admission and surgical debridement


B. Application of futura splint and fracture clinic review
C. Application of tubigrip bandage and fracture clinic review
D. Admission for open reduction and fixation
E. Discharge with reassurance
F. Commence oral prednisolone
G. Commence oral diclofenac

Which of the following options is the best management plan? Each option may be used once, more
than once or not at all.

30. A 42 year old skier falls and impacts his hand on his ski pole. On examination he is tender
in the anatomical snuffbox and on bimanual palpation. X-rays with scaphoid views show
no evidence of fracture.

You answered Admission and surgical debridement

The correct answer is Application of futura splint and fracture clinic review

A fracture may still be present and should be immobilised until repeat imaging can be
performed. If clinical suspicion persists then subsequent imaging should be with MRI
scanning or CT if MRI is contra-indicated.

31. A 43 year old man falls over landing on his left hand. Although there was anatomical
snuffbox tenderness no x-rays either at the time or subsequently have shown evidence of
scaphoid fracture. He has been immobilised in a futura splint for two weeks and is now
asymptomatic.

You answered Admission and surgical debridement

The correct answer is Discharge with reassurance

This patient is at extremely low risk of having sustained a scaphoid injury and may be
discharged.

32. A builder falls from scaffolding and lands on his left hand he suffers a severe laceration to
his palm. An x-ray shows evidence of scaphoid fracture that is minimally displaced.

Admission and surgical debridement

This is technically an open fracture and should be debrided prior to attempted fixation
(which should occur soon after).
Scaphoid fractures:
80% of all carpal fractures
80% occur in men
80% occur at the waist of the scaphoid

Please rate this question:

Discuss and give feedback


Next question

Scaphoid fractures

 Scaphoid fractures are the commonest carpal fractures.


 Surface of scaphoid is covered by articular cartilage with small area available for blood
vessels (fracture risks blood supply)
 Forms floor of anatomical snuffbox
 Risk of fracture associated with fall onto outstretched hand (tubercle, waist, or proximal third)
 Ulnar deviation AP needed for visualization of scaphoid
 Immobilization of scaphoid fractures difficult

Management
Non-displaced fractures - Casts or splints
- Percutaneous scaphoid fixation

Displaced fracture Surgical fixation, usually with a screw

Complications

 Non union of scaphoid


 Avascular necrosis of the scaphoid
 Scapholunate disruption and wrist collapse
 Degenerative changes of the adjacent joint

Next question
Theme: Paediatric orthopaedics

A. Musculoskeletal pain

B. Congenital dysplasia of the hip

C. Slipped upper femoral epiphysis

D. Transient synovitis

E. Septic arthritis

F. Perthes disease

G. Tibial fracture

Please select the most likely diagnosis for the scenario given. Each option may be used once, more
than once or not at all.

33. A 4 year boy presents with an abnormal gait. He has a history of recent viral illness. His WCC is 11
and ESR is 30.

You answered Musculoskeletal pain

The correct answer is Transient synovitis

Viral illnesses can be associated with transient synovitis. The WCC should ideally be > 12 and the
ESR > 40 to suggest septic arthritis.

34. A 6 year old boy presents with groin pain. He is known to be disruptive in class. He reports that he
is bullied for being short. On examination he has an antalgic gait and pain on internal rotation of
the right hip.

You answered Musculoskeletal pain

The correct answer is Perthes disease

This child is short, has hyperactivity (disruptive behaviour) and is within the age range for Perthes
disease. Hyperactivity and short stature are associated with Perthes disease.

35. An obese 12 year old boy is referred with pain in the left knee and hip. On examination he has an
antaglic gait and limitation of internal rotation. His knee has normal range of passive and active
movement.

You answered Musculoskeletal pain

The correct answer is Slipped upper femoral epiphysis

Similar theme to September 2012 Exam


Slipped upper femoral epiphysis is commonest in obese adolescent males. The x-ray will show
displacement of the femoral epiphysis inferolaterally. Treatment is usually with rest and non
weight bearing crutches.

Beware of attributing gait disorders to benign processes in young children without careful clinical and
radiological assessment.

Please rate this question:

Discuss and give feedback

Next question

Paediatric orthopaedics

Diagnosis Mode of presentation Treatment Radiology

Developmental Usually diagnosed in infancy Splints and harnesses or Initially no obvious


dysplasia of the by screening tests. May be traction. In later years change on plain films
hip bilateral, when disease is osteotomy and hip and USS gives best
unilateral there may be leg realignment procedures may resolution until 3
length inequality. As be needed. In arthritis a joint months of age. On
disease progresses child replacement may be needed. plain films Shentons
may limp and then early However, this is best line should form a
onset arthritis. More deferred if possible as it will smooth arc
common in extended almost certainly require
breech babies. revision

Perthes Disease Hip pain (may be referred Remove pressure from joint X-rays will show
to the knee) usually to allow normal flattened femoral
occurring between 5 and 12 development. Physiotherapy. head. Eventually in
years of age. Bilateral Usually self-limiting if untreated cases the
disease in 20%. diagnosed and treated femoral head will
promptly. fragment.

Slipped upper Typically seen in obese Bed rest and non-weight X-rays will show the
femoral male adolescents. Pain is bearing. Aim to avoid femoral head displaced
epiphysis often referred to the knee. avascular necrosis. If severe and falling
Limitation to internal slippage or risk of it occurring inferolaterally (like a
rotation is usually seen. then percutaneous pinning of melting ice cream
Knee pain is usually present the hip may be required. cone) The Southwick
2 months prior to hip angle gives indication
slipping. Bilateral in 20%. of disease severity

Next question
Theme: Eponymous fractures

A. Smith's
B. Bennett's
C. Monteggia's
D. Colles'
E. Galeazzi
F. Pott's
G. Barton's

Link the most appropriate eponymously named fracture to the scenario described. Each scenario
may be used once, more than once or not at all.

36. A 28 year old man falls on the back of his hand. On x-ray he has a fractured distal radius
demonstrating volar displacement of the fracture.

Smith's

This is a Smith fracture (reverse Colles' fracture); unlike a Colles' this is a high velocity
injury and may require surgical correction. Note that Colles' fractures are usually dorsally
displaced.

37. A 38 year old window cleaner falls from his ladder. He lands on his left arm and notices an
obvious injury. An x-ray and clinical examination demonstrate that he has a fracture of the
proximal ulna and associated radial dislocation.

You answered Smith's

The correct answer is Monteggia's

This constellation of injuries is referred to as a Monteggia's fracture.

38. A 32 year old man falls from scaffolding and sustains an injury to his forearm. Clinical
examination and x-ray shows that he has sustained a radial fracture with dislocation of the
inferior radio-ulna joint.

You answered Smith's

The correct answer is Galeazzi

Isolated fracture of the radius alone can occur but is rare. Always check for associated
injury.
Please rate this question:

Discuss and give feedback


Next question

Eponymous fractures

Colles' fracture (dinner fork deformity)

 Fall onto extended outstretched hand


 Classical Colles' fractures have the following 3 features:

1. Transverse fracture of the radius


2. 1 inch proximal to the radio-carpal joint
3. Dorsal displacement and angulation

Smith's fracture (reverse Colles' fracture)

 Volar angulation of distal radius fragment (Garden spade deformity)


 Caused by falling backwards onto the palm of an outstretched hand or falling with wrists
flexed

Bennett's fracture

 Intra-articular fracture of the first carpometacarpal joint


 Impact on flexed metacarpal, caused by fist fights
 X-ray: triangular fragment at ulnar base of metacarpal
Image sourced from Wikipedia

Monteggia's fracture

 Dislocation of the proximal radioulnar joint in association with an ulna fracture


 Fall on outstretched hand with forced pronation
 Needs prompt diagnosis to avoid disability
Image sourced from Wikipedia

Galeazzi fracture

 Radial shaft fracture with associated dislocation of the distal radioulnar joint
 Direct blow

Pott's fracture

 Bimalleolar ankle fracture


 Forced foot eversion

Barton's fracture

 Distal radius fracture (Colles'/Smith's) with associated radiocarpal dislocation


 Fall onto extended and pronated wrist
 Involvement of the joint is a defining feature

Next question
A 54-year-old man presents to the Emergency Department with a 2 day history of a swollen, painful
left knee. You aspirate the joint to avoid admission to the orthopaedic wards. Aspirated joint fluid
shows calcium pyrophosphate crystals. Which of the following blood tests is most useful in revealing
an underlying cause?

Transferrin saturation

ACTH

ANA

Serum ferritin

LDH

This is a typical presentation of pseudogout. An elevated transferrin saturation may indicate


haemochromatosis, a recognised cause of pseudogout.

A high ferritin level is also seen in haemochromatosis but can be raised in a variety of infective and
inflammatory processes, including pseudogout, as part of an acute phase response.
Please rate this question:

Discuss and give feedback


Next question

Pseudogout

Pseudogout is a form of microcrystal synovitis caused by the deposition of calcium pyrophosphate


dihydrate in the synovium

Risk factors

 hyperparathyroidism
 hypothyroidism
 haemochromatosis
 acromegaly
 low magnesium, low phosphate
 Wilson's disease
Features

 knee, wrist and shoulders most commonly affected


 joint aspiration: weakly-positively birefringent rhomboid shaped crystals
 x-ray: chondrocalcinosis

Management

 aspiration of joint fluid, to exclude septic arthritis


 NSAIDs or intra-articular, intra-muscular or oral steroids as for gout

Next question
A 19 year old soldier has just returned from a prolonged marching exercise and presents with a
sudden onset, severe pain, in the forefoot. Clinical examination reveals tenderness along the second
metatarsal. Plain x-rays are taken of the area, these demonstrate callus surrounding the shaft of the
second metatarsal. What is the most likely diagnosis?

Stress fracture

Mortons neuroma

Osteochondroma

Acute osteomyelitis

Freiberg's disease

Theme from 2011 Exam


A short history of pain together with clinical examination and radiological signs affecting the second
metatarsal favour a stress fracture. The fact that callus is present suggests that immobilisation is
unlikely to be beneficial. Freibergs disease is an anterior metatarsalgia affecting the head of the
second metarsal, it typically occurs in the pubertal growth spurt. The initial injury was thought to be
due to stress microfractures at the growth plate. The key feature in the history which distinguishes
the injury as being stress fracture is the radiology. In Freibergs disease the x-ray changes include;
joint space widening, formation of bony spurs, sclerosis and flattening of the metatarsal head.

Please rate this question:

Discuss and give feedback

Next question

Stress fractures

Repetitive activity and loading of normal bone may result in small hairline fractures. Whilst these
may be painful they are seldom displaced. Surrounding soft tissue injury is unusual. They may
present late following the injury, in which case callus formation may be identified on radiographs.
Such cases may not require formal immobilisation, injuries associated with severe pain and
presenting at an earlier stage may benefit from immobilisation tailored to the site of injury.

Next question
A 65-year-old Asian female presents with an extracapsular neck of femur fracture. Investigations
show:

Calcium 2.07 mmol/l (2.20-2.60 mmol/l)

Phosphate 0.66 mmol/l (0.8-1.40 mmol/l)

ALP 256 IU/l (44-147 IU/l)

What is the most likely diagnosis?

Bone tuberculosis

Hypoparathyroidism

Myeloma

Osteomalacia

Paget's disease

Osteomalacia

 low: calcium, phosphate


 raised: alkaline phosphatase

The low calcium and phosphate combined with the raised alkaline phosphatase point towards
osteomalacia.
Please rate this question:

Discuss and give feedback


Next question

Osteomalacia

Basics

 normal bony tissue but decreased mineral content


 rickets if when growing
 osteomalacia if after epiphysis fusion

Types

 vitamin D deficiency e.g. malabsorption, lack of sunlight, diet


 renal failure
 drug induced e.g. anticonvulsants
 vitamin D resistant; inherited
 liver disease, e.g. cirrhosis

Features

 rickets: knock-knee, bow leg, features of hypocalcaemia


 osteomalacia: bone pain, fractures, muscle tenderness, proximal myopathy

Investigation

 low calcium, phosphate, 25(OH) vitamin D


 raised alkaline phosphatase
 x-ray: children - cupped, ragged metaphyseal surfaces; adults - translucent bands (Looser's
zones or pseudofractures)

Treatment

 calcium with vitamin D tablets

Next question
A 78-year-old woman is discharged following a fractured neck of femur. On review she is making
good progress but consideration is given to secondary prevention of further fractures. Unfortunately
the orthogeriatricians are all on annual leave and the consultant has asked you to arrange suitable
management. Which is the best option?

Alendronate

Alendronate, calcium and vitamin D supplementation

Strontium

Arrange a DEXA scan

Hormone replacement therapy

A bisphosphonate, calcium and vitamin D supplementation should be given to all patients aged over
75 years after having a fracture. A DEXA scan is only needed of the patient is aged below 75 years.
Hormone replacement therpay has been shown to reduce vertebral and non vertebral fractures,
however the risks of cardiovascular disease and breast malignancy make this a less favourable
option.
Please rate this question:

Discuss and give feedback


Next question

Osteoporosis: secondary prevention

NICE guidelines were updated in 2008 on the secondary prevention of osteoporotic fractures in
postmenopausal women.

Key points include

 Treatment is indicated following osteoporotic fragility fractures in postmenopausal women


who are confirmed to have osteoporosis (a T-score of - 2.5 SD or below).
 In women aged 75 years or older, a DEXA scan may not be required 'if the responsible
clinician considers it to be clinically inappropriate or unfeasible'
 Vitamin D and calcium supplementation should be offered to all women unless the clinician is
confident they have adequate calcium intake and are vitamin D replete
 Alendronate is first-line
 Around 25% of patients cannot tolerate alendronate, usually due to upper gastrointestinal
problems. These patients should be offered risedronate or etidronate (see treatment criteria
below)
 Strontium ranelate and raloxifene are recommended if patients cannot tolerate
bisphosphonates (see treatment criteria below)

Supplementary notes on treatment

Bisphosphonates

 Alendronate, risedronate and etidronate are all licensed for the prevention and treatment of
post-menopausal and glucocorticoid-induced osteoporosis
 All three have been shown to reduce the risk of both vertebral and non-vertebral fractures
although alendronate, risedronate may be superior to etidronate in preventing hip fractures
 Ibandronate is a once-monthly oral bisphosphonate

Vitamin D and calcium

 Poor evidence base to suggest reduced fracture rates in the general population at risk of
osteoporotic fractures - may reduce rates in frail, housebound patients

Raloxifene - selective oestrogen receptor modulator (SERM)

 Has been shown to prevent bone loss and to reduce the risk of vertebral fractures, but has
not yet been shown to reduce the risk of non-vertebral fractures
 Has been shown to increase bone density in the spine and proximal femur
 May worsen menopausal symptoms
 Increased risk of thromboembolic events
 May decrease risk of breast cancer

Strontium ranelate

 'Dual action bone agent' - increases deposition of new bone by osteoblasts and reduces the
resorption of bone by osteoclasts
 Strong evidence base, may be second-line treatment in near future
 Increased risk of thromboembolic events

Next question
Which of the following statements relating to avascular necrosis is false?

When associated with fracture may occur despite the radiological evidence of fracture
union.

Pain and stiffness will typically precede radiological evidence of the condition.

Drilling of affected bony fragments may be used to facilitate angiogenesis where


arthroplasty is not warranted.

The earliest detectable radiological evidence is a radiolucency of the affected area


coupled with subchondral collapse.

It is less likely when prompt anatomical alignment of fracture fragments is achieved.

Avascular necrosis- radiological changes occur late.

Radiolucency and subchondral collapse are late changes. The earliest evidence on plain films is the
affected area appearing as being more radio-opaque due to hyperaemia and resorption of the
neighboring area. It may be diagnosed earlier using bone scans and MRI.
Please rate this question:

Discuss and give feedback


Next question

Avascular necrosis

 Cellular death of bone components due to interruption of the blood supply, causing bone
destruction
 Main joints affected are hip, scaphoid, lunate and the talus.
 It is not the same as non union. The fracture has usually united.
 Radiological evidence is slow to appear.
 Vascular ingrowth into the affected bone may occur. However, many joints will develop
secondary osteoarthritis.

Causes
P ancreatitis
L upus
A lcohol
S teroids
T rauma
I diopathic, infection
C aisson disease, collagen vascular disease
R adiation, rheumatoid arthritis
A myloid
G aucher disease
S ickle cell disease

Presentation
Usually pain. Often despite apparent fracture union.

Investigation
MRI scanning will show changes earlier than plain films.

Treatment
In fractures at high risk sites anticipation is key. Early prompt and accurate reduction is essential.

Non weight bearing may help to facilitate vascular regeneration.

Joint replacement may be necessary, or even the preferred option (e.g. Hip in the elderly).
Next question
Theme: Diseases affecting the spine

A. Spondylolysis
B. Spina bifida occulta
C. Spondylolisthesis
D. Meningomyelocele
E. Meningocele
F. Scoliosis - non structural
G. Scoliosis
H. Ankylosing spondylitis
I. Scheuermann's disease

Please select the most likely underlying diagnosis for the condition described. Each condition may
be used once, more than once or not at all.

44. A 19 year old female is involved in an athletics event. She has just completed the high
jump when she suddenly develops severe back pain and weakness affecting both her legs.
on examination she has a prominent sacrum and her lower back is painful.

You answered Spondylolysis

The correct answer is Spondylolisthesis

Theme from September 2012 Exam


Young athletic females are the group most frequently affected by spondylolythesis who
have a background of spondylolysis. Whilst the latter condition is a risk factor for
spondylolythesis the former condition is most likely in a young athletic female who
presents with sudden pain.

45. A 15 year old boy is brought to the clinic by his mother who is concerned that he has a
mark overlying his lower spine. On examination the boy has a patch of hair overlying his
lower lumbar spine and a birth mark at the same location. Lower limb neurological
examination is normal.

You answered Spondylolysis

The correct answer is Spina bifida occulta

Spina bifida occulta is a common condition and may affect up to 10% of the population.
The more severe types of spina bifida have more characteristic skin changes. Occasionally
the unwary surgeon is persuaded to operate on these "cutaneous" changes and we would
advocate performing an MRI scan prior to any such surgical procedure in this region.

46. A 19 year old female presents to the clinic with progressive pain in her neck and back. The
condition has been progressively worsening over the past 6 months. She has not presented
previously because she was an inpatient with a disease flare of ulcerative colitis. On
examination she has a stiff back with limited spinal extension on bending forwards.

You answered Spondylolysis

The correct answer is Ankylosing spondylitis

Ankylosing spondylitis is associated with HLA B27, there is a strong association with
ulcerative colitis in such individuals. The clinical findings are usually of a kyphosis
affecting the cervical and thoracic spine. Considerable symptomatic benefit may be
obtained using non steroidal anti inflammatory drugs. These should be used carefully in
patients with inflammatory bowel disease who may be taking steroids.

Please rate this question:

Discuss and give feedback


Next question

Diseases affecting the vertebral column

Ankylosing  Chronic inflammatory disorder affecting the axial skeleton


spondylitis  Sacro-ilitis is a usually visible in plain films
 Up to 20% of those who are HLA B27 positive will develop the
condition
 Affected articulations develop bony or fibrous changes
 Typical spinal features include loss of the lumbar lordosis and
progressive kyphosis of the cervico-thoracic spine

Scheuermann's  Epiphysitis of the vertebral joints is the main pathological process


disease  Predominantly affects adolescents
 Symptoms include back pain and stiffness
 X-ray changes include epiphyseal plate disturbance and anterior
wedging
 Clinical features include progressive kyphosis (at least 3 vertebrae
must be involved)
 Minor cases may be managed with physiotherapy and analgesia,
more severe cases may require bracing or surgical stabilisation

Scoliosis  Consists of curvature of the spine in the coronal plane


 Divisible into structural and non structural, the latter being
commonest in adolescent females who develop minor postural
changes only. Postural scoliosis will typically disappear on
manoeuvres such as bending forwards
 Structural scoliosis affects > 1 vertebral body and is divisible into
idiopathic, congential and neuromuscular in origin. It is not
correctable by alterations in posture
 Within structural scoliosis, idiopathic is the most common type
 Severe, or progressive structural disease is often managed surgically
with bilateral rod stabilisation of the spine

Spina bifida  Non fusion of the vertebral arches during embryonic development
 Three categories; myelomeningocele, spina bifida occulta and
meningocele
 Myelomeningocele is the most severe type with associated
neurological defects that may persist in spite of anatomical closure of
the defect
 Up to 10% of the population may have spina bifida occulta, in this
condition the skin and tissues (but not not bones) may develop over
the distal cord. The site may be identifiable by a birth mark or hair
patch
 The incidence of the condition is reduced by use of folic acid
supplements during pregnancy

Spondylolysis  Congenital or acquired deficiency of the pars interarticularis of the


neural arch of a particular vertebral body, usually affects L4/ L5
 May be asymptomatic and affects up to 5% of the population
 Spondylolysis is the commonest cause of spondylolisthesis in
children
 Asymptomatic cases do not require treatment

Spondylolisthesis  This occurs when one vertebra is displaced relative to its immediate
inferior vertebral body
 May occur as a result of stress fracture or spondylolysis
 Traumatic cases may show the classic "Scotty Dog" appearance on
plain films
 Treatment depends upon the extent of deformity and associated
neurological symptoms, minor cases may be actively monitored.
Individuals with radicular symptoms or signs will usually require
spinal decompression and stabilisation

Next question
Theme: Management of fractures

A. Discharge home with arm sling and fracture clinic appointment


B. Discharge home with futura splint and fracture clinic appointment
C. Admit for open reduction and fixation
D. Fasciotomy
E. Active observation for progression of neurovascular compromise
F. Reduction of fracture in casualty and application of plaster backslab,
followed by discharge home.

Please select the most appropriate immediate management for the fracture scenarios given. Each
option may be used once, more than once or not at all.

47. A 22 year old rugby player falls onto an outstretched hand and sustains a fracture of the
distal radius. The x-ray shows a dorsally angulated comminuted fracture.

You answered Discharge home with arm sling and fracture clinic appointment

The correct answer is Admit for open reduction and fixation

Unlike an osteoporotic fracture in an elderly lady this is a high velocity injury and will
require surgical fixation.

48. A 10 year old boy undergoes a delayed open reduction and fixation of a significantly
displaced supracondylar fracture. On the ward he complains of significant forearm pain
and paraesthesia of the hand. Radial pulse is normal.

You answered Discharge home with arm sling and fracture clinic appointment

The correct answer is Fasciotomy

The delay is the significant factor here. These injuries often have neurovascular
compromise and inactivity now places him at risk of developing complications. In
compartment syndrome the loss of arterial pulsation occurs late.

49. A 28 year old man falls onto an outstretched hand. On examination there is tenderness of
the anatomical snuffbox. However, forearm and hand x-rays are normal.

You answered Discharge home with arm sling and fracture clinic appointment

The correct answer is Discharge home with futura splint and fracture clinic
appointment
This could well be a scaphoid fracture and should be temporarily immobilised pending
further review. A futura splint will immobilise better than an arm sling for this problem.

Please rate this question:

Discuss and give feedback


Next question

Fracture management

 Bony injury resulting in a fracture may arise from trauma (excessive forces applied to bone),
stress related (repetitive low velocity injury) or pathological (abnormal bone which fractures
during normal use of following minimal trauma)
 Diagnosis involves not just evaluating the fracture ; such as site and type of injury but also
other associated injuries and distal neurovascular deficits. This may entail not just clinical
examination but radiographs of proximal and distal joints.
 When assessing x-rays it is important to assess for changes in length of the bone, the
angulation of the distal bone, rotational effects, presence of material such as glass.

Fracture types
Fracture type Description

Oblique fracture Fracture lies obliquely to long axis of bone

Comminuted fracture >2 fragments

Segmental fracture More than one fracture along a bone

Transverse fracture Perpendicular to long axis of bone

Spiral fracture Severe oblique fracture with rotation along long axis of bone

Open Vs Closed
It is also important to distinguish open from closed injuries. The most common classification system
for open fractures is the Gustilo and Anderson classification system (given below):
Grade Injury

1 Low energy wound <1cm

2 Greater than 1cm wound with moderate soft tissue damage

3 High energy wound > 1cm with extensive soft tissue damage

3 A (sub group of 3) Adequate soft tissue coverage

3 B (sub group of 3) Inadequate soft tissue coverage

3 C (sub group of 3) Associated arterial injury

Key points in management of fractures

 Immobilise the fracture including the proximal and distal joints


 Carefully monitor and document neurovascular status, particularly following reduction and
immobilisation
 Manage infection including tetanus prophylaxis
 IV broad spectrum antibiotics for open injuries
 As a general principle all open fractures should be thoroughly debrided ( and internal fixation
devices avoided or used with extreme caution)
 Open fractures constitute an emergency and should be debrided and lavaged within 6 hours
of injury

Next question
A 4 year old boy falls and sustains a fracture to the growth plate of his right wrist. Which of the
following systems is used to classify the injury?

Salter - Harris system

Weber system

Gustilo - Anderson system

Garden system

None of the above

The mnemonic 'SALTR' can be used to help remember the first five types. This mnemonic requires
the reader to imagine the bones as long bones, with the epiphyses at the base.

 I "S" = Slip (separated or straight across). Fracture of the cartilage of the physis (growth
plate)
 II "A" = Above. The fracture lies above the physis, or Away from the joint.
 III "L" = Lower. The fracture is below the physis in the epiphysis.
 IV "TE" = Through Everything. The fracture is through the metaphysis, physis, and epiphysis.
 V "R" = Rammed (crushed). The physis has been crushed

The Salter - Harris system is most commonly used. The radiological signs in Type 1 and 5 injuries
may be identical. Which is unfortunate as type 5 injuries do not do well (and may be missed!). One
of our users has helpfully supplied a mnemonic for remembering the types (see above).
Please rate this question:

Discuss and give feedback


Next question

Paediatric fractures

Paediatric fracture types


Type Injury pattern

Complete fracture Both sides of cortex are breached


Toddlers fracture Oblique tibial fracture in infants

Plastic deformity Stress on bone resulting in deformity without cortical disruption

Greenstick fracture Unilateral cortical breach only

Buckle fracture Incomplete cortical disruption resulting in periosteal haematoma only

Growth plate fractures


In paediatric practice fractures may also involve the growth plate and these injuries are classified
according to the Salter- Harris system (given below):

Type Injury pattern

I Fracture through the physis only (x-ray often normal)

II Fracture through the physis and metaphysis

III Fracture through the physis and epiphyisis to include the joint

IV Fracture involving the physis, metaphysis and epiphysis

V Crush injury involving the physis (x-ray may resemble type I, and appear normal)

As a general rule it is safer to assume that growth plate tenderness is indicative of an underlying
fracture even if the x-ray appears normal. Injuries of Types III, IV and V will usually require surgery.
Type V injuries are often associated with disruption to growth.

Non accidental injury

 Delayed presentation
 Delay in attaining milestones
 Lack of concordance between proposed and actual mechanism of injury
 Multiple injuries
 Injuries at sites not commonly exposed to trauma
 Children on the at risk register

Pathological fractures
Genetic conditions, such as osteogenesis imperfecta, may cause pathological fractures.

Osteogenesis imperfecta

 Defective osteoid formation due to congenital inability to produce adequate intercellular


substances like osteoid, collagen and dentine.
 Failure of maturation of collagen in all the connective tissues.
 Radiology may show translucent bones, multiple fractures, particularly of the long bones,
wormian bones (irregular patches of ossification) and a trefoil pelvis.

Subtypes

 Type I The collagen is normal quality but insufficient quantity.


 Type II- Poor collagen quantity and quality.
 Type III- Collagen poorly formed. Normal quantity.
 Type IV- Sufficient collagen quantity but poor quality.

Osteopetrosis

 Bones become harder and more dense.


 Autosomal recessive condition.
 It is commonest in young adults.
 Radiology reveals a lack of differentiation between the cortex and the medulla described as
marble bone.

Next question
Theme: Bone disease

A. Osteogenesis imperfecta
B. Osteoporosis
C. Rickets
D. Pagets disease
E. Chondrosarcoma
F. Metastatic breast cancer

Please select the most likely diagnosis for the scenario given. Each option may be used once, more
than once or not at all.

51. A 66 year old lady presents with pain in her right hip. It has been increasing over the
previous three weeks and waking her from sleep. On examination she is tender on internal
rotation. Blood tests reveal a mildly elevated serum calcium and alkaline phosphatase
levels.

You answered Osteogenesis imperfecta

The correct answer is Metastatic breast cancer

Increasing pain at rest, together with increased serum calcium and alkaline phosphatase are
most likely to represent metastatic tumour to bone. Chondrosarcomas do occur in the
pelvis but are not associated with increased serum calcium and typically have a longer
history.

52. A 73 year old man presents with pain in the right leg. It is most uncomfortable on walking.
On examination he has a deformity of his right femur, which on x-ray is thickened and
sclerotic. His serum alkaline phosphatase is elevated, but calcium is within normal limits.

You answered Osteogenesis imperfecta

The correct answer is Pagets disease

This is a typical scenario for Pagets disease.

53. A 73 year old lady presents with pain in her left hip. She was walking around the house
when she tripped over a rug and fell over. Apart from temporal arteritis which is well
controlled with prednisolone she is otherwise well. On examination he leg is shorted and
externally rotated.Her serum alkaline phosphatase and calcium are normal.

You answered Osteogenesis imperfecta


The correct answer is Osteoporosis

The combination of age, female gender and steroids coupled with hip pain on minor
trauma are strongly suggestive of osteoporosis.

Please rate this question:

Discuss and give feedback


Next question

Bone disease

Disease Features Treatment

Pagets  Focal bone resorption followed by Bisphosphonates


excessive and chaotic bone deposition
 Affects (in order): spine, skull, pelvis
and femur
 Serum alkaline phosphatase raised
(other parameters normal)
 Abnormal thickened, sclerotic bone on
x-rays
 Risk of cardiac failure with >15%
bony involvement
 Small risk of sarcomatous change

Osteoporosis  Excessive bone resorption resulting in Bisphosphonates, calcium


demineralised bone and vitamin D
 Commoner in old age
 Increased risk of pathological fracture,
otherwise asymptomatic
 Alkaline phosphatase normal, calcium
normal

Secondary bone  Bone destruction and tumour Radiotherapy, prophylactic


tumours infiltration fixation and analgesia
 Mirel scoring used to predict risk of
fracture
 Appearances depend on primary
(e.g.sclerotic - prostate, lytic - breast)
 Elevated serum calcium and alkaline
Disease Features Treatment

phosphatase may be seen

Next question
Theme: Shoulder pain

A. Impingement syndrome
B. Rotator cuff tear
C. Adhesive capsulitis
D. Calcific tendonitis
E. Biceps tendon rupture
F. Parsonage - Turner syndrome
G. Labral tear

Please select the most likely cause for shoulder pain from the list. Each option may be used once,
more than once or not at all.

54. A 63 year old lady undergoes an axillary clearance for breast cancer. She makes steady
progress. However, 8 weeks post operatively she still suffers from severe shoulder pain.
On examination she has reduced active movements in all planes and loss of passive
external rotation.

You answered Impingement syndrome

The correct answer is Adhesive capsulitis

Theme from January 2016 Exam


Frozen shoulder passes through an initial painful stage followed by a period of joint
stiffness. With physiotherapy the problem will usually resolve although it may take up to 2
years to do so.

55. A 78 year old man complains of a long history of shoulder pain and more recently
weakness. On examination active attempts at abduction are impaired. Passive movements
are normal.

You answered Impingement syndrome

The correct answer is Rotator cuff tear

Rotator cuff tears are common in elderly people and may occur following minor trauma or
as a result of long standing impingement. Tears greater than 2cm should generally be
repaired surgically. The length of the history in this scenario is suggestive of a tear
complicating impingement.

56. A 28 year old man complains of pain and weakness in the shoulder. He has recently been
unwell with glandular fever from which he is fully recovered. On examination there is
some evidence of muscle wasting and a degree of winging of the scapula. Power during
active movements is impaired.
You answered Impingement syndrome

The correct answer is Parsonage - Turner syndrome

This is a peripheral neuropathy that may complicate viral illnesses and usually resolves
spontaneously.
Deep seated pain in the proximal forearm especially during the night and at rest may be due to
tumour, especially metastatic lesions.

Please rate this question:

Discuss and give feedback


Next question

Shoulder disorders

Shoulder fractures and dislocations


Fractures
Proximal humerus
Background

 Third most common fragility fracture in the elderly.


 Results from low energy fall in predominantly elderly females, or from high energy trauma in
young males.
 Can be associated with nerve injury (commonly axillary), and fracture-dislocation of the
humeral head. Detailed neurological assessment is essential for all upper limb injuries.

Anatomy
Osteology
Consists of articular head, greater tuberosity, lesser tuberosity, metaphysis and diaphysis. Between
the articular head and the tuberosities is the anatomical neck (previous physis). Between the
tuberosities and the metaphysis is the surgical neck.
The supraspinatus, infraspinatus and teres minor muscles attach to the greater tuberosity. The
subscapularis muscle attaches to the lesser tuberosity.

Vascular Supply
Humeral head is supplied by the anterior and posterior humeral circumflex arteries. Anatomical neck
fractures are at greatest risk of osteonecrosis.

Imaging
Imaging aims to both delineate the fracture pattern, and confirm/exlude the presence of an
associated dislocation.

 Radiographs - True anteroposterior (AP), axillary lateral and/or scapula Y view.


 CT - indicated to better define intra-articular involvement and to aid pre-operative planning.
MRI is not useful for fracture imaging.

Classification
Description of the fracture is often more useful than classification. Particular attention should be paid
to humeral alignment, fracture displacement, and greater tuberosity position (rotator cuff will pull the
GT supero-posterioly, which can cause impingement problems with malunion).
- Neer Classification: Most commonly used. Describes fracture as 2,3,or 4 part depending upon the
number main fragments. Also comments on the degree of displacement. Fragments:
-greater tuberosity
-lesser tuberosity
- articular surface
- shaft
Displacement: >1cm or angulation >45 degrees.

Treatment
The vast majority of proximal humeral fractures are minimally displaced, and therefore can be
managed conservatively. This involves immobilisation in a polysling, and progressive mobilisation.
Pendular exercise can commence at 14 days, and active abduction from 4-6 weeks.

Irreducible fracture dislocation is an indication for operative management. Other indications include
large displacement, younger patient, head splitting (intra-articular fractures). However, the recent
PROFHER trial (1) has suggested no benefit to operative intervention on patient outcome (it must be
applied cautiously as majority of patients were elderly with extraarticular fractures). Options available
for surgical management include:

ORIF Most commonly used. Plate and screw fixation. Can reconstruct
complex fractures.

Intramedullary nail Suitable for extra-articular configuration, predominantly surgical neck


+/- GT fractures.

Used for un-reconstructable fractures in the older patient who has good
Hemiarthroplasty glenoid quality.

Total shoulder Unconstructable fractures where high functioning shoulder is required


arthroplasty (hemiarthroplasty will cause glenoid erosion)

Reverse shoulder Total shoulder arthroplasty that provides better functional outcome than
arthroplasty conventional total shoulder replacement.
Scapula
Background
Uncommon fractures usually associated with high energy trauma. Most commonly involve scapula
body or spine (50%), glenoid fossa and glenoid neck. Important to exclude associated life
threatening injury.

Imaging
Plain radiographs should include true anteroposterior (AP), axillary lateral and/or scapula Y view. CT
scanning is useful for defining intra-articular involvement, displacement and for three dimensional
reconstruction.

Classification
Based on the location of the fracture (coracoid, acromion, glenoid neck, glenoid fossa, scapula
body). Beware of ipsilateral glenoid neck and clavicle fracture -floating shoulder - where limb is
effectively dissociated from axial skeleton.

Treatment
The vast majority of scapula fractures are amenable to conservative management, consisting of
sling immobilisation for two weeks followed by early rehabilitation. Floating shoulder will usually
require fixation, and consideration of surgery should also be given to intra-articular and
displaced/angulated glenoid fractures.

Dislocations

Types
Dislocations around the shoulder joint include glenohumeral dislocation, acromioclavicular joint
disruption and sternoclavicular dislocation. Only glenohumeral dislocation will be covered here.

Glenohumeral dislocation
Diagnosis, classification and management are covered here.

Background
Shoulder dislocation is commonly seen in A&E. It has a high recurrence rate that is as high as 80%
in teenagers. Initial management requires emergent reduction to prevent lasting chondral damage.

Early assessment and management


Usually a traumatic cause (multi-directional instability in frequent dislocations requires discussion
with orthopaedics and is not covered here). Careful history, examination and documentation of
neurovascular status of the limb, in particular the axillary nerve (regimental badge sensation). This
should be re-assessed post manipulation. Early radiographs to confirm direction of dislocation.

Initial management consists of emergent closed reduction under under entanox and analgesia, but
often requires conscious sedation. Arm should then be immobilised in a polysling, and XR to confirm
relocation.

Imaging - True anteroposterior (AP), axillary lateral and/or scapula Y view. Reduced humeral head
should lie between acromion and coracoid on lateral/scapula view.

Types
Reduction
Direction Features Cause Examination techniques
Reduction
Direction Features Cause Examination techniques

Anterior Most Usually traumatic - anterior Loss of shoulder


Common force on arm when shoulder contour - sulcus sign. Hippocratic.
>90% is abducted, eternally Humeral head can be Milch.
rotated felt anteriorly. Stimson.

Kocher not
advised due to
complication of
fracture

Posterior 50% 50% traumatic, but Shoulder locked in Gentle lateral


missed in classically post seizure or internal rotation. XR traction to
A&E electrocution may show lightbulb adducted arm.
appearance.

Inferior Rare Associated with pectorals As for primary injury Management of


and rotator cuff tears, and primary injury
glenoid fracture

Rare Associated with As for primary injury Management of


Superior acrominon/clavicle fracture primary injury

Associated injuries

 Bankart lesion - avulsion of the anterior glenoid labrum with an anterior shoulder dislocation
(reverse Bankart if poster labrum in posterior dislocation).
 Hill Sachs defect - chondral impaction on posteriosuperior humeral head from contact with
gleonoid rim. Can be large enough to lock shoulder, requiring open reduction. (Reverse Hill
Sachs in posterior dislocation).
 Rotator cuff tear - increases with age.
 Greater or lesser tuberosity fracture - increases with age.
 Humeral neck fracture - shoulder fracture dislocation. More common in high energy trauma
and elderly. Should be discussed with orthopaedics prior to any attempted reduction.

Rotator Cuff Disease


Rotator cuff disease is a spectrum of conditions that ranges from subacromial impingement to rotator
cuff tears and eventually to rotator cuff arthropathy (arthritis).

Anatomy
The rotator cuff is a group of four muscles that are important in shoulder movements, and
maintenance of glenohumeral stability.
Scapular Humeral
Muscle attachment attachment Action Innervation

Supraspinatus Supraspinatus Superior facet of Initiation of Suprascapular nerve


fossa greater tuberosity abduction of
humerus

Infraspinatus Infraspinatus Posterior facet of External rotation Suprascapular nerve


fossa greater tuberosity of humerus

Teres Minor Lateral border Inferior facet of External rotation Axillary Nerve
greater tuberosity of humerus

Subscapularis Subscapular Lesser tuberosity Internal rotation Upper and lower


fossa of humerus subscapular nerve

 The inferior rotator cuff muscles (infraspinatus, teres minor, and subscapularis) balance the
superior pull of the deltoid. Injury/tear results in upward migration of the humeral head on the
glenoid (can be seen on AP radiograph).
 Likewise, the anterior muscles (subscapularis) are balanced with the posterior muscles
(infraspinatus, teres minor).

Subacromial Impingement

 The most common cause of shoulder pain, which results from impingement of the superior
cuff on the undersurface of the acromion, and an inflammatory bursitis.
 Associated with certain types of acromial morphology (Bigliani classification).
 Presents as insidious pain which is exacerbated by overhead activities.

Rotator Cuff Tear


 Often presents as an acute event on the background of chronic subacromial impingement in
the older patient, but can present as an avulsion injury in younger patients.
 Majority of tears are to the superior cuff (supraspinatus, infraspinatus, teres minor), though a
tear to subscapularis is associated with subcoracoid impingement.
 Tears present as pain and weakness when using the muscles in question.

Rotator Cuff Arthropathy

 Defined as shoulder arthritis in the setting of rotator cuff dysfunction. Results from superior
migration due to the loss of rotator cuff function and integrity. Unopposed deltoid pulls the
humeral head superiorly.
 Associated with massive chronic cuff tears.

Imaging

Plain radiographs

 AP of the shoulder may show superior migration of the humerus with a cuff tear, and features
of arthritis with arthropathy. Other causes of pain may also be identified (e.g. calcific
tendonitis/fracture)
 Outlet view is useful for defining the acromial morphology

USS

 Allows dynamic imaging of the cuff, and is inexpensive. However, it is very user dependent.

MRI

 Best imaging modality for cuff pathology.


 Also allows imaging of the rest of the shoulder. When intra-articular pathology is suspected,
can be combined with an arthrogram for improved sensitivity and specificity.

Treatment

Subacromial impingement

 Physiotherapy, oral anti-inflammatory medication


 Subacromial steroid injection can settle inflammation
 Arthroscopic subacromial decompression by shaving away the undersurface of the
acromion, more space is created for the rotator cuff. Cuff integrity is assessed also at time of
surgery, and can be repaired if necessary.

Rotator cuff tear

 When considering repair of a cuff tear, the age and activity of the patient, the nature of the
tear (degenerative vs. acute traumatic), and the size and retraction of the tear should be
considered when making a surgical plan.
 Mild tears or tears in the elderly can be managed conservatively, as outlined above.
 Moderate tears can be repaired arthroscopically. Massive or retracted tears will often require
an open repair (occasionally with a tendon transfer). Subacromial decompression is
performed at the same time to reduce impingement, symptoms and recurrence.

Calcific tendonitis
Calcific tendonitis involves calcific deposits within tendons anywhere in the body, but most
commonly in the rotator cuff (specifically the supraspinatus tendon). When present in the shoulder, it
is associated with subacromial impingement and pain.

Pathology

 More common in women aged 30-60 years.


 Association with diabetes and hypothyroidism

There are three stages of calcification

 Formative phase characterized by calcific deposits


 Resting phase deposit is stable, but presents with impingement problems
 Resorptive phase phagocytic resorption. Most painful stage.

Presentation

 Similar in presentation to subacromial impingement, with pain especially with over head
activities. Atraumatic in nature.

Imaging

 Plain radiographs show calcification of the rotator cuff, usually within 1.5cm of its insertion on
the humerus. Supraspinatus outlet views can show level of impingment. Further imaging is
rarely needed.
Treatment

 Non-operative NSAIDS, steroid injection (controversial, but practiced) and physiotherapy.


Approximately 75% will resolve by 6 months with conservative management.
 Ultrasound guided or surgical needle barbotage can break down deposits and resolve
symptoms. Occasionally surgical excision is required.

Adhesive capsulitis (Frozen Shoulder)

 Pain and loss of movement of shoulder joint, which involves fibroplastic proliferation of
capsular tissue, causing soft tissue scarring and contracture. Patients present with a painful
and decreased arc of motion.
 Associated with prolonged immobilization, previous surgery, thyroid disorders (AI) and
diabetes
 Classically three stages which can take up to two years to resolve:

Stage one the freezing and painful stage


Stage two the frozen and stiff stage
Stage three the thawing stage, where shoulder movement slowly improves

Imaging

 Plain radiographs to exclude other causes of a painful shoulder


 MRI arthrogram may show capsular contracture, and again may be used to exclude cuff
pathology. However, often not performed as diagnosis is largely clinical.

Treatment

 Non-operative NSAIDS, steroid injection and physiotherapy. Patience is required as


condition can take up to 2 years to improve.
 Operative MUA or arthroscopic adhesiolysis (release of adhesions) can expedite recovery,
followed by intensive physiotherapy.

Glenohumeral Arthritis
Shoulder arthritis presents with the normal symptoms of arthritis, however primary osteoarthritis is
not as commonly the primary cause as seen in other large joints.. A large proportion of shoulder
arthritis is rheumatoid (RA), post traumatic, or secondary to rotator cuff arthropathy (discussed
above).

Treatment
 Surgical options include hemiarthroplasty, total shoulder replacement and reverse geometry
total shoulder replacement (used when rotator cuff function is absent).

References
1. JAMA. 2015;313(10):1037-1047. doi:10.1001/jama.2015.1629
Next question
Theme: Knee injuries

A. Anterior cruciate ligament rupture


B. Posterior cruciate ligament rupture
C. Medial collateral ligament tear
D. Lateral collateral ligament tear
E. Torn meniscus
F. Chondromalacia patellae
G. Dislocated patella
H. Fractured patella
I. Tibial plateau fracture

What is the most likely injury for scenario given? Each option may be used once, more than once or
not at all.

57. A 38 year old man is playing football when he slips over during a tackle. His knee is
painful immediately following the fall. Several hours later he notices that the knee has
become swollen. Following a course of non steroidal anti inflammatory drugs and rest the
situation improves. However, complains of recurrent pain. On assessment in clinic you
notice that it is impossible to fully extend the knee, although the patient is able to do so
when asked.

You answered Anterior cruciate ligament rupture

The correct answer is Torn meniscus

Theme from September 2012 Exam


Twisting sporting injuries followed by delayed onset of knee swelling and locking are
strongly suggestive of a menisceal tear. Arthroscopic menisectomy is the usual treatment.

58. A 34 year old woman is a passenger in a car during an accident. Her knee hits the
dashboard. On examination the tibia looks posterior compared to the non injured knee.

You answered Anterior cruciate ligament rupture

The correct answer is Posterior cruciate ligament rupture

In ruptured posterior cruciate ligament the tibia lies back on the femur and can be drawn
forward during a paradoxical draw test.

59. A 28 year old professional footballer is admitted to the emergency department. During a
tackle he is twisted with his knee flexed. He hears a loud crack and his knee rapidly
becomes swollen.
Anterior cruciate ligament rupture

This is common in footballers as the football boot studs stick to the ground and high
twisting force is applied to a flexed knee. Rapid joint swelling also supports the diagnosis.

Please rate this question:

Discuss and give feedback


Next question

Knee injury

Types of injury

Ruptured anterior  Sport injury


cruciate ligament  Mechanism: high twisting force applied to a bent knee
 Typically presents with: loud crack, pain and RAPID joint
swelling (haemoarthrosis)
 Poor healing
 Management: intense physiotherapy or surgery

Ruptured posterior  Mechanism: hyperextension injuries


cruciate ligament  Tibia lies back on the femur
 Paradoxical anterior draw test

Rupture of medial  Mechanism: leg forced into valgus via force outside the leg
collateral ligament  Knee unstable when put into valgus position

Menisceal tear  Rotational sporting injuries


 Delayed knee swelling
 Joint locking (Patient may develop skills to "unlock" the knee
 Recurrent episodes of pain and effusions are common, often
following minor trauma

Chondromalacia  Teenage girls, following an injury to knee e.g. Dislocation


patellae patella
 Typical history of pain on going downstairs or at rest
 Tenderness, quadriceps wasting
Dislocation of the  Most commonly occurs as a traumatic primary event, either
patella through direct trauma or through severe contraction of
quadriceps with knee stretched in valgus and external rotation
 Genu valgum, tibial torsion and high riding patella are risk
factors
 Skyline x-ray views of patella are required, although displaced
patella may be clinically obvious
 An osteochondral fracture is present in 5%
 The condition has a 20% recurrence rate

Fractured patella  2 types:

i. Direct blow to patella causing undisplaced fragments


ii. Avulsion fracture

Tibial plateau fracture  Occur in the elderly (or following significant trauma in young)
 Mechanism: knee forced into valgus or varus, but the knee
fractures before the ligaments rupture
 Varus injury affects medial plateau and if valgus injury, lateral
plateau depressed fracture occurs
 Classified using the Schatzker system (see below)

Schatzker Classification system for tibial plateau fractures


Type Anatomical description Features

1 Vertical split of lateral Fracture through dense bone, usually in the young. It may be
condyle virtually undisplaced, or the condylar fragment may be
pushed inferiorly and tilted

2 Vertical split of the lateral The wedge fragment (which may be of variable size), is
condyle combined with an displaced laterally; the joint is widened. Untreated, a valgus
adjacent load bearing part deformity may develop
of the condyle

3 Depression of the articular The split does not extend to the edge of the plateau.
surface with intact condylar Depressed fragments may be firmly embedded in
rim subchondral bone, the joint is stable
Type Anatomical description Features

4 Fragment of the medial Two injuries are seen in this category; (1) a depressed
tibial condyle fracture of osteoporotic bone in the elderly. (2) a high
energy fracture resulting in a condylar split that runs from
the intercondylar eminence to the medial cortex. Associated
ligamentous injury may be severe

5 Fracture of both condyles Both condyles fractured but the column of the metaphysis
remains in continuity with the tibial shaft

6 Combined condylar and High energy fracture with marked comminution


subcondylar fractures

Next question
A 10 year old boy is referred to the orthopaedic clinic with symptoms of right knee pain. He has
suffered pain for the past 3 months and the pain typically lasts for several hours. On examination he
walks with an antalgic gait and has apparent right leg shortening. The right knee is normal but the
right hip reveals pain on internal and external rotation. Imaging shows flattening of the femoral head.
Which of the following is the most likely underlying diagnosis?

Osteogenesis imperfecta

Child abuse

Osteosarcoma

Osteopetrosis

Perthes disease

This is a typical description of Perthes disease. Management involves keeping the femoral head in
the acetabulum by braces, casts or surgery.
Please rate this question:

Discuss and give feedback


Next question

Perthes disease

Perthes disease

 Idiopathic avascular necrosis of the femoral epiphysis of the femoral head


 Impaired blood supply to femoral head, causing bone infarction. New vessels develop and
ossification occurs. The bone either heals or a subchondral fracture occurs.

Clinical features

 Males 4x's greater than females


 Age between 2-12 years (the younger the age of onset, the better the prognosis)
 Limp
 Hip pain
 Bilateral in 20%
Diagnosis
Plain x-ray, Technetium bone scan or magnetic resonance imaging if normal x-ray and symptoms
persist.

Catterall staging
Stage Features

Stage 1 Clinical and histological features only

Stage 2 Sclerosis with or without cystic changes and preservation of the articular surface

Stage 3 Loss of structural integrity of the femoral head

Stage 4 Loss of acetabular integrity

Management

 To keep the femoral head within the acetabulum: cast, braces


 If less than 6 years: observation
 Older: surgical management with moderate results
 Operate on severe deformities

Prognosis
Most cases will resolve with conservative management. Early diagnosis improves outcomes.
Next question
Which statement relating to talipes equinovarus is untrue?

It has an annual incidence of around 1 in 1000 in the UK.

The muscles involved in the disorder are intrinsically abnormal.

The cuboid is classically displaced medially.

All cases should be treated with an Ilizarov frame initially unless there is minor
deformity.

The talocalcaneal angle is typically less than 20 degrees in club foot.

In most cases of Club Foot conservative measures should be tried first. The Ponsetti method is a
popular approach. Severe cases may benefit from Ilizarov frame re-aligment.
Please rate this question:

Discuss and give feedback


Next question

Talipes Equinovarus

Congenital talipes equinovarus.


Features:

 Equinus of the hindfoot.


 Adduction and varus of the midfoot.
 High arch.

Most cases in developing countries. Incidence in UK is 1 per 1000 live births. It is more common in
males and is bilateral in 50% cases. There is a strong familial link(1). It may also be associated with
other developmental disorders such as Down's syndrome.

Key anatomical deformities (2):

 Adducted and inverted calcaneus


 Wedge shaped distal calcaneal articular surface
 Severe Tibio-talar plantar flexion.
 Medial Talar neck inclination
 Displacement of the navicular bone (medially)
 Wedge shaped head of talus
 Displacement of the cuboid (medially)

Management
Conservative first, the Ponseti method is best described and gives comparable results to surgery. It
consists of serial casting to mold the foot into correct shape. Following casting around 90% will
require a Achilles tenotomy. This is then followed by a phase of walking braces to maintain the
correction.

Surgical correction is reserved for those cases that fail to respond to conservative measures. The
procedures involve multiple tenotomies and lengthening procedures. In patients who fail to respond
surgically an Ilizarov frame reconstruction may be attempted and gives good results.

References
1. Wynne-Davies R, Littlejohn A, Gormley J. Aetiology and interrelationship of some common
skeletal deformities. (Talipes equinovarus and calcaneovalgus, metatarsus varus, congenital
dislocation of the hip, and infantile idiopathic scoliosis). J Med Genet. 1982 Oct;19(5):321-8.
2. Horn BD, Davidson RS. Current treatment of clubfoot in infancy and childhood. Foot Ankle Clin.
2010 Jun;15(2):235-43.
3. Clarke NM, Uglow MG, Valentine KM. Comparison of Ponseti Versus Surgical Treatment in
Congenital Talipes Equinovarus. J Foot Ankle Surg. 2011 Jun 14.
Next question
Which of the following is least likely to impair bone fracture healing?

Radiotherapy

Osteoporosis

Administration of non steroidal anti inflammatory drugs

Preservation of periosteum

Presence of osteomyelitic sequestra

Periosteal preservation helps fractures to heal.


Please rate this question:

Discuss and give feedback


Next question

Fracture healing

Bone fracture
- Bleeding vessels in the bone and periosteum
- Clot and haematoma formation
- The clot organises over a week (improved structure and collagen)
- The periosteum contains osteoblasts which produce new bone
- Mesenchymal cells produce cartilage (fibrocartilage and hyaline cartilage) in the soft tissue around
the fracture
- Connective tissue + hyaline cartilage = callus
- As the new bone approaches the new cartilage, endochondral ossification occurs to bridge the gap
- Trabecular bone forms
- Trabecular bone is resorbed by osteoclasts and replaced with compact bone

Factors affecting fracture healing

 Age
 Malnutrition
 Bone disorders: osteoporosis
 Systemic disorders: diabetes, Marfan's syndrome and Ehlers-Danlos syndrome cause
abnormal musculoskeletal healing.
 Drugs: steroids, non steroidal anti inflammatory agents.
 Type of bone: Cancellous (spongy) bone fractures are usually more stable, involve greater
surface areas, and have a better blood supply than cortical (compact) bone fractures.
 Degree of Trauma: The more extensive the injury to bone and surrounding soft tissue, the
poorer the outcome.
 Vascular Injury: Especially the femoral head, talus, and scaphoid bones.
 Degree of Immobilization
 Intra-articular Fractures: These fractures communicate with synovial fluid, which contains
collagenases that retard bone healing.
 Separation of Bone Ends: Normal apposition of fracture fragments is needed for union to
occur. Inadequate reduction, excessive traction, or interposition of soft tissue will prevent
healing.
 Infection

Next question
Theme: Disorders of the hip

A. Perthes disease

B. Developmental dysplasia of the hip

C. Osteoarthritis

D. Slipped upper femoral epiphysis

E. Septic arthritis

F. Rheumatoid arthritis

G. Intra capsular fracture of the femoral neck

H. Extra capsular fracture of the femoral neck

Please select the most likely underlying diagnosis for the scenario given. Each option may be used
once, more than once or not at all.

63. An obese 14 year old boy presents with difficulty running and mild knee and hip pain. There is no
antecedent history of trauma. On examination internal rotation is restricted but the knee is
normal with full range of passive movement possible and no evidence of effusions. Both the C-
reactive protein and white cell count are normal.

You answered Perthes disease

The correct answer is Slipped upper femoral epiphysis

Slipped upper femoral epiphysis is the commonest adolescent hip disorder. It occurs most
commonly in obese males. It may often present as knee pain which is usually referred from the
ipsilateral hip. The knee itself is normal. The hip often limits internal rotation. The diagnosis is
easily missed. X-rays will show displacement of the femoral epiphysis and the degree of its
displacement may be calculated using the Southwick angle. Treatment is directed at preventing
further slippage which may result in avascular necrosis of the femoral head.

64. A 6 year old boy presents with pain in the hip it is present on activity and has been worsening
over the past few weeks. There is no history of trauma. He was born by normal vaginal delivery at
38 weeks gestation On examination he has an antalgic gait and limitation of active and passive
movement of the hip joint in all directions. C-reactive protein is mildly elevated at 10 but the
white cell count is normal.

Perthes disease

This is a typical presentation for Perthes disease. X-ray may show flattening of the femoral head
or fragmentation in more advanced cases.

65. A 30 year old man presents with severe pain in the left hip it has been present on and off for
many years. He was born at 39 weeks gestation by emergency caesarean section after a long
obstructed breech delivery. He was slow to walk and as a child was noted to have an antalgic gait.
He was a frequent attender at the primary care centre and the pains dismissed as growing pains.
X-rays show almost complete destruction of the femoral head and a narrow acetabulum.

You answered Perthes disease

The correct answer is Developmental dysplasia of the hip

Developmental dysplasia of the hip. Usually diagnosed by Barlow and Ortolani tests in early
childhood. Most Breech deliveries are also routinely subjected to USS of the hip joint. At this
young age an arthrodesis may be preferable to hip replacement.

Early plain x-ray changes in Perthes Disease:


Widening of the joint space.
Sub chondral linear lucency.

Please rate this question:

Discuss and give feedback

Next question

Paediatric orthopaedics
Diagnosis Mode of presentation Treatment Radiology

Developmental Usually diagnosed in infancy Splints and harnesses or Initially no obvious


dysplasia of the by screening tests. May be traction. In later years change on plain films
hip bilateral, when disease is osteotomy and hip and USS gives best
unilateral there may be leg realignment procedures may resolution until 3
length inequality. As be needed. In arthritis a joint months of age. On
disease progresses child replacement may be needed. plain films Shentons
may limp and then early However, this is best line should form a
onset arthritis. More deferred if possible as it will smooth arc
common in extended almost certainly require
breech babies. revision

Perthes Disease Hip pain (may be referred Remove pressure from joint X-rays will show
to the knee) usually to allow normal flattened femoral
occurring between 5 and 12 development. Physiotherapy. head. Eventually in
years of age. Bilateral Usually self-limiting if untreated cases the
disease in 20%. diagnosed and treated femoral head will
promptly. fragment.

Slipped upper Typically seen in obese Bed rest and non-weight X-rays will show the
femoral male adolescents. Pain is bearing. Aim to avoid femoral head displaced
epiphysis often referred to the knee. avascular necrosis. If severe and falling
Limitation to internal slippage or risk of it occurring inferolaterally (like a
rotation is usually seen. then percutaneous pinning of melting ice cream
Knee pain is usually present the hip may be required. cone) The Southwick
2 months prior to hip angle gives indication
slipping. Bilateral in 20%. of disease severity

Next question
Theme: Paediatric fractures

A. Non accidental injury


B. Accidental fracture
C. Rickets
D. Metabolic bone disease of prematurity
E. Hypophosphataemic rickets
F. Osteopetrosis
G. Osteogenesis imperfecta
H. Hypoparathyroidism
I. Osteoporosis

Please select the most likely explanation for each of the following injury scenarios. Each option may
be used once, more than once or not at all.

66. A toddler aged 3 years presents to the Emergency Department with swelling of his leg and
is found to have a spiral fracture of the tibia. His mother reports that he had tripped and
fallen the previous day but she had not noticed any sign of injury at the time. She is a
single parent with little family support. The child is not on the child protection register.

Non accidental injury

Delayed presentation is unusual and should raise concern. In addition spiral fractures are
usually the result of rotational injury which is not compatible with the mechanism
proposed by the parent.

67. A 5 month baby boy presents with swelling of his right arm and is found to have a spiral
fracture of the humerus. He had been in the care of her mother's boyfriend who reported
that he had nearly dropped him that day when reaching for his bottle and had inadvertently
pulled on his arm to save him. He was immediately taken to the Emergency Department.

You answered Non accidental injury

The correct answer is Accidental fracture

The mechanism fits with the fracture pattern and the presentation is not delayed.

68. An infant is admitted with symptoms and signs of respiratory infection and is found to
have several posterior rib fractures on chest radiograph. He was born prematurely at 37
weeks' gestation and was observed overnight on the special care baby unit for tachypnoea
which settled by the following day. On assessment it is also apparent that his head
circumference has increased at an excessive rate and has crossed 3 centiles since birth.
Non accidental injury

Posterior rib fractures are extremely unusual in neonates. The change in head size may be
accounted for by hydrocephalus which may occur as a sequelae from head injury.

Please rate this question:

Discuss and give feedback


Next question

Paediatric fractures

Paediatric fracture types


Type Injury pattern

Complete fracture Both sides of cortex are breached

Toddlers fracture Oblique tibial fracture in infants

Plastic deformity Stress on bone resulting in deformity without cortical disruption

Greenstick fracture Unilateral cortical breach only

Buckle fracture Incomplete cortical disruption resulting in periosteal haematoma only

Growth plate fractures


In paediatric practice fractures may also involve the growth plate and these injuries are classified
according to the Salter- Harris system (given below):

Type Injury pattern

I Fracture through the physis only (x-ray often normal)

II Fracture through the physis and metaphysis


III Fracture through the physis and epiphyisis to include the joint

IV Fracture involving the physis, metaphysis and epiphysis

V Crush injury involving the physis (x-ray may resemble type I, and appear normal)

As a general rule it is safer to assume that growth plate tenderness is indicative of an underlying
fracture even if the x-ray appears normal. Injuries of Types III, IV and V will usually require surgery.
Type V injuries are often associated with disruption to growth.

Non accidental injury

 Delayed presentation
 Delay in attaining milestones
 Lack of concordance between proposed and actual mechanism of injury
 Multiple injuries
 Injuries at sites not commonly exposed to trauma
 Children on the at risk register

Pathological fractures
Genetic conditions, such as osteogenesis imperfecta, may cause pathological fractures.

Osteogenesis imperfecta

 Defective osteoid formation due to congenital inability to produce adequate intercellular


substances like osteoid, collagen and dentine.
 Failure of maturation of collagen in all the connective tissues.
 Radiology may show translucent bones, multiple fractures, particularly of the long bones,
wormian bones (irregular patches of ossification) and a trefoil pelvis.

Subtypes

 Type I The collagen is normal quality but insufficient quantity.


 Type II- Poor collagen quantity and quality.
 Type III- Collagen poorly formed. Normal quantity.
 Type IV- Sufficient collagen quantity but poor quality.

Osteopetrosis
 Bones become harder and more dense.
 Autosomal recessive condition.
 It is commonest in young adults.
 Radiology reveals a lack of differentiation between the cortex and the medulla described as
marble bone.

Next question
In paediatric orthopaedic surgery, which of the following does not fulfill the Kocher criteria for septic
arthritis?

ESR > 40mm/h

Positive blood culture

Fever

White cell count > 12, 000

Non weight bearing on the affected side

Kocher criteria
1. Non weight bearing on affected side
2. ESR > 40 mm/hr
3. Fever
4. WBC count of >12,000 mm 3
- When 4/4 criteria are met, there is a 99% chance that the child has septic arthritis

The Kocher criteria do not consider blood culture results.


Please rate this question:

Discuss and give feedback


Next question

Septic arthritis- Paediatric

Septic arthritis

 Staph aureus commonest organism


 Urgent washout and antibiotics otherwise high risk of joint destruction

Diagnosis

 Plain x-rays
 Consider aspiration
 Utilise the Kocher criteria (see below)
Kocher criteria:
1. Non weight bearing on affected side
2. ESR > 40 mm/hr
3. Fever
4. WBC count of >12,000 mm 3
- when 4/4 criteria are met, there is a 99% chance that the child has septic arthritis

Treatment
Surgical drainage of the affected joint is required, this should be done as soon as possible since
permanent damage to the joint may occur. In some cases repeated procedures are necessary.
Appropriate intravenous antibiotics should be administered.

Next question
Theme: Ankle fractures

A. Surgical fixation

B. Below knee amputation

C. Aircast boot

D. Application of full leg plaster cast to include midfoot

E. Application of below knee plaster cast to include the midfoot

F. Application of external fixation device

G. Application of compression bandage and physiotherapy.

Please select the most appropriate management for the injury type described. Each option may be
used once, more than once or not at all.

70. A 24 year old man falls sustaining an inversion injury to his ankle. On examination he is tender
over the lateral malleolus only. On x-ray there is a fibular fracture that is distal to the
syndesmosis.

You answered Surgical fixation

The correct answer is Application of below knee plaster cast to include the midfoot

Theme from 2010 Exam


These distal injuries are generally managed conservatively. Conservative management will involve
a below knee cast, this will need to extend to the midfoot. It can be substituted for an aircast boot
once radiological union is achieved.

71. An 86 year old lady stumbles and falls whilst opening her front door. On examination her ankle is
swollen with both medial and lateral tenderness. X rays demonstrate a fibular fracture at the level
of the syndesmosis.

You answered Surgical fixation

The correct answer is Application of below knee plaster cast to include the midfoot
Although, this is a potentially unstable injury operative fixation in this age group generally gives
poor results owing to poor quality bone. A below knee cast should be applied in the first instance.
If this fails to provide adequate control it can be extended above the knee.

72. A 25 year old man suffers an injury whilst playing rugby involving a violent twist to his left lower
leg. On examination both malleoli are tender and the ankle joint is slightly swollen. On x-ray there
is a spiral fracture of the fibula and widening of the ankle mortise.

Surgical fixation

This is a variant of the Weber C fracture in which disruption of the tibio-fibular syndesmosis
occurs leading to joint disruption. Surgical repair is warranted.

Please rate this question:

Discuss and give feedback

Next question

Ankle injuries

An ankle fracture relates to a fracture around the tibio-talar joint. It generally refers to a fracture
involving the lateral, and/or medial and/or posterior malleolus. Pilon and Tillaux fractures are also
considered to be ankle fractures, but are not covered here.
Ankle fractures are common. They effect men and women in equal numbers, but men have a higher
rate as young adults (sports and contact injuries), and women a higher rate post-menopausal
(fragility type fracture).

Osseous anatomy
The ankle (or mortise) joint consists of the distal tibia (tibial plafond and posterior malleolus), the
distal fibula (lateral malleolus), and the talus. The main movement at the ankle joint is plantar and
dorsiflexion.

Ligamentous anatomy
Medial side: Deltoid ligament. This is divided into superficial and deep portions. It is the primary
restraint to valgus tilting of the talus.
Lateral side: Lateral ligament complex consisting from anterior to posterior of the anterior talofibular
ligament (ATFL), calcaneofibular ligament (CFL), and the posterior talofibular ligament (PTFL).
Together they resist valgus stress to the ankle, and are a restraint to anterior translation of the talus
within the mortise joint.

Syndesmosis: The syndesmosis is a ligament complex between the distal tibia and fibula, holding
the two bones together. It is fundamental to the integrity of the ankle joint, and its disruption leads to
instability. It consists of (from anterior to posterior) the anterior-inferior tibiofibular ligament (AITFL),
the transverse tibiofibular ligament (TTFL), the interosseous membrane, and the posterior-inferior
tibiofibular ligament (PITFL).

Presentation and initial management


Patients will present following a traumatic event with a painful, swollen ankle, and reluctance/inability
to weight bear. The Ottawa rules can be applied to differentiate between an ankle fracture and
sprain, but can be unreliable.
In high energy injuries, management should follow ATLS principles to identify more significant
injuries first. Neurovascular status of the foot should be documented, and open injuries should be
excluded. If an open injury is identified, it should be managed in line with BOAST 4 principles1. If an
obvious deformity exists, it should be reduced as soon as possible with appropriate analgesia or
conscious sedation. Radiographs of clearly deformed or dislocated joints are not necessary, and
removing the pressure on the surrounding soft tissues from the underlying bony deformity is the
priority. If the fracture pattern is not clinically obvious then plain radiographs are appropriate and will
guide the subsequent manipulation during plaster-of-paris below knee backslab application.

Imaging
AP, lateral and mortise views (20o internal rotation) are essential to evaluate fracture displacement
and syndesmotic injury. Decreased tibiofibular overlap, medial joint clear space and lateral talar shift
all indicate a syndesmotic injury. (In subtle cases of shift, imaging the uninjured ankle can be helpful
as a proportion of the population have little or no tibiotalar overlap 2.)

Where there is suspicion of syndesmosis involvement in the absence of radiographic evidence,


stress radiographs can be diagnostic.
Complex fracture patterns (and increasingly posterior malleolar fractures) are best defined using
CT.

Classification
The most commonly used classifications are Lauge-Hansen and Danis-Weber.

Lauge-Hansen
Comprises two parts: first part is the foot position, and the second part is the force applied. Useful for
understanding the forces involved and therefore predict the ligamentous or bony injury. Results in
four injury patterns:
Supination - Adduction (SA) - 10-20%
Supination - External rotation (SER) - 40-75%
Pronation - Abduction (PA) - 5-20%
Pronation - External rotation (PER) - 5-20%

Not often used in clinical practice but good for understanding the principles of ankle fracture.

Danis-Weber
Commonly used. Based on the level of the fibula fracture in relation to the syndesmosis. The more
proximal, the greater the risk of syndesmotic injury and therefore fracture instability.
A - fracture below the level of the syndesmosis
B - fracture at the level of the syndesmosis / level of the tibial plafond
C - fracture above the level of the syndesmosis. This includes Maisonneuve fractures (proximal
fibula fracture), which can be associated with ankle instability. Beware the high fibula fracture - it
may be an ankle fracture!

The Weber classification is based purely on the the lateral side. All injuries can include a medial or
posterior bony or ligamentous injury which also dictates fracture stability (bimalleolar and trimalleolar
fractures are more unstable).

Treatment
When deciding upon treatment for an ankle fracture, one must consider both the fracture and the
patient. Diabetic patients and smokers are at greater risk of post-operative complication, especially
wound problems and infection. Likewise, the long term outcome of post-traumatic arthritis from a
malunited ankle fracture is extremely important for a young patient, but not as relevant in the elderly.
Therefore, normal surgical decision processes apply as with all fractures.

Defining stability of an ankle fracture underpins the treatment decision.


Weber A - Unimalleolar Weber A Weber fractures by definition are stable and therefore can be
mobilised fully weight bearing in an ankle boot.

Weber C - Fractures tend to include syndesmotic disruption and are usually bimalleolar (either bony
or ligamentous). They are therefore unstable and usually require operative fixation. In addition to the
fracture fixation, the syndesmosis usually requires reconstruction/augmentation with screws to
restore the joint integrity and function.

Weber B - B fractures vary greatly. They can be part of a trimalleolar injury and therefore extremely
unstable, requiring fixation. Alternatively, a uni-malleolar Weber B fracture can be a stable injury,
and therefore mobilised immediately in an ankle boot. Defining the stability can be challenging, and
often involves stress radiographs, or a trial of mobilisation and repeat radiographs. Defining stability
is the subject of much ongoing research. However, treating undisplaced ankle fractures in a below
knee plaster, non-weight bearing for six weeks is still widely practised, and a safe approach.

When operative fixation is appropriate, it is usually via open reduction and internal fixation using
plates and screws. It must be carried out when soft tissue swelling has settled in order to minimise
the risk of wound problems. This can often take a week to settle.
The use of fibula nails is expanding, but is not yet mainstream. Ankle fractures can also be treated
with external fixation, or with a hind foot nail in patients who need fixation but where soft tissue or
bone quality is poor.

Post operative management


Ankle fractures generally take 6 weeks to unite enough to prevent secondary displacement. This is
therefore an appropriate time period to keep a cast on in a conservatively managed patient. Weight
bearing post-operatively depends on the quality of the fixation and bone quality, and preference
varies between surgeons, ranging from aggressive early mobilisation to a period of non-weight
bearing. Return to activities takes approximately three months, and often requires assistance of a
physiotherapist to improve range-of-movement and muscle strengthening.

References
1. http://www.boa.ac.uk/publications/boa-standards-trauma-boasts/
2. Shah AS, Kadakia AR, Tan GJ, Karadsheh MS, Wolter TD, Sabb B. Radiographic evaluation of
the normal distal tibiofibular syndesmosis. Foot Ankle Int. 2012;33(10):870-6

Next question
Theme: Management of hip fractures

A. MRI scan

B. Hemiarthroplasty

C. Bone scintigraphy

D. Conservative management

E. Total hip replacement

F. Insertion of intra medullary nail

G. Hip arthrodesis

H. Internal fixation

For each fracture scenario please select the most appropriate management option from the list.
Each option may be used once, more than once or not at all.

73. An otherwise fit 74 year old man presents with pain in the right hip following minimal trauma. On
examination his leg is shortened and externally rotated. Plain films demonstrate a displaced
intracapsular fracture of the femoral neck.

You answered MRI scan

The correct answer is Total hip replacement

In otherwise fit patients aged over 70, the best long term functional outcomes are obtained with
total hip arthroplasty.

74. A 72 year old retired teacher is admitted to A&E with a fall and hip pain. He is normally fit and
well. He lives with his son in a detached, 2 storey house. A hip x-ray confirms an subtrochanteric
fracture.

You answered MRI scan

The correct answer is Insertion of intra medullary nail


Intramedullary devices are normally recommended for reverse oblique, transverse
subtrochanteric fractures.

75. A 72 year old lady stumbles and falls. On examination she is tender in the left groin and unable to
weight bear. Attempts at internal rotation produce severe pain. Plain films show of the hip show
no obvious fracture.

MRI scan

In those patients who present with a suspected hip fracture, but normal plain films the most
accurate investigation is an MRI or CT scan.

Please rate this question:

Discuss and give feedback

Next question

Hip fractures

Background
Neck of femur (NOF) fracture is a common orthopaedic presentation, with over 65000 fractures in
the UK per year. Like many orthopaedic injuries, there is a bimodal age distribution. It is imperative
to distinguish between the high energy injury in a young patient, and the low energy osteoporotic
fracture in the elderly, as their management aims are very different:

Young patient - Usually high energy trauma (e.g road traffic accident, horse riding) and needs
treating in accordance with Advanced Trauma Life Support (ATLS) principles. Will often have
associated injuries. Aim is to retain the patients own anatomy, and optimise their function.

Elderly patient - Predominantly female, fall from standing height (fragility fracture). Often patients
have multiple comorbidities that will ultimately dictate their prognosis. Aim of orthopaedic treatment
is to immediately regain patient mobility so that morbidity (infection, thromboembolic events,
pressure sores etc) and mortality associated with prolonged bed rest is avoided. Left untreated, a
neck of femur fracture can be considered a terminal event. Historically, mortality associated with
elderly hip fracture is 10% at one month, and 30% at one year. However, this has been improved in
the UK with the introduction of multidisciplinary, orthogeriatric lead care and the National Hip
Fracture Database and Best Practice Tariff.

Pertinent anatomy
Osteology - normal neck-shaft angle is 130 +/- 7 degrees, and 10 +/- 7 degrees of neck anteversion.
Vascular supply - The predominant blood supply to the femoral head and neck is from the medial
and lateral femoral circumflex arteries (branches of profunda femoris). These anastomose and
pierce the joint capsule at the base of the neck, mainly posteriorly. There is a small vascular
contribution from the artery of the ligament teres. Understanding the blood supply is fundamental to
the decision making process in treating NOF fractures.

Presentation and initial management


Typically, patients present with pain in the hip/groin, a shortened, abducted, externally rotated leg
(due to the unopposed pull of the muscles that act across the hip joint) and the inability to straight-
leg-raise. With undisplaced fractures, signs are more subtle.
High energy injuries should be treated in line with ATLS principles. All patients should be fluid
resuscitated, have adequate pain relief (often with a fascio-iliiaca nerve block), and be optimised for
surgery. In addition, elderly patients should be assessed by an orthogeriatrician.

Imaging
Anteroposterior and cross-table lateral plain radiographs are sufficient to diagnose the majority of
NOF fractures. If the fracture extends below the level of the lesser trochanter, or there is any
possibility of pathological fracture, full length femur views are essential to plan surgery.

Where there is a high index of suspicion of fracture, but plain radiographs are inconclusive, gold
standard investigation is MRI. However, if unavailable within 24 hours, or if the patient will not
tolerate MRI, CT is appropriate. The majority of fractures can be seen with modern CT techniques,
and so this is becoming first line in many hospitals.

Classification
There has been a move away from named classification systems towards descriptive classification
systems.
Two main types of NOF exist: Intra-capsular, and extra-capsular. Extra-capsular fractures are further
divided into pertrochanteric or subtrochanteric (within 5cm distal to the lesser trochanter). All
fractures are then described as undisplaced, minimally displaced, or displaced.
Femoral neck and head blood supply disruption is common with intracapsular NOF fractures, and
rare with extracapsular fractures. This fundamental principle underpins the practise of arthroplasty
for intracapsular fractures, and fixation for extracapsular fractures.

If you wish to use a named classification system, the most commonly used are below:
Elderly intracapsular - Garden Classification
Young intrasapsular - Pauvels Classification
Intertrochanteric - Evans
Subtrochanteric - Russell Taylor

Treatment
In general, NOF fractures are treated operatively except if the patient is deemed unlikely to survive
an anaesthetic. Best Practice Tarif (BPT) dictates that surgery should happen within 36 hours, as
delay of greater than 48 hours is associated with increased morbidity and mortality. Below are
suggested algorithms for the treatment of NOF. There are some areas of debate/controversy which
are detailed below.

Image sourced from Wikipedia

* The priority with the young patient is to retain the femoral head if possible, even with a displaced
intracapsular fracture. The risk of avascular necrosis and non-union (and therefore revision surgery)
associated with internal fixation needs weighing up against the sequelae of total hip replacement in
the young (wear, dislocation, revision). Discussion is necessary with the patient, on a case by case
basis.

** Undisplaced fractures in the elderly can be treated with internal fixation, often with cannulated
screws. This is appropriate for valgus impacted subcapital fractures which are inherently stable, to
prevent secondary displacement. This does still carry the risk of AVN or non-union, and therefore a
future revision. For this reason, many surgeons advocate arthroplasty as a single surgery.

*** NICE guidance - patients who fulfil these criteria should be offered total hip replacement which
conveys better function and prosthetic survivorship, compared with hemiarthroplasty, but at an
increased risk of dislocation.
Image sourced from Wikipedia

* Intertrochanteric fractures vary greatly in their stability. If the trochanter (and therefore lateral wall),
and medial calcar is in tact, then the fracture configuration bears stability. This can be treated with a
DHS, as collapse of the fracture is predictable. Where either or both structures are involved in the
fracture, stability becomes compromised and many surgeons will favour using an intramedullary
device. This is an ongoing debate, and difficult to test in an exam setting.

Post operative management


Patients should be mobilised fully weight bearing where possible. Care is multidisciplinary in its
delivery. Elderly patients should have orthogeriatrician assessment of comorbidity, and bone health
with secondary prevention measures if appropriate. There should be early involvement of
physiotherapy and occupational therapy services. For further guidance see sources listed below.

NICE clinical guidance on hip fracture: https://www.nice.org.uk/guidance/cg124


Best Practice Tarif:
www.nhfd.co.uk/20/hipfractureR.../Best%20Practice%20Tariff%20User%20Guide.pdf
National Hip Fracture Database: www.nhfd.co.uk/

Next question
Theme: Paediatric orthopaedics

A. USS hip

B. Hip x-ray

C. Anteroposterior pelvic x-ray

D. CT scan

E. MRI scan

F. Technetium bone scan

G. USS knee

H. X-ray knee

I. Discharge and reassure

For each of the following scenarios which is the most appropriate investigation? Each option may be
used once, more than once or not at all.

76. An obese 12 year old boy presents with knee pain. On examination he has pain on internal
rotation of the hip. His knee is clinically normal.

You answered USS hip

The correct answer is Hip x-ray

The main differential diagnosis in a boy over 10 years old is of slipped upper femoral epiphysis.
Knee pain is a common presenting feature. An anteroposterior pelvic x-ray may miss a minor slip,
therefore request a hip film.

77. A baby is delivered in the breech position. Barlows and Ortolani tests are normal

USS hip

This child is at risk of developmental dysplasia of the hip (up to 20% will have DDH), so should
have the hip joints scanned to exclude this.

78. A 5 year old boy presents with a painful limp. The symptoms have been present for 8 weeks. Two
hip x-rays have been performed and appear normal.

You answered USS hip

The correct answer is Technetium bone scan

Perthes disease should be suspected in boys over 4 years old presenting with a limp. Early disease
can be missed on x-ray, therefore a bone scan should be performed. MRI is less sensitive than the
bone scan.

Please rate this question:

Discuss and give feedback

Next question

Paediatric orthopaedics

Diagnosis Mode of presentation Treatment Radiology

Developmental Usually diagnosed in infancy Splints and harnesses or Initially no obvious


dysplasia of the by screening tests. May be traction. In later years change on plain films
hip bilateral, when disease is osteotomy and hip and USS gives best
unilateral there may be leg realignment procedures may resolution until 3
length inequality. As be needed. In arthritis a joint months of age. On
disease progresses child replacement may be needed. plain films Shentons
may limp and then early However, this is best line should form a
onset arthritis. More deferred if possible as it will smooth arc
common in extended almost certainly require
breech babies. revision
Perthes Disease Hip pain (may be referred Remove pressure from joint X-rays will show
to the knee) usually to allow normal flattened femoral
occurring between 5 and 12 development. Physiotherapy. head. Eventually in
years of age. Bilateral Usually self-limiting if untreated cases the
disease in 20%. diagnosed and treated femoral head will
promptly. fragment.

Slipped upper Typically seen in obese Bed rest and non-weight X-rays will show the
femoral male adolescents. Pain is bearing. Aim to avoid femoral head displaced
epiphysis often referred to the knee. avascular necrosis. If severe and falling
Limitation to internal slippage or risk of it occurring inferolaterally (like a
rotation is usually seen. then percutaneous pinning of melting ice cream
Knee pain is usually present the hip may be required. cone) The Southwick
2 months prior to hip angle gives indication
slipping. Bilateral in 20%. of disease severity

Next question
A 5 year old boy is playing in a tree when he falls and lands on his right forearm. He is brought to the
emergency department by his parents. On examination he has bony tenderness and bruising. An X-
ray is taken and shows unilateral cortical disruption and development of periosteal haematoma.
What is the most likely diagnosis?

Buckle fracture

Greenstick fracture

Toddlers fracture

Complete fracture

None of the above

Greenstick fractures are common childhood injuries. Unilateral cortical disruption is the main
radiological feature, since involvement of both cortices makes the injury a complete fracture. Buckle
fractures will show periosteal haematoma formation only.
Please rate this question:

Discuss and give feedback


Next question

Paediatric fractures

Paediatric fracture types


Type Injury pattern

Complete fracture Both sides of cortex are breached

Toddlers fracture Oblique tibial fracture in infants

Plastic deformity Stress on bone resulting in deformity without cortical disruption


Greenstick fracture Unilateral cortical breach only

Buckle fracture Incomplete cortical disruption resulting in periosteal haematoma only

Growth plate fractures


In paediatric practice fractures may also involve the growth plate and these injuries are classified
according to the Salter- Harris system (given below):

Type Injury pattern

I Fracture through the physis only (x-ray often normal)

II Fracture through the physis and metaphysis

III Fracture through the physis and epiphyisis to include the joint

IV Fracture involving the physis, metaphysis and epiphysis

V Crush injury involving the physis (x-ray may resemble type I, and appear normal)

As a general rule it is safer to assume that growth plate tenderness is indicative of an underlying
fracture even if the x-ray appears normal. Injuries of Types III, IV and V will usually require surgery.
Type V injuries are often associated with disruption to growth.

Non accidental injury

 Delayed presentation
 Delay in attaining milestones
 Lack of concordance between proposed and actual mechanism of injury
 Multiple injuries
 Injuries at sites not commonly exposed to trauma
 Children on the at risk register

Pathological fractures
Genetic conditions, such as osteogenesis imperfecta, may cause pathological fractures.
Osteogenesis imperfecta

 Defective osteoid formation due to congenital inability to produce adequate intercellular


substances like osteoid, collagen and dentine.
 Failure of maturation of collagen in all the connective tissues.
 Radiology may show translucent bones, multiple fractures, particularly of the long bones,
wormian bones (irregular patches of ossification) and a trefoil pelvis.

Subtypes

 Type I The collagen is normal quality but insufficient quantity.


 Type II- Poor collagen quantity and quality.
 Type III- Collagen poorly formed. Normal quantity.
 Type IV- Sufficient collagen quantity but poor quality.

Osteopetrosis

 Bones become harder and more dense.


 Autosomal recessive condition.
 It is commonest in young adults.
 Radiology reveals a lack of differentiation between the cortex and the medulla described as
marble bone.

Next question
Which of the following is not typically seen in patients with a femoral neck fracture?

Malunion

Non union

Avascular necrosis

Shortening

External rotation

Malunion would be unusual with a femoral neck fracture. Because it is a weight bearing joint, if the
fracture is not united then it does not heal at all. It is for this reason that most femoral neck fractures
are fixed. Avascular necrosis is a well recognised complication and a total hip replacement or
hemiarthroplasty is usually considered in the elderly.

Please rate this question:

Discuss and give feedback

Next question

Hip fractures

Background
Neck of femur (NOF) fracture is a common orthopaedic presentation, with over 65000 fractures in
the UK per year. Like many orthopaedic injuries, there is a bimodal age distribution. It is imperative
to distinguish between the high energy injury in a young patient, and the low energy osteoporotic
fracture in the elderly, as their management aims are very different:

Young patient - Usually high energy trauma (e.g road traffic accident, horse riding) and needs
treating in accordance with Advanced Trauma Life Support (ATLS) principles. Will often have
associated injuries. Aim is to retain the patients own anatomy, and optimise their function.
Elderly patient - Predominantly female, fall from standing height (fragility fracture). Often patients
have multiple comorbidities that will ultimately dictate their prognosis. Aim of orthopaedic treatment
is to immediately regain patient mobility so that morbidity (infection, thromboembolic events,
pressure sores etc) and mortality associated with prolonged bed rest is avoided. Left untreated, a
neck of femur fracture can be considered a terminal event. Historically, mortality associated with
elderly hip fracture is 10% at one month, and 30% at one year. However, this has been improved in
the UK with the introduction of multidisciplinary, orthogeriatric lead care and the National Hip
Fracture Database and Best Practice Tariff.

Pertinent anatomy
Osteology - normal neck-shaft angle is 130 +/- 7 degrees, and 10 +/- 7 degrees of neck anteversion.
Vascular supply - The predominant blood supply to the femoral head and neck is from the medial
and lateral femoral circumflex arteries (branches of profunda femoris). These anastomose and
pierce the joint capsule at the base of the neck, mainly posteriorly. There is a small vascular
contribution from the artery of the ligament teres. Understanding the blood supply is fundamental to
the decision making process in treating NOF fractures.

Presentation and initial management


Typically, patients present with pain in the hip/groin, a shortened, abducted, externally rotated leg
(due to the unopposed pull of the muscles that act across the hip joint) and the inability to straight-
leg-raise. With undisplaced fractures, signs are more subtle.
High energy injuries should be treated in line with ATLS principles. All patients should be fluid
resuscitated, have adequate pain relief (often with a fascio-iliiaca nerve block), and be optimised for
surgery. In addition, elderly patients should be assessed by an orthogeriatrician.

Imaging
Anteroposterior and cross-table lateral plain radiographs are sufficient to diagnose the majority of
NOF fractures. If the fracture extends below the level of the lesser trochanter, or there is any
possibility of pathological fracture, full length femur views are essential to plan surgery.

Where there is a high index of suspicion of fracture, but plain radiographs are inconclusive, gold
standard investigation is MRI. However, if unavailable within 24 hours, or if the patient will not
tolerate MRI, CT is appropriate. The majority of fractures can be seen with modern CT techniques,
and so this is becoming first line in many hospitals.

Classification
There has been a move away from named classification systems towards descriptive classification
systems.
Two main types of NOF exist: Intra-capsular, and extra-capsular. Extra-capsular fractures are further
divided into pertrochanteric or subtrochanteric (within 5cm distal to the lesser trochanter). All
fractures are then described as undisplaced, minimally displaced, or displaced.
Femoral neck and head blood supply disruption is common with intracapsular NOF fractures, and
rare with extracapsular fractures. This fundamental principle underpins the practise of arthroplasty
for intracapsular fractures, and fixation for extracapsular fractures.

If you wish to use a named classification system, the most commonly used are below:
Elderly intracapsular - Garden Classification
Young intrasapsular - Pauvels Classification
Intertrochanteric - Evans
Subtrochanteric - Russell Taylor

Treatment
In general, NOF fractures are treated operatively except if the patient is deemed unlikely to survive
an anaesthetic. Best Practice Tarif (BPT) dictates that surgery should happen within 36 hours, as
delay of greater than 48 hours is associated with increased morbidity and mortality. Below are
suggested algorithms for the treatment of NOF. There are some areas of debate/controversy which
are detailed below.

Image sourced from Wikipedia

* The priority with the young patient is to retain the femoral head if possible, even with a displaced
intracapsular fracture. The risk of avascular necrosis and non-union (and therefore revision surgery)
associated with internal fixation needs weighing up against the sequelae of total hip replacement in
the young (wear, dislocation, revision). Discussion is necessary with the patient, on a case by case
basis.

** Undisplaced fractures in the elderly can be treated with internal fixation, often with cannulated
screws. This is appropriate for valgus impacted subcapital fractures which are inherently stable, to
prevent secondary displacement. This does still carry the risk of AVN or non-union, and therefore a
future revision. For this reason, many surgeons advocate arthroplasty as a single surgery.

*** NICE guidance - patients who fulfil these criteria should be offered total hip replacement which
conveys better function and prosthetic survivorship, compared with hemiarthroplasty, but at an
increased risk of dislocation.

Image sourced from Wikipedia

* Intertrochanteric fractures vary greatly in their stability. If the trochanter (and therefore lateral wall),
and medial calcar is in tact, then the fracture configuration bears stability. This can be treated with a
DHS, as collapse of the fracture is predictable. Where either or both structures are involved in the
fracture, stability becomes compromised and many surgeons will favour using an intramedullary
device. This is an ongoing debate, and difficult to test in an exam setting.

Post operative management


Patients should be mobilised fully weight bearing where possible. Care is multidisciplinary in its
delivery. Elderly patients should have orthogeriatrician assessment of comorbidity, and bone health
with secondary prevention measures if appropriate. There should be early involvement of
physiotherapy and occupational therapy services. For further guidance see sources listed below.

NICE clinical guidance on hip fracture: https://www.nice.org.uk/guidance/cg124


Best Practice Tarif:
www.nhfd.co.uk/20/hipfractureR.../Best%20Practice%20Tariff%20User%20Guide.pdf
National Hip Fracture Database: www.nhfd.co.uk/

Next question
Theme: Eponymous fractures

A. Smith's
B. Bennett's
C. Monteggia's
D. Colle's
E. Galeazzi
F. Pott's
G. Barton's

Which is the most likely eponymous fracture for the scenario given. Each option may be used once,
more than once or not at all.

81. A 14 year old boy jumps off a 10 foot wall and lands on both feet. An x-ray shows a
bimalleolar fracture of the right ankle.

You answered Smith's

The correct answer is Pott's

82. A 22 year old drunk man is involved in a fight. He hurts his thumb when he punches his
opponent.

You answered Smith's

The correct answer is Bennett's

83. A 63 year nurse falls on an extended and pronated wrist. An x-ray shows a distal radial
fracture with radiocarpal dislocation.

You answered Smith's

The correct answer is Barton's

Please rate this question:

Discuss and give feedback


Next question
Eponymous fractures

Colles' fracture (dinner fork deformity)

 Fall onto extended outstretched hand


 Classical Colles' fractures have the following 3 features:

1. Transverse fracture of the radius


2. 1 inch proximal to the radio-carpal joint
3. Dorsal displacement and angulation

Smith's fracture (reverse Colles' fracture)

 Volar angulation of distal radius fragment (Garden spade deformity)


 Caused by falling backwards onto the palm of an outstretched hand or falling with wrists
flexed

Bennett's fracture

 Intra-articular fracture of the first carpometacarpal joint


 Impact on flexed metacarpal, caused by fist fights
 X-ray: triangular fragment at ulnar base of metacarpal

Image sourced from Wikipedia


Monteggia's fracture

 Dislocation of the proximal radioulnar joint in association with an ulna fracture


 Fall on outstretched hand with forced pronation
 Needs prompt diagnosis to avoid disability

Image sourced from Wikipedia

Galeazzi fracture

 Radial shaft fracture with associated dislocation of the distal radioulnar joint
 Direct blow

Pott's fracture

 Bimalleolar ankle fracture


 Forced foot eversion

Barton's fracture
 Distal radius fracture (Colles'/Smith's) with associated radiocarpal dislocation
 Fall onto extended and pronated wrist
 Involvement of the joint is a defining feature

Next question
Theme: Fracture management

A. Application of external fixator


B. Open reduction and internal fixation
C. Fasciotomy
D. Skeletal traction

For the following upper limb injuries please select the most appropriate initial management. Each
option may be used once, more than once or not at all.

84. A 32 year old man falls from a ladder and sustains a fracture of his proximal radius. On
examination, he has severe pain in his forearm and diminished distal sensation. There is a
single puncture wound present at the fracture site.

You answered Application of external fixator

The correct answer is Fasciotomy

Theme from April 2012 Exam


Pain and neurological symptoms in a tight fascial compartment coupled with a high
velocity injury carry a high risk of compartment syndrome and prompt fasciotomy should
be performed.

85. A 32 year old man falls a sustains a fracture of his distal humerus. The fracture segment is
markedly angulated and unstable. There is a puncture site overlying the fracture site.

Application of external fixator

Wide exposure to plate the humerus is generally inadvisable owing to its many important
anatomical relations. Both intramedullary nailing and external fixation are reasonable
treatments. However, in the presence of an open fracture application of an external fixator
and appropriate tissue debridement would be most appropriate.

86. A 24 year old man sustains a distal radius fracture during a game of rugby. Imaging shows
a comminuted fracture with involvement of the articular surface.

You answered Application of external fixator

The correct answer is Open reduction and internal fixation

Meticulous anatomical alignment of the fracture segments is crucial to avoid the


development of osteoarthritis and risk of malunion.
Please rate this question:

Discuss and give feedback


Next question

Fracture management

 Bony injury resulting in a fracture may arise from trauma (excessive forces applied to bone),
stress related (repetitive low velocity injury) or pathological (abnormal bone which fractures
during normal use of following minimal trauma)
 Diagnosis involves not just evaluating the fracture ; such as site and type of injury but also
other associated injuries and distal neurovascular deficits. This may entail not just clinical
examination but radiographs of proximal and distal joints.
 When assessing x-rays it is important to assess for changes in length of the bone, the
angulation of the distal bone, rotational effects, presence of material such as glass.

Fracture types
Fracture type Description

Oblique fracture Fracture lies obliquely to long axis of bone

Comminuted fracture >2 fragments

Segmental fracture More than one fracture along a bone

Transverse fracture Perpendicular to long axis of bone

Spiral fracture Severe oblique fracture with rotation along long axis of bone

Open Vs Closed
It is also important to distinguish open from closed injuries. The most common classification system
for open fractures is the Gustilo and Anderson classification system (given below):

Grade Injury

1 Low energy wound <1cm


Grade Injury

2 Greater than 1cm wound with moderate soft tissue damage

3 High energy wound > 1cm with extensive soft tissue damage

3 A (sub group of 3) Adequate soft tissue coverage

3 B (sub group of 3) Inadequate soft tissue coverage

3 C (sub group of 3) Associated arterial injury

Key points in management of fractures

 Immobilise the fracture including the proximal and distal joints


 Carefully monitor and document neurovascular status, particularly following reduction and
immobilisation
 Manage infection including tetanus prophylaxis
 IV broad spectrum antibiotics for open injuries
 As a general principle all open fractures should be thoroughly debrided ( and internal fixation
devices avoided or used with extreme caution)
 Open fractures constitute an emergency and should be debrided and lavaged within 6 hours
of injury

Next question
Theme: Fracture management

A. Copious lavage and generous surgical debridement, followed by


external fixation
B. Intramedullary nail
C. Open reduction and internal fixation
D. Immobilisation in plaster cast
E. External fixation using a frame device
F. Amputation
G. Application of external fixation device
H. Primary closure of wound and application of plaster cast

Please select the most appropriate management for the fractures described. Each option may be
used once, more than once or not at all.

87. A 55 year old motorcyclist is involved in a road traffic accident and sustained a Gustilo
and Anderson IIIc type fracture to the distal tibia. He was trapped in the wreckage for 7
hours during which time he bled profusely from the fracture site. He has an established
distal neurovascular deficit.

You answered Copious lavage and generous surgical debridement, followed by


external fixation

The correct answer is Amputation

This man is unstable, and at 7 hours after extraction, the limb is not viable. The safest
option is primary amputation.

88. A 25 year old ski instructor who falls off a ski lift and sustains a spiral fracture of the mid
shaft of the tibia. Attempts to achieve satisfactory position in plaster have failed.
Overlying tissues are healthy.

You answered Copious lavage and generous surgical debridement, followed by


external fixation

The correct answer is Intramedullary nail

This would be a good case for intramedullary nailing. Open reduction and external fixation
would strip off otherwise healthy tissues and hence is unsuitable. In some units the injury
may be managed with an Ilizarov frame device but the majority would treat with IM
nailing.

89. A 35 year old mechanic is hit by a fork lift truck. He sustains a Gustilo and Anderson type
IIIA fracture of the shaft of the left femur.
Copious lavage and generous surgical debridement, followed by external fixation

At the tissues are in better shape than in the first case and as there is no associated vascular
injury the patient may be suitable for debridement of the area and external fixation. If
debridement leaves a tissue defect then plastic surgical repair will be needed at a later
stage.
Delayed treatment of open fractures with significant vascular injury may be best treated by primary
amputation.

Please rate this question:

Discuss and give feedback

Fracture management

 Bony injury resulting in a fracture may arise from trauma (excessive forces applied to bone),
stress related (repetitive low velocity injury) or pathological (abnormal bone which fractures
during normal use of following minimal trauma)
 Diagnosis involves not just evaluating the fracture ; such as site and type of injury but also
other associated injuries and distal neurovascular deficits. This may entail not just clinical
examination but radiographs of proximal and distal joints.
 When assessing x-rays it is important to assess for changes in length of the bone, the
angulation of the distal bone, rotational effects, presence of material such as glass.

Fracture types
Fracture type Description

Oblique fracture Fracture lies obliquely to long axis of bone

Comminuted fracture >2 fragments

Segmental fracture More than one fracture along a bone

Transverse fracture Perpendicular to long axis of bone

Spiral fracture Severe oblique fracture with rotation along long axis of bone
Open Vs Closed
It is also important to distinguish open from closed injuries. The most common classification system
for open fractures is the Gustilo and Anderson classification system (given below):

Grade Injury

1 Low energy wound <1cm

2 Greater than 1cm wound with moderate soft tissue damage

3 High energy wound > 1cm with extensive soft tissue damage

3 A (sub group of 3) Adequate soft tissue coverage

3 B (sub group of 3) Inadequate soft tissue coverage

3 C (sub group of 3) Associated arterial injury

Key points in management of fractures

 Immobilise the fracture including the proximal and distal joints


 Carefully monitor and document neurovascular status, particularly following reduction and
immobilisation
 Manage infection including tetanus prophylaxis
 IV broad spectrum antibiotics for open injuries
 As a general principle all open fractures should be thoroughly debrided ( and internal fixation
devices avoided or used with extreme caution)
 Open fractures constitute an emergency and should be debrided and lavaged within 6 hours
of injury
Theme: Paediatric gastrointestinal disorders

A. Liver transplant
B. Rectal biopsy
C. Roux-en-Y portojejunostomy
D. Ramstedt pyloromyotomy
E. Abdominal CT scan
F. Upper GI contrast study
G. Laparotomy and formation of stoma
H. Laparotomy
I. Colonoscopy
J. Gastroscopy

What is the best management option for these children? Each option may be used once, more than
once or not at all.

1. A male infant, born at term appears well following delivery. Six hours later, he is noted to
have bilious vomiting by the paediatricians. On examination he seems well and his
abdomen is soft and non tender.

You answered Liver transplant

The correct answer is Upper GI contrast study

Bilious vomiting in neonates is a surgical emergency and is intestinal malrotation and


volvulus until otherwise proven. It is investigated with an upper GI contrast study. Contrast
should be seen to exit the stomach and the location of the DJ flexure is noted (it lies to the
left of the midline). If this is not the case, or the study is inconclusive, a laparotomy is
performed.

2. A 2 month old baby presents with jaundice. He has an elevated conjugated bilirubin level.
Diagnosis is confirmed by cholangiography during surgery.

You answered Liver transplant

The correct answer is Roux-en-Y portojejunostomy

This child has biliary atresia. The aim is to avoid liver transplantation (however, most will
come to transplant in time).

3. A male infant, born at term by normal vaginal delivery is well. However, 72 hours
following delivery, he has still not passed meconium. On examination, his abdomen is soft
and not particularly distended. He has a normally sited anus.
You answered Liver transplant

The correct answer is Rectal biopsy

Delayed passage of meconium is suggestive of Hirschsprung's disease and the investigation


of choice is full thickness suction rectal biopsy. If Hirschprungs is confirmed, then the
correct treatment is laparotomy and stoma formation. At between 9 and 12 months of age,
definitive surgery (usually resection and primary anastomosis) is performed.

Please rate this question:

Discuss and give feedback


Next question

Paediatric Gastrointestinal disorders

Pyloric stenosis  M>F


 5-10% Family history in parents
 Projectile non bile stained vomiting at 4-6 weeks of life
 Diagnosis is made by test feed or USS
 Treatment: Ramstedt pyloromyotomy (open or laparoscopic)

Acute appendicitis  Uncommon under 3 years


 When occurs may present atypically

Mesenteric  Central abdominal pain and URTI


adenitis  Conservative management

Intussusception  Telescoping bowel


 Proximal to or at the level of, ileocaecal valve
 6-9 months age
 Colicky pain, diarrhoea and vomiting, sausage shaped mass, red jelly
stool.
 Treatment: reduction with air insufflation

Malrotation  High caecum at the midline


 Feature in exomphalos, congenital diaphragmatic hernia, intrinsic
duodenal atresia
 May be complicated by development of volvulus, infant with
volvulus may have bile stained vomiting
 Diagnosis is made by upper GI contrast study and USS
 Treatment is by laparotomy, if volvulus is present (or at high risk of
occurring then a ladds procedure is performed

Hirschsprung's  Absence of ganglion cells from myenteric and submucosal plexuses


disease  Occurs in 1/5000 births
 Full thickness rectal biopsy for diagnosis
 Delayed passage of meconium and abdominal distension
 Treatment is with rectal washouts initially, thereafter an anorectal
pull through procedure

Oesophageal  Associated with tracheo-oesophageal fistula and polyhydramnios


atresia  May present with choking and cyanotic spells following aspiration
 VACTERL associations

Meconium ileus  Usually delayed passage of meconium and abdominal distension


 Majority have cystic fibrosis
 X-Rays may not show a fluid level as the meconium is viscid
(depends upon feeding), PR contrast studies may dislodge meconium
plugs and be therapeutic
 Infants who do not respond to PR contrast and NG N-acetyl cysteine
will require surgery to remove the plugs

Biliary atresia  Jaundice > 14 days


 Increased conjugated bilirubin
 Urgent Kasai procedure

Necrotising  Prematurity is the main risk factor


enterocolitis  Early features include abdominal distension and passage of bloody
stools
 X-Rays may show pneumatosis intestinalis and evidence of free air
 Increased risk when empirical antibiotics are given to infants beyond
5 days
 Treatment is with total gut rest and TPN, babies with perforations
will require laparotomy

Next question
What is the most common abdominal emergency in children under 1 year of age?

Appendicitis

Intussusception

Intestinal malrotation

Pyloric stenosis

Mid gut volvulus

The commonest emergency in this age group is inguinal hernia followed by intussusception.
Appendicitis is commoner in those older than 1 year of age.
Please rate this question:

Discuss and give feedback


Next question

Paediatric Gastrointestinal disorders

Pyloric stenosis  M>F


 5-10% Family history in parents
 Projectile non bile stained vomiting at 4-6 weeks of life
 Diagnosis is made by test feed or USS
 Treatment: Ramstedt pyloromyotomy (open or laparoscopic)

Acute appendicitis  Uncommon under 3 years


 When occurs may present atypically

Mesenteric  Central abdominal pain and URTI


adenitis  Conservative management

Intussusception  Telescoping bowel


 Proximal to or at the level of, ileocaecal valve
 6-9 months age
 Colicky pain, diarrhoea and vomiting, sausage shaped mass, red jelly
stool.
 Treatment: reduction with air insufflation

Malrotation  High caecum at the midline


 Feature in exomphalos, congenital diaphragmatic hernia, intrinsic
duodenal atresia
 May be complicated by development of volvulus, infant with
volvulus may have bile stained vomiting
 Diagnosis is made by upper GI contrast study and USS
 Treatment is by laparotomy, if volvulus is present (or at high risk of
occurring then a ladds procedure is performed

Hirschsprung's  Absence of ganglion cells from myenteric and submucosal plexuses


disease  Occurs in 1/5000 births
 Full thickness rectal biopsy for diagnosis
 Delayed passage of meconium and abdominal distension
 Treatment is with rectal washouts initially, thereafter an anorectal
pull through procedure

Oesophageal  Associated with tracheo-oesophageal fistula and polyhydramnios


atresia  May present with choking and cyanotic spells following aspiration
 VACTERL associations

Meconium ileus  Usually delayed passage of meconium and abdominal distension


 Majority have cystic fibrosis
 X-Rays may not show a fluid level as the meconium is viscid
(depends upon feeding), PR contrast studies may dislodge meconium
plugs and be therapeutic
 Infants who do not respond to PR contrast and NG N-acetyl cysteine
will require surgery to remove the plugs

Biliary atresia  Jaundice > 14 days


 Increased conjugated bilirubin
 Urgent Kasai procedure

Necrotising  Prematurity is the main risk factor


enterocolitis  Early features include abdominal distension and passage of bloody
stools
 X-Rays may show pneumatosis intestinalis and evidence of free air
 Increased risk when empirical antibiotics are given to infants beyond
5 days
 Treatment is with total gut rest and TPN, babies with perforations
will require laparotomy

Next question
Theme: Paediatric gastrointestinal disorders

A. Duodenal atresia
B. Hypertrophy of the pyloric sphincter
C. Budd Chiari Syndrome
D. Intussceception
E. Oesophageal atresia
F. Congenital diaphragmatic hernia
G. Cystic fibrosis
H. Intestinal malrotation
I. Gastroenteritis

Please select the most likely diagnosis for the scenario given. Each option may be used once, more
than once or not at all.

5. A two week old child is brought to the emergency department by his parents. He was slow
to establish on feeds but was discharged home three days following delivery. During the
past 7 hours he has been vomiting and the vomit is largely bile stained. On examination he
has a soft, scaphoid abdomen.

You answered Duodenal atresia

The correct answer is Intestinal malrotation

Theme from April 2013 Exam


The combination of scaphoid abdomen and bilious vomiting is highly suggestive of
intestinal malrotation and volvulus. An urgent upper GI contrast study and ultrasound is
required.

6. A 4 week old baby is developing well and develops profuse and projectile vomiting after
feeds. He has been losing weight and the vomit is described as being non bilious.

You answered Duodenal atresia

The correct answer is Hypertrophy of the pyloric sphincter

A history of projective vomiting and weight loss is a common story suggestive of pyloric
stenosis. The vomit is often not bile stained. Diagnosis is further suggested by
hypochloraemic metabolic alkalosis and a palpable tumour on test feeding.

7. A 1 day old child is born by emergency cesarean section for foetal distress. On examination
he has decreased air entry on the left side of his chest and a displaced apex beat. Abdominal
examination demonstrates a scaphoid abdomen but is otherwise unremarkable.
You answered Duodenal atresia

The correct answer is Congenital diaphragmatic hernia

Displaced apex beat and decreased air entry are suggestive of diaphragmatic hernia. The
abdomen may well be scaphoid in some cases. The underlying lung may be hypoplastic and
this correlates directly with prognosis.

Please rate this question:

Discuss and give feedback


Next question

Paediatric Gastrointestinal disorders

Pyloric stenosis  M>F


 5-10% Family history in parents
 Projectile non bile stained vomiting at 4-6 weeks of life
 Diagnosis is made by test feed or USS
 Treatment: Ramstedt pyloromyotomy (open or laparoscopic)

Acute appendicitis  Uncommon under 3 years


 When occurs may present atypically

Mesenteric  Central abdominal pain and URTI


adenitis  Conservative management

Intussusception  Telescoping bowel


 Proximal to or at the level of, ileocaecal valve
 6-9 months age
 Colicky pain, diarrhoea and vomiting, sausage shaped mass, red jelly
stool.
 Treatment: reduction with air insufflation

Malrotation  High caecum at the midline


 Feature in exomphalos, congenital diaphragmatic hernia, intrinsic
duodenal atresia
 May be complicated by development of volvulus, infant with
volvulus may have bile stained vomiting
 Diagnosis is made by upper GI contrast study and USS
 Treatment is by laparotomy, if volvulus is present (or at high risk of
occurring then a ladds procedure is performed

Hirschsprung's  Absence of ganglion cells from myenteric and submucosal plexuses


disease  Occurs in 1/5000 births
 Full thickness rectal biopsy for diagnosis
 Delayed passage of meconium and abdominal distension
 Treatment is with rectal washouts initially, thereafter an anorectal
pull through procedure

Oesophageal  Associated with tracheo-oesophageal fistula and polyhydramnios


atresia  May present with choking and cyanotic spells following aspiration
 VACTERL associations

Meconium ileus  Usually delayed passage of meconium and abdominal distension


 Majority have cystic fibrosis
 X-Rays may not show a fluid level as the meconium is viscid
(depends upon feeding), PR contrast studies may dislodge meconium
plugs and be therapeutic
 Infants who do not respond to PR contrast and NG N-acetyl cysteine
will require surgery to remove the plugs

Biliary atresia  Jaundice > 14 days


 Increased conjugated bilirubin
 Urgent Kasai procedure

Necrotising  Prematurity is the main risk factor


enterocolitis  Early features include abdominal distension and passage of bloody
stools
 X-Rays may show pneumatosis intestinalis and evidence of free air
 Increased risk when empirical antibiotics are given to infants beyond
5 days
 Treatment is with total gut rest and TPN, babies with perforations
will require laparotomy

Next question
A 1-day-old baby girl is noted to become profoundly cyanotic whilst feeding and crying. A diagnosis
of congenital heart disease is suspected. What is the most likely cause?

Transposition of the great arteries

Coarctation of the aorta

Patent ductus arteriosus

Hypoplastic left heart

Ventricular septal defect

Congenital heart disease

 Cyanotic: TGA most common at birth, Fallot's most common overall


 Acyanotic: VSD most common cause

Theme from April 2011 exam


It is important to be aware of common congenital cardiac abnormalities. The main differentiating
factor is whether the patient is cyanotic or acyanotic. In the neonate, TGA is the most common
condition presenting with profound cyanosis.

The other options are causes of acyanotic congenital heart disease


Please rate this question:

Discuss and give feedback


Next question

Congenital heart disease

Acyanotic - most common causes

 Ventricular septal defects (VSD) - most common, accounts for 30%


 Atrial septal defect (ASD)
 Patent ductus arteriosus (PDA)
 Coarctation of the aorta
 Aortic valve stenosis
VSDs are more common than ASDs. However, in adult patients ASDs are the more common new
diagnosis as they generally present later.

Cyanotic - most common causes

 Tetralogy of Fallot
 Transposition of the great arteries (TGA)
 Tricuspid atresia
 Pulmonary valve stenosis

Next question
A 6 month old boy is brought to the clinic by his mother. She is concerned that his testes are not
located into the scrotum. She has noticed them only when he is in the bath, but not at any other
time. What is the most likely underlying diagnosis?

Retractile testis

Ectopic testis

Undescended testis

Testicular agenesis

Intersex child

Theme from April 2012 Exam


A testis that appears in warm conditions or which can be brought down on clinical examination and
does not immediately retract is usually a retractile testis.
Please rate this question:

Discuss and give feedback


Next question

Cryptorchidism

A congenital undescended testis is one that has failed to reach the bottom of the scrotum by 3
months of age. At birth up to 5% of boys will have an undescended testis, post natal descent occurs
in most and by 3 months the incidence of cryptorchidism falls to 1-2%. In the vast majority of cases
the cause of the maldescent is unknown. A proportion may be associated with other congenital
defects including:

Patent processus vaginalis


Abnormal epididymis
Cerebral palsy
Mental retardation
Wilms tumour
Abdominal wall defects (e.g. gastroschisis, prune belly syndrome)

Differential diagnosis
These include retractile testes and, in the case of absent bilateral testes the possibility of intersex
conditions. A retractile testis can be brought into the scrotum by the clinician and when released
remains in the scrotum. If the examining clinician notes the testis to return rapidly into the inguinal
canal when released then surgery is probably indicated.
Reasons for correction of cryptorchidism

 Reduce risk of infertility


 Allows the testes to be examined for testicular cancer
 Avoid testicular torsion
 Cosmetic appearance

Males with undescended testis are 40 times as likely to develop testicular cancer (seminoma) as
males without undescended testis
The location of the undescended testis affects the relative risk of testicular cancer (50% intra-
abdominal testes)

Treatment

 Orchidopexy at 6- 18 months of age. The operation usually consists of inguinal exploration,


mobilisation of the testis and implantation into a dartos pouch.
 Intra-abdominal testis should be evaluated laparoscopically and mobilised. Whether this is a
single stage or two stage procedure depends upon the exact location.
 After the age of 2 years in untreated individuals the Sertoli cells will degrade and those
presenting late in teenage years may be better served by orchidectomy than to try and
salvage a non functioning testis with an increased risk of malignancy.

Next question
Which of the following statements relating to omphalocele is false?

The herniated organs lie outside the peritoneal sac

Cardiac abnormalities co-exist in 25%

The caecum is usually right sided

The defect occurs through the umbilicus

Mortality may be as high as 15%

Gastroschisis: Isolated abnormality, bowel lies outside abdominal wall through defect located to right
of umbilicus.
Exomphalos: Liver and gut remain covered with membranous sac connected to umbilical cord. It is
associated with other developmental defects.

They are contained within the peritoneal sac and therefore do not have the fluid losses seen in
gastroschisis. True malrotation is unusual and minor variants may not result in a requirement for
surgery.
Please rate this question:

Discuss and give feedback


Next question

Paediatric Gastrointestinal disorders

Pyloric stenosis  M>F


 5-10% Family history in parents
 Projectile non bile stained vomiting at 4-6 weeks of life
 Diagnosis is made by test feed or USS
 Treatment: Ramstedt pyloromyotomy (open or laparoscopic)

Acute appendicitis  Uncommon under 3 years


 When occurs may present atypically

Mesenteric  Central abdominal pain and URTI


adenitis
 Conservative management

Intussusception  Telescoping bowel


 Proximal to or at the level of, ileocaecal valve
 6-9 months age
 Colicky pain, diarrhoea and vomiting, sausage shaped mass, red jelly
stool.
 Treatment: reduction with air insufflation

Malrotation  High caecum at the midline


 Feature in exomphalos, congenital diaphragmatic hernia, intrinsic
duodenal atresia
 May be complicated by development of volvulus, infant with
volvulus may have bile stained vomiting
 Diagnosis is made by upper GI contrast study and USS
 Treatment is by laparotomy, if volvulus is present (or at high risk of
occurring then a ladds procedure is performed

Hirschsprung's  Absence of ganglion cells from myenteric and submucosal plexuses


disease  Occurs in 1/5000 births
 Full thickness rectal biopsy for diagnosis
 Delayed passage of meconium and abdominal distension
 Treatment is with rectal washouts initially, thereafter an anorectal
pull through procedure

Oesophageal  Associated with tracheo-oesophageal fistula and polyhydramnios


atresia  May present with choking and cyanotic spells following aspiration
 VACTERL associations

Meconium ileus  Usually delayed passage of meconium and abdominal distension


 Majority have cystic fibrosis
 X-Rays may not show a fluid level as the meconium is viscid
(depends upon feeding), PR contrast studies may dislodge meconium
plugs and be therapeutic
 Infants who do not respond to PR contrast and NG N-acetyl cysteine
will require surgery to remove the plugs

Biliary atresia  Jaundice > 14 days


 Increased conjugated bilirubin
 Urgent Kasai procedure

Necrotising  Prematurity is the main risk factor


enterocolitis  Early features include abdominal distension and passage of bloody
stools
 X-Rays may show pneumatosis intestinalis and evidence of free air
 Increased risk when empirical antibiotics are given to infants beyond
5 days
 Treatment is with total gut rest and TPN, babies with perforations
will require laparotomy

Next question
Theme: Bilious vomiting in neonates

A. Biliary atresia

B. Intestinal malrotation

C. Ileal atresia

D. Necrotising enterocolitis

E. Duodenal atresia

F. Meconium ileus

G. Viral gastroenteritis

H. Pyloric stenosis

Please select the most likely underlying cause of bilious vomiting for the situation described. Each
option may be used once, more than once or not at all.

11. A male infant is born prematurely at 26 weeks gestation by emergency cesarean section.
Following the birth he develops respiratory distress syndrome and is ventilated. He begins to
improve twelve days after birth. Then he becomes unwell and develops abdominal distension and
passes bloody stools and vomits a small quantity of bile stained vomit.

You answered Biliary atresia

The correct answer is Necrotising enterocolitis

Necrotising enterocolitis often has a delayed presentation and affected infants will typically pass
bloody stools. Plain films may show air in the intestinal wall (Pneumatosis).

12. A male infant is born by spontaneous vaginal delivery at 39 weeks gestation. He is well after the
birth, established on bottle feeding and discharged home. His parents are concerned because he
subsequently becomes unwell and vomits a large quantity of bile stained vomit approximately 2
days after discharge home. On examination he looks ill and his abdomen is soft and non
distended.
You answered Biliary atresia

The correct answer is Intestinal malrotation

Intestinal malrotation with volvulus will typically compromise the vascularisation and lumenal
patency of the gut. This will cause bilious vomiting and the vascular insufficiency will produce a
clinical picture of illness at odds with the lack of overt abdominal signs. Delay in diagnosis and
surgery will result in established necrosis, perforation and peritonitis.

13. A female infant is born by cesarean section at 38 weeks gestation for foetal distress. The
attending paediatricians notice that she has a single palmar crease and a mongoloid slant to her
eyes. Soon after the birth the mother tries to feed the child who has a projectile vomit about 10
minutes after feeding. On examination she has a soft, non distended abdomen.

You answered Biliary atresia

The correct answer is Duodenal atresia

Proximally sited atresia will produce high volume vomits which may or may not be bile stained.
Abdominal distension is characteristically absent. Whilst under resuscitated children may be a
little dehydrated they are seldom severely ill. The presence of Trisomy 21 (palmar and eye signs)
increases the likelihood of duodenal atresia.

Please rate this question:

Discuss and give feedback

Next question

Bilious vomiting in neonates

Causes of intestinal obstruction with bilious vomiting in neonates

Disorder Incidence and Age at Diagnosis Treatment


causation presentation

Duodenal 1 in 5000 (higher Few hours after AXR shows Duodenoduodenostomy


atresia in Downs birth "double bubble
syndrome) sign, contrast
study may
confirm

Malrotation Usually cause by Usually 3-7 days Upper GI Ladd's procedure


with incomplete after birth, contrast study
volvulus rotation during volvulus with may show DJ
embryogenesis compromised flexure is more
circulation may medially placed,
result in peritoneal USS may show
signs and abnormal
haemodynamic orientation of
instability SMA and SMV

Jejunal/ ileal Usually caused by Usually within 24 AXR will show Laparotomy with primary
atresia vascular hours of birth air-fluid levels resection and anastomosis
insufficiency in
utero, usually 1
in 3000

Meconium Occurs in Typically in first 24- Air - fluid levels Surgical decompression,
ileus between 15 48 hours of life on AXR, sweat serosal damage may require
and20% of those with abdominal test to confirm segmental resection
babies with cystic distension and cystic fibrosis
fibrosis, bilious vomiting
otherwise 1 in
5000

Necrotising Up to 2.4 per Usually second Dilated bowel Conservative and


enterocolitis 1000 births, risks week of life loops on AXR, supportive for non
increased in pneumatosis perforated cases,
prematurity and and portal laparotomy and resection in
inter-current cases of perforation of
illness venous air ongoing clinical
deterioration

Next question
Theme: Paediatric gastrointestinal disorders

A. Meconium ileus
B. Biliary atresia
C. Oesophageal atresia
D. Pyloric stenosis
E. Intussusception
F. Malrotation
G. Hirschsprung disease
H. Mesenteric adenitis.

What is the most likely diagnosis for each scenario given? Each option may be used once, more
than once or not at all.

14. A 3 day old baby presents with recurrent episodes of choking and cyanotic episodes. There
is a history of polyhydramnios.

You answered Meconium ileus

The correct answer is Oesophageal atresia

Diagnosis is confirmed when an nasogastric tube fails to reach the stomach.

15. A 3 day old neonate is developing increasing problems with feeding. On examination she
has a pan systolic murmur and her forearms have not developed properly.

You answered Meconium ileus

The correct answer is Oesophageal atresia

This child has VACTERL, which is a combination of Vertebral, Ano-rectal, Cardiac,


Tracheo-oesophageal, Renal and Radial limb anomalies. Half of babies with oesophageal
atresia will have VACTERL.

16. A 2 year old child has central abdominal pain. He has had a recent upper respiratory tract
infection.

You answered Meconium ileus

The correct answer is Mesenteric adenitis.

Mesenteric adenitis may complicate upper respiratory tract infection and clinical exclusion
of appendicitis can be difficult.
Please rate this question:

Discuss and give feedback


Next question

Paediatric Gastrointestinal disorders

Pyloric stenosis  M>F


 5-10% Family history in parents
 Projectile non bile stained vomiting at 4-6 weeks of life
 Diagnosis is made by test feed or USS
 Treatment: Ramstedt pyloromyotomy (open or laparoscopic)

Acute appendicitis  Uncommon under 3 years


 When occurs may present atypically

Mesenteric  Central abdominal pain and URTI


adenitis  Conservative management

Intussusception  Telescoping bowel


 Proximal to or at the level of, ileocaecal valve
 6-9 months age
 Colicky pain, diarrhoea and vomiting, sausage shaped mass, red jelly
stool.
 Treatment: reduction with air insufflation

Malrotation  High caecum at the midline


 Feature in exomphalos, congenital diaphragmatic hernia, intrinsic
duodenal atresia
 May be complicated by development of volvulus, infant with
volvulus may have bile stained vomiting
 Diagnosis is made by upper GI contrast study and USS
 Treatment is by laparotomy, if volvulus is present (or at high risk of
occurring then a ladds procedure is performed

Hirschsprung's  Absence of ganglion cells from myenteric and submucosal plexuses


disease  Occurs in 1/5000 births
 Full thickness rectal biopsy for diagnosis
 Delayed passage of meconium and abdominal distension
 Treatment is with rectal washouts initially, thereafter an anorectal
pull through procedure

Oesophageal  Associated with tracheo-oesophageal fistula and polyhydramnios


atresia  May present with choking and cyanotic spells following aspiration
 VACTERL associations

Meconium ileus  Usually delayed passage of meconium and abdominal distension


 Majority have cystic fibrosis
 X-Rays may not show a fluid level as the meconium is viscid
(depends upon feeding), PR contrast studies may dislodge meconium
plugs and be therapeutic
 Infants who do not respond to PR contrast and NG N-acetyl cysteine
will require surgery to remove the plugs

Biliary atresia  Jaundice > 14 days


 Increased conjugated bilirubin
 Urgent Kasai procedure

Necrotising  Prematurity is the main risk factor


enterocolitis  Early features include abdominal distension and passage of bloody
stools
 X-Rays may show pneumatosis intestinalis and evidence of free air
 Increased risk when empirical antibiotics are given to infants beyond
5 days
 Treatment is with total gut rest and TPN, babies with perforations
will require laparotomy

Next question
Theme: Neonatal gastrointestinal disease

A. Ano-rectal atresia
B. Pyloric stenosis
C. Hirschsprungs disease
D. Duodenal atresia
E. Meconium ileus
F. Intussusception
G. Necrotising enterocolitis
H. Intestinal volvulus
I. Tracheo-oesophageal fistula

Please select the most likely diagnosis to account for the case described. Each option may be used
once, more than once or not at all.

17. A newborn baby boy presents with mild abdominal distension and failure to pass
meconium after 24 hours. X- Ray reveals dilated loops of bowel with fluid levels. The
anus appears normally located.

You answered Ano-rectal atresia

The correct answer is Hirschsprungs disease

Hirschsprung's disease is an absence of ganglion cells in the neural plexus of the intestinal
wall. It is more common in boys than girls. The delayed passage of meconium together
with distension of abdomen is the usual clinical presentation. A plain abdominal x ray will
demonstrate dilated loops of bowel with fluid levels and a barium enema can be helpful
when it demonstrates a cone with dilated ganglionic proximal colon and the distal
aganglionic bowel failing to distend.

18. A premature infant (30-week gestation) presents with distended and tense abdomen. She is
passing blood and mucus per rectum, and she is also manifesting signs of sepsis.

You answered Ano-rectal atresia

The correct answer is Necrotising enterocolitis

Necrotising enterocolitis is more common in premature infants. Mesenteric ischemia


causes bacterial invasion of the mucosa leading to sepsis. Terminal ileum, caecum and the
distal colon are commonly affected. The abdomen is distended and tense, and the infant
passes blood and mucus per rectum. X -Ray of the abdomen shows distended loops of
intestine and gas bubbles may be seen in the bowel wall.

19. A newborn baby boy presents with gross abdominal distension. He is diagnosed with
cystic fibrosis and his abdominal x ray shows distended coils of small bowel, but no fluid
levels.

You answered Ano-rectal atresia

The correct answer is Meconium ileus

One in 15,000 newborns will have a distal small bowel obstruction secondary to abnormal
bulky and viscid meconium. Ninety percent of these infants will have cystic fibrosis and
the abnormal meconium is the result of deficient intestinal secretions. This condition
presents during the first days of life with gross abdominal distension and bilious vomiting.
x Ray of the abdomen shows distended coils of bowel and typical mottled ground glass
appearance. Fluid levels are scarce as the meconium is viscid.

Please rate this question:

Discuss and give feedback


Next question

Paediatric Gastrointestinal disorders

Pyloric stenosis  M>F


 5-10% Family history in parents
 Projectile non bile stained vomiting at 4-6 weeks of life
 Diagnosis is made by test feed or USS
 Treatment: Ramstedt pyloromyotomy (open or laparoscopic)

Acute appendicitis  Uncommon under 3 years


 When occurs may present atypically

Mesenteric  Central abdominal pain and URTI


adenitis  Conservative management

Intussusception  Telescoping bowel


 Proximal to or at the level of, ileocaecal valve
 6-9 months age
 Colicky pain, diarrhoea and vomiting, sausage shaped mass, red jelly
stool.
 Treatment: reduction with air insufflation
Malrotation  High caecum at the midline
 Feature in exomphalos, congenital diaphragmatic hernia, intrinsic
duodenal atresia
 May be complicated by development of volvulus, infant with
volvulus may have bile stained vomiting
 Diagnosis is made by upper GI contrast study and USS
 Treatment is by laparotomy, if volvulus is present (or at high risk of
occurring then a ladds procedure is performed

Hirschsprung's  Absence of ganglion cells from myenteric and submucosal plexuses


disease  Occurs in 1/5000 births
 Full thickness rectal biopsy for diagnosis
 Delayed passage of meconium and abdominal distension
 Treatment is with rectal washouts initially, thereafter an anorectal
pull through procedure

Oesophageal  Associated with tracheo-oesophageal fistula and polyhydramnios


atresia  May present with choking and cyanotic spells following aspiration
 VACTERL associations

Meconium ileus  Usually delayed passage of meconium and abdominal distension


 Majority have cystic fibrosis
 X-Rays may not show a fluid level as the meconium is viscid
(depends upon feeding), PR contrast studies may dislodge meconium
plugs and be therapeutic
 Infants who do not respond to PR contrast and NG N-acetyl cysteine
will require surgery to remove the plugs

Biliary atresia  Jaundice > 14 days


 Increased conjugated bilirubin
 Urgent Kasai procedure

Necrotising  Prematurity is the main risk factor


enterocolitis  Early features include abdominal distension and passage of bloody
stools
 X-Rays may show pneumatosis intestinalis and evidence of free air
 Increased risk when empirical antibiotics are given to infants beyond
5 days
 Treatment is with total gut rest and TPN, babies with perforations
will require laparotomy
Which of the following statements relating to biliary atresia is untrue?

It most commonly presents as prolonged conjugated jaundice in the neonatal period.

Evidence of portal hypertension at diagnosis is seldom present in the UK.

It may be confused with Alagille syndrome.

The Kasai procedure is best performed in the first 8 weeks of life.

Survival following a successful Kasai procedure is approximately 45% at 5 years.

Alagille syndrome autosomal dominant disorder characterised by presence of paucity of bile ducts
and cardiac defects. Only the embryonic form of biliary atresia is associated with cardiac and other
embryological defects.

Biliary atresia usually presents with obstructed jaundice. A Kasai procedure is best performed in the
first 8 weeks of life. If a Kasai procedure is successful most patients will not require liver
transplantation. 45% of patients post Kasai procedure will require transplantation. However, overall
survival following a successful Kasai procedure is 80%.
Please rate this question:

Discuss and give feedback


Next question

Biliary atresia

 1 in 17000 affected
 Biliary tree lumen is obliterated by an inflammatory cholangiopathy causing progressive liver
damage

Clinical features

 Infant well in 1st few weeks of life


 No family history of liver disease
 Jaundice in infants > 14 days in term infants (>21 days in pre term infants)
 Pale stool, yellow urine (colourless in babies)
 Associated with cardiac malformations, polysplenia, situs inversus
Investigation

 Conjugated bilirubin (prolonged physiological jaundice or breast milk jaundice will cause a
rise in unconjugated bilirubin, whereas those with obstructive liver disease will have a rise in
conjugated bilirubin)
 Ultrasound of the liver (excludes extrahepatic causes, in biliary atresia infant may have tiny
or invisible gallbladder)
 Hepato-iminodiacetic acid radionuclide scan (good uptake but no excretion usually seen)

Management

 Early recognition is important to prevent liver transplantation.


 Nutritional support.
 Roux-en-Y portojejunostomy (Kasai procedure).
 If Kasai procedure fails or late recognition, a liver transplant becomes the only option.

Next question
Theme: Administration of intravenous fluids

A. 0.9% Saline
B. 5% Dextrose
C. 20% Glucose
D. 0.18% saline/ 4% glucose
E. 0.45% saline/ 15% glucose
F. 0.45% saline/ 2.5% glucose
G. 4.5% albumin
H. 10% Pentastarch
I. 10% Dextrose

For the scenario given please select the most appropriate type of intravenous fluid for the scenario
given. Each option may be used once, more than once or not at all.

21. A 4 year old boy is undergoing an elective orchidopexy.

0.9% Saline

Isotonic fluids should be used in this setting and 0.9% saline is the safest option.

22. A 2 day old boy is recovering from an inguinal herniotomy he has yet to feed and the
nursing staff would like a prescription for an initial fluid to be given on return to the ward.
His potassium is within normal limits.

You answered 0.9% Saline

The correct answer is 10% Dextrose

Neonates require 10% dextrose solutions as they are at risk of developing hypoglycaemia.

23. A 4 year boy with learning difficulties has developed swallowing problems and is awaiting
a PEG tube. He required maintenance IV fluids and the nursing staff require choice of
fluid for the next bag. He has just been given 250ml of 0.9% saline.

You answered 0.9% Saline

The correct answer is 5% Dextrose

5% Dextrose would the routine choice for water replacement.

Please rate this question:


Discuss and give feedback
Next question

Paediatric fluid management

Since 2000 there have been at least 4 reported deaths from fluid induced hyponatraemia in children.
This led to the National Patient Safety Agency introducing revised guidelines in 2007.
Indications for IV fluids include:

 Resuscitation and circulatory support


 Replacing on-going fluid losses
 Maintenance fluids for children for whom oral fluids are not appropriate
 Correction of electrolyte disturbances

Fluids to be avoided
Outside the neonatal period saline / glucose solutions should not be given. The greatest risk is with
saline 0.18 / glucose 4% solutions. The report states that 0.45% saline / 5% glucose may be used.
But preference should be given to isotonic solutions and few indications exist for this solution either.

Fluids to be used

 0.9% saline
 5% glucose (though only with saline for maintenance and not to replace losses)
 Hartmann's solution

Potassium should be added to maintenance fluids according patients plasma potassium levels
(which should be monitored).

Intraoperative fluid management


Neonates should receive glucose 10% during surgery.
Other children should receive isotonic crystalloid.

Maintenance fluids
Weight Water requirement/kg/day Na mmol/kg/day K mmol/kg/day

First 10Kg body weight 100ml 2-4 1.5-2.5

Second 10Kg body weight 50ml 1-2 0.5-1.5

Subsequent Kg 20ml 0.5-1.0 0.2-0.7


Glucose will need to be given to neonates- usually 10% at a rate of 60ml/Kg/day.

Reference
NPSA -reducing risk of hyponatraemia when administering intravenous fluids to children. Issue date
March 2007. Further references included in this document.
Next question
A 3 day old baby develops dyspneoa. A chest x-ray is performed and shows a radio-opaque shadow
with an air-fluid level in the chest. It is located immediately anterior to the 6th hemivertebra. Which of
the following is the most likely underlying diagnosis?

Bronchogenic cyst

Congenital diaphragmatic hernia

Infection with Staphylococcus aureus

Oesphageal duplication cyst

Hiatus hernia

A midline cystic mass of an infant in this age group is most likely to be a bronchogenic cyst. Hiatus
hernia is unusual in the neonatal period. Oesophageal duplication cysts are very rare and respiratory
symptoms are less common than with bronchogenic cysts. Midline congenital hernias are rare and
would usually not include an air/ fluid level in the rare event that they occur at this site.

Please rate this question:

Discuss and give feedback

Next question

Bronchogenic cysts

Overview
Bronchogenic cysts most commonly arise as a result of anomalous development of the ventral
foregut. They are most commonly single, although multiple cysts are described.

They often lie near the midline and most frequently occur in the region of the carina. They may be
attached to the tracheobronchial tree, although they are seldom in direct connection with it.

Cases may be asymptomatic or present with respiratory symptoms early in the neonatal period.
They are the second most common type of foregut cysts (after enterogenous cysts) in the middle
mediastinum. Up to 50% of cases are diagnosed prior to 15 years of age.

Investigation
Many cases are diagnosed on antenatal ultrasound. Others may be detected on conventional chest
radiography as a midline spherical mass or cystic structure. Once the diagnosis is suspected a CT
scan should be performed.

Treatment
Thorascopic resection is the ideal treatment. Very young babies can be operated on once they reach
six weeks of age.

Next question
Theme: Paediatric umbilical disorders

A. Omphalitis

B. Umbilical hernia

C. Umbilical granuloma

D. Paraumbilical hernia

E. Persistent vitello-intestinal duct

F. Persistent urachus

Please select the most likely underlying disorder for the umbilical condition described. Each option
may be used once, more than once or not at all.

25. A 2 week old baby is referred to the surgical team by the paediatricians. They are concerned
because the child has a painful area of macerated tissue at the site of the umbilicus. On
examination a clear- yellowish fluid is seen to be draining from the umbilicus when the baby cries.

You answered Omphalitis

The correct answer is Persistent urachus

A patent urachus will present with umbilical urinary discharge. The skin may become macerated if
not properly cared for. The discharge is most likely to be present when intra-abdominal pressure
is raised. It is associated with posterior urethral valves.

26. A premature neonate is born by emergency cesarean section at 29 weeks gestation. He initially
seems to be progressing well. However, the team are concerned because he becomes
systemically septic and on examination has a swollen and erythematous umbilicus.

Omphalitis

Infection from omphalitis may spread rapidly and cause severe sepsis especially in
immunologically compromised, premature neonates.
27. A baby boy is born by elective cesarean section at 39 weeks gestation. He initially seems to
progress well and is discharged from hospital the following day. The parents bring the child to the
clinic at 10 days of age and are concerned at the presence of a profuse and foul smelling
discharge at the site of the umbilicus. On examination the umbilicus has some prominent
granulation tissue. When the baby cries a small trickle of brownish fluid is seen to pass from the
umbilicus.

You answered Omphalitis

The correct answer is Persistent vitello-intestinal duct

A persistent vitello-intestinal duct may allow the persistent and ongoing discharge of small bowel
content from the umbilicus. This fluid may be very irritant to the surrounding skin.

Please rate this question:

Discuss and give feedback

Next question

Paediatric umbilical disorders

Embryology
During development the umbilicus has two umbilical arteries and one umbilical vein. The arteries are
continuous with the internal iliac arteries and the vein is continuous with the falciform ligament
(ductus venosus). After birth the cord dessicates and separates and the umbilical ring closes.

Umbilical hernia
Up to 20% of neonates may have an umbilical hernia, it is more common in premature infants. The
majority of these hernias will close spontaneously (may take between 12 months and three years).
Strangulation is rare.

Paraumbilical hernia
These are due to defects in the linea alba that are in close proximity to the umbilicus. The edges of a
paraumbilical hernia are more clearly defined than those of an umbilical hernia. They are less likely
to resolve spontaneously than an umbilical hernia.

Omphalitis
This condition consists of infection of the umbilicus. Infection with Staphylococcus aureus is the
commonest cause. The condition is potentially serious as infection may spread rapidly through the
umbilical vessels in neonates with a risk of portal pyaemia, and portal vein thrombosis. Treatment is
usually with a combination of topical and systemic antibiotics.

Umbilical granuloma
These consist of cherry red lesions surrounding the umbilicus, they may bleed on contact and be a
site of seropurulent discharge. Infection is unusual and they will often respond favorably to chemical
cautery with topically applied silver nitrate.

Persistent urachus
This is characterised by urinary discharge from the umbilicus. It is caused by persistence of the
urachus which attaches to the bladder. They are associated with other urogenital abnormalities.

Persistent vitello-intestinal duct


This will typically present as an umbilical discharge that discharges small bowel content. Complete
persistence of the duct is a rare condition. Much more common is the persistence of part of the duct
(Meckels diverticulum). Persistent vitello-intestinal ducts are best imaged using a contrast study to
delineate the anatomy and are managed by laparotomy and surgical closure.

Next question
Which one of the following is least associated with Tetralogy of Fallot?

Right ventricular outflow tract obstruction

Overriding aorta

Pan systolic murmur

Left-to-right shunt

Right ventricular hypertrophy

Right-to-left shunting is characteristic of Fallot's. In some patients there can be bidirectional shunting
(if there is mild pulmonary stenosis) and a few patients can even have pink tetralogy when there is a
predominant shunt from left to right due to minimal infundibular stenosis.
Please rate this question:

Discuss and give feedback


Next question

Tetralogy of Fallot

Tetralogy of Fallot (TOF) is the most common cause of cyanotic congenital heart disease*. It
typically presents at around 1-2 months, although may not be picked up until the baby is 6 months
old

The four characteristic features are:

 ventricular septal defect (VSD)


 right ventricular hypertrophy
 right ventricular outflow tract obstruction, pulmonary stenosis
 overriding aorta

The severity of the right ventricular outflow tract obstruction determines the degree of cyanosis and
clinical severity

Other features

 cyanosis
 causes a right-to-left shunt
 ejection systolic murmur due to pulmonary stenosis (the VSD doesn't usually cause a
murmur)
 a right-sided aortic arch is seen in 25% of patients
 chest x-ray shows a 'boot-shaped' heart, ECG shows right ventricular hypertrophy

Management

 surgical repair is often undertaken in two parts


 cyanotic episodes may be helped by beta-blockers to reduce infundibular spasm

*however, at birth transposition of the great arteries is the more common lesion as patients with TOF
generally present at around 1-2 months
Next question
A 3 year old boy is brought to the clinic with symptoms of urinary hesitancy and poor stream. Which
of the following is the most likely underlying diagnosis?

Benign prostatic hypertrophy

Posterior urethral valves

Neurogenic bladder

Urethral calculus

Hypospadias

Theme from September 2015 Exam


Posterior urethral valves are one of the commonest causes of poor urinary stream and hesitancy in
children. Prostatic disorders are rare.
Hypospadias is associated with urine that is difficult to control, but should not produce hesitancy.

Please rate this question:

Discuss and give feedback

Next question

Urethral valves

Posterior urethral valves are the commonest cause of infravesical outflow obstruction in males. They
may be diagnosed on ante natal ultrasonography. Because the bladder has to develop high
emptying pressures in utero, the child may develop renal parenchymal damage. This translates to
renal impairment noted in 70% of boys at presentation. Treatment is with bladder catheterisation.
Endoscopic valvotomy is the definitive treatment of choice with cystoscopic and renal follow up.

Next question
Which of the following is not a feature of oesphageal atresia in neonates?

High incidence of polyhydramnios

Risk of recurrence in subsequent pregnancies of 80%

Distal tracheoesphageal fistula is the commonest variant

High incidence of associated imperforate anus

Absence of gastric bubble on antenatal ultrasound

Most are sporadic and risk in subsequent pregnancies is not increased.


Please rate this question:

Discuss and give feedback


Next question

Paediatric Gastrointestinal disorders

Pyloric stenosis  M>F


 5-10% Family history in parents
 Projectile non bile stained vomiting at 4-6 weeks of life
 Diagnosis is made by test feed or USS
 Treatment: Ramstedt pyloromyotomy (open or laparoscopic)

Acute appendicitis  Uncommon under 3 years


 When occurs may present atypically

Mesenteric  Central abdominal pain and URTI


adenitis  Conservative management

Intussusception  Telescoping bowel


 Proximal to or at the level of, ileocaecal valve
 6-9 months age
 Colicky pain, diarrhoea and vomiting, sausage shaped mass, red jelly
stool.
 Treatment: reduction with air insufflation

Malrotation  High caecum at the midline


 Feature in exomphalos, congenital diaphragmatic hernia, intrinsic
duodenal atresia
 May be complicated by development of volvulus, infant with
volvulus may have bile stained vomiting
 Diagnosis is made by upper GI contrast study and USS
 Treatment is by laparotomy, if volvulus is present (or at high risk of
occurring then a ladds procedure is performed

Hirschsprung's  Absence of ganglion cells from myenteric and submucosal plexuses


disease  Occurs in 1/5000 births
 Full thickness rectal biopsy for diagnosis
 Delayed passage of meconium and abdominal distension
 Treatment is with rectal washouts initially, thereafter an anorectal
pull through procedure

Oesophageal  Associated with tracheo-oesophageal fistula and polyhydramnios


atresia  May present with choking and cyanotic spells following aspiration
 VACTERL associations

Meconium ileus  Usually delayed passage of meconium and abdominal distension


 Majority have cystic fibrosis
 X-Rays may not show a fluid level as the meconium is viscid
(depends upon feeding), PR contrast studies may dislodge meconium
plugs and be therapeutic
 Infants who do not respond to PR contrast and NG N-acetyl cysteine
will require surgery to remove the plugs

Biliary atresia  Jaundice > 14 days


 Increased conjugated bilirubin
 Urgent Kasai procedure

Necrotising  Prematurity is the main risk factor


enterocolitis  Early features include abdominal distension and passage of bloody
stools
 X-Rays may show pneumatosis intestinalis and evidence of free air
 Increased risk when empirical antibiotics are given to infants beyond
5 days
 Treatment is with total gut rest and TPN, babies with perforations
will require laparotomy

Next question
A 12 day old infant is brought to the emergency department by his anxious mother who notices that
he has developed a right sided groin swelling. On examination the testes are correctly located but it
is evident that the child has a right sided inguinal hernia. It is soft and easily reduced. What is the
most appropriate management?

Surgery over the next few days

Reassure and discharge

Surgery at 1 year of age

Surgery once the child is 6 months old

Application of a hernia truss

Inguinal hernia in infants = Urgent surgery

The high incidence of strangulation necessitates an urgent herniotomy be performed. In infants with
a reducible hernia this can be performed on a daycase list during the same week. Deferring surgery
on the basis of age is not justified.

Please rate this question:

Discuss and give feedback

Next question

Paediatric inguinal hernia

Inguinal hernias are a common disorder in children. They are commoner in males as the testis
migrates from its location on the posterior abdominal wall, down through the inguinal canal. A patent
processus vaginalis may persist and be the site of subsequent hernia development.
Children presenting in the first few months of life are at the highest risk of strangulation and the
hernia should be repaired urgently. Children over 1 year of age are at lower risk and surgery may be
performed electively. For paediatric hernias a herniotomy without implantation of mesh is sufficient.
Most cases are performed as day cases, neonates and premature infants are kept in hospital
overnight as there is a recognised increased risk of post operative apnoea.

Next question
Theme: Paediatric gastrointestinal disorders

A. Meckel diverticulum
B. Pyloric stenosis
C. Acute appendicitis
D. Mesenteric adenitis
E. Intussusception
F. Malrotation
G. Hirschsprung disease

What is the most likely diagnosis for each scenario given? Each option may be used once, more
than once or not at all.

32. A 48 hour old neonate develops increasing abdominal distension. He had a normal
delivery but has yet to pass any meconium. Following digital rectal examination liquid
stool is released.

You answered Meckel diverticulum

The correct answer is Hirschsprung disease

Theme from April 2016


Hirschsprungs may present either with features of bowel obstruction in the neonatal period
or more insidiously during childhood. After the PR there may be an improvement in
symptoms. Diagnosis is by full thickness rectal biopsy.

33. A 7 month old girl presents with vomiting and diarrhoea. She is crying and drawing her
legs up. There is a a sausage shaped mass in the abdomen.

You answered Meckel diverticulum

The correct answer is Intussusception

Sausage shaped mass (colon shaped) is common in intussusception. The other common
sign is red jelly stool.

34. A 1 month old baby girl presents with bile stained vomiting. She has an exomphalos and a
congenital diaphragmatic hernia.

You answered Meckel diverticulum

The correct answer is Malrotation


Exomphalos and diaphragmatic herniae are commonly associated with malrotation.

Please rate this question:

Discuss and give feedback


Next question

Paediatric Gastrointestinal disorders

Pyloric stenosis  M>F


 5-10% Family history in parents
 Projectile non bile stained vomiting at 4-6 weeks of life
 Diagnosis is made by test feed or USS
 Treatment: Ramstedt pyloromyotomy (open or laparoscopic)

Acute appendicitis  Uncommon under 3 years


 When occurs may present atypically

Mesenteric  Central abdominal pain and URTI


adenitis  Conservative management

Intussusception  Telescoping bowel


 Proximal to or at the level of, ileocaecal valve
 6-9 months age
 Colicky pain, diarrhoea and vomiting, sausage shaped mass, red jelly
stool.
 Treatment: reduction with air insufflation

Malrotation  High caecum at the midline


 Feature in exomphalos, congenital diaphragmatic hernia, intrinsic
duodenal atresia
 May be complicated by development of volvulus, infant with
volvulus may have bile stained vomiting
 Diagnosis is made by upper GI contrast study and USS
 Treatment is by laparotomy, if volvulus is present (or at high risk of
occurring then a ladds procedure is performed

Hirschsprung's  Absence of ganglion cells from myenteric and submucosal plexuses


disease  Occurs in 1/5000 births
 Full thickness rectal biopsy for diagnosis
 Delayed passage of meconium and abdominal distension
 Treatment is with rectal washouts initially, thereafter an anorectal
pull through procedure

Oesophageal  Associated with tracheo-oesophageal fistula and polyhydramnios


atresia  May present with choking and cyanotic spells following aspiration
 VACTERL associations

Meconium ileus  Usually delayed passage of meconium and abdominal distension


 Majority have cystic fibrosis
 X-Rays may not show a fluid level as the meconium is viscid
(depends upon feeding), PR contrast studies may dislodge meconium
plugs and be therapeutic
 Infants who do not respond to PR contrast and NG N-acetyl cysteine
will require surgery to remove the plugs

Biliary atresia  Jaundice > 14 days


 Increased conjugated bilirubin
 Urgent Kasai procedure

Necrotising  Prematurity is the main risk factor


enterocolitis  Early features include abdominal distension and passage of bloody
stools
 X-Rays may show pneumatosis intestinalis and evidence of free air
 Increased risk when empirical antibiotics are given to infants beyond
5 days
 Treatment is with total gut rest and TPN, babies with perforations
will require laparotomy

Next question
A 6 year old child develops ballooning of the foreskin on micturition and is brought to the clinic by his
anxious mother. One examination the foreskin is non retractile but otherwise normal. By which age
are 95% of all foreskins retractile

2 years

16 years

8 years

5 years

10 years

By 16 years of age almost all foreskins should be retractile and if they are not circumcision should be
considered at around this time.

Please rate this question:

Discuss and give feedback

Next question

Paediatric urology- foreskin disorders

Disorders of the foreskin


At birth and in the neonatal period the normal foreskin is non retractile due to the presence of
adhesions between the foreskin and glans. In most cases these will separate spontaneously. By the
end of puberty 95% of foreskins can be retracted. In some children the non-retractile foreskin may
balloon during micturition. This is a normal variant and requires no specific treatment.

Balanitis This is inflammation of the glans penis. It may occur in both circumcised and non-
circumcised individuals.

Posthitis This is inflammation of the foreskin. It may occur as a result of infections such as
gonorrhoea and other STD's. It may also complicate diabetes. Posthitis may progress
to phimosis and as this may make cleaning of the glans difficult and allow progression
to balanoposthitis.

Paraphimosis Prolonged retraction of the foreskin proximal to the glans may allow oedema to occur.
This may then make foreskin manipulation difficult. It can usually be managed by
compression to reduce the oedema and replacement of the foreskin. Where this fails a
dorsal slit may be required and this followed by delayed circumcision.

Phimosis This is inability to retract the foreskin and may be partial or complete. It may occur
secondary to balanoposthitis or balanitis xerotica obliterans. Depending upon the
severity and symptoms treatment with circumcision may be required.

Balanitis This is a dermatological condition in which scarring of the foreskin occurs leading to
xerotica phimosis. It is rare below the age of 5 years. Treatment is usually with circumcision.
obliterans

Next question
A neonate is noted to have colonic obstruction, what is the most likely cause?

Imperforate anus

Meconium plugs

Colonic atresia

Anal atresia

Sigmoid volvulus

Cystic fibrosis is the most common cause of meconium plugs and such cases can be treated with
intestinal lavage, this should be followed by formal testing for CF.
Please rate this question:

Discuss and give feedback


Next question

Paediatric Gastrointestinal disorders

Pyloric stenosis  M>F


 5-10% Family history in parents
 Projectile non bile stained vomiting at 4-6 weeks of life
 Diagnosis is made by test feed or USS
 Treatment: Ramstedt pyloromyotomy (open or laparoscopic)

Acute appendicitis  Uncommon under 3 years


 When occurs may present atypically

Mesenteric  Central abdominal pain and URTI


adenitis  Conservative management

Intussusception  Telescoping bowel


 Proximal to or at the level of, ileocaecal valve
 6-9 months age
 Colicky pain, diarrhoea and vomiting, sausage shaped mass, red jelly
stool.
 Treatment: reduction with air insufflation

Malrotation  High caecum at the midline


 Feature in exomphalos, congenital diaphragmatic hernia, intrinsic
duodenal atresia
 May be complicated by development of volvulus, infant with
volvulus may have bile stained vomiting
 Diagnosis is made by upper GI contrast study and USS
 Treatment is by laparotomy, if volvulus is present (or at high risk of
occurring then a ladds procedure is performed

Hirschsprung's  Absence of ganglion cells from myenteric and submucosal plexuses


disease  Occurs in 1/5000 births
 Full thickness rectal biopsy for diagnosis
 Delayed passage of meconium and abdominal distension
 Treatment is with rectal washouts initially, thereafter an anorectal
pull through procedure

Oesophageal  Associated with tracheo-oesophageal fistula and polyhydramnios


atresia  May present with choking and cyanotic spells following aspiration
 VACTERL associations

Meconium ileus  Usually delayed passage of meconium and abdominal distension


 Majority have cystic fibrosis
 X-Rays may not show a fluid level as the meconium is viscid
(depends upon feeding), PR contrast studies may dislodge meconium
plugs and be therapeutic
 Infants who do not respond to PR contrast and NG N-acetyl cysteine
will require surgery to remove the plugs

Biliary atresia  Jaundice > 14 days


 Increased conjugated bilirubin
 Urgent Kasai procedure

Necrotising  Prematurity is the main risk factor


enterocolitis  Early features include abdominal distension and passage of bloody
stools
 X-Rays may show pneumatosis intestinalis and evidence of free air
 Increased risk when empirical antibiotics are given to infants beyond
5 days
 Treatment is with total gut rest and TPN, babies with perforations
will require laparotomy

Next question
An 18 month old boy is brought to the emergency room by his parents. He was found in bed with a
nappy filled with dark red blood. He is haemodynamically unstable and requires a blood transfusion.
Prior to this episode he was well with no prior medical history. What is the most likely cause?

Necrotising enterocolitis

Anal fissure

Oesophageal varices

Meckels diverticulum

Crohns disease

Meckels diverticulum is the number one cause of painless massive GI bleeding requiring a
transfusion in children between the ages of 1 and 2 years.

Other causes of GI bleeding include:


Older than 2
Site Newborn 1 month to 1 year 1 to 2 years years

Upper Haemorrhagic disease, Oesophagitis/ Peptic ulcer Varices


GI tract swallowed maternal gastritis disease
blood

Lower Anal fissure/ NEC Anal fissure/ Polyps / Meckels IBD/ Polyps/
GI tract Intussusception diverticulum Intussusception

Please rate this question:

Discuss and give feedback


Next question

Meckel's diverticulum

 Congenital abnormality resulting in incomplete obliteration of the vitello-intestinal duct


 Normally, in the foetus, there is an attachment between the vitello-intestinal duct and the yolk
sac.This disappears at 6 weeks gestation.
 The tip is free in majority of cases.
 Associated with enterocystomas, umbilical sinuses, and omphaloileal fistulas.
 Arterial supply: omphalomesenteric artery.
 2% of population, 2 inches long, 2 feet from the ileocaecal valve.
 Typically lined by ileal mucosa but ectopic gastric mucosa can occur, with the risk of peptic
ulceration. Pancreatic and jejunal mucosa can also occur.

Clinical

 Normally asymptomatic and an incidental finding.


 Complications are the result of obstruction, ectopic tissue, or inflammation.
 Removal if narrow neck or symptomatic. Options are between wedge excision or formal
small bowel resection and anastomosis.

Next question
What is the investigation of choice to look for renal scarring in a child with vesicoureteric reflux?

Abdominal x-ray

Ultrasound

DMSA

CT KUB

Micturating cystourethrogram

Please rate this question:

Discuss and give feedback


Next question

Vesicoureteric reflux

Vesicoureteric reflux (VUR) is the abnormal backflow of urine from the bladder into the ureter and
kidney. It is relatively common abnormality of the urinary tract in children and predisposes to urinary
tract infection (UTI), being found in around 30% of children who present with a UTI. As around 35%
of children develop renal scarring it is important to investigate for VUR in children following a UTI

Pathophysiology of VUR

 ureters are displaced laterally, entering the bladder in a more perpendicular fashion than at
an angle
 therefore shortened intramural course of ureter
 vesicoureteric junction cannot therefore function adequately

The table below summarises the grading of VUR

{Grade}

I Reflux into the ureter only, no dilatation


II Reflux into the renal pelvis on micturition, no dilatation

III Mild/moderate dilatation of the ureter, renal pelvis and calyces

IV Dilation of the renal pelvis and calyces with moderate ureteral tortuosity

V Gross dilatation of the ureter, pelvis and calyces with ureteral tortuosity

Investigation

 VUR is normally diagnosed following a micturating cystourethrogram


 a DMSA scan may also be performed to look for renal scarring

Next question
Which of the following statements relating to necrotising enterocolitis is false?

It has a mortality of 30%

Most frequently presents in premature neonates less than 32 weeks gestation.

Should be managed by early laparotomy and segmental resections in most cases.

Pneumostosis intestinalis may be visible on plain abdominal x-ray.

May be minimised by use of breast milk over formula feeds.

Most cases will settle with conservative management with NG decompression and appropriate
support. Laparotomy should be undertaken in patients who progress despite conservative
management or in whom compelling indications for surgery exist (eg free air).
Please rate this question:

Discuss and give feedback


Next question

Paediatric Gastrointestinal disorders

Pyloric stenosis  M>F


 5-10% Family history in parents
 Projectile non bile stained vomiting at 4-6 weeks of life
 Diagnosis is made by test feed or USS
 Treatment: Ramstedt pyloromyotomy (open or laparoscopic)

Acute appendicitis  Uncommon under 3 years


 When occurs may present atypically

Mesenteric  Central abdominal pain and URTI


adenitis  Conservative management

Intussusception  Telescoping bowel


 Proximal to or at the level of, ileocaecal valve
 6-9 months age
 Colicky pain, diarrhoea and vomiting, sausage shaped mass, red jelly
stool.
 Treatment: reduction with air insufflation

Malrotation  High caecum at the midline


 Feature in exomphalos, congenital diaphragmatic hernia, intrinsic
duodenal atresia
 May be complicated by development of volvulus, infant with
volvulus may have bile stained vomiting
 Diagnosis is made by upper GI contrast study and USS
 Treatment is by laparotomy, if volvulus is present (or at high risk of
occurring then a ladds procedure is performed

Hirschsprung's  Absence of ganglion cells from myenteric and submucosal plexuses


disease  Occurs in 1/5000 births
 Full thickness rectal biopsy for diagnosis
 Delayed passage of meconium and abdominal distension
 Treatment is with rectal washouts initially, thereafter an anorectal
pull through procedure

Oesophageal  Associated with tracheo-oesophageal fistula and polyhydramnios


atresia  May present with choking and cyanotic spells following aspiration
 VACTERL associations

Meconium ileus  Usually delayed passage of meconium and abdominal distension


 Majority have cystic fibrosis
 X-Rays may not show a fluid level as the meconium is viscid
(depends upon feeding), PR contrast studies may dislodge meconium
plugs and be therapeutic
 Infants who do not respond to PR contrast and NG N-acetyl cysteine
will require surgery to remove the plugs

Biliary atresia  Jaundice > 14 days


 Increased conjugated bilirubin
 Urgent Kasai procedure

Necrotising  Prematurity is the main risk factor


enterocolitis  Early features include abdominal distension and passage of bloody
stools
 X-Rays may show pneumatosis intestinalis and evidence of free air
 Increased risk when empirical antibiotics are given to infants beyond
5 days
 Treatment is with total gut rest and TPN, babies with perforations
will require laparotomy

Next question
A 6 week old baby boy is brought to the clinic by his mother. She is concerned because although the
left testis is present in the scrotum the right testis is absent. She reports that it is sometimes palpable
when she bathes the child. on examination the right testis is palpable at the level of the superficial
inguinal ring. What is the most appropriate management?

Discharge

Re-assess in 5 years

Laparoscopy

Re-assess in 6 months

Orchidopexy

Undescended testes are not uncommon in young children. They may be present in 4% of term
infants, but only in 1.3% children at 3 months of age. In this scenario the testis is retractile and can
be managed expectantly.
Please rate this question:

Discuss and give feedback


Next question

Cryptorchidism

A congenital undescended testis is one that has failed to reach the bottom of the scrotum by 3
months of age. At birth up to 5% of boys will have an undescended testis, post natal descent occurs
in most and by 3 months the incidence of cryptorchidism falls to 1-2%. In the vast majority of cases
the cause of the maldescent is unknown. A proportion may be associated with other congenital
defects including:

Patent processus vaginalis


Abnormal epididymis
Cerebral palsy
Mental retardation
Wilms tumour
Abdominal wall defects (e.g. gastroschisis, prune belly syndrome)

Differential diagnosis
These include retractile testes and, in the case of absent bilateral testes the possibility of intersex
conditions. A retractile testis can be brought into the scrotum by the clinician and when released
remains in the scrotum. If the examining clinician notes the testis to return rapidly into the inguinal
canal when released then surgery is probably indicated.

Reasons for correction of cryptorchidism

 Reduce risk of infertility


 Allows the testes to be examined for testicular cancer
 Avoid testicular torsion
 Cosmetic appearance

Males with undescended testis are 40 times as likely to develop testicular cancer (seminoma) as
males without undescended testis
The location of the undescended testis affects the relative risk of testicular cancer (50% intra-
abdominal testes)

Treatment

 Orchidopexy at 6- 18 months of age. The operation usually consists of inguinal exploration,


mobilisation of the testis and implantation into a dartos pouch.
 Intra-abdominal testis should be evaluated laparoscopically and mobilised. Whether this is a
single stage or two stage procedure depends upon the exact location.
 After the age of 2 years in untreated individuals the Sertoli cells will degrade and those
presenting late in teenage years may be better served by orchidectomy than to try and
salvage a non functioning testis with an increased risk of malignancy.

Next question
An 8 week old infant is brought to clinic with a history of 18 days of jaundice. The mother is breast
feeding. He was a full term baby. There is no family history of liver disease. What is the most
appropriate next step?

Liver USS

Unconjugated bilirubin measurement

Conjugated bilirubin measurement

Reassure and discharge

ERCP

This baby is a full term and has > 14 days of jaundice, therefore needs an urgent conjugated bilirubin
check to rule out biliary atresia. If physiological jaundice the unconjugated bilirubin levels will be
increased. Isotope scanning may be used in diagnosis, but a definitive diagnosis is normally made
during a laparotomy.
Please rate this question:

Discuss and give feedback


Next question

Biliary atresia

 1 in 17000 affected
 Biliary tree lumen is obliterated by an inflammatory cholangiopathy causing progressive liver
damage

Clinical features

 Infant well in 1st few weeks of life


 No family history of liver disease
 Jaundice in infants > 14 days in term infants (>21 days in pre term infants)
 Pale stool, yellow urine (colourless in babies)
 Associated with cardiac malformations, polysplenia, situs inversus

Investigation
 Conjugated bilirubin (prolonged physiological jaundice or breast milk jaundice will cause a
rise in unconjugated bilirubin, whereas those with obstructive liver disease will have a rise in
conjugated bilirubin)
 Ultrasound of the liver (excludes extrahepatic causes, in biliary atresia infant may have tiny
or invisible gallbladder)
 Hepato-iminodiacetic acid radionuclide scan (good uptake but no excretion usually seen)

Management

 Early recognition is important to prevent liver transplantation.


 Nutritional support.
 Roux-en-Y portojejunostomy (Kasai procedure).
 If Kasai procedure fails or late recognition, a liver transplant becomes the only option.

Next question
Which of the following statements relating to Hirschsprungs disease is false?

It is more common in males.

Is typically associated with a dilated aganglionic segment of bowel.

May present with delayed passage of meconium.

Mucosal biopsies are inadequate for diagnosis.

Disease extending beyond 30cm of colon and rectum is unusual.

There is a transition zone from the contracted aganglionic segment (the abnormal area) to dilated
normal bowel on barium enema. Males are more frequently affected than females. Surgery may
involve a pull through procedure. A number of patients will have ongoing evacuatory disturbance.
Please rate this question:

Discuss and give feedback


Next question

Paediatric Gastrointestinal disorders

Pyloric stenosis  M>F


 5-10% Family history in parents
 Projectile non bile stained vomiting at 4-6 weeks of life
 Diagnosis is made by test feed or USS
 Treatment: Ramstedt pyloromyotomy (open or laparoscopic)

Acute appendicitis  Uncommon under 3 years


 When occurs may present atypically

Mesenteric  Central abdominal pain and URTI


adenitis  Conservative management

Intussusception  Telescoping bowel


 Proximal to or at the level of, ileocaecal valve
 6-9 months age
 Colicky pain, diarrhoea and vomiting, sausage shaped mass, red jelly
stool.
 Treatment: reduction with air insufflation

Malrotation  High caecum at the midline


 Feature in exomphalos, congenital diaphragmatic hernia, intrinsic
duodenal atresia
 May be complicated by development of volvulus, infant with
volvulus may have bile stained vomiting
 Diagnosis is made by upper GI contrast study and USS
 Treatment is by laparotomy, if volvulus is present (or at high risk of
occurring then a ladds procedure is performed

Hirschsprung's  Absence of ganglion cells from myenteric and submucosal plexuses


disease  Occurs in 1/5000 births
 Full thickness rectal biopsy for diagnosis
 Delayed passage of meconium and abdominal distension
 Treatment is with rectal washouts initially, thereafter an anorectal
pull through procedure

Oesophageal  Associated with tracheo-oesophageal fistula and polyhydramnios


atresia  May present with choking and cyanotic spells following aspiration
 VACTERL associations

Meconium ileus  Usually delayed passage of meconium and abdominal distension


 Majority have cystic fibrosis
 X-Rays may not show a fluid level as the meconium is viscid
(depends upon feeding), PR contrast studies may dislodge meconium
plugs and be therapeutic
 Infants who do not respond to PR contrast and NG N-acetyl cysteine
will require surgery to remove the plugs

Biliary atresia  Jaundice > 14 days


 Increased conjugated bilirubin
 Urgent Kasai procedure

Necrotising  Prematurity is the main risk factor


enterocolitis  Early features include abdominal distension and passage of bloody
stools
 X-Rays may show pneumatosis intestinalis and evidence of free air
 Increased risk when empirical antibiotics are given to infants beyond
5 days
 Treatment is with total gut rest and TPN, babies with perforations
will require laparotomy

Next question
A 4 year old is admitted with right iliac fossa pain and is due to undergo an appendicectomy. The
nursing staff would like to give the child an infusion of intravenous fluid whilst waiting for theatre.
Assuming electrolytes are normal, which of the following is an appropriate fluid for infusion in this
situation?

10% Dextrose solution

0.9% Saline solution

0.45% saline/ 5% glucose solution

Gelofusin

None of the above

0.45% saline/ 5% glucose solutions carry a risk of hyponatraemia and is contra indicated- see below.
Please rate this question:

Discuss and give feedback


Next question

Paediatric fluid management

Since 2000 there have been at least 4 reported deaths from fluid induced hyponatraemia in children.
This led to the National Patient Safety Agency introducing revised guidelines in 2007.
Indications for IV fluids include:

 Resuscitation and circulatory support


 Replacing on-going fluid losses
 Maintenance fluids for children for whom oral fluids are not appropriate
 Correction of electrolyte disturbances

Fluids to be avoided
Outside the neonatal period saline / glucose solutions should not be given. The greatest risk is with
saline 0.18 / glucose 4% solutions. The report states that 0.45% saline / 5% glucose may be used.
But preference should be given to isotonic solutions and few indications exist for this solution either.

Fluids to be used
 0.9% saline
 5% glucose (though only with saline for maintenance and not to replace losses)
 Hartmann's solution

Potassium should be added to maintenance fluids according patients plasma potassium levels
(which should be monitored).

Intraoperative fluid management


Neonates should receive glucose 10% during surgery.
Other children should receive isotonic crystalloid.

Maintenance fluids
Weight Water requirement/kg/day Na mmol/kg/day K mmol/kg/day

First 10Kg body weight 100ml 2-4 1.5-2.5

Second 10Kg body weight 50ml 1-2 0.5-1.5

Subsequent Kg 20ml 0.5-1.0 0.2-0.7

Glucose will need to be given to neonates- usually 10% at a rate of 60ml/Kg/day.

Reference
NPSA -reducing risk of hyponatraemia when administering intravenous fluids to children. Issue date
March 2007. Further references included in this document.
Next question
Theme: Paediatric gastrointestinal disorders

A. Appendicectomy
B. Active observation
C. Discharge
D. Ultrasound of the abdomen
E. Colonoscopy
F. Pneumatic reduction under fluoroscopic guidance
G. Laparotomy

Please select the most appropriate form of management from the list above. Each option may be
used once, more than once or not at all.

44. A 5 year old girl has been unwell for 3 days with occasional vomiting and lethargy, she
had one episode of diarrhoea. On examination she has a soft abdomen with tenderness in
the region of the right iliac fossa. Her temperature is 38.1. Urine dipstick shows leucocytes
(+) and protein (+).

Appendicectomy

The most likely diagnosis is appendicitis. This can often present with less robust signs in
paediatric than adult practice.

45. A 6 day old baby was born prematurely at 33 weeks. He has been suffering from
respiratory distress syndrome and has been receiving ventilatory support on NICU. He has
developed abdominal distension and is increasingly septic. Ultrasound of the abdomen
shows free fluid and evidence of small bowel dilatation. His blood pressure has remained
labile despite inotropic support

You answered Appendicectomy

The correct answer is Laparotomy

He has necrotising enterocolitis and whilst this is often initially managed medically a
laparotomy is required if the situation deteriorates.

46. A 5 year old child has been unwell with a sore throat and fever for several days. He
progresses to develop periumbilical abdominal discomfort and passes diarrhoea. This
becomes blood stained. The paediatricians call you because the ultrasound has shown a
'target sign'.

You answered Appendicectomy


The correct answer is Pneumatic reduction under fluoroscopic guidance

This child has an intussusception. The lymphadenopathy will have initiated it. A target
sign is seen on ultrasound and is the side on view of multiple layers of bowel wall.
Reduction using fluoroscopy with air is the usual first line management. Ileo-colic
intussceceptions are generally most reliably reduced using this method, long ileo-ileal
intussceceptions usually result in surgery.

Please rate this question:

Discuss and give feedback


Next question

Paediatric Gastrointestinal disorders

Pyloric stenosis  M>F


 5-10% Family history in parents
 Projectile non bile stained vomiting at 4-6 weeks of life
 Diagnosis is made by test feed or USS
 Treatment: Ramstedt pyloromyotomy (open or laparoscopic)

Acute appendicitis  Uncommon under 3 years


 When occurs may present atypically

Mesenteric  Central abdominal pain and URTI


adenitis  Conservative management

Intussusception  Telescoping bowel


 Proximal to or at the level of, ileocaecal valve
 6-9 months age
 Colicky pain, diarrhoea and vomiting, sausage shaped mass, red jelly
stool.
 Treatment: reduction with air insufflation

Malrotation  High caecum at the midline


 Feature in exomphalos, congenital diaphragmatic hernia, intrinsic
duodenal atresia
 May be complicated by development of volvulus, infant with
volvulus may have bile stained vomiting
 Diagnosis is made by upper GI contrast study and USS
 Treatment is by laparotomy, if volvulus is present (or at high risk of
occurring then a ladds procedure is performed

Hirschsprung's  Absence of ganglion cells from myenteric and submucosal plexuses


disease  Occurs in 1/5000 births
 Full thickness rectal biopsy for diagnosis
 Delayed passage of meconium and abdominal distension
 Treatment is with rectal washouts initially, thereafter an anorectal
pull through procedure

Oesophageal  Associated with tracheo-oesophageal fistula and polyhydramnios


atresia  May present with choking and cyanotic spells following aspiration
 VACTERL associations

Meconium ileus  Usually delayed passage of meconium and abdominal distension


 Majority have cystic fibrosis
 X-Rays may not show a fluid level as the meconium is viscid
(depends upon feeding), PR contrast studies may dislodge meconium
plugs and be therapeutic
 Infants who do not respond to PR contrast and NG N-acetyl cysteine
will require surgery to remove the plugs

Biliary atresia  Jaundice > 14 days


 Increased conjugated bilirubin
 Urgent Kasai procedure

Necrotising  Prematurity is the main risk factor


enterocolitis  Early features include abdominal distension and passage of bloody
stools
 X-Rays may show pneumatosis intestinalis and evidence of free air
 Increased risk when empirical antibiotics are given to infants beyond
5 days
 Treatment is with total gut rest and TPN, babies with perforations
will require laparotomy

Next question
A 4 year old boy presents with symptoms of dysuria and urinary frequency. A urine dipstick is
positive for blood and nitrites. A UTI is suspected. Which of the following follow up strategies is most
appropriate?

Watchful waiting

Cystoscopy

DMSA scan

CT scan of pelvis

Renal MRI

A first presentation of an uncomplicated UTI (even in male children) may be managed expectantly.
More than 1 UTI in a six month period should prompt further investigation. NICE guidelines from
2007 suggest that recurrent UTI and those with a non e-coli UTI should be imaged with USS and
DMSA several months later.
Please rate this question:

Discuss and give feedback

Urinary tract infection- paediatric

 UTI's may occur in 5% of young girls and 1-2% males. The incidence is higher in premature
infants.
 E-Coli accounts for 80% cases.
 In children with UTI it is important to establish whether there is underlying urinary stasis or
vesico-ureteric reflux (or both).
 Pyelonephritis in children carries the risk of renal scarring 10% and this translates into a 10%
risk of developing end stage renal disease.

Diagnosis

 Pyrexia lasting for more than 3 days mandates urine testing.


 Samples may be taken from mid-stream urine samples or supra pubic aspiration. Urine
collected from nappies usually have faecal contaminants. In samples showing mixed growth
contamination of the sample has usually occurred.
 As in adults >105 colony forming units of a single organism are usually indicative of a UTI.
Management

 A single isolated UTI (in girls) may be managed expectantly.


 > 2 UTI's (or 1 in males) in a 6 month period should prompt further testing.
 Voiding cystourethrograms show the greatest anatomical detail and is the ideal first line test
in males; isotope cystography has a lower radiation dose and is the first line test in girls.
 USS should also be performed. Renal cortical scintigraphy should be performed when renal
scarring is suspected.
Which of the following is not directly affected by warfarin?

Protein C

Factor II

Factor VII

Factor IX

Factor VIII

Warfarin affects synthesis of factors II, VII, IX, X and protein C.

Theme from September 2013 Exam


Please rate this question:

Discuss and give feedback


Next question

Warfarin

Warfarin is an oral anticoagulant which inhibits the reduction of vitamin K to its active hydroquinone
form, which in turn acts as a cofactor in the formation of clotting factor II, VII, IX and X (mnemonic =
1972) and protein C

Factors that may potentiate warfarin

 Liver disease
 P450 enzyme inhibitors, e.g.: amiodarone, ciprofloxacin
 Cranberry juice
 Drugs which displace warfarin from plasma albumin, e.g. NSAIDs
 Inhibit platelet function: NSAIDs

Side-effects

 Haemorrhage
 Teratogenic
 Skin necrosis: when warfarin is first started biosynthesis of protein C is reduced. This results
in a temporary procoagulant state after initially starting warfarin, normally avoided by
concurrent heparin administration. Thrombosis may occur in venules leading to skin
necrosis.

Next question
A 56 year old lady with idiopathic thrombocytopenic purpura has a platelet count of 50. She is due to
undergo a splenectomy. What is the optimal timing of a platelet transfusion in this case?

24 hours pre-operatively

2 hours pre-operatively

Whilst making the skin incision

After ligation of the splenic artery

On removal of the spleen

ITP causes splenic sequestration of platelets. Therefore a platelet transfusion should be carefully
timed. Too soon and it will be ineffective. Too late and unnecessary bleeding will occur. The optimal
time is after the splenic artery has been ligated.
Please rate this question:

Discuss and give feedback


Next question

Splenectomy

Indications

 Trauma: 1/4 are iatrogenic


 Spontaneous rupture: EBV
 Hypersplenism: hereditary spherocytosis or elliptocytosis etc
 Malignancy: lymphoma or leukaemia
 Splenic cysts, hydatid cysts, splenic abscesses

Splenectomy
Technique
Trauma

 GA
 Long midline incision
 If time permits insert a self retaining retractor (e.g. Balfour/ omnitract)
 Large amount of free blood is usually present. Pack all 4 quadrants of the abdomen. Allow
the anaesthetist to 'catch up'
 Remove the packs and assess the viability of the spleen. Hilar injuries and extensive
parenchymal lacerations will usually require splenectomy.
 Divide the short gastric vessels and ligate them.
 Clamp the splenic artery and vein. Two clamps on the patient side are better and allow for
double ligation and serve as a safety net if your assistant does not release the clamp
smoothly.
 Be careful not to damage the tail of the pancreas, if you do then this will need to be formally
removed and the pancreatic duct closed.
 Wash out the abdomen and place a tube drain to the splenic bed.
 Some surgeons implant a portion of spleen into the omentum, whether you decide to do this
is a matter of personal choice.
 Post operatively the patient will require prophylactic penicillin V and pneumococcal vaccine.

Elective
Elective splenectomy is a very different operation from that performed in the emergency setting. The
spleen is often large (sometimes massive). Most cases can be performed laparoscopically. The
spleen will often be macerated inside a specimen bag to facilitate extraction.

Complications

 Haemorrhage (may be early and either from short gastrics or splenic hilar vessels
 Pancreatic fistula (from iatrogenic damage to pancreatic tail)
 Thrombocytosis: prophylactic aspirin
 Encapsulated bacteria infection e.g. Strep. pneumoniae, Haemophilus
influenzae and Neisseriameningitidis

Post splenectomy changes

 Platelets will rise first (therefore in ITP should be given after splenic artery clamped)
 Blood film will change over following weeks, Howell Jolly bodies will appear
 Other blood film changes include target cells and Pappenheimer bodies
 Increased risk of post splenectomy sepsis, therefore prophylactic antibiotics and
pneumococcal vaccine should be given.

Post splenectomy sepsis

 Typically occurs with encapsulated organisms


 Opsonisation occurs but then not recognised

Next question
A 19 year old man has a skin lesion excised from his back. He is reviewed clinically at 4 months post
procedure and the surgeon notes that the scar has begun to contract. Which of the following
facilitates this process?

Myofibroblasts

Neutrophils

Granuloma formation

Macrophages

Fibroblasts

As wounds mature the fibroblast population differentiates into myofibroblasts (usually 6 weeks and
beyond), these have a contractile phenotype and therefore help in contracting the wound. Immature
fibroblasts, though able to adhere to the ECM, do not have this ability.
Please rate this question:

Discuss and give feedback


Next question

Wound healing

Surgical wounds are either incisional or excisional and either clean, clean contaminated or dirty.
Although the stages of wound healing are broadly similar their contributions will vary according to the
wound type.

The main stages of wound healing include:

Haemostasis

 Minutes to hours following injury


 Vasospasm in adjacent vessels, platelet plug formation and generation of fibrin rich clot.

Inflammation

 Typically days 1-5


 Neutrophils migrate into wound (function impaired in diabetes).
 Growth factors released, including basic fibroblast growth factor and vascular endothelial
growth factor.
 Fibroblasts replicate within the adjacent matrix and migrate into wound.
 Macrophages and fibroblasts couple matrix regeneration and clot substitution.

Regeneration

 Typically days 7 to 56
 Platelet derived growth factor and transformation growth factors stimulate fibroblasts and
epithelial cells.
 Fibroblasts produce a collagen network.
 Angiogenesis occurs and wound resembles granulation tissue.

Remodeling

 From 6 weeks to 1 year


 Longest phase of the healing process and may last up to one year (or longer).
 During this phase fibroblasts become differentiated (myofibroblasts) and these facilitate
wound contraction.
 Collagen fibres are remodeled.
 Microvessels regress leaving a pale scar.

The above description represents an idealised scenario. A number of diseases may distort this
process. Neovascularisation is an important early process. Endothelial cells may proliferate in the
wound bed and recanalise to form a vessel. Vascular disease, shock and sepsis can all compromise
microvascular flow and impair healing.

Conditions such as jaundice will impair fibroblast synthetic function and immunity with a detrimental
effect in most parts of the healing process.

Problems with scars:

Hypertrophic scars
Excessive amounts of collagen within a scar. Nodules may be present histologically containing
randomly arranged fibrils within and parallel fibres on the surface. The tissue itself is confined to the
extent of the wound itself and is usually the result of a full thickness dermal injury. They may go on
to develop contractures.

Image of hypertrophic scarring. Note that it remains confined to the boundaries of the original
wound:
Image sourced from Wikipedia

Keloid scars
Excessive amounts of collagen within a scar. Typically a keloid scar will pass beyond the boundaries
of the original injury. They do not contain nodules and may occur following even trivial injury. They
do not regress over time and may recur following removal.

Image of a keloid scar. Note the extension beyond the boundaries of the original incision:

Image sourced from Wikipedia

Drugs which impair wound healing:


 Non steroidal anti inflammatory drugs
 Steroids
 Immunosupressive agents
 Anti neoplastic drugs

Closure
Delayed primary closure is the anatomically precise closure that is delayed for a few days but before
granulation tissue becomes macroscopically evident.

Secondary closure refers to either spontaneous closure or to surgical closure after granulation tissue
has formed.
Next question
Which of the following blood products can be administered to a non ABO matched recipient?

Whole blood

Platelets

Packed red cells

Stem cells

Cryoprecipitate

In the UK, platelets either come from pooling of the platelet component from four units of whole
donated blood, called random donor platelets, or by plasmapharesis from a single donor. The
platelets are suspended in 200-300 ml of plasma and may be stored for up to 4 days in the
transfusion laboratory where they are continually agitated at 22 oC to preserve function. One adult
platelet pool raises the normal platelet count by 30,000 to 60,000 platelets litre. ABO identical or
compatible platelets are preferred but not necessary in adults; but rhesus compatibility is required in
recipients who are children and women of childbearing age to prevent haemolytic disease of the
newborn.
Please rate this question:

Discuss and give feedback


Next question

Blood products - cross matching

Whole blood fractions

Fraction Key points

Packed red cells Used for transfusion in chronic anaemia and cases where infusion of large
volumes of fluid may result in cardiovascular compromise. Product obtained
by centrifugation of whole blood.

Platelet rich Usually administered to patients who are thrombocytopaenic and are bleeding
Fraction Key points

plasma or require surgery. It is obtained by low speed centrifugation.

Platelet Prepared by high speed centrifugation and administered to patients with


concentrate thrombocytopaenia.

Fresh frozen  Prepared from single units of blood.


plasma  Contains clotting factors, albumin and immunoglobulin.
 Unit is usually 200 to 250ml.
 Usually used in correcting clotting deficiencies in patients with hepatic
synthetic failure who are due to undergo surgery.
 Usual dose is 12-15ml/Kg-1.
 It should not be used as first line therapy for hypovolaemia.

Cryoprecipitate  Formed from supernatant of FFP.


 Rich source of Factor VIII and fibrinogen.
 Allows large concentration of factor VIII to be administered in small
volume.

SAG-Mannitol Removal of all plasma from a blood unit and substitution with:
Blood

 Sodium chloride
 Adenine
 Anhydrous glucose
 Mannitol

Up to 4 units of SAG M Blood may be administered. Thereafter whole blood


is preferred. After 8 units, clotting factors and platelets should be considered.

Cross matching
Must be cross matched Can be ABO incompatible in adults

Packed red cells Platelets

Whole blood FFP


Must be cross matched Can be ABO incompatible in adults

Cryoprecipitate

Next question
Which of the following would be the optimal fluid management option for a 45 year old man due to
undergo an elective right hemicolectomy?

Remain "nil by mouth" for at least 6 hours pre-operatively and avoid intra venous fluids

Remain "nil by mouth" for at least 6 hours pre-operatively and receive supplementary
intravenous 5% dextrose to replace lost calories

Allow him free access to oral fluids only until 30 minutes prior to surgery

Administer a carbohydrate based loading drink 3 hours pre operatively, and avoid
intravenous fluids

Administer a carbohydrate based loading drink 6 hours pre-operatively and administer 5%


dextrose saline thereafter

Patients for elective surgery should not have solids for 6 hours pre-operatively. However, clear fluids
may be given up to 2 hours pre-operatively. Enhanced recovery programmes are now the standard
of care in many countries around the world and involve administration of carbohydrate loading
drinks.
The routine administration of 5% dextrose in the scenarios given above would convey little in the
way of benefit and increase the risks of electrolyte derangement post operatively.
Please rate this question:

Discuss and give feedback


Next question

Pre operative fluid management

Fluid management has been described in the British Consensus guidelines on IV fluid
therapy for Adult Surgical patients (GIFTASUP) and by NICE (CG174 December 2013)

The Recommendations include:

 Use Ringer's lactate or Hartmann's when a crystalloid is needed for resuscitation or


replacement of fluids. Avoid 0.9% N. Saline (due to risk of hyperchloraemic acidosis) unless
patient vomiting or has gastric drainage.
 Use 4%/0.18% dextrose saline or 5% dextrose in maintenance fluids. It should not be used
in resuscitation or as replacement fluids.
 Adult maintenance fluid requirements are: Na 50-100 mmol/day and K 40-80 mmol/day in
1.5-2.5L fluid per day.
 Patients for elective surgery should NOT be nil by mouth for >2 hours (unless has disorder of
gastric emptying).
 Patients for elective surgery should be given carbohydrate rich drinks 2-3h before. Ideally
this should form part of a normal pre op plan to facilitate recovery.
 Avoid mechanical bowel preparation.
 If bowel prep is used, simultaneous administration of Hartmann's or Ringer's lactate should
be considered.
 Excessive fluid losses from vomiting should be treated with a crystalloid with potassium
replacement. 0.9% N. Saline should be given if there is hypochloraemia. Otherwise
Hartmann's or Ringer lactate should be given for diarrhoea/ileostomy/ileus/obstruction.
Hartmann's should also be given in sodium losses secondary to diuretics.
 High risk patients should receive fluids and inotropes.
 An attempt should be made to detect pre or operative hypovolaemia using flow based
measurements. If this is not available, then clinical evaluation is needed i.e. JVP, pulse
volume etc.
 In Blood loss or infection causing hypovolaemia should be treated with a balanced crystalloid
or colloid (or until blood available in blood loss). A critically ill patient is unable to excrete Na
or H20 leading to a 5% risk of interstitial oedema. Therefore 5% dextrose as well as colloid
should be given.
 If patients need IV fluid resuscitation, use crystalloids that contain sodium in the range 130-
154 mmol/l, with a bolus of 500 ml over less than 15 minutes (NICE Guidance CG 174).

Next question
A 63 year old man undergoes a laparotomy and small bowel resection. Twelve hours post
operatively he is noted to have a decreased urine output. Which of the hormones listed below is
most likely to be responsible?

Cortisol

Atrial natriuretic hormone

Vasopressin

Insulin

Glucagon

Theme from January 2013 Exam


Vasopressin is released in increased quantities following most operative procedures and will tend to
cause water retention. For this reason, excessive administration of intravenous fluids in an attempt
to force a diuresis may cause fluid overload in post operative patients.
Please rate this question:

Discuss and give feedback


Next question

Response to surgery

Sympathetic nervous system

 Noradrenaline from sympathetic nerves and adrenaline from adrenal medulla


 Blood diverted from skin and visceral organs; bronchodilatation, reduced intestinal motility,
increased glucagon and glycogenolysis, insulin reduced
 Heart rate and myocardial contractility are increased

Acute phase response

 TNF-α, IL-1, IL-2, IL-6, interferon and prostaglandins are released


 Excess cytokines may cause SIRS
 Cytokines increase the release of acute phase proteins
Endocrine response

 Hypothalamus, pituitary, adrenal axis


 Increases ACTH and cortisol production:

increases protein breakdown


increases blood glucose levels

 Aldosterone increases sodium re-absorption


 Vasopressin increases water re-absorption and causes vasoconstriction

Vascular endothelium

 Nitric oxide produces vasodilatation


 Platelet activating factor enhances the cytokine response
 Prostaglandins produce vasodilatation and induce platelet aggregation

Next question
A 48 year old lady is being prepared for a Whipples procedure. A right sided subclavian line is
inserted and then anaesthesia is induced. Following intubation the patient becomes progressively
hypoxic and haemodynamically unstable. What is the most likely underlying explanation?

Drug allergy

Simple pneumothorax

Tension pneumothorax

Halothane toxicity

Haemothorax

Theme from September 2014 Exam


Central lines (and particularly subclavian lines) are risk factors for the development of
pneumothorax. In the context of positive pressure ventilation a tension pneumothorax is a strong
possibility and would be associated with haemodynamic instability.

Please rate this question:

Discuss and give feedback

Next question

Intravenous access

Venous access
A number of routes for establishing venous access are available.

Peripheral venous cannula


Easy to insert with minimal morbidity. Wide lumen cannulae can provide rapid fluid infusions. When
properly managed infections may be promptly identified and the cannula easily re sited. Problems
relate to their peripheral sites and they are unsuitable for the administration of vaso active drugs,
such as inotropes and irritant drugs such as TPN (except in the very short term setting).
Central lines
Insertion is more difficult and most operators and NICE advocate the use of ultra sound.
Coagulopathies may lead to haemorrhage following iatrogenic arterial injury. Femoral lines are
easier to insert and iatrogenic injuries easier to manage in this site however they are prone to high
infection rates. Internal jugular route is preferred. They have multiple lumens allowing for
administration of multiple infusions. The lumens are relatively narrow and thus they do not allow
particularly rapid rates of infusion.

Intraosseous access
This is typically undertaken at the anteromedial aspect of the proximal tibia and provides access to
the marrow cavity and circulatory system. Although traditionally preferred in paediatric practice they
may be used in adults and a wide range of fluids can be infused using these devices.

Tunneled lines
Tunneled lines such as Groshong and Hickman lines are popular devices for patients with long term
therapeutic requirements. These devices are usually inserted using ultrasound guidance into the
internal jugular vein and then tunneled under the skin. A cuff of woven material is sited near the end
and helps to anchor the device into the tissues. These cuffs require formal dissection to allow the
device to be removed. Tunneled lines can be linked to injection ports that are located under the skin.
These are especially popular in paediatric practice.

Peripherally inserted central cannula


Referred to as PICC lines, these are popular methods for establishing central venous access.
Because they are inserted peripherally they are less prone to major complications relating to device
insertion than conventional central lines.

Next question
Theme: Local anaesthetics

A. 1% xylocaine with 1 in 200,000 adrenaline


B. 1% Lignocaine
C. 0.5% Bupivacaine with 1 in 200,000 adrenaline
D. 0.5% Bupivicaine
E. Prilocaine 1%
F. Procaine 1%
G. Cocaine 25%
H. Cocaine 10%

Please select the local anaesthetic formulation most appropriate to the procedure indicated. Each
option may be used once, more than once or not at all.

8. A 25 year old male presents with epistaxis, the ENT SpR plans to cauterise the bleeding
point with silver nitrate.

1% xylocaine with 1 in 200,000 adrenaline

Historically cocaine was popular for the management of epistaxis. Some surgeons will still
routinely use cocaine paste for this indication. Its popularity stems from the fact that it
causes vasospasm. However, systemic absorption carries the risk of adverse reactions.
Where it is used the correct dose is 4%. Topically applied short acting local anaesthetic
agents with adrenaline may produce similar effects, with lower risks of toxicity.

9. An 18 year old boy requires a Zadeks procedure.

You answered 1% xylocaine with 1 in 200,000 adrenaline

The correct answer is 1% Lignocaine

This is excision of the toe nail and a fast acting local anaesthetic is indicated. Adrenaline
should be avoided in this setting as it can cause digital ischaemia

10. A 72 year old woman fractured her distal radius. A Biers Block is planned to facilitate
reduction of the fracture.

You answered 1% xylocaine with 1 in 200,000 adrenaline

The correct answer is Prilocaine 1%

This is the best local anaesthetic for this. Bupivacaine may cause cardiotoxicity and should
be avoided.
Local anaesthetics: avoid use of adrenaline in extremities

Please rate this question:

Discuss and give feedback


Next question

Local anaesthetic agents

Lidocaine

 An amide
 Local anaesthetic and a less commonly used antiarrhythmic (affects Na channels in the
axon)
 Hepatic metabolism, protein bound, renally excreted
 Toxicity: due to IV or excess administration. Increased risk if liver dysfunction or low protein
states. Note acidosis causes lidocaine to detach from protein binding.
 Drug interactions: Beta blockers, ciprofloxacin, phenytoin
 Features of toxicity: Initial CNS over activity then depression as lidocaine initially blocks
inhibitory pathways then blocks both inhibitory and activating pathways. Cardiac arrhythmias.
 Increased doses may be used when combined with adrenaline to limit systemic absorption.

Cocaine

 Pure cocaine is a salt, usually cocaine hydrochloride. It is supplied for local anaesthetic
purposes as a paste.
 It is supplied for clinical use in concentrations of 4 and 10%. It may be applied topically to the
nasal mucosa. It has a rapid onset of action and has the additional advantage of causing
marked vasoconstriction.
 It is lipophillic and will readily cross the blood brain barrier. Its systemic effects also include
cardiac arrhythmias and tachycardia.
 Apart from its limited use in ENT surgery it is otherwise used rarely in mainstream surgical
practice.

Bupivacaine

 Bupivacaine binds to the intracellular portion of sodium channels and blocks sodium influx
into nerve cells, which prevents depolarization.
 It has a much longer duration of action than lignocaine and this is of use in that it may be
used for topical wound infiltration at the conclusion of surgical procedures with long duration
analgesic effect.
 It is cardiotoxic and is therefore contra indicated in regional blockage in case the tourniquet
fails.
 Levobupivicaine (Chirocaine) is less cardiotoxic and causes less vasodilation.
Prilocaine

 Similar mechanism of action to other local anaesthetic agents. However, it is far less
cardiotoxic and is therefore the agent of choice for intravenous regional anaesthesia e.g.
Biers Block.

All local anaesthetic agents dissociate in tissues and this contributes to their therapeutic effect. The
dissociation constant shifts in tissues that are acidic e.g. where an abscess is present, and this
reduces the efficacy.

Doses of local anaesthetics


Agent Dose plain Dose with adrenaline

Lignocaine 3mg/Kg 7mg/Kg

Bupivacaine 2mg/Kg 2mg/Kg

Prilocaine 6mg/Kg 9mg/Kg

These are a guide only as actual doses depend on site of administration, tissue vascularity and co-
morbidities.

Maximum total local anaesthetic doses

 Lignocaine 1% plain - 3mg/ Kg - 200mg (20ml)


 Lignocaine 1% with 1 in 200,000 adrenaline - 7mg/Kg - 500mg (50ml)
 Bupivicaine 0.5% - 2mg/kg- 150mg (30ml)

Maximum doses are based on ideal body weight

Effects of adrenaline
Adrenaline may be added to local anaesthetic drugs. It prolongs the duration of action at the site of
injection and permits usage of higher doses (see above). It is contra indicated in patients taking
MAOI's or tricyclic antidepressants. The toxicity of bupivacaine is related to protein binding and
addition of adrenaline to this drug does not permit increases in the total dose of bupivacaine, in
contrast to the situation with lignocaine.

References
An excellent review is provided by:
French J and Sharp L. Local Anaesthetics. Ann R Coll Surg Engl 2012; 94: 76-80.
Next question
Which statement relating to the peri operative management of patients with diabetes mellitus is
false?

They should be placed first on the operating list

An intravenous sliding scale should be used in all cases

Potassium supplementation is likely to be required in diabetics on a sliding scale

Electrolyte abnormalities are more common after major visceral resections

Blood glucose monitoring is required during general anaesthesia

This is not the case and some type 2 diabetics may be managed using a watch and wait policy with
regular blood glucose monitoring. The cellular shifts of potassium with sliding scales may cause
problems with electrolyte management which should be anticipated.
Please rate this question:

Discuss and give feedback


Next question

Preparation for surgery

Elective and emergency patients require different preparation.

Elective cases

 Consider pre admission clinic to address medical issues.


 Blood tests including FBC, U+E, LFT's, Clotting, Group and Save
 Urine analysis
 Pregnancy test
 Sickle cell test
 ECG/ Chest x-ray

Exact tests to be performed will depend upon the proposed procedure and patient fitness.

Risk factors for development of deep vein thrombosis should be assessed and a plan for
thromboprophylaxis formulated.

Diabetes
Diabetic patients have greater risk of complications.
Poorly controlled diabetes carries high risk of wound infections.
Patients with diet or tablet controlled diabetes may be managed using a policy of omitting medication
and checking blood glucose levels regularly. Diabetics who are poorly controlled or who take insulin
will require a intravenous sliding scale. Potassium supplementation should also be given.
Diabetic cases should be operated on first.

Emergency cases
Stabilise and resuscitate where needed.
Consider whether antibiotics are needed and when and how they should be administered.
Inform blood bank if major procedures planned particularly where coagulopathies are present at the
outset or anticipated (e.g. Ruptured AAA repair)
Don't forget to consent and inform relatives.

Special preparation
Some procedures require special preparation:

 Thyroid surgery; vocal cord check.


 Parathyroid surgery; consider methylene blue to identify gland.
 Sentinel node biopsy; radioactive marker/ patent blue dye.
 Surgery involving the thoracic duct; consider administration of cream.
 Pheochromocytoma surgery; will need alpha and beta blockade.
 Surgery for carcinoid tumours; will need covering with octreotide.
 Colorectal cases; bowel preparation (especially left sided surgery)
 Thyrotoxicosis; lugols iodine/ medical therapy.

References
Management of adults with diabetes undergoing surgery and elective procedures. NHS Diabetes.
April 2011.
Next question
Theme: Surgical complications

A. Anastamotic leak
B. Chyle leak
C. Air leak
D. Biliary leak
E. Deep vein thrombosis
F. Portal vein thrombosis
G. Biliary obstruction

Please select the most likely complication for the scenario given. Each option may be used once,
more than once or not at all.

12. A 67 year old female undergoes an oesophagogastrectomy for carcinoma of the distal
oesophagus. She complains of chest pain. The following day there is brisk bubbling into
the chest drain when suction is applied.

You answered Anastamotic leak

The correct answer is Air leak

Damage to the lung substance may produce an air leak. Air leaks will manifest themselves
as a persistent pneumothorax that fails to settle despite chest drainage. When suction is
applied to the chest drainage system, active and persistent bubbling may be seen. Although
an anastomotic leak may produce a small pneumothorax, a large volume air leak is more
indicative of lung injury.

13. A 20 year old man has a protracted stay on ITU following a difficult appendicectomy for
perforated appendicitis with pelvic and sub phrenic abscesses. He has now deteriorated
further and developed deranged liver function tests.

You answered Anastamotic leak

The correct answer is Portal vein thrombosis

Such marked intra-abdominal sepsis may well produce coagulopathy and the risk of portal
vein thrombosis.

14. A 63 year old man undergoes an Ivor - Lewis oesophagogastrectomy for carcinoma of the
distal oesophagus. The following day a pale opalescent liquid is noted to be draining from
the right chest drain.

You answered Anastamotic leak


The correct answer is Chyle leak

Damage to the lymphatic duct may occur during this procedure and some surgeons
administer a lipid rich material immediately prior to surgery to facilitate its identification
in the event of iatrogenic damage.

Please rate this question:

Discuss and give feedback


Next question

Surgical complications

Complications occur in all branches of surgery and require vigilance in their detection. In many
cases anticipating the likely complications and appropriate avoidance will minimise their occurrence.
For the purposes of the MRCS the important principles to appreciate are:

 The anatomical principles that underpin complications


 The physiological and biochemical derangements that occur
 The most appropriate diagnostic modalities to utilise
 The principles which underpin their management

This is clearly a very broad area and impossible to cover comprehensively. There is considerable
overlap with other topic areas within the website.

Avoiding complications

Some points to hopefully avert complications:

 World Health Organisation checklist- now mandatory prior to all operations


 Prophylactic antibiotics - right dose, right drug, right time.
 Assess DVT/ PE risk and ensure adequate prophylaxis
 MARK site of surgery
 Use tourniquets with caution and with respect for underlying structures
 Remember the danger of end arteries and in situations where they occur avoid using
adrenaline containing solutions and monopolar diathermy.
 Handle tissues with care- devitalised tissue serves as a nidus for infection
 Be very wary of the potential for coupling injuries when using diathermy during laparoscopic
surgery
 The inferior epigastric artery is a favorite target for laparoscopic ports and surgical drains!
Anatomical principles
Understanding the anatomy of a surgical field will allow appreciation of local and systemic
complications that may occur. For example, nerve injuries may occur following surgery in specific
regions. The table below lists some of the more important nerves to consider and mechanisms of
injury

Nerve Mechanism

Accessory Posterior triangle lymph node biopsy

Sciatic Posterior approach to hip

Common peroneal Legs in Lloyd Davies position

Long thoracic Axillary node clearance

Pelvic autonomic nerves Pelvic cancer surgery

Recurrent laryngeal nerves During thyroid surgery

Hypoglossal nerve During carotid endarterectomy

Ulnar and median nerves During upper limb fracture repairs

These are just a few. The detailed functional sequelae are particularly important and will often be
tested. In addition to nerve injuries certain procedures carry risks of visceral or structural injury.
Again some particular favorites are given below:

Structure Mechanism

Thoracic duct During thoracic surgery e.g. Pneumonectomy, oesphagectomy


Structure Mechanism

Parathyroid glands During difficult thyroid surgery

Ureters During colonic resections/ gynaecological surgery

Bowel perforation Use of Verres Needle to establish pneumoperitoneum

Bile duct injury Failure to delineate Calots triangle carefully and careless use of diathermy

Facial nerve Always at risk during Parotidectomy

Tail of pancreas When ligating splenic hilum

Testicular vessels During re-do open hernia surgery

Hepatic veins During liver mobilisation

Again many could be predicted from the anatomy of the procedure.

Physiological derangements
A very common complication is bleeding and this is covered under the section of haemorrhagic
shock. Another variant is infection either superficial or deep seated. The organisms are covered
under microbiology and the features of sepsis covered under shock. Do not forget that
immunocompromised and elderly patients may present will atypical physiological parameters.

Selected physiological and biochemical issues are given below:

Complication Physiological/ Biochemical Problem

Arrhythmias following Susceptibility to hypokalaemia (K+ <4.0 in cardiac patients)


cardiac surgery
Complication Physiological/ Biochemical Problem

Neurosurgical electrolyte SIADH following cranial surgery causing hyponatraemia


disturbance

Ileus following Fluid sequestration and loss of electrolytes


gastrointestinal surgery

Pulmonary oedema following Loss of lung volume makes these patients very sensitive to fluid
pneumonectomy overload

Anastamotic leak Generalised sepsis causing mediastinitis or peritonitis depending


on site of leak

Myocardial infarct May follow any type of surgery and in addition to direct cardiac
effects the decreased cardiac output may well compromise grafts
etc.

Try making a short list of problems and causes specific to your own clinical area.

Diagnostic modalities
Depends largely on the suspected complication. In the acutely unwell surgical patient the following
baseline investigations are often helpful:

 Full blood count, urea and electrolytes, C- reactive protein (trend rather than absolute value),
serum calcium, liver function tests, clotting (don't forget to repeat if on-going bleeding)
 Arterial blood gases
 ECG (+cardiac enzymes if MI suspected)
 Chest x-ray to identify collapse/ consolidation
 Urine analysis for UTI

These will often identify the most common complications.

Special tests

 CT scanning for identification of intra-abdominal abscesses, air and if luminal contrast is


used an anastomotic leak
 Doppler USS of leg veins- for identification of DVT
 CTPA for PE
 Sending peritoneal fluid for U+E (if ureteric injury suspected) or amylase (if pancreatic injury
suspected)
 Echocardiogram if pericardial effusion suspected post cardiac surgery and no pleural window
made.

Management of complications
The guiding principal should be safe and timely intervention. Patients should be stabilised and if an
operation needs to occur in tandem with resuscitation then generally this should be of a damage
limitation type procedure rather than definitive surgery (which can be more safely undertaken in a
stable patient the following day).

Remember that recent surgery is a contra indication to thrombolysis and that in some patients IV
heparin may be preferable to a low molecular weight heparin (easier to reverse).

As a general rule laparotomies for bleeding should follow the core principle of quadrant packing and
then subsequent pack removal rather than plunging large clamps into pools of blood. The latter
approach invariable worsens the situation is often accompanied by significant visceral injury
particularly when done by the inexperienced. If packing controls a situation it is entirely acceptable
practice to leave packs in situ and return the patient to ITU for pack removal the subsequent day.
Next question
Theme: Wound closure

A. Split thickness skin grafting


B. Full thickness skin graft
C. Local flap
D. Leave wound as it is and apply a simple dressing
E. Primary closure using interrupted 3/0 silk
F. Primary closure using 4/0 interrupted nylon
G. Use of vacuum assisted closure device

Please select the most appropriate wound closure modality for the scenario given. Each option may
be used once, more than once or not at all.

15. A 68 year old man undergoes a wide local excision of a squamous cell carcinoma from the
lateral aspect his nose. At the completion of the operation the alar cartilage is visible.

You answered Split thickness skin grafting

The correct answer is Local flap

This type of wound should be managed with a local rotational flap.

16. A 68 year old man has a seborrhoiec wart on his left cheek this is removed by use of
curretage leaving a superficial defect approximately 1cm in diameter.

You answered Split thickness skin grafting

The correct answer is Leave wound as it is and apply a simple dressing

This type of superficial wound will re-epithelialise satisfactorily without grafting.

17. A 72 year old man undergoes a ray amputation for an infection complicating long standing
diabetes.

You answered Split thickness skin grafting

The correct answer is Use of vacuum assisted closure device

Ray amputations for diabetic foot infections do not heal well and should never be
primarily closed. The use of vacuum assisted closure devices has been shown to improve
healing rates.

Please rate this question:


Discuss and give feedback
Next question

Wound healing

Surgical wounds are either incisional or excisional and either clean, clean contaminated or dirty.
Although the stages of wound healing are broadly similar their contributions will vary according to the
wound type.

The main stages of wound healing include:

Haemostasis

 Minutes to hours following injury


 Vasospasm in adjacent vessels, platelet plug formation and generation of fibrin rich clot.

Inflammation

 Typically days 1-5


 Neutrophils migrate into wound (function impaired in diabetes).
 Growth factors released, including basic fibroblast growth factor and vascular endothelial
growth factor.
 Fibroblasts replicate within the adjacent matrix and migrate into wound.
 Macrophages and fibroblasts couple matrix regeneration and clot substitution.

Regeneration

 Typically days 7 to 56
 Platelet derived growth factor and transformation growth factors stimulate fibroblasts and
epithelial cells.
 Fibroblasts produce a collagen network.
 Angiogenesis occurs and wound resembles granulation tissue.

Remodeling

 From 6 weeks to 1 year


 Longest phase of the healing process and may last up to one year (or longer).
 During this phase fibroblasts become differentiated (myofibroblasts) and these facilitate
wound contraction.
 Collagen fibres are remodeled.
 Microvessels regress leaving a pale scar.
The above description represents an idealised scenario. A number of diseases may distort this
process. Neovascularisation is an important early process. Endothelial cells may proliferate in the
wound bed and recanalise to form a vessel. Vascular disease, shock and sepsis can all compromise
microvascular flow and impair healing.

Conditions such as jaundice will impair fibroblast synthetic function and immunity with a detrimental
effect in most parts of the healing process.

Problems with scars:

Hypertrophic scars
Excessive amounts of collagen within a scar. Nodules may be present histologically containing
randomly arranged fibrils within and parallel fibres on the surface. The tissue itself is confined to the
extent of the wound itself and is usually the result of a full thickness dermal injury. They may go on
to develop contractures.

Image of hypertrophic scarring. Note that it remains confined to the boundaries of the original
wound:

Image sourced from Wikipedia

Keloid scars
Excessive amounts of collagen within a scar. Typically a keloid scar will pass beyond the boundaries
of the original injury. They do not contain nodules and may occur following even trivial injury. They
do not regress over time and may recur following removal.

Image of a keloid scar. Note the extension beyond the boundaries of the original incision:
Image sourced from Wikipedia

Drugs which impair wound healing:

 Non steroidal anti inflammatory drugs


 Steroids
 Immunosupressive agents
 Anti neoplastic drugs

Closure
Delayed primary closure is the anatomically precise closure that is delayed for a few days but before
granulation tissue becomes macroscopically evident.

Secondary closure refers to either spontaneous closure or to surgical closure after granulation tissue
has formed.
Next question
Which of the following statements relating to pre-operative fluid management is false?

5% dextrose should be given cautiously in the elderly

Patients undergoing elective colonic resections may continue to drink water up to 2 hours
prior to surgery

Normal saline increases the risk of hyperchloraemic acidosis

A 70kg man will need approximately 100mmol of sodium daily

Carbohydrate rich beverages and loading drinks can cause ileus therefore should be
avoided

Carbohydrate loading is one of the enhanced recovery principles.


Please rate this question:

Discuss and give feedback


Next question

Pre operative fluid management

Fluid management has been described in the British Consensus guidelines on IV fluid
therapy for Adult Surgical patients (GIFTASUP) and by NICE (CG174 December 2013)

The Recommendations include:

 Use Ringer's lactate or Hartmann's when a crystalloid is needed for resuscitation or


replacement of fluids. Avoid 0.9% N. Saline (due to risk of hyperchloraemic acidosis) unless
patient vomiting or has gastric drainage.
 Use 4%/0.18% dextrose saline or 5% dextrose in maintenance fluids. It should not be used
in resuscitation or as replacement fluids.
 Adult maintenance fluid requirements are: Na 50-100 mmol/day and K 40-80 mmol/day in
1.5-2.5L fluid per day.
 Patients for elective surgery should NOT be nil by mouth for >2 hours (unless has disorder of
gastric emptying).
 Patients for elective surgery should be given carbohydrate rich drinks 2-3h before. Ideally
this should form part of a normal pre op plan to facilitate recovery.
 Avoid mechanical bowel preparation.
 If bowel prep is used, simultaneous administration of Hartmann's or Ringer's lactate should
be considered.
 Excessive fluid losses from vomiting should be treated with a crystalloid with potassium
replacement. 0.9% N. Saline should be given if there is hypochloraemia. Otherwise
Hartmann's or Ringer lactate should be given for diarrhoea/ileostomy/ileus/obstruction.
Hartmann's should also be given in sodium losses secondary to diuretics.
 High risk patients should receive fluids and inotropes.
 An attempt should be made to detect pre or operative hypovolaemia using flow based
measurements. If this is not available, then clinical evaluation is needed i.e. JVP, pulse
volume etc.
 In Blood loss or infection causing hypovolaemia should be treated with a balanced crystalloid
or colloid (or until blood available in blood loss). A critically ill patient is unable to excrete Na
or H20 leading to a 5% risk of interstitial oedema. Therefore 5% dextrose as well as colloid
should be given.
 If patients need IV fluid resuscitation, use crystalloids that contain sodium in the range 130-
154 mmol/l, with a bolus of 500 ml over less than 15 minutes (NICE Guidance CG 174).

Next question
Theme: ASA scoring

A. ASA 1

B. ASA 2

C. ASA 3

D. ASA 4

E. ASA 5

The American society of anaesthesiologists physical status scoring system is a popular method for
stratifying patients physical status. Please select the most appropriate ASA grade for each of the
following scenarios. Each option may be used once, more than once or not at all.

19. A 66 year old man is admitted following a collapse whilst waiting for a bus. Clinical examination
confirms a ruptured abdominal aortic aneurysm. He is moribund and hypotensive

You answered ASA 1

The correct answer is ASA 5

Theme from 2009 Exam

Patients who are moribund and will not survive without surgery are graded as ASA 5.

20. A 23 year old man with a 4cm lipoma on his flank is due to have this removed as a daycase. He is
otherwise well.

ASA 1

Absence of co-morbidities and small procedure with no systemic compromise will equate to an
ASA score of 1.

21. A 72 year old man is due to undergo an inguinal hernia repair. He suffers from COPD and has an
exercise tolerance of 10 yards. He also has pitting oedema to the thighs.
You answered ASA 1

The correct answer is ASA 4

Severe systemic disease of this nature is a constant threat to life. Especially as he also has
evidence of cardiac failure.

Please rate this question:

Discuss and give feedback

Next question

American Society of anesthesiologists physical status scoring system (ASA)

ASA Description
grade

1 No organic physiological, biochemical or psychiatric disturbance. The surgical pathology is


localised and has not invoked systemic disturbance

2 Mild or moderate systemic disruption caused either by the surgical disease process or though
underlying pre-existing disease

3 Severe systemic disruption caused either by the surgical pathology or pre-existing disease

4 Patient has severe systemic disease that is a constant threat to life

5 A patient who is moribund and will not survive without surgery

Next question
Theme: Wound infections

A. <5%
B. 5-10%
C. 15-25%
D. 25-40%
E. 0%
F. 75-100%

Please select the anticipated risk of surgical site infections for the procedures described. Each
option may be used once, more than once or not at all.

22. A patient is undergoing a Hartmans procedure for perforated sigmoid diverticular disease.

You answered <5%

The correct answer is 25-40%

This is a 'dirty' procedure and carries an SSI risk of 25-40 %.

23. A 23 year old male is undergoing an elective inguinal hernia repair.

<5%

This is a clean procedure and carries the lowest risk of SSI.

24. A 43 year old women is undergoing a laparoscopic choelcystectomy for uncomplicated


biliary colic.

You answered <5%

The correct answer is 5-10%

This is a clean contaminated procedure as the cystic duct is divided. Inadvertent spill of
bile converts the operation to a contaminated one and the risk of infection rises.

Please rate this question:

Discuss and give feedback


Next question

Surgical site infection


 Surgical site infections may occur following a breach in tissue surfaces and allow normal
commensals and other pathogens to initiate infection. They are a major cause of morbidity
and mortality.
 Surgical site infections (SSI) comprise up to 20% of all healthcare associated infections and
at least 5% of patients undergoing surgery will develop an SSI as a result.
 In many cases the organisms are derived from the patient's own body. Measures that may
increase the risk of SSI include:
 Shaving the wound using a razor (disposable clipper preferred)
 Using a non iodine impregnated incise drape if one is deemed to be necessary
 Tissue hypoxia
 Delayed administration of prophylactic antibiotics in tourniquet surgery

Preoperatively

 Don't remove body hair routinely


 If hair needs removal, use electrical clippers with single use head (razors increase infection
risk)
 Antibiotic prophylaxis if:

- placement of prosthesis or valve


- clean-contaminated surgery
- contaminated surgery

 Use local formulary


 Aim to give single dose IV antibiotic on anaesthesia
 If a tourniquet is to be used, give prophylactic antibiotics earlier

Intraoperatively

 Prepare the skin with alcoholic chlorhexidine (Lowest incidence of SSI)


 Cover surgical site with dressing
 A recent meta analysis has confirmed that administration of supplementary oxygen does not
reduce the risk of wound infection. In contrast to previous individual RCT's(1)
 Wound edge protectors do not appear to confer benefit (2)

Post operatively
Tissue viability advice for management of surgical wounds healing by secondary intention

Use of diathermy for skin incisions


In the NICE guidelines the use of diathermy for skin incisions is not advocated(3). Several
randomised controlled trials have been undertaken and demonstrated no increase in risk of SSI
when diathermy is used(4).

References
1. Brar M et al.. Perioperative supplemental oxygen in colorectal patients: a meta analysis. J Surg
A 48 year old lady has a metallic heart valve and requires a paraumbilical hernia repair.
Perioperatively she is receiving intra venous unfractionated heparin. To perform the surgery safely a
normal coagulation state is required. Which of the following strategies is routine standard practice?

Administration of 10 mg of vitamin K the night prior to surgery and stopping the heparin
infusion 6 hours pre operatively

Stopping the heparin infusion 6 hours pre operatively

Stop the heparin infusion on induction of anaesthesia

Stopping the heparin infusion 6 hours pre operatively and administration of intravenous
protamine sulphate on commencing the operation

None of the above

Patients with metallic heart valves will generally stop unfractionated heparin 6 hours pre operatively.
Unfractionated heparin is generally cleared from the circulation within 2 hours so this will allow plenty
of time and is the method of choice in the elective setting. Protamine sulphate will reverse heparin
but is associated with risks of anaphylaxis and is thus not generally used unless immediate reversal
of anticoagulation is needed, e.g. coming off bypass.
Please rate this question:

Discuss and give feedback


Next question

Heparin

 Causes the formation of complexes between antithrombin and activated thrombin/factors


7,9,10,11 & 12

Advantages of low molecular weight heparin

 Better bioavailability
 Lower risk of bleeding
 Longer half life
 Little effect on APTT at prophylactic dosages
 Less risk of HIT
Complications

 Bleeding
 Osteoporosis
 Heparin induced thrombocytopenia (HIT): occurs 5-14 days after 1st exposure
 Anaphylaxis

In surgical patients that may need a rapid return to theatre, administration of unfractionated heparin
is preferred; as low molecular weight heparins have a longer duration of action and are harder to
reverse.
Next question
Theme: Management of complications

A. Intra venous calcium


B. Intra venous potassium
C. Immediate removal of skin clips on ward
D. Removal of skin clips in theatre
E. Laryngoscopy
F. Intravenous thyroxine

Please select the most appropriate intervention for the scenario given. Each option may be used
once, more than once or not at all.

26. A 22 year old lady undergoes a total thyroidectomy for Graves disease. 6 hours post
operatively she develops respiratory stridor and develops a small haematoma in the neck

You answered Intra venous calcium

The correct answer is Immediate removal of skin clips on ward

This is true emergency and evacuation and release of pressure must be performed
immediately, in this case by removal of skin clips on the ward

27. A 44 year old lady undergoes a total thyroidectomy for recurrent multinodular goitre. 3
days post operatively she is still troubled by a hoarse voice.

You answered Intra venous calcium

The correct answer is Laryngoscopy

Unfortunately one of the laryngeal nerves may have been injured and this will be best
demonstrated by laryngoscopy.

28. A 48 year old lady undergoes a redo thyroidectomy for a multinodular goitre. 24 hours
post operatively she develops oculogyric crises and diffuse muscle spasm.

Intra venous calcium

She has most likely developed hypocalcaemic tetany and will require immediate calcium
supplementation.

Please rate this question:


Discuss and give feedback
Next question

Surgical complications

Complications occur in all branches of surgery and require vigilance in their detection. In many
cases anticipating the likely complications and appropriate avoidance will minimise their occurrence.
For the purposes of the MRCS the important principles to appreciate are:

 The anatomical principles that underpin complications


 The physiological and biochemical derangements that occur
 The most appropriate diagnostic modalities to utilise
 The principles which underpin their management

This is clearly a very broad area and impossible to cover comprehensively. There is considerable
overlap with other topic areas within the website.

Avoiding complications

Some points to hopefully avert complications:

 World Health Organisation checklist- now mandatory prior to all operations


 Prophylactic antibiotics - right dose, right drug, right time.
 Assess DVT/ PE risk and ensure adequate prophylaxis
 MARK site of surgery
 Use tourniquets with caution and with respect for underlying structures
 Remember the danger of end arteries and in situations where they occur avoid using
adrenaline containing solutions and monopolar diathermy.
 Handle tissues with care- devitalised tissue serves as a nidus for infection
 Be very wary of the potential for coupling injuries when using diathermy during laparoscopic
surgery
 The inferior epigastric artery is a favorite target for laparoscopic ports and surgical drains!

Anatomical principles
Understanding the anatomy of a surgical field will allow appreciation of local and systemic
complications that may occur. For example, nerve injuries may occur following surgery in specific
regions. The table below lists some of the more important nerves to consider and mechanisms of
injury

Nerve Mechanism
Nerve Mechanism

Accessory Posterior triangle lymph node biopsy

Sciatic Posterior approach to hip

Common peroneal Legs in Lloyd Davies position

Long thoracic Axillary node clearance

Pelvic autonomic nerves Pelvic cancer surgery

Recurrent laryngeal nerves During thyroid surgery

Hypoglossal nerve During carotid endarterectomy

Ulnar and median nerves During upper limb fracture repairs

These are just a few. The detailed functional sequelae are particularly important and will often be
tested. In addition to nerve injuries certain procedures carry risks of visceral or structural injury.
Again some particular favorites are given below:

Structure Mechanism

Thoracic duct During thoracic surgery e.g. Pneumonectomy, oesphagectomy

Parathyroid glands During difficult thyroid surgery

Ureters During colonic resections/ gynaecological surgery

Bowel perforation Use of Verres Needle to establish pneumoperitoneum


Structure Mechanism

Bile duct injury Failure to delineate Calots triangle carefully and careless use of diathermy

Facial nerve Always at risk during Parotidectomy

Tail of pancreas When ligating splenic hilum

Testicular vessels During re-do open hernia surgery

Hepatic veins During liver mobilisation

Again many could be predicted from the anatomy of the procedure.

Physiological derangements
A very common complication is bleeding and this is covered under the section of haemorrhagic
shock. Another variant is infection either superficial or deep seated. The organisms are covered
under microbiology and the features of sepsis covered under shock. Do not forget that
immunocompromised and elderly patients may present will atypical physiological parameters.

Selected physiological and biochemical issues are given below:

Complication Physiological/ Biochemical Problem

Arrhythmias following Susceptibility to hypokalaemia (K+ <4.0 in cardiac patients)


cardiac surgery

Neurosurgical electrolyte SIADH following cranial surgery causing hyponatraemia


disturbance

Ileus following Fluid sequestration and loss of electrolytes


gastrointestinal surgery

Pulmonary oedema following Loss of lung volume makes these patients very sensitive to fluid
Complication Physiological/ Biochemical Problem

pneumonectomy overload

Anastamotic leak Generalised sepsis causing mediastinitis or peritonitis depending


on site of leak

Myocardial infarct May follow any type of surgery and in addition to direct cardiac
effects the decreased cardiac output may well compromise grafts
etc.

Try making a short list of problems and causes specific to your own clinical area.

Diagnostic modalities
Depends largely on the suspected complication. In the acutely unwell surgical patient the following
baseline investigations are often helpful:

 Full blood count, urea and electrolytes, C- reactive protein (trend rather than absolute value),
serum calcium, liver function tests, clotting (don't forget to repeat if on-going bleeding)
 Arterial blood gases
 ECG (+cardiac enzymes if MI suspected)
 Chest x-ray to identify collapse/ consolidation
 Urine analysis for UTI

These will often identify the most common complications.

Special tests

 CT scanning for identification of intra-abdominal abscesses, air and if luminal contrast is


used an anastomotic leak
 Doppler USS of leg veins- for identification of DVT
 CTPA for PE
 Sending peritoneal fluid for U+E (if ureteric injury suspected) or amylase (if pancreatic injury
suspected)
 Echocardiogram if pericardial effusion suspected post cardiac surgery and no pleural window
made.

Management of complications
The guiding principal should be safe and timely intervention. Patients should be stabilised and if an
operation needs to occur in tandem with resuscitation then generally this should be of a damage
limitation type procedure rather than definitive surgery (which can be more safely undertaken in a
stable patient the following day).

Remember that recent surgery is a contra indication to thrombolysis and that in some patients IV
heparin may be preferable to a low molecular weight heparin (easier to reverse).

As a general rule laparotomies for bleeding should follow the core principle of quadrant packing and
then subsequent pack removal rather than plunging large clamps into pools of blood. The latter
approach invariable worsens the situation is often accompanied by significant visceral injury
particularly when done by the inexperienced. If packing controls a situation it is entirely acceptable
practice to leave packs in situ and return the patient to ITU for pack removal the subsequent day.
Next question
Theme: Use of blood products in surgery

A. Wait and see


B. Vitamin K
C. Fresh frozen plasma
D. Cryoprecipitate
E. Platelet cells
F. Packed red cells
G. Human Prothrombin Complex
H. Blood from the cell saver salvaged during surgery
I. Human Prothrombin Complex and vitamin K

For each coagulation or bleeding problem please select the most appropriate item. Each item may
be used once, more than once or not at all.

29. A 74 year old male is undergoing a revision total hip replacement for aseptic loosening of
the prosthesis. He has lost 1500ml of blood during the procedure. This has been collected
in a cell saver.

You answered Wait and see

The correct answer is Blood from the cell saver salvaged during surgery

This blood, which has been correctly collected can then be filtered and re-infused.

30. A 74 year old male with colon cancer sustains an iatrogenic splenic injury during surgery.
He is bleeding profusely.

You answered Wait and see

The correct answer is Packed red cells

The cell saver is inappropriate because the cells will be contaminated with malignant cells
and faecal matter from the open bowel.

31. A 53 year old cleaner is admitted with a fall. She is haemodynamically unstable and a CT
has shown a massive retroperitoneal haematoma. She is on warfarin.

You answered Wait and see

The correct answer is Human Prothrombin Complex and vitamin K

Each hospital has different protocols and would recommend discussion with a
haematologist. However Human Prothrombin Complex with vitamin K is indicated in this
situation, as the condition is life threatening.

Please rate this question:

Discuss and give feedback


Next question

Blood products

Whole blood fractions

Fraction Key points

Packed red cells Used for transfusion in chronic anaemia and cases where infusion of large
volumes of fluid may result in cardiovascular compromise. Product obtained
by centrifugation of whole blood.

Platelet rich Usually administered to patients who are thrombocytopaenic and are bleeding
plasma or require surgery. It is obtained by low speed centrifugation.

Platelet Prepared by high speed centrifugation and administered to patients with


concentrate thrombocytopaenia.

Fresh frozen  Prepared from single units of blood.


plasma  Contains clotting factors, albumin and immunoglobulin.
 Unit is usually 200 to 250ml.
 Usually used in correcting clotting deficiencies in patients with hepatic
synthetic failure who are due to undergo surgery.
 Usual dose is 12-15ml/Kg-1.
 It should not be used as first line therapy for hypovolaemia.

Cryoprecipitate  Formed from supernatant of FFP.


 Rich source of Factor VIII and fibrinogen.
 Allows large concentration of factor VIII to be administered in small
volume.

SAG-Mannitol Removal of all plasma from a blood unit and substitution with:
Blood

 Sodium chloride
 Adenine
 Anhydrous glucose
 Mannitol

Up to 4 units of SAG M Blood may be administered. Thereafter whole blood


is preferred. After 8 units, clotting factors and platelets should be considered.

Cell saver devices


These collect patients own blood lost during surgery and then re-infuse it. There are two main types:

 Those which wash the blood cells prior to re-infusion. These are more expensive to purchase
and more complicated to operate. However, they reduce the risk of re-infusing contaminated
blood back into the patient.
 Those which do not wash the blood prior to re-infusion.

Their main advantage is that they avoid the use of infusion of blood from donors into patients and
this may reduce risk of blood borne infection. It may be acceptable to Jehovah's witnesses. It is
contraindicated in malignant disease for risk of facilitating disease dissemination.

Blood products used in warfarin reversal


In some surgical patients the use of warfarin can pose specific problems and may require the use of
specialised blood products

Immediate or urgent surgery in patients taking warfarin(1) (2):

1. Stop warfarin

2. Vitamin K (reversal within 4-24 hours)


-IV takes 4-6h to work (at least 5mg)
-Oral can take 24 hours to be clinically effective

3. Fresh frozen plasma


Used less commonly now as 1st line warfarin reversal
-30ml/kg-1
-Need to give at least 1L fluid in 70kg person (therefore not appropriate in fluid overload)
-Need blood group
-Only use if human prothrombin complex is not available

4. Human Prothrombin Complex (reversal within 1 hour)


-Bereplex 50 u/kg
-Rapid action but factor 6 short half life, therefore give with vitamin K
References
1. Dentali, F., C. Marchesi, et al. (2011). "Safety of prothrombin complex concentrates for rapid
anticoagulation reversal of vitamin K antagonists. A meta-analysis." Thromb Haemost 106(3): 429-
438.

A 22 year old man presents with a peri anal abscess, which is managed by incision and drainage.
The perineal wound measures 3cm by 3cm. Which of the following is best management option?

Primary closure with interrupted mattress sutures

Delayed primary closure with interrupted mattress sutures

Allow the wound to heal by secondary intention

Insert a seton through the cavity into the rectum to allow a mature fistula track to develop

Perform a V-Y flap 2 weeks later

Peri anal abscess are typically managed by secondary intention healing. Any attempt at early
closure is at best futile and at worst dangerous. Insertion of a seton may be considered by an
experienced colorectal surgeon, and only if the tract is clearly identifiable with minimal probing.
There is seldom a need for flaps, ongoing discharge usually indicates a fistula (managed
separately).
Please rate this question:

Discuss and give feedback


Next question

Wound healing

Surgical wounds are either incisional or excisional and either clean, clean contaminated or dirty.
Although the stages of wound healing are broadly similar their contributions will vary according to the
wound type.

The main stages of wound healing include:

Haemostasis

 Minutes to hours following injury


 Vasospasm in adjacent vessels, platelet plug formation and generation of fibrin rich clot.

Inflammation

 Typically days 1-5


 Neutrophils migrate into wound (function impaired in diabetes).
 Growth factors released, including basic fibroblast growth factor and vascular endothelial
growth factor.
 Fibroblasts replicate within the adjacent matrix and migrate into wound.
 Macrophages and fibroblasts couple matrix regeneration and clot substitution.

Regeneration

 Typically days 7 to 56
 Platelet derived growth factor and transformation growth factors stimulate fibroblasts and
epithelial cells.
 Fibroblasts produce a collagen network.
 Angiogenesis occurs and wound resembles granulation tissue.

Remodeling

 From 6 weeks to 1 year


 Longest phase of the healing process and may last up to one year (or longer).
 During this phase fibroblasts become differentiated (myofibroblasts) and these facilitate
wound contraction.
 Collagen fibres are remodeled.
 Microvessels regress leaving a pale scar.

The above description represents an idealised scenario. A number of diseases may distort this
process. Neovascularisation is an important early process. Endothelial cells may proliferate in the
wound bed and recanalise to form a vessel. Vascular disease, shock and sepsis can all compromise
microvascular flow and impair healing.

Conditions such as jaundice will impair fibroblast synthetic function and immunity with a detrimental
effect in most parts of the healing process.

Problems with scars:

Hypertrophic scars
Excessive amounts of collagen within a scar. Nodules may be present histologically containing
randomly arranged fibrils within and parallel fibres on the surface. The tissue itself is confined to the
extent of the wound itself and is usually the result of a full thickness dermal injury. They may go on
to develop contractures.

Image of hypertrophic scarring. Note that it remains confined to the boundaries of the original
wound:

Image sourced from Wikipedia

Keloid scars
Excessive amounts of collagen within a scar. Typically a keloid scar will pass beyond the boundaries
of the original injury. They do not contain nodules and may occur following even trivial injury. They
do not regress over time and may recur following removal.

Image of a keloid scar. Note the extension beyond the boundaries of the original incision:

Image sourced from Wikipedia


Drugs which impair wound healing:

 Non steroidal anti inflammatory drugs


 Steroids
 Immunosupressive agents
 Anti neoplastic drugs

Closure
Delayed primary closure is the anatomically precise closure that is delayed for a few days but before
granulation tissue becomes macroscopically evident.

Secondary closure refers to either spontaneous closure or to surgical closure after granulation tissue
has formed.
Next question
A surgeon is considering using lignocaine to provide local anaesthesia for a minor surgical
procedure. Which of the following may attenuate its action?

Hyperkalaemia

Administration with adrenaline

Administration with bupivicaine

Administration with sodium bicarbonate

Use in tissues which are infected

Local anaesthetics are relatively ineffective when used in infected tissues.

Most anaesthetic agents are amine bases that become ionised due to the relative alkalinity of
tissues. In active infection there may acidosis of the tissues and therefore local anasthetics may be
less effective. Some surgeons mix sodium bicarbonate as it is reported to reduce the pain
experienced by patients during administration.
Please rate this question:

Discuss and give feedback


Next question

Local anaesthetic agents

Lidocaine

 An amide
 Local anaesthetic and a less commonly used antiarrhythmic (affects Na channels in the
axon)
 Hepatic metabolism, protein bound, renally excreted
 Toxicity: due to IV or excess administration. Increased risk if liver dysfunction or low protein
states. Note acidosis causes lidocaine to detach from protein binding.
 Drug interactions: Beta blockers, ciprofloxacin, phenytoin
 Features of toxicity: Initial CNS over activity then depression as lidocaine initially blocks
inhibitory pathways then blocks both inhibitory and activating pathways. Cardiac arrhythmias.
 Increased doses may be used when combined with adrenaline to limit systemic absorption.
Cocaine

 Pure cocaine is a salt, usually cocaine hydrochloride. It is supplied for local anaesthetic
purposes as a paste.
 It is supplied for clinical use in concentrations of 4 and 10%. It may be applied topically to the
nasal mucosa. It has a rapid onset of action and has the additional advantage of causing
marked vasoconstriction.
 It is lipophillic and will readily cross the blood brain barrier. Its systemic effects also include
cardiac arrhythmias and tachycardia.
 Apart from its limited use in ENT surgery it is otherwise used rarely in mainstream surgical
practice.

Bupivacaine

 Bupivacaine binds to the intracellular portion of sodium channels and blocks sodium influx
into nerve cells, which prevents depolarization.
 It has a much longer duration of action than lignocaine and this is of use in that it may be
used for topical wound infiltration at the conclusion of surgical procedures with long duration
analgesic effect.
 It is cardiotoxic and is therefore contra indicated in regional blockage in case the tourniquet
fails.
 Levobupivicaine (Chirocaine) is less cardiotoxic and causes less vasodilation.

Prilocaine

 Similar mechanism of action to other local anaesthetic agents. However, it is far less
cardiotoxic and is therefore the agent of choice for intravenous regional anaesthesia e.g.
Biers Block.

All local anaesthetic agents dissociate in tissues and this contributes to their therapeutic effect. The
dissociation constant shifts in tissues that are acidic e.g. where an abscess is present, and this
reduces the efficacy.

Doses of local anaesthetics


Agent Dose plain Dose with adrenaline

Lignocaine 3mg/Kg 7mg/Kg

Bupivacaine 2mg/Kg 2mg/Kg


Agent Dose plain Dose with adrenaline

Prilocaine 6mg/Kg 9mg/Kg

These are a guide only as actual doses depend on site of administration, tissue vascularity and co-
morbidities.

Maximum total local anaesthetic doses

 Lignocaine 1% plain - 3mg/ Kg - 200mg (20ml)


 Lignocaine 1% with 1 in 200,000 adrenaline - 7mg/Kg - 500mg (50ml)
 Bupivicaine 0.5% - 2mg/kg- 150mg (30ml)

Maximum doses are based on ideal body weight

Effects of adrenaline
Adrenaline may be added to local anaesthetic drugs. It prolongs the duration of action at the site of
injection and permits usage of higher doses (see above). It is contra indicated in patients taking
MAOI's or tricyclic antidepressants. The toxicity of bupivacaine is related to protein binding and
addition of adrenaline to this drug does not permit increases in the total dose of bupivacaine, in
contrast to the situation with lignocaine.

References
An excellent review is provided by:
French J and Sharp L. Local Anaesthetics. Ann R Coll Surg Engl 2012; 94: 76-80.
Next question
Which of the following statements relating to use of total parenteral nutrition is untrue?

It may cause steatosis and derangement of liver function tests

Administration via a central line or PICC line is preferable to peripheral administration

It is highly irritant to vessel walls

It should be administered when a patient has an albumin less than15

Administration of TPN for periods of less than 1 week is unlikely to produce noticable
benefits

Albumin is a poor indicator of overall nutrition and the decision to start TPN should not be based on
this parameter alone. Patients should ideally be fed enterally where possible and if this is likely to
occur within 5-7 days then starting TPN is unlikely to confer benefit.
Please rate this question:

Discuss and give feedback


Next question

Nutrition Monitoring-NICE guidelines

 Weight: daily if fluid balance concerns, otherwise weekly reducing to monthly


 BMI: at start of feeding and then monthly
 If weight cannot be obtained: monthly mid arm circumference or triceps skin fold thickness
 Daily electrolytes until levels stable. Then once or twice a week.
 Weekly glucose, phosphate, magnesium, LFTs, Ca, albumin, FBC, MCV

levels if stable

 2-4 weekly Zn, Folate, B12 and Cu levels if stable


 3-6 monthly iron and ferritin levels, manganese (if on home parenteral regime)
 6 monthly vitamin D
 Bone densitometry initially on starting home parenteral nutrition then every 2 years

Next question
Which of the following statements relating to use of tourniquets in surgery is false?

The use of an esmarch bandage tourniquet to exsanguinate the limb reduces the incidence
of neuropraxia.

Excessive inflation pressures are amongst the commonest causes of nerve injury related to
tourniquet use.

Tourniquet deflation causes a fall in CVP.

Children require lower inflation pressures than adults.

In patients developing neuropraxia related to tourniquets the radial nerve is most


frequently affected.

The use of esmarch bandage tourniquet increases the risk of nerve injury as it increases pressure in
the limb. Limb elevation is safer.
Please rate this question:

Discuss and give feedback


Next question

Tourniquets

Tourniquets are used during surgery to minimise blood loss and ensure a clear operative field. They
must be correctly applied and monitored. They are applied to extremities and in most cases are
inflated using a pressure monitoring system.

There are a number of systemic effects that can accompany tourniquet use, these can be divided
into those which occur following inflation and those that occur once the tourniquet is deflated.

Post inflation
Increased systemic vascular resistance, increased CVP and increased BP
Slower gradual increase in BP over time
Induced hypercoagulable state
Slow increase in core temperature

Post deflation
Fall in CVP, BP and SVR
Increased end tidal carbon dioxide
Enhanced fibrinolysis
Fall in core temperature
Raised serum potassium and lactate levels
Contra indications
Absolute Relative

AV fistula Sickle cell disease


Severe peripheral vascular disease History of thromboembolic
Previous vascular surgery events
Bone fracture or thrombosis at the site of tourniquet Skin grafts
application Localised infection
Lymphoedema

Local complications

 Damage to skin
 Damage to muscle (rarely compartment syndrome)
 Damage to vessels
 Neuropraxia

Next question
In relation to patients with type 1 diabetes mellitus undergoing surgery, which of the following
statements is untrue?

They should not receive oral carbohydrate loading drinks as part of enhanced recovery
programmes

When a variable rate insulin infusion is required 0.45% sodium chloride with either
0.15% or 0.3% potassium are the fluids of choice

Hourly intraoperative blood glucose measurements are required

Insulin infusions are only required in patients who will miss more than two meals or who
are nil by mouth for greater than 12 hours

Blood glucose levels persistently greater than 12 should initiate a change in therapy

Insulin should not be stopped in patients with type 1 diabetes and omission of more than one meal
will usually require a variable rate insulin infusion

Type 1 diabetics who take insulin should have this continued through the perioperative period.
Fluid guidelines in diabetics differ and are not well covered in NPSA fluid guidelines.
Please rate this question:

Discuss and give feedback


Next question

Preparation for surgery

Elective and emergency patients require different preparation.

Elective cases

 Consider pre admission clinic to address medical issues.


 Blood tests including FBC, U+E, LFT's, Clotting, Group and Save
 Urine analysis
 Pregnancy test
 Sickle cell test
 ECG/ Chest x-ray

Exact tests to be performed will depend upon the proposed procedure and patient fitness.
Risk factors for development of deep vein thrombosis should be assessed and a plan for
thromboprophylaxis formulated.

Diabetes
Diabetic patients have greater risk of complications.
Poorly controlled diabetes carries high risk of wound infections.
Patients with diet or tablet controlled diabetes may be managed using a policy of omitting medication
and checking blood glucose levels regularly. Diabetics who are poorly controlled or who take insulin
will require a intravenous sliding scale. Potassium supplementation should also be given.
Diabetic cases should be operated on first.

Emergency cases
Stabilise and resuscitate where needed.
Consider whether antibiotics are needed and when and how they should be administered.
Inform blood bank if major procedures planned particularly where coagulopathies are present at the
outset or anticipated (e.g. Ruptured AAA repair)
Don't forget to consent and inform relatives.

Special preparation
Some procedures require special preparation:

 Thyroid surgery; vocal cord check.


 Parathyroid surgery; consider methylene blue to identify gland.
 Sentinel node biopsy; radioactive marker/ patent blue dye.
 Surgery involving the thoracic duct; consider administration of cream.
 Pheochromocytoma surgery; will need alpha and beta blockade.
 Surgery for carcinoid tumours; will need covering with octreotide.
 Colorectal cases; bowel preparation (especially left sided surgery)
 Thyrotoxicosis; lugols iodine/ medical therapy.

References
Management of adults with diabetes undergoing surgery and elective procedures. NHS Diabetes.
April 2011.
Next question
A 72 year old man is recovering from an inguinal hernia repair when he suffers from an extensive
CVA. He is managed on the rehabilitation unit. However, he is still not able to feed safely and
repeated swallowing assessments have shown that he tends to aspirate. Which of the following is
the best option for long term feeding?

PEG tube feeding

Feeding jejunostomy

Total parenteral nutrition

Long term naso gastric tube feeding

Withold feeding and palliate

Theme from September 2014 exam


A PEG tube is the best long term option although they are associated with a significant degree of
morbidity. A feeding jejunostomy would require a general anaesthetic. TPN is not a good option.
Long term naso gastric feeding is usually unsatisfactory.
Please rate this question:

Discuss and give feedback


Next question

Enteral Feeding

 Identify patients as malnourished or at risk (see below)


 Identify unsafe or inadequate oral intake with functional GI tract
 Consider for enteral feeding
 Gastric feeding unless upper GI dysfunction (then for duodenal or jejunal tube)
 Check NG placement using aspiration and pH (check post pyloric tubes with AXR)
 Gastric feeding > 4 weeks consider long-term gastrostomy
 Consider bolus or continuous feeding into the stomach
 ITU patients should have continuous feeding for 16-24h (24h if on insulin)
 Consider motility agent in ITU or acute patients for delayed gastric emptying. If this doesn't
work then try post pyloric feeding or parenteral feeding.
 PEG can be used 4 hours after insertion, but should not be removed until >2 weeks after
insertion.

Surgical patients due to have major abdominal surgery: if malnourished, unsafe swallow/inadequate
oral intake and functional GI tract then consider pre operative enteral feeding.
Patients identified as being malnourished

 BMI < 18.5 kg/m2


 unintentional weight loss of > 10% over 3-6/12
 BMI < 20 kg/m 2 and unintentional weight loss of > 5% over 3-6/12

AT RISK of malnutrition

 Eaten nothing or little > 5 days, who are likely to eat little for a further 5 days
 Poor absorptive capacity
 High nutrient losses
 High metabolism

Reference
Stroud M et al. Guidelines for enteral feeding in adult hospital patients. Gut 2003; 52(Suppl VII):vii1 -
vii12.
Next question
Theme: Anaesthetic agents

A. Halothane
B. Propofol
C. Ketamine
D. Etomidate
E. Sodium thiopentone
F. Flumazenil
G. Naloxone
H. Sevoflurane

Please select the drug which most closely matches the description given. Each option may be used
once, more than once or not at all.

38. An agent which reverses the action of midazolam

You answered Halothane

The correct answer is Flumazenil

Flumazenil antagonises the effects of benzodiazepines by competition at GABA binding


sites. Since may benzodiazepines have longer half lives than flumazenil patients still
require close monitoring after receiving the drug.

39. An agent which is associated with hepatotoxicity

Halothane

Halothane is hepatotoxic. Despite this it remains in mainstream use. It should be avoided


in patients with hepatic dysfunction, and scavengers should be used in theatres as
accumulation of the drug may be injurious to theatre staff.

40. An anaesthetic agent which has anti emetic properties

You answered Halothane

The correct answer is Propofol

Propofol is rapidly metabolised and has mild/ moderate anti emetic properties. It is the
agent of choice in most day case operations for this reason.

Please rate this question:


Discuss and give feedback
Next question

Anaesthetic agents

The table below summarises some of the more commonly used IV induction agents
Agent Specific features

Propofol  Rapid onset of anaesthesia


 Pain on IV injection
 Rapidly metabolised with little accumulation of metabolites
 Proven anti emetic properties
 Moderate myocardial depression
 Widely used especially for maintaining sedation on ITU, total IV
anaesthesia and for daycase surgery

Sodium  Extremely rapid onset of action making it the agent of choice for rapid
thiopentone sequence of induction
 Marked myocardial depression may occur
 Metabolites build up quickly
 Unsuitable for maintenance infusion
 Little analgesic effects

Ketamine  May be used for induction of anaesthesia


 Has moderate to strong analgesic properties
 Produces little myocardial depression making it a suitable agent for
anaesthesia in those who are haemodynamically unstable
 May induce state of dissociative anaesthesia resulting in nightmares

Etomidate  Has favorable cardiac safety profile with very little haemodynamic
instability
 No analgesic properties
 Unsuitable for maintaining sedation as prolonged (and even brief) use
may result in adrenal suppression
 Post operative vomiting is common

Next question
A 57 year old man is coming off the cardiac bypass circuit following a successful coronary artery
bypass procedure. Which drug should be administered to normalise the patients clotting prior to
decannulation and chest closure?

Intravenous vitamin K

Protamine sulphate

Aprotinin

Fresh frozen plasma

None of the above

Since cardiac bypass circuits are thrombogenic large doses of intravenous heparin are administered.
This is reversed with protamine sulphate. FFP may be effective but would carry a significant risk of
fluid overload.
Please rate this question:

Discuss and give feedback


Next question

Heparin

 Causes the formation of complexes between antithrombin and activated thrombin/factors


7,9,10,11 & 12

Advantages of low molecular weight heparin

 Better bioavailability
 Lower risk of bleeding
 Longer half life
 Little effect on APTT at prophylactic dosages
 Less risk of HIT

Complications
 Bleeding
 Osteoporosis
 Heparin induced thrombocytopenia (HIT): occurs 5-14 days after 1st exposure
 Anaphylaxis

In surgical patients that may need a rapid return to theatre, administration of unfractionated heparin
is preferred; as low molecular weight heparins have a longer duration of action and are harder to
reverse.
Next question
A 34 year old man is suffering from septic shock and receives and infusion of Dextran 70. Which of
the following complications may potentially ensue?

Anaphylaxis

Vomiting

Acute hepatic failure

Digital necrosis

Deep vein thrombosis

Dextran 40 and 70 have higher incidence of anaphylaxis than either gelatins or starches.

Dextrans are branched polysaccharide molecules. Dextran 40 and 70 are available. The higher
molecular weight dextran 70 may persist for up to 8 hours. They inhibit platelet aggregation and
leucocyte plugging in the microcirculation. Thereby improving flow through the microcirculation,
primarily of use in sepsis.
Unlike many other intravenous fluids Dextrans are a recognised cause of anaphylaxis.
Please rate this question:

Discuss and give feedback


Next question

Post operative fluid management

Composition of commonly used intravenous fluids mmol-1

Na K Cl Bicarbonate Lactate

Plasma 137-147 4-5.5 95-105 22-25 -

0.9% Saline 153 - 153 - -

Dextrose / saline 30.6 - 30.6 - -


Hartmans 130 4 110 - 28

Post operative fluid management


In the UK the GIFTASUP and NICE (CG174 2013) guidelines (see reference below) were devised to
try and provide some consensus guidance as to how intravenous fluids should be administered. A
decade ago it was a commonly held belief that little harm would occur as a result of excessive
administration of normal saline and many oliguric post operative patients received enormous
quantities of IV fluids. As a result they developed hyperchloraemic acidosis. With greater
understanding of this potential complication, the use of electrolyte balanced solutions (Ringers
lactate/ Hartmans) is now favored over normal saline.
The other guidance includes:

 Fluids given should be documented clearly and easily available


 Assess the patient's fluid status when they leave theatre
 If a patient is haemodynamically stable and euvolaemic, aim to restart oral fluid intake as
soon as possible
 Review patients whose urinary sodium is < 20
 If a patient is oedematous, hypovolaemia if present should be treated first. This should then
be followed by a negative balance of sodium and water, monitored using urine Na excretion
levels
 Solutions such as Dextran 70 should be used in caution in patients with sepsis as there is a
risk of developing acute renal injury

References
NICE guidance CG174. Intravenous fluid therapy in adults. December 2013.

British Consensus Guidelines on Intravenous Fluid Therapy for Adult Surgical Patients. GIFTASUP
(2009)
Next question
In relation to operating in the elderly which statement is false?

A 30 minute increment in operation length is associated with increase in mortality in


patients over the age of 80

Hypoalbuminaemia is associated with increased mortality

Statins given preoperatively reduce perioperative cardiac events

Elevated brain (or B-type) natriuretic peptide (BNP) levels before undergoing non cardiac
surgery is associated with high risk of cardiac mortality and all cause mortality

Beta blockers should be stopped acutely prior to surgery due to risk of perioperative
hypotension

Beta blockers should not be stopped acutely prior to surgery as there may be a rebound effect
associated with increased complications.

Brain natriuretic peptide is a neurohormone synthesized in the cardiac ventricles. Levels have been
used to assess prognosis in heart failure and acute coronary syndromes. Preoperative elevated
brain natriuretic peptide levels identify patients undergoing non cardiac surgery at high risk of
cardiac mortality and all cause mortality.

All patients with peripheral vascular disease should take statins prior to vascular surgery as studies
have shown a 50% risk reduction and a reduction in perioperative cardiac events.

Reference
1. Dernellis J, Panaretou M. Assessment of cardiac risk before non-cardiac surgery: brain natriuretic
peptide in 1590 patients. Heart 2006;92:1645-1650
2. Poldermans, D et al Fluvastatin and Perioperative Events in Patients Undergoing Vascular
Surgery. NEJM 2009; 361:980-989
Please rate this question:

Discuss and give feedback


Next question

Proactive care of older people undergoing surgery (POPS)

 Comprehensive geriatric assessment


 MDT assessment preoperatively
 Main predictors of complications are co-morbidities cardiac disease and reduced functional
capacity - preoperative assessment is the key to preventing adverse postoperative outcomes
 Patients screened for risk factors (albumin <30, co morbidities)
 Management plan made and disseminated to all involved
 Patients education: pain relief, post op exercises, nutrition

Outcomes:

 Fewer postoperative medical complications


 Reduced length of stay by 4.5 days

References
Proactive care of older people undergoing surgery (POPS)
Danielle Harari et al.
Age and Ageing 2007 36(2):190-196
Next question
Which of the following intravenous fluid solutions has the greatest chloride content?

Dextrose / saline

Normal saline

Hartmanns solution

Ringers lactate

5% dextrose

Normal saline has the highest chloride content and excessive administration of normal saline is a
recognised risk factor for the development of hyperchloraemic metabolic acidosis.

Please rate this question:

Discuss and give feedback

Next question

Intra operative fluid management

Composition of commonly used intravenous fluids mmol-1

Na K Cl Bicarbonate Lactate

Plasma 137-147 4-5.5 95-105 22-25 -

0.9% Saline 153 - 153 - -


Dextrose / saline 30.6 - 30.6 - -

Hartmans 130 4 110 - 28

Recommendations for intra operative fluid management


The latest set of NICE guidelines produced in 2013 relating to intravenous fluids did not specifically
address the requirements of intra operative fluid administration. The reason for this is that
administration of fluids in this specific situation does not lend itself to rigid algorithms.
With the introduction of enhanced recovery programmes 10 years ago there was an increasing
emphasis of the concept of fluid restriction. Historically, patients received very large volumes of
saline rich solutions peri-operatively. Clearing the sodium load of a single litre of saline may take up
to 36 hours or more. This can have deleterious effects on the tissues including the development of
oedema. This results in poor perfusion, increased risk of ileus and wound breakdown. A tailored
approach to fluid administration is now practiced and far greater usage is made of cardiac output
monitors in providing goal directed fluid therapy.

References
British Consensus Guidelines on Intravenous Fluid Therapy for Adult Surgical Patients
GIFTASUP (2009) Revised May 2011.

Frost P. Intravenous fluid therapy in adult inpatients. BMJ 2015 (350): 31-34.

Next question
A 43 year old lady with a metallic heart valve has just undergone an elective paraumbilical hernia
repair. In view of her metallic valve, she is given unfractionated heparin perioperatively. How should
the therapeutic efficacy be monitored, assuming her renal function is normal?

Therapeutic monitoring is not required

Measurement of APTT

Measurement of INR

Measurement of Prothromin time

None of the above

Unlike low molecular weight heparins that do not require monitoring unfractionated heparin does
require monitoring, this is done by measuring the APTT.
Please rate this question:

Discuss and give feedback


Next question

Heparin

 Causes the formation of complexes between antithrombin and activated thrombin/factors


7,9,10,11 & 12

Advantages of low molecular weight heparin

 Better bioavailability
 Lower risk of bleeding
 Longer half life
 Little effect on APTT at prophylactic dosages
 Less risk of HIT

Complications
 Bleeding
 Osteoporosis
 Heparin induced thrombocytopenia (HIT): occurs 5-14 days after 1st exposure
 Anaphylaxis

In surgical patients that may need a rapid return to theatre, administration of unfractionated heparin
is preferred; as low molecular weight heparins have a longer duration of action and are harder to
reverse.
Next question
A patient receives atropine as pre medication prior to a laparotomy. Which of the following is least
likely to occur?

Pupillary dilation

Dry mouth

Urinary retention

Bradycardia

Decreased salivation

Since it inhibits vagal tone, the use of atropine will typically result in an increased heart rate.

Please rate this question:

Discuss and give feedback

Next question

Atropine

Atropine is a muscarinic receptor antagonist (competitive antagonist for the muscarinic acetylcholine
receptor). It therefore inhibits parasympathetic activity.It was traditionally used as a premedication
for anaesthesia because it reduced bronchial secretions, salivary secretions and bradycardia from
increased vagal tone on anaesthetic induction. Modern anaesthetic techniques have reduced the
need for routine use of this drug. Its other effects include urinary retention and pupillary dilatation.

Next question
Theme: Pre operative preparation

A. Methylene Blue intravenously 1 hour pre-operatively


B. Lugol's iodine
C. 100ml single cream given 4 hours prior to surgery
D. Carbohydrate loading drink 2 hours prior to surgery
E. Picolax sachet
F. Fleet enema
G. Intravenous calcium chloride
H. 1mg lorazepam orally 30 minutes pre operatively
I. Patent blue dye intravenously

For each procedure please select the most appropriate procedure specific preparation required.
Each option may be used once, more than once or not at all.

47. A 45 year old man is due to undergo an Ivor Lewis oesophagectomy for a carcinoma of the
distal oesophagus.

You answered Methylene Blue intravenously 1 hour pre-operatively

The correct answer is 100ml single cream given 4 hours prior to surgery

This will facilitate identification of the thoracic duct if it is inadvertently divided during
the operation.

48. A 32 year old man is due to undergo a right hemicolectomy for a large caecal sessile
polyp.

You answered Methylene Blue intravenously 1 hour pre-operatively

The correct answer is Carbohydrate loading drink 2 hours prior to surgery

This is now a standard feature of colonic enhanced recovery programmes.The


administration of carbohydrate rich loading drinks results in lower incidence of ileus. The
drink is usually administered 2 hours pre-operatively and is rapidly absorbed from the GI
tract.

49. A 67 year old women is due to undergo a parathyroidectomy for a parathyroid adenoma.

Methylene Blue intravenously 1 hour pre-operatively

Though not universally adopted, many endocrine surgeons will administer methylene blue
as it will facilitate identification of the parathyroid glands.
Please rate this question:

Discuss and give feedback


Next question

Preparation for surgery

Elective and emergency patients require different preparation.

Elective cases

 Consider pre admission clinic to address medical issues.


 Blood tests including FBC, U+E, LFT's, Clotting, Group and Save
 Urine analysis
 Pregnancy test
 Sickle cell test
 ECG/ Chest x-ray

Exact tests to be performed will depend upon the proposed procedure and patient fitness.

Risk factors for development of deep vein thrombosis should be assessed and a plan for
thromboprophylaxis formulated.

Diabetes
Diabetic patients have greater risk of complications.
Poorly controlled diabetes carries high risk of wound infections.
Patients with diet or tablet controlled diabetes may be managed using a policy of omitting medication
and checking blood glucose levels regularly. Diabetics who are poorly controlled or who take insulin
will require a intravenous sliding scale. Potassium supplementation should also be given.
Diabetic cases should be operated on first.

Emergency cases
Stabilise and resuscitate where needed.
Consider whether antibiotics are needed and when and how they should be administered.
Inform blood bank if major procedures planned particularly where coagulopathies are present at the
outset or anticipated (e.g. Ruptured AAA repair)
Don't forget to consent and inform relatives.

Special preparation
Some procedures require special preparation:

 Thyroid surgery; vocal cord check.


 Parathyroid surgery; consider methylene blue to identify gland.
 Sentinel node biopsy; radioactive marker/ patent blue dye.
 Surgery involving the thoracic duct; consider administration of cream.
 Pheochromocytoma surgery; will need alpha and beta blockade.
 Surgery for carcinoid tumours; will need covering with octreotide.
 Colorectal cases; bowel preparation (especially left sided surgery)
 Thyrotoxicosis; lugols iodine/ medical therapy.

References
Management of adults with diabetes undergoing surgery and elective procedures. NHS Diabetes.
April 2011.
Next question
Theme: Local anaesthetics

A. 1% xylocaine with 1 in 200,000 adrenaline


B. 1% Lignocaine
C. 0.5% Bupivacaine with 1 in 200,000 adrenaline
D. 0.5% Bupivacaine
E. Prilocaine 1%
F. Procaine 1%
G. Cocaine 4%
H. Cocaine 10%

Please select the local anaesthetic formulation most appropriate to the procedure indicated. Each
option may be used once, more than once or not at all.

50. A 28 year old man has a sebaceous cyst of the scalp that requires excision.

1% xylocaine with 1 in 200,000 adrenaline

As scalp wounds can bleed profusely an adrenaline containing solution is preferred.


Xylocaine is similar to lignocaine in its onset and duration of action.

51. A 32 year old man has an appendicectomy performed through a Lanz incision, which
anaesthetic would you infiltrate the wound with to provide post operative analgesia.

You answered 1% xylocaine with 1 in 200,000 adrenaline

The correct answer is 0.5% Bupivacaine

A long acting local anaesthetic is preferred.There is little advantage to adding a short


acting local anaesthetic agent since by the time the patient has recovered following surgery
the bupivacaine will be active.

52. A 43 year old man is due to undergo a vasectomy.

You answered 1% xylocaine with 1 in 200,000 adrenaline

The correct answer is 1% Lignocaine

Plain lignocaine will suffice. This will give rapid onset of action. Bupivacaine will take too
long to take effect. There would be little additional benefit derived for adding adrenaline.
Use of adrenaline with local anaesthetic agents prolongs duration of actions and allows
administration of larger doses.
Please rate this question:

Discuss and give feedback


Next question

Local anaesthetic agents

Lidocaine

 An amide
 Local anaesthetic and a less commonly used antiarrhythmic (affects Na channels in the
axon)
 Hepatic metabolism, protein bound, renally excreted
 Toxicity: due to IV or excess administration. Increased risk if liver dysfunction or low protein
states. Note acidosis causes lidocaine to detach from protein binding.
 Drug interactions: Beta blockers, ciprofloxacin, phenytoin
 Features of toxicity: Initial CNS over activity then depression as lidocaine initially blocks
inhibitory pathways then blocks both inhibitory and activating pathways. Cardiac arrhythmias.
 Increased doses may be used when combined with adrenaline to limit systemic absorption.

Cocaine

 Pure cocaine is a salt, usually cocaine hydrochloride. It is supplied for local anaesthetic
purposes as a paste.
 It is supplied for clinical use in concentrations of 4 and 10%. It may be applied topically to the
nasal mucosa. It has a rapid onset of action and has the additional advantage of causing
marked vasoconstriction.
 It is lipophillic and will readily cross the blood brain barrier. Its systemic effects also include
cardiac arrhythmias and tachycardia.
 Apart from its limited use in ENT surgery it is otherwise used rarely in mainstream surgical
practice.

Bupivacaine

 Bupivacaine binds to the intracellular portion of sodium channels and blocks sodium influx
into nerve cells, which prevents depolarization.
 It has a much longer duration of action than lignocaine and this is of use in that it may be
used for topical wound infiltration at the conclusion of surgical procedures with long duration
analgesic effect.
 It is cardiotoxic and is therefore contra indicated in regional blockage in case the tourniquet
fails.
 Levobupivicaine (Chirocaine) is less cardiotoxic and causes less vasodilation.
Prilocaine

 Similar mechanism of action to other local anaesthetic agents. However, it is far less
cardiotoxic and is therefore the agent of choice for intravenous regional anaesthesia e.g.
Biers Block.

All local anaesthetic agents dissociate in tissues and this contributes to their therapeutic effect. The
dissociation constant shifts in tissues that are acidic e.g. where an abscess is present, and this
reduces the efficacy.

Doses of local anaesthetics


Agent Dose plain Dose with adrenaline

Lignocaine 3mg/Kg 7mg/Kg

Bupivacaine 2mg/Kg 2mg/Kg

Prilocaine 6mg/Kg 9mg/Kg

These are a guide only as actual doses depend on site of administration, tissue vascularity and co-
morbidities.

Maximum total local anaesthetic doses

 Lignocaine 1% plain - 3mg/ Kg - 200mg (20ml)


 Lignocaine 1% with 1 in 200,000 adrenaline - 7mg/Kg - 500mg (50ml)
 Bupivicaine 0.5% - 2mg/kg- 150mg (30ml)

Maximum doses are based on ideal body weight

Effects of adrenaline
Adrenaline may be added to local anaesthetic drugs. It prolongs the duration of action at the site of
injection and permits usage of higher doses (see above). It is contra indicated in patients taking
MAOI's or tricyclic antidepressants. The toxicity of bupivacaine is related to protein binding and
addition of adrenaline to this drug does not permit increases in the total dose of bupivacaine, in
contrast to the situation with lignocaine.

References
An excellent review is provided by:
French J and Sharp L. Local Anaesthetics. Ann R Coll Surg Engl 2012; 94: 76-80.
Next question
Theme: Acid - base disorders

A. Respiratory alkalosis
B. Type 1 respiratory failure
C. Type 2 respiratory failure
D. Metabolic alkalosis
E. Metabolic acidosis with normal anion gap
F. Metabolic acidosis with increased anion gap

Please match the condition with the blood gas result. Each option may be used once more than
once or not at all.

53. pH 7.48, pO2 10.1, Bicarbonate 30, pCO2 4.5, Chloride<10meq

You answered Respiratory alkalosis

The correct answer is Metabolic alkalosis

This would be typical result of prolonged vomiting.

54. pH 7.49, pO2 8.5, pCO2 2.4, Bicarbonate 22, Chloride 12meq

Respiratory alkalosis

The hyperventilation results in decreased carbon dioxide levels, causing a respiratory


alkalosis (non compensated).

55. pH 7.20, pO2 7.5, Bicarbonate 22, pCO2 8.1, Chloride 10

You answered Respiratory alkalosis

The correct answer is Type 2 respiratory failure

This is a sign of acute type 2 respiratory failure (non compensated). This is the result of
carbon dioxide retention.

Please rate this question:

Discuss and give feedback


Next question

Disorders of acid - base balance


Disorders of acid- base balance are often covered in the MRCS part A, both in the SBA and EMQ
sections.

The acid-base normogram below shows how the various disorders may be categorised

Image sourced from Wikipedia

Metabolic acidosis

 This is the most common surgical acid - base disorder.


 Reduction in plasma bicarbonate levels.
 Two mechanisms:

1. Gain of strong acid (e.g. diabetic ketoacidosis)


2. Loss of base (e.g. from bowel in diarrhoea)
- Classified according to the anion gap, this can be calculated by:
(Na+ + K+) - (Cl- + HCO3-).
- If a question supplies the chloride level then this is often a clue that the anion gap should be
calculated. The normal range = 10-18 mmol/L

Normal anion gap ( = hyperchloraemic metabolic acidosis)

 Gastrointestinal bicarbonate loss: diarrhoea, ureterosigmoidostomy, fistula


 Renal tubular acidosis
 Drugs: e.g. acetazolamide
 Ammonium chloride injection
 Addison's disease

Raised anion gap

 Lactate: shock, hypoxia


 Ketones: diabetic ketoacidosis, alcohol
 Urate: renal failure
 Acid poisoning: salicylates, methanol

Metabolic acidosis secondary to high lactate levels may be subdivided into two types:

 Lactic acidosis type A: (Perfusion disorders e.g.shock, hypoxia, burns)


 Lactic acidosis type B: (Metabolic e.g. metformin toxicity)

Metabolic alkalosis

 Usually caused by a rise in plasma bicarbonate levels.


 Rise of bicarbonate above 24 mmol/L will typically result in renal excretion of excess
bicarbonate.
 Caused by a loss of hydrogen ions or a gain of bicarbonate. It is due mainly to problems of
the kidney or gastrointestinal tract

Causes

 Vomiting / aspiration (e.g. Peptic ulcer leading to pyloric stenosis, nasogastric suction)
 Diuretics
 Liquorice, carbenoxolone
 Hypokalaemia
 Primary hyperaldosteronism
 Cushing's syndrome
 Bartter's syndrome
 Congenital adrenal hyperplasia

Mechanism of metabolic alkalosis

 Activation of renin-angiotensin II-aldosterone (RAA) system is a key factor


 Aldosterone causes reabsorption of Na+ in exchange for H+ in the distal convoluted tubule
 ECF depletion (vomiting, diuretics) → Na+ and Cl- loss → activation of RAA system → raised
aldosterone levels
 In hypokalaemia, K+ shift from cells → ECF, alkalosis is caused by shift of H + into cells to
maintain neutrality

Respiratory acidosis

 Rise in carbon dioxide levels usually as a result of alveolar hypoventilation


 Renal compensation may occur leading to Compensated respiratory acidosis

Causes

 COPD
 Decompensation in other respiratory conditions e.g. Life-threatening asthma / pulmonary
oedema
 Sedative drugs: benzodiazepines, opiate overdose

Respiratory alkalosis

 Hyperventilation resulting in excess loss of carbon dioxide


 This will result in increasing pH

Causes

 Psychogenic: anxiety leading to hyperventilation


 Hypoxia causing a subsequent hyperventilation: pulmonary embolism, high altitude
 Early salicylate poisoning*
 CNS stimulation: stroke, subarachnoid haemorrhage, encephalitis
 Pregnancy

*Salicylate overdose leads to a mixed respiratory alkalosis and metabolic acidosis. Early stimulation
of the respiratory centre leads to a respiratory alkalosis whilst later the direct acid effects of
salicylates (combined with acute renal failure) may lead to an acidosis
Next question
Theme: Intravenous fluids

A. Gelofusine
B. Dextran 70
C. 0.4%/0.18% dextrose saline
D. 5% dextrose
E. 1000ml 3% sodium chloride solution
F. Hartmann's solution
G. Pentastarch
H. Fluid restriction 500mls
I. Fluid restriction 1L
J. 0.9% sodium chloride with 40mmol potassium chloride per litre

Which of the following is the best fluid management for the scenario given? Each option may be
used once, more than once or not at all.

56. A 53 year old alcoholic male presents with acute pancreatitis. He is clinically dehydrated.
His blood results show normal renal function and electrolytes.

You answered Gelofusine

The correct answer is Hartmann's solution

This patient needs fluid replacement due to large third space losses. Hartmann's solution is
recommended. N. Saline would put this patient at risk of hyperchloraemic acidosis.

57. A 45 year old man with previous laparotomy is admitted with adhesional small bowel
obstruction. He is managed with prolonged nasogastric drainage. His U+E's are as follows:
Sodium 129

Potassium 3.4

Urea 8.4

Creatinine 89

You answered Gelofusine

The correct answer is 0.9% sodium chloride with 40mmol potassium chloride per
litre

The potassium will decline further if this deficiency is not addressed. Remember that
potassium is predominantly an intracellular cation. U+E's measure the serum potassium
which is relatively buffered by the intra cellular stores. Therefore a fall in serum potassium
represents a very real intracellular deficiency. This requires supplementary potassium to
correct the defect.

58. A 19 year old lady is admitted with pyelonephritis. She is in septic shock with a blood
pressure of 95/60 and pulse rate of 110. Apart from an allergy to corn she has no other
significant medical history.

You answered Gelofusine

The correct answer is Hartmann's solution

A 2012 Cochrane review concluded that colloids and crystalloids had the same effects on
outcome. Since crystalloids were more cost effective, these are now the preferred agents.
Hypertonic saline (3% sodium chloride) is rarely used in clinical practice and almost never for the
purposes of volume replacement.

Please rate this question:

Discuss and give feedback


Next question

Pre operative fluid management

Fluid management has been described in the British Consensus guidelines on IV fluid
therapy for Adult Surgical patients (GIFTASUP) and by NICE (CG174 December 2013)

The Recommendations include:

 Use Ringer's lactate or Hartmann's when a crystalloid is needed for resuscitation or


replacement of fluids. Avoid 0.9% N. Saline (due to risk of hyperchloraemic acidosis) unless
patient vomiting or has gastric drainage.
 Use 4%/0.18% dextrose saline or 5% dextrose in maintenance fluids. It should not be used
in resuscitation or as replacement fluids.
 Adult maintenance fluid requirements are: Na 50-100 mmol/day and K 40-80 mmol/day in
1.5-2.5L fluid per day.
 Patients for elective surgery should NOT be nil by mouth for >2 hours (unless has disorder of
gastric emptying).
 Patients for elective surgery should be given carbohydrate rich drinks 2-3h before. Ideally
this should form part of a normal pre op plan to facilitate recovery.
 Avoid mechanical bowel preparation.
 If bowel prep is used, simultaneous administration of Hartmann's or Ringer's lactate should
be considered.
 Excessive fluid losses from vomiting should be treated with a crystalloid with potassium
replacement. 0.9% N. Saline should be given if there is hypochloraemia. Otherwise
Hartmann's or Ringer lactate should be given for diarrhoea/ileostomy/ileus/obstruction.
Hartmann's should also be given in sodium losses secondary to diuretics.
 High risk patients should receive fluids and inotropes.
 An attempt should be made to detect pre or operative hypovolaemia using flow based
measurements. If this is not available, then clinical evaluation is needed i.e. JVP, pulse
volume etc.
 In Blood loss or infection causing hypovolaemia should be treated with a balanced crystalloid
or colloid (or until blood available in blood loss). A critically ill patient is unable to excrete Na
or H20 leading to a 5% risk of interstitial oedema. Therefore 5% dextrose as well as colloid
should be given.
 If patients need IV fluid resuscitationn, use crystalloids that contain sodium in the range 130-
154 mmol/l, with a bolus of 500 ml over less than 15 minutes (NICE Guidance CG 174).

Next question
Theme: Thromboprophylaxis

A. Oral dabigatran alone


B. Oral dabigatran with compression stockings
C. Low molecular weight heparin and compression stockings
D. Warfarin
E. Low molecular weight heparin and pneumatic compression stockings
F. Low molecular weight heparin alone
G. No thromboprophylaxis
H. Unfractionated heparin and compression stockings
I. Unfractionated heparin alone
J. Unfractionated heparin and pneumatic compression stockings

Please select the most appropriate thromboprophylactic regime in the surgical scenarios described
below. Each regime may be used once, more than once or not at all.

59. A 30 year old male is admitted electively for a right inguinal hernia repair under local
anaesthesia. He is otherwise well but his grandfather died from a pulmonary embolism.

You answered Oral dabigatran alone

The correct answer is No thromboprophylaxis

Inguinal hernia repairs under local anaesthetic have a short operative time and patients are
usually ambulant immediately afterwards. His family history is unlikely to be significant
and he is at very low risk.

60. A 5 year old boy undergoes a closure of a loop colostomy.

You answered Oral dabigatran alone

The correct answer is No thromboprophylaxis

In paediatric surgical practice the use of heparin type agents is rare. This is because, even
with abdominal surgery, children are ambulant soon after surgery and DVT's vanishingly
rare in this population.

61. An 83 year old man is admitted for an abdomino-perineal excision of the colon and rectum
for a distal rectal tumour. His co-mobidities include diabetes and intermittent claudication.
His renal function is normal.

You answered Oral dabigatran alone


The correct answer is Low molecular weight heparin and pneumatic compression
stockings

Pelvic cancer surgery carries a very high risk of development of deep vein thrombosis. In a
patient with normal renal function the use of a low molecular weight heparin is standard.
However, many surgeons would only use this in the post operative setting. Intermittent
compression devices in claudicants are not without risk, but on balance probably outweigh
the risk of DVT in this specific case. The perfusion of the feet should be closely monitored
and compression stopped if concerns develop.

Please rate this question:

Discuss and give feedback


Next question

Thromboprophylaxis in surgical patients

Deep vein thrombosis may develop insidiously in many surgical patients. Untreated it may progress
to result in pulmonary embolism.
The following surgical patients are at increased risk of deep vein thrombosis:

 Surgery greater than 90 minutes at any site or greater than 60 minutes if the procedure
involves the lower limbs or pelvis
 Acute admissions with inflammatory process involving the abdominal cavity
 Expected significant reduction in mobility
 Age over 60 years
 Known malignancy
 Thrombophilia
 Previous thrombosis
 BMI >30
 Taking hormone replacement therapy or the contraceptive pill
 Varicose veins with phlebitis

Mechanical thromboprophylaxis

 Early ambulation after surgery is cheap and is effective


 Compression stockings (contra -indicated in peripheral arterial disease)
 Intermittent pneumatic compression devices
 Foot impulse devices

Therapeutic agents
Agent Mode of action Uses

Low molecular Binds antithrombin Thromboprophylaxis or treatment of thromboembolic


weight heparin causing inhibition events in those with normal renal function. It is given as
of factor Xa once daily subcutaneous injection

Unfractionated Binds antithrombin Effective anticoagulation, administered intravenously it


heparin III affecting has a rapid onset and its therapeutic effects decline
thrombin and factor quickly on stopping and infusion. Its activity is
Xa measured using the APTT. If need be it can be reversed
using protamine sulphate

Dabigatran Orally administered Used prophylaxis in hip and knee surgery. It does not
direct thrombin require therapeutic monitoring. It has no known antidote
inhibitor and should not be used in any patient in whom there is a
risk of active bleeding or imminent likelihood of
surgery

Next question
Theme: Management of complications

A. Gastrograffin contrast enema


B. Barium enema
C. Oral gastrograffin and CT
D. Barium meal
E. Ultrasound of the thorax
F. Endoanal ultrasound
G. Anorectal physiology studies
H. Biofeedback
I. Abdominal CT scan with IV contrast

Please select the most appropriate intervention from the list given. Each option may be used once,
more than once or not at all.

62. A 65 year old male with carcinoma of the oesophagus undergoes endoscopic dilatation.
Following which he develops pleuritic chest pain and sub cutaneous emphysema.

You answered Gastrograffin contrast enema

The correct answer is Oral gastrograffin and CT

This is consistent with oesophageal perforation. Gastrograffin and CT will accurately


delineate the site of perforation and guide further therapy. Barium may produce a
mediastinitis and should not be used.

63. A frail 73 year old lady is admitted with intractable faecal incontinence. She undergoes a
laparoscopic defunctioning of the rectum with an end colostomy. 48 hours later her stoma
has still not worked and her abdomen is distended and painful.

Gastrograffin contrast enema

Occasionally the wrong end of bowel is brought up and fashioned as the end stoma,
effectively leaving the bowel obstructed. A gastrograffin enema will easily demonstrate if
this is the case.

64. A 43 year old man develops fast atrial fibrillation 5 days following a low anterior resection
of the rectum for cancer. On examination he has lower abdominal tenderness and a WCC
19.

You answered Gastrograffin contrast enema

The correct answer is Abdominal CT scan with IV contrast


An anastomotic leak is the most likely occurrence and may be visualised using CT
scanning. A gastrograffin enema may demonstrate the leak but a CT scan will also provide
information to guide management.

Please rate this question:

Discuss and give feedback


Next question

Surgical complications

Complications occur in all branches of surgery and require vigilance in their detection. In many
cases anticipating the likely complications and appropriate avoidance will minimise their occurrence.
For the purposes of the MRCS the important principles to appreciate are:

 The anatomical principles that underpin complications


 The physiological and biochemical derangements that occur
 The most appropriate diagnostic modalities to utilise
 The principles which underpin their management

This is clearly a very broad area and impossible to cover comprehensively. There is considerable
overlap with other topic areas within the website.

Avoiding complications

Some points to hopefully avert complications:

 World Health Organisation checklist- now mandatory prior to all operations


 Prophylactic antibiotics - right dose, right drug, right time.
 Assess DVT/ PE risk and ensure adequate prophylaxis
 MARK site of surgery
 Use tourniquets with caution and with respect for underlying structures
 Remember the danger of end arteries and in situations where they occur avoid using
adrenaline containing solutions and monopolar diathermy.
 Handle tissues with care- devitalised tissue serves as a nidus for infection
 Be very wary of the potential for coupling injuries when using diathermy during laparoscopic
surgery
 The inferior epigastric artery is a favorite target for laparoscopic ports and surgical drains!

Anatomical principles
Understanding the anatomy of a surgical field will allow appreciation of local and systemic
complications that may occur. For example, nerve injuries may occur following surgery in specific
regions. The table below lists some of the more important nerves to consider and mechanisms of
injury

Nerve Mechanism

Accessory Posterior triangle lymph node biopsy

Sciatic Posterior approach to hip

Common peroneal Legs in Lloyd Davies position

Long thoracic Axillary node clearance

Pelvic autonomic nerves Pelvic cancer surgery

Recurrent laryngeal nerves During thyroid surgery

Hypoglossal nerve During carotid endarterectomy

Ulnar and median nerves During upper limb fracture repairs

These are just a few. The detailed functional sequelae are particularly important and will often be
tested. In addition to nerve injuries certain procedures carry risks of visceral or structural injury.
Again some particular favorites are given below:

Structure Mechanism

Thoracic duct During thoracic surgery e.g. Pneumonectomy, oesphagectomy

Parathyroid glands During difficult thyroid surgery


Structure Mechanism

Ureters During colonic resections/ gynaecological surgery

Bowel perforation Use of Verres Needle to establish pneumoperitoneum

Bile duct injury Failure to delineate Calots triangle carefully and careless use of diathermy

Facial nerve Always at risk during Parotidectomy

Tail of pancreas When ligating splenic hilum

Testicular vessels During re-do open hernia surgery

Hepatic veins During liver mobilisation

Again many could be predicted from the anatomy of the procedure.

Physiological derangements
A very common complication is bleeding and this is covered under the section of haemorrhagic
shock. Another variant is infection either superficial or deep seated. The organisms are covered
under microbiology and the features of sepsis covered under shock. Do not forget that
immunocompromised and elderly patients may present will atypical physiological parameters.

Selected physiological and biochemical issues are given below:

Complication Physiological/ Biochemical Problem

Arrhythmias following Susceptibility to hypokalaemia (K+ <4.0 in cardiac patients)


cardiac surgery

Neurosurgical electrolyte SIADH following cranial surgery causing hyponatraemia


disturbance
Complication Physiological/ Biochemical Problem

Ileus following Fluid sequestration and loss of electrolytes


gastrointestinal surgery

Pulmonary oedema following Loss of lung volume makes these patients very sensitive to fluid
pneumonectomy overload

Anastamotic leak Generalised sepsis causing mediastinitis or peritonitis depending


on site of leak

Myocardial infarct May follow any type of surgery and in addition to direct cardiac
effects the decreased cardiac output may well compromise grafts
etc.

Try making a short list of problems and causes specific to your own clinical area.

Diagnostic modalities
Depends largely on the suspected complication. In the acutely unwell surgical patient the following
baseline investigations are often helpful:

 Full blood count, urea and electrolytes, C- reactive protein (trend rather than absolute value),
serum calcium, liver function tests, clotting (don't forget to repeat if on-going bleeding)
 Arterial blood gases
 ECG (+cardiac enzymes if MI suspected)
 Chest x-ray to identify collapse/ consolidation
 Urine analysis for UTI

These will often identify the most common complications.

Special tests

 CT scanning for identification of intra-abdominal abscesses, air and if luminal contrast is


used an anastomotic leak
 Doppler USS of leg veins- for identification of DVT
 CTPA for PE
 Sending peritoneal fluid for U+E (if ureteric injury suspected) or amylase (if pancreatic injury
suspected)
 Echocardiogram if pericardial effusion suspected post cardiac surgery and no pleural window
made.
Management of complications
The guiding principal should be safe and timely intervention. Patients should be stabilised and if an
operation needs to occur in tandem with resuscitation then generally this should be of a damage
limitation type procedure rather than definitive surgery (which can be more safely undertaken in a
stable patient the following day).

Remember that recent surgery is a contra indication to thrombolysis and that in some patients IV
heparin may be preferable to a low molecular weight heparin (easier to reverse).

As a general rule laparotomies for bleeding should follow the core principle of quadrant packing and
then subsequent pack removal rather than plunging large clamps into pools of blood. The latter
approach invariable worsens the situation is often accompanied by significant visceral injury
particularly when done by the inexperienced. If packing controls a situation it is entirely acceptable
practice to leave packs in situ and return the patient to ITU for pack removal the subsequent day.
Next question
A 63 year old man is recovering following an open extended right hemicolectomy for carcinoma of
the colonic splenic flexure. Two days post operatively he develops a persistent pyrexia. What is the
least likely cause?

Ileus

Atelectasis

Anastomotic leak

Wound infection

Urinary tract infection

An ileus in itself is seldom a cause of a pyrexia. It may serve as a proxy marker of other
complications. In this scenario atelectasis would be the most likely underlying cause, as open
extended right hemicolectomies will necessitate a long midline incision. Anastomotic leaks are less
common after right sided colonic surgery and the timeframe for it is rather short (but are possible).
Both wound infections and UTI's ,may complicate major abdominal surgery at any stage. We remind
you to check the wording of the question, it asks for the "least likely" cause of pyrexia.
Please rate this question:

Discuss and give feedback


Next question

Pyrexia- post operative

Many surgical patients will develop a pyrexia post operatively. The cause and investigation depends
upon the nature of the infection.
The following scenarios may account for post operative pyrexia:
Cause Features

Anastomotic leak  Swinging pyrexia


 Ileus
 Increasing abdominal pain
 Raised inflammatory markers

Wound infection  Evidence of superficial erythema, discharge of pus or increasing


pain
 Usually mild pyrexia (unless major or deep seated wound infection)
 May be accompanied by evidence of wound dehisence
 Inflammatory markers raised

Atelectasis  Usually complicates abdominal surgery


 Most common after midline laparotomies (pain impairs ventilation)
 Pyrexia usually mild and non swinging
 Most patients will have chest signs on examination
 Inflammatory markers raised

Central line sepsis  Patients with complex venous access


 May have marked pyrexia
 Access site may show evidence of erythema
 Diagnosis is by blood culture from line, line removal and subsequent
tip culture
 Groin lines and those for TPN have the highest risk
 Inflammatory markers raised

Urinary tract  Common in surgical patients


infection  Usually occur in patients with indwelling urinary catheters
 Diagnosis is by dipstick and CSU and signs of raised inflammatory
markers
 Treatment is with antibiotics (to cover hospital acquired organisms)

Next question
A 72 year old man is due to undergo an oesophagectomy for malignancy. His BMI is 17.5. What is
the best feeding regime immediately following surgery?

Total parenteral nutrition.

Feeding jejunostomy.

Feeding duodenostomy.

Liquid diet orally.

Soft solids orally.

Theme from April 2012 Exam


This patient has a condition causing poor absorption, loss of nutrients and high metabolism. Enteral
feeds should be used where possible and many surgeons will site a jejunostomy for this purpose.
Oral diet is not permitted following a resection until the anastamosis has had time to heal.
Please rate this question:

Discuss and give feedback


Next question

Enteral Feeding

 Identify patients as malnourished or at risk (see below)


 Identify unsafe or inadequate oral intake with functional GI tract
 Consider for enteral feeding
 Gastric feeding unless upper GI dysfunction (then for duodenal or jejunal tube)
 Check NG placement using aspiration and pH (check post pyloric tubes with AXR)
 Gastric feeding > 4 weeks consider long-term gastrostomy
 Consider bolus or continuous feeding into the stomach
 ITU patients should have continuous feeding for 16-24h (24h if on insulin)
 Consider motility agent in ITU or acute patients for delayed gastric emptying. If this doesn't
work then try post pyloric feeding or parenteral feeding.
 PEG can be used 4 hours after insertion, but should not be removed until >2 weeks after
insertion.

Surgical patients due to have major abdominal surgery: if malnourished, unsafe swallow/inadequate
oral intake and functional GI tract then consider pre operative enteral feeding.
Patients identified as being malnourished

 BMI < 18.5 kg/m2


 unintentional weight loss of > 10% over 3-6/12
 BMI < 20 kg/m 2 and unintentional weight loss of > 5% over 3-6/12

AT RISK of malnutrition

 Eaten nothing or little > 5 days, who are likely to eat little for a further 5 days
 Poor absorptive capacity
 High nutrient losses
 High metabolism

Reference
Stroud M et al. Guidelines for enteral feeding in adult hospital patients. Gut 2003; 52(Suppl VII):vii1 -
vii12.
Next question
A 38 year old lady presents with abdominal pain. On investigation, her serum calcium is found to be
3.5mmol/L. What is the most appropriate initial management?

Intravenous bisphosphonates

Oral bisphosphonates

Intravenous calcitonin

Intravenous 0.9% sodium chloride

Intravenous frusemide

The immediate treatment of hypercalcaemia involves intravenous fluid resuscitation. This may be
complemented with the use of bisphosphonates and sometimes diuretics. However, fluids are
administered first and because this question asks what the most appropriate initial treatment is,
intravenous fluids are the most appropriate answer. Normal saline is usually preferred for this over
dextrose containing solutions.
Please rate this question:

Discuss and give feedback


Next question

Management of hypercalcaemia

 Free Ca is affected by pH (increased in acidosis) and plasma albumin concentration


 ECG changes include: Shortening of QTc interval
 Urgent management is indicated if:

Calcium > 3.5 mmol/l

Reduced consciousness

Severe abdominal pain

Pre renal failure

Management:
 Airway Breathing Circulation
 Intravenous fluid resuscitation with 3-6L of 0.9% Normal saline in 24 hours
 Concurrent administration of calcitonin will also help lower calcium levels
 Medical therapy (usually if Corrected calcium >3.0mmol/l)

Bisphosphonates

 Analogues of pryrophosphate
 Prevent osteoclast attachment to bone matrix and interfere with osteoclast activity
 Inhibit bone resorption.

Agents

Drug Side effects Notes

IV Pamidronate pyrexia, leucopaenia Most potent agent

IV Zoledronate response lasts 30 days Used for malignancy associated hypercalcaemia

Calcitonin

 Quickest onset of action however short duration (tachyphylaxis) therefore only given with a
second agent.

Prednisolone

 May be given in hypercalcaemia related to sarcoidosis, myeloma or vitamin D intoxication.

Next question
Theme: Bowel preparation

A. No preparation required
B. Phosphate enema
C. Bowel preparation with oral laxatives (e.g. picolax)
D. Senokot tablets
E. Oral pergolide
F. Rectal lavage with saline
G. 60 ml oral lactulose

Please select the most appropriate form of bowel preparation for the procedures given. Each agent
may be selected once, more than once or not at all.

68. A 56 year old man with a change in bowel habit requires a colonoscopy.

You answered No preparation required

The correct answer is Bowel preparation with oral laxatives (e.g. picolax)

Endoscopy requires full bowel preparation. In elderly patients, this can cause electrolyte
disturbance and renal compromise and it is important to check the patients urea and
electrolytes beforehand.

69. A 44 year old man with carcinoma of the hepatic flexure requires a right hemicolectomy.

No preparation required

Formal bowel preparation for right sided colonic resection is unnecessary. The formal
bowel preparation of elective patients for right sided resection results in increased post
operative morbidity and delayed discharge.

70. A 34 year old man with rectal bleeding requires a flexible sigmoidoscopy.

You answered No preparation required

The correct answer is Phosphate enema

For a limited endoscopy a simple enema will suffice. .

Please rate this question:

Discuss and give feedback


Next question

Preparation for surgery

Elective and emergency patients require different preparation.

Elective cases

 Consider pre admission clinic to address medical issues.


 Blood tests including FBC, U+E, LFT's, Clotting, Group and Save
 Urine analysis
 Pregnancy test
 Sickle cell test
 ECG/ Chest x-ray

Exact tests to be performed will depend upon the proposed procedure and patient fitness.

Risk factors for development of deep vein thrombosis should be assessed and a plan for
thromboprophylaxis formulated.

Diabetes
Diabetic patients have greater risk of complications.
Poorly controlled diabetes carries high risk of wound infections.
Patients with diet or tablet controlled diabetes may be managed using a policy of omitting medication
and checking blood glucose levels regularly. Diabetics who are poorly controlled or who take insulin
will require a intravenous sliding scale. Potassium supplementation should also be given.
Diabetic cases should be operated on first.

Emergency cases
Stabilise and resuscitate where needed.
Consider whether antibiotics are needed and when and how they should be administered.
Inform blood bank if major procedures planned particularly where coagulopathies are present at the
outset or anticipated (e.g. Ruptured AAA repair)
Don't forget to consent and inform relatives.

Special preparation
Some procedures require special preparation:

 Thyroid surgery; vocal cord check.


 Parathyroid surgery; consider methylene blue to identify gland.
 Sentinel node biopsy; radioactive marker/ patent blue dye.
 Surgery involving the thoracic duct; consider administration of cream.
 Pheochromocytoma surgery; will need alpha and beta blockade.
 Surgery for carcinoid tumours; will need covering with octreotide.
 Colorectal cases; bowel preparation (especially left sided surgery)
 Thyrotoxicosis; lugols iodine/ medical therapy.
The following are contra indications to the use of lignocaine for local anaesthesia except:

Accelerated idioventricular rhythm

Current treatment with flecainide

3rd degree heart block without pacemaker

Severe sino atrial block

Protein C deficiency

Lignocaine is widely used as a local anaesthetic. As a class IB antiarrhythmic it should not be used
in people with unstable disorders of cardiac rhythm and ideally should not be co-administered with
other anti-arhythmics.
Please rate this question:

Discuss and give feedback


Next question

Local anaesthetic agents

Lidocaine

 An amide
 Local anaesthetic and a less commonly used antiarrhythmic (affects Na channels in the
axon)
 Hepatic metabolism, protein bound, renally excreted
 Toxicity: due to IV or excess administration. Increased risk if liver dysfunction or low protein
states. Note acidosis causes lidocaine to detach from protein binding.
 Drug interactions: Beta blockers, ciprofloxacin, phenytoin
 Features of toxicity: Initial CNS over activity then depression as lidocaine initially blocks
inhibitory pathways then blocks both inhibitory and activating pathways. Cardiac arrhythmias.
 Increased doses may be used when combined with adrenaline to limit systemic absorption.

Cocaine

 Pure cocaine is a salt, usually cocaine hydrochloride. It is supplied for local anaesthetic
purposes as a paste.
 It is supplied for clinical use in concentrations of 4 and 10%. It may be applied topically to the
nasal mucosa. It has a rapid onset of action and has the additional advantage of causing
marked vasoconstriction.
 It is lipophillic and will readily cross the blood brain barrier. Its systemic effects also include
cardiac arrhythmias and tachycardia.
 Apart from its limited use in ENT surgery it is otherwise used rarely in mainstream surgical
practice.

Bupivacaine

 Bupivacaine binds to the intracellular portion of sodium channels and blocks sodium influx
into nerve cells, which prevents depolarization.
 It has a much longer duration of action than lignocaine and this is of use in that it may be
used for topical wound infiltration at the conclusion of surgical procedures with long duration
analgesic effect.
 It is cardiotoxic and is therefore contra indicated in regional blockage in case the tourniquet
fails.
 Levobupivicaine (Chirocaine) is less cardiotoxic and causes less vasodilation.

Prilocaine

 Similar mechanism of action to other local anaesthetic agents. However, it is far less
cardiotoxic and is therefore the agent of choice for intravenous regional anaesthesia e.g.
Biers Block.

All local anaesthetic agents dissociate in tissues and this contributes to their therapeutic effect. The
dissociation constant shifts in tissues that are acidic e.g. where an abscess is present, and this
reduces the efficacy.

Doses of local anaesthetics


Agent Dose plain Dose with adrenaline

Lignocaine 3mg/Kg 7mg/Kg

Bupivacaine 2mg/Kg 2mg/Kg

Prilocaine 6mg/Kg 9mg/Kg

These are a guide only as actual doses depend on site of administration, tissue vascularity and co-
morbidities.
Maximum total local anaesthetic doses

 Lignocaine 1% plain - 3mg/ Kg - 200mg (20ml)


 Lignocaine 1% with 1 in 200,000 adrenaline - 7mg/Kg - 500mg (50ml)
 Bupivicaine 0.5% - 2mg/kg- 150mg (30ml)

Maximum doses are based on ideal body weight

Effects of adrenaline
Adrenaline may be added to local anaesthetic drugs. It prolongs the duration of action at the site of
injection and permits usage of higher doses (see above). It is contra indicated in patients taking
MAOI's or tricyclic antidepressants. The toxicity of bupivacaine is related to protein binding and
addition of adrenaline to this drug does not permit increases in the total dose of bupivacaine, in
contrast to the situation with lignocaine.

References
An excellent review is provided by:
French J and Sharp L. Local Anaesthetics. Ann R Coll Surg Engl 2012; 94: 76-80.
Next question
A 43 year old lady has undergone a total thyroidectomy for multinodular goitre. You are called to see
her because of respiratory distress. On examination she has a marked stridor, her wound seems
healthy but there is a swelling within the operative site. What is the most likely explanation for this
problem?

Bilateral superior laryngeal nerve injury

Hypocalcaemic tetany

Anxiety

Contained haematoma

Unilateral recurrent laryngeal nerve injury

In this setting a contained haematoma is the most likely cause. This will impair venous return
resulting in laryngeal oedema and respiratory compromise.
Please rate this question:

Discuss and give feedback


Next question

Thyroid disease

Patients may present with a number of different manifestations of thyroid disease. They can be
broadly sub classified according to whether they are euthyroid or have clinical signs of thyroid
dysfunction. In addition it needs to be established whether they have a mass or not.

Assessment

 History
 Examination including USS
 If a nodule is identified then it should be sampled ideally via an image guided fine needle
aspiration
 Radionucleotide scanning is of limited use

Thyroid Tumours

 Papillary carcinoma
 Follicular carcinoma
 Anaplastic carcinoma
 Medullary carcinoma
 Lymphoma's

Multinodular goitre

 One of the most common reasons for presentation


 Provided the patient is euthyroid and asymptomatic and no discrete nodules are seen, they
can be reassured.
 In those with compressive symptoms surgery is required and the best operation is a total
thyroidectomy.
 Sub total resections were practised in the past and simply result in recurrent disease that
requires a difficult revisional resection.

Endocrine dysfunction

 In general these patients are managed by physicians initially.


 Surgery may be offered alongside radio iodine for patients with Graves disease that fails with
medical management or in patients who would prefer not to be irradiated (e.g. pregnant
women).
 Patients with hypothyroidism do not generally get offered a thyroidectomy. Sometimes
people inadvertently get offered resections during the early phase of Hashimotos thyroiditis,
however, with time the toxic phase passes and patients can simply be managed with
thyroxine.

Complications following surgery

 Anatomical such as recurrent laryngeal nerve damage.


 Bleeding. Owing to the confined space haematoma's may rapidly lead to respiratory
compromise owing to laryngeal oedema.
 Damage to the parathyroid glands resulting in hypocalcaemia.

Further sources of information


1. http://www.acb.org.uk/docs/TFTguidelinefinal.pdf- Association of Clinical Biochemistry guidelines
for thyroid function tests.

2. British association of endocrine surgeons website- http://www.baets.org.uk


Next question
Which of the following drugs is not positively inotropic?

Dopamine

Glucagon

Theophylline

Sodium thiopentone

Dobutamine

Inotropes are a class of drugs that increase the force or cardiac contractility. This may improve
cardiac output. Increased blood pressure may have direct beneficial effects for the heart in that it
improves myocardial perfusion pressure. Dopamine and dobutamine are both commonly used
inotropes, they should be administered via a central line and in a monitored setting. Glucagon and
theophylline are also positive inotropes (although not commonly used for this purpose). In contrast
sodium thiopentone causes marked myocardial depression.

Please rate this question:

Discuss and give feedback

Next question

Inotropes and cardiovascular receptors

Inotropes are a class of drugs which work primarily by increasing cardiac output. They should be
distinguished from vasoconstrictor drugs which are used specifically when the primary problem is
peripheral vasodilatation.

Catecholamine type agents are commonly used and work by increasing cAMP levels by adenylate
cyclase stimulation. This in turn intracellular calcium ion mobilisation and thus the force of
contraction. Adrenaline works as a beta adrenergic receptor agonist at lower doses and an alpha
receptor agonist at higher doses. Dopamine causes dopamine receptor mediated renal and
mesenteric vascular dilatation and beta 1 receptor agonism at higher doses. This results in
increased cardiac output. Since both heart rate and blood pressure are raised, there is less overall
myocardial ischaemia. Dobutamine is a predominantly beta 1 receptor agonist with weak beta 2 and
alpha receptor agonist properties. Noradrenaline is a catecholamine type agent and predominantly
acts as an alpha receptor agonist and serves as a peripheral vasoconstrictor.

Phosphodiesterase inhibitors such as milrinone act specifically on the cardiac phosphodiesterase


and increase cardiac output.

Inotrope Cardiovascular receptor action

Adrenaline α-1, α-2, β-1, β-2

Noradrenaline α-1,( α-2), (β-1), (β-2)

Dobutamine β-1, (β 2)

Dopamine (α-1), (α-2), (β-1), D-1,D-2

Minor receptor effects in brackets

Effects of receptor binding

α-1, α-2 vasoconstriction

β-1 increased cardiac contractility and HR

β-2 vasodilatation

D-1 renal and spleen vasodilatation

D-2 inhibits release of noradrenaline

Next question
A 28 year old man undergoes a laparotomy for perforated duodenal ulcer and broad spectrum
antibiotics are administered. Post operatively he has hearing impairment. Which of the following
agents is the most likely underlying culprit?

Gentamicin

Ciprofloxacin

Metronidazole

Ampicillin

Co-trimoxazole

Ototoxicity is a recognised adverse reaction with the aminoglycoside antibiotics.


Please rate this question:

Discuss and give feedback


Next question

Antibiotics: mechanism of action

The lists below summarise the site of action of the commonly used antibiotics

Inhibit cell wall formation

 penicillins
 cephalosporins

Inhibit protein synthesis

 aminoglycosides (cause misreading of mRNA)


 chloramphenicol
 macrolides (e.g. erythromycin)
 tetracyclines
 fusidic acid
Inhibit DNA synthesis

 quinolones (e.g. ciprofloxacin)


 metronidazole
 sulphonamides
 trimethoprim

Inhibit RNA synthesis

 rifampicin

Next question
A 62 year old lawyer has a transurethral resection of the prostate which took 1 hour to perform. The
ST2 contacts you as the patient has become agitated. He has a HR 105 bpm and his blood pressure
is 170/100 mmHg. He is fluid overloaded. His blood results reveal a Na of 120mmol/l. What is the
most likely cause?

Over administration of 0.9% Normal Saline

Syndrome of inappropriate antidiuretic hormone secretion

Congestive cardiac failure

TUR syndrome

Acute renal failure

Complications of Transurethral Resection: TURP

T ur syndrome
U rethral stricture/UTI
R etrograde ejaculation
P erforation of the prostate

TUR syndrome occurs when irrigation fluid enters the systemic circulation. The triad of features are:

1. Hyponatraemia: dilutional
2. Fluid overload
3. Glycine toxicity

Management involves fluid restriction and the treatment of the complications associated with the
hyponatraemia.

Please rate this question:

Discuss and give feedback


Post prostatectomy syndromes

Transurethral prostatectomy is a common and popular treatment for benign prostatic hyperplasia.
The procedure involves insertion of a resectoscope via the penile urethra. The bladder and prostate
are irrigated and strips of prostatic tissue removed using diathermy.

Indications for surgery in patients with BPH


Refractory urinary retention
Recurrent urinary tract infections due to prostatic hypertrophy
Recurrent gross haematuria
Renal insufficiency secondary to bladder outlet obstruction
Permanently damaged or weakened bladders
Large bladder diverticula that do not empty well secondary to an enlarged prostate

Complications include haemorrhage, urosepsis, retrograde ejaculation and electrolyte disturbances


from the irrigation fluids used during surgery.

Risk factors for increased morbidity following TURP


Glands >45g
Operating time > 90 minutes
Acute urinary retention as presenting feature
A 44 year old man undergoes a distal gastrectomy for cancer. He is slightly anaemic and therefore
receives a transfusion of 4 units of packed red cells to cover both the existing anaemia and
associated perioperative blood loss. He is noted to develop ECG changes that are not consistent
with ischaemia. What is the most likely cause?

Hyponatraemia

Hyperkalaemia

Hypercalcaemia

Metabolic alkalosis

Hypernatraemia

The transfusion of packed red cells has been shown to increase serum potassium levels. The risk is
higher with large volume transfusions and with old blood.
Please rate this question:

Discuss and give feedback


Next question

Blood transfusion reactions

Acute transfusion reactions present as adverse signs or symptoms during or within 24 hours of a
blood transfusion. The most frequent reactions are fever, chills, pruritus, or urticaria, which typically
resolve promptly without specific treatment or complications. Other signs occurring in temporal
relationship with a blood transfusion, such as severe dyspnoea, pyrexia, or loss of consciousness
may be the first indication of a more severe potentially fatal reaction.
The causes of adverse reactions are multi-factorial. Immune mediated reactions, some of the most
feared, occur as a result of component mismatch, the commonest cause of which is clerical error.
More common, non immune mediated, complications may occur as a result of product
contamination, this may be bacterial or viral.
Transfusion related lung injury is well recognised and there are two proposed mechanisms which
underpin this. One involves the sequestration of primed neutrophils within the recipient pulmonary
capillary bed. The other proposed mechanism suggests that HLA mismatches between donor
neutrophils and recipient lung tissue is to blame.
The table below summarises the main types of transfusion reaction.

Immune mediated Non immune mediated


Immune mediated Non immune mediated

Pyrexia Hypocalcaemia

Alloimmunization CCF

Thrombocytopaenia Infections

Transfusion associated lung injury Hyperkalaemia

Graft vs Host disease

Urticaria

Acute or delayed haemolysis

ABO incompatibility

Rhesus incompatibility

Next question
Which of the following muscle relaxants will tend to incite neuromuscular excitability following
administration?

Atracurium

Suxamethonium

Vecuronium

Pancuronium

None of the above

Suxamethonium may induce generalised muscular contractions following administration. This may
raise serum potassium levels.

Please rate this question:

Discuss and give feedback

Next question

Muscle relaxants

Suxamethonium  Depolarising neuromuscular blocker


 Inhibits action of acetylcholine at the neuromuscular junction
 Degraded by plasma cholinesterase and acetylcholinesterase (affected by lack
of acetylcholinesterase)
 Fastest onset and shortest duration of action of all muscle relaxants
 Produces generalised muscular contraction prior to paralysis
 Adverse effects include hyperkalaemia, malignant hyperthermia, delayed
recovery
Atracurium  Non depolarising neuromuscular blocking drug
 Duration of action usually 30-45 minutes
 Generalised histamine release on administration may produce facial flushing,
tachycardia and hypotension
 Not excreted by liver or kidney, broken down in tissues by hydrolysis
 Reversed by neostigmine

Vecuronium  Non depolarising neuromuscular blocking drug


 Duration of action approximately 30 - 40 minutes
 Degraded by liver and kidney and effects prolonged in organ dysfunction
 Effects may be reversed by neostigmine

Pancuronium  Non depolarising neuromuscular blocker


 Onset of action approximately 2-3 minutes
 Duration of action up to 2 hours
 Effects may be partially reversed with drugs such as neostigmine

Next question
A 23 year old man is recovering from an appendicectomy. The operation was complicated by the
presence of perforation. He is now recovering on the ward. However, his urine output is falling and
he has been vomiting. Which of the following intravenous fluids should be initially administered,
pending analysis of his urea and electrolyte levels?

Hartmans solution

Dextran 70

Pentastarch

Gelofusin

5% Dextrose

Theme from January 2011 Exam

He will have sequestration of electrolyte rich fluids in the abdomen and gut lumen. These are best
replaced by use of Hartmans solution in the first instance.
Please rate this question:

Discuss and give feedback


Next question

Post operative fluid management

Composition of commonly used intravenous fluids mmol-1

Na K Cl Bicarbonate Lactate

Plasma 137-147 4-5.5 95-105 22-25 -

0.9% Saline 153 - 153 - -

Dextrose / saline 30.6 - 30.6 - -


Hartmans 130 4 110 - 28

Post operative fluid management


In the UK the GIFTASUP and NICE (CG174 2013) guidelines (see reference below) were devised to
try and provide some consensus guidance as to how intravenous fluids should be administered. A
decade ago it was a commonly held belief that little harm would occur as a result of excessive
administration of normal saline and many oliguric post operative patients received enormous
quantities of IV fluids. As a result they developed hyperchloraemic acidosis. With greater
understanding of this potential complication, the use of electrolyte balanced solutions (Ringers
lactate/ Hartmans) is now favored over normal saline.
The other guidance includes:

 Fluids given should be documented clearly and easily available


 Assess the patient's fluid status when they leave theatre
 If a patient is haemodynamically stable and euvolaemic, aim to restart oral fluid intake as
soon as possible
 Review patients whose urinary sodium is < 20
 If a patient is oedematous, hypovolaemia if present should be treated first. This should then
be followed by a negative balance of sodium and water, monitored using urine Na excretion
levels
 Solutions such as Dextran 70 should be used in caution in patients with sepsis as there is a
risk of developing acute renal injury

References
NICE guidance CG174. Intravenous fluid therapy in adults. December 2013.

British Consensus Guidelines on Intravenous Fluid Therapy for Adult Surgical Patients. GIFTASUP
(2009)
Next question
A 73 year old man develops disseminated intravascular coagulation following an abdominal aortic
aneurysm repair. He receives an infusion of cryoprecipitate. What is the major constituent of this
infusion?

Factor VIII

Factor IX

Protein C

Protein S

Factor V

Please rate this question:

Discuss and give feedback


Next question

Cryoprecipitate

 Blood product made from plasma


 Usually transfused as 6 unit pool
 Indications include massive haemorrhage and uncontrolled bleeding due to haemophilia

Composition
Agent Quantity

Factor VIII 100IU

Fibrinogen 250mg

von Willebrand factor Variable


Agent Quantity

Factor XIII Variable

Next question
A 32 year old man presents to the acute surgical unit with acute pancreatitis. Over the next few days
he becomes dyspnoeic and his saturations are 89% on air. A CXR shows bilateral pulmonary
infiltrates. His CVP pressure is 16mmHg. What is the most likely diagnosis?

Cardiac failure

Pneumococcal pneumonia

Staphylococcal pneumonia

Pneumocystis carinii

Adult respiratory distress syndrome

Theme from January 2012 Exam


Theme from September 2013 Exam
Acute pancreatitis is known to precipitate ARDS. ARDS is characterised by bilateral pulmonary
infiltrates and hypoxaemia. Note that pulmonary oedema is excluded by the CVP reading <
18mmHg.
Please rate this question:

Discuss and give feedback


Next question

Adult respiratory distress syndrome

Defined as an acute condition characterized by bilateral pulmonary infiltrates and severe hypoxemia
(PaO2/FiO2 ratio < 200) in the absence of evidence for cardiogenic pulmonary oedema (clinically or
pulmonary capillary wedge pressure of less than 18 mm Hg).
In is subdivided into two stages. Early stages consist of an exudative phase of injury with associated
oedema. The later stage is one of repair and consists of fibroproliferative changes. Subsequent
scarring may result in poor lung function.

Causes

 Sepsis
 Direct lung injury
 Trauma
 Acute pancreatitis
 Long bone fracture or multiple fractures (through fat embolism)
 Head injury (causes sympathetic nervous stimulation which leads to acute pulmonary
hypertension)

Clinical features

 Acute dyspnoea and hypoxaemia hours/days after event


 Multi organ failure
 Rising ventilatory pressures

Management

 Treat the underlying cause


 Antibiotics (if signs of sepsis)
 Negative fluid balance i.e. Diuretics
 Recruitment manoeuvres such as prone ventilation, use of positive end expiratory pressure
 Mechanical ventilation strategy using low tidal volumes, as conventional tidal volumes may
cause lung injury (only treatment found to improve survival rates)

Next question
Which of the anaesthetic agents below is most likely to induce adrenal suppression?

Sodium thiopentone

Midazolam

Propofol

Etomidate

Ketamine

Etomidate is a recognised cause of adrenal suppression, this has been associated with increased
mortality when used as a sedation agent in the critically ill.
Please rate this question:

Discuss and give feedback


Next question

Anaesthetic agents

The table below summarises some of the more commonly used IV induction agents
Agent Specific features

Propofol  Rapid onset of anaesthesia


 Pain on IV injection
 Rapidly metabolised with little accumulation of metabolites
 Proven anti emetic properties
 Moderate myocardial depression
 Widely used especially for maintaining sedation on ITU, total IV
anaesthesia and for daycase surgery

Sodium  Extremely rapid onset of action making it the agent of choice for rapid
thiopentone sequence of induction
 Marked myocardial depression may occur
 Metabolites build up quickly
 Unsuitable for maintenance infusion
 Little analgesic effects

Ketamine  May be used for induction of anaesthesia


 Has moderate to strong analgesic properties
 Produces little myocardial depression making it a suitable agent for
anaesthesia in those who are haemodynamically unstable
 May induce state of dissociative anaesthesia resulting in nightmares

Etomidate  Has favorable cardiac safety profile with very little haemodynamic
instability
 No analgesic properties
 Unsuitable for maintaining sedation as prolonged (and even brief) use
may result in adrenal suppression
 Post operative vomiting is common

Next question
A 63 year old man is commenced on an infusion of packed red cells following a total hip
replacement. Which of the following adverse events is most likely?

ABO mismatching

Immune mediated intolerance of rhesus incompatible blood

Pyrexia

Jaundice

Graft versus host disease

Mild pyrexia during blood transfusion is the most common event and commonly occurs during
transfusion.

Please rate this question:

Discuss and give feedback

Next question

Blood transfusion reactions

Acute transfusion reactions present as adverse signs or symptoms during or within 24 hours of a
blood transfusion. The most frequent reactions are fever, chills, pruritus, or urticaria, which typically
resolve promptly without specific treatment or complications. Other signs occurring in temporal
relationship with a blood transfusion, such as severe dyspnoea, pyrexia, or loss of consciousness
may be the first indication of a more severe potentially fatal reaction.
The causes of adverse reactions are multi-factorial. Immune mediated reactions, some of the most
feared, occur as a result of component mismatch, the commonest cause of which is clerical error.
More common, non immune mediated, complications may occur as a result of product
contamination, this may be bacterial or viral.
Transfusion related lung injury is well recognised and there are two proposed mechanisms which
underpin this. One involves the sequestration of primed neutrophils within the recipient pulmonary
capillary bed. The other proposed mechanism suggests that HLA mismatches between donor
neutrophils and recipient lung tissue is to blame.
The table below summarises the main types of transfusion reaction.

Immune mediated Non immune mediated

Pyrexia Hypocalcaemia

Alloimmunization CCF

Thrombocytopaenia Infections

Transfusion associated lung injury Hyperkalaemia

Graft vs Host disease

Urticaria

Acute or delayed haemolysis

ABO incompatibility

Rhesus incompatibility

Next question
Theme: Diagnosis and management of post operative complications

A. Trans anal ultrasound scan


B. Upper abdominal ultrasound scan
C. Total abdominal ultrasound scan
D. PET CT scan
E. ERCP
F. Small bowel MRI Scan
G. Abdominal CT scan with oral and IV contrast
H. Non contrast abdominal CT scan
I. Laparotomy
J. Laparoscopy

For the following post operative scenarios please select the most appropriate investigation or
management. Each option may be used once, more than once or not at all.

8. A 7 year old boy develops a persistent fever following an open appendicectomy for
gangrenous appendicitis. On examination he has erythema of the wound and some
abdominal distension.

You answered Trans anal ultrasound scan

The correct answer is Total abdominal ultrasound scan

Theme from September 2013 Exam


This patient has risk factors for a wound infection which is not in itself an indication for
scanning. However, he also had abdominal distension and this, together with the history of
distension would generally attract a recommendation for imaging. A USS will show an
abdominal wall collection and more importantly any phrenic or pelvic collections. Unlike
adult practice, CT scanning is rarely performed in children.

9. A 56 year old man is 8 days following a left hemicolectomy. He has developed a swinging
pyrexia over the past 48 hours and has an ileus clinically.

You answered Trans anal ultrasound scan

The correct answer is Abdominal CT scan with oral and IV contrast

This would most likely be the result of an anastomotic leak with abscess formation.
Detailed imaging is required to allow accurate diagnosis and planning of management.

10. A 43 year old lady underwent an acute cholecystectomy for cholecystitis. A drain is left
during the procedure. Over the next 5 days the drain has been accumulating between 100-
200ml of bile per 24 hour period.
You answered Trans anal ultrasound scan

The correct answer is ERCP

The most likely cause of a bile leak in this scenario would be a dislodged clip from the
cystic duct. Whilst it may be tempting to try and plan to manage this surgically the
anatomy is often unfavorable and the duct very difficult to identify. An ERCP has the
advantage of demonstrating the cause of the leak and allowing placement of a stent. This
will usually allow the resolution of most leaks without the need for surgery.

Please rate this question:

Discuss and give feedback


Next question

Surgical complications

Complications occur in all branches of surgery and require vigilance in their detection. In many
cases anticipating the likely complications and appropriate avoidance will minimise their occurrence.
For the purposes of the MRCS the important principles to appreciate are:

 The anatomical principles that underpin complications


 The physiological and biochemical derangements that occur
 The most appropriate diagnostic modalities to utilise
 The principles which underpin their management

This is clearly a very broad area and impossible to cover comprehensively. There is considerable
overlap with other topic areas within the website.

Avoiding complications

Some points to hopefully avert complications:

 World Health Organisation checklist- now mandatory prior to all operations


 Prophylactic antibiotics - right dose, right drug, right time.
 Assess DVT/ PE risk and ensure adequate prophylaxis
 MARK site of surgery
 Use tourniquets with caution and with respect for underlying structures
 Remember the danger of end arteries and in situations where they occur avoid using
adrenaline containing solutions and monopolar diathermy.
 Handle tissues with care- devitalised tissue serves as a nidus for infection
 Be very wary of the potential for coupling injuries when using diathermy during laparoscopic
surgery
 The inferior epigastric artery is a favorite target for laparoscopic ports and surgical drains!

Anatomical principles
Understanding the anatomy of a surgical field will allow appreciation of local and systemic
complications that may occur. For example, nerve injuries may occur following surgery in specific
regions. The table below lists some of the more important nerves to consider and mechanisms of
injury

Nerve Mechanism

Accessory Posterior triangle lymph node biopsy

Sciatic Posterior approach to hip

Common peroneal Legs in Lloyd Davies position

Long thoracic Axillary node clearance

Pelvic autonomic nerves Pelvic cancer surgery

Recurrent laryngeal nerves During thyroid surgery

Hypoglossal nerve During carotid endarterectomy

Ulnar and median nerves During upper limb fracture repairs

These are just a few. The detailed functional sequelae are particularly important and will often be
tested. In addition to nerve injuries certain procedures carry risks of visceral or structural injury.
Again some particular favorites are given below:

Structure Mechanism
Structure Mechanism

Thoracic duct During thoracic surgery e.g. Pneumonectomy, oesphagectomy

Parathyroid glands During difficult thyroid surgery

Ureters During colonic resections/ gynaecological surgery

Bowel perforation Use of Verres Needle to establish pneumoperitoneum

Bile duct injury Failure to delineate Calots triangle carefully and careless use of diathermy

Facial nerve Always at risk during Parotidectomy

Tail of pancreas When ligating splenic hilum

Testicular vessels During re-do open hernia surgery

Hepatic veins During liver mobilisation

Again many could be predicted from the anatomy of the procedure.

Physiological derangements
A very common complication is bleeding and this is covered under the section of haemorrhagic
shock. Another variant is infection either superficial or deep seated. The organisms are covered
under microbiology and the features of sepsis covered under shock. Do not forget that
immunocompromised and elderly patients may present will atypical physiological parameters.

Selected physiological and biochemical issues are given below:

Complication Physiological/ Biochemical Problem

Arrhythmias following Susceptibility to hypokalaemia (K+ <4.0 in cardiac patients)


Complication Physiological/ Biochemical Problem

cardiac surgery

Neurosurgical electrolyte SIADH following cranial surgery causing hyponatraemia


disturbance

Ileus following Fluid sequestration and loss of electrolytes


gastrointestinal surgery

Pulmonary oedema following Loss of lung volume makes these patients very sensitive to fluid
pneumonectomy overload

Anastamotic leak Generalised sepsis causing mediastinitis or peritonitis depending


on site of leak

Myocardial infarct May follow any type of surgery and in addition to direct cardiac
effects the decreased cardiac output may well compromise grafts
etc.

Try making a short list of problems and causes specific to your own clinical area.

Diagnostic modalities
Depends largely on the suspected complication. In the acutely unwell surgical patient the following
baseline investigations are often helpful:

 Full blood count, urea and electrolytes, C- reactive protein (trend rather than absolute value),
serum calcium, liver function tests, clotting (don't forget to repeat if on-going bleeding)
 Arterial blood gases
 ECG (+cardiac enzymes if MI suspected)
 Chest x-ray to identify collapse/ consolidation
 Urine analysis for UTI

These will often identify the most common complications.

Special tests
 CT scanning for identification of intra-abdominal abscesses, air and if luminal contrast is
used an anastomotic leak
 Doppler USS of leg veins- for identification of DVT
 CTPA for PE
 Sending peritoneal fluid for U+E (if ureteric injury suspected) or amylase (if pancreatic injury
suspected)
 Echocardiogram if pericardial effusion suspected post cardiac surgery and no pleural window
made.

Management of complications
The guiding principal should be safe and timely intervention. Patients should be stabilised and if an
operation needs to occur in tandem with resuscitation then generally this should be of a damage
limitation type procedure rather than definitive surgery (which can be more safely undertaken in a
stable patient the following day).

Remember that recent surgery is a contra indication to thrombolysis and that in some patients IV
heparin may be preferable to a low molecular weight heparin (easier to reverse).

As a general rule laparotomies for bleeding should follow the core principle of quadrant packing and
then subsequent pack removal rather than plunging large clamps into pools of blood. The latter
approach invariable worsens the situation is often accompanied by significant visceral injury
particularly when done by the inexperienced. If packing controls a situation it is entirely acceptable
practice to leave packs in situ and return the patient to ITU for pack removal the subsequent day.
Next question
A patient with tachycardia and hypotension is to receive vasopressors. Which of the following
conditions are most likely to be treated with vasopressors?

Hypovolaemic shock

Septic shock

Neurogenic shock

Cardiogenic shock

None of the above

Theme from April 2012 Exam


The term septic shock has a precise meaning and refers to refractory systemic arterial hypotension
in spite of fluid resuscitation. Patients will therefore usually require inotropes. Individuals suffering
from neurogenic shock will usually receive intravenous fluids to achieve a mean arterial pressure of
90mmHg. If this target cannot be achieved then these patients will receive inotropes. Hypovolaemic
shock requires fluids and the management of cardiogenic shock is multifactorial and includes
inotropes, vasodilators and intra-aortic balloon pumps.

Please rate this question:

Discuss and give feedback

Next question

Inotropes and cardiovascular receptors

Inotropes are a class of drugs which work primarily by increasing cardiac output. They should be
distinguished from vasoconstrictor drugs which are used specifically when the primary problem is
peripheral vasodilatation.

Catecholamine type agents are commonly used and work by increasing cAMP levels by adenylate
cyclase stimulation. This in turn intracellular calcium ion mobilisation and thus the force of
contraction. Adrenaline works as a beta adrenergic receptor agonist at lower doses and an alpha
receptor agonist at higher doses. Dopamine causes dopamine receptor mediated renal and
mesenteric vascular dilatation and beta 1 receptor agonism at higher doses. This results in
increased cardiac output. Since both heart rate and blood pressure are raised, there is less overall
myocardial ischaemia. Dobutamine is a predominantly beta 1 receptor agonist with weak beta 2 and
alpha receptor agonist properties. Noradrenaline is a catecholamine type agent and predominantly
acts as an alpha receptor agonist and serves as a peripheral vasoconstrictor.

Phosphodiesterase inhibitors such as milrinone act specifically on the cardiac phosphodiesterase


and increase cardiac output.

Inotrope Cardiovascular receptor action

Adrenaline α-1, α-2, β-1, β-2

Noradrenaline α-1,( α-2), (β-1), (β-2)

Dobutamine β-1, (β 2)

Dopamine (α-1), (α-2), (β-1), D-1,D-2

Minor receptor effects in brackets

Effects of receptor binding

α-1, α-2 vasoconstriction

β-1 increased cardiac contractility and HR

β-2 vasodilatation

D-1 renal and spleen vasodilatation

D-2 inhibits release of noradrenaline


Infusion with which of the following blood products is most likely to result in an urticarial reaction?

Packed red cells

Fresh frozen plasma

Platelets

Cryoprecipitate

Factor VIII concentrate

Pyrexia is the most common adverse event in transfusing packed red cells
Urticaria is the most common adverse event following infusion of FFP

Please rate this question:

Discuss and give feedback

Next question

Blood transfusion reactions

Acute transfusion reactions present as adverse signs or symptoms during or within 24 hours of a
blood transfusion. The most frequent reactions are fever, chills, pruritus, or urticaria, which typically
resolve promptly without specific treatment or complications. Other signs occurring in temporal
relationship with a blood transfusion, such as severe dyspnoea, pyrexia, or loss of consciousness
may be the first indication of a more severe potentially fatal reaction.
The causes of adverse reactions are multi-factorial. Immune mediated reactions, some of the most
feared, occur as a result of component mismatch, the commonest cause of which is clerical error.
More common, non immune mediated, complications may occur as a result of product
contamination, this may be bacterial or viral.
Transfusion related lung injury is well recognised and there are two proposed mechanisms which
underpin this. One involves the sequestration of primed neutrophils within the recipient pulmonary
capillary bed. The other proposed mechanism suggests that HLA mismatches between donor
neutrophils and recipient lung tissue is to blame.
The table below summarises the main types of transfusion reaction.

Immune mediated Non immune mediated

Pyrexia Hypocalcaemia

Alloimmunization CCF

Thrombocytopaenia Infections

Transfusion associated lung injury Hyperkalaemia

Graft vs Host disease

Urticaria

Acute or delayed haemolysis

ABO incompatibility

Rhesus incompatibility

Next question
Theme: Feeding options

A. Feeding jejunostomy
B. Percutaneous endoscopic gastrostomy
C. Total parenteral nutrition
D. Naso gastric feeding tube
E. Elemental diet orally
F. Normal oral intake

Please select the most appropriate method of delivering nutrition in each of the following scenarios.
Each option may be used once, more than once or not at all.

13. A 28 year old man is comatose, from head injuries, on the neurosurgical intensive care
unit. Apart from a parietal fracture there is no bony injury. He is recovering well and
should be extubated soon.

You answered Feeding jejunostomy

The correct answer is Naso gastric feeding tube

Theme from April 2012 Exam


Theme from January 2013 Exam
The feeding of head injured patients was reviewed in a 2008 Cochrane report. They
concluded that the overall evidence base was poor. However, there was a trend for the
enteral route, with NG feeding in the later stages following injury. This is contra indicated
if there are signs of basal skull fractures.

14. A 56 year old man has undergone a potentially curative oesophagectomy for carcinoma.

Feeding jejunostomy

Feeding jejunostomy is the standard of care in most centres. Naso jejunal tubes are
preferred by some surgeons. However, if they become displaced the only alternative then
becomes TPN.

15. A 43 year old man is recovering from a laparoscopic low anterior resection with loop
ileostomy.

You answered Feeding jejunostomy

The correct answer is Normal oral intake

Early feeding in this situation is both safe and will enhance recovery.
Please rate this question:

Discuss and give feedback


Next question

Nutrition options in surgical patients

Oral intake  Easiest option


 May be supplemented by calorie rich dietary supplements
 May contra indicated following certain procedures

Naso gastric feeding  Usually administered via fine bore naso gastric feeding tube
 Complications relate to aspiration of feed or misplaced tube
 May be safe to use in patients with impaired swallow
 Often contra indicated following head injury due to risks
associated with tube insertion

Naso jejunal feeding  Avoids problems of feed pooling in stomach (and risk of
aspiration)
 Insertion of feeding tube more technically complicated (easiest
if done intra operatively)
 Safe to use following oesophagogastric surgery

Feeding jejunostomy  Surgically sited feeding tube


 May be used for long term feeding
 Low risk of aspiration and thus safe for long term feeding
following upper GI surgery
 Main risks are those of tube displacement and peritubal leakage
immediately following insertion, which carries a risk of
peritonitis

Percutaneous  Combined endoscopic and percutaneous tube insertion


endoscopic  May not be technically possible in those patients who cannot
gastrostomy undergo successful endoscopy
 Risks include aspiration and leakage at the insertion site

Total parenteral  The definitive option in those patients in whom enteral feeding
nutrition is contra indicated
 Individualised prescribing and monitoring needed
 Should be administered via a central vein as it is strongly
phlebitic
 Long term use is associated with fatty liver and deranged LFT's

Next question
Theme: Anaesthetic agents

A. Etomidate
B. Ketamine
C. Propofol
D. Sodium thiopentone
E. Methohexitone
F. Metaraminol
G. Midazolam

Please select the most appropriate anaesthetic induction agent for the procedure described. Each
option may be used once, more than once or not at all.

16. A 32 year old man is admitted for a trendelenberg procedure for varicose veins. He is
known to have porphyria.

You answered Etomidate

The correct answer is Propofol

This is a daycase procedure for which propofol is ideal. Sodium thiopentone and etomidate
are contraindicated in porphyria.

17. A 77 year old lady with unstable ischaemic heart disease requires an emergency femoral
hernia repair. She is volume depleted and slightly hypotensive.

You answered Etomidate

The correct answer is Ketamine

Ketamine is not negatively inotropic and will not depress cardiac output. Propofol and
Sodium thiopentone will produce myocardial depression. Some doctors may also consider
etomidate. However, it may cause adrenal suppression and post operative vomiting- which
she is at high risk of developing.

18. A 22 year old man is brought to theatre for an emergency apppendicectomy for generalised
peritonitis. He is vomiting.

You answered Etomidate

The correct answer is Sodium thiopentone

Most anaesthetists would use sodium thiopentone for a rapid sequence induction (which
this man will need).
Propofol- Ideal agent for daycase- wears off rapidly, good antiemetic effect.

Sodium thiopentone- Fast onset of action- prone to accumulation. Depresses cardiac output.
Ketamine- Little haemodynamic instability. Good analgesic properties. Nightmares and restlessness.

Please rate this question:

Discuss and give feedback


Next question

Anaesthetic agents

The table below summarises some of the more commonly used IV induction agents
Agent Specific features

Propofol  Rapid onset of anaesthesia


 Pain on IV injection
 Rapidly metabolised with little accumulation of metabolites
 Proven anti emetic properties
 Moderate myocardial depression
 Widely used especially for maintaining sedation on ITU, total IV
anaesthesia and for daycase surgery

Sodium  Extremely rapid onset of action making it the agent of choice for rapid
thiopentone sequence of induction
 Marked myocardial depression may occur
 Metabolites build up quickly
 Unsuitable for maintenance infusion
 Little analgesic effects

Ketamine  May be used for induction of anaesthesia


 Has moderate to strong analgesic properties
 Produces little myocardial depression making it a suitable agent for
anaesthesia in those who are haemodynamically unstable
 May induce state of dissociative anaesthesia resulting in nightmares

Etomidate  Has favorable cardiac safety profile with very little haemodynamic
instability

You might also like